All IU DES 10437 _2023 pdf.pdfjajjsjnjis

NobelPel1 57 views 190 slides May 20, 2024
Slide 1
Slide 1 of 2488
Slide 1
1
Slide 2
2
Slide 3
3
Slide 4
4
Slide 5
5
Slide 6
6
Slide 7
7
Slide 8
8
Slide 9
9
Slide 10
10
Slide 11
11
Slide 12
12
Slide 13
13
Slide 14
14
Slide 15
15
Slide 16
16
Slide 17
17
Slide 18
18
Slide 19
19
Slide 20
20
Slide 21
21
Slide 22
22
Slide 23
23
Slide 24
24
Slide 25
25
Slide 26
26
Slide 27
27
Slide 28
28
Slide 29
29
Slide 30
30
Slide 31
31
Slide 32
32
Slide 33
33
Slide 34
34
Slide 35
35
Slide 36
36
Slide 37
37
Slide 38
38
Slide 39
39
Slide 40
40
Slide 41
41
Slide 42
42
Slide 43
43
Slide 44
44
Slide 45
45
Slide 46
46
Slide 47
47
Slide 48
48
Slide 49
49
Slide 50
50
Slide 51
51
Slide 52
52
Slide 53
53
Slide 54
54
Slide 55
55
Slide 56
56
Slide 57
57
Slide 58
58
Slide 59
59
Slide 60
60
Slide 61
61
Slide 62
62
Slide 63
63
Slide 64
64
Slide 65
65
Slide 66
66
Slide 67
67
Slide 68
68
Slide 69
69
Slide 70
70
Slide 71
71
Slide 72
72
Slide 73
73
Slide 74
74
Slide 75
75
Slide 76
76
Slide 77
77
Slide 78
78
Slide 79
79
Slide 80
80
Slide 81
81
Slide 82
82
Slide 83
83
Slide 84
84
Slide 85
85
Slide 86
86
Slide 87
87
Slide 88
88
Slide 89
89
Slide 90
90
Slide 91
91
Slide 92
92
Slide 93
93
Slide 94
94
Slide 95
95
Slide 96
96
Slide 97
97
Slide 98
98
Slide 99
99
Slide 100
100
Slide 101
101
Slide 102
102
Slide 103
103
Slide 104
104
Slide 105
105
Slide 106
106
Slide 107
107
Slide 108
108
Slide 109
109
Slide 110
110
Slide 111
111
Slide 112
112
Slide 113
113
Slide 114
114
Slide 115
115
Slide 116
116
Slide 117
117
Slide 118
118
Slide 119
119
Slide 120
120
Slide 121
121
Slide 122
122
Slide 123
123
Slide 124
124
Slide 125
125
Slide 126
126
Slide 127
127
Slide 128
128
Slide 129
129
Slide 130
130
Slide 131
131
Slide 132
132
Slide 133
133
Slide 134
134
Slide 135
135
Slide 136
136
Slide 137
137
Slide 138
138
Slide 139
139
Slide 140
140
Slide 141
141
Slide 142
142
Slide 143
143
Slide 144
144
Slide 145
145
Slide 146
146
Slide 147
147
Slide 148
148
Slide 149
149
Slide 150
150
Slide 151
151
Slide 152
152
Slide 153
153
Slide 154
154
Slide 155
155
Slide 156
156
Slide 157
157
Slide 158
158
Slide 159
159
Slide 160
160
Slide 161
161
Slide 162
162
Slide 163
163
Slide 164
164
Slide 165
165
Slide 166
166
Slide 167
167
Slide 168
168
Slide 169
169
Slide 170
170
Slide 171
171
Slide 172
172
Slide 173
173
Slide 174
174
Slide 175
175
Slide 176
176
Slide 177
177
Slide 178
178
Slide 179
179
Slide 180
180
Slide 181
181
Slide 182
182
Slide 183
183
Slide 184
184
Slide 185
185
Slide 186
186
Slide 187
187
Slide 188
188
Slide 189
189
Slide 190
190
Slide 191
191
Slide 192
192
Slide 193
193
Slide 194
194
Slide 195
195
Slide 196
196
Slide 197
197
Slide 198
198
Slide 199
199
Slide 200
200
Slide 201
201
Slide 202
202
Slide 203
203
Slide 204
204
Slide 205
205
Slide 206
206
Slide 207
207
Slide 208
208
Slide 209
209
Slide 210
210
Slide 211
211
Slide 212
212
Slide 213
213
Slide 214
214
Slide 215
215
Slide 216
216
Slide 217
217
Slide 218
218
Slide 219
219
Slide 220
220
Slide 221
221
Slide 222
222
Slide 223
223
Slide 224
224
Slide 225
225
Slide 226
226
Slide 227
227
Slide 228
228
Slide 229
229
Slide 230
230
Slide 231
231
Slide 232
232
Slide 233
233
Slide 234
234
Slide 235
235
Slide 236
236
Slide 237
237
Slide 238
238
Slide 239
239
Slide 240
240
Slide 241
241
Slide 242
242
Slide 243
243
Slide 244
244
Slide 245
245
Slide 246
246
Slide 247
247
Slide 248
248
Slide 249
249
Slide 250
250
Slide 251
251
Slide 252
252
Slide 253
253
Slide 254
254
Slide 255
255
Slide 256
256
Slide 257
257
Slide 258
258
Slide 259
259
Slide 260
260
Slide 261
261
Slide 262
262
Slide 263
263
Slide 264
264
Slide 265
265
Slide 266
266
Slide 267
267
Slide 268
268
Slide 269
269
Slide 270
270
Slide 271
271
Slide 272
272
Slide 273
273
Slide 274
274
Slide 275
275
Slide 276
276
Slide 277
277
Slide 278
278
Slide 279
279
Slide 280
280
Slide 281
281
Slide 282
282
Slide 283
283
Slide 284
284
Slide 285
285
Slide 286
286
Slide 287
287
Slide 288
288
Slide 289
289
Slide 290
290
Slide 291
291
Slide 292
292
Slide 293
293
Slide 294
294
Slide 295
295
Slide 296
296
Slide 297
297
Slide 298
298
Slide 299
299
Slide 300
300
Slide 301
301
Slide 302
302
Slide 303
303
Slide 304
304
Slide 305
305
Slide 306
306
Slide 307
307
Slide 308
308
Slide 309
309
Slide 310
310
Slide 311
311
Slide 312
312
Slide 313
313
Slide 314
314
Slide 315
315
Slide 316
316
Slide 317
317
Slide 318
318
Slide 319
319
Slide 320
320
Slide 321
321
Slide 322
322
Slide 323
323
Slide 324
324
Slide 325
325
Slide 326
326
Slide 327
327
Slide 328
328
Slide 329
329
Slide 330
330
Slide 331
331
Slide 332
332
Slide 333
333
Slide 334
334
Slide 335
335
Slide 336
336
Slide 337
337
Slide 338
338
Slide 339
339
Slide 340
340
Slide 341
341
Slide 342
342
Slide 343
343
Slide 344
344
Slide 345
345
Slide 346
346
Slide 347
347
Slide 348
348
Slide 349
349
Slide 350
350
Slide 351
351
Slide 352
352
Slide 353
353
Slide 354
354
Slide 355
355
Slide 356
356
Slide 357
357
Slide 358
358
Slide 359
359
Slide 360
360
Slide 361
361
Slide 362
362
Slide 363
363
Slide 364
364
Slide 365
365
Slide 366
366
Slide 367
367
Slide 368
368
Slide 369
369
Slide 370
370
Slide 371
371
Slide 372
372
Slide 373
373
Slide 374
374
Slide 375
375
Slide 376
376
Slide 377
377
Slide 378
378
Slide 379
379
Slide 380
380
Slide 381
381
Slide 382
382
Slide 383
383
Slide 384
384
Slide 385
385
Slide 386
386
Slide 387
387
Slide 388
388
Slide 389
389
Slide 390
390
Slide 391
391
Slide 392
392
Slide 393
393
Slide 394
394
Slide 395
395
Slide 396
396
Slide 397
397
Slide 398
398
Slide 399
399
Slide 400
400
Slide 401
401
Slide 402
402
Slide 403
403
Slide 404
404
Slide 405
405
Slide 406
406
Slide 407
407
Slide 408
408
Slide 409
409
Slide 410
410
Slide 411
411
Slide 412
412
Slide 413
413
Slide 414
414
Slide 415
415
Slide 416
416
Slide 417
417
Slide 418
418
Slide 419
419
Slide 420
420
Slide 421
421
Slide 422
422
Slide 423
423
Slide 424
424
Slide 425
425
Slide 426
426
Slide 427
427
Slide 428
428
Slide 429
429
Slide 430
430
Slide 431
431
Slide 432
432
Slide 433
433
Slide 434
434
Slide 435
435
Slide 436
436
Slide 437
437
Slide 438
438
Slide 439
439
Slide 440
440
Slide 441
441
Slide 442
442
Slide 443
443
Slide 444
444
Slide 445
445
Slide 446
446
Slide 447
447
Slide 448
448
Slide 449
449
Slide 450
450
Slide 451
451
Slide 452
452
Slide 453
453
Slide 454
454
Slide 455
455
Slide 456
456
Slide 457
457
Slide 458
458
Slide 459
459
Slide 460
460
Slide 461
461
Slide 462
462
Slide 463
463
Slide 464
464
Slide 465
465
Slide 466
466
Slide 467
467
Slide 468
468
Slide 469
469
Slide 470
470
Slide 471
471
Slide 472
472
Slide 473
473
Slide 474
474
Slide 475
475
Slide 476
476
Slide 477
477
Slide 478
478
Slide 479
479
Slide 480
480
Slide 481
481
Slide 482
482
Slide 483
483
Slide 484
484
Slide 485
485
Slide 486
486
Slide 487
487
Slide 488
488
Slide 489
489
Slide 490
490
Slide 491
491
Slide 492
492
Slide 493
493
Slide 494
494
Slide 495
495
Slide 496
496
Slide 497
497
Slide 498
498
Slide 499
499
Slide 500
500
Slide 501
501
Slide 502
502
Slide 503
503
Slide 504
504
Slide 505
505
Slide 506
506
Slide 507
507
Slide 508
508
Slide 509
509
Slide 510
510
Slide 511
511
Slide 512
512
Slide 513
513
Slide 514
514
Slide 515
515
Slide 516
516
Slide 517
517
Slide 518
518
Slide 519
519
Slide 520
520
Slide 521
521
Slide 522
522
Slide 523
523
Slide 524
524
Slide 525
525
Slide 526
526
Slide 527
527
Slide 528
528
Slide 529
529
Slide 530
530
Slide 531
531
Slide 532
532
Slide 533
533
Slide 534
534
Slide 535
535
Slide 536
536
Slide 537
537
Slide 538
538
Slide 539
539
Slide 540
540
Slide 541
541
Slide 542
542
Slide 543
543
Slide 544
544
Slide 545
545
Slide 546
546
Slide 547
547
Slide 548
548
Slide 549
549
Slide 550
550
Slide 551
551
Slide 552
552
Slide 553
553
Slide 554
554
Slide 555
555
Slide 556
556
Slide 557
557
Slide 558
558
Slide 559
559
Slide 560
560
Slide 561
561
Slide 562
562
Slide 563
563
Slide 564
564
Slide 565
565
Slide 566
566
Slide 567
567
Slide 568
568
Slide 569
569
Slide 570
570
Slide 571
571
Slide 572
572
Slide 573
573
Slide 574
574
Slide 575
575
Slide 576
576
Slide 577
577
Slide 578
578
Slide 579
579
Slide 580
580
Slide 581
581
Slide 582
582
Slide 583
583
Slide 584
584
Slide 585
585
Slide 586
586
Slide 587
587
Slide 588
588
Slide 589
589
Slide 590
590
Slide 591
591
Slide 592
592
Slide 593
593
Slide 594
594
Slide 595
595
Slide 596
596
Slide 597
597
Slide 598
598
Slide 599
599
Slide 600
600
Slide 601
601
Slide 602
602
Slide 603
603
Slide 604
604
Slide 605
605
Slide 606
606
Slide 607
607
Slide 608
608
Slide 609
609
Slide 610
610
Slide 611
611
Slide 612
612
Slide 613
613
Slide 614
614
Slide 615
615
Slide 616
616
Slide 617
617
Slide 618
618
Slide 619
619
Slide 620
620
Slide 621
621
Slide 622
622
Slide 623
623
Slide 624
624
Slide 625
625
Slide 626
626
Slide 627
627
Slide 628
628
Slide 629
629
Slide 630
630
Slide 631
631
Slide 632
632
Slide 633
633
Slide 634
634
Slide 635
635
Slide 636
636
Slide 637
637
Slide 638
638
Slide 639
639
Slide 640
640
Slide 641
641
Slide 642
642
Slide 643
643
Slide 644
644
Slide 645
645
Slide 646
646
Slide 647
647
Slide 648
648
Slide 649
649
Slide 650
650
Slide 651
651
Slide 652
652
Slide 653
653
Slide 654
654
Slide 655
655
Slide 656
656
Slide 657
657
Slide 658
658
Slide 659
659
Slide 660
660
Slide 661
661
Slide 662
662
Slide 663
663
Slide 664
664
Slide 665
665
Slide 666
666
Slide 667
667
Slide 668
668
Slide 669
669
Slide 670
670
Slide 671
671
Slide 672
672
Slide 673
673
Slide 674
674
Slide 675
675
Slide 676
676
Slide 677
677
Slide 678
678
Slide 679
679
Slide 680
680
Slide 681
681
Slide 682
682
Slide 683
683
Slide 684
684
Slide 685
685
Slide 686
686
Slide 687
687
Slide 688
688
Slide 689
689
Slide 690
690
Slide 691
691
Slide 692
692
Slide 693
693
Slide 694
694
Slide 695
695
Slide 696
696
Slide 697
697
Slide 698
698
Slide 699
699
Slide 700
700
Slide 701
701
Slide 702
702
Slide 703
703
Slide 704
704
Slide 705
705
Slide 706
706
Slide 707
707
Slide 708
708
Slide 709
709
Slide 710
710
Slide 711
711
Slide 712
712
Slide 713
713
Slide 714
714
Slide 715
715
Slide 716
716
Slide 717
717
Slide 718
718
Slide 719
719
Slide 720
720
Slide 721
721
Slide 722
722
Slide 723
723
Slide 724
724
Slide 725
725
Slide 726
726
Slide 727
727
Slide 728
728
Slide 729
729
Slide 730
730
Slide 731
731
Slide 732
732
Slide 733
733
Slide 734
734
Slide 735
735
Slide 736
736
Slide 737
737
Slide 738
738
Slide 739
739
Slide 740
740
Slide 741
741
Slide 742
742
Slide 743
743
Slide 744
744
Slide 745
745
Slide 746
746
Slide 747
747
Slide 748
748
Slide 749
749
Slide 750
750
Slide 751
751
Slide 752
752
Slide 753
753
Slide 754
754
Slide 755
755
Slide 756
756
Slide 757
757
Slide 758
758
Slide 759
759
Slide 760
760
Slide 761
761
Slide 762
762
Slide 763
763
Slide 764
764
Slide 765
765
Slide 766
766
Slide 767
767
Slide 768
768
Slide 769
769
Slide 770
770
Slide 771
771
Slide 772
772
Slide 773
773
Slide 774
774
Slide 775
775
Slide 776
776
Slide 777
777
Slide 778
778
Slide 779
779
Slide 780
780
Slide 781
781
Slide 782
782
Slide 783
783
Slide 784
784
Slide 785
785
Slide 786
786
Slide 787
787
Slide 788
788
Slide 789
789
Slide 790
790
Slide 791
791
Slide 792
792
Slide 793
793
Slide 794
794
Slide 795
795
Slide 796
796
Slide 797
797
Slide 798
798
Slide 799
799
Slide 800
800
Slide 801
801
Slide 802
802
Slide 803
803
Slide 804
804
Slide 805
805
Slide 806
806
Slide 807
807
Slide 808
808
Slide 809
809
Slide 810
810
Slide 811
811
Slide 812
812
Slide 813
813
Slide 814
814
Slide 815
815
Slide 816
816
Slide 817
817
Slide 818
818
Slide 819
819
Slide 820
820
Slide 821
821
Slide 822
822
Slide 823
823
Slide 824
824
Slide 825
825
Slide 826
826
Slide 827
827
Slide 828
828
Slide 829
829
Slide 830
830
Slide 831
831
Slide 832
832
Slide 833
833
Slide 834
834
Slide 835
835
Slide 836
836
Slide 837
837
Slide 838
838
Slide 839
839
Slide 840
840
Slide 841
841
Slide 842
842
Slide 843
843
Slide 844
844
Slide 845
845
Slide 846
846
Slide 847
847
Slide 848
848
Slide 849
849
Slide 850
850
Slide 851
851
Slide 852
852
Slide 853
853
Slide 854
854
Slide 855
855
Slide 856
856
Slide 857
857
Slide 858
858
Slide 859
859
Slide 860
860
Slide 861
861
Slide 862
862
Slide 863
863
Slide 864
864
Slide 865
865
Slide 866
866
Slide 867
867
Slide 868
868
Slide 869
869
Slide 870
870
Slide 871
871
Slide 872
872
Slide 873
873
Slide 874
874
Slide 875
875
Slide 876
876
Slide 877
877
Slide 878
878
Slide 879
879
Slide 880
880
Slide 881
881
Slide 882
882
Slide 883
883
Slide 884
884
Slide 885
885
Slide 886
886
Slide 887
887
Slide 888
888
Slide 889
889
Slide 890
890
Slide 891
891
Slide 892
892
Slide 893
893
Slide 894
894
Slide 895
895
Slide 896
896
Slide 897
897
Slide 898
898
Slide 899
899
Slide 900
900
Slide 901
901
Slide 902
902
Slide 903
903
Slide 904
904
Slide 905
905
Slide 906
906
Slide 907
907
Slide 908
908
Slide 909
909
Slide 910
910
Slide 911
911
Slide 912
912
Slide 913
913
Slide 914
914
Slide 915
915
Slide 916
916
Slide 917
917
Slide 918
918
Slide 919
919
Slide 920
920
Slide 921
921
Slide 922
922
Slide 923
923
Slide 924
924
Slide 925
925
Slide 926
926
Slide 927
927
Slide 928
928
Slide 929
929
Slide 930
930
Slide 931
931
Slide 932
932
Slide 933
933
Slide 934
934
Slide 935
935
Slide 936
936
Slide 937
937
Slide 938
938
Slide 939
939
Slide 940
940
Slide 941
941
Slide 942
942
Slide 943
943
Slide 944
944
Slide 945
945
Slide 946
946
Slide 947
947
Slide 948
948
Slide 949
949
Slide 950
950
Slide 951
951
Slide 952
952
Slide 953
953
Slide 954
954
Slide 955
955
Slide 956
956
Slide 957
957
Slide 958
958
Slide 959
959
Slide 960
960
Slide 961
961
Slide 962
962
Slide 963
963
Slide 964
964
Slide 965
965
Slide 966
966
Slide 967
967
Slide 968
968
Slide 969
969
Slide 970
970
Slide 971
971
Slide 972
972
Slide 973
973
Slide 974
974
Slide 975
975
Slide 976
976
Slide 977
977
Slide 978
978
Slide 979
979
Slide 980
980
Slide 981
981
Slide 982
982
Slide 983
983
Slide 984
984
Slide 985
985
Slide 986
986
Slide 987
987
Slide 988
988
Slide 989
989
Slide 990
990
Slide 991
991
Slide 992
992
Slide 993
993
Slide 994
994
Slide 995
995
Slide 996
996
Slide 997
997
Slide 998
998
Slide 999
999
Slide 1000
1000
Slide 1001
1001
Slide 1002
1002
Slide 1003
1003
Slide 1004
1004
Slide 1005
1005
Slide 1006
1006
Slide 1007
1007
Slide 1008
1008
Slide 1009
1009
Slide 1010
1010
Slide 1011
1011
Slide 1012
1012
Slide 1013
1013
Slide 1014
1014
Slide 1015
1015
Slide 1016
1016
Slide 1017
1017
Slide 1018
1018
Slide 1019
1019
Slide 1020
1020
Slide 1021
1021
Slide 1022
1022
Slide 1023
1023
Slide 1024
1024
Slide 1025
1025
Slide 1026
1026
Slide 1027
1027
Slide 1028
1028
Slide 1029
1029
Slide 1030
1030
Slide 1031
1031
Slide 1032
1032
Slide 1033
1033
Slide 1034
1034
Slide 1035
1035
Slide 1036
1036
Slide 1037
1037
Slide 1038
1038
Slide 1039
1039
Slide 1040
1040
Slide 1041
1041
Slide 1042
1042
Slide 1043
1043
Slide 1044
1044
Slide 1045
1045
Slide 1046
1046
Slide 1047
1047
Slide 1048
1048
Slide 1049
1049
Slide 1050
1050
Slide 1051
1051
Slide 1052
1052
Slide 1053
1053
Slide 1054
1054
Slide 1055
1055
Slide 1056
1056
Slide 1057
1057
Slide 1058
1058
Slide 1059
1059
Slide 1060
1060
Slide 1061
1061
Slide 1062
1062
Slide 1063
1063
Slide 1064
1064
Slide 1065
1065
Slide 1066
1066
Slide 1067
1067
Slide 1068
1068
Slide 1069
1069
Slide 1070
1070
Slide 1071
1071
Slide 1072
1072
Slide 1073
1073
Slide 1074
1074
Slide 1075
1075
Slide 1076
1076
Slide 1077
1077
Slide 1078
1078
Slide 1079
1079
Slide 1080
1080
Slide 1081
1081
Slide 1082
1082
Slide 1083
1083
Slide 1084
1084
Slide 1085
1085
Slide 1086
1086
Slide 1087
1087
Slide 1088
1088
Slide 1089
1089
Slide 1090
1090
Slide 1091
1091
Slide 1092
1092
Slide 1093
1093
Slide 1094
1094
Slide 1095
1095
Slide 1096
1096
Slide 1097
1097
Slide 1098
1098
Slide 1099
1099
Slide 1100
1100
Slide 1101
1101
Slide 1102
1102
Slide 1103
1103
Slide 1104
1104
Slide 1105
1105
Slide 1106
1106
Slide 1107
1107
Slide 1108
1108
Slide 1109
1109
Slide 1110
1110
Slide 1111
1111
Slide 1112
1112
Slide 1113
1113
Slide 1114
1114
Slide 1115
1115
Slide 1116
1116
Slide 1117
1117
Slide 1118
1118
Slide 1119
1119
Slide 1120
1120
Slide 1121
1121
Slide 1122
1122
Slide 1123
1123
Slide 1124
1124
Slide 1125
1125
Slide 1126
1126
Slide 1127
1127
Slide 1128
1128
Slide 1129
1129
Slide 1130
1130
Slide 1131
1131
Slide 1132
1132
Slide 1133
1133
Slide 1134
1134
Slide 1135
1135
Slide 1136
1136
Slide 1137
1137
Slide 1138
1138
Slide 1139
1139
Slide 1140
1140
Slide 1141
1141
Slide 1142
1142
Slide 1143
1143
Slide 1144
1144
Slide 1145
1145
Slide 1146
1146
Slide 1147
1147
Slide 1148
1148
Slide 1149
1149
Slide 1150
1150
Slide 1151
1151
Slide 1152
1152
Slide 1153
1153
Slide 1154
1154
Slide 1155
1155
Slide 1156
1156
Slide 1157
1157
Slide 1158
1158
Slide 1159
1159
Slide 1160
1160
Slide 1161
1161
Slide 1162
1162
Slide 1163
1163
Slide 1164
1164
Slide 1165
1165
Slide 1166
1166
Slide 1167
1167
Slide 1168
1168
Slide 1169
1169
Slide 1170
1170
Slide 1171
1171
Slide 1172
1172
Slide 1173
1173
Slide 1174
1174
Slide 1175
1175
Slide 1176
1176
Slide 1177
1177
Slide 1178
1178
Slide 1179
1179
Slide 1180
1180
Slide 1181
1181
Slide 1182
1182
Slide 1183
1183
Slide 1184
1184
Slide 1185
1185
Slide 1186
1186
Slide 1187
1187
Slide 1188
1188
Slide 1189
1189
Slide 1190
1190
Slide 1191
1191
Slide 1192
1192
Slide 1193
1193
Slide 1194
1194
Slide 1195
1195
Slide 1196
1196
Slide 1197
1197
Slide 1198
1198
Slide 1199
1199
Slide 1200
1200
Slide 1201
1201
Slide 1202
1202
Slide 1203
1203
Slide 1204
1204
Slide 1205
1205
Slide 1206
1206
Slide 1207
1207
Slide 1208
1208
Slide 1209
1209
Slide 1210
1210
Slide 1211
1211
Slide 1212
1212
Slide 1213
1213
Slide 1214
1214
Slide 1215
1215
Slide 1216
1216
Slide 1217
1217
Slide 1218
1218
Slide 1219
1219
Slide 1220
1220
Slide 1221
1221
Slide 1222
1222
Slide 1223
1223
Slide 1224
1224
Slide 1225
1225
Slide 1226
1226
Slide 1227
1227
Slide 1228
1228
Slide 1229
1229
Slide 1230
1230
Slide 1231
1231
Slide 1232
1232
Slide 1233
1233
Slide 1234
1234
Slide 1235
1235
Slide 1236
1236
Slide 1237
1237
Slide 1238
1238
Slide 1239
1239
Slide 1240
1240
Slide 1241
1241
Slide 1242
1242
Slide 1243
1243
Slide 1244
1244
Slide 1245
1245
Slide 1246
1246
Slide 1247
1247
Slide 1248
1248
Slide 1249
1249
Slide 1250
1250
Slide 1251
1251
Slide 1252
1252
Slide 1253
1253
Slide 1254
1254
Slide 1255
1255
Slide 1256
1256
Slide 1257
1257
Slide 1258
1258
Slide 1259
1259
Slide 1260
1260
Slide 1261
1261
Slide 1262
1262
Slide 1263
1263
Slide 1264
1264
Slide 1265
1265
Slide 1266
1266
Slide 1267
1267
Slide 1268
1268
Slide 1269
1269
Slide 1270
1270
Slide 1271
1271
Slide 1272
1272
Slide 1273
1273
Slide 1274
1274
Slide 1275
1275
Slide 1276
1276
Slide 1277
1277
Slide 1278
1278
Slide 1279
1279
Slide 1280
1280
Slide 1281
1281
Slide 1282
1282
Slide 1283
1283
Slide 1284
1284
Slide 1285
1285
Slide 1286
1286
Slide 1287
1287
Slide 1288
1288
Slide 1289
1289
Slide 1290
1290
Slide 1291
1291
Slide 1292
1292
Slide 1293
1293
Slide 1294
1294
Slide 1295
1295
Slide 1296
1296
Slide 1297
1297
Slide 1298
1298
Slide 1299
1299
Slide 1300
1300
Slide 1301
1301
Slide 1302
1302
Slide 1303
1303
Slide 1304
1304
Slide 1305
1305
Slide 1306
1306
Slide 1307
1307
Slide 1308
1308
Slide 1309
1309
Slide 1310
1310
Slide 1311
1311
Slide 1312
1312
Slide 1313
1313
Slide 1314
1314
Slide 1315
1315
Slide 1316
1316
Slide 1317
1317
Slide 1318
1318
Slide 1319
1319
Slide 1320
1320
Slide 1321
1321
Slide 1322
1322
Slide 1323
1323
Slide 1324
1324
Slide 1325
1325
Slide 1326
1326
Slide 1327
1327
Slide 1328
1328
Slide 1329
1329
Slide 1330
1330
Slide 1331
1331
Slide 1332
1332
Slide 1333
1333
Slide 1334
1334
Slide 1335
1335
Slide 1336
1336
Slide 1337
1337
Slide 1338
1338
Slide 1339
1339
Slide 1340
1340
Slide 1341
1341
Slide 1342
1342
Slide 1343
1343
Slide 1344
1344
Slide 1345
1345
Slide 1346
1346
Slide 1347
1347
Slide 1348
1348
Slide 1349
1349
Slide 1350
1350
Slide 1351
1351
Slide 1352
1352
Slide 1353
1353
Slide 1354
1354
Slide 1355
1355
Slide 1356
1356
Slide 1357
1357
Slide 1358
1358
Slide 1359
1359
Slide 1360
1360
Slide 1361
1361
Slide 1362
1362
Slide 1363
1363
Slide 1364
1364
Slide 1365
1365
Slide 1366
1366
Slide 1367
1367
Slide 1368
1368
Slide 1369
1369
Slide 1370
1370
Slide 1371
1371
Slide 1372
1372
Slide 1373
1373
Slide 1374
1374
Slide 1375
1375
Slide 1376
1376
Slide 1377
1377
Slide 1378
1378
Slide 1379
1379
Slide 1380
1380
Slide 1381
1381
Slide 1382
1382
Slide 1383
1383
Slide 1384
1384
Slide 1385
1385
Slide 1386
1386
Slide 1387
1387
Slide 1388
1388
Slide 1389
1389
Slide 1390
1390
Slide 1391
1391
Slide 1392
1392
Slide 1393
1393
Slide 1394
1394
Slide 1395
1395
Slide 1396
1396
Slide 1397
1397
Slide 1398
1398
Slide 1399
1399
Slide 1400
1400
Slide 1401
1401
Slide 1402
1402
Slide 1403
1403
Slide 1404
1404
Slide 1405
1405
Slide 1406
1406
Slide 1407
1407
Slide 1408
1408
Slide 1409
1409
Slide 1410
1410
Slide 1411
1411
Slide 1412
1412
Slide 1413
1413
Slide 1414
1414
Slide 1415
1415
Slide 1416
1416
Slide 1417
1417
Slide 1418
1418
Slide 1419
1419
Slide 1420
1420
Slide 1421
1421
Slide 1422
1422
Slide 1423
1423
Slide 1424
1424
Slide 1425
1425
Slide 1426
1426
Slide 1427
1427
Slide 1428
1428
Slide 1429
1429
Slide 1430
1430
Slide 1431
1431
Slide 1432
1432
Slide 1433
1433
Slide 1434
1434
Slide 1435
1435
Slide 1436
1436
Slide 1437
1437
Slide 1438
1438
Slide 1439
1439
Slide 1440
1440
Slide 1441
1441
Slide 1442
1442
Slide 1443
1443
Slide 1444
1444
Slide 1445
1445
Slide 1446
1446
Slide 1447
1447
Slide 1448
1448
Slide 1449
1449
Slide 1450
1450
Slide 1451
1451
Slide 1452
1452
Slide 1453
1453
Slide 1454
1454
Slide 1455
1455
Slide 1456
1456
Slide 1457
1457
Slide 1458
1458
Slide 1459
1459
Slide 1460
1460
Slide 1461
1461
Slide 1462
1462
Slide 1463
1463
Slide 1464
1464
Slide 1465
1465
Slide 1466
1466
Slide 1467
1467
Slide 1468
1468
Slide 1469
1469
Slide 1470
1470
Slide 1471
1471
Slide 1472
1472
Slide 1473
1473
Slide 1474
1474
Slide 1475
1475
Slide 1476
1476
Slide 1477
1477
Slide 1478
1478
Slide 1479
1479
Slide 1480
1480
Slide 1481
1481
Slide 1482
1482
Slide 1483
1483
Slide 1484
1484
Slide 1485
1485
Slide 1486
1486
Slide 1487
1487
Slide 1488
1488
Slide 1489
1489
Slide 1490
1490
Slide 1491
1491
Slide 1492
1492
Slide 1493
1493
Slide 1494
1494
Slide 1495
1495
Slide 1496
1496
Slide 1497
1497
Slide 1498
1498
Slide 1499
1499
Slide 1500
1500
Slide 1501
1501
Slide 1502
1502
Slide 1503
1503
Slide 1504
1504
Slide 1505
1505
Slide 1506
1506
Slide 1507
1507
Slide 1508
1508
Slide 1509
1509
Slide 1510
1510
Slide 1511
1511
Slide 1512
1512
Slide 1513
1513
Slide 1514
1514
Slide 1515
1515
Slide 1516
1516
Slide 1517
1517
Slide 1518
1518
Slide 1519
1519
Slide 1520
1520
Slide 1521
1521
Slide 1522
1522
Slide 1523
1523
Slide 1524
1524
Slide 1525
1525
Slide 1526
1526
Slide 1527
1527
Slide 1528
1528
Slide 1529
1529
Slide 1530
1530
Slide 1531
1531
Slide 1532
1532
Slide 1533
1533
Slide 1534
1534
Slide 1535
1535
Slide 1536
1536
Slide 1537
1537
Slide 1538
1538
Slide 1539
1539
Slide 1540
1540
Slide 1541
1541
Slide 1542
1542
Slide 1543
1543
Slide 1544
1544
Slide 1545
1545
Slide 1546
1546
Slide 1547
1547
Slide 1548
1548
Slide 1549
1549
Slide 1550
1550
Slide 1551
1551
Slide 1552
1552
Slide 1553
1553
Slide 1554
1554
Slide 1555
1555
Slide 1556
1556
Slide 1557
1557
Slide 1558
1558
Slide 1559
1559
Slide 1560
1560
Slide 1561
1561
Slide 1562
1562
Slide 1563
1563
Slide 1564
1564
Slide 1565
1565
Slide 1566
1566
Slide 1567
1567
Slide 1568
1568
Slide 1569
1569
Slide 1570
1570
Slide 1571
1571
Slide 1572
1572
Slide 1573
1573
Slide 1574
1574
Slide 1575
1575
Slide 1576
1576
Slide 1577
1577
Slide 1578
1578
Slide 1579
1579
Slide 1580
1580
Slide 1581
1581
Slide 1582
1582
Slide 1583
1583
Slide 1584
1584
Slide 1585
1585
Slide 1586
1586
Slide 1587
1587
Slide 1588
1588
Slide 1589
1589
Slide 1590
1590
Slide 1591
1591
Slide 1592
1592
Slide 1593
1593
Slide 1594
1594
Slide 1595
1595
Slide 1596
1596
Slide 1597
1597
Slide 1598
1598
Slide 1599
1599
Slide 1600
1600
Slide 1601
1601
Slide 1602
1602
Slide 1603
1603
Slide 1604
1604
Slide 1605
1605
Slide 1606
1606
Slide 1607
1607
Slide 1608
1608
Slide 1609
1609
Slide 1610
1610
Slide 1611
1611
Slide 1612
1612
Slide 1613
1613
Slide 1614
1614
Slide 1615
1615
Slide 1616
1616
Slide 1617
1617
Slide 1618
1618
Slide 1619
1619
Slide 1620
1620
Slide 1621
1621
Slide 1622
1622
Slide 1623
1623
Slide 1624
1624
Slide 1625
1625
Slide 1626
1626
Slide 1627
1627
Slide 1628
1628
Slide 1629
1629
Slide 1630
1630
Slide 1631
1631
Slide 1632
1632
Slide 1633
1633
Slide 1634
1634
Slide 1635
1635
Slide 1636
1636
Slide 1637
1637
Slide 1638
1638
Slide 1639
1639
Slide 1640
1640
Slide 1641
1641
Slide 1642
1642
Slide 1643
1643
Slide 1644
1644
Slide 1645
1645
Slide 1646
1646
Slide 1647
1647
Slide 1648
1648
Slide 1649
1649
Slide 1650
1650
Slide 1651
1651
Slide 1652
1652
Slide 1653
1653
Slide 1654
1654
Slide 1655
1655
Slide 1656
1656
Slide 1657
1657
Slide 1658
1658
Slide 1659
1659
Slide 1660
1660
Slide 1661
1661
Slide 1662
1662
Slide 1663
1663
Slide 1664
1664
Slide 1665
1665
Slide 1666
1666
Slide 1667
1667
Slide 1668
1668
Slide 1669
1669
Slide 1670
1670
Slide 1671
1671
Slide 1672
1672
Slide 1673
1673
Slide 1674
1674
Slide 1675
1675
Slide 1676
1676
Slide 1677
1677
Slide 1678
1678
Slide 1679
1679
Slide 1680
1680
Slide 1681
1681
Slide 1682
1682
Slide 1683
1683
Slide 1684
1684
Slide 1685
1685
Slide 1686
1686
Slide 1687
1687
Slide 1688
1688
Slide 1689
1689
Slide 1690
1690
Slide 1691
1691
Slide 1692
1692
Slide 1693
1693
Slide 1694
1694
Slide 1695
1695
Slide 1696
1696
Slide 1697
1697
Slide 1698
1698
Slide 1699
1699
Slide 1700
1700
Slide 1701
1701
Slide 1702
1702
Slide 1703
1703
Slide 1704
1704
Slide 1705
1705
Slide 1706
1706
Slide 1707
1707
Slide 1708
1708
Slide 1709
1709
Slide 1710
1710
Slide 1711
1711
Slide 1712
1712
Slide 1713
1713
Slide 1714
1714
Slide 1715
1715
Slide 1716
1716
Slide 1717
1717
Slide 1718
1718
Slide 1719
1719
Slide 1720
1720
Slide 1721
1721
Slide 1722
1722
Slide 1723
1723
Slide 1724
1724
Slide 1725
1725
Slide 1726
1726
Slide 1727
1727
Slide 1728
1728
Slide 1729
1729
Slide 1730
1730
Slide 1731
1731
Slide 1732
1732
Slide 1733
1733
Slide 1734
1734
Slide 1735
1735
Slide 1736
1736
Slide 1737
1737
Slide 1738
1738
Slide 1739
1739
Slide 1740
1740
Slide 1741
1741
Slide 1742
1742
Slide 1743
1743
Slide 1744
1744
Slide 1745
1745
Slide 1746
1746
Slide 1747
1747
Slide 1748
1748
Slide 1749
1749
Slide 1750
1750
Slide 1751
1751
Slide 1752
1752
Slide 1753
1753
Slide 1754
1754
Slide 1755
1755
Slide 1756
1756
Slide 1757
1757
Slide 1758
1758
Slide 1759
1759
Slide 1760
1760
Slide 1761
1761
Slide 1762
1762
Slide 1763
1763
Slide 1764
1764
Slide 1765
1765
Slide 1766
1766
Slide 1767
1767
Slide 1768
1768
Slide 1769
1769
Slide 1770
1770
Slide 1771
1771
Slide 1772
1772
Slide 1773
1773
Slide 1774
1774
Slide 1775
1775
Slide 1776
1776
Slide 1777
1777
Slide 1778
1778
Slide 1779
1779
Slide 1780
1780
Slide 1781
1781
Slide 1782
1782
Slide 1783
1783
Slide 1784
1784
Slide 1785
1785
Slide 1786
1786
Slide 1787
1787
Slide 1788
1788
Slide 1789
1789
Slide 1790
1790
Slide 1791
1791
Slide 1792
1792
Slide 1793
1793
Slide 1794
1794
Slide 1795
1795
Slide 1796
1796
Slide 1797
1797
Slide 1798
1798
Slide 1799
1799
Slide 1800
1800
Slide 1801
1801
Slide 1802
1802
Slide 1803
1803
Slide 1804
1804
Slide 1805
1805
Slide 1806
1806
Slide 1807
1807
Slide 1808
1808
Slide 1809
1809
Slide 1810
1810
Slide 1811
1811
Slide 1812
1812
Slide 1813
1813
Slide 1814
1814
Slide 1815
1815
Slide 1816
1816
Slide 1817
1817
Slide 1818
1818
Slide 1819
1819
Slide 1820
1820
Slide 1821
1821
Slide 1822
1822
Slide 1823
1823
Slide 1824
1824
Slide 1825
1825
Slide 1826
1826
Slide 1827
1827
Slide 1828
1828
Slide 1829
1829
Slide 1830
1830
Slide 1831
1831
Slide 1832
1832
Slide 1833
1833
Slide 1834
1834
Slide 1835
1835
Slide 1836
1836
Slide 1837
1837
Slide 1838
1838
Slide 1839
1839
Slide 1840
1840
Slide 1841
1841
Slide 1842
1842
Slide 1843
1843
Slide 1844
1844
Slide 1845
1845
Slide 1846
1846
Slide 1847
1847
Slide 1848
1848
Slide 1849
1849
Slide 1850
1850
Slide 1851
1851
Slide 1852
1852
Slide 1853
1853
Slide 1854
1854
Slide 1855
1855
Slide 1856
1856
Slide 1857
1857
Slide 1858
1858
Slide 1859
1859
Slide 1860
1860
Slide 1861
1861
Slide 1862
1862
Slide 1863
1863
Slide 1864
1864
Slide 1865
1865
Slide 1866
1866
Slide 1867
1867
Slide 1868
1868
Slide 1869
1869
Slide 1870
1870
Slide 1871
1871
Slide 1872
1872
Slide 1873
1873
Slide 1874
1874
Slide 1875
1875
Slide 1876
1876
Slide 1877
1877
Slide 1878
1878
Slide 1879
1879
Slide 1880
1880
Slide 1881
1881
Slide 1882
1882
Slide 1883
1883
Slide 1884
1884
Slide 1885
1885
Slide 1886
1886
Slide 1887
1887
Slide 1888
1888
Slide 1889
1889
Slide 1890
1890
Slide 1891
1891
Slide 1892
1892
Slide 1893
1893
Slide 1894
1894
Slide 1895
1895
Slide 1896
1896
Slide 1897
1897
Slide 1898
1898
Slide 1899
1899
Slide 1900
1900
Slide 1901
1901
Slide 1902
1902
Slide 1903
1903
Slide 1904
1904
Slide 1905
1905
Slide 1906
1906
Slide 1907
1907
Slide 1908
1908
Slide 1909
1909
Slide 1910
1910
Slide 1911
1911
Slide 1912
1912
Slide 1913
1913
Slide 1914
1914
Slide 1915
1915
Slide 1916
1916
Slide 1917
1917
Slide 1918
1918
Slide 1919
1919
Slide 1920
1920
Slide 1921
1921
Slide 1922
1922
Slide 1923
1923
Slide 1924
1924
Slide 1925
1925
Slide 1926
1926
Slide 1927
1927
Slide 1928
1928
Slide 1929
1929
Slide 1930
1930
Slide 1931
1931
Slide 1932
1932
Slide 1933
1933
Slide 1934
1934
Slide 1935
1935
Slide 1936
1936
Slide 1937
1937
Slide 1938
1938
Slide 1939
1939
Slide 1940
1940
Slide 1941
1941
Slide 1942
1942
Slide 1943
1943
Slide 1944
1944
Slide 1945
1945
Slide 1946
1946
Slide 1947
1947
Slide 1948
1948
Slide 1949
1949
Slide 1950
1950
Slide 1951
1951
Slide 1952
1952
Slide 1953
1953
Slide 1954
1954
Slide 1955
1955
Slide 1956
1956
Slide 1957
1957
Slide 1958
1958
Slide 1959
1959
Slide 1960
1960
Slide 1961
1961
Slide 1962
1962
Slide 1963
1963
Slide 1964
1964
Slide 1965
1965
Slide 1966
1966
Slide 1967
1967
Slide 1968
1968
Slide 1969
1969
Slide 1970
1970
Slide 1971
1971
Slide 1972
1972
Slide 1973
1973
Slide 1974
1974
Slide 1975
1975
Slide 1976
1976
Slide 1977
1977
Slide 1978
1978
Slide 1979
1979
Slide 1980
1980
Slide 1981
1981
Slide 1982
1982
Slide 1983
1983
Slide 1984
1984
Slide 1985
1985
Slide 1986
1986
Slide 1987
1987
Slide 1988
1988
Slide 1989
1989
Slide 1990
1990
Slide 1991
1991
Slide 1992
1992
Slide 1993
1993
Slide 1994
1994
Slide 1995
1995
Slide 1996
1996
Slide 1997
1997
Slide 1998
1998
Slide 1999
1999
Slide 2000
2000
Slide 2001
2001
Slide 2002
2002
Slide 2003
2003
Slide 2004
2004
Slide 2005
2005
Slide 2006
2006
Slide 2007
2007
Slide 2008
2008
Slide 2009
2009
Slide 2010
2010
Slide 2011
2011
Slide 2012
2012
Slide 2013
2013
Slide 2014
2014
Slide 2015
2015
Slide 2016
2016
Slide 2017
2017
Slide 2018
2018
Slide 2019
2019
Slide 2020
2020
Slide 2021
2021
Slide 2022
2022
Slide 2023
2023
Slide 2024
2024
Slide 2025
2025
Slide 2026
2026
Slide 2027
2027
Slide 2028
2028
Slide 2029
2029
Slide 2030
2030
Slide 2031
2031
Slide 2032
2032
Slide 2033
2033
Slide 2034
2034
Slide 2035
2035
Slide 2036
2036
Slide 2037
2037
Slide 2038
2038
Slide 2039
2039
Slide 2040
2040
Slide 2041
2041
Slide 2042
2042
Slide 2043
2043
Slide 2044
2044
Slide 2045
2045
Slide 2046
2046
Slide 2047
2047
Slide 2048
2048
Slide 2049
2049
Slide 2050
2050
Slide 2051
2051
Slide 2052
2052
Slide 2053
2053
Slide 2054
2054
Slide 2055
2055
Slide 2056
2056
Slide 2057
2057
Slide 2058
2058
Slide 2059
2059
Slide 2060
2060
Slide 2061
2061
Slide 2062
2062
Slide 2063
2063
Slide 2064
2064
Slide 2065
2065
Slide 2066
2066
Slide 2067
2067
Slide 2068
2068
Slide 2069
2069
Slide 2070
2070
Slide 2071
2071
Slide 2072
2072
Slide 2073
2073
Slide 2074
2074
Slide 2075
2075
Slide 2076
2076
Slide 2077
2077
Slide 2078
2078
Slide 2079
2079
Slide 2080
2080
Slide 2081
2081
Slide 2082
2082
Slide 2083
2083
Slide 2084
2084
Slide 2085
2085
Slide 2086
2086
Slide 2087
2087
Slide 2088
2088
Slide 2089
2089
Slide 2090
2090
Slide 2091
2091
Slide 2092
2092
Slide 2093
2093
Slide 2094
2094
Slide 2095
2095
Slide 2096
2096
Slide 2097
2097
Slide 2098
2098
Slide 2099
2099
Slide 2100
2100
Slide 2101
2101
Slide 2102
2102
Slide 2103
2103
Slide 2104
2104
Slide 2105
2105
Slide 2106
2106
Slide 2107
2107
Slide 2108
2108
Slide 2109
2109
Slide 2110
2110
Slide 2111
2111
Slide 2112
2112
Slide 2113
2113
Slide 2114
2114
Slide 2115
2115
Slide 2116
2116
Slide 2117
2117
Slide 2118
2118
Slide 2119
2119
Slide 2120
2120
Slide 2121
2121
Slide 2122
2122
Slide 2123
2123
Slide 2124
2124
Slide 2125
2125
Slide 2126
2126
Slide 2127
2127
Slide 2128
2128
Slide 2129
2129
Slide 2130
2130
Slide 2131
2131
Slide 2132
2132
Slide 2133
2133
Slide 2134
2134
Slide 2135
2135
Slide 2136
2136
Slide 2137
2137
Slide 2138
2138
Slide 2139
2139
Slide 2140
2140
Slide 2141
2141
Slide 2142
2142
Slide 2143
2143
Slide 2144
2144
Slide 2145
2145
Slide 2146
2146
Slide 2147
2147
Slide 2148
2148
Slide 2149
2149
Slide 2150
2150
Slide 2151
2151
Slide 2152
2152
Slide 2153
2153
Slide 2154
2154
Slide 2155
2155
Slide 2156
2156
Slide 2157
2157
Slide 2158
2158
Slide 2159
2159
Slide 2160
2160
Slide 2161
2161
Slide 2162
2162
Slide 2163
2163
Slide 2164
2164
Slide 2165
2165
Slide 2166
2166
Slide 2167
2167
Slide 2168
2168
Slide 2169
2169
Slide 2170
2170
Slide 2171
2171
Slide 2172
2172
Slide 2173
2173
Slide 2174
2174
Slide 2175
2175
Slide 2176
2176
Slide 2177
2177
Slide 2178
2178
Slide 2179
2179
Slide 2180
2180
Slide 2181
2181
Slide 2182
2182
Slide 2183
2183
Slide 2184
2184
Slide 2185
2185
Slide 2186
2186
Slide 2187
2187
Slide 2188
2188
Slide 2189
2189
Slide 2190
2190
Slide 2191
2191
Slide 2192
2192
Slide 2193
2193
Slide 2194
2194
Slide 2195
2195
Slide 2196
2196
Slide 2197
2197
Slide 2198
2198
Slide 2199
2199
Slide 2200
2200
Slide 2201
2201
Slide 2202
2202
Slide 2203
2203
Slide 2204
2204
Slide 2205
2205
Slide 2206
2206
Slide 2207
2207
Slide 2208
2208
Slide 2209
2209
Slide 2210
2210
Slide 2211
2211
Slide 2212
2212
Slide 2213
2213
Slide 2214
2214
Slide 2215
2215
Slide 2216
2216
Slide 2217
2217
Slide 2218
2218
Slide 2219
2219
Slide 2220
2220
Slide 2221
2221
Slide 2222
2222
Slide 2223
2223
Slide 2224
2224
Slide 2225
2225
Slide 2226
2226
Slide 2227
2227
Slide 2228
2228
Slide 2229
2229
Slide 2230
2230
Slide 2231
2231
Slide 2232
2232
Slide 2233
2233
Slide 2234
2234
Slide 2235
2235
Slide 2236
2236
Slide 2237
2237
Slide 2238
2238
Slide 2239
2239
Slide 2240
2240
Slide 2241
2241
Slide 2242
2242
Slide 2243
2243
Slide 2244
2244
Slide 2245
2245
Slide 2246
2246
Slide 2247
2247
Slide 2248
2248
Slide 2249
2249
Slide 2250
2250
Slide 2251
2251
Slide 2252
2252
Slide 2253
2253
Slide 2254
2254
Slide 2255
2255
Slide 2256
2256
Slide 2257
2257
Slide 2258
2258
Slide 2259
2259
Slide 2260
2260
Slide 2261
2261
Slide 2262
2262
Slide 2263
2263
Slide 2264
2264
Slide 2265
2265
Slide 2266
2266
Slide 2267
2267
Slide 2268
2268
Slide 2269
2269
Slide 2270
2270
Slide 2271
2271
Slide 2272
2272
Slide 2273
2273
Slide 2274
2274
Slide 2275
2275
Slide 2276
2276
Slide 2277
2277
Slide 2278
2278
Slide 2279
2279
Slide 2280
2280
Slide 2281
2281
Slide 2282
2282
Slide 2283
2283
Slide 2284
2284
Slide 2285
2285
Slide 2286
2286
Slide 2287
2287
Slide 2288
2288
Slide 2289
2289
Slide 2290
2290
Slide 2291
2291
Slide 2292
2292
Slide 2293
2293
Slide 2294
2294
Slide 2295
2295
Slide 2296
2296
Slide 2297
2297
Slide 2298
2298
Slide 2299
2299
Slide 2300
2300
Slide 2301
2301
Slide 2302
2302
Slide 2303
2303
Slide 2304
2304
Slide 2305
2305
Slide 2306
2306
Slide 2307
2307
Slide 2308
2308
Slide 2309
2309
Slide 2310
2310
Slide 2311
2311
Slide 2312
2312
Slide 2313
2313
Slide 2314
2314
Slide 2315
2315
Slide 2316
2316
Slide 2317
2317
Slide 2318
2318
Slide 2319
2319
Slide 2320
2320
Slide 2321
2321
Slide 2322
2322
Slide 2323
2323
Slide 2324
2324
Slide 2325
2325
Slide 2326
2326
Slide 2327
2327
Slide 2328
2328
Slide 2329
2329
Slide 2330
2330
Slide 2331
2331
Slide 2332
2332
Slide 2333
2333
Slide 2334
2334
Slide 2335
2335
Slide 2336
2336
Slide 2337
2337
Slide 2338
2338
Slide 2339
2339
Slide 2340
2340
Slide 2341
2341
Slide 2342
2342
Slide 2343
2343
Slide 2344
2344
Slide 2345
2345
Slide 2346
2346
Slide 2347
2347
Slide 2348
2348
Slide 2349
2349
Slide 2350
2350
Slide 2351
2351
Slide 2352
2352
Slide 2353
2353
Slide 2354
2354
Slide 2355
2355
Slide 2356
2356
Slide 2357
2357
Slide 2358
2358
Slide 2359
2359
Slide 2360
2360
Slide 2361
2361
Slide 2362
2362
Slide 2363
2363
Slide 2364
2364
Slide 2365
2365
Slide 2366
2366
Slide 2367
2367
Slide 2368
2368
Slide 2369
2369
Slide 2370
2370
Slide 2371
2371
Slide 2372
2372
Slide 2373
2373
Slide 2374
2374
Slide 2375
2375
Slide 2376
2376
Slide 2377
2377
Slide 2378
2378
Slide 2379
2379
Slide 2380
2380
Slide 2381
2381
Slide 2382
2382
Slide 2383
2383
Slide 2384
2384
Slide 2385
2385
Slide 2386
2386
Slide 2387
2387
Slide 2388
2388
Slide 2389
2389
Slide 2390
2390
Slide 2391
2391
Slide 2392
2392
Slide 2393
2393
Slide 2394
2394
Slide 2395
2395
Slide 2396
2396
Slide 2397
2397
Slide 2398
2398
Slide 2399
2399
Slide 2400
2400
Slide 2401
2401
Slide 2402
2402
Slide 2403
2403
Slide 2404
2404
Slide 2405
2405
Slide 2406
2406
Slide 2407
2407
Slide 2408
2408
Slide 2409
2409
Slide 2410
2410
Slide 2411
2411
Slide 2412
2412
Slide 2413
2413
Slide 2414
2414
Slide 2415
2415
Slide 2416
2416
Slide 2417
2417
Slide 2418
2418
Slide 2419
2419
Slide 2420
2420
Slide 2421
2421
Slide 2422
2422
Slide 2423
2423
Slide 2424
2424
Slide 2425
2425
Slide 2426
2426
Slide 2427
2427
Slide 2428
2428
Slide 2429
2429
Slide 2430
2430
Slide 2431
2431
Slide 2432
2432
Slide 2433
2433
Slide 2434
2434
Slide 2435
2435
Slide 2436
2436
Slide 2437
2437
Slide 2438
2438
Slide 2439
2439
Slide 2440
2440
Slide 2441
2441
Slide 2442
2442
Slide 2443
2443
Slide 2444
2444
Slide 2445
2445
Slide 2446
2446
Slide 2447
2447
Slide 2448
2448
Slide 2449
2449
Slide 2450
2450
Slide 2451
2451
Slide 2452
2452
Slide 2453
2453
Slide 2454
2454
Slide 2455
2455
Slide 2456
2456
Slide 2457
2457
Slide 2458
2458
Slide 2459
2459
Slide 2460
2460
Slide 2461
2461
Slide 2462
2462
Slide 2463
2463
Slide 2464
2464
Slide 2465
2465
Slide 2466
2466
Slide 2467
2467
Slide 2468
2468
Slide 2469
2469
Slide 2470
2470
Slide 2471
2471
Slide 2472
2472
Slide 2473
2473
Slide 2474
2474
Slide 2475
2475
Slide 2476
2476
Slide 2477
2477
Slide 2478
2478
Slide 2479
2479
Slide 2480
2480
Slide 2481
2481
Slide 2482
2482
Slide 2483
2483
Slide 2484
2484
Slide 2485
2485
Slide 2486
2486
Slide 2487
2487
Slide 2488
2488

About This Presentation

des


Slide Content

!"#កទី ១
Basice Science 2076
No Question
1
An HIV-positive patient asks you if you can tell him the chances of him progressing to
symptomatic AIDS. Which one of the following tests would be most useful?
A CD4 lymphocyte count
B HIV antibody test
C HIV RT PCR
D HIV p24 antigen
ANSWER:C
2
Which of the following viruses causes an acute febrile rash and produces disease in
immunocompetent children but has been associated with transient aplastic crises in persons
with sickle cell disease?
A Rubeola
B Varicella-zoster
C Parvovirus
D Rubella
ANSWER:C
3
Infection with herpes simplex virus, a common human pathogen, is best described by which
of the following statements?
A The CNS and visceral organs are usually involved
B It rarely recurs in a host who has a high antibody titer
C It can be reactivated by emotional disturbances or prolonged exposure to sunlight
D Initial infection usually occurs by intestinal absorption of the virus
ANSWER:C
4
The latest and most effective therapy for AIDS patients includes azidothymidine (AZT),
dideoxyinosine (DDI), and saquinavir or similar agents. Use of these three drugs would
inhibit which of the following viral processes?
A RNase, DNase
B gp120 formation
C p24 antibody expression
D Reverse transcriptase, protease
ANSWER:D

5
An HIV-positive patient prior to being treated with AZT, DDI, and saquinavir has a CD4
lymphocyte count and an HIV RNA viral load test done. Results are as follows: CD4: 50 CD4
lymphocytes per microliter HIV RNA: 750,000 copies per ml Which of the following
statements best describes the above patient?
A This patient is no longer in danger of opportunistic infection
B The 5-year prognosis is excellent
C The patient’s HIV screening test is most likely negative
D The viral load of 750,000 copies per ml suggests that the patient will respond to triple therapy
ANSWER:D
6
This HIV-positive patient with a viral load of 750,000 copies of HIV RNA/ml and a total
CD4 count of 50 is at an increased risk for a number of infectious diseases. For which of the
following diseases is the patient at no more added risk than an immunocompetent host?
A Pneumocystic pneumonia
B Mycobacterial disease
C Kaposi’s sarcoma
D Pneumococcal pneumonia
ANSWER:D
7
Infectious mononucleosis, a viral disorder that can be debilitating, is characterized by which
of the following statements?
A It is most prevalent in children less than 14 years old
B It is caused by a rhabdovirus
C The causative pathogen is an Epstein-Barr virus
D Affected persons respond to treatment with the production of heterophil antibodies
ANSWER:C
8
A tube of monkey kidney cells is inoculated with nasopharyngeal secretions. During the next
7 days, no cytopathic effects (CPEs) are observed. On the eighth day, the tissue culture is
infected accidentally with a picornavirus; nevertheless, the culture does not develop CPEs.
The most likely explanation of this phenomenon is that
A The nasopharyngeal secretions contained hemagglutinins
B The nasopharyngeal secretions contained rubella virus
C Picornavirus does not produce CPEs
D Picornavirus does not replicate in monkey kidney cells

ANSWER:B
9
The clinical picture of arbovirus infection fits one of three categories: encephalitis,
hemorrhagic fever, or fever with myalgia. One of the characteristics of arboviruses is that
they
A Are transmitted by arthropod vectors
B Are usually resistant to ether
C Usually cause symptomatic infection in humans
D Are closely related to parvoviruses
ANSWER:A
10
Which one of the following statements best describes interferon’s suspected mode of action
in producing resistance to viral infection?
A It stimulates a cell-mediated immunity
B It stimulates humoral immunity
C Its direct antiviral action is related to the suppression of messenger RNA formation
D Its action is related to the synthesis of a protein that inhibits translation or transcription
ANSWER:D
11
Coronaviruses are recognized by club-shaped surface projections that are 20 nm long and
resemble solar coronas. These viruses are characterized by their ability to
A Infect infants more frequently than adults
B Cause the common cold
C Grow well in the usual cultured cell lines
D Grow profusely at 50°C
ANSWER:B
12
Which of the following antiviral agents is a purine nucleoside analogue that has shown
promise with Lassa fever, influenza A and B, and respiratory syncytial virus (RSV)?
A Amantadine
B Rimantadine
C Vidarabine
D Ribavirin
ANSWER:D
13 Echoviruses are cytopathogenic human viruses that mainly infect the
A Respiratory system
B Central nervous system

C Blood and lymphatic systems
D Intestinal tract
ANSWER:D
14
The most sensitive test for the diagnosis of herpes simplex (HSV)
meningitis in a newborn infant is
A HSV IgG antibody
B HSV polymerase chain reaction (PCR)
C HSV culture
D Tzanck smear
ANSWER:B
15
Acute hemorrhagic conjunctivitis (AHC) is a contagious ocular infection characterized by
pain, swelling of the eyelids, and subconjunctival hemorrhages. AHC has been reported to be
caused by which of the following viruses?
A Coronavirus
B Reovirus
C Rhinovirus
D Enterovirus
ANSWER:D
16
Mumps virus accounts for 10 to 15% of all cases of aseptic meningitis in the United States.
Infection with mumps virus
A Is preventable by immunization
B Will usually cause mumps orchitis in postpubertal males
C Is maintained in a large canine reservoir
D Usually produces severe systemic manifestations
ANSWER:A
17
The serum of a newborn infant reveals a 1:32 cytomegalovirus (CMV) titer. The child is
clinically asymptomatic. Which of the following courses of action would be advisable?
A Repeat the CMV titer immediately
B Obtain an anti-CMV IgM titer from the baby
C Obtain a CMV titer from all siblings
D Obtain an anti-CMV IgM titer from the mother
ANSWER:B

18
A 3-year-old child presents at the physician’s office with symptoms of coryza, conjunctivitis,
low-grade fever, and Koplik’s spots. The causative agent of this disease belongs to which
group of viruses?
A Adenovirus
B Herpesvirus
C Picornavirus
D Paramyxovirus
ANSWER:D
19
One of the most common sexually transmitted diseases that may lead to cervical carcinoma is
caused by which of the following viruses?
A Cytomegalovirus
B Papillomavirus
C Epstein-Barr virus
D Herpes simplex virus
ANSWER:B
20
Which virus is the leading cause of the croup syndrome in young children and, when
infecting mammalian cells in culture, will hemabsorb red blood cells?
A Group B coxsackievirus
B Rotavirus
C Parainfluenza virus
D Adenovirus
ANSWER:C
21
Hepatitis E, a recently characterized hepatitis virus, is best described by which of the
following statements?
A It is not a threat to the blood supply
B It is a major cause of blood-borne hepatitis
C It is prevalent in North America
D It is a single-stranded DNA virus
ANSWER:A
22
Meningitis is characterized by the acute onset of fever and stiff neck. Aseptic meningitis may
be caused by a variety of microbial agents. During the initial 24 h of the course of aseptic
meningitis, an affected person’s cerebrospinal fluid is characterized by
A Decreased protein content

B Elevated glucose concentration
C Lymphocytosis
D Polymorphonuclear leukocytosis
ANSWER:D
23
A nurse develops clinical symptoms consistent with hepatitis. She recalls sticking herself with
a needle approximately 4 months before after drawing blood from a patient. Serologic tests
for HBsAg, antibodies to HBsAg, and hepatitis A virus (HAV) are all negative; however, she
is positive for IgM core antibody. The nurse
A Does not have hepatitis B
B Has hepatitis A
C Is in the late stages of hepatitis B infection
D Is in the “window” (after the disappearance of HBsAg and before the appearance
ANSWER:D
24
Eastern equine encephalitis virus is associated with a high fatality rate. Control of the disease
could be possible by eradication of
A Horses
B Birds
C Mosquitoes
D Fleas
ANSWER:C
25
Adults who have had varicella as children occasionally suffer a recurrent form of the disease,
shingles. The agent causing these diseases is a member of which of the following viral
families?
A Herpesvirus
B Poxvirus
C Adenovirus
D Myxovirus
ANSWER:A
26 Rhinovirus is primarily transmitted by
A Droplet aerosolization
B Sexual activity
C Fecal-oral route
D Fomites

ANSWER:D
27
German measles virus (rubella), a common cause of exanthems in children, is best described
by which of the following statements?
A Measles (rubeola) and German measles (rubella) are caused by the same virus
B Incubation time is approximately 3 to 4 weeks
C Vesicular rashes are characteristic
D Onset is abrupt with cough, coryza, and fever
ANSWER:D
28 The presence of Negri inclusion bodies in host cells is characteristic of
A Mumps
B Infectious mononucleosis
C Congenital rubella
D Rabies
ANSWER:D
29
Kuru is a fatal disease of certain New Guinea natives and is characterized by tremors and
ataxia; Creutzfeldt-Jakob disease (CJD) is characterized by both ataxia and dementia. These
diseases are thought to be caused by
A Slow viruses
B Cell wall–deficient bacteria
C Environmental toxins
D Prions
ANSWER:D
30
According to recommendations issued by the U.S. Public Health Service, which of the
following statements regarding vaccination against smallpox is true?
A Pregnant women should be vaccinated in the first trimester
B Persons who have eczema should be vaccinated soon after diagnosis
C Persons who have immune deficiencies should be vaccinated every 5 years
D Persons traveling abroad need not be vaccinated
ANSWER:D
31
Hepatitis D virus (delta agent) is a defective virus that can replicate only in cells already
infected with which of the following viruses?
A Hepatitis A virus
B Epstein-Barr virus

C Hepatitis G virus
D Hepatitis B virus
ANSWER:D
32
A patient presents with keratoconjunctivitis. The differential diagnosis should include
infection with which of the following viruses
A Parvovirus
B Adenovirus
C Epstein-Barr virus
D Respiratory syncytial virus
ANSWER:B
33
A hospital worker is found to have hepatitis B surface antigen. Subsequent tests reveal the
presence of e antigen as well. The worker most likely
A Is infective and has active hepatitis
B Is infective but does not have active hepatitis
C Is not infective
D Is evincing a biologic false-positive test for hepatitis
ANSWER:A
34 Alphavirus causes which one of the following viral diseases?
A Marburg virus disease
B St. Louis encephalitis
C Western equine encephalitis
D Dengue
ANSWER:C
35
Several antiviral compounds have been developed during the last decade. One such
compound is ribavirin, a synthetic nucleoside structurally related to guanosine. Ribavirin
therapy has been successfully used against
A Respiratory syncytial virus
B Herpes simplex virus
C Hepatitis B
D Group A coxsackievirus
ANSWER:A

36
An immunocompromised person with history of seizures had an MRI that revealed a
temporal lobe lesion. Brain biopsy results showed multinucleated giant cells with intranuclear
inclusions. The most probable cause of the lesion is
A Hepatitis C virus
B Herpes simplex virus
C Listeria monocytogenes
D Coxsackievirus
ANSWER:B
37
Which of the following procedures or clinical signs is most specific for the diagnosis of
infectious mononucleosis caused by the Epstein-Barr virus?
A
Laboratory diagnosis is based on the presence of “atypical lymphocytes” and EBV-specific
antibody
B Growth in tissue culture cells
C Heterophile antibodies in serum
D Lymphadenopathy and splenomegaly on physical examination
ANSWER:B
38
An infant, seen in the ER, presents with a fever and persistent cough. Physical examination
and a chest x-ray suggest pneumonia. Which of the following is most likely the cause of this
infection?
A Rotavirus
B Adenovirus
C Coxsackievirus
D Respiratory syncytial virus
ANSWER:D
39 Which one of the following groups of people may be at increased risk for HIV infection?
A Members of a household in which there is a person who is HIV-positive
B Receptionists at a hospital
C Factory workers whose coworkers are HIV-positive
D Foreign service employees who are hospitalized in Zaire for bleeding ulcers
ANSWER:D
40
An obstetrician sees a pregnant patient who was exposed to rubella virus in the eighteenth
week of pregnancy. She does not remember getting a rubella vaccination. The best
immediate course of action is to

A Terminate the pregnancy
B Order a rubella antibody titer to determine immune status
C Reassure the patient because rubella is not a problem until after the thirtieth week
D Administer rubella immune globulin
ANSWER:B
41
Mad Cow Disease has been highly publicized in Great Britain. This disease, which is similar
to scrapie, is caused by
A A prion
B A virus
C Rickettsiae
D An autoimmune reaction
ANSWER:A
42
A patient has all the gastrointestinal symptoms of infection with hepatitis A virus (HAV), yet
all the tests for HAV-IgG and HAV-IgM are nonreactive. A possible cause of this infection is
A Hepatitis B surface antigen
B Hepatitis C
C Hepatitis D
D Hepatitis E
ANSWER:D
43
A 70-year-old nursing home patient refused the influenza vaccine and subsequently
developed influenza. She died of acute pneumonia 1 week after contracting the “flu.” The
most common cause of acute postinfluenzal pneumonia is
A Legionella
B Listeria
C Staphylococcus aureus
D Klebsiella
ANSWER:C
44 Which of the following viruses is primarily transmitted by the fecaloral route?
A St. Louis encephalitis virus
B Colorado tick fever virus
C Coxsackievirus
D Yellow fever virus

ANSWER:C
45
Hantavirus is an emerging pathogen that is best described by which of the following
statements?
A Influenza-like symptoms are followed rapidly by acute respiratory failure
B Hemolysis is common in infected patients
C It is acquired by inhalation of aerosols of the urine and feces of deer
D Transmission from human to human is common
ANSWER:A
46 Erythema infectiosum (fifth disease), a self-limited disease of children, is caused by
A Measles
B Parvovirus
C Rubella
D Human herpesvirus type 6
ANSWER:B
47 Which one of the following viruses may be human tumor virus?
A Epstein-Barr virus (EBV)
B HIV
C Papillomavirus
D Varicella-zoster virus (VZV)
ANSWER:C
48 Parvovirus infection, the cause of a mild exanthem in children, is characterized by
A Epidemic acute respiratory disease
B Gastroenteritis
C Whooping cough–like disease
D Acute hemolytic anemia
ANSWER:D
49
Cytomegalovirus (CMV) infection is common. Which one of the following statements best
characterizes CMV?
A It can be transmitted across the placental barrier
B While a common infection, CMV is almost always symptomatic
C The CMV can be cultured from red blood cells of infected patients
D Unlike other viral infections, CMV is not activated by immunosuppressive therapy
ANSWER:A

50 Human rotaviruses are characterized by which of the following statements?
A They produce an infection that is primarily seen in adults
B They produce cytopathic effects in many conventional tissue culture systems
C They are lipid-containing RNA viruses possessing a double-shelled capsid
D
They can be sensitively and rapidly detected in stools by the enzyme-linked immunosorbent
assay (ELISA) technique
ANSWER:D
51
Subacute sclerosing panencephalitis virus (SSPE) is best described by which of the following
statements?
A It is a progressive disease involving both white and gray matter
B It is a late CNS manifestation of mumps
C It is a common event occurring in 1 of 300,000 cases of mumps
D Viral DNA can be demonstrated in brain cells
ANSWER:A
52
Rotavirus is a double-stranded RNA virus with a double-walled capsid. Which one of the
following statements best describes rotavirus?
A There are no related animal viruses
B It is a major cause of neonatal diarrhea
C It is readily cultured from the stool of infected persons
D Maternal antibody does not appear to be protective
ANSWER:B
53 Paramyxoviruses are most commonly associated with which of the following diseases?
A Fifth disease
B Rubella
C Croup
D Tonsillitis
ANSWER:C
54 Human papillomavirus is most commonly associated with
A Rectal polyps
B Prostate cancer
C Condyloma acuminatum
D Hepatic carcinoma
ANSWER:D

55
Reverse transcriptase is an enzyme unique to the retroviruses. Which one of the following is
a function of the enzyme reverse transcriptase?
A DNase activity
B RNA-dependent RNA polymerase activity
C RNA isomerase activity
D RNA-dependent DNA polymerase activity
ANSWER:D
56
St. Louis encephalitis, a viral infection, was first recognized as an entity in 1933. Which of
the following best describes SLE?
A It is transmitted to humans by the bite of an infected tick
B It is caused by a togavirus
C It is the major arboviral cause of central nervous system infection in the United States
D It may present initially with symptoms similar to influenza
ANSWER:C
57
There is considerable overlap of signs and symptoms seen in congenital and perinatal
infections. In a neonate with “classic” symptoms of congenital cytomegalovirus (CMV)
infection, which one of the following tests would be most useful in establishing a diagnosis?
A CMV IgG titer on neonate’s serum at birth
B CMV IgG titer on mother’s serum at birth of infant
C CMV IgM titer on neonate’s serum at birth and at 1 month of age
D Total IgM on neonate’s serum at birth
ANSWER:C
58
Interferon, a protein that inhibits viral replication, is produced by cells in tissue culture when
the cells are stimulated with which of the following?
A Botulinum toxin
B Synthetic polypeptides
C Viruses
D Chlamydiae
ANSWER:C
59
Which one of the following statements best describes the cytopathic effects of viruses on host
cells?
A Usually morphological in nature

B Often associated with changes in mitochondrial membranes
C Pathognomonic for an infecting virus
D Rarely fatal to the host cell
ANSWER:A
60
A 17-year-old girl presents with cervical lymphadenopathy, fever, and pharyngitis.
Infectious mononucleosis is suspected. The most rapid and clinically useful test to make this
diagnosis is
A IgM antibody to viral core antigen (VCA)
B IgG antibody to VCA
C Antibody to Epstein-Barr nuclear antigen (EBNA)
D Culture
ANSWER:A
61 Which one of the following viruses would be most likely to establish a​​ latent infection?
A Adenovirus
B Measles virus
C Influenza virus
D Parvovirus
ANSWER:A
62
A regimen that includes appropriately administered gamma globulin may be contraindicated
in which one of the following diseases?
A Hepatitis A
B Hepatitis B
C Rabies
D Infectious mononucleosis
ANSWER:D
63 Atypical lymphocytosis is most likely to be found in which one of the following diseases?
A Encephalitis caused by herpes simplex virus (HSV)
B Mononucleosis induced by Epstein-Barr virus
C Parvovirus infection
D Chronic hepatitis C
ANSWER:B
64
A patient has arthralgia, a rash, lymphadenopathy, pneumonia but no fever. Which of the
following diseases is most likely based on these symptoms?

A Dengue fever
B St. Louis encephalitis
C Infectious mononucleosis
D Hepatitis
ANSWER:A
65
Hepatitis C (HCV) is usually a clinically mild disease, with only minimal elevation of liver
enzymes. Hospitalization is unusual. Which one of the following statements best characterizes
HCV?
A Few cases progress to chronic liver disease
B It often occurs in posttransfusion patients
C HBV but not HCV infections occur in IV drug abusers
D It is a DNA virus
ANSWER:B
66
Which of the following markers is usually the first viral marker detected after hepatitis B
infection?
A HBeAg
B HBsAg
C HBcAg
D Anti-HBc
ANSWER:B
67
Which of the following may be the only detectable serological marker during the early
convalescent phase of HBV infection (window phase)?
A HBeAg
B HBsAg
C HBcAg
D Anti-HBc
ANSWER:D
68
Which one of the following markers is closely associated with HBV infectivity and DNA
polymerase activity?
A HBeAg
B HBsAg
C HBcAg
D Anti-HBc

ANSWER:A
69
Which of the following is found within the nuclei of infected hepatocytes and not usually in
the peripheral circulation?
A HBeAg
B HBsAg
C HBcAg
D HbeAb
ANSWER:D
70 Which one of the following viruses is the leading cause of congenital malformations?
A Rabies
B Rhinovirus
C Cytomegalovirus
D Respiratory syncytial virus
ANSWER:C
71 Orchitis, which may cause sterility, is a possible manifestation of which of the following?
A Rabies
B Rhinovirus
C Cytomegalovirus
D Mumps
ANSWER:D
72 Which of the following is a leading cause of pneumonia primarily in infants?
A Rabies
B Rhinovirus
C Cytomegalovirus
D Respiratory syncytial virus
ANSWER:D
73 Which of the following causes a fatal encephalitis for which a vaccine is available?
A Rabies
B Rhinovirus
C Cytomegalovirus
D Respiratory syncytial virus
ANSWER:A

74
Traditional vaccination for the common cold is virtually impossible because there are
multiple serotypes of which one of the following viruses?
A Rabies
B Rhinovirus
C Cytomegalovirus
D Respiratory syncytial virus
ANSWER:B
75 Which of the following is available and effective for hepatitis A?
A Acyclovir
B Killed virus vaccine
C Inactivated virus vaccine
D Live virus vaccine
ANSWER:B
76 Patients should be vaccinated annually for influenza with which of the following vaccines?
A Immune serum globulin
B Killed virus vaccine
C Inactivated virus vaccine
D Live virus vaccine
ANSWER:C
77 The vaccine for measles is best characterized as a
A Bacterin
B Killed virus vaccine
C Inactivated virus vaccine
D Live virus vaccine
ANSWER:D
78 Which one of the following would be the treatment of choice for HSV infection?
A Acyclovir
B Killed virus vaccine
C Herpes immune globulin
D Azythromycin
ANSWER:A
79 Which of the following best describes the presently available vaccine for hepatitis B?
A Synthetic peptide vaccine

B Killed virus vaccine
C Inactivated virus vaccine
D Recombinant viral vaccine
ANSWER:D
80
Chicken pox is a common disease of childhood. It is caused by which of the following
viruses?
A Cytomegalovirus
B Rotavirus
C Varicella-zoster virus
D Adenovirus
ANSWER:C
81
Excluding influenza, which one of the following viruses is a common cause of acute
respiratory disease?
A Cytomegalovirus
B Rotavirus
C Varicella-zoster virus
D Adenovirus
ANSWER:D
82
Human warts are not only cosmetically unsightly but may lead to cancer of the cervix. They
are caused by which one of the following viruses?
A Cytomegalovirus
B Rotavirus
C Varicella-zoster virus
D Papillomavirus
ANSWER:D
83
A vaccine is available for one of the most common causes of infantile gastroenteritis.
However, it has recently been recalled. The virus is
A Cytomegalovirus
B Rotavirus
C Varicella-zoster virus
D Adenovirus
ANSWER:B

84
A child has mononucleosis-like symptoms yet the test for mononucleosis and the EBV titers
are negative. One of the causes of heterophilenegative mononucleosis is
A Cytomegalovirus
B Herpes simplex virus
C Varicella-zoster virus
D Adenovirus
ANSWER:A
85
Malaise and fatigue with increased “atypical” lymphocytes and a reactive heterophil antibody
test is most commonly caused by
A Toxoplasma
B Borrelia burgdorferi
C Epstein-Barr virus
D Parvovirus
ANSWER:C
86
Lethargy, malaise, and fatigue are observed in a patient 2 weeks after eating raw hamburger
at a restaurant. The most likely infectious cause is
A Toxoplasma
B Cytomegalovirus
C E. coli
D Salmonella
ANSWER:A
87
Burkitt’s lymphoma is characterized by elevated “early antigen” tests with a restricted pattern
of fluorescence. This disease is caused by
A Cytomegalovirus
B B. burgdorferi
C Epstein-Barr virus
D Lymphogranuloma venereum
ANSWER:C
88
This virus may be detected by the polymerase chain reaction (PCR) in a variety of cells of
patients with nasopharyngeal carcinoma.
A Measles
B Mumps
C Rubella

D Parvovirus
ANSWER:B
89
This virus causes a mononucleosis-like syndrome caused by a latent herpesvirus; it is often a
congenital infection. Large amounts of the virus are excreted in the urine; thus, urine
becomes the fluid of choice for diagnosis of this disease.
A Epstein-Barr virus
B Cytomegalovirus
C HHV-6
D Parvovirus
ANSWER:B
90
Assume you are asked by a resident what the most appropriate specimen is for the detection
of a particular virus. Human papillomavirus
A Cervical tissue
B Synovial fluid
C Blood
D Skin
ANSWER:A
91
Assume you are asked by a resident what the most appropriate specimen is for the detection
of a particular virus. Cytomegalovirus
A Cervical tissue
B Synovial fluid
C Blood
D Skin
ANSWER:C
92
Assume you are asked by a resident what the most appropriate specimen is for the detection
of a particular virus. Enterovirus
A Cervical tissue
B Synovial fluid
C Blood
D Cerebrospinal fluid
ANSWER:D
93
Assume you are asked by a resident what the most appropriate specimen is for the detection
of a particular virus. Varicella-zoster virus (VZV)

A Cervical tissue
B Synovial fluid
C Blood
D Skin
ANSWER:D
94
Assume you are asked by a resident what the most appropriate specimen is for the detection
of a particular virus. Adenovirus 40/41
A Cervical tissue
B Synovial fluid
C Blood
D Stool
ANSWER:D
95
Which of the following is transmitted by the fecal-oral route; can be acquired from shellfish;
and often causes acute jaundice, diarrhea, and liver function abnormalities?
A Rotavirus
B Adenovirus 40/41
C Norwalk virus
D Hepatitis A virus
ANSWER:D
96
Which of the following is the second most common cause of pediatric gastroenteritis? Unlike
other similar viruses, this virus causes only gastroenteritis.
A Rotavirus
B Adenovirus 40/41
C Norwalk virus
D Astrovirus
ANSWER:B
97
Which of the following is the most common cause of pediatric gastroenteritis? It is difficult
to grow in cell culture but can be detected easily by immunologic methods (ELISA).
A Rotavirus
B Adenovirus 40/41
C Norwalk virus
D Astrovirus

ANSWER:A
98
Which of the following is a common cause of epidemic gastroenteritis, particularly aboard
cruise ships and in summer camps? It may be detected by ELISA methods or electron
microscopy.
A Rotavirus
B Adenovirus 40/41
C Norwalk virus
D Astrovirus
ANSWER:C
99
Which of the following is a cause of mild gastroenteritis? It can be transmitted by the fecal-
oral route but not by food consumption.
A Rotavirus
B Adenovirus 40/41
C Norwalk virus
D Astrovirus
ANSWER:D
100
IgM antibody to the viral particle is the method of choice for laboratory diagnosis of which
one of the following hepatitis viruses?
A Hepatitis A
B Hepatitis B
C Hepatitis C
D Hepatitis D
ANSWER:A
101
This virus belongs to the family of flaviviruses and its reservoir is strictly human.
Transmission is blood-borne so the blood supply is routinely screened for this virus.
A Hepatitis A
B Hepatitis B
C Hepatitis C
D Hepatitis D
ANSWER:C
102
Vaccination for this hepatic disease is with viral surface antigen and usually provides
immunity.
A Hepatitis A

B Hepatitis B
C Hepatitis C
D Hepatitis D
ANSWER:B
103
This hepatitis virus is a calicivirus. The reservoir is in pigs, and humans acquire it via the
fecal-oral route.
A Hepatitis A
B Hepatitis B
C Hepatitis C
D Hepatitis E
ANSWER:D
104
This hepatitis virus is a defective virus in that it cannot replicate independently without the
presence of hepatitis B virus.
A Hepatitis A
B Hepatitis B
C Hepatitis C
D Hepatitis D
ANSWER:D
105
Which of the following is the causative agent of a variety of cutaneous warts (plantar,
common, and flat) and is associated with cervical neoplasia?
A Human papillomavirus
B West Nile virus
C Tick-borne encephalitis virus
D Polyomavirus
ANSWER:A
106
Recently appearing in the United States, this virus is carried by birds, transmitted by
mosquitoes, and infects humans and horses.
A Human papillomavirus
B West Nile virus
C Tick-borne encephalitis virus
D Polyomavirus
ANSWER:B

107
Which of the following viruses causes progressive multifocal leukoencephalopathy (PML), a
disease causing demyelination in the central nervous system?
A Human papillomavirus
B West Nile virus
C Tick-borne encephalitis virus
D Polyomavirus
ANSWER:B
108 This virus is transmitted by the same arthropod that transmits babesiosis and ehrlichiosis.
A Human papillomavirus
B West Nile virus
C Tick-borne encephalitis virus
D Polyomavirus
ANSWER:C
109
This virus is a single-stranded RNA orthomyxovirus. Annual vaccination is necessary
because of antigenic drift and shift.
A Measles virus
B Influenza virus
C Respiratory syncytial virus
D Parainfluenza virus
ANSWER:B
110
This virus is a single-stranded RNA paramyxovirus. The rash known as Koplik’s spots is
pathognomonic.
A Measles virus
B Influenza virus
C Respiratory syncytial virus
D Parainfluenza virus
ANSWER:A
111 This virus is the leading cause of bronchiolitis and communityacquired pneumonia in infants.
A Measles virus
B Influenza virus
C Respiratory syncytial virus
D Parainfluenza virus

ANSWER:C
112 This is a paramyxovirus and causes the syndrome known as croup.
A Measles virus
B Influenza virus
C Respiratory syncytial virus
D Parainfluenza virus
ANSWER:D
113
This is a double-stranded DNA virus. It is responsible for 15% of pediatric respiratory
infections and 10 to 15% of acute diarrhea in children.
A Measles virus
B Influenza virus
C Respiratory syncytial virus
D Adenovirus
ANSWER:D
114
A patient with a peptic ulcer was admitted to the hospital and a gastric biopsy was performed.
The tissue was cultured on chocolate agar incubated in a microaerophilic environment at
37°C for 5 to 7 days. At 5 days of incubation, colonies appeared on the plate and were
curved, Gramnegative rods, oxidase-positive. The most likely identity of this organism is
A Campylobacter jejuni
B Vibrio parahaemolyticus
C Haemophilus influenzae
D Helicobacter pylori
ANSWER:D
115
An inhibitor was designed to block a biologic function in H. influenzae. If the goal of the
experiment was to reduce the virulence of H. influenzae, the most likely target would be
A Exotoxin liberator
B Endotoxin assembly
C Flagella synthesis
D Capsule formation
ANSWER:D

116
An experimental compound is discovered that prevents the activation of adenyl cyclase and
the resulting increase in cyclic AMP. The toxic effects of which of the following bacteria
might be prevented with the use of this experimental compound?
A Vibrio cholerae
B Corynebacterium diphtheriae
C Pseudomonas
D Listeria monocytogenes
ANSWER:A
117
There are millions of cases of leprosy (Hansen’s disease) worldwide, but predominately in
Asia and Africa. The clinical spectrum of Hansen’s disease is best characterized by
A Immunologic anergy
B Chronic pneumonitis
C Peripheral neuritis
D Bacilli in lesions that digest tissues
ANSWER:C
118
At a church supper in Nova Scotia, the following meal was served: baked beans, ham,
coleslaw, eclairs, and coffee. Of the 30 people who attended, 4 senior citizens became ill in 3
days; 1 eventually died. Two weeks after attending the church supper, a 19-year-old girl
gave birth to a baby who rapidly became ill with meningitis and died in 5 days.
Epidemiologic investigation revealed the following percentages of people who consumed the
various food items: baked beans, 30%; ham, 80%; coleslaw, 60%; eclairs, 100%; and coffee,
90%. Which of the following statements is true?
A This is not a case of food poisoning because only 4 people became ill
B
A relationship between the death of the baby and the food consumed at the church supper
can be ruled out
C
Based on the epidemiologic investigation, the eclairs can be isolated as the source of the
disease
D
Additional epidemiologic data should include the percentage of those who ate a particular
food item who became ill
ANSWER:D

119
At a church supper in Nova Scotia, the following meal was served: baked beans, ham,
coleslaw, eclairs, and coffee. Of the 30 people who attended, 4 senior citizens became ill in 3
days; 1 eventually died. Two weeks after attending the church supper, a 19-year-old girl
gave birth to a baby who rapidly became ill with meningitis and died in 5 days.
Epidemiologic investigation revealed the following percentages of people who consumed the
various food items: baked beans, 30%; ham, 80%; coleslaw, 60%; eclairs, 100%; and coffee,
90%. Which of the following statements is true? Microbiologic analysis revealed no growth
in the baked beans, ham, or coffee; many Gram-positive beta-hemolytic, short, rod-shaped
bacteria in the coleslaw; and rare Gram-positive cocci in the eclairs. The most likely cause of
this outbreak is
A Staphylococcus aureus
B Listeria
C Clostridium perfringens
D Clostridium botulinum
ANSWER:B
120
A 21-year-old college student complained of malaise, low-grade fever, and a harsh cough,
but not of muscle aches and pains. An x-ray revealed a diffuse interstitial pneumonia in the
left lobes of the lung. The WBC count was normal. The student has been ill for a week.
Based on the information given, the most likely diagnosis is
A Mycoplasma pneumonia
B Pneumococcal pneumonia
C Staphylococcal pneumonia
D Influenza
ANSWER:A
121
A 21-year-old college student complained of malaise, low-grade fever, and a harsh cough,
but not of muscle aches and pains. An x-ray revealed a diffuse interstitial pneumonia in the
left lobes of the lung. The WBC count was normal. The student has been ill for a week.
Based on the information given, which of the following laboratory tests would most rapidly
assist you in making the diagnosis?
A Cold agglutinins
B Viral culture
C Complement fixation (CF) test
D Gram stain of sputum

ANSWER:A
122
A 21-year-old college student complained of malaise, low-grade fever, and a harsh cough,
but not of muscle aches and pains. An x-ray revealed a diffuse interstitial pneumonia in the
left lobes of the lung. The WBC count was normal. The student has been ill for a week. The
following laboratory data were available within 2 days: cold agglutinins—negative;
complement fixation (M. pneumoniae)—1:64; viral culture—pending, but negative to date;
bacterial culture of sputum on blood agar and MacConkey’s agar—normal oral flora. In
order to confirm the diagnosis, which of the following procedures could be ordered to
achieve a specific and sensitive diagnosis?
A Culture of the sputum on charcoal yeast extract
B A repeat cold agglutinin test
C A DNA probe to the 16S ribosomal RNA of an organism lacking a cell wall
D A repeat CF test in 5 days
ANSWER:C
123
Pathogenic mechanisms involved in tuberculosis can be primarily attributed to which of the
following?
A Toxin production by the mycobacteria
B Specific cell adhesion sites
C Cell-mediated hypersensitivity
D Humoral immunity
ANSWER:C
124
The class of antibiotics known as the quinolones are bactericidal. Their mode of action on
growing bacteria is thought to be
A Inhibition of DNA gyrase
B Inactivation of penicillin-binding protein II
C Inhibition of β-lactamase
D Prevention of the cross-linking of glycine
ANSWER:A
125
Vancomycin-indeterminate S. aureus (VISA) has recently been reported in the United States.
Which one of the statements concerning VISA is the most correct?
A Minimum inhibitory concentration (MIC) for vancomycin is at least 1.0 mcg/mL
B VISA isolates are usually methicillin susceptible (methicillin-resistant S. aureus, MRSA)
C VISAs have emerged because of the extended use of vancomycin for MRSAs

D Patients with VISA isolates need not be isolated
ANSWER:C
126
A sputum sample was brought to the laboratory for analysis. Gram stain revealed the
following: rare epithelial cells, 8 to 10 polymorphonuclear leukocytes per high-power field,
and pleomorphic Gram-negative rods. As the laboratory consultant, which of the following
interpretations should you make?
A The sputum specimen is too contaminated by saliva to be useful
B There is no evidence of an inflammatory response
C The patient has pneumococcal pneumonia
D The patient has Vincent’s disease
ANSWER:C
127
An isolate from a wound culture is a Gram-negative rod identified as Bacteroides fragilis.
Anaerobic infection with B. fragilis is characterized by
A A foul-smelling discharge
B A black exudate in the wound
C An exquisite susceptibility to penicillin
D A heme-pigmented colony formation
ANSWER:A
128
Virtually all prokaryotic cells (bacteria, both Gram-positive and Gramnegative) contain
peptidoglycan as well as specific enzymes for its synthesis. All of the following statements
concerning Gram-positive and Gram-negative bacteria are true except
A The extent of cross-linking of peptidoglycan is a function of different species of bacteria
B The peptidoglycan-synthesizing enzymes can be antibiotic targets
C Both Gram-positive and Gram-negative bacteria contain significant amounts of teichoic acid
D
With the exception of the structures that are cross-linked, peptidoglycan structure is common
to most bacteria
ANSWER:C
129
L. monocytogenes causes a variety of diseases, including food poisoning. Listeria are small,
Gram-positive, motile rod-shaped bacteria. Which of the following best describes these
microorganisms?
A Listeria are facultative intracellular pathogens
B Once infected, the immune system cannot destroy Listeria

C Listeria cannot be cultivated on artificial media
D Flagella are produced both at room temperature and at 37°C
ANSWER:A
130
A 30-year-old male patient was seen by the emergency service and reported a 2-week history
of a penile ulcer. He noted that this ulcer did not hurt. Which one of the following
conclusions/actions is most valid?
A Draw blood for a herpes antibody test
B Perform a dark-field examination of the lesion
C Prescribe acyclovir for primary genital herpes
D Even if treated, the lesion will remain for months
ANSWER:B
131
The laboratory reports that the Venereal Disease Research Laboratory (VDRL) test
performed on the above patient is reactive at a dilution of 1:4 (4 dils). The patient also
reports to you that he has recently been diagnosed with hepatitis A. Which one of the
following actions would be most appropriate?
A Report this patient to the health department, as he has syphilis
B Order a confirmatory test such as the fluorescent treponemal antibody test (FTA)
C Repeat the VDRL test
D Order a rapid reagin test (RPR)
ANSWER:B
132
In the above patient, which one of the following test combinations for syphilis is most
appropriate?
A FTA-Abs (IgG)/FTA-Abs (IgM)
B RPR/FTA-Abs
C RPR/culture of the lesion
D VDRL/RPR
ANSWER:B
133
Assume that the patient absolutely denied any contact, sexual or otherwise, with a person
who had syphilis. Assume also that both the RPR and the FTA Abs were positive on this
patient. Which one of the following tests could be used to show that this patient probably
does not have syphilis?
A VDRL
B Quantitative RPR

C Treponema pallidum immobilization (TPI) test
D Frei test
ANSWER:C
134
A 55-year-old man who is being treated for adenocarcinoma of the lung is admitted to a
hospital because of a temperature of 38.9°C (102°F), chest pain, and a dry cough. Sputum is
collected. Gram’s stain of the sputum is unremarkable and culture reveals many small Gram-
negative rods able to grow only on a charcoal yeast extract agar. This organism most likely is
A Klebsiella pneumoniae
B Mycoplasma pneumoniae
C Legionella pneumophila
D Chlamydia trachomatis
ANSWER:C
135
A patient was hospitalized after an automobile accident. The wounds became infected and
the patient was treated with tobramycin, carbenicillin, and clindamycin. Five days after
antibiotic therapy was initiated, the patient developed severe diarrhea and
pseudomembranous enterocolitis. Antibioticassociated diarrhea and the more serious
pseudomembranous enterocolitis can be caused by
A Clostridium sordellii
B Clostridium perfringens
C Clostridium difficile
D S. aureus
ANSWER:C
136
Assuming that the average achievable serum level of gentamicin is 6 to 8 mcg/mL, which of
the following bacteria is susceptible to gentamicin?
A E. coli with a minimal inhibitory concentration (MIC) of 10 mcg/mL
B E. coli with an MIC of 12 mcg/mL
C Klebsiella with an MIC of 0.25 mcg/mL
D Klebsiella with an MIC of 6.0 mcg/mL
ANSWER:C
137
A child comes to an emergency room because of an infected dog bite. The wound is found
to contain small Gram-negative rods. The most likely cause of infection is
A E. coli

B H. influenzae
C Pasteurella multocida
D Brucella canis
ANSWER:C
138
A patient complained to his dentist about a draining lesion in his mouth. A Gram’s stain of
the pus showed a few Gram-positive cocci, leukocytes, and many branched Gram-positive
rods. The most likely cause of the disease is
A Actinomyces israelii
B Actinomyces viscosus
C Diphtheriae
D Propionibacterium acnes
ANSWER:A
139
Group B streptococcus sepsis in an infant is preventable. Which one of the following
procedures is most likely to reduce the incidence of group B streptococcal disease?
A Intrapartum antibiotic treatment
B Use of a polysaccharide vaccine
C Screening of pregnant females in the last trimester
D Identification of possible high-risk births
ANSWER:A
140
There has been much speculation on the pathogenesis of group B streptococcal disease in the
neonate. One of the most likely pathogenic mechanisms is
A Complement C5a, a potent chemoattractant, activates PMNs
B The streptococci are resistant to penicillin
C The alternative complement pathway is activated
D
In the absence of specific antibody, opsonization, phagocyte recognition, and killing do not
proceed normally
ANSWER:D
141
A man who has a penile chancre appears in a hospital’s emergency service. The VDRL test is
negative. The most appropriate course of action for the physician in charge would be to
A Send the patient home untreated
B Repeat the VDRL test in 10 days
C Perform dark-field microscopy for treponemes

D Swab the chancre and culture on Thayer-Martin agar
ANSWER:C
142
Fever of unknown origin in a farmer who raises goats would most likely be caused by which
of the following organisms?
A Brucella melitensis
B Clostridium novyi
C T. pallidum
D Histoplasma capsulatum
ANSWER:A
143
Cholera is a toxicogenic dysenteric disease common in many parts of the world. In the
treatment of patients who have cholera, the use of a drug that inhibits adenyl cyclase would
be expected to
A Kill the patient immediately
B Eradicate the organism
C Increase fluid secretion
D Block the action of cholera toxin
ANSWER:D
144
A box of ham sandwiches with mayonnaise prepared by a person with a boil on his neck was
left out of the refrigerator for the on-call interns. Three doctors became violently ill
approximately 2 h after eating the sandwiches. The most likely cause is
A S. aureus enterotoxin
B Coagulase from S. aureus in the ham
C S. aureus leukocidin
D C. perfringens toxin
ANSWER:A
145
S. aureus causes a wide variety of infections, ranging from wound infection to pneumonia.
Treatment of S. aureus infection with penicillin is often complicated by the
A Inability of penicillin to penetrate the membrane of S. aureus
B Production of penicillinase by S. aureus
C Production of penicillin acetylase by S. aureus
D Lack of penicillin binding sites on S. aureus
ANSWER:B

146
Symptoms of C. botulinum food poisoning include double vision, inability to speak, and
respiratory paralysis. These symptoms are consistent with
A Invasion of the gut epithelium by C. botulinum
B Secretion of an enterotoxin
C Endotoxin shock
D Ingestion of a neurotoxin
ANSWER:D
147
In people who have sickle cell anemia, osteomyelitis usually is associated with which of the
following organisms?
A Micrococcus
B Escherichia
C Pseudomonas
D Salmonella
ANSWER:D
148 The treatment of choice for a patient with C. jejuni enterocolitis is
A Erythromycin
B Ciprofloxacin
C Ampicillin
D Pepto-Bismol
ANSWER:A
149
A hyperemic edema of the larynx and epiglottis that rapidly leads to respiratory obstruction
in young children is most likely to be caused by
A K. pneumoniae
B M. pneumoniae
C Neisseria meningitidis
D H. influenzae
ANSWER:D
150
A 70-year-old female patient was readmitted to a local hospital with fever and chills
following cardiac surgery at a major teaching institution. Blood cultures were taken and a
Gram-positive coccus grew from the blood cultures within 24 hours. Initial tests indicated
that this isolate was resistant to penicillin. The most likely identification is
A Streptococcus pneumoniae
B Neisseria

C Group A streptococcus
D Enterococcus
ANSWER:D
151
A 70-year-old female patient was readmitted to a local hospital with fever and chills
following cardiac surgery at a major teaching institution. Blood cultures were taken and a
Gram-positive coccus grew from the blood cultures within 24 hours. Initial tests indicated
that this isolate was resistant to penicillin. Further testing revealed that the isolate possessed
the group D antigen, was not β-lactamase-positive, but was resistant to vancomycin. The
most likely identification of this isolate is
A Enterococcus faecalis
B Enterococcus durans
C Enterococcus cassiflavus
D Enterococcus faecium
ANSWER:D
152
Acute hematogenous osteomyelitis is often diagnosed by isolation of the organism from the
blood and is caused most often by
A Proteus mirabilis
B Streptococcus faecalis
C Staphylococcus epidermidis
D S. aureus
ANSWER:D
153 Diphtheria toxin is produced only by those strains of C. diphtheriae that are
A Glucose fermenters
B Sucrose fermenters
C Lysogenic for β-prophage
D Of the mitis strain
ANSWER:C
154
A 28-year-old menstruating woman appeared in the emergency room with the following
signs and symptoms: fever, 104°F (40°C); WBC, 16,000/μL; blood pressure, 90/65 mmHg; a
scarlatiniform rash on her trunk, palms, and soles; extreme fatigue; vomiting; and diarrhea.
The patient described in the case above most likely has
A Scalded skin syndrome
B Toxic shock syndrome

C Guillain-Barré syndrome
D Chickenpox
ANSWER:B
155
A 28-year-old menstruating woman appeared in the emergency room with the following
signs and symptoms: fever, 104°F (40°C); WBC, 16,000/ μL; blood pressure, 90/65 mmHg;
a scarlatiniform rash on her trunk, palms, and soles; extreme fatigue; vomiting; and diarrhea.
Culture of the menstrual fluid in the case cited would most likely reveal a predominance of
A S. aureus
B S. epidermidis
C C. perfringens
D C. difficile
ANSWER:A
156
A 28-year-old menstruating woman appeared in the emergency room with the following
signs and symptoms: fever, 104°F (40°C); WBC, 16,000/ μL; blood pressure, 90/65 mmHg;
a scarlatiniform rash on her trunk, palms, and soles; extreme fatigue; vomiting; and diarrhea.
The most characteristic finding not yet revealed in the case just presented would be
A Travel to Vermont
B Recent exposure to rubella
C A retained tampon
D Heavy menstrual flow
ANSWER:C
157
A new latex agglutination (LA) reagent for H. influenzae polysaccharide antigen in
cerebrospinal fluid was evaluated. Results were compared with the isolation of H. influenzae
from the CSF. Results were as follows: LA POS, CULT POS: 25 LA POS, CULT NEG: 5 LA
NEG, CULT POS: 5 LA NEG, CULT NEG: 95 The sensitivity of LA is
A 0%
B 30%
C 85%
D 95%
ANSWER:C

158
A new latex agglutination (LA) reagent for H. influenzae polysaccharide antigen in
cerebrospinal fluid was evaluated. Results were compared with the isolation of H. influenzae
from the CSF. Results were as follows: LA POS, CULT POS: 25 LA POS, CULT NEG: 5 LA
NEG, CULT POS: 5 LA NEG, CULT NEG: 95 The specificity of LA is
A 0%
B 30%
C 80%
D 95%
ANSWER:D
159
A new latex agglutination (LA) reagent for H. influenzae polysaccharide antigen in
cerebrospinal fluid was evaluated. Results were compared with the isolation of H. influenzae
from the CSF. Results were as follows: LA POS, CULT POS: 25 LA POS, CULT NEG: 5 LA
NEG, CULT POS: 5 LA NEG, CULT NEG: 95 The negative predictive value of LA is
A 10%
B 80%
C 95%
D 110%
ANSWER:C
160
A new latex agglutination (LA) reagent for H. influenzae polysaccharide antigen in
cerebrospinal fluid was evaluated. Results were compared with the isolation of H. influenzae
from the CSF. Results were as follows: LA POS, CULT POS: 25 LA POS, CULT NEG: 5 LA
NEG, CULT POS: 5 LA NEG, CULT NEG: 95 The incidence of H. influenzae meningitis in
the general population is less than 1%. If during an epidemic the incidence rose to 3%, the
negative predictive value of the LA test would
A Increase
B Decrease
C Remain the same
D Be impossible to calculate
ANSWER:B
161
Methicillin-resistant S. aureus (MRSA) was isolated from 7 patients in a 14-bed intensive care
unit. All patients were isolated and the unit closed to any more admissions. Which one of the
following reasons best explains these rigorous methods to control MRSA?

A MRSA is inherently more virulent than other staphylococci
B
The alternative for treatment of MRSA is vancomycin, an expensive and potentially toxic
antibiotic
C MRSA causes toxic shock syndrome
D
MRSA spreads more rapidly from patient to patient than antibiotic-susceptible staphylococci
do
ANSWER:B
162
A patient with AIDS returned from Haiti with acute diarrhea. The stool revealed an oval
organism (8 to 9 μm in diameter) that was acid-fast and fluoresced blue under ultraviolet
light. The most likely identification of this organism is
A Cyclospora
B Giardia
C Enterocytozoon
D Cryptosporidium
ANSWER:A
163
A 2-year-old infant is brought to the emergency room with hemolytic uremic syndrome and
thrombocytopenia. Which one of the following bacteria would most likely be isolated from a
stool specimen?
A Shigella
B Salmonella
C Aeromonas
D E. coli 0157/H7
ANSWER:D
164
E. coli causes disease by a variety of different methods. Which one of the following E. coli
types is characterized by the presence of LT (heat-labile) and ST (heat-stable) proteins?
A Enteroinvasive (EIEC)
B Enterotoxigenic (ETEC)
C Enterohemorrhagic (EHEC)
D Enteropathogenic (EPEC)
ANSWER:B

165
Yersinia pestis, the causative agent of plague, is enzootic in the United States west of the one-
hundredth meridian. Human plague can be bubonic or pneumonic. The primary
epidemiologic difference between the two clinical forms of plague is
A Season of the year
B Route of infection
C Age of the patient
D Health of the animal vector
ANSWER:B
166
A 9-year-old child is brought to the emergency room with the chief complaint of enlarged,
painful axillary lymph nodes. The resident physician also notes a small, inflamed, dime-sized
lesion surrounding what appears to be a small scratch on the forearm. The lymph node is
aspirated and some pus is sent to the laboratory for examination. A Warthin-Starry silver
impregnation stain reveals many highly pleomorphic, rod-shaped bacteria. The most likely
cause of this infection is
A Y. pestis
B Yersinia enterocolitica
C Mycobacterium scrofulaceum
D Bartonella henselae
ANSWER:D
167
Recently, there have been sensational media reports of patients infected with invasive, “flesh-
eating” bacteria that spread rapidly through the tissues. This necrotizing fasciitis is usually
caused by
A S. aureus
B Group A streptococci
C Micrococcus
D Bacillus cereus
ANSWER:B
168
The most effective noninvasive test for the diagnosis of Helicobacterassociated gastric ulcers
is
A Detection of H. pylori antigen in stool
B Growth of H. pylori from a stomach biopsy
C Growth of H. pylori in the stool
D IgM antibodies to H. pylori

ANSWER:A
169
The following test results were observed in a woman tested in November who reported being
in the woods in Pennsylvania during the past summer, was bitten by a tick, and now has
Bell’s palsy: Lyme IgG antibody 1:1280; Lyme IgM antibody negative. Which one of the
following courses of action is most appropriate?
A
Order tests for syphilis (VDRL, FTA-ABS) because there are cross-reactions reported with
Borrelia burgdorferi
B Ask the patient if she has a severe headache
C Consider treatment of the patient with an appropriate antibiotic such as tetra-cycline
D Ask the patient if she has had a urinary tract infection with E. coli
ANSWER:C
170
Mycobacterium avium is a major opportunistic pathogen in AIDS patients. M. avium from
AIDS patients can be best characterized by which one of the following statements?
A The majority of M. avium isolates from AIDS patients are nonpigmented
B M. avium isolates from AIDS patients are of multiple serovars
C Few isolates from AIDS patients are acid-fast
D M. avium can be isolated from the blood of AIDS patients
ANSWER:D
171
Rheumatic fever (RF) is a disease seen in children and young adults. Which one of the
following statements best typifies the disease?
A
It is characterized by inflammatory lesions that may involve the heart, joints, subcutaneous
tissues, and the central nervous system
B
The pathogenesis is related to the similarity between a staphylococcal antigen and a human
cardiac antigen
C Prophylaxis with benzathine penicillin is of little value
D It is a complication of group A streptococcal skin disease but usually not of pharyngitis
ANSWER:A
172
If a quellung test was done on the following bacterial isolates, which one would you expect
to be positive?
A S. pneumoniae
B Enterobacter
C Haemophilus parainfluenzae
D C. diphtheriae

ANSWER:A
173
Bacteria cause disease in a number of ways. One mechanism of pathogenesis is the secretion
of potent protein toxins. All the following diseases are caused by microbial protein toxins,
but one toxin has been used for a variety of maladies. It is
A Tetanus
B Botulism
C Bacillary (Shigella) dysentery
D Diphtheria
ANSWER:B
174
A 2-year-old child was admitted to the hospital with acute meningitis. The Gram stain
revealed Gram-positive short rods, and the mother indicated that the child had received “all”
of the meningitis vaccinations. What is the most likely cause of the disease?
A N. meningitidis, group A
B N. meningitidis, group C
C Listeria
D S. pneumoniae
ANSWER:C
175 The most common portal of entry for C. tetani, the cause of tetanus, is the
A Skin
B Gastrointestinal tract
C Respiratory tract
D Genital tract
ANSWER:A
176 The most common way in which tuberculosis is acquired is via the
A Skin
B Gastrointestinal tract
C Respiratory tract
D Genital tract
ANSWER:C
177
Shigellosis is common in travelers to developing countries. Infection is commonly acquired
through the
A Skin
B Gastrointestinal tract

C Respiratory tract
D Genital tract
ANSWER:B
178 A person who contracts gonorrhea is most likely to have acquired it via the
A Skin
B Gastrointestinal tract
C Respiratory tract
D Genital tract
ANSWER:D
179
There are a variety of “unusual” bacteria that infect humans. While rare, disease caused by
these microorganisms is serious and occasionally difficult to identify. Branhamella is best
characterized as
A A Gram-negative pleomorphic rod that can cause endocarditis
B The causative agent of trench fever
C
A Gram-negative rod, fusiforme-shaped, that is associated with periodontal disease but may
cause sepsis
D The causative agent of sinusitis, bronchitis, and pneumonia
ANSWER:D
180 Cardiobacterium is best characterized as
A A Gram-negative pleomorphic rod that can cause endocarditis
B The causative agent of trench fever
C
A Gram-negative rod, fusiforme-shaped, that is associated with periodontal disease but may
cause sepsis
D The causative agent of rat-bite fever
ANSWER:A
181 Capnocytophyga is best characterized as
A A Gram-negative pleomorphic rod that can cause endocarditis
B The causative agent of trench fever
C
A Gram-negative rod, fusiforme-shaped, that is associated with periodontal disease but may
cause sepsis
D The causative agent of rat-bite fever
ANSWER:C

182
An enterococcus (E. faecium) was isolated from a urine specimen (100,000 cfu/mL).
Treatment of the patient with ampicillin and gentamicin failed. The most clinically
appropriate action is
A Do no further clinical workup
B Suggest to the laboratory that low colony counts may reflect infection
C Determine if fluorescent microscopy is available for the diagnosis of actinomycosis
D Consider vancomycin as an alternative drug
ANSWER:D
183
A patient with symptoms of urinary tract infection had a culture taken, which grew 5 × 103
E. coli. The laboratory reported it as “insignificant.” The most clinically appropriate action is
A Do no further clinical workup
B Suggest to the laboratory that low colony counts may reflect infection
C Determine if fluorescent microscopy is available for the diagnosis of actinomy-cosis
D Consider vancomycin as an alternative drug
ANSWER:B
184
A patient appeared in the emergency room with a submandibular mass. A smear was made of
the drainage and a bewildering variety of bacteria were seen, including branched, Gram-
positive rods. The most clinically appropriate action is
A Do no further clinical workup
B Suggest to the laboratory that low colony counts may reflect infection
C Determine if fluorescent microscopy is available for the diagnosis of actinomy-cosis
D Consider vancomycin as an alternative drug
ANSWER:C
185 The antibiotic therapy of choice for legionellosis is
A Penicillin
B Ampicillin
C Erythromycin
D Vancomycin
ANSWER:C
186 The antibiotic of choice for pneumococcal pneumonia is
A Penicillin
B Ampicillin

C Erythromycin
D Vancomycin
ANSWER:A
187 The antibiotic of choice for streptococcal pharyngitis is
A Penicillin
B Ampicillin
C Erythromycin
D Vancomycin
ANSWER:A
188 The therapy of choice for pseudomembranous enterocolitis is
A Penicillin
B Ampicillin
C Erythromycin
D Vancomycin
ANSWER:D
189
Although cholera, a Vibrio infection, has been rarely seen in the United States, there have
been recent outbreaks of classic cholera associated with shellfish harvested from the Gulf of
Mexico. Vibrios are shaped like curved rods, and infections more common than cholera may
be caused by a variety of curved-rod bacteria. C. jejuni is best described as
A “String-test”-positive isolate; three serotypes—Ogawa (AB), Inaba (AC), Hikojima (ABC)
B Human pathogen, halophilic, lactose-positive; produces heat-labile, extracellular toxin
C
Human pathogen, halophilic, lactose-negative, sucrose-negative; causes gastrointestinal
diseases primarily from ingestion of cooked seafood
D Cause of gastroenteritis; reservoir in birds and mammals, optimal growth at 42°C
ANSWER:D
190
Although cholera, a Vibrio infection, has been rarely seen in the United States, there have
been recent outbreaks of classic cholera associated with shellfish harvested from the Gulf of
Mexico. Vibrios are shaped like curved rods, and infections more common than cholera may
be caused by a variety of curved-rod bacteria. V. cholerae, the causative agent of cholera, is
best described as
A “String-test”-positive isolate; three serotypes—Ogawa (AB), Inaba (AC), Hikojima (ABC)
B Human pathogen, halophilic, lactose-positive; produces heat-labile, extracellular toxin

C
Human pathogen, halophilic, lactose-negative, sucrose-negative; causes gastrointestinal
diseases primarily from ingestion of cooked seafood
D Cause of gastroenteritis; reservoir in birds and mammals, optimal growth at 42°C
ANSWER:A
191
Although cholera, a Vibrio infection, has been rarely seen in the United States, there have
been recent outbreaks of classic cholera associated with shellfish harvested from the Gulf of
Mexico. Vibrios are shaped like curved rods, and infections more common than cholera may
be caused by a variety of curved-rod bacteria. V. parahaemolyticus, first described in Japan,
is best characterized as
A “String-test”-positive isolate; three serotypes—Ogawa (AB), Inaba (AC), Hikojima (ABC)
B Human pathogen, halophilic, lactose-positive; produces heat-labile, extracellular toxin
C
Human pathogen, halophilic, lactose-negative, sucrose-negative; causes gastrointestinal
diseases primarily from ingestion of cooked seafood
D Cause of gastroenteritis; reservoir in birds and mammals, optimal growth at 42°C
ANSWER:C
192
Although cholera, a Vibrio infection, has been rarely seen in the United States, there have
been recent outbreaks of classic cholera associated with shellfish harvested from the Gulf of
Mexico. Vibrios are shaped like curved rods, and infections more common than cholera may
be caused by a variety of curved-rod bacteria. Vibrio vulnificus, which can be found in the
oceans and bays in temperate and tropical climates, is best characterized as
A “String-test”-positive isolate; three serotypes—Ogawa (AB), Inaba (AC), Hikojima (ABC)
B Human pathogen, halophilic, lactose-positive; produces heat-labile, extracellular toxin
C
Human pathogen, halophilic, lactose-negative, sucrose-negative; causes gastrointestinal
diseases primarily from ingestion of cooked seafood
D Cause of gastroenteritis; reservoir in birds and mammals, optimal growth at 42°C
ANSWER:B
193
Each bacterium listed below is a small, Gram-negative rod. Some of them are fastidious and
may be difficult to culture and identify. They have unique microbiological features and each
of them causes distinctive clinical disease. Y. enterocolitica, formerly a Pasteurella, is best
described by which of the following statements?
A
Commonly inhabits the canine respiratory tract and is an occasional pathogen for humans;
strongly urease-positive

B
Pits agar, grows both in carbon dioxide and under anaerobic conditions, and is part of the
normal oral cavity flora
C
Typically infects cattle, requires 5 to 10% carbon dioxide for growth, and is inhibited by the
dye thionine
D
Manifests different biochemical and physiologic characteristics, depending on growth
temperature, and causes a spectrum of human disease, most commonly mesenteric
lymphadenitis
ANSWER:D
194
Each bacterium listed below is a small, Gram-negative rod. Some of them are fastidious and
may be difficult to culture and identify. They have unique microbiological features and each
of them causes distinctive clinical disease. Brucella abortus, one of the three species causing
brucellosis, a possible bioterrorism agent, is best described by which of the following
statements?
A
Commonly inhabits the canine respiratory tract and is an occasional pathogen for humans;
strongly urease-positive
B
Pits agar, grows both in carbon dioxide and under anaerobic conditions, and is part of the
normal oral cavity flora
C
Typically infects cattle, requires 5 to 10% carbon dioxide for growth, and is inhibited by the
dye thionine
D
Typically is found in infected animal bites in humans and can cause hemorrhagic septicemia
in animals
ANSWER:C
195
Each bacterium listed below is a small, Gram-negative rod. Some of them are fastidious and
may be difficult to culture and identify. They have unique microbiological features and each
of them causes distinctive clinical disease. Bordetella bronchiseptica could be confused with
the agent of whooping cough. It is best described by which of the following statements?
A
Commonly inhabits the canine respiratory tract and is an occasional pathogen for humans;
strongly urease-positive
B
Pits agar, grows both in carbon dioxide and under anaerobic conditions, and is part of the
normal oral cavity flora
C
Typically infects cattle, requires 5 to 10% carbon dioxide for growth, and is inhibited by the
dye thionine

D
Typically is found in infected animal bites in humans and can cause hemorrhagic septicemia
in animals
ANSWER:A
196
Each bacterium listed below is a small, Gram-negative rod. Some of them are fastidious and
may be difficult to culture and identify. They have unique microbiological features and each
of them causes distinctive clinical disease. P. multocida is a very common organism and is
best described by which of the following statements?
A
Commonly inhabits the canine respiratory tract and is an occasional pathogen for humans;
strongly urease-positive
B
Pits agar, grows both in carbon dioxide and under anaerobic conditions, and is part of the
normal oral cavity flora
C
Typically infects cattle, requires 5 to 10% carbon dioxide for growth, and is inhibited by the
dye thionine
D
Typically is found in infected animal bites in humans and can cause hemorrhagic septicemia
in animals
ANSWER:D
197
To isolate specific bacteria from clinical specimens, it is necessary to use a variety of artificial
media, some of which is selective, others of which are nonselective. N. gonorrhoeae is a
fastidious pathogen and found in sites often contaminated with normal flora. The best
medium for isolation is
A Sheep blood agar
B Löffler’s medium
C Thayer-Martin agar
D Thiosulfate citrate bile salts sucrose medium
ANSWER:C
198
To isolate specific bacteria from clinical specimens, it is necessary to use a variety of artificial
media, some of which is selective, others of which are nonselective. V. cholerae, the
causative agent of cholera, is best isolated using
A Sheep blood agar
B Löffler’s medium
C Thayer-Martin agar
D Thiosulfate citrate bile salts sucrose medium
ANSWER:D

199
To isolate specific bacteria from clinical specimens, it is necessary to use a variety of artificial
media, some of which is selective, others of which are nonselective. M. tuberculosis can be
found in the sputum of patients with tuberculosis. After digestion of the sputum, isolation is
best accomplished using
A Sheep blood agar
B Löffler’s medium
C Thayer-Martin agar
D Löwenstein-Jensen medium
ANSWER:D
200
To isolate specific bacteria from clinical specimens, it is necessary to use a variety of artificial
media, some of which is selective, others of which are nonselective. C. diphtheriae may be
difficult to isolate from the nasopharynx without the use of special media. The medium of
choice is
A Sheep blood agar
B Löffler’s medium
C Thayer-Martin agar
D Thiosulfate citrate bile salts sucrose medium
ANSWER:B
201
To isolate specific bacteria from clinical specimens, it is necessary to use a variety of artificial
media, some of which is selective, others of which are nonselective. S. aureus has a
distinctive appearance on which one of the following media?
A Sheep blood agar
B Löffler’s medium
C Thayer-Martin agar
D Thiosulfate citrate bile salts sucrose medium
ANSWER:A
202
In order to recognize abnormal bacteria, it is necessary to know which bacteria are
predominant normal flora of certain body sites. Which of the following is the predominant
organism on skin commonly seen as a blood culture contaminant?
A α-hemolytic streptococci
B Lactobacillus
C S. epidermidis
D Escherichia coli

ANSWER:C
203
In order to recognize abnormal bacteria, it is necessary to know which bacteria are
predominant normal flora of certain body sites. Which of the following is the predominant
flora of the mouth that is the major cause of dental caries?
A α-hemolytic streptococci
B Lactobacillus
C S. epidermidis
D E. coli
ANSWER:A
204
In order to recognize abnormal bacteria, it is necessary to know which bacteria are
predominant normal flora of certain body sites. Which of the following is the most prevalent
microorganism in the vagina that may also be protective?
A α-hemolytic streptococci
B Lactobacillus
C S. epidermidis
D E. coli
ANSWER:B
205 Streptococcus mutans is best described as
A
A facultative anaerobe that often inhabits the buccal mucosa early in a neonate’s life and can
cause bacterial endocarditis
B A β-hemolytic organism that causes a diffuse, rapidly spreading cellulitis
C An anaerobic, filamentous bacterium that often causes cervicofacial osteomyelitis
D A facultative anaerobe that is highly cariogenic and sticks to teeth by synthesis of a dextran
ANSWER:D
206 Streptococcus salivarius, a common isolate in the clinical laboratory, is best described as
A
A facultative anaerobe that often inhabits the buccal mucosa early in a neonate’s life and can
cause bacterial endocarditis
B A β-hemolytic organism that causes a diffuse, rapidly spreading cellulitis
C An anaerobic, filamentous bacterium that often causes cervicofacial osteomyelitis
D A facultative anaerobe that is highly cariogenic and sticks to teeth by synthesis of a dextran
ANSWER:A

207 A. israelii is one of many actinomycetes and is best described as
A
A facultative anaerobe that often inhabits the buccal mucosa early in a neonate’s life and can
cause bacterial endocarditis
B A β-hemolytic organism that causes a diffuse, rapidly spreading cellulitis
C An anaerobic, filamentous bacterium that often causes cervicofacial osteomyelitis
D A facultative anaerobe that is highly cariogenic and sticks to teeth by synthesis of a dextran
ANSWER:C
208 A. viscosus, another ubiquitous actinomycete, can best be described as
A
A facultative anaerobe that often inhabits the buccal mucosa early in a neonate’s life and can
cause bacterial endocarditis
B A β-hemolytic organism that causes a diffuse, rapidly spreading cellulitis
C An anaerobic, filamentous bacterium that often causes cervicofacial osteomyelitis
D A facultatively anaerobic, rod-shaped bacterium that sticks to teeth and is cariogenic
ANSWER:D
209
Diphtheriae causes diphtheria, a rare disease in the United States. diphtheriae is best
characterized by which of the following statements?
A It secretes erythrogenic toxin that causes the characteristic signs of scarlet fever
B
It produces toxin that blocks protein synthesis in an infected cell and carries a lytic
bacteriophage that produces the genetic information for toxin production
C
It produces at least one protein toxin consisting of two subunits, A and B, that cause severe
spasmodic cough usually in children
D It requires cysteine for growth
ANSWER:B
210
Bordetella pertussis is a Gram-negative rod that causes severe respiratory disease. Which one
of the following statements best characterizes this microorganism?
A It secretes erythrogenic toxin that causes the characteristic signs of scarlet fever
B
It produces toxin that blocks protein synthesis in an infected cell and carries a lytic
bacteriophage that produces the genetic information for toxin production
C
It produces at least one protein toxin consisting of two subunits, A and B, that cause severe
spasmodic cough usually in children
D It requires cysteine for growth
ANSWER:C

211
Francisella tularensis is the bacterium that causes tularemia, a disease not uncommonly seen
in hunters. Which one of the following statements best characterizes this bacterium?
A It secretes erythrogenic toxin that causes the characteristic signs of scarlet fever
B
It produces toxin that blocks protein synthesis in an infected cell and carries a lytic
bacteriophage that produces the genetic information for toxin production
C
It produces at least one protein toxin consisting of two subunits, A and B, that cause severe
spasmodic cough usually in children
D It requires cysteine for growth
ANSWER:D
212
E. coli 0157/H7 is a newsworthy microorganism that has been isolated from foods (meat)
and beverages (cider) with increasing frequency. Its characteristic biology is best described
by which of the following statements?
A It secretes erythrogenic toxin that causes the characteristic signs of scarlet fever
B
It produces toxin that blocks protein synthesis in an infected cell and carries a lytic
bacteriophage that produces the genetic information for toxin production
C
It produces at least one protein toxin consisting of two subunits, A and B, that cause severe
spasmodic cough usually in children
D
It secretes exotoxin that has been called “verotoxin” and “Shiga-like toxin”; infection is
mediated by specific attachment to mucosal membranes
ANSWER:D
213
Streptococcus pyogenes is a toxigenic bacterium causing a variety of diseases. Which of the
following statements best characterizes this organism?
A It secretes erythrogenic toxin that causes the characteristic signs of scarlet fever
B
It produces toxin that blocks protein synthesis in an infected cell and carries a lytic
bacteriophage that produces the genetic information for toxin production
C
It produces at least one protein toxin consisting of two subunits, A and B, that cause severe
spasmodic cough usually in children
D It requires cysteine for growth
ANSWER:A
214
N. meningitidis causes meningitis in all age groups. A characteristic physiological trait is that
it
A
Possesses N-acetylneuraminic acid capsule and adheres to specific tissues by pili found on the
bacterial cell surface

B Has capsule of polyglutamic acid, which is toxic when injected into rabbits
C Synthesizes protein toxin as a result of colonization of vaginal tampons
D
Causes spontaneous abortion and has tropism for placental tissue due to the presence of
erythritol in allantoic and amniotic fluid
ANSWER:A
215
Brucella is pathogenic for humans and animals. Which one of the following statements best
characterizes this organism?
A It has capsule of polyglutamic acid, which is toxic when injected into rabbits
B It synthesizes protein toxin as a result of colonization of vaginal tampons
C
It causes spontaneous abortion and has tropism for placental tissue due to the presence of
erythritol in allantoic and amniotic fluid
D It secretes two toxins, A and B, in large bowel during antibiotic therapy
ANSWER:C
216
An 18-year-old male patient appeared at the emergency room with a 3-day history of fever,
dry cough, difficulty breathing, and muscle aches and pains. His chest x-ray showed a
diffuse left upper lobe infiltrate. The following 5 questions focus on the etiology of
“atypical” or communityacquired pneumonia: Mycoplasma pneumoniae pneumonia
(walking pneumonia) may be rapidly identified by which of the following procedures?
A Detection of specific antigen in urine
B Cold agglutinin test
C Electron microscopy of sputum
D Culture of respiratory secretions in HeLa cells after centrifugation of the inoculated tubes
ANSWER:D
217
An 18-year-old male patient appeared at the emergency room with a 3-day history of fever,
dry cough, difficulty breathing, and muscle aches and pains. His chest x-ray showed a
diffuse left upper lobe infiltrate. The following 5 questions focus on the etiology of
“atypical” or communityacquired pneumonia: Influenza can be treated; therefore, specific
detection of the virus becomes much more important. Which of the following would be best
for detection of influenza?
A Detection of specific antigen in urine
B Cold agglutinin test
C Electron microscopy of sputum

D Detection of antigen in respiratory secretions
ANSWER:A
218 Legionnaires’ disease is most rapidly diagnosed by which one of the following procedures?
A Detection of specific antigen in urine
B Cold agglutinin test
C Electron microscopy of sputum
D Detection of antigen in respiratory secretions
ANSWER:D
219
An 18-year-old male patient appeared at the emergency room with a 3-day history of fever,
dry cough, difficulty breathing, and muscle aches and pains. His chest x-ray showed a
diffuse left upper lobe infiltrate. The following 5 questions focus on the etiology of
“atypical” or communityacquired pneumonia: Chlamydia pneumoniae has recently been
implicated in respiratory disease primarily in children. Which of the following would best
isolate this fastidious bacterium?
A Detection of specific antigen in urine
B Cold agglutinin test
C Electron microscopy of sputum
D Culture of respiratory secretions in HeLa cells after centrifugation of the inoculated tubes
ANSWER:D
220
An 18-year-old male patient appeared at the emergency room with a 3-day history of fever,
dry cough, difficulty breathing, and muscle aches and pains. His chest x-ray showed a
diffuse left upper lobe infiltrate. The following 5 questions focus on the etiology of
“atypical” or communityacquired pneumonia: Bordetella bronchitis, sometimes called
whooping cough, can best be detected by which of the following procedures?
A Fluorescent antibody detection of the organism in sputum
B Cold agglutinin test
C Direct microscopy of sputum by Gram stain
D Culture of respiratory secretions on Regan-Lowe agar
ANSWER:D
221 Which of the following bacterial transport methods is energyindependent?
A Facilitated diffusion

B Simple diffusion
C Proton gradient energized active transport
D Group translocation
ANSWER:B
222
Iron is essential in bacterial metabolism. When bacteria invade the human host they must
capture iron in order to survive. Which of the macromolecules listed below is important in
bacterial iron metabolism?
A Transferrin
B Lactoferrin
C Ferric oxide
D Siderophores
ANSWER:D
223
An aliquot of Escherichia coli is treated with ethylenediaminetetraacetic acid (EDTA). The
first wash is analyzed and found to contain alkaline phosphatase, DNase, and penicillinase.
The anatomic area of the cell affected by the EDTA is most likely to have been the
A Periplasmic space
B Mesosomal space
C Chromosome
D Plasma membrane
ANSWER:A
224
A bacterium is examined and is found to lack superoxide dismutase, catalase, and
peroxidase. Which of the following statements best describes this bacterium?
A This bacterium is an anaerobe
B This bacterium will survive in an O2 environment
C This bacterium is more virulent than one containing the three enzymes
D This bacterium does not produce superoxide
ANSWER:A
225
Analysis of the metabolites produced by an organism’s fermentation of glucose reveals small
amounts of 6-phosphogluconic acid. This fermentation organism is most likely to be
A Enterobacter
B Escherichia
C Leuconostoc

D Enterococcus faecalis
ANSWER:C
226
The formation of adenosine triphosphate (ATP) is essential for the maintenance of life. In
mammalian systems, the number of moles of ATP formed per gram atom of oxygen
consumed (the P/O ratio) is 3; in bacteria, however, the P/O ratio may be only 1 or 2. The
primary reason for the lower P/O ratio in bacteria is
A Absence of nicotinamide adenine dinucleotide (NAD)
B Loss of oxidative phosphorylation coupling sites
C Less dependence on ATP as an energy source
D Absence of a nonphosphorylative bypass reaction
ANSWER:B
227
Reversion of Neisseria gonorrhoeae from a fimbriated (fim 1) to a nonfimbriated (fim 2)
state would result in which one of the following phenomena?
A Inability of N. gonorrhoeae to colonize the mucosal epithelium
B Reversion to a Gram-positive stain
C Death of the organism
D Loss of serologic specificity
ANSWER:A
228
An unknown isolate is recognized serologically as Salmonella enteritidis serovar newport. A
mutant of this organism has lost region 1 (O-specific polysaccharide) of its
lipopolysaccharide. This mutant would be identified as
A Salmonella typhi
B Salmonella newport
C S. enteritidis
D S. enteritidis serovar newport
ANSWER:C
229
Certain enzymes catalyze the cross-linking of peptidoglycan, a unique constituent of
bacterial cell walls. Which of the following may be a factor in antibiotic resistance, the target
of which is cell-wall synthesis?
A Bactoprenol
B Reverse transcriptase
C RNA polymerase
D Penicillin-binding proteins (PBPs)

ANSWER:D
230
Ideally, an antibiotic should focus on a microbial target not found in mammalian cells. By
this standard, which of the following antibiotic agents would be expected to be most toxic to
humans?
A Penicillin
B Mitomycin
C Cephalosporin
D Bacitracin
ANSWER:B
231 E. coli has two major porins located in the outer membrane. The function of porins is the
A Stabilization of the mesosome
B Metabolism of phosphorylated intermediates
C Transfer of small molecules through the outer membrane
D Serologic stabilization of the O antigen
ANSWER:C
232
Early attempts at the polymerase chain reaction (PCR) used E. coli DNA polymerase. This
was replaced with DNA polymerase from Thermus aquaticus (“Taq” polymerase). The
primary advantage in using this enzyme is
A It is cheaper than E. coli polymerase
B Specificity is increased because nonspecific hybridization of primers does not occur
C Use of Taq polymerase results in fewer PCR cycles
D Use of Taq polymerase enables lower temperatures to be used
ANSWER:B
233
You have been asked to design a nucleic acid amplification test for a rarely isolated
bacterium. There are several questions that you must ask in order to develop a test that could
be used to diagnose disease. Assume, initially, that the polymerase chain reaction (PCR) will
amplify any DNA, human or microbial. The best way to prevent contamination of the PCR
process is to
A Wear gloves
B Wash benches with bleach
C Use universal precautions
D Incorporate self-sterilizing agents into the PCR mixture
ANSWER:D

234
You have been asked to design a nucleic acid amplification test for a rarely isolated
bacterium. There are several questions that you must ask in order to develop a test that could
be used to diagnose disease. You must choose primers for this PCR. Primers are small pieces
of nucleic acid that recognize a pair of unique sites on the bacterial chromosome. For an
optimum test to be developed, which of the following characteristics is the least desirable?
A Ability to be constructed by a synthesizer
B Uniqueness to the organism that you wish to detect
C Complementary to sequences on the bacterial chromosome
D That the sequences are widely recognized by many bacterial species
ANSWER:D
235
You have been asked to design a nucleic acid amplification test for a rarely isolated
bacterium. There are several questions that you must ask in order to develop a test that could
be used to diagnose disease. You have chosen the primers for the PCR that you have
designed. You have also developed a reaction mixture that contains, among other substances,
a polymerase enzyme. After the primer pairs have been amplified, they must be detected.
Which of the following detection methods is most sensitive (that is, will detect the highest
number of amplicons)?
A Southern blot
B Ethidium staining of the amplified products (amplicons)
C Microscopy
D Capture of the amplicons on a solid phase followed by an enzyme immuno-assay
ANSWER:A
236
The purpose of gene cloning is to produce large amounts of genes in pure form. The
sequence of the cloning process is critical to the production of clones. Which of the
following steps initializes the cloning process?
A Isolation and fragmentation of source DNA
B Amplification of source DNA
C Detection and purification of clones
D Joining of host DNA to a cloning vector
ANSWER:A

237
Nucleic acid probes are not only useful for searching for genes in the cloning host but also
for searching for genes or gene fragments in patient specimens. Which one of the following
statements is true of nucleic acid probes?
A Only DNA can be used as a probe
B Primers are labeled to allow detection, but probes are unlabeled
C
Nucleic acid probes are not as sensitive as traditional culture methods for detection of
pathogenic microorganisms
D
Probes can be designed so that they can detect very specific pieces of a nucleic acid, for
example, a penicillin-resistant gene
ANSWER:D
238
DNA from a host sample can be amplified by a process known as the polymerase chain
reaction (PCR). Which of the following is required for PCR?
A Knowledge of the genetic sequence to be amplified
B An single nucleotide primer
C An ultracentrifuge
D A universal probe to detect the amplified product
ANSWER:A
239
DNA can be transferred from one bacterium to another by a number of processes. Uptake by
a recipient cell of soluble DNA released from a donor cell is defined as
A Conjugation
B Recombination
C Competence
D Transformation
ANSWER:D
240
DNA can be transferred from one bacterium to another by a number of processes. Transfer
of a donor chromosome fragment by a temperate bacterial virus is defined as
A Conjugation
B Recombination
C Competence
D Transduction
ANSWER:D
241
DNA can be transferred from one bacterium to another by a number of processes. Direct
transfer of a plasmid between two bacteria is defined as

A Conjugation
B Recombination
C Competence
D Transformation
ANSWER:A
242 A broad-spectrum antibiotic in the general class of thienamycins is
A Piperacillin
B Cefoperazone
C Ceftriaxone
D Imipenem
ANSWER:D
243 Third-generation cephalosporin with good activity against Borrelia burgdorferi is
A Piperacillin
B Cefoperazone
C Ceftriaxone
D Ciprofloxacin
ANSWER:C
244 Broad-spectrum penicillin with antipseudomonas activity is
A Piperacillin
B Cefoperazone
C Ceftriaxone
D Ciprofloxacin
ANSWER:A
245 Third-generation cephalosporin with primary activity against Pseudomonas aeruginosa is
A Piperacillin
B Cefoperazone
C Ceftriaxone
D Ciprofloxacin
ANSWER:B
246 Quinolone antibiotic with broad Gram-negative and Gram-positive activity is
A Piperacillin
B Cefoperazone
C Ceftriaxone

D Ciprofloxacin
ANSWER:D
247 Which one of the following antibiotics inhibits dihydrofolate reductase?
A Penicillin
B Amdinocillin
C Amphotericin
D Trimethoprim
ANSWER:D
248 Which one of the following antibiotics binds to penicillin-binding protein-2 (PBP-2)?
A Penicillin
B Amdinocillin
C Amphotericin
D Chloramphenicol
ANSWER:B
249
Which one of the following antibiotics inhibits the final peptide bond between d-alanine and
glycine?
A Penicillin
B Amdinocillin
C Amphotericin
D Chloramphenicol
ANSWER:A
250 Which one of the following antibiotics binds sterols and alters membrane permeability?
A Penicillin
B Amdinocillin
C Amphotericin
D Chloramphenicol
ANSWER:C
251
Which one of the following antibiotics attaches to 50S ribosome and inhibits peptidyl
transferase?
A Penicillin
B Amdinocillin
C Amphotericin
D Chloramphenicol

ANSWER:D
252
Bacteria or their components may be characterized by unique constituents or structures.
Bacterial lipopolysaccharide (LPS) is characterized by
A Phospholipid
B Ribitol teichoic acid
C Glycolipids (waxes)
D Ketodeoxyoctonate
ANSWER:D
253
Bacteria or their components may be characterized by unique constituents or structures.
Mycobacterium cell walls are characterized by
A Phospholipid
B Ribitol teichoic acid
C Glycolipids (waxes)
D Ketodeoxyoctonate
ANSWER:C
254 Mycoplasmas differ from chlamydiae in that they are
A Susceptible to penicillin
B Able to grow on artificial cell-free media
C Able to cause urinary tract infection
D Able to stain well with Gram’s stain
ANSWER:B
255 Q fever
A Is an illness confined to the upper respiratory tract
B Has an incubation period of 4 to 6 weeks
C Is most commonly found in tropical regions
D Is an acute febrile illness caused by Coxiella burnetii
ANSWER:D
256 Rickettsiae, which include the spotted fevers, Q fever, typhus, and scrub typhus, are
A Obligate intracellular parasites
B Stable outside the host cell
C Easily stained (Gram-negative) with a Gram stain
D Maintained in nature with humans as the mammalian reservoir
ANSWER:A

257
A man with chills, fever, and headache is thought to have “atypical” pneumonia. History
reveals that he raises chickens and that approximately 2 weeks ago he lost a large number of
them to an undiagnosed disease. The most likely diagnosis of this man’s condition is
A Anthrax
B Q fever
C Relapsing fever
D Ornithosis (psittacosis)
ANSWER:D
258
An ill patient denied being bitten by insects. However, he had spent some time in a milking
barn and indicated that it was dusty. Of the following rickettsial diseases, which one has he
most likely contracted?
A Scrub typhus
B Rickettsialpox
C Brill-Zinsser disease
D Q fever
ANSWER:D
259
Which of the following mycoplasmas has been implicated as a cause of nongonococcal
urethritis (NGU)?
A Mycoplasma hominis
B M. pneumoniae
C M. fermentans
D Ureaplasma urealyticum
ANSWER:D
260 Which of the following statements best describes human monocytic ehrlichiosis (HME)?
A The HME agent grows on artificial media
B It is a fatal disease transmitted by the bite of a dog
C Clinical diagnosis is based on the presence of erythema migrans (EM)
D
Diagnosis is usually made serologically but morulae may be seen in the cytoplasm of
monocytes
ANSWER:D
261
Lymphogranuloma venereum (LGV) is a venereal disease caused by serotype L1, L2, or L3
of Chlamydia trachomatis. The differential diagnosis should include which of the following?

A Psittacosis
B Chancroid
C Shingles
D Babesiosis
ANSWER:B
262
An inhibitor of ATP synthesis would be expected to retard most severely the penetration of
the host cell by which of the following organisms?
A Chlamydia psittaci
B C. trachomatis
C U. urealyticum
D Rickettsia rickettsii
ANSWER:D
263 C. trachomatis can be distinguished from C. psittaci by which of the following criteria?
A C. trachomatis is sensitive to sulfonamides
B C. trachomatis has a different lipopolysaccharide antigen
C C. trachomatis can be stained with Giemsa
D C. psittaci is an obligate prokaryotic parasite
ANSWER:A
264
Chlamydiae have an unusual three-stage cycle of development. The correct sequence of
these events is
A
Penetration of the host cell, synthesis of elementary body progeny, development of an initial
body
B
Penetration of the host cell, development of an initial body, synthesis of elementary body
progeny
C
Development of an initial body, synthesis of elementary body progeny, penetration of the
host cell
D
Synthesis of elementary body progeny, development of an initial body, penetration of the
host cell
ANSWER:B
265
Rickettsiae are Gram-negative bacteria that cause a wide range of diseases. The agent of
Rocky Mountain spotted fever (RMSF) is best characterized by the statement that it
A Grows on 7% sheep blood agar
B Has an “atypical” Gram-negative cell wall

C Is energy-deficient and cannot phosphorylate glucose
D Is normal flora of the mosquito gut
ANSWER:C
266
Human granulocytic ehrlichiosis (HGE) is a disease transmitted to humans by the bite of a
tick, Ixodes scapularis. Which of the following statements about HGE is most correct?
A Clinical diagnosis is based on the presence of erythema migrans
B HGE is a self-limiting disease
C HGE is caused by Ehrlichia chaffeensis
D
HGE is characterized by an acute onset of fever, severe headache, and influenzalike
symptoms
ANSWER:D
267
The “spotted fever” group of rickettsial diseases is caused by a variety of rickettsial species.
While not critical for treatment of disease, the speciation of these organisms is essential for
epidemiologic studies. Of the following rickettsiae, which one is found in the United States
and is a member of the spotted fever group?
A Rickettsia sibirica
B R. conorii
C R. akari
D R. prowazekii
ANSWER:C
268
A 36-year-old man presents at his physician’s office complaining of fever and headache. On
examination, he had leukopenia and increased liver enzymes, and inclusion bodies were seen
in his monocytes. History revealed that he was an outdoorsman and remembered removing a
tick from his leg. Which of the following diseases is most likely causing the symptoms
described?
A Lyme disease
B Ehrlichiosis
C Rocky Mountain spotted fever
D Q fever
ANSWER:B
269
Typhus, spotted fever, and scrub typhus share which of the following manifestations of
disease?
A Short incubation period (<48 h)

B Fever, rash, and rickettsemia
C Common vector
D Similar geographic distribution
ANSWER:B
270
C. trachomatis is a well-known cause of venereal disease. This organism is also implicated in
which of the following?
A Classic trachoma infection
B Sexually transmitted cardiac disease in adults
C Perinatal retinitis
D Middle-ear infection in young children
ANSWER:A
271
Which one of the following statements best characterizes lymphogranuloma venereum
(LGV)?
A The causative agent is C. trachomatis
B In the United States, it is more common among women
C It is most common in temperate regions
D Penicillin is effective in early treatment
ANSWER:A
272
rachoma is one of the leading causes of blindness. Which of the following best typifies the
disease?
A It is caused by C. trachomatis
B It is best treated with systemic cephalosporins and ophthalmic tetracycline
C It affects 400 million people in the Pacific Rim
D It is a form of chronic uveitis
ANSWER:A
273
Chlamydiae are small Gram-negative rods once thought to be viruses. Which of the
following best characterizes chlamydiae as distinct from viruses?
A Independent synthesis of proteins
B Susceptibility to antimicrobial agents
C Intracellular reproduction
D Synthesis of ATP
ANSWER:A

274
Chlamydia pneumoniae, sometimes known as Chlamydia “TWAR,” is the most recent
Chlamydia species to be associated with human disease. Which one of the following
statements best describes C. pneumoniae?
A C. pneumoniae infections are generally severe
B C. pneumoniae infections are uncommon—up to 10% of adults may show specific antibody
C Nonpsittacine birds are reservoirs of C. pneumoniae
D C. pneumoniae has been associated with myocardial infarction
ANSWER:D
275
Members of the family of Mycoplasma have a variety of clinical and epidemiological
features. Which one of the following organisms causes primary atypical pneumonia in
humans?
A M. hominis
B M. orale
C M. pneumoniae
D M. fermentans
ANSWER:C
276
Members of the family of Mycoplasma have a variety of clinical and epidemiological
features. Which one of the following organisms is associated with nongonococcal urethritis
in humans?
A M. hominis
B M. orale
C M. pneumoniae
D U. urealyticum
ANSWER:D
277
Members of the family of Mycoplasma have a variety of clinical and epidemiological
features. Which one of the following organisms normally inhabits the healthy human oral
cavity?
A M. hominis
B M. orale
C M. pneumoniae
D M. fermentans
ANSWER:B

278
Members of the family of Mycoplasma have a variety of clinical and epidemiological
features. Which one of the following organisms normally inhabits the female genital tract but
may cause acute respiratory illness?
A M. hominis
B M. orale
C M. pneumoniae
D M. fermentans
ANSWER:A
279
The following group of organisms is characterized as rather difficult to cultivate with some
interesting molecular and physiologic similarities. The causative agent of lymphogranuloma
venereum is
A B. (Rochalimaea) henselae
B E. chaffeensis
C C. trachomatis
D R. rickettsii
ANSWER:C
280
The following group of organisms is characterized as rather difficult to cultivate with some
interesting molecular and physiologic similarities. Which one of the following is transmitted
by the bite of a hard Ixodes tick?
A B. (Rochalimaea) henselae
B E. chaffeensis
C C. trachomatis
D R. rickettsii
ANSWER:B
281
The following group of organisms is characterized as rather difficult to cultivate with some
interesting molecular and physiologic similarities. Which of the following microorganisms is
the causative agent of catscratch fever?
A B. (Rochalimaea) henselae
B E. chaffeensis
C C. trachomatis
D R. rickettsii
ANSWER:A

282
The following group of organisms is characterized as rather difficult to cultivate with some
interesting molecular and physiologic similarities. Which of the following is a Gram-negative
rickettsia with tropism for mononuclear cells and causes Q fever?
A B. (Rochalimaea) henselae
B E. chaffeensis
C C. trachomatis
D C. burnetii
ANSWER:D
283
The following group of organisms is characterized as rather difficult to cultivate with some
interesting molecular and physiologic similarities. Which of the following is the causative
agent of Rocky Mountain spotted fever?
A B. (Rochalimaea) henselae
B E. chaffeensis
C C. trachomatis
D R. rickettsii
ANSWER:D
284 Infected tissues demonstrating budding fungal cells are seen in
A Coccidioidomycosis, chromomycosis, aspergillosis
B Tinea versicolor, tinea nigra, candidiasis
C Blastomycosis, paracoccidioides, dermatophytosis
D Candidiasis, cryptococcosis, and sporotrichosis
ANSWER:D
285
A 6-year-old girl presents to the clinic with scaly patches on the scalp. Primary smears and
culture of the skin and hair were negative. A few weeks later, she returned and was found to
have inflammatory lesions. The hair fluoresced under Wood’s light and primary smears of
skin and hair contained septate hyphae. On speaking with the parents, it was discovered that
there were several pets in the household. Which of the following is the most likely agent?
A Microsporum audouinii
B Microsporum canis
C Trichophyton tonsurans
D Trichophyton rubrum
ANSWER:B

286
A patient with AIDS has a persistent cough and has shown progressive behavioral changes in
the past few weeks after eating an undercooked hamburger. A cerebrospinal fluid (CSF)
sample is collected and an encapsulated, yeast-like organism is observed. Based only on these
observations, what is the most likely organism?
A Toxoplasma
B Cryptosporidium
C Candida
D Cryptococcus
ANSWER:D
287
A clinical diagnosis of meningitis is confirmed with a latex agglutination test on CSF for the
capsular polysaccharide of the organism. The most likely causative agent is
A Candida albicans
B Cryptococcus
C Paracoccidioides brasiliensis
D Histoplasma capsulatum
ANSWER:B
288
A section of tissue from the foot of a person assumed to have eumycotic mycetoma shows a
white, lobulated granule composed of fungal hyphae. In the United States, the most common
etiologic agent of this condition is a species of
A Acremonium
B Nocardia
C Actinomyces
D Pseudallescheria (Petriellidium)
ANSWER:D
289
The formation of granulomas is seen in major systemic fungal infections. Which of the
following groups of fungi is most likely to cause granulomas?
A Aspergillus, Coccidioides, Cryptococcus
B Mucor, Candida, Malassezia
C Cladosporium, Aspergillus, Microsporum
D Coccidioides, Blastomyces, Histoplasma
ANSWER:D
290
Infection with Sporothrix schenckii (formerly Sporotrichum schenckii) is an occupational
hazard for gardeners. The portal of entry for this organism is the

A Lymphatic system
B Respiratory tract
C Skin
D Mouth
ANSWER:C
291 C. albicans is recognized in microscopic examination of infected tissues by the presence of
A Spherules containing endospores
B Metachromatic granules
C Yeasts and pseudohyphae
D Asci containing 2–8 ascospores
ANSWER:C
292
The mechanism of mucosal invasion by C. albicans is at least partially understood. Which
one of the following modifications in the structure or function of this yeast would be most
likely to affect its invasive ability?
A Loss of ability to produce ethanol from glucose
B Loss of ability to produce germ tubes or hyphae
C Reduced ability to grow at 37°C
D Loss of ability to produce a polysaccharide capsule
ANSWER:B
293
You have been designated as coordinator of construction of a bone marrow transplant unit
(BMTU). There will be extensive removal of walls and floors in order to install the laminar
flow rooms required for a BMTU. From the standpoint of frequency and lethality, which one
of the following fungi should be your biggest concern?
A Aspergillus
B Candida
C Wangiella
D Cryptococcus
ANSWER:A
294
H. capsulatum, a dimorphic fungus, is found in soil heavily contaminated with bird
droppings. Which of the following statements best describes the presence of the organism in
tissue biopsies?
A Yeasts with broad-based bud
B Single-cell yeasts with pseudohyphae

C Arthrospores
D Oval budding yeasts inside macrophages
ANSWER:D
295 Which of the following best describes an infection with Coccidioides immitis?
A A negative complement-fixing (CF) antibody test
B Inhaled arthrospores form thick-walled spherules filled with endospores
C “Fungus ball” formation
D Thrush
ANSWER:B
296
Inhalation of fungal spores can cause primary lung infections. Of the following organisms,
which one is most likely to be associated with this mode of transmission?
A C. immitis
B S. schenckii
C C. albicans
D T. tonsurans
ANSWER:A
297
An immunocompromised patient is suspected of having an infection with A. fumigatus.
Which of the clinical conditions is most likely to occur?
A Wound infection
B Urinary tract infection
C Invasive aspergillosis causing thrombosis and infarction
D Thrush
ANSWER:C
298
Patients who have disseminated coccidioidomycosis may usually demonstrate which one of
the following?
A A negative coccidioidin skin test and a rising complement-fixing (CF) titer
B A negative coccidioidin skin test and a stable CF titer
C A positive skin test and a mildly elevated CF titer
D Absence of CF antibodies
ANSWER:A
299 Aspergillus is best described by which of the following statements?
A
Widespread in environment; conidia may be inhaled; microscopic appearance in specimen
reveals dichotomous branching and septate hyphae

B
Round, black sporangia filled with endospores; sporangia unbranched, rising from a runner
called a stolon
C
Single-tipped sporangiophores; no rhizoids or stolons; nonseptate hyphae, which show
branching
D
Yeast forms with budding blastoconidia often showing pseudohyphae; positive germ tube
test; chlamydospores present
ANSWER:A
300 Mucor is best described by which of the following statements?
A
Widespread in environment; conidia may be inhaled; microscopic appearance in specimen
reveals dichotomous branching and septate hyphae
B
Round, black sporangia filled with endospores; sporangia unbranched, rising from a runner
called a stolon
C
Single-tipped sporangiophores; no rhizoids or stolons; nonseptate hyphae, which show
branching
D
Yeast forms with budding blastoconidia often showing pseudohyphae; positive germ tube
test; chlamydospores present
ANSWER:C
301 Rhizopus is best described by which of the following statements?
A
Widespread in environment; conidia may be inhaled; microscopic appearance in specimen
reveals dichotomous branching and septate hyphae
B
Round, black sporangia filled with endospores; sporangia unbranched, rising from a runner
called a stolon
C
Single-tipped sporangiophores; no rhizoids or stolons; nonseptate hyphae, which show
branching
D
Yeast forms with budding blastoconidia often showing pseudohyphae; positive germ tube
test; chlamydospores present
ANSWER:B
302
The naming of fungi is very confusing to the nonmycologist. For this reason, the clinician
who may treat fungal infections should have a working knowledge of fungal taxonomy.
Most of the fungi known to cause infection in humans have been recognized for many years
by their asexual stage (anamorph). The sexual stage (teleomorph) of many of these familiar
fungi has now been discovered. The appropriate teleomorph for Trichophyton
mentagrophytes is

A Ajellomyces capsulata
B Ajellomyces dermatitidis
C Arthroderma van breuseghemii
D Filobasidiella neoformans
ANSWER:C
303
on, the clinician who may treat fungal infections should have a working knowledge of
fungal taxonomy. Most of the fungi known to cause infection in humans have been
recognized for many years by their asexual stage (anamorph). The sexual stage (teleomorph)
of many of these familiar fungi has now been discovered. The appropriate teleomorph for
Microsporum gypseum is
A Ajellomyces capsulata
B Ajellomyces dermatitidis
C Arthroderma van breuseghemii
D Nannizzia incurvata
ANSWER:D
304
The naming of fungi is very confusing to the nonmycologist. For this reason, the clinician
who may treat fungal infections should have a working knowledge of fungal taxonomy.
Most of the fungi known to cause infection in humans have been recognized for many years
by their asexual stage (anamorph). The sexual stage (teleomorph) of many of these familiar
fungi has now been discovered. The appropriate teleomorph for Cryptococcus neoformans is
A Ajellomyces capsulata
B Ajellomyces dermatitidis
C Arthroderma van breuseghemii
D Filobasidiella neoformans
ANSWER:D
305
The naming of fungi is very confusing to the nonmycologist. For this reason, the clinician
who may treat fungal infections should have a working knowledge of fungal taxonomy.
Most of the fungi known to cause infection in humans have been recognized for many years
by their asexual stage (anamorph). The sexual stage (teleomorph) of many of these familiar
fungi has now been discovered. The appropriate teleomorph for Blastomyces dermatitidis is
A Ajellomyces capsulata

B Ajellomyces dermatitidis
C Arthroderma van breuseghemii
D Filobasidiella neoformans
ANSWER:B
306
The naming of fungi is very confusing to the nonmycologist. For this reason, the clinician
who may treat fungal infections should have a working knowledge of fungal taxonomy.
Most of the fungi known to cause infection in humans have been recognized for many years
by their asexual stage (anamorph). The sexual stage (teleomorph) of many of these familiar
fungi has now been discovered. The appropriate teleomorph for Histoplasma capsulatum is
A Ajellomyces capsulata
B Ajellomyces dermatitidis
C Arthroderma van breuseghemii
D Filobasidiella neoformans
ANSWER:A
307 Barrel-shaped arthroconidia are characteristic of which one of the following fungi?
A E. floccosum
B C. immitis
C Phialophora verrucosa
D M. canis
ANSWER:B
308
Sporulation from flask-shaped, pigmented projections is commonly observed in which one
of the following fungi?
A E. floccosum
B C. immitis
C P. verrucosa
D M. canis
ANSWER:C
309 Clavate macroconidia are characteristic of which one of the following fungi?
A E. floccosum
B C. immitis
C P. verrucosa
D M. canis

ANSWER:A
310 Broad-based budding cells are characteristic of which one of the following fungi?
A E. floccosum
B C. immitis
C P. verrucosa
D B. dermatitidis
ANSWER:D
311
Rough-walled macroconidia of 8 to 15 cells are commonly observed in which one of the
following fungi?
A E. floccosum
B C. immitis
C P. verrucosa
D M. canis
ANSWER:D
312
Fungal skin diseases may not be named according to the etiologic agent but rather are called
tinea or a dermatophytosis. Tinea corporis is caused by which of the following?
A E. floccosum
B Malassezia furfur
C M. canis
D Exophiala werneckii
ANSWER:C
313
Fungal skin diseases may not be named according to the etiologic agent but rather are called
tinea or a dermatophytosis. Tinea cruris is caused by which of the following?
A E. floccosum
B M. furfur
C M. canis
D E. werneckii
ANSWER:A
314
Fungal skin diseases may not be named according to the etiologic agent but rather are called
tinea or a dermatophytosis. Tinea pedis is caused by which of the following?
A E. floccosum
B M. furfur
C M. canis

D E. werneckii
ANSWER:A
315
Fungal skin diseases may not be named according to the etiologic agent but rather are called
tinea or a dermatophytosis. Tinea capitis is caused by which of the following?
A E. floccosum
B M. furfur
C M. canis
D E. werneckii
ANSWER:C
316
Fungal skin diseases may not be named according to the etiologic agent but rather are called
tinea or a dermatophytosis. Tinea versicolor is caused by which of the following?
A E. floccosum
B M. furfur
C M. canis
D E. werneckii
ANSWER:B
317
Harmless molds commonly seen growing on bread, as well as on a wide variety of other
products or foodstuffs, grow equally well in the human host when the host’s defenses are
compromised. Which one of the following fungi causes subcutaneous zygomycosis? It is
most often seen in Africa and Asia; the infections are most often seen on the trunk or limbs
and begin as painless, small nodules; and the hyphae are 6 to 25 mm in length with irregular
branching:
A A. fumigatus
B C. albicans
C Conidiobolus coronata
D Basidiobolus ranarum
ANSWER:D
318
Harmless molds commonly seen growing on bread, as well as on a wide variety of other
products or foodstuffs, grow equally well in the human host when the host’s defenses are
compromised. Which one of the following fungi causes subcutaneous zygomycosis? The
infections usually involve the nasal area with swelling of nose and cheeks, and cases are seen
in Africa with rare cases in the Caribbean and South America.
A A. fumigatus

B C. albicans
C C. coronata
D R. arrhizus
ANSWER:C
319
Harmless molds commonly seen growing on bread, as well as on a wide variety of other
products or foodstuffs, grow equally well in the human host when the host’s defenses are
compromised. Which one of the following fungi causes allergic bronchopulmonary disease
resulting in marked elevation of serum IgE?
A A. fumigatus
B C. albicans
C C. coronata
D R. arrhizus
ANSWER:A
320
Harmless molds commonly seen growing on bread, as well as on a wide variety of other
products or foodstuffs, grow equally well in the human host when the host’s defenses are
compromised. Which one of the following fungi is often isolated from blood, urine, and
sputum in invasive disease? It is difficult to determine whether isolation from one body site
or body fluid is suggestive of colonization or infection.
A A. fumigatus
B C. albicans
C C. coronata
D R. arrhizus
ANSWER:B
321
Harmless molds commonly seen growing on bread, as well as on a wide variety of other
products or foodstuffs, grow equally well in the human host when the host’s defenses are
compromised. Which one of the following fungi causes rhinocerebral zygomycosis and is
usually associated with acute diabetes?
A A. fumigatus
B C. albicans
C C. coronata
D R. arrhizus
ANSWER:D

322
A person who had recently consumed half a box of raspberries came down with severe
watery diarrhea. Surprisingly, the patient felt tired for a few days and had a low-grade fever.
The most likely cause of infection is
A Cryptosporidium
B Escherichia coli 0157:H7
C Isospora
D Cyclospora
ANSWER:D
323
A 30-year-old female stored her contact lenses in tap water. She noticed deterioration of
vision and visited an ophthalmologist who diagnosed her with severe retinitis. Culture of the
water as well as vitreous fluid would most likely reveal
A Naegleria
B Pneumocystis
C Acanthamoeba
D Babesia
ANSWER:C
324
A recently recognized organism secretes potent toxins, has been responsible for significant
fish kills, and has also caused disease in humans. This organism has been identified as
A Listeria monocytogenes
B Giardia lamblia
C Vibrio parahaemolyticus
D Pfiesteria piscicida
ANSWER:B
325
Human infection with the beef tapeworm, Taenia saginata, usually is less serious than
infection with the pork tapeworm, T. solium, because
A Acute intestinal stoppage is less common in beef tapeworm infection
B Larval invasion does not occur in beef tapeworm infection
C Toxic by-products are not given off by the adult beef tapeworm
D The adult beef tapeworms are smaller
ANSWER:B

326
A man coughed up a long (4 to 6 cm) white worm and his chief complaint was abdominal
tenderness. He reports that he goes to sushi bars at least once a week. The following parasites
have been observed in people who eat raw fish: Anisakis, Pseudoterranova, Eustrongylides,
and Angiostrongylus. Which of the following would best differentiate the specific parasitic
agent?
A Identification of specific species of fish involved
B Study of distinctive morphology of the parasite
C Specific antibody tests
D Antigen detection in tissues
ANSWER:B
327
A survey of 100 healthy adults reveals that 80% have IgG antibodies to Toxoplasma. Which
one of the following statements would help to explain this finding?
A The potential for Toxoplasma infection is widespread and the disease is mild and self-limiting
B Toxoplasmosis is caused by eating meat; therefore, all meat eaters have had toxoplasmosis
C A variety of parasitic infections induce the formation of Toxoplasma antibody
D The test for Toxoplasma antibodies is highly nonspecific
ANSWER:A
328
In the United States, certain enteric protozoan and helminthic infections were previously
considered to be exotic illnesses related to foreign travel or to contaminated food or water.
However, sexual transmission of these diseases has produced a “hyperendemic” infection rate
among male homosexuals. The most common infection seen in this group is
A Giardiasis
B Ascariasis
C Amebiasis
D Enterobiasis
ANSWER:A
329
Analysis of a patient’s stool reveals small structures resembling rice grains; microscopic
examination shows these to be proglottids. The most likely organism in this patient’s stool is
A Enterobius vermicularis
B Ascaris lumbricoides
C Necator americanus

D T. saginata
ANSWER:D
330
Trypanosoma cruzi initially penetrates through the mucous membranes on the skin and then
multiplies in a lesion known as a chagoma. In the chronic stage of the disease, the main
lesions are often observed in the
A Spleen and pancreas
B Heart and digestive tract
C Liver and spleen
D Digestive tract and respiratory tract
ANSWER:B
331
A woman, recently returned from Africa, complains of having paroxysmal attacks of chills,
fever, and sweating; these attacks last a day or two at a time and recur every 36 to 48 h.
Examination of a stained blood specimen reveals ringlike and crescent-like forms within red
blood cells. The infecting organism most likely is
A Plasmodium falciparum
B Plasmodium vivax
C Trypanosoma gambiense
D Wuchereria bancrofti
ANSWER:A
332
A young man, recently returned to the United States from Vietnam, has severe liver disease.
Symptoms include jaundice, anemia, and weakness. An intermediate form of the organism
shown in the photomicrographs on the preceding page lives in
A Mosquitoes
B Pigs
C Snails
D Cows
ANSWER:C
333
One of the most clinically significant infections in patients with AIDS is Pneumocystis carinii
pneumonia (PCP). PCP is a treatable disease; therefore, rapid diagnosis is essential. The
method of choice for detection of P. carinii in respiratory specimens is
A Methenamine-silver stain
B Toluidine blue stain
C Direct fluorescent antibody (DFA) microscopy

D Indirect fluorescent antibody (IFA) microscopy
ANSWER:C
334
There are five varieties of cockroaches: the German cockroach, the brown-banded
cockroach, the oriental cockroach, the American cockroach, and the smoky brown
cockroach. A characteristic of cockroaches is their
A Transmission of Salmonella
B Toxic sting
C Function as a vector for Borrelia burgdorferi
D Function as a secondary host for rickettsiae
ANSWER:A
335
A “parasite” that may be a fungus is the initial clinical manifestation in up to 60% of patients
with AIDS. This organism is
A Microsporidium
B Cryptosporidium
C Pneumocystis
D Blastocystis
ANSWER:C
336
A medical technologist visited Scandinavia and consumed raw fish daily for 2 weeks. Six
months after her return home, she had a routine physical and was found to be anemic. Her
vitamin B12 levels were below normal. The most likely cause of her vitamin B12 deficiency
anemia is
A Excessive consumption of ice-cold vodka
B Infection with parvovirus B 19
C Infection with the fish tapeworm D. latum
D Infection with Yersinia
ANSWER:C
337
A renal transplant patient was admitted for graft rejection and pneumonia. A routine
evaluation of his stool showed rhabditiform larvae. Subsequent follow-up revealed similar
worms in his sputum. He had no eosinophils in his peripheral circulation. The most likely
organism is
A Necator
B Hymenolepsis
C Ascaris

D Strongyloides
ANSWER:D
338 Which of the following best describes lice?
A They have wings
B They cause tissue edema
C They transmit epidemic typhus, relapsing fever, and trench fever
D Pediculus humanus is the only species of louse
ANSWER:C
339
Scabies is caused by a small mite that burrows into the skin. The disease is best described by
which one of the following statements?
A It is caused by a species of Sarcoptes
B Secondary bacterial infection is rare
C It is synonymous with Kawasaki’s syndrome
D d. It is best diagnosed by biopsy of the inflammatory region around the mite bite
ANSWER:A
340
Amebae that are parasitic in humans are found in the oral cavity and the intestinal tract.
Which one of the following statements best describes these intestinal amebae?
A They are usually nonpathogenic
B They can cause peritonitis and liver abscesses
C They are usually transmitted as trophozoites
D They occur most abundantly in the duodenum
ANSWER:B
341
Schistosomiasis is a disease characterized by granulomatous reactions to the ova or to
products of the parasite at the place of oviposition. Clinical manifestations include which one
of the following?
A Bladder wall hyperplasia
B Pulmonary embolism
C Splenomegaly
D Cardiac abnormalities
ANSWER:C
342
B. burgdorferi, the causative agent of Lyme disease, has been isolated from a variety of ticks
such as Ixodes scapularis, Amblyomma, Dermacentor, and Ixodes pacificus. Which one of
the following statements is true of Lyme disease?

A I. scapularis and I. dammini are different types of ticks
B White-tailed deer, an important reservoir for I. scapularis, are dying because of Lyme disease
C Only a small percentage of people who get bitten by a tick develop Lyme disease
D Dermacentor and Amblyomma are significant vectors of B. burgdorferi to humans
ANSWER:C
343
Microsporidia are spore-forming, obligate, intracellular parasites. Which of the following
statements best describes microsporidia?
A There is one genus of microsporidia
B Microsporidia have been detected only in patients with AIDS
C
Microsporidia are too small to be observed in stool specimens even with the use of special
stains
D Microsporidia are found only in a variety of vertebrate and invertebrate hosts
ANSWER:D
344
Flukes (trematodes) have a great impact on worldwide public health, causing both morbidity
and mortality. Which of the following organisms penetrates skin, is endemic in Africa and
Latin America, and has a large lateral spine on its eggs?
A Paragonimus
B Clonorchis
C S. mansoni
D Schistosoma japonicum
ANSWER:C
345
Flukes (trematodes) have a great impact on worldwide public health, causing both morbidity
and mortality. Which of the following organisms may be ingested with raw fish, affects the
liver, and has an operculated egg?
A Paragonimus
B Clonorchis
C S. mansoni
D S. japonicum
ANSWER:B
346
Flukes (trematodes) have a great impact on worldwide public health, causing both morbidity
and mortality. Which of the following organisms penetrates skin, is endemic in Asia, and has
a small lateral spine on its eggs?

A Paragonimus
B Clonorchis
C S. mansoni
D S. japonicum
ANSWER:D
347
Flukes (trematodes) have a great impact on worldwide public health, causing both morbidity
and mortality. Which of the following organisms penetrates skin, is endemic in Africa and
the Middle East, has large terminal spines on its eggs, and is found in urine samples?
A Paragonimus
B Clonorchis
C S. mansoni
D S. haematobium
ANSWER:D
348 G. lamblia is best diagnosed by
A Sigmoidoscopy and aspiration of mucosal lesions
B Baermann technique
C Dilution followed by egg count
D Enzyme immunoassay (EIA)
ANSWER:D
349 The best method for the detection of Strongyloides larvae is
A Sigmoidoscopy and aspiration of mucosal lesions
B Baermann technique
C Dilution followed by egg count
D Enzyme immunoassay (EIA)
ANSWER:B
350 Ascaris are best observed in human specimens by which one of the following?
A Sigmoidoscopy and aspiration of mucosal lesions
B Baermann technique
C Dilution followed by egg count
D Enzyme immunoassay (EIA)
ANSWER:C

351
Certain parasites are found most commonly in particular body contents. A protozoan with
characteristic jerky motility is most commonly observed in
A Vaginal secretions
B Duodenal contents
C Blood
D Biopsied muscle
ANSWER:A
352
Certain parasites are found most commonly in particular body contents. A helminth that is
naturally transmitted by ingestion of pork, bear, or walrus meat could be detected in
A Vaginal secretions
B Duodenal contents
C Blood
D Biopsied muscle
ANSWER:D
353
Certain parasites are found most commonly in particular body contents. A tissue-dwelling
trematode that may be found in feces can also be detected in
A Vaginal secretions
B Duodenal contents
C Blood
D Sputum
ANSWER:D
354
Certain parasites are found most commonly in particular body contents. Cysts of a protozoan
adhere to a piece of nylon yarn coiled in a gelatin capsule which is swallowed. These cysts
are usually found in
A Vaginal secretions
B Duodenal contents
C Blood
D Biopsied muscle
ANSWER:B
355
Certain parasites are found most commonly in particular body contents. A parasite
resembling malaria that infects both animals and humans and is carried by the same tick that
transmits B. burgdorferi (the bacterium that causes Lyme disease) would most likely be
observed in

A Vaginal secretions
B Duodenal contents
C Blood
D Biopsied muscle
ANSWER:C
356
It is determined an infant suffers from Bruton’s agammaglobulinemia. Which of the
following pathogens will present the most serious threat to this child?
A Measles virus
B Mycobacterium tuberculosis
C Chlamydia trachomatis
D Varicella-zoster virus (VZV)
ANSWER:C
357 Megakaryocytes are minimally immunoresponsive; such cells are also known as
A B lymphocytes
B T lymphocytes
C Cytokines
D Platelets
ANSWER:D
358
One of the most remarkable aspects of the human immune system is its diversity, that is, the
ability to recognize a wide range of antigens and to mount a specific antibody response. This
is called clonal selection. At the cellular level, which of the following are primarily
responsible for such specificity?
A Cytotoxic T cells
B Hypervariable regions in domains of B cells
C The major histocompatibility complex
D Specific T cell receptors
ANSWER:B
359
A young girl has had repeated infections with Candida albicans and respiratory viruses since
the time she was 3 months old. As part of the clinical evaluation of her immune status, her
responses to routine immunization procedures should be tested. In this evaluation, the use of
which of the following vaccines is contraindicated?
A Diphtheria toxoid
B Bordetella pertussis vaccine

C Tetanus toxoid
D BCG
ANSWER:D
360
A latent, measles-like viral infection and, presumably, a defect in cellular immunity is
associated with which of the following diseases?
A Progressive multifocal leukoencephalopathy (PML)
B Multiple sclerosis (MS)
C Creutzfeldt-Jakob disease
D Subacute sclerosing panencephalitis (SSPE)
ANSWER:D
361
In humans, two closely linked genetic loci, each made up of two alleles, compose the
histocompatibility locus A (HL-A). Paired first and second locus antigens are called
haplotypes. The HL-A haplotypes (separated by a semicolon) of a child’s parents are given
below. Father 3,25;7,12 Mother 1,3;8,9 Assuming that no cross-over events have occurred,
the child’s histotype could be which of the following?
A 1,3;7,8
B 7,12;1,3
C 3,3;7,9
D 1,25;7,12
ANSWER:B
362
A 19-year-old college student develops a rash. She works part-time in a pediatric AIDS
clinic. Her blood is drawn and tested for specific antibody to the chickenpox virus (varicella-
zoster). Which of the following antibody classes would you expect to find if she is immune
to chickenpox?
A IgA
B IgD
C IgE
D IgG
ANSWER:D
363
A 34-year-old male patient visits a physician with complaints of fatigue, weight loss, night
sweats, and “swollen glands.” The physician also observes that he has an oral yeast infection.
Which of the following tests would most likely reveal the cause of his problems?
A A test for CD8 lymphocytes

B A human T-lymphotropic virus type I (HTLV-I) test
C An HIV ELISA test
D A test for infectious mononucleosis
ANSWER:C
364
A second patient makes an appointment. This patient, a 30-year-old male, requests a routine
HIV test. The HIV ELISA was weakly positive and is repeated with the same results. The
Western blot result is as shown in the preceding figure. The patient denies any risk factors for
HIV. Which of the following is the most likely cause of a falsely positive HIV test?
A A yeast infection
B Test cross-reactivity with HTLV
C Test cross-reactivity with Epstein-Barr virus
D A recent “flu” shot
ANSWER:D
365
Patients with C5 through C9 complement deficiencies would be most likely to have which of
the following infections?
A AIDS
B Meningococcal infection
C Pneumococcal infection
D Giardiasis
ANSWER:B
366
There are at least 10 properties of cytokines. Which of the following is one of these
characteristics?
A Mitogenesis
B B-cell lipids
C Lipopolysaccharide (LPS) activation
D T-cell differentiation
ANSWER:A
367
Immunity may be natural or acquired. Which of the following best describes acquired
immunity?
A Increase in C-reactive protein (CRP)
B Presence of natural killer (NK) cells
C Complement cascade

D Maternal transfer of antibody
ANSWER:D
368
A hapten is a nonimmunogenic small protein. Which of the following statements best
describes haptens?
A Haptens activate T cells
B Penicillin is a hapten
C Haptens do not react with specific antibody
D Haptens bind the major histocompatibility complex (MHC)
ANSWER:B
369 The major role of T cells in the immune response includes which one of the following?
A
Recognition of epitopes presented with major histocompatibility complex molecules on all
surfaces
B Complement fixation
C Phagocytosis
D Production of antibodies
ANSWER:A
370
Specific immunological unresponsiveness is called tolerance. Which one of the following
statements best describes immunological tolerance?
A Immunologic maturity of the host does not play a major role
B It occurs only with polysaccharide antigens
C It is related to the concentration of antibody
D It is prolonged by administration of immunosuppressive drugs
ANSWER:D
371
It appears that HIV binds selectively to CD4 glycoproteins. Thus, HIV shows a selective
infection with the destruction of helper T cells. Which of the following cells exhibit CD4
glycoprotein on their cell surface?
A Macrophages
B Polymorphonuclear leukocytes
C Suppressor lymphocytes
D Columnar epithelial cells
ANSWER:A
372
Interleukin 1 (IL-1) is a potent cytokine. It is best described by which one of the following
statements?

A Synthesis of IL-1 is inhibited in activated macrophages
B It can be produced by natural killer cells
C It exerts its effects on T and B cells as a costimulator
D It is multimeric and consists of more than one protein
ANSWER:B
373
Survival of allografts is increased by choosing donors with few major histocompatibility
complex (MHC) mismatches with recipients and by use of immunosuppression in recipients.
Which one of the following procedures might be a useful measure of immunosuppression?
A Administration of corticosteroids to recipient
B Lymphoid irradiation of donor
C Administration of immunoglobulin to recipient
D Destruction of donor B cells
ANSWER:A
374
Relative to the primary immunological response, secondary and later booster responses to a
given hapten-protein complex can be associated with which one of the following?
A Lower titers of antibody
B Increased antibody affinity for the hapten
C Decreased antibody avidity for the original hapten-protein complex
D Maintenance of the same subclass, or idiotype, of antibody produced
ANSWER:B
375
Which one of the following hypotheses may be sufficient to explain nonprecipitation in
antigen-antibody system?
A The antigen has a multivalent determinant
B The antigen has a single, nonrepeated determinant
C The antibody has been cleaved to divalent Fab′ ligands
D The antibody has been cleaved to divalent Fab′2 ligands
ANSWER:C
376
IgA antibody is the first line of defense against infections at the mucous membrane. It is
usually an early specific antibody. Which of the following statements regarding IgA is not
true?
A Complement fixation tests for IgA antibody will be positive if specific IgA antibody is present

B IgA is not found in saliva, therefore an IgA diagnostic test on saliva would have no value
C IgA can be destroyed by bacterial proteases
D IgA is absent in colostrum
ANSWER:C
377
Complement is a series of important host proteins which provide protection from invasion by
foreign microorganisms. Which one of the following statements best describes complement?
A Complement inhibits phagocytosis
B Microorganisms agglutinate in the presence of complement but do not lyse
C Complement plays a minor role in the inflammatory response
D
Complement protects the host from pneumococcal and Haemophilus infection through
complement components C1, C2, and C4
ANSWER:D
378
The following five questions relate to immunoglobulins, IgG, M, A, D, and E. Which
immunoglobulin has no known function, but is present on the surface of B lymphocytes? It
may function as an antigen receptor.
A IgG
B IgM
C IgE
D IgD
ANSWER:D
379
The following five questions relate to immunoglobulins, IgG, M, A, D, and E. Which of the
following is the immunoglobulin that is initially seen on the primary immune response? It is
present as a monomer on B cell surfaces but as a pentamer in serum.
A IgG
B IgM
C IgE
D IgA
ANSWER:B
380
The following five questions relate to immunoglobulins, IgG, M, A, D, and E. Which
immunoglobulin mediates immediate hypersensitivity and is involved in immune response to
parasitic infections?
A IgG

B IgM
C IgE
D IgA
ANSWER:C
381
he following five questions relate to immunoglobulins, IgG, M, A, D, and E. Which
immunoglobulin is the primary antibody in saliva, tears, and intestinal and genital secretions?
A IgG
B IgM
C IgE
D IgA
ANSWER:D
382
The following five questions relate to immunoglobulins, IgG, M, A, D, and E. Which
immunoglobulin is the predominant antibody in the secondary immune response? It has four
subclasses.
A IgG
B IgM
C IgE
D IgA
ANSWER:A
383 IgM antibody (1:200) to Borrelia burgdorferi is associated with
A Fifth disease
B Susceptibility to chickenpox
C Possible subacute sclerosing panencephalitis (SSPE)
D Acute Lyme disease
ANSWER:D
384 Elevated IgG and IgM antibody titers to parvovirus suggest a diagnosis of
A Fifth disease
B Susceptibility to chickenpox
C Possible subacute sclerosing panencephalitis (SSPE)
D Possible hepatitis B infection
ANSWER:A
385 A negative varicella antibody titer in a young woman signifies

A Fifth disease
B Susceptibility to chickenpox
C Possible subacute sclerosing panencephalitis (SSPE)
D Possible hepatitis B infection
ANSWER:B
386 A patient has an increased antibody titer to delta agent. You would most likely suspect
A Fifth disease
B Susceptibility to chickenpox
C Possible subacute sclerosing panencephalitis (SSPE)
D Possible hepatitis B infection
ANSWER:D
387
A patient with neurological problems has an elevated cerebrospinal fluid (CSF) antibody titer
to measles virus. You would most likely suspect
A Fifth disease
B Susceptibility to chickenpox
C Possible subacute sclerosing panencephalitis (SSPE)
D Possible hepatitis B infection
ANSWER:C
388
Infection with Epstein-Barr virus (EBV) results in the development of virus-specific
antibodies. The pattern of these antibodies helps to stage the illness. EBNA-Ab
A
Appears 2 weeks to several months after onset and is present more often in atypical cases of
infectious mononucleosis
B Appears 3 to 4 weeks after onset; titers correlate with severity of clinical illness
C Arises early in the course of the illness; detectable titers persist a lifetime
D Appears late in the course of the disease and persists a lifetime
ANSWER:D
389
Infection with Epstein-Barr virus (EBV) results in the development of virus-specific
antibodies. The pattern of these antibodies helps to stage the illness. EA-EBV (anti-D) Ab
A
Appears 2 weeks to several months after onset and is present more often in atypical cases of
infectious mononucleosis
B Appears 3 to 4 weeks after onset; titers correlate with severity of clinical illness
C Arises early in the course of the illness; detectable titers persist a lifetime
D Appears late in the course of the disease and persists a lifetime

ANSWER:B
390
Infection with Epstein-Barr virus (EBV) results in the development of virus-specific
antibodies. The pattern of these antibodies helps to stage the illness. EBV-VCA (IgG) Ab
A
Appears 2 weeks to several months after onset and is present more often in atypical cases of
infectious mononucleosis
B Appears 3 to 4 weeks after onset; titers correlate with severity of clinical illness
C Arises early in the course of the illness; detectable titers persist a lifetime
D Appears late in the course of the disease and persists a lifetime
ANSWER:C
391
Infection with Epstein-Barr virus (EBV) results in the development of virus-specific
antibodies. The pattern of these antibodies helps to stage the illness. EBV-VCA (IgM) Ab
A
Appears 2 weeks to several months after onset and is present more often in atypical cases of
infectious mononucleosis
B Appears 3 to 4 weeks after onset; titers correlate with severity of clinical illness
C Arises early in the course of the illness; detectable titers persist a lifetime
D Arises early in the course of the illness, and then titers fall rapidly
ANSWER:D
392 A xenograft is best described as a
A Transplant from one region of a person to another
B Transplant from one person to a genetically identical person
C Transplant from one species to the same species
D Transplant from one species to another species
ANSWER:D
393 An allograft is best described as a
A Transplant from one region of a person to another
B Transplant from one person to a genetically identical person
C Transplant from one species to the same species
D Transplant from one species to another species
ANSWER:C
394 An autograft is best described as a
A Transplant from one region of a person to another
B Transplant from one person to a genetically identical person
C Transplant from one species to the same species

D Transplant from one species to another species
ANSWER:A
395 An isograft is best described as a
A Transplant from one region of a person to another
B Transplant from one person to a genetically identical person
C Transplant from one species to the same species
D Transplant from one species to another species
ANSWER:B
396
Antigenic determinants on immunoglobulins are used to classify antibodies. An isotype is
characterized by
A Determinant exposed after papain cleavage to an F(ab) fragment
B
Determinant from one clone of cells and probably located close to the antigenbinding site of
the immunoglobulin
C
Determinant inherited in a Mendelian fashion and recognized by crossimmunization of
individuals in a species
D Heavy-chain determinant recognized by heterologous antisera
ANSWER:D
397
Antigenic determinants on immunoglobulins are used to classify antibodies. An allotype is
characterized by
A Determinant exposed after papain cleavage to an F(ab) fragment
B
Determinant from one clone of cells and probably located close to the antigenbinding site of
the immunoglobulin
C
Determinant inherited in a Mendelian fashion and recognized by crossimmunization of
individuals in a species
D Heavy-chain determinant recognized by heterologous antisera
ANSWER:C
398
Antigenic determinants on immunoglobulins are used to classify antibodies. An idiotype is
characterized by
A Determinant exposed after papain cleavage to an F(ab) fragment
B
Determinant from one clone of cells and probably located close to the antigenbinding site of
the immunoglobulin
C
Determinant inherited in a Mendelian fashion and recognized by crossimmunization of
individuals in a species

D Heavy-chain determinant recognized by heterologous antisera
ANSWER:B
399
Complement-fixation (CF) testing is an important serological tool. One has to understand the
conditions under which complement is bound and RBCs are lysed. Anti-Mycoplasma
antibody + complement + hemolysin-sensitized red blood cells (RBC) + anti-RBC antibody
results in
A Complement is bound, red blood cells are lysed
B Complement is bound, red blood cells are not lysed
C Complement is not bound, red blood cells are lysed
D Complement is not bound, red blood cells are not lysed
ANSWER:C
400
Complement-fixation (CF) testing is an important serological tool. One has to understand the
conditions under which complement is bound and RBCs are lysed. Anti-Mycoplasma
antibody + Mycoplasma antigen + complement + hemolysin-sensitized red blood cells + anti-
RBC antibody results in
A Complement is bound, red blood cells are lysed
B Complement is bound, red blood cells are not lysed
C Complement is not bound, red blood cells are lysed
D Complement is not bound, red blood cells are not lysed
ANSWER:B
401
Complement-fixation (CF) testing is an important serological tool. One has to understand the
conditions under which complement is bound and RBCs are lysed. Anti-Mycoplasma
antibody + Mycoplasma antigen + complement + hemolysin-sensitized red blood cells + anti-
RBC antibody results in
A Complement is bound, red blood cells are lysed
B Complement is bound, red blood cells are not lysed
C Complement is not bound, red blood cells are lysed
D Complement is not bound, red blood cells are not lysed
ANSWER:B

402
Most, but not all, cases of hepatitis are caused by hepatitis A virus (HAV), hepatitis B virus
(HBV), or non-A, non-B hepatitis virus (hepatitis C virus). While the laboratory diagnosis of
HAV is usually accomplished by the detection of IgG and IgM antibodies to HAV, the
diagnosis of HBV is more complex. Finding IgG antibodies to core antigen, antibodies to e
antigen, and antibodies to surface antigen reflects
A Acute infection (incubation period)
B Acute infection (acute phase)
C Post infection (acute phase)
D Immunization
ANSWER:C
403
Most, but not all, cases of hepatitis are caused by hepatitis A virus (HAV), hepatitis B virus
(HBV), or non-A, non-B hepatitis virus (hepatitis C virus). While the laboratory diagnosis of
HAV is usually accomplished by the detection of IgG and IgM antibodies to HAV, the
diagnosis of HBV is more complex. Finding HBsAg positive and HBeAg positive reflects
A Acute infection (incubation period)
B Acute infection (acute phase)
C Post infection (acute phase)
D Immunization
ANSWER:A
404
Most, but not all, cases of hepatitis are caused by hepatitis A virus (HAV), hepatitis B virus
(HBV), or non-A, non-B hepatitis virus (hepatitis C virus). While the laboratory diagnosis of
HAV is usually accomplished by the detection of IgG and IgM antibodies to HAV, the
diagnosis of HBV is more complex. Finding HBsAg positive, HBeAg positive, and IgM core
antibody positive reflects
A Acute infection (incubation period)
B Acute infection (acute phase)
C Post infection (acute phase)
D Immunization
ANSWER:B

405
Most, but not all, cases of hepatitis are caused by hepatitis A virus (HAV), hepatitis B virus
(HBV), or non-A, non-B hepatitis virus (hepatitis C virus). While the laboratory diagnosis of
HAV is usually accomplished by the detection of IgG and IgM antibodies to HAV, the
diagnosis of HBV is more complex. Finding HBsAg positive, no antibodies to HBsAg, and
other tests variable reflects
A Acute infection (incubation period)
B Acute infection (acute phase)
C Post infection (acute phase)
D HBV carrier state
ANSWER:D
406
Most, but not all, cases of hepatitis are caused by hepatitis A virus (HAV), hepatitis B virus
(HBV), or non-A, non-B hepatitis virus (hepatitis C virus). While the laboratory diagnosis of
HAV is usually accomplished by the detection of IgG and IgM antibodies to HAV, the
diagnosis of HBV is more complex. Finding antibodies to HBsAg reflects
A Acute infection (incubation period)
B Acute infection (acute phase)
C Post infection (acute phase)
D Immunization
ANSWER:D
407
There are a variety of immunologic tests available for the detection of both antigen and
antibody. Which of the following tests combines features of gel diffusion and
immunoelectrophoresis and is applicable only to negatively charged antigens?
A Latex agglutination (LA)
B Enzyme-linked immunosorbent assay (ELISA)
C Enzyme multiplied immunoassay test (EMIT)
D Counterimmunoelectrophoresis (CIE)
ANSWER:D
408
There are a variety of immunologic tests available for the detection of both antigen and
antibody. Which of the following tests depends on the presence of protein A on certain
strains of Staphylococcus aureus?
A Latex agglutination (LA)
B Enzyme-linked immunosorbent assay (ELISA)
C Enzyme multiplied immunoassay test (EMIT)

D Coagglutination (COA)
ANSWER:D
409
There are a variety of immunologic tests available for the detection of both antigen and
antibody. Which of the following tests is homogeneous immunoassay and is preferred for
detection of low-molecular-weight substances?
A Latex agglutination (LA)
B Enzyme-linked immunosorbent assay (ELISA)
C Enzyme multiplied immunoassay test (EMIT)
D Counterimmunoelectrophoresis (CIE)
ANSWER:C
410
There are a variety of immunologic tests available for the detection of both antigen and
antibody. Which of the following tests is used extensively to detect microbial antigens
rapidly (5 min or less)? Inert particles are sensitized with either antigen or antibody.
A Latex agglutination (LA)
B Enzyme-linked immunosorbent assay (ELISA)
C Enzyme multiplied immunoassay test (EMIT)
D Counterimmunoelectrophoresis (CIE)
ANSWER:A
411
There are a variety of immunologic tests available for the detection of both antigen and
antibody. Which of the following tests is heterogeneous immunoassay? Its detection system is
based on enzymatic activity.
A Latex agglutination (LA)
B Enzyme-linked immunosorbent assay (ELISA)
C Enzyme multiplied immunoassay test (EMIT)
D Counterimmunoelectrophoresis (CIE)
ANSWER:B
412
Antistreptolysin titer of 400 international units (IU) indicates which one of the following
diseases?
A Chronic infectious mononucleosis
B Primary syphilis
C Scarlet fever
D Primary atypical pneumonia
ANSWER:C

413 Hemagglutination inhibition titer (>1:20) suggests which one of the following diseases?
A Chronic infectious mononucleosis
B Primary syphilis
C Scarlet fever
D Immunity to rubella (German measles)
ANSWER:D
414 Reactive cold agglutinins suggests which one of the following diseases?
A Chronic infectious mononucleosis
B Primary syphilis
C Scarlet fever
D Primary atypical pneumonia
ANSWER:D
415
Epstein-Barr virus (EBV) VCA-IgG 1:80 and EBV antibody to early antigen (EA) 1:320
suggest which one of the following diseases?
A Chronic infectious mononucleosis
B Primary syphilis
C Scarlet fever
D Primary atypical pneumonia
ANSWER:A
416 A reactive rapid plasma reagin (RPR) test suggests which one of the following diseases?
A Chronic infectious mononucleosis
B Primary syphilis
C Scarlet fever
D Primary atypical pneumonia
ANSWER:B
417
A 31-year-old male patient complains of fatigue, yeast infection in his mouth, and enlarged
lymph nodes under his arms. He said that he was involved in “high-risk” behavior 6 years
ago while on a trip to eastern and southern Africa. He also indicated that his “HIV test” was
negative. Which one of the following options would be most appropriate?
A Initiate treatment for HIV disease
B Order a test for human T cell leukemia virus (HTLV)
C Repeat the test for HIV-1

D Order an HIV test which would include antibodies to HIV-1 and HIV-2
ANSWER:D
418
Which of the listed terms is described by: “All the chemical processes that take place in the
organelles and cytoplasm the cells of the body”?
A Metabolism
B Cellular respiration
C Homeostasis
D Physiology
ANSWER:A
419 Which major organ lies deep to the right hypochondriac region?
A The stomach
B The spleen
C The liver
D The duodenum
ANSWER:C
420 Which plane of the body divides it into dorsal and ventral regions?
A Transverse
B Axial
C Coronal
D Sagittal
ANSWER:C
421 To which of the following does the “tissue level” of structural organisation refer?
A atoms, ions, molecules and electrolytes
B mitochondria, ribosomes, nucleus, endoplasmic reticulum
C nephron, alveolus, villus, lobule
D muscle, nervous, connective, epithelial
ANSWER:D
422 The directional term “superior” in anatomy means which of the following?
A cephalic
B ventral
C caudal
D dorsal
ANSWER:A

423 Which of the following is the best definition of physiology?
A The microscopic study of tissues and cells
B The study of how the body works.
C All the chemical processes that take place in the organelles of the body’s cells.
D The body’s automatic tendency to maintain a relatively constant internal environment.
ANSWER:B
424 The “anatomical position” could be described as which of the following?
A Lying down prone
B Lying down supine
C Standing displaying the ventral surface of the body
D Standing with arms and legs abducted
ANSWER:C
425 Which choice best describes the location of the majority of the musculo-skeletal system?
A It is in the dorsal cavity
B It is in the ventral cavity
C It is in the abdomino-pelvic cavity
D It is not located in a body cavity
ANSWER:D
426 Which of the following is/are the contents of the ventral cavity?
A heart and lungs
B brain and spinal cord
C viscera
D gut, kidneys, liver, pancreas, spleen, bladder, internal reproductive organs.
ANSWER:C
427 Which of the stated relationships is correct?
A the heart is inferior to the clavicle
B the shoulder is distal to the carpals
C the phalanges are proximal to the metacarpals
D the eye is medial to the eyebrows
ANSWER:A
428 Which of the following is/are the contents of the dorsal body cavity?
A heart and lungs
B brain and spinal cord

C viscera
D gut, kidneys, liver, pancreas, spleen, bladder, internal reproductive organs.
ANSWER:B
429 Which of the stated relationships is correct?
A the heart is superior to the large intestine
B the shoulder is distal to the metacarpals
C the phalanges are proximal to the carpals
D the eye is medial to the nose
ANSWER:A
430 What is the study of how body parts function called?
A histology
B physiology
C homeostasis
D metabolism
ANSWER:B
431 Which of the following correctly describes the two named body parts?
A the elbow is proximal to the shoulder
B the phalanges are distal to the carpals
C the ribs are proximal to the sternum
D the elbow is distal to the knee
ANSWER:B
432 Which one of the following statements is correct?
A the diaphragm separates the brain and spinal cord
B the ventral cavity contains the male and female reproductive system
C the abdomino-pelvic cavity contains the spinal cord.
D the dorsal cavity contains the brain and spinal cord
ANSWER:D
433 Complete the sentence correctly: “Cervical vertebrae are……
A superior to the rib cage.
B inferior to the thoracic vertebrae.
C located between the thoracic and sacral vertebrae.
D fused into a single bone called the sacrum.
ANSWER:A

434 The dorsal body cavity contains which of the following organs?
A The brain.
B The brain and spinal cord.
C The brain, spinal cord and heart.
D The brain, spinal cord, heart and kidneys.
ANSWER:B
435 What does the process known as anabolism refer to?
A the use of energy for producing chemical substances.
B the breaking down phase of metabolism.
C all the chemical process that take place in the organelles of the cells.
D the supply of nutrients to the body’s cells.
ANSWER:A
436 To what does the term “hypochondriac” refer?
A A condition of having too few chondria.
B The region of abdomen inferior to the ribs.
C A person who often complains of an ailment.
D Having insufficient cartilage in the knees.
ANSWER:B
437
If a medical image displays internal anatomy in mid-sagittal section, which of the following
describes the section?
A
A vertical section through the nose and umbilicus that divides the body into right and left
halves.
B A cross-section through the midriff at about the level of the liver.
C A cross-section through the upper chest at about the level of the shoulders.
D
A vertical section through the midpoint of the clavicle and through either the right or left
thigh.
ANSWER:A
438 Which of the following best describes the “anatomical position”?
A
Standing vertically, arms held horizontally, legs apart so that the tips of the head, hands and
feet lie on an imaginary circle, drawn around the body.
B
Standing “to attention”, with hands held so that thumbs are ventral while the fifth digit is
dorsal.

C
Standing “at ease” with hands clasped behind your back while adjacent and dorsal to the
sacrum.
D
Standing vertically, arms parallel and lateral to the ribs with hands inferior to the elbows and
supinated.
ANSWER:D
439 Which term describes the location of the adrenal glands with reference to the kidneys?
A proximal
B distal
C superior
D inferior
ANSWER:C
440 Which of the following terms is NOT used to identify a region of the abdomen?
A left hypochondriac
B hypogastric
C epigastric
D right sacral
ANSWER:D
441 What structure separates the thoracic cavity from the abdominal cavity?
A The mediastinum
B The diaphragm
C The peritoneum
D The pylorus
ANSWER:B
442 Which structure within the cell produces ATP (adenosine triphosphate)?
A the mitochondria
B the nucleus
C peripheral proteins
D the endoplasmic reticulum
ANSWER:A
443 Which of the following is NOT a component of the cell plasma membrane?
A cholesterol
B proteins
C microfilaments

D phospholipids
ANSWER:C
444 Which list below contains the four types of tissue?
A extracellular fluid, skeletal tissue, glandular tissue, connective tissue.
B extracellular fluid, muscle tissue, glandular tissue, cartilaginous tissue.
C neural tissue, skeletal tissue, epithelial tissue, cartilaginous tissue.
D Neural tissue, muscle tissue, epithelial tissue, connective tissue.
ANSWER:D
445 Except for one, the following are types of cells. Which one is NOT a type of cell?
A platelets
B leucocytes
C macrophages
D osteoblasts
ANSWER:A
446
In which part of a cell does the process of making ATP from oxygen and glu- cose take
place?
A lysosomes
B ribosomes
C mitochondria
D golgi apparatus
ANSWER:C
447 Which of the following is a function of membrane proteins?
A to process lipids and proteins for secretion through the plasma membrane
B to act as receptors for hormones
C to synthesise proteins from amino acids
D to act as a cytoskeleton to support and shape the cell
ANSWER:B
448 What is the difference between simple squamous cells and simple columnar cells?
A squamous cells are flattened while columnar cells are taller than they are wide.
B simple squamous cells are one layer thick while simple columnar cells are several layers thick.
C simple squamous cells are epithelial tissue while simple columnar cells are connective tissue.

D squamous cells are flattened while columnar cells are cuboidal.
ANSWER:A
449 Which of the following is NOT an example of a cell?
A macrophages
B lysosomes
C plasmocytes
D chondroblasts
ANSWER:B
450
Which cell organelles contain an acidic environment capable of digesting a wide variety of
molecules?
A Lysosomes
B Ribosomes
C Centrosomes
D Golgi complex
ANSWER:A
451
Which form of transport through the plasma membrane requires the expendi- ture of energy
by the cell?
A Facilitated diffusion
B Osmosis
C Active transport
D Diffusion
ANSWER:C
452 Which of the tissue types below consists of a single layer of cells?
A stratified squamous epithelial tissue
B glandular epithelium
C areolar connective tissue
D simple columnar epithelial tissue
ANSWER:D
453 One of the following is NOT a serous membrane. Which one?
A pleura
B peritoneum
C mucosa
D pericardium

ANSWER:C
454 Which of the following is NOT made predominantly from epithelial tissue?
A In the dermis
B In exocrine glands
C In endocrine glands
D In the endothelium of blood vessels
ANSWER:A
455 What are tendons and ligaments composed of?
A Dense connective tissue
B Liquid connective tissue
C Muscular tissue
D Epithelial tissue
ANSWER:A
456 What is the composition of the intercellular matrix in connective tissue?
A Cells and fibres
B Serous and mucus membranes and lamina propria
C Protein fibres and ground substance
D Interstitial fluid
ANSWER:C
457 Which statement about the plasma membrane is INCORRECT?
A It is selectively permeable.
B It is composed of two layers of glycoprotein molecules.
C It contains receptors for specific signalling molecules.
D The plasma membranes of adjacent cells are held together by desmosomes.
ANSWER:B
458 Which of the following is NOT epithelial tissue?
A the epidermis
B glandular tissue
C the internal lining of blood vessels
D the dermis
ANSWER:D
459 Which of the following is NOT a cell found in connective tissue?
A adipocytes

B chondroblasts
C keratinocytes
D osteoblasts
ANSWER:C
460
What tissue has cells that are closely packed and that have one surface attached to a basement
membrane and the other free to a space?
A epithelial tissue
B muscle tissue
C connective tissue
D nervous tissue
ANSWER:A
461
What is the name of the mechanism that ensures that there is a higher concen- tration of
sodium ions in the extracellular fluid than in the intracellular fluid?
A Facilitated diffusion
B The sodium-potassium pump
C Secondary active transport
D Osmosis
ANSWER:B
462 What are lysosomes, centrosomes and ribosomes example of?
A stem cells
B organelles within a cell
C sensory receptors in the dermis
D exocrine glands
ANSWER:B
463 What does simple columnar epithelial tissue refer to? Tissue with
A a single layer of cells longer than they are wide.
B a single layer of cells whose length, breadth and depth are about the same size.
C several layers of cells, all of the same type.
D several layers of cells but without a basement membrane.
ANSWER:A
464 Which of the following is NOT an example of connective tissue?
A blood
B bone

C tendon
D epidermis
ANSWER:D
465 What is the function of phospholipids in the plasma membrane?
A to maintain the intracellular fluid at a similar composition to that of the interstitial fluid.
B to form channels to selectively allow passage of small molecules.
C to act as receptors for signalling chemicals.
D to present a barrier to the passage of water-soluble molecules.
ANSWER:D
466 Which one of the following cell types is found in epithelial tissue?
A plasma cells
B leucocytes
C keratinocytes
D chondroblasts
ANSWER:C
467 Which of the following is NOT part of the plasma membrane of a cell?
A integral proteins
B glycoproteins
C plasma proteins
D peripheral proteins
ANSWER:C
468 A major role for mitochondria is to
A transcribe the information in DNA (deoxyribonucleic acid)
B produce ATP (adenosine triphosphate)
C synthesise proteins from amino acids
D use enzymes to lyse molecules
ANSWER:B
469 Choose the tissue below that is one of the four primary types of body tissue.
A epidermal tissue
B epithelial tissue
C interstitial tissue
D osseous tissue
ANSWER:B

470 What are the primary types of tissue in the body?
A Muscle, nervous, connective and epithelial
B Muscle, nervous, connective, osseous and epithelial
C Muscle, nervous, connective, osseous, blood and epithelial
D Muscle, nervous, connective, glandular and epithelial
ANSWER:A
471 What is the name of the membrane that surrounds the lungs?
A visceral peritoneum
B parietal peritoneum
C visceral pleura
D dura mater
ANSWER:C
472 What is a role performed by mitochondria?
A contain enzymes capable of digesting molecules
B produce ATP
C synthesise proteins
D synthesise fatty acids, phospholipids & cholesterol
ANSWER:B
473 Which of the following is NOT found in the plasma membrane?
A proteins
B cholesterol
C endoplasmic reticulum
D phospholipids
ANSWER:C
474 Which one of the following cell types is found in epithelial tissue?
A mast cells
B adipocytes
C chondroblasts
D keratinocytes
ANSWER:D
475 Which of the following is NOT part of the plasma membrane of a cell?
A phospholipid
B glycoprotein

C chromatin
D cholesterol
ANSWER:C
476 A major role for mitochondria is to
A synthesise fatty acids, phospholipids & steroids
B deliver lipids and proteins to plasma membrane for secretion
C synthesise proteins from amino acids
D produce ATP (adenosine triphosphate)
ANSWER:D
477 Choose the tissue below that is NOT one of the four primary types of body tissue.
A connective tissue
B muscular tissue
C nervous tissue
D osseous tissue
ANSWER:D
478 What is the purpose of mitochondria?
A to store the nucleolus and chromatin
B to produce adenosine triphosphate
C to support and shape the cell.
D they produce enzymes to break down molecules
ANSWER:B
479
The plasma membrane of a cell contains molecules that have a hydrophobic end and a
hydrophilic end What are they called?
A phospholipids
B cholesterol
C integral proteins
D glycoproteins
ANSWER:A
480 Adipocytes are found in which type of tissue?
A muscle tissue
B epithelial tissue
C nervous tissue
D connective tissue

ANSWER:D
481 What is the role of mitochondria? To:
A function in cell division
B synthesise proteins
C form part of the plasma membrane
D synthesise fatty acids, phospholipids and steroids.
ANSWER:C
482 Which one of the following cell types is found in epithelial tissue?
A mast cells
B adipocytes
C chondroblasts
D melanocytes
ANSWER:D
483 What is the difference between “loose” connective tissue (CT) and “dense” connective tissue?
A Fibres occupy most of the volume in dense CT
B Dense CT includes cartilage, loose CT does not.
C Loose CT has a good blood supply while dense CT does not.
D Loose CT has no fibres (and dense CT does).
ANSWER:A
484 Facilitated diffusion refers to the process of
A movement along a concentration gradient assisted by protein carrier molecules.
B movement of ions and molecules along a concentration gradient.
C transport of molecules and ions against their concentration gradient.
D water movement through a semi-permeable membrane
ANSWER:A
485 What do fibroblasts, chondroblasts, osteoblasts and haemocytoblasts have in common?
A they are all types of white blood cell.
B they are all macrophages.
C they are all immature cells.
D they are all types of epithelial cell.
ANSWER:C
486 Which is NOT true of connective tissue (CT)?

A the cells are closely packed
B the tissue contains protein fibres and ground substance.
C types include loose CT, dense CT and liquid CT.
D CT contains white blood cells.
ANSWER:A
487 Active transport across the plasma membrane may be described by which statement?
A active transport requires energy from ATP.
B active transport is also known as endocytosis.
C active transport moves molecules along their concentration gradient.
D active transport is the movement of lipid-soluble molecules through the plasma membrane.
ANSWER:A
488 Which of the following cell types denotes an immature cell?
A macrophages
B monocytes
C osteoblasts
D ribosomes
ANSWER:C
489 Choose the membrane that is NOT a serous membrane.
A pleura
B peritoneum
C pericardium
D lamina propria
ANSWER:D
490 Which organelle is the site of ATP production?
A the nucleus
B endoplasmic reticulum
C mitochondria
D golgi apparatus
ANSWER:C
491 Which of the following is ONE major function of epithelial cells?
A movement
B secretion
C support of other cell types

D transmit electrical signals
ANSWER:B
492 What are the major types of tissue in the body?
A nervous, muscle, epithelial, connective.
B squamous, cuboidal, columnar, transitional.
C osteocytes, chondrocytes, leucocytes, adipocytes.
D protein, adipose, cartilage, osseous.
ANSWER:A
493 Which of the following is NOT one of the organelles within a cell?
A desmosome
B endoplasmic reticulum
C mitochondrion
D golgi apparatus
ANSWER:A
494 Which list contains the main body tissue types?
A glandular, connective, osseous, nervous
B epithelial, nervous, connective, muscle.
C endothelial, connective, muscle, cartilaginous
D epithelial, cartilaginous, muscle, glandular
ANSWER:B
495
The process of “diffusion” through a membrane may be described by which of the
following?
A
the movement of ions and molecules away from regions where they are in high
concentration towards regions where they are in lower concentration.
B
the use of energy from ATP to move ions and small molecules into regions where they are in
lower concentration.
C the plasma membrane engulfs the substance and moves it through the membrane.
D the use of energy from ATP to move water molecules against their concen- tration gradient.
ANSWER:A
496
The process of “active transport” through a membrane may be described by which of the
following?

A
the movement of ions and small molecules away from regions where they are in high
concentration.
B
the use of energy from ATP to move ions and small molecules into regions where they are in
lower concentration.
C the plasma membrane engulfs the substance and moves it through the membrane.
D
the use of energy from ATP to move ions and small molecules against their concentration
gradient.
ANSWER:D
497 Which of the following is the smallest living structural unit of the body?
A atom
B molecule
C organelle
D cell
ANSWER:D
498 Which of the following enables ions such as sodium to cross a plasma membrane?
A phospholipid bilayer
B peripheral proteins
C integral proteins
D desmosomes
ANSWER:C
499
Cell membranes can maintain a difference in electrical charge between the inte- rior of the
cell and the extracellular fluiWhat is this charge difference called?
A excitability
B the membrane potential
C the action potential
D the sodium-potassium pump
ANSWER:B
500
The resting membrane potential of a cell is the consequence of which of the following
concentrations of ions?
A High K+ and Cl− outside the cell and high Na+ and large anions inside the cell.
B High K+ and Na+ outside the cell and high Cl− and large anions inside the cell.
C High Cl− and Na+ outside the cell and high K+ and large cations inside the cell.
D High Ca+ and Na+ outside the cell and high K+ and large cations inside the cell.

ANSWER:C
501 What is one function of mitochondria? To
A produce enzymes to break down molecules
B produce molecules of ATP
C hold adjacent cells together
D allow passage of molecules through the plasma membrane
ANSWER:B
502 Membrane proteins perform the following functions EXCEPT one. Which One?
A form the glycocalyx
B act as receptor proteins
C form pores to allow the passage of small solutes
D behave as enzymes.
ANSWER:A
503 Facilitated diffusion differs from active transport because facilitated diffusion:
A requires energy from ATP
B moves molecules from where they are in lower concentration to higher concentration
C moves molecules from where they are in higher concentration to lower concentration.
D involves ions & molecules that pass through membrane channels.
ANSWER:C
504 Which of the following is NOT a connective tissue?
A blood
B mesothelium
C fat
D tendon
ANSWER:B
505 The cells that are found in tendons are called:
A osteocytes
B adipocytes
C haemocytoblasts
D fibroblasts
ANSWER:D
506 Which one of the following terms best describes the structure of the cell membrane:
A fluid mosaic model

B static mosaic model
C quaternary structure
D multilayered structure
ANSWER:A
507 Which one of the following terms best describes a phospholipiIt consists of a:
A polar head and polar tail
B non-polar head and a polar tail
C polar head and non-polar tail
D non-polar head and a non-polar tail
ANSWER:C
508 One of the functions of integral proteins in cell membranes is to:
A maintain the rigid structure of the cell
B support mechanically the phospholipids
C interact with the cytoplasm
D form channels for transport functions
ANSWER:D
509 Which one of the following best describes what a cell membrane consists of?
A lipids, proteins, ribosomes
B lipids, cholesterol, proteins
C cholesterol, proteins, cytoplasm
D lipids, proteins, cytoplasm
ANSWER:B
510
Which one of the following organelles is considered as the “energy producing” centre of the
cell?
A rough endoplasmic reticulum
B Golgi apparatus
C mitochondria
D ribosomes
ANSWER:C
511 What is the major function of lysosomes? They:
A package proteins
B detoxify toxic substances
C catalyse lipid metabolism

D digest unwanted particles within the cell
ANSWER:D
512 What is the purpose of the “sodium/potassium pump”
A to perform endocytosis.
B to move sodium and potassium by facilitated diffusion.
C to perform bulk transport through the plasma membrane.
D To produce a concentration gradient for sodium ions
ANSWER:D
513 Which of the following is NOT a type of cell?
A ribosome
B haemocytoblast
C neutrophil
D phagocyte
ANSWER:A
514
What is the name of the mechanism that ensures that there is a higher concen- tration of
sodium ions in the extracellular fluid than in the intracellular fluid?
A Facilitated diffusion
B The sodium-potassium pump
C Secondary active transport
D Osmosis
ANSWER:B
515
What is the name given to the type of transport where glucose or an amino acidbinds to a
receptor protein on the plasma membrane, which then moves the molecule into the cell
without the expenditure of energy?
A facilitated diffusion
B bulk transport
C secondary active transport
D active transport
ANSWER:A
516
What is the name given to the movement of glucose or amino acids from the gut into the
cells lining the gut, when they bind to a transport protein that has also bound a sodium ion.
The sodium ion is entering the cell along its concentration gradient.
A facilitated diffusion

B the sodium potassium pump
C active transport
D secondary active transport
ANSWER:D
517 Mitochondria produce which of the following?
A ATP
B DNA
C RNA
D proteins
ANSWER:A
518
Why does the plasma membrane of a cell present a barrier to the movement of electrolytes
through it?
A There are no channels in the membrane for the passage of electrolytes.
B Electrolytes are not soluble in the lipid of the membrane.
C Electrolytes are too large to pass through membrane channels.
D Membrane proteins electrically repel charged particles.
ANSWER:B
519 Which of the following statements about “leak channels” in the plasma mem-brane is correct?
A Proteins that form these channels bind to solutes to allow them to pass into the cell.
B
They are passageways formed by proteins to allow water and ions to move passively through
the membrane.
C They allow small ions and molecules to move between adjacent cells.
D They are formed by glycoprotein and proteoglycans to allow hormones to enter cells.
ANSWER:B
520 What are the primary types of body tissue?
A connective tissue, blood, muscle tissue, nervous tissue, epithelial tissue.
B muscle tissue, osseous tissue, epithelial tissue, nervous tissue, blood, con- nective tissue.
C nervous tissue, epithelial tissue, muscle tissue, connective tissue
D epithelial tissue, connective tissue, adipose tissue, muscle tissue, nervous tissue.
ANSWER:C
521
Epithelial and connective tissue differ from each other in which of the follow- ing
characteristics?

A epithelial tissue contains fibres but connective tissue does not.
B connective tissue is avascular but epithelial tissue is well-vascularised.
C cells in epithelial tissue are closely packed, whereas in connective tissue they are not.
D
connective tissue includes tissue that makes up glands, but epithelial tissue does not occur in
glands.
ANSWER:C
522 Which of the following is a component of the plasma membrane of a cell?
A plasma
B glycolipid
C plasma proteins
D cholesterol
ANSWER:D
523
What term is used to describe the movement of dissolved particles along (or down) their
concentration gradient?
A endocytosis
B active transport
C osmosis
D diffusion
ANSWER:D
524 Which of the following molecules cannot pass through the plasma membrane?
A water molecules
B non-polar molecules
C amino acid based hormones
D fat-soluble molecules
ANSWER:C
525 Which of the following is a connective tissue?
A pancreas
B spinal cord
C muscle
D blood
ANSWER:D
526 Which of the following is an epithelial tissue?
A adipose tissue

B the adrenal gland
C the heart
D blood
ANSWER:B
527 What is the major component of the plasma membrane of a cell?
A phospholipid
B glycolipid
C integral protein
D cholesterol
ANSWER:A
528 Which one of the following is NOT a function of membrane proteins?
A they form a structure called a glycocalyx
B they attach cells to each other
C they form passageways to allow solutes to pass through the membrane
D they from receptors which can bind messenger molecules
ANSWER:A
529 Facilitated diffusion through a membrane involves which of the following scenarios?
A
the diffusion of water through a selectively permeable membrane along its concentration
gradient.
B the movement of a molecule against its concentration gradient with the expenditure of energy
C
the plasma membrane surrounding (engulfing) the molecule & the molecule moving into the
cell.
D
a molecule binding to a receptor which moves the molecule through the membrane without
the expenditure of energy
ANSWER:D
530 The diffusion of water through a membrane is referred to as
A secondary active transport
B bulk transport
C osmosis
D endocytosis
ANSWER:C
531 What is the tissue that covers the body surface and lines internal tubes called?

A epithelial tissue
B connective tissue
C glandular epithelium
D muscle tissue
ANSWER:A
532 Which of the following is true for connective tissue?
A it consists of cells, a basement membrane and intercellular matrix
B its cells are closely packed and held together by protein fibres.
C it has a high rate of cell division and no blood supply
D it is made of cells, protein fibres and ground substance
ANSWER:D
533
The cell membrane’s resting potential (about −0 mV inside with respect to the outside) is due
mainly to which of the following mechanisms?
A The sodium potassium pump.
B
The diffusion of cations and anions through the membrane along their con- centration
gradients.
C The diffusion of sodium and potassium across the cell membrane.
D The presence inside the cell of anions too large to passively cross the cell membrane.
ANSWER:A
534 The term “chromatin” would be used in reference to which of the following?
A genetic substance
B cellular energy
C membrane support
D nuclear membrane
ANSWER:A
535 In protein synthesis, where dose translation occur? In the:
A cytoplasm between ribosomes, tRNA and mRNA
B nucleus between ribosomes, tRNA and mRNA
C nucleus between DNA and mRNA
D cytoplasm between DNA and mRNA
ANSWER:A
536 If the DNA strand sequence of bases is CTT AGA CTA ATA, what would the tRNA read?
A GAA TCT GAT TAT

B CUU AGA CUA AUA
C GAA UCU GAU UAU
D GUU ACA GUA AUA
ANSWER:C
537 Which one of the following statements best describes DNA?
A single stranded, deoxyribonucleic acid
B single stranded, ribonucleic acid
C double stranded, deoxyribonucleic acid
D double stranded, ribonucleic acid
ANSWER:C
538 In which phase of mitosis would chromosomes line up at the centre of the spindle:
A anaphase
B interphase
C prophase
D metaphase
ANSWER:D
539 In a cell cycle which phase takes the longest time to complete:
A anaphase
B interphase
C prophase
D telophase
ANSWER:B
540 What is the purpose of meiosis? To produce:
A DNA
B somatic cells
C diploid cells
D haploid cells
ANSWER:D
541 What results from the events that occur during metaphase of mitosis?
A The nuclear membranes form around two nuclei.
B The chromosomes are aligned on a plane in the centre of the cell.
C The chromosomes become visible and attach to the spindle fibres.
D The chromatids from each chromosome separate and move to opposite sides of the cell.

ANSWER:B
542
What is the name of the process of division of a somatic cell’s nucleus into two daughter
nuclei?
A prophase
B cytokinesis
C mitosis
D meiosis
ANSWER:C
543
In a strand of DNA, what is the combination of deoxyribose and phosphate and base known
as?
A A ribosome
B A chromatid
C A codon
D A nucleotide
ANSWER:D
544 What happens during anaphase of mitosis?
A spindle fibres pull each chromatid to opposite sides of the cell
B the sense and non-sense strands “unzip” along their hydrogen bonds
C RNA polymerase forms a complementary strand by reading the sense strand
D the cell cytoplasm divides into two cells
ANSWER:A
545
The process by which information is read from DNA, encoded and transported outside the
nucleus is known as:
A translation
B transcription
C encoding
D catalysis
ANSWER:B
546 How many nucleotides are required to code for a single amino acid?
A twenty
B five
C three
D one

ANSWER:C
547
The combination of a sugar, a base and at least one phosphate group is given the general
term of:
A nucleoside
B amino acid
C polypeptide
D nucleotide
ANSWER:D
548 The nucleus of the cell contains the master nucleic acid:
A DNA
B RNA
C mRNA
D tRNA
ANSWER:A
549
Which of the following is the correct combination of the components for the nucleic acid
DNA?
A Phosphate, Ribose, Uracil
B Phosphate, Deoxyribose, Proline
C Phosphate, Ribose, Thymine
D Phosphate, Deoxyribose, Adenine
ANSWER:D
550
In the ribosome of a cell, the mRNA is read to produce the particular amino acid sequence
for the formation of a protein. What is this process called?
A Translation
B Transcription
C Transportation
D Transmutation
ANSWER:A
551 Which of the base pairings in DNA would be correct?
A A–T pair
B A–G pair
C C–T pair
D C–A pair

ANSWER:A
552 The combination of a sugar and a base is given the general term of:
A nucleoside
B amino acid
C polypeptide
D nucleotide
ANSWER:A
553
The nucleic acid which carries the information for protein synthesis from the cell nucleus to
the ribosomes is:
A DNA
B RNA
C mRNA
D tRNA
ANSWER:C
554
Which of the following is the correct combination of the components for the nucleic acid
RNA?
A Phosphate, Ribose, Uracil
B Phosphate, Deoxyribose, Proline
C Phosphate, Ribose, Thymine
D Phosphate, Deoxyribose, Adenine
ANSWER:A
555 In the nucleus of the cell DNA is used as a template to form mRNWhat is the process called?
A Translation
B Transcription
C Transportation
D Transmutation
ANSWER:B
556 Which statement is true of the chromosomes within a sperm?
A chromosomes is the diploid number
B
chromosomes came from the father, chromosomes came from the mother, while one of
either the Y or the X came from the father or mother respectively.
C Some of the came from the father and the rest came from the mother.

D chromosomes and the Y came from the father, while chromosomes came from the mother.
ANSWER:C
557 What can be correctly said of data that are “normally distributed”?
A
The upper and lower values of the distribution describe the healthy range of physiological
values.
B
The standard deviation characterises the dispersion of data and the variance characterises the
central tendency of the data.
C
The mean and range are statistics that are strictly only applicable to nor- mally distributed
data.
D Sixty eight percent of all data values will be within one standard deviation from the mean.
ANSWER:D
558
If someone’s height is measured while the person is wearing shoes, the height will be
overestimateThis type of error is known as which of the following?
A Absolute error
B Parallax error
C Calibration error
D Zeroing error
ANSWER:D
559 Which of the following metric prefixes is used to denote one thousandth of a gram?
A micro
B milli
C centi
D kilo
ANSWER:B
560 What is the standard deviation used for?
A as a measure of central tendency
B as a measure of dispersion
C as a measure of spread of data that are normally distributed
D as a measure of the error of the mean value
ANSWER:C
561 What information does the “standard deviation” of a mean value tell us?
A It gives us the healthy range of values for the measured physiological quantity.
B It is the range within which % of measured values are found.

C It tells us that the measured values are normally distributed.
D It tells us the number of values that were used to calculate the mean.
ANSWER:B
562
What does the standard deviation of the mean represent? For values that are normally
distributed, it represents:
A the value above and below the mean that includes % of all data values
B the difference between the highest data value and the lowest data value
C the average of the difference between each data value and the mean value.
D the spread of the normal distribution.
ANSWER:A
563 Which of the following statements applies to the statistic known as the “stan- dard deviation”?
A It is a measure of central tendency.
B It is only applicable to qualitative measurements.
C Standard deviation is also known as the “variance”.
D % of all data lie within two standard deviations of the mean.
ANSWER:D
564 Which of the following units is NOT part of the Australian metric system of units?
A mmHg for measuring blood pressure
B degree Celsius for measuring temperature
C Pascal for measuring pressure
D second for measuring time
ANSWER:A
565 In the Australian Metric System of units, what does the prefix micro stand for?
A one thousand
B one thousandth
C one million
D one millionth
ANSWER:D
566 Which of the following measurements is a semi-quantitative one?
A a blood pressure of 0/0 mmHg
B a blood glucose level of + + +
C a state of anxiety measure of “calm”

D the patient’s name is Tim Cruise
ANSWER:B
567
Systematic errors arise from some inadequacy of equipment or technique. Which of the
following is NOT an example of systematic error?
A parallax error
B calibration error
C random error
D zeroing error
ANSWER:C
568 The median is a measure of central tendency. It may be defined as:
A the value that has half the values greater than it and half less than it
B the value that occurs most often
C the distribution of values that has the mode, mean and average equal to each other
D the sum of all values divided by the number of values.
ANSWER:A
569 Which of the following statements involves a nominal measurement?
A James has a height of 0 cm.
B Barry’s blood pressure is elevated.
C Gino was born in Italy.
D More than % of Australians receive a pension.
ANSWER:C
570 In the Australian Metric System of measurement what does the prefix “milli” stand for?
A one thousandth
B one thousand
C one millionth
D one million
ANSWER:A
571
The millimetre of mercury is a unit commonly used for the measurement of blood pressure.
Which of the following statements about this unit is true?
A It is part of the Australian Metric System but not part of the SI system.
B It is part of the SI system but not part of the Australian Metric System.
C It belongs to both the SI system and the Australian Metric System.
D It does not belong to either the SI system or the Australian Metric System.

ANSWER:D
572
The chemical formula CHO contains much information. However, what information is NOT
provided by the formula?
A the number of atoms in a molecule
B the name of the substance
C the elements that make up the substance
D whether the substance is covalent or ionic
ANSWER:B
573 Which of the particles listed below is the smallest?
A an atom
B a proton
C an ion
D a molecule
ANSWER:B
574 The chemical name for sodium is which of the following?
A Na
B So
C K
D Si
ANSWER:A
575 A molecular compound may be defined by which of the following?
A atoms from non-metal elements covalently bonded.
B atoms from metal elements covalently bonded
C atoms from metal elements and non-metal elements covalently bonded
D atoms from non-metal elements ironically bonded
ANSWER:A
576
Many drugs are neutralised to form salts and administered in this form. What is the main
advantage of administering the salt form of the drug? It is usually:
A less toxic
B more soluble in water
C more pleasant to taste
D more soluble in lipid
ANSWER:B

577 One of the following is INCORRECT. Which one?
A metal atoms will form compounds with non-metal atoms.
B metal atoms will form compounds with metal atoms.
C non-metal atoms will form compounds with non-metal atoms.
D metal atoms will not form compounds with metal atoms.
ANSWER:B
578 Choose the correct statement about hydrogen bonds. They
A are stronger than covalent bonds.
B act between the H in one –OH or –NH group, and the O or N in another.
C operate within molecules.
D act between the H in one –OH or –NH group, and the H in another.
ANSWER:B
579
Choose the ending that will correctly complete the sentence: When atoms of a metal element
and atoms of a non-metal element react, the result is
A a covalent compound consisting of molecules
B a covalent compound consisting of ions
C an ionic compound consisting of ions in a lattice
D an ionic compound consisting of molecules in a lattice
ANSWER:C
580 In a water molecule, the bond between oxygen (O) and hydrogen (H) is
A a covalent bond and a polar bond
B an ionic bond
C a covalent bond
D an ionic bond and forms an electrolyte
ANSWER:A
581
The chemical elements can be divided into metal elements and non-metal ele-ments. Which
of the statements about metals and non-metals is correct?
A metals lose electrons to become charged particles called cations
B most of the elements are non-metals.
C non-metals are located at the left hand side of the periodic table
D metals have low melting points and are good conductors of heat
ANSWER:A

582
of the chemical elements are essential to the human body. Four bulk ele- ments, are
macrominerals and are trace elements. Which are the four bulk elements?
A calcium, carbon, hydrogen & oxygen
B nitrogen, carbon, hydrogen & oxygen
C calcium, nitrogen, carbon & oxygen
D carbon, oxygen, phosphorus & iron
ANSWER:B
583 Most non-molecular compounds form by the chemical combination of:
A molecules with molecules
B non-metals with non-metals
C metals with metals
D metals with non-metals
ANSWER:D
584
Molecules are relatively easy to separate from one another. This means that the bonds
between them are:
A ionic
B covalent
C relatively weak
D relatively strong
ANSWER:C
585 What happens when a sodium atom reacts to form a compound? The atom will
A gain one electron
B lose one electron
C gain two electrons
D lose two electrons
ANSWER:B
586 Which of the following type of bond between atoms is the weakest?
A ionic bond
B polar bond
C covalent bond
D hydrogen bond
ANSWER:D

587 Ionic, non-molecular compounds are likely to have which of the following sets of properties?
A high melting point, often soluble in organic liquids, in pure form do not conduct electricity.
B low melting point, no strong odour, soluble in water, electrical conductivity in solution.
C high melting point, electrical conductivity in solution, no strong odour, often soluble in water
D
low melting point, strong odour, soluble in organic liquids, in pure form do not conduct
electricity.
ANSWER:C
588 Which of the following best describes a molecule?
A The simplest structure in an ionic compound.
B One thousandth of a mole.
C The particles of which covalent compounds are composed.
D The smallest particle of an element.
ANSWER:C
589 Which of the following is true of metal elements?
A The four most common metal elements in the body have the symbols: C, H, O and N.
B They form compounds with non-metals.
C When they form compounds, they gain electrons.
D In the body, ions of metal atoms have a negative charge.
ANSWER:B
590 What is the name given to the particles that make up a covalent compound?
A ions
B molecules
C anions and cations
D metal atoms
ANSWER:B
591 Compounds may be described by which of the following sentences?
A Pure substances that contain two or more elements.
B Materials that are composed of particles called molecules.
C Materials composed of more than one part, and the parts may be present in any proportion.

D Substances that appear on the right hand side of the periodic table.
ANSWER:A
592 “Covalent” is the term applied to which of the following bonds?
A Those between an ion and all the surrounding oppositely charged ions.
B The bond between an electrolyte and the surrounding water molecules in a solution.
C Those between non-metal atoms.
D Those between atoms on the left hand side of the periodic table.
ANSWER:C
593 Which of the following is a definition of a molecule? The particle that composes:
A covalent compounds
B non-metal elements
C electrolytes
D ionic compounds
ANSWER:A
594 What is the smallest particle of a non-metal element known as?
A a molecule
B an atom
C an ion
D a neutron
ANSWER:B
595 What is the difference between ions and molecules?
A ions have an electrical charge whereas molecules do not.
B ions are from metal elements only whereas molecules contain only non- metal elements
C
ions arise from compounds between non-metal elements, whereas mole- cules arise from
metal and non-metal elements
D an ion may be formed from a single atom but molecules always involve more than one atom
ANSWER:D
596 Of the four different types of matter listed below which is not an example of an element?
A hydrogen
B oxygen
C water
D gold

ANSWER:C
597 Which of the following statements about atoms is FALSE?
A They are mostly empty space.
B Nearly all their mass is concentrated in the nucleus.
C In a neutral atom protons and electrons are equal in number.
D The nucleus contains equal numbers of protons and neutrons.
ANSWER:D
598
In which of the following sequences are particles listed in order of increasing size from left
to right?
A electron, atom, proton, molecule
B molecule, atom, proton, electron
C atom, proton, electron, molecule
D electron, proton, atom, molecule
ANSWER:D
599 Which of the following is the name of a subatomic particle?
A anion
B cation
C molecule
D neutron
ANSWER:D
600
Two atoms have the same mass number but different atomic numbers. Which of the
following statements concerning these atoms is TRUE?
A Each has the same number of neutrons in its nucleus.
B They are isotopes.
C They are atoms of different elements.
D Each has the same number of protons in its nucleus.
ANSWER:C
601 Which of the following statements concerning isotopes is FALSE?
A They contain the same number of protons in their atoms.
B They contain the same number of electrons in their atoms.
C They contain the same number of neutrons in their atoms.
D They have very similar chemical properties.
ANSWER:C

602
Which of the following properties is least likely to be possessed by a covalent,molecular
substance?
A strong odour
B high solubility in water
C melting point above 00 °C
D low electrical conductivity
ANSWER:C
603 Which item from the following list is an electrolyte?
A Ca++
B oxygen gas (O) dissolved in water
C table salt (solid form of Na+Cl-)
D glucose (solid form of the sugar)
ANSWER:C
604 What does the term “electrolyte” refer to?
A an uncharged dissolved particle.
B the smallest particle of an element.
C a substance that will conduct electricity when dissolved in water.
D negatively charged sub-atomic particles.
ANSWER:C
605 Which of the following is NOT an electrolyte (or does not contain electrolytes)?
A K+
B nitrate ions
C haemoglobin
D a % solution of sodium chloride
ANSWER:C
606 Which is the best definition of an electrolyte?
A an atom that dissociates into ions.
B a substance that will conduct electricity when it is dissolved in water.
C molecules of solid, liquid or gas that will conduct electricity in solution.
D a substance that will conduct electricity.
ANSWER:B
607 What is the condition known as hyperkalemia characterised by? A
A higher than normal concentration of potassium in the blood.

B lower than normal concentration of potassium in the blood.
C serum sodium concentration greater than 0 mmol/l.
D serum sodium concentration greater than mmol/l.
ANSWER:A
608 A sudden and severe loss of potassium due to diuretic abuse is likely to result in:
A hypothermia
B hyponatremia
C hypokalemia
D hypoventilation
ANSWER:C
609
Which of the following statements relating to a patient with severe loss of potassium due to
diuretic abuse is TRUE?
A the serum levels of potassium are > mmol/L
B an ECG is probably not warranted
C the condition may be treated by administering oral glucose and potassium
D
one course of action is to decrease the intake of potassium and to undergo ion-exchange
resin treatment
ANSWER:C
610 Which strategy would be most effective in dealing with a severe case of dehydration?
A Oral administration of a hypertonic solution.
B Intravenous administration of distilled water.
C Intravenous administration of isotonic glucose.
D Intravenous administration of hypotonic sodium chloride.
ANSWER:C
611 The “extra-cellular” fluid compartment of the body includes which of the following?
A interstitial, trans-cellular and connective tissue fluids
B vascular, connective tissue and interstitial fluids
C intra-cellular and transcellular fluid
D vascular and connective tissue fluid
ANSWER:B
612 Fluid and electrolyte balance in the body is maintained by which of the following?
A the hormone aldosterone
B keeping accurate account of the patient’s fluid balance chart

C the nephron of the kidney
D the hypothalamus of the brain
ANSWER:C
613 Which general description of the components of a solution is correct?
A solvent and solute
B solvent and liquid
C solute and solder
D liquid and solid
ANSWER:A
614
Given that the healthy range of sodium ion concentration in the blood is – mmol/L, if the
measured concentration of a blood sample was 0 mmol/L, what would the condition be
called?
A hyperkalemia
B hypokalemia
C hypernatremia
D hyponatremia
ANSWER:D
615 Extracellular fluid includes which of the following liquids?
A blood plasma
B blood plasma and interstitial fluid
C blood plasma and interstitial fluid and connective tissue fluid
D blood plasma and interstitial fluid and connective tissue fluid and liquid inside cells
ANSWER:C
616 What can be said about a solution that conducts electricity?
A the solute is a polar molecule
B the solvent is a non-polar liquid
C the solution contains dissolved ions
D the solution contains dissolved molecules
ANSWER:C
617 Which of the body’s fluid compartments does the vascular compartment form part of?
A interstitial fluid
B extracellular fluid
C intracellular fluid

D transcellular fluid
ANSWER:B
618 To what condition does the term “hypokalemia” refer?
A too little phosphorus in the blood
B too much sodium in the blood
C too little potassium in the blood
D too little sodium in the blood
ANSWER:C
619 Which of the following is correct for intra-cellular fluid (ICF) and extra-cellular fluid (ECF)?
A the ECF is part of the ICF
B the majority of the body’s water is in the ECF
C the ICF contains more sodium than the ECF
D the ICF contains more potassium than the ECF
ANSWER:D
620
In the context of fluid (water) balance, the body is said to have “two compart- ments”. What
are they?
A the vascular and the interstitial compartments
B the intracellular and the interstitial compartments
C the lymph and the vascular compartments
D the extracellular and the intracellular compartments
ANSWER:D
621 What is an insufficient concentration of potassium in the blood known as?
A hypokalemia
B hyponatremia
C hypopotassemia
D hypocalcemia
ANSWER:A
622 If a solution is shown to be able to conduct electricity, then what is true?
A A: the solution is free of impurities
B B: the solution contains a dissolved electrolyte
C C: the solution contains dissolved molecules
D D: the solution is an aqueous solution

ANSWER:B
623 In the vascular compartment of the body, what is the solvent?
A blood
B plasma
C serum
D water
ANSWER:D
624 The extracellular fluid compartment consists of which of the following?
A vascular and transcellular
B interstitial, vascular and connective tissue fluid
C intra-cellular and transcellular
D trans-cellular, intra-cellular and connective tissue fluid
ANSWER:B
625 Electrolyte balance is achieved largely by:
A the kidneys and aldosterone
B drinking sufficient water
C anti-diuretic hormone and isotonic fluids
D ensuring that daily water intake is the same as daily water output
ANSWER:A
626 What may hyponatremia be described as?
A insufficient potassium in the blood
B insufficient iron in the blood
C insufficient sodium in the blood
D excess sodium in the blood
ANSWER:C
627
Which of the following ways of expressing a solution’s concentration is written as a number
without units?
A % concentration
B Molarity
C Osmotic pressure
D Specific gravity
ANSWER:D

628
Consider two aqueous solutions of different concentration separated by a semi- permeable
membrane. In this situation, osmosis results in:
A water molecules moving to the side where the solution concentration is lower.
B the more concentrated solution becoming even more concentrated.
C the more dilute solution becoming even more dilute.
D the more concentrated solution becoming more dilute.
ANSWER:D
629 Osmosis may be defined as which of the following?
A
The diffusion of water molecules across a semi-permeable membrane from the solution with
higher water concentration into the solution of lower water concentration.
B
The movement of water molecules across a semi-permeable membrane from the solution of
higher concentration into the solution of lower concentration.
C
The diffusion of solute particles across a semi-permeable membrane from the solution of
higher concentration into the solution of lower concentration.
D
The movement of water molecules across a semi-permeable membrane from the solution of
lower concentration into the solution of higher concentration.
ANSWER:D
630
What is the movement of water molecules across a plasma membrane from the side where the
solution concentration is more dilute to the side where the solu- tion is more concentrated
called?
A osmosis
B reverse osmosis
C diffusion
D hydration
ANSWER:A
631
The movement of water molecules through a plasma membrane from the side where there is
a higher concentration water molecules to the side where there are fewer, is best known as:
A diffusion
B osmosis
C pinocytosis
D hydrolysis
ANSWER:B

632
The diffusion of water molecules across a cell membrane from the side where the solution
concentration is more dilute to the side where it is greater, is known as which of the
following?
A osmosis
B filtration
C hydrolysis
D buffer action
ANSWER:A
633
If a semi-permeable membrane separates two aqueous solutions with different osmotic
pressures, what will be the direction of water flow between solutions? From:
A higher osmotic pressure to the solution of lower osmotic pressure.
B lower osmotic pressure to the solution of higher osmotic pressure.
C higher concentration to the solution of lower concentration.
D higher hydrostatic pressure to the solution of lower hydrostatic pressure.
ANSWER:B
634 During dialysis, what moves across a semi-permeable membrane (and how)?
A
Water molecules by diffusion from the region of high solute concentration to the region of
low solute concentration.
B
Water molecules by filtration from the region of high hydrostatic pressure to the region of
low hydrostatic pressure.
C
Solutes by diffusion from the region of high solute concentration to the region of low solute
concentration.
D
Solutes by filtration from the region of low hydrostatic pressure to the region of high
hydrostatic pressure.
ANSWER:C
635 A suitable definition of osmosis would be movement:
A
of solute particles through a plasma membrane from the side where their concentration is
greatest to the side where it is lower.
B
of water molecules through a plasma membrane from the side where their concentration is
greatest to the side where it is lower.
C
of a substance from a region where it is in high concentration to where its concentration is
lower
D caused by a hydrostatic pressure difference.

ANSWER:B
636 The difference between dialysis and diffusion is that
A dialysis involves the movement of water molecules.
B diffusion involves movement against the concentration gradient.
C dialysis involves passive movement through a cell membrane.
D diffusion is caused by a hydrostatic pressure difference.
ANSWER:C
637 Osmosis involves the movement of:
A
water molecules through a membrane from a region of higher concentration of water
molecules to a region of lower water molecule concentration.
B
solute particles from a region of higher solution concentration to a region of lower solution
concentration.
C
water molecules from a region of lower concentration of water to a region of higher water
molecule concentration.
D
solute particles through a membrane from a region of lower solute concen- tration to a
region of higher solute concentration.
ANSWER:A
638 What does “osmosis” refer to?
A the constant random motion of ions and molecules.
B
the movement of ions and molecules from regions of high concentration to regions of low
concentration.
C the movement of water molecules through a semi-permeable membrane.
D
the movement of water molecules through a semi-permeable membrane from the side with
higher water concentration to the side with lower water concentration.
ANSWER:D
639 What is the difference between filtration and diffusion?
A Diffusion can occur through a biological membrane whereas filtration cannot.
B Filtration can occur through a biological membrane whereas diffusion cannot.
C
Filtration is the movement of molecules caused by a pressure difference but diffusion does
not involve a difference in pressure.
D
Diffusion is the movement of molecules caused by a pressure difference but filtration does
not involve a difference in pressure.
ANSWER:C

640 What is the difference between osmosis and dialysis?
A
Dialysis involves the movement of solute molecules whereas osmosis refers to water
molecules.
B
Osmosis involves the movement of solute molecules whereas dialysis refers to water
molecules.
C
Osmosis involves movement of molecules across a membrane but dialysis does not involve a
membrane.
D
Dialysis involves movement of molecules across a membrane but osmosis does not involve a
membrane.
ANSWER:A
641
Which one of the following processes that describe movement of the particles in a solution
does NOT require passing through a membrane?
A diffusion
B filtration
C dialysis
D osmosis
ANSWER:A
642
Which statement about the osmotic pressure of an aqueous solution is correct? Osmotic
pressure:
A is an indication of the force with which pure water moves into that solution.
B is a measure of the tendency of water to move into the solution.
C is the drawing power of water and depends on the number of molecules in the solution.
D of a solution is called its osmolarity in mosmol/kg.
ANSWER:B
643 Diffusion is the term given to the process where:
A
molecules move along their concentration gradient from high concentration to low
concentration.
B water moves along its concentration gradient from low concentration to high concentration.
C ATP is used to move ions along their concentration gradient.
D
a membrane protein, by changing shape after binding to a molecule, moves the molecule
across the plasma membrane.
ANSWER:A

644
By what name is the movement of solute particles through a selectively perme- able
membrane, in the direction of their concentration gradient known?
A diffusion
B dialysis
C osmosis
D filtration
ANSWER:B
645
Blood has a slightly higher osmotic pressure than the interstitial fluid that sur- rounds
capillaries. What is the effect of this?
A Water will tend to move from the interstitial fluid into the capillaries.
B The solution concentration of blood is less than the solution concentration of interstitial fluid.
C Water will tend to move from the capillaries into the interstitial fluid.
D Capillaries will expand in diameter.
ANSWER:A
646
Consider a patient undergoing kidney dialysis, whose blood has bicarbonate at a
concentration of mmol/L and urea at mmol/L. The dialysing liquid has bicarbonate at
mmol/L and urea at 0 mmol/L. In which direction will these substances flow?
A
bicarbonate will flow from patient’s blood to dialysing liquid, urea will flow from patient’s
blood into dialysing liquid.
B
bicarbonate will flow from dialysing liquid to patient’s blood, urea will flow from dialysing
liquid into patient’s blood.
C
bicarbonate will flow from dialysing liquid to patient’s blood, urea will flow from patient’s
blood into dialysing liquid.
D
bicarbonate will flow from patient’s blood to dialysing liquid, urea will flow from dialysing
liquid to patient’s blood.
ANSWER:C
647 Which is the best description for the osmotic pressure of a solution?
A
A: the pressure that needs to be applied to the solution while it is separated from pure water
by a membrane, to prevent a net flow of water through the membrane into the solution.
B
B: the force with which pure water moves through a membrane into that solution as a result
of its solute concentration.

C
C: the movement of particles through a membrane, where the movement is caused by a
hydrostatic pressure.
D D: it is the force of attraction for water by undissolved particles in the solution.
ANSWER:A
648 In which of the following situations would the osmotic pressure of blood be the greatest?
A in a patient whose blood osmolarity is 0 mosmol/L
B in a patient whose blood osmolarity is 0 mosmol/L and whose urine spe- cific gravity is .00
C in a patient with hyperthermia
D in a patient who is dehydrated
ANSWER:D
649 Osmosis involves movement of water from where the:
A water concentration is lower to where it is higher
B solute concentration is higher to where it is lower
C solution is more concentrated to where it is less concentrated
D water concentration is higher to where it is lower
ANSWER:D
650 What is “osmotic pressure”?
A the pressure exerted by a solution due to its concentration
B a measure of solution concentration expressed in the units of pressure
C the pressure exerted by the blood colloidal plasma proteins
D the pressure that drives water movement out of the arterial end of capillaries
ANSWER:B
651
If a red blood cell (rbc) is placed in a solution that has a greater concentration than that
inside the rbc, what will happen?
A the rbc will crenate
B the rbc will haemolyse
C there will be a net movement of water out of the rbc into the solution
D there will be no net movement of water out of the rbc
ANSWER:C
652 One of the following is an example of osmosis. Which one?
A water moving from the glomerulus of a nephron into the Bowman’s capsule
B water leaving a blood capillary from close to its arteriole end, to enter the interstitial fluid

C
water entering a red blood cell that is in a 0.% sodium chloride solution, by passing through
its plasma membrane
D water evaporating from perspiration on the skin
ANSWER:C
653
The Na+/K+ ATPase pump in the plasma membrane moves Na out of the cell and K into the
cell against their concentration gradient. Then Na reenters the cell and K leaks out of the cell,
along their concentration gradients through their membrane channels. What is the movement
of Na and K along their concentra- tion gradients called?
A Active transport
B Diffusion
C Facilitated diffusion
D Osmosis
ANSWER:B
654 Which of the following statements is FALSE?
A
Filtration is movement of water caused by a difference in hydrostatic pres- sure, while
diffusion results from a difference in concentration.
B
Both diffusion and filtration will tend to continue till there is an equal amount on both sides
of the membrane.
C
Any hypertonic solution has a concentration lower than that of blood while a hypotonic
solution has a concentration greater than the blood.
D
Water moves into a red blood cell resulting in haemolysis, and out of a cell by a process
called plasmolysis.
ANSWER:C
655 What information does the molarity of a solution provide?
A the density of a solution
B the number of dissolved particles per litre of solution
C the mass of a mole of the substance
D the tonicity of the solution
ANSWER:B
656
What term is applied to an intravenous solution that would cause a net move- ment of water
out of red blood cells?
A hypertonic
B supertonic

C epitonic
D hypotonic
ANSWER:A
657 What is a definition of an osmole? The amount of substance that:
A must be dissolved to produce × 0 solute particles.
B must be dissolved to produce an osmotic pressure of .0 mmHg.
C must be dissolved to produce an isotonic solution.
D contains × 0 particles.
ANSWER:A
658 What can be correctly said of an isotonic intravenous solution? An isotonic solution:
A causes water to move out of red blood cells.
B causes no net movement of water into or out of red blood cells.
C has the same solutes in the same solution concentration as blood plasma
D causes water to move into red blood cells.
ANSWER:B
659 What would be the concentration of a solution that causes red blood cells placed in it to swell?
A hypotonic
B isotonic
C hypertonic
D iso-osmotic
ANSWER:A
660 The unit milliosmoles per litre (mosmol/L) refers to which of the following?
A the number of particles in solution, in multiples of × 00 per litre.
B 0 times the number of moles of particles in a litre of solution.
C the number of molecules per litre of solution.
D the number of moles per millilitre of solution.
ANSWER:A
661 A hypertonic solution is one which
A has an osmotic pressure that is different to that inside red blood cells
B has an osmolarity less than that of red blood cells
C causes no net movement of water through the membrane of red blood cells
D has an osmolarity greater than that of red blood cells

ANSWER:D
662 The unit millimoles per litre (mmol/L) refers to which of the following?
A the number of particles in solution, in multiples of × 00 per litre.
B 0 times the number of moles of particles in a litre of solution.
C the number of molecules per litre of solution.
D the number of moles per millilitre of solution.
ANSWER:A
663 An isotonic solution is one which
A has an osmotic pressure that is different to red blood cells
B has an osmolarity less than that of red blood cells
C causes no net movement of water between the solution and red blood cells
D has an osmolarity greater than that of red blood cells
ANSWER:C
664 A hypotonic solution may be characterised by which of the following?
A a solution whose osmolarity is greater than that of blood.
B one that causes red blood cells to crenate.
C a solution within the range 0–00 mosmol/L
D one that causes a net water movement into red blood cells.
ANSWER:D
665 Which of the following statements could be applied to a hypertonic solution?
A it causes red blood cells to shrink and crenate.
B it causes red blood cells to swell and perhaps lyse.
C it is a solution with an osmolarity less than that of blood.
D it causes movement of water into red blood cells.
ANSWER:A
666 To what does the term “mole” refer?
A the smallest particle of a molecular compound.
B the amount of solute that must be dissolved in water to make an isotonic solution.
C an amount of substance that contains .0 × 0 particles.
D a group of or more atoms bonded together.
ANSWER:C
667
What do solutions of % glucose, . % sucrose and 0. % sodium chloride have in common?
They:

A all have the same concentration
B are all hypotonic to plasma
C all contain the same number of dissolved particles per unit volume.
D are all hypertonic to plasma
ANSWER:C
668
An intravenous fluid that is hypertonic to blood would have what effect on the red blood
cells?
A it would have no effect.
B the number of red blood cells would increase
C red blood cells would lyse.
D it would cause red blood cells to crenate.
ANSWER:D
669 An intravenous fluid that is isotonic to blood would have what effect on the red blood cells?
A it would have no effect.
B it would cause red blood cells to crenate.
C red blood cells would lyse.
D the blood volume would increase.
ANSWER:A
670 What may be said of isotonic solutions? They have:
A had added the same number of moles of solid substance per volume of solution.
B the same number of grams of solute per volume of solution.
C the same percent concentration.
D the same number of dissolved particles per volume of solution.
ANSWER:D
671 Which of the following statements about an “osmole” is correct? An osmole is the:
A same as a mole for ionic substances.
B formula weight of a substance expressed in grams.
C number of moles multiplied by the number of molecules in the chemical formula.
D amount of substance that must be dissolved in order to produce . × 0 dissolved particles.
ANSWER:D
672
Red blood cells are added to a hypotonic solution of glucose. Which of the fol- lowing best
describes what you would observe?

A The cells would most likely sink to the bottom unaffected
B The cells would shrink due to water loss.
C The cells would coagulate.
D The cells would swell and burst due to intake of water
ANSWER:D
673 A solution that is said to be isotonic to blood has the same:
A percent concentration as blood.
B number of moles of dissolved particles as blood.
C number of osmoles per litre of dissolved particles as blood.
D number of dissolved particles as blood.
ANSWER:C
674
If the following amounts of the given substances were dissolved in water, which would result
in four osmoles of dissolved particles?
A moles of CHO
B moles of Na+Cl−
C moles of CHO
D moles of (Na+)SO
ANSWER:B
675 What is the difference between “molarity” and “osmolarity”?
A
molarity applies only to covalent compounds while osmolarity applies only to ionic
compounds.
B osmolarity is molarity multiplied by two.
C
the molarity and osmolarity of a solution is the same for dissolved ionic compounds, but are
different for dissolved covalent compounds.
D
osmolarity refers to the concentration of dissolved particles in a solution which may not be
the same as the number of moles of substance that was dissolved per litre of solution.
ANSWER:D
676
Human blood has an osmolarity that lies within the range 0– 00 mosmol/L. Which of the
following statements is correct?
A an isotonic solution has osmolarity that is either less than 0 or greater than 00 mosmol/L
B a hypotonic solution has osmolarity between 0 and 00 mosmol/L
C a hypertonic solution has osmolarity between 0 and 00 mosmol/L
D an isotonic solution has osmolarity between 0 and 00 mosmol/L

ANSWER:D
677 What distinguishes an osmole from a mole?
A In a sample of substance, the number of osmoles is twice the number of moles
B
Both mole and osmole may be used in reference to ionic compounds, while covalent
compounds are described by mole alone.
C
the mass of a mole is the sum of the relative atomic masses (RAM) of the atoms in the
formula stated as grams. An osmole is half of this mass.
D
an osmole applies only to covalent molecular substances, whereas a mole is validly applied to
both covalent and ionic substances.
ANSWER:B
678 A solution that is isotonic to blood plasma is one which
A contains 0. % glucose
B must contain the same solutes as blood and in the same concentration as in blood
C has an osmolarity between 0 and 00 mosmol/L
D causes no net movement of water into or out of cells
ANSWER:D
679
Which of the following quantities of substance would, when dissolved in one litre of water,
produce a solution with the highest osmolarity?
A mole of glucose molecules (CHO)
B mole of Na+Cl− (sodium chloride)
C mole of haemoglobin molecules
D osmole of K+ (potassium ions)
ANSWER:B
680
Given the concentration of Na+ inside the cell is about 0 mmol/L while it is 0 mmol/L in the
extracellular fluiThe values for K+ are 0 mmol/L inside the cell and mmol/L extracellularly.
In what directions would these ions diffuse?
A Na+ would diffuse into the cell while K+ would diffuse out.
B K+ would diffuse into the cell while Na + would diffuse out.
C Both Na+ and K+ would diffuse out of the cell.
D Both Na+ and K+ would diffuse into of the cell.
ANSWER:A
681
Which of the following statements best describes what all isotonic solutions have in
common? They:

A contain the same concentration of glucose molecules
B contain the same total concentration of particles
C contain the same concentration of sodium chloride
D have the same specific gravity
ANSWER:B
682 Which of the following statements is CORRECT?
A An acid is an electron donor and a base is an electron acceptor.
B An acidic solution has a pH less than and a basic solution has a pH greater than .
C Neutralization of a strong acid by a strong base gives only water.
D The pH of saliva is normally in the range of .–.
ANSWER:B
683 Which statement about buffers below is most correct?
A A buffer is any acid and base which together control the concentration of pH in the blood.
B
A buffer is the solution which allows CO to be lost from the lungs in order to control the pH
of the lungs.
C
A buffer is a weak base and its acid salt or a weak acid and its basic salt. The ratio of the two
components helps maintain blood pH levels.
D A buffer is a mixture of two acids which together help to maintain blood pH
ANSWER:C
684 Which of the following statements is FALSE?
A An acid is a proton donor and a base is a proton acceptor.
B An acidic solution has a pH greater than and a basic solution has a pH less than .
C Neutralization of an acid by a base gives a solution of salt in water.
D The pH of the stomach is normally in the range of .–.
ANSWER:B
685 Which of the following statements is closest to a correct definition of an acid?
A a substance that ionises in a solution to produce hydronium and hydroxide ions.
B a substance that reacts with water to produce hydroxide ions.
C a substance that dissociates in water to produce a solution with pH greater than .0.
D a substance that reacts partly with water to produce a low concentration of hydronium ions.
ANSWER:D
686 The pH of a buffered solution depends on:

A the ratio of the components of the buffer solution.
B the amount of acid added to the buffer solution.
C the amount of base added to the buffer solution.
D the amount of acid and of base added to the solution.
ANSWER:A
687 The long term acid-base balance in the body is controlled by
A the phosphate and carbonic acid/bicarbonate buffers in the blood
B the kidneys and the lungs
C the phosphate, carbonic acid/bicarbonate, and protein buffers in the blood and cells.
D the kidneys
ANSWER:B
688
Acidity is stated as a pH value. If the pH of urine sample “A” is and the pH of urine sample
“B” is , then which of the following is true?
A the most acidic sample is sample B.
B sample A has 0 times the hydroxide ion concentration of sample B.
C the B sample has 0 times the hydrogen ion concentration of sample A.
D sample A has 0 times the hydrogen ion concentration of sample B.
ANSWER:D
689
One form of acid base imbalance in the body is called acidosis. In this situation, which of the
following is true?
A the blood is less alkaline than it should be.
B the blood’s pH is less than .0
C the blood is less acidic than it should be.
D the concentration of hydrogen ions in the blood is less than it should be.
ANSWER:A
690
One of the buffer systems in the blood is the carbonic acid/bicarbonate buffer. It helps to
maintain the body’s acid-base balance by destroying any excess:
A hydrogen ions in the blood.
B acid or base in the blood.
C hydroxide ions formed in the blood.
D bicarbonate ions formed in the blood.
ANSWER:B
691 The major buffer system in the extracellular compartment is the:

A the protein buffer
B carbonic acid/bicarbonate buffer
C ammonia buffer
D phosphate buffer
ANSWER:B
692 If a patient was suffering from “acidosis”, what would this mean?
A blood pH is not sufficiently alkaline.
B blood pH is acidic.
C there is too little hydronium ion in the plasma.
D blood pH is too acidic.
ANSWER:A
693 What happens when an acid is added to a buffered solution?
A the solution becomes acidic
B the pH of the solution decreases significantly
C the pH of the solution decreases very slightly
D the pH of the solution increases slightly
ANSWER:C
694 Which of the statements below is correct?
A
The dihydrogen phosphate component of the phosphate buffer releases hydrogen ions into
the lungs for excretion, and in the process, reverts to monohydrogen phosphate.
B Haemoglobin as it passes through the lungs, releases hydronium ions, which are breathed out.
C Carbon dioxide that is dissolved in blood diffuses into the alveoli and is breathed out.
D
Hydronium ions react with bicarbonate ions to form carbonic acid which moves into the
lungs for exhalation.
ANSWER:C
695 Fructose is a simple sugar or carbohydrate. What is it an example of?
A monosaccharide
B disaccharide
C polysaccharide
D oligosaccharide
ANSWER:A

696
Cells use glucose as an energy source, but store it as glycogen. When needed the glycogen is
broken down by a process called:
A glycolysis
B glycogenesis
C gluconeogenesis
D glycogenolysis
ANSWER:D
697
A lipid such as oleic acid contains a number of double bonds in the carbon chain. Because of
this, what term is applied to it?
A monounsaturated
B diunsaturated
C polyunsaturated
D saturated
ANSWER:C
698 What are the structural components of proteins?
A Amino acids
B Fatty acids
C Peptides
D Monosaccharides
ANSWER:A
699
Proteins perform a wide range of functions in the body including which one of the
following?
A They are our major source of energy
B They act as enzymes
C They are used to make sex hormones
D They are necessary for the absorption of vitamins A, D, E and K
ANSWER:B
700 Which of the following refers to a carbohydrate?
A diglycerol
B adenosine diphosphate
C disaccharide
D dipeptide
ANSWER:C

701 What are the bonds that maintain the secondary and tertiary structure of proteins?
A hydrogen bonds
B covalent bonds
C ionic bonds
D peptide bonds
ANSWER:A
702 Which of the following formulae would be most likely to represent a lipid?
A CHNOS
B CH0O
C CHO
D Ca0(PO)(OH)
ANSWER:B
703 What are the four most common elements found in proteins?
A C, H, N, Ca
B C, O, N, Fe
C C, H, N, O
D N, C, H, Na
ANSWER:C
704 To what type of fatty acids is the term “saturated” applied?
A Those with four single bonds around each carbon atom.
B Those with at least one double bond between carbon atoms.
C Those that are not implicated in coronary heart disease.
D Those which are essential in our diet.
ANSWER:A
705 Which of the following is a polysaccharide?
A glucuronidase
B glucagon
C glucose
D glycogen
ANSWER:D
706 What holds the primary structure of a protein together?
A hydrogen bonds
B covalent bonds

C peptide bonds
D ionic bonds
ANSWER:C
707 Which of the following are NOT proteins?
A glycolipids
B enzymes
C haemoglobin
D albumin
ANSWER:A
708 What is a “saturated fat”?
A one that contains cholesterol
B a triglyceride that has three fatty acids
C one where the carbon atoms that are connected by single bonds
D one that must be included in our diet
ANSWER:C
709 What is a function of carbohydrates in the body?
A to act as enzymes
B to provide energy
C function as local hormones
D to provide the building blocks for proteins
ANSWER:B
710 In what form is most of the lipid component of our food in?
A triglycerols
B polysaccharides
C complex carbohydrates
D polypeptides
ANSWER:A
711 In organic compounds, How many bonds do carbon atoms always form?
A four single bonds
B four covalent bonds
C two double bonds
D four ionic bonds
ANSWER:B

712
A molecule of an organic compound can be thought of as having two portions: a radical
with an attached functional group. What is the correct definition of a functional group?
A it consists of carbon atoms and hydrogen atoms
B it is responsible for most of the properties of the molecule.
C it has a modest influence on the properties of the molecule.
D it consists of a ring of carbon atoms.
ANSWER:B
713 The one medicine may have many names. What is the “generic name” of a medicine?
A the name that describes the effect of the drug on the body.
B the systematic name of the chemical involved.
C the name given to the product by a manufacturer of the drug.
D the name proposed by the inventor and approved by a government agency.
ANSWER:D
714 When a protein is denatured, which aspect of its structure is affected the​​​ LEAST?
A primary structure
B secondary structure
C tertiary structure
D quaternary structure
ANSWER:A
715
The element carbon is said to be tetravalent. This means it always has the fol- lowing number
of covalent bonds in neutral compounds.
A one
B two
C three
D four
ANSWER:D
716 Which of the following structures represents a molecule belonging to the alco-hol family?
A CH – COOH
B CH – OH
C CH – O – CH
D CH – CO – CH
ANSWER:B
717 Which statement describing the chemical reactivity of organic molecules is FALSE?

A Reactivity depends on the nature of the functional group.
B Reactivity is independent of the number and nature of the carbon rings
C Reactivity is dependent on the nature and size of the radical group.
D Reactivity in the body may depend on the internal body factors such as temperature and pH.
ANSWER:B
718 Which of the following structures represents molecules belonging to the amide family?
A CH – SH
B CH – NH
C CH – CO – NH
D CH – OH
ANSWER:C
719 A hydrocarbon is a type of compound that contains:
A Only carbon atoms
B Carbon and Hydrogen atoms
C Carbon, Hydrogen and Oxygen atoms
D Water and carbon atoms
ANSWER:B
720
A hydrocarbon molecule which contains at least one double bond could be an example of
which type of compound:
A Alkane
B Alkene
C Alkyne
D Alcapone
ANSWER:B
721 Which of the following structures represents a molecule belonging to the ketone family?
A CH–CO–NH
B CH–NH
C CH – O – CH
D CH – CO – CH
ANSWER:D
722 Which statement below is correct?
A dendrites are produced by dendrocytes

B glycolipids are produced by lipocytes
C keratin is produced by keratinocytes
D melanin is produced by merkel cells
ANSWER:C
723 Where are sebaceous glands found?
A In the digestive system
B In the hypodermis
C In the dermis
D In the stratum corneum
ANSWER:C
724 When a medicine is delivered via a patch attached to the skin, it is said to be delivered:
A transdermally
B subcutaneously
C topically
D intramuscularly
ANSWER:A
725
Full thickness burns to more than 0 % of the skin surface is a life-threatening situation. Why
is this?
A The synthesis of vitamin D (calcitriol) is severely compromised.
B The loss of skin sensation prevents access to environmental information.
C The body is not able to prevent water loss from the burnt area.
D Soft internal tissues are abraded by the external environment.
ANSWER:C
726 Which is a notable feature of the stratum corneum layer of the integument? It:
A is where melanocytes and keratinocytes are rapidly dividing.
B is richly supplied with blood capillaries.
C consists of keratin filled cells with glycolipid in between cells.
D has protruding epidermal ridges that push the overlying epidermis into “fingerprints”.
ANSWER:C
727 When a pharmaceutical is administered hypodermically, it is
A wiped onto the skin.
B applied via a patch that adheres to the skin.
C injected into the dermis.

D injected into subcutaneous fat.
ANSWER:D
728 Which of the following is NOT a sensory receptor of the skin?
A a Meissner corpuscle
B an apocrine gland
C a root hair plexus
D a nociceptor
ANSWER:B
729 What is the protein that fills the outermost dead cells of the epidermis?
A granstein
B dermin
C melanin
D keratin
ANSWER:D
730 What is the name given to the most superficial layer of the integument?
A stratum corneum
B papillary dermal layer
C stratum lucidum
D superficial fascia
ANSWER:A
731 Which of the following statements is INCORRECT?
A sudiferous glands secrete sebum
B sebaceous glands secrete oil
C apocrine glands secrete sweat
D ceruminous glands secrete cerumen
ANSWER:A
732 To what in the skin does the term “nociceptors” refer?
A sensors that detect movement of hair follicles
B any lamellated sensory corpuscle
C the sensory receptor that is associated with the Merkel cells of the epidermis
D free nerve endings with large receptive fields that detect pain
ANSWER:D
733 Choose the incorrect statement below.

A Keratinocytes produce keratin
B Merkel cells are associated with a sensory nerve ending.
C Melanocytes produce melanin
D Dendrocytes produce dendrocidin.
ANSWER:D
734 Which layer of the integument contains rapidly dividing keratinocytes?
A stratum lucidum
B papillary dermal layer
C stratum germinativum
D reticular dermal layer
ANSWER:C
735 Which of the following situations could produce life threatening fluid loss and infection?
A Stomach ulcers
B Full thickness skin burn
C Severe muscle tear
D Displaced bone fracture
ANSWER:B
736 What do sudiferous glands do?
A Secrete sebum into a hair follicle
B Secrete sweat through a duct to the skin
C Secrete sweat through a duct to the skin or into a hair follicle
D Secrete cerumen through a duct to the skin or into a hair follicle
ANSWER:C
737
Which cell type produces a pigment that affords the skin some protection against ultraviolet
radiation?
A keratinocytes
B melanocytes
C dendrocytes
D merkel cells
ANSWER:B
738 Which layer of the integument is the most superficial layer?
A the hypodermis
B stratum granulosum

C stratum corneum
D reticular dermal layer
ANSWER:C
739
A drug that is applied to the skin and exerts its therapeutic effect systemically is said to be
administered:
A transdermally
B topically
C intradermally
D subcutaneously
ANSWER:A
740
Which is the most superficial layer of the integument that also has capillaries, lymphatics and
sensory neurons?
A reticular dermal layer
B papillary dermal layer
C stratum granulosum
D stratum lucidum
ANSWER:B
741 Which skin layer is the most superficial?
A stratum lucidum
B stratum corneum
C papillary dermal layer
D reticular dermal layer
ANSWER:B
742 Which glands secrete “oil” into a hair follicle?
A apocrine
B eccrine
C ceruminous
D sebaceous
ANSWER:D
743 What must be the case for a drug to be administered transdermally?
A It must be water soluble
B It must be lipid soluble
C It must be injected subcutaneously

D It must not irritate the skin’s chemo-receptors.
ANSWER:B
744
The skin participates in the production of vitamin D when which of the follow- ing occurs?
When
A calcium is present
B signalled to by the hormone PTH
C exposed to ultraviolet radiation
D the precursor molecule is produced by the liver
ANSWER:C
745 The integument consists of which of the following layers?
A epidermis and dermis
B epidermis, dermis and hypodermis
C
stratum germinativum, stratum spinosum, stratum granulosum, stratum lucidum and stratum
corneum
D stratum corneum, dermis and reticular dermal layer
ANSWER:A
746 One of the following is NOT a type of sweat glanWhich one?
A eccrine gland
B merocrine gland
C endocrine gland
D apocrine gland
ANSWER:C
747 Three functions of the skin are to:
A store fat, produce sweat, receive stimuli.
B synthesise vitamin D, excrete bile, and provide a barrier to the entry of bacteria.
C produce keratin, assist in the immune response, and produce lymphocytes.
D produce melanin, secrete sebum, minimise water loss.
ANSWER:D
748 Which of the following is NOT a cell?
A macrophage
B chondroblast
C lysosome
D melanocyte

ANSWER:C
749 What purpose is vitamin D (calcitriol) used for?
A required for several stages of haemostasis
B required for uptake of calcium from the gut
C required for erythropoiesis
D required for uptake of intrinsic factor
ANSWER:B
750 Which layer of the skin is the most superficial?
A epidermis
B dermis
C papillary dermal layer
D stratum germinativum
ANSWER:A
751 The skin begins the production of vitamin D in which of the following situa- tions? When
A exposed to ultraviolet radiation.
B signalled to by the hormone PTH
C calcium is present
D it adds an hydroxyl group to a cholesterol molecule.
ANSWER:A
752 What do the apocrine glands of the skin secrete?
A apocrin
B cerumin
C milk
D sweat
ANSWER:D
753 Another name for oil glands in the skin is:
A sebaceous glands
B eccrine glands
C merocrine glands
D apocrine glands
ANSWER:A
754 What are three functions of the skin? To:
A store fat, produce sebum, trap a layer of air beneath hair to insulate against heat loss.

B synthesise vitamin D, excrete bile, protect against abrasion.
C produce melanin, regulate body temperature, minimise water loss.
D produce keratin, assist in the immune response, produce dendrocytes.
ANSWER:C
755
Which of the following lists layers of the integument in the order from most superficial first,
to deep?
A epidermis, hypodermis, dermis
B epidermis, papillary dermal layer, reticular dermal layer.
C dermis, stratum germinativum, stratum corneum
D stratum corneum, stratum germinativum, epidermis.
ANSWER:B
756 What is another name for sweat glands?
A ceruminous glands
B sebaceous glands
C sudiferous glands
D apocrine glands
ANSWER:C
757 Which statement about vitamin D is INCORRECT?
A It is required for uptake of calcium from the gut
B It is made in the skin, liver and kidneys.
C It is an essential part of our diet.
D It is a hormone
ANSWER:C
758 If a drug is administered “transdermally”, which of the following applies?
A it is absorbed through the skin and acts locally.
B it is injected into the dermis
C it is absorbed through the skin and acts systemically.
D it is injected into the subcutaneous fat layer
ANSWER:C
759 The “stratum corneum” is part of the skin that:
A contains the youngest, rapidly-dividing cells.
B anchors the skin to muscle while allowing it to slide over muscle.
C contains collagen, elastin and reticular fibres.

D protects the body against heat, chemicals and bacteria.
ANSWER:D
760 A drug that is administered “transdermally” is one that:
A acts at (or close to) the skin area to which it is administered.
B is injected into the muscle
C is applied to the epidermis
D is inserted into the anus
ANSWER:C
761 The hypodermis is which of the following?
A the outer layer of the skin.
B the inner layer of the skin.
C the superficial fascia and not regarded as part of the skin.
D not vascularised, getting its nutrients by diffusion.
ANSWER:C
762 Sudiferous glands are also known as:
A eccrine or apocrine glands.
B sebaceous or oil glands.
C ceruminous or apocrine glands.
D mammary or eccrine glands.
ANSWER:A
763 Which is the outermost layer of the skin?
A dermis
B epidermis
C stratum lucidum
D reticular dermal layer
ANSWER:B
764 One of the following is NOT part of the integumentary system. Which one?
A the hypodermis
B sebaceous glands
C finger nails.
D the stratum corneum.
ANSWER:A

765
People with full thickness burns to more than 0 % of their body are in a life- threatening
situation. This is due to which of the following?
A The body’s inability to thermoregulate.
B The loss of the ability to produce vitamin D.
C The buildup of urea and uric acid which would otherwise have been excreted by the skin.
D The body’s inability to prevent water loss
ANSWER:D
766
Which one of the following cell types is responsible for forming the skin’s abil- ity to tan on
exposure to sunlight?
A melanocytes.
B keratinocytes.
C dendrocytes.
D lymphocytes.
ANSWER:A
767 Which list below contains functions that are NOT performed by the integumen- tary system?
A protection, secretion of sebum, role in immunity.
B body temperature regulation, excretion, synthesis of vitamin D.
C perception of stimuli, production of sweat, protection
D body temperature regulation, synthesis of vitamin E, social function.
ANSWER:D
768 Full thickness burns to more than 0 % of the body surface is life-threatening because of the:
A fluid loss and inability to produce vitamin D.
B loss of ability to thermoregulate and infection.
C inability to excrete lactic acid, urea and uric acid, loss of thermoregulation.
D fluid loss and loss of the barrier against infection
ANSWER:D
769 Which one of the following cell types is responsible for forming the skin’s stratum corneum?
A melanocytes.
B keratinocytes.
C dendrocytes.

D lymphocytes.
ANSWER:B
770 Which list of structures is NOT all part of the integumentary system?
A sebaceous glands, hair, nails, mammary glands.
B meissner’s corpuscles, hypodermis, eccrine sweat glands, oil glands.
C apocrine sweat glands, sebaceous glands, merkel disc, hair follicles
D melanocytes, keratinocytes, merkel cells, dendrocytes.
ANSWER:B
771 Which of the following is NOT a function of the skin?
A fat storage
B waterproofing the body
C production of vitamin D
D immunity
ANSWER:A
772 What is one difference between the dermis and the epidermis? The
A
epidermis is composed of fibrous connective tissue while the dermis is com- posed of
epithelial cells.
B dermis is the most exterior layer.
C dermis is not part of the skin while the epidermis is.
D dermis is vascularised while the epidermis is not.
ANSWER:D
773 What is the most common protein in the epidermis?
A elastin
B keratin
C melanin
D cholesterol
ANSWER:B
774 What is the fibrous protein in our skin that protects it from abrasion?
A melanin
B keratin
C sebum
D elastin
ANSWER:B

775 Which stratum of the epidermis protects the body against water loss and abrasion?
A stratum germinativum
B stratum basale
C stratum lucidum
D stratum corneum
ANSWER:D
776 On which part of the integument is most of the body’s normal flora located?
A dermis
B epidermis
C microdermis
D hypodermis
ANSWER:B
777 Which is the deepest layer of the integument?
A the epidermis
B the dermis
C the stratum corneum
D the papillary dermal layer
ANSWER:B
778 One of the following is NOT a gland found in the integument. Which one?
A sudiferous gland
B mammary gland
C pineal gland
D sebaceous gland
ANSWER:C
779 All but one of the following are sensory receptors in the skin. Which one is NOT?
A Reticular dermal layer
B Merkel discs
C Nociceptors
D Pacinian corpuscles
ANSWER:A
780 What is the secretion produced by sudiferous glands?
A sweat
B sebum

C cerumin
D merocrin
ANSWER:A
781 One of the following statements about the stratum corneum is correct. Which one?
A cells in this layer undergo cell division to replace the skin.
B it consists of dead cells.
C it contains collagen, elastin and reticular fibres.
D the layer has sensory receptors known as Merkel discs, Meissner’s and Pacinian corpuscles.
ANSWER:B
782
Choose one answer below that completes the sentence so that it makes a true statement:
Positive feedback:
A is the way the body maintains homeostasis
B is a response that opposes a stimulus
C is a response that maintains a dynamic state of equilibrium
D is a response that enhances a stimulus
ANSWER:D
783
The human body’s ability to maintain a relatively constant internal temperature is an
example of what?
A Respiratory heat loss
B Homeostasis
C Vasodilation and evaporative heat loss
D Positive feedback
ANSWER:B
784
Some of the body’s homeostatic responses rely on “negative feedback”. Which of the
following happens in negative feedback?
A
The body ignores changes in a physiological variable that are directed towards the set point
for that variable.
B
The body ignores changes in a physiological variable that are directed away from the set
point for that variable
C The body’s response acts to oppose the change in the physiological variable.
D The body’s response acts to enhance the change in the physiological variable
ANSWER:C

785
In homeostasis, what is it that produces the response that moves the physiologi- cal variable
back towards the middle of its healthy range?
A the effector
B the receptor
C the integrating centre
D the efferent pathway
ANSWER:A
786 The “afferent pathway” in the description of a feedback loop in homeostasis, refers to the:
A circulating blood.
B pathway from the integrating centre to the effector.
C outgoing signal.
D path taken by the signal produced by a stimulus.
ANSWER:D
787 Homeostasis usually returns the body to a healthy state after stressful stimuli by
A negative feedback
B positive feedback
C means of the immune system
D means of the nervous system
ANSWER:A
788 What does the word “homeostasis” refer to?
A the steps leading to repair of a blood vessel and the coagulation of blood
B the maintenance of internal body conditions within narrow limits
C the controlled response that opposes the influence that caused it.
D the production of blood cells in active bone marrow
ANSWER:B
789
How does homeostasis return the body to a healthy state after stressful stimuli? By producing
a
A responses that oppose the stress
B learned behaviour
C reflex action
D buffering mechanism
ANSWER:A
790 What causes the body to maintain a relatively constant internal environment?

A positive feedback
B homeostasis
C reflexes
D pH buffers
ANSWER:B
791
Homeostasis relies on feedback to achieve its aims. “Negative” feedback refers to which
situation below? The body’s response:
A travels from the effector to the integrating centre via the afferent pathway.
B opposes the stressful stimulus.
C is to decrease the set point.
D enhances the stressful stimulus.
ANSWER:B
792
The term “homeostasis” is described by which one of the following statements? “The body’s
ability to:
A respond to a stimulus or stress in such a way as to enhance the stress.
B maintain a relatively constant internal temperature.
C respond to a stimulus or stress in such a way as to reduce the stress.
D maintain a relatively constant internal environment.
ANSWER:D
793
Synthesis and release of most hormones is regulated by negative feedback. Negative
feedback means
A A rise in hormone levels affects the target organ which acts to inhibit further hormone release
B
A rise in hormone levels affects the target organ which acts to stimulate further hormone
release
C The effect of hormones on target cells does not control further release of hormone
D
Neural stimuli result in the release of oxytocin and antidiuretic hormone from the
hypothalamus
ANSWER:A
794 Which of the following is a correct definition of “positive feedback”?
A The process by which the body maintains homeostasis.
B A mechanism in which the body’s response to a stimulus, opposes the stimulus.
C A mechanism whereby the body responds to a stimulus by acting to enhance the stimulus.

D
The dynamic equilibrium maintained by an integrating centre which causes an effector to
respond to the stimulus received by the receptor.
ANSWER:C
795 Which of the following is a correct definition of “negative feedback”?
A The process by which the body maintains homeostasis.
B A mechanism in which the body’s response opposes the stimulus.
C A mechanism whereby the body responds to a stimulus by acting to enhance the stimulus.
D
The dynamic equilibrium maintained by an integrating centre which causes an effector to
respond to the stimulus received by the receptor.
ANSWER:B
796 Homeostasis refers to maintaining :
A a constant internal body environment through negative feedback.
B
body conditions relatively constant within a narrow range through negative and positive
feedback.
C adequate concentrations of respiratory gases.
D blood glucose level within the healthy range.
ANSWER:B
797 What does the term “homeostasis” refer to?
A the chemical processes that take place in the organelles of the cells.
B the body’s tendency to maintain a relatively constant internal environment.
C the body’s use of energy to produce chemical substances and parts for growth.
D any body response that opposes the stimulus that initiated the response.
ANSWER:B
798 The body returns to a healthy state after stressful stimuli thanks to which of the following?
A positive feedback
B metabolism
C anabolism
D negative feedback
ANSWER:D
799 Which of the following would be a negative feedback response by the body to hyperthermia?
A Shivering
B Sweating

C Vasoconstriction of blood vessels in the dermis
D An increase in metabolic rate
ANSWER:B
800
How would the hypothalamus respond if its osmoreceptors noticed an increase in plasma
osmotic pressure? It would send a message to:
A the posterior pituitary to release more ADH
B the posterior pituitary to release less ADH
C the adrenal cortex to release less aldosterone
D the atria to release more ANP
ANSWER:A
801
During the delivery of a baby the baby’s head is pushing against the cervix causing the
cervix wall to stretch. This stretching causes nerve impulses to be sent to the hypothalamus
which directs the posterior pituitary to release oxyto- cin in the blooOxytocin stimulates the
uterus to contract which pushes the baby’s head deeper into the cervix, stretching it further.
This situation is a description of which of the following?
A negative feedback
B positive feedback
C homeostasis
D an afferent pathway to an integrating centre
ANSWER:B
802 Which of the following statements about positive feedback and homeostasis is correct?
A
They are regulation mechanisms that control most fluctuations in the inter- nal environment
of the body.
B The response to the stimulus serves to exaggerate the feedback effect.
C
This type of feedback response only involves an effector not a specific stim- ulus receptor
site.
D This feedback mechanism involves adjustments at the organ level but not at the cellular level.
ANSWER:B
803 Which of the following is NOT a bone of the axial skeleton?
A deltoid
B ethmoid
C sphenoid

D hyoid
ANSWER:A
804 Which of the following is a function of the skeletal system?
A haemopoiesis
B haemostasis
C peristalsis
D glycogenolysis
ANSWER:A
805 In which of the following bone structures do osteocytes live?
A osteons
B canaliculi
C lacunae
D lamellae
ANSWER:C
806 Which bone is most superior?
A manubrium
B occipital bone
C cervical vertebra #
D patella
ANSWER:B
807 What is a “trochanter”?
A part of a femur
B a feature of the pelvis
C a projection that forms part of an articulation
D a groove in which lies a tendon
ANSWER:A
808 One of the functions of bones is to make red blood cells. What is this process known as?
A Haemolysis
B Haemopoiesis
C Haematuria
D Haemostasis
ANSWER:B
809 Where do osteocytes reside?

A In lamellae
B In endosteum
C In trabeculae
D In lacunae
ANSWER:D
810 Which of the following describes the movements known as pronation and supination?
A The flexing of the arm with respect to the forearm around the elbow.
B The swivelling of the foot to the medial and lateral directions.
C The twisting of the wrist while the elbow is held motionless.
D The rotation at the shoulder that causes the arm to describe a cone shape.
ANSWER:C
811 Which of the following is NOT a “long” bone?
A the humerus
B the tibia
C a carpal
D a metacarpal
ANSWER:C
812 Which one of the following is a bone that is embedded within a tendon?
A sphenoid
B hyoid
C ethmoid
D sesamoid
ANSWER:D
813 In which one of the following structures do osteocytes reside? In the
A haversian canals
B lacunae
C trabeculae
D endosteum
ANSWER:B
814 Which bone of the head has a synovial joint?
A The sphenoid
B The maxilla
C The mandible

D The hyoid
ANSWER:C
815 What are the bones of the fingers known as?
A short bones
B metacarpals
C carpals
D phalanges
ANSWER:D
816 Which of the following comprise seven bones?
A Cervical vertebrae
B Carpals
C Cranial bones
D Lumbar vertebrae
ANSWER:A
817 Which term below refers to a depression in a bone?
A tuberosity
B fossa
C tubercle
D condyle
ANSWER:B
818 What body part is able to perform pronation and supination?
A the forearm
B the foot
C the thigh
D the wrist
ANSWER:A
819 Where are blood vessels in compact bone found?
A in the canaliculi
B in the periosteum
C in the lacunae
D in the central canal
ANSWER:D
820 Which of the following is NOT a depression or cavity on a bone?

A tuberosity
B facet
C meatus
D sinus
ANSWER:A
821 One of the following lists contains only bones in the appendicular skeleton. Which one?
A patella, ethmoid, femur, coccyx, tibia
B clavicle, fibula, metatarsal, phalange, radius
C humerus, scapula, occipital, metacarpal, sternum
D ulna, radius, phalange, mandible, coxal
ANSWER:B
822 A synovial joint is also known as one of the following, which one?
A synarthrosis
B immovable joint
C slightly moveable joint
D freely moveable joint
ANSWER:D
823 What is contained within the medullary canal of a long bone?
A trabeculae
B lamellae
C marrow
D osteoblasts and osteoclasts
ANSWER:C
824 Where in the skeleton is the scapula located?
A in the axial skeleton
B in the appendicular skeleton
C in the carpal region
D in the shoulder girdle
ANSWER:D
825 Which of the following bone markings forms part of an articulation?
A the deltoid tuberosity of the humerus
B the lateral condyle of the femur
C the greater trochanter of the femur

D the greater tubercule of the humerus
ANSWER:B
826 Where is the epiphyseal plate of a long bone located?
A in the diaphysis
B between the diaphysis and the epiphysis
C in the epiphysis
D in the medullary canal
ANSWER:B
827 In which structure are osteoclasts and osteoblasts found?
A in the periosteum
B in the haversian canals
C in the lacunae of osteons
D in the trabeculae of osteons
ANSWER:A
828 Which of the following is a NOT a projection from a bone surface?
A trochanter
B tubercle
C trabeculum
D tuberosity
ANSWER:C
829 Which of the listed bones is superior to the rest?
A manubrium
B xiphoid process
C coccyx
D femur
ANSWER:A
830 Choose the correct sentence. Compact bone contains
A lamellae and osteocytes but no osteons.
B trabeculae, canaliculi and osteons.
C haversian systems and canaliculi but no osteons.
D osteons and lamellae but no trabeculae.
ANSWER:D
831 Which of the following bone markings is NOT a projection for muscle attachment?

A fossa
B tuberosity
C tubercle
D trochanter
ANSWER:A
832 Which of the list below is a cell that reabsorbs bone?
A osteon
B osteoblast
C osteocyte
D osteoclast
ANSWER:D
833 The formula for the inorganic salts in bone is
A NHCCOOH
B CHO
C Ca0(PO)OH
D CH(CH)CH = CH(CH)COOH
ANSWER:C
834 Freely moveable joints are also known as
A fibrous joints
B cartilaginous joints
C amphiarthroses
D synovial joints
ANSWER:D
835 Which of the following is a projection from a bone surface?
A fossa
B fissure
C foramen
D facet
ANSWER:D
836 Which of the listed bones is the most inferior?
A ethmoid
B sphenoid
C femoid

D hyoid
ANSWER:D
837 Choose the correct sentence. Cancellous bone contains
A lamellae and osteocytes but no trabeculae.
B trabeculae, canaliculi and osteons.
C haversian systems and canaliculi but no osteons.
D trabeculae and lamellae but no osteons.
ANSWER:D
838 Which of the following bones is part of the cranium?
A occipital
B mandible
C hyoid
D carpal
ANSWER:A
839 The appendicular skeleton includes all of the following EXCEPT one. Which one?
A the pectoral girdle
B the thoracic cage
C the phalanges
D the lower limbs
ANSWER:B
840 What is the name given to the central tunnel of an osteon that contains blood vessels?
A canaliculus
B endosteum
C haversian canal
D medullary canal
ANSWER:C
841 Which of the following is an example of a synovial joint? The joint between the:
A tibia and fibula
B sternum and rib number
C thoracic vertebrae and
D proximal ends of the radius and ulna
ANSWER:D
842 Which list contains the bones of the pelvic and pectoral girdles?

A coxal, scapulae, manubrium, ilium
B clavicles, cervical, coccyx, innominate
C clavicles, scapulae, coxal
D clavicles, scapulae, sacrum, coxal
ANSWER:C
843 Synovial joints have all of the following features EXCEPT one. Which one?
A are surrounded by an articular capsule.
B have synovial fluid filling the space between articulating bones.
C have synovial membrane covering the articulating bone surfaces.
D are supported by reinforcing ligaments.
ANSWER:C
844 The manubrium and the xiphoid process are located on which part of the skeleton?
A the lower jaw
B the sternum
C the pelvis
D the hand
ANSWER:B
845 Carpals refers to
A the points of attachment of ribs to vertebrae
B bones of the wrist
C bones that are embedded within a tendon
D the thumbs
ANSWER:B
846 Haemopoiesis refers to
A blood cell formation in bone marrow
B the process of blood clotting
C the crenation of red blood cells in a hypotonic solution
D an excessively large proportion of red blood cells to plasma.
ANSWER:A
847 Articulating bones are joined by
A aponeuroses
B tendons
C fasciculi

D ligaments
ANSWER:D
848 On which bone is the greater trochanter found?
A pelvic
B femur
C radius
D humerus
ANSWER:B
849 What does “articulation” refer to?
A the joining of a ligament to a bone.
B the contact made between a tendon and a bone
C the contact between two bones.
D the connection between a muscle and a bone
ANSWER:C
850 To which bones does the word phalanges apply? Those in the
A fingers and toes
B wrist and ankle
C ankle and foot
D fingers and hand
ANSWER:A
851 The axial skeleton groups together which sets of bones?
A the arms and hands, the legs and feet, shoulder girdle and pelvic girdle.
B the head, shoulder girdle, arms and hands.
C
the thoracic cage, vertebral column, shoulder girdle, the pelvic girdle, the skull and facial
bones.
D bones of the skull and face, thoracic cage and vertebral column.
ANSWER:D
852 How do synovial joints differ from the other types of bone articulation?
A they have a joint cavity.
B the bones are joined by fibrous tissue.
C the articulating bones are joined by cartilage.
D the articulating bone surfaces are covered by tendons.
ANSWER:A

853 What are the cells that are found in the lacunae of compact bone called?
A osteocytes.
B osteons.
C osteoblasts.
D osteoclasts.
ANSWER:A
854 The appendicular skeleton groups together which sets of bones?
A the arms and hands, the legs and feet, shoulder girdle and pelvic girdle.
B the head, shoulder girdle, arms and hands.
C
the thoracic cage, vertebral column, shoulder girdle, the pelvic girdle, the skull and facial
bones.
D bones of the skull and face, thoracic cage and vertebral column.
ANSWER:A
855 Synovial joints differ from the other types of joint between bones in the body because:
A they are immovable joints.
B they are slightly moveable
C the bones are joined by cartilage.
D the ends of the articulating bones are covered by hyaline cartilage.
ANSWER:D
856 What does the term “haversian canal” refer to in bone?
A the larger examples of foramina.
B a groove that receives a muscle’s tendon.
C the centre of an osteon that contains blood capillaries.
D the space within a long bone that contains marrow.
ANSWER:C
857 What is the structure that attaches one bone to another?
A ligament
B cartilage
C tendon
D diaphysis
ANSWER:A
858 Which of the following describes what an “epiphysis” is?
A The shaft of a long bone.

B The line that separates the shaft from the end of a long bone.
C The membrane that surrounds a bone.
D The end of a long bone.
ANSWER:D
859 To what does the term “osteon” refer in bone?
A the bone cells (osteocytes) in lacunae.
B a small rounded projection on a bone.
C concentric cylinders of calcified bone matrix.
D the membrane covering the outside of a bone.
ANSWER:C
860 Blood cell formation (haemopoiesis) occurs in which of the following structures?
A red marrow
B yellow marrow
C medullary cavity
D epiphyseal plate
ANSWER:A
861 Compact bone differs from spongy (cancellous) bone because compact bone:
A does not contain osteons
B is used to form short bones
C contains marrow
D has Haversian canals
ANSWER:D
862 What is the role of hyaline cartilage in the body?
A it attaches muscle to bone.
B it reinforces joints by tying one bone to another.
C it covers articulating bone surfaces.
D it produces synovial fluid.
ANSWER:C
863 Where does the increase in the length of a long bone take place? At the:
A diaphysis ossification centres
B epiphyseal plates
C cartilaginous plates
D medullary canal

ANSWER:B
864 The human skeleton consists of which of the following?
A pectoral girdle, the hip girdle and the bones of the limbs.
B axial skeleton and the appendicular skeleton.
C cranial bones, the thoracic cage, the two girdles and the limb bones.
D appendicular skeleton, skull bones and the vertebral skeleton.
ANSWER:B
865 The functions of bones may be stated as:
A fat storage, movement, mineral storage, protection, blood cell formation.
B mineral storage, haemopoiesis, movement, leverage, protection.
C blood cell formation, hormone production, movement, support, protection.
D support, storage, movement, haemopoiesis, protection
ANSWER:D
866 What is true of synovial joints? They:
A are also known as amphiarthroses
B all have an articular disc to aid shock absorption.
C have a fluid-filled space between the articulating bones.
D have articulating bones held together by cartilage.
ANSWER:C
867 The tibia articulates distally with which one of the following?
A tarsals
B metatarsals
C phalanges
D femur
ANSWER:A
868 Which of the following is an amphiarthrotic joint?
A symphysis pubis
B suture in the skull
C elbow
D shoulder
ANSWER:A
869 What is the term “osteon” used for?
A a bone cell

B an Haversian system in compact bone
C the bony structure in spongy bone
D the space in a bone where a bone cell lives.
ANSWER:B
870 In a long bone, which of the following parts are involved in an articulation?
A epiphysis
B metaphysis
C diaphysis
D symphysis
ANSWER:A
871 Hypochondriac refers to which of the following?
A someone who complains chronically of ill health
B the abdominal region inferior to your ribs
C an abnormally low level of chondria in the body
D that part of your head surrounding your chin.
ANSWER:B
872 Where are the bones known as the humerus and radius located?
A in the axial skeleton
B in the arm
C in the leg
D in the arm and leg respectively
ANSWER:B
873 Where does blood cell formation occur? In the
A blood
B endosteum
C Haversian canal
D red marrow
ANSWER:D
874 On what bone does the acetabulum occur?
A occipital
B humerus
C pelvis
D tibia

ANSWER:C
875 Where is the xiphoid process?
A on the sternum
B on the humerus
C on the temporal bone
D on the tibia
ANSWER:A
876 What is the metaphysis?
A the shaft of a long bone.
B the region that separates the narrow shaft of a long bone from its end.
C the end of a long bone.
D the canal inside a long bone that contains marrow.
ANSWER:B
877 What is a distinguishing feature of synovial joints?
A there is fluid between the articulating bones
B they are immovable joints
C the articulating bones are held together by tendons
D they involve a “ball and socket” articulation
ANSWER:A
878 What term is applied to moving the thigh laterally away from the midline of the body?
A extension
B adduction
C abduction
D flexion
ANSWER:C
879 Where is the hyoid bone?
A in the sternum
B in the wrist
C in the knee
D in the neck
ANSWER:D
880 What is the occipital bone?
A it is one of the carpals

B it is a bone of the cranium
C it is one of the vertebrae
D it is a facial bone
ANSWER:B
881 What is a “foramen”?
A a basin-like depression serving as an articular surface.
B a raised roughening which is a site for muscle attachment.
C a hole through a bone for a nerve or blood vessel.
D a sharp slender projection of bone.
ANSWER:C
882 Which statement correctly defines an osteon?
A the arrangement of trabeculae and osteocytes that make up spongy bone.
B the membrane that lines the medullary cavity.
C concentric cylinders of calcified bone matrix.
D the distal or proximal end of a long bone.
ANSWER:C
883 What is the role of Ca++ in muscle contraction?
A Ca causes an action potential to travel along the sarcolemma
B Ca binds to troponin changing its shape.
C Ca attaches to the binding site of myosin, energising it.
D Ca engages with the binding site of actin causing the power stroke
ANSWER:B
884 Which of the following muscle structures is the largest?
A sarcomere
B fascicle
C myofibril
D muscle fibre
ANSWER:B
885 Which of the following muscles is NOT named after its location in the body?
A biceps brachii
B sternocleidomastoid
C rectus abdominus
D flexor carpi radialis

ANSWER:B
886 Which feature is shared by cardiac muscle cells and skeletal muscle cells?
A striations
B intercalated discs
C branching
D involuntary nature
ANSWER:A
887 What structures attach a muscle to a bone?
A a tendon
B a fasciculus
C a sarcomere
D an internal intercostal
ANSWER:A
888 Which of the following muscles is NOT named after its location in the body?
A latissimus dorsi
B adductor longus
C rectus femoris
D biceps brachii
ANSWER:B
889 Which of the following muscle structures is the smallest?
A sarcomere
B fasciculus
C myofibril
D muscle fibre
ANSWER:A
890 A feature of skeletal muscle that is NOT shared with cardiac or smooth muscle is:
A striations
B branched cells
C intercalated discs
D many nuclei
ANSWER:D
891 Which muscles extend the leg?
A quadriceps

B hamstrings
C gluteus muscles
D soleus, gastrocnemius & tibialis anterior
ANSWER:A
892 What is the role of acetylcholine in muscle cell contraction?
A it is a neurotransmitter
B it binds to troponin causing it to change shape
C it supplies the energy for contraction
D it engages with the binding site on actin
ANSWER:A
893 What is a sarcomere?
A it is the plasma membrane of a muscle cell
B it is the cytoplasm of a muscle cell
C it is a section of myofibril
D it is a bundle of thick and thin myofilaments
ANSWER:C
894 Which of the following muscles IS named after its location in the body?
A sartorius
B triceps brachii
C soleus
D trapezius
ANSWER:B
895 Which of the following is the smallest unit in a muscle?
A muscle fibre
B myosin
C fasciculus
D myofibril
ANSWER:B
896 Skeletal muscle cells can be characterised as:
A unstriated, involuntary, multinucleate
B unstriated, voluntary, multinucleate
C striated, voluntary, uninucleate
D striated, voluntary, multinucleate

ANSWER:D
897 The muscles involved in mastication include which of the following?
A sternocleidomastoid, scalene
B sartorius, gracilis, soleus
C temporalis, masseter, buccinator
D orbicularis oculi, mentalis
ANSWER:C
898 What action does the flexor carpi ulnaris perform?
A it flexes the lower arm
B the same as the extensor carpi ulnaris.
C it flexes the fingers
D the same as the flexor carpi radialis
ANSWER:D
899 Which one of the following is not made of skeletal muscle?
A the diaphragm
B pyloric sphincter
C vastus lateralis
D the tongue
ANSWER:B
900 Which of the following muscles is named using the criterion of its size?
A sternocleidomastoid
B gluteus medius
C flexor digitorum profundus
D trapezius
ANSWER:B
901 What is the cytoplasm of a skeletal muscle cell called?
A sarcolemma
B sarcomere
C sarcoplasm
D fasciculus
ANSWER:C
902 What does the term “origin” refer to in the musculoskeletal system?
A The point of attachment of a muscle to the “moveable” bone.

B The line that separates the shaft from the end of a long bone.
C The point of attachment of a muscle to the “stationary” bone.
D The end of a long bone.
ANSWER:C
903 Which of the following muscles causes the wrist to bend?
A extensor digitorum
B extensor carpi ulnaris
C flexor digitorum profundus
D abductor pollicis longus
ANSWER:B
904
Which of the following muscles increases the angle between the bones of the fingers and
hand?
A extensor digitorum
B extensor carpi ulnaris
C flexor digitorum profundus
D abductor pollicis longus
ANSWER:A
905 What is the protein of thick myofilaments in a skeletal muscle cell?
A tropomyosin
B myosin
C actin
D acetylcholine
ANSWER:B
906
What are the muscles known as triceps brachii, biceps femoris and quadriceps femoris named
according to? Their:
A relative size and location of muscle’s origin
B number of origins and location in the body
C shape of muscle and direction of muscle fibres
D number of insertions and location in the body
ANSWER:B
907
A skeletal muscle fibre (cell) consists of many sections (units) which contract. What is the
name given to one of the units that contract?
A sarcomere

B sarcolemma
C sarcoplasm
D fasciculus
ANSWER:A
908
The neurotransmitter that causes an action potential to occur in a muscle cell membrane is
called:
A inorganic phosphate (HPO −)
B adenosine diphosphate (ADP)
C calcium (Ca++)
D acetylcholine (ACh)
ANSWER:D
909 Which statement about thick or thin myofilaments is CORRECT?
A Thick myofilaments contain the three proteins myosin, tropomyosin and troponin.
B Thin myofilaments contain the three proteins actin, tropomyosin and troponin.
C
Thick myofilaments contain about 00 myosin molecules each of which has a binding site for
a cross bridge.
D Thin myofilaments contain about 00 myosin molecules each of which has a cross bridge.
ANSWER:B
910 What is the role of calcium ions in muscle contraction? To
A bind to troponin, thus changing its shape and pulling it away from the actin molecule.
B cause the myosin cross-bridge to detach from its binding site.
C cause the action potential to propagate along the sarcolemma.
D bind with ADP during aerobic respiration to produce ATP to provide energy.
ANSWER:A
911 The space between the ribs is filled with:
A intercostal muscle
B costal cartilage
C intercostal space
D pleura
ANSWER:A
912 Which of the following describes skeletal muscle?
A striated, voluntary, multinucleate, individually named.
B striated, branched, uninucleate, involuntary.

C not striated, uninucleate, voluntary, individually named.
D not striated, multinucleate, involuntary, with intercalated discs.
ANSWER:A
913 Which list is in the correct order of DECREASING size?
A muscle fibre, sarcomere, myofilament, myofibril.
B muscle, fasciculus, muscle fibre, myofibril.
C sarcomere, fasciculus, myofibril, myofilament.
D muscle, muscle fibre, myosin, myofibril.
ANSWER:B
914
What are the muscles known as gluteus maximus, gluteus medius and gluteus minimus
named according to? Their:
A size.
B shape.
C whimsy.
D direction of their muscle fibres.
ANSWER:A
915 Select the one INCORRECT statement about skeletal muscles
A An “agonist” opposes or reverses a particular movement
B A muscle’s attachment point to a stationary bone is called its “origin”.
C A skeletal muscle cell is a “syncytium”.
D Muscles that immobilise a bone are called “fixators”.
ANSWER:A
916 Which of the following groupings of muscle type and their characteristics is INCORRECT?
A skeletal, striated, voluntary
B smooth, visceral, involuntary
C cardiac, striated, voluntary
D skeletal, striated, syncytium
ANSWER:C
917
Microscopically, muscle fibres contain parallel myofibrils. What are the units joined end to
end within a myofibril called? A:
A myofilament
B motor unit
C myosin

D sarcomere
ANSWER:D
918 The part of a skeletal muscle cell that is able to contract is called
A sarcoplasm
B sarcolemma
C sarcomere
D sarcoplasmic reticulum
ANSWER:C
919 What is the role of Ca++ in the contraction of a muscle cell?
A Ca++ binds to troponin to change its shape which reveals actin’s binding site.
B Ca++ attaches to the binding site of actin
C Ca++ detaches from ATP as it forms ADP
D Ca++ causes the myosin head to detach from the binding site of actin
ANSWER:A
920
The energy for muscle contraction is derived from the mechanisms below EXCEPT for one.
Which one is NOT a method of producing ATP?
A anaerobic glycolysis.
B aerobic respiration.
C direct phosphorylation of ADP by creatinine phosphate.
D anaerobic digestion of lactic acid.
ANSWER:D
921 With respect to the flexion of the forearm, which of the following statements is correct?
A the origin of the biceps brachii is on the radius and its insertion is on the scapula
B the origin of the biceps brachii is on the ulna and its insertion is on the scapula
C the agonist muscle is the biceps brachii and the antagonist is the triceps brachii
D the agonist muscle is the biceps brachii and the antagonist is the brachialis
ANSWER:C
922
Patients confined to bed and those with plaster casts immobilising a bone frac- ture suffer
muscle wasting. What is the term used for this condition?
A Disuse atrophy
B Denervation atrophy
C Muscle dystrophy
D Muscle hypertrophy

ANSWER:A
923
What is the source of the ATP used by muscles for vigorous activity that may last for 0 to
seconds?
A glycolysis of glucose in the cell cytoplasm forms ATP.
B the ATP that is stored in muscle cells as ATP.
C aerobic respiration in the mitochondria produces the ATP
D creatinine phosphate in muscle and ADP react to form the required ATP.
ANSWER:D
924 Which muscle and bone listed below do NOT work together in combination?
A humerus and biceps femoris
B quadriceps and tibia
C femur and gluteal muscles
D radius and biceps brachii
ANSWER:A
925 Which of the following muscles is named after its origin and insertion points?
A tibialis anterior
B extensor digitorum longus
C rectus femoris
D sternocleidomastoid
ANSWER:D
926 Which of the following is NOT a common intramuscular injection site?
A gluteus medius
B deltoid
C gluteus maximus
D vastus lateralis
ANSWER:C
927 What structure attaches a muscle to a bone?
A a meniscus
B a ligament
C a cartilage
D a tendon
ANSWER:D
928 The muscle known as the “transversus abdominus” is named according to

A its size and number of origins
B the direction of its muscle fibres and its action
C its action and its location in the body
D location in the body and direction of muscle fibres
ANSWER:D
929 When a muscle contracts, exactly what structure gets shorter?
A the fascicles of a muscle
B the myosin molecules of a myofilament
C the actin molecules of a myofilament
D the sarcomeres of a myofibril
ANSWER:D
930 What is the neurotransmitter that crosses the neuromuscular junction?
A Acetylcholine (ACh)
B Adrenalin (epinephrine)
C Noradrenalin (norepinephrine)
D Ca++
ANSWER:A
931 What does aerobic respiration refer to?
A glycolysis in the cytoplasm in the absence of oxygen.
B oxidative phosphorylation in the mitochondria in the presence of oxygen.
C glycolysis in the liver in the presence of oxygen
D gluconeogenesis in the liver in the absence of oxygen
ANSWER:B
932 Which of the following statements is WRONG? The end products of:
A protein digestion are transported to the liver via the hepatic portal vein
B triglyceride digestion are transported to the liver via the lymphatic system
C carbohydrate digestion are transported to the liver in the blood
D triglyceride digestion are transported via the lymphatic system
ANSWER:B
933 One of the following processes is NOT part of mechanical digestion. Which One?
A hydrolysis
B peristalsis
C segmentation

D mastication
ANSWER:A
934 Which type of cell produces hydrochloric acid?
A Zymogenic cells
B Parietal cells
C Chief cells
D Enteroendocrine cells
ANSWER:B
935 Which of the following glands are accessory organs of the digestive system?
A adrenal glands
B pancreatic islets
C gastric glands
D salivary glands
ANSWER:D
936 What is the role of gastrin in the digestive system?
A to stimulate release of bile and pancreatic juice
B to stimulate gastric secretion
C to activate pepsinogen
D to hydrolyse proteins to polypeptides
ANSWER:B
937 Which liver cells produce bile?
A Kupffer cells
B sinusoids
C hepatocytes
D the acini
ANSWER:C
938 What are the end products of carbohydrate digestion?
A chylomicrons
B amino acids
C free fatty acids
D monosaccharides
ANSWER:D
939 What feature of the small intestine enhances its ability to absorb digested food?

A its large surface area
B the gaps between adjacent epithelial cells
C secretion of the hormone absorptin
D its longer length compared to the large intestine
ANSWER:A
940
Which of the following gut structures are listed in the correct order that food would pass
through them, from input to exit?
A pyloric sphincter, ileum, jejunum, transverse colon.
B pancreas, jejunum, ascending colon, sigmoid colon.
C ileum, duodenum, descending colon, ascending colon.
D duodenum, ileum, caecum, transverse colon.
ANSWER:D
941 Which statement about the layers of the alimentary canal is correct?
A The serosa absorbs the products of digestion.
B The mucosa protects against self-digestion.
C The sub-mucosa is involved in segmentation and peristalsis
D The muscularis externa is dense connective tissue.
ANSWER:B
942
Which of the following pairs of substances are NOT secreted by the stomach as part of
“gastric juice”?
A hydrochloric acid and pepsinogen
B hormones and intrinsic factor
C nuclease and amylase
D mucus and gastrin
ANSWER:C
943 What are some products of lipid digestion?
A free bases and pentose sugars
B fructose and glucose
C amino acids and small peptides
D free fatty acids and monoglycerols
ANSWER:D
944 From which of the gut structures below is most digested food absorbed?
A duodenum

B stomach
C ileum
D ascending colon
ANSWER:C
945 Which of the following is an active enzyme?
A procarboxypeptidase
B pepsin
C telophase
D trypsinogen
ANSWER:B
946 Which of the following is a function of the liver?
A Recycling of non-viable red blood cells
B Conversion of pyruvic acid to lactic acid
C Synthesis of plasma proteins
D Production of renin
ANSWER:C
947 What is the term applied to the production of glucose from non-carbohydrate molecules?
A deamination
B transamination
C glycogenolysis
D gluconeogenesis
ANSWER:D
948
Which of the following terms is used to describe the changing of large food molecules into
smaller molecules?
A mechanical digestion
B deglutition
C segmentation
D hydrolysis
ANSWER:D
949 What is the purpose of “intrinsic factor” in gastric juice?
A to activate pepsinogen
B to assist with the absorption of vitamin B
C to protect the stomach lining against hydrochloric acid

D it stimulates the release of gastrin
ANSWER:B
950
Which of the following does NOT contribute to increasing the surface area of the small
intestine?
A the brush border
B plicae circulars
C intestinal crypts
D villi
ANSWER:C
951 What are the end-products of carbohydrate digestion?
A monosaccharides
B disaccharides
C glucose
D trisaccharides
ANSWER:A
952 Name the major cell type in a liver lobule.
A Kupffer cells
B hepatocytes
C sinusoids
D epithelial cells
ANSWER:B
953 To what does the term “gluconeogenesis” refer?
A the conversion of glycogen to glucose.
B the removal of an amine group from an amino acid.
C the production of glucose from non-carbohydrate molecules.
D the conversion of disaccharides to monosaccharides.
ANSWER:C
954 Which of the following is TRUE of bile?
A it converts inactive pancreatic enzymes to active form.
B needed in the small intestine for the digestion of fats.
C synthesised by the gall bladder.
D needed in the small intestine for the emulsification of fats.
ANSWER:D

955 Why are the blood capillaries in the liver lobules so permeable?
A to allow the products of digestion to leave the blood for processing in the liver.
B to allow fatty acids to leave the liver cells to enter the blood.
C to allow plasma proteins that are synthesised in the liver to enter the blood.
D to allow red blood cells at the end of their life to leave the blood to be recycled in the liver.
ANSWER:C
956 Which layer of the gastro-intestinal tract is in contact with the contents of the gut?
A Muscularis externa
B Mucosa
C Serosa
D Sub-mucosa
ANSWER:B
957
What is the name given to the process of moving the gut contents along the tract in the right
direction?
A Peristalsis
B Emesis
C Segmentation
D Deglutition
ANSWER:A
958 Which of the following substances is NOT produced by the cells of the gastric glands?
A mucus
B hydrochloric acid
C gastrin
D pepsin
ANSWER:A
959 Which hormone stimulates the release of bile and pancreatic juice?
A cholecystokinin
B secretin
C intestinal gastrin
D pepsin
ANSWER:A
960 What molecules are the products of protein hydrolysis?

A Monoglycerols and free fatty acids
B Monosaccharides and disaccharides
C Amino acids
D Amino acids and small peptides
ANSWER:D
961 What happens to the products of lipid digestion in the gut?
A They are actively transported into the epithelial cells lining the gut.
B They diffuse into epithelial cells and are reconstituted into triglycerides.
C They are transported to the liver by the hepatic portal vein.
D
They diffuse through the plasma membrane of epithelial cells, then diffuse into blood
capillaries
ANSWER:B
962 One of these processes is NOT part of carbohydrate metabolism in the liver. Which one?
A Production of ATP from glucose
B Production of glucose from glycogen
C Production of glucose from amino acids
D Production of glycogen from glucose
ANSWER:A
963
The liver contains “leaky capillaries” known as sinusoids. This enables what liver product to
enter the blood stream?
A Angiotensinogen
B Kupffer cells
C Plasma proteins
D Cholesterol
ANSWER:C
964 What is the function of bile salts?
A To assist the absorption of digested lipids
B To emulsify lipids
C To hydrolyse lipids
D To digest lipids
ANSWER:B
965 Digestion of food molecules is necessary so that:
A Indigestible food molecules are separated from digestible food molecules

B Essential amino acids and fatty acids may be absorbed by the body
C Excretion of waste products can occur via the bowel
D Food may be converted into particles small enough to pass into the cells of the gut wall
ANSWER:D
966 Where is the gastro-oesophageal sphincter?
A between the stomach and the duodenum
B between the stomach and the caecum
C at the entrance to the stomach
D before the external anal sphincter
ANSWER:C
967 What are the end products resulting from the digestion of carbohydrates?
A monosaccharides
B monoglycerols
C pentose sugars
D amino acids
ANSWER:A
968
Which sections of the gut perform the majority of the digestion of food and absorption of
the digested products?
A stomach and duodenum
B jejunum and ileum
C ascending colon and transverse colon
D duodenum and jejunum
ANSWER:D
969 Which enzyme below digests proteins?
A nuclease
B maltase
C carboxypeptidase
D transaminase
ANSWER:C
970 To what does the term “gluconeogenesis” refer?
A the conversion of non-carbohydrate molecules to glucose
B the formation of non-essential amino acids from a keto-acid
C the removal of an amine group from a molecule

D the release of glucose from stored glycogen
ANSWER:A
971
A lobule of the liver contains several blood vessels. Which one carries nutrient- rich blood
from the small intestine?
A hepatic artery proper
B hepatic portal vein
C central vein
D bile ductule
ANSWER:B
972 Which of the following could NOT be used to describe pepsinogen?
A it is a protein
B it is a hormone
C it is related to an enzyme
D it is inactive
ANSWER:B
973 What is the function of the oesophagus in digestion?
A It is a site of mechanical digestion.
B It transfers food from the mouth to the stomach.
C The oesophagus secretes amylase to begin carbohydrate digestion.
D the oesophagus secretes hydrochloric acid.
ANSWER:B
974
What is the purpose of the mucosal barrier between the cells of the stomach wall and the
stomach contents?
A It prevents the enzymes in the stomach contents from digesting the stomach
B It converts pepsinogen to its active form.
C It prevents bacteria in the stomach from invading the stomach wall.
D It prevents undigested food molecules from being absorbed by the stom- ach lining.
ANSWER:A
975 Which parts of the alimentary canal prepare food for chemical digestion?
A the mouth, oesophagus and stomach.
B the mouth, stomach and small intestine.
C the mouth, stomach and duodenum.
D the teeth, stomach and pancreas.

ANSWER:C
976 What food is digested by lipase?
A nucleic acids
B carbohydrates
C polypeptides
D triglycerides
ANSWER:D
977 What food is digested into monoglycerols?
A protein
B lipid
C nucleic acid
D starch
ANSWER:B
978 What is the function of bile?
A bile hydrolyses polypeptides.
B bile emulsifies fats and oils.
C bile activates procarboxypeptidase.
D bile stimulates the pancreas to secrete pancreatic juice.
ANSWER:B
979
The liver is able to deaminate amino acids forming ammonia in the process. What happens to
the ammonia?
A It is phagocytosed by Kupffer cells.
B It is used in transamination to form non-essential amino acids.
C It is converted to bile to be excreted via the gut.
D It is converted to urea for excretion by the kidneys.
ANSWER:D
980 What happens to the products of digestion of lipids? They are absorbed into a:
A capillary and transported by the blood to the liver.
B capillary and transported by the blood to the heart.
C lacteal and transported by the lymph to the heart.
D lacteal and transported by the lymph to the liver.
ANSWER:C
981 If blood glucose is high, what does the liver do about it?

A The liver converts glucose to glycogen or triglycerides.
B The liver performs glycogenolysis.
C The liver performs gluconeogenesis.
D The liver transaminates glucose to produce amino acids.
ANSWER:A
982 Which is an enzyme secreted by the gastric glands?
A pepsin
B gastrin
C Cholecystokinin
D intrinsic factor
ANSWER:A
983 What are the products of protein digestion?
A monoglycerols and fatty acids
B dipeptides, tripeptides and amino acids
C bases, pentose sugars and nitrate ions
D monosaccharides and disaccharides
ANSWER:B
984 Which three sections does the small intestine consists of?
A ileum duodenum, caecum
B antrum, jejunum, duodenum
C rectum, ileum, duodenum
D ileum, duodenum, jejunum
ANSWER:D
985 What name is given to the movement of food material through the gastrointes- tinal tract?
A peristalsis
B segmentation
C deglutition
D bowel movement
ANSWER:A
986 Correctly complete the sentence: Pepsinogen is
A converted to pepsin by hydrochloric acid
B converted to pepsin by intrinsic factor
C secreted by the pancreas

D involved in production of carbohydrate digesting enzymes
ANSWER:A
987 Emulsification is the name of the process carried out by:
A lipase
B bile
C micelles
D lacteals
ANSWER:B
988 The pH of the stomach and the pH of the small intestine are BEST described (respectively) as:
A acidic and alkaline/basic
B strongly acidic and weakly alkaline/basic
C acidic and weakly alkaline/basic
D strongly acidic and strongly alkaline/basic
ANSWER:B
989
What feature do procarboxypeptidase, pepsinogen, fibrinogen, chymotryp- sinogen have in
common?
A they are all enzymes.
B they are all produced by the pancreas.
C they are all inactive.
D they all digest proteins.
ANSWER:C
990 Which layer of the alimentary canal is responsible for absorbing the products of digestion?
A muscularis interna
B mucosa
C serosa
D submucosa
ANSWER:B
991 Protein is digested to polypeptides by which of the following?
A pepsinogen
B intrinsic factor
C hydrochloric acid
D pepsin

ANSWER:D
992 What are the products of carbohydrate digestion?
A monosaccharides
B amino acids
C monoglycerides
D monogylcerols
ANSWER:A
993 What does bile do?
A Bile stimulates the release of lipase
B Bile emulsifies fat
C Bile digests fat
D Bile hydrolyses fat
ANSWER:B
994 What is the function of the hepatic portal vein?
A to return blood from the liver to the heart
B to transport blood rich in amino acids, monosaccharides and free fatty acids to the liver
C to transport the products of protein and carbohydrate digestion from gut to the liver
D to allow the nutrients absorbed from the gut to bypass the liver.
ANSWER:C
995 Which of the following are the end products of protein hydrolysis?
A monoglycerols
B keto acids and non-essential amino acids
C polypeptides
D amino acids and di- and tripeptides
ANSWER:D
996 One of the following is NOT a function of the liver. Which one?
A recycling of red blood cells
B storage of fat soluble vitamins
C removal and recycling of lactic acid
D activation of vitamin D
ANSWER:A
997 What converts pepsinogen to pepsin in the stomach?
A hydrochloric acid

B gastrin
C intrinsic factor
D pepsinase
ANSWER:A
998 What the products of hydrolysis of lipids?
A monosaccharides
B chylomicrons
C amino acids and small peptides
D free fatty acids and monoglycerols
ANSWER:D
999 Why are sinusoids the type of capillaries found within a liver lobule?
A to allow for mixing of blood from the hepatic artery and the hepatic portal vein.
B so that liver synthesised plasma proteins may enter the blood
C to allow worn out red blood cells to leave the blood stream.
D in order for the products of digestion to be removed from the blood.
ANSWER:B
1000
Which of the following presents the structures through which chyme travels in the correct
sequence?
A oesophagus, ileum, duodenum, ileo-cecal valve, transverse colon, rectum
B stomach, duodenum, ileum, transverse colon, ileo-cecal valve, rectum
C stomach, duodenum, transverse colon, ileum, rectum, ileo-cecal valve,
D ileo-cecal valve, stomach, duodenum, transverse colon, ileum, rectum
ANSWER:B
1001 What is the function of bile salts?
A to digest dietary fats through hydrolysis
B to excrete the products haeme breakdown
C to emulsify ingested fats and oils
D to activate trypsinogen and chymotrypsinogen
ANSWER:C
1002 Which of the functions below is NOT performed by the liver?
A production of glucagon
B synthesis of lipoproteins to transport fatty acids
C deamination of amino acids to form keto-acids

D conversion of non-carbohydrate molecules to glucose
ANSWER:A
1003 What is the function of gastrin?
A to facilitate the absorption of vitamin B from the gut
B to inhibit gastric secretion
C to stimulate gastric secretion
D to stimulate pancreatic secretion
ANSWER:C
1004
Which list of sections of the intestine has them in correct order from nearest to furthest from
the mouth?
A duodenum caecum, jejunum, ileum
B caecum, sigmoid colon, transverse colon, rectum
C duodenum, ileum, rectum, jejunum,
D jejunum, ileum, caecum, ascending colon
ANSWER:D
1005
Forward movement of food material through the gastrointestinal tract is achieved by which
process?
A peristalsis
B emesis
C deglutition
D hydrolysis
ANSWER:A
1006 What are the cells in the pancreas that secrete “pancreatic juice” called?
A hepatocytes
B Peyer’s patches
C the acini
D islets of Langerhans
ANSWER:C
1007 The lowest pH is found in which of the listed body sites?
A pancreas
B stomach
C duodenum
D blood

ANSWER:B
1008 Which of the following is a function of bile?
A To attach to vitamin B to allow it to be absorbed.
B To activate trypsinogen, chymotrypsinogen and procarboxypeptidase.
C To digest fats.
D To disperse large lipid globules into smaller droplets.
ANSWER:D
1009 What is the process that splits carbohydrates in the gut into smaller molecules called?
A catalysis
B hydrolysis
C catabolism
D glycogenolysis
ANSWER:B
1010 What is true about the “muscularis externa”?
A It is muscle used for peristalsis.
B It secretes mucus
C It refers to superficial skeletal muscles
D It is composed of connective tissue
ANSWER:A
1011
The surface area available for absorption in the small intestine is increased by all of the
following structures EXCEPT one. Which one?
A villi
B haustra
C plicae circularis
D microvilli
ANSWER:B
1012 The majority of dietary lipids are ingested in the form of
A trisaccharides
B tripeptides
C cholesterol
D triglycerides
ANSWER:D
1013 Emulsification is the process where

A procarboxypeptidase and chymotrypsinogen become active enzymes
B chyme is moved backwards and forwards across the surface of the small intestine
C fat droplets are dispersed into smaller droplets
D dietary fat is digested by lipase
ANSWER:C
1014 Kupffer cells are macrophages. Where are they found?
A In the lymphatics of the submucosa and devour bacteria that escape the gut
B In the lumen of the large intestine and feed on our normal flora to produce vitamin K
C They are in the stomach wall as part of the mucosal barrier
D They occur in liver sinusoids and engulf bacteria in blood coming from the gut
ANSWER:D
1015 One of the following statements is UNTRUE. Which one?
A Glucose is produced during the manufacture of ATP
B Glycogenolysis is the process of releasing glucose from glycogen
C Gluconeogenesis is the conversion of amino acids to glucose
D The monosaccharides galactose and fructose can be converted to glucose
ANSWER:A
1016 Monosaccharides are the product of digestion of what substance?
A proteins
B carbohydrates
C triglycerides
D dipeptides
ANSWER:B
1017 One of the following is NOT a function of the large intestine. Which one?
A absorption of electrolytes
B synthesis of some vitamins
C absorption of water
D digestion of fats.
ANSWER:D
1018
How are the pH of the small intestine and the pH of the stomach (respectively) best
described?
A alkaline (basic) and acidic
B weakly alkaline (basic) and strongly acidic

C pH of and pH of
D pH of and pH of .
ANSWER:B
1019 Which of the following is an enzyme?
A amylase
B gastrin
C intrinsic factor
D pepsinogen
ANSWER:A
1020
Choose the list which has the selected structures of the alimentary canal in the same order that
chyme would pass through them.
A larynx, jejunum, ileum, descending colon, transverse colon, sigmoid colon.
B mouth, pharynx, oesophagus, large intestine, small intestine, anus.
C oesophagus, stomach, duodenum, ascending colon, sigmoid colon, rectum.
D stomach, duodenum, ileum, descending colon, transverse colon, ascend- ing colon.
ANSWER:C
1021
What feature do procarboxypeptidase, pepsinogen, trypsinogen, chymotryp- sinogen have in
common?
A they are all enzymes.
B they are all produced by the pancreas.
C they all digest proteins.
D they are all inactive.
ANSWER:D
1022
The products of fat digestion are absorbed into the epithelial cells of the intes- tinal wall
differently from the way products of protein and carbohydrate diges- tion are. The reason is:
A the products of protein and carbohydrate digestion are smaller.
B the products of fat digestion are actively transported across the plasma membrane.
C the products of fat digestion are smaller.
D monoglycerides are soluble in the plasma membrane.
ANSWER:D
1023 Which fluid within the body is likely to have the LOWEST pH?
A the chyme in the ileum.

B saliva.
C the blood.
D the chyme in the stomach.
ANSWER:D
1024 What pancreatic enzyme digests lipids to free fatty acids and monoglycerides?
A lipase
B bile
C cholecystokinin
D lingual lipase
ANSWER:A
1025 What is the product of carbohydrate digestion?
A glucose
B monosaccharides
C glycogen
D ATP
ANSWER:B
1026 Which of the following structures produce bile?
A the gall bladder
B the liver
C the pancreas
D the duodenum
ANSWER:B
1027 What is a function of the stomach?
A absorb the products of digestion.
B participate in deglutition.
C participate in mechanical digestion.
D release cholecystokinin.
ANSWER:C
1028 Which of the following is NOT a function of the mucosa of the small intestine?
A Protection against infectious disease.
B Secretion of digestive enzymes.
C The absorption of end products of digestion.
D Segmentation.

ANSWER:D
1029 Which fluid within the body is likely to have the HIGHEST pH?
A the contents of the ileum.
B the contents of the start of the duodenum.
C the blood.
D the contents of the stomach.
ANSWER:A
1030 What are the products of protein digestion?
A polypeptides
B monosaccharides
C amino acids
D free fatty acids and monoglycerides
ANSWER:C
1031 Which one of the following is a function of the liver?
A lipase
B digestive enzymes
C insulin
D plasma proteins
ANSWER:D
1032 A function of the LARGE intestine is to:
A absorb the products of digestion
B absorb water
C participate in mechanical digestion
D release intestinal gastrin
ANSWER:B
1033 Which fluid within the body is likely to have the LOWEST pH?
A the contents of the ilium.
B the contents of the start of the duodenum.
C urine in the bladder.
D the contents of the stomach.
ANSWER:D
1034 One of the following is NOT a function of the stomach. Which one?
A digestion of fats

B digestion of proteins
C mechanical digestion
D storage of food
ANSWER:A
1035 Which of the following pancreatic juice enzymes aids in the digestion of proteins?
A amylase
B lipase
C nuclease
D trypsin
ANSWER:D
1036 What role do the Kupffer cells of the liver perform?
A they are sinusoids
B they are hepatocytes
C they are macrophages
D they de-aminate amino acids
ANSWER:C
1037 What is the process in the digestion of food molecules that produces their monomers called?
A polymerisation
B hydrolysis
C isomerisation
D deamination
ANSWER:B
1038 Fatty acids are transported around the body by the blood in structures known as:
A micelles
B chylomicrons
C triglycerols
D low density lipoproteins
ANSWER:B
1039
The wall of the alimentary canal is made up of the four layers (not in any order): the
muscularis externa, serosa, mucosa and sub-mucosWhich layer absorbs the end products of
digestion?
A sub-mucosa

B muscularis externa
C serosa
D mucosa
ANSWER:D
1040 Which one of the following is NOT secreted in pancreatic juice?
A amylase
B trypsinogen
C pepsinogen.
D lipase
ANSWER:C
1041 In the stomach which cells secrete pepsinogen?
A parietal cells
B zymogenic cells
C Kupffer cells
D enteroendocrine cells
ANSWER:B
1042
Which part of the gastro-intestinal tract contains three distinct layers of smooth muscle in its
walls?
A rectum
B small intestine
C stomach
D oesophagus
ANSWER:C
1043 Which digestive enzyme in pancreatic juice digests proteins?
A trypsin
B lipase
C pepsin
D amylase
ANSWER:A
1044 Which digestive enzyme in saliva breaks down starch?
A trypsin
B lipase
C pepsin

D amylase
ANSWER:D
1045 Which hormone is responsible for contraction of the gall bladder?
A secretin
B gastrin
C cholecystokinin
D intrinsic factor
ANSWER:C
1046 All of the following statements regarding the liver are true except one, which one?
A It can convert amino acids to glucose during periods of fasting
B
Blood from the hepatic artery and portal vein travels away from the central vein of each
lobule
C It contains special phagocytic cells which remove worn-out blood cells from the circulation
D It converts ammonia to urea
ANSWER:B
1047 During which phase are our gastric secretions stimulated by the sight and smell of food?
A gastric
B digestive
C cephalic
D intestinal
ANSWER:C
1048 Which statement best describes the process of Glycogenesis
A The digestion of glycogen in the diet.
B The conversion of fat into glycogen in muscle tissue
C The conversion of glucose into glycogen in the liver
D The conversion of glycogen into glucose in muscle tissue
ANSWER:C
1049 Which of the following is a function of the normal flora of the large intestine?
A To hydrolyse cellulose
B To synthesise blood clotting proteins
C To synthesise B vitamins and vitamin K
D To secrete intrinsic factor

ANSWER:C
1050 Why is insulin not given as an oral drug?
A It is too irritating to the gastro-intestinal mucosa
B It is altered by passing through the liver
C It is too big a molecule to be absorbed through the plasma membrane
D It would be digested by enzymes in the stomach
ANSWER:D
1051 Which one of the listed molecule types are absorbed from the gut?
A starch
B monosaccharides
C cellulose
D polypeptides
ANSWER:B
1052 What are the products of carbohydrate digestion?
A free bases and pentose sugars
B monosaccharides
C cellulose and disaccharides
D free fatty acids and monoglycerides
ANSWER:B
1053 One of the functions of the liver is to produce:
A blood cells
B digestive enzymes
C insulin and glucagon
D glycogen from glucose
ANSWER:D
1054 What is a function of the SMALL intestine? To:
A temporarily store ingested food
B absorb the products of digestion
C participate in mechanical digestion
D secrete hydrochloric acid
ANSWER:B
1055 Which of the following organs is an accessory organ of the gastrointestinal tract?
A jejunum

B appendix
C caecum
D pancreas
ANSWER:D
1056 What is the name of the hormone that inhibits the stomach from secreting gastric juice?
A gastrin
B pepsin
C enterogastrin
D cholecystokinin
ANSWER:D
1057 What role do the Kupffer cells of the liver perform?
A they transport plasma proteins
B they perform gluconeogenesis
C they are macrophages
D they produce bile
ANSWER:C
1058 Fatty acids are absorbed from the gut into structures known as:
A triglycerides
B sinusoids
C capillaries
D lacteals
ANSWER:D
1059 Which statement below about hormones is true?
A Hormones are enzymes that catalyse reactions
B Hormones are released into the blood circulation
C Hormones affect all cells of the body
D Hormones are released by neurones at synapses
ANSWER:B
1060 Which hormones are soluble in blood?
A Steroid hormones
B Hormones produced by the adrenal cortex
C The sex hormones
D Those released by the pituitary gland

ANSWER:D
1061 Which statement about the hypothalamus is correct?
A The hypothalamus is connected to the brain by the infundibulum
B The hypothalamus is composed of glandular epithelial tissue
C The hypothalamus secretes “releasing hormones”
D The hypothalamus secretes epinephrine and norepinephrine
ANSWER:C
1062 What hormone does the thyroid produce?
A thyroid stimulating hormone
B calcitriol
C thyroxine
D parathyroid hormone
ANSWER:C
1063 What hormone(s) does the adrenal medulla produce?
A aldosterone
B epinephrine and norepinephrine
C corticosteroids
D glucocorticoids
ANSWER:B
1064 What is produced by the beta cells of the pancreas?
A angiotensin converting enzyme
B glucocorticoids
C glucagon
D insulin
ANSWER:D
1065 Which gland or organ releases erythropoietin?
A The kidneys
B The adrenal glands
C The anterior pituitary
D The pancreas
ANSWER:A
1066 What effect does parathyroid hormone have?
A It increases plasma Ca+ concentration

B It decreases plasma Ca+ concentration
C It increases the rate of ATP formation
D It stimulates the thyroid gland to produce thyroxine
ANSWER:A
1067 Which one of the following is NOT part of the endocrine system?
A the islets of Langerhans (pancreatic islets)
B the thyroid gland
C the acini cells of the pancreas
D the parathyroid glands
ANSWER:C
1068 What is the difference between an exocrine gland and an endocrine gland?
A An endocrine gland secretes neurotransmitters (an exocrine gland does not).
B An endocrine gland secretes via a tube to the destination (an exocrine gland does not).
C An exocrine gland secretes into the blood (an endocrine gland does not).
D An endocrine gland secretes into the blood (an exocrine gland does not).
ANSWER:D
1069 By what term are hormones derived from tyrosine also known?
A amino acid derivatives
B peptide hormones
C steroid hormones
D corticosteroids
ANSWER:A
1070 Which hormones have their receptors inside their target cell?
A amino acid based hormones
B hormones with a membrane carrier mechanism or that are lipid soluble
C steroid hormones and peptide hormones of less than 0 amino acids
D lipid soluble hormones
ANSWER:B
1071 Where in the body is the hypothalamus located?
A On the inferior surface of the brain
B In the cortex of the adrenal gland
C In the anterior pituitary gland
D On the dorsal surface of the thyroid gland

ANSWER:A
1072 From where are antidiuretic hormone and oxytocin released?
A the anterior pituitary
B the posterior pituitary
C the adrenal cortex
D the adrenal medulla
ANSWER:B
1073 Which hormone has the element iodine as part of its molecule?
A calcitonin
B haemoglobin
C thyroxine
D parathyroid hormone
ANSWER:C
1074 What effect does aldosterone have?
A It causes glucose to be absorbed from the blood
B It cause Na+ to be absorbed in the kidneys
C It causes Ca++ to be absorbed from the gut
D It causes K+ to be absorbed from the filtrate.
ANSWER:B
1075 Which of the following is a part of the endocrine system?
A the thalamus
B the pancreatic islets (islets of Langerhans)
C the renal glands
D the salivary glands
ANSWER:B
1076 Which of the following is an amino acid derivative hormone?
A epinephrine
B tyrosine
C testosterone
D prostaglandin
ANSWER:A
1077 Which statement below is true of steroid hormones?
A they do not have a specific receptor to bind with.

B they are not lipid soluble do bind to receptor proteins on the cell membrane.
C they are lipid soluble so diffuse through the cell membrane.
D they cross the cell membrane via a carrier mechanism.
ANSWER:C
1078
Which structure controls the endocrine system and integrates the activities of the nervous and
endocrine systems?
A the infundibulum
B the pituitary gland
C the thalamus
D the hypothalamus
ANSWER:D
1079 One of the statements below is true. Which one?
A
the anterior pituitary produces testosterone from cholesterol and releases it when releasing
hormones arrive from the hypothalamus.
B the hypothalamus produces ADH and oxytocin which are stored in the pos- terior pituitary.
C
the posterior pituitary contains autonomic centres that exert neural control over the adrenal
glands.
D the thalamus produces ADH and oxytocin which are stored in the anterior pituitary
ANSWER:B
1080 Iodine is an essential component of which hormone?
A thyroid hormones
B aldosterone
C thyroid stimulating hormones
D parathyroid hormone
ANSWER:A
1081 Which hormone(s) increases the reabsorption of Ca++ from the filtrate in the kidney tubule?
A calcitonin
B mineralocorticoids
C parathyroid hormone
D aldosterone
ANSWER:C

1082 What effect does aldosterone have? It causes:
A angiotensin to be formed from angiotensinogen
B Na+ to be absorbed from the filtrate
C Na+ and Ca++ to be absorbed from the filtrate and K+ to be secreted into the filtrate
D Na+ to be absorbed from the filtrate and K+ to be secreted into the filtrate
ANSWER:D
1083 Which of the following is NOT part of the endocrine system?
A The thymus
B The pineal gland
C The acini cells of the pancreas
D The posterior pituitary gland
ANSWER:C
1084 Peptide hormones are produced (and/or released) by which structure?
A The adrenal cortex
B The gonads
C The hypothalamus
D The kidneys
ANSWER:C
1085 Which statement applies to steroid hormones?
A They are transported dissolved in blood
B They bind to receptor proteins on the outside of the plasma membrane
C They cross the plasma membrane by using a protein carrier mechanism.
D They bind to receptors in the cell cytoplasm or nucleus
ANSWER:D
1086 Which endocrine organ produces “releasing hormones” and “inhibitory hormones”?
A thyroid
B anterior pituitary
C hypothalamus
D thalamus
ANSWER:C
1087 What hormone is produced by the parafollicular cells of the thyroid gland?
A Parathyroid hormone
B Calcitonin

C Thyroid hormone
D Thyroxine
ANSWER:B
1088 What hormones are produced by the adrenal medulla?
A Epinephrine and norepinephrine
B Insulin and glucagon
C Aldosterone and erythropoietin
D Testosterone and estrogen
ANSWER:A
1089 What is one mechanism of hormone action?
A They act as second messengers in the cytoplasm
B They act as enzymes for reactions
C They act as receptor proteins
D They activate genes in the nucleus
ANSWER:D
1090 What is the effect of ADH (antidiuretic hormone)?
A allows walls of collecting duct to become permeable to water
B inhibits the reabsorption of Na+
C causes an increase in the volume of urine produced
D it promotes diuresis
ANSWER:A
1091 Which one of the following is NOT true of peptide hormones?
A they are water soluble
B they are derived from amino acids
C their receptors are located in the cell cytoplasm
D they are transported dissolved in blood
ANSWER:C
1092 Which hormones are produced by the adrenal medulla?
A gonadocorticoids
B steroid hormones
C mineralocorticoids
D catecholamines
ANSWER:D

1093 Which structure integrates the activities of the endocrine system and the ner- vous system?
A the hypothalamus
B the thalamus
C the posterior pituitary
D the anterior pituitary
ANSWER:A
1094 Which structure produces ADH and oxytocin?
A the thalamus
B the anterior pituitary
C the hypothalamus
D the posterior pituitary
ANSWER:C
1095 Which structure is composed of glandular epithelial tissue?
A the thalamus
B the anterior pituitary
C the posterior pituitary
D the hypothalamus
ANSWER:B
1096 Which hormone is the one made in greatest quantity by the thyroid gland?
A calcitonin
B thyroid stimulating hormone
C tri-iodothyronine
D thyroxine
ANSWER:D
1097 Which of the following organ(s) are NOT endocrine organs?
A renal
B adrenal
C thyroid
D parathyroid
ANSWER:A
1098 To which group of hormones does aldosterone belong?
A catecholamines
B glucocorticoids

C mineralocorticoids
D gonadocorticoids
ANSWER:C
1099 Where are the receptors for almost all of the amino acid derived hormones located?
A on the mitochondria
B in the nucleus
C on the outside of the plasma membrane
D on the inside of the plasma membrane
ANSWER:C
1100 Which structure produces the hormones ADH and oxytocin?
A the posterior pituitary
B the anterior pituitary
C the thalamus
D the hypothalamus
ANSWER:D
1101 The hypothalamus produces “releasing hormones”. What do these releasing hormones do?
A They direct the posterior pituitary to release hormones.
B They direct the anterior pituitary to release hormones.
C They direct the gonads to release hormones.
D They act as “second messengers” when hormones bind to their receptor site.
ANSWER:B
1102 What is the role of glucagon and insulin?
A glucagon raises blood glucose level and inhibits gluconeogenesis.
B glucagon lowers blood glucose level and stimulates glycogenolysis.
C insulin raises blood glucose level and stimulates gluconeogenesis
D insulin lowers blood glucose level inhibits glycogenolysis.
ANSWER:D
1103 Which of the following hormones CANNOT cross the plasma membrane?
A sex hormones
B amino acid based hormones
C thyroid hormones
D steroids
ANSWER:B

1104 Which of the following “controls” the endocrine system?
A the posterior pituitary
B the thalamus
C the anterior pituitary
D the hypothalamus
ANSWER:D
1105 Which part of the pituitary gland is comprised of neural tissue?
A the posterior pituitary
B the pars intermedia
C the adenohypophysis
D the anterior pituitary
ANSWER:A
1106 The adrenal medulla produces which of the following?
A weak androgens
B mineralocorticoids
C testosterone and oestrogen
D epinephrine and norepinephrine
ANSWER:D
1107 What effect does insulin have?
A it increases metabolic rate
B it causes the breakdown of glycogen to glucose
C it lowers blood sugar level
D it stimulates gluconeogenesis
ANSWER:C
1108
Which of the following is a substantial difference between amino acid based hormones and
steroid hormones?
A
endocrine glands release steroid hormones while amino acid hormones are released from
exocrine glands.
B amino acid hormones are circulating hormones while steroid hormones are local hormones.
C amino acid hormones are fat soluble while steroid hormones are not.
D steroid hormones can pass through the plasma membrane while amino acid hormones cannot.

ANSWER:D
1109 What type of molecule is cAMP, or what role does it play?
A a second messenger
B an amino acid based hormone
C a catecholamine
D a steroid hormone
ANSWER:A
1110 Which one is NOT a mode of action of hormones on their target cell?
A a hormone may stimulate the synthesis of an enzyme in the target cell.
B a hormone may activate an enzyme by altering its shape.
C a hormone may deactivate an enzyme by altering its structure.
D some hormones are enzymes that promote a chemical reaction in a cell.
ANSWER:D
1111 What may be correctly said about oxytocin and antidiuretic hormone?
A they are made and released from the posterior pituitary
B they are made in the hypothalamus and stored and released from the poste- rior pituitary
C they are made in the hypothalamus and stored and released from the ante- rior pituitary
D they are made and released from the anterior pituitary
ANSWER:B
1112 Which hormones are produced by the adrenal medulla?
A glucocorticoids
B mineralocorticoids
C adrenalin and noradrenalin
D gonadocorticoids
ANSWER:C
1113 Which cells produce insulin?
A the acini cells of the pancreas
B parafollicular cells of the thymus
C alpha cells of the islets of Langerhans
D beta cells of the islets of Langerhans
ANSWER:D
1114 What are the two parts of the pituitary gland known as?
A the thalamus and the hypothalamus

B anterior and posterior
C alpha and beta cells
D cortex and medulla
ANSWER:B
1115 Which of the following could be a definition of a hormone?
A chemicals released to communicate between adjacent cells in contact.
B a chemical messenger released into blood to coordinate activities in distant tissues.
C a chemical messenger released by a neurone at a synapse.
D a chemical messenger in the extracellular fluid between cells of a single tissue.
ANSWER:B
1116 Which two hormones are stored in the posterior pituitary prior to their release?
A luteinising hormone and follicle stimulating hormone
B adrenalin and noradrenalin
C calcitonin and calcitriol
D oxytocin and antidiuretic hormone
ANSWER:D
1117 What is a function of calcitonin?
A accelerating Ca+ release from bone
B stimulating Ca+ excretion by the kidneys
C reducing Ca+ deposition in bone
D stimulating the formation of calcitriol in the kidneys
ANSWER:B
1118 The hormones thyroxine and tri-iodothyronine contain which element?
A cobalt
B iron
C iodine
D manganese
ANSWER:C
1119 What is produced in the adrenal cortex?
A cholesterol
B catecholamines
C adrenalin and noradrenalin
D corticosteroids

ANSWER:D
1120 What is the function of insulin?
A enhance the transport of glucose through the plasma membrane into the cell
B promote glycogenolysis
C promote gluconeogenesis
D to raise blood sugar level
ANSWER:A
1121 Which of the following statements about endocrine hormones is always true?
A They are secreted by neurones
B They are derived from amino acids
C They are produced by exocrine glands
D They are released into the bloodstream
ANSWER:D
1122 Which of the following statements about corticosteroids is true?
A They may also act as neurotransmitters
B They are transported dissolved in blood
C They are produced by the adrenal gland
D They are amino acid derivatives
ANSWER:C
1123 Which structure produces epinephrine and norepinephrine?
A adrenal pelvis
B the anterior pituitary
C adrenal medulla
D adrenal cortex
ANSWER:C
1124 Which cells produce insulin?
A the acini
B the alpha cells
C the beta cells
D the islets of Langerhans
ANSWER:C
1125 What is the function of erythropoietin (EPO)?
A stimulate bone marrow to produce red blood cells

B decrease the plasma concentration of Ca++
C increase the plasma concentration of Ca++
D to raise blood sugar level
ANSWER:A
1126 Endocrine communication involves hormones of two types
A first messengers and second messengers
B steroid hormones and amino acid based hormones
C amino acid derivatives and peptide hormones
D peptide hormones and corticosteroids
ANSWER:B
1127 Which of the following statements is FALSE?
A peptide hormones are not able to penetrate the cell membrane
B thyroid hormone can cross the membrane
C steroid hormones bind to receptors on the outside of the cell membrane
D catecholamines are not lipid soluble
ANSWER:C
1128 The posterior pituitary does which one of the following?
A produces growth hormone, prolactin and tropic hormones
B secretes regulatory hormones that control endocrine cells in the anterior pituitary
C exerts neural control over other endocrine glands
D stores and releases oxytocin and antidiuretic hormone
ANSWER:D
1129 What does insulin do?
A it lowers blood sugar level
B it causes the breakdown of glycogen to glucose
C it increases metabolic rate
D it hydrolyses glucose into ATP
ANSWER:A
1130 An amino acid based hormone binds to its receptor. This has the effect of:
A activating an enzyme to produce cAMP
B causing it to diffuse through the cell to trigger a cascade of reactions
C activate a G-protein
D allowing it to move along the membrane

ANSWER:C
1131
The structure that secretes regulatory hormones that control the pituitary gland is known as
the:
A hypothalamus
B hypophysis
C hypothyroid
D hypothymus
ANSWER:A
1132 What is the difference between endocrine glands and exocrine glands?
A endocrine glands produce hormones whereas exocrine glands do not.
B exocrine glands secrete into the blood stream whereas endocrine glands do not.
C
endocrine glands are controlled by the autonomic nervous system whereas exocrine glands
are not.
D
exocrine glands secrete steroid hormones whereas endocrine glands secrete amino acid-based
hormones.
ANSWER:A
1133 By what other name is adrenaline also known?
A noradrenaline
B epinephrine
C androgen
D ANP
ANSWER:B
1134 Which hormones does the pancreas produce?
A epinephrine and norepinephrine.
B oxytocin and antidiuretic hormone.
C glucagon and insulin.
D glucocorticoids and aldosterone
ANSWER:C
1135 Which statement is NOT CORRECT about hormones:
A They are chemical substances that alter cell activity
B They regulate metabolic function of other cells in the body
C Steroid hormones are amino acid-based and are synthesized from cholesterol
D They are produced in glands and transported via the blood stream

ANSWER:C
1136 What does the anterior lobe of the pituitary gland synthesise and release?
A Growth hormone-releasing hormone
B Corticotropin-releasing hormone
C Thyroid-stimulating hormone
D Gonadotropin-releasing hormone
ANSWER:C
1137 How do calcitonin or parathyroid hormone control blood calcium levels?
A Calcitonin acts to increase blood calcium levels
B Parathyroid hormone release is inhibited by increased calcium levels
C Parathyroid hormone stimulates bone resorbing cells to take up calcium
D Calcitonin inhibits parathyroid hormone
ANSWER:B
1138 What controls the blood glucose level?
A The action of insulin
B The action of glucagon
C The action of insulin and glucagon
D The action of insulin, glucagon and glycogen
ANSWER:C
1139 Complete the following sentence correctly: The hypothalamus:
A Is the major link between the nervous and the endocrine systems.
B Is situated in the brain superior to the thalamus.
C Produces a hormone that stimulates the thyroid gland.
D Does not produce anti-diuretic hormone (ADH).
ANSWER:A
1140 Which of the following secretes growth hormone?
A the adrenal glands
B The thyroid gland
C The posterior lobe of the pituitary gland
D The anterior lobe of the pituitary gland
ANSWER:D
1141 Blood calcium levels are controlled by hormones from which structures?
A The parathyroid glands and the thyroid gland

B The anterior lobe of the pituitary gland and the thyroid gland
C The parathyroid glands and the anterior lobe of the pituitary gland
D The adrenal cortex and the hypothalamus.
ANSWER:A
1142
The hormones known as “catecholamines” (adrenaline, noradrenaline and dopamine) are not
lipid soluble. Therefore their receptor sites are:
A On the inside of the plasma membrane.
B On the outside of the plasma membrane.
C In the cell cytoplasm.
D In the cell nucleus.
ANSWER:B
1143 Where are all of the glomeruli of the kidney located?
A in the medulla
B in the columns
C in the pyramids
D in the cortex
ANSWER:D
1144 What structure does the blood from the afferent arteriole enter?
A the peritubular capillaries
B the vasa recta
C the glomerulus
D Bowman’s capsule
ANSWER:C
1145 Which part of the nephron is impermeable to water?
A Proximal convoluted tubule
B Distal convoluted tubule in the presence of ADH
C Ascending limb of the loop of Henle
D Descending limb of the loop of Henle
ANSWER:C
1146 Which of the following happens as we descend deeper into the kidney medulla?
A the concentration of the interstitial fluid doesn’t change
B the concentration of the interstitial fluid increases
C the concentration of the filtrate within the tubule increases

D the concentration of the interstitial fluid decreases
ANSWER:B
1147 What molecule catalyses the formation of angiotensin I?
A carbonic anhydrase
B calcitriol
C erythropoietin
D renin
ANSWER:D
1148 How are cortical nephrons different from juxtamedullary nephrons?
A cortical nephrons lie almost entirely outside the renal medulla.
B cortical nephrons have an associated vasa recta.
C cortical nephrons have a longer tubule.
D there are fewer cortical nephrons.
ANSWER:A
1149 Which part of the renal tubule is impermeable to water?
A the ascending limb of the loop of Henle
B the collecting duct
C the proximal convoluted tubule
D the thin portion of the loop of Henle
ANSWER:A
1150
In the glomerulus, what is the method by which solutes are transferred from the blood to the
Bowman’s capsule?
A diffusion
B active transport
C secretion
D filtration
ANSWER:D
1151 How does the juxtaglomerular apparatus respond when systemic blood pres- sure is too high?
A the juxtaglomerular cells send a message to the afferent arteriole to dilate.
B The macula densa sends a message to the efferent arteriole to constrict.
C The macula densa sends a message to the afferent arteriole to constrict.
D The granular cells release renin which causes systemic arterioles to constrict.

ANSWER:C
1152 What is the body’s response to a rise in blood plasma osmotic pressure?
A The anterior pituitary releases ADH which makes the renal tubule perme- able to water.
B The posterior pituitary releases ADH which makes the renal tubule perme- able to water.
C The juxtaglomerular apparatus releases renin which promotes diuresis
D The glomerular filtration rate increases so more urine is produced.
ANSWER:B
1153 What is the kidney tubule’s response to a rise in blood pH?
A Bicarbonate ions are created from carbonic acid and absorbed into the blood.
B
Hydronium ions are secreted into the filtrate, where they are buffered by bicarbonate ions in
the filtrate.
C
Bicarbonate ions are secreted into the filtrate, while hydronium ions are absorbed from the
filtrate into the blood.
D
Hydronium ions are secreted into the filtrate, while bicarbonate ions are absorbed from the
filtrate into the blood.
ANSWER:C
1154 How does the composition of the filtrate change as it travels through the loop of Henle?
A
In the ascending limb, the volume decreases and in the descending limb, the concentration
increases.
B
In the descending limb, the volume decreases and in the ascending limb, the concentration
decreases.
C
In the descending limb, the volume decreases and in the ascending limb, the concentration
increases.
D
In the ascending limb, the volume decreases and in the descending limb, the concentration
decreases.
ANSWER:B
1155
What is the entry point to the kidney for the renal artery, renal vein, lymphat- ics and nerves
called?
A renal pyramid
B renal hilus
C renal capsule
D renal column
ANSWER:B

1156 In what part of the kidney are the glomeruli located?
A In the cortex
B In the medulla
C In the hilus
D In the minor calyces
ANSWER:A
1157
Four sections of the vasculature of the kidney tubule are listed below. Which one lists them
in correct order of blood flow from left to right?
A efferent arteriole, glomerulus, afferent arteriole, peritubular capillaries.
B afferent arteriole, glomerulus, efferent arteriole, peritubular capillaries.
C peritubular capillaries, afferent arteriole, glomerulus, efferent arteriole.
D glomerulus, afferent arteriole, peritubular capillaries, efferent venule.
ANSWER:B
1158
What method does the glomerulus of the kidney nephron use to remove the dissolved
substances from the blood to the filtrate?
A active transport
B diffusion along the concentration gradient
C high hydrostatic pressure
D osmosis
ANSWER:C
1159
If the glomerular filtration rate is too high, the macula densa sends a message to the afferent
arteriole. What is the effect of this message?
A granular cells of arteriole walls release renin
B afferent arteriole dilates
C it inhibits the action of ATP and adenosine on the afferent arteriole
D afferent arteriole constricts
ANSWER:D
1160 What is the effect of antidiuretic hormone on the kidney tubules?
A It causes Na+ to be absorbed from the filtrate into the tubular cells.
B It causes the concentration of urine to decrease.
C It causes the filtrate volume to increase.
D It causes the walls of the collecting duct to become permeable to water.
ANSWER:D

1161
The descending limb of the Loop of Henle is permeable to water so water dif- fuses out of
the descending limb into the interstitial fluiWhat happens to this water?
A it flows through the renal papillae into the minor calyces to become urine.
B it diffuses into the ascending limb of the Loop of Henle.
C it diffuses into the peritubular capillaries and ascending vasa recta for return to the blood.
D it diffuses into the filtrate for elimination from the body.
ANSWER:C
1162
The concentration of blood is 0 to 00 mosmol/L, but may rise to 00 mosmol/L in which
situation?
A In the vasa recta of the kidney
B In severe dehydration
C In the peritubular capillaries of the kidney
D In severe over-hydration
ANSWER:A
1163 Which one of the following is NOT produced by the kidneys?
A Aldosterone
B Renin
C Erythropoietin
D Calcitriol
ANSWER:A
1164
What is the collective term applied to the proximal and distal convoluted tubules, the loop of
Henle (i.e. the nephron loop) and the glomerular capsule?
A The renal corpuscle
B The renal tubule
C The nephron
D The renal capsule
ANSWER:B
1165
When systemic blood pressure increases, how does the kidney respond to maintain
glomerular filtration rate?
A The afferent arteriole dilates
B The efferent arteriole constricts
C The efferent arteriole dilates
D The afferent arteriole constricts

ANSWER:D
1166 What may correctly be said of the juxtaglomerular apparatus (or complex)?
A The juxtaglomerular cells are chemoreceptors
B The granular cells are chemoreceptors
C The macula densa cells are chemoreceptors
D The macula densa cells are mechanoreceptors
ANSWER:C
1167
What part of the nephron performs the majority of the reabsorption of materi- als from the
filtrate?
A The Bowman’s capsule and glomerulus
B The loop of Henle (the nephron loop)
C The distal convoluted tubule and collecting duct
D The proximal convoluted tubule
ANSWER:D
1168 What is the name of the tube that connects the bladder to the kidney?
A renal tubule
B ureter
C urethra
D collecting duct
ANSWER:B
1169 Which of the following may be said of the renal medulla?
A it is the more superficial part of the kidney.
B it contains all of the glomeruli.
C it produces adrenaline and noradrenaline.
D it contains the pyramids and columns.
ANSWER:D
1170 What influences and structures facilitate blood filtration in the renal corpuscle?
A high osmotic pressure in the capillaries and sinusoidal capillaries
B high hydrostatic pressure in the capillaries and fenestrated capillaries
C high osmotic pressure in the capillaries and fenestrated capillaries
D high hydrostatic pressure in the capillaries and sinusoidal capillaries
ANSWER:B
1171 In which part of the nephron does most of the reabsorption of water and sol- utes occur?

A the collecting duct
B the nephron loop (loop of Henle)
C the vasa recta
D the proximal convoluted tubule
ANSWER:D
1172 Which statement about kidney anatomy is correct?
A The cortex is superficial to the medulla and contains all of the glomeruli.
B The cortex is deep to the medulla and contains the collecting tubules.
C The pyramids are in the cortex and contain the collecting tubules.
D The pyramids are in the medulla and contain all of the glomeruli.
ANSWER:A
1173 The renal tubule of the nephron includes which of the following structures?
A Proximal convoluted tubule, vasa recta, Bowman’s capsule, collecting duct.
B
Distal convoluted tubule, ascending limb of Loop of Henle, Bowman’s capsule, proximal
convoluted tubule.
C
Descending limb of Loop of Henle, collecting duct, distal convoluted tubule, ascending limb
of Loop of Henle.
D Glomerulus, proximal convoluted tubule, distal convoluted tubule, Bowman’s capsule.
ANSWER:B
1174 How does the descending limb of the loop of Henle differ from the ascending limb?
A the descending limb is impermeable to water but permeable to sodium chloride.
B the ascending limb is permeable to water but impermeable to sodium chloride.
C the descending limb is permeable to water but impermeable to sodium chloride.
D the ascending limb is permeable to both water and to sodium chloride.
ANSWER:C
1175
What effect is achieved by having an arteriole that supplies blood and another that drains
blood from the glomerulus?
A oxygen rich blood can be supplied to the nephron after blood leaves the glomerulus
B the blood pressure within the glomerulus can be manipulated.
C reabsorption of water and nutrients from the filtrate is facilitated.
D the concentration gradient within the kidney’s medulla can be maintained.
ANSWER:B
1176 What is the resulting effect of renin being released by the kidney?

A Angiotensin II is formed
B aldosterone is released
C macula densa sends paracrine message to afferent arterioles.
D efferent arterioles are constricted
ANSWER:A
1177 What is the effect on the kidney caused by increasing the release of ADH?
A the collecting duct becomes more permeable to water
B the ascending limb of the loop of Henle becomes impermeable to water
C the descending limb of the loop of Henle becomes permeable to water
D the collecting duct becomes impermeable to water.
ANSWER:A
1178 The kidneys produce all of the following EXCEPT one. Which one?
A erythropoietin
B angiotensinogen
C hydronium ions
D bicarbonate ions
ANSWER:B
1179 Which one of the following is part of the renal tubule?
A glomerulus
B vasa recta
C collecting duct
D macula densa
ANSWER:D
1180 From which part of the nephron is the greatest proportion of Na+ absorbed from the filtrate?
A the Bowman’s capsule (i.e. renal capsule)
B the proximal convoluted tubule
C the ascending limb of the loop of Henle
D the distal convoluted tubule
ANSWER:B
1181 Complete the following sentence correctly. Angiotensin II
A stimulates the adrenal glands to release aldosterone
B increases potassium reabsorption from the filtrate

C increases sodium (Na+) excretion at the kidneys.
D reduces our thirst
ANSWER:A
1182 What part of the renal tubule is NOT able to reabsorb water?
A the descending limb of the loop of Henle
B the proximal convoluted tubule
C the ascending limb of the loop of Henle
D the distal convoluted tubule
ANSWER:C
1183 The influence (or influences) that drives blood filtration in the kidney is
A
difference in osmolarity between blood in the glomerulus and filtrate in the Bowman’s
capsule
B
fluid pressure difference between blood in the glomerulus and filtrate in the Bowman’s
capsule
C
the osmotic pressure difference between blood in the glomerulus and fil- trate in the
Bowman’s capsule
D diffusion along the concentration gradient between blood and filtrate, and active transport
ANSWER:B
1184 What does the renal system consists of?
A kidneys, urethra, bladder, ureter.
B adrenal glands, kidneys, ureter, urethra, bladder.
C adrenal glands, kidneys, ureters, urethra, bladder.
D kidneys, urethra, bladder, ureters.
ANSWER:D
1185 Which of the statements about the capillaries of the glomerulus is NOT true?
A glomerular capillaries are fenestrated (i.e. porous).
B blood enters and leaves the glomerulus via arterioles.
C
the blood pressure in glomerular capillaries is higher ( mmHg) than in the capillaries in the
rest of the body.
D glomerular capillaries have smooth muscle in their walls.
ANSWER:D
1186 Which hormone causes increased sodium reabsorption in the kidney?
A angiotensin I

B antidiuretic hormone
C vasopressin
D aldosterone
ANSWER:D
1187 Which type of anti-hypertensive drug aims to prevent vasoconstriction?
A beta blockers
B diuretics
C ACE inhibitors
D calcium channel blockers
ANSWER:D
1188 The kidney produces all but one of the following. Which one?
A calcitriol
B atrial natriuretic peptide
C renin
D bicarbonate ions
ANSWER:B
1189 The filtrate that is formed in the kidney contains all of the following except one. Which one?
A metabolic wastes
B electrolytes
C plasma proteins
D nutrients
ANSWER:C
1190
What is the term applied to the first process in urine formation, where some components of
blood pass into the Bowman’s capsule?
A filtration
B active transport
C dialysis
D osmosis
ANSWER:A
1191 To what does the “juxtaglomerular apparatus” refer?
A to those nephrons whose loop of Henle penetrate deep into the medulla

B
to the lamina densa and podocytes that form filtration slits around the cap- illaries of the
glomerulus
C to the capillaries that surround the loop of Henle of juxtamedullary nephrons
D to certain cells of the distal convoluted tubule where it touches the afferent arteriole.
ANSWER:D
1192 In what part (or parts) of the renal tubule reabsorb the least material from the filtrate?
A the distal convoluted tubule
B the loop of Henle
C the proximal convoluted tubule
D the distal convoluted tubule and the collecting duct together
ANSWER:D
1193 Which statement about kidney anatomy is correct? Renal pyramids are in the:
A medulla and end in a papilla that empties into a minor calyx
B medulla and end in a column that empties into a major calyx
C cortex and end in a papilla that empties into a minor calyx
D cortex and end in a column that empties into a minor calyx
ANSWER:A
1194 The kidneys produce all of the following except one. Which one?
A the enzyme renin
B the hormone erythropoietin
C antidiuretic hormone
D the vitamin calcitriol
ANSWER:C
1195
In the nephron, if the afferent arteriole dilates and the efferent arteriole con- stricts, which of
the following would be true?
A The glomerular filtration rate would decrease.
B The pressure in the glomerulus will decrease.
C The absorption of sodium and chloride ions form the filtrate would increase.
D Before these events, the granular cells would have released renin.
ANSWER:D
1196 The majority of material reabsorbed from the filtrate is reabsorbed from the renal tubule:
A after the Loop of Henle
B in the descending limb of the Loop of Henle

C before the Loop of Henle
D in the ascending limb of the Loop of Henle
ANSWER:C
1197 What effect does aldosterone have?
A increases the absorption of Na+ from the kidney tubules.
B makes the kidney tubules more permeable to water.
C catalyses the formation of angiotensin I.
D blocks the release of ADH
ANSWER:A
1198 The kidneys produce all of the following EXCEPT one. Which one?
A erythropoietin
B aldosterone
C renin
D active vitamin D
ANSWER:B
1199 Which statement about the descending limb of the Loop of Henle is true?
A it is freely permeable to water
B it is impermeable to water
C it is impermeable to water when ANP is present
D it is permeable to water when ADH is present
ANSWER:A
1200 In which structure does blood filtration in the kidney occur? The:
A macula densa
B renal corpuscle
C major calyx
D vasa recta
ANSWER:B
1201 From which part of the nephron is the greatest proportion of Na+ absorbed from the filtrate?
A the proximal convoluted tubule
B the ascending limb of the loop of Henle
C the distal convoluted tubule
D the collecting duct in the presence of aldosterone

ANSWER:A
1202 What is the purpose of ANP in urine production?
A stimulate the reabsorption of Na+
B stimulate the reabsorption of Ca++
C inhibit the reabsorption of Na+
D stimulate the reabsorption of water
ANSWER:C
1203
By what process(es) do water and solutes move from blood in the glomerulus into the
Bowman’s capsule?
A diffusion
B osmosis and diffusion
C filtration
D dialysis
ANSWER:C
1204 Complete the following sentence correctly. Antidiuretic hormone
A stimulates our thirst
B causes the wall of the collecting duct of the nephron to increase in perme- ability to water.
C increases sodium (Na+) excretion at the kidneys.
D stimulates the adrenal glands to release aldosterone
ANSWER:B
1205 Complete the following sentence correctly. Angiotensin II
A stimulates the adrenal glands to release aldosterone
B causes the wall of the collecting duct of the nephron to increase in perme- ability to water
C increases sodium (Na+) excretion at the kidneys.
D reduces our thirst
ANSWER:A
1206 Complete the following sentence correctly. Atrial natriuretic peptide
A causes the wall of the collecting duct of the nephron to increase in perme- ability to water
B increases sodium (Na+) excretion at the kidneys.
C causes peripheral vasoconstriction
D stimulates the adrenal glands to release aldosterone
ANSWER:B

1207
What is the place where the arteries, veins, lymphatics and nerves enter or leave the kidney
called?
A The carina
B The reno-atrio notch
C The renal pelvis
D The hilus
ANSWER:D
1208 The nephrons of the kidney consist of
A Bowman’s capsule, a loop of Henle, a collecting duct and a renal tubule
B a juxtaglomerular apparatus and collecting duct
C a glomerulus and a juxtaglomerular apparatus
D a glomerulus, a proximal convoluted tubule, loop of Henle, and a distal convoluted tubule
ANSWER:D
1209 What part of the renal tubule does NOT reabsorb water?
A the juxtaglomerular apparatus
B the ureter
C the ascending limb of the loop of Henle
D the collecting duct
ANSWER:C
1210 The influence(s) that drives blood filtration in the kidney is
A
dialysis through a semi-permeable membrane due to the different osmolar- ity of blood and
filtrate
B fluid pressure difference between blood and filtrate
C osmotic pressure difference between blood and filtrate
D diffusion along a concentration gradient between blood and filtrate, and active transport
ANSWER:B
1211 What does the presence of aldosterone in the blood cause?
A calcium to be absorbed from the DCT
B the collecting duct to become permeable to water
C more bicarbonate to be formed in the tubule cells
D more sodium to be reabsorbed from the DCT
ANSWER:D
1212 What does the term “oliguria” refer to?

A a daily urine production of much more than litres
B a daily urine production of less than 00 ml
C production of less than 0 ml of urine in a day
D the condition of excessive concentration of urea in the blood
ANSWER:B
1213 What is the role of aldosterone?
A to convert angiotensinogen into angiotensin I.
B to inhibit the absorption of Na+.
C to promote the absorption of Na+.
D to promote the absorption of Ca++.
ANSWER:C
1214 One way to increase the glomerular filtration rate is to dilate:
A the afferent arteriole and to constrict the efferent arteriole.
B the efferent arteriole and to constrict the afferent arteriole.
C both the afferent arteriole and the efferent arteriole.
D the efferent arteriole and to increase the permeability of the capillary endothelium.
ANSWER:A
1215 The nephron of the kidney consists of which of the following structures?
A glomerulus, renal tubule and collecting duct.
B bowman’s capsule, proximal convoluted tubule, loop of Henle, and distal convoluted tubule.
C glomerulus and renal tubule
D renal tubule and collecting duct.
ANSWER:C
1216 What name is given to the blood vessel that drains blood from the glomerulus after filtration?
A vasa recta
B afferent arteriole
C efferent arteriole
D efferent vein
ANSWER:C
1217
Through which structure must the filtrate move to enter the Bowman’s capsule from the
glomerulus?

A The capillary endothelial cell walls
B The capillary endothelial wall and basement membrane
C The capillary endothelial wall, basement membrane and podocytes
D The capillary endothelial wall, basement membrane, podocytes and the vasa recta
ANSWER:C
1218 What is the function of angiotensin II?
A causes constriction of systemic arteries.
B causes the collecting ducts to become permeable to water.
C causes the formation of atrial natriuretic hormone.
D causes constriction of the efferent arterioles.
ANSWER:A
1219 What is the term meaning the production of urine?
A oliguria
B diuresis
C hypouria
D anuria
ANSWER:B
1220 Which section of the renal tubule is permeable to urea?
A descending limb of the loop of Henle.
B ascending limb of the loop of Henle.
C collecting duct in the presence of aldosterone.
D proximal convoluted tubule.
ANSWER:D
1221
What name is given to the blood vessel that connects the capillaries of the glomerulus to the
vasa recta?
A macula densa
B afferent arteriole
C efferent arteriole
D afferent vein
ANSWER:C
1222 Which of the following chemicals is produced by the kidney?
A angiotensinogen
B bicarbonate ions

C sodium ions
D vitamin C
ANSWER:B
1223
What could be concluded of a person who (during the previous three hours) has produced a
total of 00 ml of urine that is bright yellow and has a strong (but not unpleasant) odour?
A They are well hydrated.
B Their urine will have a high specific gravity.
C They are an uncontrolled diabetic.
D They have more than the usual concentration of bilirubin in their urine.
ANSWER:B
1224 In which section of the renal tubule is most water reabsorbed?
A descending limb of the loop of Henle.
B ascending limb of the loop of Henle.
C collecting duct.
D proximal convoluted tubule.
ANSWER:D
1225 Under what conditions will the kidney produce concentrated urine? If:
A glomerular filtration rate is low.
B glomerular filtration rate is high.
C atrial natriuretic peptide and aldosterone are present in blood.
D antidiuretic hormone and aldosterone are present in the blood.
ANSWER:D
1226 Which one of the following is a function of the renal system?
A produce bile
B produce the enzyme renin
C produce the hormone aldosterone
D produce vitamin K
ANSWER:B
1227 The functional unit of the kidney that filters blood and produces urine is called the:
A medulla
B glomerulus
C neurone

D nephron
ANSWER:D
1228
In the kidney, the filtrate passes through several structures on its way to becoming urine.
Which of the following lists presents these structures in the correct order?
A
collecting duct, glomerulus, proximal convoluted tubule, distal convoluted tubule, loop of
Henle.
B
proximal convoluted tubule, collecting duct, glomerulus, loop of Henle, distal convoluted
tubule.
C
glomerulus, proximal convoluted tubule, loop of Henle, distal convoluted tubule, collecting
duct.
D
glomerulus, collecting duct, proximal convoluted tubule, distal convoluted tubule, loop of
Henle.
ANSWER:C
1229 Which of the following statements about the structures in the loop of Henle is correct?
A its ascending limb is permeable to water
B its descending limb is impermeable to urea
C its descending limb is impermeable to water
D its ascending limb is impermeable to solutes
ANSWER:B
1230 What is renin?
A an enzyme released by the juxtaglomerular cells of the kidney when arte- rial pressure falls.
B it catalyses the formation of angiotensin II in the lungs.
C it is a rapid acting, intense vasoconstrictor of arterioles.
D a protein that stimulates the adrenal glands to release aldosterone.
ANSWER:A
1231 Which of the following are organic wastes produced by the body?
A Uric acid and ammonium ions
B Amino acids and potassium ions
C Albumin and globulin
D Urea and sodium ions
ANSWER:A
1232 What is indicated if the specific gravity of a patient’s urine is high?

A the patient has kidney disease
B the urine’s concentration is high
C the patient is well hydrated
D the urine density is low
ANSWER:B
1233 Which ion does aldosterone stimulate the kidneys to reabsorb?
A calcium
B sodium
C potassium
D bicarbonate
ANSWER:B
1234 If a urine specific gravity was measured to be .0, an interpretation would be that:
A the person was dehydrated
B the person was well hydrated
C the urine sample had a density less than water
D the urine was dilute
ANSWER:A
1235 If a urine specific gravity was measured to be .00, an interpretation would be that:
A the person was dehydrated
B the person was well hydrated
C the urine sample had a density less than water
D the urine was concentrated
ANSWER:B
1236 Which of the following statements about urine specific gravity is WRONG?
A if urine specific gravity is .00, the urine is dilute.
B a urine specific gravity value of .0 = 0 mmol/L.
C if urine specific gravity is .00, the person is dehydrated.
D a specific gravity value of .00 is equal to a urine density of .00 g/ml.
ANSWER:B
1237
Given that the specific gravity of a urine sample is .00, which of the follow- ing statements is
correct?
A the patient is dehydrated
B the sample contains .00 mmol/L of dissolved particles

C the sample contains .00 millimole of dissolved solutes
D the urine has a density of .00 g/ml
ANSWER:D
1238 To which of the following would the term “white cell” NOT be applied?
A erythrocyte
B leucocyte
C lymphocyte
D monocyte
ANSWER:A
1239 In the haemostasis process, what forms as a result of the extrinsic and intrinsic pathways?
A fibrin
B thrombin
C a platelet plug
D prothrombinase
ANSWER:D
1240
The blood group known as the ABO system is based on the presence of what proteins on
blood cells?
A antibodies
B antigens
C agglutinins
D immunoglobulins
ANSWER:B
1241 What is found in blood serum that is also in blood plasma?
A blood cells
B platelets
C plasma proteins
D clotting factors
ANSWER:C
1242 What is the term “formed elements” used to mean in a description of blood?
A white blood cells, red blood cells and platelets
B blood plasma
C blood serum
D the clotting factors in blood

ANSWER:A
1243 What is the SECOND step in the three phases of haemostasis listed below?
A The vascular phase
B The intrinsic pathway
C The extrinsic pathway
D The platelet phase
ANSWER:D
1244 What type of blood may a patient with blood type “B+” be infused with? Any blood that is
A positive for rhesus antigen D
B negative for rhesus antigen D
C negative for antigen B
D negative for antigen A
ANSWER:D
1245 What is the first process that occurs after a blood vessel is damaged?
A coagulation
B platelet plug formation
C vasoconstriction
D haemolysis
ANSWER:C
1246 Which blood cells are involved in protecting the body from pathogens and for- eign cells?
A erythrocytes
B leucocytes
C platelets
D haemoglobin
ANSWER:B
1247 Which is the most abundant plasma protein?
A alpha- and beta- globulins
B albumin
C mitochondria
D haemoglobin
ANSWER:B
1248 Which characteristic of blood refers to the concentration of solutes?
A salinity

B pH
C osmolality
D viscosity
ANSWER:C
1249 Which type of white blood cell is responsible for engulfing pathogens during phagocytosis?
A thrombocyte
B neutrophil
C erythrocyte
D basophil
ANSWER:B
1250 What does “Rhesus positive” refer to?
A The presence of antigen D on the surface of red blood cells
B The final factor involved in blood clotting
C The presence of the rhesus antibody/agglutinin in the blood
D A deficiency of Factor VIII that results in haemophilia
ANSWER:A
1251 What are red blood cells primarily composed of?
A alpha- and beta- globulins
B albumin
C mitochondria
D haemoglobin
ANSWER:D
1252 Which is the LEAST common type of white blood cell?
A lymphocyte
B basophil
C thrombocyte
D neutrophil
ANSWER:B
1253 In the process of haemostasis, which phase involves the intrinsic and extrinsic pathways?
A the platelet phase
B the clot lysis phase
C the vascular phase

D the coagulation phase
ANSWER:D
1254 In haemostasis, which molecule polymerises to become the insoluble blood clot?
A factor X
B thrombin
C fibrin
D plasmin
ANSWER:C
1255 Which enzyme converts fibrinogen to fibrin?
A serotonin
B thrombin
C renin
D secretin
ANSWER:B
1256 Which of the following is NOT a macrophage?
A Kupffer cell
B Monocyte
C Dendrocyte
D Megakaryocyte
ANSWER:D
1257 What can be said about a person who has the “A” antigen on their red blood cells?
A their blood contains anti-B agglutinins
B their blood contains anti-A agglutinins
C their blood contains anti-A and anti-B agglutinins
D their blood contains neither anti-A nor anti-B agglutinins
ANSWER:A
1258 Which one of the following is NOT a plasma protein?
A keratin
B albumin
C ferritin
D globulin
ANSWER:A
1259 What substance is produced by the first step in the blood clotting (coagulation) process?

A thrombin
B prothrombin
C factor X
D prothrombinase
ANSWER:D
1260 Which statement about neutrophils is correct?
A they have no nucleus
B they contain haemoglobin
C they function as a body defence mechanism
D eosinophils are one type of neutrophil
ANSWER:C
1261 What are red blood cells also known as?
A erythrocytes
B thrombocytes
C monocytes
D eosinophils
ANSWER:A
1262 In blood clotting, what activates “factor X”
A prothrombinase
B thrombin
C the extrinsic pathway
D tissue plasminogen activator
ANSWER:C
1263 A person’s blood group is determined by:
A the agglutinogens circulating in their plasma
B the antigens on the surface of their red blood cells
C the antibodies on the surface of their red blood cells
D the agglutinins circulating in their plasma
ANSWER:B
1264 If a blood sample is taken for DNA testing, which of the following would be examined?
A leucocytes
B erythrocytes
C thrombocytes

D plasma proteins
ANSWER:A
1265 What is the major task of red blood cells?
A to transport carbon dioxide
B to ensure haemostasis
C to provide immunity
D to transport oxygen
ANSWER:D
1266 Which cell in the list below is the MOST common white blood cell?
A basophils
B lymphocytes
C monocytes
D neutrophils
ANSWER:D
1267 What substance is the product of the second step in the blood clotting process?
A thrombin
B prothrombin
C prothrombin activator
D fibrin
ANSWER:A
1268 A person whose blood group is “B positive” has which of the following?
A the rhesus D antigen and the B antigen on their rbc, and the anti-A agglutinin.
B the rhesus D antigen and the B antigen on their rbc, and the anti-B agglutinin.
C the rhesus D antigen and the A antigen on their rbc, and the anti-B agglutinin
D no rhesus D antigen and the B antigen on their rbc, and the anti-A agglutinin
ANSWER:A
1269 The role of platelets in blood clotting includes all of the following EXCEPT one. Which one?
A to form a plug in the hole of the damaged blood vessel
B to convert prothrombin to thrombin
C to release chemicals to attract other platelets
D to adhere to exposed collagen fibres in damaged blood vessels
ANSWER:B

1270 If someone’s ABO blood group is “type A”, this means that
A they have the type A antigen on their red blood cells
B their blood contains anti-A agglutinins
C they can receive blood from a type B donor
D they may donate blood to a type B recipient
ANSWER:A
1271 Which statement below about vitamin K is true?
A It is water soluble.
B It is essential for prothrombin production by the liver.
C It is part of the “extrinsic pathway” of formation of prothrombin activator.
D It destroys fibrin so allowing a clot to gradually dissolve.
ANSWER:B
1272 What is the function of the plasma proteins in blood?
A to transport oxygen.
B to regulate electrolyte balance
C to exert osmotic pressure and so help maintain blood volume.
D to function as a non-specific body defence mechanism.
ANSWER:C
1273 The term “formed elements” used in relation to the blood include which of the following?
A fibrinogen.
B white blood cells.
C electrolytes.
D plasma proteins.
ANSWER:B
1274
Which blood cell fits the following description: multi-lobed nucleus, incon- spicuous
cytoplasmic granules, most common type of blood cell except for red blood cells?
A neutrophil
B eosinophil
C basophil
D lymphocyte
ANSWER:A
1275 What constitutes blood plasma?
A whole blood without the formed elements.

B blood without the red blood cells.
C whole blood without blood cells and clotting factors.
D blood minus blood cells and proteins.
ANSWER:A
1276 Which of the following is not a type of white blood cell?
A leucocyte
B eosinophil
C erythrocyte
D neutrophil
ANSWER:C
1277 Which of the following formed elements of the blood is important in the forma- tion of clots?
A erythrocytes
B lymphocytes
C monocytes
D thrombocytes
ANSWER:D
1278 Leucocytes may be correctly described as what?
A cells with nuclei that do not contain haemoglobin.
B cells without nuclei, that contain haemoglobin.
C white blood cells with granules in their cytoplasm.
D neutrophilic.
ANSWER:A
1279 What are lymphocytes? Blood cells that:
A mature and proliferate in the bone marrow.
B contain haemoglobin.
C are involved in the body’s immune response
D mature into macrophages.
ANSWER:C
1280 Which of the following statements about platelets is INCORRECT? They:
A adhere to collagen fibres of damaged tissue
B
release phospholipids which combine with “clotting factors” to produce prothrombin
activator.

C are cell fragments derived from megakayoblasts
D are part of the “extrinsic pathway” for the formation of prothrombin activator.
ANSWER:B
1281 Finish the sentence correctly. Plasma proteins:
A help maintain blood volume due to colloid osmotic pressure.
B are regarded as formed elements of the blood.
C are low molecular weight proteins.
D are part of the blood serum.
ANSWER:A
1282 The colloid osmotic pressure of blood is due to which of the following?
A proteins in the blood
B proteins in the interstitial fluid
C sodium and chloride ions dissolved in blood
D the water component of the blood
ANSWER:A
1283 Which one of the following terms refers to an abnormally low number of white blood cells?
A thrombocytosis
B haemostasis
C leukopenia
D cytokinesis
ANSWER:C
1284 Which of the following three proteins are known as “plasma proteins”?
A albumin, globulin, haemoglobin
B insulin, glucagon, haemoglobin
C fibrin, globulin, albumin
D albumin, fibrinogen, globulin
ANSWER:D
1285 Which are the two most common types of white blood cells?
A neutrophils and lymphocytes
B erythrocytes and neutrophils
C neutrophils and eosinophils
D monocytes and lymphocytes

ANSWER:A
1286 Blood plasma contains “plasma proteins”. Which of the following lists the plasma proteins?
A insulin, kaolin, bilirubin
B cholesterol, urea, glucagon
C Na+, K+, Ca+, Mg+
D albumins, fibrinogen, globulins
ANSWER:D
1287
What causes the blood’s osmotic pressure to be greater than the osmotic pres- sure of the
surrounding interstitial fluid that is outside of the capillaries?
A
there is a higher concentration of sodium and chloride ions in the blood than the interstitial
fluid.
B there is a higher concentration of water in the blood than in the interstitial fluid.
C the plasma proteins in blood.
D the hydrostatic pressure produced by the heart’s contractions.
ANSWER:C
1288 What does the term “neutrophil” refer to?
A An affinity for neutrons.
B An abnormally low number of cells.
C A type of white blood cell.
D An immature cell that will become a neutrocyte.
ANSWER:C
1289 One of the following cells does NOT occur in blooWhich one?
A erythrocytes
B basophils
C leucocytes
D osteocytes
ANSWER:D
1290 What would a person with type A blood also have?
A antibody A
B antigen A
C agglutinin A
D agglutinogen B
ANSWER:B

1291 Blood flow through the heart follows which of the sequences listed below?
A from left atrium, then mitral valve, right ventricle, aorta, left ventricle
B from right atrium, then mitral valve, right ventricle, pulmonary trunk, left ventricle.
C from pulmonary trunk, then tricuspid valve, left atrium, aortic valve, aorta
D from vena cava, then right ventricle, pulmonary trunk, left ventricle, aorta.
ANSWER:D
1292 What feature does cardiac muscle possess that is missing in skeletal muscle?
A striations
B multiple nuclei
C voluntary control
D intercalated discs
ANSWER:D
1293 What is the name of the valve between the left atrium and the left ventricle?
A mitral valve
B tricuspid valve
C semi-lunar valve
D aortic valve
ANSWER:A
1294 What is meant by a diastolic blood pressure of 00 mm Hg?
A the maximum pressure at the start of the aorta during ventricular contraction.
B the minimum pressure at the start of the aorta before the start of a ventricu- lar contraction.
C
the maximum pressure at the start of the aorta and pulmonary trunk during ventricular
contraction.
D The minimum blood pressure measured when resting.
ANSWER:B
1295 What is the main function of mitral valve?
A to increase the pressure inside the left atrium during systole
B to prevent a drop in pressure in the aorta during diastole
C to prevent backflow from left ventricle to left atrium during systole
D to add additional blood from left atrium to left ventricle during atrial systole
ANSWER:C
1296
The Frank-Starling law of the heart describes the proportional relationship between which of
the following pairs?

A Stroke volume and cardiac output
B Stroke volume and end-diastolic volume
C The blood volume in the ventricles and stroke volume
D Systemic vascular resistance and stroke volume
ANSWER:C
1297 What will cause the sinoatrial (SA) node to depolarize more frequently?
A Acetylcholine
B Norepinephrine
C Parasympathetic stimulation
D Vagus nerve
ANSWER:B
1298 How are cardiac cells mechanically attached to each other? By their:
A mitochondria
B intercalated discs
C gap junctions
D sarcolemma
ANSWER:B
1299
Starting at the APEX of the heart and moving superiorly, what is the correct order in which
you would encounter the four anatomical structures below?
A valves, chordae tendonae, papillary muscle, ventricle
B ventricle, papillary muscle, chordae tendonae, valves
C papillary muscle, chordae tendonae, ventricle, valves
D chordae tendonae, valves, ventricle, papillary muscle
ANSWER:B
1300 Which period of the heart cycle is completely occupied by the ventricles relaxing?
A atrial systole
B atrial diastole
C ventricular systole
D ventricular diastole
ANSWER:D
1301
Through which valve does blood flow when it moves from the right atrium into the right
ventricle?
A the tricuspid valve

B the mitral valve
C the pulmonary valve
D the bicuspid valve
ANSWER:A
1302 How is the fibrous pericardium attached to the surrounding structures?
A laterally to the pleural surfaces of the lungs.
B posteriorly to the sternum.
C anteriorly to trachea, main-stem bronchi and oesophagus.
D inferiorly to the clavicles.
ANSWER:A
1303
A drug, such as cocaine, which stimulates the heart but does directly inhibit the heart’s ability
to relax, would be considered a:
A Sympatholytic
B Sympathomimetic
C Parasympatholytic
D Parasympathomimetic
ANSWER:B
1304
Why is the myocardium of the right ventricle (RV) thinner than that of the left ventricle
(LV)?
A the RV pumps into the pulmonary circuit which has less resistance than the systemic circuit.
B the RV pumps a smaller volume of blood than the LV.
C the RV pumps blood out with a slower exit speed than the RV.
D the RV chamber has a smaller volume than the LV.
ANSWER:A
1305
Through which valve does blood flow when it moves from the left atrium into the left
ventricle?
A the semilunar valve
B the mitral valve
C the tricuspid valve
D the bicuspid valve
ANSWER:B
1306 Which period of the heart cycle is completely occupied by the ventricles contracting?

A atrial systole
B atrial diastole
C ventricular systole
D ventricular diastole
ANSWER:C
1307 Which statement below describes blood flow through the mitral valve?
A blood flows from the right atrium into the right ventricle
B blood flows from the right ventricle into the pulmonary artery
C blood flows from the left ventricle into the aorta
D blood flows from the left atrium into the left ventricle
ANSWER:D
1308 Which structure has the thickest wall?
A the aorta
B the inter-atrial septum
C the left ventricle
D the right ventricle
ANSWER:C
1309 Which tissue is supplied with blood via the coronary arteries?
A the lungs
B the myocardium
C the corona
D the aorta
ANSWER:B
1310 What is the innermost layer of the heart wall known as?
A epicardium
B pericardium
C visceral pericardium
D endocardium
ANSWER:D
1311 Where is the mitral valve of the heart located? Between the
A left atrium and left ventricle
B left ventricle and the aorta
C right ventricle and the pulmonary trunk

D right atrium and right ventricle
ANSWER:A
1312 Choose the structure known as the pacemaker of the heart from the following.
A atrio-ventricular node
B sino-atrial node
C atrio-ventricular bundle
D the bundle of His
ANSWER:B
1313 Where is the aortic valve located?
A between the right atrium and right ventricle
B between the right ventricle and the pulmonary trunk
C between the left ventricle and the aorta
D between the left atrium and left ventricle
ANSWER:C
1314 By what name is the heart muscle known?
A epicardium
B myocardium
C pericardium
D endocardium
ANSWER:B
1315 The heart receives its own oxygenated blood supply via the
A coronary arteries
B the pulmonary veins
C the coronary sinus
D the foramen ovale
ANSWER:A
1316 Which name is NOT applied to the valve between the left ventricle and the left atrium?
A atrioventricular valve
B semilunar valve
C the bicuspid valve
D the mitral valve
ANSWER:B
1317 Where does the pulmonary trunk deliver its blood to?

A the left atrium
B the right ventricle
C the lungs
D the left ventricle
ANSWER:C
1318 The heart can be made to beat faster by which of the following?
A sympathetic stimulation of the SA node
B sympathetic stimulation of the AV node
C parasympathetic stimulation of the SA node
D parasympathetic stimulation of the AV node
ANSWER:A
1319 What is the outermost layer of the heart wall known as?
A epicardium
B pericardium
C parietal membrane
D endocardium
ANSWER:A
1320 The valve between the atrium and the ventricle that pumps oxygenated blood is called:
A the right atrioventricular valve
B the semilunar valve
C the mitral valve
D the tricuspid valve
ANSWER:C
1321 The mitral valve of the heart is located between the
A right atrium and right ventricle
B left ventricle and the aorta
C right ventricle and the pulmonary trunk
D left atrium and left ventricle
ANSWER:D
1322 Complete the sentence correctly. The left ventricle pumps:
A more blood than the right ventricle
B blood at a lower pressure than the right ventricle
C less blood than the right ventricle

D blood at a higher pressure than the right ventricle
ANSWER:D
1323 What is ventricular systole? It
A refers to contraction of the ventricles
B occurs at the same time as contraction of the atria
C occurs while the bicuspid valve is open
D refers to relaxation of the ventricles
ANSWER:A
1324 Which is correct? In its passage through the heart, blood is pumped into the pulmonary trunk:
A after leaving the left ventricle
B after leaving the left atrium
C after passing through the right AV valve
D after passing through the left AV valve
ANSWER:C
1325 Cardiac muscle cells differ from skeletal muscle cells in that:
A skeletal muscle cells are voluntary but cardiac muscle cells are not.
B skeletal muscle cells are branched but cardiac muscle cells are not.
C cardiac muscle cells are multinucleate but skeletal muscle cells are not.
D cardiac muscle cells are a syncytium while skeletal muscle does not.
ANSWER:A
1326 Which chamber of the heart has the thickest myocardium?
A left ventricle
B right ventricle
C left atrium
D right atrium
ANSWER:A
1327 Why is the myocardium of the left ventricle thicker than that of the right ventricle?
A The left ventricle has to pump a greater volume of blood than the right ventricle.
B The resistance of the systemic circulation is greater than that of the pulmo- nary circulation.
C The left ventricle has to pump blood to the brain against gravity.
D The right ventricle is assisted by the “respiratory pump”.

ANSWER:B
1328 What supplies blood to the myocardium?
A the coronary circulation.
B the vena cavae.
C the vasa recta.
D the pulmonary circulation.
ANSWER:A
1329
Which of the following heart structures are listed in the correct sequence of blood flow
through them?
A right atrium, bicuspid valve, pulmonary valve, left ventricle.
B tricuspid valve, right ventricle, left atrium, mitral valve.
C pulmonary valve, left atrium, tricuspid valve, left ventricle.
D right ventricle, left atrium, aortic valve, left ventricle.
ANSWER:B
1330 The tricuspid valve separates which two structures?
A right ventricle and pulmonary trunk
B right ventricle and right atrium
C left ventricle and aorta
D left ventricle and left atrium
ANSWER:B
1331 Why is the myocardium of the right ventricle thinner than that of the left ventricle?
A the left ventricle has to pump a greater volume of blood than the right ventricle.
B it results from left ventricular hypertrophy due to increased peripheral resistance.
C it pumps blood into the low resistance pulmonary circulation.
D it pumps blood into the high resistance systemic circulation.
ANSWER:C
1332 Which of the following events occur during late ventricular diastole?
A the atria are relaxed, the ventricles are filling passively, the atrioventricular valves are open
B
the ventricles are starting to contract, the atrioventricular valves are closed, the semilunar
valves are open
C the atria contract, the ventricles are relaxed, the atrioventricular valves are open
D
the atria are relaxed, the ventricles are starting to relax, the atrioventricular valves are closed,
the semilunar valves are closed.

ANSWER:C
1333 In a normal ECG trace, what does a QRS wave indicate?
A depolarisation of the atria
B repolarisation of the atria
C depolarisation of the ventricles
D repolarisation of the ventricles
ANSWER:C
1334 What would be a possible consequence of the SA node failing to depolarise?
A the entire heart would not contract
B the heart rate will decrease
C the ventricles would not contract
D the heart rate will increase
ANSWER:B
1335 Which of the following does limb lead II of a typical electrocardiogram represent?
A A graph of the variation of voltage produced by the heart against time.
B The voltage at right arm (RA) plus the voltage at left leg (LL).
C The electrical events that precede the contraction of the ventricles.
D
The projection of the electric dipole vector of the heart on the line from left arm (LA) to
right arm (RA).
ANSWER:A
1336 Which of the following events occur during early ventricular systole?
A the atria are relaxed, the ventricles are filling passively, the atrioventricular valves are open
B
the ventricles are starting to contract, the atrioventricular valves are closed, the semilunar
valves are closed
C the atria contract, the ventricles are relaxed, the atrioventricular valves are open
D
the atria are relaxed, the ventricles are starting to relax, the atrioventricular valves are
opening, the semilunar valves are closing.
ANSWER:B
1337
When listening to the “lub-dup” sound of the heart with a stethoscope, what is the cause of
the “dup” sound?
A The blood flowing through the open semilunar valves
B The blood flowing through the open atrioventricular valves
C The turbulent blood flow through closing atrioventricular valves

D The turbulent blood flow through closing semilunar valves
ANSWER:D
1338 What structure in the heart prevents backflow of blood into the right atrium?
A The tricuspid valve
B The bicuspid valve
C The mitral valve.
D The foramen ovale
ANSWER:A
1339 Why does the lumen of a large vein have a larger diameter than the lumen of a large artery?
A They need to withstand higher pressure than arteries.
B Veins contain the majority of the blood volume.
C This allows blood to return to the heart rapidly.
D So that vasoconstriction can produce a greater change in diameter than for arteries.
ANSWER:B
1340
Different types of capillaries are distinguished by their structure. Which of the following is
NOT a structural difference between capillaries?
A
The endothelial cells of some capillaries are joined by tight junctions while for others there
are gaps between the cells.
B Some capillaries have pores that allow movement between the plasma and interstitial fluid.
C
Some capillaries have an arterial end and a venous end, while others begin in the tissues and
drain into a venule.
D Some capillaries are surrounded by a basement membrane while others are not.
ANSWER:C
1341 What causes venous blood to return to the heart?
A The pumping action of the heart.
B The squashing action of muscles, and valves in the veins.
C Rhythmic vasoconstriction and valves in the veins.
D Gravity, valves and the negative pressure generated by the atria emptying.
ANSWER:B
1342
When cardiac ejection ceases during diastole, what is the most important factor maintaining
blood flow in arteries of the body?
A Contraction of skeletal muscle

B Closing the venous valves
C Elastic recoil of the arteries close to heart
D Contraction of the atria
ANSWER:C
1343
In which organs would be found continuous, fenestrated, and sinusoid capillar- ies,
respectively?
A Brain, small intestine, liver
B Bone marrow, brain, spleen
C Liver, bone marrow, brain
D Small intestine, liver, brain
ANSWER:A
1344 What is the advantage of having a wide lumen in veins?
A It provides less resistance to the blood on its way to heart
B It controls the opening and closing of the valves
C It produces high pressure on the blood on its way to heart
D It helps the pre-capillary sphincters to stay open for a longer time
ANSWER:A
1345
What is the pulse that we feel at the anterior medial part of the elbow (when in anatomical
position) called?
A Radial pulse
B Ulnar pulse
C Carotid pulse
D Brachial pulse
ANSWER:D
1346 Which capillaries allow cells and plasma proteins to enter or leave their lumen?
A Continuous
B Fenestrated
C Sinusoidal
D Anastomatic
ANSWER:C
1347 Which of the following does NOT assist in returning the blood to the heart through the veins?
A valves in the veins

B the “respiratory pump”
C the effect of gravity
D the pumping action of the heart
ANSWER:D
1348 Which of the following materials is found in the walls of capillaries?
A endothelium
B elastic fibres
C collagen fibres
D smooth muscle
ANSWER:A
1349 Which of the following arteries do NOT arise from the ascending aorta?
A Brachiocephalic trunk
B Left Brachiocephalic
C Left Common Carotid
D Left Subclavian
ANSWER:B
1350
What is the pulse we feel in the anterior lateral wrist (in anatomical position – the normal
wrist pulse) called?
A Radial pulse
B Ulnar pulse
C Dorsalis pedis pulse
D Brachial pulse
ANSWER:A
1351
Which type of capillary is required to allow the liver to perform its function of producing
plasma proteins?
A Continuous
B Fenestrated
C Sinusoidal
D Anastomatic
ANSWER:C
1352
What is it called when plaque dislodges from a lesion in a blood vessel wall and then moves
“downstream” to lodge in the capillary bed feeding the digestive tract? We call this a:

A Myocardial infarction
B Stroke
C Pulmonary embolism
D Mesenteric embolism
ANSWER:D
1353 Which of the following is found in the walls of capillaries?
A endothelial cells and basement membrane
B tunica externa
C tunica media
D smooth muscle
ANSWER:A
1354
In which of the four lists below are the types of capillaries listed in order of permeability
with the first being the least permeable and the last being the most permeable type of
capillary?
A blood brain barrier capillaries, continuous capillaries, fenestrated capillar- ies, sinusoids.
B
continuous capillaries, blood brain barrier capillaries, fenestrated capillar- ies, glomerular
capillaries.
C fenestrated capillaries, blood brain barrier capillaries, continuous capillar- ies, sinusoids.
D liver sinusoids, blood brain barrier capillaries, continuous capillaries, fenes- trated capillaries.
ANSWER:A
1355 Which of the following assists in returning the blood to the heart through the veins?
A valves in the veins, the effect of breathing, gravity.
B valves in the veins, the effect of breathing, squashing action of muscles, gravity.
C the effect of gravity, the pumping action of the heart.
D the effect of breathing, squashing action of muscles, the right ventricle.
ANSWER:B
1356 Why do arteries have more elastic and muscular tissue than veins?
A arteries need to expand and contract as blood flows through them.
B arteries need carry a greater volume of blood than do veins.
C to ensure that blood flows only in the direction away from the heart.
D in order to support the larger diameter of arteries compared to veins.
ANSWER:A

1357 Which capillaries have walls that allow the easiest passage of materials through them?
A fenestrated capillaries
B capillaries of the blood-brain barrier
C sinusoids
D continuous capillaries
ANSWER:C
1358 Which factor below does NOT assist venous return of blood?
A breathing
B gravity
C smooth muscle contraction
D skeletal muscle contraction
ANSWER:C
1359 The usefulness of having elastic arteries is in:
A their ability to regulate blood pressure.
B their ability to expand as the heart pumps blood into them
C the assistance they give to venous return.
D their ability to produce vasoconstriction and vasodilation
ANSWER:B
1360
What are capillaries that have endothelial cells joined by “tight junctions” and have
“intercellular clefts” between cells called?
A the blood-brain barrier
B continuous capillaries
C fenestrated capillaries
D sinusoids
ANSWER:B
1361 Vasoconstriction and vasodilation of blood vessels is facilitated by the
A elastic fibres in vessel walls
B parasympathetic division of the nervous system
C smooth muscle in vessel walls
D tunica intima of the blood vessel
ANSWER:C
1362 One of the following does NOT help blood to move through arteries. Which one?
A the influence of gravity.

B the action of breathing and the movement of the diaphragm.
C the elastic recoil of artery walls.
D the pumping action of the heart.
ANSWER:B
1363 Veins in the limbs have which one of the following characteristics?
A thin walls composed of epithelium.
B vessel walls with a thick layer of smooth muscle
C pulsatile flow
D valves
ANSWER:D
1364
Exchange between the blood and the interstitial fluid occurs most readily through which type
of capillary?
A venules
B fenestrated capillaries
C sinusoids
D arterioles
ANSWER:C
1365 What is meant by the “pulmonary circulation”? The flow of blood:
A out the aorta and back through the vena cavae.
B from the heart through the lungs and back to the heart.
C into the coronary arteries and back through the coronary sinus.
D into the vena cavae and out to the pulmonary trunk via the right ventricle.
ANSWER:B
1366 What is a small artery called?
A an anastomosis
B an arteriole
C an efferent artery
D a distributing artery
ANSWER:B
1367 How do arteries differ from veins?
A arteries have a larger diameter than veins.
B arteries have more elastic tissue than veins.
C there is a greater volume of blood in the arteries than in the veins.

D arteries have valves but veins do not.
ANSWER:B
1368 The wall of a capillary consists of one layer (or coat). What is it called?
A tunica intima
B tunica externa
C lamina propria
D tunica media
ANSWER:A
1369
Which part(s) of the cardiovascular system has/have a single layer of endothe- lial cells as the
innermost layer of the structure(s)?
A arteries, capillaries and veins
B arteries and veins
C heart, arteries, veins and capillaries
D capillaries
ANSWER:C
1370 Which of the following describes an artery? An artery is a blood vessel that
A has a thinner wall than a vein of comparable size
B carries blood away from the heart
C when located in the limbs, has valves
D carries oxygenated blood
ANSWER:B
1371
Some capillaries allow plasma proteins and phagocytes to enter and leave the blooWhich
type?
A sinusoids
B fenestrated capillaries
C continuous capillaries with intercellular clefts
D lacteals
ANSWER:A
1372 Arteries may be characterized as:
A elastic tubes that carry oxygenated blood.
B elastic tubes that carry blood away from the heart.
C muscular tubes that have valves.
D muscular tubes with a larger diameter than veins.

ANSWER:B
1373 Some capillaries are called “sinusoids”. These are capillaries that:
A are fenestrated.
B have endothelium without gaps between cells.
C have endothelium with gaps between adjacent cells.
D have pores in the endothlial cell walls.
ANSWER:C
1374 What can be said about the endothelium of fenestrated capillaries?
A the endothelial cells have pores (windows) to allow rapid movement of sol- utes and water.
B
gaps between adjacent endothelial cells allow free exchange between blood and interstitial
fluid.
C their tunica intima is composed of a complete lining of endothelial cells.
D They do not have an endothelium.
ANSWER:A
1375 Which of the following definitions best describes veins?
A a vessel that carries de-oxygenated blood.
B a vessel that carries blood towards the heart.
C vessels that contain valves.
D vessels through which blood flows under the influence of pressure produced by the heart.
ANSWER:B
1376
Fenestrated capillaries permit the exchange of nutrients and wastes between cells and the
blood because of which feature of their structure?
A
gaps between adjacent endothelial cells allow free exchange between blood and interstitial
fluid.
B they are in close proximity to most cells.
C their tunica intima is composed of a single layer of endothelial cells.
D their endothelium has pores (windows) to allow the rapid movement of water and solutes.
ANSWER:D
1377 Which of the following is a difference between arteries and veins?
A artery walls have more elastic tissue and smooth muscle than veins.
B veins have three distinct “tunics” in their walls whereas arteries have only two.
C blood flow in veins is pulsatile while that is arteries is continuous.
D the walls of veins have more elastic tissue and smooth muscle than in arteries.

ANSWER:A
1378
Capillaries permit the exchange of nutrients and wastes between cells and the blood because
of which feature of their structure?
A
adjacent endothelial cells are separated by gaps which allow free exchange between blood
and interstitial fluid.
B they are in close proximity to most cells.
C
they have a single layer, the tunica intima, consisting of a single layer of endothelial cells in
their walls.
D
their endothelial cells have pores (windows) to allow the rapid movement of water and
solutes.
ANSWER:C
1379
People with occupations that involve standing all day are at risk of varicose veins. Why
should this be true?
A Veins have a larger lumen than arteries so hold more blood which distends them.
B Veins have only two tunics so their walls are more easily stretched into varicosity.
C
Gravity acting on blood in the legs puts stress on the vessel walls increasing the likelihood
that they will stretch and distort.
D When standing still, the valves stay open and blood accumulates in the leg veins.
ANSWER:C
1380 Which statement is INCORRECT? Colloid osmotic pressure:
A is the difference in solution concentration between plasma and interstitial fluid.
B
refers to the greater solution concentration of plasma, compared to intersti- tial fluid, due to
the plasma proteins.
C causes water to be drawn into blood from the interstitial fluid.
D is the pressure exerted on capillary walls due to the collision of the plasma proteins.
ANSWER:D
1381 Which tunic(s) are ABSENT from the walls of blood capillaries?
A tunica intima
B tunics externa and intima
C tunics media and externa
D tunics interna and media
ANSWER:C

1382
The volume of gas trapped within a space will increase as pressure decreases, the physics law
that refers to this is:
A Henry’s law
B Poiseuille’s law
C Hugh’s law
D Boyle’s law
ANSWER:D
1383 Increasing the number of red cells per millilitre of blood will:
A increase the viscosity and increase the rate of flow of the blood
B decrease the viscosity and increase the rate of flow of the blood.
C increase the viscosity and decrease the rate of flow of the blood.
D decrease the viscosity and decrease the rate of flow of the blood.
ANSWER:C
1384
According to Boyle’s law, what will happen in a fixed amount of gas if its vol- ume
decreases?
A the pressure of the gas will decrease
B the pressure of the gas will increase
C the temperature of the gas will decrease
D the temperature of the gas will increase
ANSWER:B
1385
Consider Boyle’s law. If the pressure being exerted on an enclosed volume of gas (at a
constant temperature) is increased by 0 %, by what percentage would its volume change?
A volume would increase by 0 %
B volume would increase by %
C volume would decrease by %
D volume would decrease by 0 %
ANSWER:D
1386 What formula, or law or principle refers to the partial pressure of a gas?
A Boyle’s law
B Henry’s law
C Pascal’s principle
D Poiseuille’s law
ANSWER:B

1387 Which of the following is NOT used as a unit of pressure?
A pascals
B newtons per square metre
C millimetres of mercury
D millilitres of water
ANSWER:D
1388
When performing cardio-pulmonary resuscitation (CPR) why do you use the “heel” of the
hand rather than the whole palm and fingers?
A
A greater force can be exerted by the “heel” of the hand which results in a greater pressure
being exerted.
B
Using the “heel” places a smaller area of the hand in contact with the ster- num, hence allows
a greater force to be exerted.
C
The same force can be applied to a smaller area of the sternum which is less able to resist the
greater downward pressure.
D The resuscitator’s wrist is less likely to be damaged by applying CPR in this way.
ANSWER:C
1389
Which of these statements about pressure in static liquids is NOT true? Pressure at a point in a
liquid
A depends on the height of liquid above it.
B acts equally in all directions.
C depends on the depth it is below the surface.
D depends on the volume of liquid above it.
ANSWER:D
1390 Which of the following is closest to a statement of Poiseuille’s law?
A Volume flow rate is proportional to the fourth power of the radius of the tube.
B The pressure in flowing fluids is lowest where the speed of flow is greatest.
C The amount of gas that will dissolve in a liquid is proportional to its partial pressure.
D When the flow of fluid is producing sound, then turbulent flow is present.
ANSWER:A
1391 The quantity called ‘pressure’ is defined as the
A mass per unit area.
B force per unit area.
C height of mercury supported by the atmosphere.

D newton per square metre.
ANSWER:B
1392
The unit of pressure called the ‘pascal’ (symbol Pa) is the name given to which of the
following combination of units?
A newton per metre (N/m).
B newton per square metre (N/m).
C millimetres of mercury (mm Hg).
D force per area (F/A).
ANSWER:B
1393 Which of the following statements about pressure is true?
A As boxes are stacked on top of each other, the pressure that they exert on the floor decreases.
B Very small forces will exert small pressures.
C By standing on wide flat boards, the pressure exerted on the floor by your body is decreased.
D Objects of large mass will produce large pressures.
ANSWER:C
1394
The measurement of cerebrospinal fluid (CSF) pressure is made while the patient is lying
down rather than sitting up. The horizontal posture prevents a false high reading due to
A the weight of the ‘head’ of CSF in the spinal cord.
B possible movement of the patient while sitting up.
C pressure on the lumbar vertebrae when the back is vertical.
D the greater muscle tone needed to maintain a sitting position.
ANSWER:A
1395
A patient lying on a mattress of enclosed air is less likely to develop decubitis ulcers because
the:
A mass of the patient is decreased.
B force exerted by the patient is decreased.
C patient’s surface area in contact with the mattress is increased.
D pressure exerted by the patient on the mattress is increased.
ANSWER:C
1396
A bedridden patient is less likely to develop bed sores while lying on a water beThis is
because:

A the force applied to their bony projections is acting over a tiny area.
B the flexibility of the bed assists the blood circulation.
C
the weight of the patient is being supported by parts of the body that are adapted for weight-
bearing.
D the patient experiences a uniform pressure over most of the lower surface of their body.
ANSWER:D
1397
Pascal’s principle (‘pressure applied to an enclosed fluid at rest is transmitted to every
portion of the fluid and to the walls of the containing vessel’) may be used to understand
which of the following phenomena?
A An air mattress minimises the pressure applied to a bed-ridden patient’s body.
B A worker can walk on wet concrete without sinking by standing on wide boards.
C That a pulse can be felt as blood flows through a superficial artery.
D The collapsing of a plastic intravenous bag as the liquid runs out.
ANSWER:C
1398 The consideration of pressure in gases differs from pressure in liquids because
A pressure at any point in a gas acts differently in different directions.
B in a gas mixture, the pressure exerted by each different gas must be considered.
C the pressure in a liquid increases with depth but does not for a gas.
D liquids are virtually incompressible whereas gases are compressible.
ANSWER:D
1399
Which of the following statements from kinetic molecular theory can be used to explain why
a region of low pressure will result in gas particles rushing into that region?
A Particles of a gas are widely separated, consequently can be easily compressed.
B Gas particles are moving at very high speed in random directions.
C There are a great many sub-microscopic particles per unit volume.
D Gas particles will exert a force on colliding with the walls of their container.
ANSWER:B
1400
The kinetic molecular theory of gases provides us with some valuable insights into the
behaviour of gases. One correct prediction of the theory is: That gas pressure is:
A due to the force exerted by gas molecules as they collide with the walls of their container.
B due to the forces of attraction between gas particles.
C inversely proportional to the gas temperature.
D proportional to the partial pressures of the different gases that make up the gas.

ANSWER:A
1401 Which of the following is a statement of Henry’s law?
A
The partial pressure of a gas, in a mixture of gases, is the contribution it makes to the total
pressure of the mixture.
B
In a mixture of gases, the total pressure is the sum of the pressures exerted by each of the
gases alone.
C
Pressure applied to any point in a gas is transmitted equally and undimin- ished to all parts of
the gas and to the walls of the container.
D
The quantity of gas that will dissolve in a liquid at a given temperature is proportional to the
partial pressure of the gas and to its solubility coefficient.
ANSWER:D
1402
Which of the following sentences best represents a statement of the Bernoulli effect? Pressure
in a…
A flowing fluid is greatest where its speed is greatest.
B fluid acts equally in all directions.
C flowing fluid is lowest where its speed is lowest.
D flowing fluid is lowest where its speed is greatest.
ANSWER:D
1403 A statement of Boyle’s law could be:
A as pressure of a gas increases, its solubility increases
B as volume of a gas increases, its pressure decreases.
C as volume of a gas decreases, its pressure decreases.
D as pressure of a gas increases, its solubility decreases.
ANSWER:B
1404
When a person is standing upright, what can be said about the arterial blood pressure in their
feet?
A It will be greater than arterial pressure in the aorta.
B It will be less than the arterial pressure in the aorta.
C It will be the same as the arterial pressure in the aorta.
D It will be the same as the venous blood pressure in the feet.
ANSWER:A
1405
A plaque in a coronary vessel wall will result in a decrease in the lumen diam- eter of a
coronary artery and in less oxygen being delivered to the heart muscle. Why is this?

A
A protruding plaque increases the distance blood needs to travel and this increases resistance
to flow.
B A narrow artery restricts blood flow by increasing blood’s viscosity.
C
The constricted artery causes the pressure gradient to decrease which results in lower blood
flow.
D The decrease in artery radius will cause blood volume flow rate to decrease.
ANSWER:D
1406 What units is a blood pressure of 0/0 is expressed in?
A Millimetres of mercury
B Centimetres of blood
C Centimetres of mercury
D Pascals
ANSWER:A
1407 What effect will increasing the concentration of red cells in blood have? It will:
A decrease blood viscosity and increase aortic pressure
B increase blood viscosity and decrease aortic pressure
C increase blood viscosity and increase aortic pressure
D decrease blood viscosity and decrease aortic pressure
ANSWER:C
1408 By which means will the sympathetic response raise blood pressure?
A Vasoconstriction due to stimulation of alpha receptors
B Vasoconstriction due to stimulation of beta receptors
C Increase cardiac output due to stimulation of alpha receptors
D Increased stroke volume due to stimulation of alpha receptors
ANSWER:A
1409
Consider Pascal’s principle. If the heart exerts a pressure of 0 mmHg on the blood in the
aorta, where else in the body will the blood pressure be 0 mm Hg?
A in the brachial artery of a seated person
B in the superior vena cava of a standing person.
C in the capillaries of the feet of a supine person
D in the anterior tibial artery of a person who is standing
ANSWER:A

1410
If an atheroma reduces the diameter of an artery from mm to mm, what effect would this
have on the blood flow through the artery? (According to Poiseuille’s law, volume flow rate
through a blood vessel is proportional to its radius to the fourth power i.e. V ∝ R).
A blood flow would decrease to half (/) the value for an artery with diameter mm.
B blood flow would decrease to a quarter (/) of the value for an artery with diameter mm.
C blood flow would decrease to an eighth (/) of the value for an artery with diameter mm.
D blood flow would decrease to one sixteenth (/) of the value for an artery with diameter mm.
ANSWER:D
1411
If the radius of a blood vessel is halved, the blood flow through it drops to one sixteenth its
previous value. This statement could be describing which of the following?
A a decrease in pressure gradient
B atherosclerosis
C vasoconstriction
D Poiseuille’s law
ANSWER:D
1412
A man who is standing has a resting systolic blood pressure of 0 mmHg at the start of his
aortWhat will be the pressure in the arteries of his feet?
A about 0 mmHg, as arterial blood pressure decreases with distance from the heart.
B
about 0 mmHg as arterial blood pressure does not fall appreciably until just before blood
enters the capillaries.
C about 00 mmHg as the “head” of liquid increases the blood pressure in the feet.
D
about 0 mmHg as blood pressure will drop in the absence of venous return when the
“skeletal muscle pump” is not operating.
ANSWER:C
1413 To what does the term “cardiac output” refer?
A the speed of blood flow through the aorta.
B the volume of blood flowing through the aorta per minute.
C the volume of blood pumped by the heart with each beat.
D the number of heart beats per minute.
ANSWER:B
1414 A resting blood pressure stated as 0/0 (in units of mmHg) refers to
A maximum pressure in the aorta/minimum pressure in the aorta

B maximum pressure in the aorta/maximum pressure in the vena cavae
C diastolic pressure/systolic pressure
D left ventricular systolic/right ventricular systolic
ANSWER:A
1415 Which of the following does NOT influence the resistance to blood flow?
A diameter of the arterioles
B temperature of the blood
C haematocrit
D radius of the veins
ANSWER:D
1416 Colloid osmotic pressure
A is due to all of the dissolved particles in blood
B is the difference in pressure between the arterial end and the venous end of a capillary
C is the difference in pressure between the inside of a capillary and the inter- stitial fluid
D is due to the plasma proteins in the blood
ANSWER:D
1417
Due to an atheromatous plaque, a man’s coronary artery has narrowed to one third of its
healthy diameter, all other things being equal, his blood flow rate (ml/min) in that artery
would
A be one third of the unobstructed value
B be one eighty-first of its healthy value
C be one ninth of its healthy value
D be one twenty-seventh of its healthy value
ANSWER:B
1418
Say a diastolic blood pressure reading was 0 mm Hg. This is consistent with which one of the
following statements?
A the diastolic reading is 0 mm Hg greater than atmospheric pressure.
B pressure is measured in length units.
C the diastolic reading is 0 mm Hg less than atmospheric pressure.
D this is the maximum pressure produced by contraction of the myocardium.
ANSWER:A
1419
If mean arterial pressure is kept constant while a small artery changes its radius from mm to
mm, what will happen?

A blood volume flow rate through the artery will double.
B blood volume flow rate through the artery will increase to four times its previous value.
C blood volume flow rate through the artery will be sixteen times the original value.
D blood speed will halve so there will be no change in volume flow rate.
ANSWER:C
1420 When “colloid osmotic pressure” is used in relation to the blooWhat is being referred to?
A the movement of water molecules across the membrane of a red blood cell.
B the filtration pressure in the glomeruli of the kidneys.
C the osmotic pressure forcing water and solute out of capillaries.
D the osmotic pressure due to the plasma proteins.
ANSWER:D
1421
According to Poiseuille’s law of fluid flow, the volume flow rate, V, is propor- tional to
radius to the fourth power. What would happen to the volume flow rate of blood if the
diameter of an artery decreased to 0 % (/) of its original diameter? V would:
A be one fifth (0.) of its original value.
B be one twenty-fifth (0.0) its original value.
C be / (0.00) of its original value.
D five times its original value.
ANSWER:C
1422 If a blood pressure is stated as over 0. (in units of kilopascals) this means:
A pulse pressure is kPa.
B diastolic pressure is 0. kPa below atmospheric pressure.
C systolic pressure is . kPa above atmospheric pressure
D systolic pressure is kPa above atmospheric pressure
ANSWER:D
1423
As a result of the pumping action of the heart, we are able to feel pulsations in our superficial
arteries (e.g. the radial pulse). This effect is an example of which of the following?
A Pascal’s principle.
B Torricelli’s law.
C The Bernoulli effect.
D Starling’s law of the heart.
ANSWER:A

1424
Suppose that a person who is standing still, has a mean arterial pressure in the aorta of kPa (
mmHg). The blood pressure at the start of the arterioles of the feet will be:
A about the same since healthy arteries present very little resistance to blood flow.
B less because blood pressure decreases along arteries as distance from the heart increases.
C more because of the hydrostatic pressure exerted by the column of blood in the arteries.
D less because of the decreased venous return which results from the person’s inactivity.
ANSWER:C
1425
A systolic blood pressure which is stated as kPa (0 mmHg) means that the pressure in the
arterial blood is:
A negative kPa.
B kPa above zero kilopascals.
C kPa greater than atmospheric pressure.
D kPa less than atmospheric pressure.
ANSWER:C
1426
If a stenosis reduces the size of a blood vessel to half of the original diameter, the volume
flow rate through the vessel will be reduceWhich relationship below determines the extent of
the decrease in flow?
A Poiseuille’s law.
B Bernoulli’s theorem.
C Dalton’s law.
D Pascal’s principle.
ANSWER:A
1427
During an auscultatory blood pressure determination, the Korotkoff sounds that are listened
for are produced because
A the partial pressure of the blood has been increased.
B the blood flow is turbulent.
C of the viscosity of the blood.
D the volume flow rate has decreased.
ANSWER:B
1428
When taking a subject’s blood pressure you are actually measuring the differ- ence between
total pressure and which other pressure?
A gauge pressure
B blood pressure

C atmospheric pressure
D standard atmospheric pressure
ANSWER:C
1429 Which of the following would increase arterial blood pressure?
A a decrease in sympathetic impulses along the cardio-accelerator nerves
B an increase in parasympathetic impulses along the vagus nerve
C a decrease in sympathetic impulses along vasomotor nerves
D an increase in sympathetic impulses along vasomotor nerves
ANSWER:D
1430
Blood flow is largely regulated at a tissue level. Which of the following could be said
regarding this process?
A A rise in the blood level of O will result in vasodilation
B A raised CO level results in vasodilation
C Acidaemia directly increases vasopressin (ADH) release
D A raised CO blood level will result in an increased serum alkalinity
ANSWER:B
1431 Which of the following statements regarding antihypertensive medication is correct?
A Calcium channel blockers are a class of drug used to reverse a decrease in blood volume.
B Diuretic medication principally affects peripheral resistance.
C Beta blockers target cardiac sympathetic innervation.
D ACE inhibitors promote the effects of the renin-angiotensin aldosterone system.
ANSWER:C
1432
Which of the following is a class of antihypertensive medication which specifi- cally target a
reduction in blood volume?
A Beta blockers
B Calcium Channel Blockers
C Diuretics
D Anticoagulants
ANSWER:C
1433
Which of the following events would you expect to observe in response to a drop in a
patients’ blood pressure?
A Renin is converted to Angiotensinogen
B Angiotensin is converted into Aldosterone

C ADH (Anti Diuretic Hormone) will be released by the posterior pituitary gland
D Baroreceptors signal the SA node to slow
ANSWER:C
1434 Which of the following responses best describes term “pulse pressure”
A A mean measurement of the systolic and diastolic readings
B A measurement calculated from / of the diastolic added to the systolic value
C The lowest audible Korotkoff sound when recording blood pressure
D A measurement of the difference in pressure between systolic and diastolic readings
ANSWER:D
1435 What may correctly be said about baroreceptors?
A They are located in the walls of the aortic arch and the inferior vena cava
B
A drop in blood pressure triggers the baroreceptor reflex which causes vaso- dilation and an
increased heart rate
C
They promote vasoconstriction and an increased force of myocardial con- traction in the
hypotensive patient
D They respond directly to alterations in circulating oxygen levels
ANSWER:C
1436 What does Angiotensin II do?
A It is a weak vasoconstrictor and requires activation by Angiotensinogen
B It acts via several mechanisms that cause blood pressure to increase
C It reduces blood pressure through decreasing vascular smooth muscle tone
D It causes an increase in urine output by triggering ADH release
ANSWER:B
1437 What does administering Beta-Blocking medication do?
A It targets adrenergic neurotransmission to Beta receptors
B It targets cholinergic neurotransmission to decrease blood pressure
C It acts principally upon Beta and Alpha receptors
D It exposes the Beta receptors to enhance neurotransmission
ANSWER:A
1438 Cardiac output does NOT depend on one of the following, which one?
A the rate of venous return to the heart
B the blood viscosity
C the volume flow rate through the circulatory system

D the pressure drop (between start of aorta and start of capillaries)
ANSWER:C
1439 Which of the following is most unlikely to increase blood viscosity?
A leucocytosis
B dehydration
C hypothermia
D an infusion of packed red blood cells
ANSWER:A
1440 Which of the following would cause blood pressure to DECREASE?
A an increase in heart rate
B an increase in total peripheral resistance
C an increase in heart stroke volume
D an increase in parasympathetic impulses along the vagus nerve
ANSWER:D
1441 Which of the following is the best definition of hypertension?
A a systolic pressure of more than 0 mmHg and a diastolic pressure of more than 0 mmHg
B a diastolic pressure of more than 0 mmHg and a systolic pressure of more than 0 mmHg
C a blood pressure of more than 0/0 mmHg measured after minutes of inactivity
D a blood pressure of less than 0/0 mmHg measured after minutes of inactivity
ANSWER:C
1442 In which part of the brain is the cardiovascular control centre located?
A the neurohypophysis
B the cerebrum
C the hypothalamus
D the medulla oblongata
ANSWER:D
1443 When blood pressure drops, which of the following responses would happen?
A atrial natriuretic peptide is released from the heart
B the kidneys release renin which catalyses the formation of angiotensin I
C the rate of sodium excretion by the kidneys increases
D the secretion of anti-diuretic hormone is inhibited
ANSWER:B
1444 Which of the following are two of the factors that influence arterial blood pressure?

A peripheral resistance and gravity.
B cardiac output and the partial pressure of oxygen dissolved in blood.
C blood volume and blood osmotic pressure
D cardiac output and peripheral resistance
ANSWER:D
1445
What would be the effect produced if the cardiovascular centre increased the rate of
parasympathetic impulses it sends out?
A heart rate would increase
B heart rate would decrease
C vasoconstriction of blood vessels would increase
D vasodilation of blood vessels would increase
ANSWER:B
1446 Which of the following does NOT influence the resistance to blood flow?
A diameter of the arterioles
B temperature of the blood
C haematocrit
D diameter of the veins
ANSWER:D
1447 Which will produce a decrease in arterial blood pressure?
A vasoconstriction.
B increased parasympathetic stimulation.
C increased blood osmolarity.
D increased cardiac output.
ANSWER:B
1448 What achieves short term control of blood pressure?
A hormonal mechanisms
B the kidneys
C changes in concentration of chemicals such as O, CO, H+, K+.
D neural mechanisms
ANSWER:D
1449 What does the term “systolic pressure” refer to?
A the value, in mmHg, that appears in the denominator of a blood pressure measurement
B the peak pressure in the blood due to the contraction of the left ventricle.

C the minimum pressure in the aorta prior to left ventricular contraction
D the difference between maximum and minimum arterial blood pressures.
ANSWER:B
1450 What are the receptors that are sensitive to blood pressure called?
A pacinian corpuscles
B nociceptors
C baroreceptors
D chemoreceptors
ANSWER:C
1451 Which situation below would make the heart beat faster?
A an increase in sympathetic impulses along the cardioaccelerator nerves
B an increase in parasympathetic impulses along the cardioaccelerator nerves
C an increase in sympathetic impulses along the vagus nerves
D an increase in parasympathetic impulses along the vagus nerves
ANSWER:A
1452 Which of the following will NOT increase cardiac output?
A increasing strength of contraction
B increasing stroke volume
C increasing heart rate
D increasing total peripheral resistance
ANSWER:D
1453 Which of the following will increase cardiac output?
A an increase in sympathetic impulses
B an increase in parasympathetic impulses
C a faster stream of impulses from the baroreceptors
D an increase in vasodilation
ANSWER:A
1454 Angiotensin II does all of the following except one. Which one?
A stimulates thirst
B causes the release of aldosterone
C causes the release of ADH
D stimulates peripheral vasodilation
ANSWER:D

1455 What is the consequence when ADH is released?
A blood osmolarity increases
B the permeability of the collecting ducts to water is increased
C peripheral vasodilation increases
D blood pressure decreases
ANSWER:B
1456 If dietary salt intake is excessive, which of the following will NOT occur?
A Less ADH will be secreted
B The osmolarity of extracellular fluids will increase
C The thirst centre will be stimulated
D The extracellular fluid volume increases
ANSWER:A
1457
The antihypertensive drugs known as “ACE inhibitors” function by doing which of the
following?
A preventing the release of ADH
B blocking the formation of angiotensin II
C blocking the release of renin
D preventing the entry of Ca++ to vascular smooth muscle.
ANSWER:B
1458 Which hormone produces a decrease in arterial blood pressure?
A vasopressin
B ANP
C ADH
D angiotensin II
ANSWER:B
1459 Which completed statement is NOT true? Peripheral resistance:
A increases if diameter of blood vessels increases
B is greater if the total length of blood vessels is greater
C increases if viscosity of blood increases
D is greater than pulmonary resistance
ANSWER:A
1460 Complete the following sentence correctly for a person at rest. Hypertension
A occurs when blood volume is too low

B refers to the increased blood pressure in the legs while standing
C indicates that resistance to blood flow is low
D is when systolic blood pressure is more than 0 mmHg
ANSWER:D
1461 An increase in parasympathetic impulses along the vagus nerve causes
A dilation of the arterioles
B a decrease in blood pressure
C an increase in vasoconstriction
D increases the force of myocardial contraction
ANSWER:B
1462 Which of the following would cause a rise in mean arterial blood pressure?
A sympathetic impulses along the cardio-accelerator nerves.
B changing from a standing position to a supine position.
C generalized vasodilation of blood vessels.
D a severe haemorrhage.
ANSWER:A
1463 What is the role of angiotensin II? To:
A decrease blood pressure by promoting vasodilation of veins.
B increase blood pressure by promoting vasoconstriction of arterioles
C decrease blood pressure by promoting excretion of water in urine.
D increases blood pressure by promoting absorption of Na+.
ANSWER:B
1464 One of the following is a vasodilator. Which one?
A atrial natriuretic peptide (ANP).
B angiotensin II.
C epinephrine (adrenaline).
D an increase in sympathetic impulses.
ANSWER:A
1465 To what does the term ventricular systole refer?
A relaxation of the ventricles.
B relaxation of the atria.
C contraction of the myocardium.
D contraction of the ventricles

ANSWER:D
1466 What is the pressure gradient produced by the left ventricle equal to?
A the mean arterial pressure.
B the difference between mean arterial pressure and pressure at the start of the capillaries.
C
the difference between mean arterial pressure and pressure at the start of the capillaries,
divided by the distance between the start of the aorta and capillaries.
D
the mean arterial pressure divided by the distance between the start of the aorta and
capillaries.
ANSWER:C
1467 What is one of the determinants of the resistance to blood flow?
A blood viscosity.
B cardiac output.
C heart rate.
D the blood osmolarity.
ANSWER:A
1468 Which one of the following will make blood pressure fall?
A increased cardiac output
B increased heart rate
C increased vasodilation
D increased peripheral resistance
ANSWER:C
1469 Hypertension in adults may be defined as:
A Excessive decrease in blood pressure.
B Mean arterial pressure greater than 0 mm Hg.
C Systolic blood pressure less than 00 mm Hg when resting
D Systolic blood pressure persistently greater than 0 mm Hg when resting
ANSWER:D
1470
Which of the following statements about the cardiovascular control centre of the brain is
FALSE?
A increased impulses along parasympathetic fibres causes vasoconstriction.
B increased output along the sympathetic fibres INcrease heart rate.
C output along of the parasympathetic fibres DEcreases heart rate.
D decreased output along the sympathetic fibres causes dilation of arterioles.

ANSWER:A
1471 Which of the following does Angiotensin II cause to happen?
A Atrial natriuretic peptide to be released.
B The collecting ducts in the kidney to become permeable to water.
C The release of antidiuretic hormone to be supressed.
D Aldosterone to be released.
ANSWER:D
1472 One of the following does NOT contribute to peripheral resistance. Which one?
A heart rate.
B blood viscosity.
C diameter of blood vessels.
D length of blood vessels
ANSWER:A
1473
Which of the following statements about the cardiovascular control centre of the brain is
TRUE?
A increased impulses along parasympathetic fibres causes vasoconstriction.
B output along the sympathetic fibres INcrease heart rate.
C output along the parasympathetic fibres INcrease heart rate.
D output along the sympathetic fibres DEcrease heart rate.
ANSWER:B
1474 Which three hormones have a role in regulating blood pressure?
A angiotensin II, ADH and ANP
B renin, angiotensin II and ADH
C vasopressin, ADH and ANP
D angiotensin II, ACE and ADH
ANSWER:A
1475 Which of the following statements about the cardiovascular centre of the brain is FALSE?
A it consists of the cardiac centre and the vasomotor centre.
B output along fibres of the sympathetic nervous system DEcrease heart rate.
C output along fibres of the parasympathetic nervous system DEcrease heart rate.
D output along fibres of the sympathetic nervous system INcrease heart rate.
ANSWER:B
1476 Which statement about cardiac output is correct?

A cardiac output is peripheral resistance multiplied by stroke volume.
B mean arterial pressure multiplied by peripheral resistance is cardiac output.
C cardiac output is heart rate multiplied by stroke volume.
D cardiac output is blood volume multiplied by heart rate.
ANSWER:C
1477 Which of the following would produce a DECREASE in heart rate?
A sympathetic impulses along the cardioaccelerator nerves.
B increased sympathetic impulses along the vasomotor nerves
C decreased sympathetic impulses along the vasomotor nerves.
D parasympathetic impulses along the vagus nerve.
ANSWER:D
1478
A rise in arterial blood pressure stretches the vessel walls which contain baro- receptors.
Which of the following responses does this produce? The barorecep- tors send a:
A slower stream of impulses to the vasomotor centre which inhibits it.
B faster stream of impulses to the vasomotor centre which inhibits it.
C slower stream of impulses to the vasomotor centre which stimulates it.
D faster stream of impulses to the vasomotor centre which stimulates it.
ANSWER:B
1479 Why is the blood pressure in the pulmonary arteries less than in the aorta?
A the expansion and contraction of the lungs pumps blood through its blood vessels.
B blood flowing to the lungs does not need to overcome gravity.
C pulmonary resistance is greater than the peripheral resistance.
D pulmonary resistance is less than peripheral resistance
ANSWER:D
1480
When blood pressure is measured in the brachial artery by the auscultatory method, why
should the arm be at the same level as the heart?
A
if the arm is lower than the heart, brachial artery pressure will be lower as blood flow is
assisted by gravity.
B
if the arm is higher than the heart, brachial artery pressure will be higher as more force is
required to pump blood up hill.
C to avoid any hydrostatic pressure effects on the brachial artery pressure.
D there is no reason for it, the practice is part of “nursing ritual”.
ANSWER:C

1481 Why is blood pressure stated in units of “mm Hg”? Because…
A the first barometers operated with mercury.
B standard International (SI) units are not required in human biology.
C the haemoglobin molecule contains an atom of mercury.
D blood pressure is a length.
ANSWER:A
1482
Blood may flow in the aorta with a speed of 0 cm/s and in the capillaries with a speed of only
0. m/s. Why is there such a large difference in speed?
A the capillaries are much further from the heart than the aorta.
B
the very narrow capillaries present a large resistance to blood flow com- pared to the large
diameter aorta.
C
the total cross-sectional area of the lumens of the capillaries is much greater than the cross-
sectional area of the aorta.
D the length of the aorta is short compared to the length of a capillary.
ANSWER:C
1483 Which statement about cardiac output is correct? Cardiac output is:
A the sum of volume of blood pumped by left and right ventricles per minute.
B the mean arterial pressure divided by total peripheral resistance
C the mean arterial pressure multiplied by stroke volume.
D the blood volume multiplied by heart rate.
ANSWER:B
1484 In which of the following situations would blood pressure be increased?
A antidiuretic hormone (ADH) secretion is inhibited.
B the kidneys absorb less water before it is excreted as urine.
C the extracellular fluid volume decreases.
D the extracellular fluid volume increases.
ANSWER:D
1485
In the circulatory system, why does an increase in cardiac output cause an increase in volume
flow rate (of blood)? Because:
A an increased cardiac output causes vasoconstriction.
B as cardiac output increases so blood viscosity increases.
C as cardiac output increases, the resistance of the systemic circulation to blood flow decreases.

D cardiac output and volume flow rate are the same thing
ANSWER:D
1486
The Korotkoff sounds that are listened for during a blood pressure measure- ment by the
auscultatory method are caused by:
A turbulent blood flow in the aorta
B the difference between systolic pressure and cuff pressure.
C turbulent flow in the collapsed brachial artery
D the opening and closing of the heart valves
ANSWER:C
1487 What is “autoregulation”? It is the adjustment of blood flow to each tissue due to:
A hormonal control
B neural control
C systemic factors
D local factors
ANSWER:D
1488 Which three factors affect blood pressure?
A heart rate, stroke volume and total peripheral resistance.
B cardiac output, heart rate and pulse pressure.
C total peripheral resistance, cardiac output and blood volume.
D diastolic pressure, systolic pressure and pulse pressure.
ANSWER:C
1489 Which of the following statements could be applied to “external respiration”?
A Exchange of gases between alveolar air and the blood in pulmonary capillaries.
B Exchange of dissolved gases between blood in tissue capillaries and the body tissues.
C The production of CO from organic molecules in the cells by using O.
D The inhalation of atmospheric air into the lungs followed by exhalation.
ANSWER:A
1490
Which anatomical structures does the “conducting zone” of the lower respira- tory tract
contain?
A Eustachian tube, larynx and trachea.
B Primary, secondary and tertiary bronchi and bronchioles.
C Nares, conchae, olfactory mucosa and sinuses.
D Nasopharynx and larynx.

ANSWER:B
1491 What is the function of the cilia on the cells that line the bronchial tree?
A They help mix the inhaled fresh air with the residual air contained in the bronchial tree.
B They slow the movement of air to allow for efficient exchange of gases.
C They move the mucus on the cell surface up out of the bronchial tree.
D They filter particles from inhaled air.
ANSWER:C
1492 One of the following statements is correct. Which one?
A The visceral pleura is attached to the chest wall and the parietal pleura is attached to the lung.
B The two lungs and their associated structures are known as the pneumothorax.
C The hilum is a serous membrane that surrounds each lung separately.
D A negative pressure is maintained between the two lung pleura.
ANSWER:D
1493 What term is applied to the volume of air that moves into the lungs while breathing at rest?
A anatomical dead space
B inspiratory reserve capacity
C tidal volume
D residual volume
ANSWER:C
1494
Severing the nerves that innervate the breathing muscles may lead rapidly to death. Will a
spinal cord break between the level of cervical vertebrae and leave the victim able to
breathe? Choose the answer with the correct reason.
A
No. The breathing muscles are innervated by spinal nerves that leave the spinal cord at the
level of each thoracic vertebra.
B Yes. The diaphragm will work as it is innervated by nerves arising from C to C.
C
No. The breathing muscles are innervated by autonomic impulses from the respiratory centre
which is located in the brain stem.
D
Yes. The muscles of breathing are innervated by the sympathetic nervous system which is
unaffected by damage to the somatic nervous system.
ANSWER:B
1495
Which molecule or ion dissolved in blood is able to stimulate the central che- moreceptors of
the brain’s respiratory centre?

A CO
B HO+
C O
D Ca++
ANSWER:A
1496 Between which two anatomical structures does the Larynx lie?
A The nares and the choanae
B The epiglottis and the trachea
C The choanae and the glottis
D The glottis and the epiglottis
ANSWER:B
1497 What is the function of the ciliated cells of the respiratory epithelium?
A to trap inhaled particles not removed by the nasal cavity
B to secrete a mucus layer onto the epithelium
C to move mucus and trapped particles up the bronchial tree
D to secrete surfactant that decreases water surface tension
ANSWER:C
1498
Why is it that bronchioles can constrict and so reduce their diameter while sec- ondary
bronchi and respiratory bronchioles cannot constrict?
A
Bronchioles have smooth muscle but no cartilage in their walls while sec- ondary bronchi are
supported by cartilage.
B
Bronchioles have smooth muscle but no cartilage in their walls while respi- ratory
bronchioles are supported by cartilage.
C
Bronchioles have cartilage but no smooth muscle in their walls while sec- ondary bronchi are
supported by cartilage.
D
Bronchioles have cartilage but no smooth muscle in their walls while respi- ratory
bronchioles only have smooth muscle in their walls.
ANSWER:A
1499
The goal of respiration is to control the concentration of which substances dis- solved in the
blood?
A oxygen
B oxygen and carbon dioxide
C oxygen, carbon dioxide and hydrogen ions

D oxygen, carbon dioxide, hydrogen ions and ATP
ANSWER:C
1500
The walls of the following structures are all supported by cartilage except for one of them.
Which one?
A bronchioles
B trachea
C bronchi
D larynx
ANSWER:A
1501 What constitutes the respiratory membrane?
A the parietal and visceral pleurae and enclosed pleural fluid
B capillary and alveolar epithelial cells, their basement membranes and adja- cent fluid
C the alveolar surface fluid and epithelial cells
D alveolar epithelial and septal cells, ciliated cells, macrophages and surfactant.
ANSWER:B
1502 What is the “cribriform plate”?
A That part of the nose with three folds of tissue called conchae.
B The structure that separates the nose from the nasopharynx.
C Part of the ethmoid bone through which olfactory nerves pass.
D The nose structure through which air is warmed and humidified as it passes.
ANSWER:C
1503 What is a good definition of “internal respiration”?
A The exchange of gases between body tissues and capillary blood.
B Ventilation of the lungs.
C The production of ATP and CO from small molecules using O.
D The exchange of gases between the alveoli and pulmonary capillaries.
ANSWER:A
1504 Which statement may be used to define a bronchiole?
A They are the airways that branch from the left and right primary bronchi.
B They are kept open by “C” shaped rings of cartilage.
C Their walls have supporting cartilage between smooth muscle.
D Their walls contain smooth muscle but no cartilage.
ANSWER:D

1505 One statement below about the larynx is correct. Which one?
A It has walls lined with ciliated cells
B It has walls made of cartilage
C It has walls made of bone
D It has walls made of muscle
ANSWER:B
1506 To what does the term “external respiration” refer?
A ventilation of the lungs (breathing)
B exchange of gases between alveolar air and lung capillaries
C the production by cells of ATP from small molecules and oxygen
D exchange of dissolved gases between capillary blood and body tissues
ANSWER:B
1507 What distinguishes bronchioles from the larger bronchi?
A bronchioles have no cartilage in their walls
B bronchioles have smooth muscle in their walls
C bronchioles collapse between exhalation and inhalation
D the alveoli open onto these air passages
ANSWER:A
1508 Which structures are included in the respiratory membrane?
A alveolar fluid and surfactant
B alveolar fluid, surfactant and epithelial cells of alveoli
C alveolar fluid, surfactant, epithelial cells of alveoli and basement membrane of epithelial cell
D
alveolar fluid, surfactant, epithelial cells of alveoli, basement membrane of epithelial cell and
endothelial cell of capillary
ANSWER:D
1509 How is the diaphragm innervated?
A from the respiratory centre in concert with chemoreceptors that detect blood oxygen level
B by the spinal nerves arising from thoracic vertebrae at the same level
C by the phrenic nerve arising from vertebrae C to C
D by the vagus nerve arising from the medulla oblongata
ANSWER:C
1510 What mechanism transports the largest portion of oxygen around the body?

A oxygen is carried bound to plasma proteins
B oxygen is transported in solution dissolved in blood plasma
C oxygen is bound to haemoglobin within red blood cells
D oxygen is transported as bicarbonate after reacting with water to form car- bonic acid
ANSWER:C
1511 Which structures constitute the “upper respiratory tract”?
A nose, pharynx and larynx
B larynx, epiglottis and bronchi
C trachea, bronchi and bronchioles
D terminal bronchioles, alveoli and pleurae
ANSWER:A
1512
The lists below include of four respiratory tract structures. Which list has them in the order
that inhaled air would pass through them on the way to the lungs?
A glottis, pharynx, conchae, trachea
B nares, pharynx, larynx, conchae
C conchae, pharynx, larynx, trachea
D pharynx, conchae, trachea, glottis
ANSWER:C
1513 What passes through the foramina of the cribriform plate of the ethmoid bone?
A inhaled air on its way through the nose.
B tubes that drain the sinuses of the facial bones.
C nerve fibres associated with the sense of smell.
D blood vessels that supply the nasal mucosa.
ANSWER:C
1514 What function is served by the goblet/mucus cells of the bronchial “tree”?
A they trap small inhaled particles.
B they secrete mucus onto the surface of the airways
C they increase the surface area available for gas exchange.
D they move mucus up the bronchial tree.
ANSWER:B
1515
The central chemoreceptors in the brain stem increase breathing rate in response to which
stimulus?
A an increase in CO concentration in the CSF

B an increase in CO and H+ concentration in the CSF
C a decrease in O concentration in the CSF
D a decrease in O concentration in the blood
ANSWER:B
1516
In what form is the majority of carbon dioxide that is generated by cellular respiration,
transported to the lungs?
A as dissolved carbon dioxide in the blood plasma.
B bound to haemoglobin in red blood cells.
C as carbonic acid inside red blood cells.
D as bicarbonate ions in the blood plasma.
ANSWER:D
1517 A bronchiole differs from tertiary (and smaller) bronchi in that it:
A has cartilage in its wall (& bronchi do not)
B does not have cartilage in its wall (& bronchi do)
C has smooth muscle in its wall (& bronchi do not)
D does not have smooth muscle in its wall (& bronchi do)
ANSWER:B
1518 The term “cellular respiration” is applied to:
A exchange of gases in the lungs
B ventilation of the lungs (breathing)
C exchange of gases in the body tissues
D the production of ATP in the cells
ANSWER:D
1519 The respiratory centre in the brain is sensitive to
A an increase in H+ concentration in the CSF.
B a decrease in O concentration in the blood.
C an increase in H+ concentration in the blood.
D a decrease in O concentration in the CSF.
ANSWER:A
1520 Trauma that severs the spinal cord between C and C will
A mean that the phrenic nerve has been severed
B mean that artificial ventilation will be required to sustain life
C cause the diaphragm to lose innervation but allow the intercostal muscles to operate

D cause the intercostal muscles to lose innervation but allow the diaphragm to operate
ANSWER:D
1521 What happens when carbon dioxide levels in the blood decrease to below normal?
A pH of the blood decreases.
B the blood becomes more acidic.
C the concentration of hydrogen ions in the blood decreases.
D pH does not change.
ANSWER:C
1522 In a healthy person, which of these lung volumes should be the largest?
A tidal volume
B vital capacity
C expiratory reserve volume
D residual volume
ANSWER:B
1523
What term refers to the exchange of gases between alveolar air and blood in the pulmonary
capillaries?
A inhalation
B internal respiration
C ventilation
D external respiration
ANSWER:D
1524 What is a cavity in a skull bone that is lined with mucus membrane?
A sinus
B bronchiole
C glottis
D larynx
ANSWER:A
1525 What is the function of ciliated cells in the lungs?
A they form part of the respiratory membrane
B to move mucus out of the bronchial tree
C to secrete surfactant onto the lining of the alveoli
D to phagocytose inhaled bacteria
ANSWER:B

1526 What ensures that the lungs expand as the chest wall expands?
A secreted surfactant
B negative pressure between the pleura
C serous liquid secreted by the pleura
D the elastic recoil of the alveolar tissue
ANSWER:B
1527
Which of the following is responsible for increasing respiratory activity under normal
conditions?
A decreased CO level in blood
B decreased O level in blood
C increased CO level in blood
D increased blood pH
ANSWER:C
1528 To what does “internal respiration” refer?
A inhalation
B exchange of gases between alveolar air and blood in the pulmonary capillaries.
C exchange of gases between capillary blood and interstitial fluid.
D the production of ATP from organic molecules using oxygen.
ANSWER:C
1529 Which structures are called bronchioles?
A respiratory passageways that have cartilage in their walls
B the tubes that open from the left and right primary bronchi.
C the tubes that enter a lobule
D the tubes that enter an alveolar sac.
ANSWER:C
1530 Which of the following is NOT part of the respiratory membrane?
A the basement membrane of alveolar epithelial cells
B the plasma membrane of red blood cells
C capillary endothelial cells
D alveolar fluid and surfactant
ANSWER:B
1531 How is the diaphragm innervated?
A by the phrenic nerves arising from C to C

B by the vagus nerves from the respiratory centre
C by cranial nerves arising from C to C
D by nerves arising from the spinal cord from T to T.
ANSWER:A
1532 Which molecule has the greatest effect in controlling lung ventilation?
A oxygen in the blood
B hydrogen ions in the blood
C carbon dioxide in the blood
D oxygen in the cerebrospinal fluid
ANSWER:C
1533 Which is the incorrect statement among the four below?
A more oxygen is carried bound to haemoglobin than dissolved in plasma.
B more carbon dioxide is carried bound to haemoglobin than dissolved in plasma.
C haemoglobin buffers hydrogen ions derived from carbon dioxide.
D carbonic anhydrase is the enzyme that binds oxygen to haemoglobin.
ANSWER:D
1534 Which structures comprise the lower respiratory tract?
A pharynx, larynx, trachea
B larynx, trachea, bronchi
C nose, pharynx, larynx
D trachea, bronchi, lungs
ANSWER:D
1535 A bronchiole differs from a bronchus in that it is:
A unable to change its diameter (& a bronchus can)
B able to change its diameter (& a bronchus cannot)
C a smaller diameter tube than is a bronchus
D a larger diameter tube than is a bronchus
ANSWER:B
1536 With regard to lung ventilation, what does “dead space” refer to?
A air in the conducting zone of the bronchial tree
B air remaining in the alveoli after an exhalation at maximal effort.
C air between the parietal and visceral pleura
D the difference between the volume of a maximum inhalation and the tidal volume.

ANSWER:A
1537 In which form is the majority of CO transported in the blood?
A as a dissolved solute
B bound to plasma proteins
C as carbonic acid molecules
D as bicarbonate (HCO ) ions
ANSWER:D
1538 Which of the following lists the components of the respiratory membrane?
A alveolar epithelial cells, capillary endothelial cells and their basement membranes
B
fluid and surfactant alveolar epithelial cells, capillary endothelial cells and their basement
membranes
C visceral pleura, parietal pleura and serous fluid.
D ciliated epithelial cells, mucus cells and secreted mucus
ANSWER:B
1539 To which of the following does the term “respiration” NOT apply?
A The conversion of carbon dioxide to bicarbonate ions for transport to the lungs
B The exchange of gases between alveolar air and capillary blood
C The derivation of energy from organic molecules in the cells
D The exchange of gases between capillary blood and body tissues
ANSWER:A
1540 The “lower respiratory tract” includes all those structures below which of the following?
A the internal nares
B larynx
C trachea
D conducting zone
ANSWER:B
1541 What are the cells that produce surfactant called?
A mucus cells
B ciliated cells
C alveolar macrophages
D type II pneumocytes
ANSWER:D
1542 Which of the following statements is correct?

A tidal volume is maximum volume that can be inhaled and exhaled
B FEV is the maximum volume of air that can be forcefully exhaled in second
C
expiratory reserve volume is the maximum volume of air that can be exhaled after a deep
inhalation
D vital capacity is the expiratory reserve volume added to the inspiratory reserve volume
ANSWER:B
1543 What happens when carbon dioxide levels in the blood increase?
A pH of the blood increases.
B the blood becomes more alkaline.
C the number of hydrogen ions in the blood decreases.
D the blood becomes more acidic.
ANSWER:D
1544 Chemoreceptors in the medulla oblongata are sensitive to
A increases in blood oxygen content.
B increases in blood carbon dioxide.
C increases in blood pH.
D both choices and B.
ANSWER:D
1545
What does sympathetic nervous system stimulation to the smooth muscle lay- ers in the
bronchioles cause?
A bronchoconstriction
B bronchodilation
C an increase in tidal volume
D increase in activity of the cilia
ANSWER:B
1546
Patients with diabetes mellitus who neglect insulin therapy rapidly metabolize lipids, and
there may be an accumulation of the acidic by-products of lipid metabolism in the blooWhat
effect would this have on respiration?
A increase in respiratory rate
B decrease in respiratory rate
C decrease in respiratory rate if oxygen is reduced
D no influence on respiratory rate
ANSWER:A

1547 Which of these structures has no cartilage in it?
A Epiglottis
B Trachea
C Bronchi
D Alveoli
ANSWER:D
1548
Where does the actual gas exchange between inspired air and the blood in the capillaries
occur? In the:
A bronchi
B bronchioles
C alveolar ducts and alveoli
D respiratory bronchioles.
ANSWER:C
1549 Which of these statements concerning ventilation is NOT correct?
A During inspiration, the pressure in the alveoli is less than atmospheric pressure.
B Contraction of the neck muscles decreases the volume of the thoracic cavity.
C When the diaphragm contracts, thoracic cavity volume increases.
D During quiet breathing, passive recoil of the lung and thoracic wall cause expiration.
ANSWER:B
1550 With regard to the respiratory centre, which of the following is TRUE?
A Blood oxygen concentration affects the respiratory centre.
B Anaesthetics don’t affect respiration.
C Raised intracranial pressure increases ventilation.
D Narcotic drugs may depress ventilation
ANSWER:D
1551 In the control of respiration, which of the following is NOT TRUE?
A
Peripheral chemoreceptors respond to changes in oxygen and carbon diox- ide concentration
in the blood
B
Chemoreceptors in the central nervous system respond to changes in carbon dioxide
concentration in the blood
C
Respiration responds to smaller changes in the blood concentration of oxy- gen than carbon
dioxide
D Central chemoreceptors are sensitive to changes in the pH of the cerebro- spinal fluid

ANSWER:B
1552 Gas exchange takes place in the:
A larynx
B bronchioles
C alveoli
D pleura
ANSWER:C
1553 The walls of the trachea are held open by which of the following?
A nerve impulses
B rings of cartilage
C fine bones
D smooth muscle contractions
ANSWER:B
1554 Oxygen and carbon dioxide cross the respiratory membrane by the process of:
A counter-current exchange
B diffusion
C active transport
D oxygen – carbon dioxide pump
ANSWER:B
1555 Which of the following is NOT TRUE?
A Raised intracranial pressure may depress respiration
B Anaesthetics never affect respiration
C Receptors in the airways trigger sneezing
D Narcotic drugs may suppress respiration
ANSWER:B
1556 Normal expiration in a person at rest is due to
A elastic tissue in the lung
B contraction of abdominal muscles
C contraction of the expiratory muscles
D diffusion
ANSWER:A
1557 A molecule that is important in maintaining normal lung structure is:
A immunoglobulin

B haemoglobin
C peroxidase
D surfactant
ANSWER:D
1558
The volume of air which moves in and out of the lungs during a normal quiet respiratory
cycle is called the:
A tidal volume
B vital capacity
C ventilatory volume
D pulmonary capacity
ANSWER:A
1559 What are the main muscles involved in normal inspiration?
A muscles of the neck
B abdominal muscles
C intercostal muscles
D intercostals and the diaphragm
ANSWER:D
1560 What is the number of breaths per minute called? The:
A respiratory rate
B respiratory speed
C pulmonary index
D respiratory volume
ANSWER:A
1561
A man runs up a flight of stairs. His respiratory rate rose from breaths per minute (resting) to
bpm (at the top of stairs). The increase was probably a result of:
A increased blood pH
B increased concentration of CO in the blood
C decreased concentration of O in the blood
D decreased concentration of CO in the blood
ANSWER:B
1562
Why is oxygen therapy – allowing a patient to breathe in an atmosphere in which the
proportion of oxygen is greater than 0 % – beneficial? Because the

A
partial pressure of CO in the inhaled air is decreased making it easier to clear CO from the
lungs.
B
partial pressure of oxygen in the lungs is increased allowing more oxygen to dissolve in the
alveolar fluid.
C
oxygen molecule is smaller than the nitrogen molecule so a greater number of moles of air
can be drawn into the lungs with each breath.
D
patient is required to inhale less frequently and this reduces the strain on the respiratory
system.
ANSWER:B
1563 What are the membranes that surround each lung called? The
A parietal and visceral membranes.
B parietal and visceral meninges
C pleura
D peritoneum
ANSWER:C
1564 Which membrane surrounds the lungs?
A the pericardium
B the pleura
C the mediastinum
D the diaphragm
ANSWER:B
1565 Which of the following buffer systems of the body is affected by the action of the lungs?
A protein
B monohydrogen-phosphate/dihydrogen-phosphate
C ammonia/ammonium
D carbonic acid/bicarbonate
ANSWER:D
1566
What does it mean if oxygenated blood leaving the alveolar capillaries has an oxygen partial
pressure of 00 mmHg?
A oxygen will have diffused from the blood into the alveoli.
B the partial pressure of oxygen in the alveoli was 00 mmHg
C the partial pressure of oxygen in the alveoli was less than 00 mmHg
D the partial pressure of oxygen in the alveoli was more than 00 mmHg

ANSWER:B
1567 When the diaphragm contracts which of the following will happen in the lungs?
A air pressure will increase, volume will decrease and exhalation will occur.
B air pressure will decrease, volume will increase and exhalation will occur.
C air pressure will decrease, volume will increase and inhalation will occur.
D air pressure will increase, volume will increase and inhalation will occur.
ANSWER:C
1568 What does it mean when the concentration of dissolved oxygen in blood is 00 mmHg?
A
the oxygen dissolved in blood exerts a pressure of 00 mmHg over and above the blood
pressure generated by the heart.
B 00 mmHg of the blood pressure, is due to the dissolved oxygen within it.
C
the blood had been exposed to air in the lungs that contained oxygen at a partial pressure of
00 mmHg.
D the dissolved oxygen exerts an osmotic pressure of 00 mmHg.
ANSWER:C
1569
A pump used to clear an airway of obstructing liquid expands the volume in its chamber to
produce a:
A negative pressure and liquid is pushed by atmospheric pressure into the chamber.
B positive pressure and liquid is pushed by atmospheric pressure into the chamber.
C negative pressure and liquid is sucked up into the chamber.
D positive pressure and liquid is sucked by atmospheric pressure into the chamber.
ANSWER:A
1570
Boyle’s law may be stated as: “when the volume of an enclosed gas expands, its pressure
decreases”. Thus the pressure in our lungs will:
A increase as our diaphragm contracts.
B be negative as we breathe out.
C be negative as our rib cage moves up and out.
D be positive as our rib cage moves up and out
ANSWER:C
1571
Boyle’s law may be stated: Provided that the temperature does not change, the volume of a
fixed amount of gas decreases as its pressure increases (and vice versa). Which statement
concerning the pressure of the air in the lungs is con- sistent with Boyle’s law?
A It will decrease as the chest expands.

B It decreases as we breathe out.
C It increases when we contract our diaphragm.
D It decreases as our intercostal muscles relax.
ANSWER:A
1572
Boyle’s law states that pressure multiplied by volume is a constant value. Hence which of the
following statements is correct?
A The amount of air in the lungs will increase when the pressure inside them increases.
B A positive pressure is produced in the lungs when the chest expands
C A negative pressure is produced in the lungs when the chest expands.
D The amount of air in the lungs will decrease when the pressure in them decreases.
ANSWER:C
1573
If the partial pressure of oxygen in the atmosphere was halved, what effect would this have
on the amount of oxygen that would now dissolve in the alveo- lar fluid? It would
A decrease to one quarter its former value.
B decrease to one half its former value.
C be about the same as before.
D increase to one and a half times its former value.
ANSWER:B
1574
If the partial pressure of oxygen in the air contained in the alveoli of the lungs is kPa, then
the partial pressure of oxygen dissolved in the alveolar fluid will be:
A Very close to kPa
B Significantly greater than kPa
C Significantly less than kPa.
D
Unable to be determined without the solubility coefficient of oxygen and the temperature of
the alveolar fluid.
ANSWER:A
1575
One of the reasons that the mixture of gases in the air contained in the alveoli of the lungs,
differs from atmospheric air is that alveolar air is:
A saturated with water vapour whereas atmospheric air is not.
B at a higher temperature than atmospheric air.
C at a higher pressure than atmospheric air.
D enriched with nitrogen as it diffuses into the alveoli from the blood.
ANSWER:A

1576
If the pressure between the visceral pleura of the lungs and the parietal pleura of the thoracic
cage is − mmHg, then
A the lung will collapse.
B exhalation is occurring.
C the pressure is above atmospheric pressure.
D the lungs will fill with air.
ANSWER:D
1577
What are the conditions under which O diffuses from the blood plasma into red blood cells
and attaches to haemoglobin? When partial pressure of oxygen dis- solved in plasma is
A 0 mmHg in the systemic capillaries.
B 00 mmHg in the alveolar capillaries.
C 0 mmHg in the alveolar capillaries.
D 00 mmHg in the systemic capillaries.
ANSWER:B
1578
After an inhalation, the partial pressure of O in the alveoli is 0 mmHg and of CO is 0 mmHg.
What will be the concentration of these gases dissolved in blood in the capillaries leaving the
alveoli?
A O = 0 mm CO = 0 mmHg
B O = 0 mmHg, CO = mmHg
C O = 0 mmHg, CO = mmHg
D O = 00 mmHg, CO = 0 mmHg
ANSWER:D
1579 If arterial blood entering the lung has pCO = mmHg & alveolar air has pCO = 0 mmHg then
A CO will move out of blood into the alveoli
B CO will move out of alveoli into the blood
C blood leaving the alveoli will have pCO = mmHg
D there will be an increase in the concentration of carbonic acid in the blood
ANSWER:A
1580
Recall Boyle’s law applied to the lungs. Which of the following would occur as air pressure
in the lungs increases?
A volume increases

B volume decreases
C the lungs expand
D the diaphragm contracts
ANSWER:B
1581 Inhalation of air into the lungs is correctly described by which of the following?
A
the action of the diaphragm and the ribs create a positive pressure in the thoracic cavity
which causes air to move into the lungs.
B
the volume of the thoracic cavity is increased as muscles relax, thus increas- ing the pressure
and air is forced into the lungs.
C
the thoracic cavity decreases in volume as muscles relax and pressure decreases so external
air is forced into the lungs.
D
the lungs expand as muscles contract, this creates a negative pressure so air is forced into the
lungs.
ANSWER:D
1582
Air flows from a region of high pressure to a region of lower pressure. To explain the
process of inhalation and exhalation, you need this fact and which other?
A Boyle’s law
B Dalton’s law
C Henry’s law
D Charles’ law
ANSWER:A
1583 Which word correctly completes the statement: “All motor neurons are…”
A interneurons
B multipolar
C bipolar
D unipolar
ANSWER:B
1584 In the peripheral nervous system, which cells form the myelin sheath?
A Ependymal cells
B Schwann cells
C Astrocytes
D Oligodendrocytes
ANSWER:B

1585
What is the gap between the plasma membranes of a neuron that conducts an incoming
signal and the cell that is going to receive the signal called?
A neuromuscular junction
B intercellular cleft
C synaptic cleft
D intercalated disc
ANSWER:C
1586 What name is given to the cells in the nervous system that produce nerve impulses?
A neurotransmitters
B nerves
C neurons
D neuroglia
ANSWER:C
1587 Which structure carries incoming impulses towards the nerve cell body?
A axon hillock
B axon
C dendrite
D synaptic knobs
ANSWER:C
1588 Which neurons are unipolar?
A neurons in the central nervous system
B neurons in the retina
C sensory neurons
D motor neurons
ANSWER:C
1589
Which glial cells are responsible for forming the myelin sheath around periph- eral nerve
cells?
A Astrocytes
B Schwann cells
C Satellite cells
D Oligodendrocytes
ANSWER:B

1590
Inactive muscle and nerve cells maintain a resting membrane potential. This potential results
in:
A the outside of the cell being negative
B the inside of the cell being positive
C the inside and outside of the cell having the same charge
D the inside of the cell being negative
ANSWER:D
1591 When an action potential arrives at a synapse, what happens first?
A a neurotransmitter is released into the synaptic cleft
B extracellular Na+ crosses the post-synaptic membrane
C Choline in the synaptic cleft enters the nerve cell and is converted to acetyl choline
D extracellular Ca++ enters the nerve cell
ANSWER:D
1592
What is the last part of a nerve cell that is involved when a nerve impulse passes to another
cell?
A synaptic knob
B axon hillock
C dendrite
D axon
ANSWER:A
1593 Which of the following statements is true of neuroglia?
A they are the cells that link motor neurons to sensory neurons
B it is the non-cellular material that lies between neurons
C they have only one dendrite and one axon
D they produce the myelin sheath
ANSWER:D
1594 What can correctly be said about somatic motor neurons?
A they are unipolar neurons
B their cell bodies are in the dorsal root ganglia
C their cell bodies are located in the central nervous system
D they are bipolar neurons
ANSWER:C

1595
What is the effect of the movement of Na+ into a nerve cell followed very soon by the
movement of K+ out of the nerve cell?
A this establishes the resting membrane potential
B these movements are known as depolarisation and repolarisation
C these movements repolarise the cell
D it changes the membrane potential from about −0 mV to about −0 mV
ANSWER:B
1596
There is a space between a neuron and the cell it stimulates, that is crossed by a
neurotransmitter. What is it called?
A synaptic cleft
B voltage-gated channel
C synapse
D post-synaptic membrane
ANSWER:A
1597 Which of the following CAN cross the blood-brain barrier to enter the brain?
A K+
B O
C proteins
D most pharmaceuticals
ANSWER:B
1598 What name is used for a nerve cell?
A neuron
B neuroglia
C ganglion
D astrocyte
ANSWER:A
1599 What is the name of the nerve cell structure that carries incoming impulses towards the cell?
A dendrite
B axon
C cell body
D ganglion
ANSWER:A

1600 Which is the major type of nerve cell in the CNS?
A Anaxonic
B Unipolar
C Bipolar
D Multipolar
ANSWER:D
1601 What is the purpose of the myelin sheath around an axon?
A To control the chemical environment around the nerve cell.
B To phagocytose microbes
C To prevent movement of ions through the nerve cell membrane
D To form the blood-brain barrier.
ANSWER:C
1602 Which nerve cells carry impulses from the brain to the muscles?
A Sensory
B Motor
C Afferent
D Association
ANSWER:B
1603 Which of the following describes an “action potential”?
A The high concentration of Na+ and Cl− outside the cell, and of K+ inside the cell.
B The voltage change that moves along the cell membrane until it reaches the axon hillock.
C
The movement of a neurotransmitter from the pre-synaptic membrane to the post-synaptic
membrane.
D
The movement of Na+ across the cell membrane into the cell, followed by the movement of
K+ out of the cell.
ANSWER:D
1604 What part of the neurone carries the “action potential”?
A the cell body
B the dendrites
C the synaptic knobs
D the axon
ANSWER:D
1605 What type of neurones are motor neurones?

A Anaxonic
B Multipolar
C Bipolar
D Unipolar
ANSWER:B
1606
What event during the action potential causes the resting membrane potential to change from
about −0 mV to about +0 mV?
A K+ ions moving into the cell
B K+ ions moving out of the cell
C Na+ ions moving into the cell
D Na+ ions moving out of the cell
ANSWER:C
1607 Where are the cell bodies of somatic motor neurones found?
A in the peripheral nervous system
B in the central nervous system
C in the dorsal root ganglia
D in the spinal cord
ANSWER:D
1608 What does the term “synapse” refer to?
A the plasma membrane of the axon terminal of a nerve cell.
B
that part of the plasma membrane of the cell being stimulated, that is oppo- site the axon
terminal.
C the gap between the stimulating nerve cell and the receiving cell.
D the place where signal transmission between a nerve cell and the cell it is stimulating occurs.
ANSWER:D
1609 Which statement is true of a multi-polar neuron?
A has many axons attached to the cell body
B is the major type of neuron in the peripheral nervous system
C all sensory neurons are multi-polar
D has many dendrites attached to the cell body
ANSWER:D
1610 Which of the following is a true statement about an “action potential”?

A It refers to the movement of a neuro-transmitter along an axon.
B It travels away from the cell body along the axon.
C It causes K+ to rush into the cell.
D It travels between the dendrite and the axon hillock.
ANSWER:B
1611 Complete the sentence. Neuroglia
A are bundles of axons
B contain cell bodies outside the central nervous system
C are a type of neuron
D include ependymal cells, astrocytes and satellite cells
ANSWER:D
1612 Which of the three structures listed below constitute a nerve cell?
A dendrites, ganglion, myelin sheath
B dendrites, cell body, axon
C neuron, neuroglia, synaptic process
D cell body, synaptic knobs, efferent fibre
ANSWER:B
1613 What are the major type of nerve cells in the CNS?
A multipolar
B sensory
C interneurons
D unipolar
ANSWER:A
1614 Which sequence of ion movements describes the action potential?
A Na+ move out of cell then K+ move in
B K+ move in to cell then Na+ move out
C K+ move out of cell then Na+ move in
D Na+ move into cell then K+ move out
ANSWER:D
1615
What is the space between a neuron and the following neuron, muscle or gland that it
stimulates called?
A Synaptic vesicle
B Ion channel

C Synaptic cleft
D Receptor
ANSWER:C
1616 What feature do the dendrites of a nerve cell have?
A they transmit an action potential
B they contain the cell nucleus and organelles
C they carry incoming impulses to the cell body
D they are connected to the cell body by the axon hillock
ANSWER:C
1617 Most sensory neurones may be described as which of the following?
A multipolar
B bipolar
C having cell bodies within the CNS
D unipolar
ANSWER:D
1618 What is the type of neuroglia that forms the myelin sheath on neurons outside of the CNS?
A Oligodendrocytes
B Satellite cells
C Schwann cells
D Microglia
ANSWER:C
1619 A multipolar neuron has more than one what?
A dendrite attached to the cell body
B axon attached to the cell body
C synaptic terminal attached to the axon
D cell body
ANSWER:A
1620 What is the depolarisation and repolarisation of a nerve cell membrane called?
A graded potential
B action potential
C threshold potential
D resting membrane potential
ANSWER:B

1621 Which best describes a nerve?
A dendrites, cell bodies, axons, Schwann cells
B dendrites, cell bodies, axon hillock, axon terminals, vesicles
C dendrites, cell bodies, axon hillock, axon terminals, Schwann cells, neurotransmitters
D axons, blood vessels, connective tissue, Schwann cells
ANSWER:D
1622 Which of the following would conduct an action potential with the greatest speed?
A myelinated, large diameter fibres
B myelinated, small diameter fibres
C unmyelinated, large diameter fibres
D unmyelinated, small diameter fibres
ANSWER:A
1623 Which of the following is a characteristic of an action potential?
A The signal is graded
B It results due to an influx of potassium ions
C it is an all or none response
D It results from an initial outflow of sodium ions
ANSWER:C
1624 What is meant by an absolute refractory period?
A
at least ms must elapse from the time of the first action potential before a second can be
initiated
B
an action potential cannot be initiated during this period regardless of the strength of the
stimulus
C
an action potential can be initiated if the strength of the stimulus is higher than normal (>0
mV)
D
an action potential can be initiated if the strength of the stimulus is lower than normal (<0
mV)
ANSWER:B
1625 What would happen if a neuron lost its myelin sheath?
A
Na+ would leak out of the axon leaving too few ions to stimulate the Na channels at the next
node to open.
B The neuron would die.

C
More Na+ channels would be exposed allowing freer entry so conduction speed would
increase.
D More K+ channels would be exposed allowing freer exit so the cell would hyperpolarise.
ANSWER:A
1626
Some cells in the body can maintain an electric potential across their cell mem- brane. How
do they do this?
A
By using the sodium-potassium pump to continually eject positive sodium and potassium
ions from the cell.
B By allowing negative chloride ions to enter the cell along their concentra- tion gradient.
C By trapping large cations inside the cell membrane.
D By keeping unequal concentrations of various ions on each side of the cell membrane.
ANSWER:D
1627
In nerve fibres with myelin sheaths, which of the following is true about the electrical
conduction?
A It is ‘saltatory’, so propagates at higher speed
B It requires more energy to send an impulse
C The conduction between adjacent axons is enhanced (‘cross talk’ is increased)
D It is slower due to the separation between the ‘nodes of Ranvier’
ANSWER:A
1628 When we say that the cell membrane is polarised we mean that
A the outside of the cell is negative with respect to the inside
B the inside of the cell is negative with respect to the outside
C there are more Na+ ions and less K+ ions inside the cell than outside.
D Na+ ions have moved out of the cell and K+ ions have moved in.
ANSWER:B
1629 Depolarisation of the cell membrane involves:
A sodium channels opening to allow Na+ to flow in.
B potassium channels opening to allow K+ to flow in.
C chloride pumps quickly pumping large amounts of Cl− outside.
D electrical attraction between K+ inside and Cl− outside.
ANSWER:A
1630 Which of the following statements about the action potential is FALSE?
A The action potential lasts about four milliseconds.

B It is triggered by anions crossing the cell membrane.
C The sequence: ‘Na ions moving in, K ions moving out’ constitutes the action potential.
D Repolarisation follows depolarisation of the cell membrane.
ANSWER:B
1631 When is an action potential initiated? When
A the resting membrane potential changes from −0 mV to +0 mV.
B a nerve impulse has caused some muscle action to be produced.
C the potassium ‘gates’ in the cell membrane open and potassium ions flood into the cell.
D a stimulus, which is above the threshold level, is applied to a receptor.
ANSWER:D
1632 Which of the lists of structures include all of the central nervous system?
A cerebellum, cerebrum, spinal cord, diencephalon, brainstem
B midbrain, spinal cord, autonomic nerves, pons, diencephalon
C midbrain, cerebellum, special sense organs, medulla oblongata
D cerebrum, sensory neurons, motor neurons, cerebellum
ANSWER:A
1633
What are the three meninges and two named “spaces” that surround the brain, in order from
superficial to deep (outermost to innermost)?
A pia, arachnoid, sub-arachnoid, dura, septa
B sub-arachnoid, epidural, dura, pia, arachnoid
C arachnoid, sun-arachnoid, pia, epidural, dura
D epidural, dura, arachnoid, sub-arachnoid, pia
ANSWER:D
1634 Which of the following is NOT composed of “gray matter”?
A spinothalamic tract
B cerebral cortex
C basal nuclei
D post-central gyrus
ANSWER:A
1635 Where in the brain is the “primary motor area”?
A Midbrain
B Thalamus
C Basal nuclei

D Pre-central gyrus
ANSWER:D
1636 The hypothalamus does ALL of the following EXCEPT one. Which one?
A It is the autonomic control centre.
B It directs lower CNS centres to perform actions.
C It produces the rigidly programmed, automatic behaviours necessary for survival
D It performs many homeostatic roles.
ANSWER:C
1637 Which of the following structures together make up the brainstem?
A medulla oblongata, pons, midbrain, cerebellum
B medulla oblongata, pons, midbrain
C medulla oblongata, pons, midbrain, thalamus
D medulla oblongata, pons, midbrain, pineal gland
ANSWER:B
1638 In which of the following places would you NOT find cerebrospinal fluid?
A the sub-arachnoid space
B the third ventricle of the brain
C the epidural space
D the central canal of the spinal cord
ANSWER:C
1639 What is the name of the lobe of the brain that is immediately superior to the cerebellum?
A dorsal
B occipital
C posterior
D parietal
ANSWER:B
1640 In which part of the brain is the thalamus found?
A diencephalon
B cerebrum
C cerebellum
D brainstem
ANSWER:A
1641 Where is the autonomic control centre for most of body homeostasis located?

A In the limbic system
B In the brainstem
C In the hypothalamus
D In the cerebellum
ANSWER:C
1642 Which four structures together make up the brain?
A cerebrum, diencephalon, brainstem and cerebellum
B cerebrum, thalamus, brainstem and cerebellum
C cerebrum, diencephalon, meninges and cerebellum
D spinal cord, diencephalon, brainstem and medulla oblongata
ANSWER:A
1643 Which of the following are NOT part of the cerebral cortex?
A motor areas, sensory areas and association areas
B pre-central gyrus and post-central gyrus
C white matter and basal nuclei
D the lateral ventricles and the thalamus
ANSWER:D
1644 Which part of the brain allows us to control skilled voluntary muscle movements?
A basal nuclei
B cerebellum
C pre-central gyrus
D thalamus
ANSWER:C
1645 Which of the following roles is NOT performed by the hypothalamus?
A autonomic control of heat activity and blood pressure
B relaying visual and auditory information to the cerebral cortex
C production of hormones for the posterior pituitary
D body temperature regulation
ANSWER:B
1646 Which layer of membrane around the brain is the most superficial?
A Dura mater
B Meningeal mater
C Arachnoid mater

D Pia mater
ANSWER:A
1647
Which of the following substances is prevented from entering the brain by the blood-brain
barrier?
A glucose
B nicotine
C pharmaceuticals
D alcohol
ANSWER:C
1648 What part of the brain contains the motor areas and the sensory areas?
A cerebrum
B diencephalon
C brainstem
D cerebellum
ANSWER:A
1649 What part of the brain contains the main visceral control centre of body homeostasis?
A cerebrum
B diencephalon
C brainstem
D cerebellum
ANSWER:B
1650
What part of the brain subconsciously provides precise timing for the move- ments of
learned skeletal muscle contraction?
A cerebrum
B diencephalon
C brainstem
D cerebellum
ANSWER:D
1651 Where does the spinal cord start and finish?
A It extends from the foramen magnum to L – L
B It extends from the foramen magnum to the sacrum
C
It starts at the superior part of the medulla oblongata and extends to the inferior part of the
cauda equina.

D It extends from C to L
ANSWER:A
1652 Where is the cerebral spinal fluid found?
A Between the pia mater and the brain
B Between the dura mater and the arachnoid mater
C Between the dura mater and the pia mater
D Between the arachnoid mater and the brain
ANSWER:D
1653 Which of the following substances CANNOT cross the blood-brain barrier?
A metabolic wastes, toxins, K+
B O, CO and HO
C fats, fatty acids, fat soluble substances
D alcohol, nicotine, anaesthetics
ANSWER:A
1654 What part of the brain contains the midbrain, the pons and the medulla oblongata?
A the diencephalon
B the cerebrum
C the cerebellum
D the brainstem
ANSWER:D
1655 Which of the following statements is INCORRECT?
A The pituitary gland dangles from the hypothalamus by the infundibulum
B The post-central gyrus houses the primary motor cortex
C The thalamus surrounds the third ventricle
D White matter consists of myelinated axons of neurons
ANSWER:B
1656 What is true about the spino-thalamic tract?
A it is a descending pathway that carries sensory information
B it is a descending pathway that carries motor instructions
C it is an ascending pathway that carries sensory information
D it is an ascending pathway that carries motor instructions
ANSWER:C
1657 What is linked to the posterior gray horn of the spinal cord?

A the dorsal root of the spinal nerve that carries motor fibres
B the dorsal root of the spinal nerve that carries sensory fibres
C the ventral root of the spinal nerve that carries sensory fibres
D the ventral root of the spinal nerve that carries motor fibres
ANSWER:B
1658 What is the blood-brain barrier?
A It is the inner two meninges that surround the brain.
B It is formed by the capillaries of choroid plexus and ependymal cells.
C It is the endothelial cells of capillaries that supply the brain, and their astrocytes.
D It is the cerebrospinal fluid that bathes the brain.
ANSWER:C
1659 What part of the brain is known as the cerebrum?
A it is that part of the diencephalon that surrounds the third ventricle.
B it makes up the majority of the brain stem
C the superficial part consisting of sulci and gyri
D the dorsal inferior part adjacent to the occipital bone
ANSWER:C
1660 What and where is the pre-central gyrus?
A It is the site of sensory function and is in the frontal lobe.
B It is the site of sensory function and is in the parietal lobe.
C It is the site of motor function and is in the frontal lobe.
D It is the site of motor function and is in the parietal lobe.
ANSWER:C
1661
What part of the brain receives sensory input before passing it on to another part of the brain
for interpretation or action?
A pons
B hypothalamus
C post-central gyrus
D thalamus
ANSWER:D
1662
What part of the brain contains the autonomic control centre whose orders reg- ulate food
intake, water balance and body temperature?
A hypothalamus

B thalamus
C medulla oblongata
D cerebellum
ANSWER:A
1663 What are the “ascending tracts” of the spinal cord and what do they do?
A They are white matter and they transmit sensory information to the brain.
B They are gray matter and they transmit sensory information to the brain.
C They are white matter and they transmit motor information to the brain.
D They are gray matter and they transmit motor information to the brain.
ANSWER:A
1664 In what part of the brain is the “decussation of the pyramids” found
A pons
B medulla oblongata
C midbrain
D hypothalamus
ANSWER:B
1665 What is the likely result of an injury that severs the spinal cord between C and C?
A respiratory failure and death.
B paraplegia
C hemiplegia
D quadriplegia
ANSWER:D
1666 To what does the “blood-brain barrier” refer?
A the three meninges that surround the brain and spinal cord.
B the tight junctions between endothelial cells of the capillaries that serve the brain.
C the structures that prevent fat-soluble molecules from entering the brain from the blood.
D the structure that produces cerebrospinal fluid from blood.
ANSWER:B
1667 What functions are controlled from the pre-central gyrus of the frontal lobe?
A automatic visceral functions
B conscious perception of many sensory inputs
C subconscious timing and co-ordination of skeletal muscle.
D voluntary control of skeletal muscle

ANSWER:D
1668 What is the primary function of the cerebellum?
A it regulates such things as body temperature, water balance and emotional responses
B it refines/adjusts learned motor movements so that they are performed smoothly.
C it controls our automatic functions such as breathing, digestion and cardio- vascular functions
D it is the origin of our conscious thoughts and intellectual functions
ANSWER:B
1669 What do the descending tracts of the spinal cord contain?
A white matter and transmit sensory information
B white matter and transmit motor commands
C gray matter and transmit sensory information
D gray matter and transmit motor commands
ANSWER:B
1670 Which of the following lists all of the main sections of the brain?
A cerebrum, brainstem, midbrain, medulla oblongata
B cerebrum, cerebral cortex, cerebellum, mesencephalon
C cerebellum, diencephalon, brainstem, cerebrum
D cerebral cortex, midbrain, diencephalon, cerebellum
ANSWER:C
1671 One of the following is not a meninge. Which one?
A Pia mater
B Alma mater
C Arachnoid mater
D Dura mater
ANSWER:B
1672
What part of the brain subconsciously provides the appropriate pattern of smooth co-
ordinated skeletal muscle contraction for movements that we have learned?
A the cerebellum
B the brainstem
C the cerebrum
D the diencephalon
ANSWER:A

1673 In what respect does “gray matter” differ from “white matter”
A gray matter refers to the CNS while white matter refers to the PNS.
B white matter makes up the autonomic nervous system, gray matter does not.
C
gray matter is found in the cerebrum, while white matter occurs in the cer- ebellum and the
diencephalon.
D gray matter contains the cell bodies of nerve cells white matter contains axons.
ANSWER:D
1674 Which of the following is NOT a meninge?
A cerebra mater
B pia mater
C dura mater
D arachnoid mater
ANSWER:A
1675 What is the most superficial part of the brain called?
A diencephalon
B cerebral cortex
C cerebellum
D mesencephalon
ANSWER:B
1676 What do neurones in the pre-central gyrus do?
A they receive information from general sense receptors in muscle and skin
B they communicate with motor, sensory and multi-modal association areas
C they allow conscious control of skilled voluntary muscle movements
D they process and relay auditory and visual input
ANSWER:C
1677 What is the role of the hypothalamus?
A it receives sensory input and relays it to the cerebral cortex
B it is the autonomic control centre which directs the function of the lower CNS
C it uses past experience to analyse and act on sensory input
D
it integrates sensory information from association areas and performs abstract intellectual
functions
ANSWER:B
1678 With respect to the spinal cord, where is the epidural space?

A external to the dura mater
B between the arachnoid and pia maters
C between the arachnoid and dura maters
D between the two layers of the dura
ANSWER:A
1679 Which part of the brain controls breathing, heart function, vasoconstriction and swallowing?
A mesencephalon
B cerebellum
C diencephalon
D brainstem
ANSWER:D
1680 The blood brain barrier functions to protect the brain from
A lipid soluble drugs, alcohol and nicotine
B fluctuations in oxygen and carbon dioxide concentrations
C neurotransmitters, bacteria and neurotoxins
D dehydration and fluctuating blood glucose level
ANSWER:C
1681 The precentral gyrus is the area of brain which houses
A the primary motor cortex
B our association areas
C the primary somatosensory area
D our higher intellectual functions
ANSWER:A
1682 In the spinal cord, the ascending tracts contain
A white matter and transmit sensory information
B white matter and transmit motor commands
C gray matter and transmit sensory information
D gray matter and transmit motor commands
ANSWER:A
1683 What is found between the arachnoid and pia mater?
A adipose tissue
B venous sinuses

C choroid plexus
D cerebrospinal fluid
ANSWER:D
1684 What is found in the epidural space?
A adipose tissue
B venous sinuses
C choroid plexus
D cerebrospinal fluid
ANSWER:A
1685 One of the functions of the prefrontal cortex is:
A making conscious decisions
B controlling motor functions
C detecting and integrating sensory information
D enabling word recognition
ANSWER:A
1686 What does the term ‘decussation’ mean and where does it occur?
A decussation = span. It occurs between the superior part of the brain and the inferior
B decussation = cross. It occurs between the anterior part of the brain and the posterior
C decussation = cross over. It occurs between the pons and the spinal cord
D decussation = associate. It occurs between the sensory area and the sensory association area
ANSWER:C
1687 The central sulcus separates gyri involved with which two major functions?
A vision and taste
B vision and hearing
C motor and sensory
D emotion and memory
ANSWER:C
1688 What type of nerves are found in the dorsal root?
A only afferent nerves
B only efferent nerves
C both afferent and efferent nerves
D only ganglionic nerves
ANSWER:A

1689
An epidural block involves injecting anaesthetic into the epidural space. What is the main
reason for this?
A Anaesthetic in this space only affects spinal nerves in the immediate vicin- ity of the injection.
B
The anaesthetic will be readily distributed along the spinal cord by the cere- bral spinal fluid
from this space.
C There is less chance of damaging the spinal cord when inserting the needle into this space.
D The epidural space is highly vascularised and so will the anaesthetic will be quickly absorbed.
ANSWER:A
1690 The central sulcus of the brain lies between which two lobes?
A parietal and occipital
B temporal and occipital
C frontal and temporal
D frontal and parietal
ANSWER:D
1691
If a person suffers a stroke and damage occurs to the occipital lobe of the brain which
function is the most likely to be affected?
A the ability to write
B speech
C hearing
D vision
ANSWER:D
1692 If a person had a pre-frontal lobotomy what would be the physiological consequences?
A Movement would be impaired
B Sensory function would be impaired
C The ability to assess the consequence of actions would be impaired
D Speech would be impaired
ANSWER:C
1693 What is the function of the thalamus?
A connects two cerebral hemispheres
B connects cerebellum to midbrain
C connects areas within same hemisphere

D it is a relay centre
ANSWER:D
1694 What is the function of the corpus callosum?
A connects two cerebral hemispheres
B connects cerebellum to midbrain
C connects areas within same hemisphere
D it is a relay centre
ANSWER:A
1695 Which one of the following is NOT a function of the cerebral spinal fluid?
A to produce hormones
B to transport nutrients around the brain
C to protect the spinal cord
D to cushion the brain
ANSWER:A
1696 Which best describes the function of the association area of the temporal lobe?
A It perceives of movement
B It interprets the meaning of sound patterns
C It recognises of geometric shapes and faces
D It perceives meaningful information from different senses
ANSWER:B
1697 What is the blood-brain barrier? An adaptation of the capillaries serving the brain that:
A prevents fat-soluble molecules from entering the brain.
B inhibits all substances from passing from the blood stream into the brain
C selectively inhibits many substances from passing from the blood stream into the brain
D operates from birth to prevent foreign molecules entering the brain.
ANSWER:C
1698 What is the purpose of inserting a needle into the epidural space? It:
A Allows access to the cerebro-spinal fluid
B Enables the cerebro-spinal fluid pressure to be measured
C Enables permanent drainage of cerebro-spinal fluid to treat hydrocephalus
D Allows access to administer analgesia and anaesthesia
ANSWER:D
1699 Which of the following statements about the blood-brain barrier is TRUE? It

A consists of the meninges which surround the brain
B is a protective mechanism which limits entry of alcohol into the brain
C
is poorly developed in the newborn who are therefore less sensitive to drugs which act on the
brain
D is unable to prevent entry of lipid-soluble toxins into the brain
ANSWER:D
1700
Spinal nerves are formed from a dorsal root and a ventral root. What is true of the ventral
root?
A they contain sensory neurons carrying afferent impulses
B they contain sensory neurons carrying efferent impulses
C they contain motor neurons carrying afferent impulses
D they contain motor neurons carrying efferent impulses
ANSWER:D
1701 Which of the following sends sensory information to the brain?
A The afferent division of the peripheral nervous system.
B The efferent division of the peripheral nervous system.
C The somatic nervous system.
D The autonomic nervous system.
ANSWER:A
1702 What part of the nervous system prepares the body for action during extreme situations?
A the limbic system
B the sympathetic division
C the efferent system
D the parasympathetic division
ANSWER:B
1703
To what does the following description apply? “An unlearned and involuntary but
predictable motor response to a stimulus, that is rapid and does not involve any processing
by the brain.”
A spinal reflex
B autonomic reflex
C cranial reflex
D learned reflex
ANSWER:A

1704 Which statement about the sympathetic and/or parasympathetic divisions is correct?
A All sympathetic neurons release ACh as a neurotransmitter.
B Sympathetic division fibres emerge from brain & sacral spinal cord.
C Parasympathetic division stimulates adrenal gland to release norepineph- rine & epinephrine.
D Some organs are innervated by both sympathetic division and parasympa- thetic division.
ANSWER:D
1705 Which one of the following parts of the nervous system carries impulses towards the brain?
A peripheral nervous system
B somatic nervous system
C autonomic nervous system
D parasympathetic division
ANSWER:A
1706 What statement is true about spinal reflexes?
A They cannot be inhibited or reinforced by the brain.
B They do not involve processing by the brain.
C They involve processing by the brainstem.
D They are all simple monosynaptic pathways.
ANSWER:B
1707 The nervous system is divided into two divisions. What are they called?
A somatic and autonomic
B central and peripheral
C afferent and efferent
D sympathetic and parasympathetic
ANSWER:B
1708 What do “sympathetic” and “parasympathetic” divisions refer to?
A the central nervous system
B the efferent neurons of the peripheral nervous system
C the autonomic nervous system
D the somatic nervous system
ANSWER:C
1709 Which of the following neural pathways or tracts carry sensory information?

A cortico-bulbar tracts
B spino-thalamic tracts
C cortico-spinal tracts
D reticulo-spinal tracts
ANSWER:B
1710 Nerve impulses carried by the parasympathetic division travel along which nerve fibres?
A cranial nerves I and II
B the spino-cerebellar pathway
C the spinal nerves
D the vagus nerves
ANSWER:D
1711 Which of the following is true of parasympathetic neurons?
A they emerge from the thoracic and lumbar vertebrae
B they all release ACh as a neurotransmitter
C they all release NE as a neurotransmitter
D they have short pre-ganglionic fibres and long post-ganglionic fibres
ANSWER:B
1712 Adrenergic receptors are so named because they:
A are located in the adrenal glands
B bind epinephrine and norepinephrine
C are located in the kidneys
D bind acetylcholine
ANSWER:B
1713 What is the purpose of a neurotransmitter?
A To pass a nerve impulse along a nerve cell axon
B To pass a nerve impulse onto another cell
C To pass a nerve impulse onto a muscle cell
D To pass a nerve impulse onto another nerve cell
ANSWER:B
1714 What innervates the diaphragm?
A The spinal nerves from T to T
B The vagus nerve
C The phrenic nerve

D The sciatic nerve
ANSWER:C
1715
Consider the following pairs of terms. Which pair has a term that refers to a part of the
nervous system that carries sensory information to the brain, and a term that refers to a part
that carries motor commands to the peripheral tissues?
A parasympathetic division; sympathetic division
B somatic nervous system; autonomic nervous system
C afferent division; efferent division
D central nervous system; peripheral nervous system
ANSWER:C
1716 Which organisational entity of the brain is divided into an afferent and an effer- ent division?
A the central nervous system
B the peripheral nervous system
C the somatic nervous system
D the autonomic nervous system
ANSWER:B
1717
Which neurotransmitter do all motor neurons release at their synapses with skeletal muscle
cells?
A ACh
B ATP
C GABA
D norepinephrine
ANSWER:A
1718 Which muscle(s) are NOT controlled by the autonomic nervous system?
A cardiac muscle
B the diaphragm
C skeletal muscle
D smooth muscle
ANSWER:C
1719
Which type of receptor always produces stimulation of the post-synaptic cell when bound by
a neurotransmitter?
A nicotinic

B muscarinic
C alpha adrenergic
D beta adrenergic
ANSWER:A
1720 What is one effect that the sympathetic division of the autonomic nervous sys- tem have?
A increases gut motility and digestive secretions
B causes bronchioles to constrict
C decreases heart rate
D stimulates sweating from sweat glands
ANSWER:D
1721 To what part of the nervous system does the somatic nervous system belong?
A efferent, central nervous system
B efferent, peripheral nervous system
C afferent, peripheral nervous system
D afferent, central nervous system
ANSWER:B
1722 What is the parasympathetic division of the autonomic nervous system respon- sible for?
A rapid predictable motor responses without processing by the brain
B conserving energy and maintaining body activities without conscious brain control.
C preparing the body for energetic activity without conscious brain control.
D gathering sensory information from the viscera that is not interpreted by the brain.
ANSWER:B
1723
The following receptors for neurotransmitters may be stimulatory or inhibitory EXCEPT for
one of them. Which One?
A nicotinic receptors
B muscarinic receptors
C adrenergic alpha receptors
D adrenergic beta receptors
ANSWER:A
1724 What do “sympathetic” and “parasympathetic” refer to? Divisions of:
A the central nervous system
B the efferent neurons of the peripheral nervous system
C the autonomic nervous system

D the somatic nervous system
ANSWER:C
1725 What structure(s) does a neurotransmitter molecule cross?
A synaptic cleft
B synaptic cleft and the post-synaptic membrane
C pre-synaptic membrane and the synaptic cleft
D post-synaptic membrane
ANSWER:C
1726 What is a spinal reflex?
A it involves rapid processing by the brain and a predictable response
B it involves stimulation of a motor neurone by a sensory neurone without a synapse
C it is a rapid, predictable, learned and involuntary motor response
D it is a predictable, unlearned and involuntary motor response
ANSWER:D
1727 A “post-ganglionic cholinergic fibre” refers to a neuron that
A runs from CNS to a ganglion and releases noradrenaline
B synapses with an effector cell and whose neurotransmitter stimulates alpha and beta receptors.
C runs from a ganglion to an effector cell and releases norepinephrine
D synapses with an effector cell and releases acetylcholine (ACh)
ANSWER:D
1728 Which part of the nervous system prepares you for vigorous activity (“to fight or flee”)?
A sympathetic
B parasympathetic
C somatic
D autonomic
ANSWER:A
1729 What is that part of the nervous system that carries commands to the skeletal muscles called?
A somatic nervous system
B autonomic nervous system
C central nervous system
D sympathetic division

ANSWER:A
1730 Which neurotransmitter do all motor neurons release at their synapses?
A acetylcholine
B norepinephrine
C dopamine
D adenosine triphosphate
ANSWER:A
1731 Which neurotransmitter is released by all parasympathetic neurons?
A norepinephrine
B acetylcholine
C nicotine
D muscarine
ANSWER:B
1732
Which choice correctly ends the following sentence? The parasympathetic divi- sion is part
of the:
A autonomic nervous system
B somatic nervous system
C afferent division of the peripheral nervous system
D central nervous system
ANSWER:A
1733 What may accurately be said of the post-synaptic membrane?
A It is attached to the transmitting axon
B It has receptors for a neurotransmitter
C It is before the synaptic cleft
D It is part of a neurone
ANSWER:B
1734 Which statement about the vagus nerve is true?
A it lies within the cerebrospinal tract
B it arises from the pons
C it has a sensory function in vision and olfaction
D it carries parasympathetic motor impulses
ANSWER:D
1735 One thing that could NOT be correctly said of the sympathetic division is that:

A it stimulates the adrenal gland to release adrenaline and noradrenaline
B its fibres emerge from the spinal cord at the thoracic and lumbar vertebrae
C it promotes the conservation of the body’s energy
D it supplies the smooth muscle of blood vessels
ANSWER:C
1736 If the term “cholinergic” is applied, to a synapse what does it mean?
A the target organs are innervated by the sympathetic nervous system
B the receptors are nicotinic
C the result is always stimulatory
D acetylcholine is released at the synapse
ANSWER:D
1737 What determines the response of the post-synaptic cell to autonomic impulses?
A the neurotransmitter that binds to the cell
B the type of receptor on the cell
C whether the innervation is sympathetic or parasympathetic
D whether the fibre is pre-ganglionic or post-ganglionic
ANSWER:B
1738 What is the nerve that that carries most of the parasympathetic signals?
A phrenic
B vagus
C sciatic
D trigeminal
ANSWER:B
1739 Which is an example of a cholinergic receptor?
A nicotinic receptor
B adrenergic receptor
C alpha receptor
D beta receptor
ANSWER:A
1740 Which statement about neurotransmitters and/or receptors is correct?
A all somatic motor neurons release ACh at their synapse.
B noradrenalin is the major neurotransmitter of the parasympathetic division
C nicotinic receptors when bound by ACh are always inhibitory

D noradrenalin binds to nicotinic and muscarinic receptors
ANSWER:A
1741 The peripheral nervous system is divided into
A sympathetic division and parasympathetic division
B brain and spinal cord
C somatic system and autonomic system
D motor division and sensory division
ANSWER:D
1742 A neurotransmitter is a molecule that crosses the
A synaptic cleft
B synaptic cleft and the post-synaptic membrane
C presynaptic membrane and the synaptic cleft
D post-synaptic membrane
ANSWER:C
1743 What characterises a spinal reflex?
A It involves rapid processing by the brain and a predictable response.
B It is a predictable, unlearned and involuntary motor response.
C It is a rapid, predictable, learned and involuntary motor response.
D
It involves stimulation of a motor neurone by a sensory neurone that origi- nates within the
CNS.
ANSWER:B
1744
When their neurotransmitter binds to them, which of the following receptors is always
stimulatory?
A muscarinic
B cholinergic
C nicotinic
D adrenergic
ANSWER:C
1745
Two ways that cells can communicate within the body are by synaptic commu- nication or
by endocrine communication. A difference between the two is:
A
endocrine communication involves a chemical messenger whereas synaptic communication
does not.

B
the action caused by synaptic communication may persist for several hours whereas that
caused by endocrine communication persists for several minutes.
C
endocrine communication controls cellular activities in distant tissues whereas synaptic
communication affects the adjacent cell.
D
synaptic communication occurs between adjacent cells whereas endocrine communication
occurs between cells of the same tissue.
ANSWER:C
1746 Which of the following is NOT indicative of the stimulation of the parasympa- thetic system?
A constriction of skeletal muscle blood vessels and vasodilation of renal blood vessels
B excitation of the blood flow to the kidneys and smooth muscles of the GI tract
C inhibition of heart rate and smooth muscles of the urinary bladder
D excitation of the smooth muscles of the urinary bladder and GI tract
ANSWER:A
1747 Sensory receptors convert stimuli into what?
A neurotransmitters
B action potentials
C graded potentials
D voltage-gated channels
ANSWER:C
1748 What is the function of a spinal nerve?
A transmit sensory information
B transmit both sensory and motor information
C connect sensory and motor neurons
D transmit autonomic nervous system information
ANSWER:B
1749 Sensory receptors convert stimuli that are above threshold into what?
A graded potentials
B neurotransmitters
C action potentials
D motor activity in muscles
ANSWER:C
1750 What is the function of an efferent neuron?

A transmit sensory information
B transmit motor information
C connect sensory and motor neurons
D transmit both sensory and motor information
ANSWER:B
1751 Choose the correct statement about the eye:
A Bipolar and ganglion cells occur in the retina except at the fovea.
B Rod and cone cells occur in the retina except at the fovea.
C Bipolar and ganglion cells occur in the choroid except at the optic disc
D Rod and cone cells occur in the choroid except at the optic disc.
ANSWER:A
1752 Which part of the retina has the greatest sensitivity to light?
A the optic disc
B macula lutea
C the choroid
D fovea centralis
ANSWER:D
1753 The deterioration of sight with age is known by which term?
A protanopia
B presbyopia
C hyperopia
D scotopia
ANSWER:B
1754
What is the place where the blood vessels and nerve fibres come together and leave the
posterior chamber of the eye called?
A macula lutea
B optic disc
C fovea centralis
D choroid
ANSWER:B
1755
Accommodation refers to the eye’s ability to focus light from objects whatever their distance
from the eye. How is this achieved?
A by altering the distance between the cornea and the eye’s lens

B by altering the distance between the lens and the retina
C by altering the shape of the eye’s lens
D by altering the shape of the cornea
ANSWER:C
1756 What does the ciliary muscle do when accommodation (focussing) in the eye occurs?
A It contracts and the tension on the ciliary fibres increases making the eye lens less convex.
B
It relaxes and the tension on the ciliary fibres decreases allowing the eye lens to become more
convex.
C
It contracts and the pull of the ciliary fibres decreases allowing the lens to become more
convex.
D It relaxes and the pull of the ciliary fibres increases to make the lens less convex.
ANSWER:C
1757
Which cells of the retina are responsible for detecting light in scotopic (i.e. low light)
conditions?
A bipolar cells
B rod cells
C ganglion cells
D cone cells
ANSWER:B
1758
Accommodation refers to our ability to bring objects at any distance into sharp visual focus
by altering which of the following?
A the distance between our eye and the object to be viewed
B the curvature of our cornea
C the curvature of the eye’s lens
D the distance between the lens and retina
ANSWER:C
1759 What is the purpose of the optic chiasma?
A
to allow images from each eye to cross over to the other side of the brain prior to crossing
back at the decussation of pyramids.
B
to allow fibres from the medial aspect of one eye to join fibres from the lateral aspect of the
other eye to form an optic tract.
C to allow the fibres from the lateral aspect of each eye to come together as an optic tract.

D
to allow light entering the left eye to be interpreted by the right hand side of the occipital
lobe (and vice versa).
ANSWER:B
1760 What is presbyopia (old-age vision) due to?
A the loss of elasticity of the lens of the eye
B the change in the curvature of the cornea
C the gradual loss of cone cells from the retina
D the deviation from a spherical eye-ball shape with aging.
ANSWER:A
1761 In which region of the eye does the most detailed vision occur? The
A fovea centralis
B optic disc
C macula lutea
D ciliary body
ANSWER:A
1762 How does accommodation in the eye occur? When our ciliary muscle
A contracts and the tension on the ciliary fibres increases allowing a rounder lens
B relaxes and the tension on the ciliary fibres decreases allowing a rounder lens
C relaxes and the pull of our ciliary fibres flattens the lens
D contracts and the pull of our ciliary fibres flattens the lens
ANSWER:C
1763
What is the name of the structure that allows nerve fibres from the medial aspect of each
retina to join fibres from the lateral aspect of the retina of the other eye?
A optic chiasma
B optic nerve
C optic radiation
D lateral geniculate body
ANSWER:A
1764 What is the light sensitive cell in the retina that responds to colour called?
A macula
B macula lutea
C cone
D rod

ANSWER:C
1765
People over years of age eventually require reading glasses. This condition (known as
presbyopia) is the result of which of the following conditions?
A the loss of elasticity in the lens
B the development of cataracts in the lens
C the decrease in the refractive index of the cornea
D the degeneration of the cone cells of the retina.
ANSWER:A
1766
What is meant by the term “accommodation” when referring to our vision? It refers to the
ability of our eye to:
A alter the thickness of the lens to focus on objects whatever their distance from us.
B alter the amount of refraction occurring in the cornea.
C alter the diameter of the pupil to cope with situations of different light intensity.
D
use either rods or cones for vision depending on whether we are viewing during daylight or
at night.
ANSWER:A
1767 Choose the answer that correctly completes the sentence. The “lens” of the human eye
A is a biconcave lens.
B produces more refraction than the cornea.
C can have its focal length altered.
D consists of rods and cones.
ANSWER:C
1768 Which statement about refraction of light in the human eye is correct?
A Most of it occurs in the lens.
B Its extent is governed by the size of the iris opening to the eye.
C Most of it occurs as light enters the cornea from air.
D The angle of incidence is equal to the angle of refraction.
ANSWER:C
1769
The process of adjusting the eye’s lens to view objects at different distances from the eye is
called:
A accommodation.
B presbyopia.
C refraction.

D hyperopia.
ANSWER:A
1770 Glaucoma is an eye disease which affects vision. It is caused by:
A
blockage of the flow of aqueous humor through the canal of Schlemm and loss of
intraocular pressure in the vitreous humor.
B detachment of the retina and subsequent loss of vision in this part of the eye.
C cataracts that form in the eye’s lens which prevent light from reaching the retina.
D
increased intraocular pressure which collapses the blood capillaries that perfuse the retina so
part of it dies.
ANSWER:D
1771 Which one of the following statements is INCORRECT?
A A convex lens causes light rays to converge.
B The fluid between the cornea and the eye’s lens is a lens.
C The cornea is a lens.
D The eye’s lens is a convex lens.
ANSWER:B
1772 In the human eye where does the greatest refraction occur?
A in the lens of the eye.
B at the retina.
C as light passes from air into the cornea.
D as light passes from the lens into the vitreous humor.
ANSWER:C
1773
In order to focus on objects that are very close to the eye, what must happen to the human
lens? It must:
A increase its focal length
B increase the amount of refraction it causes
C be stretched to a thinner shape
D decrease its dioptre value
ANSWER:B
1774 Myopia may be corrected with a lens that is
A bifocal.
B concave.
C cylindrical.

D convex.
ANSWER:B
1775 The change in vision that occurs with ageing is called
A protanopia.
B hyperopia.
C deuteranopia.
D presbyopia.
ANSWER:D
1776 As part of the normal ageing process, our eyes deteriorate because
A the ciliary muscles gradually lose their tone.
B the lens loses its flexibility.
C parts of the retina detach from the underlying blood vessels.
D the distance between the lens and the retina gradually changes.
ANSWER:B
1777 In which colour ranges do the three pigments in the retina have their major sensitivities?
A red, green and blue.
B red, blue and yellow.
C green, yellow and red.
D green, yellow and blue.
ANSWER:A
1778 By which of the following pathways does sound entering the ear reach the organ of Corti?
A basilar membrane, middle ear, oval window, endolymph,
B tympanic membrane, ossicles, oval window, cochlear fluid
C tectorial membrane, Eustachian tube, ossicles, cochlear fluid
D oval window, ear canal, auditory tube, endolymph
ANSWER:B
1779
Which part of the ear contains the apparatus that we use to distinguish between different
frequencies of sound?
A The cochlea
B The Eustachian (or auditory) tube
C The tensor tympani
D The auditory meatus
ANSWER:A

1780 What is the range of frequencies that the human ear is most sensitive to?
A 0 Hz to 00 Hz
B ,000 Hz to 0,000 Hz
C 00 Hz to 000 Hz
D 0 Hz to 0,000 Hz
ANSWER:C
1781 Which one of the following statements is WRONG?
A the middle ear lies between the tympanic membrane and the oval and round windows
B the outer ear is vented by the Eustachian (or auditory) tube
C the stapes is located in the middle ear
D the tectorial membrane and the basilar membrane are located in the inner ear
ANSWER:B
1782
The ossicles of the ear pass on sound vibrations to the fluid in the inner ear. In what structure
is this fluid located?
A the organ of Corti
B the cochlea
C the Eustachian tube
D the saccule and utricle
ANSWER:B
1783
Sound produces vibrations in the cochlear fluid of the inner ear. The movement of the fluid
then produces motion in which of the following?
A tectorial membrane
B basilar membrane
C otolithic membrane
D crista ampullaris
ANSWER:B
1784 What is the function of the middle ear ossicles?
A To protect the cochlea from excessively loud noises.
B To increase the sound intensity by resonating for sounds of frequencies near 000 Hz
C To amplify the sound intensity that reaches the tympanic membrane.
D To cause the sound energy of waves in air to be transmitted into the cochlear fluid
ANSWER:D

1785 Sound waves are conducted from the air outside the ear to the inner ear by the processes of:
A absorption, transmission and refraction.
B reflection, transmission and scattering.
C resonance, leverage and amplification.
D resonance, diffraction and refraction.
ANSWER:C
1786
Sound level (measured in decibels, dB) is a subjective measure of the loudness of a sounA
sound of 0 dB
A will produce hearing damage if the ear is subjected to it chronically.
B is beyond the normal audible frequency range of human hearing.
C cannot be heard by the human ear even though it carries energy.
D will be perceived to be the same loudness at all audible frequencies.
ANSWER:A
1787
Of the following lists of four anatomical features, which one has them in the correct order of
the path taken by sound energy as it is transmitted through the ear?
A tectorial membrane, malleus, oval window, cochlear fluid.
B tectorial membrane, incus, round window, organ of Corti.
C tympanic membrane, malleus, oval window, cochlear fluid.
D tympanic membrane, stapes, round window, organ of Corti.
ANSWER:C
1788 Which of the following is NOT a small bone involved in hearing?
A Meatus
B Malleus
C Stapes
D Incus
ANSWER:A
1789
Which one of the following lists of anatomical features is in the correct order of the path
taken by sound energy as it is transmitted through the ear?
A Tympanic membrane, malleus, oval window, cochlear fluid.
B Basilar membrane, incus, round window, fluid of Corti.
C Tympanic membrane, stapedius, round window, fluid of Corti.
D Basilar membrane, malleus, oval window, cochlear fluid.

ANSWER:A
1790
The loudness of a sound wave as perceived by the human ear, depends on which of the
following pairs of wave properties?
A speed and frequency.
B intensity and frequency.
C amplitude and phase.
D speed and intensity.
ANSWER:B
1791
If the frequency of a sound wave is increased from 0 Hz to 000 Hz, its loud- ness also
increases. This occurs because:
A the ear is more sensitive to 000 Hz than to 0 Hz.
B sounds of higher frequency carry higher energy.
C as frequency increases the sound intensity also increases.
D loudness is proportional to frequency.
ANSWER:A
1792
Our ears are most sensitive to sounds with frequencies that lie between about 000 Hz and 00
Hz. The reason for this is that
A sounds with these frequencies have the largest decibel rating
B the majority of human speech sounds are composed of frequencies that lie in this range
C
the external ear canal has dimensions that allow it to resonate with a fre- quency that is in this
range
D the largest part of the basilar membrane is receptive to this range of frequencies
ANSWER:C
1793 What can be said about noise-induced hearing loss?
A It affects sound frequencies near 000 Hz most.
B It is also called presbycusis.
C It is likely to be caused by sounds above dB.
D It is due to otosclerosis.
ANSWER:A
1794 For which condition is a hearing aid most successful at treating?
A conductive hearing loss.
B perceptive deafness.
C nerve deafness.

D sensorineural hearing loss.
ANSWER:A
1795 What is the purpose of the diaphragm on the bell of a stethoscope? To:
A prevent external sounds from interfering with auscultation.
B eliminate any air gap between the skin and stethoscope.
C resonate with the sound being listened to.
D transmit the body sounds to the earpieces.
ANSWER:C
1796
Which structure lies on the boundary between the middle and inner ear and has the stapes
bound to it?
A Ampulla
B Oval window
C Round window
D Tympanic membrane
ANSWER:B
1797 Which membrane lies over the hair cells found in the organ of Corti?
A Basilar
B Tectorial
C Vestibular
D Cochlear
ANSWER:B
1798
The inner ear (or internal ear) maybe described as a series of tubes. What are the tubes filled
with?
A air
B perilymph
C endolymph
D perilymph and endolymph
ANSWER:D
1799
What are the cells found in the maculae of the utricle and saccule that are responsible for our
sense of equilibrium?
A Supporting cells
B Otoliths
C Hair cells

D Epithelial cells
ANSWER:C
1800 Which of the following auditory structures are filled with fluid?
A The inner ear
B The middle ear
C The external meatus
D the Eustachian tube
ANSWER:A
1801 Where are the male ejaculatory ducts?
A In the testicles before the epididymis.
B In the penis.
C Between the bulbourethral glands and the urethra.
D At the end of the vas deferens (ductus deferens).
ANSWER:D
1802 What is the function of luteinising hormone?
A It stimulates the interstitial (Leydig) cells to produce testosterone.
B It stimulates sustentacular (Sertoli) cells to produce sperm.
C It stimulates the anterior pituitary to release follicle stimulating hormone.
D It stimulates the ovary to develop follicles.
ANSWER:A
1803 Which statement is true?
A Males have two “X” chromosomes
B Females do not produce any primordial follicles after they are born.
C Females have one “X” chromosome
D Fertilisation occurs in the pelvic cavity before the start of the fallopian tube.
ANSWER:B
1804
In which of the following lists are the structures of the male reproductive tract listed in
correct order from testes to urethra?
A ejaculatory ducts, seminiferous tubules, epididymis, vas deferens.
B seminiferous tubules, epididymis, vas deferens, ejaculatory ducts.
C epididymis, ejaculatory ducts, seminiferous tubules, vas deferens.
D vas deferens, seminiferous tubules, epididymis, ejaculatory ducts.
ANSWER:B

1805 One of the following is NOT a secondary sex characteristiWhich one?
A the adult male body shape
B the thicker vocal cords of a male
C pubic hair
D the penis
ANSWER:D
1806 What does gonadotropin releasing hormone (GnRH) do?
A stimulates the anterior pituitary to release LH.
B stimulates the anterior pituitary to release both LH and FSH.
C stimulates the anterior pituitary to release FSH.
D stimulates the corpus luteum to release progesterone.
ANSWER:B
1807 Where should fertilisation of the egg by a sperm occur?
A in the cervix
B in the uterus
C in the Fallopian tube
D in the abdominal cavity between ovary and Fallopian tube.
ANSWER:C
1808 Which of the following is/are NOT associated with the male reproductive system?
A estrogens
B androgens
C FSH and LH
D the tunica vaginalis
ANSWER:A
1809 Which of the listed structures does the male reproductive tract pass through?
A the prostate
B the bulbourethral gland
C the seminiferous vesicles
D the bladder
ANSWER:A
1810 Where is the hormone progesterone produced?
A by the thecal cells that surround the follicle
B in the anterior pituitary

C in the corpus luteum
D by the developing follicle
ANSWER:C
1811 What is the name of the tube that carries sperm from the testes to the prostate gland?
A vas deferens
B ejaculatory duct
C seminiferous tubule
D urethra
ANSWER:A
1812 What effect does luteinising hormone have?
A it stimulates the growth of a few follicles each month
B stimulates ovulation and maintains the corpus luteum
C it prepares the uterus for pregnancy
D it establishes and maintains the secondary sex characteristics
ANSWER:B
1813 Which of the following do the testes produce?
A capacitated spermatozoa
B about 0 % of the ejaculate
C slightly acidic fluid.
D physically mature spermatozoa
ANSWER:D
1814 What do the thecal cells that surround the follicle produce?
A mucus
B luteinising hormone
C androgens
D estrogens
ANSWER:C
1815
In the sequence of events known as the ovarian cycle, which of the following does NOT
occur?
A the anterior pituitary releases FSH and LH
B FSH stimulates a follicle to develop
C the hypothalamus releases GnRH
D the developing follicle produces progesterone

ANSWER:D
1816 How do sperm cells differ from other cells in the male body?
A they contain chromosomes
B they all contain an X chromosome
C they all contain a Y chromosome
D they undergo mitosis
ANSWER:A
1817 Where does fertilisation of the ovum normally occur?
A in the cervical canal
B in the ovary
C in the uterus
D in the Fallopian tube
ANSWER:D
1818 What hormone is released by the corpus luteum in the greatest quantity?
A progesterone
B estrogens
C luteinising hormone
D follicle stimulating hormone
ANSWER:A
1819 Which list of structures in the male reproductive tract has them in correct sequence?
A seminiferous tubules, epididymis, vas deferens, urethra, ejaculatory duct
B epididymis, seminiferous tubules, vas deferens, ejaculatory duct, urethra
C seminiferous tubules, epididymis, vas deferens, ejaculatory duct, urethra
D epididymis, seminiferous tubules, vas deferens, urethra, ejaculatory duct
ANSWER:C
1820 Choose the correct statement about LH or FSH.
A LH targets Leydig cells of the testes which produce testosterone
B LH targets Sertoli cells of the testes which promotes spermiogenesis
C FSH targets Leydig cells of the testes which promotes spermiogenesis
D FSH targets Sertoli cells of the testes which produce testosterone
ANSWER:A
1821 Which cells produce the majority of estrogens?
A the cells of the corpus luteum

B the cells of anterior pituitary
C endometrial cells
D granulosa cells of the follicle
ANSWER:D
1822 What is the function of progesterone?
A to stimulate the development of follicles
B to maintain the corpus luteum
C prepare and maintain the uterus for pregnancy
D to stimulate ovulation
ANSWER:C
1823 If a cell is said to be “haploid”, what is meant?
A it has chromosomes
B it has chromosomes that all consist of one chromatid
C it has the “n” number of chromosomes
D it is NOT a gamete (or sex cell)
ANSWER:A
1824 Which of the following could be accurately said of oogenesis and spermiogenesis?
A they both occur after puberty
B they both cease after menopause
C the former occurs before birth while the latter continues from puberty to death
D the former is promoted by FSH while the latter is promoted by LH
ANSWER:C
1825 Male sterilisation (vasectomy) involves the cutting of which tube?
A ejaculatory duct
B epididymis
C urethra
D ductus deferens
ANSWER:D
1826 Which cells develop into the corpus luteum?
A granulosa cells
B interstitial cells
C cells of the antrum
D thecal cells

ANSWER:A
1827 What is the function of the hormone progesterone?
A to maintain secondary sex characteristics
B to pause meiosis until the ovum is fertilised
C to stimulate estrogen production
D to prepare the uterus for pregnancy
ANSWER:D
1828 Which of the following statements is correct?
A ova all contain a Y chromosome
B half of the ova carry an X chromosome and half carry a Y chromosome
C half of the sperm cells carry an X chromosome and half carry a Y chromosome
D sperm all carry an X chromosome
ANSWER:C
1829 From which source does the majority of the volume of a male ejaculation come?
A epididymis
B seminiferous tubules
C seminal vesicles
D prostate gland
ANSWER:C
1830 What is a fertilised egg known as?
A ovum
B zygote
C embryo
D blastocyst
ANSWER:B
1831 What term is applied to the second two weeks of the menstrual (uterine) cycle
A menses
B secretory phase
C luteal phase
D proliferation
ANSWER:B
1832 What is the section of the male reproductive tract within which sperm are pro- duced called?

A the urethra
B the epididymis
C the vas deferens
D the seminiferous tubules
ANSWER:D
1833 What is the role of progesterone?
A to stimulate follicle development
B to stimulate the maturation of the uterine lining
C to stimulate the oocyte to complete meiosis I
D to stimulate the release of FSH
ANSWER:B
1834 Which statement about the “granulosa cells” is NOT correct?
A they produce estrogens and inhibin
B they form a single layer around the primary follicle
C they form the corpus luteum
D one of them will develop into the ovum
ANSWER:D
1835 In the male reproductive tract, where are sperm produced? In the:
A seminiferous tubules.
B epididymis.
C Sertoli cells of the testes.
D Leydig cells of the testes.
ANSWER:A
1836 What is the “external urethral meatus” is another name for?
A the shaft of the penis.
B the opening of the tube at the end of the penis.
C the prostate gland.
D the scrotum.
ANSWER:B
1837 What is the role of luteinising hormone (LH) in the female?
A it causes release of FSH from the anterior pituitary.
B it maintains the secondary sex characteristics.
C it allows the frequency of pulses of GnRH to increase.

D it causes ovulation.
ANSWER:D
1838 What is the chromosome complement of a sperm?
A chromosomes ( pairs including a pair of Y chromosomes).
B chromosomes ( pairs plus one X chromosome and one Y).
C chromosomes including one X chromosome OR one Y.
D chromosomes including one X chromosome AND one Y
ANSWER:C
1839 Which one of the following statements is INcorrect?
A corpus luteum releases progesterone.
B hypothalamus releases GnRH.
C anterior pituitary releases FSH.
D granulosa cells produce androgens.
ANSWER:D
1840 Which one of the following is true about the secretory phase of the menstrual cycle?
A It occurs as the uterine epithelium regrows under the stimulation of estrogens.
B It begins at ovulation and continues while the corpus luteum is intact.
C It refers to the release of progesterone by the corpus luteum.
D During this phase the follicle develops prior to ovulation.
ANSWER:B
1841 What is the function of the epididymis?
A Production of sperm.
B Stores sperm and facilitates their maturation.
C Stores sperm and produces seminal fluid.
D Carries semen out through the penis.
ANSWER:B
1842 In males, what is the function of luteinising hormone (LH)?
A It stimulates interstitial cells in the testes to produce testosterone.
B It stimulates sustentacular cells in the testes to produce sperm.
C It promotes the maturation of spermatozoa.
D It stimulates the anterior pituitary to release FSH.
ANSWER:A
1843 One of the following is NOT a function of testosterone, which one?

A Stimulates spermiogenesis.
B Maintains secondary sex characteristics.
C Maintains glands of the reproductive tract.
D Stimulates anterior pituitary to release FSH and LH.
ANSWER:D
1844
After ejaculation, sperm travel through the structures of the female reproduc- tive tract in
which order?
A Vagina, uterus, fallopian tube, ovary.
B Cervix, vagina, uterus, fallopian tube.
C Vagina, cervix, uterus, fallopian tube.
D Cervix, urethra, uterus, fallopian tube.
ANSWER:C
1845 What is the name given to a young woman’s first menstrual period?
A Menarche.
B Menses.
C Eclampsia
D Amenorrhea.
ANSWER:A
1846 The hormone progesterone is released by which structure?
A Anterior pituitary.
B Corpus luteum.
C Hypothalamus.
D The adrenal glands.
ANSWER:B
1847 What is the result of meiosis in males? The production of:
A One spermatid and three polar bodies
B One spermatid and two polar bodies
C Two primary spermatocytes
D Four spermatids
ANSWER:D
1848 Sperm gain motility as they pass through which structure? The
A Lumen of the seminiferous tubule
B Prostatic part of the urethra

C Ductus deferens
D Epididymis
ANSWER:B
1849 Which of the following lists the structures of the female perineal area the cor- rect order?
A Clitoris, vaginal opening, urethral opening, anus
B Clitoris, urethral opening, vaginal opening, anus
C Urethral opening, clitoris, vagina, cervix
D Anus, clitoris, urethral opening, vaginal opening.
ANSWER:B
1850 What are the phases of the ovarian cycle?
A Menarche, menstrual cycle, menopause
B Luteal phase, menses and proliferative phase
C Follicular phase and the luteal phase
D Menses, proliferative phase and secretory phase
ANSWER:C
1851 Which of the following is NOT a function of estrogens?
A stimulating bone and muscle growth
B maintain female secondary sex characteristics
C maintaining the corpus luteum
D initiating growth and repair of the endometrium
ANSWER:C
1852 Which statement is NOT correct?
A Testosterone is required to maintain the adult male secondary sex characteristics
B Testosterone increases the sex drive in both sexes
C Testosterone inhibits the closure of the epiphyseal plate in the long bones
D Testosterone stimulates the growth and maturation of the male genitalia
ANSWER:C
1853 What does gonadotrophin-releasing hormone stimulate?
A The hypothalamus
B The anterior lobe of pituitary gland
C Spermatogenesis or oogenesis
D The production of testosterone or oestrogen
ANSWER:B

1854 Select one INCORRECT statement from the following:
A Meiosis results in the reduction of chromosome numbers in cells from n to n.
B The primordial follicle cells are present in females at birth.
C Spermatogenesis and oogenesis result in the production of either sperm or ova.
D Puberty is the time of life when the reproductive organs begin to mature.
ANSWER:C
1855 Which of the following produces male sex hormones?
A The interstitial cells
B Corpus luteum
C Anterior lobe of the pituitary gland
D Seminal vesicles
ANSWER:A
1856
The menstrual cycle can be divided into phases. Starting from day one of menstruation,
which of the following orders are the phases in?
A Menstrual, secretory, proliferative
B Proliferative, secretory, menstrual
C Menstrual, proliferative, secretory
D Secretory, proliferative, menstrual
ANSWER:C
1857 Which organ does NOT add a secretion to semen?
A Testes
B Prostate gland
C Penis
D Seminal vesicles
ANSWER:C
1858 The amplitude of a wave is related to which of the following?
A the distance between two successive crests.
B the number of wavelengths that pass by per second.
C the speed of the wave’s travel.
D the amount of energy it carries.
ANSWER:D
1859 Which is the BEST definition of a wave?
A The method by which the energy carried by visible light is propagated.

B
Travelling oscillations in the magnitude and direction of electric and mag- netic fields that do
not require a material medium.
C
A periodic disturbance in some property of the medium, the medium itself remains
(relatively) at rest.
D A mechanism for the transfer of energy without the transfer of matter.
ANSWER:D
1860 What does the term “wavelength” mean when applied to a wave?
A The number of complete cycles that pass by in one second.
B The distance between two successive crests (or compressions).
C The time it takes for one wavelength to pass by.
D How fast a wave is moving in its direction of propagation.
ANSWER:B
1861 Which of the statements about sound and light is INCORRECT?
A Sound is a mechanical wave while light does not require a medium to travel in.
B Light is a transverse wave phenomenon while sound is a longitudinal wave phenomenon.
C The speed of light is much greater than the speed of sound.
D Ultrasound and ultraviolet light have frequencies less than infrasound and infrared light.
ANSWER:D
1862 A wave may be defined as which of the following?
A The oscillation of a particle of the medium.
B A series of crests and compressions that propagate through space.
C A mechanism for the transfer of energy without transferring matter.
D The transport of the medium due to the oscillation of its particles.
ANSWER:C
1863 Which of the following definitions of a wave is the best?
A A wave is an event or disturbance that is localised at a particular location.
B A mechanical wave is a periodic disturbance in a material medium.
C Waves are a means of transferring energy without transferring matter.
D
Waves are a phenomenon characterised by their wavelength and their dis- placement of the
medium from the mean position.
ANSWER:C
1864 Which quantity most closely describes the amount of energy that is transported by a wave?
A Frequency.

B Amplitude.
C Wavelength.
D Velocity.
ANSWER:B
1865 What is the amplitude of a wave?
A the maximum displacement from the rest position.
B the distance between adjacent troughs.
C the energy carried by the wave.
D the frequency multiplied by the wavelength.
ANSWER:A
1866
Waves have properties (such as wavelength, frequency, period, speed, ampli- tude, intensity,
direction and phase) which can be measureSome of these properties are relateThat is, if one
of a pair of related properties is known, then the other can be worked out. Which of the
following lists contain proper- ties that are all Unrelated to each other?
A Amplitude, period, intensity, phase
B Frequency, amplitude, period, wavelength
C direction, speed, amplitude, phase
D Phase, wavelength, speed, frequency
ANSWER:C
1867 What is the quantity that is most characteristic of an electromagnetic wave?
A amplitude.
B wavelength.
C frequency.
D velocity.
ANSWER:C
1868 In a wave, what is the distance between two adjacent troughs called?
A The period
B The displacement
C The amplitude
D The wavelength
ANSWER:D
1869 Visible light waves are examples of
A electromagnetic waves.

B mechanical waves.
C longitudinal waves.
D compressional waves.
ANSWER:A
1870 Which phrase would best describe waves of dim violet light?
A High frequency and high amplitude.
B Low frequency and high amplitude.
C High frequency and low amplitude.
D Low frequency and low amplitude.
ANSWER:C
1871 The phenomenon of refraction is due to
A light rays bending when they enter a different medium
B the decrease in speed when a ray enters a less dense medium
C the difference in refractive indices of two media
D the different speeds with which different frequencies of light travel through media.
ANSWER:C
1872
Consider the situation where a light wave travelling in air strikes a glass surface with an angle
of incidence of 0o. Which of the following statements is true?
A The angle formed by the incident ray and the normal will be 0°.
B The angle of refraction will be greater than 0°.
C The angle formed by the incident ray and the glass surface is 0°.
D There will be no refracted ray. That is, total internal reflection will occur.
ANSWER:A
1873 The term ‘refraction’ may be applied to which of the following?
A The effect on light rays as they enter the cornea of the eye.
B Light rays from an object meeting at a point to form a sharp image.
C
A light ray travelling away from a surface at the same angle that the light ray approached the
surface.
D A light ray travelling through glass with a faster speed than it would have in air.
ANSWER:A
1874 What is a convex lens? One that
A can accommodate to different focal lengths.
B will cause light rays to diverge.

C is thicker in the middle than at the edge.
D will correct for myopia (short sightedness).
ANSWER:C
1875
In the electromagnetic spectrum, the frequencies that we call visible light have values that lie
above
A the ultraviolet and below the infrared
B the infrared and below microwaves
C microwaves but below ultraviolet
D the infrared but below the radio range
ANSWER:C
1876
In which of the following sequences are the types of electromagnetic radiation listed in
correct order of energy with lowest energy first and highest energy last?
A visible, ultraviolet C, ultraviolet A, x-rays.
B microwaves, infrared, ultraviolet, gamma rays.
C x-rays, ultraviolet, visible, microwaves.
D infrared, microwaves, x-rays, gamma rays.
ANSWER:B
1877 The energy carried by a photon of electromagnetic radiation is proportional to its
A frequency
B speed
C wavelength
D amplitude
ANSWER:A
1878 Ultraviolet radiation is damaging to the eye because
A the heat produced as it is absorbed distorts the cornea.
B it causes an increase in the pressure in the eyeball which results in glaucoma.
C the energy of the radiation destroys the cones in the fovea.
D The energy of ultraviolet radiation is mainly absorbed in the lens which harms the cells.
ANSWER:D
1879
To what radiation does the term ‘non-ionising radiation’ refer? To radiation that consists of
photons with energy that is:
A not high enough to knock electrons out of atoms but is high enough to pro- duce ions.
B sufficient to generate ions in the material that the photons enter.

C not sufficient to generate ions in the material that the photons enter.
D high enough to knock electrons out of atoms but not high enough to produce ions.
ANSWER:C
1880 An oscillating magnetic field will be produced by:
A an electric field of constant magnitude.
B a permanent magnet moving at constant speed.
C the direct current that powers portable radios.
D the alternating current in household electrical appliances.
ANSWER:D
1881
What can be said about the collimated beam of electromagnetic energy pro- duced by an Nd-
YAG laser?
A It has a higher frequency than visible light energy.
B It stimulates the production of light as it passes through air.
C It is produced when electrons change energy levels.
D It consists of waves of two or more frequencies.
ANSWER:C
1882
Consider an electron that is in its ground state in an atom. When this electron moves to
another state in the atom it is said to have
A gained energy and to be in an excited state
B gained energy and to be in a metastable state
C lost energy and undergone stimulated emission
D lost energy and to be part of a population inversion
ANSWER:A
1883 What property of a LASER makes it suitable for surgical procedures? A LASER:
A beam’s energy can be focussed onto a very small spot to vaporise tissue.
B can be passed down an optical fibre in an endoscope.
C beam produces monochromatic (all of one wavelength) photons.
D emits photons which all have the same energy.
ANSWER:A
1884 What is a concave lens? One that
A can accommodate to different focal lengths.
B will cause light rays to converge.
C is thicker in the middle than at the edge.

D will correct for myopia (short sightedness).
ANSWER:D
1885 Which of the following conditions may be corrected by a diverging (concave lens)?
A Hyperopia
B Myopia
C Presbyopia
D Red minus dichroma
ANSWER:B
1886 Which of the following statements about sound and light is INCORRECT?
A they both extend over a range of frequencies
B sound is a longitudinal wave and light is a transverse wave
C the speed of travel of light is fast while sound travels relatively slowly
D sound will travel through a vacuum while light requires a transparent medium.
ANSWER:D
1887 What is the measure known as “sound level”?
A a subjective measure of the perceived loudness of a sound (in dB).
B
a graph of the levels of sound of different frequencies that we perceive as equal in loudness
(in phon).
C
the frequency of the sound that produces the loudest response in a healthy hearing
mechanism (in Hz).
D the objectively measured amount of sound energy carried by a sound wave (in W/m).
ANSWER:A
1888 How may sound waves be characterised? As:
A
Longitudinal waves because the oscillations that they cause are along a line at right angles to
the direction of propagation.
B Mechanical waves because they can only travel through a material medium.
C Waves because they allow for the oscillation of a medium without the trans- fer of energy.
D
Compressions and rarefactions because the nature of the oscillation of their electric and
magnetic fields.
ANSWER:B
1889 As the frequency of a sound in air is made to decrease, which of the following will happen?
A The period will increase.

B The wavelength will decrease.
C The amplitude will decrease.
D The velocity will decrease.
ANSWER:A
1890 What is the unit called the decibel (dB) used in the measurement of?
A sound frequency (pitch).
B sound intensity (energy).
C sound pressure.
D sound level (loudness).
ANSWER:D
1891
How much louder does a sound of 0 dB sound when compared to the same sound played at
0 dB?
A two times louder
B times louder
C 0 times louder
D 00 times louder
ANSWER:D
1892 What frequencies are attributed to the inaudible sound known as Ultrasound?
A greater than kHz.
B greater than MHz.
C greater than 0,000 Hz or less than 0 Hz.
D less than 0 Hz.
ANSWER:B
1893
Prior to an imaging examination using ultrasound, the skin surface is coated with a gel
substance known as a “coupling agent”. What is the purpose of the gel?
A To reduce the friction between the skin and the ultrasound transducer.
B To eliminate air, which would reflect the ultrasound, from between the skin and transducer.
C
Since ultrasound travels faster through denser materials, the time delay caused by travelling
through air is avoided by using gel.
D
To avoid an unpleasantly cold sensation that would otherwise be produced by the ultrasound
transducer.
ANSWER:B

1894 Which statement about a Doppler ultrasound stethoscope is true?
A
They amplify the echo produced when ultrasound strikes a boundary between two tissues of
different impedance.
B
Their operation depends on the reflected ultrasound being at a different fre- quency to the
emitted ultrasound.
C They emit ionising radiation.
D
The depth of penetration of ultrasound into tissue increases as the frequency of ultrasound
increases.
ANSWER:B
1895 Which of the following is considered to be the diagnostic ultrasound frequency range?
A mHz to mHz
B 0 Hz to 0 Hz
C 0 Hz to 0 kHz
D MHz to 0 MHz
ANSWER:D
1896 Ultrasound radiation may be characterised as which one of the following?
A ionising radiation
B audible radiation
C longitudinal waves
D electromagnetic waves
ANSWER:C
1897 What is Ultrasound?
A frequencies less than 0 Hz and more than 0 000 Hz.
B a mechanical wave (it requires a medium).
C a form of ionising radiation.
D more penetrating (in human tissues) as its frequency increases.
ANSWER:B
1898 What will a sound of frequency 0,000 Hz be?
A audible.
B painful to listen to.
C ultrasonic.
D close to the threshold of hearing.
ANSWER:C

1899 What is ultrasound least useful for examining?
A heart.
B lungs.
C kidneys.
D uterus.
ANSWER:B
1900
Basically, what does a Doppler ultrasound examination for peripheral vascular disease
involve?
A the measurement of blood speed.
B listening to the reflected ultrasound frequencies.
C using ultrasound to produce an image on a screen.
D increasing blood flow by warming the deep tissues.
ANSWER:A
1901
When ultrasound strikes the boundary between two different body tissues, the amount of
reflection that will occur is proportional to the difference in what?
A the speed of sound in the tissues.
B the density of the two tissues.
C elasticity in the tissues.
D impedance between the two tissues.
ANSWER:D
1902
Which statement about the differences between medical imaging using x-rays and a nuclear
medicine scan using gamma rays is correct?
A
an x-ray procedure leaves the patient with residual radioactivity while nuclear medicine does
not.
B
a gamma ray source can be switched off after which no gamma radiation is produced while
an x-ray source will continue to produce radiation until the source decays.
C
x-rays produce an image of internal anatomy while a nuclear medicine scan provides
information about the functioning of an organ or tissue.
D
a beam of gamma rays is fired at the patient and detected on the other side, while x-rays are
produced by the nucleus of a radionuclide incorporated in the patient’s body.
ANSWER:C
1903
Which of the following imaging modalities does NOT involve the use of “ion- ising
radiation”?

A mammography
B ultrasound
C a scintigram using technetium
D a chest x-ray
ANSWER:B
1904 When inspecting an x-ray image, the order of densities from blackest to whitest is:
A Bone, water, fat, air
B Air, fat, water, bone
C Air, water, fat, bone
D Bone, air, water, fat
ANSWER:B
1905 Which of the following imaging modalities uses x-rays?
A computed tomography (CT).
B single photon emission computed tomography (SPECT).
C positron emission tomography (PET).
D nuclear medicine scan (scintigram).
ANSWER:A
1906 Radiation which is “ionising” includes which of the following?
A x-rays and gamma rays
B infra-red radiation
C radiation emitted by mobile phones
D microwaves
ANSWER:A
1907
When compared to visible light, which is not very penetrating, why can radia- tion such as x-
rays and gamma rays pass right through the human body? Because:
A the density of the human body is relatively low.
B they have no mass and no charge.
C atoms in the body are mostly empty space.
D they have very high energy.
ANSWER:D
1908 Which of the following is true?
A A patient exposed to diagnostic x-rays will emit x-rays for a short time after the procedure.

B
A cancer patient treated with a megavoltage beam of x-rays will emit x-rays for a short time
after the treatment.
C
For a short time after having a bone scan using the radionuclide technetium m, the patient
will emit gamma rays.
D
The human body does not contain any radioactive material unless it has been exposed to
man-made radioactive material.
ANSWER:C
1909 What does the term Ionising radiation refer to?
A the radiation that is emitted by ionised atoms.
B
that part of the electromagnetic spectrum with wavelengths less than 00 nm which has
enough energy to produce ions.
C alpha, beta and gamma rays spontaneously emitted from radionuclides.
D radiation with enough energy to produce ionisation in the material which absorbs it.
ANSWER:D
1910 Which of the following is a correct use of the unit known as the “electron volt” (eV)?
A One electron volt is the amount of radioactivity that results in one disinte- gration per second.
B
Radiopharmaceuticals contain gamma photon emitting radionuclides whose energy is usually
in the range 00–0 keV
C One electron volt is equal to . × 0 joules of energy.
D A photon of visible light has energy of about . MeV.
ANSWER:B
1911 What may the term “ionising radiation” be applied to?
A all electromagnetic radiation
B radiation that produces ions when it interacts with matter
C infra-red radiation
D radiation that is emitted by ions
ANSWER:B
1912 What does it mean when an x-ray tube is operated at an accelerating voltage of 0 kV?
A the maximum energy that an x-ray photon can have will be 0 keV
B the characteristic x-rays will have energy 0 keV
C all of the x-ray photons will have an energy of 0 keV
D the x-ray beam will contain photons with every energy from 0 keV up to 0 keV

ANSWER:A
1913
Why do some x-ray photons will pass through the human body without deflec- tion?
Because:
A carbon, hydrogen and oxygen atoms are transparent to x-rays.
B the energy of diagnostic x-rays is too low to produce interactions
C
the wavelength of x-rays is too long to interact with an object with the dimensions of the
human body.
D the interior of atoms is mostly empty space
ANSWER:D
1914
How will increasing the filtration of an x-ray beam reduce the intensity of the x-ray
spectrum?
A equally at all frequencies
B more at lower frequencies than at higher frequencies
C more at higher frequencies than at lower frequencies
D only at lower frequencies
ANSWER:B
1915 What is a difference between x-rays and gamma rays?
A x-rays are ionising radiation and gamma rays are not
B gamma rays have higher energies than x-rays
C gamma rays can be turned off by switching the power supply off.
D
x-rays are produced in an electrical machine whereas gamma rays emerge from an atomic
nucleus.
ANSWER:D
1916 What is the purpose of adding filtration to an x-ray beam? To:
A Prevent high energy photons entering the patient
B Increase the mean energy of the beam.
C Decrease the scattered radiation
D Increase the ratio of low energy photons to high energy photons.
ANSWER:B
1917 What is the purpose of an intensifying screen?
A It converts a small number of x-ray photons into a large number of visible light photons.
B It converts low energy x-ray photons into high energy visible light photons.
C It improves the absorption efficiency of x-rays.

D It protects the radiologist’s eyes from the damage that would be caused by x-rays.
ANSWER:A
1918
One difference between the x-radiation in the primary beam and the scattered radiation is
that:
A Photons in the primary beam degrade contrast in radiographic images.
B Scattered radiation is more penetrating than the primary beam.
C Scattered radiation may be absorbed in the imager.
D Scattered radiation is travelling at an angle to the main beam.
ANSWER:D
1919 The advantage of Computed Tomography (CT) over conventional radiography is:
A CT delivers lower doses than conventional radiography.
B CT images are faster to acquire than conventional radiographs.
C CT produces a cross-sectional image that is not obscured by overlying ana- tomical structures.
D CT projects a D structure onto a D image.
ANSWER:C
1920 The contrast of a CT image displayed on a monitor may be increased by
A increasing the window width
B increasing the window level
C decreasing the window width
D decreasing the window level
ANSWER:C
1921
Exposing a foetus or young baby to x-rays should be avoideWhat is the cause of the danger
most likely to be due to?
A Denaturing of cells due to the increase in temperature in cells absorbing radiation.
B Damage to a cell’s DNA.
C The baby becoming radioactive.
D The formation of a blood clot.
ANSWER:B
1922 Which of the following statements about radioactivity is NOT correct?
A some of the atoms in our body are radioactive.
B radioactivity occurs naturally in the environment.
C radioactivity is associated with the nucleus of an atom.

D radioactivity is involved in diagnostic x-rays.
ANSWER:D
1923
In radiotherapy, why is the patient’s irradiation treatment “fractionated”, that is, consist of
(say) 0 sessions and spread over (say) weeks – rather than given all at session?
A
Fractionation allows time for the normal healthy tissue that is also irradi- ated, to recover in
between irradiations.
B
Extremely high energy electrons bombard the target of a linear accelerator, fractionation is
necessary to allow the x-ray target to cool.
C In order to irradiate the tumor over a period of time that it is growing.
D
Irradiating in a single session takes too long, people cannot remain immo- bile for the time it
would require.
ANSWER:A
1924
Why are radioisotopes that emit low energy (00–0 keV) gamma rays pre- ferred for the
diagnostic procedures of nuclear medicine? Because:
A high energy gamma rays are too easily stopped by body tissue.
B
radioisotopes that emit gammas within this energy range have a half-life that is ideal for
diagnostic procedures.
C
photons of this energy are sufficiently penetrating to escape from the body but are able to be
detected.
D charged particles are too difficult to shield against.
ANSWER:C
1925
Given the statement: “Your exposure to radiation varies inversely with the square of your
distance from the source of radiation”, what is the correct way to finish the sentence? “If you
increased the distance between you and a patient with a radioactive implant, from to m, your
exposure would”:
A increase by a factor of nine.
B decrease by a factor of one ninth.
C decrease by a factor of one third.
D increase by a factor of three.
ANSWER:B
1926 What may the term “radioactive” be correctly used to describe?
A a diagnostic x-ray machine used to produce radiographs.
B a linear accelerator used to produce x-rays for radiotherapy.

C a patient undergoing a nuclear medicine scan.
D a patient undergoing a CT (computed tomography) examination.
ANSWER:C
1927 What may the term radioactivity correctly used to refer to?
A The spontaneous emission of electromagnetic radiation from the nucleus of an atom.
B The particles or photons emitted from an unstable nucleus.
C The emission of particulate radiation from a radionuclide.
D The alpha, beta or X-radiation which emanates from some atomic nuclei.
ANSWER:B
1928 What does the physical half-life of a pure sample of radioactive material refer to?
A the amount of time taken for half of the radioactive atoms to decay.
B half of the time that it would take for all of the radioactive atoms to decay.
C the average time taken for any particular radioactive atom to decay.
D the time it takes for half of a sample of ingested radioisotope to be cleared from the body.
ANSWER:A
1929
Which of the following is the principal unit used when measuring the energy of ionising
radiation?
A joule
B electron-volt
C gray
D sievert
ANSWER:B
1930 What time does the half-life of a radioactive sample refer to? The time for the
A activity to halve.
B count rate to double
C parent nuclei to decay
D number of nuclei to halve
ANSWER:A
1931 ‘Half-life’ when applied to atoms of a radioactive isotope refers to the:
A midpoint of the time span for which the isotope will emit its radiation
B effective time for which the isotope is considered to be dangerous.
C length of time taken for half of the isotope to emit its radiation.
D time after which the radioactivity of the sample is half of its original value.

ANSWER:C
1932 What is a radionuclide with a short half-life (say hours) said to be?
A highly radioactive.
B weakly radioactive.
C of high penetrating ability.
D of low penetrating ability.
ANSWER:A
1933 Which of the following does NOT contribute to the background radiation?
A cosmic rays
B fluorescent lights
C radon gas
D uranium
ANSWER:B
1934 Which one of the following sources contributes to the background radiation?
A medical x-rays of bones.
B potassium 0 and carbon in our bodies.
C ultraviolet B radiation from the sun.
D microwave radiation.
ANSWER:B
1935 Which one of the following radiations, all of MeV energy, is the least penetrating?
A alpha rays
B beta rays
C gamma rays
D x-rays
ANSWER:A
1936 Which is the most penetrating nuclear radiation?
A alpha particles of energy MeV.
B beta particles of 0. MeV energy.
C x-rays with energy of MeV.
D gamma rays with 0 keV of energy.
ANSWER:C

1937
Suppose a gamma source is placed 0 cm from a radiation detector and in suc- cession, a cm
thick slab of each of the following materials is used to shield the source. For which one
would the count rate be lowest?
A lead
B wood
C aluminium
D cardboard
ANSWER:A
1938
Why are gamma rays are able to penetrate “solid” walls but alpha or beta rays cannot?
Because
A the speed of gamma rays ( × 0 m/s) is very much faster than either alpha or beta radiation.
B gamma rays do not have an electric charge whereas alpha is charged + and beta -.
C to a gamma ray, the atoms of the wall appear to be mostly empty space.
D gamma rays possess much more energy than either alpha or beta rays.
ANSWER:B
1939 What does the unit the sievert describe?
A radioactivity
B absorbed radiation dose
C absorbed radiation equivalent dose
D energy of radiation
ANSWER:C
1940
Why is keeping one’s distance from a source of radiation effective in minimis- ing exposure?
Because
A exposure is inversely proportional to distance
B exposure decreases as the inverse square of distance
C exposure decreases exponentially with distance
D electromagnetic radiation is absorbed by air
ANSWER:B
1941
Why is an absorbed radiation dose of 0 grays, absorbed over day (i.e. acutely) more
damaging to living tissue than the same dose received as 000 exposures of 0.0 grays over 0
years (i.e. chronically)?
A 0.0 grays is below the threshold dose that is known to damage cells.
B Rapidly dividing cells are more susceptible to damage from acute doses of radiation.

C Healthy cells can recover from low levels of radiation if the whole body is not exposed.
D Cells can repair radiation damage if given time between exposures to do so.
ANSWER:D
1942 A radiopharmaceutical is comprised of two components. These are
A a gamma emitter and a beta emitter.
B technetium and a non-radioactive carrier.
C a non-radioactive carrier and a radionuclide.
D a radionuclide and a radioactive carrier.
ANSWER:C
1943
Which one of the following statements about the radionuclide selected for inclusion in
radiopharmaceuticals is NOT correct?
A it should have a short half life
B it should emit alpha or beta rays
C it should emit gamma rays in the range 00–0 keV
D all the atoms should be radioactive
ANSWER:B
1944
Why is a radionuclide with a short half-life and which emits low energy gamma radiation
preferred for in vivo diagnosis? Because they
A have a low activity.
B are highly penetrating.
C emit their radiation in a short time span.
D are very damaging to cancerous tissue.
ANSWER:C
1945
Why are radioactive isotopes of the stable elements that occur normally in the body useful
for tracing metabolic pathways? Because
A such isotopes undergo chemical reactions that are identical to those of sta- ble isotopes.
B radioisotopes are indistinguishable from non-radioactive isotopes of the same element.
C
nuclear radiation can be detected outside the body and be used to produce an image of
internal structures.
D
radioactive forms of elements that exist naturally in the body do not pro- duce toxic effects
when used as a radiopharmaceutical.
ANSWER:A
1946 X rays for radiation therapy are produced by

A a linear accelerator
B cobalt 0
C an afterloading brachytherapy device
D technetium m
ANSWER:A
1947
In a radiotherapy treatment plan for a deep tumour, which of the following is part of the
treatment plan?
A Saturating the healthy tissue with oxygen to minimise its sensitivity to radiation.
B Using a single large dose to destroy all the tumour cells quickly.
C Using low energy alpha particles in order to minimise dose to healthy tissue.
D Splitting of the total dose into a number of smaller doses given daily.
ANSWER:D
1948
Why is it possible to trace metabolic pathways in the body using radioactive isotopes of the
naturally occurring elements in the body? Because, radioactive isotopes
A are used in such small quantities that they produce no toxic effects.
B have a very short half-life so soon decay to safe levels.
C are chemically identical to non-radioactive isotopes of the same element.
D are physically identical to non-radioactive isotopes of the same element.
ANSWER:C
1949
How has remote-controlled after-loading reduced the dose to staff involved in
brachytherapy?
A Patient exposure to radiation is reduced.
B The radionuclides are only in place in an operating theatre.
C Staff members are never exposed to radiation during the treatment.
D Only one staff member at a time is required in the brachytherapy room.
ANSWER:C
1950 What does the ‘maximum permissible dose’ of radiation (00 mSv/ yr period) refer to?
A The average dose for the general population as a whole.
B The dose for an individual not exposed to radiation through their work.
C The dose allowed to people exposed to radiation through their work.
D The dose above which radiation is likely to cause harm to humans.
ANSWER:C
1951 What does the ‘relative biological effectiveness’ (RBE) of radiation depends on?

A The recommended maximum permissible dose of radiation.
B The amount of radiation absorbed by the whole body.
C The dose of radiation that is actually absorbed in the tissue.
D The energy deposited in the tissue per millimetre of distance travelled through the tissue.
ANSWER:D
1952 Which of the following body parts is the least sensitive to radiation?
A lens of the eye
B red bone marrow
C gonads
D hands
ANSWER:D
1953
If a thickness of . mm of lead can absorb half of the MeV gamma photons that enter the lead,
then what fraction of gamma rays will be absorbed by twice this thickness (i.e. . mm)?
A Three quarters
B Seven eighths
C Fifteen sixteenths
D Sixteen sixteenths
ANSWER:A
1954
Given that the half-value layer of lead for a gamma photon of energy 0. MeV is 0. cm,
which of the situations listed below would result in the LEAST exposure to radiation?
Staying in the same room as the gamma source for
A minutes at a distance of m from the source with 0. cm of lead shielding.
B 0 minutes at 0. m with 0. cm of lead as shielding.
C minutes at m from the source using no shielding.
D 0 minutes at a distance of m using 0. cm of lead as shielding.
ANSWER:A
1955 Which of the following is NOT a principle used to set radiation safety standards?
A There is no completely safe dose.
B Any dose given must show a positive net benefit.
C Any dose received should be as low as reasonably achievable.
D Any dose received should not exceed that due to natural background radiation.
ANSWER:D

1956 Which is the correct statement about the behaviour of electrical charges?
A an electron and a proton will repel each other.
B two like charges would repel each other.
C like charges would attract each other.
D unlike charges repel each other.
ANSWER:B
1957
In an Australian domestic AC electrical circuit, what is the potential difference (voltage) in
the three wires?
A active wire oscillates between +0 V and −0 V, while neutral and earth are both at 0 V.
B neutral wire at –0 V, active at +0 V, earth at 0 V.
C active wire at +0 V, while neutral and earth are both at 0 V.
D neutral wire and active wire both at +0 V, earth at −0 V.
ANSWER:A
1958 When can electrical microshock occur? When:
A
contact between the skin and the 0 V domestic supply results in a current greater than 00
milliamps.
B a conductor carrying a current greater than microamp enters the body through the skin.
C a conductor carrying a current greater than 00 microamps enters the body through the skin.
D a current of to amps passes through the heart forcing it to clamp shut.
ANSWER:C
1959
Inside a typical household electrical appliance that has a three prong plug, what is the earth
wire connected to?
A The fuse.
B The neutral wire.
C The metal casing
D The on/off switch.
ANSWER:C
1960
Choose the alternative which correctly completes the following sentence: ‘There are two
types of electric charge called…
A protons and electrons and they attract each other.’
B positive and negative and they attract each other.’
C anions and cations and they repel each other.’

D electrons and ions and they repel each other.’
ANSWER:B
1961 If an ion has a positive charge then what do we know about it? That it
A will attract another ion with a positive charge.
B has gained some protons.
C has lost some electrons.
D has more electrons than protons.
ANSWER:C
1962 The unit of potential difference is the volt. What does the number of volts tell us?
A
How much energy one coulomb of electrons loses as it moves through the potential
difference.
B How much difficulty one coulomb of electrons has in moving through a circuit.
C How many coulombs of electrons are moving per second.
D How much energy per second electrons lose as they move through the circuit.
ANSWER:A
1963 Which three electrical quantities does Ohm’s law relate to each other?
A current, resistance and potential difference
B potential difference, current and voltage
C resistance, charge and ohms
D charge, current and potential difference
ANSWER:A
1964
Given Ohm’s law i.e. potential difference (V) is the product of current (I) and resistance (R),
which of the following statements is true? If potential difference is fixed (at say, 0 V):
A a high resistance means that a low current will flow.
B a high resistance means that a high current will flow.
C a low resistance means that a low current will flow.
D current will be fixed whatever the resistance.
ANSWER:A
1965 What is one difference between static electricity and current electricity?
A
Static electricity flows in the human body, while current electricity flows in electrical
appliances.
B No useful purpose has been found for static electricity.

C In current electricity, charges are moving whereas in static electricity, charges do not move.
D Direct current involves static electricity, while alternating current involves current electricity.
ANSWER:C
1966 What is one difference between direct current (DC) and alternating current (AC)?
A DC can produce a fatal shock, whereas AC cannot.
B AC can supply power to portable devices, but DC cannot.
C AC can be transmitted over long distances more cheaply than but DC.
D DC can be easily transformed to a different voltage, but AC cannot.
ANSWER:C
1967 Which one of the following statements is true?
A An electric current exists when electrons are moving through insulators.
B
Electrical resistance is a measure of the difficulty with which electrons move through a
conductor.
C Electrons are the carriers of electrical charge within our bodies.
D Static electricity results from a build up of charge on conductors.
ANSWER:B
1968
In order for the ‘on-off’ switch and the fuse to operate as intended in AC cir- cuits, which
wires must they be placed on?
A switch on active wire, fuse on earth wire.
B switch on earth wire, fuse on neutral wire.
C both switch and fuse on active wire.
D both switch and fuse on earth wire.
ANSWER:C
1969
When electrical equipment becomes faulty, the casing (if made of metal) may become live.
In this case what is the function of the earth wire on the equipment?
A To provide a path to ground that has high electrical resistance.
B To provide a path to ground that has low electrical resistance.
C To provide a path to ground for excess heat to flow through.
D To melt and thus break the circuit so that current can no longer flow.
ANSWER:B

1970
Suppose that while crawling around inside the roof space of your house, you touch a bare
wire with your bare hand while touching the brick wall with the other bare hanWhich of the
following wires would be most likely to deliver a fatal shock?
A The neutral wire.
B The active wire.
C The telephone wire.
D The earth wire.
ANSWER:B
1971 In an electrical device, a correctly installed earth wire provides protection against what?
A The electrical device receiving too high a current.
B It protects the fuse or circuit breaker from too much current.
C Electrocution of a person touching the faulty electrical device.
D It protects against stray currents that may exist in the ground (the soil).
ANSWER:C
1972
The electrical resistance of the body, measured from hand to hand, will be dif- ferent on
different occasions. Why is this? Because the resistance of the skin:
A Increases as the skin gets drier.
B Increases as the skin gets damper.
C Decreases as the skin gets drier.
D Decreases as the hands are brought closer together.
ANSWER:A
1973
What could be a correct definition of macro-electrocution? ‘That phenomenon which results
from a prolonged macro-shock produced by:
A an electric current flowing directly to the heart without having to cross the skin.’
B the contact of bare skin to alternating voltages of over 00,000 V.’
C a fatal current in direct contact with unprotected skin.’
D switching on a faulty device that was earthed and had a fuse on the active wire.’
ANSWER:C
1974
In the fluids of the human body, what are the carriers of charge that move through fluids
called?
A cations
B ions
C anions

D electrons
ANSWER:B
1975
Complete the sentence: ‘Defibrillation’ is the process where for a few millisec- onds, a direct
current of about:
A A is applied directly to the heart through a conducting path that bypasses the skin
B 00 mA is applied to the chest wall through two ‘paddles’
C A is applied the chest wall through two ‘paddles’
D 00 mA is applied directly to the heart through a conducting path that bypasses the skin
ANSWER:C
1976 What are the letters ECG short for? Electro-
A encephalogram.
B colonogram.
C cardiogram.
D cryogram.
ANSWER:C
1977
The potential differences generated by the heart muscle as it contracts and relaxes can be
measured by placing electrodes on the surface of the body. What is the record of these
electrical events correctly called?
A An electrocardiogram
B limb lead II
C An electrocardiograph
D An EEG
ANSWER:A
1978 What are the deflections seen on an electrocardiogram trace due to?
A pressure differences created by ventricular contraction.
B the closing and opening of heart valves.
C the de- and repolarisation of the cells of the myocardium.
D
variation in the electrical properties of oxygenated blood and deoxygenated blood as it
moves through the heart.
ANSWER:C
1979 The electrocardiogram chart is a graph. What is plotted against the vertical axis?
A The voltage produced by the myocardial cells.
B The elapsed time.

C The rate of contraction of the myocardium.
D The current flowing in the conduction system of the heart.
ANSWER:A
1980
Consider a cardiac monitor whose time-base control is set so that the ECG trace moves
horizontally at mm/seIf the patient’s heart rate is beats per min- ute, how far apart would the
peaks of the ECG trace be?
A mm
B mm
C 0 mm
D mm
ANSWER:C
1981
Why does a correctly installed Earth wire provides protection against electrocu- tion by a
faulty electrical device? Because:
A it is made of thick copper wire so presents almost zero resistance
B a fuse is located on eh Earth wire which will “blow” if excessive current flows.
C The Earth wire is connected to ground and to the metal case of the appliance.
D The on/off switch is on the Earth wire so that power can be turned off.
ANSWER:C
1982 Choose the INCORRECT statement from the four below.
A mass is the amount of matter contained in an object.
B gravity is the name of the force that acts between any objects with mass.
C weight is the pressure with which earth’s gravity acts on an object.
D for a standing person, base of support is the area bounded by their feet.
ANSWER:C
1983 Which of the following is correct?
A Fixed traction employs a moveable pulley to provide mechanical advantage.
B In suspended traction the traction force is equal to the tension in the cord.
C
In suspended traction, counter-traction is provided by the friction between the patient and
the bed.
D
The traction force must be equal in magnitude to but opposite in direction to the counter-
traction force.
ANSWER:D

1984
If a traction hanging mass is kg, use Newton’s second law (or otherwise) to determine the
gravitational force acting on the mass.
A newtons
B newtons
C 0 newtons
D kilograms
ANSWER:C
1985
Consider the action of moving from having both feet flat on the ground to standing on “tip
toes”. Which of the following correctly identifies the fulcrum, the load force, the effort force
and the lever?
A
Fulcrum is the heel, load is the body’s weight, effort is the pull of the ham- strings, lever is
the tarsal bones of the foot.
B
Fulcrum is the ball of the foot, load is the body’s weight, effort is the pull of the calf
muscles, lever is the tarsal and metatarsal bones of the foot.
C
Fulcrum is the ankle, load is the weight of the foot, effort is the pull of the hamstrings, lever
is the tibia & fibula bones.
D
Fulcrum is the knee, load is the weight of the foot, effort is the pull of the calf muscles, lever
is the tibia & fibula bones.
ANSWER:B
1986 A person whose mass is kg would have a weight closest to:
A kg because mass and weight are directly proportional to each other (on Earth).
B 0 kg because weight = mass ´0 (approximately)
C N because F=- F (Newton’s rd law)
D 0 N because F = m ´ a (Newton’s nd law)
ANSWER:D
1987 Which statement is consistent with “good lifting technique”?
A Keep your centre of gravity and that of the object being lifted as close as possible.
B Maximise the length of the effort arms of the body levers you use to lift the load.
C Use the relatively strong back muscles rather than the relatively weak thigh muscles.
D Minimise your base of support and bend your legs at the knees before you lean over.
ANSWER:A
1988
In a Hamilton-Russell traction system, a moveable pulley provides a mechani- cal advantage.
What does this mean? That the

A weight of the hanging mass is greater than the traction force.
B weight of the hanging mass is equal to the traction force.
C traction force is greater than the weight of the hanging mass.
D traction force is greater than the counter-traction force
ANSWER:C
1989
In straight leg traction (Buck’s extension) the force of counter traction is sup- plied by
friction (between the patient and the bed) and also by:
A the component of the patient’s weight that is perpendicular to the bed.
B the component of the patient’s weight that is parallel to the bed.
C gravity acting on the hanging mass.
D using cords to attach the patient to the head of the bed.
ANSWER:B
1990 A person whose mass is 0 kg would have a weight closest to:
A 0 kg because mass and weight are directly proportional to each other (on Earth).
B 0 N because F=- F (Newton’s rd law)
C 00 kg because weight = mass ´ 0 (approximately)
D 00 N because F = m ´ a (Newton’s nd law)
ANSWER:D
1991 A person who is standing on both of their feet is “stable” when:
A their base of support is as wide as is comfortably possible.
B their centre of gravity is above their base of support.
C their centre of gravity is close to their base of support.
D the position of their centre of gravity and base of support coincide.
ANSWER:B
1992
Use the equation P = F ¸ A to determine the pressure on the knee joint (area = 00 cm) when a
0 kg person is standing on one leg (assume that 0 kg is the body mass above the knee). Hint:
convert cm to m
A kPa
B 0 kPa
C 00 kPa
D 000 kPa
ANSWER:C
1993 Which statement concerning friction is INCORRECT?

A Friction within liquids is greater than that between dry solids.
B Within the human body friction is reduced by fluids such as saliva, serous fluid, mucus etc.
C Friction exists whenever two surfaces are in contact.
D Sliding friction is less than static friction.
ANSWER:A
1994 One of the following statements is WRONG. Which one?
A A patient’s inertia may be decreased by using several people to assist in their transfer.
B
The position of a person’s centre of gravity may be altered by altering the position of their
arms and legs.
C Base of support may be increased by adopting a wide stance with your feet.
D Friction between a patient and the bed may be reduced by using a slide sheet.
ANSWER:A
1995
Which one of the following is a nurse affecting, by positioning her body close to that of the
patient while executing a patient handling procedure?
A the mass of the patient to be shifted
B the maximum effort that her muscles are able to produce
C the distance between the position of muscle insertion and the fulcra of her limbs
D the distance between the centre of gravity of the patient and that of the nurse
ANSWER:D
1996
What is a nurse attempting to minimise by getting close to a patient during a manual
handling procedure?
A the load arm
B the size of the fulcrum
C the effort arm
D the patient’s weight
ANSWER:A
1997 Good manual handling technique on an unconscious patient involves which of the following?
A using the muscles of the arms
B instructing the patient on how to shift themself.
C using the muscles of the legs
D using the muscles of the back
ANSWER:C

1998
Most of our bones that articulate at freely movable joints can be described as third class
levers. What does this mean?
A they are “efficient” levers
B the “effort arm” is longer than the “load arm”
C the muscle’s tendon is inserted between the load and the joint
D the muscle’s tendon is inserted close to the joint
ANSWER:C
1999 Good manual handling technique generally requires extensive use of which muscles?
A biceps brachii and triceps brachii
B erector spinae and abdominal muscles
C gluteus maximus and rectus abdominus
D quadriceps and hamstrings
ANSWER:D
2000 What is the purpose of using of a slide sheet in patient manual handling?
A to extend the patient’s base of support
B to facilitate raising the patient’s centre of gravity
C to minimise friction between the patient and the bed
D to minimise the patient’s inertia
ANSWER:C
2001 What is the purpose of “counter-balancing” in manual handling? To
A ensure that your large leg muscles are used to shift a patient
B ensure a firm grip on the slide sheet
C increase the size of your base of support
D use gravity, acting on your weight, to shift a patient
ANSWER:D
2002 Most manual handling manoeuvres require bending at the knees. This is so that
A the strong thigh muscles are used.
B a wide base of support can be adopted.
C a stable body position is achieved.
D the back can be used as a third class lever.
ANSWER:A
2003 Which of the following is NOT a reason for using a slide sheet?
A To provide the handler with convenient hand holds while shifting the patient.

B To increase the patient’s base of support.
C To reduce the friction between the patient and their bed.
D To reduce the risk of damaging fragile skin.
ANSWER:B
2004 Which of the following statements is characteristic of “third-class levers”?
A the fulcrum lies between the effort and the load.
B the muscular effort involved in shifting them exceeds the load that is shifted.
C they are efficient levers.
D the effort and the load are equally distant from the fulcrum.
ANSWER:B
2005
Levers are acted upon by forces known as the load, effort, and fulcrum. “Third class” levers
are characterised by having the:
A effort located between the other two forces
B load located between the other two forces
C fulcrum located between the other two forces
D resistance located between the other two forces
ANSWER:A
2006
Levers are acted upon by forces known as the effort, load/resistance and ful- crum. “Third
class” levers are characterised by having the:
A effort located between the other forces
B load located between the other forces
C fulcrum located between the other forces
D resistance located between the other forces
ANSWER:A
2007
Levers are acted upon by forces known as the load, effort, and fulcrum. “First class” levers
are characterised by having the:
A effort located between the other two forces
B load located between the other two forces
C fulcrum located between the other two forces
D the effort and the load on opposite sides of the fulcrum
ANSWER:C
2008 What is the reason that a rd class lever is inefficient?
A the muscle’s effort force is applied closer to the fulcrum than the load force.

B the load force is applied closer to the fulcrum than the muscle’s effort force
C the fulcrum separates the effort from the load
D the load force is greater than the muscle’s effort force
ANSWER:A
2009 Which of the following is not an example of a force?
A Tension.
B Friction.
C Inertia.
D Weight.
ANSWER:C
2010
If the imaginary line joining a person’s centre of gravity to the centre of the Earth passes
through the person’s base of support, what can we say about that person? They are:
A unstable.
B balanced.
C using their back as a lever.
D not doing any work.
ANSWER:B
2011
A person (assumed to be healthy, awake and on their feet) is stable – that is, will not
overbalance and fall – as long as
A they have a large base of support
B their centre of gravity is close to the ground
C their centre of gravity remains within their body
D their centre of gravity is above their base of support
ANSWER:D
2012 What does the term ‘friction’ refer to?
A The tension force generated when a muscle contracts.
B A force that acts in the opposite direction to a motion.
C The resistance force that is overcome by an effort force.
D The force of gravity that causes an object to fall.
ANSWER:B
2013
Which one of the following describes what could happen to an object when a balanced force
is acting on it? The object
A starts to move.

B changes its direction but not its speed.
C object changes its shape.
D nothing happens.
ANSWER:D
2014 Which one describes when an unbalanced force is acting?
A an object’s centre of gravity is above its base of support.
B a moving object maintains a constant speed and direction.
C a muscle contracts and causes a limb to move.
D a soldier is standing rigidly to attention.
ANSWER:C
2015
In cars, the aim of safety features such as seatbelts, padding on the dashboard, collapsible
steering columns, airbags and body panels that crumple progressively, is to minimise the
unbalanced force on occupants during a crash. Bearing Newton’s second law in minHow do
such features achieve? They
A minimise the occupant’s deceleration.
B maximise the occupant’s deceleration.
C prevent whiplash injuries.
D convert an unbalanced force into a net force.
ANSWER:A
2016 Which of the following is NOT consistent with Newton’s second law?
A Weight = mass × .
B Acceleration = weight ¸ mass
C Mass = acceleration ¸ weight
D Force = mass ´ acceleration
ANSWER:C
2017 What is the best definition of the WEIGHT of an object? Weight is the:
A force of attraction between the Earth and the object.
B tendency of a body to maintain its state of motion.
C amount of matter contained in the body.
D mass of the object multiplied by its acceleration.
ANSWER:A
2018 What is the meaning of ‘work’ in the scientific sense?
A An artist’s completed painting.

B The amount of energy transferred between objects.
C The functions performed during the course of paid employment.
D Sustained physical or mental activity.
ANSWER:B
2019
In which of the following cases is the greatest amount of work being done on the object that
is experiencing the force?
A 0,000 N is exerted on a wall for 0 s.
B 000 N is used to raise an object by a height of 0 m.
C 000 N is used to push an object a distance of m.
D 000 N is used to pull an object over 0 m.
ANSWER:D
2020 What may be said about the measurable quantities which are referred to as vectors?
A They are arrow-headed line segments.
B They include time, mass, pressure and energy.
C They need a magnitude to be completely defined.
D They include force, velocity, acceleration and electric field strength.
ANSWER:D
2021
Why are the third class lever systems of the human musculoskeletal system inefficient?
Because
A third-class lever systems are the least efficient.
B the muscle insertion is closer to the joint than the load is.
C muscles can contract only by about twenty per cent of their relaxed length.
D the force of muscle tension is less than the weight of the load.
ANSWER:B
2022 What is the aim of ‘correct lifting technique’? To
A maintain balance by keeping the centre of gravity over the base of support.
B avoid working with heavy loads that are on the ground.
C use the bones and muscles of the leg.
D keep the back straight while using it as a lever.
ANSWER:C
2023 Which of the following is NOT considered part of good lifting technique?
A Keeping the feet close together.
B Keeping the back virtually straight.

C Standing close to the object to be lifted.
D Bending the knees.
ANSWER:A
2024
A requirement in traction is that the traction force is equal in magnitude but in the opposite
direction to the counter traction force. This requirement is really a statement of:
A Ohm’s law.
B Pascal’s principle.
C Newton’s first law.
D Newton’s third law.
ANSWER:D
2025
Traction forces may be represented by vectors. In Hamilton-Russell traction, which of the
following vectors is equal in magnitude to the traction force?
A The component of the patient’s weight that is perpendicular to the bed.
B The component of the patient’s weight that is parallel to the bed.
C The vector in subtracted from the patient’s weight.
D The resulting vector when the vectors in and are added.
ANSWER:B
2026
Fixed traction may be applied by a device such as a ‘Thomas splint’. In this case, the counter-
traction force is supplied by
A the push of the appliance on a fixed point on the body (such as the ischial tuberosity).
B an adhesive bandage wrapped around the lower leg.
C the pull of a wire that is made taut by turning a wingnut attached to the patient’s foot.
D the weight of the patient’s leg and the friction between the patient and the bed.
ANSWER:A
2027 The magnitude of the traction force in a Hamilton-Russell traction is deter- mined by
A The vector addition of the forces in the cords
B The hanging mass multiplied by ..
C The component of the patient’s weight that is perpendicular to the bed.
D The number of pulleys in the system.
ANSWER:A
2028
In a Hamilton-Russell traction system, the traction force is greater than the weight of the
hanging mass because:
A three cords are attached to the patient’s leg.

B there is a moveable pulley attached to the patients foot
C four pulleys are in the system.
D the traction force is parallel to the femur.
ANSWER:B
2029 Which of the following statements about the pulleys used in traction systems is correct?
A A moveable pulley changes the direction of the traction force.
B
A moveable pulley changes the direction of the traction force and provides a mechanical
advantage.
C A fixed pulley changes the direction of the traction and provides a mechani- cal advantage.
D A fixed pulley is one that is attached to the limb undergoing traction.
ANSWER:B
2030 What is one of the roles of the pulley in a traction system?
A To supply the counter traction.
B To enable the vector addition of forces.
C To change the direction of the traction force
D To prevent the hanging masses from resting on the floor.
ANSWER:C
2031
When a particular energy value is ascribed to a food what type of energy is being referred
to? Its
A translational kinetic energy.
B gravitational potential energy
C chemical potential energy
D average kinetic energy per molecule
ANSWER:C
2032 Energy may be defined from the concepts of
A force and heat.
B heat and joules.
C force and work.
D work and temperature.
ANSWER:C
2033 Which of the following statements about work is NOT correct?
A More work occurs when a force acts over a small distance (W = F ÷ s)

B Simple machines allow us to perform work more easily.
C Work (done on an object) is the amount of energy that transfers to that object.
D It takes energy to perform work.
ANSWER:A
2034 Work is done when an object is moved by a force. What is energy?
A the capacity to do work.
B the rate at which work is done.
C a force that results in no movement.
D the transformation of work from one form to another.
ANSWER:A
2035 What is a person’s metabolic rate defined as? The rate
A of energy utilisation by their body.
B of energy utilisation by their body during ‘absolute rest’.
C at which they consume oxygen.
D at which they produce heat.
ANSWER:A
2036
Consider a person whose diet provides a daily energy intake of 0,00 kJ and whose daily
activities result in their body consuming 00 kJ of energy daily. If this situation continues for
several months the possible outcome would be:
A a gradual rise in basal metabolic rate.
B The person increases their mass by several kilograms of fat tissue
C The energy requirements of daily activities will rise to consume 0,00 kJ
D An increase in radiation from the body to dissipate excess energy
ANSWER:B
2037
In order to lose body fat, diet and exercise must be organized so that the energy value of the
food intake is
A less than the energy used daily.
B more than the energy used daily.
C equal to the daily energy use.
D greater than the daily exercise.
ANSWER:A

2038
A block of wood (a poor conductor of heat) whose temperature is °C is placed in contact
with a block of steel (a good conductor of heat) of the same size but whose temperature is 0
°Both are touched with a hand whose skin temperature is °Which of the following is true?
A The steel block will feel colder than the wood.
B The wood block will withdraw more heat from the hand than will the steel block.
C Heat will flow from the steel block to the wood block.
D Heat will flow from the steel block to the hand.
ANSWER:A
2039 Which of the statements about heat is true? Heat is
A one of the forms of infra-red radiation.
B transmitted through a solid object by convection currents.
C a measure of the temperature of an object.
D the flow of energy from one body to another at a lower temperature.
ANSWER:D
2040
What does kinetic theory allows us to understand that the temperature of an object measures?
The
A heat that it contains.
B average kinetic energy of its particles.
C hotness or coldness of it.
D number of degrees kelvin it is.
ANSWER:B
2041 What may the ‘thermal energy’ of the particles in an object be defined as?
A the amount of heat that is contained in the object.
B sum of the random translational, rotational and vibrational kinetic energies.
C
sum of the random translational, rotational and vibrational kinetic energies and the work
done to overcome the intermolecular forces.
D average random kinetic energy of the particles.
ANSWER:B
2042
Which of the following factors contributes LEAST to the human sensation of ‘hot’ or ‘cold’
when an object is touched?
A The amount of heat in the object.
B The temperature of the object being touched.

C The thermal conductivity of the object.
D The local skin temperature.
ANSWER:A
2043 How does a clinical (or fever) thermometer differ from a standard thermometer?
A It contains mercury.
B It is a maximum reading thermometer.
C It measures temperature in kelvins.
D It contains a capillary tube.
ANSWER:B
2044 On a normal winter day, by what means does the human body lose most of its heat?
A convection.
B conduction.
C radiation.
D evaporation.
ANSWER:C
2045 What factor does NOT affect the amount of heat lost from the human body by radiation?
A The area of skin facing the external environment.
B The temperature difference between the skin and the surroundings.
C The surface area of the body.
D The mass of the body.
ANSWER:D
2046
By what means does a person sitting in the shade of a tree on a very hot dry summer day lose
most of their heat? By
A radiation.
B conduction.
C convection.
D evaporation.
ANSWER:D
2047
Why is heat loss from a hot object prevented when a material that is a poor conductor of
heat, is wrapped around a hot object? Because
A
the particles of the poor conductor easily transfer the kinetic energy of their vibrations to
their neighbors.
B water vapour is prevented from escaping to the air.

C it reflects radiated heat back into the hot object.
D air trapped within the poor conductor prevents convection currents from occurring.
ANSWER:D
2048
What is the reason that the temperature of a substance does not rise while it is in the act of
melting, even if heat is added to it?
A
Melting will not occur until the latent heat of vaporisation has been sup- plied, this prevents a
rise in temperature until all the solid has melted.
B
The added heat energy is used to increase the vibrational and rotational kinetic energies
rather than the temperature (i.e. translational kinetic energy) of the particles
C
The presence of colder unmelted particles alongside particles that are in liquid form causes
their temperature to remain low.
D
Any energy added to the substance is used to break the bonds that hold the particles into the
solid form so does not contribute to a temperature rise.
ANSWER:D
2049 Which of the following forms of heat therapy relies mainly on conduction?
A heat lamp.
B microwave diathermy.
C hot water bottle.
D ultrasound waves.
ANSWER:C
2050 What is the most effective way to deposit heat in bones and joints?
A ultrasound.
B infrared radiation.
C conductive heating (heat packs).
D diathermy.
ANSWER:A
2051 Which of the following is a biological mechanism of preventing heat loss?
A the production and evaporation of sweat
B increasing muscular activity
C vasoconstriction of superficial blood vessels
D seeking a warm environment
ANSWER:C

2052
Which of the following statements is correct? As a means of losing heat from the body,
evaporation of sweat works:
A provided that the surrounding air is not saturated with water vapour.
B provided that the surrounding environment is at a lower temperature that the body.
C because sweat is at a lower temperature than blood.
D because a film of sweat acts as an absorber of infra-red radiation from the surroundings.
ANSWER:A
2053
When the vibrating atoms of an object (at a temperature of 0 °C) pass on energy to the more
slowly vibrating atoms in an adjacent object with which it is in contact (and which is at a
lower temperature), what is this energy transfer known as?
A insulation
B convection
C radiation
D conduction
ANSWER:D
2054 How does the body attempt to cope with hyperthermia?
A by increasing muscular activity
B with peripheral vasodilation and sweating
C with peripheral vasoconstriction and shivering
D by reducing heat loss by radiation and convection
ANSWER:B
2055 In what situation will the ability to lose heat by evaporation of sweat be diminished?
A When the body is dehydrated.
B
When the ambient temperature of the surrounding environment is signifi- cantly higher than
body temperature.
C When the relative humidity of the surrounding air is very low.
D When very little bare skin is exposed.
ANSWER:A
2056
Which of the listed factors does NOT affect the amount of heat that the human body loses by
radiation?
A the temperature difference between the skin and the surroundings
B an individual’s behaviour
C being wrapped in a “space blanket” with a silver foil lining.

D the area of uncovered skin
ANSWER:D
2057 Which mechanism of heat loss from the human body is minimised by wearing clothes?
A convection of air
B radiation to the environment
C warming of inhaled air to body temperature before exhaling
D evaporation of sweat
ANSWER:A
2058 Evaporation of sweat cools our body because the evaporating water molecules
A have a high heat capacity.
B transfer kinetic energy away from us.
C radiate heat away from us.
D remove heat by conduction.
ANSWER:B
2059 What causes the cooling effect that we experience when sweating?
A the emission of infra-red radiation
B conduction of heat to the surrounding air then convection
C dripping off of warm sweat from our skin
D evaporating water molecules taking their kinetic energy with them
ANSWER:D
2060 How does the evaporation of sweat work as a heat loss mechanism?
A sweat is produced at a lower temperature than skin so cools the body by conduction.
B sweat promotes vasodilation which promotes heat loss by infra-red radiation.
C
the water molecules with the greatest energy evaporate, leaving the remain- ing ones at a
lower temperature.
D sweat flows across the skin surface so promotes heat loss by convection.
ANSWER:C
2061 Which heat loss avenues are reduced by wearing clothes?
A radiation and evaporation.
B conduction and convection.
C evaporation and convection.
D radiation and conduction.
ANSWER:B

2062
What may be said about the amount of energy in the form of infra-red electro- magnetic
waves that is radiated from our bodies?
A It depends on our body’s temperature.
B It would be greater if our layer of subcutaneous fat was thicker.
C
It may be increased by contact between the body and an object with tem- perature lower than
the body’s temperature
D It may be increased by exposing more bare skin.
ANSWER:A
2063
Except for one situation described below, water molecules changing state from liquid to gas
are involved in the cooling effect. Which one?
A a cold wind blowing against your skin
B drying off after a swim.
C the evaporation of sweat
D the exhalation of breath from the lungs
ANSWER:A
2064 What is the healthy human core body temperature?
A Below °C except during fever
B It lies between . and . °C
C It lies between and °C
D It lies between and °C
ANSWER:B
2065 How does subcutaneous adipose tissue assist the body to regulate its temperature?
A It conducts heat more readily than lean tissue so promotes heat loss.
B It stores heat energy so acts like a “heat sink”.
C It produces sweat for secretion via sweat glands.
D It conducts heat less readily than lean tissue so insulates the body.
ANSWER:D
2066 When is radiation an effective form of heat loss from the body?
A When we expose a greater amount of bare skin.
B When our body temperature is greater than that of our surroundings.
C When blood vessels close to the body surface are vasoconstricted.
D When our body temperature is less than that of our surroundings.
ANSWER:B

2067 Which choice explains how the evaporation of sweat “cools” our body?
A
Evaporating water molecules carry with them more than the average amount of kinetic
energy which leaves the remaining molecules with a lower aver- age kinetic energy.
B
The body loses more heat through the infrared radiation emitted by sweat than it gains from
the infrared radiation emitted by the surroundings.
C
Sweat is at a lower temperature than our core body temperature so sweat on our skin cools us
by conduction.
D
The water molecules in sweat are at a higher temperature than our core body temperature so
losing sweat leaves us cooler due to the mass of water lost.
ANSWER:A
2068 Why does an ice pack applied to a bruise reduce swelling?
A Less fluid leaks from the bruise due to the diminished nerve impulses.
B It causes vasoconstriction.
C It reduces the metabolic rate at the local site.
D It increases the viscosity of blood below the ice pack.
ANSWER:B
2069
Why will the metal bell of a stethoscope that is at the room’s air temperature, produce the
sensation of cold when placed on the patient’s skin?
A The skin is at a lower temperature than the bell.
B The stethoscope bell is a good conductor of heat.
C The stethoscope bell is a poor conductor of heat.
D Sweat evaporating from under the bell cools the skin.
ANSWER:B
2070 Why does perspiring cause heat energy to be lost from the body? Because:
A being at a lower temperature than the skin, sweat cools skin by conduction.
B evaporating water molecules remove heat in the form of their own kinetic energy.
C the presence of sweat on the skin prevents infra-red radiation being absorbed.
D sweat on the skin allows heat to be lost to the air by conduction.
ANSWER:B
2071
In which situation will the skin lose heat by conduction to an object that is in contact with it?
When the
A object is a good conductor of heat.
B skin is not covered by clothing.

C object is a poor conductor of heat.
D object is at a lower temperature than the skin
ANSWER:D
2072 Which of the following is the most acceptable definition of the term “power”?
A the rate of doing work.
B that which is stored and can be fully recovered and converted to kinetic energy.
C the concept applied to that which gives an object the ability to do work.
D the sum of an object’s potential and kinetic energies.
ANSWER:A
2073
Which of the following correctly states the principle of conservation of energy in terms of
the human body? (assume no foodstuffs are consumed and no urine or faeces are excreted).
A
Q = s × m × ΔT (Q = energy, s = specific heat of tissue, m = body mass, T = body
temperature)
B
The energy stored in the human body is equal to the energy lost from the body plus the
work done by the body.
C
The energy value of the food we eat must exceed the energy value of the muscular activity
we perform.
D
The change in the energy stored in the body is equal to the heat lost from the body plus the
work done by the body.
ANSWER:D
2074 Which of the statements is a description of basal metabolic rate in a human?
A
The sum total of the energy released per minute by all of the chemical reac- tions that occur
in the body.
B the rate of energy utilisation during “absolute rest”.
C The power generated by the body’s activities.
D The oxygen consumption (in l/min) of an individual.
ANSWER:B
2075 Which is the correct distinction between temperature and heat?
A
Temperature is a measure of the amount of heat in an object and heat is the energy that flows
between objects which have different temperatures.
B
Heat is a measure of the energy contained within an object and temperature is the objective
measurement of heat.

C
Temperature is a measure of the average kinetic energy of particles and heat is the energy
that flows between objects as a result of a temperature difference.
D
Heat is the energy that flows from a cold object to a hotter object and tem- perature measures
the average random kinetic energy of the particles.
ANSWER:C
2076
A cold pack, applied to reduce swelling, is more effective if it contains melting ice at 0 °C
rather than water at 0 °Why is this?
A Because ice cools by conduction whereas water cools by convection.
B Because initially the melting ice is colder than the cold water.
C Because ice has a higher latent heat of vaporisation than water.
D Because melting ice remains at 0 °C until it has all melted.
ANSWER:D

!"#កទី ៤
Obstetrics & Gynecolog
No Question
1 All are complication of abruptio placenta EXCEPT:
A Macrosomia
B IUFD
C DIC
D PPH
ANSWER:A
2 Velamentous insertion of the cord is associated with an increased risk for:
A Premature rupture of the membranes.
B Fetal bleeding before labor.
C Torsion of the umbilical cord.
D Fetal malformation.
ANSWER:B
3 Anti-D prophylaxis:
A Should be given to all sensitized Rhesus negative women after delivery
B Should be given to all Rhesus negative women after amniocen tesis.
C Should be given to all Rhesus positive women who give birth to Rhesus
D negative babies.
ANSWER:B
4 In Rhesus Iso-immunization, the following test may be helpful :
A Rhesus antibody titer in liquor
B Maternal serum bilirubin level
C Liquor bilirubin level
D Maternal hemoglobin
ANSWER:C
5 RH disease :
A Occurs when the mother is Rh+
B Occurs when the father is RHC.
C Occurs when the fetus is Rh+ve
D Can never occurs in the 1st pregnancy
ANSWER:C
6 Rh isoimmunization Anti-D immunoglobulin should be given:

A After every abortion occurring more than 8 weeks gestation
B To all Rh negative females who have an Rh positive baby
C Postpartum only to Rh negative female who are sensitized
D Postpartum to Rh positive female with Rh negative husbands
ANSWER:B
7 RH incompatibility occurs with :
A Rh –ve father & Rh+ve mother
B Rh –ve mother & Rh –ve father
C Rh–vemother&Rh+vefather
D Rh +ve mother & Rh +ve father
ANSWER:C
8 In ABO incompatibility :
A The mother has to be blood group AB
B The father has to be B1 group O
C It doesn't protect against RH disease
D It is an antigen antibody reaction
ANSWER:D
9 Fetal RBCs can be distinguished from maternal RBCs by their :
A Shape
B Resistance to acidelution
C Lack of Rh factor
D Lower amounts of hemoglobin
ANSWER:B
10 Anti-D immunoglobulin should be given:
A To Rh-negative mothers after every abortion occurring be yond 6-8 week's gestation.
B To all sensitized Rh-negative females who have Rh-positive.
C Postpartum only to Rh-negative females who are sensitized regardless of the fetal blood type.
D Postpartum to Rh-positive females with Rh-negative husbands.
ANSWER:A
11 Fetal RBCs can be distinguished from maternal RBCs by their:
A Shape.
B Resistance to acidelution.
C Lack of Rh factor.

D Lower amount of hemoglobin.
ANSWER:B
12 If blood must be given without adequate cross matching, the best to use is:
A AB Rh-positive.
B AB Rh-negative.
C O Rh-positive.
D O Rh-negative.
ANSWER:D
13 Regarding Postpartum haemorrhage:
A May occur as aconsequence of Antepartum haemorrhage.
B Ends with Hypercoagulable state
C Hysterectomy is the first the first line of treatment
D Always complicate intrauterine fetal death (IUFD)
ANSWER:A
14 In Abruptio placenta: Which is true?
A It is bleeding from abnormally situated placenta
B Has minimum effect on the fetus
C Causes painless bleeding
D Can be a cause of postpartum hemorrhage
ANSWER:D
15 Postpartum hemorrhage can occur due to all the followings EXCEPT:
A Fetal macrosomia.
B Polyhydramnios.
C Placenta brevia.
D Post date pregnancy.
ANSWER:D
16 Disseminated intravascular coagulation (DIC) :
A Can be managed by leukocyte transfusion
B Can cause decrease fibrinogen degeneration products
C Can cause decrease PT , PTT
D Can been countered in case of IUFD
ANSWER:D

17
All of the following circumstances should alert an obstetrician to an increased likelihood of postpartum
hemorrhage EXCEPT:
A Prolonged labor
B Rapid labor
C Post date pregnancy
D Oxytocin stimulation
ANSWER:C
18 In DIC: Disseminating intravascular coagulation
A The level of FDP (Fibrinogen degeneration products) is low
B Platelet count is high
C Bleeding time is prolonged
D PT and PTT are normal
ANSWER:C
19 Clinical Presentation of DIC include the following EXCEPT:
A Bleeding from IV sites
B Hematuria
C Failure active surgical hemostasis
D Increase temperature (fever)
ANSWER:D
20 Clinical causes of DIC ( disseminating intravascular coagulation ) include the following EXCEPT :
A Ectopic pregnancy
B Septic abortion
C Mildpre-eclampsia
D The use of tampons
ANSWER:C
21 The treatment of DIC may include the following EXCEPT:
A Heparin
B Packed RBCs
C Platelet transfusion
D Leukocyte transfusion
ANSWER:D
22 Ergometrine to control post-partum hemorrhage :
A Is contraindicated in patient with high blood pressure

B It will not act on the smooth muscle of the blood vessels
C Intravenous root is the only way to be given
D It can be used for induction of labor
ANSWER:A
23
A 28 years old patient complains of the amenorrhea after D & C for postpartum bleeding. The most likely
diagnosis:
A Gonadal dysgenesis
B Sheehan's syndrome
C Kallman's syndrome
D Asherman's syndrome
ANSWER:D
24 Post-partum hemorrhage may have higher incidence in all the following conditions EXCEPT:
A Multiple pregnancies
B Polyhydramnios
C Macrosomic baby
D Placenta previa
ANSWER:B
25 All the following are causes of DIC in pregnancy, EXCEPT :
A Fetal demise.
B Abortion.
C Placental abruption.
D Placentaprevia.
ANSWER:D
26 Regarding postpartum hemorrhage the following are true EXCEPT:
A blood loss of 500 ml or more after vaginal delivery.
B Blood loss of 1000 ml or more after C-section.
C Atonic hemorrhageis less common than traumatic one.
D Multiple pregnancy may predispose to postpartum hemorrhage.
ANSWER:C
27 The following statements are correct EXCPT:
A Syntometrine is composed of syntocin on & Ergometrine.
B Ergometrine is contraindicated in cardiac patient.
C Syntometrine is used prophylcatically in the management of 3RD stage of labor.

D Syntometrine is contraindicated in cardiac patients.
ANSWER:A
28 The following are always indications for Caesarean Section
A Hydrocephalus
B Abruptio placenta
C Preterm Labor
D Active primary genital herpes
ANSWER:D
29 Obstructed labor: Which is true?
A Diagnosis only when the cervix is fully dilated
B Usually predicted before onset of labor
C More common in developed countries
D Mento-posterior position could beacause
ANSWER:D
30 Prolapse of umbilical cord: Which is true?
A Not an indication for caesarean section when baby viable at 36 weeks
B Diagnosed when membranes are still intact
C Is more common when fetus acquires an abnormallie
D Incidence is 5%
ANSWER:C
31 The best uterine scar a patient can have for Caesarian section is
A Transverse upper segment
B Longitudinal upper segment
C Transverse lower segment
D Longitudinal lower segment
ANSWER:C
32 Which of the following is not a basic component of an obstetric forceps ?
A Blade
B Handle
C Lack
D Stem
ANSWER:D
33 Which instrument is not a basic of a laparoscopic set :

A Trochar
B Hegar dilator
C Veress needle
D Light source
ANSWER:B
34 Which of the followings is a contraindication to a trial of labor after cesarean delivery?
A Prior classical incision.
B Prior cesarean delivery for dystocia.
C Prior IUFD.
D Ultrasound estimation of fetal weight of 3500g.
ANSWER:A
35 Indications for instrumental delivery include all the followings EXCEPT:
A Prolonged second stage of labor.
B Fetal distress.
C Transverse lie.
D Breech presentation.
ANSWER:C
36 Obstructed labor:
A Diagnosed only when the cervix is fully dilated.
B Usually predicted before the onset of labor.
C More common in developed countries.
D Mento-posterior position could beacause.
ANSWER:D
37 Prerequisites for instrumental delivery include all the followings EXCEPT:
A Cephalic presentation.
B Engaged head.
C Full dilation of the cervix.
D The presence of epidural analgesia.
ANSWER:D
38 Vacuum extraction (ventouse):
A Causes more maternal birth canal injuries than the forceps.
B Can be used when the cervix is 7 cm dilated.
C Can be applied when the vertex is minus 2 station.

D Can cause Cephalohematoma to the baby.
ANSWER:D
39 Which of the following change in puberty is influenced by the estrogen:
A Growth of the acinar buds of the breast
B Epiphyseal fusion
C Proliferatve phase
D All of the above
ANSWER:D
40 not exclusevely in the stomach121. Pelvic ultrasound is helpful in the diagnosis of:
A Endometrial carcinoma
B Asherman’s syndrome
C Ascites
D Ovulation detection
ANSWER:D
41 Glycogen is seen in the lumina of endometrial glands :
A During the luteal phase
B During pregnancy only
C During pre and post ovulatory
D During proliferative phase only
ANSWER:A
42 The Wolfian duct in the female :
A Develops into the fallopian tube
B Forms the ovary
C Forms the round ligament
D Regresses and becomes vestigial
ANSWER:D
43 Large amount of alkaline phosphatase may be demonstrated in the endometrium of :
A Decidua
B Secretory phase
C Proliferative phase
D All of the above
ANSWER:C
44 Oxytocin and vasopressin are transferred from hypothalamus to neurohypophysis through:

A Venous channels
B Lymphatics
C Nerve axons
D All of the above
ANSWER:C
45 The function of round ligament is :
A Vestigial with no apparent function
B To prevent retrodisplacement of the uterus
C To prevent uterine prolapse
D To provide nerve supply of the upper vagina
ANSWER:B
46 The definitive epithelium of vagina is derived from :
A Wolfian duct
B Mullerian duct
C Urogenital epithelium
D Coelomic epithelium
ANSWER:C
47 Causes of post partum amenorrhoea may be :
A Anorexia nervosa
B Cervical atresia
C Chlorpromazaine therapy
D Any of the above
ANSWER:D
48 The cyclic production of pituitary hormones is dependant upon:
A Normal menstruation
B An intact pituitary- portal system
C An adult anterior pituitary gland
D All of the above
ANSWER:B
49 The clots passed with menorrhagia perhaps indicate
A No endometrial regeneration
B No terminal arteriolar spasm
C Large amount of bleeding

D All of the above
ANSWER:C
50 Monilial vagintis occurs frquently during pregnancy because :
A Glycosuria is commoner
B The vagina contains more glycogen
C Higher vaginal acidity suppresses other organisms
D All of the above
ANSWER:D
51
Physical exam reveals the uterus to be about 6 wk size. Vaginal bleeding is scanty with no discernible tissue in the
cervical os. There are no palpable adnexal masses. The uterus is mildly tender. Ultrasonographic exam does not
reveal a gestational sac. Which of the following should be recommended?
A Dilatation & curettage.
B Culdocentesis.
C Observation followed by serial B-HCG determinations.
D Diagnostic laparoscopy.
ANSWER:D
52 Which of the following statements is incorrect regarding levonorgestrel releasing intrauterine system:
A There is increased incidence of menorrhagia
B This system can be used as hormone replacement therapy
C This method is useful for the treatment of endometerial hyperplasia
D Irregular uterine bleeding can be problem initially
ANSWER:A
53 Myxoma peritonei may occur as a consequence of rupture of which ovarian cyst ?
A Dermoid
B Struma ovarii
C Serous cystadenoma
D Mucinous cystadenoma
ANSWER:D
54 Lutein and theca lutein cysts may be associated with all the following except :
A Mole
B Chorionepithelioma
C Stein-leventhal syndrome(PCO)
D Pregnancy

ANSWER:C
55 The preferred treatment of ruptured tubo-ovarian abscess is :
A Cul-de-sac drainage
B Removal of uterus , tubes and involved ovary
C Removal of uterus , tubes and ovaries
D Removal of ruptured tube and ovary
ANSWER:C
56 The cysts of Stein -Leventhal ovary or PCOD are of which kind?
A Lutein
B Germinal inclusion
C Follicular
D Theca lutein
ANSWER:C
57 Clinical findings of PCOD include all except :
A Obesity
B Olgomenorrhoea
C Infertility
D Tall stature
ANSWER:D
58 Presence of pyometra in a post menopausal females strongly suggests:
A Diabetes mellitus
B Degenerating myoma
C Senile endometritis
D Malignancy
ANSWER:D
59 The most common symptom associated with adenomyosis is :
A Infertility
B Menorrhagia
C Haematometra
D Dyspareunia
ANSWER:B
60
Medadteam.org More than you dream125. A 63 old lady presents with abdominal mass & weight loss , was
diagnosed as having an ovarian tumor , the most common ovarian tumour in this woman would be…:

A epithelial tumour
B germ cell tumour
C stromal tumour
D sex cord tumour
ANSWER:A
61
There is a 5% incidence of primary extrauterine malignancy associated with endometrial cancer, the most frequent
site for such is :
A Stomach
B lung
C Breast
D Bone
ANSWER:C
62 The cause of virilizing adrenal hyperplasia is :
A Defect in cortisol synthesis
B defect in ACTH synthesis
C Defect in testosterone synthesis
D All of the above
ANSWER:A
63 Anterior pituitary function may be blocked by:
A Blood levels of steroids
B Emotional factors
C Sensory stimuli
D All of the above
ANSWER:D
64 Subnuclear vaculoes in the endometrial mucosa are evidence of activity of:
A Cholesterol
B Progesterone
C Pregnendiol
D Androstenendione
ANSWER:B
65 The commonest cause of death in cancer cervix is :
A Infection
B Uraemia

C Haemorrhage
D Cachexia
ANSWER:B
66 Failure to find sperm in postcoital examination may be due to :
A Excessive oestrogen effect on cervical mucous
B Excessive vaginal lactic acid
C Oligospermia
D All of the above
ANSWER:C
67 The differential diagnosis of vaginal cysts include :
A Cystocele
B Urethral diverticulum
C Urethrocoele
D All of the above
ANSWER:D
68 Factors in cervical cancer development EXCEPT:
A HIV infection
B Chlamydia infection
C Breast cancer
D Smoking
ANSWER:C
69 Non-neoplastic ovarian cysts include all of the following except:
A follicular cyst
B theca lutein cyst
C dermoid cyst
D corpus luteum cyst
ANSWER:C
70 Which of the following ovarian tumor is most prone to undergo torsion duringpregnancy?
A Serous cystadenoma
B Mucinous cystadenoma
C Dermoid cyst
D Theca lutein cyst
ANSWER:C

71 Magnesium sulphate toxicity include all EXCEPT:
A CNS depression
B This drug acts only on motor end plate
C Respiratory depression
D muscle relaxant
ANSWER:B
72 In DUB all are right except,:
A may be associated with hypothyroidism
B may be associated with post-menopausal bleeding
C may be associated with functional ovarian cysts
D may present as menorrhagia
ANSWER:B
73 Metabolic causes of anovulatory DUB include all except :
A Hypothyroidism
B Halban’s syndrome
C Cushing’s syndrome
D Hyperthyroidism
ANSWER:B
74 The primary drainage of the lower vagina is to :
A external iliac nodes
B Sacral nodes
C Femoral nodes
D superficial inguinal nodes
ANSWER:D
75 The most common causative organism in acute bartholinitis is
A Staphylococcus
B Streptococcus
C Colon bacillus
D Gonococcus
ANSWER:D
76 The uterine artery supplies the
A Vagina
B Lower cevix

C Ovary
D All of the above
ANSWER:D
77 Common accompaniments salpingitis are :
A Pelvic mass
B Bleeding
C Pain
D All of the above
ANSWER:C
78 Cervical amputation :
A Is followed frequently by abortion
B Is associated with high incidence of post operative sterility
C Is not frequently followed by cervical dystocia in patients who become pregnant
D All of the above
ANSWER:D
79 The effect of ovarian steroid on anterior pituitary is
A Direct stimulation
B Direct inhibition
C Mediated via hypothalamus
D Unknown
ANSWER:C
80 What are the signs of ovulation on Ultrasonography :
A Irregular follicle wall
B Collapse of follicle
C Fluid in cul de sac
D All of the above
ANSWER:D
81 The uterus is held in anteflexed position by :
A The ventral pull of round ligament
B The dorsal pull of uterosacral ligaments
C Its weight
D All of the above
ANSWER:D

82 The most common symptom of adenomyosis is :
A Dysmenorrhoea
B Menorrhagia
C Pain
D Fever
ANSWER:B
83 The commonest cause of stress incontinence is
A Constipation
B Raised intra abdominal pressure
C Congenital weakness of sphincter
D Childbirth trauma
ANSWER:D
84 Perforation of the uterus while doing endometrial biopsy in non pregnant uterus, needs
A Laparoscopy
B Observation
C Immediate laparotomy
D Hysterectomy
ANSWER:A
85 Least common type of uterine anomaly in patients with recurrent pregnancy loss :
A Unicornuate
B Arcuate
C Septate
D Bicornuate
ANSWER:A
86 Diagnosis of stress incontinence coded by which of the following before taking the patient for surgery
A History
B Subjective demonstration of stress incontinence
C Objective demonstration of stress incontinence
D Urodynamic studies
ANSWER:D
87 The following are the factors associated with CIN EXCEPT
A Onset of coitus at early stage
B Multiple sexual partners

C Lower socioeconomic status
D Nulliparity
ANSWER:D
88 Best treatment for severe stress incontinence without prolapse is
A Pelvic floor exercise
B Kelly’s repair
C Burch colposuspension
D MMK operation
ANSWER:B
89 60. Bartholin’s gland duct opens in…..
A Upper third of labia majora
B Middle third of labia majora
C Upper third of labia minora
D Middle third of labia minora
ANSWER:D
90
A 54-year-old woman comes to the physician for an annual examination. She has no complaints. For the past year,
she has been taking tamoxifen for the prevention of breast cancer. She was started on this drug after her physician
determined her to be at high risk on the basis of her strong family history, nulliparity, and early age at menarche.
She takes no other medications. Examination is within normal limits. Which of the following is this patient most
likely to develop while taking tamoxifen?
A Breast cancer
B Elevated LDL cholesterol
C Endometrial changes
D Myocardial infarction
ANSWER:C
91 Ovarian precursors of oestradiol include :
A Oestrone
B Androstenedione
C Testosterone
D All of the above
ANSWER:D
92 Female patient with endometrial hyperplasia could be all of these except:
A thecoma

B fibroma
C Brenner tumor
D follicular cyst
ANSWER:B
93 Endometroid cyst, on examination:
A adenexal tenderness
B cyst felt in thin people
C cyst fixed and tender
D All of the above
ANSWER:D
94 Considering epithelial neoplasm of the ovaries all true except :
A the commonest
B mucinous cystadenoma lined by tubal epithelium
C Brenner tumor lined by urinary tract epithelium
D embryologically arise from wolffian epithelium .
ANSWER:B
95 The Commonest ovarian neoplasm complicated with torsion during pregnancy:
A fibroma
B teratoma
C simple serous cyst
D thecoma .
ANSWER:B
96 Female patient with acute abdomen , CBC normal , B-HCG negative , No vaginal bleeding , Mostly is :
A hemorrhagic teratoma
B disturbed ectopic pregnancy
C appendicitis
D peritonitis .
ANSWER:A
97 Considering mucinous cystadenoma :
A the commonest neoplasm
B usually bilateral
C sometimes fill the entire abdominal cavity
D lined by tubal epithelium .

ANSWER:C
98 Considering Brenner tumor all true except :
A potential malignant is common
B histologically has epithelial nests and coffe bean nuclei
C vaginal bleeding reported with it
D usually in childbearing women
ANSWER:D
99 Considering malignant ovarian neoplasm histologically may be all except :
A epithelial tumors
B germ cells tumor
C cystic and solid tumors
D sex cord tumors .
ANSWER:C
100 For endometrial cyst all true except :
A choclate cyst on TVS
B laparoscope is indicated
C C125 is a specific test
D associated with dysmenoorrhoea .
ANSWER:C
101
Female patient with history of induction of ovulation present with tender lowerabdominal pain and discomfort ,
TVS show cyst , Next step is :
A assurance sending home
B hold ovarian stimulatin drug
C laparotomy
D None of the above
ANSWER:B
102 Considering endometroid cyst :
A not uncommon
B due to menstrual reaction
C torsion is common
D A & B
ANSWER:D
103 Considering endometrial cyst ttt all true except :

A GNRH is of benefit
B laparosope idicated in small cyst
C laparotomy is preferred
D recurrence is not common
ANSWER:D
104 Considering the follicular cyst it is rarely associated with :
A endometrial hyperplasia
B acute abdomen
C polycystic ovary
D On PV in obese patient it may rupture .
ANSWER:B
105
Female patient with history of endometriosis , menstrual disorders complaining from pain on right iliac fossa , on
examination there was tenderness on right iliac fossa with no rebound pain no rigidity , on CBC it was normal ,
most likely :
A peritonitis
B appendicitis
C follicular cyst
D None of the above
ANSWER:C
106 Considering a case of follicular cyst it need all of following except :
A assurance follow up
B OCP
C usually surgical removal
D repeated US
ANSWER:C
107 Considering the follicular cyst all of following is true except :
A associated with metropathia hemorrhagica
B OCP indicated in ttt
C the second common functional cyst
D TAS is the gold standard diagnostic method
ANSWER:C
108 Considering the endometroid cyst :
A associated with dysmenorrhoea

B associated with pelvic pain
C associatd with pelvic endometriosis
D All of the above
ANSWER:D
109
Female patient with history of hydatiform and complaining of lower abdominal pain , on examination there was
tenderness on palpation and the lab result show high level of HCG , most likely to be :
A follicular cyst
B theca lutein cyst
C corpus luteum cyst
D None of the above
ANSWER:B
110
A 29-year-old G4P4 is found to have an abnormal smear signed out as atypical glandular cells, favouring
neoplasia. She undergoes a colposcopy with cervical biopsies. One of the ectocervical biopsies demonstrated
adenocarcinima in the situ. The most appropriate next step is:
A Vaginal hysterectomy
B Radical hysterectomy/Radiotherapy
C Cold-knife conization of the cervix
D Loop excision of the cervical tranformation zone
ANSWER:C
111 The following about human papilloma virus (HPV) infection are correct EXCEPT:
A It is the most common viral STDs.
B It may lead CIN and cervical cancer.
C It is due to RNA virus.
D Infection may be warty or flat condyloma.
ANSWER:C
112 The lymphatic drainage of the cervix is to the following lymph nodes EXCEPT:
A The femoral lymph nodes.
B The internal iliac lymph nodes.
C The para-cervical lymph nodes.
D The pre-sacral lymh nodes.
ANSWER:A
113 The commonest secondary change in uterine fibroids is:
A Fatty degeneration

B Myxomatous degeneration.
C Hyaline degeneration
D Cystic degeneration
ANSWER:C
114 The following is correct about the ovarian ligaments:
A Contain ureters.
B Contain ovarian arteries.
C Are attached laterally to pelvic wall.
D Lie anterior to the broad ligament.
ANSWER:B
115 Involves pelvic LN clearance, hysterectomy, removal of the parametrium and theupper third of the vagina.
A Wartman’s hysterectomy
B Wertheim’s hysterectomy
C Wertheims Trachelectomy
D Radical trachelectomy
ANSWER:B
116 5 year survival for someone with stage 3-4 cervical carcinoma
A 10-30%!!!
B 80-95%
C 2-10%
D 65-80%
ANSWER:A
117
Cervical carcinoma spread and staging: Microinvasion of the basement membrane, <7mm across, with no
lymph/vascular space invasion
A Stage 1b
B Stage 3
C Stage 4
D Stage 1a
ANSWER:D
118
A 42-year-old G4P4 has had postcoital bleeding for the past four months. She has not had a Pap test since the
delivery of her last child 7 years ago. Speculum examination shows a vaginal discharge and a 1 cm exophytic
lesion of the posterior cervical lip. The most appropriate next step is:
A Perform a Pap smear

B Perform a cold-knife conization
C Give the patient a course of intravaginal Metronidazole gel followed by reexamination in 6 weeks
D Perform a punch biopsy of the lesion
ANSWER:D
119 The area where cervical carcinoma usually originates
A Neoplastic zone
B Metaplastic field
C Retrograde area
D Transformation zone
ANSWER:D
120 Cervical carcinoma is most common between the ages of
A 45-55
B 16-18
C 18-22
D 35-45
ANSWER:A
121 Cervical carcinoma characteristically spreads in the
A Tissue
B Lymph
C Bone
D Blood
ANSWER:B
122 Which of the following is thought to be protective against CIN?
A HIV
B Oral contraceptive usage
C Long term sexual abstinence
D Smoking
ANSWER:C
123 Cervical carcinoma that can be treated with cone biopsy
A Stage 5
B Stage 1a
C Stage 3
D Stage 2a

ANSWER:B
124 The presence of atypical cells within the squamous epithelium
A Cervical dyskaryosis
B Nabothian follicles
C Dysplasic dyskaryosis
D Cervical intraepithelial neoplasia
ANSWER:D
125 5 year survival for someone with stage 1a cervical carcinoma
A 95%
B 10%
C 30%
D 80%
ANSWER:A
126 Cervical carcinoma spread and staging: Invasion of the lower vagina or pelvicwall, or causing ureteric obstruction
A Stage 1a
B Stage 4
C Stage 3
D Stage 2a
ANSWER:C
127 HPV types _________ are the most significant and account for 70% of allcervical cancers
A 45 and 46
B 31 and 33
C 14 and 16
D 16 and 18
ANSWER:D
128 Ovarian tumors which may produce chorionic gonadotrophins include :
A Dysgerminoma
B Teratoma
C Choriocarcinoma
D All of the above
ANSWER:D
129 Pathology of endometriosis may be explained by :

A coelemic metaplasia
B endometrial hyperplasia
C retrograde menstruation
D intraperitoneal immunologic deficit
ANSWER:C
130 The cysts in Polycystic Ovarian syndrome are formed by:
A Failure of atretic follicles to undergo apoptosis
B Oocyte proliferation
C Multiple corpus lutea
D Cystic degeneration of ovarian cortex
ANSWER:A
131 An ‘ in situ ‘ stage has not been officially recognized in which of the following :
A Ovarian carcinoma
B Endometrial carcinoma
C Cervical carcinoma
D Vulvar carcinoma
ANSWER:A
132 The gastrointestinal primary of a Krukenberg tumour of the ovary is most oftenfound in the :
A Gall bladder
B Rectum
C Pylorus
D Colon
ANSWER:C
133 Functional ovarian cysts include:
A Follicular cysts.
B Endometriomas.
C Dermoid cysts.
D fibromas.
ANSWER:A
134 In contrast to a malignant ovarian tumor, a benign tumor has which of the following gross features?
A Excrescences on the surface.
B Peritoneal implants.
C Intra-cystic papillations.

D Free mobility.
ANSWER:D
135
A 54-year-old woman is found to have endometrial hyperplasia on endometrialbiopsy. A functional ovarian
tumor to be suspected is a:
A Lipid cell tumor.
B Granulosa-theca cell tumor.
C Sertoli-Leydig yumor.
D Muncious cystadenocarcinoma.
ANSWER:B
136
A uni-locular ovarian cyst measuring 4.4 cm by 4.9 cm found on routine ultrasonograrhy during the 8th week of
gestation . best management for this case is
A observation and repeated ultrasonography
B laparoscoic aspiration of the cyst
C immediate laparotomy and cystectomy
D immediate laparotomy and ovariectomy
ANSWER:A
137 Germ cell tumours include all the following except
A choriocarcinoma
B gonadoblastoma
C endodermal sinus tumour
D begnin cystic teratoma
ANSWER:B
138 Which is the major cause of cancer death in women?
A Breast cancer
B Cervical cancer
C Endometrial cancer
D Lung cancer
ANSWER:A
139 Endometrial carcinomas associated with estrogen therapy “ caused by unopposedestrogen therapy “ :
A well differentiated
B are deeply invasive
C are sensitive to progesterone therapy
D generally have poor prognosis

ANSWER:C
140 Ovarian cancer:
A Separate FIGO staging systems exist for epithelial and sex-cord/stromal ovarian tumors
B Granulosa Cell Tumor has an excellent prognosis because most patients present with early-stage disease
C Meigs’ syndrome consists of ascites; hydrothorax and a malignant ovarian tumor
D Krukenberg tumours are metastatic ovarian neoplasms originating exclusively in the stomach
ANSWER:B
141 Regarding ovarian tumours
A adenocarcinoma is more commonly bilateral than mucinous
B the use of oral contraceptives is a risk factor for ovarian cancer
C Sertoli-Leydig tumours of the ovary are typically estrogen secreting
D Fat saturation MRI images are of value in diagnosing cystic teratomas
ANSWER:A
142 A Krukenberg tumour is an ovarian neoplasm which :
A Is primary in the ovary
B Is associated with hydrothorax
C Is secondary to any GIT cancer
D Shows characteristic mucoid epithelial change
ANSWER:D
143 CA-125 is ?
A A mucin-type glycoprtein
B A ganglioside
C A tumor-specific transplantation antigen
D Useful for ovarian cancer screening in the general patient population
ANSWER:A
144
A young female came to you with complaint of oligomenorrhea ,hirsutism & weightgain ,US reveals bulky
ovaries with subcapsular cysts. Most likely diagnosis is
A ovarian cancer
B cushing syndrome
C PCOD
D DM
ANSWER:C

145
A large cystic tumour is detected in a woman in routine antenatal examination.The most common complication she
can encounter?
A Torsion
B rupture
C hemorrhage
D infection
ANSWER:A
146
A 18-year-old woman comes to the physician for an annual examination. She has no complaints. She has been
sexually active for the past 2 years. She uses the oral contraceptive pill for contraception. She has depression for
which she takes fluoxetine. She takes no other medications and has no allergies to medications. Her family history
is negative for cancer and cardiac disease. Examination is unremarkable. Which of the following screening tests
should this patient most likely have?
A Colonoscopy
B Mammogram
C Pap smear
D Pelvic ultrasound
ANSWER:C
147 Hilus or Leydig cell tumour may be associated with :
A Reinke crystals
B Oestrogen effect on endometrium
C Clinical virilism
D All of the above
ANSWER:D
148
A multiparous woman aged 40 years, presents with menorahagia and progressively increasing dysmenorrhoea.
Most probable diagnosis is:
A Ca Cervix
B Ca Endometrium
C Adenomyosis
D DUB
ANSWER:C
149 Considering dysgerminoma all true except :
A the commonest germ cell tumor
B usually in young females

C lymphatic spread is so late
D elevate lactic dehydrogenase level .
ANSWER:C
150 Ordering accord to the commonest cancers in female genital tract the right is :
A cervical , endometrial ,ovarian
B ovarian , cervical , endometrial
C endometrial , cervical , ovarian
D endometrial , ovarian , cervical .
ANSWER:C
151 Female came to the ER with Bp 80/60 and pulse 125 with history of acute abdomen , next step is
A laparotomy
B iv fluids
C CBC
D PV examination
ANSWER:B
152 Considering ovarian cancer :
A surgery is preferred to be last line
B early discovered with good prognosis
C chemotherapy is good in most tumors
D germ cell tumors show good response to chemotherapy
ANSWER:D
153
A 48-year-old woman has noted a small amount of irregular vaginal bleeding for the past 2 months. She has a
pelvic examination that reveals no cervical lesions, and a Pap smear that shows no abnormal cells. Next, an
endometrial biopsy is performed, and there is microscopic evidence for endometrial hyperplasia. An abdominal
ultrasound reveals a solid right ovarian mass. Which of the following neoplasms is this woman is most likely to
have?
A Mature cystic teratoma
B Choriocarcinoma
C Sertoli-Leydig cell tumor
D Fibrothecoma
ANSWER:D
154 Vaginal adenocarcinomas in children is caused by
A Virus

B Administration of DES to pregnant mothers
C Hormonal changes
D All of the above
ANSWER:B
155 Carcinoma cervix with involvement of upper 2/3 of vagina is stage
A II
B II B
C III A
D III B
ANSWER:A
156
A 47-year-old woman has noted a pressure sensation, but no pain, in her pelvic region for the past 5 months. On
physical examination there is a right adnexal mass. An ultrasound scan shows a 10 cm fluid-filled cystic mass in
the right ovary. A fine needle aspirate of the mass is performed and cytologic examination of clear fluid aspirated
from the mass reveals clusters of malignant epithelial cells surrounding psammoma bodies. Which of the following
neoplasms is she most likely to have?
A Endometrial adenocarcinoma
B Ovarian serous cystadenocarcinoma
C Mesothelioma
D Ovarian mature cystic teratoma
ANSWER:B
157 Ovarian masses:
A Are malignant in presence of ascites
B Include benign teratomas
C Of germ cell origin may secrete hormones
D May be confused with develpomental abnormalities of renal tract
ANSWER:B
158
A 4-year-old girl is noted to have breast enlargement and vaginal bleeding. On physical examination, she is noted
to have a 9-cm pelvic mass. Which of the following is the most likely etiology?
A Cystic teratoma
B Dysgerminoma
C Endodermal sinus tumor
D Granulosa cell tumor
ANSWER:D

159 140. Before puberty, what is the ratio between the cervical length and uterine body ?
A 1 : 2
B 2 : 1
C 1 : 3
D 1 : 4
ANSWER:B
160 141. As regard mastalgia:
A in cyclic mastalgia pain is usaully max. postmenestrual
B is treaeted surgically
C bromocriptine may be used
D gammaleinoliec acid is contraindicated
ANSWER:C
161 Pap smear
A the next step in dysplastic smear is colposcopy
B is simple but inaccurate
C should be carried out every 5 years
D has no role in screening of assymptomatic women
ANSWER:A
162
A 40-years-old female with history of fibroid on investigation showed CIN-2 changes. Treatment of choice in this
case is :
A Hysterectomy
B Conization
C Cryotherapy
D Laser ablation
ANSWER:A
163 Dysfunctional Uterine Bleeding (DUB) is defined as abnormal uterine bleeding ?
A Secondary to hormonal dysfunction
B Caused by cancer
C In a patient with von Willebrand’s disease
D With no organic cause
ANSWER:D
164 Abnormal Uterine bleeding (AUB) is defined by all of the following except ?
A Excessive Blood loss (>80 ml) during menses

B Menstrual length less than 7 days
C An interval of less than 21 days between the starts of successive menses
D Irregular bleeding episodes between menses
ANSWER:B
165 Post menopausal bleeding does not occur in….
A Use of combined OCP’s
B Atrophic vaginitis
C Endometrial hyperplasia
D CA-Endometrium
ANSWER:A
166 Bicornute uterus may predispose to all the following except:
A recurrent PTL
B primary amenorrhea
C retention of placenta after delivery
D menorrhagia
ANSWER:B
167
A couple presented in OPD with H/0 infertility since last 2 years. Husbands semen analysis was advised. What is
WHO criterion – for minimum sperm count in normal semen?
A 10 million.
B 20 million.
C 30 million.
D 40 million.
ANSWER:B
168
A 23 years old primigravida presents with abdominal pain, syncope and vaginal spotting. Assessment reveals that
she has an ectopic pregnancy. The most common site of pregnancy is:
A Ampull
B Isthmus.
C Fimbrial end.
D Abdomin.
ANSWER:A
169 Mean age for menopause is:
A 40 years.
B 45 years.

C 51 years.
D 48 years.
ANSWER:C
170 Second degree uterovaginal prolapse is characterized by:
A Complete protrusion of uterus outside introitus.
B Descent of genital tract within vagina.
C Descent of genital tract upto introitus.
D Descent of genital tract outside the introitus.
ANSWER:D
171
A 63 years old lady presents with abdominal mass and weight loss, was diagnosed as having an ovarian tumour.
The most common ovarian tumour in this woman would be:
A Epithelial tumour.
B Germ cell tumour.
C Stromal tumour.
D Sex cord tumour.
ANSWER:A
172
A young girl, 23 years old is presented with complaint of abdominal pain, menorrhagia and 18 weeks size mass
arising from hypogastrium. The most likely diagnosis is:
A Endometriosis.
B Pelvic inflammatory disease.
C Ovarian cyst.
D Fibroid uterus.
ANSWER:D
173
A 25 years old school teacher Para 1 wants to use oral contraceptive pills for contraception. She is asking about the
mode of action of oral contraceptive pills. The mechanism of action of oral contraceptive pills is:
A Inhibiting ovulation by suppression of serum FSH.
B Inducing endometrial atrophy.
C Increasing cervical mucous hostility.
D Inducing endometritis.
ANSWER:A
174 Women complaining of milky whitish discharge with fishy odour. No history of itching. Most likely diagnosis is:
A Bacterial vaginosis.

B Trichomoniasis.
C Candidiasis.
D Malignancy.
ANSWER:A
175
A young medical student has come to you with complaints of oligomenorrrhea, hirsuitism and weight gain,
ultrasound reveals bulky ovaries with sub-capsular cysts. Most likely diagnosis is:
A Ovarian cancer.
B Cushing’s syndrome.
C Polycystic ovarian disease.
D Diabetes mellitus.
ANSWER:C
176
A 43 year old, lecturer has come to you with complaints of heavy but regular menstrual bleeding with flooding
and clots. There is no anatomical reason for heavy flow. The most effective remedy for reducing her menstrual
flow is:
A Tranexemic acid.
B Dilatation and Curettage.
C Depomedroxy progesterone acetate.
D Misoprostol.
ANSWER:A
177
A 39 years old women Para 6 has presented with complaint of post coital bleeding for the past three months. Your
first investigation should be:
A Dilatation & Curettage.
B Cone biopsy of cervix.
C Pap smear.
D Colposcopy.
ANSWER:C
178
A 28 years old woman has 14 weeks size irregular uterus. She does not complain of abdominal pain or
menorrhagia. Her pap smear is normal. The best next step in her management would be:
A Continued observation.
B Endometrial biopsy.
C Hysterectomy.
D Pelvic ultrasonography.
ANSWER:D

179 The most common cause of rectovaginal fistula is:
A Obstetrical.
B Irradiation of the pelvis.
C Carcinoma.
D Crohn’s disease.
ANSWER:A
180
A 40 years old multiparous woman complains of involuntary loss of urine associated with coughing, laughing,
lifting or standing. The history is most suggestive of:
A Fistul
B Stress incontinence.
C Urge incontinence.
D Urethral diverticulum.
ANSWER:B
181
A 28 years old G3 P2 has presented with complaints of brownish vaginal discharge, passage of vesicles and
excessive vomiting. Ultrasound scan shows snowstorm appearance in uterus with no fetus. The most likely
diagnosis is:
A Septic induced abortion.
B Twin pregnancy.
C Gestational trophoblastic disease.
D Ectopic pregnancy.
ANSWER:C
182 The maximum number of oogonia are formed at what ageof female life:
A One month intrauterine.
B Five month intrauterine.
C At birth.
D At puberty.
ANSWER:B
183 Menarche usually occurs at age of:
A 8 and 10 years.
B 11 and 13 years.
C 14 and 16 years.
D 17 and 18 years.
ANSWER:B

184 The most common cause of vesicovaginal fistula (VVF) in under developed countries would be:
A Obstetrical injuries.
B Pelvic irradiation.
C Carcinoma .
D Haemorrhoidectomy.
ANSWER:A
185 A 28 year old patient complains of amenorrhea after having dilatation and curettage. The most likely diagnosis is:
A Kallman’s Syndrome.
B Turner’s Syndrome.
C Asherman’s Syndrome.
D Pelvic inflammatory disease.
ANSWER:C
186
A large cystic ovarian tumour is detected in a woman on routine antenatal check up. The most common
complication she can encounter is:
A Torsion.
B Rupture.
C Haemorrhage.
D Degeneration.
ANSWER:A
187 Which of the following is used to take cervical smear:
A Colposcope.
B Vaginoscope.
C Ayre’s spatula.
D Laparoscope.
ANSWER:C
188 Normal duration of menstrual cycle is:
A 1-3 days.
B 1-4 days.
C 2-7 days.
D 7-10 days.
ANSWER:C

189
a 20 year old medical student presents with five years history of weight gain, irregular periods and worsening
fascial hair. What is the most likely diagnosis?
A Polycystic ovarian disease.
B Hypothyroidism.
C Obesity.
D Cushing’s Syndrome.
ANSWER:A
190
28 years old woman with previous history of having baby with Down’s Syndrome is now 12 weeks pregnant.
Which of the following would you suggest to her:
A Amniocentesis.
B Obstetric ultrasound.
C Chorionic villus sampling.
D Fetal blood sampling.
ANSWER:C
191
A newly married girl comes to gynae OPD with history of dysuria, burning, micturition and sore perineum. What
is your likely diagnosis:
A Trichomonas vaginalis.
B Candida infection.
C Trauma due to coitus.
D Honey moon cystitis.
ANSWER:D
192 Gonadotropin releasing hormone (GnRH) stimulates the release of:
A ACTH.
B Growth hormone.
C Leutinising Hormone (LH).
D Thyroid stimulating hormone (TSH).
ANSWER:C
193 Serum prolactin levels are highest in which of the following conditions:
A Menopause.
B Ovulation.
C Parturition.
D Sleep.
ANSWER:C

194 Main uterine support is:
A Uterosacral ligaments.
B Round ligaments.
C Transverse cervical ligaments.
D Ovarian ligaments.
ANSWER:C
195 The most likely cause of abnormal uterine bleeding in 13years old girl is:
A Uterine cancer.
B Ectopic pregnancy.
C Anovulation.
D Systemic bleeding diatheses.
ANSWER:C
196 Which of the following pubertal event would occur even in the absence of ovarian estrogen production:
A Thelarche.
B Menarche.
C Pubarche.
D Skeletal growth.
ANSWER:C
197
58 years old woman has presented with complaints of postmenopausal bleeding for the past two weeks. The most
essential investigation would be:
A Colposcopy.
B Pap smear.
C Cone biopsy.
D D & C (dilatation & Curettage).
ANSWER:D
198 The most common symptom of endometrial hyperplasia is:
A Vaginal discharge.
B Vaginal bleeding.
C Amenorrhea.
D Pelvic pain.
ANSWER:B
199
56 years old woman has come to you with the complaints of hot flushes irritability, joint pains with lack of sleep.
Most appropriate treatment would be:

A Hysterectomy.
B Vitamins.
C Combined oestrogen, progesterone preparations.
D Phytooestrogens.
ANSWER:C
200 Which of the following is used as an emergency contraceptive:
A Combined oral contraceptive pills.
B Progesterone only.
C Depoprovera.
D Levonorgestril (EM-Kit).
ANSWER:D
201
A 48-year-old woman presents with intermenstrual bleeding for two months and episodes of bleeding occurring
any time in the cycle. There is no associated pain. Differential diagnosis for intermenstrual bleeding does not
include:
A endocervical polyp
B cervical malignancy
C endometrial polyp
D ovarian teratoma
ANSWER:D
202 All of the following drugs are associated with hyperprolactinaemia, apart from:
A reserpine
B progesterone-only contraceptive pill
C methyldopa
D ranitidine
ANSWER:B
203 All of the following are effects of premature menopause, apart from:
A decreased cardiovascular risk
B infertility
C osteoporosis
D vasomotor symptoms
ANSWER:A
204
A 32-year-old woman presents to the gynaecology clinic with infrequent periods. A hormone profi le is done and
all of the following are consistent with polycystic ovarian syndrome, apart from:

A increased androgen levels
B normal FSH
C normal oestradiol
D decreased LH
ANSWER:D
205
A 28-year-old woman attends the colposcopy clinic after an abnormal smear test. The smear is reported as severe
dyskaryosis and she has an intrauterine contraceptive device in situ. All of the following statements are likely to be
true, apart from:
A the cervix is macroscopically normal
B acetic acid is applied and an irregular white area is apparent to the left of the cervical os
C Lugol’s iodine is applied and the same area stains dark brown while the rest of the cervix stains pale
D a biopsy is taken
ANSWER:C
206
A 24-year-old woman presents with the absence of periods for nine months. She started her periods at the age of
13 years and had a regular 28-day cycle until 18 months ago. The periods then became irregular, occurring every
two to three months until they stopped completely. The following are all included in the differential diagnosis of
secondary amenorrhoea, apart from:
A excessive exercise
B hyperprolactinaemia
C hyperthyroidism
D premature ovarian failure
ANSWER:C
207 The following statements regarding adenomyosis are true, apart from one.
A It tends to occur in women over 35 years.
B Risk factors include increased parity, termination and quick labours.
C The condition commonly occurs in association with endometriosis.
D With each period, bleeding occurs from the endometrial tissue into the smooth muscle.
ANSWER:B
208
A 20-year-old woman is referred with a problem of post-coital bleeding. Over the past two months it has occurred
on six occasions and there has been a small amount of bright red blood noticed after intercourse. There is no
associated pain. The following investigations should initially be performed, apart from:
A cervical smear
B endocervical swab for chlamydia

C colposcopy
D endocervical swab for gonorrhoea
ANSWER:C
209 The following are all consistent with the diagnosis of antiphospholipid syndrome except:
A a hydatidiform mole
B severe early-onset pre-eclampsia
C c arterial or venous thrombosis
D mid-trimester fetal loss
ANSWER:A
210 The following are all causes of recurrent miscarriage, apart from:
A parental chromosomal abnormality
B activated protein C-resistance
C uncontrolled hypothyroidism
D chlamydia infection
ANSWER:D
211 Which of the following statements concerning the anterior pituitary is true?
A It develops in the embryo from a down-growth of the hypothalamus.
B It secretes antidiuretic hormone (ADH).
C It is regulated by hypothalamic-releasing hormones.
D It secretes its hormones into the pituitary portal system.
ANSWER:C
212 Which one of the following statements about the implantation of the human embryo is true?
A It will occur at any time over a period of about 14 days.
B It will occur whether or not the zona pellucida is present.
C It will occur when the cytotrophoblast contacts the endometrial epithelium and begins to invade the maternal tissue.
D It will occur with the inner cell mass closest to the endometrium.
ANSWER:D
213 Which one of the following statements about puberty is true?
A Puberty is preceded by falling plasma levels of adrenal androgens.
B The fi rst menstrual period is called the adrenarche.
C The pubertal growth spurt is the fi rst sign of puberty.
D Pubic hair growth is stimulated in girls by oestrogen.

ANSWER:A
214 Which is the most appropriate statement concerning pulmonary embolism?
A It is now rarely fatal, with the introduction of modern diagnostic tests and treatments.
B It gives an area of lung which is unventilated on a ventilationperfusion scan.
C It does not usually show up on a CT pulmonary angiogram.
D It may give symptoms similar to pneumonia.
ANSWER:D
215
One of the following is true. It is recognised that the positive predictive value of initial mammography for breast
cancer within the national screening programme in the UK is 16%. This means that:
A 16% of people who have breast cancer are detected on initial mammography
B 84% of people without breast cancer have a normal mammogram
C 16% of initial mammograms are abnormal
D a patient with an abnormal initial mammogram has a 16% chance of having breast cancer
ANSWER:D
216 One of the following is true. Successful fertilisation and subsequent normal embryonic development:
A require at least two spermatozoa
B require the retention of the cortical granules in the oocyte
C are most likely when the oocytes have been ovulated in an immature stage
D require exclusion of the second polar body
ANSWER:D
217 One of the following is true. The increase in maternal blood volume in pregnancy occurs as a result of:
A peripheral vasoconstriction
B a reduction in progesterone
C decreased synthesis of vasopressin
D increased aldosterone synthesis
ANSWER:D
218
One of the following is true. Decreased peripheral resistance in pregnancy has been attributed to an increase in
synthesis of:
A angiotensin
B endothelin
C nitric oxide
D renin
ANSWER:C

219
A 25-year-old woman on liver enzyme inducers is requesting contraceptive advice. The method providing her
with the most reliable form of contraception would be:
A combined oral contraceptive pill
B Depo-Provera injection
C diaphragm
D male condom
ANSWER:B
220
A 35-year-old woman comes requesting long-term reversible contraception. You advise that the method that can
provide the longest protection is:
A contraceptive implant
B copper intrauterine device
C Depo-Provera injection
D intrauterine hormonal system (IUS)
ANSWER:B
221 Regarding cervical cancer, which is the true statement?
A HPV types 6 and 12 are high risk for developing cervical cancer.
B The new vaccines can prevent invasive carcinoma but not CIN.
C As soon as the new vaccination is introduced, cervical screening programmes can cease.
D HPV types 16 and 18 account for the majority of cervical cancer in the UK.
ANSWER:D
222 Regarding the menstrual cycle, which is the true statement?
A Menstruation occurs with vasodilation of the spiral arteries.
B The LH surge triggers menstruation.
C The Graafi an follicle develops during the luteal phase.
D Both the follicle and the corpus luteum secrete oestradiol.
ANSWER:D
223 Regarding Müllerian duct abnormalities which is the true statement?
A occur about 1 in 500
B the commonest uterine abnormality is septate uterus
C occur not infrequently with gastrointestinal abnormalities
D surgical correction of a septate uterus is followed by fetal salvage in <60% of cases
ANSWER:B
224 Choose the correct statement: Uterine leiomyosarcomas:

A are associated with exposure to tamoxifen
B originate from leiomyomas
C pelvic radiotherapy has a signifi cant impact on survival
D commonly metastasise to the brain
ANSWER:A
225 Which one of the following statements about the menopause is correct?
A Progesterone levels rise after the menopause.
B LH levels rise after the menopause.
C The pituitary stops secreting LH and FSH at the menopause.
D Menstrual cycles remain regular until the last menstrual perio
ANSWER:B
226 Choose the correct statement: The female reproductive tract plays important roles in sperm transport by:
A trapping most spermatozoa in the cervical crypt for many days
B regulating sperm transport so that cells reach the site of fertilisation around the time of ovulation
C allowing sperm transport at all stages of the ovarian cycle
D preventing spermatozoa from swimming out of the peritoneal cavity
ANSWER:B
227 Which one of the following statements is true: Semen analysis:
A identifi es men with high-quality fertile spermatozoa
B identifi es men with low sperm concentrations that might affect fertility
C can always be used to predict fertility
D cannot identify abnormal spermatozoa
ANSWER:B
228 One of the following is true. A malignant tumour arising in the mesenchymal tissue is called:
A adenoma
B carcinoma
C lymphoma
D sarcoma.
ANSWER:D
229 One of the following is true. Affording moral status to a human embryo/fetus means that it now has:
A an inalienable right to life
B a right to life
C a right to consideration

D a right dependent on moral consensus
ANSWER:D
230 At term amniotic fluid volume is
A 800ml
B 500 ml
C 400 ml
D 600 ml
ANSWER:A
231 what is the principal carbohydrate present in Amniotic fluid ?
A Glucose
B Fructose
C Mannose
D Galactose
ANSWER:A
232 Oligohydramnios is related which of the following condition ?
A Renal Agenesis
B esophageal atresia
C anencephaly
D down's syndrome
ANSWER:A
233 Early amniocentesis is done in which period of pregnancy
A 12-14 wks
B 14-16 wks
C 16-18 wks
D 9-11 wks
ANSWER:B
234 Immune rejection of fetus prevented by
A HCG
B HPL
C oestrogen
D progesterone
ANSWER:A
235 what happens to GFR in a case of Pre-eclampsia ?

A GFR Decreases
B GFR increases
C remains same
D None of the above
ANSWER:A
236 Shortest diameter of pelvic Cavity
A Interspinous
B transverse
C antero-posterior
D oblique
ANSWER:A
237 Large Chorioangioma associated with
A polyhydroamnios
B oligohydramnios
C All of the above
D None of the above
ANSWER:A
238 Commonest presentation of Choriocarcinoma
A vaginal bleeding
B abdominal pain
C breathlessness
D perforation of the uterus
ANSWER:A
239 Frog eye appearance is seen in
A Anencephaly
B acardia
C down's syndrome
D patau's syndrome
ANSWER:A
240 what of the following is seen in Partial mole
A Triploidy
B haploidy
C polyploidy

D diploidy
ANSWER:A
241 Cervical changes in pregnancy are all except ?
A increased collagen
B increased Hyaluronic acid
C Increased glands
D increased vascularity
ANSWER:b
242
A 20-year-old female presents to your office for routine well-woman examination. She has a history of acne, for
which she takes minocycline and isotretinoin on a daily basis. She also has a history of epilepsy that is well
controlled on valproic aciD. She also takes a combined oral contracep- tive birth control pill containing
norethindrone acetate and ethinyl estra- diol. She is a nonsmoker but drinks alcohol on a daily basis. She is
concerned about the effectiveness of her birth control pill, given all the medications that she takes. She is
particularly worried about the effects of her medications on a developing fetus in the event of an unintended preg-
nancy. Which of the following drugs has the lowest potential to cause birth defects?
A Alcohol
B Isotretinoin A.ccutane)
C Tetracyclines
D Progesterone
ANSWER:D
243
A patient presents for prenatal care in the second trimester. She was born outside the United States and has never
had any routine vaccinations. Which of the following vaccines is contraindicated in pregnancy?
A Hepatitis A
B Tetanus
C Typhoid
D Measles
ANSWER:D
244
Your 25-year-old patient is pregnant at 36 weeks gestation. She has an acute urinary tract infection (UTI). Which
of the following medications is contraindicated in the treatment of the UTI in this patient?
A Ampicillin
B Nitrofurantoin
C Trimethoprim/sulfamethoxazole
D Cephalexin

ANSWER:C
245
You diagnose a 21-year-old woman at 12 weeks gestation with gonorrhea cervicitis. Which of the following is the
most appropriate treatment for her infection?
A Doxycycline
B Chloramphenicol
C Tetracycline
D Ceftriaxone
ANSWER:D
246
A 36-year-old G0 who has been epileptic for many years is contem- plating pregnancy. She wants to go off her
phenytoin because she is con- cerned about the adverse effects that this medication may have on her unborn fetus.
She has not had a seizure in the past 5 years. Which of the following is the most appropriate statement to make to
the patient?
A
Babies born to epileptic mothers have an increased risk of structural anomalies even in the absence of
anticonvulsant medications.
B
She should see her neurologist to change from phenytoin to valproic acid because valproic acid is not associated
with fetal anomalies.
C She should discontinue her phenytoin because it is associated with a 1% to 2 % risk of spina bifidA.
D
Vitamin C supplementation reduces the risk of congenital anomalies in fetuses of epileptic women taking
anticonvulsants.
ANSWER:A
247
At 1 year of age, a child has six deciduous teeth, which are discolored and have hypoplasia of the enamel. Match
the appropriate scenario with the antibiotic most likely responsible for the clinical findings presenteD.
A Tetracycline
B Streptomycin
C Nitrofurantoin
D Chloramphenicol
ANSWER:A
248
During routine auditory testing of a 2-day-old baby, the baby failed to respond to high-pitched tones. Match the
appropriate scenario with the antibiotic most likely responsible for the clinical findings presenteD.
A Tetracycline
B Streptomycin
C Nitrofurantoin
D Chloramphenicol

ANSWER:B
249
A 24-year-old primigravida with twins presents for routine ultra- sonography at 20 weeks gestation. Based on the
ultrasound findings, the patient is diagnosed with dizygotic twins. Which of the following is true regarding the
membranes and placentas of dizygotic twins?
A They are dichorionic and monoamniotic only if the fetuses are of the same sex.
B They are dichorionic and monoamniotic regardless of the sex of the fetuses.
C They are monochorionic and monoamniotic if they are conjoined twins.
D They are dichorionic and diamniotic regardless of the sex of the twins.
ANSWER:D
250
After delivery of a term infant with Apgar scores of 2 at 1 minute and 7 at 5 minutes, you ask that umbilical cord
blood be collected for pH. The umbilical arteries carry which of the following?
A Oxygenated blood to the placenta
B Oxygenated blood from the placenta
C Deoxygenated blood to the placenta
D Deoxygenated blood from the placenta
ANSWER:C
251
During the routine examination of the umbilical cord and placenta after a spontaneous vaginal delivery, you
notice that the baby had only one umbilical artery. Which of the following is true regarding the finding of a single
umbilical artery?
A It is a very common finding and is insignificant.
B It is a rare finding in singleton pregnancies and is therefore not significant.
C It is an indicator of an increased incidence of congenital anomalies of the fetus.
D It is equally common in newborns of diabetic and nondiabetic mothers.
ANSWER:C
252
A 22-year-old G1P0 at 28 weeks gestation by LMP presents to labor and delivery complaining of decreased fetal
movement. She has had no pre- natal carE. On the fetal monitor there are no contractions. The fetal heart rate is
150 beats per minute and reactivE. There are no decelerations in the fetal heart tracing. An ultrasound is
performed in the radiology department and shows a 28-week fetus with normal-appearing anatomy and size con-
sistent with dates. The placenta is implanted on the posterior uterine wall and its margin is well away from the
cervix. A succenturiate lobe of the pla- centa is seen implanted low on the anterior wall of the uterus. Doppler flow
studies indicate a blood vessel is traversing the cervix connecting the two lobes. This patient is most at risk for
which of the following?
A Premature rupture of the membranes

B Fetal exsanguination after rupture of the membranes
C Torsion of the umbilical cord caused by velamentous insertion of the umbilical cord
D Amniotic fluid embolism
ANSWER:B
253
healthy 34-year-old G1P0 patient comes to see you in your office for a routine OB visit at 12 weeks gestational
agE. She tells you that she has stopped taking her prenatal vitamins with iron supplements because they make her
sick and she has trouble remembering to take a pill every day. A review of her prenatal labs reveals that her
hematocrit is 39%. Which of the following statements is the best way to counsel this patient?
A
Tell the patient that she does not need to take her iron supplements because her prenatal labs indicate that she is not
anemic and therefore she will not absorb the iron supplied in prenatal vitamins
B Tell the patient that if she consumes a diet rich in iron, she does not need to take any iron supplements
C Tell the patient that if she fails to take her iron supplements, her fetus will be anemic
D
Tell the patient that she needs to take the iron supplements even though she is not anemic in order to meet the
demands of pregnancy
ANSWER:D
254
A pregnant patient of yours goes to the emergency room at 20 weeks gestational age with complaints of hematuria
and back pain. The emer- gency room physician orders an intravenous pyelogram (IVP) as part of a workup for a
possible kidney stonE. The radiologist indicates the absence of nephrolithiasis but reports the presence of bilateral
hydronephrosis and hydroureter, which is greater on the right side than on the left. Which of the following
statements is true regarding this IVP finding?
A
The bilateral hydronephrosis is of concern, and renal function tests, including BUN and creatinine, should be run
and closely monitoreD.
B These findings are consistent with normal pregnancy and are not of concern.
C The bilateral hydronephrosis is of concern, and a renal sonogram should be ordered emergently.
D
The findings indicate that a urology consult is needed to obtain recommendations for further workup and
evaluation.
ANSWER:B
255
During a routine return OB visit, an 18-year-old G1P0 patient at 23 weeks gestational age undergoes a urinalysis.
The dipstick done by the nurse indicates the presence of trace glucosuriA. All other parameters of the urine test are
normal. Which of the following is the most likely etiology of the increased sugar detected in the urine?
A The patient has diabetes.
B The patient has a urine infection.

C The patient’s urinalysis is consistent with normal pregnancy.
D The patient’s urine sample is contaminate
ANSWER:C
256
A 33-year-old G2P1 is undergoing an elective repeat cesarean section at term. The infant is delivered without any
difficulties, but the placenta cannot be removed easily because a clear plane between the placenta and uterine wall
cannot be identifieD. The placenta is removed in pieces. This is followed by uterine atony and hemorrhagE. Match
the descriptions with the appropriate placenta typE.
A Succenturiate placenta
B Vasa previa
C Placenta previa
D Placenta accrete
ANSWER:D
257 The shortest distance between the sacral promontory and the symphysis pubis is called which of the following?
A Interspinous diameter
B True conjugate
C Diagonal conjugate
D Obstetric (OB) conjugate
ANSWER:D
258
A patient presents in labor at term. Clinical pelvimetry is performeD. She has an oval-shaped pelvis with the
anteroposterior diameter at the pelvic inlet greater than the transverse diameter. The baby is occiput posterior. The
patient most likely has what kind of pelvis?
A A gynecoid pelvis
B An android pelvis
C An anthropoid pelvis
D A platypelloid pelvis
ANSWER:C
259
On pelvic examination of a patient in labor at 34 weeks, the patient is noted to be 6 cm dilated, completely effaced
with the fetal nose and mouth palpablE. The chin is pointing toward thematernal left hip. This is an example of
which of the following?
A Transverse lie
B Mentum transverse position
C Occiput transverse position
D Brow presentation

ANSWER:B
260
A patient comes to your office with her last menstrual period 4 weeks ago. She denies any symptoms such as
nausea, fatigue, urinary frequency, or breast tenderness. She thinks that she may be pregnant because she has not
had her period yet. She is very anxious to find out because she has a history of a previous ectopic pregnancy and
wants to be sure to get early prenatal carE. Which of the following actions is most appropriate at this time?
A No action is needed because the patient is asymptomatic, has not missed her period, and cannot be pregnant.
B Order a serum quantitative pregnancy test.
C Listen for fetal heart tones by Doppler equipment.
D Perform an abdominal ultrasoun
ANSWER:A
261
A patient presents for her first initial OB visit after performing a home pregnancy test and gives a last menstrual
period of about 8 weeks ago. She says she is not entirely sure of her dates, however, because she has a long history
of irregular menses. Which of the following is the most accurate way of dating the pregnancy?
A Determination of uterine size on pelvic examination
B Quantitative serum human chorionic gonadotropin (HCG) level
C Crown-rump length on abdominal or vaginal ultrasound
D Determination of progesterone level along with serum HCG level
ANSWER:C
262
A healthy 31-year-old G3P2002 patient presents to the obstetrician’s office at 34 weeks gestational age for a
routine return visit. She has had an uneventful pregnancy to datE. Her baseline blood pressures were 100 to
110/60 to70, and she has gained a total of 20 lb so far. During the visit, the patient complains of bilateral pedal
edema that sometimes causes her feet to ache at the end of the day. Her urine dip indicates trace protein, and her
blood pressure in the office is currently 115/75. She denies any other symptoms or complaints. On physical
examination, there is pitting edema of both legs without any calf tenderness. Which of the following is the most
appropriate response to the patient’s concern?
A Prescribe Lasix to relieve the painful swelling.
B
Immediately send the patient to the radiology department to have venous.Doppler studies done to rule out deep
vein thromboses.
C Admit the patient to L and D to rule out preeclampsiA.
D Reassure the patient that this is a normal finding of pregnancy and no treatment is neede
ANSWER:D

263
A 28-year-old G1P0 presents to your office at 18 weeks gestational age for an unscheduled visit secondary to
right-sided groin pain. She describes the pain as sharp and occurring with movement and exercisE. She denies any
change in urinary or bowel habits. She also denies any fever or chills. The application of a heating pad helps
alleviate the discomfort. As her obstetrician, what should you tell this patient is the most likely etiology of this
pain?
A Round ligament pain
B Appendicitis
C Preterm labor
D Kidney stone
ANSWER:A
264
A 19-year-old G1P0 presents to her obstetrician’s office for a routine OB visit at 32 weeks gestation. Her
pregnancy has been complicated by gestational diabetes requiring insulin for control. She has been noncompli- ant
with diet and insulin therapy. She has had two prior normal ultra- sounds at 20 and 28 weeks gestation. She has no
other significant past medical or surgical history. During the visit, her fundal height measures 38 cm. Which of the
following is the most likely explanation for the discrepancy between the fundal height and the gestational age?
A Fetal hydrocephaly
B Uterine fibroids
C Polyhydramnios
D Breech presentation
ANSWER:C
265
A 30-year-old G2P1001 patient comes to see you in the office at 37 weeks gestational age for her routine OB visit.
Her first pregnancy resulted in a vagi- nal delivery of a 9-lb 8-oz baby boy after 30 minutes of pushing. On doing
Leopold maneuvers during this office visit, you determine that the fetus is breech. Vaginal examination
demonstrates that the cervix is 50% effaced and 1 to 2 cm dilateD. The presenting breech is high out of the pelvis.
The esti- mated fetal weight is about 7 lB. The patient denies having any contractions. You send the patient for a
sonogram, which confirms a fetus with a double footling breech presentation. There is a normal amount of
amniotic fluid present and the head is hyperextended in the “stargazer” position. Which of the following is the best
next step in the management of this patient?
A Allow the patient to undergo a vaginal breech delivery whenever she goes into labor.
B Send the patient to labor and delivery immediately for an emergent cesarean section.
C Schedule a cesarean section at or after 41 weeks gestational agE.
D Schedule an external cephalic version in the next few days.

ANSWER:D
266
A 29-year-old G1P0 presents to the obstetrician’s office at 41 weeks gestation. On physical examination, her
cervix is 1 centimeter dilated, 0% effaced, firm, and posterior in position. The vertex is presenting at –3 station.
Which of the following is the best next step in the management of this patient?
A Send the patient to the hospital for induction of labor since she has a favorable Bishop scorE.
B
Teach the patient to measure fetal kick counts and deliver her if at any time there are less than 20 perceived fetal
movements in 3 hours.
C Order BPP testing for the same or next day.
D Schedule the patient for induction of labor at 43 weeks gestation.
ANSWER:C
267
Your patient had an ultrasound examination today at 39 weeks gestation for size less than dates.The ultrasound
showed oligohydramnios with an amniotic fluid index of 1.5 centimeters. The patient’s cervix is unfavorablE.
Which of the following is the best next step in the management of this patient?
A Admit her to the hospital for cesarean delivery.
B Admit her to the hospital for cervical ripening then induction of labor.
C Write her a prescription for misoprostol to take at home orally every 4 hours until she goes into labor.
D Perform stripping of the fetal membranes and perform a BPP in 2 days.
ANSWER:B
268
An 18-year-old G2P1001 with the first day of her last menstrual period of May 7 presents for her first OB visit at
10 weeks. What is this patient’s estimated date of delivery?
A February 10 of the next year
B February 14 of the next year
C December 10 of the same year
D December 14 of the same year
ANSWER:B
269
A new patient presents to your office for her first prenatal visit. By her last menstrual period she is 11 weeks
pregnant. This is the first pregnancy for this 36-year-old woman. She has no medical problems. At this visit you
observe that her uterus is palpable midway between the pubic symphysis and the umbilicus. No fetal heart tones
are audible with the Doppler stethoscopE. Which of the following is the best next step in the manage- ment of this
patient?
A Reassure her that fetal heart tones are not yet audible with the Doppler stetho- scope at this gestational agE.
B
Tell her the uterine size is appropriate for her gestational age and schedule her for routine ultrasonography at 20
weeks.

C Schedule genetic amniocentesis right away because of her advanced maternal agE.
D Schedule an ultrasound as soon as possible to determine the gestational age and viability of the fetus.
ANSWER:D
270
A 16-year-old primigravida presents to your office at 35 weeks gesta- tion. Her blood pressure is 170/110 mm Hg
and she has 4+ proteinuria on a clean catch specimen of urinE. She has significant swelling of her face and
extremities. She denies having contractions. Her cervix is closed and unef- faceD. The baby is breech by bedside
ultrasonography. She says the baby’s movements have decreased in the past 24 hours. Which of the following is
the best next step in the management of this patient?
A Send her to labor and delivery for a BPP.
B Send her home with instructions to stay on strict bed rest until her swelling and blood pressure improvE.
C Admit her to the hospital for enforced bed rest and diuretic therapy to improve her swelling and blood pressurE.
D Admit her to the hospital for cesarean delivery.
ANSWER:D
271
A 29-year-old G3P2 presents to the emergency center with com- plaints of abdominal discomfort for 2 weeks. Her
vital signs are: blood pressure 120/70 mm Hg, pulse 90 beats per minute, temperature 36.94°C, respiratory rate 18
breaths per minutE. A pregnancy test is positive and an ultrasound of the abdomen and pelvis reveals a viable 16-
week gestation located behind a normal-appearing 10 × 6 × 5.5 cm uterus. Both ovaries appear normal. No free
fluid is noteD. Which of the following is the most likely cause of these findings?
A Ectopic ovarian tissue
B Fistula between the peritoneum and uterine cavity
C Primary peritoneal implantation of the fertilized ovum
D Tubal abortion
ANSWER:D
272
A 32-year-old G2P1 at 28 weeks gestation presents to labor and delivery with the complaint of vaginal bleeding.
Her vital signs are: blood pressure 115/67 mm Hg, pulse 87 beats per minute, temperature 37.0°C, respiratory rate
18 breaths per minutE. She denies any contraction and states that the baby is moving normally. On ultrasound the
placenta is anteriorly located and completely covers the internal cervical os. Which of the following would most
increase her risk for hysterectomy?
A Desire for sterilization
B Development of disseminated intravascular coagulopathy D.IC)
C Placenta accreta
D Prior vaginal delivery

ANSWER:C
273
A patient at 17 weeks gestation is diagnosed as having an intrauter- ine fetal demisE. She returns to your office 5
weeks later and her vital signs are: blood pressure 110/72 mm Hg, pulse 93 beats per minute, tempera- ture
36.38°C, respiratory rate 16 breaths per minutE. She has not had a miscarriage, although she has had some
occasional spotting. Her cervix is closed on examination. This patient is at increased risk for which of the
following?
A Septic abortion
B Recurrent abortion
C Consumptive coagulopathy with hypofibrinogenemia
D Future infertility
ANSWER:C
274
A 24-year-old presents at 30 weeks with a fundal height of 50 cm. Which of the following statements concerning
polyhydramnios is true?
A Acute polyhydramnios rarely leads to labor prior to 28 weeks.
B The incidence of associated malformations is approximately 3%.
C Maternal edema, especially of the lower extremities and vulva, is rarE.
D Complications include placental abruption, uterine dysfunction, and postpartum hemorrhage
ANSWER:D
275
A 20-year-old G1 at 32 weeks presents for her routine obstetric (OB) visit. She has no medical problems. She is
noted to have a blood pressure of 150/96 mm Hg, and her urine dip shows 1+ protein. She complains of a
constant headache and vision changes that are not relieved with rest or a pain reliever. The patient is sent to the
hospital for further management. At the hospital, her blood pressure is 158/98 mm Hg and she is noted to have
tonic-clonic seizurE. Which of the following is indicated in the manage- ment of this patient?
A Low-dose aspirin
B Dilantin (phenytoin)
C Antihypertensive therapy
D Magnesium sulfate
ANSWER:C
276
A 32-year-old G5P1 presents for her first prenatal visit. A complete obstetrical, gynecological, and medical history
and physical examination is donE. Which of the following would be an indication for elective cerclage placement?
A Three spontaneous first-trimester abortions
B Twin pregnancy

C Three second-trimester pregnancy losses without evidence of labor or abruption
D History of loop electrosurgical excision procedure for cervical dysplasia
ANSWER:C
277
Uterine bleeding at 12 weeks gestation accompanied by cervical dilation without passage of tissuE. Match above
description with the correct type of abortion.
A Complete abortion
B Incomplete abortion
C Threatened abortion
D Inevitable abortion
ANSWER:D
278
Passage of some but not all placental tissue through the cervix at 9 weeks gestation. Match above description with
the correct type of abortion.
A Complete abortion
B Incomplete abortion
C Threatened abortion
D Inevitable abortion
ANSWER:B
279
Fetal death at 15 weeks gestation without expulsion of any fetal or maternal tissue for at least 8 weeks. Match
above description with the correct type of abortion.
A Complete abortion
B Incomplete abortion
C Threatened abortion
D Missed abortion
ANSWER:D
280
Uterine bleeding at 7 weeks gestation without any cervical dilation. Match above description with the correct type
of abortion.
A Complete abortion
B Incomplete abortion
C Threatened abortion
D Missed abortion
ANSWER:C
281
Expulsion of all fetal and placental tissue from the uterine cavity at 10 weeks gestation. Match above description
with the correct type of abortion.

A Complete abortion
B Incomplete abortion
C Threatened abortion
D Missed abortion
ANSWER:A
282
A 20-year-old G1P0 presents to your clinic for follow-up for a suc- tion dilation and curettage for an incomplete
abortion. She is asymptomatic without any vaginal bleeding, fever, or chills. Her examination is normal. The
pathology report reveals trophoblastic proliferation and hydropic degenera- tion with the absence of vasculature;
no fetal tissue is identifieD. A chest x-ray is negative for any evidence of metastatic diseasE. Which of the
following is the best next step in her management?
A Weekly human chorionic gonadotropin (hCG) titers
B Hysterectomy
C Single-agent chemotherapy
D Combination chemotherapy
ANSWER:A
283
A 22-year-old G1P0 presents to your clinic for follow-up of evacuation of a complete hydatidiform molE. She is
asymptomatic and her examination is normal. Which of the following would be an indication to start single-agent
chemotherapy?
A A rise in hCG titers
B A plateau of hCG titers for 1 week
C Return of hCG titer to normal at 6 weeks after evacuation
D Appearance of liver metastasis
ANSWER:A
284
A 32-year-old female presents to the emergency department with abdominal pain and vaginal bleeding. Her last
menstrual period was 8 weeks ago and her pregnancy test is positivE. On examination she is tachycardic and
hypotensive and her abdominal examination findings reveal peritoneal signs, a bedside abdominal ultrasound
shows free fluid within the abdominal cavity. The decision is made to take the patient to the operating room for
emergency exploratory laparotomy. Which of the following is the most likely diagnosis?
A Ruptured ectopic pregnancy
B Hydatidiform mole
C Incomplete abortion D. Missed abortion
D Torsed ovarian corpus luteal cyst
ANSWER:A

285
A 27-year-old has just had an ectopic pregnancy. Which of the following events would be most likely to
predispose to ectopic pregnancy?
A Previous cervical conization
B Pelvic inflammatory disease (PID)
C Use of a contraceptive uterine device (IUD)
D Induction of ovulation
ANSWER:B
286
A 34-year-old G2P1 at 31 weeks gestation presents to labor and delivery with complaints of vaginal bleeding
earlier in the day that resolved on its own. She denies any leakage of fluid or uterine contractions. She reports
good fetal movement. In her last pregnancy, she had a low trans- verse cesarean delivery for breech presentation
at term. She denies any medical problems. Her vital signs are normal and electronic external monitoring reveals a
reactive fetal heart rate tracing and no uterine contractions. Which of the following is the most appropriate next
step in the management of this patient?
A Send her home, since the bleeding has completely resolved and she is
B experiencing good fetal movements
C Perform a sterile digital examination
D Perform a sterile speculum examination
ANSWER:D
287
A 34-year-old G2P1 at 31 weeks gestation with a known placenta previa presents to the hospital with vaginal
bleeding. On assessment, she has normal vital signs and the fetal heart rate tracing is 140 beats per minute with
accelerations and no decelerations. No uterine contractions are demonstrated on external tocometer. Heavy vaginal
bleeding is noteD. Which of the following is the best next step in the management of this patient?
A Administer intramuscular terbutaline
B Administer methylergonovine
C Admit and stabilize the patient
D Perform cesarean delivery
ANSWER:C

288
A 40-year-old G2P1001 presents to your office for a routine OB visit at 30 weeks gestational agE. Her first
pregnancy was delivered 10 years ago and was uncomplicateD. She had a normal vaginal delivery at 40 weeks
and the baby weighed 7 lB. During this present pregnancy, she has not had any complications, and she reports no
significant medical history. She is a non- smoker and has gained about 25 lb to datE. Despite being of advanced
maternal age, she declined any screening or diagnostic testing for Down syndromE. Her blood pressure range has
been 100 to 120/60 to 70. During her examination, you note that her fundal height measures only 25 cm. Which
of the following is a likely explanation for this patient’s decreased fundal height?
A Multiple gestation
B Hydramnios
C Fetal growth restriction
D The presence of fibroid tumors in the uterus
ANSWER:C
289
A 26-year-old G1 at 37 weeks presents to the hospital in active labor. She has no medical problems and has a
normal prenatal course except for fetal growth restriction. She undergoes an uncomplicated vaginal delivery of a
female infant weighing 1950 g. The infant is at risk for which of the following complications?
A Hyperglycemia
B Fever
C Hypertension
D Hypertension
ANSWER:D
290
A 39-year-old G1P0 at 39 weeks gestational age is sent to labor and delivery from her obstetrician’s office because
of a blood pressure reading of 150/100 mm Hg obtained during a routine OB visit. Her baseline blood pressures
during the pregnancy were 100 to 120/60 to 70. On arrival to labor and delivery, the patient denies any headache,
visual changes, nausea, vomiting, or abdominal pain. The heart rate strip is reactive and the toco- dynamometer
indicates irregular uterine contractions. The patient’s cervix is 3 cm dilateD. Her repeat blood pressure is 160/90
mm Hg. Hematocrit is 34.0, platelets are 160,000, SGOT is 22, SGPT is 15, and urinalysis is neg- ative for protein.
Which of the following is the most likely diagnosis?
A Preeclampsia
B Chronic hypertension
C Chronic hypertension with superimposed preeclampsia
D Gestational hypertension
ANSWER:D

291
A 20-year-old G1 at 36 weeks is being monitored for preeclampsia; she rings the bell for the nurse because she is
developing a headache and feels funny. As you and the nurse enter the room, you witness the patient undergoing
a tonic-clonic seizurE. You secure the patient’s airway, and within a few minutes the seizure is over. The patient’s
blood pressure monitor indicates a pressure of 160/110 mm Hg. Which of the following medications is
recommended for the prevention of a recurrent eclamptic seizure?
A Hydralazine
B Magnesium sulfate
C Labetalol
D Pitocin
ANSWER:B
292
A 22-year-old G1 at 14 weeks gestation presents to your office with a history of recent exposure to her 3-year-old
nephew who had a rubella viral infection. In which time period does maternal infection with rubella virus carry
the greatest risk for congenital rubella syndrome in the fetus?
A Preconception
B First trimester
C Second trimester
D Third trimester
ANSWER:B
293
A 30-year-old class D diabetic is concerned about pregnancy. She can be assured that which of the following risks
is the same for her as for the general population?
A Preeclampsia and eclampsia
B Infection
C Fetal cystic fibrosis
D Postpartum hemorrhage after vaginal delivery
ANSWER:C
294
A 23-year-old G1P0 reports to your office for a routine OB visit at 28 weeks gestational agE. Labs drawn at her
prenatal visit 2 weeks ago reveal a 1-hour glucose test of 128, hemoglobin of 10.8, and a platelet count of 80,000.
All her other labs were within normal limits. During the present visit, the patient has a blood pressure of 120/70
mm Hg. Her urine dip is negative for protein, glucose, and blooD. The patient denies any com- plaints. The only
medication she is currently taking is a prenatal vitamin. She does report a history of epistaxis on occasion, but no
other bleeding. Which of the following medical treatments should you recommend to treat the thrombocytopenia?
A No treatment is necessary

B Stop prenatal vitamins
C Oral corticosteroid therapy
D Intravenous immune globulin
ANSWER:A
295
A 20-year-old G1 at 38 weeks gestation presents with regular painful contractions every 3 to 4 minutes lasting 60
seconds. On pelvic examina- tion, she is 3 cm dilated and 90% effaced; an amniotomy is performed and clear fluid
is noteD. The patient receives epidural analgesia for pain man- agement. The fetal heart rate tracing is reactivE.
One hour later on repeat examination, her cervix is 5 cm dilated and 100% effaceD. Which of the following is the
best next step in her management?
A Begin pushing
B Initiate Pitocin augmentation for protracted labor
C No intervention; labor is progressing normally
D Perform cesarean delivery for inadequate cervical effacement
ANSWER:C
296
A 30-year-old G2P0 at 39 weeks is admitted in active labor with spontaneous rupture of membranes occurring 2
hours prior to admission. The patient noted clear fluid at the timE. On examination, her cervix is 4 cm dilated and
completely effaceD. The fetal head is at 0 station and the fetal heart rate tracing is reactivE. Two hours later on
repeat examination her cervix is 5 cm dilated and the fetal head is at +1 station. Early decelerations are noted on
the fetal heart rate tracing. Which of the following is the best next step in her labor management?
A Administer terbutaline
B Initiate amnioinfusion
C Initiate Pitocin augmentation
D Perform cesarean delivery for arrest of descent
ANSWER:C
297
A 27-year-old G2P1 at 38 weeks gestation was admitted in active labor at 4 cm dilated; spontaneous rupture of
membranes occurred prior to admission. She has had one prior uncomplicated vaginal delivery and denies any
medical problems or past surgery. She reports an allergy to sulfa drugs. Currently, her vital signs are normal and
the fetal heart rate tracing is reactivE. Her prenatal record indicates that her Group B streptococcus (GBS) culture
at 36 weeks was positivE. What is the recommended antibiotic for prophylaxis during labor?
A Cefazolin
B Clindamycin
C Erythromycin

D Penicillin
ANSWER:D
298
A 23-year-old G1 at 38 weeks gestation presents in active labor at 6 cm dilated with ruptured membranes. On
cervical examination the fetal nose, eyes, and lips can be palpateD. The fetal heart rate tracing is 140 beats per
minute with accelerations and no (oxytocin) decelerations. The patient’s pelvis is adequatE. Which of the
following is the most appropriate management for this patient?
A Perform immediate cesarean section without labor.
B Allow spontaneous labor with vaginal delivery.
C
Perform forceps rotation in the second stage of labor to convert mentum posterior to mentum anterior and to allow
vaginal delivery.
D
Allow patient to labor spontaneously until complete cervical dilation is achieved and then perform an internal
podalic version with breech extraction.
ANSWER:B
299
A 32-year-old G3P2 at 39 weeks gestation presented to the hospital with ruptured membranes and 4 cm dilateD.
She has a history of two prior vaginal deliveries, with her largest child weighing 3800 g at birth. Over the next 2
hours she progresses to 7 cm dilateD. Two hours later, she remains 7 cm dilateD. The estimated fetal weight by
ultrasound is 3200 g. Which of the following labor abnormalities best describes this patient?
A Prolonged latent phase
B Protracted active-phase dilation
C Hypertonic dysfunction
D Secondary arrest of dilation
ANSWER:D
300
You are following a 38-year-old G2P1 at 39 weeks in labor. She has had one prior vaginal delivery of a 3800-g
infant. One week ago, the esti- mated fetal weight was 3200 g by ultrasounD. Over the past 3 hours her cervical
examination remains unchanged at 6 cm. Fetal heart rate tracing is reactivE. An intrauterine pressure catheter
(IUPC) reveals two contractions in 10 minutes with amplitude of 40 mm Hg each. Which of the following is the
best management for this patient?
A Ambulation
B Sedation
C Administration of oxytocin
D Cesarean section
ANSWER:C

301
A primipara is in labor and an episiotomy is about to be cut. Compared with a midline episiotomy, which of the
following is an advantage of mediolateral episiotomy?
A Ease of repair
B Fewer breakdowns
C Less blood loss
D Less extension of the incision
ANSWER:D
302
A 27-year-old woman (G3P2) comes to the delivery floor at 37 weeks gestation. She has had no prenatal carE.
She complains that, on bending down to pick up her 2-year-old child, she experienced sudden, severe back pain
that now has persisted for 2 hours. Approximately 30 minutes ago she noted bright red blood coming from her
vaginA. By the time she arrives at the delivery floor, she is contracting strongly every 3 minutes; the uterus is quite
firm even between contractions. By abdominal palpation, the fetus is vertex with the head deeply engageD. Fetal
heart rate is 130 beats per minutes. The fundus is 38 cm above the symphysis. Blood for clotting is drawn, and a
clot forms in 4 minutes. Clotting studies are sent to the laboratory. Which of the following actions can most likely
wait until the patient is stabilized?
A Stabilizing maternal circulation
B Attaching a fetal electronic monitor
C Inserting an intrauterine pressure catheter
D Administering oxytocin
ANSWER:D
303
A 24-year-old primigravid woman, at term, has been in labor for 16 hours and has been dilated to 9 cm for 3
hours. The fetal vertex is in the right occiput posterior position, at +1 station, and moldeD. There have been mild
late decelerations for the past 30 minutes. Twenty minutes ago, the fetal scalp pH was 7.27; it is now 7.20. For
above clinical description, select the most appropriate procedure.
A External version
B Internal version
C Midforceps rotation
D Low transverse cesarean section
ANSWER:D
304
You have just delivered an infant weighing 2.5 kg (5.5 lb) at 39 weeks gestation. Because the uterus still feels
large, you do a vaginal examination. A second set of membranes is bulging through a fully dilated cervix, and
you feel a small part presenting in the saC. A fetal heart is auscultated at 60 beats per minutE. For above clinical
description, select the most appropriate procedurE.

A External version
B Internal version
C Midforceps rotation
D Low transverse cesarean section
ANSWER:B
305
A 24-year-old woman (G3P2) is at 40 weeks gestation. The fetus is in the transverse lie presentation. For above
clinical description, select the most appropriate procedurE.
A External version
B Internal version
C Midforceps rotation
D Low transverse cesarean section
ANSWER:A
306
A nulliparous woman is in active labor C.ervical dilation 5 cm with complete effacement, vertex at 0 station); the
labor curve shows pro- tracted progression without descent following the administration of an epidural block. An
IUPC shows contractions every 4 to 5 minutes, peaking at 40 mm Hg. Select the most appropriate treatment for
above clinical situation.
A Epidural block
B Meperidine D.emerol) 100 mg intramuscularly
C Oxytocin intravenously
D Midforceps delivery
ANSWER:C
307
A nulliparous woman has had arrest of descent for the past 2 hours and arrest of dilation for the past 3 hours. The
cervix is dilated to 7 cm and the vertex is at +1 station. Monitoring shows a normal pattern and adequate
contractions. Fetal weight is estimated at 7.5 lB. Select the most appropriate treatment for above clinical situation.
A Epidural block
B Meperidine D.emerol) 100 mg intramuscularly
C Oxytocin intravenously
D Cesarean section
ANSWER:D
308
Appears to lengthen the second stage of labor. Match above description with the most appropriate type of obstetric
anesthesiA.
A Paracervical block

B Pudendal block
C Spinal block
D Epidural block
ANSWER:C
309
A 23-year-old G1 at 40 weeks gestation presents to the hospital with the complaint of contractions. She states they
are occurring every 4 to 8 minutes and each lasts approximately 1 minutE. She reports good fetal movement and
denies any leakage of fluid or vaginal bleeding. The nurse places an external tocometer and fetal monitor and
reports that the patient is having contractions every 2 to 10 minutes. The nurse states that the con- tractions are
mild to palpation. On examination the cervix is 2 cm dilated, 50% effaced, and the vertex is at −1 station. The
patient had the same cervical examination in your office last week. The fetal heart rate tracing isn140 beats per
minute with accelerations and no decelerations. Which of the following stages of labor is this patient in?
A Active labor
B Latent labor
C False labor
D Stage 1 of labor
ANSWER:C
310
A 28-year-old G1 at 38 weeks had a normal progression of her labor. She has an epidural and has been pushing
for 2 hours. The fetal head is direct occiput anterior at +3 station. The fetal heart rate tracing is 150 beats per
minute with variable decelerations. With the patient’s last push the fetal heart rate had a prolonged deceleration to
the 80s for 3 minutes. You recommend forceps to assist the delivery owing to the nonreassuring fetal heart rate
tracing. Compared to the use of the vacuum extractor, forceps are associated with an increased risk of which of the
following neonatal complications?
A Cephalohematoma
B Retinal hemorrhage
C Jaundice
D Corneal abrasions
ANSWER:D
311
You performed a forceps-assisted vaginal delivery on a 20-year-old G1 at 40 weeks for maternal exhaustion. The
patient had pushed for 3 hours with an epidural for pain management. A second-degree episiotomy was cut to
facilitate delivery. Eight hours after delivery, you are called to see the patient because she is unable to void and
complains of severe pain. On examination you note a large fluctuant purple mass inside the vaginA. What is the
best management for this patient?

A Apply an ice pack to the perineum
B Embolize the internal iliac artery
C Incision and evacuation of the hematoma
D Perform dilation and curettage to remove retained placenta
ANSWER:C
312
A 20-year-old G1 at 41 weeks has been pushing for 21/2 hours. The fetal head is at the introitus and beginning to
crown. It is necessary to cut an episiotomy. The tear extends through the sphincter of the rectum, but the rectal
mucosa is intact. How should you classify this type of episiotomy?
A First-degree
B Second-degree
C Third-degree
D Fourth-degree
ANSWER:C
313
A 25-year-old G1P0 patient at 41 weeks presents to labor and delivery complaining of gross rupture of
membranes and painful uterine contractions every 2 to 3 minutes. On digital examination, her cervix is 3 cm
dilated and completely effaced with fetal feet palpable through the cervix. The estimated weight of the fetus is
about 6 lb, and the fetal heart rate tracing is reactivE. Which of the following is the best method to achieve
delivery?
A Deliver the fetus vaginally by breech extraction
B Deliver the baby vaginally after external cephalic version
C Perform an emergent cesarean section
D Perform an internal podalic version
ANSWER:C
314
A 25-year-old G1 at 37 weeks presents to labor and delivery with gross rupture of membranes. The fluid is noted
to be clear and the patient is noted to have regular painful contractions every 2 to 3 minutes lasting for 60 seconds
each. The fetal heart rate tracing is reactivE. On cervical examination she is noted to be 4 cm dilated, 90% effaced
with the presenting part a −3 station. The presenting part is soft and felt to be the fetal buttock. A quick bedside
ultrasound reveals a breech presentation with both hips flexed and knees extendeD. What type of breech
presentation is described?
A Frank
B Incomplete, single footling
C Complete
D Double footling

ANSWER:A
315
On postoperative day 3 after an uncomplicated repeat cesarean delivery, the patient develops a fever of 38.2°C
(100.8°F). She has no com- plaints except for some fullness in her breasts. On examination she appears in no
distress; lung and cardiac examinations are normal. Her breast exam- ination reveals full, firm breasts bilaterally
slightly tender with no erythema or masses. She is not breast-feeding. The abdomen is soft with firm, non- tender
fundus at the umbilicus. The lochia appears normal and is non- odorous. Urinalysis and white blood cell count are
normal. Which of the following is a characteristic of the cause of her puerperal fever?
A Appears in less than 5% of postpartum women
B Appears 3 to 4 days after the development of lacteal secretion
C Is almost always painless
D Is less severe and less common if lactation is suppressed
ANSWER:D
316
A 38-year-old G3P3 begins to breast-feed her 5-day-old infant. The baby latches on appropriately and begins to
sucklE. In the mother, which of the following is a response to suckling?
A Decrease of oxytocin
B Increase of prolactin-inhibiting factor
C Increase of hypothalamic dopamine
D Increase of hypothalamic prolactin
ANSWER:D
317
A 24-year-old G1P1 presents for her routine postpartum visit 6 weeks after an uncomplicated vaginal delivery.
She states that she is having prob- lems sleeping and is feeling depressed over the past 2 to 3 weeks. She reveals
that she cries on most days and feels anxious about taking care of her newborn son. She denies any weight loss or
gain, but states she doesn’t feel like eating or doing any of her normal activities. She denies suicidal or homicidal
ideation. Which of the following is true regarding this patient’s condition?
A A history of depression is not a risk factor for developing postpartum depression.
B
Prenatal preventive intervention for patients at high risk for postpartum depression is best managed alone by a
mental health professional.
C Young, multiparous patients are at highest risk.
D About 8% to 15% of women develop postpartum depression.
ANSWER:D

318
A 21-year-old G1 at 40 weeks, who underwent induction of labor for severe preeclampsia, delivered a 3900-g
male infant via vaginal delivery after pushing for 21/2 hours. A second-degree midline laceration and side- wall
laceration were repaired in the usual fashion under local analgesiA. The estimated blood loss was 450 cC.
Magnesium sulfate is continued post- partum for the seizure prophylaxis. Six hours after the delivery, the patient
has difficulty voiding. Which is the most likely cause of her problem?
A Preeclampsia
B Infusion of magnesium sulfate
C Vulvar hematoma
D Ureteral injury
ANSWER:C
319
A 32-year-old G2P2 develops fever and uterine tenderness 2 days after cesarean delivery for nonreassuring fetal
heart tones. She is placed on intravenous penicillin and gentamicin for her infection. After 48 hours of antibiotics
she remains febrile, and on examination she continues to have uterine tenderness. Which of the following bacteria
is resistant to these antibiotics and is most likely to be responsible for this woman’s infection?
A Proteus mirabilis
B Bacteroides fragilis
C Escherichia coli
D α-Streptococci
ANSWER:B
320
A 21-year-old G2P2 calls her physician 7 days postpartum because she is concerned that she is still bleeding from
the vaginA. She describes the bleeding as light pink to bright red and less heavy than the first few days
postdelivery. She denies fever or any cramping pain. On examination she is afebrile and has an appropriately
sized, nontender uterus. The vagina con- tains about 10 cc of old, dark blood. The cervix is closeD. Which of the
fol- lowing is the most appropriate treatment?
A Antibiotics for endometritis
B High-dose oral estrogen for placental subinvolution
C Oxytocin for uterine atony
D Reassurance
ANSWER:D

321
A 28-year-old G2P2 presents to the hospital 2 weeks after vaginal delivery with the complaint of heavy vaginal
bleeding that soaks a sanitary napkin every hour. Her pulse is 89 beats per minute, blood pressure 120/76 mm Hg,
and temperature 37.1°C (98.9°F). Her abdomen is non- tender and her fundus is located above the symphysis
pubis. On pelvic examination, her vagina contained small blood clots and no active bleeding is noted from the
cervix. Her uterus is about 12 to 14 weeks size and non- tender. Her cervix is closeD. An ultrasound reveals an 8-
mm endometrial stripE. Her hemoglobin is 10.9, unchanged from the one at her vaginal delivery. β-hCG is
negativE. Which of the following potential treatments would be contraindicated?
A Methylergonovine maleate (Methergine)
B Oxytocin injection (Pitocin)
C Ergonovine maleate E.rgotrate)
D Dilation and curettage
ANSWER:D
322
A 22-year-old G1P0 has just undergone a spontaneous vaginal delivery. As the placenta is being delivered, a red
fleshy mass is noted to be protruding out from behind the placentA. Which of the following is the best next step in
management of this patient?
A Begin intravenous oxytocin infusion
B Call for immediate assistance from other medical personnel
C Continue to remove the placenta manually
D Shove the placenta back into the uterus
ANSWER:D
323
Following a vaginal delivery, a woman develops a fever, lower abdom- inal pain, and uterine tenderness. She is
alert, and her blood pressure and urine output are gooD. Large gram-positive rods suggestive of clostridia are seen
in a smear of the cervix. Which of the following is most closely tied to a decision to proceed with hysterectomy?
A Close observation for renal failure or hemolysis
B Immediate radiographic examination for hydrosalpinx
C High-dose antibiotic therapy
D Gas gangrene
ANSWER:D

324
Three days ago you delivered a 40-year-old G1P1 by cesarean section following arrest of descent after 2 hours of
pushing. Labor was also signif- icant for prolonged rupture of membranes. The patient had an epidural, which was
removed the day following delivery. The nurse pages you to come to see the patient on the postpartum floor
because she has a fever of 38.8°C (102°F) and is experiencing shaking chills. Her blood pressure is 120/70 mm Hg
and her pulse is 120 beats per minutE. She has been eating a regular diet without difficulty and had a normal
bowel movement this morning. She is attempting to breast-feed, but says her milk has not come in yet. On
physical examination, her breasts are mildly engorged and ten- der bilaterally. Her lungs are clear. Her abdomen is
tender over the fundus, but no rebound is present. Her incision has some serous drainage at the right apex, but no
erythema is noteD. Her pelvic examination reveals uterine tenderness but no masses. Which of the following is the
most likely diagnosis?
A Pelvic abscess
B Septic pelvic thrombophlebitis
C Wound infection
D Endometritis
ANSWER:D
325
You are doing postpartum rounds on a 23-year-old G1P1 who is postpartum day 2 after an uncomplicated vaginal
delivery. As you walk in the room, you note that she is crying. She states she can’t seem to help it. She denies
feeling sad or anxious. She has not been sleeping well because of getting up every 2 to 3 hours to breast-feed her
new baby. Her past medical history is unremarkablE. Which of the following is the most appropriate treatment
recommendation?
A Time and reassurance, because this condition is self-limited
B Referral to psychiatry for counseling and antidepressant therapy
C Referral to psychiatry for admission to a psychiatry ward and therapy with
D Haldol
ANSWER:A
326
20-year-old G1P1 is postpartum day 2 after an uncomplicated vaginal delivery of a 6-lb 10-oz baby boy. She is
trying to decide whether to have you perform a circumcision on her newborn. The boy is in the well- baby
nursery and is doing very well. In counseling this patient, you tell her which of the following recommendations
from the American Pediatric Association?
A Circumcisions should be performed routinely because they decrease the incidence of male urinary tract infections.
B Circumcisions should be performed routinely because they decrease the incidence of penile cancer.

C Circumcisions should be performed routinely because they decrease the incidence of sexually transmitted diseases.
D Circumcisions should not be performed routinely because of insufficient data regarding risks and benefits.
ANSWER:D
327
You are counseling a new mother and father on the risks and benefits of circumcision for their 1-day-old son. The
parents ask if you will use analgesia during the circumcision. What do you tell them regarding the
recommendations for administering pain medicine for circumcisions?
A
Analgesia is not recommended because there is no evidence that newbornsundergoing circumcision experience
pain.
B Analgesia is not recommended because it is unsafe in newborns.
C Analgesia in the form of oral Tylenol is the pain medicine of choice recom- mended for circumcisions.
D Analgesia in the form of a penile block is recommende
ANSWER:D
328
You are asked to assist in the well-born nursery with neonatal carE. Which of the following is a part of routine
care in a healthy infant?
A Administration of ceftriaxone cream to the eyes for prophylaxis for gonorrhea and chlamydia
B Administration of vitamin A to prevent bleeding problems
C Administration of hepatitis B vaccination for routine immunization
D Cool-water bath to remove vernix
ANSWER:C
329
You are making rounds on a 29-year-old G1P1 who underwent an uncomplicated vaginal delivery at term on the
previous day. The patient is still very confused about whether she wants to breast-feed. She is a very busy lawyer
and is planning on going back to work in 4 weeks, and she does not think that she has the time and dedication that
breast-feeding requires. She asks you what you think is best for her to do. Which of the following is an accurate
statement regarding breast-feeding?
A Breast-feeding decreases the time to return of normal menstrual cycles.
B Breast-feeding is associated with a decreased incidence of sudden infant death syndromE.
C Breast-feeding is a poor source of nutrients for required infant growth.
D Breast-feeding is associated with an increased incidence of childhood obesity.
ANSWER:B
330
A 22-year-old G1P1 who is postpartum day 2 and is bottle-feeding complains that her breasts are very engorged
and tender. She wants you to give her something to make the engorgement go away. Which of the following is
recommended to relieve her symptoms?

A Breast binder
B Bromocriptine
C Estrogen-containing contraceptive pills
D Pump her breasts
ANSWER:A
331
A 36-year-old G1P1 comes to see you for a routine postpartum exam- ination 6 weeks after an uncomplicated
vaginal delivery. She is currently nursing her baby without any major problems and wants to continue to do so for
at least 9 months. She is ready to resume sexual activity and wants to know what her options are for birth control.
She does not have any medical problems. She is a nonsmoker and is not taking any medications except for her
prenatal vitamins. Which of the following methods may decrease her milk supply?
A Intrauterine device
B Progestin only pill
C Depo-Provera
D Combination oral contraceptives
ANSWER:D
332
A 30-year-old G3P3, who is 8 weeks postpartum and regularly breast-feeding calls you and is very concerned
because she is having pain with intercourse secondary to vaginal dryness. Which of the following should you
recommend to help her with this problem?
A Instruct her to stop breast-feeding
B Apply hydrocortisone cream to the perineum
C Apply testosterone cream to the vulva and vagina
D Apply estrogen cream to the vagina and vulva
ANSWER:D
333
A 25-year-old G1P1 comes to see you 6 weeks after an uncompli- cated vaginal delivery for a routine postpartum
examination. She denies any problems and has been breast-feeding her newborn without any diffi- culties since
leaving the hospital. During the bimanual examination, you note that her uterus is irregular, firm, nontender, and
about a 15-week sizE. Which of the following is the most likely etiology for this enlarged uterus?
A Subinvolution of the uterus
B The uterus is appropriate size for 6 weeks postpartum
C Fibroid uterus
D Adenomyosis
ANSWER:C

334
A 39-year-old G3P3 comes to see you on day 5 after a second repeat cesarean delivery. She is concerned because
her incision has become very red and tender and pus started draining from a small opening in the inci- sion this
morning. She has been experiencing general malaise and reports a fever of 38.8°C (102°F). Physical examination
indicates that the Pfan- nenstiel incision is indeed erythematous and is open about 1 cm at the left corner, and is
draining a small amount of purulent liquiD. There is tender- ness along the wound edges. Which of the following
is the best next step in the management of this patient?
A Apply Steri-Strips to close the wound
B Administer antifungal medication
C Probe the fascia
D Take the patient to the OR for debridement and closure of the skin
ANSWER:C
335
A 30-year-old G3P3 is postoperative day 4 after a repeat cesarean delivery. During the surgery she received 2
units of packed red blood cells for a hemorrhage related to uterine atony. She is to be discharged home today. She
complains of some yellowish drainage from her incision and redness that just started earlier in the day. She states
that she feels feverish. She is breast-feeding. Her past medical history is significant for type 2 dia- betes mellitus
and chronic hypertension. She weighs 110 kg. Her vital signs are temperature 37.8°C (100.1°F), pulse 69 beats per
minute, respi- ratory rate 18 breaths per minute, and blood pressure is 143/92 mm Hg. Breast, lung, and cardiac
examinations are normal. Her midline vertical skin incision is erythematous and has a foul-smelling purulent
discharge from the lower segment of the wounD. It is tender to touch. The uterine fundus is not tender. Which of
the following is not a risk factor for her condition?
A Diabetes
B Corticosteroid therapy
C Preoperative antibiotic administration
D Anemia
ANSWER:C
336
You are following up on the results of routine testing of a 68-year-old G4P3 for her well-woman examination.
Her physical examination was nor- mal for a postmenopausal woman. Her Pap smear revealed parabasal cells, her
mammogram was normal, lipid profile was normal, and the urinalysis shows hematuriA. Which of the following is
the most appropriate next step in the management of this patient?
A Colposcopy
B Endometrial biopsy
C Renal sonogram
D Urine culture

ANSWER:D
337
A 74-year-old woman presents to your office for well-woman exam- ination. Her last Pap smear and mammogram
were 3 years ago. She has hypertension, high cholesterol, and osteoarthritis. She stopped smoking 15 years ago,
and denies alcohol usE. Based on this patient’s history which of the following medical conditions should be this
patient’s biggest concern?
A Alzheimer disease
B Breast cancer
C Cerebrovascular disease
D Heart disease
ANSWER:D
338
A 17-year-old G1P1 presents to your office for her yearly well- woman examination. She had an uncomplicated
vaginal delivery last year. She has been sexually active for the past 4 years and has had six different sexual
partners. Her menses occurs every 28 days and lasts for 4 days. She denies any intermenstrual spotting, postcoital
bleeding, or vaginal dis- chargE. She denies tobacco, alcohol, or illicit drug usE. Which of the fol- lowing are
appropriate screening tests for this patient?
A Pap test
B Pap test and gonorrhea and chlamydia cervical cultures
C Pap test and herpes simplex cultures
D Pap test and hemoglobin level assessment
ANSWER:B
339
A 26-year-old woman presents to your office for her well-woman examination. She denies any medical problems
or prior surgeries. She states that her cycles are monthly. She is sexually active and uses oral contracep- tive pills
for birth control. Her physical examination is normal. As part of preventive health maintenance, you recommend
breast self-examination and instruct the patient how to do it. Which of the following is the best fre- quency and
time to perform breast self-examinations?
A Monthly, in the week prior to the start of the menses
B Monthly, in the week after cessation of menses
C Monthly, during the menses
D Every 3 months, in the week prior to the start of the menses
ANSWER:B

340
A married 41-year-old G5P3114 presents to your office for a routine examination. She reports being healthy
except for a history of migraine headaches. All her Pap smears have been normal. She developed gestational
diabetes in her last pregnancy. She drinks alcohol socially, and admits to smoking occasionally. Her grandmother
was diagnosed with ovarian cancer when she was in her fifties. Her blood pressure is 140/90 mm Hg; height is 5 ft
5 in; weight is 150 lB. Which of the following is the most common cause of death in women of this patient’s age?
A HIV
B Cardiac disease
C Accidents
D Cancer
ANSWER:D
341
A 36-year-old G2P2 presents for her well-woman examination. She has had two spontaneous vaginal deliveries
without complications. Her largest child weighed 3500 g at birth. She uses oral contraceptive pills and denies any
history of an abnormal Pap smear. She does not smoke, but drinks about four times per week. Her weight is 70 kg.
Her vital signs are normal. After place- ment of the speculum, you note a clear cyst approximately 2.5 cm in size
on the lateral wall of the vagina on the right sidE. The cyst is nontender and does not cause the patient any
dyspareunia or discomfort. Which of the following is the most likely diagnosis of this mass?
A Bartholin duct cyst
B Gartner duct cyst
C Lipoma
D Hematoma
ANSWER:B
342
A 50-year-old G4P4 presents for her well-woman examination. She had one cesarean delivery followed by three
vaginal deliveries. Her menses stopped 1 year ago and she occasionally still has a hot flash. She tells you that about
10 years ago she was treated with a laser conization for carcinoma in situ of her cervix. Since that time, all of her
Pap tests have been normal. What recommendation should you make regarding how frequently she should
undergo Pap smear testing?
A Every 3 months
B Every 6 months
C Every year
D Every 2 years
ANSWER:C

343
A 45-year-old G3P3 presents for her yearly examination. She last saw a doctor 7 years ago after she had her last
chilD. She had three vaginal deliveries, the last of which was complicated by gestational diabetes and
preeclampsiA. She has not been sexually active in the past year. She once had an abnormal Pap smear for which
she underwent cryotherapy. She denies any medical problems. Her family history is significant for coronary artery
disease in her dad and a maternal aunt who developed ovarian cancer at the age of 67. Which of the following is
best screening approach for this patient?
A Pap smear
B Pap smear and mammography
C Pap smear, mammography, and cholesterol profile
D Pap smear, mammography, cholesterol profile, and fasting blood sugar
ANSWER:D
344
A 30-year-woman presents to your office with the fear of developing ovarian cancer. Her 70-year-old
grandmother recently died from ovarian cancer. You discuss with her the risks factors and prevention for ovarian
cancer. Which of the following can decrease a woman’s risk of ovarian cancer?
A Use of combination oral contraceptive therapy
B Menopause after age 55
C Nonsteroidal anti-inflammatory drugs
D Nulliparity
ANSWER:A
345
A 42-year-old G4P3104 presents for her well-woman examination. She has had three vaginal deliveries and one
cesarean delivery for breech. She states her cycles are regular and denies any sexually transmitted diseases.
Currently she and her husband use condoms, but they hate the hassle of a coital-dependent methoD. She is
interested in a more effective contraception because they do not want any more children. She reports occasional
migraine headaches and had a serious allergic reaction to anesthesia as a child when she underwent a
tonsillectomy. She drinks and smokes socially. She weighs 78 kg, and her blood pressure is 142/89 mm Hg.
During her office visit, you counsel the patient at length regarding birth control methods. Which of the following
is the most appropriate contraceptive method for this patient?
A Intrauterine device
B Bilateral tubal ligation
C Combination oral contraceptives
D Diaphragm
ANSWER:A

346
A 48-year-old G2P2 presents for her well-woman examination. She had two uneventful vaginal deliveries. She
had a vaginal hysterectomy for fibroids and menorrhagiA. She denies any medical problems, but has not seen a
doctor in 6 years. Her family history is significant for stroke, dia- betes, and high blood pressurE. On examination
she is a pleasant female, stands 5 ft 3 in tall, and weighs 85 kg. Her blood pressure is 150/92 mm Hg, pulse 70
beats per minute, respiratory rate 14 breaths per minute, and tem- perature 37°C (98.4°F). Her breast, lung,
cardiac, abdomen, and pelvic examinations are normal. The next appropriate step in the management of this
patient’s blood pressure is which of the following?
A Beta-blocker
B Calcium channel blocker
C Diuretic
D Diet, exercise, weight loss, and repeat blood pressure in 2 months
ANSWER:D
347
A 50-year-old woman is diagnosed with cervical cancer. Which lymph node group would be the first involved in
metastatic spread of this disease beyond the cervix and uterus?
A Common iliac nodes
B Parametrial nodes
C External iliac nodes
D Paracervical or ureteral nodes
ANSWER:D
348
A 54-year-old woman undergoes a laparotomy because of a pelvic mass. At exploratory laparotomy, a unilateral
ovarian neoplasm is discovered that is accompanied by a large omental metastasis. Frozen section diagnosis
confirms metastatic serous cystadenocarcinomA. Which of the following is the most appropriate intraoperative
course of action?
A Excision of the omental metastasis and ovarian cystectomy
B Omentectomy and ovarian cystectomy
C Excision of the omental metastasis and unilateral oophorectomy
D Omentectomy, total abdominal hysterectomy, and bilateralsalpingo-oophorectomy
ANSWER:D
349
A 58-year-old woman is seen for evaluation of a swelling in her right vulvA. She has also noted pain in this area
when walking and during coitus. At the time of pelvic examination, a mildly tender, fluctuant mass is noted just
outside the introitus in the right vulva in the region of the Bartholin glanD. Which of the following is the most
appropriate treatment?
A Marsupialization

B Administration of antibiotics
C Surgical excision
D Incision and drainage
ANSWER:C
350
A 51-year-old woman is diagnosed with invasive cervical carcinoma by cone biopsy. Pelvic examination and
rectal-vaginal examination reveal the parametrium to be free of disease, but the upper portion of the vagina is
involved with tumor. Intravenous pyelography (IVP) and sigmoidoscopy are negative, but a computed
tomography C.T) scan of the abdomen and pelvis shows grossly enlarged pelvic and periaortic nodes. This patient
is classified at which of the following stages?
A IIa
B IIb
C IIIa
D IIIb
ANSWER:A
351
A 35-year-old G3P3 with a Pap smear showing high-grade squamous intraepithelial lesion of the cervix C.IN III)
has an inadequate colposcopy. Cone biopsy of the cervix shows squamous cell cancer that has invaded only 1 mm
beyond the basement membranE. There are no confluent tongues of tumor, and there is no evidence of lymphatic
or vascular invasion. The margins of the cone biopsy specimen are free of diseasE. How should you classify or
stage this patient’s disease?
A Carcinoma of low malignant potential
B Microinvasive cancer, stage Ia1
C Atypical squamous cells of undetermined significance
D Carcinoma in situ
ANSWER:B
352
A 35-year-old G3P3 with a Pap smear showing high-grade squamous intraepithelial lesion of the cervix C.IN III)
has an inadequate colposcopy. Cone biopsy of the cervix shows squamous cell cancer that has invaded only 1 mm
beyond the basement membranE. There are no confluent tongues of tumor, and there is no evidence of lymphatic
or vascular invasion. The margins of the cone biopsy specimen are free of diseasE. The patient above now asks
you for your advice on how to treat her cervical diseasE. Your best recommendation is for the patient to undergo
which of the following?
A Treatment with external beam radiation
B Implantation of radioactive cesium into the cervical canal
C Simple hysterectomy

D Simple hysterectomy with pelvic lymphadenectomy
ANSWER:C
353
A pregnant 35-year-old patient is at highest risk for the concurrent development of which of the following
malignancies?
A Cervix
B Ovary
C Breast
D Vagina
ANSWER:A
354
A 22-year-old G3P0030 obese female comes to your office for a rou- tine gynecologic examination. She is single,
but is currently sexually activE. She has a history of five sexual partners in the past, and became sexually active at
age 15. She has had three first-trimester voluntary pregnancy ter- minations. She uses Depo-Provera for birth
control, and reports occasion- ally using condoms as well. She has a history of genital warts, but denies any prior
history of abnormal Pap smears. The patient denies use of any illicit drugs, but admits to smoking about one pack
of cigarettes a day. Her physical examination is normal. However, 3 weeks later you receive the results of her Pap
smear, which shows a high-grade squamous intraepithe- lial lesion (HGSIL). Which of the following factors in this
patient’s history does not increase her risk for cervical dysplasia?
A Young age at initiation of sexual activity
B Multiple sexual partners
C History of genital warts
D Use of Depo-Provera
ANSWER:D
355
A 57-year-old menopausal patient presents to your office for evaluation of postmenopausal bleeding. She is
morbidly obese and has chronic hypertension and adult onset diabetes. An endometrial sampling done in the
office shows complex endometrial hyperplasia with atypia, and a pelvic ultrasound done at the hospital
demonstrates multiple, large uterine fibroids. Which of the following is the best treatment option for this patient?
A Myomectomy
B Total abdominal hysterectomy
C Oral contraceptives
D Uterine artery embolization
ANSWER:B

356
You see five postmenopausal patients in the cliniC. Each patient has one of the conditions listed, and each patient
wishes to begin hormone replacement therapy today. Which one of the following patients would you start on
therapy at the time of this visit?
A Mild essential hypertension
B Liver disease with abnormal liver function tests
C Malignant melanoma
D Undiagnosed genital tract bleeding
ANSWER:A
357
A mother brings her 12-year-old daughter in to your office for consul- tation. She is concerned because most of
the other girls in her daughter’s class have already started their perioD. She thinks her daughter hasn’t shown any
evidence of going into puberty yet. Knowing the usual first sign of the onset of puberty, you should ask the
mother which of the following questions?
A Has her daughter had any acne?
B Has her daughter started to develop breasts?
C Does her daughter have any axillary or pubic hair?
D Has her daughter started her growth spurt?
ANSWER:B
358
A 55-year-old woman presents to your office for consultation regard- ing her symptoms of menopausE. She
stopped having periods 8 months ago and is having severe hot flushes. The hot flushes are causing her
considerable stress. What should you tell her regarding the psychological symptoms of the climacteric?
A They are not related to her changing levels of estrogen and progesteronE.
B They commonly include insomnia, irritability, frustration, and malaisE.
C They are related to a drop in gonadotropin levels.
D They are not affected by environmental factors.
ANSWER:B
359
While evaluating a 30-year-old woman for infertility, you diagnose a bicornuate uterus. You explain that
additional testing is necessary because of the woman’s increased risk of congenital anomalies in which organ
system?
A Skeletal
B Hematopoietic
C Urinary
D Central nervous
ANSWER:C

360
A 39-year-old G3P3 complains of severe, progressive secondary dysmenorrhea and menorrhagiA. Pelvic
examination demonstrates a tender, diffusely enlarged uterus with no adnexal tenderness. Results of endometrial
biopsy are normal. Which of the following is the most likely diagnosis?
A Endometriosis
B Endometritis
C Adenomyosis
D Uterine sarcoma
ANSWER:C
361
A 28-year-old G3P0 has a history of severe menstrual cramps, pro- longed, heavy periods, chronic pelvic pain,
and painful intercoursE. All of her pregnancies were spontaneous abortions in the first trimester. A
hysterosalpingogram (HSG) she just had as part of the evaluation for recurrent abortion showed a large uterine
septum. You have recommended surgical repair of the uterus. Of the patient’s symptoms, which is most likely to
be corrected by resection of the uterine septum?
A Habitual abortion
B Dysmenorrhea
C Menometrorrhagia
D Dyspareunia
ANSWER:A
362
During the evaluation of infertility in a 25-year-old female, a ysterosalpingogram showed evidence of Asherman
syndromWhich one of the following symptoms would you expect this patient to have?
A Hypomenorrhea
B Oligomenorrhea
C Menorrhagia
D Metrorrhagia
ANSWER:A
363
A couple presents for evaluation of primary infertility. The evaluation of the woman is completely normal. The
husband is found to have a left varicocelE. If the husband’s varicocele is the cause of the couple’s infertility, what
would you expect to see when evaluating the husband’s semen analysis?
A Decreased sperm count with an increase in the number of abnormal forms
B Decreased sperm count with an increase in motility
C Increased sperm count with an increase in the number of abnormal forms
D Increased sperm count with absent motility
ANSWER:A

364
A 25year-old woman presents to your office for evaluation of primary infertility. She has regular periods every 28
days. She has done testing at home with an ovulation kit, which suggests she is ovulating. A hysterosalpingogram
demonstrates patency of both fallopian tubes. A progesterone level drawn in the mid–luteal phase is lower than
expecteD. A luteal phase defect is suspected to be the cause of this patient’s infertility. Which of the following
studies performed in the second half of the menstrual cycle is helpful in making this diagnosis?
A Serum estradiol levels
B Urinary pregnanetriol levels
C Endometrial biopsy
D Serum follicle-stimulating hormone (FSH) levels
ANSWER:C
365
A 45-year-old woman who had two normal pregnancies 15 and 18 years ago presents with the complaint of
amenorrhea for 7 months. She expresses the desire to become pregnant again. After exclusion of pregnancy,
which of the following tests is next indicated in the evaluation of this patient’s amenorrhea?
A Hysterosalpingogram
B Endometrial biopsy
C Thyroid function tests
D LH and FSH levels
ANSWER:D
366 Which of the following pubertal events in girls is not estrogen dependent?
A Menses
B Vaginal cornification
C Hair growth
D Reaching adult height
ANSWER:C
367
You suspect that your infertility patient has an inadequate luteal phasE. She should undergo an endometrial biopsy
on which day of her menstrual cycle?
A Day 3
B Day 8
C Day 14
D Day 26
ANSWER:D

368
You have recommended a postcoital test for your patient as part of her evaluation for infertility. She and her
spouse should have sexual intercourse on which day of her menstrual cycle as part of postcoital testing?
A Day 3
B Day 8
C Day 14
D Day 21
ANSWER:C
369
You ask a patient to call your office during her next menstrual cycle to schedule a hysterosalpingogram as part of
her infertility evaluation. Which day of the menstrual cycle is best for performing the hysterosalpingogram?
A Day 3
B Day 8
C Day 14
D Day 21
ANSWER:B
370
You have recommended that your infertility patient return to your office during her next menstrual cycle to have
her serum progesterone level checked. Which is the best day of the menstrual cycle to check her proges- terone
level if you are trying to confirm ovulation?
A Day 3
B Day 8
C Day 14
D Day 21
ANSWER:D
371
An 86-year-old woman presents to your office for her well-woman examination. She has no complaints. On pelvic
examination performed in the supine and upright positions, the patient has second-degree prolapse of the uterus.
Which of the following is the best next step in the management of this patient?
A Reassurance
B Placement of a pessary
C Vaginal hysterectomy
D Le Fort procedure
ANSWER:A

372
An 81-year-old woman presents to your office complaining that her uterus fell out 2 months ago. She has multiple
medical problems, includ- ing chronic hypertension, congestive heart failure, and osteoporosis. She is limited to
sitting in a wheelchair because of her health problems. Her fallen uterus causes significant pain. On physical
examination, the patient is frail and requires assistance with getting on the examination tablE. She has com- plete
procidentia of the uterus. Which of the following is the most appro- priate next step in the management of this
patient?
A Reassurance
B Placement of a pessary
C Vaginal hysterectomy
D Le Fort procedure
ANSWER:B
373
A 78-year-old woman with chronic obstructive pulmonary disease, chronic hypertension, and history of
myocardial infarction requiring angioplasty presents to your office for evaluation of something hanging out of her
vaginA. She had a hysterectomy for benign indications at age 48. For the past few months, she has been
experiencing the sensation of pelvic pressurE. Last month she felt a bulge at the vaginal opening. Two weeks ago
something fell out of the vaginA. On pelvic examination, the patient has total eversion of the vaginA. There is a
superficial ulceration at the vaginal apex. Which of the following is the best next step in the management of this
patient?
A Biopsy of the vaginal ulceration
B Schedule abdominal sacral colpopexy
C Place a pessary
D Prescribe topical vaginal estrogen cream
ANSWER:D
374
A 28-year-old woman presents to your office with symptoms of a uri- nary tract infection. This is her second
infection in 2 months. You treated the last infection with Bactrim DS for 3 days. Her symptoms never really
improved. Now she has worsening lower abdominal discomfort, dysuria, and frequency. She has had no fever or
flank pain. Physical examination shows only mild suprapubic tenderness. Which of the following is the best next
step in the evaluation of this patient?
A Urine culture
B Intravenous pyelogram
C Cystoscopy
D Wet smear
ANSWER:A

375
A 28-year-old G3P3 presents to your office for contraceptive coun- seling. She denies any medical problems or
sexually transmitted diseases. You counsel her on the risks and benefits of all contraceptive methods. Which of the
following is the most common form of contraception used by reproductive-age women in the United States?
A Pills
B Condom
C Diaphragm
D Permanent sterilization
ANSWER:D
376
A 20-year-old woman presents to your office for her well-woman examination. She has recently become sexually
active and desires an effective contraceptive methoD. She has no medical problems, but family history is
significant for breast cancer in a maternal aunt at the age of 42. She is worried about getting cancer from taking
birth control pills. You discuss with her the risks and benefits of contraceptive pills. You tell her that which of the
following neoplasms has been associated with the use of oral contraceptives?
A Breast cancer
B Ovarian cancer
C Endometrial cancer
D Hepatic adenoma
ANSWER:D
377
An intrauterine pregnancy of approximately 10 weeks gestation is confirmed in a 30-year-old G5P4 woman with
an IUD in placE. The patient expresses a strong desire for the pregnancy to be continueD. On examina- tion, the
string of the IUD is noted to be protruding from the cervical os. Which of the following is the most appropriate
course of action?
A Leave the IUD in place without any other treatment.
B Leave the IUD in place and continue prophylactic antibiotics throughout pregnancy.
C Remove the IUD immediately.
D Terminate the pregnancy because of the high risk of infection.
ANSWER:C
378
A 19-year-old woman presents for voluntary termination of pregnancy 6 weeks after her expected (missed)
menses. She previously had reg- ular menses every 28 days. Pregnancy is confirmed by β-human chorionic
gonadotropin (β-hCG), and ultrasound confirms expected gestational agE. Which of the following techniques for
termination of pregnancy would be safe and effective in this patient at this time?
A Dilation and evacuation D.&E)

B Hypertonic saline infusion
C Suction dilation and curettage D.&C)
D 15-methyl α-prostaglandin injection
ANSWER:C
379
A 22-year-old nulliparous woman has recently become sexually activE. She consults you because of painful
coitus, with the pain located at the vaginal introitus. It is accompanied by painful involuntary contraction of the
pelvic muscles. Other than confirmation of these findings, the pelvic examination is normal. Which of the
following is the most common cause of this condition?
A Endometriosis
B Psychogenic causes
C Bartholin gland abscess
D Vulvar atrophy
ANSWER:B
380
Five patients present for contraceptive counseling, each requesting that an IUD be inserteD. Which of the
following is a recognized contraindication to the insertion of an IUD?
A Pelvic inflammatory disease
B Previous pregnancy with an IUD
C Dysfunctional uterine bleeding
D Cervical conization
ANSWER:A
381
A couple presents to your office to discuss permanent sterilization. They have three children and are sure they do
not want any morE. You discuss the risk and benefits of surgical sterilization. Which of the following statements is
true regarding surgical sterilizations?
A They cannot be performed immediately postpartum.
B
They have become the second most common method of contraception for white couples between 20 and 40 years
of age in the United States.
C They can be considered effective immediately in females B.ilateral tubal ligation).
D They can be considered effective immediately in males (vasectomy).
ANSWER:C
382
A couple presents to your office to discuss sterilization. They are very happy with their four children and do not
want any morE. You discuss with them the pros and cons of both female and male sterilization. The 34-year- old
male undergoes a vasectomy. Which of the following is the most frequent immediate complication of this
procedure?

A Infection
B Impotence
C Hematoma
D Spontaneous reanastomosis
ANSWER:C
383
A woman with multiple sexual partners.For above female patient seeking contraception, select the method that is
medically contraindicated for that patient.
A Oral contraceptives
B IUD
C Condoms
D Laparoscopic tubal ligation
ANSWER:B
384
A woman with a history of deep vein thrombosis. For above female patient seeking contraception, select the
method that is medically contraindicated for that patient.
A Oral contraceptives
B IUD
C Condoms
D Laparoscopic tubal ligation
ANSWER:A
385
A woman with moderate cystocelE. For above female patient seeking contraception, select the method that is
medically contraindicated for that patient.
A Oral contraceptives
B IUD
C Condoms
D Diaphragm
ANSWER:D
386
A woman with severely reduced functional capacity as a result of chronic obstructive lung diseasE. For above
female patient seeking contraception, select the method that is medically contraindicated for that patient.
A Oral contraceptives
B IUD
C Condoms
D Laparoscopic tubal ligation
ANSWER:D

387
A woman with a known latex allergy. For above female patient seeking contraception, select the method that is
medically contraindicated for that patient.
A Oral contraceptives
B IUD
C Condoms
D Laparoscopic tubal ligation
ANSWER:C
388
Nausea during first cycle of pills. For above situation involving oral contraceptives, select the most appropriate
responsE.
A Stop pills and resume after 7 days.
B Continue pills as usual.
C Continue pills and use an additional form of contraception.
D Take an additional pill.
ANSWER:B
389 Pill forgotten for 1 day. For above situation involving oral contraceptives, select the most appropriate responsE.
A Stop pills and resume after 7 days.
B Continue pills as usual.
C Continue pills and use an additional form of contraception.
D Take an additional pill.
ANSWER:D
390
Pill forgotten for 3 continuous days. For above situation involving oral contraceptives, select the most appropriate
responsE.
A Stop pills and resume after 7 days.
B Continue pills as usual.
C Continue pills and use an additional form of contraception.
D Take an additional pill.
ANSWER:C
391
Light bleeding at midcycle during first month on pill. For above situation involving oral contraceptives, select the
most appropriate responsE.
A Stop pills and resume after 7 days.
B Continue pills as usual.
C Continue pills and use an additional form of contraception.
D Take an additional pill.

ANSWER:B
392
A 20-year-old woman presents to your office with the complaint of abdominal pain. Through further questioning,
the woman reveals that she was sexually assaulted at a party 3 weeks ago by a male friend whom she recently
started dating. She states that she has not revealed this to anyone else and has not informed the police because she
was drinking. Her abdominal and pelvic examinations are normal. Which of the following is the best management
to offer this patient?
A Counsel patient to sue male frienD.
B Provide an antidepressant.
C Provide emergency contraception.
D Test for and treat sexually transmitted infections.
ANSWER:D
393
You are called to the emergency department to evaluate an 18-year- old woman for a vulvar laceration. She is
accompanied by her mother and father. The father explains that the injury was caused by a fall onto the sup- port
bar on her bicyclE. You interview the woman alone and find out that her father has been sexually assaulting her.
Which of the following statements best describes injuries related to sexual assault?
A Most injuries are considered major and require surgical correction.
B Most injuries require hospitalization.
C More than 50% of victims will have an injury.
D Vaginal and vulvar lacerations are common in virginal victims.
ANSWER:D
394
You are evaluating a 19-year-old woman for a sexual assault. She denies any medical problems or allergies to
medications. Her pregnancy test is negativE. Which of the following antibiotic prophylaxes do you recommend
for sexually transmitted infections?
A No antibiotic prophylaxis is indicated
B Flagyl 500 mg PO twice daily for 7 days
C Rocephin 250 mg IM
D Doxycycline 100 mg PO twice daily for 7 days plus Rocephin 250 mg IM
ANSWER:D
395
After your evaluation and treatment of a rape victim has been com- pleted, you discharge the patient to homE.
When is the best time to schedule a follow-up appointment for the patient?
A 24 to 48 hours
B 1 week
C 6 weeks

D 12 weeks
ANSWER:A
396
A 22-year-old woman comes to your office at 10 weeks of gestation for her initial prenatal visit. She has been
referred to you by friends who are your patients. She would like you to be her physician, deliver her baby, and
then care for her chilD. Her uterus feels 10 weeks by size on bimanual exam, and her blood pressure is 100/70
mmHg. All other aspects of the initial complete physical examination are normal. The patient asks you about birth
plans during her initial visit and inquires as to your attitudes toward pregnant couples who wish to participate in
decision making regarding the conduct of labor and delivery. How would you respond?
A birth plans are not a good idea; usually something goes wrong and the couple is disappointed
B birth plans are not a good idea; they frequently lead to unresolved guilt in the couple
C birth plans should be avoided; perinatal morbidity and mortality are usually increased
D
birth plans are a good idea; they involve the couple in the planning for their baby’s delivery and can be a very
important part of the prenatal, postnatal and postpartum care
ANSWER:D
397
A 22-year-old woman comes to your office at 10 weeks of gestation for her initial prenatal visit. She has been
referred to you by friends who are your patients. She would like you to be her physician, deliver her baby, and
then care for her chilD. Her uterus feels 10 weeks by size on bimanual exam, and her blood pressure is 100/70
mmHg. All other aspects of the initial complete physical examination are normal. The couple inquires about the
routine administration of intravenous (IV) fluids during labor. Concerning this issue, which of the following
statements is false?
A the use of routine IV fluids does not limit ambulation in the first stage of labor
B if epidural analgesia is to be administered, an IV line must be in place
C if the first stage of labor is prolonged, an IV line should be in place to prevent dehydration
D if a patient has a history of a severe postpartum hemorrhage, an IV lineshould be established
ANSWER:A
398
A 22-year-old woman comes to your office at 10 weeks of gestation for her initial prenatal visit. She has been
referred to you by friends who are your patients. She would like you to be her physician, deliver her baby, and
then care for her chilD. Her uterus feels 10 weeks by size on bimanual exam, and her blood pressure is 100/70
mmHg. All other aspects of the initial complete physical examination are normal. The patient tells you it is very
important to her that she has a “natural” delivery and does not want any narcotics or medications that will “hurt the
baby.” How do you respond?
A many women think they do not want narcotics or an epidural, but most change their mind
B intravenous narcotics are totally safe and have no complications

C epidural analgesia is no longer associated with any complications and is completely safe
D
massage, standing in a warm shower, alternating position, and walking with intermittent monitoring can all be used
to decrease the need for pain relief during labor
ANSWER:D
399
A 22-year-old woman comes to your office at 10 weeks of gestation for her initial prenatal visit. She has been
referred to you by friends who are your patients. She would like you to be her physician, deliver her baby, and
then care for her chilD. Her uterus feels 10 weeks by size on bimanual exam, and her blood pressure is 100/70
mmHg. All other aspects of the initial complete physical examination are normal.The couple has some specific
requests that they add to their birth plan at the 36-week visit, suggested by a frienD. Which of the following would
not be advisable?
A having the patient’s mother present for support
B allowing the father to cut the umbilical cord
C putting the baby to breast before giving vitamin K and eye ointment
D allowing all birth plan actions to take place regardless of any unexpected emergencies
ANSWER:A
400
A 22-year-old woman comes to your office at 10 weeks of gestation for her initial prenatal visit. She has been
referred to you by friends who are your patients. She would like you to be her physician, deliver her baby, and
then care for her chilD. Her uterus feels 10 weeks by size on bimanual exam, and her blood pressure is 100/70
mmHg. All other aspects of the initial complete physical examination are normal. The couple’s final question
concerns “routine episiotomy.” They have been told that the medical profession is “cut happy” and that the vast
majority of episiotomies are unnecessary. Which of the following statements regarding routine episiotomy is true?
A episiotomy pain may be more severe and last longer than the pain from perineal lacerations
B episiotomy repairs heal more rapidly than do vaginal and perineal tears
C dyspareunia is more common after vaginal lacerations and perineal tear than after episiotomy
D episiotomy reduces the rate of subsequent pelvic relaxation problems
ANSWER:A
401
A 22-year-old woman comes to your office at 10 weeks of gestation for her initial prenatal visit. She has been
referred to you by friends who are your patients. She would like you to be her physician, deliver her baby, and
then care for her chilD. Her uterus feels 10 weeks by size on bimanual exam, and her blood pressure is 100/70
mmHg. All other aspects of the initial complete physical examination are normal.7. Which of the following
statements regarding the presence or absence of a supportive person (or coach) in labor is true?
A the presence of a support person or coach decreases the need for analgesia in labor

B
the presence of a support person or coach decreases the need for operative interventions such as forceps or
vacuum extraction
C the presence of a support person or coach decreases the cesarean delivery rate
D the presence of a support person increases the risk of malpractice-related lawsuits
ANSWER:A
402
A 30-year-old woman, on her first office visit, expresses her wish to become pregnant. She has never been
pregnant, denies any chronic health conditions, is not overweight, and exercises regularly. As you seek further
information to assist you in providing care, the following issues arise. Which of the following presents a risk
during the pregnancy?
A history of smoking with discontinuation 2 months ago
B history of spousal abuse
C family history of cardiovascular disease
D current use of one or two drinks after work three or four times a week
ANSWER:B
403
A 30-year-old woman, on her first office visit, expresses her wish to become pregnant. She has never been
pregnant, denies any chronic health conditions, is not overweight, and exercises regularly. As you seek further
information to assist you in providing care, the following issues arisE. During the physical examination, which of
the following findings increase(s) the risk of the pregnancy?
A elevated blood pressure
B a retroverted uterus
C a thrombosed hemorrhoid
D All of the above
ANSWER:A
404
A 30-year-old woman, on her first office visit, expresses her wish to become pregnant. She has never been
pregnant, denies any chronic health conditions, is not overweight, and exercises regularly. As you seek further
information to assist you in providing care, the following issues arisE. The patient relates to you that her sister had
a baby who suffered from spina bifidA. She questions the efficacy of folic acid:
A folic acid is known to reduce the occurrence of neural tube defects (NTDs) by60% to 70%
B the amount of folic acid found in prenatal vitamins is sufficient in this case
C
the fetus of this patient has a high risk of developing NTDs; therefore, she should start taking folic acid 12 months
before intended pregnancy
D folic acid reduces the recurrence of NTDs but does not prevent first occurrences
ANSWER:A

405
Primigravida at 8 Weeks of Gestation. A 24-year-old primigravida comes to your office at 8 weeks of gestation for
her first prenatal visit. She has asked you to be her family doctor and to look after her during the entire
pregnancy. You agree to provide her pregnancy carE. During your first visit, you explain your general
philosophy regarding prenatal care and perinatal carE. Which of the following regarding routine prenatal care is
true?
A that the number of routine office visits be significantly reduced for women at low risk
B
that focus should be on the total health and well-being of the family, including medical, psychological, social, and
environmental barriers affecting health
C
that provision of systematic health care start long before pregnancy because it was proved to be beneficial to the
physical and emotional well-being of the prospective mother and child
D All of the above
ANSWER:D
406
A 24-year-old primigravida comes to your office at 8 weeks of gestation for her first prenatal visit. She has asked
you to be her family doctor and to look after her during the entire pregnancy. You agree to provide her
pregnancy carE. During your first visit, you explain your general philosophy regarding prenatal care and perinatal
carE. Your patient had the first day of her last menstrual period on September 9, 2006. According to Nägele’s
rule, what is the patient’s estimated date of delivery A.ssume a 28-day cycle)?
A 02-Jun-07
B 16-Jun-07
C 02-Jul-07
D July 9, 2007
ANSWER:B
407
A 24-year-old primigravida comes to your office at 8 weeks of gestation for her first prenatal visit. She has asked
you to be her family doctor and to look after her during the entire pregnancy. You agree to provide her
pregnancy carE. During your first visit, you explain your general philosophy regarding prenatal care and perinatal
carE. Which of the following tests are not recommended at her initial prenatal visit?
A complete blood count C.BC)
B rapid plasma reagin (screening for syphilis)
C screening for gestational diabetes
D hepatitis B virus screen
ANSWER:C

408
After discussion of the visits and tests that will be done during the pregnancy, your patient indicates that she is
concerned about “getting fat.” She has been a smoker since age 16 years in an attempt to remain thin and she
assumes she will continue to smoke after she delivers. She is concerned about breast-feeding for the same reason
since people have told her that she would “have to eat for the baby” if she breast-feeds. She is 1.55 m tall and
weighs 59 kg. How much weight gain do you recommend?
A 2.25 to 7 kg
B 4.5 to 9 kg
C 6.8 to 11.4 kg
D 11.4 to 16 kg
ANSWER:D
409
After discussion of the visits and tests that will be done during the pregnancy, your patient indicates that she is
concerned about “getting fat.” She has been a smoker since age 16 years in an attempt to remain thin and she
assumes she will continue to smoke after she delivers. She is concerned about breast-feeding for the same reason
since people have told her that she would “have to eat for the baby” if she breast-feeds.17. In counseling your
patients on weight, which of the following is not a complication associated with excessive weight gain?
A infant macrosomia
B gestational diabetes
C shoulder dystocia
D intrauterine growth retardation
ANSWER:D
410
After discussion of the visits and tests that will be done during the pregnancy, your patient indicates that she is
concerned about “getting fat.” She has been a smoker since age 16 years in an attempt to remain thin and she
assumes she will continue to smoke after she delivers. She is concerned about breast-feeding for the same reason
since people have told her that she would “have to eat for the baby” if she breast-feeds.18. Which of the following
statements would not be regarded as reasonable nutritional advice for your patient?
A supplementation with iron if anemia is detected
B supplementation with folic acid 1 mg daily throughout pregnancy
C supplementation with vitamin A
D supplementation with calcium (Recommended Dietary Allowance, 1000 to
ANSWER:C

411
A Pregnant Woman Concerned about Consuming Ibuprofen A newly pregnant woman who comes to the office is
very concerned about the fact that she took ibuprofen for a headache during the past week. She is asking whether
it is safe to take during pregnancy. Which of the following statements about taking ibuprofen during pregnancy is
true?
A ibuprofen is considered safe during all stages of pregnancy
B
ibuprofen is considered relatively safe during the first and second trimester but should be avoided if possible in the
third trimester
C ibuprofen should never be taken during pregnancy; the patient should be counseled to consult a geneticist
D ibuprofen can be taken in the first trimester but should be avoided if possible in the second trimester
ANSWER:D
412
An 18-year-old primigravida comes to your office for her initial prenatal visit. The pregnancy was unanticipated
and she is quite disconcerteD. She denies any medical problems or prior surgery. Her body mass index is 29. She
has been taking prenatal vitamins for 1 month. Her mother hands you a list of symptoms that are bothering her
daughter. The patient is quite nauseated and “throws up constantly.” She dramatically states that she has lost “at
least 10 pounds in the past 6 weeks.” Neither “preggie pops” nor the “wrist bands she bought at the pharmacy”
help. She desires other options for ending the nauseA. Other complaints include blurred vision, bleeding gums,
and a vaginal dischargE. Her mother is concerned that she contracted a sexually transmitted disease from her
boyfrienD. The patient informs you that she had a well woman exam 2 months ago. The Pap exam was normal
and cultures for gonorrhea and chlamydia were negativE. On physical examination, the patient is well hydrated
and has actually gained 6 pounds. The uterus is 10 weeks’ sizE. The cervix is closed, firm, and not effaceD. There
is a whitish copious discharge but no odor or cervical motion tenderness. The remainder of her physical exam,
including a urinalysis, is normal. Which of the following hormones is thought to have the greatest influence on
nausea and vomiting in pregnancy (NVP)?
A progesterone
B estrogen
C thyroid-stimulating hormone
D human chorionic gonadotropin (hCG)
ANSWER:D

413
An 18-year-old primigravida comes to your office for her initial prenatal visit. The pregnancy was unanticipated
and she is quite disconcerteD. She denies any medical problems or prior surgery. Her body mass index is 29. She
has been taking prenatal vitamins for 1 month. Her mother hands you a list of symptoms that are bothering her
daughter. The patient is quite nauseated and “throws up constantly.” She dramatically states that she has lost “at
least 10 pounds in the past 6 weeks.” Neither “preggie pops” nor the “wrist bands she bought at the pharmacy”
help. She desires other options for ending the nauseA. Other complaints include blurred vision, bleeding gums,
and a vaginal dischargE. Her mother is concerned that she contracted a sexually transmitted disease from her
boyfrienD. The patient informs you that she had a well woman exam 2 months ago. The Pap exam was normal
and cultures for gonorrhea and chlamydia were negativE. On physical examination, the patient is well hydrated
and has actually gained 6 pounds. The uterus is 10 weeks’ sizE. The cervix is closed, firm, and not effaceD. There
is a whitish copious discharge but no odor or cervical motion tenderness. The remainder of her physical exam,
including a urinalysis, is normal. Which of the following would not be indicated as initial advice or treatment for
women with NVP?
A eating dry, carbohydrate-rich foods and drinking clear liquids may help alleviate symptoms
B providing the patient with a prescription for an antiemetic (i.E., promethazine)
C avoiding foods with strong seasoning or odors
D informing the patient that symptoms usually resolve at approximately 14 weeks of gestation
ANSWER:B
414
An 18-year-old primigravida comes to your office for her initial prenatal visit. The pregnancy was unanticipated
and she is quite disconcerteD. She denies any medical problems or prior surgery. Her body mass index is 29. She
has been taking prenatal vitamins for 1 month. Her mother hands you a list of symptoms that are bothering her
daughter. The patient is quite nauseated and “throws up constantly.” She dramatically states that she has lost “at
least 10 pounds in the past 6 weeks.” Neither “preggie pops” nor the “wrist bands she bought at the pharmacy”
help. She desires other options for ending the nauseA. Other complaints include blurred vision, bleeding gums,
and a vaginal dischargE. Her mother is concerned that she contracted a sexually transmitted disease from her
boyfrienD. The patient informs you that she had a well woman exam 2 months ago. The Pap exam was normal
and cultures for gonorrhea and chlamydia were negativE. On physical examination, the patient is well hydrated
and has actually gained 6 pounds. The uterus is 10 weeks’ sizE. The cervix is closed, firm, and not effaceD. There
is a whitish copious discharge but no odor or cervical motion tenderness. The remainder of her physical exam,
including a urinalysis, is normal. Which of the following remedies is no more effective than placebo in reducing
symptoms of NVP?
A pyridoxine (vitamin B6)
B P6 acupressure

C ginger capsules
D antiemetics (promethazine)
ANSWER:B
415
An 18-year-old primigravida comes to your office for her initial prenatal visit. The pregnancy was unanticipated
and she is quite disconcerteD. She denies any medical problems or prior surgery. Her body mass index is 29. She
has been taking prenatal vitamins for 1 month. Her mother hands you a list of symptoms that are bothering her
daughter. The patient is quite nauseated and “throws up constantly.” She ramatically states that she has lost “at least
10 pounds in the past 6 weeks.” Neither “preggie pops” nor the “wrist bands she bought at the pharmacy” help.
She desires other options for ending the nauseA. Other complaints include blurred vision, bleeding gums, and a
vaginal dischargE. Her mother is concerned that she contracted a sexually transmitted disease from her boyfrienD.
The patient informs you that she had a well woman exam 2 months ago. The Pap exam was normal and cultures
for gonorrhea and chlamydia were negativE. On physical examination, the patient is well hydrated and has
actually gained 6 pounds. The uterus is 10 weeks’ sizE. The cervix is closed, firm, and not effaceD. There is a
whitish copious discharge but no odor or cervical motion tenderness. The remainder of her physical exam,
including a urinalysis, is normal. How should the patient be counseled regarding her vaginal discharge?
A she was likely exposed to gonorrhea or chlamydia in the past 2months
B decreased estrogen and vaginal blood flow in pregnancy contributes to leukorrhea of pregnancy
C foul-smelling discharge, dysuria, and pruritis are not associated with leukorrhea
D leukorrhea of pregnancy is usually blood tinged and of thick consistency
ANSWER:C
416
A 36-year-old multigravida at 34 weeks of gestation. She works as a stockbroker at a large brokerage housE.
During the past 2 weeks, she has developed worsening edema in her bilateral lower extremities. It is worse at the
end of the day and generally resolves somewhat by the next morning. Although XYZ has made some lifestyle
changes (she no longer wears high heels to work), the symptoms are getting worsE. At her routine visit, she is
concerned about “severe abdominal pain.” She describes the pain as inguinal, stabbing, and intermittent. XYZ
comments that she also has significant low back pain. The pain is dull, constant, and located over the lower lumbar
spinE. She has no loss of bladder or bowel function and no neurologic abnormalities on exam. The low back pain
is not related to the inguinal pain. Which of the following statements about lower extremity edema during
pregnancy is true?
A avoiding standing for long periods of time improves symptoms
B decreased sodium and water retention leads to fluid shifts
C decreased vascular permeability worsens dependent edema

D lower extremity pitting edema late in pregnancy is highly suggestive
ANSWER:A
417
A 34-year-old female (gravida 2, para 1) presents to the clinic with bleeding during pregnancy. She reports that it
has been 6 weeks since her last menstrual perioD. She had a positive home pregnancy test 1 week ago and is
scheduled for her first obstetrical appointment in 3 weeks. She is complaining of light vaginal bleeding without
abdominal cramping or backachE. She states that her symptoms began this morning. She has no orthostatic
symptoms. There are no other systemic symptoms, including fever, abdominal pain, or vomiting. Her previous
medical and obstetrical history is uncomplicateD. Physical examination shows that she is tearful. Vital signs reveal
temperature 97.8°F, pulse 76 beats/minute, blood pressure 126/78 mmHg, and respiratory rate 20 breaths/minutE.
Her vital signs do not significantly change with orthostatic testing. Her abdomen is soft and flat. She has active
bowel sounds. Pelvic examination shows a small amount of bright red bleeding coming from the cervical os. The
uterus is parous and consistent with her dating history. Adnexal structures are normal to bimanual exam. Her urine
pregnancy test is positivE. Vaginal bleeding in pregnancy before 20 weeks of gestation is defined as
A complete abortion
B threatened abortion
C incomplete abortion
D inevitable abortion
ANSWER:B
418
A 34-year-old female (gravida 2, para 1) presents to the clinic with bleeding during pregnancy. She reports that it
has been 6 weeks since her last menstrual perioD. She had a positive home pregnancy test 1 week ago and is
scheduled for her first obstetrical appointment in 3 weeks. She is complaining of light vaginal bleeding without
abdominal cramping or backachE. She states that her symptoms began this morning. She has no orthostatic
symptoms. There are no other systemic symptoms, including fever, abdominal pain, or vomiting. Her previous
medical and obstetrical history is uncomplicateD. Physical examination shows that she is tearful. Vital signs reveal
temperature 97.8°F, pulse 76 beats/minute, blood pressure 126/78 mmHg, and respiratory rate 20 breaths/minutE.
Her vital signs do not significantly change with orthostatic testing. Her abdomen is soft and flat. She has active
bowel sounds. Pelvic examination shows a small amount of bright red bleeding coming from the cervical os. The
uterus is parous and consistent with her dating history. Adnexal structures are normal to bimanual exam. Her urine
pregnancy test is positivE.Which of the following conditions is the most common complication of a recognized
pregnancy in Cambodia?
A diabetes
B threatened abortion

C incomplete abortion
D hypertension
ANSWER:B
419
A 34-year-old female (gravida 2, para 1) presents to the clinic with bleeding during pregnancy. She reports that it
has been 6 weeks since her last menstrual perioD. She had a positive home pregnancy test 1 week ago and is
scheduled for her first obstetrical appointment in 3 weeks. She is complaining of light vaginal bleeding without
abdominal cramping or backachE. She states that her symptoms began this morning. She has no orthostatic
symptoms. There are no other systemic symptoms, including fever, abdominal pain, or vomiting. Her previous
medical and obstetrical history is uncomplicateD. Physical examination shows that she is tearful. Vital signs reveal
temperature 97.8°F, pulse 76 beats/minute, blood pressure 126/78 mmHg, and respiratory rate 20 breaths/minutE.
Her vital signs do not significantly change with orthostatic testing. Her abdomen is soft and flat. She has active
bowel sounds. Pelvic examination shows a small amount of bright red bleeding coming from the cervical os. The
uterus is parous and consistent with her dating history. Adnexal structures are normal to bimanual exam. Her urine
pregnancy test is positivE.In the management of this patient, you decide she is clinically stablE. The local hospital
is able to provide timely testing for you. Which of the following tests is least helpful at this time?
A complete blood count
B quantitative human chorionic gonadotropin -hCG) level
C vaginal probe ultrasound examination
D vaginal pH testing
ANSWER:C

420
A 34-year-old female (gravida 2, para 1) presents to the clinic with bleeding during pregnancy. She reports that it
has been 6 weeks since her last menstrual perioD. She had a positive home pregnancy test 1 week ago and is
scheduled for her first obstetrical appointment in 3 weeks. She is complaining of light vaginal bleeding without
abdominal cramping or backachE. She states that her symptoms began this morning. She has no orthostatic
symptoms. There are no other systemic symptoms, including fever, abdominal pain, or vomiting. Her previous
medical and obstetrical history is uncomplicateD. Physical examination shows that she is tearful. Vital signs reveal
temperature 97.8°F, pulse 76 beats/minute, blood pressure 126/78 mmHg, and respiratory rate 20 breaths/minutE.
Her vital signs do not significantly change with orthostatic testing. Her abdomen is soft and flat. She has active
bowel sounds. Pelvic examination shows a small amount of bright red bleeding coming from the cervical os. The
uterus is parous and consistent with her dating history. Adnexal structures are normal to bimanual exam. Her urine
pregnancy test is positivE.During a follow-up visit at your clinic, this patient notes that bleeding has stoppeD. She
has no pain or cramping. Her testing shows a quantitative -hCG level of 950 mIU/mL, and no gestational sac is
noted on pelvic ultrasounD. You decide to do the following:
A refer to surgery for ectopic pregnancy
B B.repeat quantitative -hCG level in 48 hours
C inform the patient that she likely has completed her miscarriage, and no further workup is needed
D inform the patient that she has a nonviable pregnancy
ANSWER:B
421
A 28-year-old (gravida 1, para 0) patient comes to see you for a follow-up clinic visit. She experienced vaginal
bleeding in early pregnancy. Initially, she presented with light vaginal bleeding at 10 weeks of gestation. Her
initial ultrasound was reassuring, with normal fetal growth and definite heartbeat. A few days later, she began
having heavy bleeding. Follow-up testing showed an incomplete abortion. You discussed surgical, medical, and
expectant management. She chose expectant management and is here for follow-up. Many women choose
expectant management for spontaneous abortion. Which of the following statements is true when comparing
expectant management with surgical management of a spontaneous abortion?
A surgical procedure
B women tend to experience more bleeding with surgical treatment of spontaneous abortion
C
women with very heavy bleeding and orthostatic symptoms can be managed expectantly as long as good follow-
up is available
D more women undergoing expectant management will experience incomplete abortion
ANSWER:D

422
A 28-year-old (gravida 1, para 0) patient comes to see you for a follow-up clinic visit. She experienced vaginal
bleeding in early pregnancy. Initially, she presented with light vaginal bleeding at 10 weeks of gestation. Her
initial ultrasound was reassuring, with normal fetal growth and definite heartbeat. A few days later, she began
having heavy bleeding. Follow-up testing showed an incomplete abortion. You discussed surgical, medical, and
expectant management. She chose expectant management and is here for follow-up. Medical regimens exist as
treatment options for spontaneous abortion. Misoprostol is part of many of these regimens. Which of the following
statements is A.re) true regarding use of misoprostol in the medical management of spontaneous abortion?
A misoprostol is Food and Drug Administration (FDA) approved for labor induction of term pregnancies
B misoprostol is FDA approved for medical management of spontaneous abortion
C misoprostol can cause gastrointestinal side effects, including nausea and diarrhea
D
there is a minimal risk of pelvic cramping when using oral misoprostol for medical management of spontaneous
abortion
ANSWER:C
423
A 28-year-old (gravida 1, para 0) patient comes to see you for a follow-up clinic visit. She experienced vaginal
bleeding in early pregnancy. Initially, she presented with light vaginal bleeding at 10 weeks of gestation. Her
initial ultrasound was reassuring, with normal fetal growth and definite heartbeat. A few days later, she began
having heavy bleeding. Follow-up testing showed an incomplete abortion. You discussed surgical, medical, and
expectant management. She chose expectant management and is here for follow-up. In an uncomplicated
pregnancy, which of the following factors does not increase the risk for spontaneous abortion?
A cigarette smoking
B sexual activity
C alcohol use
D advanced maternal age
ANSWER:B

424
A 28-year-old (gravida 1, para 0) patient comes to see you for a follow-up clinic visit. She experienced vaginal
bleeding in early pregnancy. Initially, she presented with light vaginal bleeding at 10 weeks of gestation. Her
initial ultrasound was reassuring, with normal fetal growth and definite heartbeat. A few days later, she began
having heavy bleeding. Follow-up testing showed an incomplete abortion. You discussed surgical, medical, and
expectant management. She chose expectant management and is here for follow-up.The patient continues to be
managed expectantly and experiences a completed spontaneous abortion without need for surgical
instrumentation. She is now concerned that she will experience recurrent abortion. What is the definition of
recurrent abortion?
A any number of spontaneous abortions that concern a patient
B two or more consecutive spontaneous abortions
C two or more nonconsecutive spontaneous abortions that occur during a patient’s lifetime
D three or more consecutive spontaneous abortions
ANSWER:D
425
A 28-year-old (gravida 1, para 0) patient comes to see you for a follow-up clinic visit. She experienced vaginal
bleeding in early pregnancy. Initially, she presented with light vaginal bleeding at 10 weeks of gestation. Her
initial ultrasound was reassuring, with normal fetal growth and definite heartbeat. A few days later, she began
having heavy bleeding. Follow-up testing showed an incomplete abortion. You discussed surgical, medical, and
expectant management. She chose expectant management and is here for follow-up.In the 6 months following
miscarriage, women are at increased risk for which of the following disorders?
A depressive disorder
B anxiety disorder
C obsessivecompulsive disorder
D All of the above
ANSWER:D
426
The patient is a 23-year-old woman whose family has a history of diabetes mellitus. She is currently 28 weeks of
gestation. Your patient goes into labor at 40 weeks of gestation, gradually increasing to fully dilateD. She then
pushes for 3 hours until you elect to do a vacuum extraction because of maternal exhaustion. You notice
immediate “turtling” of the infant’s heaD. The following are appropriate steps in using a vacuum extractor except:
A applying the cup over the sagittal suture 3 cm in front of the posteriorfontanelle
B applying continuous pressure against the vacuum until it disengages three times
C halting the procedure if there is no progress after three consecutive pulls
D releasing the vacuum when the jaw is reachable

ANSWER:B
427
A Primigravida with Hypertension. A 35-year-old pregnant woman (gravida 2, para 1) comes into the office for
her 18-week prenatal appointment. Her blood pressure is 140/94 mmHg when taken by your nurse and is
confirmed by your own measurement. She has no protein in her urine and has no headaches, blurred vision,
nausea, or vomiting. The rest of the examination is consistent with dates; there is no lower extremity edemA. Her
diagnosis is
A chronic hypertension
B preeclampsia/eclampsia
C gestational hypertension
D labile hypertension
ANSWER:C
428
A Primigravida with Hypertension. A 35-year-old pregnant woman (gravida 2, para 1) comes into the office for
her 18-week prenatal appointment. Her blood pressure is 140/94 mmHg when taken by your nurse and is
confirmed by your own measurement. She has no protein in her urine and has no headaches, blurred vision,
nausea, or vomiting. The rest of the examination is consistent with dates; there is no lower extremity edemA.The
patient wants to know if she has an increased risk in this pregnancy. Your explain that: pregnancy complicated by
chronic hypertension can be easily
A pregnancy complicated by chronic hypertension can be easily managed
B
she has an increased risk of preeclampsia, eclampsia, intrauterine growth restriction (IUGR), cesarean section, and
bleeding
C
with ultrasound monitoring as well as frequent benign prostatic hyperplasia B.PH) symptom index scores she will
be safe
D chronic hypertension is not related to eclampsia
ANSWER:B
429
A Primigravida with Hypertension. A 35-year-old pregnant woman (gravida 2, para 1) comes into the office for
her 18-week prenatal appointment. Her blood pressure is 140/94 mmHg when taken by your nurse and is
confirmed by your own measurement. She has no protein in her urine and has no headaches, blurred vision,
nausea, or vomiting. The rest of the examination is consistent with dates; there is no lower extremity edemA. The
appropriate course of action to evaluate her elevated blood pressure includes.
A blood clotting studies, lactic acid dehydrogenase level
B starting her on an angiotensin-converting enzyme A.CE) inhibitor
C starting her on Aldomet (methyldopa)
D watchful waiting

ANSWER:D
430
A Primigravida with Hypertension. A 35-year-old pregnant woman (gravida 2, para 1) comes into the office for
her 18-week prenatal appointment. Her blood pressure is 140/94 mmHg when taken by your nurse and is
confirmed by your own measurement. She has no protein in her urine and has no headaches, blurred vision,
nausea, or vomiting. The rest of the examination is consistent with dates; there is no lower extremity edemA.Your
patient calls you at night complaining of a severe headache and thinks she is seeing “doublE.” She is now 30
weeks pregnant. You tell her to go to the emergency room. Your presumptive diagnosis is
A transient ischemic attack in pregnancy
B preeclampsia superimposed on chronic hypertension
C eclampsia
D hemolysis, elevated liver enzymes, low platelet count (HELLP) syndrome
ANSWER:B
431
A Primigravida with Hypertension. A 35-year-old pregnant woman (gravida 2, para 1) comes into the office for
her 18-week prenatal appointment. Her blood pressure is 140/94 mmHg when taken by your nurse and is
confirmed by your own measurement. She has no protein in her urine and has no headaches, blurred vision,
nausea, or vomiting. The rest of the examination is consistent with dates; there is no lower extremity edemA.The
patient’s blood pressure in the emergency room is 160/110 mmHg, and she has severe pedal edema and
hyperflexiA. You will
A hospitalize the patient, start her on hydralazine, and draw lab tests
B
hospitalize her for observation and start her on hydralazinintravenously (IV); do a complete ultrasound and
biophysical profile
C at this time, there is no laboratory evidence of preeclampsia, so she should be treated as an outpatient.
D her edema and hyperflexia are sufficient evidence of her severe preeclampsia
ANSWER:B
432
A Primigravida with Hypertension. A 35-year-old pregnant woman (gravida 2, para 1) comes into the office for
her 18-week prenatal appointment. Her blood pressure is 140/94 mmHg when taken by your nurse and is
confirmed by your own measurement. She has no protein in her urine and has no headaches, blurred vision,
nausea, or vomiting. The rest of the examination is consistent with dates; there is no lower extremity edemA.Fetal
indications for delivery of this patient’s baby include all the following except:
A signs of IUGR
B suspected abruptio placentae
C oligohydramnios
D an amniotic fluid index of 10

ANSWER:D
433
A Primigravida with Hypertension. A 35-year-old pregnant woman (gravida 2, para 1) comes into the office for
her 18-week prenatal appointment. Her blood pressure is 140/94 mmHg when taken by your nurse and is
confirmed by your own measurement. She has no protein in her urine and has no headaches, blurred vision,
nausea, or vomiting. The rest of the examination is consistent with dates; there is no lower extremity
edemA.Treatment of acute severe hypertension (sustained blood pressures higher than 160 systolic and 105
diastolic) in pregnancy include the following except:
A labetalol (Normodyne) 20 mg
B nifedipine (Procardia) 10 mg orally
C hydralazine A.presoline) 5 mg IV
D methyldopa A.ldomet) 250 mg orally
ANSWER:D
434
A Primigravida with Hypertension. A 35-year-old pregnant woman (gravida 2, para 1) comes into the office for
her 18-week prenatal appointment. Her blood pressure is 140/94 mmHg when taken by your nurse and is
confirmed by your own measurement. She has no protein in her urine and has no headaches, blurred vision,
nausea, or vomiting. The rest of the examination is consistent with dates; there is no lower extremity edemA.The
patient complains of a severe headache during labor, her blood pressure climbs to 150/100 mmHg, and she now
has 3+ protein on a urine sample collected by the nursE. The most appropriate treatment for your patient at this
time would be
A labetalol (Normodyne) 20 mg IV
B magnesium sulfate 2-g loading dose and then run at 1 g/hour
C magnesium sulfate 4-g loading dose and then run at 2 g/hour
D hydralazine A.presoline) 10 mg IM
ANSWER:C
435
A Primigravida with Hypertension. A 35-year-old pregnant woman (gravida 2, para 1) comes into the office for
her 18-week prenatal appointment. Her blood pressure is 140/94 mmHg when taken by your nurse and is
confirmed by your own measurement. She has no protein in her urine and has no headaches, blurred vision,
nausea, or vomiting. The rest of the examination is consistent with dates; there is no lower extremity edemA.Which
of the following statements regarding eclampsia is true?
A eclampsia should be treated with intravenous diazepam
B eclampsia may occur with a diastolic blood pressure less than 90 mmHg
C eclamptic seizures frequently occur during delivery
D phenytoin may be administered intravenously to a patient having a preeclamptic seizure

ANSWER:A
436
A Primigravida with Hypertension. A 35-year-old pregnant woman (gravida 2, para 1) comes into the office for
her 18-week prenatal appointment. Her blood pressure is 140/94 mmHg when taken by your nurse and is
confirmed by your own measurement. She has no protein in her urine and has no headaches, blurred vision,
nausea, or vomiting. The rest of the examination is consistent with dates; there is no lower extremity edemA.Which
one of the following intrapartum conditions is associated with preeclampsia/eclampsia?
A postpartum hemorrhage
B postdates pregnancy with induction
C maternal hyperglycemia
D prolonged first stage of labor
ANSWER:A
437
A Primigravida with Hypertension. A 35-year-old pregnant woman (gravida 2, para 1) comes into the office for
her 18-week prenatal appointment. Her blood pressure is 140/94 mmHg when taken by your nurse and is
confirmed by your own measurement. She has no protein in her urine and has no headaches, blurred vision,
nausea, or vomiting. The rest of the examination is consistent with dates; there is no lower extremity edemA.The
patient delivers vaginally. The following are considered steps to use in the active management of the third stage of
labor except
A administration of a uterine tonic prior to delivery of the infant
B administration of a uterine tonic prior to delivery of the placenta
C relatively rapid cord clamping and cutting
D application of controlled traction to the cord
ANSWER:A
438
MB is a 24-year-old (gravida 1, para 0) female who is 30 weeks pregnant. Her last menstrual period is certain. She
presented for care at 9 weeks of gestation and has kept her monthly follow-up appointments. Her initial body mass
index was 21, and she has gained 3kg during the past 8 weeks. Fundal height has been consistent with dates, but
today the fundal measurement is 25 cm. She smokes one and a half packs of cigarettes a day and is unable to cut
down. She denies alcohol or other substance usE. She denies any recent infections. An ultrasound at 16 weeks was
consistent with her last menstrual period and showed normal fetal anatomy. A repeat ultrasound shows estimated
fetal weight consistent with a 25-week gestation (fetal weight below the 10th percentile for 30-week gestation).
The amniotic fluid index is normal.Which of the following statements regarding intrauterine growth restriction
(IUGR) is true?
A the term describes a fetus with an estimated weight that is less than expected for gestational age
B the 3rd percentile is generally the cutoff used to define IUGR

C IUGR is interchangeable with the term small for gestational age (SGA)
D IUGR is a term for infants with genetic anomalies whose weight is at the low end of the growth curve
ANSWER:A
439
MB is a 24-year-old (gravida 1, para 0) female who is 30 weeks pregnant. Her last menstrual period is certain. She
presented for care at 9 weeks of gestation and has kept her monthly follow-up appointments. Her initial body mass
index was 21, and she has gained 3kg during the past 8 weeks. Fundal height has been consistent with dates, but
today the fundal measurement is 25 cm. She smokes one and a half packs of cigarettes a day and is unable to cut
down. She denies alcohol or other substance usE. She denies any recent infections. An ultrasound at 16 weeks was
consistent with her last menstrual period and showed normal fetal anatomy. A repeat ultrasound shows estimated
fetal weight consistent with a 25-week gestation (fetal weight below the 10th percentile for 30-week gestation).
The amniotic fluid index is normal.What is the leading cause of fetal growth restriction in human pregnancies?
A poor maternal weight gain
B placental insufficiency
C maternal diabetes
D maternal toxoplasmosis exposure
ANSWER:B
440
A 24-year-old (gravida 1, para 0) female who is 30 weeks pregnant. Her last menstrual period is certain. She
presented for care at 9 weeks of gestation and has kept her monthly follow-up appointments. Her initial body mass
index was 21, and she has gained 3kg during the past 8 weeks. Fundal height has been consistent with dates, but
today the fundal measurement is 25 cm. She smokes one and a half packs of cigarettes a day and is unable to cut
down. She denies alcohol or other substance usE. She denies any recent infections. An ultrasound at 16 weeks was
consistent with her last menstrual period and showed normal fetal anatomy. A repeat ultrasound shows estimated
fetal weight consistent with a 25-week gestation (fetal weight below the 10th percentile for 30-week gestation).
The amniotic fluid index is normal.All of the following types of maternal substance use are linked with IUGR
except
A marijuana
B tobacco
C methadone
D cocaine
ANSWER:A

441
A 35-year-old (gravida 1, para 0) female visits your office at 40 weeks of gestation. Her last menstrual period
(LMP) is “certain.” Her history is significant for regular menstrual cycles and no oral contraceptive use within 3
months of becoming pregnant. A 13-week ultrasound was consistent with her LMP. The pregnancy has been
unremarkablE. The patient is quite concerned because today is her due date and she is not in labor. She states that
her mother did not “go into labor until after 44 weeks” and she is worried that “late babies run in the family.” On
physical examination, fundal height is 39 cm. Her weight has increased by 1 pound and vital signs are stablE. Fetal
heart tones are 140. The cervix is closed, thick, and high. Estimated fetal weight is 3.5kg. Which of the following
gestational ages is considered post-term?
A 270 days
B 280 days
C 287 days
D 294 days
ANSWER:D
442
A 35-year-old (gravida 1, para 0) female visits your office at 40 weeks of gestation. Her last menstrual period
(LMP) is “certain.” Her history is significant for regular menstrual cycles and no oral contraceptive use within 3
months of becoming pregnant. A 13-week ultrasound was consistent with her LMP. The pregnancy has been
unremarkablE. The patient is quite concerned because today is her due date and she is not in labor. She states that
her mother did not “go into labor until after 44 weeks” and she is worried that “late babies run in the family.” On
physical examination, fundal height is 39 cm. Her weight has increased by 1 pound and vital signs are stablE. Fetal
heart tones are 140. The cervix is closed, thick, and high. Estimated fetal weight is 3.5kg.Which of the following
conditions is not associated with increased risk of post-term pregnancy?
A placental sulfatase insufficiency
B fetal anencephaly
C female gender of fetus
D primiparity
ANSWER:C

443
A 35-year-old (gravida 1, para 0) female visits your office at 40 weeks of gestation. Her last menstrual period
(LMP) is “certain.” Her history is significant for regular menstrual cycles and no oral contraceptive use within 3
months of becoming pregnant. A 13-week ultrasound was consistent with her LMP. The pregnancy has been
unremarkablE. The patient is quite concerned because today is her due date and she is not in labor. She states that
her mother did not “go into labor until after 44 weeks” and she is worried that “late babies run in the family.” On
physical examination, fundal height is 39 cm. Her weight has increased by 1 pound and vital signs are stablE. Fetal
heart tones are 140. The cervix is closed, thick, and high. Estimated fetal weight is 3.5kg.What is the most
common cause of post-term pregnancy?
A incorrect dating
B fetal anencephaly
C genetic predisposition
D multiparity
ANSWER:A
444
A 35-year-old (gravida 1, para 0) female visits your office at 40 weeks of gestation. Her last menstrual period
(LMP) is “certain.” Her history is significant for regular menstrual cycles and no oral contraceptive use within 3
months of becoming pregnant. A 13-week ultrasound was consistent with her LMP. The pregnancy has been
unremarkablE. The patient is quite concerned because today is her due date and she is not in labor. She states that
her mother did not “go into labor until after 44 weeks” and she is worried that “late babies run in the family.” On
physical examination, fundal height is 39 cm. Her weight has increased by 1 pound and vital signs are stablE. Fetal
heart tones are 140. The cervix is closed, thick, and high. Estimated fetal weight is 3.5kg.The patient progresses to
41 weeks. She now wants to be managed expectantly because she heard that the risk of cesarean is higher if she is
induceD. Which of the following statements about labor induction at 41 weeks is not true?
A induction at 41 weeks does not increase the risk of cesarean
B post-term induction of labor reduces the risk of perinatal death
C the reduction in risk of perinatal death with induction is very small compared to that of expectant management
D induction at 41 weeks increases the risk for cesarean
ANSWER:D

445
A 35-year-old (gravida 1, para 0) female visits your office at 40 weeks of gestation. Her last menstrual period
(LMP) is “certain.” Her history is significant for regular menstrual cycles and no oral contraceptive use within 3
months of becoming pregnant. A 13-week ultrasound was consistent with her LMP. The pregnancy has been
unremarkablE. The patient is quite concerned because today is her due date and she is not in labor. She states that
her mother did not “go into labor until after 44 weeks” and she is worried that “late babies run in the family.” On
physical examination, fundal height is 39 cm. Her weight has increased by 1 pound and vital signs are stablE. Fetal
heart tones are 140. The cervix is closed, thick, and high. Estimated fetal weight is 3.5kg.The patient agrees to
proceed with labor induction. Her cervical exam at this time is as follows: 2-cm dilation, 60% effacement, −2
station, firm consistency, and posterior. Which of the following statements is correct?
A an oxytocin induction is indicated
B the Bishop score indicates a high likelihood of vaginal delivery with induction
C cervical ripening with prostaglandins is indicated
D the chance of successful induction with prostaglandins is low
ANSWER:C
446
A 35-year-old (gravida 1, para 0) female visits your office at 40 weeks of gestation. Her last menstrual period
(LMP) is “certain.” Her history is significant for regular menstrual cycles and no oral contraceptive use within 3
months of becoming pregnant. A 13-week ultrasound was consistent with her LMP. The pregnancy has been
unremarkablE. The patient is quite concerned because today is her due date and she is not in labor. She states that
her mother did not “go into labor until after 44 weeks” and she is worried that “late babies run in the family.” On
physical examination, fundal height is 39 cm. Her weight has increased by 1 pound and vital signs are stablE. Fetal
heart tones are 140. The cervix is closed, thick, and high. Estimated fetal weight is 3.5kg.Which of the following
statements about the use of prostaglandin for cervical ripening is incorrect?
A pitocin is contraindicated if prostaglandins are used during an induction for post-term pregnancy
B prostaglandin E2 D.inoprostone) and prostaglandin E1 (misoprostol) are two options for post-term induction
C no standardized dosing regimen is established for these medications
D higher doses are associated with uterine hyperstimulation
ANSWER:C
447
An 18-year-old (gravida 1) female at 39 weeks and 5 days gestation. The pregnancy has been uneventful. She
arrives on the floor with her mother, boyfriend, and two friends. Her presenting complaint is contractions for 3
hours. The contractions are 5 minutes apart and irregular. She denies bleeding, fluid leakage, or decreased fetal
movement. On physical examination, her cervix is dilated to 3 cm and is 20% effaced, firm, and posterior. A non-
stress test is reassuring. She is monitored for 2 hours and has no significant cervical changE. 59. Women admitted
to labor and delivery in this patient’s stage of labor are at increased risk for all of the following except

A cesarean delivery
B shoulder dystocia
C amnionitis
D intrauterine pressure catheter placement
ANSWER:B
448
An 18-year-old (gravida 1) female at 39 weeks and 5 days gestation. The pregnancy has been uneventful. She
arrives on the floor with her mother, boyfriend, and two friends. Her presenting complaint is contractions for 3
hours. The contractions are 5 minutes apart and irregular. She denies bleeding, fluid leakage, or decreased fetal
movement. On physical examination, her cervix is dilated to 3 cm and is 20% effaced, firm, and posterior. A non-
stress test is reassuring. She is monitored for 2 hours and has no significant cervical changE.What is the working
diagnosis at this time?
A active labor
B failure to progress
C latent labor
D Braxton–Hicks contractions
ANSWER:C
449
An 18-year-old (gravida 1) female at 39 weeks and 5 days gestation. The pregnancy has been uneventful. She
arrives on the floor with her mother, boyfriend, and two friends. Her presenting complaint is contractions for 3
hours. The contractions are 5 minutes apart and irregular. She denies bleeding, fluid leakage, or decreased fetal
movement. On physical examination, her cervix is dilated to 3 cm and is 20% effaced, firm, and posterior. A non-
stress test is reassuring. She is monitored for 2 hours and has no significant cervical changE.Which of the
following outcomes is not associated with continuity of care during pregnancy?
A women require less medication for pain relief in labor
B neonates are less likely to require resuscitation at delivery
C women are more likely satisfied with their intrapartum care
D operative vaginal delivery is more common
ANSWER:D

450
The patient is sent homE. She returns 2 days later with continued contractions that are now 3 minutes apart and
regular. On sterile vaginal exam, her cervix is 4 cm dilated, 50% effaced, −2 position, mid-station, and soft. The
patient is admitted to labor and delivery. Her mother and boyfriend are quite excited and want to know exactly
when the baby will deliver. The patient wants to talk about whether an epidural is a good ideA. Her mother wants
to know when her daughter will receive an enema and “be shaveD.” She also warns the patient that an episiotomy
is required for the baby to deliver safely. What is the expected rate of cervical dilatation during active labor in
nulliparous women?
A 0.2 cm/hour
B 0.5 cm/hour
C 1 cm/hour
D 1.5 cm/hour
ANSWER:C
451
The patient is sent homE. She returns 2 days later with continued contractions that are now 3 minutes apart and
regular. On sterile vaginal exam, her cervix is 4 cm dilated, 50% effaced, −2 position, mid-station, and soft. The
patient is admitted to labor and delivery. Her mother and boyfriend are quite excited and want to know exactly
when the baby will deliver. The patient wants to talk about whether an epidural is a good ideA. Her mother wants
to know when her daughter will receive an enema and “be shaveD.” She also warns the patient that an episiotomy
is required for the baby to deliver safely.Which statement regarding active management of labor is false?
A early amniotomy and oxytocin are performed to correct prolonged labor
B it reduces the duration of labor
C interventions are triggered if cervical progress deviates more than 2 hours from the normal progress line
D interventions are indicated in primiparas without adequate cervical change after a 4-hour period
ANSWER:D
452
The patient is sent homE. She returns 2 days later with continued contractions that are now 3 minutes apart and
regular. On sterile vaginal exam, her cervix is 4 cm dilated, 50% effaced, −2 position, mid-station, and soft. The
patient is admitted to labor and delivery. Her mother and boyfriend are quite excited and want to know exactly
when the baby will deliver. The patient wants to talk about whether an epidural is a good ideA. Her mother wants
to know when her daughter will receive an enema and “be shaveD.” She also warns the patient that an episiotomy
is required for the baby to deliver safely.Which of these types of general care during labor is supported by
evidence-based studies?
A perineal shaving
B routine enemas

C restriction of oral fluid and food intake
D continuous support during labor
ANSWER:D
453
The patient is sent homE. She returns 2 days later with continued contractions that are now 3 minutes apart and
regular. On sterile vaginal exam, her cervix is 4 cm dilated, 50% effaced, −2 position, mid-station, and soft. The
patient is admitted to labor and delivery. Her mother and boyfriend are quite excited and want to know exactly
when the baby will deliver. The patient wants to talk about whether an epidural is a good ideA. Her mother wants
to know when her daughter will receive an enema and “be shaveD.” She also warns the patient that an episiotomy
is required for the baby to deliver safely.Which of the following statements regarding amniotomy is true?
A numerous studies support the benefit of amniotomy for augmentation of labor
B it is associated with increased need for oxytocin
C more mild and moderate variables are noted on external fetal monitoring in patients who undergo amniotomy
D it is associated with a 30-minute reduction in the duration of labor
ANSWER:C
454
The patient is sent homE. She returns 2 days later with continued contractions that are now 3 minutes apart and
regular. On sterile vaginal exam, her cervix is 4 cm dilated, 50% effaced, −2 position, mid-station, and soft. The
patient is admitted to labor and delivery. Her mother and boyfriend are quite excited and want to know exactly
when the baby will deliver. The patient wants to talk about whether an epidural is a good ideA. Her mother wants
to know when her daughter will receive an enema and “be shaveD.” She also warns the patient that an episiotomy
is required for the baby to deliver safely.How should expectant women be counseled regarding episiotomy?
A routine episiotomy facilitates delivery and is indicated to avoid perineal damage
B extension of the episiotomy into the rectum is very rare
C mediolateral episiotomy is superior to a midlateral approach
D episiotomy should only be performed for specific indications
ANSWER:D

455
The patient is sent homE. She returns 2 days later with continued contractions that are now 3 minutes apart and
regular. On sterile vaginal exam, her cervix is 4 cm dilated, 50% effaced, −2 position, mid-station, and soft. The
patient is admitted to labor and delivery. Her mother and boyfriend are quite excited and want to know exactly
when the baby will deliver. The patient wants to talk about whether an epidural is a good ideA. Her mother wants
to know when her daughter will receive an enema and “be shaveD.” She also warns the patient that an episiotomy
is required for the baby to deliver safely.Which of the following is not included in active management of the third
stage of labor?
A administration of oxytocin after delivery of the anterior shoulder
B controlled cord traction to expedite delivery of the placenta
C use of McRoberts’ maneuver to expedite delivery of the fetal head
D delivery of the placenta by maternal pushing
ANSWER:D
456
A 27-year-old female (gravida 2, para 1) at 39 weeks of gestation presents to labor and delivery in active labor.
Her pregnancy has been uncomplicated and her prior two deliveries were vaginal. Her cervix is checked by the
nurse and judged to be 6 cm, 90% effaced, midposition, and soft. The fetus is not engaged and is thought to be
vertex. Initial fetal monitoring shows a heart rate in the 140s with good accelerations and is reassuring.
Contractions are 4 minutes apart and the patient is comfortablE. Twenty minutes later, the patient experiences a
large gush of clear fluid, and severe variable decelerations appear on the fetal heart rate monitor. What is the most
likely diagnosis at this time?
A uterine rupture
B placental abruption
C placenta previa
D cord prolapse
ANSWER:D
457
A 27-year-old female (gravida 2, para 1) at 39 weeks of gestation presents to labor and delivery in active labor.
Her pregnancy has been uncomplicated and her prior two deliveries were vaginal. Her cervix is checked by the
nurse and judged to be 6 cm, 90% effaced, midposition, and soft. The fetus is not engaged and is thought to be
vertex. Initial fetal monitoring shows a heart rate in the 140s with good accelerations and is reassuring.
Contractions are 4 minutes apart and the patient is comfortablE. Twenty minutes later, the patient experiences a
large gush of clear fluid, and severe variable decelerations appear on the fetal heart rate monitor.Which of the
following conditions is not considered a risk factor for cord prolapse?
A grand multiparity
B female fetus

C abnormally long umbilical cord
D prematurity
ANSWER:B
458
A 27-year-old female (gravida 2, para 1) at 39 weeks of gestation presents to labor and delivery in active labor.
Her pregnancy has been uncomplicated and her prior two deliveries were vaginal. Her cervix is checked by the
nurse and judged to be 6 cm, 90% effaced, midposition, and soft. The fetus is not engaged and is thought to be
vertex. Initial fetal monitoring shows a heart rate in the 140s with good accelerations and is reassuring.
Contractions are 4 minutes apart and the patient is comfortablE. Twenty minutes later, the patient experiences a
large gush of clear fluid, and severe variable decelerations appear on the fetal heart rate monitor.Which of the
following statements about cord prolapse diagnosis is false?
A cord prolapse is likely when prolonged fetal bradycardia is seen in the presence of ruptured membranes
B ruptured membranes are a prerequisite
C mean cervical dilatation at diagnosis is 7 cm
D the diagnosis is confirmed when the umbilical cord is palpable in the vagina ahead of the fetal presenting part
ANSWER:C
459
A 27-year-old female (gravida 2, para 1) at 39 weeks of gestation presents to labor and delivery in active labor.
Her pregnancy has been uncomplicated and her prior two deliveries were vaginal. Her cervix is checked by the
nurse and judged to be 6 cm, 90% effaced, midposition, and soft. The fetus is not engaged and is thought to be
vertex. Initial fetal monitoring shows a heart rate in the 140s with good accelerations and is reassuring.
Contractions are 4 minutes apart and the patient is comfortablE. Twenty minutes later, the patient experiences a
large gush of clear fluid, and severe variable decelerations appear on the fetal heart rate monitor.What is the
recommended immediate management of this patient?
A emergent primary cesarean delivery
B operative vaginal delivery using forceps
C operative vaginal delivery using a vacuum extractor
D manual elevation of the presenting fetal part
ANSWER:D

460
No Prenatal Care and Bleeding A 23-year-old female (gravida 6, para 3114) presents to labor and delivery with
severe abdominal pain. She has no prenatal care, and she thinks her last menstrual period was approximately 9
months ago. She denies a history of medical problems or surgery. All previous deliveries were vaginal. She
smokes one and a half packs of cigarettes a day and admits to remote “crank” usE. Fundal height measures 39 cm,
and there is copious bleeding from the vaginA. The fetal monitor shows contractions every minute with elevated
baseline uterine tonE. Fetal tachycardia at 180 beats/minute, and late decelerations are also present.What is the
most likely diagnosis?
A uterine rupture
B placenta previa
C placental abruption
D vasa previa
ANSWER:C
461
No Prenatal Care and Bleeding. A 23-year-old female (gravida 6, para 3114) presents to labor and delivery with
severe abdominal pain. She has no prenatal care, and she thinks her last menstrual period was approximately 9
months ago. She denies a history of medical problems or surgery. All previous deliveries were vaginal. She
smokes one and a half packs of cigarettes a day and admits to remote “crank” usE. Fundal height measures 39 cm,
and there is copious bleeding from the vaginA. The fetal monitor shows contractions every minute with elevated
baseline uterine tonE. Fetal tachycardia at 180 beats/minute, and late decelerations are also present.Maternal risks
associated with this diagnosis include all of the following except
A death
B hysterectomy
C disseminated intravascular coagulation
D myocardial infarction
ANSWER:D
462
No Prenatal Care and Bleeding. A 23-year-old female (gravida 6, para 3114) presents to labor and delivery with
severe abdominal pain. She has no prenatal care, and she thinks her last menstrual period was approximately 9
months ago. She denies a history of medical problems or surgery. All previous deliveries were vaginal. She
smokes one and a half packs of cigarettes a day and admits to remote “crank” usE. Fundal height measures 39 cm,
and there is copious bleeding from the vaginA. The fetal monitor shows contractions every minute with elevated
baseline uterine tonE. Fetal tachycardia at 180 beats/minute, and late decelerations are also present.Which
gestational age has the highest incidence of placental abruption?
A 24 to 26 weeks
B 30 to 32 weeks

C 32 to 34 weeks
D 38 to 40 weeks
ANSWER:A
463
No Prenatal Care and Bleeding. A 23-year-old female (gravida 6, para 3114) presents to labor and delivery with
severe abdominal pain. She has no prenatal care, and she thinks her last menstrual period was approximately 9
months ago. She denies a history of medical problems or surgery. All previous deliveries were vaginal. She
smokes one and a half packs of cigarettes a day and admits to remote “crank” usE. Fundal height measures 39 cm,
and there is copious bleeding from the vaginA. The fetal monitor shows contractions every minute with elevated
baseline uterine tonE. Fetal tachycardia at 180 beats/minute, and late decelerations are also present.Which of the
following conditions is not strongly associated with placental abruption?
A maternal smoking
B maternal opiate use
C chorioamnionitis
D history of previous placental abruption
ANSWER:B
464
A 37-year-old female (gravida 2, para 1001) at 39 weeks of gestation progresses to complete and pushing. Her
pregnancy has been complicated by type 2 diabetes, for which she takes metformin. She has gained 45 pounds
during the pregnancy, despite both nutritional consultation and repeated counseling. She is 5 feet 2 inches tall and
has a prepregnancy body mass index of 34. Descent of the fetal head is slower than anticipated with “positive
turtle sign” during contractions. The head is delivered after 2 hours of pushing. The anterior shoulder is difficult
to deliver without increased traction. Sixty seconds pass without successful delivery.What is the most important
action to take at this time?
A flex the maternal hips and bring the knees up to the chest
B ask the nurse to apply suprapubic pressure
C call for additional help
D perform an episiotomy
ANSWER:C

465
A 37-year-old female (gravida 2, para 1001) at 39 weeks of gestation progresses to complete and pushing. Her
pregnancy has been complicated by type 2 diabetes, for which she takes metformin. She has gained 45 pounds
during the pregnancy, despite both nutritional consultation and repeated counseling. She is 5 feet 2 inches tall and
has a prepregnancy body mass index of 34. Descent of the fetal head is slower than anticipated with “positive
turtle sign” during contractions. The head is delivered after 2 hours of pushing. The anterior shoulder is difficult
to deliver without increased traction. Sixty seconds pass without successful delivery.Which of the following
statements about shoulder dystocia is true?
A clavicular fracture occurs in approximately 1% of cases
B a previously well-oxygenated fetus can tolerate 4 or 5 minutes of severe hypoxia without residual damage
C brachial plexus injuries usually involve the C3 and C4 nerve roots
D fractures involving the growth plate usually heal well with little or no long-term problems
ANSWER:B
466
A 37-year-old female (gravida 2, para 1001) at 39 weeks of gestation progresses to complete and pushing. Her
pregnancy has been complicated by type 2 diabetes, for which she takes metformin. She has gained 45 pounds
during the pregnancy, despite both nutritional consultation and repeated counseling. She is 5 feet 2 inches tall and
has a prepregnancy body mass index of 34. Descent of the fetal head is slower than anticipated with “positive
turtle sign” during contractions. The head is delivered after 2 hours of pushing. The anterior shoulder is difficult
to deliver without increased traction. Sixty seconds pass without successful delivery.Which of the following
statements regarding macrosomia and shoulder dystocia is true?
A diabetes and maternal obesity have strong positive predictive value for shoulder dystocia
B 30% of macrosomic infants deliver without shoulder dystocia
C fetal macrosomia is suspected when the estimated fetal weight is more than 4000 g
D 40% to 60% of cases of shoulder dystocia occur in infants who weigh less than 4000 g
ANSWER:D
467
Hypotension and Bleeding after Delivery. The patient in Clinical Case Problem 3 delivers atraumatically using a
combination of McRoberts’ maneuver, suprapubic pressure, and an episiotomy. Profuse vaginal bleeding is noted
both prior to and following delivery of the placentA. The patient becomes lightheaded and tachycardiC. Blood
pressure drops to 60/40 mmHg.Which of the following is the least likely cause of this problem?
A uterine atony
B uterine rupture
C retained placental parts
D vaginal or cervical lacerations
ANSWER:B

468
Hypotension and Bleeding after Delivery. The patient in Clinical Case Problem 3 delivers atraumatically using a
combination of McRoberts’ maneuver, suprapubic pressure, and an episiotomy. Profuse vaginal bleeding is noted
both prior to and following delivery of the placentA. The patient becomes lightheaded and tachycardiC. Blood
pressure drops to 60/40 mmHg.Which of the following steps is not included in active management of the third
stage of labor?
A administration of pitocin immediately following delivery of the anterior shoulder
B controlled cord traction
C immediate uterine massage after delivery of the placenta
D administering 400 to 600 μg of misoprostol orally
ANSWER:D
469
A 61-year-old postmenopausal female comes to your office for a routine health exam. She has a history of
osteoarthritis, and she smokes one pack of cigarettes per day. She fractured her left wrist at age 50 years after
falling down some stairs. Her mother has osteoporosis and fractured her hip after a fall. Her diet is low in calcium-
rich foods, and she is not currently taking a calcium supplement. She is on no medications. Her blood pressure is
120/80 mmHg, her height is 160cm, and she weighs 52kg. The rest of her physical exam is normal.Which of the
following is not an established major risk factor for osteoporosis?
A low body weight
B current smoking
C history of fragility fracture in first-degree relative
D low calcium intake
ANSWER:D
470
A 61-year-old postmenopausal female comes to your office for a routine health exam. She has a history of
osteoarthritis, and she smokes one pack of cigarettes per day. She fractured her left wrist at age 50 years after
falling down some stairs. Her mother has osteoporosis and fractured her hip after a fall. Her diet is low in calcium-
rich foods, and she is not currently taking a calcium supplement. She is on no medications. Her blood pressure is
120/80 mmHg, her height is 160cm, and she weighs 52kg. The rest of her physical exam is normal.Which of the
following is not an associated risk factor for osteoporosis?
A low calcium intake
B sedentary lifestyle
C cigarette smoking
D obesity
ANSWER:D

471
A 61-year-old postmenopausal female comes to your office for a routine health exam. She has a history of
osteoarthritis, and she smokes one pack of cigarettes per day. She fractured her left wrist at age 50 years after
falling down some stairs. Her mother has osteoporosis and fractured her hip after a fall. Her diet is low in calcium-
rich foods, and she is not currently taking a calcium supplement. She is on no medications. Her blood pressure is
120/80 mmHg, her height is 160cm, and she weighs 52kg. The rest of her physical exam is normal.What is the
most common presenting fracture in osteoporosis?
A wrist fracture C.olles’ fracture)
B vertebral compression fracture
C femoral neck fracture
D tibial fracture
ANSWER:B
472
A 61-year-old postmenopausal female comes to your office for a routine health exam. She has a history of
osteoarthritis, and she smokes one pack of cigarettes per day. She fractured her left wrist at age 50 years after
falling down some stairs. Her mother has osteoporosis and fractured her hip after a fall. Her diet is low in calcium-
rich foods, and she is not currently taking a calcium supplement. She is on no medications. Her blood pressure is
120/80 mmHg, her height is 160cm, and she weighs 52kg. The rest of her physical exam is normal.Which of the
following sites for osteoporotic fracture is most commonly associated with morbidity and mortality?
A Ward’s triangle (hip)
B the femoral neck (hip)
C the thoracic vertebrae (spine)
D the lumbar vertebrae (spine)
ANSWER:B
473
A 61-year-old postmenopausal female comes to your office for a routine health exam. She has a history of
osteoarthritis, and she smokes one pack of cigarettes per day. She fractured her left wrist at age 50 years after
falling down some stairs. Her mother has osteoporosis and fractured her hip after a fall. Her diet is low in calcium-
rich foods, and she is not currently taking a calcium supplement. She is on no medications. Her blood pressure is
120/80 mmHg, her height is 160cm, and she weighs 52kg. The rest of her physical exam is normal.Which of the
following conditions is not associated with an increased risk for osteoporosis?
A hyperparathyroidism
B rheumatoid arthritis
C history of solid organ transplant
D history of osteoarthritis

ANSWER:D
474
A 61-year-old postmenopausal female comes to your office for a routine health exam. She has a history of
osteoarthritis, and she smokes one pack of cigarettes per day. She fractured her left wrist at age 50 years after
falling down some stairs. Her mother has osteoporosis and fractured her hip after a fall. Her diet is low in calcium-
rich foods, and she is not currently taking a calcium supplement. She is on no medications. Her blood pressure is
120/80 mmHg, her height is 160cm, and she weighs 52kg. The rest of her physical exam is normal.Which of the
following is not a therapy approved by the Food and Drug Administration (FDA) for the prevention of
osteoporosis?
A bisphosphonates
B selective estrogen receptor modulators (SERMS)
C calcium supplementation
D teriparatide
ANSWER:D
475
A 41-year-old female comes to your office after finding a breast lump during a routine self-examination. She has
been examining her breasts regularly for the past 5 years; this is the first lump she has founD. On examination,
there is a lump located in the right breast. The lump’s anatomic location is in the upper outer quadrant. It is
approximately 3 cm in diameter and is not fixed to skin or musclE. It has a hard consistency. There are three
axillary nodes present on the right side; each node is approximately 1 cm in diameter. No lymph nodes are present
on the left.What is the first diagnostic procedure that should be performed in this patient?
A ultrasound of the breast
B mammography
C fine needle biopsy
D All of the above
ANSWER:B
476
A 41-year-old female comes to your office after finding a breast lump during a routine self-examination. She has
been examining her breasts regularly for the past 5 years; this is the first lump she has founD. On examination,
there is a lump located in the right breast. The lump’s anatomic location is in the upper outer quadrant. It is
approximately 3 cm in diameter and is not fixed to skin or musclE. It has a hard consistency. There are three
axillary nodes present on the right side; each node is approximately 1 cm in diameter. No lymph nodes are present
on the left.What is the definitive procedure that should be performed in this patient?
A ultrasound of the breast
B mammography

C biopsy
D All of the above
ANSWER:C
477
A Painful Bilateral Breast Masses That Wax and Wane with Her Period A 42-year-old female comes to your office
with bilateral breast masses that are painful and seem to “come and go” depending on the stage of the menstrual
cyclE. There is significant pain with these masses during menstruation. On examination, there are two areas of
dense tissue, one in each breast, and each is approximately 4 cm in diameter. No axillary lymph nodes are
palpablE.What is the most likely diagnosis in this patient?
A carcinoma of the breast
B mammary dysplasia (fibrocystic disease)
C fibroadenoma
D Paget’s disease of the breast
ANSWER:B
478
A Painful Bilateral Breast Masses That Wax and Wane with Her Period A 42-year-old female comes to your office
with bilateral breast masses that are painful and seem to “come and go” depending on the stage of the menstrual
cyclE. There is significant pain with these masses during menstruation. On examination, there are two areas of
dense tissue, one in each breast, and each is approximately 4 cm in diameter. No axillary lymph nodes are
palpablE.If medical treatment is indicated and prescribed for the condition described here, which of the following
should be considered as the therapeutic agent of first choice?
A hormone therapy: the oral contraceptive pill
B hormone therapy: danazol
C a thiazide diuretic
D vitamin E
ANSWER:A
479
A 23-year-old female consults her physician because of a breast mass; the mass is mobile, firm, and approximately
1 cm in diameter. It is located in the upper outer quadrant of the right breast. No axillary lymph nodes are
present.What is the most likely diagnosis in this patient?
A carcinoma of the breast
B mammary dysplasia (fibrocystic disease)
C fibroadenoma
D Paget’s disease of the breast
ANSWER:C

480
A 23-year-old female consults her physician because of a breast mass; the mass is mobile, firm, and approximately
1 cm in diameter. It is located in the upper outer quadrant of the right breast. No axillary lymph nodes are
present.What is the treatment of choice for the condition described here?
A modified radical mastectomy
B lumpectomy
C biopsy
D radical mastectomy
ANSWER:C
481
A 33-year-old female comes to your office with a 2-month history of a bloody unilateral left nipple dischargE.
She also has noted a small and soft lump just beneath the areola on the left sidE. On examination, there is a 4-mm
soft mass located just inferior to the left areolA. No other abnormalities are present in either breast. What is the
most likely diagnosis in this patient?
A carcinoma of the breast
B fibroadenoma
C intraductal papilloma
D fibrocystic breast disease
ANSWER:C
482
A 23-year-old healthy, nulliparous female comes to your office for her annual physical and Papanicolaou (Pap)
test. Her last menstrual period was 7 days ago. She has been on oral contraceptive pills (OCPs) for several years
and tells you that she stopped taking them recently to “give her body a break.” She heard from friends and
relatives that using OCPs for a long time increases the risk of future health problems, including infertility. She is
currently sexually active with one male partner for the past 6 months. They use condoms and withdrawal
inconsistently. The patient reports a history of chlamydia several years ago for which she and her partner were
treateD. She does not want to be pregnant anytime in the near futurE. She smokes a pack of cigarettes a day. On
examination, her blood pressure is 120/80 mmHg, her weight is 200 pounds, and she is 5 feet 5 inches tall B.ody
mass index is 33). The rest of her examination is unremarkable except for some mild facial acnE. You perform a
pelvic examination, a Pap test, and gonorrhea and chlamydia cultures.What would you tell your patient regarding
the use of the “withdrawal method?”
A it is a highly effective method of contraception but not sexually transmitted disease (STD) protection
B it is a highly effective method of STD protection but not contraception
C it has a less than 1% failure rate with “perfect use”
D it has up to a 24% failure rate with “typical use”
ANSWER:D

483
A 23-year-old healthy, nulliparous female comes to your office for her annual physical and Papanicolaou (Pap)
test. Her last menstrual period was 7 days ago. She has been on oral contraceptive pills (OCPs) for several years
and tells you that she stopped taking them recently to “give her body a break.” She heard from friends and
relatives that using OCPs for a long time increases the risk of future health problems, including infertility. She is
currently sexually active with one male partner for the past 6 months. They use condoms and withdrawal
inconsistently. The patient reports a history of chlamydia several years ago for which she and her partner were
treated. She does not want to be pregnant anytime in the near future. She smokes a pack of cigarettes a day. On
examination, her blood pressure is 120/80 mmHg, her weight is 200 pounds, and she is 5 feet 5 inches tall B.ody
mass index is 33). The rest of her examination is unremarkable except for some mild facial acne. You perform a
pelvic examination, a Pap test, and gonorrhea and chlamydia cultures. Which of the following statements is true
regarding the use of any estrogen-containing hormonal contraceptive method for this patient?
A estrogen is contraindicated because she is a smoker
B estrogen may increase her risk of endometrial cancer
C estrogen is contraindicated because of her history of chlamydia
D estrogen is contraindicated because of her obesity
ANSWER:A
484
A 23-year-old healthy, nulliparous female comes to your office for her annual physical and Papanicolaou (Pap)
test. Her last menstrual period was 7 days ago. She has been on oral contraceptive pills (OCPs) for several years
and tells you that she stopped taking them recently to give her body a break. She heard from friends and relatives
that using OCPs for a long time increases the risk of future health problems, including infertility. She is currently
sexually active with one male partner for the past 6 months. They use condoms and withdrawal inconsistently. The
patient reports a history of chlamydia several years ago for which she and her partner were treated. She does not
want to be pregnant anytime in the near future. She smokes a pack of cigarettes a day. On examination, her blood
pressure is 120/80 mmHg, her weight is 200 pounds, and she is 5 feet 5 inches tall B.ody mass index is 33). The
rest of her examination is unremarkable except for some mild facial acne. You perform a pelvic examination, a
Pap test, and gonorrhea and chlamydia cultures. You counsel the patien tabout her contraceptive options. All the
following are true except
A she cannot get an intrauterine device (IUD) because she has never had a child
B she may have an increased risk of contraceptive failure on the transdermal contraceptive patch (OrthoEvra)
C local skin irritation is the most common side effect experienced by transdermal contraceptive patch users
D the vaginal contraceptive ring (NuvaRing) is a soft, flexible ring that is self-inserted and removed by the patient
ANSWER:A

485
A 23-year-old healthy, nulliparous female comes to your office for her annual physical and Papanicolaou (Pap)
test. Her last menstrual period was 7 days ago. She has been on oral contraceptive pills (OCPs) for several years
and tells you that she stopped taking them recently to “give her body a break.” She heard from friends and
relatives that using OCPs for a long time increases the risk of future health problems, including infertility. She is
currently sexually active with one male partner for the past 6 months. They use condoms and withdrawal
inconsistently. The patient reports a history of chlamydia several years ago for which she and her partner were
treateD. She does not want to be pregnant anytime in the near futurE. She smokes a pack of cigarettes a day. On
examination, her blood pressure is 120/80 mmHg, her weight is 200 pounds, and she is 5 feet 5 inches tall B.ody
mass index is 33). The rest of her examination is unremarkable except for some mild facial acnE. You perform a
pelvic examination, a Pap test, and gonorrhea and chlamydia cultures.Your patient decides that she wants to restart
combined oral contraceptives C.OCs) since she has used the pills in the past and would like to have regular and
predictable menstrual cycles. Which option would not be ideal for this patient?
A progestin-only pills (POPs)
B COCs containing 35 μg of ethinyl estradiol
C COCs containing 20 μg of ethinyl estradiol
D monophasic COCs
ANSWER:A
486
A 23-year-old healthy, nulliparous female comes to your office for her annual physical and Papanicolaou (Pap)
test. Her last menstrual period was 7 days ago. She has been on oral contraceptive pills (OCPs) for several years
and tells you that she stopped taking them recently to “give her body a break.” She heard from friends and
relatives that using OCPs for a long time increases the risk of future health problems, including infertility. She is
currently sexually active with one male partner for the past 6 months. They use condoms and withdrawal
inconsistently. The patient reports a history of chlamydia several years ago for which she and her partner were
treateD. She does not want to be pregnant anytime in the near futurE. She smokes a pack of cigarettes a day. On
examination, her blood pressure is 120/80 mmHg, her weight is 200 pounds, and she is 5 feet 5 inches tall B.ody
mass index is 33). The rest of her examination is unremarkable except for some mild facial acnE. You perform a
pelvic examination, a Pap test, and gonorrhea and chlamydia cultures.You counsel your patient about starting
COCs. Which of the following statements regardingCOC initiation in this patient is true?
A she must wait until the first Sunday after her period begins to start her COCs
B nausea and breast tenderness are uncommon side effects of COCs
C if she develops any breakthrough bleeding, she should stop the COCs immediately
D weight gain is an unlikely consequence of COC use

ANSWER:D
487
A 23-year-old healthy, nulliparous female comes to your office for her annual physical and Papanicolaou (Pap)
test. Her last menstrual period was 7 days ago. She has been on oral contraceptive pills (OCPs) for several years
and tells you that she stopped taking them recently to “give her body a break.” She heard from friends and
relatives that using OCPs for a long time increases the risk of future health problems, including infertility. She is
currently sexually active with one male partner for the past 6 months. They use condoms and withdrawal
inconsistently. The patient reports a history of chlamydia several years ago for which she and her partner were
treateD. She does not want to be pregnant anytime in the near futurE. She smokes a pack of cigarettes a day. On
examination, her blood pressure is 120/80 mmHg, her weight is 200 pounds, and she is 5 feet 5 inches tall B.ody
mass index is 33). The rest of her examination is unremarkable except for some mild facial acnE. You perform a
pelvic examination, a Pap test, and gonorrhea and chlamydia cultures.Which of the following statements regarding
long-term COC use is true?
A there is strong evidence that long-term COC use increases ovarian cancer risk
B there is strong evidence that long-term COC use increases breast cancer risk
C there is strong evidence that long-term COC use decreases cervical cancer risk
D there is strong evidence that long-term COC use decreases endometrial cancer risk
ANSWER:D
488
A 23-year-old healthy, nulliparous female comes to your office for her annual physical and Papanicolaou (Pap)
test. Her last menstrual period was 7 days ago. She has been on oral contraceptive pills (OCPs) for several years
and tells you that she stopped taking them recently to “give her body a break.” She heard from friends and
relatives that using OCPs for a long time increases the risk of future health problems, including infertility. She is
currently sexually active with one male partner for the past 6 months. They use condoms and withdrawal
inconsistently. The patient reports a history of chlamydia several years ago for which she and her partner were
treateD. She does not want to be pregnant anytime in the near futurE. She smokes a pack of cigarettes a day. On
examination, her blood pressure is 120/80 mmHg, her weight is 200 pounds, and she is 5 feet 5 inches tall B.ody
mass index is 33). The rest of her examination is unremarkable except for some mild facial acnE. You perform a
pelvic examination, a Pap test, and gonorrhea and chlamydia cultures.All of the following conditions may be
improved with the use of estrogen-containing hormonal contraceptives except
A iron-deficiency anemia
B cholelithiasis
C dysmenorrhea
D ectopic pregnancy
ANSWER:B

489
A 40-year-old female (gravida 2, para 2) comes to your office for her 6-week postpartum visit. She had an
uncomplicated pregnancy, normal spontaneous vaginal delivery, and routine postpartum coursE. She and her
baby are doing well. She has not gotten her period yet. She is breast-feeding and supplementing with formula
intermittently. She does not want to get pregnant again, at least not for another few years. The patient has no major
medical problems, does not smoke, and has already returned to her aerobics class. She has no history of STDs or
abnormal Pap tests. She desires a reliable birth control method that she does not have to remember to take every
day or remember to use every time she has sex with her husbanD. Her examination is completely normal.Of the
following choices, which would be the most appropriate contraceptive method for this patient at this time?
A bilateral tubal ligation B.TL) or vasectomy
B transdermal contraceptive patch or vaginal contraceptive ring
C COC pills
D a levonorgestrel IUD or Depo-Provera
ANSWER:D
490
A 40-year-old female (gravida 2, para 2) comes to your office for her 6-week postpartum visit. She had an
uncomplicated pregnancy, normal spontaneous vaginal delivery, and routine postpartum coursE. She and her
baby are doing well. She has not gotten her period yet. She is breast-feeding and supplementing with formula
intermittently. She does not want to get pregnant again, at least not for another few years. The patient has no major
medical problems, does not smoke, and has already returned to her aerobics class. She has no history of STDs or
abnormal Pap tests. She desires a reliable birth control method that she does not have to remember to take every
day or remember to use every time she has sex with her husbanD. Her examination is completely normal.All of the
following would be appropriate management strategies except
A expectant management
B uterine aspiration
C medical management with vaginal misoprostol
D exploratory laparoscopy
ANSWER:D

491
A 40-year-old female (gravida 2, para 2) comes to your office for her 6-week postpartum visit. She had an
uncomplicated pregnancy, normal spontaneous vaginal delivery, and routine postpartum coursE. She and her
baby are doing well. She has not gotten her period yet. She is breast-feeding and supplementing with formula
intermittently. She does not want to get pregnant again, at least not for another few years. The patient has no major
medical problems, does not smoke, and has already returned to her aerobics class. She has no history of STDs or
abnormal Pap tests. She desires a reliable birth control method that she does not have to remember to take every
day or remember to use every time she has sex with her husbanD. Her examination is completely normal.Which of
the following statements is true regarding the use of estrogen-containing hormonal contraceptives in this patient?
A estrogen is contraindicated in women older than 40 years of age
B estrogen may increase the patient’s breast milk production
C estrogen may delay the onset of menopause
D estrogen may help regulate menses and/or reduce perimenopausal Symptoms
ANSWER:D
492
A 40-year-old female (gravida 2, para 2) comes to your office for her 6-week postpartum visit. She had an
uncomplicated pregnancy, normal spontaneous vaginal delivery, and routine postpartum coursE. She and her
baby are doing well. She has not gotten her period yet. She is breast-feeding and supplementing with formula
intermittently. She does not want to get pregnant again, at least not for another few years. The patient has no major
medical problems, does not smoke, and has already returned to her aerobics class. She has no history of STDs or
abnormal Pap tests. She desires a reliable birth control method that she does not have to remember to take every
day or remember to use every time she has sex with her husbanD. Her examination is completely normal.Which of
the following statements is true regarding the use of Depo-Provera in this patient?
A she will have rapid return to fertility following cessation of use
B Depo-Provera will not adversely affect her quantity or quality of breast milk
C Depo-Provera is contraindicated if she has a seizure disorder
D Depo-Provera will accelerate her age of onset of menopause
ANSWER:B

493
A 40-year-old female (gravida 2, para 2) comes to your office for her 6-week postpartum visit. She had an
uncomplicated pregnancy, normal spontaneous vaginal delivery, and routine postpartum coursE. She and her
baby are doing well. She has not gotten her period yet. She is breast-feeding and supplementing with formula
intermittently. She does not want to get pregnant again, at least not for another few years. The patient has no major
medical problems, does not smoke, and has already returned to her aerobics class. She has no history of STDs or
abnormal Pap tests. She desires a reliable birth control method that she does not have to remember to take every
day or remember to use every time she has sex with her husbanD. Her examination is completely normal.Which of
the following statements is true regarding the use of a copper IUD (ParaGard T 380A) in this patient.
A she will have an increased risk of ectopic pregnancy
B there is usually a long delay in return to fertility following removal of the copper IUD
C the copper IUD is contraindicated in breastfeeding mothers
D the copper IUD may increase her symptoms if she suffers from dysmenorrhea or menorrhagia
ANSWER:D
494
A 40-year-old female (gravida 2, para 2) comes to your office for her 6-week postpartum visit. She had an
uncomplicated pregnancy, normal spontaneous vaginal delivery, and routine postpartum coursE. She and her
baby are doing well. She has not gotten her period yet. She is breast-feeding and supplementing with formula
intermittently. She does not want to get pregnant again, at least not for another few years. The patient has no major
medical problems, does not smoke, and has already returned to her aerobics class. She has no history of STDs or
abnormal Pap tests. She desires a reliable birth control method that she does not have to remember to take every
day or remember to use every time she has sex with her husbanD. Her examination is completely normal.Your
patient asks you about sterilization options in the futurE. Which of the following statements about vasectomies and
tubal ligations is true?
A vasectomies are usually performed in an outpatient office under local anesthesia
B current vasectomy and tubal ligation procedures are easily reversible
C vasectomies increase prostate cancer risk
D tubal ligations increase the risk of ectopic pregnancy
ANSWER:A

495
A 34-year-old female who is a long-term patient of yours presents to the office for a routine blood pressure check.
She was recently diagnosed with hypertension and diabetes. Her medications include metformin,
hydrochlorothiazide, and a multivitamin. Her blood pressure today is 150/100 mmHg, and her body mass index is
30. She is currently sexually active with her husband, and they use the “rhythm” method only. She reports that her
menses have been irregular and vary from 20 to 45 days apart. The patient is worried about the risks of hormonal
contraception given her medical conditions. She does not want to be pregnant for several years.You advise her to
do all the following except
A exercise on most days of the week for 30 minutes
B eat a high-fiber, low-fat diet
C continue to use the rhythm method C.alendar method) only
D consider an IUD
ANSWER:C
496
A 34-year-old female who is a long-term patient of yours presents to the office for a routine blood pressure check.
She was recently diagnosed with hypertension and diabetes. Her medications include metformin,
hydrochlorothiazide, and a multivitamin. Her blood pressure today is 150/100 mmHg, and her body mass index is
30. She is currently sexually active with her husband, and they use the “rhythm” method only. She reports that her
menses have been irregular and vary from 20 to 45 days apart. The patient is worried about the risks of hormonal
contraception given her medical conditions. She does not want to be pregnant for several years.Which of the
following statements about barrier methods is true?
A the diaphragm must be inserted at least 24 hours prior to intercourse
B the cervical cap (FemCap) is less effective in parous women compared to nulliparous women
C the cervical cap has a lower pregnancy failure rate compared to the diaphragm
D barrier methods are not safe for medically complicated patients
ANSWER:B
497
A very tearful 21-year-old female (gravida 0, para 0) walks into your office on a Tuesday morning. She tells you
that she had sexual intercourse with her boyfriend Friday night. They used a condom, but it brokE. They
previously had intercourse with a condom the week beforE. Her last menstrual period was approximately 3 weeks
ago and was normal in flow and duration. She had been given a sample pack of Ortho-Tri-Cyclen during her
initial gynecologic examination 2 weeks ago, but she did not have a chance to start them yet. She would be
devastated ifshe got pregnant. She is a heavy smoker (two packs per day) but otherwise has no medical problems,
denies bleeding or other symptoms, and her examination is normal. Her most recent Pap smear and
gonococcus/chlamydia results were normal.Which of the following statements regarding the use of emergency
contraceptive pills E.CPs) in this patient is true?

A ECPs are contraindicated because it has been longer than 72 hours
B ECPs are contraindicated because she is a heavy smoker
C ECPs could have been prescribed to this patient over the phone without an examination
D ECPs would be contraindicated if either her Pap or her gonococcus/chlamydia test was abnormal
ANSWER:C
498
A very tearful 21-year-old female (gravida 0, para 0) walks into your office on a Tuesday morning. She tells you
that she had sexual intercourse with her boyfriend Friday night. They used a condom, but it brokE. They
previously had intercourse with a condom the week beforE. Her last menstrual period was approximately 3 weeks
ago and was normal in flow and duration. She had been given a sample pack of Ortho-Tri-Cyclen during her
initial gynecologic examination 2 weeks ago, but she did not have a chance to start them yet. She would be
devastated ifshe got pregnant. She is a heavy smoker (two packs per day) but otherwise has no medical problems,
denies bleeding or other symptoms, and her examination is normal. Her most recent Pap smear and
gonococcus/chlamydia results were normal.Which of the following best describes the effects of giving women
advance supplies of ECPs?
A women are more likely to stop routine birth control
B women are less likely to use condoms
C women are more likely to use ECPs when needed
D the rate of unintended pregnancy declines
ANSWER:C
499
You receive a call at 3 am from your prenatal patient who is worried about bleeding and cramping that began
several hours ago. This is the fourth pregnancy for your patient, which was a planned pregnancy. She has had two
uncomplicated, spontaneous vaginal deliveries and one elective abortion in the past. Her prenatal course to date
has been uncomplicateD. Two weeks ago, you obtained a first trimester ultrasound for dating purposes that
revealed a 6-week intrauterine pregnancy. She denies any fever, nausea, vomiting, dizziness, lightheadedness,
shortness of breath, or arm or chest pain. Her cramps are becoming more intense, but she is managing to control
the pain with a heating paD. She reports using approximately three sanitary pads in the past 6 hours for bleeding,
none of which were soaked through. The patient is home with her husband, who is a well-known patient of yours
as well. They are very anxious and want to know what to do next.You advise your patient to
A come to your office first thing in the morning for an evaluation
B take some ibuprofen and see you at her next scheduled prenatal visit
C rush to the emergency room because of suspected ectopic pregnancy
D rush to the emergency room for an immediate dilation and curettage &C)

ANSWER:A
500
You receive a call at 3 am from your prenatal patient who is worried about bleeding and cramping that began
several hours ago. This is the fourth pregnancy for your patient, which was a planned pregnancy. She has had two
uncomplicated, spontaneous vaginal deliveries and one elective abortion in the past. Her prenatal course to date
has been uncomplicateD. Two weeks ago, you obtained a first trimester ultrasound for dating purposes that
revealed a 6-week intrauterine pregnancy. She denies any fever, nausea, vomiting, dizziness, lightheadedness,
shortness of breath, or arm or chest pain. Her cramps are becoming more intense, but she is managing to control
the pain with a heating paD. She reports using approximately three sanitary pads in the past 6 hours for bleeding,
none of which were soaked through. The patient is home with her husband, who is a well-known patient of yours
as well. They are very anxious and want to know what to do next.She follows your advicE. The next day you see
the patient and her husband in your officE. She appears tearful, though calm. Her temperature is 98.4°F, blood
pressure is 120/80 mmHg, pulse is 80 beats/minute, and respiratory rate is 16 breaths/minutE. She reports that
since she spoke to you, she has passed a few dime-sized clots but no obvious tissuE. She continues to have lower
abdominal cramping. You perform a speculum exam, which reveals some blood in the vaginal vault and a small
amount of tissue protruding from an open, dilated cervical os. A bimanual exam reveals a 6-week-size uterus with
minimal tenderness but no peritoneal signs. The most likely diagnosis is
A missed abortion
B recurrent spontaneous abortion
C complete abortion
D incomplete abortion
ANSWER:C
501
A 23-year-old female graduate student presents to the office for a “personal problem” as reported by your nursE.
When you enter the room, she is noticeably tearful. She has regular menses, and her last menstrual period was
approximately 6 weeks ago. Today, she denies fever, vaginal bleeding, and abdominal pain. You perform a high-
sensitivity urine pregnancy test, which is positivE. On examination, the uterus is approximately 6 weeks in size
with no adnexal tenderness or masses. You tell the patient that she is approximately 6 weeks pregnant. The patient
is quiet and will not make eye contact with you.Which the following is the most appropriate next step in
management?
A congratulate the patient and schedule her initial prenatal visit
B ask the patient how she feels about being pregnant
C state that the urine pregnancy test is probably - hCG test is necessary to confirm the diagnosis
D tell her to go home and come back after she is ready to talk

ANSWER:B
502
A 23-year-old female graduate student presents to the office for a “personal problem” as reported by your nursE.
When you enter the room, she is noticeably tearful. She has regular menses, and her last menstrual period was
approximately 6 weeks ago. Today, she denies fever, vaginal bleeding, and abdominal pain. You perform a high-
sensitivity urine pregnancy test, which is positivE. On examination, the uterus is approximately 6 weeks in size
with no adnexal tenderness or masses. You tell the patient that she is approximately 6 weeks pregnant. The patient
is quiet and will not make eye contact with you.The patient’s pregnancy options could include all the following
except
A continuing the pregnancy and becoming a parent
B continuing the pregnancy and pursuing adoption for the baby
C ending the pregnancy by medication abortion E..g., mifepristone and methotrexate)
D pursuing any of the above options based solely on what her partner wants
ANSWER:D
503
A 24-year-old female comes to the emergency room with a 2-day history of lower abdominal pain, fever, chills,
and malaisE. The patient also complains of nausea and multiple episodes of vomiting in the past 24 hours. On
physical examination, there is bilateral adnexal tenderness, muco-purulent cervical discharge, and cervical motion
tenderness. The patient has a temperature of 40°C. Her last menstrual period was 4 weeks ago, and her pregnancy
test is negativE. She admits to being sexually active but denies a history of any sexually transmitted diseases
(STDs). She is currently not using birth control.What is the most likely diagnosis in this patient?
A acute appendicitis
B acute pelvic inflammatory disease (PID)
C uncomplicated cervicitis
D ectopic pregnancy
ANSWER:B
504
A 24-year-old female comes to the emergency room with a 2-day history of lower abdominal pain, fever, chills,
and malaisE. The patient also complains of nausea and multiple episodes of vomiting in the past 24 hours. On
physical examination, there is bilateral adnexal tenderness, muco-purulent cervical discharge, and cervical motion
tenderness. The patient has a temperature of 40°C. Her last menstrual period was 4 weeks ago, and her pregnancy
test is negativE. She admits to being sexually active but denies a history of any sexually transmitted diseases
(STDs). She is currently not using birth control. What is the most appropriate intervention for this patient?
A hospitalize the patient for parenteral treatment

B hospitalize the patient for immediate laporoscopy
C begin outpatient treatment with follow-up within 24 hours
D begin outpatient treatment with follow-up in 1 week
ANSWER:A
505
A 24-year-old female comes to the emergency room with a 2-day history of lower abdominal pain, fever, chills,
and malaisE. The patient also complains of nausea and multiple episodes of vomiting in the past 24 hours. On
physical examination, there is bilateral adnexal tenderness, muco-purulent cervical discharge, and cervical motion
tenderness. The patient has a temperature of 40°C. Her last menstrual period was 4 weeks ago, and her pregnancy
test is negativE. She admits to being sexually active but denies a history of any sexually transmitted diseases
(STDs). She is currently not using birth control.If hospitalization was chosen for this patient, which of the
following is an acceptable first-line parenteral regimen for her condition?
A intravenous (IV) ampicillin and gentamicin
B IV cefoxitin and oral doxycycline
C IV ceftriaxone only
D IV ciprofloxacin only
ANSWER:B
506
A 24-year-old female comes to the emergency room with a 2-day history of lower abdominal pain, fever, chills,
and malaisE. The patient also complains of nausea and multiple episodes of vomiting in the past 24 hours. On
physical examination, there is bilateral adnexal tenderness, muco-purulent cervical discharge, and cervical motion
tenderness. The patient has a temperature of 40°C. Her last menstrual period was 4 weeks ago, and her pregnancy
test is negativE. She admits to being sexually active but denies a history of any sexually transmitted diseases
(STDs). She is currently not using birth control.Which of the following statements regarding the relationship
between combined oral contraceptive pills (OCPs) and this patient’s condition is true?
A OCPs decrease the risk of this condition
B OCPs increase the risk of this condition
C OCPs have no influence on this condition
D OCPs are contraindicated in patients with this condition
ANSWER:A

507
A 24-year-old female comes to the emergency room with a 2-day history of lower abdominal pain, fever, chills,
and malaisE. The patient also complains of nausea and multiple episodes of vomiting in the past 24 hours. On
physical examination, there is bilateral adnexal tenderness, muco-purulent cervical discharge, and cervical motion
tenderness. The patient has a temperature of 40°C. Her last menstrual period was 4 weeks ago, and her pregnancy
test is negativE. She admits to being sexually active but denies a history of any sexually transmitted diseases
(STDs). She is currently not using birth control.200. Which of the following organisms is not associated with the
condition described in this case?
A Neisseria gonorrhea
B Chlamydia trachomatis
C Gardnerella hominis
D Group A-hemolytic streptococcus
ANSWER:D
508
A 24-year-old female comes to the emergency room with a 2-day history of lower abdominal pain, fever, chills,
and malaisE. The patient also complains of nausea and multiple episodes of vomiting in the past 24 hours. On
physical examination, there is bilateral adnexal tenderness, muco-purulent cervical discharge, and cervical motion
tenderness. The patient has a temperature of 40°C. Her last menstrual period was 4 weeks ago, and her pregnancy
test is negativE. She admits to being sexually active but denies a history of any sexually transmitted diseases
(STDs). She is currently not using birth control.All of the following are direct risk factors for PID except
A having new or multiple sexual partners
B living in an area with a high prevalence of N. gonorrhea and/or t rachomatis
C being age 25 years or younger
D prior or current use of an intrauterine device
ANSWER:C
509
A 24-year-old heterosexually active male comes to your office with complaints of a 2-day history of dysuriA. He
denies fever, urgency, frequency, or hematuriA. Physical examination reveals no suprapubic or costovertebral
tenderness. Urologic examination reveals mucupurulent urethral discharge, nontender testes, normal prostate, and
no penile lesions. Urine analysis is positive for leukocyte esterase, but it is negative for nitrite and blooD. You send
a swab of his urethral discharge for gram stain.The patient’s urethral gram stain reveals 20 WBCs per high power
fielD. There are no intracellular gram-negative diplococci seen. What is the most likely diagnosis in this patient?
A gonorrhea
B acute prostatitis

C epididymitis
D nongonococcal urethritis (NGU)
ANSWER:D
510
A 24-year-old heterosexually active male comes to your office with complaints of a 2-day history of dysuriA. He
denies fever, urgency, frequency, or hematuriA. Physical examination reveals no suprapubic or costovertebral
tenderness. Urologic examination reveals mucupurulent urethral discharge, nontender testes, normal prostate, and
no penile lesions. Urine analysis is positive for leukocyte esterase, but it is negative for nitrite and blooD. You send
a swab of his urethral discharge for gram stain.What is the most likely organism causing this condition?
A C. trachomatis
B Ureaplasma urealyticum
C Trichomonas vaginalis
D N. gonorrhea
ANSWER:A
511
A 23-year-old female graduate student presents to your office for her annual gynecologic examination. She has
been sexually active for 4 years with the same partner. She is up-to-date with cervical cancer screening, and her
Papanicolaou (Pap) smears have all been normal. The patient appears worried and says she wants to be checked
for “that HPV virus.” Several of her friends have had abnormal Pap smears and were told that the human
papillomavirus (HPV) was responsible for these findings. She asks how to prevent getting HPV and whether there
are treatments to “get rid of it.” On examination, her external genitalia and cervix appear normal without evidence
of lesions. Bimanual examination reveals a small, anterverted uterus with no masses.You inform the patient that
A there is nothing she can do to prevent getting HPV except stay in a monogamous relationship
B consistent condom use will protect her from HPV transmission
C she is a candidate for the HPV vaccine
D she is not eligible for the HPV vaccine because she is already sexually active
ANSWER:C

512
A 23-year-old female graduate student presents to your office for her annual gynecologic examination. She has
been sexually active for 4 years with the same partner. She is up-to-date with cervical cancer screening, and her
Papanicolaou (Pap) smears have all been normal. The patient appears worried and says she wants to be checked
for “that HPV virus.” Several of her friends have had abnormal Pap smears and were told that the human
papillomavirus (HPV) was responsible for these findings. She asks how to prevent getting HPV and whether there
are treatments to “get rid of it.” On examination, her external genitalia and cervix appear normal without evidence
of lesions. Bimanual examination reveals a small, anterverted uterus with no masses.Which of the following
statements is true about HPV?
A the majority of cervical cancers can be attributed to HPV 16 and 18
B the majority of cervical cancers can be attributed to HPV 6 and 11
C the majority of genital warts can be attributed to HPV 16 and 18
D it is a rare STD predominantly seen in sex workers
ANSWER:A
513
A 25-year-old sexually active female comes to your office with a 2-week history of “growths” in the vulvar
region. On examination, you find multiple “cauliflower” verrucous lesions on the labia majora and minorA.Which
of the following statements about syphilis is true?
A primary syphilis is associated with a single, painful chancre
B secondary syphilis is associated with skin lesions and lymphadenopathy
C latent syphilis is associated with constitutional symptoms
D treatment for primary syphilis is oral penicillin
ANSWER:B
514
A 25-year-old sexually active female comes to your office with a 2-week history of “growths” in the vulvar
region. On examination, you find multiple “cauliflower” verrucous lesions on the labia majora and minorA.What
is the treatment of choice in patients who are not allergic to penicillin for primary, secondary, or early latent
syphilis (syphilis acquired within the preceding year without evidence of disease)?
A benzathine penicillin G 2.4 million units IM in a single dose
B benzathine penicillin 2.4 million units IM in three doses doses, at 1-week intervals
C aqueous crystalline penicillin G IV 18 to 24 million units/day for 10 to 14 days
D levofloxacin 250 mg orally a day for 7 days
ANSWER:A
515
A 25-year-old sexually active female comes to your office with a 2-week history of “growths” in the vulvar
region. On examination, you find multiple “cauliflower” verrucous lesions on the labia majora and minorA.What
is the most likely diagnosis in this patient?

A condyloma lata
B condyloma acuminatum
C herpes simplex type 1
D herpes simplex type 2
ANSWER:B
516
A 25-year-old sexually active female comes to your office with a 2-week history of “growths” in the vulvar
region. On examination, you find multiple “cauliflower” verrucous lesions on the labia majora and minorA.All of
the following are acceptable treatments for this condition except
A podophyllin
B trichloracetic acid
C carbon dioxide laser
D acyclovir
ANSWER:D
517
A 25-year-old sexually active female comes to your office with a 2-week history of “growths” in the vulvar
region. On examination, you find multiple “cauliflower” verrucous lesions on the labia majora and minorA.The
patient should be counseled that
A treatment for genital warts prevents further recurrences
B treatment for genital warts prevents transmission to her partner
C she should have a Pap smear every 6 months from now on
D recurrence of genital warts is common
ANSWER:D
518
A 24-year-old female comes to your office with a 2-day history of dysuria accompanied by painful genital lesions
that have coalesced to form ulcers. The patient also has fever, malaise, myalgias, and headachE. There is no
previous history of this condition. She has had three sexual partners in the past and inconsistently uses barrier
contraceptive methods.You tell the patient the most likely diagnosis is
A herpes simplex infection
B chancroid
C lymphogranuloma venereum
D granuloma inguinale
ANSWER:A

519
A 24-year-old female comes to your office with a 2-day history of dysuria accompanied by painful genital lesions
that have coalesced to form ulcers. The patient also has fever, malaise, myalgias, and headachE. There is no
previous history of this condition. She has had three sexual partners in the past and inconsistently uses barrier
contraceptive methods.Which of the following statements concerning the patient’s condition is false?
A transmission of infection can occur during asymptomatic periods
B duration of viral shedding may be reduced with appropriate therapy
C time needed to heal lesions may be reduced with appropriate therapy
D subclinical viral shedding can be eliminated with appropriate suppressive therapy
ANSWER:D
520
A 24-year-old female comes to your office with a 2-day history of dysuria accompanied by painful genital lesions
that have coalesced to form ulcers. The patient also has fever, malaise, myalgias, and headachE. There is no
previous history of this condition. She has had three sexual partners in the past and inconsistently uses barrier
contraceptive methods.Strategies for the screening and diagnosis of HIV should include
A mandatory testing
B consent for HIV testing with an opportunity to decline
C further testing for STDs only if symptoms are present
D a chest radiograph
ANSWER:B
521
A 24-year-old female comes to your office with a 2-day history of dysuria accompanied by painful genital lesions
that have coalesced to form ulcers. The patient also has fever, malaise, myalgias, and headachE. There is no
previous history of this condition. She has had three sexual partners in the past and inconsistently uses barrier
contraceptive methods.Which of the following accurately describes the natural history of HIV?
A acute retroviral syndrome is usually asymptomatic
B antiretroviral therapy has no effect on the rate of immune system decline
C the median time between HIV infection and AIDS is 10 years in untreated patients
D opportunistic infections generally occur when CD4 counts are greater than 1000
ANSWER:C
522
A 27-year-old nulligravida female comes to your office with her husbanD. They are concerned about not having
conceived after a year of regular, unprotected intercoursE. The patient denies any major medical illnesses, and she
takes no medications. The husband reports he is healthy and has never fathered a chilD. Both the patient and her
husband are visibly upset and somewhat tearful while discussing their frustrations about not being pregnant yet.
They express that they are anxious to begin “all the tests necessary” as soon as possible so they can have a child
without further delay.What is the most appropriate diagnosis for this couple’s condition?

A primary sterility
B secondary sterility
C primary infertility
D secondary infertility
ANSWER:C
523
A 27-year-old nulligravida female comes to your office with her husbanD. They are concerned about not having
conceived after a year of regular, unprotected intercoursE. The patient denies any major medical illnesses, and she
takes no medications. The husband reports he is healthy and has never fathered a chilD. Both the patient and her
husband are visibly upset and somewhat tearful while discussing their frustrations about not being pregnant yet.
They express that they are anxious to begin “all the tests necessary” as soon as possible so they can have a child
without further delay.Infertility is defined as failure to conceive with unprotected regular sexual intercourse after
A 1 month
B 3 months
C 6 months
D 1 year
ANSWER:D
524
A 27-year-old nulligravida female comes to your office with her husbanD. They are concerned about not having
conceived after a year of regular, unprotected intercoursE. The patient denies any major medical illnesses, and she
takes no medications. The husband reports he is healthy and has never fathered a chilD. Both the patient and her
husband are visibly upset and somewhat tearful while discussing their frustrations about not being pregnant yet.
They express that they are anxious to begin “all the tests necessary” as soon as possible so they can have a child
without further delay.What is the most appropriate initial step in this couple’s evaluation?
A basal body temperature charting
B history and physical examination of both partners
C semen analysis
D referral to a reproductive specialist
ANSWER:B

525
A 27-year-old nulligravida female comes to your office with her husbanD. They are concerned about not having
conceived after a year of regular, unprotected intercoursE. The patient denies any major medical illnesses, and she
takes no medications. The husband reports he is healthy and has never fathered a chilD. Both the patient and her
husband are visibly upset and somewhat tearful while discussing their frustrations about not being pregnant yet.
They express that they are anxious to begin “all the tests necessary” as soon as possible so they can have a child
without further delay.The patient’s initial evaluation does not reveal any abnormalities. You discuss with the
patient that the next step is to confirm the presence of ovulation. All of the following are acceptable methods for
assessing ovulation except
A basal body temperature charting
B urine luteinizing hormone (LH) levels
C urine follicle-stimulating hormone (FSH) levels
D cervical mucus changes
ANSWER:D
526
A 27-year-old nulligravida female comes to your office with her husbanD. They are concerned about not having
conceived after a year of regular, unprotected intercoursE. The patient denies any major medical illnesses, and she
takes no medications. The husband reports he is healthy and has never fathered a chilD. Both the patient and her
husband are visibly upset and somewhat tearful while discussing their frustrations about not being pregnant yet.
They express that they are anxious to begin “all the tests necessary” as soon as possible so they can have a child
without further delay.All of the following may be direct causes of female infertility except
A previous uncomplicated abortion
B pelvic inflammatory disease (PID)
C endometriosis
D polycystic ovarian syndrome (PCOS)
ANSWER:A
527
A 27-year-old nulligravida female comes to your office with her husbanD. They are concerned about not having
conceived after a year of regular, unprotected intercoursE. The patient denies any major medical illnesses, and she
takes no medications. The husband reports he is healthy and has never fathered a chilD. Both the patient and her
husband are visibly upset and somewhat tearful while discussing their frustrations about not being pregnant yet.
They express that they are anxious to begin “all the tests necessary” as soon as possible so they can have a child
without further delay.Evaluation for tubal patency or “pelvic factor” is best accomplished by
A transvaginal ultrasound

B hysteroscopy
C hysterosalpingogram (HSG)
D pelvic magnetic resonance imaging (MRI)
ANSWER:C
528
A 27-year-old nulligravida female comes to your office with her husbanD. They are concerned about not having
conceived after a year of regular, unprotected intercoursE. The patient denies any major medical illnesses, and she
takes no medications. The husband reports he is healthy and has never fathered a chilD. Both the patient and her
husband are visibly upset and somewhat tearful while discussing their frustrations about not being pregnant yet.
They express that they are anxious to begin “all the tests necessary” as soon as possible so they can have a child
without further delay.The postcoital test is performed to assess which of the following?
A interaction of sperm with cervical mucus prior to ovulation
B interaction of sperm with cervical mucus after ovulation
C interaction of sperm with cervical mucus anytime during the cycle
D interaction of sperm with cervical mucus in mid-luteal phase
ANSWER:A
529
A 27-year-old nulligravida female comes to your office with her husbanD. They are concerned about not having
conceived after a year of regular, unprotected intercoursE. The patient denies any major medical illnesses, and she
takes no medications. The husband reports he is healthy and has never fathered a chilD. Both the patient and her
husband are visibly upset and somewhat tearful while discussing their frustrations about not being pregnant yet.
They express that they are anxious to begin “all the tests necessary” as soon as possible so they can have a child
without further delay.Appropriate initial screening for male infertility includes which of the following?
A two semen analyses done at least 1 month apart
B serum testosterone and FSH levels
C postejaculatory urinalysis
D scrotal ultrasonography
ANSWER:A

530
A 27-year-old nulligravida female comes to your office with her husbanD. They are concerned about not having
conceived after a year of regular, unprotected intercoursE. The patient denies any major medical illnesses, and she
takes no medications. The husband reports he is healthy and has never fathered a chilD. Both the patient and her
husband are visibly upset and somewhat tearful while discussing their frustrations about not being pregnant yet.
They express that they are anxious to begin “all the tests necessary” as soon as possible so they can have a child
without further delay.It is appropriate to initiate an infertility evaluation after 6 months of trying to conceive in
which of the following conditions?
A the woman is older than age 35 years
B the man is older than age 40 years
C the woman has used Depo-Provera within the previous year
D the woman has used oral contraceptive pills for at least 10 years
ANSWER:A
531
A 21-year-old woman, who is known to have beta thalassemia major, attends the clinic for preconception
counselling.What is the most relevant initial pre-pregnancy investigation to predict maternal complications of
pregnancy?
A Cardiac MRI
B Chest X-Ray
C ECG
D Echocardiogram
ANSWER:D
532
A 34-year-old woman attends for her booking in her third pregnancy. She had a caesarean section in her first
pregnancy 4 years ago and has had a successful vaginal birth after caesarean section (VBAC) 2 years ago. She has
a BMI OF 26. What is the best predictor for a successful VBAC?
A BMI of less than 30
B Less than 35 years old
C Previous vaginal birth
D Short inter-pregnancy interval
ANSWER:C
533
A 28-year-old woman attends the mental health antenatal clinic at 12 weeks for a booking assessment. This is her
first baby. Which condition gives her the highest risk of puerperal psychosis?
A Anorexia nervosa
B Bipolar affective disorder
C Moderate depression

D Obsessive compulsive disorder
ANSWER:B
534
A woman has an instrumental delivery of a baby weighing 3950 g in her first pregnancy. A Grade 3C tear of the
anal sphincter is identifieD. An appropriate overlapping repair using 3/0 PDS is performeD. Prior to discharge, she
asks about the long-term risk of faecal or flatal incontinencE. What percentage risk would you advise?
A 10%
B 20%
C 30%
D 40%
ANSWER:D
535
After an initial pregnancy resulted in a spontaneous loss in the first trimester, your patient is concerned about the
possibility of this recurring. An appropriate answer would be that the chance of recurrence
A Depends on the genetic makeup of the prior abortus
B Is no different than it was prior to the miscarriage
C Is increased to approximately 50%
D Is increased most likely to greater than 50%
ANSWER:B
536
A 24-year-old woman has had three first-trimester spontaneous abortions. Which of the following statements
concerning chromosomal aberrations in abortions is true?
A 45,X is more prevalent in chromosomally abnormal term babies than in abortus products
B Approximately 20% of first-trimester spontaneous abortions have chromosomal abnormalities
C Trisomy 21 is the most common trisomy in abortuses
D Despite the relatively high frequency of Down syndrome at term, most Down fetuses abort spontaneously
ANSWER:D
537
A 26-year-old patient has had three consecutive spontaneous abortions early in the second trimester. As part of an
evaluation for this problem, the least useful test would be
A Hysterosalpinogram
B Chromosomal analysis of the couple
C Endometrial biopsy in the luteal phase
D Postcoital test
ANSWER:D

538
A 24-year-old woman is in a car accident and is taken to an emergency room, where she receives a chest x-ray and
a film of her lower spinE. It is later discovered that she is 10 weeks pregnant. She should be counseled that
A The fetus has received 50 rads
B Either chorionic villus sampling C.VS) or amniocentesis is advisable to check for fetal chromosomal abnormalities
C At 10 weeks, the fetus is particularly susceptible to derangements of the central nervous system
D The fetus has received less than the assumed threshold for radiation damage
ANSWER:D
539
One of your patients, a 25-year-old G0, comes to your office for preconception counseling. She is a long-distance
runner and wants to continue to train during her pregnancy. This patient wants to know if there are any potential
adverse effects to her fetus if she pursues a program of regular exercise throughout gestation. You advise her of
which of the following true statements regarding exercise and pregnancy?
A
During pregnancy, women should stop exercising because such activity is commonly associated with intrauterine
growth retardation in the fetus
B Exercise is best performed in the supine position to maximize venous return and cardiac output
C It is acceptable to continue to exercise throughout pregnancy as long as thematernal pulse does not exceed 160
D Non-weight-bearing exercises are optimal because they minimize the risks of maternal and fetal injuries
ANSWER:D
540
A 47-year-old woman has achieved a pregnancy via in vitro fertilization (IVF) using donor eggs from a 21-year-
old donor and sperm from her 46- year-old husbanD. She has a sonogram performed at 7 weeks gestational age
that shows a quintuplet pregnancy. A 5-mm nuchal translucency is discovered in one of the embryos. Implications
of this include which of the following?
A The embryo has a high risk of neural tube defect
B The embryo has a high risk of cardiac malformation
C The nuchal translucency will enlarge by 20 weeks
D If the nuchal translucency resolves, the risk of a chromosome abnormality is comparable to that of other embryos
ANSWER:B
541
A 24-year-old white woman has a maternal serum α-fetoprotein (MSAFP) at 17 weeks gestation of 6.0 multiples
of the median (MOM). The next step should be
A A second MSAFP test
B Ultrasound examination
C Amniocentesis

D Amniography
ANSWER:B
542
A 41-year-old had a baby with Down syndrome 10 years ago. She is anxious to know the chromosome status of
her fetus in a current pregnancy. The test that has the fastest lab processing time for karyotype is
A Amniocentesis
B Cordocentesis
C Chorionic villus sampling C.VS)
D Doppler flow ultrasound
ANSWER:C
543 A 39-year-old wants first-trimester prenatal diagnosis. Advantages of early amniocentesis over CVS include
A Amniocentesis can be performed earlier in pregnancy
B Amniocentesis is usually less painful
C Second-trimester diagnosis allows for safer termination of pregnancy when termination is chosen by the patient
D CVS has a higher complication rate than first-trimester amniocentesis
ANSWER:D
544
You see a healthy 40-year-old multiparous patient for preconception counseling. She is extremely worried about
her risk of having a baby with spina bifidA. Five years ago, this patient delivered a baby with anencephaly who
died shortly after birth. How would you counsel this woman regarding future pregnancies?
A She does not have a recurrence risk of a neural tube defect above that of the general population
B She has an increased risk of having another baby with anencephaly because she is over 35 years old
C
When she becomes pregnant, she should undergo diagnostic testing for fetal neural tube defects with a first-
trimester chorionic villus sampling
D
When she becomes pregnant, she should avoid hyperthermia in the first trimester because both maternal fevers and
the use of hot tubs have been associated with an increased risk of neural tube defects
ANSWER:D
545
A 36-year-old G1 undergoes a triple screen test at 16 weeks of pregnancy to evaluate her risk of having a baby
with Down syndrome because she is worried about being of “advanced maternal agE.” Her maternal serum AFP
level comes back elevateD. This patient is extremely concerned and comes into your office to get additional
counseling and recommendations. All of the following are true statements that you can tell her except
A
An elevated serum AFP level indicates that she is at risk for having a baby with Down syndrome, and you
recommend that she undergo a chorionic villus sampling to definitely determine the fetus’s chromosomes
B An ultrasound should be performed to confirm the gestational age of the fetus and rule out any fetal anomalies

C
Fetal neural tube defects, multifetal gestations, and fetal abdominal wall defects are all possible etiologies of an
elevated MSAFP
D
Unexplained elevated MSAFP levels have been associated with poor pregnancy outcome, such as placental
abruption, oligohydramnios, or fetal death in utero
ANSWER:A
546
An obese, 25-year-old G1P0 comes to your office at 8 weeks gestational age for her first prenatal visit. She is
delighted to be pregnant and wants to do whatever is necessary to ensure a healthy pregnancy. She is currently 5
ft, 2 in. tall and weighs 300 lB. She is concerned because she is overweight and wants you to help her with a strict
exercise and diet regimen so that she can be more healthy during the pregnancy. During your counseling session
with the patient, you advise her of all of the following except
A Marked obesity in pregnancy increases the risk of developing diabetes, hypertension, and fetal macrosomia
B
She should gain at least 25 lb during the pregnancy because nutritional deprivation can result in impaired fetal
brain development and intrauterine fetal growth retardation
C Obese women will still have adequate fetal growth in the absence of any weight gain during pregnancy
D She should avoid initiating a vigorous exercise program to get in shape
ANSWER:B
547
A 26-year-old G1P1 comes to see you in your office for preconception counseling because she wants to get
pregnant again. She denies a history of any illegal drug use but admits to smoking a few cigarettes each day and
occasionally drinking some beer. When you advise her not to smoke or drink at all during this pregnancy, she gets
defensive because she smokes and drinks very little, and she did the same during her previous pregnancy 2 years
ago and her baby was just finE. Which of the following statements is true regarding the effects of tobacco and
alcohol on pregnancy?
A
Small amounts of alcohol, such as a glass of wine or beer a day at dinner time, are safe; only binge drinking of
large amounts of alcohol has been associated with fetal alcohol syndrome
B Fetal alcohol syndrome can be diagnosed prenatally via identifying fetal anomalies on sonogram done antenatally
C Cigarette smoking is associated with an increased risk of spontaneous abortion
D In most studies, cigarette smoking has been associated with an increased risk of congenital anomalies
ANSWER:C
548
A 36-year-old G0 who has been epileptic for many years is contemplating pregnancy and wants to go off her
anticonvulsant medications because she is concerned regarding the adverse effects that these medications may have
on her unborn fetus. All of the following statements are true regarding epilepsy and pregnancy except

A
Babies born to epileptic mothers have an increased risk of structural anomalies even in the absence of
anticonvulsant medications
B
Epileptic women should have their neurologist attempt to wean them off their anticonvulsant medications prior to
conceiving
C Phenytoin D.ilantin) is associated with a 1 to 2% risk of spina bifida
D
Folic acid supplementation reduces the risk of congenital anomalies in fetuses of epileptic women taking
anticonvulsants
ANSWER:C
549
A patient who works as a nurse in the surgery intensive care unit at a local community hospital comes to see you
for her annual gynecologic exam. She tells you that she plans to go off her oral contraceptives because she plans
to attempt pregnancy in the next few months. This patient has many questions regarding updating her
immunizations and whether or not she can do this when pregnant. All of the following statements are
trueregarding immunizations and pregnancy except
A
The patient should be checked for immunity against the rubella (Germanmeasles) virus prior to conception
because the rubella vaccine is a live virus and should not be given during pregnancy
B
The patient should be given the tetanus toxoid vaccination prior to becoming pregnant because it is a live virus
that has been associated with multiple fetal anomalies when administered during pregnancy
C
The Centers for Disease Control and Prevention recommends that all pregnant women should be vaccinated
against the influenza virus after the first trimester
D
If the patient does not have a history of having chickenpox in the past, she should be screened for immunity
because most people without a history of the infection are immune; if the patient is nonimmune, she should be
given the vaccine prior to achieving pregnancy because the vaccine is a live virus
ANSWER:B
550
A 20-year-old G2P1 patient comes to see you at 17 weeks gestational age to review the results of her triple test
done 1 week ago. You tell the patient that her MSAFP level is 2.0 MOM. The patient’s obstetrical history consists
of a term vaginal delivery 2 years ago without complications. What do you tell your patient regarding how to
proceed next?
A
Explain to the patient that the blood test is diagnostic of a neural tube defect and she should consult with a
pediatric neurosurgeon as soon as possible
B
Tell the patient that the blood test result is most likely a false-positive result and she should repeat the test at 20
weeks
C Refer the patient for an ultrasound to confirm dates
D Offer the patient immediate chorionic villus sampling to obtain a fetal karyotype

ANSWER:C
551
You see a 42-year-old patient in your office who is now 5 weeks pregnant with her fifth baby. She is very
concerned regarding the risk of Down syndrome because of her advanced maternal agE. After extensive genetic
counseling, she has decided to undergo a second-trimester amniocentesis to determine the karyotype of her fetus.
Prior to performing the procedure,you inform the patient that all of the following are possible complications of the
amniocentesis except
A Amniotic fluid leakage
B Chorioamnionitis
C A fetal loss rate of less than 0.5%
D Limb reduction defects
ANSWER:D
552
A healthy 25-year-old G1P0 at 40 weeks gestational age comes to your office to see you for a routine OB visit.
The patient complains to you that on several occasions she has experienced dizziness, light-headedness, and
feeling as if she is going to pass out when she lies down on her back to take a nap. What is the appropriate plan of
management for this patient?
A Do an ECG
B Monitor her for 24 h with a Holter monitor to rule out an arrhythmia
C Do an arterial blood gas analysis
D Reassure her that nothing is wrong with her and encourage her not to lie flat on her back
ANSWER:D
553
A 42-year-old primigravida presents to your office for a routine OB visit at 34 weeks gestational agE. She voices
concern because she has noticed an increasing number of spidery veins appearing on her abdomen. She is upset
with the unsightly appearance of these veins and wants to know what you recommend to get rid of them. How do
you advise this patient?
A Tell her that this is not a serious condition and give her a referral to a vascular surgeon to have the veins removed
B Tell her that you are concerned that she may have serious liver disease and order liver function tests
C Refer her to a dermatologist for further workup and evaluation
D
Tell her that the appearance of these blood vessels is a normal occurrence with pregnancy and will resolve
spontaneously after delivery
ANSWER:D

554
A 32-year-old G2P1001 at 20 weeks gestational age presents to the emergency room complaining of constipation
and abdominal pain for the past 24 h. The patient also admits to bouts of nausea and emesis since eating a very
spicy meal at a new Thai restaurant the evening beforE. She denies a history of any medical problems. During her
last pregnancy, the patient underwent an elective cesarean section at term to deliver a fetus in the breech
presentation. The emergency room doctor who examines her pages you and reports that the patient has a low-
grade fever of 100°F, with a normal pulse and blood pressurE. She is minimally tender to deep palpation with
hypoactive bowel sounds. She has no rebound tenderness. The patient has a WBC of 13,000, and electrolytes are
normal. What is the appropriate next step in the management of this patient?
A
The history and physical exam are consistent with constipation, which is commonly associated with pregnancy; the
patient should be discharged with reassurance and instructions to give herself a soapsuds enema and follow a high-
fiber diet with laxative use as needed
B The patient should be prepped for the operating room immediately to have an emergent appendectomy
C
The patient should be reassured that her symptoms are due to the spicy meal consumed the evening before and
should be given Pepto-Bismol to alleviate the symptoms
D The patient should be sent to radiology for an upright abdominal x-ray
ANSWER:D
555
A healthy 34-year-old G1P0 patient comes to see you in your office for a routine OB visit at 12 weeks gestational
agE. She tells you that she has stopped taking her prenatal vitamins with iron supplements because they make her
sick and she has trouble remembering to take a pill every day. A review of her prenatal labs reveals that her
hematocrit is 39%. Which of the following statements is the correct way to counsel this patient?
A
Tell the patient that she does not need to take her iron supplements because her prenatal labs indicate that she is not
anemic and therefore she will not absorb the iron supplied in prenatal vitamins
B Tell the patient that if she consumes a diet rich in iron, she does not need to take any iron supplements
C Tell the patient that if she fails to take her iron supplements, her fetus will be anemic
D
Tell the patient that she needs to take the iron supplements even though she is not anemic in order to meet the
demands of pregnancy
ANSWER:D
556
A 17-year-old primipara at 41 weeks wants an immediate cesarean section. She is being followed with biophysical
profile testing. You tell her that
A
Biophysical profile testing includes amniotic fluid volume, fetal breathing, fetal body movements, fetal body tone,
and contraction stress testing
B The false-negative rate of the biophysical profile is 10%
C False-positive results on biophysical profile are rare

D Spontaneous decelerations during biophysical profile testing are associated with significant fetal morbidity
ANSWER:D
557
A patient at 17 weeks gestation is diagnosed as having an intrauterine fetal demisE. She returns to your office 5
weeks later and has not had a miscarriage, although she has had some occasional spotting. This patient is at
increased risk for
A Septic abortion
B Recurrent abortion
C Consumptive coagulopathy with hypofibrinogenemia
D Future infertility
ANSWER:C
558
A 24-year-old presents at 30 weeks with a fundal height of 50 cm. Which of the following statements concerning
polyhydramnios is true?
A Acute polyhydramnios always leads to labor prior to 28 weeks
B The incidence of associated malformations is approximately 3%
C Maternal edema, especially of the lower extremities and vulva, is rare
D Complications include placental abruption, uterine dysfunction, and postpartum hemorrhage
ANSWER:D
559
A 19-year-old woman comes to the emergency room and reports that she fainted at work earlier in the day. She
has mild vaginal bleeding. Her abdomen is diffusely tender and distendeD. In addition, she complains of shoulder
and abdominal pain. Her temperature is 97.6°F, pulse rate is 120/min, and blood pressure is 96/50 mmHg. To
confirm the diagnosis suggested by the available clinical data, the best diagnostic procedure is
A Pregnancy test
B Posterior colpotomy
C Dilation and curettage
D Culdocentesis
ANSWER:D
560
A 27-year-old has just had an ectopic pregnancy. Which of the following events would be most likely to
predispose to ectopic pregnancy?
A Previous tubal surgery
B Pelvic inflammatory disease (PID)
C Use of a contraceptive uterine device (IUD)
D Induction of ovulation
ANSWER:B

561
A maternal fetal medicine specialist is consulted and performs an indepth sonogram. The sonogram indicates that
the fetuses are both male,and the placenta appears to be diamniotic and monochorioniC. Twin B is noted to have
oligohydramnios and to be much smaller than twin A. In this clinical picture, all of the following are concerns for
twin A except
A Congestive heart failure
B Anemia
C Hypervolemia
D Polycythemia
ANSWER:B
562
A 33-year-old has an infection in pregnancy. Which of the following is a reinfection, and therefore not a risk to
the fetus?
A Group B coxsackievirus
B Rubella virus
C Chickenpox virus
D Shingles
ANSWER:D
563
Viremia and the presence of rubella virus in the throat of infected persons bear which of the following
relationships to the onset of the rubella rash?
A They precede the rash by 5 to 7 days
B They precede the rash by 1 to 2 days
C They occur coincidentally with the rash
D They occur 1 to 2 days after the rash
ANSWER:A
564
A pregnant woman is discovered to be an asymptomatic carrier of Neisseria gonorrhoeaE. A year ago, she was
treated with penicillin for a gonococcal infection and developed a severe allergic reaction. Treatment of choice at
this time is
A Tetracycline
B Ampicillin
C Spectinomycin
D Chloramphenicol
ANSWER:C
565
A 22-year-old has just been diagnosed with toxoplasmosis. You try to determine what her risk factors werE. The
highest risk association is

A Eating raw meat
B Eating raw fish
C Having a dog
D Being English
ANSWER:A
566
A 24-year-old woman appears at 8 weeks of pregnancy and reveals a history of pulmonary embolism 7 years ago
during her first pregnancy. She was treated with intravenous heparin followed by several months of oral warfarin
C.oumadin) and has had no further evidence of thromboembolic disease for over 6 years. Which of the following
statements about her current condition is true?
A Having no evidence of disease for over 5 years means that the risk of thromboembolism is not greater than normal
B Impedance plethysmography is not a useful study to evaluate for deep venous thrombosis in pregnancy
C Doppler ultrasonography is not a useful technique to evaluate for deep venous thrombosis in pregnancy
D The patient should be placed on low-dose heparin therapy throughout pregnancy and puerperium
ANSWER:D
567
An 18-year-old has asymptomatic bacteriuria at her first prenatal visit at 15 weeks gestation. Which of the
following statements is true?
A The prevalence of ASB during pregnancy may be as great as 30%
B There is a decreased incidence of ASB in multiparas with sickle cell trait
C Fifteen percent of women develop a urinary tract infection after an initial negative urine culture
D
Ten percent of women with ASB subsequently develop an acute symptomatic urinary infection during that
pregnancy
ANSWER:D
568
A 21-year-old has a positive purified protein derivative (PPD) and is about to be treated for tuberculosis. She can
be reassured that her risk of which of the following is minimal?
A A flulike syndrome caused by rifampin
B A peripheral neuropathy caused by isoniazid
C Optic neuritis caused by INH
D Ototoxicity as a side effect of streptomycin
ANSWER:C

569
A 27-year-old woman (gravida 3, para 2) comes to the delivery floor at 37 weeks gestation. She has had no
prenatal carE. She complains that, on bending down to pick up her 2-year-old child, she experienced sudden,
severe back pain that now has persisted for 2 h. Approximately 30 min ago she noted bright red blood coming
from her vaginA. By the time she arrives at the delivery floor, she is contracting strongly every 3 min; the uterus is
quite firm even between contractions. By abdominal palpation, the fetus is vertex with the head deeply engageD.
Fetal heart rate is 130/min. The fundus is 38 cm above the symphysis. Blood for clotting is drawn, and a clot
forms in 4 min. Clotting studies are sent to the laboratory. Which of the following actions can wait until the patient
is stabilized?
A Stabilizing maternal circulation
B Attaching a fetal electronic monitor
C Inserting an intrauterine pressure catheter
D Administering oxytocin
ANSWER:D
570
A 16-year-old G1P0 at 38 weeks gestation comes to the labor and delivery suite for the second time during the
same weekend that you are on call. She initially presented to L and D at 2:00 P.M. Saturday afternoon
complaining of regular uterine contractions. Her cervix was 50/1/−1 vertex, and she was sent home after walking
for 2 h in the hospital without any cervical changE. It is now Sunday night at 8:00 P.M., and the patient returns to
L and D with increasing pain. She is exhausted because she did not sleep the night before because her contractions
kept waking her up. The patient is placed on the external fetal monitor. Her contractions are occurring every 2 to
3 min. You reexamine the patient and determine that her cervix is unchangeD. What is the best next step in the
management of this patient?
A Perform artificial rupture of membranes to initiate labor
B Administer an epidural
C Administer Pitocin to augment labor
D Administer 10 mg intramuscular morphine
ANSWER:D

571
A 75-year-old G2P2 presents to your GYN office for a routine exam. She tells you that she does not have an
internist and does not remember the last time she had a physical exam. She says she is very healthy and denies
taking any medication, including hormone replacement therapy. She is a nonsmoker and has an occasional
cocktail with her dinner. She does not have any complaints. In addition, she denies any family history of cancer.
The patient tells you that she is a widow and lives alone in an apartment in town. Her grown children have families
of their own and live far away. She states that she is self-sufficient and spends her time visiting friends and
volunteering at a local museum. Her blood pressure is 140/70. Her height is 5 ft, 4 in. and she weighs 130 lB. Her
physical exam is completely normal. All of the following are appropriate screening tests to order for this patient
except
A Colonoscopy
B Mammogram
C Bone densitometry
D TB skin test
ANSWER:D
572
A 75-year-old G2P2 presents to your GYN office for a routine exam. She tells you that she does not have an
internist and does not remember the last time she had a physical exam. She says she is very healthy and denies
taking any medication, including hormone replacement therapy. She is a nonsmoker and has an occasional
cocktail with her dinner. She does not have any complaints. In addition, she denies any family history of cancer.
The patient tells you that she is a widow and lives alone in an apartment in town. Her grown children have families
of their own and live far away. She states that she is self-sufficient and spends her time visiting friends and
volunteering at a local museum. Her blood pressure is 140/70. Her height is 5 ft, 4 in. and she weighs 130 lB. Her
physical exam is completely normal. You recommend to the patient that she receive all the following vaccinations
except
A Hepatitis B vaccine
B Pneumococcal vaccine
C Influenza vaccine
D Tetanus-diphtheria booster every 10 years
ANSWER:A

573
A 75-year-old G2P2 presents to your GYN office for a routine exam. She tells you that she does not have an
internist and does not remember the last time she had a physical exam. She says she is very healthy and denies
taking any medication, including hormone replacement therapy. She is a nonsmoker and has an occasional
cocktail with her dinner. She does not have any complaints. In addition, she denies any family history of cancer.
The patient tells you that she is a widow and lives alone in an apartment in town. Her grown children have families
of their own and live far away. She states that she is self-sufficient and spends her time visiting friends and
volunteering at a local museum. Her blood pressure is 140/70. Her height is 5ft, 4 in. and she weighs 130 lB. Her
physical exam is completely normal.You send the patient to the laboratory for some screening tests. All of the
following are appropriate tests to order in this patient except
A Lipid profile
B CA-125
C TSH
D Urinalysis
ANSWER:B
574
A 75-year-old G2P2 presents to your GYN office for a routine exam. She tells you that she does not have an
internist and does not remember the last time she had a physical exam. She says she is very healthy and denies
taking any medication, including hormone replacement therapy. She is a nonsmoker and has an occasional
cocktail with her dinner. She does not have any complaints. In addition, she denies any family history of cancer.
The patient tells you that she is a widow and lives alone in an apartment in town. Her grown children have families
of their own and live far away. She states that she is self-sufficient and spends her time visiting friends and
volunteering at a local museum. Her blood pressure is 140/70. Her height is 5ft, 4 in. and she weighs 130 lB. Her
physical exam is completely normal. The urinalysis comes back positive for blooD. What would be the next
appropriate step in the management of this patient?
A Intravenous pyelogram
B Urine culture
C Renal sonogram
D Referral to urologist
ANSWER:B

575
A 75-year-old G2P2 presents to your GYN office for a routine exam. She tells you that she does not have an
internist and does not remember the last time she had a physical exam. She says she is very healthy and denies
taking any medication, including hormone replacement therapy. She is a nonsmoker and has an occasional
cocktail with her dinner. She does not have any complaints. In addition, she denies any family history of cancer.
The patient tells you that she is a widow and lives alone in an apartment in town. Her grown children have families
of their own and live far away. She states that she is self-sufficient and spends her time visiting friends and
volunteering at a local museum. Her blood pressure is 140/70. Her height is 5 ft, 4 in. and she weighs 130 lB. Her
physical exam is completely normal. What is the leading cause of death in women of this patient’s age?
A Breast cancer
B Lung cancer
C Heart disease
D Cerebrovascular disease
ANSWER:C
576
A 16-year-old Go African American presents to your office for a routine annual gynecologic exam. She reports
that she has previously been sexually active, but currently is not dating anyonE. She has had three sexual partners
in the past and says she diligently used condoms. She is a senior in high school and is doing well academically and
has many friends. She lives at home with her parents and a younger sibling. She denies any family history of
medical problems, but says her 80-year-old grandmother was recently diagnosed with breast cancer. She denies
any other family history of cancer. She says she is healthy and has no history of medical problems or surgeries.
She reports having had chickenpox. She smokes tobacco and drinks beer occasionally, but denies any illicit drug
usE. She had her first Pap smear and gynecologic exam last year with another doctor and reports that everything
was normal. Her menses started at age 13 and are regular and light. She denies any dysmenorrhea. Her blood
pressure is 90/60. Her height is 5 ft, 6 in. and she weighs 130 lB. What is the leading cause of death in teenagers?
A Suicide
B Homicide
C Motor vehicle accidents
D Cancer
ANSWER:C

577
A 16-year-old Go African American presents to your office for a routine annual gynecologic exam. She reports
that she has previously been sexually active, but currently is not dating anyone. She has had three sexual partners
in the past and says she diligently used condoms. She is a senior in high school and is doing well academically and
has many friends. She lives at home with her parents and a younger sibling. She denies any family history of
medical problems, but says her 80-year-old grandmother was recently diagnosed with breast cancer. She denies
any other family history of cancer. She says she is healthy and has no history of medical problems or surgeries.
She reports having had chickenpox. She smokes tobacco and drinks beer occasionally, but denies any illicit drug
usE. She had her first Pap smear and gynecologic exam last year with another doctor and reports that everything
was normal. Her menses started at age 13 and are regular and light. She denies any dysmenorrheA. Her blood
pressure is 90/60. Her height is 5 ft, 6 in. and she weighs 130 lB. All of the following are appropriate screening
tests to order in this patient except
A Pap test
B HIV test
C Gonorrhea and chlamydia cervical cultures
D Hepatitis C virus testing
ANSWER:D
578
A 16-year-old Go African American presents to your office for a routine annual gynecologic exam. She reports
that she has previously been sexually active, but currently is not dating anyonE. She has had three sexual partners
in the past and says she diligently used condoms. She is a senior in high school and is doing well academically and
has many friends. She lives at home with her parents and a younger sibling. She denies any family history of
medical problems, but says her 80-year-old grandmother was recently diagnosed with breast cancer. She denies
any other family history of cancer. She says she is healthy and has no history of medical problems or surgeries.
She reports having had chickenpox. She smokes tobacco and drinks beer occasionally, but denies any illicit drug
usE. She had her first Pap smear and gynecologic exam last year with another doctor and reports that everything
was normal. Her menses started at age 13 and are regular and light. She denies any dysmenorrheA. Her blood
pressure is 90/60. Her height is 5 ft, 6 in. and she weighs 130 lB. Which of the following vaccines is appropriate to
administer to this patient?
A Hepatitis A vaccine
B Pneumococcal vaccine
C Varicella vaccine
D Hepatitis B vaccine
ANSWER:D

579
A 16-year-old Go African American presents to your office for a routine annual gynecologic exam. She reports
that she has previously been sexually active, but currently is not dating anyonE. She has had three sexual partners
in the past and says she diligently used condoms. She is a senior in high school and is doing well academically and
has many friends. She lives at home with her parents and a younger sibling. She denies any family history of
medical problems, but says her 80-year-old grandmother was recently diagnosed with breast cancer. She denies
any other family history of cancer. She says she is healthy and has no history of medical problems or surgeries.
She reports having had chickenpox. She smokes tobacco and drinks beer occasionally, but denies any illicit drug
usE. She had her first Pap smear and gynecologic exam last year with another doctor and reports that everything
was normal. Her menses started at age 13 and are regular and light. She denies any dysmenorrheA. Her blood
pressure is 90/60. Her height is 5 ft, 6 in. and she weighs 130 lB. You explain to the patient how to perform a
breast self-exam. All of the following are important counseling issues to review with her except
A The exam should be performed in both the supine and standing positions
B The BSE should be performed monthly
C Asymmetry of the breasts is common
D The best time to perform the breast exam is immediately preceding menses
ANSWER:D
580
A 16-year-old Go African American presents to your office for a routine annual gynecologic exam. She reports
that she has previously been sexually active, but currently is not dating anyonE. She has had three sexual partners
in the past and says she diligently used condoms. She is a senior in high school and is doing well academically and
has many friends. She lives at home with her parents and a younger sibling. She denies any family history of
medical problems, but says her 80-year-old grandmother was recently diagnosed with breast cancer. She denies
any other family history of cancer. She says she is healthy and has no history of medical problems or surgeries.
She reports having had chickenpox. She smokes tobacco and drinks beer occasionally, but denies any illicit drug
usE. She had her first Pap smear and gynecologic exam last year with another doctor and reports that everything
was normal. Her menses started at age 13 and are regular and light. She denies any dysmenorrheA. Her blood
pressure is 90/60. Her height is 5 ft, 6 in. and she weighs 130 lB. What would be the most appropriate instrument
to use when performing the Pap smear test in this patient?
A Graves speculum
B Pederson speculum
C Pediatric speculum
D Vaginoscope
ANSWER:B

581
A 50-year-old woman is diagnosed with cervical cancer. Which lymph node group would be the first involved in
metastatic spread of this disease beyond the cervix and uterus? Select the one best response to each question.
A Common iliac nodes
B Parametrial nodes
C External iliac nodes
D Paracervical or ureteral nodes
ANSWER:D
582
Select the one best response to each question. A 21-year-old woman presents with left lower quadrant pain. An
anterior 7-cm firm adnexal cyst is palpateD. Ultrasound confirms a complex left adnexal mass with solid
components that appear to contain bone and teeth. What percentage of these tumors are bilateral?
A Less than 1%
B 2 to 3%
C 10 to 15%
D 50%
ANSWER:C
583
Select the one best response to each question. A 54-year-old woman undergoes a laparotomy because of a pelvic
mass. At exploratory laparotomy, a unilateral ovarian neoplasm is discovered that is accompanied by a large
omental metastasis. Frozen section diagnosis confirms metastatic serous cystadenocarcinoma. The most appropriate
intraoperative course of action is
A Excision of the omental metastasis and ovarian cystectomy
B Omentectomy and ovarian cystectomy
C Excision of the omental metastasis and unilateral oophorectomy
D Omentectomy, total abdominal hysterectomy, and bilateral salpingooophorectomy
ANSWER:D
584
Select the one best response to each question.A 58-year-old woman is seen for evaluation of a swelling in her right
vulvA. She has also noted pain in this area when walking and during coitus. At the time of pelvic examination, a
mildly tender, fluctuant mass is notedjust outside the introitus in the right vulva in the region of the Bartholin’s
glanD. What is the most appropriate treatment?
A Marsupialization
B Administration of antibiotics
C Surgical excision
D Incision and drainage
ANSWER:C

585
Select the one best response to each question.A 51-year-old woman is diagnosed with invasive cervical carcinoma
by cone biopsy. Pelvic examination and rectal-vaginal examination reveal the parametrium to be free of disease,
but the upper portion of the vagina is involved with tumor. Intravenous pyelography (IVP) and sigmoidoscopyare
negative, but a computed tomography C.T) scan of the abdomen and pelvis shows grossly enlarged pelvic and
periaortic nodes. This patient is classified as stage
A IIa
B IIb
C IIIa
D IIIb
ANSWER:A
586
A 35-year-old G3P3 with a Pap smear showing high-grade squamous intraepithelial lesion of the cervix C.IN III)
has an inadequate colposcopy.Cone biopsy shows squamous cell cancer that has invaded only 1 mm beyond the
basement membranE. There are no confluent tongues of tumor,and there is no evidence of lymphatic or vascular
invasion. The margins of the cone biopsy specimen are free of diseasE. How would you classify or stage this
patient’s disease?
A Carcinoma of low malignant potential
B Microinvasive cancer
C Atypical squamous cells of undetermined significance
D Carcinoma in situ
ANSWER:B
587
A 35-year-old G3P3 with a Pap smear showing high-grade squamous intraepithelial lesion of the cervix C.IN III)
has an inadequate colposcopy. Cone biopsy shows squamous cell cancer that has invaded only 1 mm beyond the
basement membranE. There are no confluent tongues of tumor, and there is no evidence of lymphatic or vascular
invasion. The margins of the cone biopsy specimen are free of diseasE. Of the following, appropriate therapy for
this lesion is
A External beam radiation
B Implantation of radioactive cesium
C Simple hysterectomy
D Simple hysterectomy with pelvic lymphadenectomy
ANSWER:C
588
A woman is found to have a unilateral invasive vulvar carcinoma that is 2 cm in diameter but not associated with
evidence of lymph node spreaD. Initial management should consist of
A Chemotherapy

B Radiation therapy
C Simple vulvectomy
D Radical vulvectomy and bilateral inguinal lymphadenectomy
ANSWER:D
589
A patient is receiving external beam radiation for treatment of metastatic endometrial cancer. The treatment field
includes the entire pelvis. Which of the following tissues within this radiation field is the most radiosensitive?
A Vagina
B Ovary
C Rectovaginal septum
D Bladder
ANSWER:B
590
An intravenous pyelogram (IVP) showing hydronephrosis in the workup of a patient with cervical cancer
otherwise confined to a cervix of normal size would indicate stage
A I
B II
C III
D IV
ANSWER:C
591
A pregnant 35-year-old patient is at highest risk for the concurrent development of which of the following
malignancies?
A Cervix
B Ovary
C Breast
D Vagina
ANSWER:A
592
Stage Ib cervical cancer is diagnosed in a young woman who wishes to retain her ability to have sexual
intercoursE. Your consultant has therefore recommended a radical hysterectomy. Assuming that the cancer is
confined to the cervix and that intraoperative biopsies are negative, which of the following structures would not be
removed during the radical hysterectomy?
A Uterosacral and uterovesical ligaments
B Pelvic nodes
C The entire parametrium on both sides of the cervix

D Both ovaries
ANSWER:D
593
A 24-year-old woman presents with new-onset right lower quadrant pain, and you palpate an enlarged, tender
right adnexA. Which of the following sonographic characteristics of the cyst in this patient suggests the need for
surgical exploration now instead of observation for one menstrual cycle?
A Lack of ascites
B Unilocularity
C Papillary vegetation
D Diameter of 5 cm
ANSWER:C
594
A 70-year-old woman presents for evaluation of a pruritic lesion on the vulvA. Examination shows a white, friable
lesion on the right labia majora that is 3 cm in diameter. No other suspicious areas are noteD. Biopsy of the lesion
confirms squamous cell carcinomA. In this patient, lymphatic drainage characteristically would be first to the
A External iliac lymph nodes
B Superficial inguinal lymph nodes
C Deep femoral lymph nodes
D Periaortic nodes
ANSWER:B
595
A 7-year-old girl is seen by her pediatrician for left lower quadrant pain. You are consulted because an ovarian
neoplasm is identified by ultrasounD.Of the following, the most common ovarian tumor in this type of patient is
A Germ cell
B Papillary serous epithelial
C Fibrosarcoma
D Brenner tumor
ANSWER:A
596
A 41-year-old woman undergoes exploratory laparotomy for a persistent adnexal mass. Frozen section diagnosis is
serous carcinomA. Assuming that the other ovary is grossly normal, what is the likelihood that the contralateral
ovary is involved in this malignancy?
A 5%
B 15%
C 33%

D 50%
ANSWER:C
597
A postmenopausal woman presents with pruritic white lesions on the vulvA. Punch biopsy of a representative area
is obtaineD. Which of the following histologic findings is consistent with the diagnosis of lichen sclerosus?
A Blunting or loss of rete pegs
B Presence of thickened keratin layer
C Acute inflammatory infiltration
D Increase in the number of cellular layers in the epidermis
ANSWER:A
598
At the time of annual examination, a patient expresses concern over exposure to sexually transmitted diseases.
During your pelvic examination, a singular, indurated, nontender ulcer is noted on the vulvA. Venereal Disease
Research Laboratory (VDRL) and fluorescent treponemal antibody (FTA) tests are positivE. Without treatment, the
next stage of this disease is clinically characterized by
A Optic nerve atrophy and generalized paresis
B Tabes dorsalis
C Gummas
D Macular rash over the hands and feet
ANSWER:D
599
A 24-year-old patient has returned from a yearlong stay in the tropics. Four weeks ago she noted a small vulvar
ulceration that spontaneously healeD. Now there is painful inguinal adenopathy with malaise and fever. You are
considering the diagnosis of lymphogranuloma venereum (LGV). The diagnosis can be established by
A Staining for Donovan bodies
B The presence of antibodies to Chlamydia trachomatis
C Positive Frei skin test
D Culturing Haemophilus ducreyi
ANSWER:B
600
One day after a casual sexual encounter with a bisexual man recently diagnosed as antibody-positive for human
immunodeficiency virus (HIV),a patient is concerned about whether she may have become infecteD. A negative
antibody titer is obtaineD. To test for seroconversion, when is the earliest you should reschedule repeat antibody
testing after the sexual encounter?
A 1 to 2 weeks
B 3 to 4 weeks
C 6 to 12 weeks

D 12 to 15 weeks
ANSWER:C
601
A 55-year-old postmenopausal female presents to her gynecologist for a routine exam. She denies any use of
hormone replacement therapy and does not report any menopausal symptoms. She denies the occurrence of any
abnormal vaginal bleeding. She has no history of any abnormal Pap smears and has been married for 30 years to
the same partner. She is currently sexually active with her husband on a regular basis. Two weeks after her exam,
her Pap smear comes back as atypical glandular cells of undetermined significance A.GUS). What is the next most
appropriate step in the management of this patient?
A Re-Pap in 4 to 6 months
B HPV testing
C Hysterectomy
D Colposcopy, endometrial biopsy, endocervical curettage
ANSWER:D
602
A 24-year-old G0 presents to your office complaining of vulvar discomfort.More specifically, she has been
experiencing intense burning and pain with intercoursE. The discomfort occurs at the vaginal introitus primarily
with penile insertion into the vaginA. The patient also experiences the same pain with tampon insertion and when
the speculum is inserted during a gynecologic exam. The problem has become so bad that she can no longer have
sex, which is causing problems in her marriagE. She is otherwise healthy and denies any medical problems. She is
experiencing regular menses and denies any dysmenorrheA. On physical exam, the region of the vulva around the
opening of the vagina appears erythematous and inflamed and is tender to touch with a cotton swab. What is the
most likely diagnosis?
A Vulvar vestibulitis
B Atrophic vaginitis
C Contact dermatitis
D Lichen sclerosus
ANSWER:A

603
A 24-year-old G0 presents to your office complaining of vulvar discomfort. More specifically, she has been
experiencing intense burning and pain with intercoursE. The discomfort occurs at the vaginal introitus primarily
with penile insertion into the vaginA. The patient also experiences the same pain with tampon insertion and when
the speculum is inserted during a gynecologic exam. The problem has become so bad that she can no longer have
sex, which is causing problems in her marriagE. She is otherwise healthy and denies any medical problems. She is
experiencing regular menses and denies any dysmenorrheA. On physical exam, the region of the vulva around the
opening of the vagina appears erythematous and inflamed and is tender to touch with a cotton swaB. All of the
following are appropriate treatments for this disorder except
A Tricyclic antidepressants
B Surgical excision of the vestibular glands
C Topical xylocaine
D Topical steroids
ANSWER:C
604
A 23-year-old G0 comes to your office complaining of a right nipple discharge that is bloody. She reports that the
discharge is spontaneous and not associated with any nipple pruritus, burning, or discomfort. On physical exam,
you do not detect any dominant breast masses or adenopathy. All of the following conditions should be
considered in the differential diagnosis except
A Breast cancer
B Duct ectasia
C Intraductal papilloma
D Pituitary adenoma
ANSWER:D
605
A 28-year-old G0, LMP 1 week ago, presents to your gynecology clinic complaining of a mass in her left breast
that she discovered on a routine breast self-exam in the shower. When you perform a breast exam on her, you
palpate a 2-cm firm, nontender mass in the upper inner quadrant of the left breast that is well circumscribed and
mobilE. You do not detectany skin changes, nipple discharge, or lymphadenopathy. What is this patient’s most
likely diagnosis?
A Fibrocystic breast change
B Fibroadenoma
C Breast carcinoma
D Fat necrosis
ANSWER:B

606
You have a patient who has undergone a routine screening ultrasound at 20 weeks gestation. The patient phones
you immediately following the ultrasound because during the procedure the radiologist commented that she has
several fibroid tumors in her uterus. This is the patient’s first pregnancy and she is most concerned regarding the
possible sequelae these growths may have on the outcome of her pregnancy. As her obstetrician, you counsel the
patient that all of the following are possible complications that can occur in the pregnancy as a result of
leiomyomas except
A Fibroid necrosis and degeneration
B Fetal malpresentation
C Progression to leiomyosarcoma
D Preterm labor
ANSWER:C
607
A 50-year-old G3P3 with a history of fibroids comes to see you complaining of menometrorrhagiA. Her LMP was
5 weeks ago and so heavy that she could not leave the house, fearing she would bleed through her clothes. She
also complains of occasional hot flushes and emotional lability. She does not have any medical problems and is
not taking any medications. She is a nonsmoker and denies any alcohol or drug usE. Her gynecologic history is
significant for cryotherapy to the cervix 10 years ago for moderate dysplasiA. She has had three cesarean sections
and a tubal ligation. All of the following are reasonable next steps in the evaluation of this patient except
A Fractional D and C and hysteroscopy
B Blood tests for TSH, PRL, and BHCG
C Pelvic ultrasound
D Conization of the cervix
ANSWER:D
608
An 18-year-old consults you for evaluation of disabling pain with her menstrual periods. The pain has been
present since menarche and is accompanied by nausea and headachE. History is otherwise unremarkable, and
pelvic examination is normal. You diagnose primary dysmenorrhea and recommend initial treatment with which
of the following?
A Ergot derivatives
B Antiprostaglandins
C Gonadotropin-releasing hormone (GnRH) analogues
D Danazol
ANSWER:B

609
A 28-year-old nulligravid patient complains of bleeding between her periods and increasingly heavy menses. Over
the past 9 months she has had two dilation and curettages D.&Cs), which have failed to resolve her symptoms, and
oral contraceptives and antiprostaglandins have not decreased the abnormal bleeding. Of the following options,
which is most appropriate at this time?
A Perform a hysterectomy
B Perform hysteroscopy
C Perform endometrial ablation
D Treat with a GnRH agonist
ANSWER:B
610
During the evaluation of secondary amenorrhea in a 24-year-old woman, hyperprolactinemia is diagnoseD. Which
of the following conditions could cause increased circulating prolactin concentration and amenorrhea in this
patient?
A Stress
B Primary hyperthyroidism
C Anorexia nervosa
D Congenital adrenal hyperplasia
ANSWER:A
611
A 45-year-old woman who had two normal pregnancies 15 and 18 years ago presents with the complaint of
amenorrhea for 7 months. She expresses the desire to become pregnant again. After exclusion of pregnancy,
which of the following tests is next indicated in the evaluation of this patient’s amenorrhea?
A Hysterosalpingogram
B Endometrial biopsy
C Thyroid function tests
D LH and FSH levels
ANSWER:D
612
A 22-year-old woman consults you for treatment of hirsutism. She is obese and has facial acne and hirsutism on
her face and periareolar regions and a male escutcheon. Serum LH level is 35 mIU/mL and FSH is 9 mIU/mL.
Androstenedione and testosterone levels are mildly elevated, but serum DHAS is normal. The patient does not wish
to conceive at this timE. Which of the following single agents is the most appropriate treatment of her condition?
A Oral contraceptives
B Corticosteroids
C GnRH

D Parlodel
ANSWER:A
613
An 18-year-old college student who has recently become sexually active is seen for severe primary
dysmenorrheA. She does not want to get pregnant, and has failed to obtain resolution with heating pads and mild
analgesics. Which of the following medications is most appropriate for this patient?
A Prostaglandin inhibitors
B Narcotic analgesics
C Oxytocin
D Oral contraceptives
ANSWER:D
614
A 19-year-old patient presents to your office with primary amenorrheA. She has normal breast and pubic hair
development, but the uterus and vagina are absent. Diagnostic possibilities include
A XYY syndrome
B Gonadal dysgenesis
C Müllerian agenesis
D Klinefelter syndrome
ANSWER:C
615
A 23-year-old woman presents for evaluation of a 7-month history of amenorrheA. Examination discloses
bilateral galactorrhea and normal breast and pelvic examinations. Pregnancy test is negativE. Which of the
following classes of medication is a possible cause of her condition?
A Antiestrogens
B Gonadotropins
C Phenothiazines
D Prostaglandins
ANSWER:C
616
A 9-year-old girl has breast and pubic hair development. Evaluation demonstrates a pubertal response to a
gonadotropin-releasing hormone (GnRH) stimulation test and a prominent increase in luteinizing hormone (LH)
pulses during sleep. These findings are characteristic of patients with
A Theca cell tumors
B Iatrogenic sexual precocity
C Premature thelarche
D Constitutional precocious puberty
ANSWER:D

617
A 29-year-old G0 who comes to your OB/GYN office complaining of PMS. On taking a more detailed history,
you learn that the patient suffers from emotional lability and depression for about 10 days prior to hermenses. She
reports that once she begins to bleed she feels back to normal.The patient also reports a long history of
premenstrual fatigue, breast tenderness, and bloating. Her previous health care provider placed her on oral
contraceptives to treat her PMS 6 months ago. She reports that the pills have alleviated all her PMS symptoms
except for the depression and emotional symptoms. Which of the following would be the best treatment for this
patient’s problem?
A Spironolactone
B Evening primrose oil
C Fluoxetine
D Progesterone supplements
ANSWER:C
618
A 50-year-old woman complains of leakage of urinE. After genuine stress urinary incontinence, the most common
cause of urinary leakage is
A Detrusor dyssynergia
B Unstable bladder
C Unstable urethra
D Urethral diverticulum
ANSWER:B
619 A 65-year-old woman complains of leakage of urinE. The most common cause of this condition in such patients is
A Anatomic stress urinary incontinence
B Urethral diverticula
C Overflow incontinence
D Unstable bladder
ANSWER:D
620
A 59-year-old woman undergoes vaginal hysterectomy and anteroposterior repair for uterine prolapsE. Which of
the following is a complication of this procedure that often develops within 2 weeks of surgery?
A Dyspareunia
B Stress urinary incontinence
C Nonfistulous fecal incontinence
D Enterocele
ANSWER:B

621
A 53-year-old postmenopausal woman, gravida 3, para 3, presents for evaluation of troublesome urinary leakage
6 weeks in duration. Of the following choices, which is the most appropriate first step in this patient’s evaluation?
A Urinalysis and culture
B Urethral pressure profiles
C Intravenous pyelogram
D Cystourethrogram
ANSWER:A
622
A postmenopausal woman is undergoing evaluation for fecal incontinencE. She has no other diagnosed medical
problems. She lives by herself and is self-sufficient, oriented, and an excellent historian. Physical examination is
completely normal. Which of the following is the most likely cause of this patient’s condition?
A Rectal prolapse
B Diabetes
C Obstetric trauma
D Senility
ANSWER:C
623
You are discussing surgical options with a patient with symptomatic pelvic relaxation. Partial colpocleisis (Le Fort
procedure) may be moreappropriate than vaginal hysterectomy and anterior and posterior A.&P) repair for
patients who
A Do not desire retained sexual function
B Need periodic endometrial sampling
C Have had endometrial dysplasia
D Have cervical dysplasia that requires colposcopic evaluation
ANSWER:A
624
For the treatment of stress urinary incontinence, you are assisting in a procedure in which the periurethral tissue is
attached to the symphysis pubis. The disadvantages of this Marshall-Marchetti-Krantz procedure compared with
other surgical alternatives for treatment of stress urinaryincontinence include which of the following?
A Urinary retention
B Increased incidence of urinary tract infections
C High failure rate
D Osteitis pubis
ANSWER:D

625
A patient is seen on the second postoperative day after a difficult abdominal hysterectomy complicated by
hemorrhage from the left uterine artery pediclE. Multiple sutures were placed into this area to control bleeding.
The patient now has fever, left back pain, left costovertebral angle tenderness, and hematuriA. An ultrasound
examination shows that fluid has accumulated in the left flank. A ureteral injury is diagnoseD. If the injury had
been recognized at the time of surgery, which of the following procedures could have been recommended?
A Percutaneous nephrostomy
B Placement of a ureteral stent without anastomosis
C Intraperitoneal drainage without anastomosis
D Ureteral reimplantation into the bladder
ANSWER:D
626
A 59-year-old G4P4 presents to your GYN office complaining of losing urine when she coughs, sneezes, or
engages in certain types of strenuous physical activity. The problem has gotten increasingly worse over the past
few years, to the point where the patient finds her activities of daily living compromised secondary to fear of
embarrassment. She denies any other urinary symptoms such as urgency, frequency, or hematuriA. In addition,
she denies any problems with her bowel movements. Her prior surgeries include a tonsillectomy and
appendectomy. She has adult-onset diabetes and her blood sugars are well controlled with oral glucophagE. The
patient has no history of any gynecologic problems in the past. She has four children that were delivered via
spontaneous vaginal deliveries; their weights ranged between 8 and 9 lB. She is currently sexually active with her
partner of 25 years. She has been menopausal for 4 years and has never taken any hormone replacement therapy.
Her height is 5 ft, 6 in., and she weighs 190 lB. Her blood pressure is 130/80. Based on the patient’s history, what
is the most likely diagnosis?
A Overflow incontinence
B Stress incontinence
C Urinary tract infection
D Detrusor instability
ANSWER:B
627
A 90-year-old G5P5 with multiple medical problems is brought into your gynecology clinic accompanied by her
granddaughter. The patient has hypertension, chronic anemia, coronary artery disease, and osteoporosis. She is
mentally alert and oriented and lives in an assisted living facility. She takes numerous medications, but is very
functional at the current timE. She is a widow and not sexually activE. Her chief complaint is a sensation of
heaviness and pressure in the vaginA. She denies any significant urinary or bowel problems. On performance of a
physical exam, you note that the cervix is at the level of the introitus. Based on the physical exam, what is the most
likely diagnosis?

A Normal exam
B First-degree uterine prolapse
C Second-degree uterine prolapse
D Third-degree uterine prolapse
ANSWER:C
628
A 40-year-old G3P3 comes to your office for a routine annual GYN exam. She tells you that she gets up several
times during the night to voiD. On further questioning, she admits to you that during the day she sometimes gets
the urge to void, but sometimes cannot quite make it to the bathroom. She attributes this to getting older and is not
extremely concerned, although she often wears a pad when she goes out in case she loses some urinE. This patient
is very healthy otherwise and does not take any medication on a regular basis. She has had three normal
spontaneous vaginal deliveries of infants weighing between 7 and 8 lB. An office dipstick of her urine does not
indicate any blood, bacteria, WBCs, or protein. Based on her office presentation and history, what is this patient’s
most likely diagnosis?
A Urinary stress incontinence
B Urinary tract infection
C Overflow incontinence
D Bladder dyssynergia
ANSWER:D
629
An 18-year-old G0 comes to see you complaining of a 3-day history of urinary frequency, urgency, and dysuriA.
She panicked this morning when she noticed the presence of bright red blood in her urinE. She also reports some
midline lower abdominal discomfort. She had intercourse for the first time 5 days ago and reports that she used
condoms. On physical exam, there is no discharge from the cervix or in the vagina and the cervix appears normal.
Bimanual exam is normal except for mild suprapubic tenderness. There is no flank tenderness, and the patient’s
temperature is normal. What is the most likely diagnosis?
A Chlamydia cervicitis
B Pyelonephritis
C Cystitis
D Bladder dyssynergia
ANSWER:C
630
A 22-year-old nulliparous woman has recently become sexually activE. She consults you because of painful
coitus, with the pain located at the vaginal introitus. It is accompanied by painful involuntary contraction of the
pelvic muscles. Other than confirmation of these findings, the pelvic examination is normal. Of the following,
what is the most common cause of this condition?

A Endometriosis
B Psychogenic causes
C Bartholin’s gland abscess
D Vulvar atrophy
ANSWER:B
631
A 62-year-old woman presents for annual examination. Her last spontaneous menstrual period was 9 years ago,
and she has been reluctant to use postmenopausal hormone replacement because of a strong family history of
breast cancer. She now complains of diminished interest in sexual activity. Which of the following is the most
likely cause of her complaint?
A Decreased vaginal length
B Decreased ovarian function
C Alienation from her partner
D Untreatable sexual dysfunction
ANSWER:B
632
A 39-year-old patient is contemplating discontinuing birth control pills in order to conceivE. She is concerned
about her fertility at this age, and inquires about when she can anticipate resumption of normal menses. You
counsel her that by 3 months after discontinuation of birth control pills, the following proportion of patients will
resume normal menses
A 99%
B 95%
C 80%
D 50%
ANSWER:C
633
A 31-year-old, gravida 3, para 3 Jehovah’s Witness begins to bleed heavily 2 days after a cesarean section. She
refuses transfusion and says that she would rather die than receive any blood or blood products. You personally
feel that you cannot watch her die and do nothing. Appropriate actions that you can take under these
circumstances include
A Telling the patient to find another physician who will care for her
B Transfusing her forcibly
C Letting her die, giving only supportive care
D Getting a court order and transfusing
ANSWER:C

634
A 27-year-old woman who has previously received no prenatal care presents at term. On ultrasound, she is shown
to have a placenta previa, but she refuses a cesarean section for any reason. Important points to consider in her
management include
A The obstetrician’s obligation to the supposedly normal fetus supersedes the obligation to the healthy mother
B The inclusion of several people in this complex situation raises the legal risk to the physician
C Child abuse statutes require the physician to get a court order to force a cesarean section
D A hospital ethics committee should be convened to evaluate the situation
ANSWER:D
635
Your patient is a 44-year-old G4P4 with symptomatic uterine fibroids that are unresponsive to medical therapy.
The patient has severe menorrhagia to the point that when she gets her menses, she cannot leave the housE. You
recommend to her that she undergo a total abdominal hysterectomy.You counsel her that she may need a blood
transfusion if she suffers a lot of bleeding during the surgical procedurE. Her current hematocrit is 25.0. The
patient is a Jehovah’s Witness who adamantly refuses to have a blood transfusion, even if it results in her death. All
of the following are ethical concerns that need to be considered when working through this case except
A Legal issues
B Patient preferences
C Quality of life issues
D Medical indications
ANSWER:A
636
Your patient is a 44-year-old G4P4 with symptomatic uterine fibroids that are unresponsive to medical therapy.
The patient has severe menorrhagia to the point that when she gets her menses, she cannot leave the housE. You
recommend to her that she undergo a total abdominal hysterectomy.You counsel her that she may need a blood
transfusion if she suffers a lot of bleeding during the surgical procedurE. Her current hematocrit is 25.0. The
patient is a Jehovah’s Witness who adamantly refuses to have a blood transfusion, even if it results in her death.
The patient’s insurance company refuses to pay for the surgical procedurE. The ethical area involved is
A Autonomy
B Justice
C Contextual issue
D Patient preference
ANSWER:B

637
Your patient is a 44-year-old G4P4 with symptomatic uterine fibroids that are unresponsive to medical therapy.
The patient has severe menorrhagia to the point that when she gets her menses, she cannot leave the housE. You
recommend to her that she undergo a total abdominal hysterectomy. You counsel her that she may need a blood
transfusion if she suffers a lot of bleeding during the surgical procedurE. Her current hematocrit is 25.0. The
patient is a Jehovah’s Witness who adamantly refuses to have a blood transfusion, even if it results in her death.
Respect for the patient’s autonomy or own wishes requires that which of the following be assessed?
A The needs of society
B The duty not to inflict harm
C The impact that the treatment will have on the patient’s quality of life
D Consideration of what is the best treatment
ANSWER:A
638
Your patient is a 44-year-old G4P4 with symptomatic uterine fibroids that are unresponsive to medical therapy.
The patient has severe menorrhagia to the point that when she gets her menses, she cannot leave the housE. You
recommend to her that she undergo a total abdominal hysterectomy.You counsel her that she may need a blood
transfusion if she suffers a lot of bleeding during the surgical procedurE. Her current hematocrit is 25.0. The
patient is a Jehovah’s Witness who adamantly refuses to have a blood transfusion, even if it results in her death.
Prior to performing the abdominal hysterectomy, you give the patient informed consent. All of the following are
key elements of informed consent except
A The patient must have the ability to comprehend medical information
B Alternatives to the procedure must be presented
C If the patient is incapable of providing consent, the procedure should be abandoned
D The risks of the procedure must be presented
ANSWER:C
639
Your patient is a 44-year-old G4P4 with symptomatic uterine fibroids that are unresponsive to medical therapy.
The patient has severe menorrhagia to the point that when she gets her menses, she cannot leave the housE. You
recommend to her that she undergo a total abdominal hysterectomy.You counsel her that she may need a blood
transfusion if she suffers a lot of bleeding during the surgical procedurE. Her current hematocrit is 25.0. The
patient is a Jehovah’s Witness who adamantly refuses to have a blood transfusion, even if it results in her death.
The patient requests that you do not talk at all to husband about her medical carE. This request falls under the
ethical concept of
A Informed consent
B Confidentiality

C Nonmaleficence
D Advanced directive
ANSWER:B
640
Ms Jones is a 28-year-old female who has agreed to be a gestational surrogate for a couple who cannot bear
children. She presents to your office for prenatal carE. Which of the following is your responsibility as an
obstetrician caring for a gestational surrogate?
A Consult with intended parents regarding all clinical interventions and management of the pregnancy.
B
Discuss the health of the surrogate and progress of the pregnancy with the intended parents without consent of the
surrogate mother.
C
Make recommendations for prenatal care in accordance with the agreement between the gestational surrogate and
the intended parents.
D Provide appropriate care regardless of the patient’s plans to keep or relinquish the future chilD.
ANSWER:D
641
A 24-year-old patient who you have been seeing for routine gynecological care reports that she is considering
becoming a surrogate mother for a couple she knows at work. As her physician, what is your responsibility to her
in preparing her to become a surrogate?
A Contact the intended parents so that you can provide care for them also.
B Explain to her that you will require an additional fee to care for her pregnancy since she will be a surrogatE.
C Recommend that she utilize the same legal counsel as the intended parents.
D Refer her to mental health counseling.
ANSWER:D
642
A 63-year-old patient is seen for routine examination. An excoriated 2-cm lesion is found on her left labium
majus, which, she states, has been present for at least 3 months. What is the next best step in the management of
this patient?
A prescribe hydrocortisone cream
B schedule colposcopy
C perform excisional biopsy
D prescribe Burow’s solution soaks
ANSWER:C
643
An 18-year-old woman consults you for a painful swelling of her left labium that has progressively worsened over
the past 3 days. She has been treating the discomfort with over-the-counter analgesics and warm sitz baths. On
examination, a 6-cm swollen, red, tender, tense cystic mass is present in the base of the left labium majus. What is
the most appropriate next step in the care of this patient?

A excision of the mass
B dry heat
C oral antibiotics
D incision and drainage of the mass
ANSWER:D
644
A 21-year-old G0P0 healthy college student presents to Student Health Center, complaining of severe vulvar
puritius. She has a BMI of 24, uses condoms with coitus, and finished her last menses 4 days prior. Last month she
was diagnosed with and successfully treated for manila vaginitis. She denies any other symptoms including vaginal
dischargE. What is the most likely diagnosis?
A vaginal trichomoniasis
B leukemia
C personal hygiene products
D secondary syphilis
ANSWER:C
645
\A 79-year-old woman presents to your office with a 1-cm fleshy outgrowth from her urethrA. It has a slightly
infected appearance and bleeds on contact. You perform a biopsy, and the report states “transitional and stratified
squamous epithelium with underlying loose connective tissuE.” Which of the following is the most likely
diagnosis?
A urethral leiomyoma
B hidradenitis suppurativa
C senile urethritis
D urethral caruncle
ANSWER:D
646
A patient consults you with complaints of recurrent, painful, draining vulvar lesions. Examination shows multiple
abscesses and deep scars in the labiA. A foul-smelling discharge from the lesions is noteD. During the review of
systems, the patient reports the occasional appearance of similar lesions in the axillA. Which of the following is the
most likely diagnosis?
A herpetic vulvitis
B hidradenitis suppurativa
C lymphogranuloma venereum
D granuloma inguinale
ANSWER:B

647
A 20-year-old patient complains of painful vulvar ulcers present for 72 hours. Examination reveals three tender,
punched-out lesions with a yellow exudate but no induration. Which of the following is the most likely diagnosis?
A chancroid
B granuloma inguinale
C herpes
D lymphogranuloma venereum
ANSWER:A
648
A 17-year-old girl is seen at a local clinic desiring contraception because she thinks she will soon become sexually
activE. During her examination, an ulcerative lesion is seen in the vaginal fornix. It has a rolled, irregular edge
with a reddish-appearing granular basE. The lesion is mildly tender to palpation. This lesion is most likely which
of the following?
A vaginal intraepithelial neoplasia
B vulvar carcinoma
C syphilis
D an ulcer caused by the use of tampons
ANSWER:D
649 Which of the following is the most common benign neoplasm of the cervix and endocervix?
A polyp
B leiomyoma
C nabothian cyst
D endometriosis
ANSWER:A
650
A 15-year-old patient has had menstrual bleeding every 2 to 4 weeks since menarche 1 year ago. The bleeding can
be both heavy and light. It sometimes lasts as long as 2 weeks. Which of the following is the next best step in the
management of her problem?
A obtain a pregnancy test
B perform an endometrial biopsy
C obtain pelvic ultrasonography
D initiate cyclic progestin therapy
ANSWER:D
651
A 47-year-old woman complains of postcoital bleeding, nearly as heavy as menses. Which of the following is the
most likely origin of her bleeding?

A cervical polyps
B cervical ectropion
C cervical carcinoma
D cervical nabothian cysts
ANSWER:C
652
An obese 63-year-old woman presents with a 3-month history of continuous scanty vaginal bleeding. She denies
the use of hormone replacement therapy. Adequate history and physical examination in the office reveal no other
abnormalities. A Pap smear is negativE. Which of the following is the next most appropriate step in her
management?
A begin estrogen replacement therapy
B sample the endometrium
C perform colposcopic evaluation of the cervix
D obtain random biopsies of the cervix
ANSWER:B
653
A patient being treated for prothrombin deficiency develops abnormal uterine bleeding. An anatomic lesion has
been ruled out. Further management to control the bleeding should begin with which of the following?
A gonadotropin-releasing hormone (GnRH) antagonists
B medroxyprogesterone acetate
C conjugated equine estrogens
D OC pills
ANSWER:D
654
A patient complains of heavy but regular menstrual periods. An anatomic cause of the magnitude of her flow has
been ruled out. Which of the following has been shown to be most effective in reducing rather than eliminating
her menstrual flow?
A tranexamic acid
B dilation and curettage
C depot medroxyprogesterone acetate D.MPA)
D misoprostol
ANSWER:A
655
A patient in her forties presents with dysfunctional bleeding. You want to do an endometrial biopsy. Because she
has no insurance, she would prefer not to have the procedure unless it is likely to show important pathology. An
endometrial sampling is likely to be reported as showing endometrial hyperplasia in a patient who is which of the
following?

A obese
B postmenopausal
C using cyclic combination OCs
D using DMPA
ANSWER:A
656
A 35-year-old accountant complains of episodic bloating, breast tenderness, dyspareunia, irritability, and
depression, which leave her with “only 1 good week a month.” She is currently using condoms and foam for birth
control because she “felt terrible” on OCs. Pelvic examination is normal. Which of the following is the best
diagnostic course?
A begin a prospective diary of symptoms for the next 2 months
B obtain a serum progesterone level during the last half of her menstrual cycle
C obtain a serum estrogen level during the first half of her menstrual cycle
D perform a transvaginal ultrasound examination of the posterior cul-de-sac
ANSWER:A
657
A 33-year-old patient has been diagnosed as having adenomyosis. Which of the following symptoms is most
consistent with this diagnosis?
A dyspareunia
B mood swings
C painful defecation
D secondary dysmenorrhea
ANSWER:D
658
A patient has secondary dysmenorrheal and a fixed pelvis. At laparoscopy, lesions are biopsied that are thought to
represent endometriosis. The diagnosis of endometriosis is confirmed histologically by identifying extragenital
implants containing which of the following?
A endometrial glands and stroma
B hypertrophic smooth muscle
C hemorrhage and iron pigment deposits
D fibrosis
ANSWER:A
659
Which of the following is the most common indication for treatment of uterine leiomyomata in a 42-year-old
woman?
A interference with reproductive function
B rapid enlargement

C pain
D excessive uterine bleeding
ANSWER:D
660
A 45-year-old patient with uterine leiomyomata found on pelvic examination complains of excessive uterine
bleeding. Which of the following should be the next step in the management of this patient?
A myomectomy
B hysterectomy
C ultrasonography
D endometrial biopsy
ANSWER:D
661
A 26-year-old patient is found to have an 8-week size, irregular uterus. She does not complain of pain or
excessive menstrual bleeding. Her Pap smear is normal, and a pregnancy test is negativE. Which of the following
is the best step in the management of this patient?
A continued observation
B endometrial biopsy
C cervical conization
D hysterectomy
ANSWER:A
662
A 23-year-old woman complains of heavy, painful menstrual periods every 2 weeks. On further questioning, you
find that every other episode of bleeding is actually very brief, consisting of only 2 days of spotting. At these
times, the pain is also only an occasional twingE. During the heavy bleeding, the pain is crampy, nearly constant,
located centrally in the pelvis, and lasts 3 days. She reports that this has been her pattern of menstrual pain since
her early teens. A BBT curve is biphasic, compatible with normal ovulatory cycles 28 days in length. Her physical
examination is normal. In addition to primary dysmenorrhea, which of the following is the most likely diagnosis?
A anovulatory bleeding
B progressive endometriosis
C chronic constipation
D mittelschmerz
ANSWER:D

663
A 25-year-old patient with her last menstrual period (LMP) 3 weeks ago is being followed for a 5 cm x 4 cm x 4
cm right ovarian cystic mass. She comes to the emergency department complaining of sudden right-sided low
abdominal pain and nausea that has been constant for 2 hours. She had intermittent spasms of pain for a week
preceding this episode (when you first felt the cyst). All these pain episodes resolved within minutes. The patient
denies fever or recent coitus (none in 6 months). Examination demonstrates a 10 cm x 8 cm x 6 cm right pelvic
mass that is very tender. White blood cell (WBC) count is 12,500/mL and temperature is 100.2°F. She has had no
prior surgery. The patient undergoes diagnostic laparoscopy, and a black mass is seen replacing the entire right
ovary. Which of the following is the most appropriate management of this patient?
A removal of the ovary
B antibiotic therapy
C Clostridium antitoxin
D reverse torsion and oophoropexy
ANSWER:A
664
A 58-year-old G2P2 patient presents with complaints of severe vulvar pruritus. She is 10 years postmenopausal.
Her examination is consistent with atrophic vulvitis. Which of the following is the most effective treatment of
vulvar pruritus associated with atrophic vulvitis?
A antihistamines
B hydrocortisone
C alcohol injections
D topical estrogen therapy
ANSWER:D
665
A 53-year-old woman is diagnosed with anovulatory dysfunctional bleeding. Which of the following is the most
appropriate medical therapy?
A orally administered estrogen for the first 25 days of each month
B vaginal estrogen cream two to three times per week
C orally administered progesterone 5 to 10 mg daily for 10 days each month
D testosterone tablets 10 mg/d
ANSWER:C
666
A 63-year-old patient presents with symptoms of vaginal itching, vaginal dryness, and dys-pareuniA. Which of the
following is the most appropriate medical therapy?
A orally administered estrogen for the first 25 days of each month
B vaginal estrogen cream daily
C orally administered progesterone 5 to 10 mg daily for 10 days each month

D testosterone tablets 10 mg/d
ANSWER:B
667
A 19-year-old woman is seen in the emergency department with a history of amenorrhea for 8 weeks, 1 week of
unilateral adnexal pain. On physical examination, she is found to have an acute abdomen with tenderness and
absent bowel sounds. Laboratory evaluations reveal a hematocrit that is 23%, and a positive pregnancy test. Which
of the following is the most likely diagnosis?
A ectopic pregnancy
B pelvic inflammatory disease (PID)
C endometriosis
D appendicitis
ANSWER:A
668
A 23-year-old G1P1 patient is using barrier contraception and is 1 week past onset of her last mensE. She is found
to have bilaterally equal adnexal pain; cervical motion tenderness; direct abdominal tenderness; temperature,
101.3°F; and WBC, 12,000/mL. Which of the following is the most likely diagnosis?
A ectopic pregnancy
B PID
C endometriosis
D urinary tract infection (UTI)
ANSWER:B
669
A 16-year-old G0P0 patient reports delayed onset of menses, the sudden onset of severe pain, and syncopE. A
serum pregnancy test is negativE. Her CBC reveals an Hct of 42% and a WBC of 8,000. Which of the following is
the most likely diagnosis?
A ectopic pregnancy
B PID
C endometriosis
D ruptured corpus luteum cyst of the ovary
ANSWER:D
670
A 21-year-old patient is seen for a physical examination prior to her return to collegE. She has been healthy and is
using OCs for the past 3 years. On physical examination, you note a 2-mm pigmented flat lesion with irregular
margins on the left labiA. What is the most appropriate next step in the management of this lesion?
A follow up in 6–12 months
B discontinue OCs
C excisional biopsy of the lesion

D wide local excision of lesion with 5 mm margins
ANSWER:C
671
A 36-year-old patient presents for evaluation of complaints of chronic vaginal infection. She reports little vaginal
discharge, but rather a 1-year history of progressively worsening vulvar discomfort that has escalated to pain
sufficient to preclude intercourse and tampon usE. Inspection of the vulva demonstrates focal inflammation,
punctation, and ulceration of the perineal and vaginal epithelium. An attempt to perform a bimanual examination
of the pelvic organs reveals intense pain and tenderness at the posterior introitus and vestibulE. Which of the
following is the most appropriate next step in the management of this lesion?
A topical anesthetics and antidepressant treatment
B reduction of dietary oxylates
C excisional biopsy of the lesions
D interferon injections of the vaginal introitus
ANSWER:A
672
A 32-year-old G0P0 patient presents complaining of secondary dysmenorrhea that is increasing in severity. The
pain is triggered by deep thrusting with coitus. Which of the following is the most common cause of deep-thrust
dyspare-unia?
A endometriosis
B depression
C vaginismus
D vestibulitis
ANSWER:A
673
Treatments of primary dysmenorrhea are directed toward addressing the cause, which is associated with elevations
in which of the following?
A estrogen
B progesterone
C FSH
D prostaglandin F2alpha
ANSWER:D
674
A 20-year-old woman at 12 weeks’ gestation is involved in a serious automobile accident and is brought to the
emergency department with multiple traumas. The emergency department physician believes that imaging studies
of the abdomen are needed to assess the patient’s acute injuries. Regarding this imaging, what should you counsel
the managing team?
A Imaging at this stage of pregnancy should not be carried out.

B Imaging should be limited to no more than two views of the abdomen.
C Imaging can only be done if the uterus is shielded during the procedurE.
D There are no contraindications to the needed tests.
ANSWER:D
675 Uterine leiomyomata are thought to arise from which of the following?
A embryonic rests
B vascular smooth muscle cells
C degenerative uterine smooth muscle cells
D pluripotent endometrial epithelium
ANSWER:B
676
On pelvic examination of a 28-year-old mul-tiparous patient, several 3–5 mm yellowish translucent or opaque,
raised cystic structures are seen on the surface of the cervix. The patient is asymptomatiC. What is the most
appropriate next step in the management of these findings?
A excisional biopsy
B incision and drainage of cysts
C oral antibiotic therapy
D counseling and reassurance
ANSWER:D
677
A 56-year-old woman has a biopsy-proven vulvar intraepithelial neoplasia (VIN III). She undergoes a wide
excision and returns 3 months later with vulvar pruritus. What should you advise the patient?
A Steroid cream on the vulva will reduce the itching.
B She may need a repeat biopsy.
C There is minimal chance of cancer.
D There is minimal chance of recurrencE.
ANSWER:B
678
A 65-year-old woman presents with complaints of vulvar redness, pruritus, and occasional weeping from the skin.
Examination reveals ery-thematous, eczematoid of the labia minora, and periclitoral areA. This is consistent with
Paget’s disease of the vulvA. Which of the following characterizes Paget’s disease of the vulva?
A recurrences are infrequent after treatment
B frequent association with other invasive carcinomas
C appears as a solitary hypopigmented lesion
D is treated with laser vaporization
ANSWER:B

679 Which of the following types of vulvar cancer occurs most commonly?
A Paget’s
B squamous
C melanoma
D adenocarcinoma
ANSWER:B
680
A 48-year-old woman presents with a large verrucous lesion of her vulvA. It is not particularly painful, but the
appearance is worrisome to the patient. Such a lesion is most likely which of the following?
A clear cell carcinoma
B condyloma acuminata
C adenocarcinoma
D hidradenoma
ANSWER:B
681 Which of the following is the most common symptom of vulvar carcinoma in elderly women?
A abnormal bleeding
B a foul smell
C pruritus
D vulvar atrophy
ANSWER:C
682
A 1-cm vulvar carcinoma with tumor-positive unilateral nodes and no distant spread would be in which FIGO
(International Federation of Gynecology and Obstetrics) stage?
A I
B II
C III
D IV
ANSWER:C
683
A 58-year-old woman has a 1-cm vulvar ulcer. A biopsy shows invasive squamous cell carcinoma with more than
1 mm of stromal invasion. Which of the following is the preferred treatment?
A Burow’s soaks
B 5-fluorouracil (5-FU) cream
C radiotherapy
D radical local excision and ipsilateral inguinofemoral lymph node dissection
ANSWER:D

684
A 58-year-old woman has a 1-cm vulvar ulcer. A biopsy shows invasive squamous cell carcinoma with more than
1 mm of stromal invasion. If the lymph nodes in this case are negative, the 5-year survival should be
approximately what percentage?
A 12
B 25
C 52
D 90
ANSWER:D
685
A 58-year-old woman has a 1-cm vulvar ulcer. A biopsy shows invasive squamous cell carcinoma with more than
1 mm of stromal invasion. This patient undergoes radical vulvectomy. Which of the following is the most
common complication of radical vulvectomy?
A debilitating edema of the lower extremities
B pulmonary embolism
C necrotizing fasciitis
D breakdown of the surgical wound
ANSWER:D
686
A 72-year-old woman has had a radical vulvectomy for stage II squamous cell vulvar cancer. She wants to know
the most likely site of recurrence if the tumor comes back. Where would the tumor most likely appear?
A at the site of tumor resection
B in the bladder or rectum
C in the scalene lymph nodes
D the chest
ANSWER:A
687 Which of the following tumors of the vulva has the best prognosis?
A stage I verrucous carcinoma
B melanoma
C stage I squamous cell cancer of vulva
D basal cell carcinoma
ANSWER:D
688
A 56-year-old woman presents with painless mild vaginal spotting. She had a hysterectomy at age 40 for persistent
cervical dysplasiA. She is otherwise healthy and takes no medications. On further review of symptoms, she has
occasional urgency and dysuriA. On pelvic examination, a 0.5-cm lesion is felt and visualized in the anterior
vaginA. What is the next step in the evaluation or treatment of this lesion?

A refer to gynecologic oncologist
B perform directed punch biopsy of the lesion
C perform Papanicolaou (Pap) smear of the vagina and vaginoscopy
D perform laser ablation therapy
ANSWER:B
689
A 30-year-old woman presents for her annual examination. On history, she reports that her mother was prescribed
diethylstilbestrol D.ES) during the pregnancy with her. Which of the following conditions is she most at risk for as
a result?
A endometrial adenocarcinoma
B ovarian adenocarcinoma
C clear cell adenocarcinoma of the vagina
D ovarian cysts
ANSWER:C
690
Which of the following is a malignant tumor of the vagina of young children that appears clinically as a mass of
grape-like edematous polyps?
A emphysematous vaginitis
B squamous cell carcinoma
C sarcoma botryoides
D adenocarcinoma
ANSWER:C
691
A 72-year-old woman complains of vaginal bleeding. On evaluation, a 2-cm vaginal lesion is found in the upper
third of the anterior vaginA. On bimanual and rectovaginal examination, the mass extends to the lateral pelvic
wall. On biopsy, vaginal carcinoma is confirmeD. What stage cancer does this patient most likely have?
A 0
B I
C II
D III
ANSWER:D
692
A 72-year-old woman complains of vaginal bleeding. On evaluation, a 2-cm vaginal lesion is found in the upper
third of the anterior vaginA. On bimanual and rectovaginal examination, the mass extends to the lateral pelvic
wall. On biopsy, vaginal carcinoma is confirmeD. Which of the following is the most likely histology of vaginal
carcinoma in this woman?
A melanoma

B verrucous
C clear cell
D squamous cell
ANSWER:D
693
A 72-year-old woman complains of vaginal bleeding. On evaluation, a 2-cm vaginal lesion is found in the upper
third of the anterior vaginA. On bimanual and rectovaginal examination, the mass extends to the lateral pelvic
wall. On biopsy, vaginal carcinoma is confirmeD. Which of the following is the best treatment for her?
A total vaginectomy
B upper vaginectomy
C chemotherapy
D combination radiation and chemotherapy
ANSWER:D
694 Which of the following is the most common method used to diagnose cervical intraepithe-lial neoplasia C.IN​?
A complaints of abnormal discharge
B postcoital bleeding
C chronic pelvic pain
D abnormal Pap smears
ANSWER:D
695 Which of the following reflects the etiology of cervical dysplasia and cervical cancer?
A Human papillomavirus (HPV) is the major causal agent.
B They are associated with obesity.
C They are associated with nulliparity.
D There is a strong genetic component to the development of cervical cancer.
ANSWER:A
696 Which of the following reflects HPV?
A Only 20% of sexually experienced women will be infected with HPV.
B The virus is transient for most women.
C Most women with HPV will go on to develop warts, CIN, or cancer.
D
Other cofactors such as cigarette smoking and altered immune response have not been shown to be related to the
development of cervical neoplasiA.
ANSWER:B

697
A 40-year-old woman is seen for a routine examination. Her menses have been regular, and she has no
complaints. Findings, including those on pelvic examination, are normal. Ten days later, her Pap smear is returned
as “high-grade squamous intraepithelial lesion.” Which of the following options is the best course of action?
A immediate wide-cuff hysterectomy
B repeated Pap smears at 3-month intervals
C fractional dilation and curettage D.&C
D colposcopy with biopsy
ANSWER:D
698
A 40-year-old woman is seen for a routine examination. Her menses have been regular, and she has no
complaints. Findings, including those on pelvic examination, are normal. Ten days later, her Pap smear is returned
as “high-grade squamous intraepithelial lesion.” The colposcope permits one to do which of the following?
A view the cervix at 1–4 power magnification
B see the entire transition zone in all patients
C choose the most suspicious areas on the cervical portio to biopsy
D treat invasive cancer with a biopsy
ANSWER:C
699
A 40-year-old woman is seen for a routine examination. Her menses have been regular, and she has no
complaints. Findings, including those on pelvic examination, are normal. Ten days later, her Pap smear is returned
as “high-grade squamous intraepithelial lesion.” Under colposcopic examination, a distinct area of acetowhite
change is noted with associated coarse pattern vessels and punctation. This is consistent with what histologic
finding on directed biopsy?
A CIN I
B atrophy
C squamous cell cancer
D CIN II-III
ANSWER:D
700
A 40-year-old woman is seen for a routine examination. Her menses have been regular, and she has no
complaints. Findings, including those on pelvic examination, are normal. Ten days later, her Pap smear is returned
as “high-grade squamous intraepithelial lesion.”Conization of the cervix would be inappropriate in which of the
following instances?
A when there is disparity between Pap smear and biopsy results

B when colposcopy is inadequate
C when microinvasion is diagnosed by biopsy
D when deeply invasive cancer is shown on a biopsy
ANSWER:D
701
A 40-year-old woman is seen for a routine examination. Her menses have been regular, and she has no
complaints. Findings, including those on pelvic examination, are normal. Ten days later, her Pap smear is returned
as “high-grade squamous intraepithelial lesion.”This patient has biopsy-proven CIN III. She requests cryotherapy
for treatment. Cryotherapy is appropriate to consider in which clinical circumstance?
A CIN III
B a patient with well-circumscribed, small lesion of mild dysplasia C.IN 1
C invasive carcinoma
D a patient who wishes to preserve fertility
ANSWER:B
702
A 25-year-old woman presents with irregular vaginal bleeding. She is otherwise healthy and uses condoms for
contraception. She smokes occasionally and takes no medications. Her aunt had cervical cancer and she is worried
that she may also have cervical cancer. What is the most common symptom associated with cervical cancer?
A no symptom
B pain with intercourse
C vaginal bleeding
D weight loss
ANSWER:A
703
When sampling the cervix for a Pap smear, it is critical to sample which area since it is the most likely source of
cervical cancer. Where do most cervical cancers arise?
A on the portio vaginalis
B at the internal os
C in the endocervix
D at the squamocolumnar junction
ANSWER:D
704 What percentage of clinical stage I carcinomas of the cervix will have lymphatic spread?
A 0–1
B up to 7
C 15

D 25
ANSWER:C
705 If a nonhealing ulcer is seen on the cervix, it is best evaluated by which of the following?
A repeat examination
B Pap smear
C punch biopsy
D cone biopsy
ANSWER:C
706
A 48-year-old woman presents for her routine annual examination. Her last Pap smear was more than 10 years
ago. She has had occasional abnormal Pap smear during her lifetime, but no treatments were recommendeD.
Otherwise, she has mild hypertension and is on a beta-blocker. On examination, she has a normal pelvic
examination, but her Pap smear reveals high-grade squamous intraepithelial lesion (SIL). A colposcopically
directed biopsy reveals invasive squamous cell carcinomA. Which of the following should be the most appropriate
next step in the care of the patient?
A metastatic evaluation
B conization
C radical hysterectomy
D radiation therapy
ANSWER:A
707
A 34-year-old woman G1 is 16 weeks pregnant and has a Pap smear suspicious for cancer. How do you advise
her?
A have colposcopy with biopsy
B have colposcopy, but biopsy is too risky in pregnancy
C have a repeat Pap smear in 3 months
D undergo a termination of pregnancy and then undergo complete evaluation
ANSWER:A
708
A 65-year-old woman presents with vaginal discharge and rare mild bleeding. She had a cone of the cervix for
CIN III 20 years ago. She has since had a complete hysterectomy for uterine fibroids. You perform a pelvic
examination and see an irregular area in the vaginA. Vaginal col-poscopy and directed biopsy reveal vaginal
intraepithelial neoplasia (VAIN). VAIN is most commonly found in which part of the vagina?
A the upper one-third
B the mid-vagina
C the distal vagina

D at the hymenal ring
ANSWER:A
709
The preferred treatment for a 1.5-cm stage I vaginal carcinoma confined to the upper one-third of the lateral
vagina in a 29-year-old woman would be which of the following?
A intravaginal 5-FU
B upper vaginectomy
C simple hysterectomy and upper vaginectomy
D radical hysterectomy, bilateral pelvic lymphadenectomy, and upper vaginectomy
ANSWER:D
710
A 42-year-old presents with a history of post-coital spotting. Examination of the cervix reveals a raised/reddened
well-circumscribed lesion next to the os. Which of the following is the most likely diagnosis?
A carcinoma
B condyloma lata
C ectropion
D cervical polyp
ANSWER:D
711 The treatment of carcinoma of the cervix during pregnancy should depend on all except which of the following?
A the recommendation of the oncologist
B the religious and moral beliefs of the patient
C the trimester of the pregnancy
D the length of the cervix
ANSWER:D
712
A 42-year-old woman with cervical cancer undergoes a radical hysterectomy and requires postoperative radiation.
During the radiation therapy she returns complaining of watery vaginal discharge and recurrent urinary tract
infections. Which of the following would be the first test to perform to evaluate the most likely cause of the
discharge?
A intravenous pyelogram (IVP)
B cystoscopy
C wet mount
D inject diluted methylene blue in sterile water into the bladder and examine the vagina
ANSWER:D

713
A 35-year-old woman with stage IIB squamous cell carcinoma of the cervix will receive radiation. Regarding
reproductive changes, how should you advise her?
A Ovaries are radioresistant.
B Fertility is maintaineD.
C Radiation will likely result in endometrial ablation.
D Younger patients are more susceptible to radiation-induced castration.
ANSWER:C
714
A 46-year-old obese woman smokes two packs of cigarettes a day. She had a radical hysterectomy with a para-
aortic and pelvic lym-phadenectomy for stage IB squamous cell carcinoma of the cervix. At surgery she was
found to have dense pelvic small-bowel adhesions from a prior ruptured appendix and appendectomy. Lymph
nodes were positive for cancer cells. In discussing postoperative radiation, you counsel her that she has an
increased rate of radiation-related complication because of which of the following?
A obesity
B excision of lymph nodes
C decreased bowel motility from adhesions
D age
ANSWER:C
715
A 24-year-old healthy woman has her routine examination and Pap smear. Her Pap smear is atypical squamous
cells of undetermined significance A.SCUS). Which of the following reflects our current knowledge about ASCUS?
A the risk of CIN II or III on biopsy is 1%
B the risk of invasive cervical cancer is 0.1%
C represent a minority of abnormal Pap smears per year in U.S. women
D requires immediate colposcopy
ANSWER:B
716
A 48-year-old postmenopausal woman presents for routine gynecologic examination. The examination is normal;
however, the Pap smear returns atypical glandular cells A.GCs). What would be the most appropriate management
for this patient?
A repeat Pap smear in 4 to 6 months
B treat with intravaginal estrogen and repeat Pap smear
C perform cone biopsy of the cervix
D perform colposcopy, cervical and endometrial biopsies
ANSWER:D

717
A 30-year-old woman presents for her annual examination. She inquires whether she should receive the HPV
vaccinE. For which patient population is the HPV vaccine FDA approved?
A women of all ages
B only women who are virginal and have never had an abnormal Pap smear
C men, for the prevention of female cervical dysplasias and cancer
D girls and women aged 9 to 26 years
ANSWER:D
718
A 44-year-old multiparous obese woman complains of abnormal vaginal bleeding of 5 months’ duration. Pelvic
examination demonstrates a small, anteverted uterus and a normal-appearing cervix. No adnexal masses are
present. A serum pregnancy test is negative, and a cervical Papanicolaou (Pap) smear is normal. Prolactin and
thyroid-stimulating hormone (TSH) levels are normal.Which of the following is the most efficient next step in the
evaluation of this patient?
A dilation and curettage D.&C
B endometrial biopsy
C endometrial cytology
D transvaginal sonography
ANSWER:B
719
A 44-year-old multiparous obese woman complains of abnormal vaginal bleeding of 5 months’ duration. Pelvic
examination demonstrates a small, anteverted uterus and a normal-appearing cervix. No adnexal masses are
present. A serum pregnancy test is negative, and a cervical Papanicolaou (Pap) smear is normal.Prolactin and
thyroid-stimulating hormone (TSH) levels are normal.Tissue sampling in this patient reveals endometrial
hyperplasiA. What is the most common symptom associated with this condition?
A vaginal discharge
B vaginal bleeding
C amenorrhea
D pelvic pain
ANSWER:B
720
A 44-year-old multiparous obese woman complains of abnormal vaginal bleeding of 5 months’ duration. Pelvic
examination demonstrates a small, anteverted uterus and a normal-appearing cervix. No adnexal masses are
present. A serum pregnancy test is negative, and a cervical Papanicolaou (Pap) smear is normal. Prolactin and
thyroid-stimulating hormone (TSH) levels are normal.Which of the following factors is protective against
endometrial hyperplasias?
A obesity

B tamoxifen
C oral contraceptive pills (OCPs).
D early menarche or late menopause
ANSWER:C
721
A 49-year-old woman experiences irregular vaginal bleeding of 3 months’ duration. You perform an endometrial
biopsy, which obtains copious tissue with a velvety, lobulated texturE. The pathologist report shows proliferation
of glandular and stromal elements with dilated endometrial glands, consistent with simple hyperplasiA. Cytologic
atypia is absent. Which of the following is the best way to advise the patient?
A She should be treated to estrogen and progestin hormone therapy.
B The tissue will progress to cancer in approximately 10% of cases.
C The tissue may be weakly premalignant and progresses to cancer in approximately 1% of cases.
D She requires a hysterectomy.
ANSWER:C
722
A 48-year-old woman is referred to you for irregular vaginal bleeding of 6 months’ duration. Her referring
physician removed tissue protruding through the cervix 3 months ago. Microscopic examination of the tissue
shows a mass with cystic hyperplasia and a central vascular channel surrounded on three sides by epithelium. The
vaginal bleeding has continueD. Which of the following is the best way to advise the patient?
A Risk of developing endometrial cancer is increased 10-folD.
B Bleeding is from an endometrial polyp.
C Histology of the tissue may not reflect the source of the bleeding.
D Uterus should be probed with a forceps to remove more tissuE.
ANSWER:C
723
A 58-year-old woman on combined estrogen and progesterone hormone replacement has postmenopausal
bleeding. You obtain a pelvic ultrasound that shows an endometrial stripe thickness of 12 mm. Which of the
following is most correct?
A
If the endometrial stripe thickness had been less than 5 mm, you would have told the patient that no further
evaluation was needeD.
B An endometrial stripe thickness of 5 to 10 mm confers no risk of endometrial cancer.
C She has a greater than 50% risk of having adenocarcinoma of the endometrium.
D
The endometrial stripe thickness in premenopausal women is interpreted similar to the endometrial stripe thickness
dimensions in postmenopausal women.
ANSWER:A

724
An internist calls you for consultation regarding a 55-year-old postmenopausal woman with some vaginal
spotting. On examination, a small, round, bright red mass was noted to protrude through the cervical os. It bled
during the Pap smear. The Pap smear result was normal. You should advise the internist to do which of the
following?
A Recheck the mass in 6 months and refer if it enlarges.
B Refer the patient for probable polyp removal.
C Refer the patient for cone biopsy.
D Tell the patient not to worry since the Pap smear is negativE.
ANSWER:B
725
A 44-year-old female biochemist has complex hyperplasia without atypia on endometrial biopsy. You prescribe
40-mg megestrol acetate daily. She inquires about the mechanism of action and regression ratE. Which of the
following explanations is most correct?
A The regression of endometrial hyperplasia takes at least 12 months.
B Progestins oppose estrogen action in endometrial tissue by reducing the amount of estrogen receptors.
C Hyperplastic endometrium has few progesterone receptors so a large dose of progestin is needeD.
D If regression of endometrial hyperplasia occurs within 3 months, it will recur if she stops the medication.
ANSWER:B
726
A 45-year-old woman complains of pelvic pressure and abnormal uterine bleeding. Ultrasound reveals an
enlarged uterus with an intramural 4 cm mass. Which of the following is the most common uterine neoplasm?
A sarcoma
B adenocarcinoma
C adenomyosis
D leiomyoma
ANSWER:D
727
During a presentation to a group on women’s health a discussion of gynecologic/reproductive cancers including
their etiology risk, factors, and normal clinical course is presenteD. Which of the following types of cancer is the
leading cause of gynecologic/reproductive cancer death in women?
A cervical
B uterine
C ovarian
D breast
ANSWER:D

728
A 69-year-old postmenopausal woman is being admitted for surgical treatment of endometrial cancer. She has no
health insurance and would like to know which is the most important preoperative screening test to look for
metastasis?
A chest X-ray
B hysterosalpingogram
C pelvic ultrasound
D intravenous pyelogram (IVP)
ANSWER:D
729
A healthy 65-year-old woman is seen for postmenopausal bleeding. The pelvic examination is normal. A
fractional D&C demonstrates adenocarcinoma of the endometrium. Histologically, endometrial glands are
confluent without solid areas of tumor cells. The endocervical curettage shows normal endocervical cells. The
cervical Pap smear and other preoperative investigations are normal. Which of the following statements most likely
reflects this patient’s endometrial carcinoma stage or treatment.
A Invasion of tumor through most of the myometrium will not be founD.
B Invasion of tumor into pelvic lymph nodes will not occur.
C Steroid hormone receptors will not be present in tumor tissuE.
D
Therapy depends on surgical and histologic evaluation of pelvic viscera, peritoneal cavity, and retroperitoneal
lymph nodes.
ANSWER:D
730
A healthy 65-year-old woman is seen for postmenopausal bleeding. The pelvic examination is normal. A
fractional D&C demonstrates adenocarcinoma of the endometrium. Histologically, endometrial glands are
confluent without solid areas of tumor cells. The endocervical curettage shows normal endocervical cells. The
cervical Pap smear and other preoperative investigations are normal. Exploratory laparotomy is negative for
metastatic diseasE. The uterus is opened in the operating room and found to have tumor invasion into the
myometrium. Histologic examination of the uterus confirms tumor invasion beyond the inner half of the
myometrium. Peritoneal washings and pelvic and paraaortic nodes are negative for malignancy. What should you
advise this patient?
A no further therapy
B radiation therapy
C hormonal therapy
D single-agent chemotherapy
ANSWER:B

731
A healthy 65-year-old woman is seen for postmenopausal bleeding. The pelvic examination is normal. A
fractional D&C demonstrates adenocarcinoma of the endometrium. Histologically, endometrial glands are
confluent without solid areas of tumor cells. The endocervical curettage shows normal endocervical cells. The
cervical Pap smear and other preoperative investigations are normal. This patient underwent postoperative
radiation. During radiation therapy she develops nausea, anorexia, diarrhea, and mild abdominal pain. Which of
the following is the most likely diagnosis?
A radiation cystitis
B radiation enteritis
C radiation proctitis
D enterovaginal fistula
ANSWER:B
732
A pulmonary nodule is discovered on the chest radiogram of a healthy 82-year-old woman. Four years ago, she
was treated for endometrial adenocarcinomA. Excision of the nodule shows moderately differentiated endometrial
adenocarcinoma-containing progesterone receptors. There is no other evidence of metastatic diseasE. What should
you advise this patient?
A exploratory laparotomy
B lobectomy
C radiation therapy
D progestin therapy
ANSWER:D
733
A 52-year-old patient undergoes a hysterectomy for a rapidly growing uterine mass. At surgery the frozen biopsy
is reported as a sarcomA. What is the most common uterine sarcoma?
A leiomyosarcoma
B endometrial stromal sarcoma
C endolymphatic stromal myosis
D malignant mixed müllerian tumor
ANSWER:D
734
A 55-year-old woman undergoes a total abdominal hysterectomy and bilateral salpingo-oophorectomy for a
rapidly enlarging pelvic mass. A frozen section is sent, although the pathologist tells you he cannot distinguish
leiomyosarcomas very well on frozen section. Nonetheless, the specimen looks very suspicious. You still have her
abdomen open in the operating room. Which of the following statements describes the optimal next step in the
evaluation and management for this patient?
A Radical parametrectomy should be performeD.

B Lymphadenectomy should be performeD.
C Radiation to the pelvis has no effect on pelvic recurrence of sarcomA.
D There are no additional benefits from intraoperative radiation, radical surgery, or optimal cytoreduction.
ANSWER:D
735
A 40-year-old woman is found on pelvic examination to have an enlarged uterus. Ultrasound reveals a well-
circumscribed intramural mass consistent with the leiomyomA. The patient asks: what is the incidence of
sarcomatous degeneration in a uterine leiomyoma?
A <1%
B 3%
C 10%
D 15%
ANSWER:A
736 Which of the following is a factor predisposing to the development of malignant mixed müllerian tumors?
A prenatal exposure to diethylstilbestrol D.ES)
B exposure to mumps virus
C family history of ovarian cancer
D previous pelvic irradiation
ANSWER:D
737
A 38-year-old nulliparous woman presents requesting a bilateral salpingo-oophorectomy. Her mother died of
ovarian cancer at the age 64, and her sister at the age 48. There is no family history of other cancers. You advise
her that her risk of developing ovarian cancer is what percentage?
A 1–2%
B 7%
C 20%
D 30–40%
ANSWER:B
738
The same patient gets on the Internet and returns asking about the hereditary types of epithelial ovarian cancer.
Which of the following statements is true?
A A site-specific defect transmitting the trait for only ovarian carcinoma is common.
B A BRCA1 gene mutation increases her lifetime risk of ovarian cancer to 10%.
C Lynch type II cancer syndrome includes ovarian malignancy.
D Fifty percent of ovarian cancer is hereditary.
ANSWER:C

739
A 56-year-old healthy woman develops vague complaints and presents to her primary care physician. Which of
the following accurately describes symptoms that could be associated with a diagnosis of ovarian cancer?
A there are no identifiable symptoms in women with ovarian cancer
B symptoms are usually present for years prior to a diagnosis
C shortness of breath and cough
D urinary urgency and bloating
ANSWER:D
740 Which of the following is a cornerstone for detection of ovarian neoplasia?
A CA-125
B human chorionic gonadotropin (hCG)
C pelvic examination
D pelvic ultrasound
ANSWER:C
741 Ovarian neoplasms most commonly arise from which of the following cell lines?
A ovarian epithelium
B ovarian stroma
C ovarian germ cells
D ovarian sex cords
ANSWER:A
742 Which of the following postmenopausal women is most protected from ovarian epithelial carcinoma?
A a married woman using perineal talc powder
B an unmarried woman with a history of breast cancer
C a nun with a history of late menopause
D a multiparous woman who used OCPs and now postmenopausal
ANSWER:D
743 Which of the following statements accurately reflects the natural history of ovarian epithelial carcinoma?
A The incidence of ovarian carcinoma increases with age until the seventh decade of lifE.
B Elderly women are less likely than younger women to have disease diagnosed at an advancedstagE.
C Most women with ovarian cancer do not have any symptoms prior to dissemination of diseasE.
D Seventy-five percent of all ovarian tumors in women older than 50 years are malignant.
ANSWER:A

744
A 35-year-old woman desiring fertility undergoes exploratory laparotomy for a 12-cm pelvic mass. At surgery, a
large, lobulated, right ovarian mass is observeD. It has a smooth external capsule and a bluishgray appearancE.
The uterus, fallopian tubes, and left ovary appear normal. Abdominal exploration is negative for metastatic
diseasE. A right salpingo-oophorectomy is performeD. The tumor is opened intraoperatively and found to be
divided by septa into lobules. Frozen section of the tumor shows a mucinous cystadenoma of low malignant
potential.One would base the remainder of the surgical intervention at this time on which of the following
statements regarding mucinous cystadenoma of low malignant potential?
A
Spread of the tumor outside the ovary occurs 30–40% of the time in the form of intraperitoneal growth of mucin-
producing cells.
B It has 1–2% incidence of bilaterality.
C It has a 5-year survival rate of 60%.
D It comprises atypical epithelial proliferation without stromal invasion.
ANSWER:D
745
A 35-year-old woman desiring fertility undergoes exploratory laparotomy for a 12-cm pelvic mass. At surgery, a
large, lobulated, right ovarian mass is observeD. It has a smooth external capsule and a bluishgray appearancE.
The uterus, fallopian tubes, and left ovary appear normal. Abdominal exploration is negative for metastatic
diseasE. A right salpingo-oophorectomy is performeD. The tumor is opened intraoperatively and found to be
divided by septa into lobules. Frozen section of the tumor shows a mucinous cystadenoma of low malignant
potential.Two days after surgery, you receive the pathology report of the ovarian tumor. It is a mucinous
cystadenoma of low malignant potential mixed with well-differentiated carcinomA. The tumor has not invaded the
ovarian capsule, lymphatics, or mesovarium. Omental and retroperitoneal lymph node biopsies and peritoneal
washings are negative for tumor cells. How do you advise this patient?
A biopsy of the contralateral ovary
B removal of the uterus and contralateral adnexum
C postoperative chemotherapy
D no further therapy
ANSWER:D
746
A 54-year-old healthy woman comes for an annual examination. Her last menstrual period (LMP) was 4 years
ago. The physical examination is normal. Pelvic examination shows vaginal atrophy and a small, mobile uterus.
The right ovary is 2.5 x 4.5 cm in diameter. The left ovary is nonpalpablE. Vaginal ultrasonography shows that
the right ovary is similar in size to that of a premenopausal ovary. What should you advise this patient?
A The ovaries of a postmenopausal woman are usually palpablE.

B The right ovary of a postmenopausal woman is usually palpable by right-handed examiners.
C A palpable ovary in a postmenopausal woman is suspicious for malignancy.
D The right ovary is still producing significant amounts of estrogen.
ANSWER:C
747
A 65-year-old woman has abdominal distention of 3 months’ duration. Abdominal percussion causes a wavelike
movement of fluid around a central tympanitic areA. Pelvic examination shows a right ad-nexal mass. It is 8 cm in
size, nodular, and fixed in the pelvis. The left ovary is nonpalpablE. Blood chemistries, urinalysis, cervical Pap
smear, mammography, and chest X-ray are normal. Stool gua-iac examination and gastrointestinal studies are also
normal. A serum CA-125 level is 250 U/mL (normal, <35 U/mL). Which of the following statements reflects CA-
125?
A It is a circulating antigenic marker for germ cell ovarian carcinomA.
B It is found in normal fetal and adult ovaries.
C It is secreted by mesothelial cells of the pleura, pericardium, and peritoneum.
D It is not useful in monitoring tumor progression.
ANSWER:C
748
A 65-year-old woman has abdominal distention of 3 months’ duration. Abdominal percussion causes a wavelike
movement of fluid around a central tympanitic areA. Pelvic examination shows a right ad-nexal mass. It is 8 cm in
size, nodular, and fixed in the pelvis. The left ovary is nonpalpablE. Blood chemistries, urinalysis, cervical Pap
smear, mammography, and chest X-ray are normal. Stool gua-iac examination and gastrointestinal studies are also
normal. A serum CA-125 level is 250 U/mL (normal, <35 U/mL). Which of the following is the most likely
diagnosis?
A gonadoblastoma
B Meigs’ syndrome
C Krukenberg’s tumors
D serous cystadenocarcinoma
ANSWER:D
749
A 65-year-old woman has abdominal distention of 3 months’ duration. Abdominal percussion causes a wavelike
movement of fluid around a central tympanitic areA. Pelvic examination shows a right ad-nexal mass. It is 8 cm in
size, nodular, and fixed in the pelvis. The left ovary is nonpalpablE. Blood chemistries, urinalysis, cervical Pap
smear, mammography, and chest X-ray are normal. Stool gua-iac examination and gastrointestinal studies are also
normal. A serum CA-125 level is 250 U/mL (normal, <35 U/mL). Her surgical treatment should do which of the
following?
A remove all gross disease if the risk of fatal complications is minimal

B avoid resection of bowel
C be done through a Pfannenstiel incision
D be done laparoscopically
ANSWER:A
750 Malignant changes occur in what percentage of streak ovaries when a Y chromosome is present?
A 5
B 25
C 45
D 65
ANSWER:B
751
A 5-year-old girl experiences early breast development. She is taller than her peers. Her mother has noticed blood
at the girl’s introitus. Serum gonadotropin levels are low and are unchanged after intravenous administration of
gonadotropin-releasing hormone (GnRH). Which of the following is the most likely diagnosis?
A Sertoli-Leydig cell tumor
B granulosa cell tumor
C hilar cell tumor
D fibroma
ANSWER:D
752
A 48-year-old woman is taken to surgery for a solid pelvic mass of 6x7 cm and marked ascites. At laparotomy the
adnexa is removed and sent for frozen section examination. The report returns as metastic adenocarcinoma to the
ovary. Although uncommon, metastatic tumors to the ovary most often originate from which of the following?
A breast
B stomach
C large intestine
D vagina
ANSWER:D
753
A hormonally active neoplasm is likely to cause clinical signs through the hormonal effect that would prompt
earlier evaluation and hence diagnosis. Which one of the following tumors is likely to be found primarily based on
physical size and location of the neoplasm?
A Sertoli-Leydig cell tumor
B granulosa cell tumor

C hilar cell tumor
D fibroma
ANSWER:D
754
A 43-year-old woman has stage III epithelial ovarian carcinomA. She had surgical debulking and has received
five courses of carboplatin and paclitaxel. She is now due for her sixth coursE. She comes to clinic complaining of
fatigue and myalgias. She has a temperature of 101.3°F. On examination, you find no obvious source of the fever.
WBC count is 1,000/mm3 (normal 4, 500 to 11,000/mm3). Your next course of action is to do which of the
following?
A send home and instruct to check temperature twice a day
B give a broad-spectrum antibiotic as outpatient
C admit to hospital and observe
D admit to hospital and start antibiotics
ANSWER:D
755
A 23-year-old woman at 29 weeks’ gestation ruptures her amniotic membranE. She is admitted to the hospital, and
3 days later, she is found to have a temperature of 101.1°F, a white blood cell (WBC) count of 15,000, fetal
tachycardia, and mild tenderness over her lower abdomen. Which of the following is the most likely diagnosis for
this patient?
A intra-amniotic infection
B lower urinary tract infection (UTI)
C pyelonephritis
D genital herpes
ANSWER:A
756
Three weeks after delivery, a 29-year-old primipara, who is breast-feeding twin girls, presents to the clinic,
complaining of a tender right breast mass. On physical examination, you find a 5-cm fluctuant, swollen, reddened
mass in her right breast that is exquisitely tender to the touch. Axillary lymph nodes on the ipsilateral side are
enlarged and tender. What is the most appropriate next step in the management of this patient?
A excisional biopsy of the mass
B needle aspiration of the mass
C intravenous antibiotic therapy for the mother and infants
D incision and drainage of the mass plus oral antibiotics for the mother
ANSWER:D

757
Two weeks after the birth of her infant, a new mother brings the child in to see you. The child’s eyes are
edematous, with conjunctival erythema and a mucopurulent dischargE. Your evaluation and treatment should
include which of the following?
A a pelvic examination (using a small scope) of the infant
B culture maternal genital tract for GC and chlamydia
C anaerobic cultures of the infant’s and mother’s eyes
D immunoglobulin M (IgM) titers of the infant
ANSWER:B
758
Three days after an elective termination of pregnancy, a 29-year-old woman presents to the emergency
department with a history of mild abdominal pain and fever, a physical examination showing pelvic tenderness,
and a purulent cervical dischargE. A Gram’s stain of the cervical discharge shows gramnegative intracellular
diplococci. What is the most likely causative agent in this patient’s case?
A Neisseria gonorrhoeae
B Chlamydia trachomatis
C Escherichia coli
D Treponema pallidum
ANSWER:A
759
A 31-year-old woman in her first trimester presents for her initial visit. She complains of a painless raised lesion in
her vulvA. Examination reveals a chancrE. Rapid plasma reagin (RPR) test with elevated titer is positive along with
fluorescent treponemal antibody absorption (FTA-ABS), confirming the diagnosis of primary syphilis. The patient
should be offered which of the following counseling or treatment options?
A Offer the woman termination, as the fetus will be infected and develop congenital syphilis.
B Immediately treat her with parenteral penicillin G.
C
The causative agent for syphilis, T. pallidum, does not cross the placenta; therefore, there is no risk for congenital
syphilis.
D Penicillin-allergic women should be treated with erythromycin.
ANSWER:A
760
A woman has a stillborn infant covered with a petechial rash. Which of the following infections would be most
likely?
A herpes zoster
B herpes simplex
C Listeria monocytogenes
D human papillomavirus (HPV)

ANSWER:C
761
Regarding immunization during pregnancy, which of the following vaccines would be the safest to receive during
pregnancy?
A mumps
B polio
C rabies
D rubella
ANSWER:B
762
A patient presents to labor and delivery in active labor and has a precipitous delivery within 15 minutes of arrival.
Her prenatal care has been erratiC. During the repair of a second-degree laceration, you note ulcerations on the
labia consistent with herpes simplex virus (HSV). On further questioning of the patient, her history is consistent
with a primary outbreak of herpes. You send cultures but recommend immediate treatment of the newborn for
HSV while waiting for the results. When the patient inquiries why, you tell her that HSV infection in pregnancy is
associated with a neonatal mortality rate for untreated infected infants. What is this mortality rate?
A 10%
B 25%
C 50%
D 75%
ANSWER:C
763
An infant, seemingly well when born, demonstrates microcephaly, chorioretinitis, deafness, and delayed
development later in lifE. Which of the following is the most likely cause?
A type 2 herpes hominis virus acquired at the time of delivery
B CMV infection during pregnancy
C vitamin K deficiency in the newborn
D late-onset group B streptococcal infection
ANSWER:B
764
A 19-year-old woman who has never had chick-enpox has just been exposed to the disease A.pproximately 36
hours ago) at 16 weeks’ gestation. What is the most appropriate next step in the management of this patient?
A reassurance only
B a measurement of maternal varicella titer 3 weeks after exposure
C a measurement of maternal varicella titer 6 weeks after exposure
D the administration of varicella zoster immune globulin (VZIG)

ANSWER:D
765
A 27-year-old gravida you have been following throughout her pregnancy presents at 22 weeks’ gestation not
feeling well. She complains of fever, cough, a runny nose, conjunctivitis, and on examination has white spots
surrounded by a halo of erythema on her buccal mucosa and an erythematous maculopapular rash on her
abdomen. What is the most likely cause of this patient’s condition?
A varicella zoster
B rubella
C rubeola (measles)
D syphilis
ANSWER:C
766
A 25-year-old G1 is newly pregnant. She is found to be rubella nonimmunE. She asks you about the implication
of this. You should inform her about which of the following?
A
A significant percentage of fetuses of women who develop rubella infection during pregnancy will develop
congenital rubella syndromE.
B Rubella infection increases maternal mortality.
C Treatment with antiviral medications is effectivE.
D She should receive rubella immunization during this pregnancy.
ANSWER:A
767
Of the following individuals, who would theoretically be at highest demographic risk for toxoplasmosis infection
during pregnancy?
A country western singer
B medical technologist
C plumber
D cat breeder
ANSWER:D
768
A 25-year-old sexually active woman complains of a “fishy” smelling gray-white vaginal dischargE. You examine
this on wet mount and see epithelial cells with clusters of bacteria obscuring their borders. The vaginal pH is 5.5.
This infection has been most closely implicated in which of the following complications of pregnancy?
A intrauterine growth restriction
B preterm birth
C congenital cataracts
D learning disabilities during childhood

ANSWER:B
769
A 45-year-old Laotian woman is visiting her daughter. She comes to your office complaining of frequent
intermenstrual bleeding for years. You examine her and feel that her pelvis is “firmly fixed,” with little mobility of
the organs. You perform an endometrial biopsy. The pathology report returns stating that “frequent giant cells,
caseous necrosis, and granuloma formation” are seen. Which of the following is the most likely cause of this
woman’s condition?
A syphilis
B trachomatis
C tuberculosis
D N. gonorrhoeae
ANSWER:C
770
A 43-year-old woman has had a history of frequency, urgency, and dysuria for the past 8 years. She has had five
negative urine cultures and urinalyses in the last year. Cystoscopy 1 month ago showed a normal bladder and
reddened urethrA. An intravenous pyelogram (IVP) is normal. What is the most likely diagnosis?
A surreptitious use of antibiotics by the patient to mask her laboratory results
B tuberculous urethritis
C vulvar vestibulitis syndrome
D urethral syndrome
ANSWER:D
771
A 51-year-old woman presents complaining of dysuria, dyspareunia, frequency of urination, dribbling of urine
from the urethra when she stands after voiding, and a painful swelling under her urethrA. Which of the following
is the most likely diagnosis?
A simple cystitis
B urethral syndrome
C infection of the Skene’s glands
D infected urethral diverticulum
ANSWER:D
772
On the evening after a vaginal hysterectomy, a patient develops a temperature of 100.4°C. You are called to
evaluate her. Which of the following do you consider most likely prior to examining the patient?
A She probably has a UTI.
B Ureteral obstruction is likely.
C Her fever may be factitious.
D The temperature elevation is most likely unrelated to a surgical infection.

ANSWER:D
773
On the evening after a vaginal hysterectomy, a patient develops a temperature of 100.4°C. You are called to
evaluate her. The same patient continues to have fever in the 102°F to 104°F range over the next few days. A
pelvic examination is repeated and a midline, tender mass approximately 8 cm in diameter is noted over the
vaginal cuff. What is the most appropriate next step in this patient’s management?
A obtain an ESR and WBC, and start or change antibiotics
B get an infectious disease consult
C send a vaginal culture to assess the coverage of your antibiotics
D open the vaginal cuff in the midline
ANSWER:D
774
The hospital is reviewing its protocols to decrease the iatrogenic infection rate within the hospital. For which of the
following procedures would prophylactic antibiotics be appropriate?
A amniocentesis
B laparoscopy
C tubal sterilization
D vaginal hysterectomy
ANSWER:D
775
A 35-year-old woman undergoes a cesarean section after a failed induction for postmaturity. Three days after
surgery, she develops a high spiking fever. Ampicillin and gentamicin are administereD. Complete physical
examination shows no abnormality except a tender uterus. Blood, urine, and sputum cultures are negativE. On the
fifth day after surgery, a hectic (spiking) fever is still present. The antibiotics are changed to ampicillin,
gentamicin, and clindamycin in high dosagE. Forty-eight hours later, the fever persists, and examination shows a
tender uterus. A chest X-ray is normal. Pelvic CT is consistent with parametrial thrombosed vessels but no abscess.
Which of the following is the next best step in managing this patient?
A reoperate to find the source of the fever
B anticoagulate the patient with heparin
C get an infectious disease consult
D discontinue all antibiotic medication and reculture the patient
ANSWER:B

776
A 34-year-old woman (gravida 2, para 1) is at 13 weeks’ gestation by last menstrual period (LMP) with a desired
pregnancy. She presents to the emergency department very anxious with a 10-hour history of low abdominal
cramping and vaginal bleeding. Her temperature is 102.2°F, and her uterus is markedly tender on bimanual
examination. Ultrasound shows an intrauterine pregnancy with a crownrump length consistent with her LMP and
fetal cardiac activity present. Her cervix is dilated by 1 cm. Her WBC count is 26,000. What is the best
management for this patient?
A place a cervical cerclage immediately after administering antibiotics
B administer antibiotics and expectantly manage her
C evacuate her uterus after administering antibiotics
D
administer antibiotics, and if she does not spontaneously abort after 24 hours of observation, place a cervical
cerclage
ANSWER:C
777
An asymptomatic 24-year-old African-American woman with sickle cell trait is found on routine prenatal
screening at 14 weeks’ gestation to have symptomatic bacteriuria (105 colonies/mL). What is her risk of
developing pyelonephritis if untreated?
A 5–10%
B 20–30%
C 40–50%
D 60–70%
ANSWER:B
778
An asymptomatic 24-year-old African-American woman with sickle cell trait is found on routine prenatal
screening at 14 weeks’ gestation to have symptomatic bacteriuria (105 colonies/mL). What is the most likely
organism to be cultured?
A group B streptococcus
B Klebsiella pneumoniae
C trachomatis
D coli
ANSWER:D
779
An asymptomatic 24-year-old African-American woman with sickle cell trait is found on routine prenatal
screening at 14 weeks’ gestation to have symptomatic bacteriuria (105 colonies/mL).What is an appropriate choice
of antibiotic therapy for this patient pending culture results?
A ampicillin
B tetracycline

C ciprofloxacin
D nitrofurantoin
ANSWER:D
780
A mother brings her 4-year-old daughter in for complaints of itching “down there” and staining on the underwear.
Which of the following conditions is the most likely cause of vulvovaginal symptoms in children?
A foreign body
B lichen sclerosis
C nonspecific
D physiologic leukorrhea
ANSWER:C
781
A 6-year-old girl is referred by her pediatrician for a friable mass in the genital region. You suspect a urethral
prolapsE. Which of the following is the most common symptom of urethral prolapse in the prepubertal, unestroge-
nized girl?
A dysuria
B hematuria
C painless genital bleeding
D urinary frequency
ANSWER:C
782
A mother has brought in her 8-year-old daughter because of the development of breasts over the past year. They
are now to the size that she is requiring a bra and is being teased by the other children. Premature thelarche differs
from true precocious puberty in that premature thelarche is associated with which of the following?
A axillary hair development
B isolated breast development
C pubic hair development
D spontaneous ovulations
ANSWER:B
783
A 14-year-old girl is brought into the office by her mother because of a concern of a lack of menarchE. Her
mother is worried that something is wrong since she has not started menstruating. Based on a complete history and
limited physical and thorough application of a knowledge of normal pubarche changes, you may be able to calm
the mother. Which of the following occurs earliest in preadolescent girls entering puberty?
A axillary hair growth
B breast development

C menarche
D peak growth velocity
ANSWER:B
784
A 16-year-old girl is seen in the emergency department for evaluation of nausea and vomiting. Her vital signs are
blood pressure, 80/40 mm kg; pulse, 130 bpm; and temperature, 102.2°F. Physical examination shows
conjunctivitis, oropharyngeal hyperemia, and a sunburn-like macular rash over the face, proximal extremities, and
trunk. Palpation of the extremities elicits muscle tenderness. Pelvic examination is normal, and a bloody tampon is
present in the vaginA. Which of the following is the most likely diagnosis?
A erysipelas
B human immunodeficiency virus (HIV)
C Kawasaki disease
D toxic shock syndrome (TSS)
ANSWER:D
785
A 14-year-old girl has a chronic cough with copious expectoration. A biopsy of the respiratory mucosa shows
ciliated epithelium devoid of dynein arms. Which of the following conditions is most likely to occur in later life?
A abnormal vaginal bleeding
B chronic diarrhea
C infertility
D pelvic pain
ANSWER:C
786
An 8-year-old girl is brought in by the mother after finding her crying and having bloody underwear. She will not
tell her mother what happeneD. On examination, there are injuries consistent with vaginal penetration. You advise
the mother that it is very important to allow the authorities to speak with the daughter about what happeneD. What
is the percentage of sexually abused children who know their assailant?
A 15
B 35
C 55
D 75
ANSWER:D

787
An 8-year-old girl is brought in by the mother after finding her crying and having bloody underwear. She will not
tell her mother what happeneD. On examination, there are injuries consistent with vaginal penetration. You advise
the mother that it is very important to allow the authorities to speak with the daughter about what happeneD.
Which of the following is the most commonly reported form of incest?
A brother-sister
B father-daughter
C father-son
D mother-son
ANSWER:B
788
A father brings his 9-year-old daughter to the office after he picked her up for his joint custody visit. He and his
ex-wife have been in a long drawn-out custody battlE. His daughter told him that her mom’s new boyfriend was
touching and poking her “down there” last night while mom was shopping. When childhood sexual assault is
suspected within the past 72 hours, which of the following should be the next action of the physician?
A bring family members together for an interview
B contact mental health workers
C notify the police
D perform a complete physical examination
ANSWER:D
789
A father brings his 9-year-old daughter to the office after he picked her up for his joint custody visit. He and his
ex-wife have been in a long drawn-out custody battlE. His daughter told him that her mom’s new boyfriend was
touching and poking her “down there” last night while mom was shopping. Which of the following is a legal but
not a medical responsibility of the physician caring for an alleged sexual assault victim?
A collecting samples of hair and vaginal secretions, and microscopic evaluation of motile sperm
B obtaining a complete gynecologic history
C obtaining informed consent from patient
D offering postcoital hormonal prophylaxis to prevent pregnancy if reproductive age
ANSWER:A
790 Which of the following legal theories describes the failure of a physician to disclose the risks of a procedure?
A abandonment
B breach of duty
C informed consent
D intentional tort

ANSWER:B
791
Professional liability insurance that protects against claims made during the policy period, regardless of when the
suit is filed, is which of the following?
A claims-made policy
B occurrence policy
C tail policy
D nose policy
ANSWER:B
792
An 8-year-old girl is brought to your office soon after suffering a fall on her brother’s bicyclE. Her mother
reports that the girl’s foot slipped off the bicycle pedal, which resulted in the girl falling on the center bar of the
bicyclE. The girl complains of sharp pain between her legs. There has been no obvious bleeding and no other
injuries are apparent. The girl is in moderate distress with a pulse of 110 bpm, blood pressure of 118/68 mm Hg,
and respirations of 28/min. Physical examination is normal with the exception of inspection of the vulva where a 6-
cm tender bluish mass is present in the area of the right labia majorA. No further examination is possible because
of the girl’s discomfort. Which management is most directly related to an uncomplicated outcome?
A topical application of ice
B use of prophylactic antibiotics
C bed rest for the next 24 hours
D examination under anesthesia
ANSWER:A
793 Which legal document sets out a patient’s wishes regarding her future health status, including end-oflife issues?
A living will
B proxy directive
C advanced directive
D informed consent
ANSWER:C
794 What is a common emotional sequella of rape?
A mania
B depression
C rage
D bipolar disorders
ANSWER:B

795
A 6-year-old girl is seen for a 10-day history of intense vulvar itching leading to excoriation. The vulva are noted
to be diffusely inflameD. What is the most likely cause for these symptoms?
A Candida albicans infection
B foreign body
C sexual assault
D Enterobius vermicularis
ANSWER:A
796 Which of the following sales activities would be considered unethical in medical practice?
A sale of a pessary fitted in the office
B sale of prescription medications to be taken at home
C sale of cookies for a local charity
D sale of annual flu vaccine given in the office
ANSWER:B
797
A 46-year-old patient has her serum cholesterol tested in your laboratory, and it is 280 mg/dL. Which of the
following statements regarding this finding would be accurate to tell her?
A Elevated serum cholesterol is the most significant risk factor in the development of ischemic cardiac diseasE.
B Further evaluation will be needeD.
C She needs to see an internist immediately.
D The pizza she had for lunch the previous day probably is responsible for the elevation.
ANSWER:B
798
A 70-year-old woman who is in good health comes to your office for the first timE. Her only disease prevention
issue is that she smokes. While discussing this with her, which of the following should you tell her?
A Assure her that 70 years of age is too old to be worrying about quitting smoking.
B At 70 years of age, smoking is up to her, and she is certainly mature enough to make her own decisions.
C If she got to be 70-year-old and smokes, it is probably good for her.
D
Inform her that her life expectancy may be 15 to 20 more years or longer, and that if she would like to try to quit
smoking, you will assist her.
ANSWER:D
799
There is a great deal of debate about what is the best screening strategy regarding the use of mammography.
Which of the following is true about the most important feature of mammography?
A It allays fears in women.
B It can detect lesions as small as 1 mm.
C It essentially misses no cancer.

D It leads to a reduction in mortality in breast cancer in women aged 50 to 64 years.
ANSWER:D
800
Many common illnesses are related to life choices that increase risks for developing medical complications. Such
life choices include smoking, overeating, and use of illegal substances. The clinician needs to determine where the
patient is regarding approaching change in behavior to direct management. A pregnant patient who is smoking 10
cigarettes a day has already arranged with family not to smoke around her, has established a quit date, and is
enquiring about nicotine replacement. Which of the following best describes the mindset of this patient?
A action
B contemplative
C maintenance
D precontemplative
ANSWER:A
801
A 43-year-old woman, 5 ft 4 in. tall (163 cm) and weighing 176 lb (80 kg), presents for her annual examination
and assistance on losing weight. She asks “How many calories would I have to cut out each day to lose 1 pound
per week?” Which of the following choices is the best response?
A A.100
B B.500
C C.900
D 1 300
ANSWER:B
802
A 43-year-old woman, 5 ft 4 in. tall (163 cm) and weighing 176 lb (80 kg), presents for her annual examination
and assistance on losing weight. You also encourage her to exercisE. Given her current status, which of the
following is the best initial form of exercise for her to maximize the calories burned per week?
A bicycling
B jogging/running
C swimming
D walking
ANSWER:D
803
A 27-year-old woman presents for her annual examination and renewal of her oral contraceptive (OC) pills. Her
history and physical examination were unremarkable except for a non-tender 4 × 4 cm enlargement of the left
lobe of her thyroiD. Which of the following is key in the evaluation of this finding?
A TSH level

B free T4 and free T3 levels
C radioactive iodine scan
D ultrasound with fine-needle aspirate
ANSWER:D
804
As part of a premarital examination, a 24-year-old teacher would like a measles vaccination because she is
nonimmunE. She asks, “Do I need to avoid pregnancy after getting this vaccination?” Which of the following is
the most appropriate response?
A no, it is a killed vaccine and unnecessary
B no, it is a form of passive immunization and therefore noninfective
C yes, for 6 weeks
D yes, for 12 weeks because it is a live, attenuated vaccine
ANSWER:D
805
A 28-year-old woman who has been splenec-tomized as a result of a car accident wonders if there is any special
immunization she should have as a result. Of the following choices, how should you answer her?
A measles
B mumps
C meningococcus
D pneumococcus
ANSWER:D
806
A patient presents to you with excruciating pain, and you confirm the diagnosis of a kidney stonE. You would like
to provide the greatest amount of pain relief possible until the stone can be treated or passes. Which of the
following narcotics has the greatest analgesic potency A.dministered parenterally) when compared to morphine?
A codeine
B hydromorphone
C meperidine
D methadone
ANSWER:B
807
A 77-year-old woman comes to see you with her daughter. The mother has recently moved in with the daughter
because it is difficult for her to care for herself completely. At night, the mother seems to become confused, does
not know where she is, and cannot recognize her daughter. Which of the following is this?
A a normal variant of aging and should be accepted
B an indication that the mother needs to be restrained at night for fear of hurting herself

C an indication of possible early dementia or organic brain syndrome, needing further evaluation
D controllable by administration of sleep medication early in the evening
ANSWER:C
808
A 19-year-old patient comes in for an annual refill of her birth control pills. For the past 6 years her family
physician has treated her for nodular cystic acne with what the patient feels are poor results. Which of the
following should you recommend?
A isotretinoin
B stopping her OCs
C oral tetracycline
D topical benzoyl peroxide
ANSWER:A
809
A woman and her husband are planning a trip to Mexico for their 25th anniversary. She has heard about
“traveler’s diarrhea” and wonders what advice you can give her. You should tell her to do which of the following?
A not to worry about it; very few people ever get it
B she will get it no matter what she does, and it will subside quickly on its own
C to be sure to drink plenty of water, not to eat uncooked meat or vegetables, and she will likely be fine
D to take trimethoprim-sulfa tablets along and begin them at the first signs of diarrhea
ANSWER:D
810
A 39-year-old woman comes in complaining that every night just after going to bed she awakens with a severe,
substernal burning that is relieved when she drinks a glass of milk. She is allergic to codeine and has a known
gallstonE. Physical examination shows she is 5 ft 4 in. tall and weighs 209 lB. Her general examination is normal.
There is no abdominal tenderness. Her stool is guaiac negativE. She would like to know what to do for long-term
relief. Of the following choices, how should you advise her?
A antacids before bedtime
B an upper gastrointestinal X-ray series
C cholecystectomy
D weight loss and no eating within 3 hours of bedtime
ANSWER:D
811
A 34-year-old multigravid patient comes to your office with two complaints. She has difficulty having bowel
movements. They occur about twice weekly and are associated with significant straining. She has also noticed a
painful mass at the anus and some bright red blood on her toilet paper. On

A
examination you see a bluish lump about 2 cm across the anus, a mild recto-cele, and a normal digital rectal
examination. A stool specimen is guaiac negativE. Of the following choices, how should you advise her?
B that a biopsy of the mass is necessary
C to have a colonoscopy
D
to drink 8 to 10 glasses of water each day, increase the amount of fiber in her diet, and soak the area in a tub of
warm water twice daily
ANSWER:C
812
A 27-year-old woman has just begun a new job as an administrative assistant after working for years as an account
representative at another bank. Over the last few weeks, she has developed a chronic, bilateral, nonpulsatile
headache that begins every afternoon. Her 79-year-old aunt has recently died of a cerebral aneurysm, and she had
a cousin who she believes died of a “brain tumor.” Her neurologic examination is within normal limits. What do
you tell her about the origin of her headaches?
A Given her family history, an angiogram is indicateD.
B It is a common migraine headache, and she will need further evaluation.
C She needs to see a neurologist.
D The headaches are most likely stress-related and can be managed without further testing.
ANSWER:D
813
A 48-year-old patient (gravida 2, para 2) presents for an annual examination. She has had a tubal ligation for
contraception. She reports her menses occur every 25 to 28 days and are “normal.” Her history and examination,
including stool guaiac and skin, are unremarkable other than she appears somewhat pale on examination. Which
of the following laboratory results should prompt an evaluation for a cause other than simply iron deficiency from
menstrual loss?
A hemoglobin < 11.5g/dL
B increased total iron-binding capacity (TIBC)
C microcytic hypochromic cells on peripheral smear
D normal to high reticulocyte count once corrected for the anemia
ANSWER:D
814
A 19-year-old woman (gravida 0, para 0) presents for her annual sports physical. She is 5 ft 10 in. tall and weighs
110 lB. She states that she has been having this weight for “a while” and attributes it to being the star forward for
her nationally ranked college soccer team. She does note her last menses was more than 3 months ago. Her urine
human chorionic gonadotropin (h C G) is negativE. Which of the following findings would be inconsistent that
her presentation is due to athletic involvement and instead raise the concern of an anorexic disorder?

A increased exercise tolerance
B increased physical activity
C lanugo hair
D low body weight
ANSWER:C
815
A 14-year-old patient presents complaining of knee pain that has been increasing over the past few months. Since
she is the goalie for the high school soccer team, you suspect patellofemoral dysfunction. Which of the following
would decrease a postpubertal girl’s risk for this problem?
A hyperelastic joints
B increased angle from knee to pelvic girdle
C shallow trochlear groove configuration
D tight vastus medialis obliquus musculature
ANSWER:D
816
A 38-year-old woman presents with the complaints of symmetric polyarthritis, especially in the hands and wrists,
marked morning stiffness that lasts for up to an hour, and nodules over her elbows. These complaints have been
present and increasing over the past 3 to 4 months. Of the following differential list, which is the most likely
diagnosis?
A ankylosing spondylitis
B gout
C osteoarthritis
D rheumatoid arthritis
ANSWER:D
817
A patient asks what she can do to minimize her risks of developing skin cancer. You inform her of which of the
following?
A application of sunscreen is best just before sun exposure
B high altitudes are less dangerous for sun exposure dermal injury
C one will get sun damage on overcast days
D she should use a sunblocker with a sun protection factor (SPF) rating of 8 or less
ANSWER:C
818
During her annual examination, a patient states that she is very concerned about developing skin cancer due to a
strong family history. You instruct her in the components of a skin examination that involves evaluating a lesion
for danger signs for melanoma, which would include which of the following?
A asymmetry of appearance

B consistent dark black pigmentation
C diameter of 4 mm
D nonraised surface
ANSWER:A
819
Which of the following elements is not necessary for the plaintiff to prove to win a medical professional liability
claim?
A duty
B breach of duty
C causation
D intention
ANSWER:D
820
A woman wishes to know how she can prevent toxic shock syndrome (TSS). Which of the following would be the
most protective?
A use tampons with high absorbency
B change tampons often
C maintain a neutral vaginal pH
D use pads rather than tampons
ANSWER:D
821
A 22-year-old patient is involved in a skiing accident with severe head injuries. Six months after her apparent
recovery she has not had a menstrual cyclE. She also complains of hot flushes. Her luteinizing hormone (LH),
follicle-stimulating hormone (FSH), and estradiol levels are all very low. Which of the following is most likely to
have been injured?
A periventricular nucleus
B supraoptic nucleus
C suprachiasmatic nucleus
D arcuate nucleus
ANSWER:D
822
You are examining a 34-year-old woman (gravida 3, para 2) at 38-5/7 weeks’ gestation. She is in labor (5 cm).
There is no fetal part in the pelvis. Ultrasound report notes a transverse lie with the fetal back toward the maternal
legs. Which of the following is the procedure of choice?
A expectant management anticipating spontaneous vaginal delivery
B tocolysis
C external version

D cesarean delivery
ANSWER:D
823 Normally, the pregnant woman hyperventilates. Which of the following compensates for this?
A increased tidal volume
B respiratory alkalosis
C decreased Pcc2 of the blood
D decreased plasma bicarbonate
ANSWER:D
824
A 22-year-old woman experiences amenorrhea of 6 months’ duration. Physical examination demonstrates normal
breast development and normal pelvic organs. There is no hirsutism or galactorrheA. Serum thyroid-stimulating
hormone (TSH) and prolactin levels are normal. A serum pregnancy test is negativE. What would be the next
course of action?
A administer progesterone
B administer estrogen followed by progesterone
C measure circulating estrogen levels
D measure circulating testosterone levels
ANSWER:A
825
A 33-year-old woman cannot feel the string of her intrauterine contraceptive device (IUCD). Her last menstrual
period (LMP) was 1 week ago. A serum pregnancy test is negativE. Which of the following is the best immediate
action?
A insert another IUCD to replace the lost one
B obtain an abdominal radiogram
C obtain a pelvic ultrasound
D probe the cervical canal gently to pull down the string
ANSWER:D
826
A 35-year-old woman complains of irregular vaginal bleeding and abdominal pain. Her LMP was 8 weeks ago. A
laparoscopic tubal fulguration was performed 2 years ago for permanent sterilization. She wants to know if her
symptoms are related to the previous surgical sterilization. The most appropriate response is to advise her that a
known complication of female sterilization is which of the following?
A dysmenorrhea
B anovulation
C irregular bleeding
D pregnancy

ANSWER:D
827
A 58-year-old woman consults you for vulvar pruritus. On pelvic examination, you note thin, atrophic skin with
whitish coloration over the entire vulvA. Which of the following diagnoses is most likely in this patient?
A vulvar carcinoma
B vulvar intraepithelial neoplasia
C hyperkeratosis
D lichen sclerosis
ANSWER:D
828
A 21-year-old woman comes to your office complaining of severe vulvo-vaginal itching and discharge. She just
finished a course of antibiotics for an uncomplicated urinary tract infection, and she states that her urinary
symptoms have resolved. She has been sexually active with the same male partner for more than a year. They use
latex condoms, and she has been taking oral contraceptive pills for the past 3 months. She has no medical
problems or history of sexually transmitted infections (STIs). Her annual Papanicolaou (Pap) tests have all been
normal. On inspection of the external genitalia, you note vulvar erythema, fissures, and swelling. On speculum
examination, you note a thick, white, curdy discharge adherent to the vaginal walls with no odor. She has no
vulvovaginal or cervical lesions. You perform a gross and microscopic examination of the vaginal dischargE. The
vaginal pH is 4, the whiff test is negative, the wet mount (saline-prepped slide) reveals no evidence of clue cells or
trichomonads, and the KOH prepped slide reveals several pseudohyphaE.This patient returns 2 weeks later stating
that she has not responded to the treatment you prescribed. What should you do next?
A repeat the same treatment, but double the dose
B repeat the same treatment, but double the duration of use
C reconsider the diagnosis, and reevaluate the patient
D apply topical metronidazole gel to her vulvar and vaginal areas
ANSWER:C

829
A 21-year-old woman comes to your office complaining of severe vulvo-vaginal itching and dischargE. She just
finished a course of antibiotics for an uncomplicated urinary tract infection, and she states that her urinary
symptoms have resolved. She has been sexually active with the same male partner for more than a year. They use
latex condoms, and she has been taking oral contraceptive pills for the past 3 months. She has no medical
problems or history of sexually transmitted infections (STIs). Her annual Papanicolaou (Pap) tests have all been
normal. On inspection of the external genitalia, you note vulvar erythema, fissures, and swelling. On speculum
examination, you note a thick, white, curdy discharge adherent to the vaginal walls with no odor. She has no
vulvovaginal or cervical lesions. You perform a gross and microscopic examination of the vaginal dischargE. The
vaginal pH is 4, the whiff test is negative, the wet mount (saline-prepped slide) reveals no evidence of clue cells or
trichomonads, and the KOH prepped slide reveals several pseudohyphaE.Which of the following is not included in
the classification for uncomplicated vulvovaginal candidiasis as defined by the Centers for Disease Control and
Prevention CDC)?
A sporadic and infrequent episodes
B mild to moderate signs and symptoms
C occurring in pregnant women
D albicans
ANSWER:C
830
A 21-year-old woman comes to your office complaining of severe vulvo-vaginal itching and dischargE. She just
finished a course of antibiotics for an uncomplicated urinary tract infection, and she states that her urinary
symptoms have resolveD. She has been sexually active with the same male partner for more than a year. They use
latex condoms, and she has been taking oral contraceptive pills for the past 3 months. She has no medical
problems or history of sexually transmitted infections (STIs). Her annual Papanicolaou (Pap) tests have all been
normal. On inspection of the external genitalia, you note vulvar erythema, fissures, and swelling. On speculum
examination, you note a thick, white, curdy discharge adherent to the vaginal walls with no odor. She has no
vulvovaginal or cervical lesions. You perform a gross and microscopic examination of the vaginal dischargE. The
vaginal pH is 4, the whiff test is negative, the wet mount (saline-prepped slide) reveals no evidence of clue cells or
trichomonads, and the KOH prepped slide reveals several pseudohyphaE.Which of the following statements
regarding prophylactic antifungal therapy for recurrent VVC is true?
A
there is no evidence to support that prophylactic antifungal therapy for recurrent VVC reduces the risk of
recurrence
B
prophylactic therapy for recurrent VCC is not necessary for nonimmunocompromised patients since their
symptoms are not severe
C prophylactic therapy for recurrent VCC is effective indefinitely, even after therapy has been discontinued

D
oral flucanozole therapy (150 mg orally once every 3 days for 2 weeks, followed by 150 mg orally each week for
6 months) has been shown to decrease the number of VVC episodes in women suffering from recurrent VVC
ANSWER:D
831
A 29-year-old woman comes to your office with a 2-week history of a persistent, malodorous vaginal dischargE.
The unpleasant “fishy” odor appears to worsen after sex. She denies any vaginal itching, urinary symptoms, or
any other complaints. She is in a long-standing monogamous relationship with her husband, who is asymptomatiC.
She has no history of sexually transmitted diseases (STDs) or abnormal Pap test results. She has been douching
weekly for the past several months. On examination, there is a thin, milky, off-white discharge present at the
introitus without any evidence of vulvar irritation. On speculum examination, the discharge is homogeneous and
pooling on the floor of the vagina with no signs of vaginal or cervical inflammation. You perform a gross and
microscopic examination of the vaginal discharge: The pH is 6, the whiff test is strongly positive, the wet mount
slide reveals the presence of several clue cells but no trichomonads or polymorphonuclear/white blood cells
(WBCs), and the KOH slide reveals no evidence of pseudohyphae or budding yeast cells. What is the most likely
diagnosis in this patient?
A physiologic discharge
B trichomoniasis
C candidiasis
D bacterial vaginosis
ANSWER:D
832
A 29-year-old woman comes to your office with a 2-week history of a persistent, malodorous vaginal dischargE.
The unpleasant “fishy” odor appears to worsen after sex. She denies any vaginal itching, urinary symptoms, or
any other complaints. She is in a long-standing monogamous relationship with her husband, who is asymptomatiC.
She has no history of sexually transmitted diseases (STDs) or abnormal Pap test results. She has been douching
weekly for the past several months. On examination, there is a thin, milky, off-white discharge present at the
introitus without any evidence of vulvar irritation. On speculum examination, the discharge is homogeneous and
pooling on the floor of the vagina with no signs of vaginal or cervical inflammation. You perform a gross and
microscopic examination of the vaginal discharge: The pH is 6, the whiff test is strongly positive, the wet mount
slide reveals the presence of several clue cells but no trichomonads or polymorphonuclear/white blood cells
(WBCs), and the KOH slide reveals no evidence of pseudohyphae or budding yeast cells. Which of the following
statements regarding this patient’s condition is A.re) true?
A it is considered a sexually transmitted infection

B treating the partner will prevent recurrence
C it has no association with preterm labor
D it is the result of an overgrowth of lactobacilli in the vagina
ANSWER:D
833
A 29-year-old woman comes to your office with a 2-week history of a persistent, malodorous vaginal dischargE.
The unpleasant “fishy” odor appears to worsen after sex. She denies any vaginal itching, urinary symptoms, or
any other complaints. She is in a long-standing monogamous relationship with her husband, who is asymptomatiC.
She has no history of sexually transmitted diseases (STDs) or abnormal Pap test results. She has been douching
weekly for the past several months. On examination, there is a thin, milky, off-white discharge present at the
introitus without any evidence of vulvar irritation. On speculum examination, the discharge is homogeneous and
pooling on the floor of the vagina with no signs of vaginal or cervical inflammation. You perform a gross and
microscopic examination of the vaginal discharge: The pH is 6, the whiff test is strongly positive, the wet mount
slide reveals the presence of several clue cells but no trichomonads or polymorphonuclear/white blood cells
(WBCs), and the KOH slide reveals no evidence of pseudohyphae or budding yeast cells. Which of the following
is no longer an acceptable treatment for this patient’s condition, according to the CDC?
A metronidazole (500 mg orally twice a day for 7 days)
B metronidazole (2 g orally for a single dose)
C metronidazole gel 0.75% (5 g intravaginally at bedtime for 5 days)
D clindamycin (300 mg orally twice a day for 7 days)
ANSWER:B
834
A 29-year-old woman comes to your office with a 2-week history of a persistent, malodorous vaginal dischargE.
The unpleasant “fishy” odor appears to worsen after sex. She denies any vaginal itching, urinary symptoms, or
any other complaints. She is in a long-standing monogamous relationship with her husband, who is asymptomatiC.
She has no history of sexually transmitted diseases (STDs) or abnormal Pap test results. She has been douching
weekly for the past several months. On examination, there is a thin, milky, off-white discharge present at the
introitus without any evidence of vulvar irritation. On speculum examination, the discharge is homogeneous and
pooling on the floor of the vagina with no signs of vaginal or cervical inflammation. You perform a gross and
microscopic examination of the vaginal discharge: The pH is 6, the whiff test is strongly positive, the wet mount
slide reveals the presence of several clue cells but no trichomonads or polymorphonuclear/white blood cells
(WBCs), and the KOH slide reveals no evidence of pseudohyphae or budding yeast cells. What is the most
common class of organisms associated with this patient’s condition?

A aerobic bacteria
B anaerobic bacteria
C virus
D fungi/yeast
ANSWER:B
835
A 29-year-old woman comes to your office with a 2-week history of a persistent, malodorous vaginal dischargE.
The unpleasant “fishy” odor appears to worsen after sex. She denies any vaginal itching, urinary symptoms, or
any other complaints. She is in a long-standing monogamous relationship with her husband, who is asymptomatiC.
She has no history of sexually transmitted diseases (STDs) or abnormal Pap test results. She has been douching
weekly for the past several months. On examination, there is a thin, milky, off-white discharge present at the
introitus without any evidence of vulvar irritation. On speculum examination, the discharge is homogeneous and
pooling on the floor of the vagina with no signs of vaginal or cervical inflammation. You perform a gross and
microscopic examination of the vaginal discharge: The pH is 6, the whiff test is strongly positive, the wet mount
slide reveals the presence of several clue cells but no trichomonads or polymorphonuclear/white blood cells
(WBCs), and the KOH slide reveals no evidence of pseudohyphae or budding yeast cells. Treatment for BV is
indicated for all of the following patients except
A all nonpregnant women who have signs and symptoms of BV
B women who have evidence of BV based on Pap smear
C all pregnant women who have signs and symptoms of BV
D women who have a reported history of allergy to metronidazole
ANSWER:B
836
A 17-year-old woman comes to your office with her partner complaining of severe vaginal itching and
malodorous dischargE. She denies any vaginal bleeding or urinary symptoms. She has been sexually active with a
new partner for the past 3 months. On external genital examination, you note vulvar edema and erythemA.
Speculum examination reveals copious, frothy, yellow-green, malodorous discharge with petechial-like lesions on
the cervix. A bimanual examination reveals no cervical motion tenderness and no uterine or adnexal masses or
tenderness. You perform a gross and microscopic examination of the vaginal discharge: The pH is 6, the whiff test
is slightly positive, the wet mount reveals several motile flagellated organisms and many WBCs (>10/HPF) but no
clue cells, and there are no pseudohyphae or budding yeast cells noted on the KOH slidE.What is the most likely
diagnosis in this patient?
A candidiasis
B trichomoniasis
C bacterial vaginosis

D physiologic discharge
ANSWER:B
837
A 17-year-old woman comes to your office with her partner complaining of severe vaginal itching and
malodorous dischargE. She denies any vaginal bleeding or urinary symptoms. She has been sexually active with a
new partner for the past 3 months. On external genital examination, you note vulvar edema and erythemA.
Speculum examination reveals copious, frothy, yellow-green, malodorous discharge with petechial-like lesions on
the cervix. A bimanual examination reveals no cervical motion tenderness and no uterine or adnexal masses or
tenderness. You perform a gross and microscopic examination of the vaginal discharge: The pH is 6, the whiff test
is slightly positive, the wet mount reveals several motile flagellated organisms and many WBCs (>10/HPF) but no
clue cells, and there are no pseudohyphae or budding yeast cells noted on the KOH slidE.All of the following
statements are true regarding the patient’s condition except
A it is a sexually transmitted infection
B it is a potential cause of preterm labor
C males with this condition are usually symptomatic
D Pap tests are not reliable diagnostic tests for this condition
ANSWER:C
838
A 17-year-old woman comes to your office with her partner complaining of severe vaginal itching and
malodorous dischargE. She denies any vaginal bleeding or urinary symptoms. She has been sexually active with a
new partner for the past 3 months. On external genital examination, you note vulvar edema and erythemA.
Speculum examination reveals copious, frothy, yellow-green, malodorous discharge with petechial-like lesions on
the cervix. A bimanual examination reveals no cervical motion tenderness and no uterine or adnexal masses or
tenderness. You perform a gross and microscopic examination of the vaginal discharge: The pH is 6, the whiff test
is slightly positive, the wet mount reveals several motile flagellated organisms and many WBCs (>10/HPF) but no
clue cells, and there are no pseudohyphae or budding yeast cells noted on the KOH slidE.All of the following are
acceptable treatments for her condition except
A clindamycin phosphate cream (5 g intravaginally at bedtime for 5 to 7 days)
B tinidazole (2 g orally in a single dose)
C metronidazole (500 mg orally twice a day for 7 days)
D metronidazole gel (5 g intravaginally twice a day for 7 days)
ANSWER:D

839
A 17-year-old woman comes to your office with her partner complaining of severe vaginal itching and
malodorous dischargE. She denies any vaginal bleeding or urinary symptoms. She has been sexually active with a
new partner for the past 3 months. On external genital examination, you note vulvar edema and erythemA.
Speculum examination reveals copious, frothy, yellow-green, malodorous discharge with petechial-like lesions on
the cervix. A bimanual examination reveals no cervical motion tenderness and no uterine or adnexal masses or
tenderness. You perform a gross and microscopic examination of the vaginal discharge: The pH is 6, the whiff test
is slightly positive, the wet mount reveals several motile flagellated organisms and many WBCs (>10/HPF) but no
clue cells, and there are no pseudohyphae or budding yeast cells noted on the KOH slidE.Which of the following
recommendations should you give her at this time?
A her partner should be treated for trichomonas only if he has symptoms
B screening for other STIs is unnecessary since trichomoniasis is notan STI
C she can continue with normal sexual activity during the course of her treatment
D her partner should be treated for trichomonas even if he is asymptomatic
ANSWER:D
840
A Female with an “ASC-US” Pap Test Result. A 26-year-old woman comes to your office for her health
maintenance examination. She is married with two children, and she has no major medical illnesses. She reports a
10 pack-year history of cigarette smoking. She has had 10 heterosexual partners in her lifetime and denies a
history of sexually transmitted disease (STD). All of her Papanicolaou (Pap) tests have been normal. Her physical
examination, including pelvic, is unremarkablE. A week later, you receive her Pap result, which reads “satisfactory
for evaluation, ASC-US.”Which of the following would be appropriate as initial management for this patient?
A repeat the Pap test in 1 year
B perform an endocervical curettage only
C perform human papilloma virus (HPV) DNA testing
D perform cryotherapy
ANSWER:C
841
A Female with an “ASC-US” Pap Test Result. A 26-year-old woman comes to your office for her health
maintenance examination. She is married with two children, and she has no major medical illnesses. She reports a
10 pack-year history of cigarette smoking. She has had 10 heterosexual partners in her lifetime and denies a
history of sexually transmitted disease (STD). All of her Papanicolaou (Pap) tests have been normal. Her physical
examination, including pelvic, is unremarkablE. A week later, you receive her Pap result, which reads “satisfactory
for evaluation, ASC-US.”All of the following are known risk factors for carcinoma of the cervix except

A multiple sexual partners
B early age of first intercourse
C infection with “high-risk” HPV subtypes
D alcohol use
ANSWER:D
842
A Female with an “ASC-US” Pap Test Result. A 26-year-old woman comes to your office for her health
maintenance examination. She is married with two children, and she has no major medical illnesses. She reports a
10 pack-year history of cigarette smoking. She has had 10 heterosexual partners in her lifetime and denies a
history of sexually transmitted disease (STD). All of her Papanicolaou (Pap) tests have been normal. Her physical
examination, including pelvic, is unremarkablE. A week later, you receive her Pap result, which reads “satisfactory
for evaluation, ASC-US.”What is the most appropriate approach to a patient who undergoes cervical cancer
screening with liquid-based cytology and the Pap returns as “satisfactory for evaluation, ASC-US, positive for
high-risk HPV type”?
A repeat Pap test in 4 to 6 months
B repeat HPV DNA testing in 4 to 6 months
C colposcopy
D continue annual Pap tests
ANSWER:C
843
A Female with an “ASC-US” Pap Test Result. A 26-year-old woman comes to your office for her health
maintenance examination. She is married with two children, and she has no major medical illnesses. She reports a
10 pack-year history of cigarette smoking. She has had 10 heterosexual partners in her lifetime and denies a
history of sexually transmitted disease (STD). All of her Papanicolaou (Pap) tests have been normal. Her physical
examination, including pelvic, is unremarkablE. A week later, you receive her Pap result, which reads “satisfactory
for evaluation, ASC-US.”Carcinoma of the cervix is associated with which HPV types?
A 6, 11
B 16, 18, 31, 45
C 40, 42
D 53, 54
ANSWER:B

844
A Female with an “ASC-US” Pap Test Result. A 26-year-old woman comes to your office for her health
maintenance examination. She is married with two children, and she has no major medical illnesses. She reports a
10 pack-year history of cigarette smoking. She has had 10 heterosexual partners in her lifetime and denies a
history of sexually transmitted disease (STD). All of her Papanicolaou (Pap) tests have been normal. Her physical
examination, including pelvic, is unremarkablE. A week later, you receive her Pap result, which reads “satisfactory
for evaluation, ASC-US.”Which of the following statements is true?
A the risk of invasive carcinoma with ASC-US is less than 1.0%
B AGC is associated with endometrial neoplasia, not cervical neoplasia
C approximately 75% of women with LSIL have histologically confirmed high-grade cervical lesions C.IN 2/3)
D approximately 25% of women with HSIL have histologically confirmed high-grade cervical lesions C.IN 2/3)
ANSWER:A
845
A Female with an “ASC-US” Pap Test Result. A 26-year-old woman comes to your office for her health
maintenance examination. She is married with two children, and she has no major medical illnesses. She reports a
10 pack-year history of cigarette smoking. She has had 10 heterosexual partners in her lifetime and denies a
history of sexually transmitted disease (STD). All of her Papanicolaou (Pap) tests have been normal. Her physical
examination, including pelvic, is unremarkablE. A week later, you receive her Pap result, which reads “satisfactory
for evaluation, ASC-US.”Your colleague asks your opinion about liquidbased cytology for cervical cancer
screening. You explain that advantages of liquid-based cytology include
A it is less expensive than conventional Pap tests
B it permits reflex HPV testing
C collection of a cervical specimen is easier than with conventionalPap
D the patient is more comfortable during cervical sampling than with conventional Pap
ANSWER:B
846
A 33-year-old female (gravida 2, para 2) comes to your office for a routine annual examination. She has never
smoked and has no history of STDs. She is in a stable, monogamous relationship with her husbanD. Her previous
Pap smears have been normal. Her physical examination is normal, including pelvic examination. You perform a
Pap smear at this timE. Two weeks later, the Pap smear comes back as “satisfactory for evaluation, consistent with
LSIL.”Which of the following would be most appropriate as initial management for this patient?
A continue routine screening because she has no other risk factors for cervical dysplasia
B repeat a Pap test in 4 to 6 months
C perform HPV DNA typing
D perform colposcopy

ANSWER:D
847
A 33-year-old female (gravida 2, para 2) comes to your office for a routine annual examination. She has never
smoked and has no history of STDs. She is in a stable, monogamous relationship with her husbanD. Her previous
Pap smears have been normal. Her physical examination is normal, including pelvic examination. You perform a
Pap smear at this timE. Two weeks later, the Pap smear comes back as “satisfactory for evaluation, consistent with
LSIL.”The patient returns after having a colposcopy that was satisfactory (the entire squamocolumnar junction was
visualized). Her cervical biopsy was consistent with “CIN 1,” and her endocervical curettage E.CC) was “negative
for neoplasiA.” All of the following are acceptable management plans for biopsy-confirmed CIN 1 except
A repeat Pap smear and colposcopy at 12 months
B perform LEEP or cryotherapy
C perform HPV DNA testing at 12 months
D perform total hysterectomy
ANSWER:D
848
A 33-year-old female (gravida 2, para 2) comes to your office for a routine annual examination. She has never
smoked and has no history of STDs. She is in a stable, monogamous relationship with her husbanD. Her previous
Pap smears have been normal. Her physical examination is normal, including pelvic examination. You perform a
Pap smear at this timE. Two weeks later, the Pap smear comes back as “satisfactory for evaluation, consistent with
LSIL.”Which part of the cervix is most vulnerable to dysplastic changes?
A the squamous epithelium
B the columnar epithelium
C the squamocolumnar junction
D the superior lip of the cervix
ANSWER:C
849
You are seeing a 14-year-old girl today for a routine exam. She has never been sexually activE. Her mother
accompanies her to the visit and wants to know what your opinion is regarding the new “HPV (human
papillomavirus) shot.” You tell her that the quadrivalent HPV vaccine
A is not necessary because the patient is a virgin and HPV is only transmitted sexually
B is not appropriate for the patient because she is too young
C is not appropriate for the patient because she is too old
D provides 99% to 100% protection from HPV types 6, 11, 16, and 18
ANSWER:D

850
A 23-year-old woman comes to your office with a 6-month history of fatigue, anxiety, emotional lability,
difficulty concentrating, and insomniA. She also complains of breast tenderness, abdominal bloating, and food
cravings. She denies any menstrual irregularities or prodromal life stressors. These symptoms recur on a regular
basis during the week leading up to her menstrual period but completely resolve within the first 3 days of menses.
She denies any suicidal ideations. However, she tearfully admits that she feels totally incapacitated when she is
symptomatic and that these symptoms are adversely affecting her personal and professional lifE. What is the most
likely diagnosis in this patient?
A generalized anxiety disorder
B dysmenorrhea
C major depression
D premenstrual dysphoric disorder syndrome (PMDD)
ANSWER:D
851
A 23-year-old woman comes to your office with a 6-month history of fatigue, anxiety, emotional lability,
difficulty concentrating, and insomniA. She also complains of breast tenderness, abdominal bloating, and food
cravings. She denies any menstrual irregularities or prodromal life stressors. These symptoms recur on a regular
basis during the week leading up to her menstrual period but completely resolve within the first 3 days of menses.
She denies any suicidal ideations. However, she tearfully admits that she feels totally incapacitated when she is
symptomatic and that these symptoms are adversely affecting her personal and professional lifE.What is the main
characteristic that differentiates this condition from major depression?
A the type of symptoms
B the severity of symptoms
C the duration of this condition
D the timing of the symptoms relative to the menstrual cycle
ANSWER:D
852
A 23-year-old woman comes to your office with a 6-month history of fatigue, anxiety, emotional lability,
difficulty concentrating, and insomniA. She also complains of breast tenderness, abdominal bloating, and food
cravings. She denies any menstrual irregularities or prodromal life stressors. These symptoms recur on a regular
basis during the week leading up to her menstrual period but completely resolve within the first 3 days of menses.
She denies any suicidal ideations. However, she tearfully admits that she feels totally incapacitated when she is
symptomatic and that these symptoms are adversely affecting her personal and professional lifE.What is the main
characteristic that differentiates this condition from premenstrual syndrome (PMS)?
A the type of symptoms
B the severity of symptoms

C the duration of this condition
D the timing of the symptoms relative to the menstrual cycle
ANSWER:B
853
A 51-year-old woman has been experiencing progressive symptoms of profuse night sweats and frequent hot
flushes occurring both day and night. She finds her emotional state increasingly labilE. She is also experiencing
sleep disturbances and anxiety. She denies any other complaints. Her last period was approximately 12 months
ago. She has no history of medical problems or affective disorders. Her pulse is 78 beats/ minute, and her blood
pressure is 122/74 mmHg. Her pelvic examination reveals atrophic external genitalia, a small anteverted uterus,
and no adnexal masses. The rest of her examination is completely normal.What is the most likely diagnosis in this
patient?
A pheochromocytoma
B hyperthyroidism
C menopause
D generalized anxiety disorder
ANSWER:C
854
A 51-year-old woman has been experiencing progressive symptoms of profuse night sweats and frequent hot
flushes occurring both day and night. She finds her emotional state increasingly labilE. She is also experiencing
sleep disturbances and anxiety. She denies any other complaints. Her last period was approximately 12 months
ago. She has no history of medical problems or affective disorders. Her pulse is 78 beats/ minute, and her blood
pressure is 122/74 mmHg. Her pelvic examination reveals atrophic external genitalia, a small anteverted uterus,
and no adnexal masses. The rest of her examination is completely normal.What is the most effective treatment
option for this patient?
A thyroid replacement
B estrogen with progestin (hormone therapy [HT])
C antidepressants
D estrogen alone E.strogen therapy [ET])
ANSWER:B

855
A 51-year-old woman has been experiencing progressive symptoms of profuse night sweats and frequent hot
flushes occurring both day and night. She finds her emotional state increasingly labilE. She is also experiencing
sleep disturbances and anxiety. She denies any other complaints. Her last period was approximately 12 months
ago. She has no history of medical problems or affective disorders. Her pulse is 78 beats/ minute, and her blood
pressure is 122/74 mmHg. Her pelvic examination reveals atrophic external genitalia, a small anteverted uterus,
and no adnexal masses. The rest of her examination is completely normal.Alternative therapies, with demonstrated
efficacy, for this patient’s condition might include
A black cohosh
B soy isoflavones
C red clover
D selective serotonin reuptake inhibitors (SSRIs)/ selective serotonin and norepinephrine reuptake inhibitor (SSNRIs)
ANSWER:D
856
A 51-year-old woman has been experiencing progressive symptoms of profuse night sweats and frequent hot
flushes occurring both day and night. She finds her emotional state increasingly labilE. She is also experiencing
sleep disturbances and anxiety. She denies any other complaints. Her last period was approximately 12 months
ago. She has no history of medical problems or affective disorders. Her pulse is 78 beats/ minute, and her blood
pressure is 122/74 mmHg. Her pelvic examination reveals atrophic external genitalia, a small anteverted uterus,
and no adnexal masses. The rest of her examination is completely normal.The HT C.ombined estrogen/progestin)
arm of the Women’s Health Initiative (WHI) randomized, controlled trial (RCT) was stopped prematurely
primarily because patients in the treatment group demonstrated an increased relative risk for what condition?
A breast cancer
B endometrial cancer
C colon cancer
D osteoporotic fractures
ANSWER:A

857
A 51-year-old woman has been experiencing progressive symptoms of profuse night sweats and frequent hot
flushes occurring both day and night. She finds her emotional state increasingly labilE. She is also experiencing
sleep disturbances and anxiety. She denies any other complaints. Her last period was approximately 12 months
ago. She has no history of medical problems or affective disorders. Her pulse is 78 beats/ minute, and her blood
pressure is 122/74 mmHg. Her pelvic examination reveals atrophic external genitalia, a small anteverted uterus,
and no adnexal masses. The rest of her examination is completely normal.Which of the following statements
regarding postmenopausal osteoporosis is true?
A the most rapid loss of bone density occurs within the first 5 years of menopause
B surgical menopause is a lower risk factor for osteoporosis than natural menopause
C the protective effects of estrogen on bone density are maintained after discontinuation
D all women should undergo bone density testing at menopause
ANSWER:A
858
A 51-year-old woman has been experiencing progressive symptoms of profuse night sweats and frequent hot
flushes occurring both day and night. She finds her emotional state increasingly labilE. She is also experiencing
sleep disturbances and anxiety. She denies any other complaints. Her last period was approximately 12 months
ago. She has no history of medical problems or affective disorders. Her pulse is 78 beats/ minute, and her blood
pressure is 122/74 mmHg. Her pelvic examination reveals atrophic external genitalia, a small anteverted uterus,
and no adnexal masses. The rest of her examination is completely normal.Your patient also complains of chronic
urinary urgency and frequency. She admits that she needs to wear a pad and also notes leakage of urine whenever
she coughs, laughs, or sneezes. She has no history of urinary tract infections (UTIs), diabetes, or kidney problems.
The most likely diagnosis for this patient is
A urge incontinence
B stress incontinence
C mixed incontinence
D overflow incontinence
ANSWER:C
859
A 51-year-old woman has been experiencing progressive symptoms of profuse night sweats and frequent hot
flushes occurring both day and night. She finds her emotional state increasingly labilE. She is also experiencing
sleep disturbances and anxiety. She denies any other complaints. Her last period was approximately 12 months
ago. She has no history of medical problems or affective disorders. Her pulse is 78 beats/ minute, and her blood
pressure is 122/74 mmHg. Her pelvic examination reveals atrophic external genitalia, a small anteverted uterus,
and no adnexal masses. The rest of her examination is completely normal.Initial workup for this patient would
include all of the following except

A urinalysis
B postvoid residual
C voiding diary
D bladder ultrasound
ANSWER:D
860
A 14-year-old female comes to your office with a 6-month history of lower midabdominal pain. The pain is
colicky in nature, radiates to the back and upper thighs, begins with onset of menses, and lasts for 2 to 4 days. She
has missed several days of school during the past 2 months because the pain was so severE. Menarche began 18
months ago, and her menses became regular 6 months ago. The patient is not sexually activE. Physical
examination, including abdomen and pelvis, is normal. The patient has normal secondary sexual
development.What is the most likely etiology of this patient’s pain?
A primary dysmenorrhea
B pelvic inflammatory disease (PID)
C secondary dysmenorrhea
D endometriosis
ANSWER:A
861
A 14-year-old female comes to your office with a 6-month history of lower midabdominal pain. The pain is
colicky in nature, radiates to the back and upper thighs, begins with onset of menses, and lasts for 2 to 4 days. She
has missed several days of school during the past 2 months because the pain was so severE. Menarche began 18
months ago, and her menses became regular 6 months ago. The patient is not sexually activE. Physical
examination, including abdomen and pelvis, is normal. The patient has normal secondary sexual development.The
etiology of this patient’s conditions is related to
A increased levels of prostaglandin
B decreased levels of prostaglandin
C increased levels of cyclic adenosine monophosphate C.AMP)
D decreased levels of cAMP
ANSWER:A
862
A 14-year-old female comes to your office with a 6-month history of lower midabdominal pain. The pain is
colicky in nature, radiates to the back and upper thighs, begins with onset of menses, and lasts for 2 to 4 days. She
has missed several days of school during the past 2 months because the pain was so severE. Menarche began 18
months ago, and her menses became regular 6 months ago. The patient is not sexually activE. Physical
examination, including abdomen and pelvis, is normal. The patient has normal secondary sexual
development.What would you recommend as initial treatment of choice?

A nonsteroidal anti-inflammatory drugs (NSAIDs)
B oral contraceptive pills (OCPs)
C gonadotropin-releasing hormone (GnRH) agonist
D acetaminophen
ANSWER:A
863
A 14-year-old female comes to your office with a 6-month history of lower midabdominal pain. The pain is
colicky in nature, radiates to the back and upper thighs, begins with onset of menses, and lasts for 2 to 4 days. She
has missed several days of school during the past 2 months because the pain was so severE. Menarche began 18
months ago, and her menses became regular 6 months ago. The patient is not sexually activE. Physical
examination, including abdomen and pelvis, is normal. The patient has normal secondary sexual development.The
pathophysiology of this patient’s pain is associated with
A vasodilation of the uterine arteries
B vasoconstriction of the uterine arteries
C vasodilation of the pelvic veins
D vasodilation of the uterine veins
ANSWER:B
864
A 14-year-old female comes to your office with a 6-month history of lower midabdominal pain. The pain is
colicky in nature, radiates to the back and upper thighs, begins with onset of menses, and lasts for 2 to 4 days. She
has missed several days of school during the past 2 months because the pain was so severE. Menarche began 18
months ago, and her menses became regular 6 months ago. The patient is not sexually activE. Physical
examination, including abdomen and pelvis, is normal. The patient has normal secondary sexual
development.When does the disorder described usually begin?
A 13 to 16 years of age
B within 3 years of onset of the larche B.reast development)
C within 5 years of onset of the larche
D within 3 years of onset of menarche (first menses)
ANSWER:D
865
A 14-year-old female comes to your office with a 6-month history of lower midabdominal pain. The pain is
colicky in nature, radiates to the back and upper thighs, begins with onset of menses, and lasts for 2 to 4 days. She
has missed several days of school during the past 2 months because the pain was so severE. Menarche began 18
months ago, and her menses became regular 6 months ago. The patient is not sexually activE. Physical
examination, including abdomen and pelvis, is normal. The patient has normal secondary sexual
development.Which of the following is not usually associated with primary dysmenorrhea?

A pain beginning with onset of menses
B pain peaking during heaviest flow
C pain responsive to NSAIDs
D endometriosis
ANSWER:D
866
A 14-year-old female comes to your office with a 6-month history of lower midabdominal pain. The pain is
colicky in nature, radiates to the back and upper thighs, begins with onset of menses, and lasts for 2 to 4 days. She
has missed several days of school during the past 2 months because the pain was so severE. Menarche began 18
months ago, and her menses became regular 6 months ago. The patient is not sexually activE. Physical
examination, including abdomen and pelvis, is normal. The patient has normal secondary sexual
development.Which of the following is more consistent with premenstrual syndrome (PMS) than with primary
dysmenorrhea?
A symptoms that interfere with patient’s daily function
B symptoms that are cyclic in nature
C abdominal symptoms associated with menses
D symptoms with onset during late luteal phase
ANSWER:D
867
A 14-year-old female comes to your office with a 6-month history of lower midabdominal pain. The pain is
colicky in nature, radiates to the back and upper thighs, begins with onset of menses, and lasts for 2 to 4 days. She
has missed several days of school during the past 2 months because the pain was so severE. Menarche began 18
months ago, and her menses became regular 6 months ago. The patient is not sexually activE. Physical
examination, including abdomen and pelvis, is normal. The patient has normal secondary sexual development.The
patient returns 6 months later. She has tried several different NSAIDs, using the correct doses and regimens you
prescribeD. She had partial relief of her pain but still experiences such bothersome symptoms that she still misses
school occasionally. At this time, you recommend that she
A continue the NSAIDs only
B discontinue the NSAIDs and begin oxycodone
C add OCPs
D switch to danazol
ANSWER:C

868
A 24-year-old nulligravida woman comes to your office with an 18-month history of cyclic, debilitating pelvic
pain related to menses. Her menses is regular and heavy, requiring 10 to 15 thick pads on the days of heaviest
flow. She denies ever being diagnosed with a sexually transmitted infection (STI). She and her husband have been
engaging in regular intercourse without contraception for 1 year in an attempt to conceivE. On pelvic
examination, you find a normalsized, immobile, retroverted uterus with nodularity and tenderness on palpation of
the uterosacral ligaments.You inform the patient that the most likely diagnosis is
A uterine fibroid
B endometriosis
C adenomyosis
D PID
ANSWER:B
869
A 24-year-old nulligravida woman comes to your office with an 18-month history of cyclic, debilitating pelvic
pain related to menses. Her menses is regular and heavy, requiring 10 to 15 thick pads on the days of heaviest
flow. She denies ever being diagnosed with a sexually transmitted infection (STI). She and her husband have been
engaging in regular intercourse without contraception for 1 year in an attempt to conceivE. On pelvic
examination, you find a normalsized, immobile, retroverted uterus with nodularity and tenderness on palpation of
the uterosacral ligaments.You further explain that her pain is described most accurately as
A primary dysmenorrhea
B secondary dysmenorrhea
C premenstrual syndrome
D psychogenic pain
ANSWER:B
870
A 24-year-old nulligravida woman comes to your office with an 18-month history of cyclic, debilitating pelvic
pain related to menses. Her menses is regular and heavy, requiring 10 to 15 thick pads on the days of heaviest
flow. She denies ever being diagnosed with a sexually transmitted infection (STI). She and her husband have been
engaging in regular intercourse without contraception for 1 year in an attempt to conceivE. On pelvic
examination, you find a normalsized, immobile, retroverted uterus with nodularity and tenderness on palpation of
the uterosacral ligaments.Which of the following studies would establish a diagnosis in this condition?
A hysteroscopy
B ultrasound
C laparoscopy
D hysterosalpingogram (HSG)
ANSWER:C

871
A 24-year-old nulligravida woman comes to your office with an 18-month history of cyclic, debilitating pelvic
pain related to menses. Her menses is regular and heavy, requiring 10 to 15 thick pads on the days of heaviest
flow. She denies ever being diagnosed with a sexually transmitted infection (STI). She and her husband have been
engaging in regular intercourse without contraception for 1 year in an attempt to conceivE. On pelvic
examination, you find a normalsized, immobile, retroverted uterus with nodularity and tenderness on palpation of
the uterosacral ligaments.Which of the following is not an appropriate medical therapy for this condition?
A danazol
B GnRH agonist
C continuous OCPs
D clomiphene
ANSWER:D
872
A 24-year-old nulligravida woman comes to your office with an 18-month history of cyclic, debilitating pelvic
pain related to menses. Her menses is regular and heavy, requiring 10 to 15 thick pads on the days of heaviest
flow. She denies ever being diagnosed with a sexually transmitted infection (STI). She and her husband have been
engaging in regular intercourse without contraception for 1 year in an attempt to conceivE. On pelvic
examination, you find a normalsized, immobile, retroverted uterus with nodularity and tenderness on palpation of
the uterosacral ligaments.Which of the following is least consistent with secondary dysmenorrhea?
A normal pelvic examination
B onset of pain after the age of 25 years
C onset of pain during adolescence
D pain relief with NSAIDs
ANSWER:C
873
A 24-year-old nulligravida woman comes to your office with an 18-month history of cyclic, debilitating pelvic
pain related to menses. Her menses is regular and heavy, requiring 10 to 15 thick pads on the days of heaviest
flow. She denies ever being diagnosed with a sexually transmitted infection (STI). She and her husband have been
engaging in regular intercourse without contraception for 1 year in an attempt to conceivE. On pelvic
examination, you find a normalsized, immobile, retroverted uterus with nodularity and tenderness on palpation of
the uterosacral ligaments.Other causes of secondary dysmenorrhea include all of the following except
A PID
B chronic use of OCPs
C uterine fibroids
D IUD
ANSWER:B

874
A 35-year-old female presents to your office with concerns about heavy menstrual periods for the past year that
occur at irregular intervals. She explains that sometimes her menses comes twice a month but other times will skip
2 months in a row. Her menses may last 7 to 10 days and require 10 to 15 thick sanitary napkins on the heaviest
days. She admits to some fatigue, but she denies any lightheadedness. She has no pain with menses or intercoursE.
She denies any vaginal discharge or any other symptoms. She is a nonsmoker. She has had normal Pap smears in
the past. She is in a stable monogamous relationship with her husband and denies a history of sexually transmitted
infections (STIs). On physical examination, her blood pressure is 120/80 mmHg and her body mass index B.MI) is
32. Her physical examination is normal, including pelvic exam.The patient’s bleeding pattern is best described as
A menometorrhagia
B polymenorrhea
C menorrhagia
D metorrhagia
ANSWER:A
875
A 35-year-old female presents to your office with concerns about heavy menstrual periods for the past year that
occur at irregular intervals. She explains that sometimes her menses comes twice a month but other times will skip
2 months in a row. Her menses may last 7 to 10 days and require 10 to 15 thick sanitary napkins on the heaviest
days. She admits to some fatigue, but she denies any lightheadedness. She has no pain with menses or intercoursE.
She denies any vaginal discharge or any other symptoms. She is a nonsmoker. She has had normal Pap smears in
the past. She is in a stable monogamous relationship with her husband and denies a history of sexually transmitted
infections (STIs). On physical examination, her blood pressure is 120/80 mmHg and her body mass index B.MI) is
32. Her physical examination is normal, including pelvic exam.Which of the following tests is not appropriate for
the initial workup of this patient?
A complete blood count C.BC)
B assessment for history of bleeding dyscrasia
C free testosterone and dehydroepiandrosterone sulfate D.HEAS)
D urine pregnancy test
ANSWER:C

876
A 35-year-old female presents to your office with concerns about heavy menstrual periods for the past year that
occur at irregular intervals. She explains that sometimes her menses comes twice a month but other times will skip
2 months in a row. Her menses may last 7 to 10 days and require 10 to 15 thick sanitary napkins on the heaviest
days. She admits to some fatigue, but she denies any lightheadedness. She has no pain with menses or intercoursE.
She denies any vaginal discharge or any other symptoms. She is a nonsmoker. She has had normal Pap smears in
the past. She is in a stable monogamous relationship with her husband and denies a history of sexually transmitted
infections (STIs). On physical examination, her blood pressure is 120/80 mmHg and her body mass index B.MI) is
32. Her physical examination is normal, including pelvic exam.The most likely diagnosis is
A DUB
B PID
C endometrial carcinoma
D bleeding dyscrasia
ANSWER:A
877
A 35-year-old female presents to your office with concerns about heavy menstrual periods for the past year that
occur at irregular intervals. She explains that sometimes her menses comes twice a month but other times will skip
2 months in a row. Her menses may last 7 to 10 days and require 10 to 15 thick sanitary napkins on the heaviest
days. She admits to some fatigue, but she denies any lightheadedness. She has no pain with menses or intercoursE.
She denies any vaginal discharge or any other symptoms. She is a nonsmoker. She has had normal Pap smears in
the past. She is in a stable monogamous relationship with her husband and denies a history of sexually transmitted
infections (STIs). On physical examination, her blood pressure is 120/80 mmHg and her body mass index B.MI) is
32. Her physical examination is normal, including pelvic exam.What is the most likely underlying mechanism for
this patient’s abnormal bleeding?
A a coagulation defect
B anovulation
C uterine pathology
D cervical pathology
ANSWER:B

878
3A 35-year-old female presents to your office with concerns about heavy menstrual periods for the past year that
occur at irregular intervals. She explains that sometimes her menses comes twice a month but other times will skip
2 months in a row. Her menses may last 7 to 10 days and require 10 to 15 thick sanitary napkins on the heaviest
days. She admits to some fatigue, but she denies any lightheadedness. She has no pain with menses or intercoursE.
She denies any vaginal discharge or any other symptoms. She is a nonsmoker. She has had normal Pap smears in
the past. She is in a stable monogamous relationship with her husband and denies a history of sexually transmitted
infections (STIs). On physical examination, her blood pressure is 120/80 mmHg and her body mass index B.MI) is
32. Her physical examination is normal, including pelvic exam.Your patient returns to discuss test results. Her
hemoglobin is 10.8 g/dL. She does not desire future fertility and has no method of birth control at this timE.
Which of the following therapies would not be an appropriate medical management option for this patient?
A iron supplementation
B cyclic progestin
C medroxyprogesterone acetate injection D.epo-Provera)
D combined oral contraceptives
ANSWER:B
879
25-year-old female (gravida 0, para 0) presents to your office complaining of not having her period for the past 6
months. She previously had regular cycles since menarche at age 13 years. Her blood pressure is 100/70 mmHg,
and her BMI is 19. Her physical exam is unremarkable, including pelvic exam. She has normal secondary sexual
development. Upon further questioning, she reveals that she has been training for a marathon and has lost
approximately 10 pounds in the past 2 months. She does not have an eating disorder. She is currently sexually
active with one partner and desires contraception.Which of the following best describes this patient’s bleeding
pattern?
A primary amenorrhea
B secondary amenorrhea
C dysmenorrhea
D oligomenorrhea
ANSWER:B

880
A 25-year-old female (gravida 0, para 0) presents to your office complaining of not having her period for the past
6 months. She previously had regular cycles since menarche at age 13 years. Her blood pressure is 100/70 mmHg,
and her BMI is 19. Her physical exam is unremarkable, including pelvic exam. She has normal secondary sexual
development. Upon further questioning, she reveals that she has been training for a marathon and has lost
approximately 10 pounds in the past 2 months. She does not have an eating disorder. She is currently sexually
active with one partner and desires contraception.Which of the following would be the least likely cause for this
patient’s bleeding pattern?
A pregnancy
B hypothyroidism
C hypothalamic amenorrhea
D Turner’s syndrome
ANSWER:D
881
A 25-year-old female (gravida 0, para 0) presents to your office complaining of not having her period for the past
6 months. She previously had regular cycles since menarche at age 13 years. Her blood pressure is 100/70 mmHg,
and her BMI is 19. Her physical exam is unremarkable, including pelvic exam. She has normal secondary sexual
development. Upon further questioning, she reveals that she has been training for a marathon and has lost
approximately 10 pounds in the past 2 months. She does not have an eating disorder. She is currently sexually
active with one partner and desires contraception.What is the most appropriate initial step in the evaluation of this
patient’s condition?
A progestin challenge
B hysteroscopy
C pelvic ultrasound
D Depo-Provera shot
ANSWER:A
882
A 25-year-old female (gravida 0, para 0) presents to your office complaining of not having her period for the past
6 months. She previously had regular cycles since menarche at age 13 years. Her blood pressure is 100/70 mmHg,
and her BMI is 19. Her physical exam is unremarkable, including pelvic exam. She has normal secondary sexual
development. Upon further questioning, she reveals that she has been training for a marathon and has lost
approximately 10 pounds in the past 2 months. She does not have an eating disorder. She is currently sexually
active with one partner and desires contraception.The patient’s laboratory studies come back normal. She had a
positive response to a progestin challengE. At this time, what would be the most beneficial medical therapy for this
patient?
A combined oral contraceptives

B monthly progestin pills on days 1 through 10
C monthly progestin pills on days 18 through 28
D NSAIDs
ANSWER:A
883
A 55-year-old postmenopausal woman with a history of type II diabetes presents to your office for her annual
gynecological exam. She experienced menopause approximately 3 years ago. She mentions to you that she has
had recurrent episodes of irregular “menstrual-like” vaginal bleeding, occurring every 4 to 8 weeks, for the past 6
months. She describes the bleeding as lasting from 1 to 7 days, requiring one to five pads a day. The patient has
never been on hormone therapy (HT). She complains of some fatigue but is otherwise feeling well. Her Pap
smears have always been normal. Sexual history is significant for a new sexual partner for the past 6 months. Her
blood pressure is 130/80 mmHg and her BMI is 42. The rest of her physical exam, including pelvic, is
normal.You perform a Pap smear and a gonorrhea/chlamydia screen. You also check a CBC and TSH. What else
do you recommend to the patient at this time?
A transvaginal ultrasound
B dilation and curettage D.&C)
C combined oral contraceptives
D oral progestin challenge
ANSWER:A
884
A 55-year-old postmenopausal woman with a history of type II diabetes presents to your office for her annual
gynecological exam. She experienced menopause approximately 3 years ago. She mentions to you that she has
had recurrent episodes of irregular “menstrual-like” vaginal bleeding, occurring every 4 to 8 weeks, for the past 6
months. She describes the bleeding as lasting from 1 to 7 days, requiring one to five pads a day. The patient has
never been on hormone therapy (HT). She complains of some fatigue but is otherwise feeling well. Her Pap
smears have always been normal. Sexual history is significant for a new sexual partner for the past 6 months. Her
blood pressure is 130/80 mmHg and her BMI is 42. The rest of her physical exam, including pelvic, is normal.A
transvaginal ultrasound is performed and is read as “no structural abnormalities, normal sized uterus and ovaries, 7
mm endometrial stripe noteD.” What should be the next step in this patient’s management?
A repeat the ultrasound in 6 months
B give cyclic progestin
C perform an endometrial biopsy
D give cyclic oral contraceptives
ANSWER:C

885
A 27-year-old nulligravida female presents to your office for routine exam. Upon gynecological history, you
discover that she has a 5-year history of oligomenorrhea, with only approximately two or three menses a year. She
denies intercycle spotting or premenstrual symptoms. Her last menses was 3 months ago. Her blood pressure is
120/75 mmHg and her BMI is 34. Her physical exam reveals a moderate amount of facial hair and facial acnE.
Her pelvic examination is unremarkablE.What condition do you suspect in this patient?
A adrenal tumor
B polycystic ovary syndrome (PCOS)
C hypothyroidism
D hyperprolactinoma
ANSWER:B
886
A 27-year-old nulligravida female presents to your office for routine exam. Upon gynecological history, you
discover that she has a 5-year history of oligomenorrhea, with only approximately two or three menses a year. She
denies intercycle spotting or premenstrual symptoms. Her last menses was 3 months ago. Her blood pressure is
120/75 mmHg and her BMI is 34. Her physical exam reveals a moderate amount of facial hair and facial acnE.
Her pelvic examination is unremarkablE.All of the following laboratory studies are appropriate for initial
evaluation except
A TSH
B luteinizing hormone (LH)
C follicle-stimulating hormone (FSH)
D transvaginal ultrasound
ANSWER:D
887
A 37-year-old G1P1001 female comes to your office with a 3-day history of progressive pelvic pain. She notes
some vaginal spotting but no frank bleeding. She denies any fevers, chills, diarrhea, vaginal discharge, or urinary
symptoms. Her last menstrual period was 6 weeks ago. She is married and has been trying to conceive for the past
6 months. She and her husband have one child already, a result of in vitro fertilization (IVF). She is afebrile, and
her pulse and blood pressure are normal. On speculum examination, her os appears closed and there is a small
amount of dark brownish-red blood pooled in the fornix. There is no mucopurulent discharge or cervical motion
tenderness. On bimanual examination, her uterus feels slightly enlarged and boggy, and the left adnexa is tender
without any obvious mass. A wet prep is normal except for many red blood cells.Which of the following is the
most likely diagnosis?
A acute cervicitis
B ectopic pregnancy
C acute pelvic inflammatory disease (PID)

D completed spontaneous abortion
ANSWER:B
888
A 37-year-old G1P1001 female comes to your office with a 3-day history of progressive pelvic pain. She notes
some vaginal spotting but no frank bleeding. She denies any fevers, chills, diarrhea, vaginal discharge, or urinary
symptoms. Her last menstrual period was 6 weeks ago. She is married and has been trying to conceive for the past
6 months. She and her husband have one child already, a result of in vitro fertilization (IVF). She is afebrile, and
her pulse and blood pressure are normal. On speculum examination, her os appears closed and there is a small
amount of dark brownish-red blood pooled in the fornix. There is no mucopurulent discharge or cervical motion
tenderness. On bimanual examination, her uterus feels slightly enlarged and boggy, and the left adnexa is tender
without any obvious mass. A wet prep is normal except for many red blood cells.What is the most appropriate
initial test that should be performed to support your diagnosis?
A urine or serum -human chorionic gonadotropin -hCG)
B hysterosalpingogram
C culdocentesis
D pelvic/transvaginal ultrasound
ANSWER:A
889
A 37-year-old G1P1001 female comes to your office with a 3-day history of progressive pelvic pain. She notes
some vaginal spotting but no frank bleeding. She denies any fevers, chills, diarrhea, vaginal discharge, or urinary
symptoms. Her last menstrual period was 6 weeks ago. She is married and has been trying to conceive for the past
6 months. She and her husband have one child already, a result of in vitro fertilization (IVF). She is afebrile, and
her pulse and blood pressure are normal. On speculum examination, her os appears closed and there is a small
amount of dark brownish-red blood pooled in the fornix. There is no mucopurulent discharge or cervical motion
tenderness. On bimanual examination, her uterus feels slightly enlarged and boggy, and the left adnexa is tender
without any obvious mass. A wet prep is normal except for many red blood cells.A Serum-hCG is ordered and is
reported soon thereafter to be 5000 mIU/mL. Based on this level of serum -hCG, you would expect which of the
following?
A an intrauterine pregnancy visible on transvaginal ultrasound only
B an intrauterine pregnancy visible on transabdominal ultrasound only
C an intrauterine pregnancy visible on both transvaginal and transabdominal ultrasound
D no intrauterine pregnancy yet because it is still too early
ANSWER:C

890
A 37-year-old G1P1001 female comes to your office with a 3-day history of progressive pelvic pain. She notes
some vaginal spotting but no frank bleeding. She denies any fevers, chills, diarrhea, vaginal discharge, or urinary
symptoms. Her last menstrual period was 6 weeks ago. She is married and has been trying to conceive for the past
6 months. She and her husband have one child already, a result of in vitro fertilization (IVF). She is afebrile, and
her pulse and blood pressure are normal. On speculum examination, her os appears closed and there is a small
amount of dark brownish-red blood pooled in the fornix. There is no mucopurulent discharge or cervical motion
tenderness. On bimanual examination, her uterus feels slightly enlarged and boggy, and the left adnexa is tender
without any obvious mass. A wet prep is normal except for many red blood cells.In which anatomic site do most
ectopic pregnancies occur?
A the ampulla of the fallopian tube
B the isthmus of the fallopian tube
C the interstitial portion of the fallopian tube
D the interstitial portion of the ovary
ANSWER:A
891
A 37-year-old G1P1001 female comes to your office with a 3-day history of progressive pelvic pain. She notes
some vaginal spotting but no frank bleeding. She denies any fevers, chills, diarrhea, vaginal discharge, or urinary
symptoms. Her last menstrual period was 6 weeks ago. She is married and has been trying to conceive for the past
6 months. She and her husband have one child already, a result of in vitro fertilization (IVF). She is afebrile, and
her pulse and blood pressure are normal. On speculum examination, her os appears closed and there is a small
amount of dark brownish-red blood pooled in the fornix. There is no mucopurulent discharge or cervical motion
tenderness. On bimanual examination, her uterus feels slightly enlarged and boggy, and the left adnexa is tender
without any obvious mass. A wet prep is normal except for many red blood cells.A transvaginal ultrasound reveals
a mass in the adnexal and no evidence of an intrauterine pregnancy. Which of the following medical treatments is
appropriate for this condition?
A intravenous estrogen
B combined oral contraceptives C.ontain estrogen and progestin)
C progestin-only pills
D IM methotrexate
ANSWER:D

892
A 37-year-old G1P1001 female comes to your office with a 3-day history of progressive pelvic pain. She notes
some vaginal spotting but no frank bleeding. She denies any fevers, chills, diarrhea, vaginal discharge, or urinary
symptoms. Her last menstrual period was 6 weeks ago. She is married and has been trying to conceive for the past
6 months. She and her husband have one child already, a result of in vitro fertilization (IVF). She is afebrile, and
her pulse and blood pressure are normal. On speculum examination, her os appears closed and there is a small
amount of dark brownish-red blood pooled in the fornix. There is no mucopurulent discharge or cervical motion
tenderness. On bimanual examination, her uterus feels slightly enlarged and boggy, and the left adnexa is tender
without any obvious mass. A wet prep is normal except for many red blood cells.Which of the following
statements is true regarding the clinical presentation of ectopic pregnancy?
A the majority of women present with fever higher than 100.4°F
B the majority of women report vasovagal symptoms
C the majority of women report amenorrhea or abnormal menses
D the majority of women have peritoneal signs
ANSWER:C
893
A 37-year-old G1P1001 female comes to your office with a 3-day history of progressive pelvic pain. She notes
some vaginal spotting but no frank bleeding. She denies any fevers, chills, diarrhea, vaginal discharge, or urinary
symptoms. Her last menstrual period was 6 weeks ago. She is married and has been trying to conceive for the past
6 months. She and her husband have one child already, a result of in vitro fertilization (IVF). She is afebrile, and
her pulse and blood pressure are normal. On speculum examination, her os appears closed and there is a small
amount of dark brownish-red blood pooled in the fornix. There is no mucopurulent discharge or cervical motion
tenderness. On bimanual examination, her uterus feels slightly enlarged and boggy, and the left adnexa is tender
without any obvious mass. A wet prep is normal except for many red blood cells. Major complications of ectopic
pregnancy include which of the following?
A intraabdominal hemorrhage
B hypovolemic shock
C fetal death
D A & B
ANSWER:D

894
A 21-year-old woman comes to your office complaining of severe vulvo-vaginal itching and dischargE. She just
finished a course of antibiotics for an uncomplicated urinary tract infection, and she states that her urinary
symptoms have resolveD. She has been sexually active with the same male partner for more than a year. They use
latex condoms, and she has been taking oral contraceptive pills for the past 3 months. She has no medical
problems or history of sexually transmitted infections (STIs). Her annual Papanicolaou (Pap) tests have all been
normal. On inspection of the external genitalia, you note vulvar erythema, fissures, and swelling. On speculum
examination, you note a thick, white, curdy discharge adherent to the vaginal walls with no odor. She has no
vulvovaginal or cervical lesions. You perform a gross and microscopic examination of the vaginal dischargE. The
vaginal pH is 4, the whiff test is negative, the wet mount (saline-prepped slide) reveals no evidence of clue cells or
trichomonads, and the KOH prepped slide reveals several pseudohyphaE.What is the most likely diagnosis in this
patient?
A physiologic discharge
B bacterial vaginosis
C vulvovaginal candidiasis (VVC)
D trichomoniasis
ANSWER:C
895
A 21-year-old woman comes to your office complaining of severe vulvo-vaginal itching and dischargE. She just
finished a course of antibiotics for an uncomplicated urinary tract infection, and she states that her urinary
symptoms have resolveD. She has been sexually active with the same male partner for more than a year. They use
latex condoms, and she has been taking oral contraceptive pills for the past 3 months. She has no medical
problems or history of sexually transmitted infections (STIs). Her annual Papanicolaou (Pap) tests have all been
normal. On inspection of the external genitalia, you note vulvar erythema, fissures, and swelling. On speculum
examination, you note a thick, white, curdy discharge adherent to the vaginal walls with no odor. She has no
vulvovaginal or cervical lesions. You perform a gross and microscopic examination of the vaginal dischargE. The
vaginal pH is 4, the whiff test is negative, the wet mount (saline-prepped slide) reveals no evidence of clue cells or
trichomonads, and the KOH prepped slide reveals several pseudohyphaE.You treat the patient accordingly and her
symptoms resolvE. She returns 6 months later for her routine Pap smear. The Pap smear results return as
satisfactory for evaluation, negative for intraepithelial lesion or malignancy, fungal organisms morphologically
consistent with Candida species. The patient is asymptomatic, and speculum and pelvic examination are normal.
What is the next most appropriate step?
A treat the patient for VVC only if her wet prep is positive for pseudohyphae
B treat the patient for VVC only if a vaginal culture is positive for Candida albicans
C treat the patient for VVC only if both a wet prep and vaginal culture are positive for yeast

D no intervention is required at this time
ANSWER:D
896
A 21-year-old woman comes to your office complaining of severe vulvo-vaginal itching and dischargE. She just
finished a course of antibiotics for an uncomplicated urinary tract infection, and she states that her urinary
symptoms have resolveD. She has been sexually active with the same male partner for more than a year. They use
latex condoms, and she has been taking oral contraceptive pills for the past 3 months. She has no medical
problems or history of sexually transmitted infections (STIs). Her annual Papanicolaou (Pap) tests have all been
normal. On inspection of the external genitalia, you note vulvar erythema, fissures, and swelling. On speculum
examination, you note a thick, white, curdy discharge adherent to the vaginal walls with no odor. She has no
vulvovaginal or cervical lesions. You perform a gross and microscopic examination of the vaginal dischargE. The
vaginal pH is 4, the whiff test is negative, the wet mount (saline-prepped slide) reveals no evidence of clue cells or
trichomonads, and the KOH prepped slide reveals several pseudohyphaE.Which of the following has not been
shown to increase the risk for recurrence of this condition?
A high-carbohydrate diets
B diabetes mellitus
C oral contraceptives
D frequent/prolonged antibiotic use
ANSWER:A
897 The Expected date of delivery of a human pregnancy can be calculated:
A From a change in the patient's weight.
B As 10 lunar months after the time of ovulation.
C As 40weeksafterlastmenstrualperiod.
D As 280 days from the last full moon.
ANSWER:C
898 The last menstrual period was June 30. the expected date of delivery (EDD) is​​ approximately:
A March 23.
B April 7.
C March 28.
D March7.
ANSWER:D
899 Uterine Cervix:
A Is the portion of the uterus below the isthmus
B External OS cell lining is columnar epithelium

C Laterally is attached to the round ligament
D The cervical canal is covered with stratified squamous epithelium
ANSWER:A
900 The main support of the uterus is provided by
A The round ligament
B The cardinal ligament
C The infandilo-pelvic ligament
D The integrity of the pelvis
ANSWER:B
901 The most important muscle in the pelvic floor is:
A Bulbo cavernousus.
B Ischio-cavernosus.
C Levator ani.
D Superficial transverse Perineal muscle.
ANSWER:C
902 The pelvis includes which of the following bones:
A Trochanter, hip socket, ischium, sacrum & pubis.
B Ilium, ischium, pubis, sacrum & coccyx.
C Ilium, ischium & pubis.
D Sacrum, Ischium, ilium & pubis.
ANSWER:B
903 The joint between the two pubic bones is called the:
A Sacroiliac joint.
B Pubis symphysis.
C Sacrococcygeal joint.
D Piriformis.
ANSWER:B
904 The greatest diameter of the fetal head is:
A Occipitofrontal.
B Occipitomental.
C Suboccipit bregmati
D Bitemporal.
ANSWER:B

905 Molding of the fetal head:
A Usually cause brain damage.
B Becomes progressively easier as gestational age increase.
C Increase the difficulty of delivery.
D Does not have time to occuring breech delivery.
ANSWER:D
906 The main blood supply of the vulva is:
A Inferior hemorrhoidal artery.
B Pudendal artery.
C Ilioinguinal artery.
D Femoral artery.
ANSWER:B
907 The following are typical in the female bony pelvis EXCEPT:
A Has a transverse diameter of the inlet greater than the antero-posterior diameter.
B Has an obstetric conjugate of 11-12 cm.
C Is funnel shaped.
D Has an obtuse greater sciatic notch.
ANSWER:C
908 Regarding fetal head, Choose the CORRECT answer:
A Can be delivered vaginally in persistent occipito-mental Presentation.
B Will show Spalding's sign within 12 hours of Intrauterine death.
C Can be delivered vaginally in persistent brow presentation.
D Considered to been gaged when the Biparietal diameter passes the level of the pelvic inlet.
ANSWER:D
909 The ovarian artery is a branch of:
A Common iliac artery.
B Internal iliac artery.
C Aorta.
D Hypogastric artery.
ANSWER:C
910 The normal lining of the fallopian tube is:
A Squamous epithelium.
B Transitional epithelium.

C Cuboidal epithelium.
D Columnar epithelium with cilia.
ANSWER:D
911 The cilia of the fallopian tube has the following function:
A Remove the zona pellucida which surrounds the ovum.
B Transport the ovum towards the uterus.
C Enhance the rapid division of the zygote.
D Transport the ovum towards the peritoneal cavity.
ANSWER:B
912 Hyperextension of the fetal head is found in:
A Vertex presentation
B Face presentation
C Shoulder presentation
D Breach presentation
ANSWER:B
913 Stages of labor
A The first stage commences at the time of membrane rupture
B The cervix dilates at consistent rate of 3 cm per hour in the first stage
C The third stage end with the delivery of the placenta and membranes
D Forceps or ventose may be useful in slow progress of the late 1st stage
ANSWER:C
914 All the following characteristics are applied to a pelvis favorable to vaginal delivery EXCEPT:
A Sacral promontory can not be felt.
B Obstetric conjugate is less than 10cm.
C Ischial spines are not prominent.
D Subpubic arch accepts 2 fingers.
ANSWER:B
915 In the fetus:
A The coronal suture lies between the two parietal bones.
B The umbilical artery normally contains one artery and two veins.
C Fetal lie describes the long axis of the fetus to the long axis of the mother.
D Entanglement of the umbilical cord is common in diamniotic twins.
ANSWER:C

916
Which of the following terms best describes the pelvic type of small posterior saggital diameter, convergent
sidewalls, prominent ischial spines, and narrow pubic arch?
A Android.
B Gynecoid.
C Anthropoid.
D Platypelloid.
ANSWER:A
917 The second stage of labor involves:
A Separation of the placenta.
B Effacement of the cervix.
C Expulsion of the placenta.
D Expulsion of the fetus.
ANSWER:D
918 A pelvic inlet is felt to be contracted if :
A The anterio-posterior diameter is only 12 cm.
B The transverse diameter is only 10cm.
C Platypelloid pelvis.
D The mother is short.
ANSWER:B
919 During clinical pelvimerty, which of the following is routinely measured:
A Bi-ischeal diameter.
B Transverse diameter of the inlet.
C Shape of the pubic arch.
D Flare of the iliac crest.
ANSWER:C
920 At term, the ligaments of the pelvis change. This can result in:
A Increasing rigidity of the pelvis.
B Degeneration of pelvic ground substance.
C Decreasing width of the symphysis.
D Enlargement of the pelvic cavity.
ANSWER:D
921 During clinical pelvimetry, which of the following is routinely measured:
A True conjugate.

B Transverse diameter of the inlet.
C Shape of the pubic arch.
D Flare of the iliac crest.
ANSWER:C
922 During the delivery, the fetal head follow the pelvic axis. The axis is best​​ described as:
A A straight line.
B A curved line, 1ST directed anteriorly then caudal.
C Acurvedline, 1ST directed posteriorly then caudal.
D A curved line, 1ST directed posteriorly then cephalic.
ANSWER:C
923 A head of level (one fifth) indicates:
A Indicates that one fifth of the head is below the pelvic brim.
B Indicates that the head is engaged.
C Indicated that forceps may not be used.
D Indicates that head is at the level of the ischial spines.
ANSWER:B
924 In a vertex presentation, the position is determined by the relationship of what fetal part to the Mother's pelvis:
A Mentum.
B Sacrum.
C Acromian.
D Occiput.
ANSWER:D
925 Signs of Placental separation after delivery include:
A Bleeding.
B Changes of uterine shape from discoid to globular.
C Lengthening of the umbilical cord.
D All of the above
ANSWER:D
926 The persistence of which of the following is usually incompatible with spontaneous delivery at term:
A Occiput left posterior
B Mentum posterior.
C Mentum anterior.
D Occiput anterior.

ANSWER:B
927 An unstable lie is related to all of the following EXCEPT:
A Prematurity.
B Grand multiparty.
C Placenta previa.
D Fundal fibroid.
ANSWER:D
928 The relation of the fetal parts to one another determines:
A Presentation of the fetus.
B Lie of the fetus.
C Attitude of the fetus.
D Position of the fetus.
ANSWER:D
929 The relationship of the long axis of the fetus to the long axis of the mother is called:
A Lie.
B Presentation.
C Position.
D Attitude.
ANSWER:A
930 Engagement is strictly defined as:
A When the presenting part goes through the pelvic inlet.
B When the presenting part is level with the ischial spines.
C When the greatest Biparietal diameter of the fetal head passes the pelvic inlet.
D When the greatest Biparietal diameter of the fetal head is at the level of ischial spines.
ANSWER:C
931 The fetal head may undergo changes in shape during normal delivery. The most common etiology listed is:
A Cephalohematoma.
B Molding.
C Subdural hematoma.
D Hydrocephalus.
ANSWER:B
932 If the large fontanel is the presenting part, what is the presentation?
A Vertex.

B Sinciput.
C Breech.
D Face.
ANSWER:B
933 Methods of determining fetal presentation & position include:
A Cullen's sign.
B Leopold's maneuver.
C Mauriceau-Smelli-Veit maneuver.
D Carful history taking.
ANSWER:B
934 A transverse lie of the fetus is least likely in the presence of:
A Placenta previa.
B Pelvic contraction.
C Preterm fetus.
D Normal term fetus.
ANSWER:D
935
A patient sustained a laceration of the premium during delivery, it involved the muscles of Perineal body but not
the anal sphincter. Such a laceration would be classified as :
A First degree
B Second degree
C Third degree
D Forth degree
ANSWER:B
936 An unstable lie is associated with all the following EXCEPT :
A Prematurity
B Grand multiparity
C Placenta previa
D Cervical fibroid
ANSWER:D
937
A primipara is in labor & and an episiotomy is about to be cut. Compared with a midline episiotomy, an advantage
of mediolateral episiotomy.
A Ease of repair,
B Fewer break downs.

C Lower blood loss.
D Less extension of the incision.
ANSWER:D
938 Which of the following statements about episiotomy if FALSE:
A Median (midline) episiotomy is generally considered to be less painful the mediolateral episiotomy.
B Mediolateral or lateral episiotomy may be associated with more blood loss than median one.
C
Indications for episiotomy include avoiding an imminent Perineal tear, the use of forceps, breech delivery, & the
delivery of premature infants.
D The earlier the episiotomy is done during delivery, generally the more beneficial it will be un speeding delivery.
ANSWER:D
939 The first stage of labor :
A Separation of the placenta.
B Effacement of the cervix.
C Expulsion of the placenta.
D Ends with fully Dilation of the cervix.
ANSWER:D
940 Repetitive late decelerations most commonly indicate:
A Fetal academia.
B Fetal hypoxia.
C Fetal sleep state.
D Fetal efforts of maternal sedation.
ANSWER:B
941 Electronic fetal monitoring:
A Has high specificity but low sensitivity.
B Has low specificity but high sensitivity.
C Has low specificity & sensitivity.
D Has high specificity & sensitivity.
ANSWER:B
942 What is the uterine blood flow at term:
A 50 ml/min.
B 100 to 150 ml/min.
C 300 to750 ml/min.

D 500 to 750 ml/min.
ANSWER:D
943 Regarding Fetal blood pH:
A Can only be measured postnatally.
B Is not a reliable way of assessing fetal distress.
C Is dangerous to perform & should not be done.
D Can be measured during labor.
ANSWER:D
944 The following are major indicators of fetal asphyxia:
A Old meconium at the time of induction of labor.
B Loss of acceleration.
C Deep type I deceleration in the 2ND stage of labor.
D Type II (late) decelerations with tachycardia.
ANSWER:D
945 Which of the following is NOT a characteristic of normal labor:
A Progressive cervical dilation.
B Increasing intensity of contractions.
C Uterine relaxation between contractions.
D Moderate bleeding.
ANSWER:D
946 Bishop score includes all the followings EXCEPT:
A Dilation of the cervix.
B Position of the cervix.
C The presenting part of the fetus.
D Length of the cervix.
ANSWER:C
947 During which of the following conditions would the serum Prolactin level be greatest:
A sleep.
B Ovulation.
C Parturition.
D Suckling.
ANSWER:D
948 Regarding Prostaglandins:

A Maintain the corpus luteum of early pregnancy.
B Have no role in the development of menorrhagia.
C Are involved in the onset of labor.
D Have no rule in the development of dysmenorrhea.
ANSWER:C
949 Early deceleration is :
A Associated with unengaged head of the fetus.
B Associated usually with brain asphyxia.
C A decrease in the fetal heart beat that peaks after the peak of uterine contraction.
D Results from increased vagal tone secondary to head compression.
ANSWER:D
950 The normal cord pH is :
A 6.1.
B 6.2.
C 7.0.
D 7.2.
ANSWER:D
951 The bishop score is used to predict :
A The state of the fetus at the time of delivery.
B The success rate of the induction of the labor.
C The fetal condition in the uterus.
D The maternal well being in labor.
ANSWER:B
952 Which of the following fetal scalp pH results should prompt immediate delivery:
A 7.30.
B 7.22.
C 7.18.
D 7.26.
ANSWER:C
953 Cephalopelvic disproportion in the absence of gross pelvic abnormality can be diagnosed by:
A Ultrasound.
B A maternal stature of less than 158 cm.
C Trial of labor.

D X-ray pelvimetry.
ANSWER:C
954 Umbilical cord prolapse is associated with all the following, EXCEPT :
A Post maturity.
B Cephalo pelvic disproportion.
C Multiparity.
D Anencephaly.
ANSWER:D
955 In a Case of labor with meconium stained amniotic fluid, your next step is:
A Amnio-infusion
B Close observation
C Fetal scalp blood sample
D Immediate C/S
ANSWER:C
956 In patient with bicornuate uterus when getting pregnant can get all these complication, EXCEPT:
A Polyhydramnios
B Abortion
C Preterm labor
D Abnormal fetal lie
ANSWER:A
957 In Turner's syndrome:
A A chromosomal structure of 45 XY is characteristic
B Secondary amenorrhea is usual
C Ovaries are streak
D The ovaries are multicystic
ANSWER:C
958 Sexual differentiation
A Development of male genitalia depends on the presence of functioning testes and responsive end organs
B Due to the absence of testes XX fetus exposed to androgens in uteri will NOT be musculinized
C The development of the female genital requires presence of the ovary
D 45 XO fetus will have normal ovaries
ANSWER:A
959 Turner syndrome: Which is true?

A Genetically is 46 X O
B Has testis in inguinal area
C Usually presents with primary amenorrhea.
D Has low I.Q
ANSWER:C
960 Androgen insensitivity syndrome: Which is true?
A Genotype is 46 XX
B Phenotype they are female but with ill-developed breast
C Usually have secondary amenorrhea
D They have no uterus
ANSWER:D
961 The karyotype of patient with Androgen insensitivity Syndrome is
A 46XX
B 46XY
C 47XXY
D 45XO
ANSWER:B
962 In Turner syndrome the following are usually present EXCEPT:
A the ovary are usually well developed.
B The nipple are widely spaced
C The girls are of short stature
D Has web neck
ANSWER:A
963 Regarding Androgen insensitivity syndrome all true EXCEPT:
A The chromosomal sex is 46XX.
B Scant or no pubic and axillary hair.
C No uterus
D Normal female external genetailia
ANSWER:A
964 Which is not true about Turner's syndrome:
A Short stature
B Buccal smear is chromatin positive
C The majority are 45XO

D Very low urinary estrogen titer
ANSWER:B
965 Androgen Insensitivity Syndrome:
A The characteristic features include normal uterus and breast development, and ambiguous genitalia.
B The Karyotyping is 46XX.
C They have normal female testosterone level.
D Gonadectomy must be performed after puberty because of the increased risk of malignancy.
ANSWER:D
966
While evaluating a 30-year-old woman for infertility, you diagnosed a bicornuate uterus. You explain that
additional testing is necessary because of the woman's increased risk of congenital anomalies in which system?
A Skeletal.
B Hematopoietic.
C Urinary.
D Central nervous.
ANSWER:C
967 Bicornuate uterus can cause all of the following EXCEPT:
A Abortions
B Abnormal fetal lie.
C Infertility.
D Congenital anomalies of the baby.
ANSWER:D
968 In Turner’s syndrome patients, all of the following are true EXCEPT:
A The streak ovaries should be removed surgically due to 25% tendency to be malignant.
B Are usually less than 5 feet tall.
C Have raised FSH levels.
D Have female internal genitalia.
ANSWER:A
969 The adenxea Uteri include all of the following EXCEPT:
A Ovary.
B Fallopian tubes.
C Uterus.
D Broad ligament.
ANSWER:C

970 In the development of external genitalia:
A Genital tubercles from the labia minora.
B Genital smoothing from the labia majora.
C Genital fold from scrotum in male.
D Chlydrotestone is essential for muscularity of external genitalia.
ANSWER:C
971 Congenital uterine malformations causes all of the following EXCEPT:
A Spontaneous abortions.
B Premature labor.
C Pregnancy induced hypertension.
D Abnormal fetal lie.
ANSWER:C
972 Ovarian Dysgenesis is associated with the elevation of which of the following hormones.
A Pituitary Gonadotropins.
B Estradiol.
C Estriol.
D Pregnandiol.
ANSWER:A
973 Confirmation of the diagnosis of Turner syndrome is best done by:
A Gyn PV examination.
B Pregnantriol estimation.
C Hysterosalpingography (HSG)
D Chromosomal analysis (Karyotyping).
ANSWER:D
974 In cases of androgen insensitivity syndrome the following findings are true EXCEPT:
A The chromosomal sex is 46XX
B Scant or no pubic or axillary hair.
C No uterus.
D Normal female external genitalia.
ANSWER:A
975 In testicular feminization syndrome:
A There are usually normal testes.
B Kalman's syndrome is a recognized cause.

C Breast development is usually lacking.
D There's usually very low testosterone level.
ANSWER:A
976 In Turner's syndrome, the following are usually observed EXCEPT:
A The ovaries are usually well developed.
B The nipples are widely spaced.
C The girl is of short stature.
D Has a webbed neck.
ANSWER:A
977 Follicular growth (in non-ovulating follicle) is usually followed by:
A Ovulation.
B Cyst formation.
C Atresia.
D Arrest.
ANSWER:C
978 The normal sequence of pubertal changes in the female is:
A Thelarche, Maximal growth velocity, menarche.
B Maximal growth velocity, Thelarche, menarche.
C Thelarche, menarche, maximal growth velocity.
D Menarche, maximal growth velocity, Thelarche.
ANSWER:A
979 The barr body is:
A The condensed non functioning X chromosome.
B The darkest, widest band found on chromosomes.
C On extra lobe on the female polymorpholnuclear leukocytes.
D Found only in females.
ANSWER:A
980 The most common cause of precocious puberty is :
A Idiopathic.
B Gonadoblastoma.
C Albright syndrome.
D Abnormal skull development.
ANSWER:A

981 The normal sequence of puberty is :
A Thelarche, adrenarche, growth, menarche.
B Menarche, adrenarche, thelarche ,growth.
C Growth. thelarche, adrenarche, menarche
D Adrenarche , thelarche, growth, menarche
ANSWER:A
982 Which of the following is suggestive of ovulation:
A Basal body temperature drop at least 0.5C in the second half of the cycle
B Day 21 estrogen level is elevated
C Progesterone level on day ten of the cycle is elevated
D Regular cycle with dysmenorrhea
ANSWER:D
983 The luteal phase of the menstrual cycle is associated with:
A High luteinizing hormone level
B High progesterone levels
C High prolactin level
D Low basal body temperature
ANSWER:B
984 The follicular phase of menstrual cycle is characterized by:
A Endometrial gland proliferation.
B Decreased Ovarian Estradiol production.
C Progesterone dominance.
D A fixed length of 8 days.
ANSWER:A
985
A sample of cervical mucus is taken on day 12 of the menstrual cycle. The mucus is thin, clear, & stretchy. It
placed on a slide and allowed to air dry. When placed under microscopic, what would you expect:
A Calcium citrate.
B Clear fields, devoid of bacteria Cell.
C Thick mucus with background bacteria.
D Afren pattern characteristic of estrogen.
ANSWER:D
986 Estrogen hormone is produced from all of the following organs EXCEPT:
A Corpus luteum.

B Anterior pituitary (anterior lobe of hypophysis).
C Placenta.
D Testes,
ANSWER:B
987 An involuted corpus luteum becomes a hyalinized mass known as a:
A Corpus delicti.
B Corpus granulosa.
C Graafian follicle.
D Corpus albicans.
ANSWER:D
988 Which of the following is the best method to predict the occurrence of ovulation:
A Thermogenic shift in basal body temperature.
B LHsurge.
C Endometrial decidulaization.
D Profuse, thin, acellular cervical mucous.
ANSWER:B
989 Luteal phase deficiency:
A Has inadequate luteal progesterone production.
B Has inadequate follicular estrogen production.
C Can be corrected by estrogen.
D Associated with delayed menstruation.
ANSWER:A
990
On Examination of endometrial tissue obtained from a biopsy reveals simple columnar epithelium with no sub
nuclear vacuoles. The stroma is edematous, & a tortuous gland contains secretions. These findings are consistent
which stage of menstrual cycle:
A Mid-proliferative.
B Late proliferative.
C Early secretory.
D Mid-secretory.
ANSWER:D
991 Ovulation occurs:
A 36 hours after LH surge
B hours after LH surge.

C After Prolactin surge.
D After follicles ripened in the ovary.
ANSWER:A
992 Regarding Estrogen hormone:
A It is produced in corpus luteum.
B It is responsible for secretory changes in endometrium.
C It is mainly secreted as E3 by the ovary.
D Can not be detected in the blood of postmenopausal.
ANSWER:A
993 Regarding Human Chorionic Gonadotropin, all of the following are true EXCEPT:
A It is produced by the placenta.
B Is reversible for the maintenance of corpus luteum.
C It's level doubles every 48 hours in ectopicpregnancy.
D Reaches a peak concentration in maternal serum by 10 weeks gestation.
ANSWER:C
994 Endometrial changes during the menstrual cycle:
A The basal layer of the endometrium is responsive to hormonal stimulation.
B The functional layer of the endometrium remains intact throughout the menstrual cycle.
C The increased thickness of the endometrium during the proliferative phase is due to estrogen action.
D Estrogen induces secretory changes in the endometrium & reduces mitotic activity.
ANSWER:C
995 Inadequate luteal phase is associated with all of the following EXCEPT:
A Insufficient secretion on FSH in the antecedent follicular phase.
B Induction of ovulation with Clomiphene citrate.
C Induction of ovulation with human menopausal gonadotropins.
D Administration of progesterone in the luteal phase.
ANSWER:A
996 In the days after ovulation, all of the following occurs EXCEPT:
A The basal temperature rises.
B The endometrium undergoes secretory changes.
C The plasma progesterone concentration falls.
D Cervical mucous becomes more viscous & scanty.
ANSWER:C

997 The following hormones are secreted from the anterior pituitary gland EXCEPT:
A FSH.
B HCG.
C LH.
D TSH.
ANSWER:B
998 Ovulation may be indicated by all the following, EXCEPT :
A Endometrial biopsy revealing secretary changes.
B Upward shift in the basal temperature.
C Changing of cervical mucous to thick and scanty.
D Mid-cycle elevation in Prolactin.
ANSWER:D
999 Which of the following is the primary source of estrogen ?
A Theca interna cells.
B Theca externa cells.
C Granulosa cells.
D Interstitial cells.
ANSWER:C
1000 Gonadotropin-releasing hormone (GnRH) stimulates the release of:
A Opiate peptides.
B Adrenocorticotropic hormone (ACTH).
C LH.
D Growth hormone.
ANSWER:C
1001 Raised FSH levels are found in all of the following conditions EXCEPT:
A Postmenopausal women.
B Turner's Syndrome.
C Women on Combined Oral Contraceptive Pills.
D Gonadal dysgenesis.
ANSWER:C
1002 Estrogen have all of the following actions, EXCEPT:
A Produce proliferation of the endometrium.
B Development of secondary sexual characteristics.

C Fusion of the epiphysis.
D Prevention of thrombosis.
ANSWER:D
1003 Which of the following pubertal events in is NOT mediated by gonadal estrogen production?
A Menstruation.
B Pubic hair growth.
C Breast development.
D Skeletal growth
ANSWER:B
1004 Which of the following statement regarding Prolactin is true?
A Prolactin levels decreases shortly after sleep.
B Prolactin levels increase levels increase after ingesting high glucose meals.
C Prolactin levels decreases during surgery.
D Prolactin levels increase during stress.
ANSWER:D
1005 Besides infertility, the most common symptoms of a luteal phase defect:
A Vaginal dryness.
B Early abortion.
C Tubal occlusion.
D Breast tenderness.
ANSWER:B
1006 The following are presumptive skin signs of pregnancy except:
A Chloasma
B Maculo-papular rash
C Linea Nigra
D Stretch Marks
ANSWER:B
1007 The resting pulse in pregnancy is:
A Decreased by 20 bpm.
B Decreased by 10 to 15 bpm.
C Unchanged.
D Increased by 10 to 15 bpm.
ANSWER:D

1008 Normally, pregnancy in 2ND trimester is characterized by all of the following EXCEPT:
A Elevated fasting plasma glucose.
B Decreased fasting plasma glucose.
C Elevated postprandial plasma insulin.
D Elevated postprandial plasma glucose.
ANSWER:A
1009 All are CORRECT, EXCEPT, Pregnancy is associated with:
A Increase cardiac output
B Increase venous return
C Increase peripheral resistance
D Increase pulse rate
ANSWER:C
1010 During normal pregnancy: Which is true?
A Estradiol is the principal circulating estrogen
B The blood pressure increases in first and second trimester
C The tidal volume is reduced
D In The second half of pregnancy, amniotic fluid is mostly contributed fetal urine
ANSWER:D
1011 In normal pregnancy, levels of all of the following hormones increases EXCEPT:
A Total thyroxine (T4)
B Parathyroid hormone (PTH) in the 2 ND & 3 RD trimesters.
C Free cortisol.
D Prolactin.
ANSWER:B
1012 The increase in blood volume in normal pregnancy is made up of:
A Plasma only.
B Erythrocytes only.
C More plasma than erythroblasts.
D More Erythrocytes than plasma.
ANSWER:C
1013 In the fetus, the most well oxygenated blood is allowed into the systemic circulation by the:
A Ductus arteriosus.
B Foramen ovale.

C Rt. Ventricle.
D Ligamentum teres.
ANSWER:C
1014 Changes in the urinary tract system in pregnancy include:
A Increase the glomerular filtration rate (GFR).
B Decrease in renal plasma flow (RPF).
C Marked increase in both GFR & RPF when the patient is supine.
D Increase in the amount of dead space in the urinary tract.
ANSWER:A
1015 Lowered Hemoglobin during normal pregnancy is a physiological finding. It's mainly due to:
A low iron stores in all women.
B Blood lost to the placenta
C Increased plasma volume.
D Increased cardiac output resulting in greater red cell destruction.
ANSWER:C
1016 The Maternal blood volume in normal pregnancy:
A remains stable.
B Decreases 10%.
C Increases 10%
D Increases up to 40%
ANSWER:D
1017 During pregnancy, maternal estrogen levels increases markedly. Most of this estrogen is produced by the:
A Ovaries.
B Adrenals.
C Testes.
D Placenta.
ANSWER:D
1018 During normal pregnancy, the renal glomerular filtrate rate (GFR) can increase as much as:
A 10%.
B 25%.
C 50%.
D 75%.
ANSWER:C

1019 Fetal blood is returned to the umbilical arteries & the placenta through:
A Hypogastric arteries.
B Ductus venosus.
C Portal vein.
D Inferior vena cava.
ANSWER:A
1020 In normal physiological changes in pregnancy, all of the following are increased EXCEPT:
A Glomerular filtration rate.
B Stroke volume.
C Peripheral resistance.
D Plasma volume.
ANSWER:C
1021 Regarding Renal changes in pregnancy, all of the following are true EXCEPT:
A Blood flow is increased by 10%.
B Glomerular filtration rate is increased by 50%.
C Plasma urea will be reduced.
D Glycosuria could be normal.
ANSWER:A
1022 During Pregnancy, all of the following are CORRECT EXCEPT:
A There will be hyperplasia & hypertrophy of the uterine muscle.
B Estradiol will increase the columnar epithelial of the endocervix.
C Estrogen will increase the glandular duct.
D Progestin & HPL will decrease the number of glands.
ANSWER:D
1023 Regarding renal tract during pregnancy, the following are true EXCEPT:
A The ureters are dilated.
B The renal pelvis calyces are dilated.
C The right side is affected more then the left side.
D The bladder stone increases.
ANSWER:D
1024 As pregnancy advances, which of the following hematological changes occurs?
A Plasma volume increases proportionally more than red cell volume.
B Red cell volume increases proportionally more than plasma volume.

C Plasma volume increases & red cell volume remains constant.
D Red cell volume decreases & plasma volume remains constant.
ANSWER:A
1025 Which of the following would normally be expected to increase during pregnancy:
A Plasma creatinine.
B Thyroxin-binding globulin.
C Hematocrit.
D Core temperature.
ANSWER:B
1026 The supine position is important during late pregnancy because it may cause all of the following EXCEPT:
A Complete occlusion of the inferior vena cave.
B A significant decrease in maternal ventilatory capacity.
C Hypotension & syncope.
D A significant reduction in renal blood flow & glomerular filtration.
ANSWER:B
1027 Which of the following is probably responsible for physiologic hyperventilation during pregnancy?
A Large fluctuations in plasma bicarbonate.
B Increased estrogen production.
C Increased progesterone production.
D Decreased functional residual volume.
ANSWER:C
1028 Normally the pregnant woman hyperventilates. This is compensated by:
A Increased tidal volume.
B Respiratory alkalosis.
C Decreased Pco2 of the blood.
D Decreased plasma bicarbonate.
ANSWER:D
1029 The resting pulse in pregnancy is :
A Decreased by 30 beats /min.
B Decreased by 10-15 beats/min.
C Increased by 10-15 beats/min.
D Increase by 30 beats/min.
ANSWER:C

1030 After birth, all of the following vessels constrict EXCEPT:
A Ductus arteriosus.
B Umbilical arteries.
C Ductus venosus.
D Hepatic portal vein.
ANSWER:D
1031 Select the most correct statement about fetal & neonatal IgM:
A It is almost entirely maternal in origin.
B It is approximately 75% maternal & 25% fetal in origin.
C It is almost entirely fetal in origin
D It is 25% maternal, 75% fetal in origin.
ANSWER:C
1032 Which one is true about the placenta:
A 10% maternal contribution only
B U.C covered with chorion
C U.C contain wharlton jell
D Placental lobes are the functional units.
ANSWER:D
1033 Spinnbarkheit is a term which means :
A Crystallization of the cervical mucous.
B Thickening of the cervical mucous.
C Mucous secretion of the cervix.
D Threading of the cervical mucous.
ANSWER:D
1034 Regarding Placental function:
A hCG is a glycoprotein composed of alpha & β subunits. It's secreted by cytotrophoblast.
B Human placenta lactogen enhances insulin action & improves glucose tolerance.
C
Placental Corticotropin releasing hormone increases ACTH & cortisol & causes vasoconstriction of the feto-
placental blood vessels.
D Estrogen is secreted by the feto-placental unit responsible for the growth of the myometrium & angiogenesis.
ANSWER:D
1035 Regarding placental anatomy:
A The decidua capsularis forms part of the placenta.

B The fetal side of the placenta is divided into 30-40 cotyledons.
C The intervillous space contains fetal blood.
D Fetal blood vessels develop in the mesenchymal core of the chorioni cvilli.
ANSWER:D
1036 Which of the following does NOT accurately describes the placenta in humans:
A 15-20 cm in diameter.
B 2-4 cm thick.
C Weighs about 1/6 of what the term infant does.
D Delivered from maternal & fetal tissue.
ANSWER:D
1037 Maternal serum Prolactin levels in pregnancy are highest:
A At the end of gestation just before delivery of the infant.
B Just after the delivery of the infant.
C As the placenta is released.
D The 3 RD to 4 TH day postpartum.
ANSWER:D
1038 All of the following causes Oligohydromnios EXCEPT:
A Renal agenesis
B Poor placental perfusion
C Post term pregnancy
D Anencephaly
ANSWER:D
1039 Placental insufficiency is caused by all the following, EXCEPT:
A Smoking in pregnancy.
B Post maturity.
C Dietary insufficiency in pregnancy.
D Hypertensive disorder in pregnancy.
ANSWER:C
1040 All the following hormones are products of placental synthesis, EXCEPT :
A HCG.
B HPL.
C Prolactin.
D Progesterone.

ANSWER:C
1041 We can detect the fetal heart beat by Sonography (Transvaginal) at:
A 5 weeks
B 6 weeks
C 7 weeks
D 8 weeks
ANSWER:B
1042 Pregnant lady with polyhydramnios, the cause could be:
A Fetus with oesophageal-atresia
B Fetus with polycystic kidney disease
C Fetal growth restriction
D Hyperprolactinaemia during pregnancy
ANSWER:A
1043 Polyhydramnios is associated with the following condition
A Intrauterine growth restriction
B Fetal kidney agenesis
C Diabetes insipidus
D Tracheo-oesophageal fistula
ANSWER:D
1044 All the following are possible causes of Polyhydramnios, EXCEPT:
A IUGR
B Multiple pregnancy
C Fetus with hydrops fetalis
D Fetus with duodenal atresia or neural tube defect
ANSWER:A
1045 Which of the following causes of polyhydramnios is more common:
A Twin pregnancy.
B Diabetes.
C Hydrops fetalis.
D Idiopathic.
ANSWER:D
1046
Using your knowledge of normal maternal physiology, which of the following would employ if a 38 weeks’
pregnant patient become faint while lying supine on your examination table:

A Blood transfusion.
B Turning the patient on her side.
C Oxygen by face mask.
D I.V. saline solution.
ANSWER:B
1047 Skin changes during pregnancy should include:
A Chloasma.
B Striae.
C Palmer erythema.
D All of the above
ANSWER:D
1048 The number of chromosomes in the human somatic cell is:
A 24
B 44
C 46
D 48
ANSWER:C
1049 Regarding Oogenesis & ovulation:
A Primary oocytes are formed after birth until puberty.
B The 1ST meiotic division is arrested in the diplotene stage until just before ovulation.
C Oogenesis is completed in 72 hours.
D The ova survive for 3 days after ovulation.
ANSWER:B
1050 Regarding Fertilization & implantation:
A Fertilization occurs in the inner third of the fallopian tube.
B The sperm head penetrates through the corona radiata & zona pellucida while the tail remains outside.
C The 2ND meiotic division is completed before fertilization.
D Implantation occurs at the morula stage.
ANSWER:B
1051 The second meiotic division of the oocyte is normally completed:
A at the stage of the primary follicle.
B At the stage of the Graafian follicle.
C In the peritoneal cavity.

D After the sperm penetrates the secondary oocyte
ANSWER:D
1052 Physiological changes in the reproductive system include :
A There is no change in the vagina.
B The uterus 1 st enlarges by hyperplasia then by hypertrophy.
C There is no change in the cervix.
D Estrogen has no role in the changes that occur during pregnancy.
ANSWER:B
1053 Sure sign of pregnancy is:
A Amenorrhea
B Hegar's sign
C Nausea and vomiting
D Auscultation of fetal heart
ANSWER:D
1054 If your patient is 8 weeks pregnant which one of the following USS measurement is most useful
A Crown rump length
B Biparietal diameter
C Femur length
D Placental site
ANSWER:A
1055 The following statement are all TRUE about vomiting in pregnancy, EXCEPT:
A May be cured by admission to hospital
B Is commonest in the third trimester
C Associated with multiple pregnancy
D Is associated with trophoblastic disease
ANSWER:B
1056 First trimester pregnancy may be terminated by
A Prostaglandin inhibitor
B Anti-progesterone
C β sympathomimetic agonist
D Synthetic estrogen
ANSWER:B
1057 The following ultrasonic measurements may be used to confirm or establish gestational age:

A Crown rump length
B Nuchal pad thickening
C Amniotic fluid volume
D Yolk sac volume
ANSWER:A
1058 Antenatal booking investigations include all of the following, EXCEPT:
A Complete blood count
B Blood sugar
C Hepatitis screening
D Thyroid function
ANSWER:D
1059 An Ultrasound in the 1st trimester of pregnancy is done for
A Placental localization
B Detecting of fetal weight
C Assessment of amniotic fluid volume
D Dating of the pregnancy
ANSWER:D
1060 A serum progesterone value less than 5ng/ml can exclude the diagnosis of viable pregnancy with a certainty of:
A 20%
B 40%
C 60%
D 100%
ANSWER:D
1061 In normal pregnancy, the value of β-hCG doubles every:
A 2 days.
B 4 days.
C 8 days.
D 10 days.
ANSWER:A
1062 The β-hCG curve in maternal serum in a normal pregnancy peaks at:
A 6 weeks of pregnancy.
B 8 weeks of pregnancy.
C 10 weeks of pregnancy.

D 14 weeks of pregnancy.
ANSWER:C
1063 The followings are considered normal symptoms of pregnancy EXCEPT:
A Backache due to an increased lumbar lordosis.
B Lower abdominal pain and groin pain due to stretch of round ligaments.
C Visual disturbance.
D Calf pain due to muscle spasm.
ANSWER:C
1064
A woman in early pregnancy is worried because of several small raised nodules on areola of both breasts. There
are no other findings. Your immediate management should be:
A Reassurance after thorough examination.
B Needle aspiration of the nodules.
C Surgical removal of the areola.
D Mammography.
ANSWER:A
1065 The source of progesterone that maintains the pregnancy during early 1ST trimester:
A Placenta.
B Corpus luteum.
C Corpus albicans.
D Adrenal glands.
ANSWER:B
1066 Counseling of a pregnant patient during early prenatal care should include detection of & information on:
A Smoking.
B Alcohol abuse.
C Drug abuse.
D All of the above
ANSWER:D
1067 Ultrasound examination used for:
A Fetal weight.
B Presence of multiple gestation.
C Whether abdominal masses are cystic or solid.
D All of the above
ANSWER:D

1068
Which of the following medications, when given before & during pregnancy may help to protect neural tube
defects?
A Vitamin B6.
B Iron.
C Folic acid.
D Zinc.
ANSWER:C
1069 Which of the following is/are needed by women in increased amount during pregnancy?
A Iron.
B Folic acid.
C Protein.
D All of the above
ANSWER:D
1070 Often, an increase in vaginal discharge may be noted during pregnancy, It may be:
A Bacterial.
B Caused by Trichomonas.
C Caused by Candidiasis.
D All of the above
ANSWER:D
1071
The following measures are usually performed during a routine antenatal visit for a healthy uncomplicated
pregnancy at 36 weeks gestations' EXCEPT:
A Symphysis-fundal height.
B Maternal blood pressure.
C Maternal weight.
D Midsteam urine specimen (MSU) for culture & sensitivity.
ANSWER:D
1072 Which of the following is NOT a presumptive symptom/sign of pregnancy:
A Cessation of menstruation.
B Quickening.
C Nausea & vomiting.
D Breast changes.
ANSWER:B
1073 Probable sign of pregnancy include:

A Detection of fetal movement.
B Enlargement of the abdomen.
C X-ray demonstrating a fetus.
D Lower abdominal cramps.
ANSWER:B
1074 The softening of the cervical isthmus that occurs early in gestation is called:
A Hegar's sign.
B Chadwick's sign.
C Braxton Hick's contraction.
D Von fernwald's sign.
ANSWER:A
1075
During early pregnancy, a pelvic examination may reveal that one adnexia is slightly enlarged. This is most likely
due to:
A A parovarian cyst.
B Fallopian tube hypertrophy.
C Ovarian neoplasm.
D Corpus luteal cyst.
ANSWER:D
1076 Booking investigations include all the following, EXCEPT :
A Liver function test.
B Glucose challenge test.
C CB
D US.
ANSWER:A
1077 Antenatal care can prevent all the following complications, EXCEPT :
A Anemia due to iron deficiency or folic acid deficiency.
B UTI of pyelonephritis.
C Macrosomia.
D Preterm labor.
ANSWER:D
1078 High alpha feto protein found in? EXCEPT
A IUFD
B Multiple pregnancy

C Some Ovarian Cancer
D Trisomy 21
ANSWER:D
1079 Of the following laboratory studies, which test might be done routinely at booking:
A Electrolytes.
B Urinary estriol.
C Serum glumatic-oxaloacetic transaminase.
D Hemoglobin.
ANSWER:D
1080 The following drugs cross the placenta to the fetus, EXCEPT :
A Heparin
B Tetracycline
C Warfarin
D Diazepam
ANSWER:A
1081 Which of these drugs don’t cross the placenta?
A Heparin
B Warfarin
C Tetracycline
D Degoxin
ANSWER:A
1082
Hypoplasia & yellow discoloration of the primary teeth has occurred in infants​​​ whose pregnant mothers were
treated with drug:
A Sulphonaudes.
B Penicillin.
C Streptomycin.
D Tetracycline.
ANSWER:D
1083 In fetal circulation:
A Oxygenated blood goes along the umbilical arteries
B The fetal lung is bypassed by means of ductus venosus
C The foramen ovale connects the two ventricles
D Most of the blood entering the right atrium flows into the left atrium

ANSWER:D
1084 Components of biophysical profile include all of the following, EXCEPT:
A Fetal movement
B Placental thickness
C Fetal tone
D Fetal breathing movement
ANSWER:B
1085 Antenatal fetal monitoring can NOT be accomplished by:
A Fetal kick chart.
B Fetal scalp sampling.
C Non-stress test.
D Obstetric U/S & Biophysical profile.
ANSWER:B
1086 Which of the following procedures allow the earliest retrieval of DNA for prenatal diagnosis in pregnancy:
A Fetoscopy.
B Amniocentesis.
C Chorionic Villi Sampling (CVS)
D Percutaneous Umbilical Blood Sampling (PUBS)
ANSWER:C
1087 Regarding the biophysical profile:
A Is usually done in labor.
B Never include an non-stress test.
C Includes fetal movement, fetal tone, fetal breathing, fetal heartrate & amniotic fluid.
D Includes a Doppler study.
ANSWER:C
1088 Fetal assessment include the following EXCEPT:
A Fetal biophysical profile.
B Fetal Doppler velocimetry.
C Fetal biometry.
D Fetal blood sugar sample
ANSWER:D
1089 Patients with high risk pregnancy should have:
A Follow-up in ANC every 6 weeks

B Fetal kick chart.
C Fetal maternal transfusion
D Fetal biophysical profile.
ANSWER:D
1090 A biophysical profile includes all of the following assessment parameters EXCEPT:
A Fetal movement.
B Fetal weight.
C Fetal tone.
D Fetal breathing movements.
ANSWER:B
1091 A low APGAR score at one minute:
A Is highly correlated with late neurologic sequelae.
B Indicates an academic newborn.
C Has the same significance in premature & term infants.
D Indicates the need for immediate resuscitation.
ANSWER:D
1092 APGAR's score includes all the followings EXCEPT:
A Skin color.
B Muscle tone.
C Blood pH.
D Heart rate.
ANSWER:C
1093 Apgar's score consists of all the following, EXCEPT :
A Newborn breathing.
B Newborn tone.
C Newborn heart rate.
D Newborn pH
ANSWER:D
1094 Immediate therapy for infants with suspected meconium should routinely include :
A Corticosteroid
B Antibiotics
C Sodium bicarbonate
D Clearing of the airway

ANSWER:D
1095 Multiple pregnancy increases
A In white people more than black
B With advancing maternal age
C With Bromocriptine use for infertility treatment
D If first pregnancy
ANSWER:B
1096 In twin deliveries: Which is true?
A The first twin is at greater risk than the second
B They usually go post date
C Epidural analgesia is best avoided
D There is increased risk of postpartum hemorrhage
ANSWER:D
1097 The most common cause of uterine size-date disproportion:
A Fetal macrosomia
B Polyhydramnios
C Inaccurate last menstrual period date
D Multiple pregnancy
ANSWER:C
1098 Which of the following is known to be the commonest presentation in twins?
A Breech, cephalic
B Cephalic, breech
C Cephalic, cephalic
D Breech, breech
ANSWER:C
1099 The risk of postpartum uterine atony is associated with:
A Hypotension.
B Epidural anesthesia.
C Median episiotomy.
D Twin pregnancy.
ANSWER:D
1100 The major cause of the increased risk of morbidity & mortality among twin gestation is:
A Gestational diabetes.

B Placenta previa.
C Malpresentation.
D Preterm delivery.
ANSWER:D
1101 Multiple gestations should be suspected in all of the following condition EXCEPT:
A Maternal weight gain is greater than expected.
B The uterus is larger than expected.
C Maternal AFP is elevated.
D Maternal perception of fetal movement occurs earlier than expected in gestation.
ANSWER:D
1102 The most common cause of perinatal death in mono-amniotic twin is:
A Cord entrapment.
B Cord prolapse.
C Twin-twin transfusion syndrome.
D Lethal congenital anomalies.
ANSWER:A
1103 If twin A is in a transverse lie & twin B is vertex. The most appropriate route for delivery is:
A C-section.
B Internal podalic version followed by breech extraction.
C Both
D Neither.
ANSWER:A
1104 All of the following increased in multiple gestation EXCEPT:
A Blood loss at delivery.
B The incidence of congenital anomalies.
C The incidence of cephalo-pelvic disproportion.
D The incidence of placental abruption.
ANSWER:C
1105 Regarding Hyperemesis gravidarum, which one of the following items is TRUE?
A Is a complication of multiple pregnancy
B Not known to happen in molar pregnancy
C Worsen in missed abortion
D Liver function test is not required

ANSWER:A
1106 Monozygotic twins, All of the following are correct EXCEPT :
A Has a constant incidence of 1:250 births
B Has a constant incidence 1 : 600 births
C Is not related to induction of ovulation
D Constitutes 1/3 of twins
ANSWER:B
1107 The following are complications of multiple pregnancy EXCEPT :
A Increase incidence of pre-eclamptic toxemia
B Polyhydramnios
C Increase incidence of preterm labor
D Increase incidence of gestational diabetes
ANSWER:D
1108 Twins pregnancy :
A Presentation of the second twin dictate the mode of delivery
B Internal podalic version should not be performed for the second twin
C Monozygotic twins always bearing same sex
D Commonly goes post mature
ANSWER:C
1109 The following are fetal complications in multiple pregnancy EXCEPT :
A Increase incidence of perinatal mortality & morbidity
B Increase incidence of prematurity
C Increase incidence of mal-presentation
D Increase incidence of congenital abnormalities particularly sacral agenesis
ANSWER:D
1110 The following are true for dizygotic twins EXCEPT:
A Fertilization of more than one egg by more than one sperm
B Most common type of twins represents 2/3 of cases
C Both twins are identical & of the same sex.
D There are two chorions & two amnions
ANSWER:C
1111 The following factors affect the incidence of dizygotic multiple pregnancy EXEPT :
A Induction of ovulation

B Increase maternal age
C Heredity
D Nuliparity
ANSWER:D
1112 The following statements about multiple pregnancy are true EXCEPT:
A Its occurrence in West Africa.
B Its incidence is increased by increased age and parity.
C Twin to twin transfusion common in monochromic twins.
D Can be diagnosed by ultrasound only after 12 weeks.
ANSWER:D
1113 The following are complication of multiple gestation EXCEPT :
A Increase incidence of pre-eclampsia toxemia
B Polyhydramnios
C Increase incidence of premature labor
D Increase incidence of gestational diabetes
ANSWER:D
1114 Twins can be diagnosed by :
A Large uterus after delivery of the first twin
B Uterus bigger than date during pregnancy
C Ultrasonography
D All of the above
ANSWER:D
1115 The following are common in twins pregnancy EXCEPT :
A Increase incidence of premature labor
B Increase incidence of APH
C Increase incidence PPH
D Increase incidence perinatal mortality
ANSWER:B
1116 Regarding twin pregnancies all of the following are correct EXCEPT:
A It has a higher incidence of preterm labor
B Mal-presentation of one of the main factors leading to increase incidence of C/S
C
Abruptio placenta may occur with the sudden decompression of the uterus immediately after delivery of the first
twin

D Identical or monozygotic twin arise from fertilization of two ovum
ANSWER:D
1117 All are true about monozygotic pregnancy, EXCEPT :
A The 1 st commonly presents as breech.
B Pregnancy induced hypertension is common.
C There is only one placenta.
D Polyhydramnios is frequently present.
ANSWER:A
1118 Excessive increased level of β-HCG is expected in :
A Ectopic pregnancy.
B Pregnancy of diabetic mothers.
C Twin pregnancy.
D Incomplete abortion.
ANSWER:C
1119 Regarding multiple pregnancy. All the following are true, EXCEPT :
A Is frequently complicated by premature labor.
B Is associated with an increased risk of post partum hemorrhage.
C Occurs in approximately 1 in 80 pregnancies.
D Often causes prolonged labor.
ANSWER:D
1120 Multiple pregnancy increases:
A In white people more than black
B With advancing maternal age
C With Bromocriptine use for infertility treatment
D If first pregnancy
ANSWER:B
1121 Multiple Gestation is frequently associated with all of the following EXCEPT:
A Hypertension.
B Hydramnios.
C Fertility drugs.
D Post-maturity.
ANSWER:D
1122 Regarding missed abortion, all of the following are CORRECT, EXCEPT:

A Patient may present with loss of the symptoms of pregnancy
B Per vaginal bleeding may be one of the presenting symptom
C Immediate evacuation should be done once the diagnosis is made
D Disseminated intra-vascular coagulation may occur as a sequele of missed abortion
ANSWER:C
1123
14 weeks pregnant woman had abortion and she was told that it is a complete abortion. This is true regarding
complete abortion:
A Uterus is usually bigger than date
B Cervical OS is opened with tissue inside the cervix
C Need to have evacuation of the uterus
D After complete abortion there is minimal or no pain and minimal or no bleeding
ANSWER:D
1124 In patients with three consecutive spontaneous abortion in the second trimester the most useful investigation is:
A Chromosomal analysis
B Hysterosalpingogram
C Endometrial biopsy
D Post coital test
ANSWER:B
1125 Regarding cervical incompetence, all of the following are true, EXCEPT:
A Typically causes painful abortions
B Typically causes mid-trimester abortions
C Is treated by Shirodkar suture (cervical cerculage) which is best preformed early in the second trimester
D May lead to premature rupture of the membrane
ANSWER:A
1126 Causes of first trimester abortion
A Chromosomal abnormalities
B Cervical incompetence
C Bicornuate uterus
D Gestational hypertension
ANSWER:A
1127 Management of a patient with threatened abortion includes all of the following, EXCEPT:
A Ultrasound
B Physical examination

C CBC
D Dilatation and curettage.
ANSWER:D
1128 A 25-year-old primigravida with 8 weeks threatened abortion, ultrasound would most likely reveal:
A Thickened endometrium with no gestational sac
B Feral heart motion in the adnexia
C Empty gestational sac
D An intact gestational sac with fetal
ANSWER:D
1129 Regarding incomplete abortion, all are true, EXCEPT:
A There is a history of tissue passed per vagina
B The cervix is open on vaginal examination
C Ultrasound shows retained product of conception
D Ultrasound shows intact gestational sac non-viable fetus
ANSWER:D
1130 Most common cause of first trimester abortion
A Chromosomal abnormalities
B Syphilis
C Rhesus isoimmunization
D Cervical incompetence
ANSWER:A
1131 Incompetent cervix
A Is a cause for early pregnancy loss
B Is best diagnosed by history
C Is a cause for fetal congenital abnormalities
D Is not encounted with uterine anomities
ANSWER:B
1132 Of the proposed etiologies for recurrent pregnancy wastage, the least likely is:
A Maternal trauma.
B Maternal balanced translocation.
C Paternal balanced translocation.
D Luteal phase deficiency.
ANSWER:A

1133 The most common etiology for spontaneous abortion of a recognized first trimester gestation:
A Chromosomal anomaly in 50-60% of gestations.
B Chromosomal anomaly in 20-30% of gestations.
C Maternal hypothyroidism.
D Maternal Diabetes.
ANSWER:A
1134 In threatened abortion, which one of the following items is TRUE?
A The cervix is open
B Evacuation is the best treatment
C All patients should be admitted
D In the majority of cases pregnancy will continue without any complication
ANSWER:D
1135 In case of threatened abortion :
A Fetal heart is present
B Cervix is dilated
C There is a history of passing tissue per vagina.
D Patients needs immediate evacuation
ANSWER:A
1136 Missed abortion may cause one of the following complication:
A Bone marrow depression
B Rupture uterus
C High positive serum β-hCG
D Coagulopathy
ANSWER:D
1137 Management of a patient with threatened abortion includes all the followings EXCEPT:
A Ultrasound.
B Physical exam.
C CB
D Immediate dilation and curettage
ANSWER:D
1138 In threatened abortion at 15 weeks gestation in a nulliparous patient:
A Pain is characteristic.
B The internal os is often opened.

C Fainting is characteristi
D Vaginal bleeding is usually mild.
ANSWER:D
1139 Regarding threatened abortion:
A Anti-D should be given to Rh- positive mother.
B All patients should be admitted to the hospital.
C Ultrasound should be done to confirm the diagnosis.
D Vaginal examination will reveal severe pain.
ANSWER:C
1140 Therapy in threatened abortion should include:
A Progesterone IM
B Prolonged bed rest
C Restricted activity
D Prostaglandin suppositories
ANSWER:C
1141 During the first & second trimester of pregnancy, the most common pathologic cause of vaginal bleeding :
A Hydatiform mole
B Abruptio placenta
C Ectopic pregnancy
D Abortion
ANSWER:D
1142 Which of the following items may be associated with a mid trimester abortion:
A Recurrent pelvic infection
B Maternal smoking
C Uterine anomalies
D Sickle cell disease
ANSWER:C
1143 Bleeding in early pregnancy could be causes by all of the following, EXCEPT :
A An ectopic pregnancy
B Trophoblastic disease
C Carcinoma of the ovary
D Invasive carcinoma of the cervix
ANSWER:C

1144 Which of the following is correct in the treatment of a case of threatened abortion :
A Bed rest
B Oral stillbosterol
C Curettage
D Urgent admission to hospital
ANSWER:A
1145 Abortion :
A Hasan incidence of 15% of all pregnancy
B eks missed abortion is usually managed with suction curettage
C In threatened abortion, the cervix is always open
D 1st trimester abortion is usually causes by incompetent cervical os
ANSWER:A
1146 Etiological factors in spontaneous abortion include :
A Chromosomal abnormalities
B Placental abnormalities
C Maternal disease
D All of the above
ANSWER:D
1147 A major hazard of a late missed abortion :
A A positive human chorionic gonadotropin (hCG) titer
B Systemic allergies
C Bone marrow depression
D Coagulopathy
ANSWER:D
1148 A missed abortion is :
A Death of the fetus at 36 weeks of gestation
B In which the products of conception are expelled completely
C In which the products of conception are partially expelled
D Death of the fetus before 24 weeks gestation
ANSWER:D
1149 Cervical cerculage:
A Closure of incompetence cervix
B Effective in prevention of all types of abortion

C Should be removed at 32 weeks of pregnancy
D Protect against exposure of the pregnant lady to infection
ANSWER:A
1150 A 25 primigravida with 8 weeks threatened abortion. The US would most likely reveal :
A Thickened endometrium with no gestational sac.
B Fetal heart motion in the adnexia.
C Empty gestational sa
D An intact gestational sac with fetal heart motion.
ANSWER:D
1151
weeks pregnant woman had abortion and she was told that it is a complete abortion. This is true regarding
complete abortion:
A Uterus is usually bigger than date
B Cervical OS is opened with tissue inside the cervix
C Need to have evacuation of the uterus
D After complete abortion there is minimal or no pain and minimal or no bleeding
ANSWER:D
1152 Regarding Cervical incompetence, one is true:
A Cone biopsy is not a predisposing factor
B Cerculage is contraindicated
C In not encountered with uterine anomalies
D Best diagnosed by Hx
ANSWER:D
1153 In patients with three consecutive spontaneous abortion in the second trimester the most useful investigation is:
A Chromosomal analysis
B Hysterosalpingogram
C Endometrial biopsy
D Post coital test
ANSWER:B
1154 Therapy for threatened abortion should include:
A Progesterone IM.
B D & C
C Prolonged bed rest.
D Restricted activity.

ANSWER:D
1155
Repeated 2ND trimester abortions, especially when associated with a lack of painful uterine contractions, suggests
most strongly:
A Defective germ plasm.
B Uterine myoma.
C Maternal hyperthyroidism.
D Incompetent cervical os.
ANSWER:D
1156 Inevitable abortion is usually associated with all of the following EXCEPT:
A Pain.
B Dilated cervix.
C Bleeding.
D Fever.
ANSWER:D
1157 Threatened abortion is characterized by:
A The presence of empty sac by ultrasound.
B Disappearance of pregnancy symptoms.
C Passage of vaginal vesicles.
D Vaginal bleeding.
ANSWER:D
1158 In spontaneous abortion:
A Uncontrolled blood sugar increases the risk of abortion in diabetic patients.
B If the patient has mild bleeding with opened internal os this is considered to be threatened abortion.
C Cervical incompetence is the most common cause of abortion in the 1ST trimester.
D History of bleeding & passing of tissue per vagina indicates the need for curettage without the need for US.
ANSWER:A
1159 Early bleeding in pregnancy may be caused by the following, EXCEPT:
A Incomplete abortion
B Cervical cancer
C Threatened abortion
D Vasa praevia
ANSWER:D
1160 Regarding ectopic pregnancy, all of the following are true, EXCEPT:

A Is associated with uterine enlargement
B Is situated in the ovary in about 0.5% of all cases
C Is more dangerous when it is situated in the isthmus of the fallopian tube
D Can only be diagnosed after it has ruptured
ANSWER:D
1161 Acceptable management of ruptured ectopic pregnancy
A Observation followed by Methotrexate
B Diagnostic laparoscopy followed by observation
C Repeat ultrasound next 24 hours to confirm the diagnosis
D Exploratory laparotomy and salpingectomy
ANSWER:D
1162
The following are factors affecting the choice of Methotrexate as a choice of treatment for ectopic pregnancy,
EXCEPT:
A Size of the ectopic
B Presence or absence of cardiac activity
C Level of β-hCG
D Parity of the patient
ANSWER:D
1163 The most common cause of ectopic pregnancy is:
A History of pelvic inflammatory disease
B Congenital anomalies of the tube
C Endometriosis
D Tubal surgery
ANSWER:A
1164
Following evacuation of a molar pregnancy, β-hCG titers will fall to untraceable levels in about 90% of patients
within:
A 2 weeks.
B 4 weeks.
C 8 weeks.
D 12 weeks.
ANSWER:D
1165 The most common symptom of ectopic pregnancy is :
A Profuse vaginal bleeding.

B Abdominalpain.
C Syncope.
D Dyspareunia.
ANSWER:B
1166 Acceptable management of possible rupture ectopic pregnancy would include all of the following EXCEPT:
A Exploratory laparotomy.
B Diagnostic laparoscopy.
C Partial salpingectomy.
D Observation followed by Methotrexate
ANSWER:D
1167
If the above described patient has had a previous term pregnancy prior to her ectopic pregnancy, her chances of
subsequent intrauterine pregnancy would be about:
A 80%
B 60%
C 40%
D 20%
ANSWER:A
1168 The commonest site of ectopic pregnancy is :
A Peritoneal cavity
B Mesosalpinx
C Ovary
D Ampullaofthefallopiantube
ANSWER:D
1169 Etiological factor of ectopic pregnancy include all of the following EXCEPT:
A Gonococcal Salpangitis
B Tubal surgery
C Combined OCP
D TB salpingitis
ANSWER:C
1170 In ectopic pregnancy :
A Rarely diagnosed before 12 weeks of gestation
B Usually present as obstetric emergency before 12 weeks gestation
C IM progesterone is useful

D 50% continued up to term
ANSWER:B
1171 Patient with ectopic pregnancy:
A Present with heavy per-vaginal bleeding
B Methotrexate is the treatment of the choice in patient with ruptured one
C May complain of shoulder pain.
D Present with cardio vascular collapse in all the cases
ANSWER:C
1172 In the management of ectopic pregnancy:
A β-hCG titer has a role in the Dx & management of ectopic pregnancy.
B D&C is the treatment of choice
C Methotrexate should be used if the patient hemodynamically unstable
D Shoulder pain is referred from bowel irritation
ANSWER:A
1173 Methotrexate Treatment in ectopic is contraindicated if:
A β-hCG < 1500
B No fetal heart.
C Hemoperitonium
D Gestational sac < 3 cm
ANSWER:C
1174
The following are factors affecting the choice of Methotrexate as a choice of treatment for ectopic pregnancy,
EXCEPT:
A Size of the ectopic
B Presence or absence of cardiac activity
C Level of β-hCG
D Parity of the patient
ANSWER:D
1175 The most common cause of ectopic pregnancy is:
A History of pelvic inflammatory disease
B Congenital anomalies of the tube
C Endometriosis
D Tubal surgery
ANSWER:A

1176 Lower abdominal pain and six weeks gestation:
A Vaginal examination is contraindicated.
B Right iliac fossa pain is diagnostic of appendicitis.
C Placental abruption should be considered.
D USS has reliable diagnostic information.
ANSWER:D
1177 The endometrial change of ectopic pregnancy:
A Glandular cystic hyperplasia
B Decidual transformation
C Secroteraty changes with chorial cell
D Atypical hyperplasia
ANSWER:B
1178 In ectopic pregnancy :
A The ovarian ectopic pregnancy is the most common site
B More than 90% of ectopic pregnancies occurs in the fallopian tube
C The majority of tubal pregnancies occurs in the fimbria
D The most common symptoms of ectopic pregnancy is vaginal bleeding without abdominal pain
ANSWER:B
1179 Which of the following is a contraindication to medical treatment in ectopic pregnancy?
A An intact tubal pregnancy.
B The size is less than 3cm.
C The presence of hemoperitonium.
D The absence of fetal cardiac activity.
ANSWER:C
1180 The quantitative β subunit of HCG in a serum of patient with ectopic pregnancy will :
A Rise in a rate greater than expected.
B Rise at rate consistent with the normal curve.
C Rise at a slower than expected.
D Plateau.
ANSWER:C
1181 Antepartum haemorrhage may be caused by the following, EXCEPT:
A Placenta previa
B Cervical cancer

C Abruptio placenta
D Ectopic pregnancy
ANSWER:D
1182 Each of the following typical feature of placenta previa, EXCEPT:
A Painless bleeding
B First episode of bleeding is usually self limited
C May be associated with post coital bleeding
D Commonly as sociated with coagulopathy
ANSWER:D
1183
A 33 year old woman at 37 weeks gestation confirmed by early sonogram presents with moderate to sever
vaginal bleeding, and is note by sonogram to have placenta previa, which of the following is the best
management for her.
A Induction of labor
B Give tocolytic drugs
C Caesarean section
D Expectant management
ANSWER:C
1184 Regarding Abruptio placenta:
A Postpartum hemorrhage occurs only when there in hypofibrinogenemia
B Maternal anemia is a major cause for abruptio placenta
C Fetus is not usually affected
D Associated with antecedent hypertension
ANSWER:D
1185 Regarding Placenta previa:
A Is diagnosed when the placenta occupies the funds and start to bleed
B Recognized to be complicated by postpartum hemorrhage
C The fetal heart rate is usually abnormal.
D Less common in patients with repeat caesarian section
ANSWER:B
1186 In placenta previa: Which is true?
A It is common primigravida
B May cause abnormal lie
C Causes recurrent painful bleeding

D All patients should be induced with prostaglandin pessaries
ANSWER:B
1187 Antepartum hemorrhage: Which is true?
A Is any bleeding from the genital tract during any stage of pregnancy
B Requires assessment by vaginal examination
C May be caused by cervical carcinoma
D Is always painless
ANSWER:C
1188
A pregnant woman presents with a placenta praevia of a major defect and fetus is malformed. Which of the
following will be the best management?
A Caesarian section
B Oxytocin drip
C Rupture of membranes
D Induce with PG E2
ANSWER:A
1189 Abruptio placenta:
A Is defined as premature separation of low lying placenta.
B There is no increase risk of recurrence.
C The etiology of placental abruption is usually known.
D The most predisposing condition is chronic maternal Hypertension.
ANSWER:D
1190 Management of Placental Abruption includes all of the following EXCEPT:
A Coagulation studies.
B Expectant management in cases of IUFD.
C Augmentation of labor.
D Artificial rupture of Amniotic membrane.
ANSWER:B
1191 The most common risk factors for placental abruption:
A Diabetes.
B Increased maternal age, Multi parity, hypertension & cigarette smoking.
C Intrauterine growth retardation.
D Rh isoimmunization.
ANSWER:B

1192 The following is a contraindication for the use of amnio-hook:
A Plcenta previa
B Abruptio placenta
C Breech presentation
D IUGR
ANSWER:A
1193 Regarding Abruptio placenta:
A Is defined as premature separation of low lying placenta
B There is no increase risk of recurrence
C The etiology of placental abruption is usually known
D Chronic maternal hypertension is a known cause
ANSWER:D
1194 Management of placental abruption includes all of the following EXCEPT:
A Coagulation studies
B Tocolytic drugs if the baby is premature
C Augmentation of labor
D Artificial rupture of amniotic membrane
ANSWER:B
1195 The followings are causes of Antepartum hemorrhage EXCEPT:
A Abruptio placenta.
B Placenta brevia.
C Cervical polyp.
D Rhisoimmunization.
ANSWER:D
1196
Extensive bleeding into the myometrium & beneath the uterine serosa in severe cases of abruption placenta may
result in :
A A couvelaire uterus.
B Active uterus
C Placental perfusion
D Normal fetal heart rate
ANSWER:A
1197 Which of the following is NOT a complication of abruption placenta ?
A Postpartum hemorrhage

B Consumptive hemorrhage
C Fetal demise
D Subsequent ectopic
ANSWER:D
1198 One of the following contraindications for the use of amnio-hook :
A Placental previa
B Abruptio placenta
C Breech presentation
D IUGR
ANSWER:A
1199 Which of the following signs is most useful in predicting the absence of placental abruption following trauma :
A Absence of uterine contraction
B Absence of vaginal bleeding
C Presence of normal fetal heart tones
D Absence of tense, painful uterus
ANSWER:D
1200
24 -year- old patient ( G2 P1 + 0 ). At 34 weeks of gestation presented to emergency with vaginal bleeding. Which
one of the following is NOT TRUE ?
A Admit the patient
B Resuscitate the patient
C Do digital examination immediately
D Cross-match blood
ANSWER:C
1201 In the management of placenta previa centralis :
A Once diagnosis is made, the treatment should urgent caesarean section
B Patient may stay at home if she is living near the hospital
C Vaginal examination should be done carefully to confirm diagnosis
D If the pregnancy has advanced to 37 weeks, it is usually best to perform C/S
ANSWER:D
1202 The condition of placental abruption is associated with :
A External cephalic version
B Nulliparus women, among whom it is more common
C Direct trauma which may be the main cause

D A clinical diagnosis
ANSWER:D
1203
An 18 year old woman is noted to have a marginal placenta previa on an US at 24 weeks gestation. Which of the
following is the most appropriate management?
A Schedule cesarean delivery at 38 weeks.
B Schedule an amniocentesis at 36 weeks & delivery by C-section if the fetal lung is mature.
C Reassess placental position at 33-34 weeks.
D Recommend termination of pregnancy.
ANSWER:C
1204 The following may be signs of abruptio placenta, EXCEPT :
A Vaginal bleeding.
B Absence of uterine contractions.
C Blood stained amniotic fluid.
D Abnormal fetal heart rate.
ANSWER:B
1205 In Placenta previa, all help in the diagnosis, EXCEPT:
A Constant lower abdominal pain
B Mal presentation
C Painless vaginal bleeding
D US
ANSWER:A
1206
33 year female at 37 weeks gestation confirmed by early sonography presents with moderate severe vaginal
bleeding, she is noted to have placenta previa, which of the following is the best management for her :
A Induction of labor
B C-Section.
C Expectant management
D Artificial rupture of membrane
ANSWER:B
1207 Routine pelvic examination is contraindicated in which of the following situations during pregnancy:
A Carcinoma of the cervix.
B Gonorrhea.
C Prolapsed cord.
D Placenta previa.

ANSWER:D
1208 Which of the following is NOT a complication of abruptio placenta:
A Postpartum hemorrhage.
B Consumptive hemorrhage.
C Fetal demise.
D Subsequent ectopic.
ANSWER:D
1209 Placenta previa is more likely to be found in a pregnancy associated with:
A Multiple pregnancy.
B Previous manual removal of placenta.
C Pyometra.
D Previous C-section.
ANSWER:D
1210 In placenta previa:
A Common in primigravida.
B Presents with vaginal bleeding with abdominal pain.
C Hx. Of repeated C-section is a risk factor.
D Characterized by bleeding at 10weeks gestation.
ANSWER:C
1211 Lady with infertility with bilateral tubal block at cornua. Best method ofmanagement is :
A Laparoscopy & Hysteroscopy
B Hydrotubation
C IVF
D Tuboplasty
ANSWER:A
1212 Women with postmenopausal bleeding need endometrial sampling if endometrial onUS is thicker than
A 1mm
B 2mm
C 5mm
D 8mm
ANSWER:C
1213 Which of the following change in puberty is influenced by the estrogen:
A Growth of the acinar buds of the breast

B Epiphyseal fusion
C Proliferatve phase
D All of the above
ANSWER:D
1214 Sub urethral diverticula may occur as a sequelae to infection of:
A Bartholin’s gland
B Skene’s gland
C Clitoral gland
D Vulvovaginal gland
ANSWER:B
1215 Large amount of alkaline phosphatase may be demonstrated in the endometrium of : a. Decidua
A Secretory phase
B Proliferative phase
C All of the above
D None of the above
ANSWER:C
1216 The function of round ligament is :
A Vestigial with no apparent function
B To prevent retrodisplacement of the uterus
C To prevent uterine prolapse
D To provide nerve supply of the upper vagina
ANSWER:B
1217 The definitive epithelium of vagina is derived from :
A Wolfian duct
B Mullerian duct
C Urogenital epithelium
D Coelomic epithelium
ANSWER:C
1218 Causes of post partum amenorrhoea may be :
A Anorexia nervosa
B Cervical atresia
C Chlorpromazaine therapy
D Any of the above

ANSWER:D
1219 The cyclic production of pituitary hormones is dependant upon:
A Normal menstruation
B An intact pituitary- portal system
C An adult anterior pituitary gland
D All of the above
ANSWER:B
1220 The clots passed with menorrhagia perhaps indicate
A No endometrial regeneration
B No terminal arteriolar spasm
C Large amount of bleeding
D All of the above
ANSWER:C
1221 Monilial vagintis occurs frquently during pregnancy because :
A Glycosuria is commoner
B The vagina contains more glycogen
C Higher vaginal acidity suppresses other organisms
D All of the above
ANSWER:D
1222 Which of the following statements is incorrect regarding levonorgestrel releasing intrauterine system:
A There is increased incidence of menorrhagia
B This system can be used as hormone replacement therapy
C This method is useful for the treatment of endometerial hyperplasia
D Irregular uterine bleeding can be problem initially
ANSWER:A
1223 Myxoma peritonei may occur as a consequence of rupture of which ovarian cyst ? a. Dermoid
A Struma ovarii
B Serous cystadenoma
C Mucinous cystadenoma
D Cystadenofibroma
ANSWER:D
1224 Subnuclear vaculoes in the endometrial mucosa are evidence of activity of: a. Cholesterol
A Progesterone

B Pregnendiol
C Androstenendione
D Oestrogen
ANSWER:B
1225 Clinical findings of PCOD include all except :
A Obesity
B Olgomenorrhoea
C Infertility
D Tall stature
ANSWER:D
1226 Non-neoplastic ovarian cysts include all of the following except: a. follicular cyst
A theca lutein cyst
B dermoid cyst
C corpus luteum cyst
D endmetroid cyst
ANSWER:C
1227 Which of the following ovarian tumor is most prone to undergo torsion during pregnancy?
A Serous cystadenoma
B Mucinous cystadenoma
C Dermoid cyst
D Theca lutein cyst
ANSWER:C
1228 Magnesium sulphate toxicity include all EXCEPT:
A CNS depression
B This drug acts only on motor end plate
C Respiratory depression
D muscle relaxant
ANSWER:B
1229 The uterus is held in anteflexed position by :
A The ventral pull of round ligament
B The dorsal pull of uterosacral ligaments
C Its weight
D All of the above

ANSWER:D
1230 What are the signs of ovulation on Ultrasonography :
A Irregular follicle wall
B Collapse of follicle
C Fluid in cul de sac
D All of the above
ANSWER:D
1231 Perforation of the uterus while doing endometrial biopsy in non pregnant uterus, needs
A Laparoscopy
B Observation
C Immediate laparotomy
D Hysterectomy
ANSWER:A
1232 Diagnosis of stress incontinence coded by which of the following before taking the patient for surgery
A History
B Subjective demonstration of stress incontinence
C Objective demonstration of stress incontinence
D Urodynamic studies
ANSWER:D
1233 Bartholin’s gland duct opens in.....
A Upper third of labia majora
B Middle third of labia majora
C Upper third of labia minora
D Middle third of labia minora
ANSWER:D
1234 Female patient with endometrial hyperplasia could be all of these except:
A thecoma
B fibroma
C Brenner tumor
D follicular cyst .
ANSWER:B
1235 Endometroid cyst, on examination:
A adenexal tenderness

B cyst felt in thin people
C cyst fixed and tender
D All of the above
ANSWER:D
1236 Ovarian precursors of oestradiol include :
A Oestrone
B Androstenedione
C Testosterone
D All of the above
ANSWER:D
1237 Considering epithelial neoplasm of the ovaries all true except :
A the commonest
B mucinous cystadenoma lined by tubal epithelium
C Brenner tumor lined by urinary tract epithelium
D embryologically arise from wolffian epithelium .
ANSWER:B
1238 The Commonest ovarian neoplasm complicated with torsion during pregnancy:
A fibroma
B teratoma
C simple serous cyst
D thecoma .
ANSWER:B
1239 Female patient with acute abdomen , CBC normal , B-HCG negative , No vaginal bleeding , Mostly is :
A hemorrhagic teratoma
B disturbed ectopic pregnancy
C appendicitis
D Peritonitis
ANSWER:A
1240 Considering mucinous cystadenoma :
A the commonest neoplasm
B usually bilateral
C sometimes fill the entire abdominal cavity
D lined by tubal epithelium .

ANSWER:C
1241 Considering Brenner tumor all true except :
A potential malignant is common
B histologically has epithelial nests and coffe bean nuclei
C vaginal bleeding reported with it
D usually in childbearing women
ANSWER:D
1242 Considering malignant ovarian neoplasm histologically may be all except :
A epithelial tumors
B germ cells tumor
C cystic and solid tumors
D sex cord tumors .
ANSWER:C
1243 For endometrial cyst all true except :
A choclate cyst on TVS
B laparoscope is indicated
C C125 is a specific test
D associated with dysmenoorrhoea .
ANSWER:C
1244
Female patient with history of induction of ovulation present with tender lower abdominal pain and discomfort ,
TVS show cyst , Next step is :
A assurance sending home
B hold ovarian stimulatin drug
C laparotomy
D None of the above
ANSWER:B
1245 Considering endometroid cyst :
A not uncommon
B due to menstrual reaction
C torsion is common
D A & B
ANSWER:D
1246 Considering endometrial cyst ttt all true except :

A GNRH is of benefit
B laparosope idicated in small cyst
C laparotomy is preferred
D recurrence is not common
ANSWER:D
1247 Considering the follicular cyst it is rarely associated with :
A endometrial hyperplasia
B acute abdomen
C polycystic ovary
D On PV in obese patient it may rupture .
ANSWER:B
1248
Female patient with history of endometriosis , menstrual disorders complaining from pain on right iliac fossa , on
examination there was tenderness on right iliac fossa with no rebound pain no rigidity , on CBC it was normal ,
most likely :
A peritonitis
B appendicitis
C follicular cyst
D None of the above
ANSWER:C
1249 Considering a case of follicular cyst it need all of following except :
A assurance follow up
B OCP
C usually surgical removal
D repeated US
ANSWER:C
1250 Considering the follicular cyst all of following is true except :
A associated with metropathia hemorrhagica
B OCP indicated in ttt
C the second common functional cyst
D TAS is the gold standard diagnostic method
ANSWER:C
1251 Considering the endometroid cyst :
A associated with dysmenorrhoea

B associated with pelvic pain
C associatd with pelvic endometriosis
D All of the above
ANSWER:D
1252
Female patient with history of hydatiform and complaining of lower abdominal pain , on examination there was
tenderness on palpation and the lab result show high level of HCG , most likely to be :
A follicular cyst
B theca lutein cyst
C corpus luteum cyst
D None of the above
ANSWER:B
1253
A 29-year-old G4P4 is found to have an abnormal smear signed out as atypical glandular cells, favouring
neoplasia. She undergoes a colposcopy with cervical biopsies. One of the ectocervical biopsies demonstrated
adenocarcinima in the situ. The most appropriate next step is:
A Vaginal hysterectomy
B Radical hysterectomy/Radiotherapy
C Cold-knife conization of the cervix
D Loop excision of the cervical tranformation zone
ANSWER:C
1254 The severity of CIN is graded
A 03-Jan
B 1a-4a
C I-III+ I-IV
D A-C
ANSWER:A
1255
A 42-year-old G4P4 has had postcoital bleeding for the past four months. She has not had a Pap test since the
delivery of her last child 7 years ago. Speculum examination shows a vaginal discharge and a 1 cm exophytic
lesion of the posterior cervical lip. The most appropriate next step is:
A Perform a Pap smear
B Perform a cold-knife conization
C Give the patient a course of intravaginal Metronidazole gel followed by re-examination in 6 weeks
D Perform a punch biopsy of the lesion
ANSWER:D

1256 The cysts in Polycystic Ovarian syndrome are formed by:
A Failure of atretic follicles to undergo apoptosis
B Oocyte proliferation
C Multiple corpus lutea
D Cystic degeneration of ovarian cortex
ANSWER:A
1257 Functional ovarian cysts include:
A Follicular cysts.
B Endometriomas.
C Dermoid cysts.
D fibromas.
ANSWER:A
1258 llowing changes occur in urinary system during normal pregnancy:
A Increase in renal blood flow
B Increase in glomerular filtration rate
C Increase in capacity of kidney pelvis
D All of the above
ANSWER:D
1259 chanism of labor in abortion stick ( use of stick to induce abortion)is due to
A Necrosis of uterine endometrium and stimulation of uterine contraction
B Oxytocin present in the stick
C Prostaglandins present in the stick
D All of the above
ANSWER:A
1260
woman delivers a 9 lb baby with midline episiotomy & develops a 3rd degree tear. Inspection shows that the
following structures are intact.
A Anal sphincter
B Perineal body
C Rectal mucosa
D Perineal muscles
ANSWER:C
1261 Engagement is said to occur when.......
A The fetal head is within the maternal pelvis

B The biparietal diameter of the fetal head is through the plane of the inlet.
C The presenting part is just above the level of ischial spines.
D The vertex is in transverse position.
ANSWER:B
1262 The following hormone is not produced by the placenta...
A HCG
B HPL
C Prolactin
D Estriol
ANSWER:C
1263 Engagement is said to occur when.......
A The fetal head is within the maternal pelvis
B The biparietal diameter of the fetal head is through the plane of the inlet.
C The presenting part is just above the level of ischial spines.
D The vertex is in transverse position
ANSWER:B
1264 Engaging diameter, in fully extended head :
A Mento occipital
B Submentobregmatic
C Biparietal
D Mentovertica
ANSWER:B
1265
Following a vaginal delivery, a woman develops a fever, lower abdominal pain anduterine tenderness. She is alert,
and her blood pressure and urine output are good. Large gram positive rods suggestive of clostridia are seen in a
smear of cervix. management should include all except :
A Immediate radiographic examination for gas in uterus
B High dose antibiotic therapy
C Hysterectomy
D close observation for renal failure or hemolysis
ANSWER:C
1266
A multiparous woman aged 40 years, presents with menorahagia and progressively increasing dysmenorrhoea.
Most probable diagnosis is:
A Ca Cervix

B Ca Endometrium
C Adenomyosis
D DUB
ANSWER:C
1267 Considering dysgerminoma all true except :
A the commonest germ cell tumor
B usually in young females
C lymphatic spread is so late
D elevate lactic dehydrogenase level .
ANSWER:C
1268 Ordering accord to the commonest cancers in female genital tract the right is :
A cervical , endometrial ,ovarian
B ovarian , cervical , endometrial
C endometrial , cervical , ovarian
D endometrial , ovarian , cervical .
ANSWER:C
1269 Female came to the ER with Bp 80/60 and pulse 125 with history of acute abdomen , next step is
A laparotomy
B iv fluids
C CBC
D PV examination
ANSWER:B
1270
A 40-years-old female with history of fibroid on investigation showed CIN-2 changes. Treatment of choice in
this case is :
A Hysterectomy
B Conization
C Cryotherapy
D Laser ablation
ANSWER:A
1271 Post menopausal bleeding does not occur in....
A Use of combined OCP's
B Atrophic vaginitis
C Endometrial hyperplasia

D CA-Endometrium
ANSWER:A
1272 The most dangerous symptom during pregnancy is:
A PV bleeding
B Ankle swelling
C Hyperemesis
D Cramps
ANSWER:A
1273 The Arius-Stella reaction may be seen with all except :
A Ectopic pregnancy
B Birth control pills
C Abortion
D Trophoblastic disease
ANSWER:B
1274 The passage of decidual cast in cases of ectopic pregnancy usually means :
A Impending tubal rupture
B Reabsorption of embryo
C Pregnancy was intrauterine
D Death of embryo
ANSWER:D
1275 What is the most common side effect with MTX therapy for ectopic pregnancy
A Transient pelvic pain 3 - 7 days after starting treatment
B Stomatitis
C Bone marrow suppression
D Gastritis
ANSWER:B
1276
A primigravida presents to casualty at 32 weeks gestation with acute painabdomen for 2 hours, vaginal bleeding
and decreased fetal movements. She should be managed by;
A Immediate cesarean section
B Immediate induction of labor
C Tocolytic therapy
D Magnesium sulphate therapy
ANSWER:A

1277 Placenta previa, all true except :
A Shock out of proportion of bleeding
B Malpresentation
C Head not engaged
D Painless bleeding
ANSWER:A
1278
A 34wk GA lady presented with vaginal bleeding of an amount more of that ofher normal cycle. O/E uterine
contracts every 4 min, bulged membrane, the cervix is 3 cm dilated, fetus is in a high transverse lie and the
placenta is on the posterior fundus. US showed translucency behind the placenta and the CTG (Cardiotocography)
showed FHR of 170, the best line of management is:
A C/S immediately.
B give oxytocin.
C do rupture of the membrane.
D amniocentisis
ANSWER:A
1279 Most important cause of immediate post partum hemorrhage:
A laceration of cervix
B laceration of vagina
C uterine atony
D placental fragment retention
ANSWER:C
1280 Factors favoring long anterior rotation include all except
A Correction of the deflexion
B Adequate pelvis
C good pelvic floor
D rupture of membranes
ANSWER:D
1281 Shock is out of proportion to the amount of bleeding in :
A 1ry postpartum haemorrhage
B Retained placenta
C Acute puerperal inversion of uterus
D Hypofibrinogenemia
ANSWER:B

1282 Fetal hyperinsulinemia leads to:
A Fetal macrosomia causes difficult vaginal delivery
B Inhibition of pulmonary surfactant causing Intrauterine asphyxia
C Decrease serum K causing respiratory distress syndrome
D Neonatal hypoglycemia with myocardial injury
ANSWER:A
1283 Glycogen is seen in the lumina of endometrial glands :
A During the luteal phase
B During pregnancy only
C During pre and post ovulatory
D During proliferative phase only
E At the time of ovulation only
ANSWER:A
1284 The Wolfian duct in the female :
A Develops into the fallopian tube
B Forms the ovary
C Forms the round ligament
D Regresses and becomes vestigial
E None of the above
ANSWER:D
1285 Oxytocin and vasopressin are transferred from hypothalamus to neurohypophysisthrough:
A Venous channels
B Lymphatics
C Nerve axons
D All of the above
E None of the above
ANSWER:C
1286 The levator ani muscle :
A Is a voluntary muscle
B Is attached laterally to the ”white line of the pelvis ”
C Is composed of pubococcygeus and iliococcygeus muscle
D Contracts to prevent spillage of urine during strain
E All of the above

ANSWER:E
1287
Physical exam reveals the uterus to be about 6 wk size. Vaginal bleeding is scanty with no discernible tissue in the
cervical os. There are no palpable adnexal masses. The uterus is mildly tender. Ultrasonographic exam does not
reveal a gestational sac. Which of the following should be recommended?
A Dilatation & curettage.
B Culdocentesis.
C Observation followed by serial B-HCG determinations.
D Diagnostic laparoscopy.
E Laparotomy
ANSWER:D
1288 The preferred treatment of ruptured tubo-ovarian abscess is :
A Cul-de-sac drainage
B Removal of uterus , tubes and involved ovary
C Removal of uterus , tubes and ovaries
D Removal of ruptured tube and ovary
E Removal of adenexae and drainage
ANSWER:C
1289 The cysts of Stein -Leventhal ovary or PCOD are of which kind?
A Lutein
B Germinal inclusion
C Follicular
D Theca lutein
E Endometrial
ANSWER:C
1290 Pyogenic infections of genital tract usually spread via :
A Mucous membrane
B Veins
C Lymphatics
D Fistulous tracts
E B & C
ANSWER:E
1291 Presence of pyometra in a post menopausal females strongly suggests:
A Diabetes mellitus

B Degenerating myoma
C Senile endometritis
D Malignancy
E Sexual promiscuity
ANSWER:D
1292 The most common symptom associated with adenomyosis is :
A Infertility
B Menorrhagia
C Haematometra
D Dyspareunia
E Metrorrhagia
ANSWER:B
1293 Adenomyosis is often associated with all of the following except :
A Endometrial hyperplasia
B Myoma
C Endometriosis
D Mymetrial hypertrophy
E Subinvolution of uterus
ANSWER:E
1294 Interstitial uterine myomas most often cause menorrhagia due to :
A Secondary degeneration
B Rupture into endometrial cavity
C Pressure necrosis
D Inhibition of uterine contractility
E Prolapse
ANSWER:D
1295
There is a 5% incidence of primary extrauterine malignancy associated with endometrial cancer, the most frequent
site for such is :
A Stomach
B lung
C Breast
D Bone
E Spleen

ANSWER:C
1296 The cause of virilizing adrenal hyperplasia is :
A Defect in cortisol synthesis
B defect in ACTH synthesis
C Defect in testosterone synthesis
D All of the above
E None of the above
ANSWER:A
1297 Anterior pituitary function may be blocked by:
A Blood levels of steroids
B Emotional factors
C Sensory stimuli
D All of the above
E None of the above
ANSWER:D
1298 Common ovulation induced drugs are
A Clomiphene citrate
B Tamoxifen or the newer letrozole
C Gonadotrophins
D GnRh analogue down regulation protocols
E All of the above
ANSWER:E
1299 The commonest cause of death in cancer cervix is :
A Infection
B Uraemia
C Haemorrhage
D Cachexia
E Distant metastasis
ANSWER:B
1300 Failure to find sperm in postcoital examination may be due to :
A Excessive oestrogen effect on cervical mucous
B Excessive vaginal lactic acid
C Oligospermia

D All of the above
E None of the above
ANSWER:C
1301 The differential diagnosis of vaginal cysts include :
A Cystocele
B Urethral diverticulum
C Urethrocoele
D All of the above
E None of the above
ANSWER:D
1302 Factors in cervical cancer development EXCEPT:
A HIV infection
B Chlamydia infection
C Breast cancer
D Smoking
E Immunosuppression
ANSWER:C
1303 A synthetic progestin. What is the most likely explanation for the contraceptive action of this drug?
A Replacement of the LH surge by an FSH surge.
B Abolition of the LH surge
C Enhanced positive feedback of the hypothalamic-pituitary-gonadal axis.
D Increased conversion of testosterone to estradiol.
E Inadequate decidualization of the uterus.
ANSWER:E
1304
All of the following mechanisms might account for a reduced risk of upper genital tract infection in users of
progestin releasing IUDs, except:
A Reduced retrograde menstruation
B Decreased ovulation
C Thickened cervical mucus
D Decidual changes in the endometrium
E All of the above
ANSWER:E
1305 In DUB all are right except,:

A may be associated with hypothyroidism
B may be associated with post-menopausal bleeding
C may be associated with functional ovarian cysts
D may present as menorrhagia
E may be present as metropathia hemorrhagica
ANSWER:B
1306 Metabolic causes of anovulatory DUB include all except :
A Hypothyroidism
B Halban’s syndrome
C Cushing’s syndrome
D Hyperthyroidism
E diabetes mellitus
ANSWER:B
1307 The primary drainage of the lower vagina is to :
A external iliac nodes
B Sacral nodes
C Femoral nodes
D superficial inguinal nodes
E internal iliac nodes
ANSWER:D
1308 The most common causative organism in acute bartholinitis is
A Staphylococcus
B Streptococcus
C Colon bacillus
D Gonococcus
E Trichomonas
ANSWER:D
1309 The uterine artery supplies the
A Vagina
B Lower cevix
C Ovary
D All of the above
E None of the above

ANSWER:D
1310 Common accompaniments salpingitis are :
A Pelvic mass
B Bleeding
C Pain
D All of the above
E None of the above
ANSWER:C
1311 he ovary of new born may contain :
A Small folicular cysts
B Corpora lutea
C Lutenized grnulosa cells
D All of the above
E None of the above
ANSWER:E
1312 Cervical amputation :
A Is followed frequently by abortion
B Is associated with high incidence of post operative sterility
C Is not frequently followed by cervical dystocia in patients who become pregnant
D All of the above
E None of the above
ANSWER:D
1313 The effect of ovarian steroid on anterior pituitary is
A Direct stimulation
B Direct inhibition
C Mediated via hypothalamus
D Unknown
E Direct stimulation and inhibition
ANSWER:C
1314 The most common symptom of adenomyosis is :
A Dysmenorrhoea
B Menorrhagia
C Pain

D Fever
E None of the above
ANSWER:B
1315 The commonest cause of stress incontinence is
A Constipation
B Raised intra abdominal pressure
C Congenital weakness of sphincter
D Childbirth trauma
E Estrogen deficiency
ANSWER:D
1316 Least common type of uterine anomaly in patients with recurrent pregnancy loss :
A Unicornuate
B Arcuate
C Septate
D Bicornuate
E Didelphys
ANSWER:A
1317 The following are the factors associated with CIN EXCEPT
A Onset of coitus at early stage
B Multiple sexual partners
C Lower socioeconomic status
D Nulliparity
E H/o veneral disease
ANSWER:D
1318 Best treatment for severe stress incontinence without prolapse is
A Pelvic floor exercise
B Kelly's repair
C Burch colposuspension
D MMK operation
E Urethral collagen implant
ANSWER:B

1319
A 19-year-old female comes to the physician because of left lower quadrant pain for 2 months. She states that she
first noticed the pain 2 months ago but now it seems to be growing worse. She has had no changes in bowel or
bladder function. She has no fevers or chills and no nausea, vomiting, or diarrhea. The pain is intermittent and
sometimes feels like a dull pressure. Pelvic examination is significant for a left adnexal mass that is mildly tender.
Urine hCG is negative. Pelvic ultrasound shows a 6 cm complex left adnexal mass with features consistent with a
benign cystic teratoma (dermoid). Which of the following is the most appropriate next step in management?
A Repeat pelvic examination in 1 year
B Repeat pelvic ultrasound in 6 weeks
C Prescribe the oral contraceptive pill
D Perform hysteroscopy
E perform laparotomy
ANSWER:E
1320
A 54-year-old woman comes to the physician for an annual examination. She has no complaints. For the past year,
she has been taking tamoxifen for the prevention of breast cancer. She was started on this drug after her physician
determined her to be at high risk on the basis of her strong family history, nulliparity, and early age at menarche.
She takes no other medications. Examination is within normal limits. Which of the following is this patient most
likely to develop while taking tamoxifen?
A Breast cancer
B Elevated LDL cholesterol
C Endometrial changes
D Myocardial infarction
E Osteoporosis
ANSWER:C
1321
A 62-year-old woman comes to the physician because of bleeding from the vagina. She states that her last
menstrual period came 11 years ago and that she has had no bleeding since that time. She has hypertension and
type 2 diabetes mellitus. Examination shows a mildly obese woman in no apparent distress. Pelvic examination is
unremarkable. An endometrial biopsy is performed that shows grade I endometrial adenocarcinoma. Which of the
following is the most appropriate next step in management?
A Chemotherapy
B Cone biopsy
C Dilation and curettage
D Hysteroscopy

E Hysterectomy
ANSWER:E
1322 The following about human papilloma virus (HPV) infection are correct EXCEPT:
A It is the most common viral STDs.
B It may lead CIN and cervical cancer.
C It is due to RNA virus.
D Infection may be warty or flat condyloma.
E Infection is usually associated with others STDs.
ANSWER:C
1323 The lymphatic drainage of the cervix is to the following lymph nodes EXCEPT:
A The femoral lymph nodes.
B The internal iliac lymph nodes.
C The para-cervical lymph nodes.
D The pre-sacral lymh nodes.
E The Obturator lymph nodes
ANSWER:A
1324 The commonest secondary change in uterine fibroids is:
A Fatty degeneration
B Myxomatous degeneration.
C Hyaline degeneration
D Cystic degeneration
E Calcification
ANSWER:C
1325 The following is correct about the ovarian ligaments:
A Contain ureters.
B Contain ovarian arteries.
C Are attached laterally to pelvic wall.
D Lie anterior to the broad ligament.
E Are homologous to part of the gubernaculums testis in the male
ANSWER:B
1326 Cervical polyps
A causes spontaneous abortion
B are cause of antepartum hge

C cause watery vaginal discharge
D are covered by squamous epithelium
E cause intermenstrual bleeding
ANSWER:E
1327 Involves pelvic LN clearance, hysterectomy, removal of the parametrium and the upper third of the vagina.
A Wartman’s hysterectomy
B Wertheim’s hysterectomy
C Wertheims Trachelectomy
D Radical trachelectomy
E Trachelems hysterectomy
ANSWER:B
1328 5 year survival for someone with stage 3-4 cervical carcinoma
A 10-30%!!!
B 80-95%
C 2-10%
D 65-80%
E 45-60%
ANSWER:A
1329
Cervical carcinoma spread and staging: Microinvasion of the basement membrane, <7mm across, with no
lymph/vascular space invasion
A Stage 1b
B Stage 3
C Stage 4
D Stage 1a
E Stage 2a
ANSWER:D
1330 Acetic acid turns a portion of the cervix _____ in a patient with a CIN
A Blue
B Brown
C Orange
D White
E Green
ANSWER:E

1331 Typical cells are found only in the lower third of the epithelium
A CIN III
B CIN I
C CIN V
D CIN IV
E CIN II
ANSWER:E
1332 The area where cervical carcinoma usually originates
A Neoplastic zone
B Metaplastic field
C Retrograde area
D Transformation zone
E Transition field
ANSWER:D
1333 Cervical carcinoma is most common between the ages of
A 45-55
B 16-18
C 18-22
D 35-45
E 25-35
ANSWER:A
1334 Cervical carcinoma characteristically spreads in the
A Tissue
B Lymph
C Bone
D Blood
E Mucus
ANSWER:B
1335 Which of the following is thought to be protective against CIN?
A HIV
B Oral contraceptive usage
C Long term sexual abstinence
D Smoking

E Long term steroid use
ANSWER:C
1336 Cervical carcinoma that can be treated with cone biopsy
A Stage 5
B Stage 1a
C Stage 3
D Stage 2a
E Stage 4a
ANSWER:B
1337 The presence of atypical cells within the squamous epithelium
A Cervical dyskaryosis
B Nabothian follicles
C Dysplasic dyskaryosis
D Cervical intraepithelial neoplasia
E Cervicitis
ANSWER:D
1338 5 year survival for someone with stage 1a cervical carcinoma
A 95%
B 10%
C 30%
D 80%
E 60%
ANSWER:A
1339
Cervical carcinoma spread and staging: Invasion of the lower vagina or pelvic wall, or causing ureteric
obstruction
A Stage 1a
B Stage 4
C Stage 3
D Stage 2a
E Stage 1b
ANSWER:C
1340 HPV types _________ are the most significant and account for 70% of all cervical cancers
A 45 and 46

B 31 and 33
C 14 and 16
D 16 and 18
E 12 and 14
ANSWER:D
1341 Ovarian tumors which may produce chorionic gonadotrophins include :
A Dysgerminoma
B Teratoma
C Choriocarcinoma
D All of the above
E None of the above
ANSWER:D
1342 Pathology of endometriosis may be explained by :
A coelemic metaplasia
B endometrial hyperplasia
C retrograde menstruation
D intraperitoneal immunologic deficit
E lymphatic diffusion
ANSWER:C
1343 An ‘ in situ ‘ stage has not been officially recognized in which of the following :
A Ovarian carcinoma
B Endometrial carcinoma
C Cervical carcinoma
D Vulvar carcinoma
E Vaginal carcinoma
ANSWER:A
1344 The gastrointestinal primary of a Krukenberg tumour of the ovary is most often found in the :
A Gall bladder
B Rectum
C Pylorus
D Colon
E Small intestine
ANSWER:C

1345 In contrast to a malignant ovarian tumor, a benign tumor has which of the following gross features?
A Excrescences on the surface.
B Peritoneal implants.
C Intra-cystic papillations.
D Free mobility.
E Capsule rupture.
ANSWER:D
1346
A uni-locular ovarian cyst measuring 4.4 cm by 4.9 cm found on routine ultrasonograrhy during the 8th week of
gestation . best management for this case is
A observation and repeated ultrasonography
B laparoscoic aspiration of the cyst
C immediate laparotomy and cystectomy
D immediate laparotomy and ovariectomy
E laparotomy and cystectomy postponed to 14 weeks
ANSWER:A
1347 Germ cell tumours include all the following except
A choriocarcinoma
B gonadoblastoma
C endodermal sinus tumour
D begnin cystic teratoma
E solid teratoma
ANSWER:B
1348 Which is the major cause of cancer death in women?
A Breast cancer
B Cervical cancer
C Endometrial cancer
D Lung cancer
E Ovarian cancer
ANSWER:A
1349 Pelvic ultrasound is helpful in the diagnosis of:
A Endometrial carcinoma
B Asherman’s syndrome
C Ascites

D Ovulation detection
E Endometriosis
ANSWER:D
1350 A Krukenberg tumour is an ovarian neoplasm which :
A Is primary in the ovary
B Is associated with hydrothorax
C Is secondary to any GIT cancer
D Shows characteristic mucoid epithelial change
E None of the above
ANSWER:D
1351 CA-125 is ?
A A mucin-type glycoprtein
B A ganglioside
C A tumor-specific transplantation antigen
D Useful for ovarian cancer screening in the general patient population
E An antigen which is commonly expressed by mucinous ovarian carcinomas
ANSWER:A
1352
A 63 old lady presents with abdominal mass & weight loss , was diagnosed as having an ovarian tumor , the most
common ovarian tumour in this woman would be...:
A epithelial tumour
B germ cell tumour
C stromal tumour
D sex cord tumour
E trophoblastic tumour
ANSWER:A
1353
A large cystic tumour is detected in a woman in routine antenatal examination. The most common complication
she can encounter?
A Torsion
B rupture
C hemorrhage
D infection
E degeneration
ANSWER:A

1354
A 18-year-old woman comes to the physician for an annual examination. She has no complaints. She has been
sexually active for the past 2 years. She uses the oral contraceptive pill for contraception. She has depression for
which she takes fluoxetine. She takes no other medications and has no allergies to medications. Her family history
is negative for cancer and cardiac disease. Examination is unremarkable. Which of the following screening tests
should this patient most likely have?
A Colonoscopy
B Mammogram
C Pap smear
D Pelvic ultrasound
E Sigmoidoscopy
ANSWER:C
1355 Hilus or Leydig cell tumour may be associated with :
A Reinke crystals
B Oestrogen effect on endometrium
C Clinical virilism
D All of the above
E None of the above
ANSWER:D
1356
A 47-year-old woman has noted a pressure sensation, but no pain, in her pelvic region for the past 5 months. On
physical examination there is a right adnexal mass. An ultrasound scan shows a 10 cm fluid-filled cystic mass in
the right ovary. A fine needle aspirate of the mass is performed and cytologic examination of clear fluid aspirated
from the mass reveals clusters of malignant epithelial cells surrounding psammoma bodies. Which of the following
neoplasms is she most likely to have?
A Endometrial adenocarcinoma
B Ovarian serous cystadenocarcinoma
C Mesothelioma
D Ovarian mature cystic teratoma
E Adenocarcinoma of fallopian tube
ANSWER:B
1357 Ovarian masses:
A Are malignant in presence of ascites
B Include benign teratomas
C Of germ cell origin may secrete hormones

D May be confused with develpomental abnormalities of renal tract
E If malignant can be reliably staged pre-operatively
ANSWER:B
1358
A 4-year-old girl is noted to have breast enlargement and vaginal bleeding. On physical examination, she is
noted to have a 9-cm pelvic mass. Which of the following is the most likely etiology?
A Cystic teratoma
B Dysgerminoma
C Endodermal sinus tumor
D Granulosa cell tumor
E Mucinous tumor
ANSWER:D
1359 Current modes of investigation for infertility to check functioning of tubes are all of the following execpt:
A Air insufflation
B Sonosalpingography
C Hysterrosalpingography
D Laparoscopic chromotubation
E All of the above
ANSWER:E
1360 Pap smear
A the next step in dysplastic smear is colposcopy
B is simple but inaccurate
C should be carried out every 5 years
D has no role in screening of assymptomatic women
E All of the above
ANSWER:A
1361 Dysfunctional Uterine Bleeding (DUB) is defined as abnormal uterine bleeding ?
A Secondary to hormonal dysfunction
B Caused by cancer
C In a patient with von Willebrand's disease
D With no organic cause
E Caused by an endometrial polyp
ANSWER:D
1362 Abnormal Uterine bleeding (AUB) is defined by all of the following except ?

A Excessive Blood loss (>80 ml) during menses
B Menstrual length less than 7 days
C An interval of less than 21 days between the starts of successive menses
D Irregular bleeding episodes between menses
E Extended (>35 days) intervals between menses
ANSWER:B
1363 Which is the least frequent site of an ectopic pregnancy?
A Fallopian tube
B Cervix
C Ovary
D Abdominal cavity
E Between the leaves of broad ligament
ANSWER:D
1364
Perforation tends to occur earliest when an ectopic pregnancy is located in whichportion of fallopian tube ? a.
Isthmic
A Isthmic
B Interstitial
C Ampullary
D Infundibular
E No difference
ANSWER:A
1365 Which of the following does not occur in post partum pituitary necrosis :
A signs of hypoglycaemia
B Asthenia
C Amenorrhoea
D Galactorrhoea
E Decreased libido
ANSWER:C
1366 Exposure of female fetus to androgens may arrest differentiation of :
A Mullerian duct
B Ovary
C Urogenital sinus
D All of the above

E None of the above
ANSWER:C
1367 The gold standard in diagnosing ectopic pregnancy
A Laparoscopy
B Culdocenteris
C Beta HCG
D US
E Progesterone
ANSWER:A
1368 Which method of terminating a molar gestation is never indicated
A Suction curettage
B Prostaglandic
C Hypertonicsaline
D Hysterotomy
E Hyterectomy
ANSWER:C
1369 Invasive molar tissue is most commonly found in
A Myometrium
B Vaginal wall
C Ovary
D Liver
E Lungs
ANSWER:A
1370
A 31-year-old, HIV-positive woman, gravida 3, para 2, at 32-weeks' gestationcomes to the physician for a prenatal
visit. Her prenatal course is significant for the fact that she has taken zidovudine throughout the pregnancy.
Otherwise, her prenatal course has been unremarkable. She has no history of mental illness. She states that she has
been weighing the benefits and risks of cesarean delivery in preventing transmission of the virus to her baby. After
much deliberation, she has decided that she does not want a cesarean delivery and would like to attempt a vaginal
delivery. Which of the following is the most appropriate next step in management?
A Contact psychiatry to evaluate the patient
B Contact the hospital lawyers to get a court order for cesarean delivery
C Perform cesarean delivery at 38 weeks

D Perform cesarean delivery once the patient is in labor
E Respect the patient's decision and perform the vaginal delivery
ANSWER:E
1371
A 19-year-old primigravid woman at 42 weeks' gestation comes the labor anddelivery ward for induction of labor.
Her prenatal course was uncomplicated. Examination shows her cervix to be long, thick, closed, and posterior.
The fetal heart rate is in the 140s and reactive. The fetus is vertex on ultrasound. Prostaglandin (PGE2) gel is
placed intravaginally. One hour later, the patient begins having contractions lasting longer than 2 minutes. The
fetal heart rate falls to the 70s. Which of the following is the most appropriate next step in management?
A Administer general anesthesia
B Administer terbutaline
C Perform amnioinfusion
D Start oxytocin
E Perform cesarean delivery
ANSWER:B
1372 Which one of the following is a risk factor for developing DVT?
A Family history of thromboembolic disease.
B Factor V Leiden.
C Antiphospholipid syndrome.
D Sepsis.
E All of the above
ANSWER:E
1373
A 22-year-old woman in labor progresses to 7 cm dilation, and then has nofurther progress. She therefore
undergoes a primary cesarean section. Examination 2 days after the section shows a temperature of 39.1 C (102.4
F), blood pressure of 110/70 mm Hg, pulse of 90/min, and respirations of 14/min. Lungs are clear to auscultation
bilaterally. Her abdomen is moderately tender. The incision is clean, dry, and intact, with no evidence of
erythema. Pelvic examination demonstrates uterine tenderness. Which of the following is the most appropriate
pharmacotherapy?
A Ampicillin
B Ampicillin-gentamicin
C Clindamycin-gentamicin
D Clindamycin-metronidazole
E Metronidazole

ANSWER:D
1374
A 19-year-old nulliparous woman in her 35th week of pregnancy presents withnausea, blurred vision and a weight
gain of 4.5 kg per week. Her blood pressure is 160/110 mmHg. Which of the following tests is the most suitable
for the assessment of fetal status?
A amniocentesis for the measurement of the lecithin/ sphingomyelin (L/S) ratio
B amniocentesis for the measurement of the creatinine level of the amnotic fluid
C sonographiccephalometry
D a non-stress test (NST)
E an oxytocin challenge test (OCT)
ANSWER:D
1375
A 31-year-old woman comes to the physician for follow-up after an abnormalPap test and cervical biopsy. The
patient's Pap test showed a high-grade squamous intraepithelial lesion (HGSIL). This was followed by colposcopy
and biopsy of the cervix. The biopsy specimen also demonstrated HGSIL. The patient was counseled to undergo a
loop electrosurgical excision procedure (LEEP). Which of the following represents the potential long-term
complications from this procedure?
A Abscess and chronic pelvic inflammatory disease
B Cervical incompetence and cervical stenosis
C Constipation and fecal incontinence
D Hernia and intraperitoneal adhesions
E Urinary incontinence and urinary retention
ANSWER:B
1376 Most common indication for C/S :
A malpresentations
B antepartum hge
C prematurity
D previous c/s
E contracted pelvis
ANSWER:D
1377 A woman with a complete mole is most likely to present with which of thesymptoms?
A Vaginal Bleeding
B Excessive uterine size
C Hypermesis
D Prominent theca lutein cysts

E Pre-eclampsia
ANSWER:A
1378 The following are eitiological factors of atonic postpartum hge except :
A prolonged labour
B overdistension of uterus
C full bladder
D cervical lacerations
E accidental he
ANSWER:D
1379 If the foetus is lying accros the uterus, with the head in the flank
A Transverse lie
B Cephalic lie
C Breech lie
D Frank lie
E Oblique lie
ANSWER:A
1380 Refers to the part of the foetus that occupies the lower segment of the uterusor pelvic
A The show
B The version
C The engagement
D The lie
E The presentation
ANSWER:E
1381 Which is the most common cause of abnormal lie?
A Polyhydramnios
B Twin pregnancy
C Uterine deformity
D Pelvic tumour
E Placenta praevia
ANSWER:A
1382 Which of the following statements regarding vaginal breech birth is FALSE?
A Increased risk if footling
B In about 30% there is slow cervical dilatation in the first stage

C CTG is advised
D Pushing is not encouraged until the buttocks are visible
E Epidural analgesia is mandatory
ANSWER:E
1383 Refers to a maneuver which attempts to turn a breech baby to a cephalicpresentation
A VEC
B CEV
C ECR
D EVC
E ECV
ANSWER:E
1384 Refers to the part of the foetus that occupies the lower segment of the uterusor pelvis
A The show
B The version
C The engagement
D The lie
E The presentation
ANSWER:E
1385 After what age gestation would abnormal lie warrant hospital admission
A 37
B 40
C 38
D 39
E 36
ANSWER:A
1386 The most common type of breech
A Starling breech
B Flexed breech
C Explicit breech
D Footling breech
E Extended breech
ANSWER:E
1387 Breech presentations occurs in ___ of term pregnancies

A 1%
B 3%
C 8%
D 4-10%
E 5-6%
ANSWER:B
1388 Flexion of the fetal head occurs when it meets resistance from :
A Pelvic floor
B Cervix
C Pelvic walls
D Any of the above
E None of the above
ANSWER:A
1389 Leopold maneuvers refers to :
A delivery of head
B External version
C Internal version
D Breech extraction
E Examination of abdomen.
ANSWER:E
1390 All of the following can be used in hypertension in Pregnancy except
A Hydralazine
B Labetolol
C Captopril
D Alpha methyl DOP
ANSWER:C
1391 All are complications of illegal /Septic abortion except
A Cerebral Hemorrhage
B DIC
C ARF
D Bacterial Shock
ANSWER:A
1392 Cervical lesion (ectopy):

A It is an ulcer of the cervix.
B Should be treated in pregnant females.
C Pap smear is advisable before management.
D Commonly cause pain, dyspareunia & low back pain.
ANSWER:D
1393 Engagement all true except :
A the biparietal diameter in the pelvic inlet
B grasped by 1st pelvic grip
C at the onset of labor in multiparas
D Preferring an empty bladder.
ANSWER:B
1394 The foetal well-being can be assessed by all of the following, except ?
A non-stress test
B contraction stress test
C ultrasound
D oxytocin sensitivity test!!!
ANSWER:D
1395 Which is contraindicated in trial of labour following Caesarian Section ?
A History of Classical CS
B Breech
C X-ray pelivmetry not available
D No previous vaginal delivery
ANSWER:B
1396
A 20 year old full-term primigravida is brought to the casualty with labour pains for last 24 hours and a hand
prolapse. On examination, she has pulse 96/min, BP 120/80 mm Hg, and mild pallor. The abdominal examination
reveals the uterine height at 32 weeks, the foetus in transverse lie and absent foetal heart sounds. On vaginal
examination, the left arm of the foetus is prolapsed and the foetal ribs are palpable. The pelvis is adequate. What
would be the best management option ?
A External cephalic version
B Decapitation and delivering the baby vaginally
C Internal podalic version
D Lower Segment Caesarean section
ANSWER:B

1397 The indications of an elective caesarean section include all of the following, except ?
A Placenta Praevia
B Cephalopelvic disproportion
C Previous lower segment caesarean section
D Carcinoma Cervix
ANSWER:C
1398 Ectopic pregnancy is differentiated from abortion by the fact that in ectopicpregnancy :
A Pain appears after vaginal bleeding
B There is slight amount of bleeding
C No enlargement of uterus
D Histological examination of products of expulsion shows villi
ANSWER:D
1399
These conditions are always an indication for caesarean section. Which one of these is an absolute indication for
C/S :
A Twin pregnancy
B Breech presentation
C Severe PET
D Major degree placenta previa
ANSWER:D
1400 Indication for cesarean section include all of the following EXCEPT:
A Previous caesarean section
B Failed forceps delivery
C Cervical cerculage.
D Primigravida.
ANSWER:D
1401 Pre-requisite for a forceps delivery include all of the following EXCEPT :
A A complete dilated cervix
B An empty bladder
C Thevertexintheoccipito-anteriorposition
D Ruptured membrane
ANSWER:C
1402 Which of the following neonatal morbidities is not related to forceps delivery?
A Fractured skull

B Sepsis
C Nerve palsies
D Cephalohematoma
ANSWER:B
1403 Kjelland's forceps :
A May be used if the head is not engaged
B Have no cephalic curve
C Have knobs on the shank which point towards the sinciput
D Have a sliding lock in the order to correct asynclitism
ANSWER:D
1404 An absolute indication foe an elective C-section is :
A Previous CS.
B Antepartum hemorrhage.
C Twin pregnancy.
D Placenta previa totalis.
ANSWER:D
1405 Immediate complications of C-section include all the following, EXCEPT :
A Complications of anesthesia.
B Bladder injury.
C Thromboembolism.
D Colon injury.
ANSWER:C
1406 Immediate complication of C/S include all except:
A Bladder injury
B Hemorrhage
C Thromboembolism (also DM is not immediate)
D Complication of anesthesia
ANSWER:C
1407 One of the following is an absolute indication for C-section:
A Grade IV placenta previa.
B Abruptio placenta.
C Cervical cerculage.
D Breech presentation.

ANSWER:A
1408 Pre-requisite for instrumental delivery include all of the following EXCEPT:
A Cervix fully dilated.
B Ruptured membranes.
C Fetal head engaged.
D Fetal head at 2 station.
ANSWER:D
1409 A newborn is noted to have a darkened swelling of the scalp that does not cross the midline. This is most likely a:
A Caput succedaneum.
B Subdural hemorrhage.
C Cephalohematoma.
D Subarachnoid hemorrhage.
ANSWER:C
1410 Which of the following anesthetic technique will produce the greatest uterine relaxation?
A Spinal block.
B Caudal.
C Nitrous oxide.
D Halothane.
ANSWER:D
1411 What is the most common cause of anesthetic death in obstetrics :
A Failed intubation.
B Hemorrhage.
C Stroke.
D Aspiration pneumonitis
ANSWER:D
1412 Epidural nerve block in labor:
A Involves the injection of an anesthetic agent into the subarachnoid space.
B Causes transient hypertension.
C Increases the rate of forceps delivery.
D Increases the length of the 1St stage of labor.
ANSWER:C
1413 Which of the following anesthetic technique will produce the greatest uterine relaxation?

A Spinal block
B Caudal
C Nitrous oxide
D Halothane
ANSWER:D
1414 Most common cause of anesthesia death in pregnancy:
A Aspiration pneumonia
B Medication reaction
C Stroke
D Hemorrhage
ANSWER:A
1415 The following are absolute indication for C-section EXCEPT:
A Face presentation.
B Shoulder presentation.
C Cervical cancer.
D Fibroids in the lower uterine segment.
ANSWER:A
1416 All of these drugs can be used as tocolytic to stop labor, EXCEPT:
A Salbutamol ventolin
B Diazepam (valium)
C Calcium channel blocker
D Indomethacin non steroidal anti inflammatory drugs
ANSWER:B
1417
In cases with premature rupture of membranes, all the following are acceptable in the conservative management
except:
A Frequent vaginal examination to assess cervical dilatation
B serial complete blood count to diagnose rising of WBC
C Close monitoring of maternal vital signs
D Ultrasound to assess fetal weight and amount of liquor
ANSWER:A
1418 Regarding Surfactant:
A Is secreted by type I pneumocytes
B Are glycoprotein.

C After 38 weeks the ratio to sphingomyelins is 2:1.
D Its secretion is suppressed by betamethzone.
ANSWER:C
1419 The following has a recognized relation with spontaneous preterm labor:
A Fetus with anencephaly
B Oligohydromnios
C Maternal hypothyroidism
D Bacterial Vaginosis
ANSWER:D
1420 All the following are possible of premature labor, EXCEPT:
A Multiple pregnancy
B Polyhydramnios
C Bicorrnuate utures
D Anecephaly
ANSWER:D
1421 A positive nitrazine test is:
A Strong evidence of rupture of the membranes.
B Presumptive evidence of intact membranes.
C An evidence of intact membranes.
D Presumptive evidence of intact membranes.
ANSWER:A
1422 The most serious complication of preterm premature rupture of membrane (PPROM) at 28 weeks:
A Fetal compression anomaly.
B Pulmonary hypoplasia.
C Intrauterine infection.
D Limb contraction.
ANSWER:C
1423 The following are obstetrics causes for premature labor EXCEPT :
A Multiple pregnancy
B Multiparity
C Premature preterm rupture of the membrane
D Cervical incompetence
ANSWER:B

1424 In the diagnosis of premature rupture of the membrane, all of the following are true EXCEPT:
A History of fluid loss per vagina
B Visualization of amniotic fluid in the vagina by sterile speculum
C Positive Nitrazine test
D Positive methylene blue test
ANSWER:D
1425 Rupture of membranes is suspected with all of the followings EXCEPT:
A Positive Nitrazine test.
B Contractions seen on the CTG.
C Pooling of amniotic fluid on speculum examination.
D Observing amniotic fluid drain through the cervix during speculum examination.
ANSWER:B
1426 The following are known causes of preterm labor EXCEPT:
A Maternal hypoxia
B Intrauterine fetal death
C Polyhydramnios
D Multiple pregnancy
ANSWER:A
1427 Early signs of premature labor include all of following EXCEPT:
A Increased vaginal discharge
B Increased uterine contraction
C Low back pain
D Cervical dilatation to 4 cm
ANSWER:A
1428 Rupture of membranes is suspected with all the following, EXCEPT :
A +ve nitrazine test.
B +ve fern test.
C Pooling of amniotic fluid on speculum examination.
D Contraction seen on the CTG.
ANSWER:D
1429 In cases of premature rupture of membrane, all the following is acceptable in conservative management, EXCEPT:
A Serial CBC to diagnose rising WBC.

B Closed monitoring of maternal vital signs.
C Frequent vaginal examination to assess cervical dilation.
D US to asses fetal weight and the amount of liquor.
ANSWER:C
1430
In cases with premature rupture of membranes, all the following are acceptable in the conservative management
except:
A Frequent vaginal examination to assess cervical dilatation.
B serial complete blood count to diagnose rising of WBC.
C Close monitoring of maternal vital signs.
D Ultrasound to assess fetal weight and amount of liquor.
ANSWER:A
1431 A premature birth has been defined as:
A Before 37 completed weeks' gestation.
B Prior to the period of viability.
C Weighing less than 1000 g.
D Weighing more than 1000g but less than 2500g.
ANSWER:A
1432 Preterm labor is defined as labor which starts:
A Before 24 weeks of gestation.
B Before 37 completed weeks.
C Before the viability of the fetus.
D When the fetus weighing < 1000 gm.
ANSWER:B
1433 Premature rupture of membrane is most strictly defined as rupture at any time prior to:
A A stage of fetal viability.
B The 2ND stage of labor.
C The 32ND week of gestation.
D The onset of labor.
ANSWER:D
1434 Contraindications to tocolytic therapy includes the following EXCEPT:
A Severe PET.
B Severe antepartum hemorrhage.
C IUGR.

D Preterm breech presentation.
ANSWER:D
1435 Drugs used to inhibit preterm labor include:
A Phenobarbitone.
B Prostaglandins.
C Ca+ channel inhibitors.
D Anticholinergic drugs.
ANSWER:C
1436 Premature rupture of membrane is :
A Rupture of membranes before the onset of labor.
B Rupture of membranes before fetal lung maturity.
C Rupture of membranes before term.
D Rupture of membranes before 28 weeks gestation.
ANSWER:A
1437 All of these drugs can be used as tocolytic to stop labor, EXCEPT:
A Salbutamol ventolin
B Methotrexate
C Calcium channel blocker
D Indomethacin non steroidal anti inflammatory drugs Retodrine (_- agonist)
ANSWER:B
1438 The following are known contraindications to the usage of Tocolysis in pregnancy EXCEPT:
A Cardiac disease.
B Severe hypertension.
C Clinical chorioamnionitis.
D Intrauterine death.
ANSWER:A
1439 Dexamethasone is indicated in which of the following condition :
A Premature labor to prevent neonatal respiratory distress syndrome
B Ectopic pregnancy to enhance fetal lung maturity
C Spontaneous rupture of membrane at 39 week
D At 38 weeks severe abruption placenta
ANSWER:A
1440 Complication of pregnancy that predispose to preterm labor includes all the following EXCEPT :

A polyhydramnios
B Urinary tract infection
C 25 years old primigravida.
D Premature rupture of membranes
ANSWER:C
1441 Which of the following complications is associated with ruptured appendix & peritonitis ?
A Fetal growth restriction
B Oligohydromnios
C Chorioamnionitis
D Preterm birth
ANSWER:D
1442 Inhibition of prostaglandin synthesis are not generally used for tocolysis because they:
A Are effective
B Produce marked hypertension
C May cause premature closure of the fetal ductus arteriosus
D Are too expensive
ANSWER:C
1443
In cases with premature rupture of membranes, all the following are acceptable in the conservative management
except:
A Frequent vaginal examination to assess cervical dilatation.
B serial complete blood count to diagnose rising of WBC.
C Close monitoring of maternal vital signs.
D Ultrasound to assess fetal weight and amount of liquor.
ANSWER:A
1444
Retodrine is a b adrenergic receptor stimulator that is used to arrest preterm labor. Which of the following is a
major maternal risk of its use?
A Hypertension.
B Decreased plasma glucose.
C Decreased serum potassium.
D Cardiac arrhythmias.
ANSWER:D
1445 Prostaglandin synthesis is inhibited by:
A progesterone.

B Indomethacin.
C ACTH.
D Prolactin-inhibiting factor.
ANSWER:B
1446 Side effects of β sympathomimetics include all of the following, EXCEPT:
A Tachycardia
B Pulmonary oedema
C Headache
D Premature closure of ductus arteriosus
ANSWER:D
1447 In prenatal infection
A Rubella can be prevented by administration of rubella vaccine during pregnancy
B Toxoplasma is a virus
C HIV virus infect the baby more readily when delivered vaginally than caesarean section
D Cytomegalovirus causes macrosomic babies
ANSWER:C
1448 Transplacental infection occur with all, EXCEPT:
A Cytomegalovirus.
B Toxoplasma
C Rubella
D Gonorrhea
ANSWER:D
1449 In HIV infection which of the following is CORRECT ?
A It is DNA virus
B Virus can be isolated from saliva
C Breast feeding increase transmission to the baby
D Caesarean section increase transmission of infection to the baby
ANSWER:C
1450 The following may be indicative of chorioamnionitis EXCEPT:
A Maternal pyrexia
B Maternal tachycardia.
C Tender uterus
D Fetal bradycardia

ANSWER:D
1451 Genital tract Candida occurs more frequent in all these patients EXCEPT :
A Diabetic
B On long term antibiotic therapy
C Thyrotoxicosis
D On oral contraceptive pills
ANSWER:C
1452 Regarding Rubella immunization :
A Rubella negative patients should be vaccinated during pregnancy
B Rubella vaccine is a Toxoid
C The majority of pregnant patients are rubella non immune
D Pregnancy should be avoided for 3 months after vaccination
ANSWER:D
1453 Regarding Rubella vaccination, choose the correct answer :
A It's a live attenuated virus.
B Should be given in pregnancy in non-immune mother.
C Should be given to all pregnant mothers in the 1st antenatal visit.
D Pregnancy should be avoided for one year after the vaccination.
ANSWER:A
1454 The following are characteristic findings in neonatal rubella infection, EXCEPT :
A Deafness.
B Spina bifida.
C Congenital heart disease.
D Cataracts.
ANSWER:B
1455 The following maternal infections may cross the placenta:
A Herpes genitals.
B Parvovirus B19.
C Toxoplasmosis.
D Chicken pox.
ANSWER:C
1456 Which of the following is the predominant bacteria in vagina during pregnancy:
A Peptostreptococci.

B Listeria monocytogenes.
C Lactobacilli.
D Streptococcus agalatia.
ANSWER:C
1457 The following can be associated with toxoplasmosis infection during pregnancy EXCEPT:
A Rhinitis
B Brain calcification
C Hepatic Splenomegaly
D Spina bifida
ANSWER:D
1458 All these tests can be useful in management of intrauterine fetal growth restriction (IUGR) EXCEPT:
A Fetal kick chart
B Cardiotocography CTG non stress test
C Chorionic villus sampling
D Biophysical profile
ANSWER:C
1459 The following are possible complications of IUGR EXCEPT :
A Intrauterine death
B Severe hypoxia and fetal distress in labor
C Meconium aspiration
D Postmaturity
ANSWER:D
1460 The definition of intrauterine growth restriction (IUGR):
A Infant with birth weight below 10 th percentile for a given gestation age
B Infant with birth weight below 25th percentile for a given gestation age
C Infant with birth weight of 2.8 kg
D Infant with birth weight below 50th percentile for a given gestation age
ANSWER:A
1461 Maternal causes for intrauterine growth restriction may one of these EXCEPT:
A Hypertensive diseases with pregnancy
B Chronic renal diseases with pregnancy
C Smoking and alcoholism
D Rheumatic mitral stenosis

ANSWER:D
1462 Risk factors for shoulder dystocia include all the followings EXCEPT:
A Maternal obesity.
B Macrosomia.
C Maternal diabetes.
D IUGR.
ANSWER:D
1463 Causes of IUGR include all the followings EXCEPT:
A Constitutional small mother.
B Fetal urinary tract anomalies.
C Premature rupture of membranes.
D Placental insufficiency.
ANSWER:A
1464 The most accurate method for diagnosis of IUFD :
A No fetal movement by the mother
B Decrease in symptoms & signs of pregnancy
C Recurrent bleeding per vagina
D Absence of fetal heart by realtime UUS movement
ANSWER:D
1465 The following are causes for a uterus that is large for gestation during pregnancy, EXCEPT :
A Multiple pregnancy
B IUGR
C Fibroid
D Polyhydramnios
ANSWER:B
1466 Causes of IUGR include all the followings EXCEPT:
A Constitutional small mother.
B Fetal urinary tract anomalies.
C Premature rupture of membranes.
D Placental insufficiency.
ANSWER:A
1467 The most serious maternal complication of IUFD:
A Acute amnionitis.

B Acute psychosis.
C Pelvic thrombophlebitis.
D Hypofibrinogenemia.
ANSWER:D
1468 IUGR may occur in all of the following EXCEPT:
A Pre-eclampsia.
B Congenital anomaly.
C Gestational diabetes mellitus.
D Sickle cell anemia.
ANSWER:C
1469 In cases of IUFD:
A A cause can be indentified in most cases.
B Immediate delivery is indicated.
C More common with good antenatal care.
D Careful examination of the new born is important.
ANSWER:D
1470 If IUGR is encountered:
A Immediate delivery is indicated.
B C-section is the only way in those cases.
C Steroids should not be administrated in causes post maturity.
D Congenital anomalies should be ruled out.
ANSWER:D
1471 The commonest cause of IUFD is :
A True knots in the cord.
B Gestational diabetes mellitus.
C Unexplained.
D Infections.
ANSWER:C
1472 The most common chromosomal abnormality in the abortuses is:
A Blanced translocation.
B Unblanced translocstion.
C Triploidy.
D Trisomy.

ANSWER:D
1473 Which is a contraindication of induction of labor
A History of upper segment caesarian section
B Sever P E T at 36 weeks
C Gestational diabetes on insulin at 39 weeks
D Post term pregnancy
ANSWER:A
1474 Which of the following is an indication for induction of labor:
A Placenta previa.
B Postterm gestation.
C Cord presentation.
D Prior classical C-section.
ANSWER:B
1475 The following are indications for induction of labor EXCEPT :
A Sever pre-eclampsia
B Diabetes mellitus-controlled with high dose of insulin
C Prolonged rupture of membrane without uterine contraction
D History of previous 3 C/S.
ANSWER:D
1476 The most common reason for postdate pregnancy is:
A Inaccurate gestational age.
B Fetal anencephaly.
C Oligohydramnios.
D IUGR.
ANSWER:A
1477 Which of the followings is an indication for induction of labor?
A Placenta brevia.
B Postterm gestation.
C Cord presentation.
D Prior classical cesarean section.
ANSWER:B
1478 In postterm pregnancy all the followings are true EXCEPT:
A Associated with meconium stained liquor.

B The fetus has long nails.
C May results in oligohydraminos.
D It is a gestational age beyond 43 weeks.
ANSWER:D
1479 Indications for induction of labor include all the following EXCEPT :
A Prolonged pregnancy
B Severe pre-eclampsia
C IUGR
D Previous 3 LSCS (lower segment c/s)
ANSWER:D
1480 Risks and complications of induction of labor include all the following, EXCEPT :
A Failed induction.
B Atonic postpartum hemorrhage.
C Uterine hyperstimulation leading to fetal hypoxia.
D Prostaglandin may cause hypothermia due to its direct effect on thermo regularity centers in the brain.
ANSWER:D
1481 Which of the following has NOT been shown to stimulate (induce) labor:
A Amniotomy.
B Prostaglandins.
C Enemas.
D Overeating.
ANSWER:D
1482 Regarding induction of labor all statements are false, EXCEPT:
A Chance of success is not dependent on the cervical bishop score.
B Is indicated in patient with mitral stenosis.
C Vaginal prostaglandin pessaries reduce the induction of delivery interval.
D n indicated at 40 weeks gestation.
ANSWER:C
1483 What is the 1/2 life of Oxytocin:
A < 1 min.
B 3 min.
C 10 - 15 min.
D 0 - 30 min.

ANSWER:B
1484 Amnio-hook (artificial ROM) may cause all except:
A Amniotic fluid embolus
B IUGR
C Abruptio placenta
D Fetal distress
ANSWER:B
1485 Complications of artificial rupture of membranes include all the following, EXCEPT :
A Abruptio placenta.
B Amniotic fluid embolism.
C Fetal distress.
D Meconium aspiration.
ANSWER:D
1486 A contraindication to the use of Oxytocin for stimulating labor at term is:
A Dead fetus.
B Hypertonic uterine dysfunction.
C Hypotonic uterine dysfunction.
D Twin gestation.
ANSWER:B
1487 Compound presentation is most consistently associated with :
A Prematurity.
B Advanced maternal age.
C Uncoordinated uterine contractions.
D Diabetic pregnant woman.
ANSWER:A
1488 Complete breech means:
A Flexion at hip joint and extension in knee joint
B Flexion at hip joint and flexion at knee joint
C Extension at the hip joint
D Flexion at knee joint and extension at the hip joint
ANSWER:B
1489 Regarding shoulder presentation, the following are true EXCEPT:
A Cord prolapse is common

B 3rd degree Perineal tear is common
C Deep transverse arrest can occur*
D More common in primigravida
ANSWER:D
1490 Breech presentation: Which is true?
A Constitutes 10% of all term deliveries
B Common in post term labor
C Vacuum extraction can be used when cervix is fully dilated
D Forceps can be used for after coming head
ANSWER:D
1491 Which of the following is contraindication for delivery using vacuum extraction?
A Face presentation
B Second twins in vertex presentation
C post term pregnancy
D Occipito transverse position
ANSWER:A
1492 Umbilical cord prolapse is most likely to occur with:
A Frank breech.
B Complete breech.
C Single footling breech.
D Double footling breech.
ANSWER:C
1493 The most frequent severe complication of vaginal breech delivery:
A Cord prolapse.
B Spinal cord injury.
C Head entrapment.
D Cord avulsion.
ANSWER:C
1494 The major cause of serious neonatal morbidity & mortality for infants with breech presentation is:
A Birth trauma.
B IUGR.
C Cord prolapse.
D Associated congenital anomalies.

ANSWER:A
1495 The most common complication of External Cephalic Versions:
A Placental separation.
B Fetomaternal hemorrhage.
C Persistent fetal bradycardia.
D Inability to convert the fetus to the vertex presentation.
ANSWER:C
1496 An absolute contraindication for vaginal breech delivery is:
A Footling breech presentation.
B Hyperextension of the head.
C Prolonged latent phase of labor.
D A large fetus with estimated fetal weight > 3000 gm.
ANSWER:A
1497
A 24-year-old G1 P0, Rh-negative , 36 weeks a breech presentation and is considering external cephalic version.
She should be told :
A She should be offered general anesthesia
B The procedure can be done with oligohydraminos
C Prophylaxis with anti-globulin D can wait until after delivery
D Tocolysis with intravenous ritodrine has been shown to improve the results of external version
ANSWER:D
1498 The following are possible causes of breech presentation EXCEPT :
A Prematurity
B Subserous fundal fibroid
C Multiple pregnancy
D Placenta previa
ANSWER:B
1499 Which of the following is contraindicated for delivery using Vacuum extraction?
A Face presentation
B 38 weeks gestation
C Chorioamnionitis
D Post-term pregnancy
ANSWER:A
1500 The incidence of breech presentation at term is :

A 20%
B 10%
C 3.5%
D 15%
ANSWER:C
1501 With regard to breech presentation :
A In vaginal delivery the fetus is prone to Cephalohematoma
B Hyperextension of the fetal head is a favorable
C External cephalic version (ECV), reduces the breech presentation at term 1%
D Mid trimester amniocentesis is likely to result in breech presentation at term
ANSWER:C
1502 External cephalic version is contraindicated in all of these conditions EXCEPT
A Scarred uterus
B Multiple pregnancy
C Placenta previa
D Gestational diabetes
ANSWER:D
1503 Regarding Breech presentation, the following are true EXCEPT:
A Accounts for up to 3 % term pregnancies
B May be diagnosed on clinical examination of the abdomen
C May be associated with fetal abnormality
D Is a contraindication for vaginal delivery
ANSWER:C
1504
An infant presents as a breech presentation and delivered without assistance as far as the umbilicus. The reminder
of the body is manually assisted by the obstetrician. This is called :
A Version and extraction
B Spontaneous breech delivery
C Assisted breech delivery
D Total breech extraction
ANSWER:C
1505
At 39 week gestation, a fetus was felt to be breech presentation as judged by information gained through
Leopold's maneuvers. The breech was well down in the pelvis, and the uterus was irritable. Pelvimetry was within
normal limits and the estimated fetal weight was 3.4 kg. which of the following should be done ?

A Cesarean section
B External cephalic version
C Internal podalic version
D Oxytocin induction
ANSWER:A
1506 All are causes of breech EXCEPT:
A Prematurity.
B Congenital anomalies.
C Hydrocephalus.
D Increased maternal age.
ANSWER:D
1507 All of the following associated with increase incidence of breech presentation, EXCEPT:
A Placenta previa
B Müllerian anomaly
C Uterine leiomyoma
D Nulliparity
ANSWER:D
1508 The following are contraindication to external cephalic version, EXCEPT:
A Contracted pelvis
B Placenta previa
C Multiple pregnancy
D Presence of cervical suture insite
ANSWER:D
1509 Face presentation :
A All cases must be delivered by C-section.
B All cases can be delivered vaginally.
C The presenting diameter is occipitofrontal.
D Mentoposterior position must be delivered by C-section.
ANSWER:D
1510 Regarding Puerperium:
A Refer to the first 6 months after delivery
B The lochia usually persist for 7 weeks
C The uterine fundus should not be palpable abdominally by 14 days after delivery

D The incidence of post partum depression is 50%
ANSWER:C
1511 The most common complication of breast feeding is:
A Amenorrhea.
B Pregnancy.
C Excessive weight loss.
D Puerperal mastitis.
ANSWER:D
1512 Oxytocin in the Puerperium is associated with :
A Involution of the uterus
B Initiation of lactation
C Resumption of menses
D Sub-involution of the uterus
ANSWER:A
1513 Of the following, the greatest predisposing cause of puerperal infection is:
A Retained placental tissue.
B Iron deficiency
C Coitus during late pregnancy
D Poor nutrition
ANSWER:A
1514 Which of the following is the most likely causes of a fever in a women on the second day postpartum:
A Pneumonia
B Endometritis.
C Mastitis
D Cholycystitis
ANSWER:B
1515 Postpartum, the deciduas becomes necrotic and is normally cast off within five to six days as :
A Decidual cast
B Placental remnants
C Lochia
D Carunculae myrtiforms
ANSWER:C
1516 A syndrome of amnenorrhea-glactorrhea developing postpartum is :

A Ahumada del Castillo
B Ciari-formmel
C Budd-chiari
D Sheehan's
ANSWER:B
1517 What is the recurrence risk of postpartum depression ?
A 5%.
B 20%.
C 70%.
D 95%.
ANSWER:C
1518 Immediately after the completion of a normal labor, the uterus should be :
A Firm & contracted at the level of the umbilicus.
B At the level of the symphysis pubis.
C Immobile.
D Atonic.
ANSWER:A
1519 In the mother, suckling leads to which of the following response:
A Decrease of Oxytocin.
B Increase of Prolactin-inhibiting hormone.
C Increase of hypothalamic dopamine.
D Increase of hypophyseal Prolactin.
ANSWER:D
1520 After parturition, endometrium regenerates from the decidual:
A Basal zone.
B Compact zone.
C Functional zone.
D Parietal layer.
ANSWER:A
1521 A routine postnatal check include all of the following EXCEPT:
A Breast examination.
B Pap smear.
C Serum blood sugar 2 hr. p. p.

D Bimanual vaginal examination.
ANSWER:C
1522 Risk factors of post partum Endometritis include all of the following, EXCEPT:
A Prolonged labor
B Prolonged rupture of membranes
C Multiple vaginal exams
D Gestational diabetes.
ANSWER:D
1523 Symptoms and signs of puerperal endometritis include all the following, EXCEPT :
A Malodorous vaginal discharge.
B Lower abdominal pain.
C Fever.
D Involution of the uterus.
ANSWER:D
1524 All the following methods inhibit lactation EXCEPT :
A Restriction of fluid and diuretics
B Tight breast binder and analgesics
C estrogen hormone in large dose
D Thyroxin hormone
ANSWER:A
1525 Breast feeding :
A Should be discontinued if breast infection is suspected
B Should be started until 3 days postpartum
C Should be replaced by bottle feeds if the patient had a caesarean section
D Has a role in involution of the fetus
ANSWER:D
1526 In the mother suckling leads to which of the following responses?
A Decrease Oxygen
B Increase of prolactin-inhibiting factor
C Increase of hypothalamic dopamine
D Increase of hypothalamic Prolactin.
ANSWER:D
1527 Breast feeding accelerates the involution of the uterus through:

A The increased level of prolactin.
B The release of oxytocin.
C The increased level of estrogen.
D The decreased level of progesterone.
ANSWER:B
1528 Therapeutic termination of early pregnancy is indicated in:
A Uterine fibroid
B Maternal pulmonary hypertension
C Placenta previa
D Maternal blood sugar >12 mmol/L
ANSWER:B
1529 In Pre-eclampsia, right upper quadrant part abdominal pain is due.
A Tension of the liver capsule
B Cholecystitis
C Pancreatitis
D Gastric ulcer
ANSWER:A
1530 Immediate appropriate response to an initial eclamptic seizure include all of the following, EXCEPT:
A Ultrasound for fetal growth
B Maintain adequate oxygenation
C Administer magnesium sulphate
D Prevent maternal injury
ANSWER:A
1531 The most important reason to give antihypertensive drug for hypertension in pregnancy is to decrease the:
A Incidence of IUGR
B Incidence of oligohydraminos
C Incidence of fetal death
D Risk of maternal complications such as stroke
ANSWER:D
1532 Risk factors for pre- eclampsia include all of the following, EXCEPT:
A Elderly primigravida
B African ethnicity
C Positive family history of hypertension

D Positive history of macrosomic baby
ANSWER:D
1533 Pre-eclampsia is associated with an increase risk of all of the following, EXCEPT:
A Delivery of a small for gestational age infant
B Placental abruptio
C Pulmonary edema
D Prolonged duration of labor
ANSWER:D
1534 The following are known complications of pre-eclampsia EXCEPT :
A Abruptio placenta.
B Uterine rupture.
C Prematurity.
D Placental insufficiency.
ANSWER:B
1535 Severely pre-eclamptic patients have a decrease in :
A Response to pressor amines.
B Plasma volume.
C Total body sodium.
D Uric acid.
ANSWER:B
1536 What is the most common cause of acute renal failure in pregnancy ?
A Drug abuse.
B SLE.
C Pre-eclampsia and eclampsia.
D Sickle cell disease.
ANSWER:C
1537 Regarding essential hypertension in pregnancy, all the following is true, EXCEPT
A Commonly associated with +ve family Hx of hypertension.
B Usually diagnosed in the 3 rd trimester.*
C More common in women over the age of 35.
D It's usually not associated with significant proteinuria.
ANSWER:B

1538
A pre-eclamptic patient has just delivered and has a soft uterus with moderate bleeding. Examination reveals no
laceration. Of the options below, the BEST choice is :
A 0.2 mg IV ergometrine.
B 0.5 mg oral ergometrine.
C 5 units of oral oxytocin.
D 20 units Oxytocin in a 500 ml of D 5 W given IV.
ANSWER:D
1539 Pregnancy induced proteinuric hypertension is associated with all the following, EXCEPT :
A An increase in serum uric acid level.
B Plasma volume decreases.
C An increase in the incidence of IUGR.
D An increase in creatinine clearance.
ANSWER:D
1540 Regarding pre-eclampsia, proteinuria is defined as :
A 100 mg/24 hour urine.
B 200 mg/24 hour urine.
C 300 mg/24 hour urine.
D > 500 mg/24 hour urine.
ANSWER:C
1541 Which adverse pregnancy outcome isn't increased in pregnancies complicated by chronic hypertension :
A Fetal growth restriction.
B Preterm birth.
C Spontaneous preterm rupture of membranes.
D Postterm birth
ANSWER:D
1542 A syndrome seen in pre-eclampsia called HELLP syndrome is characterized by all of the following EXCEPT:
A Elevation of Liver enzymes.
B Hemolysis.
C Low platelet count.
D Prolongation of the Prothrombin time.
ANSWER:D
1543 The most common presenting prodromal sign or symptom in patient with eclampsia is:
A Right upper quadrant pain.

B Edema.
C Headache.
D Visual disturbance.
ANSWER:C
1544 Appropriate response to an initial eclamptic seizure include all of the following EXCEPT:
A Attempt to abolish the seizure by administrating I.M. diazepam.
B Maintain adequate Oxygenation.
C Administer Mg sulphate by either the I.M. or I.V. route.
D Prevent maternal injury.
ANSWER:A
1545 The most consistent finding in Eclampsia patients is:
A Hyperreflexia.
B 4+ proteinuria.
C Generalized edema.
D Convulsions.
ANSWER:D
1546 Eclampsia occurring prior to 20 weeks of gestation is most commonly seen in women with:
A A history of chronic hypertension.
B Multiple gestation.
C Gestational trophoblastic diseases.
D A history of seizure disorder.
ANSWER:C
1547 Of the following, the most common complication of eclampsia is:
A Mg intoxication.
B Recurrent seizures following administration of Mg sulphate.
C Intracranial hemorrhage.
D Maternal death.
ANSWER:C
1548 The reason of using antihypertensive treatment in pregnancy is to:
A Reduce the placental blood flow.
B Reduce the risk of CVA in the fetus.
C Reduce the risk of CVA in the mother.
D Prevent hypertensive renal disease.

ANSWER:C
1549 Which of the following laboratory tests would be most suggestive of preeclampsia?
A Elevated bilirubin.
B Decreased hematocrit.
C Elevated lactate dehydrogenase (LDH).
D Elevated uric acid.
ANSWER:D
1550 HELLP Syndrome includes all the followings EXCEPT:
A Hemolysis.
B Increased AST.
C Increased platelets.
D Increased ALT.
ANSWER:C
1551 The proteinuria in eclampsia contains?
A Only albumin
B Only globulin
C More albumin than globulin
D More globulin than albumin
ANSWER:C
1552 Which adverse pregnancy outcome is not increased in pregnancies complicated by chronic hypertension :
A Fetal growth restriction
B Pre-term birth
C Spontaneous preterm rupture of membrane
D Post-term birth
ANSWER:D
1553 Pregnancy induced proteinuria hypertension is associated with all following, EXCEPT :
A Increases in serum uric acid level
B Plasma volume decreases
C Increases in the incidence of IUGR
D Increase in creatinine clearance
ANSWER:D
1554
A pre-eclamptic patient has just delivered & has a soft uterus with moderate bleeding. Examination reveals no
laceration. Of the options below, the BEST choice :

A 0.2 mg IV ergometrine
B 0.5 mg oral ergometrine
C 5 units oral oxytocin
D 20 units oxytocin in a 500 ml of D5W given IV
ANSWER:D
1555 Of the following, the most common cause of maternal death from eclampsia is:
A Infection.
B Uremia.
C Congestive heart failure.
D Cerebral hemorrhage.
ANSWER:D
1556 The criteria for severe pre-eclampsia include all the following, EXCEPT:
A Diastolic blood pressure of 110 mmhg or more.
B Proteinuria more than 5g/24 hours.
C Presence of epigastric pain .
D Decreased hematocrit.
ANSWER:D
1557 The immediate appropriate response to an eclamptic seizure includes all the following, EXCEPT :
A Monitor the fetal heart rate.
B US done for fetal growth.
C Administer Mg sulphate.
D Maintain adequate oxygenation.
ANSWER:B
1558 All the following factors increase the risk of the development of pre-eclampsia, EXCEPT :
A Closed spaced pregnancies.
B Pre-existing diabetes.
C Multiple gestation.
D Pre-eclampsia with a previous pregnancy.
ANSWER:A
1559 Risk factors to preeclampsia include all the following EXCEPT:
A Premature delivery.
B Placenta abruption.
C Renal failure.

D Polycythemia.
ANSWER:D
1560 Pregnancy induced hypertension is more common in all the following EXCEPT:
A Primigravida.
B Multiple pregnancy.
C Patients with pre-existing hypertension.
D Women with UTI.
ANSWER:D
1561 Which of the following laboratory tests would be most suggestive of preeclampsia?
A Elevated bilirubin.
B Decreased Hematocrit.
C Elevated lactate dehydrogenase (LDH).
D Elevated uric acid.
ANSWER:D
1562 Early clinical evidence of magnesium sulfate toxicity would show:
A Flushing
B Diplopia
C Decreased oxygen saturation
D Loss of deep tendon reflexes
ANSWER:D
1563 The following are true regarding patients with essential hypertension in labor, EXCEPT :
A Shouldn't have epidural analgesia.
B Can be safely given IV syntocinon.
C Shouldn't be given ergometrin as a routine in the 3rd stage.
D Should have continuous fetal heart rate monitoring.
ANSWER:A
1564 Markedly obese pregnant patients often experience all the following complications, EXCEPT :
A Hypertension.
B Diabetes mellitus.
C Thromboembolism.
D Fetal growth restriction.
ANSWER:D
1565 What is the mechanism of action of alpha methyl dopa?

A Increase peripheral vascular resistance.
B Increase sodium and water retention.
C Acts centrally to decrease sympathetic activity.
D Relaxes arterial smooth muscle.
ANSWER:C
1566 Gestational diabetes is associated with an increase risk of all the following, EXCEPT:
A Cesarean section
B Shoulder dystocia
C Fetal macrosomia
D Intrauterine growth restriction
ANSWER:D
1567 Control of gestational diabetes is accomplished with the following, EXCEPT:
A Insulin
B Diet
C Oral hypoglycemic agents
D Exercise
ANSWER:C
1568 Infants of mother with gestational diabetes have an increased risk of the following, EXCEPT:
A Hypoglycemia
B Hypoinsulinemia
C Hypocalcemia
D Hyper bilirubine
ANSWER:B
1569 The best screening test for gestational diabetes
A Fasting blood sugar
B Random blood sugar
C Glucose challenge test
D Glucose tolerance test
ANSWER:C
1570 Glucose tolerance test :
A Is used as a screening test for diabetes.
B Is considered to be a diagnostic test for diabetes.
C Is performed in a non-fasting state.

D Should be avoided during pregnancy as it needs a loading dose of glucose.
ANSWER:B
1571 Indications of glucose tolerance test GTT in pregnancy include all the following EXCEPT :
A Previous Hx of gestational diabetes mellitus.
B Hx of macrosomic baby.
C Glycosuria in one occasion.
D Hx of unexplained fetal death.
ANSWER:C
1572 Regarding Good control of diabetes in pregnancy :
A Maintains blood sugar level between 8 and 12 mmol/L.
B Is achieved by twice daily injection of insulin.
C Increases the incidence of polyhydramnios.
D Has no effect on the incidence of congenital abnormalities.
ANSWER:B
1573 In the Saudi population, the most common problem during pregnancy is :
A Diabetes.
B Toxemia.
C Heart disease.
D UTI.
ANSWER:A
1574 Maternal complications associated with polyhydramnios include :
A High blood pressure.
B Urinary tract anomalies.
C Diabetes.
D Postmature pregnancy.
ANSWER:C
1575 Which of the following items in a pregnant patient's History suggests the possibility of her having diabetes :
A IUGR.
B Past Hx of twins.
C 1st trimester bleeding.
D Unexplained stillbirths.
ANSWER:D
1576 During pregnancy, blood tests for diabetes are more abnormal than in nonpregnant state. This is due to :

A Decreased insulin.
B Increased absorption from the GI tract.
C Increased placental lactogen.
D Estrogen decreases and progesterone increases.
ANSWER:C
1577 The test that is used to diagnose diabetes is :
A Glucose challenge test.
B FBS & random blood sugar.
C Glucose tolerance test.
D Hb A1C.
ANSWER:C
1578 Which of the following is a known complication of diabetes in pregnancy :
A It increases the incidence of congenital abnormalities.
B Development of pregnancy induced hypertension.
C Development of oligohydraminos.
D Development of retinopathy.
ANSWER:A
1579 Gestational diabetes should be suspected in all the following conditions, EXCEPT:
A A strong family Hx of diabetes.
B Previous Hx of big baby.
C Polydipsia and polyuria.
D Hx of unexplained fetal loss.
ANSWER:C
1580 Infants of diabetic mothers are at risk of one of the following :
A Low Hb.
B Hypercalcemia.
C Hyperglycemia.
D Respiratory distress syndrome.
ANSWER:D
1581 Insulin requirements of pregnant diabetic women are greatest during :
A The 1st half of pregnancy.
B The 2 nd half of pregnancy.
C During lactation.

D The immediate postpartum period.
ANSWER:B
1582 Gestational Diabetes Mellitus (GDM) is associated with:
A Increased risk of spontaneous abortions.
B Increased risk of fetal cardiac malformations.
C Increased risk of fetal CNS malformations.
D IUGR.
ANSWER:A
1583 Infants of mothers with GDM have an increased risk of all of the following EXCEPT:
A Hypoglycemia.
B Hyperglycemia.
C Hypocalcaemia.
D Hyperbilirubinemia.
ANSWER:B
1584 GDM is associated with an increased risk of all the following EXCEPT:
A C-section.
B Shoulder dystocia.
C Fetal Macrosomia.
D IUGR.
ANSWER:D
1585 Infants of mothers with GDM are at an increased risk of becoming:
A Obese adults.
B Type II diabetes.
C Neither
D Both.
ANSWER:D
1586 Compared to Type II diabetes, type I diabetes is associated with all of the following EXCEPT:
A Greater incidence of pre-eclampsia.
B Greater incidence of preterm delivery.
C Greater risk of maternal hypoglycemia.
D Reduced risk of IUGR.
ANSWER:D
1587 GDM substantially increase the mother's risk for the ultimate development of:

A Type I DM.
B TypeIIDM.
C Neither.
D Both.
ANSWER:B
1588 Which of the followings is a known complication of diabetes in pregnancy?
A Fetal microsomia (small baby).
B Oligohydraminos.
C Iron deficiency anemia.
D Fungal infection.
ANSWER:D
1589 Regarding gestational diabetes mellitus (GDM):
A It is the most common cause of IUGR.
B The best screening test is random blood sugar.
C The diagnostic test is glucose tolerance test (GTT).
D All patients should be treated by insulin, as diet alone is not enough.
ANSWER:C
1590 Diabetes in pregnancy can cause all the following congenital anomalies EXCEPT:
A Sacral agenesis.
B Central nervous system abnormalities.
C Lower limb hypoplasia.
D Yellow teeth discoloration.
ANSWER:D
1591 Infants of diabetic mothers are at risk of one of the following :
A Low hemoglobin
B Hypocalcaemia
C Hyperglycemia
D Microsomia
ANSWER:B
1592
The serum insulin level in the newborn infant of a diabetic mother in comparison to the infant of a euglycemic
mother is generally :
A Higher
B The same as euglycemic

C Lower
D Extremely labile
ANSWER:A
1593 With overt diabetes, what is the most common fetal malformation?
A Congenital heart disease.
B Caudal regression.
C Renal agenesis.
D Neural tube defect.
ANSWER:A
1594 All the following are considered to be complications of gestational diabetes, EXCEPT :
A Polyhydramnios.
B IUGR.
C Fetal macrosomia.
D Sudden unexpected intrauterine fetal death.
ANSWER:B
1595 Regarding gestational diabetes, choose the correct answer :
A Cleft lip is the most common congenital anomaly seen with gestational diabetes.
B Fetal lung maturity is accelerated in diabetic patient compared to non diabetic mothers.
C Macrosomia is one of the complications of uncontrolled gestational diabetes.
D Gestational diabetes usually starts before 12 weeks gestation.
ANSWER:C
1596 The most common congenital anomaly associated with diabetes is:
A Congenital heart disease.
B Neural tube defect.
C Sacral agenesis.
D Renal agenesis
ANSWER:A
1597 Regarding gestational diabetes mellitus :
A The incidence of this disease is very low in KSA.
B Potential diabetic patients should have only glucose challenge test as a diagnostic test.
C
Intrauterine growth retardation is more common in patients with gestational diabetes than those patients with long
standing diabetes mellitus.
D Oral hypoglycemic agents are not safe during pregnancy.

ANSWER:D
1598 Polyhydramnios is commonly found with
A IUGR
B Placenta previa
C Diabetes insipidus
D Diabetes mellitus
ANSWER:D
1599 Risk factors for the development of gestational diabetes include all the following, EXCEPT :
A Obesity.
B Family Hx of diabetes.
C Previous Hx of IUFD.
D Previous Hx of IUGR.
ANSWER:D
1600 In a pregnant female which of the following depicts the level of iron:
A Transferrin level
B Serum ferritin level
C Haemoglobin level
D Iron binding capacity
ANSWER:A
1601 The most common type of anemia in pregnancy is due to :
A Iron deficiency.
B Sickle cell disease.
C Folate deficiency.
D Hemolytic disease.
ANSWER:A
1602 Folic acid deficiency results in :
A Microcytic anemia.
B Megaloblastic anemia.
C Aplastic anemia.
D G6PD deficiency.
ANSWER:B

1603
A 29 year old primigravida woman at 16 weeks of gestation reports irritability, palpitation and difficulty in
gaining weight. Physical examination is normal except for a pulse of 104 beats/min and lid lag. Laboratory studies
reveal Hb of 10.1g/dl, serum free thyroxin of 10 ng/dl and thyrotropin of 0.1 mU1ow. What is the most likely
diagnosis?
A Thyroid storm.
B Hypothyroidism.
C Depression.
D Hyperthyroidism.
ANSWER:D
1604 According to WHO, Hb in a pregnant woman shouldn't be less than:
A <8
B <10
C <9
D <11
ANSWER:B
1605 Disseminated intravascular coagulation has a recognized association with:
A IUFD
B Multiple pregnancy
C Iron deficiency
D Diabetic mother
ANSWER:A
1606 Risk Factors for DVT include all, EXCEPT:
A Smoking
B Operative delivery
C Lupus anticoagulation
D Hyperthyroidism
ANSWER:D
1607 In pregnant patients with rheumatic heart disease, all of the following is true, EXCEPT:
A Should be always given iron to avoid anemia.
B Commonly develop atrial fibrillation.
C Should be given prophylactic antibiotic in labor.
D Usually remain asymptomatic.
ANSWER:B

1608 Which of the following isn't considered a high risk pregnancy ?
A Gestational diabetes.
B Cardiac disease in pregnancy.
C Candida infection in pregnancy.
D Bleeding in pregnancy.
ANSWER:C
1609
Regarding cardiac disease in pregnancy, according to the New York Heart Association classification, a patient
with cardiac disease & slight limitation of physical activity would be:
A Class O.
B Class I.
C Class II.
D Class III.
ANSWER:C
1610 Which of the following accounts for most heart disease in pregnancy ?
A Cardiomyopathy
B Previous myocardial infarction
C Hypertension
D Congenital heart disease (CHD)
ANSWER:D
1611
What is the most serious disadvantage of switching from Warfarin to heparin during pregnancy in women with
mechanical heart prosthesis ?
A The risk of embryopathy is increased.
B The risk of thrombo-embolism is increased.
C The need for self-injection.
D Hemorrhage is more likely.
ANSWER:C
1612
If delivery occurs unexpectedly while the patient is on Heparin and excessive bleeding occurs, what is the
appropriate initial intervention?
A Administration of protamine acetate.
B Administration of vitamin K .
C Administaration of tranexemic acid.
D Ligation of the hypogastric arteries.
ANSWER:A

1613 Which of the following symptoms in pregnancy is most suggestive of heart disease ?
A Tachypnea.
B Syncope with exertion.
C Tachycardia.
D Peripheral edema.
ANSWER:B
1614
Regarding anticoagulants in pregnancy, which of the following laboratory parameters should be maintained at a
level of 1.5 to 2.5 times baseline value?
A Bleeding time.
B Partial thromboplastin time.
C Prothrombin time.
D Thrombin time.
ANSWER:B
1615 UTI in pregnancy :
A Occurs in about 10% of pregnancies.
B Can be easily diagnosed when there is more than 1000 bacteria per ml on culture.
C Is commoner in multigravida when compared with primigravida.
D May lead to preterm labor.
ANSWER:D
1616 All are the risks associated with macrosomia, EXCEPT:
A Maternal obesity
B Prolonged Pregnancy
C Previous large infant
D Short Stature
ANSWER:D
1617 Which of the following is not associated with large infants?
A Diabetic mothers
B Multiparity
C Large parents
D Maternal smoking
ANSWER:D
1618 Perinatal mortality refers to:
A Number of stillbirths per 1,000 total births.

B Number of stillbirths & neonatal deaths per 1,000 total births.
C Number of stillbirths & neonatal deaths per 1,000 live births.
D Number of neonatal deaths per 1,000 total births.
ANSWER:B
1619 Markedly obese pregnant patients often experience all of the following complications EXCEPT :
A Hypertension
B Diabetes Mellitus
C Thromboembolism
D Fetal growth restriction
ANSWER:D
1620 All of the following are possible causes of acute abdomen in pregnancy EXCEPT :
A Large ovarian cyst
B Large fetus
C Large appendix
D Large peptic ulcer
ANSWER:B
1621
A patient in the third trimester of pregnancy is seen in the emergency room and while being examined has a
convulsion. The doctor should immediately:
A Obtain neurologic consultation
B Obtain psychiatric consultation
C Give IV valium
D Protect the patient from self harm
ANSWER:D
1622 Fetoprotein can be elevated in all the following, EXCEPT :
A Anencephaly.
B Duodenal atresia.
C Closed meningiocele.
D Fetal death.
ANSWER:D
1623 The mechanism of Action of alpha methyldopa:
A Increased peripheral vascular resistance.
B Increased sodium & water diuresis.
C Act centrally to decrease sympathetic.

D Relaxes arterial smooth muscles.
ANSWER:C
1624 Maternal aspiration of gastric contents during labor is most often due to:
A Gastric hypotonicity.
B Pelvic pain.
C Pneumothorax.
D Anesthesia.
ANSWER:D
1625 Oligohydromnios is characteristically associated with:
A Rh isoimmunization.
B Renal agenesis.
C Diabetes mellitus.
D Hypothyroidism.
ANSWER:B
1626 Which of the following contraceptive methods should NOT be used by a patient with coronary heart disease?
A Combined oral contraceptive pills
B Male condom
C Female condom
D Diaphragm
ANSWER:A
1627 Permanent sterilization include all of the following, EXCEPT:
A Vasectomy
B Tubal ligation
C Mirena IUCD
D Hysterectomy
ANSWER:C
1628 The combined contraceptive pills: Which is true?
A Pre-dispose to pelvis inflammatory disease
B Predispose to ovarian cyst
C Predispose to benign breast cyst
D Contraindicated in a patient with history of DVT
ANSWER:D
1629 The following is an absolute contraindications to the combined oral contraceptive pills:

A Varicose veins
B Previous history of viral hepatitis
C Prosthetic heart valve
D Diabetes mellitus
ANSWER:C
1630 The following conditions are aggravated by oral contraceptive pills
A Hirsutism
B Endometriosis
C Dysmenorrhea
D Genital fungal infection
ANSWER:D
1631 Regarding Intra uterine contraceptive device (IUCD)
A Reduces pelvic inflammatory disease
B Inhibits ovulation
C Increases incidence of endometrial cancer
D If pregnancy occurs there is increased risk of ectopic pregnancy.
ANSWER:D
1632 Combined Oral Contraceptive Pills contain:
A A synthetic estrogen alone
B A progestin alone
C Both estrogen and progestin
D Bromocriptine
ANSWER:C
1633 Mechanisms of Oral Contraceptive Pills include all of the following EXCEPT:
A Ovulation suppression
B Enhanced ovarian androgen production
C Altered cervical mucus
D Altered endometrium
ANSWER:B
1634 Postulated mechanism of the IUCD include all of the following action EXCEPT:
A Altered tubal motility
B Altered endometrium
C Altered cervical mucus

D Cupper has spermicidal effect
ANSWER:C
1635 Regarding injectable progesterone contraception, all of the following is true, EXCEPT:
A Medroxyparogesterone acetate is the most commonly used
B May cause irregular uterine bleeding
C May cause amenorrhea.
D Should not be given to lactating mother.
ANSWER:D
1636 With regards to contraception failure, the pearl index refers to:
A Numbers of Pregnancies in years.
B Number of pregnancies in 1 woman-year.
C Number of pregnancies in 100 woman-year.
D Number of pregnancies in 100 woman-years over pregnancy losses.
ANSWER:C
1637 Contraindication of OCP, EXCEPT:
A IHD
B Previous DVT
C Previous CVA
D 35 Y, smokes>15 cigarette perday.
ANSWER:D
1638 Combined oral contraceptive pills are associated with :
A Dysmenorrhea
B Menorrhagia
C Polymenorrhoea
D Intermenstrual bleeding.
ANSWER:D
1639 OCP have the following beneficial effects EXCPET :
A Decrease endometrial cancer
B Decrease benign breast disease
C Decrease iron deficiency anemia
D Decrease the incidence of renal failure
ANSWER:D
1640 Side effect of OCP include all the following EXCEPT:

A Break through bleeding
B Dysmenorrhea.
C Nausea
D Mastalgia
ANSWER:B
1641 The use of OCP reduces the risk of all the following EXCEPT:
A Ectopic pregnancy
B Hepatic adenoma.
C Salpangitis
D Ovarian cancer
ANSWER:B
1642 Which of the following statements regarding the use of IUCD is TRUE :
A Most IUD were withdrawn from the market due to the financial burden
B The Contraceptive effectiveness of IUD is similar to that of barrier methods
C The risk of Salpangitis in IUD wearer is positively correlated with duration of use
D Contraceptive effectiveness of copper-containing IUD is higher than that of the inert device
ANSWER:D
1643 All are true regarding OCP EXCEPT :
A Increase viscosity of cervical mucosa
B Can cause venous thrombosis
C Can cause mild hypertension
D Usually cause amenorrhea
ANSWER:D
1644 IUCD are relatively contraindicated in the following patient EXCEPT:
A With past history of chronic Salpangitis.
B With Submucous fibroid
C With history of ectopic pregnancy
D With history of Csection
ANSWER:D
1645 Lowest pregnancy rate in 100 women using the method for 1 year :
A IUCD
B OCP
C Condom

D Diaphragm
ANSWER:A
1646 The use of progesterone only pills :
A Ovulation is not regularly inhibited
B Protection against is as good as combined pill
C There is a risk of DVT
D Breakthrough bleeding is rare
ANSWER:A
1647 A contraceptive method that prevent transmission of STD is:
A Condom.
B OCP.
C IUCD.
D Spermicide.
ANSWER:A
1648 The goal for developing a successful contraception include all of the following EXCEPT:
A Regulate fertility.
B Stabilize population growth.
C Reducing maternal morbidity.
D Diminishing the incidence.
ANSWER:D
1649 What is the effect of oral contraceptives on the incidence of DVT & embolism?
A Decreases the incidence.
B Doesn't affect the incidence.
C Increases thrombosis but not pulmonary embolism.
D Increases the incidence.
ANSWER:D
1650 The mechanism of action of OCPs include all the following, EXCEPT:
A Enhances ovarian androgen production.
B Alter the cervical mucous.
C Alter the endometrium.
D Ovulation suppression.
ANSWER:A
1651 Postulated mechanism of action of IUD includes all the following, EXCEPT :

A Alter the cervical mucous.
B Alter the endometrium.
C Inhibits fertilization.
D Alter the tubal motility.
ANSWER:A
1652 Which of the following contraceptive methods should not be used by a patient with coronary heart disease?
A COCP.
B Male condoms.
C Female condoms.
D Diaphragm.
ANSWER:A
1653 Estrogens commonly used in oral contraceptive pills include.
A Ethinyl estradiol.
B Ethinyl estriol.
C Northisterone.
D Ethisterone.
ANSWER:A
1654 The goals for developing a successful contraceptive include all of the following EXCEPT:
A Regulating fertility.
B Stabilizing population.
C Reducing maternal morbidity.
D Diminishing the incidence.
ANSWER:D
1655 Regarding IUCD, all of the following are INCORRECT, EXCEPT:
A Inhibition of ovulation is the main mechanism of action.
B Has a failure of 6%.
C Statistically increases the incidence of ectopic pregnancy.
D Is commonly associated with excessive menstrual loss.
ANSWER:C
1656 The following are known contraindications to the use of combined oral contraceptive pills (COCP), EXCEPT:
A Deep venous thrombosis or pulmonary embolism.
B Active liver disease.
C Benign intracranial hypertension.

D Repeated LSCS.
ANSWER:D
1657
The combined oral contraceptive pill, besides being used as a method of contraception, can be used to treat the
following, EXCEPT:
A Endometriosis.
B Ovulation pain.
C Menorrhagia.
D Mucinous ovarian cyst.
ANSWER:D
1658 Progesterone only pills work as contraceptives by:
A Suppressing ovulation.
B Altering the cervical mucosa.
C Producing endometrial hyperplasia.
D Reducing libido.
ANSWER:A
1659 Combined oral contraceptive pills containing estrogen & progesterone produce the following effects, EXCEPT:
A Act by preventing ovulation.
B Can cause hypertension.
C Can cause venous thrombosis.
D Usually cause amenorrhea.
ANSWER:D
1660 Regarding Progesterone only pills preparation:
A Affect the cervical mucous.
B Reliably inhibit ovulation.
C Are contraindicated in mild hypertension.
D Are reliable as the combined preparation.
ANSWER:B
1661 Which of the following is an absolute contraindication to the use of COCP?
A Varicose veins.
B DM.
C Seizure disorders.
D Recent history of deep venous thrombosis.

ANSWER:D
1662 IUCD may prevent pregnancy by all of the following mechanism EXCEPT:
A Creating chronic endometritis.
B Inducing endometrial atrophy.
C Inhibiting ovulation.
D Altering tubal motility.
ANSWER:C
1663 Vulvovaginal Candidiasis:
A Cause muco-purulent cervicitis.
B Frequently associated with systemic symptoms.
C Maybe diagnosed microscopically by mixing discharge with KOH.
D Is treated with doxycycline.
ANSWER:C
1664 Bacterial Vaginosis:
A Is rare vaginal infection.
B Is always symptomatic.
C Is usually associated with profound inflammatory reaction.
D Causes fishy discharge which results from bacterial amine production.
ANSWER:D
1665 Trichomoniasis:
A Associated with cytological findings in PAP smear.
B Associated with pregnancy & Diabetes mellitus.
C Is sexually transmitted parasite which causes pruritic discharge.
D May cause overt warts.
ANSWER:C
1666 All the followings can be transmitted sexually EXCEPT:
A HIV.
B Chlamydia.
C Gonorrhea.
D Vaginal Candidiasis.
ANSWER:D
1667 Regarding gonococcal infection of the female genital tract, the following are common site of infection:
A bartholine gland

B Skene gland
C Urethra & cervix
D Rectal crypt
ANSWER:C
1668 Vaginal Trochomitis is :
A sexually transmitted disease
B Yeast infection
C Cause Salpangitis
D Is more common in diabetic
ANSWER:A
1669 Candida occur in all of the following EXCEPT:
A Diabetic
B Long term anti biotic
C Thyrotoxicosis
D OCP
ANSWER:C
1670 All the followings can be transmitted sexually EXCEPT:
A HIV.
B Clamydia.
C Gonorrhea.
D Vaginal Candidiasis.
ANSWER:D
1671 Genital tract Candida occurs more frequent in all these patients, EXCEPT:
A Diabetic.
B On long term antibiotic therapy.
C Thyrotoxicosis.
D On oral contraceptive pills.
ANSWER:C
1672 Regarding Vaginal Trichomoniasis, choose the CORRECT answer:
A Is a yeast infection.
B Is generally sexually transmitted.
C Frequently cause Salpangitis.
D Is more common in diabetic patients.

ANSWER:B
1673 The treatment choice for gardenerella vaginosis is :
A Ampicillin.
B Metronidazole.
C Clindamycin.
D Gentamycin.
ANSWER:B
1674 Which of the following is NOT detected by a cervical smear?
A Herpes Simplex Virus
B Human Papilloma Virus
C Actinomycosis
D Trichomonas
ANSWER:D
1675 Metronidazole is indicated for which of the following conditions:
A Bacterial vaginosis
B Trichomonas
C Prophylaxis before vagina surgery
D All of the above
ANSWER:D
1676 Vaginal Trichomoniasis, choose the correct answer :
A Is a yeast infection
B Is generally sexually transmitted
C Frequently cause salpingitis
D Is more common in diabetic patients
ANSWER:B
1677 All the following infection can be transmitted through sexual intercourse, EXCEPT:
A Genital herpes
B Human Immunodeficiency Virus (HIV)
C Chlamydia
D Vaginal Candidiasis
ANSWER:D
1678 Treatment of pelvic inflammatory disease (PID) include the following, EXCEPT:
A Oral doxycycline

B Removal of IUCD
C Clindamycin
D Dilatation and curettage (D&C)
ANSWER:D
1679 The differential diagnosis of vulvar swelling includes all the followings EXCEPT:
A Bartholin's cyst.
B Hematoma.
C Condyloma.
D Nabothian cyst.
ANSWER:D
1680 The terminology of pelvic inflammatory diseases indicates:
A Infection of the vagina.
B Infection of Bartholin's glands
C Infection of Skene's glands.
D Endometritis and salpingo-oophoritis.
ANSWER:D
1681 Late sequele of Salpangitis:
A Endometriosis
B Endometrial hyperplasia
C Fallopian tube cancer
D Hydrosalpinx
ANSWER:D
1682 Recto-vaginal fistula result from all of the following EXCEPT:
A obstetrical delivery
B Irradiation to the pelvis
C Carcinoma
D Obesity
ANSWER:D
1683 The etiology of PID include all of the following except:
A TB.
B Hepatitis virus.
C Chlamydia.
D Group B streptococci.

ANSWER:B
1684 The differential diagnosis of vulvar swelling includes all the followings EXCEPT:
A Bartholin's cyst
B Hematoma.
C Condyloma.
D Nabothian cyst.
ANSWER:D
1685
A 38 year old woman is seen for the evaluation of a swelling in her right vulva. She has also noted pain in this
area when walking and during coitus. On examination a mildly tender flactuant mass was noticed just outside the
introits in the right vulva. What the most likely diagnosis?
A Bartholin's abscess.
B Lymphogranuloma venerum.
C Chancroid.
D Vulva carcinoma.
ANSWER:A
1686 Treatment of pelvic inflammatory disease include the following, EXCEPT:
A Oral doxycycline
B Removal of IUCD
C Clindamycin
D Dilatation and curettage (D&C)
ANSWER:D
1687
On bimanual examination, bilateral adnexal masses were palpated. A vaginal US was done for her and it showed
bilateral tubo-ovarian abscesses. What is the most appropriate next step in her management:
A Admit the patient for emergency laparoscopic drainage of the abscess.
B Admit the patient and give her IV antibiotic.
C Treat with multiple oral antibiotics as an outpatient.
D Call interventional radiotherapy to perform "CT guided Percutaneous aspiration".
ANSWER:B
1688 Bartholin's abscess:
A Is often asymptomatic.
B Is usually bilateral.
C Is most commonly due to gonococcus infections.
D Is best treated surgically.

ANSWER:D
1689 The mechanism of infertility in pelvic inflammatory disease (PID) include the following EXCEPT:
A Polycystic ovary.
B Peritubal adhesions.
C Hydrosalpinx.
D Pyosalpinx.
ANSWER:A
1690 Disorders of pelvic support are commonly associated with all of the following EXCEPT:
A Multiparity. (children)
B Aging.
C Alcohol consumption.
D Pelvic trauma.
ANSWER:C
1691 All the following are true, about ovarian hyperstimulation, EXCEPT:
A It can follow any of ovulation induction drugs
B In severe type admission to the intensive care unit may be required
C The ovaries will be very small in size have unilateral cyst.
D It can diagnosed clinically and by ultrasound
ANSWER:C
1692 All of the following are true about PCO EXCEPT:
A To diagnose PCO hormonal analysis can be done at any time of the cycle
B It is associated with reversed FSH:LH ratio
C It is associated with increase resistance to insulin
D Can be associated with increased prolactin level
ANSWER:A
1693 Evaluation of infertile couples:
A History of regular cycles is suggestive of anovulation
B History of severe dysmenorrhea and dyspareunia is suggestive of endometriosis
C Endometriosis is diagnosed by hysteroscopy
D Hysterosalpingogram is the only means of assisting ectopic pregnancy
ANSWER:B
1694 Treatment of infertility
A Clomiphene citrate helpful in patients with anovulatory premature ovarian failure

B Ovarian drilling is helpful in the treatment of infertility endometriosis
C IVF is not helpful in the treatment of infertility due to male factor
D Hyperstimulation syndrome can occurs in patient treated by gonadotropins
ANSWER:D
1695 All of the following possible causes of infertility, EXCEPT:
A Previous laparotomy for any reason
B Smoking
C High body mass index
D Uterine subserous fibroids
ANSWER:D
1696 The following value of semen analysis indicates abnormal semen quality:
A Volume less than 2 ml
B Count of 40 million / ml
C Motility 60%
D Abnormal form 30%
ANSWER:A
1697 Regarding human fertility: Which is true?
A Fertilization usually occurs 5-7 days before implantation and before the extrusion of the second polar body
B Subfertility only treated with IVF
C An adverse male factor is detectable in 60% of couples with low fertility
D Mumps in adulthood has no effect on male fertility
ANSWER:A
1698 The commonest cause for infertility
A Tubal obstruction
B Male factor
C Chronic anovulation
D Hyper androgens
ANSWER:B
1699 All the following are possible causes of anovulation, EXCEPT:
A High body mass index
B Anorexia nervosa
C Polycystic ovarian syndrome
D Sickle cell trait

ANSWER:D
1700 Polycystic ovarian diseases, all of the following can be seen, EXCEPT:
A Acne
B Streak ovaries
C Insulin resistance
D Hirsutism
ANSWER:B
1701 Induction of ovulation agents:
A Clomiphene citrate is an anti-androgen.
B Clomiphene citrate is given by injection.
C Gonadotropins is given orally.
D Causes multiple follicles.
ANSWER:D
1702 In polycystic ovary syndrome patients, all the followings are true EXCEPT:
A They are usually underweight with low body mass index.
B They have hirsutism.
C They have Oligomenorrhoea.
D They have high Prolactin level.
ANSWER:A
1703 Which of the following is suggestive of ovulation:
A Basal body temperature drop at least 0.5 C in the second half of the cycle
B Day 21 Estrogen level is elevated
C Progesterone level on day ten of the cycle is elevated.
D Regular cycle with dysmenorrhea.
ANSWER:D
1704 Which of the following is a basic investigation for the male infertility :
A Semen analysis.
B Sperm penetration assay of cervical mucous.
C Sperm penetration assay of hamster ova.
D Splits ejaculate analysis.
ANSWER:A
1705 Characteristic of normal semen analysis all true EXCEPT:
A Volume > 2ml

B pH of 7.2-7.8
C Sperm count > 2o million /ml.
D Normal sperm morphology is < 20%
ANSWER:D
1706 In Secondary infertility:
A Is frequently due to tubal problems
B The problem is often a chromosomal defect in women.
C Is frequently due to male factor
D Commonly associated with endometriosis.
ANSWER:A
1707 In an infertile couple all are correct EXCEPT:
A Fibroid can cause in fertility.
B Ovulation can be induced by Clomiphene.
C Gonadotropins can cause hyperstimulation.
D Polycystic ovary is a common problem.
ANSWER:A
1708 Regarding Hyperprolactenemia all are true EXCEPT:
A Can be drug induced
B Stress can play role
C Can cause infertility
D In case of pituitary adenoma surgical removal is the best choice
ANSWER:D
1709 Galactorrhea (non-gestational lactation) may result from all of the following EXCEPT:
A pituitary adenoma
B Hypothyroidism
C Renal failure
D Intrapartum hemorrhage
ANSWER:D
1710 The test used to diagnose ovulation on day 21 in a 28 days menstrual cycle is:
A Estrogen.
B FSH.
C Progestrone.
D LH.

ANSWER:D
1711 What is the association of ovulation induction for fertility and multiple births?
A Decreases the incidence multiple pregnancy.
B Increases the incidence multiple pregnancy.
C Increases the incidence of only dizygotic twins.
D Doesn't affect the incidence of twins.
ANSWER:B
1712 All of the following are true about PCO except:
A To diagnose PCO hormonal analysis can be done at any time of the cycle
B It is associated with reversed FSH:LH ratio
C It is associated with increase resistance to insulin
D Can be associated with increased prolactin level
ANSWER:A
1713 According to WHO, Sperm count shouldn't be less than :
A 50 million /ml
B 60 million/ml.
C 90 million/ml.
D 20 million/ml.
ANSWER:D
1714 PCO, all true EXCEPT:
A Hirsutism.
B Imperforate hymen
C Infertility.
D Acne.
ANSWER:B
1715
A woman with bilateral proximal tubal blockage which was diagnosed by Hysterosalpingiogram and her
husband have normal spermogram. There best chance to have a pregnancy is by:
A Doing a laproscopic opening of the tube and then expectant management.
B Giving the woman clomide for 6 months and expectant management.
C Doing controlled ovarian hyperstimulation and IUI.
D Doing IVF of the couple.
ANSWER:D
1716 Hyperprolactenemia:

A Associated with positive progesterone challenge test.
B Ovulation can be induced with dopamine antagonist.
C If macro-adenoma is the cause, the management is surgical.
D Galactorrhea is present in 30% of cases.
ANSWER:D
1717
The most likely cause of infertility in a 30 year old women, P2+4 who had salpingitis after her last abortion, 3
years ago is:
A Submucous fibroid.
B Asherman's syndrome (Uterine synechiae).
C Blocked tubes.
D Tuberculous endometritis.
ANSWER:C
1718 The following agents are used in ovulation induction in patients undergoing assisted conception, EXCEPT:
A Clomiphene Citrate.
B Human menopausal gonadotropin.
C GRH analogues.
D Aspirin.
ANSWER:D
1719 Regarding ovulation can be diagnosed by the following measures, EXCEPT:
A Measuring day 14 serum progesterone.
B Observing a rise in basal body temperature in the 2ND half of the menstrual cycle.
C Study of the cervical mucous.
D Endometrial histology.
ANSWER:A
1720 Hysterosalpingiogram (HSG) is contraindicated in the following EXCEPT:
A Suspicious pregnancy.
B PID.
C Congenital malformations of the uterus.
D Presence of abnormal uterine bleeding.
ANSWER:C
1721 Hirsutism can be found in all of the following conditions, EXCEPT:
A Polycystic ovary syndrome
B On Danazol therapy

C Adrenal hyperplasia
D Patient on oral contraceptive pills
ANSWER:D
1722 Regarding Hyperprolactenemia, all of the following are CORRECT EXCEPT:
A Can be drug induced.
B In cases on pituitary adenoma surgical removal is the best option.
C Stress can play a role.
D Can cause infertility.
ANSWER:B
1723 All of the following are true regarding TB infection EXCEPT:
A Is transmitted by hematogenous spread
B It causes infertility ,abnormal uterine bleeding ,pelvic pain
C It causes severe tenderness & cervical excitation on pelvic examination
D 2/3 have evidence of old pulmonary TB.
ANSWER:C
1724 Regarding Infertility:
A A patient who is P 0 +3 is defined as primary infertility.
B Painful menstruation suggests unovulatory menstrual cycles.
C Measuring progesterone level in the follicular phase is the diagnostic feature for ovulation.
D Including the husband in the investigation is essential.
ANSWER:D
1725 Recognized feature of Sheehan syndrome :
A Menorrhagia
B Galactorrhea
C Insulin resistance
D Hypothyroidism
ANSWER:D
1726 The earliest sign of Sheehan syndrome is :
A Secondary amenorrhea
B Failure of lactation
C Loss of axillary + pubic hair
D P.V bleeding
ANSWER:B

1727 In Sheehan syndrome, changes that take place include the following EXCEPT :
A Complete lactation failure
B Feeling of lethargy
C Genital atrophy
D Increased Basal Metabolic Rate (B.M.R)
ANSWER:D
1728
A 26 years old lady presented with secondary amenorrhea and FSH and LH are found to be high, your diagnosis
will be?
A Sheehan syndrome
B Asherman syndrome
C Premature ovarian failure.
D Imperforated hymen
ANSWER:C
1729 Patients with the following condition present with primary amenorrhea:
A Bicornuate uterus
B Polycystic ovary syndrome
C Imperforate hymen
D Sheehan’s Syndrome
ANSWER:C
1730 The presentation of Asherman's Syndrome typically involves:
A Hypomenorrhea and amenorrhea.
B Galactorrhea.
C Menorrhagia.
D Metrorrhagia.
ANSWER:A
1731 Secondary amenorrhea is a recognized feature of:
A Imperforated hymen.
B Testicular feminization syndrome.
C Pregnancy.
D Sickle cell anemia.
ANSWER:C
1732 Pituitary insufficiency & amenorrhea is best treated with :
A Low potent estrogen

B FSH with GnRH.
C Cyclic progesterone
D Clomide
ANSWER:B
1733 The following are true about Asherman syndrome EXCEPT:
A Usually causes primary amenorrhea
B Causes Secondary amenorrhea
C May result from excessive dilation & curettage
D Usually diagnosed by Hysterosalpingogram
ANSWER:A
1734 Feature characteristically associated with imperforated hymen in a 16 yearold girl include :
A Acute retention of the urine
B Absence of secondary sexual characteristics.
C Hirsutism.
D Short stature.
ANSWER:A
1735 Regarding Primary amenorrhea ,all the following are correct EXCEPT:
A Failure of breast development by age of 10 require investigation
B kalman syndrome is recognized cause
C it may be due to systemic disease
D turner is known cause
ANSWER:A
1736 In Sheehan syndrome ,changes that take place include all of the following EXCEPT:
A Genital atrophy
B Feeling of lethargy
C Complete lactation failure
D Increased basal metabolic rate
ANSWER:D
1737
A 48-year-woman who had two normal pregnancies 13 and 15 years ago presents with the complaint of
amenorrhea for 7 months. She expresses the desire to become pregnant again. After exclusion of pregnancy,
which of the following tests is next indicated in the evaluation of this patient's amenorrhea?
A Hysterosalpingogram.
B Endometrial biopsy.

C Thyroid function test.
D LH and FSH levels.
ANSWER:D
1738 In Polycystic Ovary Syndrome patients, all the followings are true EXCEPT:
A They are usually underweight with low body mass index.
B They have hirsutism.
C They have oligomenorrhea.
D They have high prolactin level.
ANSWER:A
1739 To diagnose a woman with PCOS you need to have :
A More than 10 cysts on each ovary measuring 2 mm.
B Hypomenorrhea or amenorrhea.
C Hyperandrogenism either biochemical or clinical.
D Only 2 of the above.
ANSWER:D
1740 Secondary amenorrhea may be due to the following, EXCEPT:
A Endometriosis.
B Thyrotoxicosis.
C Pregnancy.
D Anorexia.
ANSWER:B
1741 The ovary in polycystic ovarian syndrome characteristically shows :
A Fibrosis.
B Theca cells hyperplasia.
C Granulosa cells hyperplasia.
D Increased fatty infilteration.
ANSWER:B
1742 Which of the following features is associated with an imperforated hymen :
A Hirsutism.
B Closed spina bifida.
C Absence of secondary sex characteristics.
D Cyclic lower abdominal pain with primary amenorrhea.
ANSWER:D

1743 Polycystic Ovarian syndrome (PCOS):
A Estradiol levels are high.
B There is increased sensitivity to insulin.
C If Ovulation can be induced, the chance of conception is high.
D Androstenedione levels are high.
ANSWER:D
1744 Polycystic Ovarian syndrome:
A Estradiol levels are high.
B There is increased sensitivity to insulin.
C Has no change in FSH:LH ratio.
D Andostendione levels are high.
ANSWER:D
1745 The following is related to premature ovarian failure:
A There are decreased levels of estrogen.
B Is associated with autoimmune disease.
C There are decrease levels of FSH.
D It is seen in association with autoimmune disease.
ANSWER:A
1746 The following are know criteria for diagnosing Polycystic ovarian syndrome (PCOS), EXCEPT:
A Decreased body weight.
B Hormonal evidence of androgen excess.
C Chronic anovulation.
D Recurrent miscarriages.
ANSWER:A
1747 A Feature of Polycystic ovarian syndrome:
A Elevated level of serum FSH.
B Endometrial hyperplasia.
C Breast atrophy.
D Loss of androgen level.
ANSWER:A
1748 Secondary amenorrhea can be due to the following EXCEPT:
A Asherman syndrome.
B Sheehan's syndrome.

C Galactorrhea.
D Sickle cell anemia.
ANSWER:D
1749
A 19-year-old women comes to your office with a compliant of never having had menses. Physical examination
shows that she is 1.37 m tall, & weighs 42 kg. she lacks breast & pubic hair development. There is webbing of her
neck & cubitus valgus. Which of the following is likely to be true?
A Testicular feminization.
B Klinefelter syndrome.
C Turner's syndrome.
D Congenital adrenal hyperplasia.
ANSWER:C
1750 The most common mass associated with amenorrhea in a reproductive age women is:
A Follicular cyst.
B Corpus luteal cyst.
C Benign cystic teratoma.
D Pregnancy.
ANSWER:D
1751 Urge incontinence
A Is due to pelvic anatomic defect.
B Patient loses small amount of urine.
C Can be diagnosed with stress test.
D Can be treated medically.
ANSWER:D
1752 Which of the following factors predispose to genital prolapse:
A Repeated LSCS
B Multiparty
C Pelvic inflammatory disease
D Endometriosis
ANSWER:B
1753 The treatment of cystocele in a 32 years old may include the following, EXCEPT:
A Pelvic floor exercise.
B Weight loss.
C Vaginal hysterectomy.

D Anterior colporrhaphy.
ANSWER:C
1754 Urinary stress incontinence is:
A The loss is of large amount of urine when intra-abdominal pressure increases.
B The loss is of small amount of urine when intra-abdominal pressure increases.
C Inability to control the bladder of all urine
D Inability to pass urine
ANSWER:B
1755 In Urogenital prolapse, the following statement are correct EXCEPT:
A Urethrocele is prolapse of the urethra in the vagina.
B Obesity is the risk factor.
C Inguinal hernia is manifestation
D Rectoceles are associated with chronic constipation
ANSWER:C
1756 An Enterocele:
A Contain part of the peritoneum.
B Is lined by peritoneum
C Herniates into the bladder
D Can be treated medically
ANSWER:B
1757 Regarding Rectocele
A Can be treated medically
B Surgery is indicated in all patients
C Constipation is not a predisposing factor
D Should only be corrected if it is interfering with defecation and lifestyle
ANSWER:D
1758 True incontinence can result from the following EXCEPT:
A Pelvic irradiation
B Trauma
C Prolonged obstructed labor
D Epidural analgesia
ANSWER:D

1759
49 years old women comes in complaining that several years it feels as though "her organ are progressively
falling out her vagina" along with this, she complains of losing urine with straining, occasional urgency, and
sometimes a feeling of incomplete emptying of her bladder with voiding. On further examination you will find :
A Rectocele
B Cystocele
C Enterocele
D Complete uterine prolapse
ANSWER:B
1760
38yearsoldmultigravidawomencomplainsofthepainlesslossofurine, beginning immediately after coughing,
laughing, lifting, or straining. Immediate cessation of the activity stops the urine loss .this history is must
suggestive of :
A Fistula
B Stress incontinence
C Urge incontinence
D Urethral incontinence.
ANSWER:B
1761 The work up of patients with incontinence include all of the following EXCEPT:
A Culture & sensitivity
B Stress test
C pelvis U/S
D Cystometric study
ANSWER:C
1762 Detrusor instability should be managed by:
A Drug therapy.
B Electrical therapy.
C Operative therapy
D Psychiatric therapy.
ANSWER:A
1763 Clinical symptoms of uterine prolapse include the following EXCEPT:
A Cervical ulcer.
B Dyspareunia.
C Amenorrhea.

D Urinary retention.
ANSWER:C
1764 Stress incontinence: the following statement are correct EXCEPT:
A Is more common in older women.
B Can be treated with estrogen.
C More common in women after menopause.
D Is always isolated.
ANSWER:D
1765 Stress incontinence:
A Is more common in multiparus.
B Means passing urine frequently.
C Can be treated by antibiotic.
D Bladder drilling is an effective treatment.
ANSWER:A
1766 True incontinence can result from the following EXCEPT:
A Pelvic irradiation
B Trauma
C Prolonged obstructed labor
D Recurrent cystitis.
ANSWER:D
1767 An increased risk of Osteoporosis is associated with:
A Bromocriptine use.
B Excess glucocorticosteroid usage.
C Androgen excess in the female.
D Early menarche.
ANSWER:B
1768 The use of estrogen alone in menopausal women increase the risk of :
A Endometrial cancer
B Ovarian cancer
C Bone cancer
D Bowel cancer
ANSWER:A
1769 Recently the main risk of hormone replacement therapy after menopause is:

A fracture the neck of femur
B Cancer of the colon
C Hirsutism
D Cancer of breast
ANSWER:D
1770 In the menopause :
A Osteoporosis is common in black women
B They have increase tendency of fracture
C Post menopausal bleeding is always due to endometrial carcinoma
D HRT is always indicated
ANSWER:B
1771 Postmenopausal women:
A Malignancy is the commonest cause of postmenopausal bleeding.
B FSH and LH are characteristically low.
C Fibroid uterus tends to grow bigger.
D Hormonal replacement therapy increases the risk of breast cancer.
ANSWER:D
1772 Which of the following is an indication for estrogen treatment?
A Fibroid.
B Threatened abortion.
C Endometriosis.
D Postmenopausal atrophic vaginitis.
ANSWER:D
1773 The following is a symptom of Climacteric(menopause):
A obesity.
B Premenstrual tension.
C Hot flushes.
D Menorrhagia.
ANSWER:C
1774 Regarding hot flushes in the menopausal period. All the following are true, EXCEPT :
A They are reduced by exercise.
B They are less in obese women.
C They are more in smoking women.

D It usually disappears in 1 month.
ANSWER:D
1775 Which of the following statement is INCORRECT:
A Climacteric is the phase during which a woman passes form the reproductive stage to non-reproductive stage.
B Pre-menopause is the time during climacteric during which the periods becomes irregular.
C Menopause is the final menstruation which occurs during the climacteric.
D Premature menopause is cessation of menstruation before the age of 40.
ANSWER:B
1776 The severity of menopausal vasomotor symptoms correlates best with:
A Mental status at the time if climacteric.
B Rate of estrogen withdrawal.
C Serum estrogen concentration.
D Serum FSH concentration.
ANSWER:B
1777 Which of the following is NOT a physical finding associated with the climacteric:
A Atrophic vaginitis.
B Clitoromegaly.
C Vaginal stenosis.
D Small labia minora.
ANSWER:B
1778 Recognized feature of postmenopausal women is:
A Low FSH & LH.
B Increased bone mineral density.
C High FSH.
D High level of estrogen.
ANSWER:C
1779 After menopause: Which is true?
A There is increase in vaginal acidity
B Gonadotropins secretion falls.
C Recurrent vaginal bleeding should be investigated by endometrial biopsy.
D Malignancy is the leading cause of post menopausal bleeding.
ANSWER:C
1780 All the following are possible causes of menorrhagia, EXCEPT

A Uterine fibroid
B Adenomyosis
C Pelvic inflammatory disease
D Combined oral contraceptive pills
ANSWER:D
1781 After menopause
A There is increase vaginal acidity.
B Gonadotropins level falls.
C There is increase in bone density.
D Any postmenopaus al bleeding should be investigated by endometrial sampling.
ANSWER:D
1782 Menorrhagia is:
A Intermittent irregular vaginal bleeding
B Commonly presents as postmenopausal bleeding
C Heavy menstrual cycle more than 80 ml
D Infrequent spaced cycles every 45 days
ANSWER:C
1783 Amount of blood loss during the normal menses :
A 10 to 25 ml.
B 25 to 75 ml.
C 80 to 120 ml.
D 125 to 175 ml.
ANSWER:B
1784 Post menopausal bleeding :
A May be considered as a normal variation
B Ultrasound can exclude uterine malignancy
C Hysteroscopy is contraindicated
D Cervical cancer need exclusion.
ANSWER:D
1785 All the following are possible causes of menorrhagia, EXCEPT
A Uterine fibroid
B Combined oral contraceptive pills
C Pelvic inflammatory disease

D Endometrial hyperplasia
ANSWER:B
1786 Which statement regarding menstrual abnormality is correct ?
A Menorrhagia is defined as a "frequent period".
B Bleeding due to uterine polyps is called dysfunctional uterine bleeding.
C Oligomenorrhea means periods with more than 35 days apart.
D Menorrhagia means a "heavy period".
ANSWER:C
1787 Regarding Menorrhagia:
A Progesterone mediated IUCD can treat this condition.
B D & C is not required for diagnosis & management in a 40 years or more.
C In adolescent, it is important to rule out malignancy.
D Asherman syndrome is a main cause.
ANSWER:A
1788 Initial management for patient with post menopausal bleeding is:
A pap smear.
B Ultrasound.
C CT scan.
D History & physicalexamination.
ANSWER:D
1789 The most common cause of menstrual abnormality in reproductive-aged women is:
A Ectopic pregnancy.
B Uterine leiomyomas.
C Adenomyosis.
D An ovulation.
ANSWER:D
1790 The most common symptom of endometrial hyperplasia:
A Vaginal discharge & itching.
B Vaginal bleeding.
C Amenorrhea.
D Pelvic pain.
ANSWER:B
1791 Menorrhagia occurring at the age of puberty may be due to:

A Folic acid deficiency anemia.
B Congenital adrenal hyperplasia.
C Von Will brand’s disease. (Congenital coagulopathy)
D Androgen insensitivity disease.
ANSWER:C
1792 Regarding Adenomyosis:
A Is the presence of endometrial glands and stroma out side the uterus
B Can be diagnosed by D&C
C Can be detected by hysteroscopy
D Can cause severe dysmenorrhea
ANSWER:D
1793 The treatment of endometriosis include all the following EXCEPT:
A Birth control pills (BCP).
B Oral progesterone.
C Estrogen.
D Depoprovera.
ANSWER:C
1794 These are possible sites for endometriosis deposits EXCEPT:
A Brain.
B Peritoneum.
C Uterosacral ligaments.
D Ovaries.
ANSWER:A
1795 Regarding Adenomyosis, the following are true EXCEPT:
A Causes dysmenorrhea
B Usually associated with menorrhagia
C The uterine size not affected by the disease
D Hysterectomy is the treatment of choice to cure this condition.
ANSWER:C
1796 Secondary dysmenorrhea can be caused by all of the following EXCEPT:
A Endometrial polyp
B Endometriosis
C PID

D OCP
ANSWER:D
1797 Regarding Endometriosis :
A is the presence of endometrial tissue in the myometrium
B The liver is the most common site
C Implantation, lymphatic–vascular metastasis & coelomic metaplasia are the theories about the pathogenesis
D OCP are contraindicated as they worsen the symptom
ANSWER:C
1798 Pelvic endometriosis :
A Is a cause of 1ry dysmenorrheal
B Associated with excessive vaginal discharge
C Associated with prolonged use of IUCD
D Can be diagnosed by laparoscopy
ANSWER:D
1799 The following are theories of endometriosis EXCEPT:
A Retrograde menstruation
B Endometrial hyperplasia
C Immunological factor
D Coilemic metaplasia
ANSWER:B
1800 A woman with symptomatic endometriosis is likely to complain of all of the following EXCEPT:
A Dyspareunia.
B Mood swings.
C Painful defecation.
D Severe dysmenorrhea.
ANSWER:B
1801 Primary dysmenorrhea is most likely caused by:
A Uterine hypercontractility.
B Increased parity.
C High levels of estrogen.
D Coitus during menses.
ANSWER:A
1802 The following are theories for endometriosis, EXCEPT:

A Coelimic metaplasia.
B Endometrial hyperplasia.
C Retrograde menstruation.
D Immunological factor.
ANSWER:B
1803 Regarding Endometriosis, all of the following are CORRECT, EXCECPT:
A Is frequently associated with infertility.
B Causes deep dyspareunia.
C Is often asymptomatic.
D Causes postcoital bleeding.
ANSWER:D
1804 Endometriosis is :
A The presence of endometrial glands outside the uterus.
B The presence of endometrial stroma outside the uterus.
C The presence of endometrial glands & Stroma outside the uterus.
D A disease of menopause.
ANSWER:C
1805 Dyspareunia in endometriosis can be caused by the following EXCEPT:
A Endometriomas.
B Flexed retroversion of the uterus.
C Uterosacral implants.
D The use of the birth control pill.
ANSWER:D
1806 The operation of Endometriosis should be:
A Tailored to the patient's age, symptoms, & extent of disease.
B Total abdominal hysterectomy (TAH) & bilateral Salpingo-oophorectomy (BSO).
C Removal of implants only.
D Presacral neuroectomy.
ANSWER:A
1807 Each of the following can be a treatment modality for endometriosis EXCEPT :
A Depoprovera medroxy progesterone
B GnRH analogues
C Estrogen skin patches

D Oral contraceptive pills
ANSWER:C
1808
A 40 year-old woman complains of menorrhagia and dysmenorrhea that progressed gradually, the most likely
diagnosis is:
A Endometrial cancer.
B Adenomyosis.
C Cervical cancer.
D Ovarian cyst.
ANSWER:B
1809
The diagnosis of endometriosis is often strongly suspected from patient's initial history expressing the following
EXCEPT:
A Infertility
B Dysmenorrhea.
C Vaginal dryness.
D Dyspareunia.
ANSWER:C
1810 Premenstrual tension should be initially treated with:
A COCP.
B Hysterectomy.
C Aldosterone.
D Diuretics.
ANSWER:A
1811 Which of the following tumors is LEAST likely to be hormonally active:
A Sertoli-lyeding cell tumor.
B Granulosa cell tumor.
C Hair cell tumor.
D Fibroma.
ANSWER:D
1812 The most common malignancy in the female reproductive organs:
A Carcinoma of the cervix.
B Carcinoma of the ovary.
C Carcinoma of the breast.
D Carcinoma of the uterine corpus.

ANSWER:C
1813 Laparoscopy is used in the diagnosis of the following EXCEPT:
A Ectopic pregnancy.
B Endometriosis.
C Submucous fibroid.
D Tubal patency.
ANSWER:C
1814 The Commonest uterine fibroid to cause excessive bleeding is :
A Submucous fibroid.
B Subserous fibroid.
C Intramural fibroid.
D Cervical fibroid.
ANSWER:A
1815 Acceptable treatment for uterine fibroids includes all of the following EXCEPT:
A No treatment.
B Myomectomy during pregnancy if red degeneration occurs.
C Myomectomy.
D Hysterectomy.
ANSWER:B
1816 Uterine fibroid: Which is true?
A Is commoner in white people than black.
B All should be treated immediately.
C Sarcomatous change occurs in 1%.
D Can cause obstructed labor.
ANSWER:D
1817 The following is TRUE regarding degenerative changes in uterine fibroids :
A Sarcoma occurs in 2%.
B Cystic degeneration is common with the use of oral contraceptive pills.
C Hyaline degeneration causes acute pain.
D Red degeneration should be managed conservatively.
ANSWER:D
1818 The following are TRUE about Leiomyomas EXCEPT:
A Usually multiple.

B Usually malignant.
C Usually discrete.
D Usually spherical or irregular lobulated.
ANSWER:B
1819 Fibromyoma of the uterus:
A Is pre malignant tumor
B Symptoms are related to the site
C Estrogen dependent tumor
D Can be associated with endometriosis & endometrial hyperplasia.
ANSWER:C
1820 Uterine fibroid:
A Commonly associated with endometriosis
B Are tumors of Striated muscle.
C Commonly present with post menopausal bleeding
D progesterone dependent tumor
ANSWER:D
1821 Regarding red degeneration in fibroids, all are true EXCEPT:
A Common during pregnancy.
B Causes acute abdominal pain.
C Surgery is the 1 ST line treatment.
D Caused by ischemic necrosis.
ANSWER:C
1822 Regarding Submucous uterine fibroids all of the following are correct EXCEPT:
A May become polypoidal.
B Can become infected.
C Frequently cause infertility.
D Often present with menorrhagia.
ANSWER:C
1823 Regarding uterine leiomyomata may undergo the following changes EXCEPT:
A Hyaline degeneration.
B Squamous metaplasia.
C Atrophy.
D Calcification.

ANSWER:B
1824 Uterine leiomyomata generally require no treatment. When treatment is indicated, it is most frequently because of :
A Interference with reproductive function.
B Rapid enlargement with the hazards of Sarcomatous degeneration.
C Pain.
D Excessive uterine bleeding.
ANSWER:D
1825 These are possible symptoms caused by fibroids EXCEPT:
A Pelvic pain.
B Subfertility.
C Pressure symptoms.
D Deep vein thrombosis.
ANSWER:D
1826 True statement regarding leiomyomas is:
A The severity of the bleeding is proportional to the size and he number of myomas
B Intermenstrual pain is present in more than 50% of women with myomas that have not undergone degeneration
C Intermenstrual bleeding is the most common menstrual disorders
D The majority of the women with multiple myomas are fertile
ANSWER:D
1827
A 20 year old lady pregnant in 1st trimester came complaining of lower abdominal pain, in examination a mass
continued with the uterus was found what is the diagnosis:
A Red degeneration of fibroid
B Ectopic pregnancy
C Uterine rupture
D Rupture placenta
ANSWER:A
1828 Types of uterine fibroid degeneration include all EXCEPT :
A Red degeneration
B Yellow degeneration
C Calcification
D Hyaline degeneration

ANSWER:B
1829 Regarding uterine fibroid in pregnancy, all are true EXCEPT:
A Are often asymptomatic.
B May present with abdominal pain.
C May cause obstructed labor.
D Should be removed surgically if it becomes symptomatic.
ANSWER:D
1830 Which of the following is an early symptom of ovarian cancer?
A Pelvic pain.
B It's usually a symptomatic.
C Dysuria.
D Constipation.
ANSWER:B
1831 Ovarian neoplasm most commonly arise from:
A Ovarian epithelium.
B Ovarian stroma.
C Ovarian germ cells.
D Ovarian sex cords.
ANSWER:A
1832 Which of the following tumors is NOT an ovarian epithelial neoplasm:
A Serous tumor.
B Mucinous tumor.
C Endometrioid tumor.
D Endodermal sinus tumor.
ANSWER:D
1833 Metastatic tumors to the ovary rarely originate from the:
A Breast.
B Stomach
C Large intestine.
D Vagina.
ANSWER:D
1834 The following are Suspicious criteria for malignancy in ovarian tumors EXCEPT:
A Presence of Ascites.

B Presence of cachexia.
C Edema of the lower limbs.
D Elevated serum Estradiol.
ANSWER:D
1835 Ovarian cancer is more likely to occur in all of the following EXCEPT:
A Nulliparus women.
B Women who have breast cancer
C Patients with history of prolonged use of oral contraceptive pills.
D Women with a family history of ovarian cancer.
ANSWER:C
1836 Regarding Ovarian cysts in pregnancy:
A The dermoid accounts for up to 50% of total.
B They are commonly malignant in origin.
C Corpus luteal accounts for the majority of cases.
D Torsion does not occur during pregnancy.
ANSWER:C
1837 Which of the following ovarian tumors is the most common:
A Arrhenoblastoma.
B Granulosa cell tumors.
C Granulosa cell tumor.
D Mucinous Cystadenoma.
ANSWER:D
1838 In ovarian tumors all these are germ cell tumor EXCEPT :
A Teratoma
B Choriocarcinoma
C Endometrioid tumor
D Yolk sac tumor
ANSWER:C
1839 Suggestive ultrasound features of ovarian malignancy include following EXCEPT :
A Bilateral
B Presence of ascites
C Contain solid component
D Unilocular

ANSWER:D
1840 Which of the following tumors produces estrogen?
A Endodermal sinus tumors.
B Choriocarcinoma.
C Granulosa cell tumors.
D Dysgerminoma.
ANSWER:C
1841 The most common germ cell tumor is:
A Dysgerminoma.
B Endodermal sinus tumor.
C Embryonal carcinoma.
D Choriocarcinoma.
ANSWER:A
1842 Which of the following hormones is most likely to be produced by Granulosa-Cell tumors:
A Estrogen.
B Inhibin.
C Both of them.
D None of them.
ANSWER:A
1843 Which of the following ovarian tumors are thought to be derived from ovarian germinal epithelium:
A Dysgerminoma.
B Fibroma.
C Theca cell.
D Endometrioid.
ANSWER:D
1844 Which of the following tumors produces estrogen:
A Endodermal sinus tumors
B Choriocarcinoma
C Granulosa-celltumors
D Dysgerminoma
ANSWER:C

1844

!"#កទី ៥
Medicine 1673
No Question
1
A 22 year old medical student has presented to his GP because he is concerned that his urine
is dark: he’s worried that he has a urine infection. He has also had lower abdominal pain and
non-specific joint pain. He presented yesterday with a sore throat and headache. On urine dip
you note microscopic haematuria, alongside proteinuria. What investigation would be
diagnostic for the cause of his presentation?
A Renal ultrasound
B Urine protein: Creatinine ratio
C Antiglomerular basement membrane antibodies
D Renal biopsy
ANSWER:D
2
A 55-year-old male patient is admitted with a massive GI bleed. The patient is at risk for
what type of acute kidney injury?
A Post-renal
B Intra-renal
C Pre-renal
D Intrinsic renal
ANSWER:C
3 A common marker of chronic kidney disease (CKD) is
A rash.
B hematuria.
C proteinuria.
D bacteremia.
ANSWER:C
4
A female patient 40yo complains many months ago of palpitation crisis that started brutal
without particular triggered circumstance. Until now the palpitations has receded brutally and
spontaneously within some minutes, immediately followed by urine out put. She has
admitted for a lasting crisis with duration greater than 30min and accompanied by general
dizziness. BP was 90/70mmHg. ECG finding regular tachycardia with 200b/min, QRS
narrow and P waves negatives, retrogrades in D2, D3, aVF was seen 200sm after each QRS.
Question with a true answer What is a exact heart arrhythmia?

A Sinus tachycardia
B Basic atrial tachycardia
C Junctional tachycardia
D Atrial flutter
ANSWER:C
5
A male patient is transfer to private clinic with complaints of periorbital swelling which was
noticed 3 days ago. Urine dipstick is positive for protein. Blood test shows low albumin and
high cholesterol & normal creatinine. What is the most likely diagnosis?
A Orthostatic Proteinuria
B Transient Proteinuria
C Nephritic Syndrome
D Nephrotic Syndrome
ANSWER:D
6
A male patient was transfer to OPD with complaints of progressive swelling around the
eyes which has progressed to involve the legs. Urine is positive for nephrotic range
proteinuria but no hematuria. Serum albumin is low but he is otherwise well. He is diagnosed
as Nephrotic Syndrome. Which of the following will NOT be part of his initial management?
A Salt restriction
B Diuretics
C Bed rest
D Low fat diet
ANSWER:C
7
A patient 56yo, consults for exercise dyspnea that has developed many years ago, but
uncomfortable (rapid working on flat ground) for only few weeks. Heart auscultation at
mitral area hears systolic murmur 3/6, blowing, pans systolic, radiation to axillary. Physical
examination don’t fine peripheral signs of heart failure. ECG shows an atrial Fibrillation with
ventricular rate 100b/min. On chest x ray, we saw a cardiomegaly (ICT = 0.60),Bulge of
middle and inferior left portion with pulmonary hyper vascularization. A mitral regurgitation
was diagnosed. Question with simple complement What is a frequent sign of auscultation in
this disease and that not have in clinical description?
A Splitting S1

B Splitting S2
C Decreasing of S2’s intensity
D Presence of S3 gallop
ANSWER:C
8
A patient 56yo, consults for exercise dyspnea that has developed many years ago, but
uncomfortable (rapid working on flat ground) for only few weeks. Heart auscultation at
mitral area hears systolic murmur 3/6, blowing, pans systolic, radiation to axillary. Physical
examination don’t fine peripheral signs of heart failure. ECG shows an atrial Fibrillation with
ventricular rate 100b/min. On chest x ray, we saw a cardiomegaly (ICT = 0.60),Bulge of
middle and inferior left portion with pulmonary hyper vascularization. A mitral regurgitation
was diagnosed. Question with simple complement (One answer) Among of 5 auscultation
signs, the one is impossible to hear in this patient?
A Decreasing of S1
B Increasing of S2 at pulmonary area
C Presence of S3 gallop
D Presystolic rumbling
ANSWER:D
9
A patient 56yo, consults for exercise dyspnea that has developed many years ago, but
uncomfortable (rapid working on flat ground) for only few weeks. Heart auscultation at
mitral area hears systolic murmur 3/6, blowing, pans systolic, radiation to axillary. Physical
examination don’t fine peripheral signs of heart failure. ECG shows an atrial Fibrillation with
ventricular rate 100b/min. On chest x ray, we saw a cardiomegaly (ICT = 0.60),Bulge of
middle and inferior left portion with pulmonary hyper vascularization. A mitral regurgitation
was diagnosed. Question with simple complement In which functional NYHA classification
that can situate this patient?
A Class 0
B Class 1
C Class 2
D Class 3
ANSWER:C

10
A patient is suspected of having nephrotic syndrome due to a health history of Lupus. As the
nurse you know that what substance(s) will be present in the urine to confirm this diagnosis?
A Red blood cells and mild protein
B Massive red blood cells and moderate protein
C Massive protein
D Elevated potassium and sodium
ANSWER:C
11
All investigations for diagnosis of stable angina are recommended One false answer, which
one?
A ST depress and T inversed
B Stress test
C Artery coronary angiography
D Blood test (Troponin)
ANSWER:D
12 Causes of MR All answer are trues, except one, which one?
A Rheumatic fever is not frequent
B Ischemic heart disease
C Valve prolapse
D Functional
ANSWER:A
13 Causes of MS All answer are trues, except one, which one?
A Rheumatic fever
B Calcification
C Lutembacher (SVD+MS)
D Congenital
ANSWER:C
14 Chest X ray in MS One false answer, which one?
A Middle portion of left contour is normal
B Vascular redistribution at top of lung
C Interstitial edema
D Alveolar edema

ANSWER:A
15 Complications of LHF One true answer, which one
A Acute pulmonary edema
B No thrombus
C No heart arrhythmia
D Endocarditis
ANSWER:A
16 Complications of MR One false answer, which one?
A Left heart failure
B Thrombus
C Pulmonary embolism
D Heart arrhythmias
ANSWER:C
17 Complications of MS One false answer, which one?
A Acute pulmonary edema
B Thrombus
C Atrial fibrillation
D Pulmonary embolism
ANSWER:D
18 Definition of HTN One false answer, which one?
A BP ≥ 140/90mmHg
B BP ≥ 160/80mmHg
C BP ≥ 130/90mmHg
D BP = 120/80mmHG
ANSWER:D
19 Dysentery:
A The stools often contain blood or pus and can be of small volume or large?
B Liquid stools?
C Melena stools?
D All of the above
ANSWER:A
20 ESRD is commonly defined as a glomerular filtration rate (GFR) less than

A 15 mL/min/1.73 m2.
B 25 mL/min/1.73 m2.
C 35 mL/min/1.73 m2.
D 45 mL/min/1.73 m2.
ANSWER:A
21 Etiologies of left heart failure One true answer, which one?
A IHD + HTN 40%
B HTN 25%
C IHD 13%
D Other 12%
ANSWER:A
22 Except one below caused by Portal hypertension in liver cirrhosis?
A Hepatomegaly
B Splenomegaly
C Esophageal varices
D Gastropathy
ANSWER:A
23 Factors precipitating portosystemic encephalopathy except:
A High dietary protein
B Gastrointestinal haemorrhage
C Constipation
D Diarrhea
ANSWER:D
24 Hallmark sign of liver cirrhosis?
A Spider angioma
B Purpura
C Ecchymosis
D Pruritus
ANSWER:A
25 Heart ultrasound All answer are trues, except one, which one?
A Left atrium enlarge
B Calcification of mitral valve

C Surface area is decreased
D Most often absence TR
ANSWER:D
26 Hemodialysis can be provided via
A arteriovenous graft.
B arteriovenous fistula.
C temporary venous catheter.
D All of the above
ANSWER:D
27 How frequently should subjective global assessments (SGAs) be completed on CKD patients?
A Monthly
B Quarterly
C Semiannually
D Annually
ANSWER:B
28
In a patient with diabetic nephropathy and proteinuria, which of the following is not
associated with the rate of decline in GFR?
A Glycated haemoglobin (HbA1c) concentration
B Mean arterial pressure
C Serum bicarbonate
D Serum total CO 2
ANSWER:A
29 In ischemic heart disease, all answers are false, except one is true?
A Affected young person
B Most often without risk factors
C On healthy coronary artery
D Included: stable angina and acute coronary syndrome
ANSWER:D
30
In patients who are experiencing acute glomerulonephritis, the glomerulus is permeable to
what substances?
A Red blood cells and protein

B Protein and white blood cells
C Red blood cells, protein, and lipids
D Proteins
ANSWER:A
31 Indication of coronary artery angiography All answers are trues, except one, which one?
A STEMI is developing
B STEMI developed but recurrent chest pain
C Recurrent chest pain on patient with history MI
D NSTE ACS (Unstable angina)
ANSWER:D
32 Intrarenal ARF may be recognized by a distinctive presentation of
A orthostatic hypotension.
B edema of the extremities.
C very high urine osmolality.
D brownish, muddy appearance of urine.
ANSWER:C
33 Location deep septal One true answer, which one?
A V1,V2,V3
B V1,V2,V3,4
C V1,V2,V3,V4,V5,V6
D D2,D3,aVF,V1,V2,V3
ANSWER:D
34
Male patient 62yo, 70kg, has a past medical history HTN 5 years and dyspnea on exercise 1
year ago. Current treatment as digoxine 1c/d, Natrilix 1c/d and diet.This patient has called in
emergency for brutal developing of dyspnea. Physical examinationfound that BP
170/110mmHg, HR 138/min irregular. Respiratory rate 32/min, T⁰ 37⁰, conciseness is
normal, cyanosis, sweet, systolic murmur 3/6, left gallop, good urine out put just arrival.
Lung auscultation findingwheezing and crackle rale bilateral at lower lobes. Question with a
true answer An element of clinical feature is compatible with:
A Pulmonary embolism
B Asthma crisis
C Cardiogenic shock

D Left heart failure
ANSWER:D
35
Male patient 62yo, 70kg, has a past medical history HTN 5 years and dyspnea on exercise 1
year ago. Current treatment as digoxine 1c/d, Natrilix 1c/d and diet.This patient has called in
emergency for brutal developing of dyspnea. Physical examinationfound that BP
170/110mmHg, HR 138/min irregular. Respiratory rate 32/min, T⁰ 37⁰, conciseness is
normal, cyanosis, sweet, systolic murmur 3/6, left gallop, good urine out put just arrival.
Lung auscultation findingwheezing and crackle rale bilateral at lower lobes. Question with a
true answer Heart murmur can correspond on:
A MR organic
B MR functional related to LV cavity dilated
C MS
D Anemia
ANSWER:B
36 Measurement condition of BP One true answer, which one?
A One measure
B Immediately after exercise arrest
C Unilateral
D 10min after exercise arrest, bilateral, 2-3 measures, sleeping and standing position
ANSWER:D
37 Mechanism of skin changes in liver cirrhosis?
A Liver failure
B Liver steatosis
C Stellate cell activation
D All of the above
ANSWER:A
38 Mitral valve stenosis (MS) One true answer, which one?
A Incomplete closing of mitral valve
B During systole
C Most common cause is carcinoide syndrome
D Incomplete opening of mitral valve during diastole
ANSWER:D

39 Paracentesis?
A Extraction of ascitic fluid by needle
B Extraction of pleural fluid by needle
C Extraction of pericardial fluid by needle
D Extraction of synovial fluid by needle
ANSWER:A
40 Paraclinic tests of acute pancreatitis:
A Blood tests are not helpful?
B Ultrasonography can be detected?
C Urines analysis?
D Hemoculture?
ANSWER:B
41
Question with multiple complements (Multiple answers) What are the true answers in
Ventricular tachycardia’s ECG?
A Irregular tachycardia
B Wide QRS
C LBBB aspect with right axis
D Unresponsive to vagal maneuver
ANSWER:A
42
Question with one false answer All following proposals of multifocal atrial tachycardia are
trues, excepts one?
A MAT is a multiple sites of competing atrial activity
B Irregular ventricular rate greater than 100b/min
C Organized and discrete P waves with at least 3 different morphologies in the same lead.
D Cardioversion is not contraindicated in MAT.
ANSWER:D
43 Question with one false answer in AF
A Paroxysmal AF if duration less than a week
B Cardioversion by external electrical shock in AF with unstable hemodynamic situation
C Using Aspirin in AF with rheumatic fever origin is absolute recommended
D Digoxin therapy doesn’t use in AF with WPW syndrome
ANSWER:C

44
Question with simple complement (One answer) All answers are true in A Flutter’s ECG,
except one?
A Ventricular rate is often regular
B A-V conduction is often 2/1 if HR 150b/min
C With an iso-electric line return between F wave
D
Atrial waves called F wave appearance saw tooth, consisting of negative and positive phases.
Rapid up sloping and slow down sloping.
ANSWER:C
45
Question with simple complement (One answer) All answers are true in PVC’s ECG
according to malignant prognostic, except one?
A A lot of PVC
B Fixed and long coupling
C Repetitive, increased by effort
D Multifocal
ANSWER:B
46
Question with simple complement (One answer) All answers are true in PVC’s ECG, except
one?
A Ectopic beat develops early
B Aspect RBBB if ectopic focus located in right ventricle
C T wave axis is always opposite to QRS
D Wide and deformity QRS
ANSWER:B
47
Question with simple complement (One answer) All answers are true in PVC’s ECG
according to benign prognostic?
A Variable coupling
B Unifocal
C Decreasing PVC number when exercise
D On healthy heart
ANSWER:A
48
Question with simple complement (One answer) All answers in AF’s ECG are true, except
one?
A Undulation P wave in D2, D3 and V1

B Normal AF is ventricular rate between 60-100/min
C Narrow QRS duration if associated with WPW or BBB
D Ventricular rhythm is irregular
ANSWER:C
49
Question with simple complement (One answer) All answers in Ventricular tachycardia’s
ECG are true, except one?
A Regular tachycardia with wide QRS more than 100/min
B Different morphology of PVC
C Aspect mono or biphasic QRS
D Capture complex
ANSWER:B
50
Question with simple complement (One answer) All characteristic are true of ST segment,
except one?
A Early ventricular repolarization
B Horizontal aspect depression in injury
C Ending of QRS wave and ending of T wave
D Can be slightly elevated (up to 2.0mm in some precordial leads)
ANSWER:C
51
Question with simple complement (One answer) All characteristic P wave are true, except
one ?
A Depolarization of atriums
B Duration of ≤ to 0.11 seconds. Amplitude < 2.5mm in D2
C Axis between +0 and +90◦
D Shape is not smooth, notched or peaked
ANSWER:D
52
Question with simple complement (One answer) All Echocardiography’s parameters are true,
except one, which one?
A Aorta root 20-37mm
B Left atrium diameter 20-40mm
C Interventricular septum diastolic 6-12mm
D Left ventricle diastolic diameter 37-55mm
ANSWER:C

53
Question with simple complement (One answer) All risk factors of atherosclerosis are true,
except one, which one?
A High Blood Pressure
B Absence of cigarette smoking
C High levels of blood sugar
D High levels of cholesterol
ANSWER:B
54
Question with simple complement (One answer) All the auscultation elements are in alone
mild MS, except one?
A Opening snap
B S1 increased
C Diastolic rumbling with presystolic strengthening
D S2 pulmonary normal if pulmonary artery hypertension
ANSWER:D
55
Question with simple complement (One answer) Among of different etiologies of MR, what
is a false answer?
A Rheumatic fever
B Myocardial infarction
C Chest trauma
D Syphilis
ANSWER:D
56
Question with simple complement (One answer) In AV block 2nd high, all answers are true,
except one, which one?
A PR interval normal or prolong but constant
B PP interval regular
C At least one P wave blocked can pass to 2-3 or 4 blocked
D QRS can be always wide
ANSWER:D
57
Question with simple complement (One answer) In AV block 3rd degree, all answers are
true, except one, which one?
A PR interval is variable
B Narrow QRS if lower Hiss

C At least one P wave blocked can pass to 2-3 or 4 blocked
D Torsade de pointe
ANSWER:B
58
Question with simple complement (One answer) In LVH all answers are true, except one,
which one?
A Left axis
B Sokolow indice is SV1 + RV5-6 > 35mm
C Cornell indice is SV3 + RaVL < 20mm
D Systolic overload if T negative
ANSWER:C
59
Question with simple complement (One answer) In regular rhythm, what is a true answer for
calculation of HR?
A Calculation of HR in 6 second
B 1500 divided by number Small Square
C Electrical impulse originates from ectopic focus
D 300 divided by number big square
ANSWER:A
60
Question with simple complement (One answer) In sinus rhythm, all answers are true, except
one?
A PR interval normal or prolong
B Normal axis P wave
C Electrical impulse originates from ectopic focus
D All QRS are preceded by P wave
ANSWER:C
61
Question with simple complement (One answer) In supra ventricular tachycardia (AVNRT)
all answers are true, except one, which one?
A QRS wave are thin or wide
B Irregular tachycardia with HR 160-230/min
C P wave retrograde, negative in D2, D3 and aVF
D Sometime invisible or nearly from QRS
ANSWER:B

62
Question with simple complement (One answer) What are the true answers according to
center Venus pressure (CVP)?
A Decreased CVP in cardiogenic shock
B Relatively decreased in vagal shock
C Decreased CVP in hypovolemic shock
D Normal in anaphylactic shock
ANSWER:A
63
Question with simple complement (One answer) What is a false answer in left atrial
hypertrophy?
A Prolonged P wave duration in D2
B Can call mitral P wave
C Tall and sharp P wave in D2
D Changed second portion P wave in V1
ANSWER:C
64 Question with simple complement (One answer) What is a false answer in PAC’s ECG ?
A Not preceding P wave before QRS
B Mostly narrow QRS
C Sometime blocked PAC
D Wide QRS if PAC with aberration conduction
ANSWER:A
65
Question with simple complement (One answer) What is a false answer in PVC’s ECG
according to benign prognostic?
A Isolated
B On heart disease
C ROSENBAUM aspect
D Unifocal
ANSWER:B
66
Question with simple complement (One answer) What is a false answer in right atrial
hypertrophy?
A Bifid P wave in D2
B Can call pulmonary P wave
C Increased amplitude P wave

D Changed first portion P wave in V1
ANSWER:A
67 Question with simple complement (One answer) What is a false answer of atherosclerosis?
A Signs and symptoms will not depend on which arteries are affected.
B The plaques of atherosclerosis cause the three main kinds of cardiovascular disease
C Atherosclerosis usually causes no symptoms until middle or older age
D Angina if coronary artery is affected
ANSWER:A
68 Question with simple complement (One answer) What is a false characteristic of T wave?
A Isolated T wave inversion in an asymptomatic adult is generally a normal variant.
B Ending of ventricular repolarization
C Symmetric with pointed top
D (+) D1, D2, D3 and V3 to V7- (-) in V1, V2, aVR and aVF - (-) or di-phasic in D3
ANSWER:C
69 Question with simple complement (One answer) What is a true QRS axis?
A -10 and -90◦
B +80 and ± 180◦
C -90 and -120◦
D -10 and +80◦
ANSWER:D
70 Question with simple complement (One answer) What is false characteristic of PR interval?
A Duration of electrical impulse transmission from SAN to AVN
B Normal PR duration is 0.12-0.20s
C Measure from ending P valve to beginning QRS
D Prolong PR interval in AVB
ANSWER:C
71
Question with simple complement (One answer)All symptoms are true in cardiogenic shock,
except one?
A Low blood pressure (SBP &lt; 90mmHg), tachycardia, weak pulse
B Sweating, pale skin, cold hands or feet
C Absence of sign and symptoms of heart failures (LHF, RHF)
D Cardiopulmonary arrest is the same of heart attack

ANSWER:C
72
Question with simple multiple complement (One answer) What is a false answer in
myocardial injury?
A Affects ST segment
B Develops two hours after chest pain
C Absence reciprocal signs
D ST elevated in injury subepicardic
ANSWER:C
73
Select the most common type of medications that may be ordered by a physician to treat
nephrotic syndrome:
A Cardiac glycosides
B Corticosteroids &amp; Diuretics
C Antibiotics
D Antihypertensives
ANSWER:B
74 Signs of right heart failure All answer are trues, except one, which one?
A Jugular vein distension (JVD)
B Crackle bilateral
C Hazer sign positive
D Hepatomegalia
ANSWER:B
75 Stage 4 CKD may require the use of exogenous erythropoietin to manage
A anemia.
B neutropenia.
C pancytopenia.
D thrombocytopenia.
ANSWER:A
76 STEMI All answer are trues, except one, which one?
A Chest pain at rest
B Duration over 30min
C ST segment elevated tombstoning aspect
D Troponin positive

ANSWER:D
77 Symptoms of MS One false answer, which one?
A Diastolic rumbling
B Opening snap
C S1 increased
D Systolic thrill
ANSWER:D
78 The acute coronary syndrome One false answer, which one?
A Chest pain on exercise
B STEMI
C Unstable angina (NSTE ACS with troponin negative)
D NSTEMI
ANSWER:A
79 The leading cause of death among patients with ESRD is
A uremia.
B anemia.
C liver failure.
D cardiovascular complications.
ANSWER:D
80 The term 'portosystemic encephalopathy' (PSE) refers to
A A chronic neuropsychiatric syndrome secondary to chronic liver disease.
B A chronic neuropsychiatric syndrome secondary to chronic kidney disease.
C A chronic neuropsychiatric syndrome secondary to chronic heart disease.
D A chronic neuropsychiatric syndrome secondary to chronic neurologic disease.
ANSWER:A
81 The two leading causes of end-stage renal disease (ESRD) are
A allergies and diabetes.
B infection and diabetes.
C diabetes and hypertension.
D infection and hypertension.
ANSWER:C

82
The use of angiotensin-converting enzyme (ACE) inhibitors has been shown to be beneficial
in patients with
A lupus.
B hypotension.
C renal artery stenosis.
D diabetic nephropathy.
ANSWER:D
83 Triggered factors of left heart failure One true answer, which one?
A Respiratory infection
B No stop treatment
C Regime no riche in salt
D Absence of heart arrhythmia
ANSWER:A
84 Urea cycle take place in:
A Liver
B Kidney
C Lungs
D GI tract
ANSWER:A
85 What are the cardio-vascular complications of HTN, except one is false?
A Left ventricular hypertrophy (LVH)
B Ischemic heart disease (IHD)
C Heart failure
D Atrial-ventricular block
ANSWER:D
86 What function of the kidneys is responsible for water homeostasis?
A Excretory
B Metabolic
C Endocrine
D Digestive
ANSWER:A
87 What is a diastolic HTN One true answer, which one?

A BP ≥ 160/90mmHg
B SBP ≤ 160 and DBP ≥ 90mmHg
C BP at time the heart contracts
D Most often in young person
ANSWER:D
88 What is an urgency hypertensive One true answer, which one?
A Chronic elevation BP
B BP ≥ 180/110mmHg increased suddenly
C With organ damages
D Using of long acting antihypertensive drug
ANSWER:B
89 What is masked HTN One false answer, which one?
A BP is high at cabinet’s doctor
B Has complication higher than permanent HTN
C BP is increased at patient’s house
D Need self measurement BP
ANSWER:B
90 Which of the following can cause a person to develop CKD?
A Chronic condition such as diabetes or hypertension
B Taking medications as prescribed
C Seeking medical treatment in a timely manner
D Maintaining optimal blood sugar and blood pressure control
ANSWER:A
91 Which of the following drugs is a common cause of interstitial nephritis?
A Metformin
B Ranitidine
C Lithium
D Omeprazole
ANSWER:D
92 Which of the following is not a feature of the nephrotic syndrome?
A Proteinuria greater than 5 g/ day
B Hypercholesterolaemia

C Microscopic haematuria
D Peripheral oedema
ANSWER:C
93 Which of the following is not a typical symptom of kidney failure?
A Insomnia
B Hallucinations
C Itching
D Restless legs
ANSWER:B
94
You are reviewing the latest lab results for a patient with stage 4 CKD with metabolic bone
disease. He currently doesn’t take any type of phosphate binders, calcium supplements, or
vitamin D therapy. His current lab results are phosphorus: 8 mg/dL; calcium: 9 mg/dL, and
parathyroid hormone: 400 pg/mL. What course of treatment would you recommend?
A Initiate calcium supplementation
B Initiate vitamin D3 supplement
C
Initiate vitamin D sterols, initiate phosphate binder therapy, and educate on dietary
restrictions
D Educate on restricting phosphorus in diet only
ANSWER:C
95
You are reviewing the latest lab results for a patient with stage 4 CKD with metabolic bone
disease. He currently doesn’t take any type of phosphate binders, calcium supplements, or
vitamin D therapy. His current lab results are phosphorus: 8 mg/dL; calcium: 9 mg/dL, and
parathyroid hormone: 400 pg/mL. What course of treatment would you recommend?
A Initiate calcium supplementation
B Initiate vitamin D3 supplement
C
Initiate vitamin D sterols, initiate phosphate binder therapy, and educate on dietary
restrictions
D Educate on restricting phosphorus in diet only
ANSWER:C
96 Celiac disease is not triggered by ingestion of:

A Wheat
B Barley
C Rye
D Cassava
ANSWER:D
97 choose the wrong one of the following sentences the three mains subtypes ischemic
A thrombosis
B Embolism
C systemic hypoperfusion
D pleural effusion
ANSWER:D
98
Choose the wrong one of the following The celebrovascular disease is cause by one of the
several pathology process involve the blood vessel in the brain except
A the process may be intrinsic to the vessel, the process may be originate remotely
B
the process may result from inadequate blood flow due to decrease perfusion or increase
viscosity
C the process may result rupture from a vessel in the subarachnoidd apace intracerebral tissue
D the process may have an abundance of neurofibrillary tanles
ANSWER:C
99 Definition of hypertriglyceridemia
A Triglyceridemia > 150mg/dl
B Triglyceridemia >300mg/dl
C Triglyceridemia >100mg/dl
D Triglyceridemia > 500mg /dl
ANSWER:A
100 Definition of normal weight for Asian people
A BMI > 18.5kg/m2 and < 23kg/m2
B BMI > 18 kg/m2 and < 25 kg/m2
C BMI > 18 kg/m2 and < 30 kg/m2
D BMI > 18 kg/m2 and < 35 kg/m2
ANSWER:A

101 Definition of peripheral obesity for Asian population
A BMI > 23kg/m2
B BMI > 25 kg/m2
C BMI > 30 kg/m2
D BMI > 35 kg/m2
ANSWER:A
102 Direct Coombs’ test positive means:
A Patient’s red cells are coated with autoantibody
B Patient’s serum contains autoantibody
C Patient’s red cells are full of antigens
D Patient’s blood is septic
ANSWER:A
103 Function of HDL cholesterol (metabolism of HDL.C)
A Transports triglyceride to cells
B Transports cholesterol to cell
C Transports amino-acids to cells
D Scavenger of cholesterol
ANSWER:D
104 Grave’s disease resulted from stimulating of TSH receptor by:
A TSH
B Auto antibody
C Pathogen
D Thyroid hormone
ANSWER:B
105 Hydrogenations of fats
A Transform unsaturated fat to saturated fat
B Transform saturated to unsaturated fat
C Protect from cardiovascular disease
D Protect from dyslipidemia
ANSWER:A
106 Hyper LDL.C causes
A Atherosclerosis

B Pancreatitis
C Myocarditis
D All of the above
ANSWER:A
107 Immunization is:
A artificial acquired passive immunity
B artificial acquired active immunity
C natural acquired active immunity
D natural acquired passive immunity
ANSWER:B
108 In Grave’s disease the most accurate diagnosis is:
A Thyroid functional test
B Radioactive iodine imaging
C Thyroid hormone level
D Thyroid Stimulating Receptor antibodies level
ANSWER:D
109 In Myasthenia Gravis what are CT and MRI of the chest performed to look for?
A Mediastinal tumour
B Lung metastasis
C Pneumonia
D Abnormal thymus
ANSWER:D
110 Innate immune response to viral infection is:
A antiviral antibodies
B cytotoxic T lymphocytes
C helper T cells
D epithelial barrier
ANSWER:D
111 Lipids digestion
A Takes place in stomach
B Takes place in duodenum where the junction of distal common bile duct and pancreatic duct

C Takes place in caecum
D Takes place in colon
ANSWER:B
112 One egg approximately contains cholesterol
A 1000 mg
B 500 mg
C 300 mg
D 186 mg
ANSWER:D
113 Our body approximately contains triglyceride and cholesterol respectively
A 99% and 1%
B 90% and 10%
C 80% and 20%
D 70% and 30%
ANSWER:A
114 Our body approximately needs cholesterol per day
A 1000mg
B 500mg
C 300mg
D 100mg
ANSWER:C
115 Physiologic hypogammaglobulinemia of infancy usually happens:
A from birth to 3 months old
B between 3 and 6 months old
C between 6 and 9 months old
D after 9 months old
ANSWER:B
116 Source of chylomicron
A Liver
B Kidney
C Intestine
D All of the above

ANSWER:C
117 Source of VLDL
A Liver
B Kidney
C Intestine
D All of the above
ANSWER:A
118 Splenectomy is one of the treatments in warm Autoimmune Hemolytic Anemia because:
A the spleen is too large
B there is hemorrhage inside the spleen
C there is extravascular hemolysis inside the spleen
D the spleen is toxic
ANSWER:C
119 the below clinical sings of multiple sclerosis except
A visual and oculomotor abnormalities
B paresthesias, weakness, spasticity
C seizure and headache
D urinary dysfunction, mild cognitive impairment
ANSWER:C
120 the below clinical sings what are the common symptom of multiple sclerosis
A visual and oculomotor abnormalities
B paresthesias, weakness, spasticity
C urinary dysfunction, mild cognitive impairment
D All of the above
ANSWER:D
121 the following below are the medication treatment for multiple sclerosis except
A corticosteroid for acute exacerbation
B immunomodulators to prevent exacerbation
C gabapentin or tricyclic for anti depression
D antibiotic for inflammation and infection
ANSWER:D
122 The following statement of the Cause of Crohn’s disease, all are true except:

A Unknown.Host factor,Immunologic factor
B Bacteria and viruses also may play a major role
C Environmental factors,Smokers
D TB
ANSWER:D
123 The following statement of the Complication of Crohn’s disease, all are true except:
A Obstruction or blockage of the intestine due to swelling and the formation of scar tissue.
B Toxic megacolon..
C Fistulas that connect different parts of the intestine. Rectovaginal
D Vaginal bleeding or hematuria
ANSWER:D
124 The following statement of the Diagnostic of GERD disease, all are true except:
A A positive stool guaiac /Stool antigen HP.
B Endoscopy showing ulceration or inflammation of the esophagus.
C A barium swallow showing reflux
D Stool microscopic
ANSWER:D
125 The following statement of the treatment of Crohn’s disease, all are true except:
A Drugs, nutritional supplements, surgery or a combination of all three.
B Some people have long remission times.
C Depending on the location and severity of the disease
D Albendazol.
ANSWER:C
126 The following statement of the treatment of GERD disease, all are true except:
A Albendazol
B Elevate head of bed during sleeping, Weight loss, stop smoking.
C
Avoid food: coffee, chocolate, alcohol, fatty food , oranges, tight clothes, anticholinergic
drugs (atropin, hyoscine), ca+blocker, nitrate.
D Medication H2 blocker, Proton pump inhibitors.
ANSWER:A
127 The following statement of the diagnosis of Crohn’s disease, all are true except:

A
History and physical exam,laboratory tests-Blood culture (if fever), CBC, lyte, urea,
creatinine, transaminase, hept B, HCV, iron, stool microscopie, urine microscopie.
B X-ray tests may include CXRAY, barium studies of the upper and lower GI tract.
C Endoscopy tests may include flexible sigmoidoscopy and sometimes, colonoscopy.
D Stool microscopie
ANSWER:D
128 The following statements about Definition of BUDD-CHIARI SYNDROME are true, except:
A
BCS can be defined as any pathophysiologic process that results in interruption of the normal
blood flow out of the liver .
B BCS is the diminution of the normal blood flow out of the liver.
C
BCS implies thrombosis of the hepatic veins and/or the intrahepatic or suprahepatic inferior
vena cava.
D BCS-can be physiologic process that the normal blood flow out freely from the liver.
ANSWER:D
129 What are mechanisms of innate immunity?
A Mechanical barriers and surface secretion
B Humoral mechanisms
C Cellular defence mechanisms
D All of the above
ANSWER:D
130 What are the most propable causes of Systemic Lupus Erythematosus?
A Genetic influence
B Hormones
C Environmental factors
D All of the above
ANSWER:D
131 What is the characteristic of adaptive immunity?
A Effectiveness in killing pathogen
B Rapid immune response
C Specific and memory
D Live long immunity

ANSWER:C
132 What is the common etiologic of primary immunodeficiency?
A Infection at the early age
B Acute infection
C Malnutrition
D Gene defect
ANSWER:D
133 What is the gold standard method for HIV screening?
A Polymerase Chain Reaction
B Rapid testing
C ELISA
D Cell culture
ANSWER:C
134 What is the most common AIDS-associated cancer?
A Lund cancer
B Invasive cervical carcinoma
C Non-hodgkin’s lymphoma
D Kaposis’s sarcoma
ANSWER:D
135 What is the most common cause of Secondary Immunodeficiencies ?
A HIV
B Severe malnutrition
C Metabolic disorders
D Immunosuppressive drugs
ANSWER:B
136 What is the most common deficiency that causes Primary Immunodeficiency
A Antibody
B Lymphocytes
C Phagocytes
D Complement
ANSWER:A
137 What is the most commonly recognized symptoms of Celiac Disease?

A Dermatitis herpetiformis
B Extrinsic allergic alveolitis
C Malabsorption of food
D Peripheral neuropathy or epilepsy
ANSWER:C
138 What is the most dangerous clinical manifestation of Systemic Lupus Erythematosus?
A Lupus nephritis
B Cardiopulmonary
C Infection
D Masculoskeletal
ANSWER:A
139
What is the most effective drugs in suppressing HIV replication and prolonging the life of
HIV infected individual?
A Reverse transcriptase inhibitors
B Protease inhibitors
C Non-nucleoside reverse transcriptase inhibitors
D All of the above
ANSWER:D
140 What is the most effective treatment of Celiac Disease?
A Electrolite and fluid replacement
B Calcium and vitamin D administration
C Vitamin and mineral supplementation
D Gluten free diet
ANSWER:D
141
What is the most important family history to be obtained for the diagnosis of primary
immunodeficiency?
A A pedigree chart to identify a hereditary pattern
B A history of adverse reactions to immunizations or viral infections
C A history of prior surgery (eg, splenectomy, tonsillectomy, adenoidectomy)
D
A history of radiation therapy to the thymus or nasopharynx, and prior antibiotic and
immune globuline (Ig) therapies
ANSWER:A

142 Which component that plays roles in both innate and adaptive immunity?
A Natural killer cells
B Interferons
C Cytokines
D Complement
ANSWER:C
143 Which of the following is the Cause of GERD all true except:
A Certain foods (eg, coffee, big ).
B Medications (eg, calcium channel blockers, nitrates, beta-blockers).
C Obesity. Pregnancy.
D Doing exercises
ANSWER:D
144 Which of the following is the clinical of BUDD-CHIARI SYNDROME all true ,except’
A The classic triad of abdominal pain, ascitis and hepatomegaly.
B icterus, ascitis, hepatomegaly, splenomegaly, ankle edema, prominence of collateral veins.
C
Rapidly progressive : severe upper abdominal pain, mild jaundice, hepatomegaly, gross
ascites resistant to therapy, elevated liver enzymes and eventual encephalopathy.
D Lost weight gradualy.
ANSWER:D
145 Which of the following is the complication of GERD all true except
A Barrett’s esophagus
B A change in the lining of the esophagus that can increase the risk of cancer)
C Replacement of squamous cell epithelial with columnar epithelial is a kinds of metaplasia.
D Easy to heal
ANSWER:D
146
Which of the following is the Pathophysiology of BUDD-CHIARI SYNDROME all true
,except
A
The increased portal pressure causes: increased filtration of vascular fluid with the formation
of protein-rich ascites in the abdomen.
B
Collateral venous flow through alternative veins leading to oesophageal, gastric and rectal
varices.

C
Obstruction also causes centrilobular necrosis and peripheral lobule fatty change due to
ischemia.
D No change liver structure at all.
ANSWER:D
147 Which of the following is the pathophysiology of GERD all true except
A
Readiness of reflux:when gastric volume is increase (after meals, with pyloric obstruction or
gastric stasis syndrome and in acid hypersecretory state)
B Hypercalcemia, which can increase gastrin production, leading to increase acidity.
C
LES Relaxation:Pregnancy- The elevated hormone levels of pregnancy (e g, estrogen and
progesterone) relax the lower esophageal sphincter (LES) which usually prevent stomach
acid from entering the esophagus.
D Jaundice
ANSWER:D
148 Which of the following is the treatment of BUDD-CHIARI SYNDROME all true ,except
A No Supportive care,
B Anticoagulation, and thrombolysis
C Radiologic procedures (such as angioplasty, TIPS, and stenting)
D Surgical intervention (including shunting procedures and liver transplantation)
ANSWER:A
149 Which of the following statement is true regarding the clinical diagnosis of septic arthritis:
A
Acute mono arthritis, fever, synovial fluid culture positive blood culture positive (Leucocyte
>50000 cells / mm3 blood culture >75%( PMN)
B Poly arthritis, synovial fluid culture negative
C Acute mono arthritis fever, synovial fluid and blood culture are negative
D Poly arthritis fever synovial fluid culture negative, but blood culture positive
ANSWER:A
150 which of the following statement is true regarding the complications of hyperuricemia:
A Hyperuricemia can cause gout disease.
B Hyperuricemia can cause renal manifestation.
C Hyperuricemia can cause gout, renal manifestation and cardiovascular disease
D Hyperuricemia can cause sillent tissue depositioin.
ANSWER:C

151 which of the following statement is true regarding the diagnosis of gout:
A All hyperuricemia patients are gout.
B
Serum uric acid concentrations do not confirm or exclude gout because many people with
hyperuricemia do not develop gout and having acute attacks serum concentration might be
within the normal range in a third of patient.
C
Radiograph is useful for differential diagnosis and useful for confirmation of diagnosis of
early or acute gout.
D
Risk factors for gout and associated comorbidity should not be assessed including features of
metabolic syndrome
ANSWER:B
152 which of the following statement is true regarding the gout disease:
A
disease response to production or excretion of uric acid resulting in high levels of uric acid
in blood or caused by disturbed uric acid metabolism.
B The crystal arthropathies are caused by disturbed uric acid metabolism.
C Gout is secondary disease due to increased cell turnover.
D Gout is primary result to genetic defect glucose metabolism.
ANSWER:A
153 which of the following statement is true regarding the treatment of gout:
A Colchicine is the first-line medication in gout disease.
B Steroid is the first-line treatment of gout disease.
C Colchicine is the second-line treatment in gout.
D NSAIDs are the most commonly used first-line treatment of gout disease.
ANSWER:D
154
Which of the following statement is true regarding the treatment of rheumatic fever with
arthritis only:
A
Primary prevention, anti- inflammatory treatment (aspirin 75-100/kg/day 6 week and
prednisolone 2-2.5mg/kg/day for 4 weeks) supportive treatment
B
Penicillin V 250mg bid for 10days, aspirin 75-100 /kg/day for 6 weeks, prednisolone 2-
25kg/day for 4 weeks, treatment of chorea
C Treatment antibiotic, anti- inflammatory treatment of congestive cardiac failure

D
Benzatine peniciline G 600000 U IM for patient ≤ 27kg , 1.200000 U IM for patient bigger
than 27kg + aspirin 75-100 mg /kg/day 6weeks 4 divided dose, supportive management
,management of complications and secondary prevention of Rheumatic fever
ANSWER:D
155
Which of the following statement is true regarding the treatment of rheumatic fever with
carditis:
A
Benzathine penicillin G Patient ≤ 27kg 600000 U IM or Patient >27kg 1200000U IM (once
) , prednisolone 2-2.5 mg /kg/day for 2 weeks , taper over 2 weeks and while tapering add
aspirin 75mg /kg /day for 2 weeks , continue aspirin alone 100mg /kg/day for another 4
weeks ,supportive management, complications management and secondary prevention of
rheumatic fever.
B
Benzathine penicillin patient ≤ 27kg 600000 U IM or patient >27kg 1200000 U IM, Aspirin
75-100mg/kg /day 4-6weeks supportive management and prevention of recurrent attacks
C
Treatment of streptococcal tonsillophayngitis, anti- inflammatory treatment, supportive
treatment and management of complication
D
Eradication streptococcal infection, aspirin75-100mg/kg/d, and secondary prevention
recurrent infection.
ANSWER:A
156 Which of the following statement is true regarding the treatment of septic arthritis
A Antibiotic treatment, Joint drainage, surgery
B Antibiotic treatment, surgery, and blood culture
C Definitive therapy of antibiotic with arthrotomy
D
Antibiotic empiric (IV), definitive therapy of antibiotic, joint drainage (Closed needle
aspiration, arthrotomy) no weight bearing
ANSWER:D
157
Which one among the procedures below to be taken in the prevention of primary
immunodeficiency?
A Genetic counselling
B Prenatal diagnosis
C Sex determination
D All of the above

ANSWER:D
158
Which one among the sentences below that is not the characteristic of Secondary
Immunodeficiency?
A Wide spectrum of presentation and severity
B Generally resolved with the management of primary condition
C Inherited disorders of the immune system
D Extrinsic factors affecting intrinsically normal immune system
ANSWER:C
159 Which one of the below sentences is correct?
A Autoimmunity is the weak immune system in response to antigen
B Autoimmunity is the strong immune system in response to antigen
C Autoimmunity is the failure of immune system in recognizing its own constituent parts
D Autoimmunity is the auto-response of immune system to antigen
ANSWER:C
160
Which one of the clinical manifestations below that is rarely seen in predominantly antibody
defects?
A Recurrent bacterial infections
B Viral and fungal infections
C Lymphatic system hypoplasia
D Autoimmune diseases
ANSWER:B
161 Which one of the disease below that is caused by phagocyte difficiency?
A DiGeorge syndrome
B Ataxia-telangiectasia
C Chronic Granulomatous Disease
D Wiskott Aldrich syndrome
ANSWER:C
162
Which one of the elements below that does not belong to the criteria for diagnosis of
Rhumatoid Arthritis?
A Large joint affected
B Small joints affected
C Positive RF or ACPA antibodies

D Abnormal CRP or ESR
ANSWER:A
163 Which one of the mechanisms below that our immunity uses to kill virus
A Humoral immune response
B Cellular defense mechanism
C Cellular immune response
D Mechanical barriers and surface secretion
ANSWER:C
164 All vibrioses are requires salt for growth, except for
A V. cholerae
B Other vibrioses
C V. cholerae and V. mimicus
D V. mimicus
ANSWER:C
165 Appearance of HBsAg and appearance of anti-HBs given a signal
A recovery from HBV infection
B non infectivity
C infectivity and risk of chronic HBV
D protection from recurrent HBV infection
ANSWER:D
166 Cholera toxin (CTX) modifies G proteins and thereby permanently first activates
A Cyclic AMP (cAMP).
B PDE (Phosphodiesterase) enzymes
C AMP (adenosin monophosphate).
D Adenylate cyclase (AC)
ANSWER:D
167 Clinical manifestations of meningococcemia
A splenomegaly
B hepatomegaly
C hemorrhagic skin lesion (petechiae, purpura)
D diarrhea profuse
ANSWER:C

168 Clinical presentation in human of cysticercosis in muscles
A with fever, eosinophilia
B muscular pseudohypertrophy and fibrosis
C muscles are look like normal or marginally dystrophy
D strongly contraction
ANSWER:B
169 Clinical resolution VZV infection is followed by the establishment of latent infection within
A Spinal nerves
B Spinal Cord
C Nerve ending under the skin
D Sensory dorsal root ganglia
ANSWER:D
170 Congenital infection of toxoplasmosis is result in
A Non-immune women during pregnancy
B Immune women during pregnancy
C Immunodeficiency baby
D A & C
ANSWER:A
171 Cytomegalovirus is
A Varicella-Zoster Virus type 3
B causative agent of roseola
C is another herpes virus linked with Kaposi’s sarcoma
D Human Herpes Viruses Virus type 5
ANSWER:D
172 Describe the complications of typhoid fever in chronic carrier stage
A Intestinal bleeding or intestinal perforation
B Shed bacteria in their stool for decades
C Single carrier may have multiple genotypes
D intestinal perforation
ANSWER:C
173 Diagnosis of acute or chronic HCV infection generally requires testing of serum for

A anti-HCV
B anti-HCV and HCV RNA
C HCV RNA first
D anti-HCV or HCV RNA
ANSWER:B
174 Diagnosis of neurocysticercosis
A T. solium eggs in stool samples, increase white blood count
B history eating of pork meat
C calcium deposited in brain parenchyma
D Neuroimaging with CT or MRI
ANSWER:D
175 Disseminated intravascular coagulation (DIC) induced
A hemorrhage and tissue injury
B cardiogenic shock
C hypovolemic shock
D plasminogen activity
ANSWER:A
176 Disseminated intravascular coagulation (DIC) of meningococcal infection
A fibrinogen is high
B prothrombin time (PT) is prolonged
C partial thromboplastin time (PTT) prolonged is not seen
D platelet count is increased
ANSWER:B
177 During acute infection, viremia uniformly occurs as shown by
A detection of VZV DNA in serum
B peripheral blood mononuclear cells
C characteristic lesions of herpes zoster
D All above is true
ANSWER:D
178 Enhance transport of VZV to cutaneous epithelia may
A Macrophage
B B cells

C CD4 T cells and expressing skin markers
D Macrophage residential
ANSWER:C
179 Epidemiology of herpes zoster, or shingles, incidence rates progressively increase with
A decline in VZV-specific humoral-mediated immunity
B any age
C decline in VZV-specific cell-mediated immunity.
D All of the above
ANSWER:C
180 Epstein-Barr Infectious Mononucleosis
A Human Herpes Viruses Virus type 2
B Human Herpes Viruses Virus type 5
C Human herpes Virus type 3
D Human herpes Virus type 4
ANSWER:D
181 Gold standard for diagnosis of typhoid fever
A neutropenia or leukocytosis
B Widal test for antibody
C stools culture
D blood culture
ANSWER:D
182 Hepatitis A virus (HAV). Presence of IgG anti-HAV alone
A infectivity
B previous exposure to HAV
C chronic HAV
D previous exposure to HAV and immunity to recurring HAV infection
ANSWER:D
183 Hepatitis B virus (HBV). Persons with chronic HBV, particularly
A indicated to chronic persistent hepatitis B virus
B indicated to chronic active hepatitis B virus
C Most of them risk of hepatocellular carcinoma
D when viral replication persists and risk cirrhosis

ANSWER:D
184 Hepatitis C Virus (HCV). After acute exposure, anti-HCV is generally
A detected the same time with HCV RNA
B detectable before 2-4 weeks
C detectable after 2-4 weeks
D not detectable before 8-12 weeks
ANSWER:C
185 Hepatitis C Virus (HCV). After acute exposure, HCV RNA is usually detected as
A early as 4 weeks in serum
B early as 2 weeks in serum
C early as 1 weeks in serum
D early as 6 weeks in serum
ANSWER:B
186
Herpes Viruses (HVs), common morphologic features, the biologic and epidemiologic
features of each of the HVs are
A Similar
B Little different
C different
D most closely
ANSWER:C
187
Immunity is linked directly to reactivated VZV syndromes, observations indicate that a
decline in
A cell-mediated, rather than humoral
B humoral, rather than cell-mediated
C acquirer, rather than innate immunity
D innate, rather than acquirer immunity
ANSWER:A
188 In HIV infected person, risk of herpes zoster
A was not associated with the duration of HIV infection
B predicted more rapid progression to AIDS
C was associated with the duration of HIV infection
D All of the above

ANSWER:A
189
In the case of cysticerci in the brain parenchyma, four major stages have been lassified, in
stage 4 most patients bearing develop clinical signs and symptoms such as
A intracranial obstruction
B lethargy from increased blood pressure
C altered mental status, headaches, nausea, vomiting
D hypovolemic status
ANSWER:C
190 In toxoplasmosis, during CNS involvement can be documented with evidence of
A Brain atrophy
B Brain tissue necrosis
C Microglial necrosis
D B & C
ANSWER:D
191 In toxoplasmosis, lung complication have involvement of the
A Parenchyma pneumonitis
B Interstitial pneumonitis
C Pleural effusion
D All of the above
ANSWER:B
192 In toxoplasmosis, ocular complication can be documented of
A Irridocyclitis
B Hydrocephaly
C Granulomatous lesions
D A & C
ANSWER:D
193 In toxoplasmosis, oocyst is
A Tachyzoites
B Bradyzoites
C Sporozoites
D All of the above
ANSWER:C

194 In Toxoplasmosis, the hallmarks of infection develop
A Cell regeneration
B Cell necrosis
C Cell mutation
D Cell growth
ANSWER:B
195 In toxoplasmosis, tissue cyst is
A Tachyzoites
B Sporozoites
C Bradyzoites
D All of the above
ANSWER:C
196 In toxoplasmosis, trophozoite is
A Tachyzoites
B Bradyzoites
C Merozoites
D All of the above
ANSWER:A
197 Laboratory test for monitoring of typhoid fever
A Widal test with cell blood count
B blood culture
C Widal test with blood culture
D Stool culture
ANSWER:D
198 Mechanism of death in meningococcemia is
A Hypovolemic shock
B Septic shock
C Hemorrhagic shock
D High virulent
ANSWER:B
199 Meningococcal infection. Heparinization in DIC and bleeding does
A not influence prognosis

B has influence prognosis
C administration of dexamethasone is necessary
D aspirin is good prognosis
ANSWER:A
200 Meningococcal Meningitis. Lumbar puncture, cerebrospinal fluid (CSF) typically reveals
A normal protein
B a cloudy or purulent
C normal pressure
D increased glucose content
ANSWER:B
201 Patients who die from fulminant meningococcemia have
A partial adrenal insufficiency
B able to mount the normal coticosteroid response
C adrenal hemorrhages
D cardiogenic necrosis
ANSWER:C
202 Prolonged incubation period, prior to the onset of skin lesions of VZV
A requires for VZV overcome on WBCs
B requires for high dose of IFN-α production
C required for VZV to overcome local immune-mediated barriers
D requires IFN-α production by epidermal cells
ANSWER:C
203 Reactivation of this neurotropic virus leads to herpes zoster, or shingles,
A painful is not seen
B decline in VZV-specific humoral-mediated immunity.
C unilateral vesicular eruption in a restricted dermatomal distribution
D A & C
ANSWER:C
204 Role of antimotility agents in shigellosis:
A kill shigellae
B increases movement of intestine
C increases appetite

D improved penetration of shigella in intestinal mucus
ANSWER:D
205 S typhi and S paratyphi have specialized fimbriae that
A used the macrophages’ cellular machinery for their own reproduction.
B induced macrophages traveling from the gut into blood flow
C induced their host macrophages to attract more neutrophils
D adheres to Peyer’s patches
ANSWER:D
206 Serological test for acute hepatitis A virus (HAV)
A Detect of IgM and IgG is an excellent test for diagnosis of acute HAV.
B Titers of IgG anti-HAV peak after 1 month of the disease and may persist for years.
C Presence of IgG anti-HAV alone means chronic HAV
D Detect of IgM is an excellent test for diagnosis of acute HAV
ANSWER:D
207 Serological tests in toxoplasmosis, antibody IgM appears
A Early of the disease
B 1-2 weeks after start of infection
C 6-8 weeks after start of infection
D 3-4 week after start of infection
ANSWER:B
208 Shigellosis is an acute infectious inflammatory colitis due to
A Entamoeba histolytica
B Shigella dysenteriae
C Escherichia coli
D Salmonella typhi
ANSWER:B
209 Sing and symptoms of meningococcal meningitis
A nuchal and back rigidity are not seen
B Kernig and Brudzinski sings are not clearly positive
C cerebrospinal fluid glucose content is normal
D nausea and vomiting are present
ANSWER:D

210 Stages of clinical presentation of neurocysticercosis in human
A has 3 stages
B has 2 stages
C has 4 stages
D has 5 stages
ANSWER:C
211 Surface Vi antigen. S typhi has a Vi capsular antigen that
A masks toll-like receptor
B avoiding neutrophil-based inflammation
C macrophages unrecognized microorganism
D defense against pathogens by activating the innate immune system
ANSWER:B
212 The Primary VZV infection results in the diffuse vesicular rash of
A varicella or chickenpox
B herpes zoster or shingles or zona
C roseola
D Human Herpes Viruses Virus type 2
ANSWER:A
213 The salmonella that consists of Vi antigen
A Salmonella paratyphi C
B Salmonella paratyphi A
C Salmonella typhimurium
D Samonella choleresuis
ANSWER:A
214 The specific diagnosis of shigellosis is based on
A polymerase chain reaction (PCR) to detect shiga-family toxins
B culture of shigella from the stool
C Serological tests
D epidemiology
ANSWER:B
215 The specifics sing of typhoid fever
A high fever and increases heart rate

B abdominal pain, constipation in adult
C loss of consciousness
D petechial on the skin
ANSWER:B
216 The symptoms and sings of meningococcal meningitis
A can be distinguished from other meningeal pathogens by nuchal and back rigidity
B cannot be distinguished from symptoms and signs of tuberculous meningitis
C can be distinguished from other meningeal pathogens by Kernig, Brudzinski ’s sign
D cannot be distinguished from other meningeal pathogens.
ANSWER:D
217 The toxoplasma gondii in definitive host exists
A Trophozoite
B Cyst
C Oocyst
D All of the above
ANSWER:D
218 The transmission of cysticercosis
A by ingesting the eggs of T. solium from feces
B by ingesting of larvae of T. solium from cooked pork
C by ingesting from pork meat or from fish meat
D by ingesting from beef meat
ANSWER:A
219 The transmission of topxoplasmosis results from
A Transplacental transmission of trophozoites
B Careless handling of contaminated cat
C Ingestion of cysts in raw or undercooked meat
D All of the above
ANSWER:D
220 The yield of Shigella is increased if the organism is sought by
A Stool samples for gram stain
B urine culture
C hemoculture

D stool culture when the patient has fecal leukocytes or bloody diarrhea
ANSWER:D
221 Toxoplasma gondii, an obligate
A Intracellular protozoan
B Extracellular parasite
C Intravascular protozoan
D Interstitial protozoan
ANSWER:A
222 Toxoplasmosis, humans can become infected with the parasite by
A Inhaling coccidian of cats through dust or eating food
B Contact with cat
C Contact with trophozoite of coccidian of cats
D All of the above
ANSWER:A
223 Transmission of hepatitis B virus (HBV) may be:
A oral route
B vegetables and water
C blood and blood products
D breast milk
ANSWER:C
224 Transmission of hepatitis C virus (HCV) may be
A through oral drug user
B vegetables and water
C blood and blood products
D breast milk
ANSWER:C
225 Transmission of meningococcus to human
A oral route
B per cutaneous
C respiratory droplet
D blood transfusion
ANSWER:C

226 Treatment of herpes zoster is associated with
A healing of lesions
B prevention of complications, particularly postherpetic neuralgia (PHN)
C limiting the potential for reactivation
D All of the above
ANSWER:B
227 Typhoid fever causes by bacteria:
A Shigella
B Entamoeba histolytica
C Salmonella typhi
D Salmonella typhimurium
ANSWER:C
228 Varicella-Zoster Virus (VZV) infection causes
A causative agent of roseola
B Human Herpes Viruses Virus type 2
C varicella and herpes zoster (shingles or zona)
D two clinically not distinct forms of disease appeared on the skin
ANSWER:C
229 VZV-specific cell-mediated immune responses play a critical role in
A clearances of virus from blood and lymph
B healing VZV latency
C limiting the potential for reactivation
D B & C
ANSWER:C
230 VZV causes productive infection of activated CD4+ T cells, resulting in
A down-regulation of MHC class I expression.
B up-regulation of MHC class I expression
C down-regulation of MHC class II expression
D up-regulation of MHC class II expression
ANSWER:A
231 Acute encephalitis in clinical manifestation of Rabies
A periods excessive motor activity, excitation, and agitation

B confusion and hallucination are not seen
C calm and muscle spasms
D seizure and focal paralysis are not seen
ANSWER:A
232 AIDS is a disease, caused by HIV, that
A breaks down some part of the body’s immune system
B leaving a person defenseless against a variety of unusual microorganisms
C not life-threatening illnesses
D indicated a state of less transmissible
ANSWER:B
233 AIDS is a disease, when the number of CD4 cell count is
A 500 cells per mm3
B 500 to 350 cells per mm3
C 350 to 200 cells per mm3
D less than 200 cells per mm3
ANSWER:D
234 Clinical Manifestations of cellulitis tend to have
A localized symptoms over a few days’ time
B acute onset of symptoms with systemic manifestations including fever and chills
C a more indolent course with development of localized symptoms over a few days’ time
D more indolent course
ANSWER:C
235 Clinical Manifestations of erysipelas tend to have
A acute onset of symptoms
B more indolent course
C localized symptoms over a few days’ time
D upper extremities are the most common site of infection
ANSWER:A
236 Coma in cerebral malaria causes by the phenomenon of
A Cytoadherence of parasitized erythrocytes
B Hemolysis of RBCs
C Excess of plasmodium

D Cytoadherence of parasitized erythrocytes, Excess of plasmodium
ANSWER:A
237 Coma in cerebral malaria causes by the phenomenon of
A Rosette formation
B Hemolysis of RBCs
C Excess of plasmodium
D Rosette formation, Excess of plasmodium
ANSWER:A
238 Coma in cerebral malaria causes by the phenomenon of
A Sequestration of parasitized erythrocytes
B Rosette formation
C Excess of plasmodium
D Sequestration of parasitized erythrocytes, Rosette formation
ANSWER:D
239 Coma in cerebral malaria causes by the phenomenon of
A Reduce RBCs deformability
B Rosette formation
C Excess of plasmodium
D Reduce RBCs deformability, Rosette formation
ANSWER:D
240 Compared with HIV-2, for the HIV-1 is
A less transmissible
B rarely the cause of vertical transmission
C associated with a lower viral load
D associated with high virulent
ANSWER:D
241 Congenital syphilis. Treponema pallidum
A transmitted by placenta is not seen because of the strong barrier
B is transmitted by breast milk
C occurs in infants of treated or adequately treated mothers
D infant should be evaluated immediately, at 6-8 weeks of age
ANSWER:D

242 Death in clinical manifestation of Rabies
A involvement of the respiratory center produces and apneic death
B the patient with clear consciousness
C the median period of survival depend on treatment
D All of the above
ANSWER:A
243 Direct contact with transmission of leptospirosis are
A urine, blood of infected animal or exposure to a contaminated environment.
B tissue from an uninfected animal
C exposure of infected human
D urine of man
ANSWER:A
244 Field Management for venomous snake bites
A Withhold alcohol and drugs that may confound clinical assessment
B attempt to identify the snake
C Always application of pressure to delay in systemic absorption of venom
D
Withhold alcohol and drugs that may confound clinical assessment, attempt to identify the
snake
ANSWER:D
245 Gonorrhea during pregnancy is associated with
A spontaneous abortion
B premature labor
C early rupture of fetal membranes
D perinatal infant mortality
ANSWER:D
246 Human immunodeficiency virus (HIV): HIV1 and HIV2, which show
A 40% to 60% amino acid homology
B 30% to 40% amino acid homology
C 60% to 80% amino acid homology
D 40% to 50% amino acid homology
ANSWER:A
247 Human immunodeficiency virus 2 or HIV2 is

A very rapid transmissible
B rarely the cause of vertical transmission
C associated with a high virulent
D some patient high viral load
ANSWER:B
248 In AIDS, cytomegalovirus diseases (CMV) can infect multiple parts of the body, especially
A retinitis
B brain edema
C peptic ulcer
D glomerulonephritis
ANSWER:A
249 In AIDS, herpes simplex virus (HSV) is a very common virus is usually
A acquired by fecal-oral route
B from an infected mother during birth
C can cause diarrhea
D can transmit by protected sexual intercourse
ANSWER:B
250 In AIDS, Hodgkin lymphoma
A is a cancer that starts in cells called lymphocytes
B lymphocytes are in the lymph nodes and other lymphoid tissues
C is a cancer of the lymphatic system
D is cancer of bone marrow
ANSWER:C
251 In AIDS, infection with the fungus Cryptococcus neoformans can causes
A diarrhea
B peptic ulcer
C meningitis
D brain tumor
ANSWER:C
252 In AIDS, infection with the protozoan parasite Cryptosporidium can causes
A meningitis
B esophagitis

C chronic watery diarrhea
D pneumonitis
ANSWER:C
253 In AIDS, non-Hodgkin lymphoma
A is a cancer that starts in cells called neutrophil and eosinophil
B lymphocytes are in the lymph nodes and other lymphoid tissues
C is a cancer of the lymphatic system
D is a cancer like leukemia
ANSWER:B
254 In AIDS, the fungus Histoplasma capsulatum, often infects the lungs
A symptoms are similar to those of malaria
B symptoms are similar to those of pneumonia
C can involve organs other than the lungs in a short time
D can cause a watery diarrhea
ANSWER:B
255 In AIDS, the parasite Isospora belli, which can cause
A diarrhea and constipation
B brain edema
C weight loss
D pneumonia
ANSWER:C
256 In AIDS, the parasite Isospora belli, which can enter the body through contaminated
A food
B percutaneous
C inhalation
D close contact
ANSWER:A
257 In encephalitis. Hypothalamic-pituitary axis, increased intracranial pressure can cause
A Papilledema and 3rd and 6th cranial nerve palsies
B Papilledema and 4rd and 5th cranial nerve palsies
C headache on the occipital lobe
D headache on the occipital and frontal lobe

ANSWER:A
258 In epidemiology of leptospirosis, the reservoirs are
A herbivore animal
B rat, dogs, fish, and birds
C people living in the forest
D crustacean
ANSWER:B
259 In humans, rickettsiae grow principally in
A cell of visceral organ
B endothelial cells of artery
C endothelial cells of small blood vessels
D red blood cells
ANSWER:C
260 In malaria, changes in respiration can be a warning sign of
A Hypoglycemia
B Metabolic acidosis
C Pulmonary edema
D Hypoglycemia, Metabolic acidosis, Pulmonary edema
ANSWER:D
261 In neurosyphilis, adequate treatment is indicated by gradual decrease
A in WBC cell count
B protein concentration
C VDRL titer
D TPHA will be negative
ANSWER:C
262 In rickettsiosis. Diagnosis is usually based on
A symptoms
B epidemiologic evidence
C clinical examination
D epidemiologic evidence, clinical examination
ANSWER:D
263 In severe form of leptospirosis, characterized by

A cardiomyopathy
B renal dysfunction, jaundice and hemorrhagic diathesis
C hemorrhage in gastrointestinal tract
D blindness
ANSWER:B
264 Indicator of multiplication of HIV in target cell is
A gp41
B gp120
C p17
D p24
ANSWER:D
265 Invasive aspergillosis in the immunocompromised host presents
A most common among patients with acute leukemia
B recipients of tissue transplants
C acute, rapidly progressive pulmonary infiltrate
D
most common among patients with acute leukemia, recipients of tissue transplants, acute,
rapidly progressive pulmonary infiltrate
ANSWER:D
266 Leishmaniasis is a group of infections of
A skin and viscera
B tissue
C mucous membrane, skin and organ
D endothelial cell
ANSWER:C
267 Name of enzyme of HIV
A glycoproteins, gp120 and gp41
B reverse transcriptase (p64)
C DNA polymerase
D proteinase
ANSWER:B
268 Nonspecific prodrome in clinical manifestation of Rabies
A complaint of paresthesia at around the site inoculation of virus

B fatigability is not seen
C usually last 1 month
D complaint of paresthesia at around the site inoculation of virus,fatigability is not seen
ANSWER:A
269 Patients with chronic pneumonia and aspergillus in the sputum should be assumed to have
A Pneumonia causes by Aspergilus fumigatus
B Chronic lung disease causes by Aspergilus fumigatus
C Aspergillosis pneumonia with underlying immunosuppression
D
Chronic lung disease causes by Aspergilus fumigatus, Aspergillosis pneumonia with
underlying immunosuppression
ANSWER:C
270 Persons with HIV infection often have trouble with candida, especially in the
A head
B body
C mouth and vagina
D mouth only
ANSWER:C
271 Pneumocystis carinii pneumonia (PCP) is
A parenchyma pneumonia
B interstitial pneumonia
C pleural effusion
D pulmonary edema
ANSWER:B
272 Pneumocystis carinii pneumonia (PCP) occurs with
A weakened immune systems
B intact immune system
C people with CD4 cell between 350 to 200 cells per mm3
D people with CD4 cell lest than 500 cells per mm3
ANSWER:A
273 Profound dysfunction of brainstem centers in clinical manifestation of Rabies
A is the manifestations of brainstem
B cranial nerve involvement is not seen

C difficulty with deglutition in rare
D excessive salivation, spontaneous ejaculation when the patient is improve
ANSWER:A
274 Rickettsiae producing
A myocardial infarction
B cell necrosis
C vasculitis, thrombosis, cell necrosis
D organ dysfunction is not seen
ANSWER:C
275 Risk factor for acquiring gonorrhea is
A sexual intercourse with endometritis
B sexual intercourse with an infected partner
C sexual intercourse with salpingitis
D sexual intercourse with female infertility
ANSWER:B
276 Sign and symptoms of secondary syphilis
A lesion on the penis
B lesion skin and mucus
C lesion condylomata lata
D lesion firm chancre
ANSWER:C
277 Syphilis in Pregnancy. All pregnant women should have
A nontreponemal serologic test (e.g., VDRL) for syphilis
B seronegative women should be considered infected
C should be treated unless prior treatment with fall antibody titer
D is very rare transmitted to the baby
ANSWER:A
278 The filariform larvae of strongyloides stercoralis with
A Tail forked or blunt
B Tail tapered
C Tail pointed
D Tail forked or blunt, Tail tapered

ANSWER:A
279 The free-living form of strongyloides stercoralis with
A Lest transmissible
B Sharp pointed tail
C Long esophagus
D Sharp pointed tail, Long esophagus
ANSWER:B
280 The general paresis in syphilis
A is involved with dorsal roots and ganglia of spinal nerves
B is primary state syphilis
C is generalized involvement of the cerebral cortex
D is secondary state syphilis
ANSWER:C
281 The laboratory: Nontreponemal antigen tests (VDRL) in syphilis
A these tests are used as quantifiable test of antibody in syphilis
B these tests helpful in assessing disease activity
C these tests are used to diagnose of syphilis
D
these tests are used as quantifiable test of antibody in syphilis, these tests helpful in assessing
disease activity
ANSWER:D
282 The laboratory: Nontreponemal antigen tests (VDRL) in syphilis may become
A positive with adequate treatment in primary syphilis
B negative in untreated patients (50% of patients with late-stage syphilis)
C remains positive after treatment in late stage
D negative in an inadequate treatment in primary syphilis
ANSWER:C
283 The laboratory: Treponemal antibody tests (FTA-ABS and TPHA) in syphilis
A these tests are used in monitoring of the disease activity
B these tests are used to diagnose of syphilis
C these tests are used in monitoring of the disease activity
D
these tests are used in monitoring of the disease activity , these tests are used in monitoring of
the disease activity

ANSWER:B
284 The localization of lesions in secondary syphilis
A skin
B skin and mucosal surfaces
C lymphangitis
D lesion on the genitalia
ANSWER:B
285 The retrovirus that named human Immunodeficiency virus or HIV was isolated in
A 1985
B 1982
C 1983
D 1984
ANSWER:C
286 The roles of glycoproteins 120 (gp120)
A binding of HIV to every cell in the human body
B binding of HIV to CD4 T cells
C fusion of HIV to CD4 T cells
D binding and fusion of HIV to CD4 cells
ANSWER:B
287 The roles of glycoproteins 41 (gp41)
A binding of HIV to every cell in the human body
B binding of HIV to CD4 T cells
C fusion of HIV to CD4 T cells
D binding and fusion of HIV to CD4 cells
ANSWER:C
288 The Tabes dorsalis in syphilis
A degeneration of ascending fibers of sensory neurons of spinal cord
B lead to muscular hypotonia
C muscle contraction
D anesthesia
ANSWER:A
289 Transmission of Leptospirosis

A from uninfected animal
B tissue from an infected animal, exposure to a contaminated environment
C food is an important vehicle in their transmission
D from meat of pork
ANSWER:B
290
A 22 year old medical student has presented to his GP because he is concerned that his urine
is dark: he’s worried that he has a urine infection. He has also had lower abdominal pain and
non-specific joint pain. He presented yesterday with a sore throat and headache. On urine dip
you note microscopic haematuria, alongside proteinuria. What investigation would be
diagnostic for the cause of his presentation?
A Renal ultrasound
B Urine protein: creatinine ration
C Antiglomerular basement membrane antibodies
D Renal biopsy
ANSWER:D
291
A 4 year old boy has presented with sudden onset periorbital oedema, which has progressed
to oedema across his entire body. He has been lethargic, with a poor appetite and his mum
states that his urine has looked a bit ‘frothy’. On examination he has leukonychia. What is
the most appropriate initial investigation for this presentation?
A Urine dipstick
B Renal ultrasound
C Renal biopsy
D U&Es
ANSWER:A
292
A diuretic used for treatment of hypertension & heart failure that can decrease glucose
tolerance, produce hypokalemia (in high doses), aggrevate gout by interfering with uric acid
secretion, and produce a small rise in LDL.
A Amiloride
B Furosemide
C Hydrochlorothiazide
D Mannitol
ANSWER:C

293 A doctor who specializes in Kidney diseases is call a:
A Urologist
B Endocrinologist
C Nephrologist
D Immunologist
ANSWER:C
294 Acute complication of hepatitis A virus
A Acute fulminant hepatitis
B Liver cirrhosis
C Liver cancer
D Chronic hepatitis
ANSWER:A
295 Acute coronary syndrome includes the followings, except:
A Non-ST-elevation myocardial infarction (NSTEMI),
B Stable angina pectoris
C ST-elevation MI (STEMI), and
D Unstable angina
ANSWER:B
296 All the following are important electrolyte in the body except ?
A Potassium ion
B Carbon ion
C Chloride ion
D Sodium ion
ANSWER:B
297 Amlodipine is one of anti HTN medicine which is in the group of:
A ARB
B ACE inhibitor
C Centrally acting agent
D Calcium channel blocker
ANSWER:D

298
An 84 year old gentleman is seen on a medical ward round. He was admitted two weeks ago,
‘off legs’ with suspicion of a chest infection. This was treated successfully with antibiotics
and he is close to discharge. He experienced a recurrence of longstanding shoulder pain two
days ago, for which he was started on ibuprofen gel. You note that his U&Es, undertaken
this morning are deranged. His creatinine is 185 (normal 60-120), which is an increase from
133 two days previously. He has only passed 100ml of urine in the past 24 hours, with zero
urine output in the past 12 hours. He weighs 85kg. He had been clinically well until now,
with a past medical history of COPD and osteoarthritis. He is apyrexial and is not tachycardic
or tachypnoeic. What is the most likely cause of his Acute Kidney Injury?
A Sepsis
B NSAIDs
C Calculus
D Glomerulonephritis
ANSWER:C
299
An 88 year old man has been on the general medical ward for two weeks. Today he is
complaining of bone pain. He has an extensive past medical history, including COPD,
diabetes and hypertension. His most recent U&Es have come back as: creatinine 199 (normal
60-120), urea 17 (normal 2.5-6.5), chloride 177 (95-110). This has been a slow progression.
His eGFR is 13. His other bloods include: corrected calcium 2.01 (normal 2.25-2.5 mmol/L),
Calcitriol 25 (normal >50), Phosphate 2.2 (normal 0.8-1.4), PTH 70 (normal 10-55). Which
of the following processes is causing the patient’s abnormal bloods, and therefore bone pain?
A Reduced renal excretion of phosphate
B Primary hyperparathyroidism
C Poor dietary intake
D Tumour Lysis Syndrome
ANSWER:A
300
Any reduction in HbA1c is beneficial for individuals; however, the ideal target for an adult
with type 2 diabetes, managed with diet alone, should be:
A 6%
B 8%

C <7%
D 9%
ANSWER:C
301 Approximately one – third of the body water exist in the ?
A Kidney and urinary bladder
B Blood
C Extracellular fluid compartment
D Transcellular fluid compartment
ANSWER:C
302 Atherosclerosis is a disease of artery characterized by the following, except
A A healthy artery walls
B The artery which become thicken
C The artery which loss of elasticity
D Calcification of arterial walls resulting in decreased blood supply
ANSWER:A
303
Choose the correct answer about New York Heart Association (NYHA) functional
classification.
A
Class I: Patients have cardiac disease resulting in inability to carry on any physical activity
without discomfort.
B Class II: Patient have cardiac disease but without the resulting limitations of physical activity.
C Class III: Patients have cardiac disease resulting in marked limitation of physical activity.
D Class IV: Patients have cardiac disease resulting in slight limitation of physical activity.
ANSWER:C
304
Choose the incorrect answer One patient comes for health checkup. First BP reading was
145/92mmHg without any complain. What will you do next?
A Ask the patient to rest and repeat BP at least 5 minutes later.
B Immediately provide anti-HTN medicine to lower the BP.
C Take complete history and physical examination to confirm a diagnosis of hypertension.
D Screen for other cardiovascular disease risk factors.
ANSWER:B
305 Choose the incorrect statement about coronary artery disease.

A
Ischemic Heart Disease (IHD) is a condition in which there is an inadequate supply of blood
and oxygen to a portion of the myocardium.
B
The most common cause of myocardial ischemia is atherosclerotic disease of an epicardial
coronary artery.
C
Central to an understanding of the pathophysiology of myocardial ischemia is the concept of
myocardial supply and demand.
D
Even coronary artery lumen is severely reduced, myocardial perfusion is always maintained
as normal.
ANSWER:D
306 Common non-specific symptoms of chronic kidney disease include:
A Increased urination at night
B Loss of appetite
C Swelling of hands and feet
D All of the above
ANSWER:D
307 Correct ways of BP measurement, except:
A Have the patient relax, sitting on chair, feet on floor
B
The patient should avoid caffeine, exercise, and smoke for at least 30minutes before
measurement.
C
Examiner should talk to patient throughout the time of checking BP to ensure your patient is
fine.
D Ensure patient has emptied his/her bladder.
ANSWER:C
308 Definition of Cholelithiasis?
A Stone formation in the gallbladder.
B Stone formation in the choledochus
C Stone formation in bile duct
D All of the above
ANSWER:A
309 Definition of irritable bowel syndrome
A
is a chronic functional bowel disorder characterized by symptoms of abdominal pain or
discomfort that is associated with alteration in bowel habits.

B is a chronic inflammation of the bowel
C is an ulceration of the bowel
D is an infection of the bowel
ANSWER:A
310 Diagnosis of gall stone base on
A Blood tests
B Urinary tests
C Bilirubin tests
D Ultrasonography of abdomen
ANSWER:D
311 Diagnosis of irritable bowel syndrome
A History taking, Physical examination, Signs and symptom
B Blood tests
C Imaging study
D All of the above
ANSWER:A
312 Diagnostic evaluations of Acute coronary syndrome (ACS) are based on followings:
A History of chest discomfort
B Electrocardiogram finding
C Cardiac biomarkers
D All of the above
ANSWER:D
313
DM is a 62 year old man who has had poorly controlled hypertension for the past 10 years,
and now presents with signs of ankle edema, a low GFR and a serum creatinine of 2.5
mg/dL. The most effective drug for producing a diuresis and fall in blood pressure in DM is:
A Amlodipine
B Furosemide
C Hydrochlorothiazide
D Losartan
ANSWER:B
314 Edema is the accumulation of fluid in the ?

A Interstitial space
B Special fluid compartments
C Plasma
D Blood
ANSWER:A
315 Electrolyte balance in the body usually refers to the balance of ?
A Bases
B Salts
C Organic molecules
D Acids
ANSWER:B
316
Imbalance between myocardial oxygen supply and demand resulting from the following
processes, except:
A Disruption of an unstable coronary plaque due to plaque rupture
B Gradual intraluminal narrowing
C
Increased myocardial oxygen demand produced by conditions such as fever, tachycardia,
and thyrotoxicosis in the presence of fixed epicardial coronary obstruction.
D None of the above
ANSWER:D
317 Incubation period of hepatitis A virus
A 2 to 3 weeks
B 1 to 2 months
C 2 to 3 months
D 1month
ANSWER:A
318 Incubation period of hepatitis B virus
A 1 to 2 weeks
B 2 to 3 weeks
C 1 to 5 months
D 5 to 7 days
ANSWER:C
319 Macro-albuminuria detected by

A Albumin Creatinine Ratio <30 mg/g
B Albumin Creatinine Ratio > 100 mg/g
C Albumin Creatinine Ratio > 200 mg/g
D Albumin Creatinine Ratio > 300 mg/g
ANSWER:D
320 Manning Criteria for irritable bowel syndrome except one is incorrect:
A At least three of the underlying six criteria has to be present:
B (1) Abdominal pain eased after bowel movement, (2) Looser stool at the onset of pain,
C (3) More frequent bowel movement at the onset of pain, (4) Abdominal distension
D (5) Mucus per rectum, (6) Feeling of incomplete evacuation
ANSWER:A
321 Medically speaking, which term refers to the kidney function?
A Hepatic
B Renal
C Dialysis
D Urinary
ANSWER:B
322 Micro-albuminuria detected by
A Albumin Creatinine Ratio 30 to 300 mg/g
B Albumin Creatinine Ratio <30 to 100 mg/g
C Albumin Creatinine Ratio > 30 mg/g
D Albumin Creatinine Ratio <30 mg/g
ANSWER:A
323 Microscopic hematuria is the presence of
A Number of WBC > 10 /mm3 in urinalysis
B Number of WBC > 100/mm3 in urinalysis
C Number of RBC > 10/mm3 in urinalysis
D Number of RBC >100/mm3 in urinalysis
ANSWER:C
324 Normo-albuminuria detected by
A Albumin Creatinine Ratio <30 mg/g
B Albumin Creatinine Ratio <300 mg/g

C Albumin Creatinine Ratio > 300 mg/g
D Albumin Creatinine Ratio <10 mg/g
ANSWER:A
325 One of the following is not risk factor for primary (essential) hypertension.
A Obesity.
B Regular exercise.
C High salt intake.
D Excessive alcohol consumption.
ANSWER:B
326
One of the following tests is not considered as part of the test that is used to evaluate
coronary artery diseases:
A Electrocardiogram
B Stress testing
C Coronary artery bypass grafting (CABG)
D Coronary arteriography (angiography)
ANSWER:C
327 One of the followings is not considered as a cause of myocardial ischemia.
A An increase in oxygen demand due to LVH secondary to hypertension.
B A reduction in oxygen supply secondary to coronary atherosclerosis and anemia.
C Abnormal constriction or failure of normal dilation of the coronary resistance vessels.
D None of the above.
ANSWER:D
328 Prevention of hepatitis A virus
A Vaccination
B Immunoglobulin therapy
C Antiviral drug
D Immunosuppressive drug
ANSWER:A
329 Prevention of hepatitis B virus
A Vaccination
B Immunoglobulin therapy
C Antiviral drug

D Immunosuppressive drug
ANSWER:A
330
The amount of myocardial damage caused by coronary occlusion depends on the following,
except:
A The territory supplied by the affected vessel.
B Whether or not the vessel becomes totally occluded.
C The duration of coronary occlusion.
D The body mass index of the patients.
ANSWER:D
331 The condition in which sodium level are too low is referred to as ?
A Hyponatremia
B Hypokalemia
C Aldosteronism
D Hypernatremia
ANSWER:A
332 The following are groups of HTN medicine, except:
A Calcium channel blocker
B Diuretics
C Angiotensin converting enzyme inhibitor
D Third generation cephalosporine
ANSWER:D
333 The following medicines are used for Anti-anginal agents except:
A Nitrates
B Sublingual Nitroglycerin
C Metoprolol
D Rosuvastatin
ANSWER:D
334 The following points described the usual presentation of Angina, except:
A It is crescendo-decrescendo in nature
B It typically lasts 2–5 min
C
It can radiate to either shoulder and to both arms (especially the ulnar surfaces of the forearm
and hand)

D It is usually relieved by taking anti-hypertensive medicine
ANSWER:D
335 The following statements are the clinical clues to suspect Secondary hypertension, except
A 60 yrs person, with obesity, and physical inactivity, with family history of HTN.
B Severe or resistant hypertension.
C Hypertension associated with electrolyte disorders including hypokalemia.
D HTN at age less than 30 years with no family history of hypertension and no obesity.
ANSWER:A
336 The following statements are true about Hypertension (HTN), except:
A
Hypertension increases the risk of cardiovascular diseases, including coronary heart disease
(CHD), congestive heart failure (CHF), ischemic and hemorrhagic stroke, renal failure, and
peripheral arterial disease (PAD).
B The prevalence of hypertension increases with advancing age.
C Systolic blood pressure refers to the pressure inside the artery during ventricle relaxation.
D
HTN is a common condition in which the long-term force of the blood against the artery
walls is high enough that it may eventually cause health problems.
ANSWER:C
337 The followings are the long-term management for coronary artery disease, except:
A Smoking cessation
B Regular exercise
C Control of hypertension and diabetes
D None of the above
ANSWER:D
338 The followings are the Major manifestations of acute rheumatic fever, except
A Arthritis (usually migratory polyarthritis predominantly involving the large joints)
B Carditis and valvulitis (eg, pancarditis)
C Central nervous system involvement (eg, Sydenham chorea)
D
Elevated acute phase reactants (erythrocyte sedimentation rate [ESR], C-reactive protein
[CRP])
ANSWER:D
339 The followings are the Minor manifestations of acute rheumatic fever, except
A Subcutaneous nodules

B Fever
C
Elevated acute phase reactants (erythrocyte sedimentation rate [ESR], C-reactive protein
[CRP])
D Prolonged PR interval on electrocardiogram
ANSWER:A
340 The intracellular fluid compartment refers to all the water found in ?
A The bones of the body
B Area outside the body cell
C Areas within the gastrointestinal tract
D All cells of the body
ANSWER:D
341 The main hormone secreted by the Thyroid gland
A T4
B T3
C A and B Both
D TSH
ANSWER:C
342
The management plan for coronary artery disease should include the following components,
except
A Identification and treatment of aggravating conditions.
B Treatment of risk factors that will decrease the occurrence of adverse coronary outcomes.
C Consideration of revascularization.
D None of the above.
ANSWER:D
343 The most prevalent electrolyte in the extracellular fluid is ?
A Potassium
B Calcium
C Phosphate
D Sodium
ANSWER:D
344 The test for checking mean plasma glucose concentration over the previous 8-10 weeks is:
A Hemoglobin A1c

B Oral glucose tolerance test (OGTT)
C Fructosamine test
D Fasting plasma glucose concentration
ANSWER:A
345 The thyroid produces hormones that regulate what bodily function?
A Metabolism
B Temperature
C Blood pressure
D Digestion
ANSWER:A
346 There are …. stages of kidney disease:
A 3
B 5
C 6
D 7
ANSWER:B
347 Thyroid disorders are sometimes mistaken for which disease or condition?
A Crohn's disease
B Menopause
C Pregnancy
D Posttraumatic stress disorder
ANSWER:B
348 Urinary infection is the presence of
A Number of WBC > 10 /mm3 in urinalysis
B Number of WBC > 100/mm3 in urinalysis
C Number of RBC > 10/mm3 in urinalysis
D Number of RBC >100/mm3 in urinalysis
ANSWER:A
349 Viral hepatitis A spread by
A Faeco-oral
B Vertical
C Carrier state

D All of the above
ANSWER:A
350 Weight loss, rapid heart rate, and heat sensitivity are likely symptoms of...
A Hypothyroidism (underactive thyroid)
B Hyperthyroidism (overactive thyroid)
C Thyroid cancer
D All of the above
ANSWER:B
351
What are women with a history of gestational diabetes mellitus at an increased risk of
developing?
A Type 1 and type 2 diabetes mellitus
B Type 1.5 diabetes
C Type 2 diabetes mellitus
D Type 1 diabetes mellitus
ANSWER:C
352 What is the first-line drug for patients with type 2 diabetes and obesity?
A Acarbose
B Metformin
C Sulphonylureas
D Insulin
ANSWER:B
353
When the patient is asked to localize the sensation, he or she typically places a hand over the
sternum, sometimes with a clenched fist, to indicate a squeezing, central, substernal
discomfort. This sign is called:
A Murphy’s sign
B Rovsing’s sign
C Levine’s sign
D Kernig sign
ANSWER:C
354 Where is the thyroid gland located?
A At the base of the spine
B In the abdomen

C In the neck
D In the brain
ANSWER:C
355 Which is not correct about atherosclerosis?
A
Lesions are most common at branch points, at sites of low shear, where a predilection to
plaque formation has been observed.
B Fatty streak is the first phase in atherosclerosis
C
Fibrous cap atheromas are defined as plaques with a well-defined lipid core covered by a
fibrous cap, which may be relatively acellular.
D
Intraplaque hemorrhage mainly results from decrease blood supply like in case of
dehydration.
ANSWER:D
356 Which of the following conditions is not likely associated with atherosclerotic plaque rupture?
A Acute coronary syndrome
B Hypotension secondary from hypovolemia
C Ischemic stroke
D Peripheral artery occlusive disease
ANSWER:B
357 Which of the following confirmed values meet the diagnostic threshold for diabetes?
A fasting blood glucose ? 140 mg/dl
B random glucose &gt; 160 mg/dl
C 2 hour post prandial glucose ≥ to 126 mg/dl
D fasting blood glucose ≥ 126 mg/dl
ANSWER:D
358 Which of the following is correct for Isolated systolic hypertension?
A Both systolic and diastolic blood pressure are higher than normal range.
B Only diastolic blood pressure is higher than normal range.
C Only systolic blood pressure is higher than normal range.
D Both systolic and diastolic blood pressure are within normal range.
ANSWER:C
359 Which of the following is NOT an example of an electrolyte?

A Creatinine
B Potassium
C Magnesium
D Calcium
ANSWER:A
360 Which of the following statements is correct?
A Insulin suppresses the activity of glycogen synthase
B Insulin mediates glucose uptake in the brain
C
"Prediabetes" is a condition characterized by an increased risk for the future development of
type 2 diabetes
D
The rise in insulin concentration after meal ingestion is reduced in type 1 but not in type 2
diabetes
ANSWER:C
361 Which of the followings is not considered as part of the treatment for NSTEMI?
A Anti-angina agents
B Anti-platelet therapy
C Low-molecular-weight heparin (LMWH)
D Proton pump inhibitor
ANSWER:D
362
Which of these factors is not the pathogenesis contributing to the development of
atherosclerosis?
A Endothelial dysfunction
B Dyslipidemia
C Hypertension
D Regularly taking Atorvastatin
ANSWER:D
363 Which patient is at most risk for fluid volume deficiency ?
A A patient who has been vomiting and having diarrhea for 2 days
B A patient with continuous nasogastric suction
C A patient with an abdominal wound vac at intermittent suction
D All of the above
ANSWER:D

364 Which patient is at most risk for the hypomagnesemia ?
A A 55 year old chronic alcoholic
B A 57 year old with hyperthyroidism
C A patient reporting overuse of antacid and laxative
D A 28 year old suffering from hypoglycemia
ANSWER:A
365
Your 60 year old male hypertensive patient who had an MI a year ago is now showing signs
of CHF. You therefore add spironolactone to his drug regimen. What side effect should you
warn him about?
A Gynecomastia
B Hypokalemia
C Lupus
D Ototoxicity
ANSWER:A
366
A female patient 40yo complains many months ago of palpitation crisis that started brutal
without particular triggered circumstance. Until now the palpitations has receded brutally and
spontaneously within some minutes, immediately followed by urine out put. She has
admitted for a lasting crisis with duration greater than 30min and accompanied by general
dizziness. BP was 90/70mmHg. ECG finding regular tachycardia with 200b/min, QRS
narrow and P waves negatives, retrogrades in D2, D3, aVF was seen 200sm after each
QRS.Question with a true answer What is a exact heart arrhythmia?
A Sinus tachycardia
B Basic atrial tachycardia
C Junctional tachycardia
D Atrial flutter
ANSWER:C

367
A patient 56yo, consults for exercise dyspnea that has developed many years ago, but
uncomfortable (rapid working on flat ground) for only few weeks. Heart auscultation at
mitral area hears systolic murmur 3/6, blowing, pans systolic, radiation to axillary. Physical
examination don’t fine peripheral signs of heart failure. ECG shows an atrial Fibrillation with
ventricular rate 100b/min. On chest x ray, we saw a cardiomegaly (ICT = 0.60), Bulge of
middle and inferior left portion with pulmonary hyper vascularization. A mitral regurgitation
was diagnosed.Question with simple complement A male patient 40yo has HTN with IHD
complication, what antihypertensive drugs is preferable to treat his hypertension.
A Betabloquant
B Diuretic
C ACE
D Central alpha stimulant
ANSWER:A
368
A patient 56yo, consults for exercise dyspnea that has developed many years ago, but
uncomfortable (rapid working on flat ground) for only few weeks. Heart auscultation at
mitral area hears systolic murmur 3/6, blowing, pans systolic, radiation to axillary. Physical
examination don’t fine peripheral signs of heart failure. ECG shows an atrial Fibrillation with
ventricular rate 100b/min. On chest x ray, we saw a cardiomegaly (ICT = 0.60), Bulge of
middle and inferior left portion with pulmonary hyper vascularization. A mitral regurgitation
was diagnosed. Question with simple complement What is a frequent sign of auscultation in
this disease and that not have in clinical description?
A Splitting S1
B Splitting S2
C Decreasing of S2’s intensity
D Presence of S3 gallop
ANSWER:C

369
A patient 56yo, consults for exercise dyspnea that has developed many years ago, but
uncomfortable (rapid working on flat ground) for only few weeks. Heart auscultation at
mitral area hears systolic murmur 3/6, blowing, pans systolic, radiation to axillary. Physical
examination don’t fine peripheral signs of heart failure. ECG shows an atrial Fibrillation with
ventricular rate 100b/min. On chest x ray, we saw a cardiomegaly (ICT = 0.60), Bulge of
middle and inferior left portion with pulmonary hyper vascularization. A mitral regurgitation
was diagnosed. Question with simple complement (One answer) Among of 5 auscultation
signs, the one is impossible to hear in this patient?
A Decreasing of S1
B Increasing of S2 at pulmonary area
C Presence of S3 gallop
D Presystolic rumbling
ANSWER:D
370
A patient 56yo, consults for exercise dyspnea that has developed many years ago, but
uncomfortable (rapid working on flat ground) for only few weeks. Heart auscultation at
mitral area hears systolic murmur 3/6, blowing, pans systolic, radiation to axillary. Physical
examination don’t fine peripheral signs of heart failure. ECG shows an atrial Fibrillation with
ventricular rate 100b/min. On chest x ray, we saw a cardiomegaly (ICT = 0.60), Bulge of
middle and inferior left portion with pulmonary hyper vascularization. A mitral regurgitation
was diagnosed. Question with simple complement In which functional NYHA classification
that can situate this patient?
A Class 0
B Class 1
C Class 2
D Class 3
ANSWER:C

371
A patient 56yo, consults for exercise dyspnea that has developed many years ago, but
uncomfortable (rapid working on flat ground) for only few weeks. Heart auscultation at
mitral area hears systolic murmur 3/6, blowing, pans systolic, radiation to axillary. Physical
examination don’t fine peripheral signs of heart failure. ECG shows an atrial Fibrillation with
ventricular rate 100b/min. On chest x ray, we saw a cardiomegaly (ICT = 0.60), Bulge of
middle and inferior left portion with pulmonary hyper vascularization. A mitral regurgitation
was diagnosed. Question with simple complement For intense chest pain, developed brutal
on patient 45yo, a diagnosis of MI is suspected for theFollowing symptoms excepts one,
what is?
A Location retrosternal
B Increase intensity during deep inspiration
C Squeezing
D Radiation to jaw, shoulder and left arm
ANSWER:B
372
All investigations for diagnosis of stable angina are recommended One false answer, which
one?
A ST depress and T inversed
B Stress test
C Artery coronary angiography
D Blood test (Troponin)
ANSWER:D
373 All proposals are not corrects for treatment of hypertensive crisis, except one is true?
A Oral therapy
B Long acting antihypertensive drug
C Decreased BP 15% in first hour and 25% over the first 6h with Loxen…
D Not should have continuous BP monitoring
ANSWER:C
374 All proposals are used for treatment of moderate HTN, except one is false?
A Life style change, diet
B Monotherapy in initial treatment
C Goal BP < 140/90mmHg
D Betablocker is first choice therapy if HTN uncomplicated

ANSWER:D
375 blood test for iron over load
A Serum iron increases
B Transferrin saturation >70%
C Transferrin saturation > 45%
D Serum ferritin: >300 µg/L for men and postmenopausal women
ANSWER:B
376 Born pain in hyperparathyroidism causes by
A Reabsorption of calcium from bone to blood
B Absorption of calcium from GI tract to blood
C Reabsorption of calcium from kidney to blood
D All of the above
ANSWER:A
377 Causes of MR All answer are trues, except one, which one?
A Rheumatic fever is not frequent
B Ischemic heart disease
C Valve prolapse
D Functional
ANSWER:A
378 Causes of MS All answer are trues, except one, which one?
A Rheumatic fever
B Calcification
C Lutembacher (SVD+MS)
D Congenital
ANSWER:C
379 Chest X ray in MS One false answer, which one?
A Middle portion of left contour is normal
B Vascular redistribution at top of lung
C Interstitial edema
D Alveolar edema
ANSWER:A
380 Chvostek sign:

A
Positive sign is ipsilateral twitching of the upper lip upon tapping the facial nerve on the
cheek.
B
Positive sign is contralateral twitching of the upper lip upon tapping the facial nerve on the
cheek.
C
Positive sign is painful carpal spasm after 3-minute occlusion of brachial artery with blood
pressure cuff.
D All of the above
ANSWER:A
381 Complications of LHF One true answer, which one
A Acute pulmonary edema
B No thrombus
C No heart arrhythmia
D Endocarditis
ANSWER:A
382 Complications of mechanical prosthetic valve One false answer, which one?
A Endocarditis
B Sudden death
C Malaria
D Hepatitis
ANSWER:B
383 Complications of MR One false answer, which one?
A Left heart failure
B Thrombus
C Pulmonary embolism
D Heart arrhythmias
ANSWER:C
384 Complications of MS One false answer, which one?
A Acute pulmonary edema
B Thrombus
C Atrial fibrillation
D Pulmonary embolism
ANSWER:D

385 Complications of Wilson`s disease except one
A Liver cirrhosis
B Parkinson disease
C Neurologic disorders
D Liver cancer
ANSWER:B
386 Count The following statement of the 12 cranial nerve are correct except
A The olfactory nerve , the optic nerve , Oculomotor nerve (trochlear nerve , trigeminal nerve ,
B Abducens nerve , facial nerve , vestibule-cochlear nerve ,
C Glossopharyngeal nerve , Vagus nerve , accessory nerve , and the hypoglossal nerve .
D Sympathetic and parasympathetic nerve.
ANSWER:D
387 Definition of HTN One false answer, which one?
A BP ≥ 140/90mmHg
B BP ≥ 160/80mmHg
C BP ≥ 130/90mmHg
D BP = 120/80mmHG
ANSWER:D
388 Hemochromatosis signs and symptoms occurs
A In female later than male
B In male later than female
C In both sexes the same decade
D All of the above
ANSWER:A
389 Hypo-parathyroidism causes
A Hyper-calcemia
B Hypo-calcemia
C Hyper-kalemia
D Hypo-kalemia
ANSWER:B
390 In ischemic heart disease, all answers are false, except one is true?

A Affected young person
B Most often without risk factors
C On healthy coronary artery
D Included: stable angina and acute coronary syndrome
ANSWER:D
391 In new born baby hypo-calcemia occurs in
A Hyper-calcemic mother
B Hypo-calcemic mother
C Hypo- kalemic mother
D Hyper-kalemic mother
ANSWER:A
392 INR control One true answer, which one?
A INR 1,5-2
B INR 2-3
C INR 3-3,5
D No risk bleeding on old patient
ANSWER:B
393 Location deep septal One true answer, which one?
A V1,V2,V3
B V1,V2,V3,4
C V1,V2,V3,V4,V5,V6
D D2,D3,aVF,V1,V2,V3
ANSWER:D
394
Male patient 62yo, 70kg, has a past medical history HTN 5 years and dyspnea on exercise 1
year ago. Current treatment as digoxine 1c/d, Natrilix 1c/d and diet.This patient has called in
emergency for brutal developing of dyspnea. Physical examination found that BP
170/110mmHg, HR 138/min irregular. Respiratory rate 32/min, T⁰ 37⁰, conciseness is
normal, cyanosis, sweet, systolic murmur 3/6, left gallop, good urine out put just arrival.
Lung auscultation finding wheezing and crackle rale bilateral at lower lobes. Question with a
true answer Heart murmur can correspond on:
A MR organic
B MR functional related to LV cavity dilated

C MS
D Anemia
ANSWER:B
395
Male patient 62yo, 70kg, has a past medical history HTN 5 years and dyspnea on exercise 1
year ago. Current treatment as digoxine 1c/d, Natrilix 1c/d and diet.This patient has called in
emergency for brutal developing of dyspnea. Physical examination found that BP
170/110mmHg, HR 138/min irregular. Respiratory rate 32/min, T⁰ 37⁰, conciseness is
normal, cyanosis, sweet, systolic murmur 3/6, left gallop, good urine out put just arrival.
Lung auscultation finding wheezing and crackle rale bilateral at lower lobes. Question with a
true answer An element of clinical feature is compatible with:
A Pulmonary embolism
B Asthma crisis
C Cardiogenic shock
D Left heart failure
ANSWER:D
396 Management of Hypercalcemia
A Loop diuretic
B Thiazide diuretic
C Reduce water intake
D Immobilization
ANSWER:A
397 management of tetany
A IV calcium gluconate: 1 or 2 g, each infused over a period of 10 minutes.
B Oral calcium
C Rehydration
D Anti-convulsion drug
ANSWER:A
398 Measurement condition of BP One true answer, which one?
A One measure
B Immediately after exercise arrest
C Unilateral
D 10min after exercise arrest, bilateral, 2-3 measures, sleeping and standing position

ANSWER:D
399 Mitral valve stenosis (MS) One true answer, which one?
A Incomplete closing of mitral valve
B During systole
C Most common cause is carcinoide syndrome
D Incomplete opening of mitral valve during diastole
ANSWER:D
400 Most common bacteria for pyelonephritis
A Escherichia coli
B Staphylococcus
C Streptococcus
D Klebsiella
ANSWER:A
401 Percutaneous mitral dilatation All answer are trues, except one, which one?
A Little mitral valve calcification and severe stenosis
B Cordage is soft
C Absence MR
D Presence thrombus inside LA
ANSWER:D
402 Question with one false answer in AF
A Paroxysmal AF if duration less than a week
B Cardioversion by external electrical shock in AF with unstable hemodynamic situation
C Using Aspirin in AF with rheumatic fever origin is absolute recommended
D Digoxin therapy doesn’t use in AF with WPW syndrome
ANSWER:C
403 Risk factors of pyelonephritis (3)
A Frequency of recent sexual intercourse/spermicide use
B New sex partner within the prior year
C Stress incontinence in the previous 30 days
D Pregnancy
ANSWER:A
404 Symptoms of MS One false answer, which one?

A Diastolic rumbling
B Opening snap
C S1 increased
D Systolic thrill
ANSWER:D
405 The acute coronary syndrome One false answer, which one?
A Chest pain on exercise
B STEMI
C Unstable angina (NSTE ACS with troponin negative)
D NSTEMI
ANSWER:A
406 The following statement of Doll’s eyes maneuver are true except.
A Oculocephalic-reflex: Holding open the upper eyelids and turn the head quickly
B Intact brainstem: eyes move in the opposite direction
C Loss of doll’s eye movements in a comatose patient,
D suggests no lesion of midbrain
ANSWER:D
407 The following statement about Hypokalemic-are true except
A Induced by heavy meals =&gt; more insulin, which induce K+ goes into the cells
B
Muscle membrane is hypopolarized =&gt; nerve stimulation fails to elicit a muscle action
potential
C Diarrhea profuse
D Prolong fever
ANSWER:D
408 The following statement of Signs of Meningeal Irritation are true except
A Neck stiffness : patient can’t flex neck to touch chin to chest ,due to pain and spasm.
B
Brudzinski’s sign : examiner flexes sign neck of supine patient . Patient tries to reduce neck
pain by flexing hip ,
C
Kernig’s sign : patient resists sign straightening knee when hip is flexed to 90 degrees , due
to pain
D Both led cant flexed
ANSWER:D

409 The following statement of acute abdominal pain are true except
A Appendicitis
B Acute cholecystitis
C Acute cholangitis
D A caecal carcinoma, or an ovarian tumor
ANSWER:D
410 The following statement of acute upper GI bleeding are true except
A Esophagitis
B Esophagus Varice rupture
C Hemorrhagie form of gastritis
D Anus hemorrhoid bleeding
ANSWER:D
411 The following statement of cause of dysphagia are correct except
A Oropharyngeal , dysphagia
B Thyromegaly dysphagia
C Esophageal dysphagia
D Tonsilitis
ANSWER:D
412 The following statement of cause of esophageal dysphagia are correct except
A
Structural disorders-Esophageal Webs (upper)- secondary to GERD, upper sphincter no
good relaxation. sphincter GERD.
B
Schatzki Ring (lower) is located at the gastroesophageal junction lower sphincter no good
relaxation .
C Motility disorders Achalasia Diffuse esophageal spasm ,scleroderma
D Hematemesis
ANSWER:D
413 The following statement of Myasthenia gravis -are true, except
A Auto-antibodies against Ach receptors
B Weakness of cranial nerves and proximal muscles and palpebral.
C NOT Response quickly to pyridostigmine, anticholinesterase
D Long-term with immunosuppression: steroids, azathioprine, cyclosporine
ANSWER:C

414 The following statement of Polio are the most accurate true except
A Poliovirus (orofecal transmission)
B Damage the motor neurons of the brain stem and the ventral horns of the spinal cord
C Pain, weakness, paralysis, muscle atrophy
D No Supportive therapy
ANSWER:D
415 The following statement of the neuron’s definition are correct ,except;
A Nerve cells and nerve fibers)
B
Electrically excitable cells in the nervous system that function to process and transmit
information.`
C Neurons, also known as nerve cells, send and receive signals from your brain.
D
The cell body not carries genetic information, maintains the neuron’s structure, and provides
energy to drive activities.
ANSWER:D
416 The Following statement of the three basic function of CNS are correct except;
A Sensory Functions: Sensory receptors detect both internal and external stimuli.
B
B-Integrative Functions: CNS integrates sensory input and makes decisions regarding
appropriate responses
C Motor Functions: Response to integration decisions.
D The central nervous system consists of only the brain.
ANSWER:D
417 The following statement of type of neuron are true except
A Unipolar
B Bipolar
C Multipolar
D Pseudomultipolar
ANSWER:D
418 The treatment of stable angina One true answer, which one?
A Stop exercise and give Trinitrin sublingual or spray
B Admission in ICU
C Discharge treatment included BASIC
D Tenormin + risordan is not recommended

ANSWER:B
419 The treatment of unstable angina in ICU All answers are trues, except one, which one?
A Established D5% and monitoring ECG
B Unfractionated heparin
C Plavix + aspirin
D Adalate if HTN
ANSWER:C
420 Treatment of MR All treatment are trues, except one, which one?
A Medications
B Valvuloplasty in functional MR
C Annuloplasty
D Replacement
ANSWER:B
421 Treatment of RHF One true answer, which one?
A Lasilix
B ACE
C Digoxin (Sinus rhythm)
D Betablocker
ANSWER:A
422 Treatment of STEMI developing in ICU All answer are true, except one, which one?
A D5 PIV%
B Heparin (IV)
C Plavix, Aspirin
D No coronary artery angiography followed by angioplasty
ANSWER:D
423 Treatment Wilson`s disease
A Lifetime treatment with penicillamine, 1-1.5 g daily, is effective in chelating copper.
B Chelating iron
C Phlebotomy
D All of the above
ANSWER:A
424 Triggered factors of left heart failure One true answer, which one?

A Respiratory infection
B No stop treatment
C Regime no riche in salt
D Absence of heart arrhythmia
ANSWER:A
425 Trousseau sign:
A
Positive sign is ipsilateral twitching of the upper lip upon tapping the facial nerve on the
cheek.
B
Positive sign is contralateral twitching of the upper lip upon tapping the facial nerve on the
cheek.
C
Positive sign is painful carpal spasm after 3-minute occlusion of brachial artery with blood
pressure cuff.
D All of the above
ANSWER:C
426 Valve replacement One false answer, which one?
A Very calcification
B Severe stenosis
C Mechanical prosthetic valve in on patient
D Control INR
ANSWER:C
427 What are causes of Fulminant hepatic failure? The following statement are correct except
A
Drug acetominophen , isoniazid, sulfonamides, tetracycline, NSAIDS ,herbal remedies,
cocaine.
B
Virus-hepatitisA, B,C,E,herpes simplex virus, cytomegalovirus, EpsteinBarr virus,parvovirus,
varicella zostervirus,
C Virus syndrome
D
Other-mushroom intoxication, bacillus cereus, BUDD-Chiara syndrome, Acute fatty liver of
pregnancy, HELLP syndrome, autoimmune hepatitis.
ANSWER:C
428 What are the -Coordination : cerebellar function? the following statement are true except
A Gait :ataxia ( cerebellar disease )
B Tandem gait,. Finger to nose,. Heel to shin,. Rapid alternating movement

C Romberg ‘s test
D Patient sitting down
ANSWER:D
429 What are the cause of jaundice? The following statement are true except
A Cause-Prehepatic-neonatal, haemolytic, gilbert syndrome,
B
Hepatocellular –viral hepatitis, cirrhosis, alcoholic hepatitis, hepatic metastases, drug induced
, autoimmune hepatitis, liver abscess, hepatoma.lymphoma, leptospirosis, BUDD-Chiari
syndrome,
C
Cholestatic-common duct stones, pancreatic cancer, primary biliary cirrhosis,
cholangiocarcinomasclerosing cholangitis, Benign stricture pancreatitis
D Viral syndrome
ANSWER:D
430 What are the cause of ascites? The following statement are correct except
A Causes-due either to chronic liver disease or carcinomatous
B
Rapid onset of ascites is a feature of decompensated ciorrhosis, malignasncy (including
hepatoma), portal or splenic vein thrombosis or Budd chiari syndrome.
C Ascites in an alcoholic cirrhotic with pancreatitis may be due to hepatic decompensation .
D Virus syndrome
ANSWER:D
431 What are the disease can cause of Dysphagia? The following statement are correct except
A Stroke
B GI cancer
C Scleroderma
D URTI
ANSWER:D
432 What are the treatment of ascites?. The following statement are true except
A Investigations-paracentesis,
B IV furosemide,
C Propranolol, Spironolactone
D IVfluid of NSSO,9%
ANSWER:D
433 What are the Acute intestinal ischemia? The following statement are true except

A Severe abdominal pain in an elderly patient
B Atrial fibrillation, or vasculitis with abdominal pain .
C Rectal bleeding
D Chronic diarrhea
ANSWER:D
434 What are the assess memory? the following statement are true except
A Short-term memory (or recent memory)
B Long-term memory (or remote memory)
C Ask about date of birth, names of school attended
D Give 3 words to the patient, but the patient cant’s say “cat”, “pen”,
ANSWER:D
435 What are the cardio-vascular complications of HTN, except one is false?
A Left ventricular hypertrophy (LVH)
B Ischemic heart disease (IHD)
C Heart failure
D Atrial-ventricular block
ANSWER:D
436 What are the cause of Acute hepatic failure? The following statement are correct except
A Paracetamol overdose and viral hepatitis
B Alcohol, Wilson’s disease, Budd Chiari syndrome,
C HELLP syndrome( hemolysis).
D Viral syndrome
ANSWER:D
437 What are the cause of Parkinson Disease?The following statement are correct except
A Certain nerve cells (neurons) in the brain gradually break down or die.
B
Many of the symptoms are due to a loss of neurons that produce a chemical messenger in
your brain called dopamine.
C
When dopamine levels decrease, it causes abnormal brain activity, leading to impaired
movement and other symptoms of Parkinson's disease.
D NeuroSyphylis.
ANSWER:D
438 What are the cause of Peripheral neuropathy? the following statement are true except

A Metabolic: diabetes, hypothyroid, alcoholism, Vit B deficiency, uremia
B Toxic: drugs (nitrofurantoin, vincristin), arsenic/lead, organophosphates
C Infection: HIV, CMV, HZV, Leprosy, Lyme, Immune/inflammation: GB
D Gastrointestinal bleeding
ANSWER:D
439 What are the Causes of acute cerebellar ataxia? the following statement are true except
A Autoimmunity to Purkinje cells or other neural cells in the cerebellum
B CNS vasculitis, Multiple sclerosis, Infection ,Bleeding ,Tumor
C Direct injury, Toxins(eg,alcohol),Genetic disorders
D URTI
ANSWER:D
440 What are the cerebellum? the following statement are true except
A The cerebellum is largely involved in "coordination".
B
Persons whose cerebellum doesn't work well are generally clumsy and unsteady. They may
look like they are drunk even when they are not.
C
Acute cerebellum ataxia-is sudden , uncoordinated muscle movement due to diseases or
injury to the cerebellum in the brain
D Parkinson’s disease
ANSWER:D
441 What are the cerebral cortex ?the following statement are true except
A Cognition
B Voluntary Movement.
C Sensation
D Speech movement
ANSWER:D
442 What are the clinical gastric carcinoma?The following statement are correct except
A Dyspepsia is usual,.
B Anorexia and weight loss.
C Haematemesis
D A supraclavicular node (Virchow’s node) is uncommon.
ANSWER:A
443 What are the definition of acute pancreatitis? The following statement are correct except

A
Abdominal pain as burning persistent radiated to the back , nausea vomiting and fever
gradually worse by eating
B Blood test ,Rise amylase four times the upper limit of normal .
C U/S abdomen
D Abdominal pain as burning at epigastric no radiated any where
ANSWER:D
444 What are the function of CNS ?The following statement are correct except
A The central nervous system (CNS) consists of the brain and spinal cord.
B
This body system is responsible for integrating and coordinating the activities of the entire
body
C
Through these physical structures, thought, emotion, and sensation are experienced, and
body movements are coordinated.
D CNS controlled only thermoregulation.
ANSWER:D
445 What are the Hepatorenal syndrome? The following statement are correct except
A
Characterzed by renal vasoconstriction that leads to severe reduction in glomerullar function
with minimal histology abnormalities.
B
Characterized by a rapidly progressive reduction in kidney function, defined by adoubleing
of the initial serum creatinine to greater than 2,5mg /dl(221micromoles/L or 50% reduction
of the initial 24h creatinin clearance to a level less than 20ml/min/1.73 in less than 2 weeks.
C Is the late state of complication of cirrhosis.
D Severe dehydration due to diarrhea.
ANSWER:D
446 What are the Mechanical intestinal obstruction? The following statement are true except
A Ileus,
B volvulus
C intussusception
D Intestinal polyp
ANSWER:D
447 What are the peritonitis ?The following statement are correct except:
A Fever, abdominal pain severe, guarding, rebound tenderness and rigidity .

B PE-abdominal Bowel sound absent.
C Local-appendicitis, cholecystitis, diverticulitis, Pancreatitis
D Acute diarrhea
ANSWER:D
448 What are the prophylaxie of portal hypertension.The following statement are correct except
A
AS Variceal prophylaxis- Sclerotherapy or banding is not indicated unless bleeding has
occurred.
B Varices recur after obliteration in about 40%
C Propranolol 20-40mg three time daily decrease the risk of bleeding
D Prophylaxie by cetirizine
ANSWER:D
449 What are the Spontaneous bacterial peritonitis The following statement are correct except
A Is the complication of the cirrhosis
B The prevalence of spontaneous bacterial peritonitis (SBP)-` 50% to 15% is diagnostic.
C
when ascitis fluid bacterial culture-is positive and ascitis fluid absolute polymorphonuclear
count is elevated&gt;250.
D URTI due to virus.
ANSWER:D
450 What are the Toxic dilatation of the colon?.The following statement are true, except
A
Dilatation of the colon with signs of systemic upset is becoming less common as acute attack
of ulcerative colitis.
B Colitis usually caused by severe ulcerative colitis
C abdominal pain as cramp F/U diarrhea chronic fever, neutrophils, tachycardia.
D Radiology –No colonic dilatation (widest diameter &lt;60 cm
ANSWER:D
451 What are the Zollinger Ellison syndrome?The following statement are correct except
A Gastrin secreting –neuroendocrine tumors
B Recurrent severe duodenum ulceration and diarrhea,
C
Localization is difficult because multiple tumours,in the duodenal sub-mucosa, and
metastases are common
D As the same GERD

ANSWER:D
452 What is a diastolic HTN ? One true answer, which one?
A BP ≥ 160/90mmHg
B SBP ≤ 160 and DBP ≥ 90mmHg
C BP at time the heart contracts
D Most often in young person
ANSWER:D
453 What is an ascites ?The following statement are correct except
A Free fluid in the peritoneal cavity .
B
It is thought that peripheral vasodilatation is the initial event , possibly due to impaired
hepatic metabolic of endogenous vasodilators.
C
This decreases renal blood flow, which stimulates the renin-angiotensin-aldosterone axis,
leading to salt and water retension.
D causes fluid to accumulate intra lumen cavity
ANSWER:D
454 What is an urgency hypertensive ? One true answer, which one?
A Chronic elevation BP
B BP ≥ 180/110mmHg increased suddenly
C With organ damages
D Using of long acting antihypertensive drug
ANSWER:B
455 What is masked HTN One false answer, which one?
A BP is high at cabinet’s doctor
B Has complication higher than permanent HTN
C BP is increased at patient’s house
D Need self measurement BP
ANSWER:B
456 What is the Budd Chiari syndrome ?he following statement are correct except
A
Characterized by hepatic venous outflow tract obstruction (including the supra-hepatic
inferior vena cava)
B
Can be defined as any pathophysiologic process that results in interruption or diminution of
the normal blood flow out of the liver

C
implies thrombosis of the hepatic veins and/or the intrahepatic or suprahepatic inferior vena
cava
D Characterized by hepatic venous inflow and outflow tract obstruction.
ANSWER:D
457 What is the treatment of varical prophylaxia.The following statement are true except.
A The highest risk of rebleeding is in the first 6 weeks after the initial bleed.
B
Propranolol 20-40mg three time daily decreases the risk of bleeding and is indicated if it can
be tolerated. It is the treatment of choice for portal hypertensive gastropathy.;
C IV Omeprazol
D
Endoscopic variceal ligation every 2 4 weeks, obliterate varices, endoscopy 1-3months after
obliteration , then every 6-12 months
ANSWER:C
458
A 42-year-old man underwent allogeneic bone marrow transplantation from an unrelated
donor for acute myeloid leukemia in second remission. He develops hepatomegaly and fever
3 months after the infusion of the allogeneic cells. Radiographic imaging of the abdomen
reveals extensive intraabdominal lymphadenopathy as well as an enlarged liver, which
appears to be diffusely infiltrated. There is no ascites noted. Which of the following is the
likely mechanism for the patient’s current deterioration? ?
A Venoocclusive disease of the liver
B Overgrowth of EBV-infected lymphocytes
C Recurrence of acute myeloid leukemia
D Graft-versus-host disease
ANSWER:B
459
A 43-year-old nurse whose job requires frequent hand washing has noted a small erosive
skin lesion between the third interdigital web of the right hand (Plate A). The best therapy
for this condition would be ?
A topical 5-fluorouracil
B topical clotrimazole
C oral griseofulvin
D topical clotrimazole
ANSWER:B

460
A 24-year-old man is concerned because of the appearance of several light brown spots on
his trunk (Plate D). The lesions (limited to the chest, back, abdomen, and upper arms) are flat
and sharply marginated and have a fine scale that is easily scraped off. The most appropriate
diagnostic study is ?
A Giemsa stain of scraped material (Tzanck preparation)
B bacterial culture of the lesions
C fungal culture of the lesions
D microscopic examination of potassium hydroxide- treated scrapings
ANSWER:D
461 Which statements concerning Acinetobacter is correct ?
A This organism is often confused with Neisseria on Gram stain
B This organism is often mistakenly identified as a diphtheroid on Gram stain.
C
This organism is a member of the Enterobacteriaceae family on the basis of its appearance on
routine laboratory culture media.
D This organism is usually sensitive to penicillin and ampicillin
ANSWER:A
462 Correct statements concerning melioidosis include which of the following? ?
A Infection is usually caused by person-to-person transmission.
B Patients with pneumonia usually have relatively few organisms in the sputum.
C The diagnosis usually depends on serologic testing.
D
Therapy with a combination of two or three antibiotics is recommended for severely ill
patients.
ANSWER:D
463
For the past 2 days, a 24-year-old woman has had fever and pain in the left wrist, right
ankle, and left knee. Nine painful skin lesions are present on the distal extremities,
predominantly about the jo ?
A herpes simplex
B meningococcemia
C gonococcemia
D erythema multiforme
ANSWER:C

464
A 19 year old man, presents to the Emergency Department with a supra-condylar fracture of
the distal humerus. Later that evening, after surgical fixation he is noted to have a wrist drop
with a patch of numbness on the dorso-radial aspect of the wrist.What is this most likely to be
due to?
A A radial nerve injury
B An ulnar nerve injury
C A musculo-cutaneous nerve injury
D An anxilliary nerve injury
ANSWER:A
465
In the management of pneumothorax, what is the best indication that the intercostal catheter
can be removed?
A The size of the pneumothorax is less than 25% of the lung volume on chest x-ray
B There is lack of bubbling in underwater seal drain with suction
C There is lack of bubbles with cough in underwater seal drain
D There is presence of swing and no bubbling in underwater seal drain
ANSWER:A
466
A 75 year old man, presents with painless loss of vision in the left eye one hour ago.
VAR:6/9, VAL:light perception only. There is a left afferent pupil defect. He has been on an
ACE inhibitor for hypertension. What do you see when you look into his left eye with a
direct ophthalmoscope?
A Signs of age-related macular degeneration
B Some blot haemorrhages and hard exudates near the macula
C A dense nuclear cataract makes the view of a probably normal optic disc and macula difficult
D
The macula has an orange/red spot and the surrounding retina looks pale, with marked
attenuation of the retinal arterioles and segmentation of blood in the retinal veins
ANSWER:D
467
A 26 year old woman has a history of wheezing as a child. She has suffered from seasonal
rhinitis in recent years. She is brought in by ambulance from a night club having developed
severe shortness of breath. Which of the following findings is in keeping with a severe
asthma attack?
A Respiratory rate of 8/min

B Blood pressure of 85/60 mmHg
C Inspiratory rhonchi on auscultation
D 88 percent oxygen saturation on pulse oximetry
ANSWER:D
468
A 26 year old nurse has had wheezy bronchitis as a child. She went to a night club and
suffered an attack of asthma. She is given nebulised salbutamol in the Emergency
Department. Which of the following is the best explanation of why Salbutamol produces
bronchodilation?
A Antagonises muscarinic cholinergic receptors on airway smooth muscle
B Increases cyclic AMP in airway smooth muscle cells
C Inhibits the sodium channels in airway epithelial cells
D Increases the firing rate of neurones in the brain stem respiratory centre
ANSWER:B
469
A 25 year old man who sustains a clean, fairly superficial, 3cm long laceration to his left
forearm on a piece of broken glass. Which one of the following statements about
management is true?
A Prophylactic antibiotics are required
B Thorough cleansing with saline is the decontamination method of choice
C Immediate tetanus prophylaxis is required as this is a tetanus-prone wound
D X-ray is required even if the whole length and depth of the wound can be visualized
ANSWER:B
470 Regarding the anatomy of the lungs, which of the following statements is correct?
A The left lung is larger and heavier than the right
B The lungs are supplied with oxygenated blood by the pulmonary arteries
C The lungs are invested by parietal pleura
D The apices of the lungs may pass above the thoracic inlet (1st rib)
ANSWER:D
471
A 30 year old woman, presents with a 2-day history of fever, vomiting and mild right-sided
flank pain. She has evidence of right renal angle tenderness on examination. Urinalysis
shows the presence of polymorphonuclear leucocytes and nitrates. What would be the next
most appropriate investigation?
A Abdominal ultrasound

B Urine microscopy and culture
C Intravenous pyelogram
D Serum creatinine levels
ANSWER:B
472 Which of the following does the median nerve innervate?
A All the muscles moving the thumb
B Opponens pollicis
C Adductor pollicis
D The skin on the dorsum of the 5th digit
ANSWER:B
473
A 69 year old man presents to the doctor with an acutely swollen right knee. He has had
previous episodes of acute pain and swelling in his big toes and ankles over the last 10 years.
What is the most appropriate initial investigation to determine the diagnosis?
A X-ray of the knee
B Microscopy of synovial fluid
C Full blood count
D Serum uric acid
ANSWER:B
474
A 75 year old woman who presents with a six-month history of pain in her right knee, which
is worse with prolonged walking. She has no relevant past history. Physical examination
reveals a small effusion and crepitus on movement. Her body mass index is 32. You make a
diagnosis of osteoarthritis Which of the following is the most important initial aspect of
treatment?
A Paracetamol
B Non-steroidal anti-inflammatory drugs
C Arthroscopy plus lavage
D Avoidance of walking
ANSWER:B
475
Mrs Betty Smith is a 75 year old woman with osteoarthritis for whom you prescribe
diclofenac. Which one of the following statements about diclofenac sodium is correct?
A It inhibits both Cox I and Cox II isoenzymes
B It does not affect platelet aggregation

C It does not reduce renal blood flow in normal kidneys
D It has no analgesic properties
ANSWER:A
476
A 22 year old woman who is an active intravenous drug user presents to her general
practitioner, six weeks pregnant. On investigation she is found to be positive for anti-
hepatitis C virus (HCV). Her hepatitis A and B and HIV serology are negative. What advice is
most appropriate for this patient at this point?
A The risk of transmission of Hepatitis C to the fetus is > 80%
B She should be vaccinated against hepatitis A and B unless she is already immune
C She should commence antiviral treatment
D She can expect to develop symptoms of hepatitis as her pregnancy progresses
ANSWER:B
477 What is the most important direct action of the hormone 1,25 Dihydroxyvitamin D?
A Decrease bone resorption
B Increase gut calcium absorption
C Reduce urine calcium losses
D Potentiate the action of parathyroid hormone
ANSWER:B
478
With regard to the pathology of rheumatoid arthritis, which of the following statements is
correct?
A Rheumatoid arthritis involves amphiarthroses (fibrocartilagenous joints)
B Pannus destroys articular cartilage and underlying bone
C Rheumatoid erosions are typically subchondral
D Rheumatoid nodules are typically caseating granulomas
ANSWER:B
479
A 28 year old man, is involved in a head-on vehicle collision. He had allegedly been
drinking at a pub with friends and left after a dispute. He was initially trapped in the car with
the major impact on his right thigh. He is brought into the Emergency Department
conscious, but pale with a pulse of 135bpm and blood pressure of 90/60mmHg. He has
deformity of his mid thigh with visible bone protruding from a wound. Which of the
following is the correct terminology to describe his fracture?
A Greenstick fracture

B Pathological fracture
C Compound fracture
D Impacted fracture
ANSWER:C
480
A 28 year old man, is involved in a head-on vehicle collision. He had allegedly been
drinking at a pub with friends and left after a dispute. He was initially trapped in the car with
the major impact on his right thigh. He is brought into the Emergency Department
conscious, but pale with a pulse of 135bpm and blood pressure of 90/60mmHg. He has
deformity of his mid thigh with visible bone protruding from an anterior wound. Which of
the following is the most likely early complication?
A Femoral artery injury
B S1 nerve root injury
C Ruptured bladder
D Pulmonary embolus
ANSWER:A
481
Soft tissue healing is an ordered process. Which of the following is NOT a stage in this type
of wound healing?
A Acute inflammatory reaction
B Formation of fibrous tissue
C Granuloma formation
D Formation of the coagulum
ANSWER:C
482
A 28 year old man is travelling in Thailand when he is involved in a motor vehicle accident.
He is taken to a local hospital with visible bone protruding from a wound in his mid-thigh.
He undergoes transfusion with multiple units of whole blood as part of his immediate
treatment. Six weeks later he develops jaundice. He undergoes a series of tests to investigate
his jaundice. Which of the results suggests that his jaundice is likely to be related to his blood
transfusion?
A The presence of Anti-Hepatitis C Virus antibody
B Elevated Alanine amino-transferase
C The presence of Anti-Hepatitis B Surface Antigen antibodies
D Elevated gamma-glutamyl transferase

ANSWER:A
483
A 54 year old asthmatic woman, presents for a routine check up and isfound to have a blood
pressure of 150/90mmHg. Examination reveals she is moderately overweight (BMI 28
kg/m²), in sinus rhythm with a heart rate of 80 beats per minute and has an audible fourth
heart sound. Which one of the following clinical features is NOT relevant to the possible
causation of hypertension in this patient?
A A history of snoring
B The detection of a carotid bruit
C The detection of an abdominal bruit
D A history of regular alcohol consumption
ANSWER:B
484
Gail Tran is a 39 year old woman who is 20 weeks pregnant. You have just received her
amniocentesis result which shows: 47 XY Trisomy 21. What is the most common cause of a
left to right cardiac shunt in an infant with Trisomy 21?
A Ventricular septal defect
B Atrial septal defect
C Tetralogy of Fallot
D Atrial septal defect with pulmonary hypertension
ANSWER:A
485 Regarding the blood supply to the heart, which of the following statements is correct?
A The left and right coronary arteries arise from the arch of the aorta
B The left and right coronary arteries are usually of equal length
C The anterior interventricular artery is a branch of the right coronary artery
D The right coronary artery descends on the heart between the right atrium andventricle
ANSWER:D
486 Regarding the anatomy of the heart, which one of the following statements is most correct?
A The coronary arteries are the first branches of the arch of the aorta
B Papillary muscles are located in the atria and the ventricles of the heart
C The coronary sinus drains directly into the right atrium
D The openings of the pulmonary veins into the left atrium are usually guarded by valves
ANSWER:C

487
SOK, aged 63, is in left ventricular failure following a myocardial infarct. Which of the
following is the most appropriate to maximise life expectancy?
A Digoxin - to increase the force of cardiac contraction
B ACE inhibitors - to inhibit the renin-angiotensin system
C Beta-adrenoreceptor agonists - to increase the force of cardiac contraction
D Alpha-adrenoreceptor antagonists to reduce systemic vascular resistance
ANSWER:B
488
A 36 year old woman who presents with palpitations and is found to have significant mitral
stenosis. Which of the following arrhythmias is the most likely cause of her palpitations?
A Atrial fibrillation
B Ventricular fibrillation
C Atrial tachycardia
D Ventricular tachycardia
ANSWER:A
489
Where should the normal apex beat be palpated whilst undertaking a cardiovascular
examination?
A Left 5th intercostal space, anterior anxillary line
B Left 5th intercostal space, mid-clavicular line
C Left 6th intercostal space, mid-clavicular line
D Left 6th intercostal space, anterior axillary line
ANSWER:B
490
Mr. Wong, a 53 year old office worker, is found by his General Practitioner to have a blood
pressure of 150/95mmHg. Which one of the following findings is most likely?
A Normal renal function
B A renal artery bruit
C An elevated adrenocorticotropic hormone (ACTH) level
D An elevated adrenaline level
ANSWER:A
491 Which one of the following is the specific treatment for paracetamol overdose?
A Activated charcoal
B Naloxone

C Celecoxib
D N-acetyl cysteine
ANSWER:D
492
The cranial nerves IX, X, XII are characterised by the fact that they all emerge from which
one of the following structures?
A Midbrain
B Pons
C Medulla
D Thalamus
ANSWER:C
493 Which one of the following statements about the skeleton is true?
A The ulna is lateral to the radius
B The metacarpals are numbered from 1 to 5 from the medial side
C The metatarsals are numbered from 1 to 5 from the medial side
D There are 14 phalanges in the hand and 13 phalanges in the foot
ANSWER:C
494
A 30 year old woman presents with polyuria and is diagnosed with diabetes mellitus. She has
a blood glucose level of 24 mmol/L. Which of the following is the strongest indication to
commence insulin treatment immediately?
A Her very high blood glucose level
B Her age
C Her symptomatic hyperglycaemia
D Presence of moderate blood ketones
ANSWER:A
495
Macrocytosis refers to enlarged red cells while the term megaloblastosis refers to a specific
morphological appearance observed in haemopoietic cells forming in the absence of
adequate amounts of either vitamin B12 or folate. Which of the following morphological
features in a peripheral blood smear indicates megaloblastic haemopoiesis?
A Hypersegmented neutrophils
B Target cells
C Platelet clumps
D Reduced neutrophil granulation

ANSWER:A
496
Which one of the following is the correct order of tissues lining the gastrointestinal tract from
the lumen (inside) outwards?
A Epithelium, lamina propria, muscularis mucosa, submucosa, muscularis externa
B Epithelium, muscularis mucosa, lamina propria, submucosa, muscularis externa
C Epithelium, lamina propria, muscularis externa, submucosa, muscularis mucosa
D Epithelium, lamina propria, muscularis mucosa, muscularis externa, submucosa
ANSWER:A
497 Which of the following is true regarding surgical therapy for colorectal cancer?
A Adjuvant therapy is indicated for all patients with colorectal cancer
B The goal of adjuvant therapy is to improve the chances of cure
C Adjuvant radiotherapy is indicated for patients with colon cancer
D Adjuvant chemotherapy for colon cancer should continue for at least 12 months
ANSWER:B
498 What is the laryngeal prominence (Adam's apple) in the neck formed by?
A Epiglottis
B Hyoid bone
C Thyroid cartilage
D Cricoid cartilage
ANSWER:C
499
What is the most useful guide to the likely post-operative opioid requirements of a patient
undergoing a laparotomy?
A The weight of the patient
B The estimated lean body mass of the patient
C The magnitude of the surgery
D The age of the patient
ANSWER:C
500
A 55 year old woman, presents to the Emergency Department with a 24 hour history of right
upper quadrant pain, passing dark coloured urine and a fever of 38.7ºC. Abdominal
ultrasound shows multiple gallstones in the gallbladder and adilated common bile duct of 1.5
cm. What is the most likely diagnosis?
A Acute appendicitis

B Acute pancreatitis
C Acute cholecystitis
D Acute cholangitis
ANSWER:D
501 Which of the following statements about the autonomic nervous system is most accurate?
A
The cell bodies of sympathetic preganglionic neurons are located in the ventral horn of the
thoracic and lumbar segments of the spinal cord
B
In response to stress, sympathetic nerves regulating different effectors tend to be activated
simultaneously
C
Sympathetic preganglionic neurons receive direct inputs from primary afferent nerves that
enter the spinal cord at the same segmental level
D Most sympathetic and parasympathetic ganglia are remote from their target organs
ANSWER:B
502
A 19 year old girl presents to the Emergency Department with a one day history of
drowsiness, headache, fever and photophobia. There is no history of trauma, and she doesn't
have a rash. Which of the following is the most appropriate next step in managing this
patient?
A Prescribe opioids to treat her headache
B Administer intravenous antiobiotics urgently after blood cultures
C CT scan of the brain
D Lumbar puncture with CSF sampling
ANSWER:D
503
Radiotherapy with or without chemotherapy can be used to cure (without surgery) all of the
cancers listed EXCEPT which of the following below?
A Lymphoma
B Anal cancer
C Colon cancer
D Cervix cancer
ANSWER:A
504
Which of the following pharmacological agents used in the management of type 2 diabetes is
NOT associated with significant weight gain?
A Metformin

B Sulphonylureas
C Short acting insulin
D Thiazolidinediones
ANSWER:A
505
Which of the following is NOT a recognised side effect of non-steroidal antiinflammatory
drugs (NSAIDs)?
A Peptic ulceration
B Squamous cell carcinoma of the skin
C Renal impairment
D Aggravation of asthma in some patients
ANSWER:B
506
A patient with Parkinson's disease has been treated successfully for several years with a
levodopa based medication. He has recently developed prominent involuntary writhing
movements of the limbs. Which of the following terms describes this phenomenon?
A Hypokinesia
B Akinesia
C Bradykinesia
D Dyskinesia
ANSWER:D
507
A 64 year old female presents with a two-year history of perianal itching and irritation more
noticeable in the past month. Inspection of the perineum shows an area of hyperkeratosis,
leucoplakia and an associated large irregular ulcer on the anal verge in the mid-line
posteriorly. What should be the next step in her management?
A Perform a wide local excision
B Perform an incisional biopsy
C Patient to apply rectogesic ointment 5 percent (nitroglycerin-base)
D Perform an internal anal sphincterotomy
ANSWER:B
508
Which of the following is the least important factor contributing to Total lung capacity
(TLC)?
A Age
B Height

C Lung elastic recoil
D Gender
ANSWER:D
509
Which of the following has the least effect on the diffusing capacity for carbon monoxide of
the lung at rest?
A The physical properties of carbon monoxide
B Pulmonary capillary blood volume
C Thickness of the blood-gas barrier
D Haemoglobin level in the blood
ANSWER:A
510
A 45 year old man is involved in a motor vehicle accident. He presents 2 weeks later with
pain involving the right shoulder, scapula and upper limb. What sensory findings would be
suggestive of a C6 radiculopathy?
A Numbness in a patch over the deltoid
B Parasthesia over the upper trapezius
C Numbness in the axilla
D Paraesthesia of the index finger and thumb
ANSWER:D
511 Which of the following joints are affected most commonly by osteoarthritis?
A Metacarpophalangeal joint
B Ankle joint
C Elbow joint
D Distal interphalangeal joint
ANSWER:D
512 How does chronic suppurative otitis media most commonly present?
A Recurrent painless aural discharge
B Acute aural discharge accompanied by pain
C Hearing loss
D Facial palsy
ANSWER:C
513
Mary is a 17 year old student who presents complaining of recurrent nasal discharge,
irritation and sneezing. What is the most likely cause of her complaint?

A Deviated nasal septum
B Infective sinusitis
C Nasal foreign body
D Allergic rhinitis
ANSWER:D
514 Which of the following is NOT an indication for tracheotomy?
A Acute airway obstruction
B Prolonged ventilation
C Recurrent bronchial toilet
D Moderate sleep apnoea
ANSWER:D
515
Chandra, a 50 year old Indian migrant, presents with a two week history of high spiking
fever and right upper quadrant pain. A CT scan of the abdomen shows features of a liver
abscess. Ultrasound guided aspiration of the collection shows fluid that looks like anchovy
sauce. What is the most likely diagnosis?
A Staphylococcus liver abscess
B Amoebic liver abscess
C Hydatid liver abscess
D Tuberculous liver abscess
ANSWER:B
516
A 17 year old girl, presents with a 1-day history of central abdominal pain that then localises
to the right lower quadrant of the abdomen. This is associated with nausea, decreased
appetite and a temperature of 37.8°C. What is the most appropriate initial management of this
patient?
A β HCG
B Pelvic ultrasound
C Urgent O&G review to exclude pelvic inflammatory disease
D Laparoscopy
ANSWER:B

517
A 24 year old student, presents at the Emergency Department with sudden onset of a cough
and high fever. Chest X-ray shows a left lower lobar consolidation and further history
reveals frequent sinus infections requiring antibiotics, and two episodes of lobar pneumonia
in the past 5 years. Which of the following organisms is most likely to be responsible for his
repeated infections?
A Haemophilus influenza
B Streptococcus pneumonia
C Moraxella catarrhalis
D Pneumocystis jiroveci
ANSWER:B
518
Type I diabetes mellitus is caused by an auto-immune process. Which of the following
mechanisms is responsible for this?
A Anti-insulin antibodies
B Anti-glutamic acid decarboxylase antibodies
C Infiltrate of islets by CD4+ and CD8+ T cells
D Ant-islet cell antibodies
ANSWER:C
519
A 39 year old receptionist with a history of chronic asthma, has her blood pressure checked
by her GP. It is found to be 165/95 mmHg. The patient’s body mass index is 32 kg/m² (NR:
20 -25kg/m 2 ) and the remainder of the physical examination is normal. Urinalysis is
negative for blood and protein. Which of the following anti-hypertensive agents is
contraindicated in this case?
A Perindopril
B Atenolol
C Nifedipine
D Hydrochlorothiazide
ANSWER:B
520
An 85 year old woman comes to see you, her local doctor, with her daughter. Her daughter
tells you that her mother has become increasingly confused over the last week. She has also
developed increasing generalised weakness, poor mobility and her overall ability to carry out
her activities of daily living has deteriorated. Which of the following investigations would
you perform first?

A Electrolytes
B Computed tomography (CT) scan of the brain
C Full blood count
D Dipstick urine analysis
ANSWER:B
521 What is the main vein draining the brain?
A Internal jugular vein
B External jugular vein
C Internal carotid vein
D External carotid vein
ANSWER:A
522
Which of the following micro-organisms is the commonest cause of communityacquired
pneumonia?
A Haemophilus influenzae
B Klebsiella pneumonia
C Streptococcus pneumoniae
D Pseudomonas aeruginosa
ANSWER:B
523 How is an ""open"" fracture defined?
A There is a full thickness break of the bone
B Requires surgical (open) reduction
C Requires antibiotic therapy
D Associated with a breach in the overlying skin
ANSWER:D
524 Which of the following is NOT a stage in normal soft tissue wound healing?
A Acute inflammatory reaction
B Formation of fibrous tissue
C Granuloma formation
D Formation of the coagulum
ANSWER:C
525
Which of the following is the main risk factor for ischaemic heart disease in men and women
older than 75?

A Hypertension
B Age
C Cholesterol
D Smoking
ANSWER:C
526
Which of the following tests is most likely to establish a definitive microbiological diagnosis
in a patient with community-acquired pneumonia?
A Gram stain of sputum
B Bacterial culture of sputum
C Bacterial culture from a throat swab
D Blood culture
ANSWER:A
527 Which of the following investigations is useful in the monitoring of rheumatoid arthritis?
A C- reactive protein
B Anti DNA antibodies
C X-rays of the lumbar spine
D Serum uric acid level
ANSWER:A
528
A 45 year old woman presents with a 3 month history of arthritis. Physical examination
reveals tender swollen metacarpo-phalangeal joints in a symmetrical pattern. What is the most
likely diagnosis?
A Rheumatoid arthritis
B Systemic lupus erythematosis
C Reactive arthritis
D Gout
ANSWER:A
529
A 58 year old man is treated with diuretics for his mild essential hypertension. His blood gas
results are shown below: PaO2 = 92 mmHg (normal range: 80-100 mmHg), PaCO2 = 44
mmHg (normal range: 35-45 mmHg), pH = 7.50 (normal range: 7.35-7.45), BE = +10
mmol/L (normal range: -2-+2 mmol/L) , Which of the following best describes this picture?
A Compensated metabolic acidosis

B Uncompensated metabolic alkalosis
C Compensated respiratory acidosis
D Uncompensated respiratory alkalosis
ANSWER:B
530
Jacob Butler is an 18 year old motorcyclist who is brought to hospital by ambulance
following a single-vehicle crash. He has a compound fracture of the right femur and is
agitated and disorientated. Which of the following is NOT an immediate priority?
A Assess air-entry and position of the trachea
B Check upper airways to ensure patency
C Urgent full blood count
D Urgent blood sugar level
ANSWER:D
531 Hyperalgesia is defined as which of the following?
A Loss of sensitivity to pain
B Increased sensitivity to touch
C Increased sensitivity to pain
D Pain response to non-noxious stimulus
ANSWER:C
532 Regarding the uterine tubes, which of the following statements is NOT correct?
A They have smooth muscle in their wall
B They connect to the junction of body and fundus of uterus
C They are most dilated at the isthmus
D They are located in the upper part of the broad ligament
ANSWER:C
533 Regarding the scapula, which one of the following statements is correct?
A It helps form the acromioclavicular joint
B It is not involved in force transfer from the upper limb to the thorax
C The suprascapular notch transmits the radial nerve
D Only two of the musculotendinous cuff muscles attach to it
ANSWER:A
534
Which of the following terms best describes the usual response of cardiac myocytes to
hypertension?

A Atrophy
B Metaplasia
C Hyperplasia
D Hypertrophy
ANSWER:D
535
During pregnancy, which one of the following infectious diseases has the lowest risk of
causing congenital infection?
A Toxoplasmosis
B Tuberculosis
C Rubella
D Listeriosis
ANSWER:B
536 Which of the following statements about right-sided heart failure is true?
A Pitting oedema of dependent parts is a common sign
B Pulmonary oedema is a consequence
C When the disorder is a result of lung disease, left ventricular hypertrophy is likely
D The liver compensates with cirrhosis and shrinkage
ANSWER:A
537 Maternal cardiovascular changes during pregnancy include which one of the following?
A A decrease in cardiac output
B Systemic vasoconstriction
C Vasoconstriction of the renal arterioles
D A decrease in red blood cell production
ANSWER:D
538 Regions that are innervated by the ulnar nerve include which of the following?
A The lateral epicondyle of the humerus
B Skin over the palmar surface of the thumb
C The 1st and 2nd (lateral) lumbrical muscles
D Skin over the palmar surface of the little (5th) finger
ANSWER:D
539
In the context of drug use, which one of the following statements best describes the
difference between addiction and dependence?

A Dependence is the diagnostic term
B There is no difference between the terms
C A withdrawal syndrome is always present in addiction
D Addiction is an antiquated term which has been replaced with dependence
ANSWER:C
540 Regarding genital chlamydial infections, which of the following statements is FALSE?
A The period of infectivity is 7-14 days after infection
B It can be treated with a single dose of an antibiotic
C It can lead to pelvic inflammatory disease (PID) in females
D Polymerase chain reaction (PCR) assays are useful for diagnosis
ANSWER:A
541
If a patient has an isolated iron deficiency, which of the following sets of results would you
expect to find?
A Low serum iron, normal serum transferrin and low serum ferritin
B Low serum iron, high serum transferrin and low serum ferritin
C Low serum iron, high serum transferrin and normal serum ferritin
D Low serum iron, low serum transferrin and high serum ferritin
ANSWER:B
542 Which of the following is a feature of the inferior vena cava?
A It is formed by the union of brachiocephalic veins
B It empties blood into the right atrium
C It drains the lower lobe of the lungs
D It is formed by the union of portal and superior mesenteric veins
ANSWER:B
543
Smoking is a known risk factor for a number of cancers. Smoking cessation can modify a
person’s risk of these cancers. Despite this, smoking cessation has NOT been shown to
reduce the risk of which of the following?
A Oesophageal cancer
B Pancreatic cancer
C Colon cancer
D Lung cancer
ANSWER:D

544
Which of the following agents/devices used in the treatment of pulmonary embolism is
associated with the highest risk of serious bleeding?
A Unfractionated heparin
B Low molecular weight heparin
C Tissue plasminogen activator (TPA)
D Inferior vena caval filters
ANSWER:C
545
An obese 65 year old female with renal failure has had a laparotomy. On day five post
operative she develops copious serous ooze from her midline abdominal wound. She is
afebrile. This is likely to be due to which of the following?
A Wound abscess
B Wound dehiscence
C Normal wound healing
D Allergic reaction to the sutures
ANSWER:B
546
A 15 year old male presents with central periumbilical pain, a low grade temperature and
nausea. Over the next couple of hours, the pain shifts to the right iliac fossa. Which of the
following is the most likely diagnosis?
A Mesenteric lymphadenitis
B Acute appendicitis
C Crohn's disease
D Pelvic inflammatory disease
ANSWER:B
547 What is an advantage of tramadol over morphine?
A Fewer interactions with antidepressants compared with morphine
B Less respiratory depression
C Less likely to cause seizures
D Less diarrhea
ANSWER:B

548
Sara presents to the Emergency Department with 5 days of cough, 2 days of fever and
increased lethargy. Her oxygen saturation is 90 percent, blood pressure is 90/50 mmHg,
heart rate 120/min and temperature is 38.5°C. On examination, she has bronchial breathing
at her right lung base. What is the most likely diagnosis?
A Bronchitis
B Pneumonia
C Pulmonary embolism
D Pulmonary haemorrhage
ANSWER:B
549 How do you calculate the Mean Arterial Pressure (or MAP)?
A Diastolic BP + systolic BP
B (Diastolic BP / 3) + systolic BP
C Diastolic BP + 1/3 (systolic BP - diastolic BP)
D Diastolic BP + 1/3 (systolic BP + diastolic BP)
ANSWER:C
550
Sara, a 25 year old woman, presents to the Emergency Department with 5 days of cough, 2
days of fever and increased lethargy. Her oxygen saturation on air is 90%, blood pressure is
90/50 mmHg, heart rate is 120 bpm and temperature is 38.5ºC. On examination, she has
bronchial breathing at her right lung base. You have assessed Ms Jones and have decided to
give her IV fluids. How much fluid will you give before reassessing her?
A 1 litre normal saline rapidly
B 1 litre 5% glucose rapidly
C 1 litre normal saline over 2 hours
D 1 litre 5% glucose over 1 hour
ANSWER:C
551
Anna is a 50 year old woman who is brought to the Emergency Department after collapsing
at home while standing at the sink. She says that she remembers feeling hot and nauseated
before developing tunnel vision and then passing out. She is usually well and takes a single
anti-hypertensive agent. On physical examination, she is now well. There are no significant
cardiovascular findings and no focal neurological deficits. What is the most likely cause for
her syncopal episode?

A Vasovagal syncope
B Transient ischaemic attack
C Orthostatic hypotension
D Ventricular tachycardia
ANSWER:C
552
Susan is a 35 year old woman who is brought into the Emergency Department by ambulance
after being found lying in the gutter. She is able to speak but is unable to give coherent
answers to questions. Her initial vital signs are as follows: heart rate 120 bpm, blood pressure
80/40 mmHg, temperature 38ºC and respiratory rate 28bpm. What would be your first step
on reaching her bedside?
A Perform a neurological examination
B Remove her clothing, looking for signs of injury
C Ask for an urgent 12 lead electrocardiogram (ECG)
D nsert an intravenous cannula
ANSWER:D
553 Which device would deliver oxygen at a concentration close to 100 percent?
A Self-inflating bag- valve- mask device with reservoir and oxygen flow at 15L/min
B Simple face mask with oxygen flow at 15L/min
C Venturi device set at 50% and oxygen flow at 30L/min instead of the usual 15L/min
D
Loose fitting mask that has an oxygen reservoir bag with a one-way valve and oxygen flow
at 15L/min
ANSWER:A
554
A 45 year old morbidly obese male presents with a two day history of pain in the left calf
and swelling of the ankle. Which of the following is the correct management?
A Elastic stockings and aspirin with a review in 5 days
B Bed rest with elevation of the left leg
C Venous duplex ultrasound scan of the left leg
D Subcutaneous heparin for 6 days
ANSWER:C
555 Which of the following is most useful for identifying the cause of syncope?
A Electroencephalogram (EEG)
B Electrocardiogram (ECG)

C Cerebral Computerised Tomography (CT)
D History from patient
ANSWER:D
556
Management of atrial fibrillation of onset within 4 hours of presentation includes all of the
following EXCEPT:
A Urgent cardioversion to prevent systemic embolisation
B Rate control with IV anti-arrhythmic drugs
C Echocardiography
D Commencement of continuous ECG monitoring
ANSWER:C
557
A 20 year old university student presents to you with a one week history of fatigue, fevers,
night sweats and poor appetite. He was well prior to this episode. Examination reveals
multiple tender lymph nodes in the cervical and axillary areas and splenomegaly palapable
3cm below the costal margin. What would most appropriate initial management include?
A Perform a monospot and viral serology
B Referral for abdominal (staging) CT scan
C Referral for excision biopsy of a lymph node
D Referral for a fine needle lymph node aspiration biopsy
ANSWER:A
558 Red cell fragments (schistocytes) are commonly seen in which of the following conditions?
A Acute pulmonary embolism
B Autoimmune haemolytic anaemia
C Disseminated intravascular coagulation
D Falciparum malaria
ANSWER:B
559
Multiple myeloma is characterised by plasma cell infiltration of which of the following
organs?
A Lymph nodes
B Spleen
C Kidneys

D Bone marrow
ANSWER:D
560
Which of the following statements regarding the use of diagnostic ultrasound for medical
imaging is correct?
A A fizzy drink is the best preparation for an ultrasound of the upper abdomen
B The higher the frequency of the ultrasound used the better the spatial resolution obtained
C
The use of ultrasound in women of child bearing age is not recommended because of
radiation concerns
D
The use of ultrasound for guidance of percutaneous procedures is decreasing as CT scans
and MRI units become more widely available
ANSWER:D
561 Which of the following is most likely to cause damage to cardiac muscle?
A Cyclophosphamide
B Methotrexate
C 5-fluorouracil
D Doxorubicin
ANSWER:D
562
A 24 year old man admitted to the Emergency Department following a motor vehicle
accident. He has a contused chest wall and is in respiratory distress. His left chest is resonant
on percussion and no breath sounds can be heard on auscultation. His trachea is deviated to
the right. What is the most likely diagnosis?
A Acute Respiratory Distress Syndrome (ARDS)
B Pulmonary embolism
C Spontaneous pneumothorax
D Tension pneumothorax
ANSWER:D
563
Alex is a 25 year old woman who presents to the Emergency Department with sharp central
chest pain. You make a provisional diagnosis of pericarditis. Which of the following is
TRUE of this condition?
A It has a high fatality rate in young people
B It is sometimes associated with pulsus paradoxus
C It commonly presents with a pleuritic pain, worse on sitting up

D It commonly presents with ECG changes such as PR elevation
ANSWER:B
564 Regarding unstable angina pectoris, which of the following statements is true?
A A normal ECG and normal serial troponins makes the diagnosis very unlikely
B The absence of cardiac risk factors excludes the disease
C Pain commonly radiates to the back
D It is treated with thrombolysis or percutaneous coronary intervention
ANSWER:D
565
Charlie Springer is a 25 year old man who has had a motor bike accident. On arrival at the
Emergency Department, his vital signs are heart rate is 130/minute, blood pressure of 90/60
mmHg, respiratory rate of 32/minute, temperature is 36°C. His peripheral oxygen saturation
is 97 percent on room air. What would be your initial order for IV fluids?
A Two packs of O negative blood over half an hour
B A litre of normal saline over 2 hours
C A litre of 4 percent dextrose and 0.18 percent saline over an hour
D A litre of normal saline over half an hour
ANSWER:D
566
Cholinesterase inhibitors have NOT been associated with which of the following potential
adverse effects?
A Tachycardia
B Diarrhoea
C Muscle cramps
D Pancreatitis
ANSWER:D
567 Which of the following statements about septic arthritis is correct?
A The infection is most commonly due to Escherichia coli
B The clinical features can be similar to gout
C It rarely presents as a monoarthritis
D It usually presents as a pyrexia of unknown origin
ANSWER:C

568
A 24 year old man presents with a six month history of low back and buttock pain. His pain
is worse in the morning and improves during the day. He has restriction of movement of his
lumbar spine. His symptoms are helped by non-steroidal antiinflammatory drugs. Which of
the following clinical features is he most likely to develop?
A Arthritis involving the small joints of the hands and feet
B Pulmonary fibrosis involving the lower lobes
C Aortic valve incompetence
D Acute iritis
ANSWER:D
569
A 75 year old man presents a year after sustaining a stroke. He complains of worsening
urinary frequency, urgency and nocturnal enuresis. Which of the following statements is
true?
A The most likely cause is benign prostatic hypertrophy
B
The appropriate imaging investigation is computed tomography (CT) scan of the urinary
tract
C Cystoscopy is mandatory
D The most likely cause is neurogenic overactive bladder
ANSWER:D
570
Profound hypothyroidism in the elderly is likely to cause which of the following electrolyte
disturbances?
A Hyponatraemia
B Hypernatraemia
C Hypokalaemia
D Hyperkalaemia
ANSWER:A
571
A 50 year old woman presents with painless rectal bleeding on defaecation. There is no
family history of bowel cancer, no weight loss and no change in bowel habits. What is the
recommended investigation?
A Haemoccult testing
B Colonoscopy
C Flexible sigmoidoscopy

D Proctoscopy and haemoccult test
ANSWER:B
572
What is the significance of the finding of localised tenderness and guarding on abdominal
examination?
A The patient's pain is severe
B There is an inflammatory process involving the parietal peritoneum
C There is intraperitoneal free fluid
D There is haemorrhage within a hollow viscus
ANSWER:B
573 A Regarding analgesic doses of ketamine, which of the following statements is correct?
A It should be given carefully to asthmatics as it may worsen bronchospasm
B It is likely to cause nausea and vomiting
C It antagonises the effects of opiates
D It is unlikely to cause respiratory depression
ANSWER:B
574
A 56 year old man is brought into the Emergency Department following a car accident.
When you assess his respiratory system you find that he has reduced chest expansion on the
right with a midline trachea. The percussion note is dull on the right posteriorly and laterally
but resonant on the left. He is very tender in the right axilla. Breath sounds are absent at the
right base and generally vesicular with occasional crackles elsewhere. What is the most likely
diagnosis based on these findings?
A Haemothorax on the right
B Simple pneumothorax on the right
C Right lower lobe aspiration pneumonitis
D Acute severe asthma
ANSWER:A
575
In an emergency setting for a patient with an unknown blood group, which is the most
appropriate type of fresh frozen plasma (FFP) to administer?
A O negative
B O positive
C AB negative
D AB positive

ANSWER:C
576 Aminoglycosides (e.g. gentamycin) cause deafness by damaging which of the following?
A Basilar membrane
B Vestibular nerve
C Cochlear hair cells
D Cochlear duct
ANSWER:B
577 With regard to the use of antibiotics in cholera, which of the following statements is correct?
A They prevent the spread of infection in the community
B They reduce the risk of bacteraemia
C They reduce the production of the toxin
D They can shorten the duration of diarrhoea
ANSWER:A
578 Which oral condition has the LEAST association with Type 1 and Type 2 diabetes?
A Candidiasis
B Periodontitis
C Oral cancer
D Lichenoid reactions
ANSWER:C
579
Plasmodium vivax and Plasmodium ovale can relapse months or years after an initial
infection. Which one of the following parasite stages is responsible for reactivation of
malaria infection?
A Hypnozoite
B Merozoite
C Trophozoite
D Sporozoite
ANSWER:A
580
Which of the following treatments is most likely to be associated with a potential life
threatening attack of asthma?
A Inhaled corticosteroids
B Inhaled corticosteroid plus a long acting beta agonist

C Multiple courses of oral corticosteroids for acute asthma
D Steroid nasal sprays for concurrent allergic rhinitis
ANSWER:C
581
A 75 year old woman brought into the Emergency Department by her daughter. Her
daughter tells you she has become more confused over the last 2 days and she is normally
independent and otherwise healthy woman. On examination she is afebrile, her heart rate is
100 beats per minute and her blood pressure is 160/90 mmHg. Pulse oximetry is 97 percent
on room air. There are no focal findings on neurological examination and her chest is clear.
What is the most likely cause of her confusion?
A Transient ischaemic attack
B Pneumonia
C Dementia
D Vitamin B12 deficiency
ANSWER:A
582
Which of the following statements describes the mechanism of action of statin therapy for
lipid-modification?
A They predominantly reduce triglyceride levels
B Their major action is on intestinal cholesterol absorption
C They inhibit HMGCoA reductase
D They predominantly reduce HDL
ANSWER:C
583
A 26 year old woman, presents with a 7 month history of abdominal pain. Investigations
reveal her to have Crohn's disease affecting a short segment of the terminal ileum. There is
no evidence of disease elsewhere, and she remains systemically well. Of the following, which
would be the most appropriate initial therapy?
A Oral olsalazine
B Oral budesonide
C Oral azathioprine
D Oral cyclosporine
ANSWER:B

584
A 45 year old man presents having had a large haematemesis and melaena. He had four
forceful vomits containing old food and bile prior to the episode of haematemesis. What is
the most likely cause of the haematemesis?
A Duoenal ulcer
B Gastric ulcer
C Mallory-Weiss tear
D Oesophageal varicies
ANSWER:C
585
A thin 45 year old alcoholic man presents 6 weeks after an admission for alcohol induced
acute pancreatitis with a history of mild to moderate epigastric discomfort and a poor oral
intake due to early satiety. Examination reveals he is afebrile and has an 8cm firm, round,
slightly tender mass in the epigastrium. What is the most likely diagnosis?
A Distended stomach due to gastric outlet obstruction
B A pancreatic abscess
C A pancreatic pseudocyst
D A mucinous cystic tumour of the pancreas
ANSWER:C
586
A 74 year old man with advanced chronic obstructive pulmonary disease (COPD) presents to
his family doctor with increasing exertional dyspnoea. This problem has slowly evolved over
the previous 2-3 years. He is now having difficulty walking around the shopping centre and
can no longer do the gardening. Investigations suggest his symptoms are due to progression
of the COPD. Which of the following management strategies is NOT appropriate to consider
in this patient?
A Inhaled corticosteroid and long acting bronchodilator therapy
B Domiciliary oxygen
C Lung transplantation
D Pulmonary rehabilitation program
ANSWER:C
587 In which of the following is topical anaesthesia with lignocaine NOT effective?
A Cornea
B Posterior pharynx
C Auditory canal

D Urethra
ANSWER:C
588 Regarding adult basic life support in cardiac arrest, which of the following is correct?
A
The ratio of compressions to ventilations may be altered depending on the number of
rescuers
B The recommended compression:ventilation ratio is 30:2
C Five initial rescue breaths should be given
D A radial pulse should be checked before commencing chest compressions
ANSWER:B
589
A 25 year old man had a late night out drinking alcohol heavily. At 2 o’clock in the
morning he vomited profusely. This was followed by an acute, severe pain in his chest
radiating to his back and neck. What is the most likely diagnosis?
A Acute myocardial infarction
B Pulmonary embolus
C Peptic ulcer disease
D Oesophageal perforation
ANSWER:D
590 Which of the following is the most sensitive investigation for pancreatitis?
A Lipase - with levels > 3X normal
B Amylase - if levels 3X normal
C Abdominal X-ray
D Computed Tomography (CT) scan
ANSWER:A
591
By definition, severe acute pancreatitis includes all EXCEPT which of the following
complications?
A Acute renal failure
B Acute respiratory distress syndrome (ARDS)
C Acute bowel perforation
D Pancreatic pseudocysts
ANSWER:C
592
Which of the following most accurately describes the ECG abnormality seen with a delay in
AV node conduction?

A A QRS duration of >0.16 seconds
B A PR interval of >0.12 seconds
C A QT interval of 420 milliseconds
D A PR interval of >0.20 seconds
ANSWER:D
593
A72 year old woman, presents with 2 days of melaena on the background of a 2 month
history of burning epigastric pain relieved by food. An upper endoscopy demonstrates a
gastric ulcer in the prepyloric region. Urease test is positive, indicating the presence of H.
pylori. She is treated with standard triple therapy for H. pylori infection, followed by
omeprazole for a further 2 months, with good relief of her symptoms. She presents for
follow up. Which of the following would represent the most appropriate management?
A Check H. pylori eradication with faecal culture
B Check H. pylori eradication with anti H. pylori antibody assay
C Check H. pylori eradication with urea breath test
D Repeat upper endoscopy
ANSWER:D
594 Haematemesis is NOT a feature of which of the following?
A Jejunal angiodysplasia
B Dieulafoy lesion
C Duodenal ulcer
D Gastrointestinal stromal tumour of the stomach
ANSWER:A
595
A 20 year old woman with a history of smoking and mild asthma presents with sudden onset
of sharp chest pain which on questioning is worse on inspiration. On examination her resting
respiratory rate is 30 breaths/min and she is clearly distressed. What do you consider to be
the most clinically valuable next step in examination or investigation?
A Determine if the trachea is midline
B Listen for decreased breath sounds
C Order an urgent chest X-ray
D Organise pulse oximetry
ANSWER:A

596
A 40 year old man with a history of alcohol abuse presents with a one week history of
shortness of breath, fever and lateral chest pain. His temperature is 39ºC and he is
hypotensive. A chest X-ray shows a large D shaped collection posteriorly at the base of his
right lung. Which of the following choices is the most appropiate next step?
A Ultrasound scan of the collection
B Computed tomography (CT) of the chest
C Diagnostic pleural tap and gram stain
D Intercostal drain insertion and gram stain
ANSWER:D
597
An HIV-positive patient recently returned from the tropics presents with severe diarrhoea (up
to 30 times per day). What is the most likely cause?
A Escherichia coli
B Staphylococcus aureus
C Cryptosporidium
D Entamoeba histolytica
ANSWER:C
598
A 34-year-old coronary care nurse accidentally stabs himself with a used needle from a
patient infected with the hepatitis C virus. He attends the occupational health department and
asks for advice.Which would be the most appropriate next step suggested by the
occupational health doctor?
A Monthly hepatitis C antibody testing
B Monthly hepatitis C PCR testing
C 6 months' ribavirin therapy
D 6 months' lamivudine therapy
ANSWER:B
599
A 25-year-old man presents with 4 days of fever, retro-orbital pain and severe myalgia
following travel to the Indian subcontinent. He has red eyes and a faint, blanching,
maculopapular rash. A peripheral smear for malarial parasites is negative and his white cell
count and chest X-ray are normal. What is the most likely diagnosis?
A Malaria
B Typhoid fever
C Bubonic plague

D Dengue fever
ANSWER:D
600
A 19-year-old female university student presents with fever and headache. On examination
she is conscious but has neck stiffness. The cerebrospinal fluid Gram stain shows intracellular
Gram-negative diplococci. The most probable diagnosis is?
A Meningococcal meningitis
B Haemophilus influenzae meningitis
C Streptococcus pneumonia meningitis
D Listeria monocytogenes
ANSWER:A
601
A 22-year-old farm worker is admitted to hospital with a 2-day history of headache, fever,
severe myalgia and a petechial rash. He is known to suffer from mild asthma, which is well
controlled by inhaled steroids. He is jaundiced, has a tachycardia and has not passed urine
for over 14 hours. His urea level is raised and liver function tests indicate hepatocellular
damage. What is the most likely diagnosis?
A Brucellosis
B Weil's disease
C Lyme disease
D Rat-bite fever
ANSWER:B
602
A 31-year-old man has just returned from a holiday to recuperate after the death of his
partner. He has been suffering night sweats, a chronic cough and shortness of breath on
exercise. Over the past 6 months he has lost a few kilograms in weight and suffered from
intermittent diarrhoea. On auscultation the lung fields appear relatively clear. Laboratory
testing reveals a relative lymphopenia, with the CD4 lymphocyte subfraction reduced at only
85/ mm3 (normal 200-800). There is desaturation on blood gas monitoring associated with
exercise. Other blood tests reveal a raised lactate dehydrogenase level. Chest X-ray reveals
diffuse pulmonary infiltrates. What diagnosis fits best with this clinical picture?
A Tuberculosis
B Pneumocystis jiroveci pneumonia
C Histoplasmosis

D Cryptococcosis
ANSWER:B
603
A 45-year-old man of Sudanese origin is admitted with a history of low-grade fever for over
7 days. He migrated to the UK 1 year ago and has a past history of well-controlled asthma.
His temperature chart shows that on some days there is a doubled rise in his temperature
during 24 hours. Examination shows a massively enlarged spleen and mild hepatomegaly.
His full blood count shows a mild microcytic and hypochromic anaemia along with
granulocytopenia and thrombocytopenia. Which one of the following investigations will
establish a diagnosis?
A Bone marrow aspirate
B Widal test
C Xenodiagnosis
D Examination of a wet blood film taken at night
ANSWER:A
604
A 10-year-old boy is complaining of pain in his right leg. He is pyrexial and a diagnosis of
osteomyelitis has been made. Which of the following is the most likely pathogen?
A Streptococcus viridans
B Staphylococcus aureus
C Cornybacterium diphtheriae
D Neisseria meningitides
ANSWER:B
605
A 33-year-old man has recently returned from a holiday in Pakistan. He is complaining of
episodes of abdominal spasms followed by loose stools containing blood and mucus.Which
one of the following pathogens is not likely to be causative of his disorder?
A Schistosoma mansoni
B Shigella dysenteriae
C Salmonella typhi
D Campylobacter jejuni
ANSWER:A

606
A 55-year-old man of no fixed address is admitted to the hospital because of self-neglect. A
chest X-ray has shown bilateral apical cavitation and hyperinflated lung fields consistent with
COPD. Sputum cultures have grown Mycobacterium avium complex (MAC).Which one of
the following statements is correct?
A Patient should be treated with a standard 6 months of anti-tuberculosis drugs
B Patient should be notified within 1 week of diagnosis
C Patient should be barrier nursed for 2 weeks
D This organism is most likely to affect patients with pre-existing lung disease
ANSWER:D
607
A 19-year-old athlete was admitted to hospital after having been found wandering around in
a confused state. His girlfriend is concerned that his behaviour has been very odd during the
day. No illicit drugs are found on him. Clinically, he is disoriented and vague about his
history but does admit to headaches. Histemperature is high at 38.3°C. On review by the
medical team, his level of consciousness deteriorated, his Glasgow Coma Score (GCS) being
11 out of 15. A CT scan of his head was normal. His CSF was clear, but the opening pressure
was raised at 23 cmH2O. His CSF protein concentration was 0.9 g/l and the glucose level was
normal. The CSF showed 300 white cells, mainly lymphocytes. No organisms were seen on
Gram stain. The results of CSF PCR is awaited. Which one of the following treatments would
you start immediately?
A Intravenous benzylpenicillin
B Intravenous acyclovir
C Intravenous anti-fungal therapy
D Intravenous steroid therapy
ANSWER:B
608
A 60-year-old woman is convalescing in hospital following total right knee replacement
surgery undertaken three weeks ago. She develops headache, chills and a fever of 39.2°C.
On examination the right knee is red, hot and very tender. Synovial fluid aspirate reports the
growth of Gram-positive cocci. Which one of the following is the MOST likely organism?
A Staphylococcus epidermidis
B Pseudomona aeruginosa
C Streptococcus pneumoniae

D Staphylococcus aureus
ANSWER:A
609
A 33-year-old woman has returned from a holiday in Africa. She has been diagnosed as
suffering from malaria due to Plasmodium falciparum.Which one of the following statements
is FALSE?
A Following successful treatment, fever recurs due to persistence of the parasite in the liver
B Cough and mild diarrhoea are a common presentation
C Splenectomy increases the risk of infection
D Jaundice is usually due to haemolysis and hepatitis
ANSWER:A
610
A 30-year-old man presents with an acute onset of pain and blurred vision of his right eye.
On examination there is conjunctival injection and dendritic ulceration is seen on his
cornea.What is the diagnosis?
A Herpes simplex virus keratitis
B Foreign body
C Candida keratitis
D Trachoma
ANSWER:A
611
A 36-year-old woman presents complaining of a yellowish-green vaginal discharge that
started 1 week ago. On examination her vagina is swollen and erythematous.What is the most
sensitive diagnostic test?
A Colposcopy
B Blood cultures
C Serology
D Vaginal fluid microspcopy and culture
ANSWER:D
612
A 36-year-old woman presents complaining of a yellowish-green vaginal discharge that
started 1 week ago. On examination her vagina is swollen and erythematous. Given the likely
diagnosis, what is the most appropriate treatment?
A Ampicillin
B Nystatin
C Metronidazole

D Fluconazole
ANSWER:C
613
A 25-year-old homosexual man complains of a 9-day history of mucopurulent anal
discharge, anal bleeding and pain while opening his bowels. What is the next step in the
diagnosis?
A Colonoscopy
B Erythrocyte sedimentation rate
C Stained specimen microscopy
D Specimen culture
ANSWER:C
614
A 25-year-old homosexual man complains of a 9-day history of mucopurulent anal
discharge, anal bleeding and pain while opening his bowels.What is the most likely diagnosis?
A Candidiasis
B Gonorrhoea
C Crohn's disease
D Salmonella infection
ANSWER:B
615
A 29-year-old homosexual man has been complaining of anal warts for the last 6 months.
They have gradually increased in size and he has also noticed some fresh blood when
opening his bowels. On examination there are grey lesions, approximately 5 mm in size,
around his anus. What is the most likely cause for these lesions?
A Human papillomavirus
B Neisseria gonorrhoea
C Candida albicans
D Human immunodeficiency virus
ANSWER:B

616
You are asked to review a 54-year-old asylum seeker from Eastern Europe. He has suffered
a cough and weight loss of some 14 kg over the past few months. He admits to occasional
night sweats. He is a heavy smoker of some 40 cigarettes per day. Blood testing reveals that
he is HIV-positive. Chest X-ray reveals multiple calcified lymph nodes, fibrosis and hilar
retraction. Initial sputum culture is unremarkable. What is the most likely diagnosis in this
case?
A Bronchial carcinoma
B Sarcoidosis
C Silicosis
D Pulmonary tuberculosis
ANSWER:D
617
A 45-year-old business traveller noticed some moderate diarrhoea 3 days after he arrived
inKorea. The diarrhoea lasted for 4 days. What is the most likely cause for his diarrhoea?
A Legionella
B Staphylococcus
C Enterotoxic Escherichia coli
D Giardia lamblia
ANSWER:C
618
A 32-year-old man from Uganda is referred to hospital with a high eosinophil count by his
GP following routine blood tests. He is entirely asymptomatic and has no past medical
history of note. Which of the following organisms is LEAST likely to be responsible?
A Strongyloides stercoralis
B Entamoeba histolytica
C Schistosoma mansoni
D Schistosoma haematobium
ANSWER:B
619
A 29-year-old Catholic priest returns from a trip to Brazil with fevers and deranged LFTs. He
has an ALT of 2500 U/l and bilirubin of 75 mmol/l. He attended a travel clinic and was
vaccinated prior to travel. He also took mefloquine malaria prophylaxis. What is the most
likely diagnosis?
A Malaria
B Hepatitis A

C Hepatitis B
D Hepatitis E
ANSWER:D
620
A 30-year-old man presents with an acute onset of pain and blurred vision of his right eye.
On examination there is conjunctival injection and dendritic ulceration is seen on his cornea.
Given the likely diagnosis, what is the most important treatment?
A Topical steroids
B Topical aciclovir
C Ampicillin ointment
D Topical nystatin
ANSWER:B
621
A young homosexual man contacts his GP complaining of a short episode of lethargy, fever
and swollen neck glands. HIV infection is diagnosed. Which is the most likely cell receptor
through which the virus infects the body?
A CD13
B CD4
C CD8
D CD2
ANSWER:B
622
A 49-year-old man presents with an episode of acute self-limiting hepatitis. Hepatitis A is
diagnosed. What is the most likely mode of transmission?
A Sexually
B Blood transfusion
C Needle-stick injury
D Contaminated food
ANSWER:D
623
A 46-year-old patient has been chronically infected with the hepatitis B virus for the last 8
years. He has an increased risk of developing which disease?
A Coronary artery disease
B COPD
C Pancreatic cancer
D Hepatocellular cancer

ANSWER:D
624
A 33-year-old homosexual patient who is HIV positive but in the stable phase of the disease
is best monitored with which biomarker?
A C-reactive protein
B CD4 lymphocyte count
C Erythrocyte sedimentation rate
D Polymerase chain reaction
ANSWER:B
625
An 18-year-old woman has been diagnosed with human papillomavirus infection. What is
the most significant long-term risk following this infection?
A Coronary artery disease
B Endometriosis
C Infertility
D Cervical cancer
ANSWER:D
626
A 23-year-old man presents to his GP complaining of fevers, headache, malaise and muscle
pain. Shortly after his return to the UKlast week from a walking trip in the United States, he
says he used a cigarette to burn off a tick on his leg. On examination he has a rash on the
palms and soles of his feet. What diagnosis fits best with this clinical picture?
A Infectious mononucleosis
B Rocky Mountain spotted fever
C Reiter's syndrome
D Influenza
ANSWER:B
627 What is the cause of myocarditis caused by diphtheria?
A Hypoxia
B Superinfection with streptococci
C Toxins
D Massive bacteraemia
ANSWER:C

628
A 23-year-old woman presents to the Sexual Health Clinic. She had unprotected sex after an
office party 4 days ago. She is currently taking antibiotics for a respiratory tract infection.
There is intense difficulty passing urine, accompanied by burning, itching and pain over her
labia. On examination there is a crop of vesicles with ulceration. What diagnosis fits best with
this clinical picture?
A Syphilis
B Herpes zoster infection
C Herpes simplex infection (HSV-1)
D Herpes simplex infection (HSV-2)
ANSWER:D
629
A 38-year-old man presents some 3 weeks after a stag party weekend in Prague. He has
developed a painless ulcer on his penis. You suspect that he may have syphilis. Which of the
following percentages is the best estimate of how many untreated syphilis patients go on to
develop late-stage CNS or cardiovascular complications?
A 80%
B 90%
C 10%
D 30%
ANSWER:D
630
An HIV-positive patient attends clinic. He is on his first antiretroviral regimen, which
includes stavudine, DDI and nevirapine. He is well but complains of wasting of his temporal
areas and arms with an increase in the size of his abdomen. You do some screening tests, the
results of which are shown below: U&E normal; LFT normal; glucose 7.9 mmol/l; amylase
80 U/l; cholesterol 8.8 mmol/l; TGs 12.7 g/l; FBC normal; CD4 count 870 cells/mm3; HIV
viral load < 50 copies/ml. Which advice is the most appropriate?
A Stop the antiretroviral therapy and start atorvastatin 40 mg
B Switch the stavudine to abacavir and start pravastatin 10 mg
C Switch the stavudine to abacavir and start atorvastatin 40 mg
D Switch the nevirapine to nelfinavir and start pravastatin 10 mg
ANSWER:B

631
A 45-year-old Christian missionary returned from a tropical assignment in Central America
10 months ago. Since then he has undertaken no foreign travel. He presents with fever,
malaise and rigors. On examination he has mild jaundice, liver tenderness and a palpable
spleen. Blood testing reveals that he is anaemic. What infective agent fits best with this
clinical picture?
A Plasmodium falciparum
B Plasmodium malariae
C Tropical sprue
D Plasmodium vivax
ANSWER:D
632
A 23-year-old man who lives with his male partner consults you for an opinion. He has
suffered anal discharge and pruritis for the past 3 days. There are also some symptoms of
dysuria. A urethral smear reveals intracellular diplococci. What is the most likely infective
agent to fit with this clinical picture?
A Neisseria gonorrhoeae
B Chlamydia trachomatis
C Treponema pallidum
D Herpes simplex-type 1
ANSWER:A
633
A 36-year-old woman presents complaining of a yellowish-green vaginal discharge that
started 1 week ago. On examination her vagina is swollen and erythematous. What is the
most likely diagnosis?
A Candidiasis
B Trichomoniasis
C AIDS
D Papillomavirus infection
ANSWER:B

634
A 19-year-old student presented to his university GP complaining of a severe sore throat,
headache and malaise.On examination there was a severe exudative pharyngitis with grossly
enlarged inflamed tonsils. There was some evidence of cervical lymphadenopathy. He was
diagnosed with a streptococcal infection and received a course of ampicillin. Unfortunately
he re-presents to the GP with a maculopapular rash, still feeling unwell. Blood testing reveals
a relative lymphocytosis with atypical lymphocytes. What is the most likely cause of this
clinical picture?
A Stevens-Johnson syndrome
B Epstein-Barr virus (EBV)
C Toxoplasmosis
D Cytomegalovirus
ANSWER:B
635
A 22-year-old man has returned from a period travelling, during which he visited
central/sub-Saharan Africa. He presents to the GP complaining of urinary frequency,
perineal itching and inflammation and also of painless haematuria. What diagnosis fits best
with this clinical picture?
A Infection with Schistosoma mansoni
B Infection with Schistosoma japonicum
C Infection with Schistosoma haematobium
D Syphilis
ANSWER:C
636
A 32-year-old woman has just returned from a holiday in the Middle-East. She had to spend
much of the flight in the toilet and has been brought by ambulance from the airport. On
admission she is severely dehydrated and gives a history of passing voluminous watery stools
that look like rice water, mixed with mucus and blood. Blood testing reveals a raised
haemoglobin, markedly raised urea and raised creatinine. Blood glucose is measured at only
3.1 mmol/l (normal 3.0-6.0). What diagnosis fits best with this clinic picture?
A Cholera
B Typhoid fever
C Shigella
D Salmonella

ANSWER:A
637
A 19-year-old student visits you complaining of fevers and headaches over the past week or
two. She has just started university after a world tour during her gap-year. There is also
complaint of muscle ache, a sore throat, general malaise and of a general lack of appetite and
vague abdominal pain. She remembers a short period of diarrhoea a couple of weeks ago.
On examination there are a few faint maculopapular blanching lesions on the chest. What
diagnosis fits best with this clinical picture?
A Malaria
B Tuberculosis
C Typhoid fever
D Amoebic liver abscess
ANSWER:C
638
A 32-year-old farmer's wife presents with fever and malaise, feeling generally 'washed-out'
and off her food. She has recently been helping out with lambing on the farm. On
examination there is generalised lymph node swelling and a palpable liver edge. Her white
blood cell count is just below the normal range. What diagnosis fits best with this clinical
picture?
A Tuberculosis
B Subacute bacterial endocarditis
C Brucellosis
D Amoebic liver abscess
ANSWER:C
639
An epidemic of diarrhoea and vomiting has broken out on the elderly care wards. Your
catering suppliers assure you that their food is unlikely to be responsible as they follow the
strictest hygiene procedures. A total of 15 patients on the ward have become unwell with a
sudden onset of diarrhoea and vomiting. Patients infected earlier have recovered with
rehydration therapy after about 48 h. Examination of faeces by electron microscopy has
revealed circular virus particles with radiating spokes. Which virus is most likely to be
responsible for this outbreak?
A Enteric adenovirus
B Small, round-structured virus
C Rotavirus

D Astrovirus
ANSWER:C
640
A 19-year-old student is admitted directly to emergency after being taken ill on a return
flight from Central America. It is understood that he was on the last leg of a round-the-world
ticket. His vaccination history is unavailable. He had suffered a flu-like illness around 10
days ago, from which he had recovered. On examination in emergency he is pyrexial at
39.0°C, has extensive bruising, with bleeding around the gum line, and deep jaundice. What
diagnosis fits best with this clinical picture?
A Malaria
B Influenza
C Weil's disease
D Yellow fever
ANSWER:D
641
An 18-year-old student presents to his GP with a 1-day history of rash, which has followed a
3-day history of cold-like symptoms and conjunctivitis. The rash began as a maculopapular
eruption in the postauricular region, but has rapidly spread to his face and upper body. On
examination white papules are visible inside his mouth. What diagnosis fits best with this
clinical picture?
A Scarlet fever
B German measles
C Measles
D Enterovirus infection
ANSWER:C
642
A 17-year-old young woman is undertaking a summer placement at a nursery school before
applying to study medicine. She has received a full programme of childhood vaccinations.
Her main complaints are difficulty swallowing, sore throat, malaise and fever. On
examination she has 5-10 grey ulcers on her buccal mucosa. There is also a vesicular rash
affecting her hands and feet. What is the most likely cause of this clinical picture?
A Erythema multiforme
B Herpes simplex infection
C Coxsackievirus infection

D Pemphigus
ANSWER:C
643
An 18-year-old woman complains of malaise, tiredness, headache and abdominal discomfort
for the last 3-4 days. She was started on ampicillin 2 days ago and has developed a rash. She
has lymphadenopathy and exudative tonsillitis. Her white cell count shows abnormal
lymphocytosis. What is the most likely diagnosis?
A German measles
B Chickenpox
C Infectious mononucleosis
D Herpes simplex infection
ANSWER:C
644 Infection with which virus is the most frequent cause of blindness in patients with AIDS?
A Herpes simplex virus
B Varicella zoster virus
C Epstein-Barr virus
D Cytomegalovirus
ANSWER:D
645
A 70-year-old man known to have NIDDM was admitted with pain and swelling in the left
ear and face. On examination the external ear is red, tender and swollen. There is a small
amount of purulent discharge from the external auditory canal with crust covering the skin.
The left side of the face is swollen, with tenderness over the left temporal bone. The primary
microorganism most probably responsible for this infection is?
A Pseudomonas aeruginosa
B Staphylococcus aureus
C Streptococcus pneumoniae
D Listeria monocytogenes
ANSWER:A
646 Which one of the following statements is true with regard to Legionnaires' disease?
A Legionella pneumophila is a Gram-positive rod
B
The urinary antigen test for Legionella species has low sensitivity and is not particularly
specific

C
Hyponatraemia occurs significantly more often in Legionnaires' disease than in other
pneumonias
D
The beta-lactam group of drugs are now regarded as the drug of choice against Legionella
species
ANSWER:C
647
A 65-year-old man known to have COPD presented with progressive respiratory failure. He
was treated in ITU with mechanical ventilation and improved. After extubation he was
transferred to the ward. On the second day on the ward, his temperature spiked and he
developed a productive cough with a yellow-greenish sputum. Blood results showed
leucocytosis. A chest X-ray revealed a right-sided middle and lower lobe pneumonia. What is
the most probable cause of his pneumonia?
A Pneumococcal pneumonia
B Aspiration pneumonia
C Pseudomonas pneumonia
D Staphylococcal pneumonia
ANSWER:C
648
A 48-year-old woman is admitted with a couple of days' history of fever with rigors and
breathlessness. On examination she looks extremely unwell and is confused, cyanosed, has a
respiratory rate of 36/min and a systolic blood pressure of 86 mmHg. There is dullnesson
percussion and bronchial breathing at her right base. The chest radiograph reveals
consolidation. Which of the following would be the most appropriate antibiotic regimen to
use?
A Oral amoxicillin
B Oral amoxicillin and oral clarithromycin
C Intravenous cefotaxime and intravenous clarithromycin
D Intravenous ceftazidime and intravenous vancomycin
ANSWER:C
649
A 46-year-old meat-factory worker is found to have Q fever pneumonia. Which of the
following statements is correct?
A He requires high-dose penicillin for his treatment
B The organism is usually inhaled from infected dust
C There is no long-term sequel of the disease

D The organism responsible is Coxiella pneumoniae
ANSWER:B
650
A 65-year-old man with severe rheumatoid arthritis (RA) is admitted with a right pleural
effusion. He has been complaining of dyspnoea on exertion for the last three months. He has
never smoked and has not worked for over 20 years when he was diagnosed to be suffering
from rheumatoid arthritis.Which of the following is true?
A Pleural effusions with rheumatoid arthritis occur in over 50% of patients
B
A glucose level in pleural fluid of < 1.6 mmol/l is characteristic of a rheumatoid pleural
effusion
C Pleural effusions associated with RA have low levels of cholesterol
D The most appropriate treatment is chemical pleurodesis
ANSWER:B
651
Which pulmonary function test may be altered to a similar degree in both restrictive lung
disease and obstructive lung disease?
A Residual volume
B Tidal volume
C Total lung capacity
D Forced expiratory volume in 1 second/forced vital capacity (FEV1/FVC) ratio
ANSWER:B
652
An 82-year-old man living alone in a bungalow came to the clinic complaining of feeling
generally unwell for about the last 3-4 months and of losing about 9.5 kg (21 lbs) in weight
during this period. On further enquiry he said he had been having night sweats for the last
month. He also has a past history of angina and arthritis and was on medication. On
examination he did not look well. He was pyrexic and without lymphadenopathy. Bibasal
crepitus on the lower zone was heard on chest auscultation. He had hepatosplenomegaly and
clubbing. Investigations showed WBC 12.3 x 106/l (neutrophils 52%, lymphocytes 39%),
Hb 9.1 g/dl, with all other routine investigations being normal. A chest X-ray showed 1-2 cm
diameter miliary shadows all over the lung field. The Mantoux test was negative. No bacteria
grew in a sputum culture.What is the probable cause of the illness and the X-ray finding?
A Sarcoidosis
B Mycoplasma pneumonia

C Staphylococcal pneumonia
D Miliary tuberculosis
ANSWER:D
653
A 65-year-old man came to clinic with a history of proximal muscle weakness. He has had a
cough for 8 weeks. There is associated pain in the small joints of the hands, and has small
haemorrhages in the nail folds. He is apyrexic and on examination there is no
lymphadenopathy or clubbing. Bibasal crackles can be heard and his chest X-ray reveals
diffuse reticular infiltrates. Lung function tests show a restrictive pattern. What is the
underlying cause of his interstitial lung disease?
A Cryptogenic fibrosing alveolitis
B SLE
C Ankylosing spondylitis
D Polymyositis
ANSWER:D
654
At the time of discharge of a 75-year-old non-smoker, with known COPD, it was decided
that, according to the criteria, he should be having long-term oxygen therapy in home. What
is not considered as a lone criterium for LTOT among the following options?
A Arterial blood gas showing pa(O2) 7.5 kPa
B Cor pulmonale
C FEV1 < 1.5 litres despite maximal treatment
D FVC < 2 litres despite maximal treatment
ANSWER:A
655
A 67-year-old woman has been diagnosed as suffering from bronchiectasis on a high-
resolution computed tomography (HRCT) scan of the lung. Which one of the following
statements is NOT true?
A She is at risk of developing a pneumothorax
B She is at risk of developing a brain abscess
C Massive haemoptysis is the commonest cause of death in her age group
D Recurrent chest infections are likely at her age
ANSWER:C

656
A 32-year-old Black woman presents with a 3-month history of a non-productive cough,
dyspnoea and pleuritic chest pain, especially with climbing stairs. She reports intermittent
fevers of up to 39°C and a 3.5-kg weight loss. She complains of wrist and ankle pain that has
interfered with her work. She smokes two packets of cigarettes per day. Her full blood count
is normal and serum ANA is negative. On examination there are red nodules over her lower
legs. What is the most likely diagnosis?
A Goodpasture's syndrome
B Adenocarcinoma of the lung
C Systemic lupus erythematosus
D Sarcoidosis
ANSWER:D
657
A patient with small-cell lung cancer has a serum sodium concentration of 121
mmol/l.Which of the following is the most likely cause?
A Sodium-restricted diet
B Sodium-reduced water drinking
C SIADH
D Liver metastases
ANSWER:C
658
As the medical SHO on call you are summoned to A&E to see a 25-year-old man whose
condition has suddenly deteriorated. He arrived about 45 minutes earlier with a 2-hour
history of central pleuritic-type chest pain and breathlessness. He collapsed while awaiting
radiography. He is now agitated and cyanosed. His pulse is 128/min and BP 76/40 mmHg.
Oxygen saturation is reading 76% with the patient breathing high-flow oxygen via a re-
breathing mask. On respiratory examination you hear reduced breath sounds in the right
lung field with deviation of the trachea towards the left. On percussion it is resonant
bilaterally. What immediate course of action should you take?
A Contact the ITU for urgent mechanical ventilation
B Insert a large-bore needle into the left, second intercostal space
C Insert a large-bore needle into the right, second intercostal space
D Check his arterial blood gases and start nebulisation with salbutamol
ANSWER:C

659
The 18-year-old son of an immigrant from Bangladesh who recently came to the UK has
been complaining of tiredness, weight loss and generally not feeling well for the last month.
He presents to his GP because of haemoptysis, especially in the morning. What is the most
likely diagnosis?
A Pneumothorax
B Pulmonary embolism
C Lung cancer
D Pulmonary tuberculosis
ANSWER:D
660
A 36-year-old primary schoolteacher from the East End of London presents with increasing
shortness of breath accompanied by sudden-onset, right-sided pleuritic chest pain. She gives
a history of influenza for a few days before this acute presentation and also says she suffered
a pulmonary embolus 2 years ago while taking the contraceptive pill and describes her pain
as identical to that occasion. On further questioning it transpires that her mother had suffered
from recurrent deep vein thrombosis. Arterial blood gases reveal a p(O2) of 7.2 kPa on a re-
breather mask, with a p(CO2) of 3.2 kPa. Her chest X-ray reveals a wedge-shaped area of
consolidation affecting her right middle and lower lobes. The white blood cell count is
normal. Which diagnosis fits best with this clinical picture?
A Recurrent pulmonary embolism
B Staphlyoccal pneumonia
C Pneumothorax
D Tuberculosis
ANSWER:A
661
A patient with tuberculosis was initially treated with streptomycin, which was later changed
to a combination of isoniazid, rifampicin, pyrazinamide and ethambutol. Abnormal liver
functions are noted on this, his follow-up, visit. Which drug is most likely to be responsible?
A Streptomycin
B Ethambutol
C Isoniazid
D Pyrazinamide

ANSWER:C
662 Which is the most common malignant neoplasm of the lung?
A Carcinoid tumour
B Squamous-cell carcinoma of the bronchus
C Metastatic carcinoma
D Adenocarcinoma of the bronchus
ANSWER:C
663
A patient with cystic fibrosis presents with a severe bronchopneumonia. What is the most
likely pathogen?
A Streptococcus pyogenes
B Streptococcus pneumoniae
C Pseudomonas aeruginosa
D Klebsiella pneumoniae
ANSWER:C
664
A 16-year-old girl presents to A&E with a severe asthma attack.What is the most important
therapy to relieve her bronchoconstriction?
A Propranolol
B Salbutamol
C Oxygen
D Glucocorticosteroids
ANSWER:B
665
A 36-year-old lorry driver who smokes heavily presents with a 2-day history of
coughassociated with fever. He also complains of right-sided chest pain on inspiration. On
examination he is slightly cyanosed. His temperature is 38°C, respiratory rate 38/min, BP
100/70 mmHg and pulse 130/min. He has basal crepitations and dullness to percussion at the
right lung base. What is the most important next step in confirming the diagnosis?
A ESR (Erythrocyte sedimentation rate)
B d-Dimer
C Chest X-ray
D Sputum sample
ANSWER:C

666
A 33-year-old man is found to have strongly positive aspergillus precipitins in his blood and
complains of a cough with intermittent bloody sputum. He has a business working as a
builder, particularly involved in renovating farm houses and barns. What is the most likely
diagnosis?
A Allergic bronchopulmonary aspergillosis (ABPA)
B Colonising aspergillosis
C Invasive aspergillosis
D Asperger's syndrome
ANSWER:B
667
A 30-year-old asthmatic patient has the following drug regimen: regular inhaled
corticosteroids, regular inhaled long-acting b 2-agonists (salmeterol) and inhaled short-acting
b 2-agonists when required. Although her compliance is good, her symptoms are still not
satisfactorily controlled. What is the next step in her therapy?
A Antibiotics
B Oral leukotriene-receptor antagonists
C Oral steroids
D Oral cromoglycate
ANSWER:B
668
A 30-year-old asthmatic patient has the following drug regimen: regular inhaled
corticosteroids, regular inhaled long-acting b 2-agonists (salmeterol), oral leukotriene-
receptor antagonists and inhaled short-acting b 2-agonists when required. Although her
compliance is good, her symptoms are still not satisfactorily controlled. What is the next step
in her therapy?
A Oral steroids
B Oral theophylline
C Switch to nebuliser
D Oral cromoglycate
ANSWER:B
669
A 24-year-old woman is brought to casualty with thorax injuries after a road traffic accident.
Her chest X-ray shows multiple rib fractures and a right-sided shadow suggestive of a
haemothorax. What is the next step in her management?
A Blood transfusion

B Intubation and ventilation
C CT thorax
D Intercostal drain insertion
ANSWER:D
670
A 30-year-old shepherd presents with a 1-week history of headaches, rhinitis and pharyngitis
associated with high fever and you elicit a systolic murmur on examination. Three days ago
he noticed a painful cough and blood-stained phlegm. What is the most likely diagnosis?
A Tuberculosis
B Borreliosis
C Brucellosis
D Q-fever
ANSWER:D
671
A 69-year-old former coal-miner is referred to you by the on-call team. There is a smoking
history and he has been managed by his GP for COPD. He has been admitted with dyspnoea
that is now so bad that he is unable to manage at home and cannot walk from the chair to the
bathroom. There is a cough productive of black sputum. Lung function tests show a mixed
restrictive and obstructive picture. A chest X-ray shows marked changes with massive
fibrotic masses predominantly in the upper lobes. There are also changes consistent with lung
destruction and emphysema. His rheumatoid factor is positive. Which diagnosis fits best with
this clinical picture?
A Progressive Massive Fibrosis (PMF)
B Chronic obstructive pulmonary disease
C Tuberculosis
D Asthma
ANSWER:A

672
A 29-year-old office secretary has been suffering from intermittent pain and tenderness
affecting her elbows, wrist and ankles for last 2 years. Symptomatic relief had being obtained
from NSAIDs. For last 3 months she has been increasingly unwell, and with night sweats,
fever and a weight loss of about 6.4 kg (14 lbs). She also developed a non-productive cough
and left-sided pleuritic chest pain. She smoked 30 cigarettes per day. On examination she
had temperature of 37.8°C. The syno vium was palpable over her wrist joints. On respiratory
examination her left lower zone was dull to percussion with decreased breath sounds. A chest
X-ray confirmed a left-sided pleural effusion in addition to some fibrotic patches on both the
upper zones. Aspiration showed a straw-coloured fluid with a protein concentration of 46 g/l
and a glucose concentration of 1.6 mmol/l. The fluid contained many lymphocytes but no
malignant cells. A culture was sterile on the fifth day. What is the probable cause of the
pleural effusion?
A Sarcoidosis
B Rheumatoid pleural effusion
C Tuberculosis
D Pleural effusion secondary to lung malignancy
ANSWER:C
673
A 33-year-old HIV-positive man presents for review. He is poorly compliant with
antiretroviral therapy and his recent CD4 count is only 90/ml blood. He complains of a
gradual-onset headache, fever, malaise, night sweats and a cough associated with
haemoptysis. He is emaciated. On examination there is widespread lymphadenopathy, there
are crackles and wheeze on auscultation of his chest, and tenderness over the liver edge.
Blood testing reveals a normochromic normocytic anaemia, he has a low white count, urea
and creatinine levels are raised and liver function tests are abnormal. Sputum samples reveal
acid- and alcohol-fast bacilli (AFB). Chest X-ray reveals calcified lymph nodes, cavitation
and areas of lung fibrosis and hilar retraction. Which diagnosis fits best with this clinical
picture?
A Primary pulmonary tuberculosis
B Miliary tuberculosis
C Bacterial pneumonia
D Pulmonary fibrosis
ANSWER:B

674
A 56-year-old man has a chest X-ray performed as he has become breathless on exertion and
has inspiratory crackles. The chest X-ray reveals upper lobe lung fibrosis. Which of the
following is the most likely explanation?
A Cryptogenic fibrosing alveolitis
B Langerhans' cell histiocytosis
C Asbestosis
D Connective tissue fibrosing alveolitis
ANSWER:B
675
A 30-year-old woman presents with shortness of breath. This began gradually, around 2.5
years ago, but now she is breathless on climbing a flight of stairs. There is no past history of
note. On examination the JVP is raised, carotid pulse volume is reduced and there is evidence
of right ventricular hypertrophy. There are right-sided murmurs on cardiac auscultation. Her
chest X-ray shows pulmonary artery enlargement, ECG shows right axis deviation and right
ventricular hypertrophy. Arterial blood gases reveal hypoxia and hypercapnia, a lung
perfusion scan is normal. Cardiac catheterisation reveals that right-sided pressures are
markedly raised. Which diagnosis best fits with this clinical picture?
A Chronic thromboembolic disease
B Right ventricular failure
C Primary pulmonary hypertension (PPH)
D Cryptogenic fibrosing alveolitis
ANSWER:C
676
A 58-year-old memorial stonemason presents to the chest clinic. Over the past few years he
has noted a gradual increase in shortness of breath, with cough and occasional wheeze. He is
a non-smoker and has no other past history of note. His chest X-ray is abnormal with small
rounded opacities and irregular upper zone fibrosis. There is hilar lymphadenopathy with
'eggshell' calcification. Pulmonary function testing reveals a restrictive picture and there is
mild hypoxia. Which diagnosis best fits with this clinical picture?
A Silicosis
B Asthma
C Idiopathic pulmonary fibrosis
D Tuberculosis

ANSWER:A
677
A 67-year-old patient with lung cancer complains of a cough and has difficulty breathing.
He has swelling of his face, neck, upper body and arms. What is the most likely diagnosis?
A Side-effects from radiotherapy
B Allergic reaction
C Superior vena cava syndrome
D Side-effect from chemotherapy
ANSWER:C
678
A 42-year-old cotton worker from Southern India is staying with her brother in London .
While here, they pay for a private medical consultation. She reports what sounds like a work-
related illness. During the first hour at work after the weekend she reports severe shortness of
breath, cough and chest tightness, this appears to gradually ease during the week, only to
return after the next weekend off. Which diagnosis best fits this clinical picture?
A Occupational asthma
B Chronic obstructive pulmonary disease
C Byssinosis
D Berylliosis
ANSWER:C
679
A 50-year-old woman patient presents with increasing dyspnoea. She is obese, smokes and
takes oestrogens for menopausal symptoms. On examination you find clinical, electrocardial
and radiological findings of aright-sided heart failure without signs of left ventricular failure.
What is the most likely cause for the cor pulmonale?
A Asthma
B Recurrent pneumonias
C Recurrent small pulmonary embolisms
D Bronchiectasis
ANSWER:C

680
A 38-year-old woman presents with recurrent chest infections. For some time she has noticed
that her nails are yellow and misshapen and that she often has oedematous legs after standing
all day in the shop where she works. Examination reveals evidence of lower lobe
consolidation and possible pleural effusion. This is confirmed on chest X-ray. Her nails are
very abnormal, thickened and yellow and she has bilateral lymphoedema affecting her legs.
Which diagnosis fits best with this clinical picture?
A Bronchiectasis
B Yellow-nail Syndrome
C Underlying bronchial carcinoma
D COPD
ANSWER:B
681
A patient with small-cell lung cancer has a serum sodium concentration of 121 mmol/l. The
patient is asymptomatic. What is the most appropriate therapy?
A Fluid restriction
B Glucocorticoids
C Start chemotherapy
D Start radiotherapy
ANSWER:A
682
A 26-year-old man presents with fever, headache and a non-productive cough. The chest X-
ray shows increased interstitialmarkings. The laboratory examination shows an elevated LDH
and anaemia with the presence of cold agglutinins. What is the most appropriate treatment?
A Erythromycin
B Piperacillin
C Clindamycin
D Ampicillin
ANSWER:A
683
A 37-year-old man who speaks little English comes to the TB clinic. His notes are missing
but he is able to tell you that he has been on treatment for almost 1 year. What is the most
likely reason he has been treated for this length of time?
A Pulmonary TB
B Lymph-node TB

C TB meningitis
D Bony TB
ANSWER:C
684
A morbidly obese 36 year-old man presents for review. His main reason for attendance is
that his wife is concerned about his loud snoring and the fact that he stops breathing during
the night for periods of up to 8-10 seconds, followed by coughing, snoring and arousal.
Recently he has become hypertensiveand is also on treatment for impotence. His 24-h
urinary free cortisol level is normal. Which diagnosis best fits this picture?
A Cushing's disease
B Obstructive sleep apnoea
C Simple snoring
D Simple obesity
ANSWER:B
685
An adolescent girl has chronic cough and recurrent respiratory infections over the past two
to three years. Which one of the following pieces of clinical information in her history would
point most strongly to the development of bronchiectasis?
A Pale stools and low weight
B History of wheeze
C Previous whooping cough in early childhood
D Serum precipitins to Aspergillus fumigatus
ANSWER:C
686
A 64-year-old mechanic and lifelong smoker noticed haemoptysis a few days after he had a
cold. Clinical examination is unremarkable. His chest X-ray shows bilateral hilar enlargement
and mediastinal widening. What is the next step in obtaining the diagnosis?
A CT thorax
B Bronchoscopy
C Ventilation-perfusion scan
D Sputum sample
ANSWER:A
687
A 65-year-old patient with new-onset chronic obstructive pulmonary disease (COPD) asks
you about his prognosis. Which of the following single tests is the most important predictor
of survival in patients with COPD?

A Blood gases
B Chest X-ray
C FEV1
D Exercise tolerance
ANSWER:C
688
Which treatment improves the long-term prognosis in patients with chronic obstructive
pulmonary disease (COPD)?
A Inhaled steroids
B Oral steroids
C Inhaled b2-agonists
D Long-term domiciliary oxygen therapy
ANSWER:D
689
A 16-year-old boy is brought to casualty after a fire in his parents' house. He inhaled a lot of
smoke and has a hoarse voice, stridor and burned nasal hairs. Due to deterioration in his
peak flow rate and arterial blood gases, he has been intubated, ventilated and transferred to
the intensive care unit where his condition is now stable. In terms of investigation, what is the
most important next step in assessing this boy's condition?
A CT thorax
B Chest X-ray
C Bronchoscopy
D Ventilation-perfusion scan
ANSWER:C
690
A 21-year-old man presents with episodic pain in his buttocks, low back pain and stiffness
that is worst in the mornings. A lateral X-ray of his lower spine shows blurring of the upper
and lower vertebral rims at the thoracolumbar junction. He is found to be HLA-B27-positive.
Given the likely diagnosis, what would be the most appropriate treatment for him?
A Aspirin
B Prednisolone
C Diclofenac
D Physiotherapy
ANSWER:D

691
A 10-year-old girl presents with a high fever, rash and hip and knee joint pains. A slit-lamp
examination of her eyes is normal. Blood tests are negative for autoantibodies. What is the
most likely diagnosis?
A Persistent oligoarthritis
B Still's disease
C Polyarticular arthritis
D Enthesitis-related arthritis
ANSWER:B
692
A 73-year-old woman complains of dry, gritty eyes, a constant sensation of mouth
drynessand swollen parotid glands. You suspect that she has Sjögren's syndrome. Which of
the following tests is not likely to help in establishing the diagnosis?
A Serum antibody to Ro antigen
B Lymph node biopsy
C Schirmer's test
D Labial biopsy
ANSWER:B
693
A 55-year-old man presents with bowed legs, low back pain and increasing deafness over the
past 6 months. His father was similarly affected at 60 years of age and died of bone cancer.
Given the likely clinical diagnosis, what would be the most characteristic finding in a blood
test?
A Decreased serum calcium levels
B Elevated serum phosphate
C Elevated serum alkaline phosphatase
D Elevated ESR
ANSWER:C
694
A 62-year-old woman is being treated for a pathological fracture of her right femur. An X-
ray of the femur shows patchy sclerosis, thickening of the trabeculae and
dedifferentiation.Given the likely diagnosis, what treatment would be most appropriate for
her?
A Calcium supplements
B Calcitonin
C Hormone replacement therapy

D Tiludronate
ANSWER:D
695
A 65-year-old woman being treated for epilepsy presents with bone pain and muscle
weakness. Blood tests show increased serum alkaline phosphatase, normal plasma calcium
and low serum phosphate levels. An X-ray of the femur reveals linear areas of low density
surrounded by sclerotic borders. What is the most likely diagnosis?
A Osteoporosis
B Osteomalacia
C Paget's disease
D Osteosarcoma
ANSWER:B
696
A 35-year-old man presents with abdominal pain, joint pains, fever and weight loss. He gives
a history of passing bulky, malodorous stools over the past month that are difficult to flush
away. A biopsy of the small bowel shows stunted villi with PAS (Periodic acid-Schiff)-
positive macrophages. What is a characteristic finding in this condition?
A Occult blood in stools
B The presence of bacilli within macrophages on electron microscopy
C Mesenteric thickening with lymph node enlargement on CT scan
D Positive HLA-B27
ANSWER:B
697
A 46-year-old woman with a history of duodenal ulcer has developed progressively
worsening pain, swelling and stiffness of her metacarpophalangeal, distal interphalangeal and
wrist joints. On examination there is wasting of the muscles of the hand and limited
movement of the joints. Given the most likely diagnosis, which drug would be most
appropriate for her condition?
A Prednisolone
B Sulfasalazine
C Ibuprofen
D Aspirin
ANSWER:B

698
A 27-year-old woman is referred to the rheumatology clinic by her gynaecologist with a
history of swelling and pain in her right big toe and left knee. The only positive finding on
gynaecological examination was the presence of cervicitis. An endocervical swab tested
positive for chlamydial infection. What is the most probable diagnosis in this case?
A Acute gouty arthritis
B Rheumatoid arthritis
C Reactive arthritis
D Septic arthritis
ANSWER:C
699
A 30-year-old woman presents with severe scaly, erythematous lesions all over her body.
She has also developed swelling and severe pain in her distal interphalangeal joints. Given
the diagnosis of psoriasis, what treatment is likely to be most suitable for her?
A Topical corticosteroids
B Methotrexate
C Coal tar
D Long-wave ultraviolet radiation (psoralen ultraviolet A; PUVA)
ANSWER:B
700
A 22-year-old homosexual man gives a history of high-grade fever associated with pustules
on his hands and severe joint pain 4 weeks ago. His left knee is now swollen and red.
Cultures from blood and joint aspirate are negative. Urethral discharge shows the presence of
gonococci. He has a history of penicillin sensitivity. Which drug would be most suitable in
this condition?
A Ciprofloxacin
B Amoxicillin
C Prednisolone
D Erythromycin
ANSWER:A
701
A 47-year-old woman presents with an inability to raise her arms over her shoulders. Over
the past few weeks she has also been having difficulty swallowing food. Onexamination
there is muscle wasting and the muscles are tender with reduced tendon reflexes. Her serum
creatine kinase is elevated. What is the most likely diagnosis?
A Polymyalgia rheumatica

B Polymyositis
C Hypocalcaemia
D Painful arc syndrome
ANSWER:B
702
A 73-year-old man presents with severe back pain. An X-ray shows the presence of
osteolytic lesions in his vertebrae and pelvis. He also complains of malaise but no other
symptoms of note.Based on the clinical findings and radiology, what is the most probable
diagnosis?
A Paget's disease of bone
B Prostatic carcinoma
C Multiple myeloma
D Hypocalcaemia
ANSWER:C
703
A 65-year-old woman with a past medical history of osteoarthritis only affecting her hand
joints and diet-controlled diabetes mellitus complains of a sudden onset of pain, swelling and
stiffness in her right knee. Examination shows that the right knee is swollen, erythematous
and tender. Which of the following tests is most likely to lead to a diagnosis?
A X-ray of the knee
B Autoimmune screen
C Serum uric acid level
D Aspiration and examination of the synovial fluid
ANSWER:D
704
A 28-year-old man presents to the clinic with painful knees and ankles. He is noted to have a
rash on the glans penis. He has a history of urethritis due to Chlamydia trachomatis . He has
also recently attended the ophthalmology department for an episode of uveitis. What is the
most likely diagnosis?
A Reiter's syndrome
B Reactive arthritis
C Gouty arthritis
D Septic arthritis
ANSWER:A

705
A 35-year-old woman is diagnosed with systemic lupus erythematosus. What is the most
common finding on blood testing that would be of help in supporting your clinical findings?
A Anti double-stranded DNA
B Rheumatoid factor
C VDRL-positive
D Low complement levels
ANSWER:D
706
A 65-year-old woman presents with severe pain and stiffness of her shoulders and neckthat is
worse in the mornings and lasts for more than an hour. Physical examination is
unremarkable. Blood tests show a mild normocytic normochromic anaemia. Her ESR is 77
mm/1st hour. What is the most likely diagnosis?
A Polymyalgia rheumatica
B Polymyositis
C Hypocalcaemia
D Painful arc syndrome
ANSWER:A
707
A 58-year-old woman complains of severe unilateral temporal headaches and jaw pain when
eating. A provisional diagnosis of giant-cell arteritis is made.Which of the following is a
characteristic clinical feature of this condition?
A All peripheral arteries are involved
B It does not occur in the absence of polymyalgia rheumatica
C Negative temporal artery biopsy excludes the disorder
D Treatment is monitored by measuring CRP levels
ANSWER:D
708
A 45-year-old man presents with fever, malaise, weight loss and myalgia over the past
month. You suspect polyarteritis nodosa and arrange to perform some blood tests. Which
abnormality might you most expect to find?
A Elevated creatinine
B Anaemia
C Leucopenia
D Thrombocytosis

ANSWER:A
709
A young African-American woman is diagnosed as having systemic lupus erythematosus.
What is the characteristic epidemiological feature of this condition?
A It is about twice as common in women than in men
B The highest incidence is amongst Caucasian women
C The age of onset is usually over 40 years
D It is associated with HLA-B8 and -DR3 in Caucasians
ANSWER:D
710
A middle-aged man with red scaly patches on his elbows and knees presents with pain in the
distal interphalangeal joints. You suspect psoriatic arthritis. Which of the following features is
most strongly linked in men to this condition?
A Age of onset 20-30 years
B Occurrence of arthropathy at the same time as the skin lesions
C Minimal destruction of cartilage and bone
D Involvement mainly of the distal interphalangeal joints
ANSWER:D
711
A 39-year-old woman complains of swelling, stiffness and pain in her fingers. She also tells
her doctor that in winter her fingers often turn dark in colour. Her autoimmune screen shows
the presence of anticentromere antibody. Which of the following is she most likely to have?
A Rheumatoid arthritis
B Systemic lupus erythematosus
C CREST variant of scleroderma
D Polyarteritis nodosa
ANSWER:C
712
A 62-year-old woman presents with severe pain and stiffness in her shoulder muscles and
pelvis for the past 3 weeks that is worse in the mornings. Her ESR is raised. What is the most
likely diagnosis?
A Polymyalgia rheumatica
B Polymyositis
C Pseudogout
D Psoriatic arthritis

ANSWER:A
713
A 22-year-old woman presents with red scaly plaques on her elbows, knees, lower back and
scalp. She also has pitting and yellow-brown discoloration of her nails and painful deformed
finger and toe joints. Given the likely clinical diagnosis, what would be the most appropriate
treatment, taking current UK guidelines into account?
A NSAIDs
B Sulfasalazine
C Methotrexate
D Corticosteroids
ANSWER:C
714
A 50-year-old diabetic woman with a history of osteoarthritis of her knees suddenly
develops pain and swelling in her right knee. On examination the knee is red, hot, swollen
and very tender. Which investigation would be most helpful in the management of this case?
A Plain X-ray of the knee
B Joint aspiration and Gram-staining
C Joint fluid microscopy
D Blood culture
ANSWER:B
715
A 20-year-old man presents with a raised red and scaly lesion on his glans penis, red
discolouration and pain in both eyes and pain and swelling of his right knee. Over the past
few days he has noticed painless red plaques on his hands and feet. A diagnosis of Reiter's
syndrome is suspected. Which additional clinical feature would best support this diagnosis?
A History of a flu-like illness 4-6 weeks prior to symptoms
B Presence of keratoderma blenorrhagica
C Family history of ulcerative colitis
D Positive gonococcal culture of urethral discharge
ANSWER:B
716
A 75-year-old woman presents with chronic back pain. An X-ray of the spine shows
vertebral crush fractures and evidence of osteoporosis.Which of the following blood results
would be most in keeping with this diagnosis?

A Low calcium levels
B Elevated phosphate levels
C Normal or high alkaline phosphatase levels
D Decreased 1,25-dihydroxycholecalciferol levels
ANSWER:C
717
A 25-year-old mechanic complains of stiffness and low back pain that is worse in the
mornings. He is HLA-B27-positive. A provisional diagnosis of ankylosing spondylitis is
made. What would be the most characteristic finding on an X-ray of the lower spine?
A Narrowing of disc space
B Erosion of the apophyseal joints
C Osteophyte formation
D Spondylolisthesis
ANSWER:B
718
A 22-year-old college student complains of stiffness and low back pain that is worse in the
mornings. An X-ray shows obliteration of the sacroiliac joints. Given the likely clinical
diagnosis, what would be the most appropriate treatment for him?
A Spinal osteotomy
B Spinal extension exercises
C Bedrest and immobilisation
D Prednisolone
ANSWER:B
719
A 55-year-old woman on procainamide develops drug-induced lupus erythematosus. What is
the most characteristic clinical feature of this condition?
A It may occur with chlorpromazine
B It commonly involves the kidneys
C It rarely causes pulmonary disease
D The symptoms may be alleviated with long-term steroids
ANSWER:A
720
A 79-year-old woman who drinks 30 units of alcohol per week presents with a red, hot
swollen ankle. Which investigation may yield a definitive diagnosis?
A Blood culture
B Joint aspiration and microscopy

C Joint aspiration and culture
D X-ray of the ankle
ANSWER:B
721
A 4-year-old girl with a 1-day history of increasing hip pain is unable to stand. Her WCC is
20 x 109 /l, ESR 90 mm/1st h and CRP 275 mg/l. A radiograph of the hip shows a widened
joint space. What is the most likely diagnosis?
A Perthe's disease
B Slipped upper femoral epiphysis
C Septic arthritis
D Congenital dislocation of hip
ANSWER:C
722
A 5-year-old Asian boy, who has been having episodes of fever and a persistent cough for
the past 3 weeks, now complains of right hip pain. Blood tests show: WCC 19 x 109 /l, ESR
110 mm/1st h and CRP 102 mg/l. An X-ray of the hip joint shows diffuse rarefaction. What
is the most likely diagnosis?
A Septic arthritis
B Tuberculous arthritis
C Osteomyelitis
D Reactive arthritis
ANSWER:B
723
A 65-year-old woman who lives alone complains of increasing pain in her left knee and
episodes of the joint 'giving way'. She is no longer able to climb stairs. Valgus deformity
with instability is also noted. What treatment would be most appropriate for her?
A Oral NSAIDs
B Intra-articular steroid injections
C Joint replacement
D Physiotherapy
ANSWER:C
724
A 22-year-old man who suffers from inflammatory bowel disease has developed pain and
stiffness in his lower back over the past 6 months. Examination reveals tenderness over both
sacroiliac joints. He tests positive for the HLA-B27 gene. What is the most probable
diagnosis?

A Prolapsed intervertebral disc
B Rheumatoid arthritis
C Ankylosing spondylitis
D Osteoarthritis
ANSWER:C
725
A 72-year-old man with heart disease is on diuretics. He complains of stiff, painful hands
and knees.On examination Heberden's nodes are seen. What is the most appropriate
treatment?
A Regular paracetamol
B Allopurinol
C Oral NSAIDs with gastric protection
D Knee replacement
ANSWER:A
726
A 35-year-old woman complains of bilateral stiff and painful joints in her hands and feet for
the past 3 months. The stiffness lasts for more than an hour in the mornings. On examination
her fingers are swollen and stiff. Movement is painful. Which test would be most relevant in
this case?
A X-ray of the hands and feet
B Serum uric acid levels
C Joint aspirate for crystals
D Plasma rheumatoid factor
ANSWER:D
727
A 65-year-old diabetic woman on indometacin and glibenclamide has had a blood test that
shows evidence of renal failure, hyperkalaemia and hyperchloraemia. What is the most likely
underlying cause of her biochemical abnormalities?
A Renal tubular acidosis
B Renal artery stenosis
C Diabetic nephropathy
D Minimal-change nephropathy
ANSWER:A

728
A 74-year-old man has an acutely painful, red and swollen knee. He is suffering from
congestive cardiac failure, chronic renal impairment and is currently on digoxin and
furosemide. What treatment would be most suitable for the symptomatic relief of his joint
pain?
A Aspirin
B Paracetamol
C Colchicine
D Diclofenac
ANSWER:C
729
A 57-year-old man with longstanding osteoarthritis of his right hip is seen in the clinic prior
to admission for hip replacement. He has a history of peptic ulcer and is on lansoprazole.
Which anticoagulant formulation would offer him the best protection against postoperative
thrombosis?
A Subcutaneous low molecular weight heparin
B Subcutaneous unfractionated heparin
C Warfarin
D Intravenous heparin
ANSWER:A
730
A 12-year-old girl complains of pain in her hip and knee joints, as well as fever, bloody
diarrhoea and abdominal pain. A barium enema shows rose-thorn ulcers. What is the most
characteristic feature seen on colonoscopy in this condition?
A Red-raw mucosa
B Pseudopolyps
C Discrete ulcers
D Colonic dilatation
ANSWER:C
731
A 67-year-old man known to be hypothyroid says he woke up in the morning with a
painful, warm, red and swollen right knee. An X-ray of his knee shows calcification of the
meniscus only. What is the most likely diagnosis?
A Acute gout
B Osteoarthritis
C Rheumatoid arthritis

D Pyrophosphate arthropathy
ANSWER:D
732
A 37-year-old gym instructor gives a 2-year history of numbness and burning of his fingers
precipitated by cold. He now feels tightness in the fingers and is unable to extend his fingers
completely. He also complains of a progressive difficulty in swallowing food. You suspect
limited cutaneous scleroderma. Which of the following blood investigations would most aid
in the diagnosis?
A Normocytic normochromic anaemia
B Microangiopathic haemolytic anaemia
C Anticentromere antibodies
D Antinuclear antibodies
ANSWER:C
733
A 20-year-old college student complains of increasing back pain and early morning stiffness.
An X-ray of his lower back shows erosion of the apophyseal joints and obliteration of the
sacroiliac junction. Given the most likely clinical diagnosis from these findings, what
treatment would be most suitable for him?
A Oral NSAIDs
B Colchicine
C Intra-articular steroid injections
D Spinal osteotomy
ANSWER:A
734
A 62-year-old woman complains of general lethargy, morning stiffness, inability to comb
her hair because of arm pain. There is no muscle tenderness. Her ESR is 57 mm/1st
h.Electromyography of the deltoid muscle is normal. What is the most likely diagnosis?
A Guillain-Barrè syndrome
B Polymyositis
C Polymyalgia rheumatica
D Multiple sclerosis
ANSWER:C
735
A 45-year-old woman with rheumatoid arthritis presents with leg ulcers and swollen neck
glands. A diagnosis of Felty's syndrome is made. Which typical finding on blood testing
would you expect to find in her case?

A Positive Coombs' test
B Normocytic normochromic anaemia
C Lymphocytosis
D Reticulocytosis
ANSWER:B
736
A 65-year-old woman presents a 1-month history of weakness in the hip region and inability
to walk or get up from a chair. On examination there is wasting of the pelvic girdle muscles.
She has also developed a purple discolouration of the eyelids. You suspect adult
dermatomyositis. Which of the following investigations will be most helpful in the diagnosis
and management of this condition?
A Raised serum creatine phosphokinase
B Presence of rheumatoid factor
C Presence of antinuclear antibody
D Myositis-specific antibodies
ANSWER:A
737
A 20-year-old football player presents with a swollen left knee and locking. He complains of
pain after exercise. On examination wasting over the quadriceps and lateral aspect of the
joint are noted. Arthroscopy of the joint reveals three loose bodies in the synovial cavity.
What is the most likely diagnosis?
A Chip fractures of the joint surfaces
B Osteoarthritis
C Synovial chondromatosis
D Osteochondritis dissecans
ANSWER:D
738
A 24-year-old man suddenly develops severe back pain while lifting some luggage. He is
unable to straighten up and subsequently develops numbness and weakness in his left leg
followed by retention of urine. He is unable to move his toes. Given the likely clinical
diagnosis, which plan of management is likely to be required?
A Lumbar traction
B NSAIDs
C Extension exercises
D Laminectomy and fusion

ANSWER:D
739
A 70-year-old man became wheezy while walking uphill, and tripped hitting his left eye. He
now complains of pain in this eye; the cornea is hazy, the globe feels very firm to palpation
and there is a hyphaema obscuring most of the iris. Which of the following is the most
appropriate treatment?
A Examination under anaesthesia
B Topical β-blockers
C Intravenous carbonic anhydrase inhibitors
D Topical anticholinergics
ANSWER:C
740
A 60-year-old diabetic complains of floaters of 24 hours' duration, followed by a sudden
painless loss of vision in his right eye. His corrected visual acuities are 6/60 (right eye) and
6/12 (left eye). His right retina cannot be visualised, and the left retina contains scattered
pigmented spots in the periphery. What is the most appropriate management plan?
A Stop daily aspirin
B Admit for laser treatment within 48 hours
C Observe and review in ophthalmology out-patients
D Prescribe oral acetazolamide
ANSWER:C
741 What is the typical opthalmological finding in patients with subacute bacterial endocarditis?
A Cherry red macula
B Janeway lesion
C Roth's spots
D Retinal artery aneurysms
ANSWER:C
742
A teenager is referred from his optician with a diagnosis of Lisch nodules of the iris. What is
the most likely sign to observe on examination?
A Ash-leaf spots on the trunk
B Axillary freckles
C Ectopia lentis

D Haemangioblastoma of the spinal cord
ANSWER:B
743
A 68-year-old man presents to his GP with a one-week history of blurred vision affecting his
left eye. On examination, his visual acuities are 6/6 right eye and 6/18 left eye. No papillary
defect is noted. Dilated fundal examination reveals evidence of widespread retinal
haemorrhages in all quadrants of the left retina associated with dilated tortuous retinal veins.
The right fundus appears entirely normal. The patient has not seen his GP within the last ten
years. He describes himself as fit and well and on no regular medication. However, a
subsequent examination reveals a blood pressure of 185/100 and a random blood sugar of
12 mmol/L. Which of the following is the most likely ocular diagnosis?
A Branch retinal vein occlusion
B Central retinal vein occlusion
C Hypertensive retinopathy
D Diabetic retinopathy
ANSWER:B
744
A 25-year-old woman presents with loss of weight, gritty eyes and double vision on looking
up. Which of the following is the most important investigation?
A CT brain
B Exophthalmometry
C Formal perimetry
D Hess chart
ANSWER:C
745
A 25-year-old man develops cirrhosis of the liver and is referred from the gastroenterology
clinic to the ophthalmology clinic for evidence of Wilson's disease. What should the
gastroenterologist ask the ophthalmologist to look for?
A Corneal arcus
B Hudson-Stahli lines
C Kayser-Fleischer rings
D Cornea verticillata
ANSWER:C

746
A 45-year-old woman is found by her optometrist to have band keratopathy and is referred
to the eye clinic. Which of the following investigations is likely to be helpful in determining
an underlying cause?
A Cholesterol
B Ferritin
C Serum calcium
D Gamma GT
ANSWER:C
747
A patient with diabetic retinopathy is treated with panretinal photocoagulation in the eye
clinic and followed up in the diabetic clinic. Which of the following features found 6 months
after treatment is an indication for further laser treatment?
A Visual field constriction
B Vitreous haemorrhage
C Retinal burns
D Optic atrophy
ANSWER:B
748
A 26-year-old man with aortic regurgitation is referred from cardiology to the eye clinic to
look for features of Marfan's syndrome. Which of the following ocular features suggests the
diagnosis?
A High hyperopia
B Band keratopathy
C Dislocated lenses
D Raised intraocular pressure
ANSWER:C
749 Altitudinal hemianopia is a cardinal feature in a patient who?
A Denies the fact he is blind
B Is 72 years old with macular degeneration
C Is 70 years old with headache, vomiting and swelling of the optic disc
D Is a 74-year-old man with multiple cholesterol emboli on fundoscopy
ANSWER:D

750
A sixty-year-old man presents with a week's history of painless diplopia first noticed when
reading. The images are constantly horizontally and vertically separated, although he
comments that the degree of separation varies. On examination the visual acuities are 6/6 in
either eye. There is no pupil abnormality. There is a left ptosis, partially covering the pupil
and reduced abduction and depression of the left eye, both in abduction and adduction, with
other ocular movements appearing normal. There is no other abnormality on examination.
Which one of the following is the MOST likely diagnosis?
A Orbital apex syndrome
B Ocular myaesthenia gravis
C Third nerve palsy
D Fourth nerve palsy
ANSWER:B
751
A 22-year-old man with ulcerative colitis and chronic lower back pain complains of a red
painful eye. Which one of the following is likely to be present on examination?
A Purulent discharge
B Photophobia on ophthalmoscopy
C A dilated pupil
D Profound visual loss
ANSWER:B
752
A 32-year-old woman presents as an emergency with sudden, painless loss of vision in her
right eye. She has been a type-1 diabetic for the past 16 years. On examination, her vision is
reduced to hand movements in the right eye. Pupil reactions are normal. Dilated fundal
examination reveals an irregular red reflex with evidence of a vitreous haemorrhage. Which
of the following would be the most likely causative retinal abnormality?
A Microaneurysm
B Cotton wool spot
C Hard exudates
D Neovascularisation at the optic disc
ANSWER:D

753
An 82-year-old woman presents with sudden loss of vision in her left eye. On further
questioning, she complains of a left-sided headache over the past few weeks, associated with
tenderness of her head when she brushes her hair. On examination, her vision is reduced to
counting fingers in the left eye. A left relative afferent pupillary defect is present.
Fundoscopy reveals a pale, swollen left optic disc with some flame-shaped haemorrhages.
The right eye is entirely normal. An urgent erythrocyte sedimentation rate (ESR) is elevated
at 72 mm/hr (< 30). Which of the following management options should be carried out first?
A Temporal artery biopsy
B Ophthalmic outpatient review
C Administration of high-dose systemic steroids
D Automated visual field assessment
ANSWER:C
754
A patient presents to the medical clinic with diplopia. Which of the following suggests that
the trochlear nerve is involved?
A The diplopia is horizontal
B The diplopia is episodic
C The diplopia is worse at night
D The diplopia is torsional
ANSWER:D
755
A 26-year-old man presents with sudden-onset headache and double vision. The A&E
doctor diagnoses IIIrd (oculomotor) nerve palsy. Which of the following is the most likely
cause?
A Posterior communicating artery aneurysm
B Acoustic neuroma
C Diabetes mellitus
D Extradural haematoma
ANSWER:A
756
A 70-year-old man presents with a gradually worsening, droopy, right upper eyelid. There is
a right ptosis and anisocoria, greater when the room lights are off than in light conditions.
Which of the following is the best investigation?
A Anti-acetylcholine receptor antibodies

B Blood glucose
C Chest X-ray
D Dilute pilocarpine eye-drop test
ANSWER:C
757
An 80-year-old man is referred to the psychogeriatrician with features of dementia. The
neurology SpR is called because the SHO has noted an abnormality of eye movements. The
patient is not on any medication. The SpR confirms that the eye movements are limited in
upward gaze on pursuit movements, but that the eyes move upwards normally when he
flexes the patient's neck. Which of the following conclusions is valid?
A The patient has an abnormal vestibulo-ocular response
B The patient has a supranuclear gaze palsy
C The patient has tardive dyskinesia
D The patient has an oculogyric crisis
ANSWER:B
758
A 19-year-old student presents with a complaint of visual loss. Which of the following
suggests a conversion reaction?
A The pupils are unequal in size
B There is a spiralling visual field loss
C Optokinetic nystagmus (OKN) cannot be elicited
D The discs show temporal pallor
ANSWER:B
759
During a routine insurance medical examination, a GP notices that a 30-year-old female
patient has absent ankle jerks and unequal pupils. Which of the following is the most likely
diagnosis?
A Holmes-Adie syndrome
B Horner's syndrome
C Third nerve palsy
D Mescaline ingestion
ANSWER:A
760
A patient is examined in the diabetic clinic and found to have circinate hard exudates in both
fundi, with reduced visual acuity. What is the most likely diagnosis?
A Normal fundi

B Background retinopathy
C Maculopathy
D Preproliferative retinopathy
ANSWER:C
761
A 70-year-old woman presents with a sudden loss of vision in one eye. Which of the
following clinical findings suggests a diagnosis of giant-cell arteritis?
A Central retinal artery occlusion (CRAO)
B Pale disc swelling
C Isolated central scotoma
D Abnormal consensual light response
ANSWER:B
762
A patient with controlled ocular myasthenia gravis develops an acute infection. Which of the
following antibiotics is contraindicated?
A Aminoglycosides
B Metronidazole
C Macrolides
D Quinolones
ANSWER:A
763
A 70-year-old woman presents with a sudden loss of vision in one eye. Which of the
following investigations most strongly supports a diagnosis of temporal arteritis?
A An ESR of 40 mm/hour
B Abnormal CRP
C Giant-cell infiltrate in a temporal artery biopsy
D Homonymous hemianopia on visual field
ANSWER:C
764
A 40-year-old man presents to his GP with unequal pupils. The GP considers that IIIrd
(oculomotor) nerve palsy is the likely diagnosis. Which of the following statements is true?
A The pupil on the affected side is smaller
B The pupil on the affected side reacts normally to light
C The pupil on the contralateral side reacts normally to light
D The pupil on the affected side reacts normally to accommodation
ANSWER:C

765
A patient is seen with features of proliferative retinopathy in one eye and background
diabetic retinopathy in the other. Which of the following tests is most likely to provide an
explanation?
A Chest X-ray
B Electrocardiography
C Coagulation screen
D Carotid Doppler
ANSWER:D
766
A diabetic patient with diplopia is found to have a third nerve palsy. Which of the following
clinical features would most point to a compressive cause?
A Ptosis
B Impaired adduction
C Pupil involvement
D Impaired elevation
ANSWER:C
767
A patient is referred to the dermatology clinic with facial flushing and early rhinophyma.
The dermatologist seeks an ophthalmic opinion. Which of the following findings supports a
diagnosis of acne rosacea?
A Iritis
B Cataract
C Central retinal vein occlusion
D Keratitis
ANSWER:C
768
The medical SHO telephones the consultant to say she has diagnosed a patient as having
Behçet's disease. The patient presented to A&E with reduction in vision in one eye. Which of
the following features supports her diagnosis?
A The patient is of Celtic extraction
B There is a strong family history of blindness
C The patient has a small-joint polyarthropathy
D The patient has active oral ulceration
ANSWER:D

769
A patient is examined in the diabetic clinic and found to have dark cluster haemorrhages in
both fundi. What is the most likely diagnosis?
A Normal fundi
B Background retinopathy
C Maculopathy
D Pre-proliferative retinopathy
ANSWER:D
770
A 30-year-old man, under investigation for abdominal cramps and passing blood rectally,
presents with an acutely painful, red and photophobic eye. What is the most likely sign on
ocular examination?
A Conjunctival purulent discharge
B White corneal stromal infiltrate
C Mydriasis of the affected eye
D Hypopyon
ANSWER:D
771
A 40-old woman with AIDS presents to the GU clinic with a shadow in her vision in one eye.
Which of the following supports a diagnosis of cytomegalovirus retinitis?
A Conjunctival injection
B Mydriasis on the affected side
C Retinal haemorrhages
D Macular oedema
ANSWER:C
772
A patient is examined in the diabetic clinic and found to have a vitreous haemorrhage
precluding a view of his fundi. He is admitted for bed rest. After 3 days the fundi can be
visualised. What is the most likely diagnosis?
A Proliferative retinopathy
B Background retinopathy
C Maculopathy
D Preproliferative retinopathy
ANSWER:A

773
A diabetic 46-year-old man is found in the diabetic clinic to have reduced visual acuity.
During a telephone referral to the eye clinic, the ophthalmologist asks if the patient has any
risk factors for macular oedema. Which of the following should the referring physician bring
to his attention?
A Background diabetic retinopathy
B Low glycosylated haemoglobin
C Hypercholesterolaemia
D Proteinuria
ANSWER:D
774
A patient is being referred to the eye clinic with suspected early diabetic retinopathy. Which
of the following tests, if abnormal, most strongly supports the diagnosis?
A Electroretinography (ERG)
B Fluorescein angiography
C Ocular ultrasound
D Visual evoked potentials (VEP)
ANSWER:B
775
A 30-year-old man is referred to the dermatology clinic and found to have café-au-lait spots.
Which of the following features subsequently found in the eye clinic suggests a diagnosis of
neurofibromatosis?
A Busacca nodules
B Lisch nodules
C Brushfield spots
D Koeppe nodules
ANSWER:B
776
A 32-year-old woman with rheumatoid arthritis presents to the rheumatology clinic with a
complaint of severe pain and reduced vision in one of her eyes. The medical SpR notes that
one eye is dusky red in colour and that the vision in this eye is reduced to 6/36. What is the
appropriate management?
A The patient should be booked for an urgent CT scan
B The patient should be started on artificial tears
C The patient should be started immediately on immunosuppressant therapy
D The patient should be referred urgently to the ophthalmology clinic

ANSWER:D
777
A 75 year old gentleman with type 2 diabetes melllitus presents for his retinal screening. On
the retinal photograph there is evidence of cotton wool spots and flame haemorrhages. There
also appears to be new vessel formation very close to the optic disc. What is the next stage in
management?
A Fenofibrate
B Insulin
C Referral for laser photocoagulation
D Referral for fluorescein angiography
ANSWER:C
778 Which of the following ocular signs would you find in acne rosacea?
A Cataract
B Uveitis
C Ptosis
D Keratitis
ANSWER:D
779
A 68 year old male with previous history of TIA presents with unilateral painless vision loss.
The retina is pale and the macula appears as a cherry red spot. What is the most likely cause
of the vision loss?
A Retinal detachment
B Central retinal artery occlusion
C Age related macular degeneration
D Retinal vein occlusion
ANSWER:B
780
A 9 year old boy is brought to his GP. his mother has noticed him tripping up frequently. he
states he has difficulty seeing in the dark. He is otherwise very well. A full neurological
examination is performed and is normal. Fundoscopy is performed and reveals mid
peripheral bone spicules. Which of the following is the most likely diagnosis?
A Retinoblastoma
B Retinitis Pigmentosa
C Glaucoma
D Alport's Syndrome

ANSWER:B
781
A 30-year-old presents to her GP with reduced vision in her left eye. She has previously
presented with abnormal sensory changes in her leg which have now resolved. On
examination her visual acuity is greatly reduced and there is evdence of a relevant afferent
pupil defect. All movements of the eye are very painful. On fundoscopy and neurological
examination there is nil of note. What is the most likely diagnosis?
A Optic neuritis
B Age related macular degeneration
C Orbital tumour
D Retinal vein occlusion
ANSWER:A
782
A 68 year old femalee presents with central visual blurring. On examination there appears to
be small yellow deposits within the maculae and a small haemorrhage at the maculae.
Fluoroscein angiography is performed and shows neovascularisation within the macula of
both eyes. You suspect wet age related macular degeneration. Which of the following is an
important risk factor for the development?
A Hypertension
B Smoking
C Cataract surgery
D Glaucoma
ANSWER:B
783
A 34-year-old professional footballer is evaluated for symptoms of 'dizziness' during
exercise. Physical examination reveals a laterally displaced apical impulse. On auscultation,
there is a 2/6 mid-systolic murmur in the aortic area that increases on sudden standing. The
ECG shows LVH and Q waves in the V2-V5 leads. What is the most likely diagnosis?
A Young-onset hypertension
B Acute MI
C Aortic stenosis
D Hypertrophic cardiomyopathy
ANSWER:D

784
What is the most likely lipid abnormality in a 48-year-old Asian man with good glycaemic
control?
A Elevated high-density lipoprotein (HDL)
B Elevated low-density lipoprotein (LDL)
C Low HDL/elevated triglycerides
D Low HDL/elevated LDL
ANSWER:C
785
A 65-year-old woman with a history of heavy smoking presents for review. She has woken
during the early hours of the morning for the second time with shortness of breath so bad
that she had to fling open the windows. On examination there are crackles in the lung bases,
her chest X-ray shows bilateral fluffy perihilar shadowing. ECG reveals smallanterior Q
waves and a sinus tachycardia of 105 bpm. What diagnosis fits best with this clinical picture?
A Cryptogenic fibrosing alveolitis
B Pulmonary embolus
C Exacerbation of COPD
D Pulmonary oedema
ANSWER:D
786
A 42-year-old painter presents to A&E with symptoms of vertigo, diplopia and gait
unsteadiness at the end of a busy afternoon painting the interior of a property. On
examination there is a markedly lower blood pressure in the left arm. What diagnosis fits best
with this clinical picture?
A Anterior circulation transient ischaemic attack
B Subclavian steal syndrome
C Vestibular neuronitis
D Posterior circulation transient ischaemic attack
ANSWER:B
787
A 70-year-old-man reverts to atrial fibrillation after several attempts at cardioversion, but
remains symptomatic despite rate control with digoxin and metoprolol. He developed
pulmonary fibrosis with amiodarone. Which of the following will be the next step in the
management of this patient?
A Switch metoprolol to amlodipine

B Double the dose of digoxin
C Perform radiofrequency ablation of the AV node, and implant a pacemaker
D Make another attempt at cardioversion
ANSWER:C
788
A 40-year-old man is noted to have palmar crease xanthomas. Which form of lipid disorder
is most likely?
A Familial hypercholesterolaemia
B Familial mixed hypercholesterolaemia
C Broad b disease
D Familial hypertriglyceridaemia
ANSWER:C
789
A 78-year-old-man presents to Casualty with a history of syncope. An ECG shows complete
heart block. Which of the following physical signs is consistent with the diagnosis?
A Regular cannon 'a' waves on JVP
B Soft first heart sound
C Low-volume pulse
D Basal systolic murmur
ANSWER:D
790
A 56-year-old man has known tricuspid regurgitation. Which part of the jugular venous
waveform is likely to be most prominent?
A a wave
B c wave
C v wave
D x descent
ANSWER:C
791
A 50-year-old woman presents with an acute myocardial infarction, and thrombolysis is
being considered.Which one of the following would be an absolute contraindication for
thrombolytic therapy?
A Background diabetic retinopathy
B
Past history of a minor stroke 5 years ago with full recovery and no evidence of underlying
cerebrovascular lesion

C Resting blood pressure 220/130 mmHg
D Dyspeptic symptoms
ANSWER:C
792
A 17-year-old youth is brought to the GP by his mother. He was previously seen 2 weeks
earlier suffering from acute pharyngitis. His teeth are in generally poor condition, but
otherwise there is no previous medical history. On examination he is febrile with a
temperature of 38.2 A°C, and has a polyarthritis affecting his knees, ankles, wrists and
elbows. He also appears to have subcutaneous nodules over his elbows, and mitral
regurgitation on cardiovascular examination. What diagnosis fits best with this clinical
picture?
A Bacterial endocarditis
B Juvenile rheumatoid arthritis
C Scarlet fever
D Rheumatic fever
ANSWER:D
793
A 56-year-old man presents with a cardiac rhythm disorder. Which one of the following
scenarios would be an indication for temporary transvenous cardiac pacemaker insertion?
A Asymptomatic 2.8 s sinus pauses
B
A short period of complete heart block complicating inferior myocardial infarction, (pre-
thrombolysis) with blood pressure 110/70 mmHg
C Asymptomatic complete heart block with broad complex ventricular complexes at 35 bpm
D
Mobitz II AV block complicating anterior myocardial infarction with blood pressure 110/70
mmHg
ANSWER:D
794
A 70-year-old man undergoes successful DC cardioversion for atrial fibrillation (AF). Which
one of the following factors best predicts long-term maintenance of sinus rhythm following
this procedure?
A Age under 75 years
B Normal left ventricular function
C Warfarin therapy
D AF duration less than 6 months prior to cardioversion
ANSWER:D

795
A 38-year-old man of Chinese descent who smokes 60 cigarettes per day presents to his GP.
He is developing pain at rest in his legs, and is unable to walk more than a few yards due to
ischaemic pain. On examination there is prolonged capillary refill and necrotic ulcers at the
tips of his toes. There is also evidence of thrombophlebitis. What diagnosis fits best with this
clinical picture?
A Buerger's disease
B Simple peripheral vascular disease
C Polyarteritis nodosa
D Familial hypercholesterolaemia
ANSWER:A
796
A 45-year-old man with a strong family history of ischaemic heart disease presents with
atypical chest pains. Electrocardiographic (ECG) exercise testing shows J point depression of
1 mm with a heart rate of 120 beats/min (bpm). What is the most appropriate next step?
A Coronary angiography
B Dobutamine stress echocardiography
C Radionuclide myocardial perfusion scanning
D Reassure and discharge
ANSWER:D
797
A 67-year-old lady during pre-operative assessment is found to have a small pericardial
effusion located posteriorly on routine Echocardiography. Electrocardiogram (ECG) is
entirely normal. What is the next most appropriate step in her management?
A Cardiac catheterisation
B Reassure
C Pericardiocentesis
D Diuretics
ANSWER:B
798
A 62-year-old patient presents with atrial fibrillation of unknown duration. Which drug may
slow his ventricular rate over a prolonged period but is unlikely to result in cardioversion?
A Adenosine
B Amlodipine
C Digoxin

D Flecanide
ANSWER:C
799
A 55-year-old man who has sustained an acute MI subsequently presents with heart failure.
As well as other treatments the cardiologist has recommended that abeta-blocker be
commenced. According to currently available evidence which of the following beta-blockers
would be most appropriate?
A Celiprolol
B Labetalol
C Bisoprolol
D Propranolol
ANSWER:C
800
A 67-year-old lady is found to have a small pericardial effusion located posteriorly on
routine echocardiography. There is no haemodynamic compromise.What is the next most
appropriate step in her management?
A Diagnostic tap
B Mammography
C Tuberculosis screen
D Reassure
ANSWER:D
801
A 65-year-old woman with a history of heavy smoking presents for review. She has woken
during the early hours of the morning for the second time with shortness of breath so bad
that she had to fling open the windows. On examination there are crackles in the lung bases,
her chest X-ray shows bilateral fluffy perihilar shadowing. ECG reveals small anterior Q
waves and a sinus tachycardia of 105 bpm. What diagnosis fits best with this clinical picture?
A Cryptogenic fibrosing alveolitis
B Pulmonary embolus
C Exacerbation of COPD
D Pulmonary oedema
ANSWER:D

802
A 69-year-old man has been admitted to the emergency department with syncope. He felt
hot, complained of nausea and then fainted.His electrocardiogram (ECG) was normal. His
brother suffers from adult onset epilepsy. What is the most appropriate investigation?
A Electroencephalogram (EEG)
B 24-h ECG
C Tilt test
D Echocardiography
ANSWER:C
803
An elderly gentleman is admitted with syncope. He also complains of shortness of breath and
is diagnosed as having aortic stenosis. Which of the following conditions when associated
with aortic stenosis would indicate a poor prognosis?
A Aortic regurgitation
B Left ventricular failure
C Electrocardiography (ECG) changes
D Endocarditis
ANSWER:B
804
A 46-year-old man is admitted with shortness of breath, headache, blood pressure 190/110
mmHg and fundoscopy showing retinal haemorrhages and papilloedema. ECG revealed left
ventricular hypertrophy, chest x-ray did not reveal any evidence of left ventricular failure.
Which of the following agents would be the most appropriate management?
A Intravenous labetalol
B Intravenous sodium nitroprusside
C Atenolol
D Doxasosin MR oral
ANSWER:C
805
A patient with underlying ischaemic heart disease had two transient episodes of loss of
consciousness but feels fine at present. Both episodes were preceded by afeeling of dizziness,
""vision going black"" and witnesses report that the subject went very pale and then
collapsed, lying motionless for a few seconds before making a rapid recovery. No abnormal
movements were seen during the period of unconsciousness. What investigation will you
order next?

A Echocardiography
B Computed tomography (CT) of the head
C 24-hour electrocardiogram (ECG)
D Cardiac catheterisation
ANSWER:C
806
An elderly man is admitted to the ICU and put on intermittent positive-pressure ventilation.
Which of the following statements is true when compared to spontaneous ventilation?
A Lung volumes are decreased
B Pulmonary vascular resistance is decreased
C Systemic blood pressure rises
D Venous return and cardiac output fall
ANSWER:D
807
A 65-year-old male patient with stable angina complains of shortness of breath after walking
two flights of stairs. He has normal left ventricular function on the echocardiogram and a
positive exercise tolerance test (3 mm ST depression at stage III). What is the most
appropriate therapy?
A Atenolol
B Simvastatin
C Isosorbide mononitrate
D Angiotensin-converting enzyme (ACE) inhibitor
ANSWER:A
808
A 65-year-old female patient with severe heart failure presents with increasing shortness of
breath. Her current pharmacological treatment consists of an angiotensin-converting enzyme
(ACE) inhibitor, loop diuretic and β-blocker. What is the most appropriate management?
A Add digoxin
B Add spironolactone
C Stop β-blocker
D Stop ACE inhibitor
ANSWER:B

809
A 67-year-old man with chronic heart failure is reviewed in terms of his drug therapy.
Which of the following treatments has no proven mortality benefit?
A Bisoprolol
B Digoxin
C Enalapril
D Nitrates and hydralazine
ANSWER:B
810
A 35-year-old-woman of African origin presents with a 4-month history of increasing
swelling over her feet and abdominal distension. She has no history of cough, orthopnoea or
breathlessness on exertion. Her heart rate is 98 beats/minute: irregularly irregular. Her JVP is
markedly raised and she has pitting lower limb oedema. The heart sounds are soft, and there
are no audible murmurs. Abdominal examination reveals hepatomegaly along with ascites.
Chest X-ray reveals a normal cardiac size and clear lung fields. A lateral X-ray shows
calcification around the heart border. Urinalysis is normal. Her ECG shows a low QRS
voltage and lateral T-wave changes. What is the likely diagnosis?
A Dilated cardiomyopathy
B Cirrhosis of the liver
C Constrictive pericarditis
D Restrictive cardiomyopathy
ANSWER:C
811
A 51-year-old married father of three presents with epigastric abdominal pain and weight
loss. Computed tomography CT. of the abdomen reveals an extensive midpancreatic mass
with obvious liver metastases. Needle biopsy of one of the hepatic lesions is consistent with
pancreatic adenocarcinomA. At this point the patient’s primary care physician should tell the
patient
A that a referral to an oncologist will be made
B
the nature of histology and refer the patient to an oncologist and defer discussion of the
prognosis
C
that he has an extremely serious life-threatening illness, but that all measures will be
undertaken to extend the quantity and quality of the patient’s life
D that we now have effective therapy for pancreatic carcinoma
ANSWER:C

812
A 35-year-old woman with a severe anxiety neurosis develops acute appendicitis. The
primary care physician, the surgical consultant, and the infectious disease consultant all
recommend urgent appendectomy. The patient wants to be treated only with antibiotics
because she is concerned about the risks of anesthesiA. The patients states she understands
the reasons for the physician’s recommendations and that she might die if she does not have
this operation, yet she feels strongly that this is what is best for her. At this point the most
appropriate course of action would be to
A call a psychiatrist
B tell the patient that you cannot treat her and refer her to another hospital
C
declare the patient legally incompetent, restrain her, and perform surgery since it will be life
saving
D treat the patient with antibiotics
ANSWER:A
813
What is the most important principle in discussions with a surrogate decision maker for a
patient who is unconscious and incapable of making an important decision regarding
advanced care, especially if there is no documentation of the patient’s desires?
A
Without knowing the patient’s specific wishes, prolongation of life must be carried out at all
costs
B The physician should make a judgment based on the medical facts
C
The physician should decide as the patient would under the circumstances, assuming the
patient knew all information
D The surrogate makes the decision based on what he or she thinks is best for the patient
ANSWER:C
814
A 52-year-old postmenopausal woman comes for a routine visit to her primary care
physician. She asks about the value of hormone replacement therapy. Which of the
following statements is correct regarding this situation?
A
Use of tamoxifen slows the development of osteoporosis but has no effect on the incidence
of breast cancer
B The use of tamoxifen reduces the risk of breast cancer but has no effect on osteoporosis
C The use of tamoxifen reduces both the risk of osteoporosis and breast cancer
D
Calcium and estrogen slow the development of osteoporosis and reduce the frequency of
breast cancer

ANSWER:C
815
Which of the following is a correct statement concerning the differences in the etiology and
incidence of ischemic heart disease in men and women?
A The incidence of ischemic heart disease is higher in women than in men
B
Women are more likely to present with angina as the initial symptom of ischemic heart
disease, and men with myocardial infarction
C The risk of morbidity and mortality after a myocardial infarction is lower in women than men
D
Women have a lower perioperative mortality rate after coronary artery bypass surgery than
men
ANSWER:B
816
A 35-year-old woman pregnant with her first child develops edemA. She presents to her
obstetrician, who finds that her blood pressure is 170/115 and that she has bipedal edema
and bilateral rales on pulmonary examination. Laboratory studies reveal 6 g protein in a 24-
h urine collection, elevated hepatic transaminases, and a platelet count of 80,000/ L. The
patient is currently at 31 weeks of pregnancy and is admitted to the hospital and put on
bedrest. Her blood pressure and the status of the fetus are closely monitoreD. Which of the
following additional measures represents the most appropriate treatment?
A
Magnesium sulfate: 6-g bolus over 15 min followed by 1 to 3 g/h by continuous infusion
pump
B Intravenous labetalol
C Intravenous diazepam
D Oral losaarten
ANSWER:B
817
A 30-year-old Hispanic woman in the second trimester of pregnancy receives a 100-g oral
glucose challengE. She has elevated values of serum glucose at each of the 1-,2-, and 3-h
time points. Which of the following statements concerning this clinical situation is correct?
A A trial of caloric restriction and minimal intake of concentrated sweets should be undertaken
B The patient should be given subcutaneous insulin therapy
C The patient should be treated with oral hypoglycemic agents

D The patient should be treated with magnesium sulfate
ANSWER:A
818
A 75-year-old widower who lives alone is brought to his primary care physician by his
daughter because she feels that he has been confused over the past few days. Prior to this he
was quite independent and cognitively intact. The patient has a history of hypertension and is
on hydrochlorothiazidE. He is due for bilateral cataract extraction in 2 months. Of note, he
reports a fall in his living room about two weeks earlier, at which time he sustained a hip
bruise and bumped his heaD. At this time his general physical examination is unremarkable;
his neurologic examination is normal except for the mental status component, which
discloses defects in short-term memory. Which of the following diagnostic studies is most
likely to explain this patient’s condition?
A Neuropsychiatric battery
B CT scan of head
C Electroencephalogram
D Serum chemistry panel
ANSWER:B
819
An 80-year-old woman with severe osteoarthritis is wheelchair-bounD. She develops a
pressure ulcer on her right buttock that involves the dermis; however, subcutaneous tissues
and deeper structures do not appear to be involveD. In addition to efforts to increase
mobility and to relieve pressure, the most appropriate therapy is
A surgical debridement
B dry dressings
C wet dressings changed when dry
D wet dressings changed when wet
ANSWER:D
820
A 15-year-old boy is concerned about his lack of pubic hair, deep voice, and muscle growth.
On physical examination you confirm that his testicles have not enlarged and he lacks pubic
hair. Which of the following statements about this situation is correct?
A The patient will most likely progress through puberty without intervention
B Testosterone should be administered
C Growth hormone should be given
D A hormonally active adrenal tumor is likely

ANSWER:A
821
A 16-year-old adolescent is seen in your cliniC. On physical examination you note that he
has small testes for his stated age and has poorly developed secondary sexual characteristics.
In addition, on physical examination there is notable gynecomastiA. He is rather tall, with
abnormally long upper and lower limbs. A buccal smear is obtained and examined
microscopically. How many chromatinpositive inclusion bodies are seen?
A 0
B 1
C 2
D 3
ANSWER:B
822
A 3-month-old child arrives in your clinic who has profound hypotoniA. On physical
examination, in addition to the hypotonia, he is noted to have a brachycephalic head with a
flat occiput and a low bridge nasal structurE. The hands are short and broad, and a single
crease is noted on the fifth finger. The feet show a characteristic wide gap between the first
and second toes and the furrow is extending along the proximal plantar surface. A full
karyotype is performed and shows an abnormality. However, the total number of
chromosomes is normal at 46. What is the most likely explanation for this child’s clinical
syndrome?
A Fragile X syndrome
B Down’s syndrome
C Prader-Willi syndrome
D Cri-du-chat syndrome
ANSWER:B
823
A 45-year-old man comes to your office with a history of stage I colon cancer. The patient’s
father was diagnosed at age 49 with colon cancer, and his brother the patient’s unclE. also
had colon cancer diagnosed at age 47. The patient’s grandmother had endometrial cancer
diagnosed at age 51. This patient most likely has a defect in which of the following genes?
A p53
B APC
C MSH-2 gene affecting DNA mismatched repair

D Retinoblastoma gene RB.
ANSWER:C
824
A 35-year-old woman comes to your clinic for a consultation. She is 17 weeks pregnant with
her second chilD. She is G2 P1. Her prior pregnancy was complicated by neonatal
alloimmune thrombocytopenia NATP.. Analysis of the patient’s serum reveals circulating
anti-PIA1 antibodies. Which of the following statements concerning NATP is true?
A
If the gene frequency of PIA2 is 0.02, then the likelihood of her second child having NATP
is low
B
Given the gene frequency of PIA2 of 0.02, then the likelihood of her second child having
NATP approaches 100%
C The incidence of NATP is approximately 1/20,000 neonates
D
NATP is unrelated to the circulating anti-PIA1 antibodies because IgG antibodies do not
cross the placental barrier
ANSWER:B
825
A 35-year-old woman develops a generalized seizure and is brought to the emergency room.
Your goal is to reduce the chance for additional seizures by therapy that will achieve a
therapeutic plasma phenytoin level as soon as possiblE. However, you infuse the phenytoin
at a rate of 50 mg/min over 20 min. What is the reason for not administering the drug more
rapidly?
A Rapid administration slows drug clearance
B High blood levels during a rapid load could produce drug toxicity
C The equilibrium phase would be reached too quickly
D High levels can exacerbate the seizures
ANSWER:B
826
A 75-year-old patient with advanced metastatic prostate cancer and a long history of renal
failure has severe bone pain. He is given meperidinE. Two days later he develops a
generalized seizurE. What is the likely mechanism of this complication?
A Buildup of meperidine
B Buildup of meperidine metabolite levels
C Brain metastases
D Hypercalcemia
ANSWER:B

827
A 72-year-old man with chronic atrial fibrillation presents with bruising. He is otherwise
well, although he admits to being concerned about a failing memory and is taking a host of
over-the-counter remedies in an effort to deal with this problem. His only medicines are
furosemide, digoxin, and warfarin. His physical examination, with the exception of
ecchymoses and irregular heartbeat, is normal; there is no evidence of cognitive impairment.
His serum chemistry profile and complete blood count are normal. However, coagulation
tests reveal an INRof 7. What is the most likely cause of this patient’s coagulopathy?
A Warfarin overdose
B Interaction between digoxin and warfarin
C Use of dietary herbal supplements
D Interaction between furosemide and warfarin
ANSWER:C
828
A 55-year-old man with a history of seasonal allergic rhinitis develops a low-grade fever and
cough. He complains to his physician that he is producing copious amounts of greenish
sputum and coughing quite a bit. Since the patient is known to be allergic to penicillin, the
physician prescribes erythromycin. The patient is also taking terfenadine because of his
allergic symptoms. Which of the following is a potential complication in this clinical
scenario?
A Decreased bioavailability of erythromycin with failure to treat pulmonary infection
B Congestive heart failure
C Increased bleeding
D Stevens-Johnson syndrome
ANSWER: B
829
A 25-year-old man who was recently admitted to a psychiatric hospital with the diagnosis of
severe depression complicated by psychosis is brought to the emergency room because of
worsening mental status and fever. The patient is unable to give a history because he is
profoundly confused and claims to be on Mars. The psychiatrist informs you that the patient
has been started recently on haloperidol and amitriptylinE. Physical findings include a rectal
temperature of 40.6 C 105 F., muscle rigidity, and dry skin. A cooling blanket is ordered,
and you administer acetaminophen. Which of the following agents would be most
appropriately ordered at this time?

A Bromocriptine
B Atropine
C Levarterenol
D Chlorpheniramine
ANSWER:A
830
A 23-year-old woman presents for a routine physical examination. The patient gives a
history of trying to diet, but she also admits to binge intake and extensive use of laxatives.
The physical examination discloses a woman of normal height and weight and is
unremarkable except for chipping and erosion of the front teeth. This patient is likely to
A have disrupted menstrual cycles
B experience a normal life span
C have osteopenic bones
D have elevated serum anti-goblet cell antibodies
ANSWER:B
831
A strain of obese mice homozygous for a mutation in the ob gene has a failure to elaborate
normal amounts of the peptide leptin. Which of the following statements about leptin is
correct?
A Leptin is secreted by cells in the hypothalamus
B Overproduction causes increased energy expenditurE.
C Patients with nongenetic causes of obesity do not respond to exogenous leptin.
D Obese people have normal levels of serum leptin.
ANSWER:C
832
A 32-year-old obese woman presents to you for advice regarding weight loss. She has no
significant medical problems at this timE. She requests a pill to help her lose weight. You
should
A tell her that a low-calorie diet and exercise are the best ways to lose weight
B prescribe phentermine
C prescribe fenfluramine
D prescribe a combination of phenylpropanolamine plus fenfluramine
ANSWER:A

833
You are evaluating a 45-year-old homeless individual with known chronic schizophreniA.
He appears to be generally malnourished with decreased temporal and proximal extremity
muscle mass. Which of the following immunologic/hematologic abnormalities is most likely
to be found in this situation?
A Depressed absolute lymphocyte count
B Depressed absolute monocyte count
C Depressed absolute granulocyte count
D Hypogammaglobulinema
ANSWER:A
834
Why do patients with the carcinoid syndrome develop pellagra loss of appetite, generalized
weakness, abdominal pain, vomiting, stomatis and bright red glossitis, and a skin rash that is
pigmented and scaling in sunlightexposed areas.?
A Failure to elaborate serotonin
B Vitamin B deficiency 12
C Thiamine deficiency
D Niacin deficiency
ANSWER:D
835
A 23-year-old individual who believes in megavitamin therapy develops severe headaches.
Ophthalmologic examination discloses papilledemA. The cause for this problem is
A vitamin A intoxication
B vitamin B intoxication 6
C vitamin D intoxication
D vitamin E intoxication
ANSWER:A
836
A 65-year-old woman complains of a 20-lb weight loss over the past 6 months. She is of
average build and has not been on a diet. She states that she has just not been very hungry.
Physical examination is unrevealing. Complete blood count is normal, as is urinalysis,
multiphased chemical screen, thyroid-stimulating hormone, chest x-ray, and stool for blooD.
The next most appropriate study would be
A abdominal CT
B serum protein electrophoresis

C colonoscopy
D blood cultures
ANSWER:A
837
A 45-year-old truck driver is involved in a serious motor vehicle accident. He has fractured
multiple bones and sustained a pneumothorax and a hepatic laceration requiring emergency
surgery. In order to maintain energy balance, what must this patient receive per day?
A 2500 kcal
B 3500 kcal
C 4500 kcal
D 5500 kcal
ANSWER:D
838
A 50-year-old man with recent acute pancreatitis receiving his third week of nutritional
support via a central venous catheter might be expected to develop which of the following
complications at this time?
A Zinc deficiency
B Osteomalacia
C Microcytic anemia
D Hyperosmolar coma
ANSWER:D
839
A 70-year-old man of Irish descent returns to his physician for a routine check of his blood
pressurE. He is a vigorous, retired executive who except for mild hypertension is healthy.
After his examination, as he is getting dressed, he states that his wife has been nagging him to
mention a spot on his nose as shown in Plate P.. He is certain that this lesion, which has been
present for several years, is of no significancE. The most likely diagnosis for this lesion is
A dermal nevus
B sebaceous hyperplasia
C clear cell acanthoma
D basal cell carcinoma
ANSWER:D

840
A 52-year-old woman sees her physician for an “insurance physical.” Physical examination
reveals only a pigmented lesion as shown in Plate Q. present on one foot. The woman states
that the lesion apparently was present at birth and does not itch or bleed; it is, however, not
as homogeneous in color as it used to b E. Which of the following statements about the
condition described is true?
A Bleeding and tenderness would be the first signs of malignant degeneration.
B It is unlikely that the lesion, present since birth, is malignant.
C The diagnostic procedure of choice is an incisional biopsy of this lesion.
D Change in color of the lesion is a suspicious sign for potential malignancy.
ANSWER:D
841
A 58-year-old man presents with fatiguE. His physical examination is normal except for the
presence of splenomegaly. CBC discloses hematocrit, 29%; platelet count, 90,000/ L; WBC,
2700/ L; and an essentially normal red cell morphology differential 12% monocytes, 12%
granulocytes, and 76% lymphocytes.. A bone marrow aspirate and biopsy were performeD.
The aspirate was dry and the biopsy is pending. Based on the available information, the most
likely diagnosis in this case is
A chronic lymphocytic leukemia CLL.
B hairy cell leukemia
C chronic myeloid leukemia CML.
D myelofibrosis
ANSWER:B
842
A 58-year-old chronic alcoholic and heavy smoker presents with a 3-cm, firm, right
midcervical neck mass. An excisional biopsy reveals squamous cell carcinomA. Which of
the following is the most appropriate approach at this time?
A Bronchoscopy, esophagoscopy, and laryngoscopy
B CT of the neck
C CT of the brain
D Neck dissection
ANSWER:A

843
A 55-year-old woman presents to the emergency department because her family notes that
she has yellow skin. The patient has lost 7 kg 15 lbs. over the past 3 months but states that
this is because she has been dieting in preparation for her daughter’s wedding. Her past
medical history is significant only for vitiligo. Her physical examination is unremarkable
except for the presence of scleral icterus and a yellow tinge to the skin. Laboratory
evaluation reveals hematocrit of 17%, WBC count of 2500/ L, and platelet count of 70,000/
L. Serum chemistries are normal except for direct bilirubin of 51 mol/L 3 mg/dL. and
indirect bilirubin of 12 mol/L 0.7 mg/dL.. The patient’s reticulocyte count is 3%. MCV is
108 fL. Which one of the following additional laboratory findings would most likely be
associated with this patient’s clinical syndrome?
A Clonal chromosomal abnormalities on karyotypic analysis of the bone marrow
B Antiparietal cell antibody
C Extrahepatic biliary obstruction
D Decreased gastric fluid pH
ANSWER:B
844
A 27-year-old woman presents with stage II breast and lymph node involvement. right
breast cancer. Her family history is markedly positive for other tumors. One of her sisters
developed an osteogenic sarcoma at age 17, her brother was diagnosed with acute leukemia
at age 5, her mother died of breast cancer, and she has two uncles with soft-tissue sarcomas,
both developing this disease when in their thirties. This patient’s peripheral blood
lymphocytes would be most likely to reveal which of the following abnormalities?
A Retinoblastoma gene mutation
B p53 gene mutation
C Translocation between chromosomes 9 and 22
D Translocation between chromosomes 8 and 14
ANSWER:B

845
A patient with a myelodysplastic syndrome subtype, refractory anemia with ringed
sideroblasts. has been transfusion-dependent for the past 2 years. The patient has received a
total of 50 units of packed red blood cells. His physical examination is normal except for
hyperpigmentation. Laboratory evaluation reveals mild glucose intolerancE. A trial of
erythropoietin was unsuccessful. Which of the following would be the most important
therapeutic approach at this time?
A Granulocyte colony-stimulating factor G-CSF.
B Phlebotomy
C Ascorbic acid
D Desferrioxamine
ANSWER:D
846
A 26-year-old woman has painful mouth ulcers. Six weeks ago, she was started on
propylthiouracil for hyperthyroidism. She is afebrile, and physical examination is
unremarkable except for several small oral aphthous ulcers. White blood cell count is 200/ L
15% neutrophils, 80% lymphocytes, 5% monocytes.; hemoglobin concentration, hematocrit,
and platelet count are normal. The woman’s physician should stop the prophylthiouracil and
A schedule a follow-up outpatient appointment
B arrange for HLA typing of her siblings in preparation for bone marrow transplantation
C prescribe oral prednisone, 1 mg/kg
D hospitalize her for broad-spectrum antibiotic therapy
ANSWER:A
847
A 30-year-old black woman with long-standing sickle cell anemia presents with severe pain
in the chest and abdomen 1 week after having an upper respiratory infection. No
intrathoracic or intraabdominal pathology was immediately obvious on routine physical
examination and laboratory evaluation. The most appropriate therapeutic intervention at this
point is
A hypertransfusion
B hydration and narcotic analgesia
C hydroxyurea
D broad-spectrum antibiotics
ANSWER:B

848
A 25-year-old, previously healthy woman presents with jaundice, confusion, and fever.
Initial physical examination is unremarkable except for scattered petechiae on the lower
extremities, scleral icterus, and disorientation on mental status examination. Laboratory
examination discloses the following: hematocrit, 27%; white cell count, 12,000/ L; platelet
count, 10,000/ L; bilirubin, 85 mol/ L 5 mg/dL.; direct bilirubin, 10 mol/L 0.6 mg/dL.;
urea nitrogen, 21 mmol/L 60 mg/dL.; creatinine,400 mol/L 4.5 mg/dL.. Red blood cell
smear discloses fragmented red blood cells and nucleated red blood cells. Prothrombin,
thrombin, and partial thromboplastin times are all normal. The most effective and
appropriate therapeutic maneuver is likely to be
A plasmapheresis
B administration of aspirin
C administration of high-dose glucocorticoids
D administration of high-dose glucocorticoids plus cyclophosphamide
ANSWER:A
849
A 72-year-old man with known benign prostatic hypertrophy develops fever and flank pain.
He rapidly becomes very ill. He presents to the emergency room with a blood pressure of
80/40 mmHg, heart rate of 120, and a temperature of 39.5 C 103 F.. His urine shows
numerous white cells. His laboratory examination is remarkable for a white count of 2000,
hematocrit of 28%, and platelet count of 10,000. The PT and PTT are elevateD. The most
appropriate way to deal with this patient’s coagulopathy is
A intravenous -aminocaproic acid
B intravenous antibiotics
C platelets and fresh-frozen plasma
D antithrombin 3 concentrates
ANSWER:B
850
A 65-year-old woman on hemodialysis for chronic renal failure requires an urgent dental
extraction for an abscessed tooth. Of the following, the most appropriate agent to administer
to reduce the risk of significant bleeding would be
A desmopressin
B conjugated estrogen
C erythropoietin
D fresh-frozen plasma

ANSWER:A
851
A 36-year-old woman presents with a firm painless mass in her right thigh just superior to
her kneE. A CT scan reveals a 4- by 5-cm solid mass attached to the musclE. You refer the
patient to a surgeon, who performs an incisional biopsy. The pathology indicates high-grade
fibrosarcoma with several mitoses per 10 high-power fields. The most appropriate
management at this point is
A complete excision of the mass
B radiation therapy
C chemotherapy with a doxorubicin-containing regimen
D radiation therapy plus chemotherapy with a doxorubicin containing regimen
ANSWER:A
852
A 35-year-old woman develops hirsutism, deepening voice, and clitorimegaly. A pelvic
examination reveals a left ovarian mass. Assuming appropriate diagnostic and staging tests
are performed, given this clinical presentation, if the patient requires chemotherapy she
should be treated in a fashion analogous to the management of
A epithelial ovarian cancer
B lymphoma
C testicular cancer
D soft tissue sarcoma
ANSWER:C
853
A 65-year-old woman presents with severe pelvic pain with radiation down both legs. There
is no evidence of a sensory level. Physical examination is unremarkable except for a
colostomy in the right lower quadrant. There is a history of an abdominal-perineal resection
for rectal cancer 3 years ago. Postoperatively she received pelvic irradiation and adjuvant
chemotherapy with 5-FU. The most likely cause for this patient’s pain is
A pelvic recurrence of rectal cancer
B bony metastasis
C secondary leukemia with bone pain
D post-radiation radiculitis
ANSWER:A

854
A 52-year-old man noted a pigmented lesion in the area of his left flank. Excisional biopsy
revealed a malignant melanoma 2.5 mm in thickness. The patient then underwent a definitive
resection of the tumor with 2-cm margins that were not involveD. Chest x-ray and liver
function tests are normal. The patient should receive
A interferon-
B interleukin-2
C dacarbazine
D dacarbazine plus carmustine
ANSWER:A
855
A 43-year-old man presents with severe mid-thoracic back pain. His past medical history is
remarkable for the removal of a malignant melanoma depth 1.5 mm. approximately 3 years
ago. The patient’s back pain is severe and has been waking him up at night over the past
week. Physical examination is unremarkablE. Plain films of the spine reveal loss of the left
pedicle in the fifth thoracic vertebrA. Magnetic resonance images are obtained, and the
patient is begun on steroids. Which of the following treatment modalities is most appropriate
in this situation?
A Surgery
B Radiation therapy
C Chemotherapy
D Hormonal therapy
ANSWER:A
856
A 65-year-old man develops superficial thrombophlebitis in multiple sites including the arms
and chest. He has had several episodes in the past couple of months, each of which lasted a
few days. Which of the following neoplasms is most closely associated with this patient’s
clinical problem?
A Prostate carcinoma
B Lung carcinoma
C Pancreatic carcinoma
D Acute promyelocytic leukemia
ANSWER:C

857
A 40-year-old woman undergoes her first mammogram. The study reveals a cluster of
microcalcifications in the right breast. Needle biopsy reveals a focus of lobular carcinoma in
situ no invasion.. At this point the patient should be offered
A quadrantectomy and lymph node dissection on the ipsilateral side
B quadrantectomy with irradiation
C
right breast mastectomy with irradiation depending on lymph node status at the time of
surgery
D resection followed by annual mammography and semiannual physical exam
ANSWER:D
858
A 55-year-old man complains of numbness in both legs and progressive inability to walk
over the past 2 months. Physical examination is normal except for a decreased perception of
light touch and pain in the lower extremities as well as bilateral leg weakness. There is no
sensory level. Laboratory workup is remarkable for a hematocrit of 30% and elevated total
protein. Serum protein electrophoresis reveals an M spikE. The etiology of this patient’s
weakness is most likely
A necrosis of central nervous system gray and white matter
B inflammation of dorsal root ganglia
C tumor-elaborated immunoglobulin that is reacting with myelin components
D
elaboration of tumor-associated protein that elicits an immune response that is cross-reactive
with peripheral nerves
ANSWER:C
859
A 55-year-old man who recently underwent hip sur- V-92. ContinueD. gery develops a
pulmonary embolism and is placed on unfractionated heparin. After 3 days of therapy, his
platelet count previously normal. is now 50,000/ l. What is the mechanism of the
thrombocytopenia in this case?
A The induction of an anti-platelet antibody
B Platelet-heparin binding causing platelet aggregation
C Platelet aggregation due to the formation of a heparin platelet factor IV antibody complex
D Heparin-induced bone marrow suppression
ANSWER:C

860
A 55-year-old Japanese businessman visiting the United States has been in excellent health
until 6 months ago, when he first noted mild upper abdominal fullness after meals. On
examination the man is noted to have hyperpigmented, heaped-up velvety lesions as shown
in Plate S. confined to the neck, axillae, and groin. Which of the following conditions is
associated with the skin findings?
A Non-Hodgkin’s lymphoma
B Anorexia nervosa
C Acute leukemia
D Adenocarcinoma of the stomach
ANSWER:D
861
A 76-year-old man has developed over the past 3 months a large number of occasionally
pruritic lesions on his trunk Plate T.. These lesions are probably a manifestation of
A hypertriglyceridemia
B Malignancy
C severe drug reaction
D disseminated candidiasis
ANSWER:B
862
A 21-year-old woman with relapsed acute lymphoblastic leukemia is treated with a five-drug
induction regimen cyclophosphamide, daunorubicin, vincristine, prednisone, and L-
asparaginasE. . On the sixth day after the initiation of this therapy the patient develops a
fever and is started on intravenous ceftazidimE. The patient defervesces but develops
another fever 5 days later and is started on amphotericin Ten days later the patient, still on
oral steroids, remains febrile, neutropenic, and thrombocytopenic and is noted to have
shortness of breath. Chest x-rays show a densely consolidated pulmonary infiltrate in the left
lung zonE. A sputum culture demonstrates normal oral flora and several colonies of
Aspergillus. The most appropriate conclusion to draw is that
A the patient most likely has invasive pulmonary aspergilliosis
B the Aspergillus is a contaminant; the patient most likely has bacterial pneumonia
C biopsy is not required for a definitive diagnosis
D the patient most likely has viral pneumonitis
ANSWER:A

863
A 28-year-old Egyptian farmer presents with left flank pain. Ultrasonography reveals
enlargement of the left ureter and hydronephrosis of the left kidney. Cystoscopy reveals a
mass extending from the left ureter into the bladder. Parasitic ova 150 by 50 mm. are noted
in the urine and in a biopsy of the ureteral mass. Which of the following statements is correct?
A Renal failure is likely in the absence of treatment.
B The lesion is not reversible by chemotherapy.
C
In the absence of treatment, the patient has an increased risk for transitional cell carcinoma of
the bladder.
D The patient is suffering from schistosomiasis.
ANSWER:D
864
A 35-year-old patient undergoing initial therapy for acute myeloid leukemia has tolerated
the chemotherapy well. However, 6 days after the initiation of chemotherapy and 10 days
after the insertion of an indwelling transthoracic intravenous device Hickman catheter., he
develops a fever. Examination is negative except for erythema and tenderness at the insertion
site and along the subcutaneous tunnel. Blood cultures and chest x-ray are negativE. The
most appropriate course of action at this point is to
A remove the line and insert a new one over a guidewire
B begin intravenous vancomycin
C begin intravenous vancomycin and gentamicin
D remove the line
ANSWER:D
865
A 70-year-old man with a history of heavy smoking and moderately severe chronic
obstructive pulmonary disease COPD. has been feeling poorly. He reports cough, chills,
pleuritic chest pain, and low-grade fever. Chest xray reveals a small dense infiltrate in the
right lower lobE. Gram’s stain of the patient’s sputum reveals numerous gram-negative
cocci, many of which occur in pairs. The most appropriate therapy would be
A penicillin/clavulanic acid
B tetracycline
C ciprofloxacin
D trimethoprim/sulfamethoxazole TMP/SMZ.
ANSWER:A

866
A 35-year-old man with a history of abrasion of the right hand presents with acute pain in
the right shoulder. His physical examination reveals a temperature of 39.5 C 103 F. and
rigor, and he appears to be quite ill. There is dusky erythema and edema of the right
shoulder and right upper extremity with marked tenderness. Within a few hours the patient is
unresponsive and is found to be hypotensivE. Laboratory evaluation reveals an elevated
serum, creatinine, thrombocytopenia, and elevated hepatic transaminases. The soft tissues in
the left upper extremity have begun to necrosE. Blood culture, obtained at the time of initial
presentation, has already turned positivE. The organism that is most likely to be responsible
for this clinical syndrome is
A group A streptococci
B group D streptococci
C Staphylococcus aureus
D Bacteroides fragilis
ANSWER:A
867
A 53-year-old black man who received a renal allograft 7 months ago is now receiving
azathioprine and prednisonE. He presents to the hospital 1 week after developing fever,
night sweats, and anorexiA. He also complains of coughing and chest pain. Chest film
reveals biapical infiltrates with an apparent cavity in the left upper lobE. Auramine-
rhodamine staining reveals the presence of microorganisms consistent with tubercle bacilli.
The patient’s creatinine is 106 mol/L 1.2 mg/dL.. The treatment of choice at this time would
be
A isoniazid, rifampin, and pyrazinamide
B isoniazid, rifampin, pyrazinamide, and ethambutol
C isoniazid and rifampin
D rifampin, pyrazinamide, and ethambutol
ANSWER:B
868
A 25-year-old intravenous drug abuser with fever has blood cultures obtained, and 24 h later
a report from the microbiology laboratory indicates the presence of gram-positive cocci in
clusters. The identification of the organism and sensitivities are pending. The most
appropriate antibiotic choice would be
A penicillin
B nafcillin

C vancomycin
D TMP/SMZ
ANSWER:C
869
Four months after having undergone a siblingdonor renal allograft, a 38-year-old man is has
done well and has had no evidence of graft rejection or major problems stemming from his
chronic immunosuppressive therapy cyclosporine and prednisonE. . He now develops a
fever to 39 C 102 F., headache, and a stiff neck. MRI of the brain with gadolinium
enhancement reveals no abnormalities.The most likely cause of the patient’s current clinical
problem is infection with
A Listeria monocytogenes
B Mycobacterium tuberculosis
C Toxoplasma gondi
D H. influenzae
ANSWER:A
870
A 12-year-old girl presents with painful epitrochlear lymphadenopathy associated with low-
grade fever and malaisE. The patient has a cat and also gave a history of a papillary lesion in
the left forearm about 1 week or 10 days ago. The most likely etiologic agent in this situation
is
A Bartonella henselae
B Staphylococcus aureus
C Epstein-Barr virus
D Sporothrix schenkii
ANSWER:A
871
A 73-year-old previously healthy man is hospitalized because of the acute onset of dysuria,
urinary frequency, fever, and shaking chills. His temperature is 39.5 C 103.1 F., blood
pressure is 100/60 mmHg, pulse is 140 beats per minute, and respiratory rate is 30 breaths
per minutE. Which of the following interventions would be the most important in the
treatment of this acute illness?
A Catheterization of the urinary bladder
B Initiation of antibiotic therapy
C Infusion of Ringer’s lactate solution
D Infusion of dopamine hydrochloride

ANSWER:B
872
A 65-year-old Greek woman visiting her children in New York City complains of upper
abdominal pain. The patient is brought to the family physician, who notices icteric sclera and
a mass in the right upper quadrant. CT reveals a 10-cm multiloculated cyst with mural
calcification that is compressing the common bile duct. Which of the following statements is
correct concerning this clinical situation?
A Treatment with the antiamebic agent chloroquine is indicateD.
B Treatment with an antiechinococcal agent such as albendazole is sufficient.
C The adult parasite resides in the patient’s intestinE.
D Infection was probably caused by exposure to infected dogs.
ANSWER:D
873
A 60-year-old man from North Carolina presents with a fever and progressive confusion. His
wife reports that he experienced 1 week of fever, headache, and malaise before the more
profound neurologic condition occurreD. Just prior to the presentation at the hospital he had
a generalized seizurE. The IgM capture enzyme immunoassay of the patient’s CSF was
positive for the eastern equine encephalitis virus. An MRI would most likely reveal
A normal findings
B meningeal enhancement
C hydrocephalus
D lesions in the basal ganglia
ANSWER:D
874
A 50-year-old woman emigrated from El Salvador 10 years ago and currently resides in
Washington, DC. She complains of shortness of breath. Chest x-ray reveals biventricular
cardiac enlargement. An echocardiographic study shows biventricular enlargement, thin
ventricular walls, and an apical aneurysm. The patient has no history of alcohol abuse,
thyroid disease, risk factors for atherosclerotic heart disease, or family history of
hemochromatosis. In considering a potential etiology for the patient’s current problem,
which of the following statements is correct?
A The etiologic agent can be demonstrated on Giemsa stain of the peripheral blooD.
B Other manifestations of infection could include involvement of the gastrointestinal tract.
C The vector for the transmission of this disease is the tsetse fly.

D Glucocorticoids may be beneficial.
ANSWER:B
875
A 60-year-old insulin-dependent man with diabetes mellitus has had purulent drainage from
his left ear for 1 week. Suddenly, fever, increased pain, and vertigo develop. The most likely
causative agent is
A Aspergillus
B Mucor
C Pseudomonas
D Staphylococcus aureus
ANSWER:C
876
A 35-year-old man is seen 6 months after a cadaveric renal allograft. The patient has been on
azathioprine and prednisone since that procedurE. He has felt poorly for the past week with
fever to 38.6 C 101.5 F., anorexia, and a cough productive of thick sputum. Chest x-ray
reveals a left lower lobe 5 cm. nodule with central cavitation. Examination of the sputum
reveals long, crooked, branching, beaded gram-positive filaments. The most appropriate
initial therapy would include the administration of which of the following antibiotics?
A Penicillin
B Erythromycin
C Sulfisoxazole
D Ceftazidime
ANSWER:C

877
A previously healthy 28-year-old man describes several episodes of fever, myalgia, and
headache that have been followed by abdominal pain and diarrheA. He has experienced up
to 10 bowel movements per day. Physical examination is unremarkablE. Laboratory
findings are notable only for a slightly elevated leukocyte count and an elevated erythrocyte
sedimentation ratE. Wright’s stain of a fecal sample reveals the presence of neutrophils.
Colonoscopy reveals is inflamed mucosA. Biopsy of an affected area discloses mucosal
infiltration with neutrophils, monocytes, and eosinophils; epithelial damage, including loss of
mucus; glandular degeneration; and crypt abscesses. The patient notes that several months
ago he was at a church barbecue where several people contracted a diarrhea1 illness. While
this patient could have inflammatory bowel disease, which of the following pathogens is
most likely to be responsible for his illness?
A Campylobacter
B S. aureus
C E. coli
D Salmonella
ANSWER:A
878
A 62-year-old gardener who has chronic lymphocytic leukemia develops lymphangitis and a
painless, nodular lesion on his wrist. Subsequently, he becomes severely ill with cavitary
right-upper-lobe pneumonia; Sporothrix schenckii is isolateD. He should be treated with
A chloramphenicol
B potassium iodide
C penicillin
D amphotericin B
ANSWER:D
879
A 22-year-old female intravenous drug abuser and admitted prostitute is seen in the
emergency room with a complaint of vaginal dischargE. She is afebrile and has no
complaint or physical findings except that she has yellow mucopurulent discharge emanating
from the cervical os. Gram’s stain of the discharge reveals polymorphonuclear leukocytes,
but organisms are not seen. The patient has no primary care doctor. The most appropriate
action at this time is to

A
reat for both chlamydial and gonorrheal infection with cefixime, 400 mg PO, plus
azithromycin, 1 g PO
B
schedule the patient for colposcopy to cauterize the endocervical cells that have migrated
onto the visible ectocervix
C treat for chlamydial infection with doxycycline, 100 mg PO bid for 7 days
D treat for gonorrheal infection with ciprofloxacin, 500 mg PO
ANSWER:A
880
Four days after he and his friends were killing muskrats along a rural creek, a boy becomes
ill with headache, fever, and a macular rash. On examination, axillary adenopathy is noted,
but otherwise the examination is normal. Which of the following tests would be most helpful
in proving that this boy has tularemia?
A Blood culture
B Aspiration and culture of an axillary lymph node
C Determination of serum agglutinins for Francisella tularensis
D Bone marrow culture
ANSWER:B
881
A 10-year-old boy is seen in a rural Arizona clinic because of prostration, fever of 40 C 104
F., and severe headachE. Examination is negative for rash, stiff neck, joint tenderness, and
chest and abdominal abnormalities. However, several tender, enlarged lymph nodes are
palpated in the left axilla, which is very edematous. The test most likely to be of greatest help
in the immediate management of this boy would be
A blood culture
B examination of a blood smear
C biopsy of an axillary lymph node
D aspiration and Gram stains of an axillary lymph node
ANSWER:D

882
A 10-year-old boy presents with an abnormal appearing face. The boy lives in Rhode Island
and has been playing outside a good deal this summer. He has been feeling poorly for a
week with complaints of muscle aches and headachE. His mother has noticed that her son
has a low-grade fever and an oval rash on the back measuring about 10 cm in diameter.
Physical examination reveals evidence of the oral erythema on the posterior thorax and
evidence of right facial droop. Routine laboratory studies are unremarkablE. A lumbar
puncture reveals an opening pressure of 80 mmHg, total protein of 0.46 g/L 46 mg/dL., and
glucose of 5.0 mmol/L 90 mg/dL. with 10 white cells, all of which are lymphocytes. The
most specific diagnostic study would be
A polymerase chain reaction– based DNA detection
B Borrelia serology
C blood culture for Borrelia
D cerebrospinal fluid culture for Borrelia
ANSWER:C
883
A 22-year-old gay man from New Orleans presents with a 2-week history of fever, anorexia,
and progressive diffuse lymphadenopathy. Physical findings reveal an emaciated young man
who has several tongue ulcers. Hepatomegaly is noteD. Laboratory examination reveals
pancytopenia, an elevated alkaline phosphatase, and hyperkalemiA. A chest radiograph
reveals a miliary pattern of diffuse infiltration. A tongue biopsy reveals the presence of
hyphae that bear both large and small spores. The correct diagnosis is
A histoplasmosis
B coccidioidomycosis
C cryptococcosis
D blastomycosis
ANSWER:A
884
A 40-year-old Canadian who operates a tropical fish store sees his physician because of a
nonhealing ulcer on his left arm. He is afebrile and gives no history of night sweats, weight
loss, or other constitutional symptoms. Biopsy of the lesion shows granulomatous
inflammation and rare acid-fast organisms. A tuberculin test is negativE. This man most
likely has an infection caused by
A Mycobacterium tuberculosis

B M. ulcerans
C M. kansasii
D M. marinum
ANSWER:D
885
A 35-year-old HIV-infected homosexual man presents with fever, pain of the right upper
quadrant, and a CT of the liver that shows a 10-cm, oval hypoechoic cyst in the right lobE.
An ELISA assay detects the presence of antibodies to Entamoeba histolytica; cysts from the
same organism are found in a stool specimen. Which of the following is the most appropriate
next step in management?
A Administration of metronidazole
B Administration of chloroquine
C Drainage of the hepatic lesion for therapeutic purposes
D Aspiration of the hepatic lesion for diagnosis
ANSWER:A
886
A 38-year-old woman has undergone allogeneic bone marrow transplantation for acute
myelogenous leukemia in second remission from a matched unrelated donor; she now
develops pneumonia proven to be due to cytomegalovirus. The patient is placed on
ganciclovir, but the pneumonia progresses. Which of the following drugs may potentially be
effective in this situation?
A Acyclovir
B IFN- -2
C Almuvidine
D Foscarnet
ANSWER:D
887
A 23-year-old woman who will be doing Peace Corps work in Bolivia has had all her routine
immunizations and has been vaccinated against hepatitis A and Which of the following is the
most reasonable recommendation for prophylaxis against common infectious diseases in the
country of her destination?
A Chloroquine
B Yellow fever vaccine plus mefloquine
C Yellow fever vaccine
D Yellow fever vaccine plus chloroquine

ANSWER:B
888
A 35-year-old Samoan presents with recurrent fever, headache, photophobia, and painful
lymphangitis in the left leg. The best way to diagnose filariasis caused by Wuchereria
bancrofti is
A biopsy of any inflamed lymph nodes to demonstrate the adult worm
B demonstration of microfilariae in blood taken between 9 P.M. and 2 A. M
C observation of intense itching after a single dose of diethylcarbamazine
D demonstration of microfilariae after injection of blood into mice
ANSWER:B
889
An 18-year-old sexually active woman from the innercity presents with fever, pleuritic pain
of the right upper quadrant, and lower abdominal pain. Pelvic examination reveals
mucopurulent cervicitis and tenderness after the production of cervical motion. The right
upper quadrant, uterine fundus, and adnexa are slightly tender. The white blood cell count
and erythrocyte sedimentation rate are elevated, but the results of the remainder of the
laboratory examination, including liver function tests, are normal. Which of the following
agents is the most likely cause of this clinical syndrome?
A Herpes simplex virus
B Treponema pallidum
C Neisseria gonorrhoeae
D Chlamydia trachomatis
ANSWER:C
890
A 65-year-old retired banker who spends the summer on Nantucket Island off the
Massachusetts coast returned to his home in Boston early in September. He noted the gradual
onset of a febrile illness with chills, sweats, myalgias, and yellow eyes. His doctor palpated
the spleen and noted a macrocytic anemia, hyperbilirubinemia, and a high serum level of
lactic dehydrogenase on laboratory examination. Which of the following would be the most
helpful diagnostic procedure at this point?
A Blood culture
B Examination of leukocytes on blood film
C Examination of erythrocytes on blood film
D Splenic biopsy
ANSWER:C

891
A 35-year-old intravenous drug abuser with HIV infection is being managed with
combination antiretroviral therapy. The patient was doing well on his current medical
regimen, which consists of lamivudine and saquinavir as well as methodone, TMP/SMZ, and
fluconazolE. Although he has been stable clinically of late, efavirenz has recently been
added to his medical regimen in an attempt to decrease a rising viral loaD. After 1 week of
therapy with efavirenz, the patient develops abdominal cramps, malaise, sweats, and anxiety.
The most likely reason for the patient’s symptoms is
A reduced plasma methadone concentration.
B increased fluconazole levels
C infection with Pneumocystis due to decreased TMP/SMZ levels
D lamivudine toxicity secondary to decreased albumin binding
ANSWER:A
892
A 42-year-old man underwent allogeneic bone marrow transplantation from an unrelated
donor for acute myeloid leukemia in second remission. He develops hepatomegaly and fever
3 months after the infusion of the allogeneic cells. Radiographic imaging of the abdomen
reveals extensive intraabdominal lymphadenopathy as well as an enlarged liver, which
appears to be diffusely infiltrateD. There is no ascites noteD. Which of the following is the
likely mechanism for the patient’s current deterioration?
A Venoocclusive disease of the liver
B Overgrowth of EBV-infected lymphocytes
C Recurrence of acute myeloid leukemia
D Graft-versus-host disease
ANSWER:B
893
A 43-year-old nurse whose job requires frequent hand washing has noted a small erosive
skin lesion between the third interdigital web of the right hand Plate A. . The best therapy
for this condition would be
A topical 5-fluorouracil
B topical clotrimazole
C oral griseofulvin
D topical hydrocortisone
ANSWER:B

894
For the past 2 days, a 24-year-old woman has had fever and pain in the left wrist, right
ankle, and left kneE. Nine painful skin lesions are present on the distal extremities,
predominantly about the joints as shown in Plate B. . The most likely diagnosis is
A herpes simplex
B meningococcemia
C gonococcemia
D erythema multiforme
ANSWER:C
895
A 26-year-old man from Cape Cod sees his physician because of a 3-week history of an
expanding, slightly burning ring of redness as shown in Plate C. that first surrounded a red
papule on the posterior neck. He complains of headaches, generalized muscle aches,
anorexia, and malaisE. On examination, he is noted to be febrile [38.3 C 101 F.]; his rash is
slightly raised and slightly tender and displays central clearing but no scaling, even after
vigorous scraping. Which of the following vectors has been strongly associated with the type
of rash described above?
A Kissing bug
B Spider
C Flea
D Tick
ANSWER:D
896
A 24-year-old man is concerned because of the appearance of several light brown spots on
his trunk Plate D. The lesions limited to the chest, back, abdomen, and upper arms. are flat
and sharply marginated and have a fine scale that is easily scraped off. The most appropriate
diagnostic study is
A Giemsa stain of scraped material Tzanck preparation.
B bacterial culture of the lesions
C fungal culture of the lesions
D microscopic examination of potassium hydroxide treated scrapings
ANSWER:D

897
A 67-year-old man presents with a history of headache for 5 days and 2 days of swelling of
the right part of the forehead and right eye see Plate E. . A Tzanck preparation of the lesion
reveals multinucleate giant cells on Giemsa stain. The patient was admitted to the hospital and
begun on intravenous acyclovir. The most important next step would be
A ophthalmologic consultation
B administration of systemic glucocorticoids to prevent postherpetic neuralgia
C C. administration of antistaphylococcal antibiotics to prevent secondary bacterial infection
D application of iodine-containing solution to prevent secondary bacterial infections
ANSWER:A
898
A 25-year-old homosexual man presents with a diffuse maculopapular rash over his trunk,
head, neck, palms, and soles. Generalized lymphadenopathy is also present. He has a history
of 4 weeks of anal pain. Which of the following tests is likely to identify the etiologic agent?
A Antinuclear antibody
B Blood culture
C Serum rapid plasma reagin RPR.
D Skin biopsy
ANSWER:C
899
An 18-year-old man pictured in Plate F. presents because of unsightly facial inflammation.
Which of the following statements is correct?
A
Closed comedones whiteheads. are less commonly associated with the inflammatory lesions
than are open comedones blackheads.
B
Glucocorticoids, although not indicated except in the most severe cases, would likely result
in improvement.
C Patients on systemic retinoic acid may experience very dry skin and hypertriglyceridemia.
D Systemic antibiotic therapy is unlikely to be helpful.
ANSWER:C
900
A 59-year-old man presents to the emergency department ED. complaining of new onset
chest pain that radiates to his left arm. He has a history of hypertension,
hypercholesterolemia, and a 20-pack-year smoking history. His electrocardiogram ECG. is
remarkable for T-wave inversions in the lateral leads. Which of the following is the most
appropriate next step in management?

A Give the patient two nitroglycerin tablets sublingually and observe if his chest pain resolves.
B Place the patient on a cardiac monitor, administer oxygen, and give aspirin.
C Call the cardiac catheterization laboratory for immediate percutaneous intervention PCI..
D Order a chest x-ray; administer aspirin, clopidogrel, and heparin.
ANSWER:B
901
A 36-year-old woman presents to the ED with sudden onset of leftsided chest pain and mild
shortness of breath that began the night beforE. She was able to fall asleep without difficulty
but woke up in the morning with persistent pain that is worsened upon taking a deep breath.
She walked up the stairs at home and became very short of breath, which made her come to
the ED. Two weeks ago, she took a 7-hour flight from Europe and since then has left-sided
calf pain and swelling. What is the most common ECG finding for this patient’s presentation?
A Tachycardia or nonspecific ST-T–wave changes
B Atrial fibrillation
C Right-axis deviation
D Right-atrial enlargement
ANSWER:A
902
A 29-year-old tall, thin man presents to the ED after feeling short of breath for 2 days. In the
ED, he is in no acute distress. His BP is 115/70 mm Hg, HR is 81 beats per minute, RR is 16
breaths per minute, and oxygen saturation is 98% on room air. Cardiac, lung, and abdominal
examinations are normal. An ECG reveals sinus rhythm at a rate of 79. A chest radiograph
shows a small right-sided less than 10% of the hemithorax. spontaneous pneumothorax. A
repeat chest x-ray 6 hours later reveals a decreased pneumothorax. Which of the following is
the most appropriate next step in management?
A Discharge the patient with follow-up in 24 hours
B Perform needle decompression in the second intercostal space, midclavicular line
C Insert a 20F chest tube into right hemithorax
D Observe for another 6 hours
ANSWER:A

903
A 42-year-old man found vomiting in the street is brought to the ED by emergency medical
services EMS.. He has a known history of alcohol abuse with multiple presentations for
intoxication. Today, the patient complains of acute onset, persistent chest pain associated
with dysphagia, and pain upon flexing his neck. His BP is 115/70 mm Hg, HR is 101 beats
per minute, RR is 18 breaths per minute, and oxygen saturation is 97% on room air. As you
listen to his heart, you hear a crunching sounD. His abdomen is soft with mild epigastric
tenderness. The ECG is sinus tachycardia without ST-T–wave abnormalities. On chest x-ray,
you note lateral displacement of the left mediastinal pleural. What is the most likely
diagnosis?
A Aspiration pneumonia
B Acute pancreatitis
C Pericarditis
D Esophageal perforation
ANSWER:D
904
A 65-year-old man with a history of chronic hypertension presents to the ED with sudden-
onset tearing chest pain that radiates to his jaw. His BP is 205/110 mm Hg, HR is 90 beats
per minute, RR is 20 breaths per minute, and oxygen saturation is 97% on room air. He
appears apprehensivE. On cardiac examination you hear a diastolic murmur at the right
sternal border. A chest x-ray reveals a widened mediastinum. Which of the following is the
preferred study of choice to diagnose this patient’s condition?
A Electrocardiogram ECG.
B Transthoracic echocardiography TTE.
C Transesophageal echocardiography TEE.
D Computed tomography CT. scan
ANSWER:C
905
A 47-year-old man with a history of hypertension presents to the ED complaining of
continuous left-sided chest pain that began while snorting cocaine 1 hour ago. The patient
states he never experienced chest pain in the past when using cocainE. His BP is 170/90 mm
Hg, HR is 101 beats per minute, RR is 18 breaths per minute, and oxygen saturation is 98%
on room air. The patient states that the only medication he takes is alprazolam to “calm his
nerves.” Which of the following medications is contraindicated in this patient?
A Metoprolol

B Diltiazem
C Aspirin
D Lorazepam
ANSWER:A
906
A 32-year-old woman presents to the ED with a persistent fever of 101°F over the last 3
days. The patient states that she used to work as a convenience store clerk but was fired 2
weeks ago. Since then, she has been using drugs intravenously daily. Cardiac examination
reveals a heart murmur. Her abdomen is soft and nontender with an enlarged spleen. Chest
radiograph reveals multiple patchy infiltrates in both lung fields. Laboratory results reveal
white blood cells WBC. 14,000/μL with 91% neutrophils, hematocrit 33%, and platelets
250/μL. An ECG reveals sinus rhythm with first-degree heart block. Which of the following
is the most appropriate next step in management?
A
Obtain four sets of blood cultures, order a transthoracic echocardiogram TTE. , and start
antibiotic treatment.
B
Order a monospot test and recommend that the patient refrain from vigorous activities for 1
month.
C
Administer a nonsteroidal anti-inflammatory drug NSAID. and inform the patient she has
pericarditis.
D Administer isoniazid INH. and report the patient to the Department of Health.
ANSWER:A
907
A 31-year-old man who works for a moving company presents to the ED because he
thought he was having a heart attack. He does not smoke, and jogs 3 days a week. His father
died of a heart attack in his sixties. He describes a gradual onset of chest pain that is worse
with activity and resolves when he is at rest. His HR is 68 beats per minute, BP is 120/70
mm Hg, and RR is 14 breaths per minutE. On examination, his lungs are clear and there is
no cardiac murmur. You palpate tenderness over the left sternal border at the third and
fourth riAn ECG reveals sinus rhythm at a rate of 65. A chest radiograph shows no infiltrates
or pneumothorax. Which of the following is the most appropriate next step in management?
A Administer aspirin and send for a troponin.
B Administer aspirin, clopidogrel, and heparin and admit for acute coronary syndrome ACS.

C Administer ibuprofen and reassure the patient that he is not having a heart attack.
D Inject corticosteroid into the costochondral joint to reduce inflammation.
ANSWER:C
908
A 21-year-old woman presents to the ED complaining of lightheadedness. Her symptoms
appeared 45 minutes ago. She has no other symptoms and is not on any medications. She has
a medical history of mitral valve prolapsE. Her HR is 170 beats per minute and BP is
105/55 mm Hg. Physical examination is unremarkablE. After administering the appropriate
medication, her HR slows down and her symptoms resolvE. You repeat a 12-lead ECG that
shows a rate of 89 beats per minute with a regular rhythm. The PR interval measures 100
msec and there is a slurred upstroke of the QRS complex. Based on this information, which
of the following is the most likely diagnosis?
A Ventricular tachycardia
B Wolff-Parkinson-White WPW. Syndrome
C Atrial fibrillation
D Lown-Ganong-Levine LGL. syndrome
ANSWER:B
909
While eating dinner, a 55-year-old man suddenly feels a piece of steak “get stuck” in his
stomach. In the ED, he complains of dysphagia, is drooling, and occasionally retches. On
examination, his BP is 130/80 mm Hg, HR is 75 beats per minute, RR is 15 breaths per
minute, and oxygen saturation is 99% on room air. He appears in no respiratory distress.
Chest x-ray is negative for air under the diaphragm. Which of the following is the most
appropriate next step in management?
A Administer 1-mg glucagon intravenously while arranging for endoscopy.
B Administer a meat tenderizer such as papain to soften the food bolus.
C Administer 10-mL syrup of ipecac to induce vomiting and dislodge the food bolus.
D Perform the Heimlich maneuver until the food dislodges.
ANSWER:A

910
A 59-year-old man presents to the ED with left-sided chest pain and shortness of breath that
began 2 hours prior to arrival. He states the pain is pressure-like and radiates down his left
arm. He is diaphoretiC. His BP is 160/80 mm Hg, HR 86 beats per minute, and RR 15
breaths per minutE. ECG reveals 2-mm ST-segment elevation in leads I, aVL, V3 to V6.
Which of the following is an absolute contraindication to receiving thrombolytic therapy?
A Systolic BP greater than 180 mm Hg
B Patient on Coumadin and aspirin
C Total hip replacement 3 months ago
D Previous hemorrhagic stroke
ANSWER:D
911
A 67-year-old woman is brought to the ED by paramedics complaining of dyspnea, fatigue,
and palpitations. Her BP is 80/50 mm Hg, heart is 139 beats per minute, and RR is 20 breaths
per minutE. Her skin is cool and she is diaphoretiC. Her lung examination reveals bilateral
crackles and she is beginning to have chest pain. Her ECG shows a narrow complex irregular
rhythm with a rate in the 140s. Which of the following is the most appropriate immediate
treatment for this patient?
A Synchronized cardioversion
B Metoprolol
C Digoxin
D Coumadin
ANSWER:A
912
A 61-year-old woman with a history of congestive heart failure CHF. is at a family picnic
when she starts complaining of shortness of breath. Her daughter brings her to the ED where
she is found to have an oxygen saturation of 85% on room air with rales halfway up both of
her lung fields. Her BP is 185/90 mm Hg and pulse rate is 101 beats per minutE. On
examination, her jugular venous pressure JVP. is 6 cm above the sternal anglE. There is
lower extremity pitting edemA. Which of the following is the most appropriate first-line
medication to lower cardiac preload?
A Metoprolol
B Morphine sulfate
C Nitroprusside

D Nitroglycerin
ANSWER:D
913
A 56-year-old woman with a history of ovarian cancer presents to the ED with acute onset of
right-sided chest pain, shortness of breath, and dyspneA. Her BP is 131/75 mm Hg, HR is
101 beats per minute, respirations are 18 breaths per minute, and oxygen saturation is 97%
on room air. You suspect this patient has a pulmonary embolism PE. . Which of the
following tests is most likely to be abnormal?
A Arterial blood gas
B Oxygen saturation
C D-dimer
D Chest radiograph
ANSWER:C
914
A 61-year-old woman with a history of diabetes and hypertension is brought to the ED by
her daughter. The patient states that she started feeling short of breath approximately 12
hours ago and then noticed a tingling sensation in the middle of her chest and became
diaphoretiC. An ECG reveals ST-depression in leads II, III, and aVF. You believe that the
patient had a non–ST-elevation MI. Which of the following cardiac markers begins to rise
within 3 to 6 hours of chest pain onset, peaks at 12 to 24 hours, and returns to baseline in 7
to 10 days?
A Myoglobin
B Creatinine kinase CK.
C Creatinine kinase-MB CK-MB.
D Troponin I
ANSWER:D
915
A 27-year-old man complains of chest palpitations and lightheadedness for the past hour. He
has no past medical history and is not taking any medications. He drinks a beer occasionally
on the weekend and does not smoke cigarettes. His HR is 180 beats per minute, BP is 110/65
mm Hg, and oxygen saturation is 99% on room air. An ECG reveals a HR of 180 beats per
minute with a QRS complex of 90 msec with a regular rhythm. There are no discernable P
waves. Which of the following is the most appropriate medication to treat this dysrhythmia?
A Digoxin

B Lidocaine
C Amiodarone
D Adenosine
ANSWER:D
916
A 55-year-old man presents to the ED at 2:00 AM with left-sided chest pain that radiates
down his left arm. He takes a β-blocker for hypertension, a proton-pump inhibitor for
gastroesophageal reflux disease, and an antilipid agent for high cholesterol. He also took
sildenafil the previous night for erectile dysfunction. His BP is 130/70 mm Hg and HR is 77
beats per minutE. Which of the following medication is contraindicated in thispatient?
A Aspirin
B Unfractionated heparin
C Nitroglycerin
D Metoprolol
ANSWER:C
917
A 71-year-old man is playing cards with some friends when he starts to feel a pain in the left
side of his chest. His fingers in the left hand become numb and he feels short of breath. His
wife calls the ambulance and he is brought to the hospital. In the ED, an ECG is performeD.
Which of the following best describes the order of ECG changes seen in an MI?
A Hyperacute T wave, ST-segment elevation, Q wave
B Q wave, ST-segment elevation, hyperacute T wave
C Hyperacute T wave, Q wave, ST-segment elevation
D ST-segment elevation, Q wave, hyperacute T wave
ANSWER:A
918
A 63-year-old insurance agent is brought to the ED by paramedics for shortness of breath
and a RR of 31 breaths per minutE. The patient denies chest pain, fever, vomiting, or
diarrheA. His wife says he ran out of his “water pill” 1 week ago. His BP is 185/90 mm Hg,
HR is 101 beats per minute, oxygen saturation is 90% on room air, and temperature is
98.9°F. There are crackles midway up both lung fields and 2+ pitting edema midway up his
legs. An ECG shows sinus tachycardiA. The patient is sitting up and able to speak to you.
After placing the patient on a monitor and inserting an IV, which of the following is the most
appropriate next step in management?

A
Obtain blood cultures, complete blood cell CBC. count, and begin empiric antibiotic
therapy.
B Order a statim STAT. portable chest x-ray.
C Administer oxygen via nasal cannula and have the patient chew an aspirin.
D
Administer oxygen via non-rebreather, furosemide, nitroglycerin, and consider noninvasive
respiratory therapy.
ANSWER:D
919
Which of the following patients has the lowest clinical probability for the diagnosis of
pulmonary embolism?
A A 21-year-old woman 2 days after a cesarean delivery
B
A 55-year-old woman on estrogen replacement therapy who underwent a total hip
replacement procedure 3 days ago
C
A 39-year-old man who smokes cigarettes occasionally and underwent an uncomplicated
appendectomy 2 months ago
D A 62-year-old man with pancreatic cancer
ANSWER:C
920
A 75-year-old man goes out to shovel snow from his driveway. After 5 minutes of
shoveling, he feels short of breath, chest pain, and then passes out. He awakens minutes later
to his wife shaking him. In the ED, he denies chest pain or dyspneHis BP is 1605 mm Hg,
HR is 71 beats per minute, and oxygen saturation is 97% on room air. On examination, you
hear a harsh systolic ejection murmur. An ECG reveals a sinus rhythm with left ventricular
hypertrophy. Which of the following is the most likely diagnosis?
A Asystolic cardiac arrest
B Brugada syndrome
C Subclavian steal syndrome
D Aortic stenosis
ANSWER:D

921
While playing a match of tennis, a 56-year-old man with a medical history significant only
for acid reflux disease starts to feel substernal chest pain that radiates into his left arm and
shortness of breath. His pain feels better after drinking antacid, but since it is not completely
resolved, his partner calls 911. Upon arrival, EMS administers aspirin and sublingual
nitroglycerin. After 20 minutes, the man’s symptoms resolvE. He is brought to the ED for
further evaluation where his ECG shows sinus rhythm without any ischemic abnormalities.
You order a chest radiograph and send his blood work to the laboratory for analysis. Which
of the following statements regarding the diagnosis of acute MI is most accurate?
A A normal ECG rules out the diagnosis of acute MI.
B
One set of negative cardiac enzymes is sufficient to exclude the diagnosis of MI in this
patient.
C Troponin may not reach peak levels for at least 12 hours.
D Relief of symptoms by antacids essentially rules out a cardiac cause of his chest pain.
ANSWER:C
922
A 62-year-old woman presents to the ED with general weakness, shortness of breath, and
substernal chest pain that radiates to her left shoulder. Her BP is 155/80 mm Hg, HR is 92
beats per minute, and RR is 16 breaths per minutE. You suspect that she is having an acute
MI. Which of the following therapeutic agents has been shown to independently reduce
mortality in the setting of an acute MI?
A Nitroglycerin
B Aspirin
C Unfractionated heparin
D Lidocaine
ANSWER:B

923
A 22-year-old college student went to the health clinic complaining of a fever over the last 5
days, fatigue, myalgias, and a bout of vomiting and diarrheA. The clinic doctor diagnosed
him with acute gastroenteritis and told him to drink more fluids. Three days later, the student
presents to the ED complaining of substernal chest pain that is constant. He also feels short of
breath. His temperature is 100.9°F, HR is 119 beats per minute, BP is 120/75 mm Hg, and
RR is 18 breaths per minutE. An ECG is performed revealing sinus tachycardiA. A chest
radiograph is unremarkablE. Laboratory tests are normal except for slightly elevated WBCs.
Which of the following is the most common cause of this patient’s diagnosis?
A Streptococcus viridans
B Staphylococcus aureus
C Coxsackie B virus
D Atherosclerotic disease
ANSWER:C
924
A 51-year-old woman presents to the ED after 5 consecutive days of crushing substernal
chest pressure that woke her up from sleep in the morning. The pain resolves spontaneously
after 20 to 30 minutes. She is an avid rock climber and jogs 5 miles daily. She has never
smoked cigarettes and has no family history of coronary diseasE. In the ED, she experiences
another episode of chest pain. An ECG reveals ST-segment elevations and cardiac
biomarkers are negativE. The pain is relieved with sublingual nitroglycerin. She is admitted
to the hospital and diagnostic testing reveals minimal coronary atherosclerotic diseasE.
Which of the following is the most appropriate medication to treat this patient’s condition?
A Aspirin
B Calcium channel blocker CCB.
C β-Blocker
D H2-Blocker
ANSWER:B

925
While discussing a case presentation with a medical student, a nearby patient who just
returned from getting an ankle radiograph done yells out in pain. You walk over to him
and ask what is wrong. He states that since returning from the radiology suite, his automatic
implantable cardioverter defibrillator AICD. is discharging. You hook him up to the monitor
and note that his rhythm is sinus. You observe a third shock while the patient is in sinus
rhythm. Which of the following is the most appropriate next step in management?
A Send the patient back to the radiology suite for another radiograph to desensitize his AICD.
B
Administer pain medication and wait until the device representative arrives at the hospital to
power off the AICD.
C
Admit the patient to the telemetry unit to monitor his rhythm and find the cause of his AICD
dischargE.
D Place a magnet over the AICD generator to inactivate it and thereby prevent further shocks.
ANSWER:D
926
A 55-year-old man presents to the ED with chest pain and shortness of breath. His BP is
170/80 mm Hg, HR is 89 beats per minute, and oxygen saturation is 90% on room air.
Physical examination reveals crackles midway up both lung fields and a new holosystolic
murmur that is loudest at the apex and radiates to the left axillA. ECG reveals ST elevations
in the inferior leads. Chest radiograph shows pulmonary edema with a normalsized cardiac
silhouettE. Which of the following is the most likely cause of the cardiac murmur?
A Critical aortic stenosis
B Papillary muscle rupture
C Pericardial effusion
D CHF
ANSWER:B

927
A 22-year-old man presents to the ED with a history consistent with an acute MI. His ECG
reveals ST elevations and his cardiac biomarkers are positivE. He has been smoking half a
pack of cigarettes per day for the last 3 months. He drinks alcohol when hanging out with his
friends. His grandfather died of a heart attack at 80 years of age. The patient does not have
hypertension or diabetes mellitus and takes no prescription medications. A recent cholesterol
check revealed normal levels of total cholesterol, lowdensity lipoprotein LDL., and high-
density lipoprotein HDL.Which of the following is the most likely explanation for his
presentation?
A Cigarette smoking
B Family history of heart attack at age 80 years
C Incorrectly placed leads on the ECG
D Undisclosed cocaine use
ANSWER:D
928
A 29-year-old man is brought to the ED by EMS for a syncopal episode that occurred
during a basketball gamE. A friend states that the patient just dropped to the ground shortly
after scoring a basket on a fast-break. On examination, you note a prominent systolic
ejection murmur along the left sternal border and at the apex. An ECG reveals left
ventricular hypertrophy, left atrial enlargement, and septal Q waves. You suspect the
diagnosis and ask the patient to perform the Valsalva maneuver while you auscultate his
heart. Which of the following is most likely to occur to the intensity of the murmur with this
maneuver?
A Decrease
B Increase
C Remain unchanged
D Disappear
ANSWER:B
929
A 68-year-old woman with recently diagnosed uterine cancer is brought to the ED by her
daughter. The patient complains of acute onset right-sided chest pain that is sharp in
character and worse with inspiration. Her BP is 135/85 mm Hg, HR 107 beats per minute, RR
20 breaths per minute, and oxygen saturation 97% on room air. Physical examination reveals
a swollen and tender right calf. ECG is sinus tachycardiA. Which of the following is the most
appropriate next step in management?

A Start heparin therapy prior to diagnostic study
B Administer thrombolytics
C Order a ventilation-perfusion scan
D Order a CT angiogram
ANSWER:A
930
A 57-year-old man complains of chest palpitations and lightheadedness for the past hour.
Five years ago he underwent a cardiac catheterization with coronary artery stent placement.
He smokes half a pack of cigarettes daily, and drinks a glass of wine at dinner. His HR is 140
beats per minute, BP is 115/70 mm Hg, and oxygen saturation is 99% on room air. An ECG
reveals a wide complex tachycardia at a rate of 140 that is regular in rhythm. An ECG from
6 months ago shows a sinus rhythm at a rate of 80. Which of the following is the most
appropriate medication to treat this dysrhythmia?
A Digoxin
B Diltiazem
C Amiodarone
D Adenosine
ANSWER:C
931
A 55-year-old man with hypertension and a one-pack-per-day smoking history presents to
the ED complaining of three episodes of severe heavy chest pain this morning that radiated
to his left shoulder. In the past, he experienced chest discomfort after walking 20 minutes
that resolved with rest. The episodes of chest pain this morning occurred while he was
reading the newspaper. His BP is 155/80 mm Hg, HR 76 beats per minute, RR 15 breaths
per minutE. He does not have chest pain in the ED. An ECG reveals sinus rhythm with a
rate of 72. A troponin I is negativE. Which of the following best describes this patient’s
diagnosis?
A Variant angina
B Stable angina
C Unstable angina
D Non–ST-elevation MI
ANSWER:C

932
A 55-year-old woman with a past medical history of diabetes walks into the emergency
department ED. stating that her tongue and lips feel like they are swollen. During the
history, she tells you that her doctor just started her on a new blood pressure BP.
medication. Her only other medication is a baby aspirin. Her vitals at triage are: BP 130/70
mm Hg, heart rate HR. 85 beats per minute, respiratory rate RR. 16 breaths per minute,
oxygen saturation 99% on room air, and temperature 98.7°F. On physical examination, you
detect mild lip and tongue swelling. Over the next hour, you notice that not only are her
tongue and lips getting more swollen, but her face is starting to swell, too. What is the most
likely inciting agent?
A Metoprolol
B Furosemide
C Aspirin
D Lisinopril
ANSWER:D
933
A 45-year-old woman presents to the ED immediately after landing at the airport from a
transatlantic flight. She states that a few moments after landing she felt short of breath and
felt pain in her chest when she took a deep breath. Her only medications are oral
contraceptive pills and levothyroxinE. She is a social drinker and smokes cigarettes
occasionally. Her BP is 130/75 mm Hg, HR is 98 beats per minute, temperature is 98.9F,
RR is 20 breaths per minute, and oxygen saturation is 97% on room air. You send her for a
duplex ultrasound of her legs, which is positive for deep vein thrombosis. What is the most
appropriate management for this patient?
A
Place patient on a monitor, provide supplemental oxygen, and administer unfractionated
heparin.
B
Place patient on a monitor, order a chest computed tomography CT. scan to confirm a
pulmonary embolism PE. , and then administer unfractionated heparin.
C Place patient on a monitor and administer aspirin.
D
Instruct the patient to walk around the ED so that she remains mobile and does not
exacerbate thrombus formation.
ANSWER:A

934
A 54-year-old undomiciled woman presents to the ED with severe cough, general malaise,
and subjective fevers for the last week. She also describes coughing up “chicken livers”
during this time and reports that her symptoms are getting progressively worse. Her initial
vitals include a HR of 100 beats per minute, a BP of 145/66 mm Hg, temperature of 99.9 °F,
and an RR of 16 breaths per minute with an oxygen saturation of 95% on room air. She
states that she has a history of alcohol abuse, but denies taking any medications or illicit
drugs. A chest radiograph shows a lobar pneumoniA. Given this patient’s clinical
presentation, which of the following is this patient at most risk for contracting?
A Streptococcus pneumoniae
B Klebsiella pneumoniae
C Mycoplasma pneumoniae
D Legionella pneumophila
ANSWER:B
935
A tall, thin 18-year-old man presents to the ED with acute onset of dyspnea while at rest. The
patient reports sitting at his desk when he felt a sharp pain on the right side of his chest that
worsened with inspiration. His past medical history is significant for peptic ulcer diseasE. He
reports taking a 2-hour plane trip a month ago. His initial vitals include a HR of 100 beats
per minute, a BP of 120/60 mm Hg, an RR of 16 breaths per minute, and an oxygen
saturation of 97% on room air. On physical examination, you note decreased breath sounds
on the right side. Which of the following tests should be performed next?
A Electrocardiogram ECG.
B Chest radiograph
C Ventilation perfusion scan V/Q scan.
D Upright abdominal radiograph
ANSWER:B
936
A 46-year-old alcoholic man presents to the ED with cough, fever, and rigors for 2 days. He
describes his sputum as rust-coloreD. His vital signs are: temperature 101.1 ˚F, HR 94, BP
125/75 mm Hg, RR 20, and pulse oxygen of 97% on room air. Auscultation reveals crackles
in the left-lower lobE. Chest radiograph is significant for a left-lower lobar infiltratE. Which
of the following organism is the most common cause of community acquired bacterial
pneumonia?

A Haemophilus influenza
B Streptococcus pneumoniae
C Klebsiella pneumonia
D Group A streptococci
ANSWER:B
937
A 30-year-old obese woman with no significant past medical history presents to the ED
complaining of shortness of breath and coughing up blood-streaked sputum. The patient
states that she traveled to Moscow a month ago. Upon returning to the United States, the
patient developed a persistent cough associated with dyspneA. She was seen by a
pulmonologist, who diagnosed her with bronchitis and prescribed an inhaler. However, over
the following weeks, the patient’s symptoms worsened, and she developed pleuritic chest
pain. In the ED, she lets you know that she smokes half a pack per day. Her vitals include a
temperature of 99°F, BP of 105/65 mm Hg, HR of 124 beats per minute, RR of 22 breaths
per minute, and an oxygen saturation of 94% on room air. Physical examination is
noncontributory, except for rales at the left-mid lung. Her ECG reveals sinus tachycardia
with large R waves in V1 to V3 and inverted T waves. Given this patient’s history and
presentation, what is the most likely etiology of her symptoms?
A Mycoplasma pneumoniae “walking” pneumoniA.
B Q fever pneumonia
C Pneumocystis jiroveci pneumonia PCP.
D PE
ANSWER:D
938
A 24-year-old woman is brought to the ED after being found on a nearby street hunched
over and in mild respiratory distress. Upon arrival, she is tachypneic at 24 breaths per minute
with an oxygen saturation of 97% on face mask oxygen administration. Upon physical
examination, the patient appears to be in mild distress with supraclavicular retractions.
Scattered wheezing is heard throughout bilateral lung fields. Which of the following
medications should be administered first?
A β2-Agonist nebulizer treatment
B Magnesium sulfate
C Epinephrine
D Anticholinergic nebulizer treatment

ANSWER:A
939
An 81-year-old woman presents to the ED with acute onset of shortness of breath just before
arrival. She refuses to answer questions for the interview, but repeatedly states that she is
feeling short of breath. Her initial vitals include a HR of 89 beats per minute, a BP of 168/76
mm Hg, and an RR of 18 breaths per minute with an oxygen saturation of 89% on room air.
A portable chest x-ray appears normal. Her physical examination is unremarkable, except for
a systolic ejection murmur. Intravenous IV. access is successfully obtaineD. After placing
the patient on oxygen and a monitor, which of the following should be performed first?
A Evaluation of troponin level
B Evaluation of D-dimer level
C ECG
D Repeat chest x-ray
ANSWER:C
940
A 30-year-old man is brought to the ED by emergency medical service EMS. in respiratory
distress. His initial vitals include a HR of 109 beats per minute, a BP of 180/90 mm Hg, and
an RR of 20 breaths per minute with an oxygen saturation of 92% on room air. A chest x-ray
shows a bilateral diffuse infiltrative process. A subsequent toxicologic screen is positivE.
Which of the following agents is most likely responsible for this patient’s presentation?
A Cannabis
B Opioid
C Crack cocaine
D Methamphetamine
ANSWER:C
941
A 26-year-old woman presents to the ED with an acute onset of dyspnea after falling down a
few steps. The patient denies any loss of consciousness and reports feeling short of breath.
Her initial chest x-ray appears normal; however she continues to be symptomatic with stable
vital signs. Which of the following procedures should be performed next?
A Repeat upright chest x-ray
B Inspiratory and expiratory chest radiographs

C Chest CT scan
D Chest thoracostomy
ANSWER:B
942
As you evaluate a patient with shortness of breath, you appreciate decreased breath sounds at
the left-lung basE. You suspect the patient has a small pleural effusion. Which of the
following views of the chest is this small pleural effusion most likely to be detected?
A Supine
B Lateral decubitus right-side down
C Lateral decubitus left-side down
D Lateral
ANSWER:C
943
A 32-year-old firefighter presents to the ED in acute respiratory distress. He was taken to the
ED shortly after extinguishing a large fire in a warehousE. His initial vitals include a HR of
90 beats per minute, a BP of 120/55 mm Hg, and an RR of 18 breaths per minute with an
oxygen saturation of 98% on 2-L nasal cannulA. An ECG shows a first-degree heart block.
Upon physical examination, there are diffuse rhonchi bilaterally. The patient is covered in
soot and the hairs in his nares are singeD. Given this clinical presentation, which of the
following may be responsible for this patient’s respiratory distress?
A Reactive airway disease
B Foreign body aspiration
C Decompression sickness
D Thermal burns
ANSWER:D
944
A 76-year-old man presents to the ED in acute respiratory distress, gasping for breath while
on face mask. Paramedics state that he was found on a bench outside of his apartment in
respiratory distress. Initial vitals include a HR of 90 beats per minute, a BP of 170/90 mm
Hg, and an RR of 33 breaths per minute with an oxygen saturation of 90%. Upon physical
examination, the patient is coughing up pink, frothy sputum, has rales two-thirds of the way
up both lung fields, and has pitting edema of his lower extremities. A chest radiograph
reveals bilateral perihilar infiltrates, an enlarged cardiac silhouette, and a small right-sided
pleural effusion. After obtaining IV access and placing the patient on a monitor, which of the
following medical interventions is most appropriate?

A Morphine sulfate only
B Nitroglycerin only
C Nitroglycerin and a loop diuretic
D Aspirin
ANSWER:C
945
A 67-year-old man is brought to the ED in respiratory distress. His initial vitals include a HR
of 112 beats per minute, a BP of 145/88 mm Hg, and an RR of 18 breaths per minute with
an oxygen saturation of 92% on room air. He is also febrile at 102°F. After obtaining IV
access, placing the patient on a monitor, and administering oxygen via nasal cannula, a chest
radiograph is performed and shows patchy alveolar infiltrates with consolidation in the lower
lobes. On review of systems, the patient tells you that he had five to six watery bowel
movements a day for the last 2 days with a few bouts of emesis. Which of the following
infectious etiologies is most likely responsible for the patient’s presentation?
A Streptococcus pneumoniae
B Legionella pneumophila
C Mycoplasma pneumoniae
D Chlamydophila pneumoniae
ANSWER:B
946
A 58-year-old man presents to the ED with progressive dyspnea over the course of 1 week.
Upon arrival, he is able to speak in full sentences and states that he stopped taking all of his
medications recently. Initial vitals include a HR of 92 beats per minute, a BP of 180/100 mm
Hg, and an RR of 16 breaths per minute with an oxygen saturation of 94% on room air.
Upon physical examination, the patient has bibasilar crackles, jugular venous distention, and
pedal edemA. Which of the following medication regimens was the patient most likely on?
A Loop diuretic only
B Aspirin only
C Loop diuretic and β-blocker
D Loop diuretic, β-blocker, and angiotensin-converting enzyme ACE. Inhibitor
ANSWER:D

947
A 32-year-old woman presents to the ED with a 1-month history of general malaise, mild
cough, and subjective fevers. She states that she is human immunodeficiency virus HIV.
positive and her last CD4 count, 6 months ago, was 220. She is not on antiretroviral therapy
or any other medications. Initial vitals include a HR of 88 beats per minute, a BP of 130/60
mm Hg, and an RR of 12 breaths per minute with an oxygen saturation of 91% on room air.
Her chest radiograph shows bilateral diffuse interstitial infiltrates. Subsequent laboratory tests
are unremarkable except for an elevated lactate dehydrogenase level. Given this patient’s
history and physical examination, which of the following is the most likely organism
responsible for her clinical presentation?
A Coccidioides immitis
B Mycobacterium tuberculosis
C Pneumocystis jiroveci
D Mycoplasma pneumoniae
ANSWER:C
948
A 27-year-old woman presents to the ED complaining of an intensely pruritic rash all over
her body, abdominal cramping, and chest tightness. She states that 1 hour ago she was at
dinner and accidentally ate some shrimp. She has a known anaphylactic allergy to shrimp.
Her BP is 115/75 mm Hg, HR is 95 beats per minute, temperature is 98.9°F, RR is 20 breaths
per minute, and oxygen saturation is 97% on room air. She appears anxious, and her skin is
flushed with urticarial lesions. Auscultation of her lungs reveals scattered wheezes with
decreased air entry. Which of the following is the most appropriate next step in management?
A
Administer oxygen via non-rebreather, place a large-bore IV, begin IV fluids, and
administer methylprednisolone intravenously.
B
Administer oxygen via non-rebreather, place a large-bore IV, begin IV fluids, and
administer methylprednisolone and diphenhydramine intravenously.
C
Administer oxygen via non-rebreather, place a large-bore IV, begin IV fluids, administer
methylprednisolone and diphenhydramine intravenously, and give subcutaneous
epinephrinE.
D
Administer oxygen via non-rebreather, place a large-bore IV, begin IV fluids, and start
aerosolized albuterol.
ANSWER:C

949
An 82-year-old woman becomes acutely short of breath while at rest on the rehabilitation
unit. She is brought into the ED with an oxygen saturation of 86% on room air and in acute
respiratory distress. Her initial ECG is within normal limits and unchanged from a recent
previous examination. Her initial chest x-ray is also negativE. Upon chest auscultation, there
are equal bilateral breath sounds with some scattered rhonchi. Her nurse tells you that 2 days
ago she underwent internal fixation of a right-femur fracture and has been on anticoagulant
therapy. Given the history and presentation of this patient, what is the most likely etiology of
her symptoms?
A Venous thromboembolism
B Air embolism
C Fat embolism
D Pulmonary hemorrhage
ANSWER:C
950
A 72-year-old man presents to the ED with worsening dyspneA. His initial vitals include a
HR of 93 beats per minute, BP of 110/50 mm Hg, and RR of 20 breaths per minute with an
oxygen saturation of 88% on room air. The patient appears thin and anxious. He is using
accessory muscles to breathE. Despite distant breath sounds, you hear end-expiratory
rhonchi and a prolonged expiratory phasE. An ECG shows peaked P waves in leads II, III,
and aVF. Given this patient’s history and physical examination, which of the following
conditions does this patient most likely have?
A Chronic bronchitis
B Asthma
C Emphysema
D Congestive heart failure CHF.
ANSWER:C
951
A 71-year-old woman presents to the ED after a reported mechanical fall 2 days ago. Her
initial vitals include a HR of 55 beats per minute, a BP of 110/60 mm Hg, an RR of 14
breaths per minute, and an oxygen saturation of 96% on room air. The patient does not
appear to be taking deep breaths. Her physical examination is significant for decreased
breath sounds bilaterally and tenderness to palpation along the right side of her chest. After
initial stabilization, which of the following is the diagnostic test of choice for this patient’s
condition?

A Chest x-ray
B Chest CT scan
C ECG
D Rib radiographs
ANSWER:D
952
A 29-year-old woman presents to the ED for hyperventilation. Her initial vitals include an
RR of 28 breaths per minute with an oxygen saturation of 100% on room air. She is able to
speak in full sentences and tells you that she cannot breathe and that her hands and feet are
cramping up. She denies any trauma, past medical history, or illicit drug usE. Chest
auscultation reveals clear breath sounds bilaterally. A subsequent chest radiograph is normal.
Upon reevaluation, the patient reports that she is breathing better. Her vitals include an RR of
12 breaths per minute with an oxygen saturation of 100% on room air. Which of the
following conditions is most likely the etiology of this patient’s symptoms?
A Pneumothorax
B Hemopneumothorax
C Pleural effusion
D Anxiety attack
ANSWER:D
953
A 42-year-old man presents to the ED via ambulance after activating EMS for dyspneA. He
is currently on an oxygen face mask and was administered one nebulized treatment of a β2-
agonist by the paramedics. His initial vitals include an RR of 16 breaths per minute with an
oxygen saturation of 96% on room air. The patient appears to be in mild distress with some
intercostal retractions. Upon chest auscultation, there are minimal wheezes localized over
bilateral lower lung fields. The patient’s symptoms completely resolve after two more
nebulizer treatments. Which of the following medications, in addition to a rescue β2-agonist
inhaler, should be prescribed for outpatient use?
A Magnesium sulfate
B EpiPen
C Corticosteroids
D Cromolyn sodium
ANSWER:C

954
A 22-year-old woman is brought to the ED by paramedics who state that they found the
patient hunched over on a park bench barely breathing. The patient is rousable only to
painful stimuli. Her initial vitals include a HR of 78 beats per minute, a BP of 125/58 mm
Hg, and a respiratory rate of 6 breaths per minute with an oxygen saturation of 94% on 2-L
nasal cannulA. Upon physical examination, the patient has clear breath sounds bilaterally
and no signs of traumA. Her pupils are 2 mm bilaterally and reactive to light. Which of the
following agents may be used to restore this patient’s respirations?
A Oxygen
B Flumazenil
C Anticholinergic inhaler treatment
D Naloxone
ANSWER:D
955
A 43-year-old undomiciled man is brought to the ED after being found intoxicated on the
street. He is currently rousable and expresses a request to be left alonE. Initial vitals include
a HR of 92 beats per minute, a BP of 125/80 mm Hg, and an RR of 14 breaths per minute
with an oxygen saturation of 93% on room air. His rectal temperature is 101.2°F. A chest
radiograph shows infiltrates involving the right-lower lobE. Given this clinical presentation,
what initial antibiotic coverage is most appropriate for this patient?
A Gram-negative coverage only
B Gram-positive coverage only
C Broad-spectrum with anaerobic coverage
D PCP coverage
ANSWER:C
956
A 32-year-old man is brought into the ED by EMS with fever, shortness of breath, and
stridor. The patient was treated yesterday in the ED for a viral syndromE. His BP is 90/50
mm Hg, HR is 110 beats per minute, temperature is 101.2°F, and his RR is 28 breaths per
minutE. A chest radiograph reveals a widened mediastinum. The patient is endotracheally
intubated, given a 2-L bolus of normal saline, and started on antibiotics. His BP improves to
110/70 mm Hg and he is transferred to the intensive care unit ICU.. You see a friend that
accompanied the patient to the hospital and ask him some questions. You find out that the
patient is a drum maker and works with animal hides. What is the most likely organism that is
responsible for the patient’s presentation?

A Streptococcus pneumoniae
B Corynebacterium diphtheriae
C Bacillus anthracis
D Haemophilus influenzae
ANSWER:C
957
A 62-year-old man presents to the ED with gradual dyspnea over the last few weeks. He
reports that he is a daily smoker and has not seen a physician in years. Upon physical
examination, there are decreased breath sounds on the right as compared to the left. A chest
radiograph indicates blunting of the right costophrenic angle with a fluid linE. A
thoracentesis is performeD. Given this patient’s history, which of the following most likely
describes his effusion?
A Transudative effusion
B Exudative effusion
C Transudative and exudative effusion
D Lactate dehydrogenase < 200 U
ANSWER:B
958
A 49-year-old woman presents to the ED with difficulty breathing after a morning jog. Her
initial vitals include a HR of 60 beats per minute, a BP of 120/55 mm Hg, and an RR of 20
breaths per minute with an oxygen saturation of 94% on room air. Upon physical
examination, the patient appears to be in mild distress with audible wheezing. She is able to
speak in partial sentences and states that she occasionally uses an inhaler. Given this patient’s
history and physical examination, which of the following measures should be taken next?
A Peak expiratory flow
B Chest radiograph
C β-Natriuretic peptide level
D Rectal temperature
ANSWER:A

959
An 81-year-old diabetic woman with a history of atrial fibrillation is transferred to your
emergency department ED. from the local nursing homE. The note from the facility states
that the patient is complaining of abdominal pain, having already vomited oncE. Her vital
signs in the ED are temperature 100.1°F, blood pressure BP. 105/75 mm Hg, heart rate HR.
95 beats per minute, and respiratory rate RR. 18 breaths per minutE. You examine the
patient and focus on her abdomen. Considering that the patient has not stopped moaning in
pain since arriving to the ED, you are surprised to find that her abdomen is soft on palpation.
You decide to order an abdominal radiographic series. Which of the findings on plain
abdominal film is strongly suggestive of mesenteric infarction?
A Sentinel loop of bowel
B No gas in the rectum
C Presence of an ileus
D Pneumatosis intestinalis
ANSWER:D
960
A husband and wife present to the ED with 1 day of subjective fever, vomiting, watery
diarrhea, and abdominal cramps. They were at a restaurant a day before for dinner and both
ate the seafood special, which consisted of raw shellfish. In the ED, they are both tachycardic
with temperatures of 99.8°F and 99.6°F for him and her, respectively. Which of the
following is responsible for the majority of acute episodes of diarrhea?
A Parasites
B Viruses
C Enterotoxin-producing bacteria
D Anaerobic bacteria
ANSWER:B
961
A 79-year-old man was being commemorated at an awards dinner for his 50 years of service
at the local bank. While eating a steak dinner, he felt food get stuck in his stomach. He drank
a glass of water, but shortly thereafter vomited the water up. He is a bit anxious and decides
to come to the ED for further evaluation. His BP is 155/70 mm Hg, HR is 98 beats per
minute, RR is 18 breaths per minute, and oxygen saturation is 99% on room air. What is the
most common area for an esophageal foreign body to lodge in an adult?

A Aortic arch
B Cricopharyngeus muscle
C Tracheal bifurcation
D Lower esophageal sphincter
ANSWER:D
962
As you palpate the right upper quadrant RUQ. of a 38-year-old woman’s abdomen, you
notice that she stops her inspiration for a brief moment. During the history, the patient states
that over the last 2 days she gets pain in her RUQ that radiates to her back shortly after
eating. Her vitals include a temperature of 100.4°F, HR of 95 beats per minute, BP of 130/75
mm Hg, RR of 16 breaths per minutE. What is the initial diagnostic modality of choice for
this disorder?
A Plain film radiograph
B Computed tomography CT. scan
C Ultrasonography
D Radioisotope cholescintigraphy HIDA scan.
ANSWER:C
963
A 31-year-old man from Florida presents to the ED complaining of severe pain that starts in
his left flank and radiates to his testiclE. The pain lasts for about 1 hour and then improves.
He had similar pain last week that resolved spontaneously. He noted some blood in his urine
this morning. His BP is 145/75 mm Hg, HR is 90 beats per minute, temperature is 98.9°F,
and his RR is 24 breaths per minutE. His abdomen is soft and nontender. As you examine
the patient, he vomits and has trouble lying still in his stretcher. Which of the following is the
most appropriate next step in management?
A Call surgery consult to evaluate the patient for appendicitis.
B Order an abdominal CT.
C
Start intravenous IV. fluids and administer an IV nonsteroidal anti-inflammatory drug
NSAID. and antiemetiC.
D Perform an ultrasound to evaluate for an abdominal aortic aneurysm AAA. .
ANSWER:C

964
A 67-year-old man is brought to the ED by emergency medical service EMS.. His wife
states that the patient was doing his usual chores around the house when all of a sudden he
started complaining of severe abdominal pain. He has a past medical history of coronary
artery disease and hypertension. His BP is 85/70 mm Hg, HR is 105 beats per minute,
temperature is 98.9°F, and his RR is 18 breaths per minutE. On physical examination, he is
diaphoretic and in obvious pain. Upon palpating his abdomen, you feel a large pulsatile
mass. An electrocardiogram ECG. reveals sinus tachycardiA. You place the patient on a
monitor, administer oxygen, insert two large-bore IVs, and send his blood to the laboratory.
His BP does not improve after a 1-L fluid bolus. Which of the following is the most
appropriate next step in management?
A Order a CT scan to evaluate his aortA.
B Call the angiography suite and have them prepare the room for the patient.
C Order a portable abdominal radiograph.
D Call surgery and have them prepare the operating room OR. for an exploratory laparotomy.
ANSWER:D
965
A 57-year-old woman presents to the ED with a basin in her hand and actively vomiting.
You insert an IV catheter, start IV fluids, and administer an antiemetic agent. The patient
feels much better but also complains of severe crampy abdominal pain that comes in waves.
You examine her abdomen and note that it is distended and that there is a small midline scar
in the lower abdomen. Upon auscultation, you hear high-pitched noises that sound like
“tinkles.” Palpation elicits pain in all four quadrants but no rebound tenderness. She is guaiac
negativE. Which of the following is the most common cause of this patient’s presentation?
A Travel to Mexico
B Ethanol abuse
C Hysterectomy
D Hernia
ANSWER:C

966
An undomiciled 41-year-old man walks into the ED complaining of abdominal pain, nausea,
and vomiting. He tells you that he has been drinking beer continuously over the previous 18
hours. On examination, his vitals are BP 150/75 mm Hg, HR 104 beats per minute, RR 16
breaths per minute, oxygen saturation 97% on room air, temperature of 99.1°F rectally, and
finger stick glucose 81 mg/dL. The patient is alert and oriented, his pupils anicteriC. You
notice gynecomastia and spider angiomatA. His abdomen is soft but tender in the RUQ.
Laboratory tests reveal an AST of 212 U/L, ALT 170 U/L, alkaline phosphatase of 98 U/L,
total bilirubin of 1.9 mg/dL, international normalized ratio INR. of 1.3, WBC 12,000/μL.
Urinalysis shows 1+ protein. Chest x-ray is unremarkablE. Which of the following is the
most appropriate next step in management?
A Place a nasogastric tube in the patient’s stomach to remove any remaining ethanol
B Supportive care by correcting any fluid and electrolyte imbalances
C Administer hepatitis B immune globulin
D Send viral hepatitis titers
ANSWER:B
967
A 48-year-old man with a past medical history of hepatitis C and cirrhosis presents to the ED
complaining of acute onset abdominal pain and chills. His BP is 118/75 mm Hg, HR is 105
beats per minute, RR is 16 breaths per minute, temperature is 101.2°F rectally, and oxygen
saturation is 97% on room air. His abdomen is distended, and diffusely tender. You decide to
perform a paracentesis and retrieve 1 L of cloudy fluiD. Laboratory analysis of the fluid
shows a neutrophil count of 550 cells/mm3. Which of the following is the most appropriate
choice of treatment?
A Metronidazole
B Vancomycin
C Cefotaxime
D Neomycin and lactulose
ANSWER:C

968
A 24-year-old man woke up from sleep 1 hour ago with severe pain in his right testiclE. He
states that he is sexually active with multiple partners. On examination, the right scrotum is
swollen, tender, and firm. You cannot elicit a cremasteric reflex. His BP is 145/75 mm Hg,
HR is 103 beats per minute, RR is 14 breaths per minute, temperature is 98.9°F, and oxygen
saturation is 99% on room air. Which of the following is the most appropriate next step in
management?
A
Administer one dose of ceftriaxone and doxycycline for 10 days and have him follow-up
with a urologist.
B Swab his urethra, send a culture for gonorrhea and Chlamydia, and treat if positivE.
C Order a statim STAT. color Doppler ultrasound and urologic consultation.
D Treat the patient for epididymitis and have him return if symptoms persist.
ANSWER:C
969
A 28-year-old man presents to the ED complaining of constant vague, diffuse epigastric
pain. He describes having a poor appetite and feeling nauseated ever since eating sushi last
night. His BP is 125/75 mm Hg, HR is 96 beats per minute, temperature is 100.5°F, and his
RR is 16 breaths per minutE. On examination, his abdomen is soft and moderately tender in
the right lower quadrant RLQ.. Laboratory results reveal a WBC of 12,000/μL. Urinalysis
shows 1+ leukocyte esterasE. The patient is convinced that this is food poisoning from the
sushi and asks for some antaciD. Which of the following is the most appropriate next step in
management?
A Order a plain radiograph to look for dilated bowel loops.
B Administer 40 cc of Maalox and observe for 1 hour.
C Send the patient for an abdominal ultrasounD.
D Order an abdominal CT scan.
ANSWER:D
970
A 41-year-old woman presents to the ED complaining of pain in her RUQ that is steady but
gets worse with eating over the past 2 days. The pain also radiates to the right side of her
midback. She denies vomiting. Her only medication is an oral contraceptivE. Her BP is
140/75 mm Hg, HR is 80 beatsm per minute, temperature is 98.7°F, and RR is 16 breaths per
minutE. Laboratory tests are within normal limits. An abdominal ultrasound reveals stones
in her gallbladder, but no thickened wall or pericholecystic fluiD. What is the most likely
diagnosis?

A Cholangitis
B Urolithiasis
C Cholecystitis
D Biliary colic
ANSWER:D
971
A 23-year-old woman presents to the ED in moderate pain in her left lower quadrant LLQ..
She states that the pain began suddenly and is associated with nausea and vomiting. She had
a bout of diarrhea yesterday. This is the second time this month that she experienced pain in
this location, however, never with this severity. Her BP is 120/75 mm Hg, HR is 101 beats
per minute, temperature is 99.5°F, and RR is 18 breaths per minutE. She has a tender LLQ
on abdominal examination and a tender adnexa on pelvic examination. Which of the
following is the most appropriate diagnostic test for the patient?
A CT scan
B MRI
C X-ray
D Doppler ultrasound
ANSWER:D
972
A 78-year-old woman is brought to the ED by EMS complaining of vomiting and
abdominal pain that began during the night. EMS reports that her BP is 90/50 mm Hg, HR is
110 beats per minute, temperature is 101.2°F, and RR is 18 breaths per minutE. After giving
her a 500-mL bolus of NS, her BP is 115/70 mm Hg. During the examination, you notice
that her face and chest appear jaundiceD. Her lungs are clear to auscultation and you do not
appreciate a murmur on cardiac examination. She winces when you palpate her RUQ. An
ultrasound reveals dilation of the common bile duct and stones in the gallbladder. What is the
most likely diagnosis?
A Cholecystitis
B Acute hepatitis
C Cholangitis
D Pancreatic cancer
ANSWER:C

973
A 23-year-old woman presents to the ED complaining of lower abdominal pain and vaginal
spotting for 2 days. Her menstrual cycle is irregular. She has a history of ovarian cysts and is
sexually active but always uses condoms. Her BP is 115/75 mm Hg, HR is 75 beats per
minute, temperature is 98.9°F, and RR is 16 breaths per minute. Which of the following tests
should be obtained next?
A Chlamydia antigen test
B β-Human chorionic gonadotropin β-hCG.
C Transvaginal ultrasound
D Abdominal radiograph
ANSWER:B
974
A 71-year-old obese man is brought to the ED complaining of constant left middle quadrant
abdominal pain with radiation into his back. His past medical history is significant for
hypertension, peripheral vascular disease, peptic ulcer disease, kidney stones, and gallstones.
He smokes a pack of cigarettes and consumes a pint of vodka daily. His BP is 145/80 mm
Hg, HR is 90 beats per minute, temperature is 98.9°F, and RR is 16 breaths per minutE.
Abdominal examination is unremarkablE. An ECG is read as sinus rhythm with a HR of 88
beats per minutE. An abdominal radiograph reveals normal loops of bowel and curvilinear
calcification of the aortic wall. Which of the following is the most likely diagnosis?
A Abdominal aortic aneurysm
B Nephrolithiasis
C Pancreatitis
D Small bowel obstruction SBO.
ANSWER:A
975
A 51-year-old man presents to the ED complaining of epigastric pain that radiates to his
back. He states that he drinks a 6 pack of beer daily. You suspect he has pancreatitis. His BP
is 135/75 mm Hg, HR is 90 beats per minute, temperature is 100.1°F, and his RR is 17
breaths per minutE. Laboratory results reveal WBC 13,000/μL, hematocrit 48%, platelets
110/μL, amylase 1150 U/L, lipase 1450 IU, lactate dehydrogenase LDH. 150 U/L, sodium
135 mEq/L, potassium 3.5 mEq/L, chloride 105 mEq/L, bicarbonate 23 mEq/L, BUN 15
mg/dL, creatinine 1.1 mg/dL, and glucose 125 mg/dL. Which of the following laboratory
values are most specific for pancreatitis?

A Elevated amylase
B Hyperglycemia
C Elevated lipase
D Elevated LDH
ANSWER:C
976
A 51-year-old man describes 1 week of gradually worsening scrotal pain and dysuriA. He is
sexually active with his wifE. His temperature is 100.1°F, HR 81 beats per minute, BP
140/75 mm Hg, and oxygen saturation is 99% on room air. On physical examination, his
scrotal skin is warm and erythematous. A cremasteric reflex is present. The posterior left
testicle is swollen and tender to touch. Color Doppler ultrasonography demonstrates
increased testicular blood flow. Urinalysis is positive for leukocyte esterasE. What is the
most likely diagnosis?
A Epididymitis
B Testicular torsion
C UTI
D Testicular tumor
ANSWER:A
977
A 22-year-old man presents to the ED complaining of dysuria for 3 days. He states that he
has never had this feeling beforE. He is currently sexually active and uses a condom most of
the timE. He denies hematuria but notes a yellowish discharge from his urethrA. His BP is
120/75 mm Hg, HR is 60 beats per minute, and temperature is 98.9°F. You send a clean
catch urinalysis to the laboratory that returns positive for leukocyte esterase and 15 white
blood cells per high power field WBCs/hpf.. Which of the following is the most appropriate
next step in management?
A
Send a urethral swab for culture and administer 125-mg ceftriaxone intramuscularly and 1-g
azithromycin orally.
B Send urine for culture and administer SMX/TMP orally.
C Discharge the patient with strict instructions to return if his symptoms worsen.
D Order a CT scan to evaluate for a kidney stonE.
ANSWER:A

978
A 40-year-old woman presents to the ED complaining of fever and 1 day of increasingly
severe pain in her RUQ. She denies nausea or vomiting and has no history of fatty food
intolerancE. The patient returned from a trip to Mexico 6 months ago. About 2 weeks ago
she experienced intermittent diarrhea with blood-streaked mucus. Her BP is 130/80 mm Hg,
HR is 107 beats per minute, temperature is 102°F, and RR is 17 breaths per minutE. Physical
examination reveals decreased breath sounds over the right lung basE. Abdominal
examination shows tenderness to percussion over the RUQ and normal active bowel sounds.
There is no Murphy sign. Her WBC is 20,500/μL. Chest radiograph reveals a small right-
pleural effusion. Which of the following is the most likely diagnosis?
A Amebic abscess
B Cholecystitis
C Cryptosporidium
D Enterobiasis
ANSWER:A
979
A 59-year-old woman presents to the ED complaining of worsening lower abdominal pain
over the previous 3 days. She describes feeling constipated recently and some burning when
she urinates. Her BP is 135/75 mm Hg, HR is 89 beats per minute, temperature is 101.2°F,
and her RR is 18 breaths per minutE. Her abdomen is mildly distended, tender in the LLQ,
and positive for rebound tenderness. CT scan is consistent with diverticulitis with a 7cm
abscess. Which of the following is the most appropriate management for this condition?
A Reserve the OR for emergent laparotomy.
B
Start treatment with ciprofloxacin and metronidazole and plan for CT-guided draining of
the abscess.
C Give an IV dose of ciprofloxacin and have the patient follow-up with her primary physician.
D
Start treatment with ciprofloxacin and metronidazole and plan for an emergent barium
enemA.
ANSWER:B

980
A 29-year-old man presents to the ED complaining of RLQ pain for 24 hours. He states that
the pain first began as a dull feeling around his umbilicus and slowly migrated to his right
sidE. He has no appetite, is nauseated, and vomited twicE. His BP is 130/75 mm Hg, HR is
95 beats per minute, temperature is 100.9°F, and his RR is 16 breaths per minutE. His WBC
is 14,000/μL. As you palpate the LLQ of the patient’s abdomen, he states that his RLQ is
painful. What is the name of this sign?
A Blumberg sign
B Rovsing sign
C Obturator sign
D Raynaud sign
ANSWER:B
981
A 60-year-old man is brought to the ED complaining of generalized crampy abdominal pain
that occurs in waves. He has been vomiting intermittently over the last 6 hours. His BP is
150/75 mm Hg, HR is 90 beats per minute, temperature is 99.8°F, and his RR is 16 breaths
per minutE. On abdominal examination you notice an old midline scar the length of his
abdomen that he states was from surgery after a gunshot wound as a teenager. The abdomen
is distended with hyperactive bowel sounds and mild tenderness without rebounD. An
abdominal plain film confirms your diagnosis. Which of the following is the most
appropriate next step in management?
A
Begin fluid resuscitation, bowel decompression with a nasogastric tube, and request a
surgical consult.
B
Begin fluid resuscitation, administer broad-spectrum antibiotics, and admit the patient to the
medical service.
C Begin fluid resuscitation, give the patient stool softener, and administer a rectal enema.
D
Begin fluid resuscitation, administer broad-spectrum antibiotics, and observe the patient for
24 hours.
ANSWER:A

982
A 73-year-old man who is a 1-pack-per-day smoker and has a medical history of
hypertension and peripheral vascular disease presents to the ED complaining of
midabdominal and right flank pain. He states that he had this same pain 1 week ago and that
it got so bad that he passed out. His BP is 125/75 mm Hg, HR is 85 beats per minute,
temperature is 98.7°F, and his RR is 17 breaths per minutE. Physical examination reveals a
bruit over his abdominal aorta and a pulsatile abdominal mass. Which of the following is the
most appropriate initial test to evaluate this patient?
A Angiography
B Ultrasound
C MRI
D Plain radiograph
ANSWER:B
983
A 25-year-old G3P1011 presents to the ED with a 6-hour history of worsening lower
abdominal pain, mostly in the RLQ. She also noticed some vaginal spotting this morning.
She is nauseated, but did not vomit. Her last menstrual period was 2 months ago, but her
cycles are irregular. She is sexually active and has a history of pelvic inflammatory diseasE.
Her BP is 120/75 mm Hg, HR is 95 beats per minute, temperature is 99.2°F, and RR is 16
breaths per minutE. Her abdomen is tender in the RLQ. Pelvic examination reveals right
adnexal tenderness. Her WBC count is slightly elevated and her β-hCG is positivE. After
establishing IV access, which of the following is the most appropriate next step in
management?
A Call the OR to prepare for laparoscopy
B Order an emergent CT scan of the abdomen
C Perform a transvaginal ultrasound
D Order a urinalysis
ANSWER:C

984
A 59-year-old man presents to the ED complaining of vomiting and sharp abdominal pain in
the epigastric area that began abruptly this afternoon. He describes feeling nauseated and has
no appetitE. Laboratory results reveal WBC 18,000/μL, hematocrit 48%, platelets 110/μL,
AST 275 U/L, ALT 125 U/L, alkaline phosphatase 75 U/L, amylase 1150 U/L, lipase 1450
IU, LDH 400 U/L, sodium 135 mEq/L, potassium 3.5 mEq/L, chloride 110 mEq/L,
bicarbonate 20 mEq/L, BUN 20 mg/dL, creatinine 1.5 mg/dL, and glucose 250 mg/dL.
Which of the following laboratory results correlate with the poorest prognosis?
A Amylase 950, lipase 1250, LDH 400
B Lipase 1250, LDH 400, bicarbonate 20
C Lipase 1250, creatinine 1.5, potassium 3.5
D WBC 18,000, LDH 400, glucose 250
ANSWER:D
985
A 19-year-old woman presents to the ED with 1 hour of acute onset progressively worsening
pain in her RLQ. She developed nausea shortly after the pain and vomited twice over the last
hour. She had similar but less severe pain 2 weeks ago that resolved spontaneously. Her BP is
123/78 mm Hg, HR is 99 beats per minute, temperature is 99.1°F, and her RR is 16 breaths
per minute. On physical examination, the patient appears uncomfortable, not moving on the
gurney. Her abdomen is nondistended, diffusely tender, worst in the RLQ. Pelvic
examination reveals a normal sized uterus and moderate right-sided adnexal tenderness.
Laboratory results reveal WBC 10,000/μL, hematocrit 38%, and a negative urinalysis and β-
hCG. Pelvic ultrasound reveals an enlarged right ovary with decreased flow. Which of the
following is the most appropriate management for this patient?
A Admit to the gynecology service for observation
B Administer IV antibiotics and operate once inflammation resolves
C Immediate laparoscopic surgery
D Order an abdominal CT
ANSWER:C

986
An 18-year-old woman presents to the ED complaining of acute onset of RLQ abdominal
pain. She also describes the loss of appetite over the last 12 hours, but denies nausea and
vomiting. Her BP is 124/77 mm Hg, HR is 110 beats per minute, temperature is 102.1°F, RR
is 16 breaths per minute, and oxygen saturation is 100% on room air. Abdominal
examination reveals lower abdominal tenderness bilaterally. On pelvic examination you elicit
cervical motion tenderness and note cervical exudates. Her WBC is 20,500/μL and β-hCG is
negative. Which of the following is the most appropriate next step in management?
A Bring her to the OR for an appendectomy
B Begin antibiotic therapy
C Perform a culdocentesis
D Bring her to the OR for immediate laparoscopy
ANSWER:B
987
A 25-year-old man presents to the ED complaining of dull periumbilical pain that migrated
to his RLQ over the last hour. He states that he has no appetite and vomited twicE. His BP is
125/75 mm Hg, HR is 87 beats per minute, temperature is 100.6°F, and RR is 16 breaths per
minutE. Laboratory results reveal WBC 11,000/μL, hematocrit 48%, platelets 170/μL. On
physical examination, the patient complains of pain when you flex his knee with internal
rotation at his hip. What is the name of this sign?
A Obturator
B Psoas
C Rovsing
D McBurney
ANSWER:A
988
A 27-year-old man is seen in the ED for a leak around a surgical G-tube that was placed 2
weeks ago and has been used for enteral feeding for 1 week. Inspection reveals the tube is
pulled out from the stoma, but is still in the cutaneous tissuE. The abdomen is soft and
nondistended and there are no signs of skin infection. Which of the following is the most
appropriate next step in management?
A
Insert a Foley catheter into the tract and aspiratE. If gastric contents are aspirated the tube
can be used for feeding.
B
Insert a Foley catheter into the tract, instill water-soluble contrast, and obtain an abdominal
radiograph prior to using for feeding.

C Remove the tube and admit the patient for observation.
D Remove the tube and immediately obtain a CT scan of the abdomen.
ANSWER:B
989
A 30-year-old man presents to the ED complaining of sudden onset of abdominal bloating
and back pain lasting for 2 days. The pain woke him up from sleep 2 nights ago. It radiates
from his back to his abdomen and down toward his scrotum. He is in severe pain and is
vomiting. His temperature is 101.2°F and HR is 107 beats per minutE. A CT scan reveals a 9-
mm obstructing stone of the left ureter with hydronephrosis. Urinalysis is positive for 2+
blood, 2+ leukocytes, 2+ nitrites, 40 to 50 WBCs, and many bacteria. You administer pain
medicine, antiemetics, and antibiotics. Which of the following is the most appropriate next
step in management?
A Admit for IV antibiotics and possible surgical removal of stonE.
B Observe in ED for another 6 hours to see if stone passes.
C Discharge with antibiotics and pain medicinE.
D Discharge patient with instructions to consume large amounts of water.
ANSWER:A
990 For which of the following patients is an abdominal CT scan contraindicated?
A
A 52-year-old man with abdominal pain after blunt trauma, negative focused assessment
with sonography for trauma FAST. examination, BP 125/78 mm Hg, and HR 109 beats per
minute
B A 22-year-old female with RLQ pain, negative β-hCG, temperature 100.6°F
C
A 45-year-old man with abdominal pain, temperature 100.5°F, WBC 11,200/μL, BP 110/70
mm Hg, HR 110 beats per minute, and lipase 250 IU
D
A 70-year-old man with abdominal pain, an 11-cm pulsatile mass in the epigastrium, BP of
70/50 mm Hg, and HR of 110 beats per minute
ANSWER:D
991
A 71-year-old woman presents to the ED with 12-hours of emesis and abdominal pain. Her
temperature is 101.2°F, BP is 100/79 mm Hg, and HR is 104 beats per minutE. Physical
examination reveals a tender 2 × 2.-cm bulge with erythema below the inguinal ligament
and abdominal distension. An occasional high-pitched bowel sound is heard. After placing
an IV line and nasogastric tube, which of the following is the most appropriate course of
management?

A Administer broad-spectrum antibiotics and then obtain a CT scan of abdomen
B Administer broad-spectrum antibiotics and attempt reduction
C Administer broad-spectrum antibiotics and prepare the patient for the OR
D Administer broad-spectrum antibiotics and obtain a plain radiograph
ANSWER:C
992
You are working in the ED on a Sunday afternoon when four people present with acute
onset vomiting and crampy abdominal pain. They were all at the same picnic and ate most
of the same foods. The vomiting began approximately 4 hours into the picniC. They deny
having any diarrheA. You believe they may have “food poisoning” so you place IV lines,
administer IV fluids, and observE. Over the next few hours, the patients begin to improve,
the vomiting stops and their abdominal pain resolves. Which of the following is the most
likely cause of their presentation?
A Scombroid fish poisoning
B Staphylococcal food poisoning
C Clostridium perfringens food poisoning
D Campylobacter
ANSWER:B
993
A 23-year-old woman presents to the ED with RLQ pain for the last 1 to 2 days. The pain is
associated with nausea, vomiting, diarrhea, anorexia, and a fever of 100.9°F. She also reports
dysuriA. The patient returned 1 month ago from a trip to Mexico. She is sexually active
with one partner but does not use contraception. She denies vaginal bleeding or dischargE.
Her last menstrual period was approximately 1 month ago. She has a history of
pyelonephritis. Based on the principles of emergency medicine, what are the three priority
considerations in the diagnosis of this patient?
A Perihepatitis, gastroenteritis, cystitis
B Ectopic pregnancy, appendicitis, pyelonephritis
C Pelvic inflammatory disease PID. , gastroenteritis, cystitis
D Ectopic pregnancy, PID, menstrual cramps
ANSWER:B

994
A 24-year-old woman presents to the ED after being sexually assaulteD. She is a college
student with no past medical history. Her BP is 130/75 mm Hg, HR is 91 beats per minute,
temperature is 98.6°F, and RR is 16 breaths per minute. On physical examination you
observe vaginal trauma and scattered bruising and abrasions. Which of the following
medications should be offered to the patient in this scenario?
A Ceftriaxone, azithromycin, metronidazole, antiretrovirals, emergency contraception
B Ceftriaxone, tetanus, metronidazole, antiretrovirals, emergency contraception
C Ceftriaxone, azithromycin, tetanus, metronidazole, emergency contraception
D Ceftriaxone, azithromycin, tetanus, metronidazole, antiretrovirals, emergency contraception
ANSWER:D
995
A 71-year-old man presents to the ED with diffuse, crampy abdominal pain that began 1
hour after eating lunch today. The pain is intermittent over the last 8 hours with increasing
severity. He also complains of nausea and chills, and vomited once on his way to the ED. He
has not had a bowel movement or flatus since the pain began. His past medical history
includes prostate cancer, left total hip replacement, appendectomy 25 years ago, right iliac
artery aneurysm repair 5 years ago, incisional hernia repair 4 years ago, and irritable bowel
syndromE. Which of the following is the most common cause of SBO in adults?
A Bezoar
B Adhesion
C Incarcerated hernia
D Gallstone ileus
ANSWER:B

996
An 18-year-old man presents to the ED with nausea and vomiting complaining of testicular
pain for the past hour that began while playing volleyball. He recalls having similar pain 1
week ago that resolved spontaneously after 10 minutes. He was recently well and reports no
fever, diarrhea, urinary frequency, or dysuriA. Physical examination reveals vital signs
within normal limits. The patient appears in moderate discomfort, holding his scrotum. His
abdomen is soft and nontender. His right hemiscrotum is swollen, erythematous, and
diffusely tender. It is not possible to palpate the testis separate from the epididymis. The right
cremasteric reflex is absent. His left testis has a horizontal lie and is nontender. You suspect
testicular torsion. What is the correct way to attempt manual detorsion?
A Elevate the painful testis until there is pain relief.
B Rotate the testes in a lateral to medial direction as if you were closing a book.
C Rotate the testes in a medial to lateral direction as if you were opening a book.
D Rotate the testes in an inferior to superior direction.
ANSWER:C
997
A 22-year-old woman is brought to the ED by ambulance complaining of sudden onset of
severe abdominal pain for 1 hour. The pain is in the RLQ and is not associated with nausea,
vomiting, fever, or diarrheA. On the pelvic examination you palpate a tender right adnexal
mass. The patient’s last menstrual period was 6 weeks ago. Her BP is 95/65 mm Hg, HR is
124 beats per minute, temperature is 99.8°F, and RR is 20 breaths per minutE. Which of the
following are the most appropriate next steps in management?
A Provide her oxygen via face mask and administer morphine sulfatE.
B
Administer morphine sulfate, order an abdominal CT with contrast, and call an emergent
surgery consult.
C Send the patient’s urine for analysis and order an abdominal CT.
D
Bolus 2-L NS, order a type and crossmatch and β-hCG, and call gynecology for possible
surgery.
ANSWER:D

998
A 33-year-old woman presents to the ED complaining of fever, vomiting, and gradually
worsening RUQ pain. She states that her pain radiates to her back. Her BP is 130/75 mm Hg,
HR is 90 beats per minute, temperature is 100.9°F, and RR is 17 breaths per minutE. While
examining her abdomen you palpate her RUQ and notice that she momentarily stops her
inspiration. What is the name of this classic sign?
A Grey-Turner sign
B Kernig sign
C McMurray sign
D Murphy sign
ANSWER:D
999
A 21-year-old girl presents to the ED complaining of diarrhea, abdominal cramps, fever,
anorexia, and weight loss for 3 days. Her BP is 127/75 mm Hg, HR is 91 beats per minute,
and temperature is 100.8°F. Her abdomen is soft and nontender without rebound or
guarding. WBC is 9200/μL, β-hCG is negative, urinalysis is unremarkable, and stool is
guaiac positivE. She tells you that she has had this similar presentation four times over the
past 2 months. Which of the following extraintestinal manifestations is associated with Crohn
disease but not ulcerative colitis?
A Ankylosing spondylitis
B Erythema nodosum
C Nephrolithiasis
D Thromboembolic disease
ANSWER:C
1000
A 23-year-old woman presents to the ED complaining of pain with urination. She has no
other complaints. Her symptoms started 3 weeks ago. During this time, she has been to the
clinic twice, with negative urine cultures each timE. Her condition has not improved with
antibiotic therapy with sulfonamides or quinolones. Physical examination is normal. Which
of the following organisms is most likely responsible for the patient’s symptoms?
A Staphylococcus aureus
B Herpes simplex virus
C Chlamydia trachomatis
D Escherichia coli
ANSWER:C

1001
A 43-year-old man presents to the ED complaining of progressively worsening abdominal
pain over the past 2 days. The pain is constant and radiates to his back. He also describes
nausea and vomiting and states he usually drinks 6 pack of beer daily, but has not had a
drink for 2 days. His BP is 144/75 mm Hg, HR is 101 beats per minute, temperature is
99.8°F, and RR is 14 breaths per minutE. He is lying on his side with his knees flexeD.
Examination shows voluntary guarding and tenderness to palpation of his epigastrium.
Laboratory results reveal WBC 10,500/μL, hematocrit 51%, platelets 225/μL, and lipase 620
IU. An abdominal radiograph reveals a nonspecific bowel gas pattern. There is no free air
under the diaphragm. Which of the following is the most appropriate next step in
management?
A Admit to the hospital for medical management and supportive care
B Send home with antibiotic therapy
C Admit to the hospital for endoscopy
D Admit to the hospital for exploratory laparotomy
ANSWER:A
1002
A 58-year-old woman is brought to the emergency department ED. by emergency medical
service EMS. after slipping on a patch of ice while walking to work and hitting her head on
the cement pavement. Bystanders acknowledged that the patient was unconscious for
approximately 1 minutE. On arrival, her vital signs are: blood pressure BP. 155/75 mm Hg,
heart rate HR. 89 beats per minute, respiratory rate RR. 18 breaths per minute, and pulse
oxygenation 98% on room air. She has a 5-cm laceration to the back of her head that is
actively bleeding. You ask the patient what happened but she cannot remember. You inform
her that she is in the hospital as a result of a fall. Over the next 10 minutes she asks you
repeatedly what happened and where she is. You do not find any focal neurologic deficits.
As you bring the patient to the CT scanner she vomits oncE. CT results show a normal brain
scan. Which of the following is the most likely diagnosis?
A Cerebral concussion
B Diffuse axonal injury
C Cerebral contusion
D Posttraumatic epilepsy
ANSWER:A

1003
A 41-year-old man, the restrained driver in a high-speed motor vehicle collision, is brought
to the ED by EMS. The patient is breathing without difficulty with bilateral and equal breaths
sounds. He has strong pulses peripherally indicating a BP of at least 90 mm Hg. The HR is
121 beats per minute. His Glasgow Coma Scale GCS. is 14. A secondary survey reveals
chest wall bruising. You suspect a cardiac injury. Which of the following locations most
commonly involve cardiac contusions?
A Right atrium
B Right ventricle
C Left atrium
D Left ventricle
ANSWER:B
1004
A 25-year-old man is brought into the trauma resuscitation room after his motorcycle is
struck by another vehiclE. EMS reports that the patient was found 20 ft away from his
motorcycle, which was badly damageD. His vital signs include a BP of 90/60 mm Hg, HR of
115 beats per minute, RR of 22 breaths per minute, and pulse oxygenation of 100% on
facemask. Which of the following is the smallest amount of blood loss that produces a
decrease in the systolic BP in adults?
A Loss of 5% of blood volume
B Loss of 10% of blood volume
C Loss of 15% to 30% of blood volume
D Loss of 30% to 40% of blood volume
ANSWER:D
1005
You are notified by the EMS dispatcher that there is a multiple-car collision on the local
highway with many injuries. He states that there are two people dead at the scene, one person
is critically injured and hypotensive, and three people have significant injuries, but with
stable vital signs. Which of the following is the leading cause of death and disability in
trauma victims?
A Abdominal injury
B Thoracic injury
C Back injury
D Head injury
ANSWER:D

1006
Paramedics bring a 17-year-old high school football player to the ED on a backboard and
with a cervical collar. During a football game, the patient “speared” another player with his
helmet and subsequently experienced severe neck pain. He denies paresthesias and is able to
move all of his extremities. A cervical spine CT scan reveals multiple fractures of the first
cervical vertebra. Which of the following best describes this fracture?
A Odontoid fracture
B Hangman’s fracture
C Jefferson fracture
D Clay-shoveler’s fracture
ANSWER:C
1007
A 20-year-old man presents to the ED with multiple stab wounds to his chest. His BP is 85/50
mm Hg and HR is 123 beats per minutE. Two large-bore IVs intravenous. are established
and running wide open. On examination, the patient is mumbling incomprehensibly, has
good air entry on lung examination, and you notice jugular venous distension. As you are
listening to his heart, the nurse calls out that the patient has lost his pulse and that she cannot
get a BP reading. Which of the following is the most appropriate next step in management?
A Atropine
B Epinephrine
C Bilateral chest tubes
D ED thoracotomy
ANSWER:D
1008
A 22-year-old man calls the ED from a local bar stating that he was punched in the face 10
minutes ago and is holding his front incisor tooth in his hand. He wants to know what is the
best way to preserve the tooth. Which of the following is the most appropriate advice to give
the caller?
A Place the tooth in a napkin and bring it to the ED
B Place the tooth in a glass of water and bring it to the ED
C Place the tooth in a glass of beer and bring it to the ED
D Pour some water over the tooth and place it immediately back into the socket
ANSWER:D

1009
A 19-year-old man is brought into the trauma room by EMS after a headon cycling accident.
The patient was not wearing a helmet. Upon presentation his BP is 125/75 mm Hg, HR is 105
beats per minute, RR is 19 breaths per minute, and oxygen saturation is 100% on mask. His
eyes are closed, but open to commanD. He can move his arms and legs on commanD.
When you ask him questions, he is disoriented but able to conversE. What is this patient’s
GCS score?
A 11
B 12
C 13
D 14
ANSWER:C
1010
An 18-year-old man presents to the ED after getting stabbed in his abdomen. His HR is 140
beats per minute and BP is 90/40 mm Hg. He is yelling that he is in pain. Two large-bore IVs
are inserted into his antecubital fossa and fluids are running wide open. After 2 L of fluids,
his BP does not improvE. Which of the following is the most common organ injured in stab
wounds?
A Liver
B Small bowel
C Stomach
D Colon
ANSWER:A
1011
A 61-year-old man presents to the ED with chest wall pain after a motor vehicle collision. He
is speaking full sentences, breath sounds are equal bilaterally, and his extremities are well-
perfuseD. His BP is 150/75 mm Hg, HR is 92 beats per minute, and oxygen saturation is
97% on room air. Chest radiography reveals fractures of the seventh and eighth ribs of the
right anterolateral chest. He has no other identifiable injuries. Which of the following is the
most appropriate treatment for this patient’s rib fractures?
A Apply adhesive tape on the chest wall perpendicular to the rib fractures
B Insert a chest tube into the right thorax
C Bring the patient to the OR for surgical fixation
D Analgesia and incentive spirometry
ANSWER:D

1012
A 27-year-old man bought to the ED by paramedics after a motor vehicle collision. His RR is
45 breaths per minute, oxygen saturation is 89%, HR is 112 beats per minute, and BP is
115/75 mm Hg. You auscultate his chest and hear decreased breath sounds on the left. Which
of the following is the most appropriate next step in management?
A Order a STAT chest radiograph
B Perform a pericardiocentesis
C Perform a tube thoracostomy
D Perform an ED thoracotomy
ANSWER:C
1013
A physician is deciding whether to use a new test to screen for disease X in his practice. The
prevalence of disease X is 5%. The sensitivity of the test is 85%, and the specificity is 75%.
In a population of 1000, how many patients will have the diagnosis of disease X missed by
this test?
A 50
B 42
C 8
D 4
ANSWER:C
1014
How many patients will be erroneously told they have diagnosis X on the basis of the results
of this test?
A 713
B 505
C 237
D 42
ANSWER:C
1015
Which type of health care delivery system encourages physicians to see more patients but to
provide fewer services?
A Capitation
B Fee-for-service
C Fixed salary compensation
D Out-of-pocket
ANSWER:A

1016
The curve that graphically represents the family of cutoff points for a positive vs. negative
test is a receiver operating characteristic (ROC) curve. The area under this curve is a
quantitative measure of the information content of a test. The ROC axes are
A negative predictive value vs. (1 – positive predictive value)
B positive predictive value vs. (1 – negative predictive value)
C sensitivity vs. (1 – specificity)
D specificity vs. (1 – sensitivity)
ANSWER:C
1017
A patient is seen in the clinic for evaluation of chest pain. The patient is 35 years old and has
no medical illnesses. She reports occasional intermittent chest pain that is unrelated to
exercise but is related to eating spicy food. The physician’s pretest probability for coronary
artery disease causing these symptoms is low; however, the patient is referred for an exercise
treadmill test, which shows ST depression after moderate exercise. Using Bayes’ theorem,
how does one interpret these test results?
A
The pretest probability is low, and the sensitivity and specificity of exercise treadmill testing
in females are poor; therefore, the exercise treadmill test is not helpful in clinical decision
making in this case.
B
Regardless of the pretest probability, the abnormal result of this exercise treadmill testing
requires further evaluation.
C
Because the pretest probability for coronary artery disease is low, the patient should be
referred for further testing to rule out this diagnosis.
D
Because the pretest probability was low in this case, a diagnostic test with a low sensitivity
and specificity is sufficient to rule out the diagnosis of coronary artery disease.
ANSWER:A
1018
An effective way to measure the accuracy of a diagnostic test is a positive likelihood ratio
[sensitivity/(1 – specificity)], which is also defined as the ratio of the probability of a positive
test result in a patient with disease to the probability of a positive test result in a patient
without disease. What other piece of information is needed along with a positive likelihood
ratio to estimate the possibility of a given disease in a certain patient with a positive test result?
A Disease prevalence in the patient’s geographic region
B Negative predictive value of the test

C Positive predictive value of the test
D Pretest probability of the disease in a patient
ANSWER:D
1019
Drug X is investigated in a meta-analysis for its effect on mortality after a myocardial
infarction. It is found that mortality drops from 10 to 2% when this drug is administered.
What is the absolute risk reduction conferred by drug X?
A 0,02
B 0,08
C 0,2
D 2
ANSWER:B
1020 How many patients will have to be treated with drug X to prevent one death?
A 2
B 8
C 12,5
D 50
ANSWER:C
1021
A healthy 23-year-old female is referred to your clinic after being seen in the emergency
department for intermittent severe chest pain. During her visit, she is ruled out for cardiac
ischemia, with negative biomarkers for cardiac ischemia and unremarkable
electrocardiograms. An exercise single photon emission CT (SPECT) myocardial perfusion
test was performed, and a reversible exercise-induced perfusion defect was noted. The test
was read as positive. The patient was placed on aspirin. She is quite concerned that she
continues to have chest pain intermittently on a daily basis without any consistency in
regards to time or antecedent activity. She is otherwise active and feeling well. She smokes
socially on weekends. She has no family history of early coronary disease. What would be
the best next course of action?
A Cardiac catheterization
B CT of her coronary arteries
C Dobutamine stress echocardiogram
D Evaluation for non-cardiac source of her chest pain
ANSWER:D

1022 Which of the following statements regarding gender health is true?
A Alzheimer’s disease affects men and women at equal rates.
B Alzheimer’s disease affects men two times more commonly than women.
C
In a recent placebo-controlled trial, postmenopausal hormone therapy did not show
improvement in disease progression in women with Alzheimer’s disease.
D
Women with Alzheimer’s disease have higher levels of circulating estrogen than women
without Alzheimer’s disease.
ANSWER:C
1023 All of the following statements regarding women’s health are true except
A
Coronary heart disease mortality rates have been falling in men over the past 30 years, while
increasing in women.
B
Women have longer QT intervals on resting ECG, predisposing them to higher rates of
ventricular arrhythmia.
C
Women are more likely than men to have atypical symptoms of angina such as nausea,
vomiting, and upper back pain.
D
Women with myocardial infarction (MI) are more likely to present with ventricular
tachycardia, whereas men are more likely to present with cardiogenic shock.
ANSWER:D
1024
When ordering an evaluation of coronary artery disease in a female patient, all of the
following are true except
A Exercise stress testing has more false positives in women than in men.
B Exercise stress testing has more false negatives in women than in men.
C
Women are less likely than men to undergo angioplasty and coronary artery bypass grafting
(CABG).
D
Women undergoing coronary artery bypass surgery have lower 5- and 10-year survival rates
than men.
ANSWER:D
1025 Which of the following statements regarding cardiovascular risk is true?
A Aspirin is effective as a means of primary prevention in women for coronary heart disease.
B
Cholesterol-lowering drugs are less effective in women than in men for primary and
secondary prevention of coronary heart disease.

C
Low high-density lipoprotein (HDL) and diabetes mellitus are more important risk factors
for men than for women for coronary heart disease.
D
Total triglyceride levels are an independent risk factor for coronary heart disease in women
but not in men.
ANSWER:D
1026
Which of the following alternative medicines has shown proven benefit compared to placebo
in a large randomized clinical trial?
A Echinacea root for respiratory infection
B Ginkgo biloba for improving cognition in the elderly
C
Glucosamine/chondroitin sulfate for improving performance and slowing narrowing of the
joint space in patients with moderate to severe osteoarthritis
D Saw palmetto for men with symptomatic benign prostatic hyperplasia (BPH)
ANSWER:C
1027
You prescribe an extended-release antihypertensive agent for your patient at a dosing
interval of 24 h. The half-life of the agent is 48 h. Three days later the patient’s blood
pressure is not controlled. At this point you should
A add a second agent
B double the dose of the current agent
C increase the frequency of the current dose to twice/day
D recheck the blood pressure in 1 week
ANSWER:D

1028
A 56-year-old patient arrives in your clinic with worsening somnolence, per his wife. You
have followed him for several years for his long-standing liver disease related to heavy
alcohol use in the past and hepatitis C infection, as well as chronic low back pain related to
trauma. He has recently developed ascites but has had a good response to diuretic therapy.
He has no history of gastrointestinal bleeding, he denies fever, chills, abdominal pain,
tremor, or any recent change in his medicines, which include furosemide, 40 mg daily;
spironolactone, 80 mg daily; and extended-release morphine, 30 mg twice a day. He is
afebrile with normal vital signs. His weight is down 5 kg since initiating diuretic therapy.
Physical examination is notable for a somnolent but conversant man with mild jaundice,
pinpoint pupils, palmar erythema, spider hemangiomas on his chest, a palpable nodular liver
edge at the costal margin, and bilateral 1+ lower extremity edema. He does not have asterixis,
abdominal tenderness, or an abdominal fluid wave. Laboratory results compared to 3 months
previously reveal an increased INR, from 1.4 to 2.1; elevated total bilirubin, from 1.8 to 3.6
mg/dL; and decreased albumin from 3.4 to 2.9 g/L; as well as baseline elevations of his
aspartate and alanine aminotransferases (54 U/L and 78 U/L, respectively). Serum NH4 is 16.
What would be a sensible next step for this patient?
A Decrease his morphine dose by 50% and reevaluate him in a few days
B Initiate antibiotic therapy
C Initiate haloperidol therapy
D Initiate lactulose therapy
ANSWER:A
1029
A homeless male is evaluated in the emergency department. He has noted that after he slept
outside during a particularly cold night his left foot has become clumsy and feels “dead.” On
examination, the foot has hemorrhagic vesicles distributed throughout the foot distal to the
ankle. The foot is cool and has no sensation to pain or temperature. The right foot is
hyperemic but does not have vesicles and has normal sensation. The remainder of the
physical examination is normal. Which of the following statements regarding the
management of this disorder is true?
A Active foot rewarming should not be attempted.
B During the period of rewarming, intense pain can be anticipated.
C Heparin has been shown to improve outcomes in this disorder.
D Immediate amputation is indicated.

ANSWER:B
1030
A 78-year-old female is seen in the clinic with complaints of urinary incontinence for several
months. She finds that she is unable to hold her urine at random times throughout the day;
this is not related to coughing or sneezing. The leakage is preceded by an intense need to
empty the bladder. She has no pain associated with these episodes, though she finds them
very distressing. The patient is otherwise independent in the activities of daily living, with
continued ability to cook and clean for herself. Which of the following statements is true?
A The abrupt onset of similar symptoms should prompt cystoscopy.
B First-line therapy for this condition consists of desmopressin.
C Indwelling catheters are rarely indicated for this disorder.
D Referral to a genitourinary surgeon is indicated for surgical correction.
ANSWER:C
1031
All of the following statements regarding medications in the geriatric population are true
except
A
Falling albumin levels in the elderly lead to increased free (active) levels of some
medications, including warfarin.
B Fat-soluble drugs have a shorter half-life in geriatric patients.
C Hepatic clearance decreases with age.
D
The elderly have a decreased volume of distribution for many medications because of a
decrease in total body water.
ANSWER:B
1032
Which of the following class of medicines has been linked to the occurrence of hip fractures
in the elderly?
A Benzodiazepines
B Opiates
C Angiotensin-converting enzyme inhibitors
D Beta blockers
ANSWER:A
1033
Patients taking which of the following drugs should be advised to avoid drinking grapefruit
juice?
A Amoxicillin

B Aspirin
C Atorvastatin
D Prevacid
ANSWER:C
1034
A recent 18-year-old immigrant from Kenya presents to a university clinic with fever, nasal
congestion, severe fatigue, and a rash. The rash started with discrete lesions at the hairline
that coalesced as the rash spread caudally. There is sparing of the palms and soles. Small
white spots with a surrounding red halo are noted on examination of the palate. The patient
is at risk for developing which of the following in the future?
A Encephalitis
B Epiglottitis
C Opportunistic infections
D Postherpetic neuralgia
ANSWER:A
1035
You are a physician working in an urban emergency department when several patients are
brought in after the release of an unknown gas at the performance of a symphony. You are
evaluating a 52-year-old female who is not able to talk clearly because of excessive
salivation and rhinorrhea, although she is able to tell you that she feels as if she lost her sight
immediately upon exposure. At present, she also has nausea, vomiting, diarrhea, and muscle
twitching. On physical examination the patient has a blood pressure of 156/92, a heart rate of
92, a respiratory rate of 30, and a temperature of 37.4°C (99.3°F). She has pinpoint pupils
with profuse rhinorrhea and salivation. She also is coughing profusely, with production of
copious amounts of clear secretions. A lung examination reveals wheezing on expiration in
bilateral lung fields. The patient has a regular rate and rhythm with normal heart sounds.
Bowel sounds are hyperactive, but the abdomen is not tender. She is having diffuse
fasciculations. At the end of your examination, the patient abruptly develops tonic-clonic
seizures. Which of the following agents is most likely to cause this patient’s symptoms?
A Arsine
B Cyanogen chloride
C Nitrogen mustard
D Sarin

ANSWER:D
1036 All the following should be used in the treatment of this patient except
A atropine
B decontamination
C diazepam
D phenytoin
ANSWER:D
1037
A 24-year-old male is brought to the emergency department after taking cyanide in a suicide
attempt. He is unconscious on presentation. What drug should be used as an antidote?
A Atropine
B Methylene blue
C Sodium nitrite with sodium thiosulfate
D Sodium nitrite alone
ANSWER:C
1038
A 40-year-old female is exposed to mustard gas during a terrorist bombing of her office
building. She presents to the emergency department immediately after exposure without
complaint. The physical examination is normal. What is the next step?
A
Admit the patient for observation because symptoms are delayed 2 h to 2 days after exposure
and treat supportively as needed.
B Administer 2-pralidoxime as an antidote and observe for symptoms.
C
Irrigate the patient’s eyes and apply ocular glucocorticoids to prevent symptoms from
developing.
D Discharge the patient to home as she is unlikely to develop symptoms later.
ANSWER:A

1039
A 24-year-old healthy man who has just returned from a 1-week summer camping trip to the
Ozarks presents to the emergency room with fever, a severe headache, mild abdominal pain,
and severe myalgias. He is discharged home but 1 day later feels even worse and therefore
returns. Temperature is 38.4°C; heart rate is 113 beats/min; blood pressure is 120/ 70.
Physical examination is notable for a well-developed, well-nourished, but diaphoretic and
distressed man. He is alert and oriented to time and place. His lungs are clear to auscultation.
He has no heart murmur. His abdomen is mildly tender with normal bowel sounds.
Neurologic examination is nonfocal. There is no evidence of a rash. Laboratory evaluation is
notable for a platelet count of 84,000/µL. A lumbar puncture is notable for 5 monocytes, no
red blood cells, normal protein levels, and normal glucose levels. What should be the next
step in this patient’s management?
A Atovaquone
B Blood cultures and observation
C Doxycycline
D Rimantadine
ANSWER:C

1040
A 23-year-old woman with a chronic lower extremity ulcer related to prior trauma presents
with rash, hypotension, and fever. She has had no recent travel or outdoor exposure and is
up to date on all of her vaccinations. She
does not use IV drugs. On examination, the ulcer looks clean with a well-granulated base and
no erythema, warmth, or pustular discharge. However, the patient does have diffuse
erythema that is most prominent on her palms, conjunctiva, and oral mucosa. Other than
profound hypotension and tachycardia, the remainder of the examination is nonfocal.
Laboratory results are notable for a creatinine of 2.8 mg/dL, aspartate aminotransferase of
250 U/L, alanine aminotransferase of 328 U/L, total bilirubin of 3.2 mg/dL, direct bilirubin
of 0.5 mg/dL, INR
of 1.5, activated partial thromboplastin time of 1.6 × control,
and platelets at 94,000/μL. Ferritin is 1300 μg/mL.
The patient is started on broad-spectrum antibiotics after appropriate blood cultures are
drawn and is resuscitated
with IV fluid and vasopressors. Her blood cultures are
negative at 72 h: at this point her fingertips start to
desquamate. What is the most likely diagnosis?
A Juvenile rheumatoid arthritis (JRA)
B Leptospirosis
C Staphylococcal toxic shock syndrome
D Streptococcal toxic shock syndrome
ANSWER:C
1041
The Centers for Disease Control and Prevention (CDC) has designated several biologic agents
as category A in their ability to be used as bioweapons. Category A agents include agents that
can be easily disseminated or transmitted, result in high mortality, can cause public panic,
and require special action for public health preparedness. All the following agents are
considered category A except
A Bacillus anthracis
B Francisella tularensis
C ricin toxin from Ricinus communis
D smallpox
ANSWER:C

1042
A 50-year-old alcoholic woman with well-controlled cirrhosis eats raw oysters from the
Chesapeake Bay at a cookout. Twelve hours later she presents to the emergency department
with fever, hypotension, and altered sensorium. Her extremity examination is notable for
diffuse erythema with areas of hemorrhagic bullae on her shins. What is the most likely
diagnosis?
A Escherichia coli sepsis
B Hemolytic uremic syndrome
C Meningococcemia
D Vibrio vulnificus infection
ANSWER:D
1043 Hyperthermia is defined as
A a core temperature >40.0°C
B a core temperature >41.5°C
C
an uncontrolled increase in body temperature despite a normal hypothalamic temperature
setting
D an elevated temperature that normalizes with antipyretic therapy
ANSWER:C
1044
A patient in the intensive care unit develops a temperature of 40.8°C, profoundly rigid tone,
and hemodynamic shock 2 min after a succinylcholine infusion is started. Immediate therapy
should include
A intravenous dantrolene sodium
B acetaminophen
C external cooling devices
D A & C
ANSWER:D
1045
A 68-year-old alcoholic arrives in the emergency department after being found in the snow
on a cold winter night in Chicago. His core temperature based on rectal and esophageal
probe is 27°C. Pulse is 30 beats/min and blood pressure is 75/40 mmHg. He is immobile and
lacks corneal, oculocephalic, and peripheral reflexes. He is immediately intubated and placed
on a cardiac monitor. He then converts to ventricular fibrillation: a defibrillation attempt at 2
J/kg is not successful. What should be the next immediate step in management?

A
Active rewarming with forced-air heating blankets, heated humidified oxygen, heated
crystalloid infusion
B Amiodarone infusion
C Insertion of a transvenous pacemaker
D Passive rewarming with numerous blankets for insulation
ANSWER:A
1046
In the evaluation of malnutrition, which of the following proteins has the shortest half-life
and thus is most predictive of recent nutritional status?
A Albumin
B Fibronectin
C Retinol-binding protein complex
D Prealbumin
ANSWER:B
1047
Fecal occult blood testing (FOBT) was shown to decrease colon cancer–related mortality
from 8.8/1000 persons to 5.9/1000 persons over a 13-year period. What is the approximate
absolute risk reduction (ARR) of this intervention in the studied population?
A 0,5
B 0,3
C 0,03
D 0,003
ANSWER:D
1048
Which preventative intervention leads to the largest average increase in life expectancy for a
target population?
A A regular exercise program for a 40-year-old man
B Getting a 35-year-old smoker to quit smoking
C Mammography in women aged 50–70
D Pap smears in women aged 18–65
ANSWER:B
1049 All of the following patients should receive a lipid screening profile except
A a 16-year-old male with type 1 diabetes
B a17-year-old female teen who recently began smoking
C a 23-year-old healthy male who is starting his first job

D a 48-year-old woman beginning menopause
ANSWER:B
1050
A 46-year-old female presents to her primary care doctor complaining of a feeling of
anxiety. She notes that she always had been what she describes as a “worrier,” even in grade
school. The patient has always avoided speaking in public and recently is becoming anxious
to the extent where she is having difficulty functioning at work and in social situations. She
has difficulty falling asleep at night and finds that she is always “fidgety” and has a
compulsive urge to move. The patient owns a real estate company that has been in decline
since a downturn in the local economy. She recently has been avoiding showing homes for
sale. Instead, she defers to her partners because she finds that she is nervous to the point of
being unable to speak to her clients. She has two children, ages 16 and 12, who are very
active in sports. She feels overwhelmed with worry over the possibility of injury to her
children and will not attend their sports events. You suspect that the patient has a generalized
anxiety disorder. All of the following statements regarding this diagnosis are true except
A
The age at onset of symptoms is usually before 20 years, although the diagnosis usually
occurs much later in life.
B
Over 80% of these patients will have concomitant mood disorders such as major depression,
dysthymia, or social phobia.
C As in panic disorder, shortness of breath, tachycardia, and palpitations are common.
D
Experimental work suggests that the pathophysiology of generalized anxiety disorder
involves impaired binding of benzodiazepines at the γ-aminobutyric acid (GABA) receptor.
ANSWER:C
1051
For which of the following herbal remedies is there the best evidence for efficacy in treating
the symptoms of benign prostatic hypertrophy?
A Saint John’s wort
B Gingko
C Kava
D Saw palmetto
ANSWER:D

1052
Which of the following personality traits is most likely to describe a young female with
anorexia nervosa?
A Depressive
B Borderline
C Anxious
D Perfectionist
ANSWER:D
1053 Why is it necessary to coadminister vitamin B6 (pyridoxine) with isoniazid?
A Vitamin B6 requirements are higher in tuberculosis patients.
B Isoniazid causes decarboxylation of γ-carboxyl groups in vitamin K–dependent enzymes.
C Isoniazid interacts with pyridoxal phosphate.
D Isoniazid causes malabsorption of vitamin B6.
ANSWER:C
1054 The prevalence of hypertension in American persons aged >65 years old is
A 10–35%
B 35–60%
C 60–85%
D >85%
ANSWER:C
1055 Diabetes is associated with all of the following in the elderly except
A cerebrovascular accident
B cognitive decline
C fall risk
D myocardial infarction
ANSWER:B
1056 Which of the following is the best indicator of prognosis and longevity in a geriatric patient?
A Functional status
B Life span of first-degree relatives
C Marital status
D Number of medical comorbidities
ANSWER:A

1057
Diagnostic criteria for delirium as a cause of a confused state in a hospitalized patient include
all of the following except
A agitation
B altered level of consciousness
C disorganized thinking
D fluctuating mental status
ANSWER:A
1058 Fall risks in the elderly include all of the following except
A creatinine clearance <65 mL/min
B diabetes mellitus
C fear of falling
D Hypertension
ANSWER:D
1059
A stage 1 decubitus ulcer (nonblanchable erythema of intact skin or edema and induration
over a bony pressure point) can progress to a stage 4 decubitus ulcer (full-thickness skin loss
with tissue necrosis as well as damage to bone, muscle and tendons) over what period of
time?
A Several hours
B 1–2 days
C 1–2 weeks
D 1–2 months
ANSWER:B
1060
A 74-year-old woman complains of leaking urine when she coughs, laughs, or lifts her
groceries. She denies polydipsia and polyuria. She delivered four children vaginally and
underwent total abdominal hysterectomy for fibroids 20 years earlier. She has mild fasting
hyperglycemia that is controlled with diet. What is likely to be the best management for her
problem?
A Bladder retraining exercises (planned urinations every 2 h)
B Doxazosin plus finasteride
C Metformin
D Surgery
ANSWER:D

1061
A 38-year-old man with multiple sclerosis develops acute flaccid weakness in his left arm
and left leg. Physical examination reveals normal sensorium, normal cranial nerve function,
1/5 strength in his left upper extremity, 0/5 strength in his left lower extremity, impaired
proprioception in his left leg, intact proprioception in his right leg, decreased pain and
temperature sensation in his right arm and leg, and normal light touch/pain and temperature
sensation in his right leg. Where is his causative lesion most likely to be?
A Cervical nerve roots
B High cervical spinal cord
C Medulla
D Pons
ANSWER:B
1062
A 32-year-old man with a history of HIV infection presents to the hospital with nausea,
abdominal distention and projectile vomiting that developed over the previous 8–12 h. He
denies fevers, chills, diaphoresis, melena, or diarrhea. Over the past 3 months, he has lost 30
lb in the context of advanced HIV infection. He has never had abdominal surgery. On
examination, his abdomen is distended, with high-pitched intermittent bowel sounds and
guarding but no rebound. A periumbilical bruit is also detected. Abdominal x-ray reveals a
smallbowel obstruction with a probable cut-off point in the mid duodenum. What is the
diagnostic test of choice for diagnosing the cause of the underlying obstruction?
A Abdominal CT with abdominal angiogram
B Enteroscopy
C Laparoscopy
D Serum carcinoembryonic antigen (CEA) level
ANSWER:A
1063
A 64-year-old man with primary light chain amyloidosis develops orthostatic symptoms
despite maintaining adequate oral intake. He also notes early satiety, with bloating and
vomiting if he eats too rapidly. To combat this, he has decreased the size of his meals but eats
twice as frequently during the day, with some positive effect. What is the most likely
explanation for his gastrointestinal symptoms?
A Diverticulosis
B Gastric cancer

C Gastroparesis
D Irritable bowel syndrome
ANSWER:C
1064
A 42-year-old man with a history of end-stage renal disease is on hemodialysis and has been
taking a medication chronically for nausea and vomiting. Over the past week he has
developed new-onset involuntary lip smacking, grimacing, and tongue protrusion. This side
effect is most likely due to which of the following antiemetics?
A Erythromycin
B Methylprednisolone
C Ondansetron
D Prochlorperazine
ANSWER:D
1065
Which of the following is not a common cause of persistent cough lasting more than 3
months in a nonsmoker?
A Asthma
B Gastroesophageal reflux disease
C Lisinopril
D Mycoplasma infection
ANSWER:D
1066
A 74-year-old man with known endobronchial carcinoma of his left mainstem bronchus
develops massive hemoptysis (1 L of frank hemoptysis productive of bright red blood) while
hospitalized. All of the following should be considered in his initial management except
A bronchial artery embolization
B cough suppressants
C direct bronchoscopic electrocautery
D placing the patient in the lateral decubitus position with his right side down
ANSWER:D
1067
A patient with proteinuria has a renal biopsy that reveals segmental collapse of the
glomerular capillary loops and overlying podocyte hyperplasia. The patient most likely has
A diabetes

B HIV infection
C multiple myeloma
D systemic lupus erythematosus
ANSWER:B
1068
A 35-year-old woman comes to your clinic complaining of shortness of breath. It is
immediately apparent that she has a bluish tinge of her face, trunk, extremities, and mucus
membranes. Which of the following diagnoses is most likely?
A Atrial septal defect
B Myocarditis
C Raynaud’s phenomenon
D Sepsis
ANSWER:A
1069
A 43-year-old man with alcoholic liver disease complains of dyspnea upon sitting up.
Physical examination is notable for chest spider angiomas and palmar erythema. His arterial
oxygen saturations fall from 96% to 88% upon transition from lying to sitting. His lung
fields are clear and heart sounds are crisp. Abdominal examination is notable for a palpable
nodular liver edge but no fluid wave or shifting dullness. He has 1+ lower extremity edema.
What is the most likely cause of his dyspnea?
A Chronic thromboembolic disease
B Congestive heart failure
C Pulmonary arteriovenous fistula
D Portal hypertension
ANSWER:C
1070
A 30-year-old woman complains of lower extremity swelling and abdominal distention. It is
particularly troublesome after her daily shift as a toll booth operator and is at its worst during
hot weather. She denies shortness of breath, orthopnea, dyspnea on exertion, jaundice,
foamy urine, or diarrhea. Her symptoms occur independently of her menstrual cycle.
Physical examination is notable for 2+ lower extremity edema, flat jugular venous pulsation,
no hepatojugular reflex, normal S1 and S2 with no extra heart sounds, clear lung fields, a
benign slightly distended abdomen with no organomegaly, and normal skin. A complete
metabolic panel is within normal limits, and a urinalysis shows no proteinuria. What is the
most likely diagnosis?

A Cirrhosis
B Congestive heart failure
C Cyclical edema
D Idiopathic edema
ANSWER:D
1071
All the following factors are associated with a greater risk of ventricular arrhythmia versus
anxiety/panic attack in a patient complaining of palpitations except
A history of congestive heart failure
B history of coronary artery disease
C history of diabetes mellitus
D palpitations lasting >15 min
ANSWER:D
1072
A 25-year-old healthy woman visits your office during the fifth month of pregnancy. Her
blood pressure is 142/86 mmHg. What should be your next step in management?
A Have her return to your clinic in 2 weeks for a blood pressure check
B Initiate an angiotensin-converting enzyme inhibitor
C Initiate a beta blocker
D Recheck her blood pressure in the seated position in 6 h
ANSWER:D
1073
A 33-year-old woman with diabetes mellitus and hypertension presents to the hospital with
seizures during week 37 of her pregnancy. Her blood pressure is 156/92 mmHg. She has 4+
proteinuria. Management should include all of the following except
A emergent delivery
B intravenous labetalol
C intravenous magnesium sulfate
D intravenous phenytoin
ANSWER:D
1074 Which cardiac valvular disorder is the most likely to cause death during pregnancy?
A Aortic regurgitation
B Aortic stenosis
C Mitral regurgitation
D Mitral stenosis

ANSWER:D
1075
A 27-year-old woman develops left leg swelling during week 20 of her pregnancy. Left
lower extremity ultrasonogram reveals a left iliac vein deep vein thrombosis (DVT). Proper
management includes
A bedrest
B catheter-directed thrombolysis
C enoxaparin
D inferior vena cava filter placement
ANSWER:C
1076
All of the following should be components of the routine evaluation of any patient
undergoing medium- or high-risk non-cardiac surgery except
A 12-lead resting electrocardiogram
B chest radiograph
C detailed history
D physical examination
ANSWER:B
1077
Noninvasive cardiac imaging/stress testing should be considered in patients with how many
of the following six proven risk factors (high-risk surgery, ischemic heart disease, congestive
heart failure, cerebrovascular disease, diabetes mellitus, and renal insufficiency) for
perioperative cardiac events (including pulmonary edema, myocardial infarction, and heart
block)?
A 1
B 2
C 3
D 4
ANSWER:C

1078
A 72-year-old white man with New York Heart Association II ischemic cardiomyopathy,
diabetes mellitus, and chronic renal insufficiency (creatinine clearance = 42 mL/ min)
undergoes dobutamine echocardiography prior to carotid endarterectomy. He is found to
have 7-mm ST depressions in his lateral leads during the test and develops dyspnea at 70%
maximal expected dosage, requiring early cessation of the stress test. His current medicines
include an angiotensin-converting enzyme inhibitor, a beta blocker, and aspirin. What would
be your advice to the patient?
A Cancel the carotid endarterectomy
B Proceed to cardiac catheterization
C Maximize medical management
D Proceed directly to carotid endarterectomy
ANSWER:B
1079
Parkinson’s disease can often be differentiated from the atypical Parkinsonian syndromes
(multiple system atrophy, progressive supranuclear palsy) by the presence of which of the
following?
A Axial stiffness
B Pill-rolling tremor
C Shuffling gait
D Stooped posture
ANSWER:B
1080
A wide-based gait with irregular lurching and erratic foot placement but no subjective
dizziness characterizes which type of gait ataxia?
A Cerebellar dysfunction
B Frontal gait abnormality
C Inner ear dysfunction
D Parkinsonian syndromes
ANSWER:A

1081
A patient with a narrow-based gait instability complains that he needs to look at his feet while
he walks to prevent falling. He feels wobbly standing with his eyes closed and notes frequent
falls. On examination, he has no difficulty initiating gait, his stride is regular, strength is
normal, and there is no tremor. Review of routine blood work drawn 3 months prior reveals
a hematocrit of 29% with an elevated mean corpuscular volume. Which of the following is
the most likely diagnosis?
A Amyotrophic lateral sclerosis
B Cerebellar tumor
C Pernicious anemia
D Parkinson’s disease
ANSWER:C
1082
A 23-year-old female patient complains of visual blurriness. On examination, her pupils are
equally round. Shining a flashlight into her right eye causes equal, strong constriction in
both of her eyes. When the light is flashed into her left eye, both pupils dilate slightly though
not to their previous size prior to light confrontation. Where is there most likely to be
anatomic damage?
A Left cornea
B Left optic nerve or retina
C Optic chiasm
D Right cornea
ANSWER:B
1083
A patient complains of blurred vision in both eyes particularly in the periphery with the right
being worse than the left. Visual field examination with finger confrontation reveals a
decreased vision in the left periphery in the left eye and right periphery in the right eye.
Where is there most likely to be a lesion?
A Bilateral optic nerves
B Left lateral geniculate body
C Left occipital cortex
D Suprasellar space
ANSWER:D
1084 Which of the following methods is most effective for the diagnosis of corneal abrasions?
A Fluorescein and cobalt-blue light examination

B Intraocular pressure measurement
C Lid eversion for foreign body examination
D Oculoplegia and dilation
ANSWER:A
1085 Which of the following criteria best differentiates episcleritis from conjunctivitis?
A Concurrent connective tissue disease such as lupus
B Lack of discharge
C More diffuse ocular involvement
D Reduced eye motility
ANSWER:B
1086
Which diagnosis can be easily confused with adenoviral conjunctivitis and is a major cause
of blindness in the United States?
A Endophthalmitis
B Herpes simplex virus keratitis
C Angle-closure glaucoma
D Uveitis
ANSWER:B
1087
A 34-year-old male patient is referred to your clinic after a new diagnosis of anterior uveitis.
All of the following diseases should be screened for by history and physical and/or
laboratory examination because they may cause anterior uveitis except
A ankylosing spondylitis
B Lyme disease
C sarcoidosis
D toxoplasmosis
ANSWER:D
1088
A 22-year-old female is referred to your clinic after being started on glucocorticoids for a
new diagnosis of left optic neuritis seen on examination with disc pallor, and it is confirmed
with quantitative visual field mapping. What further evaluation is indicated?
A Antinuclear antibodies
B Brain MRI
C Erythrocyte sedimentation rate
D No further evaluation unless symptoms recur

ANSWER:B
1089
A 69-year-old male dialysis patient with poorly controlled diabetes, heart failure and chronic
indwelling catheters presents with fever and loss of vision in the left eye developing over the
past 6 h. Vital signs are notable for a temperature of 101.3°F, heart rate of 105/min, and
blood pressure of 125/85. Which test is most likely to confirm the diagnosis?
A Blood cultures
B Blood smear
C Brain MRI
D Rheumatic panel
ANSWER:A
1090
Exposure to which of the following types of radiation would result in thermal injury and
burns but would not cause damage to internal organs because the particle size is too large to
cause internal penetration?
A Alpha radiation
B Beta radiation
C Gamma radiation
D Neutron particles
ANSWER:B
1091
A “dirty” bomb is detonated in downtown Boston. The bomb was composed of cesium-137
with trinitrotoluene. In the immediate aftermath, an estimated 30 people were killed due to
the power of the blast. The fallout area was about 0.5 mile, with radiation exposure of ~1.8
gray (Gy). An estimated 5000 people have been potentially exposed to beta and gamma
radiation. Most of these individuals show no sign of any injury, but about 60 people have
evidence of thermal injury. What is the most appropriate approach to treating the injured
victims?
A All individuals who have been exposed should be treated with potassium iodide.
B All individuals who have been exposed should be treated with Prussian blue.
C
All individuals should be decontaminated prior to transportation to the nearest medical center
for emergency care to prevent exposure of health care workers.
D
Severely injured individuals should be transported to the hospital for emergency care after
removing the victims’ clothes, as the risk of exposure to health care workers is low.

ANSWER:D
1092
A 54-year-old man is admitted to the hospital with severe nausea, vomiting, and diarrhea.
These symptoms began 36 h ago. He briefly improved for a few hours yesterday, but today
has progressively worsened. He states he is concerned about possible poisoning because of
his role in espionage and counterterrorism for the U.S. government. He met with an
informant 2 days previously at a hotel bar, where he drank three cups of coffee but did not
eat. He does state that he left the table to place a phone call during the meeting and is
concerned that his coffee may have been contaminated. He otherwise is quite healthy and
takes no medications. On physical examination, he appears ill. The vital signs are: blood
pressure 98/60 mmHg, heart rate 112 beats/min, respiratory rate 24 breaths/min, SaO2 94%,
and temperature 37.4°C. Head, ears, eyes, nose, and throat examination shows pale mucous
membranes. Cardiovascular examination is tachycardic, but regular. His lungs are clear. The
abdomen is slightly distended with hyperactive bowel sounds. There is no tenderness or
rebound. Extremities show no edema, but a few scattered petechiae are present. Neurologic
examination is normal. A complete blood count is performed. The results are: white blood
cell (WBC) count 150/µL, red blood cell count 1.5/µL, hemoglobin 4.5 g/dL, hematocrit
15%, platelet count 11,000/µL. The differential on the WBC count is 98% PMNs, 2%
monocytes, and 0% lymphocytes. A blood sample is held for HLA testing. A urine sample is
positive for the presence of radioactive isotopes, which are determined to be polonium-210,
a strong emitter of alpha radiation. The mode of exposure is presumed to be ingestion. What
is the best approach to the treatment of this patient?
A Bone marrow transplantation
B Gastric lavage
C Supportive care and dimercaprol
D Supportive care only
ANSWER:C

1093
Several victims are brought to the emergency room after a terrorist attack in the train station.
An explosive was detonated that dispersed an unknown substance throughout the station, but
several people reported a smell like that of horseradish or burned garlic. Prior to transport to
the emergency room, exposed individuals had their clothing removed and underwent
showering and decontamination. On initial presentation, there was no apparent injury except
eye irritation. Over the next few hours, most of those exposed complain of nasal congestion,
sinus pain, and burning in the nares. Beginning about 2 h after exposure, many of the
exposed individuals began to notice diffuse redness of the skin, particularly in the neck,
axillae, antecubital fossae, and external genitalia. In addition, a few people also developed
blistering of the skin. Hoarseness, cough, and dyspnea are noted as well. What is the most
likely chemical agent that was released in the terrorist attack?
A Chlorine
B Cyanide
C Mustard gas
D Phosgene oxime
ANSWER:C
1094
A 7-month-old child is brought to clinic by his parents. He was the product of a healthy
pregnancy, and there were no perinatal complications. The parents are concerned that there
is something wrong; he is very hyperactive and is noted to have a ‘mousy’ odor. On
examination the child is found to have mild microcephaly, hypopigmentation and eczema.
Laboratory studies are sent and a diagnosis is made. How could this clinical scenario have
been prevented?
A Screening at 6 months of age for urine ketones
B Screening at birth for phenylalanine in blood
C Screening at birth for chromosomal abnormalities
D Genetic screening of parents prior to delivery
ANSWER:B
1095
A 35-year-old woman with a history of degenerative joint disease comes to clinic
complaining of dark urine over the past several weeks. She has had arthritis in her hips,
knees, and shoulders for about 2 years. On examination, she is noted to have gray-brown
pigmentation of the helices of both ears. Which of the following disorders is most likely?

A Alkaptonuria
B Hawkinsinuria
C Homocystinuria
D Hyperprolinemia type I
ANSWER:A
1096
A 22-year-old man presents to a local emergency room with severe muscle cramps and
exercise intolerance. His symptoms have been worsening over a period of months. He has
noticed that his urine is frequently dark. Examination reveals tenderness over all major
muscle groups. A creatine phosphokinase (CK) is markedly elevated. He reports a normal
childhood but since age 18 has noticed worsening exercise intolerance. He no longer plays
basketball and recently noticed leg fatigue at two flights of stairs. After intense exercise, he
occasionally has red-colored urine. Which of the following is the most likely diagnosis?
A Glucose-6-phosphatase deficiency
B Lactate dehydrogenase deficiency
C McArdle disease (type V glycogen storage disease)
D Pyruvate kinase deficiency
ANSWER:C
1097
An enzymatic assay of muscle tissue is sent and a diagnosis is made. Which of the following
represents a major source of morbidity in this disease which should be explained thoroughly
to the patient?
A Fulminant liver failure
B Myocarditis and subsequent heart failure
C Progressive proximal muscle weakness
D Rhabdomyolysis leading to renal failure
ANSWER:D

1098
A 21-year-old woman comes to clinic to establish new primary care. She has a history of
type III glycogen storage disease (debranching deficiency), for which she takes a high-
protein, high-carbohydrate diet. She has a normal physical examination except for short
stature, mild weakness, and a slightly enlarged liver. She works as an administrative assistant
and is planning to be married in the next 6 months. She is concerned about her long-term
prognosis and the chances of the disease developing in a child. All of the following
statements about her prognosis are true except
A Cardiomyopathy is a possible complication.
B Chronic liver disease is a possible complication.
C Dementia is a possible complication.
D Her child will not have the disease unless her fiancé is a carrier.
ANSWER:C
1099
A 36-year-old man comes to your office asking for genetic testing for Alzheimer’s disease.
He has no cognitive complaints but notes that all four of his grandparents have had
Alzheimer’s and his father has mild cognitive impairment at the age of 62. His Mini-Mental
Status Examination is 29/30, losing one point on the serial-7’s examination. He requests
testing for the apolipoprotein E allele (ε4). This request is an example of which of the
following?
A Early-onset dementia
B Genetic discrimination
C Predisposition testing
D Presymptomatic testing
ANSWER:C
1100
A recently married couple comes to see you in clinic for prenatal counseling. They are both
in their mid-thirties and have read extensively on the internet about pregnancy and
increasing maternal age. They want to know the risk of miscarriage as well as the risk of
having a child with Down syndrome. Which of the following is true regarding chromosome
disorders and increasing maternal age?
A About half of trisomy conceptions will survive to term.
B In women under the age of 25, trisomy occurs in <1% of all pregnancies.
C Women over the age of 42 have a 33% chance of a trisomic conception.
D The risk of Down syndrome increases 1% per year of maternal age.

ANSWER:C
1101 In what percentage of pregnancies do chromosomal disorders occur?
A 10–25%
B 0.1–0.5%
C 1–2%
D 2–5%
ANSWER:A
1102 All the following disorders can cause ambiguous sexual differentiation except
A 21-hydroxylase deficiency
B androgen insensitivity syndrome
C Klinefelter syndrome
D mixed gonadal dysgenesis
ANSWER:C
1103
An 18-year-old female is evaluated in an outpatient clinic for a complaint of amenorrhea.
She reports that she feels as if she never developed normally compared with other girls her
age. She has never had a menstrual period and complains that she has had only minimal
breast growth. Past medical history is significant for a diagnosis of borderline hypertension.
In childhood the patient frequently had otitis media and varicella infections. She received the
standard vaccinations. She recently graduated from high school and has no learning
difficulties. She is on no medications. On physical examination, the patient is of short stature
with a height of 56 in. Blood pressure is 142/88. The posterior hairline is low. The nipples
appear widely spaced, with only breast buds present. The patient has minimal escutcheon
consistent with Tanner stage 2 development. Her external genitalia appear normal. Bimanual
vaginal examination reveals an anteverted, anteexed uterus. The ovaries are not palpable.
What is the most likely diagnosis?
A Hypothyroidism
B Hyperthyroidism
C Malnutrition
D Turner syndrome (gonadal dysgenesis)
ANSWER:D

1104
A 30-year-old male is seen for a physical examination when obtaining life insurance. The
last time he saw a physician was 15 years ago. He has no complaints. Past medical history is
notable for scoliosis that was surgically corrected when the patient was a teenager and a
recent shoulder dislocation. He takes no medications and does not smoke, drink, or use illicit
drugs. Family history is notable for a father and a brother with colon cancer at ages 45 and
50 years, respectively. Physical examination is notable for normal vital signs, a tall habitus
with hypermobile joints, normal skin, and ectopia lentis. Rectal examination is normal, and
stool is guaiac-negative. The remainder of the examination is normal. Appropriate
recommendations for follow-up should include which of the following annual studies?
A Colonoscopy
B Echocardiography
C Fecal occult blood testing
D Serum periodic acid–Schiff (PSA) measurement
ANSWER:B
1105 All the following diseases are caused by errors in DNA repair except
A ataxia-telangiectasia (AT)
B Fanconi’s anemia (FA)
C fragile X (FX) syndrome
D hereditary nonpolyposis colorectal cancer (HNPCC)
ANSWER:C
1106
A 45-year-old male is evaluated for weakness and a progressive change in mental status.
After extensive evaluation, he is diagnosed with a mitochondrial disorder. All of the
following statements about mitochondrial disorders are true except
A The mitochondrial genome does not recombine.
B Inheritance is maternal.
C The proportion of wild-type and mutant mitochondria in different tissues is identical.
D Cardiomyopathy is a feature of many mitochondrial disorders.
ANSWER:C

1107
Prader-Willi syndrome (PWS) is a rare disorder that is characterized by diminished fetal
activity, obesity, mental retardation, and short stature. A deletion on the paternal copy of
chromosome 15 is the cause. A deletion on the same site on chromosome 15, but on the
maternal copy, results in a different syndrome: Angelman’s syndrome. This syndrome is
characterized by mental retardation, seizures, ataxia, and hypotonia. What is the name of the
genetic mechanism that results in this phenomenon?
A Genetic anticipation
B Genetic imprinting
C Lyonization
D Somatic mosaicism
ANSWER:B
1108 All the following are inherited disorders of connective tissue except
A Alport syndrome
B Ehlers-Danlos syndrome
C Marfan syndrome
D McArdle’s disease
ANSWER:D
1109
A 30-year-old male comes to your office for genetic counseling. His brother died at age 13
years with TaySachs disease. His sister is unaffected. The patient and his wife wish to have
children. Which of the following statements concerning Tay Sachs disease is true?
A It is seen most commonly in Scandinavian populations.
B It is caused by mutations in the galactosidase gene.
C Most patients die in the third or fourth decade of life.
D Death occurs as a result of progressive neurologic decline.
ANSWER:D
1110 All the following statements about Gaucher disease are true except
A Bone pain is common.
B Disease frequency is highest in Ashkenazi Jews.
C Inheritance is autosomal recessive.
D Splenomegaly is rare.
ANSWER:D

1111
Diseases that are inherited in a multifactorial genetic fashion (i.e., not autosomal dominant,
autosomal recessive, or X-linked) and are seen more frequently in persons bearing certain
histocompatibility antigens include
A gluten-sensitive enteropathy
B neurofibromatosis
C adult polycystic kidney disease
D Wilson’s disease
ANSWER:A
1112
Chronic hypoxia causes biochemical changes whereby oxygen delivery to tissues is not
impaired. In comparison to someone living at sea level, which of the following changes
would be expected in a healthy person acclimated to living at high altitude?
A Basal temperature <37°C
B Serum pH >7.45
C Increased red blood cell levels of 2,3-diphosphoglycerate
D Hemoglobin concentration <10 mg/dL
ANSWER:C
1113
Which of the following would be present in an individual who has lost nondeclarative
memory?
A Inability to recall a spouse’s birthday
B Inability to recall how to tie one’s shoe
C Inability to recognize a photo that was taken at one’s wedding
D Inability to recognize a watch as an instrument for keeping time
ANSWER:B
1114
A 24-year-old woman presents for a routine checkup and complains only of small masses in
her groin. She states that they have been present for at least 3 years. On physical
examination, she is noted to have several palpable 1-cm inguinal lymph nodes that are
mobile, nontender, and discrete. There is no other lymphadenopathy on examination. What
should be the next step in management?
A Bone marrow biopsy
B CT scan of the chest, abdomen, and pelvis
C Reassurance
D Fine-needle aspiration for culture and cytopathology

ANSWER:C
1115
All the following diseases are associated with massive splenomegaly (spleen extends 8 cm
below the costal margin or weighs >1000 g) except
A autoimmune hemolytic anemia
B chronic lymphocytic leukemia
C cirrhosis with portal hypertension
D None of the above
ANSWER:C
1116
The presence of Howell-Jolly bodies, Heinz bodies, basophilic stippling, and nucleated red
blood cells in a patient with hairy cell leukemia prior to any treatment intervention implies
which of the following?
A Diffuse splenic infiltration by tumor
B Disseminated intravascular coagulation (DIC)
C Hemolytic anemia
D Pancytopenia
ANSWER:A
1117 Which of the following is true regarding infection risk after elective splenectomy?
A Patients are at no increased risk of viral infection after splenectomy.
B Patients should be vaccinated 2 weeks after splenectomy.
C Splenectomy patients over the age of 50 are at greatest risk for postsplenectomy sepsis.
D Staphylococcus aureus is the most commonly implicated organism in postsplenectomy sepsis.
ANSWER:A
1118
A 64-year-old man comes to your office complaining of erectile dysfunction. He is not able
to generate an erection. His past medical history is significant for coronary artery bypass
grafting many years ago, status post-carotid endarterectomy, and a mildly reduced left
ventricular ejection fraction. His medications include aspirin, carvedilol, simvastatin,
lisinopril and furosemide. He does not take nitrates. On physical examination, you note
normal-sized testes and a normal prostate. There are no fibrotic changes along the penile
corpora. His libido is intact. What is the most likely cause of this patient’s erectile
dysfunction?
A Disturbance of blood flow

B Low testosterone
C Medication related
D Psychogenic
ANSWER:A
1119
You perform a nocturnal tumescence study on the patient in the preceding scenario. He does
not have any erections during rapid-eye-movement sleep. Which treatment modality do you
offer at this time?
A Couple sex therapy
B Implantation of a penile prosthesis
C Intraurethral alprostadil
D Vardenafil
ANSWER:D
1120
The wife of the patient in the preceding scenario also reports to you that she has experienced
a low sexual desire lately. She is not distressed by this and the couple reports no conflict as a
result of her low desire. She is 61 years old and also has a history of a coronary artery bypass
graft remotely. She experienced menopause at the age of 53. Her medications include an
aspirin, metoprolol, simvastatin, verapamil, and a multivitamin. She asks whether an oral
agent will assist with her sexual desire. What is the best answer for this patient?
A
Phosphodiesterase type 5 (PDE-5) inhibitors have been shown to improve sexual function in
premenopausal women
B PDE-5 inhibitors have been shown to improve sexual function in postmenopausal women
C PDE-5 inhibitors have no role in the treatment of female sexual dysfunction
D PDE-5 inhibitors treat orgasmic disorder but not sexual arousal disorder
ANSWER:C

1121
A 54-year-old male patient of yours presents to your clinic complaining of unexplained
weight loss. On review of his chart, you do notice that he has lost 8% of his total body
weight in the past year. He has well-treated hypertension for which he takes a thiazide
diuretic. Other than recently being widowed, he has no pertinent social history. He is a
lifelong nonsmoker and worked as a hospital administrator. An extensive review of systems
is unrevealing. Your physical examination reveals no masses or other pathology. A brief
psychiatric examination shows no signs of depression. You perform initial testing with a
complete blood count; electrolytes, renal function, liver function, urinalysis, thyroid-
stimulating hormone, and a chest x-ray, which are unrevealing. He is up to date on his
routine cancer screening. What is the next step in the workup of this patient?
A Chest CT scan
B Close follow-up
C Positron emission tomography (PET) scan
D Total-body CT scan
ANSWER:B
1122
You are conducting research on a novel nonsteroidal anti-inflammatory drug (NSAID). To
ascertain the safety profile of the drug you recruit 100 volunteers who lack the ability to
produce IgE. All subjects receive the drug. A minority of participants experience an
anaphylactic reaction within minutes of ingesting the drug. IgE levels are undetectable in all
100 subjects. What is the most likely explanation for this phenomenon?
A The drug itself directly triggered the immune system in a minority of patients.
B The IgE receptor in the patients with anaphylaxis is constitutively activated.
C The patients who had anaphylaxis have received this drug before.
D The patients who had anaphylaxis overexpress CD8+ T cells.
ANSWER:A
1123 Anthrax spores can remain dormant in the respiratory tract for how long?
A 1 week
B 2 weeks
C 3 weeks
D 6 weeks
ANSWER:D

1124
Twenty recent attendees at a National Football League game arrive at the emergency
department complaining of shortness of breath, fever, and malaise. Chest roentgenograms
show mediastinal widening on several of these patients, prompting a concern for inhalational
anthrax as a result of a bioterror attack. Antibiotics are initiated and the Centers for Disease
Control and Prevention is notified. What form of isolation should be instituted for these
patients?
A Airborne
B Contact
C Droplet
D None
ANSWER:D
1125
You are working in an urban-based intensive care unit and two cases of severe pneumonia
are admitted. Francisella tularensis is cultured from both patients’ sputum samples. Neither
patient recalls contact with wild or domesticated animals in the past 2 weeks. You should do
all of the following except
A
Alert the Centers for Disease Control and Prevention (CDC) authorities about the potential
for a bioterrorist attack.
B Alert the microbiology laboratory director.
C Institute droplet precaution for the involved patients.
D Treat with broad-spectrum antibiotics.
ANSWER:C
1126 All the following are well-documented physical effects of smoking marijuana except
A decreased sperm count
B chronic bronchial irritation
C delayed gastric emptying
D exercise-induced angina
ANSWER:C

1127
A young man is brought to the emergency department by his parents. For the past 12 h he
has barricaded himself in his room out of fear of being taken away by “the guys in black.”
He fears he is losing control and fears that he is going to die. His parents found him
trembling and sweating in his room with various pills and plant leaves in his possession. He
feels like he is choking and that he is about to die at any minute. On examination, his pupils
are dilated and he has a heart rate of 143 beats/min. What substance is most likely to have
caused these symptoms?
A Heroin
B Lysergic acid diethylamide (LSD)
C Marijuana
D Methamphetamine
ANSWER:B
1128
Which of the following is a distinguishing feature of amphetamine overdose versus other
causes of sympathetic overstimulation due to drug overdose or withdrawal?
A Hallucination
B Hot, dry, flushed skin and urinary retention
C History of benzodiazepine abuse
D
Markedly increased blood pressure, heart rate, and end-organ damage in the absence of
hallucination
ANSWER:D
1129 Which of the following findings suggests an opiate overdose?
A Anion gap metabolic acidosis with a normal lactate
B Hypotension and bradycardia in an alert patient
C Mydriasis
D Therapeutic response to naloxone
ANSWER:D
1130
A patient with metabolic acidosis, reduced anion gap, and increased osmolal gap is most
likely to have which of the following toxic ingestions?
A Lithium
B Methanol
C Oxycodone
D Propylene glycol

ANSWER:A
1131
Which of the following is true regarding drug effects after an overdose in comparison to a
reference dose?
A Drug effects begin earlier, peak earlier, and last longer
B Drug effects begin earlier, peak later, and last longer
C Drug effects begin earlier, peak later, and last shorter
D Drug effects begin later, peak earlier, and last shorter.
ANSWER:B
1132
A 28-year-old man with bipolar disorder, who is on lithium, is found in his room 2 days
after not showing up to work. He is arousable but dysarthric and has a markedly abnormal
gait when trying to walk. Upon arrival at the emergency department, he has a grand mal
seizure. The seizure is not sustained but recurs an hour after 6 mg lorazepam is infused IV.
In the postictal stage, he is not arousable to sternal rub and lacks a gag reflex. His serum
sodium returns at 158 meq/L. In reference to his seizures, all of the following are next steps
in his management except
A barbiturates
B benzodiazepines
C endotracheal intubation
D phenytoin
ANSWER:D
1133 What is the main contributor to the resting energy expenditure of an individual?
A Adipose tissue
B Exercise level
C Lean body mass
D Resting heart rate
ANSWER:C
1134 Which type of bite represents a potential medical emergency in an asplenic patient?
A Cat bite
B Dog bite
C Fish bite
D Human bite
ANSWER:B

1135
A 36-year-old man with HIV/AIDS (CD4+ lymphocyte count = 112/µL) develops a scaly,
waxy, yellowish, patchy, crusty, pruritic rash on and around his nose. The rest of his skin
examination is normal. Which of the following is the most likely diagnosis?
A Molluscum contagiosum
B Psoriasis
C Reactivation herpes zoster
D Seborrheic dermatitis
ANSWER:D
1136
A 28-year-old woman returns from a 6-week trip to Tanzania in March. She calls your office
2 weeks later complaining of new symptoms of fever, mild abdominal pain, and headache.
She feels like she has the flu. What should you do next?
A Ask her to come to the clinic in the next 24 h.
B Emergently refer her to the emergency department.
C Write her a prescription for oseltamivir and call her in 24 h to ensure improvement.
D Write her a prescription for a respiratory fluoroquinolone.
ANSWER:B
1137
A 30-year-old female with end-stage renal disease who receives her dialysis through a
tunneled catheter in her shoulder presents with fever and severe low back pain. On
examination, she is uncomfortable and diaphoretic but hemodynamically stable. She has a
soft 2/6 early systolic flow murmur. Her line site is red and warm with no pustular exudates.
She is very tender over her lower back. Neurologically, she is completely intact. There is no
evidence of Janeway lesions, Osler nodes, or Roth spots. Her white count is 16,700 with 12%
bands. Immediate evaluation should include all of the following except
A MRI of the lumbar spine
B removal of her dialysis catheter
C transthoracic echocardiogram
D two sets of blood cultures followed by vancomycin as well as gram-negative coverage
ANSWER:C
1138
Which of the following scenarios is most likely associated with the lowest risk of HIV
transmission to a health care provider after an accidental needle stick from a patient with
HIV?
A The needle is visibly contaminated with the patient’s blood.

B The needle stick injury is a deep tissue injury to the health care provider.
C
The patient whose blood is on the contaminated needle has been on antiretroviral therapy for
many years with a history of resistance to many available agents but most recently has had
successful viral suppression on current therapy.
D
The patient whose blood is on the contaminated needle was diagnosed with acute HIV
infection 2 weeks ago.
ANSWER:C
1139
In the inpatient setting, extended-spectrum β-lactamase (ESBL)-producing gram-negative
infections are most likely to occur after frequent use of which of the following classes of
antibiotics?
A Carbapenems
B Macrolides
C Quinolones
D Third-generation cephalosporins
ANSWER:D
1140 In the developed world, seroprevalence of Helicobacter pylori infection is currently
A decreasing
B increasing
C staying the same
D unknown
ANSWER:A
1141
In a patient who has undergone a traumatic splenectomy, what test can be ordered to
establish lack of splenic function?
A CT scan of the abdomen
B Neutrophil migration studies
C Peripheral blood flow cytometry
D Peripheral blood smear
ANSWER:D

1142
A 26-year-old female college student presents with tender epitrochlear and axillary tender,
firm, 3-cm lymph nodes on her left side. She has a 0.5-cm painless nodule on her left second
finger. She reports low-grade fever and malaise over 2 weeks. She enjoys gardening, exotic
fish collecting, and owns several pets including fish, kittens, and a puppy. She is sexually
active with one partner. She traveled extensively throughout rural Southeast Asia 2 years
before her current illness. The differential diagnosis includes all of the following except
A Bartonella henselae infection
B lymphoma
C Sporothrix schenkii infection
D Staphylococcal infection
ANSWER:C
1143
Sensitive and specific serum or urine diagnostic tests exist for all of the following invasive
fungal infections except
A blastomycosis
B coccidioidomycosis
C cryptococcosis
D histoplasmosis
ANSWER:A
1144
A 34-year-old man seeks the advice of his primary care physician because of an
asymptomatic rash on his chest. There are coalescing light brown to salmon-colored macules
present on the chest. A scraping of the lesions is viewed after a wet preparation with 10%
potassium hydroxide solution. There are both hyphal and spore forms present, giving the
slide an appearance of “spaghetti and meatballs.” In addition, the lesions fluoresce to a
yellowgreen appearance under a Wood’s lamp. Tinea versicolor is diagnosed. Which of the
following microorganisms is responsible for this skin infection?
A Fusarium solani
B Malassezia furfur
C Sporothrix schenkii
D Trichophyton rubrum
ANSWER:B

1145
A 19-year-old woman with anorexia nervosa undergoes surgery for acute appendicitis. The
postoperative course is complicated by acute respiratory distress syndrome, and she remains
intubated for 10 days. She develops wound dehiscence on postoperative day 10. Laboratory
data show a white blood cell count of 4000/ µL, hematocrit 35%, albumin 2.1 g/dL, total
protein 5.8 g/dL, transferrin 54 mg/dL, and iron-binding capacity 88 mg/dL. You are
considering initiating nutritional therapy on hospital day 11. Which of the following is true
regarding the etiology and treatment of malnutrition in this patient?
A She has marasmus, and nutritional support should be started slowly.
B She has kwashiorkor, and nutritional support should be aggressive.
C
She has marasmic kwashiorkor, kwashiorkor predominant, and nutritional support should be
aggressive.
D
She has marasmic kwashiorkor, marasmus predominant, and nutritional support should be
slow.
ANSWER:C
1146
You are seeing a patient in follow-up 2 weeks after hospitalization. The patient is recovering
from nosocomial pneumonia due to a resistant Pseudomonas spp. His hospital course was
complicated by a deep venous thrombosis. The patient is currently on IV piperacillin/
tazobactam and tobramycin via a tunneled catheter, warfarin, lisinopril, hydrochlorothiazide,
and metoprolol. Laboratory data this morning show an INR of 8.2. At hospital discharge his
INR was stable at 2.5. He has no history of liver disease. What is the most likely cause of the
elevated INR?
A The patient has inadvertently overdosed.
B
The patient has developed a recurrent deep venous thrombosis, which has affected the
laboratory data.
C The patient is deficient in vitamin K and needs supplementation.
D The warfarin prescription was written incorrectly at the time of discharge.
ANSWER:C

1147
A 51-year-old alcoholic man is admitted to the hospital for upper gastrointestinal bleeding.
From further history and physical examination, it becomes apparent that his bleeding is from
gingival membranes. He is intoxicated and complains of fatigue. Reviewing his chart you
find that he had a hemarthrosis evacuated 6 months ago and has been lost to follow-up since
then. He takes no medications. Laboratory data show platelets of 250,000, INR of 0.9. He
has a diffuse hemorrhagic eruption on his legs (Figure 3, Color Atlas). What is the
recommended treatment for this patient’s underlying disorder?
A Folate
B Niacin
C Thiamine
D Vitamin C
ANSWER:D
1148
While working in the intensive care unit, you admit a 57-year-old woman with acute
pancreatitis and oliguric renal failure. Respiratory rate is 26 breaths/min, heart rate is 125
beats/min, and temperature is 37.2°C. Physical examination shows marked abdominal
tenderness with normoactive bowel sounds. A CT scan shows an inflamed pancreas without
hemorrhage. You calculate her APACHE-I score to be 28. When deciding on when to initiate
nutritional replacement in this patient, which of the following statements is true?
A Bowel rest is the cornerstone of treatment for acute pancreatitis.
B
Administering parenteral nutrition within 24 h will decrease the risk of infection and
mortality.
C
Enteral feeding supports gut function by secretion of gastrointestinal hormones that stimulate
gut trophic activity.
D
In severe systemic response to inflammation, feeding can be withheld initially because the
patient is likely to have adequate, spontaneous oral intake in the first 7 days.
ANSWER:C
1149
The resting energy expenditure is a rough estimate of total caloric needs in a state of energy
balance. Of these two patients with stable weights, which person has the highest resting
energy expenditure (REE): Patient A, a 40-year-old man who weighs 90 kg and is sedentary,
or Patient B, a 40year-old man who weighs 70 kg and is very active?
A 40-year-old man who weighs 90 kg and is sedentary

B 40-year-old man who weighs 70 kg and is very active
C REE is the same
D Not enough information given to calculate the REE
ANSWER:B
1150 All of the following clinical features are common in patients with anorexia nervosa except
A Avoid food-related occupations
B Distorted body image
C Engage in binge eating
D Exercise extensively
ANSWER:A
1151
You diagnose anorexia nervosa in one of your new clinic patients. When coordinating a
treatment program with the psychiatrist, what characteristics should prompt consideration for
inpatient treatment instead of scheduling an outpatient assessment?
A Amenorrhea
B Exaggeration of food intake
C Irrational fear of gaining weight
D Weight <75% of expected body weight
ANSWER:D
1152
It is hospital day 16 for a 49-year-old homeless patient who is recovering from alcohol
withdrawal and delirium tremens. She spent the first 9 days of this hospitalization in the
intensive care unit but is now awake, alert, and conversant. She has a healing decubitus ulcer,
and her body mass index is 19 kg/m2. Laboratory data show an albumin of 2.9 g/dL and a
prothrombin time of 18 s (normal range). Is this patient malnourished?
A Cannot be determined, need more information.
B No. Given her heavy alcohol intake, her prothrombin time is expected to be delayed.
C
No. She has a low resting energy expenditure and her intact mental state argues against
malnutrition.
D
Yes, this degree of hypoalbuminemia is uncommon in cirrhosis and is likely due to
malnutrition.
ANSWER:A

1153
A 42-year-old male patient wants your opinion about vitamin E supplements. He has read
that taking high doses of vitamin E can improve his sexual performance and slow the aging
process. He is not vitamin E deficient. You explain to him that these claims are not based on
good evidence. What other potential side effect should he be concerned about?
A Deep venous thrombosis
B Hemorrhage
C Night blindness
D Peripheral neuropathy
ANSWER:B
1154
Doing rounds in the oncology center, you are see a patient with carcinoid syndrome. Due to
the increased conversion of tryptophan to serotonin, this patient has developed niacin
deficiency. All of the following are components of the pellagra syndrome except
A dermatitis
B dementia
C diarrhea
D dyslipidemia
ANSWER:D
1155
An 86-year-old woman with chronic obstructive pulmonary disease (COPD), congestive
heart failure, and insulin-requiring type 2 diabetes mellitus is admitted to the intensive care
unit with an exacerbation of her COPD. She is intubated and treated with glucocorticoids and
nebulized albuterol. She is also continued on her glargine insulin, aspirin, pravastatin,
furosemide, enalapril, and metoprolol. On hospital day 8, parenteral nutrition is begun via
catheter in the subclavian vein. Her insulin requirements increase on hospital day 9 due to
episodes of hyperglycemia. On hospital day 10, she develops rales and an increasing oxygen
requirement. A chest radiograph shows bilateral pulmonary edema. Laboratory data show
hypokalemia, hypomagnesemia, and hypophosphatemia and a normal creatinine. Her weight
has increased by 3 kg since admission. Urine sodium is <10 meq/dL. All of the following
changes in her nutritional regimen will improve her volume status except
A combination of glucose and fat in the parenteral nutrition mixture
B decreasing the sodium content of the mixture to <40 meq per day
C increasing the protein content of the parenteral nutrition mixture

D reducing the overall glucose content
ANSWER:C
1156
A new study has been published showing a benefit of 25 mg/day of vitamin X. The
recommended estimated average requirement of vitamin X is 10 mg/day, 2 standard
deviations below the amount published in the study. The tolerable upper limit of vitamin X is
unknown. Your patient wants to know if it is safe to consume 25 mg/day of vitamin X.
Which is the most appropriate answer?
A
Two standard deviations above the estimated average requirement defines the tolerable upper
limit.
B 25 mg/day is probably too much vitamin X in 1 day.
C 25 mg/day is statistically in a safe range of the estimated average requirement.
D The study was not designed to assess safety and therefore should not influence practice.
ANSWER:C
1157
An elevation in which of the following hormones is consistent with the effects of anorexia
nervosa?
A Cortisol
B Gonadotropin-releasing hormone (GnRH)
C Leptin
D Thyroxine (T4)
ANSWER:A
1158
14. Which of the following statements regarding anorexia nervosa (AN) and bulimia
nervosa (BN) is true?
A
Patients with the purging subtype of BN tend to be heavier than those with the nonpurging
subtype.
B
Patients with the restricting subtype of AN are more emotionally labile than those with the
purging subtype.
C
Patients with the restricting subtype of AN are more likely to abuse illicit drugs than those
with the purging subtype.
D The mortality of BN is lower than that of AN.
ANSWER:D

1159
You are seeing a pediatric patient from Djibouti in consultation who was admitted with a
constellation of symptoms including diarrhea, alopecia, muscle wasting, depression, and a
rash involving the face, extremities, and perineum. The child has hypogonadism and
dwarfism. You astutely make the diagnosis of zinc deficiency, and laboratory test confirm
this (zinc level <70 µL/dL). What other clinical findings is this patient likely to manifest?
A Dissecting aortic aneurysm
B Hypochromic anemia
C Hypoglycemia
D Hypopigmented hair
ANSWER:D
1160
You are rotating on a medical trip to impoverished areas of China. You are examining an 8-
year-old child whose mother complains of him being clumsy and sickly. He has had many
episodes of diarrheal illnesses and pneumonia. His “clumsiness” is most pronounced in the
evening when he has to go outside and do his chores. On examination, you notice
conjunctival dryness with white patches of keratinized epithelium on the sclera. What is the
cause of this child’s symptoms?
A Autoimmune neutropenia
B Congenital rubella
C Spinocerebellar ataxia (SCA) type 1
D Vitamin A deficiency
ANSWER:D
1161
After being stranded alone in the mountains for 8 days, a 26-year-old hiker is brought to the
hospital for evaluation of a right femoral neck fracture. He has not had anything to eat or
drink for the past 6 days. Vital signs are within normal limits. Weight is 79.5 kg, which is 1.8
kg less than he weighed 6 months ago. Laboratory data show a creatinine of 2.5 mg/dL,
blood urea nitrogen of 52 mg/dL, glucose 96 mg/dL, albumin 4.1 mg/dL, chloride 105
meq/L, and ferritin on 173 ng/mL. Which of the following statements is true regarding his
risk of malnourishment?
A He has protein-calorie malnutrition due to the rate of weight loss.
B He has protein-calorie malnutrition due to his elevated ferritin.
C He is at risk, but a normal individual can tolerate 7 days of starvation.
D He is not malnourished because he is not hypoglycemic after 6 days of no food or water.

ANSWER:C
1162
You are doing rounds in the intensive care unit on an intubated patient who is recovering
from a stroke and has diabetic gastroparesis. When suctioning the patient in the morning, she
coughs profusely, with thick green secretions. You are concerned about the possibility of
aspiration pneumonia. All of the following measures are useful in preventing aspiration
pneumonia in an intubated patient except
A combined enteral and parenteral nutrition
B elevating the head of the bed to 30°
C physician-directed methods for formula advancement
D post-ligament of Treitz feeding
ANSWER:C
1163
Which of these features represents a critical distinction between anorexia nervosa and bulimia
nervosa?
A Binge eating
B Electrolyte abnormalities
C Self-induced vomiting
D Underweight
ANSWER:D
1164
You are counseling a patient who is recovering from long-standing anorexia nervosa (AN).
She is a 22-yearold woman who suffered the effects of AN for 8 years with a nadir body
mass index of 17 kg/m2 and many laboratory abnormalities during that time. Which of the
following characteristics of AN is least likely to improve despite successful lasting treatment
of the disorder?
A Amenorrhea
B Delayed gastric emptying
C Lanugo
D Low bone mass
ANSWER:D
1165
Which source of stem cell is incorrectly paired with the challenge associated with their
clinical application?
A
Bone marrow mesenchymal stem cells: Transplanted cells may not differentiate into the
desired cell type

B Embryonic stem cells: High potential to form teratomas
C Organ-specific multipotent stem cells: Difficult to isolate from tissues other than bone marrow
D Umbilical cord blood stem cells: Graft-versus-host disease
ANSWER:D
1166
You are seeing a patient in follow-up in whom you have begun an evaluation for an elevated
hematocrit. You suspect polycythemia vera based on a history of aquagenic pruritus and
splenomegaly. Which set of laboratory tests are consistent with the diagnosis of polycythemia
vera?
A Elevated red blood cell mass, high serum erythropoietin levels, normal oxygen saturation
B Elevated red blood cell mass, low serum erythropoietin levels, normal oxygen saturation
C Normal red blood cell mass, high serum erythropoietin levels, low arterial oxygen saturation
D Normal red blood cell mass, low serum erythropoietin levels, low arterial oxygen saturation
ANSWER:B
1167
A new screening test for thyroid cancer has been introduced into the population. In the first
year, 1000 positive tests lead to correct identification of thyroid cancer in the screened
population. Over the next year, 250 cases of thyroid cancer are detected among those who
initially had a negative test. What is the sensitivity of this new screening test?
A 0,25
B 0,67
C 0,8
D Not enough information to calculate
ANSWER:C
1168
A patient with alcoholic cirrhosis has increasing ascites despite dietary sodium control and
diuretics. A paracentesis shows clear, turbid fluid. There are 2300 white blood cells (WBCs)
and 150 red blood cells per mi- croliter. The WBC differential shows 75% lymphocytes.
Fluid protein is 3.2 g/dL and the serum-ascites albumin gradient (SAAG) is 1.0 g/dL. What is
the most appropri- ate next study in this patient’s management?
A Adenosine deaminase activity of the ascitic fluid
B CT scan of the liver

C Peritoneal biopsy
D None; consider transplant evaluation
ANSWER:A
1169
You are managing a patient who complains of ab- dominal pain. The pain is located in the
epigastric area and radiates to the back. Leaning forward improves the pain. The rest of the
physical examination is unremark- able and there is no jaundice. The total bilirubin is 0.7
mg/ dL and CA 19-9 level is within the normal range. An ultra- sound of the abdomen
shows a 2.5-cm well-circumscribed mass in the tail of the pancreas. There is no ductal dila-
tion. A CT scan confirms the presence of a 2.5-cm spicu- lated mass in the tail of the
pancreas with no surrounding lymphadenopathy or local extension. What is the next most
appropriate step in this patient’s management?
A Magnetic resonance cholangiopancreatography
B Refer for surgical resection
C Serial CA 19-9 measurement
D Ultrasound-guided biopsy
ANSWER:B

1170
A 45-year-old woman presents to the emergency room for evaluation of fatigue, fever, and
acute onset of joint pain and swelling of the right knee, left ankle, and right second toe. She
reports that she was ill with a diarrheal illness about 2 weeks ago. She did not seek evaluation
as the symptoms resolved spontaneously over 48 h. She did lose about 2.3 kg, which she has
been unable to regain. Three days ago, she developed a feeling of malaise with fevers and
pain in her right second toe. Additional IX-17. (Continued) joints have become inflamed
over the ensuing 72 h. She denies any prior similar episodes. She is not currently sexually
active and estimates her last sexual activity to be 8 months prior to presentation. She has a
history of seasonal rhinitis, but is taking no medications currently. On examination, she is
febrile at 38.4°C. Her left eye has evidence of conjunctival injection. There is a superficial
ulcer on the inside of her lower lip that is not painful. The right knee is warm to touch with
an effusion. Passive movement results in pain. The left ankle is similarly warm and painful.
The right second great toe has the appearance of a “sausage digit.” There is also pain with
palpation at the tendinous insertion of both Achilles tendons. There are no genital ulcers or
discharge. No rash is present. Arthrocentesis is performed and is consistent with
inflammatory arthritis without crystals or organisms seen on Gram stain. Cervical probes for
Neisseria gonorrhoeae and Chlamydia trachomatis are negative. Reactive arthritis following
Campylobacter infection is suspected with positive serum antibodies to C. jejuni. Which of
the following statements is true regarding this diagnosis?
A
Chronic joint symptoms affect 15% of individuals, and recurrences of the acute syndrome
may occur.
B Presence of HLA-B27 antigen predicts individuals who are likely to have a better prognosis.
C
Reactive arthritis is self-limited and should be expected to resolve spontaneously over the
next 2 weeks.
D
The causative organism has no effect on long-term outcomes following an initial episode of
reactive arthritis.
ANSWER:A
1171
An 18-year-old man with ankylosing spondylitis (AS) is concerned about the development
of disability due to his disease. Which of the following statements is true regarding the
development and treatment of disability in AS?

A
Anti-TNF-α (tumor necrosis factor α) inhibitors are now first-line therapy and have been
shown to limit disability while being safe for long-term therapy.
B
Despite the development of ankylosis of the spine, spinal fracture is a rare complication,
affecting <10% of individuals with AS.
C
Maintenance of an exercise program to maintain posture and range of motion is important in
limiting disability.
D
Nonsteroidal anti-inflammatory drugs (NSAIDs) decrease pain but have no effect on the
development of disability in AS.
ANSWER:C
1172
A 42-year-old woman is brought to the emergency room by ambulance for altered mental
status. The glucose level by fingerstick monitoring was below the measurement capabilities
of the monitor (<40 mg/dL). After 2 ampules of 50% dextrose, the patient’s fingerstick
glucose remains at 42 mg/dL. She remains unconscious and had a 1-min seizure while in
transport. She has no history of diabetes mellitus. Her family denies that she has been
recently ill, but recently she has been depressed. She works as a registered nurse on a medical
floor of the hospital. Which of the following tests would confirm an overdose of exogenous
insulin?
A
Plasma glucose <55 mg/dL, plasma insulin >18 pmol/L, and plasma C-peptide levels
undetectable
B
Plasma glucose <55 mg/dL, plasma insulin >18 pmol/L, and plasma C-peptide levels >0.6
ng/mL
C Plasma glucose <55 mg/dL, plasma insulin <18 pmol/L, and plasma glucagon <12 pmol/L
D Plasma glucose <55 mg/dL, plasma insulin <18 pmol/L, and C-peptide levels undetectable
ANSWER:C
1173
Differentiating primary dysmenorrhea from other causes of chronic cyclical pelvic pain is
important because there is a specific treatment for primary dysmenorrhea. What is the
pathophysiology/treatment for primary dysmenorrhea?
A Ectopic endometrium/oral contraceptives
B History of sexual abuse/counseling
C Increased stores of prostaglandin precursors/antiinflammatory medication
D Ruptured graafian follicle/oral contraceptives
ANSWER:C

1174
At the midpoint of the menstrual cycle, a luteinizing hormone (LH) surge occurs via an
estrogen-mediated pathway. Though chronic low levels of estrogen are inhibitory to LH
release, gradually rising estrogen levels stimulate LH secretion. This relationship between
estrogen and LH is an example of which endocrine regulatory system?
A Autocrine regulation
B Negative feedback control
C Paracrine regulation
D Positive feedback control
ANSWER:D
1175
The mineralocorticoid receptor in the renal tubule is responsible for the sodium retention and
potassium wasting that is seen in mineralocorticoid excess states such as aldosterone-secreting
tumors. However, states of glucocorticoid excess (e.g., Cushing’s syndrome) can also present
with sodium retention and hypokalemia. What characteristic of the mineralocorticoid-
glucocorticoid pathways explain this finding?
A Higher affinity of the mineralocorticoid receptor for glucocorticoids
B Oversaturation of the glucocorticoid degradation pathway in states of glucocorticoid excess
C Similar, but distinct, DNA binding sites producing the same metabolic effect
D Upregulation of the mineralocorticoid-binding protein in states of glucocorticoid excess
ANSWER:B
1176
You are researching a cell line with an altered membrane structure that makes the cell
membrane impermeable to extracellular molecules of all size and charge. You then expose
the cell line to varying concentrations of various hormones. Of the following hormones,
which one should no longer exert an effect on this cell line?
A Dopamine
B Gonadotropin-releasing hormone
C Insulin
D Vitamin D
ANSWER:D

1177
The parents of a 14-year-old boy want your opinion about treatment of their child’s lipid
disorder. The family emigrated from South Africa to the United States recently. The child
has had cutaneous xanthomas on the hands, elbows, heels, and buttocks since childhood. In
South Africa, he underwent thoracotomy for a problem with his aortic valve 3 years ago. He
currently experiences exertional dyspnea, and his diet consists mostly of unhealthy, fatty
foods. On examination, you appreciate bruits in the femoral arteries and abdominal aorta.
His most recent lipid profile shows a total cholesterol of 734 mg/dL and a low-density
lipoprotein (LDL) of 376 mg/dL. What is the most appropriate step in this patient’s
evaluation?
A Genetic test for familial defective apoB100
B Rule out congenital syphilis
C Rule out hypothyroidism
D Screen the parents for Münchhausen-by-proxy syndrome
ANSWER:C
1178
A 62-year-old woman presents to your clinic complaining of fatigue and lethargy over a
period of 6 months. She cannot recall exactly when these symptoms started, but feels that
they are worsening with time. She describes dry skin and has noted that she is losing hair. On
examination she is mildly bradycardic at 52 beats/min with normal blood pressure and has
dry, coarse skin. There are areas of alopecia and mild lower extremity edema is noted. Which
of the following is the most likely clinical diagnosis and which test would be indicated for
screening for the diagnosis?
A Hyperthyroidism: thyroid-stimulating hormone (TSH)
B Hyperthyroidism: unbound T4
C Hypothyroidism: TSH
D Hypothyroidism: unbound T4
ANSWER:C

1179
You are asked to see a 15-year-old African-American girl because of anovulation. She has
never experienced menarche, and her mother is concerned since most women in her family
experience menarche around 13 years of age. The patient has prominent nipples and the
areola are part of the breast. Pubic hair is dark, curly, and coarse and is abundant in the
pubic area and inner thigh. There is no facial hair, and muscular development is age and sex
appropriate. She does have cyclical pelvic pain. What is the next step in her evaluation?
A Examination with a speculum
B MRI of the abdomen and pelvis
C Serum follicle-stimulating hormone (FSH)
D Serum prolactin
ANSWER:A
1180
A 24-year-old man with advanced HIV infection presents to the emergency department with
a tan painless nodule on the lower extremity (Figure IV-2, Color Atlas). He is afebrile and
has no other lesions. He does not take antiretroviral therapy, and his last CD4+ T cell count
was 20/µL. He lives with a friend who has cats and kittens. A biopsy shows lobular
proliferation of blood vessels lined by enlarged endothelial cells and a mixed acute and
chronic inflammatory infiltrate. Tissue stains show gramnegative bacilli. Which of the
following is most likely to be effective therapy for the lesion?
A Azithromycin
B Cephazolin
C Interferon α
D Penicillin
E Vancomycin
ANSWER:A

1181
A 38-year-old homeless man presents to the emergency room with a transient ischemic attack
characterized by a facial droop and left arm weakness lasting 20 min, and left upper
quadrant pain. He reports intermittent subjective fevers, diaphoresis, and chills for the past 2
weeks. He has had no recent travel or contact with animals. He has taken no recent
antibiotics. Physical examination reveals a slightly distressed man with disheveled
appearance. His temperature is 38.2°C; heart rate is 90 beats per minute; blood pressure is
127/74 mmHg. He has poor dentition. Cardiac examination reveals an early diastolic
murmur over the left 3d intercostal space. His spleen is tender and 2 cm descended below the
costal margin. He has tender painful red nodules on the tips of the third finger of his right
hand and on the fourth finger of his left hand that are new. He has nits evident on his
clothes, consistent with body louse infection. White blood cell count is 14,500, with 5% band
forms and 93% polymorphonuclear cells. Blood cultures are drawn followed by empirical
vancomycin therapy. These cultures remain negative for growth 5 days later. He remains
febrile but hemodynamically stable but does develop a new lesion on his toe similar to those
on his fingers on hospital day 3. A transthoracic echocardiogram reveals a 1-cm mobile
vegetation on the cusp of his aortic valve and moderate aortic regurgitation. A CT scan of
the abdomen shows an enlarged spleen with wedge-shaped splenic and renal infarctions.
What test should be sent to confirm the most likely diagnosis?
A Bartonella serology
B Epstein-Barr virus (EBV) heterophile antibody
C HIV polymerase chain reaction (PCR)
D Peripheral blood smear
E Q fever serology
ANSWER:A
1182
A 26-year-old woman comes to your clinic complaining of 3–4 weeks of a malodorous
white vaginal discharge. She recently began having unprotected sexual intercourse with a
new male partner. He is asymptomatic. Her only medication is oral contraceptives.
Examination reveals a thin white discharge that evenly coats the vagina. Further examination
of the discharge reveals that it has a pH of 5.0 and has a “fishy” odor when 10% KOH is
added to the discharge. Microscopic examination reveals vaginal cells coated with
coccobacillary organisms. Which of the following therapies is indicated?
A Acyclovir, 400 mg PO tid × 7 days

B Metronidazole, 2 g PO × 1
C Metronidazole, 500 mg PO bid × 7 days
D Fluconazole, 100 mg PO × 1
E Vaginal douching
ANSWER:C
1183
A 51-year-old woman is diagnosed with Plasmodium falciparum malaria after returning
from a safari in Tanzania. Her parasitemia is 6%, hematocrit is 21%, bilirubin is 7.8 mg/dL,
and creatinine is 2.7 mg/dL. She is still making 60 mL of urine per hour. She rapidly
becomes obtunded. Intensive care is initiated, with frequent creatinine checks, close
monitoring for hypoglycemia, infusion of phenobarbital for seizure prevention, mechanical
ventilation for airway protection, and exchange transfusion to address her high parasitemia.
Which of the following regimens is recommended as first-line treatment for her malarial
infection?
A Chloroquine
B Intravenous artesunate
C Intravenous quinine
D Intravenous quinidine
E Mefloquine
ANSWER:B
1184
A 9-year-old boy is brought to a pediatric emergency room by his father. He has had 2 days
of headache, neck stiffness, and photophobia and this morning had a temperature of 38.9°C
(102°F). He has also had several episodes of vomiting and diarrhea overnight. A lumbar
puncture is performed, which reveals pleocytosis in the cerebrospinal fluid (CSF). Which of
the following is true regarding enteroviruses as a cause of aseptic meningitis?
A An elevated CSF protein rules out enteroviruses as a cause of meningitis.
B Enteroviruses are responsible for up to 90% of aseptic meningitis in children.
C Lymphocytes will predominate in the CSF early on, with a shift to neutrophils at 24 h.
D Symptoms are more severe in children than in adults.
E They occur more commonly in the winter and spring.
ANSWER:B

1185
A 56-year-old man with a history of hypertension and cigarette smoking is admitted to the
intensive care unit after 1 week of fever and nonproductive cough. Imaging shows a new
pulmonary infiltrate, and urine antigen test for Legionella is positive. Each of the following
is likely to be an effective antibiotic except
A azithromycin
B aztreonam
C levofloxacin
D tigecycline
E trimethoprim/sulfamethoxazole
ANSWER:B
1186
Which of the following statements regarding HIV epidemiology in the United States is true as
of 2005?
A HIV incidence is currently decreasing among men who sleep with men.
B Heterosexual contact accounts for the majority of current HIV cases.
C
Minority women aged 13–19 from the southeastern United States account for a growing
proportion of prevalent HIV cases.
D
The proportion of cases due to high-risk heterosexual contact has decreased dramatically
over the past 20 years.
E The proportion of prevalent HIV cases due to injection drug use is currently increasing.
ANSWER:C
1187
A 48-year-old female presents to her physician with a 2-day history of fever, arthralgias,
diarrhea, and headache. She recently returned from an ecotour in tropical sub-Saharan
Africa, where she went swimming in inland rivers. Notable findings on physical examination
include a temperature of 38.7°C (101.7°F); 2-cm tender mobile lymph nodes in the axilla,
cervical, and femoral regions; and a palpable spleen. Her white blood cell count is 15,000/µL
with 50% eosinophils. She should receive treatments with which of the following
medications?
A Chloroquine
B Mebendazole
C Metronidazole
D Praziquantel
E Thiabendazole

ANSWER:D
1188
A 39-year-old woman received a liver transplant 2 years ago and is maintained on
prednisone, 5 mg, and cyclosporine A, 8 mg/kg per day. She has had two episodes of
rejection since transplant, as well an episode of cytomegalovirus syndrome and Nocardia
pneumonia. She intends on taking a 2-week gorilla-watching trip to Rwanda and seeks your
advice regarding her health while abroad. Which of the following potential interventions is
strictly contraindicated?
A Malaria prophylaxis
B Meningococcal vaccine
C Rabies vaccine
D Typhoid purified polysaccharid vaccine
E Yellow fever vaccine
ANSWER:E
1189
A 17-year-old woman presents to the clinic complaining of vaginal itchiness and malodorous
discharge. She is sexually active with multiple partners, and she is interested in getting tested
for sexually transmitted diseases. A wet-mount microscopic examination is performed, and
trichomonal parasites are identified. Which of the following statements regarding
trichomoniasis is true?
A A majority of women are asymptomatic.
B No treatment is necessary as disease is self-limited.
C The patient’s sexual partner need not be treated.
D Trichomoniasis can only be spread sexually.
E Trichomoniasis is 100% sensitive to metronidazole.
ANSWER:D
1190 The most common clinical presentation of infection with Babesia microti is
A acute hepatitis
B chronic meningitis
C generalized lymphadenopathy
D
overwhelming hemolysis, high-output congestive heart failure, respiratory failure, and
disseminated intravascular coagulation
E self-limited flulike illness
ANSWER:E

1191
When given as a first-line agent for invasive Aspergillus infection, voriconazole commonly
causes all of the following side effects except
A drug-drug interactions
B hepatotoxicity
C photosensitivity skin rashes
D renal toxicity
E visual disturbances
ANSWER:D
1192
A 42-year-old man with AIDS and a CD4+ lymphocyte count of 23 presents with shortness
of breath and fatigue in the absence of fevers. On examination, he appears chronically ill
with pale conjunctiva. Hematocrit is 16%. Mean corpuscular volume is 84. Red cell
distribution width is normal. Bilirubin, lactose dehydrogenase, and haptoglobin are all within
normal limits. Reticulocyte count is zero. White blood cell count is 4300, with an absolute
neutrophil count of 2500. Platelet count is 105,000. Which of the following tests is most
likely to produce a diagnosis?
A Bone marrow aspirate and biopsy
B Iron studies
C Parvovirus B19 IgG
D Parvovirus B19 polymerase chain reaction (PCR)
E Parvovirus B19 IgM
ANSWER:D
1193 All of the following are risk factors for the development of Legionella pneumonia except
A glucocorticoid use
B HIV infection
C neutropenia
D recent surgery
E tobacco use
ANSWER:C

1194
A 38-year-old female pigeon keeper who has no significant past medical history, is taking no
medications, has no allergies, and is HIV-negative presents to the emergency room with a
fever, headache, and mild nuchal rigidity. Neurologic examination is normal. Head CT
examination is normal. Lumbar puncture is significant for an opening pressure of 20
cmH2O, white blood cell count of 15 cells/ µL (90% monocytes), protein of 0.5 g/L (50
mg/mL), glucose of 2.8 mmol/L (50 mg/dL), and positive India ink stain. What is the
appropriate therapy for this patient?
A Amphotericin B for 2 weeks
B Amphotericin B with flucytosine for 2 weeks
C Amphotericin B for 2 weeks followed by oral fluconazole, 400 mg daily
D Amphotericin B for 10 weeks followed by oral fluconazole, 400 mg daily for 6–12 months
E Ceftriaxone and vancomycin for 2 weeks
ANSWER:D
1195
While attending the University of Georgia, a group of friends go on a 5-day canoeing and
camping trip in rural southern Georgia. A few weeks later, one of the campers develops a
serpiginous, raised, pruritic, erythematous eruption on the buttocks. Strongyloides larvae are
found in his stool. Three of his companions, who are asymptomatic, are also found to have
strongyloides larvae in their stool. Which of the following is indicated in the asymptomatic
carriers?
A Fluconazole
B Ivermectin
C Mebendazole
D Mefloquine
E Treatment only for symptomatic illness
ANSWER:B

1196
A 79-year-old man has had a diabetic foot ulcer overlying his third metatarsal head for 3
months but has not been compliant with his physician’s request to offload the affected foot.
He presents with dull, throbbing foot pain and subjective fevers. Examination reveals a
putrid-smelling wound notable also for a pus-filled 2.5 cm wide ulcer. A metal probe is used
to probe the wound and it detects bone as well as a 3-cm deep cavity. Gram stain of the pus
shows gram-positive cocci in chains, gram-positive rods, gram-negative diplococci, enteric-
appearing gram-negative rods, tiny pleomorphic gram-negative rods, and a predominance of
neutrophils. Which of the following empirical antibiotic regimens is recommended while
blood and drainage cultures are processed?
A Ampicillin/sulbactam, 1.5 g IV q4h
B Clindamycin, 600 mg PO tid
C Linezolid, 600 mg IV bid
D Metronidazole, 500 mg PO qid
E Vancomycin, 1g IV bid
ANSWER:A
1197 All of the following regarding herpes simplex virus (HSV)-2 infection are true except
A Approximately one in five Americans harbors HSV2 antibodies.
B
Asymptomatic shedding of HSV-2 in the genital tract occurs nearly as frequently in those
with no symptoms as in those with ulcerative disease.
C Asymptomatic shedding of HSV-2 is associated with transmission of virus.
D HSV-2 seropositivity is an independent risk factor for HIV transmission.
E Seroprevalence rates of HSV-2 are lower in Africa than in the United States.
ANSWER:E
1198
What is the most common manifestation of Coccidioides infection in an immunocompetent
host?
A Acute pneumonia
B Asymptomatic seroconversion
C Hypersensitivity phenomena such as erythema nodosum
D Meningitis
E Self limited flulike illness
ANSWER:B

1199
You are a physician working on a cruise ship traveling from Miami to the Yucatán
Peninsula. In the course of 24 h, 32 people are seen with acute gastrointestinal illness that is
marked by vomiting and watery diarrhea. The most likely causative agent of the illness is
A enterohemorrhagic Escherichia coli
B norovirus
C rotavirus
D Shigella
E Salmonella
ANSWER:B
1200 What is the best method for diagnosis?
A Acute and convalescent antibody titers
B Demonstration of Norwalk toxin in the stool
C Electron microscopy
D Isolation in cell culture
E Polymerase chain reaction (PCR) to identify the Norwalk-associated calcivirus
ANSWER:E
1201
A 32-year-old man presents with jaundice and malaise. He is found to have acute hepatitis B
with positive hepatitis B virus (HBV) DNA and E antigen. Which of the following antiviral
agents are approved as part of a therapeutic regimen for mono-infection with hepatitis B?
A Efavirenz
B Ganciclovir
C Lamivudine
D Rimantadine
E Tenofovir
ANSWER:C
1202
Which of the following factors is the most important determinant of the rate of disease
progression from initial HIV infection to clinical diagnosis of AIDS?
A Age
B CD4+ lymphocyte count 6 months after infection
C Cytomegalovirus (CMV) IgG status

D HIV resistance panel at infection
E HIV viral load set point 6 months after initial infection
ANSWER:E
1203
The standard starting regimen for acid-fast bacilli smear–positive active pulmonary
tuberculosis is
A isoniazid
B isoniazid, rifampin
C isoniazid, moxifloxacin, pyrazinamide, ethambutol
D isoniazid, rifampin, pyrazinamide, ethambutol
E rifampin, moxifloxacin, pyrazinamide, ethambutol
ANSWER:D
1204
All the following are common manifestations of cytomegalovirus (CMV) infection following
lung transplantation except
A bronchiolitis obliterans
B CMV esophagitis
C CMV pneumonia
D CMV retinitis
E CMV syndrome (fever, malaise, cytopenias, transaminitis, and CMV viremia)
ANSWER:D
1205
Which of the following statements regarding severe acute respiratory syndrome (SARS) is
true?
A SARS displays poor human-to-human transmission.
B SARS is more severe among children than adults.
C The etiologic agent of SARS is in the Adenovirus family.
D There have been no reported cases of SARS since 2004.
E There is no known environmental reservoir for the virus causing SARS.
ANSWER:D
1206
A 72-year-old woman is admitted to the intensive care unit with respiratory failure. She has
fever, obtundation, and bilateral parenchymal consolidation on chest imaging. Which of the
following is true regarding the diagnosis of Legionella pneumonia?
A Acute and convalescent antibodies are not helpful due to the presence of multiple serotypes.

B Legionella can never be seen on a Gram stain.
C Legionella cultures grow rapidly on the proper media.
D Legionella urinary antigen maintains utility after antibiotic use.
E Polymerase chain reaction (PCR) for Legionella DNA is the “gold standard” diagnostic test.
ANSWER:D
1207
Which of the following has resulted in a significant decrease in the incidence of trichinellosis
in the United States?
A
Adequate therapy that allows for eradication of infection in index cases before person-to-
person spread can occur
B Earlier diagnosis due to a new culture assay
C Federal laws limiting the import of foreign cattle
D Laws prohibiting the feeding of uncooked garbage to pigs
E Requirements for hand-washing by commercial kitchen staff who handle raw meat
ANSWER:D
1208
A 23-year-old woman is newly diagnosed with geni-tal herpes simplex virus (HSV)-2
infection. What can you tell her that the chance of reactivation disease will be during the first
year after infection?
A 0,05
B 0,25
C 0,5
D 0,75
E 0,9
ANSWER:E
1209 The most common cause of traveler’s diarrhea in Mexico is
A Campylobacter jejuni
B Entamoeba histolytica
C enterotoxigenic Escherichia coli
D Giardia lamblia
E Vibrio cholerae
ANSWER:C

1210
A patient comes to clinic and describes progressive muscle weakness over several weeks. He
has also experienced nausea, vomiting, and diarrhea. One month ago he had been
completely healthy and describes a bear hunting trip in Alaska, where they ate some of the
game they killed. Soon after he returned, his gastrointestinal (GI) symptoms began, followed
by muscle weakness in his jaw and neck that has now spread to his arms and lower back.
Examination confirms decreased muscle strength in the upper extremities and neck. He also
has slowed extraocular movements. Laboratory examination shows panic values for elevated
eosinophils and serum creatine phosphokinase. Which of the following organisms is most
likely the cause of his symptoms?
A Campylobacter
B Cytomegalovirus
C Giardia
D Taenia solium
E Trichinella
ANSWER:E
1211
Abacavir is a nucleoside transcription inhibitor that carries which side effect unique for HIV
antiretroviral agents?
A Fanconi’s anemia
B Granulocytopenia
C Lactic acidosis
D Lipoatrophy
E Severe hypersensitivity reaction
ANSWER:E

1212
A 30-year-old healthy woman presents to the hospital with severe dyspnea, confusion,
productive cough, and fevers. She had been ill 1 week prior with a flulike illness
characterized by fever, myalgias, headache, and malaise. Her illness almost entirely
improved without medical intervention until 36 h ago, when she developed new rigors
followed by progression of the respiratory symptoms. On initial examination, her
temperature is 39.6°C, pulse is 130 beats per minute, blood pressure is 95/60 mmHg,
respiratory rate is 40, and oxygen saturation is 88% on 100% face mask. On examination she
is clammy, confused, and very dyspneic. Lung examination reveals amphoric breath sounds
over her left lower lung fields. She is intubated and resuscitated with fluid and antibiotics.
Chest CT scan reveals necrosis of her left lower lobe. Blood and sputum cultures grow
Staphylococcus aureus. This isolate is likely to be resistant to which of the following
antibiotics?
A Doxycycline
B Linezolid
C Methicillin
D Trimethoprim/sulfamethoxazole (TMP/SMX)
E Vancomycin
ANSWER:C
1213 Helicobacter pylori colonization is implicated in all of the following conditions except
A duodenal ulcer disease
B gastric adenocarcinoma
C gastric mucosa-associated lymphoid tissue (MALT) lymphoma
D gastroesophageal reflux disease
E peptic ulcer disease
ANSWER:D
1214
A 24-year-old woman presents with diffuse arthralgias and morning stiffness in her hands,
knees, and wrists. Two weeks earlier she had a self-limited febrile illness notable for a red
facial rash and lacy reticular rash on her extremities. On examination, her bilateral wrists,
metacarpophalangeal joints, and proximal interphalangeal joints are warm and slightly
boggy. What test is most likely to reveal her diagnosis?
A Antinuclear antibody
B Chlamydia trachomatis ligase chain reaction of the urine

C Joint aspiration for crystals and culture
D Parvovirus B19 IgM
E Rheumatoid factor
ANSWER:D
1215
Which of the following statements regarding Clostridium difficile–associated disease relapses
is true?
A A first recurrence does not imply greater risk of further recurrences.
B Most recurrences are due to antibiotic resistance.
C
Recurrent C. difficile–associated disease has been associated with a higher risk of colon
cancer.
D Recurrent disease is associated with serious complications.
E Testing for clearance of C. difficile is warranted after treating recurrences.
ANSWER:D
1216
A 38-year-old man with HIV/AIDS presents with 4 weeks of diarrhea, fever, and weight
loss.Which of the following tests makes the diagnosis of cytomegalovirus (CMV) colitis?
A CMV IgG
B Colonoscopy with biopsy
C Serum CMV polymerase chain reaction (PCR)
D Stool CMV antigen
E Stool CMV culture
ANSWER:B

1217
A 46-year-old veterinary researcher who frequently operates on rats presents to the
emergency room with jaundice and scant hemoptysis. She recalls having a fairly deep cut on
her hand during an operation about 14 days prior. She has had no recent travel or other
animal exposures. Her illness started ~9 days prior with fever, chills, severe headache, intense
myalgias, and nausea. She also noted bilateral conjunctival injection. Thinking that she had
influenza infection, she stayed home from work and started to feel better 5 days into the
illness. However, within a day her symptoms had returned with worsening headache, and
soon thereafter she developed jaundice. On initial evaluation, her temperature is 38.6°C,
pulse is 105 beats per minute, and blood pressure is 156/89 mmHg with O2 saturations of
92% on room air. She appears acutely ill and is both icteric and profoundly jaundiced. Her
liver is enlarged and tender, but there are no palpable masses and she has no splenomegaly.
Laboratory results are notable for a BUN of 64, creatinine of 3.6, total bilirubin of 64.8
(direct 59.2), AST = 84, ALT = 103, alkaline phosphatase = 384, white blood cell (WBC)
count is 11,000 with 13% bands and 80% polymorphonuclear forms, hematocrit of 33%,
and platelets = 142. Urinalysis reveals 20 WBCs/hpf, 3+ protein, and granular casts.
Coagulation studies are within normal limits. Lumbar puncture reveals a sterile pleocytosis.
CT scan of the chest shows diffuse flamelike infiltrates consistent with pulmonary
hemorrhage. What is the likely diagnosis?
A Acute interstitial pneumonitis
B Acute myeloid leukemia
C Polyarteritis nodosum
D Rat bite fever (Streptobacillus moniliformis infection)
E Weil’s syndrome (Leptospira interrogans infection)
ANSWER:E
1218
A 17-year-old boy in Arkansas presents to a clinic in August with fever, headache, myalgias,
nausea, and anorexia 8 days after returning from a 1-week camping trip. Physical
examination is remarkable for a temperature of 38.6°C and a generally fatigued but nontoxic
appearing, well-developed young man. He does not have a rash, and orthostatic vital sign
measurements are negative. What would be a reasonable course of action?
A Initiate ceftriaxone, 1g IM × 1
B Initiate doxycycline, 100 mg PO bid

C Initiate oseltamivir, 75 mg PO qd
D Reassure the patient and order a heterophile antibody titer (Monospot)
E Reassure the patient and order rickettsial serologies
ANSWER:B
1219
A 26-year-old woman presents to the emergency department with fever, chills, backache,
and malaise. She reports a habit of active IV drug use; last use was 2 days ago. Her vital signs
show a temperature of 38.4°C, heart rate of 106/minute, respiratory rate of 22/minute, blood
pressure of 114/61 mmHg, and oxygen saturation of 98% on 2 L per nasal cannula. A chest
x-ray and subsequent chest CT scan demonstrate multiple peripheral nodular infiltrates with
cavitation. Blood cultures are sent to the laboratory and are pending. At this point in the
workup, how many minor criteria are met from the Duke criteria for the clinical diagnosis of
infective endocarditis?
A 0
B 1
C 2
D 3
E 5
ANSWER:D
1220 Which of the following is true regarding influenza prophylaxis?
A
Patients receiving an intramuscular influenza vaccine should be warned of the increased risk
of Guillain-Barré syndrome.
B Patients with hypersensitivity to eggs should not receive the intramuscular vaccine.
C
The intramuscular influenza vaccine is a live, attenuated strain of influenza that is based on
isolates from the previous year’s strains of influenza A and B.
D The intramuscular influenza vaccine should not be given to immunocompromised hosts.
E
The intranasal spray, “Flu-mist,” is an inactivated virus preparation based on the previous
year’s strains of influenza A and B.
ANSWER:B
1221
Which of the following is the most common manifestation of initial (primary) herpes
simplex virus (HSV)1 infection?
A Asymptomatic infection
B Genital ulcers

C Gingivostomatitis and pharyngitis
D Orolabial ulcers
E Trigeminal neuralgia
ANSWER:C
1222
A patient presents to the clinic complaining of nausea, vomiting, crampy abdominal pain,
and markedly increased flatus. The patient has not experienced any diarrhea or vomiting but
notes that he has been belching more than usual and he describes a “sulfur-like” odor when
he does so. He returned from a 3-week trip to Peru and Ecuador several days ago and notes
that his symptoms began about a week ago. Giardiasis is considered in the differential. Which
of the following is true regarding Giardia?
A Boiling water prior to ingestion will not kill Giardia cysts.
B
Giardia is a disease of developing nations; if this patient had not travelled, there would be no
likelihood of giardiasis.
C Hematogenous dissemination and eosinophilia are common.
D Ingestion of as few as 10 cysts can cause human disease.
E Lack of diarrhea makes the diagnosis of Giardia very unlikely.
ANSWER:D
1223
An 18-year-old man presents with a firm, nontender lesion around his anal orifice. The
lesion is about 1.5 cm in diameter and has a cartilaginous feel on clinical examination. The
patient reports that it has progressed to this stage from a small papule. It is not tender. He
reports recent unprotected anal intercourse. Bacterial culture of the lesion is negative. A rapid
plasmin reagin (RPR) test is also negative. Therapeutic interventions should include
A IM ceftriaxone, 1g
B IM penicillin G benzathine, 2.4 million U
C oral acyclovir, 200 mg 5 times per day
D observation
E surgical resection with biopsy
ANSWER:B

1224
A 17-year-old woman with a medical history of mild intermittent asthma presents to your
clinic in February with several days of cough, fever, malaise, and myalgias. She notes that
her symptoms started 3 days earlier with a headache and fatigue, and that several students
and teachers at her high school have been diagnosed recently with “the flu.” She did not
receive a flu shot this year. Which of the following medication treatment plans is the best
option for this patient?
A Aspirin and a cough suppressant with codeine
B Oseltamivir, 75 mg PO bid for 5 days
C Rimantadine, 100 mg PO bid for 1 week
D Symptom-based therapy with over-the-counter agents
E Zanamivir, 10 mg inhaled bid for 5 days
ANSWER:D
1225
One month after receiving a 14-day course of omeprazole, clarithromycin, and amoxicillin
for Helicobacter pylori–associated gastric ulcer disease, a 44-year-old woman still has mild
dyspepsia and pain after meals. What is the appropriate next step in management?
A Empirical long-term proton pump inhibitor therapy
B Endoscopy with biopsy to rule out gastric adenocarcinoma
C H. pylori serology testing
D Reassurance
E Urea breath test
ANSWER:E
1226
Which of the following medications used as antimycobacterial drugs require dose reduction
for patients with an estimated glomerular filtration rate <30 mL/min?
A Isoniazid
B Pyrazinamide
C Rifabutin
D Rifampin
E Streptomycin
ANSWER:E
1227
Which of the following statements regarding the currently licensed human papillomavirus
(HPV) vaccine (Gardasil) is true?

A It does not protect against genital warts.
B It is an inactivated live virus vaccine.
C
It is targeted towards all oncogenic strains of HPV but is only 70% effective at decreasing
infection in an individual.
D Once sexually active, women will derive little protective benefit from the vaccine.
E Vaccinees should continue to receive standard Pap smear testing.
ANSWER:E
1228
A 25-year-old woman presents with 1 day of fever to 38.3°C (101°F), sore throat,
dysphagia, and a number of grayish-white papulovesicular lesions on the soft palate, uvula,
and anterior pillars of the tonsils (Figure IV-57, Color Atlas). The patient is most likely
infected with which of the following?
A Candida albicans
B Coxsackievirus
C Herpesvirus
D HIV
E Staphylococcus lugdunensis
ANSWER:B
1229
There is wide concern among many members of the general public regarding which of the
following vaccines as a potential cause of autism?
A DTap (diphtheria and tetanus toxoid and acellular pertussis) vaccine
B Hepatitis B vaccine
C Hib (Haemophilus influenza type b) vaccine
D Human papilloma virus (HPV) vaccine
E Measles-mumps-rubella (MMR) vaccine
ANSWER:E

1230
A 19-year-old female from Guatemala presents to your office for a routine screening
physical examination. At age 4 years she was diagnosed with acute rheumatic fever. She does
not recall the specifics of her illness and remembers only that she was required to be on bed
rest for 6 months. She has remained on penicillin V orally at a dose of 250 mg bid since that
time. She asks if she can safely discontinue this medication. She has had only one other flare
of her disease, at age 8, when she stopped taking penicillin at the time of her emigration to
the United States. She is currently working as a day care provider. Her physical examination
is notable for normal point of maximal impulse (PMI) with a grade III/VI holosystolic
murmur that is heard best at the apex of the heart and radiates to the axilla. What do you
advise the patient to do?
A
An echocardiogram should be performed to determine the extent of valvular damage before
deciding if penicillin can be discontinued.
B Penicillin prophylaxis can be discontinued because she has had no flares in 5 years.
C She should change her dosing regimen to IM benzathine penicillin every 8 weeks.
D
She should continue on penicillin indefinitely as she had a previous recurrence, has
presumed rheumatic heart disease, and is working in a field with high occupational exposure
to
E
She should replace penicillin prophylaxis with polyvalent pneumococcal vaccine every 5
years.
ANSWER:D
1231
In a patient with bacterial endocarditis, which of the following echocardiographic lesions is
most likely to lead to embolization?
A 5-mm mitral valve vegetation
B 5-mm tricuspid valve vegetation
C 11-mm aortic valve vegetation
D 11-mm mitral valve vegetation
E 11-mm tricuspid valve vegetation
ANSWER:D
1232
Testing for latent Mycobacterium tuberculosis infection is indicated in HIV patients at the
time of initial diagnosis for all of the following reasons except
A
Active tuberculosis treatment success rates are lower in HIV-infected patients compared to
HIV-uninfected patients.

B
Drug interactions between drug regimens for active tuberculosis therapy and highly active
antiretroviral therapy are challenging to manage.
C
HIV-associated active tuberculosis is more likely to be extrapulmonary and can be
diagnostically challenging.
D HIV-infected patients with active tuberculosis have high 6-month HIV-related mortality rates.
E
The rate of progression from latent tuberculosis to active tuberculosis is higher in HIV-
infected persons compared to HIV-uninfected persons.
ANSWER:A
1233
A 19-year-old man presents to the emergency department with 4 days of watery diarrhea,
nausea, vomiting, and low-grade fever. He recalls no unusual meals, sick contacts, or travel.
He is hydrated with IV fluid, given antiemetics and discharged home after feeling much
better. Three days later two out of three blood cultures are positive for Clostridium
perfringens. He is called at home and says that he feels fine and is back to work. What should
your next instruction to the patient be?
A Return for IV penicillin therapy
B Return for IV penicillin therapy plus echocardiogram
C Return for IV penicillin therapy plus colonoscopy
D Return for surveillance blood culture
E Reassurance
ANSWER:E
1234 All of the following are clinical manifestations of Ascaris lumbricoides infection except
A asymptomatic carriage
B fever, headache, photophobia, nuchal rigidity, and eosinophilia
C nonproductive cough and pleurisy with eosinophilia
D right upper quadrant pain and fever
E small-bowel obstruction
ANSWER:D

1235
An 87-year-old nursing home resident is brought by ambulance to a local emergency room.
He is obtunded and ill-appearing. Per nursing home staff, the patient has experienced low-
grade temperatures, poor appetite, and lethargy over several days. A lumbar puncture is
performed, and the Gram stain returns gram-positive rods IV-65. and many white blood
cells. Listeria meningitis is diagnosed and appropriate antibiotics are begun. Which of the
following best describes a clinical difference between Listeria and other causes of bacterial
meningitis?
A More frequent nuchal rigidity.
B More neutrophils are present on the cerebrospinal fluid (CSF) differential.
C Photophobia is more common.
D Presentation is often more subacute.
E White blood cell (WBC) count is often more elevated in the CSF.
ANSWER:D
1236
Which of the following antibiotics has the weakest association with the development of
Clostridium difficile– associated disease?
A Ceftriaxone
B Ciprofloxacin
C Clindamycin
D Moxifloxacin
E Piperacillin/tazobactam
ANSWER:E
1237
All of the following statements regarding human T cell lymphotropic virus-I (HTLV-I)
infection are true except
A Acute T cell leukemia is associated with HTLV-I infection.
B HTLV-I endemic regions include southern Japan, the Caribbean, and South America.
C
HTLV-I infection is associated with a gradual decline in T cell function and
immunosuppression.
D HTLV-I is transmitted parenterally, sexually, and from mother to child.
E Tropical spastic paraparesis is associated with HTLV-I infection.
ANSWER:C

1238
A 33-year-old woman is undergoing consolidation chemotherapy for acute myelocytic
leukemia with cytarabine plus daunorubicin. She developed a fever 5 days prior which has
persisted despite the addition of cefepime and vancomycin to her prophylactic antibiotic
regimen of norfloxacin, fluconazole, and acyclovir. Other than diaphoresis and chills during
her periodic fevers, she remains largely asymptomatic except for a general sense of malaise
and nausea associated with her chemotherapy, as well as oral pain due to mucositis. She
remains neutropenic despite administration of hematopoietic growth factors. Blood, urine,
and sputum cultures all remain negative. What is the next step in her management?
A Addition of metronidazole
B Addition of tobramycin
C Change fluconazole to caspofungin
D Chest roentgenogram
E High-resolution CT plus serum galactomannan enzyme immunoassay
ANSWER:E
1239
Which of the following organisms is most likely to cause infection of a shunt implanted for
the treatment of hydrocephalus?
A Bacteroides fragilis
B Corynebacterium diphtheriae
C Escherichia coli
D Staphylococcus aureus
E Staphylococcus epidermidis
ANSWER:E
1240
A 3-year-old boy is brought by his parents to clinic. They state that he has experienced
fevers, anorexia, weight loss, and, most recently, has started wheezing at night. He had been
completely healthy until these symptoms started 2 months ago. The family had travelled
through Europe several months prior and reported no unusual exposures or exotic foods.
They have a puppy at home. On examination, the child is ill-appearing and is noted to have
hepatosplenomegaly. Laboratory results show a panic value of 82% eosinophils. Total white
blood cells are elevated. A complete blood count is repeated to rule out a laboratory error
and eosinophils are 78%. Which of the following is the most likely organism or process?

A Cysticercus
B Giardiasis
C Staphylococcus lugdunensis
D Toxocariasis
E Trichinellosis
ANSWER:D
1241
An otherwise healthy 5-year-old child presents with low-grade fevers, sore throat, and red,
itchy eyes. He attends summer camp, where several other campers were ill. On examination,
the patient is noted to have pharyngitis and bilateral conjunctivitis. Which of the following is
the most likely etiologic agent?
A Adenovirus
B Enterovirus
C Influenza virus
D Metapneumovirus
E Rhinovirus
ANSWER:A
1242
A 35-year-old male is seen 6 months after a cadaveric renal allograft. The patient has been
on azathioprine and prednisone since that procedure. He has felt poorly for the past week
with fever to 38.6°C (101.5°F), anorexia, and a cough productive of thick sputum. Chest x-
ray reveals a left lower lobe (5 cm) nodule with central cavitation. Examination of the
sputum reveals long, crooked, branching, beaded gram-positive filaments. The most
appropriate initial therapy would include the administration of which of the following
antibiotics?
A Ceftazidime
B Erythromycin
C Penicillin
D Sulfisoxazole
E Tobramycin
ANSWER:D

1243
A 53-year-old male with a history of alcoholism presents with an enlarging mass at the angle
of the jaw. The patient describes the mass slowly enlarging over a period of 6 weeks with
occasional associated pain. He has also noted intermittent fevers throughout this period.
Recently, he has developed yellowish drainage from the inferior portion of the mass. He
takes no medications and has no other past history. He drinks six beers daily. On physical
examination, the patient has a temperature of 37.9°C (100.2°F). His dentition is poor. There
is diffuse soft tissue swelling and induration at the angle of the mandible on the left. It is
mildly tender, and no discrete mass is palpable. The area of swelling is ~8 × 8 cm. An
aspirate is sent for Gram stain and culture. The culture initially grows Eikenella corrodens.
After 7 days you receive a call reporting growth of a gram-positive bacillus branching at
acute angles on anaerobic media. What organism is causing this man’s clinical presentation?
A Actinomyces
B Eikenella corrodens
C Mucormycosis
D Nocardia
E Peptostreptococcus
ANSWER:A
1244 What is the most appropriate therapy for this patient?
A Amphotericin B
B Itraconazole
C Penicillin
D Surgical debridement
E Tobramycin
ANSWER:C
1245
A 40-year-old male smoker with a history of asthma is admitted to the inpatient medical
service with fever, cough, brownish-green sputum, and malaise. Physical examination shows
a respiratory rate of 15, no use of accessory muscles of breathing, and bilateral polyphonic
wheezes throughout the lung fields. There is no clubbing or skin lesions. You consider a
diagnosis of allergic bronchopulmonary aspergillosis. All the following clinical features are
consistent with allergic bronchopulmonary aspergillosis except
A bilateral, peripheral cavitary lung infiltrates

B elevated serum IgE
C peripheral eosinophilia
D positive serum antibodies to Aspergillus species
E positive skin testing for Aspergillus species
ANSWER:A
1246
All of the following factors increase the risk for Clostridium difficile–associated disease
except
A antacids
B antecedent antibiotics
C difficile colonization
D enteral tube feeds
E increasing length of hospital stay
ANSWER:C
1247
A 19-year-old man presents to an urgent care clinic with urethral discharge. He reports three
new female sexual partners over the past 2 months. What should his management be?
A
Nucleic acid amplification test for Neisseria gonorrhoeae and Chlamydia trachomatis and
return to clinic in 2 days
B Cefpodoxime, 400 mg PO × 1, and azithromycin, 1g PO × 1 for the patient and his partners
C
Nucleic acid amplification test for N. gonorrhoeae and C. trachomatis plus cefpodoxime, 400
mg PO × 1, and azithromycin, 1 g PO × 1, for the patient
D
Nucleic acid amplification test for N. gonorrhoeae and C. trachomatis plus cefpodoxime, 400
mg PO × 1, and azithromycin, 1g PO × 1, for the patient and his recent partners
E
Nucleic acid amplification test for N. gonorrhoeae and C. trachomatis plus cefpodoxime, 400
mg PO × 1, azithromycin, 1g PO × 1, and flagyl, 2 g PO
ANSWER:D
1248
During the first 2 weeks following solid organ transplantation, which family of infection is
most common?
A Cytomegalovirus (CMV) and Epstein-Barr virus (EBV) reactivation
B
Humoral immunodeficiency–associated infections (e.g., meningococcemia, invasive
Streptococcus pneumoniae infection)
C Neutropenia-associated infection (e.g., aspergillosis, candidemia)

D
T cell deficiency–associated infections (e.g., Pneumocystis jiroveci, nocardiosis,
cryptococcosis)
E
Typical hospital-acquired infections (e.g., central line infection, hospital-acquired
pneumonia, urinary tract infection)
ANSWER:E
1249
A 19-year-old college student is brought to the emergency department by friends from his
dormitory for confusion and altered mental status. They state that many colleagues have
upper respiratory tract infections. He does not use alcohol or illicit drugs. His physical
examination is notable for confusion, fever, and a rigid neck. Cerebrospinal fluid (CSF)
examination reveals a white blood cell count of 1800 cells/µL with 98% neutrophils, glucose
of 1.9 mmol/L (35 mg/dL), and protein of 1.0 g/L (100 mg/dL). Which of the following
antibiotic regimens is most appropriate as initial therapy?
A Ampicillin plus vancomycin
B Ampicillin plus gentamicin
C Cefazolin plus doxycycline
D Cefotaxime plus doxycycline
E Cefotaxime plus vancomycin
ANSWER:E
1250
In addition to antibiotics, which of the following adjunctive therapies should be
administered to improve the chance of a favorable neurologic outcome?
A Dexamethasone
B Dilantin
C Gabapentin
D L-Dopa
E Parenteral nutrition
ANSWER:A
1251
Which of the following viruses is the leading cause of respiratory disease in infants and
children?
A Adenovirus
B Enterovirus
C Human respiratory syncytial virus
D Parainfluenza virus

E Rhinovirus
ANSWER:C
1252
Several family members present to a local emergency room 2 days after a large family
summer picnic where deli meats and salads were served. They all complain of profuse
diarrhea, headaches, fevers, and myalgias. Their symptoms began ~24 h after the picnic. It
appears that everyone who ate Aunt Emma’s bologna surprise was afflicted. Routine cultures
of blood and stool are negative to date. Which of the following is true regarding Listeria
gastroenteritis?
A Antibiotic treatment is not necessary for uncomplicated cases.
B Carriers are asymptomatic but can easily spread infection via the fecal-oral route.
C Gastrointestinal (GI) illness can result from ingestion of a single organism.
D Illness is toxin-mediated, and organisms are not present at the time of infection.
E Person-to-person spread is a common cause of outbreaks.
ANSWER:A
1253
Which clinical entity is the most difficult to distinguish from osteomyelitis in a diabetic foot
on any currently available medical imaging (plain film, CT, MRI, ultrasound, and three-
phase bone scan)?
A Abscess
B Cellulitis
C Fracture
D Neuropathic osteopathy
E Tumor
ANSWER:D
1254
The human enterovirus family includes poliovirus, coxsackieviruses, enteroviruses, and
echovirus. Which of the following statements regarding viral infection with one of the
members of this group is true?
A Among children infected with poliovirus, paralysis is common.
B Enteroviruses cannot be transmitted via blood transfusions and insect bites.
C In utero exposure to maternal enteroviral antibodies is not protective.
D
Infections are most common in adolescents and adults, though serious illness is most
common in young children.
E Paralysis from poliovirus infection was more commonly seen in developing countries.

ANSWER:B
1255
Which of the following sexually transmitted infections (STIs) is the most common in the
United States?
A Gonorrhea
B Herpes simplex virus (HSV) 2 infection
C HIV-1 infection
D Human papilloma virus infection
E Syphilis
ANSWER:D
1256
A 38-year-old woman presents to the emergency department with severe abdominal pain.
She has no past medical or surgical history. She recalls no recent history of abdominal
discomfort, diarrhea, melena, bright red blood per rectum, nausea, or vomiting prior to this
acute episode. She ate ceviche (lime-marinated raw fish) at a Peruvian restaurant 3 h prior to
presentation. On examination, she is in terrible distress and has dry heaves. Temperature is
37.6°C; heart rate is 128 beats per minute; blood pressure is 174/92 mmHg. Examination is
notable for an extremely tender abdomen with guarding and rebound tenderness. Bowel
sounds are present and hyperactive. Rectal examination is normal and guaiac test is negative.
Pelvic examination is unremarkable. White blood cell count is 6738/µL; hematocrit is 42%.
A complete metabolic panel and lipase and amylase levels are all within normal limits. CT of
the abdomen shows no abnormality. What is the next step in her management?
A CT angiogram of the abdomen
B Pelvic ultrasonography
C Proton pump inhibitor therapy and observation
D Right upper quadrant ultrasonography
E Upper endoscopy
ANSWER:E
1257
Which of the following clinical features can be used to rule out malaria in favor of another
tropical febrile illness in a returning traveler?
A Diarrhea
B Lack of paroxysmal nature of the fevers
C Lack of splenomegaly

D Severe myalgias and retroorbital headache
E None of the above
ANSWER:E
1258
Which of the following serology patterns places a transplant recipient at the lowest risk of
developing cytomegalovirus (CMV) infection after renal transplantation?
A Donor CMV IgG negative, recipient CMV IgG negative
B Donor CMV IgG negative, recipient CMV IgG positive
C Donor CMV IgG positive, recipient CMV IgG negative
D Donor CMV IgG positive, recipient CMV IgG positive
E The risk is equal regardless of serology results
ANSWER:A
1259 Which of the following statements regarding liver abscesses is true?
A Amebic liver abscess should be ruled out only by direct sampling and culture of pus.
B
Alkaline phosphatase is the most likely liver function test to be abnormal in the presence of a
liver abscess.
C
Candida species are most commonly isolated from patients with abscesses that develop as a
result of peritoneal or pelvic pathology.
D Patients with liver abscesses nearly always have right upper quadrant pain.
E All of the above
ANSWER:B
1260
All of the following antifungal medications may be used for the treatment of Candida
albicans fungemia except
A amphotericin B
B caspofungin
C fluconazole
D terbinafine
E voriconazole
ANSWER:D

1261
A 40-year-old male is admitted to the hospital with 2–3 weeks of fever, tender lymph nodes,
and right upper quadrant abdominal pain. He reports progressive weight loss and malaise
over a year. On examination, he is found to be febrile and frail with temporal wasting and
oral thrush. Matted, tender anterior cervical lymphadenopathy <1 cm and tender
hepatomegaly are noted. He is diagnosed with AIDS (CD4+ lymphocyte count = 12/µL and
HIV RNA 650,000 copies/mL). Blood cultures grow Mycobacterium avium. He is started on
rifabutin and clarithromycin, as well as dapsone for Pneumocystis prophylaxis, and
discharged home 2 weeks later after his fevers subside. He follows up with an HIV provider
4 weeks later and is started on tenofovir, emtricitabine and efavirenz. Two weeks later he
returns to clinic with fevers, neck pain, and abdominal pain. His temperature is 39.2°C, heart
rate is 110 beats per minute, blood pressure is 110/64 mmHg, and oxygen saturations are
normal. His cervical nodes are now 2 centimeters in size and extremely tender, and one has
fistulized to his skin and is draining yellow pus that is acid-fast bacillus stain–positive. His
hepatomegaly is pronounced and tender. What is the most likely explanation for his
presentation?
A Cryptococcal meningitis
B HIV treatment failure
C Immune reconstitution syndrome to Mycobacterium avium
D Kaposi’s sarcoma
E Mycobacterium avium treatment failure due to drug resistance
ANSWER:C
1262
Which of the following statements regarding prevention of human respiratory syncytial virus
(HRSV) infection in children is true?
A
All children who are admitted to the hospital more than twice a year should be vaccinated
against HRSV.
B Barrier precautions remain the only effective means of prevention.
C Children should be vaccinated at birth.
D Inactivated, whole-virus vaccine should be considered in children <2 years old.
E
RSV immune globulin should be given monthly to children <2 years old who were born
prematurely.
ANSWER:E

1263
A 52-year-old woman with alcoholic cirrhosis, portal hypertension, esophageal varices, and
history of hepatic encephalopathy presents to the hospital with confusion over several days.
Her husband remarks that the patient has been adherent to her medicines. These medicines
include labetalol, furosemide, aldactone, and lactulose. Physical examination is notable for
temperature of 38.3°C, heart rate of 115 bpm, blood pressure of 105/62 mmHg, respiratory
rate of 12 breaths per minute, and oxygen saturation of 96% on room air. The patient is
extremely drowsy, only intermittently able to answer questions, and disoriented. She has
slight asterixis. Lungs are clear. Cardiac examination is unremarkable. Her abdomen is
distended and tense but nontender. She has 3+ lower extremity edema extending to her
thighs. She is guaiac negative. Her cranial nerves and extremity strength are symmetric and
normal. Laboratory studies reveal a leukocyte count of 4830/µL, hematocrit = 33% (baseline
= 30%), and platelet count of 94,000/µL. Basic metabolic panel is unremarkable. What is an
essential component of the diagnostic workup?
A CT scan of the head
B Esophagastroduodenoscopy
C Paracentesis
D Therapeutic trial of lactulose
E Serum ammonia level
ANSWER:C
1264
A 64-year-old female is admitted to the hospital with altered mental status. She recently
returned from a summer white-water rafting trip in Colorado. Her husband reports increasing
confusion, alternating lethargy and agitation, and visual hallucinations over the past 3 days.
There is no history of drug abuse or psychiatric illness. She takes no medications. Her
physical examination is notable for a temperature of 39°C (102.2°F), myoclonic jerks, and
hyperreflexia. She is delirious and oriented to person only when aroused. There is no nuchal
rigidity. Cerebrospinal fluid (CSF) examination reveals clear fluid with a white blood cell
count of 15 cells/µL with 100% lymphocytes, protein of 1.0 g/L (100 mg/dL), and glucose
of 4.4 mmol/L (80 mg/ dL). Gram stain of the CSF shows no organisms. You suspect
infection with West Nile virus. Which of the following studies will be most useful in making
that diagnosis?
A CSF culture

B CSF IgM antibodies
C CNS MRI
D CSF PCR
E Stool culture
ANSWER:B
1265
Which of the following represents a rare but serious extrapulmonary complication of
influenza infection?
A Diffuse eczematous rash
B Myositis
C Oligoarthritis
D Purulent conjunctivitis
E Secondary bacterial pneumonia caused by Staphylococcus aureus
ANSWER:B
1266
You are a physician for an undergraduate university health clinic in Arizona. You have
evaluated three students with similar complaints of fever, malaise, diffuse arthralgias, cough
without hemoptysis, and chest discomfort, and one of the patients has a skin rash on her
extremities consistent with erythema multiforme. Chest radiography is similar in all three,
with hilar adenopathy and small pleural effusions. Upon further questioning you learn that
all three students are in the same archaeology class and participated in an excavation 1 week
ago. Your leading diagnosis is
A mononucleosis
B primary pulmonary aspergillosis
C primary pulmonary coccidioidomycosis
D primary pulmonary histoplasmosis
E streptococcal pneumonia
ANSWER:C

1267
A 34-year-old recent immigrant from Burundi presents with fever, headache, severe
myalgias, photophobia, conjunctival injection, and prostration. He lived in a refugee camp
for the previous 10 years. In the camp, he was treated for several unknown febrile illnesses.
Since arriving in the United States 7 years ago, he has worked as a computer analyst and
lived only in a metropolitan Northwest city with no significant travel. Initial blood cultures
are negative. Five days into the illness he develops hypotension, pneumonitis,
encephalopathy, and gangrene of his distal digits as well as a petechial, hemorrhagic rash
over his entire body except for his face. A biopsy of his rash reveals immunohistochemical
changes consistent with a rickettsial infection. Which of the following rickettsial pathogens is
most likely in this patient?
A Coxiella burnetii (Q fever)
B Rickettsia africae (African tick-borne fever)
C Rickettsia prowazekii (Louse-borne typhus)
D Rickettsia rickettsii (Rocky Mountain spotted fever)
E Rickettsia typhi (Murine typhus)
ANSWER:C
1268
You are the on-call physician practicing in a suburban community. You receive a call from a
28-year-old female with a past medical history significant for sarcoidosis who is currently on
no medications. She is complaining of the acute onset of crampy diffuse abdominal pain and
multiple episodes of emesis that are nonbloody. She has not had any light-headedness with
standing or loss of consciousness. When questioned further, the patient states that her last
meal was 5 h previously, when she joined her friends for lunch at a local Chinese restaurant.
She ate from the buffet, which included multiple poultry dishes and fried rice. What should
you do for this patient?
A Ask the patient to go to the nearest emergency department for resuscitation with IV fluids.
B Initiate antibiotic therapy with azithromycin.
C
Reassure the patient that her illness is self-limited and no further treatment is necessary if she
can maintain adequate hydration.
D Refer the patient for CT to assess for appendicitis.
E
Refer the patient for admission for IV vancomycin and ceftriaxone because of her
immunocompromised state resulting from sarcoidosis.
ANSWER:C

1269
Borrelia burgdorferi serology testing is indicated for which of the following patients, all of
whom reside in Lyme-endemic regions?
A
A 19-year-old female camp counselor who presents with her second episode of an inflamed,
red and tender left knee and right ankle
B
A 23-year-old male house painter who presents with a primary erythema migrans lesion at
the site of a witnessed tick bite
C
A 36-year-old female state park ranger who presents with a malar rash, diffuse
arthralgias/arthritis of her shoulders, knees, metacarpophalangeal and proximal
interphalangeal joints; pericarditis; and
D A 42-year-old woman with chronic fatigue, myalgias, and arthralgias
E
A 46-year-old male gardener who presents with fevers, malaise, migratory
arthralgias/myalgias, and three erythema migrans lesions
ANSWER:A
1270
A 39-year-old injection drug user with a history of right-sided endocarditis and HIV
infection notes back pain and fevers over the past week. He had an abscess recently on his
right arm that he drained on his own. He is part of a needle-exchange program and always
cleans his arm before shooting heroin into the vein in his antecubital fossa. On physical
examination, he has a temperature of 38.1°C, heart rate of 124 beats per minute, and blood
pressure of 75/30 mmHg. He is in a great deal of distress and is slightly confused. He has a
4/6 left lower sternal border murmur that varies with the respiratory cycle. His jugular
venous pressure is monophasic and to the jaw when seated at 90 degrees. Lung examination
is clear. Abdomen is benign. He is very tender over his lower spine. His extremities are
warm. Leg strength is 5/5 on the right, with 4/5 left hip flexion and extension, 3/5 left knee
flexion and extension, and 3/5 left foot extension. His Babinski reflex is upgoing on the left
and downgoing on the right. What is the next step in management?
A
Avoidance of antibiotics until more definitive culture data is obtained; serial neurologic
examinations
B Urgent MRI and neurosurgical consultation; vancomycin after blood cultures are drawn
C
Urgent MRI and neurosurgical consultation; vancomycin plus cefepime after blood cultures
are drawn

D
Urgent MRI and neurosurgical consultation; avoidance of antibiotics until more definitive
culture data are obtained
E Vancomycin plus cefepime after blood cultures are drawn; serial neurologic examinations
ANSWER:C
1271
An HIV-positive patient with a CD4 count of 110/ µL who is not taking any medications
presents to an urgent care center with complaints of a headache for the past week. He also
notes nausea and intermittently blurred vision. Examination is notable for normal vital signs
without fever but mild papilledema. Head CT does not show dilated ventricles. The definitive
diagnostic test for this patient is
A cerebrospinal fluid (CSF) culture
B MRI with gadolinium imaging
C ophthalmologic examination including visual field testing
D serum cryptococcal antigen testing
E urine culture
ANSWER:A
1272 Which of the following favors a diagnosis of acute bacterial epididymitis?
A A solid nontender testicular mass
B Absence of blood flow on Doppler examination
C Concurrent urethral discharge
D Elevation of the testicle within the scrotal sac
E Lack of response to ceftriaxone plus doxycycline therapy
ANSWER:C
1273
A 19-year-old woman comes to your office after being bitten by a bat on the ear while
camping in a primitive shelter. She is unable to produce a vaccination record. On physical
examination, she is afebrile and appears well. There are two small puncture marks on the
pinna of her left ear. What is an appropriate vaccination strategy in this context?
A Intravenous ribavirin
B No vaccination
C Rabies immunoglobulins
D Rabies inactivated virus vaccine
E Rabies inactivated virus vaccine plus immunoglobulins
ANSWER:E

1274
A 26-year-old woman during a clinic is found to have a positive rapid plasmin reagin test
(1:4) and a positive fluorescent treponemal antibody-absorption test (FTA-ABS). She has
never been treated for syphilis. She recalls a large painless ulcer on her labia 9 months prior,
followed about 2 months later by a diffuse rash and oral lesions that also resolved. She has
had five sexual contacts in the past year. In addition to treating the patient, all of the
following additional interventions should be considered except
A echocardiogram looking at the aortic arch
B HIV counseling and testing
C pregnancy testing
D screening and treatment of all recent sexual contacts
E screening for other sexually transmitted diseases (STDs)
ANSWER:A
1275
Per-coital rate of HIV acquisition in a man who has unprotected sexual intercourse with an
HIV-infected female partner is likely to increase under which of the following circumstances?
A Acute HIV infection in the female partner
B Female herpes simplex virus (HSV)-2 positive serostatus
C Male nongonococcal urethritis at the time of intercourse
D Uncircumcised male status
E All of the above
ANSWER:E
1276
All of the following are associated with increased risk of pelvic inflammatory disease (PID)
except
A bacterial vaginosis
B history of salpingitis
C intrauterine device
D recent sexual exposure to a man with urethritis
E symptoms beginning on days 14–21 of the menstrual cycle
ANSWER:E
1277
Current Centers for Disease Control and Prevention (CDC) recommendations are that
screening for HIV be performed in which of the following?

A
All high-risk groups (injection drug users, men who have sex with men, and high-risk
heterosexual women)
B All U.S. adults
C Injection drug users
D Men who have sex with men
E Women who have sex with more than two men per year
ANSWER:B
1278
A 26-year-old woman presents late in the third trimester of her pregnancy with high fevers,
myalgias, backache, and malaise. She is admitted and started on empirical broad-spectrum
antibiotics. Blood cultures return positive for Listeria monocytogenes. She delivers a 5-lb
infant 24 h after admission. Which of the following statements regarding antibiotic treatment
for this infection is true?
A Clindamycin should be used in patients with penicillin allergy.
B Neonates should receive weight-based ampicillin and gentamicin.
C Penicillin plus gentamicin is first-line therapy for the mother.
D Quinolones should be used for Listeria bacteremia in late-stage pregnancy.
E Trimethoprim-sulfamethoxazole has no efficacy against Listeria.
ANSWER:B
1279
Glucocorticoids have been shown to be of benefit for treatment for all of the following
infections except
A Aspergillus fumigatus pneumonia
B Mycobacterium tuberculosis pericarditis
C Pneumocystis carinii pneumonia
D severe typhoid fever
E Streptococcus pneumoniae meningitis
ANSWER:A
1280
A 23-year-old previously healthy female letter carrier works in a suburb in which the
presence of rabid foxes and skunks has been documented. She is bitten by a bat, which then
flies away. Initial examination reveals a clean break in the skin in the right upper forearm.
She has no history of receiving treatment for rabies and is unsure about vaccination against
tetanus. The physician should
A clean the wound with a 20% soap solution

B clean the wound with a 20% soap solution and administer tetanus toxoid
C
clean the wound with a 20% soap solution, administer tetanus toxoid, and administer human
rabies immune globulin intramuscularly
D
clean the wound with a 20% soap solution, administer tetanus toxoid, administer human
rabies immune globulin IM, and administer human diploid cell vaccine
E clean the wound with a 20% soap solution and administer human diploid cell vaccine
ANSWER:D
1281
A patient is admitted with fevers, malaise, and diffuse joint pains. His initial blood cultures
reveal methicillin-resistant Staphylococcus aureus (MRSA) in all culture bottles. He has no
arthritis on examination, and his renal function is normal. Echocardiogram shows a 5-mm
vegetation on the aortic valve. He is initiated on IV vancomycin at 15 mg/kg every 12 h.
Four days later the patient remains febrile and cultures remain positive for MRSA. In
addition to a search for embolic foci of infection, which of the following changes would you
make to his treatment regimen?
A No change
B Add gentamicin
C Add rifampin
D Check the vancomycin serum peak and trough levels and consider tid dosing
E Discontinue vancomycin, start daptomycin
ANSWER:A
1282
A 23-year-old woman develops cytomegalovirus (CMV) pneumonitis 5 months after a lung
transplant. She developed severe side effects from ganciclovir while receiving prophylaxis.
Foscarnet is prescribed for this episode. Which of the following side effects is most likely?
A Bone marrow suppression
B Electrolyte wasting
C Embryotoxic
D Lethargy and tremors
E Hyperkalemia
ANSWER:B

1283
A 38-year-old woman is seen in clinic for a decrease in cognitive and executive function.
Her husband is concerned because she is no longer able to pay bills, keep appointments, or
remember important dates. She also seems to derive considerably less pleasure from caring
for her children and her hobbies. She is unable to concentrate for long enough to enjoy
movies. This is a clear change from her functional status 6 months prior. A workup reveals a
positive HIV antibody by enzyme immunoassay and Western blot. Her CD4+ lymphocyte
count is 378/µL with a viral load of 78,000/mL. She is afebrile with normal vital signs. Her
affect is blunted, and she seems disinterested in the medical interview. Neurologic
examination for strength, sensation, cerebellar function and cranial nerve function is
nonfocal. Fundoscopic examination is normal. Mini-Mental Status Examination score is
22/30. A serum rapid plasmin reagin (RPR) test is negative. MRI of the brain shows only
cerebral atrophy disproportionate to her age but no focal lesions. What is the next step in her
management?
A Antiretroviral therapy
B Cerebrospinal fluid (CSF) JV virus polymerase chain reaction (PCR)
C CSF mycobacterial PCR
D CSF VDRL test
E Serum cryptococcal antigen
ANSWER:A
1284
A 72-year-old male is admitted to the hospital with bacteremia and pyelonephritis. He is
HIV-negative and has no other significant past medical history. Two weeks into his treatment
with antibiotics a fever evaluation reveals a blood culture positive for Candida albicans.
Examination is unremarkable. White blood cell count is normal. The central venous catheter
is removed, and systemic antifungal agents are initiated. What further evaluation is
recommended?
A Abdominal CT scan to evaluate for abscess
B Chest x-ray
C Funduscopic examination
D Repeat blood cultures
E Transthoracic echocardiogram
ANSWER:C

1285
A 40-year-old man with HIV (CD4+ lymphocyte count = 180, viral load = 1000 copies/mL)
was treated for secondary syphilis based on generalized painless lymphadenopathy, a diffuse
maculopapular rash that included his palms and soles, and a preceding primary genital
chancre. He reported no neurologic or ophthalmic symptoms at the time and received one
dose of IM penicillin G benzathine. At the time of diagnosis, his rapid plasmin reagin (RPR)
titer was 1:64 and fluorescent treponemal antibody-absorption (FTA-ABS) test was positive.
He follows up a year later and is found to have an RPR titer of 1:64 and his FTA-ABS
remains positive. What is the appropriate intervention at this time?
A Aqueous penicillin G 24 mU/d IV given as 4 mU q4h × 10 days
B Doxycycline, 100 mg PO bid
C Lumbar puncture
D Penicillin desensitization
E Penicillin G benzathine 2.4 mU IM weekly × 3 doses
ANSWER:C
1286 A person with liver disease caused by Schistosoma mansoni would be most likely to have
A ascites
B esophageal varices
C gynecomastia
D jaundice
E spider nevi
ANSWER:B

1287
A previously healthy 28-year-old male describes several episodes of fever, myalgia, and
headache that have been followed by abdominal pain and diarrhea. He has experienced up to
10 bowel movements per day. Physical examination is unremarkable. Laboratory findings
are notable only for a slightly elevated leukocyte count and an elevated erythrocyte
sedimentation rate. Wright’s stain of a fecal sample reveals the presence of neutrophils.
Colonoscopy reveals inflamed mucosa. Biopsy of an affected area discloses mucosal
infiltration with neutrophils, monocytes, and eosinophils; epithelial damage, including loss of
mucus; glandular degeneration; and crypt abscesses. The patient notes that several months
ago he was at a church barbecue where several people contracted a diarrheal illness.
Although this patient could have inflammatory bowel disease, which of the following
pathogens is most likely to be responsible for his illness?
A Campylobacter
B Escherichia coli
C Norwalk agent
D Staphylococcus aureus
E Salmonella
ANSWER:A
1288
Deficits in the complement membrane attack complex (C5-8) are associated with recurrent
infections of what variety?
A Pseudomonas aeruginosa
B Catalase-positive bacteria
C Streptococcus pneumoniae
D Salmonella spp.
E Neisseria meningitis
ANSWER:E

1289
A previously healthy 17-year-old woman presents in early October with profound fatigue
and malaise, as well as fevers, headache, nuchal rigidity, diffuse arthralgias, and a rash. She
lives in a small town in Massachusetts and spent her summer as a camp counselor at a local
day camp. She participated in daily hikes in the woods but did not travel outside of the area
during the course of the summer. Physical examination reveals a well-developed young
woman who appears extremely fatigued but not in extremis. Her temperature is 37.4°C; pulse
is 86 beats per minute; blood pressure is 96/54 mmHg; respiratory rate is 12 breaths per
minute. Physical examination documents clear breath sounds, no cardiac rub or murmur,
normal bowel sounds, a nontender abdomen, no organomegaly, and no evidence of
synovitis. Several cutaneous lesions are noted on her lower extremities, bilateral axillae, right
thigh, and left groin (Figure IV-121, Color Atlas). All of the following are possible
complications of her current disease state except
A Bell’s palsy
B large joint oligoarticular arthritis
C meningitis
D progressive dementia
E third-degree heart block
ANSWER:D
1290 In the patient described above, which of the following is appropriate therapy?
A Azithromycin, 500 mg PO daily
B Ceftriaxone, 2 g IV daily
C Cephalexin, 500 mg PO bid
D Doxycycline, 100 mg PO bid
E Vancomycin, 1 g IV bid
ANSWER:D
1291
IV-123. Which of the following represents an emergent (same day) indication for cardiac
surgery in a patient with infective endocarditis?
A Culture-proven fungal endocarditis
B Culture-proven resistant organism with septic pulmonary emboli
C Prosthetic valve endocarditis 4 months after surgery
D Sinus of Valsalva abscess ruptured into right heart
E Staphylococcus lugdunensis in a patient with previous history of endocarditis

ANSWER:D
1292
Which of the following pathogens are cardiac transplant patients at unique risk for acquiring
from the donor heart early after transplant when compared to other solid organ transplant
patients?
A Cryptococcus neoformans
B Cytomegalovirus
C Pneumocystis jiroveci
D Staphylococcus aureus
E Toxoplasma gondii
ANSWER:E
1293
A 68-year-old woman has been in the medical intensive care unit for 10 days with a chronic
obstructive pulmonary disease flare and pneumonia, including the initial 6 days on a
mechanical ventilator. She just finished a course of moxifloxacin and glucocorticoid taper
when she develops abdominal discomfort over 2 days. Vital signs reveal a temperature of
38.2°C, heart rate of 94 beats per minute, blood pressure of 162/94 mmHg, respiratory rate
of 18 per minute, and oxygen saturation of 90%. On examination, she is in moderate
distress. She is not using accessory muscles but is tachypneic. She has a slight bilateral
wheeze with good air movement. Heart sounds are distant and unchanged. Her abdomen is
moderately distended and tense, with scant bowel sounds present. There is no guarding or
rebound, but she is tender throughout. Review of her records reveals no bowel movement
over the past 72 h and no stool is palpable in the rectal vault. White blood cell count has
increased from 7100/µL to 38,000/µL over the past 2 days. Abdominal plain film shows
what is read as a probable ileus in the right lower quadrant. Aside from nasogastric (NG)
tube placement with suction and NPO status, which of the following should your
management also include?
A Intravenous immunoglobulin (IVIg)
B Metronidazole, 500 mg IV tid
C Piperacillin/tazobactam, 3.37 g IV q6h
D Restart moxifloxacin, 400 PO qd
E Vancomycin, 500 mg PO qid
ANSWER:B

1294
A 25-year-old woman presents to the clinic complaining of several days of worsening
burning and pain with urination. She describes an increase in urinary frequency and
suprapubic tenderness but no fever or back pain. She has no past medical history with the
exception of two prior episodes similar to this in the past 2 years. Urine analysis shows
moderate white blood cells. Which of the following is the most likely causative agent of her
current symptoms?
A Candida
B Escherichia coli
C Enterobacter
D Klebsiella
E Proteus
ANSWER:B
1295
A 42-year-old man with poorly controlled diabetes (HbA1C = 13.3%) presents with thigh
pain and fever over several weeks. Physical examination reveals erythema and warmth over
the thigh with notable woody, nonpitting edema. There are no cutaneous ulcers. CT of the
thigh reveals several abscesses located between the muscle fibers of the thigh. Orthopedics is
consulted to drain and culture the abscesses. Which of the following is the most likely
pathogen?
A Clostridium perfringens
B Group A Streptococcus
C Polymicrobial flora
D Staphylococcus aureus
E Streptococcus milleri
ANSWER:D
1296 Regarding the epidemiology of influenza viruses, which of the following is true?
A Antigenic drift requires a change in both hemagglutinin (H) and neuraminidase (N) antigens.
B
Antigenic shift is defined by an exchange of hemagglutinin (H) and neuraminidase (N)
antigens between influenza A and influenza B viruses.
C
Avian influenza outbreaks in humans occur when human influenza A viruses undergo
antigenic shifts with influenza A from poultry.

D
Influenza C virus infections, while uncommon, are more virulent on a population basis due
to its increased ability to undergo antigenic shift.
E
The lethality associated with avian influenza is related to its ability to spread via person-to-
person contact.
ANSWER:C
1297
A 62-year-old man returns from a vacation to Arizona with fever, pleurisy, and a
nonproductive cough. All of the following factors on history and laboratory examination
favor a diagnosis of pulmonary coccidioidomycosis rather than community-acquired
pneumonia except
A eosinophilia
B erythema nodosum
C mediastinal lymphadenopathy on chest roentgenogram
D positive Coccidioides complement fixation titer
E travel limited to Northern Arizona (Grand Canyon area)
ANSWER:E
1298
A 36-year-old man with a history of hypertension presents complaining of a 3-year history
of constant fatigue, diffuse myalgias, and memory deficits. He also notes trouble with routine
tasks at work. He was diagnosed with Lyme disease 4 years ago and was briefly admitted to a
cardiac care unit for transient third-degree heart block. Symptoms at that time included
fever, malaise, arthralgias, diffuse erythema migrans, and facial nerve palsy. He received
ceftriaxone, 2 g/d for 28 days, and had complete resolution of symptoms for several months
but then developed his new constellation of problems that have gradually worsened over
time. Physical examination is totally within normal limits. Which is the appropriate next step
in management?
A Borrelia burgdorferi enzyme-linked immunosorbent assay
B Ceftriaxone, 2 g daily × 1 month
C Doxycycline, 100 mg PO daily for life
D Prednisone, 60 mg PO daily
E Symptomatic treatment
ANSWER:E

1299
A sputum culture from a patient with cystic fibrosis showing which of the following
organisms has been associated with a rapid decline in pulmonary function and a poor clinical
prognosis?
A Burkholderia cepacia
B Pseudomonas aeruginosa
C Staphylococcus aureus
D Staphylococcus epidermidis
E Stenotrophomonas maltophilia
ANSWER:A
1300
Empirical antibiotic therapy for continuous ambulatory peritoneal dialysis (CAPD) patients
with peritonitis should be directed towards which organisms?
A Enteric gram-negative rods
B Enteric gram-negative rods and yeast
C Gram-positive cocci
D Gram-positive cocci plus enteric gram-negative rods
E Gram-positive cocci plus enteric gram-negative rods plus yeast
ANSWER:D
1301
Indinavir is a protease inhibitor that carries which side effect unique for HIV antiretroviral
agents?
A Abnormal dreams
B Benign hyperbilirubinemia
C Hepatic necrosis in pregnant women
D Nephrolithiasis
E Pancreatitis
ANSWER:D
1302
A 28-year-old woman presents with fevers, headache, diaphoresis, and abdominal pain 2
days after returning from an aid mission to the coast of Papua New Guinea. Several of her
fellow aid workers developed malaria while abroad, and she stopped her doxycycline
prophylaxis due to a photosensitivity reaction 5 days prior. You send blood cultures, routine
labs, and a thick and thin smear to evaluate the source of her fevers. Which of the following
statements is accurate in reference to diagnosis of malaria?

A
A thick smear is performed to increase sensitivity in comparison to a thin smear but can only
be performed in centers with experienced laboratory personnel and has a longer processing
time.
B
Careful analysis of the thin blood film allows for prognostication based on estimation of
parasitemia and morphology of the erythrocytes.
C
In the absence of rapid diagnostic information, empirical treatment for malaria should be
strongly considered.
D
Morphology on blood smear is the current criterion used to differentiate the four species of
Plasmodium that infect humans.
E All of the above
ANSWER:E
1303
A 34-year-old injection drug user presents with a 2-day history of slurred speech, blurry
vision that is worse with bilateral gaze deviation, dry mouth, and difficulty swallowing both
liquids and solids. He states that his arms feel weak as well but denies any sensory deficits. He
has had no recent illness but does describe a chronic ulcer on his left lower leg that has felt
slightly warm and tender of late. He frequently injects heroin into the edges of the ulcer. On
review of systems, he reports mild shortness of breath but denies any gastrointestinal
symptoms, urinary retention, or loss of bowel or bladder continence. Physical examination
reveals a frustrated, nontoxic appearing man who is alert and oriented but noticeably
dysarthric. He is afebrile with stable vital signs. Cranial nerve examination reveals bilateral
cranial nerve six deficits and an inability to maintain medial gaze in both eyes. He has mild
bilateral ptosis, and both pupils are reactive but sluggish. His strength is 5/5 in all extremities
except for his shoulder shrug, which is 4/5. Sensory examination and deep tendon reflexes
are within normal limits in all four extremities. His oropharynx is dry. Cardiopulmonary and
abdominal examinations are normal. He has a 4 cm × 5 cm well-granulated lower extremity
ulcer with redness, warmth, and erythema noted on the upper margin of the ulcer. What is
the treatment of choice?
A Glucocorticoids
B Equine antitoxin to Clostridium botulinum neurotoxin
C Intravenous heparin
D Naltrexone
E Plasmapheresis

ANSWER:B
1304
In an HIV-infected patient, Isospora belli infection is different from Cryptosporidium
infection in which of the following ways?
A
Isospora causes a more fulminant diarrheal syndrome leading to rapid dehydration and even
death in the absence of rapid rehydration.
B
Isospora infection may cause biliary tract disease, whereas cryptosporidiosis is strictly limited
to the lumen of the small and large bowel.
C Isospora is more likely to infect immunocompetent hosts than Cryptosporidium.
D
Isospora is less challenging to treat and generally responds well to
trimethoprim/sulfamethoxazole treatment.
E Isospora occasionally causes large outbreaks among the general population.
ANSWER:D
1305
In a patient with known HIV infection, all of the following are an AIDS-defining criterion
except
A active pulmonary tuberculosis
B CD4+ lymphocyte count < 200/µL
C cryptococcal meningitis
D cytomegalovirus (CMV) retinitis
E herpes zoster infection involving more than one dermatome
ANSWER:E
1306
A 27-year-old man presents to your clinic with 2 weeks of sore throat, malaise, myalgias,
night sweats, fevers, and chills. He visited an urgent care center and was told that he likely
had the flu. He was told that he had a “negative test for mono.” The patient is homosexual,
states that he is in a monogamous relationship and has unprotected receptive and insertive
anal and oral intercourse with one partner. He had several partners prior to his current
partner 4 years ago but none recently. He reports a negative HIV-1 test 2 years ago and
recalls being diagnosed with Chlamydia infection 4 years ago. He is otherwise healthy with
no medical problems. You wish to rule out the diagnosis of acute HIV. Which blood test
should you order?
A CD4+ lymphocyte count
B HIV enzyme immunoassay (EIA)/Western blot combination testing
C HIV resistance panel

D HIV RNA by polymerase chain reaction (PCR)
E HIV RNA by ultrasensitive PCR
ANSWER:D
1307
A 20-year-old female is 36 weeks pregnant and presents for her first evaluation. She is
diagnosed with Chlamydia trachomatis infection of the cervix. Upon delivery, what
complication is her infant most at risk for?
A Jaundice
B Hydrocephalus
C Hutchinson triad
D Conjunctivitis
E Sensorineural deafness
ANSWER:D
1308
A 29-year-old man is being initiated on HIV antiretroviral therapy (ART) because of a rising
viral RNA. He has no significant past medical or psychiatric history and has never received
ART. His viral resistance screening shows no likely resistance mutations. Which of the
following is now considered an acceptable first-line regimen of ART for patients being
newly treated for HIV infection who have no viral resistance and no other medical or
psychiatric problems?
A Stavudine (d4T), didanosine (ddI), efavirenz (EFV)
B Tenofovir (TDF), emtricitabine (FTC), efavirenz (EFV)
C Tenofovir (TDF), emtricitabine (FTC), indinavir
D Tenofovir (TDF), lopinavir/ritonavir, atazanavir
E Zidovudine (AZT), lamivudine (3TC), abacavir (ABC)
ANSWER:B
1309
All of the following clinical findings are consistent with the diagnosis of molluscum
contagiosum except
A involvement of the genitals
B involvement of the soles of the feet
C lack of inflammation or necrosis at the site of the rash
D rash associated with an eczematous eruption
E rash spontaneously resolving over 3–4 months
ANSWER:B

1310
A 45-year-old woman with known HIV infection and medical nonadherence to therapy is
admitted to the hospital with 2–3 weeks of increasing dyspnea on exertion and malaise. Chest
radiograph shows bilateral alveolar infiltrates and induced sputum is positive for
Pneumocystis jiroveci. Which of the following clinical conditions is an indication for
administration of adjunct glucocorticoids?
A Acute respiratory distress syndrome
B CD4+ lymphocyte count < 100/µL
C No clinical improvement 5 days into therapy
D Pneumothorax
E Room air PaO2 <70 mmHg
ANSWER:E
1311 Caspofungin is a first-line agent for which of the following conditions?
A Candidemia
B Histoplasmosis
C Invasive aspergillosis
D Mucormycosis
E Paracoccidiomycosis
ANSWER:A
1312
A 19-year-old college student presents to the emergency room with crampy abdominal pain
and watery diarrhea that has worsened over 3 days. He recently returned from a volunteer
trip to Mexico. He has no past medical history and felt well throughout the trip. Stool
examination shows small cysts containing four nuclei, and stool antigen immunoassay is
positive for Giardia. Which of the following is an effective treatment regimen?
A Albendazole
B Clindamycin
C Giardiasis is self-limited and requires no antibiotic therapy
D Metronidazole
E Paromomycin
ANSWER:D

1313
A 76-year-old woman is brought in to clinic by her son. She complains of a chronic
nonproductive cough and fatigue. Her son adds that she has had low-grade fevers,
progressive weight loss over months, and “just doesn’t seem like herself.” A chest CT reveals
bronchiectasis and small (<5 mm) nodules scattered throughout the lung parenchyma. She
had a distant history of treated tuberculosis. A sputum sample is obtained, as are blood
cultures. Two weeks later, both culture sets grow acid fast bacilli consistent with
Mycobacterium avium complex. Which of the following is the best treatment option?
A Bronchodilators and pulmonary toilet
B Clarithromycin and ethambutol
C Clarithromycin and rifampin
D Moxifloxacin and rifampin
E Pyrazinamide, isoniazid, rifampin, and ethambutol
ANSWER:B
1314
What is the most common side effect of oral ribavirin when used with pegylated interferon
for the treatment of hepatitis C?
A Drug-associated lupus
B Hemolytic anemia
C Hyperthyroidism
D Leukopenia
E Rash
ANSWER:B
1315
A previously unvaccinated health care worker incurs a needle stick from a patient with
known active hepatitis B infection. What is the appropriate management for the health care
worker?
A Hepatitis B immunoglobulins
B Hepatitis B vaccine
C Hepatitis B vaccine plus hepatitis B immunoglobulins
D Hepatitis B vaccine plus lamivudine
E Lamivudine plus tenofovir
ANSWER:C

1316 Which of the following is not a common feature of severe Plasmodium falciparum malaria?
A Acute tubular necrosis
B Hematocrit <15%
C Hepatic necrosis
D Hypoglycemia
E Obtundation
ANSWER:C
1317
A 55-year-old male is admitted to the hospital with aspiration pneumonia. Over the past 8
months he has had a relentless neurologic decline characterized by dementia with severe
memory loss and decline in intellectual function. These symptoms were preceded by 2–3
months of labile mood, weight loss, and headache. Currently he is awake but unable to
answer questions. Neurologic examination is notable for normal cranial nerves and sensation.
He has marked myoclonus provoked by startle or bright lights, but it also occurs
spontaneously during sleep. Prior evaluation revealed normal serum chemistries, negative
serologic tests for syphilis, and normal cerebrospinal fluid (CSF) studies. Head CT scan is
normal. The infectious agent that caused his neurologic syndrome is most likely a
A DNA virus
B fungus
C protein-lacking nucleic acid
D protozoan
E RNA virus
ANSWER:C
1318
A previously healthy 19-year-old man presents with several days of headache, cough with
scant sputum, and fever of 38.6°C. On examination, pharyngeal erythema is noted and lung
fields are clear. Chest radiograph reveals focal bronchopneumonia in the lower lobes. His
hematocrit is 24.7%, down from a baseline measure of 46%. The only other laboratory
abnormality is an indirect bilirubin of 3.4. A peripheral smear reveals no abnormalities. A
cold agglutinin titer is measured at 1:64. What is the most likely infectious agent?
A Coxiella burnetii

B Legionella pneumophila
C Methicillin-resistant Staphylococcus aureus
D Mycoplasma pneumoniae
E Streptococcus pneumoniae
ANSWER:D
1319
A 79-year-old Filipino-American man with diabetes mellitus, coronary artery disease, and
emphysema develops the acute onset of low back pain and night sweats. Ten days prior, he
underwent a prolonged lithotripsy procedure for septic ureteral stones. He was treated for a
positive PPD 23 years ago. He moved to the United States 20 years ago and was a rice farmer
in the Philippines prior to moving. Examination reveals tenderness over the lumbar spine. He
has 5/5 strength in his lower extremities. MRI shows findings consistent with osteomyelitis of
L3 and L4, with narrowing of the disc space and a small contiguous epidural abscess that is
not compressing his spinal cord. A needle culture of the epidural abscess drawn prior to
administration of antibiotics will most likely reveal which of the following?
A Brucella melitensis
B Escherichia coli
C Mycobacterium tuberculosis
D Staphylococcus aureus
E
Polymicrobial content with gram-positive cocci in chains, enteric gram-negative rods, and
anaerobic pleomorphic forms
ANSWER:B
1320
A 64-year-old man in Wisconsin develops a high fever and malaise over 2 days. He has
spent his weekends over the past month chopping wood in his backyard. Initial laboratory
examination reveals a neutrophil count of 1000/µL, platelet count of 84,000/µL, AST of 140
U/L, and ALT of 183 U/L. A peripheral blood smear reveals prominent morulae in
neutrophils. What is the most likely diagnosis?
A Human granulocytotropic anaplasmosis
B Human monocytotropic ehrlichiosis
C Lyme disease
D Rocky Mountain spotted fever
E Systemic lupus erythematosus

ANSWER:A
1321
A 26-year-old asthmatic continues to have coughing fits and dyspnea despite numerous
steroid tapers and frequent use of albuterol over the past few months. Persistent infiltrates are
seen on chest roentgenogram. A pulmonary consultation suggests an evaluation for allergic
bronchopulmonary aspergillosis. What is the diagnostic test of choice?
A Bronchoalveolar lavage (BAL) with fungal culture
B Galactomannan enzyme immunoassay (EIA)
C High-resolution CT
D Pulmonary function tests
E Serum IgE level
ANSWER:E
1322
A patient who has undergone prosthetic valve surgery 6 weeks ago is readmitted with signs
and symptoms consistent with infective endocarditis. Which of the following is the most
likely etiologic organism?
A Candida albicans
B Coagulase-negative staphylococci
C Enterococcus
D Escherichia coli
E Pseudomonas aeruginosa
ANSWER:B
1323
A 28-year-old man is diagnosed with HIV infection during a clinic visit. He has no
symptoms of opportunistic infection. His CD4+ lymphocyte count is 150/ µL. All of the
following are approved regimens for primary prophylaxis against Pneumocystis jiroveci
infection except
A aerosolized pentamidine, 300 mg monthly
B atovaquone, 1500 mg PO daily
C clindamycin, 900 mg PO q8h, plus primaquine, 30 mg PO daily
D dapsone, 100 mg PO daily
E trimethoprim/sulfamethoxazole, 1 single-strength tablet PO daily
ANSWER:C
1324
During the late 1990s, there was a resurgence of all of the following bacterial sexually
transmitted infections (STIs) among homosexual men except

A chlamydia
B gonorrhea
C lymphogranuloma venereum
D syphilis
E all of the above had a resurgence
ANSWER:E
1325
A 47-year-old woman with known HIV/AIDS (CD4+ lymphocyte = 106/µL and viral load =
35,000/ mL) presents with painful growths on the side of her tongue (Figure IV-158, Color
Atlas). What is the most likely diagnosis?
A Aphthous ulcers
B Hairy leukoplakia
C Herpes stomatitis
D Oral candidiasis
E Oral Kaposi’s sarcoma
ANSWER:B
1326
A 45-year-old patient with HIV/AIDS presents to the emergency department. He complains
of a rash that has been slowly spreading up his right arm and is now evident on his chest and
back. The rash consists of small nodules that have a reddish-blue appearance. Some of them
are ulcerated, but there is minimal fluctuance or drainage. He is unsure when these began. He
notes no foreign travel or unusual exposures. He is homeless and unemployed, but
occasionally gets work as a day laborer doing landscaping and digging. A culture of a skin
lesion grows a Mycobacterium in 5 days. Which of the following is the most likely organism?
A M. abscessus
B M. avium
C M. kansasii
D M. marinum
E M. ulcerans
ANSWER:A

1327
A 25-year-old male is seen in the emergency department for symptoms of fevers and
abdominal swelling, early satiety, and weight loss. His symptoms began abruptly 2 weeks
ago. He was previously healthy and is taking no medications. He denies illicit drug use and
recently immigrated to the United States from Bangladesh. On physical examination,
temperature is 39.0°C (102.2°F) and pulse is 120, with normal blood pressure and
respiratory rate. The remainder of the exam is notable for cachexia and a distended abdomen
with a massively enlarged spleen. The spleen is tender and soft. The liver is not palpable.
Mild peripheral adenopathy is present. Which of the following statements is correct
regarding this patient with presumed kala azar leishmaniasis?
A He probably has normal cell counts on peripheral blood smear.
B Leishmania donovani is not endemic in Bangladesh.
C Leishmania-specific cell-mediated immunity probably is present.
D Splenic aspiration offers the highest diagnostic yield.
E Treatment can be delayed until the diagnosis is confirmed.
ANSWER:D
1328
A 41-year-old man with hepatitis C–associated ascites presents with acute abdominal pain.
Physical examination is notable for temperature of 38.3°C, heart rate of 115 beats per
minute, blood pressure of 88/48 mmHg, respiratory rate of 16 breaths per minute, and
oxygen saturation of 99% on room air. The patient is in moderate discomfort and is lying
still. He is alert and oriented. Lungs are clear. Cardiac examination is unremarkable. His
abdomen is diffusely tender with distant bowel sounds, mild guarding, and no rebound
tenderness. Laboratory studies reveal a leukocyte count of 11,630/µL with 94% neutrophils,
hematocrit of 29%, and platelet count of 24,000/µL. Paracentesis reveals 658 PMNs/µL, total
protein 1.2 g/dL, glucose 24 mg/dL, and gram stain showing gram-negative rods, gram-
positive cocci in chains, gram-positive rods, and yeast forms. All of the following are
indicated except
A abdominal radiograph
B broad-spectrum antibiotics
C drotrecogin alfa
D intravenous fluid
E surgical consultation
ANSWER:C

1329
Patients with which of the following have the lowest risk of invasive pulmonary Aspergillus
infection?
A Allogeneic stem cell transplant with graft-vs-host disease
B HIV infection
C Long-standing high-dose glucocorticoids
D Post-solid organ transplant with multiple episodes of rejection
E Relapsed/uncontrolled leukemia
ANSWER:B
1330
All the following patient characteristics are included in the calculation of the Pneumonia
Patient Outcomes Research Team (PORT) score that is used in the evaluation of patients with
community-acquired pneumonia except
A age
B coexisting illness
C laboratory findings
D radiographic findings
E smoking history
ANSWER:E
1331 Rifampin lowers serum levels of all of the following medicines except
A amiodarone
B anticonvulsants
C oyclosporine
D hormonal contraceptives
E protease inhibitors
ANSWER:A
1332
Which single clinical feature has the most specificity in differentiating Pseudomonas
aeruginosa sepsis from other causes of severe sepsis in a hospitalized patient?
A Ecthyma gangrenosum
B Hospitalization for severe burn
C Profound bandemia
D Recent antibiotic exposure
E Recent mechanical ventilation for >14 days
ANSWER:A

1333
Which of the following statements regarding varicella-zoster infection after hematopoietic
stem cell transplant is true?
A
Acyclovir prophylaxis is not warranted for patients with positive varicella-zoster virus
serologies pretransplant as the rate of zoster reactivation is low following transplantation.
B
Herpes zoster resistance is a common problem, and a change from acyclovir to foscarnet is
often required.
C
Multidermatomal and disseminated zoster can occur in transplant patients who do not receive
appropriate antiviral therapy.
D
Zoster occurs more commonly following autologous transplant of stem cells than allogeneic
transplant of stem cells.
E Zoster occurs most frequently during the first month after transplant.
ANSWER:C
1334 All of the following factors influence the likelihood of transmitting active tuberculosis except
A duration of contact with an infected person
B environment in which contact occurs
C presence of extrapulmonary tuberculosis
D presence of laryngeal tuberculosis
E probability of contact with an infectious person
ANSWER:C
1335
Which of the following individuals with a known history of prior latent tuberculosis
infection (without therapy) has the greatest likelihood of developing reactivation
tuberculosis?
A
A 28-year-old woman with anorexia nervosa, a body mass index of 16 kg/m2, and a serum
albumin of 2.3 g/dL
B A 36-year-old intravenous drug user who does not have HIV but is homeless
C
A 42-year-old man who is HIV-positive with a CD4 count of 350/µL on highly active
antiretroviral therapy
D A 68-year-old man who worked as a stone mason for many years and has silicosis
E A 73-year-old man who was infected while stationed in Korea in 1958
ANSWER:C

1336
A 50-year-old man is admitted to the hospital for active pulmonary tuberculosis with a
positive sputum acid-fast bacilli smear. He is HIV positive with a CD4 count of 85/µL and is
not on highly active antiretroviral therapy. In addition to pulmonary disease, he is found to
have disease in the L4 vertebral body. What is the most appropriate initial therapy?
A Isoniazid, rifampin, ethambutol, and pyrazinamide
B Isoniazid, rifampin, ethambutol, and pyrazinamide; initiate antiretroviral therapy
C Isoniazid, rifampin, ethambutol, pyrazinamide, and streptomycin
D Isoniazid, rifampin, and ethambutol
E Withhold therapy until sensitivities are available.
ANSWER:A
1337
All of the following individuals receiving tuberculin skin purified protein derivative (PPD)
reactions should be treated for latent tuberculosis except
A A 23-year-old injection drug user who is HIV negative has a 12-mm PPD reaction.
B
A 38-year-old fourth grade teacher has a 7-mm PPD reaction and no known exposures to
active tuberculosis. She has never been tested with a PPD previously.
C
A 43-year-old individual in the Peace Corps working in Sub-Saharan Africa has a 10-mm
PPD reaction. 18 months ago, the PPD reaction was 3 mm.
D
A 55-year-old man who is HIV positive has a negative PPD. His partner was recently
diagnosed with cavitary tuberculosis.
E
A 72-year-old man who is receiving chemotherapy for non-Hodgkin’s lymphoma has a 16-
mm PPD reaction.
ANSWER:B
1338
A 68-year-old woman seeks evaluation for an ulcerative lesion on her right hand. She
reports the area on the back of her right hand was initially red and not painful. There
appeared to be a puncture wound in the center of the area, and she thought she had a simple
scratch acquired while gardening. Over the next several days, the lesion became verrucous
and ulcerated. Now, the patient has noticed several nodular areas along the arm, one of
which ulcerated and began draining a serous fluid today. She is also noted to have an
enlarged and tender epitrochlear lymph node on the right arm. A biopsy of the edge of the
lesion shows ovoid and cigar-shaped yeasts. Sporotrichosis is diagnosed. What is the most
appropriate therapy for this patient?

A Amphotericin B intravenously
B Caspofungin intravenously
C Clotrimazole topically
D Itraconazole orally
E Selenium sulfide topically
ANSWER:D
1339
A 44-year-old man presents to the emergency room for evaluation of a severe sore throat.
His symptoms began this morning with mild irritation on swallowing and have gotten
progressively severe over the course of 12 h. He has been experiencing a fever to as high as
39°C at home and also reports progressive shortness of breath. He denies antecedent
rhinorrhea or tooth or jaw pain. He has had no ill contacts. On physical examination, the
patient appears flushed and in respiratory distress with use of accessory muscles of
respiration. Inspiratory stridor is present. He is sitting leaning forward and is drooling with
his neck extended. His vital signs are as follows: temperature 39.5°C, blood pressure 116/60
mmHg, heart rate 118 beats/min, respiratory rate 24 breaths/ min, SaO2 95% on room air.
Examination of his oropharynx shows erythema of the posterior oropharynx without
exudates or tonsillar enlargement. The uvula is midline. There is no sinus tenderness and no
cervical lymphadenopathy. His lung fields are clear to auscultation, and cardiovascular
examination reveals a regular tachycardia with a II/VI systolic ejection murmur heard at the
upper right sternal border. Abdominal, extremity, and neurologic examinations are normal.
Laboratory studies reveal a white blood cell count of 17,000 µL with a differential of 87%
neutrophil, 8% band forms, 4% lymphocytes, and 1% monocytes. Hemoglobin is 13.4 g/dL
with a hematocrit of 44.2%. An arterial blood gas on room air has a pH of 7.32, a PaCO2 of
48 mmHg, and PaO2 of 92 mmHg. A lateral neck film shows an edematous epiglottis. What
is the next most appropriate step in evaluation and treatment of this individual?
A Ampicillin, 500 mg IV q6h
B Ceftriaxone, 1 g IV q24h
C Endotracheal intubation and ampicillin, 500 mg IV q6h
D Endotracheal intubation, ceftriaxone, 1 g IV q24h, and clindamycin, 600 mg IV q6h
E Laryngoscopy and close observation
ANSWER:D

1340
A 45-year-old man from western Kentucky presents to the emergency room in September
complaining of fevers, headaches, and muscle pains. He recently had been on a camping trip
with several friends during which they hunted for their food, including fish, squirrels, and
rabbits. He did not recall any tick bites during the trip, but does recall having several
mosquito bites. For the past week, he has had an ulceration on his right hand with redness
and pain surrounding it. He also has noticed some pain and swelling near his right elbow.
None of the friends he camped with have been similarly ill. His vital signs are: blood
pressure 106/65 mmHg, heart rate 116 beats/min, respiratory rate 24 breaths/min, and
temperature 38.7°C. His oxygen saturation is 93% on room air. He appears mildly
tachypneic and flushed. His conjunctiva are not injected and his mucous membranes are dry.
The chest examination reveals crackles in the right mid-lung field and left base. His heart rate
is tachycardic but regular. There is a II/VI systolic ejection murmur heard best at the lower
left sternal border. His abdominal examination is unremarkable. On the right hand, there is
an erythematous ulcer with a punched-out center covered by a black eschar. He has no
cervical lymphadenopathy, but there are markedly enlarged and tender lymph nodes in the
right axillae and epitrochlear regions. The epitrochlear node has some fluctuance with
palpation. A chest x-ray shows fluffy bilateral alveolar infiltrates. Over the first 12 h of his
hospitalization, the patient becomes progressively hypotensive and hypoxic, requiring
intubation and mechanical ventilation. What is the most appropriate therapy for this patient?
A Ampicillin, 2 g IV q6h
B Ceftriaxone, 1 g IV daily
C Ciprofloxacin, 400 mg IV twice daily
D Doxycycline, 100 mg IV twice daily
E Gentamicin, 5 mg/kg twice daily
ANSWER:E

1341
A 24-year-old man seeks evaluation for painless penile ulcerations. He noted the first lesion
about 2 weeks ago, and since that time, two adjacent areas have also developed ulceration.
He states that there has been blood staining his underwear from slight oozing of the ulcers.
He has no past medical history and takes no medication. He returned 5 weeks ago from a
vacation in Brazil where he did have unprotected sexual intercourse with a local woman. He
denies other high-risk sexual behaviors and has never had sex with prostitutes. He was last
tested for HIV 2 years ago. He has never had a chlamydial or gonococcal infection. On
examination, there are three welldefined red, friable lesions measuring 5 mm or less on the
penile shaft. They bleed easily with any manipulation. There is no pain with palpation. There
is shotty inguinal lymphadenopathy. On biopsy of one lesion, there is a prominent
intracytoplasmic inclusion of bipolar organisms in an enlarged mononuclear cell.
Additionally, there is epithelial cell proliferation with an increased number of plasma cells
and few neutrophils. A rapid plasma reagin test is negative. Cultures grow no organisms.
What is the most likely causative organism?
A Calymmatobacterium granulomatis (donovanosis)
B Chlamydia trachomatis (lymphogranuloma venereum)
C Haemophilus ducreyi (chancroid)
D Leishmania amazonensis (cutaneous leishmaniasis)
E Treponema pallidum (secondary syphilis)
ANSWER:A
1342
A 75-year-old patient presents with fevers and wasting. He describes fatigue and malaise
over the past several months and is concerned that he has been losing weight. On
examination, he is noted to have a low-grade fever and a soft diastolic heart murmur is
appreciated. Laboratory tests reveal a normocytic, normochromic anemia. Three separate
blood cultures grow Cardiobacterium hominis. Which of the following statements is true
about this patient’s clinical condition?
A Antibiotics are not likely to improve his condition.
B Echocardiogram will likely be normal.
C He has a form of endocarditis with a high risk of emboli.
D He will likely need surgery.
E The positive blood cultures are likely a skin contaminant.
ANSWER:C

1343
A 38-year-old woman with frequent hospital admissions related to alcoholism comes to the
emergency room after being bitten by a dog. There are open wounds on her arms and right
hand that are purulent and have necrotic borders. She is hypotensive and is admitted to the
intensive care unit. She is found to have disseminated intravascular coagulation and soon
develops multiorgan failure. Which of the following is the most likely organism to have
caused her rapid decline?
A Aeromonas spp.
B Capnocytophaga spp.
C Eikenella spp.
D Haemophilus spp.
E Staphylococcus spp.
ANSWER:B
1344
A 39-year-old healthy man plans to travel to Malaysia and comes to clinic for appropriate
vaccinations. He cannot recall which vaccines he has had in the past, but reports having had
“all the usual ones” in childhood. Which of the following represents the most common
vaccine-preventable infection in travelers?
A Influenza
B Measles
C Rabies
D Tetanus
E Yellow fever
ANSWER:A
1345
A 19-year-old man plans on traveling through Central America by bus. He comes to clinic
interested in travel advice and any vaccinations he may need. He has no medical history and
takes no medicines. In addition to DEET and mosquito netting, which of the following
recommendations would be important for prophylaxis against malaria?
A Atovaquone
B Chloroquine
C Doxycycline
D Mefloquine
E Primaquine
ANSWER:B

1346
Which of the following is the most common source of fever in travelers returning from
Southeast Asia?
A Dengue fever
B Malaria
C Mononucleosis
D Salmonella
E Yellow fever
ANSWER:A
1347
A 54-year-old woman presents to the emergency room complaining of pain and redness of
her left face and cheek. The area of redness began abruptly yesterday. At that time, the area
was about 5 mm2 near the nasolabial fold. There was rapid progression of the redness to an
area that is now about 5 cm2. In addition, she is complaining of intense pain in this area. On
examination, there is a well-demarcated 5 cm2 area of erythema along her left nasolabial
fold. The borders are raised and indurated. The entire area is very tender to touch. Over the
next 24 h, the affected area begins to develop a flaccid bullae. What is the most appropriate
treatment for this patient?
A Acyclovir
B Clindamycin
C Clindamycin and penicillin
D Penicillin
E Trimethoprim and sulfamethoxazole
ANSWER:D

1348
A 68-year-old man is brought to the emergency room with altered mental status, fever, and
leg pain. His wife reports that he first complained of pain in his leg yesterday, and there was
some slight redness in this area. Over the night, he developed a fever to as high as 39.8°C
and became obtunded this morning. At that point, his family brought him to the emergency
room. Upon arrival, he is unresponsive to voice and withdraws to pain. The vital signs are:
blood pressure 88/40 mmHg, heart rate 126 beats/min, respiratory rate 28 breaths/min,
temperature 39.3°C, and SaO2 95% on room air. Examination of the right leg shows diffuse
swelling with brawny edema. The patient grimaces in pain when the area is touched. There
are several bullae filled with dark blue to purple fluid. Laboratory studies show: pH 7.22,
PaCO2 28 mmHg, PaO2 93 mmHg. The creatinine is 3.2 mg/dL. White blood cell count is
elevated at 22,660/µL with a differential of 70% polymorphonuclear cells, 28% band forms,
and 2% lymphocytes. A bulla is aspirated and the Gram stain shows gram-positive cocci in
chains. What is the most appropriate therapy for this patient?
A Ampicillin, clindamycin, and gentamicin
B Clindamycin and penicillin
C Clindamycin, penicillin, and surgical debridement
D Penicillin and surgical debridement
E Vancomycin, penicillin, and surgical debridement
ANSWER:C
1349
In the urgent care clinic, you are evaluating a 47year-old woman with poorly controlled
diabetes who has a chief complaint of “sinusitis.” She does not have a history of atopy. She
first noticed a headache 2 days ago and now feels very congested in her upper nasal
passages. She has hyperesthesia over her nasal bridge as well and is inquiring about
antibiotics to treat her infection. She has a bloody nasal discharge with occasional black
specks. On examination, the sinuses are full and tender. She has a temperature of 38.3°C.
Oral examination shows a black eschar on the roof of her mouth surrounded by discolored
hyperemic areas on the palate. What is the most appropriate intervention at this time?
A Ciprofloxacin and quarantine for possible anthrax
B ENT consultation if no improvement with oral antibiotics
C Immediate biopsy of the involved areas and lipid amphotericin

D Immediate biopsy of the lesion and voriconazole
E Intranasal decongestants and close follow-up
ANSWER:C
1350
A 63-year-old man from Mississippi comes to your office for evaluation of a chronic sore on
his thigh. He has an open sore on his anterior thigh that has been draining purulent material
for many months. The thigh is nontender but is warm to touch. The material is purulent and
foul-smelling. He has been given multiple antibiotic courses and recently finished a course of
itraconazole without relief of his symptoms. He has an intact neurovascular examination of
his lower extremities. His erythrocyte sedimentation rate is 64, white blood cell count is
15,000/µL and hemoglobin is 8 mg/dL. A plain radiograph of the affected thigh shows a
periosteal reaction of the femur with osteopenia. There is suggestion of a sinus tract between
the femur and the skin. A Gram stain of the pus shows broad-based budding yeast and you
make a presumptive diagnosis of blastomyces osteomyelitis. What is the treatment of choice
for this patient?
A Amphotericin B
B Caspofungin
C Itraconazole
D Moxifloxacin
E Voriconazole
ANSWER:A
1351
A 73-year-old male presents to the clinic with 3 months of increasing back pain. He localizes
the pain to the lumbar spine and states that the pain is worst at night while he is lying in bed.
It is improved during the day with mobilization. Past history is notable only for hypertension
and remote cigarette smoking. Physical examination is normal. Laboratory studies are
notable for an elevated alkaline phosphatase. A lumbar radiogram shows a lytic lesion in the
L3 vertebra. Which of the following malignancies is most likely?
A Gastric carcinoma
B Non-small cell lung cancer
C Osteosarcoma
D Pancreatic carcinoma
E Thyroid carcinoma

ANSWER:B
1352
Patients from which of the following regions need not be screened for glucose-6-phosphate
dehydrogenase (G6PD) deficiency when starting a drug that carries a risk for G6PD
mediated hemolysis?
A Brazil
B Russia
C Southeast Asia
D Southern Europe
E Sub-Saharan Africa
ANSWER:B
1353 All the following are vitamin K–dependent coagulation factors except
A factor X
B factor VII
C protein C
D protein S
E factor VIII
ANSWER:E
1354
A 31-year-old male with hemophilia A is admitted with persistent gross hematuria. He denies
recent trauma or any history of genitourinary pathology. The examination is unremarkable.
Hematocrit is 28%. All the following are treatments for hemophilia A except
A desmopressin (DDAVP)
B fresh-frozen plasma (FFP)
C cryoprecipitate
D recombinant factor VIII
E plasmapheresis
ANSWER:E
1355
Which of the following statements regarding incidence of and risk factors for hepatocellular
carcinoma is true?
A
A chemical toxin produced by Aspergillus species, aflatoxin B has a strong association with
development of hepatocellular carcinoma and can be found in stored grains in hot, humid
places.

B In the United States, the incidence of hepatocellular carcinoma is decreasing.
C
Nonalcoholic steatohepatitis is not associated with an increased risk for hepatocellular
carcinogen.
D
Fewer than 5% of individuals diagnosed with hepatocellular carcinoma in the United States
do not have underlying cirrhosis.
E
The risk of developing hepatocellular carcinoma in individuals with hepatitis C infection is
50%.
ANSWER:A
1356
You are asked to review the peripheral blood smear from a patient with anemia. Serum
lactate dehydrogenase is elevated and there is hemoglobinuria. This patient is likely to have
which physical examination finding? (See Figure I6, Color Atlas.)
A Goiter
B Heme-positive stools
C Mechanical second heart sound
D Splenomegaly
E Thickened calvarium
ANSWER:C
1357
All of the enzyme deficiencies that lead to porphyrias are inherited either as autosomal
dominant (AD) or autosomal recessive (AR) traits with one exception. Which of the
following most commonly occurs sporadically?
A 5-ALA dehydratase-deficient porphyria
B Acute intermittent porphyria
C Erythropoietic porphyria
D Porphyria cutanea tarda
E Variegate porphyria
ANSWER:D
1358
A 55-year-old female presents with progressive incoordination. Physical examination is
remarkable for nystagmus, mild dysarthria, and past-pointing on finger-to-nose testing. She
also has an unsteady gait. MRI reveals atrophy of both lobes of the cerebellum. Serologic
evaluation reveals the presence of anti-Yo antibody. Which of the following is the most
likely cause of this clinical syndrome?
A Non-small cell cancer of the lung

B Small-cell cancer of the lung
C Breast cancer
D Non-Hodgkin’s lymphoma
E Colon cancer
ANSWER:C
1359
A 36-year-old African-American woman with systemic lupus erythematosus presents with
the acute onset of lethargy and jaundice. On initial evaluation, she is tachycardic,
hypotensive, appears pale, is dyspneic, and is somewhat difficult to arouse. Physical
examination reveals splenomegaly. Her initial hemoglobin is 6 g/dL, white blood cell count
is 6300/µL, and platelets are 294,000/µL. Her total bilirubin is 4 g/dL, reticulocyte count is
18%, and haptoglobin is not detectable. Renal function is normal, as is urinalysis. What
would you expect on her peripheral blood smear?
A Macrocytosis and PMN’s with hypersegmented nuclei
B Microspherocytes
C Schistocytes
D Sickle cells
E Target cells
ANSWER:B
1360
You are investigating the cause for a patient’s anemia. He is a 50-year-old man who was
found to have a hematocrit of 25% on routine evaluation. His hematocrit was 47% 1 year
ago. Mean corpuscular volume is 80, mean corpuscular hemoglobin concentration is 25,
mean corpuscular hemoglobin is 25. Reticulocyte count is 5%. Review of the peripheral
blood smear shows marked numbers of polychromatophilic macrocytes. Ferritin is 340 µg/L.
What is the cause of this patient’s anemia?
A Defective erythroid marrow proliferation
B Extravascular hemolysis
C Intravascular hemolysis
D Iron-deficiency anemia
E Occult gastrointestinal bleeding
ANSWER:A
1361 All the following are associated with pure red cell aplasia except
A anterior mediastinal masses

B connective tissue disorders
C giant pronormoblasts
D low erythropoietin levels
E parvovirus B19 infection
ANSWER:D
1362
A 73-year-old man is admitted to the hospital with 3 weeks of malaise and fevers. His past
medical history is notable only for hypertension controlled with a thiazide diuretic. He
smokes one pack of cigarettes per day and works as an attorney. His physical examination is
notable only for a new systolic heart murmur heard best in the mitral region. His laboratory
examination is notable for mild anemia, an elevated white blood cell count, and occasional
red blood cells on clean catch urine. Blood cultures grow Streptococcus bovis and
echocardiogram shows a <1-cm vegetation on the mitral valve. What additional evaluation is
indicated for this patient?
A Colonoscopy
B Head CT scan
C Pulmonary embolism protocol CT scan
D Renal biopsy
E Toxicology screen
ANSWER:A

1363
A 58-year-old woman presents to the emergency room complaining of jaundice. She first
noticed a yellowish discoloration of her skin about 3 days ago. It has become progressively
worse since that time. In association with the development of jaundice, she also has noticed
clay-colored stools and pruritus. There has been no associated abdominal pain, fever, chills,
or night sweats. She has a past medical history of alcohol abuse, but has been abstinent for
the past 10 years. She has no known history of cirrhosis. On physical examination, she is
afebrile with normal vital signs. She is jaundiced. The bowel sounds are normal. The
abdomen is soft and nontender. There is no distention. The liver span is 12 cm to percussion
and is palpable at the right costal margin. The spleen tip is not palpable. Liver function
testing reveals an AST of 122 IU/L, ALT of 168 IU/L, alkaline phosphatase of 483 U/L, total
bilirubin of 22.1 mg/dL, and direct bilirubin of 19.2 mg/dL. On right upper quadrant
ultrasound, the gallbladder cannot be visualized, and there is dilatation of the intrahepatic
bile ducts but not the common bile duct. What is the most likely diagnosis?
A Cholangiocarcinoma
B Cholecystitis
C Gallbladder cancer
D Hepatocellular carcinoma
E Pancreatic cancer
ANSWER:A
1364
An 81-year-old male is admitted to the hospital for altered mental status. He was found at
home, confused and lethargic, by his son. His past medical history is significant for
metastatic prostate cancer. The patient’s medications include periodic intramuscular goserelin
injections. On examination he is afebrile. Blood pressure is 110/50 mmHg, and the pulse rate
is 110 beats/min. He is lethargic and minimally responsive to sternal rub. He has bitemporal
wasting, and his mucous membranes are dry. On neurologic examination he is obtunded.
The patient has an intact gag reflex and withdraws to pain in all four extremities. Rectal tone
is normal. Laboratory values are significant for a creatinine of 4.2 mg/dL, a calcium level of
12.4 meq/L, and an albumin of 2.6 g/dL. All the following are appropriate initial
management steps except
A normal saline
B pamidronate

C furosemide when the patient is euvolemic
D calcitonin
E dexamethasone
ANSWER:E
1365
Which of the following statements describes the relationship between testicular tumors and
serum markers?
A
Pure seminomas produce α fetoprotein (AFP) or beta human chorionic gonadotropin (β-
hCG) in more than 90% of cases.
B More than 40% of nonseminomatous germ cell tumors produce no cell markers.
C Both β-hCG and AFP should be measured in following the progress of a tumor.
D
Measurement of tumor markers the day after surgery for localized disease is useful in
determining completeness of the resection.
E
β-hCG is limited in its usefulness as a marker because it is identical to human luteinizing
hormone.
ANSWER:C
1366
A woman with advanced breast cancer being treated with tamoxifen presents to the
emergency department with nausea and vomiting. She has been tolerating her treatment well
but in the last 3 days noticed nausea, vomiting, and abdominal pain. Her symptoms are not
related to food intake, and she is having normal bowel movements. She has no fevers or
rashes. Her medications include tamoxifen, alendronate, megestrol acetate, and a
multivitamin. Abdominal examination reveals very mild tenderness diffusely, and there is no
rebound tenderness. Bowel sounds are normal. Plain radiographs and a CT scan of the
abdomen are unremarkable. Laboratory analysis reveals a normal white blood cell count.
Sodium is 130 meq/L, potassium 4.9 meq/L, chloride 99 meq/L, bicarbonate 29 meq/L,
BUN 15 mg/dL, creatinine 0.7 mg/ dL. What is the next most appropriate step in this
patient’s management?
A Antiemetics prn
B Laparoscopy
C Serum cortisol
D Small-bowel follow through
E Upper endoscopy
ANSWER:C

1367
A healthy 62-year-old woman returns to your clinic after undergoing routine colonoscopy.
Findings included two 1.3-cm sessile (flat-based), villous adenomas in her ascending colon
that were removed during the procedure. What is the next step in management?
A Colonoscopy in 3 months
B Colonoscopy in 3 years
C Colonoscopy in 10 years
D CT scan of the abdomen
E Partial colectomy
ANSWER:B
1368 Which of the following statements regarding polycythemia vera is correct?
A An elevated plasma erythropoietin level excludes the diagnosis.
B Transformation to acute leukemia is common.
C Thrombocytosis correlates strongly with thrombotic risk.
D Aspirin should be prescribed to all these patients to reduce thrombotic risk.
E Phlebotomy is used only after hydroxyurea and interferon have been tried.
ANSWER:A
1369
A 52-year-old female is evaluated for abdominal swelling with a computed tomogram that
shows ascites and likely peritoneal studding of tumor but no other abnormality. Paracentesis
shows adenocarcinoma but cannot be further differentiated by the pathologist. A thorough
physical examination, including breast and pelvic examination, shows no abnormality. CA-
125 levels are elevated. Pelvic ultrasound and mammography are normal. Which of the
following statements is true?
A
Compared with other women with known ovarian cancer at a similar stage, this patient can
be expected to have a less than average survival.
B Debulking surgery is indicated.
C Surgical debulking plus cisplatin and paclitaxel is indicated.
D Bilateral mastectomy and bilateral oophorectomy will improve survival.
E Fewer than 1% of patients with this disorder will remain disease-free 2 years after treatment.
ANSWER:C

1370
A 34-year-old female with a past medical history of sickle cell anemia presents with a 5-day
history of fatigue, lethargy, and shortness of breath. She denies chest pain or bone pain. She
has had no recent travel. Of note, the patient’s 4-year-old daughter had a “cold” 2 weeks
before the presentation. On examination she has pale conjunctiva, is anicteric, and is mildly
tachycardic. Abdominal examination is unremarkable. Laboratories show a hemoglobin of 3
g/dL; her baseline is 8 g/dL. The white blood cell count and platelets are normal.
Reticulocyte count is undetectable. Total bilirubin is 1.4 mg/dL. Lactic dehydrogenase is at
the upper limits of the normal range. Peripheral blood smear shows a few sickled cells but a
total absence of reticulocytes. The patient is given a transfusion of 2 units of packed red
blood cells and admitted to the hospital. A bone marrow biopsy shows a normal myeloid
series but an absence of erythroid precursors. Cytogenetics are normal. What is the most
appropriate next management step?
A Make arrangements for exchange transfusion.
B Tissue type her siblings for a possible bone marrow transplant.
C Check parvovirus titers.
D tart prednisone and cyclosporine.
E Start broad-spectrum antibiotics.
ANSWER:C
1371
A 22-year-old pregnant woman of northern European descent presents 3 months into her
first pregnancy with extreme fatigue, pallor, and icterus. She reports being previously
healthy. On evaluation her hemoglobin is 8 g/dL, reticulocyte count is 9%, indirect bilirubin
is 4.9 mg/dL, and serum haptoglobin is not detectable. Her physical examination is notable
for splenomegaly and a normal 3-month uterus. Peripheral smear is shown below. What is
the most likely diagnosis? (See Figure I21, Color Atlas.)
A Colonic polyp
B G6PD deficiency
C Hereditary spherocytosis
D Parvovirus B19 infection
E Thrombotic thrombocytopenic purpura
ANSWER:C

1372
A patient with acute lymphoid leukemia (ALL) is admitted with respiratory distress and chest
pain. The patient reports 1 day of shortness of breath not associated with cough. There have
been no sick contacts, and before the onset of the respiratory symptoms, the patient only
recalls fatigue. A chest radiograph shows faint diffuse interstitial infiltrates without
pulmonary edema. The cardiac silhouette is normal. An arterial blood gas shows a PaO2 = 54
mmHg, while the pulse oximetry is 97% on room air. A carbon monoxide level is normal.
All of the following laboratory abnormalities are expected in this patient except
A bcr-abl mutation
B blast count >100,000/µL
C elevated lactate dehydrogenase levels
D increased blood viscosity
E methemoglobinemia
ANSWER:E
1373
A 48-year-old male is referred for evaluation by an acute care center because of a nodule on
chest radiography. Three weeks ago he was diagnosed with pneumonia after reporting 3
days of fever, cough, and sputum production. The chest radiogram showed a small right
lower lobe alveolar infiltrate and a left upper lobe 1.5-cm round nodule. He was treated with
antibiotics and is now asymptomatic. A repeat chest radiogram shows that the right lower
lobe pneumonia is resolved, but the nodule is still present. He is asymptomatic. He smoked
one pack of cigarettes per day for 25 years and quit 3 years ago. He never had a prior chest
radiogram. CT scan shows that the nodule is 1.5 by 1.7 cm and is located centrally in the left
upper lobe, has no calcification, and has slightly scalloped edges. There is no mediastinal
adenopathy or pleural effusion. Which of the following is the appropriate next step in his
management?
A Bronchoscopy
B Mediastinoscopy
C MRI scan
D 18FDG PET scan
E Repeat chest CT in 6 months
ANSWER:D

1374
All the following types of cancer commonly metastasize to the central nervous system (CNS)
except
A ovarian
B breast
C hypernephroma
D melanoma
E acute lymphoblastic leukemia (ALL)
ANSWER:A
1375
A 54-year-old woman with atrial fibrillation is anticoagulated with warfarin, 5 mg daily. She
developed a urinary tract infection that her primary care physician has treated with
ciprofloxacin, 250 mg orally twice daily for 7 days. She presents to the emergency room
today complaining of blood in her urine and easy bruising. Her physical examination shows
ecchymoses on her arms. Her urine is bloody in appearance, but no clots are present. After
flushing the bladder with 100 mL of sterile saline, the urine returns with a slight pink hue
only. A urinalysis shows 3–5 white blood cells per high power field and many red blood
cells per high power field. There are no bacteria present. The international normalized ratio
(INR) is 7.0. What is the best approach to treatment of this patient’s coagulopathy?
A Administer vitamin K 10 mg IV.
B Administer vitamin K 2 mg SC.
C Administer vitamin K 1 mg sublingually.
D Hold further warfarin doses until the INR falls to 2.0.
E Transfuse four units of fresh-frozen plasma.
ANSWER:C
1376 Which of the following statements about cardiac toxicity from cancer treatment is true?
A Doxorubicin-based cardiac toxicity is idiosyncratic and dose-independent.
B
Anthracycline-induced congestive heart failure is reversible with time and control of risk
factors.
C
Mediastinal irradiation often results in acute pericarditis during the first few weeks of
treatment.
D
Chronic constrictive pericarditis often manifests symptomatically up to 10 years after
treatment.

E
The incidence of coronary atherosclerosis in patients who have a history of mediastinal
irradiation is the same as that in age-matched controls.
ANSWER:D
1377
A 23-year-old woman is diagnosed with a lower extremity deep venous thrombosis. Which
of the following medical conditions represents a contraindication to therapy with low-
molecular-weight heparin (LMWH)?
A Pregnancy
B Obesity
C Dialysis-dependent renal failure
D Uncontrolled diabetes mellitus
E Jaundice
ANSWER:C
1378 Which of the following pairs of chemotherapy and complication is incorrect?
A Daunorubicin—CHF
B Bleomycin—interstitial fibrosis
C Cyclophosphamide—hematuria
D Cisplatin—liver failure
E Ifosfamide—Fanconi syndrome
ANSWER:D

1379
A 70-year-old man is admitted to the cardiac care unit for complaints of chest pressure
occurring at rest radiating to his left arm with associated diaphoresis and presyncope. His
admission electrocardiogram (ECG) showed ST depressions in V4–V6. The chest pain and
ECG changes resolve with sublingual nitroglycerin. He is treated with IV heparin, aspirin,
metoprolol, and lisinopril. His cardiac catheterization shows 90% occlusion of the left
anterior descending artery, 80% occlusion of the distal circumflex artery, and 99% occlusion
of the right coronary artery. He remains in the cardiac care unit awaiting coronary artery
bypass. He has a history of rheumatic heart disease and underwent mechanical mitral valve
replacement at age 58. On admission, his hemoglobin is 12.2 g/dL, hematocrit 37.1%, white
blood cell (WBC) count 9800/µL, and platelet count 240,000/µL. His creatinine is 1.7
mg/dL. On the fourth hospital day, his hemoglobin is 10.0, hematocrit 31%, WBC count
7600/µL, and platelet count 112,000/µL. His creatinine has risen to 2.9 mg/dL after the
cardiac catheterization. What is the most appropriate treatment of the patient at this time?
A Continue heparin and give a platelet transfusion.
B Discontinue heparin infusion and start argatroban.
C Discontinue heparin and start lepirudin.
D Discontinue heparin and start warfarin.
E
Send serum to assess for the presence of heparin– platelet factor 4 (PF4) IgG antibody and
continue heparin.
ANSWER:B

1380
A 24-year-old woman presents to the emergency room complaining of a red, tender rash that
has been spreading across her arms and legs over the past 2 days. She also describes severe
diffuse muscle pain that has worsened over a week’s time. She woke up feeling as though
she could not catch her breath and has developed a dry cough over the past several days. She
is without any significant medical history but recalls that she had similar symptoms several
years ago, and was told she was having an allergic reaction. Her symptoms abated with an
oral glucocorticoid taper. She takes no prescription medications but takes a number of over-
the-counter nutritional supplements daily. She cannot describe any allergic trigger to her
previous episode or her current one. Her family history is unremarkable, and her close
contacts are not ill. She works in an office, has no pets, and has not travelled internationally.
Her laboratory results are remarkable for a leukocyte count of 12,100 cells/ µL and a total
eosinophil count of 1100/µL. Which of the following is the most likely cause of her
symptoms?
A Early stage of systemic lupus erythematosus
B Gluten allergy
C Ingestion of L-tryptophan
D Lactose intolerance
E Recent viral upper respiratory tract infection
ANSWER:C
1381
A woman wants your advice regarding Papanicolaou smears. She is 36 years old and is
monogamous with her husband since they were married 3 years ago. She has had normal
Pap smears every year for the past 6 years. She would like to avoid the yearly test. What is
your advice to this patient, based on the current screening guidelines?
A
She may discontinue screening at age 50 if she has had normal yearly Pap smears for the
previous 10 years.
B She may extend the screening interval to once every 2–3 years.
C
She may extend the screening interval to once every 5 years if she agrees to use barrier
protection.
D She may discontinue Pap screening if she receives the human papilloma virus (HPV) vaccine.
E The only indication to cease Pap testing is if she were to have a total hysterectomy.
ANSWER:B

1382 Which of the following statements about lead-time bias occurrence is true?
A
A test does not influence the natural history of the disease; patients are merely diagnosed at
an earlier date.
B
Slow-growing, less aggressive cancers are detected during screening; aggressive cancers are
not detected by screening, due to death.
C
Screening identifies abnormalities that would never have caused a problem during a person’s
lifetime.
D
The screened population differs significantly from the general population in that they are
healthier.
E A test detects disease at an earlier and more curable stage of disease.
ANSWER:A
1383 Which of the following is sufficient to make a definitive diagnosis of porphyria?
A Appropriate clinical scenario including positive family history
B Evidence of an enzyme deficiency or gene defect
C Laboratory measurements in blood indicating accumulation of porphyrin precursors
D
Laboratory measurements in urine indicating accumulation of porphyrin precursors at the
time of symptoms
E
Laboratory measurements in stool indicating accumulation of porphyrin precursors at the
time of symptoms
ANSWER:B
1384 All but which of the following statements about the lupus anticoagulant (LA) are true?
A Lupus anticoagulants typically prolong the aPTT.
B A 1:1 mixing study will not correct in the presence of lupus anticoagulants.
C Bleeding episodes in patients with lupus anticoagulants may be severe and life-threatening.
D Female patients may experience recurrent midtrimester abortions.
E
Lupus anticoagulants may occur in the absence of other signs of systemic lupus
erythematosus (SLE).
ANSWER:C
1385 The most common inherited prothrombotic disorder is
A activated protein C resistance
B prothrombin gene mutation
C protein C deficiency

D protein S deficiency
E antithrombin deficiency
ANSWER:A
1386
A 34-year-old woman presents for evaluation of left lower extremity swelling and pain. She
is obese and 8 weeks postpartum. She recently traveled 6 h by airplane to visit her parents
with her infant. She has had no dyspnea, palpitations, or syncope. She is currently on no
medications except iron tablets. She is otherwise healthy. Her vitals signs are: heart rate 86
beats/min, blood pressure 110/80 mm/Hg, temperature 37.0°C, and respiratory rate 12
breaths/min. Her weight is 98 kg, and height is 170 cm. The left lower extremity is swollen,
tender, and warm to touch. A Homan’s sign is present, but there are no palpable cords. A
lower extremity Doppler shows a thrombosis in the common and superficial femoral veins of
the left leg. You are considering outpatient treatment with enoxaparin. All of the following
statements regarding low-molecular-weight heparins (LMWH) are true except
A
In patients with uncomplicated deep venous thrombosis (DVT), LMWH is a safe and
effective alternative to IV heparin and is associated with reduced health care costs compared
to IV heparin.
B
LMWH can be safely used in pregnancy, but factor Xa levels should be monitored to ensure
adequate anticoagulation.
C
Monitoring of factor Xa levels is unnecessary in most patients as there is a predictable dose-
dependent anticoagulation effect.
D
There is a decrease in the risk of development of heparin-induced thrombocytopenia with
use of LMWH.
E
This patient’s recent pregnancy is a contraindication to use of LMWH because there is a
greater risk of bleeding with LWMH compared to IV heparin.
ANSWER:E

1387
A 65-year-old man is brought to the emergency room by ambulance after his daughter
found him to be incoherent earlier today. She last spoke with him yesterday, and at that time,
he was complaining of 2 days of myalgias, headache, and fever. He had attributed it to an
upper respiratory tract infection and did not seek evaluation from his primary care physician.
Today, he did not answer when she called his home, and she found him lying in his bed
smelling of urine. He was minimally arousable but appeared to be moving all of his
extremities. His past medical history is significant for hypertension, hypercholesterolemia,
and chronic obstructive pulmonary disease. He was evaluated 2 weeks previously for a
transient ischemic attack after an episode where he had numbness and weakness of his left
arm and leg that resolved over 6 h without intervention. His current medications include
aspirin, 81 mg daily, clopidogrel, 75 mg daily, atenolol, 100 mg daily, atorvastatin, 20 mg
daily, and tiotropium, once daily. He is allergic to lisinopril, which caused angioedema. He is
a former smoker and drinks alcohol rarely. On physical examination, he is obtunded and
minimally arousable. He is febrile with a temperature of 38.9°C. His blood pressure is 159/96
mmHg, and heart rate is 98 beats/ min. He is breathing at a rate of 24 breaths/min with a
room air oxygen saturation of 95%. He has minimal scleral icterus. The oropharynx reveals
dry mucous membranes. His cardiovascular, pulmonary, and abdominal examinations are
normal. There are no rashes. His neurologic examination is difficult to obtain. There are no
cranial nerve findings. He resists movement of his extremities but has normal strength. Deep
tendon reflexes are brisk, 3+ and equal. The laboratory values are as follows: hemoglobin
9.3 g/ dL, hematocrit 29.1%, white blood cell count 14,000/µL, and platelets 42,000/µL. The
differential demonstrates 83% neutrophils, 2% band forms, 6% lymphocytes, and 9%
monocytes. The sodium is 145 meq/L, potassium 3.8 meq/L, chloride 113 meq/L,
bicarbonate 19 meq/L, blood urea nitrogen 68 mg/dL, and creatinine 3.4 mg/dL. The
bilirubin is 2.4 mg/dL, and lactate dehydrogenase is 450 U/L. A peripheral blood smear
shows diminished platelets and many schistocytes. What is the next most appropriate step in
this patient’s care?
A Discontinue clopidogrel.
B Discontinue clopidogrel and initiate plasmapheresis.
C Initiate therapy with intravenous immunoglobulin.
D
Obtain a head CT scan and initiate treatment with factor VIIa, if subarachnoid hemorrhage is
seen.

E
Perform a lumbar puncture and start broad-spectrum antibiotic coverage with ceftazidime
and vancomycin.
ANSWER:B
1388 A primary tumor of which of these organs is the least likely to metastasize to bone?
A Breast
B Colon
C Kidney
D Lung
E Prostate
ANSWER:B
1389
The triad of portal vein thrombosis, hemolysis, and pancytopenia suggests which of the
following diagnoses?
A Acute promyelocytic leukemia
B Hemolytic-uremic syndrome (HUS)
C Leptospirosis
D Paroxysmal nocturnal hemoglobinuria (PNH)
E Thrombotic thrombocytopenia purpura (TTP)
ANSWER:D
1390
A 68-year-old man seeks evaluation for fatigue, weight loss, and early satiety that have been
present for about 4 months. On physical examination, his spleen is noted to be markedly
enlarged. It is firm to touch and crosses the midline. The lower edge of the spleen reaches to
the pelvis. His hemoglobin is 11.1 g/dL, and hematocrit is 33.7%. The leukocyte count is
6200/µL, and platelet count is 220,000/µL. The white cell count differential is 75% PMNs,
8% myelocytes, 4% metamyelocytes, 8% lymphocytes, 3% monocytes, and 2% eosinophils.
The peripheral blood smear shows teardrop cells, nucleated red blood cells, and immature
granulocytes. Rheumatoid factor is positive. A bone marrow biopsy is attempted, but no cells
are able to be aspirated. No evidence of leukemia or lymphoma is found. What is the most
likely cause of the splenomegaly?
A Chronic idiopathic myelofibrosis
B Chronic myelogenous leukemia
C Rheumatoid arthritis
D Systemic lupus erythematosus

E Tuberculosis
ANSWER:A
1391 The most common cause of high serum calcium in a patient with a known cancer is
A ectopic production of parathyroid hormone
B direct destruction of bone by tumor cells
C local production of tumor necrosis factor and IL-6 by bony metastasis
D high levels of 1,25-hydroxyvitamin D
E production of parathyroid hormone–like substance
ANSWER:E
1392
A 72-year-old man with chronic obstructive pulmo-nary disease and stable coronary disease
presents to the emergency room with several days of worsening productive cough, fevers,
malaise, and diffuse muscle aches. A chest x-ray demonstrates a new lobar infiltrate.
Laboratory measurements reveal a total white blood cell count of 12,100 cells/µL, with a
neutrophilic predominance of 86% and 8% band forms. He is diagnosed with community-
acquired pneumonia, and antibiotic treatment is initiated. Under normal, or “nonstress,”
conditions, what percentage of the total body neutrophils are present in the circulation?
A 0,02
B 0,1
C 0,25
D 0,4
E 0,9
ANSWER:A
1393
All of the following laboratory values are consistentwith an intravascular hemolytic anemia
except
A increased haptoglobin
B increased lactate dehydrogenase (LDH)
C increased reticulocyte count
D increased unconjugated bilirubin
E increased urine hemosiderin
ANSWER:A
1394 All the following match the anticoagulant with its correct mechanism of action except

A abciximab—GpIIb/IIIa receptor inhibition
B clopidogrel—inhibition of thromboxane A2 release
C fondaparinux—inhibition of factor Xa
D argatroban—thrombin inhibition
E warfarin—vitamin K—dependent carboxylation of coagulation factors
ANSWER:B
1395
All the following are late complications of bone marrow transplant preparative regimens
except
A growth retardation
B azoospermia
C hypothyroidism
D cataracts
E dementia
ANSWER:E
1396 Which of the following best describes the mechanism of action of clopidogrel?
A Activates antithrombin and inhibits clotting enzymes
B
Binds to the activated GPIIb/IIIa receptor on the platelet surface to block binding of adhesive
molecules
C Inhibits cyclooxygenase 1 (COX-1) on platelets to decrease production of thromboxane A2
D
Inhibits phosphodiesterase to block the breakdown of cyclic adenosine monophosphate
(cAMP) to inhibit platelet activation
E
Irreversibly blocks P2Y12 to prevent adenosine diphosphate (ADP)–induced platelet
aggregation
ANSWER:E

1397
A 45-year-old man is evaluated by his primary care physician for complaints of early satiety
and weight loss. On physical examination, his spleen is palpable 10 cm below the left costal
margin and is mildly tender to palpation. His laboratory studies show a leukocyte count of
125,000/µL with a differential of 80% neutrophils, 9% bands, 3% myelocytes, 3%
metamyelocytes, 1% blasts, 1% lymphocytes, 1% eosinophils, and 1% basophils.
Hemoglobin is 8.4 g/dL, hematocrit 26.8%, and platelet count 668,000/µL. A bone marrow
biopsy demonstrates increased cellularity with an increased myeloid to erythroid ratio. Which
of the following cytogenetic abnormalities is most likely to be found in this patient?
A Deletion of a portion of the long arm of chromosome 5, del(5q)
B Inversion of chromosome 16, inv(16)
C Reciprocal translocation between chromosomes 9 and 22 (Philadelphia chromosome)
D Translocations of the long arms of chromosomes 15 and 17
E Trisomy 12
ANSWER:C
1398
A 35-year-old patient comes into your office with persistent iron deficiency anemia. His past
medical history is significant for end-stage renal disease on hemodialysis, hypertension, and
rheumatoid arthritis. His medications include calcium acetate, a multivitamin, nifedipine,
aspirin, iron sulfate, and omeprazole. His hemoglobin 6 months ago was 8 mg/dL. One week
ago, it was 7.9 mg/dL. His ferritin is 8 mg/dL. He reports no bright red blood per rectum,
and his stool guaiac examinations have been repeatedly negative over the past 6 months.
What is the most likely cause of this patient’s iron deficiency anemia?
A Celiac sprue
B Colon cancer
C Hemorrhoids
D Medication effect
E Peptic ulcer disease
ANSWER:D

1399
A 32-year-old male presents complaining of a testicular mass. On examination you palpate a
1-by 2-cm painless mass on the surface of the left testicle. A chest x-ray shows no lesions,
and a CT scan of the abdomen and pelvis shows no evidence of retroperitoneal adenopathy.
The α fetoprotein (AFP) level is elevated at 400 ng/mL. Beta human chorionic gonadotropin
(β-hCG) is normal, as is LDH. You send the patient for an orchiectomy. The pathology
comes back as seminoma limited to the testis alone. The AFP level declines to normal at an
appropriate interval. What is the appropriate management at this point?
A Radiation to the retroperitoneal lymph nodes
B Adjuvant chemotherapy
C Hormonal therapy
D Retroperitoneal lymph node dissection (RPLND)
E Positron emission tomography (PET) scan
ANSWER:D
1400 All of the following statements regarding tobacco usage and cessation are correct except
A
Most Americans who quit do so on their own without involvement in an organized cessation
program.
B Over 80% of adult Americans who smoke began before the age of 18.
C Smokeless tobacco is associated with gum and dental disease, not cancer.
D
Tobacco cessation messages and programs are more effective for light smokers than for
heavy smokers.
E Tobacco use is the most modifiable cancer risk factor.
ANSWER:C
1401
A 29-year-old male is found on routine chest radiography for life insurance to have right
hilar adenopathy. He is otherwise healthy. Besides biopsy of the lymph nodes, which of the
following is indicated?
A Angiotensin-converting enzyme (ACE) level
B β-hCG
C Thyroid stimulating hormone (TSH)
D PSA
E C-reactive protein
ANSWER:B

1402
Which of the following is correct regarding small-cell lung cancer compared with non-small
cell lung cancer?
A Small-cell lung cancer is more radiosensitive.
B Small-cell lung cancer is less chemosensitive.
C Small-cell lung cancer is more likely to present peripherally in the lung.
D Small-cell lung cancer is derived from an alveolar cell.
E Bone marrow involvement is more common in nonsmall cell lung cancer.
ANSWER:A
1403 I54. Which of the following statements regarding esophageal cancer is true?
A Cigarette smoking and heavy alcohol intake are synergistic risk factors for adenocarcinoma.
B
Chronic gastric reflux is a risk factor for development of esophageal squamous cell
carcinoma.
C Esophageal cancer is most common in the middle third of the esophagus.
D
Incidence of squamous cell carcinoma has decreased over the past 30 years while
adenocarcinoma continues to increase.
E
The prognosis for patients with adenocarcinoma is consistently better than for those with
squamous cell carcinoma.
ANSWER:D
1404 I55. All the following conditions are associated with an increased incidence of cancer except
A Down’s syndrome
B Fanconi’s anemia
C Von Hippel–Lindau syndrome
D neurofibromatosis
E fragile X syndrome
ANSWER:E
1405
A 50-year-old female presents to your clinic for evaluation of an elevated platelet count. The
latest complete blood count is white blood cells (WBC) 7,000/mm3, hematocrit 34%, and
platelets 600,000/mm3. All the following are common causes of thrombocytosis except
A iron-deficiency anemia

B essential thrombocytosis
C chronic myeloid leukemia
D myelodysplasia
E pernicious anemia
ANSWER:E
1406
A 76-year-old man presents to an urgent care clinic with pain in his left leg for 4 days. He
also describes swelling in his left ankle, which has made it difficult for him to ambulate. He is
an active smoker and has a medical history remarkable for gastroesophageal reflux disease,
prior deep venous thrombosis (DVT) 9 months ago that resolved, and well-controlled
hypertension. Physical examination is revealing for 2+ edema in his left ankle. A D-dimer is
ordered and is elevated. Which of the following makes D-dimer less predictive of DVT in
this patient?
A Age >70
B History of active tobacco use
C Lack of suggestive clinical symptoms
D Negative Homan’s sign on examination
E Previous DVT in the past year
ANSWER:A
1407
A patient with longstanding HIV infection, alcoholism, and asthma is seen in the emergency
room for 1–2 days of severe wheezing. He has not been taking any medicines for months.
He is admitted to the hospital and treated with nebulized therapy and systemic
glucocorticoids. His CD4 count is 8 and viral load is >750,000. His total white blood cell
(WBC) count is 3200 cells /µL with 90% neutrophils. He is accepted into an inpatient
substance abuse rehabilitation program and before discharge is started on opportunistic
infection prophylaxis, bronchodilators, a prednisone taper over 2 weeks, ranitidine, and
highly-active antiretroviral therapy. The rehabilitation center pages you 2 weeks later; a
routine laboratory check reveals a total WBC count of 900 cells/µL with 5% neutrophils.
Which of the following new drugs would most likely explain this patient’s neutropenia?
A Darunavir
B Efavirenz
C Ranitidine
D Prednisone

E Trimethoprim-sulfamethoxazole
ANSWER:E
1408 Which of the following symptoms is most suggestive of an esophageal mass?
A Early satiety
B Liquid phase dysphagia only
C Odynophagia with chest pain
D Oropharyngeal dysphagia
E Solid phase dysphagia progressing to liquid phase dysphagia
ANSWER:E
1409
All of the following have been associated with development of a lymphoid malignancy
except
A celiac sprue
B Helicobacter pylori infection
C hepatitis B infection
D HIV infection
E human herpes virus 8 (HHV8) infection
ANSWER:C
1410
A 31-year-old female is referred to your clinic for an evaluation of anemia. She describes a
2-month history of fatigue. She denies abdominal pain but notes that her abdomen has
become slightly more distended in recent weeks. Past medical history is otherwise
unremarkable. The patient’s parents are alive, and she has three healthy siblings. Physical
examination is significant for pale conjunctiva and a palpable spleen 4 cm below the left
costal margin. Hematocrit is 31% and bilirubin is normal. The reticulocyte percentage is low.
Haptoglobin and lactic dehydrogenase (LDH) are normal. A peripheral blood smear shows
numerous teardrop-shaped red cells, nucleated red cells, and occasional myelocytes. A bone
marrow aspirate is unsuccessful, but a biopsy shows a hypercellular marrow with trilineage
hyperplasia and findings consistent with the presumed diagnosis of chronic idiopathic
myelofibrosis. You transfuse her to a hematocrit of 40%. What is the most appropriate next
management step?
A Administer erythropoietin.
B Follow up in 6 months.
C Institute combined-modality chemotherapy.

D Perform HLA matching of her siblings.
E Perform a splenectomy.
ANSWER:D
1411 All the following are suggestive of iron deficiency anemia except
A koilonychia
B pica
C decreased serum ferritin
D decreased total iron-binding capacity (TIBC)
E low reticulocyte response
ANSWER:D
1412
A 59-year-old man is admitted with a painful, blistering rash on the dorsal aspects of both
hands. He has a medical history of alcoholism and admits to a recent relapse and has been
drinking heavily over the past week. He is admitted and stabilized. A diagnosis of porphyria
cutanea tarda (PCT) is made based on increased circulating porphyrins in the blood and
decreased URO-decarboxylase activity. He is discharged to a rehabilitation facility and
follows up in your clinic 2 weeks later. He has been abstinent from alcohol but his rash has
persisted, and now he also has some blistering on the legs and feet. Which of the following
treatment modalities is most appropriate?
A Hydroxyurea
B IV iron infusion weekly while following serum iron levels
C Oral iron plus vitamin C
D
The rash of PCT can take months to resolve; the patient should continue to abstain from
alcohol and be followed closely
E Weekly phlebotomy until ferritin normalizes
ANSWER:E
1413 Which of the following hemolytic anemias can be classified as extracorpuscular?
A Elliptocytosis
B Paroxysmal nocturnal hemoglobinuria
C Pyruvate kinase deficiency
D Sickle cell anemia
E Thrombotic thrombocytopenic purpura
ANSWER:E

1414
All of the following are obstacles to the more widespread application of stem cells for
regenerative medicine except
A controlling the migration of transplanted stem cells
B identifying diseases suitable for stem cell based therapies
C identifying the pathways for differentiating stem cells into specific cell types
D overcoming ethical concerns over their harvest and use
E predicting the response of cells to the environment of the diseased organ
ANSWER:B
1415
You are asked to consult on a 34-year-old male with thrombocytopenia. He sustained a
motor vehicle collision 10 days ago, resulting in shock, internal bleeding, and acute renal
failure. An exploratory laparotomy was performed that showed a ruptured spleen requiring a
splenectomy. He also underwent an open reduction and internal fixation of the left femur.
The platelet count was 260,000 cells/µL on admission. Today it is 68,000 cells/µL. His
medications are oxacillin, morphine, and subcutaneous heparin. On examination the vital
signs are stable. The examination is significant for an abdominal scar that is clean and
healing. The patient’s left leg is in a large cast and is elevated. The right leg is swollen from
the calf downward. Ultrasound of the right leg shows a deep venous thrombosis. Antiheparin
antibodies are positive. Creatinine is 3.2 mg/dL. What is the most appropriate next
management step?
A Discontinue heparin.
B Stop heparin and start enoxaparin.
C Stop heparin and start argatroban.
D Stop heparin and start lepirudin.
E Observe the patient.
ANSWER:C
1416
A 64-year-old man with chronic lymphoid leukemia (CLL) and chronic hepatitis C presents
for his yearly follow-up. His white blood cell count is stable at 83000/µL, but his hematocrit
has dropped from 35% to 26% and his platelet count also dropped from 178,000/µL to
69,000/ µL. His initial evaluation should include all of the following except
A AST, ALT, and prothrombin time
B bone marrow biopsy
C Coomb’s test

D peripheral blood smear
E physical examination
ANSWER:B
1417
A 64-year-old man with Child-Pugh class B cirrhosis presents to his gastroenterologist
complaining of weight loss and a feeling of abdominal fullness. He was diagnosed with
hepatitis C cirrhosis 5 years previously. It is thought that the patient developed with hepatitis
C following a blood transfusion 20 years ago after a car accident. His initial presentation with
cirrhosis was volume overload and ascites. He has been successfully managed with sodium
restriction, spironolactone, and furosemide. He has no other significant medical history. On
examination today, his liver is enlarged and firm. No ascites is present. A helical CT of the
abdomen shows a single tumor in the right lobe of the liver measuring 4 cm in diameter. The
location of the mass is near the main portal pedicles. There is no evidence of vascular
invasion or metastatic lesions. The α fetoprotein level is 384 ng/mL. Biopsy of the mass is
diagnostic for hepatocellular carcinoma. What is the best approach for treatment?
A Liver transplantation
B Radiofrequency ablation
C Resection of the right hepatic lobe
D Systemic chemotherapy
E Transarterial chemoembolization
ANSWER:A
1418
Which of the following should prompt investigation for hereditary nonpolyposis colon
cancer screening in a 32-year-old man?
A
Father, paternal aunt, and paternal cousin with colon cancer with ages of diagnosis of 54, 68,
and 37 years, respectively
B Innumerable polyps visualized on routine colonoscopy
C Mucocutaneous pigmentation
D New diagnosis of ulcerative colitis
E None of the above
ANSWER:A
1419 Which of the following carries the best disease prognosis with appropriate treatment?
A Burkitt’s lymphoma

B Diffuse large B cell lymphoma
C Follicular lymphoma
D Mantle cell lymphoma
E Nodular sclerosing Hodgkin’s disease
ANSWER:E
1420
You are asked to consult on a 31-year-old male with prolonged bleeding after an oral
surgery procedure. He has no prior history of bleeding diathesis or family history of
bleeding disorders. The patient’s past medical history is remarkable for infection with the
human immunodeficiency virus, with a CD4 count of 51/ mL3. The examination is
remarkable only for spotty lymphadenopathy. The platelet count is 230,000 cells/ mL. His
international normalized ratio (INR) is 1.5. Activated partial thromboplastin time is 40 s.
Peripheral blood smear shows no schistocytes and is otherwise unremarkable. A 1:1 mixing
study corrects both conditions immediately and after a 2-h incubation. Fibrinogen level is
normal. Thrombin time is prolonged. What is the diagnosis?
A Disseminated intravascular coagulation (DIC)
B Dysfibrinogenemia
C Factor V deficiency
D Liver disease
E Factor XIII deficiency
ANSWER:B
1421
Chemoprevention strategies for cancer have met with varying levels of success. Which of the
following pairings correctly identifies an effective chemoprevention strategy with its target
effect?
A Aspirin: colon cancer
B β-Carotene: lung cancer
C Calcium: adenomatous gastrointestinal polyps
D Isotretinoin: oral leukoplakia
E Tamoxifen: endometrial cancer
ANSWER:C

1422
A 48-year-old woman is admitted to the hospital with anemia and thrombocytopenia after
complaining of profound fatigue. Her initial hemoglobin is 8.5 g/dL, hematocrit 25.7%, and
platelet count 42,000/µL. Her leukocyte count is 9540/µL, but 8% blast forms are noted on
peripheral smear. A chromosomal analysis shows a reciprocal translocation of the long arms
of chromosomes 15 and 17, t(15;17), and a diagnosis of acute promyelocytic leukemia is
made. The induction regimen of this patient should include which of the following drugs:
A All-trans-retinoic acid (ATRA, or triretinoin)
B Arsenic
C Cyclophosphamide, daunorubicin, vinblastine, and prednisone
D Rituximab
E Whole-body irradiation
ANSWER:A
1423
The patient above is started on the appropriate induction regimen. Two weeks following
initiation of treatment, the patient develops acute onset of shortness of breath, fever, and
chest pain. Her chest radiograph shows bilateral alveolar infiltrates and moderate bilateral
pleural effusions. Her leukocyte count is now 22,300/µL, and she has a neutrophil count of
78%, bands of 15%, and lymphocytes 7%. She undergoes bronchoscopy with lavage that
shows no bacterial, fungal, or viral organisms. What is the most likely diagnosis in this
patient?
A Arsenic poisoning
B Bacterial pneumonia
C Cytomegalovirus pneumonia
D Radiation pneumonitis
E Retinoic acid syndrome
ANSWER:E

1424
A 76-year-old man is admitted to the hospital with complaints of fatigue for 4 months and
fever for the past 1 week. His temperature has been as high as 38.3°C at home. During this
time, he intermittently has had a 5.5kg weight loss, severe bruising with minimal trauma, and
an aching sensation in his bones. He last saw his primary care physician 2 months ago and
was diagnosed with anemia of unclear etiology at that time. He has a history of a previous
left middle cerebral artery cerebrovascular accident which has left him with decreased
functional status. At baseline, he is able to ambulate in his home with the use of a walker and
is dependent upon a caregiver for assistance with his activities of daily living. His vital signs
are: blood pressure 158/86 mmHg, heart rate 98 beats/ min, respiratory rate 18 breaths/min,
SaO2 95%, and temperature 38°C. He appears cachectic with temporal muscle wasting. He
has petechiae on his hard palate. He has no lymph node enlargement. On cardiovascular
examination, there is a II/VI systolic ejection murmur present. His lungs are clear. The liver
is enlarged and palpable 6 cm below the right costal margin. In addition, the spleen is also
enlarged, with a palpable spleen tip felt about 4 cm below the left costal margin. There are
multiple hematomas and petechiae present in the extremities. Laboratory examination reveals
the following: hemoglobin 5.1 g/dL, hematocrit 15%, platelets 12,000/µL, and white blood
cell (WBC) count 168,000/µL with 45% blast forms, 30% neutrophils, 20% lymphocytes,
and 5% monocytes. Review of the peripheral blood smear confirms acute myeloid leukemia
(M1 subtype, myeloblastic leukemia without maturation) with complex chromosomal
abnormalities on cytogenetics. All of the following confer a poor prognosis for this patient
except
A advanced age
B complex chromosomal abnormalities on cytogenetics
C hemoglobin <7 g/dL
D prolonged interval between symptom onset and diagnosis
E WBC count >100,000/µL
ANSWER:C
1425
A 56-year-old patient inquires about screening for colon cancer. He has no risk factors for
colon cancer, other than age. Which of the following statements is true regarding which
screening test you recommend for this patient?
A 50% of patients with a positive fecal occult blood testing have colon cancer.

B
One-time colonoscopy detects more advanced lesions than one-time fecal occult blood
testing with sigmoidoscopy.
C Perforation rates for sigmoidoscopy and colonoscopy are equivalent.
D Sigmoidoscopy has not been shown to reduce mortality.
E
Virtual colonoscopy is as effective as endoscopic colonoscopy for detecting polyps <5 mm
in size.
ANSWER:B
1426
A 65-year-old man seeks evaluation for nasal congestion, headaches, and dysphagia, most
notably when he lies supine for sleeping. These symptoms have been slowly worsening for
the past month. He has no nasal discharge or fevers. On review of systems, he reports recent
hoarseness and dizziness. His past medical history is significant only for mild hypertension.
He worked as a roofing contractor and smoked one pack/day of cigarettes since age 16. On
physical examination, you note facial edema. His oropharynx is also mildly edematous, and
the tonsils are unremarkable. His external and internal jugular veins are engorged bilaterally,
and there are prominent veins on the anterior chest. Chest percussion reveals dullness in the
right base with decreased tactile fremitus. A chest radiograph shows a right upper lung mass
that on biopsy is consistent with non-small cell lung cancer. All of the following treatments
may help this patient’s symptoms except
A chemotherapy
B diuretics
C glucocorticoids
D radiation therapy
E venous stenting
ANSWER:C
1427
All of the following statements regarding the epidemiology of and risk factors for acute
myeloid leukemias are true except
A
Anticancer drugs such as alkylating agents and topoisomerase II inhibitors are the leading
cause of drug-associated myeloid leukemias.
B
Individuals exposed to high-dose radiation are at risk for acute myeloid leukemia whereas
individuals treated with therapeutic radiation are not unless they are also treated with
alkylating agents.
C Men have a higher incidence of acute myeloid leukemia than women.

D The incidence of acute myeloid leukemia is greatest in individuals <20 years.
E Trisomy 21 (Down syndrome) is associated with an increased risk of acute myeloid leukemia.
ANSWER:D
1428
A 42-year-old man presented to the hospital with right upper quadrant pain. He was found to
have multiple masses in the liver that were found to be malignant on H&E staining of a
biopsy sample. Your initial history, physical examination and laboratory tests, including
prostate-specific antigen, are unrevealing. Lung, abdominal, and pelvic CT scans are
unremarkable. He is an otherwise healthy individual with no chronic medical problems.
Which immunohistochemical markers should be obtained from the biopsy tissue?
A α Fetoprotein
B Cytokeratin
C Leukocyte common antigen
D Thyroglobulin
E Thyroid transcription factor 1
ANSWER:B
1429
A 56-year-old woman is diagnosed with chronic myelogenous leukemia, Philadelphia
chromosome–positive. Her presenting leukocyte count was 127,000/µL, and her differential
shows <2% circulating blasts. Her hematocrit is 21.1% at diagnosis. She is asymptomatic
except for fatigue. She has no siblings. What is the best initial therapy for this patient?
A Allogeneic bone marrow transplant
B Autologous stem cell transplant
C Imatinib mesylate
D Interferon-α
E Leukapheresis.
ANSWER:C
1430
All the following are associated with a reduced lifetime risk of developing breast cancer
except
A absence of a history of maternal nursing
B first full-term pregnancy before age 18 years

C menarche after age 15 years
D natural menopause before age 42 years
E surgical menopause before age 42 years
ANSWER:A
1431
All the following cause prolongation of the activated partial thromboplastin time (aPTT) that
does not correct with a 1:1 mixture with pooled plasma except
A lupus anticoagulant
B factor VIII inhibitor
C heparin
D factor VII inhibitor
E factor IX inhibitor
ANSWER:D
1432
A 53-year-old woman seeks evaluation from her primary care physician regarding primary
prevention of cardiovascular disease and stroke. She has a past medical history of type 2
diabetes mellitus for the past 5 years with a known hemoglobin A1C of 7.2%. She does not
have hypertension or known coronary artery disease. She has been obese throughout
adulthood, and her BMI is 33.6 kg/m2. She is currently perimenopausal with irregular
bleeding that last occurred 3 months ago. She is taking metformin, 1000 mg twice daily. She
has been intolerant of ibuprofen in the past due to gastrointestinal upset. She previously
smoked one pack of cigarettes daily from the ages of 18 to 38. She drinks a glass of wine
with dinner. Her family history is significant for myocardial infarction in her father at age
58, paternal uncle at age 67, and paternal grandmother at age 62. On the maternal side, her
mother died of a stroke at age 62. She is concerned that she should be taking a daily aspirin
as primary prevention of cardiovascular disease and stroke but is also concerned about
potential side effects. Which of the following statements regarding aspirin therapy is true?
A
Aspirin is indicated for primary prevention of cardiovascular disease because she has a
strong family history and has a history of diabetes mellitus.
B
Aspirin is only indicated for secondary prevention of cardiovascular and cerebrovascular
disease in women.
C
Because she is not postmenopausal, aspirin therapy is not recommended as it will increase
menstrual bleeding without significantly decreasing the risk of cardiovascular disease.

D
Her adverse reaction to ibuprofen prevents use of aspirin because there is a high degree of
cross-reactivity, and she is at risk for development of bronchospasm with aspirin use.
E
The risk of major bleeding related to use of aspirin is 1–3% per year, but use of an enteric-
coated or buffered aspirin will eliminate this risk.
ANSWER:A
1433
A 22-year-old man comes into clinic because of a swollen leg. He does not remember any
trauma to the leg, but the pain and swelling began 3 weeks ago in the anterior shin area of
his left foot. He is a college student and is active in sports daily. A radiograph of the right leg
shows a destructive lesion with a “moth-eaten” appearance extending into the soft tissue and
a spiculated periosteal reaction. Codman’s triangle (a cuff of periosteal bone formation at the
margin of the bone and soft tissue mass) is present. What is the most likely diagnosis and
optimal therapy for this lesion?
A Chondrosarcoma; chemotherapy alone is curative
B Chondrosarcoma; radiation with limited surgical resection
C Osteosarcoma; preoperative chemotherapy followed by limb-sparing surgery
D Osteosarcoma; radiation therapy
E Plasma cell tumor; chemotherapy
ANSWER:C
1434 Which of the following statements is true?
A
Factor VIII deficiency is characterized clinically by bleeding into soft tissues, muscles, and
weightbearing joints.
B Congenital factor VIII deficiency is inherited in an autosomal recessive fashion.
C Factor VIII deficiency results in prolongation of the prothrombin time.
D Factor VIII complexes with Hageman factor, allowing for a longer half-life.
E Factor VIII has a half-life of nearly 24 h.
ANSWER:A
1435 All of the following statements regarding gastric carcinoma are true except
A
Linitis plastica is an infiltrative form of gastric lymphoma with no defined margins that
carries a poorer prognosis than intestinal-type lesions.
B
Reduction of tumor bulk with surgery is the best therapeutic option for gastric
adenocarcinoma, if surgically feasible.

C
The long-term ingestion of high concentrations of nitrates in dried, smoked, or salted foods
is associated with higher rates of gastric cancer.
D The presence of palpable, firm peri-umbilical nodules is a poor prognostic sign.
E
Ulcerative lesions in the distal stomach should always undergo brush sampling and biopsy to
rule out adenocarcinoma.
ANSWER:A
1436
Which of the following statements regarding malignant spinal cord compression (MSCC) is
true?
A Less than 50% of patients who are treated while ambulatory will remain ambulatory.
B
Neurologic abnormalities on physical examination are sufficient to initiate high-dose
glucocorticoids.
C Neurologic findings often appear before pain.
D Renal cell carcinoma is the most common cause of MSCC.
E The lumbosacral spine is the most commonly affected site.
ANSWER:B
1437 All the following are characteristic of tumor lysis syndrome except
A hyperkalemia
B hypercalcemia
C lactic acidosis
D hyperphosphatemia
E hyperuricemia
ANSWER:B
1438
A 22-year-old woman comes to the emergency department complaining of 12 h of
shortness of breath. The symptoms began towards the end of a long car ride home from
college. She has no past medical history and her only medication is an oral contraceptive.
She smokes occasionally but the frequency has increased recently because of examinations.
On physical examination, she is afebrile with respiratory rate of 22 breaths/ min, blood
pressure 120/80 mmHg, heart rate 110 beats/min, SaO2 (room air) 92%. The rest of her
physical examination is normal. A chest radiograph and complete blood count are normal.
Her serum pregnancy test is negative. Which of the following is the indicated management
strategy?
A Check D-dimer and, if normal, discharge with nonsteroidal anti-inflammatory therapy.

B Check D-dimer and, if normal, obtain lower extremity ultrasound.
C
Check D-dimer and, if abnormal, treat for deep venous thrombosis/pulmonary embolism
(DVT/PE).
D Check D-dimer and, if abnormal, obtain contrast multislice CT of chest.
E Obtain contrast multislice CT of chest.
ANSWER:E
1439
The patient described above is found to have a right pulmonary embolus. She is started on
low-molecular-weight heparin and warfarin. What is the goal international normalized ratio
(INR) and the duration of therapy?
A INR 3.5; 1 month
B INR 2.5; 3 months
C INR 3.5; 3 months
D INR 2.5; 6 months
E INR 3.5; 6 months
ANSWER:D
1440
A patient asks you about the utility of performing monthly breast self-examination (BSE).
Which of the following statements is correct regarding the utility of and recommendations
regarding breast self-examination?
A Breast self-examination reduces mortality only in women who undergo breast biopsy.
B Most screening societies recommend performing BSE monthly for women >20 years.
C Self-examination leads to increased biopsy rate.
D Very few breast cancers are first detected by patients.
E Breast self-examination leads to improved survival from breast cancer.
ANSWER:C
1441
Which of the following tumor characteristics confers a poor prognosis in patients with breast
cancer?
A Estrogen receptor-positive
B Good nuclear grade
C Low proportion of cells in S-phase
D Overexpression of erbB2 (HER-2/neu)
E Progesterone receptor-positive
ANSWER:D

1442
Which of the following serum laboratory tests is most useful for predicting return of renal
function in a patient with tumor lysis syndrome and acute renal failure?
A Creatinine
B Phosphate
C Potassium
D Serum pH
E Uric acid
ANSWER:E
1443 Fondaparinux may be used to treat all of the following patients except
A
A 33-year-old woman weighing 48 kg presents with a pulmonary embolus 2 months after a
motor vehicle accident that resulted in a fractured femur.
B
A 46-year-old man with hypertension and focal segmental glomerulosclerosis with a baseline
creatinine of 3.3 mg/dL presents with a left lower extremity deep
C
A 57-year-old woman had an aortic valve replacement 7 days ago. The platelet count
preoperatively was 320,000/µL. On day 7, the platelet count is 122,000/µL.
D
A 60-year-old man presents to the hospital with chest pain and ST-segment depression in
leads II, III, and aVF on electrocardiogram. Troponin I level is 2.32 ng/mL.
E A 68-year-old man has undergone an uncomplicated right total hip replacement.
ANSWER:B
1444
A 26-year-old female who is 4 months pregnant is seen for a standard evaluation. She
reports feeling well with decreasing nausea over the last 1 month. The physical examination
is normal except for the presence of a 1.5-cm hard nodule in the upper outer quadrant of the
right breast. She does not recall the nodule being present previously and has not performed
self-examination since be coming pregnant. Which of the following is the next most
appropriate action?
A Aspiration of the nodule
B Mammogram after delivery
C Prescription of oral progesterone therapy
D Recommendation of genetic testing for BRCA-1
E Repeat physical examination after delivery
ANSWER:A
1445 Aplastic anemia has been associated with all of the following except

A carbamazepine therapy
B methimazole therapy
C non-steroidal inflammatory drugs
D parvovirus B19 infection
E seronegative hepatitis
ANSWER:D
1446
A 23-year-old man presents with diffuse bruising. He otherwise feels well. He takes no
medications, does not use dietary supplements, and does not use illicit drugs. His past
medical history is negative for any prior illnesses. He is a college student and works as a
barista in a coffee shop. A blood count reveals an absolute neutrophil count of 780/µL,
hematocrit of 18% and platelet count of 21,000/µL. Bone marrow biopsy reveals
hypocellularity with a fatty marrow. Chromosome studies of peripheral blood and bone
marrow cells are performed which exclude Fanconi’s anemia and myelodysplastic syndrome.
The patient has a fully histocompatible brother. Which of the following is the best therapy?
A Anti-thymocyte globulin plus cyclosporine
B Glucocorticoids
C Growth factors
D Hematopoietic stem cell transplant
E Red blood cell and platelet transfusion
ANSWER:D
1447
A 46-year-old woman presents with new onset ascites and severe abdominal pain: a hepatic
Doppler examination reveals hepatic vein thrombosis. She also reports tea colored urine on
occasion, particularly in the morning, as well as recurrent worsening abdominal pain. On
further evaluation, she is found to have an undetectable serum haptoglobin, elevated serum
lactase dehydrogenase, hemoglobinuria and an elevated reticulocyte count. A peripheral
smear shows no schistocytes. What is the most likely diagnosis?
A Adenocarcinoma of the ovary
B Antiphospholipid syndrome
C Aplastic anemia
D Factor V Leiden deficiency
E Paroxysmal nocturnal hemoglobinuria

ANSWER:E
1448
A Sudanese refugee is brought to see you in clinic for abdominal pain. He has had
intermittent fevers for months and has lost considerable weight. He was previously a guard
for a refugee camp in the Sudan and worked the night shift exclusively. On examination, he
is severely malnourished with temporal wasting. He has massive splenomegaly but no
palpable lymphadenopathy. Oropharynx shows no thrush. Laboratory data reveal an anemia,
neutropenia, and thrombocytopenia. Skin examination shows no discrete lesions but you and
the patient notice that the skin appears gray throughout. Malaria smears are negative and
HIV testing is negative. Chest X-ray is normal. What is the most likely diagnosis?
A Cirrhosis
B Kala-azar (visceral leishmaniasis)
C Kaposi’s sarcoma
D Miliary tuberculosis
E Sickle cell anemia
ANSWER:B
1449
A 16-year-old male has recurrent thigh hematomas. He has been active in sports all of his life
and has had 3 episodes of limb-threatening bleeding with compartment syndrome. A family
history is notable for a maternal grandfather with a similar bleeding history. Paternal family
history is not available. Laboratory analysis in clinic reveals a normal platelet count, a
normal activated partial thromboplastin time (22 s) and a prolonged prothrombin time (25
s). He takes no medications. What is the most likely reason for his coagulation disorder?
A Factor VIII deficiency
B Factor VII deficiency
C Factor IX deficiency
D Prothrombin deficiency
E Surreptitious warfarin ingestion
ANSWER:B

1450
A 52-year-old man is admitted with recurrent hemarthroses of his knees. He is an electrician
who is still working but over the last year has had recurrent hemarthroses requiring surgical
evacuation. Before one year ago, he had no medical problems. He has no other past medical
history and seldom sees a physician. He smokes tobacco regularly. His platelet count is
normal, erythrocyte sedimentation rate is 55 mm/hr, hemoglobin is 9 mg/dL and albumin is
3.1 mg/dL. Coagulation studies show a prolonged activated partial thromboplastin time
(aPTT) and a normal prothrombin time (PT). Adding plasma from a normal subject does not
correct the aPTT. What is the cause of his recurrent hemarthroses?
A Acquired inhibitor
B Factor VIII deficiency
C Factor IX deficiency
D Secondary syphilis
E Vitamin C deficiency
ANSWER:C
1451
You are evaluating a 45-year-old man with an acute upper GI bleed in the emergency
department. He reports increasing abdominal girth over the past 3 months associated with
fatigue and anorexia. He has not noticed any lower extremity edema. His past medical
history is significant for hemophilia A diagnosed as a child with recurrent elbow
hemarthroses in the past. He has been receiving infusions of factor VIII for most of his life,
and received his last injection earlier that day. His blood pressure is 85/45 mmHg with a
heart rate of 115/min. His abdominal examination is tense with a positive fluid wave.
Hematocrit is 21%. Renal function and urinalysis is normal. His aPTT is minimally
prolonged, his INR is 2.7, platelets are normal. Which of the following is most likely to yield
a diagnosis for the cause of his GI bleeding?
A Factor VIII activity level
B H. pylori antibody test
C Hepatitis B surface antigen
D Hepatitis C RNA
E Mesenteric angiogram
ANSWER:D

1452
You are managing a patient with suspected disseminated intravascular coagulopathy (DIC).
The patient has end-stage liver disease awaiting liver transplantation and was recently in the
intensive care unit with E. coli bacterial peritonitis. You suspect DIC based on a new upper
gastrointestinal bleed in the setting of oozing from venipuncture sites. Platelet count is
43000/µL, INR is 2.5, hemoglobin is 6 mg/dL and D-dimer is elevated to 4.5. What is the
best way to distinguish between new-onset DIC and chronic liver disease?
A Blood culture
B Elevated fibrinogen degradation products
C Prolonged aPTT
D Reduced platelet count
E Serial laboratory analysis
ANSWER:E
1453
A 38-year-old woman is referred for evaluation of an elevated hemoglobin and hematocrit
that was discovered during an evaluation of recurrent headaches. Until about 8 months
previously, she was in good health, but developed increasingly persistent headaches with
intermittent vertigo and tinnitus. She was originally prescribed sumatriptan for presumed
migraine headaches, but did not experience relief of her symptoms. A CT scan of the brain
showed no evidence of mass lesion. During evaluation of her headaches, she was found to
have a hemoglobin of 17.3 g/dL, and a hematocrit of 52%. Her only other symptom is
diffuse itching after hot showers. She is a non-smoker. She has no history pulmonary or
cardiac disease. On physical examination, she appears well. Her BMI is 22.3 kg/m2. Vitals
signs are BP 148/84 mmHg, HR 86/min, RR 12/min, SaO2 99% on room air. She is afebrile.
The physical examination including full neurologic examination is normal. There are no
heart murmurs. There is no splenomegaly. Peripheral pulses are normal. Laboratory studies
confirm elevated hemoglobin and hematocrit. She also has a platelet count of 650,000/µL.
Leukocyte count is 12,600/µL with a normal differential. Which of the following tests should
be performed next in the evaluation of this patient?
A Bone marrow biopsy
B Erythropoietin level
C Genetic testing for JAK2 V617F mutation
D Leukocyte alkaline phosphatase

E Red cell mass and plasma volume determination
ANSWER:E
1454
A 46-year-old man is admitted to the hospital for upper gastrointestinal (GI) bleeding. He
has a known history of peptic ulcer disease, for which he takes a protonpump inhibitor. His
last admission for upper GI bleeding was 4 years ago. After fluid resuscitation, he is
hemodynamically stable and his hematocrit has not changed in the past 8 h. Upper
endoscopy is performed. Which of the following findings at endoscopy is most reassuring
that the patient will not have a significant rebleeding episode within the next 3 days?
A Adherent clot on ulcer
B Clean-based ulcer
C Gastric ulcer with arteriovenous malformations
D Visible bleeding vessel
E Visible nonbleeding vessel
ANSWER:B
1455 Which of the following statements about alcoholic liver disease is not true?
A
Pathologically, alcoholic cirrhosis is often characterized by diffuse fine scarring with small
regenerative nodules.
B The ratio of AST to ALT is often higher than 2.
C Serum aspartate aminotransferase levels are often greater than 1000 U/L.
D Concomitant hepatitis C significantly accelerates the development of alcoholic cirrhosis.
E
Serum prothrombin times may be prolonged, but activated partial thromboplastin times are
usually not affected.
ANSWER:C
1456
In the case vignette presented above, which of the following factors at presentation predicts a
poor outcome and increased risk of death in acute pancreatitis?
A Body mass index (BMI) >25 kg/m2
B Hematocrit ≥40%
C Lipase >1000 IU/L
D PaO2 <60 mmHg
E WBC count >10,000/µL
ANSWER:D

1457
A 22-year old woman presents to the emergency de- partment with abdominal pain and
malaise. Her symp- toms began about 8 h prior to presentation, and she has no diarrhea. The
pain is mostly in the right flank cur- rently but began in the periumbilical area. She has
nausea and vomiting. Temperature is 100.3°C, blood pressure 129/90 mmHg, heart rate 101
beats/min. Physical exami- nation shows only mild diffuse abdominal tenderness. The
abdomen is soft and bowel sounds are diminished. She is tender in the right flank without
costovertebral an- gle tenderness. The genitourinary and pelvic examina- tions are normal.
White blood cell count is 10,000/µL. Urine analysis shows 2 white blood cells per high pow-
ered field, no epithelial cells, and 1 red blood cell per high powered field. A serum
pregnancy test is negative. She has no past medical history and has never had similar symp-
toms. She is not sexually active. Which of the following is the most likely diagnosis?
A Abdominal aortic aneurysm rupture
B Acute appendicitis
C Pyelonephritis
D Mesenteric lymphadenitis
E Pelvic inflammatory disease
ANSWER:B
1458
A 28-year-old male with HIV and a CD4 count of 4/µL is admitted to the hospital with
several days of epigas- tric boring abdominal pain radiating to the back with asso- ciated
nausea and bilious vomiting. He has a history of disseminated mycobacterial disease,
cryptococcal pneumonia, and injection drug use. His current medica- tions include
fluconazole, trimethoprim-sulfamethoxazole, clarithromycin, ethambutol, and rifabutin. On
physical ex- amination he has normal vital signs, decreased bowel sounds, and tender
epigastrium without rebound or guard- ing. Rectal exam is guaiac-negative. The remainder
of the examination is normal. Amylase and lipase are elevated. The patient is treated
conservatively with intravenous fluids and bowel rest, with resolution of symptoms. Right
upper quadrant ultrasound is normal, and calcium and triglycer- ides are normal. Which of
the following changes to his medical regimen should be recommended on discharge?
A Discontinue rifabutin.
B Substitute azithromycin for clarithromycin.

C Substitute dapsone for trimethoprim-sulfamethoxa- zole.
D Substitute amphotericin for fluconazole.
E Discontinue trimethoprim-sulfamethoxazole.
ANSWER:C
1459
All of the following necessitate sending bacterial stool cultures in patients with diarrhea for 2
days severe enough to keep them home from work except
A age >75
B bloody stools
C dehydration
D recent lung transplantation
E temperature >38.5°C
ANSWER:C
1460
While doing rounds in the intensive care unit, you see a 70-year-old male patient with
multisystem or- gan failure who is postoperative day 3. Review of his history reveals that he
had a perforated appendix due to a delay in the diagnosis of acute appendicitis. Prior to his
surgical intervention, he was noted to be delirious. His preoperative laboratory results
showed: sodium, 133 meq/dL, potassium, 5.2 meq/dL, chloride, 98 meq/ dL, bicarbonate,
14 meq/dL, blood urea nitrogen 85 mg/dL, creatinine, 3.2 mg/dL. Urine analysis had no
red cells, white cells, and trace protein. An electrocar- diogram showed ST-segment
depression in an area of an old myocardial infarct. Preoperative troponin I level was 0.09
mg/dL. He had no history of chronic renal in- sufficiency. What is the most likely etiology of
this pa- tient’s renal failure?
A Acute interstitial nephritis
B Congestive heart failure
C Glomerulonephritis
D Ureteral injury
E Volume depletion
ANSWER:E
1461 All the following are causes of diarrhea except
A diabetes
B hypercalcemia
C hyperthyroidism

D irritable bowel syndrome
E metoclopramide
ANSWER:B
1462
A 55-year-old white male with a history of dia- betes presents to your office with complaints
of gen- eralized weakness, weight loss, nonspecific diffuse abdominal pain, and erectile
dysfunction. The examina- tion is significant for hepatomegaly without tenderness, testicular
atrophy, and gynecomastia. Skin examination shows a diffuse slate-gray hue slightly more
pronounced on the face and neck. Joint examination shows mild swelling of the second and
third metacarpophalangeal joints on the right hand. What is the recommended test for
diagnosis?
A Serum ferritin
B Serum iron studies, including transferrin saturation
C Urinary iron quantification in 24-h collection
D Genetic screen for HFE gene mutation (C282Y and H63D)
E Liver biopsy
ANSWER:D
1463 All the following are associated with an increased risk for cholelithiasis except
A chronic hemolytic anemia
B obesity
C high-protein diet
D pregnancy
E female sex
ANSWER:C

1464
A 28-year-old man is admitted to the hospital with a large perianal abscess. He is taken to the
operating room for incision and drainage, which he tolerates well, and he is discharged home
with a 2-week course of antibiotics. He returns to the hospital 2 months later for a rash on his
shins. On examination, he has discrete red swollen nodules on both of his shins without
fluctuance. They measure ~2 cm in diameter. He has no respiratory complaints, and the rest
of his skin examination is normal. Laboratory data show a white blood cell count of 12,000
with a normal dif- ferential. Erythrocyte sedimentation rate is 64 mm/h. A chest radiograph
is normal. Thyroid-stimulating hormone is 3.27 mU/L, and a glycosylated hemoglobin is
5.3%. Which of the following conditions is he also likely to have?
A Giant cell arteritis
B Pneumocystis jirovecii pneumonia
C Sarcoidosis
D Type 1 diabetes
E Uveitis
ANSWER:E
1465
A 55-year-old male with cirrhosis is seen in the clinic to follow up a recent hospitalization
for spontane- ous bacterial peritonitis. He is doing well and finishing his course of antibiotics.
He is taking propranolol and lactulose; besides complications of end-stage liver disease, he
has well-controlled diabetes mellitus and had a basal cell carcinoma resected 5 years ago. The
cirrhosis is thought to be due to alcohol abuse, and his last drink of alcohol was 2 weeks ago.
He and his wife ask if he is a liver transplant candidate. He can be counseled in which of the
following ways?
A He is not a transplant candidate as he has a history of alcohol dependence.
B
He is not a transplant candidate now, but may be after a sustained period of proven
abstinence from alcohol.
C Because he has diabetes mellitus he is not a trans- plant candidate.
D Because he had a skin cancer he is not a transplant candidate.
E He is appropriate for liver transplantation and should be referred immediately.
ANSWER:B

1466
A 16-year-old woman had visited your clinic 1 month ago with jaundice, vomiting, malaise,
and anorexia. Two other family members were ill with similar symptoms. Based on viral
serologies, including a positive anti-hepatitis A virus (HAV) IgM, a diagnosis of hepatitis A
was made. The patient was treated conservatively, and 1 week after first presenting, she
appeared to have made a full recovery. She returns to your clinic today complaining of the
same symptoms she had 1 month ago. She is jaundiced, and an initial panel of laboratory
tests returns elevated transami- nases. Which of the following offers the best explanation of
what has occurred in this patient?
A Co-infection with hepatitis C
B Hepatitis A recurrence
C Inappropriate treatment of initial infection
D Incorrect initial diagnosis; this patient likely has hepatitis B
E Relapsing hepatitis
ANSWER:E
1467
A male patient with inflammatory bowel disease (IBD) comes to your office as a new patient.
Reviewing the medical records, you note that he has had primarily rectal disease.
Macroscopic photographs from his most recent colonoscopy show a lumpy, bumpy,
hemorrhagic mucosa with ulcerations. Histology shows a process that is limited to the
mucosa, with the deep layers unaffected. There are crypt abscesses. Which historic feature
would be surpris- ing in a patient with this form of IBD?
A Age 15–30
B Current smoker
C Fraternal twin sister does not have IBD
D Identical twin brother does not have IBD
E Intact appendix
ANSWER:B

1468
A 26-year-old male presents with persistent peri- anal pain for 2 months that is worse with
defecation. The patient notes that he occasionally sees small amounts of red blood on the
toilet tissue. He never has had blood staining the toilet bowl. He reports persistent
constipation but has not had any incontinence. He denies anal trauma. On physical
examination there is a linear ulceration with raised edges with a skin tag at the distal end.
Circular fi- bers of the hypertrophied internal sphincter are visible. What is the most
appropriate treatment of this disease?
A Sitz baths
B Placement of a mechanical loop followed by surgical resection
C Steroid enemas
D Nitroglycerin ointment
E Mesalamine enemas
ANSWER:D
1469
A 76-year-old man complains of frequent small stools that are not abnormally liquid or hard.
There is some pain with passing the stool. He has no abdominal pain, nausea, melena,
vomiting, or fever. He has approximately eight to ten bowel movements per day, which
interferes with his quality of life, though there is no fecal incontinence. What is a possible
diagnosis to explain his complaints?
A Hypothyroidism
B Neuromuscular disorder
C Proctitis
D Ulcerative colitis
E Viral gastroenteritis
ANSWER:C
1470 Which of the following proteins does not cause se- cretion of gastric acid?
A Acetylcholine
B Caffeine
C Gastrin
D Histamine
E Somatostatin
ANSWER:E

1471
A 62-year-old female has a 3-month history of dif- fuse crampy abdominal pain and watery
diarrhea and has lost 14 lb over this period. There is no prior history of ab- dominal or
gynecologic disease. She is on no regular medi- cations, is a nonsmoker, and does not
consume alcohol. Colonoscopy reveals normal colonic mucosa. Biopsies of the colon reveal
inflammation with extensive subepithelial col- lagen deposition and lymphocytic infiltration
of the epithe- lium. Which of the following is the most likely diagnosis?
A Collagenous colitis
B Crohn’s disease
C Ischemic colitis
D Lymphocytic colitis
E Ulcerative colitis
ANSWER:A
1472
A 29-year-old woman who recently immigrated to the United States from South America
presents to a local emergency room with severe abdominal pain, jaundice, and fever. No one
else at home is ill. She is un- sure how long her symptoms have been going on, but describes
a sudden worsening over the past 3 days. She has been unable to get out of bed and has not
been eat- ing well over that period of time. She has had nausea and vomiting. She denies
alcohol or illicit drug use. She is rapidly triaged and on initial laboratory studies is found to
have an ALT and AST in the thousands. She is to be admitted for inpatient management, and
viral hepatitis serologies are sent. In a patient with acute hep- atitis B, which of the following
would be the first indica- tion of infection?
A Anti-HBc (antibody to hepatitis B core antigen)
B Clinical symptoms such as fever, jaundice, and ab- dominal pain
C HBeAg (hepatitis B e antigen)
D HBsAg (hepatitis B surface antigen)
E Increased transaminases
ANSWER:D
1473
The patient described above has the following lab- oratory results: HBsAg is positive, Anti-
HBc IgM is posi- tive, and HBeAg is positive. All other serologies are negative. She is
diagnosed with acute hepatitis B. When interpreting hepatitis B serology results, the term
“window period” refers to the time between which of the following?

A Anti-HBs and anti-HBc positivity
B Clinical symptoms and anti-HBs
C HBsAg and anti-HBs positivity
D HBsAg and HBeAg positivity
E Increased transaminases and HBsAg
ANSWER:C
1474
A 57-year-old man with peptic ulcer disease experi- ences transient improvement with
Helicobacter pylori eradi- cation. However, 3 months later, symptoms recur despite acid-
suppressing therapy. He does not take nonsteroidal anti-inflammatory agents. Stool analysis
for H. pylori anti- gen is negative. Upper GI endoscopy reveals prominent gas- tric folds
together with the persistent ulceration in the duodenal bulb previously detected and the
beginning of a new ulceration 4 cm proximal to the initial ulcer. Fasting gastrin levels are
elevated and basal acid secretion is 15 meq/h. What is the best test to perform to make the
diagnosis?
A No additional testing is necessary.
B Blood sampling for gastrin levels following a meal.
C Blood sampling for gastrin levels following secretin administration.
D Endoscopic ultrasonography of the pancreas.
E Genetic testing for mutations in the MEN1 gene.
ANSWER:C
1475
A 29-year-old woman comes to see you in clinic be- cause of abdominal discomfort. She
feels abdominal discom- fort on most days of the week, and the pain varies in location and
intensity. She notes constipation as well as diarrhea, but diarrhea predominates. In
comparison to 6 months ago, she has more bloating and flatulence than she has had before.
She identifies eating and stress as aggravating factors, and her pain is relieved by defecation.
You suspect irritable bowel syndrome (IBS). Laboratory data include: white blood cell
(WBC) count 8000/µL, hematocrit, 32%, platelets, 210,000/µL, and erythrocyte
sedimentation rate (ESR) of 44 mm/h. Stool studies show the presence of lactoferrin but no
blood. Which intervention is appropriate at this time?
A Antidepressants
B Ciprofloxacin
C Colonoscopy

D Reassurance and patient counseling
E Stool bulking agents
ANSWER:C
1476
After a careful history and physical and a cost-effective workup, you have diagnosed your
patient with IBS. What other condition would you expect to find in this patient?
A Abnormal brain anatomy
B Autoimmune disease
C History of sexually transmitted diseases
D Hypersensitivity to peripheral stimuli
E Psychiatric diagnosis
ANSWER:E
1477 Which of the following statements about cardiac cirrhosis is true?
A
Prolonged passive congestion from right-sided heart failure results first in congestion and
necrosis of portal triads, resulting in subsequent fibrosis.
B
AST and ALT levels may mimic the very high levels seen in acute hepatitis infection or
acetaminophen toxicity.
C Budd-Chiari syndrome cannot be distinguished clinically from cardiac cirrhosis.
D
Venoocclusive disease is a major cause of morbidity and mortality in patients undergoing
liver trans- plantation.
E
Echocardiography is the gold standard for diagnos- ing constrictive pericarditis as a cause of
cirrhosis.
ANSWER:B
1478
A patient with known peptic ulcer disease presents with sudden abdominal pain to the
emergency depart- ment. She is thought to have peritonitis but refuses an ab- dominal
examination due to the discomfort caused by previous examinations. Which of the following
maneu- vers will provide reasonably specific evidence of peritoni- tis without manual
palpation of the abdomen?
A Bowel sounds are absent on auscultation.
B Forced cough elicits abdominal pain.
C Hyperactive bowel sounds are heard on auscultation.
D Pain is elicited with gentle pressure at the costover- tebral angle.
E Rectal examination reveals heme-positive stools.

ANSWER:B
1479
In chronic hepatitis B virus (HBV) infection, pres- ence of hepatitis B e antigen (HBeAg)
signifies which of the following?
A Development of liver fibrosis leading to cirrhosis
B Dominant viral population is less virulent and less transmissible
C Increased likelihood of an acute flare in the next 1–2 weeks
D Ongoing viral replication
E Resolving infection
ANSWER:D
1480
A 42-year-old male presents for evaluation of re- current sharp substernal chest pain that
occurs primarily at rest and radiates to both arms and the sides of the chest. He notes that the
pain is worse with eating and emotional stress. The pain lasts approximately 10 min be- fore
resolving entirely. He has undergone a full cardiac evaluation, including negative exercise
echocardiography for inducible ischemia. You suspect diffuse esophageal spasm and order a
barium swallow for further evaluation. Which of the following findings would best correlate
with your suspected diagnosis?
A
Proximal esophageal dilatation with tapered beak- like appearance distally near the
gastroesophageal junction
B
Uncoordinated distal esophageal contractions re- sulting in a corkscrew appearance of the
esophagus
C
Dilation of the esophagus with loss of peristaltic contractions in the middle and distal
portions of the esophagus
D Reflux of barium back into the distal portion of the esophagus
E A tapered narrowing in the distal esophagus with an apple core–like lesion
ANSWER:B
1481
A 26-year-old woman presents to your clinic and is interested in getting pregnant. She seeks
your advice re- garding vaccines she should obtain, and in particular asks about the hepatitis
B vaccine. She works as a receptionist for a local business, denies alcohol or illicit drug use,
and is in a monogamous relationship. Which of the following is true regarding hepatitis B
vaccination?
A Hepatitis B vaccine consists of two intramuscular doses 1 month apart.
B Only patients with defined risk factors need be vac- cinated.

C Pregnancy is not a contraindication to the hepatitis B vaccine.
D This patient’s hepatitis serologies should be checked prior to vaccination.
E Vaccination should not be administered to children under 2 years old.
ANSWER:C
1482
A 41-year-old female presents to your clinic with a week of jaundice. She notes pruritus,
icterus, and dark urine. She denies fever, abdominal pain, or weight loss. The examination is
unremarkable except for yellow dis- coloration of the skin. Total bilirubin is 6.0 mg/dL, and
direct bilirubin is 5.1 mg/dL. AST is 84 U/L, and ALT is 92 U/L. Alkaline phosphatase is
662 U/L. CT scan of the ab- domen is unremarkable. Right upper quadrant ultra- sound
shows a normal gallbladder but does not visualize the common bile duct. What is the most
appropriate next management step?
A Antibiotics and observation
B Endoscopic retrograde cholangiopancreatography (ERCP)
C Hepatitis serologies
D HIDA scan
E Serologies for antimitochondrial antibodies
ANSWER:B
1483
A 46-year-old woman with a past medical history of osteoporosis presents to the hospital
because of he- matemesis. She reports having bright-red bloody emesis for 2 h as well as
seeing “coffee-grounds” in her emesis. However, you do not witness any vomiting in the
emer- gency department. She takes calcium, vitamin D, and alendronate. Blood pressure is
108/60 mmHg, heart rate 93 beats/min, and temperature 37.6°C. Her hematocrit is 30%
(baseline 37%). You request an emergent upper en- doscopy and resuscitate the patient with
fluids. What is the role for immediate IV proton-pump inhibitor (PPI) therapy in this patient?
A It is contraindicated given her history of osteoporosis.
B It should be initiated as this will decrease further bleeding.
C It should be initiated only if high-risk ulcers are identified at the time of endoscopy.
D
It will decrease her bleeding risk, length of hospitaliza- tion, likelihood to need surgery, and
overall mortality.
E There is no indication for immediate IV PPI therapy.
ANSWER:C

1484
While waiting for endoscopy, you recheck her he- matocrit 2 h later and it remains 30%.
Vital signs are un- changed. You perform a gastric lavage, which returns clear fluid. Test of
occult blood in the lavage is negative. What is the most appropriate intervention at this time?
A Perform a CT scan of the abdomen.
B Continue current management and plan.
C Perform another gastric lavage.
D Recheck another hematocrit in 2 h.
E Request psychiatric consultation for factitious bleed- ing.
ANSWER:B
1485
A 34-year-old male reports “yellow eyes” for the last 2 days during a routine employment
examination. He states that since his early twenties he has had similar epi- sodes of yellow
eyes lasting 2 to 4 days. He denies nausea, abdominal pain, dark urine, light-colored stools,
pruri- tus, or weight loss. He has not sought prior medical at- tention because of finances,
lack of symptoms, and the predictable resolution of the yellow eyes. He takes a mul-
tivitamin and some herbal medications. On examination he is mildly obese. He is icteric.
There are no stigmata of chronic liver disease. The patient’s abdomen is soft and nontender,
and there is no organomegaly. Laboratory ex- aminations are normal except for a total
bilirubin of 3 mg/dL. Direct bilirubin is 0.2 mg/dL. AST, ALT, and alka- line phosphatase
are normal. Hematocrit, lactate dehy- drogenase (LDH), and haptoglobin are normal. Which
of the following is the most likely diagnosis?
A Crigler-Najjar syndrome type 1
B Cholelithiasis
C Dubin-Johnson syndrome
D Gilbert’s syndrome
E Medication-induced hemolysis
ANSWER:D
1486 What is the appropriate next management step for this patient?
A Genotype studies
B Peripheral blood smear
C Prednisone
D Reassurance

E Right upper quadrant ultrasound
ANSWER:D
1487
A 45-year-old male says that for the last year he occasionally has regurgitated particles from
food eaten several days earlier. His wife complains that his breath has been foul-smelling. He
has had occasional dysphagia for solid foods. The most likely diagnosis is
A gastric outlet obstruction
B scleroderma
C Achalasia
D Zenker’s diverticulum
E diabetic gastroparesis
ANSWER:D
1488 All the following cancers commonly metastasize to the liver except
A breast
B colon
C lung
D melanoma
E prostate
ANSWER:E
1489
A 38-year-old male presents to his physician with 4 to 6 months of weight loss and joint
complaints. He re- ports that his appetite is good, but he has had diarrhea with six to eight
loose, foul-smelling stools each day. He has also had migratory pain in the knees and
shoulders. Stool studies demonstrate steatorrhea. Which of the fol- lowing diagnostic tests is
most likely to be positive in this patient?
A Serum IgA antiendomysial antibodies
B Serum IgA antigliadin antibodies
C Serum PCR for Tropheryma whippelii
D Small bowel biopsy showing reduced villous height and crypt hyperplasia
E Stool Clostridium difficile toxin
ANSWER:C

1490
Inflammatory bowel disease (IBD) may be caused by exogenous factors. Gastrointestinal
flora may promote an inflammatory response or may inhibit inflammation. Probiotics have
been used to treat IBD. Which of the fol- lowing organisms has been used in the treatment of
IBD?
A Campylobacter spp.
B Clostridium difficile
C Escherichia spp.
D Lactobacillus spp.
E Shigella spp.
ANSWER:D
1491
A 61-year-old male is admitted to your service for swelling of the abdomen. You detect
ascites on clinical examination and perform a paracentesis. The results show a white blood
cell count of 300 leukocytes/µL with 35% polymorphonuclear cells. The peritoneal albumin
level is 1.2 g/dL, protein is 2.0 g/dL, and triglycerides are 320 mg/dL. Peritoneal cultures are
pending. Serum albu- min is 2.6 g/dL. Which of the following is the most likely diagnosis?
A Congestive heart failure
B Peritoneal tuberculosis
C Peritoneal carcinomatosis
D Chylous ascites
E Bacterial peritonitis
ANSWER:A
1492
A 78-year-old female nursing home resident com- plains of rectal pain and profuse watery
diarrhea for 2 days. Her nurse reports 2 weeks of constipation prior to this. A physician sent
a Clostridium difficile stool antigen test that returned negative. What is the next step in estab-
lishing a diagnosis?
A Colonoscopy
B Digital rectal examination
C Repeat C. difficile stool antigen test
D Rotavirus stool antigen
E Stool culture
ANSWER:B

1493 Which of the following is the most common cause of acute pancreatitis in the United States?
A Alcohol
B Drugs
C Gallstones
D Hypercalcemia
E Hyperlipidemia
ANSWER:C
1494
A 24-year-old woman with a history of irritable bowel syndrome (IBS) has been treated with
loperamide, psyllium, and imipramine. Because of continued abdom- inal pain, bloating, and
alternating constipation/diarrhea, she is started on alosetron, 0.5 mg bid. Five days later she is
brought to the emergency department with severe ab- dominal pain. On examination she is
in severe discom- fort. Her temperature is 39°C, blood pressure 90/55 mmHg, heart rate 115
beats/min, respiratory rate 22 breaths/min, and oxygen saturation normal. Abdominal
examination is notable for hypoactive bowel sounds, dif- fuse tenderness, and guarding
without rebound tender- ness. Her stool is heme positive. Laboratory studies are notable for a
white blood cell count of 15,800 with a left shift and a slight anion gap metabolic acidosis.
Which of the following is the most likely diagnosis?
A Appendicitis
B Clostridium difficile colitis
C Crohn’s disease
D Ischemic colitis
E Perforated duodenal ulcer
ANSWER:D
1495
An 88-year-old woman is brought to your clinic by her family because she has become
increasingly socially withdrawn. The patient lives alone and has been reluctant to visit or be
visited by her family. Family members, including seven children, also note a foul odor in
her apartment and on her person. She has not had any weight loss. Alone in the examining
room, she only complains of hemorrhoids. On mental status ex- amination, she does have
signs of depression. Which of the following interventions is most appropriate at this time?
A Head CT scan

B Initiate treatment with an antidepressant medication
C Physical examination including genitourinary and rectal examination
D Screening for occult malignancy
E Serum thyroid-stimulating hormone
ANSWER:C
1496
You are asked to consult on a 62-year-old white female with pruritus for 4 months. She has
noted progressive fatigue and a 5-lb weight loss. She has intermittent nausea but no vomiting
and denies changes in her bowel habits. There is no history of prior alcohol use, blood
transfusions, or illicit drug use. The patient is widowed and had two heterosexual partners in
her lifetime. Her past medical history is significant only for hypothyroidism, for which she
takes levothyroxine. Her family history is unremark- able. On examination she is mildly
icteric. She has spider angiomata on her torso. You palpate a nodular liver edge 2 cm below
the right costal margin. The re- mainder of the examination is unremarkable. A right upper
quadrant ultrasound confirms your suspicion of cirrhosis. You order a complete blood
count and a comprehensive metabolic panel. What is the most ap- propriate next test?
A 24-h urine copper
B Antimitochondrial antibodies (AMA)
C Endoscopic retrograde cholangiopancreatography (ERCP)
D Hepatitis B serologies
E Serum ferritin
ANSWER:B
1497
Your 33-year-old patient with Crohn’s disease (CD) has had a disappointing disease response
to gluco- corticoids and 5-ASA agents. He is interested in steroid- sparing agents. He has no
liver or renal disease. You pre- scribe once-weekly methotrexate injections. In addition to
monitoring hepatic function and complete blood count, what other complication of
methotrexate therapy do you advise the patient of?
A Disseminated histoplasmosis
B Lymphoma
C Pancreatitis
D Pneumonitis
E Primary sclerosing cholangitis

ANSWER:D
1498
An 18-year-old man presents to a rural clinic with nausea, vomiting, anorexia, abdominal
discomfort, myal- gias, and jaundice. He describes occasional alcohol use and is sexually
active. He describes using heroin and co- caine “a few times in the past.” He works as a short-
order cook in a local restaurant. He has lost 15.5 kg (34 lb) since his last visit to clinic and
appears emaciated and illappearing. On examination he is noted to have icteric sclerae and a
palpable, tender liver below the right costal margin. In regard to acute hepatitis, which of the
follow- ing is true?
A A distinction between viral etiologies cannot be made using clinical criteria alone.
B Based on age and risk factors, he is likely to have hepatitis B infection.
C He does not have hepatitis E virus, as this infects only pregnant women.
D This patient cannot have hepatitis C because his presentation is too acute.
E This patient does not have hepatitis A because his presentation is too fulminant.
ANSWER:A
1499
A 22-year-old pregnant woman presents to the emergency department with abdominal pain
and malaise. Her symptoms began about 8 h prior to presentation and she has no diarrhea.
Her pain is mostly in the right flank currently but began in the periumbilical area. She has
nausea and vomiting. She has had an uncomplicated pregnancy and she is at 24 weeks’
gestation. She receives regular obstetric care, and her last examination, including an echo,
was normal 1 week ago. Temperature is 100.3°C, blood pressure 129/90 mmHg, and heart
rate 105 beats/ min. Physical examination shows only mild abdominal tenderness. The
abdomen is soft and bowel sounds are di- minished. She is tender in the right lower quadrant
with- out costovertebral angle tenderness. The genitourinary examination is normal, and she
has a closed os. Fetal monitoring shows a normal fetal heart rate. White blood cell count is
10,000/µL. Urine analysis shows 2 white blood cells per high powered field, no epithelial
cells, and 1 red blood cell per high powered field. What is the most likely diagnosis?
A Acute appendicitis
B Fitz-Hugh–Curtis syndrome
C Mittelschmerz
D Nephrolithiasis
E Pyelonephritis

ANSWER:A
1500
A 54-year-old male presents with 1 month of diar- rhea. He states that he has 8 to 10 loose
bowel movements a day. He has lost 8 lb during this time. Vital signs and physical
examination are normal. Serum laboratory stud- ies are normal. A 24-h stool collection
reveals 500 g of stool with a measured stool osmolality of 200 mosmol/L and a calculated
stool osmolality of 210 mosmol/L. Based on these findings, what is the most likely cause of
this pa- tient’s diarrhea?
A Celiac sprue
B Chronic pancreatitis
C Lactase deficiency
D Vasoactive intestinal peptide tumor
E Whipple’s disease
ANSWER:D
1501 All the following are risk factors for developing cholangiocarcinoma except
A choledochal cyst
B cholelithiasis
C liver flukes
D sclerosing cholangitis
E working in the rubber industry
ANSWER:B
1502
A 34-year-old female presents to your clinic with 5 weeks of right upper quadrant pain. She
denies nausea, changes in bowel habits, or weight loss. Her past medical history is
unremarkable. Her only medications are a mul- tivitamin and oral contraceptives. The
examination is no- table for a palpable liver mass 2 cm below the right costal margin. Serum
α fetoprotein is normal. An abdominal CT scan shows two 3-cm hypervascular lesions in the
right hepatic lobe that are suggestive of hepatocellular ad- enoma. What is the most
appropriate next management step?
A Observation
B Discontinuation of oral contraceptives
C Referral for surgical excision
D Radiofrequency ablation (RFA)
E CT-guided biopsy

ANSWER:B
1503
A 50-year-old male without a significant past medical history or recent exposure to alcohol
presents with midepigastric abdominal pain, nausea, and vomit- ing. The physical
examination is remarkable for the ab- sence of jaundice and any other specific physical
findings. Which of the following is the best strategy for screening for acute pancreatitis?
A Measurement of serum amylase
B Measurement of serum lipase
C Measurement of both serum amylase and serum lipase
D Isoamylase level analysis
E Magnetic resonance imaging
ANSWER:C
1504
A 43-year-old man with alcohol dependence pre- sents with a sharp epigastric pain radiating
to the back. He also has had nausea with bilious emesis on three occa- sions in the past 24 h.
He has had no bright red blood or coffee-ground material in his vomitus, nor has he had
melena. His last alcohol intake was yesterday, and he nor- mally drinks a gallon of whiskey
on a daily basis. He has a history of acute pancreatitis due to alcohol. On physical
examination, he appears uncomfortable, writhing in bed. His vital signs are: heart rate 112
beats/min, blood pres- sure 156/92 mmHg, temperature 37.8°C, respiratory rate 24
breaths/min, and SaO2 96% on room air. The abdomi- nal examination reveals decreased
bowel sounds and is tympanitic to percussion. There is diffuse tenderness to palpation in the
midepigastrium without rebound. Vol- untary guarding is present. The liver span is 15 cm to
per- cussion, and a smooth liver edge is palpated 5 cm below the right costal margin. No
spleen tip is palpable. The amylase is 580 U/L, and lipase is 690 U/L. Liver function testing
reveals an AST of 280 U/L, ALT 184 U/L, alkaline phosphatase 89 U/L, and albumin 2.6
g/dL. Fecal occult blood testing is negative. Which of the following best re- flects the current
recommendations on treatment of acute pancreatitis in this patient?
A
A nasogastric tube with intermittent suctioning is necessary to prevent ongoing stimulation of
pancre- atic enzyme release by gastric secretions.
B Early oral alimentation decreases the risk of infec- tion and speeds recovery
C
Placement of a nasojejunal feeding tube will allow early institution of oral feeding and
reduce hospital length of stay.

D
Total parenteral nutrition is indicated because the patient has evidence of chronic
malnutrition and is expected to be unable to tolerate oral alimentation for >1 week.
E
Treatment with analgesia, IV fluid resuscitation, and avoidance of oral feeding will result in
improvement in 3–7 days.
ANSWER:E
1505
A 38-year-old male is seen in the urgent care cen- ter with several hours of severe abdominal
pain. His symptoms began suddenly, but he reports several months of pain in the epigastrium
after eating, with a resultant 10-lb weight loss. He takes no medications besides over- the-
counter antacids and has no other medical problems or habits. On physical examination
temperature is 38.0°C (100.4°F), pulse 130/min, respiratory rate 24/min, and blood pressure
110/50 mmHg. His abdomen has absent bowel sounds and is rigid with involuntary guarding
dif- fusely. A plain film of the abdomen is obtained and shows free air under the diaphragm.
Which of the following is most likely to be found in the operating room?
A Necrotic bowel
B Necrotic pancreas
C Perforated duodenal ulcer
D Perforated gallbladder
E Perforated gastric ulcer
ANSWER:C
1506 Which of the following is the source of this pa- tient’s peritonitis?
A Blood
B Bile
C Foreign body
D Gastric contents
E Pancreatic enzymes
ANSWER:D
1507
Which of the following extraintestinal manifesta- tions of inflammatory bowel disease
typically worsens with exacerbations of disease activity?
A Ankylosing spondylitis
B Arthritis
C Nephrolithiasis

D Primary sclerosing cholangitis
E Uveitis
ANSWER:B
1508
A 62-year-old male is evaluated in the emergency department for a complaint of vomiting
and inability to tolerate oral intake. These symptoms have gradually pro- gressed from
occasional episodes of emesis after meals to an extent where the patient has not been able to
tolerate solid foods for the last week. He notes no significant sen- sation of nausea before the
emesis. Instead, the patient describes vomiting partially digested foods within a half hour of
eating. The patient notes no abdominal pain. He has experienced an unintentional 30-lb
weight loss over 6 months. The patient has a history of diabetes mellitus that is poorly
controlled, with a glycosylated hemoglobin level of 8.9%. The patient underwent partial
gastrectomy for peptic ulcer disease at age 52. His only medication is insulin therapy. On
physical examination the patient is cachectic with a body mass index (BMI) of 17. He has
temporal wasting. The abdominal examination reveals no masses and is nontender. The
bowel sounds are normoac- tive, and the patient’s stool is hemoccult-negative. An ab-
dominal film shows an enlarged gastric bubble with decompressed small intestinal loops.
What is the most likely diagnosis?
A Small bowel obstruction
B Gastroparesis
C Esophageal stricture
D Gastric outlet obstruction
E Cholelithiasis
ANSWER:D
1509
A 25-year-old female with cystic fibrosis is diag- nosed with chronic pancreatitis. She is at
risk for all of the following complications except
A vitamin B12 deficiency
B vitamin A deficiency
C pancreatic carcinoma
D niacin deficiency
E steatorrhea
ANSWER:D
1510 All of the following statements regarding fat mal- absorption are true except

A 90% of pancreatic exocrine function must be lost before malabsorption ensues.
B
Celiac disease is a commonly overlooked cause of nonspecific, gastrointestinal symptoms and
fat mal- absorption.
C Nutritional deficiencies are uncommon.
D Steatorrhea is formally established with >7 g of fat in stool over 24 h.
E Symptoms include greasy, foul-smelling stools that are difficult to flush.
ANSWER:C
1511
A 52-year-old male with chronic hepatitis C pre- sents to your clinic with worsening right
upper quadrant pain. Examination shows a palpable right upper quadrant mass. CT scan
shows a large 5 × 5 cm mass in the right lobe of the liver. Serum α fetoprotein is elevated. A
CT- guided liver biopsy confirms the suspected diagnosis of hepatocellular carcinoma. All
the following are appropri- ate management steps except
A referral for surgical resection
B referral for radiofrequency ablation
C referral for liver transplantation
D systemic chemotherapy
E chemoembolization
ANSWER:D
1512 What is the most common cause of chronic secre- tory diarrhea in the United States?
A Carcinoid tumor
B Crohn’s disease with ileitis
C Lactose intolerance
D Lymphocytic colitis
E Medications
ANSWER:E

1513
A 26-year-old female presents to the emergency room after ingesting “lots of pills.” Her
boyfriend discov- ered her crying on the floor of their bedroom, found numerous open
bottles of acetaminophen scattered throughout the apartment, and called 911. He does not
know when she first took the pills but had last seen her 4 h before finding her on the floor.
She is nauseated and vomits once in the emergency room. Vital signs are stable. On
examination she is alert and oriented. She has some epigastric tenderness to deep palpation.
Otherwise the examination is unremarkable. Her acetaminophen level is 400 µg/mL. Liver
function tests are normal. Which of the following statements regarding her clinical condition
is not true?
A N-acetylcysteine is the treatment of choice for aceta- minophen toxicity.
B Alkalinization of the urine is not effective as a treat- ment for acetaminophen toxicity.
C
The patient should be admitted and observed for 48 to 72 h as her hepatic injury may
manifest days after the initial ingestion.
D
Liver transplantation is the only option for patients who develop fulminant hepatic failure
from aceta- minophen.
E Normal liver function tests at presentation make significant liver injury unlikely.
ANSWER:E
1514
A 37-year-old woman presents with abdominal pain, anorexia, and fever of 4 days’
duration. The ab- dominal pain is mostly in the left lower quadrant. Her past medical history
is significant for irritable bowel syndrome, diverticulitis treated 6 months ago, and status post-
appendectomy. Since her last bout of di- verticulitis she has increased her fiber intake and
avoids nuts and popcorn. Review of systems is positive for weight loss, daily chills and
sweats, and “bubbles” in her urinary stream. Her temperature is 39.6°C. A limited CT scan
shows thickened colonic wall (5 mm) and inflammation with pericolic fat stranding. She is
admitted with a presumptive diagnosis of diverticuli- tis. What is the most appropriate
management for this patient?
A A trial of rifaximin and a high-fiber diet
B Bowel rest, ciprofloxacin, metronidazole, and ampi- cillin
C Examination of the urine sediment
D Measurement of 24-h urine protein
E Surgical removal of the affected colon and exploration
ANSWER:E

1515
A 69-year-old patient presents to the emergency department with hematochezia of 4 h
duration. The pa- tient is pale but alert and oriented. Blood pressure is 107/ 82 mmHg,
respiratory rate is 24 breaths/min and heart rate is 96 beats/min. The hematocrit is 24%, with
a base- line of 32%. Which of the following represents the best ap- proach for localization of
this patient’s intestinal bleeding?
A Angiography is most appropriate for this massive gastrointestinal (GI) bleed.
B Angiography is of little utility since the patient is not stable.
C Colonoscopy is better suited to localize bleeding, if it is massive.
D Colonoscopy can be diagnostic and therapeutic in this mild GI bleed.
E Immediate surgery with intraoperative localization is appropriate.
ANSWER:A
1516
Chronic active hepatitis is most reliably distin- guished from chronic persistent hepatitis by
the presence of
A extrahepatic manifestations
B hepatitis B surface antigen in the serum
C antibody to hepatitis B core antigen in the serum
D a significant titer of anti-smooth-muscle antibody
E characteristic liver histology
ANSWER:E
1517 All the following are causes of bloody diarrhea except
A Campylobacter
B Cryptosporidia
C Escherichia coli
D Entamoeba
E Shigella
ANSWER:B

1518
A 36-year-old female with AIDS and a CD4 count of 35/mm3 presents with odynophagia
and progressive dysphagia. The patient reports daily fevers and a 20-lb weight loss. The
patient has been treated with clotrima- zole troches without relief. On physical examination
the patient is cachectic with a body mass index (BMI) of 16 and a weight of 86 lb. The
patient has a temperature of 38.2°C (100.8°F). She is noted to be orthostatic by blood
pressure and pulse. Examination of the oropharynx re- veals no evidence of thrush. The
patient undergoes EGD, which reveals serpiginous ulcers in the distal esophagus without
vesicles. No yellow plaques are noted. Multiple biopsies are taken that show intranuclear and
intracyto- plasmic inclusions in large endothelial cells and fibro- blasts. What is the best
treatment for this patient’s esophagitis?
A Ganciclovir
B Thalidomide
C Glucocorticoids
D Fluconazole
E Foscarnet
ANSWER:A
1519
A 32-year-old man who recently returned from a vacation in Thailand presents with the
acute onset of jaundice, abdominal pain, and vomiting. He is able to tol- erate small amounts
of food. His vital signs are normal, and an abdominal examination reveals a nontender liver
edge palpable 2 cm below the right costal margin. His transaminases are elevated in the
thousands, hepatitis B surface (anti-HBs) antigen is positive, and antibody to hepatitis B
surface antigen is negative. He has no previous medical history and abstains from alcohol
use. He has never received a hepatitis B vaccine series. Which of the following do you
recommend as first-line management?
A Conservative management and close follow-up
B Hepatitis B vaccine series
C Hospital admission and initiation of a liver trans- plant workup
D Immediate entecavir treatment until anti-HBs is positive
E Immediate lamivudine treatment for a planned 6- month course
ANSWER:A

1520
A 48-year-old male seeks evaluation for diarrhea and malabsorptive symptoms.
Approximately 5 years ago the patient underwent partial gastrectomy with gastroje-
junostomy for a perforated duodenal ulcer. He had done well since that time until 5 months
ago, when he devel- oped abdominal pain and bloating after eating. In addi- tion, the patient
has had profound diarrhea that occurs after eating and is worse after he eats fatty foods. He
notes that the diarrhea is foul-smelling and often leaves a greasy film in the toilet. On
physical examination the patient is thin with a body mass index of 19. The examination is
unremarkable. His stool is hemoccult-negative. Labora- tory studies are remarkable except
for an albumin of 3.1 g/dL. He is noted to have a hemoglobin of 9.6 mg/dL and a mean
corpuscular volume (MCV) of 106. What is the most likely diagnosis?
A Dumping syndrome
B Bile reflux gastropathy
C Afferent loop syndrome
D Postvagotomy diarrhea
E Zollinger-Ellison syndrome
ANSWER:C
1521
A 17-year-old Asian student complains of abdom- inal bloating and diarrhea, particularly
after eating ice cream and other milk products. Her parents have similar symptoms. The
patient denies any weight loss or systemic symptoms. The physical examination is normal.
Treat- ment with which of the following medications is most likely to reduce her symptoms?
A Cholestyramine
B Metoclopramide
C Omeprazole
D Viokase ®
E None of the above
ANSWER:E

1522
A 36-year-old male presents with fatigue and tea-col- ored urine for 5 days. Physical
examination reveals jaundice and tender hepatomegaly but is otherwise unremarkable.
Laboratories are remarkable for an aspartate aminotransfer- ase (AST) of 2400 U/L and an
alanine aminotransferase (ALT) of 2640 U/L. Alkaline phosphatase is 210 U/L. Total
bilirubin is 8.6 mg/dL. Which of the following is least likely to cause this clinical picture and
these laboratory abnormalities?
A Acute hepatitis A infection
B Acute hepatitis B infection
C Acute hepatitis C infection
D Acetaminophen ingestion
E Budd-Chiari syndrome
ANSWER:C
1523
A 69-year-old man with Parkinson’s disease is ad- mitted to the intensive care unit from a
long-term care fa- cility for diarrhea, fever, and hypotension. He initially developed diarrhea
2 days ago, and this morning was found to have a blood pressure of 72/44 mmHg, heart rate
of 130 beats/min, and temperature of 38.9°C. He began to receive IV fluids and was
transferred to the emergency de- partment. Upon arrival, he is lethargic and minimally re-
sponsive. He remains febrile and hypotensive with blood pressure 78/44 mmHg and heart
rate 122 beats/min after 1 L of normal saline. His abdomen is tense and distended, with
hypoactive bowel sounds. A plain radiograph of the abdomen shows “thumbprinting” or
free air, but the co- lon is dilated to 8 cm. Stool is positive for occult blood. The patient
undergoes colonoscopy, and the results are shown in Figure VIII-76 (Color Atlas). What is
the most likely diagnosis?
A Diverticulitis
B Ischemic colitis
C Pseudomembranous colitis
D Salmonella infection
E Ulcerative colitis
ANSWER:C

1524
One week after removal of a biliary mass, a patient still has an elevated total bilirubin. The
patient is recover- ing well and imaging of the hepatobiliary system shows no remaining
pathology. The conjugated bilirubin is decreas- ing but remains elevated out of proportion to
the patient’s recovery. What is the best explanation for this finding?
A Bilirubin bound to albumin
B Gilbert’s syndrome
C Hibernating hepatocytes
D Incomplete resection
E Occult hemolysis
ANSWER:A
1525 Which of the following statements regarding bi- lirubin metabolism is true?
A
Bacterial β-glucuronidases unconjugate the conju- gated bilirubin that reaches the distal
ileum.
B
Bilirubin solubilizes in the serum after conversion from biliverdin in the reticuloendothelial
system.
C Conjugated bilirubin is passively transported into the bile canalicular system.
D
Glutathione S-transferase B facilitates conjugated bilirubin’s transport into the bile
canalicular system.
E Most bilirubin that reaches the terminal ileum is re- absorbed as urobilinogen.
ANSWER:A
1526
The differential diagnosis of an isolated unconju- gated (indirect) hyperbilirubinemia is
limited. In a pa- tient with isolated unconjugated hyperbilirubinemia, which of these historic
findings would be unlikely?
A Calcium bilirubinate gallstones
B Cryoglobulinemia
C History of gout
D Spherocytosis
E Recurrent long-bone pain crises
ANSWER:B
1527 Which of the following statements regarding pan- creatic cancer is true?
A Five-year survival is ~5%.
B Most cases present with locally confined disease amenable to a surgical cure.

C Pancreatic adenocarcinomas occur most frequently in the pancreatic tail.
D The median age of diagnosis is 49 years.
E The most common tumor type is an islet cell tumor.
ANSWER:A
1528
In a patient with ascites, which of the following physical examination findings suggests a
superior vena cava obstruction instead of intrinsic hepatic cirrhosis?
A Bulging flanks
B Collateral venous flow downward toward the umbilicus
C Everted umbilicus
D Pulsatile liver
E Venous hum at the umbilicus
ANSWER:B
1529
You are managing a patient with stage IV pancreatic adenocarcinoma. The patient has been
treated with gem- citabine for 16 weeks, and a recent CT scan confirms growth of the mass in
the head of the pancreas over that time period. The patient has had biliary stents placed with-
out complication for obstructive jaundice. The patient’s weight is stable and he is able to
perform activities of daily living independently. The patient wants to know what “the next
step” is now that gemcitabine has seemed to fail. What is the most appropriate
recommendation at this time?
A Initiate treatment with 5-fluorouracil.
B Make a referral to home hospice care.
C Refer for debulking surgery.
D Refer for external beam radiation as an adjunct to chemotherapy.
E Suggest enrolling in a clinical trial.
ANSWER:E
1530
All of the following physical examination clues are helpful for differentiating jaundice
caused by hyperbiliru- binemia from other causes except
A greenish discoloration of the skin
B involvement of the nasolabial folds
C predominant involvement of palms, soles, and fore- head
D sparing of non-sun-exposed areas of the body
E sparing of the sclera

ANSWER:D
1531
When evaluating a patient with chronic ascites, a high (>1.1 g/dL) serum-ascites albumin
gradient (SAAG) is consistent with all of the following diagnoses except
A cirrhosis
B congestive heart failure
C constrictive pericarditis
D hepatic vein thrombosis
E nephrosis
ANSWER:E
1532
A 73-year-old woman with a medical history of obesity and diabetes mellitus presents to
your clinic complaining of right knee pain that has been progressive and is worse with
walking or standing. She has taken overthe-counter nonsteroidal anti-inflammatory drugs
without relief. She wants to know what is wrong with her knee and what may have caused it.
X-rays are performed and reveal cartilage loss and osteophyte formation. Which of the
following represents the most potent risk factor for the development of osteoarthritis?
A Age
B Gender
C Genetic susceptibility
D Obesity
E Previous joint injury
ANSWER:A
1533
A 42-year-old obese male presents to your office with complaints of paresthesias in the right
hand that are worst in the fourth and fifth fingers. Symptoms have been present
intermittently for the last 4 months. He has no other past medical history and takes no
medications. The examination is significant for an intact neurologic examination of the right
upper extremity but mild wasting of the intrinsic muscles on inspection of the right hand.
Laboratories show a normal white blood cell count, hemoglobin, and sedimentation rate.
Electrolytes and creatinine and liver function tests are normal except for a serum glucose of
148 mg/dL. What is the most likely etiology of this patient’s symptoms?
A Diabetes mellitus
B Cholesterol emboli
C Churg-Strauss disease

D Cervical spondylosis
E Neurogenic thoracic outlet syndrome
ANSWER:E
1534
A 54-year-old man is admitted for persistent lower abdominal and groin pain that began 7
months previously. Two months before his present admission, he required exploratory
laparoscopy for acute abdominal pain and presumed cholecystitis. This revealed necrotic
omental tissue IX-3. (Continued) and pericholecystitis necessitating omentectomy and
cholecystectomy. However, the pain continued unchanged. He currently describes it as
periumbilical and radiating into his groin and legs. It becomes worse with eating. The patient
has also had episodic severe testicular pain, bowel urgency, nausea, vomiting, and diuresis.
He has lost ~22.7 kg over the preceding 6 months. His past medical history is significant of
hypertension that has recently become difficult to control. Medications on admission include
aspirin, hydrochlorothiazide, hydromorphone, lansoprazole, metoprolol, and quinapril. On
physical examination, the patient appears comfortable. His blood pressure is 170/100 mmHg,
his heart rate is 88 beats/min, and he is afebrile. He has normal first and second heart sounds
without murmurs, and an S4 is present. There are no carotid, renal, abdominal, or femoral
bruits. His lungs are clear to auscultation. Bowel sounds are normal. Abdominal palpation
demonstrates minimal diffuse tenderness without rebound or guarding. No masses are
present, and the stool is negative for occult blood. During the examination, the patient
develops Raynaud's phenomenon in his right hand that persists for several minutes. His
neurologic examination is intact. Admission laboratory studies reveal an erythrocyte
sedimentation rate of 72 mm/h, a BUN of 17 mg/dL, and a creatinine of 0.8 mg/dL. The
patient has no proteinuria or hematuria. Tests for antinuclear antibodies, anti-double-
stranded-DNA antibodies, and antineutrophil cytoplasmic antibodies are negative. Liver
function tests are abnormal with an AST of 89 IU/L and an ALT of 112 IU/L. Hepatitis B
surface antigen and e antigen are positive. Mesenteric angiography demonstrates small
beaded aneurysms of the superior and inferior mesenteric veins. What is the most likely
diagnosis?
A Hepatocellular carcinoma
B Ischemic colitis
C Microscopic polyangiitis
D Mixed cryoglobulinemia

E Polyarteritis nodosa
ANSWER:E
1535
A 64-year-old African-American male is evaluated in the hospital for congestive heart
failure, renal failure, and polyneuropathy. Physical examination on admission was notable
for these findings and raised waxy papules in the axilla and inguinal region. Admission
laboratories showed a BUN of 90 mg/dL and a creatinine of 6.3 mg/dL. Total protein was
9.0 g/dL, with an albumin of 3.2 g/dL. Hematocrit was 24%, and white blood cell and
platelet counts were normal. Urinalysis was remarkable for 3+ proteinuria but no cellular
casts. Further evaluation included an echocardiogram with a thickened left ventricle and
preserved systolic function. Which of the following tests is most likely to diagnose the
underlying condition?
A Bone marrow biopsy
B Electromyogram (EMG) with nerve conduction studies
C Fat pad biopsy
D Right heart catheterization
E Renal ultrasound
ANSWER:A
1536
A 31-year-old woman presents to your clinic complaining of painful arthritis that is worse in
the mornings when she wakes up. She was recently evaluated by an ophthalmologist for
uveitis in her right eye. A recent laboratory report shows an erythrocyte sedimentation rate
of 48 mm/h. Which of the following will be helpful in distinguishing relapsing
polychondritis from rheumatoid arthritis (RA)?
A Arthritis associated with RA is nonerosive.
B Eye inflammation is absent in relapsing polychondritis.
C Relapsing polychondritis will not present with vasculitis.
D Relapsing polychondritis will present with high-titer rheumatoid factor.
E The arthritis of relapsing polychondritis is asymmetric.
ANSWER:E

1537
A 66-year-old woman with a history of rheumatoid arthritis and frequent pseudogout attacks
in her left knee presents with night sweats and a 2-day history of left knee pain. On physical
examination, her temperature is 38.6°C, heart rate is 110 beats/min, blood pressure is 104/78
mmHg, and oxygen saturation is 97% on room air. Her left knee is swollen, red, painful, and
warm. With 5° of flexion or extension, she develops extreme pain. She has evidence of
chronic joint deformity in her hands, knees, and spine. Peripheral white blood cell (WBC)
count is 16,700 cells/µL with 95% neutrophils. A diagnostic tap of her left knee reveals
168,300 WBCs per microliter, 99% neutrophils, and diffuse needle-shaped birefringent
crystals present. Gram stain shows rare gram-positive cocci in clusters. Management includes
all of the following except
A blood cultures
B glucocorticoids
C needle aspiration of joint fluid
D orthopedic surgery consult
E vancomycin
ANSWER:B
1538
A 58-year-old female presents complaining of right shoulder pain. She does not recall any
prior injury but notes that she feels that the shoulder has been getting progressively more
stiff over the last several months. She previously had several episodes of bursitis of the right
shoulder that were treated successfully with NSAIDs and steroid injections. The patient’s past
medical history is also significant for diabetes mellitus, for which she takes metformin and
glyburide. On physical examination, the right shoulder is not warm or red but is tender to
touch. Passive and active range of motion is limited in flexion, extension, and abduction. A
right shoulder radiogram shows osteopenia without evidence of joint erosion or osteophytes.
What is the most likely diagnosis?
A Adhesive capsulitis
B Avascular necrosis
C Bicipital tendinitis
D Osteoarthritis
E Rotator cuff tear
ANSWER:A

1539
A 44-year-old woman presents for evaluation of dry eyes and mouth. She first noticed these
symptoms >5 years ago and the symptoms have worsened over time. She describes her eyes
as gritty-feeling, as if there were sand in her eyes. Sometimes her eyes burn, and she states
that it is difficult to be outside in bright sunlight. In addition, her mouth is quite dry. In her
job, she is frequently asked to give business presentations and finds it increasingly difficult to
complete a 30- to 60-minute presentation. She usually has water with her at all times.
Although she reports good dental hygiene without any recent changes, her dentist has had to
place fillings twice in the past 3 years for dental caries. Her only other past medical history is
treated tuberculosis that she contracted while in the Peace Corp in Southeast Asia when in her
twenties. She takes no medication regularly and does not smoke. Ocular examination reveals
punctuate corneal ulcerations on Rose Bengal stain, and the Schirmer test shows <5 mm of
wetness after 5 min. Her oral mucosa is dry with thick mucous secretions, and the parotid
glands are enlarged bilaterally. Laboratory examination reveals positive antibodies to Ro and
La (SS-A and SS-B). In addition, her chemistries reveal a sodium of 142 mEq/L, potassium
2.6 mEq/L, chloride 115 mEq/L, and bicarbonate of 15 mEq/L. What is the most likely
cause of the hypokalemia and acidemia in this patient?
A Diarrhea
B Distal (type I) renal tubular acidosis
C Hypoaldosteronism
D Purging with underlying anorexia nervosa
E Renal compensation for chronic respiratory alkalosis
ANSWER:B
1540
A patient with end-stage renal disease on hemodialysis presents to your office with hand pain
and you diagnose carpal tunnel syndrome. A serum thyroid-stimulating hormone level is
normal. You also note bilateral knee effusions, IX-9. (Continued) which the patient states
have been there for many months. Suspecting an amyloid deposition disease, you perform a
fat pad biopsy. Which protein do you expect to find on immunohistochemical staining?
A β2-Microglobulin
B Fibrinogen α-chain
C Immunoglobulin light chain

D Serum amyloid A
E Transthyretin
ANSWER:A
1541
A 41-year-old female presents to your clinic with 3 weeks of weakness, lethargy. and
depressed mood. She notes increasing difficulty with climbing steps, rising from a chair, and
combing her hair. She has no difficulty buttoning her blouse or writing. The patient also
notes some dyspnea on exertion and orthopnea. She denies rash, joint aches, or constitutional
symptoms. She is on no medications, and the past medical history is otherwise
uninformative. The family history is notable only for coronary artery disease. The physical
examination is notable for an elevated jugular venous pressure, an S3, and some bibasilar
crackles. The neurologic examination shows some marked proximal muscle weakness in the
deltoids and biceps and the hip flexors. Distal muscle strength is normal. Sensory
examination and reflexes are normal. Laboratories are unremarkable except for a negative
antinuclear antibody screen and a creatinine kinase of 3200 IU/L. You suspect a diagnosis of
polymyositis. All the following clinical conditions may occur in polymyositis except
A an increased incidence of malignancy
B interstitial lung disease
C dilated cardiomyopathy
D dysphagia
E Raynaud’s phenomenon
ANSWER:A
1542
A 64-year-old man with congestive heart failure presents to the emergency room
complaining of acute onset of severe pain in his right foot. The pain began during the night
and awoke him from a deep sleep. He reports the pain to be so severe that he could not wear
a shoe or sock to the hospital. His current medications are furosemide, 40 mg twice daily,
carvedilol, 6.25 mg twice daily, candesartan, 8 mg once daily, and aspirin, 325 mg once
daily. On examination, he is febrile to 38.5°C. The first toe of the right foot is erythematous
and exquisitely tender to touch. There is significant swelling and effusion of the first
metatarsophalangeal joint on the right. No other joints are affected. Which of the following
findings would be expected on arthrocentesis?
A Glucose level of <25 mg/dL

B Positive Gram stain
C
Presence of strongly negatively birefringent needleshaped crystals under polarized light
microscopy
D
Presence of weakly positively birefringent rhomboidal crystals under polarized light
microscopy
E White blood cell (WBC) count >100,000/µL
ANSWER:C
1543 In this patient, which test should be performed next?
A Chest radiograph
B Echocardiogram
C Electrocardiogram
D Skin biopsy
E Urinalysis
ANSWER:E
1544
A 35-year-old female comes to the local health clinic because for the last 6 months she has
had recurrent urticarial lesions, which occasionally leave a residual discoloration. She also
has had arthralgias. The sedimentation rate now is 85 mm/h. The procedure most likely to
yield the correct diagnosis in this case would be
A a battery of wheal-and-flare allergy skin tests
B measurement of total serum IgE concentration
C measurement of C1 esterase inhibitor activity
D skin biopsy
E patch testing
ANSWER:D
1545
A 45-year-old obese man presents to the clinic several weeks after starting a jogging
regimen. He describes right-sided heel pain that has worsened over this time. The pain is
worse in the morning and when the patient is barefoot. On examination, pain can be elicited
with palpation of the inferior medial right heel. Which of the following is required to make a
definitive diagnosis of plantar fasciitis?
A Compatible history and provocative testing
B History and physical examination alone
C History, physical examination, and nuclear medicine bone scan

D History, physical examination, and heel ultrasound showing thickening of the fascia
E History, physical examination, and plain radiograph demonstrating heel spur
ANSWER:B
1546
Which of the following findings on joint aspiration is most likely to be associated with
calcium pyrophosphate deposition disease (pseudogout)?
A
Fluid, clear and viscous; white blood cell count, 400/ µL; crystals, rhomboidal and weakly
positively birefringent
B Fluid, cloudy and watery; white blood cell count, 8000/µL; no crystals
C
Fluid, dark brown and viscous; white blood cell count, 1200/µL; crystals, needle-like and
strongly negatively birefringent
D
Fluid, cloudy and watery; white blood cell count, 12,000/µL; crystals, needle-like and
strongly negatively birefringent
E
Fluid, cloudy and watery; white blood cell count, 4800/µL; crystals, rhomboidal and weakly
positively birefringent
ANSWER:E
1547
A 54-year-old female with rheumatoid arthritis is treated with infliximab for refractory
disease. All the following are potential side effects of this treatment except
A demyelinating disorders
B disseminated tuberculosis
C exacerbation of congestive heart failure
D pancytopenia
E pulmonary fibrosis
ANSWER:E

1548
A 26-year-old man presents with severe bilateral pain in his hands, ankles, knees, and
elbows. He is recovering from a sore throat and has had recent fevers to 38.9°C. Social
history is notable for recent unprotected receptive oral intercourse with a man ~1 week ago.
Physical examination reveals a well-developed man in moderate discomfort. He is afebrile.
His pharynx is erythematous with pustular exudates on his tonsils. He has tender anterior
cervical lymphadenopathy. His cardiac examination is notable for a normal S1 and S2 and a
soft ejection murmur. His lungs are clear. Abdomen is benign with no organomegaly. He has
no rash, and genital examination is normal. His bilateral proximal interphalangeal joints,
metacarpophalangeal joints, wrists, ankles, and metatarsophalangeal joints are red, warm, and
boggy with tenderness noted with both passive and active movement. A complete metabolic
panel and complete blood count are all within normal limits. His erythrocyte sedimentation
rate is 85 mm/h and C-reactive protein is 11 mg/dL. What is the most likely diagnosis?
A Acute HIV infection
B Acute rheumatic fever
C Lyme disease
D Neisseria gonorrhoeae infection
E Poststreptococcal reactive arthritis
ANSWER:E
1549
A 27-year-old female with SLE is in remission; current treatment consists of azathioprine 75
mg/d and prednisone 5 mg/d. Last year she had a life-threatening exacerbation of her
disease. She now strongly desires to become pregnant. Which of the following is the least
appropriate action to take?
A Advise her that the risk of spontaneous abortion is high.
B Warn her that exacerbations can occur in the first trimester and in the postpartum period.
C Tell her it is unlikely that a newborn will have lupus.
D
Advise her that fetal loss rates are higher if anticardiolipin antibodies are detected in her
serum.
E Stop the prednisone just before she attempts to become pregnant.
ANSWER:E

1550
A 48-year-old male has a long-standing history of ankylosing spondylitis. His most recent
spinal film shows straightening of the lumbar spine, loss of lordosis, and “squaring” of the
vertebral bodies. He currently is limited by pain with ambulation that is not improved with
nonsteroidal anti-inflammatory medications. Which of the following treatments has been
shown to improve symptoms the best at this stage of the illness?
A Celecoxib
B Etanercept
C Prednisone
D Sulfasalazine
E Thalidomide
ANSWER:B
1551
A 72-year-old woman presents to the emergency room for an episode of vision loss in her
right eye. The vision loss came on abruptly and is described as a curtain falling across her
visual field. She immediately called her daughter and upon arrival to the emergency room
40 min later, her vision had returned to normal. Recently she also has been experiencing dull
throbbing headaches for which she is taking acetaminophen, with limited relief. She has a
past medical history of hypercholesterolemia and coronary artery disease, undergoing
angioplasty and stenting of the right coronary artery 8 years previously. She does not smoke
currently but has a 40-pack-year history of tobacco, quitting only after her diagnosis of
coronary artery disease. On review of systems, the patient recalls pain in her scalp with
combing her hair, particularly on the right side, and occasional pain with chewing food. She
has also recently noticed stiffness and pain in her hips, making it difficult to stand from
seated position. On examination, she has 20/30 visual acuity in the left eye, and 20/100
visual acuity in the right eye. Funduscopic examination suggests anterior ischemic optic
neuropathy. There are no carotid bruits present, but palpation of the temporal artery is
painful. The neurologic examination is otherwise normal. The erythrocyte sedimentation rate
(ESR) is 102 mm/h. The hemoglobin is 7.9 g/dL, and hematocrit is 25.5%. A head CT shows
no acute ischemic event. Which of the following is the next most important step in the
management of this patient?
A Initiate treatment with indomethacin, 75 mg twice daily.
B Initiate treatment with prednisone, 60 mg daily.

C
Initiate treatment with unfractionated heparin adjusted based on activated partial
thromboplastin time to obtain full anticoagulation.
D Perform magnetic resonance angiography of the brain.
E Perform a temporal artery biopsy.
ANSWER:B
1552
A patient presents with 3 weeks of pain in the lower back. All the following are risk factors
for serious causes of spine pathology except
A age more than 50 years
B urinary incontinence
C duration of pain more than 2 weeks
D bed rest without relief
E history of intravenous drug use
ANSWER:C
1553
A 64-year-old man with coronary artery disease and atrial fibrillation is referred for
evaluation of fevers, arthralgias, pleuritis, and malar rash. The symptoms have developed
over the past 6 months. The pleuritis has responded to steroid therapy, but prednisone has
been unable to be tapered off due to recurrence of symptoms at daily steroid doses <15 mg
of prednisone. His medications include aspirin, procainamide, lovastatin, prednisone, and
carvedilol. You suspect drug-induced lupus due to procainamide. Antibodies directed against
which of the following proteins is most likely to be positive?
A Cardiolipin
B Double-strand DNA
C Histone
D Ribonucleoprotein (RNP)
E Ribosomal P
ANSWER:C

1554
A 28-year-old woman seeks evaluation from her primary care doctor for recurrent episodes
of hives and states that she is “allergic to cold weather.” She reports that for >10 years she
would develop areas of hives when exposed to cold temperatures, usually on her arms and
legs. She has never sought evaluation previously and states that over the past several years
the occurrence of the hives has become more frequent. Other than cold exposure, she can
identify no other triggers for development of hives. She has no history of asthma or atopy.
She denies food intolerance. Her only medication is oral contraceptive pills, which she has
taken for 5 years. She lives in a single-family home that was built 2 years ago. On
examination, she develops a linear wheal after being stroked along her forearm with a
tongue depressor. Upon placing her hand in cold water, her hand becomes red and swollen.
In addition, there are several areas with a wheal and flare reaction on the arm above the area
of cold exposure. What is the next step in the management of this patient?
A Assess for the presence of antithyroglobulin and antimicrosomal antibodies.
B Check C1 inhibitor levels.
C Discontinue the oral contraceptive pills.
D Treat with cetirizine, 10 mg daily.
E Treat with cyproheptadine, 8 mg daily.
ANSWER:D

1555
An 18-year-old man is admitted to the hospital with acute onset of crushing substernal chest
pain that began abruptly 30 min ago. He reports the pain radiating to his neck and right arm.
He has otherwise been in good health. He currently plays trumpet in his high school
marching band but does not participate regularly in aerobic activities. On physical
examination, he is diaphoretic and tachypneic. His blood pressure is 100/48 mmHg and heart
rate is 110 beats/min. His cardiovascular examination has a regular rhythm but is
tachycardic. A II/VI holosystolic murmur is heard best at the apex and radiates to the axilla.
His lungs have bilateral rales at the bases. The electrocardiogram demonstrates 4 mm of ST
elevation in the anterior leads. On further questioning regarding his past medical history, he
recalls having been told that he was hospitalized for some problem with his heart when he
was 2 years old. His mother, who accompanies him, reports that he received aspirin and γ
globulin as treatment. Since that time, he has required intermittent follow-up with
echocardiography. What is the most likely cause of this patient’s acute coronary syndrome?
A Dissection of the aortic root and left coronary ostia
B Presence of a myocardial bridge overlying the left anterior descending artery
C Stenosis of a coronary artery aneurysm
D Vasospasm following cocaine ingestion
E Vasculitis involving the left anterior descending artery
ANSWER:C
1556
A 29-year-old male with episodic abdominal pain and stress-induced edema of the lips, the
tongue, and occasionally the larynx is likely to have low functional or absolute levels of
which of the following proteins?
A C5A (complement cascade)
B IgE
C T cell receptor, α chain
D Cyclooxygenase
E C1 esterase inhibitor
ANSWER:E
1557 Which of the following joints are typically spared in osteoarthritis (OA)?
A Ankle
B Cervical spine

C Distal interphalangeal joint
D Hip
E Knee
ANSWER:A
1558
A 62-year-old woman is admitted to the hospital with pneumococcal bacteremia. Her past
medical history is notable for a history of pneumonia due to Haemophilus influenzae type
B2 years ago and hypertension. On review of systems, she reports easy bruising, peripheral
paresthesias, and symptoms of carpal tunnel syndrome. On physical examination, she has
ecchymoses on her face and arms. Her nails are dystrophic and she has alopecia. Her tongue
has indentations on both sides. Abdominal examination shows only hepatomegaly. She takes
no medications or supplements and has no significant family history. A complete blood
count shows a white blood cell count of 17,000/µL, hematocrit of 30%, and platelets of
300,000/µL. Differential shows 75% neutrophils, 20% lymphocytes. Serum albumin is 3.3
mg/dL, calcium 8.0 mg/dL total protein 8.2 mg/dL, AST 32 U/L, ALT 32 U/L, total
bilirubin 1.3 mg/ dL, alkaline phosphatase 120 U/L. What is the most likely etiology of the
patient’s current infection?
A Cyclical neutropenia
B Functional asplenism
C HIV infection
D Sickle cell anemia
E X-linked agammaglobulinemia
ANSWER:B
1559 Which of the following statements regarding rheumatoid arthritis is true?
A There is an association with the class II major histocompatibility complex allele HLA-B27.
B
The earliest lesion in rheumatoid arthritis is an increase in the number of synovial lining cells
with microvascular injury.
C
Females are affected three times more often than are males, and this difference is maintained
throughout life.
D
Africans and African Americans most commonly have the class II major histocompatibility
complex allele HLA-DR4.
E
Titers of rheumatoid factor are not predictive of the severity of rheumatoid arthritis or its
extraarticular manifestations.

ANSWER:B
1560 Which of the following definitions best fits the term enthesitis?
A
Alteration of joint alignment so that articulating surfaces incompletely approximate each
other
B Inflammation at the site of tendinous or ligamentous insertion into bone
C Inflammation of the periarticular membrane lining the joint capsule
D Inflammation of a saclike cavity near a joint that decreases friction
E A palpable vibratory or crackling sensation elicited with joint motion
ANSWER:B
1561
A 35-year-old female presents to her primary care doctor complaining of diffuse body and
joint pain. When asked to describe which of her joints are most affected, she answers, “All of
them.” There is no associated stiffness, redness, or swelling of the joints. No Raynaud’s
phenomenon has been appreciated. Occasionally she notes numbness in the fingers and toes.
The patient complains of chronic pain and poor sleep quality that she feels is due to her pain.
She previously was seen in the clinic for chronic headaches that were felt to be tension-
related. She has tried taking over-the-counter ibuprofen twice daily without relief of pain.
She has no other medical problems. On physical examination, the patient appears
comfortable. Her joints exhibit full range of motion without evidence of inflammatory
arthritis. She does have pain with palpation at bilateral suboccipital muscle insertions, at C5,
at the lateral epicondyle, in the upper outer quadrant of the buttock, at the medial fat pad of
the knee proximal to the joint line, and unilaterally on the second right rib. The erythrocyte
sedimentation rate is 12 s. Antinuclear antibodies are positive at a titer of 1:40 in a speckled
pattern. The patient is HLA-B27-positive. Rheumatoid factor is negative. Radiograms of the
cervical spine, hips, and elbows are normal. What is the most likely diagnosis?
A Ankylosing spondylitis
B Disseminated gonococcal infection
C Fibromyalgia
D Rheumatoid arthritis
E Systemic lupus erythematosus
ANSWER:C

1562
A 42-year-old woman comes to your clinic 1 week after her primary doctor diagnosed her
with fibromyalgia. She describes years of fatigue, chronic pain, poor sleep, and irritability
and is unable to work due to her symptoms. A review of her physical examination confirms
the presence of pain on digital palpation at 14 of 18 characteristic sites. While relieved at
finally having a diagnosis, she is concerned about what treatments are available. Which of
the following should be your first treatment step?
A Improve sleep and consider tricyclics
B Initiation of a pain diary and frequent, brief clinic visits
C Low-dose narcotics and a long-acting benzodiazepine
D Referral for psychotherapy with a psychologist
E Treatment of depression with a selective serotonin reuptake inhibitor (SSRI)
ANSWER:A
1563
A 23-year-old woman was diagnosed with systemic lupus erythematosus based upon the
presence of polyarthritis, malar rash with photosensitivity, and oral ulcerations. Antibodies to
double-stranded DNA, Smith protein, and antinuclear antibodies were present in high titers.
A urinalysis is normal. The patient is requesting treatment for the joint symptoms as she feels
they limit her activities of daily living. What is the best choice for initial therapy in this
individual?
A Hydroxychloroquine, 200–400 mg daily
B Methotrexate, 15 mg weekly
C Physical therapy only
D Prednisone, 1 mg/kg daily
E Quinacrine, 100 mg three times daily
ANSWER:A

1564
A 43-year-old male presents to your office complaining of weakness in the right hand for 2
days. He reports that he had been in excellent health until 2 months ago, when he was
diagnosed with hypertension. Since that diagnosis, he has lost 20 lb unintentionally and
complains of frequent headaches and abdominal pain that is worse after eating. He
previously was an injection drug user but now is maintained on methadone. His only
medications are hydrochlorothiazide 25 mg/d, methadone 70 mg/d, and lisinopril 5 mg/d.
On physical examination, the patient appears well developed and without distress. Blood
pressure is 148/94. He is not tachycardic. The examination is otherwise notable only for the
inability to extend the right wrist and fingers against gravity. Laboratory studies show an
erythrocyte sedimentation rate (ESR) of 88 mm/h, an aspartate aminotransferase (AST) of
154 IU/L, and an alanine aminotransferase (ALT) of 176 IU/L. Which of the following tests
is most useful in establishing a diagnosis?
A Hepatitis B surface antigen
B Hepatitis C viral load
C Anticytoplasmic neutrophil antibodies
D Mesenteric angiography
E Radial nerve biopsy
ANSWER:E
1565
A 23-year-old woman seeks evaluation for seasonal rhinitis. She reports that she develops
symptoms yearly in the spring and fall. During this time, she develops rhinitis with postnasal
drip and cough that disrupts her sleep. In addition, she will also note itchy and watery eyes.
When the symptoms occur, she takes nonprescription loratadine, 10 mg daily, with
significant improvement in her symptoms. What is the most likely allergen(s) that are causing
this patient’s symptoms?
A Grass
B Ragweed
C Trees
D A and B
E B & C
ANSWER:E

1566
A 45-year-old male has been hospitalized for several weeks in the intensive care unit for
postsurgical complications after gastrojejunal bypass surgery. He is noted to have persistent
fevers and on examination is found to have erythema, fluctuance, and tenderness over the
posterior surface of the left elbow. Initial management of this disorder should include all the
following except
A incision and drainage
B empirical antibiotics for gram-positive organisms
C aspiration of the collection for Gram stain and culture
D microscopic evaluation of aspirate for crystals
E pressure-relieving devices
ANSWER:A
1567
A 51-year-old male presents to your office complaining of lower back pain. When he exerts
himself or lifts items, he describes worsening of the pain and also pain in the left buttock that
radiates down the posterior left thigh. The patient denies pain at rest and any history of
trauma. You examine his lower back. Which examination maneuver is the most specific for
lumbar disk herniation?
A Right straight leg raise
B Left straight leg raise
C Right crossed straight leg raise
D Left crossed straight leg raise
E Reverse straight leg raise
ANSWER:C
1568
A 42-year-old woman is being treated with cyclophosphamide, 2 mg/kg daily, for
Wegener’s granulomatosis manifested as glomerulonephritis, tracheal stenosis, and cavitary
lung disease. All of the following are potential side effects of cyclophosphamide at this dose
except
A alopecia
B bone marrow suppression
C hemorrhagic cystitis
D infertility
E myelodysplasia
ANSWER:A

1569
A 25-year-old female presents with a complaint of painful mouth ulcerations. She describes
these lesions as shallow ulcers that last for 1 or 2 weeks. The ulcers have been appearing for
the last 6 months. For the last 2 days, the patient has had a painful red eye. She has had no
genital ulcerations, arthritis, skin rashes, or photosensitivity. On physical examination, the
patient appears well developed and in no distress. She has a temperature of 37.6°C (99.7°F),
heart rate of 86, blood pressure of 126/72, and respiratory rate of 16. Examination of the
oral mucosa reveals two shallow ulcers with a yellow base on the buccal mucosa. The
ophthalmologic examination is consistent with anterior uveitis. The cardiopulmonary
examination is normal. She has no arthritis, but medially on the right thigh there is a palpable
cord in the saphenous vein. Laboratory studies reveal an erythrocyte sedimentation rate of
68 s. White blood cell count is 10,230/mm3 with a differential of 68% polymorphonuclear
cells, 28% lymphocytes, and 4% monocytes. The antinuclear antibody and anti-dsDNA
antibody are negative. C3 is 89 mg/dL, and C4 is 24 mg/dL. What is the most likely
diagnosis?
A Behçet’s syndrome
B Systemic lupus erythematosus
C Discoid lupus erythematosus
D Sjögren’s syndrome
E Cicatricial pemphigoid
ANSWER:A
1570 What is the best initial treatment for this patient?
A Topical glucocorticoids including ophthalmic prednisolone
B Systemic glucocorticoids and azathioprine
C Thalidomide
D Colchicine
E Intralesional interferon α
ANSWER:B

1571
A 42-year-old man presents to your clinic complaining of left shoulder soreness that has
been bothering him for 8 months. He experiences intermittent pain that is worse at night.
Active abduction of his left arm over his head causes extreme pain. He describes his pain as a
dull ache in his shoulder. He cannot identify a specific trauma that led to his pain but notes
that he lifts weights and plays sports on a regular basis. On physical examination, he has
tenderness over the lateral aspect of the humeral head and pain with arm abduction. Which
of the following is the most likely cause of his symptoms?
A Acromioclavicular arthritis
B Bicipital tendonitis
C Inflammation of the infraspinatus tendon
D Inflammation of the supraspinatus tendon
E Subluxation of the left humeral head
ANSWER:D
1572
A 42-year-old male presents with complaints of a rash and joint pain. He first noticed the
rash 6 months ago. It is primarily on the hands (see Figure IX-49, Color Atlas), the extensor
surfaces of the elbows, and the knees, low back, and scalp. Although he complains of the
appearance of these lesions, they do not itch or hurt. The patient has not been previously
evaluated for them and has recently noticed changes in the nail beds. For the last 2 weeks,
the patient has had increasingly severe pain in the distal joints of the hands and feet. His
hands are so painful that he is having trouble writing and holding utensils. The patient denies
fevers, weight loss, fatigue, cough, shortness of breath, or changes in bowel or bladder
habits. Which of the following is the most likely diagnosis?
A Arthritis associated with inflammatory bowel disease
B Gout
C Osteoarthritis
D Psoriatic arthritis
E Rheumatoid arthritis
ANSWER:D
1573
Which of the following is the most common clinical manifestation of relapsing
polychondritis?
A Aortic regurgitation
B Arthritis of weight-bearing joints

C Auricular chondritis
D Reduced hearing
E Saddle-nose deformity
ANSWER:C
1574
A 60-year-old woman with a history of Sjögren’s syndrome diagnosed 20 years ago
presents to her primary care doctor complaining of facial swelling. Her xerostomia dry eye
symptoms have not changed. She is known to be positive for rheumatoid factor in addition
to Ro and La antibodies but is not thought to have rheumatoid arthritis. She previously had
cutaneous vasculitis requiring treatment with prednisone, but she has been off steroids for 5
years without evidence of recurrence. She is currently using artificial tears and cevimeline,
30 mg three times daily. On physical examination, her right parotid gland is enlarged. It is
not tender, but is firm and hard to touch. It is noted that the right parotid gland was similarly
enlarged on a visit 3 months ago. She denies systemic illness or any new symptoms. What is
the most likely diagnosis?
A Adenoid cystic carcinoma
B B cell lymphoma
C Impacted sialolith
D Mumps
E Recurrent vasculitis
ANSWER:B
1575
A 42-year-old Turkish man presents to his physician complaining of recurring ulcers in the
mouth and on his penis. He states that the ulcers are painful and last for about 2 weeks before
spontaneously resolving. In addition, he intermittently gets skin lesions that he describes as
painful nodules on his lower extremities. You suspect that he has Behçet’s syndrome. A
pathergy test is performed. What response would you expect after injecting 0.3 mL of sterile
saline under the skin?
A Development of 10 mm of induration with overlying erythema after 72 h
B Development of a 2- to 3-mm papule at the site of insertion in 2–3 days
C Development of granulomatous inflammation 4–6 weeks after the injection
D Development of an urticarial reaction within 15 min
E No reaction
ANSWER:B

1576
A 45-year-old African-American woman with systemic lupus erythematosus (SLE) presents
to the emergency room with complaints of headache and fatigue. Her prior manifestations of
SLE have been arthralgias, hemolytic anemia, malar rash, and mouth ulcers, and she is
known to have high titers of antibodies to doublestranded DNA. She currently is taking
prednisone, 5 mg daily, and hydroxychloroquine, 200 mg daily. On presentation, she is
found to have a blood pressure of 190/110 mmHg with a heart rate of 98 beats/min. A
urinalysis shows 25 red blood cells (RBCs) per high-power field with 2+ proteinuria. No
RBC casts are identified. Her blood urea nitrogen is 88 mg/dL, and creatinine is 2.6 mg/dL
(baseline 0.8 mg/dL). She has not previously had renal disease related to SLE and is not
taking nonsteroidal anti-inflammatory drugs. She denies any recent illness, decreased oral
intake, or diarrhea. What is the most appropriate next step in the management of this patient?
A
Initiate cyclophosphamide, 500 mg/m2 body surface area IV, and plan to repeat monthly for
3–6 months.
B Initiate hemodialysis.
C
Initiate high-dose steroid therapy (IV methylprednisolone, 1000 mg daily for 3 doses,
followed by oral prednisone, 1 mg/kg daily) and mycophenolate mofetil, 2 g daily.
D Initiate plasmapheresis.
E Withhold all therapy until renal biopsy is performed.
ANSWER:C

1577
A 52-year-old female has poorly controlled rheumatoid arthritis on prednisone 5 mg daily
and etanercept 50 mg weekly by subcutaneous injection. Despite this, she has ongoing
symptoms with severe pain in the wrists, hands, feet, and ankles. She also has destructive
arthritis causing swan-neck and boutonnière deformities in the hands as well as plantar
subluxation of the metatarsal heads that prevents ambulation. She has subcutaneous nodules
on the extensor surfaces of the arms. She presents to the emergency room complaining of
fevers and dysuria. On physical examination temperature is 39.1°C (102.3°F). Heart rate is
112, and blood pressure is 122/76. The examination is unremarkable except for right
costovertebral angle tenderness and splenomegaly. Laboratory studies at the time of
presentation reveal a white blood cell count of 2300/mm3 with 15% polymorphonuclear
cells, 75% lymphocytes, 8% monocytes, and 2% eosinophils. She is also anemic with a
hemoglobin of 9.2 mg/dL and a hematocrit of 28.7%. The mean corpuscular volume is 88
fL. The platelet count is 132,000/mm3. A peripheral blood smear shows normocytic anemia
without anisocytosis or poikilocytosis. She is found to have Escherichia coli bacteremia
related to a urinary tract infection. She is treated with ceftriaxone and does well. However,
she remains anemic and neutropenic. The patient undergoes a bone marrow biopsy that
shows hypercellularity with a lack of mature neutrophils. What is the most likely diagnosis?
A Acute myelogenous leukemia
B B cell lymphoma
C Disseminated Mycobacterium tuberculosis infection
D Felty’s syndrome
E Idiosyncratic reaction to etanercept
ANSWER:D
1578 What is the most common extraarticular manifestation of ankylosing spondylitis?
A Anterior uveitis
B Aortic regurgitation
C Cataracts
D Inflammatory bowel disease
E Third-degree heart block
ANSWER:A

1579
A 42-year-old female presents to the physician with 3 months of worsening dyspnea on
exertion, malaise, and weakness. She reports that the symptoms have worsened gradually and
are associated with low-grade fever, anorexia, and an 8-lb weight loss. She has trouble
climbing stairs because of leg weakness and shortness of breath. Recently she has noticed that
her arms tire while she is brushing her teeth or combing her hair. Her mother also
commented that the patient seems to have difficulty rising from the couch. Her writing is
normal, and she has no sensory symptoms. Physical examination is notable for a temperature
of 37.8°C (100°F), bilateral lung crackles, and diminished strength in the deltoids,
quadriceps, and psoas muscles. Laboratory studies are notable for an elevated creatine
kinase. Chest radiography shows bilateral interstitial infiltrates, and lung volumes are reduced
to 70% of the predicted values. Which of the following autoantibodies is most likely to be
present in this patient?
A Antiglomerular basement membrane antibody
B Antihistone antibody
C Anti-Jo-1 antibody
D Antimicrosomal antibody
E Antineutrophil cytoplasmic antibody (ANCA)
ANSWER:C
1580
A 43-year-old man presents to your clinic complaining of bilateral knee pain. He states that
the pain worsens with walking and is not present at rest. He has been experiencing knee pain
for many months and has had no relief from over-the-counter analgesics. He has a history of
hypertension and obesity. Which of the following represents the best initial treatment strategy
for this patient?
A Avoidance of walking for several weeks
B Light daily walking exercises
C Low-dose, long-acting narcotics
D Oral steroid pulse
E Weight loss
ANSWER:E

1581
A 53-year-old woman presents to your clinic complaining of fatigue and generalized pain
that have worsened over 2 years. She also describes irritability and poor sleep and is
concerned that she is depressed. She reveals that she was recently separated from her
husband and has been stressed at work. Which of the following elements of her presentation
meet American College of Rheumatology criteria for fibromyalgia?
A Diffuse chronic pain and abnormal sleep
B Diffuse pain without other etiology and evidence of major depression
C Major depression, life stressor, chronic pain, and female gender
D Major depression and pain on palpation at 6 of 18 tender point sites
E Widespread chronic pain and pain on palpation at 11 of 18 tender point sites
ANSWER:E
1582
A 62-year-old female complains of aching joints. She notes intermittent stiffness and pain in
the knees, hips, wrists, and hands. She also describes easy fatigability, dyspepsia, a dry
cough, and itchy red eyes and also has trouble keeping her dentures in place. There is a
history of diabetes but no other significant history. Medications include insulin and
naproxen. She has no HIV risk factors. Examination is significant for dry mucous
membranes in the oropharynx. There is no evidence of joint destruction or active
inflammation. Laboratory studies show a negative antinucleolar antibody but a positive Ro/
SSA autoantigen. What is the most likely diagnosis?
A Sarcoidosis
B Sjögren’s syndrome
C Rheumatoid arthritis
D Psychogenic illness
E Vitamin A deficiency
ANSWER:B
1583
A 46-year-old woman is referred to your clinic by her primary care physician. She describes
fatigue and diffuse muscle aches that have been worsening over a period of 6 months. She
also has not been sleeping well. Her primary doctor evaluated her and sent screening
laboratory tests, which returned with a positive rheumatoid factor. She has read about
rheumatoid arthritis on the Internet and is very concerned that she has the disease based on
her symptoms and her positive test. Which of the following is true in regard to diagnosing
rheumatoid arthritis (RA)?

A
10% of healthy individuals will test positive for antibodies to cyclic citrullinated polypeptides
(anti-CCP).
B Erythrocyte sedimentation rate (ESR) is elevated in 70% of patients with active disease.
C In early disease, rheumatoid factor is more accurate than anti-CCP.
D Fewer than one-third of unselected patients with positive rheumatoid factor will have RA.
E Radiographs should be performed in this patient to help with diagnosis.
ANSWER:D
1584
IX-62. A 34-year-old woman is seen in the allergy clinic for complaint of chronic rhinitis.
She reports that she first developed seasonal rhinitis in her early twenties, limited to the fall
of the year. At that point, she would use diphenhydramine on an as-needed basis, but she
limited her use because of the sedating side effects. Since she moved into her current home 5
years ago, her symptoms have become continuous. She states her nose stays congested, and
she has constant postnasal drip. She awakens frequently at night with a cough and complains
of daytime fatigue due to inadequate sleep. She currently is taking fexofenadine, 180 mg
daily, but states she feels no relief from her symptoms. At night, she will occasionally take
diphenhydramine because of its sedating side effects. Her past medical history is significant
for eczema, for which she uses topical steroid creams, and frequent migraines requiring
propranolol for prophylaxis. She is allergic to ragweed. She has no other known
environmental allergens but has never had skin-prick testing. She does not smoke cigarettes
or consume alcohol. She works as a librarian in a new building. Her home is a wooden single
family home built in the 1930s. There is carpeting throughout the first floor of the home,
including the bedroom. The basement is not finished and has been flooded in the past. She
has a 1-year old cat that lives indoors. On physical examination, she has large and boggy
nasal turbinates. Her posterior oropharynx shows evidence of cobblestoning. Her lungs are
clear without wheezes. Skinprick testing demonstrates allergic responses to ragweed, grass,
cat and dog dander, and dust mites. All of the following would be appropriate initial therapy
in this patient except
A immunotherapy for cat dander
B intranasal mometasone furoate
C oral montelukast
D removal of carpets and drapes from the bedroom
E weekly laundry of the bedding at temperatures >54.4°C

ANSWER:A
1585
IX-63. A 46-year-old woman presents to your clinic with multiple complaints. She
describes fatigue and general malaise over 2–3 months. Her appetite has decreased. She
thinks she has unintentionally lost ~5.5 kg. Lately she notes pain and stiffness in her fingers
on both hands that is worse in the morning and with repetitive movement. She has a
grandmother and a sister who have rheumatoid arthritis (RA), and she is very concerned that
she now has it as well. Which of her complaints represents the most common manifestation
of established RA?
A Fatigue and anorexia for >2 months with concomitant joint pain
B Morning joint stiffness lasting for >1 h
C Pain in symmetric joints worsened with movement
D Positive family history with two relatives with RA
E Weight loss >4.5 kg during period of active disease
ANSWER:C
1586
A 23-year-old man seeks evaluation for low back pain. He states that when he first awakens
there is a dull aching pain in his lower lumbar and gluteal region. When he first noticed the
pain about 6 months ago, he thought the pain might be related to his mattress, but it has
worsened even after buying a new mattress. Most mornings, it takes about 45–60 min to
loosen up after he has awakened, but the pain will recur if he is idle. He is currently in law
school and finds it increasingly difficult to remain in classes because of back pain. When he
exercises, the pain lessens. There are occasional nights that the pain will awaken him from
sleep, and he will have to move around and stretch his back to improve the pain. On physical
examination, there is pain with palpation at the iliac crests, ischial tuberosities, greater
trochanters, and heels. With maximal inspiration, the chest expands 4 cm, and there is
decreased flexion of the lumbar spine. A radiograph of the pelvis shows erosions and
sclerosis of the sacroiliac joints bilaterally. Which of the following tests is most likely to be
positive in this individual?
A Alkaline phosphatase
B Antibodies directed against cyclic citrullinated peptides (CCP)
C Antinuclear antibodies
D HLA-B27
E Rheumatoid factor

ANSWER:D
1587
An 84-year-old man is seen by his primary care provider with symptoms of acute gouty
arthritis in the first great toe and ankle on the left. He has a prior history of gout presenting
similarly. His past medical history is significant for myelodysplasia, congestive heart failure,
hypercholesterolemia, and chronic kidney disease. He is taking pravastatin, aspirin,
furosemide, metolazone, lisinopril, and metoprolol XL. His baseline creatinine is 2.4 mg/dL,
and uric acid level 9.3 mg/dL. His most recent complete blood count results are white blood
cell count 2880/µL, hemoglobin 8.2 g/dL, hematocrit 26.2%, and platelet 68,000/ µL. Which
of the following medication regimens are most appropriate for the treatment of this patient?
A Allopurinol, 100 mg once daily
B Colchicine, 1 mg IV once, then 0.5 mg IV every 6 h until improvement
C Indomethacin, 25 mg three times daily
D Prednisone, 40 mg once daily
E Probenecid, 250 mg twice daily
ANSWER:D
1588
A patient with primary Sjögren’s syndrome that was diagnosed 6 years ago and treated with
tear replacement for symptomatic relief notes continued parotid swelling for the last 3
months. She has also noted enlarging posterior cervical lymph nodes. Evaluation shows
leukopenia and low C4 complement levels. What is the most likely diagnosis?
A Chronic pancreatitis
B Secondary Sjögren’s syndrome
C HIV infection
D Lymphoma
E Amyloidosis
ANSWER:D

1589
A 19-year-old recent immigrant from Ethiopia comes to your clinic to establish primary
care. She currently feels well. Her past medical history is notable for a recent admission to
the hospital for new-onset atrial fibrillation. As a child in Ethiopia, she developed an illness
that caused uncontrolled flailing of her limbs and tongue lasting ~1 month. She also has had
three episodes of migratory large-joint arthritis during her adolescence that resolved with
pills that she received from the pharmacy. She is currently taking metoprolol and warfarin
and has no known drug allergies. Physical examination reveals an irregularly irregular heart
beat with normal blood pressure. Her Point of Maximal Impluse (PMI) is most prominent at
the mid clavicular line and is normal in size. An early diastolic rumble and 3/6 holosystolic
murmur are heard at the apex. A soft early diastolic murmur is also heard at the left third
intercostal space. You refer her to a cardiologist for evaluation of valve replacement and
echocardiography. What other intervention might you consider at this time?
A Glucocorticoids
B Daily aspirin
C Daily doxycycline
D Monthly penicillin G injections
E Penicillin G injections as needed for all sore throats
ANSWER:D
1590 What is the most common cause of hypothyroidism worldwide?
A Autoimmune disease
B Graves’ disease
C Iatrogenic causes
D Iodine deficiency
E Medication side effects
ANSWER:D

1591
A 23-year-old woman presents to clinic complaining of months of weight gain, fatigue,
amenorrhea, and worsening acne. She cannot identify when her symptoms began precisely,
but she reports that without a change in her diet she has noted a 12.3-kg weight gain over the
past 6 months. She has been amenorrheic for several months. On examination she is noted to
have truncal obesity with bilateral purplish striae across both flanks. Cushing’s syndrome is
suspected. Which of the following tests should be used to make the diagnosis?
A 24-h urine free cortisol
B Basal adrenocorticotropic hormone (ACTH)
C Corticotropin-releasing hormone (CRH) level at 8 A.M.
D Inferior petrosal venous sampling
E Overnight 1 mg dexamethasone suppression test
ANSWER:A
1592
Secretion of gonadotropin releasing-hormone (GnRH) normally stimulates release of
luteinizing hormone (LH) and follicle-stimulating hormone (FSH) which promote
production and release of testosterone and estrogen. Which mechanism below best explains
how long-acting gonadotropin-releasing hormone agonists (e.g., leuprolide) decrease
testosterone levels in the management of prostate cancer?
A
GnRH agonists also promote production of sex hormone–binding globulin, which decreases
the availability of testosterone
B Negative feedback loop between GnRH and LH/FSH
C Sensitivity of LH and FSH to pulse frequency of GnRH
D
Translocation of the cytoplasmic nuclear receptor into the nucleus with constitutive
activation of GnRH
E
ANSWER:C

1593
A 44-year-old woman seeks evaluation for irregular menstrual cycles with heavy menstrual
bleeding. She reports that her menses had been regular with 28-day cycles since her early
twenties. However, for the past 6 months, her cycles have been 22–25 days with heavy
associated bleeding that is unusual for her. She has had rare hot flashes and sleep
disturbance. She is requesting assistance in controlling these symptoms. You suspect she is
perimenopausal, and hormonal testing on day 2 of her menses confirms this suspicion. You
are considering treatment with oral contraceptives for control of her symptoms and to protect
against unintended pregnancy. All of the following would be considered contraindications to
use of oral contraceptive pills except
A breast cancer
B cigarette smoking
C kidney disease
D liver disease
E prior history of deep venous thrombosis
ANSWER:C
1594 All the following are risk factors for the development of osteoporotic fractures except
A African-American race
B current cigarette smoking
C female sex
D low body weight
E physical inactivity
ANSWER:A
1595 All the following drugs are associated with an increased risk of osteoporosis in adults except
A cyclosporine
B dilantin
C heparin
D prednisone
E ranitidine
ANSWER:E

1596
A 34-year-old woman presents to your clinic with a variety of complaints that have been
worsening over the past year or so. She notes fatigue, amenorrhea, and weight gain. She
states that her primary physician diagnosed her with hypothyroidism several months ago,
and she has been faithfully taking thyroid hormone replacement. Her thyroid-stimulating
hormone (TSH) has been in the normal range over the last two laboratory checks. When her
symptoms did not improve on synthroid, she was sent to your clinic for further evaluation. A
diagnosis of panhypopituitarism is considered. All of the following are consistent with
normal pituitary function except
A
basal elevation of follicle-stimulating hormone (FSH) and luteinizing hormone (LH) in a
postmenopausal woman
B elevation of aldosterone after infusion of cosyntropin
C elevation of growth hormone after ingestion of a glucose load
D elevation of cortisol after injection of regular insulin
E elevation of TSH after infusion of thyrotropinreleasing hormone (TRH)
ANSWER:C
1597
A 33-year-old male with end-stage renal disease who is on hemodialysis complains of
decreased libido, inability to maintain erections, increasing fatigue, and mild weakness. He
has been on a stable hemodialysis regimen for 8 years, and all his electrolytes are normal.
Further evaluation reveals a reduced serum testosterone level. Measurement of which of the
following will distinguish primary from secondary hypogonadism?
A Aldosterone
B Cortisol
C Estradiol
D Luteinizing hormone
E Thyroid-stimulating hormone
ANSWER:C

1598
A 44-year-old male is involved in a motor vehicle collision. He sustains multiple injuries to
the face, chest, and pelvis. He is unresponsive in the field and is intubated for airway
protection. An intravenous line is placed. The patient is admitted to the intensive care unit
(ICU) with multiple orthopedic injuries. He is stabilized medically and on hospital day 2
undergoes successful open reduction and internal fixation of the right femur and right
humerus. After his return to the ICU, you review his laboratory values. TSH is 0.3 mU/L,
and the total T4 level is normal. T3 is 0.6 µg/dL. What is the most appropriate next
management step?
A Initiation of levothyroxine
B A radioiodine uptake scan
C A thyroid ultrasound
D Observation
E Initiation of prednisone
ANSWER:D
1599 All the following biochemical markers are a measure of bone resorption except
A serum alkaline phosphatase
B serum cross-linked N-telopeptide
C serum cross-linked C-telopeptide
D urine hydroxyproline
E urine total free deoxypyridinoline
ANSWER:A
1600
A 54-year-old woman is referred to endocrinology for evaluation of osteoporosis after a
recent evaluation of back pain revealed a compression fracture of the T4 vertebral body. She
is perimenopausal with irregular menstrual periods and frequent hot flashes. She does not
smoke. She otherwise is well and healthy. Her weight is 70 kg, and height is 168 cm. A bone
mineral density scan shows a T-score of –3.5 SD and a Z-score of –2.5 SD. All of the
following tests are indicated for the evaluation of osteoporosis in this patient except
A 24-h urine calcium
B follicle-stimulating hormone and luteinizing hormone levels
C serum calcium
D renal function panel
E vitamin D levels (25-hydroxyvitamin D)

ANSWER:B
1601
A 67-year-old woman presents to clinic after a fall on the ice a week ago. She visited the
local emergency room immediately after the fall, where hip radiographs were performed and
were negative for fracture or dislocation. They did reveal fusion of the sacroiliac joints and
coarse trabeculations in the ilium, consistent with Paget disease. A comprehensive metabolic
panel was also sent at that visit and is remarkable for an alkaline phosphatase of 157 U/L,
with normal serum calcium and phosphate levels. She was discharged with analgesics and
told to follow up with her primary care doctor for further management of her radiographic
findings. She is recovering from her fall and denies any long-standing pain or immobility of
her hip joints. She states that her father suffered from a bone disease that caused him
headaches and hearing loss near the end of his life. She is very concerned about the
radiographs and wants to know what they mean. Which of the following is the best treatment
strategy at this point?
A Initiate physical therapy and non-weight bearing exercises to strengthen the hip.
B
No treatment; she is asymptomatic. Follow radiographs and laboratory findings every 6
months.
C Prescribe vitamin D and calcium.
D Start an oral bisphosphonate.
E Start high-dose prednisone with rapid taper over 1 week.
ANSWER:D
1602
A 26-year-old woman presents with 2 weeks of nausea, vomiting, and jaundice. She has
been previously healthy and has no past medical history. On examination, a palpable liver
edge is appreciated. Ocular findings are presented in Figure X-14 (Color Atlas). Her
transaminases and total bilirubin are elevated. Which of the following tests will lead to a
definitive diagnosis in this patient?
A Anti-smooth-muscle antibody
B Hepatitis B surface antigen
C Liver biopsy with quantitative copper assay
D Serum ceruloplasmin
E Total iron-binding capacity and ferritin
ANSWER:C

1603
A 29-year-old woman presents to your clinic complaining of difficulty swallowing, sore
throat, and tender swelling in her neck. She has also noted fevers intermittently over the past
week. Several weeks prior to her current symptoms she experienced symptoms of an upper
respiratory tract infection. She has no past medical history. On physical examination, she is
noted to have a small goiter that is painful to the touch. Her oropharynx is clear. Laboratory
studies are sent, and reveal a white blood cell count of 14,100 cells/µL with a normal
differential, erythrocyte sedimentation rate (ESR) of 53 mm/h, and a thyroid-stimulating
hormone (TSH) of 21 µΙU/mL. Thyroid antibodies are negative. What is the most likely
diagnosis?
A Autoimmune hypothyroidism
B Cat-scratch fever
C Graves’ disease
D Ludwig’s angina
E Subacute thyroiditis
ANSWER:E
1604 What is the most appropriate treatment for the patient described above?
A Iodine ablation of the thyroid
B Large doses of aspirin
C Local radiation therapy
D No treatment necessary
E Propylthiouracil
ANSWER:B
1605
The Diabetes Control and Complications Trial (DCCT) provided definitive proof that
reduction in chronic hyperglycemia
A improves microvascular complications in type 1 diabetes mellitus
B improves macrovascular complications in type 1 diabetes mellitus
C improves microvascular complications in type 2 diabetes mellitus
D improves macrovascular complications in type 2 diabetes mellitus
E improves both microvascular and macrovascular complications in type 2 diabetes mellitus
ANSWER:A

1606
A 54-year-old woman undergoes thyroidectomy for follicular carcinoma of the thyroid.
About 6 h after surgery, the patient complains of tingling around her mouth. She
subsequently develops a pins-and-needles sensation in the fingers and toes. The nurse calls
the physician to the bedside to evaluate the patient after she has severe hand cramps when
her blood pressure is taken. Upon evaluation, the patient is still complaining of intermittent
cramping of her hands. Since surgery, she has received morphine sulfate, 2 mg, for pain and
compazine, 5 mg, for nausea. She has had no change in her vital signs and is afebrile.
Tapping on the inferior portion of the zygomatic arch 2 cm anterior to the ear produces
twitching at the corner of the mouth. An electrocardiogram (ECG) shows a QT interval of
575 ms. What is the next step in evaluation and treatment of this patient?
A Administration of benztropine, 2 mg IV
B Administration of calcium gluconate, 2 g IV
C Administration of magnesium sulphate, 4 g IV
D Measurement of calcium, magnesium, phosphate, and potassium levels
E Measurement of forced vital capacity
ANSWER:B
1607
A 68-year-old woman with stage IIIB squamous cell carcinoma of the lung is admitted to the
hospital because of altered mental status and dehydration. Upon admission, she is found to
have a calcium level of 19.6 mg/dL and phosphate of 1.8 mg/dL. Concomitant measurement
of parathyroid hormone was 0.1 pg/mL (normal 10–65 pg/mL), and a screen for
parathyroid hormone–related peptide was positive. Over the first 24 h, the patient receives 4
L of normal saline with furosemide diuresis. The next morning, the patient’s calcium is now
17.6 mg/dL and phosphate is 2.2 mg/dL. She continues to have delirium. What is the best
approach for ongoing treatment of this patient’s hypercalcemia?
A Continue therapy with large-volume fluid administration and forced diuresis with furosemide.
B
Continue therapy with large-volume fluid administration, but stop furosemide and treat with
hydrochlorothiazide.
C Initiate therapy with calcitonin alone.
D Initiate therapy with pamidronate alone.
E Initiate therapy with calcitonin and pamidronate.
ANSWER:E

1608
A patient visited a local emergency room 1 week ago with a headache. She received a head
MRI, which did not reveal a cause for her symptoms, but the final report states “an empty
sella is noted. Advise clinical correlation.” The patient was discharged from the emergency
room with instructions to follow-up with her primary care physician as soon as possible. Her
headache has resolved, and the patient has no complaints; however, she comes to your office
1 day later very concerned about this unexpected MRI finding. What should be the next step
in her management?
A
Diagnose her with subclinical pan-hypopituitarism, and initiate low-dose hormone
replacement.
B Reassure her and follow laboratory results closely.
C Reassure her and repeat MRI in 6 months.
D
This may represent early endocrine malignancy— whole-body positron-emission
tomography/CT is indicated.
E
This MRI finding likely represents the presence of a benign adenoma—refer to
neurosurgery for resection.
ANSWER:B
1609
A 16-year-old previously healthy teenage boy presents to the local emergency room with a
headache that has been worsening over the course of 2 months. His parents note that “he just
hasn’t seemed like himself,” and over the past 2 weeks has been complaining of double
vision. He experienced profuse vomiting this afternoon, which prompted his visit. He also
describes weight gain over the same 2 month time period and has not been sleeping well. On
examination, he is drowsy, and funduscopic examination reveals papilledema. He has no
fever, neck stiffness, or elevated white blood cell count. Which of the following is the most
likely cause?
A Carney syndrome
B Congenital pan-hypopituitarism
C Craniopharyngioma
D McCune-Albright syndrome
E Meningioma
ANSWER:C
1610
Which of the following is the most common site for a fracture associated with osteoporosis?

A Femur
B Hip
C Radius
D Vertebra
E Wrist
ANSWER:D
1611
Postmenopausal estrogen therapy has been shown to increase a female’s risk of all the
following clinical outcomes except
A breast cancer
B hip fracture
C myocardial infarction
D stroke
E venous thromboembolism
ANSWER:B
1612
All the following therapies have been shown to reduce the risk of hip fractures in
postmenopausal women with osteoporosis except
A alendronate
B estrogen
C parathyroid hormone
D raloxifene
E risedronate
ANSWER:D
1613
A 45-year-old man is diagnosed with pheochromocytoma after presentation with confusion,
marked hypertension to 250/140 mmHg, tachycardia, headaches, and flushing. His
fractionated plasma metanephrines show a normetanephrine level of 560 pg/mL and a
metanephrine level of 198 pg/mL (normal values: normetanephrine: 18–111 pg/mL;
metanephrine: 12–60 pg/mL). CT scanning of the abdomen with IV contrast demonstrates a
3-cm mass in the right adrenal gland. A brain MRI with gadolinium shows edema of the
white matter near the parietooccipital junction consistent with reversible posterior
leukoencephalopathy. You are asked to consult regarding management. Which of the
following statements is true regarding management of pheochromocytoma is this individual?

A
Beta-blockade is absolutely contraindicated for tachycardia even after adequate alpha-
blockade has been attained.
B
Immediate surgical removal of the mass is indicated, because the patient presented with
hypertensive crisis with encephalopathy.
C
Salt and fluid intake should be restricted to prevent further exacerbation of the patient’s
hypertension.
D Treatment with phenoxybenzamine should be started at a high dose (20–30 mg three times
daily) to rap- idly control blood pressure, and surgery can be undertaken within 24–48 h.
E
Treatment with IV phentolamine is indicated for treatment of the hypertensive crisis.
Phenoxybenzamine should be started at a low dose and titrated to the maximum tolerated
dose over 2–3 weeks. Surgery should not be planned until the blood pressure is consistently
below 160/100 mmHg.
ANSWER:E
1614
Inhibition of renin activity is a contemporary target mechanism for treatment of
hypertension. All of the following physiologic alterations will cause an increase in renin
secretion except
A decreased effective circulating blood volume
B high-potassium diet
C increased sympathetic activity
D low solute delivery to the distal convoluted tubules
E upright posture
ANSWER:B
1615
Which of the following represents the likelihood of finding a pituitary microadenoma at
autopsy in the general population?
A 0.1%
B 2%
C 5%
D 11%
E 25%
ANSWER:E

1616
A 40-year-old female with Graves’ disease was recently started on methimazole. One month
later she comes to the clinic for a routine follow-up. She notes some low-grade fevers,
arthralgias, and general malaise. Laboratories are notable for a mild transaminitis and a
glucose of 150 mg/dL. All the following are known side effects of methimazole except
A agranulocytosis
B rash
C arthralgia
D hepatitis
E insulin resistance
ANSWER:E
1617
A 60-year-old woman is referred to your office for evaluation of hypercalcemia of 12.9
mg/dL. This was found incidentally on a chemistry panel that was drawn during a
hospitalization for cervical spondylosis. Despite fluid administration in the hospital, her
serum calcium at discharge was 11.8 mg/dL. The patient is asymptomatic. She is otherwise in
good health and has had her recommended age-appropriate cancer screening. She denies
constipation or bone pain and is now 8 weeks out from her spinal surgery. Today, her serum
calcium level is 12.4 mg/dL, and phosphate is 2.3 mg/dL. Her hematocrit and all other
chemistries including creatinine were normal. What is the most likely diagnosis?
A Breast cancer
B Hyperparathyroidism
C Hyperthyroidism
D Multiple myeloma
E Vitamin D intoxication
ANSWER:B

1618
A 62-year-old man presents to a local emergency room complaining of chest pressure and
feeling “like my heart is fluttering inside my chest.” He experienced similar symptoms 1
month ago that resolved spontaneously. He did not seek medical attention at that time. He
has no significant past medical history. On review of systems he notes some recent weight
loss and excessive sweating. He feels as though his appetite has increased lately. His wife
adds that he has recently taken some time off work due to fatigue; despite his time off he has
not been able to relax and has not been sleeping well. On physical examination his heart rate
is irregular at 140–150 beats/minute. Blood pressure is 134/55 mmHg. He is admitted to the
hospital and screening tests reveal an undetectable thyroid-stimulating hormone level. Which
of the following statements is true?
A
50% of hyperthyroid patients will convert from atrial fibrillation to normal sinus rhythm
with thyroid management alone.
B
A firm, small thyroid on physical examination would be compatible with a diagnosis of
Graves’ disease.
C Atrial fibrillation is the most common cardiac manifestation of hyperthyroidism.
D His excessive sweating is likely not related to hyperthyroidism.
E Hyperthyroidism leads to a high-output state for the heart, and narrowing pulse pressure.
ANSWER:A
1619
The patient described above is started on atenolol and his heart rate slows to 80 beats/min.
Which of the following additional therapies is indicated?
A Diltiazem
B Itraconazole
C Liothyronine
D Methimazole
E Phenoxybenzamine
ANSWER:D

1620
A patient presents to his primary care physician complaining of fatigue and hair loss. He has
gained 6.4 kg since his last clinic visit 6 months ago but notes markedly decreased appetite.
On review of systems, he reports that he is not sleeping well and feels cold all the time. He is
still able to enjoy his hobbies and spending time with his family, and does not believe that he
is depressed. His examination reveals diffuse alopecia and slowed deep tendon reflex
relaxation. Hypothyroidism is high on the differential for this patient. Which of the
statements regarding that diagnosis is correct?
A
A normal thyroid-stimulating hormone (TSH) excludes secondary, but not primary
hypothyroidism.
B T3 measurement is not indicated to make the diagnosis.
C The T3/T4 ratio is important for determining response to therapy.
D
Thyroid peroxidase antibodies distinguish between primary and secondary hypothyroidism.
E Unbound T4 is a better screening test than TSH for subclinical hypothyroidism.
ANSWER:B
1621 Which of the following statements regarding autoimmune hypothyroidism is true?
A 10% of 40- to 60-year-old adults have subclinical hypothyroidism.
B Absence of a goiter makes autoimmune hypothyroidism unlikely.
C Family history of autoimmune disorders does not significantly increase risk.
D It is more common in the Pacific Rim where diets are lower in iodine.
E Viral thyroiditis does not induce subsequent autoimmune thyroiditis.
ANSWER:E
1622 In regard to Graves’ disease, which of the following is true?
A It accounts for >90% of all causes of thyrotoxicosis.
B It occurs in 2% of women.
C It typically occurs in patients between 50 and 60 years of age.
D Populations with a low iodine intake have an increased prevalence.
E There is an equal male-to-female prevalence.
ANSWER:B

1623
A 16-year-old male is brought to your clinic by his parents due to concern about his weight.
He has not seen a physician for many years. He states that he has gained weight due to
inactivity and that he is less active because of exertional chest pain. He takes no medications.
He was adopted and his parents do not know the medical history of his biologic parents.
Physical examination is notable for Stage 1 hypertension and body mass index of 30 kg/ m2.
He has xanthomas on his hands, heels, and buttocks. Laboratory testing shows a low-density
lipoprotein (LDL) of 210 mg/dL, creatinine of 0.7 mg/dL, total bilirubin of 3.1 mg/dL,
haptoglobin <6 mg/dL, and a glycosylated hemoglobin of 6.7%. You suspect a hereditary
lipoproteinemia due to the clinical and laboratory findings. Which test would be diagnostic
of the primary lipoprotein disorder in this patient?
A Congo red staining of xanthoma biopsy
B CT scan of the liver
C Family pedigree analysis
D Gas chromatography
E LDL receptor function in skin biopsy
ANSWER:D
1624
A 35 year-old woman presents with amenorrhea over the past 4 months. She has been trying
to get pregnant without success. She complains of a thin milky discharge from her nipples
and over the past several days has noted some blurry vision. On laboratory testing, her
prolactin level is 110 µg/L (normal: 5–20 µg/L). A head MRI is performed and reveals an 11-
mm pituitary macroadenoma. What is the next step in management?
A Follow visual fields; if worse in 1 month, refer for surgery.
B Reassure the patient and follow-up closely.
C Refer for urgent neurosurgery.
D Repeat MRI in 4 months.
E Do visual field testing and initiate a dopamine agonist.
ANSWER:E
1625
A patient is asked to undergo a testing protocol to assess adrenocortical function. After 5
days of severe sodium restriction (10 mmol/day), blood is drawn for analysis. Which
hormone abnormality may be detected using this protocol?
A Hypercorticolism

B Glucocorticoid deficiency
C Mineralocorticoid deficiency
D Mineralocorticoid excess
E Vasopressin excess
ANSWER:C
1626
A 38-year-old woman presents to her primary care doctor complaining of fatigue and
irritability. She thinks these symptoms have been worsening over a period of several months.
She has a history of mild intermittent asthma and hypertriglyceridemia. Physical examination
reveals a resting heart rate of 105 beats/min, blood pressure of 136/ 72 mmHg, bilateral
proptosis and warm, moist skin. Screening tests are sent and reveal a thyroid-stimulating
hormone (TSH) level that is undetectable and a normal unbound T4. What should be the
next step in diagnosis?
A Radionuclide scan of the thyroid
B Thyroid-stimulating antibody screen
C Thyroid peroxidase (TPO) antibody screen
D Total T4
E Unbound T3
ANSWER:E
1627
A 24-year-old female patient returns to your office to review her recent laboratory data. On
her last clinic visit, you began an evaluation for secondary amenorrhea. Her vital signs are
normal and her body mass index (BMI) is 20 kg/m2. Her β-human chorionic gonadotropin
is negative. Serum follicle-stimulation hormone (FSH) is below the lower limit of normal.
Serum testosterone is within normal limits. Morning cortisol is 24 mg/dL. Urinalysis is
unremarkable and there is no glucose in the urine. Thyroid-stimulating hormone is 3.7
mU/L. Serum prolactin is elevated. What is the most likely cause of this patient’s secondary
amenorrhea?
A Ectopic pregnancy
B Pituitary tumor
C Primary ovarian failure
D Uterine outflow obstruction
E Malnutrition
ANSWER:B

1628
A couple seeks advice regarding infertility. The female partner is 35 years old. She has never
been pregnant and was taking oral contraceptive pills from age 20 until age 34. It is now 16
months since she discontinued her oral contraceptives. She is having menstrual cycles
approximately once every 35 days, but occasionally will go as long as 60 days between
cycles. Most months, she develops breast tenderness about 2–3 weeks after the start of her
menstrual cycle. When she was in college, she was treated for Neisseria gonorrhoeae that was
diagnosed when she presented to the student health center with a fever and pelvic pain. She
otherwise has no medical history. She works about 60 h weekly as a corporate attorney and
exercises daily. She drinks coffee daily and alcohol at social occasions only. Her body mass
index (BMI) is 19.8 kg/m2. Her husband, who is 39 years old, accompanies her to the
evaluation. He also has never had children. He was married previously from the ages of
24–28. He and his prior wife attempted to conceive for about 15 months, but were
unsuccessful. At that time, he was smoking marijuana on a daily basis and attributed their
lack of success to his drug use. He has now been completely free of drugs for 9 years. He
suffers from hypertension and is treated with lisinopril, 10 mg daily. He is not obese (BMI,
23.7 kg/m2). They request evaluation for their infertility and request help with conception.
Which of the following statements is true in regards to their infertility and likelihood of
success in conception?
A
Determination of ovulation is not necessary in the female partner as most of her cycles occur
regularly, and she develops breast tenderness mid-cycle indicative of ovulation
B
Lisinopril should be discontinued immediately because of the risk of birth defects associated
with its use
C
The female partner should be assessed for tubal patency by a hysterosalpingogram. If
significant scarring is found, in vitro fertilization should be strongly considered to decrease
the risk of ectopic pregnancy.
D
The prolonged use of oral contraceptives for >10 years has increased the risk of anovulation
and infertility
E
The use of marijuana by the male partner is directly toxic to sperm motility, and this is the
likely cause of their infertility.
ANSWER:C

1629
A 22-year-old male seeks evaluation from his primary care doctor for gynecomastia that has
developed over the past 2 years. He states he did not enter puberty until much later than his
friends and has only had sparse growth of facial and axillary hair. He continues to have poor
libido and rarely desires sexual intercourse, even though he has been in a monogamous
relationship for the past 8 months. His girlfriend is increasingly frustrated by his lack of
sexual desire and also urged him to seek medical evaluation. He has no other medical history
and was born prematurely at 34 weeks’ gestation. His birth weight was 2400 g (50th
percentile). His early development was normal. During elementary school, he was held back
in third grade because of learning difficulties and thereafter was in special educational classes
to assist him with reading and mathematics. He is taking no medications. On physical
examination, he is 188 cm tall and has eunuchoid features. His facial, axillary, and genital
hair is sparse. Gynecomastia is present. The testes are small, measuring 2.8 cm in length.
What is the most likely diagnosis in this patient?
A Androgen insensitivity syndrome (testicular feminization)
B Klinefelter syndrome
C Mixed gonadal dysgenesis (45,X/46,XY mosaicism)
D Testicular dysgenesis
E True hermaphroditism
ANSWER:B
1630 All the following drugs may interfere with testicular function except
A cyclophosphamide
B ketoconazole
C metoprolol
D prednisone
E spironolactone
ANSWER:C
1631
A 65-year-old man with a central left upper lobe lung mass presents with renal stones and
generalized bone pain. He is found to have a calcium level of 16.4 mg/dL with a phosphate
level of 1.2 mg/dL. A bone scan is normal. Which of the following laboratory tests is most
likely to establish a diagnosis?
A Adrenocorticotropic hormone (ACTH)
B Cortisol

C Magnesium level
D Parathyroid hormone (intact PTH or PTHi)
E Parathyroid hormone–related peptide (PTHrp)
ANSWER:E
1632 A biopsy of the lung mass in the patient in Question X-51 will most likely show:
A Bronchoalveolar lung carcinoma
B Bronchial carcinoid
C Poorly differentiated adenocarcinoma
D Small cell carcinoma
E Squamous cell carcinoma
ANSWER:E
1633 All the following are direct actions of parathyroid hormone (PTH) except
A increased calcium resorption from bone
B increased calcium resorption from the kidney
C increased calcium resorption from the gastrointestinal tract
D increased synthesis of 1,25 dihydroxyvitamin D
E decreased phosphate resorption from the kidney
ANSWER:C

1634
A 45-year-old Caucasian woman seeks advice from her primary care physician regarding her
risk for osteoporosis and the need for bone density screening. She is a lifelong nonsmoker
and drinks alcohol only socially. She has a history of moderate-persistent asthma since age
12. She is currently on fluticasone, 44 mg/puff twice daily, with good control currently. She
last required oral prednisone therapy about 6 months ago when she had influenza that was
complicated by an asthma flare. She took prednisone for a total of 14 days. She has had
three pregnancies and two live births at ages 39 and 41. She currently has irregular periods
occurring approximately every 42 days. Her follicle-stimulating hormone level is 25 mIU/L
and 17β-estradiol level is 115 pg/mL on day 12 of her menstrual cycle. Her mother and
maternal aunt both have been diagnosed with osteoporosis. Her mother also has rheumatoid
arthritis and requires prednisone therapy, 5 mg daily. Her mother developed a compression
fracture of the lumbar spine at age 68. On physical examination, the patient appears well and
healthy. Her height is 168 cm. Her weight is 66.4 kg. The chest, cardiac, abdominal,
muscular, and neurologic examinations are normal. What do you tell the patient about the
need for bone density screening?
A As she is currently perimenopausal, she should have a bone density screen every other year
until she completes menopause and then have bone densitometry measured yearly thereafter.
B
Because of her family history, she should initiate bone density screening yearly beginning
now.
C Bone densitometry screening is not recommended until after completion of menopause.
D
Delayed childbearing until the fourth and fifth decade decreases her risk of developing
osteoporosis
E
Her use of low-dose inhaled glucocorticoids increases her risk of osteoporosis threefold, and
she should undergo yearly bone density screening.
ANSWER:C

1635
A 55-year-old male is admitted to the intensive care unit with 1 week of fever and cough. He
was well until 1 week before admission, when he noted progressive shortness of breath,
cough, and productive sputum. On the day of admission the patient was noted by his wife to
be lethargic and unresponsive. 911 was called, and the patient was intubated in the field and
then brought to the emergency department. His medications include insulin. The past
medical history is notable for alcohol abuse, diabetes mellitus, and chronic renal
insufficiency. Temperature is 38.9°C (102°F). He is hypotensive with a blood pressure of
76/40 mmHg. Oxygen saturation is 86% on room air. On examination, the patient is sedated
and intubated. Jugular venous pressure is normal. There are decreased breath sounds at the
right lung base with egophony. Heart sounds are normal. The abdomen is soft. There is no
peripheral edema. Chest radiography shows a right lower lobe infiltrate with a moderate
pleural effusion. An electrocardiogram is normal. Sputum Gram stain shows gram-positive
diplococci. White blood cell count is 23 × 103/µL, with 70% polymorphonuclear cells and
6% bands. Blood urea nitrogen is 80 mg/dL, and creatinine is 6.1 mg/dL. Plasma glucose is
425 mg/dL. He is started on broad-spectrum antibiotics, intravenous fluids, omeprazole, and
an insulin drip. A nasogastric tube is inserted, and tube feedings are started. On hospital day
2 plasma phosphate is 1.0 mg/dL. All of following are causes of hypophosphatemia except
A sepsis
B renal failure
C insulin
D alcoholism
E malnutrition
ANSWER:B
1636
A 50-year-old male presents to the clinic for a routine health examination. A comprehensive
metabolic panel shows a serum calcium level of 11.2 mg/dL. Serum phosphate is 3.0 mg/dL.
Serum creatinine is normal. He denies bone pain, lethargy, weakness, or weight loss. What is
the most common cause of hypercalcemia in outpatients?
A Malignancy
B Medications
C Milk-alkali syndrome
D Primary hyperparathyroidism

E Granulomatous disease
ANSWER:D
1637 All of the following would be indicated in the workup of infertility except
A endometrial biopsy
B hysterosalpingogram
C measurement of testosterone and dehydroepiandosterone in the female partner
D
measurement of testosterone, follicle-stimulating hormone (FSH) and luteinizing hormone
(LH) in the male partner
E semen analysis
ANSWER:A
1638
A 21-year-old patient presents to the emergency room with several days of severe nausea
and vomiting. His roommate noted that he hadn’t been able to keep down solid foods and
that his eyes and skin have taken on a “yellow” appearance. The patient has no past medical
history and denies alcohol or substance abuse. He takes no medications. He is admitted to the
hospital for further management. On examination he is ill-appearing and jaundiced. The
liver edge is palpable and he has mild abdominal tenderness. He has a normal neurologic
examination. His total bilirubin is 2.0 mg/dL (normal range 0.1–1.2 mg/dL) and AST is 86
U/L (normal range 0–37 U/L). His prothrombin time is normal. Viral hepatitis serologies are
all negative. A liver biopsy and quantitative copper assay are performed and the patient is
subsequently diagnosed with Wilson disease. Which of the following is the most appropriate
treatment for this patient?
A Dimercaprol
B Immediate liver transplant
C Penicillamine
D Trientine and liver transplant evaluation
E Zinc
ANSWER:E
1639 The World Health Organization (WHO) recently defined osteoporosis operationally as
A
a patient with a bone density less than the mean of age-, race-, and gender-matched controls
B
a patient with a bone density less than 1.0 standard deviation (SD) below the mean of race-
and gendermatched controls

C
a patient with a bone density less than 1.0 SD below the mean of age-, race-, and gender-
matched controls
D
a patient with a bone density less than 2.5 SD below the mean of race-and gender-matched
controls
E
a patient with a bone density less than 2.5 SD below the mean of age-, race-, and gender-
matched controls
ANSWER:D
1640
A 54-year-old man has been hospitalized following a myocardial infarction. He has a history
of hypertension, hypertriglyceridemia, obesity, mild renal insufficiency and diet-controlled
diabetes. His condition is improving, and on the third day of admission his serum urate is
elevated at 8.8 mg/dL. He denies any joint complaints and cannot recall ever having
symptoms compatible with gout. His serum creatinine is 1.5 mg/dL, which is close to his
baseline over the past 3 years. In regard to this patient’s hyperuricemia, which of the
following is true?
A >70% of hospitalized patients will have increased serum urate levels.
B His hyperuricemia is unrelated to his heart disease.
C His renal disease is likely caused by his increased serum urate levels.
D No treatment is needed.
E This patient should be discharged on low-dose allopurinol.
ANSWER:D
1641
A 25-year-old female visits her primary care physician after 3 years of intermittent
abdominal pain, peripheral neuropathy, and increasing episodes of anxiety and
hallucinations. The physician suspects acute intermittent porphyria. She can make this
diagnosis by doing which of the following tests?
A Sunlight administration with observation for rash
B Urine porphobilinogen (PBG) level during an attack
C Peripheral blood testing for an HFE C282Y mutation
D Urine PBG level when the patient is well
E Hypoglycemic provocation testing
ANSWER:B
1642 All the following are side effects of HMG-CoA reductase inhibitors (statins) except
A hepatitis

B myopathy
C dyspepsia
D headache
E pulmonary fibrosis
ANSWER:E
1643 Which of the following is consistent with a diagnosis of subacute thyroiditis?
A
A 38-year-old female with a 2-week history of a painful thyroid, elevated T4, elevated T3,
low TSH, and an elevated radioactive iodine uptake scan
B
A 42-year-old male with a history of a painful thyroid 4 months ago, fatigue, malaise, low
free T4, low T3, and elevated TSH.
C
A 31-year-old female with a painless enlarged thyroid, low TSH, elevated T4, elevated free
T4, and an elevated radioiodine uptake scan
D
A 50-year-old male with a painful thyroid, slightly elevated T4, normal TSH, and an
ultrasound showing a mass
E A 46-year-old female with 3 weeks of fatigue, low T4, low T3, and low TSH
ANSWER:B
1644
All of the following statements regarding hypoglycemia in diabetes mellitus are true except
A
Individuals with type 2 diabetes mellitus experience less hypoglycemia than those with type
1 diabetes mellitus.
B
Recurrent episodes of hypoglycemia predispose to the development of autonomic failure
with defective glucose counterregulation and hypoglycemia unawareness.
C
The average person with type 1 diabetes mellitus has two episodes of symptomatic
hypoglycemia weekly.
D
Thiazolidinediones and metformin cause hypoglycemia more frequently than sulfonylureas.
E From 2–4% of deaths in type 1 diabetes mellitus are directly attributable to hypoglycemia.
ANSWER:D
1645 Which of the following forms of contraception have theoretical efficacy of >90%?
A Condoms
B Intrauterine devices
C Oral contraceptives

D Spermicides
E All of the above
ANSWER:E
1646
A patient is seen in the clinic for follow-up of type 2 diabetes mellitus. Her hemoglobin A1C
has been poorly controlled at 9.4% recently. The patient can be counseled to expect all the
following improvements with improved glycemic control except
A decreased microalbuminuria
B decreased risk of nephropathy
C decreased risk of neuropathy
D decreased risk of peripheral vascular disease
E decreased risk of retinopathy
ANSWER:D
1647
A healthy 53-year-old man comes to your office for an annual physical examination. He has
no complaints and has no significant medical history. He is taking an over-the-counter
multivitamin and no other medicines. On physical examination he is noted to have a
nontender thyroid nodule. His thyroid-stimulating hormone (TSH) level is checked and is
found to be low. What is the next step in his evaluation?
A Close follow-up and measure TSH in 6 months
B Fine-needle aspiration
C Low-dose thyroid replacement
D Positron emission tomography followed by surgery
E Radionuclide thyroid scan
ANSWER:E
1648
During a routine checkup, a 67-year-old male is found to have a level of serum alkaline
phosphatase three times the upper limit of normal. Serum calcium and phosphorus
concentrations and liver function test results are normal. He is asymptomatic. The most likely
diagnosis is
A metastatic bone disease
B primary hyperparathyroidism
C occult plasmacytoma
D Paget’s disease of bone
E osteomalacia

ANSWER:D
1649
A 78-year-old man presents to your clinic and describes headaches and back pain. These
have been chronic complaints, and he thinks they are getting worse despite conservative
management. His wife believes he is experiencing some hearing loss. She describes how over
the past several months he needs to turn up the volume on the television and has a difficult
time talking to his children on the telephone. Physical examination is largely unremarkable;
straight leg raise is normal. Based on Rinne and Weber tests, the patient appears to have some
mild sensorineural hearing loss on the right side. A comprehensive chemistry panel shows an
elevated alkaline phosphatase of 170 U/L. Paget disease is now high on the differential.
Which of the following is true regarding this diagnosis?
A Family history is not predictive.
B Hearing loss is the most common symptom.
C Nuclear medicine bone scan is required for diagnosis.
D Serum calcium and phosphate are usually abnormal.
E The pelvis, skull, and vertebrae are most commonly affected.
ANSWER:E
1650 Which of the following is the most common sign of Cushing’s syndrome?
A Amenorrhea
B Hirsutism
C Obesity
D Purple skin striae
E Skin hyperpigmentation
ANSWER:C
1651
A patient receives CT of the head as part of a “virtual wellness physical exam” he received as
a gift certificate from his family. A 7-mm sellar mass “most consistent with a pituitary
adenoma” is reported, and he comes to your office very concerned that he has a life-
threatening brain tumor. A full panel of endocrine laboratory measurements reveals no
abnormalities, and besides his anxiety he reports feeling quite healthy. What is the next step
in management?
A Perform positron emission tomography/CT (PET-CT) to evaluate for metabolic activity.
B Reassure the patient that this finding is common and benign; take no action.
C Reassure and repeat laboratory measurements in 6 months.

D Reassure and repeat head imaging in 1 year.
E Refer to neurosurgery.
ANSWER:D
1652
Which of the following statements regarding hormone release from the anterior pituitary is
true?
A All hormones are released in a pulsatile manner.
B
Follicle-stimulating hormone (FSH) and luteinizing hormone (LH) release are suppressed
prior to puberty and after menopause.
C
Somatostatin acts in a feedback loop to inhibit adrenocorticotropin hormone (ACTH) release.
D Thyroid-stimulating hormone (TSH) is released primarily at night.
E
With the exception of prolactin, none of the anterior pituitary hormones are present in a
fetus until week 28 of gestation.
ANSWER:A
1653 All the following are features of lipoprotein lipase deficiency except
A low levels of plasma chylomicrons
B acute pancreatitis
C hepatosplenomegaly
D xanthomas
E autosomal recessive inheritance
ANSWER:A
1654
A 21-year-old female with a history of type 1 diabetes mellitus is brought to the emergency
room with nausea, vomiting, lethargy, and dehydration. Her mother notes that she stopped
taking insulin 1 day before presentation. She is lethargic, has dry mucous membranes, and is
obtunded. Blood pressure is 80/40 mmHg, and heart rate is 112 beats/min. Heart sounds are
normal. Lungs are clear. The abdomen is soft, and there is no organomegaly. She is
responsive and oriented × 3 but diffusely weak. Serum sodium is 126 meq/L, potassium is
4.3 meq/L, magnesium is 1.2 meq/L, blood urea nitrogen is 76 mg/ dL, creatinine is 2.2
mg/dL, bicarbonate is 10 meq/L, and chloride is 88 meq/L. Serum glucose is 720 mg/dL. All
the following are appropriate management steps except
A arterial blood gas
B intravenous insulin

C intravenous potassium
D 3% sodium solution
E intravenous fluids
ANSWER:D
1655 All the following are effects of hypercalcemia except
A diarrhea
B confusion
C polyuria
D a shortened QT interval
E nephrolithiasis
ANSWER:A
1656 All of the following are actions of parathyroid hormone except
A direct stimulation of osteoblasts to increase bone formation
B direct stimulation of osteoclasts to increase bone resorption
C increased reabsorption of calcium from the distal tubule of the kidney
D inhibition of phosphate reabsorption in the proximal tubule of the kidney
E stimulation of renal 1-α-hydroxylase to produce 1,25-hydroxycholecalciferol
ANSWER:B
1657 Which of the following statements regarding hypothyroidism is true?
A Hashimoto’s thyroiditis is the most common cause of hypothyroidism worldwide.
B
The annual risk of developing overt clinical hypothyroidism from subclinical
hypothyroidism in patients with positive thyroid peroxidase (TPO) antibodies is 20%.
C
Histologically, Hashimoto’s thyroiditis is characterized by marked infiltration of the thyroid
with activated T cells and B cells.
D A low TSH level excludes the diagnosis of hypothyroidism.
E
Thyroid peroxidase antibodies are present in less than 50% of patients with autoimmune
hypothyroidism.
ANSWER:C

1658
You are evaluating a patient for secondary causes of hypertension. The patient is a 39-year-
old woman who has hypertension despite using four different classes of antihypertensive
medications, including a diuretic at therapeutic doses. She mainly has diastolic hypertension
and has been found to have hypokalemia on several routine blood chemistry analyses. You
hold her diuretics and provide her with potassium supplementation for 14 days, after which
you find the serum potassium is in the normal range. She denies licorice ingestion. Plasma
renin activity is low. After days of saline loading, aldosterone levels are elevated. A CT scan
of the adrenal glands reveal no masses. An overnight dexamethasone suppression test shows
no aldosterone suppression. What is the most likely diagnosis?
A Conn’s syndrome
B Cortical nodular hyperplasia
C Glucocorticoid remediable aldosteronism
D Liddle’s syndrome
E Renin-secreting tumor
ANSWER:B
1659
A 17-year-old woman is evaluated in your office for primary amenorrhea. She does not feel
as if she has entered puberty in that she has never had a menstrual period and has sparse
axillary and pubic hair growth. On examination, she is noted to be 150 cm tall. She has a low
hairline and slight webbing of her neck. Her follicle-stimulating hormone level is 75
mIU/mL, luteinizing hormone is 20 mIU/mL, and estradiol level 2 pg/mL. You suspect
Turner syndrome. All of the following tests are indicated in this individual except
A buccal smear for nuclear heterochromatin (Barr body)
B echocardiogram
C karyotype analysis
D renal ultrasound
E thyroid-stimulating hormone (TSH)
ANSWER:A
1660
A 30-year-old male, the father of three children, has had progressive breast enlargement
during the last 6 months. He does not use any drugs. Laboratory evaluation reveals that both
LH and testosterone are low. Further evaluation of this patient should include which of the
following?

A
Blood sampling for serum glutamic-oxaloacetic transaminase (SGOT) and serum alkaline
phosphatase and bilirubin levels
B Measurement of estradiol and human chorionic gonadotropin (hCG) levels
C A 24-h urine collection for the measurement of 17 ketosteroids
D Karyotype analysis to exclude Klinefelter syndrome
E Breast biopsy
ANSWER:B
1661 Obesity is associated with an increased incidence of all the following except
A diabetes mellitus
B cancer
C hypertension
D biliary disease
E chronic obstructive lung disease
ANSWER:E
1662 Which of the following statements regarding Paget disease is true?
A 1% of patients over the age of 50 have evidence of Paget disease.
B A majority of patients with disease will experience symptoms at the time of diagnosis.
C The disease frequency has decreased over the past 20 years.
D There is a significant female predominance.
E While prevalent worldwide, Paget disease is most common in Asia.
ANSWER:C
1663
Which of the following statements is true about familial hypocalciuric hypercalcemia (FHH)?
A It is inherited in an autosomal recessive pattern.
B The cause is a defect in the parathyroid hormone receptor.
C Clinical symptoms first manifest in the third and fourth decades of life.
D Treatment is rarely necessary.
E Renal calcium reabsorption is more than 99%.
ANSWER:D
1664 Which of the following is true of Wilson disease?
A Early diagnosis is crucial as highly effective therapy is available.
B It is inherited in an autosomal dominant pattern.

C Serum copper levels are usually two to three times above normal.
D The frequency of disease in the general population is ~1%.
E The liver and pancreas are the most commonly affected organs.
ANSWER:A
1665
A 48-year-old female is undergoing evaluation for flushing and diarrhea. Physical
examination is normal except for nodular hepatomegaly. A CT scan of the abdomen
demonstrates multiple nodules in both lobes of the liver consistent with metastases in the liver
and a 2-cm mass in the ileum. The 24-h urinary 5-HIAA excretion is markedly elevated. All
the following treatments are appropriate except
A diphenhydramine
B interferon-α
C octreotide
D odansetron
E phenoxybenzamine
ANSWER:E
1666
While undergoing a physical examination during medical student clinical skills, this patient
develops severe flushing, wheezing, nausea, and light-headedness. Vital signs are notable for
a blood pressure of 70/30 mmHg and a heart rate of 135/min. Which of the following is the
most appropriate therapy?
A Albuterol
B Atropine
C Epinephrine
D Hydrocortisone
E Octreotide
ANSWER:E

1667
A 66-year-old Asian woman seeks treatment for osteoporosis. She fell and fractured her right
hip, requiring a surgical intervention 3 months ago. She was told while hospitalized that she
had osteoporosis but had not previously been evaluated for this. During the hospitalization,
she developed a deep venous thrombosis (DVT) with pulmonary embolus, for which she is
currently taking warfarin. She completed menopause at age 52. She is a former smoker,
quitting about 6 years ago. She has always been thin, and her current body mass index
(BMI) is 19.2 kg/m2. Her laboratory studies show a calcium of 8.7 mg/dL, phosphate 3 mg/
dL, creatinine 0.8 mg/dL, and 25-hydroxyvitamin D levels of 18 ng/mL (normal >30
ng/mL). A dual-energy x-ray absorptiometry scan of bone mineral density has a T-score of
–3.0. What is the best initial therapy for this patient?
A Calcitonin, 200 IU intranasally daily
B Calcium carbonate, 1200 mg, and vitamin D 400 IU daily
C Ethinyl estradiol, 5 µg, and medroxyprogesterone acetate 625 mg daily
D Raloxifene, 60 mg daily
E Risedronate, 35 mg once weekly
ANSWER:E
1668 All of the following would be expected to increase prolactin levels except
A chest wall trauma
B hyperthyroidism
C pregnancy
D renal failure
E sexual orgasm
ANSWER:B

1669
A 35-year-old male is referred to your clinic for evaluation of hypercalcemia noted during a
health insurance medical screening. He has noted some fatigue, malaise, and a 4-lb weight
loss over the last 2 months. He also has noted constipation and “heartburn.” He is
occasionally nauseated after large meals and has water brash and a sour taste in his mouth.
The patient denies vomiting, dysphagia, or odynophagia. He also notes decreased libido and
a depressed mood. Vital signs are unremarkable. Physical examination is notable for a clear
oropharynx, no evidence of a thyroid mass, and no lymphadenopathy. Jugular venous
pressure is normal. Heart sounds are regular with no murmurs or gallops. The chest is clear.
The abdomen is soft with some mild epigastric tenderness. There is no rebound or
organomegaly. Stool is guaiac-positive. Neurologic examination is nonfocal. Laboratory
values are notable for a normal complete blood count. Calcium is 11.2 mg/dL, phosphate is
2.1 mg/dL, and magnesium is 1.8 meq/dL. Albumin is 3.7 g/dL, and total protein is 7.0
g/dL. TSH is 3 µIU/mL, prolactin is 250 µg/L, testosterone is 620 ng/dL, and serum insulin-
like growth factor 1 (IGF-1) is normal. Serum intact parathyroid hormone level is 135
pg/dL. In light of the patient’s abdominal discomfort and heme-positive stool, you perform
an abdominal computed tomography (CT) scan that shows a lesion measuring 2 cm by 2 cm
in the head of the pancreas. What is the diagnosis?
A Multiple endocrine neoplasia (MEN) type 1
B MEN type 2a
C MEN type 2b
D Polyglandular autoimmune syndrome
E Von–Hippel Lindau (VHL) syndrome
ANSWER:A
1670
Your 60-year-old patient with a monoclonal gammopathy of unclear significance presents
for a follow-up visit and to review recent laboratory data. His creatinine is newly elevated to
2.0 mg/dL, potassium is 3.7 mg/dL, calcium is 12.2 mg/dL, low-density lipoprotein (LDL) is
202 mg/dL and triglycerides are 209 mg/dL. On further questioning he reports 3 months of
swelling around the eyes and “foamy” urine. On examination, he has anasarca. Concerned
for multiple myeloma and nephrotic syndrome, you order a urine protein/creatinine ratio,
which returns at 14:1. Which treatment option would be most appropriate to treat his lipid
abnormalities?

A Cholesterol ester transfer protein inhibitor
B Dietary management
C HMG-CoA reductase inhibitors
D Lipid apheresis
E Niacin and fibrates
ANSWER:C
1671
All of the following statements regarding asymptomatic adrenal masses (incidentalomas) are
true except
A All patients with incidentalomas should be screened for pheochromocytoma.
B
Fine-needle aspiration may distinguish between benign and malignant primary adrenal
tumors.
C In patients with a history of malignancy, the likelihood the mass is a metastasis is ~50%.
D The majority of adrenal incidentalomas are nonsecretory.
E The vast majority of adrenal incidentalomas are benign.
ANSWER:B
1672 Which of the following studies is most sensitive for detecting diabetic nephropathy?
A Serum creatinine level
B Creatinine clearance
C Urine albumin
D Glucose tolerance test
E Ultrasonography
ANSWER:C
1673
A 28-year-old woman seeks counseling before getting pregnant. She had a brother who died
at age 9 of the LeschNyhan syndrome, and she is a known carrier of the genetic defect. She
has no significant past medical history, and her husband has no significant family history.
Which of the following statements is true?
A Her children have no risk of disease since she is not symptomatic.
B
Her husband should be screened for carrying the genetic defect of Lesch-Nyhan syndrome.
C If she has a daughter, the child has a 50% chance of being a carrier.
D
If she has an affected son, starting him on allopurinol from birth will prevent clinical
manifestations of disease.

E She should start taking allopurinol to decrease her risk of gout and urate nephropathy.
ANSWER:C

!"#កទី ៣
Pediatrics 3246
No Question
1
You have just confirmed the diagnosis of cystic fibrosis in a 3-year-old child. The parents
are concerned about future pregnancies. You explain to them that the pattern of genetic
transmission of cystic fibrosis is
A autosomal dominant
B autosomal recessive
C X-linked recessive
D X-linked dominant
ANSWER:B
2
A 12-year-old boy just returned from Boy Scout Camp in Wisconsin. He now has fever,
myal- gia, and a 10-cm skin lesion which looks​​ like a target. You suspect Lyme disease, most
likely contracted by which of the following?
A ingestion of unripe fruit
B ingestion of spoiled fruit
C drinking of contaminated water
D the bite of a tick
ANSWER:D
3
A 2-year-old boy presents with extremity swelling and proteinuria and is found on urine
analysis. Minimal-change disease is suspected and you explain to the child’s parents that this
diagnosis
A is the most common cause of nephrotic syndrome in childhood
B has a peak incidence in children between 10 and 15 years of age
C usually results in end-stage renal dis- ease in 5–10 years
D is characterized by normal serum lipids and cholesterol
ANSWER:A
4
You have just prescribed phenytoin for a 12- year-old boy with new onset of epilepsy. Of
the following side effects, which is most likely to occur in this patient?
A lymphoma syndrome
B Raynaud phenomenon
C acute hepatic failure
D gingival hyperplasia
ANSWER:D

5
A term infant is born to a mother who has been using crack cocaine. This infant is at
increased risk for which of the following?
A anemia
B intrauterine growth retardation
C hypercalcemia
D macrosomia
ANSWER:B
6
You examine an 18-year-old male college stu- dent with a 5-day history of fever, sore
throat, and fatigue. Physical examination reveals an exudative tonsillitis and bilateral
enlarged and slightly tender posterior cervical lymph nodes. The spleen is palpable 3 cm
below the rib cage. Which agent is most likely responsible for this patient’s illness?
A Group A -hemolytic streptococcus
B Adenovirus
C Toxoplasma gondii
D Epstein-Barr virus
ANSWER:D
7
A term infant requires intubation in the deliv- ery room after aspiration of thick meconium
and is brought to the neonatal intensive care unit. Which of the following is the most likely
risk factor for meconium aspiration syndrome in this infant?
A chromosomal anamoly
B congenital heart disease
C cystic fibrosis
D fetal distress
ANSWER:D
8
A newborn infant with stigmata of Down syn- drome has a heart murmur. Which of the fol-
lowing cardiac lesions is most likely in this baby?
A hypoplastic left heart syndrome
B total anamolous venous return
C coarctation of the aorta
D atrioventricular defect
ANSWER:D
9
A mother of a 2-month-old wants more informa- tion about immunizations. Which of the
follow- ing statements regarding immunization against Haemophilus influenzae type b (Hib)
is correct?
A It is indicated for high-risk children only.

B Hib vaccine can be administered effec- tively as early as 2 months of age.
C Hib vaccine should not be given to children who have had allergic reactions to eggs.
D
Hib vaccine should not be administered to children with a history of reaction to DTaP
immunization.
ANSWER:B
10
Acellular pertussis vaccine is recommended for infants, children, adolescents, and adults.
Compared to the previously available whole cell vaccine, which of the following best
describes these products?
A they are more immunogenic
B they are less expensive
C they are associated with fewer side effects
D they require fewer doses
ANSWER:C
11
It is recommended that young infants should sleep in the supine rather than in the prone posi-
tion. This is based on data suggesting that the prone position is associated with an increased
incidence of which of the following?
A delayed eruption of the first deciduous teeth
B gastroesophageal reflux and aspiration
C macrognathia
D sudden infant death
ANSWER:D
12
A 3-month-old infant presents with poor growth and inadequate weight gain. There is no
history of vomiting or diarrhea. Except for the appear- ance of malnutrition and lack of
subcutaneous fat, the physical examination is normal. What is the most likely cause of this
child’s failure to thrive?
A renal disease
B a metabolic disorder
C tuberculosis
D a nonorganic cause
ANSWER:D
13
A 2-year-old child is being evaluated because the mother notes that her right eye has been
turning in. Physical examination documents strabismus with a right esotropia. Attempts to
visualize the fundi are unsuccessful, but it is noted that the red reflex is replaced by a yellow-
white pupillary reflex in the right eye. This child most likely has which of the following?

A retinitis pigmentosa
B retinoblastoma
C rhabdomyosarcoma
D severe hyperopia
ANSWER:B
14
A 2-year-old child is admitted because of weak- ness proceeding to coma. According to the
par- ents, he had been well until several hours prior to admission, when they noted diarrhea,
cough, wheezing, and sweating. Physical examination reveals a comatose child with diffuse
weakness and areflexia. Pupils are pinpoint and unrespon- sive. Examination of the chest
reveals generalized wheezing. Oral secretions are copious. Which of the following should
you administer at this time?
A adrenaline
B atropine
C cefotaxime
D methylprednisolone
ANSWER:B
15
A 3-week-old infant is admitted with vomiting of 5 days’ duration. Physical examination
reveals a rapid heart rate, evidence of dehydration, and ambiguous genitalia. Serum
electrolytes are Na+120 meq/L, K+7.5 meq/L, HCO3 12 meq/L, BUN 20 mg/dL. In addition
to intravenous fluid replacement with normal saline, administration of which of the
following would be most important?
A diuretics
B potassium exchange resin
C glucose and insulin
D hydrocortisone
ANSWER:D
16
A previously well 12-year-old girl presents to clinic because of painful swellings on the front
of the legs of about 3 days’ duration. Examination reveals tender erythematous nodules, 1–2
cm in diameter, on the extensor surfaces of the lower legs. The remainder of the physical
examination is unremarkable. Which of the following is most likely to confirm the cause of
this condition?
A stool smear and culture
B urine analysis and BUN
C throat culture
D slit-lamp examination of the eye

ANSWER:C
17
An 18-year-old boy presents with cough, chest pain, and low-grade nightly fevers of several
weeks duration. He has a 4-year history of smoking two packs of cigarettes per day. Chest x-
ray reveals a large mass in the mediastinum with extension into the right upper chest. Which
of the following is the most likely diagnosis?
A adenocarcinoma
B squamous cell carcinoma
C small cell carcinoma
D lymphoma
ANSWER:D
18
A 12-year-old child is seen because of a rash and severe headache which began 2 weeks after
returning from vacation in Massachusetts. The skin lesion began as a red macule on the
thigh, which gradually expanded over 1 week to reach approximately 15 cm in diameter
with red borders and central clearing. The lesion is slightly painful. A few days after the
onset of the skin manifesta- tion, the child developed severe headache, myal- gias,
arthralgias, and malaise. Low-grade fever was present. The mother recalls that the child was
bitten by a tick about 1 week prior to the onset of symptoms. This patient’s disorder is
probably best treated with which of the following?
A corticosteroids
B diphenhydramine
C methotrexate
D doxycycline
ANSWER:D
19
An 8-year-old child is hospitalized because of paroxysms of severe colicky abdominal pain
which does not radiate to the back or the groin. Physical examination is unremarkable except
for generalized abdominal tenderness. An exploratory laparotomy reveals an edematous
intestine without specific lesions. The appendix appears normal but is removed.
Postoperatively the abdominal pain persists, and hematuria develops. Values for BUN and
creatinine are normal. On the second postoperative day, tender swelling of both ankles and
knees is noted. Which of the following additional find- ings would most likely be present in
this child?
A shock
B meningitis
C hepatitis
D a purpuric rash

ANSWER:D
20
A 10-year-old boy has been having episodes of repetitive and semipurposeful movements
of the face and shoulders. The parents believe these movements are worse when the child is
under emotional stress. They also volunteer that they have never noted the movements while
the patient is asleep. The movements have been present for more than 6 months. The parents
are now especially concerned because the child has developed repetitive episodes of throat
clearing and snorting. Physical and neurologic examinations are entirely normal. During the
examination you note that the child has some blinking of the right eye, twitching of the right
face, and grimacing. You ask him to stop these movements, and he is temporarily suc-
cessful in doing so, but the movements recur. The home situation, social history, and child’s
devel- opment and social adjustment appear normal. A head CT scan is normal. Of the
following, which would be the most appropriate next step?
A order an electroencephalogram
B prescribe carbamazepine
C prescribe corticosteroids
D prescribe haloperidol
ANSWER:D
21
A 3-month-old infant is hospitalized because of recurrent right focal seizures that are now
gen- eralized in nature. Birth and perinatal history are unremarkable. You note that the child
has a flat, purplish-red skin lesion on the left side of the face extending onto the forehead.
The remainder of the examination including a com- plete neurologic examination is within
normal limits. The results of a lumbar puncture are normal. You order a CT scan of the head
and anticipate seeing which of the following?
A agenesis of the corpus callosum
B a porencephalic cyst
C gyriform calcifications
D hydrocephalus
ANSWER:C

22
On routine examination of the children of a migrant farm worker, you notice that a 12-year-
old child who has received little previous medical care is short and mentally retarded.
Physical exam- ination reveals that the liver is enlarged to 5 cm below the right rib cage, and
the spleen is enlarged 6 cm below the left rib cage. Lumbodorsal kypho- sis is prominent.
The child has a peculiar facies with thick lips and a large tongue. Attempts to visualize the
retina are unsuccessful because of clouding of the corneas. You expect that examination of
this child’s urine will reveal which of the following?
A dermatan and heparan sulfate
B galactose
C mannose
D the odor of maple syrup
ANSWER:A
23
A 6-year-old girl presents with unilateral non- painful, nonsuppurative conjunctivitis and
preauricular lymphadenitis. What is the most likely causative organism?
A Mycobacterium avium
B Bartonella henselae
C Adenovirus
D Staphylococcus aureus
ANSWER:B
24
You suspect the diagnosis of Werdnig-Hoffman disease in an infant with severe hypotonia.
Which other finding will support this diagnosis?
A normal deep tendon reflexes
B seizures
C fasciculations of the tongue
D recurrent fevers
ANSWER:C
25
A 16-month-old girl presents with acute onset of truncal ataxia with vomiting, nystagmus,
and dysarthria. She is afebrile and has no nuchal rigidity. Which of the following historical
items would help you to identify a cause for these symptoms?
A elevated lead level at the age of one year
B febrile seizure episode at the age of 13 months
C sore throat with blisters on palate 3 weeks ago
D febrile illness with rash 2 months ago
ANSWER:C

26
You follow a patient with craniosynostosis and congenital malformations of the head and
face in your clinic. What other congenital malforma- tions will you most likely find in this
patient?
A trachea and esophagus malformations
B heart malformations
C genitourinary tract malformations
D extremity malformations
ANSWER:D
27
A 12-year-old boy has migratory arthritis with red, warm, and swollen joints. He has
serologic evidence of recent group A streptococcal infec- tion. Arthritis in this condition is
characterized by which of the following?
A usually nonpainful
B heals without deformity
C appears after the fever subsides
D seen only in patients with concurrent carditis
ANSWER:B
28
A 2-month-old infant has severe dyspnea and cyanosis. Chest roentgenogram reveals
minimal cardiomegaly and a diffuse reticular pattern of the lung fields. Which of the
following best explains these findings?
A acute viral myocarditis
B hypoplastic left heart syndrome
C pulmonary artery atresia
D total anomalous pulmonary drainage with venous obstruction
ANSWER:D
29
You suspect the diagnosis of a brain tumor in a 2-year-old girl with a recent history of
ataxia, slurred speech, and early morning vomiting. Which statement about childhood brain
tumors is true?
A Most are located in the midline and/or below the tentorium cerebri.
B Brain tumors are a rare type of cancer in childhood.
C Signs of increased intracranial pressure are rare on presentation.
D Seizures are the presenting complaint in most cases.
ANSWER:A
30
An 8-month-old child has vomiting and scream- ing episodes for 12 hours. Physical
examination reveals a sausage-shaped mass in the right upper quadrant. Which of the
following would be most useful?

A passage of nasogastric tube
B examination of a stool specimen for ova and parasites
C blood culture
D barium enema study
ANSWER:D
31
A 4-year-old boy has failed to grow and has evidence of exocrine pancreatic insufficiency.
What is the most likely cause for this?
A cystic fibrosis
B biliary atresia
C Swachman-Diamond syndrome
D congenital absence of the pancreas
ANSWER:A
32
A 5-year-old girl presents with fever and headache. Imaging of the brain reveals a ring-
enhancing lesion. Which of the following is the most likely underlying condition in this
child?
A chronic renal failure
B idiopathic or familial epilepsy
C congenital cyanotic heart disease
D chronic or recurrent tonsillitis
ANSWER:C
33
A 5-year-old girl diagnosed with pauciarticular juvenile idiopathic arthritis has a positive anti-
nuclear antibody test. Which of the following would most likely be found in this patient?
A pericarditis
B nephritis
C uveitis
D splenomegaly
ANSWER:C
34
A 2-year-old African-American child presents with painful swelling of the hands and feet.
Laboratory evaluation reveals hemoglobin of 9 g/dL with white blood cell count of 11,500
and platelet count of 250,000. Which additional laboratory test will support your diagnosis?
A skeletal survey
B VDRL testing
C bone marrow aspiration

D hemoglobin electrophoresis
ANSWER:D
35
A 2-week-old infant presents with apnea. The infant was born at term after an
uncomplicated pregnancy. The mother of this baby had rhin- orrhea and cough that started 3
weeks ago and now she has a severe persistent cough with post-tussive emesis. Which
treatment should be initiated?
A ceftriaxone
B amoxicillin
C azithromycin
D vancomycin
ANSWER:C
36
An 18-month-old toddler has microcytic anemia. Which dietary history finding best explains
this?
A pica
B lack of fresh vegetables in the diet
C intake of inadequate amounts of fruit juice
D intake of large amounts of unmodified cow’s milk
ANSWER:D
37
A 2-year-old toddler has a large abdominal mass and pancytopenia. Which of the follow- ing
diagnoses would most likely be established by bone marrow aspiration?
A hepatoblastoma
B neuroblastoma
C renal cell carcinoma
D rhabdomyosarcoma
ANSWER:B
38
A 14-month-old boy has a 4-month history of intermittent diarrhea. He frequently has
explo- sive bowel movements containing food parti- cles. He is growing well, is otherwise
healthy, and has a normal physical examination. What should be the next step?
A reassurance of parents
B stool culture
C total serum qualitative immunoglobulin measurement
D qualitative fecal fat
ANSWER:A

39
A child with polyosteotic fibrous dysplasia of the bones and abnormal skin pigmentation is
diagnosed with McCune-Albright syndrome. What other problem is this patient most likely
to develop?
A anemia
B deafness
C precocious puberty
D multiple neurofibromas
ANSWER:C
40
A normal 6-month-old infant has a continuous cardiac murmur and bounding peripheral
pulses. What step should be taken next?
A karyotype evaluation
B surgical or catheter correction of the defect
C life-long endocarditis prophylaxis for at- risk procedures
D repeating examination at the age of 12 months
ANSWER:B
41
A newborn has delayed passage of meconium stools and barium enema radiograph shows
dilated proximal colon and small obstructed distal colon. What should be the next diagnostic
test?
A abdominal CT-scan
B stool studies
C rectal suction biopsies
D sweat chloride testing
ANSWER:C
42
An 8-year-old boy is referred for new-onset seizures. Which of the following would mostly
support a diagnosis of complex partial (psy- chomotor) seizures?
A normal mental state, consciousness, and responsiveness during seizure
B a brief tonic-clonic phase
C automatisms
D three-per-second spike-and-wave pat- tern on EEG
ANSWER:C
43
A 14-year-old female has progressive headaches. Examination shows bilateral papillary
edema. CT-scan of the brain is normal. What should be the next diagnostic test?
A lumbar puncture with opening pressure
B MRI of the brain
C orbital CT-scan

D urine toxicology screen
ANSWER:A
44 An infant is born to a mother who is HBsAg positive. What should be the next step?
A Check hepatitis B serology on infant and give hepatits B immune globulin if indicated.
B Give infant hepatitis B immune globulin.
C Vaccinate infant with hepatitis B vaccine.
D Give infant hepatitis B immune globulin and hepatitis B vaccine.
ANSWER:D
45
A 38-week infant is born to a mother with ges- tational diabetes. Birth weight is 4255 g.
What would you expect to see most commonly in this infant?
A neural tube defect
B small left colon syndrome
C cardiomegaly
D hydronephrosis
ANSWER:C
46
A newborn has been diagnosed with aniridia. Which of the following tests should be per-
formed on this patient?
A chest radiograph
B alpha-fetoprotein measurement
C renal function testing
D renal ultrasound
ANSWER:D
47
A 12-year-old girl develops jaundice, progres- sive tremors, and emotional lability. You are
most likely to find which of the following during physical examination?
A head circumference greater than 95th percentile
B brown discoloration of the limbic region of the cornea
C bilateral conductive hearing loss
D generalized lymphadenopathy
ANSWER:B
48
A 12-month-old infant is unable to sit by herself and parents have noticed an exaggerated
star- tle response. What are you most likely to find on physical examination?
A holosystolic murmur
B absent knee-jerk reflex
C syndactyly
D cherry red macular spot

ANSWER:D
49
An infant has been diagnosed with congenital hypoparathyroidism. What are you most likely
to find on evaluation?
A microcephaly
B hyponatremia
C hyperkalemia
D candidiasis
ANSWER:D
50
A 14-month-old boy has severe eczema, recur- rent sinus and ear infections, and thrombocy-
topenia. What is the inheritance pattern of this disorder?
A X-linked
B autosomal dominant
C autosomal recessive
D random mutation
ANSWER:B
51
A 12-year-old girl has had progressive muscle weakness over the past weeks. She has also
devel- oped an erythematous, scaly rash on the face, arms and thighs, and a lacy rash on her
upper eyelids. What is the next best laboratory study?
A rheumatoid factor
B erythrocyte sedimentation rate (ESR)
C urine analysis
D serum creatinine kinase
ANSWER:D
52
A 12-month-old girl has been diagnosed with transient erythroblastopenia of childhood
(TEC). Which statement about this disorder is correct?
A Corticosteroid treatment is usually beneficial.
B Red blood cell transfusions may be necessary.
C Hepatosplenomegaly is usually present.
D Spontaneous recovery is uncommon.
ANSWER:B
53
A 2-year-old girl is listless and pale. You obtain a complete blood count and find that the
patient has severe megaloblastic anemia. What additional history explains this?
A eats only organically grown products
B drinks exclusively goat milk
C has required phototherapy in neonatal period

D has required multiple antibiotics for middle ear infections
ANSWER:B
54
A 12-month-old child has had poor weight gain. The child started to have loose stools at the
age of 8 months and has a very poor appetite. On examination, you see a clingy, irritable
child with very little subcutaneous fat and a protu- berant abdomen. What is the next best
test?
A IgA-endomysial antibody
B urine analysis
C sweat chloride
D quantitative immunoglobulins
ANSWER:A
55 A 7-year-old girl develops secondary nocturnal enuresis. What is the next best study?
A renal ultrasound
B voiding cystourethrogram
C abdominal radiograph
D urine analysis
ANSWER:D
56
A 5-year-old girl suffers from a second episode with meningococcal meningitis. What is the
best next laboratory study?
A quantitative immunoglobulin levels
B T-cell subset analysis
C CH50
D quantitative nitroblue tetrazolium test
ANSWER:C
57
A 2-year-old child is referred to you for evalu- ation of child abuse. On physical
examination, you find a pale child with diffuse petechiae and bilateral proptosis with
periorbital ecchy- moses. Which of the following statements is true about this condition?
A Age at presentation correlates directly with survival.
B A full skeletal survey should be obtained next.
C Hematuria is a common finding.
D Spontaneous regression has occurred in some children.
ANSWER:D
58
Which study is the most important to obtain in a 2-year-old child with Beckwith-Wiedemann
syndrome and an abdominal mass?
A hepatobiliary scintigraphy

B upper gastrointestinal endoscopy
C urine catecholamine levels
D serum alpha-fetoprotein level
ANSWER:D
59
A 9-year-old African-American child presents with anemia and stroke. What is the most
likely finding with hemoglobin electrophoresis?
A HbS 45%
B HbA 65%
C HbA2 15%
D HbF 15%
ANSWER:D
60
A 2-year-old boy from Sudan has failure to thrive, chronic diarrhea, and severe candidiasis.
You suspect HIV infection. Which of the fol- lowing organisms would most likely be found
on stool examination?
A rotavirus
B Salmonella
C Cryptosporidium species
D Giardia
ANSWER:C
61
A 5-year-old boy has severe pharyngitis and culture is positive for group A streptococci. Of
the following suppurative and nonsuppura- tive complications of group A streptococcal
pharyngitis and skin infections, which is asso- ciated only with pharyngeal infections?
A scarlatina
B toxic shock syndrome
C rheumatic fever
D necrotizing fasciitis
ANSWER:C
62
A 7-year-old boy has abdominal pain and a rash that started several days ago. On
examination, you notice a palpable purpuric rash over his calves and buttocks with swelling
of both ankles. Abdominal examination is unremark- able. What is the most likely laboratory
finding?
A decreased platelet count
B hypochromic microcytic anemia
C elevated blood urea nitrogen and creatinine

D normal clotting parameters
ANSWER:D
63
A mother with mild mental retardation has a 10-year-old son with severe mental retardation.
The boy is tall, has a long face with prominent jaw and large ears. Which statement about his
condition is true?
A Premutation carriers generally have phenotypic manifestations.
B Inheritance is autosomal dominant.
C It is the most common form of inherited mental retardation.
D Affected males typically have microorchidism.
ANSWER:C
64
A 4-month-old female infant has generalized hypotonia, small ears, inner epicanthal folds,
clinodactyly, and wide-spaced first and second toes. For which of the following problems is
she most at risk?
A leukemia
B patent ductus arteriosus
C seizure disorder
D hearing loss
ANSWER:D
65
What is the most important test to obtain a diagnosis in a 14-year-old girl with primary
amenorrhea, and short stature, who has a his- tory of repaired coarctation of the aorta in
infancy?
A sweat chloride testing
B karyotyping
C fluorescent in situ hybridization (FISH) of chromosome 22q11
D pelvic ultrasonography
ANSWER:B
66
What are the blood requirements for transfusion of a patient with hypocalcemia, heart defect,
and recurrent infections?
A leukocyte depleted
B HLA matched
C CMV negative
D irradiated
ANSWER:D
67
What hematologic abnormality should you suspect in a newborn with bilateral absence of
radii?

A thrombocytopenia
B anemia
C neutropenia
D pancytopenia
ANSWER:A
68
What laboratory abnormality do you expect to find in a 3-year-old child with severe mental
retardation, coarse facies, hazy corneas, hepatosplenomegaly, and multiple skeletal x-ray
abnormalities?
A increased serum homocystine
B deficiency of leucocyte hexosaminidase A
C urinary excretion of dermatan sulfate and heparan sulfate
D deficiency of liver glucose-6-phosphatase activity
ANSWER:C
69
A 3-month-old infant is brought to the hospital because of altered mental status changes.
Examination shows a sleepy baby who is difficult to arouse. Fundoscopic examination shows
reti- nal hemorrhages. Examination otherwise is unre- markable. What is the best next
diagnostic test?
A spinal tap
B hematology profile with smear review
C CT-scan of the head
D skull radiographs
ANSWER:C
70
A previously healthy 5-year-old girl has acute onset of edema and oliguria. Laboratory
studies reveal hypoalbuminemia and hypercholes- terolemia. Which of the following is the
major cause for mortality with this condition?
A bacterial peritonitis
B acute renal failure
C hyperlipidemia
D congestive heart failure
ANSWER:A
71
A previously healthy 9-year-old boy has had diarrhea for 6 weeks that started after he
returned from camp. He has had anorexia, abdominal cramps with abdominal disten- sion,
and a 4 pound weight loss. His stools are large, foul-smelling but do not contain blood. What
is the best treatment?
A avoidance of lactose in diet

B gluten-free diet
C oral prednisone
D metronidazole
ANSWER:D
72
A 16-year-old adolescent has morbid obesity. Which of the following conditions is the most
common cause for pulmonary insufficiency in obese adolescents?
A pneumothorax
B gastric esophageal reflux disease
C congestive heart failure
D sleep apnea
ANSWER:D
73
The parents of an 8-year-old boy are concerned about their son’s short stature. What should
be the most important next step?
A measurement of body mass index
B determination of genital maturation stage
C bone age measurement
D determination of height velocity
ANSWER:D
74
A 7-year-old boy has chronic fecal soiling but only rarely has a voluntary bowel movement.
What is the most common explanation for his problem?
A Hirschsprung disease
B functional fecal retention
C hypothyroidism
D lead poisoning
ANSWER:B
75
A 6-month-old boy is found to have very low levels of IgG, IgM, and IgA. Which of the fol-
lowing organisms is most likely to cause prob- lems in this patient?
A enterovirus
B herpesvirus
C Shigella
D Escherichia coli
ANSWER:A
76
An infant is born to a mother who acquired primary CMV infection during pregnancy. What
will be the most likely finding in this infant?
A hepatosplenomegaly and jaundice

B subclinical infection
C microcephaly and intrauterine growth retardation
D sensorineural hearing loss
ANSWER:B
77
A healthy adolescent is found to have elevated blood pressure on several occasions. Which
statement is correct?
A Obesity is rarely associated with hyper- tension among adolescents.
B Essential hypertension is the most com- mon cause of hypertension among adolescents.
C The incidence in Caucasian adolescents is twice that of African-American adolescents.
D Most pediatric patients with hyperten- sion are symptomatic.
ANSWER:B
78
A 14-year-old girl has irregular menstrual bleeding since menarche 1 year ago. What is the
most common cause for this?
A immature hypothalamic-pituitary- ovarian axis
B polycystic ovarian syndrome
C blood dyscrasia
D systemic illness
ANSWER:A
79
A 3-year-old girl has a mild febrile illness with mild URI symptoms. She has an
erythematous rash on both cheeks. Her pregnant mother had arthralgias of the hands wrists,
knees, and ankles a week ago. What should be the next action?
A Closely monitor the child’s sibling who has spherocytosis.
B Exclude the child from daycare until rash has resolved.
C Exclude the pregnant daycare providers until no further cases are diagnosed for 2 weeks.
D Give the mother IVIG.
ANSWER:A
80 A 14-year-old boy has an acutely painful and swollen scrotum. What should be the next step?
A fine needle aspiration
B bone marrow aspiration
C surgical exploration
D oral antibiotics
ANSWER:C

81
A 16-year-old boy presents with fever, fatique, and sore throat. Examination reveals exuda-
tive pharyngitis, generalized lymphadenopa- thy, and mild splenomegaly. Laboratory studies
show elevated WBC count with presence of atypical lymphocytes. What is the best action?
A no participation in contact sports for next 2–4 weeks
B 2-week treatment with oral prednisone
C 2-week treatment with oral acyclovir and prednisone
D 10 days of oral penicillin
ANSWER:A
82
A 4-year-old girl had bloody diarrhea for sev- eral days. One week later she develops perior-
bital edema and fatique. What is the most likely laboratory finding?
A elevated PT and PTT
B positive ANA
C decreased C3 and C4
D anemia and thrombocytopenia
ANSWER:D
83
An infant with failure to thrive has rectal pro- lapse. What test will most likely provide the
diagnosis?
A abdominal CT-scan
B rectal biopsies
C liver function testing
D sweat chloride test
ANSWER:D
84
A 16-month-old toddler has painless rectal bleeding. His stools have currant jelly consis-
tency. Physical examination of the patient, including rectal examination, is completely
normal. A routine barium study is normal. Which of the following is true for this condition?
A Plain abdominal radiographs are usu- ally diagnostic.
B It is the most common congenital gas- trointestinal anomaly.
C The abnormality typically is located within 1 cm of the ileocecal valve.
D Most common presentation is partial or complete bowel obstruction.
ANSWER:B
85
A 5-day-old infant boy is jaundiced. The total bilirubin level is 14 mg/dL and the direct
bilirubin is 4 mg/dL. Which of the following tests is the most appropriate?
A blood type and direct antibody test on the infant’s blood

B blood type and direct antibody test on the mother’s blood
C urine analysis and culture
D hepatitis serology
ANSWER:C
86
An 18-year-old boy presents with acute severe chest pain. EKG
an acute myocardial infarction. Cardiac cather- ization reveals a coronary artery aneurysm
with thrombosis. Which constellation of symp- toms in his past could explain this finding?
A conjunctivitis, fever, cervical lymphadenopathy
B meningitis, conjunctivitis, pallor
C cervical lymphadenopathy, hepatitis, rash
D fever, irritability, pancreatitis
ANSWER:A
87
A 2-year-old girl has persistent seborrheic der- matitis in the diaper area. In addition she has
chronically draining infected ears. In which location do bony lesions most often occur in
patients with this disorder?
A ribs
B femur
C sternum
D skull
ANSWER:D
88
A 3-year-boy with severe hypotonia and mild mental retardation is severely obese. He is
obsessed with eating and does not have a sense of satiation. What abnormality will you most
likely find?
A macrocephaly
B height greater than 95%
C large hands and feet
D micropenis and cryptorchidism
ANSWER:D
89
A 3-year-old girl develops petechiae and bruises on her extremities while she is recov- ering
from a cold. She is brought to medical attention after she has a transient nosebleed. Physical
examination shows a toddler with widespread petechiae and bruising who oth- erwise looks
healthy. What is the best treat- ment for this patient?
A plasmapheresis
B intravenous gammaglobulin

C vincristine and methotrexate
D intravenous antibiotics
ANSWER:B
90 A 3-month-old infant has persistent stridor. What is the most likely cause?
A vascular ring
B laryngomalacia
C tracheomalacia
D laryngeal cleft
ANSWER:B
91
A newborn girl with ambiguous genitalia has severe vomiting with weight loss. What will be
the most likely finding?
A decreased urinary excretion of 17-ketosteroids
B decreased plasma dehydroepiandros- terone sulfate level
C increased plasma cortisol
D hyponatremia and hyperkalemia
ANSWER:D
92
A 9-year-old has hematuria and an increased serologic titer to antistreptolysin O (ASO).
What will be the most likely finding?
A decreased serum C3
B IgA deposits in kidney biopsy
C decreased serum albumin
D decreased urinary protein/creatine ratio
ANSWER:A
93
A 6-year-old girl with short stature has webbing of the neck, a low posterior hairline, a
broad chest, and cubitus valgus. Which organ is affected most frequently in patients with this
syndrome?
A heart
B kidneys
C ovaries
D thyroid
ANSWER:C
94
A 15-year-old female has a 1 year history of secondary amenorrhea. She is an avid gymnast
and has an intese fear of becoming fat. Her weight is at 80% of ideal body weight. For what
long-term irreversible complication is this patient at greatest risk?
A cardiac arrythmias

B hypothyroidism
C visual impairment
D osteoporosis
ANSWER:D
95
A newborn boy was diagnosed prenatally with bilateral hydronephrosis, distended bladder,
and oligohydramnios. What will be the most likely diagnosis?
A urethral strictures
B anterior urethral valves
C prune-belly syndrome
D posterior urethral valves
ANSWER:D
96
A healthy 2-day-old infant has multiple, firm, yellow-white papules on an erythematous base
that are widely dispersed over much of the skin. What will be the most likely microscopic
finding?
A multinucleated giant cells
B eosinophilic infiltrate
C cytoplasmic inclusion bodies
D gram-positive cocci in clusters
ANSWER:B
97
A 13-month-old toddler has a tibia fracture after an insignificant fall. Other family mem-
bers have blue sclerae and recurrent fractures in childhood. For what other problem is this
toddler at increased risk?
A presenile hearing loss
B mental retardation
C seizures
D recurrent pneumonia
ANSWER:A
98
A 4-week-old male infant has vomiting and a hypochloremic metabolic alkalosis. What is
the next best study?
A urine organic acids
B urine 17-hydroxy progesterone
C stool culture
D abdominal ultrasound
ANSWER:D

99
A newborn boy has deficiency of the abdomi- nal muscles and urinary tract abnormalities.
What other anomaly will you most likely find?
A imperforate anus
B undescended testes
C mental retardation
D congenital heart disease
ANSWER:B
100
A 15-month-old boy has strabismus and a white pupillary reflex. Ophthalmologic exam-
ination reveals a white retinal mass. This patient is at increased risk for development of which
other tumor?
A leukemia
B lymphoma
C osteosarcoma
D Ewing sarcoma
ANSWER:C
101
A previously healthy 3-month-old infant devel- ops generalized weakness with difficulty in
sucking, swallowing, and crying, and labored breathing. No fever is present. Which study
will most likely provide the diagnosis?
A stool culture
B blood culture
C head CT-scan
D nerve conduction velocity testing
ANSWER:A
102
A 3-year-old boy presents with acute right leg pain and a limp. There is no history of
trauma. He holds his right hip in external rotation and flexion and he has mild restriction of
range of motion. He appears otherwise well and is afebrile. His WBC is normal and ESR is 25
mm/h. What is the best treatment option at this time?
A intravenous antibiotics
B surgical drainage of the right hip joint
C anti-inflammatory drugs and bed rest
D oral antibiotics
ANSWER:C
103
You have followed a 7-month-old infant who has failed to gain weight. Birth weight was
3250 g; the child currently weighs 5.5 kg. In your office, the baby takes an 8-oz bottle with
ease and does not vomit. What is the next best step?

A placement of nasogastric feeding tube
B hospitalization of the child with unlim- ited feedings
C contact child protective services for placement in foster care
D a barium swallowing study
ANSWER:B
104
A patient with streptococcal pharyngitis devel- ops tender red bumps along her entire tibia.
What is the most likely diagnosis?
A sarcoidosis
B cellulitis
C thrombophlebitis
D erythema nodosum
ANSWER:D
105
A 2-year-old child develops apnea, cyanosis, and loss of consciousness with repeated
generalized clonic jerks after being scolded by his mother. On examination, the child appears
completely normal. What is the best treatment option?
A tegretol
B valproic acid
C antiarrhythmics
D counseling of parents
ANSWER:D
106
A 16-year-old high school soccer player com- plains of chronic knee pain that has not been
associated with an injury. The pain is worse upon going upstairs and after sitting for pro-
longed periods. The only abnormal finding on examination is peripatellar tenderness. What
is the best next action?
A arthroscopy
B thigh strengthening exercise
C knee brace
D immobilization with cast
ANSWER:B
107
You evaluate an 8-year-old girl with hyperactiv- ity and inattentiveness. Which of the
following manifestations is required to make a diagnosis of attention-deficit hyperactivity
disorder?
A occurrence before the age of 10 years
B concurrent learning disability
C impulsivity

D history of birth trauma
ANSWER:C
108
You counsel the new parents of a baby boy with hypospadias about circumcision. Which
informa- tion will you most likely share with the parents?
A There is clearly an increased risk for penile cancer in uncircumcised males.
B Urinary tract infections are 10–15 times more common in uncircumcised infants.
C Circumcision reduces the risk of sexually transmitted diseases.
D Complications following circumcision are very rare.
ANSWER:B
109
A 14-year-old boy has had several measurements of blood pressure. His systolic blood
pressure has been above 99th percentile for age and dias- tolic blood pressure has ranged
between 90th to 94th percentile. What should you advise this young man?
A Complete restriction of exercise is necessary.
B Patient can participate in competitive sports if there are no signs of target organ damage.
C
Participation in competitive sports need to be restricted until hypertension is under adequate
control.
D Complete restriction of exercise with exception of isometric activities is necessary.
ANSWER:C
110
An infant is diagnosed with a given disorder below. Which of these poses the greatest recur-
rence risk for this patient’s future siblings?
A Hirschsprung disease
B cystic fibrosis
C ventricular septal defect
D trisomy 21
ANSWER:B
111
A term infant with microcephaly, jaundice, and thrombocytopenia is thought to have
congeni- tal CMV infection. Your attending physician notes though that 1.5% of all
newborns may have asymptomatic congenital cytomegalovirus infection. Which of the
following is the most commonly reported sequelae of such infections?
A chorioretinitis
B sensorineural hearing loss
C thrombocytopenia
D poor growth
ANSWER:B

112
A 2-year-old infant has acquired sensorineural hearing loss. His mother is asking what the
most likely cause could be. What is the least likely etiology of acquired sensorineural hear-
ing loss in a toddler?
A hyperbilirubinemia
B congenital infection
C aminoglycoside therapy
D neuroblastoma
ANSWER:D
113
You are a practicing pediatrician in a state where cystic fibrosis is not a routine part of
newborn screening. Which of the following symptoms in a newborn infant would prompt
you to test for cystic fibrosis?
A pneumonia
B intrauterine growth retardation
C meconium ileus
D wheezing
ANSWER:C
114
An infant is born precipitously to a mother without prenatal care and you are requested to
determine the infant’s gestational age. Of the following, which physical finding is most
indicative of a full-term infant?
A veins and tributaries are seen over the abdomen
B long lanugo is present on the back
C palpable breast tissue of less than 1 cm
D pitting edema over the tibia
ANSWER:C
115
A 1-week-old term newborn is in your office for a well-child assessment, physical
examination findings are consistent with oral candidiasis (thrush). Which of the following is
a correct statement regarding thrush in a term newborn?
A responds well to topical therapy with nystatin
B requires systemic therapy with amphotericin B
C requires both topical (nystatin) and sys- temic (amphotericin) therapy
D requires investigation to rule out 22q11 deletion syndrome
ANSWER:A
116
During the delivery room resuscitation of a vigorous term newborn, which of the following
should be performed first?
A Verify the airway is clear, dry, and stim- ulate the infant.

B The heart rate should be auscultated.
C Breath sounds should be auscultated.
D The mouth and trachea should be suctioned.
ANSWER:A
117
A low-risk newborn infant has pathologic unconjugated hyperbilirubinemia, which is
appopriately diagnosed within the first few hours of life due to the astute observation of
rapid, progressive jaundice by the nurse. She is asking why the infant was not jaundiced
imme- diately after birth. Which of the following best describes the major route for excretion
of biliru- bin in the fetus in utero?
A via the kidney
B transplacental passage
C degradation to biliverdin
D reincorporation into hemoglobin
ANSWER:B
118
You are the attending physician for a newborn infant with hemolytic jaundice. The mother
did not receive prenatal care with this pregnancy or her prior pregnancy. The direct Coombs
test is positive. The mother’s blood type is A- and the baby’s blood type is O+. Her first baby
did not have hemolytic jaundice. What is the most likely cause of the hemolytic jaundice?
A ABO incompatibility
B Toxoplasmosis
C Rh incompatibility
D rubella
ANSWER:C
119
The blood bank has received an order for an intrauterine transfusion. A fetus with which of
the following would most likely require trans- fusion prior to birth?
A erythroblastosis fetalis
B sickle cell anemia
C spherocytosis
D fetal distress and bradycardia
ANSWER:A
120
The obstetrician performing the intrauterine transfusion is counseling the mother regard- ing
complications of this procedure. Which of the following is the most common complication
of intrauterine transfusion?
A a transfusion reaction (mismatch)

B graft-versus-host reaction
C premature onset of labor
D acquired immunodeficiency syndrome (AIDS)
ANSWER:C
121
During a routine prenatal visit a mother states she has been reading about kernicterus. She is
very concerned that her infant will develop kernicterus if she provides breast milk for the
infant since she has also read that some breast- fed infants have high bilirubin levels. Which
of the following is most predictive for the devel- opment of kernicterus?
A hyperbilirubinemia within the first 24 hours of life
B peak conjugated bilirubin level
C peak unconjugated bilirubin level
D duration of hyperbilirubinemia
ANSWER:C
122
A medical student in your office is assessing a 1-week-old term infant. The mother has been
exclusively breast-feeding and the student is not sure how to interpret the change in the
infant’s weight compared to the birth weight. Which of the following is the expected weight
flux for an infant in the first week of life?
A gain approximately 30 g/day
B gain approximately 60 g/day
C neither gain nor lose weight
D lose approximately 5%–10% of its birth weight
ANSWER:D
123
A 3-day-old infant is requiring phototherapy for hyperbilirubinemia. Which of the following
risk factors best predicts the occurrence of ABO isoimmune hemolytic disease in a newborn?
A first pregnancy
B more than four pregnancies
C prior Rh disease
D maternal blood type is O and infant is type A
ANSWER:D
124
The patient in Question 15 was hospitalized for 2 weeks, treated with phototherapy for 12
days, and required a red cell transfusion during the hospitalization. Which of the following is
the most common serious late clinical mani- festation of ABO disease?
A kernicterus
B congestive heart failure

C gallstones
D bilirubinuria
ANSWER:B
125
Prevention of bilirubin encephalopathy or ker- nicterus is one of the goals for the
appropriate diagnosis and treatment of hyperbilirubine- mia. Which of the following
mechanisms has a role in preventing these adverse outcomes?
A Unconjugated bilirubin is not lipid soluble.
B Unconjugated bilirubin is tightly bound to albumin.
C Unconjugated bilirubin is tightly bound to hemoglobin.
D The blood–brain barrier is impermeable to unconjugated bilirubin.
ANSWER:B
126
You are formulating a differential diagnosis for a newborn infant with respiratory distress.
Which of the following is most closely associ- ated with the development of neonatal respi-
ratory distress syndrome (hyaline membrane disease)?
A gestational age
B birth weight
C cesarean section delivery
D maternal diabetes
ANSWER:A
127
Which of the following statements is true regarding infants of comparable weight and
gestational age (> 1500 g) in the United States regarding mortality rate?
A
There is no difference in mortality rates between males and females and African Americans
and Caucasians.
B
Males have a higher mortality rate than females, and African Americans have a higher
mortality rate than Caucasians.
C Males have a lower mortality rate
D
Males have a higher mortality rate than females, and African Americans have a lower
mortality rate than Caucasians.
ANSWER:D
128
A 29-week-gestation infant is being resusci- tated in the delivery room. Surfactant is given
through the endotracheal tube. Which of the following is the most physiologically active
component of surfactant?
A surfactant protein A
B surfactant protein B
C neutral lipid

D phospholipid
ANSWER:D
129
A 25-week-gestation infant is born to a 25-year-old primigravida, who has had
preeclampsia. Which of the following is a true statement describing the neonatal mortality
rate for this infant?
A
It decreases with increasing gestational age from 30 weeks through 43 completed weeks of
gestation.
B It is not related to birth weight.
C It is not related to race.
D It is higher than the mortality rates of adolescents.
ANSWER:D
130
During the resuscitation of a preterm infant, a medical student in the delivery room is asking
why surfactant is so important to neonatal lung physiology. Which of the following is true of
surfactant production as it relates to respiratory distress syndrome in a premature infant?
A Surfactant is synthesized and stored in the type I alveolar cells.
B Surfactant is synthesized and stored in the type II alveolar cells.
C Surfactant is produced by the pulmonary alveolar macrophages.
D Surfactant is stored in the interstitial spaces in the lungs.
ANSWER:B
131
A mother presents in active labor, she did not receive prenatal care and is unsure when her
last menstrual period was. Using ultrasound, the estimated gestational age is 30 weeks. Which
of the following best describes the aver- age birth weight of a 30-week gestation infant?
A 500 g
B 1000 g
C 1500 g
D 2000 g
ANSWER:C
132
A mother presents for prenatal care with a com- plicated medical history as listed in the
following choices. Which of the conditions most predis- poses her fetus to congenital heart
disease?
A hypertension
B diabetes mellitus
C atherosclerotic coronary vascular disease

D anemia
ANSWER:B
133
On a discharge examination you hear a heart murmer and consult the cardiologist. An
echocardiogram was notable for a small ven- tricular septal defect and a patent foramen
ovale. You are notifying the parents of the results and explain to them the role of the fora-
men ovale in fetal life. Which of the following statements are true?
A Blood flows through the foramen ovale from the right ventricle to the left ventricle.
B Blood flows through the foramen ovale from the left ventricle to the right ventricle.
C Blood flows through the foramen ovale from the left atrium to the right atrium.
D Blood flows through the foramen ovale from the right atrium to the left atrium.
ANSWER:D
134
A full-term newborn has a diffuse rash on the day of anticipated discharge. You diagnose
ery- thema toxicum and must discuss this finding with the parents. Which of the following is
true?
A more common among term than prema- ture infants
B usually associated with fever and a gen- eral toxic state
C uncommon before the fifth day of life
D usually associated with an elevated peripheral white blood cell count
ANSWER:A
135
A mother is hospitalized with high blood pressue at 35 weeks gestation. The perinatologist
recom- mends obtaining a lecithin–sphingomyelin ratio of the amniotic fluid to aid in the
decision for delivery. For which of the following systems does the lecithin–sphingomeylin
ratio indicate maturity?
A central nervous system
B lungs
C liver
D kidneys
ANSWER:B
136
A hospitalized neonate is given the diagnosis of bronchopulmonary dysplasia (BPD). Which
of the following best describes the pathophysiol- ogy of BPD?
A An inflammatory insult to the lungs late in fetal development.
B Failure of development of pulmonary arterioles during early fetal life.
C Failure of development of the bronchial buds during early fetal life.
D
The use of oxygen and positive-pressure breathing in the treatment of respiratory distress
syndrome.

ANSWER:D
137
A 32-week-gestation infant is now 24 hours old and has had progressive respiratory dis-
tress. Given the infant’s clinical course and the radiographic appearance of the lungs the deci-
sion has been made, in consultation with the neonatologist, to give surfactant replacement
therapy for respiratory distress syndrome. Which of the following is true?
A is considered experimental
B is only useful in infants with birth weight less than 1500 g
C has no known complications
D requires tracheal intubation to administer
ANSWER:D
138
You are examining the chest x-ray of a 4-hour- old infant born at 30 weeks gestation. The
infant is breathing 100 times per minute while breathing 100% oxygen and intubation is
immi- nent. You suspect respiratory distress syndrome. Which of the following is the
characteristic roentgenographic findings of the infant with respiratory distress syndrome?
A lobar atelectasis and interstitial edema
B bilateral patchy densities and pneu- mothorax
C diffuse reticulogranular changes and air bronchograms
D diffuse hyperaeration and cardiomegaly
ANSWER:C
139
After surfactant therapy, the infant with respi- ratory distress syndrome is treated with con-
tinuous positive airway pressure. Which of the following is the major goal of continuous pos-
itive airway pressure?
A prevent infection
B prevent pneumothorax
C improve cardiac output
D raise arterial Po2
ANSWER:D
140
A 30-week-gestation infant with respiratory distress syndrome is weaning from the venti-
lator. He currently has an oxygen saturation of 100% while breathing 50% oxygen on
minimal ventilator settings. In a premature infant with respiratory distress syndrome, which
of the fol- lowing may be an adverse effect of supple- mental oxygen therapy?
A alveolar proteinosis
B atelectasis
C fire or explosion

D retinopathy of prematurity
ANSWER:D
141
The father of a 26-week-gestation infant, who required intubation at birth, is asking why the
infant’s lungs were not mature. Which of the following is the pathophysiology mechanism of
respiratory distress syndrome in the prema- ture infant?
A increased production of pulmonary surfactant
B decreased production of pulmonary surfactant
C increased metabolism of pulmonary surfactant
D decreased metabolism of pulmonary surfactant
ANSWER:B
142
A healthy term infant is circumcised and expe- riences excessive blood loss eventually requir-
ing transfusion. The most likely diagnosis is which of the following?
A factor IX deficiency
B factor VIII deficiency
C von Willebrand disease
D disseminated intravascular coagulopathy
ANSWER:B
143
An irregular red reflex is noted on the initial examination of an infant. The infant is referred
to the opthalmologist for evaluation of a cataract. Which of the following is most likely to be
asso- ciated with a cataract in the newborn?
A maple syrup urine disease
B glucose-6-phosphate dehydrogenase deficiency
C phenylketonuria
D galactosemia
ANSWER:D
144
A mother has brought her infant into your office for the first newborn visit. She is very
concerned about a pigmented skin lesion that was not discussed with her in the hospital.
Which of the following is true of pigmented lesions known as slate gray spots?
A They never occur in white infants.
B They are identified in over 40% of African American infants.
C They consist of small, well-demarcated lesions approximately 2 mm in diameter.
D Malignant degeneration is common.
ANSWER:B

145
A newborn infant presents with cyanosis and mild tachypnea at about 6 hours of life. The
infant is placed in 95% oxyhood and satura- tions normalize. Which of the following is the
most likely diagnosis in this infant?
A cyanotic congenital heart disease
B lung disease
C central nervous system disease
D liver disease
ANSWER:B
146
A 35-week-gestation infant is delivered weighing3.9 kg, with an omphalocele and a large
tongue. No other abnormalities are detected. Which of the following is the most likely
diagnosis?
A congenital hypothyroidism
B trisomy 18
C trisomy 13
D Beckwith-Wiedemann syndrome
ANSWER:D
147
A 7-day-old infant develops white, cheesy patches on the tongue and buccal mucosa with
mild inflammation of the mucosa. Which of the following organisms is most likely the cause
of these oral lesions?
A Candida albicans
B Listeria monocytogenes
C Escherichia coli
D group A streptococcus
ANSWER:A
148
You are interviewing a family with a newborn infant. They are new to your practice and you
elicit a family history of seizures in both the mother and maternal grandfather. During
examination of the infant you note a skin find- ing that may be associated with the family his-
tory of seizures. Which of the following is the most likely skin finding in this infant?
A hypopigmented patch
B harlequin color change
C salmon patch on the nasal glabella
D hemangioma of the thigh
ANSWER:A

149
A term, otherwise healthy, neonate has isolated premature synostosis of the sagittal suture.
Which of the following is the most likely asso- ciated condition?
A scaphocephaly
B increased intracranial pressure
C microcephaly
D hydrocephalus
ANSWER:A
150
A healthy newborn is noted to have numerous 3 mm vesicles on the chest and neck, along
with several similar-sized hyperpigmented macules in the same distribution. Which of the
following is the most likely diagnosis?
A mucocutaneous herpes simplex infection
B acne neonatorum
C erythema toxicum
D transient neonatal pustular melanosis
ANSWER:D
151
A newborn infant gags and chokes with feed- ings. He has just had an apneic spell during a
feed resulting in significant cyanosis. Which of the following is the most likely diagnosis?
A proximal esophageal atresia
B hypoplastic left heart syndrome
C group B streptococcal sepsis
D neonatal herpes simplex infection
ANSWER:A
152
As a medical student in the newborn nursery, you correctly identify a scalp swelling as a
cephalohemtoma. Your attending physician then asks you to describe how you would
differenti- ate this from caput succedaneum. Which of the following best describes the way
to differentiate a cephalohematoma from caput succedaneum?
A absence of a history of prolonged or dif- ficult labor
B limitation of swelling to the area over one bone
C a normal neurologic examination
D a prolonged prothrombin time
ANSWER:B

153
A 40-year-old couple is in your office for genetic counseling prior to having their first child.
They are very concerned about advanc- ing maternal age and the possible genetic prob- lems
it could cause for their child. Which of the following is the most likely disorder asso- ciated
with advancing maternal age?
A autosomal recessive disorders
B nondisjunction chromosome disorders
C autosomal dominant disorders
D X-linked disorders
ANSWER:B
154
The same couple in Question 46 also is inquiring about the genetic affects of the father’s
age. Which of the following is the most important disorder associated with advancing
paternal age?
A autosomal recessive disorders
B nondisjunction chromosome disorders
C autosomal dominant disorders
D X-linked disorders
ANSWER:C
155
A normal newborn is screened for hypo- glycemia after birth as per the normal nursery
protocol at your hospital. In the asymptomatic term neonate, evaluation and treatment of
hypoglycemia should be initiated when the glucose level is at or below which of the fol-
lowing levels?
A 10 mg/dL
B 20 mg/dL
C 40 mg/dL
D 60 mg/dL
ANSWER:C
156
A nurse working in the newborn nursery calls your office because she is concerned about the
appearance of the feet of a newborn infant just delivered. Based on her description, the
infant has congenital clubfoot. Which of the following systems is most commonly also
involved with this disorder?
A the central nervous system
B the hematopoietic system
C the gastrointestinal system
D the cardiovascular system
ANSWER:A

157
A newborn infant has micrognathia, glossop- tosis, and cleft soft palate. These findings are
consistent with Pierre Robin sequence. Which of the following life-threatening events is asso-
ciated with these findings?
A heart failure
B seizures
C intestinal obstruction
D upper airway obstruction
ANSWER:D
158
A consultant in the neonatal intensive care unit is recommending a trial of pyridoxine for a
patient. Which of the following problems in a newborn infant might respond to a pharmaco-
logic dose of pyridoxine?
A blindness
B seizures
C jaundice
D rash
ANSWER:B
159
The geneticist is evaluating a patient in the newborn intensive care unit. The possibility of a
disorder with a mitochondrial inheritance pattern has been discussed with the family. Which
of the following is true of mitochondrial inheritance?
A These disorders commonly follow a paternal line of inheritance.
B These disorders have not been identified in humans thus far.
C Only females are affected.
D These disorders commonly follow a maternal line of inheritance.
ANSWER:D
160
A family with an infant in the neonatal inten- sive care unit is very concerned that their child
will have long-term neurologic abnormalities. Of the following, which correlates best with
subsequent neurologic abnormalities?
A fetal bradycardia
B failure to breathe at birth
C a low 1-minute Apgar score
D seizures in the first 36 hours of life
ANSWER:D
161
An 8-day-old infant develops inflammatory papules and pustules on the forehead, nose, and
malar areas of the face. The child is other- wise well, and the remainder of the physical
examination is normal. Which of the follow- ing is the most likely diagnosis?

A congenital syphilis
B impetigo
C neonatal acne
D staphylococcal pustulosis
ANSWER:C
162
You are counseling a mother with an abnor- mal serum alpha-fetoprotein test during a
follow-up prenatal visit. The maternal serum alpha-fetoprotein test is most useful in diag-
nosing which of the following?
A duodenal atresia
B clubfoot
C cleft lip and palate
D myelomeningocele
ANSWER:D
163
A newborn with trisomy 21 develops bilious emesis on the first day of life. An abdominal x-
ray reveals a “double bubble sign.” Which of the following is the most likely diagnosis?
A annular pancreas
B duodenal atresia
C gastric volvulus
D pyloric stenosis
ANSWER:B
164
A 1700-g infant was asphyxiated at birth, after a successful resuscitation, the infant had
numerous apneic episodes. On the third day of life, the infant began to vomit. Abdominal
dis- tention and bloody stools were noted. Which of the following is the most likely
diagnosis?
A congenital aganglionic megacolon
B intussusception
C necrotizing enterocolitis
D Shigella enteritis
ANSWER:C
165
A 2-month-old infant is in your office for a well-child examination. The mother is asking
you about a red macule on the infant’s cheek. She describes it as getting larger over the past
few weeks. You suspect this is a capillary hema- gioma. The natural history of an elevated
capil- lary or cavernous hemangioma is best described by which of the following statements?
A no significant change in size after birth.

B an increase in size during the first few years after birth and then regression.
C an increase in size during the first decade of life and then no further change.
D a slow but progressive increase in size throughout life.
ANSWER:B
166
A full-term, ill appearing, infant presents with vesicles and large bullae shortly at birth.
Which of the following is most likely?
A herpes simplex virus
B gram-negative infection
C epidermolysis bullosa
D Staphylococcus aureus
ANSWER:D
167
A newborn infant presents with skin lesions typical of erythema toxicum and you wish to
confirm the diagnosis. Wright stain of the con- tents from a lesion of erythema toxicum
usually will reveal which of the following cells?
A basophils
B eosinophils
C lymphocytes
D immature lymphocytes
ANSWER:B
168
A mother with hyperparathyroidism just deliv- ered a term infant who appears healthy.
Which of the following problems is likely to develop in her infant?
A hypercalcemia
B hypocalcemia
C parathyroid carcinoma
D hyperthyroidism
ANSWER:B
169
A screening bilirubin test is sent on a term new- born who appears jaundiced at 72 hours of
life. You unexpectedly identify conjugated hyper- bilirubinemia. Which of the following is
asso- ciated with conjugated hyperbilirubinemia?
A sequestrated blood
B tyrosinemia
C hereditary spherocytosis
D Rh incompatibility
ANSWER:B

170
A newborn infant is being evaluated for ambiguous genitalia. While awaiting results of
chromosome analysis, the infant develops hyponatremia, hypoglycemia, and low blood
pressure. What is the most likely diagnosis?
A 21-hydroxylase deficiency
B severe hypospadius
C 3-beta-hydroxysteroid dehydrogenase deficiency
D 11-beta-hydroxylase deficiency
ANSWER:A
171
You are evaluating a small-for-gestational-age infant for tachycardia. There is no prenatal
care history available, but based on the infant’s signs and symptoms and the obstetrician’s
description of the mother you suspect neonatal thyrotoxicosis. Thyrotoxicosis in the first day
of life most likely occurs in an infant born to a mother with which of the following
diagnoses?
A with untreated hypothyroidism
B with untreated Graves disease
C with Graves disease being treated with antithyroid medications
D with euthyroid goiter
ANSWER:B
172
During a prenatal care visit, a mother notifies you that her partner has been diagnosed with
herpes simplex virus (HSV). She is asking about risks for the infant if she has HSV. Which of
the following is the most important risk fac- tors for neonatal HSV infection?
A prematurity
B cold sore on mother’s lip at time of delivery
C primary maternal genital infection dur- ing pregnancy
D fetal scalp monitoring
ANSWER:C
173
As a medical student in the neonatal intensive care unit, you report a calcium level in a 1-day-
old term newborn that was flagged as abnor- mal by the laboratory. Which of the following
is true of serum calcium levels in the newborn?
A High parathyroid hormone (PTH) levels suppress the serum calcium.
B The serum calcium promptly decreases after delivery.
C The serum calcium remains essentially unchanged over the first 3 days.
D The serum calcium is usually higher on day 2 than day 1.
ANSWER:B

174
A mother of a 3-day-old newborn, born at home with a midwife, is calling your office
because she is concerned about constipation. You are deciding whether to evaluate this infant
for Hirschsprung disease and ask her about meconium passage. Meconium passage delayed
beyond which of the following time points to the diagnosis of Hirshsprung disease?
A during the process of birth
B within a few minutes of birth
C 2–4 hours of life
D 24–36 hours of life
ANSWER:D
175
On the fifth day of life, an infant is noted to have a violaceous, circumscribed, subcutaneous
nodule immediately beneath fading forceps marks on one cheek. The infant is otherwise well
appearing. Which of the following is the most likely diagnosis?
A an abscess
B a hemangioma
C a pericytoma
D subcutaneous fat necrosis
ANSWER:D
176
Your office is notified by the state screening program that a newborn in your practice is pre-
sumped positive for hypothyroidism. Which of the following is true of congenital hypothy-
roidism?
A Affected infants are usually clinically apparent by the third day of life.
B Affected infants have unusually small fontanelles.
C Affected infants appear thin.
D Prolonged hyperbilirubinemia is common.
ANSWER:D
177
Iron deficiency is the leading cause of anemia in infancy and childhood. Which of the
following infants is at greatest risk of this early on?
A a premature infant
B an infant with ABO incompatibility
C an infant with physiologic hyperbiliru- binemia
D a postmature infant
ANSWER:A

178
A term infant has respiratory distress and a blood count reveals a hematocrit of 65%. Your
resident states that fetal hemoglobin has an increased affinity for oxygen as compared with
adult hemoglobin. What effect might this result in?
A an increased mass of fetal hemoglobin
B thin red cell membrane in the fetus
C the presence of unconjugated bilirubin in the red cell
D decreased binding with 2,3-diphospho- glycerate (2,3-DPG)
ANSWER:D
179
An infant born with malformed forearms is noted to develop severe hemorrhagic manifes-
tations. Which of the following would be the most likely cause of the bleeding problem?
A vitamin K deficiency
B factor VIII deficiency
C thrombocytopenia
D disseminated intravascular coagulation
ANSWER:C
180
You are a medical student evaluating a jaun- diced infant in clinic. The infant appears
healthy and has been entirely breast-fed. The total bilirubin level is 19 mg/dL, direct biliru-
bin is 0.2 mg/dL, and the hemoglobin is normal. You have diagnosed “breast milk jaun-
dice.” Which of the following is the most typical feature of breast milk jaundice?
A It is an unconjugated hyperbilirubinemia.
B It usually appears within the first 12 hours of life.
C The elevated levels may persist for more than 60 days.
D It is generally associated with moderate anemia and reticulocytosis.
ANSWER:A
181
You are counseling a diabetic teenager who states she is trying to get pregnant. She has not
been compliant with her diabetes management over the past few years. You are trying to
stress the importance of glucose control prior to con- ception and during pregnancy by
explaining the possible complications associated with infants born to diabetic mothers. Which
of the following is the most common problem in infants of diabetic mothers?
A birth asphyxia
B sacral agenesis
C hyponatremia
D hypoglycemia
ANSWER:D

182
A 12-month-old infant with a cloudy cornea is diagnosed with primary infantile glaucoma, a
rare disorder occurring in 1:10,000 live births. You explain to the parents that it can be a
major cause of blindness. Which of the following state- ments regarding infantile glaucoma
is true?
A It is best treated with medical management.
B It is frequently seen with trisomy 21.
C It presents with the white pupil sign.
D It is associated with congenital rubella and neurofibromatosis.
ANSWER:D
183
A premature infant is now 4 hours old and you suspect respiratory distress syndrome
(hyaline membrane disease). Which of the following is the least likely first clinical
presentation of res- piratory distress syndrome?
A tachypnea
B delayed capillary refill
C cyanosis
D wheezing
ANSWER:D
184
Fetal heart rate monitoring is being used during the labor of a 40-year-old primagravida who
is now 1 week post dates. Which of the following would most commonly be seen in a
healthy baby?
A heart rate 60–80 beats per minute
B accelerations of the heart rate
C heart rate below 60 beats per minute
D decreased beat-to-beat variability in heart rate
ANSWER:B
185
You are the resident on call and are called urgently to the bedside of a newborn. The nurse is
concerned that the infant has developed a tension pneumothorax. Which of the following
clinical signs most likely represent those seen in a newborn with tension pneumothorax?
A cyanosis, apnea, and tachycardia
B apnea, hypertension, and bradycardia
C tachypnea, cyanosis, and bradycardia
D wheezing, tachycardia, and hypertension
ANSWER:C

186
You are evaluating an infant born at 39 weeks gestation weighing 2000 g. Which of the fol-
lowing would be the appropriate classification of this infant?
A low birth weight
B premature
C small for gestational age
D low birth weight and small for gesta- tional age
ANSWER:D
187
A term infant is now 10 hours of age and has a seizure. Which of the following diagnoses is
more common among term rather than pre- mature infants?
A intraventricular hemorrhage
B hemorrhagic disease of the newborn
C sepsis
D subdural hemorrhage
ANSWER:D
188
A term neonate begins vomiting during the first few days of life, and develops a distended
abdomen. The family history is positive for cystic fibrosis. Which of the following condi-
tions, if found, would most likely be related to the family history?
A annular pancreas
B duodenal atresia
C hypertrophic pyloric stenosis
D meconium ileus
ANSWER:D
189
An infant delivered precipitously to a mother who had not received prenatal care. The infant
has finding suggestive of congenital syphilis. Maternal and infant testing confirm this diag-
nosis. Which of the following is most sugges- tive of early congenital syphilis?
A disseminated intravascular coagulation
B bullous lesions of the palms and soles
C hepatitis
D dermal erythropoiesis
ANSWER:B
190
Prior to the newborn screening program in your state, you are evaluating an infant with a
skeletal survey due to concerns of trauma. Which of the following roentgenographic find-
ings in a newborn infant is most suggestive of undiagosed hypothyroidism?
A epiphyseal dysgenesis
B absence of ossification of the hamate bone

C prominent thymic shadow
D osteoporosis
ANSWER:A
191
A 5-day-old appropriate-for-gestational age male presents with tachypnea, poor feeding, and
lethargy. On examination, the neonate appears in shock with hypotension, pallor, and poor
capillary refill. Among the following, which is the most likely diagnosis?
A tetrology of Fallot
B tricuspid atresia
C transposition of the great vessels
D hypoplastic left heart syndrome
ANSWER:D
192
Incontinentia pigmenti (IP) is an X-linked hereditary disorder and is typically lethal in
males. Skin lesions may be present at birth. Which of the following best describe the initial
lesions of incontinentia pigmenti?
A hypopigmented
B scaly
C waxy papules
D inflammatory bullae
ANSWER:D
193
A newborn infant is noted to have a peculiar face with low-set ears and widely spaced eyes
with epicanthic folds. The chin is receeding, and the nose is broad and flat. Oligohydramnios
was noted prior to delivery. Examination of the pla- cental membranes reveals amnion
nodosum. Which of the following diagnoses should be suspected?
A bilateral renal agenesis
B atrioventricular canal defect
C esophageal atresia
D group B streptococcal infection
ANSWER:A
194
Hydrocephalus, chorioretinitis, and diffuse cere- bral calcifications are present in a newborn
male. Of the following pets, which is most likely to be the source of this zoonotic congenital
infection?
A dog
B cat
C horse
D rabbit

ANSWER:B
195
A 42-week gestation neonate is born by cesarean delivery following repeated fetal heart rate
decelerations. At rupture of membranes, thick meconium was noted. The infant has no
respiratory effort but slight movement and a heart rate of 90 beats per minute. Of the fol-
lowing, which is the first intervention?
A pass an umbilical artery catheter to measure pH and Po2
B start bag-and-mask ventilation with 100% oxygen
C suction the mouth and trachea to remove the meconium
D intubate the trachea and begin ventila- tion with 100% oxygen
ANSWER:C
196
An ultrasound at 33 weeks gestation reveals a small-for-gestational-age fetus with no other
abnormalities. At birth at 39 weeks, the 1.6-kg neonate requires no resuscitation. On
examina- tion, the infant is noted to be microcephalic and have a generalized rash consisting
of dozens of blue-purple macules. His abdominal examina- tion reveals hepatosplenomegaly.
Which of the following is the most likely diagnosis?
A group B streptococcal septicemia
B isoimmune thrombocytopenia
C congenital cytomegalovirus infection
D congenital toxoplasmosis
ANSWER:C
197
A term infant at 12 hours of age is normal except for jaundice. Initial laboratory values
reveal a total bilirubin of 12 mg/dL and several spherocytes on the peripheral blood smear.
Among the following, which is the most likely diagnosis?
A hereditary elliptocytosis
B hereditary spherocytosis
C glucose-6-phosphate dehydrogenase deficiency
D ABO isoimmune hemolytic disease
ANSWER:D
198
A previously healthy term infant is brought to clinic by his mother at 2 weeks of age because
of rapid breathing, poor breastfeeding, and excessive sleeping. He is afebrile. Which of the
following is the most common arrhythmia at this age?
A supraventricular tachycardia
B atrial flutter
C atrial fibrillation
D complete atrioventricular block

ANSWER:A
199
A 1-hour-old, 33-week gestation male neonate develops tachypnea, grunting respirations,
and deep intercostal retractions. A chest x-ray reveals a homogeneously grainy appearance to
both lung fields. An arterial blood gas would most likely reveal which of the following?
A respiratory alkalosis and hypoxemia
B respiratory acidosis and metabolic alkalosis
C respiratory acidosis and hypoxemia
D respiratory and metabolic alkalosis
ANSWER:C
200
A college student delivers an infant in the campus housing area after denying her preg-
nancy and experiencing 6 hours of abdominal pain. Paramedics arrive and find an infant
they estimate to be 4–5 lb in weight, crying but slightly dusky. They administer oxygen. The
newborn’s temperature is 94°F. Hypothermia in this infant would produce which of the
following?
A decreased oxygen consumption
B hypertriglyceridemia
C hypercalcemia
D hypoglycemia
ANSWER:D
201
A 30-day-old, former 24-weeks gestation, 600 g neonate had a difficult initial respiratory
course complicated by a tension pneumothorax. She had serial head ultrasound evaluations
during the first weeks of life. All previous studies revealed a normal immature brain. Now,
the head ultrasound reveals an abnormality. Among the following, which is most likely?
A grade II intraventricular hemorrhage
B grade IV intraventricular hemorrhage
C aqueductal stenosis
D periventricular leukomalacia
ANSWER:D
202
A newborn infant has signs of congestive heart failure. Physical examination does not reveal
a significant cardiac murmur. Auscultation of the head reveals a loud cranial bruit. Which of
the following is the most likely diagnosis?
A polycythemia
B hyperthyroidism

C ruptured cerebral aneurysm
D arteriovenous malformation of the great vein of Galen
ANSWER:D
203
A term 4.3-kg infant is delivered vaginally to a 33-year-old woman with juvenile-onset dia-
betes. The delivery was complicated by severe shoulder dystocia and the infant experienced
a brachial plexus injury with limited movement of the right arm. At 72 hours of age, the
infant is noted to be tachypneic but he is pink and well perfused. Which of the following is
the most likely explanation for his tachypnea?
A respiratory distress syndrome (RDS)
B diaphragmatic paralysis
C pulmonary hemorrhage
D pneumothorax
ANSWER:B
204
A term infant is delivered vaginally to an 18-year-old primagravida after an uncompli- cated
pregnancy. Mother presented with rup- ture of membranes and a low-grade fever. Labor and
delivery proceeded rapidly, maternal antibiotics were not administered prior to delivery of
the infant. Maternal screening during pregnancy for group B streptococcus was negative. At
30 hours of age the infant is mottled, not feeding well, and cries when handled. A lumbar
puncture is performed and the spinal fluid reveals 660 WBC/mm3 and a CSF protein of 290
mg/dL. Which of the fol- lowing is the most likely agent causing this infant’s infection?
A group B streptococcus
B Escherichia coli
C Listeria monocytogenes
D Haemophilus influenzae
ANSWER:A
205
A mother brings her 6-month-old son in for a checkup. She is concerned that when she is
play- ing with him and puts a toy behind her back, he does not try to find it. Object
permanence, the understanding that objects continue to exist even when not seen, is a major
milestone that occurs closest to which of the following ages?
A 9–12 months
B 15 months
C 18 months
D 24 months
ANSWER:A

206
A 6-month-old girl, comes in for her checkup with her mother and grandmother. A look in
the girl’s mouth reveals no teeth. What is the usual age for the eruption of the primary teeth?
A 2–4 months
B 4–6 months
C 6–8 months
D 8–10 months
ANSWER:C
207
The girl’s grandmother goes on to report that she is sure that her grandaughter will be left-
handed, just like her. Consistent use of a dom- inant hand is established at which of the
following ages?
A 12 months
B 24 months
C 4–6 years
D 7 years
ANSWER:C
208
A group of nursing students are learning about how vision screening is done in the primary
care office. One student asks at what age visual acuity reaches 20/20. Normal visual acuity
typ- ically occurs at which of the following ages?
A 1 week
B 6 months
C 1 year
D 4–5 years
ANSWER:D
209
The father of a 3-year-old boy brings him in with a 2-week history of purulent rhinorrhea
and an occasional headache in the frontal area. He requests that you perform sinus films to
determine if his son has frontal sinusitis. What is the earliest age at which the frontal sinuses
can be visualized radiographically?
A 3 years
B 4 years
C 6 years
D 8 years
ANSWER:C

210
A medical student is rotating with a local pedi- atrician who uses the Parents’ Evaluations of
Developmental Status (PEDS) questionnaire. Which of the following characteristics best
describes this screening test?
A Used to evaluate development of chil- dren 10–14 years
B Screens for development in children less than 8 years
C Utilizes a 100-question format
D Used to evaluate teen adjustment
ANSWER:B
211
A third-year college student reports that she took a pregnancy test which was positive this
morning. She had moderate alcohol use for the last 2 months and was worried that develop-
ment of the embryo might be affected. Which of the following gestational ages describes the
embryonic period?
A up to 5 weeks
B up to 9 weeks
C up to 12 weeks
D up to 16 weeks
ANSWER:B
212
At 4-weeks gestation, she has not experienced fetal movements. Which of the following best
describes the occurrence of first fetal move- ments, or quickening?
A 12 weeks
B 16 weeks
C 20 weeks
D 24 weeks
ANSWER:C
213
A worried mother brings her 2-year-old to the pediatrician for a well-child check. She is con-
cerned because the child still has only 50 words in his vocabulary. Which of the following
best describes the characteristic age of a toddler who speaks 50 words?
A 12 months
B 15 months
C 18 months
D 24 months
ANSWER:D
214
The mother of a 3-year-old boy reports that he is “on the go” all the time. Typically,
children stop taking daytime naps at which of the fol- lowing ages?
A 18 months

B 24 months
C 30 months
D 48 months
ANSWER:D
215
Pediatric residents are performing camp phys- icals for Girl Scouts. A 9-year-old girl asks
how she will know that she is beginning puberty. In girls, which of the following is the first
visible sign of puberty?
A development of breast buds
B appearance of early pubic hair
C menarche
D enlargement of the clitoris
ANSWER:A
216
While giving a talk about growing up to the parents of 4-H youth, a father asks the pedia-
trician if it is considered normal for a boy to have signs of puberty at age 8. The median age
for entering puberty in boys is at what age?
A 8 years
B 10 years
C 12 years
D 13 years
ANSWER:B
217
A 10-year-old female patient, who is at sexual maturity rating (SMR) stage II is the shortest
girl in her class and her mother wonders when she will have her next growth spurt. Maximal
height gain velocity in girls occurs during which of the following SMR?
A I
B II
C III
D IV
ANSWER:C
218
The grandmother of an 8-year-old boy pres- ents with him for his preschool checkup and is
concerned because he has not grown in height for the last 2 years. Which of the following
best approximates the prepubertal annual increase in height?
A 1–2 cm
B 3–4 cm
C 5–7 cm
D 8–10 cm

ANSWER:C
219
A mother concerned about her 10-year-old’s weight calls your office to find out how to cal-
culate her child’s BMI. Your office nurse tells her the BMI can be calculated only if one
knows which of the following values?
A Weight
B Height
C Weight and height
D Weight, height, and age
ANSWER:C
220
A pediatric endocrinologist has diagnosed con- stitutional growth delay in a 14-year-old
boy. Which of the following are consistent with this diagnosis?
A bone age is normal compared to chrono- logic age
B prognosis for a normal adult height is good
C family history for childhood short stature
D undernourished for the first year of life
ANSWER:B
221
A 1-week-old infant male presents to his pri- mary care provider’s office for his first visit.
His mother is concerned because he sleeps a lot. Which of the following is the average
amount of sleep per day experienced by infants in the first month of life?
A 12 hours
B 14 hours
C 16 hours
D 18 hours
ANSWER:C
222
A 100 kg, 10-year-old presents for well-child care. Which of the following history items
should be included when evaluating this obese child?
A family history of hyperparathyroidism
B vigorous physical activities performed by the child
C parents’ occupation
D instances of binge drinking by the parent
ANSWER:B

223
A preschool teacher is concerned that a 24-month-old boy occasionally repeats words
spoken to him. She sends a note with mother to his checkup. The boy’s physical and
neurologic examinations are entirely normal, and the child’s developmental landmarks are
within normal limits. Which of the following is the most likely description of this child’s
symptom?
A precocious verbal behavior
B subtle neurologic abnormality
C poor language development
D normal
ANSWER:D
224
A neonatologist is performing a physical exam- ination with medical students on an infant
greater than 24 hours old. Which of the fol- lowing findings would be considered normal for
the neonate at this age?
A has a visual preference for geometric shapes over faces
B needs 4–6 hours after birth to suck well at the breast
C is farsighted
D learns to differentiate the voice of her mother from that of other women by 4 weeks of age
ANSWER:D
225
First-time parents of a 1-month-old are anx- ious to hear what they can expect in the next
few months. Which of the following best describes the usual sequence for attainment of gross
motor milestones in a young infant?
A head control, rolling over, hands together in midline, sits without support
B head control, hands together in midline, rolling over, pulls to stand
C rolls over, sits without support, hands together in midline, pulls to stand
D sits without support, hands together in midline, pulls to stand, walks along
ANSWER:B
226
The parents of a 15 month old are concerned that their daughter does not seem to be able to
stack cubes into a tower like their son could at the same age. Her pediatrician reviews the
child’s prior fine motor development with the parents. Which of the following best describes
the correct sequence of fine motor develop- ment one should see in a healthy infant/toddler?
A thumb–finger grasp, grasps rattle, trans- fers objects from hand to hand
B grasps rattle, thumb–finger grasp, reaches for objects, transfers objects from hand to hand
C
grasps rattle, transfers objects from hand to hand, builds tower of two cubes, thumb–finger
grasp

D
grasps rattle, transfers objects from hand to hand, thumb–finger grasp, builds tower of two
cubes
ANSWER:D
227
A 6-year-old boy is proud to show off the first tooth he lost but his mother is concerned
because the tooth was one of his canines and a perma- nent tooth seems to be erupting in this
area. Which of the following best describes the correct sequence for eruption of permanent
teeth?
A central incisors, first premolars, canines, second molars
B first molars, central incisors, canines, first premolars
C central incisors, lateral incisors, canines, first molars
D first molars, lateral incisors, canines, first premolars
ANSWER:C
228
The parents of a 6-month-old boy point out that he has just cut his first tooth, and wonder
which teeth will come next. Which of the fol- lowing is the correct sequence for eruption of
primary teeth?
A central incisors, first molars, second molars, canines
B central incisors, lateral incisors, canines, first molars
C central incisors, canines, first molars, second molars
D central incisors, lateral incisors, first molars, canines
ANSWER:D
229
A term newborn infant has a normal weight but height below the 5th percentile. A large
posterior fontanelle is the only other finding. A delayed bone age is noted on skeletal survey
based on the appearance of ossification centers and further testing reveals the diagnosis of
con- genital hypothyroidism. In normal neonates, ossification centers are usually radiographi-
cally visible at birth in which of the following sites?
A patella
B lunate (carpal)
C proximal tibia
D head of the femur
ANSWER:C
230
The grandfather of a 1-year-old brings him in for a checkup and states that the boy received
a tricycle for his first birthday but wonders when he will be able to pedal to ride it. Which of
the following sequence best demonstrates the correct attainment of motor milestones?
A runs, rides tricycle, skips, hops

B runs, hops, skips, rides tricycle
C runs, goes upstairs alternating feet, rides tricycle, skips
D hops, skips, rides tricycle, goes upstairs alternating feet
ANSWER:C
231
The mother of a 30-month-old brings the pri- mary care provider a note from his parents as
teachers (PAT) (early childhood development) specialist. The PAT specialist is concerned by
the child’s lack of self-feeding skills. Which of the following social development sequences
do toddlers follow?
A feeds self, helps to undress, washes hands, domestic role-playing
B handles spoon well, plays in parallel, dresses and undresses, washes hands
C plays in parallel, feeds self, dresses and undresses, washes hands
D plays in parallel, dresses and undresses, washes hands, feeds self
ANSWER:A
232
A group of student nurses is learning about childrens’ language development. Their instruc-
tor explains that language is a critical barometer of both cognitive and emotional
development. With which of the following is speech delay most closely associated?
A 22q11 deletion syndrome
B spina bifida
C diabetes
D child abuse
ANSWER:D
233
The mother of a 2-year-old girl brings her to her pediatrician’s office for a checkup. When
asked about discipline concerns, the mother expresses worry because the child gets mad and
throws herself on the ground screaming. Which of the following statements regarding temper
tantrums is correct?
A most often indicate a serious psychoso- cial problem
B usually appear at the end of the first year
C peak prevalence is between 4 and 6 years
D routinely occur 8–10 times per day
ANSWER:B
234
A group of first-year residents is examining the National Center for Health Statistics growth
curves for girls and boys. Typically, infants and children stay within one or two growth chart
channels. A normal exception to this rule exists for which of the following?
A female infants
B during preschool years (3–6 years)

C up to age 2 years
D male infants
ANSWER:C
235
As part of a routine 18-month check-up, a med- ical student in the outpatient clinic adminis-
ters the Denver Developmental Screening Test (DDST) to the child. She reports that the child
appeared to function at about a 15–18-month level, but he was noncompliant and difficult to
test. Which of the following best explains the testing results for this child?
A advanced and was stressed by the test
B delayed and psychologically disturbed
C delayed and was stressed by the test
D normal and noncompliance is common at this age
ANSWER:D
236
The mother of a 6-year-old in first grade reports that he is not keeping up with the other
kids in any subject and wants testing to be performed. Evaluation of academic failure at
school typi- cally includes which of the following?
A hearing and visual evaluation
B personality testing
C magnetic resonance imaging (MRI) of head
D home visit
ANSWER:A
237
Vision milestones are being discussed with a group of student nurses. One student asks when
babies can begin to visually follow people as they walk across the room. Fixation and
tracking through the visual field are well developed by which of the following ages?
A at 7 months gestation
B at birth
C at 2 months
D at 6 months
ANSWER:C
238
A tired-appearing mother of a 2-month-old boy states he is feeding and sleeping well but
cries for a total of 3 hours each day. The baby’s examination is normal. Which of the follow-
ing statements about crying is correct?
A Crying increases through the entire first year of life.
B Crying usually is a result of hunger.

C
Any baby crying 3 hours per day, even in light of a normal examination, war- rants a
medical workup.
D This amount of crying is normal for this age.
ANSWER:D
239
The mother of 9-month-old, who had a normal sleeping pattern in the past, reports he has
started awakening and crying once or twice a night. Her primary care physician suggests that
this may be a manifestation of separation anx- iety. At which of the following ages does sepa-
ration anxiety usually first manifest?
A 1 week
B 3–4 months
C 8–9 months
D 2–3 years
ANSWER:C
240
A medical student is checking out a 2-month well-child visit to the attending pediatrician.
She remarks that the infant is sleeping with her parents. Which of the following statements
regarding cosleeping is true?
A Incidence of sudden infant death syn- drome (SIDS) is higher in young infants.
B Cosleeping is uncommon in countries outside of the United States.
C Cosleeping can facilitate transitioning to a child’s own crib or bed.
D The primary care provider should refer the child for sleep testing.
ANSWER:A
241
A 7-month-old infant presents for well-child care and the physician notes a well-developed
Moro reflex, palmar grasp reflex, and tonic neck reflexes on examination. The persistence of
neonatal reflexes is an indicator of develop- mental delay, he tells the mother of the infant.
At which age should the Moro reflex typically disappear?
A 3 months
B 4 months
C 6–8 months
D 12–16 months
ANSWER:C
242 At what age should the tonic neck reflex disappear?
A 3 months
B 4 months
C 6–8 months
D 12–16 months

ANSWER:C
243 At what age should the palmar grasp reflex disappear?
A 3 months
B 4 months
C 6–8 months
D 12–16 months
ANSWER:D
244
A 1-year-old girl who has been in a motor vehi- cle accident today is alert, but you note a
pos- itive Babinski (upgoing plantar) reflexes. At what age does a Babinski reflex disappear?
A 3 months
B 4 months
C 6–8 months
D 15–18 months
ANSWER:C
245
A mother brings her preteen twins (1 girl, 1 boy) in for a checkup and notes that the girl is 2
in taller than her twin. She asks the pediatric nurse practitioner if this could be a sign of
puberty. Pubertal growth spurt in females begins on average at what age?
A 8 years
B 10 years
C 12 years
D 14 years
ANSWER:B
246 The pubertal growth spurt in males occurs at which of the following ages?
A precedes the growth spurt in females by 2 years
B has its onset at the same age as in females but lasts longer
C follows the growth spurt in females by 2 years
D coincides with attainment of the ability to ejaculate
ANSWER:C
247
The father of a 13-year-old boy asks if his son’s adult height can be predicted. The peak
veloc- ity of growth during adolescence averages which of the following?
A 1–2 cm/yr
B 3–4 cm/yr
C 5–6 cm/yr
D 9–10 cm/yr

ANSWER:D
248
An adolescent male wearing a baggy T-shirt comes to his pediatrician’s office for evalua-
tion of breast development. He is very worried and embarrassed, and he inquires if there is
anything that can be done to treat it. Which of the following statements regarding gyneco-
mastia in males during adolescence is correct?
A is distinctly uncommon
B usually occurs at age 14–15 years
C is synonymous with lipomastia
D is often a continuing problem in adulthood
ANSWER:B
249
Concerns about childrens’ TV watching are being discussed with a group of parents. The age
at which a child can reliably distinguish fantasy from reality occurs at which of the following
ages?
A 4 years
B 6 years
C 7 years
D 8 years
ANSWER:C
250
A 19-year-old G3P3 reports that she had a period of depression after the birth of her second
child. She is concerned this could resur- face with the birth of the third child. Which of the
following statements is true regarding post- partum depression?
A Postpartum “blues” are rare in new mothers.
B Other family stress usually lessens in the postpartum period.
C Teenage mothers have a high rate of depression.
D Psychosis is a common feature.
ANSWER:C
251
The HEADSS inventory is a useful screening tool for adolescents. Which of the following
areas are evaluated?
A Risk for decreased intelligence quotient
B Risk for early coronary artery disease
C Risk for diabetes
D Risk for depression
ANSWER:D
252
The tired parents of an infant girl ask if there are any predictive factors for the development
of colic. Which of the following best describes risk factors for colic?

A Colic usually is associated with infants who are bottle-fed.
B Colic typically begins at 41–42 weeks gestational age regardless of gestational age at birth.
C Colic is most prevalent among cau- casian neonates.
D Colic occurs more commonly among females than males.
ANSWER:B
253
The parents of a 21-month-old boy are concerned because he used to say a few words and
more recently, he is just babbling with no recognizable words. They are worried these may
be features of autism. Which of the following is a feature that characterizes autistic spectrum
disorders?
A Pointing or waving by 18 months
B Use of 2 word phrases by 18 months
C Decreased eye contact
D Nystagmus
ANSWER:C
254
The mother of a 6-month-old girl presents for a well-child check. The baby’s weight and
length are at the 5th percentile, and her head circumference is at the 25th percentile. Which
of the following is a true statement regarding failure to thrive in infants?
A more common among Hispanic children
B extremely rare in the United States
C most often caused by an organic problem
D a major risk factor for later developmen- tal and behavioral difficulties
ANSWER:D
255
The parents of a 6-year-old boy bring him to his pediatrician to discuss his behavior patterns.
The child’s mother is concerned about his behaviors, but his father reports that his son will
“grow out of it.” Some behavior patterns are considered appropriate at certain develop-
mental stages, but are obviously pathologic if present later on in life. Other behavior patterns
are considered pathologic at all ages. Which of the following is an example of the latter?
A temper tantrums
B lying
C oppositional behavior
D truancy
ANSWER:D
256
One of this mother’s concerns is that her 6-year-old often physically fights with his sib-
lings. Childhood aggression is most commonly associated with which of the following?

A girls compared to boys
B children small for their age compared to children large for their age
C children from smaller families compared to children from larger families
D children diagnosed with attention- deficit hyperactivity disorder
ANSWER:D
257
Primary care office nurses are designing a ques- tionnaire to screen for toxin poisoning in
their practice. They wonder how to describe pica so that a parent can understand it. Pica is
the ingestion of nonnutrient substances like dirt and chalk. Which of the following statements
regarding pica is true?
A The median age of onset is 6 years.
B It is often a symptom of family disor- ganization, poor supervision, and affec- tional neglect.
C It is caused by iron poisoning and para- sitic infestations.
D It usually persists into adolescence.
ANSWER:B
258
A father brings his twins (1 boy, 1 girl) in for their kindergarten checkup. He is concerned
because both children wet the bed at night, and he wonders how long it will last. Which of
the following statements regarding nocturnal enuresis is true?
A Prevalence at age 10 years ranges from 7% to 10%.
B A marked familial pattern is often noted.
C
Workup routinely recommended in the management includes renal ultrasound and
micturating cystourethrogram to rule out kidney disease and vesci- coureteric reflux.
D Bed wetting alarms are rarely useful in the management of some children with enuresis.
ANSWER:B
259
A pediatrician has followed a child in foster care since infancy. This child has experienced
academic and social success despite difficult environmental stresses. Factors which con-
tribute to resiliency in childhood include which of the following?
A difficulty with adaptability to changes
B low level of energy and persistence
C a desire to investigate and master new situations
D family history of academic success
ANSWER:D
260
The mother of a kindergartner is concerned that her child may have a learning disability.
Which of the following would be consistent with a learning disability?
A reverses some letters when writing his name

B had delayed speech as toddler
C diagnosis before age 3 years
D has a discrepancy between intelligence and achievement in one or more areas
ANSWER:D
261
The mother of a 2-year-old girl wonders if her daughter’s motor skills are age appropriate.
Which of the following is a motor skill that most 2-year-olds have attained?
A stands on one foot for 10 seconds
B climbs stairs using alternating feet
C pedals a tricycle
D builds a tower of 8–10 cubes
ANSWER:D
262
The mother of a 2-year-old girl is concerned that her daughter is resisting efforts to be toilet-
trained. Which of the following recommendations for toilet training are advisable?
A Begin training before age 2 years
B Continue even if the child is extremely resistant
C “Time out” for “accidents”
D Sticker charts are a useful tool to track successes
ANSWER:D
263
The normal cognitive development of toddlers is being discussed with a group of medical
stu- dents. The developmental thrust of the 2-year- old is best expressed by which of the
following phrases?
A “me do it”
B “show me how”
C “that can’t be right”
D “why”
ANSWER:A
264
The mother of a 9-month-old girl reports that she is now waking up at night, after sleeping
all night for several months. Nighttime awakening at this age can be treated by which of the
following?
A Vary the bedtime routine
B Avoid the use of a transitional object such as a teddy bear
C Give a bottle of formula or breast milk to the infant in bed
D Put infant to bed while drowsy but still awake
ANSWER:D

265
In discussing the features of the body mass index (BMI) curve for children to a group of
medical students, the pediatrician reports that children experience a period of growth called
the adiposity rebound, where their body weight begins increasing to a greater degree than
their height is increasing. Adiposity rebound typically occurs at which of the fol- lowing
average ages?
A 1 week
B 1 year
C 5 years
D 10–12 years
ANSWER:C
266
In coordinating pediatric residents to perform head start (early childhood development)
physicals and developmental screenings on a group of preschool children, it is mentioned
that a child who can walk downstairs alternat- ing his or her feet, do a broad jump, and
throw a ball overhand, also would be expected to do which of the following?
A add five and five
B identify three or four coins
C name two or three colors
D multiply three times three
ANSWER:C
267
The mother of a 3-year-old girl reports that her daughter is afraid to go to bed at night.
Which of the following are recommended methods of managing nighttime fears?
A Perform an exhaustive “search” for monsters in the child’s bedroom
B Vary the bedtime schedule
C Vary the regular nighttime routine prior to bedtime
D Visit the child briefly to provide reassur- ance when the child awakens with fear
ANSWER:D
268
The parents of a child who recently turned 5 years old bring him in for a well-child check
and assessment of school readiness. Regarding school readiness, which of the following is
true?
A
Any concerns about the child’s ability to learn will probably resolve if the child delays
kindergarten entry for a year.
B Previous daycare experience is irrelevant.
C The ability to follow 1–3 step directions is an important skill for kindergarten success.
D Private schools usually have more access to specialized services than do public schools.
ANSWER:C

269
The pathophysiology of various causes of neona- tal cyanosis is being discussed with a
group of pediatric residents doing their nursery rotation. In the term newborn, which of the
following state- ments regarding fetal hemoglobin is correct?
A Nearly 100% of a neonate’s hemoglobin is fetal hemoglobin.
B Fetal hemoglobin binds oxygen less tightly than adult hemoglobin.
C Oxygenated fetal hemoglobin is blue rather than red.
D Fetal hemoglobin prevents sickling of sickle cells.
ANSWER:D
270
A 12-month-old infant presents to her primary care provider’s office with respiratory
distress. The normal respiratory rate of a 1-year-old child is which of the following?
A over 80 breaths per minute
B between 60 and 80 breaths per minute
C between 35 and 50 breaths per minute
D between 20 and 30 breaths per minute
ANSWER:D
271
A pediatric cardiologist is reviewing the elec- trocardiogram (ECG) of a term infant with
tachycardia with a group of pediatric residents. As compared to older children and adults, the
ECG of an infant normally shows which of the following findings?
A a shorter RR and shorter PR interval
B a shorter RR and longer PR interval
C a longer RR and shorter PR interval
D a longer RR and shorter PR interval
ANSWER:A
272
A 4 week male with a temperature of 101F°has undergone an evaluation for meningitis. The
results of his cerebrospinal fluid (CSF) testing return and while no white blood cells are seen
and the glucose is normal, the protein is recorded as high at 90 mg/dL. The concentration of
pro- tein in the CSF of infants during the first few weeks of life normally may be as high as
which of the following?
A 20 mg/dL
B 45 mg/dL
C 125 mg/dL
D 500 mg/dL
ANSWER:C

273
The parents of a 4-year-old boy are concerned because his speech is still not fully
understand- able. Which of the following best describes a child’s conversation at 4 years of
age?
A fully understandable, although mispro- nunciations and grammatical errors are common
B fully understandable, with few if any mispronunciations and grammatical errors
C fully understandable to the parent but not necessarily to others
D somewhat understandable although garbled and indistinct
ANSWER:A
274
A 7-year-old girl comes to her pediatrician’s office because her teacher is concerned that she
may have attention-deficit hyperactivity dis- order (ADHD). Hallmarks of ADHD include
which of the following?
A Often quiet demeanor
B History of seizures at birth
C History of recurrent ear infections
D Symptom occurrence in more than one environment
ANSWER:D
275
The father of 2- and 8-year-old siblings calls their pediatrician to discuss how he and the
childrens’ mother can prepare them for the par- ents’ upcoming divorce. Which of the
following statements is true regarding children of divorce?
A The emotional availability of one or both parents is often heightened.
B On average, girls have a more difficult time adjusting initially to divorce than boys.
C
As boys reach young adulthood, they may experience anxiety about male- female
relationships.
D Preschool-aged children may demon- strate regressive behavior
ANSWER:D
276
The mother of an otherwise healthy infant noticed recently that she could not identify the
soft spot in her infant anymore. Based on this piece of his- tory, this child is most likely to be
what age?
A 3 months
B between 3 and 9 months
C between 9 and 18 months
D between 18 and 24 months
ANSWER:C

277
The parents of a 3-month-old boy who is grow- ing well come to the pediatrician’s office for
a well-child check. Which of the following guid- ance items regarding growth and
development are appropriate to discuss at this visit?
A may leave infant unattended on couch or changing table as long as he is not rolling over yet
B caution against leaving small items on the floor because infant could pick them up
C infant still needs to be fed every 2 hours at night
D expect him to coo, smile and laugh
ANSWER:B
278
A 16-year-old girl presents to her primary care provider’s office requesting a pregnancy test
due to a late period. Which of the following factors are associated with an increased risk of
teen pregnancy?
A poor academic performance
B rigid parental support and supervision
C living in a middle class community
D history of drug or alcohol abuse in parent
ANSWER:A
279
A medical student is describing the growth pat- tern of a child with growth hormone (GH)
defi- ciency. Children with isolated GH deficiency usually have which of the following?
A have a normal bone age
B have associated mild hypothyroidism
C grow parallel to, but below, the normal growth curve
D show deceleration of growth velocity and fall away from the growth normal curve
ANSWER:D
280
Parents bring a 10-year-old boy in for evaluation of short stature. The patient is worried
about his eventual adult height. Children with consti- tutional growth delay (without
endocrine abnor- mality) generally can expect to ultimately achieve which of the following
growth descriptions?
A very short and obese
B short but of proportionate weight
C of normal height and weight
D very tall but of proportionate weight
ANSWER:C

281
The parents of a 7-year-old girl report that she would rather watch TV than play outside. It is
estimated that the average school-age American child watches television for about how many
hours a week?
A 4–8 h/week
B 10–14 h/week
C 15–20 h/week
D 35–40 h/week
ANSWER:D
282
The inpatient pediatric team is discussing the causes of failure to thrive (FTT) as they evalu-
ate a 9-month-old infant with weight and length below the 5th percentile. Which of the
following should be included in the differential diagnosis of infant who is not thriving?
A Small atrial septal defect
B Acute bacterial meningitis
C Cystic fibrosis
D Recurrent otitis media
ANSWER:D
283
High school teachers are listening to a lecture about the features of adolescence. Which of the
fol- lowing best describes the period of adolescence?
A immediately before, during, and after puberty
B of maximal physical growth
C of maximal sexual development
D of psychosocial transition from child- hood to adulthood
ANSWER:D
284
The mother of a 16-year-old girl reports that she has noticed declining scholastic grades,
refusal to participate in family activities, and an unwillingness to communicate with either
par- ents or peers. These findings in an adolescent usually indicate which of the following?
A a normal stage of development
B a normal response to peer pressure
C a normal reaction to overprotective parents
D an emotional or psychiatric problem
ANSWER:D
285
The parents of a 2-year-old boy report he is about to be thrown out of daycare due to biting
other children. For the primary care provider, which of the following facts are important to
inquire about regarding biting behavior?

A Age when parents introduced solid food
B Toileting behavior
C Age at which child’s first tooth erupted
D Concerns about the child’s developmen- tal progress
ANSWER:D
286
A hemoglobin and hematocrit is obtained from a 12-month-old who came to his
pediatrician’s office for a checkup. Which of the following best describes the normal
(average) hemoglo- bin concentration at 1 year of age?
A 17 g/dL
B 15 g/dL
C 12 g/dL
D 10 g/dL
ANSWER:C
287
A group of medical students on the newborn nursery rotation are discussing the sensory and
physical abilities of a newborn. Which of these abilities is considered normal?
A inability to sense pain
B preference for geometric shapes over mother’s face
C 20/20 vision
D grasping a caregiver’s finger
ANSWER:D
288
The parents of a 4-year-old with recurrent wheezing are both smokers. They inquire about
the child’s likelihood of becoming a smoker when he reaches adulthood. The phenomenon
of children incorporating habits demonstrated by the adults in their lives is known by which
of the following terms?
A time in
B incidental learning
C conditioned reinforcement
D fading
ANSWER:B
289
The father of a 2-month-old boy wonders why his son’s head looks so big. Which of the fol-
lowing best describes the head growth during the first month of life?
A 0.5 cm
B 1.25 cm
C 2 cm
D 5 cm

ANSWER:C
290
The mother of an 11-year-old girl who comes to her pediatrician’s office for a checkup
notes that her daughter is beginning to develop breast buds. She asks when her daughter
might have her first period. Which of the following is a true statement regarding menarche in
the adolescent girl?
A precedes the spurt in linear growth
B occurs simultaneously with Tanner stage II breast development
C
generally occurs when Tanner stage III breast and pubic hair development have been
achieved
D occurs simultaneously with Tanner stages IV to V pubic hair and breast development
ANSWER:D
291
A pediatric psychologist is discussing the con- cept of temperament with a group of
residents. Which of the following best describes the term temperament?
A A parent’s response to a tantrum.
B The emotional bond that a child feels with his parent.
C The acts of independence that a child demonstrates as she enters toddlerhood.
D Stable, individual modes of responding to the environment.
ANSWER:D
292
A 3-year-old male child from South America is brought in for an adoption physical examina-
tion. He has a 4–5 mm, foamy, mildly vascular, conjuctival lesion next to the cornea on the
medial side of both eyes. Which nutrient defi- ciency is most likely to cause this lesion?
A vitamin A
B vitamin B6
C thiamine
D copper
ANSWER:A
293
Parents appear in your office to discuss their considerations for adopting their 11-month-old
foster son. They state he has a history of failure to thrive at 6 months of age and they are con-
cerned about the possibility of long-term prob- lems. Undernutrition in the first year of life
has which of the following effects?
A has no permanent effect on physical growth or development of intelligence
B can have permanent effects on physical growth but not on development of intelligence
C can have permanent effects on develop- ment of intelligence but not on physical growth
D can have permanent effects on both physical growth and development of intelligence

ANSWER:D
294
At a 4-month well child visit, a mother is dis- tressed because her baby’s grandmother
believes she should have started feeding her baby solid food at 2 months of age. It is
generally recom- mended that beikost (infant foods other than milk) be introduced into the
infant’s diet at about what age?
A 3 weeks
B 6 weeks
C 3 months
D 6 months
ANSWER:D
295
A 23-month-old female is hospitalized for severe malnutrition. Which of the following
problems is most likely to be the result of inap- propriately rapid treatment of the severely
mal- nourished child?
A hyperkalemia
B hyponatremia
C hyperglycemia
D congestive heart failure
ANSWER:D
296
A 10-week-old child weighing 5 kg is being fed commercial infant formula. The mother is
concerned she is underfeeding her baby. You tell her that, to satisfy both his fluid and
caloric requirements, the daily intake ought to be at least how many ounces?
A 12 oz
B 18 oz
C 26 oz
D 36 oz
ANSWER:C
297
A pregnant African-American woman plans to breastfeed her baby. She has read of the nutri-
tional, psychologic, and immunologic advan- tages, but recalled breastfed babies might still
require vitamin supplementation. The most cur- rent nutritional recommendations indicate
she should supplement the baby’s nutrition with which vitamin?
A vitamin A
B vitamin B
C vitamin C
D vitamin D
ANSWER:D

298
You are in charge of the nutrition support serv- ice in the hospital where you are practicing.
The company that makes the vitamin/mineral mixture for total parenteral nutrition (TPN) has
gone out of business and for the past 2 months you have been trying to fortify the parenteral
nutrition mixtures through other sources. An 8-month-old infant with short gut, who has
been on TPN for the past 4 months, presents with a curious erythematous desquamating rash
around his mouth, buttocks, and a small amount on his hands and feet. Which nutrient
deficiency is the most likely cause?
A vitamin A
B thiamin
C riboflavin
D zinc
ANSWER:D
299
A physical examination of a 10-year-old Hispanic male reveals BMI greater than 95th per-
centile. Laboratory test results include fasting serum glucose of 98 mg/dL, fasting serum
insulin of 30 µU/mL, an ALT of 60 U/L, total cholesterol of 128 mg/dL, and triglycerides of
130 mg/dL. You have great concern that he may have which of the following?
A diabetes
B prediabetes
C nonalcoholic steatohepatitis
D metabolic syndrome
ANSWER:C
300
At 1 year of age, a boy was at the 50th percentile for height and weight. At 2 years of age,
he is at the 25th and 10th percentiles respectively. Review of systems reveals fussiness, loose
stools, and possibly stomach aches all beginning after the mother stopped breast- feeding the
boy and introduced table foods. Mother has consumed milk products lifelong, but her son
does not drink cow milk. Which is the most likely cause of these symptoms?
A toddler’s diarrhea
B lactose intolerance
C celiac disease
D cow milk protein allergy
ANSWER:C

301
At 1 year of age, a boy was at the 50th percentile for height and weight. At 2 years of age,
he is at the 25th and 10th percentiles respectively. Review of systems reveals fussiness, loose
stools, and possibly stomach aches all beginning after the mother stopped breast- feeding the
boy and introduced table foods. Mother has consumed milk products lifelong, but her son
does not drink cow milk.The most definitive test for this condition is which of the following?
A breath hydrogen test
B abdominal ultrasound
C serum antigliadin antibodies
D small bowel biopsy
ANSWER:D
302
At 1 year of age, a boy was at the 50th percentile for height and weight. At 2 years of age,
he is at the 25th and 10th percentiles respectively. Review of systems reveals fussiness, loose
stools, and possibly stomach aches all beginning after the mother stopped breast- feeding the
boy and introduced table foods. Mother has consumed milk products lifelong, but her son
does not drink cow milk. Patients with this condition should avoid which of the following?
A corn
B barley and rye
C rice and legumes
D oats
ANSWER:B
303
A 5-year-old female presents with a 3-day history of crampy abdominal pain and a rash on
her lower legs. On examination, she is cranky but nontoxic. Her abdomen is mildly diffusely
tender with normal bowel sounds. A purpuric rash is located predomi- nately on the extensor
surface of lower legs. A com- plete blood count and urine analysis are normal. Which of the
following is the best next step?
A intravenous corticosteroid therapy
B intravenous antibiotics
C hematology-oncology consult
D sickle cell screening
ANSWER:A

304
A 5-year-old female presents with a 3-day history of crampy abdominal pain and a rash on
her lower legs. On examination, she is cranky but nontoxic. Her abdomen is mildly diffusely
tender with normal bowel sounds. A purpuric rash is located predomi- nately on the extensor
surface of lower legs. A com- plete blood count and urine analysis are normal. While
hospitalized, the child’s abdominal pain increases. She becomes pale and draws up her legs.
On examination, distension of the right upper quadrant with palpable fullness is found.
Which of the following is true of this complication?
A recurrence occurs in at least 15% of children
B rupture of the appendix may be an asso- ciated complication
C usually starts with stool containing blood-stained mucus
D ileoileal involvement is typical
ANSWER:D
305
Because her son prefers other beverages over milk, a school-age boy’s mother is concerned
he may not be getting enough calcium in his diet. As you recommend calcium-rich foods,
you mention the calcium requirement for a school- age child is which of the following?
A 0.1 g/day
B 1.3 g/day
C 10.5 g/day
D 50.0 g/day
ANSWER:B
306
During a preschool physical, the mother of a 5-year-old girl states the family follows a strict
vegan diet (a diet excluding all animal products including eggs, milk, and milk products, as
well as meats and fish). In which of the following vitamins is this child likely to be deficient?
A vitamin C
B vitamin E
C vitamin B1
D vitamin B12
ANSWER:D
307
At a 2-month well child visit, a mother states her family has a history of cow milk protein
allergies and wonders if soy formula would be better for her baby. Which of the following
statements regarding soy formula is correct?
A
Infants fed exclusively with soy formula display growth comparable to infants fed cow milk
protein formula.

B
The protein in soy formulas is essen- tially nonallergenic, and clinically signif- icant soy
protein hypersensitivity is extremely rare.
C
Soy formula is most useful in children with well-documented, severe, gastroin- testinal
allergic reactions to cow milk protein.
D Soy formula should not be used by patients with a family history of celiac disease.
ANSWER:A
308
A 5-month-old male presents with poor weight gain. His diet consists only of goat milk. On
examination, he appears tired and is mildly tachycardic. The laboratory value most likely to
be elevated is which of the following?
A serum albumin
B red blood cell mean corpuscular volume
C hemoglobin
D serum vitamin B12 level
ANSWER:B
309
An 8-month-old female is hospitalized with fail- ure to thrive. She has a 1-day history of
fever and cough. On physical examination, you observe a very thin, well-hydrated alert
infant in no acute distress. You obtain a complete blood count and order a chest x-ray for
further evaluation of a one day history of fever and cough. You note the lung fields to be
normal but also note an absence of a thymic shadow. Which of the following should be your
next step?
A obtain an immunology consult
B order genetic karyotyping
C repeat the chest x-ray in 24 hours
D feed the infant as tolerated, following weight and intake closely
ANSWER:D
310
A single, 35-year-old pregnant female has not been tested previously for HIV or hepatitis.
While awaiting prenatal screening, she requests information regarding breastfeeding. You
would most accurately tell her in the United States breastfeeding of infants is contraindicated
in which of the following?
A an infant whose mother is seropositive for HIV antibody
B an infant whose mother is seropositive for hepatitis B surface antigen
C an infant whose mother is seropositive for hepatitis C antibody
D
an infant whose mother was recently immunized with tetanus, diphtheria, acellular pertussis
vaccine
ANSWER:A

311
A new mother at her baby’s 2-week well visit is concerned that she may not be able to
continue breastfeeding because her nipples are very sore and cracked, and have even bled.
Inquiring about her feeding techniques, you discover she nurses her baby about 20–25
minutes on each breast each feeding. Which of the following offers the best supportive
advice?
A Lengthen the time interval between every nursing by at least an hour.
B Nurse for a minimum of 30 minutes on each breast each feeding.
C
Offer 1 oz of formula from a bottle before each breastfeeding so that her infant will not be as
ravenous.
D Limit nursing to 5–10 minutes per breast per feeding.
ANSWER:D
312
A mother is concerned because her 20-month- old son prefers to eat with his fingers rather
than use a small spoon. Which of the following statements is correct?
A At 18 months of age, most toddlers pre- fer to feed themselves with a spoon.
B Most children learn to feed themselves independently during the second year of life.
C
Self-spoon-feeding usually begins at 10 months of age when well-defined wrist rotation
develops.
D Most children can feed themselves with a cup by 6 months of age.
ANSWER:B
313
You are following a child with ADHD. His teacher requests information on diet and ADHD.
In particular, she is interested in the role of ingestion of sugar in this condition. Which of the
following statements is true regarding controlled studies of the effects of sucrose ingestion in
children?
A Sucrose ingestion has been linked to problem behavior in children with ADHD.
B Sucrose ingestion has been linked to increased motor activity in children with ADHD.
C
Sucrose ingestion is associated with an increase in both motor activity and problem behavior
in children with ADHD.
D
Sucrose ingestion is not associated with problem behavior or increased motor activity in
children.
ANSWER:D
314
You are in a health clinic in Africa. An 18-month- old child is brought in for your help. On
inspection you note that the child is edematous, has a dark desquamating skin rash over most
of the pres- sure points on the body, has very thin hair, and what hair is left is reddish in
color. Of the following, what laboratory test is most likely to be present?
A serum albumin of 3.7 g/dL

B serum sodium of 143 meq/L
C serum albumin of 2.3 g/dL
D serum potassium of 2 meq/L
ANSWER:C
315
Recent concerns about the prevention of ather- osclerotic heart disease have led to
recommen- dations for reducing the fat content, and particularly the cholesterol content, of
the diet early in life. This can include the use of skim milk. It is recommended that the use of
skim milk begin no earlier what age?
A 4 months
B 6 months
C 1 year
D 2 years
ANSWER:D
316
Because of her own success with a low carbo- hydrate diet, the mother of an overweight 9-
year-old asks about a low carbohydrate diet for her son. As you discuss this with her, you
point out that the greatest percentage of calo- ries in the normal diet of the school-age child
comes from which of the following?
A carbohydrate
B protein
C fat
D cholesterol
ANSWER:A
317
A 3-month-old Caucasian female is brought to you for cold symptoms. On examination, you
note her head circumference is at the 50th percentile, her length at the 25th percentile, and
her weight less than the 5th percentile. She appears alert but very thin. She has skin folds on
her arms, thighs, and buttocks. Parents state she drinks four 8 oz bottles of premixed for-
mula each day. You feed the baby and she quickly drinks 7 oz. You decide to admit the
infant. How many calories will she likely need for weight gain?
A 60 kcal/kg/day
B 80 kcal/kg/day
C 100 kcal/kg/day
D 150 kcal/kg/day
ANSWER:D

318
The mother of a headstrong 2-year-old is con- cerned that he insists on drinking six bottles
of apple juice per day. Which of the following rep- resents a serious nutritional concern in
this situation?
A development of obesity
B development of diabetes mellitus
C vitamin C deficiency
D vitamin A toxicity
ANSWER:A
319
A 3-year-old male presents to your clinic for the first time for a well-child visit. You note
macro- cephaly, short fingers, and only six teeth. What additional finding is likely?
A absent thumb
B flat facial bones
C hepatosplenomegaly
D absent clavicles
ANSWER:D
320
An 18-month-old Hispanic male presents for a well-child check. His parents report he drinks
seven to eight bottles of cow milk each day and does not like solid foods. His weight is
greater than the 95th percentile while height and head circumference are at the 25th
percentile. His heart rate is 180 beats per minute and respira- tory rate is 28 breaths per
minute. On exami- nation, you find a chubby, happy, pale, and otherwise normal baby.
Which of the following tests will most likely be abnormal?
A sweat test
B serum sodium
C eosinophil count
D hemoglobin
ANSWER:D
321
A 7-year-old male presents for a well-child visit. You note his height is at 75th percentile and
weight is at greater than 95th percentile for age. His examination is otherwise normal. His
father is overweight but his mother is thin. Which of the following is the most appropriate
next step for you to take?
A obtain T4 and TSH levels
B order serum chromosomes
C refer to an endocrinologist
D counsel the family on increasing exer- cise and eliminating target foods from diet
ANSWER:D

322
The most accurate method for assessing adi- posity in the office setting is which of the fol-
lowing?
A measurement of weight
B calculation of percent above ideal body weight for height using age/sex norms
C calculation of body mass index (weight/height2)
D measurement of subcutaneous fat thickness
ANSWER:C
323
A 2-year-old Caucasian male presents with fail- ure to thrive, chronic diarrhea, and recurrent
pneumonia. Though his family history is neg- ative for cystic fibrosis, a sweat test reveals a
sodium concentration of 120 mg/dL. Which of the following is the appropriate next step in
caring for this infant?
A iron
B vitamin B12 and folic acid
C pancreatic enzyme supplementation
D copper and magnesium
ANSWER:C
324
A 15-year-old Caucasian female presents with the complaint that she is always cold. She has
no history of illness, but she has not had a period for 4 months. Her mother states she is
worried because although her daughter inter- mittently eats a large amount of food, she
seems to be losing weight. On examination, the patient is extremely thin for her age. Which
of the following is most likely?
A She feels she is too thin, but likes it that way.
B She feels she is too thin, and would like to gain weight.
C She feels she is too fat, and wants to lose weight.
D She feels she is too fat, but likes it that way.
ANSWER:C
325
A new mother asks you how much formula her 3 kg infant should be consuming daily. To
give her an idea on how much fluid the child should be taking, you calculate out the approx-
imate daily fluid requirement. Which of the following most closely resembles this child’s
daily fluid requirements?
A 900 mL
B 700 mL
C 500 mL
D 300 mL
ANSWER:D

326
A 30 kg child you are taking care of in the hos- pital is receiving IV fluids in preparation for
surgery. You are trying to determine if the fluids are running at an appropriate rate for daily
maintenance. What do you determine is the approximate daily fluid requirement for this
child?
A 3000 mL
B 2400 mL
C 1700 mL
D 1200 mL
ANSWER:C
327
A 50 kg child you are taking care of has just received replacement for her fluid deficit. You
are now trying to determine the hourly fluid rate needed to supply maintenance fluid
requirements. Which number below most accu- rately reflects the correct mL/h?
A 125 mL/h
B 115 mL/h
C 100 mL/h
D 90 mL/h
ANSWER:D
328
An infant presents with a weight of 8 kg and a 3-day history of diarrhea and vomiting. He
appears severely dehydrated with decreased sensorium, sunken fontanelle, poor skin turgor,
and decreased urine output. Which of the fol- lowing most closely estimates the fluid deficit
of this child?
A 1900–2000 mL
B 1300–1500 mL
C 900–1100 mL
D 400–500 mL
ANSWER:B
329
A 10-month-old male infant had multiple episodes of vomiting and diarrhea over the last 24
hours. The infant now has slightly sunken eyes, mildly decreased activity, and dry skin. Vital
signs are stable. Which of the following is the generally preferred method for rehydra- tion
for this patient?
A oral administration of a solution containing 75 meq/L of sodium and 5 g/dL of glucose
B oral administration of a solution containing 50 meq/L of sodium and 2 g/dL of glucose
C oral administration of a solution containing 35 meq/L of sodium and 10 g/dL of glucose
D
intravenous administration of a solution containing 154 meq/L of sodium and 5 g/dL of
glucose

ANSWER:A
330
A 16-year-old male is brought to the emergency room by his friends. They relate they were
drinking alcohol and that their friend “passed out” about 2 hours ago and is increasingly dif-
ficult to arouse. Which of the following is most useful in your immediate management of this
patient?
A serum sodium
B serum glucose
C blood alcohol level
D serum calcium
ANSWER:B
331
You are discussing the family history with the father of a boy who is new to your practice.
During your discussion the father reports that the family had a daughter who died at age 9
months from complications associated with glycogen storage disease. That child had muscle
weakness, hypotonia, and severe cardiomegaly. She did not have significant hyperlipidemia,
gout, skeletal involvement, or frequent bacterial infections. Which glycogen storage disease
was most likely in the deceased child?
A type Ia (von Gierke disease)
B type Ib (glucose-6-phosphatase microsomal transport defect)
C type II (Pompe disease)
D type III (debranching enzyme deficiency)
ANSWER:C
332
A 3-day-old infant with severe jaundice, hypo- glycemia, and anemia is further noted to
have liver nodules and cirrhosis. Serum tyrosine and methionine levels are markedly
elevated. What enzyme defect is causing this child’s liver failure?
A tyrosine aminotransferase
B phenylalanine hydroxylase
C fumarylacetoacetate hydrolase
D malylacetoacetate isomerase
ANSWER:C
333
In counselling a family regarding the diagnosis of alcaptonuria, you state that is a metabolic
dis- ease caused by a defect in or lack of homogen- tisic acid oxidase. The excess
homogentisic acid leads to which clinical finding?
A kinky hair
B absent patella
C black urine and darkly pigmented sclera, cornea, and ears

D blue sclera
ANSWER:C
334
A child with mental retardation is also noted to have severe myopia. On examination, sublux-
ation of the ocular lens (ectopia lentis) is found. This generally occurs after 3 years of age in
children with which of the following?
A hawkinsinuria
B tyrosinemia
C phenylketonuria
D homocystinuria
ANSWER:D
335
An 8-month-old Caucasian male is brought with concerns of a 4-day history of fever and
decreased food intake. The mother states that he has slept most of the day. On examination,
the patient is febrile with otherwise normal vital signs. He does not open his eyes during the
examination. The liver is noted to be slightly enlarged. His laboratory values show normal
electrolytes without acidosis. The serum glu- cose is 33 mg/dL. Liver enzymes as well as
ammonia are slightly elevated. Serum insulin is undetectable. A simultaneous urine sample is
negative for ketones. Which of the following conditions is the most likely diagnosis?
A sulfonylurea ingestion
B glycogen storage disease type I
C accelerated starvation
D medium-chain acyl-CoA dehydrogenase deficiency
ANSWER:D
336
A 15-kg toddler with group A coxsackievirus infection is refusing to drink. His serum potas-
sium is normal. For maintenance of intra- venous fluids, what concentration of KCl should
be added?
A 5 meq KCl/L
B 10 meq KCl/L
C 25 meq KCl/L
D 40 meq KCl/L
ANSWER:C

337
A 12-year-old male is brought to your office for concerns regarding excessive drinking. The
family states that the excessive drinking is also caus- ing him to urinate frequently. His
weight has decreased 6 kg in the past 5 months. His current medications include albuterol as
needed as well as lithium. A urine analysis shows no glucose, ketones, or evidence of a
urinary tract infec- tion. The specific gravity is less than 1.005. Further work-up shows the
serum sodium to be 163 meq/L, with a normal potassium and glu- cose. BUN is mildly
elevated. Urine osmolality is less than serum osmolality. The urine concentrates only
minimally when parenteral DDAVP is administered. What is the most helpful treatment for
the patient’s condition?
A fludrocortisone
B fluid restriction
C intranasal DDAVP
D hydrochlorothiazide
ANSWER:D
338
One of your patients recently underwent splenectomy due to an autosomal recessive gly-
colytic enzyme deficiency. Which of the fol- lowing is a common clinical sign in neonatal
presentation of this disorder?
A sepsis
B jaundice
C hepatomegaly
D hypertonia
ANSWER:B
339 What is the most likely clinical finding in an 8-year-old with Wilson disease?
A hepatitis-like illness
B acral cyanosis
C polyuria
D behavioral changes
ANSWER:A
340
A 2-month-old infant is noted to have bilateral cataracts. The remainder of the infant’s exam-
ination is normal; and developmentally she is on target. What early dietary adjustment could
have prevented the development of cataracts in this child?
A eliminating gluten
B providing low iron formula
C eliminating milk and milk products
D providing increased amounts of milk and milk products

ANSWER:C
341
A 10-month-old male infant with a history of umbilical hernia repair is noted to have coarse
facial features and is diagnosed with Hurler dis- ease. This condition is the most severe of
which of the following groups of inherited diseases?
A glycogen storage diseases
B glycoproteinoses
C mucopolysaccharidoses
D sphingolipidoses
ANSWER:C
342
You are obtaining the history of a 7-year-old male, who presents to your office with a
several-week history of ataxia and visual changes. Previously he has been treated for
attention deficit disorder. As you are about to examine the child he begins to seize. An X-
linked peroxisomal disease is sus- pected. Plasma elevation of which of the follow- ing
substances confirms your suspicions?
A lead
B iron
C short chain fatty acids
D very long chain fatty acids
ANSWER:D
343
Pyruvate dehydrogenase complex deficiency is an autosomal recessive disease and leads to
the development of an anion gap acidosis. Which of the following accumulates to result in an
elevated anion gap?
A beta-hydroxybutyric acid
B acetoacetic acid
C lactic acid
D hydrochloric acid
ANSWER:C
344
You are evaluating a 10-month-old infant for recurrent fractures following relatively minor
trauma. You note deep blue sclera and bowing of the lower extremities. X-ray examination
reveals generalized osteopenia. Which of the following is the most likely diagnosis?
A achondroplasia
B histiocytosis X
C osteogenesis imperfecta
D osteopetrosis

ANSWER:C
345
A 5-week-old male presents with poor feed- ing, poor growth, a peculiar odor, hypertonia,
and hyperactive reflexes. He is afebrile. History reveals no problems with labor and delivery
and early hospital discharge at 24 hours of age. He has not seen a physician since that time.
Which of the following is the most likely etiol- ogy of this infant’s condition?
A sepsis
B pyloric stenosis
C overfeeding
D phenylketonuria (PKU)
ANSWER:D
346
The most important part of management of the infant described above is to restrict consump-
tion of which of the following?
A iron
B complex carbohydrates
C short chain fatty acids
D phenylalanine
ANSWER:D
347
A 2-week-old female is admitted to the hospital for jaundice, hypoglycemia, and acidosis.
She has also recently been febrile and blood cultures are positive for Escherichia coli. The
patient is most likely deficient in which enzyme?
A galactokinase
B galactose-1-phosphate uridyl transferase
C pyruvate dehydrogenase
D galactose-1-phosphate dehydrogenase
ANSWER:B
348
You are seeing a 10-month-old Caucasian female whose mother brought her in for
wheezing. This is the child’s third visit in the past 2 months for the same complaint. Today
you also notice that her weight is less than the 3rd percentile while her length is at the 25th
percentile. What is the first choice for diagnosing the suspected disease?
A pancreatic biopsy
B molecular genetics
C evaluate stool sample for fecal fat
D sweat chloride test
ANSWER:D

349
The parents of a 3-day-old male infant present to the emergency department with their child.
They report that he was born without complica- tion and was previously doing well. Today,
he became very lethargic and started vomiting feeds. Mother reports that she had a brother
who died suddenly as a neonate. An ammonia level was drawn and was noted to be
markedly elevated. You suspect ornithine transcarbamylase defi- ciency. What is the
probability of these parents having future offspring effected by this disease?
A 50% of males would be severely affected.
B 25% of males and 25% of females would be affected.
C All offspring would be affected.
D 50% of males and 50% of females would be severely affected.
ANSWER:A
350
A 13-year-old female presents to the emergency department with a 3-day history of fever
above 104°F (40°C); vomiting; diarrhea; and diffuse, erythematous rash. She is found to
have ortho- static hypotension. Laboratory evaluation reveals decreased platelets and
elevated liver and renal function tests. The mother of the child informs you that the child is
currently menstruating. You suspect that this child has a toxin mediated infec- tion. What is
the most likely etiology of this toxin mediated infection?
A Streptococcus pyogenes
B Staphylococcus aureus
C Neisseria gonorrhoeae
D Streptococcus agalactiae
ANSWER:B
351
What are additional features commonly associ- ated with the toxin-mediated process
described in Question 1?
A pneumonia and pleural effusion
B arthritis and myositis
C exudative tonsillitis and cervical adenitis
D thrombocytopenia and conjunctival hyperemia
ANSWER:D
352
Hilar lymphadenopathy is noticed on the chest x-ray obtained for a 6-year-old male known
to have mild intermittent asthma. The child is from rural Ohio and lives on a farm with
chickens. Tuberculin skin testing is negative and the family denies history of exposure to
tuberculosis. You suspect histoplasmosis infection. What is the most likely mode of
transmission of the spores?
A ingestion

B direct contact from another person
C inhalation
D skin inoculation
ANSWER:C
353
You are seeing a healthy, full-term infant just after delivery. The mother plans on breast-
feeding the baby and asks you how long immunity transferred to the baby from the
pregnancy would be present. When does passively trans- ferred maternal IgG antibody reach
a nadir?
A 1–2 months
B 3–6 months
C 12–18 months
D 18–24 months
ANSWER:B
354
The mother from Question 3 also tells you that she has read that breast-feeding immunity is
different than that acquired in utero. What is the major source of immunity conferred in
breast milk?
A T-cell mediated
B complement mediated
C IgM
D IgA
ANSWER:D
355
The mother of a 3-year-old child in your office for a well-child visit mentions that her child
has been scratching his bottom lately. The mother reports to you that on a few occasions she
has seen what appears to be rice in his diaper. Which of the following is the most likely
explanation?
A Enterobius vermicularis infection.
B The child has malabsorption disorder.
C The child eats rice frequently.
D Strongyloides stercoralis infection.
ANSWER:A
356
You were called to the neonatal nursery to see an infant with the following features: growth
retardation, rash, and absent red reflexes. On your examination, the baby is very small, with
bluish purpuric skin lesions, and obvious cataracts. The baby also has a heart murmur. You
suspect congenital infection. What is the most likely etiology?
A HIV

B toxoplasmosis
C CMV
D rubella
ANSWER:D
357
A 6-year-old unimmunized child has fever of 104°F (40°C) and cropped vesicles on the
trunk with scattered scabbed lesions. Which of the following infections is the likely diagnosis?
A measles
B smallpox
C HHV-6
D varicella
ANSWER:D
358
The mother of a 6-month-old child seeing you for a well visit, requests immunization against
measles as she has heard it is present in the community. At what age should the first dose of
live attenuated measles vaccine (as MMR) be routinely administered in the absence of a com-
munity measles outbreak?
A 6 months of age
B 9 months of age
C 12–15 months of age
D 18–24 months of age
ANSWER:C
359
A 12-month-old child presents to your office for a well-child visit. The mother informs you
that at the 6 month visit the child had fever and localized arm swelling after diphtheria-
tetanus- acellular pertussis (DTaP) vaccine. She is won- dering if this vaccine should be held.
Which of the following is a contraindication to receiving DTaP immunization?
A encephalopathy within 7 days of previ- ous dose
B family history of seizures
C for a preterm infant who is 2 months of age
D an internationally adopted child
ANSWER:A
360
An 8-year-old child presents to the emergency department with a 3-day history of fever and
new altered mental status. Her parents report that they have been camping several times and
she has sustained a lot of mosquito bites. Mosquitoes are recognized as vectors in the
transmission of encephalitis of which of the following viruses?

A arbovirus
B coxsackievirus
C enterovirus
D influenza virus
ANSWER:A
361
A malnourished 3-year-old child from Mexico has a positive tuberculin skin test (TST > 10
mm for this age). Which of the following would be most concerning for extra-pulmonary
disease?
A fever
B hilar lymphadenopathy on roentgeno- graph
C night sweats
D hepatosplenomegaly
ANSWER:D
362
A 15-year-old female comes to your office with 4-day history of fever and cough. Chest x-
ray findings include bilateral patchy infiltrates. Oxygen saturation is normal. What is the
most likely etiology of her pneumonia?
A Streptococcus pneumoniae
B Staphylococcus aureus
C Mycoplasma pneumoniae
D Chlamydophila psittaci
ANSWER:C
363
A 3-month-old infant has had upper respira- tory symptoms for a few days and presents to
the emergency department with respiratory distress, wheezing, and hypoxia. You diagnose
bronchiolitis. What is the most common cause of acute bronchiolitis in infants?
A respiratory syncytial virus
B parainfluenza virus
C cytomegalovirus
D influenza virus
ANSWER:A
364
On tuberculin skin testing, a well 5-year-old who traveled last year to Mexico is found to
have 8 mm of induration to 5 tuberculin units (TU) of purified protein. There is no history
suggestive of contact with tuberculosis. What does this reaction likely indicate?
A sensitivity to the diluent
B subcutaneous rather than intradermal injection of test material
C cross-sensitivity to nontuberculous mycobacteria

D tuberculosis infection
ANSWER:C
365
A 4-year-old child presents with fever, night sweats, weight loss, and cough. A diagnosis of
pulmonary tuberculosis infection was made based on chest x-ray and tuberculin skin test-
ing. The mother of the child asks when her child may return to day care. When can a child
diagnosed with pulmonary tuberculosis infec- tion return to day care or school?
A as soon as effective therapy has been instituted
B 2 weeks after therapy is initiated
C once negative sputum smears are confirmed
D once therapy is completed
ANSWER:A
366
A 7-year-old child who recently traveled with her family to India presents to your office
with a 2-day history of fever, diarrhea, and tenesmus. Stool examination reveals blood and
leukocytes. You suspect infection with Salmonella typhi as the cause of her symptoms. What
additional signs/symptoms are typically found with this infection?
A rectal prolapse
B hepatosplenomegaly and abdominal pain
C intensely pruritic skin rash
D cough and lymphadenopathy
ANSWER:B
367
You are seeing an 8-month-old child in your office for recurrent thrush, candida diaper der-
matitis, and multiple recent skin pustules. In addition, he was recently hospitalized with a
hepatic abscess. You suspect that this child may have chronic granulomatous disease. Which
of the following defense mechanisms is defective and the cause for recurrent infections in
chronic granulomatous disease?
A leukocyte migration
B synthesis of collagen
C capillary permeability
D phagocyte oxidative burst
ANSWER:D
368
A mother brings her 11-month-old infant along with her 3- and 9-year-old children into
your office for a several day history of pruritic rash. You note the rash is similar in the two
older children with burrows characteristic of scabies in between the fingers, and on the flexor
sur- faces of the wrists. Which of the following rep- resents the typical manifestation in
infants?

A a maculopapular rash
B concentration of burrows in the anterior axillary folds
C characteristic sparing of the face
D a papulovesicular rash
ANSWER:D
369
You are seeing a 4-year-old child in the emergency department with abdominal pain, and
acute onset of bloody diarrhea. Due to a recent outbreak in the community related to a
popular fast food restau- rant, you suspect E coli 0157:H7. Which of the fol- lowing is seen
most commonly as a complication of Shiga-toxin-producing E coli (formerly known as
enterohemorrhagic E coli) diarrhea?
A meningitis
B hemolytic-uremic syndrome
C chronic diarrhea
D endocarditis
ANSWER:B
370
A full-term infant is born to a mother without previous prenatal screening. Laboratory eval-
uation obtained at delivery reveals a positive RPR at 1:32. You carefully examine the infant
after birth to evaluate for cutaneous manifes- tations of congenital syphilis. Which of the fol-
lowing cutaneous findings would be most characteristic of congenital syphilis?
A that the lesions are sterile
B that lesions are most numerous on the trunk
C a fleeting pink macular rash
D vesiculobullous lesions of the palms and soles
ANSWER:D
371
A 10-year-old female presents to the emergency department for forensic examination after
sexual abuse. Cultures are obtained for Neisseria gonor- rhoeae and Chlamydia trachomatis.
What is the most common manifestation of infection with N gonorrhoeae in a prepubertal
girl?
A arthritis
B conjunctivitis
C peritonitis
D vaginitis
ANSWER:D

372
A 16-year-old female presents to your office with complaints of vague abdominal discom-
fort and vaginal discharge. Pelvic examination is performed and N gonorrhoeae grows on
choco- late agar. What is the most common clinical presentation for primary symptomatic
gono- coccal disease in the adolescent female?
A hematuria
B cervicitis
C fever and shaking chills
D arthritis
ANSWER:B
373
In reviewing for your microbiology final exam- ination, you learn that staphylococcal food
poi- soning is caused by a heat-stable, preformed enterotoxin. What are the clinical
characteristics associated with this disease process?
A high fever
B onset of symptoms within minutes of ingestion of the toxin
C concurrent staphylococcal bacteremia
D vomiting and abdominal cramps, fol- lowed by diarrhea
ANSWER:D
374
You are seeing a school-age child in your office for fever and sore throat. You diagnose
streptococcal pharyngitis based on a positive rapid strepto- coccal antigen test. The mother is
concerned that her 1-year-old child may also be infected. You inform her that group A
streptococcal infection in this age group is likely to present as which of the following?
A meningitis
B scarlet fever
C fever and peritonsillar abscess
D fever and serous rhinitis
ANSWER:D
375
A 12-year-old presents to the emergency department in August with fever 102.2°F (39°C)
and intense headache. A lumbar puncture (LP) is performed, and your suspicion of aseptic
meningitis is confirmed. Which of the follow- ing CSF findings is most likely 48 hours into
the course of enteroviral meningitis?
A 5,000 WBC, 90% polymorphonuclear leukocytes
B 100 WBC, 90% eosinophils
C 150 WBC, 80% lymphocytes
D 50 WBC, 90% polymorphonuclear leukocytes

ANSWER:C
376
It is the middle of the winter and you have seen many children of different ages in your
clinic with upper respiratory symptoms. Which of the following children is most likely to
have group A streptococcal infection?
A exudative pharyngitis in a 1-year-old
B tonsillitis, rash, and fever in a 5-year-old
C cough and pharyngitis in a 15-year-old
D “slapped cheek” appearance in a 5-year-old
ANSWER:B
377
A 4 -year-old child presents to your office with fever, and increased work of breathing mani-
fested by tachypnea and retractions. A chest x-ray confirms lobar pneumonia. What is the
most likely etiology of pneumonia in this child?
A Mycoplasma pneumoniae
B Streptococcus pyogenes
C Chlamydophila pneumoniae
D Streptococcus pneumoniae
ANSWER:D
378
A 2-year-old presents to the emergency depart- ment with temperature of 104°F (40°C) and
generalized erythema that is exquisitely tender. On close examination you notice large
superfi- cial bullae. You suspect a toxin-mediated infec- tion. Which of the following toxins
is the cause of these manifestations?
A exfoliatoxin A and B
B toxic shock syndrome toxin-1
C Shiga toxin
D lipopolysaccharide
ANSWER:A
379
A 3-year-old child is admitted to the hospital in February with temperature of 101.3°C
(38.5°C), vomiting, and diarrhea. The mother describes the stools as watery, green, and very
foul smelling. The most common organism causing this pres- entation is
A adenovirus
B rotavirus
C Salmonella enteriditis
D Shigella sonnei
ANSWER:B

380
There has been a recent increase in cases of per- tussis in your community. When counseling
a family who declines immunization, you should inform them of the common manifestations
of pertussis. Which of the following is a typical presentation of pertussis infection?
A 4 or 5 days of high fever followed by croupy cough
B sudden onset of fever and cough
C gradual onset of cough, followed by abrupt onset of fever and whooping
D mild upper respiratory symptoms fol- lowed by paroxysmal cough
ANSWER:D
381
A 12-year-old child developed fever about 1 week after visiting relatives in India. The fever
has persisted for about 10 days. Diarrhea was present for a few days, and then cleared. The
child is now constipated. The child appears moderately acutely ill. The liver and spleen are
enlarged. There are palpable, small (2–4 mm) erythematous spots on the trunk only. What is
the most likely cause of this child’s infection?
A measles
B typhoid fever
C Neisseria meningitidis bacteremia
D rat-bite fever
ANSWER:B
382
You are working in a clinic in rural Mexico and examine an 8-year-old boy who has a rectal
temperature of 100°F (38°C), bilateral tender parotid swelling, and pain when you flex his
neck. He has been complaining of a headache. His immunization history is unknown. What
is the most likely cause of this child’s infection?
A brucellosis
B cysticercosis
C Epstein-Barr virus infection
D mumps
ANSWER:D
383
A previously well 3-year-old child has a tem- perature of 104°F (40°C), headache, and stiff
neck. He has a 2-day history of upper respira- tory tract symptoms and is now vomiting.
This clinical presentation most likley represents which of the following diagnoses?
A brain tumor
B optic neuritis
C subarachnoid hemorrhage

D meningitis
ANSWER:D
384
The child in the previous question has a CSF evaluation which revealed 4000 WBCs which
are 95% polymorphonuclear leukocytes. Culture grows S pneumoniae and he is treated with
ceftri- axone to which the organism is susceptible. He is noted to have subdural effusions on
CT scan of his head. Which of the following is true concern- ing subdural effusions with
bacterial meningitis?
A They are the result of inadequate treatment.
B They are indicative of a bleeding disorder.
C Finding them is a poor prognostic indicator.
D They are a common occurrence.
ANSWER:D
385
A 16-year-old adolescent presents with tem- perature of 103.1°F (39.5°C) and purpuric skin
lesions. The child is found to have a blood pres- sure of 60/30 and heart rate of 180 with
bound- ing pulses and a respiratory rate of 40. What infectious agent is the most likely to
cause the above symptoms?
A Neisseria meningitidis
B Haemophilus influenzae
C Streptococcus pneumoniae
D Staphylococcus aureus
ANSWER:A
386 Of the choices below, what is the first priority in management of the child in question #39?
A intravenous immune globulin infusion
B CSF examination
C intravenous fluid resuscitation
D taking an exposure history
ANSWER:C
387
A 3-year-old child presents with fever for 8 days, lymphadenopathy, splenomegaly, and
numer- ous reactive or atypical lymphocytes on periph- eral blood smear. The monospot test
is negative. A likely cause of this clinical picture is infection with which of the following?
A adenovirus
B respiratory syncytial virus
C herpesvirus

D Epstein-Barr virus
ANSWER:D
388
Which of the following drug regimens is the most appropriate chemoprophylaxis for adult
household contacts of a child with meningo- coccal meningitis?
A single-dose ciprofloxacin
B penicillin for 2 days
C ceftriaxone IM
D trimethoprim-sulfamethoxazole for 7 days
ANSWER:A
389
You are counseling a mother who is 37 weeks into her second pregnancy. Group B strepto-
coccal screening was negative with her prior pregnancy 2 years ago, but she was positive on
testing this time. She understands that she will receive antibiotics during labor, but is con-
cerned about possible risk of infection in her newborn. Which of the following is true regard-
ing early-onset group B streptococcal infection in the neonate?
A It is more common in infants born at less than 37 weeks gestation.
B It usually presents with meningitis.
C It is associated with good prognosis if treated promptly.
D It generally presents with osteomyelitis.
ANSWER:A
390
A 24-year-old primigravid woman presents to labor and delivery with fever, malaise,
abdominal pain, and back pain. She is found to be in labor and subsequently delivers a full-
term infant with tachypnea, grunting, and tachycardia. Blood cul- ture evaluation of the
infant reveals Listeria mono- cytogenes. Which maternal exposure is considered a risk factor
for development of listeriosis?
A ingestion of washed raw vegetables
B exposure to cat feces
C ingestion of unpasteurized dairy or prepared meat
D ingestion of shellfish
ANSWER:C
391
A 17-month-old nonimmunized child has had fever for 4 days and now has a
maculopapular rash. She is seen in clinic and diagnosed as having measles. There is a 4-
month-old sibling at home. What is the appropriate management for the sibling?
A immediate immunization with live attenuated measles vaccine
B immediate immunization with killed measles vaccine
C a single dose of immune globulin (IG)

D IG plus live attenuated measles vaccine
ANSWER:C
392
A young child with fever, cough, hepatosplenomegaly, and eosinophilia has 3 negative
examinations of the stool for ova and parasites. What is the most likely parasite to cause this
combination of symptoms?
A Ascaris lumbricoides
B Toxocara canis
C Dracunculus medinensis
D Enterobius vermicularis
ANSWER:B
393
You are working in a hospital in Africa on an international elective. A 2-year-old child pres-
ents with vomiting and a distended abdomen. Abdominal radiographs reveal acute
obstruction of the small intestine with air fluid levels. Eosinophilia is found on complete
blood count. Of the fol- lowing parasitic infections, which is most likely to present with
intestinal obstruction?
A Enterobius vermicularis
B Necator americanus
C Ascaris lumbricoides
D Strongyloides stercoralis
ANSWER:C
394
A 3-year-old child presents with 5 days of fever; conjunctivitis; red, cracked lips;
polymorphous rash; and an isolated 2 cm cervical lymph node. You diagnose Kawasaki
syndrome based on clinical findings. Which of the following seque- lae of Kawasaki
syndrome is most common?
A fulminant hepatitis
B coronary artery aneurysm
C recurrent pericarditis
D cerebral edema
ANSWER:B
395
A 15-month-old child presents to your office with a high fever, and an intense, red rash on
the cheeks with circumoral pallor. What is the most likely etiology of this febrile exanthem?
A enterovirus 71
B adenovirus
C parvovirus B19

D rubeola virus
ANSWER:C
396
A 2-week-old infant presents to the pediatric intensive care unit in shock with vesicular rash,
pneumonitis, hepatitis, and coagulopathy. The prenatal and birth history were uneventful
and the mother was healthy. What is the most likely etiology of this illness?
A group B streptococcus
B herpes simplex virus
C varicella virus
D Treponema pallidum
ANSWER:B
397
Which of the following tests should be included in the initial workup of the infant in the
previous question?
A long bone x-ray
B liver biopsy
C MRI head
D CSF evaluation
ANSWER:D
398
A family has recently moved to your area from China. The mother tells you that her
hepatitis B screen was positive on arrival to the United States. She has a 2-month-old child
and asks you what signs of infection would be in the child. What is the most common
presentation of congenital hepatitis B?
A jaundice
B arthritis
C asymptomatic infection
D papular acrodermatitis (Gianotti-Crosti syndrome)
ANSWER:C
399
In regard to the mother of the child in the previous question, the presence of which of the
following serologic markers for hepatitis B represents an increased risk of transmitting
infection?
A HBsAg
B HBeAg
C IgM anti-HBc
D antibody to HBc
ANSWER:B

400
A 10-month-old child has a temperature of 104°F (40°C) for 4 days without other signs. On
the fourth day a rose pink, maculopapular rash appears and the temperature returns to
normal. What is the most likely diagnosis?
A echovirus
B human herpes virus 6
C measles virus
D group A streptococcus
ANSWER:B
401
An 18-month-old child presents to your office with a 2-day history of fever. He is not eating
well and the mother tells you that she thinks his mouth hurts. On examination you see 3 mm
vesicles on erythematous bases on the soft palate and tonsils. What is the most likely eti-
ology of this infection?
A streptococcal pharyngitis
B herpangina
C herpes simplex virus
D human herpes virus 6
ANSWER:B
402
The child in the above question also has small vesicular lesions on his palms and soles. You
suspect hand, foot, and mouth syndrome. What virus is the most likely causative agent of this
disease?
A adenovirus
B group A streptococcus
C Arcanobacterium haemolyticum
D coxsackievirus A16
ANSWER:D
403
You are taking care of a 26-month-old child with bacterial meningitis. Blood and spinal fluid
cultures are positive for N meningitidis. The child’s household consists of the parents, one
grandparent, a 6-month-old sibling, a 5-year-old sibling, and a 14-year-old exchange
student who has been living with the family for about 1 month. Which members of the
household should receive chemoprophylaxis?
A everyone in the household
B everyone except the grandparent
C everyone except the grandparent and the visiting student
D the siblings only
ANSWER:A

404
A 4-year-old child presents with temperature of 101.3°C (38.5°C) and a sore throat. On
examina- tion, you notice exudative pharyngitis and bilat- eral, anterior cervical
lymphadenopathy. Rapid streptococcal antigen testing is negative. What is the most likely
etiology of this presentation?
A Kawasaki syndrome
B Behçet syndrome
C adenovirus
D parainfluenza virus
ANSWER:C
405
A previously well 12-year-old presents with fever, sore throat, and general malaise. On
physical examination, you notice generalized lymphadenopathy, exudative pharyngitis, and
splenomegaly. Laboratory evaluation reveals elevated serum transaminase levels. What is the
organism most likely responsible for these findings?
A Treponema pallidum
B hepatitis B virus
C rubella virus
D Epstein-Barr virus
ANSWER:D
406
A 15-year-old presents with pain, photopho- bia, and blurred vision in one eye. Examination
reveals chemosis of the affected eye and small vesicular lesions below the eye and on the
nose. Application of fluorescein dye reveals branch- ing, dendritic lesions on the cornea.
What is the most likely cause of this clinical picture?
A cytomegalovirus
B adenoviral infection
C rubella virus
D herpes simplex infection
ANSWER:D
407
A 4-month-old infant born with perinatally acquired HIV infection presents with fever,
grunting, and hypoxia. Influenza and RSV test- ing are negative. Which of the following is
the likely opportunistic infection in this infant with perinatally acquired HIV infection?
A Pneumocystis pneumonia
B disseminated cytomegalovirus infection
C disseminated toxoplasmosis
D cryptococcal meningitis
ANSWER:A

408
A newborn infant has microcephaly, periven- tricular calcifications, jaundice, and thrombo-
cytopenia. You suspect congenital infection. Which of the following is most likely?
A Epstein-Barr virus
B cytomegalovirus
C coxsackievirus B
D human immunodeficiency virus
ANSWER:B
409
A 12-year-old who went camping 2 weeks ago in Oklahoma now presents with fever and
headache. Laboratory studies demonstrate leukopenia, thrombocytopenia, elevated liver
enzymes, and hyponatremia. Of the following, what is the most likely causative agent?
A Rickettsia prowazekii
B Rickettsia typhi
C Ehrlichia chaffeensis
D Coxiella burnetii
ANSWER:C
410
A 12-year-old child presents to the pediatric intensive care unit in shock with evidence of
disseminated intravascular coagulation. Blood culture reveals lancet-shaped, gram-positive
cocci in pairs. You suspect S pneumoniae infec- tion. Which of the following underlying dis-
eases places them at increase risk for invasive pneumococcal infection in terms of incidence
and severity?
A underlying liver disease
B sickle cell disease
C valvular cardiac disease
D cystic fibrosis
ANSWER:B
411
A 7-year-old unimmunized child presents with fever and vesicular rash. You notice about
300 lesions, some of which were crusted. You sus- pect varicella infection. Which of the
following is a TRUE statement about varicella?
A It has an incubation period of 5–7 days.
B The rash is confluent, centrifugal, and pustular.
C It is associated with Koplik spots.
D It can cause visceral dissemination in the immunocompromised host.
ANSWER:D

412
You are counseling a primigravid woman who has been found to be rubella nonimmune on
prenatal laboratory evaluation. She asks you if her fetus is at risk for malformations. When is
maternal infection with rubella virus most commonly associated with congenital defects?
A in the first 4 weeks of gestation
B in the second month
C in the third or fourth month
D in the last trimester
ANSWER:A
413
A 15-year-old has exudative tonsillitis, cervical adenitis, and splenomegaly. His monospot
test is positive. Which of the following is the most common complication encountered?
A chronic fatigue lasting greater than 6 months
B hemorrhage
C pneumonia
D airway obstruction
ANSWER:D
414
An 8-year-old male presents to the emergency department with an erythematous, swollen
hand. He sustained a dog bite to the hand the day prior which broke the skin and was treated
with irrigation. Purulent material drains from the bite site with palpation. What is the most
likely organism causing this infection?
A Pasteurella multocida
B Eikenella corrodens
C Streptobacillus moniliformis
D group A streptococcus
ANSWER:A
415
A 15-year-old presents with fever, acute pharyn- gitis, and rash. Rapid streptococcal antigen
test- ing is negative. Three days later the laboratory informs you that A haemolyticum is
growing on culture. Which of the following is the most accurate description of the rash
associated with this infection?
A It has a rough, sandpaper-like texture.
B It has a tendency to involve the palms and soles.
C It has a tendency to become vesicular within 24 hours.
D It is pustular.
ANSWER:A

416
A 3-year-old child travels with his family to Mexico, where he dines on the local cuisine.
One month after return, he develops fever, anorexia, myalgias, and abdominal pain.
Serologic testing is positive for Brucella infection. Which of the following is a common
manifestation of bru- cellosis in children?
A hepatosplenomegaly
B glaucoma
C meningitis
D endocarditis
ANSWER:A
417
A 3-year-old child in day care develops abdom- inal pain with diarrhea and malaise. His
class recently had a field trip to the local petting zoo. Stool culture subsequently grows
Campylobacter jejuni. Which of the following is the most common clinical manifestation that
follows infection with Campylobacter?
A uillain-Barré syndrome
B polyarticular arthritis
C encephalitis
D inguinal lymphadenitis
ANSWER:A
418
You see a 5-year-old child in your office with a palpable lymph node in her right axilla that
is progressing in size over the last 3 days. It has become erythematous and warm. On exami-
nation, the node is indurated. You also note many scratches on the child’s arms. On further
questioning, you discover that the family has recently adopted a new kitten. What is the most
likely etiology of the child’s adenopathy?
A Coxiella burnetii
B Streptobacillus moniliformis
C Bartonella henselae
D Mycobacterium avium complex
ANSWER:C
419
A 3-month-old infant is admitted to the hospital with respiratory distress. He has had cough
and congestion for the last 2 days and is now breath- ing too fast to eat effectively. You
suspect bron- chiolitis and respiratory syncytial virus (RSV) antigen testing is positive. Which
of the following roentgenographic findings is most commonly seen in infants with
bronchiolitis caused by RSV?
A hyperinflation
B hilar adenopathy

C multilobar infiltrate
D lower lobe infiltrates
ANSWER:A
420
An 8-year-old was hiking with his Boy Scout troop a few days ago. He now presents to your
office with 2 days of abdominal pain. On further history, you realize he drank from the
stream on his hike. You suspect Giardia intestinalis infec- tion. What additional symptom
would you see most commonly in a child with giardiasis?
A fever
B watery diarrhea
C bloody diarrhea
D failure to thrive
ANSWER:B
421
After confirmation of giardiasis by antigen test- ing on the stool, you initiate therapy on the
child in the previous question. What is the most appropriate therapy for giardiasis?
A trimethoprim-sulfamethoxazole
B ketoconazole
C mebendazole
D metronidazole
ANSWER:D
422
A 3-year-old child from Somalia presents to the emergency department with a 2-week
history of low-grade fever, headache, and irritability. This morning he was difficult to arouse
and subsequently had a seizure. On examination, he has a temperature of 102.2°F (39°C),
nuchal rigidity, and somnolence. You discover that he and his parents had positive
tuberculin skin tests on arrival to the United States and suspect tuberculous meningitis. Which
of the following is characteristic of the cerebrospinal fluid in tuberculous meningitis?
A The color is usually blood tinged.
B Protein is normal.
C Culture reveals tuberculous organisms within 1 week.
D Glucose is low.
ANSWER:D
423
A 3-month-old African American infant presents to the emergency department on
Thanksgiving weekend with fever and hematochezia. From the history, you discover that the
family pre- pared chitterlings for the Thanksgiving meal. Fecal leukocytes are present. Stool
cultures are positive. What is the most likely etiology of this infant’s infection?

A Salmonella enteritidis
B Salmonella typhi
C Escherichia coli
D Yersinia enterocolitica
ANSWER:D
424
A 16-year-old sexually active male presents to your office with a painful ulcer on his penis
and fluctuant, tender lymph nodes in his left inguinal area. You aspirate the most fluctuant
node and Gram stained smear is negative. What is the most likely etiology of this
presentation?
A lymphogranuloma venereum
B herpes simplex virus
C syphilis
D chancroid
ANSWER:A
425
The mother of a newborn infant brings her 2-week- old child in for his first checkup in
November. The mother has heard reports on the local news of influenza in the community
and that young infants are at very high risk if they contract the disease. She asks you if her
child can be immu- nized. At what age can a child first receive influenza vaccine?
A birth
B 2 months
C 4 months
D 6 months
ANSWER:D
426
A 13-year-old male is taking inhaled steroids for his moderate intermittent asthma. On
examination, you find white plaques on his buccal mucosa. You suspect thrush caused by
Candida albicans. Which of the following would be most appropriate for treatment?
A itraconazole
B amphotericin B
C metronidazole
D nystatin
ANSWER:D

427
A 3-year-old female with perinatally acquired human immunodeficiency virus has history of
medical nonadherence to therapy. Her most recent CD4 count was less than 100. She
presents to your clinic with fatigue, abdominal cramping, vomiting, and diarrhea. She has
not been eating and has had significant weight loss since her last visit. Upon further
questioning, you realize that she has had diarrhea for over a month. Stool examination
reveals 4–6 m spherical cysts. What is the most likely etiology of this presentation?
A Giardia lamblia
B Entamoeba histolytica
C Cryptosporidium parvum
D Isospora belli
ANSWER:C
428
A 9-month-old presents to your office with 2-day history of temperature of 101.6°F
(38.7°C) and fussiness. Mother reports that the child has been pulling at her ear. On physical
examina- tion, you note an erythematous, bulging, right tympanic membrane with purulent
effusion. You diagnose otitis media. What organism is the most likely cause of this infection?
A Streptococcus pyogenes
B Streptococcus pneumoniae
C Haemophilus influenzae type B
D Staphylococcus aureus
ANSWER:B
429
A 14-year-old presents to your office with 2 week history of cough and purulent rhinorrhea.
He now has 2-day history of fever and frontal headache. On examination he is tender to pal-
pation over his maxillary sinuses and his mother tells you that he appears swollen over that
area. You make the clinical diagnosis of acute sinusi- tis. He has no allergies to medication.
What is the first-line therapy for this child?
A ceftriaxone
B cephalexin
C clindamycin
D amoxicillin
ANSWER:D

430
A 15-year-old male just returned from a hunt- ing trip with his father. He has an abrupt onset
of fever, chills, myalgia, and headache. He develops painful anterior cervical chain lym-
phadenopathy. On examination, you see an ulcer under his arm. His father tells you that they
removed a tick from that spot a few days prior. What is the most likely diagnosis for this
patient?
A tularemia
B Lyme disease
C Rocky Mountain spotted fever
D ehrlichiosis
ANSWER:A
431
A 7-year-old child was grabbing at a neighbor dog’s tail and was bitten on the arm. He pres-
ents to the emergency department for wound management. The dog’s behavior is normal
and it is in custody of animal control, but the owner’s do not have record of the last rabies
vaccine. The health department has not had reports of rabies in the county where the child
was bitten. Besides local wound care, what is the most appropriate intervention regarding
possible rabies exposure?
A Euthanize the dog and send brain tissue for rabies antigen testing.
B
Give the child the rabies vaccine series as well as rabies immune globulin and observe the
dog for 10 days for signs of rabies.
C Give the child rabies vaccine alone and observe the dog for 10 days for signs of rabies.
D Observe the dog for 10 days for signs of rabies.
ANSWER:D
432
A 5-year-old unimmunized child fell while playing in an old barn and sustained a lacera-
tion to his leg. After local wound care, what would be the most appropriate management
regarding tetanus prophylaxis?
A give Td only
B give DTaP only
C give Td and tetanus immune globulin
D give DTaP and tetanus immune globulin
ANSWER:D
433
A family is visiting their summer home in New England. Their 8-year-old child presents to
your office with a rash over his back. The rash started as a small, erythematous macular area
and is expanding to form an annular, erythe- matous lesion with a distinct border that is
more than 10 cm in diameter. What is the causative agent of this child’s infection?
A Rickettsia rickettsiae

B Borrelia burgdorferi
C Anaplasma phagocytophilia
D Rickettsia typhi
ANSWER:B
434 What is the term used to describe the exan- them in the patient in the above question?
A erythema nodosum
B erythema marginatum
C erythema migrans
D erythema toxicum
ANSWER:C
435
A 2-year-old African American child presents to your office with areas of alopecia. On
examina- tion you note a boggy, inflammatory mass with some purulent drainage over the
scalp and occip- ital lymphadenopathy. Cultures are obtained via skin scraping and grow
Trichophyton tonsurans. What is the best initial treatment for tinea capitis?
A topical nystatin
B oral griseofulvin
C topical miconazole
D oral itraconazole
ANSWER:B
436
You are attending in the newborn nursery and are informed that a baby was born to a
mother with human immunodeficiency virus (HIV). What is the best test to diagnose HIV
infection in this infant?
A Enzyme immunoassay (ELISA)
B RNA PCR
C Western blot
D DNA PCR
ANSWER:D
437
A 5-month-old child is admitted with severe hypoxia and respiratory failure to the pediatric
intensive care unit. A diagnosis of P jiroveci pneumonia (PCP) is made on silver staining of
bronchoalveolar lavage fluid. You suspect immunodeficiency in this patient, and HIV testing
is negative. What is the most likely immunodeficiency?
A Bruton agammaglobulinemia
B adenosine deaminase deficiency
C common variable immunodeficiency
D Job syndrome

ANSWER:B
438
A 9-year-old child is hospitalized with her second episode of meningococcemia.
Immunodeficiency is suspected. What is the most likely immune deficiency which would
predispose this child to recurrent N meningitidis infection?
A X-linked agammaglobulinemia
B Chediak-Higashi syndrome
C human immunodeficiency virus
D terminal complement deficiency
ANSWER:D
439
You are seeing a 6-month-old for well-child visit and immunizations. Anticipatory guidance
is given. What immunizations would you pre- scribe if the child were up to date at that point?
A hepatitis B, DTaP, HIB, IPV, PCV
B hepatitis A, hepatitis B, DTaP, HIB, PCV
C DTaP, HIB, IPV, PCV, MMR
D hepatitis A, DTaP,HIB, IPV, PCV
ANSWER:A
440
A 6-year-old child has recently emigrated from sub-Saharan Africa. For the last 2 weeks, he
has had fever every other day along with nausea, vomiting, arthralgia, and abdominal pain.
On further questioning, the mother tells you that the child has rigors with fever and sweats
when the fever breaks. You suspect malaria. What is the best diagnostic method for this
illness?
A aerobic and anaerobic blood culture
B serologic testing
C thick and thin blood smear preparations
D PCR on serum
ANSWER:C
441
You diagnose group A streptococcal pharyngi- tis in an 8-year-old child and treat appropri-
ately. The mother of the child asks how long she must stay home from school. When can the
child return to school?
A as soon as therapy is initiated
B 24 hours after the initiation of therapy
C 48 hours after the initiation of therapy
D 72 hours after the initiation of therapy
ANSWER:B

442
An 18-year-old male comes home from college for winter break and comes to your office
com- plaining of malaise, knee pain, and rash. On examination, you find generalized,
polymor- phic, maculopapular rash including the palms and soles, generalized
lymphadenopathy, and splenomegaly. When questioned about his sexual history, he tells you
he has had multiple partners. Of the following, what is the most likely etiology of his illness?
A primary syphilis
B secondary syphilis
C chancroid
D lymphogranuloma venereum
ANSWER:B
443
In regard to the patient in the above question, what is the most appropriate antimicrobial
therapy for his treatment?
A penicillin
B ceftriaxone
C cephalexin
D azithromycin
ANSWER:A
444
A 3-year-old child is in your office for the second time with otitis media that is not
responding to antibiotics. He has been on standard-dose amox- icillin for the last 10 days and
his tympanic membrane is still bulging with purulent effusion. You perform a
tympanocentesis and recover S pneumoniae. The parents ask you why the initial antibiotic
did not work. What is the most likely cause of pneumococcal resistance to amoxicillin?
A mutation in mef (membrane efflux pump) gene
B mutation in erm (erythromycin ribosome methylation) gene
C beta-lactamase
D penicillin-binding protein mutation
ANSWER:D
445
You are seeing a 5-year-old child with recurrent furunculosis secondary to methicillin-
resistant S aureus (MRSA). Susceptibility testing con- firms the isolate is clindamycin
susceptible and that D-zone testing is negative. You elect to treat with clindamycin. What is
the mechanism of action of clindamycin?
A inhibitor of cell wall synthesis
B inhibitor of protein synthesis
C inhibitor of bacterial DNA synthesis

D inhibitor of bacterial enzyme
ANSWER:B
446
An 18-month-old child presents to the emer- gency department with fever. The parents
inform you that they think the child has been limping on the right leg for the last week. On
examination, the only focal finding is that the right knee seems swollen and a little warm.
Arthrocentesis is performed and synovial fluid shows 75,000 WBC with 90% neutrophils.
Gram stain is negative. After 5 days, the culture grows a gram-negative coccobacillus. What
is the most likely etiology of the septic arthritis in this child?
A Staphylococcus aureus
B Streptococcus pyogenes
C Kingella kingae
D Salmonella enteritidis
ANSWER:C
447
A 15-year-old girl was at an overnight rave 24 hours ago and was given Ecstasy by a friend.
This morning, she is found comatose and resuscitation is unsuccessful. Most deaths related to
Ecstasy have been linked to which of the following?
A cerebrovascular accident
B myocardial infarction
C hyponatremia or hyperthermia
D metabolic alkalosis with compensatory respiratory acidosis
ANSWER:C
448
A2-year-old is found with an opened empty bottle of acetaminophen tablets and has pill
fragments in his mouth. The major cause of morbidity and mor- tality in acute poisoning
with acetaminophen is which of the following?
A hepatic injury
B gastric bleeding
C metabolic acidosis
D methemoglobinemia
ANSWER:A
449
A 4-year-old boy has just fallen off the top rung of the ladder while climbing up to the 3-m
diving board at the local swimming pool. He lands on the pavement and is motionless when
the lifeguard arrives there in less than a minute. Following closed head injury, which of the
fol- lowing would be most ominous?
A irritability
B vomiting

C dilated, fixed pupils
D amnesia for the event
ANSWER:C
450
An 18-month-old presents to the emergency department with a 2-day history of lethargy.
Blue and yellow bruising over the buttocks and thighs are noted and retinal hemmorhages
are seen on fundoscopic examination. Parents deny any history of bleeding or bruising other
than what you see. Laboratory studies including a urine analysis, platelet count, bleeding
time, and PT/PTT reveal normal values. Which of the following is the most likely diagnosis
to explain this child’s findings?
A von Willebrand disease
B acute lymphoblastic leukemia
C Henoch-Schöenlein purpura
D idiopathic thrombocytopenic purpura.
ANSWER:C
451 Most cases of serious physical child abuse involving children occur at what age?
A less than 1-month old
B between 1 month and 4 years old
C between 5 and 12 years old
D between 13 and 16 years old
ANSWER:B
452
A 24-month-old is seen for a well-child visit. The family resides in a 45-year-old home and
the parent notes that the child has developed a habit of eating paint which has chipped off of
the window sills. You are concerned that the child could be at risk for lead toxicity. In the
United States, the appropriate recommenda- tions for routine lead screening is at which of
the following ages?
A at the 6 months check up
B at the 6 and 9 months check up
C at the 1- and 2-year check up
D at 3- and 5-year check up
ANSWER:C
453
Which of the following combinations of signs and symptoms is most suggestive of chronic
lead poisoning?
A ataxia, fever, diarrhea, and polycythemia
B lethargy, vomiting, hallucinations, and vesicular rash
C anemia, leukopenia, thrombocytopenia, and hepatomegaly

D lethargy, abdominal cramps, constipation, and anemia
ANSWER:D
454
A 3-year-old child is brought to the emergency room 3 hours after he was found playing
with an open kerosene bottle. The parents state the child initially had some gagging and
coughing and within an hour developed labored breath- ing. At the time of your evaluation
though, the child’s examination is completely normal. What is the most appropriate action at
this time?
A discharge to home, advise parents to return if they note any problems
B obtain a chest x-ray
C observe child in emergency room for 1 hour
D induce emesis with syrup of ipecac
ANSWER:B
455
A 4-year-old child is brought to the urgent care by parents who were concerned because the
child suddenly developed unusual posture and movements. Examination reveals an alert
child who holds the head in a tilted position and has uncontrolled, writhing movements of
the hands and arms. The examination is otherwise normal. When questioned regarding
medications at home, parents state they do have routine cold medicine, acetaminophen, and
some kind of antiemetic medication in an unlocked medicine cabinet. At this time you
should do which of the following?
A perform a head computerized tomography (CT) scan
B perform a lumbar puncture
C obtain an electroencephalogram
D administer diphenhydramine intravenously
ANSWER:D
456
A 4-year-old boy was found playing with an open bottle of drain cleaner about 1 hour ago.
His mother reports that he now refuses to drink and talk but appears alert though anxious.
You should advise the mother to do which of the following?
A administer syrup of ipecac
B closely observe the child and bring to the emergency room if condition worsens
C administer milk of magnesia
D immediately bring the child to the emer- gency room for evaluation
ANSWER:D
457
Shellfish poisoning, which is caused by eating shellfish that have ingested toxic dinoflagel-
lates (red tide), is characterized by which of the following?
A blindness

B vomiting and diarrhea
C seizures and coma
D weakness and paralysis
ANSWER:D
458
A toddler presents with a known ingestion of iron tablets. By parental count of pills remain-
ing in the bottle it appears he ingested more than 60 mg/kg of elemental iron. Upon admis-
sion to the pediatric intensive care unit, he is vomiting. Which of the following chelating
agents should be administered?
A deferoxamine mesylate
B ethylene diamine tetraacetic acid (EDTA)
C British anti-Lewisite (BAL)
D hemoglobin
ANSWER:A
459
A 3-year-old child who is unresponsive pres- ents with weakness, excessive salivation, brady-
cardia, and constricted pupils. The parents are so distraught, it is difficult to get information.
However, an astute emergency room physician realizes the most likely drug or toxin to cause
these signs is which of the following?
A diphenhydramine
B phenobarbital
C ethyl alcohol
D an organophosphate
ANSWER:D
460
Following stabilization of the patient in the above question, the emergency room physician
administers a test dose of a drug and the patient clinically improves. What drug did this
physician most likely administer?
A naloxone
B diphenhydramine
C atropine
D N-acetylcysteine
ANSWER:C
461
Three symptomatic adolescents with a history of ingesting seeds of jimsonweed are arriving
via ambulance from a referring hospital emer- gency room. Which of the following best
describes the expected signs and symptoms of jimson- weed poisoning?
A agitation/hallucinations, dilated pupils
B coma, pinpoint pupils

C hallucinations, bradycardia
D coma, bradycardia
ANSWER:A
462 Ingestion of LSD will most likely result in which of the following?
A convulsions
B euphoria
C hallucinations
D sedation
ANSWER:C
463
A teenager who sniffs spot remover and then engages in stressful physical activity is at risk
for
A convulsions
B hypertension
C rhabdomyolysis
D sudden death
ANSWER:D
464 Which of the following are the most commonly used “date-rape” drug?
A amphetamines and LSD
B gamma-hydroxybutyrate and flunitrazepam
C cocaine and phenobarbital
D ephedra and codeine
ANSWER:B
465
After being lifted up by one hand, a young tod- dler refuses to use that arm and holds it
against her trunk flexed at the elbow with the forearm midway between pronation and
supination. The child most likely has which of the following?
A a shoulder dislocation
B a radial head subluxation
C a fracture of a carpal bone
D avulsion of the ulnar nerve
ANSWER:B
466
A 4-year-old child falls on an outstretched arm. The child is likely to sustain which of the
following?
A fracture displacement of the radial epiphysis
B Colles fracture
C comminuted radial and ulnar fracture

D shoulder dislocation
ANSWER:A
467 Hyperventilation due to salicylate poisoning
A is apparent on physical examination within minutes of ingestion
B is characterized by an increase in rate and depth of ventilation
C is characterized by an increase in depth of ventilation only
D is characterized by an increase in rate of ventilation only
ANSWER:B
468
Which of the following findings would be most suggestive of the form of child abuse
referred to as the abusive head trauma syndrome (formally called shaken babt syndrome)
A ecchymosis over the mastoid area
B retinal hemorrhages
C ecchymoses and petechiae over the upper arms and upper trunk
D circumferential ecchymosis on extremities
ANSWER:B
469
A 9-year-old is injured while sledding. On admission, the child appears in shock and is
complaining of pain in the left shoulder. Of immediate concern is the likely diagnosis of
A rupture of the descending aorta
B dislocation of the left shoulder
C rupture of the spleen
D rupture of the left diaphragm
ANSWER:C
470
Which of the following is the most common unintentional fatal injury to children 1–19 years
of age
A motor vehicles
B swimming pools
C firearms
D bicycles
ANSWER:A

471
A 1-year-old child is brought to the emergency room because of a swollen left thigh. The
parents, who appear very concerned, state they left the child in the care of a newly hired
housekeeper early that morning, and when they returned home in the evening they noted the
swelling. Other than tender swelling of the thigh, physical examination is entirely normal. X-
ray examination discloses a displaced fracture of the shaft of the femur; skeletal survey
reveals no other fractures or abnormalities. The grandparents, who live with the parents and
who had accompanied them on their trip, corroborate the parents’ story. After first admitting
the child for treatment of the frac- ture, what is the most appropriate next step?
A order a computerized tomography (CT) scan of the head
B order a complete coagulation profile
C order magnetic resonance imaging (MRI) of the leg
D report the incident to a child protection agency
ANSWER:D
472 Syrup of ipecac should be administered in the home to which of the following children?
A 3-year-old child who ingested lye (sodium hydroxide) 5 minutes prior
B 4-year-old child found obtunded, suspected of narcotic poisoning
C 2-year-old child who is having brief seizures, suspected of ingesting sibling’s phenytoin
D none of the above
ANSWER:D
473
A 4-year-old child is playing in the basement. The child suddenly comes upstairs, screaming
of being bitten by a spider. There is an erythe- matous 2 cm macule on the child’s face
which appears to have a central tiny puncture site. Over the next few hours the lesion
becomes larger, more painful, and in 24 hours it appears darker. The most likely
complication in this child would be which of the following?
A necrosis at the site of the bite
B renal failure
C hepatic failure
D muscle cramps and seizures
ANSWER:A
474
Management with multiple-dose activated char- coal may be indicated in the overdose of
which of the following?
A iron
B cyanide
C carbamazepine

D tricyclic antidepressants
ANSWER:C
475
A 2-year-old child is retrieved from a near- drowning episode in a pool. The child is apneic
on retrieval, but is quickly and successfully resus- citated. On arrival in the emergency room,
abnor- malities which of the following are most likely to be present and require immediate
attention?
A hyponatremia and hypokalemia
B hyponatremia and hyperkalemia
C hyperkalemia and acidosis
D acidosis and hypoxemia
ANSWER:D
476 Manifestations of the first stage of severe acute iron poisoning include
A lethargy and gastrointestinal irritation
B metabolic alkalosis and hypertension
C hemolysis and neutropenia
D renal, hepatic, and cardiac failure
ANSWER:A
477 Which of the following statements regarding automobile safety for children is correct?
A
Children, beyond the age of 1 year, or 20 lb, may ride either facing the front or rear of the
car.
B Children over 25 lb may use adult-type restraints.
C
A 1-year-old child held in the lap of a seat-belted adult is almost as safe as in an infant
restraint device.
D Infants under the age of 1 year should ride in restraint devices facing the rear of the car.
ANSWER:D
478 Which of the following statements regarding drowning and near-drowning is correct?
A Four-sided fencing around pools has not decreased the incidence of drownings.
B
Children under the age of 5 years who drown in home pools most often enter the pool by
climbing over a fence.
C The incidence of drowning peaks in ele- mentary school-age children.
D Approximately 50% of those who die are declared dead at the scene.
ANSWER:D
479 Concerning child sexual abuse, which of the following is correct?
A Boys and girls are at equal risk.
B In the majority of cases, the abuser is well known to the child.

C Physical contact is necessary to fulfill diagnostic criteria for child sexual abuse.
D An estimated 10% of children are sexually abused each year in the United States.
ANSWER:B
480
Which of the following statements regarding firearm-related injuries in children less than 19
years of age is correct?
A Intentional firearm-related injuries far outnumber unintentional firearm-related injuries.
B Half of firearm-related deaths are the result of suicide.
C Firearms account for the majority of all injury deaths in this age group.
D Firearm-related injuries result in more deaths than do motor vehicle accidents each year.
ANSWER:A
481
Injuries remain the leading cause of death among all children of age 1–19 years. For those
15–19 years of age, which of the following lists the categories of injuries from most
common to least common?
A suicide, homicide, motor vehicle accidents (MVA)
B homicide, suicide, MVA
C MVA, suicide, homicide
D MVA, homicide, suicide
ANSWER:D
482 Overdose of which of the following is most likely to be complicated by hypoglycemia?
A salicylates
B lead
C tricyclic antidepressants
D opioids
ANSWER:A
483
Last week, a 14 year old boy is caught smoking a cigarette by his mother and the teen is now
being seen in your pediatric clinic. His mother is con- cerned about short- and long-term
effects of smok- ing tobacco at such a young age. Which of the following statements
concerning tobacco/cigarette smoking in preadolescent and adolescent years is most accurate?
A The average smoker in the United States starts at age 16 years.
B 50% of adolescent smokers become adult smokers.
C
Adolescent smokers become nicotine dependent after smoking fewer ciga- rettes than the
adult smoker.
D The use of cigars and smokeless tobacco is rare in adolescence.
ANSWER:C

484
A 5-year-old, 15 kg girl presents to the emer- gency room 10 hours after ingesting 7.5 g of
acetaminophen. Plasma acetaminophen and hepatic enzymes are obtained and oral N-
acetylcysteine is started. The patient is admitted and repeat acetaminophen levels and hepatic
enzymes are followed. Peak AST levels are 300 U/L, but return to normal. Her hospital
course is uneventful. Young children are less prone to develop hepatic injury following acet-
aminophen overdoses compared to adults due to which of the following?
A Children are healthier with greater renal clearance of acetaminophen.
B Children have an active sulfate conjuga- tion pathway.
C Acetaminophen undergoes glucuronida- tion only in young pediatric patients.
D Glutathione stores are larger in adults.
ANSWER:B
485
A 24-month-old, 10 kg boy with a history of epilepsy, develops a tonic-clonic seizure at
home. EMTs arrive and are unable to get IV access. He has been seizing for 60 minutes
when you are called. The EMTs have a vial of diazepam. Which route will achieve the safest
and quickest therapeutic serum diazepam level?
A administering the diazepam via intra- muscular route
B administering the diazepam via intranasal
C administering the diazepam via oral/ sublingual route
D administering the diazepam via rectal route
ANSWER:D
486
An 8-year-old male with cystic fibrosis is admit- ted to the hospital with acute decline in pul-
monary function and fever. He is started on ceftazidime and tobramycin. This is his second
hospitalization this year. The pharmacy recom- mends tobramycin be dosed at 2.5 mg/kg IV
every 8 hours based on his pharmacokinetics from the last admission. Tobramycin levels
obtained appropriately after the third dose are: peak of 3 mg/L and trough of 1.8 mg/L. The
timing of the draws is double checked and determined to be accurate. What is the most likely
cause of the unexpected tobramycin level?
A The tobramycin dose is too low; he needs his tobramycin dose increased.
B The patient has a much larger volume of distribution than expected.
C The IV pump/system caused a delayed delivery of the drug.
D The ceftazidime is interfering with the drug level
ANSWER:C

487
A 2-year-old comes into your clinic with a chief complaint of a rash. You see multiple
circular areas with a raised boarder and a scaling inte- rior on the upper arms and back. You
also note a bald patch with a similar-looking lesion on the scalp. Which of the following
medications is the most appropriate treatment for this patient?
A griseofulvin
B ketoconazole cream
C amoxicillin
D fluconazole
ANSWER:A
488
A 6-year-old male with a known history of asthma presents to the emergency department
with the most severe asthma attack of his life. When you listen, you hear very little air move-
ment and your patient has a declining level of consciousness. After your initial measures,
you see very little improvement and a decision is made to intubate the patient’s trachea.
Which combination of drugs is most appropriate in this situation to prevent worsening
bronchospasm?
A phenobarbital, atropine, and succinyl- choline
B pancuronium and lidocaine
C ketamine, lidocaine, and rocuronium
D versed, fentanyl, and pancuronium
ANSWER:C
489
A 3-year-old girl presents to your clinic with 5 days of rhinorrhea, cough, and congestion.
Her mother has been using an over-the-counter cough and cold remedy to help control the
symptoms. What is the most appropriate coun- seling to give this mother regarding over-the-
counter medication usage in children?
A All over-the-counter medications are safe.
B All over-the-counter medications labeled pediatric or children’s are safe.
C
Many over-the-counter medications have a combination of medications that can have serious
side effects in children, always consult a physician for dosing and safety.
D It is very rare that over-the-counter cold medications have acetaminophen as an ingredient.
ANSWER:C

490
A 4-year-old with a history of renal failure, encopresis, and asthma is brought to the emer-
gency room because of lethargy and difficulty breathing. She has been taking her usual
breathing treatment and has not had any res- piratory symptoms or fevers. She had recently
been severely constipated and her parents have been administering multiple enemas to try to
help her stool with no result. Which of the fol- lowing is the most likely cause of her lethargy
and respiratory distress?
A hyperphosphatemia secondary to enemas
B albuterol overdose
C sepsis
D asthma exacerbation
ANSWER:A
491
An 11-year-old boy presents to the clinic com- plaining of itchy, watering eyes; stuffy nose;
and a “tickle” in the back of his throat. He had similar symptoms at this time last year. His
mother is concerned that he may be developing allergies. Which of the following allergy
med- ications can cause tachycardia and hypertension?
A intranasal pseudoephedrine spray
B diphenhydramine
C intranasal steroid spray
D oral decongestants (pseudoephedrine)
ANSWER:D
492
You are in the delivery room for the birth of a full-term infant who is delivered by c-section
for fetal distress. The baby is limp, cyanotic, and not making respiratory effort when placed
under the warmer. The heart rate is found to be less than 60 beats per minute. The decision is
made to place an endotracheal tube and mechanically ventilate the newborn. Which of the
following medications can be safely admin- istered via endotracheal tube?
A calcium chloride
B atropine
C amiodarone
D dopamine
ANSWER:B
493
A 1-year-old presents to the emergency depart- ment with the chief complaint of teething.
You noticed in physical examination that the patient has peri-oral cyanosis with an oxygen
satura- tion of 98% on room air. There is no significant past medical history. What is the
most likely medication that causes this condition?

A acetaminophen
B ibuprofen
C pseudoephedrine
D orajel (topical benzocaine)
ANSWER:D
494
A 5-year-old presents to the emergency depart- ment with 6 days of fever, conjunctivitis, a
“strawberry tongue,” swelling of the hands, a diffuse rash, and a large swelling that appears
to be a lymph node in the cervical area. Which medication should be included in the initial
treatment regimen for this disorder to lessen the risk of complications?
A acetaminophen
B IV antibiotics
C intravenous immunoglobin (IVIG)
D ibuprofen
ANSWER:C
495
A 14-year-old presents to the emergency depart- ment after being involved in a car accident.
He has a GCS of 15; however he lacerations which require suturing. In which of the
following locations is the use of a local anes- thetic with epinephrine contraindicated?
A shoulder
B cheek
C ear
D neck
ANSWER:C
496
A 10-year-old with a history of asthma presents to the emergency department in respiratory
dis- tress. You auscultate diffuse wheezing and the patient has increased work of breathing.
You initiate albuterol via nebulizer, oral steroids, and establish IV access. The patient is
showing signs of worsening work of breathing, which one of the following medications can
act as an acute bronchodilator?
A magnesium sulfate
B calcium chloride
C sodium phosphate
D potassium chloride
ANSWER:A

497
A 4-year-old with a seizure disorder presents to the emergency department with increased
seizure frequency. Her medication regimen is currently phenytoin twice daily. She has been
compliant with her medication and has been on the same dose for over 2 years. What is the
next best step in her management?
A add Tegretol (carbamazepine) to her regimen
B add ethosuximide to her regimen
C add phenobarbital to her regimen
D measure a serum phenytoin level
ANSWER:D
498
A 5-year-old who has just started first grade presents to your clinic with his mother. His
mother has a note from his teacher that your patient frequently misbehaves in class, gets up
out of his seat, and bothers other children. His mother says that she has a hard time control-
ling his behavior at home. He is unable to sit still during your examination and is busy play-
ing with magazines and toys when you try to talk to him. You suspect a diagnosis of ADHD
for this patient. Which of the following is the most appropriate medication to offer this
patient as a trial?
A methylphenidate
B imipramine
C haloperidol
D paroxetine
ANSWER:A
499
You are caring for a now 5-day-old infant born after 28 weeks of gestation. On physical
exam- ination you note bounding pulses and a harsh “washing machine” murmur. The
patient has also had an increasing oxygen requirement. You order an echocardiogram and
diagnose a patent ductus arteriosus. Which of the follow- ing therapies may correct this
problem?
A opamine infusion
B 100% oxygen administration
C indomethacin
D prostaglandin infusion
ANSWER:C

500
A previously healthy 12-year-old presents to the emergency department with nausea and
vomiting for 2 days, there are other sick con- tacts at home. He has no meningeal signs. An
IV is placed and he is given IV fluids for rehy- dration. The vomiting continues and you
decide to give him an antiemetic. After the medication is given, the patient begins to slowly
turn his head to the right and has extreme arching of his back. He is awake and alert. What is
the most likely cause of this reaction?
A hypocalcemia
B tetanus
C seizure disorder
D metoclopramide administration
ANSWER:D
501
A 9-year-old boy with Duchenne muscular dys- trophy presents to the pediatric emergency
room with severe respiratory distress related to pneumonia. The decision is made to intubate
the patient for respiratory failure. Which of the following paralytics are contraindicated in
this patient?
A pancuronium
B vecuronium
C atracurium
D succinylcholine
ANSWER:D
502
A 6-month-old infant with chronic lung dis- ease (CLD) on home oxygen and high-calorie
feeds presents with slow growth. You consider starting a thiazide diuretic to decrease airway
resistance and improve lung compliance. Which of the following electrolyte abnormalities
may result from the use of thiazide-type diuretics?
A hypoglycemia
B hyperuricemia
C hyperkalemia
D hypernatremia
ANSWER:B
503
A 2-year-old child presents from home with a temperature of 39°C and seizure activity. The
seizure lasts less than 15 minutes. Despite her mother’s aggressive antipyretic administration
during the child’s febrile illnesses, this is the girl’s fourth febrile seizure this year. Which of
the following drugs can decrease the rate of recurrence of febrile seizures?
A carbamazepine

B clonazepam
C phenobarbital
D phenytoin
ANSWER:C
504
A 6-year-old is brought to your office because of “poor color”. At the visit, the child is
other- wise appropriate, but slightly less active than normal, and grayish in appearance. He
admits to drinking water from a pump-well he found with his older brother. You suspect
nitrite poi- soning. Which of the following drug should you administer for treatment of acute
nitrate poisoning?
A hydralazine
B morphine
C methylene blue
D corticosteroids
ANSWER:C
505
A 12-year-old child with a surgically corrected pulmonary shunt is to undergo an invasive
dental procedure. Which of the following antibiotics would be most appropriate for pro-
phylaxis against bacterial endocarditis?
A amoxicillin
B clindamycin
C erythromycin
D oxacillin
ANSWER:A
506
A 4-year-old child has a history of developing a persistent cough that has endured the past 3
weeks. There are multiple sick contacts in the home with similar respiratory complaints. You
suspect atypical pneumonia. Which of the fol- lowing is most appropriate for the treatment
of Mycoplasma pneumoniae infection?
A cefuroxime
B chloramphenicol
C erythromycin
D penicillin
ANSWER:C

507
You are caring for a 6-month-old infant fol- lowing aortic reconstruction in the intensive
care unit. You are called with a panic value that cyanohemoglobin levels were detected on
the infant. You assess the infant and find her oral-tracheally intubated and mechanically
ventilated on numerous sedatives. Cyanide is produced in the metabo- lism of which of the
following?
A sodium nitroprusside
B nitroglycerin
C labetalol
D dobutamine
ANSWER:A
508
You have followed a 5-year-old for a series of neurologic and behavioral issues. You refer
her to a pediatric neurologist for evaluation. After her evaluation, she returns to your clinic
for follow-up and is on ethosuximide. Ethosuximide (Zarontin) is most useful in the
treatment of which symptom?
A phenobarbital overdosage
B absence (petit mal) seizures
C akinetic seizures
D tonic-clonic seizures
ANSWER:B
509
A 14-month-old male is admitted to the PICU for bleeding from his nose and gums. There is
no family history of bleeding tendency. On fur- ther history you determine the family has a
problem with mice and rats and have rodenti- cides around the house. Which of the
following should be included in initial treatment?
A vitamin C
B vitamin K
C copper sulfate
D a phenothiazine
ANSWER:B
510
A 14-year-old female presents to the emergency room 20 hours following ingestion of 8.5 g
of acetaminophen. Therapy with which of the fol- lowing should be initiated?
A deferoxamine
B physostigmine
C N-acetylcysteine
D glutathione
ANSWER:C

511
An 8-year-old with complex medical history has transferred care to your clinic. The child’s
current medication list includes: albuterol, car- bamazepine, ranitidine, and methylphenidate.
Methylphenidate is most commonly prescribed in the management of children with which of
the following?
A poor appetites
B temper tantrums
C seizure disorders
D attention-deficit hyperactivity disorders
ANSWER:D
512
A 6-month-old infant presents from day care with fever, vomiting, and lethargy. On
physical examination, you confirm the lethargy and note a full fontanel despite signs of
dehydra- tion. Which of the following would be the ini- tial antibiotic(s) of choice?
A cefuroxime and vancomycin
B ceftriaxone and vancomycin
C clindamycin
D vancomycin
ANSWER:B
513
A 3-day-old male was born at the hospital and discharged home within 24 hours. The
mother relates that he has been a poor breast-feeder. His parents bring him to the emergency
room because he has become very pale and listless over a matter of hours. On examination,
you note a lethargic newborn in shock with a large liver and a gallop. Chest x-ray reveals
increased pulmonary vasculature and cardiomegaly. You begin antibiotic therapy but suspect
congenital cardiac disease. Which of the following is the drug of choice?
A dobutamine
B digoxin
C prostaglandin E1
D milrinone
ANSWER:C
514
A 7-year-old male adopted from Romania pres- ents with hematemesis. He is known to have
severe chronic hepatitis. An upper endoscopy reveals esophageal variceal bleeding. Which of
the following agents is most likely to be beneficial in controlling the esophageal blood loss?
A octreotide
B prostaglandin E1

C nitric oxide
D prostacyclin
ANSWER:A
515
A 15-year-old adolescent comes to spend the summer with her aunt. She has an appoint-
ment to be seen in your clinic because of her need for frequent medical care. Prior to seeing
the patient you review her records and notice she is on oral pancreatic enzyme replacement
therapy. This is most helpful in patients with which of the following?
A protein-losing enteropathy
B celiac disease
C ulcerative colitis
D cystic fibrosis
ANSWER:D
516
A 2-week-old neonate presents with lethargy, hypothermia, and poor feeding. Evaluation of
the cerebrospinal fluid reveals a white blood cell count of 1200 cells/mm3 (95% polymor-
phonucleocytes) with an elevated protein and decreased glucose. Which of the following is
the initial antibiotic regiment of choice?
A ampicillin
B ceftriaxone
C cefotaxime
D gentamicin with ampicillin
ANSWER:D
517
A 14-year-old undergoing consolidation ther- apy for high-risk, chronic lymphocytic
leukemia develops burning and discomfort with urina- tion. Urine analysis demonstrates
large amount of hemoglobin, along with red and white blood cells. Urine culture is negative.
You diagnose the child with hemorrhagic cystitis. Hemorrhagic cystitis is associated most
closely with large doses of which of the following?
A cyclophosphamide
B methotrexate
C actinomycin D
D L-Asparaginase
ANSWER:A

518
A 2-month-old female presents with a 4-day history of coughing spells that last atleast 30
seconds. After the coughing spells she vomits. She is otherwise well. Laboratory eval- uation
reveals a peripheral white blood cell count of 42,000/mm3 with 86% lymphocytes. Which of
the following prophylaxis should the family and day-care contacts of this child receive?
A rifampin
B erythromycin
C gentamicin
D penicillin
ANSWER:B
519
A 4-year-old presents to the emergency depart- ment with a chief complaint of swelling
around the eyes and ankles. You note the patient to have pitting edema up to the knees
bilaterally and peri-orbital edema with no erythema. You astutely send off a urine analysis
and find that the patient has 3+ protein in the urine without signs of infection or red blood
cells. You make the diagnosis of nephrotic syndrome, likely minimal change disease. What is
the next step in this patient’s management?
A intravenous immunoglobulin (IVIG)
B corticosteroids
C kidney biopsy
D cyclophosphamide
ANSWER:B
520
A well-appearing, 10-year-old presents to the emergency department with a red rash on his
trunk, ankles, elbows, and red lesions on his tongue. You suspect that these look like
petechiae and order some laboratory tests including a complete blood count. When the
laboratory values come back, you see that the platelet count is read as 5,000/µL. The
patient’s blood type is A and he is Rh positive. You diag- nose the patient with idiopathic
thrombocy- topenia purpura. What is the next step in your management?
A intravenous immunoglobulin (IVIG)
B amoxicillin
C anti-D immunoglobulin
D platelet transfusion
ANSWER:C

521
A 5-year-old with a history of sickle cell disease presents to the emergency department with a
2-day history of cough and a 1-day history of fever. You order blood work and a chest x-
ray and she is found to have an infiltrate that is consistent with pneumonia. What is the next
step in your management?
A amoxicillin
B ceftriaxone
C azithromycin
D ceftriaxone and azithromycin
ANSWER:D
522
A 12-month-old comes to the emergency depart- ment after having 4 days of upper
respiratory tract symptoms and fever. Her mother has been giving her acetaminophen every
4 hours and an over-the-counter cold and cough remedy six times daily. She says today that
the child has been more sleepy and not as interactive. She shows you the bottle of cough and
cold remedy and you notice that it also contains acetamino- phen. You suspect
acetaminophen toxicity and this is confirmed with a serum acetaminophen level in the toxic
range, but liver enzymes are normal. How should you proceed?
A contact poison control and administer steroids
B administer furosemide and arrange for admission
C administer naloxone and monitor for improvement
D contact poison control and administer N-acetylcysteine
ANSWER:D
523
A 16-year-old presents to your office com- plaining of abdominal pain and vaginal dis-
charge. She has been sexually active with multiple partners and is concerned that she may be
pregnant. You perform a pregnancy test that is negative and during your pelvic examination
she is noted to have severe cervi- cal motion tenderness. You suspect a diagnosis of pelvic
inflammatory disease. Which of the following is the most appropriate antibiotic regiment for
this patient?
A clindamycin
B ceftriaxone
C doxycycline
D ceftriaxone plus doxycycline
ANSWER:D

524
A 15-year-old comes to your clinic complaining of frequent and heavy menstrual cycles.
She had her first period at the age of 12, had some irregularity; however began having
regular 28 day cycles at the age of 13. For the last 4 months she has been having bleeding on
and off every 1–2 weeks. She is not and has never been sex- ually active, has not had any
vaginal discharge, has no abdominal pain, or other bleeding prob- lems. She is concerned
and wants to better pre- dict her menstrual cycle. You tell her that this is likely dysfunctional
uterine bleeding. What should you recommend?
A testosterone cream
B platelet infusion
C oral contraceptive pill
D nonsteroidal anti-inflammatory drug (NSAID)
ANSWER:C
525
You are called to the newborn nursery to see a 3-day-old that has a rash. The parents are con-
cerned because they did not notice the rash until this morning and are sure that it was not
present the first 2 days of life. You find a well-appearing baby who has scattered red papules
all over the body and are in clumps on the cheeks and on the trunk, some of them even look
like vesicles. You take a scraping of one of the vesicles to examine under the microscope and
find that these vesicles contain a large number of eosinophils. What is the recommended
treatment for this patient?
A acyclovir
B topical steroids
C antidandruff shampoo
D observation
ANSWER:D
526
A 14-month-old child is brought into the emer- gency room unresponsive. He is pale, has
delayed capillary refill, and minimal respira- tory effort. Pulses are barely palpable. You are
requested to obtain vascular access. According to current resuscitation guidelines, what
should your next step be?
A Attempt a central line.
B Place an arterial line.
C Attempt a peripheral IV catheter, if unsuc- cessful place an intraosseous needle.
D Administer intracardiac medications.
ANSWER:C

527
A 24-month-old returns to your clinic after treatment for Streptococcal pharyngitis. His
mother complains that he continues to have an irregular gait after his penicillin shot. You
examine the child and find he has a foot drop. The most likely cause of this finding is which
of the following?
A reaction to penicillin
B complication of intramuscular injection into gluteal area
C complication of intramuscular injection into anterolateral thigh
D complication of the bacterial infection
ANSWER:B
528
A 10-year-old boy with Hodgkin lymphoma is undergoing chemotherapy. The oncologist
wants to start a new regimen but is worried about cardiac toxicity. Cardiomyopathy is most
likely to be caused by which of the following?
A bleomycin
B doxorubicin
C cytarabine
D etoposide
ANSWER:B
529
A 16-year-old girl with acute lymphocytic leukemia nearing the end of induction therapy
presents with tingling in her fingers.Peripheral neuropathy is the most common adverse
effect of which chemotherapeutic agent?
A methotrexate
B cyclophosphamide
C cytarabine
D vincristine
ANSWER:D
530
A 10-year-old girl with acute myeloid leukemia (AML), now day 75 status post a matched,
unrelated bone marrow transplant presents with voluminous diarrhea, nausea, and vomit-
ing. Acute graft-versus-host disease is suspected and high-dose corticosteroids are initiated.
Side- effects of corticosteroids most commonly include which of the following?
A peripheral obesity
B gigantism
C cataracts
D hypoglycemia
ANSWER:C

531
A 6-year-old girl is brought to the emergency room with head injuries after being hit by a
car. Her heart rate is 42 beats per minute, blood pressure is 160/100 mm Hg, respiratory rate
is 8 breaths per minute, and she is minimally responsive. The left pupil is larger than the
right and minimally reactive to light. The deci- sion is made to perform a rapid sequence
intu- bation since she is unable to protect her airway. Which of the following sedative agents
could cause worsening of her condition?
A thiopental
B propofol
C fentanyl
D ketamine
ANSWER:D
532
A full-term baby became cyanotic 5 hours after birth. The baby was found to have persistent
pulmonary hypertension of the newborn (PPHN) and is now being managed in the NICU on
a mechanical ventilator. The baby’s condition is worsening, which of the following
interventions is indicated?
A prostacyclin
B arachidonic acid
C leukotriene C
D nitric oxide
ANSWER:D
533
A 16-year-old male with sickle cell anemia pres- ents to clinic for evaluation of his pain man-
agement. He is having acute pain on top of his chronic pain. Which of the following opioids
is available as a long-acting and short-acting oral formulation as well as intravenous?
A fentanyl
B morphine
C oxycodone
D meperidine
ANSWER:B

534
A 3-year-old girl is admitted to the burn unit following an immersion hot water injury with
resultant second- and third-degree burns cov- ering 60% of the body surface area. Fluids are
started at maintenance plus 4 mL/kg/%BSA burned according to the Parkland formula with
half of the fluid administered over the first 8 hours and the remainder over the following 16
hours. A Foley catheter is placed to monitor urine output. Six hours after admission, the
urine output has decreased to 0.5 mL/kg/h for the past 2 hours. The most appropriate
therapy would be which of the following?
A continue to observe the urine output
B furosemide 1 mg/kg intravenously
C furosemide 2 mg/kg orally
D fluid bolus of 20 mL/kg of normal saline
ANSWER:D
535
A 2-day-old, 1.9-kg boy was delivered to a 19-year-old mother at 36 weeks gestation, with
no prenatal care. Upon preparing the infant for discharge, the patient was noted to be tachyp-
neic, with weak peripheral pulses and grayish cyanosis of the lips. Oxygen saturations were
noted to be 85% and declining. Chest x-ray shows cardiomegaly and an echocardiogram is
ordered which reveals the presence of hypoplastic left heart syndrome (HLHS). Considering
the patient’s present condition, which one of the following is the best initial intervention that
should be taken?
A give indomethacin
B consult a pediatric cardiologist for surgi- cal options
C start prostaglandin E1
D start phenylephrine
ANSWER:C
536
A 10-year-old child presents to the emergency department with an erythematous rash on his
thigh, fever, and myalgia 2 weeks after hunting with his father in eastern Pennsylvania. The
rash appeared about 4 days prior to presenta- tion and has been gradually increasing in size.
He thinks he had a bug bite in the same spot of the rash. The initial treatment of choice is
which of the following?
A streptomycin
B azithromycin
C clindamycin
D doxycycline
ANSWER:D

537
A 16-year-old male presents with fever, chills, nausea, vomiting, and headache. He was
previ- ously healthy, but over the last 24 hours he has felt awful and states he has been
unable to con- centrate for his exams. He also has a petechial rash on his trunk. Gram stain of
CSF fluid shows gram-negative diplococcus. Which of the fol- lowing is the drug of choice
for the treatment of meningococcal meningitis?
A cefotaxime
B ceftriaxone
C vancomycin
D penicillin
ANSWER:D
538
A 3-year-old female was found with her mother’s prenatal vitamins. The bottle was empty
and was estimated to contain 20–25 tablets. At 4 hours postingestion, her serum iron level is
550 mg/dL. Which of the following should be administered?
A dimercaprol (BAL)
B deferoxamine (desferal)
C edetate calcium disodium (EDTA)
D ipecac
ANSWER:B
539
A 10-year-old male is admitted to the hospital with a 4-day history of nausea, vomiting, and
abdominal pain. The pain is located in the epigastrium and radiates to the back. He is cur-
rently completing a course of azithromycin for a sinusitis diagnosed 3 days prior. Home
medica- tions include acetaminophen, metoclopramide, ranitidine, valproic acid, fluticasone
nasal spray, and loratadine. The drug most likely to cause these symptoms is which of the
following?
A acetaminophen
B ranitidine
C azithromycin
D valproic acid
ANSWER:D
540
An adolescent male presents with a urethral discharge. Gram stain of the exudate reveals
intracellular gram-negative diplococci. The initial treatment of choice is which of the
following?
A azithromycin, 1 g orally once
B doxycycline, 100 mg twice a day for 7 days
C procaine penicillin G, 1.2 million units IM once

D ceftriaxone, 125 mg IM once
ANSWER:D
541
A 16-year-old girl presents to the emergency department 12 hours after ingesting 30 extra
strength Tylenol (acetominophen) tablets. She is vomiting, diaphoretic, and lethargic. Which
of the following statements is true regarding the toxicity of acetaminophen overdose?
A primarily involves the liver
B primarily involves the central nervous system
C is most severe in patients less than 2 years of age
D is best treated with sodium bicarbonate to alkalinize the urine
ANSWER:A
542
A neonate is receiving intravenous ampicillin and gentamicin for sepsis and possible
meningitis. A blood culture reveals group B streptococcus. The antibiotic of choice is which
of the following?
A clindamycin
B penicillin
C cefuroxime
D vancomycin
ANSWER:B
543
A 14-year-old patient with acute lymphocytic leukemia presents with fever to 39.6°C. She
was most recently discharged 1 week ago after completing a round of chemotherapy. Blood
culture reveals a nonlactose fermenting gram- negative rod, which is oxidase positive. Which
of the following is the appropriate antibiotic?
A levofloxacin
B azithromycin
C trimethoprim-sulfamethoxazole
D piperacillin-tazobactam
ANSWER:D
544
A 2-year-old male presents to the emergency department with fever to 39.1°C, and abnormal
mental status for the past 12 hours. Laboratory results show a WBC of 567,000, hemoglobin
3.2, and LDH 1197. Acute lymphoblastic leukemia is suspected and the oncologist asks you
to initiate allopurinol therapy. Allopurinol is used most commonly to manage what in acute
leukemia of childhood?
A to induce remission
B to maintain remission

C to help alkalinize the urine
D to prevent hyperuricemia associated with tumor lysis syndrome
ANSWER:D
545
A 15-year-old boy with central core disease and a history of malignant hyperthermia crises is
under- going surgery. He would benefit from which agent during a malignant hyperthermia
crisis?
A ethanol
B succinylcholine
C halothane
D dantrolene
ANSWER:D
546
Gentamicin and piperacillin-tazobactam is initiated in an 8 year old with acute myeloblastic
leukemia for dual coverage of Pseudomonas aeruginosa bacteremia. Gentamicin levels will
be checked to ensure that the peaks are high enough to kill the bacteria relative to the MIC
(peak concentration to MIC or peak/MIC ratio).Which of these drugs is concentration-
dependent like the aminoglycosides?
A doxycycline
B ciprofloxacin
C vancomycin
D cefdinir
ANSWER:B
547
Gentamicin and piperacillin-tazobactam is initiated in an 8 year old with acute myeloblastic
leukemia for dual coverage of Pseudomonas aeruginosa bacteremia. Gentamicin levels will
be checked to ensure that the peaks are high enough to kill the bacteria relative to the MIC
(peak concentration to MIC or peak/MIC ratio).For the above patient, the appropriate time to
check serum gentamicin levels at steady state would be after how many half-lives?
A one
B two
C three
D five
ANSWER:D

548
A 33-week, 1200 g boy is born to a teenage mother who had poor prenatal care. The baby’s
head circumference, height, and weight are below average for his age, indicating poor
intrauterine growth, likely due to exposure to drugs during pregnancy. Intrauterine growth
retardation (IUGR) is a common adverse effect of which drug if taken by the mother during
pregnancy?
A warfarin
B acetaminophen
C lithium
D phenytoin
ANSWER:D
549
An 8-year-old girl with uncontrolled asthma is receiving inhaled corticosteroids and beta-
agonists (Advair, fluticasone/salmeterol) as well as a leukotriene antagonist (Singulair,
montelukast). The decision is made to begin theophylline; however you are worried about
the potential toxicities. What side effects should you monitor for that would indicate high
levels of theophylline?
A nystagmus
B myopathy
C polyuria
D vomiting
ANSWER:D
550
An 18-month-old male toddler presents with pallor. He drinks 64 oz of cow milk per day.
The examina- tion is significant only for an obese and playful male with pallor. Stool is
negative for blood. Which laboratory test would most likely reveal the diagnosis?
A chest x-ray
B examination of stool for ova and parasites
C complete blood count
D serum haptoglobin
ANSWER:C
551
An 18-month-old male toddler presents with pallor. He drinks 64 oz of cow milk per day.
The examina- tion is significant only for an obese and playful male with pallor. Stool is
negative for blood. Testing reveals a mean corpuscular volume of 60 fL (nL 72–86 fL) and
an elevated red cell dis- tribution width. The most likely diagnosis is
A vitamin B12 deficiency
B B-cell leukemia
C hemosiderosis

D iron deficiency anemia
ANSWER:D
552
An 18-month-old male toddler presents with pallor. He drinks 64 oz of cow milk per day.
The examina- tion is significant only for an obese and playful male with pallor. Stool is
negative for blood. Appropriate therapy is started for this patient. When should
reticulocytosis peak?
A 12–24 hours
B 1–3 days
C 5–10 days
D 2–4 weeks
ANSWER:C
553
A 10-month-old female presents in winter with 2 days of rhinorrhea, tachypnea, and
wheezing. She has respirations of 60 breaths per minute, HR of 160 beats per minute, and an
oxygen saturation of 90% on room air. Examination reveals an alert infant in mild
respiratory distress with mild intercostals retractions and coarse bilateral expiratory wheezing.
Appropriate therapy for this infant should begin with which of the following?
A intravenous access and a 20 cc/kg bolus of normal saline
B oxygen by nasal canula
C bag-valve mask followed by rapid- sequence intubation
D posterior-anterior and lateral chest radiographs
ANSWER:B
554
A 10-month-old female presents in winter with 2 days of rhinorrhea, tachypnea, and
wheezing. She has respirations of 60 breaths per minute, HR of 160 beats per minute, and an
oxygen saturation of 90% on room air. Examination reveals an alert infant in mild
respiratory distress with mild intercostals retractions and coarse bilateral expiratory wheezing.
Chest radiography reveals bilateral air trap- ping, chest hyperexpansion, and peribronchial
thickening. Which of the following is the most likely diagnosis?
A congestive heart failure
B acute respiratory distress syndrome (ARDS)
C pneumococcal pneumonia
D viral bronchiolitis
ANSWER:D

555
A 10-month-old female presents in winter with 2 days of rhinorrhea, tachypnea, and
wheezing. She has respirations of 60 breaths per minute, HR of 160 beats per minute, and an
oxygen saturation of 90% on room air. Examination reveals an alert infant in mild
respiratory distress with mild intercostals retractions and coarse bilateral expiratory
wheezing.Which of the following is the best additional therapeutic option for this patient?
A inhaled beta agonist therapy
B ribavirin
C extracorporeal membrane oxygenation
D palivizumab (Synagis)
ANSWER:A
556
A 6-year-old previouslywellmalepresentswith 1 week of worsening elbow swelling and
fever. He denies trauma. Examination reveals a male in mild distress with a temperature to
102°F. His left elbow is warm, erythematous, edematous, and tender around the joint. He is
holding it in mid-flexion and strongly resists passive movement.Which is the most likely
offending organism for his condition?
A Neiserria gonorrheae
B Group B streptococcus
C Pseudomonas aeruginosa
D Staphylococcus aureus
ANSWER:D
557
A 6-year-old previouslywellmalepresentswith 1 week of worsening elbow swelling and
fever. He denies trauma. Examination reveals a male in mild distress with a temperature to
102°F. His left elbow is warm, erythematous, edematous, and tender around the joint. He is
holding it in mid-flexion and strongly resists passive movement.Which of the following
should be performed first?
A administration of oral antibiotics
B arthrocentesis of left elbow
C administration of nonsteroidal anti- inflammatory agents
D fasciotomy of the affected limb
ANSWER:B
558
A 6-year-old previouslywellmalepresentswith 1 week of worsening elbow swelling and
fever. He denies trauma. Examination reveals a male in mild distress with a temperature to
102°F. His left elbow is warm, erythematous, edematous, and tender around the joint. He is
holding it in mid-flexion and strongly resists passive movement.Which of the following is a
poor prognostic factor for this condition?

A age greater than 6 months
B gram-positive infection
C absence of physeal involvement
D hip or shoulder involvement
ANSWER:D
559
A 1-month-old female infant is referred to your clinic for a positive newborn screen for
hypothy- roidism. On history, the mother reports she is “a good baby who sleeps all the time
but is a slow eater.” She was jaundiced for the first 2 weeks and stools twice a week.
Examination reveals an alert infant with a large tongue, cool skin, a large umbil- ical hernia,
edematous extremities, and hypotonia.The most likely cause of this infant’s condi- tion is
which of the following?
A maternal ingestion of propylthiouracil
B thyroid dysgenesis
C iodide transport defects
D thyrotropin deficiency
ANSWER:B
560
A 1-month-old female infant is referred to your clinic for a positive newborn screen for
hypothy- roidism. On history, the mother reports she is “a good baby who sleeps all the time
but is a slow eater.” She was jaundiced for the first 2 weeks and stools twice a week.
Examination reveals an alert infant with a large tongue, cool skin, a large umbil- ical hernia,
edematous extremities, and hypotonia. Which of the following management options should
be initiated next?
A levothyroxine therapy at a dose of 10–15 mcg/kg/day
B thyroid scintiscan at the next available date
C neurodevelopmental consultation
D radiographs of the legs
ANSWER:A
561
A 4-year-old previously healthy boy presents with 1 day of scrotal swelling. His mother
noted his scro- tum to be markedly swollen and thinks his eyes are puffy. Examination
reveals an afebrile child with a BP of 90/50 mm Hg. He is alert with significant bilateral
periorbital edema. His abdomen has ascites with no organomegaly. His scrotum and lower
extremities have tense pitting edema. The initial laboratory test most likely to point to the
etiology of his illness is as which of the following?
A chest radiograph
B liver biopsy

C urine analysis
D hepatitis panel
ANSWER:C
562
A 4-year-old previously healthy boy presents with 1 day of scrotal swelling. His mother
noted his scro- tum to be markedly swollen and thinks his eyes are puffy. Examination
reveals an afebrile child with a BP of 90/50 mm Hg. He is alert with significant bilateral
periorbital edema. His abdomen has ascites with no organomegaly. His scrotum and lower
extremities have tense pitting edema. Subsequent testing reveals a serum albumin of 1 g/dL,
a cholesterol level of 560 mg/dL (nL 109–189 mg/dL), and normal complement and liver
enzyme levels. The most likely diagnosis for this patient is which of the following?
A membranous glomerulonephritis
B focal segmental glomerulosclerosis
C poststreptococcal glomerulonephritis
D minimal-change disease
ANSWER:D
563
A 4-year-old previously healthy boy presents with 1 day of scrotal swelling. His mother
noted his scro- tum to be markedly swollen and thinks his eyes are puffy. Examination
reveals an afebrile child with a BP of 90/50 mm Hg. He is alert with significant bilateral
periorbital edema. His abdomen has ascites with no organomegaly. His scrotum and lower
extremities have tense pitting edema. Which of the following statements regarding this
patient’s most likely condition is true?
A It almost never responds to steroid therapy.
B Spontaneous bacterial peritonitis is not a concern.
C These patients have an increased ten- dency to hemorrhage.
D A low-salt diet is essential during flares of the illness.
ANSWER:D
564
A 7-year-old well female presents to your emer- gency department with episodic headache
and hypertension. During the episodes she is sleepy, complains of headaches, vomits, and
becomes sweaty. Her current vital signs are: T101°F,HR 150 beats per minute, BP 220/130
mm Hg. She is diaphoretic, sleepy but arousable, and clutches her head. Pupils are reactive
and papilledema is present. There is no organomegaly and femoral pulses are normal. After
determining that her airway is intact and breathing sufficient, the first course of action should
be which of the following?
A immediate CT scan of the head to evalu- ate for a mass lesion

B lumbar puncture to rule out meningitis
C administration of an antihypertensive medication
D administration of a benzodiazepine to relieve the patient’s anxiety
ANSWER:C
565
A 7-year-old well female presents to your emer- gency department with episodic headache
and hypertension. During the episodes she is sleepy, complains of headaches, vomits, and
becomes sweaty. Her current vital signs are: T101°F,HR 150 beats per minute, BP 220/130
mm Hg. She is diaphoretic, sleepy but arousable, and clutches her head. Pupils are reactive
and papilledema is present. There is no organomegaly and femoral pulses are normal. Her
vital signs are stabilized on the appropri- ate therapy. Now happy and interactive, she is
transferred to the ICU. Which option is most likely to lead to a diagnosis?
A magnetic resonance imaging of the brain
B urine for catecholamines
C ECG and echocardiography
D inquiry into a family history of essential hypertension
ANSWER:B
566
A 7-year-old well female presents to your emer- gency department with episodic headache
and hypertension. During the episodes she is sleepy, complains of headaches, vomits, and
becomes sweaty. Her current vital signs are: T101°F,HR 150 beats per minute, BP 220/130
mm Hg. She is diaphoretic, sleepy but arousable, and clutches her head. Pupils are reactive
and papilledema is present. There is no organomegaly and femoral pulses are normal.The
urinary vanillyl mandelic acid level returns at 400 mg/g creatinine (nL < 8 mg/g creatinine).
Which of the following is the best treatment for this condition?
A surgical removal of all tissue
B radiologic injection of cyclophos- phamide directly
C watchful waiting as most regress spontaneously
D oral mineralcorticoid therapy
ANSWER:A

567
You are seeing a 7-day-old male infant for a well- child check. The baby is breastfeeding
well. He has had no fever or emesis. He passed his first stool at3 days of age, and has not
passed another stool. Examination reveals an afebrile, well appearing, and vigorous baby.
The abdomen is firm and slightly distended with bowel sounds present. The perianal area is
slightly erythematous and rectal examination reveals increased tone with no stool present in
the rectal vault.The initial diagnostic approach is unsuccessful. Which test to should you
perform next to diag- nose this patient?
A serum TSH
B stool for esosinophils
C anal manometry
D rectal biopsy
ANSWER:D
568
You are seeing a 7-day-old male infant for a well- child check. The baby is breastfeeding
well. He has had no fever or emesis. He passed his first stool at3 days of age, and has not
passed another stool. Examination reveals an afebrile, well appearing, and vigorous baby.
The abdomen is firm and slightly distended with bowel sounds present. The perianal area is
slightly erythematous and rectal examination reveals increased tone with no stool present in
the rectal vault.Testing reveals an absence of the Meissner and Auerbach plexus. With proper
treatment, what is the most likely prognosis for this patient?
A he will likely be continent
B he will need a colostomy for his entire childhood
C he will need a total colectomy and likely have serious incontinence problems
D he will need a small bowel transplant and require lifelong parenteral nutrition
ANSWER:A
569
A previously well 2-year-old girl is brought to you for evaluation of a “broken elbow.” The
father reports swinging her round and round by her left arm and leg. She now is crying and
not moving her left arm as it is held in a flexed and slightly pronated position at her side. No
effusions or point tenderness are discernible.Which of the following is the most appropriate
initial management?
A intravenous line placement for sedation and reduction
B hyperpronation or supination-flexion of the arm
C complete blood count and erythrocyte sedimentation rate
D social work evaluation for abuse
ANSWER:B

570
A previously well 2-year-old girl is brought to you for evaluation of a “broken elbow.” The
father reports swinging her round and round by her left arm and leg. She now is crying and
not moving her left arm as it is held in a flexed and slightly pronated position at her side. No
effusions or point tenderness are discernible.When counseling the father regarding his child’s
condition, you should tell him which of the following?
A recurrence is unlikely
B splinting at bedtime is desirable
C gymnastics should be avoided
D children outgrow predisposition for this condition
ANSWER:D
571
A 16-year-old female living in a shelter presents with
3 days of worsening lower abdominal pain. She reports multiple unprotected sexual
encounters. She is febrile to 39°C and is ill appearing. She has bilateral lower quadrant
tenderness without rebound. Her pelvic examination reveals purulent vaginal dis- charge
with adnexal and cervical motion tenderness. Pregnancy test is negative.What is the most
appropriate management strategy at this time?
A inpatient admission with intravenous antibiotics for pelvic inflammatory disease
B social service referral for in loco parentis designation prior to initiation of therapy
C
initiation of antibiotic therapy in the emergency department and discharge with close
outpatient follow-up
D urine analysis and empiric intramuscu- lar antibiotic therapy for cystitis
ANSWER:A
572
A 16-year-old female living in a shelter presents with
3 days of worsening lower abdominal pain. She reports multiple unprotected sexual
encounters. She is febrile to 39°C and is ill appearing. She has bilateral lower quadrant
tenderness without rebound. Her pelvic examination reveals purulent vaginal dis- charge
with adnexal and cervical motion tenderness. Pregnancy test is negative.The patient remains
febrile and has worsening abdominal pain despite 72 hours of antibiotic therapy. At this time
what is the best next step?
A continue therapy at the scheduled doses
B add diphenhydramine for a potential drug reaction
C consult gynecology for imaging and drainage of a possible tubo-ovarian abscess
D repeat blood cultures and obtain a C-reactive protein to help monitor response to therapy
ANSWER:C

573
A 16-year-old female living in a shelter presents with
3 days of worsening lower abdominal pain. She reports multiple unprotected sexual
encounters. She is febrile to 39°C and is ill appearing. She has bilateral lower quadrant
tenderness without rebound. Her pelvic examination reveals purulent vaginal dis- charge
with adnexal and cervical motion tenderness. Pregnancy test is negative.The patient improves
and is ready for discharge. Which of the following is true about this patient’s condition?
A recurrence is rare
B recurrence is associated with an increased risk of infertility
C douching and oral contraceptives may decrease the risk of recurrence
D male partners with N gonorrheae are always symptomatic
ANSWER:B
574
An 11-year-old boy presents after cutting his ankle on a rusty piece of metal at the junkyard.
His father reports he is positive his son’s immunizations are cur- rent because the patient
received booster doses just prior to kindergarten at 5 years of age. Examination reveals a
healthy boy in no distress with a 4 cm bleed- ing laceration on the posterior aspect of the
lower leg.What should be the first step in management of this wound?
A radiography to evaluate for metallic particles
B irrigation of the wound with normal saline
C intravenous antibiotics directed against gram-positive organisms
D direct pressure on the wound for hemostasis
ANSWER:D
575
An 11-year-old boy presents after cutting his ankle on a rusty piece of metal at the junkyard.
His father reports he is positive his son’s immunizations are cur- rent because the patient
received booster doses just prior to kindergarten at 5 years of age. Examination reveals a
healthy boy in no distress with a 4 cm bleed- ing laceration on the posterior aspect of the
lower leg.The wound is treated. For tetanus prophylaxis, which of the following should this
patient receive?
A no prophylaxis
B Td only
C Td and tetanus immune globulin (TIG)
D DTaP
ANSWER:B

576
A 6-month-old boy is brought in by his mother for crying. Your examination reveals a thin
boy with tender swelling around the midshaft of his left femur. He cries when the leg is
manipulated, but is comfort- able when left alone. An x-ray reveals a transverse midshaft
femur fracture. The father tells you the patient’s 16-month-old brother lifted him off the bed
and dropped him on the floor earlier that evening.During the evaluation the father asks that
they be discharged immediately as he has to be at work early. What is the most appropriate
course of action?
A let them go home and follow-up with their pediatrician the next day
B splint the child with follow-up by an orthopedist
C call security to forcibly incarcerate the father
D
inform the family that the safety of the child may be at risk as the history given does not
match with his pattern of injuries
ANSWER:D
577
A 3-month-old female presents with 2 days of crying and decreased oral intake. She is
afebrile, HR is 280 beats per minute, and BP is 85/40 mm Hg. Her satu- rations are 99% on
room air. She is alert and easily consolable, her lungs are clear, and she is tachycardic with
no murmurs audible. There is no organomegaly and the peripheral pulses are normal in
strength. Which test should you order first to confirm the diagnosis?
A complete blood count and differential
B echocardiogram
C blood glucose
D electrocardiogram
ANSWER:D
578
A 3-month-old female presents with 2 days of crying and decreased oral intake. She is
afebrile, HR is 280 beats per minute, and BP is 85/40 mm Hg. Her satu- rations are 99% on
room air. She is alert and easily consolable, her lungs are clear, and she is tachycardic with
no murmurs audible. There is no organomegaly and the peripheral pulses are normal in
strength. Her heart rate and blood pressure remain unchanged after testing. The initial
treatment for the most likely cause of her condition is which of the following?
A adenosine
B defibrillation
C verapamil
D digoxin
ANSWER:A

579
A 15-year-old previously well female presents with 1 week of hair loss. She denies fever,
weight loss, or medications. On examination, she is pleasant but nervous about her hair loss.
Her scalp reveals patches of complete hair loss with small broken hair that easily pull out at
the edges. The scalp is smooth and no inflammation is seen. Microscopic examination of the
shafts reveals the stubs to resemble exclamation points.What illness does this patient most
likely have?
A tinea capitis
B alopecia areata
C trichotillomania
D hair traction alopecia
ANSWER:B
580
A 15-year-old previously well female presents with 1 week of hair loss. She denies fever,
weight loss, or medications. On examination, she is pleasant but nervous about her hair loss.
Her scalp reveals patches of complete hair loss with small broken hair that easily pull out at
the edges. The scalp is smooth and no inflammation is seen. Microscopic examination of the
shafts reveals the stubs to resemble exclamation points.You tell the patient the likely course
of this illness is which of the following?
A spontaneous resolution
B resolution after chemotherapy
C difficult to predict
D progression to total hair loss and then resolution
ANSWER:C
581
A 6-month-old female develops a persistent cough with progressively worsening paroxysms
and cyanosis. There is occasional posttussive emesis. The child is afebrile. Between coughing
spells, the physical examination is normal. At this time, what would be most important
question to ask the family regarding patient history?
A birth weight
B immunizations
C consanguinity
D early infant deaths in relatives
ANSWER:B

582
A 6-month-old female develops a persistent cough with progressively worsening paroxysms
and cyanosis. There is occasional posttussive emesis. The child is afebrile. Between coughing
spells, the physical examination is normal.The white blood cell count on the patient is
32,000/mm3, with 80% lymphocytes and 2% mononuclear cells. What is the most appropri-
ate next step at this time?
A order a bone marrow examination
B prescribe intravenous gammaglobulin
C prescribe oral azithromycin
D perform a lumbar puncture
ANSWER:C
583
A 6-month-old female develops a persistent cough with progressively worsening paroxysms
and cyanosis. There is occasional posttussive emesis. The child is afebrile. Between coughing
spells, the physical examination is normal.The most appropriate method to identify the
responsible organism is via which of the following?
A throat swab
B nasopharyngeal swab
C blood culture
D sputum culture
ANSWER:B
584
A 17-year-old female presents to your clinic with complaints of recurrent headaches for 6
months. They are described as circumferential; onset is not associated with time of day.
There has been no eme- sis and the headaches have not interfered with activ- ities. Her
weight is 140 kg and her BP is 140/90 mm Hg. Her examination reveals bilateral
papilledema, and an otherwise normal neurologic examination. What is the best next step in
management of this patient?
A oral administration of nifedipine
B determination of renin levels
C intravenous nitroprusside drip
D computerized tomography of the head
ANSWER:D

585
A 17-year-old female presents to your clinic with complaints of recurrent headaches for 6
months. They are described as circumferential; onset is not associated with time of day.
There has been no eme- sis and the headaches have not interfered with activ- ities. Her
weight is 140 kg and her BP is 140/90 mm Hg. Her examination reveals bilateral
papilledema, and an otherwise normal neurologic examination.The imaging study is normal,
reveals no mass and normal-sized ventricles. At this point you should do which of the
following?
A perform lumbar puncture
B treat with dexamethasone
C treat with acetazolamide
D refer her for opthalmalogic evaluation
ANSWER:A
586
A 17-year-old female presents to your clinic with complaints of recurrent headaches for 6
months. They are described as circumferential; onset is not associated with time of day.
There has been no eme- sis and the headaches have not interfered with activ- ities. Her
weight is 140 kg and her BP is 140/90 mm Hg. Her examination reveals bilateral
papilledema, and an otherwise normal neurologic examination.You proceed and she has
relief from her symp- toms. Despite appropriate care she begins to develop visual loss and
optic nerve atrophy is suspected. Your best course of action is to do which of the following?
A increasing the dose of dexamethasone
B increasing the frequency of lumbar punctures
C increasing the dose of acetozolamide
D refer to an ophthalmologist for evalua- tion for optic nerve fenestration
ANSWER:D
587
A 10-month-old female presents to the emergency department with a 2-day history of runny
nose and fever to 102°F. On examination, her temperature is 103°F, HR 140 beats per
minute, and a RR 30 times per minute. She is alert and playful with copious rhinorrhea. After
the examination is complete, she becomes stiff and displays tonic-clonic movements of all
four extremities.Which of the following should be your first task?
A obtain whole blood glucose
B administer intravenous lorazepam (Ativan)
C perform lumbar puncture
D establish airway patency
ANSWER:D

588
A 10-month-old female presents to the emergency department with a 2-day history of runny
nose and fever to 102°F. On examination, her temperature is 103°F, HR 140 beats per
minute, and a RR 30 times per minute. She is alert and playful with copious rhinorrhea. After
the examination is complete, she becomes stiff and displays tonic-clonic movements of all
four extremities. After 5 minutes, the seizure ceases and the res- piratory rate is 30 breaths
per minute. The patient is sleepy but arousable. Fundoscopic examination is normal as is the
remainder of the physical examination. Rectal acetaminophen is given. Which of the
following tests should be performed at this time?
A electroencephalogram
B skull radiographs
C arterial blood gas
D lumbar puncture
ANSWER:D
589
A 10-month-old female presents to the emergency department with a 2-day history of runny
nose and fever to 102°F. On examination, her temperature is 103°F, HR 140 beats per
minute, and a RR 30 times per minute. She is alert and playful with copious rhinorrhea. After
the examination is complete, she becomes stiff and displays tonic-clonic movements of all
four extremities. The patient quickly becomes alert, happy, and playful. She is afebrile and
has a normal exam- ination. All ordered tests are normal. Which of the following
management plans is most appropriate at this time?
A discharge home after instructions regarding home management of fever and seizures
B discharge home on phenytoin
C discharge home on phenobarbital
D
discharge home with alternating doses of ibuprofen and acetaminophen every 3 hours for
the next 3 days
ANSWER:A
590
A 6-year-old female presents with short stature. Her family reports she has been well and has
had no other medical problems. Her diet and review of sys- tems is unremarkable. Which of
the following would be most impor- tant to know at this time?
A maternal age at menarche
B parental growth rate and height
C paternal age at conception
D maternal age at conception
ANSWER:B

591
A 6-year-old female presents with short stature. Her family reports she has been well and has
had no other medical problems. Her diet and review of sys- tems is unremarkable. The
physical examination reveals a pleasant girl at less than the 5th percentile for height and 10th
percentile for weight. In addition to her short stature, she has a broad chest, cubi- tum
valgum, and 2/6 systolic ejection murmur at the right upper sternal border. Which test is
most likely to confirm the etiology of her short stature at this time?
A serum LH/FSH levels
B abdominal ultrasound
C growth hormone levels
D karyotype
ANSWER:D
592
A 13-month-old boy presents with 5 days of fever to 103°F. His temperature is 102.8°F, HR
160 beats per minute, and RR 36 times per minute. On examination, he is found to be
irritable with markedly injected con- junctiva, a strawberry tongue, and red, cracked lips. A
2-cm lymph node is present in the left anterior cer- vical chain. There is no meningismus.
His lungs are clear and he is tachycardic. A diffuse erythematous blanching rash is present on
his chest and extremities. No desquamation of the fingertips is noted. Among the following,
which diagnostic test is most important in establishing the disease most likely to be causing
this constellation of symptoms?
A complete blood count
B erythrocyte sedimentation rate
C viral culture
D clinical judgment
ANSWER:D
593
A 13-month-old boy presents with 5 days of fever to 103°F. His temperature is 102.8°F, HR
160 beats per minute, and RR 36 times per minute. On examination, he is found to be
irritable with markedly injected con- junctiva, a strawberry tongue, and red, cracked lips. A
2-cm lymph node is present in the left anterior cer- vical chain. There is no meningismus.
His lungs are clear and he is tachycardic. A diffuse erythematous blanching rash is present on
his chest and extremities. No desquamation of the fingertips is noted.After the diagnosis is
confirmed, therapy is ini- tiated with improvement in symptoms within 24 hours. Which of
the following complications is most likely to occur in this illness?
A thrombocytopenia
B sterility

C hydrops of the gallbladder
D ulcerative colitis
ANSWER:C
594
A 13-month-old boy presents with 5 days of fever to 103°F. His temperature is 102.8°F, HR
160 beats per minute, and RR 36 times per minute. On examination, he is found to be
irritable with markedly injected con- junctiva, a strawberry tongue, and red, cracked lips. A
2-cm lymph node is present in the left anterior cer- vical chain. There is no meningismus.
His lungs are clear and he is tachycardic. A diffuse erythematous blanching rash is present on
his chest and extremities. No desquamation of the fingertips is noted.Prior to discharge, the
parents state that their child needs his immunizations updated. You should counsel the
parents that their son should follow which course of treatment?
A stay on schedule for his vaccinations
B have measles and varicella immuniza- tions deferred for 11 months after IVIG
C
not be given diphtheria-tetanus-acellu- lar pertussis vaccine because it may potentiate a
relapse of his illness
D not receive influenza vaccine because he will be on chronic aspirin therapy
ANSWER:B
595
A 2-year-old is brought to your emergency department for refusal to walk after he tripped
and fell while run- ning. Examination reveals a well-appearing afebrile child in no distress.
His left leg has full range of motion and some point tenderness in the distal tibia.The most
appropriate first step in your evalu- ation is to obtain which of the following?
A a C-reactive protein
B a blood culture
C radiographs of the tibia
D a social work consultation
ANSWER:C
596
A 2-year-old is brought to your emergency department for refusal to walk after he tripped
and fell while run- ning. Examination reveals a well-appearing afebrile child in no distress.
His left leg has full range of motion and some point tenderness in the distal tibia.Your initial
evaluation confirms your suspi- cion. At this point you should perform which task next?
A make a referral to child protective serv- ices for abuse
B place a long-term intravenous catheter for intravenous antibiotics

C arrange an immediate orthopedic referral
D immobilize the affected limb in a splint with outpatient follow-up
ANSWER:D
597
You are called to the nursery to evaluate a 3-day- old, full-term male infant with lethargy.
The nurse reports the infant was feeding well on standard for- mula until 4 hours previously.
He has had no emesis. There is no maternal history of fever or rash. The infant currently is
afebrile with a HR 110 beats per minute, RR of 50 times per minute, and a BP of 80/45 mm
Hg. He is lethargic but the examination is otherwise unremarkable. There are no dysmorphic
features. Whole blood glucose is normal. Serum cal- cium and electrolyte results are
pending.What should be your next step in the evalua- tion of the patient?
A obtain emergent abdominal ultrasound
B
obtain complete blood count, urine analysis and lumbar puncture as well as cultures of
blood, urine, and CSF
C urine drug screen
D obtain emergent upper gastrointestinal series with small bowel follow-through
ANSWER:B
598
You are called to the nursery to evaluate a 3-day- old, full-term male infant with lethargy.
The nurse reports the infant was feeding well on standard for- mula until 4 hours previously.
He has had no emesis. There is no maternal history of fever or rash. The infant currently is
afebrile with a HR 110 beats per minute, RR of 50 times per minute, and a BP of 80/45 mm
Hg. He is lethargic but the examination is otherwise unremarkable. There are no dysmorphic
features. Whole blood glucose is normal. Serum cal- cium and electrolyte results are
pending.The neonatal evaluation for sepsis is complete and the neonate is placed on
antibiotics. The next day there is no evidence of infection and the infant has not improved.
Additional studies are obtained, including a serum ammonia which is 1150 mmol/L (nL
64–107) and a blood pH of 7.36. Medical therapy is initiated. The urinary orotic acid level
returns and is markedly ele- vated as well. Which of the following is the most likely
diagnosis?
A ornithine transcarbamylase deficiency
B carbamoyl synthetase deficiency
C methylmalonic academia
D carnitine palmitoyl transferase deficiency
ANSWER:A

599
You are called to the nursery to evaluate a 3-day- old, full-term male infant with lethargy.
The nurse reports the infant was feeding well on standard for- mula until 4 hours previously.
He has had no emesis. There is no maternal history of fever or rash. The infant currently is
afebrile with a HR 110 beats per minute, RR of 50 times per minute, and a BP of 80/45 mm
Hg. He is lethargic but the examination is otherwise unremarkable. There are no dysmorphic
features. Whole blood glucose is normal. Serum cal- cium and electrolyte results are
pending.Six hours after institution of treatment with appropriate intravenous doses of
arginine, the serum ammonia is 1200 mmol/L. The best course of action at this time is which
of the following?
A increase the rate of IV arginine to clear ammonia faster
B begin a double volume exchange transfusion
C transfer the patient to a center where hemodialysis can be performed
D discuss DNR status with the family
ANSWER:C
600
A 3-year-old girl is seen in the emergency depart- ment for bruising. Her family denies fever
or weight loss but states she had a “cold” 3 weeks ago. She is afebrile and the remaining vital
signs are normal. She is happy and playful and has generalized ecchy- moses and
petechiae.What should be the first test you obtain?
A bone marrow aspirate
B Neiserria meningitidis latex assay of the cerebrospinal fluid
C Rickettsia rickettsiae serology
D complete blood count and differential
ANSWER:D
601
A 4-year-old boy presents to the emergency depart- ment with a chief complaint of pallor.
He was well until 1 week previously when he developed bloody diarrhea that resolved with
oral antibiotics. He is afebrile with a BP of 150/100 mm Hg and a HR of 130 beats per
minute. He is alert and fundoscopic examination is normal. His examination is signifi- cant
for pallor.Which test is most likely to reveal the diagnosis?
A complete blood count and smear
B stool culture
C computerized tomography of the head
D renal ultrasound
ANSWER:A

602
The laboratory results reveal a hemoglobin of 8 g/dL, a BUN and creatinine of 40 and 1.8,
respectively. The urine output is normal. What should you tell the family regarding the treat-
ment of this disease?
A
A 4-year-old boy presents to the emergency depart- ment with a chief complaint of pallor.
He was well until 1 week previously when he developed bloody diarrhea that resolved with
oral antibiotics. He is afebrile with a BP of 150/100 mm Hg and a HR of 130 beats per
minute. He is alert and fundoscopic examination is normal. His examination is signifi- cant
for pallor.aggressive medical management and dialysis result in the majority of the patients
doing well
B high-dose steroids are essential in the treatment of this illness
C most patients eventually require renal transplantation
D antibiotics need to be continued for 21 days
ANSWER:A
603
An 18-month-old female presents to the emer- gency department with fever up to 102°F, a
barky cough, and stridor. The examination shows an RR of 60 breaths per minute with an
oxygen saturation of 95% while breathing room air, marked stridor, moderate substernal
retractions, and equal aeration without wheezes or rhonchi. As the patient is being evaluated,
initial ther- apy for this patient should begin with which of the following?
A acetaminophen per rectum
B nebulized dexamethasone
C nebulized racemic epinephrine
D humidified air only in a position of comfort
ANSWER:C
604
An 18-month-old female presents to the emer- gency department with fever up to 102°F, a
barky cough, and stridor. The examination shows an RR of 60 breaths per minute with an
oxygen saturation of 95% while breathing room air, marked stridor, moderate substernal
retractions, and equal aeration without wheezes or rhonchi. Despite therapy the patient
becomes more toxic appearing and is drooling. The parents subse- quently report the child
has received no immu- nizations. Her respiratory rate is now 70 breaths per minute and she
appears sleepy. At this point the best intervention is which of the following?
A blind intubation with a laryngeal mask airway
B humidified oxygen and observation with the patient in a position of comfort
C rapid sequence intubation in the emer- gency department
D anesthesia assistance for intubation in the operating room with surgical availability

ANSWER:D
605
A 4-year-old male presents to the office with wheez- ing and increased work of breathing for
the fifth time in the past year. On examination, his RR is 30 breaths per minute and oxygen
saturation is 98% while breathing room air. He is talking in complete sen- tences and there is
expiratory wheezing with a pro- longed expiratory phase. What is the first treatment he
should receive?
A nebulized beta agonist therapy
B intravenous beta agonist therapy
C subcutaneous epinephrine
D oxygen via 15 L nonrebreather
ANSWER:A
606
A previously well 9-year-old Hispanic male presents with 2 days of yellow eyes and
abdominal pain. He visited Mexico 1 month previously. On examination he is febrile to
101°F, has scleral icterus, and moderate right upper quadrant tenderness. The liver is moder-
ately enlarged. What would be the most helpful additional history?
A sickle cell disease in the family
B excessive carrot intake
C history of malar rash
D vaccination status prior to travel
ANSWER:D
607
A previously well 9-year-old Hispanic male presents with 2 days of yellow eyes and
abdominal pain. He visited Mexico 1 month previously. On examination he is febrile to
101°F, has scleral icterus, and moderate right upper quadrant tenderness. The liver is moder-
ately enlarged.What test would most likely confirm his diag- nosis?
A hepatitis A serology
B hepatitis B serology
C 24-hour copper excretion
D abdominal ultrasound
ANSWER:A
608
A previously well 9-year-old Hispanic male presents with 2 days of yellow eyes and
abdominal pain. He visited Mexico 1 month previously. On examination he is febrile to
101°F, has scleral icterus, and moderate right upper quadrant tenderness. The liver is moder-
ately enlarged.Diagnostic testing confirms the diagnosis and the family has questions about
the disease. You should tell them which of the following?
A an effective vaccine is not routinely available

B the recurrence risk is approximately 25%
C if he were to contract hepatitis D simul- taneously, he would likely need a liver transplant
D fulminant hepatitis is uncommon in children with hepatitis A infection
ANSWER:D
609
You are called to see a 2-hour-old male with cyanosis and tachypnea. Oxygen saturation is
80% while breathing room air and the RR is 60 breaths per minute with BP of 80/50 mm Hg.
The baby is cyan- otic and there are no murmurs. The rest of the exam- ination is
unremarkable. The baby is placed in on an FiO2 of 1.0 by head hood and arterial blood
gases reveal the PaO2 to be 30 mm Hg. What is the most likely etiology of the hypox- emia?
A methemoglobinemia
B cyanotic congenital heart disease
C sepsis
D pneumonia
ANSWER:B
610
You are called to see a 2-hour-old male with cyanosis and tachypnea. Oxygen saturation is
80% while breathing room air and the RR is 60 breaths per minute with BP of 80/50 mm Hg.
The baby is cyan- otic and there are no murmurs. The rest of the exam- ination is
unremarkable. The baby is placed in on an FiO2 of 1.0 by head hood and arterial blood
gases reveal the PaO2 to be 30 mm Hg. A chest x-ray shows normal lung fields, slightly
generous cardiothymic silhouette, and a narrow upper mediastinum. An electrocardiogram is
normal.What would the most likely diagnosis based on this information?
A transposition of the great arteries
B tetralogy of Fallot
C ventricular septal defect
D endocardial fibroelastosis
ANSWER:A
611
You are called to see a 2-hour-old male with cyanosis and tachypnea. Oxygen saturation is
80% while breathing room air and the RR is 60 breaths per minute with BP of 80/50 mm Hg.
The baby is cyan- otic and there are no murmurs. The rest of the exam- ination is
unremarkable. The baby is placed in on an FiO2 of 1.0 by head hood and arterial blood
gases reveal the PaO2 to be 30 mm Hg. The airway is stabilized and prostaglandin E1
therapy is initiated. Echocardiography con- firms the diagnosis. At this point what is the best
intervention?

A emergent cardiac catheterization and atrial septostomy
B nothing until the patient’s hypoxemia improves
C empiric antibiotics for sepsis
D emergent cardiovascular surgery consultation for immediate arterial switch
ANSWER:A
612
A 6-year-old previously well African American child presents with new-onset jaundice, dark
urine, and pallor. There is a history of a recent mild upper respiratory tract infection. Vital
signs are normal. Physical examination is remarkable for icterus and pallor. Laboratory
examination reveals hemoglobin of 7 g/dL with a normal platelet and white blood cell count.
The total bilirubin is 4 mg/dL.Which of the following historical findings is most likely to
indicate the diagnosis?
A His father has sickle cell trait.
B Baby aspirin given to him last year was associated with dark urine.
C His grandmother was diagnosed with leukemia last year.
D He was diagnosed as being iron defi- cient 4 years ago.
ANSWER:B
613
A 6-year-old previously well African American child presents with new-onset jaundice, dark
urine, and pallor. There is a history of a recent mild upper respiratory tract infection. Vital
signs are normal. Physical examination is remarkable for icterus and pallor. Laboratory
examination reveals hemoglobin of 7 g/dL with a normal platelet and white blood cell count.
The total bilirubin is 4 mg/dL.The bilirubin is predominantly unconjugated and the
reticulocyte count is 12%. Further his- tory reveals that for the past 2 days he has been given
an antibiotic that his grandfather was taking for a urinary tract infection. What is the most
likely diagnosis?
A sickle cell disease
B preleukemia
C infectious hepatitis
D glucose-6-phophate-dehydrogenase deficiency
ANSWER:D
614
A 6-year-old previously well African American child presents with new-onset jaundice, dark
urine, and pallor. There is a history of a recent mild upper respiratory tract infection. Vital
signs are normal. Physical examination is remarkable for icterus and pallor. Laboratory
examination reveals hemoglobin of 7 g/dL with a normal platelet and white blood cell count.
The total bilirubin is 4 mg/dL.What is the most appropriate management at this time?

A immediate transfusion to bring the hemoglobin to 14 g/dL
B immediate transfusion to bring the hemoglobin to 10 g/dL
C withhold transfusion and follow vital signs and hemoglobin level
D initiate prednisone at 2 mg/kg/day
ANSWER:C
615
A 16-year-old female presents with leg weakness after recovering from an upper respiratory
illness. On examination her vital signs are normal. She is unable to stand alone. Motor
strength is 5/5 in the arms and 2/5 in the legs. Deep tendon reflexes are absent in the
legs.What finding is classically associated with this illness?
A hydrocephalus
B elevated serum C-reactive protein
C myoglobulinuria
D elevated cerebrospinal fluid protein
ANSWER:D
616
A 16-year-old female presents with leg weakness after recovering from an upper respiratory
illness. On examination her vital signs are normal. She is unable to stand alone. Motor
strength is 5/5 in the arms and 2/5 in the legs. Deep tendon reflexes are absent in the
legs.Testing demonstrates marked slowing of nerve conduction velocity. What would the
most appropriate intervention be at this time?
A administration of intravenous immune globulin
B discharge and reassurance about the overall benign nature of this disease
C administration of intravenous fresh frozen plasma
D supplemental oxygen via nasal cannula
ANSWER:A
617
A 5-year-old male presents with a 5-lb weight loss over the previous 2 weeks and nighttime
enuresis. On examination, he is alert and talkative with a BP of 90/60 mm Hg and a HR of
130 beats per minute. Mucous membranes are sticky and respirations are rapid and deep.
The bedside whole blood glucose is 750 mg/dL. Intravenous access is obtained.What should
the first step in management be?
A regular insulin 0.1 U/kg IV push
B bicarbonate 1 meq/kg IV over 1 hour
C endotracheal intubation
D 20 cc/kg isotonic crystalloid solution over 1 hour
ANSWER:D

618
A 5-year-old male presents with a 5-lb weight loss over the previous 2 weeks and nighttime
enuresis. On examination, he is alert and talkative with a BP of 90/60 mm Hg and a HR of
130 beats per minute. Mucous membranes are sticky and respirations are rapid and deep.
The bedside whole blood glucose is 750 mg/dL. Intravenous access is obtained.After your
intervention, the whole blood glu- cose is 485 mg/dL. Intravenous fluids and insulin are
given, and the patient is admitted for further care. Which of the following meta- bolic
abnormalities is most likely to occur during insulin therapy?
A hypokalemia
B hyperkalemia
C hyperphosphatemia
D hypercalcemia
ANSWER:A
619
A 13-month-old female in day care presents to the outpatient clinic with 3 days of
rhinorrhea, cough, and fever to 101°F. Her examination is significant for clear nasal
congestion and red tympanic mem- branes that are mobile with insufflation. The mother asks
for antibiotics.What is the most appropriate therapy at this time?
A amoxicillin 80–90 mg/kg/day for 10 days
B amoxicillin 40–50 mg/kg/day for 5 days
C ceftriaxone 50 mg/kg IM in one dose
D acetaminophen and fluids
ANSWER:D
620
A 13-month-old female in day care presents to the outpatient clinic with 3 days of
rhinorrhea, cough, and fever to 101°F. Her examination is significant for clear nasal
congestion and red tympanic mem- branes that are mobile with insufflation. The mother asks
for antibiotics.The mother brings her child in 6 days later for persistent fever (up to 103°F)
and pulling at the left ear. On examination, the left tympanic membrane is bulging and
immobile. The bony landmarks are not visible. The mother states the day care won’t let her
back in without antibiotics. The most appropriate therapy at this time is which of the
following?
A amoxicillin 80–90 mg/kg/day for 10 days
B erythromycin 40–50 mg/kg/day for 10 days
C ceftriaxone 50 mg/kg IM in one dose
D otorhinolaryngology consult for drainage
ANSWER:A

621
A 13-month-old female in day care presents to the outpatient clinic with 3 days of
rhinorrhea, cough, and fever to 101°F. Her examination is significant for clear nasal
congestion and red tympanic mem- branes that are mobile with insufflation. The mother asks
for antibiotics.The patient returns again in 2 days with contin- ued fevers to 104°F. The child
is still fussy. There is redness, swelling, and tenderness posterior to the left ear and the pinna
is displaced forward. The most appropriate therapy at this time is which of the following?
A amoxicillin 80–90 mg/kg/day for 10 days
B amoxicillin 40–50 mg/kg/day for 5 days
C ceftriaxone 50 mg/kg IM in one dose
D otorhinolaryngology consult for drainage
ANSWER:D
622
A mother brings her 17-year-old daughter to your clinic for evaluation of 6 weeks of
fatigue, 10-lb weight loss, and listlessness. The vital signs include a HR of 70 beats per
minute and BP of 90/50 mm Hg. Examination reveals a thin girl with a flat affect, but no
other abnormalities.The best next step in this patient’s manage- ment would be which of the
following?
A obtain a serum beta-human chorionic gonadotrophin (b-HCG)
B a referral to an oncologist for a bone marrow aspirate
C obtain stat urine drug screen
D interview the patient alone
ANSWER:D
623
A mother brings her 17-year-old daughter to your clinic for evaluation of 6 weeks of
fatigue, 10-lb weight loss, and listlessness. The vital signs include a HR of 70 beats per
minute and BP of 90/50 mm Hg. Examination reveals a thin girl with a flat affect, but no
other abnormalities.Further discussion with the patient reveals signs of depression. However,
she is interactive and denies any drug use or suicidal ideation. She has friends and makes As
and Bs in school. The results of screening laboratory work are normal. Appropriate initial
management should be which of the following?
A immediate psychiatry consultation due to the medicolegal risk posed by the patient
B immediate inpatient admission with 24-hour suicide watch
C urine collection for illicit substances against the patient’s will
D
initiation of a selective serotonin reup- take inhibitor (SSRI) and scheduling of outpatient
counseling with a therapist
ANSWER:D

624
A mother brings her 17-year-old daughter to your clinic for evaluation of 6 weeks of
fatigue, 10-lb weight loss, and listlessness. The vital signs include a HR of 70 beats per
minute and BP of 90/50 mm Hg. Examination reveals a thin girl with a flat affect, but no
other abnormalities.You are called 3 weeks later as the patient is brought to an outside
emergency department because of increasing hyperactivity, loqua- ciousness, and 3 days of
insomnia. The patient’s parents state that she purchased a large amount of goods. Urine
toxicology is negative. This patient most likely has which of the following?
A a drug reaction
B bipolar disorder
C schizophrenia
D reaction to sexual abuse
ANSWER:B
625
A 7-year-old girl presents with a 3-week history of fatigue, 5-lb weight loss, and listlessness.
Examination is significant for a thin girl who appears tired. Petechiae and ecchymoses are
present over her trunk and extremities. The complete blood count reveals a white blood cell
count of 85,000/mm3, hemoglobin of 7 g/dL, and platelets of 15,000/mm3. The differential
reveals 80% blasts and 20% lymphocytes. She is febrile to 103°F and has a BP of 90/50 mm
Hg. Appropriate empiric antibiotic therapy would begin with which of the following?
A intramuscular ceftriaxone
B intravenous clindamycin and cefuroxime
C intravenous ceftazidime
D intravenous amphotericin and voriconazole
ANSWER:C
626
A 7-year-old girl presents with a 3-week history of fatigue, 5-lb weight loss, and listlessness.
Examination is significant for a thin girl who appears tired. Petechiae and ecchymoses are
present over her trunk and extremities. The complete blood count reveals a white blood cell
count of 85,000/mm3, hemoglobin of 7 g/dL, and platelets of 15,000/mm3. The differential
reveals 80% blasts and 20% lymphocytes. She is febrile to 103°F and has a BP of 90/50 mm
Hg.Bone marrow examination confirms the diag- nosis of acute lymphocytic leukemia.
Which of the following would indicate a poor prognosis?
A patient’s age (7 years)
B patient’s initial white blood count (85,000/mm3)
C hyperdiploidy
D absence of the Philadelphia chromo- some (9:22)

ANSWER:B
627
A 7-year-old girl presents with a 3-week history of fatigue, 5-lb weight loss, and listlessness.
Examination is significant for a thin girl who appears tired. Petechiae and ecchymoses are
present over her trunk and extremities. The complete blood count reveals a white blood cell
count of 85,000/mm3, hemoglobin of 7 g/dL, and platelets of 15,000/mm3. The differential
reveals 80% blasts and 20% lymphocytes. She is febrile to 103°F and has a BP of 90/50 mm
Hg.Induction chemotherapy is begun. Which of the following patterns of metabolic
abnormal- ities might be expected to occur?
A hypokalemia, hypouricemia, hypophos- phatemia
B hypokalemia, hypouricemia, hyper- phosphatemia
C hyperkalemia, hyperuricemia, hyper- phosphatemia
D hyperkalemia, hypouricemia, hypophosphatemia
ANSWER:C
628
A first-time mother brings her 6-week-old full-term male infant in because of excessive
crying. She states he is feeding well and has had no fever but seems to cry “all the time” for
the last 3 weeks. Examination reveals an alert and vigorous infant who has gained weight
very well since the 2-week check-up and is currently cooing with no abnormal- ities noted
on examination.What should the initial approach to manage- ment be?
A a full sepsis evaluation
B a skeletal survey
C a radioisotope milk scan
D counseling, reassurance, and close follow-up
ANSWER:D
629
A first-time mother brings her 6-week-old full-term male infant in because of excessive
crying. She states he is feeding well and has had no fever but seems to cry “all the time” for
the last 3 weeks. Examination reveals an alert and vigorous infant who has gained weight
very well since the 2-week check-up and is currently cooing with no abnormal- ities noted
on examination.What would be the most appropriate manage- ment option at this time?
A suggesting that a single individual, prefer- ably the mother, act as the sole caregiver
B swaddling of the infant
C warm water feedings three times a day to decrease gastric upset
D vigorous shaking of the baby back and forth to simulate uterine movements
ANSWER:B

630
A first-time mother brings her 6-week-old full-term male infant in because of excessive
crying. She states he is feeding well and has had no fever but seems to cry “all the time” for
the last 3 weeks. Examination reveals an alert and vigorous infant who has gained weight
very well since the 2-week check-up and is currently cooing with no abnormal- ities noted
on examination.If this infant had presented with acute onset of irritability and crying and an
episode of bil- ious emesis, differential diagnosis should focus on which of the following
diagnoses?
A hair tourniquet around a digit
B milk protein intolerance
C occult fracture
D malrotation with volvulus
ANSWER:D
631
A 10-month-old female is brought to the emergency department for evaluation of streaks of
blood on the surface of the stools. The parents deny any history of travel, diarrhea, or
fever.What additional history would be most likely to point to a specific diagnosis in this
patient?
A family history of hemophilia
B dietary iron intake
C history of constipation
D presence of rotavirus in day care contacts
ANSWER:C
632
A 10-month-old female is brought to the emergency department for evaluation of streaks of
blood on the surface of the stools. The parents deny any history of travel, diarrhea, or
fever.Examination reveals a small fissure without erythema in the posterior area of the anus.
What is the most appropriate treatment at this time?
A prescribing milk of magnesia
B encouraging the use of a rectal dilator to stretch the internal anal sphincter
C prescribing a stool softener and titrating the dose to desired stool consistency
D prescribing an oral antibiotic to prevent secondary infection of the fissure
ANSWER:C
633
An 8-year-old girl presents with a 3-month history of intermittent joint swelling and stiffness
and is subsequently diagnosed with juvenile idiopathic arthritis (JIA).Which of the following
types of JIA is most commonly associated with development of severe arthritis?
A systemic onset JIA

B pauciarticular JIA without ocular involvement
C pauciarticular JIA with associated ocular involvement
D polyarticular rheumatoid factor positive JIA
ANSWER:D
634
An 8-year-old girl presents with a 3-month history of intermittent joint swelling and stiffness
and is subsequently diagnosed with juvenile idiopathic arthritis (JIA).Vision loss associated
with JIA is most com- monly due to which of the following?
A optic neuritis
B retinal artery thrombosis
C retinal detachment
D chronic iridocyclitis/uveitis
ANSWER:D
635
A 4-month-old male infant presents with 3 days of profuse watery nonbloody diarrhea and
nonbilious emesis. He was being fed milk, orange juice, and rice water. Vital signs are T
99°F, HR 170 beats per minute, and BP 66/40 mm Hg. His eyes and fontanelle are sunken
and he has poor skin turgor with a capillary refill time of 3–4 seconds; pulses are
normal.This description is most consistent with which of the following?
A hypovolemic shock, compensated
B hypovolemic shock, uncompensated
C cardiogenic shock, compensated
D septic shock, uncompensated
ANSWER:B
636
A 4-month-old male infant presents with 3 days of profuse watery nonbloody diarrhea and
nonbilious emesis. He was being fed milk, orange juice, and rice water. Vital signs are T
99°F, HR 170 beats per minute, and BP 66/40 mm Hg. His eyes and fontanelle are sunken
and he has poor skin turgor with a capillary refill time of 3–4 seconds; pulses are normal.His
weight just prior to intravenous fluid ther- apy is 6.3 kg. His fluid deficit is approximately
what measurement?
A 50 cc
B 250 cc
C 650 cc
D 850 cc
ANSWER:C

637
A 4-month-old male infant presents with 3 days of profuse watery nonbloody diarrhea and
nonbilious emesis. He was being fed milk, orange juice, and rice water. Vital signs are T
99°F, HR 170 beats per minute, and BP 66/40 mm Hg. His eyes and fontanelle are sunken
and he has poor skin turgor with a capillary refill time of 3–4 seconds; pulses are
normal.Assuming normal kidney function, the initial hydrating fluid bolus in this patient
should be which of the following?
A sodium 140 meq/L
B sodium 100 meq/L
C sodium 80 meq/L
D sodium 40 meq/L
ANSWER:A
638
An8-day-oldfull-termneonatepresentswith1 day of vesicular lesions of the skin and mouth.
She is afebrile and alert. There are multiple 3–5 mm vesi- cles on an erythematous base
present on her trunk and mouth. The pregnancy was uncomplicated and the parents have no
history of genital lesions.Which of the following is the best course of action at this time?
A discharge home and follow up in 72 hours
B culture the lesions and begin treatment with oral clindamycin (Cleocin)
C
check a complete bloodcountand urine analysis and, if normal, discharge patient after
intramuscular ceftriaxone (Rocephin)
D
perform bacterial cultures of blood, urine, cerebrospinal fluid, bacterial and viral cultures of
the lesions, and initiate broad-spectrum antibacterial therapy and acyclovir
ANSWER:D
639
An8-day-oldfull-termneonatepresentswith1 day of vesicular lesions of the skin and mouth.
She is afebrile and alert. There are multiple 3–5 mm vesi- cles on an erythematous base
present on her trunk and mouth. The pregnancy was uncomplicated and the parents have no
history of genital lesions.Risk factors for vertical transmission of HSV type 2 include which
of the following?
A maternal coinfection with HSV type 1
B physician rupture of membranes just prior to delivery
C scalp electrode usage
D coinfection with the human immunode- ficiency virus
ANSWER:C

640
A 15-day-old, full-term, large-for-gestational age infant is brought to your clinic for
evaluation of a chest mass. Examination reveals an afebrile vigor- ous baby with a firm,
nontender mass palpable over the middle third of the right clavicle.What is the most
appropriate course of action for this patient?
A reassurance to the family about the condition
B weight-bearing and comparison views of the clavicles
C skeletal survey to rule out abuse
D fibroblast assay for osteopetrosis
ANSWER:A
641
A 15-day-old, full-term, large-for-gestational age infant is brought to your clinic for
evaluation of a chest mass. Examination reveals an afebrile vigor- ous baby with a firm,
nontender mass palpable over the middle third of the right clavicle.Risk factors for this
condition include which of the following?
A congenital syphilis
B breech presentation
C maternal drug use
D female gender
ANSWER:B
642
You are called to evaluate the cause of hypotoniain a 1-day-old full-term female infant born
to a 28-year-old mother. The baby is alert and moves all extremities well but is hypotonic.
She has upward slanting palpebral fissures, speckled irides, a large tongue, short fifth digits,
and bilateral transverse palmar creases. Karyotype is 46,XX.What is the most likely reason
for this result?
A the number of chromosomes was miscounted
B she has a partial translocation involving chromosome 21
C there is no chromosomal abnormality
D interference of the test by maternal-fetal blood transfusion
ANSWER:B
643
You are called to evaluate the cause of hypotoniain a 1-day-old full-term female infant born
to a 28-year-old mother. The baby is alert and moves all extremities well but is hypotonic.
She has upward slanting palpebral fissures, speckled irides, a large tongue, short fifth digits,
and bilateral transverse palmar creases. Karyotype is 46,XX.What is the frequency of Down
syndrome?
A 1/12 live births
B 1/600 live births

C 1/4000 live births
D 1/8000 live births
ANSWER:B
644
You are called to evaluate the cause of hypotoniain a 1-day-old full-term female infant born
to a 28-year-old mother. The baby is alert and moves all extremities well but is hypotonic.
She has upward slanting palpebral fissures, speckled irides, a large tongue, short fifth digits,
and bilateral transverse palmar creases. Karyotype is 46,XX.Patients with Down syndrome
are at increased risk for which of the following?
A hyperthyroidism
B arthritis of the cervical spine
C streak gonads
D cardiac malformations
ANSWER:D
645
A 3-year-old boy with sickle cell disease (hemoglo- bin SS) presents with a 1-hour history of
right-sided weakness. Examination reveals righthemiparesis. A CT scan of the brain is
normal.Acute treatment for this patient’s condition is which of the following?
A observation
B anticoagulation with heparin
C exchange transfusion
D intravenous tissue plasminogen activator
ANSWER:C
646
A 3-year-old boy with sickle cell disease (hemoglo- bin SS) presents with a 1-hour history of
right-sided weakness. Examination reveals righthemiparesis. A CT scan of the brain is
normal.The patient recovers completely, but prior to discharge develops a fever to 102°F. He
appears otherwise well. Infection with which of the fol- lowing organisms would be of most
concern in this patient?
A Mycobacterium tuberculosis
B Clostridium botulinum
C group A beta-hemolytic streptococcus
D Streptococcus pneumoniae
ANSWER:D

647
A 3-year-old boy with sickle cell disease (hemoglo- bin SS) presents with a 1-hour history of
right-sided weakness. Examination reveals righthemiparesis. A CT scan of the brain is
normal.The patient’s father inquires about the possi- bility that his son’s children will have
sickle cell disease. You should you inform him which of the following about the risk?
A is negligible
B is approximately 25%
C is approximately 50%
D cannot be determined at this time
ANSWER:D
648
During a well-child examination, the mother of a previously healthy 5-year boy inquires
whether you think her child is unusually clumsy. The mother thinks her son is going to be
strong as he has “big calves,” but her family tells her he seems to trip a lot. Your examination
reveals a pleasant male with a slightly waddling gait and mildly enlarged calves.Which of the
following physical examination findings would suggest the diagnosis of Duchenne muscular
dystrophy?
A Rovsing sign
B opisthotonus
C Gower sign
D Chovstek sign
ANSWER:C
649
During a well-child examination, the mother of a previously healthy 5-year boy inquires
whether you think her child is unusually clumsy. The mother thinks her son is going to be
strong as he has “big calves,” but her family tells her he seems to trip a lot. Your examination
reveals a pleasant male with a slightly waddling gait and mildly enlarged
calves.Complications of Duchenne muscular dys- trophy most commonly include which of
the following?
A renal failure from myoglobinuria
B ophthalmoplegia
C seizures
D aspiration pneumonia
ANSWER:D

650
During a well-child examination, the mother of a previously healthy 5-year boy inquires
whether you think her child is unusually clumsy. The mother thinks her son is going to be
strong as he has “big calves,” but her family tells her he seems to trip a lot. Your examination
reveals a pleasant male with a slightly waddling gait and mildly enlarged calves.The mother
is devastated to learn the diagno- sis and that she carries the affected gene. A true statement
about Duchenne muscular dystro- phy inheritance is which of the following?
A 50% of her daughters will be affected with the disease
B about 25% of the cases are from new mutations
C women are never affected with the disease
D the risk of recurrence is minimal
ANSWER:B
651
An 8-year-old girl is seen in the emergency depart- ment with a complaint of 1-week
worsening short- ness of breath. She denies fever or cough. Vital signs include HR 140 beats
per minute and BP 78/40 mm Hg. She is alert with mild tachypnea and is most comfortable
leaning forward. Cardiac exam- ination reveals tachycardia and muffled heart tones. What
finding is the most suggestive of peri- cardial tamponade?
A pulsus paradoxus of greater than 10–20 mm Hg
B electrocardiogram with low voltages in all leads
C chest radiograph demonstrating an enlarged cardiothymic silhouette
D egophany over the left anterior chest
ANSWER:A
652
An 8-year-old girl is seen in the emergency depart- ment with a complaint of 1-week
worsening short- ness of breath. She denies fever or cough. Vital signs include HR 140 beats
per minute and BP 78/40 mm Hg. She is alert with mild tachypnea and is most comfortable
leaning forward. Cardiac exam- ination reveals tachycardia and muffled heart tones.
Echocardiography confirms a large pericardial effusion compromising left ventricle
function. After ensuring that the airway is stable, what should the next intervention be?
A intravenous solumedrol 30 mg/kg rapid infusion
B continuous intravenous milrinone infusion
C thoracotomy and placement of a peri- cardial window
D pericardiocentesis under ultrasound guidance
ANSWER:D

653
A previously healthy 7-day-old, full-term infant presents to your clinic with jaundice which
began on the first day of age. He is afebrile with normal vital signs and growth indices. He is
breastfeeding well and has five gray stools and six wet diapers per day. Examination reveals
a vigorous infant with marked scleral icterus and jaundice to the level of the umbilicus. The
liver is palpable 3 cm below the right costal margin. Your nurse was only able to obtain a
limited sample of blood.What are the most important tests to direct further evaluation at this
time?
A complete blood count and culture
B total and direct bilirubin
C blood type and Coomb test
D aspartate aminotransferase and alanine aminotransferase
ANSWER:B
654
A previously healthy 7-day-old, full-term infant presents to your clinic with jaundice which
began on the first day of age. He is afebrile with normal vital signs and growth indices. He is
breastfeeding well and has five gray stools and six wet diapers per day. Examination reveals
a vigorous infant with marked scleral icterus and jaundice to the level of the umbilicus. The
liver is palpable 3 cm below the right costal margin. Your nurse was only able to obtain a
limited sample of blood.The child is sent to the emergency department, and testing reveals
the total bilirubin to be 16 mg/dL withaconjugatedfractionof14 mg/dL. Which of the
following tests would be most likely to reveal the cause of this infant’s conjugated
hyperbilirubinemia?
A urine analysis and culture
B urine succinylacetone
C sweat chloride test
D abdominal ultrasound
ANSWER:D
655
A previously healthy 7-day-old, full-term infant presents to your clinic with jaundice which
began on the first day of age. He is afebrile with normal vital signs and growth indices. He is
breastfeeding well and has five gray stools and six wet diapers per day. Examination reveals
a vigorous infant with marked scleral icterus and jaundice to the level of the umbilicus. The
liver is palpable 3 cm below the right costal margin. Your nurse was only able to obtain a
limited sample of blood.The above test is ordered and is abnormal. A subsequent liver
biopsy to confirm the diagnosis reveals a paucity of biliary chan- nels and plugging with the
channel diameter of greater than 150 mm. What is the best treatment for this condition?

A phenobarbital to promote bile flow
B immediate liver transplant using a parental split liver graft
C hepatocyte infusion directly into the residual portal vein
D Kasai hepatoportoenterostomy as soon as possible after diagnosis
ANSWER:D
656
A 4-year-old boy is brought in for evaluation because of perianal irritation and itching and
parental con- cern that this might be a manifestation of sexual abuse. Examination reveals a
playful boy; inspection of the perianal region demonstrates several superfi- cial excoriations
and several thread-like worms.What is the best next step in management?
A antiparasitic treatment and reassurance about the benign nature of the condition
B stool for ova and parasite examination
C social work evaluation and child protec- tive services referral
D instructions to the family regarding improved hygiene
ANSWER:A
657
A 4-year-old boy is brought in for evaluation because of perianal irritation and itching and
parental con- cern that this might be a manifestation of sexual abuse. Examination reveals a
playful boy; inspection of the perianal region demonstrates several superfi- cial excoriations
and several thread-like worms .Which of the following should you treat this condition with?
A mebendazole
B metronidazole
C diethylcarbamazine
D praziquantel
ANSWER:A
658
A 2-year-old 13-kg female is brought to the emer- gency department by her parents after
they found her with an empty bottle of cherry-flavored aceta- minophen drops about 2
hours prior. You note that the bottle contained 1 oz at a concentration of 80 mg/0.8 cc. The
parents report they just pur- chased it earlier in the day for her 2-month-old brother. No
treatment was given prior to arrival at the emergency department. The patient’s vital signs
are normal, and her physical examination is unremarkable.What should the next step in this
patient’s management be?
A intubate to secure the airway
B do nothing until 4 hours have elapsed since the purported time of ingestion
C give activated charcoal with sorbitol at a dose of 1 g/kg orally or via nasogastric tube
D induce emesis with syrup of ipecac

ANSWER:C
659
A 2-year-old 13-kg female is brought to the emer- gency department by her parents after
they found her with an empty bottle of cherry-flavored aceta- minophen drops about 2
hours prior. You note that the bottle contained 1 oz at a concentration of 80 mg/0.8 cc. The
parents report they just pur- chased it earlier in the day for her 2-month-old brother. No
treatment was given prior to arrival at the emergency department. The patient’s vital signs
are normal, and her physical examination is unremarkable.You proceed with management.
The 4-hour acet- aminophen level is 200 mg/kg. What is the best next step?
A
continue observation in the emergency department for 6 hours and discharge if the patient
remains asymptomatic
B start the antidote, N-acetylcysteine
C continue activated charcoal every 4 hours in addition to the antidote,N-acetylcysteine
D begin preparations for liver transplant
ANSWER:B
660
A 2-year-old 13-kg female is brought to the emer- gency department by her parents after
they found her with an empty bottle of cherry-flavored aceta- minophen drops about 2
hours prior. You note that the bottle contained 1 oz at a concentration of 80 mg/0.8 cc. The
parents report they just pur- chased it earlier in the day for her 2-month-old brother. No
treatment was given prior to arrival at the emergency department. The patient’s vital signs
are normal, and her physical examination is unremarkable.The toxicity of acetaminophen
comes from which of the following?
A its renal metabolite
B acetaminophen directly
C its glucoronidate metabolite
D its cytochrome P-450 metabolite
ANSWER:D
661
A previously healthy 3-year-old male is brought in by his parents with a chief complaint of
increased sleepi- ness, which began during a visit to his grandmother’s house earlier that day.
The patient’s vital signs are HR of 50 beats per minute, temperature of 97°F, RR of 8 times
per minute, and a BP of 98/65 mm Hg. Examination reveals an obtunded male toddler who
is unresponsive to sternal rub and lacks a gag reflex. He also has miosis.The first step in
managing this patient is
A bedside whole blood glucose
B urinary catherization to obtain urine drug screen
C intravenous flumazenil

D securing an airway and insuring ade- quate breathing
ANSWER:D
662
A previously healthy 3-year-old male is brought in by his parents with a chief complaint of
increased sleepi- ness, which began during a visit to his grandmother’s house earlier that day.
The patient’s vital signs are HR of 50 beats per minute, temperature of 97°F, RR of 8 times
per minute, and a BP of 98/65 mm Hg. Examination reveals an obtunded male toddler who
is unresponsive to sternal rub and lacks a gag reflex. He also has miosis.The grandmother
reports that she has multiple medications in her house and is concerned that the patient’s
problems may be due to an inges- tion. Which one of her medicines would most likely cause
the symptoms reported?
A clonidine
B nifedipine
C meperidine
D labetalol
ANSWER:C
663
Among children, short stature and infertility are most commonly associated with which of
the following?
A Klinefelter syndrome
B Beckwith-Wiedemann syndrome
C Turner syndrome
D Marfan syndrome
ANSWER:C
664
Unilateral multicystic dysplastic kidney in an infant usually presents with which of the
following?
A an abdominal mass
B hematuria
C hypertension
D nephrotic syndrome
ANSWER:A
665 The Fanconi syndrome is characterized by which of the following?
A azotemia, edema, and hypertension
B glycosuria, aminoaciduria, and phosphaturia
C hematuria, glycosuria, and proteinuria
D hypoglycemia, glycosuria, hypoglycine- mia, and glycinuria
ANSWER:B

666
The presence of bilateral renal masses and a mid- line suprapubic mass in a newborn male
infant is most suggestive of which of the following?
A bilateral Wilms tumor
B congenital neuroblastoma
C congenital rubella infection
D congenital urethral or bladder neck obstruction
ANSWER:D
667
Wilms tumor (nephroblastoma) is the most frequent malignant tumor of the genitourinary
tract in childhood. The most common pre- senting sign of this neoplasm is which of the
following?
A abdominal mass
B abdominal pain
C edema
D hematuria
ANSWER:A
668
A 5-year-old male presents with a 48-hour his- tory of headache, and meningismus.
Evaluation of the CSF reveals clear fluid with normal protein and glucose content. The CSF
cell count reveals 300 WBC/hpf, 90% lymphocytes. Which of the following is the most likely
etiologic agent?
A Streptococcus pneumoniae
B Haemophilus influenzae
C Neisseria meningitidis
D enterovirus
ANSWER:D
669
Compared to other methods of dialysis, peri- toneal dialysis offers many advantages for chil-
dren and their families including increased autonomy and flexibility, with resultant enhanced
school attendance and peer interac- tions. Peritoneal dialysis is less costly than hemodialysis
and can be used to treat even small infants. The major complication of peri- toneal dialysis,
either continuous ambulatory (CAPD) or continuous cycling (CCPD), is
A a high incidence of disequilibrium syndrome
B a high incidence of peritonitis
C a need for severe dietary restriction
D frequent electrolyte problems
ANSWER:B

670
Which of the following represents the occurrence of significant bilateral hearing loss in
infants in well newborn nurseries in the United States?
A 1/100
B 1/1000
C 1/5000
D 1/10,000
ANSWER:B
671
The peak age of onset of childhood nephrotic syndrome associated with minimal-change
morphology is which of the following?
A under 6 months of age
B between 12 and 18 months of age
C between 2 and 5 years of age
D between 5 and 10 years of age
ANSWER:C
672
The finding of a low serum concentration of C3 component of complement in a child with
neph- rotic syndrome indicates which of the following?
A a high likelihood of spontaneous remission
B a high likelihood of good response to steroid therapy
C the presence of focal segmental sclerosis
D the presence of membranoproliferative glomerulonephritis
ANSWER:D
673
The most common roentgenographic abnormality in a child with asthma is which of the
following?
A bronchiectasis
B generalized hyperinflation
C lower lobe infiltrates
D pneumomediastinum
ANSWER:B
674 The pattern of inheritance of achondroplasia is which of the following?
A autosomal recessive
B autosomal dominant
C X-linked recessive
D X-linked dominant
ANSWER:B

675
A 16-year-old male presents for a sports physi- cal. Examination reveals hypermobile joints,
pes planus, and a high-arched palate. Which of the following would be most appropriate at
this time?
A chromosomal analysis
B cardiac ultrasound
C urine for organic acids
D cerebrospinal fluid for amino acids
ANSWER:B
676
A 2-month-old infant presents with irritability and congestive heart failure. An EC is inter-
preted as characteristic of myocardial infarc- tion. Which of the following is the most likely
explanation for these findings?
A viral myocarditis
B ventricular septal defect
C endocardial fibroelastosis
D anomalous origin of the left coronary artery
ANSWER:D
677
A 6-month-old infant is found to be lethargic and cyanotic. Blood obtained by venipuncture
appears chocolate brown and does not become bright red when shaken in the presence of air.
It is especially important to seek a history of exposure to which of the following?
A automobile exhaust
B fava beans
C paint fumes
D well water
ANSWER:D
678 Peak height velocity for girls most often occurs around which sexual maturity rating (SMR)
A zero
B one
C three
D four
ANSWER:C
679 Which of the following is characteristic of a night terror?
A most often occur during the first one- third of the night
B onset usually is during the elementary school years

C rarely persists until adolescence
D occurs during REM sleep
ANSWER:A
680 The primary lesion in acne is which of the following?
A hyperplasia of the sweat gland
B sterile inflammation of the sweat gland
C plugging of the sebaceous gland
D infection of the sebaceous gland
ANSWER:C
681
A 7-year-old child is brought to the office because of chronic nasal obstruction. Examination
reveals bilateral, clear, serous discharge from the nose, but is otherwise unremarkable. Which
of the following is the most likely diagnosis?
A a defect in the cribriform plate
B allergic rhinitis
C choanal atresia
D cystic fibrosis
ANSWER:B
682 Which of the following best describes atopic dermatitis in children?
A it tends to spare the face and arms
B it is frequently associated with uveitis
C it rarely begins during the first 2 years of life
D it is characterized by pruritus and lichenification
ANSWER:D
683
A 15-year-old boy is bitten on the hand by a snake, which he then kills and brings to the
emergency room. The snake is identified as a copperhead measuring 14 in. in length. The
most likely complication to be expected in this child would be which of the following?
A fever
B local tissue necrosis
C paralysis
D renal failure
ANSWER:B
684
The major concern regarding chronic otitis media with effusion is the development of which
of the following?
A meningitis
B mastoiditis

C permanent nerve deafness
D impaired speech and language development
ANSWER:D
685
Which of the following is the most common indi- cation for surgical repair of pectus
excavatum?
A thoracic scoliosis
B cardiac dysfunction
C pulmonary compromise
D cosmetic appearance
ANSWER:D
686 Most nasal polyps in children are due to which of the following?
A allergy or immunodeficiency
B allergy or infection
C allergy or cancer
D cystic fibrosis or allergy
ANSWER:D
687
Which of the following is the most common manifestation of alpha1-antitrypsin deficiency
in infancy?
A pulmonary cysts
B myocarditis
C hepatic cirrhosis
D pancreatic insufficiency
ANSWER:C
688
The earliest indicators of Cushing syndrome (glucocorticoid excess) in children are which of
the following?
A weight gain and growth arrest
B growth arrest and acne
C acne and hypertension
D hypertension and striae
ANSWER:A
689
A 12-year-old girl presents with headache, visual changes, and papilledema. A CT scan
reveals a mass lesion in the region of the anterior pituitary gland which of the following is
the most likely diagnosis?
A chromophobe adenoma
B craniopharyngioma

C ganglioneuroma
D medulloblastoma
ANSWER:B
690
An 8-year-old male with sickle cell disease presents with 2–3 days of runny nose and mild-
to-moderate anterior chest pain. On chest x-ray a new infiltrate is noted in the right middle
lobe. Which of the following diagnoses, is most likely?
A acute chest syndrome
B right middle lobe syndrome
C aspiration pneumonia
D foreign body
ANSWER:A
691 Which of the following is frequently seen in sickle cell patients with splenic sequestration?
A hyposplenism
B pneumonia
C polycythemia
D thrombocytopenia
ANSWER:D
692
Which of the following is the most common cause of deaths in infants under 12 months of
age each year in the United States?
A sudden infant death syndrome (SIDS)
B RSV bronchiolitis
C child abuse
D infantile leukemia
ANSWER:A
693 Of the following, which is a significant risk factor for SIDS?
A firm mattress
B breastfeeding
C early introduction of solid foods
D maternal smoking during pregnancy
ANSWER:D
694
Which of the following in an 8-year-old child is most likely to indicate an underlying
psycho- logic or behavioral problem?
A enuresis
B encopresis
C motion illness

D migraine headache
ANSWER:B
695
A 3-year-old male presents with random eye movements, ataxia, and developmental delay.
Of the following, which is most likely the diagnosis?
A acute cerebellar ataxia
B migraine variant
C neuroblastoma
D retinoblastoma
ANSWER:C
696
An 11-week-old male presents with a 1-week history of poor feeding and constipation. On
examination, you note poor head control and a weak cry. Though the infant’s face is
expressionless, he displays a paradoxical alertness and has normal features. The most likely
condition of this infant is which of the following?
A bacterial meningitis
B infantile botulism
C hypothyroidism
D Hirschsprung disease
ANSWER:B
697
Chronic upper airway obstruction from enlarged tonsils and adenoids in a child may cause
which of the following?
A convulsions
B cor pulmonale
C a pneumothorax
D thymic hyperplasia
ANSWER:B
698 Febrile seizures occur most frequently at what age?
A in the first month of life
B in the first 6 months of life
C between 6 months and 5 years of age
D between 5 and 10 years of age
ANSWER:C
699 Episodes of cerebellar ataxia may be seen in which of the following?
A cystinuria
B Gaucher disease
C Hartnup disease

D oxalosis
ANSWER:C
700
An 8-year-old child develops an intensely pru- ritic rash on the legs only. There are patches
of erythematous papules and vesicles and sev- eral streaks of erythematous vesiculation. The
child is afebrile and otherwise well. The most likely diagnosis is
A eczema
B Henoch-Schönlein purpura
C poison ivy dermatitis
D scabies
ANSWER:C
701
A 2-day-old male presents with upper abdom- inal distension and bilious vomiting. He
passed a normal meconium stool shortly after birth. Abdominal x-rays suggest duodenal
obstruc- tion. An upper gastrointestinal contrast study reveals a “bird’s beak” appearance of
the distal portion of the duodenum. Which of the fol- lowing is most likely?
A malrotation with midgut volvulus
B infantile hypertrophic pyloric stenosis
C duodenal atresia
D intussusception
ANSWER:A
702 The usual presentation of an annular pancreas in childhood is which of the following?
A hypoglycemia
B hyperglycemic acidosis
C jaundice
D vomiting
ANSWER:D
703 The classic radiologic finding in duodenal atresia is which of the following?
A a totally gasless abdomen
B free air below the diaphragm
C the double bubble sign
D the anchor sign
ANSWER:C
704 Infants typically double their birth weight by what age?
A 2 weeks
B 2 months
C 4 months

D 8 months
ANSWER:C
705
Which of the following is the normal, or aver- age, hemoglobin concentration in a 12-month-
old infant?
A 8 g/dL
B 10 g/dL
C 12 g/dL
D 15 g/dL
ANSWER:C
706 Breath-holding spells are best described by which of the following?
A are most common between 4 and 6 years of age
B are a common cause of sudden infant death
C are a manifestation of infantile colic
D may terminate in cyanosis and loss of consciousness
ANSWER:D
707
Which of the following is the most common presentation of Enterobius vermicularis
infection?
A appendicitis
B diarrhea
C intussusception
D perianal pruritus
ANSWER:D
708
A 10-day-old infant is evaluated for recurrent blisters and sores, mostly on the extremities at
areas of friction or trauma. The child has been afebrile and well except for irritability.
Examination is unremarkable except for blis- ters, varying in stage from fresh to ruptured
and crusted, mostly on the extremities, espe- cially the dorsal surfaces of the hands and feet.
Which of the following would be highest on your differential diagnosis?
A bullous impetigo
B congenital syphilis
C drug-induced toxic epidermal necrolysis
D epidermolysis bullosa
ANSWER:D
709 Which of the following best describes juvenile gastrointestinal polyps?
A they occur most commonly in the ileum
B they rarely present in the first 5 years of life

C they usually present as blood-streaked stools
D they often already have malignant ele- ments when first discovered
ANSWER:C
710
A 2-week-old male presents with irritability, decreased feeding, and fever. Cerebrospinal
fluid examination reveals cloudy fluid with 5500 WBC/mm3 and a low glucose. Gram stain
revealed no bacteria. Which of the following is the most likely causative agent?
A Treponema pallidum
B cytomegalovirus
C Mycobacterium tuberculosis
D group B streptococcus
ANSWER:D
711 In comparison to cow milk, human milk con- tains more of what substance?
A protein
B sodium
C casein
D calories
ANSWER:D
712
On routine examination, a 2-month-old African- American male infant is noted to have a
mod- erate size umbilical hernia. The contents of the hernia are easily reduced, and it is
noted that the abdominal wall defect easily admits one examining finger but not quite two.
The remain- der of the examination is normal and the infant is asymptomatic. What is the
most appropriate next step in management?
A order thyroid function tests
B refer the infant to a surgeon
C instruct the parent in how to tape the defect
D advise the parent that the defect
ANSWER:D
713 Which of the following best describes chylous ascites in infancy?
A Congenital nephrosis is the most com- mon cause.
B Hypoalbuminemia and lymphopenia are common.
C The condition usually is benign and transient.
D Hypergammaglobulinemia is common.
ANSWER:B

714
A 12-year-old child presents with severe abdominal pain, nausea and vomiting, abdom- inal
distension, and epigastric tenderness. Chest roentgenogram reveals a small pleural effusion.
The child is afebrile but blood count reveals a marked leukocytosis. Which of the following
is the most likely cause for this con- dition?
A blunt abdominal trauma
B systemic lupus erythematosis
C hemolytic uremic syndrome
D alcohol ingestion
ANSWER:A
715 Which of the following best describes idiopathic scoliosis?
A it is more common in boys than in girls
B it is diagnosed by x-ray
C it is generally associated with mental retardation
D it is most commonly noted during preadolescence or adolescence
ANSWER:D
716
A 10-year-old male presents with an acute episode of nausea, vomiting, and severe testic-
ular pain. On examination, scrotal edema and erythema with lack of cremasteric relex are
noted. Color-flow Doppler ultrasonography confirms the suspected diagnosis. What is the
initial treatment of choice?
A observation for 24 hours
B immediate surgical exploration
C chemotherapy
D intravenous antibiotics
ANSWER:B
717
You examine a school-age child because of itchy scalp and note minute white-gray struc-
tures firmly attached to the hair shafts. You recommend which of the following?
A a selenium-containing shampoo
B a 1% permethrin cream rinse
C oral tetracycline
D oral and topical tetracycline
ANSWER:B
718
A 2-year-old child drinks kerosene that had been left in a glass. After the first swallow, she
cries and drops the glass. She is most likely to develop which of the following?
A aplastic anemia
B chemical pneumonitis

C coma and/or convulsions
D hepatitis
ANSWER:B
719
Congenital hypothyroidism should be included in the differential diagnosis of a newborn
with which of the following?
A coma
B prolonged jaundice
C pulmonary edema
D renal failure
ANSWER:B
720
A 2-month-old infant has had inspiratory stridor since the first month of life, but has been
otherwise well. Physical examination is unremarkable except for moderate inspiratory stridor
and retrac- tions, which are worse when the infant is supine or agitated and better when he is
prone and quiet. What is the most likely cause of these findings?
A reactive airway disease
B laryngomalacia
C viral croup
D an aspirated foreign body
ANSWER:B
721
A significant number of children with recur- rent gross or microscopic hematuria, normal
renal ultrasound, and normal tests of renal function have which of the following?
A hyperkaliuria
B hypercalciuria
C hypercalcemia
D hyperphosphatemia
ANSWER:B
722
A 12-year-old girl complains of decreasing visual acuity and a slight feeling of discomfort in
both eyes. Examination by an ophthalmolo- gist reveals anterior uveitis. You suspect that the
child may have which of the following?
A leukemia
B toxoplasmosis
C toxocara infection
D juvenile idiopathic arthritis
ANSWER:D

723
A 5-year-old female presents with intoeing. Her mother states the child is very limber and
has no complaints of pain. On examination, with the child in prone position and knees flexed
90 degrees the child can easily place her lateral malleoli on the table. What is the most likely
condition described here?
A Perthes disease
B acute slipped femoral epiphysis
C metatarsus adductus
D femoral anteversion
ANSWER:D
724
For which of the following would laboratory testing for group A streptococcus (GAS) be
rec- ommended in the United States?
A an 18-month-old with pharyngitis
B a 5-year-old with pharyngitis and anterior stomatitis with discrete ulcerative lesions
C a 10-year-old with pharyngitis, conjunc- tivitis, and cough
D all household contacts of a child with documented poststreptococcal glomeru- lonephritis
ANSWER:D
725
The most consistent finding in lymphocytic thyroiditis (Hashimoto thyroiditis) is which of
the following?
A enlargement of the thyroid gland
B hyperthyroidism
C hypothyroidism
D eosinophilia
ANSWER:A
726
A 6-month-old male presents with diaper rash that is resistant to therapy. He has intermittent
diarrhea. He is breastfeeding, and rarely seems interested in solid foods. Mother is a
vegetarian. On examination, he is thin and listless. He has dry, plaque-like, sharply
demarcated lesions around his mouth and eyes. His hair is coarse and scanty. A deficiency of
which of the fol- lowing could explain this condition?
A iron
B zinc
C vitamin A
D vitamin B6
ANSWER:B

727
A 2-year-old child has had recurrent episodes of fever, cough, and pulmonary infiltrates on
chest roentgenograms. Diagnostic evaluation reveals a normal sweat test, barium swallow,
and serum immunoglobulins, but a severe microcytic, hypochromic anemia. The child’s diet
is normal for age. Which of the following would be highest on your list of differential
diagnoses?
A cystic fibrosis
B diffuse pulmonary hemangiomatosis
C extramedullary pulmonary erythropoiesis
D primary pulmonary hemosiderosis
ANSWER:D
728
A 13-year-old female athlete presents with anterior knee pain of 2–3 weeks’ duration. She
has no history of trauma. On examination, she has good flexibility of lower extremities.
There are no effussions. There is no tenderness over the tibial tubercle, or at the inferior pole
of the patella. She has a positive patella glide test. Of the following causes of anterior knee
pain, which is most likely in this patient?
A Osgood-Schlatter disease
B chondromalacia patellae
C Sinding-Larsen-Johansson disease
D osteochondritis dissecans
ANSWER:B
729
On a 6-month well-child visit, you note a healthy well-grown male with a normal physical
exam- ination except for a closed anterior fontanelle. Which of the following should you
consider?
A hypothyroidism
B trisomy 18
C hydrocephalus
D craniosynostosis
ANSWER:D
730
A previously well 9-year-old boy develops an intensely pruritic rash consisting of raised
hive- like lesions with pink edges and pale centers. Lesions are well circumscribed and range
in size from 2 to 6 cm in diameter. The history is unremarkable and physical examination is
unrevealing except for the rash. Careful diag- nostic workup probably will disclose which of
the following?
A a food allergy
B an occult malignancy

C a collagen vascular disease
D no specific cause for the rash
ANSWER:D
731 Most patients with XYY constitution are which of the following?
A short
B obese
C behaviorally normal
D impulsive and antisocial
ANSWER:C
732
A petechial rash is noted in a 10-hour-old newborn who is otherwise appearing normal and
vigorous. There is no hepatosplenomegaly and radii are present. There are no risk factors for
sepsis. A complete blood count reveals a normal WBC count with differential hemoglobin 17
g/dL; and platelet count 11,000/mm3 with large platelets present. Which of the following
would be the most appropriate next step?
A maternal platelet count
B abdominal ultrasound
C bone marrow aspiration
D chest x-ray
ANSWER:A
733
A 2-month-old male presents with a cough that has persisted for over 2 weeks. Mother has
noted he coughs in spells and at the end of these spells, he vomits. She feels he has lost
weight. On examination, you note a thin male infant who otherwise appears normal.
Complete blood count reveals 30,000 white blood cells with 95% lymphocytes. Which of the
following is the most likely diagnosis?
A respiratory syncitial virus bronchiolitis
B pneumococcal pneumonia
C pertussis
D laryngomalacia
ANSWER:C
734 Pseudohypoparathyroidism is associated with which of the following?
A short stature
B renal failure
C long, spindly fingers
D generalized increased mineralization of bone
ANSWER:A

735 The most consistent abnormality in von Willebrand disease is which of the following?
A decreased platelet count
B prolonged bleeding time
C prolonged prothrombin time
D prolonged partial thromboplastin time
ANSWER:B
736 The most common cause of congenital hypothyroidism is which of the following?
A dysgenesis of the thyroid gland
B a defect in thyroid synthesis
C Hashimoto’s thyroiditis
D maternal ingestion of iodides
ANSWER:A
737
A 6-year-old Hispanic male presents with a first-time generalized seizure lasting less than 5
minutes. There is no history of trauma or ill- ness. The family recently immigrated to the
United States from Mexico. The child is afebrile, and his examination is completely normal.
Of the following, which would be the most likely cause for the seizure?
A herpes encephalitis
B cysticercosis
C lead poisoning
D vitamin B12 deficiency
ANSWER:B
738
A 17-year-old boy with cystic fibrosis complains of a 5–10 lb weight loss over the past
month, unaccompanied by changes in his pulmonary or gastrointestinal symptoms. You
suspect that he has developed which of the following?
A a bronchogenic carcinoma
B an eating disorder
C biliary cirrhosis
D diabetes mellitus
ANSWER:D
739 The diagnosis of cystic fibrosis is usually con- firmed by the finding which of the following?
A elevated sweat chloride
B decreased sweat chloride
C elevated serum chloride
D decreased serum chloride

ANSWER:A
740
A 2-year-old male presents with upper respi- ratory symptoms for 3 days and 1 day of fever,
irritability, and right ear pain. Examination reveals a temperature of 39°C, clear rhinorrhea, a
bulging immobile right tympanic membrane with absent light reflex. What is the most
appropriate initial therapy?
A antihistamines and ibuprofen
B antihistamines and acetaminophen
C penicillin
D amoxicillin
ANSWER:D
741 Which of the following best describes an inguinal hernia in a 2-month-old girl?
A it may contain an ovary
B it is usually a direct hernia
C it does not require surgical repair
D it is a sign of pseudohermaphroditism
ANSWER:A
742
Which of the following would be expected in a 6-month-old child with a large ventricular
septal defect?
A cyanosis
B an enlarged heart
C a continuous cardiac murmur
D decreased pulmonary vasculature on roentgenogram
ANSWER:B
743
Which of the following is most common presentation of hypoparathyroidism beyond the
neonatal period?
A syncope secondary to prolonged QT intervals
B tingling of extremities
C seizure
D bronchospasm
ANSWER:C
744 What is the most common cause of school absenteeism among adolescent females?
A depression
B asthma
C headache
D dysmenorrhea

ANSWER:D
745
An irritable 12-month-old male has a 1-week history of high fevers and macular truncal
rash. Examination reveals bulbar conjuctivitis, bright red cracked lips, and cervical
adenopathy. What is the most appropriate next step?
A initiate airborne barrier precautions
B intravenous antibiotics
C intravenous antibiotics for the infant and oral antibiotics for all household members
D intravenous gammaglobulin
ANSWER:D
746
An 8-month-old female presents with failure to thrive, constipation, fevers, and polydipsia.
On evaluation, you find hypokalemia and hyper- chloremic metabolic acidosis and suspect
Fanconi syndrome. Which of the following would be the most likely inherited cause?
A cystinosis
B cystic fibrosis
C glycogen storage disease
D Tay-Sachs disease
ANSWER:A
747
A 13-year-old male presents with a 3 cm annu- lar scaling lesion on his right posterior trunk
unresponsive to 2 weeks of a topical antifungal. On follow-up visit he presents with dozens
of elliptical scaling lesions on his trunk. He has no palmoplantar or mucous membrane
lesions and no adenopathy. What is the most likely diagnosis?
A pityriasis rosea
B secondary syphilis
C lichen planus
D tinea versicolor
ANSWER:A
748
An 18-month-old female presents for a well- child visit. On examination, there is an asym-
metric red reflex, with the left appearing normal and the right appearing cream colored. Of
the following, which is the most likely diagnosis?
A rhabdomyosarcoma
B retinoblastoma
C neuroblastoma
D leukemia
ANSWER:B

749
Which of the following represents the ethnic group with the highest neonatal and infant
mortality rates in the United States?
A American Indian
B Asian and Pacific Island
C African-American
D Hispanic
ANSWER:C
750 Which of the following best describes patients with constitutional growth delay?
A they will begin puberty at the same time as their peers
B they have a bone age equal to their chronological age
C they are usually obese
D they will achieve normal adult height
ANSWER:D
751
A 4-year-old male presents with a 1-day his- tory of abdominal pain and vomiting. He is
afebrile and has no diarrhea. He complains of knee pain bilaterally, and there is some ten-
derness of the knee joints but no effusions. Within 24 hours he develops a rash on his legs
and buttocks which is petechial and purpuric, and his platelet count is normal. What is the
most likely diagnosis?
A hemolytic uremic syndrome
B Henoch-Schönlein purpura
C acute glomerulonephritis
D Kawasaki disease
ANSWER:B
752
A 4-year-old female presents with a limp. The week prior she had an upper respiratory infec-
tion. There is no history of trauma. She has no his- tory of fever. Examination reveals
limitation of motion of the right hip joint, especially with inter- nal rotation. X-ray reveals
some swelling in soft tissues surrounding the hip joint. A complete blood count and
sedimentation rate are normal. What would be the next most appropriate step?
A aspirate the hip joint
B begin nonsteroidal anti-inflammatory agents
C CT scan of the hip
D MRI of the hip
ANSWER:B

753
A newborn is noted to have a large head and short limbs. On further examination, short
broad fingers, a small face, and low-normal length are noted. The trunk appears long and
narrow. What should be your first step to con- firm the diagnosis?
A order an ophthalmologic examination
B obtain skeletal radiographs
C order chromosome analysis
D examine the parents
ANSWER:B
754 Infants fed exclusively goat milk are susceptible to which of the following?
A vitamin A deficiency
B vitamin B6 deficiency
C vitamin E deficiency
D folate deficiency
ANSWER:D
755
A 5-month-old female presents in the winter with a 2-day history of vomiting, diarrhea, and
fever. The diarrhea is without blood or mucus. On examination, she is moderately
dehydrated and has a serum sodium of 152 meq/L. What is the most likely etiologic agent?
A Giardia lamblia
B Salmonella
C influenza
D rotavirus
ANSWER:D
756
Among adolescents of 15–19 years of age, motor vehicle accidents cause the greatest
number of deaths each year. Which of the following is the second leading cause of death in
this age group?
A homicide and legal intervention
B suicide
C heart disease
D cancer
ANSWER:A
757
The most common congenital infection is infection with cytomegalovirus, affecting
3/1000–4/1000 livebirths. Of those babies infected, approximately what percent are normal
at birth and develop normally?
A 5%
B 15%

C 25%
D 90%
ANSWER:D
758
A 12-year-old child is admitted because of the sudden onset of coma. The child had been
well until about 6 hours prior to admission, when he began to com- plain of a headache. The
headache became more severe and the child lapsed into coma. Physical exam- ination reveals
a temperature of 38.2°C. The child is flaccid and comatose. The remainder of the physical
examination is unremarkable. A lumbar puncture reveals grossly bloody spinal fluid. After
centrifuga- tion, the fluid appears xanthochromic. There are 3000 RBCs and 7 WBCs/mm3.
The protein concentration is 400 mg/dL and the glucose is 62 mg/dL.Which of the following
is the most likely etiol- ogy of the coma?
A intraventricular hemorrhage
B subarachnoid hemorrhage
C aqueductal stenosis
D viral encephalitis
ANSWER:B
759
A 12-year-old child is admitted because of the sudden onset of coma. The child had been
well until about 6 hours prior to admission, when he began to com- plain of a headache. The
headache became more severe and the child lapsed into coma. Physical exam- ination reveals
a temperature of 38.2°C. The child is flaccid and comatose. The remainder of the physical
examination is unremarkable. A lumbar puncture reveals grossly bloody spinal fluid. After
centrifuga- tion, the fluid appears xanthochromic. There are 3000 RBCs and 7 WBCs/mm3.
The protein concentration is 400 mg/dL and the glucose is 62 mg/dL Which of the following
underlying structural abnormalities would most likely have led to the above event or
condition?
A absence of the corpus callosum
B porencephalic cyst
C cerebral arteriovenous malformation
D cerebral aneurysm
ANSWER:C

760
A 12-year-old child is admitted because of the sudden onset of coma. The child had been
well until about 6 hours prior to admission, when he began to com- plain of a headache. The
headache became more severe and the child lapsed into coma. Physical exam- ination reveals
a temperature of 38.2°C. The child is flaccid and comatose. The remainder of the physical
examination is unremarkable. A lumbar puncture reveals grossly bloody spinal fluid. After
centrifuga- tion, the fluid appears xanthochromic. There are 3000 RBCs and 7 WBCs/mm3.
The protein concentration is 400 mg/dL and the glucose is 62 mg/dL.What would the most
appropriate next step at this time would be to
A obtain a CT of the head
B repeat the lumbar puncture
C administer fresh frozen plasma
D perform an exchange transfusion
ANSWER:A
761
A 21-day-old male infant is admitted because of vomiting for 12 days. Birth weight was
2925 g. The child had been normal at birth and did well for the first 9 days of life. He was
initially begun on breastfeeding only, but on the eighth day of life, supplemental feeding
with a commercially pre- pared cow milk formula was added. Vomiting began on the 10th
day of life and persisted despite discon- tinuation of the prepared formula. On the 21st day
of life, the child was hospitalized. Diarrhea had never been present. Several days prior to
admission, the stools had become hard and infrequent. On admission, the anterior fontanel is
sunken, the mucous membranes are dry, and skin turgor is poor. The diaper is dry and the
mother cannot recall when the child last urinated. The child appears poorly nourished. Wt
2850 g, HR 152 beats per minute, RR 12 times per minute, T 37.5°C. Pulses and color are
good. No abnormalities are palpated on examination.Which of the following is the best
diagnostic test for this condition?
A an unprepped barium enema
B a sweat test
C an abdominal ultrasound
D chromosomal analysis
ANSWER:C

762
A 21-day-old male infant is admitted because of vomiting for 12 days. Birth weight was
2925 g. The child had been normal at birth and did well for the first 9 days of life. He was
initially begun on breastfeeding only, but on the eighth day of life, supplemental feeding
with a commercially pre- pared cow milk formula was added. Vomiting began on the 10th
day of life and persisted despite discon- tinuation of the prepared formula. On the 21st day
of life, the child was hospitalized. Diarrhea had never been present. Several days prior to
admission, the stools had become hard and infrequent. On admission, the anterior fontanel is
sunken, the mucous membranes are dry, and skin turgor is poor. The diaper is dry and the
mother cannot recall when the child last urinated. The child appears poorly nourished. Wt
2850 g, HR 152 beats per minute, RR 12 times per minute, T 37.5°C. Pulses and color are
good. No abnormalities are palpated on examination.One would expect the initial laboratory
data to reveal which of the following?
A mixed metabolic and respiratory acidosis
B mixed metabolic and respiratory alkalosis
C normal acid-base status
D primary metabolic alkalosis
ANSWER:D
763
A 21-day-old male infant is admitted because of vomiting for 12 days. Birth weight was
2925 g. The child had been normal at birth and did well for the first 9 days of life. He was
initially begun on breastfeeding only, but on the eighth day of life, supplemental feeding
with a commercially pre- pared cow milk formula was added. Vomiting began on the 10th
day of life and persisted despite discon- tinuation of the prepared formula. On the 21st day
of life, the child was hospitalized. Diarrhea had never been present. Several days prior to
admission, the stools had become hard and infrequent. On admission, the anterior fontanel is
sunken, the mucous membranes are dry, and skin turgor is poor. The diaper is dry and the
mother cannot recall when the child last urinated. The child appears poorly nourished. Wt
2850 g, HR 152 beats per minute, RR 12 times per minute, T 37.5°C. Pulses and color are
good. No abnormalities are palpated on examination.Which of the following intravenous
solutions would be most appropriate as the initial hydrating fluid?
A 5% dextrose in water
B normal saline
C eq/L of Na, 120 meq/L of Cl, 20 meq/L of bicarbonate
D 5% glucose, 40 meq/L of Na, 40 meq/L of K, 80 meq/L of Cl
ANSWER:B

764
A 15-year-old male presents with a 2-day history of fever, chills, and cough. He complains
of aching muscles. Today he noticed his urine was red. Examination revealed a tired-
appearing adolescent with fever, pharyngitis, nasal congestion, and ten- der calf
muscles.Urine analysis reveals a positive test for hemo- globin with no red blood cells seen
on micro- scopic examination. Which of the following is most likely to reveal the source of
the red urine?
A detailed dietary history
B renal ultrasound
C intravenous pyelogram
D serum creatine kinase
ANSWER:D
765
A 15-year-old male presents with a 2-day history of fever, chills, and cough. He complains
of aching muscles. Today he noticed his urine was red. Examination revealed a tired-
appearing adolescent with fever, pharyngitis, nasal congestion, and ten- der calf muscles.You
obtain the above study and it is markedly abnormal. What is the most important next step?
A perform a lumbar puncture
B initiate intravenous antibiotics
C obtain a chest x-ray
D order serum electrolytes, BUN, and creatinine
ANSWER:D
766
A 15-year-old male presents with a 2-day history of fever, chills, and cough. He complains
of aching muscles. Today he noticed his urine was red. Examination revealed a tired-
appearing adolescent with fever, pharyngitis, nasal congestion, and ten- der calf muscles.Of
the following infectious agents, which is most likely to cause this condition?
A influenza
B group A streptococcus
C group B streptococcus
D hepatitis A
ANSWER:A

767
A 2-year-old child is hospitalized because of fever, abdominal pain, and hepatomegaly. The
child lives in a poor, crowded home on a small farm. No one else at home is ill. Physical
examination reveals a thin child with marked hepatomegaly. The spleen is not palpable and
all lymph nodes are within normal limits. The remainder of the examination is within normal
limits. Chest x-ray reveals bilateral scattered densities. White blood cell count is 14,000/mm3
with 36% eosinophils.Which of the following would be most likely to establish the diagnosis
in this patient?
A bone marrow aspiration
B stool examination
C liver biopsy
D serum enzyme immunoassay
ANSWER:D
768
A 2-year-old child is hospitalized because of fever, abdominal pain, and hepatomegaly. The
child lives in a poor, crowded home on a small farm. No one else at home is ill. Physical
examination reveals a thin child with marked hepatomegaly. The spleen is not palpable and
all lymph nodes are within normal limits. The remainder of the examination is within normal
limits. Chest x-ray reveals bilateral scattered densities. White blood cell count is 14,000/mm3
with 36% eosinophils.How did this child most likely acquire the disease?
A eating poorly cooked pork
B eating dirt
C kissing a dog
D contact with a sick bird
ANSWER:B
769
A 2-year-old child is hospitalized because of fever, abdominal pain, and hepatomegaly. The
child lives in a poor, crowded home on a small farm. No one else at home is ill. Physical
examination reveals a thin child with marked hepatomegaly. The spleen is not palpable and
all lymph nodes are within normal limits. The remainder of the examination is within normal
limits. Chest x-ray reveals bilateral scattered densities. White blood cell count is 14,000/mm3
with 36% eosinophils.This condition is which of the following?
A more prevalent in the United States than in developing countries
B more common in rural areas
C nearly always symptomatic
D most frequently present in school-aged children
ANSWER:A

770
A 6-month-old girl is admitted because of failure to thrive and persistent pneumonia. The
child was well at birth but severe, recurrent diarrhea began at about 6 weeks of age. At 8
weeks the child developed pneu- monia which responded only poorly to antibiotic ther- apy
and since hospitalization, she continues to require oxygen by nasal cannula. Physical
examination reveals a small, poorly nourished child with moderate respira- tory distress.
There are bilateral rales, oral thrush, and a monilial-appearing rash in the diaper area. The
remainder of the examination is within normal limits. The total white blood cell count is
5200/mm3 with 87% polymorphonuclear cells, 12% lymphocytes, and 1% eosinophils;
hemoglobin and platelets are normal. Serum immunoglobulin levels are: IgA not detectable;
IgG 280 mg/dL; IgM less than 5 mg/dL.Which of the following diagnoses best explains the
clinical picture?
A chronic granulomatous disease
B Wiskott-Aldrich syndrome
C hereditary agammaglobulinemia
D severe combined immunodeficiency
ANSWER:D
771
A 6-month-old girl is admitted because of failure to thrive and persistent pneumonia. The
child was well at birth but severe, recurrent diarrhea began at about 6 weeks of age. At 8
weeks the child developed pneu- monia which responded only poorly to antibiotic ther- apy
and since hospitalization, she continues to require oxygen by nasal cannula. Physical
examination reveals a small, poorly nourished child with moderate respira- tory distress.
There are bilateral rales, oral thrush, and a monilial-appearing rash in the diaper area. The
remainder of the examination is within normal limits. The total white blood cell count is
5200/mm3 with 87% polymorphonuclear cells, 12% lymphocytes, and 1% eosinophils;
hemoglobin and platelets are normal. Serum immunoglobulin levels are: IgA not detectable;
IgG 280 mg/dL; IgM less than 5 mg/dL.On further evaluation one would also expect to find
which of the following?
A increased numbers of plasma cells on bone marrow examination
B enlarged superficial lymph nodes
C absent thymic shadow on chest roentgenogram
D positive intradermal reaction to Candida antigen
ANSWER:C

772
A 6-month-old girl is admitted because of failure to thrive and persistent pneumonia. The
child was well at birth but severe, recurrent diarrhea began at about 6 weeks of age. At 8
weeks the child developed pneu- monia which responded only poorly to antibiotic ther- apy
and since hospitalization, she continues to require oxygen by nasal cannula. Physical
examination reveals a small, poorly nourished child with moderate respira- tory distress.
There are bilateral rales, oral thrush, and a monilial-appearing rash in the diaper area. The
remainder of the examination is within normal limits. The total white blood cell count is
5200/mm3 with 87% polymorphonuclear cells, 12% lymphocytes, and 1% eosinophils;
hemoglobin and platelets are normal. Serum immunoglobulin levels are: IgA not detectable;
IgG 280 mg/dL; IgM less than 5 mg/dL.Chest roentgenogram reveals bilateral patchy con-
solidations and diffuse granular densities with an almost ground-glass appearance to the
lungs. Which of the following is the most likely cause of these x-ray findings?
A pneumococcal pneumonia
B Pneumocystis jiroveci pneumonia
C pulmonary hemorrhage
D pulmonary lymphangiectasia
ANSWER:B
773
A 4-month-old boy had been well until 4 weeks prior to admission, when vomiting and poor
appetite were noted. Psychomotor development had been normal. The child was being fed
whole cow milk and strained foods. Stools were normal. On the morning of admission, the
child had a gen- eralized convulsion and was brought to the emer- gency room where the
seizure was controlled with intravenous medication. A second seizure occurred about 1 hour
later and again responded to intra- venous medication. Physical examination revealed a pale,
listless infant, poorly nourished, but in no acute distress. The height was at the 25th
percentile, the weight at the 3rd percentile, and the head cir- cumference over the 97th
percentile on a standard growth curve; T 38°C, RR 16 breaths per minute, HR 110 beats per
minute, BP 76/50 mm Hg. The ante- rior fontanel is full, but not bulging. There are no focal
neurologic signs. The remainder of the exami- nation is within normal limits.Which of the
following diagnoses is most likely?
A tuberculous meningitis
B mastoiditis
C subdural hematoma
D congenital toxoplasmosis
ANSWER:C

774
A 4-month-old boy had been well until 4 weeks prior to admission, when vomiting and poor
appetite were noted. Psychomotor development had been normal. The child was being fed
whole cow milk and strained foods. Stools were normal. On the morning of admission, the
child had a gen- eralized convulsion and was brought to the emer- gency room where the
seizure was controlled with intravenous medication. A second seizure occurred about 1 hour
later and again responded to intra- venous medication. Physical examination revealed a pale,
listless infant, poorly nourished, but in no acute distress. The height was at the 25th
percentile, the weight at the 3rd percentile, and the head cir- cumference over the 97th
percentile on a standard growth curve; T 38°C, RR 16 breaths per minute, HR 110 beats per
minute, BP 76/50 mm Hg. The ante- rior fontanel is full, but not bulging. There are no focal
neurologic signs. The remainder of the exami- nation is within normal limits.A funduscopic
examination performed after one pupil is dilated with atropine reveals dif- fuse retinal
hemorrhages. The most likely diag- nosis now is which of the following?
A tuberculous meningitis
B mastoiditis
C subdural hematoma
D congenital toxoplasmosis
ANSWER:C
775
A 4-month-old boy had been well until 4 weeks prior to admission, when vomiting and poor
appetite were noted. Psychomotor development had been normal. The child was being fed
whole cow milk and strained foods. Stools were normal. On the morning of admission, the
child had a gen- eralized convulsion and was brought to the emer- gency room where the
seizure was controlled with intravenous medication. A second seizure occurred about 1 hour
later and again responded to intra- venous medication. Physical examination revealed a pale,
listless infant, poorly nourished, but in no acute distress. The height was at the 25th
percentile, the weight at the 3rd percentile, and the head cir- cumference over the 97th
percentile on a standard growth curve; T 38°C, RR 16 breaths per minute, HR 110 beats per
minute, BP 76/50 mm Hg. The ante- rior fontanel is full, but not bulging. There are no focal
neurologic signs. The remainder of the exami- nation is within normal limits.Of the
following, what is the next most impor- tant step in diagnosis and management?
A lumbar puncture
B electroencephalography
C CT scan of the head

D bone marrow examination
ANSWER:C
776 A "nonself" substance that can provoke an immune response is called a[No] __________.
A colony-stimulating factor
B antibody
C interferon
D antigen
ANSWER:D
777 A child is consider to have neurologic abnormality if Babinski reflex can be elicited from a
A 1 year-old
B 2 year-old
C 3 year-old
D New born
ANSWER:D
778 A neurologic or developmental abnormality is suspected if primitive reflexes are:
A Absent at appropriate age, present longer than normal and asymmetric
B Absent at appropriate age, normally present and symmetric
C Present at appropriate age, present longer than normal and asymmetric
D Present at appropriate age, normally present and symmetric
ANSWER:A
779
A patient had a complete blood count (CBC) with differential test ordered. You asses the
white blood cell count results and know that this laboratory test will also assess the count of
what other types of WBCs. Which are not the type of white blood cell.
A Neutrophils
B Lymphocytes
C Eosinophils
D Erythrocytes
ANSWER:D
780 Active artificially adaptive immunity is a result of __________.
A vaccination
B antibodies passed on from mother to fetus through the placenta
C injection of an immune serum
D contact with a pathogen
ANSWER:A

781 All the following are important electrolytes in the body except:
A potassium ions
B sodium ions
C carbon ions
D chloride ions
ANSWER:C
782
Among the common dermatologic conditions below which one resulted from the presence of
pigmented cells in the deep layers of the skin?
A Erythema toxicum
B salmon patch
C Cutis marmorata
D Mongolian spots
ANSWER:D
783 calculate daily the maintenance for a normal child 27kg?
A 1640ml/d
B 2140ml/d
C 2700ml/d
D 1140ml/d
ANSWER:A
784 Chose the false statement about digestive track phenomena
A Jaundice is common in neonates because the inability of an immature liver.
B An elevated direct bilirubin is not normal
C Blood loss from the gastrointestinal tract is never normal
D the recommendation to begin solids food at 3month of age.
ANSWER:D
785
During auscultation in abdominal examination, if the bowel sound is absent, what are the
most common diseases you think of?
A Gastroenteritis or a bowel obstruction
B Gastritis or pancreatitis
C Hepatitis or Pneumonia.
D Peritonitis or a paralytic ileus.
ANSWER:D
786 During infancy, where the hematopoiesis take place?
A yolk sac
B virtually all bone marrow

C thymus
D liver
ANSWER:B
787 Embryonic Gut is divided in to….
A 3 parts: frontgut, middlegut and backgut
B 3parts: Foregut, Midgut, Hindgut
C 3parts foregut ,uppergut, lowergut
D 2parts: uppergut, lowergut
ANSWER:B
788 how many kinds of blood corpuscles (elements)?
A 5
B 4
C 2
D 3
ANSWER:D
789
Humoral immunity is a type of adaptive immunity that results in the circulation of which of
the following throughout the blood?
A Antigen
B antibody
C Macrophage
D Natural killer cell
ANSWER:B
790
If the tympanic membrane fails to move by the (-) or (+) pressure, then the child is likely to
have:
A A middle ear destruction from otitis media
B A middle ear perforation from otitis media
C A middle ear effusion from otitis media
D A tympanic membrane perforation from otitis media
ANSWER:C
791 In normal development of the teeth, Permanent Central incisor usually erupt at age:
A 4-5 year
B 7-8 year
C 3-4 month
D 7-8 month
ANSWER:B

792 In the process of osmosis:
A
water moves from a region of high solute concentration to a region of low solute
concentration
B
water moves from a region of low solute concentration to a region of high solute
concentration
C sodium ions move through a semipermeable membrane
D chloride ions follow the movement of sodium ions to a region of low concentration
ANSWER:B
793
In what week of gestation that quickening usually occurs and heartbeat can be heard by a
stethoscope?
A Week 20
B Week 22
C Week 24
D Week 28
ANSWER:A
794 Microcytic hypochromic anemia mean.
A MCV higher than normal range and MCH lower than normal range
B MCV higher than normal range and MCH higher than normal range
C MCV lower than normal range and MCH higher than normal range
D MCV lower than normal range and MCH lower than normal range
ANSWER:D
795 Most of the volume of normal human blood is composed of:
A plasma
B red cells
C platelets
D hemoglobin
ANSWER:A
796 Normal Digestive Tract Phenomena, normally regurgitation is…….. Which is not true?
A
the result of gastroesophageal reflux, occurs commonly in the 1st year of life. (in healthy
infant)
B Volumes 15-30ml
C the infant look sick a reflux
D occur from <1 to several times per day.
ANSWER:C
797 Platelets are formed from what type of cell?

A Melanocytes
B Macrophages
C Astrocytes
D Megakaryocytes
ANSWER:D
798 Platelets are formed from what type of cell?
A Melanocytes
B Macrophages
C Astrocytes
D Megakaryocytes
ANSWER:D
799 Process of formation of blood corpuscles(elements) is called?
A Hemolysis
B Hematopoiesis
C Hemozoin
D Hemolytic
ANSWER:B
800 Retinopathy should be suspected when ophthalmoscopic examination shows:
A Red retinal reflex
B Cristal clear reflex.
C Black retinal reflexes.
D White retinal reflexes.
ANSWER:D
801
The hormone erythropoietin stimulates red blood cell production in the red bone marrow.
Where in the body is erythropoietin produced?
A Kidney
B Spleen
C Liver
D Thyroid
ANSWER:A
802 The intracellular fluid compartment refers to all the water found in:
A all cells of the body
B areas within blood vessel and heart
C the bones of the body
D areas outside the body cells

ANSWER:A
803 the recommendation to begin solids food for baby is at
A 3month of age
B 4month of age
C 6month of age
D 5month of age
ANSWER:C
804 The two types of immunity in humans are:
A Intrinsic and extrinsic.
B Innate and the adaptive immunity.
C Overt and covert.
D Internal and external.
ANSWER:B
805 The two types of lymphocytes are:
A B- lymphocytes and the T- lymphocytes.
B Platelets and erythrocytes.
C T-cells and erythrocytes.
D Platelets and the T-cells.
ANSWER:A
806
There is normally a delicate equilibrium between the intravascular fluid and the interstitial
fluid (balance between hydrostatic and oncotic forces). Which is the false statement?
A There is no movement of fluid at all because of this equilibrium
B
The intravascular fluid has a higher concentration of albumin(oncotic) =&gt; fluid from
intersticial =&gt; intravacular
C
The hydrostatic pressure of the intravascular space, which is due to the pumping action of
the heart, drives fluid out of the intravascular space
D
Overall, there is usually a net movement of fluid out of the intravascular space to the
intracellular space, but this fluid is returned to the circulation via the lymphatics
ANSWER:A
807
This type of disease results from the inability of the immune system to distinguish self- from
nonself-antigens.
A immunodeficiency
B SCID
C allergy
D autoimmune disease

ANSWER:D
808 Total body water is divided between how many main compartments?
A 3
B 4
C 5
D 2
ANSWER:D
809 What are the five components of Apgar score?
A Pulse rate, Respiratory effort, Crying tone, Reflex irritability and color.
B Heart rate, limbs effort, Muscle tone, Reflex irritability and Crying.
C Heart rate, Respiratory effort, Muscle tone, Reflex irritability and color
D Heart rate, Respiratory effort, Muscle tone, Tendon Reflex and Crying.
ANSWER:C
810 What are the four vital signs pediatrician check in physical exam?
A Blood pressure, pulse, heart rate and temperature
B Blood pressure, pulse, respiratory rate and temperature
C Blood gaz, pulse, heart rate and primitive reflexes
D Blood gaz, pulse, respiratory rate and primitive reflexes.
ANSWER:B
811 What differentiate hydroceles from inguinal hernias in children?
A Overlie the testes and the spermatic cord, reducible, and can be transilluminated.
B Overlie the testes and the spermatic cord, are not reducible, and can be transilluminated.
C
Separate the testes and the spermatic cord, are not reducible, and do not often
transilluminated.
D Separate the testes and the spermatic cord, are reducible, and do not often transilluminated.
ANSWER:B
812 what does elimination half-life represent?
A
represented by the volume of blood from which a certain amount of unmetabolized drug is
removed (cleared) per unit time by any and all pathways capable of drug removal
B processes of drug absorption, distribution, metabolism and elimination
C
a drug dose administered by an extravascular route that is absorbed into the systemic
circulation
D represents the time for postabsorption blood or plasma concentrations to be reduced by 50%.
ANSWER:D

813 What is Pharmacokinetics does not describe in our body?
A absorption
B distribution
C the relationship between drug dosage and response
D metabolism
ANSWER:C
814
What is the amount of hemoglobin relative to the size of the red pack cell (hemoglobin
concentration) per volume of blood
A MCHC
B MCH
C RDW
D MCV
ANSWER:A
815 What is the Apgar score?
A Pregnancy assessment to evaluate immediate adaptation to extrauterine life
B Initial assessment of the newborn to evaluate immediate adaptation to extrauterine life.
C Score given to classify the baby growth.
D Score given to promote baby development.
ANSWER:B
816
What is the correct answer for acute abdominal pain with the source located in the proximal
bowel?
A Relief of pain after a bowel movement
B Relief of pain after lying down in bed
C Relief of pain after vomiting
D Relief of pain after defecation.
ANSWER:C
817 What is the correct answer for the presence of upper airway obstruction?
A Inspiration is prolonged and accompanied by other signs such as stridor and cough.
B Inpiration is short and often accompanied by stridor and wheezing.
C Expiration is prolonged and often accompanied by audible wheezing.
D Expiration is short and often accompanied by audible wheezing and cough.
ANSWER:B
818 What is the function of effector cells?
A Eliminate antigens
B Capture of antigens for display

C Eliminate of antigens
D Capture antigen and keep
ANSWER:C
819 What is the major cause of chronic abdominal pain in children?
A Recurrent abdominal pain syndrome
B Constipation
C Peptic ulcer disease
D Chronic antral gastritis
ANSWER:B
820 What is the most common medical cause in acute abdominal pain in children?
A Pneumonia
B Constipation
C Gastroenteritis
D Urinary Tract Infection
ANSWER:C
821 What is the name of the condition that results when a person does not have enough platelets?
A Thrombocytopenia
B Thromboangiitis
C Thrombocythemia
D Thrombopathia
ANSWER:A
822 What is the score should be for the normal newborn during Apgar Score check?
A 0 – 4
B 5 – 7
C 0 – 8
D 8 - 10
ANSWER:D
823 What is the sign indicating that the patient has acute cholecystitis?
A Rovsing's sign
B Positive iliopsoas test
C Positive Murphy's sign
D Cullen's sign
ANSWER:C
824 what the normal stool of baby usually is?

A The number, color, and consistency of stools can vary greatly.
B The earliest stools -> meconium : normally passed between 1st week of life.
C Feeding: meconium -> green-brown transition stools. 4-5 day-> white-c milk stools.
D Stool frequency is extremely fix 1-2 times a day.
ANSWER:A
825 Which are not the role of blood?
A Coagulation
B Excrete hormone
C Regulating body pH
D Messenger functions
ANSWER:B
826
Which is a symptom of not having enough red blood cells or hemoglobin (a condition called
anemia)?
A Itching
B Nausea
C Fever
D Fatigue
ANSWER:D
827 Which of the following cells is involved in cell-mediated immunity?
A T lymphocytes
B Mast cells
C Thrombocytes
D B lymphocytes
ANSWER:A
828 Which of the following cells is involved in humeral immunity?
A T lymphocytes
B Mast cells
C Thrombocytes
D B lymphocytes
ANSWER:D
829 Which of the following is NOT an innate defense mechanism of the body?
A skin
B fever
C B lymphocytes
D inflammation

ANSWER:C
830 Which of the following is not the cellular element of blood?
A T cells
B B cells
C Plasma
D Monocytes
ANSWER:C
831 Which of the following is the function of white blood cells?
A Transport oxygen.
B Maintain homeostasis.
C Defend against infection.
D Produce haemoglobin.
ANSWER:C
832 Which of the following is the function of white blood cells?
A Transport oxygen.
B Maintain homeostasis.
C Defend against infection.
D Produce haemoglobin.
ANSWER:C
833 Which of the following statements is true concerning human blood?
A The blood of all normal humans contains red and white cells, platelets, and plasma.
B Some human populations normally lack the ability to produce plasma.
C Proteins are not normal components of human blood.
D All human has the same amount of blood.
ANSWER:A
834 Which of the folowing may be a possible cause of edema in the tissues?
A decreased blood pressure
B decreased level of proteins in the plasma
C the ingestion of a large amount of carbohydrates in the diet
D reduced temperature at the skin surface
ANSWER:B
835 Which one of the bacteria below causes chronic antral gastritis in children?
A Shigella
B Salmonella
C Escherichia coli

D Helicobacter pillory
ANSWER:D
836 Which represents the average size of red blood cells on the CBC result.
A MCHC (Mean Corpuscular Hemoglobin Concentration)
B MCH (Mean Corpuscular Hemoglobin)
C RDW (Red Cell Distribution)
D MCV (Mean Corpuscular Volume)
ANSWER:D
837 Which result demonstrates polycythemia?
A WBC (white blood cell) greater than the normal range for age
B Platelets greater than the normal range for age
C RBC (red blood cell) greater than the normal range for age
D WBC less than the normal range for age
ANSWER:C
838 Which statement are false?
A The ECF is further divided into the plasma water and the Interstitial fluid
B The plasma water is 5% of body weight, higher in newborns and young infants.
C 1 yr of age, the ECF still &gt; ICF
D In fetus and newborn : the ECF &gt; ICF
ANSWER:C
839
You have accessed the CBC of a pale child and you see Microcytic (hypochromic) anemia.
which cause are less likely the cause of Microcytic (hypochromic) anemia?
A Iron deficiency (IDA)
B a thalassaemia trait
C Vitamin B12 deficiency
D Anemia of chronic disease (ACD)
ANSWER:C
840
A 25-day-old infant presents with low grated fever, irritability, poor feeding, and a bulging
fontanel. Spinal fluid (CSF) demonstrates gram-positive cocci. Which of the following is the
most likely diagnosis?
A Listeria monocytogenes
B Group A streptococci
C Group B streptococci
D Streptococcus pneumoniae
ANSWER:B

841
A 5-year boy has high fever for 5 days and is now admitted for suspect to dengue infection.
During hospitalization, the physician examines and notes that he is lethargic with positive
tourniquet, and epistaxis. The vital signs including: Temperature 36oC, Pulse: 120/min and
weak, BP: 100/85mmHg, RR: 30/min. For laboratory findings show WBC: 3500cells/mm3,
hematocrit 52%, Platelet count: 67000cells/mm3, SGOT: 70 IU/L, SGPT: 85 IU/L. For chest
X-Ray in right lateral decubitus reveals right pleural effusion. What is the likely diagnosis of
this patient? (single)
A DF
B DHF grade I
C DHF grade II
D DHF grade III
ANSWER:D
842
A boy had fever for 5 days with repeated vomiting and abdominal pain. His parent brought
him to the hospital. He was admitted with suspected to typhoid fever. After blood culture
done, the result showed salmonella typhi. Which one of the following empiric Antibiotic
should be used to treat him?
A Penicillin
B Ampicillin
C Ceftriaxone
D Gentamycin
ANSWER:C
843
A painless, slow-growing, hard mass producing cutaneous fistulas, a condition commonly
known as lumpy jaw is usually caused by
A Staphylococcus
B Actinomyces
C Nocardia
D Yersinia
ANSWER:B
844
A patient 70 kg with Acute ITP need treat by prednisolone(1 tablet:5mg) how many tablet he
got?
A 10 tablets
B 12 tablets
C 14 tablets
D 8 tablets
ANSWER:B

845
A patient has rheumatic arthritis with autoimmune hemolytic anemia and presenting Ig
antibodies it is?
A Primary and cold
B Secondary and warm
C Secondary and cold
D Warm and primary
ANSWER:B
846
A patient hospitalization with pneumonia was treated by antibiotic, the fever disappeared
today how many more day to continue antibiotic.
A 3days
B 2days
C 1days
D 5days
ANSWER:A
847
A patient was diagnosis ALL and present lymphoblast in the blood with appropriate therapy
the prognosis could be:
A Good
B Worse
C Unpredictable
D Good or worse
ANSWER:A
848 A patient with autoimmune anemia need transfusion need to be used:
A Total blood
B Pack red cell
C Platelets transfusion
D Plasma transfusion
ANSWER:B
849 Acute hemorrhage cystitis, which below sentence is INCORRECT?
A Often cause by E coli
B Can cause by Adenovirus 11 and 21
C Self-limited
D More common in girl
ANSWER:D
850 Acute post streptococcal glomerulonephritis occurs after pharyngitis around:
A Less than 1 week

B 1-2 weeks
C 2-3 weeks
D 3-4 weeks
ANSWER:B
851 Acute post streptococcal glomerulonephritis occurs after pyoderma around:
A Less than 1 week
B 1-2 weeks
C 2-3 weeks
D 3-6 weeks
ANSWER:D
852 All below statements are defined as recurrent UTI, EXCEPT:
A >= 2 episode of acute pyelonephritis
B
1 episode of pyelonephritis and >=1 episode of cystitis1 episode of pyelonephritis and
&gt;=1 episode of cystitis
C >= 3 episode of cystitis
D >= 4 episode of cystitis
ANSWER:D
853
All infants born to HIV-infected mothers test antibody-positive at birth because of passive
transfer of maternal HIV antibody across the placenta during gestation. If the child is not
infected with HIV, this HIV antibody will disappear by the age..
A 6 months
B 12 months
C 18 months
D 24 months
ANSWER:C
854 ALL relapse the treatment may be given for:
A 2-3years
B 1 year
C 2years
D 5years
ANSWER:A
855 All the following are dimensions of food security EXCEPT
A access
B utilization
C preparation

D stability
ANSWER:C
856 All the following statements concerning iron requirement in children are true EXCEPT
A breast milk provides optimal intake of iron for the first 4 months
B iron present in animal protein is more bioavailable than that found in vegetables
C iron deficiency is the most common micronutrient deficiency
D cow’s milk is a good source of bioavailable iron
ANSWER:D
857 All the following statements regarding giardiasis are true EXCEPT
A incubation period of Giardia infection usually is 1-2 wk
B most infections are asymptomatic
C symptomatic infections occur more frequently in adults than in children
D stools do not contain blood, mucus, or fecal leukocytes
ANSWER:C
858 Among of pneumonia how many percent of neonate pneumonia?
A 4%
B 3%
C 5%
D 6%
ANSWER:A
859 Among the symptoms below, which one is likely to make a diagnosis of pyelonephritis?
A Abdominal, back or flank pain, fever, vomiting, malaise, nausea, vomiting
B Dysuria, pollakiuria, vomiting, hypogastric pain, hematuria, fever
C Abdominal pain, tenesmus, fever, dysuria, urine incontinent, vomiting
D Flank pain, macroscopic hematuria, vomiting, dysuria, pollakiuria
ANSWER:A
860 Among the symptoms below, which one is likely to make a diagnosis of cyctitis?
A Epigastric pain, vomiting, dysuria, high grade fever, anorexia
B Hematuria, hypertension, edema, oliguria, low grade fever
C Massive proteinuria, hematuria, hypertension, hypogastric pain
D Suprapubic pain, pollakiuria, dysuria, urgency
ANSWER:D
861 Anemia that occurs in malaria is? (single)
A Iron deficiency anemia
B Vitamin B12 deficiency

C Hemolytic anemia
D Aplastic anemia
ANSWER:C
862 Anti-HAV IgM is detectable up to:
A 14 days
B 1 month
C 3 months
D 6 months
ANSWER:D
863 Antibiotic of choice for acute post-sreptoccacal glomerulonephritis is:
A Ceftriaxone
B Clarithromycin
C Erythromycin
D Penicillin
ANSWER:D
864 Atypical UTI is defined:
A UTI associated with functional abnormalities of the urinary tract
B UTI with anatomical abnormalities of the urinary tract
C UTI associated with anatomical or functional abnormalities of the urinary tract
D UTI without urinary symptoms
ANSWER:C
865
Based on the current recommendation of Cambodian Guidelines, all infants born to HIV-
infected mothers should be tested with_______________ for HIV diagnosis:
A HIV DNA PCR test at birth
B HIV DNA PCR test at 6 week of age
C HIV RNA PCR test at birth
D HIV antibody test at birth or at 6 week of age
ANSWER:A
866 Cefixime is a third-generation cephalosporin active against all the following EXCEPT
A Streptococci
B Staphylococci
C H. influenza
D Neisseria gonorrhoeae
ANSWER:B

867
Cephalexin is a cephalosporin active against S. aureus, Streptococcus, E. coli, Klebsiella, and
Proteus. To which generation of cephalosporins it belongs?
A 1st
B 2nd
C 3rd
D 4th
ANSWER:A
868
Cephalosporins are widely used in pediatric practice, both in oral and parenteral
formulations. Which of the following is a 2nd generation cephalosporins?
A cefazolin
B cephalexin
C cefuroxime
D ceftazidime
ANSWER:C
869 Chronic hepatitis B was classified when HBsAg persists over:
A 3 months
B 6 months
C 12 months
D 24 months
ANSWER:B
870 Complete recovery in acute post-streptococcal glomerulonephritis is over:
A 80%
B 85%
C 90%
D 95%
ANSWER:D
871 Concerns to be considered among vegetarians are all the following EXCEPT
A higher bioavailability of iron
B lower B12 levels
C risk of having lower level of fatty acids
D lower levels of calcium and vitamin D3
ANSWER:A
872 Contraindications for immediate Lumber Puncture include below Except:
A Cardiovascular disorder
B Infection of the skin overlying the site of the LP

C Severe or significant thrombocytopenia
D Very high white blood cells
ANSWER:D
873 Copper retension in cholestasis provoque:
A Jaundice
B Pruritus
C Hepatotoxicity
D Hypercholesterolemia
ANSWER:C
874
Current anti-TB treatment regimen for tuberculous meningitis in children (under 15years
old) in Cambodia is:
A 2RHZS/4RH
B 2RHZ/4RH
C 2RHZE/ 10RH
D 2RHZE/1RHZ/5RHE
ANSWER:C
875 Cytopathic injury in viral hepatitis is specifically increase:
A GT
B AST
C ALT
D PAL
ANSWER:C
876 Delaying response‘s treatment of pneumonia can identify by:
A Repeat the blood test
B Repeat the blood culture
C Repeat the X-ray
D Repeat the chest X-ray
ANSWER:D
877 Dengue infection caused by:
A Measles virus
B Dengue virus
C Poliovirus
D Myxovirus parotidis
ANSWER:B
878 Diagnosis of malaria is confirmed by:

A Blood culture
B Urine culture
C Blood smear including thick and thin smear
D Lumbar puncture
ANSWER:C
879
Diphtheria is an acute toxic infection; toxic cardiomyopathy occurs in 10-25% of patients
with respiratory diphtheria and is responsible for 50-60% of deaths. Of the following, the
mainstay of therapy is
A antitoxin
B penicillins
C erythromycin
D clindamycin
ANSWER:A
880 Dullness on percussion present in:
A Atelectasis and pneumonia and lung abces.
B Atelectasis and pneumonia and pleural effusion
C Atelectasis and pneumonia and emphysema
D Atelectasis and pneumonia and asthma
ANSWER:B
881 Duration of antibiotic therapy for pyelonephritis is?
A 3 – 5 days
B 5 – 7 days
C 7 – 10 days
D 7 – 14 days
ANSWER:D
882 Duration of antibiotic treatment in pneumonia:
A 5days
B 7days
C 10days
D 5-7days
ANSWER:D
883 Eosinophilic meningitis is defined as:
A 5 or more eosinophils/mm3 of CSF
B 10 or more eosinophils/mm3 of CSF
C 50 or more eosinophils/mm3 of CSF

D 100 or more eosinophils/mm3 of CSF
ANSWER:B
884 Essential fatty acids deficiency is associated with all the following features EXCEPT
A desquamating skin rashes
B alopecia
C thrombocytosis
D impaired immunity
ANSWER:C
885 Etiology of congestive heart failure are in the following, except one:
A PDA (persistent ductus arteriosus)
B VSD (Ventricular Septal Defect)
C Acute diarrhea
D Coartation of aorta
ANSWER:C
886 First choice for the prophylaxis of pneumocystis jirovecii (PCP) in HIV patient?
A Sulfadiazine
B Trimethoprim/sulfamethoxazole
C Fluconazole
D Ciprofloxacin
ANSWER:B
887
For Children age >5years old or adult, a CD4 T cell percentage of how much is comparable
to a CD4+ T cell count of 200/microliter?
A 10 %
B 15 %
C 20 %
D 25 %
ANSWER:B
888 For the life cycle of plasmodium as the following are correct, Except?
A
In the exo-erythrocytic cycle, after biting by female anopheline mosquito, sporozoites are
going to liver cells and develop to be schizonts.
B Only Plasmodium Vivax has hypnozoite stage
C Both Plasmodium vivax and Plasmodium Falciparum have hypnozoite stage
D
Schizont that develop in liver cell from young shizont, growing shizont and mature schizont
to be merozoite needs 7 – 10 days for Plasmodium Falciparum
ANSWER:C

889 Fulminant viral hepatitis is rarely occur in:
A Viral hepatitis A and B
B Viral hepatitis A and C
C Viral hepatitis A and D
D Viral hepatitis A and E
ANSWER:B
890 Giardiasis should be considered in children who have the following presentations EXCEPT
A acute dysenteric diarrhea
B persistent diarrhea
C failure to thrive
D malabsorption
ANSWER:A
891
Group B streptococcus (GBS), or Streptococcus agalactiae, is a major cause of neonatal
bacterial sepsis. Of the following, the MOST common syndrome associated with childhood
GBS disease beyond early infancy is
A bacteremia without a focus
B meningitis
C ventriculitis
D septic arthritis
ANSWER:A
892
Haemophilus influenzae type b (Hib) causes meningitis: The characteristic of Hib consists of
following EXCEPT:
A It is a slowly growing bacteria
B Gram negative coccobacillus
C Frequently found in the upper respiratory tract
D It spreads from one individual to another in airborne droplets or direct contact with secretion.
ANSWER:A
893 HBeAg in viral hepatitis B is considered a marker of:
A Chronic viral hepatitis B
B Liver cirrhosis
C Undetectable viral load
D HBV replication and infectivity
ANSWER:D

894 History is an important part of diagnostic process for epilepsy in children including, Except?
A seizure episodes
B circumstance of seizure
C frequency of attack (seizure)
D Answer a, b and c are correct
ANSWER:D
895 HIV belongs to which family of viruses?
A Retroviridae
B Reoviridae
C Calci viruses
D Picorna viruses
ANSWER:A
896
If meningitis caused by L. monocytogenes infection is suspected, as in young infants or
those with a T-lymphocyte deficiency, the antibiotics should be prescribed is:
A
Ceftriaxone (100 mg/kg/24 hr administered once per day or 50 mg/kg/dose, given every
12 hr)
B Ampicillin plus an aminoglycoside
C Ceftazidime and an aminoglycoside.
D Intravenous penicillin for 5-7 days is the treatment of choice
ANSWER:B
897
Impetigo (or pyoderma) has traditionally been classified into 2 clinical forms: bullous and
nonbullous. Of the following, which statement is TRUE?
A bullous impetigo is more common
B nonbullous lesions are most common on the trunk and perineum
C regional lymphadenitis is commonly associated with nonbullous lesions
D nonbullous impetigo is generally accompanied by fever
ANSWER:C
898 In acute glomerulonephritis, renal biopsy should be done in:
A Children less than 3 years old
B Patient with gross hematuria
C Patient with rapid progressive renal failure
D Patient suspected associated with nephrotic syndrome
ANSWER:C
899 In auscultation in atelectasis they can heard:

A Crackle
B Ranchi
C Crackle and rhonchi
D Silence
ANSWER:D
900
In children the nervous system is vulnerable to various disorders. Which one below is the
most important cause for nervous system disorders in children:
A Trauma
B Vascular disorders
C Degeneration disorder
D Infection
ANSWER:D
901 In cholestasis, absorption of ALL VITAMINES below are affected, except:
A Vitamin A
B Vitamin C
C Vitamin D
D Vitamin E
ANSWER:B
902 In cholestasis, fraction of direct bilirubin is over:
A 10%
B 20%
C 30%
D 40%
ANSWER:B
903 In developing world UTI affect:
A More boys than girls
B More girls than boys
C Same proportion of boys and girls
D Less than &lt;1% in low birth weight infant
ANSWER:B
904
In infants and very young toddlers chronic diarrhea can appear following infectious enteritis.
The pathogenesis of the diarrhea is not always clear and may be related to
A food protein allergy
B bacterial overgrowth
C giardiasis

D Strongyloides stercoralis
ANSWER:A
905
In meningococcal purpura, necrotic skin lesions are less common among children treated
with
A penicillin G
B ampicillin
C cefotaxime
D ceftriaxone
ANSWER:D
906 In partial treated meningitis, the CSF finding usually presents as below (except):
A Pressure: normal or elevated
B Leukocytes: 5-10,000 /mm3
C Protein:>100 mg/dL
D Glycose: >100 mg/dL
ANSWER:D
907 In physiologic jaundice is defined as:
A Transient elevation of direct belirubinemia
B Transient elevation of indirect bilirubunemia
C Increase indirect bilirubin with increase ALT
D Increase indirect bilirubin with increase AST
ANSWER:B
908 In the blue diaper syndrome, symptoms can include all the following EXCEPT
A vomiting
B diarrhea
C failure to thrive
D nephrocalcinosis
ANSWER:B
909 In viral hepatitis B, Anti-HBc IgG is detectable in:
A Acute phase
B Throughout infection
C 12 months
D Up to 10 years
ANSWER:B
910
In which of the following circumstances the diagnosis of acute rheumatic fever can be made
without strict adherence to Jones criteria?

A when chorea occurs as the only major manifestation of acute rheumatic fever
B
when indolent carditis is the only manifestation months after the apparent onset of acute
rheumatic fever
C
in a limited number of patients with recurrences of acute rheumatic fever in particularly high-
risk populations
D all of the above
ANSWER:D
911 Increase lipid level seen in nephrotic syndrome patient caused by:
A Increase lipoprotein synthesis by liver and decrease lipid cathabolism
B Patient eats too much lipid and uses high dose of steroid
C Increase lipoprotein uptake by liver to synthesis of protein
D High dose steroid usually increase lipid level in nephrotic syndrome patient
ANSWER:A
912 Increase serum alkaline phosphatase (ALP) level is found in condition with:
A Hemolysis
B Cholestasis
C Chronic liver disease
D Hepatorenal syndrome
ANSWER:B
913 Increase serum conjugate bilirubin level is found in condition with:
A Hemolysis
B Cholestasis
C Chronic liver disease
D Hepatorenal syndrome
ANSWER:B
914 Increase serum GGT level is found in condition with:
A Hemolysis
B Cholestasis
C Chronic liver disease
D Hepatorenal syndrome
ANSWER:B
915 Indication of phototherapy:
A Biliary atresia
B Increase indirect (non conjugate) bilirubinemia
C Increase direct (cobjugate) bilirubinemia

D Viral hepatitis
ANSWER:B
916
Isoniazid is accompanied by significant drug–drug interactions, which of the following is
TRUE?
A aluminum salts increase absorption of isoniazid
B isoniazid increase toxicity of carbamazepine
C rifampin decreased hepatotoxicity of isoniazid
D isoniazid decrease level of warfarin
ANSWER:B
917 Main First line treatment of hypertension in acute glomerulonephritis is:
A Low salt diet
B Inhibitor enzyme conversion drug
C Diuretic
D Calcium blocker
ANSWER:C
918 Management of edema in nephrotic syndrome patient:
A Give furosemide 1mg/kg, steroid 2mg/kg/day and potassium supplement
B Restrict water and salt intake along with steroid therapy
C Ask patient to eat high protein food 50g/kg
D Ask patient to sleep with his/her feet-up to facilitate blood flow
ANSWER:B
919 Management of the diarrhea in chronically malnourished children is based on
A intravenous therapy
B standard osmolarity oral rehydration solutions
C reduced osmolarity oral rehydration solutions
D slow resumption of feeds
ANSWER:C
920
Meropenem is a carbapenem antibiotic with broad-spectrum activity but has no activity
against
A P. aeruginosa
B S. maltophilia
C L. monocytogenes
D S. aureus
ANSWER:B

921
Metronidazole is highly effective in the treatment of infections caused by anaerobes. It can
increase the level of which of the following drugs if given at the same time
A carbamazepine
B rifampin
C phenobarbital
D phenytoin
ANSWER:D
922 N meningitidis is:
A a gram-negative diplococcus
B carried in the nasopharynx of otherwise healthy individuals.
C
Currently, is the leading cause of bacterial meningitis in children and young adults,
accounting for 59% of cases.
D All of the above.
ANSWER:D
923
Newborns with congenital chloride diarrhea present with severe life-threatening secretory
diarrhea during the 1st few wk of life. Of the following, the MOST common laboratory
finding is
A metabolic alkalosis
B hyperchloremia
C hyperkalemia
D hypernatremia
ANSWER:A
924
One of the following statement is not true regarding an Icteric Leptospirosis (Weil
Syndrome):
A
Weil syndrome is a rare (<10% of cases) severe form of leptospirosis seen more commonly
in adults (>30 yr) than in children
B
The immune phase, however, is characterized by jaundice, renal failure, thrombocytopenia,
and, in fulminant cases, hemorrhage and cardiovascular collapse
C Acute kidney failure occurs in 16-40% of cases and is the principal cause of death
D Hemorrhagic manifestations occur in >50% of cases
ANSWER:D
925
Oral rehydration is the mainstay of treatment for pediatric traveler’s diarrhea and the drug of
choice is
A metronidazol
B azithromycin

C amoxicillin
D trimethoprim-sulfamethoxazol
ANSWER:B
926 People at risk for L. monocygenes for meningitis include the following, except:
A Infants and young children
B Pregnant women
C Elderly individuals (&gt;60 years)
D Head trauma
ANSWER:D
927 Physical examination of whooping cough present:
A In all patients with pertussis fever typically absent
B In all patients with pertussis fever typically Present
C In all patients with pertussis high fever
D In all patients with pertussis fever with caught
ANSWER:A
928 Pneumonia definition:
A Inflammation of the air way and interceptum[U1][U2]
B Inflammation of the lung tissue
C Inflammation of the parenchyma of the lung
D Inflammation of the air way and alveolar
ANSWER:C
929 Possible complications of thalassemia
A Infection
B Iron overload
C Only B
D Both A and B
ANSWER:D
930
Prophylaxis for people who have had acute rheumatic fever with carditis but without residual
heart disease persist for
A 5 yr or until 21 yr of age, whichever is longer
B 10 yr or until 21 yr of age, whichever is longer
C 5 yr or until 40 yr of age, whichever is longer
D 10 yr or until 40 yr of age, whichever is longer
ANSWER:B

931 Recommended duration of therapy for manifestations of group B streptococcus are as follow
A bacteremia without a focus: 7 days
B meningitis: 3-6 weeks
C ventriculitis: at least 8 weeks
D septic arthritis: 3-4 weeks
ANSWER:D
932 Regarding epidemiology of Neisseria meningitides, all the following are true EXCEPT
A
meningococci are transmitted during close contact via aerosol droplets or exposure to
respiratory secretions
B meningococci survive for long periods in the environment
C
smoking and respiratory viral infection are associated with increased rates of carriage and
disease
D the highest rate of meningococcal disease occurs in infancy
ANSWER:B
933 Regarding leptospirosis, the following statements are true EXCEPT
A Leptospirosis is bacterial disease that affects humans and animals,
B Most human cases result from inhale the droplet containing the organism (air born-diseases).
C Leptospira are aerobic spiral bacteria with a terminal hook at 1 or both ends,
D It was recognized in 1886 by Adolf Weil , and thus was named as Weil disease
ANSWER:B
934
Regarding the clinical manifestations of leptospirosis, all the following statement are true
EXCEPT
A
The spectrum of human leptospirosis ranges from asymptomatic infection (most cases) to
severe disease with multiorgan dysfunction and death,
B The incubation period range from 7-12 days
C The septicemic phase lasting for 7-14 days
D
The onset is usually abrupt, and the illness tends to follow a biphasic course (septicemic
phase and immune phase)
ANSWER:C
935 Regarding the epidemiology of mumps, the following statements are true EXCEPT
A Incubation period range from 14-18 days
B
Viral shedding before onset of symptoms and in asymptomatic infected individuals is very
rare.

C Mumps is spread from person to person by respiratory droplets.
D
The period of maximum infectiousness is 1-2 days before to 5 days after onset of parotid
swelling.
ANSWER:B
936 Seroconversion of HBeAg to Anti-HBe is associated with:
A Disappearance of serum HBV DNA
B Liver cirrhosis
C Undetectable HBsAg
D Disappearance of serum anti-HBcIgG
ANSWER:A
937
Signs of Meningitis in infants and older children. Which one below that presents in infant,
but not in older children?
A Bulging fontanelle
B Fever
C Stiff neck
D Lethargy
ANSWER:A
938
Streptococcus pneumonia causes meningitis: The characteristic of S pneumonia consists of
following EXCEPT:
A Gram negative coccus
B It is a common colonizer in the human nasopharynx
C
It causes meningitis by escaping local host defense and phagocytic mechanisms, from
bacteremia or direct extension from otitis media or sinusitis.
D It is a common agent in meningitis associated with basilar skull fracture or CSF leak.
ANSWER:A
939 Symptoms of Meningitis and Septicemia: which statement is correct?
A Fever presents more in septicemia
B Neck stiffness presents in both meningitis and septicemia
C Photophobia (dislike bright lights) presents in septicemia than in meningitis
D Severe headache happens more frequently in meningitis than in septicemia
ANSWER:D
940 The adequate intake (AI) for the total fat in a nine-month-old boy is
A 10 gm/day
B 20 gm/day
C 30 gm/day

D 40 gm/day
ANSWER:C
941 The adequate intake (AI) for the total protein in a nine-month-old boy is
A 9 gm/day
B 11 gm/day
C 13 gm/day
D 15 gm/day
ANSWER:B
942 The below statement is correct in Nephritic Syndrome, EXCEPT:
A Abrupt onset of disease
B Raised blood pressure
C Heavy proteinuria
D Presence of red cells cast in urine microscopy
ANSWER:C
943 The below statement is correct in typical features Nephrotic Syndrome, EXCEPT:
A Insidious onset of disease
B Normal blood pressure
C Heavy proteinuria
D Presence of red cells cast in urine microscopy
ANSWER:D
944 The below urine collection methods, which one is more easy to contaminate?
A Midstream urine
B Catheter specimens
C Clean catch urine
D Bag urine specimen collection
ANSWER:D
945 The causal agent of typhoid fever is? (single)
A Shigiella
B Staphylococcus Aureus
C Salmonella typhi or Salmonella paratyphi A,B and C
D Meningococcus
ANSWER:C
946 The causes of Reye's syndrome are listed below EXCEPT:
A triggered by using aspirin to treat a viral illness or infection,
B followed by infections of pathogens such as influenza infection, varicella,

C
possible triggered by some agents such as Acetaminophen, outdated tetracycline, valproic
acid, warfarin, zidovudine, and some neoplastic drugs and including some Nonsteroidal anti-
inflammatory drugs.
D After an experience of febrile convulsion in infants.
ANSWER:D
947 The classical clinical triad of meningitis is:
A Fever , Headaches, Vomiting
B Fever, headache and neck stiffness.
C Fever, headaches and convulsion
D Fever, neck stiffness and coma
ANSWER:B
948 The classical clinical triad of meningitis syndrome is:
A Fever, headache and convulsion
B Headache, convulsion and neck stiffness
C Headache, lethargy and photophobia
D Fever, headache and neck stiffness.
ANSWER:D
949 The common drug that can cause Nephrotic syndrome is:
A Ceftriaxone
B Nonsteroidal anti-inflammatory drug
C Antituberculosis drug
D Anti-diabetes drug
ANSWER:B
950
The common signs and symptoms of meningitis and septicaemia in children are very similar.
Which of the following signs and symptoms of meningitis that was not often presented in
septicemia?
A Vomiting or refusing feeds
B Lethargy
C Stiff neck
D High temperature, fever, possibly with cold hands and feet
ANSWER:C
951 The complement C3 level in Nephrotic syndrome is:
A Low
B Normal
C Hight

D Borderline
ANSWER:B
952 The condition associated with chronic ITP:
A Viral infection
B Upper respiratory infection
C Immune disorder.
D Auto-immune disorder
ANSWER:D
953 The definitive’ diagnosis of ALL base on
A Present of atypical lymphocyte in the blood
B Anemia and thrombocytopenia and present of atypical lymphocyte in the blood
C Bone marrow aspiration
D Pancytopenia with Present of atypical lymphocyte in the blood
ANSWER:C
954
The diagnosis is made easily in patients with acute bilateral parotitis and a history of recent
exposure. The differential diagnosis of parotitis due to mumps is EXCEPT
A Coxsackievirus infection
B HIV infection
C Influenza A virus infection
D Varicella Zoster infection
ANSWER:D
955 The diagnosis of autoimmune anemia base on:
A Clinical manifestation
B History
C Combs’ test
D No answer
ANSWER:C
956
The diagnosis of meningitis mainly depends on examination of cerebrospinal fluid (CSF)
obtained by lumbar puncture. The CSF protein may reach 3,000 (mg/dL) in:
A Acute bacterial meningitis
B Fungal meningitis
C Viral meningitis
D Tuberculous meningitis
ANSWER:D

957 The following clinical manifestations were listed in WHO pediatric clinical stage 2, EXCEPT.
A Unexplained persistent hepatomegaly
B Persistent generalized lymphadenopathy (PGL)
C Pruritic Papular Eruptions (PPE)
D Extensive wart virus infection
ANSWER:B
958
The following infections may cause malabsorption in immunocompromised children
EXCEPT
A Shigella
B Salmonella
C enteropathogenic E. coli
D Giardia
ANSWER:C
959 The following statements are correct, Except:
A
Heart failure occurred when blood is still able to pump adequately maintain sufficient blood
to meet the metabolic demands of the body
B
Heart failure occurred when blood is unable to pump adequately maintain sufficient blood to
meet the metabolic demands of the body
C Cardiac output depend on stroke volume and heart rate
D Stroke volume determined by preload, afterload and contractility of myocardium
ANSWER:A
960 The following statements are correct, except?
A
Head tilt and chin lift should be done in children with head trauma and suspected cervical
spine trauma.
B
Jaw thrust is a method should be done for open airway in children with head trauma and
suspected cervical spine trauma.
C
Signs of poor systemic perfusion: weak and fast pulse, mottling of color over the extremities
or trunk, prolong CRT and oliguria (urine output < 0.5ml/kg/h)
D
Elevation of intracranial pressure when there is an increase in the volume of any intracranial
components such as brain parenchyma, blood and CSF.
ANSWER:A
961 The following statements are trues, EXCEPT: (single)
A Only plasmodium falciparum that can cause severe and complicated malaria

B
Malaria caused by plasmodium vivax is usually caused uncomplicated malaria and rarely
death.
C
All of malaria caused by plasmodium vivax or plasmodium falciparum can be caused severe
and complicated malaria
D
The mode of transmission of malaria is mostly caused by the bite of female anophele
mosquitos.
ANSWER:C
962 The greatest number of deaths globally caused by a parasite are due to:
A Schistosomiasis
B Malaria
C Filariasis
D Ascariasis
ANSWER:B
963 The ideal agent for treating fungal urinary tract infections is
A amphotericin B
B fluconazole
C voriconazole
D micafungin
ANSWER:B
964
The IgM response occurs earlier in the illness, generally peaking at 7-10 days after infection,
and usually disappears within a few weeks, but for some infections it can persist for months
such as
A measles
B mumps
C rubella
D hepatitis A
ANSWER:D
965 The indicator strongly consider to ALL is:
A Anemia
B Leucopenia
C Thrombocytopenia
D Thrombocytopenia and anemia
ANSWER:D
966 The largest protozoan that parasitizes human is
A Entamoeba histolytica

B Giardia lamblia
C Balantidium coli
D Isospora belli
ANSWER:C
967 The longest incubation period of viral hepatitis in children is:
A Hepatitis A
B Hepatitis B
C Hepatitis C
D Hepatitis D
ANSWER:B
968
The meninges refer to the membranous coverings of the brain and spinal cord. There
are three layers of meninges, known as: (which one is not correct?)
A Skull
B dura mater
C arachnoid mater
D pia mater.
ANSWER:A
969 The most common cause of Eosinophilic meningitis in human is:
A Gnathostoma spinigerum
B Angiostrongylus cantonensis
C Taenia solium
D Ascaris lumbricoides
ANSWER:B
970 The MOST common clinical manifestation of meningococcal infection is
A asymptomatic carriage
B meningococcal meningitis
C bacteremia without sepsis
D meningococcal septicemia
ANSWER:A
971 The MOST common complication of acute severe meningococcal septicemia is
A arthritis
B focal skin infarction
C endocarditis
D pneumonia
ANSWER:B

972 The MOST common complication of measles is
A aseptic meningitis
B pharyngitis
C otitis media
D pneumonia
ANSWER:C
973 The most common complication of mumps adolescent or young adult is
A Meningitis
B Orchitis
C Pancreatitis
D Thrombocytopenia
ANSWER:B
974 The most common etiology of viral hepatitis in children is:
A Viral hepatitis A
B Viral hepatitis B
C Viral hepatitis C
D Viral hepatitis D
ANSWER:A
975 The most common form histologic finding of nephrotic syndrome in children is:
A mesangial proliferation
B minimal change disease
C focal segmental glomerulosclerosis
D membranoproliferative glomerulonephritis
ANSWER:B
976 The most common pathogen responsible for pneumonia in neonate is:
A Pneumococcus
B Staphylococcus
C Streptococcus
D Group B streptococcus
ANSWER:D
977 The most common pathogens that cause UTI include:
A Escherichia Coli, Enterococcus faecalis, Klebsiella, Proteus, Enterobacter, Citrobacter
B
Enterococcus faecalis, Klebsiella, Streptococcus pneumonia, Haemophilus influenza,
Pseudomonas, Parainfluenzae

C Salmonella Typhi, Proteus, Enterobacter, Citrobacter, Entamoeba histolytica, Giardia lamblia
D Klebsiella, Proteus, Enterobacter, Rotavirus, Ebola virus, Polio virus
ANSWER:A
978 The most common reason of proteinuria in minimal change nephrotic syndrome is:
A separation of the epithelial podocyte from the glomerular basement membrane
B
appearance of gaps in the glomerular basement membranethrough which elements of blood,
including cells, may pass
C
loss of negative charge along the glomerular basement membrane and the epithelial cell
surface
D Different between oncotic pressure and hydrostatic pressure that allow protein to pass freely
ANSWER:C
979 The most common route of infection in UTI is:
A Hematogenous
B Ascending
C Fecal oral route
D Direct contact with contaminated urine
ANSWER:B
980 The most common route of infection in UTI is:
A Hematogenous
B Ascending
C Fecal oral route
D Direct contact with contaminated urine
ANSWER:B
981 The MOST common serious bacterial infection in infant aged 1-3 mo is
A pyelonephritis
B meningitis
C pneumonia
D septic arthritis
ANSWER:A
982 The most important predictive factor of ALL: are
A Age of the patient and speed of response with treatment and medication
B Speed of response with treatment and medication and age of onset of the disease.
C Age of the patient and speed of response with treatment and red cell count

D
Age of the patient at the time of diagnosis and speed of response with treatment and
leucocyte count
ANSWER:D
983 The optimal amount of blood to collect from a pediatric patient for blood culturedepend on
A age
B weight
C length
D BMI
ANSWER:B
984 The PCV 13 vaccine was licensed in USA in:
A 2010
B 2011
C 2012
D 2009
ANSWER:A
985
The phenomenon of plasma leakage in Dengue Hemorrhagic Fever (DHF) is evidenced in
the followings. Which one is not correct answer?
A Leucocytosis
B Hemoconcentration with hematocrit of patient increased 20% of baseline
C Right pleural effusion
D Ascites
ANSWER:A
986 The preferred initial ART regimen for children <3 years of age and <10 kg this is:
A ABC + 3TC + LPV/r
B AZT+3TC+NVP
C D4T+3TC+EFV
D TDF+3TC+EFV
ANSWER:A
987
The signs of systemic venous congestion in congestive heart failure in the following are not
correct, except one
A Difficulty of breathing
B Poor feeding
C Crackle rales
D Jugular vein turgescence

ANSWER:D
988 The TRUE hemoglobin cutoff to define anemia is
A 120 g/L for children 6-59 mo
B 115 g/L for children 5-11 yr
C 110 g/L for children 12-14 yr
D 110 g/L for non-pregnant women
ANSWER:B
989 There is an increase risk of infection in Nephrotic syndrome due to:
A Hypoalbuminemia
B Severe edema
C Hypercholesterol
D Lost of urinary immunoglobulin
ANSWER:D
990
To prevent first attack of acute rheumatic fever after acute group A streptococcus
pharyngitis, appropriate antibiotic therapy should be instituted before
A 3rd day of illness
B 5th day of illness
C 7th day of illness
D 9th day of illness
ANSWER:D
991 Transmission of viral hepatitis via needle stick injury is highest in:
A Viral hepatitis A
B Viral hepatitis B
C Viral hepatitis C
D Viral hepatitis D
ANSWER:B
992 Tranxudate meaning?
A The fluid build-up cause by systemic condition
B The fluid build-up cause by systemic condition and inflammation
C The fluid build-up cause by infection
D The fluid build-up cause by inflammation
ANSWER:A
993 Urine dipstick of +2, the value of proteinuria is:
A 10mg/dL
B 20mg/dL

C 100mg/dL
D 300mg/dL
ANSWER:C
994 Urine dipstick of +3, the value of proteinuria is:
A 10mg/dL
B 20mg/dL
C 100mg/dL
D 300mg/dL
ANSWER:D
995 Usually, CSF finding in acute bacterial meningitis: (Except):
A Pressure: 100-300 mm/H2O
B Leukocytes: 100-10,000 /mm3
C Protein:<45mg/dL
D Glycose: <40 mg/dL
ANSWER:C
996 Vitamin B12 and bile salts are only absorbed in
A distal ileum
B duodenum
C proximal jejunum
D colon
ANSWER:A
997 What is (are) is not correct answer for clinical examination on children with epilepsy?
A Record vital signs: temperature, respiratory rate, pulse rate and blood pressure
B Past medical history such as head injury, CNS infection, Medication
C Family history of seizure
D None of the above
ANSWER:D
998 What is the cause of chickenpox?
A Bacterial
B Parasite
C Virus
D Candida
ANSWER:C
999 What is the child at risk?
A Causing from anxiety disorder

B Causing from Depression
C Causing concern for the safety, welfare or well being of the child
D Causing from enuresis
ANSWER:C
1000
What is the specific treatment of malaria follow National Guideline of treatment for
uncomplicated P. Falciparum malaria in Cambodia?
A Ceftriaxone
B Artesunate + Mefloquine
C Ciprofloxacin
D Metronidazole
ANSWER:B
1001
When meningitis is suspected, lumbar puncture should be performed and CSF should sent
for testing (EXCEPT)
A Chemistry: protein, glucose
B Bacteriology: Gram stain, culture and antibiogram (DST)
C Cytology: WBC (leukocytes) and differentials
D Drug Toxicity testing
ANSWER:D
1002 Which antibiotics should be avoided in patients with botulism?
A penicillin
B cephalosporin
C macrolide
D aminoglycoside
ANSWER:D
1003 Which are in the following statement is not correct?
A
Characteristic of coma due to metabolic cause in elevation of intracranial pressure are pupils
are equal and reactive
B
Characteristic of coma due to head trauma cause in elevation of intracranial pressure are
pupil is mydriasis and unequal
C
Characteristic of coma due to metabolic cause in elevation of intracranial pressure are pupils
are unequal and not reactive
D
Characteristic of coma due to head trauma cause in elevation of intracranial pressure are
pinpoint pupil and equal
ANSWER:C
1004 Which indicator that is highly correlated with urine culture positive for enteric pathogen?

A Albumin in urine
B Bacteria in urine
C Nitrite in urine
D Red blood cells in urine
ANSWER:C
1005 Which indicator that is highly correlated with urine culture positive for enteric pathogen?
A Crystal in urine
B Bacteria in urine
C Nitrite in urine
D Red blood cells in urine
ANSWER:C
1006 Which is Prenatal diagnosis of fetal varicella infection ?
A Xrey
B Ultrasound
C MRI
D Scanner
ANSWER:B
1007
Which of the following enzymes is responsible for processing HIV proteins during the
production of new viruses?
A Reverse transcriptase
B Integrase
C Protease
D DNA polymerase
ANSWER:C
1008 Which of the following is a gram negative bacterium?
A Staphylococcus aureus
B Streptococcus pneumoniae
C Actinomyces
D Haemophilus influenza
ANSWER:D
1009
Which of the following is NOT true about Cotrimoxazole (CTX) prophylaxis for HIV-
infected children based on the current Cambodian National Guidelines?
A
All children diagnosed with HIV should continue or be started on CTX until children are
transitioned into adult care at age 15 years

B
Stop CTX if the child is anemic as cotrimoxazole may cause bone marrow suppression or if
Grade 3/4 toxicity rash occurs
C Start CTX immediately after positive HIV infection diagnosis was made
D Stop CTX if the child was on ART and CD4 increase to normal value
ANSWER:D
1010
Which of the following is NOT true about Definition of Treatment Failure for children
received HAART:
A
HIV VL >100 copies /ml based on 2 consecutive measurements after 3 months with
adherence support
B
HIV VL >1000 copies /ml based on 2 consecutive measurements after 3 months with
adherence support
C
Immunological Failure for Children &lt;5 years: Persistent CD4 levels &lt; 15% or &lt;200
cells/mm3
D Immunological Failure for Children <5 years: Persistent CD4 levels < 15% or<200 cells/mm3
ANSWER:A
1011
Which of the following is NOT true about immediate care of the HIV-exposed newborn
baby?
A HIV DNA-PCR at birth
B HIV antibody test by Elisa
C Infants should be vaccinated as per the Expanded Program on Immunizations (EPI) schedule
D
Nevirapine (NVP) prophylaxis is given at birth to all HIV-exposed infants to reduce MTCT
regardless of maternal ART
ANSWER:B
1012
Which of the following is NOT True about maternal risk factors for Maternal HIV
transmission to the baby?
A Low CD4 count
B High Viral load
C Advanced AIDS
D Prolonged rupture of membranes
ANSWER:A
1013
Which of the following test is likely to make a diagnosis in the first weeks of typhoid fever?
(Single)
A Urine culture

B CSF culture
C Blood culture
D Stool culture
ANSWER:C
1014 Which of the statement is CORRECT in acute glomerulonephritis:
A
A positive throat culture is always support the diagnosis of post-streptococcal
glomerulonephritis
B ASLO is always positive in post-streptococcal glomerulonephritis
C Acute glomerulonephritis commonly caused by group A beta-hemolytic streptococcus
D Red blood cell cast in urine is never found in acute glomerulonephritis
ANSWER:C
1015 Which of the statement is CORRECT in acute glomerulonephritis:
A Complement C3 is normal
B Edema and hypertension are the result of water and sodium retention
C Proteinuria in acute glomerulonephritis has similar characteristic as in nephrotic syndrome
D
Early use of antibiotic in the treatment of streptococcal infection can prevent all acute
glomerulonephritis
ANSWER:B
1016 Which of the statement is CORRECT in acute glomerulonephritis:
A Microscopic hematuria may persist for 1-2 years after the initial presentation
B Urinary protein excretion and hypertension usually normalize by 4–6 months after onset
C Renal dialysis is done around 12% of patients with acute glomerulonephritis
D Renal biopsy should be performed to confirm diagnosis and guide treatment
ANSWER:A
1017 Which of the statement is CORRECT in management of acute glomerulonephritis:
A Steroid may be used in the case of severe edema
B Restriction of salt and water is not necessary
C Ceftriaxone should be used early in streptococcal infection to prevent complication
D Furosemide is safe and effective in acute glomerulonephritis
ANSWER:D
1018 Which of the statement is INCORRECT in acute glomerulonephritis:
A
Acute post-streptococcal glomerulonephritis is caused by direct action of bacteria on
glomerular
B
Edema, hypertension, hematuria, renal insufficiency are the main features finding in acute
glomerulonephritis

C
Gross and microscopic hematuria have same value in making diagnosis of acute
glomerulonephritis
D Group A betahemolytic Streptococcus is the common cause in children
ANSWER:A
1019 Which of the statement is INCORRECT in nephrotic syndrome:
A Prednisolone is effective in minimal change disease nephrotic syndrome
B
Steroid-dependent: Recurrence of proteinuria on alternate-day steroid therapy or within 2
weeks after cessation of treatment
C
Steroid-resistant: Failure to response to initial prednisolone therapy or to 4 weeks of daily
prednisolone for relapse
D Renal transplant is best to perform on children with multiple relapse nephrotic syndrome
ANSWER:D
1020 Which one any stage is the risk factor for Neonatal bacterial infection?
A Term
B Premature
C Low birth Weigh
D Post term
ANSWER:B
1021 Which one cause of measles infection ?
A Bacterial
B Parasite
C Fungal
D Viral
ANSWER:D
1022 Which one cause of whooping cough?
A Bacterial
B Parasite
C Fungal
D Viral
ANSWER:A
1023 Which one is a correct answer of Shock in DHF?
A Cardiogenic shock
B Septic shock
C Hypovolemic shock
D Anaphylactic shock

ANSWER:C
1024 Which one is common severe complication of death in dengue hemorrhagic fever?
A Encephalopathy
B Hyponatremia
C Fluid overload
D Pneumonia
ANSWER:C
1025 Which one is correct answer concerning to vectors for transmission of plasmodium?
A Infected female Aedes mosquito
B Infected female anopheline mosquito
C Infected male Aedes mosquito
D Infected male anopheline mosquito
ANSWER:B
1026 Which one is correct statement?
A
Dengue Hemorrhagic Fever is simple form of dengue infection and is rarely a cause of death
because it doesn’t cause plasma leakage and hemorrhage.
B
Dengue fever is a severe form of dengue infection because it causes plasma leakage and
abnormal hemostasis.
C Dengue fever is higher case mortality rate than Dengue hemorrhagic fever
D Most dengue fever will develop to Dengue Shock Syndrome
ANSWER:B
1027 Which one is the calendar of vaccination of whooping caught?
A At birth, 2 months,3 months
B At birth, 1 month, 2 months
C 1month,2 months, 3 months
D month ½, 2 months1/2, 3 months1/2
ANSWER:D
1028 Which one of coagulation factors is production Vitamin K?
A Factor II,X,XI,XII in the liver
B Factor.V,X,IX,XII in the liver
C Factor III,X,V,VII in the liver
D II,VII,IX,X in the liver
ANSWER:D
1029
Which one of the following is a correct dose of Paracetamol in the guideline in order to
reduce fever in Dengue Hemorrhagic Fever in children?

A 5mg/kg/dose every 6hours
B 10mg/kg/dose every 6hours
C 15mg/kg/dose every 6hours
D 20mg/kg/dose every 6hours
ANSWER:B
1030 Which one of the following, is the correct answer?
A Dengue Fever has the case fatality rate higher than dengue hemorrhagic fever.
B
Dengue Fever is a severe form of dengue infection because it only causes a plasma leakage
syndrome.
C
Dengue Hemorrhagic Fever is a severe form of dengue infection because it only causes a
hemorrhagic syndrome.
D
Dengue Hemorrhagic Fever is a severe form of dengue infection because it causes both
plasma leakage and hemorrhagic syndrome.
ANSWER:D
1031 Which one transmission of whooping cough?
A Blood
B direct face to face
C bad hygiene
D by food
ANSWER:B
1032 Which one varicella infection in pregnancy can lead transmission and infection of fetus
A 26 weeks
B 27weeks
C 28 weeks
D eks
ANSWER:C
1033 Which part of the meninge that locates closest to the brain?
A dura mater,
B skull
C arachnoid mater
D pia mater.
ANSWER:D
1034
Without intervention up to ______% of newborns born to HIV-1-positive mothers are
infected with HIV:
A 25%

B 40%
C 75%
D 100%
ANSWER:B
1035 Worsening of acne in adolescent female on anti-tuberculous treatment is caused by
A isoniazid
B rifampin
C pyrazinamide
D ethambutol
ANSWER:A
1036
A patient with diaphragmatic hernia has respiratory distress at 1/2 hears after birth the
prognosis of this baby could be:
A Good
B Poor
C Very poor
D Fair
ANSWER:C
1037
Acute secondary peritonitis most often results from entry of enteric bacteria into the
peritoneal cavity through a necrotic defect in the wall of the intestines or other viscus. It
MOST commonly follows
A perforation of the appendix
B rupture of a Meckel diverticulum
C intussusception
D peptic ulceration
ANSWER:A
1038 All condition delivery is the cause of perinatal asphyxia, except one
A Cesarian section
B placenta previa
C premature rupture of membranes
D prolapsed cord
ANSWER:A
1039 All is the cause of maternal factor in perinatal asphyxia Except one
A Too young too old
B Anemia
C Family history perinatal asphyxia

D Hypoxia
ANSWER:C
1040 All is the manifestation of congenital infection except one
A Growth retardation
B Septicemia
C Congenial malformation
D Still-birth
ANSWER:B
1041 All is The shunt in fetal circulation except one
A Ductus arteriosus
B Aorta
C Ductus venosus
D Foramen ovale
ANSWER:D
1042 All of this is the cause of DIC of newborn except one
A Asphyxia
B Congenital Heart disease
C Respiratory distress syndrome
D Sepsis
ANSWER:B
1043 All of this is the major sign of rheumatism fever except one
A Fever
B Carditis
C Polyarthritis
D Sydenham’s chorea
ANSWER:A
1044 All the sign is the clinical manifestation of Perinatal asphyxia at birth except one
A Meconium in the amniotic fluid
B B Fluid, electrolyte and metabolic abnormalities
C Hepatomegaly
D Splenomegaly
ANSWER:D
1045
Appendicitis-like symptoms (diarrhea, vomiting, fever, and abdominal pain) may be seen in
the following foodborne bacterial illness
A Vibrio vulnificus

B Yersinia enterocolitica
C Shigella spp.
D Staphylococcus aureus (preformed enterotoxin)
ANSWER:B
1046 Cause of congenial hypothyroism is Except one
A iodine deficiency
B Genetic defects
C Agenesis or dysgenesis of thyroid gland:
D Renal failure
ANSWER:D
1047 Clinical diagnosis of birth asphyxia is, except one
A Abnormal fetal heart rate
B Meconium staining of the liquor
C Cyanosis after birth
D Apgar score < 7 at 5 minutes
ANSWER:C
1048 Clinical manifestation of fetal asphyxia is all except one
A Fetal heart rate tarchycardia to bradycardia
B Amniotic fluid blood
C Meconium staining
D Fetal movement increase to decrease
ANSWER:B
1049
Clinical manifestations of Tetralogy of Fallot: For older children with long-standing cyanosis
who have not undergone surgery may have:
A dusky blue skin,
B gray sclerae with engorged blood vessels, and
C marked clubbing of the fingers and toes.
D All of the above
ANSWER:D
1050
Coarctation of aorta is congenital heat disease that the aorta is narrowed or constricted. The
following statement is correct, EXCEP:
A Usually no symptoms exist at birth, but they can develop as early as the first week after birth.
B In general, cyanosis is developed in the first year of life.

C If the obstruction is mild the heart won’t be very overworked and symptoms may not occur.
D
In some children and adolescents, coarctation is discovered only after high blood pressure is
found.
ANSWER:B
1051 Common factors contributing to a spell (Tet Spell) as below, (EXCEPT):
A Pain
B Crying, upset
C Dehydration
D Hemoconcentration (increased hemoglobin)
ANSWER:D
1052
Concerning to the congenital heart disease with left-right shunt, all the clinical signs (found
by physical examination) are of the following, except:
A In ASD, auscultation of the heart showed systolic murmur with splitting of 2nd heart sound.
B In VSD, The auscultation of the heart showed pan-systolic murmur
C In PDA (persistent ductus arteriosus) showed continuous murmur
D None of the above
ANSWER:D
1053
Concerning to the congenital heart disease with left-right shunt, all the radiography of the
thorax are showed as the following, except:
A Cardiomegaly
B Pulmonary hypervascularisation
C Answer a and b are not correct
D Answer a and b are correct
ANSWER:C
1054
Congenital cardiac defects can be divided into 2 major groups based on the presence or
absence of cyanosis, which can be determined by physical examination aided by:
A Pulse oximetry
B ECG
C Chest X-Ray
D Result of Hemoglobin (CBC)
ANSWER:A
1055 Congenital diaphragmatic hernia refer to:
A Posterolateral

B Morgagni
C Paraoesophagus
D Hiatus
ANSWER:A
1056 Diazepam can be repeated if seizure activity still persist after:
A 5 minutes
B 10 minutes
C 15 minutes
D 20 minutes
ANSWER:B
1057 Filarial larva can be collected from man’s
A Peripheral blood at midnight
B smears of spleen
C smears of intestinal contents
D biopsy of liver
ANSWER:A
1058 Filariasis is a result of the infection of
A Fleas
B Bed bug
C Leech
D Nematode worm
ANSWER:D
1059 Filariform larvae are observed with
A Trichuris trichura
B Necator americanus
C Ascaris lumbricoides
D Enterobius vermicularis
ANSWER:C
1060 Findings that suggest advanced appendicitis on ultrasound include the following EXCEPT
A asymmetric wall thickening
B increased local tenderness to compression
C abscess formation
D associated free intraperitoneal fluid
ANSWER:B
1061 Hirsprung ‘disease also called:

A Congenital ganglionic megacolon
B Congenital aganglionic mega-intestine
C Congenital megacolon
D Congenital aganglionic megacolon
ANSWER:D
1062
In infants and very young toddlers chronic diarrhea can appear following infectious enteritis.
The pathogenesis of the diarrhea is not always clear and may be related to
A food protein allergy
B bacterial overgrowth
C giardiasis
D Strongyloides stercoralis
ANSWER:A
1063 Indication for bag mask ventilation in newborn is except one
A Polypnea
B Apnea
C Heart rate less than 100 bpm
D Rate = 40 – 60 bpm
ANSWER:A
1064 Indication for intubation in newborn is except one
A Prolonged bag and mask ventilation
B Apnea
C Bag and mask is ineffective
D Tracheal suctioning
ANSWER:B
1065 Indication of bag mask ventilation is all excep one
A Apnea
B Heart rate less than 100 bpm
C central cyanotic
D heart rate is : below 60 bpm
ANSWER:D
1066 Management of the diarrhea in chronically malnourished children is based on
A intravenous therapy
B standard osmolarity oral rehydration solutions
C reduced osmolarity oral rehydration solutions
D slow resumption of feeds

ANSWER:C
1067 Medical CAUSES OF RESPIRATORY DISTRESS in newborn is, except one
A Respiratory distress syndrome (RDS),
B Transient tachypnea of newborn
C Pierre Robin Syndrom
D Meconium aspiration Syndrom(MAS),
ANSWER:C
1068
Most complications of congenital cardiopathies with left-to-right shunt as the following,
except one:
A Brain abscess
B Congestive heart failure
C Pulmonary hypertension
D Infective endarteritis
ANSWER:A
1069 Normal full-term infants the passing meconium within:
A 24hears
B 72hears
C 36heras
D 48hears
ANSWER:D
1070 One of all is no minor sign of rheumatism fever, which
A Arthralgia
B Elevated acute phase reactants
C Erythema marginatum
D Prolonged PR interval
ANSWER:C
1071 One of all no manifestation of perinatal infection,which
A Fetal loss
B Meningitis
C Sepsis
D Preterm labor
ANSWER:A
1072 One of all no Postnatal contamination infection , which
A Person to person
B Breast milk

C Transplacental
D Nosocomial infection
ANSWER:C
1073 One of all not diagnosis of perinatal asphyxia Which
A Evidence of fetal distress
B Fetal metabolic acidosis
C Fetal alcalosis
D Abnormal neurological state
ANSWER:C
1074 Poisoning: Activated charcoal should not be given in:
A Diarrhea
B Need immediate antidote
C Paracetamol poisoning
D Antibiotic overdose
ANSWER:B
1075
Prenatal factors associated with a higher incidence of tetralogy of Fallot (TOF) include,
EXCEP:
A Maternal rubella (or other viral illnesses) during pregnancy
B maternal age older than 40 years, and diabetes
C Children with Down syndrome or DiGeorge syndrome
D Children Turner syndrome
ANSWER:D
1076 Relative frequency of major congenital heart lesions listed as below EXCEPT:
A Ventricular septal defect in 8-10%
B Atrial septal defect in 6-8%
C Tetralogy of Fallot in 5-7%
D Coarctation of Aorta in 5-7%
ANSWER:A
1077
Tetralogy of Fallot (TOF) is a cardiac anomaly that refers to a combination of four related
heart defects that commonly occur together. The four defects are: EXCEPT
A Ventricular septal defect (VSD)
B Overriding aorta
C Pulmonary stenosis
D Left ventricular hypertrophy
ANSWER:D

1078 The aganglionic segment is common at:
A Recto- sigmoid
B Recto- rectal
C Ano-rectal
D Colo-sigmoid
ANSWER:A
1079 The below statement is contra-indication for gastric lavage except:
A Recent ingestion of potentially lethal drug
B Recent ingestion of corrosive substance
C Recent ingestion of hydrocarbon
D Decrease consciousness
ANSWER:A
1080 The below statement is incorrect for Atypical Febrile Convulsion:
A Focal seizure
B Duration < 15 minutes
C Repeated seizure
D Seizure in children age 6 months to 5 years
ANSWER:B
1081 The below statement is incorrect for Typical Febrile Convulsion:
A Generalized seizure
B Duration < 15 minutes
C Repeated seizure
D Seizure in children age 6 months to 5 years
ANSWER:C
1082 The causative of Filariasis is
A Schistosome
B Trichinella
C Culex
D Wuchereria
ANSWER:D
1083 The cause of DIC in newborn is,excep one
A Severe viral disease ( Rubella, CMV, Herpese simplex…)
B Asphyxia
C Infection Gram negative, Sepsis (Nesseria meningitidis, streptococcus pneumoniae ….)
D Premature

ANSWER:D
1084 The cause of Perinatal infection (after birth)is, except one
A Trans-placental
B Direct contact
C breast feeding
D nosocomial
ANSWER:A
1085 The clinical presentation of one of the following closely mimics appendicitis
A Meckel diverticulitis
B mesenteric adenitis
C sickle cell disease
D pancreatitis
ANSWER:A
1086
The clinical presentations of an advanced large defect VSD (ventricular septal defect) are
signs of congestive heart failure that showed :
A
Signs of pulmonary venous congestion such as increased respiratory rate, dyspnea with
tachypnea, crackle rales etc.
B
Signs of systemic venous congestion such as edema, hepatomegaly, jugular veins
turgescence etc.
C Signs myocardial dysfunction such as poor skin turgor, oliguria, tachycardia, etc.
D All of the above
ANSWER:D
1087 The clinical presentations of atrial septal defect (ASD) are as the following, except one?
A Recurrent pulmonary infection
B Dyspnea with tachypnea
C Abdominal skin pinch goes back slowly
D Growth retardation
ANSWER:C
1088 The clinical symptoms of patent ductus arteriosus (PDA) are
A Recurrent pulmonary infection
B Dyspnea
C Bounding peripheral arterial pulse
D All of the above
ANSWER:D
1089 The common serious complications of Coarctation of aorta are, EXCEP:

A related to systemic hypertension, which may result in premature coronary artery disease,
B heart failure,
C hypertensive encephalopathy, or intracranial hemorrhage.
D hydrocephaly
ANSWER:D
1090 The constipation in Hirsprung ‘disease always happened at:
A 1 week after birth
B At birth
C 5 days after birth
D 2 weeks after birth
ANSWER:B
1091 The dose of Diazepam for IR route is::
A 0.1mg/kg
B 0.3mg/kg
C 0.5mg/kg
D 0.8mg/kg
ANSWER:C
1092 The factor survival in diaphragmatic hernia is:
A Intestine necrosis
B Pulmonary hyperplasia
C Pulmonary hypoplasia
D Pulmonary edema
ANSWER:C
1093 The first successful surgical procedure to manage Hisprung’ disease described by:
A Swenson
B Duhamel
C Soave
D Paster
ANSWER:A
1094
The following infections may cause malabsorption in immunocompromised children
EXCEPT
A Shigella
B Salmonella
C enteropathogenic E. coli
D Giardia

ANSWER:C
1095 The greatest number of deaths globally caused by a parasite are due to:
A Schistosomiasis
B Malaria
C Filariasis
D Ascariasis
ANSWER:B
1096
The low-osmolality World Health Organization (WHO) oral rehydration solution (ORS) has
the following per liter EXCEPT
A 75 mEq of sodium
B 50 mEq of chloride
C 20 mEq of potassium
D 75 mmol of glucose
ANSWER:B
1097 The main management of asymptomatic hydrocarbon ingestion is:
A Whole bowel irrigation
B Gastric lavage
C Cathetic
D Observe
ANSWER:D
1098 The main treatment that limit absorption of poison is:
A Whole bowel irrigation
B Gastric lavage
C Cathetic
D Absorbent
ANSWER:D
1099 The medical cause of respiratory distress of newborn is all except one
A Pneumothorax
B Respiratory distress syndrome
C congenital heart disease
D Meconium aspiration Syndrom(MAS),
ANSWER:A
1100 The medical treatment of congenital cardiopathies with left-to-right shunt are:
A Reduce preload by using diuretic drug such as Furosemide
B Reduce afterload by vasodilators such as Captopril

C Inotropic (+) drug such as Digoxin
D All of the above
ANSWER:D
1101 The minimal single acute toxic dose of acetaminophen is:
A 50mg/kg
B 100mg/kg
C 150mg/kg
D 200mg/kg
ANSWER:C
1102 The moderate stage of HIE have all the clinical sign except one
A Irritable
B lethargy,
C weak sucking
D often seizure
ANSWER:A
1103 The severe stage of HIE have all the clinical sign except one
A absent muscular tone &amp; reflex
B weak Moro reflex
C coma,
D persistent seizure
ANSWER:B
1104 The Surgical cause of respiratory distress of newborn is all except one
A Diaphragmatic hernia
B intracranial bleed
C Pierre Robin Syndrom
D Lobar emphysema
ANSWER:B
1105 The surgical type of Soave procedure is
A Rectosigmoidectomy
B Rectorectal transanal pull-through
C Endorectal pull-through
D Colostomy
ANSWER:C
1106 The Thyroid production is ,except one:
A The hormone thyroxin (T4)

B Calcitonine
C Insuline
D Triiodothyronin (T3)
ANSWER:C
1107 The thyroid function is except one:
A increase the body ‘s sensitivity to cathecolamine (such as adrenaline )
B Regulate protein , fat and carbohydrate metabolism
C Reduce kalori
D Stimulate vitamins metabolism
ANSWER:C
1108 The types of ASD (atrial septal defect) are below. Which one is correct?
A Ostium Secundum ASD is defect of atrial septum that is commonly occurred.
B Ostium primum ASD is defect of atrial septum that is commonly occurred
C Sinus venosus ASD is defect of atrial septum that is commonly occurred
D All of the above
ANSWER:A
1109
There are many anomalies associated with anorectal malformations. Of the following, the
MOST common are
A kidney anomalies
B cardiac anomalies
C esophageal atresia
D spina bifida
ANSWER:A
1110 This group of congenital heart lesions are classifies for cyanotic: EXCEP
A Tetralogy of Fallot
B Tricuspid atresia
C Coarctation of aorta
D Transposition of great vessels
ANSWER:C
1111 Treatment for most seizure need to be instituted after………of seizure activity:
A > 5 minutes
B > 8 minutes
C > 10 minutes
D > 15 minutes
ANSWER:A

1112 Vitamin B12 and bile salts are only absorbed in
A distal ileum
B duodenum
C proximal jejunum
D colon
ANSWER:A
1113 Which clinical sign of perinatal asphyxia before delivery
A Fetal Heart normal
B Abnormal heart rate or rhythm
C Decrease fetal movement
D Amniotic fluid blood
ANSWER:B
1114 Which cause of Perinatal Hypoxia ischemic encephalopathy
A Sepsis
B Perinatal asphyxia
C Premature
D Malformation
ANSWER:B
1115 Which clinical manifestation of HIE
A Hyperalert
B hypertonia
C coma,
D All of the above
ANSWER:D
1116 Which is a helminth disease
A Polio
B Filariasis
C Filaria
D Diphtheria
ANSWER:B
1117 Which one congenital infection from maternal source
A Exposure to ano-genital secretions and blood
B Direct contact
C Trans-placental
D Exposure

ANSWER:C
1118 Which one don’t cause Associated with antecedent bacterial or viral infection of GBS
A Bacterial infection C.Jejuni
B Bacterial infection Hemophilus influenzae
C Viral infection: Cytomegalovirus,Estein-Barr
D Bacterial infection Streptococcus
ANSWER:D
1119 Which one is not correct in the statements below:
A The common ASD is Ostium secundum
B The common VSD is occurred at inferior portion (muscular septum)
C
Medical closure of PDA by using Prostaglandin inhibitor such as Indomethacin is the method
of choice.
D Most PDA is spontaneously closed
ANSWER:C
1120 Which one is the Classification of HIE
A 2 stages
B 4 stages
C 3 stages
D 5 stages
ANSWER:C
1121 Which one is the diagnosis of Rheumatism fever(Jones criteria,1992 update)
A One major sign
B Two minor sign
C Two major sign
D Two major manifestation or one major + Two minor
ANSWER:D
1122 Which one no high risk delivery of neonatal distress
A Normal fetal heart rate
B Thick meconium after maternal membranes ruptured
C Fetal distress
D Premature birth
ANSWER:A
1123 Which one of the following statement is the best answer?
A ASD (Atrial septal defect) is persistent opening in atrial septum
B VSD (Ventricular septal defect) is defect in ventricular septum

C PDA (Persistent ductus arteriosus) is persistent of ductus arteriosus after birth
D All of the above
ANSWER:D
1124 Which one to prevent perinatal asphyxia
A Regular ante-natal check up
B High risk approach
C Management of Maternal complication
D All of the above
ANSWER:D
1125 WHO recommends to use dextrose to correct hypoglycemia:
A D5% 5ml/kg
B D10% 5ml/kg
C D20% 5ml/kg
D D50% 5ml/kg
ANSWER:B
1126
A 1-year-old child has folic acid deficiency since the age of 4 months, the best indicator of
this deficiency is
A significant increment of mean corpuscular volume MCV
B significant fall of reticulocytes count
C high level of lactate dehydrogenase LDH
D decrease level of RBC folate
ANSWER:D
1127
A 6-year-old boy is brought to the pediatrician because of a 3-day history of skin lesions. On
physical examination, he has multiple yellow, crusted erosions below the nares and on the
cheeks, chin, and upper extremities. The rest of the examination is normal. Which of the
following is the most appropriate treatment for this condition?
A Oral acyclovir
B Oral amoxicillin
C Oral cephalexin
D Topical ketoconazole
ANSWER:C
1128 A Trichinellosis diagnose by antibody detection the right time to perform this test is:
A 1week after infection
B 3weeks after infection
C 6week after infection

D 2week after infection
ANSWER:B
1129 About Breakthrough Varicella. All the following are true EXCEPT
A
is varicella that occurs in a person vaccinated &gt;42 days before rash onset and is caused by
wild-type VZV.
B The rash in breakthrough disease is frequently atypical and predominantly maculopapular
C Patients typically are afebrile or have low grade fever
D Patients develop mild skin lesion around 100 to 200 lesions.
ANSWER:D
1130 Albendazole is effective to treat Trichinellosis in the:
A Muscle stage
B Stomach stage
C Gastro intestinal stage
D All of the above
ANSWER:C
1131 All are Discharge Planning and Preparation,excep one :
A Sleep and activity
B General instructions for newborn care
C Kangaroo Mother care
D Newborn screening and immunization Program
ANSWER:C
1132 All below are the Benefit of breast-feeding for mothers except:
A Strong emotion bond between mother and baby
B The child growth healthier
C Contraction of uterus
D Reduce the risk of hemorrhage
ANSWER:B
1133 All is Nursing Plan and Implementation except one
A Maintenance of cardiopulmonary function
B Maintenance of a neutral thermal environment
C Phototherapy
D Promotion of adequate hydration and nutrition
ANSWER:C
1134 All of statement below are true, except:

A
Brain swelling may cause the brain push down on the brain stem and can damage the
reticular activating system
B
Bleeding in the layers of the brain may cause coma due to swelling and compression on the
injured side of the brain
C Serum level of Ammonia increased that cause coma is due to renal disease
D
Cardiac arrest causes a sudden cutoff of blood flow and oxygen to the brain and cause
hypoxia or anoxia
ANSWER:C
1135 All the cause is Transition begins before delivery except one:
A Maternal health
B Premature
C Gestational age
D physical defects/anomalies
ANSWER:B
1136 All the following are clinical features of typhoid fever EXCEPT
A incubation period is usually 30-45 days
B
macular or maculopapular rash (rose spots) may be visible around the 7th-10thday of the
illness
C
if complications not occur, the symptoms and physical findings gradually resolve within 2-4
wk
D typhoid fever usually manifests as high-grade fever with a wide variety of associated features
ANSWER:A
1137 All the following are true about Varicella in unvaccinated individuals EXCEPT:
A Simultaneous presence of lesions in the same stages of evolution
B Distribution of rash is predominantly central
C Many children have vesicular lesions on the eyelids and conjunctivae
D
The average number of varicella lesions is about 300, but healthy children may have fewer
than 10 to more than 1,500 lesions.
ANSWER:A
1138 All the following can cause human amoebic meningoencephalitis EXCEPT
A Naegleria
B Acanthamoeba
C Balamuthia
D Entamoeba histolytica

ANSWER:D
1139 All the following regarding assessment and care of infants with pertussis are true EXCEPT
A infants with potentially fatal pertussis may appear well between episodes
B a paroxysm must be witnessed before a decision is made between hospital andhome care
C suctioning of nose, oropharynx, or trachea should be performed on apreventive schedule
D feeding in the period following a paroxysm may be more successful than afternapping
ANSWER:C
1140 All the following statements regarding giardiasis are true EXCEPT
A incubation period of Giardia infection usually is 1-2 wk
B most infections are asymptomatic
C symptomatic infections occur more frequently in adults than in children
D stools do not contain blood, mucus, or fecal leukocytes
ANSWER:C
1141 Among answer is Characteristics of Newborn respirations except one
A 40-60 rate
B pH=7
C shallow and irregular
D nose breathers
ANSWER:B
1142
Among the serogroups of Neisseria meningitidis given, which of these are the most
important associated with meningococcal infection?
A A, B, C, Y and W135
B A, B, C, Y and W125
C A, B, C, X, Y and W125
D A, B, C, X and W135
ANSWER:A
1143 At-risk indicators for Cardiopulmonary is except one :
A Pallor
B Cyanosis
C Vomiting
D apnea.
ANSWER:C
1144 Brain is consider………………..cholesterol in food:
A Low
B Medium

C High
D Very high
ANSWER:C
1145 Carbohydrate Metabolism in newborn is, except one
A Glucose is the main source of energy in the first 4 to 6 hours following birth
B Blood glucose level stabilizes at values of 50 to 60 mg/dL.
C Blood glucose level stabilizes at values of 20 to 40 mg/dL.
D
Energy crunch occurs at birth with the removal of maternal glucose supply and increased
energy expenditure adjusting to extrauterine life.
ANSWER:C
1146 Characteristics of DNA or RNA are often different. Statements below are correct, EXCEP:
A DNA is a double-stranded molecule
B RNA is a single stranded molecule
C DNA has 4 nitrogen bases: adenine, cytosine, guanine and thymine
D RNA also has 4 nitrogen bases: adenine, cytosine, thymine and uracil.
ANSWER:D
1147 Choose the best answer. Coma will result:
A Either reticular activating system and cortical dysfunction
B Only reticular activating system dysfunction
C Only cortical dysfuntion
D Both reticular activating system and cortical dysfunction
ANSWER:A
1148
Coma develops over several hours and manifested irritability with fever and rash, we should
be considered to?
A Meningitis or encephalitis
B Severe hypoglycemia
C Diabetic ketoacidosis
D Head trauma
ANSWER:A
1149 Coma in children caused by metabolic disorder as the following, except:
A Hypoglycemia
B Renal failure
C Hepatic failure
D All of the above
ANSWER:D

1150 Common problem in transition is,except one:
A Birth Asphyxia
B Infection
C section
D Metabolic Problems
ANSWER:C
1151
Complications of reseola infection. Convulsions are the most common complication of
roseola and are recognized in up to one third of patients. With a peak age at 12-15 months,
seizures are also the most common complication of children with primary HHV-6 infection,
occurring approximately in:
A <5%
B 15%
C 50%
D 75%
ANSWER:B
1152 Concerning to severe and complicated malaria, any one of the following is true, except:
A Serum glucose level is 4.6 mmol/L
B Spontaneous bleeding that is indicated to disseminated intravascular coagulopathy
C Severe anemia with hemoglobin level less than 5g/dl
D Repeated generalized convulsion
ANSWER:A
1153 Consciousness requires as the following. Choose the best answer.
A
Arousal or wakefulness that is dependent on the function of the Reticular activating System
(RAS)
B
Awareness, which is mediated through cerebral cortical neurons and their reciprocal
projections to and from the major subcortical nuclei.
C Both arousal (wakefulness) and awareness
D None of the above
ANSWER:B
1154
Ebola can ONLY spread between humans by direct contact with infected body fluids
including but not limited to urine, saliva, sweat, feces, vomit, breast milk, and semen. All the
following statements are true EXCEPT
A It is thought that fruit bats of the Pteropodidae family are natural Ebola virus hosts.
B
Since then, several small outbreaks have occurred in Africa, the 2014-2015 outbreak is the
largest

C The countries affected in this outbreak include: South Africa and East Africa
D
The time between exposure and when symptoms occur (incubation periois 2 to 21 days. On
average, symptoms develop in 8 to 10 days.
ANSWER:C
1155 Examination on pupils suggested Hypoxic-ischemic injury in coma in children:
A symmetric dilated pupils that may not respond to light
B symmetric dilated pupils that react to light
C asymmetric pinpoint pupils that may not respond to light
D asymmetric pinpoint pupils that may respond to light
ANSWER:A
1156 Folic acid deficiency may develop in the following children EXCEPT
A a term infant fed on goat`s milk
B an infant fed on unfortified powdered milk
C an infant fed on pasteurized cow's milk
D a child with chronic use of phenytoin anticonvulsant
ANSWER:C
1157 Fragmentation hemolysis by mechanical injury may be seen in all the following EXCEPT
A extensive burns
B Kasabach-Merritt syndrome
C after cardiac surgery for prosthetic heart valve replacement
D thrombotic thrombocytopenic purpura (TTP)
ANSWER:A
1158 Full cream milk should be given to children age:
A From 3- 6 months
B From 6-9 months
C From 9-12 months
D Over 1 year old
ANSWER:D
1159 Giardiasis should be considered in children who have the following presentations EXCEPT
A acute dysenteric diarrhea
B persistent diarrhea
C failure to thrive
D malabsorption
ANSWER:A
1160 Herpes whitlow is HSV infect of the

A Eye
B Mouth
C Fingers
D Ear
ANSWER:C
1161
High fiber dietery food such as wholemeal bread, brown rice,….. could be gradually
introduced into children’s diet at the age:
A 1 year and older
B 2 years and older
C 5 years and older
D 7 years and older
ANSWER:A
1162 Highly allergenic food such as seafoodfood, egg white,….. may introduce to children at age:
A From 6 months
B From 12 months
C From 2 years
D From 5 years
ANSWER:B
1163
Hospital discharge of infants with pertussis is appropriate in all the following circumstances
EXCEPT
A over a 24-hr period disease severity is unchanged or diminished
B intervention is not required during paroxysms
C nutrition is adequate
D no complication has occurred
ANSWER:A
1164
Human herpesvirus 6 and human herpesvirus 7 cause infection in infancy and early
childhood. It is responsible for the majority of cases of reseola infantum. All the following
are true EXCEPT
A 95% of children being infected with HHV-6 by 2 yr of age.
B Infection with HHV-7 is also widespread but occurs later in childhood and at a slower rate,
C it is characterized by the abrupt onset of high fever, which may be accompanied by fussiness,
D The fever usually resolves a few day after the rash disaapeared.
ANSWER:D

1165
Human Herpesviruses: Of the more than 100 known herpesviruses, currently only
#…..routine infect only humans ( called human herpesviruses).
A 6
B 7
C 8
D 9
ANSWER:C
1166 If a child suspected in shock, we should be checked the child as the following:
A Pulse rate
B Temperature
C Capillary refill time (CRT)
D All of the above
ANSWER:D
1167 Immunity against common viral infections such as measles is :
A may last 4 to 8 months
B may last 2 to 3 months
C may last 1 to 2 months
D may last 9 to 11 months
ANSWER:A
1168 In coma in children, odor of exhaled breath may help in making etiologic diagnosis, except:
A sweet fruit may be suggested to diabetic Ketoacidosis
B urine like smell may be suggested to increased uremia caused by renal failure
C ammoniac odor (musty) may be suggested to hepatic coma
D none of the above
ANSWER:D
1169
In coma in children, the following investigations should be done. Which one should not be
done?
A Complete blood count (CBC) if coma with fever
B Lumbar puncture should be done if patient having signs of increased intracranial pressure
C Skull CT scan if having recent head trauma
D Serum level of creatinine and blood urea nitrogen if suspected to renal failure
ANSWER:B
1170 In hemophilia A, factor VIII level activity should be increased to 100% in
A epistaxis

B iliopsoas bleeding
C tooth extraction
D hematuria
ANSWER:B
1171 In meningitis Cerebro spinal fluid(CSF) is routinely culture in:
A Chocolate agar
B Blood agar
C A and B
D Specific media
ANSWER:C
1172 Increased intracranial pressure (IIP) in neonate is suggested by:
A bulging fontanel
B separation of sutures
C Sunsetting of the eyes
D All of the above
ANSWER:D
1173
Infants younger than 3 mo of age with suspected pertussis usually are admitted tohospital, as
are many between 3 and 6 mo of age unless witnessed paroxysms are not severe, as well as
are patients of any age if significant complications occur.Typical paroxysms that are not life
threatening have the following feature
A duration >45 sec
B blue color change
C bradycardia <60 beats/min in infants
D oxygen desaturation that spontaneously resolves at the end of the paroxysm
ANSWER:D
1174
Isoniazid is accompanied by significant drug–drug interactions, which of the following is
TRUE?
A aluminum salts increase absorption of isoniazid
B isoniazid increase toxicity of carbamazepine
C rifampin decreased hepatotoxicity of isoniazid
D isoniazid decrease level of warfarin
ANSWER:B
1175
JE is a mosquito-borne viral disease of humans as well as horses, swine, and other domestic
animals. All the following statements are true EXCEPT
A There are 4 main genotypic variants of JEV (JEV type I, II, III and IV)

B Transmitted by culex mosquito, which breeds in rice fields
C
Culex mosquito prefer to bite outdoors and are extremely active in the day time (not in the
nigh time)
D Pigs and aquatic birds (eg, egrets, herons) serve as amplifying hosts.
ANSWER:C
1176 Kernig and Brudzinski signs positive suggested to:
A Typhoid fever
B Pneumonia
C Meningitis
D Urinary Tract Infection
ANSWER:C
1177 Low-fat milk should be given to children age:
A From 6-12 months
B From 12-24 months
C Over 24 months
D Any age
ANSWER:C
1178 Malaria is:
A Bacterial infection
B Viral infection
C Protozoan Parasitic infection
D Fungal infection
ANSWER:C
1179 Management of Meconium stained amniotic fluid is except
A Airway and ventilatory support
B Circulatory support :Assure adequate perfusion
C Intubation
D Hypothermia prevention
ANSWER:C
1180 Mature milk is produced from about ……. after birth:
A 5 days
B 7 days
C 10 days
D 14
ANSWER:D

1181 MOST Campylobacter isolates are susceptible to
A aminoglycosides
B cephalosporins
C rifampin
D penicillins
ANSWER:A
1182
Most of clinical features of neonatal meningitis manifested as the following are not true.
Which one is true?
A High fever more common
B Neck stiffness
C Bulging fontanel
D Positive Kernig and Brudzinski signs
ANSWER:C
1183 Necrotizing fascitis is a:
A Fungal Infection
B Bacterial Infection
C Viral Infection
D None of the above
ANSWER:B
1184 One of the following statements regarding Giardia lamblia is TRUE
A life cycle is composed of 3 stages
B each ingested cyst produces 2 trophozoites in the caecum
C trophozoites contain 4 oval nuclei anteriorly
D
cyst viability is not affected by the usual concentrations of chlorine used to purify water for
drinking
ANSWER:D
1185 Opiate overdose symptoms and signs show as the following, except:
A decreased level of consciousness
B dilated pupils (mydriasis)
C Breathing slow down
D Heart rate slow down
ANSWER:B
1186 Pallor may be an early sign
A of hemorrhage
B of Anemia

C of Congenital Heard Disease
D of Cyanotic cardiopathy
ANSWER:A
1187
Patients with conditions leading to iron overload are at higher risk of developing infections
with
A Aeromonas
B Pseudomonas aeruginosa
C Yersinia
D Francisella tularensis
ANSWER:C
1188 Period of production of “Foremilk” is:
A Late pregnancy
B End of feed
C Beginning of feed
D Middle of feed
ANSWER:C
1189 Period of production of “Hindmilk” is:
A Late pregnancy
B Beginning of feed
C Middle of feed
D End of feed
ANSWER:D
1190 Production period of Colostrum:
A First 24 hours
B First 48 hours
C Late pregnancy until the first few days after delivery
D First week
ANSWER:C
1191 Raised intracranial pressure. Which one is the wrong answer?
A Hydrocephalus
B Cerebral abscess
C Apnea
D Cerebral tumor
ANSWER:C

1192
Rotaviruses are in the Reoviridae family and cause disease in virtually all mammals and
birds. All the following statements are true EXCEPT
A Infection typically begins after an incubation period of &lt;48 hr (range 1-7 days)
B Fever, vomiting and frequent watery stools are present in about 50-605 of cases.
C Vomiting and fever typically abate after the 4th day of illness
D Dehydration may develop and progress rapidly particularly in infants
ANSWER:C
1193 Sedative/Hypnotics drugs include:
A Benzodiazepines
B Barbiturates
C Chloral hydrates
D All of the above
ANSWER:D
1194 Skimmed milk should be given to children age:
A Any age
B Over 1 year
C Over 2 years
D Over 5 years
ANSWER:D
1195
Some viruses are oncogenic and predispose to certain cancers. Viral infection below can
cause cancers, EXCEPT:
A Epstein-Barr virus.
B Papillomavirus
C Mumps virus
D Hepatitis B or C viruses
ANSWER:C
1196 Term newborns normally pass meconium (dark green to black) like , except one
A within 8 to 24 hours of life
B within 3 to 6hours of life
C almost always by 48 hrs.
D At birth
ANSWER:B
1197 The below food should avoid giving to the child below 1 year of age:
A Egg yolk
B Apple

C Soy bean
D Salt and sodium rich food
ANSWER:D
1198 The below food should avoid giving to the child below 1 year of age:
A Banana
B Honey
C Olive oil
D Egg yolk
ANSWER:B
1199 The best assessment of iron overload for patients with thalassemia major is achieved by
A liver MRI
B bone marrow biopsy
C serum iron
D serum ferritin
ANSWER:A
1200
The best treatment for spinal cord compression at the vertebral canal with neurological
symptoms caused by extramedullary hematopoiesis in thalassemic patients is
A blood transfusion
B splenectomy
C iron chelation therapy
D local radiotherapy
ANSWER:D
1201 The classic physical examination findings in patient with mononucleosis are, EXCEPT.
A generalized lymphadenopathy (90% of cases),
B splenomegaly (50% of cases),
C hepatomegaly (10% of cases)
D purpura or petechiae (50% of cases)
ANSWER:D
1202 The common vector of C Trachomatis is:
A Mosquito
B Horse
C Dog
D Flies
ANSWER:D
1203 The component of primary assessment in coma in children:

A Evaluation of airway status
B Evaluation of breathing
C Evaluation of circulation
D All of the above
ANSWER:D
1204
The criteria for successful bone marrow transplantation in thalassemic patients include all the
following EXCEPT
A age younger than 15-year-old
B no hepatomegaly
C no bone deformities
D no iron overload
ANSWER:C
1205 The diagnosis of trichinellosis is confirm by:
A Stool examination
B Serology
C CBC
D Stool culture
ANSWER:B
1206 The Fetal Shunts is, Except one
A Ductus Venosus- hepatic system
B Foramen Ovale- between right &amp; left atrium
C Ductus Arteriosus- vein connects pulmonary artery to descending aorta
D Atrial septal defect
ANSWER:D
1207 The first laboratory marker in progressive iron deficiency anemia is
A depletion of bone marrow hemosiderin
B falling of serum ferritin
C decrease of serum iron and increase of the iron-binding capacity
D decrease hemoglobin synthesis
ANSWER:A
1208 The first priorities antiseptic to use in blood culture is:
A Alcohol
B Chlorhexidine
C A or B
D Iodine

ANSWER:B
1209 The following is the cause of cerebral hypoxemia or ischemia, except:
A Carbon monoxide poisoning
B Drowning
C Respiratory failure
D Head trauma
ANSWER:D
1210 The following statements are true, except:
A
Alert (Consciousness) is an appearance of wakefulness and awareness of the self and
environment.
B Lethargy is the state of a mild reduction in alertness
C Coma (Unconscious) is sleep like appearance and unresponsive to all external stimuli.
D None of the above
ANSWER:D
1211
The genus Yersinia is a member of the family Enterobacteriaceae and comprisesmore than
14 named species, 3 of which are established as human pathogens.Which of the following is
MOST often associated with mesenteric lymphadenitis?
A Yersinia enterocolitica
B Yersinia pseudotuberculosis
C Yersinia pestis
D Yersinia mollaretii
ANSWER:B
1212 The Glasgow Coma Scale means:
A Look at the ability to see (eye opening)
B Recognize visual stimuli , make sounds (verbal response)
C Control motor movements.
D All of the above
ANSWER:D
1213 The hallmarks of common HSV infections are:
A skin vesicles and shallow ulcers.
B skin petechia and purpura manifestation
C skin rash and itching
D exanthematous
ANSWER:A
1214 the history Considerations for Newborns in Transition Period is, except one

A Maternal…Medications
B Labor and Delivery
C physical defects/anomalies
D Resuscitation Measures
ANSWER:C
1215 The ideal agent for treating fungal urinary tract infections is
A amphotericin B
B fluconazole
C voriconazole
D micafungin
ANSWER:B
1216 The largest protozoan that parasitizes human is
A Entamoeba histolytica
B Giardia lamblia
C Balantidium coli
D Isospora belli
ANSWER:C
1217 The larva of Trichinellosis travel to blood at:
A large intestine
B Small intestine
C Intestine
D Rectum
ANSWER:B
1218 The leading protozoal cause of diarrhea in children worldwide is
A Entamoeba histolytica
B Giardia lamblia
C Balantidium coli
D Isospora belli
ANSWER:D
1219 The least common clinical feature of typhoid fever in children is
A diarrhea
B abdominal pain
C pallor
D splenomegaly
ANSWER:D

1220 The main stay of treatment for congenital hypoplastic anemia is
A corticosteroids
B androgen
C antithymocyte globulin (ATG)
D fully matched-related stem cell transplantation
ANSWER:A
1221 The meningococcal infections are predominant in which of the following age group?
A Infants only
B Children and Young adults
C Adults only
D Children and Adults
ANSWER:B
1222 The most appropriate of urine sample collection is:
A Suprapubic aspiration
B Urinary catheter
C Collecting clean-voided midstream specimens
D Simple urinary collection
ANSWER:A
1223 The most common cause of C Trachomatis is:
A Salpingitis in women
B Uretritis in men
C Pneumonia in infants
D Conjunctivitis
ANSWER:D
1224 The MOST common cause of death in kala azar is
A severe anemia
B bleeding
C secondary bacterial infection
D hepatic failure
ANSWER:C
1225 The MOST common complication of measles is
A aseptic meningitis
B pharyngitis
C otitis media
D pneumonia

ANSWER:C
1226 The MOST common complication of mumps is
A meningitis
B conjunctivitis
C optic neuritis
D pneumonia
ANSWER:A
1227 The MOST common complication of Y. enterocolitica infection in younger children is
A reactive arthritis
B erythema multiforme
C hemolytic anemia
D thrombocytopenia
ANSWER:D
1228 The MOST common finding among infants with congenital rubella syndrome is
A psychomotor retardation
B cataracts
C deafness
D patent ductus arteriosus
ANSWER:C
1229 The MOST common reason for which children receive antibiotics is
A pneumonia
B bronchitis
C otitis media
D pharyngitis
ANSWER:C
1230 The most common site of herpes labialis is
A Eye
B Abdomen
C Fingers
D Ear or Chin
ANSWER:D
1231 The preferable antibiotic in neonate with pertussis is
A azithromycin
B erythromycin
C clarithromycin

D trimethoprim-sulfamethoxazole
ANSWER:A
1232 The primary management of coma in children
A Secure the airway
B Maintain appropriate breathing
C Maintain circulation
D All of the above
ANSWER:D
1233 The transportation urine to laboratory is could be:
A less than 3 hears
B less than 4 hears
C less than 2 years
D As rapid as possible and less than 2hears
ANSWER:D
1234 The trichinellodis larva release from cyst under influence by:
A Acide chloride
B Pepsinogene
C Acid pepsin in intestine
D Acid pepsin in gastrointestinal
ANSWER:D
1235 The trichinellosis larva was killed at:
A >50oC
B >131oF
C >123oF
D >45oF
ANSWER:B
1236 These below food should avoid giving to the child below 1 year of age except:
A Small, hard food
B Slippery food
C Sticky food
D Egg yolk
ANSWER:D
1237 Tissue necrosis distinguishes them from cellulitis:
A Children are get higher fever in Tissue necrosis
B Tissue necrosis are bigger in size

C
In cellulitis, an inflammatory infectious process involves subcutaneous tissue but does not
destroy it. Necrotizing soft tissue infections characteristically manifest with a paucity of early
cutaneous signs relative to the rapidity and degree of destruction of the subcutaneous tissues.
D Tissue necrosis are red and harder than cellulitis
ANSWER:C
1238 Tongue and heart are consider………………..cholesterol in food:
A Low
B Medium
C High
D Very high
ANSWER:B
1239 Treatment For gingivostomatitis due to HSV:
A Oral acyclovir (10 mg/kg/dose 4 times a day PO for 7 days)
B Oral acyclovir (15 mg/kg/dose 5 times a day PO for 7 days)
C Oral acyclovir (25 mg/kg/dose 5 times a day PO for 7 days)
D Oral acyclovir (35 mg/kg/dose 3 times a day PO for 10 days)
ANSWER:B
1240 Treatment of mononucleosis by EBV. All the following statements are true, EXCEPT:
A There is no specific treatment for infectious mononucleosis
B
Therapy with high doses of acyclovir, with or without corticosteroids, decreases viral
replication and oropharyngeal shedding during the period of administration but does not
reduce the severity or duration of symptoms or alter the eventual outcome
C
The courses of corticosteroids for duration of 4 weeks may be helpful for complications of
infectious mononucleosis
D A recommended regimen is prednisone 1 mg/kg/day for 7 days followed by a dosage taper
ANSWER:C
1241 Treatment of Varicella. All the following are true EXCEPT:
A
Oral therapy with acyclovir (20 mg/kg/dose) given as 4 doses/day for 5 days can be used to
treat uncomplicated varicella.
B
Antiviral drug resistance is rare but has occurred in children with HIV infection who have
been treated with acyclovir for extended periods.
C Foscarnet is the only drug available for the treatment of acyclovir-resistant VZV infections.

D
Acyclovir 25mg/kg IV every 8 hr, should be initiated within 72 hr of development of initial
symptoms decreases the likelihood of progressive varicella and visceral dissemination in
high-risk patients
ANSWER:D
1242
Tularemia is a zoonotic infection caused by the gram-negative bacterium Francisella
tularensis.Of the following, the MOST common forms of tularemia diagnosed in children is
A ulceroglandular
B pneumonia
C oropharyngeal
D oculoglandular
ANSWER:A
1243
Types of Viral Disorders: Categorizing viral infections by the organ system most commonly
affected. Which organ is the most common infected?
A Respiratory system
B Gastro-intestinal system
C Central nervous system
D Hepatic system
ANSWER:A
1244 Typically, the first sign of infants with respiratory syncytial virus (RSV) is
A cough
B rhinorrhea
C low-grade fever
D increases respiratory rate
ANSWER:B
1245
Varicella-zoster virus (VZV) causes primary, latent, and recurrent infections. All the
following statements are true EXCEPT
A Varicella is a serious disease in young infants
B Within households, transmission of VZV at a rate of 65-85%
C Herpes zoster is very rare in healthy children younger than 15 y of age
D Herpes zoster is more common in winter
ANSWER:D
1246
Varicella is an acute febrile rash illness tat is common in developing countries. Of the
following, the TRUE statement is:
A It usually begins 4-7 days after exposure

B Mild abdominal pain may occur 24-48 hours before the rash appears.
C Subclinical varicella is common
D Temperature elevation (40-41oC) usually after the onset of rash
ANSWER:B
1247 We should take energy from carbohydrate….% of total energy:
A 30-40%
B 40-50%
C 50-60%
D 60-70
ANSWER:C
1248 What is the drug of choice for the treatment of meningococcal disease?
A Penicillin
B Amphotericin
C Methicillin
D Penicillin G
ANSWER:D
1249 What percent of the world’s population infected by EBV?
A 5%
B 25%
C 50%
D 75%
ANSWER:D
1250 Which antibiotics should be avoided in patients with botulism?
A penicillin
B cephalosporin
C macrolide
D aminoglycoside
ANSWER:D
1251 Which are not complication of Impertigo
A osteomyelitis, septic arthritis, pneumonia, and septicemia
B Cellulitis
C
Lymphangitis, suppurative lymphadenitis, guttate psoriasis, and scarlet fever occasionally
follow streptococcal disease.
D Angioedema
ANSWER:D

1252 Which are not virulence factors of S. aureus
A protect the organism from host defenses (slime layer, coagulase, penicillinase or β-lactamase)
B localize infection
C polysaccharide capsule for protection
D cause local tissue damage,
ANSWER:C
1253 Which condition of infant place of newborn high risk asphyxia
A Breech or abnormal
B Macrosomia
C Fetal growth retardation
D All of the above
ANSWER:D
1254 Which condition of mother place of newborn high risk asphyxia
A Diabetes Mellitus
B Anemia
C PROM with evidence of amnionitis
D All of the above
ANSWER:D
1255 Which drug is the most effective against Methyl resistant Staphylococcus aureus?
A Vancomycin
B Penicillin
C Cephalosporins
D Oxacillin
ANSWER:A
1256 Which is the treatment of Necrotizing soft tissue infections?
A Antibiotic alone
B Supporting care
C Antibiotic with antifungal
D surgical debridement, and parenteral antibiotic
ANSWER:D
1257 Which method is not the treatment of impertigo
A Topical therapy with mupirocin 2%, fusidic acid, and retapamulin 1%
B Oral corticosteroid
C Systemic therapy with oral antibiotics

D Cephalexin, 25-50 mg/kg/day
ANSWER:B
1258 Which of the following statements are false in regards to impetigo/ bacterial skin infection?
A Streptococcal skin infection is usually more superficial than staphylococcal impetigo
B Frequently complicates other skin infections such as scabies
C Perianal streptococcus may present as constipation.
D Impetigo is not common skin infection in children
ANSWER:A
1259 Which one Common Problems Seen In Newborns in Transition Period:
A Birth Asphyxia
B Metabolic Problems
C Infection
D All of the above
ANSWER:D
1260 Which one don t Characteristics of Cardiac Function Heart rate of newborn
A 120 to 160
B 100-120
C when the newborn cries the heart rate may exceed 180
D Murmurs are usually produced
ANSWER:B
1261 Which one initial Assessment of the newborn at birth
A Tone
B Breathing
C Heart rate
D All of the above
ANSWER:D
1262 Which one is a temperature of newborn
A 36.5-37.3c°.
B 34-36
C 34-35
D 33.3-35.5
ANSWER:A
1263 Which one is the best answer for cause of coma in children?
A Trauma
B Cerebral hypoxemia/ischemia

C Toxic
D Status epilepticus
ANSWER:B
1264 Which one Newborns have maternally induced immunity
A Tetanus
B diphtheria,
C measles
D All of the above
ANSWER:D
1265 Which one of organism is not the cause of neonatal meningitis:
A Neisseria meningitidis
B Listeria monocytogenes
C Group B streptococcus
D E coli
ANSWER:A
1266 Which one of the following is not true?
A Extreme pallor may be caused by severe anemia or shock
B Jaundice indicated to renal failure
C Cherry red skin suggested to CO poisoning
D Maculo-papular rash with necrosis suggested to meningococcemia
ANSWER:B
1267 Which one risk factor for baby need resuscitation newborn
A Multiple gestation (e.g. twins, triplets)
B Post term gestation (greater than 41 weeks)
C Large for dates
D All of the above
ANSWER:D
1268 Which one Stabilization of the Transitioning Newborn:
A Temperature
B Sugar
C Emotional Support for the Family
D All of the above
ANSWER:D
1269 Which one Surfactant synthesis and storage begins to occur
A Between 24-28 weeks

B Between 20-23 weeks
C Between 30-31 weeks
D Between 34-37 weeks
ANSWER:A
1270 Worsening of acne in adolescent female on anti-tuberculous treatment is caused by
A isoniazid
B rifampin
C pyrazinamide
D ethambutol
ANSWER:A
1271 The MOST common cause of under-5 mortality in developing world is
A diarrheal disease
B pneumonia
C malaria
D measles
E neonatal disease
ANSWER:E
1272 The LEAST common cause of death < 1 yr in USA is
A congenital malformations
B short gestation and low birth weight
C sudden infant death syndrome
D unintentional injuries
E bacterial sepsis of newborn
ANSWER:E
1273 In mental retardation, all the following are true EXCEPT
A constitute 1-3% of children
B 80% are mild form
C in severe form socio-economic status is a considerable factor
D higher percentage in low birthweight babies
E fragile X chromosome is a recognized cause
ANSWER:C
1274
An 8-month-old infant with confirmed diagnosis of spinal muscular atrophy type 1 referred
back from tertiary care center with "DNAR" status. This means during acute illness or an
emergency the child should
A receive standard care and adrenaline injection

B receive supportive care
C receive active resuscitation but not mechanical ventilation
D O2 is not part of treatment options
E none of the above
ANSWER:B
1275
A full term neonate with Down syndrome and esophageal atresia admitted in Pediatric
intensive care unit (PICU). The parents want “no interference" approach.Of the following,
the BEST response is to
A act according to parents well
B arrange the operation with the surgeon
C work for the patient best interest
D give supportive care only (DANR)
E wait for a clear legislation
ANSWER:C
1276
A 7-year-old boy presented to the out-patient department with fever and cough, you have
diagnosed him as a case of pneumonia and decided to admit and give parental therapy, the
child refused. Of the following, the BEST response is to
A explain to father about dangers of omitting treatment
B try to explain the risk to the child
C give the chance for oral antibiotics and accept the child decision
D inform your consultant
E inform hospital ethical committee
ANSWER:C
1277
You are treating acutely ill 6-year-old child with ascending paralysis, he eventually needs
mechanical ventilation. Now you are in a situation of using the last bed in respiratory care
unit (RCU). Of the following, the BEST approach is to
A try to convince the parents that, there is no use of ventilation
B keep the bed for more treatable condition
C try bedside ventilation
D consider this case as potential treatable condition and use the last bed
E contact a nearby region RCU and transfer the child
ANSWER:D

1278
You are managing a 10-year-old child with spastic quadriplasia, who is globally retarded,
had recurrent seizures, and severe recurrent chest infections which requires frequent
admissions to respiratory care unit (RCU) and ventilation. You are trying to convince the
mother about non-benefit of future RCU admissions. All of the following statements are true
EXCEPT
A use an obligations of no bed availability
B consider mother emotional status and religious believes
C be empathetic
D explain disease chronicity and non-response status
E explain the suffering of child in each ventilation
ANSWER:A
1279
The well-child care (anticipatory guidance) intends to promote the physical and emotional
well-being of children. The tasks of each well-child visit (which usually takes 18 min time)
include all the following EXCEPT
A disease detection
B disease prevention
C treatment plans
D health promotion
E accident prevention
ANSWER:C
1280 After the first year of life, the MOST common cause of death in children is mainly
A none-accidental injuries
B burn
C drowning
D accidents
E war
ANSWER:D
1281 The MOST successful injury prevention strategy is
A car seats
B water safety
C changing in product design
D poison prevention
E fall prevention
ANSWER:C

1282
Around 44,000-98,000 patients die in U.S. hospitals each year because of preventable
medical errors, these figures can be reduced if adapting an emerging new science of Quality
Improvement (QI). The TRUE figure of children receive recommended health care in USA
is about
A 16%
B 26%
C 36%
D 46%
E 80%
ANSWER:D
1283
Patient culture can change the doctor treatment options and even the way of behavior with
the patient. All the following culture values are expected if he is dealing with Muslim
patientsEXCEPT
A fasting during Ramadan month
B modesty and female body cover
C forbiddance of touch and shake hands of opposite sex
D routine postmortem exam
E patriarchal way of decision making
ANSWER:D
1284 In embryonic period, all the following are true EXCEPT
A formation of blastocyst by 8 days
B formation of endoderm and ectoderm by 2 weeks
C formation of mesoderm by 6 weeks
D formation of human like embryo by 8 weeks
E the crown-rump length is about 3 cm
ANSWER:C
1285 In fetal period, all the following are true EXCEPT
A by 10 weeks the midgut returns to abdomen
B by 12 weeks external genitalia becomes clearly distinguishable
C by 24 weeks surfactant production begun
D by 26 weeks recognizable human face formed
E during third trimester the weight triples
ANSWER:D
1286 All the following are recognizable teratogens EXCEPT
A ethanol

B antiepileptic medications
C toxoplasmosis
D hypothermia
E mercury
ANSWER:D
1287 Prenatal exposure to cigarette smoke is associated with
A lower birthweight
B shorter length
C changes in neonatal neurodevelopmental status
D neonatal diabetes
E learning problems
ANSWER:D
1288
A 16-year-old G1P1 mother complaining of reduced milk production and breasts
engorgement. She stated that she feels unhappy and scared most of the times. Her husband is
a soldier and she is living with his family. Of the following, the MOST appropriate action is
to
A refer her to obstetrician
B explain the appropriate way of milk expression
C advice milk stimulant medications
D apply Edinburgh postnatal scale
E stop breast feeding
ANSWER:D
1289
A mother of a 2-week-fullterm baby noted doll's-eye movement of her baby eyes. Of the
following, The MOST appropriate next action is to
A reassures her by informing her that, this is a normal reaction
B refer the baby to an ophthalmologist
C takes a detailed history of perinatal period
D order brain ultrasound
E order brain MRI
ANSWER:A
1290
The age at which the infant achieve early head control with bobbing motion when pulled to
sit is
A 2 mo
B 3 mo
C 4 mo

D 5 mo
E 6 mo
ANSWER:B
1291
The age at which the infant can reach an object, grasp it and bring it to mouth and seems
exited when see the food is
A 4 mo
B 5 mo
C 6 mo
D 7 mo
E 8 mo
ANSWER:A
1292 By the age of 7 months the infant is able to do all the following EXCEPT
A transfer object from hand to hand
B bounces actively
C cruises
D grasp uses radial palm
E roll over
ANSWER:C
1293
You are specialist explaining the developmental implication of a 6-month-old baby boy who
"transfers object hand to hand" to college students.Of the following, the BEST statement
describing that is a/an
A visuomotor coordination
B voluntary release
C comparison of objects
D ability to explore small objects
E increasing autonomy
ANSWER:C
1294 Building a tower of 6 cubes by a 22-month-old child requires
A visualmotor coordination
B ability uses objects in combination
C visual, gross, and fine motor coordination
D able to link actions to solve problems
E symbolic thought
ANSWER:C
1295 A 10-month-old child can do all the following EXCEPT

A follows one-step command without gesture
B says “mama” or “dada”
C points to objects
D speaks first real word
E inhibition to "no"
ANSWER:D
1296 You are observing a 15-month-old toddler, he was able to do all the following EXCEPT
A walks alone
B makes tower of 3 cubes
C inserts raisin in a bottle
D responds to his/her name
E identifies 1 or more parts of body
ANSWER:E
1297 The age by which the child can makes tower of 9 cubes and imitates circular stroke is
A 24 mo
B 30 mo
C 36 mo
D 42 mo
E 48 mo
ANSWER:B
1298 The child who continue to search for a hidden subject, has achieved the developmental age of
A 7 mo
B 9 mo
C 11 mo
D 13 mo
E 15 mo
ANSWER:B
1299
The child who's able to stands momentarily on 1 foot, makes tower of 10 cubes; imitates
construction of “bridge” of 3 cubes; copies circle and imitates cross is
A 24 mo old
B 30 mo old
C 36 mo old
D 42 mo old
E 48 mo old

ANSWER:C
1300
A mother to a 4-year-old child who has pauses and repetitions of initial sounds visited
outpatients department. Of the following, the MOST appropriate advice is
A this is a normal phenomenon affecting about 5% of preschool children
B there is no need for action as 80% of affected children recover by their own
C tries to reduce pressures associated with speaking
D I'll refer him to ENT specialist for further evaluation
E I'll refer him to speech therapist
ANSWER:C
1301 These facts are true regarding the developmental stage of preschool children EXCEPT
A handedness is achieved by 3 years of age
B boys are usually later than girls in achieving bladder control
C knowing gender by 4 years
D egocentric thinking
E musturbation
ANSWER:C
1302
A worried mother of a 4-year-old boy describing attacks of inconsolable crying episodes of
her child, taking long time, she stated also that he prefers to play alone. Of the following, the
MOST appropriate action is to
A reassures her that this is a normal phenomenon of temper tantrum
B seek more history regarding other skills and developmental domains
C refer her to pediatric psychiatry
D investigate social issues of the family
E investigate for the child abuse
ANSWER:B
1303
A young couples of a 2-month-old baby girl with excessive crying seek your medical
advice. Mother said that the baby is crying about 3 hr in a day, 2-3 days per week. She is
intermittently spitting but she is gaining weight adequately. She is bottle fed baby. Of the
following, the LEAST important advice is to
A master the situation in relaxed manner
B adhere to precry cues
C change milk formula
D avoid sensory overstimulation
E reassure about benign nature of illness
ANSWER:C

1304
Of those babies who have prolonged crying episodes in the first 2 mo of life, the percentage
that will remain having similar episodes is about
A 1%
B 5%
C 10%
D 30%
E 50%
ANSWER:B
1305
A medical student asked you during the morning round about his observation of infants cry
in response to the cry of another infant. Of the following, the MOST appropriate answer is
that it represent
A an early sign of empathy development
B a sign of good hearing
C a startle reflex to a loud sound
D an early sign of fear development
E an early sign of autistic behavior
ANSWER:A
1306
Between 2-6 months of life, the infant start to achieve a regular sleep–wake cycles. All the
following are true about infant sleep during this period EXCEPT
A total sleep hours are about 14-16 hr/24 hr
B sleeps about 9-10 hr concentrated at night
C sleeps 2 naps/day
D sleep electroencephalogram shows the mature pattern
E the sleep cycle time is similar to that of adults
ANSWER:E
1307
A highly careful mother of a 10-month-old baby boy complaining that her baby has
inadequate weight gain. She explained to you how she is so strict and so careful to feed her
baby by spoon, but he was always refusing. Of the following, the LEAST helpful advice is to
A respect infant independence
B offer softer diet
C use 2-spoons (1 for the child and 1 for the parent)
D use finger foods
E use high chair with tray table
ANSWER:B

1308
The child who helps to undress; puts 3 words together (subject, verb, object); and handles
spoon well, his/her MOST appropriate age is around
A 15 mo
B 18 mo
C 24 mo
D 30 mo
E 48 mo
ANSWER:C
1309
The child who plays simple games (in “parallel” with other children); helps in dressing and
washes hands, his/her MOST appropriate developmental age is around
A 15 mo
B 18 mo
C 24 mo
D 30 mo
E 48 mo
ANSWER:D
1310
The child who is able to imitates construction of “gate” of 5 cubes; draws a man with 2-4
parts besides head and identifies longer of 2 lines, his/her MOST appropriate developmental
age is around
A 24 mo
B 30 mo
C 48 mo
D 54 mo
E 60 mo
ANSWER:C
1311 The birthweight usually quadruples by the age of
A 1.5 yr
B 2 yr
C 2.5 yr
D 3 yr
E 3.5 yr
ANSWER:C
1312
Regarding the physical growth of preschool children (3-5 yr); all the following are true
EXCEPT
A 4-5 kg weight increment/yr

B height increment/yr
C head will grow only an additional 5-6 cm up to18 yr
D all 20 primary teeth have erupted by 3 yr
E average wt 20 kg and height 40 inch by 4 yr
ANSWER:A
1313
Regarding the physical growth of middle childhood (6-11 yr); all the following are true
EXCEPT
A 3-3.5 kg wt increment/yr
B 6-7 cm height increment/yr
C brain myelinization stops by 8 yr
D first deciduous tooth falls by 6 yr
E risk for future obesity
ANSWER:C
1314 The first permanent tooth to erupt is
A central incisor at 6 yr
B molar at 6 yr
C premolar at 6-7 yr
D lower canine at 6-7 yr
E upper canine at 6-7 yr
ANSWER:B
1315 The following statements about sleep are true EXCEPT
A melatonin which is secreted in dark-light cycles is secreted from hypothalamus
B slow-wave sleep is the first cycle of sleep
C rapid eye movement (REM) sleep is responsible for dreams
D both cycles are needed for sufficient sleep
E REM sleep protect brain from injury
ANSWER:A
1316 The MOST common cause of sleeping difficulty in the first 2 months of life is
A gastro-esophageal reflux
B formula intolerance
C colic
D developmentally sleeping behavior
E recurrent self resolving intussusceptions
ANSWER:D

1317
A mother of 4-month-old baby boy complaining that the baby never goes to sleep unless he
is rocked for quite long time. She needs to repeat the same issue whenever he is awake at
night. She was surprised as no one of her previous babies had the similar behavior. She feels
tired because of insufficient sleep. Of the following, the MOST common cause of this baby
problem is
A an early signs of ADHD
B behavioral insomnia of childhood, sleep-onset association
C primary sleep disorder
D restless legs syndrome
E sleep terrors
ANSWER:B
1318
Parents of a 4-year-old boy complaining that he is refusing to go to bed, he remains active
and playing. Of the following, the MOST likely cause of this child problem is
A thyrotoxicosis
B ADHD
C primary sleep disorder
D limit setting; behavioral insomnia of childhood
E behavioral insomnia of childhood, sleep-onset association
ANSWER:D
1319 All the following are included in basic principles of healthy sleep EXCEPT
A set bedtime routine
B avoid stimulating activities as playing computer games
C makes sure your child spends time outside every day
D gives a heavy meal within an hour or 2 of bedtime
E keeps the television set out of your child’s bedroom
ANSWER:D
1320 All the following are compatible with the definition of obstructive sleep apnea EXCEPT
A repeated episodes of prolonged upper airway obstruction
B apnea
C ≥30% reduction in airflow
D ≥ 30% O2 desaturation
E disrupted sleep
ANSWER:D
1321 Of the following, the MOST common cause of obstructive sleep apnea in children is
A adenotonsillar hypertrophy

B obesity
C allergies
D craniofacial abnormalities
E pharyngeal reactive edema due to gastroesophageal reflux
ANSWER:A
1322
Persons with Down syndrome are at particularly high risk for obstructive sleep apnea with up
to 70% prevalence. All the following are considered as risk factors EXCEPT
A peculiar facial anatomy
B hypotonia
C developmental delay
D central adiposity
E hypothyroidism
ANSWER:C
1323 All the following are parasomniac disorders EXCEPT
A narcolepsy
B sleepwalking
C sleep terror
D confusional arousal
E nightmare
ANSWER:A
1324
The percentage of mental illnesses that is encountered at least once in any stage during
childhood is about
A 10%
B 20%
C 30%
D 40%
E 50%
ANSWER:B
1325
Clinicians tools needed to recognize early symptoms of mental disorders are called "Mental
Health Action Signs". All the following are true EXCEPT
A involvement in many fights, wanting to badly hurt others
B not eating, throwing up, or using laxatives to make yourself lose weight
C feeling very sad or withdrawn for more than 2 mo
D sudden overwhelming fear for no reason
E worries or fears that get in the way of your daily activities

ANSWER:C
1326
All the following are true associations between psychiatric illness in childhood and their
treatment EXCEPT
A attention deficit/hyperactivity (ADHD) and atomoxetine
B anxiety and antidepressant
C aggression and atypical antipsychotic
D psychosis and typical antipsychotic
E depression and antidepressant
ANSWER:D
1327 The following medications are truly matched to their major pharmacological groups EXCEPT
A methylphenidate …stimulant
B atomoxetine …………. serotonin-norepinephrine reuptake inhibitor
C escitalopram ……… selective serotonin reuptake inhibitors
D fluoxetine………………. tricyclic antidepressants
E risperidone……………..atypical antipsychotics
ANSWER:D
1328 Which one of the following statements is FALSE regarding rumination?
A seen only in infants and those with intellectual disability
B runs episodic course
C results from neglect
D in infancy can resolve spontaneously
E behavioral treatment is the treatment of choice
ANSWER:A
1329 In pica disorder, the MOST appropriate statement is
A it is only related to eating clay and earth
B usually in children below 2 years
C not present with other eating disorders
D it need a period of more than a month to be stated
E it resulted in high fatalities
ANSWER:D
1330
A 7-year-old boy presented with recurrent eye blinking behavior and recurrent extension of
extremities, mother describe the movement as sudden, rapid, and repetitive movements, it
was present in the last 9 months. Of the following, the MOST appropriate diagnosis is

A Tourette's disorder
B persistent motor tic disorder
C provisional tic disorder
D post-viral encephalitis
E Sydenham chorea
ANSWER:C
1331 All the following are recognized stereotypic movements EXCEPT
A hand shaking
B eye blinking
C body rocking
D head banging
E self-biting
ANSWER:B
1332
A mother to a well 2-year-old girl with thumb sucking behavior, she is worried that the
behavior may continue or may cause dental problem. Of the following, the BEST response is
to
A reassurance to mother
B leave the behavior as the complications usually started after 5 years
C ignore thumb sucking and encouraging a substituted behavior
D use of bitter ointments will resolve the problem early
E asses the social status of the family
ANSWER:C
1333
A 7-year-old child brought by his father to your clinic, the child often express fear of being
injured by a car accident during transport to school. He expresses this fear to his teachers and
parents. Of the following, the TRUE description of his reaction is
A non pathological anxiety
B school phobia
C separation anxiety
D generalized anxiety disorder
E panic disorder
ANSWER:A
1334 All the following medical conditions can cause anxiety in a child EXCEPT
A antihistamine medications
B hypoparathyrodism
C prolonged school absences

D carbonated beverages
E lead poisoning
ANSWER:B
1335 Regarding school refusal, which statement is FALSE?
A it is a complex disorder
B selective mutism is overlapping
C younger children usually have separation anxiety disorder
D older children usually suffer from obsessive disorders
E somatic symptoms are common
ANSWER:D
1336
A school nurse called you to see a 6-year-old boy with school refusal, he is always crying
after parental leaving, refuses to stay in the class room, the school nurse informed you that
she tried her best during the last 4 weeks; and she met with the parents frequently. Of the
following, the BEST action is to
A refer the child to a pediatrics psychologist
B start selective serotonin reuptake inhibitor treatment
C asses the home environment
D arrange for parent management training
E give the child special attention from the teacher
ANSWER:A
1337 All the following are characteristic features of separation anxiety disorder EXCEPT
A not manifested below 3 years of age
B common up to 5% of children
C girls are more affected than boys
D child thinking often include fear of harm affecting parents
E not reported in children above 8 years of age
ANSWER:E
1338
You are asked to explain breath holding spells concept to a staff nurse during morning
round. Of the following, the BEST statement; it is a/an
A manifestation of iron deficiency anemia
B type of seizure
C age typical expressions of frustration or anger
D psychic problem manifest when the child becomes older
E sort of tic disorder which resolve by age
ANSWER:C

1339
A concerned parents of an 18-month-child which expressed a brief tonic movement after
being upset and cried. All the following are true advices EXCEPT
A investigate possibility of iron deficiency anemia
B avoid over concern behavior
C interfere early in the event by calming the child
D try to behave calmly, observe the child without being seen
E arrange for home rectal diazepam
ANSWER:E
1340
You are evaluating a 5-year-old child with breath holding spells, the history given includes
pallor with abnormal limb movement lasted for 5 minutes followed by sleep. All the
following are true responses/advices EXCEPT
A reassurance, behavioral instruction to parents and follow up
B order MRI brain
C order an ECG
D order an EEG
E neurological consultation
ANSWER:A
1341
A concerned young parents asking about frequent lying behavior experienced by their 3-
year-old girl. The following advices are true EXCEPT
A it is a method of playing with the language
B it is a part of their magical thinking
C it indicates a potential for future lying behavior
D it is an approach to avoid unwanted confrontation with adults
E it is a way to describe things as they wish
ANSWER:C
1342 All the following are true about truancy EXCEPT
A it is normal behavior in young children
B it represent disorganization within the home
C it may reflect underlying child abuse
D depression may be an association
E adolescent may be at risk of substance abuse
ANSWER:A
1343 All the following are diagnostic criteria for major depressive episode EXCEPT
A hypomanic episode
B depressed mood

C loss of interest or pleasure
D significant weight loss
E hypersomnia
ANSWER:A
1344
Approximately 90% of youths who complete suicide have a preexisting psychiatric illness.
Of the following, the MOST commonly encountered illness is
A major depression
B schizophrenia spectrum disorders
C conduct disorder
D chronic anxiety
E panic disorder
ANSWER:A
1345
A 13-year-old adolescent female attained out-patient department (OPD), with a complaint of
feeling fat especially over the stomach and thighs; she is also feeling cold, tired, weak, and
lacking energy. Examination reveals, heart rate 46 beats/min, blood pressure 70/40 mm Hg,
weight below 3rd centile, dry skin, and lanugo-type hair growth on face; lab investigations
shows hypokalemia and hypophosphatemia. Of the following, the MOST appropriate next
action is
A admission to the hospital
B referral to psychiatrist
C referral to dietician
D make an scheduled OP visit to you and to psychiatrist
E offer partial hospital program
ANSWER:A
1346 All the following are characteristic features of autistic spectrum disorder (ASD)​EXCEPT
A defective social communication
B highly restricted fixated interests
C scarceof gesture use
D stereotyped motor movements
E absence of routines
ANSWER:E
1347
Diagnosis of autistic spectrum disorder (ASD) depends partly but importantly on assessment
of language. All the following may raise your concern regarding language development and
may indicate ASD EXCEPT
A absent babbling by 6 months

B absent gestures by 12 month
C absent single words by 16 month
D absent 2-word purposeful phrases by 24 month
E loss of language or social skills at any time
ANSWER:A
1348 All the following are recognized in autistic spectrum disorder (ASD) EXCEPT
A 3 grades of severity
B superior intellectual functioning in select areas
C motor deficits
D epilepsy
E truancy
ANSWER:E
1349
The MOST consistent statement of structural MRI brain finding of autistic spectrum disorders
(ASD) is
A diffuse brain atrophy
B increase brain size
C focal fibrosis
D white matter degenerative changes
E gray matter degenerative changes
ANSWER:B
1350 All the following should raise the suspicion of autistic spectrum disorders (ASD) EXCEPT
A sibling with ASD
B playmate concern
C parental concern
D caregiver concern
E pediatrician concern
ANSWER:B
1351
Ideally all children should be subjected to routine screening for autistic spectrum disorders
(in USA) at age of
A 06 and 12 mo
B 12 and 18 mo
C 18 and 24 mo
D 24 and 30 mo
E 30 and 18 mo
ANSWER:C

1352
An 18-month-old male toddler was found to be symptomatic for autistic spectrum disorders
(ASD) by routine screening testing. The recommended evaluation include all the following
EXCEPT
A physical examination for dysmorphic features
B hearing tests
C brain CT
D wood's lamp
E chromosomal microarray
ANSWER:C
1353
A 24-month-old child who is referred by health institute after parental concern of lonely
play and delayed speech, he is pica eater. You have diagnosed him as autistic spectrum
disorder (ASD). Of the following, the MOST important next action is
A physical examination for dysmorphic features
B hearing tests
C lead level
D Wood's lamp
E chromosomal microarray
ANSWER:C
1354
You are following a 6-year-old boy with autism, he is under structured psychosocial
behavioral training program, there is frequent complains of aggression and self- injurious
behavior. Of the following, the BEST medication to control his behavior is
A methylphenidate
B risperidone
C escitalopram
D atomoxetine
E amantadine
ANSWER:B
1355 Childhood psychosis may include all the following EXCEPT
A delusions
B loss of reality testing
C disorganized speech
D catatonic behavior
E acute phobic hallucination
ANSWER:E

1356
A parents came to your clinic complaining that their 5-year-old boy had attacks of
frightening with imagination of a snake crawling over him and he is acting as trying to
remove it. You assessed the child and find no acute physical illness and he is cooperative and
quite intelligent. Of the following, the MOST appropriate explanation for the child behavior
is
A acute phobic hallucination
B early sign of schizophrenia
C delusional infestation
D night terror
E bipolar disorder
ANSWER:A
1357
The definition of intellectual function is the capacity to think in the abstract, reason, problem
solve, and comprehend. The score at which significant weaknesses in adaptive skills are
evident is equal to or below
A 55
B 60
C 65
D 70
E 75
ANSWER:D
1358
The neurodevelopmental function (cognition) is dependent mainly on the development of
the following functions EXCEPT
A sensory and motor
B language
C visual–spatial
D intellectual
E cerebeller
ANSWER:E
1359 Which statement is FALSE regarding specific learning disorders (SLD)?
A it is a type of neurodevelopmental dysfunctions
B the overall estimates of the prevalence of SLD range from 3-10%
C it focuses on academic skill development
D includes students did well in academic testing but not in intelligence testing
E terms as dysgraphia or dyscalculia had been revolutionized
ANSWER:D

1360 The major chemical modulator of attention is
A dopamine
B serotonin
C nor-epinephrine
D troponin
E epinephrine
ANSWER:A
1361 In between specific learning disorders (SLD), the MOST refractory to treat is
A reading
B mathematics
C working memory
D social cognition
E writing
ANSWER:B
1362 All the following are risk factors for specific learning disorders (SLD) EXCEPT
A extreme prematurity
B low birth weight
C history of sibs or parents with SLD
D history of admission to neonatal care unit
E chronic medical problem
ANSWER:D
1363
All the following are red flags in the history of children with specific learning disorders
(SLD) EXCEPT
A parents concern about academic performance
B inconsistency in marks report from grade to grade
C struggling with home work activities
D positive vision or hearing test
E positive standard screening test
ANSWER:B
1364
All the following are established modalities of treatment in a child with specific learning
disorders (SLD) EXCEPT
A remediation of skills
B demystification
C bypass strategies
D dietary interventions

E strengthening of strength
ANSWER:D
1365 The MOST common specific learning disorder (SLD) of childhood is
A attention-deficit/hyperactivity disorder (ADHD)
B reading disorder (dyslexia)
C spelling disorder
D arithmetical skills disorders
E mixed disorder of Scholastic Skills
ANSWER:B
1366
The behavioral changes in attention-deficit/hyperactivity disorder (ADHD) should met all the
following criteria EXCEPT
A developmentally inappropriately far comparing with other children of the same age
B must begin before age 6 yr
C must be present for at least for 6 mo
D must be present in 2 or more settings
E must not be secondary to another disorder
ANSWER:B
1367
You are assessing an eight-year-old male child with attention-deficit/hyperactivity disorder
(ADHD). Of the following, the LEAST useful test/investigation is
A thyroid function test
B lead level
C EEG
D Blood film
E polysomnography
ANSWER:D
1368
Of the following, the MOST common presenting behavior in girls with attention-
deficit/hyperactivity disorder (ADHD) is
A inattentive
B hyperactive
C impulsive
D disruptive
E combined
ANSWER:A
1369 Of the following, the TRUE statement of reading disorder is
A 25% of affected children have affected parents

B diagnosis is usually clinical
C IQ will remain same with time
D functional MRI is diagnostic
E boys are more affected than girls
ANSWER:B
1370
In USA, the newly arrived international adoptees are subjected to many lab tests. All the
following tests need to be repeated after 3-6 months after arrival EXCEPT
A hepatitis B virus serology
B hepatitis C virus serology
C human immunodeficiency serology
D tuberculin skin test
E treponemal test
ANSWER:E
1371
You are a newly employee Pediatrician to foster care children (children removed from their
original families and put in suitable institutes for protection). Those children have many
significant problems, but the MOST common one is
A chronic medical problems
B abuse and neglect
C mental health concerns
D family relationship problems
E developmental problems
ANSWER:D
1372
Witnessing violence, community violence, and media violence all are detrimental to children
as it increases the impact of violence. All the following adverse effects can be increased by
increasing the impact of violence EXCEPT
A poor school performance
B symptoms of anxiety and depression
C lower self-esteem
D cyper bullying
E post traumatic stress disorder
ANSWER:D
1373
A 9-year-old male with good school performance and normal behavior. In the last 2 months
he had changed his classroom. His teacher observed that he became introverted and his
school performance reduced dramatically. He was always described by some of his new
school mates as a "weak". Of the following, the MOST likely diagnosis is

A school phobia
B anxiety disorder
C bullying
D hypothyroidism
E depression
ANSWER:C
1374 The following factors show psychosocial impact of war on children EXCEPT
A loss of family members
B separation from community
C lack of education
D acute stress reaction
E displacement
ANSWER:E
1375 The MOST reported type of child abuse in USA is
A neglect
B physical abuse
C sexual abuse
D psychological maltreatment
E trafficking
ANSWER:A
1376
Regarding the physical abuse, the physical signs in abused children (as bite marks) may be
found in up to
A 5%
B 15%
C 25%
D 35%
E 45%
ANSWER:C
1377 The MOST common clue of physical abuse in children is
A history of inflicted trauma
B burn marks
C bruises
D fractures
E retinal hemorrhages
ANSWER:C

1378
In children with physical abuse, when the inflected trauma is burn injury. Of the following,
the MOST suggestive one is
A burn in napkins with involvement of flexures
B presence of splash mark
C unclear borders of burn
D glove and stock distribution
E folk remedies
ANSWER:D
1379 Skeletal injuries with high specificity in child abuse may include all the following EXCEPT
A tibial metaphyseal corner lesions
B posterior rib fracture
C scapular fracture
D first rib fracture
E clavicle fracture
ANSWER:E
1380
A 3-month-old baby girl admitted to pediatric intensive care unit with severe head injury
(evident by CT scan), the history given by parents was trivial and not informative. You
suspected child abuse. Of the following, the MOST helpful study to support your suspicion is
A infantogram (one shot for entire body)
B lateral and AP view X ray of the spine
C lateral and AP view X ray of skull
D radionuclide bone scan
E MRI brain
ANSWER:D
1381
A 3-month-precious baby of a keen and highly educated family presented with history of
URTI followed by deterioration of level of consciousness and seizure. CT brain shows
intracranial bleeding with no evidence of skull fracture. Fundocscopy was normal. Mother
gives history of difficult labour but with normal development, mild hypotonia, and
macrocephaly which was reassured initially by general Pediatrician. Of the following, the
MOST likely diagnosis is
A residual birth trauma
B arteriovenous malformation
C acute viral encephalitis

D glutaric aciduria type 1
E abusive head trauma (AHT)
ANSWER:D
1382
Retinal hemorrhages are an important marker of abusive head trauma (AHT). Whenever
AHT is being considered, a dilated indirect eye examination by a pediatric ophthalmologist
should be performed. All the following are characteristic of retinal hemorrhage caused by
AHT EXCEPT
A multiple
B involve more than one layer of the retina
C central
D traumatic retinoschisis
E various sizes
ANSWER:C
1383
A 3-year-old female child with repeated admissions as bleeding tendency in a form of
spontaneous ecchymosis and bleeding through the nose and urine. She had another daughter
who died because of similar undiagnosed illness. The mother is a staff nurse and she is
highly concerned about her child illness. Lab investigations usually show either PT and/or
PTT prolongation with normal platelets, which usually normalized after empiric vitamin K
administration. The father is a petrol engineer and never seen accompanying the family. Of
the following, the MOST helpful investigation is
A factor VII assay
B factor II assay
C drug level
D von Willebrand factor assay
E platelets function test
ANSWER:C
1384 In a child less than 3 years, the diagnosis of failure to thrive (FTT) is considered if
A weight is below the 3rd percentile
B weight drops down more than 2 major percentile lines
C weight for height is less than the 1st percentile
D BMI less than the 25th percentile
E midarm circumference < 15 cm
ANSWER:B

1385
In patients with failure to thrive; chromosomal abnormality, intrauterine infection, and
metabolic disorders need to be ruled out. All the following factors are suggestive of
metabolic problems EXCEPT
A insidious history
B recurrent vomiting
C neurologic symptoms
D cardiomyopathy
E renal symptoms
ANSWER:A
1386
Approaching to a child with failure to thrive based on signs and symptoms. Of the following,
the MOST common cause behind a child has spitting, vomiting, and food refusal is
A gastroesophageal reflux
B chronic tonsillitis
C food allergies
D eosinophilic esophagitis
E inflammatory bowel disease
ANSWER:A
1387
A 15-month-old male child with failure to thrive, diarrhea, and fatty stool. Of the following,
the MOST common possible cause of his illness is
A malabsorption
B intestinal parasites
C milk protein intolerance
D pancreatic insufficiency
E immunodeficiency
ANSWER:A
1388
A 2-year-old child with failure to thrive, recurrent wheezing, and pulmonary infections. Of
the following, the LEAST common cause of his illness is
A asthma
B aspiration
C food allergy
D cystic fibrosis
E immunodeficiency
ANSWER:E

1389
In a child with failure to thrive, the indications for hospitalization include severe malnutrition
or failure of outpatient management. The period after which the child needs hospitalization,
if he has not been responded to outpatient management, is about
A 1-2 wk
B 4 wk
C 2-3 mo
D 4 mo
E 5-6 mo
ANSWER:C
1390
You are treating a 10-month-old child with failure to thrive. Of the following, the minimal
accepted catch-up weight gain is about
A 10 gm/kg/day
B 20-30 gm/kg/day
C 40-50 gm/kg/day
D 60 gm/kg/day
E 80 gm/kg/day
ANSWER:B
1391 Regarding nutritional requirement during childhood, all the following are true EXCEPT
A nutrition and growth during 3-6 years of life predict adult stature and some health outcomes
B major risk period for growth stunting is between 4 and 24 months of age
C
estimated average requirement (EAR) is the average daily nutrient intake level estimated to
meet the requirements for 50% of the population
D
recommended dietary allowance (RDA) is an estimate of the daily average nutrient intake to
meet the nutritional needs of >97% of the individuals in a population
E
the 3 components of energy expenditure are the basal metabolic rate, thermal effect of food,
and energy for physical activity
ANSWER:A
1392 The adequate intake (AI) for the total digestible carbohydrates in a nine-month-old boy is
A 60 gm/day
B 95 gm/day
C 120 gm/day
D 145 gm/day
E 170 gm/day
ANSWER:B

1393 The adequate intake (AI) for the total fat in a nine-month-old boy is
A 10 gm/day
B 20 gm/day
C 30 gm/day
D 40 gm/day
E 50 gm/day
ANSWER:C
1394 The adequate intake (AI) for the total protein in a nine-month-old boy is
A 9 gm/day
B 11 gm/day
C 13 gm/day
D 15 gm/day
E 17 gm/day
ANSWER:B
1395 All the following amino acids are indispensable in human diet EXCEPT
A leucine
B methionine
C threonine
D valine
E alanine
ANSWER:E
1396 Essential fatty acids deficiency is associated with all the following features EXCEPT
A desquamating skin rashes
B alopecia
C thrombocytosis
D impaired immunity
E growth deficits
ANSWER:C
1397 All the following statements concerning iron requirement in children are true EXCEPT
A breast milk provides optimal intake of iron for the first 4 months
B iron present in animal protein is more bioavailable than that found in vegetables
C iron deficiency is the most common micronutrient deficiency
D cow’s milk is a good source of bioavailable iron
E iron supplements can interfere with zinc absorption
ANSWER:D

1398 Zinc deficiency is associated with increased risk for all the following EXCEPT
A stunting
B impaired immune function
C increased risk for respiratory diseases
D increased risk for diarrheal diseases
E increased risk for skin infections
ANSWER:E
1399
The American academy of pediatrics change the vitamin D intake recommendation in 2010
to
A 200 IU/day
B 400 IU/day
C 600 IU/day
D 800 IU/day
E 1000 IU/day
ANSWER:C
1400
Breast feeding has been suggested to have a possible protective effect against all the
following EXCEPT
A otitis media
B urinary tract infections
C septicemia
D childhood cancer
E skin infections
ANSWER:E
1401 An absolute contraindication to breastfeeding is
A hepatitis c infection of the mother
B alcohol intake
C herpes simplex lesion of the lips
D active pulmonary tuberculosis
E maternal high grade fever
ANSWER:D
1402 egarding breast engorgement; All the following are true EXCEPT
A usually happens in the first stage of lactogenesis
B poor breast feeding technique can cause engorgement
C breastfeeding immediately at signs of infant hunger will eventually prevent this

D
to reduce engorgement, breasts should be softened prior to infant feeding with a
combination of hot compresses and expression of milk
E
between feedings, cold compresses applied, and oral nonsteroidal anti- inflammatory
medications administered
ANSWER:A
1403 All the following organisms can cause mastitis EXCEPT
A Staphylococcus aureus
B Escherichia coli
C Haemophilus influenza
D Klebsiella pneumoniae
E group B streptococcus
ANSWER:E
1404 The MOST common cause of using infant formula is
A parental preference
B inborn error of metabolism
C maternal tuberculosis
D masititis
E inadequate weight gain
ANSWER:A
1405 Concerns to be considered among vegetarians are all the following EXCEPT
A higher bioavailability of iron
B lower B12 levels
C risk of having lower level of fatty acids
D lower levels of calcium and vitamin D3
E lower bioavailability of zinc
ANSWER:A
1406 All the following are dimensions of food security EXCEPT
A access
B utilization
C preparation
D stability
E availability
ANSWER:C
1407 The TRUE hemoglobin cutoff to define anemia is
A 120 g/L for children 6-59 mo

B 115 g/L for children 5-11 yr
C 110 g/L for children 12-14 yr
D 110 g/L for non-pregnant women
E 120 g/L for men
ANSWER:B
1408 The MOST commonly used index for nutritional status is
A height-for-age
B weight-for-height
C body mass index
D mid-upper arm circumference
E weight-for-age
ANSWER:E
1409
For children <5 yr, the highest global prevalence of micronutrient and trace elements
deficiencies is that of
A vitamin A deficiency
B zinc deficiency
C iron deficiency
D iodine deficiency
E copper deficiency
ANSWER:A
1410 The MOST profound consequence of undernutrition is
A premature death
B repeated infections
C stunting
D cell damage
E developmental delay
ANSWER:A
1411
Reductive adaptation” process of evolution to a state of full malnutrition, all the following
events will be ensued EXCEPT
A the liver makes glucose less readily
B less heat production
C gut produces less gastric acid and gut enzymes
D evident responses to infection
E reduced red cell mass
ANSWER:D

1412 The outward sign of cell damage in malnutrition is
A repeated infections
B wasting
C edema
D hepatomegaly
E facial appearance
ANSWER:C
1413
The recommended transition time from stabilization to rehabilitation phase in the treatment
of malnutrition is
A 1 day
B 3 days
C 5 days
D 7 days
E 10 days
ANSWER:B
1414
The WHO recommendation for antibiotic cover in the stabilization phase of treating
malnourished boy free of initial complications is to use
A oral amoxicillin
B parenteral penicillin
C oral second generation cephalosporin
D parenteral gentamicin
E parenteral third generation cephalosporin
ANSWER:A
1415 During treatment of malnutrition, the signal of entry to the rehabilitation phase is
A reduced edema
B resolution of infection
C disappearance of signs of micronutrient deficiency
D constant blood sugar level
E constant body temperature
ANSWER:A
1416
In consideration of preventing infections in stabilization phase of malnutrition in
unimmunized 10-month-old boy. Of the following, the vaccine that is recommended to be
given is
A oral polio vaccine
B acellular pertussis vaccine

C measles vaccine
D pneumococcal vaccine
E H. influenza vaccine
ANSWER:C
1417
The following vitamins and trace elements are recommended to be given during the
stabilization phase of malnutrition EXCEPT
A iron
B vitamin A
C folic acid
D zinc
E multivitamins
ANSWER:A
1418 Emergency treatment in severe malnutrition includes all the following EXCEPT
A shock
B hypoglycemia
C dehydration
D severe anemia
E infections
ANSWER:E
1419 The aim of the rehabilitation phase in the treatment of malnutrition is to
A repair cellular function
B correct fluid and electrolyte imbalance
C restore homeostasis
D catch-up growth
E prevent death from infection
ANSWER:D
1420 The hallmark of refeeding syndrome is the development of severe
A hypophosphatemia
B hypokalemia
C hypomagnesemia
D hypernatremia
E hyperglycemia
ANSWER:A
1421 Encephalopathy in refeeding syndrome is mainly a result of
A hypophosphatemia

B hypokalemia
C hypomagnesemia
D thiamine deficiency
E hyperglycemia
ANSWER:D
1422 Thrombocytopenia in refeeding syndrome is mainly a result of
A hypophosphatemia
B hypokalemia
C hypomagnesemia
D thiamine deficiency
E hyperglycemia
ANSWER:A
1423 Across all racial groups, the factor that confers protection against childhood obesity is
A gestational weight gain
B high birth weight
C maternal smoking
D breast feeding
E maternal education
ANSWER:E
1424
Adeposity rebound, the period when the body fat is typically at the lowest level, is best
represented at the age of
A 1 year
B 3 years
C 6 years
D 9 years
E 12 years
ANSWER:C
1425 The gastrointestinal hormone/response that stimulates appetite in children is
A cholecystokinin
B glucagon-like peptide-1
C peptide yy
D vagal neuronal feedback
E ghrelin
ANSWER:E
1426 All the following are childhood obesity-associated comorbidities later in life EXCEPT

A hypertension
B gallbladder disease
C tibia vara
D asthma
E type 1 diabetes mellitus
ANSWER:E
1427 Genetic disorders associated with obesity can have all the following features EXCEPT
A dysmorphic features
B cognitive impairment
C vision abnormalities
D hearing abnormalities
E tall stature
ANSWER:E
1428 The following are well recognized causes of secondary obesity EXCEPT
A myelodyplasia
B muscular dystrophy
C anti-psychotic medications
D achondroplasia
E anti-migraine medications
ANSWER:E
1429
The following physical features might be discovered during physical examination of an
obese 10-year-old boy, and each reflect the associated morbidity EXCEPT
A developmental delay suggests genetic disorder
B day time sleepiness suggests sleep apnea
C polyuria and nocturia suggest type 1 diabetes
D hip pain suggests blount disease
E abdominal pain suggests non alcoholic fatty liver disease
ANSWER:C
1430
The investigations to be done as part of initial evaluation of a newly identified obesity in a
12-year-old girl are the following EXCEPT
A fasting plasma glucose
B triglyceride level
C lipoprotein level
D liver function tests
E renal function tests

ANSWER:E
1431
The only Food and Drug Administration (FDA)-approved medication for obesity in children
<16 yr old is
A orlistat
B phentermine
C topiramate
D amylin
E lorcaserin
ANSWER:A
1432 The MOST characteristic lesion of vitamin A deficiency is
A xerophthalmia
B corneal ulcers
C Bitot spots
D keratomalacia
E keratoconus
ANSWER:A
1433 All the following are helpful for diagnosis of marginal vitamin A deficiency EXCEPT
A dark adaptation test
B conjunctival impression cytology
C relative dose response
D plasma retinol level
E diet history
ANSWER:D
1434 Chronic hypervitaminosis A is associated with all the following features EXCEPT
A seborrhea
B alopecia
C hepatosplenomegaly
D bulging fontanel
E echymosis
ANSWER:E
1435 The following are characterstic features of acute hypervitaminosis A EXCEPT
A nausea and vomiting
B drowsiness
C diplopia
D cranial nerve palsies

E seizures
ANSWER:E
1436 The following conditions are risk factors for carotenemia EXCEPT
A liver disease
B food faddist
C diabetes mellitus
D hypothyroidism
E malabsorption
ANSWER:E
1437 The classic clinical triad of Wernicke encephalopathy of thiamine deficiency is
A mental status changes, ocular signs, and ataxia
B cardiac involvement, peripheral neuritis, and aphonia
C depression, drowsiness, and poor mental concentration
D increased intracranial pressure, meningismus, and coma
E deterioration of school performance, seizures, and headache
ANSWER:A
1438 Death from thiamine deficiency is usually due to
A renal involvement
B repeated infections
C lactic acidosis
D cardiac involvement
E increased intracranial pressure
ANSWER:D
1439 Regarding thiamine deficiency
A features develop within 2-3 weeks of deficient intake
B poor mental concentration and depression are late features of the disease
C hoarseness or aphonia is a characteristic sign
D the term (dry) beriberi reflects the features of raised intracranial pressure
E the term (wet) beriberi reflects the features of renal dysfunction
ANSWER:C
1440
Thiamine-responsive megaloblastic anemia syndrome is a rare disorder, it is characterized by
the following EXCEPT
A diabetes mellitus
B peripheral neuritis
C megaloblastic anemia

D sensorineural hearing loss
E autosomal recessive inheritance
ANSWER:B
1441
Brown-Vialetto-Van Laere syndrome (BVVLS), a neurologic disorder characterized by
progressive neurologic deterioration, sensorineural hearing loss, and pontobulbar palsy
usually responds to treatment with high doses of
A niacin
B biotin
C riboflavin
D pyridoxine
E thiamin
ANSWER:C
1442 The MOST convenient way to confirm a diagnosis of pellagra in children is
A skin biopsy
B urinary 2-pyridone
C response to niacin treatment
D urinary n1-methyl-nicotinamide
E physical signs of glossitis and dermatitis
ANSWER:C
1443 The MOST characteristic manifestation of pellagra is
A dermatitis
B dementia
C diarrhea
D anorexia
E lassitude
ANSWER:A
1444 The dietary reference intake of pyridoxine for a healthy eight-month-old infant is
A 0.1 mg/day
B 0.3 mg/day
C 0.5 mg/day
D 1.0 mg/day
E 3 mg/day
ANSWER:B
1445 All the following manifestations of pyridoxine deficiency are seen in children EXCEPT
A seizures

B cheilosis
C listlessness
D peripheral neuritis
E seborrheic dermatitis
ANSWER:D
1446 Pyridoxine is not affected in patients receiving
A isoniazid
B valproate
C phenytoin
D penicillamine
E corticosteroids
ANSWER:B
1447 Folate is important during embryogenesis for the development of
A lungs
B genital organs
C gastrointestinal system
D central nervous system
E cardiovascular system
ANSWER:D
1448 All the following can cause folate deficiency EXCEPT
A celiac disease
B sickle cell anemia
C 6-mercaptopurine therapy
D inflammatory bowel disease
E methylene tetrahydrofolate reductase deficiency
ANSWER:C
1449 Clinical features of cerebral folate deficiency include the following EXCEPT
A lethargy
B microcephaly
C atatxia
D ballismus
E blindness
ANSWER:A
1450
One of the recognized non-hematological manifestations of folic acid deficiency in children
is

A repeated respiratory abscesses
B recurrent abdominal pain
C recurrent seizures
D growth retardation
E teeth decay
ANSWER:D
1451 All the following are features of hereditary folate malabsorption EXCEPT
A anal ulcers
B chronic diarrhea
C failure to thrive
D megaloblastic anemia
E opportunistic infections
ANSWER:A
1452 The best indicator of chronic folate deficiency is
A serum folate level
B RBC folate level
C urinary folate level
D bone marrow aspiration
E CSF 5- methyltetrahydrofolate level
ANSWER:B
1453 Effective supplementation of folate for prevention of neural tube defect is
A started at least 1 mo before conception, and continued through the first 2-3 mo of pregnancy
B started at least 3 mo before conception, to be stopped after positive pregnancy test
C started after positive pregnancy test and continued through the first 2-3 mo of pregnancy
D started at least 1 mo before conception, to be stopped after positive pregnancy test
E started at least 3 mo before conception, and continued through the first 2-3 mo of pregnancy
ANSWER:A
1454 The dietary reference intake (DRI) of folate for a healthy eight-month-old infant is
A 80 microgm/day
B 150 microgm/day
C 200 microgm/day
D 300 microgm/day
E 400 microgm/day

ANSWER:A
1455 Massive doses of folate given by injection have the potential to cause
A ototoxicity
B neurotoxicity
C nephrotoxicity
D severe abdominal pain
E respiratory difficulty
ANSWER:B
1456 Important mechanism for maintaining vitamin B12 nutriture include the following EXCEPT
A direct absorption
B acidic pH in the ileum
C enterohepatic circulation
D binding with intrinsic factor
E intestinal bacterial synthesis
ANSWER:B
1457 Vitamin B12 is important for all of the following EXCEPT
A hematopoiesis
B mental development
C skeletal development
D psychomotor development
E central nervous system myelination
ANSWER:C
1458 All the following are real risk factors for vitamin B12 deficiency EXCEPT
A celiac disease
B ileal resection
C
Imerslund-gräsbeck disease
D helicobacter pylori infection
E use of proton pump inhibitors
ANSWER:E
1459
Common observations with vitamin B12 deficiency in children include all the following
EXCEPT
A hypotonia
B peripheral neuritis
C developmental regression

D involuntary movements
E hyperpigmentation of the knuckles
ANSWER:B
1460 The dietary reference intake of cobalamin for a healthy eight-month-old infant is
A 0.5 microgm/day
B 1.0 microgm/day
C 1.5 microgm/day
D 2.0 microgm/day
E 2.5 microgm/day
ANSWER:A
1461 Radiographic features of scurvy are similar to that of
A copper deficiency
B magnesium deficiency
C vitamin A excess
D vitamin D excess
E mercury exposure
ANSWER:A
1462 The best indicator of the body stores of vitamin C is
A plasma ascorbate concentration
B leukocyte ascorbate concentration
C urinary ascorbate level
D urinary amino acid level
E plasma amino acid level
ANSWER:B
1463 The MOST specific but late radiographic feature of scurvy is
A ground-glass appearance of the shafts of the long bones
B pencil outlining of the epiphysis and diaphysis
C the white line of fränkelat the metaphysis
D Trümmerfeld zone at the metaphysis
E Pelkan spur at cortical ends
ANSWER:D
1464 Ingestion of large doses of vitamin C can cause
A musculoskeletal pain
B osmotic diarrhea
C severe headache

D respiratory symptoms
E hematemesis
ANSWER:B
1465 The following are recognized causes of craniotabes EXCEPT
A osteogenesis imperfect
B rickets
C syphilis
D Sotos syndrome
E normal newborn
ANSWER:D
1466 All the following are recognized causes of rickets EXCEPT
A Mccune-Albright syndrome
B epidermal nevus syndrome
C tuberus sclerosis
D neurofibromatosis
E aluminum-containing antacids
ANSWER:C
1467 Dominant features in X-linked hypophasphatemic rickets are
A head; craniotabes, frontal bossing, and caries
B back; scoliosis, lordosis, and kyphosis
C chest; rachitic rosary and Harrison groove
D extremities; coxa vara and windswept deformity
E general; listlessness and failure to thrive
ANSWER:D
1468 Inorganic phosphorus in rickets is raised in
A vitamin D deficiency
B dietary calcium deficiency
C Fanconi syndrome
D chronic renal failure
E tumor-induced rickets
ANSWER:D
1469 Parathyroid hormone level is reduced in
A vitamin D dependent rickets
B X- linked hypophosphatemic rickets
C hypophosphatasia

D dietary calcium deficiency
E vitamin D deficiency
ANSWER:C
1470 D level is reduced in
A vitamin D deficiency
B autosomal recessive hypophosphatemic rickets
C autosomal dominant hypophosphatemic rickets
D Fanconi syndrome
E tumor-induced rickets
ANSWER:A
1471 Urinary phosphorus is reduced in
A chronic renal failure
B autosomal dominant hypophosphatemic rickets
C autosomal recessive hypophosphatemic rickets
D dietary calcium deficiency
E Fanconi syndrome
ANSWER:A
1472
The following hereditary forms of rickets may help to diagnose an apparently healthy
mother with the same disease
A X- linked hypophosphatemic rickets
B autosomal dominant hypophosphatemic rickets
C autosomal recessive hypophosphatemic rickets
D Fanconi syndrome
E hereditary hypophosphatemic rickets with hypercalciuria
ANSWER:A
1473
In evaluating the cause of rickets, which test is convenient if malabsorption is a
consideration?
A PT
B PTT
C stool pH
D barium study
E hydrogen breath test
ANSWER:A
1474 The standard method for determining a patient’s vitamin D status is
A 25-hydroxylase level

B 25-hydroxy vitamin D level
C 1, 25-dihydroxy vitamin D level
D D-binding protein
E PTH level
ANSWER:B
1475 Regarding vitamin E deficiency in premature infants, one of the following is TRUE
A it does not cross the placenta
B hemolysis develops in the first week of life
C
premature formula that contains poly unsaturated fatty acids may carry a protective effect
against hemolysis
D excessive iron supplementation for premature infants may augment the oxidative stress
E causes thrombocytopenia
ANSWER:D
1476
The following conditions/agents are incorporated in a similar presentation to vitamin K
deficiency bleeding EXCEPT
A liver disease
B hereditary factor II deficiency
C high dose salicylate
D heparin
E rodent poison
ANSWER:D
1477 The first factor to be affected by deficiency of vitamin K is
A Factor I
B Factor II
C Factor VII
D Factor IX
E Factor X
ANSWER:C
1478 The baby that is least vulnerable to develop vitamin K bleeding tendency is
A a three-day old neonate of a healthy mother that startedimmediate breastfeeding after birth
B a three-year old child spent 21 days in intensive care unit receiving total parenteral nutrition
C a nine-month old infant with prolonged history of diarrhea and multiple antibiotics therapy

D
a six-month old infant with cystic fibrosis missed to be diagnosed during early neonatal
period
E a six-hour neonate of an epileptic mother that received phenobarbital during pregnancy
ANSWER:A
1479
One of the following matching is TRUE regarding the trace elements deficiencies and their
consequences
A zinc and hypogonadism
B selenium and dental caries
C iron and hypothyroidism
D fluoride and cardiomyopathy
E manganese and decreased alertness
ANSWER:A
1480
One of the following matching is TRUE regarding the trace element excess and its
consequence
A copper and congenital hypothyroidism
B manganese and hyperuricemia
C iron and abdominal pain
D zinc and cholestatic jaundice
E fluoride and nail changes
ANSWER:C
1481
The ratio of the intracellular fluid volume to the extracellular fluid volume approaches adult
levels at the age of
A 1 yr
B 2 yr
C 3 yr
D 4 yr
E 5 yr
ANSWER:A
1482
Expansion of the intravascular volume and increased intravascular pressure are the main
causes of edema in
A lymphatic obstruction
B heart failure
C protein-losing enteropathy
D nephrotic syndrome
E sepsis

ANSWER:B
1483 The critical site for the renal regulation of sodium balance is the
A nephron
B proximal tubule
C loop of Henle
D distal tubule
E collecting duct
ANSWER:E
1484 More than 40% of total body sodium is in the
A intravascular spaces
B interstitial spaces
C intracellular fluid
D bone
E gastrointestinal fluid
ANSWER:D
1485 The MOST devastating consequence of untreated hypernatremia is
A brain hemorrhage
B seizures
C central pontinemyelinolysis
D extra pontinemyelinolysis
E brain edema
ANSWER:A
1486
In a child with hypernatremic dehydration, the first priority is restoration of intravascular
volume by
A 3% saline
B normal saline
C lactated Ringer solution
D 1/3 glucose saline
E 1/2 glucose saline
ANSWER:B
1487 Pseudohyponatremia is present in all the following EXCEPT
A multiple myeloma
B immunoglobulin infusion
C protein losing enteropathy
D hypertriglyceridemia

E hypercholesterolemia
ANSWER:C
1488 Hypervolemic hyponatremia is caused by
A cirrhosis
B cerebral salt wasting
C pseudohypoaldosteronism type I
D obstructive uropathy
E bowel obstruction
ANSWER:A
1489 Asymptomatic hyponatremia is seen in
A cirrhosis
B tap water enema
C child abuse
D hyperglycemia
E tubule -interstitial nephritis
ANSWER:D
1490 Syndrome of inappropriate antidiuretic hormone secretion (SIADH) is characterized by
A extravascular volume expansion
B high serum uric acid
C high blood urea nitrogen
D euvolemic hyponatremia
E urine sodium <30 mEq/L
ANSWER:D
1491 purious hyperkalemia or pseudohyperkalemia in children is usually exaggerated by
A blood transfusions
B digitalis intoxication
C thrombocytosis
D renal failure
E malignant hyperthermia
ANSWER:C
1492 The MOST common cause of hypokalemia in children is
A alkalemia
B distal rental tubular acidosis
C gastroenteritis
D diabetic ketoacidosis

E loop diuretic
ANSWER:C
1493 Spurious hypokalemia occurs in patients with
A laxative abuse
B high white blood cell count
C hypomagnesaemia
D posthypercapnia
E Cushing syndrome
ANSWER:B
1494 The mechanism of hypokalemia in emesis is mainly due to
A gastric loss of potassium
B gastric loss of hydrochloric acid
C low aldosterone level
D dehydration
E hypomagnesaemia
ANSWER:B
1495 Renal manifestations of hypokalemia include all the following EXCEPT
A polyuria
B urinary retention
C decrease ammonia production
D interstitial nephritis
E renal cysts
ANSWER:C
1496 Medications that cause renal losses of magnesium include all the following EXCEPT
A amphotericin
B cisplatin
C lithium
D proton pump inhibitors
E mannitol
ANSWER:C
1497 ECG changes in hypomagnesaemia is characterized by
A peaking of the T waves and ST-segment depression
B flattened T wave and depressed ST segment
C flattening of the T wave and lengthening of the ST segment
D prolonged PR, QRS, and QT intervals

E appearance of a U wave
ANSWER:C
1498 Clinical manifestations of hypermagnesemia include all the following EXCEPT
A hypotonia
B hyporeflexia
C sleepiness
D hypertension
E vomiting
ANSWER:D
1499 Treatment of hypermagnesmia include all the following EXCEPT
A intravenous hydration
B loop diuretics
C Kayexalate
D exchange transfusion
E intravenous calcium gluconate
ANSWER:C
1500 The MOST common cause of hyperphosphatemia is
A acute hemolysis
B vitamin D intoxication
C renal insufficiency
D cow’s milk intake
E hypoparathyroidism
ANSWER:C
1501
Systemic calcification occurs when plasma calcium × plasma phosphorus, both measured in
mg/dL is
A >90
B >80
C >70
D >60
E >50
ANSWER:C
1502 All the following phosphorus binders are used in chronic renal insufficiency EXCEPT
A calcium carbonate
B calcium acetate
C sevelamer

D lanthanum
E aluminum
ANSWER:E
1503 Normal anion gap metabolic acidosis can occur in
A renal failure
B liver failure
C severe anemia
D malignancy
E starvation ketoacidosis
ANSWER:A
1504
The time for the kidneys to complete appropriate metabolic compensation for a primary
respiratory process is
A 12-24 hr
B 24-36 hr
C 36-48 hr
D 48-72 hr
E 72-96 hr
ANSWER:E
1505 Metabolic acidosis with increased anion gap occurs in
A ammonium chloride intake
B posthypocapnia
C urinary tract diversions
D hyporeninemic hypoaldosteronism
E severe anemia
ANSWER:E
1506 Metabolic alkalosis with urinary chloride >20 meq/l occurs in
A loop diuretics
B cystic fibrosis
C Cushing syndrome
D emesis
E post-hypercapnia
ANSWER:C
1507
In metabolic alkalosis secondary to gastric loss, all the following mechanisms prevent renal
bicarbonate loss EXCEPT
A mineralocorticoid escape phenomenon

B reduce GFR
C sodium and bicarbonate resorption
D increase aldosterone level
E hypokalemia
ANSWER:A
1508 Contraction alkalosis occurs in
A metformin
B diuretic use
C propofol
D methanol
E toluene
ANSWER:B
1509
Measuring serum concentrations of renin and aldosterone differentiates children with
metabolic alkalosis; both renin and aldosterone are elevated in
A renovascular disease
B glucocorticoid-remediable aldosteronism
C Liddle syndrome
D Cushing syndrome
E 11β-hydroxylase deficiency
ANSWER:A
1510 Increased production of CO2 occurs in all the following EXCEPT
A fever
B emesis
C excess caloric intake
D high levels of physical activity
E increased respiratory muscle work
ANSWER:B
1511 CNS manifestations of respiratory alkalosis include
A psychosis
B anxiety
C asterixis
D paresthesia
E hallucinations
ANSWER:D
1512 Respiratory acidosis occurs in

A severe anemia
B carbon monoxide poisoning
C hypophosphatemia
D hypotension
E Liddle syndrome
ANSWER:C
1513 The MOST important indication of mechanical ventilation in respiratory acidosis is
A Pco2>75 mm Hg
B concomitant metabolic acidosis
C slowly responsive underlying disease
D hypoxia that responds poorly to oxygen
E tiring patient
ANSWER:A
1514 Very-low birth weight infants can have insensible losses of
A 10-20 mL/kg/24 hr.
B 30-40 mL/kg/24 hr.
C 50-60 mL/kg/24 hr.
D 70-80 mL/kg/24 hr.
E 100-200 mL/kg/24 hr.
ANSWER:E
1515
For each 1°C increase in temperature above 38°C leads to increase in maintenance water
needs by
A 5-10%
B 10-15%
C 15-20%
D 20-25%
E 25-30%
ANSWER:B
1516 The best fluid bolus giving to a child with isolated vomiting and severe dehydration is
A normal saline
B ringer lactate
C half-normal saline
D hypertonic (3%) saline
E 5% dextrose + half-normal saline
ANSWER:A

1517
The MOST common manifestation of cerebral edema from an overly rapid decrease of
serum sodium concentration during correction of hypernatremic dehydration is
A irritability
B hyperreflexia
C spastisity
D seizure
E coma
ANSWER:D
1518
Fentanyl anesthesia (used as analgesia to provide pain relief during operative procedures in
neonates) reduces the incidence of all the following postoperative conditions EXCEPT
A acidosis
B tachycardia
C hypotension
D coagulation abnormalities
E intraventricular hemorrhage
ANSWER:B
1519
Postoperative nausea and vomiting (PONV) occurs in the immediate postoperative period,
within the 1st 1-2 hr, or several hours after surgery and anesthesia. All the following may
decrease the incidence of PONV EXCEPT
A hydration
B use of ketorolac
C preoperative fasting
D glucose supplementation
E prophylactic use of ondansetron
ANSWER:C
1520
Malignant hyperthermia is an acute hypermetabolic syndrome that is triggered by
inhalational anesthetic agents and succinylcholine. Of the following, the MOST recognized
clue to the risk of malignant hyperthermia is
A hypercarbia
B rapid onset of fever
C ventricular fibrillation
D acute hemoglobinuria
E rigid clenching of the masseter muscles
ANSWER:E

1521
Postanesthetic apnea is most common within the 1st 12 hr after surgery and has been
reported in premature infants up to 48 hr later. General anesthesia should be avoided (except
for emergency surgery) in full-term infants younger than postconceptual age of
A 44 wk
B 46 wk
C 48 wk
D 50 wk
E 52 wk
ANSWER:A
1522
Upper respiratory illness (URI) can increase airway reactivity for up to 6 wk in both normal
children and children with a history of reactive airway disease. All the following should be
taken in concern when preparing a child for operation with history of URI EXCEPT
A fever
B injected sclerae
C clear rhinorrhea
D productive cough
E increased mucous secretions
ANSWER:C
1523
After Upper respiratory illness (URI), it is generally recommended to avoid general
anesthesia in elective procedures for
A 1-2Wk
B 2-4Wk
C 4-6Wk
D 6-8Wk
E 8-10 wk
ANSWER:C
1524
The child with asthma should be free of wheezing for at least several days before surgery,
even if this necessitates an increase in β-agonist dosage and the addition of steroids. Of the
following, the MOST appropriate preoperative steroids regime in children with asthma (who
are receiving asthma therapy or who have received steroid within the last year) is
A prednisone, 1 mg/kg given 24 and 12 hr before surgery
B prednisone, 2 mg/kg given 24 and 12 hr before surgery
C dexamethason, 0.6 mg/kg given 24 and 12 hr before surgery
D hydrocortisone IV, 5 mg/kg given 24 and 12 hr before surgery

E hydrocortisone IV, 10 mg/kg given 24 and 12 hr before surgery
ANSWER:A
1525
Children with anterior mediastinal masses are at serious risk for airway compromise, cardiac
tamponade, and vascular obstruction. The indication to prohibit general anesthesia is CT scan
showing compression of the airway at the carina
A ≥30%
B ≥40%
C ≥50%
D ≥60%
E ≥70%
ANSWER:C
1526
During general anesthesia in children with Down syndrome, the anesthesiologists should be
aware of all the following EXCEPT
A macroglossia
B cardiac anomalies
C atlantoaxial instability
D upper airway obstruction
E temporo-mandibular sublaxation
ANSWER:E
1527 Of the following, the MOST acceptable hematocrit value for routine elective anesthesia is
A ≥28%
B ≥30%
C ≥32%
D ≥34%
E ≥36%
ANSWER:B
1528
Preoperative psychologic preparation programs decrease the incidence of postoperative
behavioral changes which last for up to 1 mo. Of the following, the MOST appropriate
advice is to use oral
A diazepam
B midazolam
C imipramine
D pentobarbital
E chloral hydrate
ANSWER:B

1529
Aspiration of gastric contents is a perioperative disaster and, if superimposed on lung disease,
may be rapidly fatal. All the following are true about the guidelines for preoperative fasting
EXCEPT
A clear, sweet liquids --- 2hr
B breast milk ------------- 4hr
C infant formula --------- 6hr
D solid food --------------- 8hr
E gelatin -------------------10hr
ANSWER:E
1530 Nystagmus is a sign of poisoning with all the following EXCEPT
A cocain
B alcohols
C ketamine
D anticonvulsant
E dextromethorphan
ANSWER:A
1531
According to the National Poison Data Systems of the American Association of Poison
Control Centers, approximately 50% of poisoning occurs in children
A ≤2 yr old
B ≤4 yr old
C ≤6 yr old
D ≤8 yr old
E ≤10 yr old
ANSWER:C
1532
In poisoning, obtaining an accurate problem-oriented history is of paramount importance.
All the following are historical features that suggest a possible diagnosis of poisoning in
patients without a witnessed exposure EXCEPT
A low levels of household stress
B multiple system organ dysfunction
C sudden alteration of mental status
D age of the child (toddler or adolescent)
E acute onset of symptoms without prodrome
ANSWER:A
1533 Acetone odor is a sign of poisoning with all the following EXCEPT
A cyanide

B methanol
C salicylates
D paraldehyde
E isopropyl alcohol
ANSWER:A
1534 Lacrimation is a sign of poisoning with
A atropin
B salicylate
C imipramin
D acetaminophine
E organophosphates
ANSWER:E
1535 Mydriasis is a sign of poisoning with
A opioids
B clonidine
C hypnotics
D organophosphates
E tricyclic antidepressant
ANSWER:E
1536 Constipation can be seen in poisoning with
A iron
B lead
C colchicine
D cholinergics
E antimicrobials
ANSWER:B
1537 Hematemesis is a recognized finding in the following poisoning EXCEPT
A lead
B iron
C NSAIDs
D caustics
E salicylates
ANSWER:A
1538 Which of the following poisoning may cause hyperglycemia?
A ethanol

B quinine
C caffeine
D Beta blockers
E salicylates (late)
ANSWER:C
1539 All the following poisonings may cause peripheral neuropathy EXCEPT
A lead
B arsenic
C lithium
D mercury
E organophosphates
ANSWER:C
1540 All the following poisonings may have a radiopaque shadow on KUB EXCEPT
A lead
B arsenic
C salicylates
D calcium carbonate
E enteric-coated pills
ANSWER:C
1541 Prolonged PR interval is a sign of poisoning with
A lithium
B fluconazole
C erythromycin
D carbamazepine
E diphenhydramine
ANSWER:A
1542 Prolonged QTc is a sign of poisoning with
A amiodarone
B carbamazepine
C diphenhydramine
D cardiac glycosides
E tricyclic antidepressants
ANSWER:A

1543
Antidotes are available for relatively few toxins, but early and appropriate use of an antidote
is a key element in managing the poisoned patient. The following matching are true EXCEPT
A β Blockers ---------------- glucagon
B opioids -------------------- naloxone
C iron ------------------ deferoxamine
D lead ------------- BAL (dimercaprol)
E isoniazid ------------------- thiamine
ANSWER:E
1544 The following matching are true regarding poisoning and antidotes EXCEPT
A acetaminophen ------ N-Acetylcysteine
B anticholinergics --------- physostigmine
C benzodiazepines ------------ nalorphene
D carbon monoxide---------- 100% oxygen
E organophosphates --------- pralidoxime
ANSWER:C
1545
The majority of poisonings in children are from ingestion, although exposures can also
occur via inhalational, dermal, and ocular route. All the following are true about
decontamination EXCEPT
A the goal is to minimize absorption of the toxic substance
B decontamination should be routinely employed for every poisoned patient
C water should not be used for decontamination after exposure to phosphorus
D the efficacy of the intervention decreases with increasing time since exposure
E the specific method, depends on the properties of the toxin and the route of exposure
ANSWER:B
1546 All the following poisonings may cause QTc prolongation EXCEPT
A lithium
B cisapride
C amiodarone
D antipsychotics
E clarithromycin
ANSWER:A

1547
Dermal and ocular decontamination begin with removal of any contaminated clothing and
particulate matter, followed by flushing of the affected area with tepid water or normal
saline. Of the following, water should be used for dermal decontamination after exposure to
A phosphorus
B calcium oxide
C organophosphate
D elemental sodium
E titanium tetrachloride
ANSWER:C
1548
Gastrointestinal () decontamination is a controversial topic among medical toxicologists
and is most likely to be effective in the 1st hour after an acute ingestion. GI decontamination
can be done even 1 hr after ingestion of the following EXCEPT
A antipsychotics
B anticholinergics
C massive pill ingestions
D enteric-coated salicylates
E sustained-release preparations
ANSWER:A
1549
The majority of poisonings in children are from ingestion, although exposures can also
occur via inhalational, dermal, and ocular routes. All the following are described methods of
decontamination EXCEPT
A gastric lavage
B whole-bowel irrigation
C induced emesis with ipecac
D single-dose activated charcoal
E multiple-dose activated charcoal
ANSWER:E
1550
After a review of the evidence and assessment of the risks and benefits of ipecac use, the
American Academy of Clinical Toxicology have published statements that advise to
A use ipecac with caution
B use ipecac in all poisoning
C abandon the use of ipecac
D use ipecac in some poisoning
E not use ipecac in persistently vomiting patient

ANSWER:C
1551
Although gastric lavage was used routinely for many years, in most clinical scenarios of
poisoning, the use of gastric lavage is no longer recommended due to all the following
EXCEPT
A painful
B time-consuming
C can induce tachycardia
D only removes a fraction of gastric contents
E can delay administration of activated charcoal
ANSWER:C
1552 All the following poisonings may cause QRS prolongation EXCEPT
A sotalol
B quinidine
C chloroquine
D propranolol
E procainamide
ANSWER:A
1553 All the following substances are poorly adsorbed by activated charcoal EXCEPT
A iron
B caustics
C alcohols
D hydrocarbons
E benzodiazepines
ANSWER:E
1554 Hypocalcemia is a sign of poisoning with
A ethanol
B fluoride
C Beta Blockers
D salicylates (late)
E calcium channel blockers
ANSWER:B
1555 Alkalinization of urine is MOST useful in managing toxicity with
A iron
B lead
C methotrexate

D phenothiazine
E cyclophosphoamide
ANSWER:C
1556 All the following drugs toxicity can be removed by hemodialysis EXCEPT
A iron
B methanol
C salicylates
D theophylline
E valproic acid
ANSWER:A
1557
Intralipid emulsions therapy (IET) act by squestering fat-soluble drugs and decreasing their
impact at target organs thus enhance the elimination in case of drug toxicity. All the
following can be eliminated by IET EXCEPT
A diltiazem
B verapamil
C amiodrone
D imipramine
E amitriptyline
ANSWER:C
1558 All the following methods can be used to enhance the elimination of toxins EXCEPT
A hemodialysis
B urinary alkalinization
C whole bowel irrigation
D intralipid emulsions therapy
E multiple-dose activated charcoal
ANSWER:C
1559 Of the following, the SINGLE acute toxic dose of acetaminophen is
A ≥100 mg/kg
B ≥150 mg/kg
C ≥200 mg/kg
D ≥250 mg/kg
E ≥300 mg/kg
ANSWER:C
1560 In toxic ingestion of acetaminophen, the serum level should be measured after
A 2 hr

B 4 hr
C 6 hr
D 8hr
E 10 hr
ANSWER:B
1561 All the following poisonings may cause ataxia EXCEPT
A alcohols
B imipramin
C anticonvulsant
D carbon monoxide
E dextromethorphan
ANSWER:B
1562
When considering the treatment of a patient with acetaminophen poisoning, and after
assessment of the ABCs, it is helpful to place the patient into one of 4 stages. Of the
following, stage 1 is characterize by
A nausea and vomiting
B elevated alanine aminotransferase
C high international normalized ratio
D right upper quadrant abdominal pain
E elevated aspartate aminotransferase
ANSWER:A
1563
The King’s College criteria for consideration of liver transplantation in acetaminophen
poisoning with acute liver failure (stage 3) include all the following EXCEPT
A coagulopathy (INR >6)
B acidemia (serum pH <7.3)
C serum lactic acid >3 mmol/L
D grade III or IV hepatic encephalopathy
E renal dysfunction (creatinine >3.4 mg/dL)
ANSWER:C
1564
In acetaminophen poisoning, when the initiation of N-Acetylcysteine therapy is delayed,
there is the greater the risk of acute liver failure acetylcysteine should be instituted no later
than
A 2 hr after ingestion
B 4 hr after ingestion
C 6 hr after ingestion

D 8 hr after ingestion
E 10 hr after ingestion
ANSWER:D
1565 Wintergreen odor is a sign of poisoning with
A arsenic
B thallium
C selenium
D methyl salicylate
E organophosphates
ANSWER:D
1566
The acute toxic dose of salicylates is >150 mg/kg, more significant toxicity is seen after
ingestions of >300 mg/kg and severe, potentially fatal, toxicity is described after ingestions of
A ≥350 mg/kg
B ≥400 mg/kg
C ≥450 mg/kg
D ≥500 mg/kg
E ≥550 mg/kg
ANSWER:D
1567 In early course of acute salicylism, the MOST dominate lab values are
A metabolic acidosis and hypoglycemia
B metabolic acidosis and hyperglycemia
C respiratory alkalosis and hypoglycemia
D metabolic acidosis and normoglycemia
E respiratory alkalosis and hyperglycemia
ANSWER:E
1568
Salicylate ingestions are classified as acute or chronic, and acute toxicity is far more common
in pediatric patients. Which of the following is not a recognized feature of salicylate
poisoning?
A tinnitus
B tachypnea
C diaphoresis
D bradycardia
E hyperthermia
ANSWER:D

1569
A 2.5 year-old- child brought to emergency room immediately upon finding the boy with
open bottle of some drugs (used by his grandfather). The child has vomited once, fussy, and
lethargic. Physical evaluation reveals a 13-kg child who has tachypnea and tachycardia.
Laboratory results include a blood sugar=225mg/dl, pH=7.5, PCO2=20 mEq/L, and
HCO3=20 mEq/L. Of the following, the MOST likely offending drug is
A iron
B atropine
C salicylate
D methylxanthine
E tricyclic antideprssnt
ANSWER:C
1570 Bradycardia is a sign of poisoning with
A antipsychotics
B anticholinergics
C antidepressants
D methylxanthines
E calcium channel blockers
ANSWER:E
1571
In salicylate poisoning, hyperglycemia (early) and hypoglycemia (late) have been described.
Of the following, the MOST appropriate clue to administer IV glucose is
A vomiting
B convulsion
C palpitation
D diaphoresis
E altered mental status
ANSWER:E
1572
Chronic salicylism have an insidious presentation, and patients can show marked toxicity at
significantly lower salicylate levels than in acute toxicity. Of the following, the MOST
appropriate primary mode of therapy in chronic salicylism is
A IV glucose
B hemodialysis
C urinary alkalinization
D volume resuscitation
E gastric decontamination
ANSWER:C

1573
Ibuprofen is well tolerated even in overdose. The acute dose of ibuprofen that result in more
serious effects (altered mental status and metabolic acidosis) is
A >100 mg/kg
B >200 mg/kg
C >300 mg/kg
D >400 mg/kg
E >500 mg/kg
ANSWER:D
1574
In acute toxic ingestion of ibuprofen, symptoms usually develop within 4-6 hr of ingestion
and resolve within 24 hr. All the following can result from acute ingestion of variable dose
of ibuprofen EXCEPT
A gastric ulcers
B CNS depression
C abdominal pain
D renal insufficiency
E nausea and vomiting
ANSWER:A
1575 Of the following, the MOST important manifestation of hydrocarbons toxicity is
A dysrhythmias
B hepatic toxicity
C intestinal necrosis
D bacterial pneumonitis
E aspiration pneumonitis
ANSWER:E
1576 Salivation is a sign of poisoning with all the following EXCEPT
A mercury
B ketamine
C salicylates
D corrosives
E organophosphates
ANSWER:A
1577
The most important manifestation of hydrocarbon toxicity is aspiration pneumonitis via
inactivation of the type II pneumocytes and resulting surfactant deficiency. Of the following,
the propensity of a hydrocarbon to cause aspiration pneumonitis is
A inversely proportional to its viscosity

B inversely proportional to its viscosity and volatility
C directly proportional to its viscosity and volatility
D inversely proportional to its viscosity, and directly proportional to its volatility
E directly proportional to its viscosity, and inversely proportional to its volatility
ANSWER:D
1578
Hydrocarbons with low viscosity and high volatility (kerosene, gasoline) spread rapidly
across surfaces and cover large areas of the lungs when aspirated. The quantity of aspirated
hydrocarbon chemicals that can produce significant injury is less than
A 1 mL
B 3 mL
C 5 mL
D 7 mL
E 10 mL
ANSWER:A
1579
Certain hydrocarbons have unique toxicities and can cause symptoms after ingestion,
inhalation, or dermal exposures. Of the following, the MOST likely cause of acute
myelogenous leukemia is long-term exposure to
A gasoline
B benzene
C kerosene
D nitrobenzene
E methyl alcohol
ANSWER:B
1580 Methemoglobinemia may result from exposure to
A gasoline
B benzene
C kerosene
D nitrobenzene
E carbon monoxide
ANSWER:D
1581 All the following are clinical manifestations of hydrocarbons poisoning EXCEPT
A transient mild CNS depression
B cough is the first clinical finding
C chest radiographs may be initially normal
D pneumatoceles can appear 2-3 wk after exposure

E fever and leukocytosis indicate bacterial superinfection
ANSWER:E
1582 Dysrhythmias may result from exposure to
A benzene
B kerosene
C nitrobenzene
D carbon tetrachloride
E halogenated hydrocarbons
ANSWER:E
1583
Carbon monoxide (CO) is a colorless, odorless gas produced during the combustion of any
carbon-containing fuel (the less efficient the combustion, the greater the amount of CO
produced). All the following are true about CO poisoning EXCEPT
A CO displaces nitric oxide (NO) from proteins
B NO is responsible for headache, syncope, and hypotension
C HbCO levels are well correlated with clinical signs of toxicity
D CO bind to cytochrome oxidase, disrupt cellular respiration
E CO binds to hemoglobin with an affinity >200 times that of oxygen
ANSWER:C
1584
Early symptoms of carbon monoxide (CO) poisoning are nonspecific and include headache,
malaise, nausea, vomiting, and the skin may appear
A blue
B pale
C normal
D grayish
E cherry-red
ANSWER:E
1585
In ambient air, the average half-life of HbCO is 4-6 hr, while after administration of 100%
oxygen the half-life is
A 0.5-1 hr
B 1-1.5 hr
C 1.5-2 hr
D 2-3 hr
E 3-4 hr
ANSWER:B
1586 All the following poisoning may cause QTc prolongation EXCEPT

A sotalol
B propranolol
C ketoconazole
D erythromycin
E phenothiazines
ANSWER:B
1587
Caustics include acids and alkalis as well as a few common oxidizing agents, strong acids and
alkalis can produce severe injury even in small-volume ingestions. All the following are true
about a caustic poisoning EXCEPT
A pH of <5 produce significant injury
B pH of >12 produce significant injury
C acids produce a coagulative necrosis
D alkalis produce a liquefaction necrosis
E prolong contact produce significant injury
ANSWER:A
1588
Clinical manifestations of organophosphate toxicity relate to the accumulation of
acetylcholine at peripheral nicotinic and muscarinic synapses and in the CNS. All the
following are symptoms of cholinergic excess at muscarinic receptors EXCEPT
A miosis
B emesis
C salivation
D bradycardia
E urine retention
ANSWER:E
1589
Nicotinic signs and symptoms of organophosphates poisoning include all the following
EXCEPT
A tachycardia
B hypotension
C fasciculation
D hypoventilation
E muscle weakness
ANSWER:B
1590 All the following are true about treatment of organophosphate poisoning EXCEPT
A activated charcoal is unlikely to be of benefit
B without treatment, symptoms can persist for weeks

C atropine is primarily targeted to correct bradycardia
D washing exposed skin and removing exposed clothing
E pralidoxime breaks the bond between the enzyme and the organophosphate
ANSWER:C
1591 All the following are done in the initial assessment of digoxin toxicity EXCEPT
A ECG
B serum calcium
C serum potassium
D renal function test
E serum digoxin level
ANSWER:B
1592 All the following poisoning may cause hypoglycemia EXCEPT
A quinine
B ethanol
C sulfonylureas
D beta Blockers
E calcium channel blockers
ANSWER:E
1593 All the following matching are true EXCEPT
A salicylates ----------------- sodium bicarbonate
B organophosphates --------------------- atropine
C methemoglobinemia --------- methylene blue
D calcium channel blockers ---------------- insulin
E tricyclic antidepressants ------------ flumazenil
ANSWER:E
1594
Digoxin has a very narrow therapeutic index, therapeutic plasma digoxin concentrations are
0.5-2.0 ng/mL; a level >2 ng/mL is considered toxic and a level >6 ng/mL is considered
potentially fatal. All the following medications increase serum digoxin concentration
EXCEPT
A tetracyclin
B amiodarone
C itraconazole
D clarithromycin
E spironolactone
ANSWER:A

1595
In digoxin toxicity, Fab fragments bind free digoxin in both the intravascular and the
interstitial space to form a pharmacologically inactive complex that is subsequently renally
eliminated. All the following are indications of Fab fragments EXCEPT
A renal failure
B hypertension
C ingestion >4 mg
D K+ value >5-5.5 mEq/L
E life threatening dysrhythmias
ANSWER:B
1596
Iron-containing products remain widely available, with the most potentially toxic being adult
iron preparations. Of the following, the severity of iron toxicity is MOSTLY related to the
A age of child
B time of presentation
C type of salt preparation
D number of tablets ingested
E amount of elemental iron ingested
ANSWER:E
1597
The amount of elemental iron ingested that should be referred to medical care for evaluation
is more than
A 10 mg/kg
B 20 mg/kg
C 30 mg/kg
D 40 mg/kg
E 50 mg/kg
ANSWER:D
1598
A 2.5 year-old-boy presented with tachycardia, pallor, and fatigue. The history reveals
excessive iron ingestion with unknown duration. Of the following, the MOST likely time of
ingestion is
A 30 min-6hr ago
B 6hr-24 hr ago
C 12hr-36 hr ago
D 36hr-48hr ago
E 48hr-72hr ago
ANSWER:B

1599 Cyanosis (unresponsive to oxygen) is a sign of poisoning with
A atropine
B amiodarone
C vancomycin
D carbon monoxide
E elemental mercury
ANSWER:B
1600
Iron is directly corrosive to the GI mucosa, leading to hematemesis, melena, ulceration,
infarction, and potential perforation. Of the following, the MOST appropriate method of
gastrointestinal decontamination in iron toxicity is
A induced emesis with ipecac
B gastric lavage
C activated charcoal
D whole-bowel irrigation
E all of the above
ANSWER:D
1601
Cardiovascular (CV) and CNS symptoms dominate the clinical presentation of tricyclic
antidepressant (TCA) toxicity, symptoms typically develop within 1-2 hr of ingestion, and
serious toxicity usually manifests within 6 hr of ingestion. Of the following, the MOST
common CV manifestation is
A hypotension
B sinus tachycardia
C widened QRS interval
D ventricular arrhythmias
E prominent R wave in lead aVR
ANSWER:B
1602 The MOST common cause of death in tricyclic antidepressant (TCA) overdose is
A seizures
B renal injury
C respiratory failure
D refractory hypotension
E ventricular arrhythmias
ANSWER:D
1603
Asymptomatic child with tricyclic antidepressant toxicity, should be observed with
continuous cardiac monitoring and serial ECGs for at least

A 4hr
B 6hr
C 8hr
D 10hr
E 12hr
ANSWER:B
1604
Probiotics are oral supplement or a food product that contains a sufficient number of viable
microorganisms (live microorganisms that are most often referred to as "friendly" or "good"
bacteria). Probiotics are indicated in all the following EXCEPT
A constipation
B irritable bowel syndrome
C protien losing enteropathy
D antibiotic-associated diarrhea
E inflammatory bowel disorders
ANSWER:C
1605
Dietary supplements (as herbs) are commonly used and may be helpful adjunctive treatments
for common childhood problems. All the following matching are true EXCEPT
A Aloe vera --------------------------------- lice
B Ginger -------------------------------- Nausea
C Chamomile ----------------------- dyspepsia
D Tea tree oil ----------------- acne remedies
E Peppermint -- Irritable bowel syndrome
ANSWER:A
1606 The MOST common reason for a sick child visit is
A fever
B altered mental status
C vomiting
D respiratory distress
E abdominal pain
ANSWER:A
1607
In pediatrics medical emergencies the survival rate can increase with good neurological
outcome if rapid, effective, cardiopulmonary resuscitation (CPR) is done. The survival rate
can reach up to
A 10%
B 30%

C 50%
D 70%
E 90%
ANSWER:B
1608 The first response to unwitnessed unresponsive infant is to
A activate emergency response system
B check pulse
C start rescue breathing
D start chest compression
E do endotracheal intubation
ANSWER:A
1609 The features of high quality CPR include all the following EXCEPT
A rate at least 100/min
B compression depth of 2/3 anterior-posterior diameter of the chest
C allow complete chest recoil after each compression
D minimize interruptions in chest compressions
E avoid excessive ventilation
ANSWER:B
1610 After activation of emergency medical system, the next action to unresponsive child is to
A asses the air ways
B check pulse
C give rescue breathings
D start chest compression
E attach automated external defibrillator (AED) to find shakable rhythm
ANSWER:B
1611
A 12-month-old infant found unresponsive in kindergarten, you were there as health visitor,
you asked for activation of EMS and to bring a nearby automated external defibrillator
(AED) machine, you check the pulse; it was 60/min. Your next response is to
A open the air ways
B give one breath every 3 seconds
C begin cycles of 30 compressions with 2 breaths
D begin a cycles of 15 compressions with 2 breaths
E attach and use AED
ANSWER:B
1612 The correct statement regarding the lower limit of systolic blood pressure for a neonate is

A ≥40 mm Hg + (2 × age)
B ≥50 mm Hg + (2 × age)
C ≥60 mm Hg + (2 × age)
D ≥70 mm Hg + (2 × age)
E ≥80 mm Hg + (2 × age)
ANSWER:C
1613 The MOST common precipitating event for cardiac instability in infants and children is
A electrolyte disturbances
B trauma
C respiratory insufficiency
D poisoning
E myocarditis
ANSWER:C
1614 Regarding Glasgow Coma scale in pediatrics, all the following are true EXCEPT
A in modified type it uses 15 score points
B it has 3 components
C valid as a prognostic scoring system
D score ≤8 require aggressive management
E verbal response component has 5 possible points
ANSWER:C
1615
In pediatrics advanced life support (PALS) curriculum, regarding the format of ABCDE, all
the following are true EXCEPT
A A refers to assessment of airways
B B refers to assessment of breathing
C C refers to assessment of circulation
D D refers to assessment of dehydration
E E refers to assessment of unidentified injuries
ANSWER:D
1616 In secondary assessment of critically ill child; all the following are true EXCEPT
A interviewing a wittiness by medical personnel that involved in resuscitation
B taking focused history
C performing a focused medical examination
D enquire about allergies and medications
E enquire about timing of the last meal
ANSWER:A

1617 All the following are part of tertiary assessment of critically ill child EXCEPT
A coagulation profile
B renal function
C echocardiography
D arterial and venous catheters
E lumbar puncture
ANSWER:E
1618
A 2-year-old toddler was playing with candies and sweets in playroom, he developed an
attack of wheezing and shortness of breath, the parents brought the child to emergency
room, RR was 68/min with subcostal retractions, and O2 saturation was 92%. Of the
following, the MOST proper next action is to
A arrange for urgent bronchoscopy
B arrange for bedside CXR
C arrange for CT chest
D encourage coughing
E reassure the parents
ANSWER:A
1619
You received a call from your neighbor, he had a 12-month-old baby boy who suddenly
chock with a candy. When you arrive you find the baby unconscious. Of the following, the
NEXT proper action is to
A open airway and give rescue breathing
B give 5 back blows
C give 5 chest thrusts
D a series of 5 back blows and chest thrusts
E perform endotracheal intubation
ANSWER:A
1620 All the following are an absolute indications for endotracheal intubation EXCEPT
A inability to protect the airway against aspiration
B failing to maintain adequate oxygenation
C complete airway obstruction
D failing to control blood carbon dioxide levels
E paralysis is required for a procedure
ANSWER:C
1621
All the following factors are attributing in definition of clinically significant bradycardia
EXCEPT

A heart rate
B temperature
C perfusion
D mental status
E blood pressure
ANSWER:B
1622 The MOST common pre-arrest rhythms in young children is
A bradyarrhythmia
B atrial flutter
C ventricular fibrillation
D supraventricular arrhythmia with WPW syndrome
E atrioventricular block
ANSWER:A
1623
You are evaluating a 3-year-old conscious child with bradycardia and poor perfusion in the
emergency department. Of the following, the MOST important first step in treatment is
A maintaining adequate perfusion by chest compression
B maintaining the airways and assisted breathing
C treating precipitating factors as hypoglycemia
D epinephrine therapy
E atropine therapy
ANSWER:B
1624 Atropine is used in the treatment of bradyarrhythmia that is associated with
A raised intracranial pressure
B hypoxia
C hypothermia
D tension pneumothorax
E hyperkalemia
ANSWER:A
1625
Factors in favour of SVT versus sinus tachycardia in pediatrics include the following
EXCEPT
A abrupt history
B polymorphic P wave
C absence of P wave
D poor perfusion
E constant RR

ANSWER:D
1626
You were engaged in CPR for 3-year-old child found unresponsive in the ward. You were
performing cycles of 15 chest compressions and 2 breathings alternatively with another
medical personal. After 2 min of resuscitation you evaluate the pulse which shows PEA
(pulseless electrical activity) status. The AED already is attached. Of the following, the
correct NEXT action is to
A give a shock of 0.5 joule/kg
B give a shock of 1 joule/kg
C give a shock of 2 joules/kg
D give adrenaline 0.01mg/kg iv
E give adrenaline 0.1mg/kg iv
ANSWER:D
1627
You were engaged in CPR for 3-year-old child found unresponsive in the ward. You were
performing cycles of 15 chest compressions and 2 breathings alternatively with another
medical personal. After 2 min of resuscitation the assistant told you that the pulse status is
shockable. This means that the pulse is in status of
A pulseless electrical activity PEA
B asystole
C VT
D AF
E SVT
ANSWER:C
1628 The perfusion pressure of brain (cerebral perfusion pressure) equals to
A intracranial pressure
B systolic blood pressure minus intracranial pressure
C mean blood pressure minus Intracranial pressure
D mean blood pressure
E mean blood pressure plus CSF pressure
ANSWER:C
1629
All the following are recognized brain herniation due to increase intracranial pressure
EXCEPT
A cingulate
B cerebral tonsillar
C transcalvrial
D uncal

E upward cerebellar
ANSWER:B
1630
A 6-year-old male child brought to emergency department after been hit by a car while
riding his bike. GCS was 6 and brain CT was normal. However, immediate MRI
demonstrates multiple areas of punctate hemorrhages consistent with diffuse axonal injury.
Regarding the classification of traumatic brain injury (TBI), the child in this scenario has
A mild TBI
B moderate TBI
C severe TBI
D pure axonal brain injury
E parenchymal brain hemorrhage
ANSWER:C
1631
All the following are signs of increased intracranial pressure (ICP) and impending brain
herniation EXCEPT
A pupillary dilation
B 4th cranial nerve palsy
C systemic hypertension
D bradycardia
E extensor posturing
ANSWER:B
1632 All the following are regarded as first tier therapy of increased intracranial pressure EXCEPT
A intubation
B controlled mechanical ventilation
C head of bed elevation
D sedation and analgesia
E barbiturate infusion
ANSWER:E
1633 All the following are important to diagnose brain death status in a child EXCEPT
A deep coma of unknown cause
B apnea
C absence of brainstem reflexes
D absence of motor response
E absence of confounding factors

ANSWER:A
1634 All the following are red flags in evaluating a patient with syncope EXCEPT
A syncope with exercise
B family history of syncope
C presyncopal feeling of light headedness
D history of Kawasaki disease
E injury with syncope
ANSWER:C
1635
An 11-year-old adolescent boy came to your clinic with a complaint of been collapsed while
bathing with hot water after he felt dizzy. Father confirms that the boy was extremely pale
when founded. He added that he toke few minutes to recover, also he had past history of
similar condition one month ago when he was urinating. Of the following, the MOST likely
explanation for this condition is
A long QT syndrome
B hypertrophic cardiomyopathy
C neurocardiogenic syncope
D drug abuse
E seizure disorder
ANSWER:C
1636
Of the following, the mandatory test/study for all patients presenting for the first time with
syncope is
A ECG
B EEG
C echocardiography
D holter monitoring
E complete blood count
ANSWER:A
1637 Ductus-dependent congenital heart lesions is a type of
A distributive shock
B obstructive shock
C cardiogenic shock
D hypovolemic shock
E septic shock
ANSWER:B

1638
You are treating an 8-month-old baby in intensive care unit with cold shock. In the first 15
min he received adequate fluid therapy and a colleague had started dopamine for him. The
baby condition is still unresponsive. Of the following, the BEST option now is to start
A epinephrine
B hydrocortisone
C norepinephrine
D terlipressin
E angiotensin
ANSWER:A
1639
Septic shock usually shows a combination of hypovolemic and distributive shock in addition
to
A obstructive shock
B cardiogenic shock
C acute respiratory distress syndrome
D disseminated intravascular coagulation
E decreased steroid synthesis
ANSWER:B
1640 The percentage of Fio2 that can be delivered via the nasal cannula is up to
A 30%
B 40%
C 50%
D 60%
E 80%
ANSWER:B
1641 The percentage of Fio2 that can be delivered via a mask is up to
A 35%
B 45%
C 65%
D 75%
E 85%
ANSWER:C
1642
Rapid and deep breathing without other signs of respiratory distress may be caused by the
following EXCEPT
A diabetic ketoacidosis
B renal tubular acidosis

C encephalitis
D CNS stimulants
E heart failure
ANSWER:E
1643 All the following are controlled mode of mechanical ventilation EXCEPT
A pressure support ventilation (PSV)
B conventional mechanical ventilation (CMV)
C intermittent mechanical ventilation (IMV)
D synchronized Intermittent mechanical ventilation (SIMV)
E synchronized Intermittent positive pressure ventilation (SIPPV)
ANSWER:A
1644
A 2-hour-old fullterm baby delivered to a mother with gestational diabetes by CS, he
developed an increasing respiratory distress and grunting with blood gases values indicative
of respiratory failure. Of the following, the BEST mode to be used in mechanical ventilation
is
A intermittent mechanical ventilation (IMV)
B synchronized Intermittent mechanical ventilation (SIMV)
C synchronized Intermittent positive pressure ventilation (SIPPV)
D pressure-regulated volume control (PRVC)
E pressure support ventilation (PSV)
ANSWER:D
1645 In mechanical ventilation giving adequate PEEP can mainly reduces
A volutrauma
B barotrauma
C oxytrauma
D pneumonia
E post-extubation airway obstruction
ANSWER:C
1646
All the following are recognized complications of mechanical ventilation which can result
from use of high tidal volume and/or inspiratory pressure EXCEPT
A volutrauma
B barotrauma
C oxytrauma
D decrease surfactant production
E atelectotrauma

ANSWER:E
1647
You are ventilating a fullterm baby with meconium aspiration syndrome, he was fine on the
machine, suddenly the monitor shows severe drop in O2 saturation. You checked the
ventilator; it was working well with its connections. Of the following, the MOST likely cause
is
A O2 source supply
B endotracheal tube blockade
C self extubation
D low inspiratory pressure value
E low PEEP value
ANSWER:B
1648
A 32-week-preterm baby on mechanical ventilation, he is recovering from RDS, his blood
gas is satisfactory, and you reduced the ventilatory settings in process for weaning. Of the
following, the MOST valuable indicator that the baby is ready for extubation is
A frequency of respiration 35
B fraction of inspired o2 0.55
C peak expiratory end pressure 6
D inspiratory pressure 8
E inspiratory time 0.4 sec
ANSWER:D
1649
Prior to extubation and in order to reduce airway narrowing after extubation it is advised to
use
A dexamethasone IV
B betamethasone IV
C inhaled budesonide
D nebulized budesonide
E hydrocortisone IV
ANSWER:A
1650 The MOST effective strategy to reduce ventilator-associated pneumonia (VAP) is
A reducing the period of ventilation
B elevation of the head of the bed to 30 degrees
C use of a protocol for oral decontamination
D liberal use of effective antibiotics
E frequent endotracheal tube suction
ANSWER:A

1651 In surviving drowning patients; expecting brain edema can occur within
A 5 min
B 15 min
C 30 min
D 60 min
E several hours
ANSWER:E
1652
Of the following, the MOST common factor that increases the likelihood of drowning in
adolescent (USA) is
A epilepsy
B long QT syndrome
C alcohol
D myocarditis
E external trauma
ANSWER:C
1653 All the following mechanisms are contributing in pathophysiology of drowning EXCEPT
A laryngospasm
B surfactant washout
C anoxic injury
D pulmonary aspiration
E circulatory overload
ANSWER:E
1654
All the following are component of cold water shock resulted from immersion in cold water
EXCEPT
A hypoventilation
B decrease breath holding ability
C hypertension
D ectopics
E SVT
ANSWER:A
1655
You are treating a 6-year-old child who survived drowning after prolonged CPR, the mother
is asking about the possibility of full neurological recovery of her child. Of the following,
the BEST response is that it is possible if he regained full consciousness within
A few hours
B 12 hours

C 24 hours
D 48 hours
E 72 hours
ANSWER:E
1656
When an individual is at-risk genotype and not clinically express the condition; this
phenomena is called
A genetic imprinting
B genetic anticipation
C uniparental diasomy
D non expressiveness
E non penetrance
ANSWER:E
1657
The genetic testing in an asymptomatic child with positive family history of Huntington
chorea is called
A predisposional testing
B predictive testing
C diagnostic testing
D pharmacogentic testing
E linkage testing
ANSWER:B
1658
A 5-year-old asymptomatic male child with strong positive family history of Huntington
chorea, mother is highly concern about her child possible genetic affection, she asked for
predictive genetic testing. Of the following, the MOST appropriate explanation is that
A whatever the result it will not affect the disease process
B this is an age dependent type of genetic disorder and may not manifest ever
C leave the decision to the child when he is grown up
D it may lead to discrimination
E we should weigh the benefits VS risks of testing
ANSWER:C
1659 Predispostional genetic testing is more useful in
A numerical chromosomal disorders
B multifactorial disorders
C single gene disorders
D structural chromosomal disorders
E pre arrangement for gene therapy

ANSWER:B
1660
A 4-year-old female child, a known case of ALL, she is developing an increasing toxicity to
methotrexate therapy. Of the following, the MOST useful test is
A homocysteine level
B prothrombin test
C factor V leiden
D MTHFR DNA testing
E folate level
ANSWER:D
1661 The following are indications for genetic counseling EXCEPT
A paternal age > 50 years
B maternal age > 45 years
C family history of intellectual disability
D consanguinity
E infertility
ANSWER:B
1662 The following statements are true about genetic counseling EXCEPT
A constructing family pedigree up to the third degree
B collecting health files about affected family members
C performing appropriate genetic testing
D refer to support groups
E talking about rewarding gene therapy
ANSWER:E
1663 Enzyme replacement therapy are available for the following disorders EXCEPT
A Wolman disease
B Gaucher disease
C Fabry disease
D Mucopolysaccharidosis type VI
E Pompe disease
ANSWER:A
1664 DiGeorge syndrome is an example of
A contiguous gene disorders
B single gene disorders
C mitochondrial inheritance disorders
D numeric chromosomal disorders

E imprinting disorders
ANSWER:A
1665
The phenomena of changing phenotypes (clinical presentations) associated with the same
single-gene disorder is called
A variable expressiveness
B variable penetrance
C false positive
D false negative
E validity
ANSWER:A
1666 The MOST important screening tool for genetic disorders is
A genetic study
B family history
C karyotyping
D prenatal diagnosis
E preimplantation genetic screen
ANSWER:B
1667 One of the following is explained by digenetic inheritance
A vitamin D dependent rickets
B retinitis pigmentosa
C biotinidase deficiency
D congenital lactic acidosis
E glycogen storage disease type VI
ANSWER:B
1668
Parent-to-child transmission (vertical transmission) is a character of autosomal dominant
inheritance, for many patients with an autosomal dominant disorder there is no history of an
affected family member. All the following are true explanations EXCEPT
A variable expressiveness
B new mutation
C somatic mutations (mosaicism)
D digenetic inheritance
E incomplete penetrance
ANSWER:D
1669 The following are characteristic feature of autosomal dominant inheritance EXCEPT
A vertical transmission

B male to male transmission
C males and females are equally affected
D skipped generation
E both parents carried the affected gene
ANSWER:E
1670 All the following are characteristic features of autosomal recessive inheritance EXCEPT
A vertical transmission
B no affected family members in other generations
C males and females are equally affected
D consanguinity plays important role
E 25% recurrence risk
ANSWER:A
1671
The risk of a genetic disorder for the offspring of a first-cousin marriage (consanguinity
marriage) is
A 3-5%
B 6-8%
C 9-11%
D 12-14%
E 15-17%
ANSWER:B
1672 Pseudodominant inheritance happens when
A an x-linked disorder appears as autosomal dominant
B an autosomal dominant appears as recessive trait
C an autosomal recessive disorder appears as autosomal dominant
D skipped generations appears
E an autosomal dominant appears as x-linked disorder
ANSWER:C
1673 In genetic disorders, male to male transmission occurs in Y- linked inheritance and in
A X- linked inheritance
B pseudodominant inheritance
C digenetic inheritance
D autosomal dominant disorder
E pseudogenetic inheritance
ANSWER:D

1674
Females sometimes manifest signs of X- linked recessive disorders. All the following are
possible explanations EXCEPT
A homozygosity for an X-linked trait
B 45,XO female
C 46,XY female
D uniparental disomy
E nonrandom X-inactivation
ANSWER:D
1675 All the following are features of mitochondrial inheritance EXCEPT
A non-traditional inheritance
B maternal inheritance
C male to offspring transmission
D both sexes are affected
E mitochondrial DNA point mutations
ANSWER:C
1676 The organ that can be spared in mitochondrial disorders is
A brain
B eye
C small intestine
D liver
E muscle
ANSWER:C
1677 The following represent a non-traditional inheritance EXCEPT
A mitochondrial disorders
B triplet repeat expansion diseases
C imprinting defects
D familial clustering
E uniparental disomy
ANSWER:D
1678 The following are a triplet repeat expansion disorders EXCEPT
A fragile X syndrome
B myotonic dystrophy
C Huntington disease
D spinocerebellar ataxias
E Kearns-Sayre syndrome

ANSWER:E
1679
The observation of increasing severity of the disease and early age of onset in subsequent
generation is called
A genetic imprinting
B genetic anticipation
C uniparental diasomy
D non expressiveness
E non penetrance
ANSWER:B
1680
The observation of significantly increasing number of imprinting disorders as Beckwith-
Wiedemann and Angelman syndrome are MOST likely due to
A new methods of prenatal diagnosis (amniocentesis)
B new technology of gene therapy
C assisted reproductive technology
D increase radiation exposure
E high pressure electricity chains in large cities
ANSWER:C
1681 The following genetic scenarios are possible to explain Angelman syndrome EXCEPT
A inheriting both chromosomes 15 from the father
B deletion of the maternal 15q12
C mutation in an imprinted gene
D mutation in the imprinting center
E genetic anticipation
ANSWER:E
1682 The MOST common genetic cause of Prader-Willi syndrome is
A inheriting both chromosomes 15 from the mother
B deletion of the maternal 15q12
C deletion of the paternal 15q12
D mutation in an imprinted gene
E mutation in the imprinting center
ANSWER:C
1683
The recurrence risk of multifactorial inheritance are characterized by all the following
EXCEPT
A related to the incidence of the disease in community
B similar among all 1st-degree relatives

C no sex predilection
D increased when multiple family members are affected
E greater when the disorder is more severe
ANSWER:C
1684 Chromosomal analysis is indicated in all the following EXCEPT
A recurrent premature deliveries
B postnatal problems in growth and development
C unexplained intellectual disability
D primary amenorrhea
E recurrent miscarriages (≥3)
ANSWER:A
1685 Chromosomal abnormalities in live births constitute around
A 1-2%
B 3-4%
C 5-6%
D 7-8%
E 9-10%
ANSWER:A
1686
A 17-year-old adolescent female with trisomy 21 became pregnant, the chance of affection
of her offspring with the same problem is
A Not possible
B 25%
C 50%
D 75%
E 100%
ANSWER:C
1687
All the following are recognized gastrointestinal features of Down syndrome in neonatal
period EXCEPT
A annular pancreas
B tracheoesophageal fistula
C diaphragmatic hernia
D imperforate anus
E neonatal cholestasis
ANSWER:D

1688
The MOST important reason for chromosomal analysis in persons suspected of having Down
syndrome is to
A confirm clinical diagnosis
B detect mosaicism
C reassure the family
D detect translocations
E detect rare cases of ring chromosome
ANSWER:D
1689
A chromosomal study of 22-year-old mother of a baby with Down syndrome t (14; 21)
shows that; she is the carrier for the translocation. You explained that the recurrence rate will
be approximately
A 2-4%
B 5-7%
C 8-10%
D 11-13%
E 33%
ANSWER:B
1690
A highly educated young couple has their first baby with Down syndrome; they are asking
you about his development. Of the following, the MOST appropriate answer is
A programs of stimulation will achieve eventual normal development
B motor development is the mostly affected domain with fair cognitive delay
C developmental delay will be in the initial first few years, but it will normalize later on
D developmental delay is universal, all domains of development almost equally affected
E developmental delay is universal, social development is relatively spared but he will have
ANSWER:E
1691
You are explaining the motor development to a mother with a baby with Down syndrome,
you told her that there is a wide range of time to achieve his milestone and as example the
baby can walk between
A 10-18 mo
B 12-24 mo
C 12-45 mo
D 15-50 mo
E 18-60 mo
ANSWER:C

1692
A 5-year-old child with Down syndrome complaining of intermittent symptoms of torticollis,
weakness of the lower limbs and gait disturbances. Of the following, the NEXT important
step is to
A reassure the family
B send for radiological assessment of the neck
C send for thyroid antibodies
D send for radiological assessment of the airway
E send for thyroid function test
ANSWER:B
1693
Quad screen is the four maternal serum tests that should be done for all pregnant women in
second trimester as a screening for Down syndrome, which includes the following tests
EXCEPT
A free β-human chorionic gonadotropin
B unconjugated estriol
C androsterone
D inhibin
E α-fetoprotein
ANSWER:C
1694 All the following are features of Edwards syndrome (Trisomy 18) EXCEPT
A low birth weight
B closed fists
C cardiac malformations
D short sternum
E slopping forehead
ANSWER:E
1695 All the following are features of Patau syndrome (Trisomy 13) EXCEPT
A scalp defects
B corneal abnormalities
C holoprosencephaly
D capillary hemangiomas
E prominent occiput
ANSWER:E
1696
Robertsonian translocations involve 2 acrocentric chromosomes that fuse near the
centromeric region with a subsequent loss of the short arms. It can occur in the following
chromosomes EXCEPT

A 13
B 15
C 17
D 21
E 22
ANSWER:C
1697 One of the common chromosomal deletions which can lead to retinoblastoma is
A 4p-
B 5p-
C 9p-
D 13q-
E 18p-
ANSWER:D
1698 All the following are examples of microdeletion syndromes EXCEPT
A Williams
B Prader-Willi
C Angelman
D DiGeorge
E Wolf-Hirschhorn
ANSWER:E
1699
An 18-month-old female child had been diagnosed as Turner syndrome (45, XO), FISH
analysis shows Y chromosome mosaicism. Of the following, the MOST appropriate next step
is to
A refer to endocrinologist
B reassure as the course will be milder
C advise for feminizing hormones
D arrange for laparoscopic removal of gonads
E rear the child as male gender
ANSWER:D
1700 All the following are recognized features of Turner syndrome EXCEPT
A female phenotype
B male phenotype
C hypothyroidism
D type I diabetes mellitus
E gonadal dysgenesis

ANSWER:D
1701
All the following are characteristic features of fragile X chromosome of a 3-year-old male
child EXCEPT
A intellectual disability
B autistic behavior
C macroorchidism
D hyperextensible finger joints
E characteristic facial features
ANSWER:C
1702
Inborn errors of metabolism (IEM) are hereditary biochemical disorders caused by single-
gene mutations that result in alteration of
A chromosomal number or structure
B mitochondrial synthesis or function
C primary fat structure or the amount of fat synthesized
D primary protein structure or the amount of protein synthesized
E primary carbohydrate structure or the amount of carbohydrate synthesized
ANSWER:D
1703
Inborn errors of metabolism (IEM) are hereditary biochemical disorders caused by single-
gene mutations that encode specific proteins. The following are the common characteristics
of IEM EXCEPT
A the majority of conditions are inherited as autosomal recessive
B the later the appearance of clinical symptoms, the more severe is the disease
C early diagnosis is of paramount importance before irreversible organ damage
D the affected infant is normal at birth and becomes symptomatic later on in life
E there is a variation in the severity of the phenotype based on the gene mutation
ANSWER:B
1704
All the following are widely used modalities for treatment of inborn errors of metabolism
(IEM) EXCEPT
A special diets
B gene therapy
C peritoneal dialysis
D liver transplantation
E administration of the deficient metabolite
ANSWER:B

1705
A 28-day-old girl presented with lethargy, poor feeding, and repeated vomiting for last 5
days; IV fluid and empirical antibiotics were started; later she develops repeated seizures not
responding to IV calcium, glucose, B6, and anticonvulsant drugs. Septic screen is negative;
serum ammonia is elevated with normal anion gap and normal pH. Family history reveals 2
siblings died with same scenario. Of the following, the MOST likely diagnosis is
A galactosemia
B hyperglycinemia
C organic acidemias
D phenylketonurea
E urea cycle defects
ANSWER:E
1706
You suspect a metabolic problem in a 30-day-old girl presented with poor feeding,
vomiting, lethargy, and convulsion. Previous sibling died with the same condition. Serum
ammonia, pH, HCO3, and anion gap are normal. Of the following, the MOST likely
diagnosis is
A porphyria
B galactosemia
C organic acidemia
D urea cycle defect
E aminoacidopathy
ANSWER:E
1707
A 3-year-old boy presented with history of hyperactivity and mild mental retardation. Past
history revealed repeated projectile vomiting with normal abdominal sonography. On
examination, there are eczematoid rash with lighter skin, microcephaly, and mild spasticity
with exaggerated tendon reflexes. Of the following, the MOST likely urine odor of this baby
is
A musty
B rotting fish
C sweaty feet
D boiled cabbage
E swimming pool
ANSWER:A

1708
A 6-year-old boy with eczematoid rash, abnormal face, and prominent maxilla complains
from deterioration in school performance, hyperactivity, and seizures for the last 2 years. Of
the following, the MOST likely enzymatic deficiency is
A acid β-glucosidase
B β-hexosaminidases
C acid sphingomyelinase
D phenylalanine hydroxylase
E fumarylacetoacetate hydrolase
ANSWER:D
1709
A 5-year-old mentally retarded girl presented with repeated convulsions and abnormal EE
finding. On examination, she had a light skin, small head, and exaggerated reflexes. Of the
following, the MOST appropriate method to confirm the diagnosis is
A uthrie test
B flow cytometry
C tandem mass spectrometry
D measurement of phenylketones in the urine
E measurement of plasma phenylalanine concentration
ANSWER:E
1710
In the classic phenylketonurea, the affected infant is normal at birth, intellectual disability
develops gradually if the infant remains untreated, and cognitive delay may not be evident
for the first few months. All the following are true about neonatal screening program of this
condition EXCEPT
A uthrie test was the 1st method used
B optimal time for Guthrie test at 7-14 of life
C blood phenylalanine may rise to a diagnostic level as early as 4 hr after birth
D fluorometric and tandem mass spectrometry give a low false-positive rate
E diagnosis must be confirmed by measurement of plasma phenylalanine concentration
ANSWER:B
1711
The mainstay of treatment of phenylketonurea (PKU) is especial diet. All the following
statements are true EXCEPT
A phenylalanine not totally restricted
B all patients kept on a phenylalanine-restricted diet lifelong
C especial diet should be used before and during pregnancy in mother with PKU
D especial diet should be started when blood phenylalanine levels above 10 mg/dL
E especial diet is not advocated in mild hyperphenylalaninemia whose levels are ≥ 6

ANSWER:E
1712
A 4-month-old boy presented with failure to thrive, rapid breathing, and repeated vomiting.
On examination, there is hepatomegaly and abdominal sonography revealed renal
calcification. Of the following, the MOST likely urine odor of this infant is
A mousey
B rotting fish
C sweaty feet
D boiled cabbage
E swimming pool
ANSWER:D
1713
A 10-month-old boy presented with failure to thrive, fever, jaundice, hepatomegaly, and
severe rickets. Investigations revealed hypoglycemia and normal anion gap metabolic
acidosis. Of the following, the MOST likely enzyme deficiency is
A acid β-glucosidase
B β-hexosaminidases
C acid sphingomyelinase
D phenylalanine hydroxylase
E fumarylacetoacetate hydrolase
ANSWER:E
1714
A 15-month-old boy had recurrent hospital admissions because of fever, jaundice,
subcutaneous bleeding, and hypoglycemic fits; on examination, there is hepatomegaly. In
the last attack, the child developed severe legs pain associated with retraction of the neck and
trunk. Of the following, the MOST likely diagnosis is
A galactosemia
B organic acidemia
C urea cycle defect
D tyrosinemia type 1
E maple syrup urine disease
ANSWER:D
1715
The MOST appropriate method to confirm the diagnosis of tyrosinemia type 1 is by elevated
level of
A α-fetoprotein
B plasma tyrosine
C serum methionine
D serum succinylacetone

E urinary 5-aminolevulinic acid
ANSWER:D
1716
A 10-day-old boy presented with lethargy, poor feeding, repeated vomiting, and recurrent
hypoglycemic fits not responding to IV glucose. On examination, he was hypertonic with
severe opisthotonos alternating with flaccidity. Of the following, the MOST likely diagnosis is
A tyrocinemia
B galactosemia
C phenyletonuria
D homocystinurea
E maple syrup urine disease
ANSWER:E
1717
A 27-day-old girl presented with abnormal bicycling movement, lethargy, poor feeding, and
repeated vomiting for the last 7 days, and then she developed rapid breathing, opisthotonos,
and hypertonicity with bad odor urine. Of the following, the MOST effective mode of
therapy is
A good hydration
B peritoneal dialysis
C liver transplantation
D sufficient calories and nutrients
E diet low in branched-chain amino acids
ANSWER:B
1718 The following are ocular manifestations of albinism EXCEPT
A red reflex
B strabismus
C refractive errors
D foveal hyperplasia
E lack of binocular vision
ANSWER:D
1719
A 6-year-old tall and thin boy who had failed preparatory exam for school entry found to
have myopia and subluxation of the ocular lens. Of the following, the MOST important
challenge is to know whether he is responsive to
A folic acid
B vitamin C
C vitamin E

D vitamin B6
E vitamin B12
ANSWER:D
1720
A 6-year-old girl that looks tall and thin with light skin. On examination she has a peculiar
malar flush, subluxation of the ocular lens, developmental delay, and severe hypertension.
Of the following,the MOST likely cause of hypertension is
A atherosclerosis
B hyperthyroidism
C thromboembolism
D coarctation of aorta
E renal artery stenosis
ANSWER:C
1721 The MOST appropriate treatment of homocystinuria is
A betaine
B folic acid
C vitamin C
D vitamin B6
E methionine restriction
ANSWER:D
1722
The diagnosis of homocystinuria is usually made after 3 yr of age when the ophthalmologist
found
A cataracts
B glaucoma
C astigmatism
D ectopia lentis
E retinal detachment
ANSWER:D
1723
An 11-month-old boy presented with repeated convulsions, poor eye contact, exaggerated
startle response to noise, and large head. Ophthalmic exam revealed cherry-red spot. Of the
following, the MOST likely diagnosis is
A Tyrosinemia
B Fabry disease
C Gaucher disease
D Tay-Sachs disease
E Niemann-pick disease

ANSWER:D
1724
One of the following is a characteristic feature for both Infantile Sandhoff disease and Tay-
Sachs disease
A splenomegaly
B hepatomegaly
C cardiac involvement
D retinal cherry-red spots
E mild bony abnormalities
ANSWER:D
1725
A 12-year-old boy presented with epistaxis, mild bruising, intermittent bone pain, and
massive splenomegaly. Complete blood count shows thrombocytopenia and mild anemia.
Radiological study revealed lytic lesions and Erlenmeyer flask deformity of the distal femur.
Of the following, the MOST likely diagnosis is
A Gaucher disease
B Sandhoff disease
C Tay-Sachs disease
D von Gierke disease
E Niemann-pick disease
ANSWER:A
1726
A 9-year-old boy admitted to the hospital because of fracture of left femur due to a minor
trauma. Past history revealed recurrent attacks of epistaxis and intermittent generalized bone
pain. On examination, there is massive splenomegaly. Of the following, the MOST
appropriate practical treatment of this disease is
A gene therapy
B liver transplantation
C enzyme replacement
D substrate reduction agents
E bone marrow transplantation
ANSWER:C
1727
A 2-year-old boy presented with failure to thrive, regression of developmental milestone,
and difficult to contact with surroundings. On examination, there are hepatosplenomegaly
and moderate lymphadenopathy. Of the following, the MOST likely diagnosis is
A Gaucher disease
B Sandhoff disease
C Tay-Sachs disease

D Von Gierke disease
E Niemann-pick disease
ANSWER:E
1728 All the following are recognized feature of Niemann-pick disease type B EXCEPT
A normal IQ
B progressive lung disease
C prolonged neonatal jaundice
D subtle or no neurologic involvement
E cherry-red maculae in some patients
ANSWER:C
1729 All the following lipidoses are inherited as an autosomal recessive traits EXCEPT
A Fabry disease
B Krabbe disease
C Gaucher disease
D Tay-sachs disease
E Niemann-pick disease
ANSWER:A
1730 Niemann-pick disease type A is characterized by all the following EXCEPT
A failure to thrive
B hepatosplenomegaly
C progressive lung disease
D psychomotor retardation
E rapidly progressive neurodegenerative
ANSWER:C
1731
A 7-month-old boy presented with recurrent attacks of hypoglycemic seizures especially
during an acute illness. On examination, there are thin extremities, protuberant abdomen,
and hepatomegaly. All the following are biochemical hallmarks of this disease EXCEPT
A lactic acidosis
B hypoglycemia
C hyperuricemia
D hyperlipidemia
E markedly elevated transaminase levels
ANSWER:E

1732
A 2-year-old girl presented with rapid deep breathing, hypoglycemia, recurrent epistaxis,
easy bruising, failure to thrive, fat cheek, and hepatomegaly. Of the following, the MOST
likely cause of bleeding tendency is
A vascular disorders
B clotting factors deficiency
C Impaired hepatic function
D depressed platelets count
E impaired platelet aggregation and adhesion
ANSWER:E
1733
A 2-year-old boy presented with growth retardation, hepatomegaly, and recurrent attacks of
hypoglycemia; biochemical study reveals elevated blood lactate, triglyceride, and uric acid
levels. All the following are complications of this disease EXCEPT
A pancreatitis
B renal failure
C cardiomyopathy
D hepatic adenoma
E pulmonary hypertension
ANSWER:C
1734
A 3-year-old girl presented with growth retardation, recurrent attacks of otitis media,
diarrhea, and hypoglycemic seizures. On examination, there is hepatomegaly and her
absolute neutrophil count (ANC) is 600/µL. Of the following, the MOST likely diagnosis is
A Kostmann disease
B cyclic neutropenia
C drug-induced neutropenia
D glycogen storage disease Ib
E Shwachman-Diamond syndrome
ANSWER:D
1735 The definitive diagnosis of type I glycogen storage disease GSD is by
A EMG
B liver biopsy
C muscle biopsy
D analysis of gene-based mutation
E enzyme assay of cultured skin fibroblasts
ANSWER:B

1736
In von Gierke disease; treatment is designed to maintain normal blood glucose levels which
is achieved by all the following EXCEPT
A restriction of fructose
B restriction of galactose
C administration of table sugar
D continuous nasogastric infusion of glucose
E oral administration of uncooked corn starch
ANSWER:C
1737
A 9-month-old boy presented with persistent head lag, rapid breathing, difficult feeding, and
hepatomegaly. Chest X- ray showed cardiomegaly. Of the following, the MOST appropriate
confirmatory test is
A liver biopsy
B muscle biopsy
C electrocardiography
D serum creatine phosphokinase
E enzyme assay in dried blood spots
ANSWER:E
1738
A 7-month-old girl presented with recurrent chest infection, chocking during feeding, large
protruded tongue, persistent head lag, and hepatomegaly. Chest X-ray showed cardiomegaly.
All the following are beneficial in the treatment of this patient EXCEPT
A exercise therapy
B high-protein diet
C cardiac transplantation
D nocturnal ventilatory support
E specific enzyme replacement therapy
ANSWER:C
1739
A 28-day-old boy presented with prolonged neonatal jaundice, lethargy, poor feeding, and
failure to regain birth weight. On examination there are hepatomegaly and cataract. Of the
following, the MOST likely injured organs in this disease are
A eyes, liver, and brain
B kidney, liver, and eyes
C kidney, liver, and brain
D kidney, heart , and brain
E pancreas, liver, and brain
ANSWER:C

1740 Irreversible complication of untreated galactosemia is
A sepsis
B cataract
C renal injury
D myocardial damage
E intellectual disability
ANSWER:E
1741
A 20-day-old girl presented with prolonged neonatal jaundice, lethargy, and poor feeding
treated as sepsis by empirical antibiotics and nothing by mouth. The patient got partial
response, when oral feeding resumed, the condition deteriorate again. In this condition, the
blood culture MOST likely reveals growth of
A Escherichia coli
B Staphylococcal aureus
C group B streptococcus
D Listeria monocytogens
E Pseudomonas aeruginosa
ANSWER:A
1742
A 9-month-old boy with exclusive breast feeding presented with jaundice, vomiting,
lethargy, hepatomegaly, and hypoglycemic fits when sweetened cereal added. Laboratory
findings showed prolonged clotting time, hypoalbuminemia, elevation of bilirubin and
transaminase levels, and proximal tubular dysfunction. Of the following, the MOST likely
diagnosis is
A tyrosinemia
B galactosemia
C organic acidemias
D maple syrup urine disease
E hereditary fructose intolerance
ANSWER:E
1743
All the following are common features between hereditary fructose intolerance and
galactosemia EXCEPT
A failure to thrive
B impaired renal function
C impaired hepatic function
D reducing substance in urine
E impaired intellectual development

ANSWER:E
1744 All the following should be restricted in hereditary fructose intolerance EXCEPT
A milk
B sorbitol
C fruit juice
D table sugar
E sweetened cereal
ANSWER:A
1745 The earliest radiographic sign in Hurler syndrome is
A thick ribs
B thickened calvarium
C enlarged J-shaped sella
D abnormal spacing of teeth
E premature closure of lambdoid suture
ANSWER:A
1746
Enzyme replacement is the main therapy of Hurler disease, it improve all the following
EXCEPT
A growth rate
B joint mobility
C organomegaly
D cognition functions
E episodes of sleep apnea
ANSWER:D
1747
A 9-year-old boy presented with tiredness on exertion; cardiac auscultation reveals a murmur
in aortic area; echo study showed aortic valve disease. On examination he had mild coarse
facial features, corneal clouding, joint stiffness, and mild dysostosis multiplex but normal
intelligence and stature. Of the following, the MOST likely diagnosis is
A Hurler disease
B Scheie disease
C Hunter disease
D Morquio disease
E Sanfilippo disease
ANSWER:B
1748 Hunter disease is characterized by all the following features EXCEPT
A short stature

B corneal clouding
C dysostosis multiplex
D intellectual disability
E coarse facial features
ANSWER:B
1749 All the following mucopolysaccharidoses are inherited as autosomal recessive EXCEPT
A Hurler disease
B Scheie disease
C Hunter disease
D Morquio disease
E Sanfilippo disease
ANSWER:C
1750 Which of the following mucopolysaccharidoses have normal intelligence?
A Sly disease
B Hurler disease
C Hunter disease
D Morquio disease
E Sanfilippo disease
ANSWER:D
1751
A 5-year-old boy presented with chronic diarrhea. On examination he had
hepatosplenomegaly, coarse facial features, joint stiffness, clear cornea, and short stature. Of
the following, the MOST likely diagnosis is
A Hurler disease
B Hunter disease
C Scheie disease
D Morquio disease
E Sanfilippo disease
ANSWER:B
1752
In mucopolysaccharidosis, disproportionate severe CNS involvement with mild physical
features is unique to
A Scheie disease
B Hurler disease
C Hunter disease
D Morquio disease
E Sanfilippo disease

ANSWER:E
1753 All the following are skeletal manifestations of Morquio disease EXCEPT
A kyphosis
B genua valga
C waddling gait
D short trunk and neck
E bullet-shaped phalanges
ANSWER:E
1754
Which of the following mucopolysaccharidoses is susceptible to life-threatening atlantoaxial
instability and dislocation?
A Hurler disease
B Scheie disease
C Hunter disease
D Morquio disease
E Sanfilippo disease
ANSWER:D
1755 The characteristic radiographic finding of Hurler disease is
A kyphosis
B genua valgus
C V-shaped configuration of the digits
D hook-shaped appearance of L1 vertebra
E ovoid configuration of the vertebral bodies
ANSWER:D
1756 A definitive diagnosis of mucopolysaccharidoses disorder is established by
A liver biopsy
B enzyme assay
C radiographic features
D bone marrow examination
E urinary glycosaminoglycan (GAG) test
ANSWER:B
1757 Acute intermittent porphyria (AIP) could be exacerbated by all the following EXCEPT
A surgery
B puberty
C pregnancy
D premenstrual period

E decreased carbohydrate intake
ANSWER:C
1758 All the following drugs are unsafe in acute intermittent porphyria EXCEPT
A nifedipine
B diclofenac
C cimetidine
D valproic acid
E carbamazepine
ANSWER:C
1759 All the following are clinical features of acute intermittent porphyria EXCEPT
A neuropathy
B tachycardia
C abdominal pain
D bladder dysfunction
E cutaneous photosensitivity
ANSWER:E
1760 The least common presentation of acute intermittent porphyria is
A sensory loss
B mental symptoms
C muscle weakness
D abdominal tenderness
E pain in the extremities
ANSWER:D
1761 The MOST common electrolyte abnormality in acute intermittent porphyria is
A hyperkalemia
B hyponatremia
C hypercalcemia
D hypomagnesemia
E hypophosphatemia
ANSWER:B
1762 All the following are features of congenital erythropoietic porphyria (CEP) EXCEPT
A neuropathy
B reddish urine
C erythrodontia
D photosensitivity

E neonatal hyperbilirubinemia
ANSWER:A
1763 The MOST important screening test of acute intermittent porphyria is
A plasma porphyrins
B fibroblast porphyrins
C fecal porphobilinogen
D erythrocyte porphyrins
E urinary porphobilinogen
ANSWER:E
1764
In older infants and children, hypoglycemia is defined as whole blood glucose concentration
less than
A 45 mg/dL
B 50 mg/dL
C 55 mg/dL
D 60 mg/dL
E 65 mg/dL
ANSWER:C
1765 The least well-defined long-term sequelae of severe prolonged hypoglycemia is
A cerebral palsy
B personality affection
C cognitive impairment
D recurrent seizure activity
E autonomic dysregulation
ANSWER:B
1766
The following are counter regulatory hormones which are increased as blood glucose falls
EXCEPT
A cortisol
B glucagon
C epinephrine
D thyroid hormone
E growth hormone
ANSWER:D

1767
A 3-year-old boy presented to emergency unit at 9:00 AM with history of difficult arousal
from sleep, flue like illness, and low grade fever; the mother stated that he had slept without
having his supper. Fasting blood sugar was 42 mg/dL. The following metabolic changes are
seen in this condition EXCEPT
A ketonuria
B low insulin level
C high alanine level
D high cortisol level
E high growth hormone level
ANSWER:C
1768
At 10:00 AM Lylaa`s mother brought her 3-year-old daughter to emergency department
with difficult arousal from sleep, low grade fever, and cough for the last 2 days. Fasting
blood sugar was 36 mg/dL with ketonuria. Of the following, the MOST likely diagnosis is
A galactosemia
B propionic acidemia
C ketotic hypoglycemia
D hereditary fructose intolerance
E medium-chain acyl-CoA dehydrogenase (MCAD) deficiency
ANSWER:C
1769
A 5-year-old child with recurrent attacks of ketotic hypoglycemia. Of the following, the
BEST advice to the parents during any intercurrent illness is to
A administrate a high-protein diet
B test the child’s urine for ketones
C administrate a high-carbohydrate diet
D do frequent monitoring of blood sugar
E administrate intravenous glucose administration
ANSWER:B
1770
The MOST specific and important laboratory finding in X-linked adrenoleukodystrophy
(ALD) is elevated level of
A cortisol
B aldosterone
C epinephrine
D very long chain fatty acid
E adrenocorticotrophic hormone
ANSWER:D

1771
In X-linked adrenoleukodystrophy (ALD),MRI shows characteristic cerebral white matter
lesions mainly in
A frontal lobes
B temporal lobes
C frontoparietal lobes
D parietoccipital lobes
E temporoparietal lobes
ANSWER:D
1772 All the following are secondary causes of hypercholesterolemia EXCEPT
A obesity
B cholestasis
C hypothyroidism
D anorexia nervosa
E nephrotic syndrome
ANSWER:A
1773 All the following drugs are secondary causes of hypertriglyceridemia EXCEPT
A tegretol
B estrogen
C thiazides
D β-blockers
E anabolic steroids
ANSWER:A
1774 Smith Lemli-Opitz Syndrome (SLOS) is characterized by all the following EXCEPT
A epispadias
B cleft palate
C microcephaly
D retromicrognathia
E mental retardation
ANSWER:A
1775
All the following are recognized laboratory findings of medium-chain acyl-CoA
dehydrogenase (MCAD) deficiency EXCEPT
A hypoglycemia
B hypoketonuria
C hypoketonemia
D metabolic acidosis

E elevated liver enzyme
ANSWER:D
1776
A 3-year-old boy presented to emergency unit at 10:00 AM with history of difficult arousal
from sleep, lethargy, vomiting, and seizures; the mother stated that he had slept without
having his supper. Fasting blood sugar was 38mg/dL and urinary ketone was low. Of the
following, the MOST likely diagnosis is
A galactosemia
B propionic acidemia
C ketotic hypoglycemia
D hereditary fructose intolerance
E medium-chain acyl-CoA dehydrogenase (MCAD) deficiency
ANSWER:E
1777 The MOST common presentation of primary carnitine deficiency is
A liver disease
B cardiomyopathy
C skeletal myopathy
D endocardial fibroelastosis
E hypoketotic hypoglycemia
ANSWER:B
1778 All the following are clinical abnormalities of Zellweger syndrome EXCEPT
A cataract
B micrognathia
C high forehead
D small fontanells
E redundant skin fold of neck
ANSWER:D
1779 All the following are features of neonatal adrenoleukodystrophy (ALD) EXCEPT
A hepatomegaly
B impaired liver function
C severely impaired hearing
D chondrodysplasia punctata
E pigmentary degeneration of the retina
ANSWER:D
1780 All the following are characteristic features of infantile Refsum disease EXCEPT
A ataxic gait

B hypertonia
C hearing loss
D impaired cognition
E retinal degeneration
ANSWER:B
1781 The diagnostic assay of Zellweger syndrome is by elevated level of
A bile acid
B plasmalogen
C phytanic acid
D pristanic acid
E very long chain fatty acid
ANSWER:E
1782 The normally functioning organ in Hutchinson-Gilford progeria syndrome (progeria) is
A eye
B ear
C skin
D liver
E heart
ANSWER:D
1783 The MOST common initial manifestation of X-linked adrenoleukodystrophy (ALD) is
A seizure
B hyperactivity
C visual disturbance
D increased intracranial pressure
E impaired auditory discrimination
ANSWER:B
1784 The MOST common presentation of mitochondrial β-oxidation of fatty acids disorders is
A cardiomyopathy
B muscle weakness
C renal tubulopathy
D hypoglycemic coma
E acute rhabdomyolysis
ANSWER:D
1785
The specific clue to the diagnosis of fatty acid oxidation disorders may be the finding of
hypoglycemia and

A hypoketonuria
B metabolic acidosis
C elevated liver enzyme
D elevated blood ammonia
E prolonged prothrombin time
ANSWER:A
1786 Xanthomas are a feature of all the following EXCEPT
A familial hypercholesterolemia
B familial dysbetalipoproteinemia
C familial combined hyperlipidemia
D familial chylomicronemia (Frederickson type I)
E familial hypertriglyceridemia(Frederickson type V)
ANSWER:C
1787 Major cause of neonatal mortality in full-term newborn is
A respiratory distress syndrome
B necrotizing enterocolitis
C bronchopulmonary dysplasia (BPD)
D congenital anomalies
E intraventricular hemorrhage
ANSWER:D
1788 Generalized edema of the newborn may occur in the following conditions EXCEPT
A prematurity
B nonimmune hydrops
C Turner syndrome
D congenital nephrosis
E Hurler syndrome
ANSWER:C
1789
Soft areas in the occipital region suggest the irregular calcification and wormian bone
formation usually associated with the following conditions EXCEPT
A osteogenesis imperfecta
B craniosynostosis
C cleidocranial dysostosis
D cretinism
E Down syndrome
ANSWER:B

1790 The following disorders are associated with a large anterior fontanel EXCEPT
A congenital rubella syndrome
B hypophosphatasia
C Apert syndrome
D Russell-Silver syndrome
E vitamin A deficiency
ANSWER:E
1791
Leukokoria (white pupillary reflex) in newborn infant suggests the following disorders
EXCEPT
A cataracts
B tumor
C congenital glaucoma
D chorioretinitis
E retinopathy of prematurity
ANSWER:C
1792 A single umbilical artery in a newborn infant increases the risk for
A meningomyelocele
B occult renal anomaly
C omphalocele
D gastroschisis
E omphalitis
ANSWER:B
1793 The following drugs can be given with caution to breast-feeding mother
A psychotropic drugs
B amphetamines
C bromocriptine
D chloramphenicol
E methimazole
ANSWER:A
1794 Of the following, the condition which is associated with polyhydramnios is
A renal agenesis (Potter syndrome)
B Prune-belly syndrome
C pulmonary hypoplasia
D intestinal pseudo-obstruction
E diaphragmatic hernia

ANSWER:B
1795 Low maternal serum α-fetoprotein (MSAFP) is associated with
A open neural tube defects
B trisomy 21
C gastroschisis
D omphalocele
E congenital nephrosis
ANSWER:B
1796 A delay in fetal pulmonary maturation may be associated with
A hydrops fetalis
B severe premature separation of the placenta
C premature rupture of the fetal membranes
D narcotic addiction
E maternal hypertensive and renal vascular disease
ANSWER:A
1797
True umbilical cord knots are seen in approximately 1% of births and are associated with the
following conditions EXCEPT
A short cord
B small fetal size
C polyhydramnios
D monoamniotic twinning
E fetal demise
ANSWER:A
1798 Regarding multiple gestation pregnancies, the following are true EXCEPT
A the reported incidence of spontaneous twinning is lowest in the Asian​​ races
B triplets are estimated to occur in 1 in 862 pregnancies in the USA
C the incidence of monozygotic twins is unaffected by racial or familial factors
D the overall incidence of multifetal gestation is unchanged
E polyovular pregnancies are more frequent beyond the 2nd pregnancy
ANSWER:D
1799
One of the following is a common neonatal respiratory problem associated with premature
infants
A bronchopulmonary dysplasia
B apnea
C congenital pneumonia

D pneumothorax
E pneumomediastinum
ANSWER:B
1800 One of the following drugs may cause pyloric stenosis if administered to a premature infant
A intravenous vitamin E
B indomethacin
C enteric gentamicin
D prostaglandins
E dexamethasone
ANSWER:D
1801
The following factors pose a risk for poor academic performance of premature infants
EXCEPT
A birthweight below 750 g
B periventricular leukomalacia
C antenatal exposure to magnesium sulfate
D bronchopulmonary dysplasia
E posthemorrhagic hydrocephalus
ANSWER:C
1802
The incidence of all the following are increased in large for gestational age newborn
EXCEPT
A cephalohematoma
B hypocalcemia
C hypoglycemia
D congenital heart disease
E developmental retardation
ANSWER:B
1803 Sudden onset of hypotension in a very low birthweight (VLBW) infant suggests
A pneumothorax
B necrotizing enterocolitis
C bacterial sepsis
D myocarditis
E hypoglycemia
ANSWER:A
1804 Seizures beginning in the delivery room or shortly thereafter may be due to

A hypoxic-ischemic encephalopathy
B unintentional injection of maternal local anesthetic into the fetus
C intracranial hemorrhage
D cerebral anomaly
E hypoglycemia
ANSWER:B
1805 After severe birth asphyxia, infants may have motor automatisms characterized by
A abscence of oral-buccal-lingual movements
B time-synchronized electroencephalographic discharges
C significant cortical epileptic activity
D good response to anticonvulsant therapy
E a poor prognosis
ANSWER:E
1806 Vomiting in the neonatal period is usually due to
A pyloric stenosis
B milk allergy
C overfeeding
D stress ulcer
E an inborn error of metabolism
ANSWER:C
1807 Regarding cephalohematoma, all the following are true EXCEPT
A is a subperiosteal hemorrhage
B it may extend across the midline and across suture lines
C occur in 1-2% of live births
D no discoloration of the overlying scalp
E an underlying skull fracture may be associated with 10-25% of cases
ANSWER:B
1808 Regarding intraventricular hemorrhage (IVH) of prematurity, the following are true EXCEPT
A it usually develops spontaneously
B MRI is the preferred imaging technique for screening IVH
C prophylactic administration of low-dose indomethacin reduces the incidence of severe IVH
D majority of patients with IVH have no clinical symptoms
E it may rarely manifest at birth

ANSWER:B
1809
Early postnatal exposure to dexamethasone, within the 1st wk of life, in VLBW infants, is
associated with the following EXCEPT
A metabolic derangements
B poor growth
C increased risk for sepsis
D increased risk of spontaneous bowel perforation
E cerebral palsy
ANSWER:E
1810 The following are true regarding apnea EXCEPT
A apnea is a common problem in preterm infants
B in term infants, apnea is always worrisome
C obstructive apnea is characterized by absence of airflow but persistent chest wall motion
D serious apnea is defined as cessation of breathing for longer than 20 sec
E bradycardia follows the apnea by 1-2 sec in more than 95% of cases and is most often nodal
ANSWER:E
1811
The risk for development of respiratory distress syndrome RDS increases with the following
conditions EXCEPT
A prolonged rupture of membranes
B maternal diabetes
C multiple births
D precipitous delivery
E cold stress
ANSWER:A
1812 The following are true regarding respiratory distress syndrome (RDS) EXCEPT
A signs of RDS usually appear within minutes of birth
B in most cases, the symptoms and signs reach a peak within 7days
C apnea and irregular respirations are ominous signs
D improvement is often heralded by spontaneous diuresis
E mixed respiratory-metabolic acidosis may be seen
ANSWER:B
1813 One of the following conditions can mimic RDS both clinically and radiographically
A persistent pulmonary hypertension
B meconium aspiration syndrome

C total anomalous pulmonary venous return
D pulmonary lymphangiectasia
E lobar emphysema
ANSWER:C
1814
Administration of antenatal corticosteroids to women between 24 and 34 wk of gestation
significantly reduces the following EXCEPT
A incidence and mortality of RDS
B postnatal growth
C the overall neonatal mortality
D need for and duration of ventilatory support
E incidence of severe intraventricular hemorrhage
ANSWER:B
1815 The basic defect requiring treatment in RDS is
A metabolic acidosis
B circulatory insufficiency
C hypothermia
D inadequate pulmonary exchange of oxygen and carbon dioxide
E electrolytes disturbance
ANSWER:D
1816 Regarding CPAP, the following are true EXCEPT
A its prevents collapse of surfactant-deficient alveoli
B its indicated, if oxygen saturation cannot be kept > 95%
C its improves functional residual capacity FRC
D its improves ventilation-perfusion matching
E its reduces ventilatory needs
ANSWER:B
1817
Prophylactic and rescue administrations of synthetic and natural surfactants have the
following advantages EXCEPT
A reduces adverse outcomes
B reduces neonatal mortality
C decreses the risk for pneumothorax
D decreses the risk for pulmonary interstitial emphysema
E reduces bronco-pulmonary dysplasia BPD rates
ANSWER:E

1818
Vitamin A supplementation given largely to infants < 1,000 g resulted in all the following
EXCEPT
A a decrease in death
B a decrease in bronco-pulmonary dysplasia BPD at 36 wk
C less nosocomial sepsis
D less retinopathy of prematurity
E decreases the need for extracorporeal membrane oxygenation (ECMO)
ANSWER:E
1819
Alkali therapy for the treatment of metabolic acidosis in RDS may result in all the following
EXCEPT
A skin slough from infiltration
B increased serum osmolarity
C hypernatremia
D hypocalcemia
E hyperkalemia
ANSWER:E
1820
Contributary factors for the development of broncho-pulmonary dysplasia (BPD) include the
following EXCEPT
A immaturity
B dehydration during the 1st days of life
C chorioamnionitis
D symptomatic PDA
E malnutrition
ANSWER:B
1821
Methylxanthines for the treatment of broncho-pulmonary dysplasia (BPD) have the
following effects EXCEPT
A increase respiratory drive
B decrease apnea
C improve diaphragmatic contractility
D increase pulmonary vascular resistance PVR
E diuretic effects
ANSWER:D
1822
One of the following is FALSE in the chest radiograph of transient tachypnea of the
newborn (TTN)
A prominent pulmonary vascular markings

B fluid in the intralobar fissures
C overaeration
D flat diaphragms
E diffuse reticulogranular pattern
ANSWER:E
1823 The following are true regarding meconium aspiration syndrome (MAS) EXCEPT
A (MAS) develops in 5% of meconium-stained infants
B 30% require mechanical ventilation
C 3-5% die
D overdistention of the chest may be prominent
E usually occurs in preterm or near-term infants
ANSWER:E
1824
The following are predisposing factors for persistent pulmonary hypertension of the
newborn (PPHN) EXCEPT
A anemia
B meconium aspiration syndrome
C early-onset sepsis
D hypoglycemia
E birth asphyxia
ANSWER:A
1825
Associated anomalies have been reported in up to 30% of congenital diaphragmatic hernia
cases; these include the following EXCEPT
A pulmonary hypoplasia
B CNS lesions
C esophageal atresia
D omphalocele
E cardiovascular lesions
ANSWER:A
1826
Congenital diaphragmatic hernia (CDH) can be diagnosed on prenatal ultrasonography
(between 16 and 24 wk of gestation) in > 50% of cases
A chest mass
B mediastinal shift
C gastric bubble
D olighydramnios
E a liver in the thoracic cavity

ANSWER:D
1827 A poor prognostic sign of congenital diaphragmatic hernia (CDH) is
A grunting
B use of accessory muscles
C early respiratory distress, within 6 hr of life
D cyanosis
E scaphoid abdomen
ANSWER:C
1828 Eventration of the diaphragm may be associated with the following EXCEPT
A pulmonary hypoplasia
B pulmonary sequestration
C congenital heart disease
D chromosomal trisomies
E recurrent infections
ANSWER:A
1829
All the following are true, regarding pathology of neonatal necrotizing enterocolitis (NEC)
EXCEPT
A in fatal cases, gangrene may extend from the stomach to the rectum
B the greatest risk factor for NEC is prematurity
C in most situations, a pathogen is identified
D NEC is much less common in infants fed human milk
E NEC in term infants is often a “secondary” disease
ANSWER:C
1830 Absolute indication for surgery in neonatal necrotizing enterocolitis (NEC) include
A positive result of abdominal paracentesis
B failure of medical management
C a single fixed bowel loop on radiographs
D abdominal wall erythema
E a palpable mass
ANSWER:A
1831 The following factors are associated with decreased risk of significant jaundice EXCEPT
A gestational age ≥41 wk
B male gender
C exclusive bottle-feeding
D black race

E discharge from hospital after 72 hr
ANSWER:B
1832
Jaundice, consisting of either indirect or direct bilirubin that is present at birth or appears
within the 1st 24 hr of life may be due to all the following EXCEPT
A erythroblastosis fetalis
B Crigler-Najjar syndrome
C concealed hemorrhage
D sepsis
E congenital infections
ANSWER:B
1833
A search to determine the cause of jaundice should be made in all the following conditions
EXCEPT
A if it appears in the first 24-36 hr of life
B if serum bilirubin is rising at a rate faster than 5 mg/dL/24 hr
C if serum bilirubin is >12 mg/dL in a full-term infant
D if jaundice persists after 10-14 days of life
E if direct bilirubin fraction is >1 mg/dL at any time
ANSWER:E
1834 Regarding breast milk jaundice, the following are true EXCEPT
A it develops in an estimated 2% of breast-fed term infants
B
maximal unconjugated bilirubin concentrations as high as 10-30 mg/dL reached during the
2nd-3rd week
C jaundice may persist for 3-10 wk
D phototherapy may be of benefit
E kernicterus never occur
ANSWER:E
1835 The therapeutic effect of phototherapy depends on the following factors EXCEPT
A distance between the lights and the infant
B skin color
C surface area of exposed skin
D rate of hemolysis
E in vivo metabolism and excretion of bilirubin
ANSWER:B
1836 Early ultrasonographic signs of hydrops include
A double–bowel wall sign (bowel edema)

B polyhydramnios
C ascites
D pleural effusions
E scalp edema
ANSWER:A
1837
The blood smear of infant with hemolytic disease of the newborn (erythroblastosis fetalis)
typically shows the following EXCEPT
A polychromasia
B increasedreticulocyte count
C a marked increase in nucleated RBCs
D thrombocytopenia
E low white blood cell count
ANSWER:E
1838
One of the following is FALSE in hemolytic disease of the newborn caused by blood group
A and B incompatibility
A ABO incompatibility occurs in 20-25% of pregnancies
B A1 is more antigenic than A2
C
natural antibodies against A and B factors are usually IgM antibodies that do not cross the
placenta
D
isoimmune hemolytic disease may be found in first-born infants irrespective of infant blood
group
E most cases are mild
ANSWER:D
1839
One of the following is FALSE in hemolytic disease of the newborn caused by Rh
incompatibility
A infant is Rh Positive (D, sometimes C)
B never occur in first-born infant
C maternal antibody titers can help predict the severity of fetal disease
D severe anemia is frequent
E stillbirth/hydrops is frequent
ANSWER:B
1840 One of the following is FALSE in plethora in the newborn infant (polycythemia)
A polycythemia is defined as a central Hct of 65% or higher
B peripheral (heelstick) Hct and Coulter counter results values are higher than central values
C the incidence of neonatal polycythemia is 8% in small for gestational age (SGA) infants

D many affected infants are asymptomatic
E
neonatal RBCs have decreased deformability and filterability, which predispose to stasis in
the microcirculation
ANSWER:B
1841 egarding hemorrhagic disease of the newborn, all the following are true EXCEPT
A age of onset is usually in 2-7 days old infants
B more frequent in breast-fed than in formula-fed infants
C bleeding time is normal
D
oral vitamin K is as effective as intramuscular route in the prevention and treatment of the
disease
E prothrombin time (PT), and partial thromboplastin time (PTT) are prolonged
ANSWER:D
1842 Apt test is useful for diagnosis of
A disseminated intravascular coagulopathy
B swallowed blood syndrome
C hemorrhagic disease of the newborn
D neonatal thrombocytopenic purpura
E congenital deficiency of factor VII
ANSWER:B
1843
The normal cord at term is 55 cm long. Abnormally short cords are associated with the
following EXCEPT
A fetal hypotonia
B wrapping around fetal parts
C uterine constraint
D oligohydramnios
E increased risk for complications of labor and delivery for both mother and infant
ANSWER:B
1844 Surgical repair of umbilical hernia is advised in the following conditions EXCEPT
A if hernia persists to the age of 4-5 yr
B causes symptoms
C hernias that appear before the age of 6 mo
D becomes strangulated
E becomes progressively larger after the age of 1-2 yr
ANSWER:C

1845
Generalized edema may be seen in the neonatal period with the following conditions
EXCEPT
A congenital nephrosis
B Turner syndrome
C Hurler syndrome
D hydrops fetalis
E offspring of diabetic mothers
ANSWER:B
1846 Diabetic mothers have a high incidence of all the following EXCEPT
A oligohydramnios
B preeclampsia
C pyelonephritis
D preterm labor
E chronic hypertension
ANSWER:A
1847 In Infants of diabetic mothers, the following are true EXCEPT
A hypoglycemia develops in about 25-50% of infants of diabetic mothers
B increased weight of the placenta and all infant organs
C heart failure occurs in 5-10% of infants of diabetic mothers
D the incidence of congenital anomalies is increased threefold in infants of diabetic mothers
E lumbosacral agenesis is one of most common congenital anomalies
ANSWER:B
1848 The following definitions are true EXCEPT
A
a syndrome is defined as a pattern of multiple abnormalities that are related by
pathophysiology and result from a common defined etiology
B dysplasia sequence is a poor organization of cells into tissues or organs
C
an association refers to a nonrandom collection of malformations in which there is an unclear
relationship among the malformations
D
disruption sequence is a mechanical (uterine) forces that alter structure of intrinsically normal
tissue
E
malformation sequence is a single, local tissue morphogenesis abnormality that produces a
chain of subsequent defects
ANSWER:D
1849 Chromosomal causes of congenital malformations include
A Prader-Willi syndrome

B X-linked hydrocephalus
C Achondroplasia
D Treacher Collins syndrome
E Apert disease
ANSWER:A
1850 One of the following definitions is FALSE
A brachydactyly=A condition of having short digits
B postaxial polydactyly=Extra finger or toe present on the medial side of the hand or foot
C clinodactyly=A medial or lateral curving of the fingers
D camptodactyly=Permanent flexion of one or more fingers
E syndactyly=Incomplete separation of the fingers
ANSWER:B
1851
Intrauterine transplacental infections of significance to the fetus and/or newborn include the
following EXCEPT
A hepatitis B virus (HBV)
B rubella
C toxoplasmosis
D Parvovirus B19
E varicella
ANSWER:A
1852 Agents that commonly cause nosocomialneonatal infection are
A group B streptococci GBS
B coagulase-negative staphylococci
C proteus
D gonococci
E chlamydiae
ANSWER:B
1853 One of the following is FALSE regarding intrapartum antibiotics
A reduce vertical transmission of GBS
B lessen neonatal morbidity after preterm rupture of membranes
C prevent perinatal transmission of GBS
D reduce the rates of late-onset GBS disease
E has no effect on the rates of infection with non-GBS pathogens
ANSWER:D
1854 Osteitis may be a feature of one of the following transplacental infections

A cytomegalovirus
B herpes simplex virus
C varicella-zoster virus
D rubella
E gondii
ANSWER:D
1855 Intracranial calcification may be a feature of one of the following transplacental infections
A cytomegalovirus
B herpes simplex virus
C varicella-zoster virus
D syphilis
E rubella
ANSWER:A
1856 Limb hypoplasia may be a feature of the following transplacental infections
A cytomegalovirus
B herpes simplex virus
C varicella-zoster virus
D HIV
E rubella
ANSWER:C
1857 Common respiratory tract manifestations of neonatal bacterial infections is
A ethmoiditis
B otitis media
C mastoiditis
D retropharyngeal cellulitis
E empyema
ANSWER:E
1858 Common skin and soft tissue manifestations of neonatal bacterial infections are
A facial cellulitis
B scalp abscess
C fasciitis
D breast abscess
E omphalitis
ANSWER:D

1859
One of the following is not included in the definition of the systemic inflammatory response
(SIRS) in neonates and pediatric patients
A temperature instability
B abnormal white blood cell (WBC) count
C respiratory dysfunction
D cardiac dysfunction
E perfusion abnormalities
ANSWER:B
1860 Neonatal sepsis case fatality rate is highest for
A Staphylococcus—coagulase negative
B Group B streptococcus
C Escherichia coli
D Pseudomonas
E Candida albicans
ANSWER:D
1861
Persistence of the umbilical cord beyond which time should prompt consideration of an
underlying abnormality
A 10 days
B 20 days
C 30 days
D 40 days
E 50 days
ANSWER:C
1862 The MOST important risk factor for necrotizing enterocolitis (NEC) in preterm infants is
A delivery by cesarean section
B breast feeding
C apgar score
D exposure to glucocorticoids during the first week of life
E gestational age and birth weight
ANSWER:E
1863 Pneumatosis intestinalis is pathognomonic for
A Hirschsprung's disease
B necrotizing enterocolitis (NEC)
C pseudomembranous enterocolitis
D neonatal ulcerative colitis

E meconium ileus
ANSWER:E
1864
The following factors suggest hemolytic disease as a cause of jaundice in the newborn
EXCEPT
A bilirubin rise of >0.5 mg/dL/h
B reticulocytosis >5% at birth
C onset of jaundice before 24 hours of age
D significant decrease in hemoglobin
E failure of phototherapy to lower serum bilirubin levels
ANSWER:B
1865 Which common sugar does the clinitest screen not detect?
A glucose
B fructose
C galactose
D sucrose
E lactose
ANSWER:D
1866 The following are manifestations of hypocalcemia in the neonate EXCEPT
A jitteriness
B seizures
C carpopedal spasm
D high-pitched cry
E laryngospasm
ANSWER:C
1867
When screening for intraventricular hemorrhage IVH, the best time to perform an ultrasound
is
A first day of life
B second day of life
C third day of life
D fourth day of life
E fifth day of life
ANSWER:D
1868
The antenatal administration of magnesium sulfate before preterm delivery has been
suggested to
A reduce rates of necrotizing enterocolitis

B decrease the risk of cerebral palsy in surviving infants
C improve overall survival
D decrease respiratory distress syndrome in premature infants
E reduce rates of intraventricular hemorrhage
ANSWER:B
1869
Adrenal production of androgen with development of underarm odor and faint genital hair
(adrenarche) may occur as early as
A 4 yr
B 5 yr
C 6 yr
D 7 yr
E 8 yr
ANSWER:C
1870 In males, the first visible sign of puberty is testicular enlargement, beginning as early as
A 8.5 yr
B 9.5 yr
C 10.5 yr
D 11.5 yr
E 12.5 yr
ANSWER:B
1871 Penile growth occur during SMR
A 1
B 2
C 3
D 4
E 5
ANSWER:C
1872 Peak growth occurs during SMR 4 when testis volumes reach approximately
A 3-4 cm3
B 5-6 cm3
C 7-8 cm3
D 9-10 cm3
E 11-12 cm3
ANSWER:D
1873 Sperm may be found in the urine and nocturnal emissions may be noted at SMR

A 1
B 2
C 3
D 4
E 5
ANSWER:C
1874
The first visible sign of puberty in females is the appearance of breast buds (thelarche)
between
A 5 and 9 yr
B 6and 10 yr
C 7 and 11 yr
D 8 and 12 yr
E 9 and 13 yr
ANSWER:D
1875 Menses typically begins 2.5 yr after the onset of puberty at an average age of
A 9.5 yr
B 10.5 yr
C 11.5 yr
D 12.5 yr
E 13.5 yr
ANSWER:D
1876
The belief that major structural brain development is completed in childhood is outdated. It
is now clear that neuromaturation continues into the
A 2nd decade
B 3rd decade
C 4th decade
D 5th decade
E 6th decade
ANSWER:B
1877
Urethritis is a sexually transmitted infections syndrome characterized by inflammation of the
urethra, usually caused by an infectious etiology. All of the following are true EXCEPT
A urethritis may present with meatal pruritus
B approximately 30-50% of males are asymptomatic
C on examination, the classic finding is erythema of the urethral meatus

D N. gonorrhoeae is one of the most commonly identified pathogen
E noninfectious causes of urethritis include urethral trauma or foreign body
ANSWER:C
1878
In assessing the seriousness of adolescent drug abuse score, which of the following take +2 in
the score?
A female sex
B age >15 years
C positive family history of drug abuse
D use before driving
E use of hallucinogens
ANSWER:D
1879
Which of the following scores regarded as serious according to the assessment of seriousness
of adolescent drug abuse score?
A 0-3
B 08-Mar
C 13-Aug
D 13-18
E 18-23
ANSWER:B
1880
A 13-years-old male presented with delirium and mumbling speech, tachycardia, dry flushed
skin, dilated pupils, myoclonus, slightly elevated temperature, urinary retention, and
decreased bowel sounds, later on he developed seizure and dysrhythmia. Of the following,
the MOST likely cause is
A antidepressant agents
B amphetamine
C barbiturates
D benzodiazepines
E edrophonium
ANSWER:A
1881
Alcohol acts primarily as a central nervous system depressant. It produces all the following
EXCEPT
A euphoria
B impaired short-term memory
C increased pain threshold
D hyperthermia

E respiratory depression
ANSWER:D
1882 Smoking during pregnancy is associated with an average decrease in fetal weight by
A 100 g
B 200 g
C 300 g
D 400 g
E 500 g
ANSWER:B
1883 Exposure to smokeless tobacco increases the users risk for
A lipoid pneumonia
B chronic cough
C cancers of the esophagus
D irritability
E decreased concentration
ANSWER:C
1884
Lysergic acid diethylamide (LSD) is a very potent hallucinogen that is made from lysergic
acid found in ergot, a fungus that grows on rye and other grains. All the following are
common somatic symptoms EXCEPT
A dizziness
B miosis
C nausea
D flushing
E hyperthermia
ANSWER:B
1885
A 15-year-old female presented with delusions, paranoia, tachycardia, hypertension,
hyperpyrexia, diaphoresis, piloerection, mydriasis, and hyperreflexia, later on she developed
seizure, hypotension, and dysrhythmia. Of the following, the MOST likely cause is
A antidepressant agents
B amphetamine
C barbiturates
D benzodiazepines
E edrophonium
ANSWER:B

1886 Which of the following is TRUE regarding pubertal gynecomastia?
A occurring in up to 20% of normal adolescent
B onset typically is between 7 and 9 yr
C usually regresses within 6 mo
D surgery may be indicated in severe or persistent cases
E medical therapies have been approved for use in adolescents
ANSWER:D
1887
Evaluation for pubertal delay in female should be done if she lacks any pubertal signs by the
age of
A 12 yr
B 13 yr
C 14 yr
D 15 yr
E 16 yr
ANSWER:B
1888 The age at which an evaluation for primary amenorrhea should be undertaken is
A 12 yr
B 13 yr
C 14 yr
D 15 yr
E 16 yr
ANSWER:D
1889 Primary amenorrhea generally requires evaluation, if menstruation does not occur within
A 1 yr from the onset of puberty
B 2 yr from the onset of puberty
C 3 yr from the onset of puberty
D 4 yr from the onset of puberty
E 5 yr from the onset of puberty
ANSWER:D
1890 The MOST commonly used method for contraception is
A condom
B withdrawal
C oral contraceptive pill
D spermicides
E intrauterine device

ANSWER:A
1891 The contraceptive method with the highest failure rate even with perfect use is
A progestin releasing intra uterine device
B progestin-only injection
C combined oral contraceptives
D male condom
E spermicides
ANSWER:E
1892
Depo-provera: an injectable progestin, medroxyprogesterone acetate available as a deep
intramuscular injection (150 mg) or as a subcutaneous injection (104 mg) with typicaluse,
the failure rates is 6%. Potential adverse effect include
A heavy menstrual bleeding
B dysmenorrhea
C acne
D weight gain
E osteoporosis
ANSWER:E
1893
A 9-month-old boy presented with recurrent attacks of diarrhea, thrush, and failure to thrive
shortly after birth. You suspect immune deficiency. Of the following, the MOST common
cause of this condition is
A hyper-IgE syndrome
B defect in phagocytic cells
C deficiencies in T-cell function
D defect in antibody production
E defect in complement proteins
ANSWER:C
1894
A 17-month-old girl presented with history of recurrent attacks of pneumonia since the age
of 8 months. You suspect a predominant B-cell defect. Of the following, the BEST simple
initial screening test is
A IgA measurement
B IgG measurement
C IgM measurement
D IgE measurement
E isohemagglutinins titer
ANSWER:A

1895
A very high serum concentrations of one or more immunoglobulin classes, suggest all the
following EXCEPT
A HIV infection
B chronic inflammation
C intestinal lymphangiectasia
D chronic granulomatous disease
E autoimmune lymphoproliferative syndrome
ANSWER:C
1896
A 3-year-old boy presented with recurrent attacks of pneumonia and otitis media since the
1st birthday. Your diagnosis is X-linked agammaglobulinemia. Of the following, the MOST
likely offending organism is
A CMV
B Mycoplasma
C Pneumocystis jiroveci
D Staphylococcus aureus
E Streptococcus pneumonia
ANSWER:E
1897
A 4.5-year-old boy presented with history of recurrent attack of secretary otitis media and
purulent nasal discharge, Haemophilus influenza revealed by culture of ear discharge, on
physical examination there is no tonsillar tissue and no palpable lymph nodes. Of the
following, the MOST appropriate test to confirm the diagnosis is measurement of
A flow cytometry
B IgA concenteration
C isohemagglutinins titer
D IgG and IgM concenteration
E antibodies to antigens of routine immunizations
ANSWER:A
1898
A 3-year-old girl presented with recurrent attacks of chest infection and frequent episodes of
chronic diarrhea, stool examination revealed Giardia lamblia and serum IgA level was very
low. Regarding this condition, all the following are true EXCEPT
A autosomal dominant inheritance
B phenotypically normal blood B cells
C administration of IVIG is not indicated
D environmental factors may trigger the disease
E it is associated with a celiac-like syndrome with dramatic response to gluten- free

ANSWER:E
1899
A 2-year-old boy presented with failure to thrive, neonatal hypocalcemic seizure, and three
attacks of sepsis. Examination reveals cleft palate and holo-systolic murmur. Of the
following, the genetic predisposition of this condition is
A microdeletions of chromosome 22q11.2
B interstitial deletion of the long arm of chromosome 15 (15q11-13)
C presence of two copies of paternally derived chromosome 11p15.5
D presence of mutated gene on long arm of chromosome 11 (11q22-23)
E presence of abnormal gene, on the proximal arm of the X chromosome at Xp11.22-11.23
ANSWER:A
1900 Of the following, the findings that are MOST suggestive of DiGeorge syndrome are
A cataract and a congenital heart defect
B hypotonia and a congenital heart defect
C cutis aplasia and a congenital heart defect
D hypocalcemia and a congenital heart defect
E mongoloid slant to the eyes and a congenital heart defect
ANSWER:D
1901
The MOST appropriate method to correct the immune deficiency in complete DiGeorge
syndrome is
A vaccination
B administration of IVIG
C judicious use of antibiotics
D transplantation of thymic tissue
E transplantation of hematopoietic stem cells
ANSWER:D
1902
In addition to infection and bleeding, the MOST common cause of death in Wiskott- Aldrich
syndrome is
A renal failure
B graft versus host disease
C protracted bloody diarrhea
D EBV-associated malignancies
E development of autoimmune disease
ANSWER:D
1903 The treatment of choice for a 3-year-old boy with Wiskott-Aldrich syndrome is
A splenectomy

B use of killed vaccines
C aggressive antibiotics
D administration of IVIG
E bone marrow or cord blood transplantation
ANSWER:E
1904 All the following are features of ataxia-telangiectasia EXCEPT
A hepatic manifestation
B high incidence of malignancy
C chronic sinopulmonary disease
D oculocutaneous telangiectasias
E variable humoral and cellular immunodeficiency
ANSWER:A
1905 The earliest clinical feature of ataxia-telangiectasia is
A ataxia
B jaundice
C inability to walk
D ocular telangiectasias
E cutaneous telangiectasias
ANSWER:A
1906 Severe eosinophilia, defined as an absolute eosinophil count (AEC) more than
A 750 cells/µL
B 1,000 cells/µL
C 1,500 cells/µL
D 5,000 cells/µL
E 7,500 cells/µL
ANSWER:D
1907
Which of the following statements is TRUE regarding diurnal variation of the absolute
eosinophil count (AEC)?
A higher in the evening
B lower in the afternoon
C higher in the afternoon
D lower in the early morning
E higher in the early morning
ANSWER:E
1908 Eosinophilia can be seen in all the following EXCEPT

A scabies
B urticaria
C corticosteroid therapy
D Hodgkin disease
E hypersensitivity drug reactions
ANSWER:C
1909 The following conditions can cause eosinophilia EXCEPT
A malaria
B filariasis
C giardiasis
D ascariasis
E amebiasis
ANSWER:C
1910 The highest white blood cells count is seen at
A birth
B 12 hour of life
C 1 week of life
D 1 year of life
E adolescence
ANSWER:A
1911 Severe neutropenia is defined as absolute neutrophil count (ANC) less than
A 500/µL
B 750/µL
C 1000/µL
D 1250/µL
E 1500/µL
ANSWER:A
1912 Chronic neutropenia by definition lasts longer than
A 1 mo
B 3 mo
C 6 mo
D 9 mo
E 12 mo
ANSWER:B
1913 Chronic neutropenia may be induced by

A adenovirus
B enteroviruses
C cytomegalovirus
D influenzas A and B
E respiratory syncytial virus
ANSWER:C
1914
A 4-year-old boy presented with high fever, decreased appetite, aphthous stomatitis, and
cellulitis of the right calf muscle. His absolute neutrophil count (ANC) was 600/µL. The
MOST likely organisms causing this condition are
A Staphylococcus aureus and Histoplsmosis
B Staphylococcus aureus and rickettsial pox
C Staphylococcus aureus and Cytomegalovirus
D Staphylococcus aureus and Pneumocystis carinii
E Staphylococcus aureus and gram-negative bacteria
ANSWER:E
1915 Recurrent infections with neutropenia are a distinctive feature of
A Pompe disease
B Hurler disease
C Gaucher disease
D Glycogen storage disease Ia (GSDIa)
E Glycogen storage disease Ib (GSDIb)
ANSWER:E
1916 All the following drugs can induce neutropenia EXCEPT
A penicillin
B tetracycline
C phenobarbital
D phenothiazine
E chloramphenicol
ANSWER:B
1917
A 15-year-old boy with rheumatoid arthritis, on quinine for 6 months, presented with fever,
aphthous stomatitis, and fatigue for the last 5 days. His absolute neutrophil count (ANC) was
700/µL. The MOST next appropriate therapeutic measure to be done when quinine
withdrawal fails to improve neutropenia is
A administration of IVIG
B aggressive use of antibiotics

C transplantation of hematopoietic stem cells
D administration of pulse methylprednisolone
E administration of recombinant human granulocyte colony-stimulating factor
ANSWER:E
1918
A 9-year-old boy with acute lymphoblastic leukemia on chemotherapy, presented with fever,
aphthous stomatitis, and fatigue for the last 5 days. His absolute neutrophil count (ANC) was
450/µL and hemoglobin level was 10gm/dl. The NEXT appropriate therapeutic measure is
A administration of IVIG
B transplantation of hematopoietic stem cells
C administration of pulse methylprednisolone
D aggressive use of broad-spectrum antibiotics
E administration of recombinant human granulocyte colony-stimulating factor
ANSWER:D
1919
A 13-month-old girl presented with failure to thrive, repeated chest infection, and fatty
offensive diarrhea. Hemoglobin level 8.8gm/dl, platelate count 150/mm3, white blood cell
count 3.500/mm3, with absolute neutrophil count (ANC) 500/µL. Of the following, the
MOST likely diagnosis is
A Kostmann disease
B cyclic neutropenia
C drug-induced neutropenia
D acute lymphoblastic leukemia
E Shwachman-Diamond syndrome
ANSWER:E
1920 Leukemoid reactions (WBC count > 50,000/µL) are characterized by abundant
A blasts
B band form
C myelocytes
D promyelocytes
E metamyelocytes
ANSWER:B
1921 Leukemoid reactions (WBC count >50,000/µL) are caused by all the following EXCEPT
A shigellosis
B septicemia
C salmonellosis

D chronic granulomatous disease
E leukocyte adhesion defect with infection
ANSWER:D
1922
Shifting to the left (proportion of immature neutrophil cells >5%) may occasionally be
caused by all the following EXCEPT
A burns
B trauma
C hemorrhage
D acute hemolysis
E megaloblastic anemia
ANSWER:E
1923
A 15-year-old boy presented with high fever, lymphadenopathy, hepatosplenomegaly,
recurrent skin infections, silvery hair, and photophobia. Last year, he developed ataxia. The
MOST characteristic finding in this condition is
A neutropenia
B thrombocytopenia
C prolonged bleeding time
D prolonged prothrombine time
E prolonged partial prothrombine time
ANSWER:C
1924
In patient with Chédiak-Higashi syndrome, the only curative therapy is hematopoietic stem
cell transplantation which correct all the following EXCEPT
A neuropathy
B immunologic function
C hematopoietic function
D natural killer cell deficiency
E conversion to the accelerated phase
ANSWER:A
1925
A 3-year-old boy presented with recurrent subcutaneous abscesses, lymphadenitis,
osteomyelitis, and family history of recurrent infections. The hallmark of this condition is
A hypocalcemia
B atopic dermatitis
C thrombocytopenia
D granuloma formation
E prolonged bleeding time

ANSWER:D
1926 The MOST common pathogen in chronic granulomatous disease (CGD) is
A S. aureus
B Aspergillus
C Salmonella
D Mycobacterium
E Candida albicans
ANSWER:A
1927 The diagnosis of chronic granulomatous disease CGD is MOST often made by
A DNA analysis
B flow cytometry
C neutrophil G6PD assay
D erythrocyte G6PD assay
E nitroblue tetrazolium dye test
ANSWER:B
1928
To reduce the number of bacterial infection in patients with chronic granulomatous disease
(CGD), they should be given daily
A penicillin
B cloxacillin
C interferon-γ
D itraconazole
E trimethoprim-sulfamethoxazole
ANSWER:E
1929
In chronic granulomatous disease (CGD), the best method to detect and follow up deep-
seated infection is
A ESR
B blood culture
C WBC count
D C-reactive protein
E immature to mature neutrophil ratio
ANSWER:A
1930 All the following are characteristic in properdin deficiency EXCEPT
A dermal vasculitis
B male predominance
C systemic lupus erythematosus

D susceptibility to N. meningitidis meningitis
E normal serum hemolytic complement activity
ANSWER:C
1931
All the following matching about genetic deficiencies of plasma complement components
and associated clinical findings are true EXCEPT
A C4 --------------------- discoid lupus erythematosus
B C1q------------------ systemic lupus erythematosus
C C1 INH ---------------------- hereditary angioedema
D factor H---- atypical hemolytic-uremic syndrome
E properdin --------------- N. meningitidis meningitis
ANSWER:A
1932 The complement component of both the classical and alternative pathways is
A C2
B C3
C C4
D C5
E C6
ANSWER:B
1933 All the following conditions can induce lymphopenia EXCEPT
A sepsis
B typhoid
C brucellosis
D corticosteroid use
E cartilage-hair hypoplasia
ANSWER:C
1934
The recognition of allergic rhinitis as a major chronic respiratory disease of children rests
largely on all the following features EXCEPT
A high prevalence
B effect on quality of life
C school performance
D other comorbidities
E presentation in infancy
ANSWER:E
1935 The diagnosis of allergic rhinitis is established by the time the child reaches age of
A 2 yr

B 4 yr
C 6 yr
D 8 yr
E 10 yr
ANSWER:C
1936 A significant risk factor in the development of allergic rhinitis in children is
A positive family history
B serum IgA higher than 100 iu/ml
C alcoholic mother
D diabetic mother
E father with renal disease
ANSWER:A
1937
The critical period for the development of allergic rhinitis exists when the genetically
susceptible individual at greatest risk of sensitization in
A early adolescence
B late adolescence
C school age
D toddlerhood
E early infancy
ANSWER:E
1938
The occurrence of 3 or more episodes of rhinorrhea in the first year of life is associated with
allergic rhinitis (AR) at age of
A 11 yr
B 9 yr
C 7 yr
D 5 yr
E 3 yr
ANSWER:C
1939
Early introduction of all the following decrease the risk of asthma and allergic rhinitis
EXCEPT
A rye
B fish
C egg
D meat
E wheat

ANSWER:D
1940
The exposure to which of the following early in childhood protects against the development
of atopy
A smoking
B alcohol
C rodents
D dogs
E indoor allergen
ANSWER:D
1941 The main differentiating item between intermittent versus persistent allergic rhinitis is
A school performance
B troublesome symptoms
C daily activities
D number of attacks per week
E sleep pattern
ANSWER:D
1942 Allergic shiners represent
A an upward rubbing of the nose with an open palm
B continuous open- mouth breathing
C dark circles under the eyes
D transverse crease of the nose
E conjunctival edema and itching
ANSWER:C
1943
Allergic rhinitis may be complicated by secondary bacterial infection as sinusitis. Of the
following, the MOST likely clue to infection is
A hyperemia
B swollen turbinates
C thick nasal secretions
D bluish mucus membranes
E edematous mucus membranes
ANSWER:C
1944 Causes of hormonal rhinitis include all the following EXCEPT
A hypothyroidism
B exercise
C cancer

D vaculitides
E systemic lupus erythematosis
ANSWER:E
1945 The reported rate of remission of allergic rhinitis among children is about
A 1-10%
B 11-20%
C 21-30%
D 31-40%
E 41-50%
ANSWER:B
1946 What percentage of allergic rhinitis patients has allergic conjunctivitis?
A >30%
B >40%
C >50%
D >60%
E >70%
ANSWER:D
1947
The percentage of allergic rhinitis in asthmatic patients is 78%, while the percentage of
asthma in patients with allergic rhinitis is
A 18%
B 28%
C 38%
D 48%
E 58%
ANSWER:C
1948
To avoid false-negative skin test results in atopic host, most sedating antihistamine should be
withheld for 3-4 days, and nonsedating antihistamines for 5-7 days, while montelukast
should be withheld for
A 1 day
B 3 day
C 5 day
D 7 day
E 9 day
ANSWER:A
1949 The best method for detection of allergen-specific IgE is

A skin tests
B serum immune assay for specific IgE
C nasal smear study
D eosinophilia in blood film
E total serum IgE concentration
ANSWER:A
1950
Second generation antihistamines are preferable to first generation antihistamines because
they have
A longer action
B less sedation
C more potency
D more palatable effect
E more shelf life
ANSWER:B
1951 The MOST effective therapy for persistent allergic rhinitis is
A oral desloratadine
B oral montelukast
C subcutaneous omalizumab
D nasal budesonide spray
E nasal ipratropium bromide spray
ANSWER:D
1952 Approximately 80% of all asthmatic patients report disease onset prior to the age of
A 2 yr
B 4 yr
C 6 yr
D 8 yr
E 10 yr
ANSWER:C
1953 Of the following, the major risk factor for persistent asthma is
A eczema
B allergic rhinitis
C wheezing apart from colds
D food allergen sensitization
E ≥4% peripheral blood eosinophils
ANSWER:A

1954 Of the following, the strongest identifiable factor for the persistence of childhood asthma is
A allergy
B male gender
C low birth weight
D parental asthma
E lower respiratory tract infection
ANSWER:A
1955 The following actions may help reduce the likelihood of asthma development EXCEPT
A healthy diet
B immunization
C active lifestyle
D avoidance of tobacco smoking
E prolonged breastfeeding > 4 months
ANSWER:B
1956
Recurrent coughing and wheezing occurs in 35% of preschool-age children. Those who
continue to have persistent asthma into later childhood are approximately
A one-fifth
B one-fourth
C one-third
D one-half
E two-thirds
ANSWER:C
1957 Indications of severe exacerbation of asthma include the following EXCEPT
A breathlessness
B accessory muscles use
C labored breath
D PEF or FEV1 value <70% of personal best values
E mental status changes
ANSWER:D
1958
Spirometry is a helpful objective measure of airflow limitation; it depends on patient’s ability
to properly perform a full, forceful, and prolonged expiratory maneuver. Spirometry usually
feasible in children
A ≥4 yr of age
B ≥6 yr of age

C ≥8 yr of age
D ≥10 yr of age
E ≥12 yr of age
ANSWER:B
1959 The MOST vital initial treatment in the management of severe asthma exacerbations is
A supplemental oxygen
B inhaled β -agonist
C intramuscular injection of epinephrine
D inhaled ipratropium
E intramuscular injection of β-agonist
ANSWER:A
1960
A patient with exacerbation of asthma being handled and managed in the ER may be
discharged home if all the following parameters are met EXCEPT
A normal physical activity
B PEF >70% of predicted or personal best
C exercise tolerance
D oxygen saturation >92% while the patient is breathing room air for 4 hr
E no more use of accessory muscles
ANSWER:C
1961 All the following are considered as risk factors for asthma morbidity and mortality EXCEPT
A poverty
B air pollution exposure
C female gender
D poor response to systemic steroid therapy
E sudden asphyxia episodes
ANSWER:C
1962 The best ‘‘rescue’’ medication in the treatment of acute asthma symptoms is
A oral SABA
B inhaled SABA
C oral corticosteroid
D inhaled ipratropium
E inhaled corticosteroid
ANSWER:B
1963 The best treatment option for step -6- severe persistent asthma in a 6-year-old boy is

A medium dose inhaled corticosteroids with long acting B-agonists
B high dose inhaled corticosteroids with leukotriene receptor antagonist
C low dose inhaled corticosteroids with leukotriene receptor antagonist
D high dose inhaled corticosteroids with long acting B-agonist and oral corticosteroids
E
high dose inhaled corticosteroids with long acting B-agonist and oral corticosteroidsalong
with omalizumab therapy
ANSWER:D
1964 A common finding of chest radiograph in a child with asthma is
A peribronchial thickening
B atelectasis
C pneumothorax
D bronchiectasis
E pneumomediastinum
ANSWER:A
1965
Bronchodilator response to an inhaled β-agonist (e.g., albuterol) is greater in asthmatic
patients than nonasthmatic persons, the rate of improvement in FEV1 consistent with asthma
is
A ≥10%
B ≥12%
C ≥14%
D ≥16%
E ≥18%
ANSWER:B
1966
The diurnal variation in Peak Expiratory Flow (PEF) that is consistent with asthma is more
than
A 10%
B 20%
C 30%
D 40%
E 50%
ANSWER:B
1967
In emergency department, the patient may be discharged to home if there is symptomatic
improvement, normal physical findings, PEF >70% of predicted or personal best, and
oxygen saturation >92% in room air for 4hr. Of the following, the MOST likely discharge
medication used is

A inhaled β-agonist only
B oral corticosteroid only
C inhaled corticosteroid only
D inhaled β-agonist plus oral corticosteroid
E oral β-agonist plus inhaled corticosteroid
ANSWER:D
1968
All the following are a recognized features of a ‘’well-controlled’’ asthma in a 7- year-old
boy EXCEPT
A FEV1:FVC ratio >80%
B FEV1of >80% of predicted
C daytime symptoms ≤2 days/wk
D ≥4 exacerbations in the past year
E need a rescue bronchodilator ≤2 days/wk
ANSWER:D
1969
Use of a stoplight zone system (green, yellow, red) tailored to each child’s “personal best”
PEF values can optimize effectiveness and interest. Of the following, the PEF value in yellow
zone is
A <10%
B 10-30%
C 30-50%
D 50-80%
E 80-100%
ANSWER:D
1970
All levels of persistent asthma should be treated with daily medications to improve long-term
control. All the following are long-term controller medications EXCEPT
A albuterol
B omalizumab
C montaleukast
D inhaled corticosteroid
E sustained-release theophylline
ANSWER:A
1971
Adverse effects of frequently administered β-agonist therapy in asthma include all the
following EXCEPT
A tremor
B irritability

C tachycardia
D hypokalemia
E mouth dryness
ANSWER:E
1972 Atopic dermatitis affects many children worldwide, the exact percentage approximating
A 5-10%
B 10-30%
C 30-50%
D 50-70%
E 70-90%
ANSWER:B
1973 The hallmark of atopic dermatitis is
A intense pruritus
B cutaneous reactivity
C lichenification
D fibrotic papules
E dry skin
ANSWER:E
1974 Atopic dermatitis typically begins in
A infancy
B toddlerhood
C preschooler age
D school age
E adolescence
ANSWER:A
1975 The cardinal feature of atopic dermatitis is
A skin rash
B lichenification
C fibrotic papules
D intense pruritus
E dry skin
ANSWER:D
1976 All the following are triggers for pruritus in atopic dermatitis EXCEPT
A grass
B tree nuts

C high humidity
D herpes simplex
E excessive sweating
ANSWER:C
1977 In infancy, atopic dermatitis is usually acute and spares the
A extensor surfaces of the extremities
B forehead
C scalp
D diaper area
E cheeks
ANSWER:D
1978
Older children with chronic atopic dermatitis have lichenification and a tendency to be
localized to
A flexural folds of the extremities
B forehead
C scalp
D diaper area
E cheeks
ANSWER:A
1979 The following are the major clinical features of atopic dermatitis (AD) EXCEPT
A pruritus
B family history
C relapsing dermatitis
D facial eczema in infants
E extensor eczema in adolescents
ANSWER:E
1980
A high index of suspicion of which of the following conditions is to be undertaken in a
patient with atopic dermatitis and failure to thrive
A Wiscott-Aldrich syndrome
B severe combined immune deficiency
C Histiocytosis
D hyper IgE syndrome
E chronic granulomatous disease
ANSWER:C

1981
One of the following metabolic conditions is not deemed in the differential diagnosis of
atopic dermatitis
A zinc deficiency
B pyridoxine deficiency
C cobalamin deficiency
D niacin deficiency
E phenylketonuria
ANSWER:C
1982 The first line therapy of atopic dermatitis (AD) is
A moisturizers
B cyclosporine
C antihistamine
D tar preparations
E topical corticosteroids
ANSWER:A
1983 Prevention of atopic dermatitis in infancy includes the following measures EXCEPT
A breast feeding
B feeding with a hypoallergenic hydrolyzed formula
C use of probiotics
D elimination of implicated food allergen from the mother diet
E use of special type napkins
ANSWER:E
1984 Which of the following vitamin deficiencies often accompanies severe atopic dermatitis?
A E
B C
C A
D D
E B12
ANSWER:D
1985
Predictive factors of a poor prognosis for atopic dermatitis includes all the following
EXCEPT
A widespread atopic dermatitis in childhood
B filaggrin gene null mutations
C concomitant allergic rhinitis and asthma
D family history of atopic dermatitis in parents or siblings

E late age at onset of atopic dermatitis
ANSWER:E
1986
The best choice of antibiotic in treating localized impetiginous lesions in patients with atopic
dermatitis is
A oral erythromycin
B oral cephalexin
C topical mupirocin
D topical ampicillin
E topical gentamycin
ANSWER:C
1987
IgE anti-bodies against Malassezia furfur have been found in patients with atopic dermatitis
of
A head and neck
B hands and arms
C legs and feet
D upper chest
E lower abdomen
ANSWER:A
1988
One of the following statements is TRUE regarding skin tests in identifying food allergies in
patients with atopic dermatitis
A
negative skin and blood test results for allergen-specific ige have a low predictive value for
excluding suspected allergens
B
positive results of skin or blood tests using foods often correlate with clinical symptoms and
no need to be confirmed with controlled food challenges
C extensive elimination diets are commonly required
D even with multiple majority of patients react to more than 3 foods
E
potential allergens can be identified by a careful history and performing selective skin prick
tests or in vitro blood testing for allergen- specific IgE
ANSWER:E
1989
One of the following medications used in treatment of atopic dermatitis should be
discontinued after failure to achieve good results within 4-6 weeks
A methotrexate
B azathioprine
C cyclosporine
D mycophenolate mofetil

E omalizumab
ANSWER:D
1990 Systemic corticosteroids are rarely indicated in the treatment of atopic dermatitis because
A toxic side effects after long term use
B rebound flare after therapy discontinuation
C cannot do more than what topical can
D tapering is required even after short term use
E antimetabolites can do better in modifying disease course
ANSWER:B
1991 The least potent topical steroid used in treatment of atopic dermatitis is
A fluticasone
B desonide
C betamethasone dipropionate
D hydrocortisone
E clobetasol propionate
ANSWER:D
1992 Which topical steroid is preferable to use for treatment of atopic dermatitis in the face?
A fluticasone 0.005%
B desonide 05%
C betamethasone dipropionate 0.05%
D hydrocortisone 1%
E clobetasol propionate 0.05%
ANSWER:D
1993
In atopic dermatitis, the presence of punched out erosions, vesicles, and infected skin lesions
that fail to respond to oral antibiotics suggests infection with
A Herpes zoster
B Herpes simplex
C cutaneous warts
D Trichophyton rubrum
E Molluscum contagiosum
ANSWER:B
1994
Exfoliative dermatitis may develop in patients with atopic dermatitis and extensive skin
involvement, usually caused by inappropriate therapy or superinfection with
A E. coli
B Herpes zoster

C Herpes simplex
D Streptococcal pyogen
E Trichophyton rubrum
ANSWER:C
1995 eactions to stinging and biting insects may cause
A a limited lesion confined to the primary site
B a pronounced localized reaction
C a pronounced systemic reaction due to immediate hypersensitivity
D a pronounced systemic reaction due to delayed hypersensitivity
E no reaction
ANSWER:A
1996
Systemic allergic responses to insects are attributed to IgE antibody response caused
primarily by
A ticks
B spiders
C hymenoptera
D scorpions
E triatoma (kissing bug)
ANSWER:C
1997 The MOST notorious stinging insect is
A honeybee
B harvester ants
C hornet
D bumblebee
E yellow jackets
ANSWER:E
1998 Delayed/Late reaction to stinging venom may precipitate all the following EXCEPT
A vasculitis
B serum sickness
C encephalopathy
D hemolytic anemia
E nephrotic syndrome
ANSWER:D
1999 The MOST reliable diagnostic modality for detection of venom-specific IgE is
A in vitro serum assay for venom-specific IgE

B skin test
C serum tryptase level
D plasma histamine
E radioallergosorbent assay
ANSWER:B
2000
In the presence of convincing history of a severe systemic reaction, the next diagnostic step
for those with initially negative skin test is
A in vitro serum assay for venom-specific IgE
B repeat skin test after 4-6 weeks
C serum tryptase level
D plasma histamine
E basophil histamine release test
ANSWER:A
2001 Sting sites rarely become infected possibly owing to
A cleansing the area immediately after insect
B the over use of antibacterial creams for the area
C venom constituents have antibacterial action
D vasospasm after stinging impedes bacterial invasion
E venom immunotherapy has antibacterial properties
ANSWER:C
2002
Anaphylactic reactions after a Hymenoptera sting are treated exactly like anaphylaxis from
any cause. Of the following, the drug of choice is
A oxygen
B epinephrine
C antihistamines
D corticosteroids
E intravenous fluids
ANSWER:B
2003 A TRUE indication of venom immunotherapy in a six-year-old boy is
A large local reaction with positive skin test and positive in vitro test
B generalized cutaneous reaction with positive skin test and negative in vitro test
C generalized cutaneous reaction with positive in vitro test and negative skin test
D systemic reaction with positive skin test and negative in vitro test
E systemic reaction with negative skin test and negative in vitro test
ANSWER:D

2004
Skeeter syndrome is a large local reaction to stinging and biting insects. Of the following, the
TRUE statement is
A Ig E mediated response
B usually occurs in older children
C usually followed by anaphylaxis
D mosquito is the responsible agent
E misdiagnosed as erythema nodosum
ANSWER:D
2005
The eye is a common target of allergic disorders because of its marked vascularity and direct
contact with allergens in the environment. Of the following, the MOST immunologically
active tissue of the external eye is
A iris
B sclera
C eyelids
D eyelashes
E conjunctiva
ANSWER:E
2006 The MOST common hypersensitivity response of the eye is
A allergic conjunctivitis
B vernal keratoconjunctivitis
C atopic keratoconjunctivitis
D giant papillary conjunctivitis
E contact allergy
ANSWER:E
2007 The MOST sight-threatening type of ocular allergy is
A contact allergy
B vernal keratoconjunctivitis
C giant papillary conjunctivitis
D seasonal allergic conjunctivitis
E perennial allergic conjunctivitis
ANSWER:B
2008 The MOST common presenting complain of allergic conjunctivitis is
A pain
B itching
C redness

D blurred vision
E purulent discharge
ANSWER:B
2009 Topical ophthalmic medications usually cause
A contact allergy
B allergic conjunctivitis
C vernal keratoconjunctivitis
D atopic keratoconjunctivitis
E giant papillary conjunctivitis
ANSWER:A
2010 Contact lenses are associated with
A contact allergy
B allergic conjunctivitis
C vernal keratoconjunctivitis
D atopic keratoconjunctivitis
E giant papillary conjunctivitis
ANSWER:E
2011 Conjunctivitis medicamentosa is a consequence of chronic use of
A antihistamines
B steroids
C decongestants
D anti-inflammatory
E mast cell stabilizers
ANSWER:C
2012 Non IgE mediated urticaria can be caused by
A milk
B hymenoptera
C Epstein-barr virus
D latex
E blood
ANSWER:C
2013 Chronic urticaria may be caused by
A latex
B peanut
C IV immunoglobulin

D streptococcal pharyngitis
E systemic lupus erythematosus
ANSWER:E
2014 The drug of choice for cold-induced urticaria is
A loratadine
B epinephrine IM
C cyproheptadine
D diphenhydramine
E oral corticosteroid
ANSWER:C
2015
Acute urticaria is a self-limited illness requiring little treatment. All the following can be used
EXCEPT
A loratadine
B hydroxyzine
C epinephrine IM
D H2 antihistamines
E oral corticosteroid
ANSWER:D
2016 Skin biopsy for diagnosis of possible urticarial vasculitis is recommended for
A urticarial lesions that persist at different locations for >24 hr
B those with non- pigmented or non-purpuric components
C those that burn more than itch
D those with associated collagen vascular diseases
E those appeared after peanuts ingestion
ANSWER:D
2017 The differential diagnosis of chronic urticaria includes the following EXCEPT
A cutaneous mastocytosis
B systemic mastocytosis
C complement- mediated mast cell degranulation in malignancies
D cutaneous blistering disorders
E dermatographism
ANSWER:E
2018 The MOST common childhood symptoms of hereditary angioedema is
A cutaneous non-pitting but pruritic edema associated with urticaria
B cutaneous pitting but non-pruritic edema not associated with urticaria

C cutaneous pitting and pruritic edema associated with urticaria
D cutaneous non-pitting and non-pruritic edema associated with urticaria
E cutaneous non-pitting and non-pruritic edema not associated with urticaria
ANSWER:E
2019 All the following factors can amplify anaphylaxis EXCEPT
A exercise
B vaccination
C upper respiratory tract infection
D fever
E emotional stress
ANSWER:B
2020 One of the principal pathologic features in fatal anaphylaxis is
A acute bronchial obstruction
B hypotension
C behavioral change
D abdominal pain
E vomiting
ANSWER:A
2021
Sudden collapse in the presence of cutaneous symptoms in a previously healthy child should
raise suspicion of
A vasovagal collapse
B anaphylaxis
C aspiration
D pulmonary embolism
E seizure disorder
ANSWER:B
2022 Cutaneous symptoms may be absent in anaphylaxis in
A 5%
B 10%
C 20%
D 30%
E 40%
ANSWER:C
2023
The best diagnostic test for anaphylaxis in a child exposed to an allergen 3 hours before
arrival to hospital is

A plasma histamine
B skin test
C radioallergosorbent assay
D plasma tryptase
E immunoCAP IgE test
ANSWER:D
2024
After institution of treatment of anaphylaxis, the patient should be monitored in the
emergency room for at least
A 1 hour
B 4 hours
C 6 hours
D 12 hours
E 24 hours
ANSWER:B
2025
In cases of food-associated exercise-induced anaphylaxis, the parents are advised that
children must not take exercise after ingestion of the triggering food for
A 2-3 hours
B 4-6 hours
C 8-12 hours
D 13-18 hours
E 19-24 hours
ANSWER:A
2026 All the following medications are encountered to cause serum sickness EXCEPT
A ciprofloxacin
B meropenem
C rituximab
D trimethoprim sulfate
E carbamazepine
ANSWER:A
2027
Once the offending agent causing serum sickness is discontinued, the symptoms resolve
spontaneously within
A 1-4 days
B 1-4 weeks
C 5-8 weeks
D 9-12 weeks

E 13-18 weeks
ANSWER:B
2028 Which of the following results of investigations is unlikely in serum sickness?
A elevated erythrocyte sedimentation rate
B thrombocytosis
C reduced C3 activity
D negative microbial cultures
E microscopic hematuria
ANSWER:B
2029 Skin biopsy is not usually necessary for confirming the diagnosis of serum sickness because
A the complement C3 and C4 yield better results
B the clinical examination is enough for the diagnosis
C skin test is superior to biopsy
D the histopathological findings are not specific
E it may be complicated by local infection
ANSWER:D
2030 All the following are complications of serum sickness EXCEPT
A carditis
B glomerulonephritis
C uillain-barre syndrome
D peripheral neuritis
E colitis
ANSWER:E
2031 All the following are symptoms of acute IgE mediated food allergy EXCEPT
A urticaria
B pruritis
C abdominal pain
D diarrhea
E wheezing
ANSWER:B
2032 One of the following presentations is a non-IgE mediated food allergy
A Heiner syndrome
B gastrointestinal anaphylaxis
C rhino conjunctivitis

D oral allergy syndrome
E angioedema
ANSWER:A
2033 The onset of presentation of food allergy is late in
A hen’s egg white
B peanuts
C shellfish
D soybean
E cow’s milk
ANSWER:C
2034 Food protein–induced enterocolitis syndrome is characterized by all the following EXCEPT
A manifests in the first months of life
B vomiting occurs 1-3 hours after feeding
C hypotension occurs in approximately 15% of cases
D commonly improved with ingestion of soy protein based formula
E usually resolve by the age of three years of life
ANSWER:D
2035 The MOST common cause of food protein induced enteropathy is
A peanuts
B fish
C cow’s milk
D eggs
E tree nuts
ANSWER:C
2036 The MOST severe form of food protein-induced enteropathy is
A celiac disease
B eosinophilic esophagitis
C oral allergy syndrome
D acute gastrointestinal allergy
E food protein proctocolitis
ANSWER:A
2037 Wheezing occurs in approximately which percent of IgE-mediated food allergic reactions?
A 15%
B 25%

C 35%
D 45%
E 55%
ANSWER:B
2038 Administration of the following vaccine is contraindicated in children with egg allergy
A MMR
B Influenza
C DPT
D yellow fever
E rabies
ANSWER:D
2039 The only way to establish the diagnosis of cell-mediated food reactions is
A breath hydrogen test
B endoscopy
C elimination and challenge test
D IgE testing
E skin test
ANSWER:C
2040 Unpredictable drug reactions include
A dose dependent
B drug toxicity
C drug interactions
D adverse effects
E allergic reaction
ANSWER:E
2041 Which of the following drugs can cause Stevens - Johnson syndrome?
A dapsone
B hydralazine
C sulfonamide
D procainamide
E bleomycin
ANSWER:C
2042 Which one of the following agents is implicated in causing cutaneous lupus?
A corticosteroids
B calcium-channel blockers

C allopurinol
D B-lactam antibiotics
E sulfonamide
ANSWER:B
2043 Fixed drug eruption can be caused by
A allopurinol
B cephalosporin
C nonsteroidal anti-inflammatory drug
D nitrofurantoin
E infliximab
ANSWER:C
2044
One of the following drug-induced allergic reactions is not included as an indictment of
allopurinol
A exanthem
B Stevens-Johnson syndrome
C toxic epidermal necrolysis
D pulmonary fibrosis
E interstitial nephritis
ANSWER:D
2045
Drug Rash with Eosinophilia and Systemic Symptoms (DRESS) can be caused by all the
following EXCEPT
A anticonvulsants
B sulfonamides
C B-lactam antibiotics
D minocycline
E allopurinol
ANSWER:C
2046 Systemic lupus erythematosus can be caused by
A isoniazid
B sulfonamide
C hydrochlorothiazide
D allopurinol
E nonsteroidal anti-inflammatory drug
ANSWER:A
2047 Vasculitis can be cause by

A isoniazid
B sulfonamide
C penicillamine
D calcium channel blockers
E tetracycline
ANSWER:C
2048 All the following are risk factors for adverse drug reactions EXCEPT
A prior exposure
B route of administration
C dosing schedule
D genetic predisposition
E atopy
ANSWER:E
2049 Of the following, the agent that does not share an identical side chain with penicillin is
A cephalexin
B cefotaxime
C cefixime
D ceftazidime
E ceftriaxone
ANSWER:C
2050
Nonsteroidal anti-inflammatory drugs (NSAIDs) are prescribed to decrease acute and chronic
inflammation associated with various rheumatic diseases; however, many adverse effects
have been encountered with long term use. Of the following, the NSAID that has the highest
toxicity is
A celecoxib
B meloxicam
C indomethacin
D naproxen
E ibuprofen
ANSWER:C
2051
A 7-year-old boy developed small hypopigmented depressed scars after fingernail scratches;
he has been treated with nonsteroidal antiinflammatory drugs (NSAIDs) for a rheumatic
disease 6 weeks ago. Of the following, the NSAID that is MOST likely to cause such a
unique skin reaction is
A celecoxib

B meloxicam
C indomethacin
D naproxen
E ibuprofen
ANSWER:D
2052
Hydroxychloroquine sulfate is an antimalarial drug important in the treatment of SLE and
dermatomyositis, particularly cutaneous manifestations of disease and to reduce lupus flares.
Of the following, the MOST important procedure that should be done routinely during the
course of administration is
A gastric endoscopy
B bone marrow examination
C muscle biopsy
D glucose-6-phophate dehydrogenase enzyme level assay
E ophthalmological examination
ANSWER:E
2053
Sulfasalazine is an effective drug in many rheumatic diseases; however, it is associated with
severe systemic hypersensitivity reaction. It is approved in all the following rheumatic
disease EXCEPT
A polyarticular juvenile idiopathic arthritis (JIA)
B oligoarticular JIA
C systemic JIA
D peripheral arthritis associated with juvenile ankylosing spondylitis
E enthesitis associated with juvenile ankylosing spondylitis
ANSWER:C
2054
You are meeting parents of a 14-year-old girl who has been treated with a monthly
intravenous cyclophosphamide for SLE-associated renal failure for the last 6 months; the
mother is asking about the long-term complications of this drug. All the following are long-
term complications EXCEPT
A bone marrow suppression
B bladder cancer
C leukemia
D lymphoma
E infertility
ANSWER:A

2055
Juvenile idiopathic arthritis has many subtypes. Arthritis must be present to make a diagnosis
of any subtype; involved joints oftenly have the following signs EXCEPT
A swelling
B warm
C erythema
D limitation of movement
E pain on movement
ANSWER:C
2056
Oligoarthritis, the most common subtype of juvenile idiopathic arthritis, is defined as
involving ≤4 joints within the 1st 6 mo of disease onset. Of the following, the MOST
commonly affected joint is
A hip
B knee
C elbow
D wrist
E metatarsal
ANSWER:B
2057
Oligoarthritis predominantly affects the joints of the lower extremities rather than upper
extremity joints. Which of the following joints is never a presenting sign of oligoarthritis?
A hip
B knee
C ankle
D metatarsal
E interphalangeal
ANSWER:A
2058
A 4-year-old girl recently diagnosed with persistent oligoarticular juvenile idiopathic
rheumatoid arthritis (JIA); she has 3 involved joints including the right knee, right ankle, and
left elbow; antinuclear antigen (ANA) is significantly positive. Of the following, the MOST
important step in the management of this girl is
A regular examination of locomotor system
B periodic slit-lamp examination
C periodic ANA monitoring
D periodic erythrocyte sedimentation rate (ESR) monitoring
E frequent C-reactive protein (CRP) monitoring
ANSWER:B

2059
Antinuclear antigen (ANA) measurement test is useful in some rheumatologic diseases
especially with persistent oligoarticular juvenile idiopathic rheumatoid arthritis (JIA). All the
following are more likely to be correlated with ANA positivity EXCEPT
A anterior uveitis
B younger age at disease onset
C female sex
D symmetrical arthritis
E lower number of involved joints over time
ANSWER:D
2060
Rheumatoid factor (RF)–positive polyarthritis is characterized by aggressive symmetric
inflammation of joints of both upper and lower extremities. Of the following, the extra
articular manifestation that is almost exclusively occur in RF- positive individuals is
A fever
B evanescent rash
C uveitis
D extensor surfaces nodules
E pericarditis
ANSWER:D
2061
A 5-year-old boy develops acute onset of high spiking fevers, lymphadenopathy,
hepatosplenomegaly, and purpura; he has been diagnosed with systemic Juvenile idiopathic
arthritis (JIA) since early childhood treated with antiinflammatory drugs. You suspect
macrophage activating syndrome (MAS). Of the following, the BEST test that distinguishes
MAS from a flare of the primary disease is
A leucopenia
B falling ESR
C hypofibrinogenemia
D hypertriglyceridemia
E evidence of hemophagocytosis in the bone marrow
ANSWER:E
2062
Rheumatoid factor (RF)–positive polyarthritis usually accounts for < 10% of all juvenile
idiopathic arthritis (JIA) cases. The articular manifestation pattern is characterized by
involvement of ≥5 joints in both upper and lower extremities.Of the following, the LEAST
effective drug to induce remission for this subtype of JIA is
A NSAIDs
B methotrexate

C TNF-α antagonists
D IL-1 inhibitors
E IL-6 inhibitors
ANSWER:A
2063
A 5-year-old boy has systemic juvenile idiopathic arthritis (sJIA) with systemic
manifestations including fever, hepatosplenomegaly, lymphadenopathy, and pericarditis. Of
the following, the BEST initial treatment for this boy is
A NSAIDs
B systemic glucocorticoids
C TNF-α inhibitors
D IL-1 antagonists
E IL-6 antagonists
ANSWER:B
2064
You are discussing with the medical students the role of systemic steroids for the
management of rheumatologic diseases. Your discussion should include all the following
statements EXCEPT
A they are recommended for management of severe systemic illness
B
they are used for bridge therapy during the wait for therapeutic response to a disease
modified anti rheumatic drugs (DMARD)
C they are effective for control of uveitis
D they prevent joint destruction
E they impose risks of severe toxicities
ANSWER:D
2065
Chronic uveitis is one of the extraarticular manifestations of various subtypes of juvenile
idiopathic arthritis. All the following are risk factors for the development of uveitis EXCEPT
A oligoarthritis subtype
B female gender
C ANA-positivity
D onset of arthritis earlier than 6 yr of age
E severity of arthritis
ANSWER:E
2066
The child with polyarticular JIA often has a more prolonged course of active joint
inflammation and requires early and aggressive therapy. Of the following, the predictor that
carries the WORST prognosis is

A old age at onset
B rheumatoid factor (RF) seronegativity
C absence of rheumatoid nodules
D small numbers of affected joints
E hip joint involvement
ANSWER:E
2067
Spondyloarthritis may be overlapped clinically with other forms of juvenile idiopathic
arthritis (JIA). Clinical manifestations that help to distinguish spondyloarthritis from other
forms of juvenile arthritis include all the following EXCEPT
A arthritis of the sacroiliac joints
B arthritis of the hips
C enthesitis
D asymptomatic uveitis
E gastrointestinal inflammation
ANSWER:D
2068
Enthesitis-Related Arthritis (ERA) is a subtype of rheumatic disease that falls in the juvenile
idiopathic arthritis (JIA) category. Of the following, the clinical feature that is highly
suggestive of ERA is
A symmetrical arthritis early in the disease course
B early involvement of more than 5 joints
C involvement of upper limb joints
D Inflammation of the small joints of the foot
E asymmetrical enthesitis
ANSWER:D
2069
Spondyloarthritides are complex diseases in which susceptibility is largely genetically
determined. Which of the following characteristics is never associated with such diseases?
A enthesitis
B peripheral arthritis
C axial arthritis
D HLA-B27 positivity
E rheumatoid factor positivity
ANSWER:E

2070
You are discussing with medical students Juvenile Ankylosing Spondylitis (JAS) subtype of
spondyloarthritides. One of the students asks you about the features that distinguish this
disease from the adult-onset ankylosing spondylitis (AOAS). All the following are true
regarding JAS in comparison with AOAS EXCEPT
A it is present in patients <16 yr old
B axial disease occurs more frequently early in the disease course
C inflammatory back pain is less frequent at disease onset
D enthesitis occurs more commonly
E peripheral arthritis is more common
ANSWER:B
2071
Reactive arthritis is defined as joint inflammation caused by a sterile inflammatory reaction
following a recent infection. Of the following, the LEAST likely micro-organism that may
cause reactive arthritis is
A Clostridium difficile
B Salmonella paratyphi
C Shigella flexneri
D Yersinia enterocolitica
E Campylobacter jejuni
ANSWER:A
2072
The distinction between postinfectious arthritis and reactive arthritis is not always clear. All
the following are common features of postinfectious arthritis, rather than reactive arthritis
EXCEPT
A there is a transient joint swelling or pain
B it lasts less than 6 wk
C chronic spondyloarthritis is a common sequelae
D it may be encountered after group A streptococcus infection
E rubella infection is a common causative agent
ANSWER:C
2073
You are meeting parents of 2-year-old girl who is going to receive rubella vaccine; the
mother is concerned regarding arthritis that may develop after immunization. Of the
following, the statement that should be included during the discussion is
A it is usually frequently encountered after immunization
B it typically affects the large joints of the upper extremity
C it is more common in girls
D it is uncommon in children

E arthralgia usually begins immediately after administration of immunization
ANSWER:D
2074
Postinfectious arthritis describes arthritis that occurs after infectious illnesses, mainly viruses.
Of the following, the LEAST likely causative viral infection is
A rubella
B varicella-zoster
C cytomegalovirus
D Epstein-Barr virus
E herpes-simplex virus
ANSWER:D
2075
A 4-year-old boy has an acute severe left groin pain that refers to thigh. Examination reveals
limitation of movement of the left hip joint. Lab investigations reveal normal blood count
and indices, and normal ESR; radiologic imaging shows widening of the hip joint space with
effusion. You suspect transient synovitis. Of the following, the BEST therapeutic approach
for this boy is
A systemic steroids
B methotrexate
C aspiration of joint fluid
D intravenous antibiotics
E observation
ANSWER:C
2076
Reactive arthritis has the potential for evolving to chronic arthritis, especially after bacterial
enteric infection or genitourinary tract infection (UTI). Of the following, the microorganism
that is MOST likely causing chronic arthritis after UTI is
A Escherichia coli
B Chlamydia trachomatis
C Proteus mirabilis
D Pseudomonas aeruginosa
E Staphylococcus saprophyticus
ANSWER:B
2077
Systemic lupus erythematosus (SLE) is a chronic autoimmune disease characterized by
multisystem inflammation and the presence of circulating autoantibodies directed against self-
antigens. It occurs in both children and adults. All the following are features of childhood
SLE EXCEPT
A it is has a more severe course

B there is more widespread organ involvement
C it usually presents before 5 year of age
D fever, fatigue, hematologic abnormalities, and arthritis are common clinical manifestations
E
although renal disease is asymptomatic, it is oftenly present as nephrotic syndrome in
adolescent age group
ANSWER:C
2078
Skin is a commonly involved organ by SLE. There are different cutaneous manifestations.
Of the following, the skin manifestation that is MOST suggestive of SLE in children is
A malar rash
B discoid rash
C photosensitive rash
D cutaneous vasculitis
E livedo reticularis
ANSWER:B
2079
Drug-induced lupus refers to the presence of SLE manifestations triggered by exposure to
specific medications, including antibiotics. Of the following, the drug that is MOST likely
associated with drug-induced lupus is
A isoniazid
B rifampin
C nitrofurantoin
D penicillin
E tetracycline
ANSWER:A
2080
Systemic lupus erythematosus (SLE) is often characterized by periods of flare and disease
quiescence or may follow a more smoldering disease course. All the following lab tests
correlate with active disease EXCEPT
A positive anti-nuclear antibody titer
B positive anti–double-stranded DNA level
C low serum complement level
D high erythrocyte sedimentation rate
E elevated C-reactive protein (CRP) value
ANSWER:A

2081
A 12-year-old female adolescent has a recent diagnosis of SLE; she has a faint malar rash
and mild arthritis including both elbow and knee joints as well as the joints of small fingers.
Of the following, the MOST appropriate initial therapy for this patient is
A systemic steroids
B methotrexate
C leflunomide
D cyclophosphamide
E hydroxychloroquine
ANSWER:E
2082
A 3-week-old male baby develops malar rash involving the face and periorbital area after
exposure to sun light; you suspect neonatal lupus. Of the following, the NEXT step of
management is
A measurement of anti-Ro and anti-La antibodies of the mother
B measurement of anti-Ro and anti-La antibodies of the baby
C cardiac evaluation of the mother
D cardiac evaluation of the baby
E hematological evaluation of the baby
ANSWER:D
2083
You are meeting a pregnant mother who has had a previous baby with congenital heart
block due to neonatal lupus; she has a positive anti-SSA and anti-SSB antibodies. All the
following are therapeutic modalities to prevent occurrence of congenital fetal cardiac
complications EXCEPT
A fluorinated corticosteroids
B intravenous immunoglobulin
C plasmapheresis
D hydroxychloroquine
E methotrexate
ANSWER:E
2084
Juvenile dermatomyositis (JDM) is the most common inflammatory myositis in children
characterized by skin rash and proximal muscle weakness. All the following are common
cutaneous manifestations of JDM EXCEPT
A heliotrope rash of the eyelids
B photosensitivity to ultraviolet light
C facial erythema sparing the nasolabial folds
D Gottron papules

E periungual telangiectasias
ANSWER:C
2085
Treatment of juvenile dermatomyositis (JDM) includes pharmacological and non-
pharmacological therapy. The mainstay of pharmacological therapy is corticosteroids that
should be integrated with non-pharmacological therapy. All the following are recommended
as part of non-pharmacological treatment program in JDM EXCEPT
A physical therapy
B occupational therapy
C avoidance of sun exposure
D bed rest
E social and psychological support
ANSWER:C
2086
Most complications from Juvenile dermatomyositis (JDM) are related to prolonged and
severe weakness; secondary complications from medical treatments are also common. Of the
following, the LEAST likely recognized complications of JDM is
A aspiration pneumonia
B gastrointestinal bleeding
C cardiac arrhythmias
D muscle atrophy
E skin calcifications
ANSWER:C
2087
Juvenile localized scleroderma (JLS) is generally insidious with skin manifestations that vary
according to disease subtype. Up to 25% of children with LS have extracutaneous
manifestations, including arthritis and neurological symptoms.Of the following, the subtype
that is MOST commonly associated with neurological manifestations is
A plaque morphea
B eosinophilic fasciitis
C generalized morphea
D en coup de sabre
E morphea profunda
ANSWER:D
2088
Juvenile systemic scleroderma (JSS) has a prolonged course with periods of remission and
exacerbation. It is characterized by multisystem organ involvement including skin,
locomotor system, CNS, and viscera. Of the following, the MOST common early cutaneous
manifestation is

A edema of the dorsum of the hands and fingers
B induration and fibrosis of the skin
C flexion contractures at the elbows, hips, and knees
D skin ulceration over pressure points
E subcutaneous calcifications
ANSWER:A
2089
You are discussing Raynaud phenomenon (RP) and Raynaud disease (RD) with medical
students. You state that RP is usually associated with rheumatic diseases, while RD is
independent of an underlying rheumatic disease. Of the following, the feature that is MOST
likely consistent with RD, rather than RP is
A presence in early childhood
B absence of tissue necrosis and gangrene
C asymmetric occurrence
D presence of periungual telangiectasia
E associated pain and paresthesia
ANSWER:B
2090
Behçet disease (BD) is a primary variable vessel vasculitis, characterized by exacerbations
and remissions. The hallmark of the disease is oral ulceration. All the following are
characteristic features of the oral ulcers EXCEPT
A very painful
B recurrent
C involve any location in the oral cavity
D heal with scarring
E last 3-10 days
ANSWER:D
2091
You are evaluating a 16-year-old female adolescent who has been diagnosed with Behçet
disease since the age of 12 year; she recently develops oral genital ulcers; examination
reveals multiple oral ulcers involving the hard palate, multiple genital ulcers involving the
labia majora, and erythema nodosum. There is no major organ involvement and uveitis. Of
the following, the BEST treatment for this patient is
A colchicine
B azathioprine
C cyclophosphamide
D systemic steroids
E interferon alpha

ANSWER:A
2092
Primary Sjögren syndrome, although rare, may occur in children between 9-10 year. Of the
following, the MOST common manifestation in children is
A recurrent parotitis
B sicca symptoms
C polyarthritis
D vulvovaginitis
E hepatitis
ANSWER:A
2093
Familial Mediterranean fever (FMF) is a recessively inherited autoinflammatory disease
usually characterized by recurrent self-limited episodes of fever, serositis, arthritis, and skin
rash. Of the following, the hallmark cutaneous finding of this disease is
A erysipelas-like erythema overlying the dorsum of the foot
B morbilliform rash
C migratory rash overlying area of myalgia
D cold-induced urticaria like lesions
E generalized pustulosis
ANSWER:A
2094
Amyloidosis is the most serious complication of familial Mediterranean fever (FMF), and in
its absence FMF patients may live a normal life span. Of the following, the organ that is not
affected by secondary amyloidosis of FMF is
A kidney
B lung
C nerve
D heart
E teste
ANSWER:C
2095
Amyloidosis is a disease caused by protein misfolding; the type referred as amyloid A (AA),
amyloidosis usually develops in patients with chronic inflammatory states, especially juvenile
idiopathic arthritis. In which of the following subtypes of JIA there is a highest prevalence of
AA amyloidosis?
A systemic JIA
B oligoarticular JIA
C polyarticular JIA
D psoriatic arthritis

E enthesitis-related arthritis
ANSWER:A
2096
Although there is no established therapy to AA amyloidosis, colchicine may be an effective
drug to prevent the development of amyloidosis. Of the following, the disease that is respond
to colchicine in preventing AA amyloidosis is
A juvenile idiopathic arthritis
B ankylosing spondylitis
C familial Mediterranean fever
D hyper IgD syndrome
E cryopyrin-associated periodic syndrome
ANSWER:C
2097
Kawasaki disease (KD) is an acute febrile illness of childhood characterized by vasculitis
with a predilection for the coronary arteries. Predictors of poor outcome across several
studies include all the following EXCEPT
A old age
B male gender
C persistent fever
D poor response to IVIG
E elevated C-reactive protein levels
ANSWER:A
2098
Kawasaki disease (KD) has unique clinical manifestations; however, less consistent clinical
presentation may occur. All the following are common classic clinical features of KD
EXCEPT
A bilateral non exudative bulbar conjunctivitis
B cracked lips
C edema and erythema of the hands and feet
D vesicular rash
E unilateral non suppurative cervical lymphadenopathy
ANSWER:D
2099
You are evaluating a 1-year-old boy with an established diagnosis of Kawasaki disease (KD)
since 4 days; the caring nurse asks you about cardiac involvement during this stage. All the
following are possible cardiac complications at this stage EXCEPT
A myocarditis
B pericarditis
C coronary artery aneurysm

D mitral regurgitation
E cardiogenic shock
ANSWER:C
2100
A follow up two-dimensional echocardiography is performed to a 1.5-year-old boy with
Kawasaki disease (KD) 3 weeks after the diagnosis; it shows a small solitary aneurysm of the
left anterior descending coronary artery without thrombosis. Of the following, the MOST
appropriate therapy for this child is
A aspirin for 6 months
B life-long aspirin
C IVIG and aspirin for 14 days
D aspirin and clopidogrel antiplatelet for 8 weeks
E aspirin and warfarin for 6 months
ANSWER:B
2101
Henoch-Schönlein purpura (HSP) is the most common vasculitis of childhood affecting
small vessels in the skin, joints, gastrointestinal tract, and kidney. Gastrointestinal
involvements occur in up to 80% of children with HSP. Of the following, the LEAST
gastrointestinal manifestation that may occur in children with HSP is
A abdominal pain
B vomiting and diarrhea
C paralytic ileus
D melena
E intussusception
ANSWER:E
2102
A 5-year-old boy has acute onset of abdominal pain and melena; he has been diagnosed with
Henoch-Schönlein purpura (HSP) one week before. Examination reveals symmetrical
palpable purpura overlying the lower extremities and buttock; other examinations are
unremarkable. Of the following, the BEST treatment for this boy is
A supportive measures
B steroids
C azathioprine
D cyclophosphamide
E mycophenolate mofetil
ANSWER:B

2103
Renal disease is the major long-term complication, occurring in 1-2% of children with
Henoch-Schönlein purpura (HSP). Chronic HSP renal disease is managed with a variety of
immunosuppressants. Of the following, the LEAST effective drug for the treatment of renal
disease in HSP is
A prednisolone
B azathioprine
C cyclophosphamide
D cyclosporine
E mycophenolate mofetil
ANSWER:A
2104
55. Childhood vasculitis encompasses a broad spectrum of diseases that share in common
inflammation of the blood vessels. Vascular injury includes small, medium, and large vessels.
Of the following, the disease that affects predominantly large blood vessels is
A Henoch-Schönlein purpura
B granulomatosis with polyangiitis (wegener granulomatosis)
C childhood polyarteritis nodosa
D Kawasaki disease
E Takayasu arteritis
ANSWER:E
2105 Approximately 80% of blood cultures that will be positive are identified at
A 1st 24 hr from incubation
B 25 – 48 hr from incubation
C after 3 day from incubation
D after 5 day from incubation
E after 7 day from incubation
ANSWER:A
2106 The optimal amount of blood to collect from a pediatric patient for blood culture depend on
A age
B weight
C length
D BMI
E laboratory standards
ANSWER:B

2107
The IgM response occurs earlier in the illness, generally peaking at 7-10 days after infection,
and usually disappears within a few weeks, but for some infections it can persist for months
such as
A measles
B mumps
C rubella
D hepatitis A
E varicella
ANSWER:D
2108
Vaccines are defined as whole or parts of microorganisms administered to prevent an
infectious disease. Which of the following is a live attenuated vaccine?
A hepatitis A
B hepatitis B
C pneumococcal
D varicella
E diphtheria
ANSWER:D
2109 Which of the following is a T-lymphocyte independent vaccine?
A hepatitis A
B hepatitis B
C pneumococcal
D varicella
E diphtheria
ANSWER:C
2110 Rotavirus vaccine should not be initiated for infants older than
A 11 wk
B 13 wk
C 15 wk
D 17 wk
E 19 wk
ANSWER:C
2111 The final dose of rotavirus vaccine must be administered no later than
A 6 mo of age
B 8 mo of age
C 10 mo of age

D 12 mo of age
E 18 mo of age
ANSWER:B
2112
Immunization is one of the most beneficial and cost-effective disease prevention measures.
As a result of effective and safe vaccines, which of the following diseases has been
eradicated?
A smallpox
B polio
C measles
D rubella
E pertussis
ANSWER:A
2113 Infants, children, and adolescents in Iraq are routinely immunized against
A 10 diseases
B 11 diseases
C 12 diseases
D 13 diseases
E 14 diseases
ANSWER:B
2114 The MOST common adverse reaction to intramuscular immunoglobulin is
A pain at the injection site
B flushing
C headache
D chills
E nausea
ANSWER:A
2115 All the following are major recommended indications for IVI EXCEPT
A replacement therapy for primary immunodeficiency disorders
B Kawasaki disease
C hepatitis A prophylaxis
D immune-mediated thrombocytopenia
E prophylaxis of infection following bone marrow transplantation
ANSWER:C
2116 Serious reactions to IVIG include all the following EXCEPT
A anaphylactoid events

B thromboembolic disorders
C aseptic meningitis
D carditis
E renal insufficiency
ANSWER:D
2117
A toxoid is a modified bacterial toxin that is made non-toxic but still able to induce an active
immune response against the toxin. Which of the following vaccine is a toxoid?
A hepatitis A
B hepatitis B
C pneumococcal
D varicella
E diphtheria
ANSWER:E
2118
Hepatitis A vaccine, licensed for administration to children 12 mo of age and older. The 2
doses in the series should be separated by at least
A 2 mo
B 4 mo
C 6 mo
D 1 yr
E 2 yr
ANSWER:C
2119 The minimum interval between the 2 doses of MMR is
A 2 wk
B 4 wk
C 2 mo
D 6 mo
E 1 yr
ANSWER:B
2120 The minimum age for the last dose of hepatitis B vaccine is
A 16 weeks
B 20 weeks
C 24 weeks
D 28 weeks
E 32 weeks
ANSWER:C

2121 Preterm infant, weight 1600 gm, should not receive the following vaccine at birth
A BCG
B hepatitis B, if born to a HBs Ag negative mother
C polio
D DPT
E MMR
ANSWER:B
2122
Which of the following vaccines is contraindicated for a patient with X-linked
agammaglobulinemia?
A BCG
B hepatitis B
C DPT
D MMR
E varicella
ANSWER:A
2123 Which of the following vaccines is contraindicated for a patient with chronic renal disease?
A pneumococcal
B hepatitis B
C live attenuated influenza
D varicella
E hepatitis A
ANSWER:C
2124
Waterless hand hygiene products are effective in killing most microbes but do not remove
dirt or debris and are ineffective against
A Pseudomonas
B hepatitis A
C Salmonella
D C. difficile
E S. aureus
ANSWER:D
2125
All the following are recognized skin infections or infestations in children in childcare
EXCEPT
A impetigo
B pediculosis

C scabies
D erythrasma
E tinea corporis
ANSWER:D
2126 A 3-year-old child diagnosed with shigella infection, he can return to his daycare
A 2 days after initiation of treatment
B after diarrhea resolves
C after diarrhea resolves and results of a single stool culture is negative for these organisms
D after diarrhea resolves and results of 2 stool cultures are negative for these organisms
E after diarrhea resolves and results of 3 stool cultures are negative for these organisms
ANSWER:D
2127
Oral rehydration is the mainstay of treatment for pediatric traveler’s diarrhea and the drug of
choice is
A metronidazol
B azithromycin
C amoxicillin
D trimethoprim-sulfamethoxazol
E erythromycin
ANSWER:B
2128 Fever is defined as a rectal temperature of
A ≥37.5°C
B ≥37.6°C
C ≥38°C
D ≥38.1°C
E ≥38.5°C
ANSWER:C
2129
Three mechanisms can produce fever: pyrogens, heat production exceeding loss, and
defective heat loss. Endogenous pyrogens include
A antigen–antibody complexes
B complement components
C lymphocyte products
D interferons β and γ
E androgenic steroid metabolites
ANSWER:D
2130 Drugs that are known to cause fever include

A chlorpheniramine
B allopurinol
C diphenhydramine
D acetazolamide
E adenosine
ANSWER:B
2131 Ingestion of dirt is an important clue to infection with
A amebiasis
B giardiasis
C malaria
D trichinosis
E toxoplasmosis
ANSWER:E
2132 Bulbar conjunctivitis in a child with FUO suggests
A leptospirosis
B coxsackievirus infection
C tuberculosis
D infectious mononucleosis
E lymphogranuloma venereum
ANSWER:A
2133 Multiple blood cultures may be required to detect bacteremia associated with
A malaria
B pyelonephritis
C brucellosis
D osteomyelitis
E pneumonia
ANSWER:D
2134
Primary immunodeficiencies are compromised states that result from genetic defects
affecting 1 or more arms of the immune system while secondary immunodeficiencies result
from infection, malignancy, or as an adverse effect of immunomodulating or
immunosuppressing medications. Which of the following represent secondary immune
deficiency?
A Shwachman-Diamond syndrome
B cystic fibrosis
C Chédiak-Higashi syndrome

D Omenn syndrome
E ataxia-telangiectasia
ANSWER:B
2135 Which type of fever is persistent and varies by more than 0.5°c (0.9°f)/day?
A intermittent
B hectic
C sustained
D remittent
E relapsing
ANSWER:D
2136 All the following are causes of very high temperatures (>41°C) EXCEPT
A malignant hyperthermia
B malignant neuroleptic syndrome
C infection
D drug fever
E heat stroke
ANSWER:C
2137
Relative bradycardia (when the pulse rate remains low in the presence of fever) can
accompany all the following conditions EXCEPT
A typhoid fever
B brucellosis
C leptospirosis
D visceral leishmaniasis
E drug fever
ANSWER:D
2138 The MOST common serious bacterial infection in infant aged 1-3 mo is
A pyelonephritis
B meningitis
C pneumonia
D septic arthritis
E osteomylitis
ANSWER:A
2139 Neutropenia is defined as an absolute neutrophil count of less than
A 1,000 cells/mm3
B 1,500 cells/mm3

C 2,000 cells/mm3
D 2,500 cells/mm3
E 3,000 cells/mm3
ANSWER:A
2140
Leukocyte adhesion defects are caused by defects in the β chain of integrin (CD18), which is
required for the normal process of neutrophil aggregation and attachment to endothelial
surfaces. It is characterized by all the following EXCEPT
A delayed cord separation
B recurrent infections
C ecthyma gangrenosum
D neutropenia
E survival is usually <10 yr
ANSWER:D
2141 Penicillins are the drugs of choice for pediatric infections caused by the following EXCEPT
A group A Streptococcus
B H .influenzae
C Treponema pallidum
D L. monocytogenes
E N. meningitides
ANSWER:B
2142 Cefixime is a third-generation cephalosporin active against all the following EXCEPT
A Streptococci
B Staphylococci
C H. influenza
D Neisseria gonorrhoeae
E Proteus vulgaris
ANSWER:B
2143
Cephalexin is a cephalosporin active against S. aureus, Streptococcus, E. coli, Klebsiella, and
Proteus. To which generation of cephalosporins it belongs?
A 1st
B 2nd
C 3rd
D 4th
E 5th

ANSWER:A
2144
Meropenem is a carbapenem antibiotic with broad-spectrum activity but has no activity
against
A P. aeruginosa
B S. maltophilia
C L. monocytogenes
D S. aureus
E N. meningitides
ANSWER:B
2145
Metronidazole is highly effective in the treatment of infections caused by anaerobes. It can
increase the level of which of the following drugs if given at the same time
A carbamazepine
B rifampin
C phenobarbital
D phenytoin
E vaiproic acid
ANSWER:D
2146
Cephalosporins are widely used in pediatric practice, both in oral and parenteral
formulations. Which of the following is a 2nd generation cephalosporins?
A cefazolin
B cephalexin
C cefuroxime
D ceftazidime
E ceftaroline
ANSWER:C
2147
Which cephalosporin should not be mixed or reconstituted with a calcium- containing
product, such as Ringer solution or parenteral nutrition containing calcium?
A cefazolin
B cefotaxime
C ceftriaxone
D ceftazidime
E cefepime
ANSWER:C
2148
Toxic shock syndrome is an acute and potentially severe illness characterized by all the
following EXCEPT

A desquamation on the hands and feet
B myalgias
C focal neurologic abnormalities
D conjunctival hyperemia
E strawberry tongue
ANSWER:C
2149
Which of the following represent a major criterion for the diagnosis of staphylococcal toxic
shock syndrome?
A rash
B conjunctival hyperemia
C myalgia
D thrombocytopenia
E vomiting
ANSWER:A
2150
Kawasaki disease closely resembles toxic shock syndrome clinically. However, many of the
clinical features of toxic shock syndrome are usually absent or rare in Kawasaki disease like
A fever unresponsive to antibiotics
B hyperemia of mucous membranes
C erythematous rash
D desquamation
E diffuse myalgia
ANSWER:E
2151
S. pneumoniae is the most frequent cause of bacteremia, bacterial pneumonia, otitis media,
and bacterial meningitis in children. Children at increased risk of pneumococcal infections
include those with the following conditions EXCEPT
A megaloplastic anemia
B deficiencies in humoral immunity
C HIV infection
D cerebrospinal fluid leak
E cochlear implants
ANSWER:A

2152
Group A streptococcus can be subdivided into more than 220 serotypes on the basis of the
M protein antigen, M serotyping is valuable for epidemiologic studies; specific group A
streptococcus diseases tend to be associated with certain M types. Of the following, the M
type associated with glomerulonephritis is
A 1
B 6
C 12
D 18
E 29
ANSWER:C
2153
Scarlet fever is an upper respiratory tract infection associated with a characteristic rash. Of
the following, which statement is TRUE?
A it is caused by an infection with pyrogenic endotoxin producing group A streptococcus
B the rash appears 96 hr after onset of symptoms
C the rash begins to fade after 1-2 weeks
D
before desquamation, the reddened papillae are prominent, giving the tongue a strawberry
appearance
E the milder form can be confused with Kawasaki disease
ANSWER:E
2154
Impetigo (or pyoderma) has traditionally been classified into 2 clinical forms: bullous and
nonbullous. Of the following, which statement is TRUE?
A bullous impetigo is more common
B nonbullous lesions are most common on the trunk and perineum
C regional lymphadenitis is commonly associated with nonbullous lesions
D nonbullous impetigo is generally accompanied by fever
E bullous impetigo usually involve the face and extremeties
ANSWER:C
2155
In which of the following circumstances the diagnosis of acute rheumatic fever can be made
without strict adherence to Jones criteria?
A when chorea occurs as the only major manifestation of acute rheumatic fever
B
when indolent carditis is the only manifestation months after the apparent onset of acute
rheumatic fever
C
in a limited number of patients with recurrences of acute rheumatic fever in particularly high-
risk populations
D all of the above

E none of the above
ANSWER:D
2156
Because no clinical or laboratory finding is pathognomonic for acute rheumatic fever, 5
major and 4 minor criteria with evidence of recent group A streptococcus infection are
required for the diagnosis. Migratory polyarthritis represent one of the major criteria, which
of the following statements is TRUE regarding this major criterion?
A occurs in approximately 50% of patients
B typically involves small joints
C the pain can precede and can appear to be disproportionate to the objective findings
D rheumatic arthritis is almost deforming
E
there is often a proportional relationship between the severity of arthritis and the severity of
cardiac involvement
ANSWER:C
2157 Acute rheumatic carditis usually presents as cardiac murmurs and
A tachycardia
B chest pain
C dyspnea
D syncopal attack
E tachypnea
ANSWER:A
2158
Patients with acute rheumatic carditis and more than minimal cardiomegaly should receive
prednisone 2 mg/kg/day in 4 divided doses for
A 7-10 days
B 2-3 weeks
C 6-8 weeks
D 2-3 months
E 4-6 months
ANSWER:B
2159 55.The drug of choice for Sydenham chorea is
A aspirin
B prednisone
C phenobarbital
D haloperidol
E chlorpromazine
ANSWER:C

2160
To prevent first attack of acute rheumatic fever after acute group A streptococcus
pharyngitis, appropriate antibiotic therapy should be instituted before
A 3rd day of illness
B 5th day of illness
C 7th day of illness
D 9th day of illness
E 11th day of illness
ANSWER:D
2161
Prophylaxis for people who have had acute rheumatic fever with carditis but without residual
heart disease persist for
A 5 yr or until 21 yr of age, whichever is longer
B 10 yr or until 21 yr of age, whichever is longer
C 5 yr or until 40 yr of age, whichever is longer
D 10 yr or until 40 yr of age, whichever is longer
E Lifelong
ANSWER:B
2162
Group B streptococcus (GBS), or Streptococcus agalactiae, is a major cause of neonatal
bacterial sepsis. Of the following, the MOST common syndrome associated with childhood
GBS disease beyond early infancy is
A bacteremia without a focus
B meningitis
C ventriculitis
D septic arthritis
E pneumonia
ANSWER:A
2163 Recommended duration of therapy for manifestations of group B streptococcus are as follow
A bacteremia without a focus: 7 days
B meningitis: 3-6 weeks
C ventriculitis: at least 8 weeks
D septic arthritis: 3-4 weeks
E osteomyelitis: 6-8 weeks
ANSWER:D

2164
Diphtheria is an acute toxic infection; toxic cardiomyopathy occurs in 10-25% of patients
with respiratory diphtheria and is responsible for 50-60% of deaths. Of the following, the
mainstay of therapy is
A antitoxin
B penicillins
C erythromycin
D clindamycin
E rifampin
ANSWER:A
2165
A painless, slow-growing, hard mass producing cutaneous fistulas, a condition commonly
known as lumpy jaw is usually caused by
A Staphylococcus
B Actinomyces
C Nocardia
D Yersinia
E Leptospira
ANSWER:B
2166 The MOST common secondary site involved in cases of pulmonary nocardiosis is
A brain
B skin
C kidney
D liver
E bone
ANSWER:A
2167 Which of the following is a gram negative bacterium?
A Staphylococcus aureus
B Streptococcus pneumoniae
C Actinomyces
D Haemophilus influenza
E Nocardia
ANSWER:D
2168 Regarding epidemiology of Neisseria meningitides, all the following are true EXCEPT
A
meningococci are transmitted during close contact via aerosol droplets or exposure to
respiratory secretions
B meningococci survive for long periods in the environment

C
smoking and respiratory viral infection are associated with increased rates of carriage and
disease
D the highest rate of meningococcal disease occurs in infancy
E most cases of meningococcal disease are sporadic
ANSWER:B
2169 The MOST common clinical manifestation of meningococcal infection is
A asymptomatic carriage
B meningococcal meningitis
C bacteremia without sepsis
D meningococcal septicemia
E pneumonia
ANSWER:A
2170
In meningococcal purpura, necrotic skin lesions are less common among children treated
with
A penicillin G
B ampicillin
C cefotaxime
D ceftriaxone
E meropenem
ANSWER:D
2171 The MOST common complication of acute severe meningococcal septicemia is
A arthritis
B focal skin infarction
C endocarditis
D pneumonia
E peritonitis
ANSWER:B
2172 poor prognostic factor for invasive meningococcal disease on presentation is
A hypertension
B leukocytosis
C alkalosis
D meningitis
E normal erythrocyte sedimentation rate
ANSWER:E
2173 Of the following, the MOST effective agent for prophylaxis of meningococcal disease is

A ceftriaxone
B rifampin
C ampicillin
D penicillin
E amoxicillin
ANSWER:A
2174
Gonorrhea is manifested by a spectrum of clinical presentations from asymptomatic
carriage, to the characteristic localized urogenital infections, to disseminated systemic
infection. Regarding disseminated gonococcal infection, the following statement is TRUE
A hematogenous dissemination occurs in 10-30% of all gonococcal infections
B men account for the majority of cases
C meningitis and osteomyelitis are the most common manifestations
D skin lesions found in75% of patients
E acute endocarditis is an uncommon but often fatal manifestation
ANSWER:E
2175
All patients who are presumed or proven to have gonorrhea should be evaluated for
concurrent presence of all the following infections EXCEPT
A syphilis
B hepatitis B
C HIV
D C. trachomatis
E HSV2
ANSWER:E
2176
Children who have bacteremia or arthritis caused by gonococcal infections should be treated
with ceftriaxone (50 mg/kg/day; maximum: 1 g/day if weighs <45 kg) for a minimum of
A 3 days
B 5 days
C 7 days
D 14 days
E 21 days
ANSWER:C
2177 The MOST common etiology of joint and bone infections in young children is
A Staphylococcus aureus
B Streptococcus pneumoniae

C Kingellakingae
D Haemophilus influenza
E Neisseria meningitidis
ANSWER:C
2178
Diagnosis of chancroid in infants and children is a strong evidence of sexual abuse.
Chancroid is caused by
A Haemophilus ducreyi
B Syphilis
C trachomatis
D Kingellakingae
E Neisseria gonorrhea
ANSWER:A
2179 The MOST important clinical manifestation of M.catarrhalis infection in children is
A pneumonia
B bronchitis
C otitis media
D pharyngitis
E cystitis
ANSWER:C
2180 The MOST common reason for which children receive antibiotics is
A pneumonia
B bronchitis
C otitis media
D pharyngitis
E cystitis
ANSWER:C
2181
Infants younger than 3 mo of age with suspected pertussis usually are admitted to hospital,
as are many between 3 and 6 mo of age unless witnessed paroxysms are not severe, as well as
are patients of any age if significant complications occur. Typical paroxysms that are not life
threatening have the following feature
A duration >45 sec
B blue color change
C bradycardia <60 beats/min in infants
D oxygen desaturation that spontaneously resolves at the end of the paroxysm
E post-tussive unresponsiveness

ANSWER:D
2182 All the following regarding assessment and care of infants with pertussis are true EXCEPT
A infants with potentially fatal pertussis may appear well between episodes
B a paroxysm must be witnessed before a decision is made between hospital and home care
C suctioning of nose, oropharynx, or trachea should
D feeding in the period following a paroxysm may be more successful than after napping
E
family education, recruitment as part of the team, and continued support after discharge are
essential
ANSWER:C
2183
Hospital discharge of infants with pertussis is appropriate in all the following circumstances
EXCEPT
A over a 24-hr period disease severity is unchanged or diminished
B intervention is not required during paroxysms
C nutrition is adequate
D no complication has occurred
E parents are adequately prepared for care at home
ANSWER:A
2184 The preferable antibiotic in neonate with pertussis is
A azithromycin
B erythromycin
C clarithromycin
D trimethoprim-sulfamethoxazole
E amoxycilline
ANSWER:A
2185
Echocardiography should be performed in critically ill infants with pertussis to detect
presence of
A myocarditis
B heart failure
C congenital heart defect
D pulmonary hypertension
E right atrial dilatation
ANSWER:D
2186 The least common clinical feature of typhoid fever in children is
A diarrhea
B abdominal pain

C pallor
D splenomegaly
E headache
ANSWER:E
2187 All the following are clinical features of typhoid fever EXCEPT
A incubation period is usually 30-45 days
B
macular or maculopapular rash (rose spots) may be visible around the 7th-10th day of the
illness
C
if complications not occur, the symptoms and physical findings gradually resolve within 2-4
wk
D typhoid fever usually manifests as high-grade fever with a wide variety of associated features
E
relative bradycardia, neurologic manifestations, and gastrointestinal bleeding, are rare in
children
ANSWER:A
2188 Regarding diagnosis of typhoid fever, one of the following is TRUE
A blood cultures are positive in 65-80% of the patients
B urine culture results become positive within the 1st wk
C thrombocytosis may be a marker of severe illness
D diagnosis by Widal test alone is prone to error
E leukocytosis is rare in young children
ANSWER:D
2189 All the following are clinical features of shigellosis EXCEPT
A an incubation period of 12 hr to several days
B most children never progress to the stage of bloody diarrhea
C untreated diarrhea can last more than 4 weeks
D neurologic findings are the most common extraintestinal manifestations
E neonatal shigellosis is rare
ANSWER:C
2190
Cholera is a dehydrating diarrheal disease caused by Vibrio cholerae, of more than 200
serogroups; the serogroups that have been associated with epidemics are
A O131
B O139
C O151
D O159

E O165
ANSWER:B
2191
Persons with which blood group are at increased risk for developing severe disease of
cholera?
A A
B B
C AB
D O
E Blood group not related to the severity of the disease
ANSWER:D
2192 Seizure in cholera is commonly due to
A hypocalcemia
B hyponatremia
C hypernatremia
D hypoglycemia
E fever
ANSWER:D
2193
Complicationsofcampylobacterjejunicanincludeacuteandlateonset complications that may
present after the acute infection has resolved. Of the following, the MOST common late-
onset complication is
A reactive arthritis
B immunoglobulin A nephropathy
C immune complex glomerulonephritis
D hemolytic anemia
E carditis
ANSWER:A
2194 MOST Campylobacter isolates are susceptible to
A aminoglycosides
B cephalosporins
C rifampin
D penicillins
E trimethoprim
ANSWER:A

2195
The genus Yersinia is a member of the family Enterobacteriaceae and comprises more than
14 named species, 3 of which are established as human pathogens. Which of the following is
MOST often associated with mesenteric lymphadenitis?
A Yersinia enterocolitica
B Yersinia pseudotuberculosis
C Yersinia pestis
D Yersinia mollaretii
E Yersinia rohdei
ANSWER:B
2196
Patients with conditions leading to iron overload are at higher risk of developing infections
with
A Aeromonas
B Pseudomonas aeruginosa
C Yersinia
D Francisella tularensis
E Campylobacter
ANSWER:C
2197 The MOST common complication of Y. enterocolitica infection in younger children is
A reactive arthritis
B erythema multiforme
C hemolytic anemia
D thrombocytopenia
E septicemia
ANSWER:E
2198
Tularemia is a zoonotic infection caused by the gram-negative bacterium Francisella
tularensis. Of the following, the MOST common forms of tularemia diagnosed in children is
A ulceroglandular
B pneumonia
C oropharyngeal
D oculoglandular
E typhoidal
ANSWER:A
2199
Human brucellosis is caused by organisms of the genus Brucella and continues to be a major
public health problem worldwide. All the following are true EXCEPT

A symptoms can be acute or insidious
B most patients present with fever, arthralgia/arthritis, and hepatosplenomegaly
C some present as a fever of unknown origin
D variable fever pattern
E invasion of the nervous system occurs in approximately 20% of cases
ANSWER:E
2200 Which antibiotics should be avoided in patients with botulism?
A penicillin
B cephalosporin
C macrolide
D aminoglycoside
E trimethoprim-sulfamethoxazole
ANSWER:D
2201
Tetanus is an acute spastic paralytic illness historically called lock-jaw that is caused by the
neurotoxin produced by Clostridium tetani. Of the following, the TRUE statement is
A tetanus is most often localized
B incubation period typically is 2-6 weeks
C patient remains conscious and there is no pain
D tetanic paralysis becomes more severe in the 4th wk after onset
E cephalic tetanus occurs in association with chronic otitis media
ANSWER:E
2202 Worsening of acne in adolescent female on anti-tuberculous treatment is caused by
A isoniazid
B rifampin
C pyrazinamide
D ethambutol
E streptomycin
ANSWER:B
2203
Isoniazid is accompanied by significant drug–drug interactions, which of the following is
TRUE?
A aluminum salts increase absorption of isoniazid
B isoniazid increase toxicity of carbamazepine
C rifampin decreased hepatotoxicity of isoniazid
D isoniazid decrease level of warfarin
E prednisolone decreased isoniazid metabolism

ANSWER:B
2204 100.The ideal agent for treating fungal urinary tract infections is
A amphotericin B
B fluconazole
C voriconazole
D micafungin
E caspofungin
ANSWER:B
2205
101.Candida is a common cause of oral mucous membrane infections (thrush) and perineal
skin infections (Candida diaper dermatitis) in young infants. All the following are true
EXCEPT
A
candida species are the third most common cause of bloodstream infection in premature
infants
B
up to 10% of full-term infants are colonized as the result of vertical transmission from the
mother at birth
C histamine-2 blockers facilitate Candida colonization and overgrowth
D
significant risk factors for neonatal invasive candidiasis include the presence of a central
venous catheter
E the cumulative incidence is <0.3% among infants <750 g birthweight admitted to the NICU
ANSWER:E
2206
102.NICUs with a high incidence of invasive candidiasis should consider prophylaxis with
fluconazole in infants with a birthweight of
A <750 g
B <1,000 g
C <1,500 g
D <2,000 g
E <2,500 g
ANSWER:B
2207
Antiviral chemotherapy typically requires a delicate balance between targeting critical steps
in viral replication without interfering with host cellular function. Which of the following
antiviral blocks M2 protein ion channel?
A acyclovir
B amantadine
C ganciclovir

D foscarnet
E vidarabine
ANSWER:B
2208
In neonates with HSV infection including CNS involvement, to improve
neurodevelopmental outcome, suppressive therapy with oral acyclovir should be used for
A 3 mo
B 6 mo
C 9 mo
D 12 mo
E 18 mo
ANSWER:B
2209
Acyclovir is a safe and effective therapy for herpes simplex virus (HSV) infections. All the
following are true EXCEPT
A activity against CMV is less pronounced
B activity against Epstein-barr virus is modest, both in vitro and clinically
C acyclovir therapy in a nursing mother is not a contraindication to breastfeeding
D main route of elimination is hepatic
E high doses of acyclovir are associated with neurotoxicity
ANSWER:D
2210
Ribavirin is a guanosine analog that has broad-spectrum activity against a variety of viruses,
particularly RNA viruses. All the following are true EXCEPT
A its precise mechanism of action is incompletely understood
B aerosolizedribavirineffectiveforparainfluenza,influenza,andmeasles infections
C ribavirin is generally nontoxic, particularly when administered by aerosol
D ribavirin and its metabolites concentrate in hepatocytes
E conjunctivitis and bronchospasm have been reported following exposure to aerosolized drug
ANSWER:D
2211
The portal of entry of measles virus is through the respiratory tract or conjunctivae following
contact with large droplets or small-droplet aerosols in which the virus is suspended. Patients
with measles are infectious
A 1 day before to up to 4-6 days after the onset of rash
B 2 days before to up to 4-6 days after the onset of rash
C 3 days before to up to 4-6 days after the onset of rash
D 4 days before to up to 4-6 days after the onset of rash

E 5 days before to up to 4-6 days after the onset of rash
ANSWER:C
2212
Measles is a serious infection characterized by high fever, an enanthem, cough, coryza,
conjunctivitis, and a prominent exanthem. All the following are true EXCEPT
A incubation period is 8-12 days
B Koplik spots have been reported in 50-70% of measles cases
C the rash begins on the forehead, behind the ears, and on the upper neck
D the rash fades over about 3 days in the same progression as it evolved
E in more severe cases, generalized lymphadenopathy may be present
ANSWER:D
2213 The MOST common complication of measles is
A aseptic meningitis
B pharyngitis
C otitis media
D pneumonia
E cystitis
ANSWER:C
2214
Subacute sclerosing panencephalitis (SSPE) is a chronic complication of measles with a
delayed onset and an outcome that is nearly always fatal. SSPE is characterized by
A females are affected twice as often as males
B clinical manifestations begin insidiously 3-6 yr after primary measles infection
C the hallmark of the 1st stage is massive myoclonus
D the 3rd stage is characterized by loss of critical centers
E clinical trials using isoprinosine suggest significant benefit
ANSWER:E
2215 The MOST common finding among infants with congenital rubella syndrome is
A psychomotor retardation
B cataracts
C deafness
D patent ductus arteriosus
E neonatal purpura
ANSWER:C
2216
Mumps virus is in the family Paramyxoviridae and the genus Rubulavirus. Mumps is
characterized by
A incubation period ranges from 7-11 days

B unilateral parotitis rarely becomes bilateral
C pale opening of the Stensen duct
D parotid swelling peaks in approximately 7 days
E a morbilliform rash is rarely seen
ANSWER:E
2217 The MOST common complication of mumps is
A meningitis
B conjunctivitis
C optic neuritis
D pneumonia
E thrombocytopenia
ANSWER:A
2218 Intrauterine mumps infection have been associated with
A glaucoma
B limb anomalies
C pulmonary stenosis
D renal agenesis
E no fetal malformations
ANSWER:E
2219
Mumps virus is neurotropic and is thought to enter the CNS via the choroid plexus and infect
the choroidal epithelium and ependymal cells. CNS involvement is characterized by
A symptomatic CNS involvement occurs in 40-60% of infected individuals
B encephalitis most commonly manifests 10 days after the parotitis
C CNS symptoms usually resolve in 3 days
D CSF protein content is usually elevated
E facial palsy is a less-common CNS complication
ANSWER:E
2220 Hematogenous dissemination of HSV to the central nervous system occur in
A neonates
B individuals with eczema
C severely malnourished children
D infants on steroids
E all of the above
ANSWER:A
2221 Herpes whitlow is HSV infection of the

A eye
B mouth
C fingers
D ear
E lip
ANSWER:C
2222 The MOST common cause of recurrent aseptic meningitis is
A mumps
B EBV
C adenoviruses
D HSV
E influenza virus
ANSWER:D
2223 In neonatal HSV encephalitis, skin vesicles occur in approximately
A 20%
B 40%
C 60%
D 80%
E 100%
ANSWER:C
2224
In patients with neonatal HSV infection who receive IV treatment for 2-3 weeks then
suppressive treatment for 6 months, they should be monitored by
A absolute neutrophil count ANC
B liver function tests
C renal function tests
D platelet count
E Hb%
ANSWER:A
2225
Varicella-zoster virus (VZV) causes primary, latent, and recurrent infections. All the
following statements are true EXCEPT
A varicella is a serious disease in young infants
B within households, transmission of VZV occurs at a rate of 65-86%
C Herpes zoster is more common in winter
D the lifetime risk for herpes zoster for individuals with a history of varicella is 20- 30%
E Herpes zoster is very rare in healthy children younger than 10 yr of age

ANSWER:C
2226
122.Varicella is an acute febrile rash illness that is common in Iraq. Of the following, the
TRUE statement is
A it usually begins 4-6 days after exposure
B subclinical varicella is common
C mild abdominal pain may occur 24-48 hr before the rash appears
D temperature elevation usually as high as 41.1°c
E lesions often appear first on the extremities
ANSWER:C
2227 123.All the following are true about varicella in unvaccinated individuals EXCEPT
A simultaneous presence of lesions in the same stages of evolution
B distribution of the rash is predominantly central
C many children have vesicular lesions on the eyelids
D exanthem may be much more extensive in children with skin disorders
E
hypopigmentation or hyperpigmentation of lesion sites persists for days to weeks in some
children
ANSWER:A
2228
Encephalitisandacutecerebellarataxiaarewell-describedneurologic complications of varicella.
All the following are true EXCEPT
A
morbidity from central nervous system complications is highest among patients younger than
5 yr and older than 20 yr
B nuchal
C
patients with cerebellar ataxia have a gradual onset of gait disturbance, nystagmus, and
slurred speech
D
neurologic symptoms usually begin 2-6 days after the onset of the rash but may occur
during the incubation period or after resolution of the rash
E clinical recovery is typically occurring after 96 hr, and is usually gradual
ANSWER:E
2229
Oral therapy with acyclovir (20 mg/kg/dose; maximum: 800 mg/dose) given as 4 doses/day
for 5 days can be used to treat uncomplicated varicella in individuals at increased risk for
moderate to severe varicella. It includes all the following individuals EXCEPT
A nonpregnant individuals older than 12 yr of age
B individuals older than 12 mo of age with chronic cutaneous disorders
C individuals receiving short-term corticosteroid therapy

D individuals receiving long-term salicylate therapy
E individuals with chronic hematological disorders
ANSWER:E
2230 Epstein-Barr virus (EBV) is shed in oral secretions after acute infection for
A 2 wk
B 4 wk
C 2 mo
D 4 mo
E 6 mo
ANSWER:E
2231 What percent of the world’s population infected by Epstein-Barr virus (EBV)?
A 5%
B 25%
C 50%
D 75%
E 95%
ANSWER:E
2232
Infectious mononucleosis is the best-known clinical syndrome caused by Epstein- Barr virus
(EBV). It is characterized by
A symptomatic hepatitis
B elevated liver enzymes
C massive splenic enlargement
D huge hepatomegaly
E occasional palatal petechiae
ANSWER:B
2233 The MOST common long-term sequela associated with congenital CMV infection is
A renal failure
B hearing loss
C heart failure
D vision loss
E hepatic failure
ANSWER:B

2234
Human herpesvirus 6 (HHV-6A and HHV-6B) and human herpesvirus 7 (HHV-7) cause
infection in infancy and early childhood. HHV-6B is responsible for the majority of cases of
roseola infantum (exanthema subitum or sixth disease). All the following are true EXCEPT
A 95% of children being infected with HHV-6 by 2 yr of age
B peak age of primary HHV-6B infection is 6-9 mo of life
C congenital infection with HHV-6 occurs in 1% of newborns
D congenital infection with HHV-7 is well recognized
E breast milk does not play a role in transmission of either HHV-6 or HHV-7
ANSWER:D
2235
Amantadine and rimantadine are effective only against influenza A viruses and are not
approved for use in children younger than
A 6 mo
B 18 mo
C 3 yr
D 5 yr
E 10 yr
ANSWER:D
2236 Typically, the first sign of infection in infants with respiratory syncytial virus (RSV) is
A rhinorrhea
B cough
C low-grade fever
D increased respiratory rate
E subcostal retractions
ANSWER:A
2237 For military services, vaccines are available for human adenoviruses types
A 1 and 4
B 2 and 5
C 3 and 6
D 4 and 7
E 5 and 8
ANSWER:D
2238
Respiratory tract infections are common manifestations of human adenoviruses (HAdV)
infections in children and adults. All the following are true EXCEPT
A HAdVs cause 5-10% of all childhood respiratory disease

B primary infections in infants may manifest as bronchiolitis or pneumonia
C HAdV pneumonia may manifest as features more typical of bacterial disease
D pharyngitis typically includes symptoms of coryza, sore throat, and fever
E HAdV can be identified in <5% of children with isolated pharyngitis
ANSWER:E
2239
Rotaviruses are in the Reoviridae family and cause disease in virtually all mammals and
birds. All the following statements are true EXCEPT
A infection typically begins after an incubation period of <48 hr (range: 1-7 days)
B fever, vomiting and frequent watery stools are present in about 50-60% of cases
C vomiting and fever typically abate after the 4th day of illness
D dehydration may develop and progress rapidly particularly in infants
E most severe disease typically occurs among children 4-36 mo of age
ANSWER:C
2240 Common adverse effects of nitazoxinide include
A diarrhea
B flatulence
C increased appetite
D fever
E pruritus
ANSWER:A
2241 One of the following statements regarding tinidazole is TRUE
A it is FDA approved for treatment of trichomoniasis
B it can be in infancy
C it should be given in 2 divided doses
D it is excreted via urine only
E it carries a pregnancy category A
ANSWER:A
2242 All the following can cause human amoebic meningoencephalitis EXCEPT
A Naegleria
B Acanthamoeba
C Balamuthia
D Entamoeba histolytica
E Sappinia
ANSWER:D
2243 One of the following statements regarding Giardia lamblia is TRUE

A life cycle is composed of 3 stages
B each ingested cyst produces 2 trophozoites in the caecum
C trophozoites contain 4 oval nuclei anteriorly
D
cyst viability is not affected by the usual concentrations of chlorine used to purify water for
drinking
E Giardia genotypes difference lead to wide spectrum of clinical manifestations
ANSWER:D
2244 All the following statements regarding giardiasis are true EXCEPT
A incubation period of Giardia infection usually is 1-2 wk
B most infections are asymptomatic
C symptomatic infections occur more frequently in adults than in children
D stools do not contain blood, mucus, or fecal leukocytes
E malabsorption of sugars, fats, and fat-soluble vitamins is well documented
ANSWER:C
2245 Giardiasis should be considered in children who have the following presentations EXCEPT
A acute dysenteric diarrhea
B persistent diarrhea
C failure to thrive
D malabsorption
E chronic crampy abdominal pain
ANSWER:A
2246
Pregnantmotherseekyouradviceaboutthestoolexaminationofher asymptomatic 15 -month-old
child which contain giardia cyst, your advice should be
A reassurance, no treatment
B inidazole 50 mg\kg once
C nitazoxinide 200 mg bid for 3 days
D metronidazole15 mg/kg/day in 3 divided doses for 5-7 days
E albendazole 400 mg once a day for 5 days
ANSWER:D
2247 The largest protozoan that parasitizes human is
A Entamoeba histolytica
B Giardia lamblia
C Balantidium coli
D Isospora belli

E Cryptosporidium
ANSWER:C
2248 The leading protozoal cause of diarrhea in children worldwide is
A Entamoeba histolytica
B Giardia lamblia
C Balantidium coli
D Isospora belli
E Cryptosporidium
ANSWER:E
2249 The MOST common cause of death in kala azar is
A severe anemia
B bleeding
C secondary bacterial infection
D hepatic failure
E renal failure
ANSWER:C
2250
Hand-foot-mouth disease is one of the more distinctive rash syndromes; it is MOST
frequently caused by
A coxsackievirus A6
B coxsackievirus A16
C enterovirus 71
D coxsackievirus B2
E coxsackievirus B12
ANSWER:B
2251 All the following are causes oropharyngeal dysphagia EXCEPT
A cerebral palsy
B esophagitis
C brain tumors
D cerebrovascular accidents
E hyperthyroidism
ANSWER:B
2252 All the following are common causes of emesis in childhood EXCEPT
A gastroenteritis
B toxic ingestion
C pertussis syndrome

D peptic ulcer
E otitis media
ANSWER:D
2253 Secretory diarrhea can be caused by
A neuroblastoma
B laxative abuse
C lactase deficiency
D irritable bowel syndrome
E thyrotoxicosis
ANSWER:A
2254 All the following metabolic disorders can cause constipation EXCEPT
A hypercalcemia
B hyperkalemia
C hypothyroidism
D diabetes mellitus
E diabetes insipidus
ANSWER:B
2255 All the following suggest a potentially serious organic etiology of abdominal pain EXCEPT
A age >5 yr
B fever
C flank pain
D awakening from sleep
E referred pain to shoulder
ANSWER:A
2256 All the following are common causes of gastrointestinal bleeding in infancy EXCEPT
A bacterial enteritis
B intussusception
C Meckel diverticulum
D anal fissure
E lymphonodular hyperplasia
ANSWER:C
2257 Delayed eruption of the primary teeth can be due to the following EXCEPT
A familial
B hypopituitarism

C hyperthyroidism
D cleidocranial dysplasia
E trisomy 21
ANSWER:C
2258 All the following are features of Stickler syndrome EXCEPT
A prominent joints
B autosomal recessive inheritance
C hypotonia
D mitral valve prolapse
E retinal detachment
ANSWER:B
2259 Periodontitis is often associated with the following conditions EXCEPT
A leukocyte adhesion defects
B hypophosphatasia
C leukemia
D vitamin D resistant rickets
E histiocytosis X
ANSWER:D
2260 Aphthous-like lesions may be associated with the following conditions EXCEPT
A inflammatory bowel disease
B Behcet disease
C gluten-sensitive enteropathy
D SWEET syndrome
E herpetic gingivostomatitis
ANSWER:E
2261 Eruption cyst over the erupting tooth is characterized by the following EXCEPT
A smooth
B painful
C blue or blue-black
D no treatment is indicated
E resolves after eruption of the tooth
ANSWER:B
2262
All the following criteria must be met for the consensus definition of cyclical vomiting
syndrome EXCEPT
A at least 5 attacks in any interval

B
recurrent episodes of intense vomiting and nausea lasting 1 hr to 10 days and occurring at
least 1 wk apart
C vomiting during episodes occurs ≥4 times/hr for ≥1 hr
D return to baseline health between episodes
E usually attributed to another disorder
ANSWER:E
2263 One of the following cause osmotic diarrhea
A toxigenic Escherichiacoli
B carcinoid
C neuroblastoma
D glucose-galactose malabsorption
E congenital chloride diarrhea
ANSWER:D
2264 Acute diarrhea in infancy is commonly caused by
A primary disaccharidase deficiency
B overfeeding
C Hirschsprung toxic colitis
D adrenogenital syndrome
E neonatal opiate withdrawal
ANSWER:B
2265 Advanced dentition for age and sex is seen in
A nutritional disturbances
B hypopituitarism
C cleidocranial dysplasia
D hyperthyroidism
E trisomy 21
ANSWER:D
2266 Mottled enamel (permanent teeth) may be seen in
A renal failure
B immunosuppression
C neutrophil chemotactic deficiency
D maternal infections
E uncontrolled juvenile diabetes
ANSWER:A
2267 Pierre Robin syndrome consists of the following EXCEPT

A micrognathia
B high arched palate
C cleft palate
D small tongue
E foreshortened floor of the mouth
ANSWER:D
2268 The following are features of achalasia EXCEPT
A dysphagia for solids and liquids
B may be accompanied by undernutrition
C may be accompanied by chronic cough
D may be misdiagnosed as asthma
E usually diagnosed before school age
ANSWER:E
2269
Contrast (usually barium) radiographic study of the esophagus and upper gastrointestinal
tract has poor sensitivity and specificity in the diagnosis of
A achalasia
B esophageal strictures
C GERD
D hiatal hernia
E gastric outlet obstruction
ANSWER:C
2270
Gastroesophageal reflux disease (GERD) is the most common esophageal disorder in
children of all ages. Of the following, the MOST common symptom in infants is
A excessive crying
B regurgitation
C failure to thrive
D abnormal posturing
E sleeping disturbances
ANSWER:B
2271
The following procedures satisfactorily improve gastroesophageal reflux disease (GERD)
symptoms in infants EXCEPT
A modified feeding volumes
B hydrolyzed infant formulas
C prone position, when the infant is awake and observed
D avoidance of smoke exposure

E seated position
ANSWER:E
2272
Erosive esophagitis is found in approximately 12% of children with gastroesophageal reflux
disease (GERD) symptoms and is more common in the following EXCEPT
A girls
B older children
C neurologically abnormal children
D children with severe chronic respiratory disease
E children with hiatal hernia
ANSWER:A
2273 Hypertrophic pyloric stenosis has been associated with the following EXCEPT
A eosinophilic gastroenteritis
B Apert syndrome
C Zellweger syndrome
D trisomy 21
E Cornelia de Lange syndrome
ANSWER:D
2274
Oral and intravenous atropine sulfate (pyloric muscle relaxant) has been described when
surgical treatment is not available for hypertrophic pyloric stenosis with a success rate of
A 20%
B 40%
C 60%
D 80%
E 90%
ANSWER:D
2275
Meckel diverticulum has been conveniently referred to by the “rule of 2s,” which explains
the classic presentation of this congenital anomaly. One of the following is FALSE in this rule
A found in approximately 2% of the general population
B usually located 2 feet proximal to the ileocecal valve
C approximately 2 inches in length
D contain 2 types of ectopic tissue (pancreatic or gastric)
E found twice as commonly in males
ANSWER:E

2276
Confirmation of a Meckel diverticulum can be difficult. Of the following, the MOST
sensitive study is
A plain abdominal radiographs
B barium studies
C radionuclide technetium-99m scans
D abdominal ultrasound
E abdominal CT scan
ANSWER:C
2277 Hirschsprung disease has been seen in association with the following EXCEPT
A microcephaly
B mental retardation
C autism
D cleft lip
E hydrocephalus
ANSWER:D
2278 In healthy full-term infants, meconium is passed within 48 hr of birth in
A 99%
B 90%
C 80%
D 70%
E 60%
ANSWER:A
2279
Ileus is the failure of intestinal peristalsis caused by loss of coordinated gut motility without
evidence of mechanical obstruction. In children, ileus accompanies the following metabolic
abnormalities EXCEPT
A uremia
B hypokalemia
C hypocalcemia
D hypermagnesemia
E acidosis
ANSWER:C
2280
The classic triad of intussusception, (pain, a palpable sausage-shaped abdominal mass, and
bloody or currant jelly stool) is seen in
A ≤20% of patients
B ≤30% of patients

C ≤40% of patients
D ≤50% of patients
E ≤60% of patients
ANSWER:B
2281 Regarding foreign bodies in the stomach and intestine, all the following are true EXCEPT
A coins are the most commonly ingested foreign body
B perforation is estimated to be <1% of all objects ingested.
C most objects pass though the intestine in 4-6 days
D cathartics should be avoided
E ingestion of batteries usually leads to problems
ANSWER:E
2282
A bezoar is an accumulation of exogenous matter in the stomach or intestine. Regarding
bezoar, all the following are true EXCEPT
A trichobezoars are composed of the patient’s own hair
B
lactobezoars can be attributed to the high casein or calcium content of some premature
formulas.
C phytobezoars are composed of a combination of plant and animal material
D lactobezoars usually resolve when feedings are withheld for 24-48 hr
E sunflower seed bezoars are reported to cause small bowel obstruction
ANSWER:E
2283
The usual presenting symptoms of peptic ulcer disease in infants and younger children are
the following EXCEPT
A feeding difficulty
B vomiting
C epigastric pain
D crying episodes
E hematemesis
ANSWER:C
2284
H. pylori are among the most common bacterial infections in humans. All the following are
true EXCEPT
A H. pylori is a Gram-negative, S-shaped rod
B H. pylori infection can manifest with abdominal pain or vomiting
C H. pylori is classified by the World Health Organization as a group I carcinogen
D If H. pylori is identified in a child with no symptoms, no therapy should be offered
E H.pyloricanbeassociated,thoughrarely,withchronicautoimmune thrombocytopenia

ANSWER:D
2285 One of the following features is MORE common in ulcerative colitis than in Crohn disease
A abdominal pain
B growth failure
C rectal bleeding
D mouth ulceration
E strictures
ANSWER:C
2286
A neonate is presented with watery and voluminous stool which is mistaken for urine. Of the
following, the MOST likely cause is
A congenital glucose-galactose malabsorption
B microvillus inclusion disease
C congenital bile acid malabsorption
D congenital enterokinase deficiency
E congenital trypsinogen deficiency
ANSWER:B
2287 The MOST common extraintestinal manifestation of celiac disease is
A osteoporosis
B short stature
C arthritis
D iron-deficiency anemia
E aphthous stomatitis
ANSWER:D
2288 The following neurologic manifestations are seen in celiac disease EXCEPT
A myopathy
B epilepsy
C irritability
D cerebral calcifications
E cerebellar ataxia
ANSWER:A
2289 The following conditions can be associated with celiac disease EXCEPT
A Williams syndrome
B Down syndrome
C Turner syndrome
D Klinefilter syndrome

E Sjogren syndrome
ANSWER:D
2290
Abetalipoproteinemia is a rare autosomal recessive disorder of lipoprotein metabolism
(Bassen-Kornzweig syndrome). It is characterized by the following EXCEPT
A presence of acanthocytes in the peripheral blood smear
B extremely low plasma levels of cholesterol
C very low triglycerides level
D absent deep tendon reflexes
E normal intellectual development
ANSWER:E
2291
The following infections may cause malabsorption in immunocompromised children
EXCEPT
A Shigella
B Salmonella
C enteropathogenic E. coli
D Giardia
E rotavirus
ANSWER:C
2292
In infants and very young toddlers chronic diarrhea can appear following infectious enteritis.
The pathogenesis of the diarrhea is not always clear and may be related to
A food protein allergy
B bacterial overgrowth
C giardiasis
D Strongyloides stercoralis
E eosinophilic gastroenteropathy
ANSWER:A
2293 Vitamin B12 and bile salts are only absorbed in
A distal ileum
B duodenum
C proximal jejunum
D colon
E proximal ileum
ANSWER:A
2294 Management of the diarrhea in chronically malnourished children is based on
A intravenous therapy

B standard osmolarity oral rehydration solutions
C reduced osmolarity oral rehydration solutions
D slow resumption of feeds
E nothing by mouth for 24 hour
ANSWER:C
2295 The MOST common congenital disorder associated with exocrine pancreatic insufficiency is
A Shwachman- Diamond syndrome
B Johanson-Blizzard syndrome
C Pearson bone marrow syndrome
D isolated pancreatic enzyme deficiency
E cystic fibrosis
ANSWER:E
2296 In the blue diaper syndrome, symptoms can include all the following EXCEPT
A vomiting
B diarrhea
C failure to thrive
D nephrocalcinosis
E ocular abnormalities
ANSWER:B
2297 Features of Tangier disease include the following EXCEPT
A orange tonsils
B hepatosplenomegaly
C relapsing neuropathy
D orange-brown spots on the colon and ileum
E increased plasma cholesterol
ANSWER:E
2298
Newborns with congenital chloride diarrhea present with severe life-threatening secretory
diarrhea during the 1st few wk of life. Of the following, the MOST common laboratory
finding is
A metabolic alkalosis
B hyperchloremia
C hyperkalemia
D hypernatremia
E hypercalcemia

ANSWER:A
2299 Acrodermatitis enteropathica requires long term treatment with elemental zinc
A 0.5 mg/kg/day
B 1 mg/kg/day
C 1.5mg/kg/day
D 2 mg/kg/day
E 2.5 mg/kg/day
ANSWER:B
2300 Direct person-to-person contact outbreaks of gastroenteritis are usually caused by
A Shigella
B Salmonella
C Rotavirus
D Giardia
E Clostridium difficile
ANSWER:A
2301 Guillain-Barré syndrome can be a sequel in the following foodborne bacterial illness
A Bacillus anthracis
B Bacillus cereus
C Brucella abortus
D Campylobacter jejuni
E Clostridium botulinum
ANSWER:D
2302
Antibiotics are not indicated and may be detrimental in the following foodborne bacterial
illness
A Enterotoxigenic E. coli
B Enterohemorrhagic Escherichia coli O157:H7
C Campylobacter jejuni
D Brucella abortus
E Bacillus anthracis
ANSWER:B
2303 The first-line treatment for children with Vibrio cholera is
A azithromycin
B ciprofloxacin
C doxycycline
D ampicillin

E TMP-SMX
ANSWER:A
2304
Appendicitis-like symptoms (diarrhea, vomiting, fever, and abdominal pain) may be seen in
the following foodborne bacterial illness
A Vibrio vulnificus
B Yersinia enterocolitica
C Shigella spp.
D Staphylococcus aureus (preformed enterotoxin)
E Listeria monocytogenes
ANSWER:B
2305
55. Risks associated with severe dehydration that might necessitate intravenous resuscitation
include
A age < 9 mo
B postmaturity
C fever >38°C if 3-36 mo of age
D secretary diarrhea
E depressed level of consciousness
ANSWER:E
2306
The low-osmolality World Health Organization (WHO) oral rehydration solution (ORS) has
the following per liter EXCEPT
A 75 mEq of sodium
B 50 mEq of chloride
C 20 mEq of potassium
D 75 mmol of glucose
E total osmolarity of 245 mOsm
ANSWER:B
2307
Immunoglobulin A (IgA) nephropathy is an extraintestinal manifestation of the following
enteric infection
A Salmonella
B Shigella
C Yersinia
D Campylobacter
E Cryptosporidium
ANSWER:D

2308
Continued enteral feeding in diarrhea aids in recovery from the episode, but the following
should be avoided
A rice
B potatoes
C lean meats
D fruits
E juices
ANSWER:E
2309
Zinc supplementation in children with diarrhea in developing countries leads to the
following EXCEPT
A decreased use of ORS
B reduced duration of diarrhea
C reduced severity of diarrhea
D reduced hospital admission
E improving diarrhea recovery rates
ANSWER:A
2310
Ondansetron is an effective and less-toxic antiemetic agent and can be given as a useful
adjunct to the treatment of vomiting in ambulatory settings, it is usually given
A intravenously
B intramuscularly
C subcutaneously
D sublingually
E orally
ANSWER:D
2311
Nitazoxanide is an anti-infective agent effective in the treatment of a wide variety of
pathogens including the following EXCEPT
A Giardia lamblia
B E. histolytica
C Shigella
D C. difficile
E Rotavirus
ANSWER:C
2312 Ciprofloxacin is the drug of choice for the treatment of
A Salmonella
B Shigella

C Campylobacter jejuni
D Clostridium difficile
E Cyclospora spp.
ANSWER:B
2313 Increased fecal calprotectin concentration is seen in
A fat malabsorption
B carbohydrate malabsorption
C intestinal inflammation
D pancreatic function
E reduced intestinal surface
ANSWER:C
2314 The following are alarm symptoms of functional abdominal pain EXCEPT
A persistent left lower quadrant pain
B unexplained fever
C genitourinary tract symptoms
D dysphagia
E nocturnal diarrhea
ANSWER:A
2315
A variety of laboratory tests have been used in the evaluation of children with suspected
appendicitis. Of the following, the MOST sensitive and specific one is
A leukocyte count
B urinalysis
C electrolytes
D C-reactive protein
E serum amyloid A protein
ANSWER:E
2316
Plain abdominal radiographs may be helpful in selected cases of abdominal pain/suspected
appendicitis. Plain abdominal x-rays can demonstrate the following findings in acute
appendicitis EXCEPT
A sentinel loops of bowel and localized ileus
B scoliosis from psoas muscle spasm
C a colonic air–fluid level above the right iliac fossa
D a RLQ soft-tissue mass
E a calcified appendicolith (50% of cases)
ANSWER:D

2317 Findings that suggest advanced appendicitis on ultrasound include the following EXCEPT
A asymmetric wall thickening
B increased local tenderness to compression
C abscess formation
D associated free intraperitoneal fluid
E surrounding tissue edema
ANSWER:B
2318 The clinical presentation of one of the following closely mimics appendicitis
A Meckel diverticulitis
B mesenteric adenitis
C sickle cell disease
D pancreatitis
E ovarian torsion
ANSWER:A
2319
There are many anomalies associated with anorectal malformations. Of the following, the
MOST common are
A kidney anomalies
B cardiac anomalies
C esophageal atresia
D spina bifida
E tethered cord
ANSWER:A
2320 The risk of colon cancer is 100% in the following condition
A Peutz-Jeghers syndrome
B Cowden syndrome
C Gardner syndrome
D Juvenile polyposis
E Bannayan-Riley- Ruvalcaba syndrome
ANSWER:C
2321
Hamartomatous polyps, which represent the most common intestinal tumors of childhood
have the following features EXCEPT
A occurring in 1-2% of children
B present in the 1st decade
C most often at ages 2-5 yr
D most commonly in the colon

E often multiple
ANSWER:E
2322
Inguinal hernias are one of the most common conditions seen in pediatric practice and the
most common surgical procedure performed in pediatric surgical practice. Regarding the
incidence of congenital indirect inguinal hernia, all the following are true EXCEPT
A approaches 30% in very-low birth weight infants
B more common in boys
C approximately 60% of inguinal hernias occur on the right side
D bilateral hernias is higher in boys
E an increased incidence in twins
ANSWER:D
2323 The features of the Johanson-Blizzard syndrome include the following EXCEPT
A exocrine pancreatic deficiency
B hypoplasia of the alae nasi
C congenital deafness
D hyperthyroidism
E ectodermal scalp defects
ANSWER:D
2324 The test of choice for acute pancreatitis is
A serum amylase
B serum lipase
C hyperglycemia
D hypocalcemia
E elevated γ-glutamyl transpeptidase
ANSWER:B
2325 The first choice for diagnosis of pancreatic pseudocysts is
A transabdominal ultrasonography
B CT scanning
C magnetic resonance cholangiopancreatography
D endoscopic retrograde cholangiopancreatography
E endoscopic ultrasound
ANSWER:A
2326
Stippled calcifications of the patellas and greater trochanter are found in the following
intrahepatic cholestasis
A idiopathic neonatal hepatitis

B Aagenaes syndrome
C Zellweger (cerebrohepatorenal) syndrome
D neonatal iron storage disease
E Byler disease
ANSWER:C
2327 One of the following rules out biliary atresia
A history of prematurity
B normal size of liver
C normal consistency of liver
D consistently pigmented stools
E no familial incidence
ANSWER:D
2328
The success rate for establishing good bile flow after the Kasai operation is much higher
(90%) if performed before
A 4 wk of life
B 6 wk of life
C 8 wk of life
D 10 wk of life
E 12 wk of life
ANSWER:C
2329
For patients with advanced liver disease, hepatic transplantation has a success rate >85%. If
the operation is technically feasible, it will prolong life and might correct the metabolic error
in diseases such as
A tyrosinemia
B galactosemia
C hereditary fructose intolerance
D Zellweger syndrome
E Crigler-Najjar
ANSWER:A
2330
Persistence of unconjugated hyperbilirubinemia at levels >20 mg/dL after the 1st wk of life
in the absence of hemolysis should suggest
A galactosemia
B tyrosinemia
C α1-Antitrypsin deficiency
D Crigler-Najjar type I

E Maple serum urine disease
ANSWER:D
2331
Liver histology demonstrates normal architecture, but hepatocytes contain black pigment
similar to melanin in
A Dubin-Johnson syndrome
B Rotor syndrome
C Gilbert syndrome
D Crigler-Najjar syndrome
E Zellweger syndrome
ANSWER:A
2332
Kayser-Fleischer (K-F) ring is a brown discoloration at the outer margin of the cornea; the
following are true EXCEPT
A might not be present in younger children
B requires a slit-lamp examination
C is permenant even after trearment
D is present in 95% of patients with neurologic symptoms
E is deposition of copper in Descemet’s membrane
ANSWER:C
2333
A major attempt in Wilson disease should be made to restrict dietary copper intake to <1
mg/day. Of the following, the food that should be avoided is
A liver
B meat
C egg
D chicken
E fish
ANSWER:A
2334
Neonatal iron storage disease (NISD), also known as neonatal hemochromatosis is a rapidly
fatal, progressive illness characterized by the following EXCEPT
A hypoprothrombinemia
B hypoalbuminemia
C hyperferritinemia
D hyperbilirubinemia
E hyperglycemia
ANSWER:E
2335 No parenteral transmission occur in the following hepatotropic viruses

A HAV
B HBV
C HCV
D HDV
E HEV
ANSWER:E
2336 HBV is present in high concentrations in
A serous exudates
B saliva
C vaginal fluid
D semen
E urine
ANSWER:A
2337
Risk factors for HBV infection in children and adolescents include acquisition by the
following EXCEPT
A intravenous drug
B contaminated needles
C sexual contact
D intimate contact with carriers
E sharing toys
ANSWER:E
2338 Regarding epidemiology of HBV, the following are true EXCEPT
A intrauterine infection occurs in 2.5%
B antigenemia appear 1-3 mo after birth
C HBsAg is inconsistently recovered in human milk of infected mothers
D breastfeeding of nonimmunized infants by infected mothers confer a greater risk of hepatitis
E HBV has 8 genotypes (A-H)
ANSWER:D
2339
Routine screening for HBV infection requires assay of multiple serologic markers, all the
following are true EXCEPT
A HBsAg is the first serologic marker of infection to appear
B Anti-HBc is a valuable serologic marker of acute HBV infection
C anti-HBs and anti- HBc are detected in persons immunized with hepatitis B vaccine
D HBeAg is a marker of infectivity

E HBsAg levels fall before symptoms wane
ANSWER:C
2340 HBV is usually spread by
A breastfeeding
B kissing
C hugging
D sharing utensils
E sharing needles
ANSWER:E
2341
The MOST common hepatobiliary disease associated with inflammatory bowel disease (IBD)
is
A hepatic abscess
B portal vein thrombosis
C sclerosing cholangitis
D autoimmune hepatitis
E biliary carcinoma
ANSWER:C
2342
Total parenteral nutrition (TPN) can cause a variety of liver diseases. Of the following, the
MOST severe complication is
A hepatic steatosis
B gallbladder damage
C bile duct damage
D cholestasis
E sclerosing cholangitis
ANSWER:D
2343
Autoimmune hepatitis is a clinical diagnosis based on certain diagnostic criteria; no single
test will make this diagnosis. Important positive features include the following EXCEPT
A female gender
B primary elevation in alkaline phosphatase
C elevated γ-globulin levels
D presence of autoantibodies
E characteristic histologic findings
ANSWER:B
2344
The gallbladder is congenitally absent in approximately 0.1% of the population. Hypoplasia
or absence of the gallbladder can be associated with

A biliary dyskinesia
B cystic fibrosis
C cirrhosis
D cholestasis
E Wilson disease
ANSWER:B
2345
Biliary sludge or cholelithiasis can be detected in >40% of children who are treated with
prolonged course of
A cefotaxime
B meropenem
C vancomycin
D erythromycin
E ceftriaxone
ANSWER:E
2346 The MOST common presentation of portal hypertension is
A jaundice
B splenomegaly
C esophageal hemorrhage
D ascites
E dilated cutaneous collateral vessels in the periumbilical region
ANSWER:C
2347 The MOST common indication for liver transplantation in children is
A Alagille syndrome
B congenital hepatic fibrosis
C hepatocellular carcinoma
D biliary atresia
E Wilson disease
ANSWER:D
2348
Ascites is the pathologic accumulation of fluid within the peritoneal cavity. Of the
following, the MOST common cause in children is
A neoplasia
B hepatic
C cardiac
D trauma
E infection

ANSWER:B
2349
Primary peritonitis usually refers to bacterial infection of the peritoneal cavity without a
demonstrable intraabdominal source. Of the following, the MOST common isolated bacteria
are
A group A streptococci
B Staphylococci
C Escherichia coli
D Mycobacterium tuberculosis
E Pneumococci
ANSWER:E
2350
100.Acute secondary peritonitis most often results from entry of enteric bacteria into the
peritoneal cavity through a necrotic defect in the wall of the intestines or other viscus. It
MOST commonly follows
A perforation of the appendix
B rupture of a Meckel diverticulum
C intussusception
D peptic ulceration
E necrotizing enterocolitis
ANSWER:A
2351
Grunting is produced by expiration against a partially closed glottis and is an attempt to
maintain positive airway pressure during expiration for as long as possible. It is MOST
commonly prominent in
A epiglottitis
B hyaline membrane disease
C asthma
D croup
E choanal atresia
ANSWER:B
2352
The pulmonary vascular resistance and the pulmonary artery pressure to the systemic arterial
resistance, from 3 mo after birth through childhood and adolescence is approximately
A 15% of the systemic values
B 25% of the systemic values
C 35% of the systemic values
D 45% of the systemic values

E 55% of the systemic values
ANSWER:A
2353 Chest CT is of a particular importance in the evaluation of the following lesions EXCEPT
A mediastinal lesions
B cystic parenchymal lesions
C pulmonary embolisms
D vascular rings
E bronchiectasis
ANSWER:D
2354 Rigid bronchoscopy is preferentially indicated for
A persistent pneumonia
B extracting foreign bodies
C atelectasis
D persistent wheeze
E interstitial disease
ANSWER:B
2355 The following infections can cause sudden death in infants EXCEPT
A bronchiolitis (respiratory syncytial virus)
B infant botulism
C pertussis
D Staph. pneumonia
E sepsis
ANSWER:D
2356
Declines of 50% or more in rates of sudden infant death syndrome (SIDS0 in the USA and
around the world have occurred in the past decade. The reductions in risk appear to be
related primarily to
A decreases in placing infants prone for sleep
B exclusive breast-feeding
C adequate immunizations
D avoidance of soft sleeping surface
E avoidance of bed sharing with parent(s)
ANSWER:A
2357
Foreign bodies are often placed in the nose by small children and developmentally delayed
children. Of the following, the MOST common presenting clinical symptom is
A mucopurulent nasal discharge

B foul nasal odor
C epistaxis
D nasal obstruction
E mouth breathing
ANSWER:A
2358 Common causes of childhood epistaxis include the following EXCEPT
A epistaxis digitorum
B foreign bodies
C significant gastroesophageal reflux into the nose
D dry air
E allergic rhinitis
ANSWER:C
2359 Of the following, the MOST common childhood cause of nasal polyposis is
A chronic sinusitis
B allergic rhinitis
C Samter triad
D Kartagener syndrome
E cystic fibrosis
ANSWER:E
2360 The following may be effective as a common cold treatment EXCEPT
A vitamin C
B first generation antihistamine
C topical or oral adrenergic agents
D ipratropium bromide
E honey
ANSWER:A
2361
Lemierre syndrome is characterized by septic thrombophlebitis of the internal jugular veins
with septic pulmonary emboli, producing hypoxia and pulmonary infiltrates, it is a
complication of pharyngitis caused by
A group A β-hemolytic streptococcus
B Arcanobacterium haemolyticum
C Mycoplasma pneumoniae
D Fusobacterium necrophorum
E Corynebacterium diphtheriae
ANSWER:D

2362
Persistent cough may need to be sought beyond the lungs, because cough receptors also
reside in the following regions EXCEPT
A pharynx
B paranasal sinuses
C stomach
D nose
E external auditory canal
ANSWER:D
2363
Habit cough (“psychogenic cough” or “cough tic”) must be considered in any child with a
cough that has lasted for weeks or months, that has been refractory to treatment and is
A abrupt
B loud
C barking quality
D all day, never during sleep
E paroxysmal
ANSWER:D
2364 A “silent chest” with infiltrates should arouse suspicion of all the following EXCEPT
A alveolar proteinosis
B bronchopulmonary dysplasia
C Pneumocystis jiroveci infection
D interstitial pneumonitis
E tumors
ANSWER:B
2365 The barking cough typical of croup is rare in
A laryngotracheobronchitis
B acute epiglottitis
C acute infectious laryngitis
D spasmodic croup
E measles croup
ANSWER:B
2366 Children with croup should be hospitalized for any of the following EXCEPT
A progressive stridor
B severe stridor at rest
C cyanosis
D depressed mental status

E congenital heart disease
ANSWER:E
2367 Bacterial tracheitis is characterized by all the following EXCEPT
A Staphylococcus aureus is the most commonly isolated pathogen
B incidence and severity do not differ by sex
C mean age is between 5 and 7 yr
D considered a primary bacterial illness
E more common than epiglottitis in vaccinated populations
ANSWER:D
2368 The MOST common congenital laryngeal anomaly that produces stridor is
A vocal cord paralysis
B laryngomalacia
C congenital subglottic stenosis
D congenital laryngeal web
E congenital subglottic hemangioma
ANSWER:B
2369 Risk factors for persistent wheezing include all the following EXCEPT
A parental history of asthma
B paternal smoking
C persistent rhinitis (apart from acute upper respiratory tract infections)
D eczema at <1 yr of age
E frequent episodes of wheezing during infancy
ANSWER:B
2370 Cardiovascular causes of wheezing include
A right atrial enlargement
B left atrial enlargement
C right ventricular enlargement
D left ventricular enlargement
E coarctation of aorta
ANSWER:B
2371 Regarding congenital lobar emphysema (CLE), the following are true EXCEPT
A familial occurrence has been reported
B usually no cause of CLE can be identified
C usually become apparent in the neonatal period
D many cases are diagnosed by antenatal ultrasonography

E left upper lobe is the most affected site
ANSWER:B
2372
Bronchiolitis obliterans (BO) is a rare chronic obstructive lung disease of the bronchioles and
smaller airways. An insult to the lower respiratory tract occurs, resulting in fibrosis of the
small airways. Of the following, the BEST mean of establishing its diagnosis is
A chest radiographs
B pulmonary function tests
C ventilation-perfusion scans
D chest CT
E open lung biopsy
ANSWER:E
2373
Pulmonary edema is traditionally separated into two categories according to cause
(cardiogenic and non-cardiogenic). Non-cardiogenic pulmonary edema, in its most severe
state, is also known as acute respiratory distress syndrome (ARDS). The following
radiographic features is usually present in non-cardiogenic pulmonary edema
A septal lines
B peribronchial cuffing
C pleural effusions
D air bronchograms
E even or central distribution of edema
ANSWER:D
2374
The recurrent aspiration of small quantities of gastric, nasal, or oral contents can lead to
several clinical presentations, including recurrent bronchitis or bronchiolitis; recurrent
pneumonia; atelectasis; wheezing; cough; apnea; and/or laryngospasm. The MOST common
underlying problem associated with recurrent pneumonias in hospitalized children is
A oropharyngeal incoordination
B esophageal foreign body
C nasoenteric tube
D poor oral hygiene
E bronchopulmonary dysplasia
ANSWER:A
2375 The MOST common etiology of pulmonary infiltrates with eosinophilia (PIE) is
A simple pulmonary eosinophilia
B acute eosinophilic pneumonia

C chronic eosinophilic pneumonia
D allergic bronchopulmonary aspergillosis
E parasitic Infections
ANSWER:E
2376 The MOST frequent pathogens of pneumonia in children 5 yr and older is
A Streptococcus pneumoniae
B Mycoplasma pneumoniae
C group A streptococci
D H. influenzae (type b, nontypable)
E adenovirus
ANSWER:B
2377 The MOST frequent pathogen of pneumonia in children 4 mo-4 yr is
A Streptococcus pneumoniae
B Mycoplasma pneumoniae
C group A streptococci
D H. influenzae (type b, nontypable)
E Respiratory syncytial virus
ANSWER:E
2378 Recurrent pneumonia is defined as
A 2 or more episodes in a single year, with radiographic clearing between occurrences
B 2 or more episodes in a single year, without radiographic clearing between occurrences
C 3 or more episodes in a single year, with radiographic clearing between occurrences
D 3 or more episodes in a single year, without radiographic clearing between occurrences
E 4 or more episodes in a single year
ANSWER:A
2379 The MOST consistent clinical manifestation of pneumonia is
A subcostal retractions
B grunting
C tachypnea
D fever
E cough
ANSWER:C
2380
The following are indications for admission to a hospital in children with pneumonia
EXCEPT
A age <6 mo

B sickle cell anemia
C multiple lobe involvement
D moderate to severe respiratory distress
E vomiting
ANSWER:B
2381 In the developed world, the MOST common cause of clinically significant bronchiectasis is
A primary ciliary dyskinesia
B foreign body aspiration
C aspiration of gastric contents
D immune deficiency syndromes
E cystic fibrosis
ANSWER:E
2382 The MOST common complaint in patients with bronchiectasis is
A cough and production of copious purulent sputum
B hemoptysis
C fever
D anorexia
E poor weight gain
ANSWER:A
2383 Cystic fibrosis (CF) can be presented with all the following EXCEPT
A salt depletion
B pansinusitis
C rectal polyposis
D pancreatitis
E cholelithiasis
ANSWER:C
2384 The earliest pathologic lesion in the lung of a patient with cystic fibrosis is
A bronchiolitis
B bronchitis
C bronchiolectasis
D bronchiectasis
E interstitial lung disease
ANSWER:A
2385 Arterial blood gas study in cystic fibrosis shows

A hypernatraemic hypochloraemic metabolic alkalosis
B hyponatraemic hypochloraemic metabolic acidosis
C hyponatraemic hyperchloraemic metabolic alkalosis
D hyponatraemic hypochloraemic metabolic alkalosis
E hypernatraemic hyperchloraemic metabolic acidosis
ANSWER:D
2386 The MOST common presenting feature of cystic fibrosis is
A failure to thrive
B persistent respiratory symptoms
C abnormal stools
D meconium ileus
E hepatobiliary disease
ANSWER:B
2387 Regarding cystic fibrosis, all the following are true EXCEPT
A delayed sexual development
B more than 95% of males are azoospermic
C sexual function is generally impaired
D high incidence of inguinal hernia
E diminished female fertility rate
ANSWER:C
2388 All the following conditions are associated with false-positive sweat test results EXCEPT
A eczema
B anorexia nervosa
C hypothyroidism
D Klinefelter syndrome
E malnutrition
ANSWER:E
2389
The finding of the following pathogen on culture of the lower airways (sputum) strongly
suggests a diagnosis of cystic fibrosis
A B. cepacia
B gram-negative rods
C nontuberculous mycobacterial species
D Mycoplasma
E S. aureus
ANSWER:E

2390 All the following inhalation therapies in cystic fibrosis are true EXCEPT
A β-agonists may decrease PaO2 acutely
B human recombinant DNase improves pulmonary function
C N-acetylcysteine is toxic to ciliated epithelium
D nebulized hypertonic saline improves mucociliary clearance
E aerosolized antibiotics are often used when the airways are colonized with S. aureus
ANSWER:E
2391 The following are clinical manifestations of primary ciliary dyskinesia (PCD) EXCEPT
A recurrent pneumonia
B chronic otitis media
C chronic pansinusitis
D dextrocardia
E retinitis pigmentosa
ANSWER:D
2392
The following are causes of diffuse alveolar hemorrhage (DAH) syndromes with pulmonary
capillaritis EXCEPT
A Goodpasture syndrome
B Henoch-Schonlein purpura
C Wegener granulomatosis
D Heiner syndrome
E Behcet syndrome
ANSWER:D
2393
Pathologic findings of pulmonary hemosiderosis in association with cow's milk
hypersensitivity include all the following EXCEPT
A elevations of IgA
B peripheral eosinophilia
C alveolar deposits of IgG, IgA
D alveolar deposits of C3
E high titers to cow's milk protein
ANSWER:A
2394
The classic laboratory findings of idiopathic pulmonary hemosiderosis (IPH) are the
following EXCEPT
A microcytic hypochromic anemia
B elevated reticulocyte count
C normal plasma bilirubin

D reduced serum iron
E elevated iron-binding capacity
ANSWER:C
2395 The treatment of choice of idiopathic pulmonary hemosiderosis (IPH) is
A transfusion of blood products
B systemic corticosteroids
C cyclophosphamide
D chloroquine
E lung transplantation
ANSWER:B
2396
A commonly encountered risk factor for deep venous thrombosis (DVT) and pulmonary
embolism (PE) in the pediatric population is
A central venous catheter
B hematologic malignancies
C sickle cell diease
D nephrotic syndrome
E antiphospholipid antibody syndrome
ANSWER:A
2397 The MOST common form of pulmonary malignancy in children is
A bronchial carcinoid
B adenoid cystic carcinoma
C mucoepidermoid carcinoma
D metastatic lesions
E pulmonary blastoma
ANSWER:D
2398
Pleurisy or inflammation of the pleura is often accompanied by an effusion. Of the
following, the MOST common cause of pleural effusion in children is
A tuberculosis
B heart failure
C bacterial pneumonia
D rheumatoid arthritis
E metastatic intrathoracic malignancy
ANSWER:C
2399 The MOST common cause of pneumomediastinum in older children and teenagers is
A dental extractions

B adenotonsillectomy
C tuberculosis
D mycoplasma pneumonia
E acute asthma
ANSWER:E
2400
The following conditions are associated with increased intrathoracic pressure causing
pneumothorax in children EXCEPT
A asthma
B pneumatocele
C bronchiolitis
D cystic fibrosis
E airway foreign body
ANSWER:B
2401 One of the following is a rare cause of hydrothorax in children
A cardiac disease
B renal disease
C hepatic disease
D severe nutritional edema
E vascular obstruction by neoplasms
ANSWER:E
2402 Of the following, the LEAST likely cause of hemothorax in children is
A chest trauma
B intrathoracic neoplasms
C costal exostoses
D blood dyscrasias
E rupture of an aneurysm
ANSWER:E
2403 All the following are true in chylothorax EXCEPT
A fluid may be clear
B fluid triglyceride level is >110 mg/dL
C pleural fluid:serum triglyceride ratio is >1.0
D pleural fluid:serum cholesterol ratio is <1.0
E cells are primarily B lymphocytes
ANSWER:E

2404
Bronchopulmonary dysplasia (BPD) is a syndrome characterized by signs and symptoms of
chronic lung disease that originates in the neonatal period. An accepted definition includes a
postnatal oxygen requirement for
A 7 days
B 14 days
C 21 days
D 28 days
E 35 days
ANSWER:D
2405
55.The MOST common physical finding of the pulmonary exam in bronchopulmonary
dysplasia (BPD) is
A tachypnea
B mouth breathing
C increased anteroposterior diameter of the chest
D intercostal retractions
E clear breath sounds
ANSWER:A
2406
The etiology of wheezing in bronchopulmonary dysplasia (BPD) may be due to all the
following EXCEPT
A lower airway inflammation
B bronchial smooth muscle irritation
C intraluminal exudate
D bronchial smooth muscle hypertrophy
E airway malacia
ANSWER:C
2407 The following are features of pectus excavatum (funnel chest) EXCEPT
A occurs in 1 : 400 births
B usually associated with a connective tissue disorder
C 9 : 1 male preponderance
D accounts for >90% of congenital chest wall anomalies
E positive family history in one third of cases
ANSWER:B
2408 Pectus carinatum is characterized by all the following EXCEPT
A accounting for 5-15% of congenital chest wall anomalies
B females are affected 4 times more often than males

C high familial occurrence
D common association of mild to moderate scoliosis
E association with mitral valve disease
ANSWER:B
2409 Acute respiratory insufficiency is often the MOST prominent clinical manifestation of
A Duchenne muscular dystrophy
B spinal muscular atrophy
C congenital myotonic dystrophy
D myasthenia gravis
E uillain-Barre syndrome
ANSWER:E
2410
Asphyxiating thoracic dystrophy (Jeune syndrome) is associated with the following skeletal
abnormalities EXCEPT
A narrowed thorax
B horizontal ribs
C short extremities
D bell-shaped chest cage
E low clavicles
ANSWER:E
2411
Physical examination and an upright, posteroanterior radiograph with subsequent
measurement of the angle of curvature (Cobb technique) remain the gold standard for
assessment of scoliosis. Scoliosis is defined when curves
A ≥5 degrees
B ≥10 degrees
C ≥15 degrees
D ≥20 degrees
E ≥25 degrees
ANSWER:B
2412 CHARGE association is characterized by the following cardiac feature
A endocardial cushion defect
B supravalvular aortic stenosis
C tetralogy of Fallot
D patent ductus arteriosus
E anomalous pulmonary venous return
ANSWER:C

2413 Hypertrophic cardiomyopathy is a recognized association with
A infant of diabetic mother
B Marfan syndrome
C William syndrome
D trisomy 21 (Down syndrome)
E XO (Turner syndrome)
ANSWER:A
2414 The following are characters of venous hum EXCEPT
A continuous murmur
B heard in infraclavicular region
C grades I-III/VI
D louder with patient in supine position
E changes with compression of jugular vein
ANSWER:D
2415
Chest pain in the pediatric patient often generates a significant amount of patient and parental
concern. Of the following, the MOST common cause is
A myocarditis
B pericarditis
C pleurisy
D pneumothorax
E pneumonia
ANSWER:E
2416
Congenital heart disease causing cyanosis without respiratory distress include the following
EXCEPT
A tricuspid atresia
B aortic stenosis
C Ebstein anomaly
D pulmonary atresia
E tetralogy of Fallot
ANSWER:B
2417 The MOST common cyanotic cardiac lesion to present in the newborn period is
A dextroposed transposition of the great arteries
B hypoplastic left heart syndrome
C tetralogy of Fallot
D truncus arteriosus

E pulmonary atresia
ANSWER:A
2418
The ECG shows left ventricular hypertrophy and a superior QRS axis (between 0° and –90°)
in
A tricuspid atresia
B Ebstein anomaly
C pulmonary atresia
D pulmonary stenosis
E tetralogy of Fallot
ANSWER:A
2419 The MOST common cause of death from cardiac defects in the first month of life is
A d-Transposition without associated lesions
B hypoplastic left heart syndrome
C pulmonary atresia
D truncus arteriosus
E complex single ventricle
ANSWER:B
2420 All the following are causes of heart failure in full-term neonate EXCEPT
A asphyxial cardiomyopathy
B coarctation of aorta
C hypoplastic left heart
D transposition of great arteries
E ventricular septal defect
ANSWER:E
2421 One of the following is a sign of right-sided heart failure
A edema
B tachypnea
C orthopnea
D wheezing
E pulmonary edema
ANSWER:A
2422 The MOST common cause of syncope in children is
A Wolff-Parkinson-White syndrome
B prolonged QT syndrome
C atrioventricular block

D neurocardiogenic syncope
E myocarditis
ANSWER:D
2423
Of the following, the MOST common clinical sign of coarctation of the aorta in older
children is
A cardiac enlargement
B notching of the inferior border of the ribs
C a systolic ejection click or thrill in the suprasternal notch
D differential blood pressure: arms > legs
E diminished or absent femoral or lower-extremity pulses
ANSWER:D
2424
The chest x-ray may help to differentiate the types of congenital heart defects. Increased
pulmonary markings (increased pulmonary blood flow) is seen in
A pulmonary atresia
B truncus arteriosus
C tetralogy of Fallot
D tricuspid atresia
E Ebstein anomaly
ANSWER:B
2425
The following are causes of congestive heart failure and cardiomegaly during the newborn
period but no murmur EXCEPT
A endocardial fibroelastosis
B asphyxia
C glycogen storage disease (Pompe disease)
D paroxysmal supraventricular tachycardia
E sepsis
ANSWER:A
2426 SVT in children differ from physiologic sinus tachycardia by all the following EXCEPT
A sudden onset and termination
B persistent ventricular rate of >180 bpm
C fixed RR interval on ECG
D abnormal P-wave shape or axis or absent P waves
E evident change in heart rate with activity
ANSWER:E
2427 The following cardiac lesions are at increased risk for bacterial endocarditis EXCEPT

A mitral insufficiency
B aortic stenosis
C atrial septal defect seccundum
D coarctation of the aorta
E patent ductus arteriosus
ANSWER:C
2428
The following factors are most strongly associated with the development of coronary artery
disease in patients with Kawasaki disease EXCEPT
A duration of fever of >16 days
B first-degree heart block
C cardiomegaly
D male gender
E age of <1 year
ANSWER:B
2429 The following are major side effects of PGE1 EXCEPT
A apnea
B fever
C cutaneous flushing
D seizures
E hypertension
ANSWER:E
2430 One of the following cardiac lesions have been identified more in male
A transposition of the great arteries
B atrial septal defect
C VSD
D PDA
E pulmonic stenosis
ANSWER:A
2431 In MOST patients with an ASD, the characteristic physical finding is
A a right ventricular systolic lift
B a fixed splitted 2nd heart sound
C a systolic ejection murmur
D a short, rumbling mid-diastolic murmur
E a mild left precordial bulge
ANSWER:B

2432 Indications for surgical closure of a VSD include the following EXCEPT
A patients in whom clinical symptoms and failure to thrive cannot be controlled medically
B
infants between 6 and 12 mo of age with large defects associated with pulmonary
hypertension
C patients older than 24 mo with a Qp : Qs ratio greater than 2 : 1.
D patients with a supracristal VSD of any size
E severe pulmonary vascular disease nonresponsive to pulmonary vasodilators
ANSWER:E
2433 Regarding supracristal ventricular septal defect, the following are true EXCEPT
A a supracristal VSD is complicated by aortic insufficiency in 50-90% of patients
B the incidence is higher in Asian children
C aortic insufficiency is most often not recognized until late in the 1st decade of life
D the murmur of a supracristal VSD is usually heard at the mid to upper left sternal border
E
closure of supracristal ventricular VSDs at the time of diagnosis is not recommended in an
asymptomatic child
ANSWER:E
2434 All the following are signs of coarctation of the aorta EXCEPT
A femoral pulses are weak or absent
B bounding pulses of the arms
C femoral pulse occurs slightly before the radial pulse
D blood pressure in the legs is lower than that in the arms
E precordial impulse and heart sounds are usually normal
ANSWER:C
2435 All the following are causes of chronic pulmonary venous hypertension EXCEPT
A congenital mitral stenosis
B total anomalous pulmonary venous return with obstruction
C peripheral pulmonary stenosis
D left atrial myxomas
E cor triatriatum
ANSWER:C
2436 Regarding mitral valve prolapse; all the following are true EXCEPT
A it is predominantly congenital
B it is usually sporadic
C it is more common in girls
D the dominant abnormal signs are auscultatory

E antibiotic prophylaxis is recommended during surgery and dental procedures
ANSWER:E
2437
Sinus bradycardia is due to slow discharge of impulses from the sinus node, the heart's
natural pacemaker. Sinus bradycardia in neonates is considered if sinus rate is
A <100 beats/min
B <90 beats/min
C <80 beats/min
D <70 beats/min
E <60 beats/min
ANSWER:B
2438 The following procedures may abort an attack of SVT EXCEPT
A placing of the face in ice water
B straining
C breath holding
D standing on head
E ocular pressure
ANSWER:E
2439 The following drugs may cause long Q-T syndromes (LQTS) EXCEPT
A trimethoprim/sulfamethoxazole
B erythromycin
C imipramine
D pentostam
E risperidone
ANSWER:D
2440 The MOST common cause of death in competitive athletes is
A aortic stenosis
B hypertrophic cardiomyopathy
C coronary artery abnormalities
D myocarditis
E long Q-T syndrome (LQTS)
ANSWER:B
2441 The leading causative agents for endocarditis in pediatric patients are
A group D enterococci
B viridans-type streptococci
C Pseudomonas aeruginosa

D fungal organisms
E Serratia marcescens
ANSWER:B
2442 The following is a major Duke criterion for the diagnosis of endocarditis
A new valve regurgitant flow by echocardiography
B Osler nodes
C single positive blood culture
D serologic evidence of infection
E high erythrocyte sedimentation rate
ANSWER:A
2443 The lower limit of pulse rate in neonate at rest is
A 50/min
B 60/min
C 70/min
D 80/min
E 90/min
ANSWER:C
2444
Careful evaluation of the character of the pulses is an important early step in the physical
diagnosis of congenital heart disease. A wide pulse pressure with bounding pulses may
suggest the following EXCEPT
A patent ductus arteriosus
B aortic insufficiency
C cardiomyopathy
D anemia
E anxiety
ANSWER:C
2445 Tall (>2.5 mm), narrow, and spiked P waves are seen in
A Ebstein anomaly
B ventricular septal defect [VSD]
C patent ductus arteriosus [PDA]
D severe mitral stenosis
E mitral regurgitation
ANSWER:A
2446
In utero heart failure, often with fetal pleural and pericardial effusions and generalized
ascites (non-immune hydrops fetalis) may occur in

A ventricular septal defect
B coarctation of aorta
C d-Transposition of great arteries
D Ebstein anomaly
E single ventricle
ANSWER:D
2447 Of the following, the cardiac lesion resulting in increased volume load is
A valvular pulmonic stenosis
B patent ductus arteriosus
C valvular aortic stenosis
D coarctation of the aorta
E mitral stenosis
ANSWER:B
2448 Of the following, the cyanotic cardiac lesion with decreased pulmonary blood flow is
A transposition of the great vessels
B single ventricle
C truncus arteriosus
D tricuspid atresia
E total anomalous pulmonary venous return without obstruction
ANSWER:D
2449
The predominant manifestations of primary pulmonary hypertension include the following
EXCEPT
A exercise intolerance
B precordial chest pain
C syncope
D low arterial oxygen saturation
E dizziness
ANSWER:D
2450 The following are common side effects of sildenafil EXCEPT
A flushing
B elevated liver function tests
C headache
D diarrhea
E myalgia
ANSWER:B

2451 Postoperative (cardiac surgery) pulmonary hypertension can be managed with
A hyperventilation and inhaled nitric oxide
B catecholamines
C phosphodiesterase inhibitors
D nitroprusside
E diuretics
ANSWER:A
2452
Patients who have undergone surgery entailing the use of cardiopulmonary bypass, should
be watched carefully for
A epilepsy
B learning disabilities
C anemia
D pulmonary hypertension
E arrhythmias
ANSWER:B
2453 The postpericardiotomy syndrome is characterized by all the following EXCEPT
A fever
B decreased appetite
C listlessness
D vomiting
E chest pain
ANSWER:E
2454
For one of the following, total repair really achieved, with no requirement for long- term
follow-up
A atrial septal defects
B ventricular septal defects
C valvar pulmonic stenosis
D uncomplicated tetralogy of Fallot
E isolated patent ductus arteriosis
ANSWER:E
2455 Pregnancy is contraindicated in mothers with
A severe pulmonary hypertension
B ventricular septal defects
C prosthetic valves
D atrial septal defects

E unoperated cyanotic congenital heart disease
ANSWER:A
2456 Surgical repair of VSD at the time of diagnosis should be considered in
A small supracristal VSD
B nonrestrictive VSD
C hemodynamically significant VSD
D restrictive VSD
E VSD with heart failure
ANSWER:A
2457
The MOST common long-term complication for those patients who underwent early repair
of complete atrioventricular canal before the development of pulmonary vascular disease is
A left AV valve regurgitation
B subaortic stenosis
C residual ventricular level shunts
D complete heart block
E endocarditis
ANSWER:A
2458 Patients with a small PDA have the following criteria EXCEPT
A continuous murmur heard best at the left upper sternal border
B normal peripheral pulses
C normal pulmonary artery pressure by echocardiography
D asmptomatic and live a normal life expectancy
E risk for endocarditis is extremely low
ANSWER:E
2459 Severe pulmonary valve stenosis defined as a peak gradient of
A >40 mm Hg
B >45 mm Hg
C >50 mm Hg
D >55 mm Hg
E >60 mm Hg
ANSWER:C
2460
Symptoms in patients with aortic stenosis depend on the severity of the obstruction. Critical
aortic stenosisis associated with the following EXCEPT
A cardiomegaly

B pulmonary edema
C weak pulses in all extremities
D pale skin
E maximum intensity of the murmur
ANSWER:E
2461 50.All the following are true regarding the diagnosis of coarctation of the aorta EXCEPT
A cardiac enlargement are noted with severe coarctation
B notching of the ribs is common by late childhood
C doppler is useful for demonstrating the specific site of the obstruction
D
CT and MRI are valuable noninvasive tools for evaluation of coarctation when the
echocardiogram is equivocal
E diagnostic catheterization is mandatory before surgery
ANSWER:E
2462 hone complex consist of the following EXCEPT
A coarctation of the aorta
B subvalvar aortic stenosis
C valvar aortic stenosis
D pulmonic stenosis
E mitral stenosis
ANSWER:D
2463
Paroxysmal hypercyanotic attacks (hypoxic, “blue,” or “tet” spells) are a particular problem
during the 1st 2 yr of life. They are characterized by
A early evening occurrence
B an increase in intensity of the systolic murmur
C unpredictable onset
D metabolic alkalosis
E more freguent spells in patients with marked cyanosis at rest
ANSWER:C
2464
Depending on the frequency and severity of hypercyanotic attacks in tetralogy of Fallot, all
the following procedures are true EXCEPT
A placement of the infant in the knee-chest position
B administration of oxygen
C injection of morphine subcutaneously
D rapid correction with intravenous sodium bicarbonate irrespective of the spell severity
E intravenous administration of propranolol

ANSWER:D
2465
Thetypicalradiologic configuration in tetralogyof Fallotas seeninthe anteroposterior view
consists of all the following EXCEPT
A normal heart size
B some elevation of the cardiac apex
C convexity in the region of the main pulmonary artery
D right-sided aortic arch
E diminished pulmonary vascularity
ANSWER:C
2466
55.The severity of symptoms of Ebstein anomaly and the degree of cyanosis are highly
variable and depend on the extent of
A cardiac dysrhythmias
B displacement of the tricuspid valve
C atrial right-to-left shunt
D pulmonary vascular resistance
E polycythemia
ANSWER:B
2467
D-transposition of the great vessels, a common cyanotic congenital anomaly, accounts for
≈5% of all congenital heart disease. In this anomaly, all the following are true EXCEPT
A the systemic veins return to the right atrium
B the pulmonary veins return to the left atrium
C the connections between the atria and ventricles are normal
D the aorta is posterior and to the right of the pulmonary artery
E the aorta is anterior and to the right of the pulmonary artery
ANSWER:D
2468
Obstructed total anomalous pulmonary venous return (TAPVR) is a pediatric cardiac
surgical emergency because
A prostaglandin therapy is usually not effective
B severe pulmonary congestion develop
C left atrium may be small
D left ventricle may be small
E pulmonary hypertension develop
ANSWER:A
2469
In neonates with total anomalous pulmonary venous return and marked pulmonary venous
obstruction, the chest x-ray demonstrates

A small heart
B enlarged heart
C prominent pulmonary artery
D prominent right ventricle
E subaortic stenosis
ANSWER:A
2470
In truncus arteriosus, a single arterial trunk arises from the heart and supplies the systemic,
pulmonary, and coronary circulations. All the following are true EXCEPT
A VSD is always present
B the truncus overriding the defect
C both ventricles are at systemic pressure
D heart failure ensues immediately after birth
E clinical cyanosis is usually mild
ANSWER:D
2471 Clinical manifestations of hypoplastic left-heart syndrome include
A cyanosis always obvious in the 1st 48 hr of life
B deep-blue color skin
C bounding peripheral pulses
D occasional association with Turner syndrome
E autosomal recessive inheritance is usual
ANSWER:D
2472 Polysplenia (left isomerism) is characterized by
A right-sided stomach
B decreased pulmonary blood flow
C absent gallbladder
D howell-jolly bodies
E severe cyanosis
ANSWER:C
2473 Pentalogy of Cantrell consists of the following EXCEPT
A ectopia cordis
B midline supra umbilical abdominal defect
C deficiency of the anterior diaphragm
D defect of the upper sternum
E intracardiac defect
ANSWER:D

2474 The MOST common cause of pulmonary hypertension in pediatric patients is
A idiopathic pulmonary arterial hypertension
B pulmonary venoocclusive disease
C left-sided valvular heart disease
D chronic obstructive pulmonary disease
E interstitial lung disease
ANSWER:A
2475
All patients with clinical evidence of a PDA are at increased risk for endocarditis. As result,
PDA should be considered for closure EXCEPT
A PDA with severe irreversible pulmonary hypertension
B small, hemodynamically insignificant PDA
C small, hemodynamically significant PDA
D moderate size PDA
E large PDA
ANSWER:A
2476 Phenytoin is used for treatment of
A supraventricular tachycardia
B atrial fibrillation
C atrial flutter
D ventricular tachycardia
E digitalis intoxication
ANSWER:E
2477 Side effects of propranolol may include
A hypothyroidism
B elevated triglycerides
C school performance problems
D hepatic toxicity
E pulmonary fibrosis
ANSWER:C
2478
The following criteria are indications for further investigation of premature ventricular
contractions (PVCs) that could require suppressive therapy EXCEPT
A multiform PVCs
B disappear during exercise
C R-on-T phenomenon
D extreme frequency of beats

E presence of underlying heart disease
ANSWER:B
2479
In urgent situations of supraventricular tachycardia (SVT) when symptoms of severe heart
failure have already occurred. Of the following, the initial management is
A adenosine by rapid intravenous push
B DC cardioversion (0.5-2 J/kg)
C verapamil
D digoxin
E amiodarone
ANSWER:B
2480
Ventricular tachycardia (VT) is defined as at least 3 premature ventricular contractions PVCs
at >120 beats/min. Hemodynamically unstable patients with VT should be immediately
treated with
A catheter ablation
B ICD implantation
C DC cardioversion
D amiodarone
E procainamide
ANSWER:C
2481
In 90% of cases of endocarditis, the causative agent is recovered from the first 2 blood
cultures. Antimicrobial pretreatment of the patient reduces the yield of blood cultures to
A 10%
B 30%
C 40%
D 60%
E 80%
ANSWER:D
2482
Despite the use of antibiotic agents, mortality remains high, in the range of 20-25%. Serious
morbidity occurs in 50-60% of children with documented infective endocarditis. Of the
following, the MOST common morbidity is
A heart failure
B pulmonary emboli
C mycotic aneurysms
D acquired ventricular septal defect
E heart block

ANSWER:A
2483
Anthracycline cardiotoxicity (doxorubicin [Adriamycin]) on rare occasion causes acute
inflammatory myocardial injury, but more classically results in dilated cardiomyopathy
(DCM) and occurs in up to 30% of patients given a cumulative dose of doxorubicin
exceeding
A 250 mg/m2
B 350 mg/m2
C 450 mg/m2
D 550 mg/m2
E 650 mg/m2
ANSWER:D
2484
Restrictive cardiomyopathy (RCM) accounts for <5% of cardiomyopathy cases. RCM is
characterized by all the following EXCEPT
A normal ventricular chamber dimensions
B normal myocardial wall thickness
C preserved systolic function
D normal atrial chamber dimensions
E high ventricular diastolic pressure
ANSWER:D
2485 Diastolic dysfunction and normal systolic function are recognized features of
A dilated cardiomyopathy
B hypertrophic cardiomyopathy
C left ventricular noncompaction
D restrictive cardiomyopathy
E arrhythmogenic right ventricular cardiomyopathy
ANSWER:D
2486 In Asia, the following hepatitis virus appears to be a significant cause of viral myocarditis
A hepatitis A virus
B hepatitis B virus
C hepatitis C virus
D hepatitis D virus
E hepatitis E virus
ANSWER:C
2487
The vast majority of tumors originating from the heart are benign. Of the following, the
MOST common pediatric cardiac tumors are

A fibromas
B rhabdomyomas
C myxomas
D hemangiomas
E papillomas
ANSWER:B
2488 Afterload reducers are not indicated in children with heart failure secondary to
A cardiomyopathy
B severe mitral insufficiency
C aortic insufficiency
D ventricular septal defect
E aortic stenosis
ANSWER:E
2489
Adverse reactions to angiotensin-converting enzyme inhibitors (ACEIs) include the
following EXCEPT
A hypotension
B hypokalemia
C maculopapular pruritic rash
D renal toxicity
E chronic cough
ANSWER:B
2490 The definition of hypertension in children is
A
average systolic blood pressure (SBP) and/or diastolic BP that is ≥95th percentile for age,
sex, on ≥3 occasions
B
average systolic blood pressure (SBP) and/or diastolic BP that is ≥95th percentile for age,
sex, and height on ≥2 occasions
C
average systolic blood pressure (SBP) and/or diastolic BP that is ≥95th percentile for age,
sex, and height on ≥3 occasions
D
average systolic blood pressure (SBP) and/or diastolic BP that is ≥90th percentile for age,
sex, and height on ≥3 occasions
E
average systolic blood pressure (SBP) and/or diastolic BP that is ≥99th percentile for age,
sex, and height on ≥3 occasions
ANSWER:C
2491 Renal childhood diseases responsible for chronic hypertension include
A hemolytic-uremic syndrome

B acute tubular necrosis
C congenital dysplastic kidney
D pyelonephritis
E renal trauma
ANSWER:C
2492
Measuring serum potassium is essential in a child with confirmed hypertension because
hyperkalemia may be seen in
A Gordon syndrome
B Liddle syndrome
C glucocorticoid remedial aldosteronism
D apparent mineralcorticoid excess
E pheochromocytoma
ANSWER:A
2493
The hematologic features of congenital hypoplastic anemia (Diamond-Blackfan Anemia)
include all the following EXCEPT
A normocytic anemia
B reticulocytopenia
C deficiency or absence of red blood cell bone marrow precursors
D elevated fetal hemoglobin (Hb F)
E elevated serum iron levels
ANSWER:A
2494
The MOST specific test that is helpful to differentiate Diamond-Blackfan anemia from
transient erythroblastopenia of childhood is
A hemoglobin electrophoresis
B reticulocytes count
C erythrocyte adenosine deaminase (ADA) enzyme level assay
D bone marrow examination
E mean corpuscular volume (MCV)
ANSWER:C
2495
Anemia and reticulocytopenia that occur in the 2nd half of infancy period is LEAST likely
due to
A congenital hypoplastic anemia (Diamond-Blackfan anemia)
B transient erythroblastopenia of childhood
C a protracted, prolonged course of the anemia of hemolytic disease of the newborn
D aplastic crises complicating various types of chronic hemolytic anemias

E Fanconi anemia
ANSWER:C
2496 The main stay of treatment for congenital hypoplastic anemia is
A corticosteroids
B androgen
C antithymocyte globulin (ATG)
D fully matched-related stem cell transplantation
E fully matched-unrelated stem cell transplantation
ANSWER:A
2497
All the following are true regarding Transient Erythroblastopenia of Childhood (TEC)
EXCEPT
A it is more common than congenital hypoplastic (Diamond-Blackfan) anemia
B most of the affected children are older than 12 mo at onset
C Parvovirus B19 infections is a common causative agent
D virtually all children recover within 1-2 months
E corticosteroid therapy is of no value
ANSWER:C
2498
The valuable diagnostic feature that differentiate anemia of chronic disorder from iron
deficiency anemia is
A low serum iron
B low or normal serum transferrin
C normal bone marrow cellularity
D normochromic normocytic RBC
E leukocytosis
ANSWER:B
2499 All the following are true regarding physiologic anemia of infancy EXCEPT
A it is exaggerated in premature infants
B the hemoglobin concentration range is between 9-11 g/dL
C the hemoglobin concentration rarely falls below 10 g/dL in healthy infants
D vitamin E deficiency does not play a role
E iron deficiency is a contributing factor
ANSWER:E
2500 Folic acid deficiency may develop in the following children EXCEPT
A a term infant fed on goat`s milk
B an infant fed on unfortified powdered milk

C an infant fed on pasteurized cow's milk
D a child with chronic use of phenytoin anticonvulsant
E a child with trimethoprim prophylaxis for recurrent urinary tract infection
ANSWER:C
2501
A 1-year-old child has folic acid deficiency since the age of 4 months, the best indicator of
this deficiency is
A significant increment of mean corpuscular volume MCV
B significant fall of reticulocytes count
C high level of lactate dehydrogenase LDH
D decrease level of RBC folate
E increase number of hypersegmented neutrophils
ANSWER:D
2502 The first laboratory marker in progressive iron deficiency anemia is
A depletion of bone marrow hemosiderin
B falling of serum ferritin
C decrease of serum iron and increase of the iron-binding capacity
D decrease hemoglobin synthesis
E hypochromic microcytic anemia
ANSWER:A
2503 An extremely high RBC distribution width (RDW) is MOST likely consistent with
A iron deficiency anemia
B megaloblastic anemia
C sideroblastic anemia
D thalassemia
E sickle cell anemia
ANSWER:C
2504 The laboratory finding that is MOST likely consistent with the diagnosis of spherocytosis is
A high reticulocytes count of more than 10%
B normal mean corpuscular volume MCV
C low mean corpuscular hemoglobin MCH
D high mean corpuscular hemoglobin concentration MCHC
E indirect hyperbilirubinemia
ANSWER:D
2505 In hereditary spherocytosis, all the following will resolve postsplenectomy EXCEPT

A osmotic fragility
B anemia
C reticulocytosis
D hyperbilirubinemia
E hypoplastic/aplastic crises
ANSWER:A
2506 Splenectomy is recommended in all the following conditions EXCEPT
A a 6-year-old child with hereditary spherocytosis and significant hemolysis
B
a 7-year-old child with hereditary elliptocytosis and a hemoglobin level of 7 g/dL and
corrected reticulocytes count of more than 15%
C an 8-year-old child with hereditary stomatocytosis with hemolysis
D
a 6-year-old child with thalassemia major with splenomegaly and frequent blood transfusion
requirement
E
a 12-year-old child with chronic immune thrombocytopenia not responding to all modalities
of medical treatment
ANSWER:C
2507 The best diagnostic test for paroxysmal nocturnal hemoglobinuria (PNH) is
A acidified serum hemolysis (HAM) test
B sucrose lysis test
C complement assay
D flow cytometry
E bone marrow study
ANSWER:D
2508
All the following may occur as a renal manifestation in a child with sickle cell anemia
EXCEPT
A polyuria
B hyposthenuria
C nephrotic syndrome
D hematuria
E acute renal injury
ANSWER:E
2509 The least common infection that may occur in a 6-year-old child with sickle cell anemia is
A Pneumococcus sepsis
B Haemophilus influenzae meningitis
C Salmonella osteomyelitis

D E.coli urinary tract infection
E Parvovirus B19 aplastic episode
ANSWER:D
2510
Blood transfusion therapy in a child with sickle cell anemia is indicated in all the following
conditions EXCEPT
A acute chest syndrome
B stroke
C chronic intolerable pain syndrome
D avascular necrosis of the hip joint
E splenic sequestration
ANSWER:D
2511 The best assessment of iron overload for patients with thalassemia major is achieved by
A liver MRI
B bone marrow biopsy
C serum iron
D serum ferritin
E total iron binding capacity
ANSWER:A
2512
The criteria for successful bone marrow transplantation in thalassemic patients include all the
following EXCEPT
A age younger than 15-year-old
B no hepatomegaly
C no bone deformities
D no iron overload
E full HLA-matched sibling
ANSWER:C
2513
The best treatment for spinal cord compression at the vertebral canal with neurological
symptoms caused by extramedullary hematopoiesis in thalassemic patients is
A blood transfusion
B splenectomy
C iron chelation therapy
D local radiotherapy
E vitamin D supplementation
ANSWER:D

2514
You are evaluating a 4-year-old child with ß-thalassemia major; he is on chronic transfusion
therapy since the age of 1 year; you suspect transfusion-induced hemosiderosis. Of the
following, the organ that is LEAST likely to be affected by iron deposition at this time is
A liver
B pancreas
C heart
D pituitary gland
E thyroid gland
ANSWER:C
2515
A healthy 5-mo-old boy appears pale. Examination is unremarkable. Lab findings include:
Hb 8.1 g/dl; WBC 4,800/mm3; platelets 144,000/mm3; MCV, 111 fl; blood film showed
hypersegmented neutrophils; serum B12, 65 pg/mL (low). The infant is vigorously breast
fed. Of the following, the NEXT step of the management is
A performing Schilling test
B check serum B12 of the mother
C parenteral administration of vitamin B12
D administration of both folic acid and vitamin B12
E transfuse packed RBCs
ANSWER:B
2516
A 9-month-old child with a hemoglobin concentration of 10 gm/dL and marked
microcytosis; serum iron and total iron binding capacity are within normal limits; serum
ferritin and hemoglobin electrophoresis are also normal. Of the following, the MOST likely
diagnosis is
A iron deficiency anemia
B sideroblastic anemia
C ß-thalassemia minor
D α-thalassemia trait
E anemia due to chronic disease
ANSWER:D
2517
Hereditary intrinsic factor deficiency (HIFD), formerly called congenital pernicious anemia,
differs from the typical adult pernicious anemia by all the following EXCEPT
A it is prominent at around 3 mo of age
B stomach secretes acid normally
C stomach is histologically normal

D there are no antibodies to parietal cells
E there are no associated endocrine disorders
ANSWER:A
2518
Increased incubated osmotic fragility test that is not corrected by the addition of glucose is
MOST likely suggestive of
A hereditary spherocytosis
B hereditary elliptocytosis
C hereditary stomatocytosis
D glucose-6-phosphate dehydrogenase deficiency
E pyruvate kinase deficiency
ANSWER:E
2519 All the following may cause autoimmune hemolytic anemia EXCEPT
A systemic lupus erythematosus (SLE)
B immunodeficiency
C mycoplasma pneumoniae infection
D lymphoproliferative disorder
E penicillin drug administration
ANSWER:E
2520 All the following are correct regarding treatment of cold agglutinin disease EXCEPT
A patient should avoid exposure to cold
B patient should be treated for underlying disease
C plasmapheresis is a modality of treatment
D glucocorticoids treatment is a modality of treatment
E
patient can be treated by splenectomy when there is no response to other modalities of
treatment
ANSWER:E
2521 Fragmentation hemolysis by mechanical injury may be seen in all the following EXCEPT
A extensive burns
B Kasabach-Merritt syndrome
C after cardiac surgery for prosthetic heart valve replacement
D thrombotic thrombocytopenic purpura (TTP)
E hemolytic uremic syndrome (HUS)
ANSWER:A
2522 All the following conditions may be associated with iron deficiency EXCEPT
A hookworm infestation

B prolonged intravascular hemolysis
C celiac disease
D congenital heart disease with right to left shunt
E prolonged use of isoniazid (INH)
ANSWER:E
2523 Parvo virus B19 infection may cause transient pancytopenia in
A thalassemia major
B sickle cell anemia
C hereditary spherocytosis
D hereditary elliptocytosis
E pyruvate kinase deficiency
ANSWER:B
2524
Fresh frozen plasma (FFP) transfusion is the best treatment for which of the following
conditions
A hemophilia a
B hemophilia b
C factor vii deficiency
D immunoglobulin replacement therapy
E bleeding due to warfarin therapy
ANSWER:E
2525
Transfusion of fresh frozen plasma (FFP) is efficacious for the treatment of deficiency of all
the following coagulation factors EXCEPT
A factor V
B factor X
C factor XI
D factor XIII
E protein C
ANSWER:D
2526
Fresh frozen plasma (FFP) transfusion/replacement is indicated in all the following
conditions EXCEPT
A
a 9-year-old child with chronic liver disease with prolonged clotting times who needs a liver
biopsy
B a 6-year-old child with protein S deficiency and significant deep venous thrombosis
C
an 11-year-old child with thrombotic thrombocytopenic purpura (TTP) post allogenic bone
marrow transplantation

D a 2-day-old preterm neonate with clotting deficienciesto prevent intraventricular hemorrhage
E a 1-week-old term neonate with hemorrhage secondary to vitamin K deficiency
ANSWER:D
2527 The main reason behind gamma irradiation of the blood products is to prevent
A transfusion-related bacterial infections
B transfusion-related viral infections
C graft-versus-host disease (GVHD)
D allergic reactions
E transfusion-related acute lung injury (TRALI)
ANSWER:C
2528
CMV-transmitted infection is least likely occurring in transfusion of which of the following
blood products?
A whole blood
B packed RBCs
C granulocytes
D platelets
E fresh frozen plasma
ANSWER:E
2529 A prolonged thrombin time with normal reptilase time is consistent with
A dysfibrinogenemia
B afibrinogenemia
C factor XIII deficiency
D disseminated intravascular coagulopathy (DIC) with high level of fibrin split products
E heparin overdose
ANSWER:E
2530 In hemophilia A, factor VIII level activity should be increased to 100% in
A epistaxis
B iliopsoas bleeding
C tooth extraction
D hematuria
E gum bleeding
ANSWER:B

2531
A 9-year-old male child is about to do elective tonsillectomy; preoperative investigations are
all normal except for a significant prolonged partial thromboplastin time PTT; there was no
previous bleeding after circumcision and emergency appendectomy. Of the following, the
MOST likely diagnosis is
A hemophilia A
B hemophilia B
C hemophilia C
D prekallikrein deficiency
E factor XIII deficiency
ANSWER:D
2532 Spontaneous intracranial hemorrhage is more likely to occur in
A factor VII deficiency
B factor VIII deficiency
C factor IX deficiency
D factor XI deficiency
E factor XIII deficiency
ANSWER:A
2533 Systemic amyloidosis may be associated with deficiency of
A factor VII
B factor VIII
C factor IX
D factor X
E factor XI
ANSWER:D
2534
In which of the following conditions you should use a plasma that is frozen within 24 hours
after collection?
A factor V deficiency
B factor VII deficiency
C factor IX deficiency
D factor X deficiency
E factor XI deficiency
ANSWER:A
2535 All the following are true regarding factor XIII EXCEPT
A the half-life is 5-7 days
B PT, PTT, and thrombin time (TT) are prolonged when the factor is deficient

C clot solubility test with 5m urea is a screening test for factor XIII deficiency
D factor assay is a specific test to detect its level
E the hemostatic level is only 2-3%
ANSWER:B
2536
In severe liver disease with prolongation of the bleeding time, the best effective treatment
before doing liver biopsy is
A intravenous vitamin K
B intramuscular vitamin K
C oral vitamin K
D fresh frozen plasma
E intravenous DDAVP
ANSWER:E
2537 Vitamin K-dependent clotting factors include all the following EXCEPT
A factor I
B factor II
C factor VII
D factor IX
E protein S
ANSWER:A
2538
A 16-year-old female is found to have multiple bruising and hypochromic microcytic
anemia; during her first pregnancy she noticed disappearance of the bruises; she has a past
history of uncomplicated emergency appendectomy at the age of 10 year. Of the following,
the drug MOST likely beneficial for alleviation of her symptoms is
A ferrous sulphate
B desmopressin acetate (DDAVP)
C recombinant factor VIII
D recombinant factor IX
E aminocaproic acid
ANSWER:B
2539 Low platelets count may be encountered in all the following EXCEPT
A Von Willebrand disease (VWD) type 2 B
B VWD type 2 N
C Platelet type pseudo VWD
D Bernard Soulier syndrome
E Wiskott Aldrich syndrome

ANSWER:B
2540 Which of the following is FALSE about factor V Leiden mutation?
A it is the most common inherited risk factor for thrombosis
B heterozygous individuals have less risk for thrombosis than homozygotes
C heterozygotes have an increased risk of arterial thrombosis
D thereisanincreasedfrequencyofthrombosiswhilereceiving contraceptive agents
E there is an increased risk of recurrent abortions
ANSWER:C
2541 Arterial thrombosis is the least likely cause of
A stroke
B a cold and pulseless lower extremity
C renal infarction
D myocardial infarction
E pulmonary embolism
ANSWER:E
2542 Lupus anticoagulants usually affect the following laboratory test
A bleeding time (BT)
B partial thromboplastin time (PTT)
C prothrombin time (PT)
D thrombin time (TT)
E clot solubility test
ANSWER:B
2543 Which of the following is FALSE regarding standard unfractionated heparin therapy?
A it acts by enhancing the antithrombin III activity
B it can be used safely during pregnancy
C it should not be given for a child with bacterial endocarditis
D it can be given intramuscularly
E it needs monitoring by PTT test
ANSWER:D
2544
All the following are true regarding low molecular weight heparin (LMWH) in comparison
with unfractionated heparin (UFH) EXCEPT
A it is more stable
B it is ease to titrate
C it is monitored by activated PTT
D it can be used subcutaneously

E it can be used as an outpatient
ANSWER:C
2545
A healthy 5-day-old male neonate develops bruising and melena. The pregnancy, and
delivery course were unremarkable. The mother received heparin therapy for recurrent
abortions secondary to lupus anticoagulants. The infant is vigorously breastfeeding.
Examination reveals only multiple bruises. Lab tests reveals: hemoglobin, 8.4 g/dL; WBC,
8600/mm3; platelets count, 200000/mm3; PTT, 74 seconds; PT, 28 seconds; serum
fibrinogen is normal. Of the following, the MOST likely cause of this bleeding is
A disseminated intravascular coagulopathy (DIC)
B hemophilia
C vitamin K deficiency
D heparin therapy
E lupus anticoagulants
ANSWER:C
2546
A previously healthy 10-month-old boy develops pallor over 2 weeks. Examination is
unremarkable except for pallor. Lab tests includes: hemoglobin, 4.9 g/dL; WBC, 6700/mm3;
platelets count, 180000/mm3; MCV, 79 fL; reticulocyte count 0.8%; hemoglobin
electrophoresis, normal. Of the following, the MOST likely cause for these findings is
A pure red cell aplasia
B transient erythroblastopenia of childhood
C thalassemia minor
D aplastic anemia
E parvovirus infection
ANSWER:B
2547
55.You are evaluating a 5-year-old male child with multiple ecchymoses distributed all over
the extremities and trunk; examination reveal a palpable spleen 3 cm below the left costal
margin. Lab findings include: hemoglobin, 11.3 g/dL; white blood cell count, 8700/mm3;
platelets count, 21000/mm3. Of the following, the most proper NEXT step is
A bleeding time
B coagulation profile
C platelets aggregation test
D abdominal ultrasonography
E bone marrow study
ANSWER:E

2548 Both quantitative and qualitative platelets defects are seen in
A generalized vasculitis
B Bernard-Soulier syndrome
C Glanzmann's Thrombasthenia
D type I von Willebrand disease
E uremia
ANSWER:B
2549
Which of the following viruses is more likely to be associated with chronic immune
thrombocytopenia?
A cytomegalovirus
B parvovirus
C influenza virus
D parainfluenza virus
E human immune deficiency virus
ANSWER:E
2550
Whichofthefollowingisleastlikelytobeassociatedwithprolonged thrombocytopenia in
children?
A immune thrombocytopenia
B systemic lupus erythematosus (SLE)
C HIV infection
D type 2B von Willebrand disease
E Wiskott-Aldrich syndrome (WAS)
ANSWER:A
2551 Which of the following drugs is least likely to cause thrombocytopenia in children?
A valproic acid
B phenytoin
C sulfonamides
D trimethoprim-sulfamethoxazole
E heparin
ANSWER:E
2552 Microthrombocytes are MOST likely to be seen in
A immune thrombocytopenia (ITP)
B Bernard-Soulier syndrome
C Wiskott-Aldrich syndrome (WAS)
D Glanzmann's Thrombasthenia

E type 2B von Willebrand disease
ANSWER:C
2553 Thrombocytopenia that improves with age is MOST likely a feature of
A congenital amegakaryocytic thrombocytopenia
B Bernard-Soulier syndrome
C Wiskott-Aldrich syndrome
D thrombocytopenia absent radius syndrome (TAR)
E aplastic anemia
ANSWER:D
2554
You are going to explain the role of anti-D therapy to the parents of a 4-year-old boy with
acute immune thrombocytopenia (ITP) who has several ecchymotic lesions located over
extremities and trunk. There is no mucosal bleeding. An important statement that should be
included during the discussion is
A anti-D therapy is more effective than IVIG therapy in acute ITP
B anti-D therapy is very effective to all children with ITP
C when anti-D therapy is given, monitoring of hemoglobin concentration is recommended
D anti-D therapy is used in acute ITP only
E anti-D therapy can be used as initial treatment for children with acute ITP
ANSWER:C
2555
A 9-month-old boy appears pale. Examination reveals a palpable liver 3 cm below the right
costal margin and palpable spleen 4 cm below the left costal margin. Lab findings include:
hemoglobin concentration, 6.4 g/dl; mean corpuscular volume (MCV), 67 fL; and WBC,
15,000/mm3. Of the following, The MOST valuable test to confirm the diagnosis is
A blood smear
B serum iron, total iron binding capacity, and serum ferritin
C hemoglobin electrophoresis
D bone marrow study
E abdominal ultrasonography
ANSWER:C
2556
You are explaining the risk of sepsis after splenectomy to medical students. An important
statement that should be included during this discussion is that risk is especially high in
children
A older than 5 yr at the time of surgery
B with hereditary spherocytosis

C with chronic immune thrombocytopenia (ITP)
D after trauma
E with Hodgkin lymphoma
ANSWER:E
2557
You are evaluating a 7-year-old male child with cervical lymphadenopathy; the MOST likely
finding that raise your suspicion of malignancy is
A matted nodes
B fluctuant nodes
C fixed nodes
D presence of systemic symptoms
E no regression in the size of the nodes to its normal within 2 weeks
ANSWER:C
2558
Of the following, the MOST accurate reflection of active erythropoiesis of the bone marrow
is
A hemoglobin concentration
B packed cell volume (PCV)
C reticulocyte count
D mean corpuscular volume (MCV)
E reticulocyte index
ANSWER:E
2559
Hematologic disease with upper limb orthopedic congenital abnormalities with presence of
thumbs is MOST likely seen in
A Fanconi anemia
B thrombocytopenia with Absent Radii (TAR) Syndrome
C congenital Amegakaryocytic Thrombocytopenia
D Glanzmann Thrombasthenia
E Bernard-Soulier Syndrome
ANSWER:B
2560 Which of the following is not a feature of Wiskott–Aldrich Syndrome (WAS)
A thrombocytopenia
B eczema
C recurrent otitis media and pneumonia
D propensity to develop autoimmune disorders
E giant platelets by blood smear
ANSWER:E

2561
A 5-year-old child has multiple bruises on her extremities and oral mucosal bleeding of 3
days duration; she had a mild respiratory tract infection 2 weeks before; physical
examination was normal apart from multiple ecchymoses and petechiae. Of the following,
the NEXT diagnostic step is
A complete blood count
B prothrombin time
C bleeding time
D partial thromboplastin time
E antinuclear antibody titer
ANSWER:A
2562 Of the following, the LEAST likely feature of Bernard-Soulier syndrome is
A autosomal recessive inheritance
B severe thrombocytopenia
C giant Platelets
D abnormal ristocetin test
E platelet transfusion is the only reliable therapy
ANSWER:B
2563 Each of the following may cause thrombocytopenia EXCEPT
A heparin therapy
B aspirin ingestion
C disseminated intravascular coagulation (DIC)
D systemic lupus erythematosus (SLE)
E hemolytic-uremic syndrome (HUS)
ANSWER:B
2564
Which of the following children with acute immune thrombocytopenia is MOST likely to
have a benefit from splenectomy?
A an 8-year-old girl with multiple bruises and platelets count of 10,000/ml
B a 6-year-old boy with mild epistaxis and platelets count of 20,000/ml
C
a 4-year-old boy with subdural hematoma and platelets count of 100,000/ml after receiving
IVIG
D
an 11-year-old boy with severe rectal bleeding and platelets count of 20000/ml not
responding to medical treatment
E
a 13-year-old female adolescent with moderate menstrual bleeding and platelets count of
50000/ml
ANSWER:D

2565 Of the following, the LEAST likely feature of acute immune thrombocytopenia (ITP) is
A a preceding viral infection
B isolated thrombocytopenia
C mucocutaneous bleeding
D isolated splenomegaly
E increased number of megakaryocytes in the bone marrow
ANSWER:D
2566
A 3-year-old male child develops hematoma and bruising of his right hand next day after
falling on the ground; the mother stated that her child has a poor wound healing and a
history of delayed umbilical separation during the neonatal period. Of the following, the
MOST valuable test for this case is
A bleeding time
B partial thromboplastin time
C prothrombin time
D thrombin time
E clot solubility test
ANSWER:E
2567
A 5-year-old child, with hemophilia A of severe type, presents to the emergency unit with a
groin pain after a minor trauma to his back; his blood pressure is 60/30 mm Hg; his pulse
rate is 180/min; he holds his right hip in a flexion position with internal rotation. Of the
following, the NEXT step in the management of this child is
A factor VIII replacement therapy
B intravenous 1-deamino-8-d-arginine vasopressin (DDAVP)
C factor VIII assay
D abdominal ultrasonography
E abdominal computed tomography (CT) scan
ANSWER:A
2568
In a child with a recent diagnosis of hemophilia A who developed moderate epistaxis, the
aim of the treatment is to increase the level of factor VIII to
A 5-20%
B 35-50%
C 55-60%
D 65-70%
E 80-100%
ANSWER:B

2569 Allogeneic bone marrow stem cell transplantation is least likely to be required in
A Fanconi anemia
B thrombocytopenia with Absent Radii (TAR) Syndrome
C congenital Amegakaryocytic Thrombocytopenia (CAT)
D Wiskott –Aldrich Syndrome (WAS)
E severe combined immune deficiency
ANSWER:B
2570 Inherited thrombocytopenias include all the following EXCEPT
A congenital Amegakaryocytic Thrombocytopenia
B thrombocytopenia with Absent Radii (TAR) Syndrome
C Glanzmann’s Thrombasthenia
D Bernard-Soulier Syndrome
E Wiskott–Aldrich Syndrome (WAS)
ANSWER:C
2571
Which of the following is the best treatment for a 5-year-old girl with acute immune
thrombocytopenia (ITP) and platelets count of 30,000/ml and mild petechial rash all over
her body?
A intravenous immunoglobulin (IVIG)
B anti-D gamma globulin
C corticosteroids
D platelets transfusion
E no treatment
ANSWER:E
2572 The standard care for MOST children with severe hemophilia is
A
prevention by F VIII replacement therapy to prevent spontaneous bleeding and early joint
deformities
B aggressive treatment by F VIII replacement therapy when significant bleeding occurs
C avoid trauma
D avoid aspirin and other NSAID
E avoid violent contact sports
ANSWER:A
2573
The part of the glomerulus which serves as a supporting and probably has a role in the
regulation of glomerular blood flow, filtration, and removal of macromolecules is the
A glomerular basement membrane
B mesangium

C bowman’s capsule
D parietal epithelial cell
E visceral epithelial
ANSWER:B
2574 Collectively, the glomerular endothelial cell, basement membrane, and podocytes form the
A glomerulus
B filtration membrane
C juxtamedullary nephron
D nephron
E renal corpuscle
ANSWER:B
2575
The ultra-filtrate which is filtered through the glomerular capillary walls contains all of the
following EXCEPT
A electrolytes
B low-molecular-weight proteins
C creatinine
D albumin
E glucose
ANSWER:D
2576
In contrast to the concentration of blood urea nitrogen, the serum creatinine level is
primarily influenced by
A state of hydration
B nitrogen balance
C muscle mass
D hemorrhage
E vomiting
ANSWER:C
2577 Mesangial cell disease include all the following EXCEPT
A IgA nephropathy
B membranous nephropathy
C mesangioproliferative glomerulonephritis
D diabetic nephropathy
E class II lupus nephritis
ANSWER:B

2578
Hematuria is defined as the presence of at least 5 red blood cells per microliter of urine, false-
positive results may be seen in urinary dipstick reading in all the following EXCEPT
A an alkaline urine (pH > 8)
B contamination with hydrogen peroxide
C presence of formalin
D hemoglobinuria
E myoglobinuria
ANSWER:C
2579
Rhabdomyolysis is always clinically significant as it may lead to acute renal injury, it can
occurs secondary to
A severe electrolyte abnormalities
B bacterial endocarditis
C transfusion reactions
D mycoplasma infection
E autoimmune hemolytic anemia
ANSWER:A
2580 Glomerular hematuria is associated with
A urinary microscopic findings of RBC casts
B minimal proteinuria on dipstick (<100 mg/dL)
C gross hematuria that is bright red or pink
D the presence of leukocytes cast
E normal urinary RBC morphology
ANSWER:A
2581 In hematuria, renal biopsy is indicated in all the following EXCEPT
A some children with persistent microscopic hematuria
B decreased renal function
C proteinuria
D hypertension
E persistent high serum complement C3
ANSWER:E
2582 IgA nephropathy is characterized by
A hematuria occur after skin infection
B proteinuria is often >1000 mg/24 hr
C severe hypertension
D normal serum levels of C3

E serum IgA levels have a diagnostic value
ANSWER:D
2583
A 5-year-old male presented with dark color urine 1 to 2 days after an upper respiratory tract
infection, GUE: RBC +++, albumin +; blood urea: 25 mg/dl; serum creatine: 0.6 mg/dl; with
normal serum complement. Poor prognostic indicators at follow-up include all of the
following EXCEPT
A persistent hypertension
B diminished renal function
C significant proteinuria
D histologically diffuse mesangial proliferation
E recurrent macroscopic hematuria
ANSWER:E
2584
The primary treatment of IgA nephropathy is appropriate blood pressure control and
management of significant proteinuria. Regarding the treatment, which of the following is
TRUE?
A ACEI are ineffective in reducing proteinuria
B corticosteroids not improve renal function
C tonsillectomy is currently recommended
D successful kidney transplantation
E cyclophosphamide is effective in improving renal function
ANSWER:D
2585
Alport Syndrome AS, is a genetically heterogeneous disease caused by mutations in the
genes coding for type IV collagen, it is characterized by
A asymptomatic microscopic hematuria
B proteinuria<1 g/24 hr.
C congenital sensor neural hearing loss
D ocular abnormalities in 80% of X-linked type
E leiomyomatosis of the esophagus is common
ANSWER:A
2586
Which of the following is pathognomonic in diagnosis of X-linked type of hereditary
nephritis?
A gross hematuria
B discontinuous epidermal basement membrane staining
C platelet abnormalities
D hearing loss

E macular flecks
ANSWER:B
2587
Thin basement membrane disease TBMD is defined by the presence of persistent
microscopic hematuria and isolated thinning of the GBM on electron microscopy, all the
following are true EXCEPT
A significant proteinuria is rare
B episodic gross hematuria can be present
C may be transmitted as an autosomal dominant trait
D heterozygous mutations in the COL4A4 genes
E renal insufficiency is common
ANSWER:E
2588 The best single test to document cutaneous streptococcal infection is
A low serum C3 level
B elevated antistreptolysin O titer
C positive streptozyme screen
D antideoxyribonuclease B level
E depressed serum CH50
ANSWER:D
2589
A 10-year-old male (had a boil in the left thigh and mild fever one month ago) presented
with headache, malaise, lethargy, lion pain, decreased urine output, and puffiness of the face;
E: RBC cast ++, albumin ++; low C3 level and normal C4 level. Which of the following is
TRUE about his disease?
A acute nephritic syndrome 40%
B nephrotic syndrome 50%
C acute renal failure 90%
D ↑ ASO titers 70%
E hypertension 70%
ANSWER:E
2590
Renal biopsy in acute post streptococcal glomerulonephritis should be considered in all the
following EXCEPT
A acute renal failure
B nephrotic syndrome
C absence of evidence of streptococcal infection
D low C3 level in the first 2 months
E initially normal complement levels

ANSWER:D
2591
Acute post- infectious glomerulonephritis is one of the most common glomerular causes of
gross hematuria in children. Of the following, the TRUE statement is
A can follow fungal infection
B early systemic antibiotic, eliminate the risk of glomerulonephritis
C all family members of patient need antibiotic treatment
D antibiotic affect the natural history of the disease
E recurrences are common
ANSWER:C
2592 The MOST common etiology of secondary membranous nephropathy (MN) in children is
A neuroblastoma
B measles
C systemic lupus erythematosis
D chronic hepatitis C
E penicillin
ANSWER:A
2593 The well-known complication of membranous nephropathy is
A renal vein thrombosis
B hypertension 2%
C hematuria acute renal failure
D gross hematuria
E nephritic syndrome
ANSWER:A
2594 Type I membranoproliferative glomerulonephritis is
A less common than type II membranoproliferative glomerulonephritis
B not to be mediated by immune complexes
C associated with partial lipodystrophy
D called dense deposit disease
E characterized by low C3 complement level
ANSWER:E
2595 A.23. Renal involvement in childhood systemic lupus erythematosis (SLE) is present in
A 100% of patients
B 80% of patients
C 60%of patients
D 40% of patients

E 20% of patients
ANSWER:B
2596
WHO classification of lupus nephritis is based on a combination of features including light
microscopy, immunofluorescence, and electron microscopy. Of the following, the WORST
outcome is associated with
A class I nephritis
B class II nephritis
C class III nephritis
D class IV nephritis
E class V nephritis
ANSWER:D
2597 In treatment of lupus nephritis, plasmapheresis is effective in
A all patients
B classes III and IV
C maintenance therapy
D reducing proteinuria
E accompanying thrombotic thrombocytopenic purpura
ANSWER:E
2598
Henoch-Schönlein purpura is the most common small vessel vasculitis in childhood,
aggressive therapy may be reasonable in those with
A isolated microscopic hematuria
B insignificant proteinuria
C severe systemic manifestations
D >50% crescents on renal biopsy
E IgA deposits intensity
ANSWER:D
2599
“Rapidly progressive” RPGN describes the clinical course of several forms of
glomerulonephritis whose unifying feature is the histopathologic finding of crescents in the
majority of glomeruli. Plasmapheresis has been reported to be of benefit in patients with
A post infectious glomerulonephritis
B immunoglobulin A nephropathy
C Henoch-Schönleinpurpura
D Good pasture disease
E endocarditis

ANSWER:D
2600 AII the following diseases can cause a pulmonary-renal syndrome EXCEPT
A systemic lupus erythematosis
B Henoch-Schönlein purpura
C microscopic polyangiitis
D granulomatosis with polyangiitis
E F. IgA nephropathy
ANSWER:E
2601
Hemolytic-uremic syndrome is a common cause of community acquired acute kidney injury
in young children; atypical type may be triggered by
A toxin-producing E-coli
B raw milk
C Streptococcus pneumoniae
D shiga toxin
E cyclosporine
ANSWER:E
2602
In hemolytic-uremic syndrome, thrombotic microangiopathies are associated with aII the
following EXCEPT
A genetically determined factor H deficiency
B neuraminidase-producing Streptococcus pneumonia infection
C systemic lupus erythematosis
D malignancy
E hypotension
ANSWER:E
2603 In hemolytic-uremic syndrome (HUS). Of the following, the MOSTcorrect statement is
A majority of patients of diarrhea-associated enteropathictype develops HUS
B stool culture is often positive in patients who have diarrhea-associated HUS
C kidney biopsy is rarely indicated to diagnose HUS
D partial thromboplastin and prothrombin times are usually low
E Coombs test is usually negative in in pneumococci-induced HUS
ANSWER:C
2604
In the treatment of diarrhea-associated hemolytic-uremic syndrome. One of the following
options has no beneficial role
A dialysis
B early intravenous volume expansion

C control of hypertension
D eculizumab
E red cell transfusions
ANSWER:D
2605 All the following matching are true EXCEPT
A nephrotic syndrome---Penicillin
B nephrogenic diabetes inspidus ---cisplatin
C nephrolithiasis ---furosemide
D renal tubular acidosis---lithium
E interstitial nephritis---cimetidine
ANSWER:A
2606
Renal cortical necrosis is a rare cause of acute renal failure occurring secondary to extensive
ischemic damage of the renal cortex. Of the following, the LEAST common cause is
A perinatal asphyxia
B severe congenital heart disease
C severe hemolytic-uremic syndrome
D amniotic fluid embolism
E infectious endocarditis
ANSWER:E
2607
Allthe followingcauses hematuria associated with rapid development of microangiopathic
hemolytic anemia or enlargement of the kidney(s) EXCEPT
A nephrotic syndrome
B renal vein thrombosis
C polycystic kidney disease
D Wilms tumor
E hemolytic-uremic syndrome
ANSWER:A
2608 Hypercalciuria can be seen in
A hypoparathyroidism
B corticosteroid therapy
C vitamin D deficiency
D oral thiazide diuretics therapy
E Addison`s disease
ANSWER:B

2609 Clinical manifestations of sickle cell nephropathy SSN include all the following EXCEPT
A hematuria
B polyuria
C hemolytic-uremic syndrome
D renal tubular acidosis
E nephrotic-range proteinuria
ANSWER:C
2610
Autosomal dominant polycystic kidney disease is the most common hereditary human
kidney disease, all the following are true EXCEPT
A gross or microscopic hematuria
B may be seen in neonates
C abnormal renal sonography in the absence of symptoms
D is a multiorgan disorder
E right sided valvular heart disease
ANSWER:E
2611 Hemorrhagic cystitis can occur in response to all the following EXCEPT
A cyclophosphamide
B adenovirus infection
C cyclosporine
D polyoma BK virus infection
E amyloidosis
ANSWER:C
2612 All the following are treatment options of hemorrhagic cystitis EXCEPT
A antibiotic
B intensive intravenous hydration
C forced dieresis
D analgesia
E spasmolytic drugs
ANSWER:A
2613 In exercise hematuria; one of the following is TRUE
A A blood clots rarely seen in urine
B no dysuria
C abnormal findings on cystoscopy
D resolution after 7 days
E less common in males

ANSWER:A
2614 False-ngative results in urine dipstick measurement of protein can occur in
A very high urine pH (>7.0)
B large volume of urine output
C contamination of the urine with blood
D highly concentrated urine specimen
E contamination of the urine with antiseptic
ANSWER:B
2615
Which of the following methods offer more precise information regarding urine protein
excretion?
A frothy urine
B dipstick testing
C micro albuminuria
D spot urine for protein/creatinine ratio
E 24 hr. urine for protein and creatinine excretion
ANSWER:E
2616 Transient proteinuria can be seen in all the following EXCEPT
A temperature >38.3°C
B exercise
C over hydration
D cold exposure
E seizures
ANSWER:C
2617
Orthostatic proteinuria is increased amounts of protein in the upright position. Of the
following, the MOST appropriate answer is
A usually symptomatic
B hematuria may be present
C renal dysfunction are absent
D the cause is well-known
E edema usually positive
ANSWER:C
2618 All the following are secondary causes of nephrotic syndrome EXCEPT
A measles
B malaria
C syphilis

D toxoplasmosis
E schistosomiasis
ANSWER:A
2619
Children with nephrotic syndrome are especially susceptible to infections such as cellulitis,
spontaneous bacterial peritonitis, and bacteremia, all the following are contributory factors
EXCEPT
A urinary losses of immunoglobulin G
B urinary loss of complement factors
C impaired opsonization of microorganisms
D immunosuppressive therapy
E urinary losses of immunoglobulin A
ANSWER:E
2620 Nephrotic syndrome is a hypercoagulable state resulting from
A hemodilution
B decreased platelet number
C changes in coagulation factor levels
D decrease in hepatic production of fibrinogen
E high level of antithrombin III
ANSWER:C
2621
When you investigate for first attack of minimal change nephrotic syndrome (MCNS), one
of the following is TRUE
A microscopic hematuria is present in 20% of children
B spot urine protein : creatinine ratio should be <2.0
C serum complement levels are low
D renal biopsy is routinely performed
E 2+ proteinuria
ANSWER:A
2622 In treatment of minimal change nephrotic syndrome, all the following are true EXCEPT
A tuberculosis must be ruled out prior to starting immunosuppressive therapy
B steroid therapy may be initiated without a diagnostic renal biopsy
C prednisone should be administered as a single daily dose of 60 mg/m2/day
D remission consists of a urine<1+ protein on urine dipstick for3 consecutive days
E pleural effusion, is a potential complication of parenteral albumin therapy
ANSWER:E
2623 Steroid resistance nephrotic syndrome is defined as failure to achieve remission after

A 4wk of corticosteroid therapy
B 6 wk. of corticosteroid therapy
C 8 wk. of corticosteroid therapy
D 10 wk. of corticosteroid therapy
E 12 wk. of corticosteroid therapy
ANSWER:C
2624
To reduce the risk of serious infections in children with nephrotic syndrome, all the
following immunizations can be used EXCEPT
A pneumococcal 13-valent conjugant vaccine
B influenza vaccine annually to the child
C pneumococcal 23-valent polysaccharide vaccine
D varicella-zoster vaccine
E influenza vaccination annually to their household contacts
ANSWER:D
2625 After achievement of remission in children with nephrotic syndrome, they should
A restrict their activities
B restrict sodium intake
C be considered chronically ill
D maintain an unrestricted diet
E decrease water/fluid intake
ANSWER:D
2626
Congenital nephrotic syndrome is defined as nephrotic syndrome manifesting at birth or
within the 1st 3 mo of life, The etiologies include all the following EXCEPT
A syphilis
B herpes infection
C mercury exposure
D mutations in the NPHS1
E infantile systemic lupus erythematosis
ANSWER:B
2627 55.The definitive treatment of primary congenital nephrotic syndrome is
A angiotensin-converting enzyme inhibitors
B prostaglandin synthesis inhibitors
C aggressive nutritional support
D bilateral nephrectomies
E renal transplantation

ANSWER:E
2628 All the following matching are true EXCEPT
A proximal RTA----trimethoprim
B distal RTA----amphotericin B
C hyperkalemic RTA----cyclosporine
D distal RTA----lithium
E proximal RTA----gentamicin
ANSWER:A
2629
Cystinosis is a systemic disease caused by a defect in the metabolism of cysteine that results in
accumulation of cystine crystals in most of the major organs of the body. All the following
are recognized features EXCEPT
A renal insufficiency
B rickets
C hypothyroidism
D glaucoma
E fever
ANSWER:D
2630
Treatment of cystinosis is directed at correcting the metabolic abnormalities associated with
Fanconi syndrome or chronic renal failure, and the use of
A cystine eye drops
B oral cysteine
C prostaglandin synthesis inhibitor
D growth hormone
E angiotensin-converting enzyme inhibitors
ANSWER:D
2631 Urinalysis in patients with Fanconi syndrome may show all the following EXCEPT
A urine pH is alkaline
B glycosuria
C uricosuria
D phosphaturia
E elevated urinary sodium
ANSWER:A
2632 Distal RTA can be presented with
A hypocalciuria
B hypercitraturia

C phosphaturia
D conductive deafness
E growth failure
ANSWER:E
2633
Medullary sponge kidney is a relatively rare sporadic disorder in children. It is characterized
by
A cystic dilation of the proximal tubule
B nephrolithiasis
C proximal renal tubular acidosis
D concentrated urine
E cortical nephrocalcinosis
ANSWER:B
2634 Pathogenesis of hyperkalemic (Type IV) renal tubular acidosis is due to
A hyperaldosteronism
B inhibiting ammonia genesis
C pseudohyperaldosteronism
D massive bicarbonate wasting
E all of the above
ANSWER:B
2635 Urinary indices in patients with type IV RTA include all the following EXCEPT
A may be alkaline or acidic
B elevated sodium level
C inappropriately low potassium level
D foul-smell
E aminoaciduria
ANSWER:E
2636
Nephrogenic diabetes inspidus is a disorder of water metabolism characterized by an
inability to concentrate urine, even in the presence of antidiuretic hormone. Of the
following, the MOST correct statement is
A most common inheritance is autosomal dominant
B can be caused by hypokalemia
C mutations in the AQP2 gene in X-linked form
D irritability and crying are uncommon features
E secondary form present with hyponatremia
ANSWER:B

2637 All the following are options in the treatment of nephrogenic diabetes insipidus EXCEPT
A free access to water
B furosemide
C hydrochlorothiazide
D amiloride
E indomethacin
ANSWER:B
2638
Bartter syndrome is a group of disorders characterized by hypokalemic metabolic alkalosis
with hypercalciuria and salt wasting. It can be presented with all of the following EXCEPT
A maternal polyhydramnios
B severe episodes of recurrent dehydration
C failure to thrive in classic Bartter syndrome
D deafness is associated with antenatal Bartter syndrome
E hypertension due to hyperaldosteronism
ANSWER:E
2639 The diagnosis of Bartter syndrome typically include
A markedly low serum rennin
B high urinary calcium levels
C nephrocalcinosis
D hypomagnesaemia
E renal biopsy
ANSWER:B
2640 All the following are options in the treatment of Bartter syndrome EXCEPT
A potassium supplementation
B aldosterone antagonist
C high-sodium diet
D Indomethacin
E kidney transplantation
ANSWER:E
2641
Gitelman syndrome is a rare cause of autosomal recessive hypokalemic metabolic alkalosis,
the distinctive feature is
A Hypercalciuria
B hypermagnesuria
C hypernatremia

D low aldosterone levels
E elevated prostaglandin E secretion
ANSWER:B
2642
The juvenile nephronophthisis JN is a group of inherited genetically determined cystic renal
diseases, it can be presented with all the following EXCEPT
A oliguria
B unexplained anemia
C growth failure
D chronic renal failure
E retinal degeneration
ANSWER:A
2643
Acute kidney injury AKI is a clinical syndrome in which a sudden deterioration in renal
function results in the inability of the kidneys to maintain fluid and electrolyte homeostasis.
Of the following, the MOST correct matching is
A low urine sodium (UNa< 20 mEq/L) : intrinsic AKI
B fractional excretion of sodium >2% : prerenal AKI
C elevated urine osmolality (UOsm>500 mOsm/kg) : post renal AKI
D specific gravity of <1.010 : intrinsic AKI
E red blood cell casts :prerenal AKI
ANSWER:D
2644 Medical management of acute kidney injury include all the following EXCEPT
A bladder catheter should be placed immediately
B intravenous administration of boluses isotonic saline
C furosemide may be administered as a single IV dose
D fluid restriction is essential if there is no response to a diuretic challenge
E mannitol is not effective in prevention of pigment induced renal failure
ANSWER:E
2645
Hyperkalemia can lead to cardiac arrhythmia, cardiac arrest, and death. Serum potassium can
be lowered by all the following EXCEPT
A regular insulin with glucose
B calcium gluconate
C sodium bicarbonate
D Kayexalate
E Dialysis
ANSWER:B

2646 Treatment of hypocalcemia in acute kidney injury include all the following EXCEPT
A low-phosphorus diet
B intra venous phosphate binders
C calcium should not be given intravenously
D sevelamer
E avoid aluminum-based binders
ANSWER:B
2647 Indications for dialysis in acute kidney injury include all the following EXCEPT
A anuria/oliguria
B persistent hypercalcemia
C severe metabolic acidosis
D volume overload
E blood urea nitrogen >100-150 mg/dl
ANSWER:B
2648
Many patients with acute kidney injury require dialysis support for1-3 wk, which of the
following is TRUE statement
A Intermittent hemodialysis is useful in patients with unstable hemodynamic status
B continuous renal replacement therapy is useful in patients with stable hemodynamic status
C in peritoneal dialysis ,hypo-osmolar dialysate is infused into the peritoneal cavity
D anticoagulation is not necessary in peritoneal dialysis
E urea and creatinine clearance more with peritoneal dialysis than intermittent hemodialysis
ANSWER:D
2649 Chronic kidney disease in children <5-yr-old is MOST commonly a result of
A lupus nephritis
B familial juvenile nephronophthisis
C Alport syndrome
D focal segmental glomerulosclerosis
E autosomal dominant polycystic kidney disease
ANSWER:D
2650 Pathogenesis of chronic kidney disease (CKD) include all the following EXCEPT
A hyperfiltration injury
B hyperkalemia
C proteinuria
D hypertension
E hyperphosphatemia

ANSWER:B
2651
End-stage renal disease (ESRD) represents the state in which a patient’s renal dysfunction has
progressed to the point at which homeostasis and survival can no longer be sustained with
native kidney function and maximal medical management. Of the following, the TRUE
statement is
A the ultimate goal for children is dialysis
B renal replacement therapy be initiated at stage 5 CKD
C the time to actually initiate dialysis include impaired school performance
D peritoneal dialysis is performed 3 times weekly
E peritoneal dialysis use cycler-driven therapy increases the risk of peritonitis
ANSWER:C
2652
Kidney transplantation is the optimal therapy for children with end-stage renal disease, all the
following are true EXCEPT
A relative contraindication is preexisting metastatic malignancy
B successful transplantation leads to improvement in their linear growth
C preemptive transplantation mean transplantation without prior dialysis
D optimal outcomes result when the child weighs >15 kg
E urologic problems should be addressed before surgery
ANSWER:D
2653 Recurrent disease in the renal graft occur in all cases of
A focal segmental glomerulosclerosis
B primary oxalosis
C membranoproliferative glomerulonephritis type II
D IgA nephropathy
E congenital nephrotic syndrome
ANSWER:C
2654 Indications for bilateral native nephrectomies before renal transplantation include
A Denys-Drash syndrome
B IgA nephropathy
C chronic glomerulonephritis
D Prune belly syndrome
E medullary cystic disease
ANSWER:A
2655 Before renal transplantation, hemoglobin levels should be maintain at the level between
A 7 and 8 g/dL

B 9and 10 g/dL
C 11 and 12 g/dL
D 13 and 14 g/dL
E 15 and 16 g/dL
ANSWER:C
2656 Risk factor for renal graft thrombosis include
A prior history of peritoneal dialysis
B arterial hypertension
C donor age (>2 yr.)
D recipient age (>5 yr.)
E cold ischemia time <24 hr.
ANSWER:A
2657
Infections need to be identified and treated before renal transplantation. Infectious disease
screening includes all the following EXCEPT
A tuberculosis skin test
B cytomegalovirus IgG
C measles antibody
D hepatitis A serology
E varicella titer
ANSWER:D
2658 Antithymocyte globulin and thymoglobulin act against
A interleukin-2 Receptor Antibodies
B T-lymphocyte antigens
C B cells
D CD20 epitope
E mammalian target of rapamycin
ANSWER:B
2659
Of the following, the BEST choice for maintenance immunosuppression in renal
transplantation is
A cyclosporin
B basiliximab
C daclizumab
D rituximab
E belatacept
ANSWER:A

2660 The side-effect profile of cyclosporine in children include all the following EXCEPT
A hypertrichosis
B gingival hyperplasia
C neutropenia
D coarsening facial features
E hyperlipidemia
ANSWER:C
2661
Corticosteroids remain an integral part of many immunosuppressive protocols despite their
multifaceted toxicities. In children, the MOST noteworthy side effect is
A obesity
B hypertension
C aseptic necrosis of bone
D retarded skeletal growth
E cushingoid faces
ANSWER:D
2662
Polyomavirus nephropathy is an important cause of allograft dysfunction; the increased
incidence is thought to be the result of
A reduction in immunosuppression doses
B more potent immunosuppressive regimens
C cidofovir use
D leflunomide use
E ciprofloxacin therapy
ANSWER:E
2663 The following formulas are true EXCEPT
A sodium requirement= 0.6 ×weight in kg ×(125− serum sodium in mEq/L)
B blood anion gap=*Na+ − *Cl +HCO3+
C blood anion gap=*Na − Cl -HCO3]
D urine anion gap (*urine Na + urine K++ − urine Cl−)
E estimated GFR = 0.43 × height in m/serum creatinine in mg/dL
ANSWER:E
2664 Glomerular filtration rate in full term neonate is
A 15 mL/min/1.73 m2
B 25 mL/min/1.73 m2
C 35mL/min/1.73 m2
D 45 mL/min/1.73 m2

E 55 mL/min/1.73 m2
ANSWER:B
2665 Potter syndrome (bilateral renal agenesis) is characterized by all the following EXCEPT
A widely separated eyes
B polyhydramnios
C low set ears
D broad nose
E limb anomalies
ANSWER:B
2666 Death in neonates with bilateral renal agenesis is due to
A renal failure
B hypertension
C pulmonary insufficiency
D congenital heart disease
E sepsis
ANSWER:C
2667 The MOST common cause of an abdominal mass in the newborn is
A polycystic kidney disease
B neuroblastoma
C renal vein thrombosis
D hydronephrosis
E multicystic dysplastic kidney
ANSWER:E
2668 One of the complications of the horseshoe kidney is
A renal abscess
B neuroblastoma
C bladder outlet obstruction
D familial renal dysplasia
E multicystic dysplastic kidney
ANSWER:E
2669 All the following are risk factors for urinary tract infection EXCEPT
A male gender
B tight clothing (underwear)
C pinworm infestation
D poor toilet training

E voiding dysfunction
ANSWER:A
2670 Pyelonephritis usually requires total or partial nephrectomy in
A renal abscess
B perinephric abscess
C xanthogranulomatous pyelonephritis
D pyelonephritic scarring
E acute lobar nephronia
ANSWER:C
2671 Symptoms of cystitis include
A fever
B polyuria
C nausea
D suprapubic pain
E urinary retention
ANSWER:D
2672
According to the clinical guidelines, urine sample for diagnosis of urinary tract infection in
children 2-24 mo should be from
A an adhesive collection bag
B mid-stream sample
C urine bag
D catheterized sample
E all of the above
ANSWER:D
2673 In pyelonephritis, a high risk of renal scarring is associated with an elevated
A erythrocyte sedimentation rate
B C-reactive protein
C procalcitonin
D white blood cell count
E serum ferritin
ANSWER:C
2674
Acute cystitis should be treated promptly to prevent possible progression to pyelonephritis.
Of the following, the LEAST effective drug is
A trimethoprim-sulfamethoxazole
B cephalexin

C ciprofloxacin
D nitrofurantoin
E amoxicillin
ANSWER:D
2675 All the following are indications for admission to hospital in patients with urosepsis EXCEPT
A dehydration
B unable to drink fluids
C neonatal period
D vomiting
E microscopic hematuria
ANSWER:E
2676
Regarding vesicoureteral reflux VUR (the retrograde flow of urine from the bladder to the
ureter and kidney). All the following are true EXCEPT
A it is usually congenital
B it may cause hypertension in children
C the mean age at VUR resolution is 10yr
D severity is graded according to voiding cystourethrogram
E reimplantation of the ureters corrects the condition
ANSWER:C
2677
Vesicoureteral reflux is usually discovered during evaluation for a UTI, which of the
following is TRUE
A autosomal dominant inheritance
B males are mainly affected
C antenatal hydronephrosis mainly seen in females
D constipation is a rare association
E average age at diagnosis is less than 1 year
ANSWER:A
2678
Regarding the voiding cystourethrogram (VCUG) for diagnosis of vesicoureteral reflux
(VUR). Of the following, the MOST appropriate statement is
A low-pressure VUR is occurring during voiding
B high-pressure VUR is occurring during bladder filling
C high-pressure VUR is significantly less likely to resolve spontaneously
D contrast VCUG is significantly exposed to less radiation than radionuclide cystogram
E contrast VCUG provides more anatomic information

ANSWER:E
2679
The antenatal hydronephrosis is graded by the trimester and the antero-posterior diameter of
the renal pelvis. Of the following, the MOST likely cause is
A vesicoureteral reflux
B ureteropelvic junction obstruction
C transient hydronephrosis
D posterior urethral valve
E ureterovesical junction obstruction
ANSWER:C
2680
Ureteropelvic junction obstruction is the most common obstructive lesion in childhood. Of
the following, the MOST correct statement is
A it is usually caused by extrinsic stenosis
B it mainly occurs on the right side
C there is a female preponderance
D voiding cystourethrogram (VCUG) is necessary
E it is usually bilateral
ANSWER:D
2681
Prompt surgical repair in ureteropelvic junction obstruction is indicated in infants with all the
following EXCEPT
A solitary kidney
B abdominal mass
C bilateral severe hydronephrosis
D hematuria after minimal trauma
E diminished function in the involved kidney
ANSWER:D
2682
Ureter that drains outside the bladder is referred to as an ectopic ureter. Of the following,
which statement is TRUE regarding ectopic ureter?
A male: female ratio is 2: 1
B it mostly drains into the cervix in girls
C it mostly drains into the vas deferens in boys
D urinary tract infection (UTI) is uncommon
E nephrectomy is indicated if renal function is poor
ANSWER:E
2683
Ureteroceles is a cystic dilation of the terminal ureter. Of the following, the MOST
appropriate statement is

A it is more common in boys
B it cannot be diagnosed prenatally
C it is associated with ureteral duplication in girls
D it commonly causes bladder outlet obstruction
E the orthotopic type extends into the urethra
ANSWER:C
2684
Prune-belly syndrome, also called triad syndrome or Eagle-Barrett syndrome, is
characterized by
A female predominance
B polydramnios
C small bladder
D cardiac abnormalities (in 10% of cases)
E all of the above
ANSWER:D
2685
Posterior urethral valves, is the most common cause of severe obstructive uropathies in
children. Of the following, the MOST appropriate statement
A its incidence 1 in 40,000 boys
B vesicoureteral reflux occurs in 5% of patients
C the urinary stream is strong
D Foley catheter should be avoided
E it is adiverticulum in the penile urethra
ANSWER:D
2686 Classic bladder exstrophy is characterized by
A hypospadias
B narrow based gait
C normal upper urinary tracts
D posteriorly displaced anus
E upward displaced umbilicus is
ANSWER:C
2687
Neuropathic bladder dysfunction in children is usually congenital resulting from neural tube
defects or other spinal abnormalities. All the following are options in the treatment EXCEPT
A botulinum toxin
B cholinergic drugs
C cutaneous vesicostomy

D antimicrobial prophylaxis
E clean intermittent catheterization
ANSWER:B
2688 Complications of augmentation cystoplasty includes all the following EXCEPT
A bladder calculi
B metabolic alkalosis
C malignant neoplasm
D urinary tract infection
E spontaneous perforation
ANSWER:B
2689
In children up to the age of 14 yr, the mean bladder capacity in ounces is equal to the age (in
years) plus
A 2
B 3
C 4
D 5
E 6
ANSWER:A
2690 The MOST common cause of daytime incontinence is
A an overactive bladder
B infrequent voiding
C detrusor–sphincter dyssynergia
D bladder outlet obstruction
E behavioral
ANSWER:A
2691 Staccato urinary stream is seen in
A hypospadias
B meatal stenosis
C ureteral ectopia
D posterior urethral valves
E non neurogenic bladder
ANSWER:E
2692 Pollakiuria is characterized by
A dysuria
B nocturia

C occurrence at 7-10 yr of age
D daytime incontinence
E voiding every 10-15 min during the day
ANSWER:E
2693 Family history in nocturnal enuresis is positive in
A 10%
B 30%
C 50%
D 70%
E 90%
ANSWER:C
2694 The pathogenesis of nocturnal enuresis is
A defective sleep arousal
B nocturnal polyuria
C genetic factors
D overactive bladder
E multifactorial
ANSWER:E
2695 The MOST effective way of treatment in older children with nocturnal enuresis is
A motivational therapy
B conditioning therapy
C desmopressin acetate
D oxybutynin chloride
E imipramine
ANSWER:B
2696 All the following are contraindications to circumcision in neonates EXCEPT
A hypospadias
B chordee without hypospadias
C dorsal hood deformity
D small penis
E penile torsion
ANSWER:E
2697
The most common cause of micropenis is failure of the hypothalamus to produce an
adequate amount of gonadotropin-releasing hormone. Of the following, the syndrome
associated with micropenis is

A Kallmann syndrome
B fetal hydantoin syndrome
C Apert syndrome
D de Lange syndrome
E Holt-Oram syndrome
ANSWER:A
2698 Priapism of high-flow type MOST commonly follows
A sickle cell disease
B perineal trauma
C sildenafil ingestion
D leukemia
E all of the above
ANSWER:B
2699 The MOST common male urethral anomaly associated with prune-belly syndrome is
A congenital urethral fistula
B urethral duplication
C megalourethra
D urethral hypoplasia
E urethral atresia
ANSWER:C
2700 The consequences of cryptorchidism include all the following EXCEPT
A infertility
B hydrocele
C inguinal hernia
D testicular malignancy
E poor testicular growth
ANSWER:B
2701 The MOST common cause of testicular pain in a 12- yr-old boy is
A epididymitis
B testicular tumor
C testicular torsion
D scrotal hematoma
E incarcerated inguinal hernia
ANSWER:C
2702 The risk of renal stone formation increases in the presence of

A low urine pH
B urinary citrate
C urinary magnesium
D urinary glycosaminoglycan
E urinary osteopontin
ANSWER:A
2703 Irritative symptoms of dysuria, urgency, and frequency usually indicate a calculus in the
A urethra
B bladder
C distal ureter
D middle part of ureter
E ureteropelvic junction
ANSWER:C
2704 Uric acid stones occur in
A inflammatory bowel disease
B corticosteroids therapy
C vitamin D excess
D distal renal tubular acidosis
E hyperoxaluria
ANSWER:A
2705 The MOST accurate study in a child with suspected renal stone is
A renal ultra-sonogram
B plain radiograph of the abdomen
C an unenhanced spiral CT scan
D radioisotope studies
E magnetic resonance urography
ANSWER:C
2706 The normal values for 24-hr urine calcium is
A <4mmol/1.73 m2
B >4mmol/1.73 m2
C <4 mg/kg
D >4 mg/kg
E 4 mg/kg
ANSWER:C
2707 Secondary hyperoxaluria can occur in patients with

A furosemide therapy
B sarcoidosis
C pyridoxine deficiency
D primary hyperparathyroidism
E immobility
ANSWER:C
2708 Cystinuria is characterized by
A X-linked recessive inheritance
B decreased cystine urinary excretion
C renal tubular acidosis
D alkaline urine
E faint radiopaque renal calculi
ANSWER:E
2709 Struvite calculi (secondary to urinary tract infections) are MOST likely caused by
A Escherichia coli
B Proteus
C Klebsiella
D Pseudomonas
E Staph. aurous
ANSWER:B
2710
Nephrocalcinosis refers to calcium deposition within the renal tissue. All of the following
can cause nephrocalcinosis EXCEPT
A furosemide
B distal RTA
C hypoparathyroidism
D hyperoxaluria
E Cushing syndrome
ANSWER:C
2711 Dietary treatment of renal or ureteral calculi include
A increased dietary intake of sodium
B reduced dietary intake in potassium
C increased protein diet
D encourage lemon juice intake
E calcium restriction
ANSWER:D

2712 Which of the following drugs can precipitate and form renal stones?
A acetazolamide
B indanivir
C probenecid
D theophylline
E vitamin C
ANSWER:B
2713 The least likely cause of Infectious vulvovaginitis is
A Escherichia coli
B Staphylococcus aurous
C Haemophilus influenza
D Enterobiusvermicularis
E Candida spp.
ANSWER:E
2714 Amoxicillin-clavulanate is the first-line treatment in vulvovaginal infections caused by
A Streptococcus pyogenes
B Staphylococcus aurous
C Haemophilus influenza
D Shigella
E Chlamydia trachomatis
ANSWER:B
2715 Ultra potent topical corticosteroids is the first-line therapy in
A labial agglutination
B lichen sclerosus
C psoriasis
D atopic dermatitis
E seborrheic dermatitis
ANSWER:B
2716 All the following are first-line treatment of diaper dermatitis EXCEPT
A frequent diaper changes
B increasing diaper free period
C frequent bathing
D antifungal treatment
E zinc oxide
ANSWER:D

2717 Vulvar dermatologic condition in children which necessitate evaluation of thyroid function is
A lichen sclerosus
B psoriasis
C vitiligo
D seborrheic dermatitis
E atopic dermatitis
ANSWER:C
2718 The gold standard diagnostic investigation for precocious puberty is
A Tanner maturating rate
B left wrist x-ray
C serum luteinizing hormone levels
D gonadotropins measurement
E brain MRI
ANSWER:D
2719
Lack of development of the breast is considered delayed and warrants endocrinology
evaluation by the age of
A 10 yr
B 11 yr
C 12 yr
D 13 yr
E 14 yr
ANSWER:D
2720 Amastia (complete absence of the breast) associated with all the following EXCEPT
A Poland syndrome
B ectodermal dysplasia
C Crohn disease
D congenital adrenal hypoplasia
E hypogonadotropic hypogonadism
ANSWER:D
2721 Of the following, the MOST likely cause of bloody nipple discharge in infants is
A chronic nipple irritation
B ducts of Montgomery
C hypothalamic tumors
D Intraductal cysts

E mammary duct ectasia
ANSWER:E
2722 The MOST common cause of breast pain in adolescents is
A benign fibroadenoma
B exercise
C mastitis
D lipoma
E Intraductal papilloma
ANSWER:B
2723 Of the following, the MOST common solid mass seen in adolescent girls is
A lipoma
B fibroadenoma
C lymphangioma
D hematoma
E hamartoma
ANSWER:B
2724 The MOST common tumor metastasized to the breast is
A rhabdomyosarcoma
B lymphoblastic leukemia
C neuroblastoma
D lymphoma
E cystosarcomaphyllodes
ANSWER:A
2725 The imaging modality of choice for breast abnormalities in the pediatric population is
A mammography
B CT scan
C ultrasonography
D MRI
E PET scan
ANSWER:C
2726 All the following may cause hirsutism EXCEPT
A hydrochlorothiazide
B acetazolamide
C cyclophosphamide
D anabolic steroids

E penicillamine
ANSWER:C
2727 Chocolate cysts are seen in
A functional cysts
B endometriomas
C cyst adenomas
D teratomas
E ovarian carcinoma
ANSWER:B
2728 5-yr survival is 100% in the following germ cell tumors
A gonadoblastoma
B embryonal carcinoma
C choriocarcinoma
D dysgerminoma
E endodermal sinus tumor
ANSWER:A
2729 The MOST common ovarian malignancy is
A gonadoblastoma
B embryonal carcinoma
C choriocarcinoma
D dysgerminoma
E endodermal sinus tumor
ANSWER:A
2730
The tumor marker (α-fetoprotein) is used for the diagnosis of all the following ovarian
cancers EXCEPT
A Immature teratoma
B embryonal carcinoma
C mixed germ cell
D dysgerminoma
E endodermal sinus tumor
ANSWER:D
2731 athogenesis of leiomyosarcoma is thought to be correlated with
A Epstein-Barr virus
B haemophilusinfluenzae
C enterococcus

D nocardia
E Yersinia
ANSWER:A
2732
Approved treatment for condyloma acuminate of the vulva in children include all the
following EXCEPT
A topical trichloroacetic acid
B local cryotherapy
C electro cautery
D laser ablation
E sinecatechins ointment
ANSWER:E
2733 Hemihematometra is described as
A accumulation of mucus or nonsanguineous fluid in the vagina
B atretic segment of vagina with menstrual fluid accumulation
C accumulation of serous fluid in the fallopian tube
D one cervix associated with 2 uterine horns
E the result of failure of 1 müllerian duct to descend
ANSWER:B
2734
The MOST sensitive and specific imaging technique used for evaluating müllerian
anomalies is
A ultrasound
B hysterosalpingogram
C sonohysterography (saline-infusion sonography)
D MRI
E CT scan
ANSWER:D
2735 The MOST common structural uterine anomaly is
A uterine septum
B bicornuate uterus
C unicornuate uterus
D uterine didelphys
E arcuate uterus
ANSWER:A
2736 An imperforate hymen is characterized by all the following EXCEPT
A the incidence is approximately 1 in 1,000

B normal secondary sex characters
C more often it is diagnosed at the time of menarche
D cannot be diagnosed In the newborn period
E primary amenorrhea
ANSWER:D
2737 Main causes of hypocalcemia in nephrotic syndrome in children are the following EXCEPT
A decreased activity of renal α-hydroxylase
B lowered serum albumin
C decreased intestinal absorption of calcium
D increased urinary losses of cholecalciferol-binding globulin
E prednisone therapy
ANSWER:A
2738
The following criteria are needed for the diagnosis of syndrome of inappropriate secretion of
antidiuretic hormone (SIADH) EXCEPT
A excessive urinary sodium concentration
B hyponatremia with normal serum osmolality
C elevated urine osmolality
D normal renal, adrenal, and thyroid function
E absence of volume depletion
ANSWER:B
2739 The following are main features of diabetic ketoacidosis (DKA) EXCEPT
A hyperglycemia (glucose usually >300 mg/dL)
B ketonemia (serum ketones >3 mmol/L)
C ketonuria
D venous pH <7.20
E serum HCO3<15 mEq/L
ANSWER:D
2740
What percent of newly diagnosed diabetics present with symptoms of diabetic ketoacidosis
(DKA)?
A 10%
B 20%
C 30%
D 40%
E 50%

ANSWER:C
2741 Typical insulin dosage requirement in prepubertal children after the "honeymoon" period is
A 0.4 U/kg/day
B 0.5 U/kg/day
C 0.6 U/kg/day
D 0.7 U/kg/day
E 1 U/kg/day
ANSWER:B
2742 The following features suggest constitutional delay as a cause of short stature EXCEPT
A no signs or symptoms of systemic disease
B bone age delayed beyond the height age
C period of poorest growth often occurring between the ages of 18 and 30 months
D parental or sibling history of delayed development
E height predictions consistent with family characteristics
ANSWER:B
2743 Congenitalgoiterisseeninwhichpercentofnewbornswithcongenital hypothyroidism?
A 10%
B 20%
C 30%
D 40%
E 50%
ANSWER:B
2744
During a routine physical examination, a solitary thyroid nodule is palpated in asymptomatic
10-year-old child. Of the following, the MOST likely cause is
A thyroid carcinoma
B thyroid adenoma
C thyroid abscess
D thyroid cyst
E subacute thyroiditis
ANSWER:A
2745 The pituitary gland is located in a saddle-shaped cavity of the
A frontal bone
B maxillary bone
C sphenoid bone

D ethemoid bone
E temporal bone
ANSWER:C
2746
Physiologic factors play a role in stimulating and inhibiting growth hormone (GH). One of
the following inhibit GH release
A sleep
B exercise
C hyperglycemia
D acute illness
E fasting
ANSWER:C
2747 Corticotropin-releasing hormone (CRH) and ACTH release are inhibited by
A arginine vasopressin
B oxytocin
C angiotensin II
D atrial natriuretic peptide
E cholecystokinin
ANSWER:D
2748 The MOST common cause of acquired hypopituitarism is
A craniopharyngioma
B eosinophilic granuloma (histiocytosis)
C tuberculosis
D toxoplasmosis
E meningitis
ANSWER:A
2749 Indications for GH treatment to promote growth include the following EXCEPT
A GH deficiency
B chronic renal failure before transplantation
C celiac disease
D Prader-Willi syndrome
E Noonan syndrome
ANSWER:C
2750 Reported side effects of GH treatment include the following EXCEPT
A pseudotumor cerebri
B slipped capital femoral epiphysis

C gynecomastia
D worsening of scoliosis
E 6-fold increase in the risk for type 1 diabetes
ANSWER:E
2751 The following stimulate vasopressin (VP) secretion EXCEPT
A hyperosmolality
B hypovolemia
C hypotension
D hyperglycemia
E nausea
ANSWER:D
2752 Acquired nephrogenic diabetes insipidus (NDI) is associated with the following EXCEPT
A lithium
B vancomycin
C amphotericin
D methicillin
E rifampin
ANSWER:B
2753 Neonates and young infants with central diabetes insipidus are often best treated with
A fluid therapy
B vasopressin analogs
C thiazide diuretics
D indomethacin
E amiloride
ANSWER:A
2754 Hyponatremia in cerebral salt wasting is accompanied by the following EXCEPT
A elevated urinary sodium excretion
B low urine output
C hypovolemia
D normal or high uric acid
E suppressed vasopressin
ANSWER:B
2755
Broad forehead, hypertelorism, small chin, long philtrum, camptodactyly, and fetal finger
pads are features of the following genetic overgrowth syndrome
A Perlman syndrome

B Sotos syndrome
C Weaver syndrome
D Beals syndrome
E Beckwith-Wiedemann syndrome (BWS)
ANSWER:C
2756
Precociouspubertyingirlsisdefinedbytheonsetofsecondarysexual characteristics before the age
of
A 7 yr
B 8 yr
C 9 yr
D 10 yr
E 11 yr
ANSWER:B
2757 The MOST common brain lesion causing central precocious puberty is
A postencephalitic scar
B tuberculous meningitis
C hypothalamic hamartoma
D tuberous sclerosis
E hydrocephalus
ANSWER:C
2758 All the following are true in premature thelarche EXCEPT
A most often appears in the 1st 2 yr of life
B growth and osseous maturation are usually normal
C menarche occurs at the expected age
D may be unilateral
E often persists till puberty
ANSWER:E
2759
Premature adrenarche has traditionally applied to the appearance of sexual hair before the
age of 8 yr in girls or 9 yr in boys without other evidence of maturation. It is characterized
by the following EXCEPT
A more frequent in girls
B higher prevalence in african-american
C axillary hair generally appears earlier
D adult-type axillary odor is common
E affected children are often slightly advanced in height

ANSWER:C
2760 The MOST accurate test of thyroid function is
A T4
B free T4
C T3
D thyroglobulin
E TSH
ANSWER:E
2761 Peak serum concentrations of TSH at birth in full-term infants reach
A 20 mU/L
B 40 mU/L
C 60 mU/L
D 80 mU/L
E 100 mU/L
ANSWER:C
2762
The thyroid hormones are transported in plasma bound to thyroxine-binding globulin
(TBG), a glycoprotein synthesized in the liver. TBG binds approximately 70% of T4 and
50% of T3. TBG level increase with administration of
A estrogens
B androgens
C glucocorticoids
D nicotinic acid
E l-asparaginase
ANSWER:A
2763 The MOST common cause of permanent congenital hypothyroidism is
A dyshormonogenesis
B thyroid dysgenesis
C Iodine deficiency
D defect of iodide transport
E Pendred syndrome
ANSWER:B
2764
The cord serum T4 is decreased in proportion to gestational age and birthweight. The serum
T4 gradually increases and enters the T4 range seen in term infants by the age of
A 2 wk
B 4 wk

C 6 wk
D 8 wk
E 10 wk
ANSWER:C
2765
Approximately 10% of infants with congenital hypothyroidism have associated congenital
anomalies. Of the following, the MOST common are
A nervous system anomalies
B lungs anomalies
C eye anomalies
D cardiac anomalies
E genitourinary anomalies
ANSWER:D
2766 The normal level of serum thyroid-stimulating hormone in first week of term Infants is up to
A 4.4 mIU/L
B 9.5 mIU/L
C 13.6 mIU/L
D 17.6 mIU/L
E 27.0 mIU/L
ANSWER:D
2767
Retardation of osseous development in congenitally hypothyroid infants can be shown
radiographically at birth in approximately
A 20%
B 40%
C 60%
D 80%
E 100%
ANSWER:C
2768
Levothyroxine (l-T4) given orallyis the treatment of choice in congenital hypothyroidism.
The recommended initial starting dose is
A 2-5 μg/kg/day
B 4-7 μg/kg/day
C 5-9 μg/kg/day
D 8-12 μg/kg/day
E 10-15 μg/kg/day

ANSWER:E
2769 The MOST common cause of acquired hypothyroidism is
A craniopharyngioma
B Hashimoto thyroiditis
C meningoencephalitis
D drug-induced
E irradiation
ANSWER:B
2770
Antithyroid antibodies develop and subclinical or overt hypothyroidism occurs in the
following conditions EXCEPT
A Down syndrome
B Turner syndrome
C type 1 diabetes mellitus
D
Sjِ en syndrome
E Williams syndrome
ANSWER:E
2771 Risk for subclinical hypothyroidism is seen in children with
A chronic hepatitis B infection
B chronic hepatitis C infection
C chronic hepatitis D infection
D hepatitis A infection
E hepatitis E infection
ANSWER:B
2772 The first clinical manifestation of acquired hypothyroidism is
A poor schoolwork
B goiter
C deceleration of growth
D weight gain
E constipation
ANSWER:C
2773 One of the following is not routinelydoneinchildrenwithsuspected hypothyroidism
A sonography
B serum free T4
C TSH
D antithyroglobulin

E antiperoxidase antibodies
ANSWER:A
2774
Chroniclymphocyticthyroiditisisthemostcommoncauseofacquired hypothyroidism, with or
without goiter. All the following are true EXCEPT
A more common in girls
B peak incidence during adolescence
C most of the affected children are asymptomatic
D clinical course is constant
E familial clusters are common
ANSWER:D
2775 Thyroid peroxidase antibodies are absent in the following thyroiditis syndrome
A Hashimoto thyroiditis
B painless sporadic thyroiditis
C painful subacute thyroiditis
D acute suppurative thyroiditis
E Riedel thyroiditis
ANSWER:D
2776
Endemic cretinism is the most serious consequence of iodine deficiency; it includes two
different but overlapping syndromes: a neurologic type and a myxedematous type. The
neurologic syndrome is characterized by the following EXCEPT
A intellectual disability
B deaf-mutism
C disturbances in gait
D patellar hyperreflexia
E delayed pubertal development
ANSWER:E
2777 All the following conditions are associated with Graves’ disease EXCEPT
A type 1 diabetes mellitus
B vitiligo
C iron deficiency anemia
D alopecia areata
E celiac disease
ANSWER:C
2778 Adverse reaction reported exclusively with propylthiouracil is
A severe liver disease

B agranulocytosis
C lupus-like polyarthritis syndrome
D glomerulonephritis
E antineutrophilic cytoplasmic antibody–positive vasculitis
ANSWER:A
2779
A painless nodule in the thyroid or in the neck is the usual presentation of childhood thyroid
cancer. Of the following, the MOST common site of distant metastasis is
A mediastinum
B long bones
C skull
D brain
E lungs
ANSWER:E
2780 Hypocalcemia is common in neonates between 12 and 72 hr of life, especially in infants with
A birth asphyxia
B sepsis
C exchange transfusion
D hypomagnesemia
E hyperbilirubinemia
ANSWER:A
2781
Aplasia or hypoplasia of the parathyroid glands is often associated with the
DiGeorge/velocardiofacial syndrome. All the following regarding this syndrome are true
EXCEPT
A occurs in 1 in 4,000 newborns
B caused by a deletion of chromosome 22q11.2
C hypocalcemia is transitory in the majority
D hypocalcemia can have its onset later in life
E conotruncal defects of the heart occur in 75% of cases
ANSWER:E
2782
There is a spectrum of parathyroid deficiencies with clinical manifestations varying from no
symptoms to those of complete and longstanding deficiency. Early manifestations include
A tingling of the hands and feet
B muscular pain and cramps

C carpopedal spasms
D convulsions
E delayed irregular teeth eruption
ANSWER:B
2783 Laboratory findings in hypoparathyroidism include the following EXCEPT
A low serum calcium (5-7 mg/dL)
B elevated serum phosphorus (7-12 mg/dL)
C elevated serum alkaline phosphatase
D level of 1,25(OH)2D3 is usually low
E normal serum magnesium
ANSWER:C
2784
Episodic symptomatic hypocalcemia occurs in the Kenny-Caffey syndrome, the latter is
characterized by the following EXCEPT
A medullary stenosis of the long bones
B short stature
C delayed closure of the fontanel
D advanced bone age
E eye abnormalities
ANSWER:D
2785
Type Ia accounts for the majority of patients with pseudohypoparathyroidism (PHP), they
have the following skeletal abnormalities EXCEPT
A brachydactyly
B dimpling of the dorsum of the hand
C the three middle fingers are the same length
D bowing
E thickening of the calvaria
ANSWER:C
2786 Parathyroid crisis is manifested by the following EXCEPT
A serum calcium levels >15 mg/dL
B progressive polyuria
C azotemia
D stupor
E coma
ANSWER:B

2787 TheMOSTconsistentandcharacteristicradiographicfindingofprimary hyperparathyroidism is
A gross trabeculation of the skull
B generalized rarefaction
C cysts
D resorption of subperiosteal bone
E radiographic signs of rickets
ANSWER:D
2788 All the following are causes of infantile hypercalcemia EXCEPT
A subcutaneous fat necrosis
B metaphyseal chondrodysplasia
C lactase/disaccharidase deficiency
D blue diaper syndrome
E proximal renal tubular acidosis
ANSWER:E
2789 Glucocorticoids play a major role in immune regulation. They increase circulating
A polymorphonuclear cell
B monocytes
C eosinophils
D lymphocytes
E basophils
ANSWER:A
2790
Glucocorticoids readily penetrate the blood–brain barrier and have direct effects on brain
metabolism. They have the following actions EXCEPT
A decrease brain edema
B insomnia
C increased irritability
D impaired memory
E improved ability to concentrate
ANSWER:E
2791 55.Patients with mineralocorticoid deficiency can develop the following EXCEPT
A weight loss
B hypotension
C metabolic alkalosis
D hyponatremia

E hyperkalemia
ANSWER:C
2792 The MOST common cause of primary adrenal insufficiency in infancy is
A congenital adrenal hyperplasia
B isolated glucocorticoid deficiency
C X-linked adrenal hypoplasia congenita
D Lemli-opitz syndrome
E adrenoleukodystrophy
ANSWER:A
2793
The clinical presentation of adrenal insufficiency depends on the age of the patient, the usual
presentation of adrenal insufficiency in infancy is
A ketosis
B hyperpigmentation
C orthostatic hypotension
D hypoglycemia
E hypernatremia
ANSWER:D
2794 The MOST definitive test for adrenal insufficiency is measurement of
A blood sugar
B cortisol before and after administration of ACTH
C serum sodium
D arterial blood gases
E urinary excretion of sodium and chloride
ANSWER:B
2795
Patients with autoimmune Addison disease must be closely observed for the development of
other autoimmune disorders. Of the following, the MOST commonly associated disorder in
children is
A alopecia
B vitiligo
C chronic active hepatitis
D type 1 diabetes mellitus
E hypoparathyroidism
ANSWER:E
2796
High-dose glucocorticoids (the equivalent of>10 timesphysiologic cortisol secretion) can be
administered without requiring a subsequent tapering for

A 5 day
B 7 day
C 10 day
D 14 day
E 21 day
ANSWER:B
2797
Classic 21-hydroxylase deficiency occurs in approximately 1 in 15,000- 20,000 births in
most populations. Of affected infants, the salt-losing form constitute about
A 40%
B 50%
C 60%
D 70%
E 80%
ANSWER:D
2798
The signs and symptoms of congenital adrenal hyperplasia (salt wasting form of the disease)
caused by 21-hydroxylase deficiency typically first develop in affected infants at
approximately
A 1 week of age
B 2 weeks of age
C 3 weeks of age
D 4 weeks of age
E 6 weeks of age
ANSWER:B
2799
Significantly virilized females with congenital adrenal hyperplasia (CAH) usually undergo
surgery between
A 2- 6 mo of age
B 7-12 mo of age
C 13-24 mo of age
D 5-9 year of age
E 10-14 year of age
ANSWER:A
2800
Cortisol deficiency in congenital adrenal hyperplasia (salt wasting form of the disease) is
treated with hydrocortisone
A 5-10 mg/m2/24 hr
B 10-15 mg/m2/24 hr

C 15-20 mg/m2/24 hr
D 20-25 mg/m2/24 hr
E 25-30 mg/m2/24 hr
ANSWER:C
2801
In older children with Cushing syndrome, in addition to obesity, a common early
manifestation is
A purplish striae on abdomen
B short stature
C hypertension
D hyperglycemia
E osteoporosis
ANSWER:B
2802 All the following tests are of diagnostic utility in the diagnosis of Cushing syndrome EXCEPT
A midnight cortisol levels
B nighttime salivary cortisol levels
C glucose tolerance test
D urinary excretion of free cortisol
E dexamethasone suppression test
ANSWER:C
2803 In primary aldosteronism, one of the following is low
A serum PH
B serum carbon dioxide
C serum sodium
D serum chloride
E serum calcium
ANSWER:D
2804 Clinical manifestations of pheochromocytomas include the following EXCEPT
A hypertension
B convulsions
C pulmonary edema
D obesity
E good appetite
ANSWER:D

2805
Calcification within the adrenal glands may occur in a wide variety of situations, some
serious and others of no obvious consequence. Adrenal calcifications are bilateral in
A neuroblastomas
B ganglioneuromas
C Wolman disease
D cortical carcinomas
E pheochromocytomas
ANSWER:C
2806 The major causes of incidentalomas are
A benign adenomas
B pheochromocytomas
C adrenocortical carcinoma
D hemorrhagic cysts
E myelolipomas
ANSWER:A
2807 Congenital hypogonadotropic hypogonadism include
A gonadal dysgenesis
B Klinefelter syndrome (47,XXY)
C Noonan syndrome
D cystic fibrosis
E Laurence-Moon- Biedl syndrome
ANSWER:E
2808 Acquired hypogonadotropic hypogonadism include
A chemotherapy
B malnutrition
C cystic fibrosis
D infection (e.g., mumps)
E infarction (testicular torsion)
ANSWER:B
2809
Klinefelter syndrome is the most common sex chromosomal aneuploidy in males, all the
following features are true EXCEPT
A 80% of them have a 47,xxy chromosome complement
B diagnosis is rarely made before puberty
C psychiatricdisordersmaybeapparentlongbeforedefectsinsexual development
D few complete high school

E approximately 80% of adults have gynecomastia
ANSWER:D
2810
True gynecomastia is characterized by the presence of a palpable fibroglandular mass
(located concentrically beneath the nipple and areolar region) at least
A 0.5 cm in diameter
B 1 cm in diameter
C 1.5 cm in diameter
D 2 cm in diameter
E 2.5 cm in diameter
ANSWER:A
2811 Weaker associations with gynecomastia are seen in
A spironolactone
B alkylating agents
C ketoconazole
D cimetidine
E opiates
ANSWER:E
2812 Significant gynecomastia is seen in
A hyperthyroidism
B Klinefelter syndrome
C 17-ketosteroid reductase deficiency
D 11β-hydroxylase deficiency
E Peutz-Jeghers syndrome
ANSWER:B
2813
All prepubertal gynecomastia cases, as well as pubertal cases with suspicious features, should
be investigated. Of the following, the initial laboratory evaluation should include
A karyotype
B dehydroepiandrosterone sulfate
C liver function tests
D thyroid function tests
E renal function tests
ANSWER:D
2814 Many patients with Turner syndrome are recognizable at birth because of
A loose skin folds at the nape of the neck
B cubitus valgus

C webbing of the neck
D widely spaced nipples
E hyperconvex fingernails
ANSWER:A
2815 The MOST common cardiac defect in Turner syndrome is
A aortic coarctation
B isolated nonstenotic bicuspid aortic valves
C aortic stenosis
D mitral valve prolapse
E anomalous pulmonary venous drainage
ANSWER:B
2816
The 47,XXX (trisomy) chromosomal constitution is the most frequent extra-X chromosome
abnormality in females, occurring in almost 1 in 1,000 liveborn females. It is characterized
by the following EXCEPT
A maternal meiotic nondisjunction is the most common cause
B normal female phenotype
C sexual development and menarche are delayed
D tall and gangly
E behavior disorders
ANSWER:C
2817 The MOST common cardiac defect in Noonan syndrome is
A aortic coarctation
B pulmonary valvular stenosis
C aortic stenosis
D mitral valve prolapse
E anomalous pulmonary venous drainage
ANSWER:B
2818 Familial isolated gonadotropin deficiency associated with anosmia is
A Noonan syndrome
B Bloom syndrome
C Werner syndrome
D Kallmann syndrome
E Prader-Willi
ANSWER:D
2819 Children with type 2 DM often seek medical care because of

A weight loss
B excessive weight gain
C polyuria
D polydipsia
E anorexia
ANSWER:B
2820
The current criteria for the diagnosis of type 1 and type 2 DM is a fasting blood glucose that
exceeds
A 110 mg/dL
B 115 mg/dL
C 120 mg/dL
D 125 mg/dL
E 130 mg/dL
ANSWER:D
2821 The clearest evidence of a role for viral infection in human type 1 DM is seen in
A acquired rubella infection
B live-virus rubella immunization
C enteroviral infection
D congenital rubella syndrome
E mumps infection
ANSWER:D
2822 The following factors are implicated in the pathogenesis of type 1 DM EXCEPT
A prenatal influences
B diet in infancy
C viral infections
D excessive exposure to certain infections
E psychologic stress
ANSWER:D
2823 Falsely low HbA1c levels are noted in the following conditions EXCEPT
A aplastic anemia
B pure red cell aplasia
C blood transfusions
D cirrhosis
E renal disease treated with erythropoietin
ANSWER:A

2824 Subjects with type 1 DM have evidence for celiac disease in
A 5-10%
B 11-15%
C 16-20%
D 21-25%
E 26-30%
ANSWER:A
2825
Diabetic ketoacidosis (DKA) is the end result of the metabolic abnormalities resulting from a
severe deficiency of insulin or insulin effectiveness. DKA is characterized by the following
EXCEPT
A ketonuria
B normal ion gap
C decreased serum bicarbonate
D decreased pH
E elevated effective serum osmolality
ANSWER:B
2826
To continue the insulin infusion in treaterment of DKA without causing hypoglycemia,
glucose must be added to the infusion. We typically recommend that glucose be added as a
10% solution when the serum glucose has decreased
A <100 mg/dL
B <150 mg/dL
C <200 mg/dL
D <250 mg/dL
E <300 mg/dL
ANSWER:C
2827
Diabetic ketoacidosis (DKA) may be arbitrarily classified as mild, moderate, or severe.
Venous pH in severe DKA is
A <7.05
B <7.10
C <7.15
D <7.20
E <7.25
ANSWER:C
2828
Any child with diabetic ketoacidosis (DKA) can be easily transitioned to oral intake and
subcutaneous insulin when the following criteria are found EXCEPT

A total CO2 >15 mEq/L
B pH >7.25
C sodium stable between 135 and 145 mEq/L
D no emesis
E no dehydration
ANSWER:B
2829
Cerebral edema complicating DKA remains the major cause of morbidity and mortality in
children and adolescents with T1 DM. All the following are true EXCEPT
A early bolus administration of insulin is a risk factor
B high volumes of fluid is a risk factor
C the incidence of cerebral edema in children with DKA has not changed over the past 15-20 yr
D radiographic imaging is frequently helpful in making the diagnosis of cerebral edema
E its etiology remains unknown
ANSWER:D
2830 Nonketotic hyperosmolar coma is characterized by the following EXCEPT
A severe hyperglycemia
B nonketotic acidosis
C severe dehydration
D hypothermia
E positive babinski signs
ANSWER:D
2831 HbA1c values may be spuriously elevated in
A thalassemia
B sickle cell disease
C iron deficiency anemia
D leukemia
E aplastic anemia
ANSWER:A
2832 Microvascular complications of DM include
A nephropathy
B accelerated coronary artery disease
C cerebrovascular disease
D peripheral vascular disease
E peripheral neuropathies

ANSWER:A
2833
uidelines suggest that prepubertal T1DM patients should commence screening for
retinopathy
A at diagnosis
B 1 yr after diagnosis
C 3yr after diagnosis
D 5 yr after diagnosis
E 10yr after diagnosis
ANSWER:D
2834 uidelines suggest that diabetic patients should commence screening for celiac disease
A at diagnosis
B 1 yr after diagnosis
C 3 yr after diagnosis
D 5 yr after diagnosis
E 10 yr after diagnosis
ANSWER:A
2835 Transient elevation of urinary albumin can occur in the following conditions EXCEPT
A hypoglycemia
B strenuous exercise
C urinary tract infections
D heart failure
E acute febrile illness
ANSWER:A
2836
100.Signs of insulin resistance or conditions associated with insulin resistance in T2 DM in
children include the following EXCEPT
A acanthosis nigricans
B hypertension
C dyslipidemia
D increased waist : hip ratio
E polycystic ovary syndrome
ANSWER:D
2837
101.The MOST commonly used and the only FDA-approved oral agent for the treatment of
T2 DM in children and adolescents is
A thiazolidinediones
B pramlintide

C metformin
D sulfonylureas
E acarbose
ANSWER:C
2838 102.Donohue syndrome is characterized by the following EXCEPT
A intrauterine growth restriction
B fasting hypoglycemia
C acanthosis nigricans
D postprandial hyperglycemia
E profound resistance to insulin
ANSWER:C
2839
One of the following do not appear to be of concern in cystic fibrosis–related diabetes
(CFRD)
A ketoacidosis
B macrovascular complications
C microvascular complications
D frequent infections
E energy needs
ANSWER:B
2840
104.When diabetes and thyroid disease coexist, the possibility of autoimmune adrenal
insufficiency should be considered. It may be heralded by the followings EXCEPT
A increasing insulin requirements
B increasing pigmentation of the skin
C salt craving
D asthenia
E postural hypotension
ANSWER:A
2841
Therapy with one of the following drugs usually results in significant insulin resistance
leading to glucose intolerance and overt diabetes
A streptozotocin
B rodenticide Vacor
C high-dose oral steroid
D cyclosporin
E tacrolimus
ANSWER:C

2842 The skull tends to assume a square or boxlike shape in
A Kleinfelter syndrome
B neurofibromatosis
C storage disorder
D chronic subdural hemorrhages
E hydrocephalus
ANSWER:D
2843 Smell can be tested reliably at
A 32nd wk of gestation
B birth
C three months of age
D six months of age
E nine months of age
ANSWER:A
2844 Contraindications to performing a lumbar puncture include the following EXCEPT
A suspected mass lesion of the brain
B suspected mass lesion of the spinal cord
C symptoms and signs of impending cerebral herniation
D skin infection at the site of the lumbar puncture
E thrombocytopenia with a platelet count <50 × 109/L
ANSWER:E
2845 An elevated polymorphonuclear (PMN) cells count in cerebrospinal fluid (CSF) suggests
A tuberculous meningitis
B early phase of aseptic meningitis
C fungal meningitis
D demyelinating diseases
E brain or spinal cord tumor
ANSWER:B
2846 Xanthochromia of cerebrospinal fluid (CSF) suggests the following EXCEPT
A bloody tap
B subarachnoid hemorrhage
C carotenemia
D hyperbilirubinemia
E markedly elevated CSF protein
ANSWER:A

2847
Cranial CT is a valuable diagnostic tool in the evaluation of the following conditions
EXCEPT
A skull fractures
B intracranial hemorrhages
C acute infarcts
D hydrocephalus
E impending herniation
ANSWER:C
2848
An approach to imaging of the spine in patients with cutaneous lesions is indicated in the
following conditions EXCEPT
A hairy patch
B subcutaneous mass or lipoma
C dermal sinus
D coccygeal pits
E scarlike lesions
ANSWER:D
2849 Regarding myelomeningocele, all the following are true EXCEPT
A risk of recurrence after one affected child is 3-4%
B
maternal periconceptional use of folic acid supplementation reduces the incidence of neural
tube defects (NTDs) in pregnancies at risk by at least 50%
C
anticonvulsant valproic acid causes NTDs in approximately 1-2% of pregnancies when
administered during pregnancy
D
hydrocephalus in association with a type II Chiari malformationdevelops in at least 80% of
patients with myelomeningocele
E
the lower the deformity is in the neuraxis (sacrum), the more likely is the risk of
hydrocephalus
ANSWER:E
2850 Mobius syndrome is characterized by the following EXCEPT
A bilateral facial weakness
B paralysis of the abducens nerve
C hypoplasia or agenesis of brainstem nuclei
D feeding difficulties
E mental retardation
ANSWER:E
2851 Dandy-Walker malformation is characterized by the following EXCEPT

A cystic dilatation of the fourth ventricle
B herniation of the cerebellar tonsils though the foramen magnum
C hypoplasia of the cerebellar vermis
D hydrocephalus
E an enlarged posterior fossa
ANSWER:B
2852 Familial (autosomal recessive) microcephaly is characterized by the following EXCEPT
A slanted forehead
B prominent nose and ears
C mild or borderline mental retardation
D prominent seizures
E surface convolutional markings of the brain
ANSWER:C
2853 Causes of communicating hydrocephalus include
A achondroplasia
B aqueductal stenosis
C Chiari malformation
D Dandy-Walker malformation
E Klippel-Feil syndrome
ANSWER:A
2854
Nonobstructive or communicating hydrocephalus may follow the following conditions
EXCEPT
A a subarachnoid hemorrhage
B pneumococcal meningitis
C tuberculous meningitis
D leukemic infiltrates
E a vein of Galen malformation
ANSWER:E
2855 Regarding the CSF, all the following are true EXCEPT
A approximately 25% of CSF originates from extrachoroidal sources
B in a normal child, about 20 mL/hr of CSF is produced
C total volume of CSF approximates 150 mL in an infant
D
normally, CSF flows from the lateral ventricles through the foramina of Monro into the 3rd
ventricle
E CSF is absorbed primarily by the arachnoid villi

ANSWER:C
2856 Trigoncephaly is a skull deformity that is a direct result of premature fusion of the following
A sagittal suture
B coronal suture
C lambdoid suture
D metopic suture
E multiple sutures
ANSWER:D
2857 Crouzon syndrome is characterized by the following EXCEPT
A an autosomal dominant inheritance
B brachycephaly
C ocular proptosis
D hypoplasia of the maxilla
E syndactyly
ANSWER:E
2858 Minor risk factors for recurrence of febrile seizure include the following EXCEPT
A complex febrile seizure
B age <1 yr
C family history of febrile seizures
D male gender
E lower serum sodium
ANSWER:B
2859
There are several predictors of epilepsy after febrile seizures, the highest percent of risk
factor for subsequent epilepsy after febrile seizure is
A complex febrile seizure, any type
B fever <1 hr before febrile seizure
C recurrent febrile seizures
D focal complex febrile seizure
E family history of epilepsy
ANSWER:D
2860 Dravet syndrome is characterized by all the following EXCEPT
A the most severe of the phenotypic spectrum of febrile seizures
B its onset is in the 1st yr of life
C seizures subsequently start to occur without fever

D developmental delay
E an autosomal dominant inheritance
ANSWER:E
2861 Electroencephalogram (EEG) in febrile seizure is characterized by the following EXCEPT
A an EEG need not norrmally be performed in first simple febrile seizure
B an abnormal EEG could predict the future recurrence of febrile seizures or epilepsy
C spikes during drowsiness are often seen in children with febrile seizures
D an EEG performed within 2 wk of a febrile seizure often have nonspecific slowing
E an EEG should be used to delineate the type of epilepsy
ANSWER:B
2862 Absence seizures are MOST often initially treated with
A ethosuximide
B valproate
C lamotrigine
D acetazolamide
E clonazepam
ANSWER:A
2863 enign myoclonic epilepsies are often best treated with
A clonazepam
B lamotrigine
C topiramate
D valproate
E benzodiazepines
ANSWER:D
2864
Gingival hyperplasia, coarsening of the facies, hirsutism, and cerebellovestibular symptoms
(nystagmus and ataxia) are adverse effects of
A carbamazepine
B lamotrigine
C phenytoin
D valproic acid
E succinimides
ANSWER:C
2865 Rickets is a potential side effect from all the following EXCEPT
A phenytoin
B valproate

C phenobarbital
D primidone
E carbamazepine
ANSWER:B
2866
Epilepsy surgery is often used to treat refractory epilepsy of a number of etiologies including
the following EXCEPT
A cortical dysplasia
B tuberous sclerosis
C polymicrogyria
D degenerative problems
E Sturge-Weber syndrome
ANSWER:D
2867 The MOST common cause of neonatal seizures is
A vascular events
B intracranial infections
C brain malformations
D hypoxic-Ischemic encephalopathy
E metabolic disturbances
ANSWER:D
2868 All the following are true regarding diazepam in neonatal seizures EXCEPT
A diazepam is highly lipophilic
B it is cleared very quickly
C recurrence of seizures is more than other anticonvulsants
D it carries a risk of apnea and hypotension
E it is currently recommended as a first-line agent
ANSWER:E
2869 Neuro-imaging is warranted in a child with headache in the following conditions EXCEPT
A abnormal neurologic examination
B afternoon headache
C headache in children <6 yr old
D brief cough headache
E migrainous headache in the child with no family history of migraine
ANSWER:B
2870 Tension-type headaches (TTH) are characterized by the following EXCEPT
A diffuse in location

B not affected by activity
C throbbing quality
D mild to moderate in severity
E less frequently associated with nausea and photophobia
ANSWER:C
2871 The following are complications of neurofibromatosis (type 1) NF-1 EXCEPT
A learning disability
B cataracts
C seizures
D precocious puberty
E scoliosis
ANSWER:B
2872
Definite tuberous sclerosis complex (TSC) is diagnosed when at least 2 major or 1 major
plus 2 minor features are present, all the following are minor features EXCEPT
A shagreen patch
B cerebral white matter migration lines
C multiple dental pits
D gingival fibromas
E bone cysts
ANSWER:A
2873
The facial port-wine stain of Sturge-Weber syndrome (SWS) is characterized by the
following EXCEPT
A present at birth
B tends to be unilateral
C always involves the upper face and eyelid
D its distribution is consistent with the ophthalmic division of the trigeminal nerve
E most children with facial port-wine stain have SWS
ANSWER:E
2874 All the following are features of Von Hippel–Lindau (VHL) disease EXCEPT
A its incidence is around 1: 36,000
B fifty percent have a de novo gene mutation
C hemangioblastoma of the spinal cord may be found
D renal carcinoma is the most common cause of death
E pheochromocytoma is a frequent association
ANSWER:B

2875 HACE syndrome denotes to the following EXCEPT
A posterior fossa malformations
B hemangiomas
C anal anomalies
D coarctation of the aorta
E eye abnormalities
ANSWER:C
2876
The current antiepileptic drug (AEDs) of choice for primary generalized tonic-clonic
seizures in children is
A phenytoin
B carbamazepine
C phenobarbital
D valproate
E topiramate
ANSWER:D
2877 The MOST common type of cerebral palsy (CP) associated with seizures is
A spastic hemiplegia
B spastic quadriplegia
C spastic diplegia
D hypotonic CP
E ataxic CP
ANSWER:A
2878 The following are manifestations of increased intracranial pressure in an infant EXCEPT
A increasing head circumference
B bulging fontanel
C failure to thrive
D persistent vomiting
E setting sun sign
ANSWER:D
2879 Characteristic features of pseudotumor cerebri include the following EXCEPT
A headache
B stiff neck
C papilledema
D fatigue
E abnormal CSF profile

ANSWER:E
2880 How often are EEGs abnormal in healthy children?
A 5%
B 10%
C 15%
D 20%
E 25%
ANSWER:B
2881 The MOST common precipitant of status epilepticus in children is
A CNS infection
B fever
C medication change
D trauma
E metabolic cause
ANSWER:B
2882 A 9-month-old infant can do all the following EXCEPT
A pulls to stand
B pincer grasp
C plays pat-a-cake
D imitates sounds
E comes when called
ANSWER:E
2883 A premature infant blinks in response to a bright light at
A 26 wk of corrected gestational age
B 28 wk of corrected gestational age
C 30 wk of corrected gestational age
D 32 wk of corrected gestational age
E 34 wk of corrected gestational age
ANSWER:B
2884 The MOST common cause of anatomic megalencephaly is
A Sotos syndrome
B benign familial megalencephaly
C Simpson-Golabi-Behmel syndrome
D fragile X syndrome
E Weaver syndrome

ANSWER:B
2885
Craniosynostosis is defined as premature closure of the cranial sutures. Of the following, the
MOST common form of craniosynostosis is
A frontal plagiocephaly
B occipital plagiocephaly
C scaphocephaly
D trigonocephaly
E turricephaly
ANSWER:C
2886 The following are risk factors for deformational plagiocephaly EXCEPT
A female sex
B firstborn child
C congenital torticollis
D developmental delay
E exclusive bottle feeding
ANSWER:A
2887 Major risk factors for recurrence of febrile seizure include
A fever 38-39°c
B family history of febrile seizures
C family history of epilepsy
D complex febrile seizure
E lower serum sodium at time of presentation
ANSWER:A
2888
In patients with febrile seizures, one of the following risk factors has the highest risk for
subsequent epilepsy
A recurrent febrile seizures
B fever <1 hr before febrile seizure
C neurodevelopmental abnormalities
D family history of epilepsy
E complex febrile seizures
ANSWER:C
2889
The majority of patients who had prolonged febrile seizures and encephalopathy after
vaccination and who had been presumed to have suffered from vaccine encephalopathy
(seizures and psychomotor regression occurring after vaccination and presumed to be caused
by it) turn out to have

A generalized epilepsy with febrile seizures plus(GEFS+)
B temporal lobe epilepsy secondary to mesial temporal sclerosis
C myoclonic astatic seizures
D Dravet syndrome
E focal febrile seizures plus epilepsy variant
ANSWER:D
2890 Higher risk of recurrence of the febrile seizure is associated with lower serum
A sodium
B potassium
C chloride
D calcium
E magnesium
ANSWER:A
2891
A history of personality change in a patient with seizure could suggest the following as a
cause of seizure
A intracranial tumor
B degenerative disease
C metabolic disease
D stimulants drugs
E congenital brain dysfunction
ANSWER:A
2892
uidelines on the evaluation of a first unprovoked nonfebrile seizure, the following studies
are recommended in specific clinical situations
A head CT
B head MRI
C spinal tap
D ECG
E EEG
ANSWER:C
2893
Drug therapy should be based on the type of seizure and the epilepsy syndrome as well as on
other individual factors. Of the following, the drug of first choice for focal seizures and
epilepsies is
A carbamazepine
B ethosuximide
C valproate

D lamotrigine
E clobazam
ANSWER:A
2894 Weight gain and alopecia are side effects of the following antiepileptic drugs AEDs
A primidone
B valproic acid
C oxcarbazepine
D lamotrigine
E acetazolamide
ANSWER:B
2895 The ketogenic diet is absolutely contraindicated in
A primary carnitine deficiency
B myoclonic– astatic epilepsy
C tuberous sclerosis complex
D Rett syndrome
E infantile spasms
ANSWER:A
2896
55.Discontinuation of antiepileptic drugs (AEDs) is usually indicated when children are free
of seizures for at least
A 1 yr
B 1.5 yr
C 2 yr
D 2.5 yr
E 3 yr
ANSWER:C
2897
The following factors are associated with a higher risk of seizure relapse after antiepileptic
drugs (AEDs) withdrawal EXCEPT
A younger age of epilepsy onset
B longer duration of epilepsy
C presence of multiple seizure types
D need to use more than 1 AED
E abnormal EEG before medication is discontinued
ANSWER:A
2898
Sudden unexpected death in epilepsy (SUDEP) is the most common epilepsy related
mortality in patients with chronic epilepsy. All the following are risk factors EXCEPT

A polypharmacology
B female gender
C age younger than 16 yr
D long duration of epilepsy
E frequent seizures
ANSWER:B
2899
There are 5 main neonatal seizure types: subtle, clonic, tonic, spasms, and myoclonic. One of
the following seizures is frequently not associated with electrographic discharges
A spasms
B focal clonic
C subtle
D focal tonic
E generalized myoclonic
ANSWER:C
2900 Subtle seizures include all the following EXCEPT
A blinking
B mouthing
C fluctuations in heart rate
D hypotension episodes
E apnea
ANSWER:D
2901 The following are features of Aicardi syndrome EXCEPT
A coloboma of the iris
B retinal lacunae
C agenesis of the corpus callosum
D severe seizures
E aminoaciduria
ANSWER:E
2902
Many inborn errors of metabolism cause generalized convulsions in the newborn period.
Prominent hiccups, persistent generalized seizures, and lethargy rapidly leading to coma are
features of
A propionic academia
B maple syrup urine disease
C nonketotic hyperglycinemia
D Leigh disease

E neonatal adrenoleukodystrophy
ANSWER:C
2903 The initial drug used to control acute neonatal seizures is usually
A diazepam
B midazolam
C phenobarbital
D lorazepam
E phenytoin
ANSWER:D
2904
Intravenous phenytoin is not widely used to control acute neonatal seizures because of all the
following EXCEPT
A reduced solubility
B severe local cutaneous reactions
C interaction with other drugs
D possible cardiac toxicity
E not possible to mix with saline solutions
ANSWER:E
2905
Status epilepticus is a medical emergency that should be anticipated in any patient who
presents with an acute seizure. It is defined as continuous seizure activity or recurrent seizure
activity without regaining of consciousness lasting for more than
A 5 min
B 15 min
C 30 min
D 45 min
E 60 min
ANSWER:A
2906 The following studies are needed for all patients with status epilepticus EXCEPT
A glucose
B magnesium
C complete blood count
D MRI
E CT scan
ANSWER:D
2907
Currently, the level of the evidence for refractory treatment of status epilepticus is strongest
for

A midazolam
B propofol
C levetiracetam
D phenytoin
E Phenobarbital
ANSWER:A
2908 All the following are treatment options for breath-holding spells EXCEPT
A education and reassurance of the parents
B iron therapy
C atropine sulfate
D antiepileptic drug therapy
E providing more care and interest for the patient
ANSWER:E
2909
Migraine is the most frequent type of recurrent headache that is brought to the attention of
parents and primary care providers, but it remains under recognized and undertreated,
particularly in children. Migraine is characterized by all the following EXCEPT
A episodic attacks
B moderate to severe in intensity
C focal in location on the head
D constant quality
E may be associated with nausea and vomiting
ANSWER:D
2910
The aura associated with migraine is a neurologic warning that a migraine is going to occur.
Of the following, the LEAST common type of typical auras is
A sensory aura
B dysphasic aura
C visual aura
D vertigo
E distortion
ANSWER:B
2911 The following are indications for neuroimaging in a child with a headache EXCEPT
A abnormal neurologic examination
B headache worst on first awakening
C brief cough headache
D frontal headache

E migrainous headache in the child with no family history of migraine
ANSWER:D
2912 Café-au-lait macules are not specific for NF-1; they may be seen in the following EXCEPT
A Noonan syndrome
B ataxia telangiectasia
C Fanconi anemia
D Gaucher disease
E Hurler syndrome
ANSWER:E
2913 The MOST frequent lesion associated with neurofibromatosis type 2 NF-2 is
A bilateral vestibular schwannomas
B intracranial
C cataract
D retinal hamartoma
E skin plaque
ANSWER:A
2914 Slow, writhing, continuous, and involuntary movements are called
A stereotypies
B tics
C tremor
D athetosis
E chorea
ANSWER:D
2915
A slowly progressive ataxia that involves the lower extremities to a greater degree than the
upper extremities. The Romberg test result is positive; the deep-tendon reflexes are absent
(particularly at the ankle), and the plantar response is typically extensor (Babinski sign). Of
the following, the MOST likely cause of this ataxia is
A ataxia-telangiectasia
B Friedreich ataxia
C abetalipoproteinemia
D Roussy-Levy disease
E Ramsay Hunt syndrome
ANSWER:B
2916 The MOST common acquired cause of chorea in childhood is

A Huntington disease
B paraneoplastic choreas
C benign hereditary chorea
D Sydenham chorea
E chorea gravidarum
ANSWER:D
2917 Prematurity is a major cause of
A spastic diplegia
B spastic quadriplegia
C spastic hemiplegia
D athetoid CP
E dyskinetic CP
ANSWER:A
2918 The MOST severe form of CP is
A spastic diplegia
B spastic quadriplegia
C spastic hemiplegia
D athetoid CP
E dyskinetic CP
ANSWER:B
2919 All the following can be used for treatment of cerebral palsy (CP) EXCEPT
A benzodiazepines
B baclofen
C dantrolene
D botulinum toxin
E hyperbaric oxygen
ANSWER:E
2920 Infants with Tay-Sachs disease (TSD) are characterized by the following EXCEPT
A convulsions
B blindness
C deafness
D cherry-red spots
E microcephaly
ANSWER:E

2921
Excessive irritability and crying, unexplained episodes of hyperpyrexia, vomiting, and
difficulty feeding are the symptoms of the following neuro-degenerative disease
A Tay-Sachs disease (TSD)
B Sandhoff disease
C juvenile GM2 gangliosidosis
D Krabbe disease (KD)
E metachromatic leukodystrophy
ANSWER:D
2922 Presenting symptoms in pediatric multiple sclerosis (MS) include the following EXCEPT
A hemiparesis
B optic neuritis
C ataxia
D dysarthria
E encephalopathy
ANSWER:E
2923 The MOST common focal presentation of arterial ischemic stroke is
A hemiparesis
B acute visual deficit
C speech deficit
D sensory deficit
E balance deficit
ANSWER:A
2924
The MOST common cause of childhood subarachnoid and intraparenchymal hemorrhagic
stroke (HS) is
A moyamoya disease/syndrome
B arteriovenous malformations
C cerebral sinovenous thrombosis
D hemolytic uremic syndrome
E idiopathic thrombocytopenic purpura
ANSWER:B
2925 Bilateral cortical visual dysfunction, encephalopathy, and seizures are seen in
A global hypoxic–ischemic encephalopathy
B herpes encephalitis
C multiple sclerosis
D hypertensive encephalopathy

E hypoglycemia
ANSWER:D
2926 The imaging studies are usually normal in the following stroke-like disorders of children
A alternating hemiplegia
B inborn errors of metabolism
C hypertensive encephalopathy
D hypoglycemia
E global hypoxic–ischemic encephalopathy
ANSWER:A
2927 Small vessel childhood primary angiitis of the CNS (SVcPACNS) is usually presented with
A arterial ischemic stroke
B seizures
C cognitive dysfunction
D personality changes
E loss of social control
ANSWER:B
2928
The diagnosis of diffuse CNS infections depends on examination of cerebrospinal fluid
(CSF) obtained by lumbar puncture (LP). The cerebrospinal fluid (CSF) protein may reach
3,000 (mg/dL) in
A acute bacterial meningitis
B tuberculous meningitis
C fungal meningitis
D amebic (naegleria) meningoencephalitis
E subdural empyema
ANSWER:B
2929 The organism causing CNS infection is never seen on direct examination of CSF in
A tuberculous meningitis
B acute bacterial meningitis
C partially treated bacterial meningitis
D fungal meningitis
E amebic (naegleria) meningoencephalitis
ANSWER:A
2930
T-lymphocyte defects (congenital or acquired by chemotherapy, AIDS, or malignancy) are
associated with an increased risk of infections of the CNS with
A Listeria monocytogenes

B Streptococcus pneumoniae
C Neisseria meningitidis
D Haemophilus influenzae type b
E coagulase-negative Staphylococci
ANSWER:A
2931 The CSF leukocyte count in normal healthy neonates may reach
A 10 leukocytes/mm3
B 15 leukocytes/mm3
C 20 leukocytes/mm3
D 25 leukocytes/mm3
E 30 leukocytes/mm3
ANSWER:E
2932 The following is an alternative treatment for L. monocytogenes meningitis
A vancomycin
B cefotaxime
C chloramphenicol
D intravenous trimethoprimsulfamethoxazole
E meropenem
ANSWER:D
2933
Reversible gallbladder pseudolithiasis, detectable by abdominal ultrasonography during
treatment of bacterial meningitis is usually caused by
A ceftazidime
B cefotaxime
C ceftriaxone
D meropenem
E chloramphenicol
ANSWER:C
2934
Data support the use of intravenous dexamethasone, 0.15 mg/kg/dose given every 6 hr for 2
days, in the treatment of children older than 6 wk with acute bacterial meningitis caused by
H. influenza type b. for the following reasons EXCEPT
A shorter duration of fever
B lower CSF protein
C lower mortality
D lower CSF lactate levels
E reduction in sensorineural hearing loss

ANSWER:C
2935
Seizures are common during the course of bacterial meningitis. Immediate therapy for
seizures includes intravenous diazepam (0.1- 0.2 mg/kg/dose) or lorazepam (0.05-
1.10mg/kg/dose). After immediate management of seizures, patients should receive the
following anticonvulsant to reduce the likelihood of recurrence
A valproic acid
B phenytoin
C phenobarbital
D lorazepam
E carbamazepine
ANSWER:B
2936 The MOST common neurologic sequelae of bacterial meningitis is
A recurrent seizures
B delay in acquisition of language
C hearing loss
D cognitive impairment
E visual impairment
ANSWER:C
2937 The MOST common cause of viral meningoencephalitis are
A arboviruses
B enteroviruses
C herpes simplex virus (HSV) type 1
D varicella-zoster virus
E mumps
ANSWER:B
2938
The following viruses have clinical manifestations similar to that of the enteroviruses with the
exception of more severe MRI lesions of the cerebral cortex and at times an absence of a CSF
pleocytosis
A arboviruses
B parechoviruses
C herpes simplex virus (HSV) type 1
D rabies virus
E Epstein-Barr virus
ANSWER:B

2939
Detection of viral DNA or RNA by polymerase chain reaction is the test of choice in the
diagnosis of CNS infection caused by
A arboviruses
B parechoviruses
C West Nile virus (WNV)
D rabies virus
E Epstein-Barr virus
ANSWER:B
2940 The predominant organisms causing brain abscesses in children are
A aerobic and anaerobic streptococci
B Streptococcus pneumoniae
C Enterococcus faecalis
D bacteroides spp
E Haemophilus aphrophilus
ANSWER:A
2941
A brain abscess can be treated with antibiotics without surgery in the following conditions
EXCEPT
A abscess is <2 cm in diameter
B illness is of short duration (<2 wk)
C lesion is located in the posterior fossa
D no signs of increased intracranial pressure
E child is neurologically intact
ANSWER:C
2942
Idiopathic intracranial hypertension, also known as pseudotumor cerebri, is a clinical
syndrome that mimics brain tumors and is characterized by the following EXCEPT
A increased intracranial pressure ≥280 mm Hg in nonobese, nonsedated children
B normal cerebrospinal fluid (CSF) cell count and protein content
C normal to slightly decreased ventricular size
D normal ventricular anatomy and position documented by MRI
E papilledema
ANSWER:A
2943 The following are hematologic causes of childhood pseudotumor cerebri EXCEPT
A Wiskott-Aldrich syndrome
B megaloblastic anemia
C polycythemia

D sickle cell disease
E Fanconi anemia
ANSWER:B
2944 The MOST common presenting complaint of intramedullary spinal cord tumors is
A gait disturbance
B sensory deficits
C scoliosis
D back pain
E urinary urgency
ANSWER:D
2945 The MOST common involved segments in transverse myelitis (TM) are in the
A cervical region
B thoracic region
C lumbar region
D lumbo-sacral region
E sacral region
ANSWER:B
2946 Gowers sign is fully expressed by the age of
A 3 yr
B 5 yr
C 7 yr
D 9 yr
E 11 yr
ANSWER:B
2947
Duchenne muscular dystrophy (DMD) is the most common hereditary neuromuscular
disease affecting all races and ethnic groups. All the following are features of DMD EXCEPT
A scoliosis
B contracture
C fasciculation
D cardiomyopathy
E intellectual impairment
ANSWER:C

2948
The characteristic features of Duchenne muscular dystrophy (DMD) are progressive
weakness, intellectual impairment, hypertrophy of the calves, and proliferation of connective
tissue in muscle. Of the following, the BEST initial test for diagnosis is
A PCR
B muscle biopsy
C electromyography
D serum creatine kinase
E nerve conduction study
ANSWER:D
2949
In Duchenne muscular dystrophy (DMD), ambulation is important not only for postponing
the psychologic depression but also for postponding
A lordosis
B scoliosis
C kyphosis
D ankles contracture
E pseudohypertrophy
ANSWER:B
2950
Becker muscular dystrophy (BMD) is a disease that is fundamentally similar to Duchenne
muscular dystrophy (DMD). Of the following, the MOST recognized feature of BMD is
A cardiomyopathy
B longer ambulation time
C calf pseudohypertrophy
D elevated creatine kinase
E genetic defect at the Xp21.2 locus
ANSWER:B
2951
Emery-Dreifuss Muscular Dystrophy (scapuloperoneal or scapulohumeral muscular
dystrophy) is a rare X-linked recessive dystrophy. Of the following, the MOST characteristic
feature is
A myotonia
B facial weakness
C pseudohypertrophy
D intellectual impairment
E dilated cardiomyopathy
ANSWER:E

2952
Myotonic muscular dystrophy (Steinert disease) is the second most common muscular
dystrophy in North America, Europe, and Australia, inherited as an autosomal dominant
trait. Of the following, the LEAST recognized feature is
A cataract
B myotonia
C low serum IgG
D cardiomyopathy
E intellectual impairment
ANSWER:D
2953 All the following myopathies are characterized by proximal muscle wasting EXCEPT
A Becker dystrophy
B myotonic dystrophy
C Duchenne dystrophy
D central core myopathy
E hypothyroid myopathy
ANSWER:B
2954
Limb-girdle muscular dystrophies (LGMDs) are a heterogeneous group of progressive
hereditary muscular dystrophies that mainly affect muscles of the hip and shoulder girdles.
Of the following, the presenting complaint may be
A low back pain
B cardiomyopathy
C difficulty in swallowing
D characteristic facies
E intellectual impairement
ANSWER:A
2955
The term congenital muscular dystrophy is misleading because all muscular dystrophies are
genetically determined. A distinguishing feature of the congenital dystrophies from other
muscular dystrophies, is a high association with malformations of
A liver
B lung
C brain
D bone
E kidney
ANSWER:C
2956 All the following may induce myopathy EXCEPT

A steroid use
B hypothyroidism
C hyperthyroidism
D hyperaldosteronism
E hypoparathyroidism
ANSWER:E
2957 In periodic paralysis, all the following are precipitating factors EXCEPT
A heavy fat meal
B licorice ingestion
C emotional stress
D hyperthyroidism
E amphotericin B use
ANSWER:A
2958 In spinal muscular atrophy (SMA) type 1, all the following are spared EXCEPT
A heart
B diaphragm
C intelligence
D anal sphincter
E extraocular muscles
ANSWER:B
2959
Muscle denervation is any loss of nerve supply regardless of the cause. Of the following, the
MOST specific clinical sign of denervation is
A myotonia
B weakness
C fatigability
D fasciculation
E abnormal sensation
ANSWER:D
2960 The MOST definitive diagnostic test in Werdnig-Hoffmann disease is
A EMG
B muscle biopsy
C serum creatine kinase
D molecular genetic test
E motor nerve conduction study
ANSWER:D

2961
InGuillain-Barrésyndrome,theparalysisusuallyfollowsanonspecific gastrointestinal or
respiratory infection by approximately 10 days. Of the following, the MOST likely
respiratory infection that triggers the disease is
A Chlamydia trachomitis
B Staphylococcal aureus
C Haemophilus influenzae
D Mycoplasma pneumoniae
E Streptococcal pneumonia
ANSWER:D
2962 All the following vaccines may induce Guillain-Barré syndrome (GBS) EXCEPT
A OPV
B Rota
C Rabies
D Influenza
E Conjugated meningococcal vaccine
ANSWER:B
2963
In Guillain-Barré syndrome, the onset is gradual and progresses over days or weeks. The
maximal severity of weakness is usually reached by
A 2 wk
B 4 wk
C 6 wk
D 8 wk
E 10 wk
ANSWER:B
2964
In Guillain-Barré syndrome, respiratory effort must be monitored to prevent respiratory
failure and respiratory arrest. Of the following, the MOST recognized sign of impending
respiratory failure is
A dysphagia
B tachypnea
C tachycardia
D altered sensorium
E vasomotor instability
ANSWER:A

2965
Miller-Fisher syndrome (MFS) consists of acute external and occasionally internal
ophthalmoplegia, ataxia, and areflexia. Of the following, the MOST likely cranial nerve
involved is
A 3rd
B 4th
C 5th
D 6th
E 7th
ANSWER:D
2966 The least significant finding in Miller-Fisher syndrome (MFS) is
A ataxia
B areflexia
C lower limbs weakness
D internal ophthalmoplegia
E external ophthalmoplegia
ANSWER:C
2967
uillain Barré syndrome is an autoimmune disorder often considered a postinfectious
polyneuropathy involving mainly motor but also sensory and sometimes autonomic nerves.
All the following are required in diagnosis EXCEPT
A CSF study
B muscle biopsy
C electromyography
D sural nerve biopsy
E motor nerve conduction study
ANSWER:B
2968 The best diagnostic test in Guillain-Barré syndrome (GBS) is
A CSF study
B electromyography
C sural nerve biopsy
D serum creatine kinase
E motor nerve conduction study
ANSWER:A
2969
Patients in early stages of Guillain-Barré syndrome should be admitted to the hospital for
observation because the ascending paralysis can rapidly involve respiratory muscles during
the next 24 hr. Of the following, the MOST effective treatment is

A steroids
B plasmapheresis
C antibiotics for C. jejuni
D immunosuppressive drugs
E intravenous immunoglobulin (IVIG)
ANSWER:E
2970 The last function to be recovered in Guillain-Barré syndrome (GBS) is
A sensation
B bulbar muscles
C anal sphincters
D tendon reflexes
E respiratory muscles
ANSWER:D
2971
Bell palsy is an acute unilateral peripheral facial nerve palsy that is not associated with other
cranial neuropathies or brainstem dysfunction. Of the following, the MOST traditional
treatment is
A acyclovir
B prednisone
C laser therapy
D physiotherapy
E surgical decompression
ANSWER:B
2972
Bell palsy usually develops abruptly about 2 wk after a systemic viral infection. Of the
following, the MOST common viral cause is
A Mumps virus
B Cytomegalovirus
C Epstein-Barr virus
D Herpes simplex virus
E Human herpes virus 6
ANSWER:D
2973
Charcot-Marie-Toothdisease,themostcommongeneticallydetermined neuropathy. Of the
following, the MOST severely affected nerve is
A radial
B facial
C femoral

D peroneal
E mandibular
ANSWER:D
2974 All the following are preserved in Charcot-Marie-Tooth disease EXCEPT
A intelligence
B tibial nerves
C axial muscles
D cranial nerves
E anal sphincter
ANSWER:B
2975 Of the following, the LEAST recognized presentation of Charcot-Marie-Tooth disease is
A foot drop
B claw hand
C pes cavus deformities
D stork-like contour legs
E tripping over their own feet
ANSWER:B
2976
Myasthenia gravis is a chronic autoimmune disease of neuromuscular blockade. Of the
following, the MOST characteristic feature is
A fatigue
B myalgia
C fasciculation
D sensory symptom
E abnormal pupillary response
ANSWER:A
2977
Myasthenia gravis should be differentiated from other causes of neuromuscular blockade as
organophosphate chemicals, botulism, and tick paralysis. Of the following, the MOST
specific diagnostic test is
A muscle biopsy
B electromyography
C anti-AChR antibodies
D serum creatine kinase
E nerve conduction study
ANSWER:B
2978 The earliest and MOST constant sign of myasthenia gravis is

A ptosis
B dysphagia
C slurred speech
D difficult chewing
E poor head control
ANSWER:A
2979
The Creatine kinase (CK) which is found in only 3 organs and may be separated into
corresponding isozymes: MM for skeletal muscle, MB for cardiac muscle, and BB for brain.
Of the following, the CK level is characteristically elevated in
A Myasthenia gravis
B uillain-Barré syndrome
C Werdnig-Hoffmann disease
D Duchenne muscular dystrophy
E Emery-Dreifuss Muscular Dystrophy
ANSWER:D
2980
Myotubular myopathy, maturational arrest of fetal muscle during the myotubular stage of
development at 8-15 wk of gestation. All the following are a characteristic features EXCEPT
A high arch palate
B cardiomyopathy
C polyhydramnios
D undescended testes
E decrease fetal movement
ANSWER:B
2981 Dolichocephalic head is a feature of
A Central core myopathy
B Myotubular myopathy
C Nemaline rod myopathy
D Becker muscular dystrophy
E Emery-Dreifuss Muscular Dystrophy
ANSWER:C
2982
Malignant hyperthermia is an acute hypermetabolic syndrome that is triggered by
inhalational anesthetic agents and succinylcholine. Of the following, which myopathy is
consistently associated with malignant hyperthermia?
A Central core myopathy

B Myotubular myopathy
C Nemaline rod myopathy
D Becker muscular dystrophy
E Emery-Dreifuss Muscular Dystrophy
ANSWER:A
2983
Congenital Guillain-Barré syndrome, manifest as generalized hypotonia, weakness, and
areflexia in an affected neonate, fulfilling all electrophysiologic and CSF criteria and in the
absence of maternal neuromuscular disease. Of the following, the MOST effective treatment
is
A steroids
B reassurance
C plasmapheresis
D immunosuppressive drugs
E intravenous immunoglobulin (IVIG)
ANSWER:B
2984
All the following are features of chronic inflammatory demyelinating
polyradiculoneuropathies (CIDPs) EXCEPT
A motor deficits in 94%
B sensory paresthesias in 64%
C cranial nerve involvement in <30%
D CSF shows pleocytosis and high protein
E hyporeflexia or areflexia is almost universal
ANSWER:D
2985 Persistent deviation of an eye in an infant requires evaluation at the age of
A 1 mo
B 3 mo
C 6 mo
D 9 mo
E 12 mo
ANSWER:C
2986 The MOST widely used visual acuity test for preschool children is
A Tumbling E test
B Snellen letters
C Lea symbols
D Allen figures

E HOTV test
ANSWER:A
2987
Children may have problems with going to sleep in a dark room (which may be mistaken
for a behavioral problem) Of the following, the MOST likely cause is
A dyslexia
B diplopia
C nyctalopia
D amaurosis
E amblyopia
ANSWER:C
2988 The MOST common cause of a dilated unreactive pupil is
A internal ophthalmoplegia
B tonic pupil
C ocular trauma
D pharmacologic blockade
E Hutchinson pupil
ANSWER:D
2989
Children with aniridia should be screened using renal ultrasonography every 3-6 mo until
approximately 5 yr of age if there is an 11p13 region deletion because the child is at risk for
A rhabdomyosarcoma
B neuroblastoma
C Wilms tumor
D gonadoblastoma
E lymphoblastic leukemia
ANSWER:C
2990 Horner syndrome is an important cause of
A dyscoria
B corectopia
C microcoria
D aniridia
E anisocoria
ANSWER:E
2991 Waardenburg syndrome is characterized by all the following EXCEPT
A an autosomal dominant inheritance

B lateral displacement of the inner canthi
C median white forelock
D defective hearing
E hyperpigmentation of the skin
ANSWER:E
2992 Cat’s-eye reflex is seen in all the following EXCEPT
A juvenile xanthogranuloma
B persistent hyperplastic primary vitreous
C cicatricial retinopathy of prematurity
D retinoschisis
E larval granulomatosis
ANSWER:A
2993 Hypertropia is seen in
A 3rd nerve palsy
B 4th nerve palsy
C 5th nerve palsy
D 6th nerve palsy
E 7th nerve palsy
ANSWER:B
2994 Bilateral facial palsy is a distinctive feature of
A Möbius syndrome
B Brown syndrome
C Parinaud syndrome
D Duane syndrome
E Bardet-Biedl syndrome
ANSWER:A
2995
Spasmus nutans is a special type of acquired nystagmus in childhood. In its complete form,
it is characterized by the triad of head nodding, torticollis and
A pendular nystagmus
B latent nystagmus
C Seesaw nystagmus
D downbeat nystagmus
E gaze-paretic nystagmus
ANSWER:A
2996 All the following are causes of blepharitis EXCEPT

A Molluscum virus
B Phthirus pubis
C Staphylococcus epidermidis
D Staphylococcus aureus
E Streptococci
ANSWER:E
2997
All the following are options in the treatment of congenital nasolacrimal duct obstruction
EXCEPT
A nasolacrimal massage
B cleansing of the lids with cold water
C topical antibiotic
D systemic antibiotics
E probing with topical anesthesia
ANSWER:B
2998 All the following agents are used as prophylaxis to prevent neonatorum ophthalmia EXCEPT
A 0.5% erythromycin drops
B 1% silver nitrate drops
C povidone iodine (2% solution)
D single dose of ceftriaxone
E saline irrigation
ANSWER:E
2999 Cicatricial adhesion between the conjunctiva of the lid and the globe is called
A dermoid cyst
B pterygium
C pinguecula
D symblepharon
E parinaudoculoglandular syndrome
ANSWER:D
3000
Keratoconus is characterized by progressive thinning and bulging of the central cornea (cone
shaped), which of the following is a sign of keratoconus?
A Munson sign
B Dalrymple sign
C Stellwag sign
D von Graefe sign

E Pseudohypopyon sign
ANSWER:A
3001 Epibulbar dermoids are choristomas found in 75% of
A Marfan syndrome
B craniosynostosis
C Alport syndrome
D Goldenhar syndrome
E Cogan syndrome
ANSWER:D
3002 All the following are options in the treatment of dendritic keratitis EXCEPT
A trifluridine
B topical ganciclovir
C systemic acyclovir
D cycloplegic agent
E topical corticosteroids
ANSWER:E
3003
The MOST serious organism which can rapidly destroy stromal tissue and lead to corneal
perforation is
A Neisseria gonorrhoea
B Pseudomonas aeruginosa
C Staphylococcus
D Streptococcus
E Yersinia
ANSWER:B
3004
Several metabolic diseases produce distinctive corneal changes in childhood. Fine opacities
radiating in a whorl or fan-like pattern is seen in
A cystinosis
B mucopolysaccharidosis
C gangliosidosis
D Fabry disease
E Wilson disease
ANSWER:D
3005
The ectopia lentis (displacement of the lens) is often downward and forward, and the lens
tends to be small and round in
A Marfan syndrome

B Homocystinuria
C Weill-Marchesani syndrome
D Sulfite oxidase deficiency
E Ehlers-Danlos syndrome
ANSWER:C
3006 Anterior uveitis is caused by
A toxoplasmosis
B toxocariasis
C cytomegalovirus
D rubella
E brucellosis
ANSWER:E
3007
The risk factors associated with retinopathy of prematurity (ROP) are not fully known, but
prematurity and the associated retinal immaturity at birth represent the major factors.
Contributory factors include all the following EXCEPT
A oxygenation
B apnea
C heart disease
D hypercarbia
E polycythemia
ANSWER:E
3008
Retinopathy of prematurity (ROP) are classified into 5 stages. The stage which is
characterized by the presence of a ridge and development of extraretinal fibrovascular tissue
is
A stage 1
B stage 2
C stage 3
D stage 4
E stage 5
ANSWER:C
3009
Retinoblastoma is the most common primary malignant intraocular tumor of childhood. It is
unusual for a child to present with a retinoblastoma after the age of
A 1 yr
B 2 yr
C 3 yr

D 4 yr
E 5 yr
ANSWER:C
3010 The initial sign in the majority of patients with retinoblastoma is
A strabismus
B leukocoria
C pseudohypopyon
D hyphema
E vitreous hemorrhage
ANSWER:B
3011
During the last decade there has been a dramatic shift in the treatment of retinoblastomas, by
markedly reduced use of
A external beam radiation
B systemic chemotherapy
C laser therapy
D cryotherapy
E brachytherapy
ANSWER:A
3012
Cherry-red spots (a bright to dull red spot at the center of the macula surrounded and
accentuated by a grayish white or yellowish halo) are seen in all the following EXCEPT
A Tuberous sclerosis
B Tay-Sachs disease
C Sandhoff disease
D Sulfatide lipidosis
E Niemann-Pick disease
ANSWER:A
3013 Phakomas are seen in all the following EXCEPT
A Sturge-Weber syndrome
B von Hippel-Lindau disease
C von Recklinghausen disease
D Niemann-Pick disease
E Tuberous sclerosis
ANSWER:D
3014 The first sign in hypertensive retinopathy is
A retinal edema

B flame-shaped hemorrhages
C cotton-wool spots
D papilledema
E irregular narrowing of the arteriole
ANSWER:E
3015 Retinopathy in subacute bacterial endocarditis is present in approximately
A 5% of cases
B 20% of cases
C 40% of cases
D 60% of cases
E 80% of cases
ANSWER:C
3016
Vascular tortuosity, arterial and venous occlusions, “salmon patches,” refractile deposits,
pigmented lesions, arteriolarvenous anastomoses, and neovascularization (with “sea-fan”
formations); is retinopathy of
A Iron deficiency anemia
B polycythemia vera
C leukemia
D sickling disorders
E β-thalassemia
ANSWER:D
3017
Optic neuritis is inflammation or demyelinization of the optic nerve with attendant
impairment of function, all the following may cause optic neuritis EXCEPT
A lead poisoning
B chloramphenicol
C vincristine
D methylprednisolone
E hydroxychloroquine
ANSWER:D
3018 The symptoms of infantile glaucoma include the classic triad of
A photophobia ,conjunctival injection, and blepharospasm
B tearing, photophobia , and blepharospasm
C tearing, photophobia , and ocular enlargement
D corneal enlargement, photophobia , and blepharospasm
E tearing, corneal edema , and blepharospasm

ANSWER:B
3019 Of the following, the MOST common benign tumor of the orbit is
A hemangioma
B rhabdomyosarcoma
C lymphosarcoma
D metastatic neuroblastoma
E teratoma
ANSWER:A
3020 Treatment of hyphema include all the following EXCEPT
A bed rest
B nonsteroidal anti-inflammatory drugs
C cycloplegic agent
D topical steroids
E systemicsteroids
ANSWER:B
3021 Purulent otorrhea is a sign of all the following conditions EXCEPT
A otitis externa
B otitis media with perforation of the tympanic membrane
C drainage from the middle ear through a patent tympanostomy tube
D skull base fracture
E drainage from first branchial cleft sinus
ANSWER:D
3022 Bloody otorrhea is associated with all the following EXCEPT
A neoplasm
B trauma
C foreign body
D branchial cleft sinus
E blood dyscrasia
ANSWER:D
3023 The MOST common cause of dizziness in young children is
A cholesteatoma in the mastoid or middle ear
B eustachian tube middle ear disease
C labyrinthitis
D vestibular neuronitis
E benign paroxysmal vertigo

ANSWER:B
3024 The MOST common infectious cause of congenital sensorineural hearing loss is
A Measles virus
B Cytomegalovirus
C Toxoplasma gondii
D Rubella virus
E Treponema pallidum
ANSWER:B
3025
The MOST common cause of bacterial meningitis that results in sensorineural hearing loss
after the neonatal period is
A H. influenza
B E. Coli
C Streptococcus pneumoniae
D Listeria monocytogenes
E Staphylococcus aureus
ANSWER:C
3026
Of the following, the congenital infectious pathogen implicated in sensorineural hearing loss
in children is
A Measles virus
B Mumps virus
C Streptococcus pneumoniae
D Lymphocytic choriomeningitis virus
E H. influenza
ANSWER:D
3027 All the following are causes of sudden sensorineural hearing loss EXCEPT
A autoimmune disease
B Epstein Barr virus infection
C thromboembolic event
D trauma
E Rubella virus infection
ANSWER:E
3028 All the following are included in the normal flora of the external ear canal EXCEPT
A Coagulase negative staphylococcus
B Micrococcus
C Diphtheroids

D Pseudomonas aeruginosa
E E. coli
ANSWER:E
3029 Of the following, the major trigger for otitis externa is
A eczema
B trauma
C excessive wetness
D foreign body
E previous infection
ANSWER:C
3030 The MOST common cause of otitis externa is
A Pseudomonas aeruginosa
B Enterobacter aerogenes
C Proteus mirabilis
D Streptococci
E Diphtheroids
ANSWER:A
3031 The predominant symptom of otitis externa is
A tenderness by pressure on the tragus
B otalgia
C itching
D hearing loss
E otorrhea
ANSWER:B
3032 The prominent sign of acute otitis externa is
A white cerumen
B swollen and tender canal
C opaque tympanic membrane
D otorrhea
E periauricular lymphadenopathy
ANSWER:D
3033 Necrotizing malignant otitis externa is probable with the finding of
A white cerumen
B swollen and tender canal
C facial paralysis

D otorrhea
E periauricular lymphadenopathy
ANSWER:C
3034 The MOST common causative organism of necrotizing otitis externa is
A Pseudomonas aeruginosa
B Enterobacter aerogenes
C Proteus mirabilis
D Streptococci
E Diphtheroids
ANSWER:A
3035
A major physical examination point to differentiate externa otitis from mastoiditis and otitis
media is
A visualization of tympanic membrane
B pus discharge from ear canal
C pain on manipulation of the auricle
D periauricular lymphadenopathy
E hearing loss
ANSWER:C
3036 Of the following, the treatment of choice for otomycosis is topical
A polymyxin
B ciprofloxacin
C clotrimazole
D hydrocortisone
E neomycin
ANSWER:C
3037 The MOST effective prophylaxis for recurrent otitis externa is
A instillation of dilute alcohol immediately after bathing
B avoidance of swimming during the episode
C ear protection
D topical neomycin
E use of hair dryer after swimming
ANSWER:A
3038 The peak incidence of otitis media (OM) is in the age of
A 1-2 yr of life
B 4-5 yr of life

C 6-7 yr of life
D 9-11 yr of life
E 13-15 yr of life
ANSWER:A
3039 The leading reason for physician visits and for use of antibiotics among children is
A urinary tract infection
B lower respiratory tract infection
C otitis media
D infectious gastroenteritis
E lymphadenitis
ANSWER:C
3040 The mainstay of pain management for acute otitis media is
A acetaminophen
B topical lidocaine
C tympanostomy
D myringotomy
E narcotic analgesics
ANSWER:A
3041 An important predictor for the development of recurrent and chronic otitis media is
A gender
B race
C age
D genetic background
E bottle feeding
ANSWER:C
3042 The MOST likely reason for the higher rate of otitis media in infants is
A early exposure to unusual organisms
B bottle feeding
C oral appreciation of infancy
D less developed immunologic defenses
E high incidence of respiratory tract infections
ANSWER:D
3043 Middle ear aspirates in children with bronchiolitis regularly contain
A Respiratory syncytial virus
B Parainfluenza virus

C Streptococcus pneumonia
D Pseudomonas aeruginosa
E Staphylococcus aureus
ANSWER:C
3044
Using standard culture techniques, the pathogens typically found in acute otitis media with
effusion are recoverable in
A 10%
B 30%
C 50%
D 70%
E 90%
ANSWER:B
3045
The reason behind progressive decline in the occurrence of otitis media as children grow
older is
A frequent exposure to the causative organisms
B improved immune response
C progressive reduction in eustachian tube wall compliance
D the more-use of over-the counter medications
E reduced incidence of respiratory tract infections
ANSWER:C
3046 Otitis media is a universal finding in
A Down syndrome
B cleft lip
C Patau syndrome
D Ig A deficiency
E cleft palate
ANSWER:E
3047 Symptoms of acute otitis media include the following EXCEPT
A irritability
B change in sleep habit
C upper respiratory tract symptoms
D fever
E facial pain
ANSWER:E
3048 Of the following, the LEAST sensitive and specific symptom of acute otitis media is

A pulling at the ear
B pain
C pus discharge
D hearing loss
E fullness in the ear
ANSWER:A
3049 The MOST sensitive finding of tympanic membrane in determining middle ear effusion is
A contour
B translucency
C color
D mobility
E structural changes
ANSWER:D
3050 The MOST common finding of the tympanic membrane in middle ear effusion is
A bulging
B impairment of mobility
C opacification
D white color
E reduced translucency
ANSWER:B
3051 The MOST specific finding of the tympanic membrane in acute otitis media is
A bulging
B impairment of mobility
C perforation
D pale color
E reduced translucency
ANSWER:A
3052 Purulent otorrhea of a recent onset is indicative of
A acute otitis externa
B chronic otitis externa
C acute otitis media
D chronic otitis media
E otitis media with effusion
ANSWER:C
3053 The following features specify non-typeable H. influenza otitis media EXCEPT

A purulent conjunctivitis
B often occurs in multiple family members
C often affects infants
D ineffective topical ocular antibiotics
E bulging of tympanic membrane
ANSWER:E
3054 ‘’Watchful waiting’’ and analgesics use in otitis media treatment is the right choice in
A a four-month-old infant having presumed diagnosis of acute otitis media
B a three-year-old girl having unilateral acute otitis media without otorrhea
C a one-year-old infant having bilateral acute otitis media with otorrhea
D a twelve-year-old girl having unilateral acute otitis media and fever of 390 c
E aneighteen-month-oldboyhavingbilateralacuteotitismediawithout otorrhea
ANSWER:B
3055 The MOST crucial aspect in the treatment of acute otitis media is
A choosing the right antibiotic
B skillful examination of the ear
C culture of middle ear aspirate
D accurate diagnosis
E analgesics use
ANSWER:D
3056 Of the following, the recommended first line treatment of acute otitis media is
A amoxicillin
B cefdinir
C cefuroxime
D clindamycin
E cefpodoxime
ANSWER:A
3057 All the following are at greatest risk of harboring resistant bacteria in otitis media EXCEPT
A those younger than 2 years of age
B those in regular contact with large group of children
C those who recently have received antimicrobial treatment
D those who have bilateral otitis media with toxicity
E those who are immune compromised
ANSWER:D
3058 Second line treatment for acute otitis media includes all the following agents EXCEPT

A trimethoprim
B cefdinir
C amixicillin-clavulanate
D cefuroxime
E ceftriaxone
ANSWER:A
3059
Indications for myringotomy in children with acute otitis media include all the following
EXCEPT
A severe refractory pain
B hyperpyrexia
C facial paralysis
D immune compromised
E failure of first course of antibiotics
ANSWER:E
3060 The MOST common etiologic organism behind chronic suppurative otitis media is
A Klebseilla
B Proteus
C Pseudomonas
D E. Coli
E Candida
ANSWER:C
3061 Gradenigo syndrome is the triad of
A
suppurative otitis media, paralysis of the internal rectus muscle, and pain in the contralateral
orbit
B
suppurative otitis media, paralysis of the external rectus muscle, and pain in the ipsilateral
orbit
C acute otitis media, paralysis of the external rectus muscle, and pain in the ipsilateral orbit
D acute otitis media, paralysis of the external rectus muscle, and pain in the contralateral orbit
E
suppurative otitis media, paralysis of the internal rectus muscle, and pain in the ipsilateral
orbit
ANSWER:B
3062 Bezold abscess refers to
A neck abscess that is originated from mastoiditis involving the temporal bone
B mastoid bone abscess that is originated from mastoiditis

C orbital cavity abscess that is originated from mastoiditis involving the temporal bone
D brain abscess that is originated from mastoiditis involving the temporal bone
E paranasal abscess that is originated from mastoiditis
ANSWER:A
3063 All the following are common organisms in all variants of acute mastoiditis EXCEPT
A S. Pneumoniae
B non-typable H. Influenza
C Pseudomonas aeruginosa
D Klebsiella spp.
E group A Streptococcus
ANSWER:D
3064 Otitic hydrocephalus (a complication of chronic otitis media) is commonly associated with
A meningitis
B lateral sinus thrombosis
C subdural abscess
D brain abscess
E facial paralysis
ANSWER:B
3065 The apocrine glands are located in the following areas EXCEPT
A axillae
B upper chest
C areolae
D perianal
E genital
ANSWER:B
3066 Nail growth is relatively slow; complete fingernail regrowth takes
A 2 mo
B 4 mo
C 6 mo
D 8 mo
E 10 mo
ANSWER:C
3067 The presenting symptom of Behçet disease is
A erythema nodosum
B recurrent aphthous stomatitis

C genital ulcerations
D perianal ulceration
E purpuric lesions
ANSWER:B
3068 One of the following may support a diagnosis of drug eruption
A neutrophilia
B basophilia
C eosinophilia
D lymphocytosis
E monocytosis
ANSWER:C
3069
In the use of topical medication, consideration of vehicle is as important as the specific
therapeutic agent. Acute weeping lesions respond best to
A ointments
B gels
C solutions
D wet compresses
E pastes
ANSWER:D
3070
Topical antibiotics have been used for many years to treat local cutaneous infections. Of the
following, the MOST effective topical agents currently available is
A polysporin
B bacitracin
C fusidic acid
D gentamycin
E tetracycline
ANSWER:C
3071
Topical corticosteroids are potent anti-inflammatory agents and effective antipruritic agents.
Medium-potency topical corticosteroids include
A betamethasone
B fluocinonide
C halcinonide
D clobetasol
E hydrocortisone butyrate
ANSWER:A

3072
Systemic adverse effects of high-potency and superpotent topical steroids occur with long-
term use and include
A telangiectasia
B acneiform eruptions
C purpura
D cataracts
E increased hair growth
ANSWER:D
3073
A 3-day-old neonate presented with numerous firm, yellow-white, 1-2 mm papules and
pustules with a surrounding erythematous flare in several sites of the body surface but the
palms and soles were spared. Intralesional contents demonstrated eosinophils in Wright-
stained smears and culture was sterile. Of the following, the MOST likely diagnosis is
A pyoderma
B erythema toxicum
C candidiasis
D herpes simplex
E transient neonatal pustular melanosis
ANSWER:B
3074
Redundant skin over the posterior part of the neck is common in the following syndromes
EXCEPT
A Ehlers-Danlos
B Turner
C Noonan
D Down
E Klippel-Feil
ANSWER:A
3075 Hypohidrotic ectodermal dysplasia is characterized by the following EXCEPT
A dry skin
B sparse scalp hair
C normal sexual hair growth
D anodontia
E delayed sexual development
ANSWER:E
3076
Capillary malformations (CMs) are present at birth. The most effective treatment for CM is
with the pulsed-dye laser. Therapy can begin

A in infancy
B after 5 yr
C after 10 yr
D during puberty
E after puberty
ANSWER:A
3077
A 3-day-old neonate presented with unvarying red purple hue restricted to left lower limb
with atrophic underlying subcutaneous tissue. The lesions become more pronounced during
changes in environmental temperature, physical activity, or crying. Of the following, the
MOST likely diagnosis is
A reticulate capillary malformation
B cutis marmorata telangiectatica congenital
C harlequin color change
D cutis marmorata
E cutis verticis gyrate
ANSWER:B
3078
A 25-day-old neonate presented with bright red, protuberant, compressible, and sharply
demarcated lesion on right cheek. Of the following, the MOST likely diagnosis is
A strawberry hemangioma
B cavernous hemangioma
C superficial infantile hemangioma
D deep infantile hemangioma
E tufted angioma
ANSWER:C
3079 All the following treatments can be used in Kasabach-Merritt phenomenon EXCEPT
A corticosteroids
B aminocaproic acid
C vincristine
D interferon-α
E propranolol
ANSWER:E
3080
In the usual patient with infantile hemangiomas (IH) who has no serious complications or
extensive growth resulting in tissue destruction and severe disfigurement, treatment consists
of
A expectant observation

B pulsed-dye laser therapy
C topical timolol solution
D oral propranolol
E oral corticosteroids
ANSWER:A
3081
In a disfiguring, life or vision threatening, or ulcerated infantile hemangiomas IH; the first-
line treatment is
A oral corticosteroids
B oral propranolol
C intralesional corticosteroid injection
D vincristine
E interferon-α
ANSWER:B
3082
Evaluate for PHACES in any patient with facial hemangioma involving significant area of
face, one of the following is not a feature of PHACES
A posterior fossa abnormalities
B arterial cerebrovascular abnormalities
C coarctation of the aorta
D ear abnormalities
E sternal abnormalities
ANSWER:D
3083
Ataxia-telangiectasia is transmitted as an autosomal recessive trait; the characteristic
telangiectasias develop at approximately 3 yr of age, first on
A bulbar conjunctivae
B nasal bridge
C malar areas
D external ears
E upper anterior chest
ANSWER:A
3084
The association of spindle cell hemangiomas with nodular enchondromas in the metaphyseal
or diaphyseal cartilaginous portion of long bones is known as
A Angiokeratoma Corporis Diffusum
B Osler-Weber-Rendu Disease
C Angiokeratoma of Mibelli
D Kaposiform Hemangioendothelioma

E Maffucci syndrome
ANSWER:E
3085 All the following are risk factors for development of melanoma EXCEPT
A xeroderma pigmentosum
B dark complexion
C excessive sun exposure
D giant congenital nevus
E immunosuppression
ANSWER:B
3086
An 8-year-old female child presented with a solitary, asymptomatic, smooth, dome shaped,
blue-gray papule 8 mm in diameter on the dorsal aspect of her hand. Of the following, the
MOST likely diagnosis is
A nevus spilus
B nevus of ota
C Spitz nevus
D blue nevus
E epidermal nevus
ANSWER:D
3087
The Peutz-Jeghers syndrome is characterized by melanotic macules on the lips and mucous
membranes and by gastrointestinal (GI) polyposis. Of the following, The MOST constant
feature is
A diffuse hyperpigmentation of the nails
B buccal mucosal macules
C jejunal polyposis
D melena
E episodic abdominal pain
ANSWER:B
3088 The following syndrome has a strong association with Café-au-lait spots
A Turner syndrome
B McCune-Albright syndrome
C Russell-Silver syndrome
D Noonan syndrome
E Rubinstein-Taybi syndrome
ANSWER:B
3089 The hallmark of Waardenburg type 1 is the

A white forelock
B depigmented skin
C deafness
D heterochromia irides
E unibrow (synophrys)
ANSWER:A
3090 The following is a feature of generalized (nonsegmental) vitiligo
A often occurs in the face
B usually not accompanied by other autoimmune diseases
C progressive, with flare-ups
D involves hair compartment soon after onset
E usually responsive to autologous grafting
ANSWER:C
3091 The MOST common agent implicated in the etiology of erythema multiforme (EM) is
A Mycoplasma pneumonia
B Herpes simplex virus (HSV) labialis
C Herpes simplex virus HSV genitalis
D Streptococcus pneumonia
E Staphylococcus epidermidis
ANSWER:B
3092
Erythema multiforme (EM) is characterized by an abrupt, symmetric cutaneous eruption,
MOST commonly on the
A face
B trunk
C legs
D palms and soles
E extensor upper extremities
ANSWER:E
3093 Stevens-Johnson syndrome is defined as affected body surface area
A <5%
B <10%
C <15%
D <20%
E <30%
ANSWER:B

3094 All the following are advised in the treatment of Stevens-Johnson syndrome EXCEPT
A topical ocular steroid
B topical oral anesthetics
C systemic antibiotics for documented urinary or cutaneous infections
D prophylactic systemic antibiotics
E IV immunoglobulin
ANSWER:D
3095 Toxic epidermal necrolysis is characterized by the following EXCEPT
A widespread blister formation
B skin tenderness
C target lesions
D sudden onset
E confluent erythema
ANSWER:C
3096
Severemucosalblistering;GIinvolvement;laryngealinvolvement(airway obstruction); and
urologic involvement are the extracutaneous clinical features of
A epidermolysis bullosa simplex– generalized (AD)
B epidermolysis bullosa simplex–Dowling- Meara (AD)
C junctional epidermolysis bullosa–herlitz (AR)
D dominant dystrophic epidermolysis bullosa (AD)
E recessive dystrophic epidermolysis bullosa–hallopeau- siemens (AR)
ANSWER:C
3097 Pemphigus vulgaris (PV) is a rare autoimmune blistering disorder, best treated initially with
A IVIG
B methotrexate
C azathioprine
D cyclophosphamide
E systemic methylprednisolone
ANSWER:E
3098 Dermatitis herpetiformis eruption is characterized by the following EXCEPT
A symmetric
B intensely pruritic
C mostly on knees and elbows
D usual mucous membranes involvement

E occasional hemorrhagic lesions on the palms and soles
ANSWER:D
3099 Most cases of drug-induced linear IgA dermatosis are related to
A vancomycin
B anticonvulsants
C ampicillin
D cyclosporine
E captopril
ANSWER:A
3100 Diaper dermatitis can be treated with following measures EXCEPT
A frequent changing of the diapers
B overwashing
C application of a protective barrier agent
D application of topical sucralfate
E application of 2.5% topical hydrocortisone
ANSWER:B
3101 One of the MOST common causes of allergic contact dermatitis is
A neomycin
B topical antihistamines
C topical anesthetics
D topical corticosteroids
E ethylenediamine
ANSWER:A
3102 Pityriasis alba occurs mainly in children and causes lesions that are
A hyperpigmented
B well-defined
C severely erythematous
D finely scaly
E intensely pruritic
ANSWER:D
3103
Seborrheic dermatitis is a chronic inflammatory disease most common in infancy and
adolescence, it is characterized by the following EXCEPT
A manifest in 1st mo of life
B self-resolves by 1 yr
C cradle cap

D pruritis
E post inflammatory pigmentary changes
ANSWER:D
3104 Initial management for infantile seborrheic dermatitis is
A conservative
B low-potency topical corticosteroids
C topical antifungal
D topical calcineurin inhibitors
E oral antifungal agents
ANSWER:A
3105 Phototoxic drug eruptions can be caused by
A nalidixic acid
B tetracyclines
C chlorothiazides
D sulfonamides
E barbiturates
ANSWER:A
3106
The MOST common photosensitive reaction seen in children is acute sunburn, effective
treatment of the desquamative phase is with
A topical corticosteroids
B bland emollient
C cool compresses
D aloe vera products
E calamine lotion
ANSWER:B
3107 Guttate psoriasis can be confused with the following EXCEPT
A viral exanthems
B secondary syphilis
C nummular dermatitis
D pityriasis rosea
E pityriasis lichenoides chronica (PLC)
ANSWER:C
3108 The treatment of psoriasis should be viewed as a 4-tier process. The second tier of therapy is
A phototherapy

B high-potency corticosteroids
C tar preparations
D methotrexate
E infliximab
ANSWER:A
3109
The generalized eruption of pityriasis rosea resembles a number of other diseases. Of the
following, the MOST important is
A secondary syphilis
B drug eruptions
C viral exanthems
D guttate psoriasis
E nummular dermatitis
ANSWER:A
3110
Ichthyosis vulgaris is the most common of the disorders of keratinization; with an incidence
of 1/250 live births. Onset generally occurs in the 1st yr of life. Scaling is MOST prominent
on the
A extensor aspects the legs
B abdomen
C neck
D face
E scalp
ANSWER:A
3111 In X-Linked ichthyosis, scaling is MOST pronounced on the
A sides of the neck
B elbow flexures
C knee flexures
D palms
E soles
ANSWER:A
3112
A keloid is a sharply demarcated, benign, dense growth of connective tissue that forms in the
dermis after trauma. Of the following, the LEAST characteristic feature of keloid is
A firm
B raised
C pink
D rubbery

E pruritic
ANSWER:E
3113
A 10-year-old child with anterior uveitis developed a firm, smooth, erythematous papules
that gradually enlarge to formannular plaques with a papular border and a normal, slightly
atrophic central area several centimeters in size on the dorsum of the hands and feet. Of the
following, the MOST likely diagnosis is
A tinea corporis
B rheumatoid nodules
C granuloma annulare
D necrobiosis lipoidica
E xanthomas
ANSWER:C
3114
A 12-year-old child presented with brawny edema of the face and neck that spreads rapidly
to involve the thorax and arms. The face acquires a waxy, mask-like appearance. The
involved areas feel indurated, woody, nonpitting, and are not sharply demarcated from
normal skin. Of the following, the MOST likely diagnosis is
A scleredema of buschke
B scleroderma
C morphea
D dermatomyositis
E subcutaneous fat necrosis
ANSWER:A
3115 Beaded papules on the eyelids is the classic sign of
A pseudoxanthoma elasticum
B Ehlers-danlos syndrome
C cutis laxa
D lipoid proteinosis
E anetoderma
ANSWER:D
3116 Cutis laxa is characterized by all the following EXCEPT
A widespread folds of lax skin
B characteristic facial features
C hypermobility of the joints
D normal tensile strength of the skin
E hoarse cry

ANSWER:C
3117 Classic Ehlers-Danlos syndrome is characterized by the following EXCEPT
A post-term birth
B skin hyperelasticity
C easy bruising
D severe joint hypermobility
E subcutaneous spheroids
ANSWER:A
3118
Urticaria pigmentosa is the most common form of mastocytosis. It can be confused with the
following EXCEPT
A drug eruptions
B herpes simplex
C pigmented nevi
D insect bites
E bullous impetigo
ANSWER:B
3119
55.The etiology is unknown in 30-50% of pediatric cases of erythema nodosum; one of the
MOST common known etiologies in children is
A group A streptococcal infection
B cat-scratch disease
C leprosy
D mycoplasma
E brucellosis
ANSWER:A
3120 A rare but potentially life threatening complication of subcutaneous fat necrosis is
A hyperkalemia
B hypercalcemia
C acidosis
D septicemia
E seizures
ANSWER:B
3121 Histopathologic changes in sclerema neonatorum consist of
A increased size of fat cells
B fat necrosis
C inflammation

D giant cells
E calcium crystals
ANSWER:A
3122
Infiltrative or destructive disorders that may produce atrophy of sweat glands by pressure or
scarring include the following EXCEPT
A scleroderma
B acrodermatitis chronica atrophicans
C burns
D ichthyoses
E
Sjِ gren syndrome
ANSWER:D
3123 The following drugs may cause hyperhidrosis EXCEPT
A antipyretics
B antiemetics
C insulin
D opiates
E ciprofloxacin
ANSWER:B
3124 The following disorders are associated with anhidrosis by unknown mechanisms EXCEPT
A dehydration
B uremia
C cirrhosis
D Addison disease
E hypothyroidism
ANSWER:E
3125
Apocrine bromhidrosis develops after puberty as a result of the formation of short- chain
fatty acids and ammonia by the action of the following bacteria on axillary apocrine sweat
A anaerobic diphtheroids
B group A streptococcal infection
C S. aureus
D S. epidermidis
E Pseudomonas aerogenosa
ANSWER:A
3126 Nail pitting or grooves are usually seen in the following hair disorders
A telogen effluvium

B trichotillomania
C tinea capitis
D alopecia areata
E traction alopecia
ANSWER:D
3127
Anagen effluvium is an acute, severe, diffuse inhibition of growth of anagen follicles,
resulting in loss of >80-90% of scalp hair. All the following are causes of anagen effluvium
EXCEPT
A radiation
B alkylating agents
C thallium
D thiouracil
E hypervitaminosis D
ANSWER:E
3128 Nail anomalies are common in certain congenital disorders; large nails are seen in
A Rubinstein-Taybi syndrome
B Ellis– van Creveld syndrome
C ectodermal dysplasias
D trisomy 18 syndrome
E Turner syndrome
ANSWER:A
3129
Terry nails are characterized by a white ground glass appearance of the entire or the
proximal end of the nail and a normal pink distal 1-2 mm of the nail; this finding is
associated with
A leprosy
B tuberculosis
C Hodgkin disease
D arsenic poisoning
E hypoalbuminemia
ANSWER:E
3130
Yellow nail syndrome manifests as thickened, excessively curved, slow-growing yellow nails
without lunulae. Associated systemic diseases include the following EXCEPT
A bronchiectasis
B recurrent bronchitis
C chylothorax

D chronic hepatitis
E focal edema of the limbs and face
ANSWER:D
3131
Onycholysis indicates separation of the nail plate from the distal nail bed; drugs which may
cause onycholysis include
A meronem
B vancomycin
C adriamycin
D indomethacin
E cyclophosphamide
ANSWER:D
3132
Beau lines are transverse grooves in the nail plate. They are usually indicative of periodic
trauma or episodic shutdown of the nail matrix secondary to the following systemic diseases
EXCEPT
A mumps
B measles
C pneumonia
D celiac disease
E hand-foot-and-mouth disease
ANSWER:D
3133
Nail changes may be particularly associated with various other diseases. The following
matchings are true EXCEPT
A psoriasis : pitting, yellow-brown discoloration, and thickening
B lichen planus : violaceous papules in the proximal nail bed
C Darier disease: red or white streaks that extend longitudinally
D alopecia areata: Transverse rows of fine pits
E acrodermatitis enteropathica: subungual parakeratotic scaling and thickening
ANSWER:E
3134 Periungual fibromas that appear in late childhood should suggest a diagnosis of
A mucous cysts
B junctional nevi
C Lichen planus
D tuberous sclerosis
E subungual exostoses
ANSWER:D

3135
Angular cheilitis is characterized by inflammation and fissuring at the corners of the mouth,
often with associated erosion, maceration, and crusting. Of the following, the LEAST likely
cause is
A chapping
B chronic lip lickers
C excessive salivation
D nutritional deficiencies
E contact dermatitis to toothpaste
ANSWER:D
3136
Aphthous stomatitis consists of solitary or multiple painful ulcerations occur on the labial,
buccal, lingual, sublingual, palatal, or gingival mucosa; in severe, debilitating cases, systemic
therapy with all the following may be helpful EXCEPT
A corticosteroids
B colchicine
C antibiotics
D dapsone
E thalidomide
ANSWER:C
3137
A 7-year-old child presented with painless, fluctuant, tense, 7mm, bluish papule on the floor
of the mouth. Of the following, the MOST likely diagnosis is
A mucocele
B Epstein pearls
C Fordyce spots
D noma
E canker sores
ANSWER:A
3138 The MOST common lesions that precede nonbullous impetigo are
A scabies
B burns
C insect bites
D chickenpox
E pediculosis
ANSWER:C

3139
Staphylococcal scalded skin syndrome is caused predominantly by phage group 2
staphylococci, particularly strains 71 and 55, which are present at localized sites of infection.
Of the following, the MOST common focus of infection is
A nasopharynx
B umbilicus
C urinary tract
D superficial abrasion
E conjunctivae
ANSWER:A
3140
Ecthyma resembles nonbullous impetigo in onset and appearance but gradually evolves into
a deeper, more chronic infection. Of the following, the usual causative agent is
A S. aureus
B S. pneumoniae
C H. influenzae type b
D Clostridium perfringens
E group A-β hemolytic streptococcus (GABHS)
ANSWER:E
3141
Ecthyma gangrenosa is a necrotic ulcer covered with a gray-black eschar. It usually occurs in
immunosuppressed patients with neutropenia. Of the following, the usual causative agent is
A S. aureus
B P. aeruginosa
C S. pneumoniae
D H. influenzae type b
E Clostridium perfringens
ANSWER:B
3142 Furunculosis is more common in individuals with low
A serum zinc level
B serum iron level
C serum sodium level
D serum calcium level
E serum phosphorus level
ANSWER:B
3143 Vaccination with BC characteristically produces a papule approximately
A 1 wk after vaccination

B 2 wk after vaccination
C 3 wk after vaccination
D 4 wk after vaccination
E 5 wk after vaccination
ANSWER:B
3144 All the following matching are true EXCEPT
A Tinea capitis: dermatophyte infection of the scalp
B Tinea corporis: infection of the glabrous skin
C Tinea cruris: infection of the axilla
D Tinea pedis: infection of the toe webs
E Tinea unguium: dermatophyte infection of the nail plate
ANSWER:C
3145
A 3-month-old infant presented with an intensely erythematous, confluent plaque with a
scalloped border, a sharply demarcated edge, and satellite pustules in the perianal skin,
perineum, and inguinal folds. Of the following, the MOST likely diagnosis is
A seborrheic dermatitis
B atopic dermatitis
C primary irritant contact dermatitis
D candidal diaper dermatitis
E bacterial dermatitis
ANSWER:D
3146
A 4-year-old child presented with a 2 pearly, skin-colored, smooth, dome-shaped, 4 mm
papules with a central umbilication on the face. Of the following, the MOST likely diagnosis
is
A molluscum contagiosum
B ectopic sebaceous glands
C keratoacanthoma
D warty dyskeratoma
E cryptococcosis
ANSWER:A
3147 All the following can be used in the treatment of papular urticaria EXCEPT
A oral antihistamines
B cool compresses
C potent topical corticosteroids
D topical antihistamines

E systemic steroids
ANSWER:D
3148 Scabies is transmitted only rarely by fomites because the isolated mite dies within
A 2-3 hours
B 12-24 hours
C 2-3 days
D 5-7 days
E 10-20 days
ANSWER:C
3149
In an immunocompetent host, scabies is frequently heralded by intense pruritus, particularly
at night. Of the following, the classic lesions of scabies are
A bullae
B pustules
C wheals
D red papules
E threadlike burrows
ANSWER:E
3150 The treatment of choice for scabies is
A oral ivermectin
B lindane 1% cream
C sulfur ointment 5-10%
D permethrin 5% cream
E crotamiton 10% cream
ANSWER:D
3151 The treatment of choice for head lice is
A spinosad
B pyrethroids
C malathion
D lindane shampoo
E benzyl alcohol lotion
ANSWER:C
3152
Comedonal acne, particularly of the central face, is frequently the first sign of pubertal
maturation. It occurs in
A 20% of adolescents
B 40% of adolescents

C 60% of adolescents
D 80% of adolescents
E 100% of adolescents
ANSWER:D
3153 All the following drugs can induce acneiform lesions in susceptible individuals EXCEPT
A gold
B isoniazid
C phenytoin
D vitamin B1
E phenobarbital
ANSWER:D
3154 Little evidence shows that ingestion of the following foods can trigger acne flares
A high carbohydrate foods
B high spicy foods
C high sweet foods
D high fatty foods
E no certain foods
ANSWER:E
3155
All topical preparations for acne vulgaris must be used for 6-8 wk before their effectiveness
can be assessed. Of the following, the primary topical preparation should be
A azelaic acid
B salicylic acid
C topical retinoid
D Benzoyl peroxide
E topical clindamycin
ANSWER:C
3156
Antibiotics are indicated for treatment of patients whose acne has not responded to topical
medications. Of the following, the antibiotic that may cause bluish discoloration of the skin
and mucous membranes is
A tetracycline
B doxycycline
C minocycline
D erythromycin
E trimethoprim-sulfamethoxazole
ANSWER:C

3157
Approximately 20% of normal neonates demonstrate acne in the 1st mo of life.Treatment is
usually unnecessary. If desired, the lesions can be treated effectively with
A azelaic acid
B salicylic acid
C topical retinoid
D benzoyl peroxide
E topical clindamycin
ANSWER:D
3158 The nodules MOST commonly seen in children are
A fibrofolliculomas
B epidermoid cysts
C pilar cyst
D pilomatricoma
E trichoepithelioma
ANSWER:B
3159
A 7-year-old child presented with a smooth, pearly, pink, telangiectatic ulcerated papule that
enlarge slowly on his face. Of the following, the MOST likely diagnosis is
A pyogenic granuloma
B nevocellular nevus
C epidermal inclusion cyst
D dermatofibroma
E basal cell carcinoma
ANSWER:E
3160 All the following are features of acrodermatitis enteropathica EXCEPT
A a rare autosomal recessive disorder
B caused by an inability to absorb sufficient zinc from the diet
C Initial signs and symptoms usually occur in the 2nd year of life
D the cutaneous eruption consists of vesiculobullous, eczematous, dry, scaly, skin lesions
E alopecia
ANSWER:C
3161 Ocular manifestations of acrodermatitis enteropathica include all the following EXCEPT
A photophobia
B cataract
C conjunctivitis
D blepharitis

E corneal dystrophy
ANSWER:B
3162
The replacement of elemental zinc for individuals with inherited acrodermatitis
enteropathica is equal to
A 1 mg/kg/24 hr
B 3 mg/kg/24 hr
C 5 mg/kg/24 hr
D 7 mg/kg/24 hr
E 10 mg/kg/24 hr
ANSWER:B
3163
The hair is sparse, thin, and depigmented, sometimes displaying a “flag sign “which is a
characteristic finding in
A pellagra
B scurvy
C vitamin a deficiency
D kwashiorkor
E acrodermatitis enteropathica
ANSWER:D
3164
100. Facial eruption that frequently follows a butterfly distribution and a dermatitis
encircling the neck “Casal’s necklace.” is usually seen in
A pellagra
B scurvy
C vitamin A deficiency
D kwashiorkor
E acrodermatitis enteropathica
ANSWER:A
3165 101.The MOST common risk factor for scurvy in children is
A prolonged antibiotics use
B psychiatric disease
C exclusive breast feeding
D chronic diarrhea
E cow milk protein allergy
ANSWER:B
3166
Normal full-term newborns can have up to 20-30 degree hip and knee flexion contractures,
these contractures tend to resolve by

A 1-3 mo
B 4-6 mo
C 7-9 mo
D 10-12 mo
E 13-15 mo
ANSWER:B
3167 The ossification centers that are typically present at birth include all the following EXCEPT
A distal femur
B proximal tibia
C distal humerus
D calcaneus
E talus
ANSWER:C
3168 The gait of a child become similar to that of an adult at
A 3 yr
B 5 yr
C 7 yr
D 9 yr
E 11 yr
ANSWER:C
3169 A rocker-bottom foot describes
A congenital vertical talus
B flexible flatfoot
C calcaneovalgus foot
D congenital talipes equinovarus
E metatarsus adductus
ANSWER:A
3170
Polydactyly is the most common congenital toe deformity, it may be preaxial (great toe) or
postaxial (5th toe), and occasionally one of the central toes is duplicated. Which of the
following statements is TRUE?
A it is seen in approximately 2 in 100,000 births
B it is bilateral in 10% of cases
C one-third of patients will also have polydactyly of the hand
D it may be associated with Turner syndrome

E surgical removal of the extra digit is generally performed after 2 years of age
ANSWER:C
3171 Macrodactyly is seen in the following conditions EXCEPT
A Proteus syndrome
B Neurofibromatosis
C Tuberous sclerosis
D Klippel-Trenaunay-Weber syndrome
E Alport syndrome
ANSWER:E
3172
Pediatricians must understand normal limb development so as to recognize pathologic
conditions during routine and targeted exams. Of the following, the TRUE statement is
A normal tibiofemoral angle at birth is 10-15 degrees of physiologic valgus
B physiologic varus up to 12 degrees is reached in between 3 and 4 yr of age
C normal varus of 7 degrees is achieved by 5-8 yr of age
D persistence of varus beyond 2 yr of age may be pathologic
E more than 35% of developmental genu varum and genu valgum cases need intervention
ANSWER:D
3173
In evaluation of concerns relating to the limb, the examination should assess the exact
torsional profile which include all the following EXCEPT
A foot progression angle
B femoral anteversion
C knee – knee distance
D tibial version with thigh–foot angle
E assessment of foot adduction and abduction
ANSWER:C
3174
In-toeing gait most commonly results from excessive femoral anteversion. Of the following,
the TRUE statements is
A it is more common in boys than girls (2 : 1)
B patellas are pointing outward when the foot is straight
C diagnosis is made by CT
D treatment is predominantly observation
E anteversion >25 degrees is an indication for operative intervention
ANSWER:D
3175
Which torsional defotmity can follow a slipped capital femoral epiphysis and increase
incidence of degenerative arthritis?

A external tibial torsion
B external femoral torsion
C internal tibial torsion
D internal femoral torsion
E metatarsus adductus
ANSWER:B
3176
A school age child presented with a mass behind the knee. Physical examination reveals a
firm mass in the popliteal fossa, medially located and distal to the popliteal crease. The mass
is most prominent when the knee is extended. Knee radiographs are normal. Of the
following, the MOST likely diagnosis is
A osteochondroma
B osteochondritis dissecans
C popliteal cyst
D malignancy
E juvenile idiopathic arthritis
ANSWER:C
3177
An adolescent girl presented with pain beneath the patella, walking up and down stairs
aggravates the pain; squatting, running, and other vigorous physical activities also exacerbate
the pain. There is no history of antecedent trauma and there is no swelling. Pain is often
relieved through knee extension. Of the following, the MOST likely diagnosis is
A osteochondroma
B osteochondritis dissecans
C popliteal cyst
D patellofemoral pain syndrome
E juvenile idiopathic arthritis
ANSWER:D
3178 The proximal femoral ossification center (in the center of the femoral head) appears between
A 1st and 3rd mo
B 4th and 7th mo
C 8th and 11th mo
D 12th and 15th mo
E 16th and 18th mo
ANSWER:B

3179
There is marked geographic and racial variation in the incidence of developmental dysplasia
of the hip (DDH). All the following are true EXCEPT
A 1.7 in 1,000 babies in Sweden
B 75 in 1,000 in Yugoslavia
C 188.5 in 1,000 in a district in Manitoba, Canada
D 3.7 in 1,000 babies in Chinese newborn
E 0% in African newborn
ANSWER:D
3180 The MOST reliable sign of a dislocated hip after the 2nd month of life is
A limited hip abduction
B apparent shortening of the thigh
C proximal location of the greater trochanter
D asymmetry of the glutealor thigh folds
E positive Ortolani test
ANSWER:A
3181
Transient synovitis (toxic synovitis) is a reactive arthritis and is one of the most common
causes of hip pain in young children. All the following clinical manifestations are true
EXCEPT
A it is most prevalent between 3 and 8 yr of age
B 70% of affected children have had a preceding nonspecific upper respiratory tract infection
C symptoms often develop acutely
D children are usually able to bear weight on the affected limb
E usually associated with a high grade fever
ANSWER:E
3182
Legg-Calvé-Perthes disease is a hip disorder of unknown etiology that results from
temporary interruption of the blood supply to the proximal femoral epiphysis, leading to
osteonecrosis and femoral head deformity. Of the following, the MOST common presenting
symptom is
A limp of varying duration
B pain
C failure to ambulate
D atrophy of the muscles of the thigh
E an apparent leg-length inequality
ANSWER:A

3183 Differential diagnosis of legg-calvé-perthes disease include all the following EXCEPT
A osteochondromatosis
B Schwartz-Jampel syndrome
C Marfan syndrome
D Maroteaux-Lamy syndrome
E Martsolf syndrome
ANSWER:C
3184
Scoliosis is a complex 3-dimensional spinal deformity that is defined in the coronal plane as
a curve of at least 10 degrees, measured by the Cobb method, on a posteroanterior (PA)
radiograph of the spine. Which statement is TRUE about this disease?
A adolescent idiopathic scoliosis (AIC) is 2-10 times more common in males than females
B fathers with AIS transmit the disease to 50% of their children
C high plasma melatonin levels have been noted in patients with progressive curvatures
D girls with AIS have been noted to have a smaller foramen magnum
E approximately one-third of girls with AIS have osteopenia
ANSWER:E
3185
The normal thoracic spine has 20-50 degrees of kyphosis as measured from T3 to T12. A
thoracic kyphosis in excess of the normal range of values is termed hyperkyphosis. All the
following conditions are associated with hyperkyphosis EXCEPT
A trauma causing spinal fractures
B osteogenesis imperfacta
C Marfan syndrome
D glycogen storage disease
E mucopolysaccharidoses
ANSWER:D
3186
Torticollis, literally meaning twisted neck, is not a diagnosis but rather a manifestation of a
variety of underlying conditions. All the following conditions may cause torticollis EXCEPT
A positional deformation
B Klippel-Feil syndrome
C cervical lymphadenitis
D supratentorial brain tumor
E upper lobe pneumonia
ANSWER:D

3187
Klippel-Feil syndrome includes the classic triad of a low posterior hairline, short neck, and
decreased cervical range of motion. Other associations include the following EXCEPT
A Sprengel’s deformity
B congenital scoliosis
C genitourinary anomalies
D conductive hearing loss
E congenital heart disease
ANSWER:D
3188
Polydactyly or duplication of a digit can occur either as a preaxial deformity (involving the
thumb) or as a postaxial deformity (involving the small finger) All the following syndromes
associated with polydactyly EXCEPT
A Carpenter syndrome
B Ellis-van Creveld syndrome
C Trisomy 18
D Orofaciodigital syndrome
E Rubinstein-Taybi syndrome
ANSWER:C
3189
Syndactyly is one of the common anomalies observed in the upper limb. All the following
syndromes are associated with syndactyly EXCEPT
A Carpenter syndrome
B Ellis-van Creveld syndrome
C Laurence-Moon-Biedl syndrome
D Fanconi pancytopenia
E Trisomy 13
ANSWER:B
3190
Fractures are the second most common manifestation of child abuse after skin injury
(bruises, burns/abrasions). All of the following fractures suggest nonaccidental injury
EXCEPT
A femur fractures in nonambulatory children
B distal femoral metaphyseal corner fractures
C posterior rib fractures
D scapular spinous process fractures
E distal humeral fractures
ANSWER:E

3191
Fractures of the wrist and forearm are very common fractures in children. All the following
statements are true EXCEPT
A the most common mechanism of injury is a fall on the outstretched hand
B an eighty percent of forearm fractures involve the proximal radius and ulna
C the majority of forearm fractures are torus or greenstick fractures
D
a significant malunion of a forearm diaphyseal fracture can lead to a permanent loss of
pronation and supination
E the AP and lateral radiographs of the forearm and wrist confirm the diagnosis
ANSWER:B
3192 All the following statements are true regarding clavicular fracture EXCEPT
A most common site for fracture is the junction of the middle and medial 3rd of clavicle
B biceps function is important to assess as it is a prognostic indicator for future function
C posterior medial clavicular physeal injuries are particularly problematic
D fractures heal rapidly usually in 3-6 wk
E a palpable mass of callus is usually visible in thin children
ANSWER:A
3193
Toddler fractures occur in young ambulatory children. The age range for this fracture is
typically around 1-4 yr. All the following statement are true EXCEPT
A children in this age group are usually unable to describe the area of injury well
B radiographs may show no fracture
C classic symptom is refusal to bear weight
D Inflammatory markers may be ordered to rule out infectious processes
E fracture is treated with bed rest and analgesia for approximately 2 wk
ANSWER:E
3194 All the following are indications for external fixation in pediatric fractures EXCEPT
A grades II and III open fractures
B fractures associated with severe burns
C fractures with soft-tissue loss
D pelvic fractures
E fractures with associated head injuries and spasticity
ANSWER:D
3195
The two MOST common causative organisms of osteomyelitis in children with sickle cell
anemia are
A S. aureus and streptococci
B Salmonella spp. and S. aureus

C S. aureus and Pseudomonas aeruginosa
D Enterobacteriaceae and S. aureus
E S. aureus and Bartonella henselae
ANSWER:B
3196
The second MOST common cause of osteomyelitis in children younger than 5 yr of age in
some parts of the world is
A S. aureus
B Kingella kingae
C Pseudomonas aeruginosa
D Enterobacteriaceae
E Bartonella henselae
ANSWER:B
3197 The MOST common site of osteomyelitis in children is
A femur
B tibia
C humerus
D radius
E ulna
ANSWER:A
3198
Septic arthritis in infants and children has the potential to damage the synovium, adjacent
cartilage, and bone causing permanent disability. All the following statements regarding its
epidemiology are true EXCEPT
A septic arthritis is more common in young children
B half of all cases occur by 5 yr of age
C adolescents and neonates are at risk of gonococcal septic arthritis
D majority of infections in otherwise healthy children are of hematogenous origin
E infection of joints following penetrating injuries are uncommon
ANSWER:A
3199 hich medical condition is regarded a contraindication for sport participation?
A atlantoaxial instability
B splenomegaly
C carditis
D long-QT syndrome
E advanced heart block
ANSWER:C

3200 Adverse effect of anabolic–androgenic steroids include all the following EXCEPT
A infertility
B gynecomastia
C female virilization
D hypotension
E epiphyseal closure
ANSWER:D
3201
Creatine is an amino acid mostly stored in skeletal muscle increasing muscle performance. Its
use has been increased when other supplements have been withdrawn from the market. Of
the following, the possible adverse effect include
A mouth ulcer
B carditis
C hepatitis
D nephritis
E genital sore
ANSWER:D
3202 In competitive swimming, the MOST common injuries are of the
A neck
B shoulder
C spine
D hip
E knee
ANSWER:B
3203
Osteogenesis imperfecta, the most common genetic cause of osteoporosis. Which of their
types is mild and non-deforming?
A I
B II
C III
D IV
E V
ANSWER:A
3204
The MOST severe non-lethal form of osteogenesis imperfecta which results in significant
physical disability is type
A I
B II

C III
D IV
E V
ANSWER:C
3205 The MOST common ocular manifestation in Marfan syndrome is
A dislocation of the ocular lens
B severe myopia
C flat cornea
D hypoplastic iris
E retinal detachment
ANSWER:A
3206
Current pharmacologic approaches that are considered the standard of care in Marfan
syndrome (MFS) are
A angiotensine converting enzyme inhibitors
B beta blockers
C calcium channel blockers
D diuretics
E angiotensine receptor II antagonists
ANSWER:B
3207
Children with severe traumatic brain injury (TBI) may experience autonomic dysfunction
characterized by all the following EXCEPT
A elevated temperature
B elevated heart rate
C elevated respiratory rate
D lowered blood pressure
E diaphoresis
ANSWER:D
3208
Osteopenia begins immediately after a spinal cord injury (SCI) occurs and plateaus 6- 12 mo
later. Pathologic fractures occur as a consequence of loss of bone mineral density. Of the
following, the MOST common site of fracture is
A distal tibia
B supracondylar region of the femur
C lumbosacral spine
D proximal humerus
E distal radius

ANSWER:B
3209 Abrupt withdrawal of baclofen may cause
A depression
B drowsiness
C headache
D euphoria
E seizure
ANSWER:E
3210
Oral medications are often used as an early treatment for generalized spasticity. Which of the
following works at the level of skeletal muscle to block calcium release from the
sarcoplasmic reticulum?
A dantrolene sodium
B clonidine
C tizanidine
D baclofen
E diazepam
ANSWER:A
3211
Coadministration of botulinum toxin (BTX) and one of the following drugs should be
performed with caution as the effect of the toxin may be potentiated
A aminoglycosides
B cephalosporines
C penicillins
D macrolids
E sulphonamides
ANSWER:A
3212 Risk factors for birth brachial plexus injury include the following EXCEPT
A shoulder dystocia
B birthweight greater than 4 kg
C primiparous mothers
D mothers with excessive weight gain
E diabetic mothers
ANSWER:C
3213
The baby with birth brachial plexus palsy (BBPP) will start with occupational or physical
therapy at approximately
A 1 wk of age

B 2 wk of age
C 3 wk of age
D 4 wk of age
E 5 wk of age
ANSWER:B
3214
Prenatal screening is recommended for all pregnant women to detect neural tube defect. If a
neural tube defect is present, one of the following is often elevated
A human chorionic gonadotropin
B α-fetoprotein
C estriol
D inhibin
E prolactin
ANSWER:B
3215
Hindbrain herniation or the Chiari type II malformation is seen in which percent of
individuals with myelomeningocele
A 10-20%
B 20-30%
C 40-50%
D 60-70%
E 80-90%
ANSWER:E
3216
In utero radiation exposure is associated with an excess risk of dying from leukemia before
age 10 yr. Of the following, the MOST acceptable risk percent is
A 32%
B 52%
C 75%
D 92%
E 180%
ANSWER:D
3217 The following diseases are associated with sensitivity to radiation EXCEPT
A ataxia-telangiectasia
B Edward syndrome
C Down syndrome
D Fanconi anemia
E Gardner syndrome

ANSWER:B
3218
The acute effects of radiation therapy (occurring less than 3 mo after therapy begins) are
usually related to the area of the body being irradiated. Of the following, the MOST severe
acute reactions is
A pneumonitis
B dermatitis
C mucositis
D esophagitis
E cerebral edema
ANSWER:A
3219 Primary malignancies with the highest cumulative incidence of a second neoplasm are
A soft tissues sarcoma
B cancers of bone
C Hodgkin disease
D leukemias
E central nervous system (CNS) cancers
ANSWER:C
3220 Children exposed to second-hand tobacco smoke have increased frequency of
A middle ear effusions
B bacterial respiratory illnesses
C otitis externa
D sinusitis
E tonsilitis
ANSWER:A
3221 Children who grow up on farms have elevated rates of
A mesothelioma
B leukemia
C skin tumor
D lymphoma
E carcinoma of lungs
ANSWER:B
3222
Arsine gas is colorless, odorless, nonirritating, and highly toxic. Inhalation causes no
immediate symptoms. After a latent period of 2-24 hr, exposed individuals experience
A hematuria
B sensorimotor peripheral neuropathy

C respiratory failure
D massive hemolysis
E proteinuria
ANSWER:D
3223
Acute ingestion of inorganic mercury salts (typically secondary to ingestion of a button
battery) can manifest in a few hours as
A tremor
B neuropsychiatric disturbances
C gingivostomatitis
D corrosive gastroenteritis
E necrotizing bronchiolitis
ANSWER:D
3224
Acrodynia, or pink disease, is a rare idiosyncratic hypersensitivity reaction to mercury that
occurs predominantly in children exposed to mercurous powders. The symptom complex
includes the following EXCEPT
A generalized pain
B paresthesias
C an acral red-pink rash
D photophobia
E generalized spasticity
ANSWER:E
3225
It is estimated that lead-poisoned children are identified by screening procedures rather than
through clinical recognition of lead related symptoms in
A cases
B 79% of cases
C 59% of cases
D 39% of cases
E 19% of cases
ANSWER:A
3226 Mushrooms are an ideal food because they are
A low in calories
B high in calories
C high in fat
D low in fat
E low in protein

ANSWER:A
3227 One of the following is not recommended for treatment of Amanita poisoning
A oral activated charcoal
B forced diuresis
C silibinin
D intravenous penicillin G
E acetylcysteine
ANSWER:B
3228
Mushrooms of the genera Inocybe contain muscarine or muscarine-related compounds.
These quaternary ammonium derivatives bind to postsynaptic receptors, ‘producing an
exaggerated cholinergic response. Of the following, the MOST serious complication is
A diaphoresis
B bradycardia
C hypotension
D vomiting
E bronchospasm
ANSWER:E
3229
Pediatricians are likely to experience unique problems in managing childhood victims of
biologic or chemical attack. The very rapid onset of neuromuscular symptoms after an
exposure should lead the clinician to consider
A botulism
B nerve agent intoxication
C chlorine
D phosgene
E cyanide
ANSWER:B
3230
Pediatricians are likely to experience unique problems in managing childhood victims of
biologic or chemical attack. A delayed onset of respiratory symptoms (days after exposure)
is characteristic of
A chlorine
B phosgene
C cyanide
D anthrax
E sarin
ANSWER:D

3231
Pediatricians are likely to experience unique problems in managing childhood victims of
biologic or chemical attack. Patients suffering from the sudden onset of severe
neuromuscular symptoms may have nerve agent intoxication and should be given atropine
(0.05 mg/kg) promptly for its antimuscarinic effects. Atropine has the following effects
EXCEPT
A relieves bronchospasm
B relieves bradycardia
C reduces bronchial secretions
D ameliorates diarrhea
E improve skeletal muscle paralysis
ANSWER:E
3232
In cases in which the delayed onset of respiratory symptoms may be the result of a terrorist
attack, consideration should be given to the empirical administration of an antibiotic
effective against anthrax, plague, and tularemia. A reasonable choice is
A ciprofloxacin
B rifampin
C vancomycin
D ampicillin
E clarithromycin
ANSWER:A
3233
Infection is the most common complication of bite injuries, regardless of the species of biting
animal. The MOST common microorganisms associated with cat bites are
A Staphylococcus species
B Streptococcus species
C Eikenella species
D Pasteurella species
E Proteus species
ANSWER:D
3234 The MOST important proposed field treatments for snake bites is
A tourniquets
B immobilization of injured body part at the level of the heart
C ice
D incision
E suction
ANSWER:B

3235
Specific antivenoms (AV) are available for many venomous creatures of the world,
particularly snakes, spiders, and scorpions. All the following about antivenoms are true
EXCEPT
A AV is capable of neutralizing only circulating venom
B it is beneficial to give AV locally at the bite site
C most AVs are given intravenously
D skin tests are unreliable
E intravenous AV should be started slowly
ANSWER:B
3236
A small-for-gestation-age infant is born at 30 weeks’ gestation. At 1 h of age, his serum
glucose is noted to be 20 mg/dL (normally greater than 40 mg/dL). The most likely
explanation for hypoglycemia in this infant is
A Inadequate stores of nutrients
B Adrenal immaturity
C Pituitary immaturity
D Insulin excess
E Glucagon deficiency
ANSWER:A
3237
“Normal values” (reference intervals) are difficult to establish within the pediatric
population. Many variables should be considered when developing reference intervals. All
the following are variables that commonly considered to define reference intervals EXCEPT
A genetic composition
B physiologic development
C environmental influences
D subclinical disease
E ethnicity
ANSWER:E
3238
The most commonly used reference range is generally given as the mean of the reference
population ±2 standard deviations (SD). The term "normal distribution" refers to which of
the following?
A gaussian distribution
B exponential distribution
C skewed distribution
D uniform distribution

E unimodal distribution
ANSWER:A
3239 The closeness of a measured value to a standard or known value is termed as
A accuracy
B precision
C sensitivity
D specificity
E none of the above
ANSWER:A
3240
You asked to obtain a measurement of serum sodium level in the hospital lab; you obtain a
value of 125 mmol/L by repeating the test 5 times. The reference value of serum sodium at
that lab is between 135-155 mmol/L. Your measurement is termed
A accurate but not precise
B precise but not accurate
C neither accurate nor precise
D both accurate and precise
E sensitive and specific
ANSWER:B
3241
Assume you are going to estimate the prevalence of amoebic dysentery in a small country
which harbors a total number of population of 530,000; you find that 57,000 of the
population are infected by the disease. The prevalence of this disease is closest to
A 5,33%
B 7,45%
C 10,75%
D 20,22%
E 25,30%
ANSWER:C
3242
Assume you have the results of PCR tests for Mycoplasma pneumoniae; the number of
positive tests is 83 collected from a 100 truly-infected persons. The sensitivity of this test is
A 17%
B 55%
C 60%
D 83%
E 95%
ANSWER:D

3243
Assume you are evaluating the specificity of the indirect fluorescence antibody test for rapid
diagnosis of Campylobacter enteritis. The number of normal persons tested is 150 and the
test is positive in 30 persons. The specificity of this test is
A 20%
B 40%
C 60%
D 70%
E 80%
ANSWER:E
3244
You are assigned to evaluate the outcome of fecal occult blood test as a screening test for
patients with bowel cancer. The total number of patients who underwent endoscopy is 2030;
in those whom the disease is confirmed are 30. The results of fecal occult blood test are as
follows: positive in 20 patients who has a confirmed bowel cancer (truly positive), and in
180 patients who don`t have the disease (falsely positive); and negative in 10 patients who
truly has the disease (falsely negative), and in 1820 patients who are not affected by the
disease (truly negative). All the following statements are true EXCEPT
A the prevalence of the disease for those population is 1.48%
B the sensitivity of the test is 67%
C the specificity of the test is 91%
D the positive predictive value is 20%
E the negative predictive value is 99.5%
ANSWER:D
3245
Rapid HIV antibody testing procedures using a finger stickor venipuncture to obtain whole
blood, plasma, or serum, andtests using oral fluid were approved. All the following
regarding this test are true EXCEPT
A
they are simple and accurate as to render the likelihood of an erroneous result by the user
negligible
B
a positive result does not need confirmation by Western blot analysis or immunofluorescence
assay
C
they allow for implementation of antiretroviral therapy for HIV-infected women who have
not been tested or are unaware of their HIV status
D they significantly reduce the risk of mother-to-child transmission
E
The median turnaround time for obtaining results from blood collection to patient
notification was only 66 minutes
ANSWER:D

3246
Almost all of the diseases detected in neonatal screening programs have a very low
prevalence; the strategy is to use the initial screening test to separate a highly suspect group
of patients from normal infants (i.e., to increase the prevalence) and then to follow this
suspect group aggressively. Of the following, the disease that has low prevalence rendering it
not useful for neonatal screening testing is
A phenylketonuria
B hypothyroidism
C sickle cell disease
D cystic fibrosis
E neuroblastoma
ANSWER:E

!"#កទី ២
Surgery 1598
No Question
1 Bell Clapper is another term used to describe which of the following abnormalities?
A Hydrocele
B Varicocele
C Microcalcifications
D Testicular torsion
ANSWER:D
2
________, of any part of the epididymis, can cause it to become obstructed and may form a
spermatocele.
A Collagen
B Scar
C Wound healing
D Acne vulgaris
ANSWER:B
3 1% of adult males, Common in infancy especially in premature infants
A Hydrocele risk factors
B Hydrocele differential diagnosis
C Hydrocele: incidence
D Hydrocele diagnostic studies/ referrals
ANSWER:C
4
27Y man is struck on the side of the head during a company so ball game. The patient complains of
headache, has had two episodes of emesis, and is somewhat lethargic. The most likely source of his
problems is:
A Elevated intracranial pressure
B Contre.coup injury
C Cranial nerve V injury
D Cranial nerve VII injury
ANSWER:A

5
30Y woman is stabbed in the back and presents to ED in a highly anxious state. Her HR is 120/mn
and BP= 100/50 mm Hg. On exam, she does not have motor function in her right leg, which is also
numb. On the le leg, she does not react to painful stimuli and is insensate to temperature. The most
likely etiology of her problem is:
A Central cord syndrome
B Neurogenic shock
C Anterior spinal cord injury
D Hemisection of the spinal cord.
ANSWER:D
6 4-7days
A
Who does orchitis rarely occur in even if they have mumps? How does mumps orchitis MC present?
B What are the signs and sx of orchitis?
C Mumps orchitis follows parotitis by how many days?
D What is critical that you rule out when thinking orchitis or any scrotal pain?
ANSWER:C
7
A 15Y boy awakens with sudden onset of RLQ and scrotal tenderness accompanied by nausea and
vomiting. Which of the following is the most appropriate diagnosis and represents a surgical
emergency?
A Acute prostatitis
B Acute epididymitis
C Torsion of the testicle
D Acute appendicitis
ANSWER:C
8
A 15Y boy is admitted with a history and physical findings consistent with appendicitis. Which
finding is most likely to be positive?
A Pelvic crepitus
B Iliopsoas sign
C Murphy sign
D Flank ecchymosis
ANSWER:B

9
A 17Y F presents to ED with a stab wound to the abdomen and a blow to the head that left her
groggy. Her BP= 80/10 mm Hg, HR=120 bpm, and RR= 28/mn. Her abdomen has a stab wound in
the anterior axillary line at the right costal margin. Two large.bore intravenous lines, a nasogastric
tube, and a Foley catheter are inserted. The blood pressure rises to 85 mm Hg after 2 L of Ringer's
lactate. The appropriate management is which of the following?
A Peritoneal lavage
B Ultrasound of the abdomen
C Laparoscopic assessment.of the peritoneal cavity
D Exploratory laparotomy
ANSWER:D
10
A 19Y man is brought to ED. He has no significant medical history, but he does smoke one park
cigarettes per day. He is complaining dyspnea and has a blood pressure 150/70 mm Hg. Breath
sounds are diminished on the left. What is the best initial treatment for this patient?
A Chest tube drainage
B Supplemental oxygen only
C Needle decompression the left chest
D CT scan the thorax
ANSWER:A
11
A 19Y man presents to ED with a gunshot wound through the umbilicus. The systolic blood
pressure is 70 mm Hg on palpation, and his abdomen is tightly distended. Large.bore intravenous
lines are placed, and Ringer's lactate is infused. What should be the next step?
A Peritoneal lavage
B CT an of the abdomen
C Exploratory laparotomy
D Transfusion of the patient until the systolic blood pressure reaches 90mmHg
ANSWER:C
12
A 20Y man is in a motor vehicle collision and has a fracture of the left tibia. On exam, he has a
nondisplaced fracture and terrible pain in the leg, which seems to be getting worse despite narcotic
pain medication. The pulse in the leg is present but diminished relative to the other side. The patient
also complains of numbness in the affected foot. The best initial treatment is:
A Fluid resuscitation
B Parenteral narcotics

C Operative release of the compartments of the leg
D External fixation
ANSWER:C
13
A 20Y man suffers an injury to the posterior urethra. After appropriate initial management and
follow.up care of this injury, the most likely late complication is
A ascending urinary tract infection
B retrograde ejaculation
C sterility
D urethral stricture
ANSWER:D
14
A 20Y woman presents to the emergency department with a stab wound to the abdomen. There is
minimal abdominal tenderness. Local wound exploration indicates that the knife penetrated the
peritoneum. What is the ideal use of antibiotic administration?
A Preoperatively
B Intraoperatively,if a colon injury is found
C Postoperatively, if the patient develops fever
D
Postoperatively, based on culture and sensitivity of fecal contamination found at the time of surgery
E. Intraoperatively, if any hollow viscus is found to be injured.
ANSWER:A
15
A 21.year.old male patient is brought to the emergency department or evaluation after a motor
vehicle accident. As part of this secondary survey, the patient is found to have blood at the urethral
meatus. What is the next maneuver?
A Foley catheter insertion followed by cystogram
B Urethrogram
C IVP
D CT scan
ANSWER:B
16
A 21.year.old male suffers a severe comminuted fracture of the right leg with considerable soft
tissue loss after a motorcycle accident. He has exposed bone and tendon in his wound after external
fixation. Which is the appropriate management?
A Split.thickness skin graft
B Full.thickness skin graft

C Allograft followed by full.thickness skin graft
D Muscle flap
ANSWER:D
17
A 22 years old man has had a constant swelling in his testicle for the past 2 years. There is no pain,
fever or tenderness. It is brilliantly transilluminant and is not palpable separately from the testicle.
The cord is uninvolved. What is the most likely diagnosis?
A Vaginal hydrocele
B Infantile hydrocele
C Congenital hydrocele
D Spermatocele
ANSWER:A
18
A 22Y man is shot in the chest. He presents to ED with incoherent mumbling but his eyes open to
vocal commands and no response to painful stimuli. What is his Glasgow Coma Scale score?
A 6
B 8
C 10
D 12
ANSWER:B
19
A 23Y man is shot with a handgun and found to have a through.and.through injure.to the right
transverse colon. There is little fecal contamination and no bowel devascularization. At operation,
what does he require?
A Right hemicolectomy with ileotransverse colon anastomosis
B Right hemicolectomy with ileostomy and mucous fistula
C Debridement and closure of wounds with exteriorization of colon
D Debridement and closure of wounds
ANSWER:D
20
A 23Y man, tall and thin, was jogging one evening when he suddently felt a sharp pain in the left
chest, worse on taking a deep breath and shortness of breath: Select the most likely diagnosis. .
A Tension pneumothorax
B Cardiac tamponade
C Spontaneous pneumothorax

D Open pneumothorax
ANSWER:C
21
A 24Y man is brought into ED after a fall from a ladder. His breathing is labored and he is cyanotic.
No breath sounds can be heard, even in the right lung field, which is resonant to percussion. The
first step in his management should be
A tube thoracostomy.
B obtaining a stat chest x.ray
C passing an oral endotracheal tube
D starting oxygen by a valve.mask device
ANSWER:A
22
A 25 years male presents with h/o scrotal swelling for 3 years. O/E the swelling is fluctuant swelling
and the testis is ot palpable separately felt. No h/o trauma or fever. diagnosis:
A Vaginal hydrocele
B Hematocele
C Testicular cancer
D Varicocele
ANSWER:A
23
A 25.year.old man fell down from his bicycle and hit a concrete wall on his left side. An ultra.sound
examination showed free fluid in the abdomen. A CT scan confirmed a grade III splenic injury. The
most important contraindication for a nonoperative management of the splenic injury is:
A Hemodynamic instability
B Active bleeding on CT scan
C Adult patient
D Lack of availability of blood for transfusion
ANSWER:A
24
A 25.year.old man is injured in the arm with a knife. What is the first mechanism responsible for
hemostasis?
A Extrinsic clotting system
B Vessel constriction
C Intrinsic clotting system
D Platelet activation

ANSWER:B
25
A 25.year.old woman was stabbed by her boyfriend in the left chest. On examination, she has a
1.cm stab wound just inferior to her left breast in the mid.clavicular line. There is jugular venous
distension and breath sounds are completely absent on the left side. She is becoming extremely
dyspneic and hypoxic. Select the most likely diagnosis.
A Tension pneumothorax
B Cardiac tamponade
C Spontaneous pneumothorax
D Open pneumothorax
ANSWER:A
26
A 25Y man has been playing basketball when another player fell a top his knee. He felt a popping
sensation, followed by acute pain. On exam, the lower leg moves forward freely at the knee joint.
The patient has a weakly palpable pulse in the leg, and the ABI is 0.6 (and 1.0 on the unaffected
side). The most appropriate next step is:
A Angiography
B Knee brace immobilization
C Urgent arthroscopy
D Heparinization
ANSWER:A
27
A 25Y man is admitted with a history of sudden onset of severe midepigastric abdominal pain.
Upright chest radiograph reveals free intraperitoneal air. What is the therapy for this patient?
A Upper endoscopy
B Barium swallow
C Gastrografin swallow
D Laparotomy.
ANSWER:D
28
A 25Y man is shot in the left lateral chest. In ED, his BP=120/90 mm Hg, HR= 104/mn, RR= 36/mn.
Chest x.ray shows air and fluid in the left pleural cavity. Nasogastric aspiration reveals blood.stained
fluid. What is the best step to rule out esophageal injury?
A Insertion of chest tube
B Insertion of nasogastric tube
C Esophagogram with gastrografin

D Esophagoscopy
ANSWER:D
29
A 26Y man is stabbed in the right intercostal space in the midclavicular line and presents to ED. On
examination, subcutaneous emphysema of the right chest wall, absent breath sounds, and a trachea
shifted to the left are noted. What is the most likely serious diagnosis?
A Pneumothorax
B Tension pneumothorax
C Massive hemothorax
D Hemopneumothorax
ANSWER:B
30
A 30Y man is admitted with full thickness burns covering both upper extremities and the anterior
chest. Fluid resuscitation should be begun through
A an antecubital vein
B a femoral vein
C an internal jugular vein
D a saphenous vein
ANSWER:A
31
A 30Y man is brought to ED in respiratory distress following a shotgun wound to the face. There is
a possible cervical spine injury. Which is the best way to gain rapid control of the airway?
A Nasotracheal intubation
B Percutaneous jet ventilation
C Cricothyroidotomy
D Endotracheal intubation
ANSWER:C
32
A 30Y woman is brought to ED after she stepped on a rusty nail and sustained a puncture wound to
the foot. The patient has been on a therapeutic dose of steroids for the past 5 years for ulcerative
colitis. Her last tetanus toxoid booster was 8 years ago. What should the patient receive?
A Tetanus toxoid booster B. Human immunoglobulin
B Antibiotics with anaerobic coverage
C Tetanus toxoid plus human immunoglobulin
D Tetanus toxoid plus human immunoglobulin and antibiotics with aerobic and anaerobic coverage
ANSWER:D

33
A 31Y man is shot in the back of the left chest, and the bullet exits the left anterior chest. The
patient's blood pressure is 130/90 mm Hg, RR= 28/mn, and HR=110 bpm. A chest x.ray reveals
hemothorax. A chest tube is inserted and yields 800 mL of blood; the first and second hour drainage
is 200 mL/h and 240 mL/h, respectively. What is the next step in management?
A Place a second 'chest tube.
B Collect the blood for autotransfusion
C Transfuse and observe drainage for another hour.
D Perform a left thoracotomy.
ANSWER:D
34
A 32Y F falls from the tenth floor of her apartment building in an apparent suicide attempt. Upon
presentation, the patient has obvious head and extremity injuries. Primary survey reveals that the
patient is totally apneic. By which method is the immediate need for a definitive airway in this
patient best provided?
A Orotracheal intubation
B Nasotracheal intubation
C Percutaneous cricothyroidotomy
D Intubation over a bronchoscope
ANSWER:A
35
A 34.year.old man is involved in a motorcycle crash and sustains significant damage to legs and
skin. He has a large skin defect over the majority of the back of the hand. The most appropriate
definitive management for this patient is:
A Split.thickness skin grafting
B Primary closure
C Biologic dressing
D Full.thickness skin grafting
ANSWER:D
36
A 34Y man is brought into ED with a large open knife wound to the left thigh. The patient's systolic
blood pressure is 90 mm Hg. Blood is spurting from the wound. What is the initial management step?
A Clamp the bleeding artery with a vascular clamp.
B Apply a tourniquet7.5cm above the wound.
C Apply direct pressure with sterile gauze.

D Apply PASG, and inflate both legs.
ANSWER:C
37
A 35Y man is involved in a high.speed motor vehicle collision. He arrives in ED in respiratory
distress. CXR taken during the initial evaluation reveal an air. fluid level in the left chest.
Management includes all the following except:
A Establishment a secure airway
B Immediate placement a nasogastric tube
C Urgent thoracotomy to repair the injury
D Placement adequate peripheral vascular access
ANSWER:C
38
A 40.year.old construction worker is pulled from the rubble after a building collapses and pins his
right lower leg. X.rays in the emergency department reveal a comminuted fracture of the right tibia
and fibula. The dorsal pedis and posterior tibial pulses are palpable. The patient complains of severe
pain that is accentuated with dorsiflexion of the foot. The calf feels tense. What is the appropriate
step?
A ORIF of fracture
B ORIF of fracture plus three.compartment fasciotomy
C Closed reduction and observation
D ORIF only if pulses become weak
ANSWER:B
39
A 40Y lady presents with acute abdominal pain. She describes the pain as high in the epigastrium,
perhaps moving a little to the right side. The pain comes with meals and has been present on and off
over the past months. After lunch today, the patient developed severe pain, low.grade ever, and
nausea. Her V/S, no fever, HR= 85,BP=normal. On exam, she is markedly tender under the right
costal margin and cannot take a full breath during that part of the examination. The best initial test
or this patient is:
A Abdominal CT scan
B RUQ ultrasound
C Abdominal magnetic resonance enterography (MRE)
D EGD
ANSWER:B

40
A 40Y man is hit by a car and sustains an injury to the pelvis. Which of the following is most
indicative of a urethral injury?
A Hematuria
B Scrotal ecchymosis
C Oliguria
D High.riding prostate on rectal examination
ANSWER:D
41
A 40Y man is involved in a car crash, presenting with BP=80. The patient is found to have subdural
hematoma and a supracondylar fracture of the left femur, FAST shows fluid within the abdomen.
He is taken to the OR, where intra.abdominal bleeding is controlled, and the subdural hematoma is
drained. The femur fracture should be treated by which of the following?
A Long.leg cast
B Steinmann pin insertion and traction
C Operative reduction and internal reduction
D Aspiration of knee joint
ANSWER:B
42
A 42 yo male came to you because of painless swelling, in the left testis, he gives Hx of mild trauma
few days earlier. Swelling is cystic, involving testis only and is not transilluminatable. Diagnosis is
A Varicocele.
B Tumor.
C Spermatocele.
D Hydrocele.
ANSWER:C
43
A 50Y man hears a "snap" and then feels pain in his right leg while lunging for a forearm drive
playing tennis. He walks off the court with difficulty, but his leg is swollen and painful. Findings on
x.rays of the leg and ankle in the emergency room are negative. Foot sensation is normal, but
findings on the Thompson test (failure of plantar flexion to occur after squeezing the
gastrocnemius) are positive. What is the diagnosis?
A Gastrocnemius muscle tear
B Acute thrombophlebitis
C Rupture of the Achilles tendon

D Acute compartment syndrome
ANSWER:C
44
A 55.year.old woman undergoes laparotomy for small bowel obstruction. During lysis of adhesions,
an enterotomy is made in the obstructed, but viable, bowel, and a large amount of fecal.looking
bowel contents are spilled into the abdomen. The incision would now be considered what kind of
wound?
A Clean contaminated
B Secondary
C Infected
D Contaminated
ANSWER:D
45
A 55Y man with insulin.dependent diabetes presents to the emergency department with acute
abdominal pain. V/S, HR= 130 , BP= 90/60 ,T= 38°C, RR= 28 .The abdominal examination
demonstrates diffuse peritonitis. What should be the first step in the evaluation and management of
this patient?
A Volume resuscitation
B Abdominal radiograph
C IV antibiotics
D CT scan
ANSWER:A
46
A 60Y man is hit by a pickup truck and brought to ED with BP= 70/0 mm Hg. Peritoneal lavage
showed no blood in the abdomen. The blood pressure is elevated to 85 systolic following the
administration of 2 L of Ringer's lactate. An x.ray showed a pelvic fracture. What is the next step in
management?
A Exploratory laparotomy with packing of the pelvis
B CT scan of the pelvis
C External fixation of the pelvis
D Open reduction and internal fixation (ORIF) of the pelvis
ANSWER:C
47
A 60Y man is in a car crash in which he is the driver. He did not have a seat belt or an airbag. He is
found to have multiple rib fractures over his right chest. His pulse is weaker during inspiration. what
are the most likely diagnoses?

A Flail chest
B Empyema
C Diaphragm.rupture
D Hemothorax
ANSWER:A
48
A 70Y man is brought into ED following his injury as a passenger in a car crash. He complains of
right side chest pain. Physical examination reveals RR= 42/mn and multiple broken ribs of a
segment of the chest wall that moves paradoxically with respiration. What should the next step be?
A Tube thoracostomy
B Tracheostomy
C Thoracentesis
D Endotracheal intubation
ANSWER:C
49
A 70Y man is brought to ED following a car crash. X.rays revealed a fractured rib on the left and a
fracture of the right femur. A CT scan of the abdomen showed a left.sided retroperitoneal hematoma
adjacent to the left kidney and no evidence of urine extravasation. The hematoma should be
managed by which of the following?
A Observation
B Exploratory laparotomy through a midline incision
C CT scan.guided aspiration
D Surgical exploration through a left.flank retroperitoneal approach
ANSWER:A
50
A 9Y girl presents with drainage from the midline neck. There is some surrounding cellulitis and an
apparent 2.cm mass that elevates with swallowing. The most appropriate definitive management of
this condition is:
A Antibiotics alone
B Thyroid scanning
C US.guided aspiration
D Complete surgical excision.
ANSWER:D
51 A benign cystic accumulation of sperm that arises from the head of the epididymis.

A Spermatocele
B Encysted hydrocele of the cord
C Spermatic cord
D Varicocele
ANSWER:A
52
A cluster of dilated veins in the scrotal sac, often just above the testes; occurs most often in young
adults.Scrotum feels wormlike.
A Varicocele
B Hydrocele
C Treatment
D Hydrocele and varicocele
ANSWER:A
53 A cystic translucent mass above the testicle is probably a:
A Spermatocele
B Varicocele
C Hydrocele
D Hematocele
ANSWER:C
54 A hydrocele develops between:
A Visceral and parietal layers of the tunica albuginea
B The tunica albuginea and tunica vaginalis
C Parietal and visceral layers of the tunica vaginalis
D Scrotum and tunica albuginea
ANSWER:C
55 A hydrocele is defined as an abnormal fluid collection between the:
A Tunica albuginea and the tunica vaginalis
B Spermatic cord and the tunica albuginea
C Two layers of the tunica vaginalis
D Spermatic cord and the tunica vaginalis
ANSWER:C

56
A male infant is presented in two months by his parents who have discovered a mass in the inguinal
region of very recent appearance, a little painful. It is irreducible to the abdomen, one does not feel
well at the upper pole of the inguinal ring, it is transillumination. The testicle is seen below this mass.
What diagnosis do you recall?
A Inguinal-scrotal hernia strangulated
B Strangulated inguinal hernia
C Cyst of the spermatic cord
D Vaginal hydrocele
ANSWER:C
57
A patient is brought to ED with a stab wound to the right chest in the fourth intercostal space in the
midaxillary line. The patient is hypotensive, complains shortness of breath, and is found to have
Absent breath sounds at the right side the chest. Which step should come next in the management
this patient?
A Chest radiograph
B Chest tube insertion
C Needle thoracentesis
D Local wound exploration
ANSWER:C
58
A patient is involved in a high.speed motor vehicle collision. The patient has a Glasgow Coma Scale
(GCS) score of 7 on arrival. Which of the following is not urgently indicated?
A Emergent intubation
B Placement of an intraventricular catheter
C Nasogastric tube to prevent aspiration
D Spinal cord immobilization
ANSWER:C
59 A spermatocele usually lies:
A In the head of the epididymis, superior to the testis
B In the body of the epididymis, posterior to the testis
C In the tail of the epididymis, inferior to the testis
D In the body of the epididymis, lateral to the testis
ANSWER:A
60 A tongue of peritoneum that precedes the migrating testes through the inguinal canal

A Paramesonephron
B Processus vaginalis
C Mesonephron
D Gubernaculum
ANSWER:B
61 A varicocele right recently appeared in a man of 60 years, to suspect the diagnosis:
A Recent renal trauma.
B Right testicular tumor.
C Cyst of the right kidney.
D Cancer of the right kidney.
ANSWER:D
62
A18Y man is brought to the emergency department with a stab wound just to the right of the
sternum in the sixth intercostal space. His blood pressure is 80 mm Hg. Faint heart sounds and pulses
paradoxes are noted. Auscultation of the right chest reveals decreased breath sounds. The initial
management of this patient should be which of the following?
A Aspiration of the right chest cavity
B Aspiration of the pericardium
C Echocardiogram
D Pericardial window
ANSWER:A
63
A19Y man is admitted to ED with a stab wound just below the right inguinal ligament. There is
profuse bleeding from the wound and he is in shock. The first step in local wound control should be
to
A apply compression of the bleeding vessel with a gloved finger
B place the patient in medical antishock trousers (MAST) and inflate the right leg section
C place a tourniquet on the right thigh above the wound
D use clamps and ligatures to control the bleeding
ANSWER:A
64 Abdominal pathology, Renal tumor, Venous obstruction
A Varicocele assessment findings
B Varicocele diagnostic studies
C Varicocele risk factors

D Varicocele differential diagnosis
ANSWER:C
65
Absent on the side of the swelling; stroke inner thigh and normally will produce a contraction of the
cremasteric (pulls scrotum up)
A Symptoms of Testicular Torsion
B Repair Varicocele
C Varicocele
D Cremasteric Reflex
ANSWER:D
6 All the following statements concerning early tangential excision of burn wound are true EXCEPT
A The procedure entails significant blood loss
B In a patient with severe smoke inhalation the procedure should be done under local anesthesia
C By expediting healing in bums around joints the procedure preserves joint function
D The procedure is carried out sequentially until good capillary bleeding indicates viable tissue
ANSWER:B
67 Also known as cyrptorchidism
A Undescended testis
B Direct hernia
C Varicocele
D Indirect hernia
ANSWER:A
68
An 18.year.old girl falls from a top a piece of furniture onto a hard surface but stops her all on an
outstretched hand. She only complains of pain in her wrist. What is the most likely injury?
A Distal radial fracture
B Distal ulnar fracture
C Lunate bone fracture
D Scaphoid bone fracture
ANSWER:D
69
An 18Y man is brought to ED after falling down a flight of stairs and losing consciousness for 3
minutes. A cervical collar is in place. The cervical spine is considered to be free of serious injury
following which procedure?

A A physical examination revealing no pain or tenderness
B A lateral cervical spine x.ray
C Completely negative findings on neurological examination
D Anteroposterior (AP), lateral, and odontoid views of the neck
ANSWER:D
70 An anechoic structure arising from the rete testes describes which of the following structures?
A Epididymal cyst
B Testicular cyst
C Hydrocele
D Spermatocele
ANSWER:D
71 An early sign of anterior compartment syndrome in the calf is
A absence of pulses in the foot
B firm calf muscles
C foot drop
D paresthesia between the great and second toes
ANSWER:D
72 An embryological remnant found on the upper pole of the testis
A Bell Clapper deformity
B Hypospadia
C Hydatid of Morgagni
D Glanular hydidiform
ANSWER:C
73
An episode of bacterial or viral infection (mumps, pna, tb, syphilis), s/e of epididymitis,
prostatectomy, trauma, infectious mononucleosis, flu, catherterization, or complicated UTI,
A What are some characteristics of hydrocele?
B What are some causes of orchitis?
C What is the infection that can cause infertility?
D What is Hydrocele?
ANSWER:B
74
Associated w/ prostatitis or UTI; Inflammatory process due to infection (gonorrhea, chlamydia,
trauma, urine reflux)

A Symptoms of Epididymitis
B Etiology of Orchitis
C Epididymitis
D Orchitis
ANSWER:C
75 Cluster of dilated veins around the testes
A Spermatocele
B Encysted hydrocele of the cord
C Spermatic cord
D Varicocele
ANSWER:D
76
Collection of abnormally large dilated veins (usually the internal spermatic vein) in the scrotum,
usually situated above the testis
A Varicocele
B Varicocele differential diagnosis
C Varicocele: consultation/ referral
D Varicocele etiology
ANSWER:A
77
Delayed primary closure would be the most appropriate wound closure technique for which of the
following procedures?
A Removal of perforated appendix
B Repair of wound dehiscence 1 week after elective left colectomy
C Emergency drainage of a diverticular abscess with sigmoid resection and end colostomy
D Vagotomy and pyloroplasty or bleeding duodenal ulcer/
ANSWER:A
78
Dilation of the vein that drains the testes, scrotum feels warm, left side of the scrotum; 1 cause of
sterility today
A Varicocele
B Diagnosis
C Repair Varicocele
D OR
ANSWER:A

79 Ductus deferens, testicular artery, pampniform venous plexus, genital branch of genitofemoral nerve
A Spermatocele
B Encysted hydrocele of the cord
C Spermatic cord
D Varicocele
ANSWER:C
80 During sleep
A What causes testicular torsion?
B Who gets testicular torsion?
C When does testicular torsion usually occur?
D What does intermittent/stuttering torsion mean?
ANSWER:C
81
Failure of this perioneal extension to obliterate after birth may lead ot the development of an
inguinal hernia or hydrocele
A Mesonephron
B Processus vaginalis
C Gubernaculum
D Paramesonephron
ANSWER:B
82
Finding a painless, cystic mass at the head of the epididymis, that transilluminates and can be clearly
differentiated from the ________, is generally sufficient.
A Prostate
B Reproductive system
C Testicle
D Penis
ANSWER:C
83 Fluid collects in the middle unclosed part of the processus vaginalis testis
A Spermatocele
B Encysted hydrocele of the cord
C Spermatic cord
D Varicocele

ANSWER:B
84
Following an automobile accident, a 19Y man is admitted to ED with an extensive central facial
injury. His breathing is labored and he is coughing up blood and mucous with each breath. He is
thrashing about and is combative despite efforts to restrain him. The first step in his management
should be to maintain in.line cervical traction and
A obtain a cross table lateral x.ray of the cervical spine
B pass an endotracheal tube by the nasal route
C pass an endotracheal tube by the oral route
D perform a cricothyroidotomy
ANSWER:D
85 High, horizontal, firm, tender testicle; ABSENT Cremasteric reflex
A What might you find on PE of appendage torsion?
B What is the TX for testicular torsion?
C What might you find on PE of testicular torsion?
D What is the TX for appendage torsion?
ANSWER:C
86 Horizontal (bell clapper deformity)
A What causes testicular torsion?
B What is the TX for testicular torsion?
C What is the salvage rate for testicular torsion?
D What testes position is most at risk for torsion?
ANSWER:D
87 Hydrocele is a which type of cyst
A Retention
B Distension
C Exudation
D Traumatic
ANSWER:A
88 Hydrocoele is accumulation of serous fluid between the two layers of the _________.
A The tunica albuginea.
B Tunica vasculosa.
C Tunica vaginalis.

D Tunica intima.
ANSWER:C
89 If uncertainty exists, ________ of the scrotum can confirm if it is spermatocele.
A Abdominal ultrasonography
B Medical ultrasonography
C Radiology
D Obstetric ultrasonography
ANSWER:A
90 If your pee is light or pale yellow, it means that:
A You are hydrated
B You’re dehydrated
C You don’t have enough yellow food dye in your diet
D You aren’t getting enough sun
ANSWER:A
91
In contrast to spermatoceles, both ________ and hernias may enlarge with the increased
intraabdominal pressure generated during Valsalva.
A Hydrocele
B Chylocele
C Varicoceles
D Hematocele
ANSWER:C
92
In patients who have an inappropriately high attachment of the tunica vaginalis, the testicle can
rotate freely on the spermatic cord within the tunica vaginalis. This congenital anomaly is called the
A Bell clapper deformity
B Hydatid of Morgagni
C Bellend deformity
D Hypospadia
ANSWER:A
93 In the groove between testicle & epididymis
A What is the salvage rate for testicular torsion?
B Where is the appendage of the testes located?

C Why is the Appendix Testis susceptible to torsion?
D What is the Appendix Testis formed of?
ANSWER:B
94 Infection of the testes or epididymis. Testicular tumor. Trauma to the testes or epididymis.
A Overview-Pathophysiology
B Overview-Causes
C Overview-Complications
D Overview-Communicating
ANSWER:B
95 Laparoscopic cholecystectomy is indicated for all of the following conditions except which?
A Biliary dyskinesia
B Initial treatment in patients with severe cholangitis.
C Acute cholecystitis
D Symptomatic cholelithiasis
ANSWER:B
96 Malignant neoplasms of the neck are LEAST LIKELY to cause
A central nervous system invasion
B Distant metastases
C malnutrition
D recurrent aspiration pneumonia
ANSWER:B
97 Manifestation of testicular torstion:
A Predisposes to rotary twisting of testi and spermatic cord within the scrotum
B Manifestation of testicular torsion
C Acute onset of testicular pain and swelling
D Treatment of testicular torsion - Surgery
ANSWER:C
98 Mumps, 2. Bacterial
A What is critical that you rule out when thinking orchitis or any scrotal pain?
B What are the 2 major causes of orchitis?
C
1. What labs should you consider running if suspecting orchitis? 2. What imaging should you
consider?

D What are the signs and sx of orchitis?
ANSWER:B
99 Neurogenic shock is characteized by the presence of
A cool, moist skin
B increased cardiac output
C decreased peripheral vascular resistance
D decreased blood volume
ANSWER:C
100
Often advised for a unilateral intra-abdominal testis that can not be corrected, becasue of the risk of
future malignancy
A Orchidopexy
B Orchidectomy
C Orchistatis
D Orchidotomy
ANSWER:B
101 Orchiopexy can be defined as
A Fixation of an ovary
B Uterine suspension
C Testicle removal
D Fixation of a testicle
ANSWER:D
102
Painless, swelling in the scrotum; Fluctuation in size of scrotum in communication hydrocele;
Scrotum feels heavy and enlarged; Transillumination of scrotum
A Hydrocele differential diagnosis
B Hydrocele diagnostic studies/ referrals
C Hydrocele assessment findings
D Hydrocele risk factors
ANSWER:C
103
Patency between the scrotal sac and the peritoneal cavity allows peritoneal fluids to collect in the
scrotum, especially with Valsalva's maneuver.
A Overview-Incidence
B Overview-Causes

C Overview-Noncommunicating
D Overview-Pathophysiology
ANSWER:D
104 Patient 60 kg, GCS 13, stable vital signs. Fluid management?
A D 5 %= 2 L
B 1.5 L colloids
C NSS 2.3 L
D NSS 1.6 l
ANSWER:C
105 Patient complaining of numbness and weakness in all four extremities. What needs to be done first?
A IV fluids
B Vital signs
C Hard collar
D Turn on his side
ANSWER:C
106 Patient process vaginalis is connected with the penitoneal cavity in the following type of hydrocele
A Congenital hydrocele
B Infantile hydrocele
C Vaginal hydrocele
D Encysted hydrocele of cord
ANSWER:A
107
Patient with GCS 13, hematoma left eye, dilate pupil left side, no reaction when left eye exposed to
light, but reaction when right eye exposed to light. What happened?
A Brain stem compression
B Compression oculomotor nerve
C Paralysis ciliar muscle due to direct trauma
D Compression facial nerve
ANSWER:B
108 Patient with head trauma lying on his back, GCS 9, stridor. What needs to be done first?
A Intubation

B Ambu bag
C Turn on his side
D Jaw thrush
ANSWER:C
109
Patient with open femur fracture and mild bleeding, alert and complaining of pain. What needs to be
done first?
A Pain medication
B Antibiotics and tetanos toxoid and dressing
C X.ray
D IV fluids
ANSWER:A
110
Patient with RR of 40/min, cyanosis, and contusion of left chest and dilated jugular veins. What
needs to be done first?
A IV fluids
B Chest X.ray
C Chest tube
D Clinical exam and needle puncture
ANSWER:D
111 Poorly functioning anti-reflux valves of the spermatic veins
A Varicocele etiology
B Varicocele incidence
C Varicocele assessment findings
D Varicocele diagnostic studies
ANSWER:A
112 Primary vaginal hydrocele in infant Seminal fluid is also made of secretions from:
A The accessory sex organs, the prostate gland, and Cowper's glands.
B The accessory sex organs, the epididymis and the prostate gland.
C The accessory sex organs, the epididymis, the prostate gland, the prepuce and Cowper's glands.
D The accessory sex organs, the epididymis, the prostate gland, and Cowper's glands.
ANSWER:D
113 Severe pain in the scrotum, swelling, N&V; no fever, no bacteria or WBC in urine
A Testicular Torsion

B Varicocele
C Symptoms of Testicular Torsion
D Diagnosis
ANSWER:C
114 Sperm-containing cyst that develops along the epididymus
A Spermatocele
B Encysted hydrocele of the cord
C Spermatic cord
D Varicocele
ANSWER:A
115
Spermatocele is a retention cyst of a tubule of the rete testis or the head of the epididymis distended
with a milky fluid that contains ________.
A Penis
B Reproductive system
C Spermatozoa
D Spermatogenesis
ANSWER:C
116 Spermatocele is most commonly present at:
A Tail of epididymis
B Head of the epididymis
C Behind and above the body of testis
D Anywhere
ANSWER:B
117
Spermatocelectomy can be performed on an outpatient basis, with the use of local or general
________.
A Pain management
B Surgery
C Medicine
D Anesthesia
ANSWER:D
118
Spermatoceles are ________ cysts so named because of the frequent finding of sperm in the cyst
fluid.

A Pseudo cyst
B Epididymal cysts
C Globulomaxillary cyst
D Nasolabial Cyst
ANSWER:B
119
Spermatoceles are usually found at the ________ of the epididymis, adjacent or posterior to the
superior pole of the testicle.
A Front
B Back
C Neck
D Head
ANSWER:D
120
Spermatoceles can be discovered as incidental scrotal masses found on physical examination by a
________.
A Medicine
B Physician
C Pediatrics
D Medical school
ANSWER:B
121 Spermatoceles vary in size from several ________ to many centimeters.
A Decametres
B Millimetres
C Metres
D 100 megametres
ANSWER:B
122 Surgical placement of the testes in the scrotum
A Testoplexy
B Orchistatis
C Orchidectomy
D Orchidopexy
ANSWER:D
123 Testicular torsion beyond 8 hours is characterized by: (one answer is wrong)

A Irreversible testicular ischemia.
B Quickly compromised endocrine function.
C A possible impact on the testis.
D The mobilization of twisted testicle is painful.
ANSWER:B
124 Testicular Torsion of at least ______ degrees is necessary for complete arterial occlusion?
A 180
B 360
C 540
D 720
ANSWER:A
125 The American Cancer Society (ACS) recommends routine screening for all of the following except:
A Mammography for breast cancer
B Pap smear for cervical cancer
C . Prostate.specific antigen (PSA) for prostate cancer
D Colonoscopy for colon cancer
ANSWER:C
126 The epididymis is distended, swollen and contains thick purulent material. The most likely cause is:
A Mumps
B Tuberculosis
C Gonorrhea
D Seminoma
ANSWER:C
127 The meaning of ductus is
A Vessel
B To carry away
C To lead
D Passage
ANSWER:C
128 The meaning of vas is

A Vessel
B Carry away
C Bukb
D To lead
ANSWER:A
129 The term deferens means
A Vessel
B To carry away
C Bulb
D Pasage
ANSWER:B
130 The term that means inflammation of the coiled tube that is attached to the testits is
A Cryptorchism
B Epididymitis
C Varicocele
D Testicular torison
ANSWER:B
131 The term that means undescended testicle is
A Cryptorchism
B Epididymitis
C Varicocele
D Testicular torison
ANSWER:A
132 The word root that means coat
A Tunica
B Vagin
C Vesic
D Semin
ANSWER:A
133 They are also believed to result from ________ or physical trauma.
A Balanitis
B Prostatitis

C Orchitis
D Epididymitis
ANSWER:D
134 Urine is made of:
A Water and nutrients
B Water and lemonade
C Everything you drank that day
D Water and waste products
ANSWER:D
135 Varicocele is common in left testis because :
A Left testicular vein drains into IVC which has high pressure
B Left testicular vein drains into left renal vein which has high pressure
C Left testis is lower situated
D Compression of testicular vein by rectum
ANSWER:B
136 What is NOT true about a varicocele?
A More common on the right side
B Associated with infertility
C Left varicocele can be a late sign of a tumor in an elderly man
D 10% cases are bilateral
ANSWER:A
137 What is the most common clinical cord syndrome in incomplete injuries?
A Anterior Cord
B Central Cord
C Posterior Cord
D Cauda Equina
ANSWER:B
138 Which of the following is a cyst of the epididymis?
A Spermatid
B Meosis
C Varicocele
D Spermatocele

ANSWER:D
139 Which of the following is not true of a hydrocele?
A It can be distinguished from a hematocele by transillumination.
B It is typically associated with a direct inguinal hernia
C It refers to excess fluid in a persistent processus vaginalis.
D It may be confined to the spermatic cord.
ANSWER:B
140 Which of the following is not true regarding anterior cord syndrome?
A Loss of motor below level
B Loss of proprioception/vibration
C Loss of pain and temp
D Flexion injury
ANSWER:B
141 Which of the following is the best treatment or melanoma?
A Surgical excision
B Chemotherapy
C Radiation therapy
D Immunotherapy
ANSWER:A
142 Which of the following terms means surgical fixation of the testis?
A Orchiectomy
B Orchiopexy
C Balanoplasty
D Varicocele
ANSWER:B
143 Which of the following terms refers to a dilated vein in the spermatic cord?
A Cryptorchism
B Epididymitis
C Varicocele
D Testicular torsion
ANSWER:C
144 Which of the following words mean fluid in the space of the tunica vaginalis?

A Spermatid
B Meiosis
C Hydrocele
D Varicocele
ANSWER:C
145 Which statement accurately describes a Brown Sequard syndrome?
A
Asymmetric injury with ipsilateral loss of motor function and sensation and contralateral loss of pain
and temperature.
B Symmetrical injury with loss of motor function bilaterally.
C
Asymmetric injury with ipsilateral loss of pain and temperature and contralateral loss of motor
funtion and sensation.
D Loss of awareness, but preserved motor, pain, temperature, and light tought.
ANSWER:A
146 Why is it important to pee when you have to?
A To remove toxins and other harmful things from your blood
B So your kidneys and bladder stay healthy
C So you don’t pee your pants
D All of the above
ANSWER:D
147
You are alone in ER with one nurse. Five patients are coming from the same accident. Which one do
you treat first?
A Patient with open femur fracture and mild bleeding, alert and complaining of pain.
B Patient with RR of 40/min, cyanosis, contusion of left chest and dilated jugular veins.
C Patient with coma, lies in lateral position, RR 16/min, bleeding from left ear.
D Patient complaining of numbness and weakness in all four extremities.
ANSWER:B
148 _____ is a common respiratory sign in abdominal injuries.
A Rapid and shallow breathing
B Slow and deep breathing
C Slow and shallow breathing
D Rapid and deep breathing
ANSWER:A

149 A 21 yo lady with rectal bleeding and severe pain on defecation
A Colon Ca
B Crohn’s Disease
C Fissure in ano
D Fistula in ano
ANSWER:C
150
A 25 yo patient comes in with a history of poor appetite, slowed growth, jaudice, enlarged spleen,
liver and heart. The blood tests show a deficiency in hemoglobin and an abnormal RBC. The
erythrocytes have a central color spot in the area of pallor. The US exam shows an extremely large
spleen. What do you suspect?
A Thalassemia
B Congenital spherocytosis
C Hemolytic Anemia
D Autoimmune Hemolytic anemia
ANSWER:A
151 A 25 yo woman, 6 weeks post-partum with PR bleeding, puritis and perianal pain
A Colon Ca
B Crohn’s Disease
C Fissure in ano
D Fistula in ano
ANSWER:C
152
A 32 yo woman presents with rectal bleeding, which occurs post-defecation. The bleeding is bright
red and painless. Endoscopy is normal.
A Haemorrhoids
B Crohn’s Disease
C Fissure in ano
D Fistula in ano
ANSWER:A
153
A 40-years old male presents with hematemesis, On examination, his BP was 90/60 mm Hg and
heart rate was 120/min, splenomegaly was also present. The most probable cause of his bleeding is:
A Portal hypertension

B Gastric ulcer
C Duodenal ulcer
D Dru induced GI injury
ANSWER:A
154 A 90 yo woman with PR bleeding and something coming down every time she defecates
A Rectal proplase
B Crohn’s Disease
C Fissure in ano
D Fistula in ano
ANSWER:A
155
A client's ulcerative colitis signs and symptoms have been present for longer than 1 week. The nurse
should assess the client for signs and symptoms of which of the following complications?
A Heart failure.
B Deep vein thrombosis.
C Hypokalemia.
D Hypocalcemia.
ANSWER:C
156
A client starts to experience pain while receiving an enema. The nurse notes blood in the return fluid
and rectal bleeding. What action does the nurse take first?
A Administers pain medication
B Slows down the rate of instillation
C Tells the client to breathe slowly and relax
D Stops the instillation and obtains vital signs
ANSWER:D
157
A client who has had a full-thickness burn is being discharged from the hospital. Which information
is most important for the nurse to provide prior to discharge?
A How to maintain home smoke detectors
B Joining a community reintegration program
C Learning to perform dressing changes
D Options available for scar removal
ANSWER:C

158
A client who has ulcerative colitis has persistent diarrhea. He is thin and has lost 12 lb since the
exacerbation of his ulcerative colitis. Which of the following will be most effective in helping the
client meet his nutritional needs?
A Continuous enteral feedings.
B Following a high-calorie, high-protein diet.
C Total parenteral nutrition (TPN).
D Eating six small meals a day.
ANSWER:C
159 A narrowing of the lumen of the intestine, As shown on the barium x-ray.
A Who is at higher risk for developing Crohns disease and ulcerative colitis:
B A string sign is:
C Gas forming foods:
D Following the acute phase, diet progresses to:
ANSWER:B
160
A patient is brought to the emergency department by ambulance. He has hematemesis and alteration
in mental status. The patient has tachycardia, cool clammy skin, and hypotension. The patient has a
history of alcohol abuse. What would the nurse suspect the patient has?
A Hemolytic jaundice
B Hepatic insufficiency
C Bleeding esophageal varices
D Portal hypertension
ANSWER:C
161
A patient who has had several episodes of bloody diarrhea is admitted to the emergency department.
Which action should the nurse anticipate taking?
A Obtain a stool specimen for culture.
B Administer antidiarrheal medications.
C Teach about adverse effects of nonsteroidal anti-inflammatory drugs (NSAIDs).
D Provide education about antibiotic therapy.
ANSWER:A
162 A patient with Crohn’s Disease is most likely to have the disease is what part of the GI tract?
A Rectum
B Duodenum of the small intestine

C Terminal Ileum
D Descending colon
ANSWER:C
163
A patient with esophageal varices has medication ordered to decrease portal pressure and aid in
preventing a first bleeding episode. What type of drug would the physician order?
A Antihypertensive agents
B Beta-blocking agents
C Calcium channel blockers
D Histamine-2 antagonists
ANSWER:B
164
A previously healthy 9-year-old child comes to emergency room because of fulminant upper
gastrointestinal bleeding. The hemorrhage is most likely to be the result of
A Esophageal varices
B Mallory-Weiss syndrome
C Gastritis
D A gastric ulcer
ANSWER:A
165 A procedure when the colon is cut and both ends are brought out onto the abdomen.
A What is meant by a "temporary" colostomy?
B What is a colostomy?
C What is meant by the term "double-barrel" colostomy?
D
When a client voices embarrassment over the noises that their colostomy makes on the first post-op
day, what would you say?
ANSWER:C
16
A Sengstaken-Blakemore tube is inserted in the effort to stop the bleeding esophageal varices in a
patient with complicated liver cirrhosis. Upon insertion of the tube, the client complains of difficulty
of breathing. The first action of the nurse is to:
A Notify the MD
B Encourage him to take deep breaths
C Monitor VS
D Deflate the esophageal balloon
ANSWER:D

167 Abdominal pain radiating to the left shoulder.
A Kehr Sign
B Areas of purple discolorization are suspicious for abdominal injury
C Ecchymosis around the umbilicus means bleeding into the abdominal wall
D Bluish discoloration in the flank area may indicate retroperitoneal hemorrhage
ANSWER:A
168 Abscesses, pseudopolyps, hemorrhage, perforation, toxic megacolon, & colonic dilation.
A Intestinal complications of ulcerative colitis
B Extraintestinal complications of ulcerative colitis
C Parenteral Nutrition
D Ulcerative Colitis
ANSWER:A
169 Action of 5α reductase inhibitors on B P H treatment :
A Relax smooth muscle of prostate , bladder neck
B Decrease prostate volume
C Increase diameter of prostatic urethra
D Inhibits production of DHT - Testosterone
ANSWER:D
170 Action of alfa blocker on B P H treatment :
A Relax smooth muscle of prostate , bladder neck
B Decrease prostate volume
C Increase diameter of prostatic urethra
D Inhibits production of DHT - Testosterone
ANSWER:A
171 Acute pyelonephritis
A is an inflammation of the bladder
B complicated to sepsis
C irritative symptoms is a main symptoms
D Suprapubic pain is a main symptoms
ANSWER:B

172
Acute superficial inflammatory lesions of the gastric mucosa induced when an individual is
subjected to abnormal elevated physiologic demands; Multiple smaller lesions, as opposed to one
large one in PUD.
A Clinically Significant SRMB
B SRMD
C Therapy Selection
D Duration of Treatment Dependent on:
ANSWER:B
173 All are the Signs of Pulmonary contusion except
A Hypoxia
B Flat neck veins
C Chest pain
D Decreased breath sounds
ANSWER:B
174 All are Clinic signs of tension pneumothorax except
A Pneumomediastinum
B Respiratory Distress
C Distended neck veins
D Hyperresonance
ANSWER:A
175 All are the most causes of cardiac arrhythmias in burn patients except
A hypovolemia,
B hypoxia,
C Uremia
D acidosis,
ANSWER:C
176 All are the clinic signs of tracheobronchial injury Except
A Persistent pneumothorax
B Pneumomediastinum
C Abnormal EKG-
D Subcutaneous air
ANSWER:C

177 All are the mechanisms of Blunt trauma Except
A Blast ( Pressure wave causes tissue disruption )
B
Crush (Compression) Body is compressed between an object and a hard surface Direct injury of
abdominal wall and internal structures
C Deceleration Body in motion strikes a fixed object
D Deceleration Direct injury of abdominal wall and internal structures
ANSWER:D
178 All are the sign of Cardiac tamponade except
A Subcutaneous air
B Distended neck veins
C Distant heart tones
D Paradoxical pulse
ANSWER:A
179 All are the signs of Cardiac contusion except
A Abnormal EKG-
B Decreased breath sounds
C Cardiac dysfunction
D Shock
ANSWER:B
180 All are the signs of Cardiac Tamponade Except
A Hypotension
B Distended neck veins
C Pneumomediastinum , Subcutaneous air
D Distant heart tones
ANSWER:C
181 All are the signs of the massive hemothorax except
A Paradoxical pulse
B Flat neck veins
C Shock
D Absent breath sounds
ANSWER:A
182 All Criteria suggest serious brain injury except

A pupillary inequality greater than 1 mm.
B lateralized extremity weakness regardless of the GCS score;
C markedly depressed skull fractures;
D Cardiac dysfunction
ANSWER:D
183 All the following is the risk factor of dry gangrene, except:
A Older people with diabetes
B Younger people with bleeding
C People with Raynaud’s disease
D Atherosclerosis
ANSWER:B
184 An open traumatic wound with retained necrotic tissue and perforated viscera is:
A Dirty contaminated wound
B Infected wound
C Contaminated wound
D An old traumatic wound
ANSWER:A
185 Anal fistula :
A is a small tunnel connecting the anal gland to the skin
B is a linear ulcer of the lower the anal canal
C is a symptomatic anal cushions
D is a discrete mass of tissue in the lumen of bowel
ANSWER:A
186 Anal fistula : Imaging study :
A Fistulography
B Abdominal Echography
C Abdominal C T SCANN
D Abdominal M R I
ANSWER:A
187 anal fistula symptoms :
A Perianal discharge , pain with history of anal abscess
B Anal prolapse

C Sentinel pile
D Bleeding
ANSWER:A
188 Assessment of symptoms and bother in B P H :
A I P S S
B Physical Examination
C Echography
D Urinalysis
ANSWER:A
189
Assign acuity: 75 year old male presents to ED after falling down "head over heels" approximately
15 stairs while carrying a television. Only complaint is left shoulder pain, denies LOC. BP 110/60
RR 20 HR 90. On further exam there is abrasion noted to knee and hip with hematoma to right
flank.
A ESI Priority 1, prepare for transport
B ESI Priority 2, requires priority provider assessment
C ESI Priority 3, always a safe choice
D ESI Priority 4 (Fast Track), the patient only c/o shoulder pain
ANSWER:B
190 Bacterial etiology of U T I
A Staphylococcus
B Candida
C Virus
D coli
ANSWER:D
191
Basophilic nuclear remnants (clusters of DNA) in circulating red blood cells that are usually
removed by the spleen. [During maturation in the bone marrow late erythroblasts normally expel
their nuclei, but in some cases a small portion of DNA remains.]
A Howell-Jolly bodies
B Susceptibility to bacteremia
C Already have antibodies (removed by liver)
D Regional lymph nodes
ANSWER:A

192 Benign prostatic hyperplasia ( BPH ) is :
A an increase in size of the prostate.
B an inflammation of the prostate
C a malignancy tumor of the prostate
D an abscess of the prostate
ANSWER:A
193 Can involve the joints, skin, mouth, eyes, & hematological systems.
A Central parenteral nutrition
B Intestinal complications of ulcerative colitis
C Surgical Management
D Extraintestinal complications of ulcerative colitis
ANSWER:D
194 Carbuncle is ( one correct )
A Necrotizing cellulitis
B Cluster of furunculosis
C Cluster of folliculitis
D Fulminant toxic dermatitis
ANSWER:C
195 Cause of Paradoxical pulse :
A Pneumomediastinum
B Pneumothorax
C Hemothorax
D Cardiac tamponade
ANSWER:D
196 Causes of Portal Hypertension prehepatic
A Inferior vena cava thrombosis
B Cardiac failure
C Portal vein thrombosis
D Primary biliary cirrhosis
ANSWER:C
197 Character of kidney pain
A Located at costovertebral angle

B The intensity is related to the position
C Radiated to the scrotum or vulva
D causing by irritation of thoracic nerves
ANSWER:A
198 Choose the incorrect sentence What is tertiary intention of the wound healing ?
A Heavy Contamination of Wound
B infection and inflammatory wound
C Granulation tissue grows and fill
D Surgical wound are left open 3-5 day then stapled or sutured closed.
ANSWER:C
199 Clado’s point
A A point at the junction of the intersection of rectus sheath and the bispinal line
B A point at the junction of the intersection of rectus sheath and the spinoumbilical line
C A point at outer third or inner two third of spinoumbilical line
D
A point at the junction of the intersection of rectus sheath and the bispinal line,A point over the right
side of the abdomen that is one-third of the distance from the two ASIS, on the bispinale line
ANSWER:A
200 Colostomies performed for cancer tend to be ______
A Permanent.
B A surgically created opening of the colon out onto the abdominal wall.
C Warmer than body temperature 99-100.
D Sitting.
ANSWER:A
201 Complication of B P H :
A Fever
B bladder stone
C Pyuria
D Varicocele
ANSWER:B
202 Complication of Monteaggia fracture.
A Wolkman syndrome

B Pseudarthrosis
C Avascular necrosis !" Head radius
D Compartment syndrome
ANSWER:D
203 Complication of pyelonephritis
A Sepsis
B Urinary retention
C Bladder stone
D Prostatitis
ANSWER:A
204 Complications of inguinal hernia is( one correct )
A Orchtis
B Strongulation
C Testis atrophy
D Infertility
ANSWER:B
205 Complications របស់ close fracture ភ(ាមៗ
A Shock
B Hemorrhage
C Non union
D Pseudarthrosis
ANSWER:A
206
Congenital, Acquired, Trauma, Surgical removal (Idiopathic thromboytopenia purpura - ITP),
Autosplenectomy (sickle hemoglobinopathies), Other medical conditions.
A Mechanism of bacterial infection?
B GI causes of functional hyposplenism?
C Causes of Asplenia?
D Liver diseases associated with functional hyposplenism?
ANSWER:D
207 Crohn's is a digestive disease that also involves which additional system in the body?
A The lymphatic system
B The central nervous system

C The endocrine system
D The immune system
ANSWER:D
208 Crohn’s disease:
A Is caused by mumps virus
B Is more common in Asians than in Jews
C Tends to occur in families
D Is less frequent in temperate climates than in tropical ones
ANSWER:C
209 Cullen sign (Periumbilical ecchymosis)
A is superficial oedema and bruising in the subcutaneous fatty tissue around the umbilicus
B is Crepitation or instability lower thoracic cage hemorrhage.
C Fullness or tenderness in the recto-vagina
D Is a crepitation or instability lower thoracic cage
ANSWER:A
210 Cystitis
A Is a inflammation of the bladder
B Fever is a main symptoms
C Is a inflammation of the prostate
D CT Scann is necessary to diagnosis of cystitis
ANSWER:A
211 Cystitis : urinalysis
A Look for WBC and Bacteria
B Look for red blood cell and calcium
C Look for virus and candida
D Look for proteinuria and bacteria
ANSWER:A
212 D R E of BPH :
A Nodule , Induration
B Smooth
C Pain
D Warm

ANSWER:B
213 D R E of prostate carcinoma ( Cap ) :
A Nodule , induration
B Pain
C Smooth
D warm
ANSWER:A
214 Dance sign is ( one correct )
A A right hypochondrium sausage-shaped mass and emptiness in right flank
B A right flank sausage-shaped mass and emptiness in RLQ.
C A right hypochondrium sausage-shaped mass and emptiness in RLQ.
D A right lower quadrant sausage-shaped mass and emptiness in hypogastrium
ANSWER:A
215 Definitions of Monteaggia Fracture?
A Fracture midle 1/3 របស់ ulnas
B Fracture radius proximal 1/3
C Fracture olecranon with dislocation of de head radius !" elbow
D Fracture proximal 1/3 of ulnas with dislocation of the head radius !" elbow
ANSWER:D
216
Diverticulum (17-40%), Angiodysplasia (9-21%), Colitis (2-30%), Neoplasia (11-14%), Anorectual,
Upper GI, Small bowel.
A Most common lower GI bleed in adults with hematochezia
B Most likely cause of acute lower GI bleeding in pt &gt;40 yers
C Work up of Lower Gi bleed
D Most common cause of lower Gi tract bleeding in...kids and adol
ANSWER:A
217 Double barrel colostomy is
A
The bowel is severed and both ends are brought out onto the abdomen. Only the proximal stoma is
functioning.
B
Surgical operation to make an opening (stoma) between the colon and the abdominal wall to allow
faces to be passed without going through the rectum.

C
The bowel is then sutured to the abdomen and two openings are created in the one stoma; one for
stool and the other for mucus.
D
Usually created in the transverse colon. This is one stoma with two openings; one discharges stool,
the second mucus.
ANSWER:A
218 Dry gangrene results from:
A Dehydration
B Occlusion of artery
C Occlusion of vein
D Chemical toxin
ANSWER:B
219 During your focused history and physical exam of a patient with a suspected abdominal injury:
A Remember that the radial pulse will normally be stronger than the femoral pulse.
B Keep in mind that it takes about one liter of blood to expand the abdominal girth by 2 inches.
C Discoloration around the umbilicus indicates bleeding is occurring inside the abdomen.
D Palpate the abdomen, starting from the point closest to the pain.
ANSWER:C
220 End colostomy is
A
A stoma is created from one end of the bowel. The other portion of the bowel is either removed or
sewn shut (Hartmann's procedure).
B
The bowel is then sutured to the abdomen and two openings are created in the one stoma; one for
stool and the other for mucus.
C The opening into the ostomy. Can be temp or permanent.
D
Usually created in the transverse colon. This is one stoma with two openings; one discharges stool,
the second mucus.
ANSWER:A
221 Etiology of the B P H :
A Sex
B Smoking
C Androgen hormone
D Obesity
ANSWER:C

222 Externa Hemorrhoid complication
A Bleeding
B Ulceration
C Strangulation ( thrombosis )
D Diarrhea
ANSWER:C
223 External Hemorrhoid location
A Above the mucocutaneous junction
B Below the mucocutaneous junction
C Anterior midline
D Posterior midline
ANSWER:B
224 External Hemorrhoid symptoms
A Ulceration
B Bleeding
C Skin tags
D Pain
ANSWER:C
225 For the first 4 to 6 weeks following colostomy, the patient should take:
A Low-protein
B High-carbohydrate
C Low-calorie
D Low-residue
ANSWER:D
226 Fournier’s gangrene, is a necrotizing fasciitis specifically located at ( one correct )
A Perineo-genital region
B Lower limb
C Upper limb
D Neck
ANSWER:A
227
Guideline for the adequacy of resuscitation in burn patient that mainly reflects reexpansion of the
intravascular compartment with fluid resuscitation. Except :

A Puls< 120/mn
B urine output remains an excellent (0.5 to 1.0 cc/kg/hour in the adult)
C Ischemic constrictions of muscle
D A persistent metabolic acidosis indicates inadequate perfusion
ANSWER:C
228 Half of patients have complications, Having it more than 10 yrs is a greater risk of colorectal cancer.
A Complications of Ulcerative Colitis
B Complications leading to surgical intervention
C Mildly active ulcerative colitis
D Complications of TPN
ANSWER:A
229 Howell Jolly Bodies are frequently seen in:
A Megaloblastic anemia.
B Sickle cell disease (SCD).
C Iron deficiency anemia.
D After splenectomy.
ANSWER:D
230
Hypotension; Sepsis; Hepatic failure, renal failure; High dose steroids; Multiple trauma; Severe
burns; Neuro injury/Spinal cord injury; Major surgery; Prolonged ICU admission; Hx of GI
bleeding; If 2 or more factors NEED PROPHYLAXIS.
A How is SRMD diagnosed on endoscopy?
B How does stress-related mucosal damage occur?
C What are other risk factors of SRMD?
D What are the independent risk factors of SRMD?
ANSWER:C
231
IBD characterized by diffuse inflammation of the intestinal mucosa. The result is a loss of surface
epithelium with ulceration and possibly abscess formation.
A Fistulas
B Define Ulcerative Colitis
C Define Tenesmus
D Define toxic megacolon

ANSWER:B
232 Idiopathic intussusception commonly affects in ( one correct )
A Newborn
B Toddlers
C Older children
D Adult
ANSWER:B
233 Idiopathic intussusception frequently locates at ( one correct )
A Enterocolic
B Ileo-Ileocolic
C Ileo-Ileal,
D Jejuno-Jejunal
ANSWER:A
234 Imaging studies is performed for B P H :
A Ultrasound
B M R I
C CT scan
D Cystoscopy
ANSWER:A
235 Imaging study necessary to diagnosis of UTI
A MRI
B C T Scann
C I V P
D Echography
ANSWER:D
236
In a patient with compensated liver cirrhosis presented with a history of variceal bleed. The
treatment of choice in this patient is
A Propranolol
B Liver transplantation
C TIPS (Transjugular intrahepatic portal shunt)
D Endoscopic sclerotherapy
ANSWER:D

237 In B P H, the blood test are performed to rule out Cap :
A Creatininemia
B Uremia
C P S A
D A F P
ANSWER:C
238 In burn patient Myoglobinuria can result from
A Ischemic constrictions of muscle, crush injuries, or deep thermal or electrical burns of muscle.
B poor tissue perfusion due to hypovolemia or to heart failure.
C Peripheral vasoconstriction
D Peripheral vasoconstriction
ANSWER:A
239 In newborn physical examination, the absence or misplace of anus, defines as ( one respond correct )
A Anus imperforate
B Defects in the development of the lowest portion of the intestinal and/or urogenital tracts.
C Cloacal persistency
D Anorectal agenesis
ANSWER:B
240 In one year of age ,Simple congenital hydrocele mostly is ( one correct )
A Resolved spontaneously
B Needing urgently a surgical treatment
C Haematocele complication
D Rupture the vaginal sac
ANSWER:A
241 In the usual preparation for general surgery, the client may be:
A Given ice chips
B NPO liquid before 8hours and solid food for 12 hours before
C Allowed to brush teeth and swallow water
D NPO liquid and solid 24hours before
ANSWER:B
242 Internal Hemorrhoid Classification

A Grade I : Permanent prolapse
B Grade II:Prolapse on defecation ,Reduces spontaneous
C Grade III : Painless , bleeding , No prolapse
D Grade IV : Prolapse and reduces manually
ANSWER:B
243 Internal Hemorrhoid symptoms
A Bleeding during defecation / prolapse
B Anal pain
C Skin tags
D Sentinel pile
ANSWER:A
244 IV fluid in burn patients is given:
A 1/2 of total fluid is given in the first 8 hours post burn
B ¼ of total fluid is given in the first 8 hours post burn
C the whole total fluid is given in the first 8 hours
D 1/2 of total fluid is given in the first hours post burn
ANSWER:A
245 LARINEC Score , Indicator for Necrotizing Fasciitis are based on( Select one is incorrect )
A C-Reactive Protein
B White Blood Cell Counts
C Hemoglobin
D Hemoglobin A1C
ANSWER:D
246
Less frequent diarrhea usually not bloody, colicy pain LRQ- mild to severe, abdominal tenderness,
spasms, weight loss, malaise, crampy pain after meals lower grade fever.
A Complications
B Clinical course
C Clinical manifestations for UC
D Clinical manifestations for Crohn's
ANSWER:D
247 Location of Benign prostatic hyperplasia ( BPH ) :
A Central zone

B Transition zone
C Peripheral zone
D Fibro muscular stroma zone
ANSWER:B
248 Location of the Kidney
A Costovertebral angle
B Hypogastric
C Right lower abdominal quadrant
D Left lower abdominal quadrant
ANSWER:A
249 Loop colostomy is
A
The bowel is then sutured to the abdomen and two openings are created in the one stoma; one for
stool and the other for mucus.
B The opening into the ostomy. Can be temp or permanent.
C
Surgical operation to make an opening (stoma) between the colon and the abdominal wall to allow
faces to be passed without going through the rectum.
D
The bowel is severed and both ends are brought out onto the abdomen. Only the proximal stoma is
functioning.
ANSWER:A
250 Medical treatment of B P H :
A Alfa blocker / 5 alfa reductase inhibitor
B Chemotherapy
C Vaccination
D Hormonothrerapy
ANSWER:A
251 Melena, Hematochezia.
A IBD bleeding
B Occult bleeding
C Obscure bleeding
D Overt bleeding
ANSWER:D
252 Mild anemia; Thrombocytopenia; Mild leukocytosis.

A Hypersplenism
B Massive splenomegaly causes
C Physical: splenomegaly due to hemolysis
D History: splenomegaly due to hemolysis
ANSWER:A
253 Morris’ point:
A A point an inch and a half from the umbilicus in a spinoumbilical line.
B A point that lies one-third of distance laterally on a spinoumbilical line.
C
A point at the intersection of the right edge of the rectus abdominis muscle (rectus sheath) and the
spinoumbilical line.
D A point is situated on intersection of rectus sheath and the bispinal line.
ANSWER:A
254 Mucosa edematous & red, White areas are superficial ulcers with mucous exudate.
A Mildly active ulcerative colitis
B Mild, Moderate, & Severe Ulcerative Colitis
C Ulcerative Colitis
D Complications of TPN
ANSWER:A
255 Munro’s point
A A point at the junction of the intersection of rectus sheath and the spinoumbilical line
B A point at outer third or inner two third of spinoumbilical line
C A point at the junction of the intersection of rectus sheath and the bispinal line
D
The point over the right side of the abdomen that is one-third of the distance from the two ASIS, on
the bispinale line
ANSWER:A
256 Munro’s point
A A point at the junction of the intersection of rectus sheath and the spinoumbilical line
B A point at outer third or inner two third of spinoumbilical line
C A point at the junction of the intersection of rectus sheath and the bispinal line
D
A point over the right side of the abdomen that is one-third of the distance from the two ASIS, on
the bispinale line
ANSWER:A

257 Necrotizing fasciitis is characterized by( one incorrect )
A Fulminant destruction of tissue,
B Systemic signs of toxicity: Sepsis
C Thrombosis of blood vessels,
D Needing urgently surgical drainage with culture and antibiogram
ANSWER:D
258 Non operative pneumatic reduction is ( one correct )
A A diagnostic procedure
B Indicated in the condition of intestinal perforation suspect
C Indicated only the symptoms less than 24H
D Indicated only the symptoms more than 48H
ANSWER:C
259 Palpation of abdominal pain should begin:
A Farthest from the quadrant that hurts.
B Immediately.
C In the left upper quadrant.
D In the quadrant that hurts.
ANSWER:A
260
Paralysis of lower portion of rectus abdomins muscle, involving the movement of the navel towards
the head on flexing the neck.
A Beevor Sign
B Grey Turner's Sign
C Cullen's Sign
D Seat Belt Sign
ANSWER:A
261 Patho-Etiology of Congenital Hernia and Hydrocele in children is: ( one correct )
A By excessive production of fluid within the sac
B By defective absorption of fluid
C By interference with lymphatic drainage of scrotal structure
D By connection with the peritoneal cavity via a patent processus vaginalis
ANSWER:D
262 Pesticides, Food additives, Radiation, Possibly allergies/immune system.

A Crohn's disease and ulcerative colitis are triggered by;
B Crohn's disease leads to:
C Crohn's disease most often affects the:
D Crohn's disease is an:
ANSWER:A
263 Physical examination of the anal fistula :
A Anoscopy with probe
B Palpation
C Percussion
D Auscultation
ANSWER:A
264 Physical examination of the prostate
A Auscultation
B Abdominal palpation
C Digital rectal examination
D Abdominal percussion
ANSWER:C
265 Post-operative care
A is the care you receive immediate after surgical procedure.
B is the care you receive before and during surgical procedure.
C is the care after you receive anesthesia.
D is the care after you discharge home.
ANSWER:A
26
Presence of dull percussion note in both flanks, constant on left but shifting with change of position
on right side
A Ballance Sign
B Grey Turner's Sign
C Cullen's Sign
D Seat Belt Sign
ANSWER:A
267 Prevalence of B P H :
A 30 - 40 years old > 50 %

B 50 - 60 years old > 75 % %
C 60 - 70 years old > 80 %
D > 80 years old > 90 %
ANSWER:D
268 Protrusion or swelling abdominal may indicate:
A Perforated bowel.
B Rib fracture.
C Appendicitis.
D Internal bleeding.
ANSWER:D
269 Pulses in the abdominal pain patient should be checked:
A In each extremity.
B Every 20 minutes.
C Centrally.
D Immediately.
ANSWER:A
270 Regarding dry gangrene, one of the following is correct:
A
A. Characterized by dry and shriveled skin ranging in color from brown to purplish blue or black.
B Swelling, blistering and a wet appearance are common features of dry gangrene.
C Mostly caused by infection with the bacterium Clostridium perfringens.
D None of the above.
ANSWER:A
271 Risk factors of soft tissue infections are( select one is incorrect )
A Poor perfusion
B Inclusion of dead tissue or foreign material (e.g. military wounds );
C Malnutrition and immunosuppression;
D Insipidus Diabetes
ANSWER:D
272
Rough estimate of fluid requirement is 4 mL/% TBSA/kg with 50% given over first 8 hours and
remainder over next 16 hours.A third degree cirumferential burn in the arm and forearm region,
which of the following is most important for monitoring – (U.P.P.GM.E.E. 04)

A Blood gases
B Carboxy-oxygen level
C Macroglobiunria cryoglobinuria
D Peripheral pulse and circulation
ANSWER:D
273 Signs and symptoms of B P H :
A Voiding and storage symptoms
B Urinary retention
C Urinary tract infection
D Bladder stone
ANSWER:A
274 Skin preparation for surgery should be:
A Bathing before surgery
B shaving 1h before surgery
C Bathing before surgery and shaving 1h before surgery
D prepared 2-3 days before surgery
ANSWER:C
275 Surgical indication for B P H treatment :
A BPH complication;bladder stone/urinary retention
B B P H with I P S S < 7
C B P H with I P S S between 7 - 20
D B P H 50 g
ANSWER:A
276 Surgical procedure with major break in sterile technique:
A Contaminated wound
B Clean wound
C Clean contaminated wound
D Dirty wound
ANSWER:C
277
Surgical procedures that enter respiratory, alimentary, genitourinary tract under controlled
conditions are the characteristics of:
A Class I wound

B Class II wound
C Class III wound
D Class IV wound
ANSWER:B
278 Surgical Technic of uncomplicated congenital hydrocele and/or inguinal hernia is( one correct )
A Laparoscopic repair
B High ligation of patent processus vaginalis ( PPV)
C Herniotomy with inguinal canal wall repair
D Herniotomy + simple closure at the neck of sac + mesh
ANSWER:B
279 Symptoms of lower UTI
A Fever , costovertebral pain , nausea vomiting
B Irritative symptoms , suprapubic pain , foul-smelling urine
C Lower abdominal pain , fever ,
D Flank pain and radiated to right scapular
ANSWER:B
280 Symptoms of upper UTI
A Fever , costovertebral pain , nausea vomiting
B Irritative symptoms , suprapubic pain , foul-smelling urine
C Lower abdominal pain , fever ,
D Flank pain and radiated to right scapular
ANSWER:A
281 Temporary
A Ileostomy
B Colostomies performed for cancer tend to be permanent.
C Colostomies performed for a gunshot are usually temporary
D In a double barrel colostomy, from which stoma will the stool come out?
ANSWER:C
282 The advance stage of untreated intussusception is( one correct )
A Colicky general abdominal pain
B Colicky general abdominal pain with hypovolemic shock
C Ischemic necrosis in both intussusceptum and intussuscipiens with bacterial translocation

D Colicky general abdominal pain with sepsis
ANSWER:C
283
The client has a large burned area on the right arm. The burned area appears pink, has blisters, and
is very painful. How will the nurse categorize this injury?
A Full-thickness (third Degree)
B Partial-thickness superficial (Superficial Second degree)
C Partial-thickness deep (deep second degree)
D Superficial ( First degree)
ANSWER:B
284 The clinical features of Dry Gangrene, except:
A Crepitus
B Dry
C Black Eschar
D Pain
ANSWER:A
285
The differential diagnosis, between Prolapse of the intussusceptum through the anus and rectal
prolapse, based on ( one correct )
A Anorectal finger examination
B Endorectal ultrasonography
C Rectoscopy
D Barium enemas rectocolonoscopy
ANSWER:A
286 The lower urinary tract symptoms :
A Storage and voiding symptoms
B Hematuria
C Pyuria
D Lower abdominal pain
ANSWER:A
287
The nurse teaches the patient whose surgery will result in a sigmoid colostomy that the feces
expelled through the colostomy will be
A Fluid.
B Mushy.

C Solid.
D Semi-mushy.
ANSWER:C
288 The objectives of surgery for peritonitis are, all are correct Except one
A to remove all infected material
B Abdominal distention
C prevent late complications
D correct the underlying cause
ANSWER:B
289 The Physical Examination of the prostate is :
A D R E
B Palpation
C Percussion
D Auscultation
ANSWER:A
290 The possible dry gangrene’s management are, except:
A Consult vascular surgery
B Rule out wet gangrene
C Consult with endocrinologist
D Give antibiotics
ANSWER:D
291 The process of normal voiding :
A Quickly - Easily - Clearly
B Quickly - Clearly - Easily
C Clearly - Easily - Quickly
D Clearly - Quickly - Easily
ANSWER:A
292 The signs of an abdominal injury include all of the following except:
A Lacerations and puncture wounds to the lower back
B Large bruises on the abdomen
C Indications of developing shock
D Contusions over the upper ribs

ANSWER:D
293 The treatment of B P H with I P S S < 7
A Medical
B Surgery
C Medical + surgery
D Chemotherapy
ANSWER:A
294 The type of gangrene that has no bacteria involvement in the process is known as
A Gas gangrene
B Wet gangrene
C Dry gangrene
D All of the above
ANSWER:C
295 Third rule of Leisky:
A Use cytotoxic substance, thorough cleaning with water or normal saline afterward
B Do not use cytotoxic substance, thorough cleaning with water or normal saline is preferred
C Moisten the wound if it is too dry, dry the wound if it is too wet
D Debridement of the necrotic tissue is required until the bleeding occurs
ANSWER:D
296 Transillumination test, ( figure. ) is important for the clinical diagnosis of( one correct )
A Inguino-scrotal hernia
B Hydrocele
C Epididymo-orchitis
D Testis tumor
ANSWER:B
297 U T I definition
A Is a inflammatory of the urothelium to bacteria
B is a present of bacteria in urine ( bacteriuria )
C is a present of red blood cell in urine
D is a present of virus in urine
ANSWER:A
298 UC- Ulcerative & exudative, Crohn's- granulomatous

A Region of bowel affected
B Risk of Colon cancer
C Complications
D Type of inflammation
ANSWER:D
299 UC-Less common, Crohn's-More common.
A Family history
B Complications
C Diagnostics
D Fistula, fissure, or abscess
ANSWER:B
300 Ulcerative colitis is most closely related to _________________.
A Pancreatitis
B Rectal bleeding
C Crohn's disease
D Acid reflux
ANSWER:C
301 Ulcerative Colitis with malignancy
A Has a better prognosis than Carcinoma Colon alone
B Is related to disease activity
C Is related to duration of ulcerative colitis
D Malignancy is more in ano rectal ulcerative colitis
ANSWER:C
302 Upon the wound classification system Class I or Clean wound :
A
An uninfected, no inflammation A surgical wound in which the respiratory, alimentary, genital, or
urinary tracts are entered
B
This includes open, fresh, accidental wounds. In addition, surgical procedures with major breaks in
sterile technique.
C This includes old traumatic wounds with retained devitalized tissue
D
An uninfected surgical wound in which no inflammation is encountered and the uninfected
respiratory, alimentary, genital, or urinary tract is not entered.
ANSWER:D

303 Urinalysis for localization of lower UTI
A Positive of WBC and Nitrite V1 = prostatitis
B Positive of WBC and Nitrite V2 = prostatitis
C Positive WBC and Nitrite V3 = prostatitis
D Positive WBC and Nitrite V3 = cystitis
ANSWER:C
304 What is another term for lymphatic nodules?
A Lymph follicles
B White pulp
C Peyer's patches
D Lymph node
ANSWER:A
305 What is clean wound?
A Uninfected surgical wound with no inflammation
B Respiratory, digestive, or genitourinary tract is not entered
C
Uninfected surgical wound with no inflammation, without entering respiratory, digestive, or
genitourinary tract
D
Uninfected surgical wound with no inflammation but entering respiratory, digestive, or
genitourinary tract
ANSWER:C
306 What is the acronym for the diffuse lymphatic tissue found in the intestinal tract?
A BALT
B DALT
C FALT
D GALT
ANSWER:D
307 What is the common name for someone with a stoma?
A Ostomate
B Stomite
C Colostomist
D Ileostomist
ANSWER:A

308 What is the most important aspect of management of burn injury in the first 24 hours ?
A Fluid resuscitation
B Dressing
C Escharotomy
D Antibiotics
ANSWER:A
309 What is the Post-operative care ?
A Is the care you receive immediate after surgical procedure.
B Is the care you receive before surgical procedure.
C Is the care you receive during surgical procedure.
D Is the care you receive after recovery room.
ANSWER:A
310 What type of gangrene is likely to be caused by poor circulation within an area of the body?
A Gas gangrene
B Dry gangrene
C Wet gangrene
D Viral gangrene
ANSWER:B
311 What type of gangrene is likely to be caused by poor circulation within an area of body?
A Gas gangrene
B Meleney's gangrene
C Dry gangrene
D Wet gangrene
ANSWER:C
312 When the patient discharge home is called
A Intermediate post-operative care.
B Immediate post-operative care.
C Convalescent post-operative care.
D Emergency post-operative care.
ANSWER:C
313 When the patient leave from operative room to recovery room is called
A Intermediate post-operative care.

B Immediate post-operative care.
C Convalescent post-operative care.
D Emergency post-operative care.
ANSWER:B
314 Which of the conditions below would necessitate an ostomy?
A Tumor obstructing the digestive tract lumen
B Congenital absence of one ureter
C Placement of a feeding tube into the stomach
D An auto accident in which the iliac and pubic bones are fractured
ANSWER:A
315 Which of the following is the most common symptom of rectal cancer?
A Pain.
B Mucus discharge.
C Haematochezia.
D Tenesmus.
ANSWER:C
316 Which seromarker is specific for Crohn’s disease
A ASCA
B p-ANCA
C Anti-pancreatic Ab
D Anti-Flagellin Ab
ANSWER:B
317
Which type of fluid should the nurse expect to prepare and administer as fluid resuscitation during
the emergent phase of burn recovery?
A Crystalloids
B Colloids
C Fresh-frozen plasma
D Blood transfusion
ANSWER:A
318
You are performing an US study on a patient with a history of recent blunt left sided trauma in a
MVA. Which finding below is not associated with this history?
A Splenic laceration

B Subcapsular hematoma of the spleen
C Free fluid
D Perisplenic hematoma
ANSWER:A
319
A 17-year-old boy complains of pain in the lower abdomen (mainly on the right side). Symptoms
commenced 12 hours before admission. He had noted anorexia in this period. Examination reveals
tenderness in the right iliac fossa, which was maximal 1cm below Mc Burney’s point. In
appendicitis, where does the pain frequently commence?
A In the right iliac fossa and remains there.
B In the back and moves to the right iliac fossa.
C In the rectal region and moves to the right iliac fossa.
D In the umbilical region and moves to the right iliac fossa.
ANSWER:D
320
A 55 y/o man undergoes low anterior resection of a low rectal cancer returns to clinic 6 months later
with constipation. You perform anoscopy in the office and notice a mass at the previous suture line.
Biopsies show adenocarcinoma. After a metastatic disease workup, the next appropriate step is:
A Do nothing
B Chemotherapy with Leucovorin
C 5000 rads of XRT
D Abdominal perineal resection
ANSWER:D
321
A 60 year old woman was found on routine screening to have irondeficiency anaemia. Which one
of the following is the most appropriatemanagement of the patient.
A prescribe an iron supplement
B arrange for the patient to undergo colonoscopy
C commence hormone replacement therapy
D arrange for a bone marrow aspirate
ANSWER:B
322
A patient presents with a small bowel obstruction and the early signs of sepsis. An abdominal xray
shows air in the biliary tree. Correct management is:
A ERCP with sphincterotomy

B Glucagon
C Exploratory laparotomy, removal of gallstone from small bowel and cholecystectomy
D Exploratory laparotomy, removal of gallsone from terminal ileum only
ANSWER:D
323
A ppendix shows the histology of the necrotic area, images of vascular thrombosis, an inflammatory
infiltrate small. They are:
A A normal appendix.
B A catarrhal appendicitis.
C A gangrenous appendix.
D An appendiceal abscess.
ANSWER:C
324 Acute appendicitis is most commonly associated with which of the following signs?
A Temperature above 40 Degree C.
B Frequent loose stools.
C Anorexia, abdominal pain, and right lower quadrant tenderness.
D White blood cell count greater than 20,000 per cu.mm.
ANSWER:C
325 All are the causes of acute appendicitis , Exclude ?
A Contact inflammation
B Rapid intestinal transit
C obstruction of the appendix
D Infection of Bactery
ANSWER:B
326 All are the factors which increase CVP , Exclude
A Deep inhalation
B Heart failure
C Tension pneumothorax
D Pleural effusion
ANSWER:A
327 All are the gestures regarding an acute appendicitis except ?
A Abdominal palpation
B Rectal palpation

C Examination testes
D Laparoscopy
ANSWER:D
328 All are the treatment of paralytic ileus except
A Mechanical obstruction or intra-abdominal sepsis
B Nasogastric suction
C Correction of electrolyte imbalance
D Colonoscopy
ANSWER:A
329 Anal fissure is diagnosed by
A TRUS
B Per rectal examination
C History and clinical examination with anoscope
D Proctoscopy
ANSWER:C
330 Complication of Short bowel syndrome is characterized by all of following except :
A Constipation
B Weight loss
C Gallstones
D Bacterial overgrowth
ANSWER:A
331 Complication of Short bowel syndrome is characterized by all of following except
A Malnutrition
B Diarrhea
C Kidney stone
D Hydration
ANSWER:D
332 Crohn's disease:
A Always affects the colon
B May lead to intestinal obstruction
C Is best treated surgically
D Requires a gluten free die

ANSWER:B
333 Enterocutaneous fistula (ECF) is an abnormal connection between
A bowel and bladder
B kidney and bowel
C vaginal and small intestine
D intestine and skin
ANSWER:D
334 External hemorrhoids below the dentate line are –
A Painful
B Ligation is done as management
C Skin tag is not seen in these cases
D May turn malignant
ANSWER:A
335 Hemorrhoids are common in pregnant women and people over the age of ___.
A 20
B 30
C 40
D 50
ANSWER:D
336 How does Crohn's disease affect the intestines?
A Crohn's disease causes ulcerations in the intestines.
B Crohn's disease causes the intestines to narrow.
C Crohn's disease causes the intestines to stiffen.
D
Crohn's disease causes ulcerations in the intestines, Crohn's disease causes the intestines to narrow,
Crohn's disease causes the intestines to stiffen.
ANSWER:D
337 How is Crohn's disease treated?
A Antibiotics
B Surgery
C Inflammation suppressant medications
D Antibiotics, Surgery, Inflammation suppressant medications
ANSWER:D

338
In a patient with the burn wound extending into the superficial epidermis without involving the
dermis would present with all of the following EXCEPT –
A Healing of the wound spontaneously without scar formation
B Anaesthesia at the site of burns
C Blister formation
D Painful
ANSWER:C
339 In childhood gastro-oesophageal reflux disease:
A Barium swallow is the best test
B Metoclopramide is useful treatment
C Respiratory symptoms are common
D Caused by gastrointestinal haemorrhage
ANSWER:C
340
Intravenous infusion of Vasopressin (the posterior pituitary hormone), causes splanchnic
vasoconstriction except one is not correct ?
A Variceal Hemorrhage
B decreased portal blood pressure.
C hypertension
D bradycardia.
ANSWER:A
341 Investigation of choice in recurrent anal fistula
A Endoanal USG
B Colonoscopy
C MRI
D Fistulography
ANSWER:C
342 Least common abscess is
A Perianal
B Ischiorectal
C Intersphicteric
D Supralevator
ANSWER:D

343 Local Sings findings in Peritonitis all are correct except One
A BP high and bradycardia
B Abdominal tenderness,
C Abdominal distention
D abdominal pain,
ANSWER:A
344
Local Sings findings in Peritonitis all are correct except One hematoma between dura mater and
arachnoid Causes increased intracranial pressure
A BP high and bradycardia
B Abdominal tenderness,
C Abdominal distention
D abdominal pain,
ANSWER:A
345 Most common fistula is
A intersphincteric
B Trans-sphincteric
C Suprasphincteric
D Extrasphincteric
ANSWER:B
346 Most common type of fistula-in-ano
A Inter-sphincteric
B Trans-sphincteric
C Supra-sphincteric
D Extra-sphincteric
ANSWER:A
347
Most patients who undergo bowel resection must take daily vitamin supplements aswell as
magnesium and calcium supplements. If patients have diarrhea after meals, whichof the following is
the most appropriate additional therapy?
A Cholestyramine
B Loperamide
C Teduglutide
D Vitamin B12

ANSWER:B
348
Move the patient's legs the pain felt in the right iliac fossa on extension of the hip, and the right hip
flexed for pain relief, is:
A The pointing sign.
B The psoas sign.
C The obturator sign.
D Rovsing's sign.
ANSWER:B
349 Newest treatment for anal incontinence?
A Sacral nerve stimulation
B Artificial sphincter
C Repair of sphincter
D Gluteus maximus graft
ANSWER:A
350
Obstruction due to strangulation of small bowel clamp (frange, adhesion) is characterized by,
EXCEPT ONE :
A A sudden onset.
B Severe pain.
C Vomiting late.
D Images with air-fluid folds haustraux on radiographs of the abdomen without preparation.
ANSWER:C
351 On patient liver cirrhosis which complication is life threatening?
A Hepatorenal syndrome
B Variceal hemorrhage
C Ascites
D Hepatic encephalopathy
ANSWER:B
352 Operation in the bowel obstruction may be delayed under certain circumstances except
A Pyloric obstruction
B Abdomial guarding or rigidity
C Obstruction develops immediately following an abdominal operation
D Crohn's disease,

ANSWER:B
353 Pain felt in the right iliac fossa when you press deeply in his left iliac fossa is:
A The pointing sign.
B The psoas sign.
C The obturator sign.
D Rovsing's sign.
ANSWER:D
354 Pain in the hypogastrium when internal rotation of the hip is :
A The pointing sign.
B The psoas sign.
C The obturator sign.
D Rovsing's sign.
ANSWER:C
355 platelet transfusions are necessary only when
A the platelet less than 50,000 /mm3 .
B the platelet less than 350,000/mm3
C hypertension
D prothrombin time is prolonged more than 3 seconds
ANSWER:B
356 Polyhydramnios is frequently observed in all of the following conditions except:
A Esophageal atresia.
B Duodenal atresia.
C Pyloric atresia.
D Hirschsprung's disease.
ANSWER:D
357 Position in surgery for pilonidal sinus
A sim’s position
B Trendelenburg position
C Lithotomy position
D Jackknife position
ANSWER:D
358 Regarding small bowel obstruction

A In Cambodia is most often due to an obstructed hernia.
B Causes colicky abdominal pain and vomiting.
C Abdominal distension is seen in all patients.
D All cases can be managed conservatively for the first 24 hours.
ANSWER:B
359
Rough estimate of fluid requirement is 4 mL/% TBSA/kg with 50% given over first 8 hours and
remainder over next 16 hours.A third degree cirumferential burn in the arm and forearm region,
which of the following is most important for monitoring.
A Blood gases
B Carboxy-oxygen level
C Macroglobiunria cryoglobinuria
D Peripheral pulse and circulation
ANSWER:D
360 Short bowel syndrome is characterized by all of following except :
A Steatorrhea
B Hypogastrinemia
C Diarrhea
D Weight loss
ANSWER:B
361 Sign of Pneumothorax
A Cardiac failure
B Hyperresonance
C Hypotension
D Distended neck veins
ANSWER:B
362 Signs of laryngeal or tracheal injury Which is INCORRECT?
A Hemoptysis
B Stridor
C Subcutaneous emphysema and/or crepitus
D Pulse deficit
ANSWER:D
363 Signs of Tracheobronchial injury

A Distant heart tones, Hypoxia
B Shock, Paradoxical pulse, Tachypnea
C Abnormal EKG- Tachypnea Hypotension
D Pneumomediastinum , Subcutaneous air, Persistent pneumothorax
ANSWER:D
364 Spasm of anal sphincter is most commonly associated with
A Prolapsed thrombosed hemorrhoids
B Fissure in ano
C Fistula in ano
D Mucosal prolapsed rectum
ANSWER:B
365 Stress ulcers seen in burns are
A Curling’s ulcer
B Cushing’s ulcer
C Meleney’s ulcer
D Rodent ulcer
ANSWER:A
36 The “gold standard” surgical operation for low rectal cancer is:
A Hartmann’s procedure
B anterior resection
C total mesorectal excision
D transverse colostomy
ANSWER:C
367 The best treatment for fissure –in-ano
A lateral sphincterotomy
B fissurectomy
C anal advancement flap
D Lord’s dilatation
ANSWER:A
368 The cardinal signs of obstruction syndrome Except one ?
A Abdominal pain and cramping
B Nausea, vomiting

C Unable to pass stool or gas
D Abdominal guarding.
ANSWER:D
369 The duodenum is the major portion of the small intestine where enzyme secretion Exclude ?
A Secretes lactase
B secretes lipase, amylase, trypsin, chymotrypsin and sodium bicarbonate
C Secretes petidase to split peptides
D Secretes mucus
ANSWER:C
370 The factors contributing to failure of spontaneous closure of the bowel fistular Except one ?
A Foreign body,
B Distal obstruction
C Neoplasm
D the platelet less than 350,000/mm3
ANSWER:D
371
The jejunum is the primary digestive and absorptive site in the bowel. When jejunal resection occurs
as a result of Crohn disease or cancer, significant reduction in nutrientabsorption results. The ileum
adapts by increasing the length and absorptive function of its villi. However, severe diarrhea and
bile acid malabsorption can occur if more thanwhat portion of the ileum is resected?
A 25 cm
B 50 cm
C 75 cm
D 100 cm
ANSWER:D
372 The metabolic derangement in the bowel fistula because of
A Trauma
B Foreign body
C Loss of large fluid volumes and Vomitting
D Inflammation or infection
ANSWER:C
373 The most common site for anal fissure
A 3 o’clock

B o’clock
C 2 o’çlock
D 10 o’clock
ANSWER:B
374 The most sensitive sign of traumatic aortic disruption is:
A Hypotension
B Jugular vein dissension
C Widen mediastinum
D Cyanosis
ANSWER:C
375 The timing of operation of bowel obstruction depends on three factors Excep ?
A consideration of the risk of strangulation.
B Duration of obstruction, and the
C Chronic partial obstruction
D the opportunity to improve vital organ function;
ANSWER:C
376 The timing of operation to the bowel obstruction depends on three factors Except?
A consideration of the risk of strangulation.
B Duration of obstruction, and the
C Chronic partial obstruction
D the opportunity to improve vital organ function;
ANSWER:C
377 The usual manifestations of appendicitis pelvic position include:
A Acute retention of urine.
B An abscess in the cul-de-sac.
C A psoïtis.
D A pain accurate in the digital rectal.
ANSWER:D
378 there are complications of Acute appendicitis in a young adult except ;
A Intestinal obstruction.
B An intermittent dysphagia.
C Peritonitis.

D Intra-abdominal and pelvic abscesses
ANSWER:B
379 Three mechanisms of Penetrating thoracic injuries Except
A direct blow to the chest (e.g., rib fracture),
B Respiratory Distress
C deceleration injury (e.g., pulmonary or cardiac contusion and aortic tear),
D compression injury (e.g., cardiac and diaphragm rupture).
ANSWER:B
380 To rewarm an area of the body that’s frostbitten, you should use:
A Hot water
B Warm water
C Saline solution
D Cold water
ANSWER:B
381 Treatment for complex fistula in crohns are all except
A Draining seton
B Fistulectomy
C Metronidazole
D infliximab
ANSWER:B
382 Treatment of Anal Fissure
A Injection of 5% phenol in almond oil.
B Local application of 0.2% GTN ointment.
C Anal dilatation.
D Lateral internal sphincterotomy.
ANSWER:D
383 Tuberculosis Caused by infection with a bacteria
A Mycobacterium tuberculosis
B Spread like the common cold
C Respiratory droplets in the air
D Coughing, sneezing, talking, singing….TB Q
ANSWER:A

384 Ulcerative Colitis
A Carries a risk of colonic cancer
B Can be treated with a LOFFLEX diet
C Can be caused by small intestinal bacterial overgrowth
D Can be caused by milk intolerance
ANSWER:A
385 What are the surgical management for intestinal obstruction?
A Removal of damaged intestine, Surgical resection, Laparotomy,Hartmann’s procedure
B Removal of damaged intestine, Surgical resection, Stenting, Colostomy,Removal of adhesion
C Colostomy, Hartmann’s procedure, Laparotomy, Stenting
D Removal of adhesion, Removal of damaged intestine, Hartmann’sprocedure, Stenting
ANSWER:B
386 What are treatments for hemorrhoids?
A Scarring
B Rubber banding
C Heat and cold therapies
D All of the above
ANSWER:B
387 What cause of Anal fistula
A Chronic diarrhea
B Constipation
C Prolonged sitting
D Obstruction and infection of the anorectal glands
ANSWER:D
388 What causes Crohn's disease?
A Stress
B Lack of vitamin D
C Inflammation of the bowel by specific chemicals
D The cause of Crohn's disease is unknown
ANSWER:D
389 What causes hemorrhoids?
A Pregnancy

B Being overweight
C Diarrhea
D All of the above
ANSWER:D
390 What causes of anal incontinence
A Prolonged sitting
B Pregnancy
C Obesity
D Obstetric injuries
ANSWER:D
391 What degree of frostbite is characterized by erythema but no blister?
A First
B Second
C Third
D Fourth
ANSWER:A
392 What examination to different an acute appendicitis subhepatic with acute gallstone cholecystitis?
A Xray
B Abdominal sonography
C Bilirubinemia
D WBC counted
ANSWER:B
393 What happen if air accumulate in pleural space?
A Lung collapse partial or completely
B Decrease oncotic pressure
C Increase hydrostatic pressure
D Inflammation
ANSWER:A
394 What is a hemorrhoid? A hemorrhoid is...
A A swollen vein located in the anal canal
B A tear in the lining of the anal canal
C A sexually transmitted disease

D An anal abscess
ANSWER:A
395 What is a thrombosis?
A A blood clot
B The medical term for "skin tag"
C A protruding hemorrhoid
D A painful hemorrhoid
ANSWER:A
396 What is bowel fistula?
A Is an abnormal communication between organ to skin.
B
Is an abnormal communication between a hollow viscus and some other organ or structure,
including the skin.
C Is an abnormal communication between stomach to intestine.
D Is an abnormal communication between intestine to intestine.
ANSWER:B
397 What is cardiac contusion ?
A pressure on heart =>blood or fluid cover sac of heart
B brushing of heart muscle
C laceration of heart muscle
D laceration of ventricle or arterial of heart or septum or papillary muscle
ANSWER:B
398 What is Crohn's disease?
A A chronic inflammatory disease of the intestines
B Inflammation of the bowel by specific chemicals
C Inflammation chronic medications
D moderating caloric intake
ANSWER:A
399 What is massive Hemothorax?
A <1500 ml of blood in pleural space
B >1500 ml of blood in pleural space
C <1500 ml of blood in pleural cavity
D >1500 ml of blood in lung

ANSWER:B
400 What is most important prognostic feature of rectal carcinoma
A Nodal involvement
B Size
C Histology
D Location
ANSWER:A
401 What is pneumothorax?
A Presence of air in lung parenchyma
B Presence of air in pleural space
C Presence of fluid in pleural space
D Presence of blood in pleural space
ANSWER:B
402 What is primary management in thoracic trauma?
A Medication
B Surgery
C Airway rescue and O2 supplement
D Blood test
ANSWER:C
403 What is the appropriate treatment for frostbite?
A Slow warming
B Rapid warming
C Rubbing with warm hands
D Use of blanket
ANSWER:B
404 What kind of hemorrhoids form on the skin around the anus?
A External
B Internal
C Medial
D Systematic
ANSWER:A

405 When body tissue dies because of a lack of blood supply, as happens with deep frostbite, it is called:
A The tourniquet effect
B Overexposure
C Necrosis
D Goosebumps
ANSWER:C
406 Which are the four most common cancers in the UK
A Pancreas, bladder, brain, lung
B Stomach, breast, kidney, melanoma
C Breast, lung, prostate, colorectum
D Endometrium, brain, oesophagus, non-Hodgkin lymphoma
ANSWER:C
407 Which gynaecologic condition most commonly mimics appendicitis both clinically and on CT?
A Uterine leiomyoma.
B Endometriosis.
C Hemorrhagic ovarian cyst.
D Cervical carcinoma.
ANSWER:C
408 Which injuries are considered life threatening requiring immediate needle decompression?
A Tension pneumothorax
B Cardiac contusion and rib fractures
C Clavicle fracture and pulmonary contusion
D Pleural effusion
ANSWER:A
409 Which is not a complication of Hepatic Cirrhosis?
A Variceal hemorrhage
B Hepatocellular Carcinoma
C Cronh disease
D Portal Hypertension
ANSWER:C
410 Which of the following complications of portal hypertension often require surgical intervention?

A Hypersplenism
B Variceal hemorrhage
C Ascites
D Encephalopathy
ANSWER:B
411 Which of the following is not Causes of Portal Hypertension?
A Portal vein thrombosis,
B BP high and bradycardia
C Splenic vein thrombosis,
D Splanchnic arteriovenous fistula
ANSWER:B
412 Which of the following is not suggest in the resuscitation a patient with variceal hemorrhage?
A Vasopressin
B Endoscopic therapy
C Colonoscopy
D Injection vascular sclerosants
ANSWER:C
413
Which of the following is the most appropriate nutritional regimen for a patient whoundergoes
extensive resection of the bowel and who also has excessive fluid andelectrolyte losses?
A Diet high in carbohydrates
B Diet high in fat and protein
C Hydration with iso-osmotic solution
D Long-term total parenteral nutrition (TPN)
ANSWER:D
414 Which of the following most often initiates the development of acute appendicitis?
A A viral infection.
B Acute gastroenteritis.
C Obstruction of the appendiceal lumen.
D A primary clostridial infection.
ANSWER:C
415 Which of the following statement (s) is/are true concerning postoperative ileus?

A
The use of intravenous patient-controlled analgesia has no effect on return of small bowel motor
activity.
B
The presence of peritonitis at the time of the original operation delays the return of normal bowel
function.
C The routine use of metoclopramide will hasten the return of small intestinal motor activity.
D
Contrast radiographic studies have no role in distinguishing early postoperative bowel obstruction
from normal ileus.
ANSWER:B
416 Which of the following statement(s) is/are true concerning inhalation injury?
A
The physiology of these injuries include upper airway obstruction secondary to progressive edema,
reactive bronchospasm from aerosolized irritants, and microatelectasis from loss of surfactant and
alveolar edema
B Endotracheal intubation is indicated immediately in all patients with suspected inhalation injury
C Distal airway injuries are usually caused by heat injury
D
Peak inspiratory pressures of &gt; 40 cm of water are indicated to maintain functional residual
capacity
ANSWER:A
417 Which of the following statements regarding the pathogenesis of appendicitis is false?
A Luminal obstruction is always the cause of acute appendicitis
B Luminal obstruction leads to increased pressure and distention of the appendix
C Obstruction of venous outflow and then arterial inflow results in gangrene
D
Obstruction of the lumen may occur from lymphoid hyperplasia, inspissated stool, or a foreign body
ANSWER:A
418 Which of these factors reduces the risk for cancer of the colorectum?
A Vitamin C
B Dietary fibre
C Alcohol
D Oestrogen
ANSWER:B
419 Which one of the following individuals is at highest risk of colorectal cancer:
A a 40 year old man whose mother developed colon cancer at the age of 70 years

B a 50 year old woman whose father developed colon cancer at the age of 45 years
C a 40 year old man who was diagnosed to have ulcerative colitis two years previously
D a 60 year old woman with a paternal aunt who developed rectal cancer at the age of 55 years
ANSWER:B
420
Which one of the following is indicated in an individual thought to be at high risk of colorectal
cancer:
A faecal occult blood testing
B rigid sigmoidoscopy
C MRI of the abdomen
D colonoscopy
ANSWER:D
421 Which one of the following is not right indication of thoracotomy due to Massive Haemothorax?
A >1500cc of blood evaluated immediately after chest tube.
B chest pain
C Increasing size of haemothorax on a chest film.
D Clotted haemothorax
ANSWER:B
422 Which one of the following is the most important clinical feature of the Cardiac Tamponade?
A Flat neck vein
B Tachycardia
C Beck’s Triad (3D’s)
D Fatigue
ANSWER:C
423 With regard to the clinical course of appendicitis, which of the following statements is/are true?
A
The typical history is one of vague abdominal pain, followed by periumbilical pain and, later, right
lower quadrant pain.
B Nausea and vomiting usually precede the pain
C Gross hematuria and pyuria are quite common
D Most patients present with constipation
ANSWER:A

424
A patient is diagnosed with fat embolism syndrome after sustaining fractures from a motor vehicle
accident. Which bone fracture is most commonly associated with the development of fat embolism
syndrome?
A Radial fracture
B Close Femur fracture
C Rib fracture
D Clavicle fracture
ANSWER:B
425
A patient’s D-dimer result is <500 ng/mL. The nurse knows that the D-dimer assesses _______ and
this result means?
A fibrin degradation fragment; positive for a blood clot
B platelet degradation protein; negative for a blood clot
C clotting factors; positive for a blood clot
D fibrin degradation fragment; negative for a blood clot
ANSWER:D
426 A pulsating abdominal mass usually indicates which of the following conditions?
A Abdominal aortic aneurysm
B Enlarged spleen
C Gastric distention
D Gastritis
ANSWER:A
427 All are the risk factors of the chronic leg ulcer Except ?
A High body mass index or obesity
B Diabetes
C Smoking
D Their body fat is distributed in the legs
ANSWER:D
428 All are the signs & symptoms of PE Except ?
A Shortness of Breath
B Chest pain
C Tachycardia (pulse>100)
D Low grade temperature

ANSWER:D
429
All are the indications for surgical intervention of vascular malformation of management an
arteriovenous fistula EXCEPT
A Hemorrhage
B Painful ischemia
C Congestive heart failure
D Tumor erosion
ANSWER:D
430 All are the sign of Branham Signs e when compress proximal to AVF Except :
A Decrease Heart rate or Normal
B Dyspnea
C Decreased pulsatile.
D Decrease drill
ANSWER:B
431 All the following are the factors increase risk of developing varicose vein , EXCEPT:
A Age
B Pregnancy
C Alcohol
D Long standing
ANSWER:C
432
An obese patient develops acute oedematous lower limb following a Pelvic surgery. Deep vein
thrombosis is suspected . The most useful investigation in this case would be
A Doppler imaging
B Fibrinogen uptake
C Venography
D Plethysmography
ANSWER:A
433 Based on Fountain Classification of PAD, stage 2 is defined as
A Symptomatic PAD
B Pain on exertion
C Pain at rest
D Asymptomatic PAD

ANSWER:B
434 Best method for diagnosis of Deep vein thromvosis is
A Doppler examination
B Plethysmography
C Contrast phlebography
D 1131 Fibrinogen studies
ANSWER:A
435 Brodie -Trendlenburg test demonstrates-
A Mid thigh perforation
B Deep vein thrombosis
C Sapheno femoral incompetence
D Calf perforators
ANSWER:C
436 Deep vein thrombosis is the condition in which-
A Large vein in the body begin to throb causing jerky movement
B Blood clot form in the vein
C There is inflammation of the vein
D Venography
ANSWER:B
437 Definition of varicose vein
A
permanently twisted, enlarged, elongated veins, edema, amputation. Commonly occur superficial
vein such as saphenous veins.
B
permanently twisted, enlarged, elongated veins. Commonly occur superficial vein such as saphenous
veins.
C
permanently twisted, enlarged, elongated veins, bleeding and edema. Commonly occur superficial
vein such as saphenous veins
D
permanently twisted, enlarged, elongated veins. Commonly occur superficial vein such as saphenous
veins and iliac vein.
ANSWER:B
438 Diagnosis of AV Fistula :
A Duplex Ultrasound , CT , MRA
B Physical Examination

C Taking History
D Radiology
ANSWER:A
439 Fat embolism is commonly seen in ?
A Head injuries
B Long bone fractures
C Drowning
D Hanging
ANSWER:B
440 Gluteus maximus
A forms the gluteal fold
B has four bursae beneath it
C has blood supply solely from the inferior gluteal artery
D is the chief control of the hip flextion
ANSWER:D
441 Grade I lymphedema means
A Pitting edema upto the ankle
B Pitting edema upto the knee
C Non-pitting edema
D Edema disapearing after overnight rest
ANSWER:D
442 Having a pulmonary embolism include risk of possible ?
A Sudden death
B Diabetes
C High blood pressure
D Amputation of limbs
ANSWER:A
443 In DVT all are seen except
A High fever
B Increased temperature at site
C Pain
D Tenderness

ANSWER:A
444 In which of the following areas is an abdominal aortic aneurysm most commonly located?
A Distal to the iliac arteries
B Distal to the renal arteries
C Adjacent to the aortic branch
D Proximal to the renal arteries
ANSWER:B
445 Initial anticoagulation drug use in MVT
A Warfarin
B Heparin
C Ribavarin
D Apixaban
ANSWER:B
446 Initial thrombus formation in small mesenteric vessels cause by
A Splenectomy
B Pancreatitis
C Hypercoagulation
D Local factor
ANSWER:C
447 Irregular heartbeat of abnormal heart rhythm refers to?
A Discardia
B Arrhythmia
C Atrial asynchrony
D Heart burn
ANSWER:B
448 Lymphedema is edema of a limb due to:
A Bacterial infection
B Viral infection
C Medication use
D
Lymphatic hypoplasia (primary) or to obstruction or disruption (secondary) of e.lymphatic vessels.
ANSWER:D

449 Management of AV Fistula
A Medical Therapeutic
B Endovascular embolization
C Major Surgery
D Not Treatment Because it’s resolve spontaneously
ANSWER:B
450 Mesenteric venous thrombosis can be present
A Acute, subacute, or chronic
B primary
C secondary
D Acute and chronic
ANSWER:A
451 Nicoladoni branham in arterioveinous fistular sign is –
A Compression cause bradycardia
B Compression cause tachycardia
C Hypotension
D Systolic filling
ANSWER:A
452 Nicoladoni branham sign is
A Compression cause bradycardia
B Compression cause tachycardia
C Hypotension
D Systolic filling
ANSWER:A
453 Pulsating varicose vein in a young adult is due to-
A Arteriovenous fistula
B Sapheno femoral incompetence
C Deep vein thrombosis
D Abdominal tumour
ANSWER:A
454 Regarding the adductor compartment
A adductor magnus lies between the anterior and posterior division of the obturator nerve

B adductor longus inserts into the upper two thirds of the linear aspect of the femur
C the hamstring part of adductor magnus supply by the tibial part of the sciatic nerve
D
the medial intermuscular septum separate the adductor compartment fom the posterior compartment
obtutor externus
ANSWER:C
455 Regarding the femoral artery;
A adductor magnus lies between it and the profunda femoris artery
B the profunda femoris vien lies behind the profunda femoris artery
C profunda femoris artery arise from its posterior surface
D
the lateral circumflex femoral artery separates the superficial and deep branches of the femoral nerve
ANSWER:D
456 Sign and symptoms of deep vein thrombosis (DVT) can:
A Redness, swelling, Tenderness, Warmth
B Rapid pules, Coughing up blood, Shortness of breath
C Bleeding, Thrombocytopenia, Chest pain
D dizziness, nausea , sweating
ANSWER:A
457 Symptoms of DVT included:
A High grade fever
B Swelling in the affected legs
C C.shortness of breath
D Cold of both legs
ANSWER:B
458 Tensor fasciae is supplied by;
A anterior division of femoral nerve
B superior gluteal nerve
C nerve to vastus lateralis
D inferior gluteal nerve
ANSWER:B
459 The complication of arteriovenous fistula
A Stenosis at the proximal venous limb

B Congestive heart failure
C Aneurysms
D Venous hypertension distal to the fistula
ANSWER:B
460 The increased pressure exerted by the heart pumping blood is referred to as:
A Systolic pressure
B Diastolic pressure
C Arterial pressure
D Ventricular pressure
ANSWER:A
461 The most comment Complications of Varicose Vein :
A Thrombophlebitis
B Headache
C Fever
D Rash
ANSWER:A
462 The most common cause of acquired arteriovenous fistula is;
A Bacterial infection
B Fungal infection
C Blunt trauma
D Penetrating trauma
ANSWER:D
463 The most severe form of fat embolism syndrome is characterized by what 1-3days post-injury?
A Myocardial Infarction
B Stroke
C Respiratory failure
D Pulmonary embolism
ANSWER:C
464
The nurse is assessing a patient, who has many risk factors for the development of a DVT, for signs
and symptoms of a deep vein thrombosis. What signs and symptoms below would possibly indicate
a deep vein thrombosis is present?
A Cool extremity

B Swelling
C Positive Homan’s sign
D B and C
ANSWER:D
465 the screw home movement in extension of the knee joint begin with tightening of the
A anterior cruciate ligament
B oblique popliteal ligament
C medial collateral ligament
D lateral collateral ligament
ANSWER:A
46 Tiibia anterior
A is supplied by the tibia nerve
B insert into the second metatarsal bones
C is pierded by the posterior tibia artry
D tendon perforates the superior extensior teticulum
ANSWER:D
467 Varicose veins of the lower limb.
A May require ligation of the femoral vein.
B Painful ischemia.
C May be injected with 3% phenol in almond oil.
D Predispose to deep venous thrombosis
ANSWER:D
468 VTE is term for what type of condition/s?
A Pulmonary Hypertension and Post Thrombotic Syndrome
B Ventricular Tachycardia
C Deep Vein Thrombosis and Pulmonary Embolism
D Vancomycin Resistant Enterococcus
ANSWER:A
469 What are some symptoms of DVT (a blood clot that forms in a vein)?
A Swelling
B Redness of the skin
C Pain, Weakness.

D Numbness.
ANSWER:D
470 What are valves in our veins?
A Thin walls of the veins that direct blood flow
B Cells that carry oxygen though the body and go through gas exchange at the capillaries
C Doors that help push blood forward through the arteries
D Doors that prevent blood back flow in veins
ANSWER:D
471 What are varicose veins?
A Veins that have become enlarged and disfigured
B Veins that are clotted and can no longer carry blood
C
Veins that have been transformed through deep vein thrombosis into arteries that carry blood to the
body
D Veins that have narrowed through plaque accumulation and are not as efficient at carryingblood
ANSWER:A
472 What is job strain?
A Stress on the job that causes CAD
B High work efforts from making decisions
C Muscle pain due to job demands
D High job demands with low decision authority
ANSWER:D
473 What is most frequent cause of pulmonary embolism ?
A Congestive heart failure
B Hemorrhagic fever
C Deep vein thrombosis
D Pneumonia
ANSWER:C
474 What is pulmonary embolism ?
A A pulmonary clot obstructing a pulmonary artery
B Right sided heart failure
C A blood clot obstructing the aorta
D None of the above

ANSWER:A
475 What is the definitive test used to diagnose an abdominal aortic aneurysm?
A Abdominal X-ray
B Arteriogram
C CT scan
D Ultrasound
ANSWER:B
476 What is the gold standard investigation that is used to compliment the diagnosis of varicose vein?
A MRI (Magnetic Resonance Imaging)
B MRV (Magnetic Resonance Venography)
C Duplex Ultrasound
D Angiography
ANSWER:C
477 What is the main option for repair when endovascular management fail?
A Stereotactic radiography
B Open surgery
C Sten graft
D Endovascular
ANSWER:B
478 What is the most common cause of abdominal aortic aneurysm?
A Atherosclerosis
B DM
C HPN
D Syphilis
ANSWER:A
479 What is the most common form of treatment for an AVF ?
A Surgery
B Endovascular embolization
C Amputation
D Microsurgery
ANSWER:B
480 What is the most common symptom in a client with abdominal aortic aneurysm?

A Abdominal pain
B Diaphoresis
C Headache
D Upper back pain
ANSWER:D
481
What is the name of the semi-hard substance composed of cholesterol, fat, calcium, and fibrin that
contributes to atherosclerosis?
A Lipids
B Collagen
C Plaque
D Lipoprotein
ANSWER:C
482 What is the pathway of arterial blood moving through the body?
A Arteries > Capillaries > Arterioles > Endothelial cells
B Arterioles > Arteries > Endothelial cells > Capillaries
C Capillaries > Arteries > Arterioles > Endothelial cells
D Arteries > Arterioles > Capillaries > Endothelial cells
ANSWER:D
483 What is the symptom of critical limb ischemia?
A Pain at rest
B Ulcer
C Gangrene
D Asymptomatic
ANSWER:A
484
When assessing a client for an abdominal aortic aneurysm, which area of the abdomen is most
commonly palpated?
A Right upper quadrant
B Directly over the umbilicus
C Middle lower abdomen to the left of the midline
D Midline lower abdomen to the right of the midline
ANSWER:C
485 Which of the following accurately explains the pathophysiology of Peripheral arterial disease?

A Blood clot that has form in the lower extremity
B A buildup of plaque within the artery
C The deterioration of cartilage within the joint
D Nerve damage to the legs and feet
ANSWER:B
486 Which of the following aneurysm is the rarest aneurysm among visceral artery aneurysms?
A Inferior mesenteric artery aneurysms
B Aortic aneurysm
C Splenic artery aneurysm
D Cerebral aneurysm
ANSWER:A
487 Which of the following blood vessel layers may be damaged in a client with an aneurysm?
A Externa
B Interna
C Media
D Interna and Media
ANSWER:C
488 Which of the following can help prevent blood clots?
A Move around after surgery
B Medication (anticoagulant)
C Exercise your legs while you’re sitting
D Make lifestyle changes:Lose weight,No smoking
ANSWER:D
489
Which of the following complications is of greatest concern when caring for a preoperative
abdominal aneurysm client?
A HPN
B Aneurysm rupture
C Cardiac arrythmias
D Diminished pedal pulses
ANSWER:B
490 Which of the following describe origin of the celiac trunk?

A
originates from the anterior aorta just below the diaphragm at the level of the thoracic vertebrae 12
(T12) or the first lumbar vertebra
B
originates from the posterior aorta just below the diaphragm at the level of the thoracic vertebrae 11
(T12) or the first lumbar vertebra
C
originates from the anterior aorta just below the diaphragm at the level of the thoracic vertebrae 10
(T12) or the second lumbar vertebra
D
originates from the posterior aorta just below the diaphragm at the level of the thoracic vertebrae 12
(T12) or the second lumbar vertebra
ANSWER:A
491 Which of the following groups of symptoms indicated a ruptured abdominal aneurysm?
A Lower back pain, increased BP, decreased RBC, increased WBC
B Severe lower back pain, decreased BP, decreased RBC, increased WBC
C Severe lower back pain, decreased BP, decreased RBC, decreased WBC
D Intermittent lower back pain, decreased BP, decreased RBC, increased WBC
ANSWER:C
492
Which of the following is of the greatest VTE risk to an individual? (please choose the most correct
answer)
A Immobility
B Dehydratation
C Oral contraception(the pill )
D History of VTE
ANSWER:D
493 Which of the following is the cause of Arcuate ligament syndrome ?
A compression of the celiac trunk by the median arcuate ligament of the diaphragm
B compression of the celiac trunk by the ligament of the diaphragm
C compression of the celiac trunk by the median arcuate ligament
D compression of the celiac trunk by the Lateral arcuate ligament of the diaphragm
ANSWER:A
494 Which of the following is The classic description of chronic mesenteric ischemia
A
crampy or colicky pain located in the epigastric area that begins 15–30 minutes following eating,
lasts for 2–3 hours, and gradually subsides.

B
crampy or colicky pain located in the epigastric area that begins 15–30 minutes following eating,
lasts for 2–3 hours, and without subsides.
C
crampy or colicky pain located in the epigastric area that begins 15–30 minutes before and after
eating, lasts for 2–3 hours, and gradually subsides
D
crampy or colicky pain located in the epigastric area that begins 15–30 minutes before and after
eating, lasts for 2–3 hours, and without subsides.
ANSWER:A
495 Which of the following is the common treatment for severe carotid artery disease?
A Carotid endarterectomy
B laparoscopy
C Neck dissection
D Laparostomy
ANSWER:A
496 Which of the following is the location of the SMA ?
A
A is located a few centimeters below the celiac artery, usually around the first lumbar vertebra at
20°–30° caudal angulation
B
is located a few centimeters below the celiac artery, usually around the second lumbar vertebra at
20°–30° caudal angulation
C
is located a few centimeters below the celiac artery, usually around the Third lumbar vertebra at
20°–30° caudal angulation
D
is located a few centimeters upper the celiac artery, usually around the first lumbar vertebra at
20°–30° caudal angulation
ANSWER:A
497 Which of the following is the most common way to diagnose PAD?
A Toe-brachial index
B Ankle-brachial index
C CT angiography
D C- reactive protein and Interleukin-6 levels are typically decreased
ANSWER:B
498 Which of the following is the possible complication of atherosclerotic renal artery stenosis?
A Kidney failure, requiring treatment with dialysis or a kidney transplant
B Heart damage, requiring surgery intervention

C Liver cirrhosis, requiring transplant
D Peritonitis, requiring emergency laparotomy
ANSWER:A
499 Which of the following is the possible complication of atherosclerotic renal artery stenosis?
A Shortness of breath due to a sudden buildup of fluid in the lungs
B Shortness of breath due to progressively buildup of fluid in the lungs
C Shortness of breath due to a sudden buildup of blood in the lungs
D Shortness of breath due to a progressively buildup of air in the lungs
ANSWER:A
500 Which of the following is the procedure to treat atherosclerotic renal artery stenosis?
A Renal artery bypass surgery
B Renal angioplasty without stenting
C Urethroplasty
D Hartmann
ANSWER:A
501 Which of the following is the procedure to treat atherosclerotic renal artery stenosis?
A Renal angioplasty and stenting
B Renal angioplasty without stenting
C Urethroplasty
D Hartmann
ANSWER:A
502 Which of the following is the risk factor of atherosclerotic renal artery stenosis?
A High cholesterol
B Asthma
C Female
D Children
ANSWER:A
503 Which of the following is the risk factor of atherosclerotic renal artery stenosis?
A Obesity
B Asthma
C Female
D Children

ANSWER:A
504 Which of the following is true about Carotid artery disease?
A Carotid artery disease is caused by a buildup of plaques in arteries that deliver blood to your brain
B Carotid artery disease is caused by a buildup of plaques in vein that deliver blood to your brain
C
Carotid artery disease is caused by a buildup of plaques in spinal that deliver spinal fluid to your
brain
D Carotid artery disease is caused by a buildup of plaques in arteries that deliver blood to your heart
ANSWER:A
505 Which of the following is usually related to chronic mesenteric ischemia?
A Progressive atherosclerotic narrowing of the mesenteric arteries
B Progressive atherosclerotic of aortic arteries
C Progressive atherosclerotic of the colic artery
D Progressive atherosclerotic narrowing of the celiac artery
ANSWER:A
506 Which of the following is usually the cause Chronic mesenteric ischemia?
A atherosclerosis
B Fibromuscular disease
C Trauma
D Celiac artery Damage
ANSWER:A
507 Which of the following sentence is the risk factor for carotid artery disease?
A High blood pressure
B Asthma
C Intestinal disease
D female
ANSWER:A
508 Which of the following sentence is true about prevention carotid artery disease?
A Limit cholesterol and fat
B Drink more water as much as possible
C Drink less water as much as possible
D Eat less fruit and vegetable.
ANSWER:A

509 Which of the following sentence is two main causes of atherosclerotic renal artery stenosis ?
A Buildup plaque on kidney (renal) arteries, Fibromuscular dysplasia
B Buildup plaque on kidney (renal) vein, Fibromuscular dysplasia
C Buildup plaque in kidney (renal), Fibromuscular dysplasia
D Buildup plaque around kidney (renal) vein, Fibromuscular dysplasia
ANSWER:A
510
Which of the following sounds is distinctly heard on auscultation over the abdominal region of an
abdominal aortic aneurysm client?
A Bruit
B Crackles
C Dullness
D Friction rubs
ANSWER:A
511 Which of the following statements about arteriovenous fistula are correct?
A
The local features characteristic of an arteriovenous communication are demonstrated by the
presence of a thrill and bruit with aneurysmal dilatation. Vascular permanently twisted, enlarged,
elongated veins
B An arteriovenous fistula is best managed by ligation of the feeding vessels.
C Alterations to the constitution of blood which causes hypercoagulability.
D Branum's or Nicoladoni's sign is increased heart rate when the fistula is compressed.
ANSWER:A
512 Which of the following supplied the gastrointestinal tract ?
A the celiac trunk, the superior mesenteric artery (SMA), and the inferior mesenteric artery
B Aortic Abdomina, the superior mesenteric artery (SMA), and the inferior mesenteric artery
C the celiac trunk, common hepatic artery , and the inferior mesenteric artery
D the celiac trunk, the superior mesenteric artery (SMA), Common hepatic artery
ANSWER:A
513 Which of the following test is used to detect perforator incompetence in varicose-
A Morissey’s test
B b)Trendelenberg test
C Fegan’s test- (localise)
D Homan’s test

ANSWER:B
514 Which of the following that SMA supplies?
A Pancreas
B Liver
C Intestinal
D Left Colon
ANSWER:A
515 Which of the following that SMA supplies?
A Half of the right colon
B Liver
C Intestinal
D Half of the left Colon
ANSWER:A
516 Which of these is a sleep problem that is not associated with hypertension?
A Oversleeping
B Insomnia
C Early morning awakening
D Sleep walking
ANSWER:D
517 Which of these is not a risk factor for thromboembolism
A Myocardial infarction
B Hypertension
C Estrogen therapy
D Superficial thrombophlebitis
ANSWER:D
518 Which of these is not a typical symptom of a heart attack?
A Breaking out in a hot sweat
B Heartburn and/or indigestion
C Arm pain
D Shortness of breath
ANSWER:A
519 Which one is correct of symptoms of arteriovenous fistula Except ?

A Purplish, bulging veins
B Swelling in the arms or legs
C Decreased blood pressure
D Shortness of breath
ANSWER:D
520 which one is the ratio of systolic ankle blood pressure (BP) to systolic brachial BP is normal value?
A > 1.3
B 1.0-1.30
C 0.91-0.99
D 0.40-0.90
ANSWER:B
521 which one of procedure of compressing stocking
A compression force, which is greater in the foot and ankle than in the rest of the leg
B use the laser light for thermocoagulation
C worn the shock and exercise
D injection of sclerosing agent into vessel for sclerosed
ANSWER:A
522 Which one of the following is not the investigation of chronic venous leg ulcer?
A Venography
B Color Doppler ultrasound
C C.Scialography
D Ulcer biopsy
ANSWER:C
523
Which option below is considered a positive Homan’s Sign for the assessment of a deep vein
thrombosis (DVT)?
A The patient reports pain when the foot is manually dorsiflexed.
B The patient reports pain when the foot is manually plantarflexed.
C The patient experiences pain when the leg is extended.
D the patient experiences pain when the leg is flexed.
ANSWER:A
524 Which pulmonary sign appears in fat embolism?

A Seizure
B Petechia
C Fever
D Dyspnea
ANSWER:D
525 Which statement below is incorrect about a deep vein thrombosis (DVT)?
A
"Veins that are most susceptible to a deep vein thrombosis are the peroneal, posterior tibial, popliteal
and superficial femoral."
B "DVTs tend to mostly occur in the lower extremities but can occur in the upper extremities too."
C
"A deep vein thrombosis in the lower extremity has a low probability of becoming a pulmonary
embolism."
D
"A DVT is a type of venous thromboembolism (VTE), which is a blood clot that starts in the vein."
ANSWER:C
526 Which statement below is incorrect about a deep vein thrombosis (DVT)?
A
Veins that are most susceptible to a deep vein thrombosis are the peroneal, posterior tibial, popliteal
and superficial femoral.
B DVTs tend to mostly occur in the lower extremities but can occur in the upper extremities too.
C A DVT is a type of venous thromboembolism (VTE), which is a blood clot that starts in the vein.
D
A deep vein thrombosis in the lower extremity has a low probability of becoming a pulmonary
embolism.
ANSWER:D
527 Which vein mostly affect in varicose vein
A Saphenous veins
B Popliteal veins
C Fibular veins
D Tibia veins
ANSWER:A
528 Why do women get varicose veins so much more than men?
A Because of hormonal change
B Because they have thicker blood
C Because their blood circulation more slowly

D Because more of their body fat is distributed in the legs
ANSWER:A
529
Why might women in their 30s and 40s generally be less apt to develop heart disease than same-
aged men?
A Women have less testosterone than men
B Women have greater levels of estrogen
C Pregnancy acts as a female protection against heart disease
D Women have two X chromosomes
ANSWER:B
530
A patient with a nonobstructing carcinoma of the sigmoid colon is being prepared for elective resec-
tion. To minimize the risk of post- operative infectious complications, your planning should include
A A single preoperative parenteral dose of antibiotic effective against aerobes and anaerobes
B Avoidance of oral antibiotics to pre- vent emergence of Clostridium difficile
C
Postoperative administration for 2–4 days of parenteral antibiotics effective against aerobes and
anaer- obes
D
Postoperative administration for 5–7 days of parenteral antibiotics effectiveagainst aerobes and
anaer obes
ANSWER:C
531
A 78-year-old man with a history of coronary artery disease and an asymptomatic reducible
inguinal hernia requests an elective hernia repair. You explain to him that valid reasons for delaying
the proposed surgery include
A Coronary artery bypass surgery 3 mo earlier
B A history of cigarette smoking
C Jugular venous distension
D Hypertension
ANSWER:C
532 Following celiotomy, normal bowel motility can ordinarily be presumed to have returned
A In the stomach in 4 h, the small bowel in 24 h, and the colon after the first oral intake
B In the stomach in 24 h, the small bowel in 4 h, and the colon in 3 days
C In the stomach in 3 days, the small bowel in 3 days, and the colon in 3 days
D In the stomach in 24 h, the small bowel in 24 h, and the colon in 24 h

ANSWER:B
533
Which of the following surgical interventions is least likely to provide acceptable prolongation of
life for patients with AIDS?
A Splenectomy for AIDS-related idio- pathic thrombocytopenic purpura
B Colonic resection for perforation secondary to cytomegalovirus infection
C Cholecystectomy for acalculous cholecystitis
D Tracheostomy for ventilator- dependent patients with respira tory failure
ANSWER:D
534
A cirrhotic patient with abnor- mal coagulation studies due to hepatic synthetic dysfunction requires
an urgent cholecystec- tomy. A transfusion of fresh frozen plasma is planned to minimize the risk of
bleeding due to surgery. The optimal timing of this transfusion would be
A The day before surgery
B The night before surgery
C On call to surgery
D Intraoperatively
ANSWER:C
535
On postoperative day 3, an otherwise healthy 55-year-old man recovering from a partial hepatec-
tomy is noted to have scant serosanguineous drainage from his abdominal incision. His skin sta- ples
are removed, revealing a 1.0- cm dehiscence of the upper midline abdominal fasciA. Which of the
following actions is most appropriate? Pre- and Post-operative Care
A Removing all suture material and packing the wound with moist ster- ile gauze
B Starting intravenous antibiotics
C Placing an abdominal (Scultetus) binder
D Prompt resuturing of the fascia in the operating room
ANSWER:C
536
Five days after a sigmoid colec- tomy for cancer, a patient’s skin sta- ples are removed and a large
gush of serosanguineous fluid emerges. Examination of the wound reveals an extensive fascial
dehiscencE. The most appropriate management is
A Wide opening of the wound to assure adequate drainage
B Smear and culture of the fluid and appropriate antibiotics after the smear is reviewed
C Careful reapproximation of the wound edges with tape
D Immediate return to the operating room

ANSWER:B
537 Signs and symptoms of hemo- lytic transfusion reactions include
A Hypothermia
B Hypertension
C Polyuria
D Abnormal bleeding
ANSWER:D
538
The surgeon should be particu- larly concerned about which coagu- lation function in patients
receiving anti-inflammatory or analgesic med- ications?
A APTT
B PT
C Reptilase time
D Bleeding time
ANSWER:D
539 The substrate depleted earliest in the postoperative period is
A Branched-chain amino acids
B Non-branched-chain amino acids
C Ketone
D Glycogen
ANSWER:D
540 Diagnostic abdominal laparo- scopy is contraindicated in which of the following patients?
A A patient with rebound tenderness following a tangential gunshot wound to the abdomen
B A stable patient with a stab wound to the lower chest wall
C A patient with a mass in the head of the pancreas
D A young female with pelvic pain and fever
ANSWER:A
541
A 23-year-old woman under- goes total thyroidectomy for carcinoma of the thyroid glanD. On the
second postoperative day, she begins to complain of tingling sen- sation in her hands. She appears
quite anxious and later complains of muscle cramps. Initial therapy should consist of
A 10 mL of 10% magnesium sulfate intravenously
B Oral vitamin D
C 100 μg of oral Synthroid

D Continuous infusion of calcium Gluconate
ANSWER:D
542 Hypocalcemia is associated with
A Acidosis
B Shortened QT interval
C Hypomagnesemia
D Myocardial irritability
ANSWER:C
543 The enteric fluid with an elec- trolyte (Na+, K+, C1−) content simi- lar to that of Ringer’s lactate is
A Saliva
B Contents of small intestine
C Contents of right colon
D Pancreatic secretions
ANSWER:B
544
The two solutions most commonly used to maintain fluid and electrolyte balance in the postoper
ative management of patients are 5% dextrose in 0.9% sodium chloride and lactated Ringer’s
solution. Correct statements regarding lactated Ringer’s solution include which of the following?
A It contains a higher concentration of sodium ions than does plasma
B It is most appropriate for replacement of nasogastric tube losses
C It is isosmotic with plasma
D It has a pH of less than 7.0
ANSWER:D
545 Signs and symptoms associated with early sepsis include
A Respiratory acidosis
B Decreased cardiac output
C Cutaneous vasodilation
D Increased arteriovenous oxygen difference
ANSWER:C
546 The most common physiologic cause of hypoxemia is
A Hypoventilation
B Incomplete alveolar oxygen diffusion

C Ventilation-perfusion inequality
D Pulmonary shunt flow
ANSWER:C
547 Generally accepted indications for mechanical ventilatory support include
A PaO2 of less than 70 kPa and PaCO2 of greater than 50 kPa while breathing room air
B Alveolar-arterial oxygen tension difference of 150 kPa while breathing 100% O2
C Vital capacity of 40–60 mL/kg
D Respiratory rate greater than 35 breaths/min
ANSWER:D
548
In a hemolytic reaction caused by an incompatible blood transfusion, the treatment that is most
likely to be helpful is
A Promoting a diuresis with 250 ml of 50% mannitol
B Stopping the transfusion immediately
C Acidifying the urine to prevent hemoglobin precipitation in the renal tubules
D Removing foreign bodies, such as Foley catheters, which may cause hemorrhagic complications
ANSWER:B
549
Which of the following inhalation anesthetics accumulates in air-filled cavities during general
anesthesia?
A Diethyl ether
B Nitrous oxide
C Halothane
D Methoxyflurane
ANSWER:B
550 Dopamine is a frequently used drug in critically ill patients because
A At high doses it increases splanchnic flow
B At high doses it increases coronary flow
C At low doses it decreases heart rate
D At low doses it lowers peripheral resistance
ANSWER:B
551 Compensatory mechanisms during acute hemorrhage include
A Decreased cerebral and coronary blood flow
B Decreased myocardial contractility

C Renal and splanchnic vasodilation
D Increased respiratory rate
ANSWER:D
552
An 18-year-old woman develops urticaria and wheezing after an injection of penicillin. Her blood
pressure is 120/60 mm Hg, heart rate is 155 beats/min, and respiratory rate is 30 breaths/min.
Immediate therapy should include
A Intubation
B Epinephrine
C Beta blockers
D Iodine
ANSWER:B
553 During blood transfusion, clotting of transfused blood is associated with
A ABO incompatibility
B Minor blood group incompatibility
C Rh incompatibility
D Transfusion through Ringer’s lactate
ANSWER:D
554
Among patients who require nutritional resuscitation in an intensive care unit, the best evidence that
nutritional support is adequate is
A Urinary nitrogen excretion levels
B Total serum protein level
C Serum albumin level
D Serum transferrin levels
ANSWER:C
555
The accidental aspiration of gastric contents into the tracheobronchial tree should be initially treated
by
A Tracheal intubation and suctioning
B Steroids
C Intravenous fluid bolus
D Cricothyroidotomy
ANSWER:A
556 In performing a tracheostomy, authorities agree that

A The strap muscles should be divided
B The thyroid isthmus should be preserved
C The trachea should be entered at the second or third cartilaginous ring
D Only horizontal incisions should be usedv
ANSWER:C
557 Central venous pressure C.VP) may be decreased by
A Pulmonary embolism
B Gram-negative sepsis
C Positive-pressure ventilation
D Pneumothorax
ANSWER:B
558 Wasting of the intrinsic muscles of the hand can be expected to follow injury of the
A Ulnar nerve
B Radial nerve
C Brachial nerve
D Axillary nerve
ANSWER:A
559 With regard to wound healing, which one of the following statements is correct?
A Collagen content reaches a maximum at approximately 1 wk after injury
B Monocytes are essential for normal wound healing
C Fibroblasts appear in the wound within 24–36 h after the injury
D The function of the monocyte in wound healing is limited to phagocytosis of bacteria and debris
ANSWER:B
560
While you are on duty in the emergency room, a 12-year-old boy arrives with pain and
inflammation over the ball of his left foot and red streaks extending up the inner aspect of his leg.
He remembers removing a wood splinter from the sole of his foot on the previous day. The most
likely infecting organism is
A Clostridium perfingens
B Streptococcus
C Staphylococcus
D Escherichia coli
ANSWER:B

561 The true statement regarding tendon injuries in the hand is
A Flexor digitorum superficially inserts on the distal phalanx
B The process of healing a tendon injury involves formation of a tenoma
C The tendons of flexor digitorum superficialis arise from a common muscle belly
D
The best results for repair of a flexor tendon are obtained with injuries in the fibroosseous tunnel
(zone 2)
ANSWER:B
562 Which one of the following cases is considered a clean contaminated wound?
A Open cholecystectomy for cholelithiasis
B Herniorrhaphy with mesh repair
C Lumpectomy with axillary node dissection
D Appendectomy with walled-off abscess
ANSWER:A
563
A 25-year-old man is brought to the emergency room after sustaining burns during a fire in his
apartment. He has blistering and erythema of his face, left upper extremity, and chest with frank
charring of his right upper extremity. He is agitated, hypotensive, and tachycardiaC. Which one of
the following statements concerning this patient’s initial wound management is correct? Topical
antibiotics should not be used, as they will encourage growth of resistant organisms
A Early excision of facial and hand burns is especially important
B Escharotomy should only be per- formed if neurologic impairment is imminent
C
Excision of areas of third-degree or of deep second-degree burns usually takes place 3–7 days after
injury
D
Split-thickness skin grafts over the eschar of third-degree burns should be performed immediately in
order to prevent fluid loss
ANSWER:D
564
A 25-year-old man is brought to the emergency room after sustaining burns during a fire in his
apartment. He has blistering and erythema of his face, left upper extremity, and chest with frank
charring of his right upper extremity. He is agitated, hypotensive, and tachycardiaC. Which one of
the following statements regarding the above burn patient is correct?
A
This patient should undergo immediate intubation for airway protection and oxygen administration
B Tetanus prophylaxis is not necessary if the patient has been immunized in the previous 3 years

C This burn can be estimated at 60% total body surface area using the “rule of nines”
D The most sensitive indicator of adequacy of fluid resuscitation is heart rate
ANSWER:A
565 Which of the following statements regarding carpal tunnel syndrome is correct?
A It is rarely secondary to trauma
B It may be associated with pregnancy
C It most often causes dysesthesia during waking hours
D It is often associated with vascular compromise
ANSWER:B
56 Management of leukoplakia of the oral cavity includes
A Excisional biopsy of all lesions
B Application of topical antibiotics
C Low-dose radiation therapy
D Ascertaining that dentures fit properly
ANSWER:D
567
A teenage boy falls from his bicycle and is run over by a truck. On arrival in the emergency room,
he is awake and alert and appears frightened but in no distress. The chest radiograph suggests an air-
fluid level in the left lower lung field and the nasogastric tube seems to coil upward into the left
chest. The next best step in management is
A Placement of a left chest tube
B Immediate thoracotomy
C Immediate celiotomy
D Esophagogastroscopy
ANSWER:C
568 Which of the following conditions is most likely to follow a compression type abdominal injury?
A Diaphragmatic hernia
B Superior mesenteric thrombosis
C Mesenteric vascular injury
D Avulsion of the splenic pedicle
ANSWER:A

569
A 65-year-old man who smokes cigarettes and has chronic obstructive pulmonary disease falls and
fractures the 7th, 8th, and 9th ribs in the left anterolateral chest. Chest x-ray is otherwise normal.
Appropriate treatment might include
A Strapping the chest with adhesive tape
B Immobilization with sandbags
C Tube thoracostomy
D Peritoneal lavage
ANSWER:D
570 Blunt trauma to the abdomen most commonly injures which of the following organs?
A Liver
B Kidney
C Spleen
D Intestine
ANSWER:C
571
A 27-year-old man sustains a single gunshot wound to the left thigh. In the emergency room he is
noted to have a large hematoma of his medial thigh. He complains of paresthesias in his foot. On
examination there are weak pulses palpable distal to the injury and the patient is unable to move his
foot. The appropriate initial management of this patient would be
A Angiography
B Immediate exploration and repair
C Fasciotomy of anterior compartment
D Observation for resolution of spasm
ANSWER:B
572
A 25-year-old woman arrives in the emergency room following an automobile accident. She is
acutely dyspneic with a respiratory rate of 60 breaths/min. Breath sounds are markedly diminished
on the right sidE. The first step in managing the patient should be to
A Take a chest x-ray
B Draw arterial blood for blood gas determination
C Decompress the right pleural space
D Perform pericardiocentesis
ANSWER:C

573
A 25-year-old woman arrives in the emergency room following an automobile accident. She is
acutely dyspneic with a respiratory rate of 60 breaths/min. Breath sounds are markedly diminished
on the right sidE. A chest x-ray of this woman before therapy would probably reveal
A Air in the right pleural space
B Shifting of the mediastinum toward the right
C Shifting of the trachea toward the right
D Dilation of the intrathoracic vena cava
ANSWER:A
574 Among the physiologic responses to acute injury is
A Increased secretion of insulin
B Increased secretion of thyroxine
C Decreased secretion of vasopressin A.DH)
D Decreased secretion of glucagon
ANSWER:A
575
In a stable patient, the management of a complete transection of the common bile duct distal to the
insertion of the cystic duct would be optimally performed with a
A Choledochoduodenostomy
B Loop choledochojejunostomy
C Primary end-to-end anastomosis of the transected bile duct
D Roux-en-Y choledochojejunostomy
ANSWER:D
576
No operative management of penetrating neck injuries has been advocated as an alternative to
mandatory exploration in asymptomatic patients. Which of the following findings would constitute a
relative, rather than an absolute, indication for formal neck exploration?
A Expanding hematoma
B Dysphagia
C Dysphonia
D Pneumothorax
ANSWER:D
577
Following blunt abdominal trauma, a 12-year-old girl develops upper abdominal pain, nausea, and
vomiting. An upper gastrointestinal series reveals a total obstruction of the duodenum with a “coiled
spring” appearance in the second and third portions. Appropriate management is

A Gastrojejunostomy
B Nasogastric suction and observation
C Duodenal resection
D TPN to increase the size of the retroperitoneal fat pad
ANSWER:B
578
Following traumatic peripheral nerve transection, regrowth usually occurs at which of the following
rates?
A 0.1 mm per day
B 1 mm per day
C 5 mm per day
D 1 cm per day
ANSWER:B
579
A 28-year-old man is brought to the emergency room for a severe head injury after a fall. Initially
lethargic, he becomes comatose and does not move his right sidE. His left pupil is dilated and
responds only sluggishly. The most common initial manifestation of increasing intra-cranial pressure
in the victim of head trauma is
A Change in level of consciousness
B Ipsilateral (side of hemorrhagE. pupillary dilation
C Contralateral pupillary dilation
D Hemiparesis
ANSWER:A
580
A 28-year-old man is brought to the emergency room for a severe head injury after a fall. Initially
lethargic, he becomes comatose and does not move his right sidE. His left pupil is dilated and
responds only sluggishly. Initial emergency reduction of intracranial pressure is most rapidly
accomplished by
A Saline-furosemide (Lasix) infusion
B Urea infusion
C Hyperventilation
D Intravenous dexamethasone D.ecadron)
ANSWER:C

581
A 31-year-old man is brought to the emergency room following an automobile accident in which
his chest struck the steering wheel. Examination reveals stable vital signs, but the patient exhibits
multiple palpable rib fractures and paradoxical movement of the right side of the chest. Chest x-ray
shows no evidence of pneumothorax or hemothorax, but a large pulmonary contusion is
developing. Proper treatment would consist of which of the following?
A Tracheostomy, mechanical ventilation, and positive end-expiratory pressure
B Stabilization of the chest wall with sandbags
C Stabilization with towel clips
D Immediate operative stabilization
ANSWER:A
582
A 30-year-old man is stabbed in the arm. There is no evidence of vascular injury, but he cannot flex
his three radial digits. He has injured the
A Flexor pollicis longus and flexor digitus medius tendons
B Radial nerve
C Median nerve
D Thenar and digital nerves at the wrist
ANSWER:C
583
Following a 2-h fire-fighting episode, a 36-year-old fireman begins complaining of a throbbing
headache, nausea, dizziness, and visual disturbances. He is taken to the emergency room where his
carboxyhemoglobin C.OHB. level is found to be 31%. Appropriate treatment would be to
A Begin an immediate exchange transfusion
B Transfer the patient to a hyperbaric oxygen chamber
C Begin bicarbonate infusion and give 250mg acetazolamide D.iamox) intravenously
D Administer 100% oxygen by mask
ANSWER:D
584
An elderly pedestrian collides with a bicycle-riding pizza delivery man and suffers a unilateral
fracture of his pelvis through the obturator foramen. You would manage this injury by
A External pelvic fixation
B
Angiographic visualization of the obturator artery with surgical exploration if the artery is injured or
constricted
C Direct surgical approach with internal fixation of the ischia ramus
D Short-term bed rest with gradual ambulation as pain allows after 3 days

ANSWER:D
585 Regarding high-voltage electrical burns to an extremity
A Injuries are generally more superficial than those of thermal burns
B Intravenous fluid replacement is based on the percentage of body surface area burned
C Antibiotic prophylaxis is not required
D Evaluation for fracture of the other extremities and visceral injury is indicated
ANSWER:D
586
Which of the following fractures or dislocations of the extremities induced by blunt trauma is
associated with significant vascular injuries?
A Knee dislocation
B Closed posterior elbow dislocation
C Midclavicular fracture
D Supracondylar femur fracture
ANSWER:A
587 When operating to repair civilian colon injuries
A A colostomy should be performed for colonic injury in the presence of gross fecal contamination
B
The presence of shock on admission or more than two associated intraabdominal injuries is an
absolute contraindication to primary colonic repair
C Distal sigmoidal injuries should not be repaired primarily
D Right-sided colonic wounds should not be repaired primarily
ANSWER:A
588
If injury to a major artery in an extremity is suspected, surgical exploration should be carried out
regardless of the presence of palpa- ble pulses distal to the injury. The rationale is that the presence
of pal- pable distal pulses does not reliably exclude
A Significant arterial injury
B Significant injury to adjacent motor nerve trunks
C Significant injury to adjacent long bones
D Significant injury to adjacent veins
ANSWER:A
589
Animal and clinical studies have shown that administration of lactated Ringer’s solution to patients
with hypovolemic shock may
A Increase serum lactate concentration

B Impair liver function
C Improve hemodynamics by alleviating the deficit in the interstitial fluid compartment
D Increase metabolic acidosis
ANSWER:C
590
An 18-year-old high school football player is kicked in the left flank. Three hours later he develops
hematuriA. His vital signs are stablE. The diagnostic tests performed reveal extravasation of contrast
into the renal parenchymA. Treatment should consist of
A Resumption of normal daily activity excluding sports
B Exploration and suture of the laceration
C Exploration and wedge resection of the left kidney
D Antibiotics and serial monitoring of blood count and vital signs
ANSWER:D
591
An 18-year-old high school football player is kicked in the left flank. Three hours later he develops
hematuriA. His vital signs are stablE. Initial diagnostic tests in the emergency room should include
which of the following?
A Retrograde urethrography
B Retrograde cystography
C Arteriography
D Intravenous pyelogram
ANSWER:D
592 True statements concerning penetrating pancreatic trauma include
A The major cause of death is exsanguination from associated vascular injuries
B
Management of a ductal injury to the left of the mesenteric vessels is Rouxen-Y
pancreaticojejunostomy
C Management of a ductal injury in the head of the pancreas is pancreaticoduodenectomy
D Small peripancreatic hematomas need not be explored to search for pancreatic injury
ANSWER:A
593
Rapid fluid resuscitation of the hypovolemic patient after abdominal trauma is significantly
enhanced by which of the following?
A Placement of long 18-gauge subclavian vein catheters
B Placement of percutaneous femoral vein catheters
C Bilateral saphenous vein cutdowns

D Placement of short, largebore percutaneous peripheral intravenous catheters
ANSWER:D
594
Which of the following situa- tions would be an indication for performance of a thoracotomy in the
emergency room?
A Massive hemothorax following blunt trauma to the chest
B
Blunt trauma to multiple organ systems with obtainable vital signs in the field but none on arrival in
the emergency room
C Rapidly deteriorating patient with cardiac tamponade from penetrating thoracic trauma
D Penetrating thoracic trauma and no signs of life in the field
ANSWER:C
595
A 26-year-old man sustains a gunshot wound to the left thigh. Exploration reveals that a 5-cm
portion of superficial femoral artery is destroyeD. Appropriate management includes
A Debridement and end-to-end anas- tomosis
B Debridement and repair with an interposition prosthetic graft
C Debridement and repair with an interposition arterial graft
D Debridement and repair with an interposition vein graft
ANSWER:D
596
Laryngeal obstruction. Select the proper intervention for each of the immediately life-threatening
injuries of the chest listed below:
A Endotracheal intubation
B Cricothyroidotomy
C Subxiphoid window
D Tube thoracostomy
ANSWER:B
597
Open pneumothorax. Select the proper intervention for each of the immediately life-threatening
injuries of the chest listed below:
A Endotracheal intubation
B Occlusive dressing
C Subxiphoid window
D Tube thoracostomy
ANSWER:B

598
Flail chest. Select the proper intervention for each of the immediately life-threatening injuries of the
chest listed below:
A Endotracheal intubation
B Cricothyroidotomy
C Subxiphoid window
D Tube thoracostomy
ANSWER:A
599
Tension pneumothorax. Select the proper intervention for each of the immediately life-threatening
injuries of the chest listed below:
A Endotracheal intubation
B Cricothyroidotomy
C Subxiphoid window
D Tube thoracostomy
ANSWER:D
600
Pericardial tamponadE. Select the proper intervention for each of the immediately life-threatening
injuries of the chest listed below:
A Endotracheal intubation
B Cricothyroidotomy
C Subxiphoid window
D Tube thoracostomy
ANSWER:C
601 For which of the following malignancies does histologic grade best correlate with prognosis?
A Lung cancer
B Melanoma
C Colonic adenocarcinoma
D Hepatocellular carcinoma
ANSWER:D
602
The mass appears to be fixed to the underlying muscle, but the overly- ing skin is movablE. The
next most appropriate step in management is
A Above-knee amputation
B Excisional biopsy
C Incisional biopsy

D Bone scan
ANSWER:C
603
A 50-year-old man is incidentally discovered to have non- Hodgkin’s lymphoma confined to the
submucosa of the stomach during sophagogastroduodenoscopy for dyspepsiA. Which of the
following statements is true regarding his condition?
A Surgery alone cannot be considered adequate treatment
B Combined chemotherapy and radiation therapy, without prior resecton, are not effective
C The stomach is the most common site for non-Hodgkin’s lymphoma of the gastrointestinal tract
D
Outcome (freedom from progression and overall survival) is related to the histological grade of the
tumor
ANSWER:C
604 Which of the following statements regarding malignant parotid tumors is correct?
A Acinar carcinoma is a highly aggressive malignant tumor of the parotid gland
B Squamous carcinoma of the parotid gland exhibits only moderately malignant behavior
C
Regional node dissection for occult metastases is not indicated for malignant parotid tumors because
of their low incidence and the morbidity of lymphadenectomy
D
Facial nerve preservation should be attempted when the surgical margins of resection are free of
tumor
ANSWER:D
605 Which of the following statements regarding testicular cancer is true?
A
Lymph node dissection after radical orchiectomy is useful for staging but does not increase survival
B
Seminomas and choriocarcinomas are best treated with orchiectomy and retroperitoneal lymph node
dissection
C Cryptorchidism is associated with an increased risk of testicular cancer
D Orchiectomy for a testicular mass is approached via the scrotum
ANSWER:C
606 Regarding the risk of breast cancer, which of the following statements is true?
A Breast cancer occurs more com- monly among women of the lower social classes
B
A history of breast cancer in a first- degree family relative is associated with a fourfold increase in
risk

C
Women with a first birth after age 30 years have approximately twice the risk of those with a first
birth before age 18
D Cigarette smoking increases the risk of breast cancer
ANSWER:C
607
Human immunodeficiency virus (HIV) has been isolated from many body fluids. Which of the
following is a major source of transmission?
A Tears
B Sweat
C Semen
D Urine
ANSWER:C
608 What is the most common cause of cancer death among women?
A Breast cancer
B Ovarian cancer
C Lung cancer
D Endometrial cancer
ANSWER:C
609 Which of the following statements regarding adrenal cortical insufficiency is true?
A Treatment with exogenous steroids is usually ineffective
B
It is commonly seen as a consequence of metastasis of distant cancers, such as lung or breast, to the
adrenal glands
C The most common underlying cause today is infection with resis- tant tuberculosis
D Death from untreated chronic adrenal insufficiency may occur within hours of surgery
ANSWER:C
610
A 35-year-old woman under- goes her first screening mammogram. Which of the following
mammographic findings would require a breast biopsy?
A Breast calcifications larger than 2 mm in diameter
B Five or more clustered breast microcalcifications per square centimeter
C A density that effaces with compression
D Saucer-shaped microcalcifications
ANSWER:B
611 The diagnosis of primary hy- perparathyroidism is most strongly suggested by

A Serum acid phosphatase above 120 IU/L
B Serum alkaline phosphatase above 120 IU/L
C Serum calcium above 11 mg/dL
D Urinary calcium below 100 mg/day
ANSWER:C
612 Somatostatin contributes to which of the following processes?
A Inhibition of adrenocortical cells
B Inhibition of pancreatic α cells
C Stimulation of antral gastrin cells
D Stimulation of secretin-producing cells in the duodenum
ANSWER:B
613
Which of the following statements concerning Cushing syndrome secondary to adrenal adenoma is
true?
A Postoperative corticoid therapy is required to prevent hypoadrenalism
B
Biochemical and x-ray procedures are generally unsuccessful in lateralizing the tumors
preoperatively
C Exploration of both adrenal glands is indicated
D For uncomplicated tumors, an open transperitoneal surgical approach is usually employed
ANSWER:A
614
A 40-year-old woman is found to have a 1- to 2-cm, slightly tender cystic mass in her breast; she
has no perceptible axillary adenopa- thy. What course would you follow?
A Reassurance and reexamination in the immediate postmenstrual period
B Immediate excisional biopsy
C Aspiration of the mass with cytologic analysis
D Fluoroscopically guided needle localization biopsy
ANSWER:C
615
The course of papillary carci- noma of the thyroid is best described by which of the following
statements?
A
Metastases are rare; local growth is rapid; erosion into the trachea and large blood vessels is frequent
B Local invasion and metastases almost never occur, which makes the term carcinoma misleading
C Bony metastases are frequent and produce an osteolytic pattern par- ticularly in vertebrae

D
Metastases frequently occur to cervical lymph nodes; distant metas- tases and local invasion are rare
ANSWER:D
616
As an incidental finding dur- ing an upper abdominal CT scan, a 3-cm mass in the adrenal gland is
noteD. The appropriate next step in analysis and management of this finding would be
A Observation
B CT-guided needle biopsy
C Excision of the mass
D Measurement of urine catecholamine excretion
ANSWER:A
617
A woman sustains an injury to her chest after striking the steer- ing wheel of her automobile during
a collision. Which of the following statements concerning fat necrosis of the breast is true?
A Most patients report a history of trauma
B The lesion is usually nontender and diffuse
C It predisposes patients to the development of breast cancer
D It is difficult to distinguish from breast cancer
ANSWER:D
618
The most likely diagnosis in a patient with hypertension, hypo kalemia, and a 7-cm suprarenal mass
is
A Hypernephroma
B Cushing’s disease
C Adrenocortical carcinoma
D Pheochromocytoma
ANSWER:C
619 True statements regarding Paget’s disease of the breast include that it
A Usually precedes development of Paget’s disease of bone
B Presents with nipple-areolar ecze- matous changes
C Does not involve axillary lymph nodes because it is a manifestation of intraductal carcinoma only
D Accounts for 10–15% of all newly diagnosed breast cancers
ANSWER:B

620
A 40-year-old man who has a long history of peptic ulcer disease that has not responded to medical
therapy is admitted to the hospital. His serum gastrin levels are markedly elevated; at celiotomy, a
small, firm mass is palpated in the tail of the pancreas. Correct statements concerning this patient’s
condition include which of the following?
A Histamine or a protein meal will markedly increase basal acid secretion
B Secretin administration will suppress acid secretion
C The pancreatic mass will probably be benign
D Distal pancreatectomy is the treatment of choice
ANSWER:D
621
Of the common complications of thyroidectomy, the one that may be avoided through prophy-
laxis is
A Injury to the recurrent laryngeal nerve
B Injury to the superior laryngeal nerve
C Symptomatic hypocalcemia
D Thyroid storm
ANSWER:D
622
Following correction of the patient’s hypercalcemia with hydration and gentle diuresis with
furosemide, the most likely therapeutic approach would be
A Administration of maintenance doses of steroids
B Radiation treatment for bony metastases
C Neck exploration and resection of three out of four parathyroid glands
D Neck exploration and resection of a parathyroid adenoma
ANSWER:D
623 True statements about discharge from the nipple include
A Intermittent thin or milky dis- charge can be physiologic
B Expressible nipple discharge is an indication for open biopsy
C Bloody discharge is indicative of an underlying malignancy
D Galactorrhea is indicative of an underlying malignancy
ANSWER:A
624 True statements regarding Cushing’s disease and Cushing syndrome include which of the following?
A Adrenocortical hyperplasia is the most common cause of Cushing’s disease

B Overproduction of ACTH is pathognomonic of Cushing syndrome
C Clinical manifestations of Cushing’s disease and Cushing syndrome are identical
D Cushing syndrome is caused only by neoplasms of either the pituitary or adrenal glands
ANSWER:C
625
A 34-year-old woman has recurrent fainting spells induced by fasting. Her serum insulin levels
during these episodes are markedly elevateD. Correct statements regarding this patient’s condition
include which of the following?
A The underlying lesion is probably an α-cell tumor of the pancreas
B The underlying lesion is usually multifocal
C These lesions are usually malignant
D Serum calcium levels may be elevated
ANSWER:D
626 The incidence of breast cancer
A Increases with increasing age
B Has declined since the 1940s
C Is related to dietary fat intake
D Is related to coffee intake
ANSWER:A
627
Tumor not palpable, clinically positive lymph nodes fixed to one another, no evidence of
metastases. For the clinical description select the appropriate stage of breast cancer.
A Stage I
B Stage II
C Stage III
D Stage IV
ANSWER:C
628
Tumor 5.0 cm; clinically positive, movable ipsilateral lymph nodes; no evidence of metastases. For
the clinical description select the appropriate stage of breast cancer.
A Stage I
B Stage II
C Stage III
D Stage IV
ANSWER:B

629
Tumor 2.1 cm, clinically negative lymph nodes, no evidence of metastases For the clinical
description select the appropriate stage of breast cancer.
A Stage I
B Stage II
C Stage III
D Stage IV
ANSWER:B
630
Tumor not palpable but breast diffusely enlarged and erythematous, clinically positive
supraclavicular nodes, and evidence of metastases. For the clinical description select the appropriate
stage of breast cancer.
A Stage I
B Stage II
C Stage III
D Inflammatory carcinoma
ANSWER:D
631
Tumor 0.5 cm, clinically negative lymph nodes, pathological rib fracturE. For the clinical
description select the appropriate stage of breast cancer.
A Stage I
B Stage II
C Stage III
D Stage IV
ANSWER:D
632
A 43-year-old man presents with signs and symptoms of peri- tonitis in the right lower quadrant.
The clinical impression and sup- portive data suggest acute appen- dicitis. At exploration, however,
a tumor is found; frozen section sug- gests carcinoid features. For the tumor described, choose the
most appropriate surgical procedurE. A 2.5-cm tumor at the base of the appendix.
A Appendectomy
B Segmental ileal resection
C Cecectomy
D Right hemicolectomy
ANSWER:D

633
A 43-year-old man presents with signs and symptoms of peri- tonitis in the right lower quadrant.
The clinical impression and sup- portive data suggest acute appen- dicitis. At exploration, however,
a tumor is found; frozen section sug- gests carcinoid features. For the tumor described, choose the
most appropriate surgical procedurE. A 1.0-cm tumor at the tip of the appendix.
A Appendectomy
B Segmental ileal resection
C Cecectomy
D Right hemicolectomy
ANSWER:A
634
A 43-year-old man presents with signs and symptoms of peri- tonitis in the right lower quadrant.
The clinical impression and sup- portive data suggest acute appen- dicitis. At exploration, however,
a tumor is found; frozen section sug- gests carcinoid features. For the tumor described, choose the
most appropriate surgical procedurE. A 0.5-cm tumor with serosal umbilication in the ileum.
A Appendectomy
B Segmental ileal resection
C Cecectomy
D Right hemicolectomy
ANSWER:B
635
A 43-year-old man presents with signs and symptoms of peri- tonitis in the right lower quadrant.
The clinical impression and sup- portive data suggest acute appen- dicitis. At exploration, however,
a tumor is found; frozen section sug- gests carcinoid features. For the tumor described, choose the
most appropriate surgical procedurE. A 1.0-cm tumor of the midappendix; 1-cm firm, pale lesion at
the periphery of the right lobe of the liver
A Hepatic wedgeresection and appropriate bowel resection
B Segmental ileal resection
C Cecectomy
D Right hemicolectomy
ANSWER:A

636
A 43-year-old man presents with signs and symptoms of peri- tonitis in the right lower quadrant.
The clinical impression and sup- portive data suggest acute appen- dicitis. At exploration, however,
a tumor is found; frozen section sug- gests carcinoid features. For the tumor described, choose the
most appropriate surgical procedurE. A 3.5-cm tumor encroaching onto the cecum and extensive
liver metastases
A Appendectomy
B Segmental ileal resection
C Cecectomy
D Right hemicolectomy
ANSWER:C
637
Omeprazole has been added to the H2 antagonists as a therapeu- tic approach to the management of
acute gastric and duodenal ulcers. It acts by
A Blocking breakdown of mucosal- damaging metabolites of NSAIDs
B Providing a direct cytoprotective effect
C Buffering gastric acids
D Inhibiting parietal cell hydrogen- potassium-ATPase
ANSWER:D
638
Evidence that a splenectomy might benefit a patient with immune (idiopathiC. thrombocy- topenic
purpura (ITP) includes
A A significant enlargement of the spleen
B A high reticulocyte count
C Megakaryocytic elements in the bone marrow
D An increase in the platelet count on cortisone therapy
ANSWER:D
639 Which of the following state- ments concerning imperforate anus is true?
A Imperforate anus affects males more frequently than females
B
In 90% of males, but only 50% of females, the rectum ends below the level of the levator ani
complex
C The rectum usually ends in a blind pouch
D
The chance for eventual continence is greater when the rectum has descended to below the levator
ani muscles
ANSWER:D

640
A patient with a history of familial polyposis undergoes a diagnostic polypectomy. Which of the
following types of polyps is most likely to be found?
A Villous adenoma
B Hyperplastic polyp
C Adenomatous polyp
D Retention polyp
ANSWER:C
641 What is the most common serious complication of an end colostomy?
A A.leeding
B Skin breakdown
C Parastomal hernia
D Colonic perforation during irrigation
ANSWER:C
642 Which of the following state- ments regarding pancreatic carci- noma is true?
A The majority of cases present with jaundice alone
B CT scan, angiography, and laparoscopy have been unsuccessful in predicting resectability
C If a patient is jaundiced, the resectability rate is less than 5%
D 99% of patients with pancreatic cancer have metastatic disease at the time of diagnosis
ANSWER:D
643
A 45-year-old woman is explored for a perforated duodenal ulcer 6 h after onset of symptoms. She
has a history of chronic peptic ulcer disease treated medically with minimal symptoms.The
procedure of choice is
A Simple closure with omental patch
B Truncal vagotomy and pyloroplasty
C Antrectomy and truncal vagotomy
D Highly selective vagotomy
ANSWER:C
644
A 45-year-old woman is explored for a perforated duodenal ulcer 6 h after onset of symptoms. She
has a history of chronic peptic ulcer disease treated medically with minimal symptoms. Six weeks
after surgery, the patient returns complaining of postprandial weakness, sweating, light-headedness,
crampy abdomi- nal pain, and diarrheA. The best management would be
A Antispasmodic medications E.g., Lomotil)

B Dietary advice and counseling that symptoms will probably abate within 3 month of surgery
C Dietary advice and counseling that symptoms will probably not abate but are not dangerous
D Workup for neuroendocrine tumor E.g., carcinoiD.
ANSWER:B
645
A 55-year-old man complains of chronic intermittent epigastric pain, and gastroscopy demonstrates
a 2-cm ulcer of the distal lesser curvaturE. Endoscopic biopsy yields no malignant tissuE. After a 6-
wk trial of H2 blockade and antacid therapy, the ulcer is unchange
A Proper therapy at this point is
B Repeat trial of medical therapy Local excision of the ulcer
C Billroth I partial gastrectomy
D Billroth I partial gastrectomy with vagotomy
ANSWER:C
646
A 60-year-old male alcoholic is admitted to the hospital with hematemesis. His blood pressure is
100/60 mm Hg, the physical examination reveals splenomegaly and ascites, and the initial
hematocrit is 25%. Nasogastric suction yields 300 mL of fresh blooD. After initial resuscitation, this
man should undergo
A Esophageal balloon tamponade
B Barium swallow
C Selective angiography
D Esophagogastroscopy
ANSWER:C
647
During an operation for car- cinoma of the hepatic flexure of the colon, an unexpected
discontinuous 3-cm metastasis is discovered in the edge of the right lobe of the liver. The surgeon
should
A
Terminate the operation, screen the patient for evidence of other metas-tases, and plan further
therapy after the reevaluation
B Perform a right hemicolectomy and a right hepatic lobectomy
C Perform a right hemicolectomy and a wedge resection of the metastasis
D Perform a cecostomy and schedule reoperation after a course of systemic chemotherapy
ANSWER:C

648
A 42-year-old man with no history of use of nonsteroidal anti- inflammatory drugs (NSAIDs) pre-
sents with recurrent gastritis. Infection with Helicobacter pylori is suspecteD. Which of the following
statements is true?
A Morphologically, the bacteria is a gram-positive, tennis-racket-shaped organism
B Diagnosis can be made by serologic testing or urea breath tests
C Diagnosis is most routinely achieved via culturing endoscopic scrapings
D
The most effective way to treat and prevent recurrence of this patient’s gastritis is through the use of
single drug therapy aimed at eradicating H.pylori
ANSWER:B
649 Which of the following her- nias follows the path of the spermatic cord within the cremaster muscle?
A Femoral
B Direct inguinal
C Indirect inguinal
D Spigelian
ANSWER:C
650
A 70-year-old woman has nausea, vomiting, abdominal dis- tention, and episodic, crampy mida-
bdominal pain. She has no history of previous surgery but has a long history of cholelithiasis for
which she has refused surgery. Her abdom- inal radiograph reveals a spherical density in the right
lower quadrant. Correct treatment should consist of
A Ileocolectomy
B Cholecystectomy
C Ileotomy and extraction
D Nasogastric tube decompression
ANSWER:C
651 Which of the following state- ments concerning Hirschsprung’s disease is true?
A It is initially treated by colostomy
B It is best diagnosed in the newborn period by barium enema
C It is characterized by the absence of ganglion cells in the transverse colon
D It is associated with a high inci- dence of genitourinary tract anomalies
ANSWER:A

652
Spontaneous closure of which of the following congenital abnormalities of the abdominal wall
generally occurs by the age of 4?
A Umbilical hernia
B Patent urachus
C Patent omphalomesenteric duct
D Omphalocele
ANSWER:A
653
Laparoscopic cholecystec- tomy is indicated for symptomatic gallstones in which of the following
conditions?
A Cirrhosis
B Prior upper abdominal surgery
C Suspected carcinoma of the gall bladder
D Morbid obesity
ANSWER:D
654
Infants with anorectal anom- alies tend to have other congenital anomalies. Associated abnormali-
ties include which of the following?
A Abnormalities of the cervical spine
B Hydrocephalus
C Duodenal atresia
D Heart disease
ANSWER:D
655
A 48-year-old woman devel- ops pain of the right lower quad- rant while playing tennis. The pain
progresses and the patient presents to the emergency room later that day with a low-grade fever, a
white blood count of 13,000, and com- plaints of anorexia and nausea as well as persistent, sharp
pain of the right lower quadrant. On examina- tion she is tender in the right lower quadrant with
muscular spasm and there is a suggestion of a mass effect. An ultrasound is ordered and shows an
apparent mass in the abdominal wall. Which of the fol- lowing is the most likely diagnosis?
A Acute appendicitis
B Cecal carcinoma
C Hematoma of the rectus sheath
D Torsion of an ovarian cyst

ANSWER:C
656 In determining the proper treatment for a sliding hiatal her- nia, the most useful step would be
A Barium swallow with cinefluoroscopy during Valsalva maneuver
B Flexible endoscopy
C 24-h monitoring of esophageal pH
D Measuring the size of the hernia
ANSWER:B
657 Which of the following state- ments regarding the etiology of obstructive jaundice is true?
A A markedly elevated SGOT and SGPT are usually associated with obstructive jaundice
B
When extrahepatic biliary obstruction is suspected, the first test should be endoscopic ultrasonog-
raphy E.US)
C A Klatskin tumor will result in extrahepatic ductal dilation only
D A liver-spleen scan will add signifi- cantly to the diagnostic workup for obstructive jaundice
ANSWER:B
658
A previously healthy 9-year- old child comes to the emergency room because of fulminant upper
gastrointestinal bleeding. The hemorrhage is most likely to be the result of
A Esophageal varices
B Mallory-Weiss syndrome
C Gastritis
D A gastric ulcer
ANSWER:A
659
Operative planning and pre- operative counseling for a patient with a rectal carcinoma can be best
provided if the patient is staged before surgery by
A Rectal endosonography
B Barium enema
C MRI of the pelvis
D CT scanning of the pelvis
ANSWER:A
60 Which statement regarding absorption by the small intestine is true?
A All but the fat in milk is digested and absorbed in humans by the end of the duodenum
B Complete absorption of carbohydrates in a normal meal occurs in the ileum
C In short gut syndrome, much of the dietary carbohydrate appears in the stool

D Aldosterone markedly decreases sodium transport across the gut mucosa
ANSWER:A
61
Local stimuli that inhibit the release of gastrin from the gastric mucosa include which of the fol-
lowing?
A Small proteins
B 20-proof alcohol
C Caffeine
D Acidic antral contents
ANSWER:D
62
For a symptomatic partial duodenal obstruction secondary to an annular pancreas, the operative
treatment of choice is
A A Whipple procedure
B Gastrojejunostomy
C Duodenojejunostomy
D Partial resection of the annular pancreas
ANSWER:C
63 Which of the following would be expected to stimulate intestinal motility?
A Fear
B Gastrin
C Secretin
D Acetylcholine
ANSWER:D
64 Which of the following statements concerning carcinoma of the esophagus is true?
A Alcohol has been implicated as a precipitating factor
B Squamous carcinoma is the most common type at the cardioesophageal junction
C It has a higher incidence in males
D It occurs more commonly in patients with corrosive esophagitis
ANSWER:D
65
A 30-year-old man with a duodenal ulcer is being considered for surgery because of intractable pain
and a previous bleeding epi- sodE. Serum gastrin levels are found to be over 1000 pg/mL (normal
40–150) on three separate determi- nations. The patient should be told that the operation of choice is

A Vagotomy and pyloroplasty
B Highly selective vagotomy and tumor resection
C Subtotal gastrectomy
D Total gastrectomy
ANSWER:B
66 The most common clinical presentation of idiopathic retroperitoneal fibrosis is
A Ureteral obstruction
B Leg edema
C Calf claudication
D Jaundice
ANSWER:A
67
In planning the management of a 2.8-cm epidermoid carcinoma of the anus, the first therapeutic
approach should be
A Abdominoperineal resection
B Combined radiation therapy and chemotherapy
C Local radiation therapy
D Systemic chemotherapy
ANSWER:B
68 Indications for operation in Crohn’s disease include which of the following?
A Intestinal obstruction
B Enterovesical fistula
C Free perforation
D Enterovaginal fistula
ANSWER:C
69
Which of the following organisms is most closely associ- ated with gastric and duodenal ulcer
disease?
A Campylobacter
B Cytomegalovirus
C Helicobacter
D Mycobacterium aviumintracellulare
ANSWER:C
670 Which statement regarding adenocarcinoma of the pancreas is true?

A It occurs most frequently in the body of the gland
B It carries a 1–2% 5-year survival rate
C It is nonresectable if it presents as painless jaundice
D
It can usually be resected if it pre- sents in the body or tail of the pan- creas and does not involve the
common bile duct
ANSWER:B
671 Correct statements concerning intussusception in infants include which of the following?
A Recurrence rates following treatment are high
B It is frequently preceded by a gastrointestinal viral illness
C
A 1- to 2-wk period of parenteral alimentation should precede surgical reduction when surgery is
required
D Hydrostatic reduction without surgery rarely provides successful treatment
ANSWER:B
672
A 32-year-old woman pre- sents to the hospital with a 24-h history of abdominal pain of the right
lower quadrant. She under- goes an uncomplicated appendec- tomy for acute appendicitis and is
discharged home on the fourth postoperative day. The pathologist notes the presence of a carcinoid
tumor (1.2 cm) in the tip of the appendix. Which of the following statements is true?
A The patient should be advised to undergo ileocolectomy
B The most common location of carcinoids is in the appendix
C The carcinoid syndrome occurs in more than half the patients with carcinoid tumors
D The tumor is an apudoma
ANSWER:D
673 Which of the following statements regarding direct inguinal hernias is true?
A They are the most common inguinal hernias in women
B They protrude medially to the inferior epigastric vessels
C They should be opened and ligated at the internal ring
D They commonly protrude into the scrotal sac in men
ANSWER:B
674 Which of the following state- ments regarding stress ulceration is true?
A It is true ulceration, extending into and through the muscularis mu- cosa
B It classically involves the antrum
C Increased secretion of gastric acid has been shown to play a causative role

D It frequently involves multiple sites
ANSWER:D
675 Which statement concerning cholangitis is correct?
A The most common infecting organ- ism is Staphylococcus aureus
B The diagnosis is suggested by the Charcot triad
C The disease occurs primarily in young, immunocompromised patients
D Cholecystostomy is the procedure of choice in affected patients
ANSWER:B
676
An 88-year-old man with a history of end-stage renal failure, severe coronary artery disease, and
brain metastases from lung cancer presents with acute cholecystitis. His family wants “everything
donE.” The best management option in this patient would be
A Tube cholecystostomy
B Open cholecystectomy
C Laparoscopic cholecystectomy
D Intravenous antibiotics followed by elective cholecystectomy
ANSWER:A
677 Dieulafoy’s lesion of the stomach is characterized by
A A large mucosal defect with underlying, friable vascular plexus
B Frequent rebleeding after endoscopic treatment
C Massive bleeding that requires subtotal gastrectomy
D Location in the proximal stomach
ANSWER:D
678
During an appendectomy for acute appendicitis, a 4-cm mass is found in the midportion of the
appendix. Frozen section reveals this lesion to be a carcinoid tumor. Which of the following
statements is true?
A No further surgery is indicated
B A right hemicolectomy should be performed
C There is about a 50% chance that this patient will develop the carcinoid syndrome
D Carcinoid tumors arise from islet cells
ANSWER:B
679 Correct statements regarding rectal carcinoid tumors include
A Endoscopic resection is sufficient for tumors smaller than 2 cm

B Patients frequently present with the carcinoid syndrome
C They are rapidly growing tumors
D Local recurrence is rare with complete resection of the primary lesion
ANSWER:D
680 Indications for surgical removal of polypoid lesions of the gallbladder include
A Size greater than 0.5 cm
B Presence of clinical symptoms
C Patient age of over 25 years
D Presence of multiple small lesions
ANSWER:B
681
A patient who has a total pancreatectomy might be expected to develop which of the following
complications?
A Diabetes mellitus
B Hypercalcemia
C Hyperphosphatemia
D Constipation
ANSWER:A
682
A 28-year-old previously healthy woman arrives in the emer- gency room complaining of 24 h of
anorexia and nausea and lower abdominal pain that is more intense in the right lower quadrant than
elsewherE. On examination she has peritoneal signs of the right lower quadrant and a rectal tem-
perature of 38.38°C (101.8°F). At exploration through incision of the right lower quadrant, she is
found to have a small, contained perfora- tion of a cecal diverticulum. Which of the following
statements regard- ing this situation is true?
A Cecal diverticula are acquired disorders
B Cecal diverticula are usually multiple
C Cecal diverticula are mucosal herniations through the muscularis propria
D Diverticulectomy, closure of the cecal defect, and appendectomy may be indicated
ANSWER:D
683 True statements regarding cavernous hemangiomata of the liver in adults include
A The majority become symptomatic
B They may undergo malignant transformation
C They enlarge under hormonal stimulation

D They should be resected to avoid spontaneous rupture and life threatening hemorrhage
ANSWER:C
684
A 72-year-old patient with an intractable type I ulcer along the incisura with a significant amount of
scarring along the entire length of the lesser curvature (SELECT 1 PROCEDURE. Select the
appropriate surgical procedure for the patient.
A Vagotomy and antrectomy
B Antrectomy alone
C Vagotomy and pyloroplasty
D Vagotomy and gastrojejunostomy
ANSWER:C
685
A 46-year-old patient with gastric outlet obstruction sec- ondary to ulcer disease and severe
inflammation around the pylorus and first and second portions of the duodenum (SELECT 1
PROCEDURE. Select the appropriate surgical procedure for the patient.
A Vagotomy and antrectomy
B Antrectomy alone
C Vagotomy and pyloroplasty
D Vagotomy and gastrojejunostomy
ANSWER:D
686
A 90-year-old patient with a bleeding duodenal ulcer (SELECT 1 PROCEDURE. Select the
appropriate surgical procedure for the patient.
A Vagotomy and antrectomy
B Antrectomy alone
C Vagotomy and pyloroplasty
D Vagotomy and gastrojejunostomy
ANSWER:C
687
A 36-year-old patient with a type III (pyloriC. ulcer that is refractory to medical treatment (SELECT
1 PROCEDURE. Select the appropriate surgical procedure for the patient.
A Vagotomy and antrectomy
B Antrectomy alone
C Vagotomy and pyloroplasty
D Vagotomy and gastrojejunostomy
ANSWER:A

688
The most common congenital diaphragmatic hernia in infants (SELECT 1 ABNORMALITY) Match
each description with the correct abnormality.
A Rupture of the diaphragm
B Paraesophageal hiatal hernia
C Sliding hiatal hernia
D Foramen of Bochdalek hernia
ANSWER:D
689
The hernia most likely to cause acute respiratory distress in infants (SELECT 1 ABNORMALITY)
Match each description with the correct abnormality.
A Rupture of the diaphragm
B Paraesophageal hiatal hernia
C Sliding hiatal hernia
D Foramen of Bochdalek hernia
ANSWER:D
690 Among the cardiovascular anomalies of newborns, the one most likely to present with cyanosis is
A Patent ductus arteriosus
B Coarctation of the aorta
C Transposition of the great vessels
D Ventricular septal defect
ANSWER:C
691 The superior vena cava syndrome is most frequently seen in association with
A Histoplasmosis (sclerosing mediastinitis)
B Substernal thyroid
C Thoracic aortic aneurysm
D Bronchogenic carcinoma
ANSWER:D
692
During endoscopic biopsy of a distal esophageal cancer, perfora- tion of the esophagus is suspected
when the patient complains of significant new substernal pain. An immediate chest film reveals air in
the mediastinum. You would recommend
A Placement of a nasogastric tube to the level of perforation, antibiotics, close observation
B Spit fistula C.ervical pharyngostomy), gastrostomy
C Left thoracotomy, pleural patch oversewing of perforation, drainage of mediastinum

D Esophagogastrectomy via celiotomy and right thoracotomy
ANSWER:D
693 A 3-year-old child with congenital cyanosis is most probably suffering from
A Tetralogy of Fallot
B Ventricular septal defect
C Tricuspid atresia
D Transposition of the great vessel
ANSWER:A
694 A correct statement concerning bronchial carcinoid tumors is that
A They frequently metastasize
B They most commonly arise in peripheral terminal bronchioles
C They rarely produce the carcinoid syndrome
D They are radiosensitive
ANSWER:B
695
Initial management of a patient who has a flaccid neurogenic bladder may include which of the
following measures?
A Surgical bladder augmentation
B Self-catheterization
C Supravesical urinary diversion
D Limiting fluid intake to less than 300 mL/day
ANSWER:A
696 Which of the following state- ments regarding hypospadias is correct?
A It is often associated with chordee (ventral curvature of the penis)
B It is associated with undescended testes in more than 50% of cases
C It is a rare fusion defect of the posterior male urethra
D It occurs sporadically, without evidence of familial inheritance
ANSWER:A
697
The recommended treatment for stage A (superficial and submucosal) transitional cell carcinoma of
the bladder is
A Local excision
B Radical cystectomy
C Radiation therapy

D Topical (intravesicular) chemotherapy
ANSWER:D
698
A 36-year-old man presents to the emergency room with renal coliC. A radiograph reveals a 1.5-
cm stonE. Which of the following statements regarding this disorder is correct?
A
Conservative treatment including hydration and analgesics will not result in a satisfactory outcome
B Serial kidney, ureter, bladder (KUradiographs should be used to follow this patient
C The urinalysis will nearly always reveal microhematuria
D When the acute event is correctly treated, this disease seldom recurs
ANSWER:A
699 Optimal management of bi- lateral undescended testicles in an infant is
A Immediate surgical placement into the scrotum
B
Chorionic gonadotropin therapy for 1 mo; operative placement into the scrotum before age 1 if
descent has not occurred
C Observation until the child is 2 years old because delayed descent is common
D
Observation until age 5; if no descent by then, plastic surgical scrotal prostheses before the child
enters school
ANSWER:B
700 Seminoma is accurately described by which of the following statements?
A It is the most common type of tes- ticular cancer
B Metastases to liver and bone are fre- quently found
C It does not respond to radiation
D The 5-year survival rate approaches 50%
ANSWER:A
701
A 10-year-old boy presents to the emergency room with testicular pain of 5 h duration. The pain
was of acute onset and woke the patient from sleep. On physical examination, he is noted to have a
high- riding, indurated, and markedly tender left testis. Pain is not diminished by elevation.
Urinalysis is unremarkablE. Which of the fol- lowing statements regarding the patient’s diagnosis
and treatment is true?
A
There is a strong likelihood that this patient’s father or brother has had or will have a similar event
B Operation should be delayed until a technetium scan clarifies the diag nosis

C
The majority of testicles that have undergone torsion can be salvaged if surgery is performed within
24 h
D If torsion is found, both testes should undergo orchiopexy
ANSWER:D
702 Genitourinary tuberculosis in a male patient is suggested by which of the following findings?
A Microscopic hematuria
B Bacteriuria without pyuria
C Unilateral renal cysts
D Painful swelling of the epididymis
ANSWER:A
703 Which of the following state- ments regarding carcinoma of the prostate is true?
A It has a higher incidence among American blacks than other American ethnic groups
B
A single microscopic focus of prostate cancer discovered on transurethral resection of the prostate
(TURP) is an indication for radical prostatectomy
C It arises initially in the gland’s central portion
D It commonly produces osteoclastic bony metastases
ANSWER:A
704 Which of the following state- ments regarding benign prostatic hyperplasia B.PH) is true?
A The fibrostromal proliferation of BPH occurs mainly in the outer portion of the gland
B
Assuming a voided volume greater than 100 mL, a peak urine flow rate of 30mL/s or less is good
evidence of outflow obstruction
C
Suprapubic prostatectomy for BPH involves enucleation of the entire prostate and eliminates the risk
of future prostate cancer
D
Indications for surgery include acute urinary retention and recur- rent urinary tract infections (UTIs)
ANSWER:D
705
During the course of an oper- ation on an unstable, critically ill patient, the left ureter is lacerated
through 50% of its circumferencE. If the patient’s condition is felt to be too serious to allow time for
defini- tive repair, alternative methods of management include
A Ligation of the injured ureter and ipsilateral nephrostomy
B Ipsilateral nephrectomy
C Placement of a catheter from the distal ureter through an abdominal wall stab wound

D
Placement of a suction drain adjacent to the injury without further manipulation that might convert
the partial laceration into a complete disruption
ANSWER:A
706
A pedestrian is hit by a speeding car. Radiologic studies obtained in the emergency room, including
a retrograde urethro- gram, are consistent with a pelvic fracture with a rupture of the urethra
superior to the urogenital diaphragm. Management should consist of
A Immediate percutaneous nephrostomy
B
Immediate placement of a Foley catheter through the urethra into the bladder to align and stent the
injured portions
C Immediate reconstruction of the ruptured urethra after initial stabilization of the patient
D Immediate placement of a suprapubic cystostomy
ANSWER:D
707 Meniscal tears usually result from which of the following circumstances?
A Hyperextension
B Flexion and rotation
C Simple hyperflexion
D Compression
ANSWER:B
708 Volkmann’s ischemic contracture is associated with
A Intertrochanteric femoral fracture
B Supracondylar fracture of the humerus
C Posterior dislocation of the knee
D Traumatic shoulder separation
ANSWER:B
709
In an uncomplicated disloca- tion of the glenohumeral joint, the humeral head usually dislocates
primarily in which of the following directions?
A Anteriorly
B Superiorly
C Posteriorly
D Laterally
ANSWER:A

710
The most severe epiphyseal growth disturbance is likely to result from which of the following types
of fracture?
A Fracture dislocation of a joint adja- cent to an epiphysis
B Fracture through the articular carti- lage extending into the epiphysis
C Transverse fracture of the bone shaft on the metaphyseal side of the epiphysis
D Crushing injury compressing the growth plate
ANSWER:D
711 Which of the following frac- tures is most commonly seen in healthy bones subjected to violent falls?
A Colles fracture
B Femoral neck fracture
C Intertrochanteric fracture
D Clavicular fracture
ANSWER:D
712
In a failed suicide gesture, a depressed student severs her radial nerve at the wrist. The expected
disability is
A Loss of ability to extend the wrist
B Loss of ability to flex the wrist
C Wasting of the intrinsic muscles of the hand
D Sensory loss over the thenar pad and the thumb web
ANSWER:D
713
Which of the following statements regarding compartment syndromes following orthopedic injuries
is true?
A The first sign is usually loss of pulse in the extremity
B Passive flexion of the extremity proximal to the involved compart- ment will aggravate the pain
C Surgical decompression (fasciectomy) is necessary only as a last resort
D
These syndromes are most commonly associated with supracondy- lar fractures of the humerus and
tibial shaft
ANSWER:D
714 In contrast to closed reduc- tion, open reduction of a fracture
A Produces a shorter healing time
B Decreases trauma to the fracture site

C Produces a higher incidence of nonunion
D Reduces the risk of infection
ANSWER:C
715
Epileptiform convulsion may be a causE. (SELECT 1 INJURY) For description above, select the
type of fracture or dislocation with which it is most likely to be associated.
A Navicular (scaphoiD. fracture
B Monteggia’s deformity
C Greenstick fracture
D Posterior shoulder dislocation
ANSWER:D
716
Avascular necrosis is not uncommon. (SELECT 1 INJURY) For description above, select the type of
fracture or dislocation with which it is most likely to be associateD.
A Navicular (scaphoiD. fracture
B Monteggia’s deformity
C Greenstick fracture
D Spiral fracture
ANSWER:A
717
The radial head is dislocated and the proximal third of the ulna is fractureD. (SELECT 1 INJURY)
For description above, select the type of fracture or dislocation with which it is most likely to be
associated.
A Navicular (scaphoiD. fracture
B Monteggia’s deformity
C Greenstick fracture
D Spiral fracture
ANSWER:B
718
Tenderness in the anatomist’s snuffbox may be observeD. (SELECT 1 INJURY) For description
above, select the type of fracture or dislocation with which it is most likely to be associateD.
A Navicular (scaphoiD. fracture
B Monteggia’s deformity
C Greenstick fracture
D Spiral fracture
ANSWER:A

719
Association with hyperparathyroidism (SELECT 1 DISEASE. For description above, select the type
of bone disease with which it is most likely to be associateD.
A Osteogenesis imperfecta
B Osteopetrosis
C Osteitis fibrosa cystica
D Osteomalacia
ANSWER:C
720
A defect in the mineralization of adult bone secondary to abnormalities in vitamin D metabolism
(SELECT 1 DISEASE. For description above, select the type of bone disease with which it is most
likely to be associateD.
A Osteogenesis imperfecta
B Osteopetrosis
C Osteitis fibrosa cystica
D Osteomalacia
ANSWER:D
721
Genetically determined disorder in the structure or processing of type I collagen (SELECT 1
DISEASE. For description above, select the type of bone disease with which it is most likely to be
associated.
A Osteogenesis imperfecta
B Osteopetrosis
C Osteitis fibrosa cystica
D Osteomalacia
ANSWER:A
722
Synonym for Paget’s disease (SELECT 1 DISEASE. For description above, select the type of bone
disease with which it is most likely to be associateD.
A Osteogenesis imperfecta
B Osteopetrosis
C Osteitis fibrosa cystica
D Osteitis deformans
ANSWER:D
723 Which of the following state- ments regarding glioblastoma mul- tiforme is true?
A It is a neuronal cell tumor

B It arises from the malignant degeneration of an astrocytoma
C With aggressive treatment, most patients can live up to 10 years with this disease
D It is the most common childhood intracranial neoplasm
ANSWER:B
724 Which of the following statements regarding skull fractures is true?
A Depressed fractures are those in which the patient’s level of consciousness is diminished or absent
B Compound fractures are those in which the skull is fractured and the underlying brain is lacerated
C Any bone fragment displaced more than 1 cm inwardly should be elevated surgically
D Drainage of cerebrospinal fluid via the ear or nose requires prompt surgical treatment
ANSWER:C
725
An acute increase in intracra- nial pressure is characterized by which of the following clinical find-
ings?
A Respiratory irregularities
B Decreased blood pressure
C Tachycardia
D Papilledema
ANSWER:A
726 Which of the following state ments about schwannomas is true?
A They represent central nerve tumors
B Treatment is via excision
C They arise most frequently in motor nerves
D They often degenerate to malignancy
ANSWER:B
727 Which of the following statements about craniopharyngiomas is true?
A The tumors are uniformly solid
B The tumors are usually malignant
C Children with these tumors often develop signs and symptoms of acromegaly
D The tumors may cause compression of the optic tracts and visual symptoms
ANSWER:D
728 Which of the following statements regarding cerebral contu- sions is true?
A They occur most frequently in the occipital lobes
B They may occur opposite the point of skull impact

C They are rarely accompanied by parenchymal bleeding
D They may occur spontaneously in patients receiving anticoagulants
ANSWER:B
729 True statements regarding meningiomas include that they
A Are malignant in 50% of cases
B Occur predominantly in men
C Are treated primarily by surgical excision
D Are cured, when properly treated, in nearly 95% of cases
ANSWER:C
730
While watching a golf tournament, a 37-year-old man is struck on the side of the head by a golf
ball. He is conscious and talkative after the injury, but several days later he is noted to be increas-
ingly lethargic, somewhat con- fused, and unable to move his right sidE. (SELECT 1 DIAGNOSIS).
For description above, select the type of vascular event with which it is most likely to be associateD.
A Subdural hematoma
B Epidural hematoma
C Carotid dissection
D Brain contusion
ANSWER:A
731
A 42-year-old woman complains of the sudden onset of a severe headache, stiff neck, and
photophobiA. She loses consciousness. She is later noted to have a dilated pupil. (SELECT 1
DIAGNOSIS). For description above, select the type of vascular event with which it is most likely to
be associateD.
A Subdural hematoma
B Epidural hematoma
C Carotid dissection
D Ruptured intracranial aneurysm
ANSWER:D
732
A25 year-old known substance abuser is brought to the ED with a suspected overdosE. Which of the
following is not considered a universal antidote?
A glucose
B oxygen

C calcium gluconate
D naloxone
ANSWER:C
733 Which of the following is not a classic sign of a basal skull fracture?
A Battle sign
B racoon eyes
C hemotympanum
D Freedman sign
ANSWER:D
734
A19 year-old female with a traumatic head injury is brought to the ED by EMS. She is
hemodynamically stable but requires assisted ventilation through an endotracheal tubE. She does not
open her eyes to painful stimuli and maintains an abnormal extension posturE. Which of the
following is incorrect?
A her GCS is 3
B a GCS of 8 is an indication for intubation
C isolated head injuries can cause shock
D she should be ventilated to a pCO2 of 30-35 mm Hg
ANSWER:C
735 Conductive hearing loss is a symptom of:
A presbycusis
B Meniere disease
C cholesteatoma
D Bell palsy
ANSWER:C
736 The following statements regarding epistaxis are false EXCEPT:
A Epistaxis rarely occurs in children
B It commonly results from rupture of posterior placed nasal vessels
C It may be treated by ligation of the ipsilateral internal carotid artery
D Epistaxis may be treated by cautery of Little’s area with silver nitrate
ANSWER:D
737
A52 year-old dentist comes to your office complaining of severe bilateral buttock cramps and thigh
fatigue during a tennis match and recent onset of impotence The most likely diagnosis is:

A lumbosacral disc problem
B multiple sclerosis
C Leriche syndrome
D metastatic carcinoma of the spine
ANSWER:C
738
Patients with an organic cause for impotence are often characterized by all of the following
EXCEPT:
A diabetes
B older age
C intermittent difficulty
D nocturnal penile tumescence absent
ANSWER:C
739 Which of the following is not a sign or symptom of carpal tunnel syndrome :
A loss of sensation to the proximal palm
B positive Phalen test
C positive Tinel sign
D pain involving the thumb, index, long, and part of the ring digits
ANSWER:A
740 In which of the following are systemic antibiotics not indicated :
A animal bite to hand
B laceration over metacarpal-phalangeal joint sustained after punching someone in a fight
C positive Finkelstein test
D burn wound to 75% of the dorsal aspect of the hand
ANSWER:D
741 Prolonged vomiting is associated with what electrolyte abnormality?
A hypochloremic hypokalemic metabolic acidosis
B hypochloremic hypokalemic metabolic alkalosis
C hyperchloremic metabolic acidosis
D hyperkalemia
ANSWER:B
742
Indications for immediate operative intervention rather than conservative management for arterial
insufficiency include all the following EXCEPT:

A nocturnal limb pain
B ischemic ulceration
C absent pulse
D ischemic neuropathy
ANSWER:C
743 Surgical indications for diverticulitis include all of the following EXCEPT:
A peritonitis
B persisting hemorrhage
C fistula
D palpable abdominal mass in left lower quadrant
ANSWER:D
744
All of the following radiologic abdominal plain film findings are consistent with mechanical bowel
obstruction except:
A a ""step ladder"" pattern
B dilated small bowel loops
C air-fluid levels at uniform height in same bowel loop
D absence of gas in large bowel
ANSWER:D
745
After a weekend of heavy drinking, a 28 year-old male presents with abdominal pain radiating to
the back. What is the investigative and prognostic modality of choice for the suspected diagnosis?
A abdominal ultrasound
B abdominal CT
C abdominal plain film
D endoscopic retrograde cholangiopancreatography
ANSWER:B
746
An 18 year-old motorcyclist presents in the emergency department following an accident. He has a
compound tibia and fibula fracture of the right leg and on examination the right leg has no pulses.
Your immediate treatment should be:
A immediate angiogram
B immediate surgery
C casting and/or splinting
D reduction and splinting

ANSWER:D
747
Which of the following is the most serious complication of a dis placed supracondylar fracture of
the humerus?
A compartment syndrome of the forearm
B failure to heal
C healing in a non-anatomical position
D injury to the median nerve
ANSWER:A
748 Which of the following is not a complication of untreated otitis media?
A cholesteatoma
B meningitis
C tympanic membrane perforation
D trigeminal neuralgia
ANSWER:D
749 With respect to control of micturition, all of the following are true EXCEPT:
A damage to the cerebral cortex results in hyperactivity of the detrusor
B the basal ganglia inhibits bladder overactivity
C the cerebellum coordinates emptying of the bladder
D damage to the cerebellum results in hypotonicity of the detrusor
ANSWER:D
750 Which of the following drugs will not promote urine retention?
A sympathomimetics
B anticholinergics
C cholinergic agonists
D TCAs
ANSWER:C
751
A patient with a subarachnoid hemorrhage (SAH) caused by a right anterior communicating artery
aneurysm undergoes successful surgery 2 days after the hemorrhagE. Three days later, right arm
weakness develops. The most likely diagnosis is:
A hydrocephalus
B meningitis
C repeat hemorrhage

D vasospasm
ANSWER:D
752 Complications of tonsillectomy include all of the following EXCEPT:
A secondary hemorrhage
B severe otalgia
C Quinsy
D nasopharyngeal stenosis
ANSWER:B
753 Features characteristic of acute tonsilitis include all of the following EXCEPT:
A odynophagia
B cough
C referred otalgia
D cervical lymphadenopathy
ANSWER:C
754
A 6 month-old baby has had mild inspiratory stridor for the last 2 months. The most likely cause of
this is:
A laryngomalacia
B acute epiglottitis
C croup
D tonsillar hypertrophy
ANSWER:A
755
A 48 year-old male is brought to the ED with CPR being administereD. The ECG shows electrical
activity present but you cannot palpate a carotid pulsE. Which of the following is not on the
differential for pulseless electrical activity?
A hypotension
B hypokalemia
C cardiac tamponade
D hypothermia
ANSWER:A
756
A 37 year-old male arrives at the Emergency Department unconscious. He is warm and sweaty. His
heart rate is 52 bpm, his BPis 90/60. His pupils are constricted, his eyes are teary, and he is drooling.
You assume he is suffering from a toxidromE. What antidote will you give him?

A Flumazenil
B Naloxone
C Glucagon
D Atropine
ANSWER:D
757
A 83 year-old man has fallen while walking down stairs. He is brought to the emergency department
with a 3-part intertrochanteric hip fracturE. Which of the following procedures would you choose to
perform?
A hemiarthoplasty
B total hip replacement
C multiple pin fixation
D pin and plate
ANSWER:D
758
A 45 year-old man with a history of polycystic kidney disease presents with painless gross
hematuriA. You order:
A no investigation is required since the hematuria is most likely due to the rupture of renal cyst(s)
B no investigation at this timE. Investigate if gross hematuria persists
C U/S
D U/S, urine C&S, cystosco
ANSWER:D
759
A 75 year-old man with a history of nocturia has not micturated for the last 10 hours, and is
complaining of severe lower abdominal pain. The most likely cause is:
A BPH
B prostate cancer
C renal failure
D UTI
ANSWER:A
760
A43 year-old woman is brought to the Emergency Department after being burned in a house firE.
You estimate first degree burns to 20% of her body, second degree burns to 11% of her body, and
third degree burns to 9% of her body. She weighs 60 kg and is 120 cm tall. What IV therapy would
you begin immediately?
A normal saline at 200 cc/hr for 24 hours

B normal saline at 400 cc/hr for 24 hours
C normal saline at 150 cc/hr for 8 hours, then 75 cc/hr for the next 1 hours
D normal saline at 300 cc/hr for 8 hours, then 150 cc/hr for the next 1 hours
ANSWER:D
761 Risk factors for hepatocellular carcinoma include all of the following EXCEPT:
A hepatitis A
B cirrhosis
C exogenous steroid use
D hemochromatosis
ANSWER:A
762 Which of the following is associated with biliary colic?
A epigastric pain
B rebound tenderness
C jaundice
D Murphy sign
ANSWER:A
763
A16 year-old female fell while roller-blading on her outstretched right hanD. At a nearby
emergency department X rays confirmed the diagnosis of a closed Colles fracturE. The proper
reduction technique for this wrist fracture is which of the following:
A slight extension, full pronation, and full ulnar deviation
B slight flexion, full supination, and full radial deviation
C C.slight extension, full supination, and full ulnar deviation
D slight flexion, full pronation, and full ulnar deviation
ANSWER:D
764 Which of the following is least likely to cause avascular necrosis:
A sickle cell disease
B septic arthritis
C steroid use
D constrictive dressings
ANSWER:D
765 Which of the following is true regarding mandibular fractures?
A they are predominantly unilateral

B compound fractures are rarely intraoral
C they may manifest with numbness in the V2 facial nerve distribution
D malocclussion of teeth is a common sign
ANSWER:D
76 With respect to painful scrotal swelling, all of the following are true EXCEPT:
A torsion of testicular appendages will usually subside without surgical intervention
B nausea/vomiting is very common in epididymitis
C ultrasound is helpful in determining the cause of hematocele
D torsion most commonly occurs in young males
ANSWER:B
767 Which is following statement about PSA is true?
A
every man past the age of 50 should have an annual PSA test as a primary screen for prostate cancer
B PSA is best used to follow disease progression or recurrence post-operatively
C PSA is a useless test
D increased complex PSA to total PSA ratio favors BPH over prostate cancer
ANSWER:B
768
A 40 year-old women presents to the ER with fever, no nausea or vomiting, and left flank pain
radiating to the groin. Body CT reveals 8 mm stone in the left proximal ureter. You should:
A send her home because renal stone is benign disease
B
send her home with oral analgesics and ask her to come back if the pain does not resolve in the next
24 hours
C admit, close monitoring, hydration and analgesics
D
admit, hydration, analgesics and IV antibiotics. E.g. Amp and Gent), and consider stenting if the
symptom persists
ANSWER:D
769 Nasopharyngeal carcinoma:
A is most common in people from Cambodia
B presents early with nasal pain
C is treated by wide surgical excision
D is monitored by measurement of Epstein-Barr virus antibodies
ANSWER:D

770
Factors which contribute to the development of squamous cell cancers of the head and neck include
all of the following EXCEPT:
A cigarette smoking
B alcohol ingestion
C aging
D exposure to particular emission from diesel engines
ANSWER:D
771
Which of the following statements about aspiration of a peanut into the tracheobronchial tree is
FALSE?
A the peanut is more likely to be in the left lower lobe bronchus.
B expiratory wheeze is the most likely finding on physical exam
C
the peanut should be removed with the patient under general anesthesia through an open
bronchoscope with forceps designed to grasp peanuts
D a chest x-ray may show atelectasis distal to the blocked bronchus
ANSWER:A
772
A60 year-old man presents to your office with back and leg pain and trouble urinating. On
examination, he has decreased sensation over the buttocks, normal motor power and absent ankle
jerks bilaterally. How should this patient be managed:
A plain films of the lumbar spine
B CT myelogram of the lumbar spine
C bedrest for 2 weeks with early mobilization
D emergency discectomy
ANSWER:B
773
A72 year-old man on physical examination is found to have expressive dysphasia and mild right
arm weakness. The most probable location of his lesion is:
A right parietal lobe
B left frontal lobe
C right frontal lobe
D left parietal lobe
ANSWER:B
774 Initial management of any patient with coma of undetermined cause includes all except:
A clear and secure the airway

B naloxone
C D50W 50 mL IV
D dexamethasone 1 mg IV
ANSWER:D
775
A16 year-old girl is brought to hospital by her frantic parents after a bee sting. Vitals sings are BP
70/40, RR 30 and laboured, HR 140, T 37.5. Which of the following would not be an option in her
management?
A epinephrine
B diphenhydramine
C methylprednisolone
D atropine
ANSWER:D
776
A 24 year-old woman arrives at the Emergency Department unconscious. Her BP is 90/60, her heart
rate is 60 bpm, she is breathing at 8 breaths per minute and her O2 sat is 86%. Her eyes remain
closed even after pain stimulation and the only sounds she makes are incomprehensiblE. Her elbows
and wrists are flexed with her feet extendeD. This patient’s GCS score is:
A 3
B 4
C 5
D 6
ANSWER:D
777 Which physical exam finding below is usually not associated with increasing intracranial pressure?
A deteriorating level of consciousness
B increasing heart rate and increasing blood pressure
C yawning, hiccuping, vomiting
D seizures
ANSWER:B
778 Which of the following is false regarding post-operative wound infections?
A S. aureus is the most common cause
B usually present with fever post of day 3-4
C increased likelihood in diabetics

D mainly treated with antibiotics
ANSWER:D
779
Which of the following signs and symptoms warrant surgical intervention for patients with small
bowel obstruction?
A abdominal tenderness
B air-fluid levels on abdominal x-ray
C worsening abdominal pain
D feculent vomitus
ANSWER:C
780
A59 year-old woman presents to her family physician with a 3 cm palpable, well circumscribed,
non-tender breast mass. She first noticed it several months ago and believes it has increased in size
since then. Her mother and maternal aunt were diagnosed with breast cancer in their early 50’s. She
has no other health complaints. A subsequent mammogram shows no abnormalities. Which of the
following is the next most appropriate step in management?
A prophylactic mastectomy
B repeat mammogram in 6 months
C repeat mammogram ion 1 year
D excisional biopsy
ANSWER:D
781 Which of the following is false regarding pancreatic pseudocysts?
A it is caused by duct leakage
B clinically suspected if persisting pain > 2 weeks following
C diagnosis of acute pancreatitis
D majority are treated surgically
ANSWER:C
782
An overweight, 45 year-old man presents with left lower quadrant tenderness and a one week
history of abdominal pain, loose non-bloody stools and worsening fever. Laboratory investigation
yields leukocytosis with neutrophilia and left shift. What is the diagnostic modality of choice?
A CT scan
B barium enema
C ultrasound

D plain abdominal film
ANSWER:A
783
A19 year-old woman is brought to the emergency room following involvement in a motor vehicle
accident. On examination, she has a GCS of 10 and swelling over the occipital protuberancE. The
most appropriate imaging study is:
A MRI of skull and contents
B skull films
C head CT with contrast
D head CT without contrast
ANSWER:C
784 Which of the following radiographic features is most consistent with osteoarthritis of the knee?
A marginal erosions
B juxta-articular osteopenia D.emineralization)
C loss of articular cartilage with narrowing of the radiologic joint space
D osteonecrosis A.vascular necrosis) of the medial femoral condyle
ANSWER:C
785 Which of the following statements is incorrect with respect to wound healing?
A epithelialization can occur within 24 hours following primary closure of a wound
B maximum wound strength is often achieved after 2 years
C wounds continue to gain strength after collagen synthesis has reached an equilibrium
D the incidence of wound infection increases with healing by secondary intention
ANSWER:D
786 Which of the following is not a cause of sensor-neural hearing loss:
A ossicular discontinuity
B ototoxicity
C Méniére disease
D noise
ANSWER:A
787 Which of the following is not true of Meniere’s disease?
A characterized by quadrad of vertigo, hearing loss, tinnitus and aural fullness
B vertigo burns out with time
C can be treated with diuretics

D the vertigo lasts for seconds
ANSWER:D
788
A neurosurgeon complains of a 3 week history of awakening at night with right-hand discomfort
that resolves after several minutes. On examination, he has mild weakness of thumb abduction and
diminished pain sensibility on the palmar aspect of the thumb and index finger. The most likely
diagnosis is:
A carpal tunnel syndrome
B cervical radiculopathy
C reflex sympathetic dystrophy
D tendonitis
ANSWER:A
789
A73 year-old woman presents with a 6 month history of deteriorating gait and low back discomfort,
exacerbated by walking. Examination is unremarkable except for hypoactive muscle stretch reflexes
in the legs. X-rays of the lumbosacral area shows the expected degenerative changes associated with
a woman of her agE. The most likely diagnosis is:
A acute lumbar disc hernation
B lumbar stenosis
C myopathy
D normal pressure hydrocephalus
ANSWER:B
790 Which of the following findings is inconsistent with cardiac tamponade?
A hypotension
B pulsus paradoxus
C Kussmaul sign
D jugular venous distension
ANSWER:C
791 Tension pneumothorax is best diagnosed with:
A stat CT scan
B chest x-ray
C watch and wait
D clinical exam
ANSWER:D

792 Which of the following is NOT a cause of major lower gastrointestinal hemorrhage?
A diverticulitis
B angiodysplasia
C aortoenteric fistula
D none of the above
ANSWER:A
793 All of the following have been associated with posterior shoulder dislocation EXCEPT:
A ethanol
B electricity
C exercise
D epilepsy
ANSWER:C
794 Management of an open fracture should always include each of the following EXCEPT:
A assessment of neurovascular status
B reduction and fixation of fracture
C irrigation and debridement of wound
D application of topical antibiotic
ANSWER:D
795 What is the most common type of thyroid cancer?
A medullary
B papillary
C follicular
D lymphoma
ANSWER:B
796 Which of the following is an absolute indication for a tonsillectomy?
A airway obstruction
B recurrent (>5) episodes of tonsillitis
C peritonsillar abscess
D tonsillar hypertrophy
ANSWER:A
797 Which of the following statements regarding the Glasgow coma scale is true?
A It serves as a scale to assess the long-term sequelae of head trauma

B A high score correlates with a high mortality
C It includes measurement of intracranial pressure
D It includes measurement of verbal response
ANSWER:D
798
Controlled hyperventilation (induced hypocapniA. is frequently recommended following head
traumA. The therapeutic consequences of this therapy include
A Reduction of endogenous catecholamines
B Reduction of intracellular potassium levels
C Increase in cerebrovascular resistance
D Induction of compensatory metabolic alkalosis
ANSWER:C
799
An 18-year-old man is admitted to the emergency room following a motorcycle accident. He is alert
and fully oriented, but witnesses to the accident report an interval of unresponsiveness following the
injury. Skull films disclose a fracture of the left temporal bonE. Following x-ray, the patient
suddenly loses consciousness and dilation of the left pupil is noteD. This patient should be
considered to have
A Ruptured berry aneurysm
B Acute subdural hematoma
C Epidural hematoma
D Intraabdominal hemorrhage
ANSWER:C
800
An acute increase in intracranial pressure is characterized by which of the following clinical
findings?
A Respiratory irregularities
B Decreased blood pressure
C Tachycardia
D Papilledema
ANSWER:A
801 Which of the following statements regarding cerebral contusions is true?
A They occur most frequently in the occipital lobes
B They may occur opposite the point of skull impact
C They are rarely accompanied by parenchymal bleeding

D They may occur spontaneously in patients receiving anticoagulants
ANSWER:B
802
While watching a golf tournament, a 37-year-old man is struck on the side of the head by a golf
ball. He is conscious and talkative after the injury, but several days later he is noted to be
increasingly lethargic, somewhat con- fused, and unable to move his right sidE. For description
above, select the type of vascular event with which it is most likely to be associateD.
A Subdural hematoma
B Epidural hematoma
C Carotid dissection
D Brain contusion
ANSWER:A
803
Total joint replacement is a frequently performed operation. Which of the following is correct with
regard to a joint replacement?
A It is indicated in cases of septic arthritis
B It loosens quickly if ""held in place"" by methyl methacrylate cement
C It is contraindicated in patients with poor dentition
D It is contraindicated below the age of 60 years
ANSWER:A
804
A 62 year old woman has multiple small bowel fistulae following a complicatedcourse of peritonitis.
What is the most likely abnormality that will be found on her arterial blood gas A.BG) and
biochemistry analysis?
A Respiratory alkalosis
B Metabolic acidosis with increased anion gap
C Respiratory acidosis with bicarbonate retention
D Metabolic acidosis with normal anion gap
ANSWER:B
805
A 19 year old man, presents to the Emergency Department with a supra-condylar fracture of the
distal humerus. Later that evening, after surgical fixation he is noted to have a wrist drop with a
patch of numbness on the dorso-radial aspect of the wrist. What is this most likely to be due to?
A A median nerve injury
B An ulnar nerve injury
C A musculo-cutaneous nerve injury

D A radial nerve injury
ANSWER:D
806
A 72 year old man suddenly develops pain, and weakness of his right shoulder. On examination, he
has trouble initiating abduction at the shoulder but with the arm at 90 degrees abduction strength is
near normal. Rupture of which of the following tendon is most likely to produce this deficit?
A Biceps
B Deltoid
C Supraspinatus
D Infraspinatus
ANSWER:C
807
A 25 year old man who sustains a clean, fairly superficial, 3cm long laceration to his left forearm on
a piece of broken glass. Which one of the following statements about management is true?
A Prophylactic antibiotics are required
B Thorough cleansing with saline is the decontamination method of choice
C Immediate tetanus prophylaxis is required as this is a tetanus-prone wound
D X-ray is required even if the whole length and depth of the wound can be visualised
ANSWER:B
808
A 72 year old retired office worker, has a chest X-ray prior to elective surgery. This shows a solitary
5cm nodulE. Which of the following does NOT suggest a diagnosis of lung cancer as the cause of
pulmonary nodule?
A Smoking history
B Increasing age
C Nodule size >2cm
D Nodule calcification
ANSWER:B
809
A 72 year old man has a fracture of the neck of his left femur after a fall. Six days later after internal
fixation, his left leg and thigh are swollen. His temperature is 37.5ºC and is otherwise asymptomatiC.
What is the next step in management?
A Continue observation
B Arrange a plain X-ray of his left lower limb
C Arrange a venous duplex ultrasound of his left lower limb

D Perform a blood culture and start empirical antibiotics
ANSWER:A
810
A 17 year old student who fell off his skateboard 2 days ago. He presents to the Emergency
Department with pain over the base of his thumb and tenderness over the region of the anatomical
snuffbox. There is also a small amount of swelling over the same region with no other deformity.
What is the most likely diagnosis?
A Tenosynovitis
B Scaphoid fracture
C Base of thumb arthritis
D Dislocation of the radiocarpal joint
ANSWER:B
811
A 28 year old man is involved in a head-on vehicle collision. He had allegedly been drinking at a
pub with friends and left after a disputE. He was initially trapped in the car with the major impact on
his right thigh. He is brought into the Emergency Department conscious, but pale with a pulse of
135bpm and blood pressure of 90/60mmHg. He has deformity of his mid thigh with visible bone
protruding from a wounD. Which of the following is the correct terminology to describe his
fracture?
A Greenstick fracture
B Pathological fracture
C Stress fracture
D Compound fracture
ANSWER:D
812
A 28 year old man, is involved in a head-on vehicle collision. He had allegedly been drinking at a
pub with friends and left after a disputE. He was initially trapped in the car with the major impact on
his right thigh. He is brought into the Emergency Department conscious, but pale with a pulse of
135bpm and blood pressure of 90/60mmHg. He has deformity of his mid thigh with visible bone
protruding from ananterior wounD. Which of the following is the most likely early complication?
A Femoral artery injury
B S1 nerve root injury
C Ruptured bladder
D Pulmonary embolus

ANSWER:A
813
Soft tissue healing is an ordered process. Which of the following is NOT a stage in this type of
wound healing?
A Acute inflammatory reaction
B Formation of fibrous tissue
C Granuloma formation
D Formation of the coagulum
ANSWER:C
814 Which of the following is true regarding surgical therapy for colorectal cancer?
A Adjuvant therapy is indicated for all patients with colorectal cancer
B The goal of adjuvant therapy is to improve the chances of cure
C Adjuvant radiotherapy is indicated for patients with colon cancer
D Adjuvant chemotherapy for colon cancer should continue for at least 12 months
ANSWER:B
815
A 7 year old child who weighs 25 kilograms is fasting for a series of investigations and a possible
operative procedurE. Assuming normal hydration initially, what would be an appropriate intra-
venous maintenance fluid volume?
A 110 ml/hr
B 1.5 l/day
C 25 ml/hr
D 2 ml/kg/hr
ANSWER:B
816
What is the most useful guide to the likely post-operative opioid requirements of a patient
undergoing a laparotomy?
A The weight of the patient
B The estimated lean body mass of the patient
C The gender of the patient
D The magnitude of the surgery
ANSWER:D

817
A 55 year old woman, presents to the Emergency Department with a 24 hour history of right upper
quadrant pain, passing dark coloured urine and a fever of 38.7ºC. Abdominal ultrasound shows
multiple gallstones in the gallbladder and a dilated common bile duct of 1.5 cm. What is the most
likely diagnosis?
A Acute appendicitis
B Acute pancreatitis
C Acute cholecystitis
D Acute cholangitis
ANSWER:D
818 Which of the following is true regarding the management of burns?
A All patients require prophylactic antibiotics
B Superficial (or first degreE. burns have minor blisters only and should not require pain relief
C Full thickness (or third degreE. burns are very painful and require pain relief
D
Adults should receive 3 litres of normal saline within the first 12 hours to replace fluid lost via burnt
skin
ANSWER:D
819
You are called to see a 65 year old man on the surgical warD. He underwent a hemicolectomy for
carcinoma of the colon 5 days ago, and has had increasing abdominal pain over the past 24 hours.
He is able to talk to you but looks unwell. His heart rate is 130/minute, his blood pressure is 90/60
mmHg, his temperature is 38.5ºC, and his respiratory rate is 32/minutE. His peripheral oxygen
saturation is 96 percent while receiving oxygen via a simple face mask. What should your initial
response be?
A Change the mask to a reservoir mask (non-rebreather) to increase the oxygen supply
B Request a chest X-ray
C Give one litre of normal saline as quickly as possible intravenously
D Take some blood for a cross-match
ANSWER:C
820
Which of the following is the organism most likely to cause endocarditis early after prosthetic valve
surgery?
A Fungus
B Streptococcus
C Enterococcus

D Staphylococcus
ANSWER:B
821
A 71 year old former gardener, presents with acute back pain which has steadily worsened over the
past weeks. On examination, you note thoracic kyphosis with no scoliosis. His mid-thoracic spine
is tender to palpation and his spinal mobility is limiteD. You suspect an osteoporotic fracturE. Which
of the following is the next most appropriate diagnostic test to confirm your suspicion?
A Thoracolumbar X-ray
B Whole body Tc MDP nuclear medicine bone scan
C MRI thorax
D PET scan
ANSWER:A
822
A 79 year old man with chronic atrial fibrillation who presents complaining of severe persistent
periumbilical pain over the last 12 hours. On examination he looks unwell but is afebrile and has a
pulse rate of 96. On abdominal examination he is diffusely tender but has no rebound, no localising
signs and bowel sounds are occasionally hearD.Which of the following is the most likely diagnosis?
A Perforated peptic ulcer
B Acute cholecystitis
C Acute pancreatitis
D Acute diverticulitis
ANSWER:A
823
A 64 year old female presents with a two-year history of perianal itching and irritation more
noticeable in the past month. Inspection of the perineum shows an area of hyperkeratosis,
leucoplakia and an associated large irregular ulcer on the anal verge in the mid-line posteriorly.
What should be the next step in her management?
A Perform a wide local excision
B Prescribe a steroid cream
C Patient to apply rectogesic ointment 5 percent (nitroglycerin-basE.
D Perform an incisional biopsy
ANSWER:D

824
A 45 year old man is involved in a motor vehicle accident. He presents 2 weeks later with pain
involving the right shoulder, scapula and upper limB. What sensory findings would be suggestive of
a C6 radiculopathy?
A Numbness in a patch over the deltoid
B Parasthesia over the upper trapezius
C Numbness in the axilla
D Paraesthesia of the index finger and thumb
ANSWER:D
825 Which of the following is NOT an indication for tracheotomy?
A Acute airway obstruction
B Prolonged ventilation
C Recurrent bronchial toilet
D Moderate sleep apnoea
ANSWER:D
826
A 25 year old male falls onto his outstretched hanD. Upon examination, he is tender in the
anatomical snuffbox. Which carpal bone is he most likely to have fractured?
A Hamate
B Scaphoid
C Lunate
D Capitate
ANSWER:B
827
George Ramonez, a 55 year old man, presents to the Emergency Department with haematemesis. He
looks unwell and palE. He has a pulse rate of 130/min and blood pressure of 80/50 mmHg. His
Glasgow Coma Score is 9. Physical examination shows a jaundiced patient with multiple spider
naevi on his chest, gynaecomastia and an enlarged liver. What is the most appropriate initial step in
managing this patient?
A Protect patient’s airway
B Intravenous fluid resuscitation
C Red blood cell transfusion
D Urgent gastroscopy
ANSWER:C

828
A 25 year old motorcyclist, presents with severe central chest pain to the Emergency Department
following a motor vehicle accident. His pulse rate is 130/min and blood pressure is 90/60 mmHg. A
chest X-ray shows multiple fractured ribs on the right and a widened superior mediastinum. His
abdomen is soft and non-tender with no sign of external injuries. Abdominal Focused Assessment
with Sonography for Trauma (FAST) was negativE. Which of the following is the most appropriate
management of this patient?
A MRI of the chest and abdomen
B Diagnostic peritoneal lavage
C Arch aortogram
D Exploratory thoracotomy and laparotomy
ANSWER:C
829
A 20 year old bystander outside a pub, presents to the Emergency Department with hypotension and
stab wounds to the abdomen. Her pulse rate is 140/min and blood pressure is 80/50 mmHg on
arrival. She has already had 3L of normal saline intravenous fluid resuscitation. Physical
examination shows two 2cm long stab wounds in the epigastric region of the abdomen. Following
resuscitation, which of the following is the most appropriate management of this patient?
A Trauma laparotomy
B Diagnostic laparoscopy ± laparotomy
C Diagnostic peritoneal lavage
D Repeated Focused Assessment with Sonography for Trauma (FAST)
ANSWER:A
830
A 12 year old boy, sustains a penetrating wound of the thigh while trail bike riding. He is taken to
the Emergency Department where the wound is debrideD. When questioned, his mother is certain
that he received the usual childhood immunisations during the first year of life and again before
attending school at the age of 5 years.Which of the following is the most appropriate management?
A Administer one dose of tetanus immune globulin
B Administer one dose of tetanus toxoid
C Administer a course of three tetanus toxoid injections
D Administer topical antibiotics
ANSWER:D

831
A 56 year old man who asks you, his GP, whether he should undergo prostate cancer testing. Which
of the following statements regarding prostate cancer screening is true?
A Prostate cancer screening in this man is not recommended because he has no family history
B Surgical treatment of early prostate cancer results in a reduction in mortality
C The PSA test does not detect the disease in an earlier stage
D Screening is not recommended because it is much more expensive than breast cancer screening
ANSWER:B
832 How is an "open" fracture defined?
A There is a full thickness break of the bone
B Requires surgical (open) reduction
C Requires antibiotic therapy
D Associated with a breach in the overlying skin
ANSWER:D
833 Which of the following accelerates the natural process of fracture healing?
A Multivitamin tablets
B Drinking milk
C Application of a plaster of Paris cast
D Restoring anatomical continuity of the bones
ANSWER:C
834 Which of the following is NOT a stage in normal soft tissue wound healing?
A Acute inflammatory reaction
B Formation of fibrous tissue
C Granuloma formation
D Formation of the coagulum
ANSWER:C
835
An 18 year old motorcyclist who is brought to hospital by ambulance following a single-vehicle
crash. He has a compound fracture of the right femur and is agitated and disorientateD. Which of
the following is NOT an immediate priority?
A Assess air-entry and position of the trachea
B Check upper airways to ensure patency
C Urgent full blood count
D Urgent blood sugar level

ANSWER:D
836
A 57 year old man who attends your surgery complaining that he has noticed bright blood on the
toilet paper several times over the last two months. He has a past history of haemorrhoids, diagnosed
in his 30s. On examination his perianal area looks normal and he has a normal rectal examination.
Which of the following is the most appropriate next action?
A Reassure
B Refer for colonoscopy
C Review in 3 months
D Refer for a faecal occult blood test
ANSWER:A
837
A patient is having an epidural inserted for postoperative analgesiA. He is having a laparotomy, with
a midline incision from xiphisternum to pubic symphysis. Where would be a suitable vertebral
interspace to insert the epidural catheter?
A C7/8
B T4/5
C T9/10
D L3/4
ANSWER:D
838
What is the most likely cause of hypotension in a patient with an epidural infusion of local
anaesthetic?
A Local anaesthetic toxicity
B Sympathetic blockade
C Allergy causing anaphylaxis
D Epidural abscess
ANSWER:B
839 What is the most likely cause of upper airway obstruction in an unconscious patient?
A A foreign body
B Vomitus
C The tongue
D Airway oedema
ANSWER:C

840
Which of the following compounds can be used as a part of a bowel preparation regimen for a
patient prior to large intestinal surgery?
A Polyethelene glycol
B Sodium sulphate
C Aluminium hydroxide
D Sodium thiopentone
ANSWER:A
841
A 25 year old male patient is recovering from a laparotomy he underwent earlier in the day. His
vital signs show a blood pressure of 120/80 mmHg and a pulse rate of 120 per min. His in-dwelling
catheter has drained 100 mls since coming back from the operating theatre ten hours ago. He has a
wound drain which has drained 450 mls of blood looking fluiD. Which of the following would be
the most appropriate course of action?
A
Administer a unit of colloid and send off an urgent cross match and full blood count. Reassess the
pulse rate and urine output after the fluid bolus
B Monitor closely and if the blood pressure drops then give the patient more fluids
C
Administer a unit of O negative blood and send off an urgent cross match and full blood count.
Reassess the pulse rate and urine output after the fluid bolus D. Give the patient 20 mg of lasix
D Monitor the patient closely. If the urine output increases then do nothing
ANSWER:B
842
An obese 65 year old female with renal failure has had a laparotomy. On day five post operative she
develops copious serous ooze from her midline abdominal wounD. She is afebrilE. This is likely to
be due to which of the following?
A Wound abscess
B Wound dehiscence
C Normal wound healing
D Allergic reaction to the sutures
ANSWER:B
843
Which of the following best describes a compound that can be applied topically to human tissue to
inhibit or kill micro-organisms with the aim of reducing the risk of sepsis?
A A disinfectant
B A sterilising agent
C An antiseptic

D An antibacterial
ANSWER:A
844
A 78 year old female presents febrile, tachycardic and with abdominal pain. On examination, you
note a tender mass in the left iliac fossA. After initial resuscitation and routine blood tests, what
would be the most appropriate investigation?
A Abdominal and pelvic CT scan
B Faecal occult blood
C Barium enema
D Ultrasound of the abdomen
ANSWER:A
845
A trauma patient with a suspected cervical spine injury requires intubation. Which of the following
statements is true regarding the intubation?
A Manual in-line stabilisation (MILS) makes the laryngoscopy and intubation easier
B The hard collar should be left in place
C Traction should be applied as the collar is removed
D MILS should prevent any movement of the patient's head
ANSWER:A
846
A 15 year old male presents with central periumbilical pain, a low grade temperature and nauseA.
Over the next couple of hours, the pain shifts to the right iliac fossA. Which of the following is the
most likely diagnosis?
A Mesenteric lymphadenitis
B Acute appendicitis
C Crohn's disease
D Pelvic inflammatory disease
ANSWER:B
847 What is an important difference between sedation and general anaesthesia?
A Amnesia
B Lack of movement in response to painful stimulus
C Maintenance of spontaneous respiration
D Analgesia
ANSWER:D

848
A 58 year old male presents with severe pain and bright red bleeding on defaecation. On
examination, there is a tear associated with a small inflammatory skin tag in the posterior midline of
the anus. This lesion is typical of which of the following?
A Peri-anal abscess
B Haemorrhoids
C Anal fistula
D Pilonidal sinus
ANSWER:B
849
A hirsute 20 year old male presents with severe pain in the internatal cleft. He has a low grade
temperature and on examination there is a red tender lump in that region. What is the most likely
diagnosis?
A Perianal abscess
B Pilonidal abscess
C Anal fistula
D Spina bifida
ANSWER:A
850
Graduated compressive stockings are applied to patients recovering from surgery to prevent which
of the following?
A Peripheral oedema
B Pressure ulcers
C Cellulitis
D Superficial thrombo phlebitis
ANSWER:D
851
A patient with a suspected intra-abdominal abscess would have the following pattern to their
temperature chart?
A A constant temperature of about 38°C
B A constant temperature below 36°C
C Night fevers rising to 40°C
D Temperature that varies between normal and 40°C
ANSWER:A

852
A healthy 36 year old patient is having an elective operation under generalanaesthesiA. Eight
minutes after endotracheal intubation, the pulse oximeter saturation drops from 98 percent to 88
percent. What is the most likely cause of this?
A Oesophageal intubation
B Endobronchial intubation
C Basal atelectasis
D Pulmonary embolism
ANSWER:A
853 What is the mechanism of action of local anaesthetics on nerve cells?
A Blockade of nicotinic acetylcholine receptors
B Enhancement of Gamma Aminon Butyric Acid-B (GABA-receptor activation
C Blockade of the membrane Na-K-ATPase
D Blockade of voltage-gated sodium channels
ANSWER:D
854 What is an advantage of tramadol over morphine?
A Fewer interactions with antidepressants compared with morphine
B Less respiratory depression
C Less likely to cause seizures
D Less diarrhoea
ANSWER:B
855
A 75 year old woman is brought into the Emergency Department by ambulance after a fall at homE.
She is unable to walk and complains of severe groin pain when she moves her left leg. On
examination she is lying in bed with her left leg externally rotateD. The left leg is 3 cm shorter than
the right leg. She is usually well and takes no medications. What is the most appropriate
investigation?
A Rectal and abdominal examination
B Bone scan
C Plain X-ray of the pelvis
D Plain X-ray of the hip
ANSWER:D

856
A four year old boy is brought to the Emergency Department. His mother states that he has fallen
from the backyard trampolinE. He has a swollen and obviously deformed right elbow. The radial
pulse is absent and the hand is whitE. The child says that his hand "feels funny". What is the most
likely diagnosis?
A Dislocated radial head with an ulnar fracture
B Subluxed radial head
C Proximal ulnar fracture
D Supracondylar humeral fracture
ANSWER:D
857
A four year old boy is brought to the Emergency Department. His mother states that he has fallen
from the backyard trampolinE. He has a swollen and obviously deformed right elbow. The radial
pulse is absent and the hand is whitE. The child says that his hand ""feels funny"". What is the
appropriate initial management?
A Urgent nerve conduction studies
B Straighten the arm as far as possible and immobilise in a back slab
C Place the arm in a sling and arrange orthopaedic review within 24 hours
D Infiltrate local anaesthetic around the elbow for immediate analgesia
ANSWER:C
858
A 21 year old man is brought to the Emergency Department. He has been drinking heavily and fell
while attempting to avoid arrest. His ankle is very swollen and obviously deformeD. Foot pulses are
present and the skin is unbroken but tightly stretched over the medial side of the anklE. What is the
most likely diagnosis?
A Partial tear of the medial D.eltoiD. ligament of the ankle
B Fracture dislocation of the ankle
C Fractured neck of talus
D Navicular fracture
ANSWER:B
859
A 35 year old woman is brought into the Emergency Department by ambulance after being found
lying in the gutter. She is able to speak but is unable to give coherent answers to questions. Her
initial vital signs are as follows: heart rate 120 bpm, blood pressure 80/40 mmHg, temperature 38ºC
and respiratory rate 28 bpm. What would be your first step on reaching her bedside?

A Perform a neurological examination
B Remove her clothing, looking for signs of injury
C Ask for an urgent 12 lead electrocardiogram E.CG)
D Insert an intravenous cannula
ANSWER:D
860
A 22 year old man who was out drinking last night comes into the Emergency Department
complaining of a sore right hanD. When asked how this might have happened he is evasivE. On
examination, he is tender over the ulnar aspect of the hand and wrist and has a swollen lip and a
bruise around his right eyE. What is your provisional diagnosis of his hand injury?
A A sprained wrist
B A Bennett's fracture
C A displaced Colles' fracture
D A fracture of the medial carpal bones
ANSWER:A
861
A 78 year old woman presents with diffuse abdominal pain, worsening over the past 2 days. She is
nauseated and anorexiC. She has not defecated in two days. Herbackground is notable for chronic
atrial fibrillation and hypertension. On examination, she looks very unwell and has a generally
tender abdomen on palpation. What diagnosis do you need to rule out?
A Mesenteric ischaemia
B Diverticular abscess
C Appendicitis
D Pancreatitis
ANSWER:D
862
A previously well 65 year old woman is brought in by ambulance after a collapse at homE. She is
unconscious but breathing spontaneously. She does not open her eyes to a painful stimulus but has
an abnormal flexion response in her limbs. Her only vocal response is intermittent groans. What is
her Glasgow Coma Score?
A 0
B 3
C 6
D 9
ANSWER:D

863
A 45 year old man underwent open right inguinal hernia repair with mesh for a large right
inguinoscrotal hernia 3 days ago. He presents to the Emergency Department with worsening right
testicular pain since the surgery. On physical examination,there is no groin swelling seen but marked
swelling and tenderness in his right scrotum. What is the most likely diagnosis?
A Epidiyimo-orchitis
B Torsion of testis
C Ischaemic orchitis
D Renal colic
ANSWER:A
864
A 65 year old man, presents with a 2 day history of colicky abdominal pain and vomiting. He has a
background history of open anterior resection for rectal cancer 3 years ago. On physical
examination, his abdomen is firm and distendeD. There is a midline laparotomy scar and a 6cm
lump in the middle of the scar, which is tender to palpate and not reduciblE. X-ray of the abdomen
shows multiple central bowel loops with air-fluid levels. What is the most appropriate treatment plan?
A Non-operative management
B Emergency diagnostic laparoscopy
C Gastrograffin small bowel series
D Emergency exploratory laparotomy
ANSWER:D
865
A thin, 85 year old woman who presents with a left groin lump. She complains of nausea and
vomiting and has a background history of hypertension and varicose veins. Physical examination
reveals a 1.5cm firm lump below and lateral to the left pubic tuberclE. The lump is not reducible
and she does not have a cough impulsE.What is the most likely diagnosis?
A Femoral hernia
B Inguinal hernia
C Saphena varix
D Lipoma
ANSWER:C

86
A 40 year old woman, presents with right upper quadrant pain having been the front seat passenger
in a high speed head-on motor vehicle collision. On arrival at the Emergency Department, her heart
rate is 100 beats per minute and blood pressure is 110/60 mmHg. She is alert and orientateD. Her
abdomen is mildly distended and there is mild right upper quadrant tenderness. CT scan of the
abdomen and pelvis show moderate size liver laceration with surrounding fluiD. What is the most
appropriate treatment plan?
A Admit for close and frequent observation
B Diagnostic laparoscopy
C Exploratory laparotomy
D Diagnostic peritoneal lavage
ANSWER:A
867
You are called to see a confused patient on the surgical wards, who is currently receiving oxygen
via facemask at 4L/min. Which of the following observations would suggest that the patient is
critically ill?
A Body temperature of 39.6ºC
B Heart rate of 92 bpm
C Respiratory rate 38/minute
D Blood pressure 110/50 mmHg
ANSWER:A
868
You are asked to review a 73 year old man who had a partial gastrectomy 4 days ago. He has a
history of ischaemic heart disease and chronic obstructive airway diseasE. On examination he has a
temperature of 38.3°C, a heart rate of 124/min, blood pressure of 80/40 mmHg, warm peripheries
and the jugular venous wave-form is not visiblE. What is the most likely cause for this scenario?
A Wound infection
B Pulmonary embolus
C Myocardial infarction
D Delayed bleeding from an anastomosis
ANSWER:D
869
A 45 year old morbidly obese male presents with a two day history of pain in the left calf and
swelling of the anklE. Which of the following is the correct management?
A Elastic stockings and aspirin with a review in 5 days

B Bed rest with elevation of the left leg
C Venous duplex ultrasound scan of the left leg
D Subcutaneous heparin for 6 days
ANSWER:C
870 Which of the following is NOT a symptom or a sign of acute lower limb ischaemia?
A Pain in the calf at rest
B Absent ankle pulses
C Pallor
D Paralysis of ankle/toe movement
ANSWER:D
871
The highest risk for post-operative deep vein thrombosis D.VT) is associated with which of the
following types of surgery?
A Total Hip Arthroplasty
B Gastrectomy for carcinoma
C Radical prostatectomy
D Open aortic aneurysm repair
ANSWER:D
872
What is the most effective modality for prevention of post-operative deep vein thrombosis D.VT)
following laparoscopic anterior resection for sigmoid colon cancer?
A Venous foot pump (VFP)
B Intermittent pneumatic compression (IPC.
C Low molecular weight heparin (LMWH)
D Thigh length graduated compression stocking (GCS)
ANSWER:D
873
On discharge from hospital following Total Hip Arthroplasty (THA., patients should receive which
of the following for 4 weeks for venous thromboembolism (VTE. prophylaxis?
A Graduated compression stocking (GCS)
B Aspirin
C Warfarin
D Low molecular weight heparin (LMWH)
ANSWER:B

874
A 24 year old man admitted to the Emergency Department following a motor vehicle accident. He
has a contused chest wall and is in respiratory distress. His left chest is resonant on percussion and no
breath sounds can be heard on auscultation. His trachea is deviated to the right. What is the most
likely diagnosis?
A Acute Respiratory Distress Syndrome A.RDS)
B Pulmonary embolism
C Spontaneous pneumothorax
D Tension pneumothorax
ANSWER:D
875
A 25 year old man who has had a motor bike accident. On arrival at the Emergency Department, his
vital signs are heart rate 130/minute, blood pressure 90/60 mmHg, respiratory rate 32/minute,
temperature 36ºC and his peripheral oxygen saturation is 97 percent. What should be your initial
response?
A Assess his airway and apply an oxygen mask
B Insert an IV cannula and take blood for a cross-match
C Assess his cervical spine and then apply a hard collar
D Assess his Glasgow Coma Score
ANSWER:D
876
A 25 year old man who has had a motor bike accident. On arrival at the Emergency Department, his
vital signs are heart rate is 130/minute, blood pressure of 90/60 mmHg, respiratory rate of
32/minute, temperature is 36°C. His peripheral oxygen saturation is 97 percent on room air. What
would be your initial order for IV fluids?
A Two packs of O negative blood over half an hour
B A litre of normal saline over 2 hours
C A litre of 4 percent dextrose and 0.18 percent saline over an hour
D A litre of normal saline over half an hour
ANSWER:D

877
A 24 year old woman presents with a 3 day history of lower abdominal pain and swinging fever to
39°C. She underwent laparoscopic appendicectomy for perforated appendicitis 5 days ago. The
procedure was uncomplicated and she was discharged following 2 days of hospital admission. On
physical examination, her temperature is 38.9°C, pulse rate is 120 beats/minute and blood pressure is
140/70 mmHg. Her abdomen is mildly distended and there is mild right iliac fossa tenderness. The
laparoscopic wounds are clean. What is the most likely diagnosis?
A Intraabdominal abscess
B Appendiceal stump leak
C Appendiceal stump appendicitis
D Urinary tract infection
ANSWER:A
878
A man is brought into the Emergency Department after being assaulteD. His Glasgow Coma Score
(GCS) is 8 and he has a markedly dilated pupil on the right. Which of the following is the most
important priority?
A Arrange an urgent computed tomography C.T) brain
B Give mannitol
C Give phenytoin
D Establish an airway
ANSWER:D
879 A skin flap refers to soft tissue that is characterised by which of the following?
A Can be designed in any shape, form and size
B Incorporates its own vascular supply
C Cannot be designed in all parts of the body
D Contains all layers of the skin and muscle
ANSWER:D
880 An acute attack of abdominal colic means that the pain:
A Has been present for at least 12 hours
B Is so severe that the patient cannot clearly describe it
C Is sharp and worse on inspiration
D Is constant and severe
ANSWER:C
881 Regarding analgesic doses of ketamine, which of the following statements is correct?

A It should be given carefully to asthmatics as it may worsen bronchospasm
B It is likely to cause nausea and vomiting
C It antagonises the effects of opiates
D It is unlikely to cause respiratory depression
ANSWER:B
882
A 56 year old man is brought into the Emergency Department following a car accident. When you
assess his respiratory system you find that he has reduced chest expansion on the right with a
midline tracheA. The percussion note is dull on the right posteriorly and laterally but resonant on
the left. He is very tender in the right axillA. Breath sounds are absent at the right base and generally
vesicular with occasional crackles elsewherE. What is the most likely diagnosis based on these
findings?
A Haemothorax on the right
B Simple pneumothorax on the right
C Right lower lobe aspiration pneumonitis
D Acute severe asthma
ANSWER:A
883
A 45 year old man who presents to the Emergency Department with pain in the lateral aspect of his
wrist after a fall on an outstretched hanD. On examination he is tender in his anatomical snuffbox.
On X-ray he does not have an obvious fracturE. Which fracture are you most concerned about?
A Fracture of the proximal thumb
B Fracture of the scaphoid
C Fracture of the radial styloid
D Fracture of the base of the second metacarpal bone
ANSWER:B
884
A 45 year old man, presents to Emergency Department after being knocked off his bicycle by a car.
He is talking and has no recollection of the event. Physical examination is unremarkable except for
bilateral periorbital brusing. CT scan of the head shows a small amount of air in the cranial cavity.
What is the most likely diagnosis?
A Depressed skull fracture
B Closed head injury
C Subdural haematoma

D Base of skull fracture
ANSWER:D
885
Mrs Chubb, a 60 year old woman, presents to the Emergency Department following a motor vehicle
accident. On primary survey: her airway is intact; she is breathing spontaneously with a respiratory
rate of 20 per minute, pulse rate of 90 beats per minute and blood pressure of 140/90 mmHg;
Glasgow Coma Score of 15. The secondary survey is unremarkable except for marked tenderness
over the mid sternum on palpation. No haemothorax, pneumothorax or fractures are seen on a chest
X-ray with a sternal view. What is the most likely diagnosis?
A Fractured sternum
B Soft tissue injury
C Fractured ribs
D Cardiac contusion
ANSWER:A
886
A 65 year old woman, presents to the Emergency Department following a motor vehicle accident.
Following initial assessment, she is found to have a minimally displaced fractured left claviclE. What
is the most appropriate management following 5 milligrams of intramuscular morphine injection?
A Thoracic epidural analgesia with bupivacaine
B Spinal analgesia with bupivacaine
C Open interal fixation of clavicle under general anesthesia
D Oral analgesia and rest the arm in a sling
ANSWER:D
887
A 22 year old man, presents to the Emergency Department after being tackled in rugby. He
complains of left arm pain and is unable to move his left shoulder because of pain. X ray shows a
minimally displaced fracture of the mid shaft of the humerus. His left arm is warm with good distal
pulses. Sensation in the left upper arm is also normal. What is the most appropriate management of
this humeral fracture?
A Oral analgesia and support the arm in a sling
B Open reduction and internal fixation under general anaesthesia
C External fixation under local anaesthesia
D Upper limb angiogram to exclude dissection of left brachial artery
ANSWER:A

888
A 15 year old girl presents with a 12 hour history of right iiac fossa pain. Which of the following
symptoms or signs is usually not associated with a diagnosis of acute appendicitis?
A Pain starting in the peri-umbilical region and moving to the right iliac fossA.
B Temperature of 39.5 degrees
C Localised tenderness in the right iliac fossa
D Coated tongue with bad breath
ANSWER:D
889
A 35 year old woman presents with a 24 hour history of right iliac fossa pain. She is tender with
moderate rebound tenderness in the right iliac fossA. Which of the following is the LEAST likely
differential diagnosis?
A Acute appendicitis
B Right tubo-ovarian abscess
C Mesenteric adenitis
D Ruptured ovarian follicle (Mittelschmertz syndromE.
ANSWER:B
890
A 72 year old woman presents with severe abdominal pain. She has been on the waiting list for a
laparoscopic cholecystectomy for gallstone diseasE. She is afebrile,with a normal pulse and blood
pressure and is tender in the epigastrium. Aprovisional diagnosis of acute pancreatitis is made and
confirmed with an elevated lipase and amylase level. Which of the following is NOT useful in
predicting the severity of acute pancreatitis?
A An elevated white cell count > 16,000
B Serum amylase level >1,000 I/U
C Age greater than 55
D Serum albumin < 32 g/l
ANSWER:A
891 In which of the following is topical anaesthesia with lignocaine NOT effective?
A Cornea
B Posterior pharynx
C Auditory canal
D Urethra
ANSWER:C

892
A 23 year old man presents with abdominal pain after a single low velocity gunshot wound to the
left chest. The entrance wound is 3 cm inferior and lateral to the left nipple and the exit wound 6 cm
below the left scapulA. A chest tube is inserted and drains 400 ml of blood initially and 50 ml over
the next hour. A chest X-ray reveals an expanded lung and no pleural fluiD. His initial blood
pressure was 70/20 mmHg with a pulse of 120/min. After 2 litres of saline his blood pressure is
85/55 mmHg and pulse is 110/min. An initial FAST (focused abdominal scan in traumA. scan is
negativE. Which of the following should be the next step in his management?
A A computed tomography C.T) scan of chest and abdomen
B Repeat FAST
C Continued observation
D Emergency laparotomy
ANSWER:C
893
A 48 year old woman develops sudden onset of right sided abdominal pain while playing tennis.
She presents to you with persistent sharp pain in the right lower abdomen with associated nauseA.
Examination reveals a low grade fever (37.2ºC. and tenderness to the right of the midline below the
umbilicus with an impression of a mass in this region. Which of the following is the most likely
diagnosis?
A Acute appendicitis
B Caecal carcinoma
C Haematoma of the rectus sheath
D Acute cholecystitis
ANSWER:D
894
A 72 year old man undergoes a right total knee replacement and on the 2nd post operative day is
found to have a swollen leg with pain in the calf on movement. Which of the following would be
the most likely diagnosis?
A Normal post operative swelling
B Acute joint infection
C Popliteal artery thrombosis
D Post operative bleed into the joint space
ANSWER:A

895
A 24 year old man has fallen from his motorbikE. He was paraplegic at the scene with a sensory
level at T10. You review him in the Emergency Department. His legs appear warm and well
perfused but he is tachycardic (105 bpm) and hypotensive (85/40 mmHg). His haemoglobin level is
normal (120 g/L). What is the most correct statement?
A He has undiagnosed internal bleeding and requires further imaging
B He has undiagnosed bleeding and should be given whole blood
C He has spinal shock and needs resuscitation with crystalloid
D He is a young athlete and this is normal for age
ANSWER:A
896
A 50 year old man was moving house when he felt a twinge in his back that was followed by the
sudden onset of severe pain in his left leg. It travelled from the buttock into the posterior thigh and
calf. The pain has been present for 2 weeks. He has now noticed that his calf muscles are weak on
the left sidE. He is unable to stand on his toes on the left sidE. A CT scan of the lumbar spine shows
a left lateral disc protrusion compromising the S1 nerve root. Which is the most correct statement?
A Surgical treatment should be offered
B Intensive physiotherapy for at least 2 months should be offered
C Epidural steroid injection should be offered
D An operative decision will depend on the size of the disc on MRI scanning
ANSWER:D
897
A 65 year old woman collapsed while standing at the watercooler at work. She was noted to clutch
at her head as she fell. She is brought to the Emergency Department and a computed tomography
C.T) scan is reported as showing blood in the basal cisterns. What is the most likely diagnosis?
A Subarachnoid haemorrhage
B Intracerebral haemorrhage
C Epidural haemorrhage
D Extradural haemorrhage
ANSWER:A
898
A 64-year-old man presents with headache and left-sided upper extremity weakness. The MRI
findings suggest that this is a glioblastomamultiformE. This is because the tumor exhibits which of
the following?

A It is regular in shapE.
B It is well demarcated from surrounding brain tissuE.
C
It shows a ring pattern of enhancement with intravenous contrast and has a nonenhancing necrotic
center.
D It shows an absence of surrounding white-matter edemA.
ANSWER:C
899
A 63-year-old woman presents for workup to determine the reason for a gradual hearing loss over
approximately 5 years and intermittent tinnitus over the last several months. Findings on physical
and neurologic examination are entirely within normal limits, except for the presence of
sensorineural hearing loss in the left ear. She has no cranial nerve deficits. An MRI of the brain with
gadolinium reveals the presence of an extra-axial tumor in the region of the left cerebella-pontine
anglE. What is the most likely diagnosis?
A Epidermoid tumor C.holesteatomA.
B Glioblastomamultiforme
C Meningioma
D Acoustic neuroma
ANSWER:D
900
In the management of a 64-year-old woman struck by a car, mannitol is given to do which of the
following?
A Increase CSF formation
B Increase the respiratory rate
C Increase the pulse rate
D Lower raised ICP
ANSWER:D
901
A 64-year-old woman complains of gait imbalance, headache and deterioration of mental status
over the past several months. Her vision is normal. A CT scan reveals hydrocephalus, but the lumbar
puncture pressure is unexpectedly low. What does she have?
A Meningitis
B Normal-pressure hydrocephalus
C Sigmoid sinus thrombosis
D Echinococcus
ANSWER:B

902
A 23-year-old woman complains of progressive loss of vision and papilledemA. Investigations show
normal findings on CT scan. A lumbar puncture shows marked elevation of pressurE. What is the
most likely diagnosis?
A Pseudotumorcerebri
B Corpus cavernous thrombosis
C Cavernous sinus thrombosis
D Retinoblastoma
ANSWER:A
903
During her eighth month of pregnancy, a 29-year-old woman is noted to have hydramnios. Further
testing shows anencephalus. In this case hydramnios is caused by which of the following?
A Impairment of the fetus’s swallowing mechanism
B Tumor of the fetus’s brain
C A secretory peptide from the placenta
D Excess antidiuretic hormone A.DH) from the fetus
ANSWER:A
904
A 28-year-old man presents with a history of chronic headachE. The headache is intermittent,
severe, poorly localized, and most often present when he arises in the morning. He suffered a severe
blow to the head and sustained a skull fracture at the age of 15. Findings on his physical and
neurologic examinations are within normal limits. An MRI of the brain with gadolinium reveals the
presence of a large, nonenhancing extra-axial cyst in the region of the right temporal tip. This most
likely represents which of the following?
A An arachnoid cyst
B A cystic astrocytoma
C Rathke’s cleft cyst
D A Dandy-Walker cyst (failure of proper formation of the foramina of Lushka and MagendiE.
ANSWER:A
905
A 50-year-old male is admitted as an elective right hemicolectomy for a caecal tumour. Seventy-two
hours postoperation he develops swinging pyrexia and is tender on palpation of the abdomen. What
type of operation is this?
A Clean
B Clean contaminated
C Contaminated

D Dirty
ANSWER:B
906
A 73-year-old lady presents to the outpatient department for rou- tine follow up 4 weeks after an
abdominoperineal resection, for rectal cancer. She is doing extremely well, and just as she is leav-
ing happens to mention that her left leg has become extremely painful over the past 2 weeks. On
examination, the left lower leg is erythematous, painful to touch and oedematous, peripheral pulses
intact, with no obvious traumA. What is the most likely diagnosis?
A Cellulitis
B Deep vein thrombosis
C Ruptured Baker’s cyst
D Muscle rupture
ANSWER:B
907 In administration prophylactic antibiotics , the best timing
A 1 hour before surgery
B 24 after surgery
C 48 hours after surgery
D 1 hour after surgery
ANSWER:A
908
A 26 year old man , stabbed in the centre of the abdomen ,he presented to ER with knife blade still
embedded in the abdomen . His pulse 125 b/m his BP 80/40 mmHg .all the following measure is
correct except
A Remove the knife immediately in the emergency room
B Place IV cannula and start administration of ringer lactate
C Cross match blood
D No need for imaging study of the abdomen
ANSWER:A
909 The most important factor that affects wound healing is
A Wound closure within 2 hours
B Proper choice of suture material
C Dialy change of the wound dressing
D Adequate vascularity of the edges
ANSWER:D

910 The differential diagnosis of hypercalcaemia inculd all the following except:
A Primary hyperparathyroidism
B Bone metastases
C Multiple myeloma
D Tuberculosis
ANSWER:D
911 About the anatomy of the submandibular salivary gland all the following statements are true except
A The mylohyoid muscle divide the gland into superficial and deep part
B The submandibular duct arises from superficial part of the gland
C The deep part of the gland is related to hypoglossal nerve
D The submandibular duct is related closely related to the lingual nerve
ANSWER:B
912
Which of the following characteristics is advantage of full thickness skin grafts over split thickness
skin graft
A Superior skin pigment match
B Better resistance to infection
C Large area of covarge
D Lower incidence of contacture
ANSWER:D
913 Complications and disadvantages of nasogastric tube feeding inculde all the following except
A High cost
B Pharyngitis
C Possibility of aspiration pneumonia
D Esophagitis
ANSWER:A
914 What's true regarding the management of pleomorphic adenoma of the parotid gland
A
Opreation is advised immediately after diagnosis beacuse there is high risk of malignant
transformation early
B Require only simple enculeation beacuse the tumor is enculated
C Required only simple enculeation because the tumor is enculeated
D Partial parotidectomy with conservative of the fascial nerve is preformed

ANSWER:D
915
Which of the following medications administer for hyperkalemia counteract the myocardial effects
of the potassium without reducing the serum potassium level
A Sodium polystyrene sulfonate ( kayexalatE.
B Sodium bicarbonate
C 50% dextrose
D Calcium gluconate
ANSWER:D
916
A 43 year old woman with Von willebrand disease scheduled for cholecystectomy. It can be stated
that preoperative evaluation will reveal which of the following
A Normal bleeding time , PT ,PTT
B Platelet aggregate with restocetin
C Increased bleeding time and PTT , normal PT
D Increased bleeding time and PT , normal PTT
ANSWER:C
917
A 25 year old woman present with bengin nevus on the right upper arm. She desires removal and
undergoes a clean inscision then closure of the inscision without complications. With regard to
healing process , which of the following cell types are the first infiltrating cells to enter the wound
sites. Peaking at 24 to 48 hours .
A Macrophages
B Neutrophils
C Fibroblasts
D Lymphocytes
ANSWER:A
918
A 3 years old boy is brought to ER after spilling bleach onto his lower extremities. He is diagnosed
chemical burn and all involving clothing is removeD. In addition to resuscitation. Which of the
following is the most appropriate intiail management of this patient
A Treatment of the burn wound with antimicrobial agents
B Neutralize the burn wound with weak acid
C Lavage of the burn wound with large volume of water
D Wound debridement in the operating room
ANSWER:C

919
A 60 year old man with carcinoma of esophguse is admitted with sever malnutrition . Nutritional
support is to be initiateD. What should his daily intake
A 1 kcal/kg body weight / day
B 5 kcal/ kg body weight/ day
C 15 kcal/ kg body weight/ day
D 30 kcal/ kg body weight / day
ANSWER:D
920 Which of the following statements fales regarding salivary gland tumor
A Approximately 80 % of parotid tuomr are Benign
B The most common location of adenoid cystic carcinoma is the parotid gland
C Treatment of pleomorphic adenoma of the parotid gland involved superficial
D parotidectomy
ANSWER:B
921
Four days after undergoing a right hemicolectomy for ceacal volvulus, an obese 65 year old man
has a leakage of serosanguineouse fluid between the inscision staples, the dresssing is soakeD. He
has COPD which he takes corticosteroids. His vital signs are stablE. Abdomen examination shows
distention with minimal inscisional tenderness and no erthyemA. What is the most likely diagnosis?
A Anastomosis leak
B Wound infection
C Necrotizing fascitis
D Wound dehiscence
ANSWER:D
922
A 53 year old man brought into ER after a gunshot wound to the left flank .You suspect a splenic
injury with ongoing bleeding . The patient on coumarin (warfarin ) . His INR is 3.0 what do you do
to correct coagulopathy on route to the operating room
A IV vitamin K
B Orally administered vitamin k
C Fresh frozen plasma transfusion
D PRBCs
ANSWER:C
923 the most urgent measure in management of severly injured patient

A Control of active bleeding
B Taking blood sample for grouping and cross matching
C Establishing an intervenouse line
D Securing clear airway and adaquate pulmonary ventilation
ANSWER:D
924
A 36 year old man who was hit by a car presents to ER hypotension , on examination he has
tenderness and brusing over the left lateral chest below nipple ,ultrasound examination is performed
and reveals free fluid in the abdomen . What's most likely organ to have been injured ?
A Liver
B Kidney
C Spleen
D Intestine
ANSWER:C
925
A young man complains of bright red blood on paper and in the pan after passing stool. He also
occasionally feels a 'lump come down' when straining.What is the most likely diagnosis?
A Colonic carcinoma
B Fissure
C Haemorrhoids
D Diverticular disease
ANSWER:C
926
A 50-year-old obese man complains of epigastric discomfort, and on OGD the OG (oesophageal
gastriC. junction is noted to be at an abnormal position, though intact, with no other abnormality.
What is the likely diagnosis?
A Hiatus hernia; sliding
B Hiatus hernia; rolling
C Diaphragmatic hernia
D Congenital hernia
ANSWER:B
927
During examination of the patient you notice that when he is asked to raise his head off the bed
there is a large midline mass protruding from the xiphisternum to the umbilicus. What is this likely
to be

A Congenital diaphragmatic hernia
B Incisional hernia
C Traumatic hernia
D Divarification of the rectus
ANSWER:D
928
69-year-old female presents to her GP with a 1week history of being told she looks 'yellow by her
friends. She also complains of itching. On examination there is a mass in the right upper quadrant.
She is pain freE.What sign/law is this?
A Trousseau's sign
B Homans' sign
C Courvoisier's law
D Rovsing's sign
ANSWER:C
929
An elderly gentleman underwent extensive gastric resection and anastomosis without bypass. He
reports that he is doing well, his bowels are returning to normal and his appetite is picking up,
however he feels noticeably more fatigued since the operation. What would be a concern?
A Dumping syndrome
B Anaemia: B12 defi ciency
C Steatorrhoea
D Anaemia: folate defi ciency
ANSWER:B
930
18 years old male student who lives at a hostel, presented with one day history of anal pain. On
examination there was anal fissure at 6, O'clock position. This patient should be treated by:
A Fissurectory
B Lateral sphincterotomy
C Local anesthesia, analgesic and laxative
D Manual dilatation under general anesthesia
ANSWER:C
931
A baby aged 5 months presented to the outpatient clinic with a mass in his umbilicus that appears on
crying and disappears spontaneously. Which the following statement is TRUE regarding this case?
A It should be operated upon urgently

B Operation must be postponed till the age of puberty
C The best thing is to put a truss
D It can disappear spontaneously
ANSWER:D
932
49 Years old lady developed change in bowel habit of recent onset. Barium investigation showed
apple core appearance This characteristic feature of
A Ulcerative colitis
B Cancer colon
C Ischemic colitis
D Angiopdysplesia
ANSWER:B
933
A 32-week pregnant patient presented with bright red bleeding per rectum, which was not mixed
with the stool. The most probable diagnosis
A duodenal ulcer
B Piles
C Diverticulitis
D Colonic polyps
ANSWER:B
934
19 years old male patient is suffering from abdominal pain that is releived by food or drinking
sodA. Today be presented to the casualty with 10 hours of vomiting and severe abdominal pain . On
examination the abdomen was tender & rigiD. What is the most probable diagnosis
A Pyloric stenosis
B Cancer stomach
C Bleeding ulcer
D Perforated ulcer
ANSWER:D
935
55 years old diabetic patient presented with 5 days history of throbbing anal pain, his temperature
was 38.90° C, pulse was 97 min examination revealed red tender mass in perianal areA. The BEST
managementement is:
A Hot fomentation
B Bed rest
C Systemic antibiotic

D Incision and drainage
ANSWER:D
936
A 45 year old man presents with a mass in right lower abdomen for 2 months he has altered bowel
habits & weight loss but no vomiting. Which of the Following conditions is NOT a differential
diagnosis in this patient?
A Ileo-cecal kochs
B Crohn's disease
C Carcinoma cecam
D Mesenteric ischemia
ANSWER:D
937
A man aged 32 years complains of pain and distention after meals. He feels better after inducing
vomiting. He had past history of hunger pain. The pain is periodical and comes after tension. On
abdominal examination visible peristalsis from left to right. No mass could be felt the most probable
diagnosis is
A Gastric outlet obstruction due to carcinoma of the pylorus
B Chronic gastritis
C Gastric outlet obstruction due to chronic duodenal uicer.
D Upper intestinal obstruction
ANSWER:C
938
A 45-year-old patient presents in shock complaining of sudden-onset generalized upper abdominal
pain radiating to the right iliac fossa and the tip of his right shoulder. He reports one episode of
vomiting, but none sincE. He has no past medical problems. On examination, his abdomen is rigid
and bowel sounds are absent. The diagnosis is:
A Caecal volvulus
B Pancreatitis
C Perforated duodenal ulcer
D Ascending cholangitis
ANSWER:C
939
A 2.8 kg Neonate with excessive salivation develops respiratory distress. Attempts to pass an
orogastric catheter fail because the catheter coils in the back of the throat. A chest film is obtained
and shows right upper lobe atelectasis and a gasless abdomen. The most likely diagnosis is :

A Proximal esophageal atresia without a fistulA.
B Proximal esophageal atresia with a distal tracheoesophageal (TE. fistulA.
C H-type- TE fistulA.
D Esophageal atresia with both proximal and distal TE fistulA.
ANSWER:A
940
Four days after undergoing a right hemicolectomy for cecal volvulus, obese 65 years old man has a
leakage of seroanguineous fluid between the incision staples, the dressing is soakeD. He has COPD
to which he takes corticosteroids . His vital signs are stablE. Abdomen examination shows distention
with minimal incisional tenderness and no erythemA. What's the most likely diagnosis?
A Anastomotic leak
B Wound infection
C Necrotizing fasciitis
D Wound dehiscence
ANSWER:D
941
An elderly nursing home pt is brought to the hospital with recent onset of colicky abdominal pain,
distention and constipation on examination the abdomen is markedly distended and tympaniC.
There's no marked tenderness. Plain abdominal X-Ray shows a markedly distended loop located
mainly in the right upper quadrant. The likely diagnosis is:
A Small bowel obstruction
B Large bowel obstruction
C Gallstone
D Mesenteric vascular occlusion
ANSWER:B
942
45 years old pt. Presents in shock complaining of sudden onset generalized upper abdominal pain
radiating to the right iliac fossa and the tip of his right shoulder. He reports one episode of vomiting.
But none sincE. He has no past medical problems. On examination, his abdomen is rigid and bowel
sounds are absent. The diagnosis is:
A Caecal volvulus
B Pancreatitis
C Perforated duodenal ulcer
D Ascending cholangitis
ANSWER:C

943
2.8 kg neonate with excessive salivation develops respiratory distress. Attempts to pass an orogastric
catheter fail because the catheter coils in the back of the throat. A chest film is obtained and shows
right upper lobe atelectasis and a gasless abdomen. The most likely diagnosis is:
A Proximal esophageal atresia without fistula
B Proximal esophageal atresia with a distal tracheoesophageal fistula
C H-type TE fistula
D Esophageal atresia with both proximal and distal TE fistula
ANSWER:A
944
years old Male pt has DM presented with acute RUQ pain for hours and vomiting. He is febrile
39 C and tachycardiC. He is jaundiced with total bilirubin of 3. and direct of 2.8 his WBC is
16,000 . Hb 14 , ALT 150 , AST 90 , he has past history of gallbladder stones, regarding his
diagnosis and management which is true:
A The pt has acute cholecystitis and needs I.V antibiotics and lap. cholecystectomy
B The pt has acute cholecystitis and needs ERCP and lap. Cholecystectomy
C The pt has ascending cholangitis and needs I.V antibiotics and ERCP urgently
D The pt has ascending cholangitis and needs I.V antibiotics and lap cholecystectomy
ANSWER:C
945
A 60-year-old diabetic man is admitted to the hospital with a diagnosis of acute cholecystitis. The
WBC count is 28000, and a plain film of abdomen and CT scan show evidence of intramural gas in
the gallbladder. What is the most likely diagnosis?
A emphysematous GB
B acalculous cholecystitis
C sclerosing cholangitis
D gallstone ileus
ANSWER:A
946
2,8 kg neonate with excessive salivation developed respiratory distress attempt to pass orogastric
catheter fall because the catheter in the back of throat chest film is obtain and show right upper lobe
atelectasis and gasless abdomen the most likely diagnosis
A Proximal esophagus atresia without fistula
B Proximal esophagus atresia with distal tracheoesophagus fistula
C H type tracheoesophagus fistula

D Esophagus atresia with both proximal and distal tracheoesophus fistula
ANSWER:A
947
You are asked to see a 73-year-old lady with a large ulcer over the medial malleolus of her left foot.
There is a healthy base, with granulation tissue present. There are superficial dilated veins over the
medial aspect of the leg.What is the likely diagnosis?
A Venous ulcer
B Arterial ulcer
C Marjolin’s ulcer
D Curling’s ulcer
ANSWER:A
948
A 56-year-old male complains of pain and weakness in both legs on standing and on exercisE. It is
not relieved by rest. He is a non-smoker, with no relevant family history. On examination peri-
pheral pulses are intact.What is the likely diagnosis?
A Neurogenic claudication
B Varicose veins
C Vascular claudication
D Peripheral myopathy
ANSWER:A
949
A 73-year-old lady presents to the outpatient department for rou- tine follow up 4 weeks after an
abdominoperineal resection, for rectal cancer. She is doing extremely well, and just as she is leav-
ing happens to mention that her left leg has become extremely painful over the past 2 weeks. On
examination, the left lower leg is erythematous, painful to touch and oedematous, peripheral pulses
intact, with no obvious trauma. What is the most likely diagnosis?
A Cellulitis
B Deep vein thrombosis
C Ruptured Baker’s cyst
D Muscle rupture
ANSWER:B
950
A routine sography for healthy 59 years old female shows a cluster of microcalcification . What is
the most appropriate next step in management this patient:
A Repeat mammography again
B Repeat mamography at 6 months

C Presecribe anti estrogen for 6 months
D Take a biopsy
ANSWER:D
951
A 34 years old woman present with hypertension , generlized weakness and polyuriA. Her
electrolyte panel is significant for hypokalaemia . Which of the following is the best intiail test given
her presentation and laboratory findings ?
A Plasma renin activity and plasma aldosterone concentration
B Urine electrolytes
C Plasma cortisol level
D Overnight low dose dexamethasone suppression test
ANSWER:A
952
As an accidental finding during upper Abdominal CT scan , a 3 cm mass in adrenal gland is noted .
Which of the following would Not be an appropriate step in patients work up ?
A Laparoscopic adrenalectomy
B Measurements of serum electrolytes
C Measurements of plasma aldosterone and renin
D Measurements of urine catecholamines excretion
ANSWER:A
953
A 29-year-old woman is referred by her doctor with a 3- month history of a painless lump in the
anterior aspect of the neck. She does not describe any hoarseness of voicE. Examination reveals a
lump in the left anterior triangle, which moves up and down with swallowing. FNA cytology of the
lump reveals nuclear grooves and Psamomma's bodies. What is the most likely diagnosis?
A Follicular adenoma
B Follicular carcinoma
C Papillary carcinoma
D Medullary carcinoma
ANSWER:C
954
All of the following are considered an increased a risk factor for cancer in a patient with a thyroid
mass except
A Age younger than 25 or older than 60
B Rapid growth
C Family history

D Hot nodule on thyroid uptake scan
ANSWER:D
955
60-year-old smoker presents to the urology clinic after a GP referral for haematuriA. He is pain free
with only the occasional clot seen. He is on warfarin, his most recent INR being 2.5 yesterday.
Where is the likely location of the bleeding?
A Kidney
B Ureter
C Bladder
D Prostate
ANSWER:C
956
The patient continues to have symptoms despite this treatment, and undergoes a TURP.
Postoperatively he initially does well, but his wife reports that he does not seem himself and is
becoming increasingly confused; he also complains of nauseA. His observa- tions are stable and
bladder irrigation fluid is flowing very well. Bloods are sent to the lab and reveal hyponatraemiA.
What is the likely diagnosis?
A Internal haemorrhage
B TURP syndrome
C Urinary infection
D Bladder neck stenosis
ANSWER:B
957
A 23 year old male presented with acute scrotal pain and swelling which of the following suggest
epididymitis over testiculer torsion as a cause of his presentation?
A Presence of bell clapper deformity
B Absent of cremastric reflex
C Postive perhn's sign
D Elevated WBC
ANSWER:C
958
You are called to the ward to review a patient who has lost consciousness after what the nurse
describes as a heavy fall, broken only by his head after tripping on his walking framE. She tells you
he is on warfarin for atrial fibrillation, and she is worrieD. At what point should intubation be
considered?
A GCS < 12

B GCS < 10
C GCS < 8
D GCS < 6
ANSWER:C
959
A 55-year-old woman gives a history of tiredness, aching, and a feeling of heaviness in the left
lower leg for the past 3 months. These symptoms are relieved by leg elevation. She is also awakened
frequently by calf and foot cramping, which is relieved by leg elevation, walking, or massagE. On
physical examination there are superficial varicosities, non pitting edema, and a slightly painful, 2
cm. diameter superficial ulcer 5 cm. above and behind the left medial malleolus. What is the most
appropriate diagnosis?
A Isolated symptomatic varicose veins.
B Superficial lymphatic obstruction.
C Deep venous insufficiency.
D Arterial insufficiency.
ANSWER:C
960
A 28-year-old woman developed a painful thrombosis of a superficial varix in the left upper calf 2
days previously. After spending the 2 days in bed with her leg elevated, she felt better and the
tenderness resolved; however, when out of bed she developed a twinge of right-sided chest pain
when walking and a feeling of heaviness in the calf. Which treatment is most appropriate?
A Check for leg swelling, tenderness, and Homan's sign, and obtain a Doppler ultrasound study.
B Begin antibiotics for a probable secondary bacterial infection.
C Order emergency venography, and if it is abnormal, begin heparin administration.
D
Begin ambulation and discontinue bed rest that probably caused muscle pain by hyperextension of
the kneE.
ANSWER:C
961
In a 55-year-old grocery store cashier with an 8-month history of leg edema increasing over the
course of a work day, associated with moderate to severe lower leg bursting pain, the most
appropriate investigative study or studies are:
A Doppler duplex ultrasounD.
B Brodie-Trendelenburg test.
C Ascending and descending phlebography.
D Measurement of ambulatory and resting foot venous pressurE.

ANSWER:A
962
A 35-year-old man involved in a motor vehicle accident presents with a knee dislocation that is
easily reduceD. Radiography of the knee shows no fracturE. Which of the following statements
about his treatment are true?
A If he has normal pulses he can be discharged.
B If he has normal pulses he requires either close observation or arteriography.
C
If he has absent distal pulses and severe ischemia he should undergo arteriography in the radiology
suite.
D A popliteal vein injury is best treated with ligation.
ANSWER:B
963
A 24-year-old man is involved in an industrial accident in which he sustains a crushed pelvis.
Diagnostic peritoneal lavage is positivE. At exploration, a large pelvic hematoma is founD. What is
the best treatment?
A
Explore all the major arteries and veins of the pelvis and surgically control the bleeding if possiblE.
B Do not explore the pelvic hematomA. Close the abdomen and apply a MAST suit.
C
Do not explore the pelvic hematomA. Apply a pelvic fixator and send the patient to radiology f or
possible embolization of bleeding pelvic vessels.
D Use sustained hypotensive anesthesia to try to control bleeding.
ANSWER:C
964
A43-year-old teacher underwent left parotidectomy. Upon awakening from surgery, paralysis of the
left lower lip was observeD. This complication was most likely due to injury to which of the
following:
A Parotid duct
B Facial nerve - temporal branch
C Facial nerve - cervical branch
D Facial nerve - main trunk
ANSWER:C
965
A9-year-old boy complains of a swelling on the left side of his neck in the supraclavicular region.
The swelling is translucent; a diagnosis of cystic hygroma is establisheD. What is true of cystic
hygroma?
A It arises from sweat glands in the neck.

B It is usually an anterior midline structurE.
C It may occur in the mediastinum.
D Its lesions are usually easy to enucleatE.
ANSWER:C
96
A 62-year-old man undergoes excision of a cylindroma of the submandibular glanD. He is most
likely to have an injury to which of the following?
A Maxillary branch of the trigeminal nerve
B Lingual nerve
C Vagus nerve
D Floor of the maxilla
ANSWER:B
967
A 62-year-old alcoholic presents with an indurated ulcer, 1.5 cm in length, in the left lateral aspect
of her tongue (not fixed to the alveolar ridgE. There are no clinically abnormal glands palpable in
the neck, and a biopsy of the tongue lesion reveals squamous cell carcinomA. What should she
undergo?
A Chemotherapy
B Local excision of the ulcer
C Wide excision and left radical neck dissection
D Antibiotic therapy and should be encouraged to stop smoking
ANSWER:C
968
A 59-year-old woman has discomfort in the posterior part of her tonguE. Abiopsy confirms that the
lesion is a carcinomA. What is true in carcinoma of the posterior third of the tongue?
A Lymphoid tissue is absent.
B Lymph gland spread is often encountereD.
C There is an excellent prognosis.
D The tissue is well differentiateD.
ANSWER:B
969
A 58-year-old woman undergoes excision biopsy of a tumor in the left posterior triangle of her
neck. Histology suggests that this is a metastatic cancer. What is the most likely site of the primary
tumor?
A Ovary
B Adrenal gland

C Kidney
D Piriform fossa
ANSWER:D
970
A 33-year-old female noted a discharge from a sinus in the overlying skin below the right angle of
the mandiblE. She recalls previous episodes of fullness and mild pain in this region over the past
several years. What is the most likely cause?
A Thyroglossal duct cyst
B Branchial cyst
C Teratoma
D Myeloma
ANSWER:B
971
A 5-year-old girl presents with difficulty breathing. On examination, of the oral cavity a 3-cm mass
is found in the midline on the posterior aspect of the tonguE. The most likely diagnosis is:
A Lingual tonsil
B Lingual thyroid
C Foreign body stuck to the tongue
D Dermoid
ANSWER:B
972
A 17 year old girl presented with o 2.5 cm nodule in the right lobe of the thyroid gland and
enlarged three cervical LNs confirmed by US. FNA cytology revealed malignant cells with vesicular
nuclei, the most probable diagnosis is:
A Lymphoma
B Anaplastic carcinoma
C Follicular carcinoma
D Papillary carcinoma
ANSWER:D
973
A 30-yeor old female presents for evolution of a palpable thyroid nodule Tc99 scan demonstrated a
single cold nodule it may be the following EXCEPT:
A Carcinoma
B Non-functioning adenoma
C Thyroid cyst
D Autonomous nodule

ANSWER:D
974
A64-year-old supermarket manager had an elective operation for duodenal ulcer diseasE. He has
not returned to work because he has diarrhea with more than 20 bowel movements per day.
Medication has been ineffectivE. The exact details of his operation cannot be ascertaineD. What
operation was most likely performed?
A Antrectomy and Billroth I anastomosis
B Gastric surgery combined with choleystectomy
C Truncal vagotomy
D Highly selective vagotomy
ANSWER:C
975
A40-year-old man has had recurrent symptoms suggestive of peptic ulcer disease for 4 years.
Endoscopy reveals an ulcer located on the greater curvature of the stomach. A mucosal biopsy
reveals Helicobacter. pylori. What is TRUE About H. pylori?
A Active organisms can be discerned by serology.
B It is protective against gastric carcinomA.
C It is associated with chronic gastritis.
D It causes gastric ulcer but not duodenal ulcer.
ANSWER:C
976
A 63-year-old man has an upper gastrointestinal (UGI) study as part of his workup for abdominal
pain. The only abnormal finding was in the antrum, where the mucosa prolapsed into the
duodenum. There were no abnormal findings on endoscopy. What should he do?
A Sleep with his head elevateD.
B Be placed on an H2 antagonist.
C Undergo surgical resection of the antrum.
D Be observed and treated for pain accordingly.
ANSWER:D
977
A63-year-old man underwent gastric resection for severe peptic ulcer diseasE. He had complete
relief of his symptoms but developed “dumping syndromE.” This patient is most likely to complain
of which of the following?
A Gastric intussusception
B Repeated vomiting
C Severe diarrhea

D Severe vasomotor symptoms after eating
ANSWER:D
978
A68-year-old woman has been diagnosed with a benign ulcer on the greater curvature of her
stomach, 5 cm proximal to the antrum. After 3 months of standard medical therapy, she continues to
have guaiac positive stool, anemia, and abdominal pain with failure of the ulcer to heal. Biopsies of
the gastric ulcer have not identified a malignancy. The next step in management is which of the
following?
A Treatment of the anemia and repeat all studies in 6 weeks
B Endoscopy and bipolar electrocautery or laser photocoagulation of the gastric ulcer
C
Admission of the patient for total parenteral nutrition (TPN), treatment of anemia, and endoscopic
therapy
D Surgical intervention, including partial gastric resection
ANSWER:D
979
A 50-year-old woman presents with duodenal ulcer disease and high basal acid secretory outputs.
Secretin stimulated serum gastrin levels are in excess of 1000 pg/mL. She has a long history of ulcer
disease that has not responded to intense medical therapy. What is the most likely diagnosis?
A Hyperparathyroidism
B Pernicious anemia
C Renal failure
D ZES
ANSWER:D
980
A50-year-old man presents with vague gastric complaints. Findings on physical examination are
unremarkablE. The serum albumin level is markedly reduced (1.8 g/100 mL). A barium study of
the stomach shows massive gastric folds within the proximal stomach. These findings are confirmed
by endoscopy. What is the correct diagnosis?
A Hypertrophic pyloric stenosis
B Gallstone ileus
C Mallory-Weiss tear
D Hypertrophic gastritis
ANSWER:D

981
A 36-year-old man presents with weight loss and a large palpable tumor in the upper abdomen.
Endoscopy reveals an intact gastric mucosa without signs of carcinomA. Multiple biopsies show
normal gastric mucosA. A UGI study shows amass in the stomach. At surgery, a 3-kg mass is
removeD. It is necessary to remove the left side of the transverse colon. What is the most likely
diagnosis?
A Gastric cancer
B Gastrointestinal stromal tumor (GIST)
C Choledochoduodenal fistula
D Eosinophilic gastroenteritis
ANSWER:B
982
A 64-year-old woman presents with severe upper abdominal pain and retching of 1-day duration.
Attempts to pass a nasogastric tube are unsuccessful. Xrays show an air-fluid level in the left side of
the chest in the posterior mediastinum. An incarcerated paraesophageal hernia and gastric volvulus
is diagnoseD. What is the next step in management?
A Insertion of a weighted bougie to untwist the volvulus
B Elevation of the head of the bed
C Placing the patient in the Trendelenburg position with the head of the bed lowered
D Surgery, reduction of the gastric volvulus, and repair of the hernia
ANSWER:D
983
A 78-year-old woman undergoes an uncomplicated minor surgical procedure under local
anesthesiA. At the completion of the operation, she suddenly develops pallor, sweating, bradycardia,
hypotension, abdominal pain, and gastric distension. What is the next stem in management?
A Rapid infusion of 3 L of Ringer’s lactate
B Digoxin
C Insertion of a nasogastric tube
D Morphine
ANSWER:C
984
A 67-year-old woman complains of paresthesias in the limbs. Examination shows loss of vibratory
sense, positional sense, and sense of light touch in the lower limbs. She is found to have pernicious
anemiA. Endoscopy reveals an ulcer in the body of the stomach. What does she most likely have?

A Excess of vitamin B12
B Deficiency of vitamin K
C Cancer of the stomach
D Gastric sarcoma
ANSWER:C
985
A 79-year-old retired opera singer presents with dysphagia, which has become progressively worse
during the last 5 years. He states that he is sometimes awareof a lump on the left side of his neck and
that he hears gurgling sounds during swallowing. He sometimes regurgitates food during eating.
What is the likely diagnosis?
A Carcinoma of the esophagus
B Foreign body in the esophagus
C Plummer-Vinson (Kelly-Patterson) syndrome
D Zenker’s (pharyngoesophageal) diverticulum
ANSWER:D
986
A30-year-old psychiatric patient has a barium swallow after removal of a foreign body to rule out a
small perforation of the esophagus. No perforation is seen, but an epiphrenic diverticulum is
visualizeD. An epiphrenic diverticulum may be associated with which of the following?
A Duodenal ulcer
B Gastric ulcer
C Cancer of the tongue
D Hiatal hernia
ANSWER:D
987
A 53-year-old moderately obese woman presents with heartburn aggravated mainly by eating and
lying down in the horizontal position. Her symptoms are suggestive of gastroesophangeal reflux
disease (GERC. Which of the following statements is TRUE?
A It is best diagnosed by an anteroposterior A.P) and lateral film of the chest.
B
It may be alleviated by certain drugs, especially theophylline, diazepam, and calcium channel
blockers.
C It is not relieved by cessation of smoking.
D If it is associated with dysphagia, it suggests a stricture or motility disorder.
ANSWER:D

988
A 46-year-old man present with dysphagia of recent onset. His esophogram shows a lesion in the
lower third of his esophagus. Biopsy of the lesion shows adenocarcinomA. His general medical
condition is excellent, and his metastatic workup is negativE. What should his management involve?
A Chemotherapy
B Radiation therapy
C Insertion of a stent to improve swallowing
D Surgical resection of the esophagus
ANSWER:D
989
A 44-year-old woman is scheduled for gastric surgery. She has no comorbid diseasE. The
anesthesiologist has difficulty inserting the orotracheal tubE. In between intubation attempts he uses
an ambu-bag to oxygenate the patient. The patient’s abdomen gets distended and tympany is noted
in the left upper quadrant. Suddenly the patient becomes hypotensivE. Which of the following can
cause a vosogvagal response during anesthesia?
A The gastric remnant following a distal gastrectomy
B Corrosive gastritis
C Pernicious anemia
D Acute gastric dilatation
ANSWER:D
990
A 40-year-old woman complains of heartburn located in the epigastric and retrosternal areas. She
also has symptoms of regurgitation. Endoscopy shows erythema of the esophagus consistent with
reflux esophagitis. The patient has tried conservative measures, including PPls with no improvement
in symptoms. Which of the following is TRUE?
A Manometry does not add any additional information.
B The 24-hour pH test is no longer useD.
C If endoscopy has been done, an esophagogram is unnecessary.
D Nissen fundoplication is the surgical treatment of choicE.
ANSWER:D

991
A73-year-old woman is admitted to the hospital with a mild UGI hemorrhage that stopped
spontaneously. She did not require transfusion. She had ingested large amounts of aspirin in the past
4 months to relieve the pain caused by severe rheumatoid arthritis. Endoscopy confirms the
presence of a duodenal ulcer. A biopsy is donE. What is the next step in the management of a
duodenal ulcer associated with a positive biopsy for H. pylori?
A H2 blockers
B Bipolar electrocautery of the ulcer
C Triple therapy
D Photocoagulation
ANSWER:C
992
A 50-year-old gynecologist complains of dysphagia, regurgitation, and weight loss. She also states
that she feels as if food is stuck at the level of the xiphoiD. An upright chest x-ray shows a dilated
esophagus with an air-fluid level. Which of the following is FALSE?
A A barium swallow will show a “bird’s peak” deformity
B Manometry will demonstrate that the LES fails to relax during swallowing.
C Upper endoscopy should be avoided because of the risk of complications.
D Medical treatment includes nitrates and calcium channel blockers.
ANSWER:C
993
A 50-year-old man presents to the emergency department with chest pain. The patient is evaluated
for a myocardiac infarction. The workup is negativE. On further questioning, his symptoms include
dysphagia (with both liquids and solids). Which of the following is TRUE?
A A barium swallow will always show a corkscrew esophagus.
B Manometry shows simultaneous high-amplitude contractions.
C Initial evaluation should exclude coronary artery diseasE.
D A pulsion diverticulum may be present.
ANSWER:A
994
A 60-year-old man has been having vague symptoms of upper abdominal discomfort, early satiety,
and fatiguE. He is referred to a gastroenterologist, who performs an upper endoscopy. Although a
discrete mass is not visualized, the stomach looks abnormal. It does not distend easily with
insufflation. A biopsy shows signet ring cells. Which of the following is TRUE?
A Signet ring cells are typically found in intestinal type gastric adenocarcinomA.
B Signet ring cell cancer is the most common type of gastric cancer.

C “Leather bottle stomach” is a term used to describe a nondistensible stomach infiltrated by cancer.
D The gross appearance of the stomach always shows classic findings of linitus plastica.
ANSWER:C
995
A 54-year-old man presents with a massive UGI bleed. After resuscitation, endoscopy is performed.
No esophageal varices, gastritis, or gastric ulcers are seen. After copious irrigation, a pinpoint lesion
is seen near the GE junction. What can be said about this lesion?
A It is a carcinoid.
B It is related to alcohol use.
C It is exclusively a mucosal lesion.
D Bleeding is from a submucosal vessel
ANSWER:D
996
A 15-year-old boy complains of right-sided weakness and gait impairment. ACT scan shows a large,
nonenhancing cyst in the posterior cranial fossa, with an enhancing tumor nodule in the left
cerebellum. What is the most likely diagnosis?
A An arachnoid cyst
B A cystic astrocytoma
C Rathke’s cleft cyst
D Glioblastomamultiforme
ANSWER:B
997
A 43-year-old man is treated with pyridostigmine for facial, ocular, and pharyngeal weakness due to
myasthenia gravis. Which statement is true of pyridostigmine?
A It is an anticholinesterase agentans
B It has far more side effects than neostigminE.
C Pyridostigmine and neostigmine reverse depolarizing neuromuscular blockadE.
D It causes greater muscarinic effect than neostigminE.
ANSWER:A
998
A 17-year-old male presents with 3-month history of headache, weight gain, decreased
concentration, polyuria, and polydypsiA. His headaches are mostly in morning and involvesthe
frontal region. On examination he was found to have bitemporal visual field defect and no facial
hair. MRI scan revealed a suprasellar partially calcified cystic lesion with displacement of optic
chiasm. The most likely pathology is:
A Giant aneurysm of carotid artery

B Pituitary macroadenoma
C Glioblastomamultiforme
D Craniopharyngioma
ANSWER:D
999 The following Nyhus classification of hernias is correct except for:
A Recurrent direct inguinal hernia—Type IVa.
B Indirect inguinal hernia with a normal internal inguinal ring—Type I.
C Femoral hernia—Type IIIc.
D
Indirect inguinal hernia with destruction of the transversalis fascia of Hesselbach’s triangle—Type II.
ANSWER:D
1000 Which of the following statements about the causes of inguinal hernia is correct?
A Excessive hydroxyproline has been demonstrated in the aponeuroses of hernia patients.
B
Obliteration of the processus vaginalis is a contributing factor for the development of an indirect
inguinal hernia.
C
Physical activity and athletics have been shown to have a protective effect toward the development
of inguinal hernias.
D
Elevated levels of circulating serum elastalytic activity have been demonstrated in patients with
direct herniation who smoke.
ANSWER:D
1001
A 28-year-old woman with a history of an appendectomy presents with a nontender palpable mass
in the right lower quadrant abdominal incision. The following statement(s) is/are true concerning the
diagnosis and management of this patient.
A The best diagnostic test involves imaging of the abdominal wall by either CT or MRI
B Resection of the mass with a 2 cm margin is usually adequate
C Low dose radiation is a suitable alternative to surgery for primary treatment
D Re-resection for recurrence will likely have a higher rate of recurrence than for primary resection
ANSWER:A
1002
A 77-year-old multiparous female presents with a bowel obstruction. She has no previous
abdominal operations and no abdominal wall hernias can be detected. In addition to her abdominal
symptoms, she reports pain in her right medial thigh. The following statement(s) is/are true
concerning her diagnosis and management.

A Expectant management with nasogastric suction and IV fluid replacement is indicated
B A right groin approach is indicated for exploration and repair of the presumed hernia
C The use of a polypropylene mesh will likely be necessary for repair
D
A correct diagnosis can usually be made by visualizing an external mass in the upper, medial thigh
ANSWER:C
1003 The following statement about peritonitis are all true except:
A Peritonitis is defined as inflammation of the peritoneum.
B Most surgical peritonitis is secondary to bacterial contamination.
C
Primary peritonitis has no documented source of contamination and is more common in adults than
in children and in men than in women.
D Tuberculous peritonitis can present with or without ascites.
ANSWER:C
1004 Acute appendicitis is most commonly associated with which of the following signs?
A Temperature above ؛104 F.
B Frequent loose stools.
C Anorexia, abdominal pain, and right lower quadrant tenderness.
D White blood cell count greater than 20,000 per cu. mm.
ANSWER:C
1005 Which of the following most often initiates the development of acute appendicitis?
A A viral infection.
B Acute gastroenteritis.
C Obstruction of the appendiceal lumen.
D A primary clostridial infection.
ANSWER:C
1006 The diagnosis of acute appendicitis is most difficult to establish in:
A Persons aged 60 and older.
B Women aged 18 to 35.
C Infants younger than 1 year.
D Pregnant women.
ANSWER:C
1007 The best type of x-ray to locate free abdominal air is:

A A posteroanterior view of the chest.
B A flat and upright view of the abdomen.
C Computed tomograph (CT) of the abdomen.
D A lateral decubitus x-ray, right side up.
ANSWER:D
1008 Meckel’s diverticulitis most often occurs in the:
A Proximal jejunum.
B Distal jejunum.
C Proximal ileum.
D Distal ileum.
ANSWER:D
1009 True statements regarding the pathophysiology of acute appendicitis include which of the following:
A Fecaliths are responsible for the disease process in approximately 30% of adult patients
B Lymphoid hyperplasia is a rare cause of appendicitis in young patients
C Clostridium difficile is implicated as a pathogenic organism
D Carcinoid tumors account for approximately 5% of all cases of acute appendicitis
ANSWER:A
1010 True statements regarding appendiceal neoplasms include which of the following?
A
Carcinoid tumors of the appendix less than 1.5 cm are adequately treated by simple appendectomy
B
Appendiceal carcinoma is associated with secondary tumors of the GI tract in up to 60% of patients
C
Survival following right colectomy for a Dukes’ stage C appendiceal carcinoma is markedly better
than that for a similarly staged colon cancer at 5 years
D
Mucinous cystadenocarcinoma of the appendix is adequately treated by simple appendectomy, even
in patients with rupture and mucinous ascites
ANSWER:A
1011 The most common hernia in females is:
A Femoral hernia.
B Direct inguinal hernia.
C Indirect inguinal hernia.

D Obturator hernia.
ANSWER:C
1012 Which of the following statements regarding unusual hernias is incorrect?
A
An obturator hernia may produce nerve compression diagnosed by a positive Howship-Romberg
sign.
B
Grynfeltt’s hernia appears through the superior lumbar triangle, whereas Petit’s hernia occurs
through the inferior lumbar triangle.
C Sciatic hernias usually present with a painful groin mass below the inguinal ligament.
D
Littre’s hernia is defined by a Meckel’s diverticulum presenting as the sole component of the hernia
sac.
ANSWER:C
1013
Staples may safely be placed during laparoscopic hernia repair in each of the following structures
except:
A Cooper’s ligament.
B Tissues superior to the lateral iliopubic tract.
C The transversus abdominis aponeurotic arch.
D Tissues inferior to the lateral iliopubic tract.
ANSWER:D
1014 The following statements about the repair of inguinal hernias are true except:
A The conjoined tendon is sutured to Cooper’s ligament in the Bassini hernia repair.
B The McVay repair is a suitable option for the repair of femoral hernias.
C The Shouldice repair involves a multilayer, imbricated repair of the floor of the inguinal canal.
D
The Lichtenstein repair is accomplished by prosthetic mesh repair of the inguinal canal floor in a
tension-free manner.
ANSWER:A
1015
Which of the following statement(s) is/are true concerning the diagnosis and management of
epigastric hernias?
A A large peritoneal sac containing abdominal viscera is common
B At the time of surgical repair, a careful search for other defects should be performed
C Recurrent epigastric hernias after simple closure is uncommon
D
Patients with symptoms of a painful midline abdominal mass frequently will contain incarcerated
small bowel

ANSWER:B
1016
A 28-year-old woman with a history of an appendectomy presents with a nontender palpable mass
in the right lower quadrant abdominal incision. The following statement(s) is/are true concerning the
diagnosis and management of this patient.
A The best diagnostic test involves imaging of the abdominal wall by either CT or MRI
B Resection of the mass with a 2 cm margin is usually adequate
C Low dose radiation is a suitable alternative to surgery for primary treatment
D Re-resection for recurrence will likely have a higher rate of recurrence than for primary resection
ANSWER:A
1017
A 77-year-old multiparous female presents with a bowel obstruction. She has no previous
abdominal operations and no abdominal wall hernias can be detected. In addition to her abdominal
symptoms, she reports pain in her right medial thigh. The following statement(s) is/are true
concerning her diagnosis and management.
A Expectant management with nasogastric suction and IV fluid replacement is indicated
B A right groin approach is indicated for exploration and repair of the presumed hernia
C The use of a polypropylene mesh will likely be necessary for repair
D
A correct diagnosis can usually be made by visualizing an external mass in the upper, medial thigh
ANSWER:C
1018 The following statement(s) is/are true concerning umbilical hernias in adults.
A Most umbilical hernias in adults are the result of a congenital defect carried into adulthood
B A paraumbilical hernia typically occurs in multiparous females
C The presence of ascites is a contraindication to elective umbilical hernia repair.
D Incarceration is uncommon with umbilical hernias
ANSWER:B
1019 The most helpful diagnostic radiographic procedure in small bowel obstruction is:
A CT of the abdomen.
B Contrast study of the intestine.
C Supine and erect x-rays of the abdomen.
D Ultrasonography of the abdomen.
ANSWER:C
1020 The most commonly used imaging method for diagnosis of acute cholecystitis is:

A CT of the abdomen.
B Ultrasonography of the gallbladder.
C Oral cholecystogram.
D Radionuclide (HIDA) scan of the gallbladder.
ANSWER:B
1021 Acute salpingitis occurs most often:
A After menopause.
B In patients with unilateral lower abdominal pain.
C During the menstrual cycle.
D In patients with cervical tenderness and vaginal discharge.
ANSWER:D
1022 Meckel’s diverticulitis most often occurs in the:
A Proximal jejunum.
B Distal jejunum.
C Proximal ileum.
D Distal ileum.
ANSWER:D
1023
A patient is seen in the emergency room with reproducible right lower quadrant tenderness. The
approximate incidence of finding a normal appendix on right lower quadrant exploration in similar
nonselected patients is which of the following:
A 5%
B 10%
C 20%
D 40%
ANSWER:C
1024
Of adult patients presenting to the emergency room for evaluation of acute abdominal pain, which
one of the following answers includes the most common diagnoses?
A Urologic problems, cholelithiasis, pelvic inflammatory disease
B Mittelschmerz, appendicitis, ureterolithiasis
C Nonspecific abdominal pain, appendicitis, intestinal obstruction
D Appendicitis, pelvic inflammatory disease, perforated ulcer
ANSWER:C

1025
Prospective studies have shown incidental appendectomy to be advantageous in which of the
following patient groups?
A Children undergoing staging laparotomy for malignancy who are then to enter chemotherapy
B HIV infected patients
C Patients over 50 years of age
D None of the above
ANSWER:D
1026 Visceral pain is typically:
A Well localized
B Sharp
C Mediated via spinal nerves
D Perceived to be in the midline
ANSWER:D
1027 True statements regarding appendiceal neoplasms include which of the following?
A
Carcinoid tumors of the appendix less than 1.5 cm are adequately treated by simple appendectomy
B
Appendiceal carcinoma is associated with secondary tumors of the GI tract in up to 60% of patients
C
Survival following right colectomy for a Dukes’ stage C appendiceal carcinoma is markedly better
than that for a similarly staged colon cancer at 5 years
D
Mucinous cystadenocarcinoma of the appendix is adequately treated by simple appendectomy, even
in patients with rupture and mucinous ascites
ANSWER:A
1028 Which of the following statements about the anatomic course of the esophagus is correct?
A The cervical esophagus passes behind and to the right of the trachea.
B The thoracic esophagus enters the posterior mediastinum anterior to the aortic arch.
C The thoracic esophagus passes behind the right mainstem bronchus and the pericardium.
D The esophagus deviates anteriorly and to the left as it enters the abdomen.
ANSWER:D
1029 Which of the following statements about esophageal anatomy is correct?
A
The esophagus has a poor blood supply, which is segmental in distribution and accounts for the
high incidence of anastomotic leakage.

B The esophageal serosa consists of a thin layer of fibroareolar tissue.
C
The esophagus has two distinct muscle layers, an outer, longitudinal one and an inner, circular one,
which are striated in the upper third and smooth in the distal two thirds.
D
Injury to the recurrent laryngeal nerve results in vocal cord dysfunction but does not affect
swallowing.
ANSWER:C
1030
Which of the following statements about the lower esophageal sphincter (LES) mechanism, or high-
pressure zone (HPZ), is true?
A The LES is a circular smooth muscle ring that is 3 to 5 cm. long.
B
In assessing esophageal manometric data, mean HPZ pressure less than 6 mm. Hg or overall length
less than 2 cm. is more likely to be associated with incompetence of the LES and gastroesophageal
reflux.
C
Esophageal manometry and the acid perfusion (Bernstein) test reliably identify the patient with an
incompetent LES mechanism.
D Distal HPZ relaxation occurs within 5 to 8 seconds of initiating a swallow.
ANSWER:B
1031 Which of the following statements about achalasia is/are correct?
A In most cases in North America the cause is a parasitic infestation by Trypanosoma cruzi.
B Chest pain and regurgitation are the usual symptoms.
C
Distal-third esophageal adenocarcinomas may occur in as many as 20% of patients within 10 years
of diagnosis.
D
Manometry demonstrates failure of LES relaxation on swallowing and absent or weak simultaneous
contractions in the esophageal body after swallowing.
ANSWER:D
1032 Which of the following statements about epiphrenic diverticula of the esophagus is/are correct?
A They are traction diverticula that arise close to the tracheobronchial tree.
B They characteristically arise proximal to an esophageal reflux stricture.
C The degree of dysphagia correlates with the size of the pouch.
D
The operation of choice is a stapled diverticulectomy, long esophagomyotomy, and partial
fundoplication.
ANSWER:D
1033 Which of the following statements about Schatzki’s ring is correct?

A The ring represents a panmural fibrotic stricture resulting from gastroesophageal reflux.
B Dysphagia occurs when the ring diameter is 13 mm. or less.
C The ring occurs within 1 to 2 cm. of the squamocolumnar epithelial junction.
D Schatzki’s ring indicates reflux esophagitis.
ANSWER:B
1034 Which of the following statements about pathology encountered at esophagoscopy is/are correct?
A
Reflux esophagitis should be graded as mild, moderate, or severe, to promote consistency among
different observers.
B
An esophageal reflux stricture with a 2-mm. lumen is not dilatable and is best treated with resection.
C
A newly diagnosed radiographic distal esophageal stricture warrants dilation and antireflux medical
therapy.
D
In patients with Barrett’s mucosa, the squamocolumnar epithelial junction occurs 3 cm. or more
proximal to the anatomic esophagogastric junction.
ANSWER:D
1035
Which of the following statements about the diagnosis and treatment of esophageal leiomyomas
is/are correct?
A The majority are diagnosed after they cause dysphagia and chest pain.
B Biopsy is indicated at the time of esophagoscopy, to rule out carcinoma.
C Full-thickness elliptical excision of the esophageal wall is the preferred surgical approach.
D Endoscopic ultrasonography is a reliable means of following leiomyomas conservatively.
ANSWER:D
1036 Which of the following statements regarding the pathology of esophageal carcinoma is/are correct?
A Worldwide, adenocarcinoma is the most common esophageal malignancy.
B
Squamous cell carcinoma is most common in the distal esophagus, whereas adenocarcinoma
predominates in the middle third.
C
Patients with Barrett’s metaplasia are 40 times more likely than the general population to develop
adenocarcinoma.
D
Metastases from esophageal carcinoma are characteristically localized to regional mediastinal lymph
nodes adjacent to the tumor.
ANSWER:C

1037
Which of the following statements about the surgical treatment of esophageal carcinoma is/are
correct?
A
The finding of severe dysphagia in association with Barrett’s mucosa is an indication for an
antireflux operation to prevent subsequent development of carcinoma.
B
Long-term survival is improved by radical en bloc resection of the esophagus with its contained
tumor, adjacent mediastinal tissues, and regional lymph nodes.
C
The morbidity and mortality rates for cervical esophagogastric anastomotic leak are substantially less
than those associated with intrathoracic esophagogastric anastomotic leak.
D
The leading complications of transthoracic esophagectomy and intrathoracic esophagogastric
anastomosis are bleeding and wound infection.
ANSWER:C
1038 The best management for a 48-hour-old distal esophageal perforation is:
A Antibiotics and drainage.
B Division of the esophagus and exclusion of the perforation.
C Primary repair with buttressing.
D Resection with cervical esophagostomy, gastrostomy, and jejunostomy.
ANSWER:C
1039
A 50-year-old patient develops sudden left lower chest pain and epigastric pain after vomiting. The
patient shows diaphoresis, breath sounds are decreased on the left, and there is abdominal guarding.
The most appropriate diagnostic test is:
A Aortography.
B Esophagoscopy.
C Electrocardiogram.
D Film of the chest.
ANSWER:D
1040
The following statements about the influence of diet and lifestyle on lower esophageal sphincter
(LES) function are true except one. Identify the incorrect statement.
A A high-protein diet increases LES pressure.
B A fat meal results in sustained decrease in LES pressure.
C Chocolate ingestion causes a decrease in LES pressure.
D Cigarette smoking produces no significant changes in LES pressures.
ANSWER:D

1041
When a stricture is present in association with gastroesophageal reflux, each of the following is an
acceptable repair for reflux control except one. Identify the poorest repair.
A Intrathoracic total fundoplication.
B Lengthening gastroplasty with total fundoplication.
C Total fundoplication.
D Partial fundoplication.
ANSWER:D
1042
When assessing gastroesophageal reflux disease by manometry each of the following statements is
correct except one. Identify the incorrect one.
A Absent or extremely low LES pressures have predictive value in identifying more severe reflux.
B Peristaltic dysfunction increases with increasing severity of esophagitis.
C With established reflux disease the UES is hypertensive.
D
Esophageal functional changes are worst in patients with a circumferential columnar-lined
esophagus.
ANSWER:C
1043 The presence of a nonmalignant mid- or upper esophageal stricture always indicates the presence of:
A Alkaline reflux esophagitis.
B Barrett’s esophagus.
C Idiopathic reflux disease.
D Mediastinal fibrosis.
ANSWER:B
1044
Which of the following is most reliable for confirming the occurrence of a significant esophageal
caustic injury?
A History of the event.
B Physical examination of the patient.
C Barium esophagraphy.
D Endoscopy.
ANSWER:D
1045 First-line therapy for routine peptic duodenal ulcer disease includes:
A Vagotomy and antrectomy.
B Upper endoscopy and biopsy to rule out tumor.

C Evaluation for Helicobacter pylori.
D Serum gastrin determination.
ANSWER:C
1046
Appropriate management of severe vomiting associated with gastric outlet obstruction from peptic
ulcer disease includes all of the following except:
A Nasogastric suction.
B Intravenous hydration.
C Nutritional assessment; upper endoscopy to rule out malignancy.
D Oral antacid therapy.
ANSWER:D
1047 All of the following are complications of peptic ulcer surgery except:
A Duodenal stump blowout.
B Dumping.
C Diarrhea.
D Steatorrhea.
ANSWER:D
1048 The presentation of Zollinger-Ellison syndrome includes all of the following except:
A Hyperparathyroidism in patients with multiple endocrine neoplasia type 1 (MEN 1) syndrome.
B Diarrhea.
C Migratory rash.
D Jejunal ulcers.
ANSWER:C
1049 All are true about the dumping syndrome except:
A Symptoms can be controlled with a somatostatin analog.
B Diarrhea is always part of the dumping syndrome.
C Flushing and tachycardia are common features of the syndrome.
D
Separating solids and liquids in the patient’s oral intake alleviates some of the symptoms of the
syndrome.
ANSWER:B
1050
In patients with bleeding duodenal ulcers, the endoscopic finding associated with the highest
incidence of rebleeding is:
A Visible vessel.

B Cherry-red spot.
C Clean ulcer bed.
D Duodenitis.
ANSWER:A
1051 All of the following are contraindications for highly selective vagotomy except:
A Intractable duodenal ulcer disease.
B Peptic ulcer disease causing gastric outlet obstruction.
C Fundic peptic ulceration.
D Cigarette chain smoking.
ANSWER:A
1052 All of the following statements about gastrin-releasing peptide (GRP) are true except:
A In species other than man and dog GRP is commonly referred to as bombesin.
B GRP serves as a neurotransmitter.
C GRP inhibits pancreatic secretion when given intravenously.
D GRP stimulates gastric acid secretion when given intravenously.
ANSWER:C
1053 Cholecystokinin (CCK) is believed to function in all of the following processes except:
A It physiologically delays gastric emptying.
B It appears to have a role in satiety regulation.
C It contracts the gallbladder.
D It is important in the control of the anal sphincter.
ANSWER:D
1054
All of the following measures have been recommended for control of acid secretion in patients with
Zollinger-Ellison syndrome except:
A Antrectomy.
B Highly selective vagotomy.
C Total gastrectomy.
D Vagotomy and pyloroplasty.
ANSWER:A
1055 All of the following contribute to peptic ulcer disease except:
A Cigarette smoking.
B Helicobacter pylori.

C Gastrinoma.
D Spicy foods.
ANSWER:D
1056 The sine qua non of the histologic diagnosis of a gastric pseudolymphoma is:
A Extragastric extension of the gastric lesion.
B Nodal involvement beyond the immediate stomach.
C A germinal center in the gastric lesion.
D Extension into esophagus and duodenum.
ANSWER:C
1057 All of the following statements about surgical management of gastric lymphomas are true except:
A
Stage I gastric lymphomas (small lesions confined to the stomach wall) can be cured completely
with surgical therapy alone.
B
Extensive gastric lymphomas that initially are treated with radiation and/or chemotherapy
occasionally perforate during treatment and require secondary resection.
C
Patients explored with a presumptive diagnosis of gastric lymphoma should undergo an attempt at
curative resection when this is safe and feasible.
D
Without a preoperative diagnosis resection for gastric mass should not be attempted unless
lymphoma can be excluded.
ANSWER:D
1058
Numerous epidemiologic associations have been made between (1) environmental and dietary
factors and (2) the incidence of gastric cancer, including all except:
A Dietary nitrites.
B Dietary salt.
C Helicobacter pylori infection.
D Dietary ascorbic acid.
ANSWER:D
1059 All of the following benign conditions are associated with increased rates of gastric cancer except:
A Pernicious anemia.
B Multiple endocrine neoplasia type I (MEN 1).
C Adenomatous polyps.
D Chronic atrophic gastritis.
ANSWER:B

1060 Which of the following statements concerning the pathology of gastric cancer is true?
A Distal gastric cancers are becoming more common.
B
Intestinal-type gastric tumors resemble colon carcinomas and have a better prognosis than diffuse
type.
C Early gastric cancers are confined to the mucosa and are lymph node negative.
D
Broders’ histologic grading system correlates well with survival: patients with grade IV tumors have
5-year survival rates around 65%.
ANSWER:B
1061
An 80% distal gastrectomy is performed for a 6-cm. antral cancer with extension to the muscularis
propria and three positive lymph nodes less than 3 cm. from the tumor. The stage of this tumor was:
A Stage I.
B Stage II.
C Stage III A.
D Stage III B.
ANSWER:B
1062 Which of the following statements about the surgical treatment of gastric cancer is false?
A Patients with tumors of the middle and proximal thirds should undergo total gastrectomy.
B
Adenocarcinoma of the cardia-gastroesophageal junction may require reconstruction in the
abdomen, chest, or neck.
C Palliative resection yields better results than palliative bypass.
D
Japanese patients who undergo gastric resection are, on average, 10 years younger and much leaner
than their Western counterparts.
ANSWER:A
1063 Which of the following measures of obesity correlates best with mortality?
A The 1983 Metropolitan Life Insurance Company tables for ideal body weight.
B Hydroimmersion measurements of body fat composition.
C Body mass index (BMI).
D Skinfold thickness.
ANSWER:C
1064 Which of the following is/are contraindications to gastric bypass surgery?
A Diabetes mellitus.

B Hypertension.
C Pickwickian syndrome.
D Failure to agree to long-term follow-up.
ANSWER:D
1065
A 34-year-old morbidly obese diabetic woman underwent a gastric bypass about 12 hours ago. The
operation was technically difficult but finally went well. You are called because she now has a
temperature of 99.2? F, pulse of 134, and some pain in her incision and her back. She looks well;
the incision is clean; and her examination is otherwise negative. A bolus of 500 ml. of
dextrose/lactated Ringer’s did not change her vital signs, except that her pulse rose to 140 without
an increase in urine output. Your next step should be:
A Another bolus of crystalloids.
B Posterioanterior and lateral chest films.
C Obtain white cell count, differential count, and electrolyte values.
D Call the operating room and warn them that you need to re-explore for a leak.
ANSWER:D
106
Which of the following statement(s) is/are true concerning the diagnosis and management of the
patient whose barium esophogram is shown in Figure 18-29?
A
The condition is due to neuronal generation of the myenteric plexus in the lower esophageal
sphincter
B The patient will report symptoms of vomiting of sour or bitter material
C
Despite the impressive radiologic picture, passage of the endoscope through the area of narrowing
will likely be possible
D Manometry and 24 hour pH monitoring should be performed for confirmation of the diagnosis
ANSWER:C
1067
Which of the following statement(s) is/are true concerning the blood supply and lymphatic drainage
of the esophagus?
A
The thoracic esophagus receives no direct branches from the aorta therefore allowing the technique
of transhiatal (blunt) esophagectomy
B Bleeding esophageal varices are most prominent in the mid-esophagus
C
Lymphatic drainage of the lower third of the esophagus goes entirely to the abdominal lymphatic
system

D
Nodal involvement in esophageal cancer is quite common even if the tumor is limited to the level of
the submucosa
ANSWER:D
1068 Which of the following statement(s) is/are true concerning the surgical anatomy of the esophagus?
A Surgical exposure of the cervical esophagus is best gained via the right neck
B Spontaneous esophageal perforation tends to be associated with leakage into the left chest
C Access to the entire thoracic esophagus can be obtained only via the left chest
D
The lower esophageal sphincter can be recognized distinctly by inspection of the gastroesophageal
junction
ANSWER:B
1069 Which of the following statement(s) is/are true concerning infectious esophagitis?
A
Candida albicans is not normally found in the mouth but results from the overgrowth of this fungus
in patients on broad spectrum antibiotics
B Candida esophagitis is usually self-limited and is seldom associated with chronic problems
C Systemic therapy is seldom indicated
D
Small ulcers on barium esophogram in a transplant patient complaining of dysphagia and
odynophagia are likely due to herpes simplex viral infection
ANSWER:D
1070
Which of the following statement(s) is/are correct concerning the options for resection of esophageal
carcinoma?
A
The development of reflux esophagitis seldom occurs following intrathoracic resection due to the
limited life expectancy of these patients
B
Transhiatal esophagectomy, although conceptually sound, is not technically possible in most patients
with esophageal carcinoma
C
Transhiatal resection, although less morbid, has unfavorable survival statistics compared to
transthoracic resection
D
Radical transthoracic esophagectomy with en bloc dissection of continuous lymph node bearing
tissues has not been shown to improve survival over transhiatal esophagectomy
ANSWER:D

1071
The incidence of adenocarcinoma of the esophagus is increasing at a very rapid rate, which is
largely the result of the growing prevalence of adenocarcinoma arising in Barrett’s mucosa. Which
of the following statement(s) is/are true concerning adenocarcinoma of the esophagus?
A
Barrett’s mucosa with specialized columnar epithelium characterized by veliform folds, lined by
secreting columnar and goblet cells has the highest association with carcinoma of the esophagus
B Less than 5% of patients with Barrett’s mucosa will harbor adenocarcinoma
C Severe dysplasia of Barrett’s mucosa requires frequent reexamination and biopsy
D Adenocarcinoma of the esophagus has a less aggressive behavior than squamous cell carcinoma
ANSWER:A
1072
Benign tumors of the esophagus are rare constituting less than 1% of esophageal neoplasms. Which
of the following statement(s) is/are true concerning benign esophageal neoplasms?
A Most esophageal polyps are located just above the gastroesophageal junction
B Malignant degeneration of leiomyomas of the esophagus is a frequent occurrence
C
An asymptomatic leiomyoma can be safely observed and followed with periodic barium
esophograms and endoscopic ultrasonography
D Most leiomyomas of the esophagus require esophagectomy
ANSWER:C
1073 Important stimulants of gastrin release from endocrine cells in the antrum include:
A Acidification of the antral lumen
B Small peptide fragments and amino acids from luminal proteolysis
C Locally released somatostatin
D Dietary fats
ANSWER:B
1074 Which of the following statements regarding intrinsic factor is/are correct?
A Intrinsic factor is produced in chief cells located in the gastric fundus
B
B Total gastrectomy is followed by folate deficiency due to vitamin malabsorption secondary to
intrinsic factor deficiency
C Intrinsic factor secretion, like that of acid, is stimulated by gastrin, histamine, and acetylcholine
D Intrinsic factor deficiency accompanies H pylori-caused antral gastritis
ANSWER:C

1075
A 24-year-old woman develops epigastric pain and has a diagnosis of duodenal ulcer confirmed by
esophagogastroduodenoscopy. The patient is in the third month of a pregnancy. The most
appropriate treatment would be:
A Proximal gastric vagotomy
B Misoprostol 400 mg b.i.d.
C Sucralfate 1 gm q.i.d.
D Cimetidine 400 mg b.i.d.
ANSWER:C
1076
Development of duodenal ulceration is dependent upon gastric acid secretion. Which of the
following statements correctly characterizes acid secretion in duodenal ulcer patients?
A Groups of duodenal ulcer patients demonstrate decreased basal acid secretion
B
Maximal acid output to histamine averages 40 mEq/h in duodenal ulcer patients, twice that of normal
C Tissue gastrin levels, on average, are twice normal in patients with active ulceration
D
Exogenously administered somatostatin is ineffective in suppressing acid secretion in patients with
active ulceration
ANSWER:B
1077
Which of the following statement(s) is/are correct with regard to pyloric obstruction secondary to
peptic ulceration?
A Pyloric obstruction is suggested by hypochloremic hyponatremic alkalosis
B Pyloric obstruction is suggested by hypochloremic hypokalemic alkalosis
C
Approximately 80% of patients with benign gastric outlet obstruction obtain permanent relief of
symptoms by endoscopically-directed balloon dilatation
D The lifetime risk of pyloric obstruction in peptic ulcer patients is 40%
ANSWER:B
1078
A 42-year-old man with a recently diagnosed duodenal ulcer develops melena and near-syncope.
After fluid resuscitation, upper gastrointestinal endoscopy is performed. During the examination, a 1
cm ulcer is noted in the proximal duodenum. A fresh clot is observed within the ulcer and blood is
noted to be oozing around the clot. Optimal therapy would consist of which of the following?
A Angiographic embolization of the gastroduodenal artery
B Irrigation of the clot followed by endoscopic application of a heat probe

C Transfusion and intravenous cimetidine
D Angiographic infusion of vasopressin into the gastroduodenal artery
ANSWER:B
1079
A 50-year-old patient has undergone truncal vagotomy and antrectomy with Billroth II
reconstruction two years ago. The patient now complains of recurrent postprandial pain, nausea, and
vomiting. Endoscopic examination reveals bile in the stomach; endoscopic biopsies demonstrate
histologic evidence of moderately severe gastritis. No other endoscopic abnormalities are noted.
Appropriate therapy could include:
A Octreotide administration
B Total gastrectomy
C Conversion of Billroth II gastrojejunostomy to Billroth I gastroduodenostomy
D Conversion of Billroth II gastrojejunostomy to Roux-en-Y gastrojejunostomy
ANSWER:D
1080
A 50-year-old male with a 2 year history of duodenal ulceration develops sudden, severe epigastric
pain 4 hours prior to evaluation. Physical examination reveals T 101° F, pulse 80, BP 125/90,
diminished bowel sounds, and abdominal muscular rigidity. An upright chest x-ray reveals
pneumoperitoneum. At laparotomy, an anterior perforation in the first portion of the duodenum is
observed. Optimal treatment would include:
A Omental patch of the perforation followed by truncal vagotomy and antrectomy after 8 weeks
B Omental patch of the perforation followed by truncal vagotomy and pyloroplasty after 8 weeks
C Omental patch of the perforation followed by chronic cimetidine administration
D Omental patch of the perforation plus proximal gastric vagotomy
ANSWER:D
1081
With regard to benign gastric ulceration, the most common location of disease is which of the
following?
A Along the greater curvature
B Immediately distal to the esophagogastric junction along the lesser curvature
C In the area of the incisura angularis along the lesser curvature
D Within the gastric antrum
ANSWER:C

1082
A 35-year-old smoker is involved in a house fire and receives a 45% total surface area burn. One
half of the burned surface appears to be third degree. On the third post-burn day, the patient is
noted to have bloody drainage from a nasogastric tube and a decrease of 5% in his hematocrit.
Appropriate management should include which of the following?
A Urgent upper gastrointestinal contrast study to delineate site of bleeding
B
Immediate selective arteriography via the left gastric artery to diagnose and treat presumed stress
ulceration
C Urgent esophagogastroduodenoscopy to diagnosis the cause of bleeding
D Urgent intravenous infusion of vasopressin at 0.2–0.4 IU/min
ANSWER:C
1083
A 45-year-old male presents with symptoms of epigastric pain, worsened with ingestion of food.
Physical examination is normal. Upper abdominal ultrasonography is unremarkable. Contrast
radiography reveals a 2 cm ulcer in the gastric fundus along the lesser curvature. Therapy with
omeprazole 20 mg per day is begun but symptoms persist 3 weeks later. Appropriate management
includes which of the following?
A Increase in omeprazole dose to 40 mg per day
B Addition of sucralfate 1 gm every 8 hours
C Addition of cimetidine 200 mg b.i.d.
D Esophagogastroduodenoscopy with biopsy of ulceration
ANSWER:D
1084
A 52-year-old woman is hospitalized with acute upper gastrointestinal hemorrhage. Endoscopic
examination reveal a 2.5 cm ulcer in the area of the incisura angularis. The remainder of the
endoscopic examination is normal. Continued bleeding requires operative therapy. Optimal therapy
consists of which of the following?
A Gastrotomy with oversewing of the bleeding site
B Distal gastrectomy including the area of ulceration
C Proximal gastric vagotomy and oversewing of the bleeding ulcer
D Truncal vagotomy, pyloroplasty, and oversewing of the bleeding ulcer
ANSWER:B

1085
A 25-year-old man is involved in an automobile accident with resultant injuries including bilateral
closed femur fractures, left pulmonary contusion, and closed head injury. On post-injury day 4,
significant upper gastrointestinal hemorrhage begins. Endoscopic examination reveals an area of
confluent ulceration with bleeding in the gastric fundus. Endoscopic hemostasis fails. Appropriate
immediate management includes:
A Lavage of gastric contents with iced saline
B Urgent total gastrectomy
C Selective arterial infusion of vasopressin via the left gastric artery
D Insertion of Sangstaken-Blakemore balloon
ANSWER:C
1086 Which of the following statements is correct with regard to gastric bypass for obesity?
A
Rapid weight loss following successful gastric bypass for obesity is associated with an increased risk
of developing cholelithiasis
B Marginal ulcer develops in 25% of gastric bypass patients
C
Vitamin B12 deficiency is a potential complication of gastric bypass due to gastric mucosal atrophy
D Anastamotic leak after gastric bypass is often heralded by bradycardia
ANSWER:A
1087
A patient with gastric adenocarcinoma undergoes subtotal gastrectomy. Pathological examination
reveals that the tumor penetrates to the serosa. Regional lymph nodes are not involved. Distant
metastases are not detected. What is the correct tumor stage and 5-year survival rate?
A Stage I: 90% 5-year survival
B Stage II: 45% 5-year survival
C Stage III: 15% 5-year survival
D Stage II: 15% 5-year survival
ANSWER:B
1088
A 55-year-old male is evaluated because of symptoms of epigastric pain and anorexia. Physical
examination is normal except for guaiac positivity of stool. Upper endoscopic examination reveals a
1.5 cm ulcer along the lesser curvature of the stomach proximal to the incisura angularis. Optimal
management consists of which of the following:
A Sucralfate 1 gm q.i.d. for 8 weeks
B Endoscopic biopsy of the ulcer rim

C Endoscopic cautery of the ulcer base
D Endoscopic biopsy of the ulcer base
ANSWER:B
1089 All of the following statements about the embryology of Meckel’s diverticulum are true except:
A Meckel’s diverticulum usually arises from the ileum within 90 cm. of the ileocecal valve.
B Meckel’s diverticulum results from the failure of the vitelline duct to obliterate.
C The incidence of Meckel’s diverticulum in the general population is 5%.
D Meckel’s diverticulum is a true diverticulum possessing all layers of the intestinal wall.
ANSWER:C
1090 Meckel’s diverticulum most commonly presents as:
A Gastrointestinal bleeding.
B Obstruction.
C Diverticulitis.
D Intermittent abdominal pain.
ANSWER:A
1091
Which of the following physical factors of irradiation is/are related to the potential for radiation
injury?
A The dimension of the radiation portals.
B The number of portals.
C The number of fractions.
D All of the above
ANSWER:D
1092
For which of the following consequences of radiation injury of the intestine is urgent laparotomy
required?
A Small bowel obstruction.
B Colonic perforation.
C Rectovaginal fistula.
D Malabsorption and diarrhea.
ANSWER:B

1093
In addition to its absorptive and digestive roles, the small bowel also plays a significant role in the
body’s immune system. Gut-associated lymphoid tissue (GALT) represents a major division of the
immune system. Which of the following statement(s) is/are true concerning the immunologic
functions of the small intestine?
A The B lymphocytes of the small intestine do not produce immunoglobulin A (IgA)
B
Peyer’s patches, an example of an aggregated cellular portion of the gut-associated lymphoid system
tissue, are large collections of lymphoid follicles found on the antimesenteric border of the ileum
C The major immunoglobulin of the intestinal immune system is IgM
D
IgA produced by the intestinal immune system produces the classic Fc-mediated inflammatory
reactions to antigen stimulus
ANSWER:B
1094
A 45-year-old man with a history of previous right hemicolectomy for colon cancer presents with
colicky abdominal pain which has become constant over the last few hours. He has marked
abdominal distension and has had only minimal vomiting of a feculent material. His abdomen is
diffusely tender. Abdominal x-ray shows multiple air fluid levels with dilatation of some loops to
greater than 3 cm in diameter. The most likely diagnosis is:
A Proximal small bowel obstruction
B Distal small bowel obstruction
C Acute appendicitis
D Closed-loop small bowel obstruction
ANSWER:B
1095
Which of the following statement(s) is/are true concerning laboratory tests which might be obtained
in the patient discussed above?
A
The presence of a white blood cell count > 15,000 would be highly suggestive of a closed-loop
obstruction
B Metabolic acidosis mandates emergency exploration
C An elevation of BUN would suggest underlying renal dysfunction
D There is no rapidly available test to distinguish tissue necrosis from simple bowel obstruction
ANSWER:D

1096
A 75-year-old woman is hospitalized after a fall in which she has experienced a hip fracture. Several
days after her surgical procedure, progressive painless abdominal distension is noted. Which of the
following statement(s) is/are true concerning her diagnosis and management?
A
Colon distension with a cecal diameter in excess of 12 cm should indicate the need for urgent
operation
B Endoscopic decompression may be attempted but seldom is successful
C After successful colonoscopic decompression, recurrence is unlikely
D A rectal tube as the primary treatment is generally not successful
ANSWER:D
1097 Which of the following statement(s) is/are true concerning postoperative ileus?
A
The use of intravenous patient-controlled analgesia has no effect on return of small bowel motor
activity
B
The presence of peritonitis at the time of the original operation delays the return of normal bowel
function
C The routine use of metoclopramide will hasten the return of small intestinal motor activity
D
Contrast radiographic studies have no role in distinguishing early postoperative bowel obstruction
from normal ileus
ANSWER:B
1098 The following statement(s) is/are true concerning the surgical management of Crohn’s disease.
A
Strictureplasty, although offering short-term benefits, is associated with a higher rate of recurrence
when compared to resection
B
Frozen section examination of the margin of resection is essential to prevent both recurrent disease
and early anastomotic complications
C
Conservative margins of resection are appropriate, resecting only grossly involved segments of
bowel
D
Patients with Crohn’s disease confined to the colon may be treated with total proctocolectomy with
construction of an ileal-anal pouch anastomosis
ANSWER:C
1099 The following statement(s) is/are true concerning the epidemiology of Crohn’s disease.
A
Crohn’s disease has an age distribution with peaks between the ages of 15 and 30 years and 65 and
75 years
B There is a definite female predilection for Crohn’s disease

C The disease is equally prevalent in industrialized versus underdeveloped countries
D
First and second generation relatives with Crohn’s disease have an increased prevalence when
compared to the general population
ANSWER:D
1100
An increased evidence of adenocarcinoma of the small intestine has been established with which of
the following conditions?
A Peutz-Jegher Syndrome
B Crohn’s disease
C Simple tubular adenomas of the small intestine
D Colon carcinoma
ANSWER:B
1101 Crohn’s disease:
A Is caused by Mycobacterium paratuberculosis.
B Is more common in Asians than in Jews.
C Tends to occur in families.
D Is less frequent in temperate climates than in tropical ones.
ANSWER:C
1102 Recurrence after operation for Crohn’s disease:
A Occurs after operations for ileal Crohn’s but not colonic Crohn’s.
B Is usually found just proximal to an enteric anastomosis.
C Rarely requires reoperation.
D Occurs in 1% of patients at risk per year during the first 10 years after the operation.
ANSWER:B
1103 Excision rather than bypass is preferred for surgical treatment of small intestinal Crohn’s because:
A Excision is safer.
B Bypass does not relieve symptoms.
C Excision cures the patient of Crohn’s disease but bypass does not.
D The risk of small intestine cancer is reduced.
ANSWER:D
1104 The most common indication for operation in Crohn’s disease of the colon is:
A Obstruction.
B Chronic debility.

C Bleeding.
D Perforation.
ANSWER:B
1105
Which of the following patients generally does not require surgical intervention as a consequence of
acute diverticulitis?
A A 35-year-old man with no history of diverticulitis.
B A 68-year-old man status 2 weeks post–renal transplantation.
C A 55-year-old woman with hypertension and diabetes mellitus.
D A 50-year-old man with pneumaturia.
ANSWER:C
1106 The test with the highest diagnostic yield for detecting a colovesical fistula is:
A Barium enema.
B Colonoscopy.
C Computed tomography (CT).
D Cystoscopy.
ANSWER:D
1107 Which of the following is not true of diverticular disease:
A It is more common in the United States and Western Europe than in Asia and Africa.
B A low-fiber diet may predispose to development of diverticulosis.
C It involves sigmoid colon in more than 90% of patients.
D Sixty per cent develop diverticulitis sometime during their lifetime.
ANSWER:D
1108 The most common indication for surgery secondary to acute diverticulitis is:
A Abscess.
B Colonic obstruction.
C Colovesical fistula.
D Free perforation.
ANSWER:A
1109 Which of the following statements about familial adenomatous polyposis (FAP) is/are true?
A
Inherited in an autosomal-dominant manner, this genetic defect is of variable penetrance, some
patients having only a few polyps whereas others develop thousands.
B The phenotypic expression of the disease depends mostly on the genotype.

C
Appropriate surgical therapy includes total abdominal colectomy with ileorectal anastomosis and
ileoanal pull-through with rectal mucosectomy.
D Panproctocolectomy with ileostomy is not appropriate therapy for this disease.
ANSWER:C
1110 Surgical alternatives for the treatment of ulcerative colitis include all of the following except:
A Colectomy with ileal pouch–anal anastomosis.
B Left colectomy with colorectal anastomosis.
C Proctocolectomy with Brooke ileostomy or continent ileostomy.
D Subtotal colectomy with ileostomy and Hartmann closure of the rectum.
ANSWER:B
1111 Which finding(s) suggest(s) the diagnosis of chronic ulcerative colitis as opposed to Crohn’s colitis?
A Endoscopic evidence of backwash ileitis.
B Granulomas on biopsy.
C Anal fistula.
D Rectal sparing.
ANSWER:A
1112
An 80-year-old man who has been bedridden for many years following a stroke presents with acute
onset of abdominal distention, obstipation, and colicky abdominal pain. Abdominal x-rays reveal
dilated loops of small bowel and a dilated sigmoid colon resembling a bent inner tube. Examination
reveals distention with mild direct tenderness but no rigidity or rebound tenderness. Initial
management should consist of:
A Barium enema examination.
B Laparotomy with resection of descending colon and descending colostomy.
C Multiple cleansing enemas to remove impacted feces.
D Rigid sigmoidoscopy and decompression of the sigmoid colon.
ANSWER:D
1113
Axial twisting of the right colon or cecal volvulus has been shown to be associated with each of the
following except:
A A history of abdominal operation.
B A mobile cecum.
C An obstructing lesion in the transverse or left colon.

D Inflammatory bowel disease.
ANSWER:D
1114 Sigmoid volvulus has been associated with each of the following except:
A Chronic constipation and laxative abuse.
B Chronic rectal proplapse.
C Chronic traumatic paralysis.
D Medical management of Parkinson’s disease.
ANSWER:B
1115 Which of the following statements is not true about inhereted susceptibility to colon cancer?
A There is no known genetic susceptibility to colon cancer.
B
There are known genetic susceptibilities to colon cancer, but they are always associated with
multiple adenomatous polyps.
C
There are known genetic susceptibilities to colon cancer, but they are always associated with specific
ethnic or racial groups.
D None of the above.
ANSWER:D
1116
Which of the following recommendations for adjuvant chemotherapy of colorectal carcinoma are
true?
A
Patients with Stage I or Dukes A and B1 disease should receive adjuvant treatment for 1 year with
levamisole combined with 5-FU.
B
Patients with Stage III or Dukes C disease should receive adjuvant treatment for 1 year with
levamisole combined with 5-FU.
C There is no role for adjuvant therapy for colon cancer at any stage.
D Adjuvant chemotherapy is active in colon cancer only when combined with radiotherapy.
ANSWER:B
1117 Optimal front-line treatment of squamous cell carcinoma of the rectum includes:
A Abdominal perineal resection.
B Low anterior resection when technically feasible.
C Radiation therapy.
D Combined radiation and chemotherapy.
ANSWER:D
1118 Which of the following statement(s) about complete rectal prolapse, or procidentia is/are true?

A Rectal prolapse results from intussusception of the rectum and rectosigmoid.
B The disorder is more common in men than in women.
C Continence nearly always is recovered after correction of the prolapse.
D All of the above
ANSWER:A
1119 Which of the following statements about hemorrhoids is/are not true?
A Hemorrhoids are specialized “cushions” present in everyone that aid continence.
B External hemorrhoids are covered by skin whereas internal hemorrhoids are covered by mucosa.
C Pain is often associated with uncomplicated hemorrhoids.
D Hemorrhoidectomy is reserved for third- and fourth-degree hemorrhoids.
ANSWER:C
1120
The widely accepted treatment of most localized epidermoid, cloacogenic, or transitional cell
carcinoma of the anal canal is:
A Surgical resection.
B Chemotherapy alone.
C Radiotherapy alone.
D Combined chemoradiation.
ANSWER:D
1121 Which statement(s) is/are true about hidradenitis suppurativa?
A It is a disease of the apocrine sweat glands.
B It causes multiple perianal and perineal sinuses that drain watery pus.
C The sinuses do not communicate with the dentate line.
D All of the above
ANSWER:D
1122 How much of the daily insensible water loss is due to loss in stool?
A 200 ml
B 400 ml
C 600 ml
D 800 ml
ANSWER:A

1123
A 52-year-old woman is involved in an automobile accident and sustains an open fracture of the
fight femur, compression fractures of the 10th and 11th thoracic vertebrae and right pulmonary
contusion. On the fourth day after injury, her abdomen is noted to be distended, tympanitic and
diffusely tender. Abdominal radiographs reveal gaseous distension of the ascending and transverse
segments of the colon. The cecum is 13 cm in greatest diameter. Appropriate management includes
which of the following as the next step?
A Right hemicolectomy
B Operative cecostomy
C Colonoscopy
D Contrast enema
ANSWER:C
1124 Which of the following features is/are consistent with a diagnosis of colonic inertia?
A Alternating episodes of severe constipation and normal bowel activity
B Total bowel transit time of 24 hours
C Total bowel transit time of 48 hours
D Total bowel transit time of 96 hours
ANSWER:D
1125
Which of the following statements regarding the risk of cancer in the context of ulcerative colitis
is/are correct?
A After 10 years of active disease, the risk of cancer approximates 20% to 30%
B After 10 years of active disease, the risk of cancer approximates 2% to 3%
C The risk of colon cancer in ulcerative colitis is identical to controls
D After 20 years of disease activity, the risk of colon cancer approximates 80%
ANSWER:B
1126
A 19-year-old male is seen in consultation with complaints of bloody diarrhea (10 bowel
movements per day), and weight loss (10 pounds). Physical examination reveals the presence of two
circular, 4 cm erythematous lesions on the trunk. Each lesion has an area of necrosis in the center.
The abdominal examination reveals mild hypogastric tenderness. The stool is guaiac positive. The
most appropriate next diagnostic step includes which of the following?
A Barium enema
B Flexible sigmoidoscopy
C Liver biopsy

D Chest x-ray
ANSWER:B
1127
Many patients with ulcerative colitis are operated upon electively with total abdominal colectomy,
rectal mucosectomy, formation of a small intestinal reservoir, and ileoanal anastomosis. The most
common postoperative complication after this operation is which of the following?
A Enterocutaneous fistula
B Small bowel obstruction
C Pulmonary embolism
D Urinary retention
ANSWER:B
1128
A 25-year-old woman with known ulcerative colitis presents to the emergency room with a 24-hour
history of abdominal pain, distention, and obstipation. Physical examination reveals a temperature
of 38.6° C, abdominal distention, and diffuse abdominal tenderness. Abdominal x-rays show
marked colonic dilatation, most pronounced in the transverse colon. Laboratory examination reveals
a white blood count of 19,000/mm3. Over the first 24 hours of hospitalization, symptoms are
progressive in spite of intravenous fluid resuscitation, nasogastric suctioning, and intravenous
antibiotics. The most appropriate management for this patient would include which of the following?
A Decompressive colonoscopy
B Proctocolectomy with formation of end ileostomy
C Total abdominal colectomy with formation of Hartmann pouch and end ileostomy
D Cecostomy
ANSWER:C
1129
One year following ileal pouch-anal anastomosis, the mean 24-hour stool frequency is which of the
following?
A Two to three
B Five to six
C Eight to nine
D Eleven to twelve
ANSWER:B

1130
A 30-year-old male two years postoperative after total abdominal colectomy with ileoanal
anastomosis reports a sudden increase in stool frequency, nocturnal leakage, and low-grade fevers.
Physical examination is unremarkable. Flexible endoscopic examination of the small intestinal
pouch reveals a friable erythematous mucosa. Biopsies of the mucosa are obtained. While awaiting
biopsy results, which of the following is the most appropriate empiric therapy?
A Oral corticosteroids
B Oral vancomycin
C Oral metronidazole
D Corticosteroid enema
ANSWER:C
1131
For the patient in the preceding question, after obtaining diagnostic samples, the most appropriate
management would include which of the following?
A Oral metronidazole
B Intravenous metronidazole
C Oral vancomycin
D Intravenous vancomycin
ANSWER:C
1132
A 72-year-old woman undergoes anterior resection for a rectal cancer located 7 cm proximal to the
anal verge. Pathologic examination of the resected specimen reveals invasion of the tumor into the
muscularis propria. Five of 8 lymph nodes contain microscopic tumor. There is no evidence of
disseminated disease. Appropriate subsequent management includes which of the following?
A Postoperative radiation plus intravenous 5FU
B Postoperative radiation alone
C Observation
D Postoperative radiation plus intravenous adriamycin
ANSWER:A
1133
The most common oncogene abnormality observed in association with colorectal cancer is which of
the following?
A Overexpression of the N-myc oncogene
B Amplification of the K-ras oncogene
C Suppression of the erbB oncogene

D Amplification of the L-myc oncogene
ANSWER:B
1134
Which of the following types of colonic polyps is associated with the highest incidence of malignant
degeneration?
A Tubular adenoma
B Tubulovillous adenoma
C Villous adenoma
D Hamartomatous polyp
ANSWER:C
1135
A 52-year-old man undergoes a right hemicolectomy for a carcinoma of the ascending colon.
Pathological examination of the resected specimen reveals invasion of the tumor to the level of the
muscularis propria. Three of 17 lymph nodes contain microscopic tumor. What is the correct Dukes
classification (Aster-Coller modification) and associated 5-year survival for this lesion?
A Dukes C2, 45% 5-year survival
B Dukes B1, 75% 5-year survival
C Dukes C1, 45% 5-year survival
D Dukes B3, 65% 5-year survival
ANSWER:C
1136
An pedunculated polyp, discovered incidentally at colonoscopy, is removed by snare polypectomy
from the ascending colon. Invasive cancer to the level of the submucosa is identified histologically
within the polyp. The lesion is well-differentiated. No lymphatic or vascular invasion is noted. The
cauterized margin is negative for neoplasm. Appropriate subsequent management includes which of
the following?
A Repeat endoscopy at 6 months
B Right hemicolectomy
C Subtotal colectomy
D Repeat endoscopy with fulguration of the polypectomy site
ANSWER:A

1137
A 58-year-old male undergoes resection of a Dukes C2 colon cancer via right hemicolectomy.
Three years postoperatively, rising CEA levels prompt evaluation including abdominal computed
tomography. Two lesions, each measuring 2 cm, are noted in the right hepatic lobe. No other
abnormalities are noted. A right hepatic lobectomy is performed without complication. Which of the
following most closely approximates anticipated 5-year survival?
A 85–90%
B 65–70%
C 45–50%
D 25–30%
ANSWER:D
1138
An asymptomatic 52-year-old man is undergoing screening sigmoidoscopy. In the rectum, at 6 cm
from the anal verge, a 2 cm yellow, submucosal nodule is noted. Deep endoscopic biopsies are
consistent with carcinoid. Appropriate management includes which of the following?
A Observation
B Transanal excision
C Low anterior resection
D Abdominoperineal resection
ANSWER:B
1139
A 72-year-old woman complains of anal itching and burning. Physical examination reveals an
erythematous, scaly lesion, 3 cm in circumference, within the anal canal. The intersphincteric
groove can not be appreciated in the area of the lesion. The remainder of the physical examination
is normal. Appropriate initial management includes which of the following?
A Acyclovir 200 mg QID for 10 days
B Hydrocortisone cream 0.1% topically for 14 days
C Incisional biopsy
D Metronidazole 250 mg PO QID for 14 days
ANSWER:C
1140
For the patient in the preceding question, biopsy revealed an invasive apocrine gland neoplasm. The
deep margins included striated muscle infiltrated by neoplastic cells. Appropriate management
includes which of the following?
A Primary radiation
B Abdominoperineal resection with bilateral inguinal lymph node dissection

C Abdominoperineal resection only
D Carbon dioxide laser fulguration
ANSWER:C
1141
A 43-year-old woman presents with complaints of anal pain and spotting of blood with defecation.
Physical examination reveals a 2 3 cm area of ulceration within the anal canal. The remainder of the
physical examination is normal. Incisional biopsy is positive for squamous cell carcinoma.
Appropriate management includes which of the following?
A Abdominoperineal resection
B Wide local excision, skin grafting, proximal diverting colostomy
C Primary radiation therapy
D Local excision and primary closure
ANSWER:C
1142
Recurrent episodes of sigmoid colonic diverticulitis prompt operative therapy. Which of the
following describe the appropriate margins for resection?
A Proximal margin, splenic flexure; distal margin, rectosigmoid junction
B Proximal margin, descending colon; distal margin, rectosigmoid junction
C Proximal margin, descending colon; distal margin, mid-rectum
D Proximal margin, transverse colon; distal margin, mid-rectum
ANSWER:B
1143
An elderly man presents with complaints that he is passing gas with urination. The past medical
history is positive for one episode of diverticulitis, treated medically, transurethral resection of the
prostate for benign prostatic hypertrophy, and diabetes. Which of the following diagnostic tests is
most appropriate initially?
A Computed tomography of the abdomen and pelvis
B Cystoscopy
C Barium enema
D Intravenous pyelography
ANSWER:A
1144
For the patient in the preceding question, a colovesical fistula originating from the sigmoid colon is
demonstrated. Colonoscopy reveals diverticula and excludes carcinoma. During laparotomy, a
thickened sigmoid colon is found to be adherent to the dome of the bladder. A definite fistula is not
observed. Appropriate operative management includes which of the following?

A Sigmoid resection, primary colonic anastomosis, catheter drainage of bladder
B
En-bloc resection of sigmoid colon and adjacent bladder wall, primary colonic anastomosis,
suprapubic cystostomy
C
En-bloc resection of sigmoid colon and adjacent bladder wall, formation of descending colostomy
and Hartmann’s pouch, suprapubic cystostomy
D Sigmoid resection, primary colonic anastomosis, bilateral percutaneous nephrostomies
ANSWER:A
1145
A 65-year-old woman develops obstipation, lower abdominal pain, and fever. Physical examination
reveals a temperature of 38.5°C, left lower quadrant tenderness, and an ill-defined lower abdominal
mass. White blood count is 17,500 per mm3. Intravenous hydration, broad spectrum antibiotics, and
analgesics are ordered. After 48 hours, symptoms have not improved. Appropriate management
includes which of the following?
A Barium enema
B Computed tomography of the abdomen
C Immediate laparotomy
D Intravenous pyelogram
ANSWER:B
1146 The most common complication after hemorrhoidectomy is which of the following?
A Urinary retention
B Rectal bleeding
C Incontinence
D Wound infection
ANSWER:A
1147
A 65-year-old man presents with complaints of mucous discharge and perianal discomfort. Physical
examination reveals a fistulous opening lateral to the anus. Anoscopic examination permits passage
of a probe through the fistula tract. The fistula traverses the internal anal sphincter, the
intersphincteric plane, and a portion of the external anal sphincter. The fistula is categorized as
which type?
A Intersphincteric
B Transsphincteric
C Suprasphincteric
D Extrasphincteric

ANSWER:B
1148 For the patient in the preceding question, appropriate management includes which of the following?
A
Division of the tissues over the probe with electrocautery, leaving the wound open to heal by
secondary intention
B
Division of the tissues over the probe with electrocautery, closing the wound using a pedicled skin
flap
C
Division of the internal anal sphincter using electrocautery, encircling the external sphincter with a
seton
D Proximal diverting colostomy and antibiotics
ANSWER:C
1149 Which of the following statements about the segmental anatomy of the liver are not true?
A Segments are subdivisions in both the French and American systems.
B Segments are determined primarily by the hepatic venous drainage.
C
The French anatomic system is more applicable than the American system to clinical hepatic
resection.
D Segments are important to the understanding of the topographic anatomy of the liver.
ANSWER:D
1150
Which of the following statements most accurately describes the current therapy for pyogenic
hepatic abscess?
A Antibiotics alone are adequate for the treatment of most cases.
B All patients require open surgical drainage for optimal management.
C Optimal treatment involves treatment of not only the abscess but the underlying source as well.
D Percutaneous drainage is more successful for multiple lesions than for solitary ones.
ANSWER:C
1151 Which of the following statement(s) is/are true about benign lesions of the liver?
A
Adenomas are true neoplasms with a predisposition for complications and should usually be resected.
B
Focal nodular hyperplasia (FNH) is a neoplasm related to birth control pills (BCPs) and usually
requires resection.
C
Hemangiomas are the most common benign lesions of the liver that come to the surgeon’s attention.

D Nodular regenerative hyperplasia does not usually accompany cirrhosis.
ANSWER:A
1152 Which of the following statement(s) is/are true about bile duct cancers?
A If resected, proximal lesions are usually curable.
B The more proximal the lesion, the more likely is resection to be curative.
C Radiation clearly prolongs survival.
D None of the above.
ANSWER:D
1153 Which of the following statements about hemobilia are true?
A Tumors are the most common cause.
B The primary treatment of severe hemobilia is an operation.
C Percutaneous cholangiographic hemobilia is usually minor.
D Ultrasonography usually reveals a specific diagnosis.
ANSWER:C
1154
Ligation of all of the following arteries usually causes significant hepatic enzyme abnormalities
except:
A Ligation of the right hepatic artery.
B Ligation of the left hepatic artery.
C Ligation of the hepatic artery distal to the gastroduodenal branch.
D Ligation of the hepatic artery proximal to the gastroduodenal artery.
ANSWER:D
1155
Which of the following is the most common acid-base disturbance in patients with cirrhosis and
portal hypertension?
A Metabolic acidosis.
B Respiratory alkalosis.
C Metabolic alkalosis.
D Respiratory acidosis.
ANSWER:C
1156
Which of the following is the most effective definitive therapy for both prevention of recurrent
variceal hemorrhage and control of ascites?
A Endoscopic sclerotherapy.
B Distal splenorenal shunt.

C Esophagogastric devascularization (Sugiura procedure).
D Side-to-side portacaval shunt.
ANSWER:D
1157 Which of the following treatments most effectively preserves hepatic portal perfusion?
A Distal splenorenal shunt.
B Conventional splenorenal shunt.
C Endoscopic sclerotherapy.
D Side-to-side portacaval shunt.
ANSWER:C
1158
Which of the following veins is preserved in performing the extensive esophagogastric
devascularization procedure described by Sugiura?
A Left gastric (coronary) vein.
B Short gastric vein.
C Splenic vein.
D Left gastroepiploic vein.
ANSWER:A
1159
Which of the following complications of portal hypertension often require surgical intervention (for
more than 25% of patients)?
A Hypersplenism.
B Variceal hemorrhage.
C Ascites.
D Encephalopathy.
ANSWER:B
1160 Which of the following clinical situations are considered good indications for PVS?
A
A 50-year-old cirrhotic man had an emergency portacaval shunt for bleeding varices and
postoperatively had an ascites leak and mild superficial wound infection.
B
A 57-year-old woman with primary biliary cirrhosis (PBC) has difficult to control ascites and
diuretic-induced encephalopathy.
C
A 46-year-old resistant alcoholic has chronic ascites uncontrolled by diuretics combined with repeat
paracentesis.
D
A 34-year-old woman taking BCPs had rapid onset of ascites and is found to have hepatic vein
thrombosis causing the Budd-Chiari syndrome.

ANSWER:C
1161 Which of the following statements about biliary tract problems are correct?
A Choledochal cyst should be treated by Roux-en-Y cystojejunostomy.
B
Sclerosing cholangitis is characterized by long, narrow strictures in the extrahepatic biliary duct
system.
C
Operative (needle) cholangiography is indicated in patients who at operation appear to have no
gallbladder.
D
The long cystic duct, which appears to be fused with the common duct and enters it distally, should
be dissected free and ligated at its entrance into the common duct.
ANSWER:C
1162 Which of the following are indications for cholecystectomy?
A
The presence of gallstones in a patient with intermittent episodes of right-side upper quadrant pain.
B The presence of gallstones in an asymptomatic patient.
C The presence of symptomatic gallstones in a patient with angina pectoris.
D The presence of asymptomatic gallstones in a patient who has insulin-dependent diabetes.
ANSWER:A
1163 The initial goal of therapy for acute toxic cholangitis is to:
A Prevent cholangiovenous reflux by decompressing the duct system.
B Remove the obstructing stone, if one is present.
C Alleviate jaundice and prevent permanent liver damage.
D Prevent the development of gallstone pancreatitis.
ANSWER:A
1164 Which of the following statement(s) about gallstone ileus is/are not true?
A The condition is seen most frequently in women older than 70.
B Concomitant with the bowel obstruction, air is seen in the biliary tree.
C The usual fistula underlying the problem is between the gallbladder and the ileum.
D
When possible, relief of small bowel obstruction should be accompanied by definitive repair of the
fistula since there is a significant incidence of recurrence if the fistula is left in place.
ANSWER:C
1165
Which of the following lesions are believed to be associated with the development of carcinoma of
the gallbladder?

A Cholecystoenteric fistula.
B A calcified gallbladder.
C Adenoma of the gallbladder.
D All of the above.
ANSWER:D
116 The preferred treatment for carcinoma of the gallbladder is:
A
Radical resection that includes gallbladder in continuity with the right hepatic lobe and regional
lymph node dissection.
B Radiation therapy.
C Chemotherapy.
D None of the above.
ANSWER:D
1167 Standard supportive measures for patients with mild pancreatitis include the following:
A Intravenous fluid and electrolyte therapy.
B Withholding of analgesics to allow serial abdominal examinations.
C Subcutaneous octreotide therapy.
D Nasogastric decompression.
ANSWER:A
1168 Which of the following statements about chronic pancreatitis is/are correct?
A Chronic pancreatitis is the inevitable result after repeated episodes of acute pancreatitis.
B Patients with chronic pancreatitis commonly present with jaundice, pruritus, and fever.
C Mesenteric angiography is useful in the evaluation of many patients with chronic pancreatitis.
D
For patients with disabling chronic pancreatitis and a dilated pancreatic duct with associated stricture
formation, a longitudinal pancreaticojejunostomy (Peustow procedure) is an appropriate surgical
option.
ANSWER:D
1169 Which of the following statements about adenocarcinoma of the pancreas is/are correct?
A It is the fifth most common cause of cancer death in the U.S.
B
Most cases occur in the body and tail of the pancreas, making distal pancreatectomy the most
commonly performed resectional therapy.
C
For cancers of the head of the pancreas resected by pancreaticoduodenectomy, prognosis appears to
be independent of nodal status, margin status, or tumor diameter.

D
The most accurate screening test involves surveillance of stool for carbohydrate antigen (CA 19–9).
ANSWER:A
1170
A 35-year-old woman presents with episodes of obtundation, somnolence, and tachycardia. An
insulinoma is suspected based on a random serum glucose test value of 38 mg. per dl. Which of the
following statements is/are true?
A The most important diagnostic study for insulinoma is an oral glucose tolerance test.
B It may be helpful to perform ERCP in an effort to localize the tumor.
C
Most patients with insulinoma present with extensive disease, rendering them only rarely resectable
or curable.
D
An important component of the preoperative evaluation in patients with presumed insulinoma
involves confirming elevated C-peptide or proinsulin levels and screening for anti-insulin antibodies.
ANSWER:D
1171
Pancreas divisum results from incomplete fusion of the ventral pancreatic duct with the dorsal
pancreatic duct during embryologic development. Which of the following statements correctly
describes pancreas divisum?
A
The body and tail of the pancreas drain via an accessory ampulla distal to the ampulla of Vater. The
uncinate process drains via the ampulla of Vater
B The entire pancreatic ductal system drains via the ampulla of Vater
C
The entire pancreatic ductal system drains via an accessory ampulla proximal to the ampulla of Vater
D The body and tail of the pancreas are absent. The uncinate process drains via the ampulla of Vater
ANSWER:C
1172
Orally administered glucose provokes a greater insulin response than an equivalent amount of
intravenously administered glucose. The incremental response to ingested glucose is due to the
effects of which of the following hormones?
A Gastric inhibitory peptide
B Somatostatin
C Pancreatic polypeptide
D Secretin
ANSWER:A

1173
The islets of Langerhans contain four major endocrine cell types that secrete which of the following
hormones?
A Insulin, somatostatin, glucagon, secretin
B Insulin, somatostatin, cholecystokinin, pancreatic polypeptide
C Insulin, somatostatin, glucagon, pancreatic polypeptide
D Insulin, secretin, glucagon, cholecystokinin
ANSWER:C
1174
A 50-year-old man develops acute pancreatitis due to alcohol abuse. Hyperamylasemia resolves by
the third day after admission. By the eighth hospital day, the patient is noted to have recurrent fever
(38.5°C), progressive leukocytosis (18,500 WBC/mm3), and tachypnea. The most appropriate
management includes which as the next step?
A Laparotomy with pancreatic debridement
B CT guided aspiration of peripancreatic fluid collections
C ERCP with sphincterotomy and placement of biliary stent
D Intravenous amphotericin B
ANSWER:B
1175
The patient in the above question is treated by observation for 8 weeks. He continues to be
symptomatic with epigastric pain. A repeat abdominal CT scan reveals a persistent 6 cm pseudocyst
in the region of the body of the pancreas. The pseudocyst is unilocular and demonstrates a well-
defined rim of fibrous tissue. The gastric antrum is displaced anteriorly. Using CT guidance, 300 ml
of fluid is aspirated from the lesion which is shown to be collapsed radiographically. No further
intervention is performed. What is the risk of pseudocyst recurrence after simple aspiration?
A 80–85%
B 60–65%
C 40–45%
D 20–25%
ANSWER:D
1176
In prospective, randomized trials which of the following agents or therapeutic measures has/have
been demonstrated to accelerate recovery from acute pancreatitis?
A Peritoneal lavage
B Anticholinergic blockade

C Octreotide
D None of the above
ANSWER:D
1177
A 36-year-old woman is admitted to a the hospital with upper abdominal pain, hyperamylasemia,
elevation of serum alkaline phosphatase and ultrasound evidence of cholelithiasis. With intravenous
hydration and analgesia, symptoms rapidly resolved. After 48 hours, serum amylase and alkaline
phosphatase values had returned to normal and physical examination revealed lessening tenderness
in the right upper quadrant of the abdomen. Appropriate management consists of which of the
following as the next step?
A Cholecystectomy and intraoperative cholangiography before hospital discharge
B Elective cholecystectomy at approximately 8 weeks
C
Endoscopic sphincterotomy before discharge followed by cholecystectomy at approximately 8
weeks
D Observation
ANSWER:A
1178
For the patient in the preceding question, symptomatic recurrence at 3 weeks after aspiration is
confirmed ultrasonographically. Endoscopic retrograde pancreatography does not demonstrate
communication of a major pancreatic duct with the pseudocyst. Appropriate management includes
which of the following?
A Pancreatectomy to include the pseudocyst
B Cystgastrostomy
C Repeat aspiration followed by injection of sodium morrhuate into the pseudocyst cavity
D Pancreatic debridement followed by external drainage
ANSWER:B
1179 Which of the following statement(s) relating to chronic pancreatitis is/are correct?
A In the United States, the most common cause of chronic pancreatitis is alcohol abuse
B Approximately 50% of chronic alcoholics develop chronic pancreatitis
C
Clinically significant chronic pancreatitis develops on average after five years of alcohol abuse in
men
D
The risk of alcohol-induced chronic pancreatitis can be decreased by consumption of a high-protein
diet
ANSWER:A

1180
The most appropriate test to confirm a clinical diagnosis of early chronic pancreatitis is which of the
following?
A Serum amylase determination
B Calculation of urinary amylase clearance
C Measurement of para-aminobenzoic acid absorption
D Endoscopic retrograde cholangiopancreatography
ANSWER:D
1181
A 52-year-old male, known to be alcoholic, is evaluated because of chronic abdominal pain. The
clinical diagnosis of chronic pancreatitis is supported by ERCP findings of pancreatic ductal ectasia
with alternating areas of stricture and dilatation. Several pancreatic ductal stones are also noted. With
chronic pain as the operative indication, the most appropriate procedure would be:
A 80% distal pancreatectomy with splenectomy
B Longitudinal pancreaticojejunostomy
C Distal pancreatectomy with end pancreaticojejunostomy
D Total pancreatectomy
ANSWER:B
1182
For the patient in the preceding question, the most appropriate long-term management is which of
the following?
A Endoscopic stenting of the distal common bile duct
B Choledochoduodenostomy
C Pancreaticoduodenectomy (Whipple procedure)
D Percutaneous transhepatic drainage of the common hepatic duct
ANSWER:B
1183
Which of the following is the most common cause of obstructive jaundice in patients with chronic
pancreatitis?
A Adenocarcinoma of the head of the pancreas
B Choledocholithiasis
C Fibrotic stricture of the common bile duct
D Pancreatic pseudocyst formation
ANSWER:C

1184
Alcohol-induced and hereditary chronic pancreatitis are the two most common etiologies observed
in North American patients. Most of the remaining patients fall into which of the following
categories?
A Chronic pancreatitis secondary to hyperparathyroidism
B Chronic pancreatitis caused by protein-calorie malnutrition
C Chronic pancreatitis secondary to congenital pancreatic ductal obstruction
D Idiopathic chronic pancreatitis
ANSWER:D
1185 Which of the following is the most common clinical manifestation of chronic pancreatitis?
A Epigastric pain with radiation to the hypogastrium
B Diabetes mellitus
C Steatorrhea
D Epigastric pain with radiation to the upper lumbar vertebrae
ANSWER:D
1186 For the patient in the preceding question, appropriate management includes which of the following?
A Distal pancreatectomy
B Cystjejunostomy
C Percutaneous drainage
D Primary radiotherapy and chemotherapy
ANSWER:A
1187
A 67-year-old male presents with complaints of itching, dark urine, and epigastric pain. Physical
examination reveals jaundice. Initial laboratory tests show total bilirubin of 6.5 mg/dL, alkaline
phosphatase elevated at 3 the upper limit of normal, and mild elevations in serum transaminases.
Appropriate management includes which diagnostic test next?
A Abdominal ultrasonography
B Computed tomography of the abdomen
C Magnetic resonance imaging of the abdomen
D Endoscopic retrograde cholangiography
ANSWER:A
1188
The most common cause of death in the postoperative period following pancreaticoduodenectomy
is which of the following?

A Myocardial infarction
B Intraperitoneal hemorrhage
C Pulmonary embolism
D Pneumonia
ANSWER:B
1189
Which of the following surgical procedures has the lowest incidence of recurrent jaundice when
used in the context of unresectable carcinoma of the head of the pancreas?
A Choledochoduodenostomy
B Cholecystojejunostomy
C Cholecystoduodenostomy
D Choledochojejunostomy
ANSWER:D
1190
A 45-year-old woman is evaluated for epigastric and back pain. Physical examination is normal.
Computed tomography of the abdomen reveals a 8 cm cystic lesion in the region of the tail of the
pancreas. The cyst demonstrates internal septations and papillary projections from its walls. Which
of the following diagnoses is most likely in this patient?
A Pancreatic lymphoma
B Retroperitoneal liposarcoma
C Pancreatic pseudocyst
D Pancreatic mucinous cystadenoma
ANSWER:D
1191
For the patient in the preceding question, an insulin/glucose ratio of 0.5 was documented at 28
hours of fasting. Symptoms of mental obtundation developed concurrently and were reversed by
oral glucose administration. Endoscopic ultrasonography demonstrated a 1.2 cm mass in the head of
the pancreas. Appropriate management consists of which of the following?
A Surgical enucleation of the tumor
B Total pancreatectomy
C Long-term octreotide administration
D Primary radiotherapy
ANSWER:A
1192
Neoplastic hypersecretion of the hormone vasoactive intestinal peptide is associated with which of
the following features?

A Hypokalemia, hypochlorhydria, diarrhea
B Hyperglycemia, necrolytic rash, hypoaminoacidemia
C Constipation, gallstones, hyperglycemia
D Hyperkalemia, necrolytic rash, diarrhea
ANSWER:A
1193
A patient with biochemically confirmed gastrinoma undergoes computed tomography for tumor
localization. CT reveals a 2 cm mass in the head of the pancreas and multiple nodules within right
and left lobes of the liver. Appropriate management includes which of the following?
A Omeprazole administration
B Radiotherapy
C Pancreaticoduodenectomy
D Proximal gastric vagotomy
ANSWER:A
1194
In the patient described above, which of the following are important operative steps in the
performance of a right hepatic lobectomy?
A The use of an ultrasonic dissector is essential for division of the hepatic parenchyma
B
If temporary portal inflow occlusion is used (Pringle maneuver), it is not necessary to reestablish
blood flow during the course of the parenchymal division
C The greater omentum may be used to buttress the transected liver edge
D Control of the main right hepatic vein should eliminate all forms of venous drainage
ANSWER:C
1195
Intraoperative ultrasound is now commonly used by the hepatic surgeon. Which of the following
statement(s) is/are true concerning intraoperative ultrasound and hepatic surgery?
A
An intraoperative ultrasound offers no advantage to conventional transcorporial ultrasound in
detection of hepatic lesions
B
Portal structures can be differentiated from hepatic veins by the extension of Glisson’s capsule
surrounding these structures
C It is difficult on ultrasound to differentiate a vascular structure from a mass
D The short hepatic veins are difficult to detect with intraoperative ultrasound
ANSWER:B
1196
The liver plays a vital role in carbohydrate metabolism and regulation of blood glucose. The
following statement(s) is/are true concerning carbohydrate metabolism by the liver.

A
Glycogen, a complex polymer of glucose, is synthesized by the hepatocyte in a remarkably energy
efficient process
B Glucagon stimulates glycogenesis
C
Glycolysis, the process by which glucose is converted to two molecules of pyruvate, occurs in the
liver mitochondria
D
If glycogen stores become depleted, the liver is capable of synthesizing new glucose by the process
of gluconeogenesis, which is stimulated by insulin
ANSWER:A
1197 Which of the following statement(s) is/are true concerning acute, fulminant hepatic failure?
A The most frequent cause of acute hepatic failure world-wide is hepatitis B infection
B Higher grades of encephalopathy are associated with a worse prognosis
C Hypoglycemia is a common complication of all liver diseases
D
Liver transplantation would appear indicated in all patients with hepatic coma secondary to acute
liver failure
ANSWER:B
1198
The following statement(s) is/are true concerning the management of ascites associated with chronic
liver disease.
A Spontaneous bacterial peritonitis is an insignificant complication
B Large volume paracentesis is unsafe due to excessive volume loss from the intervascular space
C
Peritoneovenous shunting is a trivial surgical procedure with minimal perioperative morbidity and
mortality
D
Transjugular intrahepatic portosystemic shunts (TIPS) can effectively treat ascites in patients
refractory to conventional medical therapy
ANSWER:D
1199
Which of the following statement(s) is/are true concerning the pathophysiology of variceal
hemorrhage?
A All patients with portal hypertension will develop esophageal varices
B All patients with esophageal varices eventually bleed
C Variceal size can predict the incidence of variceal hemorrhage
D None of the above
ANSWER:D

1200
Which of the following statement(s) is/are true concerning the use of transjugular intrahepatic
portosystemic shunts (TIPS) in the treatment of variceal bleeding?
A This procedure effectively creates an end-to-side portocaval shunt
B Procedure-related mortality is generally in excess of 20%
C TIPS has been used successfully as a pretransplant procedure to reduce portal pressure
D The placement of a TIPS is not associated with the development of encephalopathy
ANSWER:C
1201 Which of the following statement(s) is/are true concerning biopsy techniques for hepatic masses?
A
A fine needle aspiration (FNA for cytology is contraindicated for patients with hypervascular masses)
B
Percutaneous biopsy should be performed only if results may obviate the need for exploratory
laparotomy
C Needle track seeding of tumor is not a risk associated with percutaneous biopsy
D Laparoscopy and biopsy play little role in the management of liver lesions
ANSWER:B
1202
A 55-year-old woman presents with vague right upper quadrant pain and a palpable liver.
Laboratory tests are normal and a noncontrast CT scan (patient has a history of contrast allergy)
reveals an 8 cm right hepatic mass. Which of the following statement(s) is/are correct concerning the
patient’s diagnosis and management.
A
A gadilinium-enhanced MRI would be indicated to define the extent of the lesion and confirm the
diagnosis of hemangioma
B A fine needle aspiration should be performed regardless of radiographic workup
C Hepatic embolization is the treatment of choice
D The lesion should be resected because of concern for malignant degeneration
ANSWER:A
1203
A 35-year-old female presents with typical biliary colic symptoms, however her sonogram shows
no gallstones. Which of the following statement(s) is/are true concerning her diagnoses?
A
Chronic acalculous cholecystitis or gallbladder dyskinesia is seldom associated with classic biliary
colic symptoms
B
The most specific test for diagnosing gallbladder dyskinesia is CCK-enhanced cholescintigraphy
with assessment of gallbladder ejection fraction

C
An ejection fraction greater than 75% is considered abnormal and indicative of gallbladder
dyskinesia
D Cholecystectomy is not indicated for chronic acalculous cholecystitis
ANSWER:B
1204
A 32-year-old woman with symptomatic gallstones wishes to discuss nonsurgical options for her
gallstones. Which of the following statement(s) are true?
A
The best commercially available oral dissolution agent, ursodeoxycholic acid, is associated with a
complete dissolution rate of less than 50%
B If the gallstones dissolve, there is minimal risk of gallstone recurrence
C Contact dissolution is applicable regardless of stone type
D
Extracorporial shock wave lithotripsy (ESWL) in combination with oral dissolution agents is an
appropriate technique for most patients and can result in complete stone fragment clearance in over
90% of patients by one year
ANSWER:A
1205
A 48-year-old woman presents with several hours of acute right upper quadrant pain, low grade
fever, and nausea and vomiting. Which of the following statement(s) is/are true concerning her
diagnosis and management?
A A mild elevation of her bilirubin (< 3 mg/dl) would strongly suggest a common bile duct stone
B A positive bile culture can be expected in virtually 100% of patients with this scenario
C Laparoscopic cholecystectomy is clearly contraindicated
D Appropriate antibiotic coverage should include coverage for gram-negative aerobes
ANSWER:D
1206 Which of the following statement(s) is/are true concerning the solubilization of cholesterol in bile?
A Cholesterol is highly soluble in both serum and bile
B Mixed micelles are the primary transport mechanism for biliary cholesterol
C Most cholesterol found in bile is the result of excretion from serum
D Biliary vesicles are composed primarily of biliary phospholipid
ANSWER:D
1207
Which of the following statement(s) is/are true concerning a 35-year-old woman found to have
asymptomatic gallstones?
A The patient should undergo attempts at medical dissolution

B Cholecystectomy should only be performed if the laparoscopic technique is an available option
C The patient has less than 10% chance of developing significant symptoms over the next five years
D
The risk of gallbladder cancer with stones detected at such a young age, warrants cholecystectomy
ANSWER:C
1208 The following statement(s) is/are true concerning the prognosis for gallbladder cancer.
A Average survival is in the range of one year
B Five year survival rates approach 50%
C
The combination of postoperative adjuvant radiation and chemotherapy have been associated with
overall improved survival
D For most patients, the goal of treatment is palliation
ANSWER:D
1209
The management of a suspected bile duct injury depends on a number of factors, most importantly
the mode and timing of presentation. Which of the following statement(s) is/are true concerning a
patient presenting with a suspected bile leak after laparoscopic cholecystectomy?
A Laparotomy should be performed immediately
B Cholangiography should be performed to determine the nature of the injury
C Operatively-placed drains should be removed to allow the fistula to close
D The patient should be discharged to home to allow the leak to close spontaneously
ANSWER:B
1210
As the functional anatomy of the spleen is divided into red pulp, white pulp, and marginal zone,
what function is incorporated into the anatomy of the cortical zone that relates to infection control?
A Filtration of red cells, encapsulated bacteria, and other foreign material.
B Red pulp for formation of red cells.
C White pulp for its role in formation of granulocytes.
D Gray areas, so formed because of the production of platelets.
ANSWER:A
1211
During the evolution of the understanding of hematologic diseases, the indications for splenectomy
have changed. The most common indications for splenectomy are, in descending order of
frequency:
A Traumatic injury, immune thrombocytopenia, hypersplenism.

B Immune thrombocytopenic purpura, traumatic injury, hypersplenism.
C Hypersplenism, traumatic injury, immune thrombocytopenia.
D Immune thrombocytopenia, hypersplenism, traumatic injury.
ANSWER:A
1212 Useful methods for detection of splenic injury, in descending order of sensitivity, are:
A Diagnostic peritoneal lavage.
B CT.
C Ultrasonography.
D Isotope scan.
ANSWER:B
1213 The following statements about splenosis are correct:
A Autotransplantation of splenic tissue is an etiology.
B May protect against OPSS.
C May over time be “born again” and regain some immune function.
D All of the above.
ANSWER:D
1214 The following comments about immune thrombocytopenic purpura (ITP) are accurate:
A Platelet count is low.
B Circulating antiplatelet factor is present.
C Antiplatelet factor is immunoglobulin G (IgG) antibody.
D All of the above.
ANSWER:A
1215 ITP:
A Is most common in men in their 20s.
B Is frequently cured in adults by corticosteroid administration.
C Usually requires splenectomy in children.
D Is in remission in more than 80% of patients with splenectomy.
ANSWER:D
1216 Splenectomy and perioperative therapy for ITP:
A Follow successful steroid therapy.
B Respond permanently to high-dose intravenous gamma globulin.

C
Are best preceded by polyvalent vaccines for Pneumococcus, Haemophilus influenzae, and
Neisseria meningitidis.
D Cannot be done laparoscopically.
ANSWER:C
1217
Thrombotic thrombocytopenic purpura (TTP) is a syndrome characterized by all of the following
except:
A Thrombocytopenia.
B Microangiopathic hemolytic anemia.
C Deposition of platelet microthrombi.
D Afebrile.
ANSWER:D
1218 Which of the following comments does not describe hypersplenism?
A It may occur without underlying disease identification.
B It may be secondary to many hematologic illnesses.
C It is associated with work hypertrophy from immune response.
D It is associated with antibodies against platelets.
ANSWER:D
1219 Hyposplenism is a potentially lethal syndrome. Which of the following statements is incorrect?
A It is confirmed by isotope scan.
B It is always associated with an atrophic spleen.
C It may be associated with overwhelming post-splenectomy sepsis syndrome (OPSS).
D It is associated with thyrotoxicosis, corticosteroid administration, and some contrast agents.
ANSWER:B
1220
Hodgkin’s disease is a malignant lymphoma with four histologic subtypes. Which of the following is
not one of the subtypes?
A Lymphocyte predominance.
B Nodular sclerosis.
C Mixed cellularity.
D Leukocyte-lymphocyte dominance.
ANSWER:D
1221 Which of the following forces do not promote the formation of interstitial fluid?
A Increased venous pressure.

B Constrictive pericarditis.
C Hypernatremia.
D Hypoproteinemia.
ANSWER:C
1222 The most frequent cause of primary lymphedema is:
A A deficiency of transporting lymphatic channels.
B Valvular incompetence in lymphatic channels.
C Obstruction or removal of regional lymph nodes.
D Thrombosis of lymphatic channels.
ANSWER:A
1223 Most patients with lymphedema can be managed by:
A Pedicle transfer of lymphatic bearing tissue into the affected area.
B Elevation, elastic support garments, and massage therapy or mechanical pneumatic compression.
C Lymphatic bypass using an autogenous vein graft.
D
Excision of hypertrophic scarred fibrotic skin and subcutaneous tissue down to muscle fascia and
coverage with split-thickness skin grafts.
ANSWER:B
1224 Which statements about lymphangiomas are true?
A Most lesions appear during puberty.
B These lesions frequently respond to small doses of radiation therapy.
C The lesions usually grow slowly but may infiltrate local tissues.
D Malignant transformation is frequent.
ANSWER:C
1225 The best therapy for a patient with thrombotic thrombocytopenic purpura is which of the following:
A Plasmapheresis
B Corticosteroids
C Splenectomy
D Intravenous immune globulin
ANSWER:A

1226
You are consulted regarding a 50-year old male with Laennec’s cirrhosis, portal hypertension and
hypersplenism. He has no history of gastrointestinal bleeding. You would recommend which of the
following?
A Splenectomy
B Prophylactic sclerotherapy for esophageal varices
C Portosystemic shunt
D Observation
ANSWER:D
1227
A 40-year old woman with chronic immune thrombocytopenic purpura (ITP) is refractory to
corticosteroids. The approximate likelihood she will benefit from a splenectomy is approximately
which of the following?
A Less than 20%
B 40%
C 60%
D 80%
ANSWER:C
1228
When progressive enlargement of a multinodular goiter causes symptomatic tracheal compression,
the preferred management in otherwise good-risk patients is:
A Iodine treatment.
B Thyroid hormone treatment.
C Surgical resection of the abnormal thyroid.
D Radioactive iodine treatment.
ANSWER:C
1229
The most precise diagnostic screening procedure for differentiating benign thyroid nodules from
malignant ones is:
A Thyroid ultrasonography.
B Thyroid scintiscan.
C Fine-needle-aspiration biopsy (FNAB).
D Thyroid hormone suppression.
ANSWER:C
1230
The preferred operation for initial management of a thyroid nodule that is considered suspicious for
malignancy by FNAB is:

A Excision.
B Partial lobectomy.
C Total lobectomy and isthmusectomy.
D Total thyroidectomy.
ANSWER:C
1231 A familial form of medullary thyroid carcinoma (MTC) should be suspected whenever:
A The tumor is multifocal.
B The tumor is bilateral (foci of tumor are present in both thyroid lobes).
C
Pathologic examination of the resected thyroid gland reveals the presence of C-cell hyperplasia in
areas of the gland adjacent to foci of MTC.
D All of the above.
ANSWER:D
1232 All of the following are components of the MEN type 2B syndrome except:
A Multiple neuromas on the lips, tongue, and oral mucosa.
B Hyperparathyroidism.
C MTC.
D Pheochromocytoma.
ANSWER:B
1233 MEN 2A and MEN 2B syndromes are associated with germline mutations in:
A The p53 tumor suppressor gene.
B The H-ras gene.
C The N-myc gene.
D The RET proto-oncogene.
ANSWER:D
1234
Which of the following are true concerning islet cell neoplasms of the pancreas in patients with
MEN type 1?
A Islet cell neoplasms in patients with MEN 1 are characteristically multicentric.
B The most common islet cell neoplasm in patients with MEN 1 is gastrinoma.
C Islet cell neoplasms in patients with MEN 1 may be malignant.
D All of the above.
ANSWER:D
1235 Which of the following statements about the differential diagnosis of hypercalcemia is/are correct.

A
Malignant tumors typically cause hypercalcemia by ectopic production of parathyroid hormone
(PTH).
B
The diagnosis of primary hyperparathyroidism is supported by these serum levels: calcium, 10.8
mg. per dl.; chloride, 104 mmol. per liter; bicarbonate 21 mmol. per liter; phosphorus, 2.4 mg. per
dl.; elevated parathyroid hormone.
C
Familial hypocalciuric hypercalcemia is distinguished from primary hyperparathyroidism by
parathyroid imaging.
D
Although serum albumin binds calcium, the measured total calcium value is usually unaffected in
patients with severe hypoproteinemia.
ANSWER:B
1236 The parathyroid glands:
A
Develop from the second and third pharyngeal pouches, along with the palatine tonsil and the
thymus.
B
Migrate caudally in the neck in normal development but can be found anywhere from the
pharyngeal mucosa to the deep mediastinum.
C Secrete PTH and calcitonin to manage calcium homeostasis.
D Usually number four, but frequently number only two or three.
ANSWER:B
1237 Prolactinomas of the pituitary:
A Most often produce dysfunctional uterine bleeding in women.
B Most commonly produce infertility in men.
C When asymptomatic, are best treated surgically early in the microadenoma stage.
D May enlarge during pregnancy, requiring treatment with bromocriptine or surgery.
ANSWER:D
1238 Addisonian crisis, or acute adrenocortical insufficiency:
A
Occurs only in patients with known adrenal insufficiency or in those receiving long-term
supraphysiologic doses of exogenous steroids.
B Can mimic an acute abdomen with fever, nausea and vomiting, abdominal pain, and hypotension.
C
May cause electrolyte abnormalities, including hypernatremia, hypokalemia, hypoglycemia, and
hypercalcemia, as well as eosinophilia on peripheral blood smear.
D Should be diagnosed with the rapid ACTH stimulation test before steroid replacement is instituted.
ANSWER:B

1239 Hyperthyroidism can be caused by all of the following except:
A Graves’ disease.
B Plummer’s disease.
C Struma ovarii.
D Medullary carcinoma of the thyroid.
ANSWER:D
1240 Which of the following is true about the use of radioiodine to treat hyperthyroidism?
A If hyperthyroidism is secondary to radioiodine use, it will occur within 2 years of treatment.
B There is a markedly increased risk of future thyroid cancer following radioiodine therapy.
C The risk of leukemia following radioiodine therapy is approximately 10%.
D
Radioiodine may pass through the placenta and lactating breast to produce hypothyroidism in a
fetus or infant.
ANSWER:D
1241 The most common cause of goitrous hypothyroidism in adults is:
A Graves’ disease.
B Riedel’s thyroiditis.
C Hashimoto’s disease.
D de Quervain’s thyroiditis.
ANSWER:C
1242 Therapy for Hashimoto’s disease includes:
A Radioiodine.
B Antithyroid medications.
C Subtotal thyroidectomy.
D None of the above.
ANSWER:D
1243 The principal blood supply to the parathyroid glands is which of the following?
A Superior thyroid arteries
B Inferior thyroid arteries
C Thyroidea ima arteries
D Parathyroid arterial branches directly from the external carotid artery
ANSWER:B

1244
A 45-year-old woman has a solitary, nonfunctioning thyroid nodule and fine needle cytology is
nondiagnostic. Which of the following is the initial surgical procedure of choice?
A Total extracapsular thyroidectomy
B Subtotal thyroid lobectomy and resection of the isthmus
C
Total extracapsular thyroid lobectomy, resection of the isthmus, and modified unilateral neck
dissection
D Total extracapsular thyroid lobectomy and resection of the isthmus
ANSWER:D
1245 Which of the following statements regarding medullary carcinoma of the thyroid are true?
A Approximately 75% of all cases are hereditary
B The overall 10-year survival rate is less than 10%
C
Medullary carcinoma of the thyroid is associated with both multiple endocrine neoplasia IIa (MEN
IIa) and multiple endocrine neoplasia IIb (MEN IIb) syndromes
D
Prophylactic total thyroidectomy is recommended for MEN IIa and MEN IIb patients after the age
of 10 years
ANSWER:C
1246 The definitive treatment of choice for toxic multinodular goiter is?
A Total thyroidectomy
B Bilateral subtotal thyroidectomy
C Unilateral total lobectomy on the side of dominant disease
D 131I treatment
ANSWER:B
1247
A 50-year-old male has undergone an ipsilateral thyroid lobectomy and isthmus resection for what
appeared on frozen section to be a benign nodular lesion 2.0 cm in diameter. Seventy-two hours
later the final pathology returns and the diagnosis is a high-grade angioinvasive follicular
carcinoma. What do you recommend?
A 99mTc bone scan to rule out occult bone metastases
B Ipsilateral radical neck dissection
C Total thyroidectomy
D Observation with sequential 131I scans every 3 months
ANSWER:C

1248
While awaiting surgery on a HMO waiting list, a 50-year-old female with primary
hyperparathyroidism is admitted to the hospital with oliguria, confusion, nausea and vomiting,
muscle weakness and a serum calcium of 13.5 mg/dL. Of the treatment options below, which one is
the most appropriate?
A Administer 1 gm of hydrocortisone STAT
B Begin an IV EDTA (chelating agent) infusion STAT
C
Administer IV Mithramycin and calcitonin concurrently and proceed to ICU for cardiac monitoring
D Begin a normal saline infusion at 2X maintenance volume followed by 1mg/kg furosemide IV
ANSWER:D
1249
A 40-year-old male undergoes an apparently uneventful total thyroidectomy for follicular
carcinoma of the thyroid. 48 hours later he develops circumoral numbness, followed by
laryngospasm, and then has a generalized seizure. Of the following, which is the first priority?
A Proceed to OR for exploration of the operative site
B Administer 25 ml of 10% calcium gluconate intravenously
C Obtain a serum magnesium measurement and administer intravenous magnesium chloride STAT
D Obtain a CT scan of the head to evaluate the possibility of brain metastases
ANSWER:B
1250 Multiple Endocrine Neoplasia (MEN) 2b is characterized by which of the following findings?
A
Medullary carcinoma of the thyroid, pheochromocytoma, mucosal neuromas, and a distinctive
marfanoid habitus
B Parathyroid hyperplasia, pancreatic islet cell tumors, and pituitary adenomas
C Medullary carcinoma of the thyroid, pheochromocytoma, and parathyroid hyperplasia
D Parathyroid carcinoma, pheochromocytoma and chronic pancreatitis
ANSWER:A
1251 Which of the following signs/symptoms are pathognomonic of hyperparathyroidism?
A Pathologic fractures of the metacarpals
B Calcium oxalate nephrolithiasis
C Hypercalcemia causing mental status changes
D Osteitis fibrosa cystica
ANSWER:D

1252
A 10-year-old child presents with hypertension, tachycardia, nervousness and sweating. The best
initial diagnostic evaluation is which of the following?
A Radioimmunoassays for norepinephrine and epinephrine in serum
B Magnetic resonance imaging of the adrenal gland
C MIBG (131I-Methaiodobenzylguanidine) scintigraphy
D Measurement of catecholamines and their degradation products in a 24-hour urine specimen
ANSWER:D
1253
A 25-year-old male has been taking 40 mg of prednisone PO qod for ulcerative colitis for 5 years.
He undergoes an uneventful colectomy with endorectal pull-through and an ileoanal anastomosis.
Which of the following statements regarding steriod management are correct?
A On the day of surgery he should receive 100 mg hydrocortisone IV q6h
B
The postoperative steroid dose should be halved q 12 hours to reduce the risk of infectious
complications and improve would healing
C
Prophylactic treatment with a somatostatin analogue will reduce the risk of postoperative pancreatitis
D Exogenous steroid replacement can be stopped after 3 months
ANSWER:A
1254 Imaging of the adrenal gland is best achieved with which of the following techniques?
A Ultrasound
B Computed tomography (CT)
C Arteriography
D Scintigraphy with 131I-6 b-iodomethyl-19-norcholesterol (NP-59)
ANSWER:B
1255
A 45-year-old female is found to have a 2 cm solid nodule in her right adrenal gland at the time of
an abdominal CT scan following an auto accident. With regard to the adrenal lesion, she is
asymptomatic and it is found to be nonfunctional on evaluation. You would recommend which of
the following?
A Extraperitoneal right adrenalectomy through either a flank of posterior approach
B Suppression with 5 mg prednisone PO qod
C Followup CT scan in 1 to 3 months
D Excisional biopsy via laparaoscopic approach
ANSWER:C

1256
A 20-year-old male with a 10 cm left adrenal mass is found to have 10 mg of norepinephrine in a
24-hour urine collection and a plasma 18 hydroxycorticosterone level of 50 mg/dL. Initial
reoperative preparation should include which of the following?
A Treatment with spironolactone
B Intravenous potassium-loading to prevent intraoperative hypokalemia
C Treatment with phenoxybenzamine
D Treatment with labetalol
ANSWER:C
1257 Which of the following statements regarding the physiology of the adrenal gland are true?
A Release of CRH is regulated principally by negative feedback by ACTH
B Plasma 17-ketosteroid levels reflect the degree of adrenal cortisol production
C Renin undergoes enzymatic cleavage in the lung to angiotensin I
D None of the above
ANSWER:D
1258 The approximate 5-year survival rate for adrenocortical carcinoma is which of the following?
A 0%
B 20%–25%
C 50%–60%
D Nearly 100%
ANSWER:B
1259
A term neonate is noted to have ambiguous female genitalia. This infant is at risk for which of the
following potentially life-threatening problems?
A Cardiomyopathy with congestive heart failure
B Sodium wasting nephropathy with hypovolemia
C Respiratory failure from surfactant deficiency
D Spontaneous hemorrhage from thrombocytopenia
ANSWER:B
1260 Which of the following adrenal lesions can be treated definitively by medical means?
A Benign functional adrenocortical adenoma
B Adrenocortical carcinoma
C Congenital adrenal hyperplasia
D Cushing disease

ANSWER:C
1261 The most common mass lesion in the sella turcica is which of the following?
A Craniopharyngioma
B Aneurysm
C Benign pituitary cyst
D Pituitary adenoma
ANSWER:D
1262
After intraductal papilloma, unilateral bloody nipple discharge from one duct orifice is most
commonly caused by which of the following pathologic conditions?
A Paget’s disease of the nipple.
B Intraductal carcinoma.
C Inflammatory carcinoma.
D Subareolar mastitis.
ANSWER:B
1263 Which of the following conditions is associated with increased risk of breast cancer?
A Fibrocystic mastopathy.
B Severe hyperplasia.
C Atypical hyperplasia.
D Papillomatosis.
ANSWER:C
1264 Which of the following breast lesions are noninvasive malignancies?
A Intraductal carcinoma of the comedo type.
B Tubular carcinoma and mucinous carcinoma.
C Infiltrating ductal carcinoma and lobular carcinoma.
D Medullary carcinoma, including atypical medullary lesions.
ANSWER:A
1265 Which of the following are the most important and clinically useful risk factors for breast cancer?
A Fibrocystic disease, age, and gender.
B Cysts, family history in immediate relatives, and gender.
C Age, gender, and family history in immediate relatives.
D Obesity, nulliparity, and alcohol use.
ANSWER:C

126
Which of the following pathologic findings is the strongest contraindication to breast preservation
(lumpectomy with breast radiation) as primary treatment for a newly diagnosed breast cancer?
A Grade 3, poorly differentiated, infiltrating ductal carcinoma.
B Extensive intraductal cancer around the invasive lesion.
C Tumor size greater than 3 cm.
D Positive surgical margin for invasive cancer.
ANSWER:D
1267 Axillary lymph node dissection is routinely used for all of the following conditions except:
A 2-cm. pure comedo-type intraductal carcinoma.
B 1-cm. infiltrating lobular carcinoma.
C 8-mm. infiltrating ductal carcinoma.
D A pure medullary cancer in the upper inner quadrant.
ANSWER:A
1268
Failure to perform radiation after wide excision of an invasive cancer risks which of the following
outcomes?
A Recurrence of cancer in the ipsilateral breast.
B Shorter survival time.
C Regional nodal recurrence.
D Greater chance of breast cancer mortality.
ANSWER:A
1269
Which of the following treatments should never be recommended to a patient with purely
intraductal carcinoma?
A Modified radical mastectomy.
B Lumpectomy to clear surgical margins, followed by observation.
C Incisional biopsy with an involved margin, followed by radiation.
D Excisional biopsy to clear margins, followed by radiation.
ANSWER:C
1270
The proper treatment for lobular carcinoma in situ (LCIS) includes which of the following
components?
A Close follow-up.
B Radiation after excision.

C Mirror-image biopsy of the opposite breast.
D Mastectomy and regional node dissection.
ANSWER:A
1271
Which of the following statements most accurately reflects the findings of large overview analyses
of clinical trials in which adjuvant chemotherapy for early-stage breast cancer was compared to a
control group treated only with surgery?
A The benefit of adjuvant therapy is confined to young patients.
B Adjuvant therapy benefits all patients and is independent of age or node status.
C Adjuvant therapy does not work in estrogen-positive patients.
D The magnitude of benefit is very large.
ANSWER:B
1272 Which of the following statement(s) is/are true concerning radiation therapy after lumpectomy?
A The total dose given to the breast is usually in the range of 2500 to 3000 cGy
B Radiation to the axillary nodal bed is normally part of the procedure in most patients
C Long-term complications of radiation therapy include rib fractures and arm edema
D Breast edema and skin erythema usually resolves within a few weeks
ANSWER:C
1273
A 21-year-old woman presents with an asymptomatic breast mass. Which of the following
statement(s) is/are true concerning her diagnosis and treatment?
A Mammography will play an important role in diagnosing the lesion
B Ultrasonography is often useful in the differential diagnosis of this lesion
C The mass should always be excised
D The lesion should be considered pre-malignant
ANSWER:B
1274
A 45-year-old woman presents with a weeping eczematoid lesion of her nipple. Which of the
following statement(s) is/are true concerning her diagnosis and management?
A Treatment is with warm compresses and oral antibiotics
B
Biopsy of the nipple revealing malignant cells within the milk ducts is invariably associated with an
underlying invasive carcinoma
C The appropriate treatment is mastectomy
D
The lesion always represents a high-risk disease with a significant risk of subsequent metastatic
disease

ANSWER:C
1275 Which of the following statement(s) is/are correct concerning cystosarcoma phyllodes?
A The tumor is most commonly seen in post-menopausal women
B Total mastectomy is necessary for all patients with this diagnosis
C Axillary lymph node dissection is not necessary for malignant cystosarcoma phyllodes
D Most patients with the malignant variant of cystosarcoma phyllodes die of metastatic disease
ANSWER:C
1276 Which of the following statement(s) is/are true concerning adjuvant systemic therapy?
A
Adjuvant tamoxifen in post-menopausal, node-positive, ER-positive women is equivalent to
cytotoxic chemotherapy
B Tamoxifen clearly improves survival in all hormonal receptor-positive patients
C
CMF is associated with improved overall survival in both pre-menopausal and post-menopausal
node-positive patients
D There is no evidence to suggest a role for chemotherapy in node-negative patients
ANSWER:A
1277
Which of the following statement(s) is/are true concerning tissue sampling techniques for breast
masses?
A
The sensitivity of fine needle aspiration biopsy is such that mastectomy can be performed in the case
of malignant diagnosis
B
The accuracy of mammographic-directed fine needle aspiration biopsy is comparable to that
achieved for that of palpable lesions
C Core-needle biopsy showing normal breast tissue is an acceptable diagnosis
D The technique of core-needle biopsy is not applicable to radiographically detected lesions
ANSWER:B
1278 The bronchial circulation:
A Is the blood supply to the conducting airways.
B
Drains into a peribronchial venous network that may expand considerably with conditions such as
bronchiectasis and chronic obstructive pulmonary disease.
C Is an especially important consideration in pulmonary transplantation.
D All of the above.
ANSWER:D

1279
Clearance of mucus produced in the tracheobronchial tree in chronic bronchitis secondary to
smoking may:
A
Be hampered by the fact that the amount of mucus is increased by the number of mucus-producing
cells at the expense of ciliated cells.
B
Be slowed if patients have decreased lung volume and are therefore unable to generate a vigorous
cough that would cause an inflammatory process.
C Cause a decrease in diffusion capacity and associated hypoxemia.
D All of the above.
ANSWER:A
1280 The pulmonary circulation:
A Is the only vascular system in which the veins do not have the same course as the arteries.
B
Has a direct connection of vein to adjacent lung tissue by connective tissue fibers, making the
diameter of the tissue fibers dependent upon lung volume.
C Supplies the metabolic needs of the alveoli.
D All of the above.
ANSWER:C
1281 The effect of high positive end-expiratory pressures (PEEP) on cardiac output is:
A None.
B Increased cardiac output.
C Decreased cardiac output because of increased afterload to the left ventricle.
D Decreased cardiac output because of decreased effective preload to the left ventricle.
ANSWER:D
1282 Weaning patients from maximum ventilator support usually involves:
A Weaning PEEP first, tidal volume second, and the fraction of inspired oxygen (FIO 2) third.
B Weaning FIO 2 first, ventilator rate second, and PEEP third.
C Weaning FIO2 first, PEEP second, and tidal volume third.
D Weaning FIO 2 first, PEEP second, and ventilator rate third.
ANSWER:D
1283 Which of the following statements about bronchoscopy is false?
A The morbidity and mortality are approximately 0.2% and 0.08%, respectively.
B The most common complications of bronchoscopy are related to premedication of patients.

C
Adjunctive cancer therapy such as laser treatment and brachytherapy may be administered via this
route.
D
Early postoperative bronchoscopy for atelectasis is contraindicated following pulmonary resection.
ANSWER:D
1284 Flexible bronchoscopy is preferred over rigid bronchoscopy for all of the following except:
A Patients with cervical spine injuries requiring intubation.
B The evaluation of a smoke inhalation injury.
C Transcarinal needle aspiration of an enlarged subcarinal lymph node.
D The removal of a bronchus intermedius foreign body from an infant.
ANSWER:D
1285 Which of the following statements about the treatment of postintubation airway stenosis are correct?
A
Emergency management of airway obstruction due to stenosis at the level of a prior tracheal stoma
is best accomplished by establishing a new tracheostomy in normal tracheal tissue just below the scar
of the old stoma.
B
Radial lasering and dilatation usually leads to permanent resolution of postintubation tracheal
stenosis.
C
Splinting of a cervical trachea with a silicone T-tube for 6 to 8 months generally leads to permanent
resolution of stricture.
D
Acquired tracheoesophageal fistula due to intubation injury is corrected by surgical closure of the
fistula concurrent with resection and reconstruction of the damaged trachea.
ANSWER:D
1286 Which of these statements about pleural tumors is/are true?
A The commonest type of pleural tumor is primary pleural mesothelioma.
B Exposure to asbestos dust is causally related to the development of malignant mesothelioma.
C Localized benign mesotheliomas are asymptomatic.
D Complete pleurectomy for malignant mesothelioma usually results in cure.
ANSWER:B
1287
Which of the following treatments would be appropriate therapy for symptoms that persist on
medical therapy and bronchiectasis involving, in order of decreasing severity, the left lower lobe,
the right middle lobe, and the left upper lobe?

A Left pneumonectomy.
B Wedge resection of the left lower lobe.
C Left lower lobectomy.
D Simultaneous left lower lobectomy and right middle lobectomy.
ANSWER:C
1288
Which of the following would not be acceptable sequences of preoperative studies in a patient being
prepared for lingulectomy for bronchiectasis?
A CT alone.
B CT, bronchoscopy, bronchography.
C Bronchoscopy alone.
D Bronchoscopy, bronchography.
ANSWER:C
1289 Which of the following statements about pulmonary mycobacterial infection is/are correct?
A Worldwide, tuberculosis no longer represents a significant public health problem.
B
Mycobacterium tuberculosis is responsible for the majority of cases of pulmonary mycobacterial
disease.
C Mycobacterium kansasii pulmonary infection almost always requires surgical treatment.
D Atypical mycobacteria are never primary pulmonary pathogens in humans.
ANSWER:B
1290 Which statements about squamous papillomatosis of the trachea is/are correct?
A It is the most common type of benign tracheal tumor in adults.
B It is the most common type of benign tracheal tumor in children.
C Most are treated with segmental tracheal resection.
D There is no risk of malignant degeneration.
ANSWER:A
1291 Which of the following statements about pulmonary hamartomas is/are true?
A Hamartomas are benign chondromas.
B Most are located in the conducting airways.
C Wedge resection is curative.
D A lobectomy is necessary to obtain draining hilar lymph nodes.
ANSWER:C

1292
A solitary pulmonary nodule is discovered in an asymptomatic 55-year-old smoker with no
evidence of extrathoracic dissemination. The most appropriate management would be to:
A Obtain serial chest films every 3 months to determine the growth potential of the nodule.
B
Perform transthoracic needle aspiration (TTNA) before considering pulmonary resection to confirm
malignancy.
C
Conduct an extensive systematic evaluation to exclude the possibility that the nodule represents a
metastatic lesion.
D
Proceed with pulmonary resection after ascertaining that the patient would tolerate removal of the
requisite amount of lung.
ANSWER:D
1293
After thoracotomy, pulmonary resection, and mediastinal lymph node dissection, a patient is
determined to have a squamous cell carcinoma 2 cm. in diameter, located 1 cm. from the carina
along the right mainstem bronchus. Three peribronchial lymph nodes are positive for cancer, and all
other lymph node stations are negative. The correct stage, according to the TNM system, is:
A T1N0M0 Stage I.
B T1N1M0 Stage II.
C T2N1M0 Stage II.
D T3N1M0 Stage IIIa.
ANSWER:C
1294
After complete resection of Stage I non-small cell lung cancer (NSCLC), the role of adjuvant
therapy is best summarized thus as:
A Postoperative radiation therapy improves disease-free survival.
B Postoperative radiation therapy improves overall survival.
C Postoperative chemotherapy improves disease-free survival.
D Adjuvant therapy is not indicated after complete resection of Stage I NSCLC.
ANSWER:D
1295 Compared to segmentectomy or wedge resection, lobectomy for NSCLC is associated with:
A Similar operative morbidity but higher operative mortality.
B Similar operative mortality but higher operative morbidity.
C More severe postoperative pulmonary dysfunction.
D Lower incidence of locoregional recurrence.

ANSWER:D
1296 In contrast to NSCLC, small cell lung cancer (SCLC) is characterized by:
A Greater response rate to chemotherapy.
B Inability to achieve surgical cure.
C Less frequent association with paraneoplastic syndromes at the time of diagnosis.
D Lower likelihood of metastases present at the time of diagnosis.
ANSWER:A
1297
All of the following may be acceptable operative approaches to management of the thoracic outlet
syndrome except:
A Scalenectomy.
B Excision of a cervical rib.
C Thoracoplasty.
D First rib resection.
ANSWER:C
1298
Initial conservative (nonsurgical) management of the thoracic outlet syndrome may include all of
the following except:
A Weight reduction.
B Improvement of posture.
C Exercises to strengthen the muscles of the shoulder girdle.
D Pentoxifylline.
ANSWER:D
1299 Prolonged extracorporeal membrane oxygenation (ECMO):
A Is highly successful in the treatment of severe respiratory failure in newborn infants.
B Is contraindicated in adult respiratory distress syndrome (ARDS).
C Causes hemolysis and renal failure.
D Requires total systemic heparinization (activated clotting time longer than 500 seconds).
ANSWER:A
1300
A 36-year-old female developed dyspnea on exertion that has progressed over 3 months. Chest film
reveals a left anterior mediastinal mass with evidence of elevated left hemidiaphragm. CT indicates
probable invasion of the pericardium. Paratracheal or subcarinal adenopathy is not identified.
Appropriate intervention in this patient would include:

A
A median sternotomy with radical resection of the tumor, sacrificing the left phrenic nerve and
excising the involved pericardium.
B A mediastinoscopy with biopsy.
C A left anterolateral thoracotomy or median sternotomy with generous biopsy of the tumor.
D Observation with repeat chest radiography in 3 months.
ANSWER:C
1301
Which of the following would be the least appropriate in the management of acute suppurative
mediastinitis?
A Wide débridement.
B Irrigation under pressure.
C Topical antibacterials.
D Long-term systemic antibacterials.
ANSWER:D
1302 Each of the following is appropriate for managing acute suppurative mediastinitis except:
A Alloplastic material and skin flaps.
B Rectus abdominis muscle flaps.
C Omentum.
D Pectoralis major muscle flaps.
ANSWER:A
1303 Clinical features suggestive of myasthenia gravis include all of the following except:
A Proximal muscle weakness.
B Diplopia.
C Sensory deficits of the extremities.
D Dysphagia.
ANSWER:C
1304 The diagnosis of myasthenia gravis can be confirmed most reliably using:
A Anti–acetylcholine receptor antibody titers.
B The Tensilon test.
C Electromyography (EMG).
D Single-fiber EMG.
ANSWER:D
1305 All of the following statements are true about the pathogenesis of myasthenia gravis except:

A The number of functional acetylcholine receptors at the motor end plate is reduced.
B
An autoimmune mechanism involving antibodies to the acetylcholine receptor has been proposed.
C Complement system involvement has been demonstrated.
D
Clinical improvement following thymectomy is correlated with decreased acetylcholine receptor
antibody titers.
ANSWER:D
1306 Which of the following statements about the results of thymectomy for myasthenia gravis are true?
A Patients with ocular symptoms experience clinical improvement in 90% of cases.
B Clinical remission can be expected in 90% of cases.
C The response rate to thymectomy for patients with generalized symptoms is 90%.
D Patients with thymoma experience improvement in 75%.
ANSWER:C
1307 All of the following are true of the treatment of myasthenia gravis except:
A
The transcervical approach to surgical thymectomy is less likely to benefit the patient with
myasthenia gravis.
B Corticosteroids result in improvement in 80% of patients.
C Plasma exchange is associated with improvement in up to 90% of patients.
D
Medical therapy with Mestinon (pyridostigmine) is associated with remission in approximately 10%
of patients.
ANSWER:A
1308 Which of the following statements about cardiac tamponade is/are correct?
A
At least 500 ml. of fluid must be present in the pericardium of an adult to cause symptoms of
tamponade.
B
A drop in systemic blood pressure of greater than 20 mm. Hg during inspiration (pulsus paradoxus)
is a finding specific to cardiac tamponade.
C
The vast majority of patients with cardiac tamponade demonstrate a low QRS voltage, nonspecific
ST T-wave abnormalities, and electrical alternans (alternation of QRS amplitude) on the
electrocardiogram.

D
When the diagnosis is made, treatment must be instituted rapidly and may include pericardiocentesis,
creation of a pericardial window, and identification and treatment of the underlying cause.
ANSWER:D
1309
The relationship between small-cell and non-small cell lung cancers can be described by the
following:
A They differ by histology, clinical behavior and cell of origin
B Of all lung cancers, approximately 80% are non-small cell and 20% are small cell
C Both cell types are predictably responsive to chemotherapy
D The International Staging System can be applied to both tumor types
ANSWER:B
1310
For the patient in the pervious question to become an operative candidate which of the following
must be met?
A Extrathoracic metastases must be able to be controlled by another modality, e.g. radiotherapy
B Tumor doubling time must exceed 40 days
C If there is recurrence at the primary site, it must be treated before the metastatic disease
D Even if effective systemic therapy is available, resection of metastases is preferred
ANSWER:C
1311
A 61-year-old male presents with a painful mass 3.5 cm in diameter below the clavicle and attached
to the chest wall. The following is/are true:
A A CT scan is the best study to determine rib destruction
B
The lesion should be removed enbloc without biopsy to minimize the chances for local recurrence
C The chances are approximately 40% that the lesion is metastatic
D If it is metastatic, the most likely primary tumor is in the lung or pancreas
ANSWER:C
1312
82. The lesion shown (Fig. 62-6) was found on a 32-year-old male on a routine chest film required
for his employment. Which of the following is/are true?
A The stippled calcification and intact cortex of the rib are characteristic of osteochondroma
B The stippled calcification is characteristic of osteogenic sarcoma
C If the lesion is osteogenic sarcoma, the optimal treatment is resection and radiation therapy
D If the lesion is an osteochondroma, it need not be resected in this age group

ANSWER:A
1313
86. A 38-year-old man presents with facial and upper extremity edema, venous distention in the
neck and arms and a cyanotic appearance. The following is/are true statement(s):
A The most likely cause of the problem is mediastinal granulomatous disease
B A venogram should be obtained to confirm the diagnosis
C Mediastinoscopy for diagnosis is contraindicated
D If the etiology is benign disease, gradual improvement without operation is to be expected
ANSWER:D
1314 Which is not true of cardiopulmonary resuscitation (CPR)?
A Closed chest massage is as effective as open chest massage.
B The success rate for out-of-hospital resuscitation may be as high as 30% to 60%.
C The most common cause of sudden death is ischemic heart disease.
D
Standard chest massage generally provides less than 15% of normal coronary and cerebral blood
flow.
ANSWER:A
1315
Which maneuver generally is not performed early before chest compression in basic life support
outside the hospital?
A Call for help.
B Obtain airway.
C Electrical cardioversion.
D Ventilation.
ANSWER:C
1316 Which treatment would be least effective for asystole?
A External pacemaker.
B Intravenous epinephrine, 10 ml. of 1:10,000.
C Intravenous calcium gluconate, 10 ml. of 10% solution.
D Intravenous atropine, 0.5 mg.
ANSWER:C
1317 The most important factor that influences the outcome of penetrating cardiac injuries is:
A Comminuted tear of a single chamber.
B Multiple-chamber injuries.
C Coronary artery injury.

D Tangential injuries.
ANSWER:C
1318 The most useful incision in the operating room for patients with penetrating cardiac injury is:
A Left anterior thoracotomy.
B Right anterior thoracotomy.
C Bilateral anterior thoracotomy.
D Median sternotomy.
ANSWER:D
1319
In patients who present with a penetrating chest injury, injury to the heart is most likely when the
following physical sign(s) is/are present:
A Hypotension.
B Distended neck veins.
C Decreased heart sound.
D All of the above.
ANSWER:D
1320 Which of the following constitutes a true vascular ring?
A Pulmonary artery sling.
B Double aortic arch.
C Anomalous origin of right subclavian artery from the descending aorta.
D Cervical aortic arch.
ANSWER:B
1321
In an infant with suspected PDA, which of the following would be the optimal method of
confirming the diagnosis?
A Chest film.
B Cardiac catheterization.
C Retrograde aortography via an umbilical artery catheter.
D
Two-dimensional echocardiography with continuous-wave and color-flow Doppler
echocardiography.
ANSWER:D
1322
The atrial septal defect (ASD) most commonly associated with partial anomalous pulmonary venous
return (PAPVR) is:
A Secundum defect.

B Sinus venosus defect.
C Ostium primum defect.
D Complete atrioventricular (AV) canal defect.
ANSWER:B
1323 The direction of an intracardiac shunt at the atrial level is controlled by:
A The size of the defect
B The compliance of the right and left ventricles.
C The systemic oxygen saturation.
D Right atrial pressure.
ANSWER:B
1324 The ASD most commonly associated with mitral insufficiency is:
A Secundum defect
B Sinus venosus defect
C Ostium primum defect.
D Coronary sinus defect.
ANSWER:C
1325
An electrocardiogram (ECG) in a patient with a systolic ejection murmur that shows an incomplete
bundle branch block in the precordial lead is most consistent with:
A A secundum ASD.
B A sinus venosus ASD with PAPVR.
C An ostium primum ASD.
D A complete AV canal defect.
ANSWER:A
1326
ASDs can all be closed with a pericardial or prosthetic patch. Which of the following ASDs can also
be safely closed primarily without the use of a patch?
A Secundum ASD.
B Sinus venosus ASD with PAPVR.
C An ostium primum ASD.
D A complete AV canal defect.
ANSWER:A
1327 Which of the following is/are true of the surgical treatment of VSDs?
A A right ventricular approach is employed for the repair of most perimembranous VSDs.

B
Intracardiac repair is advisable for patients with intractable symptoms and for asymptomatic infants
with evidence of increasing pulmonary vascular resistance.
C Complete heart block is a common complication.
D Hospital mortality after repair of VSD in infants approaches 20%.
ANSWER:B
1328 Tetralogy of Fallot consists of all of the following features except:
A ASD.
B VSD.
C Dextroposition of the aorta.
D Pulmonary stenosis.
ANSWER:A
1329 Which of the following has the greatest impact on the physiology of tetralogy of Fallot?
A The size of the ASD.
B The size of the VSD.
C The degree of pulmonary stenosis.
D The amount of aortic overriding.
ANSWER:C
1330 Which of the following anomalies is not associated with tetralogy of Fallot?
A Absence of the left pulmonary artery.
B A right aortic arch.
C A retroesophageal subclavian artery.
D Primary pulmonary hypertension.
ANSWER:D
1331 Surgical treatment of a patient with tetralogy of Fallot can include any of the following except:
A
Maintenance of ductal patency with prostaglandins (PGE 1) to provide pulmonary blood flow while
the baby is transferred to an institution equipped to provide more definitive therapy.
B
Banding of the pulmonary artery in an acyanotic patient with tetralogy of Fallot to control
pulmonary blood flow and prevent the development of pulmonary hypertension.
C
Placement of a subclavian-to-pulmonary artery shunt on the side opposite the aortic arch in a 3-day-
old infant with severe cyanosis.
D
Closure of the VSD and transannular patching of the right ventricle onto the main pulmonary artery
in a 2-day-old infant.

ANSWER:B
1332
The predominant determinant of outcome for patients with pulmonary atresia and an intact
ventricular septum revolves around:
A The size of the ASD.
B The baby’s age at presentation.
C The size of the right ventricular cavity and tricuspid valve.
D The presence of a tricuspid—as opposed to a bicuspid—pulmonary valve.
ANSWER:C
1333 Initial management of a newborn infant with hypoplastic left heart syndrome should include:
A Intravenous administration of PGE 1.
B Supplemental oxygen.
C Routine intubation and mechanical ventilation to achieve a PCO 2 between 30 and 35 mm. Hg.
D Cardiac catheterization and balloon atrial septostomy.
ANSWER:A
1334 Which of the following arteries is most likely to be involved with serious atherosclerosis?
A The right coronary artery.
B The left coronary artery.
C The anterior descending coronary artery.
D The circumflex coronary artery.
ANSWER:C
1335 Which statement(s) about operative mortality and perioperative incidence of myocardial infarction
for elective CABG (X) versus emergency CABG following failed PTCA (Y) is/are accurate?
A
The operative mortality is higher for Y but the incidence of perioperative myocardial infarction is
unchanged between X and Y.
B
The operative mortality is unchanged between X and Y but the perioperative incidence of
myocardial infarction is higher in Y.
C The operative mortality and perioperative incidence is higher in X than in Y.
D
The operative mortality and perioperative incidence of myocardial infarction are no different for X
and for Y.
ANSWER:C

1336
Considering the results of coronary reoperation in comparison to primary CABG, choose the
incorrect statement:
A Operative morbidity and mortality are increased over those for primary CABG.
B Mortality most often stems from cardiac causes after reoperation.
C
Survival of patients after hospital discharge following coronary reoperation is nearly equivalent to
survival after primary CABG.
D Compared to primary CABG, return of anginal symptoms is delayed after reoperative CABG.
ANSWER:D
1337
Which statements are correct comparisons of gated equilibrium and initial-transit radionuclide
measurements of left ventricular function?
A
Gated equilibrium techniques provide more accurate measurements of ejection fraction than initial-
transit methods.
B
Left ventricular imaging time for a gated equilibrium study is at least 10 times that of an initial-
transit study.
C Both techniques require the same radiopharmaceuticals.
D Both techniques require a bolus injection.
ANSWER:B
1338
The radionuclide variable that contains the greatest amount of prognostic information in patients
with coronary artery disease is:
A Exercise ejection fraction.
B Change in regional wall motion from rest to exercise.
C Maximal cardiac output during exercise.
D Change in heart rate during exercise.
ANSWER:A
1339
The most effective medical therapy in ameliorating the symptoms of Kawasaki’s disease and
preventing the development of giant coronary artery aneurysms is administration of:
A Antibiotics.
B Antiviral agents.
C Aspirin.
D Gamma globulin.
ANSWER:D
1340 Indications for surgical intervention in Kawasaki’s disease include which of the following?

A The presence of multiple coronary artery aneurysms.
B Myocardial infarction and severe left ventricular dysfunction.
C The presence of a 5 mm. aneurysm in the right coronary artery.
D Progressive stenosis in the left anterior descending coronary artery.
ANSWER:D
1341
Which of the following congenital lesions of the coronary circulation causes a cardiac murmur that
is similar to the murmur produced by a PDA?
A Origin of the left coronary artery from the pulmonary artery.
B Origin of the right coronary artery from the pulmonary artery.
C Coronary artery fistula.
D Membranous obstruction of the ostium of the left main coronary artery.
ANSWER:C
1342 The congenital coronary lesion most likely to cause death in infancy is:
A Coronary artery fistula.
B Origin of the left coronary artery from the pulmonary artery.
C Origin of the right coronary artery from the pulmonary artery.
D Congenital coronary aneurysm.
ANSWER:B
1343
The congenital coronary lesion associated with minimal or absent clinical manifestations and nearly
normal life expectancy is:
A Congenital origin of both coronary arteries from the pulmonary artery.
B Congenital coronary artery fistula.
C Membranous obstruction of the ostium of the left main coronary artery.
D Congenital origin of the right coronary artery from the pulmonary artery.
ANSWER:D
1344 Under which of the following circumstances is medical management logical?
A
Moderate aortic insufficiency seen on echocardiography with normal left ventricular end-systolic
dimensions.
B
Moderate to severe aortic insufficiency seen on echocardiography with cardiomegaly on chest
roentgenography.
C
Moderate aortic insufficiency seen on echocardiography with symptoms of congestive heart failure.

D
Moderate aortic insufficiency with an end-systolic left ventricular dimension of 70 mm. as seen on
echocardiography.
ANSWER:A
1345
Which of the following are relative indications for mitral valve replacement, as opposed to mitral
valve repair?
A Extensive leaflet calcification.
B Mitral regurgitation.
C Chordal rupture of the anterior mitral leaflet.
D Significant annular dilatation.
ANSWER:A
1346 Which of the following are not true?
A
Tricuspid regurgitation due to annular dilatation alone generally does not require valve replacement.
B
Mitral valve replacement with either a bioprosthesis or a mechanical valve requires warfarin
anticoagulation.
C Tricuspid valve replacement is generally an indication for using a tissue valve.
D Chronic renal failure is a relative indication for tissue valves.
ANSWER:B
1347
Which of the following are relative indications for mechanical, as opposed to tissue, valve
replacement?
A Patient younger than 30 years.
B Young female patient who desires children.
C An elderly patient.
D Tricuspid valve replacement.
ANSWER:A
1348
The most common location of accessory pathways in patients with the Wolff-Parkinson-White
syndrome is the:
A Left free wall.
B Right free wall.
C Posterior septum.
D Anterior septum.
ANSWER:A

1349
The anatomic electrophysiologic basis of AV node re-entry tachycardia is dual AV node conduction
pathways. AV node re-entry tachycardia is most likely to occur with which of the following
electrophysiologic aberrations?
A Proximal antegrade block in the slow conduction pathway.
B Proximal retrograde block in the slow conduction pathway.
C Proximal antegrade block in the fast conduction pathway.
D Proximal retrograde block in the fast conduction pathway.
ANSWER:C
1350 All of the following statements about nonischemic ventricular tachyarrhythmias are true except:
A They usually occur in the right ventricle.
B They are usually associated with a left bundle branch block pattern during the tachycardia.
C They are usually more refractory to medical therapy than ischemic ventricular tachyarrhythmias.
D They usually occur as a result of automaticity rather than re-entry.
ANSWER:D
1351
Disadvantages of temporary pacing through skin electrodes applied to the anterior chest wall include
all of the following except:
A Skin burns.
B Painful chest wall muscle contractions.
C Ventricular fibrillation.
D Inability to pace.
ANSWER:C
1352 In adults the most common cause of acquired complete heart block is:
A Ischemic heart disease.
B Sclerodegenerative disease.
C Traumatic injury.
D Cardiomegaly.
ANSWER:B
1353 The most common indication for permanent pacing is:
A Complete heart block.
B Second-degree AV block.
C Chronic bifascicular block.
D Sick sinus syndrome.

ANSWER:D
1354 Decreasing pacemaker electrode tip size results in:
A Lower pacing thresholds.
B Improved electrogram sensing.
C Decreased battery life.
D Less patient discomfort.
ANSWER:A
1355
At the time of ventricular pacemaker implantation, lead resistance is determined at a voltage near
that of the pacemaker’s output. The calculated resistance at 5 volts should range from:
A 10 to 100 ohms.
B 125 to 250 ohms.
C 300 to 800 ohms.
D 1000 to 1500 ohms.
ANSWER:C
1356
A ventricular inhibited-demand pacemaker using the Intersociety Commission for Heart Disease
Resources (ICHD) code is designated as:
A DVI.
B VVI.
C VOO.
D VDD.
ANSWER:B
1357
In rate-modulated pacing, the pacing rate is determined by a physiologic parameter other than atrial
rate and is measured by a special sensor in the pacemaker or pacing lead. The most commonly used
physiologic parameter in rate-modulated pacemakers is:
A QT interval.
B Venous blood temperature.
C Mixed venous oxygen saturation.
D Body motion.
ANSWER:D
1358
The most common pacing mode used in patients with symptomatic bradycardia and an underlying
sinus rhythm is:
A AAI.

B DVI.
C DDD.
D VVI.
ANSWER:C
1359
A transvenous pacemaker generator pocket should be placed on the patient’s nondominant side over
the:
A Anteromedial chest wall.
B Anterolateral chest wall.
C Inferomedial chest wall.
D Inferolateral chest wall.
ANSWER:A
1360 Pacemaker-mediated tachycardia is caused by:
A Pacemaker induction of atrial fibrillation.
B Sensing of retrograde atrial activation.
C Inappropriate ventricular sensing.
D Lead fracture.
ANSWER:B
1361 Which anticoagulation treatment plan(s) is/are appropriate for a 72-year-old man with a mechanical
heart valve in place who takes Coumadin (warfarin) and now requires elective left colon resection?
A Discontinuation of Coumadin therapy on the day of the operation.
B
Discontinuation of Coumadin therapy on the day of the operation with replacement of clotting
factors with fresh frozen plasma (FFP) before the start of the surgical procedure.
C
Discontinuation of Coumadin therapy 5 days before operation with no further anticoagulation
therapy before surgery.
D
Discontinuation of Coumadin therapy 5 days before operation with the institution of intravenous
heparin as the prothrombin time normalizes.
ANSWER:D
1362 Adequate flow during cardiopulmonary bypass is best indicated by:
A Systemic blood pressure of 90/50 mm. Hg.
B Arterial PO 2 of 230 mm. Hg.
C Mixed venous hemoglobin saturation of 78%.

D Central venous pressure of 1 mm. Hg.
ANSWER:C
1363
Which of the following does not typically occur during the first few minutes of cardiopulmonary
bypass?
A Interstitial fluid increases.
B Blood flow becomes nonpulsatile.
C Platelet count decreases.
D Complement is activated.
ANSWER:A
1364
Which of the following are the most frequent complications of intra-aortic balloon counterpulsation?
A Stroke.
B Limb ischemia.
C Arrhythmias.
D Aortic thrombosis.
ANSWER:B
1365
Permanent artificial hearts are being developed that are electrically powered. Wireless techniques are
used to transmit the electrical energy across the body wall using the principle of:
A Infrared sensor.
B Inductive coupling.
C Thermionic coupling.
D High-pressure liquid chromatography (HPLC).
ANSWER:B
136
A 5-year-old girl is found on routine examination to have a pulmonic flow murmur, fixed splitting
of P2 and a right ventricular lift. The following is/are true:
A Cardiac catheterization is indicated if the chest film shows cardiomegaly
B
Radiology report of “scimitar syndrome” findings on the chest film would indicate need for an
arteriogram
C
If the catheterization report is “ostium secondum defect,” at least one pulmonary vein drains
anomalously
D Measured pulmonary vascular resistance of 14 Woods units/m2 with an ASD mandates early repair

ANSWER:B
1367 A 2-month-old infant has EKG evidence of myocardial ischemia and echocardiography suggests
anomalous origin of the left coronary artery from the pulmonary artery. The following is/are true:
A Ischemia is due to perfusion of the myocardium with inadequately oxygenated blood
B Selective coronary angiography should not be attempted because of the risk of myocardial infarction
C
Conservative treatment is preferred to allow the coronary artery to grow to a size that will allow
bypass construction
D If the infant deteriorates, ligation of the coronary at its origin is a viable option
ANSWER:D
1368
A 52-year-old man with known aortic stenosis develops angina pectoris and has a single episode of
syncope. The following is/are true:
A Onset of angina indicates concomitant coronary artery disease independent of valvular lesion
B
Percutaneous aortic balloon valvuloplasty should be considered since it has generally favorable
results
C Patient is not an operative candidate since heart failure has not occurred
D A measured transvalvular pressure gradiant > 50 mmHg would be an operative indication
ANSWER:D
1369 A 42-year-old woman has noted progressive exercise intolerance and fatigability. Examination
discloses an opening snap in the mitral area suggestive of mitral stenosis. The following is/are true:
A Critical mitral stenosis is defined as an orifice area reduced to 2 cm2
B
With a fixed mitral orifice, the change from sinus rhythm to atrial fibrillation has little effect on
cardiac output
C Mural thrombi and thromboembolism are directly related to the presence of atrial fibrillation
D Depressed cardiac output is usually due to depressed myocardial contractility
ANSWER:C
1370
A 47-year-old male with fatigue and cardiac failure has a high-pitched, decrescendo diastolic
murmur along the left sternal border and an apical diastolci rumble. His blood pressure is 148/45
mmHg. The following is/are true:
A Chest radiograph will show cor bovinum

B The apical murmur is due to the Gallavardin phenomenon
C A carotid shudder would be expected
D Abdominal exam will show a pulsatile liver
ANSWER:A
1371
In the initial management of the patient in the previous question with suspected acute MI, the
following is/are true:
A Oxygen and lidocaine should be administered prophylactically
B If chest pain persists, IV nitroglycerin should be used to limit infarct size
C Ca-channel blockers are also of value to limit infarct size
D Morphine IV can be used but has no therapeutic effect
ANSWER:B
1372 True statement(s) concerning cardiac vascular anatomy include the following:
A
In 80%–85% of cases the posterior descending coronary artery (PDA) arises from the circumflex
coronary artery
B The PDA gives off the AV nodal artery
C The great cardiac vein ascends along the right coronary artery to empty into the coronary sinus
D Thebesian veins drain from only left and right ventricles
ANSWER:B
1373
Following repair of an abdominal aortic aneurysm, a 66-year-old man develops severe chest pain,
diaphoresis, bradycardia and hypotension. The following is/are true:
A The electrocardiogram is most likely to show a prominent Q in lead 3 if this is an MI
B If Q wave is present, the infarct is subendocardial rather than transmural
C Creatine kinase measurement alone is diagnostic of MI
D Since bradycardia rarely occurs with MI, another diagnosis should be considered
ANSWER:A
1374
A 70-year-old woman with intractable angina pectoris undergoes cardiac catheterization for possible
mechanical intervention. She prefers PTCA to open correction. The following is/are true:
A A long symmetric lesion in the left main coronary artery would be appropriate for PTCA
B Multiple obstructive lesions in the same artery would be a contraindication to PTCA
C
A focal lesion in the left anterior descending coronary artery where the vessel is 1 mm in diameter
would allow PTCA
D Successful PTCA for a simple lesion carries a recurrent stenosis risk of less than 10%

ANSWER:B
1375
A 52-year-old woman with chest pain is considered for coronary arteriography on the basis of her
risk factors. The following is/are true statement(s):
A All patients with typical anginal symptoms should have coronary arteriography
B Atypical patients with borderline positive stress tests should have arteriography
C Patients who require valve procedures do not require arteriography
D
Patients in refractory heart failure awaiting cardiac transplantation should have coronary
arteriography
ANSWER:B
1376
In the workup of a 45-year-old man with suspected coronary artery disease, the following is/are true:
A Thyroid tests are included to rule out hyperthyroidism
B Typically positive stress ECG would show elevated ST segments
C Dipyridamole is a useful adjunct to thallium scanning as it increases coronary perfusion pressure
D Persisting defects on thallium scan indicate reversible myocardial ischemia
ANSWER:A
1377
A 55-year-old woman gives a history of tiredness, aching, and a feeling of heaviness in the left
lower leg for the past 3 months. These symptoms are relieved by leg elevation. She is also awakened
frequently by calf and foot cramping, which is relieved by leg elevation, walking, or massage. On
physical examination there are superficial varicosities, nonpitting edema, and a slightly painful, 2
cm. diameter superficial ulcer 5 cm. above and behind the left medial malleolus. What is the most
appropriate diagnosis?
A Isolated symptomatic varicose veins.
B Superficial lymphatic obstruction.
C Deep venous insufficiency.
D Arterial insufficiency.
ANSWER:C
1378 The best treatment plan for the patient described in the preceding question should include:
A Varicose vein ligation and stripping as soon as possible.
B Ulcer débridement, vein stripping, and skin grafting.
C Ligation of the medial perforating veins.
D Leg elevation, external Unna boot support, and ambulation without standing.

ANSWER:D
1379
In patients who develop a documented episode of deep venous thrombosis (DVT) the most
significant long-term sequela is:
A Claudication.
B Recurrent foot infections.
C Development of stasis ulcer.
D Pulmonary embolization.
ANSWER:C
1380
A 28-year-old woman developed a painful thrombosis of a superficial varix in the left upper calf 2
days previously. After spending the 2 days in bed with her leg elevated, she felt better and the
tenderness resolved; however, when out of bed she developed a twinge of right-sided chest pain
when walking and a feeling of heaviness in the calf. Which treatment is most appropriate?
A Check for leg swelling, tenderness, and Homan’s sign, and obtain a Doppler ultrasound study.
B Begin antibiotics for a probable secondary bacterial infection.
C Order emergency venography, and if it is abnormal, begin heparin administration.
D
Begin ambulation and discontinue bed rest that probably caused muscle pain by hyperextension of
the knee.
ANSWER:C
1381
In a 55-year-old grocery store cashier with an 8-month history of leg edema increasing over the
course of a work day, associated with moderate to severe lower leg bursting pain, the most
appropriate investigative study or studies are:
A Doppler duplex ultrasound.
B Brodie-Trendelenburg test.
C Ascending and descending phlebography.
D Measurement of ambulatory and resting foot venous pressure.
ANSWER:A
1382
In an otherwise healthy male with previously normal pulmonary and cardiac function, how much of
the pulmonary vascular bed must usually be occluded to produce an unstable cardiovascular state
(shock)?
A 10%.
B 20%.
C 40%.

D More than 50%.
ANSWER:D
1383 Lytic therapy in pulmonary embolism:
A Should precede anticoagulation.
B Can be considered for all patients.
C Can be considered for hemodynamically unstable patients.
D Is indicated for the majority of patients with documented pulmonary embolism.
ANSWER:C
1384 The single most important indication for emergency pulmonary embolectomy is:
A The likelihood of another episode of embolism.
B
The inability to determine whether the problem is acute pulmonary embolism or acute myocardial
infarction.
C The presence of persistent and intractable hypotension.
D Pulmonary emphysema.
ANSWER:C
1385
In prevention of the fat emboli syndrome the primary therapy can be accomplished by which of the
following?
A Systemic anticoagulation achieving a partial thromboplastin time greater than 50 seconds.
B Intravenous administration of alcohol.
C Prophylactic administration of methyl prednisolone.
D
Maintaining a serum albumin value greater than 3 gm. per 100 ml. in the days immediately
following injury.
ANSWER:D
1386
Significant tachypnea and hypoxia follow development of fat emboli syndrome, and the goal of
ventilatory support should be:
A Keeping the respiratory rate below 30.
B Preventing respiratory alkalosis.
C Reversing pulmonary shunting using positive end-expiratory pressure.
D Maintaining an adequate total volume.
ANSWER:C
1387
Which of the following statements correctly characterizes the healing of prosthetic arterial grafts in
humans?

A Complete healing occurs within 3 months of graft implantation.
B Complete healing occurs within 1 year of graft implantation.
C Prosthetic grafts do not heal completely in humans.
D Polytetrafluoroethylene (PTFE) grafts heal completely whereas Dacron grafts do not.
ANSWER:C
1388 Which of the following confirms the diagnosis of transection of the descending thoracic aorta?
A Widened mediastinum.
B Fractured first rib.
C Left pleural effusion.
D Positive aortogram.
ANSWER:D
1389
When complications occur after operating on a descending thoracic aorta, perhaps the most
devastating is:
A Recurrent nerve injury.
B Bleeding with hemothorax.
C Paraplegia.
D Renal insufficiency.
ANSWER:C
1390 The most common risk associated with carotid artery aneurysm is:
A Thrombosis of the aneurysm.
B Embolization of mural thrombus.
C Rupture of the aneurysm.
D Compression of the hypopharynx.
ANSWER:B
1391 The cause of subclavian arterial aneurysms is most often:
A Sepsis.
B A congenital defect.
C Atherosclerosis.
D Fibromuscular dysplasia.
ANSWER:C
1392 Of the visceral aneurysms, which is the most common?
A Celiac.

B Superior mesenteric.
C Hepatic.
D Splenic.
ANSWER:D
1393 An aortic abdominal aneurysm was first successfully resected by:
A Matas.
B Linton.
C Dubost.
D None of the above.
ANSWER:C
1394
Evaluation of the natural history of abdominal aortic aneurysms in patients who are followed
without any surgical procedure indicates that:
A Approximately 20% are alive at the end of 5 years.
B Seventy-five per cent of patients succumb by the end of the first year.
C Aortic rupture is quite common in this group, occurring in more than half by the second year.
D None of the above.
ANSWER:A
1395
The appropriate treatment in most situations of an aortic abdominal graft that has become infected is:
A Intravenous antibiotics and observation for future complications.
B Catheter drainage at the site of infection.
C Replacement of the infected graft with another prosthetic graft.
D Excision of the entire graft and insertion of axillobifemoral grafts.
ANSWER:D
1396
In a patient with an abdominal aortic aneurysm and a history of several previous abdominal
procedures for release of dense peritoneal adhesions causing episodes of intestinal obstruction,
consideration should be given to which one of the following at operation?
A Cardiopulmonary bypass.
B An incision from the xiphoid process to the symphysis pubis.
C Incision in the left flank with a retroperitoneal approach.
D An axillobifemoral graft.
ANSWER:C

1397
Carotid artery occlusive disease most often produces transient ischemic attacks or stroke by which of
the following mechanisms?
A Reduction of flow to the affected area of the brain through stenotic or occluded vessels.
B
Embolization of atheromatous debris and/or clot with occlusion of intracranial branches of the
carotid artery.
C Thrombosis and propagation of the clot into the intracranial branches.
D All of the above.
ANSWER:B
1398 Which of the following does not describe intermittent claudication?
A Is elicited by reproducible amount of exercise.
B Abates promptly with rest.
C Is often worse at night.
D May be an indication for bypass surgery.
ANSWER:C
1399
In terms of long-term graft patency, the best results in the femoral tibial bypass position have been
achieved with:
A A modified human umbilical cord graft.
B Polytetrafluoroethylene (PTFE [Gore-Tex]).
C Saphenous vein allograft.
D Segments of greater and lesser saphenous and cephalic veins spliced together.
ANSWER:D
1400 Which of the following statements about femoral popliteal bypass grafting is/are true?
A Long-term graft surveillance by duplex scanning has no effect on graft patency rates.
B Graft failure and amputation occur in half the patients within 5 years.
C If grafting is successful, long-term mortality is improved.
D Patency rates of 80% to 90% at 1 year currently are expected.
ANSWER:D
1401
Which of the following statements about percutaneous renal artery transluminal angioplasty (PRTA)
are true?
A Patients with renovascular hypertension are usually cured after successful PRTA.
B
Patients with renovascular hypertension due to atherosclerosis are more likely to benefit from PRTA
than those in whom it is due to fibromuscular dysplasia.

C PRTA of ostial atherosclerotic lesions is more successful than PRTA of nonostial lesions.
D
PRTA is associated with a higher morbidity and mortality than angioplasty for peripheral vascular
disease.
ANSWER:D
1402 Which of the following statements are true?
A All arterial injuries are associated with absence of a palpable pulse.
B Preoperative arteriography is required to diagnose an arterial injury.
C The presence of Doppler signals indicates that an arterial injury has not occurred.
D Patients with critical limb ischemia have paralysis and paresthesias.
ANSWER:D
1403
A patient presents with a gunshot wound of the mid-neck. Although drunk, he exhibits no
lateralizing neurologic signs. After control of his airway is achieved, he is taken directly to the
operating room for control of hemorrhage. The common carotid artery has a 2-cm. destroyed
segment. There is also a major esophageal injury. The best treatment for this carotid injury is:
A Vein graft replacement of the common carotid artery.
B Ligation of the common carotid artery.
C Ligation of the common carotid artery proximally with a subclavian carotid bypass.
D Ligation of the common carotid artery with sympathectomy.
ANSWER:A
1404
A 24-year-old man is involved in an industrial accident in which he sustains a crushed pelvis.
Diagnostic peritoneal lavage is positive. At exploration, a large pelvic hematoma is found. What is
the best treatment?
A Explore all the major arteries and veins of the pelvis and surgically control the bleeding if possible.
B Do not explore the pelvic hematoma. Close the abdomen and apply a MAST suit.
C
Do not explore the pelvic hematoma. Apply a pelvic fixator and send the patient to radiology for
possible embolization of bleeding pelvic vessels.
D Use sustained hypotensive anesthesia to try to control bleeding.
ANSWER:C
1405 Which of the following statements about acute arterial occlusion today is/are not true?
A Most arterial emboli originate in the heart as a result of underlying cardiac disease.
B It can be treated under local anesthesia.

C It is usually due to atherosclerotic disease.
D Surgical treatment can usually be avoided if the lesion is diagnosed early.
ANSWER:D
1406 Which of the following statements about lytic agents is/are true?
A They were first introduced well after the advent of balloon embolectomy.
B Streptokinase is nonantigenic.
C Systemic use is the most effective means of delivery.
D The interval to reperfusion limits their utility in the treatment of advanced ischemia.
ANSWER:D
1407 Which of the following is not an indication for postoperative heparinization?
A Suspected venous thrombosis.
B Risk of embolism following acute myocardial infarction.
C Advanced ischemia secondary to acute embolic occlusion.
D Dissolution of residual thrombus after balloon thromboembolectomy.
ANSWER:D
1408 Which of the following is/are not true of the embolectomy catheter technique?
A The balloon should be inflated by the same person who withdraws the catheter.
B Distal exploration should be carried out in all major branches of the affected extremity.
C The balloon is designed to dilate as it traverses areas of luminal narrowing.
D
Removal of adherent thrombus requires alternate catheter-based therapy in addition to balloon
exploration.
ANSWER:C
1409 Which of the following is the least reliable indicator of successful thrombectomy?
A Vigorous back-bleeding after removal of thrombotic material.
B Arteriographically demonstrated patency of all runoff vessels.
C Normal distal pulses.
D Return of normal skin color and temperature.
ANSWER:A
1410
A 21-year-old woman presents with digital color changes in response to cold stimulation. Physical
examination and laboratory data, including an autoimmune disease screen, are normal. She should
be advised that:

A
Her condition is characteristic of vasospastic Raynaud’s syndrome and, while she may be at a
slightly higher risk for developing a connective tissue disease in the future, there is no evidence of
one currently.
B Her problem with her fingers will get progressively worse and she will eventually lose fingers.
C She has scleroderma, which will manifest itself at a later date.
D Her problem is “all in her head.”
ANSWER:A
1411
Obstructive Raynaud’s syndrome can be differentiated from vasospastic Raynaud’s syndrome by the:
A Ice water test.
B Digital hypothermic challenge test.
C Antinuclear antibody levels.
D Digital blood pressure measurement.
ANSWER:D
1412 Which of the following statements about upper extremity arterial insufficiency is/are true?
A Symptomatic ischemia is more common in the upper extremity than in the lower extremity.
B
Vascular injuries from blunt trauma are more common in the upper extremity arteries than in the
lower extremity ones.
C Arteriovenous fistulas frequently follow blunt trauma.
D
The inflammatory process of arteries obstructed by an arteritis should be controlled before a bypass
graft is inserted.
ANSWER:D
1413 Which of the following statements about upper extremity edema is/are true?
A Lymphedema is more common than venous edema.
B
Signs and symptoms of venous obstruction include edema, distention of superficial veins, tightness,
aching, cyanosis, and pain.
C
Distal venous obstructions are more likely than proximal venous obstructions to cause symptoms in
the upper extremity.
D
All patients with symptomatic upper extremity venous thrombosis should receive fibrinolytic
therapy.
ANSWER:B

1414
Which of the following statements about chronic mesenteric ischemia due to atherosclerosis is/are
correct?
A
Postprandial pain in these patients is due to gastric hyperacidity and in most cases is relieved with H
2 blockers.
B Men are more often affected than women.
C
Mesenteric endarterectomy is the surgical treatment of choice, since long-term patency rates are
superior to mesenteric bypass.
D Surgical treatment is indicated to prevent intestinal infarction in symptomatic patients.
ANSWER:D
1415 Which of the following statements about angiotensin II is correct?
A It is a decapeptide.
B It is an enzyme found in high concentration in the pulmonary circulation.
C It is a direct vasoconstrictor and stimulates aldosterone production.
D It is a vasoconstrictor and inhibits aldosterone secretion.
ANSWER:C
1416 Which of the following statements about atherosclerotic obstruction of the renal arteries is true?
A Lesions are usually short.
B These lesions are usually found in the distal renal arteries, particularly just beyond branch points.
C Ostial lesions are best treated with balloon angioplasty.
D Lesions of this type are the second most common cause of renal artery stenosis.
ANSWER:A
1417 Which of the following statements about the treatment of renal artery stenosis is/are true?
A
Though a significant cause of hypertension, renal artery stenosis seldom results in loss of renal
function.
B
In patients with medically controlled renovascular hypertension there is no need to consider
revascularization.
C
Balloon angioplasty is more effective in patients with atherosclerotic disease as compared with those
with fibromuscular disease.
D
In patients with severe atherosclerosis of the aorta, bypass from the splenic or hepatic arteries should
be considered.
ANSWER:D
1418 All but one of the following statements is true. Which is not true?

A Successful clinical repair of injured veins had been effected by the turn of the twentieth century.
B
Initial large experience in managing injured veins came from the battlefields of twentieth-century
wars.
C More than 50% of repaired injured veins thrombose.
D Phlebography is useful in evaluating variable venous anatomy.
ANSWER:C
1419
Concerning in-vivo regulation of the anticoagulated state by endothelium, the following is/are true:
A Heparan-ATIII inactivates only thrombin
B Thrombomodulin serves only to bind thrombin
C Production of von Willebrand factor (VWF) inactivates platelets
D Endothelial cells can secrete tissue factor
ANSWER:D
1420
Ten years after irradiation of the neck for a tonsillar carcinoma, a 59-year-old woman is found to
have symptomatic carotid artery disease. Arteriogram shows a 70% irregular stenotic lesion. The
following is/are true:
A Replacement of the artery should be planned due to radiation induced arterial injury
B
The pathology is most likely to be an inflammatory reaction with endothelial sloughing and
thrombosis
C If atherosclerotic disease is found, the plaque will be no different than nonirradiated plaques
D The patient should be managed medically because of the radiation arterial injury
ANSWER:C
1421
A 23-year-old woman with fever, myalgia and anorexia presents with hypertension and a cool,
ischemic left arm. Angiography shows multiple stenoses of the subclavian and renal arteries. The
following is/are true:
A Coronary angiography is indicated with high likelihood of finding coronary disease
B Endarterectomy of the lesions would be preferred to transluminal angioplasty
C The presentation is more suggestive of Behcet’s disease than Takayasu arteritis
D Preferred management consists of corticosteroids
ANSWER:D
1422 Concerning the fibrinolytic system the following is/are true:
A Plasminogen is an a-globulin

B Fibrin but not fibrinogen is lysed by plasmin
C The main inhibitor of plasmin is a2-macroglobulin
D TAP is activated during fibrin bonding to plasminogen
ANSWER:D
1423
An 82-year-old man with a long history of coronary and peripheral vascular disease presents with
an acutely ischemic right lower extremity. The following is/are true:
A The first step in management should be an arteriogram
B
If intractable congestive heart failure is present, non-operative treatment with heparin would be
appropriate
C If prolonged ischemia has occurred, reperfusion should be accompanied by sodium bicarbonate
D Regardless of the period of ischemia, fasciotomy should be based on the findings postoperative
ANSWER:B
1424
Two days following coronary angiography and angioplasty, a 47-year-old male diabetic develops
painful blue toes on both feet. The following is/are true:
A It is very unlikely that there is any connection between the catheterization and the extremity problem
B The appropriate treatment is vasodilators and an antiplatelet agent
C
If both superficial femoral arteries are obstructed, the most likely etiology is in-situ microvascular
thrombosis
D If renal failure or pancreatitis develops, the outlook for long term survival is very poor
ANSWER:D
1425
A 70-year-old man presents with sudden pain and ischemic changes in his left leg. An arterial
embolus is suspected. The following is/are true:
A The most likely source of an arterial embolus is from intracardiac thrombus on a previous MI
B
If atrial fibrillation (AF) is present, it is known that chronic AF is less likely to produce embolism
than paroxysmal AF
C Currently, the most common cause of AF is ischemic rather than rheumatic heart disease
D
Aspirin is more effective than coumadin in AF for reducing risk of stroke and cardiovascular
mortality
ANSWER:C
1426
A 67-year-old man with acute popliteal arterial embolism has a negative cardiac echo for source of
the thrombus. The following is/are true:

A Most likely non-cardiac source is a thoracic aortic aneurysm
B Embolism is more common from femoral than popliteal arterial aneurysms
C Emboli from popliteal aneurysms are often clinically silent
D Embolism is rare from subclavian artery aneurysms
ANSWER:C
1427 Which of the following is/are appropriate candidates for exercise testing?
A The patient with symptoms of intermittent claudication but normal resting ankle brachial indices
B The patient with rest pain, nonhealing ulcers or gangrene
C If the resting ankle pressure is below 30–40 mmHg
D The patient with blue toe syndrome and readily palpable pedal pulses
ANSWER:A
1428
Which of the following statements is/are true regarding the use of duplex scanning as a means to
follow and monitor bypass grafts?
A Duplex scanning is accurate and cost effective
B
A localized increase in systolic velocity greater than 25% compared to adjacent segments in the graft
identifies a diameter reduction of at least 50%
C Peak systolic velocities should be less than 40 cm/sec throughout the graft
D
Arterial venous fistulas associated with in situ bypass grafts are difficult to detect with a duplex
scanner
ANSWER:A
1429 Which of the following is/are true with respect to hypertension?
A There is a threshold effect of blood pressure on the risk of cardiovascular complications
B The risk of hypertension is essentially confined to stroke
C Common antihypertensive regimens may have adverse effects on a patient’s lipid profile
D
Aggressive blood pressure reduction in patients with ischemic heart disease may increase mortality
and morbidity
ANSWER:C
1430 Which of the following is/are true regarding treatment of diabetes?
A
Strict control with insulin but not oral hypoglycemic agents markedly reduces the incidence of
cardiovascular complications in diabetic persons
B Vascular complications are directly proportional to the degree of glycemic control
C The effects of diabetes are most marked in individuals with other risk factors

D The impact of diabetes and cardiovascular risk is relatively uniform
ANSWER:C
1431
The “father of experimental surgery” who performed pioneering research, including several
transplantation procedures, was:
A Homer, the Greek who described the Chimaera in his Iliad.
B Gasparo Tagliacozzi, the Italian who described a method of reconstructing the nose.
C John Hunter, the Scot who performed autografts and xenografts.
D Emrick Ullmann, the Austrian who performed the first successful renal allograft.
ANSWER:C
1432
Transplantation terminology contains terms to describe the relationship of the graft donor to the
graft recipient. Historical terms such as “homograft” and “heterograft” have been replaced by less
ambiguous terms. The correct modern terminology for a graft between genetically nonidentical
members of the same species is:
A Allogeneic graft.
B Autogeneic graft.
C Isogeneic graft.
D Syngeneic graft.
ANSWER:A
1433
The modern era of clinical organ transplantation began with the advent of chemical
immunosuppression. The important discovery that produced the initial success of cadaveric
transplantation was:
A Cyclophosphamide.
B Azathioprine.
C Cyclosporine.
D Antilymphocyte serum.
ANSWER:B
1434 Which of the following statements about allograft rejection are true?
A
In the absence of immunosuppression, the time and intensity of rejection of transplants between
unrelated donors and recipients is highly variable.
B Allograft rejection may be mediated by antibodies or by cells.
C Allograft rejection is thought to be caused by Th2 cells.
D Acute cellular rejection is the major cause for loss of clinical organ transplants.

ANSWER:B
1435
Utilization of a living related donor instead of a cadaver donor is no longer an advantage in renal
transplantation because:
A
Public recognition of transplantation as a successful therapy has facilitated obtaining family
permission for recovery of transplantable organs. Thus, because sufficient kidneys are available
from “brain-dead” accident victims, there is no need to use related donors.
B
Cyclosporine therapy after cadaveric renal transplants has improved their outcome, which is now
comparable to related-donor transplants.
C
Modern preservation techniques can maintain viability of kidneys from cadaver donors for many
hours, consistently allowing their early function to be as good as that of kidneys from living donors.
D None of the above.
ANSWER:D
1436
As compared with the early immunosuppressive (azathioprine, steroids, antilymphocyte serum)
some newer agents have the following specific advantages:
A
Cyclosporine, which interferes with lymphokine production, exhibits neither bone marrow nor renal
toxicity.
B
Monoclonal antibody (OKT3) is more available and has greater specificity and fewer side effects
than antilymphocyte serum.
C
Tacrolimus (FK506) has properties similar to those of cyclosporine but is especially valuable for
rescue of grafts that are failing on cyclosporine therapy.
D None of the above.
ANSWER:C
1437
Survival rates for patients on dialysis are better than those for patients receiving renal allografts in
the following circumstances:
A A living related donor is available.
B A cadaver donor must be used.
C The recipient’s renal failure is secondary to diabetes.
D None of the above.
ANSWER:D
1438 Which of the following statements about posttransplantation malignancy is correct?

A
Certain immunosuppressive agents increase the incidence of malignancy in transplant recipients,
whereas others do not.
B
Those malignancies most commonly seen in the general population (breast, colon) are substantially
more common in transplant recipients.
C Lymphoproliferative states and B-cell lymphomas are associated with Epstein-Barr virus.
D None of the above.
ANSWER:C
1439
One week after receiving a cadaver renal allograft, the recipient remains oliguric and dialysis
dependent. Ultrasonography reveals a larger perigraft fluid collection. Your next step in
management includes:
A No further investigations (since perigraft collections are fairly common after renal transplantation).
B
Aspiration of the perigraft fluid collection and instillation of a fibrosis-inducing agent to obliterate
the dead space.
C Angiography for localization of a bleeding site in the renal allograft.
D Aspiration of the perigraft fluid collection for chemical analysis.
ANSWER:D
1440 Which of the following is true for hyperacute rejection?
A It is mediated by preformed cytotoxic antibody.
B It occurs late in the life of the graft.
C It is usually reversible with a bolus of steroids.
D None of the above.
ANSWER:A
1441 Hypothermia (0? to 4? C) is a critical component of successful organ cold storage because:
A Oxygen is more soluble in cold solutions and provides a continual supply for energy metabolism.
B There is no way to suppress microbial growth except by cooling and slowing the growth rate.
C
Hypothermia diminishes energy requirements and allows the limited energy reserve to keep the
organ alive.
D It slows metabolism and the enzymic processes that would destroy the cell.
ANSWER:D
1442 Which of the following are contraindications to lung transplantation?
A Age 65 years or older.

B Current corticosteroid therapy.
C History of thoracotomy.
D Ventilator-dependent respiratory failure.
ANSWER:D
1443
Which of the following is the single most useful approach for diagnosing acute lung allograft
rejection?
A Clinical diagnosis.
B Decline in spirometry and oxygenation.
C Chest radiographic abnormalities.
D Fiberoptic bronchoscopy with transbronchial lung biopsy.
ANSWER:D
1444 The most commonly used substitutes for peripheral arteries are:
A Dacron grafts.
B Expanded polytetrafluoroethylene (Gore-Tex) grafts.
C Internal, external, and/or common iliac artery autografts.
D Saphenous vein autografts.
ANSWER:D
1445
Several types of gastrointestinal autografts have been used to replace the esophagus after extirpation
of carcinomas. Successful reconstructions have been achieved most frequently with:
A Stomach.
B Jejunum.
C Ileum.
D Ascending colon.
ANSWER:A
1446
Which of the following statement(s) is/are true concerning changes in physiology following lung
transplant?
A
In patients with pulmonary hypertension, changes in right ventricular function and pulmonary
artery pressure takes weeks to months to resolve
B
In single lung transplantation, changes in pulmonary function are seen almost immediately
following transplantation
C
Patients with double lung transplants have both better pulmonary function studies as well as better
exercise capabilities

D
After single-lung transplant, ventilation perfusion mismatch persists and carbon dioxide retention is
seen
ANSWER:B
1447 Correct statement(s) concerning postoperative complications after hepatic transplantation include:
A Primary nonfunction occurs in 5 to 10% of transplanted livers in the immediate postoperative period
B A biliary leak, although a common complication, is usually of minimal clinical importance
C Portal vein thrombosis occurs much more commonly than hepatic artery thrombosis
D If postoperative bleeding is encountered, immediate return to the operating room is indicated
ANSWER:A
1448 Which of the following statement(s) is/are true concerning renal transplantation?
A
Living-related donor transplants typically can be expected to have one-year graft survival rates of
over 90%
B
Preconditioning of the recipient with the use of donor-specific blood transfusions from their living
donor improves graft survival and therefore should be used routinely
C
Pre-transplant blood transfusions result in improved graft survival following cadaveric renal
transplant in the cyclosporine era
D
Age of the recipient over 50 years is generally associated with a poorer outcome due to graft
rejection
ANSWER:A
1449
Which of the following characteristics or conditions will exclude a patient as a suitable cadaveric
organ donor?
A Active systemic bacterial infection
B Primary CNS malignancy
C Age over 65
D History of prior cholecystectomy for a possible hepatic donor
ANSWER:A
1450
There are numerous toxicities and adverse effects associated with immunosuppression. Which of the
following statement(s) is/are true concerning complications of immunosuppression?
A
Transplant recipients are susceptible primarily to infections with unusual organisms (fungus, virus,
atypical bacteria)

B
Immunosuppressive agents may blunt the inflammatory response to infection leading to a late
presentation of an infectious process
C The development of malignancy appears primarily due to direct mitogenic effects of the agent
D Lymphomas are the most common malignant tumors developing in the transplant patient
ANSWER:B
1451 Cytotoxic T cells (CTL) are capable of recognizing:
A Peptide antigens associated with major histocompatibility complex (MHC) molecules.
B Membrane-bound antigens.
C Cytoplasmic antigens.
D All of the above.
ANSWER:D
1452
Adoptive immunotherapy with lymphokine activated killer cells (LAK) and tumor infiltrate (TIL)
cells are characterized by:
A Nonspecific stimulation of effector cells.
B Expansion ex vivo of large numbers of lymphocytes.
C Infusion with interleukin 2 (IL-2).
D All of the above.
ANSWER:D
1453 Previous clinical studies with cancer vaccines have:
A Clearly demonstrated induction of tumor-specific immune response.
B Repeatedly demonstrated clinical response to large tumor burden.
C Not clearly demonstrated induction of tumor-specific immune response.
D Not been performed to date.
ANSWER:C
1454 Which of the following statements is/are true of the epidemiology and etiology of melanoma?
A Most patients are diagnosed after age 60 years.
B Skin color has no association with risk of melanoma.
C Sun exposure is the only risk factor for melanoma.
D The per capita incidence of melanoma is highest in Australia.
ANSWER:D
1455
Which of the following variables best predicts prognosis for patients with a recent diagnosis of
cutaneous melanoma and no clinical evidence of metastatic disease?

A Breslow thickness.
B Clark’s level.
C Ulceration.
D Gender.
ANSWER:A
1456
A 38-year-old man presents with a melanoma on the skin of the right calf measuring 5 mm. in
thickness. Several large nodes are palpable in the right inguinal region. Which of the following
statements about the appropriate management of this clinical problem is false?
A
In the absence of systemic disease, the primary melanoma of the right calf should be excised with at
least a 2-cm. margin.
B
Complete right inguinal node dissection should be performed if there is no evidence of systemic
metastasis.
C
If further work-up reveals multiple lung metastases of melanoma, they should be excised as soon as
possible.
D
Chemotherapy for melanoma is primarily palliative; so surgical therapy is preferred if there is no
evidence of metastatic disease beyond the inguinal region.
ANSWER:C
1457 Which of the following statements describes an ideal tumor marker?
A
The ideal tumor marker should be tumor specific; that is, in the normal population or patients with
benign diseases, false-positive test results are rare.
B
The ideal marker must have a low false-negative rate; that means that all patients with a particular
type of cancer should test positive.
C
The circulating level of an ideal tumor marker should correlate directly with the amount of viable
tumor and be a measure of the response to therapy.
D All of the above.
ANSWER:D
1458 A marker for the diagnosis of pancreatic cancer is:
A CA 15-3.
B CA 19-9.
C Alphafetoprotein (AFP).
D Carcinoembryonic antigen (CEA).
ANSWER:B

1459 Which of the following tumors may cause elevated CEA levels?
A Breast cancer.
B Colorectal cancer.
C Gastric cancer.
D All of the above.
ANSWER:D
1460
The presence of which marker is a significant poor prognosis variable for patients with breast cancer:
A CEA.
B C-erb B-2.
C AFP.
D Human chorionic gonadotropin (hCG).
ANSWER:B
1461 The most useful circulating marker for patients with hepatocellular carcinoma is:
A CA 50.
B Levels of vitamin B 12.
C CEA.
D AFP.
ANSWER:D
1462
In patients with colorectal cancer the serum CEA level is a clinically useful measure for all reasons
except:
A Prognosis.
B Detection of recurrence.
C Guiding second-look operations.
D Early diagnosis.
ANSWER:D
1463 Which serum markers are useful in the management of patients with testicular cancer?
A hCG.
B AFP.
C CA 15-3.
D Two of the above.
ANSWER:D

1464 Which tumor marker is useful for the management of patients with breast cancer?
A CA 125.
B Inhibin.
C CA 19-9.
D CA 15-3.
ANSWER:D
1465
A new marker that has possible utility in the management of patients with non–small-cell lung
cancer (NSCLC) is:
A Calcitonin.
B Neuron-specific enolase.
C CYFRA 21-1.
D Glucagon.
ANSWER:C
146
A 65-year-old man is seen two years following right hemicolectomy for a Duke’s B-2 carcinoma of
the cecum. Although asymptomatic, the CEA level has risen four-fold from a value obtained six
months previously. Computed tomography reveals a single, 3 cm lesion in the right hepatic lobe.
There is no evidence of extra-hepatic metastatic disease and the patient undergoes right hepatic
lobectomy. Which of the following correctly represents the chance of overall 5 year survival?
A 15%
B 33%
C 50%
D 6%
ANSWER:B
1467 The most serious long-term side effect of bleomycin therapy is which of the following?
A Pulmonary fibrosis
B Cataract formation
C Cardiomyopathy
D Aplastic anemia
ANSWER:A

1468
A 45-year-old woman undergoes excision of a 1 cm breast mass. Histologic examination reveals
invasive ductal carcinoma. Flow cytometric analysis is also performed which determines that a
fraction of the tumor cells are “aneuploid”. The patient asks for an explanation of this term. Which
of the following is/are correct?
A The cells have a DNA content 1 times the baseline content
B The cells have hyperchromatic nuclei
C The cells have a DNA content 2 times the baseline content
D The cells have a DNA content not an even multiple of baseline content
ANSWER:D
1469
A 45-year-old man with long-standing gastroesophageal reflux undergoes upper endoscopy that
reveals patchy areas of epithelium resembling gastric mucosa extending 5 cm proximal to the
esophagogastric junction. Biopsies are obtained. The pathologic report describes “Barrett’s
epithelium”. Which of the following processes does this finding represent?
A Cellular hyperplasia
B Cellular hypertrophy
C Metaplasia
D Carcinoma in situ
ANSWER:C
1470
The most common complication that requires alteration of planned chemotherapy regimens is which
of the following?
A Pulmonary fibrosis
B Gastrointestinal ulceration
C Hematologic suppression
D Hepatotoxicity
ANSWER:C
1471
Resistance of tumors to multiple chemotherapeutic agents is often due to the MDR gene. This gene
encodes a protein that acts by which of the following mechanisms?
A As a transmembrane efflux pump for chemotherapeutic agents
B As a DNA repair molecule
C As an isoform of superoxide dismutase
D As a membrane stabilizer
ANSWER:A

1472
The high incidence of hepatitis B infection in Africa and parts of Asia is thought to be causally
associated with increased incidence of which of the following malignancies?
A Hepatocellular carcinoma
B Esophageal cancer
C Burkitt’s lymphoma
D Gastric carcinoma
ANSWER:A
1473
The Lynch Syndrome is also known as hereditary non-polyposis colorectal cancer. Which of the
following is/are features of this syndrome?
A Left sided colon cancers
B Autosomal dominant inheritance
C Multiple polyps beginning in adolescence
D Multiple cutaneous nevi
ANSWER:B
1474
A number of clinical factors have been noted to decrease sensitivity of tumors to the effects of
ionizing radiation. Which of the following is most important in this regard?
A Increased tissue vascularity
B High tumor mitotic rate
C Tissue hypoxia
D Subcutaneous tumor location
ANSWER:C
1475
Brachytherapy involves the delivery of radiation therapy locally via specially designed catheters
placed in direct apposition to the treated tissue. The most common radioisotope used in this
application is which of the following?
A 125I
B 14C
C 3H
D 34P
ANSWER:A
1476
Patients that have acquired immunodeficiency syndrome are at increased risk for which of the
following neoplasms?
A Colorectal cancer

B Meningioma
C Kaposi’s sarcoma
D Hepatocellular carcinoma
ANSWER:C
1477
When a chemotherapeutic agent is stated to have caused a partial response this implies what degree
of reduction in measurable tumor volume?
A 0–9%
B 10–29%
C 30–49%
D 50–99%
ANSWER:D
1478
Hemophilia B has been treated in a pre-clinical model by gene transfer for which deficient clotting
factor?
A Factor II
B Factor VII
C Factor IX
D Factor X
ANSWER:C
1479
Familial hypercholesterolemia has been proposed as a disease to be treated by gene therapy. The
molecular basis of familial hypercholesterolemia is which of the following?
A Absence of hepatic low density lipoprotein receptors
B Overproduction of high density lipoprotein
C Absence of lipoprotein lipase
D Overproduction of hepatic ornithine transcarbamylase
ANSWER:A
1480
Antisense oligodeoxynucleotides have been proposed as agents for cancer-directed gene therapy.
When delivered intracellularly, antisense molecules act to block which of the following?
A Transcription
B Translation
C Post-translational processing
D Ribosylation
ANSWER:B

1481 Which of the following viruses is/are considered to be neurotropic?
A Adenovirus
B Herpes simplex virus
C Retrovirus
D Adeno-associated virus
ANSWER:B
1482 In neonates with congenital diaphragmatic hernia, which of the following statements is true?
A The defect is more common on the right side.
B Survival is significantly improved by administration of pulmonary vasodilators.
C An oxygen index of 20 is an indication for extracorporeal membrane oxygenation (ECMO).
D Mortality is the result of pulmonary hypoplasia.
ANSWER:D
1483
In neonates with necrotizing enterocolitis, which of the following findings is an indication of
significant bowel ischemia?
A Increased gastric residuals.
B Septic shock.
C Cardiac failure due to a patent ductus arteriosus.
D Erythema of the abdominal wall.
ANSWER:D
1484 The treatment of choice for neonates with uncomplicated meconium ileus is:
A Observation.
B Emergency laparotomy, bowel resection, and Bishop-Koop enterostomy.
C Intravenous hydration and a gastrograffin enema.
D Emergency laparotomy, bowel resection, and anastomosis.
ANSWER:C
1485 The pentalogy of Cantrell includes all of the following except:
A Epigastric omphalocele.
B Sternal cleft.
C Intracardiac defect.
D Pericardial cyst.
ANSWER:D
1486 In infants with duodenal atresia all the following statements are true except:

A There is an increased incidence of Down syndrome.
B Duodenal atresia can be detected by prenatal ultrasound examination.
C It may occur in infants with situs inversus, malrotation, annular pancreas, and anterior portal vein.
D It is best treated by gastroenterostomy.
ANSWER:D
1487
The initial treatment of choice for a 2.5-kg. infant with a 20.0-cm. long proximal jejunal atresia and
8.0 cm. of distal ileum is:
A
Laparotomy, nasogastric suction, proximal dilatation to lengthen the atretic jejunum, total parenteral
nutrition, and delayed anastomosis.
B Laparotomy and proximal end-jejunostomy.
C Laparotomy and immediate small bowel transplantation.
D Laparotomy, tapering jejunoplasty, and end-to-oblique jejunoileal anastomosis.
ANSWER:D
1488
A 2.8-kg. neonate with excessive salivation develops respiratory distress. Attempts to pass an
orogastric catheter fail because the catheter coils in the back of the throat. A chest film is obtained
and shows right upper lobe atelectasis and a gasless abdomen. The most likely diagnosis is:
A Proximal esophageal atresia without a fistula.
B Proximal esophageal atresia with a distal tracheoesophageal (TE) fistula.
C “H-type” TE fistula.
D Esophageal atresia with both proximal and distal TE fistula.
ANSWER:A
1489 Neonates with NEC may demonstrate all of the following findings on abdominal films except:
A Pneumatosis intestinalis.
B Portal vein air.
C Pneumoperitoneum.
D Colovesical fistula.
ANSWER:D
1490 The most common type of congenital diaphragmatic hernia is caused by:
A A defect in the central tendon.
B Eventration of the diaphragm in the fetus.
C A defect through the space of Larrey.

D A defect through the pleuroperitoneal fold.
ANSWER:D
1491 The calorie-nitrogen ratio for an infant should be maintained at:
A 75:01:00
B 100:01:00
C 50:01:00
D 150:01:00
ANSWER:D
1492 All of the following conditions are derived from the primitive embryonic foregut except:
A Bronchogenic cyst.
B Cystic adenomatoid malformation.
C Gastric duplication.
D Mesenteric cyst.
ANSWER:D
1493
For a 22-kg infant, the maintenance daily fluid requirement is approximately which of the
following?
A 1100 ml
B 1250 ml
C 1550 ml
D 1700 ml
ANSWER:C
1494
Other than the history and physical exam, which of the following tests is considered an essential
feature of the preoperative evaluation of a patient with a suspected thyroglossal duct cyst?
A Cervical ultrasound
B Thyroid scan
C Serum T3 and T4 levels
D None of the above
ANSWER:D
1495 Branchial cleft remnants most often present with which of the following clinical problems?
A Infection
B Airway obstruction
C Hemorrhage

D Malignant degeneration
ANSWER:A
1496
Proximity to which of the following structures places it at risk during surgical excision of a second
branchial cleft remnant?
A Internal carotid artery
B External carotid artery
C Hypoglossal nerve
D All of the above
ANSWER:D
1497 Standard therapy for acute epiglottitis in a child is:
A Tracheostomy
B Intravenous antibiotic treatment in an ICU setting
C Endotracheal intubation in the operating room and intravenous antibiotic therapy
D Indirect laryngoscopy and intravenous antibiotics
ANSWER:C
1498
Of the following cystic malformations of the tracheobronchial tree, which is most likely to be
asymptomatic when discovered?
A Intralobar pulmonary sequestration
B Extralobar pulmonary sequestration
C Congenital cystic adenomatoid malformation
D Congenital lobar emphysema
ANSWER:B
1499 Infants with a double aortic arch most commonly present with which of the following problems?
A Dysphagia
B High output cardiac failure related to a patent ductus arteriosus
C Positional hyperemia and edema of the right upper extremity
D Symptomatic tracheal compression
ANSWER:D
1500
Which of the following is most common after primary esophagostomy for esophageal atresia with a
distal tracheoesophageal fistula?
A Anastomotic leak
B Esophageal stricture

C Recurrent tracheoesophageal fistula
D Gastroesophageal reflux
ANSWER:D
1501 Which of the following is the most common primary lung tumor in infants and children?
A Pulmonary blastoma
B Squamous cell carcinoma
C Endobronchial carcinoid
D Leiomyoma
ANSWER:C
1502
The definitive evaluation of a child with a suspected congenital cystic abnormality of the
tracheobronchial tree is best done using which of the following?
A Rigid bronchoscopy
B Computerized tomography or magnetic resonance imaging
C Chest x-ray
D Angiography
ANSWER:B
1503
A newborn infant develops coughing, choking and cyanosis with his first feeding. He is noted to
have excessive drooling. What are the important associated anomalies that must be screened for
prior to surgical intervention?
A Right-sided aortic arch
B Hydrocephalus
C Genitourinary obstruction
D Congenital heart disease
ANSWER:C
1504
Which of the following ventilation strategies is the best initial approach for a neonate with a left
congenital diaphragmatic hernia and the following post ductal arterial blood gases: PaO2 50 mm
Hg, PaCO2 60 mm Hg, pH 7.35?
A High-frequency jet ventilation
B Permissive hypercapnia with convential pressure controlled ventilation
C Extracorporeal membrane oxygenation (ECMO)
D Induced respiratory alkalosis
ANSWER:B

1505
There is an emerging consensus that the surgical repair for congenital diaphragmatic hernia is best
done:
A Emergently at the bedside, eliminating the risks of transporting an unstable neonate
B While on extracorporeal membrane oxygenation
C When the infant is potentially extubatable
D Within the first 48 to 72 hours of life
ANSWER:C
1506
A 3-week old infant has a barium upper gastrointestinal series to evaluate vomiting. The
duodenojejunal flexure is found to be to the right of the midline as well as more caudal and anterior
than a normal ligament of Treitz. The child is seen to reflux barium spontaneously to the level of the
mid-thoracic esophagus. You would recommend which of the following?
A Barium enema
B Emergency laparotomy
C A trial of H2, blockade and cisapride therapy
D Upper gastrointestinal endoscopy
ANSWER:B
1507 The most common cause of pyogenic liver abscess in children today is which of the following?
A Perforated appendicitis
B Blunt liver injury
C Immunocompromised host
D Percutaneous liver biopsy
ANSWER:C
1508
You are asked to recommend therapy for an asymptomatic 2 year old who swallowed a small
alkaline watch battery 4 hours ago. A plain film shows the intact battery in the intestine beyond the
stomach. The best course of therapy is?
A Immediate laparotomy, enterotomy and removal of the battery
B Enteroscopy with extraction
C Laparoscopy with ultrasound localization and extraction
D Cathartics and a follow-up plain film in 48 hours if the child remains asymptomatic
ANSWER:D
1509
The operative procedure of choice for managing the most common type of choledochal cyst is
which of the following?

A Cyst gastrostomy
B Cyst jejunostomy
C Excision with Roux-en-Y hepaticojejunostomy
D Transduodenal marsupialization
ANSWER:C
1510 Which of the following is the most common liver tumor of childhood?
A Hemangioma and hemangioendothelioma
B Hepatoblastoma
C Hepatocellular carcinoma
D Mesenchymal hamaratoma
ANSWER:B
1511
The risk of biliary tract adenocarcinoma developing in a patient with a choledochal cyst left in situ is
approximately which of the following?
A Less than 1%
B 3% to 5%
C 10% to 15%
D Greater than 25%
ANSWER:B
1512 The most common cause of acute pancreatitis in childhood is which of the following?
A Pancreas divisum
B Cholelithiasis
C Trauma
D Valproic acid
ANSWER:C
1513 Which of the following statements regarding gastroschisis are true?
A Primary fascial closure can be achieved in only about 25% of these infants
B These infants have an incidence of approximately 40% to 50% of associated anomalies
C Overall survival is approximately 80% to 90%
D When the diagnosis is known prenatally, planned cesarean section is the safest method of delivery
ANSWER:C
1514
A 6-week-old child presents with generalized seizures, a serum glucose of 30 mg/dL and concurrent
hyperinsulinemia. This child’s first priority is which of the following?

A Permanent central venous access and glucose infusion
B Administration of cortisone and adrenocorticotropic hormone
C Computerized tomographic scan of the abdomen to look for an islet cell adenoma
D Urgent pancreatic resection
ANSWER:A
1515 At what age is surgical orchiopexy recommended for a child with a unilateral undescended testis?
A Promptly upon discovery, regardless of age
B 1 year
C 5 to years
D Any time prior to puberty
ANSWER:B
1516 An infant is noted to have a left flank mass shortly after birth and an ultrasound examination
demonstrates left hydronephrosis. The most common cause of this finding is which of the following?
A Neonatal Wilm’s tumor
B Congenital ureteropelvic junction obstruction
C Multicystic dysplastic kidney
D Vesicoureteral reflux
ANSWER:B
1517
A one month old female infant is brought to you for evaluation of afriable polypoid mass
prolapsing through the vaginal introitus. Your presumptive diagnosis is which of the following?
A Ectopic ureterocele
B Rectal prolapse
C Congenital adrenal hyperplasia with ambiguous genitalia
D Embryonal rhabdomyosarcoma
ANSWER:D
1518
Which of the following approaches is considered standard care for most Wilms’ tumor patients in
the United States today?
A Adriamycin and vincristine therapy followed by surgical resection
B Needle biopsy followed by either chemotherapy or resection depending upon the histology
C Primary surgical resection followed by chemotherapy
D Radiation therapy if judged unresectable on CT or MRI imaging

ANSWER:C
1519 Patients with Wilms’ tumors most frequently present with which of the following?
A Bilateral metachronous lesions
B Bilateral synchronous lesions
C An extrarenal primary
D A unifocal, unilateral lesion
ANSWER:D
1520 Hepatoblastomas are childhood liver tumors characterized by which of the following features?
A Multicentricity
B Cirrhosis in the uninvolved liver
C
Unresectable tumors subjected to cytoreductive chemotherapy may be resected with long-term
survival
D Jaundice
ANSWER:C
1521 The most ominous sign or symptom of urinary system disease is:
A Urinary frequency.
B Pyuria.
C Pneumaturia.
D Hematuria.
ANSWER:D
1522 A patient with acute urinary tract infection (UTI) usually presents with:
A Chills and fever.
B Flank pain.
C Nausea and vomiting.
D Painful urination.
ANSWER:D
1523 Renal adenocarcinomas:
A Are of transitional cell origin.
B Usually are associated with anemia.
C Are difficult to diagnose.
D Frequently are signaled by gross hematuria.
ANSWER:D

1524 Ureteral obstruction:
A Is associated with hematuria.
B
Is associated with deterioration of renal function and rising blood urea nitrogen (BUN) and
creatinine values.
C Is commonly caused by a urinary tract calculus.
D Usually requires open surgical relief of the obstruction.
ANSWER:C
1525 Stress urinary incontinence:
A Is principally a disease of young females.
B Occurs only in males.
C Is associated with urinary frequency and urgency.
D Is a disease of aging produced by shortening of the urethra.
ANSWER:D
1526 Which of the following is/are true of blunt renal trauma?
A Blunt renal trauma and penetrating renal injuries are managed similarly.
B Blunt renal trauma with urinary extravasation always requires surgical exploration.
C Blunt renal trauma must be evaluated by contrast studies using either IVP or CT.
D Blunt renal trauma requires exploration only when the patient exhibits hemodynamic instability.
ANSWER:D
1527 Carcinoma of the bladder:
A Is primarily of squamous cell origin.
B Is preferentially treated by radiation.
C May be treated conservatively by use of intravesical agents even if it invades the bladder muscle.
D May mimic an acute UTI with irritability and hematuria.
ANSWER:D
1528 The major blood supply to the testes comes through the:
A Hypogastric arteries.
B Pudendal arteries.
C External spermatic arteries.
D Internal spermatic arteries.
ANSWER:D
1529 Patients who have undergone operations for benign prostatic hypertrophy or hyperplasia:

A Require routine rectal examinations to detect the development of carcinoma of the prostate.
B Do not need routine prostate examinations.
C Have a lesser incidence of carcinoma of the prostate.
D Have a greater incidence of carcinoma of the prostate.
ANSWER:A
1530 The male contribution to a couple’s infertility is approximately:
A 10%.
B 25%.
C 50%.
D 75%.
ANSWER:C
1531 To maximize fertility potential, orchidopexy for cryptorchidism should be done before:
A Age 15 years.
B Age 12 years.
C Marriage.
D Age 2 years.
ANSWER:D
1532
Within the age group 10 to 35 years, the incidence of carcinoma of the testis in males with intra-
abdominal testes is:
A Equal to that in the general population.
B Five times greater than that in the general population.
C Ten times greater than that in the general population.
D Twenty times greater than that in the general population.
ANSWER:D
1533 The appropriate surgical treatment for suspected carcinoma of the testis is:
A Transscrotal percutaneous biopsy.
B Transscrotal open biopsy.
C Repeated examinations.
D
Inguinal exploration, control of the spermatic cord, biopsy, and radical orchectomy if tumor is
confirmed.
ANSWER:D
1534 If torsion of the testicle is suspected, surgical exploration:

A Can be delayed 24 hours and limited to the affected side.
B Can be delayed but should include the asymptomatic side.
C Should be immediate and limited to the affected side.
D Should be immediate and include the asymptomatic side.
ANSWER:D
1535 Epididymitis, either unilateral or bilateral, in a prepubertal male:
A Is a frequent diagnosis.
B Can be dealt with on an outpatient basis.
C Is a major scrotal problem in this age group.
D Is a rare phenomenon.
ANSWER:D
1536 Patients with prostatitis, especially acute suppurative prostatitis:
A Should have residual urine measured by intermittent catheterization.
B Should have bladder decompression by urethral catheter.
C Should have repeated prostatic massage.
D Should have no transurethral instrumentation if possible.
ANSWER:D
1537 Benign prostatic hypertrophy with bladder neck obstruction:
A Is always accompanied by significant symptoms.
B Is best diagnosed by endoscopy and urodynamic studies.
C Is easily diagnosed by the symptoms of frequency, hesitancy, and nocturia.
D Is always accompanied by residual urine volume greater than 100 ml.
ANSWER:B
1538
A 28-year-old white male presents with asymptomatic testicular enlargement. Which of the
following statement(s) is/are true concerning his diagnosis and management?
A
Tumor markers, b-fetoprotein (AFP) and ?-human chorionic gonadotropin (HCG) will both be of
value in the patient regardless of his ultimate tissue type
B Orchiectomy should be performed via scrotal approach
C The diagnosis of seminoma should be followed by postoperative radiation therapy
D
With current adjuvant chemotherapy regimens, retroperitoneal lymphadenectomy is no longer
indicated for non-seminomatous testicular tumors
ANSWER:C

1539
A 55-year-old male presents with severe flank pain radiating to the groin associated with nausea and
vomiting. Urinalysis reveals hematuria. A plain abdominal film reveals a radiopaque 5 mm stone in
the area of the ureterovesical junction. Which of the following statement(s) is/are true concerning
this patient’s diagnosis and management?
A A likely stone composition for this patient would be uric acid
B The stone will likely pass spontaneously with the aid of increased hydration
C Stone analysis is of relatively little importance
D
Patients with a calcium oxalate stone and a normal serum calcium level should undergo further
extensive metabolic evaluation
ANSWER:B
1540
The neurosurgeon who has had the most profound influence on the development of neurosurgery is:
A Fedor Krause of Germany.
B William Macewen of Scotland.
C Harvey Cushing of the United States.
D Egas Moniz of Portugal.
ANSWER:C
1541
The intracranial tumor most likely to be encountered in a middle-aged man with the acquired
immunodeficiency syndrome (AIDS) is:
A Glioblastoma multiforme.
B Ependymoma.
C Meningioma.
D Lymphoma.
ANSWER:D
1542 The evaluation of a comatose patient with a head injury begins with:
A The cardiovascular system.
B Pupillary reflexes.
C Establishment of an airway.
D Computed tomography (CT) of the brain.
ANSWER:C
1543
Complete excision of a brain abscess used to be the preferred method of treatment, and it is still
performed occasionally today. Most commonly, now, a brain abscess is treated by:

A Systemic antibiotic administration.
B Aspiration and drainage of the abscess through a small opening in the skull.
C Injection of antibiotics into the abscess.
D Aspiration and drainage of the abscess plus systemic antibiotic administration.
ANSWER:D
1544
A 36-year-old man developed neck and left arm pain. He noted paresthesias in the left index and
long fingers. He was found to have weakness of the left triceps muscle and a diminished left triceps
jerk. His left-sided disc herniation is most likely to be at:
A C3–C4.
B C4–C5.
C C5–C6.
D C6–C7.
ANSWER:D
1545 Which of the following statements are true?
A The Hoffmann-Tinel sign localizes the level of a nerve injury.
B Causalgia is a term used to denote the etiology of pain.
C Secondary repair of a lacerated nerve 3 to 8 weeks after injury has several advantages.
D
A surgeon who finds at delayed (3 to 8 weeks) exploration that a clinically nonfunctioning nerve is
in continuity should resect the injured portion of the nerve and suture together the ends.
ANSWER:C
1546 Which of the following lesions is not one of the cutaneous stigmata of occult spinal dysraphism?
A Midline lumbar capillary hemangioma.
B Focal hairy patch over the thoracolumbar spine.
C Dermal sinus located above the midsacrum.
D Café-au-lait spot over the thoracolumbar spine.
ANSWER:D
1547
Myelomeningoceles are congenital malformations of the spinal cord. Which of the following
findings are not commonly associated?
A Hydrocephalus.
B Chiari II malformation.
C A midline dorsal spinal mass easily noted at birth.
D Mandatory urinary incontinence.

ANSWER:D
1548
Which of the following stereotactic procedures would be performed primarily to alter the function
of the brain?
A Stereotactic biopsy of a brain tumor in the right posterior thalamus.
B Stereotactic radiotherapy of an arteriovenous malformation in the right ventrolateral thalamus.
C Stereotactic radiofrequency lesion of the right ventrolateral thalamus for Parkinson’s disease.
D Stereotactic craniotomy for excision of arteriovenous malformation in the right posterior thalamus.
ANSWER:C
1549 What is the critical difference between frame-based and frameless stereotactic procedures?
A The use of digitized imaging studies such as CT and MRI.
B The use of rendered three-dimensional images and a three-dimensional digitizer.
C Rigid fixation of the patient’s head to the operating room table.
D The presence of a lesion in the brain on digitized imaging studies.
ANSWER:B
1550 Commonest site of peptic ulcer is
A 1st part of duodenum
B 2nd part of duodenum
C Distal 1/3rd of stomach
D Pylorus of stomach
ANSWER:A
1551 Most common type of intussusception is
A ilio-colic
B ilio-ileal
C colo-colic
D caeco-colic
ANSWER:A
1552 Dumping syndrome is due to
A Diarrhea
B Presence of hypertonic content in small intestine
C Vagotomy
D Reduced gastric capacity

ANSWER:B
1553 Most common site for impaction of gall stones in gall stone ileus is
A 1st part of duodenum
B 2nd part of duodenum
C Terminal ileum
D Colon
ANSWER:C
1554 Pyoderma gagrenosum is most commonly associated with
A Ulcerative colitis
B Crohn’s disease
C Amoebic colitis
D Ischaemic colitis
ANSWER:A
1555 Colo-vesical fistula most commonly arises from
A Crohn’s disease
B Ulcerative colitis
C Ca colon
D Abdomino perineal resection
ANSWER:A
1556 Post Operative pulmonary thromboembolism is seen in all except
A Tall and thin man
B Obese male
C Pregnant female
D Oestrogen therapy
ANSWER:A
1557 Organism causing infection in patients following spenectomy
A Strep pneumoniae
B Staph aureus
C H-influenzae
D Pseudomonas
ANSWER:A
1558 Massive bleeding per rectum in 70 year old patient is due to

A Ca colon
B Diverticulosis
C Colitis
D Polyps
ANSWER:B
1559 Massive bleeding from colon in a patient is from
A Inferior mesenteric artery
B Superior mesenteric artery
C Celiac artery
D Gastro-duodenal artery
ANSWER:B
1560 Most common surgical cause of obstructive jaundice is
A CBD stone
B Stricture in CBD
C Alcoholism
D Malignancy
ANSWER:A
1561 Most common tumour of spleen is
A Lymphoma
B Sarcoma
C Haemangioma
D Metastasis
ANSWER:A
1562 Which of the following is not an absolute indication for splenectomy
A Splenic abscess
B Hereditary spherocytosis
C Fibrosarcoma
D Autoimmune haemolytic anaemi
ANSWER:D
1563 Most common site for squamous cell ca oesophagus is
A Upper third
B Middle third

C Lower third
D Gastro-oesophageal junction
ANSWER:B
1564 Commonest site of carcinoma oesophagus is
A Upper third
B Middle third
C Lower third
D Gastro-oesophageal junction
ANSWER:B
1565 True regarding congenital hypertrophic pyloric stenosis
A More common in girls
B Hypo chloremic alkalosis
C Hellers myotomy is the procedure of choice
D Most often manifests from burn
ANSWER:B
156 An ulcer that may develop in from burn tissue is
A Marjolin’s
B Rodent
C Melanoma
D Curlings
ANSWER:A
1567 Risk factor for development of gastric Ca
A Blood group O
B Duodenal ulcer
C Intestinal hyperplasia
D Intestinal metaplasia type III
ANSWER:D
1568 Pre- malignant condition with the highest probability for progression to malignancy is
A Dysplasia
B Hyperplasia
C Leucoplakia
D Erythroplakia

ANSWER:D
1569 Cork screw oesophagus is seen in which of the following conditions?
A Carcinoma oesophagus
B Scleroderma
C Achalasia cardia
D Diffuse oesophagus spasm
ANSWER:D
1570 Adenocarcinoma of oesophagus develops in
A Barret’s oesophagus
B Long standing achalasia
C Corrosive structure
D Alcohol abuse
ANSWER:A
1571 In gastric outlet obstruction in a peptic ulcer patient, the site of obstruction is most likely to be
A Antrum
B Duodenum
C Pylorus
D Pyloric canal
ANSWER:B
1572 Strong correlation with colorectal cancer is seen in
A Peuts-jegher’s polyp
B Juvenile polyposis
C Familial polyposis coli
D Hyper plastic polyp
ANSWER:C
1573 In which of the following condition is acquired (secondary) megacolon seen
A Fissure in ano
B Complete absence of parasympathetic ganglion cells
C Absence of sympathetic ganglion cell
D Rectal malignancy
ANSWER:D
1574 The diagnosis of congenital megacolon is confirmed by

A Clinical features
B Barium enema
C Rectal biopsy
D Recto-sigmoidoscopy
ANSWER:C
1575
All of the following extra intestinal manifestations of ulcerative colitis respond to colectomy except
A Primary scleoring cholangitis
B Pyoderma gangrenosum
C Episcleritis
D Peripheral arthralgia
ANSWER:A
1576 Most common cause of peripheral limb ischaemia in India is
A Trauma
B Antherosclerotic
C Burger’s disease
D Takayasu’s disease
ANSWER:B
1577 In which of the following tumours alpha fetoprotein is elevated
A Choriocarcinoma
B Neuroblastoma
C Hepatocellular carcinoma
D Seminoma
ANSWER:D
1578 Acalculous cholecystitis is seen in all the following conditions except
A Enteric fever
B Dengue haemorrhagic fever
C Leptospirosis
D Malaria
ANSWER:D
1579 Most common site of a benign peptic ulcer is
A Upper third of lesser curvature

B Greater curvature
C Pyloric antrum
D Lesser curvature near incisura angularis
ANSWER:D
1580 Most common endocrine tumour of pancreas
A Insulinoma
B Gastrinoma
C Lipoma
D Glucagonoma
ANSWER:A
1581 Virchow triad includes all except
A Venous stasis
B Injury to veins
C Blood hypercoagulability
D Venous thrombosis
ANSWER:D
1582 Most common cause of superficial thrombophlebitis is
A Trauma
B Infection
C Varicosities
D IV infusion
ANSWER:D
1583 Investigations of choice t diagnose hirschsprung’s disease is
A Rectal manometry
B Barium enema
C Rectal biopsy
D Laprotomy
ANSWER:C
1584 Dysphagia lusoria is due to
A Oesophageal diverticulum
B Aneurysm of aorta
C Oesophageal web

D Compression by aberrant blood vessels
ANSWER:D
1585
A young patient with history of dysphagia more to liquid than solids, the first investigation you will
do is
A Barium swallow
B Eosophagoscopy
C USG
D CT Scan
ANSWER:A
1586 Common cause of malabsorption
A Celiac disease
B Crohn’s disease
C Whipple’s disease
D Giardiasis
ANSWER:A
1587 Scleroderma most commonly affects
A Cardia of stomach
B Pylorus
C Duodenum
D Oeosophagus
ANSWER:D
1588 Right gastro epiploic artery is a branch of
A Right gastric artery
B Splenic artery
C Gastroduodenal artery
D Superior pancreatic duodenal artery
ANSWER:C
1589 Glucagonoma is derived from
A B cells of pancreas
B G cells
C Alpha cells
D Delta cells

ANSWER:C
1590 All are pre- malignant except
A Primary sclerosing cholangitis
B Caroli’s disease
C HBV cirrhosis
D Budd- chiarri syndrome
ANSWER:D
1591 Desloughing in burns is generally complete by
A 1 week
B 2 weeks
C 3 weeks
D 4 weeks
ANSWER:C
1592 Most important function in early burn resuscitation is
A Prevention of hypovolaemia
B Treatment of infections
C Prevention of hypothermia
D Psychological support
ANSWER:A
1593 Burn associated ulcer is called
A Cushing’s ulcer
B Curling’s ulcer
C Trophic ulcer
D Peptic ulcer
ANSWER:B
1594 Embolism in cerebral artery occurs most commonly in
A Middle cerebral artery
B Anterior cerebral artery
C Posterior cerebral artery
D Cerebellar arteries
ANSWER:A
1595 Aneurysm need repair if over

A 3.5 cm on USG
B 10cm on USG
C 5.5 cm on USG
D 7 cm on USG
ANSWER:C
1596 Mycotic aneurysm is due to
A Viruses
B Fungi
C Bacteria
D Parasites
ANSWER:C
1597 Most common site for peripheral aneurysm is
A Radial
B Popliteal
C Femoral
D Ulnar
ANSWER:B
1598 Bird beak appearance in barium radiography is seen in
A Ca oeosophagus
B Diffuse oesophageal spasm
C Plummer Vinson syndrome
D Achalasia cardia
ANSWER:D
Tags